[ { "Id": "1", "CreationDate": "2016-01-12T18:45:19.963", "Body": "

When I've printed an object I've had to choose between high resolution and quick prints. What techniques or technologies can I use or deploy to speed up my high resolution prints?

\n", "Title": "How to obtain high resolution prints in a shorter period of time?", "Tags": "|resolution|speed|quality|", "Answer": "

For FDM technologies in general with a single extruder, slicing modifications is your only options. However there will be a trade off between quality and speed. \nFor ABS, changing to a machine with a enclosed build (such as a zortrax) chamber may help and a heated build chamber (Stratasys machine) will help the quality and reliability but not the print speed directly. As ABS has a tendency to warp vase mode is not the best idea either.

\n\n

If you only need high resolution and not strength then reducing the infill percentage or even using vase mode will speed up the print. Also changing to a material that you can print at higher speeds like PLA will magnify any of the previous settings.

\n\n

If you have two extruders then changing to a wider nozzle and using that for infill may speed up the print, heating and cooling time during extruder changeover may actually make it slower.

\n\n

For other technologies there are lots of options digital light projection (DLP) and stereolithography (SLA) both provide significantly higher resolution then FDM, with DLP being the faster of the two, comparison. Take the review with a grain of salt though as the technologies are far from standardised, for example out DLP at work offers far higher resolution and speed then our SLA which is quite old.

\n\n

Sintering or melting technologies can scan the outline every layer then perform a infill of multiple layers at once to speed up the process.

\n" }, { "Id": "2", "CreationDate": "2016-01-12T18:45:51.287", "Body": "

I would like to buy a 3D printer, but I'm concerned about the health risks that are associated with its operation. Some groups of scientists say it can be harmful for humans.

\n\n

What do I need to consider before buying a 3D printer if I care about my health? Are there any safe printers?

\n", "Title": "Is 3D printing safe for your health?", "Tags": "|print-material|safety|health|", "Answer": "

I am going to address the air issue as it is currently unresolved. the third dimension offers a great answer for common safety issues.

\n

The short answer is that based on our limited knowledge at this point, there may be imperceptible health hazards related to FDM / FFF printers and therefore additional safety precautions are, in my opinion, necessary and not optional or secondary as suggested by some in the community.

\n

In other words, if you can isolate your printer in a well-vented area where people rarely go, then of course it's not a health risk, but if people will be exposed to the air of the printer for any significant periods of time, you need to do something about it. This is my situation - where I live dedicated workshops and extra rooms are luxuries that most people do not have.

\n
\n

Realistic Chance of Being Dangerous --> Treat It As Dangerous

\n

The key information at this point in time is the UFP (Ultra-Fine Particle) study that is linked in Tom's answer.

\n

Leaving out the scary / detailed parts:

\n
\n

Therefore, results herein suggest that caution should be used when\noperating these 3D printing instruments inside unvented or unfiltered\nindoor environments due to their large emissions of UFPs.

\n

One important limitation to this study is that we have no information\nabout the chemical constituents of the UFPs emitted from either type\nof 3D printer [...]

\n

[...] there may also be\ndifferences in toxicity because of differences in chemical\ncomposition.

\n
\n

This means that although many processes release UFPs (the authors of the paper compare to cooking), all UFPs are not created equal. Since the UFPs from 3D printing are still an unknown, the only real answer from a safety perspective is to treat them as dangerous.

\n
\n

This is not legal, safety, or professional advice!

\n

I am not qualified to give an opinion on what should be done but I will share what I would do:

\n\n

A note on positive vs negative pressure related to venting and filtering: if you produce positive pressure within the enclosure, you are going to be blowing all the UFPs out into your environment anyway. Negative pressure vented to a safe body of air or neutral pressure with good seals and recirculated filtering may avoid that.

\n

A note on filters: Activated carbon filters will not remove UFPs. HEPA filters may remove 3D printing UFPs.

\n
\n

Which Printer?

\n

As long as the uncertainty exists, I predict that as the market matures, filtering and enclosures will become more standard. At this point in time, the only enclosed AND HEPA filtered consumer-grade FDM printers I am aware of are the Up! Box and the Zortrax Inventure. There are a number of enclosed printers without filtering.

\n

As an alternative, at least one company has appeared with products targeted at those who are concerned about various safety aspects of 3d printing.

\n" }, { "Id": "3", "CreationDate": "2016-01-12T18:46:22.083", "Body": "

I know the minimum layer height will effect how detailed of an item you can print and the amount of time it takes to print something, but is it necessary to have an extremely low minimum layer height if you plan to print only larger objects?

\n", "Title": "How important is the minimum layer height on a 3d printer?", "Tags": "|quality|resolution|", "Answer": "

In short: no.

\n\n

Smaller layers will make your prints look nicer. That said, in my experience, most printers can handle roughly the same layer heights, no matter what the manufacturer says it can do. With a little tinkering, you can get any printer to do what you tell it to do.

\n\n

That said, for larger prints, you really won't want to lower the layer height too much. Typically, the quality difference you see in a print that's, say, .2 mm layer height (typical) vs .1mm layer height (typically high quality) is not that much different.

\n\n

Note that lowering your layer height increases the number of layers in your object, so it'll take longer to print and have more opportunities to fail along the way. For anything of significant size (150 mm + or so) it's really not worth the extra risk, I'd say.

\n" }, { "Id": "4", "CreationDate": "2016-01-12T18:50:55.973", "Body": "

Plastic is used in 3D FDM/FFF printing partly because it had a wide temperature range for its glass state - where it can be flowed with some force, but won't flow due only to gravity.

\n\n

Most metals have a very narrow, or non-existant, glass state. They transition from solid to liquid with almost no flowable-but-not-liquid state.

\n\n

Are there any metals or alloys that display a glass transition state?

\n", "Title": "Are there any metals that exhibit a large glass state?", "Tags": "|fdm|material|print-material|metal-parts|", "Answer": "

A few things are required for effective extrusion-style 3d printing materials:

\n\n\n\n

Liquid metals tend to have a conflict between \"Staying where you put it\" and \"bonding with the previous layer.\" In order for deposited metal to fully bond, the interface material needs to reach the melting point so a true fusion weld occurs. And in order to supply enough heat to remelt the interface without an additional heat source like an arc, the deposited molten metal needs to be very hot. So it will tend to run while it cools. High density and high heat capacity makes it run fast and cool slowly.

\n\n

Pretty much every DIY metal 3d print (such as made by wire-feed MIG welders) ends up looking something like this:

\n\n

\"enter\nhttps://3dprint.com/29944/diy-metal-printing-garage/

\n\n

In comparison, polymers have long molecular chains that allow them to \"diffusion weld\" and adhere WITHOUT fully remelting the interface. Molten liquid plastic will stick to solid plastic quite effectively. The interface only needs to get hot enough for appreciable diffusion to intertwine the molecular chains. This will occur between the glass point and melting point, without true fusion occurring. So you can print molten plastic at a temperature where it will stay in place long enough to harden, and still get good bonding.

\n\n

Metals also tend to be very stiff, which encourages warping. It is difficult to build a heated environment of sufficient temperature to properly stress-relieve the thermal contraction stress as the print progresses, whereas with plastic a heated build plate and warm enclosure can permit warping stresses to start relaxing as the print progresses.

\n\n

It is possible to \"FDM-style\" 3d print filament/wire made of metal alloys that have a wide range between solidus and liquidus. It has been done using solder and similar alloys. However, between the warping stresses, poor layer bonding from inadequate interface re-melting, and use of soft low-melting alloys, the resulting printed parts will usually end up being weaker than if they had simply been printed in a strong plastic. For example, PEEK is nearly as strong as aluminum, and carbon fiber or fiberglass composite plastics can exceed metals on various performance metrics. So what's the point of printing with weak, brittle metal alloys?

\n\n

Over the years, lots of people have tried FDM-style metal printing, but no one has found it worthwhile to pursue in the long run. More typical DIY metal printing approaches like 3D MIG welding following by cleanup machining will produce better results.

\n" }, { "Id": "5", "CreationDate": "2016-01-12T18:53:53.623", "Body": "

What are the main differences when using ABS over PLA and vice versa?

\n", "Title": "How is PLA different from ABS material?", "Tags": "|filament|abs|fdm|pla|", "Answer": "

Basic facts and tips for using and choosing ABS and PLA materials:

\n\n

Made from:

\n\n
ABS: Petroleum based\n\nPLA: Corn or other plants\n
\n\n

Smell:

\n\n
ABS: smell of hot plastic, \n\nPLA: gives off a smell similar to a semi-sweet cooking oil. Less intense smell than ABS. \n
\n\n

Part Accuracy:

\n\n

Both PLA and ABS are capable of building dimensionally accurate parts. However, there are characteristics that we need to memorized.\nABS: will be curling upwards of the surface in direct contact with the 3D printer\u2019s print bed. for fine and delicate features on parts involving sharp corners, such as gears, there will often be a slight rounding of the corner. \nPLA: much less part warping. but it undergoes more of a phase-change when heated and becomes much more liquid. The increased flow can also lead to stronger binding between layer.

\n\n
Recycling:\n
\n\n

ABS: Recycling number 7, most local recycling programs do not accept ABS. It generally being recycled into plastic lumber. \nPLA: it biodegrades, however the process takes a long time when outside of an industrial composting facility\nWhy we provide a vacuum sealed package for ABS and PLA products? \nlong term exposure to a humid environment without sealed package may result in detrimental effects, both to the printing process and to the quality of finished parts.

\n\n

ABS: will tend to bubble and spurt from the tip of the nozzle when printing, part accuracy, strength are reduced. Raises the risk of clogging the nozzle. ABS can be easily dried by using dry or hot air.\nPLA: bubbles or sporting at the nozzle, also discoloration and a reduction in 3D printed part are found when in long term moisture environment. \nImproving the printing outcomes \uff08Blue tape and Kapton tape pictures)\nPLA: printing temperature approximately: 190\u00b0C - 240\u00b0C\nABS: printing temperature approximately: 225\u00b0C - 250\u00b0C (recommend to use a heated print bed)

\n\n

A good first layer adhesion is of the utmost importance in obtaining the best results for the prints.

\n\n

Using Blue Tape or Kapton Tape.\nHairspray the print bed.\nTo attain optimal result for the prints, you need to consider variables such as nozzel diameter, printing speed, and layer height.\nSummary: \nABS: \nA preferred filament for the prints with engineering and professional purposes with its strength, machinability, flexibility, and higher temperature resistance. \nThe bad plastic smell is due to the petroleum based origin. \nRequired a heated print bed to attain ABS printing reliability. \nPLA:\nWide range of available colors, provide translucencies and glossy feel of the prints. \nPlant based and semi-sweet corn like smelling are appreciated over ABS.\nWhen properly cooled, PLA seems to perform higher maximum printing speeds, and sharper printed corners. Combining this with low warping on parts make it a popular plastic for household uses, toys, hobbyists, and educational purposes. \nto know more please visit abs vs pla

\n" }, { "Id": "6", "CreationDate": "2016-01-12T18:57:13.350", "Body": "

My MakerBot printer supports only two filaments at the same time.

\n\n

What are techniques to print objects with more than two colors for one object?

\n", "Title": "Multi-color printing with desktop 3D printer?", "Tags": "|filament|makerbot|dual-nozzle|color|", "Answer": "

The standard option is to change out filaments at certain times during the print. Software to add these pauses to your print automatically are around, with one (ColorPrint). This method works if you only want to change at a certain Z height, and not intermix colors on the same levels.

\n\n

The other newer development that works for many printers (this works for single nozzle printers as well) are getting Y type adapters that allow more than 1 filament to be fed into a single extruder, and use a rapid retraction to pull the filament back out past the Y connector to allow the next color to be fed into the nozzle. Several companies are putting products like this out on the market at this time. An example of this can be seen at this page.

\n\n

As mentioned in other answers, Mosaic Pallette is an option to drive more complicated multi-color prints, but is a rather expensive option at $999.

\n\n

The E3d Kraken might be a possible hotend upgrade to a Makerbot, although it would involve a fair amount of work.

\n" }, { "Id": "11", "CreationDate": "2016-01-12T19:07:53.343", "Body": "

The surfaces of my printed parts using PLA plastic look rough and uneven.

\n\n

Would changing filament to a better one make any difference?

\n\n

If not, what kind of methods can I use to achieve a smoother finish for my for 3D-printed objects?

\n", "Title": "How do I give 3D-printed parts in PLA a shiny smooth finish?", "Tags": "|filament|pla|post-processing|surface|", "Answer": "

For a cheap method to smooth out your prints, you can use clear nail polish. It will act as a resin-like coat.

\n" }, { "Id": "13", "CreationDate": "2016-01-12T19:18:57.943", "Body": "

With an ABS or PLA extrusion 3D printer, are there any potentially negative quality differences that could occur if I try to print at a higher resolution?

\n\n

I am not concerned about print time as the equipment is not under high demand. I am, however, worried the device may be more prone to fracture, likely to have defects, or have other issues I cannot currently imagine.

\n", "Title": "Resolution Drawbacks", "Tags": "|quality|resolution|fdm|", "Answer": "

It's also worth noting that the ratio of nozzle diameter to layer height affects strength. The layer height is typically set slightly smaller than the nozzle diameter, so the nozzle \"squeezes\" the new plastic onto the previous layer. This is especially important for the first layer, because it affects how well the object sticks to the bed; but it also affects inter-layer strength.

\n" }, { "Id": "14", "CreationDate": "2016-01-12T19:22:14.277", "Body": "

I would like to print parts (e.g. jewellery) for use which I don't want to look or feel like a plastic, but metal-like, so briefly people won't see much difference.

\n\n

Are there any specific type of home-printers that can achieve that? Or it's rather kind of filament that you should use?

\n", "Title": "How to print metal-like parts?", "Tags": "|filament|metal-parts|", "Answer": "

You have several options:

\n\n
    \n
  1. Printing with filaments made up of plastic and metal powder mix. Bronzefill is one example.

    \n\n

    While most may argue it doesn't look much like bronze or copper and rather clay, it can made to by some automatic polishing methods:

    \n\n\n\n

    Some don't like the idea of having a plastic and metal powder mixture and they say if it's not pure bronze/some other alloy it might as well be painted. Which is the second option:

  2. \n
  3. Spray painting an ABS or PLA print with metal color spray paint. You might need to sand or acetone bath the print first for the paint to be applied evenly.

  4. \n
  5. Metal plating can be done for more realstic texture.

    \n\n\n\n

    The 3d print should again be sanded/polished first.

  6. \n
  7. The first method of getting actual metal object from a plastic object: Printing with a metal clay, then firing it in a kiln. You'll need an appropriate extruder which can extrude paste instead of filament:

    \n\n\n\n

    There are 3D printers that can do that officially.

    \n\n\n\n

    You'll need a kiln...

  8. \n
  9. Second method of getting actual metal object from a plastic object, via metal casting (\"lost polymer casting\"). From all the above options this is the most dangerous if you're not experienced, skilled and careful.

    \n\n
  10. \n
\n" }, { "Id": "16", "CreationDate": "2016-01-12T19:25:52.270", "Body": "

I am aware of several \"clear\" filaments for a ABS or PLA printer. They, however, have a cloudy or frosted glass appearance. I do not believe this is possible to eliminate but I believe it can be reduced.

\n\n

Are there effective ways to make a print have a more transparent appearance?

\n", "Title": "Are there any ways to make a 3D print transparent?", "Tags": "|post-processing|", "Answer": "

It depends on a lot of factors, type of plastic, whether the parts need to be strong, can you use a vase print, etc. Here's a few thoughts.

\n\n

PLA - The brand of PLA makes a big difference, some can be printed very clear, some can't. Most of the transparent PLAs I've used print much more clear at around 240\u00b0C.

\n\n

ABS - I've seen some pretty impressive clear parts printed as a single layer shell in ABS and then vapor smoothed. I tend to find ABS more translucent and less transparent though.

\n\n

PETG - Again the specific PETG you use matters, but I haven't seen nearly as much variation as with PLA. I'm not sure how much temperature matters, but if it's too hot you get bubbles which will decrease clarity.

\n\n

Thin Wall Prints - I don't have much experience here, but the Smooth On XTC-3D or vapor smoothing seem to be effective.

\n\n

Solid Prints\nIf I want transparency, I usually print it at 100% infill (should be a real 100%, too much overextrusion or underextrusion will decrease transparency). Printing slower and with less cooling sometimes helps. It's easier to get the infill solid with a direct drive extruder, I couldn't get decent results with a long bowden tube (a short bowden tube works fine).

\n\n

Here's a page where I tested 10 transparent filaments, the printed samples are 2mm thick, 100% infill - http://thrinter.com/10-transparent-filaments. Those samples are all overextruded a bit, you can get better results if you dial in the extrusion precisely, but that's hard to get right, and the optimal settings may change slightly depending on the part geometry. Larger nozzles and thicker layers may help to, but I haven't experimented much with that as there are significant downsides to that approach.

\n" }, { "Id": "19", "CreationDate": "2016-01-12T19:28:09.363", "Body": "

My printed parts consist rafts, supports and other extraneous filament when printing with ABS or PLA.

\n\n

What are efficient general techniques of removing them?

\n", "Title": "How to easily get rid of rafts and support structures?", "Tags": "|support-structures|post-processing|rafts|", "Answer": "

I usually use a chisel or a flat-head screwdriver to easily remove the bottom plate that the printer auto-generates. I would also suggest using something like wire cutters or some mini pliers to pull them off.

\n" }, { "Id": "20", "CreationDate": "2016-01-12T19:30:45.060", "Body": "

I would like to understand the differences between rafts, skirts and brims. They appear in the software which I'm using to edit my 3D objects.

\n\n

Can anybody elaborate what are these and what are the main differences between them?

\n", "Title": "What are main differences between rafts, skirts and brims?", "Tags": "|rafts|brims|", "Answer": "

Rafts:

\n\n

Rafts are a few layers of plastic a placed on the printing surface before the object is printed. If non water dissoluble filament is used, a bit of an air gap will be placed between the raft and the print itself so it can be removed easily. A raft can help a print stick to the printing surface as it normally has a larger surface area than the bottom of the print. It also allows the bottom layers of the print to contact another layer of plastic so there is less spreading compared to printing on the printing bed itself.

\n\n

\"A

\n\n

Skirts:

\n\n

Skirts are loops of plastic that are extruded around the object that is being printed. On the first layer this allows the filament to get pushed through the extruder so that a steady flow can be established. A multiple layer skirt can be used to create a blockage around the print so there is less air movement on the print and the print can cool slower.

\n\n

\"A

\n\n

Brims:

\n\n

A brim is a layer of plastic that is placed on the first layer of the print go allow for better adhesion to the printing surface. It does not extend under the print as a raft does, but only goes from the edge of the print to a set distance away.

\n\n

\"A

\n\n

Credits: Images from Slic3r

\n" }, { "Id": "22", "CreationDate": "2016-01-12T19:33:04.263", "Body": "

I've acquired all the parts to build a Reprap Prusa i3 rework, the only missing part is the frame.

\n\n

I'm in doubt between a MDF cut (cheaper) or acrylic (more expensive), of course a cheaper one is my preferred option until I see any disadvantage on making it of wood.

\n\n

I thought about variables like heat and humidity and the possibility of expansion/contraction of the frame, is this a valid concern? Will I have more precision buying the acrylic one or is it irrelevant?

\n", "Title": "Is the weather a problem for MDF frames?", "Tags": "|reprap|prusa-i3-rework|", "Answer": "

I want to add the following to the already direct and very good answer of @Dani Epstein. It does not answer your question, but hopefully may help many people who are reading the question when choosing between the two materials.

\n\n

Acrylic is less stable and will probably wear off faster than a well-built MDF frame for a 3D printer! I will address the points separately.

\n\n
    \n
  1. An MDF frame has much(!) more mass than acrylic, which will reduce vibrations immediately. Almost more important, a box frame, as typical for the Prusa i3 for example, stabilizes itself intrinsically more by its 'redundant wall' design (don't beat me up over the wording here, I didn't find a better way to describe the property that results from the towers.)
    \nCompare these two images taken from reprap wiki Prusa i3:\n\"boxframe\n\"metal\nYou will probably not find an acrylic frame with this design. Shear stress as produced by the moving carriages, as well as z-wobble from the rods should theoretically be much less.
  2. \n
  3. The acrylic might not wear by 'natural causes' since it is not a biologic material as wood is, but it will wear much faster due to handling the material, maintenance and human errors. The material is very brittle. You can find lots of reports on the web where the frame cracked or broke during setup. Here is my favorite example from someone with a lot of experience (1:20-2:30): https://www.youtube.com/watch?v=wkkVk8c8XoU
  4. \n
  5. Acrylic is a bad choice in terms of its mechanical properties: It is soft (I found low tensile strength as a number to compare in the given database), has lots of thermal expansion (5 times higher than stainless steel) and melts at 160 C (so beware of anything touching your hotend). Feel free to compare it to other materials here (no MDF or plywood in that database, though ;-) ): http://www.goodfellow.com/E/Polymethylmethacrylate.html
  6. \n
\n\n

This altogether makes acrylic a very bad choice for a 3d printer frame material in my opinion. Choose MDF or plywood, you can make it look cool by painting it, too.

\n" }, { "Id": "28", "CreationDate": "2016-01-12T19:39:43.650", "Body": "

I've seen article about World's First 3D Printed Bike.

\n\n

What kind of printer is required to do that, briefly how long it takes and how much does it cost? Is this even achievable at home? Doesn't need to be that specific one.

\n", "Title": "How to 3D print a bike?", "Tags": "|cost|estimation|", "Answer": "

You will need a laser sintering or lasercusing printer, which will not be something you can buy for home use. They are horribly expensive.

\n\n

You could always print this in PLA or ABS and cast it in aluminium. Then you have to find a safe method to test the result, because casting is not quite as simple as it looks and the bike could be seriously dangerous.

\n" }, { "Id": "33", "CreationDate": "2016-01-12T19:52:25.430", "Body": "

There is a 3D desktop printer RepRap which can print most of its own components.

\n\n

Assuming each printed printer will print the next one and so on. Are there any limitation how many times this can be achieved?

\n\n

For example somebody printed for me printer and I do the same for my friends and they do the same for theirs. Can this go forever (since 3D model stays the same), or there are any serious side-effects/disadvantages of doing that continuously?

\n", "Title": "How many times printed printer can print it-self?", "Tags": "|reprap|desktop-printer|replicating-printers|", "Answer": "

The reprap printers have often been compared to plants, providing fruits to you and the possibility to reproduce themselves.

\n\n

This analogy holds in both good and bad ways. Any life form can reproduce itself only so often without artefacts (mutations) being introduced.

\n\n

It takes a bit of skill to build, configure and run a reprap printer. While the parts can be passed on, that doesn't necessarily hold for gained experience.\nChances are that the parts your printer produces are not as good as those that you have received to build the printer. At least not until you caught up on the learning curve.

\n\n

A reprap has a lot other parts that are not printed and can vary in quality independently from the printed parts. It makes a difference what steel rods are used, what driver circuit for the motors, etc. If you give printed parts away that are as good as those that you received yourself, the added parts are not necessarily as good as your.

\n\n
\n\n

My recommendation would be that you and your friends get printer parts from that somebody and you build your printers together. While giving parts to others is a great thing, building 3D printers together with friends is greater.

\n" }, { "Id": "38", "CreationDate": "2016-01-12T19:59:15.953", "Body": "

I have a few kg of 3 mm filament when I only have use for 1.75 mm.

\n\n

How can I make 1.75 mm from 3 mm filament?

\n", "Title": "Conversion of 3 mm ABS filament to 1.75 mm", "Tags": "|filament|filament-production|", "Answer": "

You could build a machine that has a nozzle with 3 mm input hole and 1.75 mm output hole, based on some designs for filament making machines. Or you could just cut the filament into little peaces and use them instead of the granulate in an original filament making machines.

\n\n

There are some open designs for such machines you can build, or you could buy one, such as Filabot.

\n\n

However, as mentioned by kaine, this is very unlikely to be worth the cost/effort. Best option for you is to try to sell the 3 mm filament to someone who has a use for it, take the money and buy some 1.75 mm filament instead.

\n" }, { "Id": "48", "CreationDate": "2016-01-12T20:16:01.823", "Body": "

I am printing a print using PLA on a Prusa i3 printer and an MK8 extruder, at 210 degrees celsius, 60 mm/sec, sliced with slic3r. The print consists of a base, with 4 tower-like projections that then join with a near-vertical overhang slope that isn't posing a problem for my printer.

\n\n

However, even before the overhang begins, I am getting large amounts of strings as the extruder head jumps between the four towers in the print, leading to a \"spiderweb\" effect between them. How can I deal with these strings, and are they a warning that there might be something amiss with my printer, or possible other failures in other parts of the print?

\n", "Title": "What are the reasons for my 3D prints having large numbers of strings between parts of a layer?", "Tags": "|quality|fdm|pla|extrusion|", "Answer": "

Here's just a few of the things you might want to look into.

\n\n\n\n

There are other things too, the extruder design makes a big difference with longer bowden tubes being more challenging to tune. The order in which the layers are printed can make a difference too in some more specialized cases.

\n" }, { "Id": "49", "CreationDate": "2016-01-12T20:16:38.260", "Body": "

When using thermoplastic-filament, this can be potentially hazardous, since constant printing can emit hazardous fumes and odors that may be emitted by heating the plastics.

\n\n

I understand it normally should be used in well ventilated areas. However I would like to use it heavily in the basement which is not well ventilated.

\n\n

Are there any practical methods of limiting such exposure? For example locking it in some special box, covering it or suck the odors? Would that help?

\n\n

Do you have any experience doing so?

\n", "Title": "Are there any methods of limiting exposure of hazardous fumes and odors emitted by heating the plastics?", "Tags": "|filament|desktop-printer|safety|", "Answer": "

Using negative pressure ventilation and a suitable organic filter will limit your exposure to toxic compounds, but won't completely remove them from your environment.

\n\n

Enclose your printer in as air-tight a box as you can manage, then use a fan to suck air out of the box. This negative pressure will ensure that any leaks in the box will not allow gasses to escape.

\n\n

The air should be blown through an organic filter. This might be done with face mask filters for painting, for instance.

\n\n

Additional filtering may be done depending on the compounds you expect the printer to produce, but the ideal situation is to set up a ventilation system to the outdoors where the products cannot concentrate and harm anyone.

\n\n

Another poorer option is to simply wear a proper organic filter mask yourself. It's not as good, since the compounds can spread through your residence or remain in the air and be inhaled when you aren't printing.

\n" }, { "Id": "59", "CreationDate": "2016-01-12T20:37:19.573", "Body": "

Is it possible to re-use ABS or PLA filament material from printed parts?

\n\n

If so, what is the techniques to reform it?

\n", "Title": "How to recycle filament material from printed parts?", "Tags": "|pla|abs|recycling|filament-production|", "Answer": "

If you're more interested in the recycling and reuse aspect than the re-print aspect, you could melt all the scrap filament onto a cookie sheet or into a bar (like in a bread ban). You could then manually work the material, or use a CNC machine to carve out your next thing.

\n

This Youtube video refers to HDPE, but the same concept will apply to other thermoplastics:

\n

\r\n \r\n

\n

This Youtube video uses a cookie sheet and failed prints:

\n

\r\n \r\n

\n" }, { "Id": "61", "CreationDate": "2016-01-12T20:40:43.047", "Body": "

I print ABS on a LulzBot Taz 5 and frequently have issues with the corners of objects lifting off the bed.

\n\n

My extruder is at 230 \u00b0C and the bed is at 90 \u00b0C for the first layer and 100 \u00b0C for the rest of the layers.

\n\n

I have experimented with using ABS slurry (ABS + acetone) on the bed for increased adhesion, building a foam enclosure for the printer, and varying the fan speed. I have noticed the problem is more common the taller the parts are and the sharper the corner is.

\n\n

Adding ABS slurry helped for smaller parts (less than an inch tall) but with my more recent larger parts the adhesion to the bed was so good that the corners of the part lifting actually peeled the PEI tape off of the bed.

\n\n

I have tried using both a skirt and a brim with no change. The skirt stays on the bed, the brim gets pulled up with the corner.

\n", "Title": "Why do the corners of my ABS object lift off the bed?", "Tags": "|extruder|abs|heated-bed|lulzbot|", "Answer": "

This post is meant to share experience with products that make your products stick to the bed. This is not meant to be a promotion of the products! In other answers I've seen answers hinting to Elmer's glue sticks.

\n\n

I have been printing directly onto the aluminium heatbed from day 2 of the i3 Prusa clone (Anet A8) printer and the glass beds of the Ultimaker 3 Extended and my HyperCube Evolution using a PVA based spray (3DLAC, but there are more products with similar effects, e.g. glue sticks). This sticks so well that PLA and PETG can only be removed after cooling down the bed completely. For ABS you could use DimaFix which can be used for ABS as it increases grip with increasing temperature where 3DLAC looses grip over 80 \u00b0C (according to theory). After trying DimaFix on high temperature beds for printing POM filament (very tough to get this to stick as this is bearing material!) I found that prints stick better with 3DLAC on glass.

\n" }, { "Id": "65", "CreationDate": "2016-01-12T20:49:28.873", "Body": "

Taken from the answer provided by @EricJohnson,

\n\n

When should I use a raft, and when should I use a brim? What advantages does each have over the other?

\n\n

Raft\n\"enter

\n\n

Brim\n\"enter

\n", "Title": "When should I use a raft, when should I use a brim?", "Tags": "|rafts|brims|", "Answer": "

I have been favoring brims recently; I am tired of the rafts becoming an integral part of my print, impossible to remove.

\n" }, { "Id": "70", "CreationDate": "2016-01-12T21:00:36.337", "Body": "

My print is warping and I suspect it is because of uneven cooling.

\n\n

How can ensure my print cools evenly? Should I put a fan on the bed? Will no fan and a tall skirt help?

\n\n

Or are my edges curling up for another reason?

\n", "Title": "My print is warping: how can I ensure that it cools evenly?", "Tags": "|adhesion|warping|cooling|", "Answer": "

Without more information we cannot address what is causing your edges to turn up. This can depend on the model being printed, the process, the material, the bed, and the method used to bond it to the bed.

\n\n

As far as the initial question, a fan will most certainly ensure uneven cooling.

\n\n

The middle of the print is, and will remain, the warmest. If you want the print to cool evenly, you'll need an enclosure to block out drafts and air currents, and you'll need to bring the heated bed temperature and enclosure temperature down slowly over a long period of time.

\n" }, { "Id": "78", "CreationDate": "2016-01-12T21:30:33.083", "Body": "

I made a test print for a small gear (~ 1.5 inches in diameter) a few months ago, with a hole through the center. On the first try, the filament (ABS) fused to the print bed, meaning that I had to spend ten minutes scraping off material to loosen it. One solution to this is to use painter's tape spread across the print bed.

\n\n

This yielded a good print during the next run. The problem with this method was that some of the tape subsequently fused to the backside of the gear; it was so tight that I had to discard the prototype. Multiple varieties of tape made no difference.

\n\n

Is there a way to continue using this tape without having it fuse to the filament?

\n", "Title": "How can I stop my print bed tape from sticking to the filament?", "Tags": "|abs|filament|maintenance|", "Answer": "

One thing that I do to keep it from sticking to the tape. Is to add glue from a glue stick down on the building pad in an even coat on top of the tape.

\n" }, { "Id": "81", "CreationDate": "2016-01-12T21:50:45.197", "Body": "

Acetone can be used to smooth ABS prints. What safety precautions should be taken during its use?

\n", "Title": "Safety precautions when using acetone", "Tags": "|abs|acetone|safety|", "Answer": "

Safety Advice for Acetone Handling/Usage

\n\n
\n\n

Please remember that you handle acetone at your own risk! By using this advice you agree to hold me harmless and not sue me as a result of using these instructions/advice. Remember that I am not a professional chemist or a lawyer (this isn't professional or legal advice)!

\n\n
\n\n

I encourage you to research safety precautions and risks on your own to build a personal body of knowledge. The most effective safety precaution available to you is knowledge; the brain is the most important piece of safety equipment! This listing of advice for acetone handling is generated from the highlights of a Material Safety Data Sheet (MSDS) for acetone. After reading this advice you should read an MSDS for acetone directly. It would actually be an excellent idea to find and read the MSDS for all the chemicals you use during 3D printing.

\n\n

Safety Equipment

\n\n\n\n

Control Your Environment

\n\n\n\n

Acetone Chemistry

\n\n\n\n

Risk Analysis

\n\n\n" }, { "Id": "83", "CreationDate": "2016-01-12T21:55:29.397", "Body": "

I would like to print multiple parts continuously (non-interactively), so\nI can leave the printer alone for a longer time. So after finish, parts could be moved somehow out from the printing area, so the next can start.

\n\n

Are there any methods of achieving that with standard desktop printers without having to use multiple printers?

\n", "Title": "How to automate printing of multiple parts continuously?", "Tags": "|desktop-printer|automation|", "Answer": "

I've recently seen a video of this being done successfully with almost no hardware modifications to the printer.

\n

Unfortunately I can't find the video itself any more, but the basic idea is to use the print head/gantry to knock the finished prints off the table, likely by adding some custom commands to the end of the sliced G-code. After the print is finished, the bed moves to the Y endstop (this was done on a bedslinger printer), the head is lowered to about half of the parts' height, then the bed is slowly moved so that the part crashes into the head, gets detached and knocked off the table, then the head is lowered even further and sweeps the part away.

\n

The only mechanical modification, namely a sheet of paper stuck to the table's edge, ensures that the part will roll clear of the bed rails, and pushes the pile of finished parts further from the printer once it's off.\nIn the case of that video there were three parts being printed at a time, so the motion was repeated for each of them, even though most of the time all three got successfully detached by the gantry.

\n

This approach is likely far from universal, and probably only works with parts that are tall enough and have a small enough footprint to easily detach from the bed; I certainly wouldn't try it if the parts have really good bed adhesion and require more than a slight force to pop off. But it looks like if the requirements are satisfied it should be quite effective.

\n

Also it's likely to be problematic on printers with box frames where it's the bed that moves in the Z direction, as the printed parts are likely to get trapped between the bed, print head and printer frame. CoreXY designs where the bed is static and the head moves in all three axes should be fine, however.

\n" }, { "Id": "84", "CreationDate": "2016-01-12T21:57:01.307", "Body": "

For standard ABS and PLA filament, most distributors recommend storing the filament in an airtight bag. Does not doing this actually make print quality worse? I have left mine in the open for a year and have had no noticeable problems.

\n", "Title": "Does filament have to be stored in an airtight environment?", "Tags": "|filament|pla|abs|storage|", "Answer": "

In most cases, you should be fine with ABS or PLA out of an airtight container. If you're worried about it, throw a few desiccant packets where you store your filament.

\n\n

However, some specialty filaments should be stored in an airtight container. PVA is notorious for absorbing the ambient humidity around it. When it's heated, the water it has absorbed starts to form bubbles, completely messing up the extrusion.

\n\n

In short, some specialty filaments, definitely. With others, it isn't necessary, but it can't hurt.

\n" }, { "Id": "92", "CreationDate": "2016-01-12T22:13:01.437", "Body": "

I'm considering purchasing Filabot or some similar filament maker.

\n\n

What kind of plastic can I use to produce my own filament? Can I use any type of plastic or just specific printable filaments? Can I also mix different types of filament together or only one type at a time?

\n", "Title": "What type of plastic can be used to produce your own filament?", "Tags": "|filament-production|filament-quality|", "Answer": "

Filament manufacturers (for example, Colorfabb) also sell pellets, price per kilo would be about 10 times less than the same plastic in filament form.

\n\n

Out of household garbage only ABS can be easily extruded into filament with Filabot-grade machine. PET AKA plastic bottles looks promising as well.

\n\n

You can also use certain products \"off-label\". For example, string trimmer string (line) is simply a nylon filament and can be used as-is in modern diameter-agnostic hot end.

\n" }, { "Id": "103", "CreationDate": "2016-01-12T23:39:20.770", "Body": "

Is there any regulation against a 3D printed weapon in the United States or Europe?

\n\n

Some time ago, I saw an article in Israel where they had one that was not detected by the x-ray and was fully functional!

\n", "Title": "Is it legal to make a fire weapon with a 3D printer?", "Tags": "|legal|", "Answer": "

Sort of.

\n\n

Fully plastic guns are banned in the United States by the Undetectable Firearms Act, because they can pass unseen through metal detectors - a huge problem. One way around this is to insert a slip of metal into the gun, thereby making it detectable by metal detectors. 3D-printed guns made of metal - were that possible - would not violate the law.

\n\n

The UFA was originally enacted in 1988, extended from 2003 to 2013, and then again until 2023. It may seem a bit unnecessary, as all-plastic guns, like the famous Liberator have structural issues that make them extremely difficult to fire when compared to a normal all-metal gun. However, then can be used.

\n\n

In Europe, gun laws vary widely by country. The European Firearms Directive is the main international agreement on gun laws, but its application varies, and it does not directly address 3D-printed guns.

\n\n

I'm not aware of a country that has an explicit law banning 3D-printed guns. The only countries that ban them either unintentionally cover them or ban guns altogether.

\n" }, { "Id": "107", "CreationDate": "2016-01-13T00:41:32.907", "Body": "

I have a 3d printer that uses ABS filament. The software I use will generate vertical supports for my objects before printing that can be easily broken off after they have been used during print to hold sharp angles up that would normally fall.

\n\n

After breaking off the stints, the print is far from smooth. Is there a material that is best suited for \"sanding\" down prints without damaging the print?

\n", "Title": "What can I use to \"sand\" my ABS prints?", "Tags": "|abs|post-processing|support-structures|support-material|surface|", "Answer": "

I normally deburr with a deburring tool:

\n\n

\"deburring

\n\n

Then I file if needed, then I hit it with a scotch brite pad:

\n\n

\"scotch

\n\n

And then I give it a quick pass with a heat gun to darken all the areas that have turned lighter from the abrasion.

\n\n

Video showing heat gun (but not scotch brite): https://www.youtube.com/watch?v=0Aj9WCabPgw

\n" }, { "Id": "132", "CreationDate": "2016-01-13T02:13:54.357", "Body": "

When you add a raft in Slic3r, the first layer of the raft prints at the first layer speed. After the raft is finished, the first layer of the print prints at the standard speed. How can I make the first layer of the actual print slow down to the first layer speed?

\n", "Title": "How can I set the 1st layer after a raft in Slic3r to print at 1st layer speed", "Tags": "|speed|slic3r|", "Answer": "

This is still work in progress, and here is what I have so far, but first:

\n\n

A useful alternative for similar problems:

\n\n

A problem very similar to this would be to use different settings for different parts of a model in Slic3r. For most settings, this can be achieved through modifier meshes.

\n\n

Post processing scripts:

\n\n

As far as I know, Slic3r does not give you the option of setting the speed of the first layer after a raft directly, but they do allow you to run post processing scripts; that is, to automatically run a set of operations - programmed by you - on the g-code output.

\n\n

Although far from trivial, you can in theory make a program that runs through the output g-code, adjusts the settings to your preference, and then saves it again at the target destination.

\n\n

Tuning overall printer speed through g-code:

\n\n

As it turns out, there is a simple g-code command that sets the overall speed of your printer's operation:

\n\n
M220 S[some number]     ; see the link above for compatible firmware\n
\n\n

A newsletter from Reprapwold explains that:

\n\n
\n

For example M220 S50 will reduce the speed to 50%\n of the original sliced G-code. If you want to hurry your print to the finish in time\n for dinner, use M220 S200, to print twice as fast (200%)

\n
\n\n

In other words, just like some printers allow you the change speed mid-print, you can use the M220 command to override the current speed used, either through a user interface such as PrintRun, or by fiddling with the original g-code itself.

\n\n

Manipulating the g-code output to adjust speed settings:

\n\n

The easiest way to achieve our goal would be to manually manipulate the output g-code file through a text editor, and insert our M220 command in appropriate places:

\n\n\n\n

In order to do this, though, we need to be able to distinguish these two points in the g-code output.

\n\n

Distinguishing insertion points:

\n\n

Slic3r offers a setting under Print Settings -> Output options -> Verbose G-code that - when enabled - inserts written comments all throughout the g-code files generated.

\n\n

If one inspects a g-code file outputted for a model with raft, one will find the comment:

\n\n
; move to first perimeter point      <- lets call this A\n
\n\n

and

\n\n
; move to next layer (x)             <- lets call this B\n
\n\n

littered several places throughout the gcode.

\n\n

It is under my impression that the first occurrence of comment A happens right after the raft is finished, and before the actual model is being printed, while the first occurrence of comment B succeeding comment A can be used to set the speed back to normal.

\n\n

It should be noted, however, that the comments in the output g-code does not seem fully consistent, and I would therefore not recommend anyone to automate this logic into a script without possibly finding other, more reliable breakpoints, and thoroughly verify these through several different models.

\n\n

I have not looked into the details of writing an automatic script for this task as of yet.

\n" }, { "Id": "134", "CreationDate": "2016-01-13T02:39:47.150", "Body": "

Assuming you have a high quality printer with a fast processor, will you see a noticeable improvement by upgrading from 16X/32X microstepping drivers to 64X/128X microstepping drivers? (e.g. smoother surface finish). In what ways do they perform differently from the more common 16X or 32X stepper drivers. I'm thinking the RAPS128, Silencioso, and Trinamic drivers vs the DRV8825, A4988 and A4988.

\n", "Title": "What are the benefits to using 128X microstepping drivers on the X and Y axis of a FFF printer?", "Tags": "|microstepping|stepper|", "Answer": "

You will likely not see a noticeable improvement by upgrading from a 16x or 32x to a 64x or 128x microstepping driver. Depending on the motors you're driving and the size of the load you could actually see a decrease in quality.

\n\n

Although microstepping increases theoretical resolution it does not necessarily increase accuracy. The reason is that microstepping significantly limits the incremental torque of the motor. This means that you may ask for a step but not get one because the torque of the step won't be enough to actually turn the shaft.

\n\n

As an example: a motor running in full steps will have 100% of its rated holding torque. Moving to 16 microsteps/full steps drops this to ~10%, 128 drops it to ~1%.

\n\n

The practical effect of this is that in high torque situations (such as printing at fast speeds) the motor may end up skipping some of the steps. In this way the increase in resolution can actually lead to a decrease in accuracy (smaller steps but they may not actually be taken).

\n\n

A relevant calculation to do would be to work out what the different number of microsteps to full steps works out to in terms of horizontal, vertical, or whatever movement the motor drives. You can do this by measuring how far the stepper moves said surface in one revolution provided you know the number of steps it takes per revolution.

\n\n

Example:

\n\n

With no microstepping: 1 turn/inch * 200 steps/turn = 200 steps/inch or .005 inch/step (127 micron resolution)

\n\n

With 16x microstepping: 16 * 200 steps/inch or .0003 inch / step (8 micron resolution)

\n\n

In this example 128x microstepping would be absolutely foolish. Every situation is different and you should use this information to make a decision based on your setup. Many manufacturers have recommendations on how far their motors can be microstepped.

\n" }, { "Id": "140", "CreationDate": "2016-01-13T04:14:31.923", "Body": "

I've had my printer for almost a year now.

\n\n

Is there something I should be doing to maintain the motors?

\n", "Title": "Do stepper motors require any maintenance?", "Tags": "|electronics|maintenance|mechanics|stepper|", "Answer": "

The stepper motor itself does not. You may want to inspect the motors for debris or dust.

\n\n

Depending on your configuration you may want to check on parts of your printer that connect to your stepper motor such as shaft couplings, pulleys, lead screws/threaded rods and belts. The stepper motor wires should occasionally be inspected for wear and strain.

\n" }, { "Id": "147", "CreationDate": "2016-01-13T05:44:21.367", "Body": "

What materials which are commonly used in 3D printing, are food-safe?

\n\n

Are there any certifications/grading process for such materials, which can help me with my cross-checking and selection?

\n\n
\n\n

I have been using an FDM printer.

\n", "Title": "Which are the food-safe materials and how do I recognize them?", "Tags": "|material|safety|food|", "Answer": "

I have looked at this a lot, both from the standpoint of my own use, and of selling items on Etsy.

\n\n

As far as I can determine, PLA and ABS are both generally safe.

\n\n

The FDA lists ABS and PLA as safe plastics for food contact, although some pigments and additives can bring their own problems. ABS is nit generally safe (per the FDA) for contact with alcohol. I don't know why.

\n\n

So, for my use, I make wine, beer, and cocktail containers from PLA, and coffee mugs from ABS.

\n\n

PETG softens too much with boiling water and does not work for coffee and tea mugs. I've tried. It fails.

\n\n

Be careful if you use acetone smoothing on ABS. The acetone enters the ABS, and even after a few days of ambient conditions, the plastic may contain enough acetone to create bubbles in the plastic when the acetone boils off in response to hot water. I had heavily smoothed this particular teacup. Perhaps if it was less exposed to acetone vapors, it may have let the disolved acetone escape faster.

\n\n

I have used non-smoothed ABS coffee mugs for months without problems.

\n\n

You will read about brass nozzles contaminating the print with lead. You will read about the ridges being bacterial breading grounds. This may be true.

\n\n

ABS still makes a fine coffee mug for personal use.

\n" }, { "Id": "149", "CreationDate": "2016-01-13T06:35:47.820", "Body": "

Suddenly, my printer has started producing prints that have a very pronounced layering. Normally, the alignment between layers is very good, and the prints look very smooth. Suddenly, the prints have become much worse and the layers are misaligned with respect to each other.

\n\n

\"enter

\n\n

The part on the left is my \"normal\" quality, while the part on the right show the deterioration. Here is another picture (in which the good part is on the right):

\n\n

\"enter

\n\n

The parts are both printed with 0.1mm layer height, and identical slicer settings/filament. I am printing on a custom-built FDM printer; the mechanism is roughly similar to that of an Ultimaker.

\n", "Title": "What can cause a sudden and dramatic loss in the inter-layer registration of my prints?", "Tags": "|fdm|abs|print-quality|", "Answer": "

It appears the heatbreak of my E3D nozzle had worked itself loose from the heatsink, allowing the nozzle to wobble around a bit. Because the nozzle was still tight against the heatbreak I didn't experience any issues with my hotend, but because the heatbreak was slightly loose the nozzle wasn't properly constrained and moving around a bit.

\n\n

A quick turn to tighten the heatsink back into the heatbreak was enough to fully resolve the issue. My prints are as smooth as ever now.

\n" }, { "Id": "150", "CreationDate": "2016-01-13T06:38:38.630", "Body": "

I am working on a robotics project and need to print some gears. These will probably by under a LOT of pressure. Which material/filament should I choose so that the gears don't wear off easily?

\n\n

PS: Newbie here...

\n\n

EDIT:\nAccording to my instuctor, it has to be some sort of plastic (not metal).\nIt also has to be lightweight...

\n", "Title": "Which are the strongest and most durable materials?", "Tags": "|filament|material|", "Answer": "

Filaments that are intended for making parts that require strange should be rated by the manufacturer for strength and flexibility.

\n\n

You need to calculate the required strength and then choose a material with higher rating

\n\n

For example the rating for filaments made by Taulman are at http://www.taulman3d.com/how-to-choose.html?m

\n" }, { "Id": "164", "CreationDate": "2016-01-13T12:25:55.097", "Body": "

Assuming I've 3D design (or I've created one) which looks very similar to Lego bricks, I am allowed to 3D print them for my personal use?

\n\n

Do I need to obtain some permission to do so, because of some patents? Or how does it work?

\n", "Title": "Can I print my own Lego bricks?", "Tags": "|legal|bricks|", "Answer": "

The patents that cover Lego bricks have expired, so you are free to print bricks using the same interlocking system. You are even allowed to offer such prints commercially.

\n\n

What is not allowed (and a violation of trademark law) is to call them \"Lego bricks\" or use Lego's logo. \"Compatible with Lego\" on the other hand, would be fine.

\n" }, { "Id": "168", "CreationDate": "2016-01-13T12:36:34.530", "Body": "

Assuming I've extracted 3D models from a game which I legally bought (such as StarCraft). I am allowed to 3D print them for personal use or give it to close friends?

\n\n

Is there any general rule, or this suppose to be specified in the license? If so, which section/clause potentially can prevent me from doing that? Or I need to contact the company who owns the game to obtain the permission?

\n", "Title": "I've exported 3D models from the game, can I print them?", "Tags": "|legal|3d-models|", "Answer": "

No. Such figures are generally covered by copyright, which means that nobody but the copyright holder is allowed to (re-)produce copies of the work. Copyright also covers personal use. In the US there are limited fair use exceptions but they do not apply here.

\n\n

The only way to do this legally is if it is specifically authorized in a license or if you get permission.

\n" }, { "Id": "171", "CreationDate": "2016-01-13T12:52:32.577", "Body": "

I would like to print edible cookies or ornamentation for a cake.

\n\n

Is printing with edible materials achievable by standard thermoplastic-like 3D desktop printer? Or you need to buy a special printer to do that?

\n", "Title": "How to print edible food?", "Tags": "|desktop-printer|food|", "Answer": "

MakerBot Industries had a mod available for their early open source machines called the Frostruder. It was basically a syringe connected to your print head. I saw this in action at the University of Washington a long time ago. Check out the legacy ReplicatorG in action!

\n\n

I like to relate 3D printing as \"A hot glue gun on rails\". The beauty is that a lot of the home machines can easily be almost anything on rails.

\n\n

So, you could always try to 3D print a solution of your own. Please keep in mind any federal regulations on food processing materials (ie ABS is not recommended for most food applications).

\n" }, { "Id": "181", "CreationDate": "2016-01-13T14:12:10.063", "Body": "

I would like to print fancy plastic cutlery sets or plastic glasses.

\n\n

Is it safe to do it? Or bad for your health, if so, why?

\n", "Title": "Can you use PLA material with food and drinks?", "Tags": "|pla|safety|health|food|", "Answer": "

As others have pointed out, PLA isn't specifically not food safe, but materials that have been printed previously can contaminate the PLA.

\n\n

Additionally, anything 3D printed is extremely porous. Once a part is used for food, moisture and bacteria will accumulate in the pores, and can never be completely cleaned out, contaminating any food that contacts it. It can't be sterilized either, because the temperatures needed for sterilization would deform or melt the plastic.

\n" }, { "Id": "182", "CreationDate": "2016-01-13T14:12:59.173", "Body": "

In my slicing software (Slic3r) some of the vertices/walls of my model seem to have disappeared, so that the inside of the model - which should be solid - is visible, while the surface appears as a thin shell.

\n\n

Why does this happen? Is it still safe to export the model for printing?

\n", "Title": "Why do some models appear broken in slicing software?", "Tags": "|slic3r|print-preparation|slicing|g-code|", "Answer": "

Had similar problems with blender <-> Slic3r, modified a stl in blender then slic3r started acting up, showing strange faces where it shouldn't.\nThe solution was (as suggested by @tbm0115) to solidify the exported object.\nJust add a Solidify modifier to the object(no need to apply), and when exporting to stl make sure to check the \"Apply modifiers\".

\n" }, { "Id": "191", "CreationDate": "2016-01-13T14:36:43.710", "Body": "

I'm interested in designing & 3D printing as a hobby (e.g. printing chess sets, small toys for family etc.)

\n\n

Conducting a Google search has brought up a range of small, cheap printers, but beyond that I don't know how to differentiate them.

\n\n

E.g. selling points include:

\n\n\n\n

My question is, which features are going to benefit a small-scale, new enthusiast to 3D printing?

\n\n

PS. The software I intend to use is Windows 10 3D design

\n\n

PPS. I'm not a graphic designer by any means, just a new enthusiast.

\n", "Title": "What criteria do I need to be aware of when buying a 3D printer for personal use?", "Tags": "|desktop-printer|", "Answer": "

The number one most important thing about a 3-d printer is the Extruder/PrintHead component. If these are buggy, the printer is worthless. Seems like most printers are pretty good with everything else. The place that they die is in the Extruder/Print Head assembly.

\n\n

The second most important thing is quality of the plastic parts. One nice thing about that is you can usually reprint the plastic parts by going to thingiverse.com and downloading them.

\n\n

So, I'm on my 2nd printer. I returned the first one to amazon and may very well return the second one. It is a frustrating game. On thing I notice is that VERY FEW printers have a LOT of ratings. Some might have 4 stars, but only 8 votes... The latest one I have had 200+ votes but a 3.5 rating. If you have 200+ votes and anything below 4.2, you've got an inconsistent and potentially crappy product.

\n\n

So, to sum up, Get details on the extruder. Does it cool properly, is the design such that the filament is properly driven and pushed? Is the hot head the only hot part, or is the tube getting heated too. Does the extruder throat near the hot head have a Teflon tube inside it?

\n\n

Read the ratings on Amazon, read the bad reviews and then look at the pictures of the product to verify them.

\n\n

Best of luck.. I'm about to return #2 printer and perhaps part out #3. Not quite sure yet..

\n" }, { "Id": "198", "CreationDate": "2016-01-13T16:47:37.063", "Body": "

I'm reading about wiring up the electronic components to my Prusa i3 using an Arduino Mega 2650 and Ramps 1.4.

\n\n

I have step sticks, a heated bed, and a Switching Power Supply 12v Dc 30a 360w (more details on that later when I can add which ones to the post).

\n\n

I've heard that if you wire it wrong and plug it in, you can do anything from starting a fire to burning out your boards.

\n\n

What are some tips of things to check before plugging it in? Are there any common mistakes that I can avoid?

\n", "Title": "Tips for not burning out my Arduino Mega or catching something on fire when wiring a Prusa i3?", "Tags": "|reprap|prusa-i3|ramps-1.4|arduino-mega-2650|", "Answer": "

Adding to the other answers:

\n\n\n" }, { "Id": "203", "CreationDate": "2016-01-13T17:57:10.633", "Body": "

I have noticed that Slic3r offers a speed setting called \"auto speed\" meant to give a constant filament pressure at the extruder, which I believe could eliminate filament grinding issues at higher printing speeds.

\n\n

According to the tooltip in Slic3r, auto speed is calculated from two parameters:

\n\n\n\n

Maximum speed speaks for itself, but how can I calculate the maximum volumetric speed of my print?

\n", "Title": "How can I calculate volumetric speed for Slic3r auto speed?", "Tags": "|slic3r|print-preparation|", "Answer": "

There's no fixed maximum volumetric speed that works for everyone, there's simply too much variables to account for.\nBy using @Ian Williams explanation you can convert from volumetric to regular speeds but you still need to test what speed works best for your setup.

\n\n

Just a few of the other variables affecting how fast material can come out consistently: temperature (nozzle & heat brake), extruder motor power, path friction between extruder and hotend, material compressibility, fluidity and glass transition temperature, ..

\n\n

There are interesting topics on RepRap forums, like this one:

\n\n

http://forums.reprap.org/read.php?262,654085

\n" }, { "Id": "204", "CreationDate": "2016-01-13T18:02:27.633", "Body": "

I upgraded to an Mk9 dual extruder, and it came with thermocouples installed instead of the thermistors I had before.

\n\n

No matter what I did with the thermocouples, the indicated temperature jumped around by as much as 30C or more. In short, after several weeks of fiddling I never got the thermocouples to work well, and replaced them with thermistors, which have been fine.

\n\n

So my question is: what is required to get thermocouples to give reliable, consistent, accurate readings? Are they just incredibly touchy?

\n\n

Some things I tried include:

\n\n\n\n

Anything I'm missing?

\n\n

Thanks!

\n\n

Steve

\n", "Title": "How to get consistent and accurate readings from thermocouples?", "Tags": "|extruder|maintenance|", "Answer": "

Thermocouples work by passively generating VERY small voltages via the Seebeck effect -- usually a few tens of millivolts. They're literally just a pair of wires made from two different special alloys, electrically connected together at the \"hot\" end. That wire junction can be mounted inside whatever kind of attachment tip or lug is desired.

\n\n

The fact that they're very simple and passive devices makes them extremely precise and consistent between TCs of the same type, MUCH more so than thermistors. Any type-K thermocouple in the world will give you the same accurate output +/-1-2C or so. You can even cut a thermocouple in half, re-twist the ends of the wires together, and it'll still work!

\n\n

However, the very small (millivolts) signal they generate is quite susceptible to electrical noise and circuit design. The signal voltage has to be greatly amplified to be useful. So it doesn't take much EMR from your heater or stepper wires to interfere with the TC reading. A frequent problem with TC circuits in 3d printers is the dreaded GROUND LOOP -- if the \"hot\" tip is electrically connected to the hot block, voltage and current on the heater and motor wires can induce small currents through the TC wires that totally screw up the millivolt signal. The amplifier picks up these stray voltages and it throws off the temp read. So, there are some important guidelines for keeping noise out of the TC wires:

\n\n\n\n

Another common issue with TC circuits is the COLD JUNCTION COMPENSATION. A thermocouple doesn't measure tip temperature, it measures the DIFFERENCE in temperature between the hot tip and the cold junction where the TC is connected to either the amp or copper wiring. The TC amp has an onboard thermistor that it uses to add the temp at the cold junction to the measured signal from the thermocouple. There are a few things you need to do to make sure the cold-junction compensation works properly:

\n\n\n\n

If you do the above, the TC will output a good signal, and the amp will read it properly. But there's one more hitch. The mainboard has to know how to understand the amp's output. 3D printer control boards that are designed exclusively for TCs, like Mightyboards, usually use digital communication between the amp and the main control chip (MCU). This is high-reliability and does not require any special firmware configuration -- support is baked in. But if you're strapping an external TC amp onto a board that is expecting thermistors, you will have to tell the firmware how to read the signal from the amp. The most common technique is for the amp to output a linear voltage signal to the MCU's normal thermistor input (ADC). Then you configure the firmware to use the appropriate \"thermistor table\" (really a voltage lookup table) for that particular amp. Depending on your controller board, you also may need to make sure the regular thermistor pull-up/pull-down resistors aren't affecting the amp's output.

\n\n

So you need to make sure:

\n\n\n\n

If you do all that, a TC should give superior accuracy and reliability over a thermistor.

\n" }, { "Id": "210", "CreationDate": "2016-01-13T20:35:45.770", "Body": "

How do I determine how much an individual print costs?

\n\n

I'd like an answer including support material, failed prints, and (ideally) wear and tear / printer maintenance costs.

\n\n

To clarify, I'm not asking how to predict the cost before printing, but rather how to calculate the actual cost after printing. Though predicting the cost beforehand is useful as well.

\n", "Title": "How do I calculate the cost of a 3D print once it's done?", "Tags": "|cost|", "Answer": "

I recently faced the problem of calculating the cost of my printed 3D models. I wanted to know what their real value had to be counted in Excel. It was really inconvenient. Then I found a program for counting, it turned out really great, even takes into account the electricity. This is not an advertisement just throwing, maybe someone also encountered such a problem.\nhttps://codecanyon.net/item/mcc-3d-model-cost-calculation-for-3d-printer/24033425\nI was interested in the question who solved the given problem in what ways?

\n" }, { "Id": "212", "CreationDate": "2016-01-13T20:46:13.137", "Body": "

When the print head changes direction, the printer must accelerate and decelerate the print head. When calibrated correctly, the printer is able to do this quickly and without causing the printer to shake too much, without drastically slowing down the print process.

\n\n

If I set it too high, my printer shakes violently, especially during infill. If I set it too low, print times are doubled or tripled.

\n\n

What process can I follow to determine (or how can I calculate) the fastest acceleration value my printer can use without causing problems in my print?

\n\n

I'd prefer a process I can follow over a formula I can plug values into, especially if the formula includes magic numbers.

\n", "Title": "How do I determine the acceleration value for my printer?", "Tags": "|calibration|", "Answer": "

Most printers use between 2000 and 5000 mm/s2.... extrusion moves are usually 2000 (on average, between different printers), travel is normally seen at 3000-5000, though for most lower end or duel direct drive extruder (heavy) printers, this value should be lower (some as low as 500-1000, some as high as 2000 for travel). Heavier extruders need lower values, as well a slower jerk values, to avoid the motors skipping. The jerk setting being set lower can actually be beneficial, as this makes for smoother extrusion after a corner or a hole (some people will see ripples near these areas as it accelerates out of the direction change - lower values make these ripples smaller, but slightly increase print time).

\n\n

I've just modified mine with a prototype carriage design that I'm working on. It's added about 100 grams to the stock weight. It was finely tuned before, so this extra weight set it over that pillow of 20-30% in the setting, and makes the X and Y skip as it jerks around the plate; especially in high resolution models, particularly high resolution circles with 200 faces on the vertical axis, because the printer tries to jerk after each node - normally that isn't an issue, but in heavy designs, the jerk setting needs to be lowered. I had my jerk at 20 for XY, and its now at 4, after having tested and failed 8 and 15 values. The acceleration was at 2000 for both extrusion and travel (some firmwares don't allow separate values for travel and extrusion moves, but if yours does, it's best to have travel value about double the print moves - when your travel is twice the speed of extrusion moves - so use that 1-1 formula in relation to speed, scale it for each desired speed set in the slicer). Mine has been tested at 2000, 1500, 1000, and 800, failing all of them, with jerk setting of 5. Now testing 500 for acceleration, and beginning to get a little confused as to why this is happening.

\n\n

Now I'm having second thoughts... After going as low as 800 from 2000, and still getting skipping (seemingly worse) am I getting this backwards here? Do I need a higher value if the motor is skipping on quick moves? My understanding is that lower is better for those problems, but perhaps I've confused myself.

\n" }, { "Id": "214", "CreationDate": "2016-01-13T20:50:38.520", "Body": "

Using a thermoplastic MDF printer with a 0.4mm extruder nozzle, I frequently have trouble with the nozzle getting clogged.

\n\n

I am not sure what's causing the clog, but my guesses are dust and/or burnt filament (from leaving the hot end on without extruding).

\n\n

What can I do to prevent, or at least minimize, the extruder nozzle getting clogged?

\n\n

Bonus question: What other common causes of clogs are there? (ie what should I watch out for besides dust and leaving the hot end on?)

\n", "Title": "How do I keep my extruder head from getting clogged?", "Tags": "|extruder|fdm|maintenance|extrusion|", "Answer": "

First, you don't.

\n\n

To reduce the likelyhood, use good plastics without contaminations.

\n\n

Do not over heat the plastic, causes faster carbon build up.

\n\n

Buy hand drills, mini drills to clear it.

\n\n

Don't switch materials around a lot. Low temp plastic residue will cook when you switch to high temp plastic. Again carbon.

\n\n

Last will come to your extruder is self. Use a hardened extruder tip, less likely to wear. All metal should help a lot too, PTEF tubes often get baked in a jam situation.

\n\n

Or do what I do. I use a giant 1mm nozzle. I have printed at least 20 lbs of plastic on the same nozzle.

\n\n

The other solutions say put oil etc.. I will just say that is not a great idea. First you will have terrible bed adhesion, second it just seems wrong. Not to mention it will be out of the extruder within one KG?. The only time its okay to use oil is if you have a bowden setup.

\n" }, { "Id": "216", "CreationDate": "2016-01-13T21:00:04.907", "Body": "

My thermoplastic FDM printer has a heated bed and uses glass as the printing surface. Sometimes the glass will chip or break entirely when I'm removing my print. This happens most often when the print has a large area in contact with the glass.

\n\n

What can I do to keep this from happening?

\n", "Title": "My heated glass print bed keeps chipping and cracking. How can I prevent this?", "Tags": "|fdm|heated-bed|hbp|fff|", "Answer": "

I have 2 suggestions.

\n\n

First, get better glass. high quality borosilicate plate glass at least 3 mm thick should shrug off even scraping with a razor.

\n\n

Second, don't scrape it with a razor, put the whole thing in the freezer (or fridge, or in front of a fan, wherever), borosilicate is known for having a very low thermal coefficient, so the plastic is going to shrink more than the glass and should pop right off

\n" }, { "Id": "229", "CreationDate": "2016-01-13T23:34:03.707", "Body": "

I am reconfiguring a Printrbot Simple Metal that has been retrofitted with a RAMPS+Arduino Mega running Marlin, with a fairly slow feedrate due to mechanical and quality limitations.

\n\n

I am finding that upon cancelling a print where something goes wrong, I have a long period of time in which the printer is continuing to print from the G-code and movement buffer, although the host software (Octoprint) has long stopped sending G-code.

\n\n

Is there a way I can either a) configure Marlin to have a smaller print buffer (since Octoprint has no trouble keeping up with the printer as it stands) or b) send a particular signal or G-code to the printer that stops it without continuing to read buffer contents?

\n", "Title": "How do I set the G-code buffer size on Marlin?", "Tags": "|g-code|firmware|marlin|", "Answer": "

Marlin does allow one to change the size of the buffers, in Configuration_adv.h. In the current version there's an ifdef that switches between two cases, one with SD support, and the other without. Both have a movement planner of size 16, which can be adjusted.

\n\n

Additionally, in the same file, BUFSIZE can be changed to modify the size of the buffer storing unparsed commands before they are parsed and enter the movement planner buffer.

\n\n

Additionally, for some firmwares, M112 will immediately shut down the printer, no matter what is in the buffer, but the reset button will need to be used, and the axes will not be homed afterward. In Marlin, it's hardcoded to call kill().

\n" }, { "Id": "233", "CreationDate": "2016-01-14T00:23:36.260", "Body": "

I often switch my print material, i.e. ABS / PLA / Wood / Flex,

\n\n

How can I best clean out my extruder between them to ensure I don't contaminate my next print?

\n", "Title": "How should I clean my extruder when changing materials?", "Tags": "|maintenance|print-material|extruder|", "Answer": "

eSun has a cleaning filament that extrudes at a wide range of temperatures. It may seem ridiculous to use filament that costs over $100 USD per kilo to purge the nozzle, but it's very convenient and in practice, you don't use much each time (it's sold in 100g packs). I'm still using some samples I received with some filament over a year ago.

\n\n

Cold pulls will probably be more effective though (as described in another answer), they're just more effort. I usually use cold pulls with nylon when I want to clear out my nozzle, either as part of routine maintenance or when switching to plastics that are sensitive to contamination (usually PETG or ninjaflex). I usually use the cleaning filament when I want to change from a dark to a light or clear filament, especially if the new filament is expensive. Most of the time I just feed in enough of the new filament to purge out the old, with the temperature set to the max of the two.

\n" }, { "Id": "239", "CreationDate": "2016-01-14T05:15:09.270", "Body": "

After multiple jams from bulging filaments on two spools I'm getting frustrated. One, right before a job was done.

\n\n

Is there something I can do to prevent these bulges in filaments from ruining jobs?

\n\n

What can I do to prevent this from happening in the future before it's a disaster?

\n\n

He's a picture of one I found using google.

\n\n

\"enter

\n", "Title": "Bulging filaments - How can catch them before they go in the printer", "Tags": "|filament|", "Answer": "

That's my picture :-)

\n\n

I use a filament monitor with an encoder wheel that pauses the print if the filament stops moving (because a bulge got stuck, or filament ran out, or the hot end jammed, or whatever). I have a bunch of these on multiple printers: http://www.toybuilderlabs.com/products/tunell-3d-printer-filament-monitor

\n\n

The Tunell monitor works with Sailfish, Marlin, and Smoothie to my knowledge. This particular one doesn't work with Repetier, but you can wire up a simple encoder wheel to use Repetier's filament monitor functionality.

\n\n

Like other people have mentioned, you should avoid cheap filament, and return spools with bulges. I personally only see bulges like that maybe every 20 spools or less.

\n" }, { "Id": "242", "CreationDate": "2016-01-14T06:54:46.547", "Body": "

My Kossel Mini printer (delta) has RAMPS1.4/Arduino Mega electronics configured to use a standard 20x4 LCD display with Marlin during printing.

\n\n

Often, navigating the menus during print can be painfully slow, and I occasionally end up making the wrong selections due to lag.

\n\n

Without starving the actual printing process of CPU resources, is there any way of speeding up the menu navigation?

\n", "Title": "Marlin menu navigation slow while printing", "Tags": "|slic3r|ramps-1.4|arduino-mega-2650|marlin|", "Answer": "

A Delta printer requires constant complex calculations to produce straight lines while printing. The firmware, therefore, spends most of its time figuring out the step and timing sequence, and only the little remaining time between interrupts and these calculations is given to the user interface.

\n\n

Marlin doesn't document any configuration parameters that would improve the user interface responsiveness, and in any case such improvement must necessarily come at the cost of printing speed and/or accuracy. The processor is being used to its maximum capacity. The only thing you might be able to do is dig into the firmware and try to change it yourself, as this is not a clear option within Marlin.

\n\n

If working with the user interface while printing is important to you, your next step should probably be to change to a faster 32 bit processor. There are a few firmwares available for ARM and other architectures which may resolve your situation.

\n" }, { "Id": "250", "CreationDate": "2016-01-14T19:34:50.137", "Body": "

I am designing a part that has to clamp around a 11mm bushing, and due to other design constraints, it has to be printed with a semicircle-shaped overhang:

\n\n

\"enter

\n\n

This is proving very challenging to print. Two copies of this part have to clamp tightly around the bushing in all directions. Support material is rather hard to remove from the very top of the arc (where the overhang angle is the highest) and I often end up removing just too little of the support material (so the part doesn't fit around the bushing) or too much (and the bushing can wobble around).

\n\n

Is there any way I can modify the design of this part (bearing in mind that it absolutely has to be printed in this orientation) to make it more tolerant of my inaccuracy when removing supports, or is there perhaps some way to manually design supports that are easier to remove (Simplify3D and Cura both don't quite cut it)?

\n", "Title": "How to print an overhanging arc", "Tags": "|fdm|support-material|", "Answer": "

I had to design something rather like this, but I made the part that wraps around the pipe (pipe in my case - bushing in yours) into a separate piece that slotted into the main arch.

\n\n

That way, the main arch could be printed with poor precision on the overhang, and the sleeve was printed on its side. It took a little work to make the slotting system fit well, but it was fine. The two arches screwed together to grip two sleeve sections. It also meant I could print the big parts quickly because their precision was less critical.

\n\n

If - as you suggest - you absolutely have to print it this way, then how about pulling the top of the arch up a little; make it a bit \"gothic\" if you see what I mean. This is a variation on tjb1's idea above, but rather than have a flat at the top, organize it so there's a gentle point. That way, the printer isn't trying to draw a critical fitting over empty space, and the area that prints poorly is away from the bushing. You might have the slack to put supports back in.

\n" }, { "Id": "253", "CreationDate": "2016-01-14T21:48:19.053", "Body": "

I'm thinking of recycling some filament from a couple of recently failed prints. I can reuse them in the future for basic prototypes, so I'm not concerned with whatever weird mixture of colors come out (they are of a few different colors).

\n\n

The thing is, I have both PLA and ABS, in small quantities. I originally intended to simply use each one separately, but it occurred to me that they could be mixed.

\n\n

If I recycle PLA and ABS together into one strand of filament, will there be any negative side effects (e.g. reduced strength)?

\n", "Title": "Can I mix ABS and PLA when recycling filament?", "Tags": "|abs|pla|filament|recycling|", "Answer": "

CNC Kitchen tested PLA up to 265\u00a0\u00b0C or 270\u00a0\u00b0C in a couple of videos, for example

\n

\r\n \r\n

\n

The plastic (it may depend on the brand) holds just fine up to 250\u00a0\u00b0C, but (from personal experience) bridges become really difficult to tune. Strength is very good.

\n

ABS can be printed properly starting from 235\u00a0\u00b0C (depending on the brand). At 235\u00a0\u00b0C layer adhesion may be suboptimal, but you are mixing it, so it's not obvious that the result will be poor.

\n

So, to summarise you have a common range 235-250\u00a0\u00b0C where you can print both plastics properly when pure, so it is reasonable to think that by mixing them you will have at least the same temperature range (the bottom may even be extended a bit, as it happens with PC-ABS which prints properly colder than pure PC).

\n

So I would say that it is a worth test with likely positive results.

\n

In fact, ABS-PLA blends exist and are very good blends for industrial use as the Terrafilum site states:

\n
\n

ABS_PLA_Blend

\n

Strong PLA Industrial Grade PLA with superior flow and\nstrength performance; designed for parts where PLA is desired but\nwhere the parts will be subjected to repeated use; ideal for parts\nthat may need bend slightly without breaking during use.

\n
\n" }, { "Id": "256", "CreationDate": "2016-01-14T22:29:05.317", "Body": "

I am operating a laser sintering machine, using polyamide 2200 powder (with a grain size of approximately 50 micrometers). During a print, a lot of powder goes unsintered and can theoretically be reused. However, using purely recycled powder degrades print quality to an unacceptable level.

\n\n

Mixing a little used powder into a larger amount of fresh powder seems to work well though. What is the greatest ratio of used to fresh powder that still gives good results, and is there anything I can do (pre- postprocessing) to allow more powder to be reused?

\n", "Title": "Optimal ratio of fresh to used nylon powder", "Tags": "|quality|recycling|sls|", "Answer": "

I currently use the 60/40 recycling mix ratio and find that it works very well. I do however wonder if there is an even more effective ratio in order to recycle used powder. I currently discard all \"cake\" powder (powder remaining in the build piston) and am only \"recycling\" the push off powder. I found this paper but it's unclear if they are reusing just the push off or both push off and cake. Any further opinions/ideas would be greatly appreciated.

\n\n

http://www.internationaljournalssrg.org/IJME/2015/Volume2-Issue7/IJME-V2I7P106.pdf

\n" }, { "Id": "258", "CreationDate": "2016-01-15T00:47:30.323", "Body": "

Usually it will either will rip the tape, or break the print somehow. Currently using ABS on a taped glass bed with a layer of hairspray for adhesion.

\n", "Title": "How to minimize damage when removing an ABS print from a heated glass print bed?", "Tags": "|heated-bed|abs|desktop-printer|", "Answer": "

Glass has a very peculiar effect under heating, that can be used to remove extremely delicate parts from the surface of it:

\n\n

Glass expands and shrinks differently to the ABS under temperature. Letting the glass bed cool down has it shrink, creating tension on the interface layer which can be exploited with a thin scraper. Putting the bed with the print into the fridge increases the tension to a point at which the bonding breaks. This results in the part popping free in several areas (sometimes everywhere) and easing the removal.

\n" }, { "Id": "259", "CreationDate": "2016-01-15T01:07:58.893", "Body": "

Let's say I print a part out of ABS and wait for it to cool. I could theoretically do this with several copies of the same printer, modified to use print beds of different compositions.

\n\n

Will the material a bed is made out of affect how long it takes a part to cool?

\n", "Title": "Does the material a bed is made out of affect the cooling time of a part?", "Tags": "|heated-bed|hbp|cooling|", "Answer": "

What bed material cools faster?

\n\n

I found an extensive list which relates various materials to their thermal conductivity, k [W/mK]; the lower thermal conductivity, the better the material insulates, and the slower the print bed will resist changes in temperature - both heating up, and cooling down.

\n\n

Here are the thermal conductivity for some common materials for 3d printer beds:

\n\n
Aluminum    205\nGlass       1.05\nAcrylic     0.2\nAir         0.024 (for reference)\n
\n\n

There is also the matter of thermal capacity, but I will not go into that right now (need to do some research myself first!).

\n\n

Will bed material affect cooling time?

\n\n

Bed material, I believe, is not necessarily related to print cooldown time: it depends on the situation, such as whether we are discussing cooldown during or after printing, and if the bed is heated or not.

\n\n
    \n
  1. If you are not using a heated bed, I believe the bed material doesn't matter at all.
  2. \n
  3. With a heated bed while printing, only the first dozen layers or so are probably affected by the rising heat sufficiently that it affects the printing process.
  4. \n
  5. With a heated bed after printing, the thermal characteristics of the bed will determine how quickly the print cools (and thus can be removed).
  6. \n
\n\n

Also remember that other physical properties, such as flatness (both cold and during heating) of the bed material is vital for successful prints, and that not all materials can tolerate heating equally well!

\n" }, { "Id": "264", "CreationDate": "2016-01-15T07:04:09.510", "Body": "

Why do we have two standard filament sizes, 1.75 mm and 3 mm? Does it really make a difference when printing? Or is the 1.75 mm just for smaller printers?

\n\n

In what situations should I be using 1.75 mm?

\n\n

When should I be using 3 mm?

\n", "Title": "When to use 1.75 mm vs 3 mm filament?", "Tags": "|fdm|filament|extruder|", "Answer": "

As I read the history, 3 mm filament was an accident of the supply chain when 3D filament printers were first being developed by hobbyists. There was a product called a \"plastic welder\" which consisted of a melting device and a source of filler material. This filler was 3 mm plastic.

\n\n

As the techniques and equipment developed, the market for filament grew to a size where it could support companies producing filament specifically for 3D printing. The benefits of 1.75 mm filament over 3 mm were, IMO, huge -- especially the easier melting and lower force needed by the extruder.

\n\n

Except for special purposes like pushing soft plastics through Bowden tubes, it seems from the marketplace that 1.75 mm filament as completely overtaken 3 mm filament.

\n\n

A possible second-order disadvantage of 1.75 mm filament can be water absorption. The surface-to-volume ratio is higher -- there is more surface per unit of the filament through which water vapor can be absorbed. It is important to keep filament dry, and sometimes necessary with both 3 mm and 1.75 mm to dry the filament in an oven before use.

\n" }, { "Id": "269", "CreationDate": "2016-01-15T13:05:05.660", "Body": "

As an extension from this question, is there any reason that you would not be able to use 1.75 mm filament in a printer that takes 3mm filament? I know you would have to change the filament size in the slicing of prints but would there be any other problems?

\n\n

Also, would using 1.75 mm filament be possible if the nozzle diameter was greater than 1.75 mm but less than 3 mm?

\n", "Title": "Can 1.75 mm filament be used in a printer that takes 3 mm filament?", "Tags": "|filament|extruder|", "Answer": "

Typically an extruder and hot end are designed for one or the other, and cannot support the other without mechanical changes.

\n\n

The extruder may not be able to grip a smaller diameter filament with enough force to assure even feeding and retraction.

\n\n

The hot end, however, is much more complex. The filament has to be pushed with force into the melting zone, which means the filament has to slide along an area inside the hot end where the filament is plastic but still put pressure on the filament ahead of it.

\n\n

When you put filament into a hot end, the filament softens before the melt zone, but since the walls of the hot end are just barely larger than the filament it has no choice but to continue pressing down on the liquid filament below.

\n\n

With a narrower diameter filament, though, the filament can heat, soften, then travel backwards along the sides of the hot end and cool in place, jamming the hot end, or at least preventing an even continuous flow of plastic.

\n\n

Some hot ends will accept a small Teflon tube that takes this space up and allows you to do this with fewer issues, and if you like to tinker you can experiment with this, but be prepared to learn a lot and fail a lot as you find out the hard lessons of hot end design.

\n\n

Generally you should upgrade your entire extruder and hot end setup to the size you want to use.

\n" }, { "Id": "271", "CreationDate": "2016-01-15T16:33:07.330", "Body": "

I've been printing for a week now on my new printer and have been getting great results, including great adhesion. However, most of my prints have had a fairly small footprint.

\n\n

Now, I'm stepping up the types of items I'm printing, and I've started to run into a problem. Long, thin parts are starting to lift off the bed, especially at the edges of the bed.

\n\n

I've read the question dealing with this issue with ABS and realize some (most) could apply to me. However, I'm printing entirely in PLA on a non-heated bed.

\n\n

My question is: What is the best way to go about troubleshooting this problem? In which order should I attempt fixes to narrow down the problem most quickly?

\n\n

Printer: Monoprice Maker Architect 3D (Very similar to Makerbot)

\n\n

Material: PLA

\n\n

Heated Bed: No

\n\n

Bed Material: Something similar to BuildTak on top of an acrylic bed. (The sheet of material came unlabeled with the printer and I haven't been able to track down exactly what it is.)

\n", "Title": "Troubleshooting poor adhesion at the edge of the bed", "Tags": "|pla|adhesion|warping|", "Answer": "

Try a dilute solution of PVA glue (approx. 5:1 water:PVA) applied to the bed or the BuildTak like sheet and allowed to dry. \nKeep the ambient temperature as high as possible (but not so high as to soften any plastic on the printer obviously). \nBig brims help - consider adding them to the model rather than just applying them in the slicer. I've used 12 mm by 0.5 mm deep circles to good effect, particularly on acutely angled parts of the model or the extremities.

\n" }, { "Id": "279", "CreationDate": "2016-01-16T00:11:52.217", "Body": "

With fdm printers, the 3D object that should be printed can be positioned anywhere in the build volume. But it's only practical to place it on the bottom, because otherwise support material would be necessary.

\n\n

stereolithography has the same problem. Even though the photopolymer can be cured at any position in the build volume, the result would drift away if it was not held in place by support material.

\n\n

The powderbed based printers (either powder+binder or any of the laser/electron beam sintering/melting variants) do not have this problem, because they continuously fill the entire build volume with powder. The support material that other printing technologies require is part of the powder based printing anyway. It would be possible to pack the build volume with many prints and print them in one go.

\n\n

Given that the machines are relatively expensive, it would be economical to increase the throughput. A company that does use such printers heavily could wait a certain amount of time until a few print queued up that fit together in the build volume and only then start the process. Do people do this?

\n", "Title": "Is packing multiple prints into the build volume a feasible workflow for powderbed printers?", "Tags": "|sls|slm|", "Answer": "

The print speed for powderbed printing depends primarily on the height of the print; the lateral extent doesn't really matter. Since powderbed printing provides its own easily-removed support structure, a packing that maximizes the number of items and minimizes the height will maximize throughput.

\n" }, { "Id": "294", "CreationDate": "2016-01-16T13:28:24.050", "Body": "

I am building a Prusa i3 Rework, and I haven't been able to find out how to attach my J-Head extruder (see below), at the moment it's just kind of held there by pressure, but it seems there are two holes on ether side of it; not sure if they're meant to hold it in place, but it seems that way.

\n\n

\"Image

\n\n

Any suggestions as to what I should use to hold it in place? This is definitely a part that gets moved around alot.

\n\n

update

\n\n

\"Image

\n", "Title": "How is a J-Head Extruder Head attached to the Prusa i3 rework?", "Tags": "|reprap|prusa-i3|prusa-i3-rework|", "Answer": "

It looks like the hotend may not be all the way in, are you sure it's not stuck?

\n\n

If it's stuck and you can get it in further, those holes should go directly through the smaller ring on top of the J-Head. You just need to run a machine screw into each hole to secure the hotend. The screw will need to at least be flush with the other side to work correctly.

\n" }, { "Id": "296", "CreationDate": "2016-01-16T14:40:27.247", "Body": "

From what I understand, when you hook up the Switching Power Supply 12v Dc 30a 360w to the wall outlet, you have to be very careful; careful not to get the wires mixed up; careful not to have anyone or anything touch the leads (in fact the first proper project I intend to print out will be a casing to fit around the switching power supply), or just order one from someone.

\n\n

Now there are three wires that go into the US wall of particular concern, and these wires come out of a standard PC cable with the female end cut off, and they hook the power supply. Like the external casing, these three wires are also insulated, and when you take the insulation off the bare wires and connect it to the power supply, you have to use Electrical connectors of some sort to connect them to the power supply's screw leads.

\n\n

I bought some electrical connectors just for this purpose, but I'm not entirely certain they will be good for this purpose, so I thought I'd check here first.

\n\n

\"Picture

\n\n

There are specifications on the back:

\n\n
\u256c\u2500\u2500\u2500\u2500\u2500\u2500\u2500\u2500\u2500\u2500\u2500\u2500\u2500\u2500\u2500\u2500\u2500\u2500\u256c\u2500\u2500\u2500\u2500\u2500\u2500\u2500\u2500\u2500\u2500\u2500\u2500\u2500\u2500\u2500\u2500\u2500\u2500\u2500\u2500\u2500\u2500\u256c\u2500\u2500\u2500\u2500\u2500\u2500\u2500\u2500\u2500\u2500\u2500\u2500\u2500\u256c\u2500\u2500\u2500\u2500\u2500\u2500\u2500\u2500\u2500\u2500\u2500\u256c\n\u256c                  \u256c        AWG           \u256c  Wire Size  \u256c Stud Size \u256c\n\u256c\u2500\u2500\u2500\u2500\u2500\u2500\u2500\u2500\u2500\u2500\u2500\u2500\u2500\u2500\u2500\u2500\u2500\u2500\u256c\u2500\u2500\u2500\u2500\u2500\u2500\u2500\u2500\u2500\u2500\u2500\u2500\u2500\u2500\u2500\u2500\u2500\u2500\u2500\u2500\u2500\u2500\u256c\u2500\u2500\u2500\u2500\u2500\u2500\u2500\u2500\u2500\u2500\u2500\u2500\u2500\u256c\u2500\u2500\u2500\u2500\u2500\u2500\u2500\u2500\u2500\u2500\u2500\u256c\n\u256c Spade Terminals                                                   \u256c\n\u256c\u2500\u2500\u2500\u2500\u2500\u2500\u2500\u2500\u2500\u2500\u2500\u2500\u2500\u2500\u2500\u2500\u2500\u2500\u256c\u2500\u2500\u2500\u2500\u2500\u2500\u2500\u2500\u2500\u2500\u2500\u2500\u2500\u2500\u2500\u2500\u2500\u2500\u2500\u2500\u2500\u2500\u256c\u2500\u2500\u2500\u2500\u2500\u2500\u2500\u2500\u2500\u2500\u2500\u2500\u2500\u256c\u2500\u2500\u2500\u2500\u2500\u2500\u2500\u2500\u2500\u2500\u2500\u256c\n\u256c  YF1.25-35 (red) \u256c        22-16         \u256c   0.5-1.5   \u256c    3.7    \u256c\n\u256c\u2500\u2500\u2500\u2500\u2500\u2500\u2500\u2500\u2500\u2500\u2500\u2500\u2500\u2500\u2500\u2500\u2500\u2500\u256c\u2500\u2500\u2500\u2500\u2500\u2500\u2500\u2500\u2500\u2500\u2500\u2500\u2500\u2500\u2500\u2500\u2500\u2500\u2500\u2500\u2500\u2500\u256c\u2500\u2500\u2500\u2500\u2500\u2500\u2500\u2500\u2500\u2500\u2500\u2500\u2500\u256c\u2500\u2500\u2500\u2500\u2500\u2500\u2500\u2500\u2500\u2500\u2500\u256c\n\u256c Ring Terminals                                                    \u256c\n\u256c\u2500\u2500\u2500\u2500\u2500\u2500\u2500\u2500\u2500\u2500\u2500\u2500\u2500\u2500\u2500\u2500\u2500\u2500\u256c\u2500\u2500\u2500\u2500\u2500\u2500\u2500\u2500\u2500\u2500\u2500\u2500\u2500\u2500\u2500\u2500\u2500\u2500\u2500\u2500\u2500\u2500\u256c\u2500\u2500\u2500\u2500\u2500\u2500\u2500\u2500\u2500\u2500\u2500\u2500\u2500\u256c\u2500\u2500\u2500\u2500\u2500\u2500\u2500\u2500\u2500\u2500\u2500\u256c\n\u256c   YF1.25-4 (red) \u256c        22-16         \u256c    .5-1.5   \u256c    4.3    \u256c\n\u256c\u2500\u2500\u2500\u2500\u2500\u2500\u2500\u2500\u2500\u2500\u2500\u2500\u2500\u2500\u2500\u2500\u2500\u2500\u256c\u2500\u2500\u2500\u2500\u2500\u2500\u2500\u2500\u2500\u2500\u2500\u2500\u2500\u2500\u2500\u2500\u2500\u2500\u2500\u2500\u2500\u2500\u256c\u2500\u2500\u2500\u2500\u2500\u2500\u2500\u2500\u2500\u2500\u2500\u2500\u2500\u256c\u2500\u2500\u2500\u2500\u2500\u2500\u2500\u2500\u2500\u2500\u2500\u256c\n\u256c Butt Splice                                                       \u256c\n\u256c\u2500\u2500\u2500\u2500\u2500\u2500\u2500\u2500\u2500\u2500\u2500\u2500\u2500\u2500\u2500\u2500\u2500\u2500\u256c\u2500\u2500\u2500\u2500\u2500\u2500\u2500\u2500\u2500\u2500\u2500\u2500\u2500\u2500\u2500\u2500\u2500\u2500\u2500\u2500\u2500\u2500\u256c\u2500\u2500\u2500\u2500\u2500\u2500\u2500\u2500\u2500\u2500\u2500\u2500\u2500\u256c\u2500\u2500\u2500\u2500\u2500\u2500\u2500\u2500\u2500\u2500\u2500\u256c\n\u256c   BF-1.2SS (red) \u256c        22-16         \u256c    .5-1.5   \u256c   n/a     \u256c\n\u256c\u2500\u2500\u2500\u2500\u2500\u2500\u2500\u2500\u2500\u2500\u2500\u2500\u2500\u2500\u2500\u2500\u2500\u2500\u256c\u2500\u2500\u2500\u2500\u2500\u2500\u2500\u2500\u2500\u2500\u2500\u2500\u2500\u2500\u2500\u2500\u2500\u2500\u2500\u2500\u2500\u2500\u256c\u2500\u2500\u2500\u2500\u2500\u2500\u2500\u2500\u2500\u2500\u2500\u2500\u2500\u256c\u2500\u2500\u2500\u2500\u2500\u2500\u2500\u2500\u2500\u2500\u2500\u256c\n
\n\n

Not sure if I should use ring or spade terminals, and I don't know what wire size to use; and I don't know what wire grade is inside a standard PC power cord or even if these are safe connectors to use for this.

\n", "Title": "Are these the right types of electrical connectors for hooking my Switching Power Supply up to a wall outlet?", "Tags": "|wiring|switching-power-supply|", "Answer": "

The block on the supply will accept the bare wire

\n\n

\"enter

\n\n

you could use the yellow in the middle on the right, but the screw on the block essentially does its own crimp.

\n" }, { "Id": "303", "CreationDate": "2016-01-16T20:21:06.633", "Body": "

This article states that 3D printing has been accomplished in outer space, on the International Space Station.

\n\n

I'm curious as to how this works differently from 3D printing on Earth. Are there any extra measures that needed to be taken to ensure that the filament would be correctly extruded onto the print bed, or during other steps?

\n", "Title": "How is 3D printing done in space?", "Tags": "|applications|", "Answer": "

The first big space-specific issue is actually air quality. You can't just open a window to air out the molten-ABS smell from the ISS!

\n\n

FFF printers put out fumes and nanoparticles. In a space station, the same air gets recycled over and over, and the air purification systems have a specific set of contaminants that they are optimized for, as well as a design capacity for air turnover and chemical removal rates that won't be adjusted just because somebody's printing a space-ratchet today. Protecting cabin air quality is a huge design factor for any experiment that goes into space.

\n\n

The Made in Space printing experiments on the ISS to date were performed in one of the vacuum experiment chambers, so any unfiltered fumes (or fire flare-ups) could be vented directly to space if required. In the long run, this isn't going to work -- other experiments may need the vacuum chamber, or \"production\" printers may be too large to fit. So the printer needs to have its own internal air purification system.

\n\n

Another MAJOR design constraint is launch survival. Rocket payloads must be designed for extreme g-forces without 1) damage, or 2) significant internal shifting of mass which would affect the payload center of gravity.

\n\n

Total payload weight is also quite important here: lifting mass to low Earth orbit is EXPENSIVE.

\n\n

Surprisingly, the microgravity environment itself isn't that big of a deal. Molten plastic is highly viscous and pretty much stays where you put it long enough to solidify, as long as it's sticking to something. But two impacts do come to mind.

\n\n\n\n

Finally, reliability is critical. Amazon doesn't deliver to the ISS (yet). Even a single stripped screw may take the printer out of commission for months until a replacement part can be fit into an upcoming supply launch. Having the printer catch on fire because something shorted would be catastrophic.

\n\n

So, really, it's all about making a printer robust enough to make it up there, operate safely, and never break. Printing upside-down is trivial in comparison.

\n" }, { "Id": "309", "CreationDate": "2016-01-17T01:23:41.323", "Body": "

\"enter

\n\n

The Sainsmart Endstops I picked up are different from the ones described in the RepRap Prusa i3 Rework electronics assembly wiki; they have 4 female plugs that go into the RAMPS 1.4 board instead of 3:

\n\n

\"enter

\n\n

Since these endstops are different, how do I hook them up, and what do the markings on them mean?

\n", "Title": "My endstops have 4 female plugs, but the examples on the RepRap Prusa i3 Site have 3; what are each of them for?", "Tags": "|prusa-i3-rework|", "Answer": "

The website shows exactly what each wire is for. Both middle wires are ground, the wire on the same side as the lever is the signal wire and the last wire is the power wire.

\n" }, { "Id": "313", "CreationDate": "2016-01-17T13:27:51.030", "Body": "

My printed objects have horizontal holes in them (as seen I the picture below):\n\"enter

\n\n

This doesn't only look bad it also makes the object break at the seems.

\n\n

Looking carefully at the printing process I can see that after a retraction there's a small amount of time the hotend isn't extruding plastic.

\n\n

Material: PLA, Printer Robo 3D R1+, Slicer: Cura

\n", "Title": "Holes/ missing layers (after retraction) in 3d printed objects", "Tags": "|extrusion|", "Answer": "

(answering my own question)

\n\n

The problem was the extrusion distance settings in Cura's advanced tab, reducing the value to 1.5mm solved the problem.

\n\n

Other problems with the same symptoms:

\n\n\n" }, { "Id": "319", "CreationDate": "2016-01-18T10:29:58.050", "Body": "

Are there any techniques for getting a smooth finish for parts printed with co-polyester (PET) filaments? More specifically, I am looking for an alternative that does not roughen the look of the part - such as using sandpaper - but rather works like acetone baths for ABS.

\n\n

In particular, I want to treat ColorFabb's XT filament made from the Eastman Amphora\u2122 3D polymer (datasheet). This is also the polymer is also used in:

\n\n\n", "Title": "How to smooth the surface of parts printed with Co-polyester (PET) filament", "Tags": "|filament|post-processing|pet|", "Answer": "

Ethyl acetate (sold as a MEK substitute) is supposed to work for vapor smoothing PET. It doesn't seem very toxic (it's used to decaffinate cofee and tea, and as a nail polish remover), but you might want to look more into it. There's a post on Printed Solid's blog where he vapor smoothed colorFabb XT and MadeSolid PET+ along with a few other filaments and got some good results.

\n\n

http://printedsolid.com/blogs/news/37035395-vapor-smoothing-3d-printed-parts-pla-colorfabb-xt-t-glase-pet

\n\n

The links in the blog don't work for me, but google was able to find slightly larger versions:

\n\n

https://cdn.shopify.com/s/files/1/0887/0138/files/blog_2014-03-20-18.38.04-1024x613.jpg?16147388421280943481

\n\n

https://cdn.shopify.com/s/files/1/0887/0138/files/blog_2014-03-21-18.16.28-1024x612.jpg?9543779874607042697

\n" }, { "Id": "322", "CreationDate": "2016-01-18T12:33:22.370", "Body": "

When designing parts that should either fit with external objects or other printed parts, what measures can one take to ensure that the dimensions of the final print are accurate and fit the other object?

\n\n

To my knowledge, you at least have two options to account for printer inaccuracy and shrinkage:

\n\n\n\n

Are there any good workflows one can use to design and print 3D-models accurately without resorting to trial and error?

\n", "Title": "How to achieve dimensional accuracy of printed parts?", "Tags": "|desktop-printer|print-preparation|", "Answer": "

I print several pats that use 2.5mm \"Pogo pins\" which are spring-loaded electric contacts. I've found that many variables will influence the size of the holes I have in my design. Flow, temperature even different brands of filament will change the final size.

\n\n

I create a profile for each part and specific filament. That way I can make changes without changing other parts/projects. Then I print a test piece with some 2.5mm holes and a few that are a few tenths of a millimeter larger and smaller. I also make holes in the test piece that are vertical and some that are horizontal as I've found that orientation to the layers makes a difference.

\n\n

I then fit the pins in my test piece and note which orientation and diameter fit best.

\n\n

After that, I lock down every variable I can think of!\nI added some desiccant beads to my filament storage bins and found even that increased the diameter of the printed holes.

\n" }, { "Id": "331", "CreationDate": "2016-01-19T02:32:34.107", "Body": "

I upgraded to a dual Mk9 extruder, and quickly discovered how critical it is to get the ends of both nozzles exactly level with each other -- that is, equally distant from the build plate at all times. Otherwise the lower one will crash against the plastic just extruded by the higher one.

\n\n

So, what's a good procedure for getting the nozzles accurately level? About all I've figure out is to move the heads down close to Z=0, and then run X and Y back and forth and eyeball and adjust; then move even closer to Z=0 and repeat. Is there a better / more efficient way?

\n", "Title": "Levelling heads for dual extruder", "Tags": "|dual-nozzle|", "Answer": "

Another option, that I found was the simplest one that worked for me:

\n\n
    \n
  1. Level your bed using just the first nozzle (temporarily lock the 2nd nozzle higher than the 1st one).
  2. \n
  3. Move the printhead at the center of the bed
  4. \n
  5. Loosen the grub screws on both nozzles (IMPORTANT, don't skip this step)
  6. \n
  7. If you're not using a glass bed, temporarily clip a sheet of glass on your bed.
  8. \n
  9. Home the Z-Axis
  10. \n
  11. Move the the head sideways, front and back a few times just to be sure (X, Y)
  12. \n
  13. Move the printhead back at the center
  14. \n
  15. Lock the grub screws on both nozzles. Lock them slowly alternating between the top and bottom screws. Also make sure the wirings don't pull on the head (it's actually better if they push the nozzles down).
  16. \n
  17. Relevel the bed, this time checking with the 2 nozzles
  18. \n
\n\n

The sheet of glass ensures a completely flat and solid surface, on top of your properly leveled bed.

\n\n

Try the above in case nothing else works for you.

\n" }, { "Id": "339", "CreationDate": "2016-01-19T15:36:34.467", "Body": "

My Replicator (you know, the one made of balsa wood) has blown a voltage regulator on its MightyBoard for the second time. The first time, MakerBot graciously replaced it. However, they replaced with the same model board with no apparent fix for the commonly faulty voltage regulator. I can't afford the now $500 replacement board, so my option is to replace the faulty component and hope no further damage was taken on the board.

\n\n

As MakerBot Industries did not need me to send in my old board, I've followed these steps to try and replace the voltage regulator on the old board.

\n\n

I've offset the voltage regulator towards the power jack, jumped the remaining pin on the voltage regulator to the remaining lead on the board (where the old voltage regulator was), but I'm confused what/where the green wire does/goes (mine is blue on my machine). The image looks like he just soldered it onto the heat sink pad.

\n\n

I tried to test it by plugging in the power and the RGB LED strip. The stepper motor driver LEDs turned on, but the LED strip and none of the other on-board LEDs turned on. I'm not very seasoned when it comes to electronics, so can anyone explain what the actual pinout should be or suggest other ways to resolve this issue?

\n\n

Update: Old Board Replacement

\n\n

So far I've replaced the regulator on my old board with some success. Before the replacement, the board wouldn't turn on (on-board LEDs wouldn't turn on). Now, all of the proper status LEDs turn on, but the board seems to be stuck in a boot sequence. The LCD screen provides two lines of block characters (as it normally does when initializing), but just hangs up there. I tried connecting the machine to my PC to try and restore the board to factory settings and install the latest firmware. However, the PC couldn't find the machine and I began to smell a bit of burning. I'm not sure if this is a result of permanent damage from the original regulator failing, or improper installation of the new regulator.

\n\n

I'm going to dig around a bit more before attempting this fix on my main board. I'll edit this post as I continue the project.

\n\n

Update: New Board Replacement

\n\n

So, I've now attempted the fix on the newest board (the reason for this question). The results of this board are the same as the old one. I actually get power on the main board, but the LCD screen just shows two solid lines. I've taken pictures of what I've done.

\n\n

\"LCD\n\"Power\n\"Ground

\n", "Title": "MakerBot Replicator 1 Voltage Regulator Fix", "Tags": "|fdm|makerbot|replicator-dual|mightyboard|fff|", "Answer": "

What's the part number of the regulator you're putting in? The pin assignments vary from one part to another, but you can probably find them from the part number online. Some parts have the ground pin also connected to a metal part of their case. The green wire from the board should be ground (no promises!)

\n\n

Assuming you've got a voltmeter, you can use it to find out which pins on the board are power and ground (easiest to do before the regulator is installed...). Then once installed, check that you're really getting 5V versus ground on the remaining (output) pin.

\n\n

Since 2 of these burned out on you, I suggest taking steps to reduce the strain on the regulator. Voltage regulators turn the excess power into heat. Some things that can help:

\n\n\n\n

Let us know how it turns out!

\n\n

Steve

\n" }, { "Id": "341", "CreationDate": "2016-01-19T21:21:46.710", "Body": "

I just received this printer and while it seems to talk to Makerbot Desktop software I'm not sure if I should be trying to update the firmware.

\n\n

The printer comes with firmware v7.2 and while Makerbot Desktop offers an upgrade to v7.5 I'm not sure if it's a good idea with this non-Makerbot branded printer.

\n\n

I've also seen information on upgrading this printer to Sailfish v7.5, is this the same thing as Makerbot firmware v7.5?

\n", "Title": "Updating firmware on Monoprice Architect printer", "Tags": "|firmware|makerbot|monoprice-maker-architect|", "Answer": "

Since the printer has no heater, I'd advise some sleuthing

\n\n
    \n
  1. Look at the motherboard. Find the big black square chip and see if it is a ATmega 1280 or 2560. Likely it's a 1280, but you never know. This will impact which firmware build you use.
  2. \n
  3. If you will eventually add a heater PCB, then figure out the size (wattage) of the power supply. It may be big enough now for a heated platform, or maybe not. I guess you can cross that bridge if/when you add a HBP (heated build platform). However, it can make a difference as to which firmware build you load as some builds of Sailfish will intentionally serialize heating so as to not put too high of a load on the power supply (PSU).
  4. \n
\n\n

Armed with the above info, you can then decide if you want a 1280 or 2560 build of Sailfish. And if you want a build which will serialize eventual use of an HBP or not.

\n\n

Serialized: MakerBot Replicator 1 Single & Dual (implied ATmega 1280), MakerBot Replicator 1 with ATmega 2560

\n\n

Non-serialized: FlashForge Creator I, II & X (implied ATmega 1280)

\n\n

For a non-serialized, 2560 build know that the FlashForge one is for a poorly behaved heater PCB and you likely won't want it. That sort of leaves you without a good, non-serialized choice. In a pinch you can use the ZYYX 3D build for a 2560. Or you can contact the Sailfish team directly: speaking with very certain knowledge, I can assure you that they'd be happy to do a targetted build for your machine. However, at present they lack info to do so (e.g., build volume, distances from endstops to center of build platform, etc.).

\n" }, { "Id": "343", "CreationDate": "2016-01-19T23:02:39.743", "Body": "

When I use Cura with the Pronterface UI it sometime just stops printing.

\n\n

When it stops the printer just stops, the UI looks like it's still printing but nothing is moving in the printer, also, trying to control the printer from the UI does nothing.

\n\n

It always stops early in the printing process, usually during the auto-leveling process or while printing the skirt, the latest it happened was during the first solid layer of a raft.

\n\n

Usually closing the printing window and re-opening it solves the problem but not always, this never happened to me with the basic UI, I couldn't find any settings that makes the problem better or worse, it just happens randomly.

\n\n

Anyone knows how to stop that from happening?

\n\n

My printer is a Robo 3D R1+

\n\n

Update

\n\n

After installing a screen on my printer I discovered Pronterface is sending a \"Wait for user\" G Code to the printer.

\n\n

Because this changes the question too much and invalidates the existing answer I've asked a new question at What makes Pronterface wait for user?

\n", "Title": "Cura with Pronterface UI stops printing", "Tags": "|software|", "Answer": "

After installing a screen on my printer I discovered Pronterface is sending a \"Wait for user\" G Code to the printer.

\n\n

Because this changes the question too much and invalidates the existing answer I've asked a new question at What makes Pronterface wait for user?

\n" }, { "Id": "344", "CreationDate": "2016-01-19T23:18:20.953", "Body": "

I'm in the process of building a 3D printer and have all the stepper motors wired up and the controller connected to the computer running pronterface.

\n\n

I can move each axis and also send g-code to the printer.

\n\n

Now how can I define the current position to be the 0/0/0 position?\nI'd like to move the printer into a position where I think should be the zero position and define it as such in pronterface somehow.

\n", "Title": "How can I set the position in Pronterface?", "Tags": "|software|pronterface|", "Answer": "

There's a handy list of G0-codes at http://reprap.org/wiki/G-code -- though the list is too big to be convenient for finding a code if you're not already sure what it's called... :(

\n" }, { "Id": "352", "CreationDate": "2016-01-20T14:06:31.790", "Body": "

Wondering if this fuse is safe to use in this switch/plug to turn on /off a 12V DC 30A Power Supply 360W Power Supply that will power a RAMPS 1.4 board for a Prusa i3 with an external led display that contains an SD Card Reader.

\n\n

I found the suggestion to use it here.

\n", "Title": "Is this fuse a good choice for my Prusa i3's power supply and RAMPS 1.4?", "Tags": "|electronics|prusa-i3-rework|switching-power-supply|ramps-1.4|", "Answer": "

No, do not use this fuse. The current rating is too high to be reasonable for your printer. It will \"work\" in the sense that your printer will get power, but it won't provide anywhere near as much protection as a lower-rated fuse.

\n\n

10A is a lot of current for mains voltage. Depending on what else you have plugged in, there is a fair chance your home's 15A breaker will trip before this fuse does, which kind of defeats the point of having it.

\n\n

Even for \"fast\" fuses, it takes a significant amount of time for them to blow when conducting their rated current. The internal fusible link has to heat up and melt before the fuse stops conducting. The less the overload current exceeds the rating, the longer that takes. A 10A fuse conducting a 10.5A short might take 30 seconds to trip. In the meantime, your printer is melting. Lower-rated fuses will trip faster for the same short and thus provide better protection.

\n\n

You need to size fuses as small as possible for the required current draw if you want to have any hope of rapidly cutting off an excessive-current event.

\n\n

I would recommend a 4A fuse in the USA for this 350w power supply. (Note: the listing title says 360 but the photos show 350.) I use 4A fuses in several printers with 120v / 350w PSUs and they do not trip. But you can do the math for yourself:

\n\n

350 watts / 120 volts / 80% efficiency = 3.64A

\n\n

The smallest fuse you can find that is larger than this value is what you should use.

\n\n

Now, we can argue over whether 80% is the right efficiency value... it could be lower. The PSU label says 6.5A input is required, but that amount of current draw implies either a <50% efficiency (which is quite poor for this kind of PSU) or would only occur for abuse/surge scenarios like starting very large motors. Such short-lived inrush events generally won't trip a fuse unless you do something dumb like lock the rotor. And none of that applies to the small microstepping-driver stepper motor systems we're working with here. This PSU should not draw more than 4A in normal 3D printer use.

\n\n

Looking at this on the other end -- how much damage will 10A do versus 4A? Lots. If the short is in the 12v system, and the PSU's short protection doesn't trip in (because it's a cheap knock-off) you would roughly multiply the AC fuse current times 10 to get the DC current. And 40A is a downright scary amount of current! Depending on wire gauge, putting 40A through heatbed wiring may make it smell and smoke. Whereas putting 100A through heatbed wiring will almost certainly start a fire.

\n\n

You're much safer with a 4A or even 6A fuse for this PSU than a 10A fuse.

\n" }, { "Id": "361", "CreationDate": "2016-01-21T00:22:14.333", "Body": "

I was just shopping for filament, and saw some glowing claims about PETG being as easy to work with as PLA, but as strong as ABS, and less brittle. Anyone know if that's actually true, or what the tradeoffs are?

\n", "Title": "Advantages of PETG filament?", "Tags": "|filament|material|print-material|", "Answer": "

I love PETG. When I first started I always used ABS because I thought it was the best and didn't see the point in using PLA. However, after a few years of playing around I no longer use ABS. I use PLA for when I am testing and PETG when I want to print something that will be used, ike parts or models.

\n\n

PETG is a little more expensive, however worth it, as it is strong and easy to use. I normally print at 220\u00b0C on the nozzle and 80\u00b0C for the bed.

\n\n

PETG has more flex to it so when you are printing parts it is less likely to break under pressure like ABS.

\n" }, { "Id": "365", "CreationDate": "2016-01-21T21:25:07.313", "Body": "

Stereolithography produces parts by projecting ultraviolet light on the top of a vat of liquid photopolymer, causing it to harden. CLIP produces parts by projecting ultraviolet light through the bottom of a vat of liquid photopolymer, causing it to harden. This seems like a minor difference, yet CLIP is reportedly much faster (I've seen numbers as high as 100x). Why is this?

\n", "Title": "Why is CLIP so much faster than SLA?", "Tags": "|clip|sla|", "Answer": "

The number of 100x could be true in some situations. I wish I could see the part(s) that they printed to measure this 100x, but that is another story.

\n\n

Looking at their videos they can move the build plate at a maximum speed of 10mm per minute. You will see the Eiffel tower video where they have to change over to standard speed for flat layers. It is because the flat layers stop resin flow and can't be printed using continuous printing.

\n\n

Prodways have shown that they can move it at 20mm per minute, but again there are things that are not advertised/mentioned. To achieve higher speeds you need to make the resins more reactive. Making resins more reactive means the resins won't last as long in the vat/bottle. So they expire sooner. It also means they could harden under normal light conditions so it makes it difficult to work with. If you look at the Prodways video you will see resin waste on the build plate. That shows that the projector brightness was set too high.

\n\n

Gizmo can print between 5 and 25 times faster than a leading brand of SLA printers, also depends on the number of parts and complexity on the build plate. The decision was made to advertise the build plate moving speed, e.g. 3mm per minute, rather than the number of times faster than anything else, because that is a value that doesn't change with the number of items on the build plate, but it does change depending on the projection area size.

\n\n

Imagine you just have a single line going upwards then most SLA printers could do it at the same speed. B9 Creator (bottom up) users have actually shown they can do continuous printing when printing really tiny items with very thin walls, because they don't have suction problems in those situations.

\n\n

Now when you add many small objects with small features on the build plate DLP printers will display the full layer in one go where laser based SLA machines need to draw out each part like an FDM machine.

\n\n

Continuous printing does have limitations. Resin needs to flow from the bottom or you will get resin starvation and cause holes so you might not be able to use it for everything. You should see it as another tool in your printing toolbox rather than the be all and end all.

\n\n

So after all that, the 100x depends on so many things, you shouldn't get stuck at looking at that number other than knowing it is a way of marketing the printers.

\n" }, { "Id": "371", "CreationDate": "2016-01-22T03:53:15.250", "Body": "

E3D-Online and Make Magazine have written about the potential damage printing carbon fiber and glow-in-the-dark filaments can do to your printer's nozzle.

\n

What I can't seem to find is what clues or warning signs to be on the lookout for if your nozzle has taken a significant amount of wear. I've printed a few hundred grams of glow filament personally and have not noticed any change in print quality.

\n

E3D says you may have "unpredictable, erratic printing" with a worn nozzle. Can anyone explain or provide examples of this and when a replacement is necessary?

\n", "Title": "How to identify nozzle wear?", "Tags": "|maintenance|carbon|nozzle|", "Answer": "

According to

\r\n \r\n
the inner diameter doesn't change much, but, as said by @0scar, the nozzle shortens and at the end you get to the inner cavity.

\n\n

Check the length and you are done.

\n" }, { "Id": "372", "CreationDate": "2016-01-22T06:03:58.597", "Body": "

I've been reading and experimenting with Acetone vapour smoothing on some printed ABS parts. My problem is that I need to selectively smoothen the printed parts which vapor smoothing doesn't allow. In particular, the cogs, whose sides I was trying to make smooth ended, up with smooth rounded tips, which was a disappointment.

\n\n

An example of what I was trying to smoothen is would be something like this: \"example

\n\n

So how can I maintain fine details (like the cog tips in the image above) while applying smoothing methods to printed parts?

\n", "Title": "Maintaining fine details while applying smoothing methods", "Tags": "|abs|post-processing|smoothing|vapor-smoothing|", "Answer": "

A technique I've used in the past is to make a acetone slurry of the same filament used to print your object, and carefully paint the details you need to smooth. You must be careful and only do a very thin coat or you may damage your print. You can add extra coat if needed to make sure the acetone has evaporatored from the previous coat of ABS filaments slurry.

\n" }, { "Id": "383", "CreationDate": "2016-01-24T12:34:09.723", "Body": "

On one of the nozzles on my printer, the filament comes out at a 45 degree angle. It seems that this causes problems with adhesion to the bed and overall quality.

\n\n\n", "Title": "Filament extrudes at an angle", "Tags": "|maintenance|extruder|extrusion|", "Answer": "

You very likely have partial clog in your hotend (the side which is clogged is the angle the extruding filament bends to) or have some plastic somehow stuck on the nozzle surface which the extruding plastic almost sticks to, like two droplets of water merging into one.

\n\n

If its the latter, clean the hotend. It's easier to remove the plastic on it while its hot and soft, but don't burn yourself.

\n\n

If it's a clog, that's more complicated.\nFirst, you can make sure if it is indeed a clog or not by extruding into the air and measuring the wire with a caliper.\nIf it's thinner than it should be and your slicer settings are fine, then its a clog.

\n\n

Ways to get rid of clog:

\n\n

1) Replace the nozzle with a new one.

\n\n

2) Break the clog with a sharp wire.

\n\n

3) Take the hotend off and clean the nozzle with acetone (\"acetone bath\").

\n\n

4) Blow torch on a removed nozzle. Dangerous thing to do for some but it works. If you're not confident or experienced to do this and the previous two solutions don't help, then go with option 1.

\n" }, { "Id": "386", "CreationDate": "2016-01-25T01:51:24.097", "Body": "

I\"m considering making my own filament, with a device like the one at http://www.thingiverse.com/thing:380987. Partly because it's another machine to build, which is cool, but also to save money on filament.

\n\n

Has anyone here tried to make their own filament? My main questions are:

\n\n\n", "Title": "Making your own filament", "Tags": "|filament|recycling|filament-production|", "Answer": "
    \n
  1. Quality depends on 3 things:

    \n\n
      \n
    1. Quality of pellets (purity, fillers, color)

    2. \n
    3. Where/how they are stored before and during the extrusion (humidity, contaminants)

    4. \n
    5. Have a filter in your extruder to get rid of random junk and air bubbles ending up in your filament (250 micron wire mesh filter)

    6. \n
  2. \n
\n\n

There's no secret formula the filament producing companies have, they just have very efficient and very fast filament producing machines (of course very expensive, too). But when it comes to vanilla ABS or PLA, it's almost the same content.

\n\n
    \n
  1. Personal experience: no. If you get the same pellets, store it in the same place and run your extruder in the same place, it should behave the same.

  2. \n
  3. I don't think there is some filament mixture you won't be able to find anywhere, but you might be able to make it yourself cheaper. Example: mixing strontium aluminate powder for glow in the dark filament (come in many colors, not just green).

  4. \n
\n\n

I'd recommend this design: http://www.instructables.com/id/Build-your-own-3d-printing-filament-factory-Filame/\nIt produces filament pretty fast (one full 1kg spoon in 3-4 hours). Just make sure you have enough experience to not electrocute yourself while assembling this as the heaters use mains power.

\n\n

I personally think the commercial \"hobby\" extruders are not worth the money. I also own the Filastruder and it's just no different and slower than the above, unless you care about a pretty plywood case for your extruder I see no advantage and since it uses off-the-shelf parts itself why bother buying a kit like that than sourcing the parts yourself?

\n" }, { "Id": "388", "CreationDate": "2016-01-25T12:37:40.963", "Body": "

When installing and using a new hotend for the first time, which steps of action should be taken before. This will probably be more applicable to chinese clones than to authentic products (is the statement true?): Should a certain cleaning procedure be carried out (removing swarf/shavings for example)? Should mechanical precision be controlled and if necessary improved (de-edging and nozzle size are two things I could think of)?

\n\n

I know the topic How should I clean my extruder when changing materials?, which is a nice addon read, but I am concerned about brand-new extruders.

\n", "Title": "Taking a new hotend into operation (cleaning, forming, etc)", "Tags": "|hotend|", "Answer": "

So far, these are my experiences to make a new hotend work properly.

\n\n\n\n

This answer is most likely not complete and totally up for discussion. I appreciate any addtional answers and comments to improve on the topic!

\n" }, { "Id": "389", "CreationDate": "2016-01-25T14:01:51.620", "Body": "

What are the specifications of the three wires inside a PC cable that is used to connect the switching power supply to a US AC outlet.

\n\n

The positive, negative and ground appear to be the same gauge stranded cable, and I've heard that it can handle 10A, but beyond that I don't really know what the rest of the specifications for the wire are.

\n", "Title": "In the standard PC Cable Wire that goes from the wall outlet to the switching power supply, are they standard and what are the wire specifications?", "Tags": "|switching-power-supply|wiring|printer-building|", "Answer": "

It's somewhat unclear what you mean by \"standard PC cable\", but virtually all desktop computers use IEC C14 sockets/IEC C13 plugs. Such connectors/sockets are rated for 10A 250V and thus you can safely assume that the cord itself will also be able to handle this voltage and current. 10A is what is specified by the IEC, certain North American standards agencies rate C13 cords for up to 15A.

\n\n

The IEC standard also specifies that the conductors inside of a C13 lead have a cross sectional area of at least 0.75mm^2 and at least 1mm^2 if the cord is longer than 2m.

\n" }, { "Id": "394", "CreationDate": "2016-01-26T05:38:44.720", "Body": "

If I need to test out some of the components of a RAMPS 1.4 based 3D Printer, can I only plug some of them into the board (not all of them) and test them out?

\n\n

I'd like to test out the NEMA 17 motors without testing the heated bed or extruder. Is this safe and why?

\n", "Title": "When building a RAMPS 1.4 based printer, can I safely plugin just some of the components to test if they are working?", "Tags": "|ramps-1.4|prusa-i3-rework|printer-building|", "Answer": "

As others have also said, this is generally fine. The main things I'd avoid are:

\n\n\n" }, { "Id": "401", "CreationDate": "2016-01-26T16:45:11.463", "Body": "

High Impact PolyStyrene (HIPS) is a frequently used filament in 3D printing. It has been touted to have simular properties when printed to ABS and is especially useful for support structures (if the user has a multi-nozzle 3d Printer) as it is soluble in Limonene.

\n\n

General Purpose PolyStyrene (GPPS) is frequently used in disposible cups, cutlery, etc. I don't believe I have seen it as a 3D printing filament. This is not styrofoam for the purpose of this question despite the identical chemical properties as the incorperated air makes it difficult to recycle.

\n\n

Would filament produced from GPPS, be usable on most FFF 3D printers that are capable of using ABS, PLA, and HIPS? Are that any particular issues observed with such filaments that would not be observed with HIPS (besides the implied decrease in impact resistance)?

\n\n

Note: For the purpose of this question, I am assuming that the filament can be produced and am concerned about whether the filament would be usable for support structures and infill.

\n", "Title": "Can general purpose polystryene (not HIPS) be used for 3D printing?", "Tags": "|filament|filament-production|", "Answer": "

In principle, it should work fine as a filament, since it's used extensively in the plastic extrusion industry, but I don't think you'd get great material properties out of it. ABS and HIPS incorporate polybutadiene into a graft polymer structure for a very good reason: the butadiene sections in the long molecular chains kind of \"stick together\" as a distinct solid phase to produce what amounts to micro-bubbles of rubber inside a matrix of hard styrene or acrylonitrile-styrene plastic. This compound microstructure is what gives HIPS and ABS favorable impact toughness and some minor flexibility.

\n\n

The flexibility is important -- the stiffer a filament is, the more it will tend to warp while printing. Based on chemistry alone, I would expect styrene to be somewhat more prone to warping than ABS. And it would certainly be more brittle. So there doesn't seem to be much reason to use it as filament.

\n\n

Interesting sidenote: PLA/PHA has very favorable mechanical properties because the PHA forms a very similar flexible microstructure inside the hard PLA matrix. PLA/PHA is good stuff because it mimics ABS and HIPS!

\n" }, { "Id": "402", "CreationDate": "2016-01-26T17:45:57.233", "Body": "

This is in with my other question about components and the other question about electricity; how can I check to see how many amps are being pulled? Can I check a component at a time to make sure I'm not going over the limit, and then just add them all in together once I've summed the amps to make sure it's safe to hook everything up. The amps shouldn't change right?

\n\n

What settings should my multimeter be set to? And to check how much it's pulling, do I just put the multimeter's leads on the green terminals on RAMPS 1.4?

\n", "Title": "How to use a multimeter to test how many amps RAMPS is pulling?", "Tags": "|ramps-1.4|", "Answer": "

If you don't want to stick a multimeter on the wire, I recommend getting a kill-a-watt meter. Pretty much, you plug it into the wall, and plug the printer into the meter, and it has a little screen that shows the result.

\n" }, { "Id": "403", "CreationDate": "2016-01-26T18:20:41.537", "Body": "

There is a little circuit board, or breadboard or something in the diagram of the wiring for the i3.

\n\n

And it's mentioned that the z-axis motors need to be wired in parallel but beyond that they don't give you much detail about parts or how the wires go in.

\n\n

Can someone provide me with some more detail on this?

\n", "Title": "How do I wire the z-axis motors in parallel on the Prusa i3?", "Tags": "|ramps-1.4|prusa-i3|printer-building|z-axis|nema-17|", "Answer": "

For some unknown reason, everywhere everybody is saying that Z stepper motors need to be connected in parallel... And this was always the only obvious way, until recently some people started to connect these motors in series.

\n\n

And I personally started to believe the right way is to connect them in series.

\n\n

All stepstick drivers are some kind of current limiting devices (you could read more about chopper mode). It is all about current.\nConnecting in series will guarantee that both motors receive the same current in all situations. And as result you could expect the same behaviour from both of them.

\n\n

The bad thing when they are in parallel, is that the motor with the bigger load will get more current and as a result the other one will get less current and could skip steps. Of course, in an ideal situation, this should never happen but don't forget about Murphy's law (\"whatever can go wrong, will go wrong\").

\n\n

One more thing - why did I change my wiring and connected my Z motors in series: At some moment I found that one Z motor was disconnected but the other one was working and this resulted in a broken printer geometry. When Z motors are in series and if one of them fails or disconnects, the other one will not work either. You will get always synchronous operation from both of them!

\n\n

I made this Z splitter that works fantastic:

\n\n

\"Z

\n" }, { "Id": "409", "CreationDate": "2016-01-27T14:26:50.577", "Body": "

I'm thinking about buliding my own 3D printer from scratch.

\n\n

Is it better to buy a starter DIY kit and try to build your printer around it, or to order separate parts for printer, and then to combine a printer?

\n", "Title": "What are the pros and cons of collecting parts yourself, versus getting a DIY kit and then modifying it?", "Tags": "|printer-building|desktop-printer|diy-3d-printer|", "Answer": "

Three great answers have already been posted, and it has been extremely interesting to read them. I shall try not to repeated what has already been said.

\n

I have sourced the parts separately for three different printers:

\n\n

I have been coding Arduinos and Pis and building robots and quadcopters for a few years now. Then, in November 2016, because I needed a prop guard for a ZMR250 quadcopter that I found hard to obtain, but easy to print, I started reading about 3D printers (mostly RepRap wiki, and then individual blogs of straight forward builds, as well as design modifications, of Prusa, P3Steel, Wilson and Delta/Kossel printers), watching countless construction videos and asking questions here on SE 3D Printers, and reading other's questions and answers, as well as going through eBay for hours at a time, looking up parts and making numerous Bill Of Materials (BOMs) and blogging the information that I gleaned. So this gave me a good grounding and starting point for when I did get around to ordering. In fact, the process is still on going...

\n

After ordering the parts, in December, piecemeal, I then had to wait for a month for the parts to arrive from China, during which time I read some more, and revised what I had already learned.

\n

I then, finally, got to work on the P3Steel, in January, but two and a half weeks later, before I had finished it, I had to move to BKK for an extended period.

\n

I suffered delays with the P3Steel build due to postal latency, obviously, but also, some partial kits where missing critical parts (see Is the 8mm x 20mm bearing axle for the X-axis idler (of a P3Steel) a custom part?), so I had to get them machined in Thailand (because it only costs around $3 to get something machined here). Hopefully, when I get back to the UK, I should have everything to hand and be able to finish the build in a few weeks maximum

\n

Once in Bangkok, I started sourcing parts for a Wilson II, and then, subsequently, a Kossel, mostly because the aluminium and steel rods are a quarter, to a half, the price that they are in Europe. Also, I had to go through the ordering process again, getting parts from China for these two printers - however, the parts from China only take two weeks to arrive to Thailand, not a month or so, for the UK. The Wilson II parts I plan to take back to the UK, in order to complete the build there, hopefully printing the plastic parts on the P3Steel, when/if the P3Steel is completed.

\n

Note, that seven, or eight months, down the line from when I first took an interest in 3D printing, I still haven't completed a single printer, yet. However, I sure as hell have learnt a lot. Note: most of the delay is due to the six month relocation away from my printer build in the UK.

\n

Also, due to my reading of the modification blogs for the Wilson and Kossel, I have recently been re-purchasing upgrades, before I have even fitted a bolt together, for the Kossel and Wilson, let alone completed either of the base builds. For example, I have just purchased Chinese aluminium vertices, rather than the plastic PLA prints that I got from Sintron. So I have ended up with a fair collection of spare (redundant?) parts, but again, it has helped me gain a great insight as to what works well, and what does not.

\n

To reiterate that which TestGeek has said, one major tip I would have is (and I read this on a forum when I was first getting into 3D printer building), if you are sourcing the parts separately, is to buy bulk (get packs of 10 pcs, 50 pcs, 100 pcs), and buy more than you require: nuts, bolts, bearings, GT2 belt, GT2 pulleys. The price comes down phenomenally, and you can resell the spares, locally, for about as much as you paid for the whole lot, online, thereby covering, or almost covering, the cost of the printer. Plus, spares come in handy for further builds down the road. Don't buy anything from the US (unless you are already in the US, I guess) - the import/postage fees are outrageous.

\n
\n

TL;DR

\n

In the same way as you learn more from building a kit as opposed to buying a pre-built printer, you will learn more sourcing the parts separately, but it might cost more, in redundant parts. Also:

\n\n" }, { "Id": "410", "CreationDate": "2016-01-27T14:30:41.597", "Body": "

There are a big variety of them that can be found at the market. Some of them have similar characteristics, the other varies from each other.

\n\n

What are props and cons of cheaper filament vs expensive filament?

\n\n

How to choose which filament to use?

\n", "Title": "How to choose a right 3D printer filament type?", "Tags": "|filament|", "Answer": "
    \n
  1. Determine what properties you need the filament to have.

    \n

    There are a very wide variety of filaments because they all have somewhat different properties. You need to determine what properties you need your final print to possess. For instance, ABS can be smoothed used acetone and PLA is biodegradable. More exotic filaments could be conductive or be extremely flexible. Do not neglect to consider safety of the plastic in your application. A full list of properties is too broad for an appropriate answer from this site.

    \n
  2. \n
  3. Determine what you printer is designed to handle.

    \n

    Most printers can handle PLA and ABS with little issue but higher temperatures are required to use some materials such as nylon and polycarbonate. Determine beforehand what your printer is designed to handle. This includes determining whether you have and appropriate heated bed or other accessories.

    \n
  4. \n
  5. Determine what you can afford.

    \n

    Printing not only requires the cost of the weight of filament in the final print but all of the support structures and misprints you produce. Not only should you look for inexpensive filaments, but you should look for ones that are good enough quality you don't waste time and money fighting problems with the raw material.

    \n
  6. \n
\n

All in all, this will vary case by case. I would recommend that you start with PLA or ABS as they are common and relatively easy to work with but that you do your own research into what each individual filament type has to offer..

\n" }, { "Id": "413", "CreationDate": "2016-01-27T16:25:12.577", "Body": "

How do I smooth 3D printed objects? What is the best / common method to do this?

\n", "Title": "Methods for smoothing 3D objects", "Tags": "|post-processing|smoothing|", "Answer": "

Get a rock/jewelry tumbler and some tumbling media such as stainless steel shot, and try tumbling your print.

\n\n

For 3D printed plastic, your print will (a) need to be sturdy, and (b) not have any fine details or small parts that you don't want to be worn away. With metal you will tumble it for hours in order to smooth and semi-polish the surface. With plastic, I would recommend a much shorter time period.

\n\n

One person tried this using screws as his tumbling media, and has some interesting results to show for it. Part 1, Part 2, Part 3, Part 4. Basically, you can get a metallic coating on your plastic print that is made up of tiny bits of the tumbling medium. You'll need to add a protective coating to keep it from rubbing off, but it's cheaper than buying metallic filament.

\n\n

(Thanks to Mark Walter's comment for the linked articles)

\n" }, { "Id": "423", "CreationDate": "2016-02-01T11:50:29.077", "Body": "

I recently found out carbon fiber and glow in the dark PLA can damage the printer nozzle, now I'm suspicious of all the \"exotic\" filaments.

\n\n

So, does wood filament cause damage to the nozzle? (under normal use, or at least what someone who only used PLA/ABS before would consider normal use)

\n\n

Let's assume a normal quality brass nozzle - not some cheap stuff that didn't even came in the correct size to begin with and not some premium reinforeced nozzle - and reasonable quality filament.

\n", "Title": "Does wood filament damage the printer nozzle?", "Tags": "|filament|filled-pla|", "Answer": "

Wood PLA is too abrasive for a brass nozzle and will wear it out until it becomes a straight pipe after about 12 hours of printing with it.

\n

This answer is based on first hand experimentation. I'm attaching photographic evidence of what to expect if you do not use hardened steel at minimum for Wood PLA.

\n

This image shows that the nozzle after a 48 hour print is a complete straight pipe. It was mostly a straight pipe after about 12 hours. I let the print complete to take stock of the entire result.\n\"Nozzle

\n

Here I am measuring a piece of filament that's been extruded after running the Wood PLA through the nozzle for 48 hours.\n\"Extruded

\n

And here is how the layers degrade. When the issue started I thought maybe I had a clog, however my extruder was exhibiting perfect extrusion behaviour. The result looks like inconsistent flow, and it is. The extruder is moving at a rate appropriate for a 0.4\u00a0mm output, but the nozzle is now 1.7\u00a0mm wide.\n\"Layer

\n" }, { "Id": "442", "CreationDate": "2016-02-02T10:35:52.800", "Body": "

In general 3D printers are compact and smaller than RP machines. That's ok. But, what's the difference? 3D printers can be used as RP machine too.

\n", "Title": "Whats the difference between a basic rapid prototyping machine and a 3D printer?", "Tags": "|rapid-prototyping|", "Answer": "

A sintered metal printer is a version of a 3D printer that is rapid, but expensive. Seen 1 for 800,000$aud\nUses laser to melt metal particles like titanium.

\n" }, { "Id": "450", "CreationDate": "2016-02-03T01:01:00.623", "Body": "

I have a Robo 3D. However A while ago, the print bed was fractured, and now it has a long crack cutting it in half. The bed still works because it is held together, by the screws holding the bed to the tracks. So I want to continue using it, because it still is fairly good, the heating element works fine, and a replacement bed is 80$, and I am unsure if the one sold on the RoBo 3D website will be compatible with my printer as I don't have the R1, but a version before that.

\n\n

\"My

\n\n

So my question is:\nHow can I best align the two glass fragments, to provide as flat a print surface as possible, and two how to best hold these two pieces in place, or if it would be best to invest in a new print bed?

\n\n

EDIT:\nHere is a image of the heating element as well to explain the situation with how it is attached\"The

\n\n

EDIT:\nThe RoBo 3D team have said that I just need to upgrade my y-axis with a object on thingiverse and then buy their new build plate. So I am going to experiment with a glass replacement, and if that fails to succeed then I will go along with their suggestion, and buy the upgrade. Thanks everyone for their help.

\n", "Title": "Broken Print Bed", "Tags": "|heated-bed|hbp|", "Answer": "

I agree with several others that your best bet is to replace the glass entirely. But short of that, you might try something like Loctite \"Glass glue\", which is essentially crazy glue for glass. Be sure to level and clamp well while drying, or you'll end up with it permanently not straight of course. You'd want it clamped to a very flat surface, and clamped pushing the broken edges together. But I think I'd just replace it.

\n" }, { "Id": "454", "CreationDate": "2016-02-03T06:42:47.657", "Body": "

I'm using Cura as my slicing/printing software and I just started using the BuildTak printing surface.

\n\n

The BuildTak is damaged by pushing a hot nozzle into it and my printer's (Robo3D R1+) autoleveling feature works by pushing the nozzle into the build surface.

\n\n

Is there a way to configure Cura so that it runs the Z probe first, then heat up the nozzle?

\n\n

My first sheet of BuildTak already has 10 small holes in it (at the homing position and at the 9 leveling touch points)

\n", "Title": "How to configure Cura to run the Z probe before heating", "Tags": "|software|ultimaker-cura|slicing|calibration|z-probe|", "Answer": "

As pointed out in Markus's comment to Tormod's answer, {print_temperature} needs to be replaced by {material_print_temperature}, so the code now becomes:

\n
G28          ;move printer to endstops (the home position)\nG92 E0       ;zero the extruded filament length\nM565 Z-1     ;set z-probe offset     <-----   ( YOU HAVE TO ADJUST THIS, READ BELOW)\nG1 Z5 F5000  ;move the printer 5mm above the bed\nG29          ;run auto-leveling\nM109 S{material_print_temperature}    ;set nozzle temperature, and wait for it heat up\n
\n" }, { "Id": "458", "CreationDate": "2016-02-03T09:30:32.887", "Body": "

I'm thinking of another extruder on my printer, and I'm curious about this one....

\n\n

Is it necessary to have both hot ends on same height? Why yes / why not? (if there is not)

\n", "Title": "Is it necessary to have both hot ends on a dual extruder printer at the same height?", "Tags": "|printer-building|hotend|dual-nozzle|calibration|layer-height|", "Answer": "

To add to the above answers, besides the obvious point that one hotend might collide with something another hotend positioned lower than it has printed, you also want both hotends to be positioned X microns above the bed at minimum height so your filament will stick to the bed properly. Position one a bit higher than the other and what it prints won't stick to the bed very well, position one a bit lower and it will hit the bed and clog/be unable to print the first layer.\nThe only reason I can think of when you'd want one hotend to be higher is a situation when you're not using both hotends and don't want the second hotend to drag on and deform a layer the main hotend has printed and is still warm and deformable. If that's an issue (I doubt) then sure, move it a bit higher.

\n\n

If there are other reasons for what you would like to have the hotends at different heights, please add that to your question. I can't think of one myself.

\n" }, { "Id": "461", "CreationDate": "2016-02-03T10:39:03.100", "Body": "

I have searched the internet and found various 3D printers with different advantages and materials which they can print - some even multi color.

\n\n

However, I cannot seem to find a printer that can print multiple material with different properties; for instance, simultaneously printing PLA and metal. Is there currently such a printer available or in development?

\n", "Title": "Printing multiple materials with different properties on the same device", "Tags": "|multi-material|", "Answer": "

For the most part, you can achieve this with a dual extruding printer. However, dual extrusion is best for either multi-color printing or printing with support material. For example, printing the part with PLA and all support material with water soluble PVA.

\n\n

In practice, printing two completely different materials is not sound engineering practice as they have the potential to not make a well enough bond to each other. So, the case of pure metal and pure plastic, the two materials will not bond well because they will not both be in the same physical state together at any point in time.

\n\n

However, your best option would be a printer like the MarkForged which uses a composite approach by combining a common binder (ABS, PLA, Nylon, etc.) and a strong material such as Carbon Fiber, Fiberglass, or Kevlar.

\n" }, { "Id": "505", "CreationDate": "2016-02-05T18:24:43.220", "Body": "

Glass is always level, easy to clean, easy to work with.

\n\n

Aluminium allows for the addition of automatic bed leveling with an inductive sensor and distributes heat a little more evenly.

\n\n

When printing mostly ABS and PLA, which one is better?

\n", "Title": "Ideal print bed: Glass or Aluminium?", "Tags": "|heated-bed|hbp|", "Answer": "

I have used glass from the beginning and it works perfect.\n(From old picture frame and old scanner)

\n\n

I have tried glass with tape,\nglass with hairspray and have had very good luck on printing on both.\nPrefer hairspray :)

\n\n

I have tried printing on the heated aluminum, but experienced the aluminum plate tend to warp due to temperature changes.

\n\n

With a glass plate on top of the aluminum, you have a flat surface that don't warp. :)

\n\n

So I would give glass thumps up!

\n\n

EDIT: Printed on Glass with glue-stick. Wow. Sticks really good.

\n" }, { "Id": "507", "CreationDate": "2016-02-05T22:18:16.727", "Body": "

I currently print with a .4mm nozzle on my extruder, and my prints seem to come out fairly accurate; would I see much of a difference if I went to a .3mm?

\n\n

What are the pros and cons of larger and smaller nozzle sizes?

\n", "Title": "Will I see a noticeable difference switching from a .4mm to a .3mm nozzle?", "Tags": "|fdm|extruder|nozzle|", "Answer": "

1) Smaller nozzle advantage: sharper \"corners\" (higher X and Y resolution)

\n\n

2) Larger nozzle advantage: faster 3d printing (because you can print the shell faster as each perimeter can be thicker so you'll need less perimeters to be printed to get the same shell thickness. Same true for infill).

\n\n

3) Smaller nozzle disadvantage (varies, debatable): higher risk of clogging. There are of course other factors which can result in clogging, but this can also affect it as even finer particles in less quality filament can get stuck now.

\n\n

4) Smallel nozzle disadvantage: since less plastic can come out at a given duration of time, it means less speedy printing of larger layer heights, if at all possible.

\n\n

I feel like I should get back to (1) and explain why it is so.\nWhen companies market their 3d printer they usually talk about the layer height (the Z resolution) completely ignoring the X and Y. This makes sense for marketing. Luckily it is not hard to explain what is usually left out and what is important for one of the answers to your question on what are advantages of smaller nozzles:

\n\n

The Z resolution depends on two main things:\n1) The stepper motor driver accuracy of your 3d printer's motherboard. Pretty much all are the same now. Claiming your 3d printer can print at 20 micron layer height doesn't say anything, pretty much any 3d printed today can.\n2) Nozzle/extruder quality, nozzle diameter. The latter surprisingly does not determine much. Because of how FFF/FMD 3d printing works, the perimeters of your 3d print can be way thinner than your nozzle diameter. This might sound odd but there's a simple explanation: the molten plastic coming out of the hotend is squished and stretched when the head moves. Because it is stretched, it can be made thinner as you increase how much it is squished. There's not much experimentation done on this and available online, only from personal experience I can say I can print perimeters 50% thinner than my 3mm nozzle. So I suppose if you wanted thinner perimeters than that, you would need to swap to a thinner nozzle.\nWhy would you want thinner perimeters? To get sharper corners or in other words better X and Y resolution for your 3d prints which is ignored by most unlike the overstated \"layer height\" (Z resolution).\nHere's an illustration to help you visualize how fatter perimeters and less sharp corners/edges on your 3d model result in \"lower X/Y resolution\" (the illustration is a 2d cutout, viewed from the top):

\n\n

\"enter

\n" }, { "Id": "510", "CreationDate": "2016-02-05T23:53:48.623", "Body": "

I always assumed that the smallest diameter that you could extrude was equal to the diameter of the extruder tip itself. Recently I heard that you can actually extrude a line that's as little as half the width of the nozzle opening.

\n\n

Is that true in theory? In practice?

\n\n

My printer's 0.3mm nozzle broke recently, and the only replacement on hand was a 1.2mm nozzle. The prints have been fast, but really coarse and imprecise. I've dialed the extrusion diameter down from about 2mm to 1.2mm for some smaller, more intricate parts, and it worked fine. Can I go down to 0.9mm or 0.6mm extrusion diameter without it screwing up my prints?

\n\n

If I can't do that, what does happen if I try to print 0.6mm out of a 1.2mm extruder nozzle?

\n\n

For this question, let's assume an FDM printer using PLA and/or ABS filament.

\n", "Title": "How thin can my extrusion be from a 1.2mm extruder?", "Tags": "|print-quality|extrusion|nozzle|", "Answer": "

The main issue with very narrow extrusion widths (less than the nozzle size) is that you get really poor \"nozzle squash.\" The plastic isn't pressed down very hard as it's extruded. That causes poor layer bonding and weaker prints.

\n\n

The flow of molten plastic coming out of the nozzle must be drawn down by tension in order to end up smaller than the nozzle opening, and that drawing effect makes it difficult to get good contact with the previous layer. So you end up with a \"stack of logs\" structure kind of like this:

\n\n

\"0.4mm\n(This photo actually shows layer height = extrusion width = nozzle size, which is another no-no, but the end result is similar.)

\n\n

What you want instead is something very \"mashed together\" and strong like this:\n\"0.4mm

\n\n

How finely the the strand can draw down -- versus smearing or breaking apart into blobs -- will depend greatly on the type of filament and the printing conditions. For example, nylon draws down extremely well because it experiences a large degree of strain crystallization, and actually gets stronger and stiffer as it's stretched. PLA tends to get more brittle when strained and may break into stringy blobs. ABS doesn't change terribly much because it's highly amorphous. Polymer type, blending agents, print speed, printing temperature, and cooling will control the drawing behavior of the filament.

\n\n

Another factor to consider is what the strand spacing is within the print. If you set the extrusion width equal to half the nozzle width, depending on the slicer, adjacent strands may be placed with pitch equal or slightly less than half the nozzle width. So after the first strand is printed (say an unimportant inner perimeter) the next strand will be printed with the nozzle partially overlapping the previous strand. This effectively creates a constriction to force the plastic out of a smaller opening to one side of the nozzle, which is not too different from printing with a smaller nozzle. Subsequent perimeter strands may then tend to \"heal up\" even if the first is low quality. This effect doesn't completely fix the problem, but it helps account for why under-width strands seem to work better than they ought to.

\n\n

Of course, even if the strands turn out ok, it's arguable whether there is any benefit. If the goal of a smaller extrusion width is more precision and sharper corners, you may not actually accomplish that goal. When the filament is being drawn down by tension in the cooling polymer, it may tend to \"cut corners\" and get pulled into a rounded arc when the nozzle changes direction. That may defeat the point of using a thinner extrusion width in the first place. Generally I only recommend using under-width strands to capture thin-wall detail that is only a little bit too thin for the nozzle, not as a way to improve overall print quality.

\n\n

In comparison, having the extrusion width equal or wider than the nozzle ensures the plastic is firmly squished downward into intimate contact with the lower layer. You can fudge this a bit and still get decent results, but it's usually best to keep the extrusion width equal or larger than the nozzle size. There's no hard cutoff where it will / won't work.

\n" }, { "Id": "516", "CreationDate": "2016-02-07T00:33:09.337", "Body": "

I'm printing cylindrical pins for a spool holder, but all my pins have irregularities on the external surface of the cylinders (see photo for partially printed pin with irregularities).

\n\n

It's hard to tell for sure, but it appears that the hot end will occasionally move a piece of soft plastic it previously laid down.

\n\n

I've tried decreasing print speed, travel speed, and layer height but I still get the irregularities.

\n\n

What parameters are most likely to effect cylinder regularity?

\n\n

\"Cylinder

\n", "Title": "What parameters affect cylinder regularity?", "Tags": "|print-quality|", "Answer": "

This is probably a result of bad quality filament. If you haven't changed much in your setups, haven't changed slicers, but may have changed spools, then this might be the result. I would try using another spool and perhaps switch suppliers. I know it's expensive and there seems to be a stigma around it, but MakerBot Industries provides quality PLA (some ABS also).

\n\n

Other than that, you've done some of what I would suggest in this case which are the following:

\n\n\n\n

My only other suggestions would be:

\n\n\n\n

I would also recommend doing some in depth maintenance to eliminate in obvious problems (such as cleaning your nozzle, rods, belts, etc.)

\n" }, { "Id": "519", "CreationDate": "2016-02-07T15:03:18.640", "Body": "

Is there anything I can do to improve the bottom side of surfaces that are printed on support?

\n\n

I always try to rotate the model so that surfaces that need to look nice are on the sides or top, and, if possible I divide the model into smaller parts to minimize the amount of support.

\n\n

But sometimes there is an object that has an irregular shape that need support no matter how I rotate it and can't be broken down into smaller objects with no support.

\n\n

And then, after removing the support I get a very ugly ridged face that takes forever to sand.

\n\n

To clarify, I'm not asking about making supports easier to remove, I don't have a problem with this, I'm talking about making the surface that touched the support better looking after removing the support.

\n\n

Is there anything I can do to improve the look of the face that is printed on supports?

\n\n

I'm using Cura to do the slicing and printing, I'm using the \"lines\" support type, my printer is the Robo3D R1+

\n", "Title": "How to improve face that are printed on support?", "Tags": "|print-quality|support-structures|", "Answer": "

The basic answer to your question is to create better bridging or overhangs on your machine. Bridging is when you are printing between two solid pieces (like a bridge). Overhangs occur when printing off of a single solid piece and coming back. Most slicing engines allow extra settings for speed, fan power, etc for these parameters, just refer to this terminology.

\n\n

A very common and simple solution in getting features to 3D print properly is to just slow down! If you're not a in a rush to get the part done, you bring down all of the feedrates in your slicing engine. MakerWare has mine at 90/150 mm/s for print and rapid. Typically I'll bring that down to about 50/90. My reasoning is the more time you give the plastic to cool, both while and after printing the layer, the more rigid that layer will be for the next one. When briding or overhanging, there will typically be a sagged area in the print. You can minimize this by providing the plastic more time to cool. Also keep in mind that printing on supports is still technically bridging (printing between two solid pieces).

\n\n

Another thing to keep in mind is adhesion from one layer to the next matter just as much between the current layer and the previous as the current strand to the one next to it. So, in some cases, increasing your shell could possibly make it easier for the infill/roof/floor strands.

\n" }, { "Id": "522", "CreationDate": "2016-02-07T21:23:27.297", "Body": "

My printer has an auto-leveling feature that works by touching the build plate with the tip of the nozzle.

\n\n

I started using a BuildTak surface and BuildTak is damaged when you push a hot nozzle into it.

\n\n

So I edited the start G-code to run the auto-leveling before heating up the hotend

\n\n

But ABS doesn't stick to the build surface unless I pre-heat the hotend and wait about a minute.

\n\n

So now I'm looking for a G-code command to put at the end of the start G-code that will make the printer wait a minute before printing

\n\n

The sequence I'm looking for is:

\n\n\n\n

Any way to insert a delay into the G-code?

\n\n

I'm using Cura to slice/print, my printer is Robo3D R1+

\n", "Title": "Is there a G-code for waiting?", "Tags": "|g-code|", "Answer": "

Thank you Fernando Baltazar for the G-code you used to solve the hot end cooling issue after bed levelling probing. I did change M190 to S60 but kept everything else. This worked.

\n
G29 ; Autonivel\nM190 S35 ; set bed temperature\nG1 Z4 F240 ; lift nozzle\nM109 S195 ; wait for temperature to be reached\nG21 ; set units to millimeters\nG90 ; use absolute coordinates\nM82 ; use absolute distances for extrusion\nM104 S210 ; set temperature\nG92 E0\nG1 E-1.5000 F1800\nG1 Z0.225 F240\n
\n" }, { "Id": "540", "CreationDate": "2016-02-09T19:47:43.250", "Body": "

I just added a fan to my printer because very small layers seem to come out very badly. For example, the 5mm PLA cube that's the top level of the test shape shown below. Watching closely, I can see that the newly-extruded fiber is pushing the previous layer(s?) around pretty freely. And when the object is finished, the little top cube is bulging, rounded, and still soft to the touch.

\n\n

The 2nd-to-top level of the object is also quite small and quick, but often comes out nicely (if anything, it was better before I added the fan).

\n\n

The fan is a squirrel-cage with about a 2.5cm square outlet, pointing at the nozzle from about 5cm away, running full speed. The extruder is a Mk9 from http://www.makergeeks.com/duexretopr.html.

\n\n

I also tried telling pronterface to wait if a level was too brief, but that setting seems not to do anything.

\n\n

What else can I try? It seems like this is a not-enough-cooling problem, but perhaps something else too?

\n\n

\"A

\n", "Title": "Mushy small top layers?", "Tags": "|fdm|print-quality|maintenance|cooling|", "Answer": "

I believe you might be having an issue with insufficient cooling. Remember the suggestion to have wait times per layer to let the previous layer cool? Well, I think with your setup and print not only do individual layers overheat, but individual parts of each layer. You either need to try more direct or dual fan cooling or maybe use a nozzle with thinner outer diameter which will have less surface to heat the layed down plastic surrounding it.

\n" }, { "Id": "542", "CreationDate": "2016-02-09T20:56:38.667", "Body": "

Most electronics use micro-controllers like an AVR, but I'm seeing ARM chips in new electronics. ARM chips are said to be more powerful, but in what areas related to 3D printing could this help? What are the features that the AVR struggles with and where an ARM could be better?

\n\n

High Speed movement? Delta printers? Graphic display?

\n\n

And is the AVR really the limitation there?

\n", "Title": "What is the benefit of using an ARM based electronics?", "Tags": "|electronics|", "Answer": "

AVR processors are 8 bit - thus they can only fetch data from memory 8 bits at a time - while an ARM is 32 bit and can fetch data 32 bits at a time. Position resolution requires a 24 bit value minimum - this means the AVR takes 3 data fetches for the position - whereas the ARM takes 1 data fetch.

\n\n

Worse still, AVR processors internally divide the clock by 3, so that a 40 MHz AVR is running at 13.3 MHz typically, while an ARM is 1 clock cycle per bus transaction and instruction processing - this includes a 32 bit by 32 bit multiple in 1 clock cycle.

\n\n

The memory map on ARM processors is 32 bit wide or 4 GB while 8 bit processors only have an address bus of 16 bits or 64 KB - which means that bank switching comes into play on any program over 64 KB - this takes instructions and time to be done - while with the ARM this is not an issue.

\n\n

The cost aspects is about the same as AVRs - it just requires redesigned firmware.

\n\n

As for FPGAs:

\n\n\n\n

The result of adding a FPGA to a AVR will cost a lot more than going to a more powerful ARM processor.

\n" }, { "Id": "554", "CreationDate": "2016-02-11T01:41:53.370", "Body": "

I want to upload some of my works to Thingiverse without making my real name public (displaying it on the profile page).

\n\n

I think it is OK to register my name to the site unless the make it public, and it is required by the terms to provide truthful and accurate information.

\n\n

I tried creating an account on the site, but I deleted it because I couldn't find the way to hide my name (set another one) from the profile page in a short time.

\n\n

I see some accounts that doesn't seem showing their real name (for example, their name on profile equals to their account ID, or at least not in two parts: first and last name as required on registration), so I guess this is archivable.

\n\n

examples:

\n\n\n\n

How can I set my name for profile page on Thingiverse after registration and logging in?

\n", "Title": "How to upload my works to Thingiverse without making my real name public", "Tags": "|makerbot|3d-models|", "Answer": "

To change your displayed name (as opposed to username) in Thingiverse:

\n\n
    \n
  1. Go to your profile page
  2. \n
  3. Click \"Edit Profile\" on the info column on the left
  4. \n
  5. At the top, next to \"Thingiverse Settings\" is another link/tab called \"Makerbot Settings\". Click that.
  6. \n
  7. Change the First Name and Last Name fields, and save.
  8. \n
\n\n

Note that neither First nor Last Name is required; if neither is provided, your username will be displayed in place of your display name.

\n" }, { "Id": "557", "CreationDate": "2016-02-11T17:54:03.270", "Body": "

What operating systems are available for running the Autodesk Spark Print Manager? (https://spark.autodesk.com/developers/reference/printer-manufacturers/integrate-your-printer/integrate-your-printer-model)

\n\n

I'm finding the documentation a little difficult to parse. It does seem that there are only build options for Windows or Mac, but I want to confirm.

\n\n

Thanks!

\n", "Title": "Autodesk Spark Print Manager Operating Systems?", "Tags": "|print-api|", "Answer": "

I checked https://spark.autodesk.com/developers/reference/desktop-applications/print-manager. It seems that Print manager is a application written in JavaScript and its source code is available on github.

\n\n

So you have to install node.js to run it. So you can use all major OS (win, mac, linux, ...).

\n" }, { "Id": "572", "CreationDate": "2016-02-13T20:50:47.553", "Body": "

There are many hot end designs out thee and one difference is the length of the melting zone.

\n\n

The melting zone is that part of the hotend where the pushed in filament is heated up and therefore starts to melt. Is it better to have a short melting zone, or a long melting zone? What are advantages of each solution. And are there materials that benefit from a short/long melting one more than others?

\n", "Title": "What is the best length of the melting zone in the hotend?", "Tags": "|hotend|3d-design|", "Answer": "

Short melting zone melts material in small amounts which is suitable for thin layers with small nozzles. In opposite, long melting zone can heat a big amount of filament which is needed for fast printing and thick layers.

\n

short melting zone

\n\n

long melting zone

\n\n

Example

\n

Example of those hotends are E3Dv6 and E3D Volcano

\n

E3Dv6

\n

\"enter

\n

E3D Volcano with a much longer melt zone than v6

\n

\"enter

\n

Speed comparison video:

\r\n \r\n

\n

Speed comparison graph:\n\"e3d

\n

Update

\n

CNC Kitchen YouTube channel did a review of 0.1\u00a0mm nozzle, where the author discusses problems with very low material flow. The main point is the usage of high-quality materials (he uses Polymaker PolyMax PLA) and avoiding materials like PVA, which tend to crystalize under heat.

\n" }, { "Id": "573", "CreationDate": "2016-02-13T21:17:30.833", "Body": "

everyone.

\n\n

I'm a relative newcomer to 3D printing, so I don't know what constitutes an unacceptably bad spool of filament.

\n\n

About 1/6 of the way into a roll of PETG (and maybe 4 hours into a 6-hour print), an over/under wrap brought things to a screeching halt. I aborted the print, then snipped the filament and started unspooling it, looking for more cross-wraps. I found a ton of them, along with a ton of kinks.

\n\n

I stopped about 1/3 of the way into the spool, still finding kinks and cross-wraps, and said to heck with it. The only way to use it would have been to run the entire length onto another spool, carefully avoiding cross-wraps, and hope the kinks wouldn't affect the print quality.

\n\n

I complained to the supplier but never even got a reply, so now I'm wondering if this is just one of those things I can expect from time to time. Any thoughts & opinions would be much appreciated.

\n\n

Cheers,

\n\n

Glasseyed

\n", "Title": "Gnarly filament: just one of those things, or grounds for complaint?", "Tags": "|filament-quality|", "Answer": "

Knot happens when you loosen the filament yourself and then leave it like that or try to manually re-spool it.\nTo avoid this kind of situation as well as the curly filament coming off the spool and getting tangled there, try to not loosen it and also build or print a \"spool guide\" for your 3d printer: http://www.thingiverse.com/search/page:1?q=filament+spool+guide&sa=

\n" }, { "Id": "582", "CreationDate": "2016-02-15T16:26:24.543", "Body": "

I was wondering if this printer(daVinci 1.0) had the ability to print very small objects, like insects, coins, or small nuts. (About the size of 1 -2 cubic centimeters)

\n\n

Here is a link to the printer on the website.

\n\n

The reason I ask is someone asked me if it was able to, but I have not been able to access the actual 3-D printer for use at this time, just manuals which I have looked through.

\n\n

So if the 3-D printer was able to print small objects, would a novice be able to do such a thing?

\n\n

Please let me know if any additional details are needed.

\n", "Title": "How small can I expect FDM 3d printers to print?", "Tags": "|desktop-printer|print-quality|", "Answer": "

Strictly by looking at the technical specifications of an FDM printer, there are a few things to note regarding the maximum print quality you can expect to achieve:

\n\n

Minimum layer height:

\n

On a finished print, the minimum layer height will affect how visible the horisontal lines of the print will be. Printing at a lower layer height can dramatically increase the smoothness of the finished part, while equally increasing the printing time - among several other things.

\n

A minimum layer height of 0.1 mm is fairly common for low-cost desktop FDM printers.

\n

Nozzle diameter:

\n

Just like the layer height defines the vertical resolution of a print, the nozzle diameter defines the sharpness of horizontal features of a print.

\n

When printing with a large nozzle diameter, all sharp edges and corners of the model will have a slight roundness to them: the larger the nozzle diameter, the more rounded sharp corners will be, and vice versa. You might think of it as making a detailed drawing with a blunt pencil.

\n

A nozzle diameter of 0.4 mm can perhaps be considered the standard for desktop FDM printers today, and will allow you to print "fairly accurate" parts.

\n

Will the daVinci work for you?

\n

Since I've never worked with the daVinci printer before, I cannot make a statement on it's user friendliness, or actual performance.

\n

In general, if you intend to use it to print fragile, complex models such as insects, I believe a resin based printer might be more right for you, since they typically will allow for much higher reproduction of details than FDM printers. Printing solid/compact structures like coins and nuts, on the other hand, can typically be handled by a well calibrated FDM printer.

\n" }, { "Id": "588", "CreationDate": "2016-02-16T18:37:23.413", "Body": "

I've got a dead MakerBot Replicator Dual and I'm not really interested in fixing it with about half the parts being proprietary, discontinued, and therefore very expensive. So, I'm wondering if it's possible to use a controller board used on some of the other RepRap machines.

\n\n

I'd assume that it's all relatively the same, mostly need a means of routing/regulating power to the servos/fans/etc. based on a g-code. If that's true, then I'm also assuming the next step is ensuring the slicing engine spits out the right format for the board (.s3g, .x3g, .gcode, etc.) and that the slicing engine knows the proper bounds for the machine.

\n\n

Side question, if I used an another controller board, would I be able to still run MakerWare/MakerBot Desktop? So far, I've preferred the MiracleGrue slicer, but I've used Cura on Octopi and it'd suffice.

\n", "Title": "Is it possible to use an aftermarket controller on a MakerBot?", "Tags": "|fdm|makerbot|replicator-dual|makerware|fff|", "Answer": "

You can put pretty much any controller in pretty much any printer, with a few noteworthy details you need to keep in mind for a Replicator 1.

\n\n
    \n
  1. Makerbots use thermocouples. Most RepRap style 3d printers use thermistors. So you would need to replace the extruder temp sensors, or choose a new controller with thermocouple support.
  2. \n
  3. Makerbots use 24v heaters and fans and PSUs. Most RepRap style 3d printers run on 12v (although this is slowly changing because 24v is better for performance and safety.) You can either pick a new controller that is known to be compatible with 24v, or replace all 24v-rated components with equivalent 12v-rated components.
  4. \n
  5. The existing heatbed thermistor circuit is probably not the same kind of circuit that your new controller will expect. Older Makerbots use a small voltage divider circuit on the HBP PCB to convert the thermistor resistance to a voltage signal, whereas RepRap style controllers typically have this circuit on the mainboard and thus need only a simple thermistor hookup. You could add a new thermistor, or modify the existing circuit, or get an entirely new heatbed.
  6. \n
  7. The existing endstops are full +5v,GND,Signal style mechanical switches with debounce capacitors and LED indicators. These should be compatible with most controller boards given proper firmware setting, but you may need to do some creative wiring if there isn't a suitable three-pin endstop port on your chosen board.
  8. \n
  9. The LCD and 5-button panel will not work with other boards. You'll need to replace that or just run a remote host via USB.
  10. \n
  11. Connectors may be different, of course. That's always an issue with any board swap.
  12. \n
  13. You will need to export RepRap style gcode instead of .x3g files. Makerware/Desktop can export gcode but there may be some minor differences in Mcode assignments from what most RepRap style firmware will expect. I couldn't tell you offhand whether this will work, but to be safe, it would probably be best to use a more traditional RepRap slicer (Slic3r, Cura) or Simplify3D.
  14. \n
\n\n

So that's a healthy to-do list. At a certain point you're practically building a new printer. But there is a significantly easier approach than installing a RepRap style controller: replace the dead Makerbot Mightyboard with a compatible community-designed Sailfish board like the Azteeg x3 by Panucatt or new CloneBoard Mini by Carl Raffle. Or, if you're ok with Chinese sourcing, a FlashForge Mightyboard will be a near 100% drop-fit replacement for your Makerbot Mightyboard. Any of these options will give you fresh new electronics with minimal hardware changes in the rest of the printer.

\n\n

Either way is fine and you'll end up with a working printer. Using another Sailfish board will certainly be easier, if you don't object to staying within the Sailfish/x3g toolchain and Makerbot/Clone parts ecosystem. Makerbot does have a very bad community reputation these days, but it's important to remember that the Replicator 1 Dual is a completely open-source machine from the \"pre-evil\" days, and compatible spare parts are widely available from literally dozens of vendors. The Rep1-derived Makerbots and Clones collectively comprise the most popular single \"flavor\" of 3d printer in the world (totaling as much as perhaps a quarter of all consumer/hobbyist 3d printers sold to date) and many/most of the spare parts for Replicator 2/2x's and clones will also fit in a genuine Rep1.

\n" }, { "Id": "590", "CreationDate": "2016-02-17T00:27:13.077", "Body": "

I am wondering if I can get some electronic and mechanical layout of the machine.

\n\n

Is it possible to make 3D printer components with 3D printer?

\n", "Title": "How to make your own 3D printer?", "Tags": "|printer-building|reprap|", "Answer": "

Yes, look at http://reprap.org for a project that's been doing what you're asking about for about a decade now.

\n" }, { "Id": "594", "CreationDate": "2016-02-17T14:42:23.657", "Body": "

I've heard alot about the need for ventilation when using 3D Printers lately. Is there any special sort of filter that would prevent the toxic gases from leaking out of the printer while it's printing it's filament? Is printing outside a solution while I build an enclosure?

\n", "Title": "What kind of filter do I need for the enclosure of a 3d printer?", "Tags": "|filament|abs|pla|ventilation|", "Answer": "

For the most part, a consumer 3D Printer will only need proper ventilation when using potentially harmful materials such as ABS. (See duplicate question). If you're printing with primarily PLA, then you don't need to worry. I print mostly with ABS and keep my machine close to a window and I haven't experienced any issues.

\n" }, { "Id": "597", "CreationDate": "2016-02-17T23:19:15.463", "Body": "

What is the cheapest (desktop) 3D printer that you can get right now?

\n\n

Is there a current price list (with different vendors)?

\n", "Title": "What is the cheapest 3D printer that you can get right now?", "Tags": "|desktop-printer|", "Answer": "

The peer-based 3D printing service called 3D Hubs conducted a survey of its users and which printers they use and suggest. This survey's results are here.

\n\n

I recall a similar Google Group conducting a similar survey a year or so ago, I'll search for the link to that as well a little later.

\n" }, { "Id": "599", "CreationDate": "2016-02-18T02:52:21.660", "Body": "

I am considering buying a 3D printer for work (scientific research). On paper, the Makerbot 5th generation seems to be the best option, because the price is right for my budget, and also because I'm generally pretty busy, so I want something that's as close to \"plug and play\" as reasonably possible. This will be my first 3D printer and I have no prior experience with the technology.

\n\n

However, any search for \"Makerbot\" brings up a raft of negative reviews from around the time of the 5th generation's first release, which mostly focus on issues with the smart extruder constantly failing and needing to be replaced under warranty. Many of these reviews point out that this may be an initial \"teething\" issue which might be fixed in later versions of the model, but now, one year later, I'm unable to find any information about whether this was indeed the case.

\n\n

So: are the initial issues with the Makerbot 5th generation's smart extruder generally considered resolved, or is it an underlying flaw of the model that won't be fixed until the next generation?

\n\n

I imagine that people might want to suggest alternative models in the same price range. That would be welcome, but I have an extra constraint, which is that I can only buy models that are available in Japan without a long delivery time - this probably limits my options quite a bit.

\n", "Title": "Are the Makerbot 5th generation's initial issues resolved", "Tags": "|makerbot|extruder|", "Answer": "

I own a 5th gen printer and after teaching myself how to dismantle and clear the smart extruder of jammed filament I can attest to the machine working better than any other 3D printer I have used. I run the 3D printing section at Tampa Hakerspace and the MakerBot is much more reliable than the prusa or wanhao which we use at the space. I had problems with the machine initially but now it works like a champ

\n" }, { "Id": "605", "CreationDate": "2016-02-19T05:42:02.453", "Body": "

While I've worked on fixing my voltage regulator on my MightyBoard, I've noticed an extra set of pins available labeled Atmega 1280 IO. I've tried finding documentation on what these pin can be used for to no avail. I'm curious if there is any use for them within the scope of MakerBot's Conveyor service or even what sort of functionality with regard to the board itself.

\n\n

Any links to documentation on this subject would be greatly appreciated.

\n", "Title": "What is the MightyBoard 1280 IO used for?", "Tags": "|makerbot|mightyboard|arduino-mega-2650|", "Answer": "

If you check the Mightyboard RevE files on Thingiverse (http://www.thingiverse.com/thing:16058/#files) you will find the schematics and PCB files (.sch and .brd) for the version of the board used in Replicator 1s and (with some minor mods) most clones. The Atmega 1280 IO header section is a bunch of breakout pins for debug functions. There are eight sets of signal/5v/gnd groupings. Four of them are currently driving debug LEDs that show flash codes for particular firmware failure modes. The other four are unused as far as I'm aware.

\n\n

The ninth and tenth pins shown in the schematic are located on the opposite end of the board, near the 8U2 chip, to give some hacking access to that chip as well. (The 8U2 handles USB comms and firmware flashing the Atmega 1280.)

\n\n

\"Schematic

\n\n

If desired, you can build your own firmware using these pins for other purposes, such as signaling to external equipment. But building Sailfish is a little more difficult than just running the latest Arduino IDE (for compiler stability reasons) so the vast majority of Mightyboard users never bother modifying their firmware.

\n" }, { "Id": "614", "CreationDate": "2016-02-21T11:27:12.620", "Body": "

There are many types of pigments made for paint, food, fabric, cosmetic and finally plastic pigments. I guess we need pigments made for plastic, or at least pigments which won't decompose, burn or lose their properties at the temperatures inside the filament extrusion machines (around 240C).

\n\n

So what type of pigment are we looking for (does that type have a name?) and where can they be found (off-the-shelf from some specialized paint store or is it a more specialized pigment for plastics which is harder to source)?

\n", "Title": "What types of pigments can be used when making your own filaments?", "Tags": "|filament|filament-production|", "Answer": "

The normal way pigment is added to filament (or any other extruded plastic product) is by mixing \"masterbatch\" pellets containing a high concentration of dye with the raw resin pellets. (https://en.wikipedia.org/wiki/Masterbatch) This is significantly easier and more reliable than trying to mix raw pigments into the plastic -- the likelihood of clumping and other issues is greatly reduced.

\n\n

You can buy these masterbatch pellets and mix up colors in whatever ratios you want to achieve specific output colors. Just make sure the masterbatch pellet carrier material is compatible with your resin. There are a wide variety of vendors, including ebay.

\n" }, { "Id": "620", "CreationDate": "2016-02-22T00:13:50.737", "Body": "

Are there general rules on how much a colored filament vs. vanilla filament affects the print results, what is affected and how those values should be changed in the slicer to achieve more similar results between different color/colored vs. pure filament spools?

\n", "Title": "What and how much does coloring in the filament affect in the print?", "Tags": "|filament|color|", "Answer": "

At this point in time I don't think there is a need to be concerned with purity of filaments based on pigmentation.

\n\n

For the most part, variations in filament quality due to coloring should be the least of your concerns compared to some of the other variables such as quality of the pellets, extrusion temperature (when manufactured), cooling rate (after extrusion), handling/storage, etc.

\n\n

Also, assuming your focused on consumer 3D printer use, the typical hardware components aren't equipped to be accurate enough to make fine adjustments with regard to the quality range driven by filament color. Even if there were capable, accurate extruder(s) installed, I think you would need a well designed feedback loop to ensure that you're reading temperatures along the full extrusion process (drive, melt, extrude, etc).

\n\n

I believe what you're asking involves more material science expertise, from a design aspect.

\n\n

However, I believe that the more \"color\" you have obviously reduces the purity of the material and thusly the material properties can suffer. Such properties as thermal resistance found in PLA and ABS. So theoretically if you have Black filament, you'll want to extrude with a lower temperature than you would with a natural \"White\" filament. I would think that the necessary difference in temperature would be a few degrees (Celsius). However, there are many other factors, such as moisture and manufacturing techniques that can take precedence over color differences.

\n\n

My advice, figure out how to \"cheaply\" analyze your material and ensure you have an accurate temperature feedback loop. If can you do that, you'll be able to drastically change the quality of consumer 3D printing.

\n" }, { "Id": "623", "CreationDate": "2016-02-22T12:08:59.097", "Body": "

I am wondering about the speed that my steppers should achieve in regular operation and what the determining factors are, from theoretical hardware facts/limits to software limitations that can influence performance.

\n\n

I do know about the basic properties of stepper motors such as the torque relation with speed, which I read some while ago from this source (for anyont stumbling into this question later):\nhttps://www.geckodrive.com/gecko/images/cms_files/Step%20Motor%20Basics%20Guide.pdf

\n\n

So far Wikipedia (https://en.wikipedia.org/wiki/Stepper_motor) easily turns up the following statements with regard to my question:

\n\n
\n

Thus when controlled by an L/R drive, the maximum speed of a stepper\n motor is limited by its inductance since at some speed, the voltage U\n will be changing faster than the current I can keep up.

\n
\n\n

and for chopper drivers:

\n\n
\n

This requires additional electronics to sense winding currents, and\n control the switching, but it allows stepper motors to be driven with\n higher torque at higher speeds than L/R drives

\n
\n\n

Obviously the amount of steps/revelation will change the speed, too

\n\n

However, all this does not help me to determine what the real life speed of my stepper should be. Additional firmware parameters clutter up my (beginner's) brain even more (and I haven't had a too deep dive into that yet).\nIn general I understand that the frequency we use to send the steps to the stepper is the main determining factor, also 'reduced' by microstepping, if applied. We cannot keep the current up as much as needed due to timing/inductive/electronic properties of the motors, so we lose torque or finally steps. A higher voltage helps to bring the current up faster, so the steepness of the current increase can also limit the maximum applicable frequency. The frequency itself is given from the controller board to the motor driver, hence is set in firmware.

\n\n

All in all, apart from what makes sense to utilize in a 3D printer setup, what are the determining factors of the stepper speeds that can be realized and how to they transform into real life values for our printers? Also I'd be interested in how the determining factors influence the reliability and print quality regardless of the mechanical difficulties that arise with print-speed.

\n\n

/edit:
\nI found these to exist on electroncs SE:
\nhttps://electronics.stackexchange.com/questions/71270/maximum-speed-of-stepper-motor
\nhttps://electronics.stackexchange.com/questions/129064/slow-down-stepper-motors-speed-using-stepper-driver-drv-8825

\n\n

However, I think the question here is relevant and specific enough for 3D printing to exist here alongside.

\n", "Title": "Real life stepper speed", "Tags": "|electronics|speed|stepper|microstepping|motor|", "Answer": "

The four main motor speed limits in a 3d printer are:

\n\n\n\n

Step generation rate limits will depend on the firmware and controller board used. There is a significant range, particularly when 32bit firmwares and RTOS-based platforms like MachineKit are compared to 8bit firmwares running on Atmega AVRs. Some examples:

\n\n\n\n

These limits typically only affect actual stepper speed when relatively fine microstepping is used. Consider an example based on a very typical Marlin printer configuration. Microstepping of at least 1/4 is strongly recommended to avoid resonance issues, so let's start with 1/16 stepping. This allows quad-stepping to effectively output 1/4-steps. Starting with finer microstepping will drop our top speed, and coarser microstepping will be louder and may encounter resonance, so this is a good starting point. Then asssume a typical 32mm-per-rev pulley on a 1.8 degree (200 steps/rev) stepper motor. This system has a resolution of 200*16/32= 100 steps/mm. At 40,000 Hz total stepping rate (quad-stepping at 10,000 Hz) we can theoretically travel at 400 mm/s before hitting Marlin's hard cap. That's probably going to be a higher RPM than we want to run the motor anyway, so it's not a very impactful limit.

\n\n

In comparison, if we started at 1/128 stepping (such as with the THB6128 driver chip) our top speed with 8bit Marlin would be 50 mm/s. That's quite slow.

\n\n

Firmware motion planners can further limits motor speed by ramping up and down to traverse turns and corners. If the model geometry has lots of sharp turns, the target speeds commanded by the gcode are likely to never actually be reached. Only long, straight line paths will have enough ramping time to hit the speed target. You can see this yourself by printing a very small model at a variety of feedrates and timing the actual print duration: at a certain point, increases in the commanded speed will make no difference on actual print times.

\n\n

Another constraint some firmwares enforce is requiring the ability to safely decelerate to a stop within the length of all the motion commands in the planner queue. This allows the printer to gracefully handle a sudden loss of input, such as as if the host computer stops transmitting USB commands, or if the SD card experiences a series of read errors. If the printer just abruptly stops from full speed when it runs out of commands, it is likely to lose position and ruin the print. Whereas if it can decelerate to a safe stopping speed, it may be able to resume after the command stream resumes.

\n\n

In terms of actual motor behavior, the inductance and back-EMF introduce significant limits because they decrease motor torque at higher RPMs. This produces a torque/RPM curve that drops off at higher speeds:

\n\n

\"enter\nhttp://www.geckodrive.com/support/step-motor-basics.html

\n\n

Note in the picture that there is a flat \"constant current\" region on the left side at low speeds. This is the range where the chopping driver is limiting coil current. On the right side of the curve, current is not able to reach the target before it must be switched back off for the next step. That reduces field strength and thus torque.

\n\n

There are two separate effects to consider here:

\n\n\n\n

For a practical discussion of speed limits, we can simplify the situation by assuming back-EMF voltage opposes the drive voltage applied to the coils. This means there is less effective voltage raising/lowering coil current, and it takes longer for the current to change, and less coil current is developed than would be the case without back-emf. This further depresses the torque/RPM curve of the motor. And when the back-EMF voltage is around the same magnitude as the drive voltage, significant motor instability can result because of complex feedback effects between rotor position and effective coil voltage.

\n\n

You can play with these effects using different motor configurations and speeds for a few popular 3D printer stepper drivers using my stepper driver simulator: https://github.com/rcarlyle/StepperSim

\n\n

In closed-loop applications with position feedback and high-end drivers that can damp various instability modes, it may be ok to run the stepper all the way down the torque curve at very high RPMs. In more typical open-loop 3D printer applications, it's generally best to stick to the low-RPM part of the torque curve before inductance and back-EMF really take over the motor behavior. Losing too much torque can easily mean skipping steps and losing position, ruining the print.

\n\n

Another important effect for steppers is mid-band resonance. This is not a normal mechanical resonance, but actually an electromechanical resonance effect. It's quite complicated, but the short explanation is that a stepper has an intrinsic 90-degree phase shift between position and torque, and when speeds rise to a point where coil inductance introduces an additional 90-degree phase shift between coil current and applied voltage, the electromechanical system has a highly-unstable 180-degree phase lag and thus creates a negative feedback loop. This can rapidly drop motor torque until the motor loses synchronicity with the driver's applied voltage and stalls.

\n\n

Mid-band resonance only occurs when coil current is limited by inductance. It cannot occur in the constant-current drive range because there is not enough phase lag between applied voltage and coil current.

\n\n

\"enter\nhttp://www.geckodrive.com/support/step-motor-basics.html

\n\n

Unfortunately, measured torque curves available from manufacturers almost never show resonance zones, because the curves are produced by loading motors with friction-brakes that damp any kind of resonance effect from building. Whereas a 3D printer drivetrain load is almost entirely inertial (accelerating a mass) which is much more prone to resonance.

\n\n

High-end drivers can automatically detect and damp mid-band resonance, but typical low-cost 3D printer drivers do not have this capability.

\n\n

It is generally prudent to avoid entering an RPM range where mid-band resonance may occur. This is easily accomplished by sticking to the left side of the torque curve, in the constant-current operating region. This ensures high torque and good stability.

\n\n

For a concrete example, consider the popular Kysan 1124090 stepper with a 24v power supply.

\n\n

\"enter\nhttps://ultimachine.com/content/kysan-1124090-nema-17-stepper-motor

\n\n

To maintain high torque and avoid mid-band resonance, we would want to keep the RPM to about 400 or less. For the same 32mm-per-rev pulley mentioned above, that would limit speeds to around 200-240 mm/s. Higher speeds are possible, but may encounter reliability issues.

\n\n

This speed, of course, assumes a 24v PSU is used, to match the measured torque curve: a 12v PSU would have a significantly lower top speed before inductance starts to limit coil current, around half as fast. Higher supply voltage greatly increases the top effective speed for the motor.

\n\n

Again, you can easily simulate this effect (using https://github.com/rcarlyle/StepperSim). This simulator output chart shows how inductance and back-EMF are keeping coil current from reaching the target:

\n\n

\"enter

\n\n

For practical drivetrain design, a motor/driver simulator is really the best tool in the toolbox for finding the point where performance will start to degrade due to inability to hit current targets. It's simply too complex to apply simple equations or rules of thumb with any accuracy. These electrical effects and the firmware step generation limits are likely to be the two main restrictions on motor speed for most printers.

\n\n

Of course, how fast you can melt plastic will tend to limit printing speeds significantly below the true motor speed limits, but that's a different discussion. For the moment, we can simply say that top motor speed tends to be unimportant for print moves in typical Cartesian printers. However, there are a few instances where they are likely to drive printer performance:

\n\n\n" }, { "Id": "628", "CreationDate": "2016-02-23T10:53:12.347", "Body": "

One way to give PLA prints a smooth finish is treatment with chloroform vapours (or other solvents, as mentioned in this answer). This method is even featured on Ultimaker website. I would like to try it on some of my prints.

\n\n

What are the practical concerns I should be aware of using chloroform vapours? I am looking for advice concerning vaporisation temperature, time of exposition that makes for a nice finish, and any other experiences.

\n\n

Caution!\nChloroform is a moderately toxic chemical! I only approach this method as I have an access to a well-equipped chemical laboratory with a fume hood.

\n\n

Disclamer:\nThe question is not about the safety issues using chloroform vapours. It is about how obtain the best post-processing results with least trial-and-error.

\n", "Title": "Practical concerns smoothing PLA print with chloroform vapours", "Tags": "|pla|post-processing|surface|", "Answer": "

Beside vaporizing with heat, you can use an atomizer and do cold vapor. The time is around 1-5 minutes at 45\u00baC for a 10x10x20 mm piece like a Marvin or a bot. I have done only gangster tests with it, so I have no larger piece info.

\n\n

Passive vapor polish does not work with chloroform since it tends to crack. It needs a fast flash on the outside only.

\n" }, { "Id": "631", "CreationDate": "2016-02-23T18:14:24.317", "Body": "

On a Cartesian printer movements are really simple. If we assume the printer has 100 steps/mm on each axis, then a move of 10mm on a axis is just 1000 Steps on that axis.

\n\n

Movements that are not aligned with one axis are also simple. Moving from x,y = 0,0 to 10,10 would be 1000 steps on x and y.

\n\n

On deltas even for simple moves more than one motor has to move. And just calculating the amount of steps on each axis needed to reach the destination probably gives a curved move.

\n\n

So what is the algorithm to calculate the steps for a given move for a delta printer?

\n", "Title": "How are delta movements calculated?", "Tags": "|delta|algorithm|", "Answer": "

There are two main special steps to the technique used by most open-source 3d printer controllers:

\n\n
    \n
  1. Divide each linear gcode segment into lots of very small subsegments (\"Segmentation\")
  2. \n
  3. Use some basic trigonometry or the pythagorean theorem to tie extruder position to carriage height for each of the three towers (\"Inverse Kinematics\") \nto find the target position for each small segment
  4. \n
\n\n

The inverse kinematics are surprisingly simple. A virtual 90 degree triangle is constructed from two known lengths to solve for the unknown third length:

\n\n\n\n

I think the best open-source reference here is Steve Grave's Rostock Kinematics document, rev3 available for download here: https://groups.google.com/d/msg/deltabot/V6ATBdT43eU/jEORG_l3dTEJ\nSome relevant pictures:\n\"enter\n\"enter

\n\n

These inverse kinematics calculations are performed for each carriage to get a \"carriage space\" target position, and this is performed for every path sub-segment.

\n\n

The results from these steps can then be reinserted back into the standard linear path interpolation techniques for the printer, in which it fires steps in the necessary ratios and at the necessary rates to produce the desired straight-line motion and acceleration/velocity profile. (How THAT is done is a different question.)

\n\n

The net effect is that the printer will move through a series of small \"linear\" carriage movements (linear meaning constant* speed with respect to time) that collectively approximate the necessary curved (quadratic position with respect to time) carriage motions required to produce a straight-line end-effector move.

\n\n

*(Constant speed before acceleration slowdowns are applied in order to obey dynamics constraints, anyway. Again, that's the subject of a different question.)

\n\n

Segmentation is very similar to the process of using a polygon to approximate a circle. If the facets are small enough, the polygon is a good approximation. Higher Segmentation rates produce less path-following error. The primary conceptual difference between drawing circle arcs and Delta motion paths is that the so-called \"faceted arc\" with Delta Segmentation is constructed in height-vs-time coordinates instead of the X-vs-Y coordinates you'd use to draw a circle on a computer screen.

\n\n

This system is used in large part because support for Delta style printers was originally bolted onto GRBL-based motion planners which were written exclusively for straight-line motion paths in Cartesian printers. It was a relatively minimal modification to the existing codebase compared to implementing full quadratic path interpolation.

\n\n

Techniques have evolved over the years. And alternate approaches are often used: for example, the dc42 fork of RepRapFirmware performs exact path-following without segmentation, by recalculating the proper time for the next step after every step. This is functionally equivalent to approximating a circle with a polygon facet count so high that every pixel on the screen gets its own facet. So it is exactly as accurate as the positioning resolution of the motors allows. The downside is that this segmentation-free technique is fairly processor-intensive, so it only works on relatively fast controllers, not the older 8bit Atmega AVR that powers most existing consumer/hobbyist printers today.

\n\n

Other techniques are possible. The academic parallel robotics control literature is a whole other world of mathematical techniques and complexity in order to produce generalized control algorithms that work for a wide range of robot mechanisms. The version we use in open-source 3d printers is quite simple and application-specific in comparison.

\n" }, { "Id": "636", "CreationDate": "2016-02-24T19:24:06.583", "Body": "

Is there a definitive scalable 3D printer?

\n\n

I've seen examples of Chinese companies printing entire houses, and I'm curious as to printers / filaments that are intended (or at least able) be scaled up for (very) large print jobs.

\n\n

Since most hobby printers can take hundreds of hours for something that can still be held in our hands, so I'm curious if there are any designs for printers that are meant to extrude material efficiently with a easily scalable printing area.

\n\n

Open sourced / free is preferable; though I'm interested in any designs that exist, commercial included.

\n", "Title": "Scalable 3D Printer", "Tags": "|filament|speed|", "Answer": "

As far as I know, right now, all the \"3D printers\" that can print houses, bridges, etc. are experimental models (and sometimes, vey elaborate art projects) - they just don't exist except as one-off creations designed as a proof-of-concept showing this can be done.

\n\n

Maybe some of those project published their plans and code but they are not designed for mass production or general use, even if you manage to build another copy of one of them it will probably be able to print just the same project the original printed.

\n\n

You specifically asked about speed, material and build area:

\n\n

Speed: the speed really depends on the size of the printer's nozzle, the typical desktop printer has a 0.4mm nozzle, if you replace it with a 1mm nozzle the printer will be approximately twice as fast (1.25 nozzle area vs 3.14 nozzle area, assuming you have an hotend designed for larger nozzles that can melt the plastic fast enough).

\n\n

Now, the \"house printer\" does not use a standard desktop hotend (see material below) and buildings don't have fine details so you can make the head much larger.

\n\n

However, as far as I know the early prototypes are still much slower then conventional construction methods.

\n\n

Material: houses are typically not made of plastic, the printers I know of have welding equipment instead of an hotend and extrude steel

\n\n

There are probably other methods but I bet all of them use typical construction materials such as steel and not thermoplastic.

\n\n

Size: and last but not least, it's obviously not practical to build a printer with as house sized platform, the house printers are actually relatively small robots that travel across the construction project, leaving material behind them and then climb over the previous layer to print the next part.

\n" }, { "Id": "637", "CreationDate": "2016-02-24T20:10:54.190", "Body": "

Our library system just put a 3D printer in one of the branches. I have used SketchUp on the library computers for a number of years just to do artsy things. Suddenly, I have the opportunity to actually print something. (I'm really not sure why the libraries have SketchUp installed. But, I have enjoyed using it.)

\n\n

There is a plug-in available for SketchUp so that it can export STL files. But, the security on the library computers will not allow me to put a file into the SketchUp plug-ins folder. And, the tech guy at the library doesn't think that the IT guys at the library will update all the copies of SketchUp at all the branches just so someone could do 3D printing.

\n\n

After doing some searching on this website, I found out that I could export a COLLADA / dae file from SketchUp. I would then import that dae file into Blender (The portable version on my flash drive). I could then export it as an STL file. \nThe process appeared to work. I could see my test object in Blender.

\n\n

I gave the STL file to the branch manager who tried to open it in the Makerbot software so that it could be sent to the printer. But, it gave him a message about the file not being recognized.

\n\n

I am not familiar with all the details in the importing and exporting processes that are going on. Is there someone out there that can give me some help?

\n\n

One problem is that, I am using the library computers. I cannot alter them. I can use what portable versions of software are out there, like Blender.

\n", "Title": "Why won't makerbot accept an STL file from Blender?", "Tags": "|software|makerbot|file-formats|blender|", "Answer": "

Makerbot will accept obj files also. \nIs there an error while importing the obj file?\nAlso you can see errors of your imported file in your makerbot\nIt will be marked in black.

\n\n

\"enter

\n\n

Please make sure your object is a watertight mesh. As I have seen its easy to make a surface model in sketchup. A 3D Printer cannot print something in surface. \nYou can also try importing to netfabb to check if the part has errors in it.

\n\n

If you want a better modeling software, I suggest you to check out OnShape. \nIts a cloud based cad software. Its free as well. (10 private files, beyond that everything is public. 100$ per month i think)\nwww.onshape.com

\n" }, { "Id": "642", "CreationDate": "2016-02-24T21:40:06.150", "Body": "

When you cut or break a PLA model (for example to remove support) it often leaves an ugly while mark where the removed piece was connected.

\n\n

Sanding also tend to leave dull white scratches on the sanded surface.

\n\n

What can I do to restore the white areas to the original filament color?

\n", "Title": "How to remove white marks from PLA", "Tags": "|pla|post-processing|", "Answer": "

Note that on lighter colours these light scratches can be hard to see. On white PLA they're almost invisible unless you hold it against the light at an angle and look at the specular reflection.

\n\n

I've had some luck on a flat surface (the face of a game tile I printed) by going through various grades of sand paper. 240, 600, 1000, 2000. Finally, finishing with a regular metal polisher like Brasso brings up a shine that doesn't seem to wash off, unlike vaseline/vegetable oil.

\n\n

It's quite a lot of work though, and even more work on curved surfaces. If heat treatment works for you, it may be preferable.

\n" }, { "Id": "645", "CreationDate": "2016-02-25T02:42:02.643", "Body": "

Cura does not seem let the full print area to be used. My printer is a Lulzbot Mini. The design illustrated below can be found here.

\n\n

\n \n\n\n \n

\n", "Title": "Cura not allowing full print area to used", "Tags": "|ultimaker-cura|", "Answer": "

If you set skirt setting to 'none', it will get you up to full bed area -1\u00a0mm at the edges, so, for example, 198x198 instead of 200x200.

\n

You need to go to 'Travel' and set 'Travel avoid distance' to zero to get the full 200x200, or whatever, bed area.

\n" }, { "Id": "647", "CreationDate": "2016-02-25T05:27:25.533", "Body": "

Over the times I hear people saying delta inverse kinematics push atmega2560 to limits. I see in my local supplier Arduino Due and RAMPS-FD is the cheapest 32bit alternatives. But there's not much documentation on it so any opinion or personal experience are welcomed.

\n\n\n\n

(I'm planning my first delta. 2560 seems to work perfectly with my previous xy bots though. )

\n", "Title": "Is Arduino Due a worth buy as controller electronics for deltabots?", "Tags": "|electronics|ramps-1.4|", "Answer": "

An 8bit Atmega can provide bare-bones delta performance with Marlin (eg 40mm/s print speed) or pretty good performance with Repetier (due to more optimized algorithms). For a small and simple delta like a bare-bones Mini Kossel, 8bit may be fine. If you want to do high-speed printing or use any of the fancier features, you should go with 32bit.

\n\n

The big issue with 8bit comes up when you want to do something that taxes the MCU. Delta kinematics already add a lot of extra math on top of all the normal printer functions like command parsing, motion planning, and heater control. For example, adding any of these to an 8bit Delta can cause issues:

\n\n\n\n

What the Due (or other 32bit board option) does for you is de-bottleneck the MCU so you can push the printer harder or utilize features that add processor load. Here's why they're so much better:

\n\n\n\n

All that said, the switch from an 8bit Atmega AVR to a 32bit ARM Cortex-M3 is still just an upgrade from a late-1990s processor to an early-2000s processor. Neither is \"modern\" by any stretch. (For example, the BeagleBone Black used by Redeem and MachineKit utterly blows away the Due in processing power and memory. But those options have steep learning curves at the moment.) The Due can still be bogged down by printing very fast with lots of features enabled. It's a big upgrade on an Atmega, but I predict it's going to be replaced by much faster controllers within the next few years. Announcements of upcoming next-gen controller boards are already trickling out.

\n\n

To summarize, the answer is yes, a Due-type board is a good buy if you want a high-performance printer. It will meet 99%+ of user's needs today. But it will be replaced in a few years, just like the Atmegas are being replaced now.

\n\n

As for specific Due shields, I strongly recommend NOT going with RAMPS-FD. It was cloned for sale by Asian companies before the design was finished, which seemingly caused the original designer to abandon it before working out all the bugs. RAMPS-FDv1 has some nasty design flaws such as heaters turning on while flashing firmware. RAMPS-FDv2 is better, but can experience unreliable and oddball behavior due to unresolved issues in the circuits intended to make it compatible with both 3v3 boards like the Due and 5v boards like the Mega. (For example, the thermistor voltage reference circuit needs modifying to work right, and there appears to be timing issues in the 3v3-5v level shifters.)

\n\n

RADDS is a good Due shield. It is a simpler, more reliable, more compact version of RAMPS-FD. It is very popular in Germany and has recently (late 2015) become available for sale in the US.

\n\n

I also recommend purchasing the Due R3-E instead of the regular Due R3. Anecdotally, the \"E\" version appears to eliminate some firmware bootup issues that can occur when the board is first powered at lower than nominal voltage.

\n\n

One possible downside to the Due is that Arduino just announced they are no longer manufacturing it. So all new Due boards from here on will be made by 3rd parties. That's not necessarily bad, since it's open source and many other companies will surely continue making them, but there won't be any more made by Arduino.

\n\n

For single-board options using the same SAM3X8E processor as the Due, the Duet 0.8.5 is becoming quite popular. (It runs RepRapFirmware, which has some great Delta features.) The Alligator runs Repetier. There are several other beta/experimental/development boards in the works too. The community appears to be moving towards the Due and Due-compatible boards over the other various options.

\n" }, { "Id": "650", "CreationDate": "2016-02-25T12:55:10.843", "Body": "

I am trying to find programs that can import and export 3MF file formats. I know that Cura 15.10 beta can open 3MF formats, but I am looking for a program that open, edit and save 3MF files. I am very interested in finding a more portable file format for my 3D designs.

\n\n

If you are not aware the 3MF format promises to provide a \"replacement\" for STL files that contains the complete model information, theoretically allowing us to slice and edit the same file.

\n\n

http://3mf.io/

\n", "Title": "Are there any software packages that can open and edit 3MF?", "Tags": "|software|", "Answer": "

Microsoft 3D Builder should have support for this format.

\n\n

Solidworks 2015 has support too. A software adoption list is placed on 3mf pages.

\n" }, { "Id": "652", "CreationDate": "2016-02-25T16:00:11.583", "Body": "

I'm having trouble getting my MendelMax 2's heatbed not to heat up when I want to print without it. I don't get how multiple sources of control relate to each other (same with extrusion temperatures, actually):

\n\n\n\n

I recently tried to print some PLA with no bed heat (using blue tape on the glass build plate). I sliced the model with bed heat off in Slic3r; changed the PLA bed temps in the printerface options panel to 0; and picked the PLA setting in the GUI main screen (and also tried setting \"0 (off)\" manually.

\n\n

When I started to print the object, the bed turned on and stayed on, maintaining 60C.

\n\n

I thought of just unplugging the heatbed, but then I expect the print would never start, since it would try to heat up the bed and never succeed.

\n\n

So how does the decision about which temperature setting to use, really get made? I have the same question about extruder temps, speeds, accelerations, and other things, too -- whose settings are really in charge?

\n", "Title": "Heatbed control with Pronterface", "Tags": "|heated-bed|slic3r|pronterface|hbp|", "Answer": "

Sli3er will control the bed temperature during a print. The options in Pronterface can be used to pre-heat the bed and hot end, or to adjust the temperature during a print.

\n\n

The slicer will put the temperatures you set in the filament tab settings into the gcode at the appropriate layers. As the printer reads these codes it will adjust the temperatures. If you manually change the temperature, it will stay at your adjustment until it encounters a line in the gcode file that tells it to change again. Usually this only happens at the start of a print, or at the transition from the first layer to the second layer.

\n\n

When you change the filament settings in Slic3r, they do not save automatically, so if you changed them, closed the program without saving your filament settings, and opened it later, the changes would be lost.

\n\n

The other place where temperature can be changed from is in the custom gcode settings, where you can specify operations to be performed at the start of the print, in between layers, or at the end of the print. Take a look there and make sure that there are no codes to change temperature in any of these fields.

\n" }, { "Id": "655", "CreationDate": "2016-02-25T23:57:07.407", "Body": "

In Cura, when you enable \"Print support structure\", is there a way to see what it will look like?

\n", "Title": "Show generated support structures in Cura", "Tags": "|ultimaker-cura|", "Answer": "

Cura generates support based on the outlines of the layers which will be printed, rather based on the triangles of the 3D model.\nIt is therefore not possible to show the generated support in the solid view.

\n\n

You can view the support in the layer view however.\nIn Cura 2 that's on the left bottom of the screen.

\n\n

\"Cura\"

\n\n

Support is classified as a helper part in the legend of the layer view.

\n" }, { "Id": "657", "CreationDate": "2016-02-26T18:25:06.273", "Body": "

I am struggling finding how to print complex shapes made of rubber, tpu, tpe...\nAre there any clear guidelines on how to print those materials without deforming and uncontrolled retraction?

\n\n

The printer is not a problem, I can print on:

\n\n\n\n

Thank you!

\n", "Title": "Proper settings for printing rubber and rubber-like materials?", "Tags": "|filament|fdm|quality|", "Answer": "

First off, you need the right extruder design. Specifically, the filament path between the drive gear or hob and hot end must be extremely well-constrained. 1.75mm TPE filaments (thermoplastic elastomers) will buckle in an instant if given the chance. That means they will try to squeeze out of any little opening in the filament drive path rather than being forced through the nozzle.

\n\n\n\n

Once your filament feed path is fully enclosed, the filament will be constrained and unable to squeeze out the side or wrap around your drive gear. That's the most important step.

\n\n

The next problem is simply loading the filament. Purging out PLA or ABS with a soft TPE can be challenging because of the force required to purge the old material. Many default loading routines are actually too fast for TPEs and will cause the TPE filament to bunch up rather than purging the previous material. It tends to extrude better when there is nothing else in the way. Some tips:

\n\n\n\n

The final issue is print settings. You ever hear the saying, \"you can't push a rope\"? That's kind of what you're fighting with TPEs. With a properly constrained feed path, you can push rubber, but not very hard. So minimizing extrusion force is the name of the game.

\n\n\n\n

With all that, it should be possible to print the softest TPEs with reasonable success. But if you really can't get it working right, plenty of higher-durometer TPEs are available now that are significantly easier to extrude.

\n" }, { "Id": "667", "CreationDate": "2016-03-01T14:04:02.353", "Body": "

We have a Makerbot Replicator 2X at our school. I have a class in the computer lab and one of our focuses is using the printer. Currently, it is not printing properly. We are using PLA at 210 degrees Celcius. The built plate is not heated (using Build Tak). This has worked very well in the past. The problem is that when the extruder goes to lay down plastic, the filament is too hot and curls up in a clump around the extruder. This is characteristic of when we first tried PLA at a higher temperature. I think that our heat sensor is not working properly. Does anyone know how we can fix this problem? Is there a way to calibrate the sensor is should we install a new one?

\n", "Title": "Heat sensor problems with Makerbot Replicator 2X", "Tags": "|extruder|", "Answer": "

Overall, it is unlikely that the problem is temp sensing accuracy. There are only a few things that will throw off the thermocouple's reading:

\n\n\n\n

You should be able to visually check for the first, and test for the second with a multimeter. The resistance between the board end of the thermocouple leads and the brass thermowell at the tip should be infinite / out of range.

\n\n

To actually check the thermocouple calibration, you have a few options:

\n\n\n\n

But, again, the problem probably isn't the TC. It's more likely either a bed tramming issue (eg too much gap between nozzle and surface) or the BuildTak is degraded and not adhering. This can happen if you do a large number of prints in the exact same place, or get the surface oily, such as with fingerprints. Try a fresh sheet of buildtak or cleaning it with rubbing alcohol and moving the print to a different location.

\n\n

In some rare cases, low-quality filament or filament stored in very high humidity may not stick well. This is pretty rare though. For the most part, if your nozzle gap is right, any extruded plastic will stick to Buildtak.

\n" }, { "Id": "669", "CreationDate": "2016-03-01T17:23:07.397", "Body": "

I am following this video series to add auto-leveling to my Prusa i3. https://youtu.be/awsI9bMndJA

\n\n

I have printed the parts I need and have the servo and ss-5 endstop in my cart at Amazon. What I need to know is what wiring I need to connect the endstop to the RAMPs board? I bought this printer as a kit and all parts were included so I am learning slowly.

\n\n

The switch\nhttp://www.amazon.com/gp/product/B00HPLBAYW?psc=1&redirect=true&ref_=ox_sc_act_title_3&smid=AZHZ102UTKBMA

\n\n

Thanks!

\n", "Title": "parts for autoleveling Prusa i3", "Tags": "|ramps-1.4|prusa-i3|", "Answer": "

You just need some wires, nothing special about them. You can use any wire that's flexible enough. For the endstop you'll need 2 wires, for the servo (if you need to extend the cable) 3. The connectors on the end are known as dupont connectors.

\n" }, { "Id": "670", "CreationDate": "2016-03-01T20:19:30.800", "Body": "

I'm building my own 3D printer (custom design, made by me and cut using a water cutting machine) but I have always had problems with the hotend clogging.

\n\n

I have had four hotends:

\n\n\n\n

Believe it or not, this hotend is the only one that has finished a whole print (about 40 minutes), all of the others, with or without fans, haven't managed to finish one!

\n\n

Well... this hotend has a 1.5mm hole so I guess that that's why it can print.

\n\n

The problem with all of the others was that the 1.75mm PLA that I'm using clogged the hotend after two minutes.

\n\n

Now I would like to buy a new, more precise, hotend. I'm using a Bowden extruder, can you suggest a good hotend?

\n\n

This is my printer:

\n\n

\"Print1\"\n\"Print2\"

\n", "Title": "Which hotend does not clog and is good to use with a Bowden 1.75 mm setup?", "Tags": "|extruder|extrusion|", "Answer": "

To add to kamuro's answer, with respect to the dust aspect, the page Filament Not Extruding/ Clogged Hot-End makes a couple of relevent points:

\n\n
\n \n
\n" }, { "Id": "673", "CreationDate": "2016-03-02T00:59:15.623", "Body": "

I've been searching, but I'm coming up empty-handed. They have a file for the TAZ with the 0.35mm nozzle, but I'm not certain if I can just change the nozzle size and be done with it.

\n", "Title": "Does anyone have a Simplify3D configuration file for a Lulzbot TAZ 5 with a 0.5mm nozzle?", "Tags": "|desktop-printer|", "Answer": "

Have you tried the configuration assistant under the \"Help\" menu?

\n\n

\"enter\nFrom S3D 3.0.2

\n\n

And yes, you can just change the nozzle size in the process \"Extruder\" tab and be done with it, if you're using auto extrusion width. If you're using manual extrusion width, also change that to be equal or greater than the nozzle size.

\n\n

S3D will handle everything else. It's not a bad idea to recalibrate extrusion multiplier (with 100% infill 20x20 calibration boxes) but it isn't strictly necessary.

\n\n

General tip for switching to a larger nozzle: while you get used to it, make sure you preview your sliced files carefully looking for areas with gaps or missing details smaller than the extrusion width.

\n" }, { "Id": "676", "CreationDate": "2016-03-02T09:29:08.800", "Body": "

I am optimizing a collection of slicing profiles and in the process of upgrading some profiles from the classic Makerbot Smart Extruder to the new Smart Extruder+.

\n\n

What are the parameters that should be changed with the new extruder?\nAre there any params to which the new extruder is more \"sensitive\"?

\n\n

Thank you!

\n", "Title": "Settings to change when upgrading from Smart Extruder to Smart Extruder+", "Tags": "|fdm|pla|makerbot|slicing|", "Answer": "

There are no \"sensitive\" params here. You just need to set it up like the regular Smart Extruder.

\n\n

Like the original design, the Smart Extruder+ is supposed to make Replicator printers simpler to use, more reliable, and more future-proof. It automatically detects when the printer's filament runs out and pauses the job, notifying users via a desktop or mobile app. It also can be quickly swapped out upon wearing out or becoming obsolete.

\n\n

Additional improvements include faster print start up and refined build plate leveling, the process of calibrating a 3D printer. The smart sensors within the extruder have been enhanced for better performance and to streamline the printing process from start to completion.

\n\n

Improved components include:

\n\n\n\n

The enhanced smart sensors are a big part of the story here though in that they allow for better speed, along with savings on the bottom line too, keeping users informed about print status from beginning to end, whether from the PC or mobile app.

\n" }, { "Id": "677", "CreationDate": "2016-03-02T18:06:22.483", "Body": "

I can not find anywhere default steps per unit for Makerbot replicaor 2. I'm using Ramp V1.4 instead original board and need to know default steps :/

\n", "Title": "default axis steps per unit Makerbot replicator 2", "Tags": "|makerbot|marlin|", "Answer": "

The Replicator 2 and 2x use 18 tooth GT2 pulleys, 1/16 microstepping, and 200 steps/rev steppers. That makes the proper steps/mm value 88.888889.

\n\n

Note that Makerbot used ~88.56 steps/mm in their defaults, which is the value you get if you calculate from the belt+pulley pitch diameter from the Gates GT2 specs. But this is the value you use for calculating belt length required in a closed loop, not for steps/mm. The 88.88... number is more accurate.

\n" }, { "Id": "683", "CreationDate": "2016-03-03T17:07:09.360", "Body": "

I have seen some lasers attached to the RepRap platform for cutting but most seem to be cutting paper, balsa wood, or merely etching. If I were wanting to build a platform for cutting wood, similar to the wood framed or boxed 3D printers on the market, what power laser would I need for that? I assume that a lower powered laser would have to travel slower but going too slow would add the possibility of catching the wood on fire (not good).

\n", "Title": "Laser cutting on RepRap", "Tags": "|reprap|replacement-parts|laser|", "Answer": "

Strapping a laser to a cheap robot made with flammable plastic parts and no enclosure is stupid in the extreme. Don't do it. Seriously, just don't. RepRaps are not suitable for laser conversions.

\n\n

You have to worry about reflected light damaging your eyes, and for the blue diode lasers currently growing in popularity, causing skin melanomas. You have to worry about mechanical vibration from jerky motion loosening fasteners. You have to worry about the workpiece catching on fire. You have to worry about the fumes being generated from etching and cutting. You have to worry about firmware freezes locking the laser on. It's dangerous to you, anybody in line of sight of the machine, and your property.

\n\n

Many of the lasers currently being sold for RepRap conversions, particularly from Russia, violate US and EU safety regulations related to power and safety interlocks and should not be on the market at all.

\n\n

The people posting videos of their RepRap laser conversions are almost always ignorant of proper laser safety requirements and regulations. It's downright scary how much blue flare you can see the camera picking up in a lot of these videos. Don't follow in those people's footsteps.

\n\n

More directly answering the question details, most people doing light etching or paper cutting type work are using small 0.5-2W diode lasers. These are particularly dangerous because they have poor focus and because they don't have the power to cut cleanly and quickly. That means more fumes, more fire risk, and more damage to the workpiece from edge charring.

\n\n

Slowing down the laser is not a good solution: dwell time is your enemy. You want a beam powerful enough to near-instantly vaporize the material, not slowly burn it away.

\n\n

An entry-level proper lasercutter for cutting things like plywood and acrylic is more likely to use a 40w CO2 laser. It should have air assist and fume extraction, and a safety-interlocked enclosure that 100% reflects or absorbs the CO2 laser light wavelength. That basically means a dedicated machine.

\n" }, { "Id": "686", "CreationDate": "2016-03-04T00:01:02.383", "Body": "

Using an FDM printer and PLA or ABS, without adding support material. What modifications can I make to improve how steep an overhang my printer can print before it starts having problems?

\n\n

The obvious first answer is to add a cooling fan, increasing the airflow over the freshly-extruded material ensuring it solidifies. What other things can improve it?

\n\n

Does lowing the temperature help? Raising it?

\n\n

Does speeding up or slowing down the print head help?

\n\n

Does increasing/decreasing the extrusion diameter, or layer height help?

\n", "Title": "How can I improve the overhang angles my printer can successfully print?", "Tags": "|print-quality|", "Answer": "

The two most important things you can do are:

\n\n\n\n

Cooling is really obvious. You need the plastic to solidify before it has a chance to sag. PLA in particular has to shed a lot of heat before it is fully solid. A fan and air guide setup using a \"squirrel-cage\" radial blower around the nozzle is optimal. A little 30mm or 40mm axial fan will not provide optimal performance.

\n\n

Low layer height when slicing is less obvious, but is extremely effective. When you use thinner layers, two things happen:

\n\n\n\n

When you combine these two effects, it is possible to exceed 70 degree overhangs with good surface quality.

\n\n

Another lesser factor is printing shells/perimeters inside-out rather than outside-in. This helps anchor the outermost strand a little better as the overhang is built. This is pretty minor though.

\n" }, { "Id": "696", "CreationDate": "2016-03-07T08:08:59.980", "Body": "

I would like to secure my hotend thermistor in a more reliable way (now it is just thermistor plugged in the hotend :D). I want to have a stainless steel tip for it to fix it inside hotend with a screw (The same approach as used for the heating cartridge). I have thermistors, but I cannot find any tips to buy separately.

\n\n

Maybe you have some links for this kind of stainless-steel tips? Or some keywords I can use to search them?

\n", "Title": "Stainless tip for thermistor", "Tags": "|extruder|hotend|thermistor|", "Answer": "

The main problem here is that the hot block has to match the temp sensor. There are a couple available commercial options for hotblocks with cartridge-and-setscrew temp sensors, but they're not thermistors:

\n\n\n\n

\"enter

\n\n\n\n

\"enter

\n\n

Other popular options for thermocouples include a brass thermowell crimped onto the tip of the TC (Makerbot and FlashForge) or an eyelet lug crimped onto the tip of the TC (Wanhao).

\n\n

The best off-the-shelf solution for most printers is probably to use an E3Dv6 hot block, which uses a screw and washer to clamp the thermistor leads so the glass bead tip can't pull free of its pocket.

\n" }, { "Id": "703", "CreationDate": "2016-03-07T18:42:44.330", "Body": "

In a design that I need to prototype, I have a coaxial, rotational joint between 2 cylinders connected to another coaxial rotational joint between 2 cylinders by a hinge. I have attached the individual part files as well as their assembly where you can use your mouse cursor to check how they move. \nClick here for the SolidWorks parts and assembly files

\n\n

Initially when I 3d printed them by splitting them in half, the outer tube did not close, even after filing the inner tube sufficiently. \n\"enter

\n\n

Could you tell me an alternative method to get this complicated joint working? I need to 3D print them. I plan to use ABS for the 3D printing.

\n\n

Thanks for your patience :)

\n\n

EDIT:\nLubrication: I plan to use grease to lubricate the sliding surfaces.

\n\n

Clearances:\nI printed them without clearances initially (this is my first ever 3D print and I wanted to know how accurate it would be). I then used Dremel and filing tools to file away about 0.5 mm of both the surfaces. It should have been enough for mating the parts. The gap between the edges of the outer shell while trying to close it around is 3-4 mm which doesn't suggest 3D printing inaccuracy. I suspect that the outer tube isn't circular enough in cross section. It meets the inner tube at the the two ends of the semicircle and not inside its belly (you can see it in the picture). I have already tried the obvious, which is removing material from the surfaces wherever the parts touch. The model seems to be too warped for any success. The parts were printed upwards from the bottom, so it is not the warping due to gravity.

\n", "Title": "Is 3D printing feasible for a coaxial rotational joint?", "Tags": "|fdm|abs|3d-models|3d-design|support-structures|", "Answer": "

Consider ball bearings

\n

Depending on the requirements for your joint, I would consider making a design that utilizes ball bearings. With ball bearings, your join will not wear down as quickly, and also move smoothly.

\n

Handling dimensional inaccuracy

\n

The topic of how to achieve dimensional accuracy of FDM printers has already been discussed in this question. You might want have a look at Ryan Carlyle's answer for a specific routine on how to calibrate a Cartesian printer.

\n

From the question linked, a summarized approach for handling dimensional inaccuracy could be:

\n
    \n
  1. Add any desired tolerances into the joint in your CAD model, without regarding the inaccuracy of the actual printer you will use.
  2. \n
  3. Tune your printer and slicer to reproduce those dimensions set by your CAD program.
  4. \n
\n

Generally speaking, the printer settings should handle filament independent dimensional accuracy, while the slicer should be used to account for different characteristics in each filament type. Correcting size through your CAD model is ill-advised, since you should be able to use the same model for multiple printers.

\n" }, { "Id": "710", "CreationDate": "2016-03-08T19:21:15.733", "Body": "

I am using a MakerBot Replicator 2x, and when it is heating the platform and extruder, it begins by heating the platform, and then after that is at 110C it goes on to heat the left extruder to 230C. Does anyone know why it does this, since it has enough power to heat both? It seems like it would take less time to heat both at once.

\n", "Title": "Why does MakerBot Replicator heat the extruder and platform separately?", "Tags": "|makerbot|heated-bed|replicator-dual|hbp|", "Answer": "

It really doesn't quite have enough power to heat everything at once. Initially bringing the bed up to temp takes a lot of current and so Makerbot's start sequences decrease stepper current and hold off on heating the extruder(s) until the bed is preheated. Once preheated, the bed's power draw decreases to a lower \"holding\" level and there is available capacity to do everything else.

\n\n

The start sequencing decreases the max average power draw by a pretty substantial amount. The decrease is about 0.6A each for the XYAB steppers and 1.7A each for two hot ends, totalling 5-6A of avoided peak power draw. (Even lower if you preheat with the steppers disabled rather than idling.)

\n\n

The history here is that one of Makerbot's design decisions with the Replicator 2 and 2x was to utilize off-the-shelf UL-listed power bricks (like a laptop might use) instead of an internal power supply. This made engineering and safety certification much easier, and completely eliminated mains-voltage wiring inside the printer, which is a big safety plus. However, the largest readily-available 24v power brick had slightly less current capacity than optimal. So Makerbot came up with a software workaround for the hardware limitation.

\n\n

If you want, you can replace the stock power brick with a ~350w PSU and preheat everything at once. Sailfish firmware builds intended for printers with properly-sized PSUs (eg FlashForge Creators) will allow simultaneous preheating.

\n" }, { "Id": "715", "CreationDate": "2016-03-09T01:37:35.403", "Body": "

I had a friend request that I print out the Bathymetry of Lake Michigan as a gift for her PhD adviser. I went to NOAA and the site had a few files for the Bathymetry of Lake Michigan.

\n\n

Files and types are located here: http://www.ngdc.noaa.gov/mgg/greatlakes/michigan.html

\n\n

I am trying to convert this into an STL file to print in my 3D printer however I am having no luck in converting this into a mesh I can use. I have tried multiple approaches with the different files that are listed but cannot seem to get it to properly convert to a mesh. I have tried the following so far:

\n\n

-Import an .xyz file into MeshLab to convert it to an STL. After I import it looks like nothing imported and I can't navigate around in the file

\n\n

-Import an ASC file into MeshLab to convert it to an STL. After I import it, I get no errors but the output is a long, thin mesh that looks nothing like lake Michigan

\n\n

-Just for a test, I imported the .xyz file into Autodesk ReCap just to see what would happen. I noticed that the points were in a long thin area similar to the ASC import but as I browsed around I noticed that the each layer of dots was the actual Bathymetry contours.

\n\n

I have tried multiple approaches and software beyond these but can't seem to get the files to convert into a printable mesh that I can manipulate to send to my 3D printer.

\n", "Title": "Point Cloud to STL File", "Tags": "|software|3d-models|", "Answer": "

You can do this using MATLAB, or possibly Octave (the open-source version of MATLAB). I'm doing this right now to print a globe with exaggerated topography and bathymetry. MATLAB has tons of tools for importing data. I'm sure you can import the Lake Michigan bathymetry data, though you may have to hunt for an importing script on the MATLAB exchange.

\n\n

As long as you can get your data imported as a 2D matrix of X, Y and Z values, you can use the surf2stl() function by Bill McDonald, available here:

\n\n

https://www.mathworks.com/matlabcentral/fileexchange/4512-surf2stl

\n\n

This creates an STL you can use. Note that it's just a surface, so you'll need to give it depth some other way so your printed object isn't paper-thin.

\n" }, { "Id": "722", "CreationDate": "2016-03-09T15:57:50.437", "Body": "

I have recently bought a spool of eSun PETG. So far I really like the filament. My only complaint is, I get lumps of charred filament deposited on my object. The slicer I used is Craft Ware and I have played with the Far Travel -> Elevation settings. I have noticed that this helps but then I have little to no adhesion to the print surface and my supports do not stick to the raft. Does any one know how to mitigated PETG from collecting on the extruder?

\n", "Title": "PETG Collecting on the Extruder", "Tags": "|filament|petg|", "Answer": "

PETG strings at working temperature to different extents depending on brand. Some of the thinner strings are flying in the air due to the fan or mechanical stress, and are finally captured by the hot end.\nI feel, PETG has two modes

\n\n\n\n

Things which do not help (at least least on my printer with direct extruder), but which I tried out individually

\n\n\n\n

Things I have not tested

\n\n\n\n

Things that worked

\n\n\n\n

Other things to consider

\n\n\n" }, { "Id": "726", "CreationDate": "2016-03-10T08:59:27.577", "Body": "

I am building what I dub Frankenstein's printer from various components I could find in the electronics dumpster to print the proteins for a P3 Steel (toolson edition).

\n\n

I want to elevate a scanner bed with an attached DVD drive motor for X-Y movements of the hotend. I plan to connect this by threaded rods to a base that also hosts the heated bed on a stury z-axis mechanism. To make this as stable as possible, I plan to conncect M8 threaded rods in the 8 edges in wooden blocks.

\n\n

I am planning on 4 vertical rods and 2 crossing rods along the diagonals of the backside of the system. Additionally each side is planned to have one diagonal connected by a rod.

\n\n

We're talking about a height of about 30cm and M8 rods. Will this introduce lots of vibrations and is it possible to avoid easily? Are there any better connection ideas or improvements I can make to this design?

\n\n

*edit: here is a really bad hand-drawn sketch:\n\"hand-drawn

\n", "Title": "Threaded rods to create frame/axis connection. Which improvements can be made and are they needed?", "Tags": "|printer-building|quality|", "Answer": "

If I understand you correctly, compared to a Mendel RepRap you are using:

\n\n\n\n

So I expect less vibration and the same print quality as a Mendel.

\n\n

Rather than put one block of wood at each corner with holes drilled at a bunch of weird skewed angles,\nthe \"1X2 split vertex\" looks like it is a lot easier to construct.

\n\n

I've heard several people claim that lots of threaded rods and associated corner connectors can be replaced with a few big sheets of wood in a 3D printer.\nSee Mendel90, RP9, SGBot, WolfStrap, etc.

\n\n

Looks like a very educational project. Good luck.

\n" }, { "Id": "729", "CreationDate": "2016-03-11T17:42:53.527", "Body": "

I have a z-axis which follows schematically the same principle as the makerbot one's (threaded rod and two leading rods with linear ball bearings carrying a level). It is from an old experimental lab doing physics or chemistry.

\n\n

The axis move gorgeously about 5 cm, but then it get's stuck on either sides of this way. Both driving threaded rod and the leading rods seem perfectly fine and should be able to allow for further movement. This is as far as I can see by eye.

\n\n

Where should I look to find further issues and how could I improve the performance? Do I need to take the construction apart?

\n", "Title": "z-axis hard to move in some areas - what could be faults, how to improve?", "Tags": "|maintenance|mechanics|z-axis|", "Answer": "

X stage binding like this is almost always caused by parallelism issues with the rods and/or screw. The two-rod-plus-screw arrangement is quite over-constrained and thus requires good alignment to move smoothly.

\n\n

Some basic troubleshooting steps:

\n\n
    \n
  1. Make sure the screw is not constrained at both ends. It is very difficult to manufacture a screw that is perfectly straight and then mount it so precisely that it can rotate with zero runout. Screws in light-duty linear motion applications should be allowed to \"wobble\" freely so any bend or runout doesn't apply side-load forces to the Z stage. This can be accomplished by putting a misalignment-tolerant coupler (like an Oldham coupler) on the driven end, and/or leaving the non-driven end free without any support bearing. A motor mount with a small amount of compliance (like a rubber stepper damper) can help when the screw is captive to the motor.
  2. \n
  3. Lubricate the screw and bearings and make sure everything is clean and in good condition.
  4. \n
  5. Allow the Z carriage to self-align the rods and screw. The proper technique for this will depend on the Z stage, but the basic idea is to loosen the rod and screw mounts on either end, run the Z carriage back and forth a few times to push the rods into position, and then only re-tighten the top and bottom hardware when the carriage is at that end. It may also be necessary to loosen the screw nut and bearings on the carriage to get everything aligned properly and smooth-running, but that does not enforce parallelism like loosening the rod and screw mounts, so is really a secondary step. It may be necessary to leave some \"float\" in one rod or the bearings on one rod (with gravity preloading out any resultant slop) if the hardware has major alignment issues.
  6. \n
  7. If the stage still binds after doing the above, it may be necessary to check if the linear hardware is bent (such as by rolling rods on a flat table) or use a file or Dremel to loosen up the mounts or whatever feature is causing the misalignment.
  8. \n
\n" }, { "Id": "732", "CreationDate": "2016-03-12T00:17:11.157", "Body": "

Some RepRap models use only a single motor for the Z axis, others use two.

\n\n

For example, there is the 3drag that has only one motor and a smooth rod on the other side. There are modifications that add a threaded rod on the other side that is connected to the motor axis with a belt - which seems to be a really good solution.

\n\n

Other printers, like the Prusa i3 or the Mendel90 have two Z motors. And after playing around with a two motor model, I find it pretty annoying when they get out of sync and I need to calibrate the axis and the print bed again. So two motors seem more like an disadvantage to me.

\n\n

Could someone please shed some light on why most RepRaps have two Z motors (nowadays)?

\n", "Title": "What's the advantage of two z motors?", "Tags": "|z-axis|reprap|motor|knowledgebase|", "Answer": "

First up: I'm no expert. All of the below is guesswork.

\n\n

I think that the main reason is that it makes for a simpler design.

\n\n

If you hold the X-Axis only on one side, you have to keep it level through rigidity. (Especially if you have the extruder motor on that axis.) If you try to control the sagging, you have a problem because the torque changes depending on the position of the extruder.

\n\n

For stepper motors to go out of sync, one has to miss steps. Regardless on whether you have one or two or seventeen motors, if you are missing steps, you need to debug it.

\n\n

I got lots of missed steps in both X and Y, and I tried everything. Until I found out that I was actually missing steps in Z, and the nozzle was hitting the build.

\n" }, { "Id": "736", "CreationDate": "2016-03-12T06:50:45.473", "Body": "

I would like to make some parts that will fit together. Specifically, I'm making an array of holders for small glass vials, which will stack together vertically to make a kind of extensible spice rack, for a scientific application.

\n\n

Of course, I could just give each module a flat top and bottom surface and glue them together. However, it would be more convenient (and more fun) to make some kind of connector, so that I can just push the parts together and pull them apart when needed. The connection will need to be relatively sturdy, and they'll need to sit flush against each other with no flexing where they join. I am a novice at 3D printing, so I'm looking for tips on how to design interlocking components, both in general and for this particular case.

\n\n

These parts will be printed in ABS using the Zortrax M200, and I'm designing them using OpenSCAD. They will almost certainly be printed lying on their side.

\n\n

One option is that there are several parametric Lego brick clones available, which I could easily incorporate into my OpenSCAD project. My only worries are that this might be overcomplicating the problem, and that printing Lego type parts on their side will result in having to scrape a lot of support material out of the hollow parts. (Though on the plus side, the lego compatibility might actually be useful, in terms of not having to manufacture additional supporting material.)

\n\n

Any tips would be welcome on how to design and print parts that can be pushed together. I am sure I can work it out myself in time, but any knowledge that will save me a design iteration or two would be very greatly appreciated.

\n", "Title": "Tips for making parts that push/snap together", "Tags": "|3d-design|", "Answer": "

There are many different ways too approach this and the question may be too broad, but here's a stab at it...

\n\n

Here are a few different ways that I've made parts that connect in the past:

\n\n

Example 1: Utilize the elasticity of the plastic by creating a semi circle to fit around another object. When fitting the part to the other object, the \"wings\" will flex out and relax around the back end of the object.

\n\n

Example 2: T-Slot style designs are a nice choice for semi-permanent or interchangeable parts. The key here is just using trapezoids and make sure the \"female\" end is slightly larger and/or tapered to make it easier to insert the slides.

\n\n

Example 3: Create snaps. This can be a bit more difficult, but can provide a more professional look to your design. It's best to start off by designing on L-Shaped slot on the parent part and a smaller L-Shaped extrusion on the child part. Then you can add a taper to the bottom area of the \"L\" to make it easier to insert into the slot.

\n\n

Example 4: Creating plugs are similar to snaps, they're just circular. Try starting off with a cylinder and joining either a sphere or a slightly larger diameter and tapered cylinder on top. Then you should be able to snap the assembly into a hole that is slightly smaller than the \"top\" of the plug.

\n\n

It's important to weigh the usefulness of some design choices against the purpose of your part. For me, if I'm designing something for the shop or something that I think I'm going to be swapping out often, I'll use the T-Slot idea. Other things that \"looking pretty\" is more important, I'll try to hide the connections by designing a type of snap that can go into the walls of the part.

\n\n

The T-Slot designs seem to print quite easily with no need for supports (depending on how gradual the angles of your trapezoid are) and provides a lot of structural strength.

\n\n

Printing snaps has been difficult for me in the past and its best not to try to unsnap the part too much as you run the risk of shearing the snap upon removal. Printing such a small feature means that it is usually not marginally strong. However, including multiple snaps in an area and designing them with tight clearances can provide a very neat/clean connection that is appealing for consumer products (for example, phone cases).

\n" }, { "Id": "737", "CreationDate": "2016-03-12T09:09:54.953", "Body": "

I would like to print a model without supports on the Zortrax M200. To do this it is mandatory to use the proprietory Z-Suite slicer. I am certain that at some point I found an option to turn off support struts (I specifically remember the text \"(not recommended)\" appearing in the UI somewhere), but it seems not to be there now. Is my memory playing tricks on me, or is there a way to enable a \"no supports\" option in Z-Suite?

\n", "Title": "How to turn off supports in Z-Suite (Zortrax slicing software)", "Tags": "|software|", "Answer": "

I've found the answer for myself by playing around a bit more. It seems that in selecting the \"support angle\", a higher angle means more supports. This was confusing me, because I had assumed \"support angle\" meant the maximum angle of overhang before a support would be generated. I guess it does actually mean that, but it counts 90\u00ba rather than 0\u00ba as vertical. In any case, supports are turned off simply by changing it to 0, in which case the text \"not recommended\" does indeed appear.

\n" }, { "Id": "740", "CreationDate": "2016-03-13T17:15:35.327", "Body": "

What software is best for the basest of n00bs when it comes to 3D parts creation?

\n\n

I have a heavy math background and know how to create explicit functions of volume, surface area, center of mass, etc. Ideally, I'd like a program that uses those strengths but I realize that most n00bs have a crippling math phobia so I'm not holding my breath.

\n\n

I tried freeCAD once and made some headway but the next time I turned my computer on, it refused to open. It was just a weakling netbook that I don't even have in my possession anymore but the computer I'm currently using is rather slow and doesn't seem to have much memory left either, so I still need something lightweight.

\n\n

tl;dr: Seeking a free, lightweight program to create .stl files that is good for n00bs that are not afraid of math.

\n", "Title": "Good (preferably free) Beginner Software for Part Creation?", "Tags": "|software|print-preparation|", "Answer": "

I've had good luck using MS 3D Builder. Most of my thingiverse models were designed with 3D Builder. Depending on your Windows and Office installations, this program may already be on your computer, but otherwise you install it via the Windows Store. This program may also make good use of your math skills; I've done more trigonometry getting dimensions right for my models than all other areas of my life combined.

\n\n

MeshMixer is another good free option. It's significantly more powerful than 3D Builder and many other free options, but I've found it much harder to get comfortable using. The visualization, especially, just hasn't been as clean, often making sharp edges appear to be rounded on the screen. Still, and I know and desire the power it offers, and I'll occasionally open it up for specific things. If nothing else, it's good for measuring/verifying the thickness of a section, and sometimes I can use it to repair models botched by other programs.

\n\n

Finally, I'll bring up OpenSCAD again. OpenSCAD effectively lets you write a program to generate your models. As such, it's especially good when creating things you will want to do more than once, where you'll tweak or customize the initial part for later uses. One way you can achieve re-use like this is creating a library of partial-designs or raw shapes which you can then resize/stretch/etc and add or mix into other parts using the other programs. The other area where OpenSCAD really shines is in prototyping. When you discover just this one section of a part shoud have been a couple millimeters longer after the initial print, it's much easier to adjust this with OpenSCAD than with many of the other options.

\n" }, { "Id": "743", "CreationDate": "2016-03-13T18:11:03.847", "Body": "

I'm building a Kossel Mini, and I'm stuck on the extruder motor holder.

\n\n

My Kossel came without instructions, I was given a set of instructions by a friend (the \"Kossel Build Guide\" by Blomker industries). I also found some instructions on the net. However, my components are different from those in the build guides.

\n\n

The question is how these components fit together.

\n\n

I'm pretty certain the stepper motor should be connected to the big black components (labeled (1) and (4) in the picture). The 4 holes in these components correspond to openings in the stepper motor; and the stepper motors for the vertical carriages, were connected in a similar way to their respective holders.

\n\n

The round thing, second from the left on the bottom, (labeled (2) in the picture) presumably goes on the axis of the stepper motor. Once again, a similar procedure was needed for the motors that will move the vertical carriages.

\n\n

The screws on the right, labeled (3), are 1 cm long and 3 mm wide.

\n\n

The other screws are 2 cm long and 3 mm wide.

\n\n

The stepper motor is a Wantai Stepper Motor Model 42BYGHW811 .

\n\n

I have e-mailed the company where I bought my kit and am awaiting their answer. Meanwhile I'm hoping to get some advice here.

\n\n

Apparently the usual stepper motor for a Kossel Mini is Nema 17. Searching on \"42BYGHW811\" I mostly find datasheets. Using \"Nema 17 mount\" or \"Nema 17 mounting screws\" as search terms, I find a lot of images with a component like (1). But nothing on how this and the other components should be put together for the motor holder for an extruder for the Kossel Mini.

\n\n

\"Parts

\n", "Title": "How to build the extruder motor holder for the Kossel Mini", "Tags": "|extruder|motor|kossel|", "Answer": "

The extruder is an MK8 derivative, I think.

\n\n

After a lot of searching, the closest that I could find (with an assembly diagram) is this product which has more or less the same parts as yours (apart from the small black bolt and sleeve - used as the axis - which you don't have): [Improved Version] YOTINO Right Hand MK8 Remote Bowden Extruder Accessories 1.75mm/3mm Filament All Metal Remote Extruder Frame Block for Reprap 3D Printer Kossel Prusa.

\n\n

These are the parts:

\n\n
\n

\"16

\n
\n\n

This is the assembly process:

\n\n
\n

\"Assembly

\n
\n\n

This is the completed extruder:

\n\n
\n

\"Completed

\n
\n\n
\n\n

Also, whilst this is not the same beast, this link shows a detailed assembly which could prove useful as a cross reference: RepRapWiki - Extruder assembly

\n\n

In addition, w.r.t the Blomker guide, there are a number of versions of the same guide that I've come across in the past:

\n\n\n" }, { "Id": "752", "CreationDate": "2016-03-14T21:48:18.690", "Body": "

I'm building a 3D printer and I've been looking around for materials suitable to make the frame.

\n\n

I have occasional access to a laser cutter which I could use to manufacture a ply box-type (UltiMaker) enclosure, but I like the ease of adjustment provided by the T-slot beam kits.

\n\n

I don't have any metal-cutting machine tools. Can aluminium beam be cut by hand with a hacksaw to a good degree of accuracy? How does one finish the cut end?

\n", "Title": "How to cut OpenBeam/MakerBeam?", "Tags": "|desktop-printer|printer-building|", "Answer": "

Misumi does provide a nice product but it's quite clear their business model is geared toward businesses and not the individual consumer.

\n\n

The 2020 extrusion seems to be a bear to get parts for, specifically T-nuts and braces. Openbeam or Makerbeam might be better options and are readily available on Amazon for very decent prices.

\n" }, { "Id": "759", "CreationDate": "2016-03-15T10:38:16.180", "Body": "

I ordered 10m of GT2 timing belt for a printer I want to build, however, the chemical smell from the belt is completely unbearable, in the sense of it gives me the feeling it is destroying something in my nose (and my nose isn't even particularly good). I tried to leave the belt outside in my garage for about three weeks now, but the smell has gone only so much. I naively assume it is the chemical to make the belt soft and bendable, would that even be a health risk having the belt around me for more extended intervals of time? Can I make the smell go away, and what would be the best way to do so?

\n", "Title": "Getting rid of hardware parts smell (GT2 timing belt)", "Tags": "|hardware|belt|", "Answer": "

It's a garbage-quality belt, and it will probably continue outgassing at a steadily-decreasing rate for a very long time.

\n\n

The fact that it has a strong chemical smell is pretty good evidence that it would not comply with regulations in civilized countries (such as REACH). I would recommend returning it or throwing it away and buying from a more reputable vendor. A large fraction of all cheap 3d printer parts these days are ridiculously far out of compliance with US/EU regs and basic common sense for contamination.

\n\n

If you're hell-bent on using a crappy knock-off belt to save a few bucks, you could put it under vacuum to try to suck out the residual chemical gas faster. Just apply the vacuum very slowly / in stages (say over 10 minutes) so you don't get decompression blisters in the rubber. You shouldn't need a hard vacuum, dropping to 0.5-1 psia or 0.05 bar absolute for a day or two should help significantly. No guarantees though.

\n" }, { "Id": "762", "CreationDate": "2016-03-15T17:09:47.277", "Body": "

PLA is flammable, but a good case can protect the main board from dirt.

\n

Is it worthwhile to protect a circuit board with a 3d printed case?

\n", "Title": "It is worthwhile to protect electronics with a 3D printed PLA case?", "Tags": "|electronics|", "Answer": "

As pointed out by Ryan Carlyle, not all 3D printing filament is flammable (such as PET and PETG), and the question therefore rather becomes:

\n\n

Can 3D printing be used to make proper electronic cases?

\n\n

And the short answer to that is yes. 3D printing allows to make customized cases of all varieties. Also, since there is huge variety of materials available, you not only restricted to cases of plastic filaments - should this be of interest.

\n\n

Then again, it all depends on what you are protecting your electronics from. If you plan on submerging your electronics into water or throw them into a fire, 3D printing might not work for you at all. If dust protection is your only concern, however, 3D printed cases should do just fine in most cases.

\n" }, { "Id": "764", "CreationDate": "2016-03-15T18:21:53.590", "Body": "

The idea is so you only need two long wires going from the power supply to the electronics boards. I'm sure I saw this done in a tutorial somewhere, but I can't for the life of me find the source. Also I just bought a kit and the build manual says to use four wires, but only two wires come with the kit, so I recalled that bit of advice in the tutorial and thought hmm either they shorted me those wires, or they just expect me to make do with two. Perhaps the jumping method is common practice? But then why have a plug on the ramps board with four inputs? Why not just two inputs and split it internally if need be?

\n", "Title": "When building the ramps 1.4 is it safe to use small wires to jump the 5A, 12V power input over to the 11 Amp input", "Tags": "|ramps-1.4|", "Answer": "

I guess that the external split is at least partly done to force the user to utilize 4 wires. 16 Amps at 12 Volts is quite a bit of power and you don't want your wires or connectors to melt. If you use twice the wire cross section by using 4 instead of 2 wires, you also reduce the resistivity and hence heat created in the wire.

\n\n

Put it the other way around: Using 2 instead of 4 wires raises the risk of your setup to catch fire.

\n\n

Still, you can use that bridging technique, if you make sure that all your wire diameters are big enough.

\n\n

I would advise you, however, to cut the wires that come with your kit in half and use the connectors in the way they were meant to. For optical appeal and less cable clutter you can still put the wires into some braided sleeve for example.

\n" }, { "Id": "768", "CreationDate": "2016-03-15T23:30:58.747", "Body": "

Backstory

\n\n

I've had issues in the past with my drive gear \"eating\" my filament. It seemed that the filament quit extruding for one reason or another and the drive gear would slowly eat away at the side of the filament.

\n\n

I eventually assumed it was the plastic filament guides causing unnecessary tension that the drive gear couldn't compete with, ultimately keeping the filament from moving forward. Thusly, allowing the drive gear to continue \"trying\".

\n\n

My solution was to hang my spools above the machine to avoid using the filament guides feeding from the back of the machine up through the top.

\n\n

Question

\n\n

Can the plastic filament guides really cause that much drag? What other variables can I expect to look out for?

\n\n

Machine: MakerBot Replicator Dual (1st Generation)

\n", "Title": "Filament Guides Drag", "Tags": "|filament|makerbot|hardware|drive-gear|", "Answer": "

The friction inside of the guide tubes is fairly minimal assuming that the lines are straight and there isn't anything else inside them. My guess would be that you may have another issue. Reducing the drag in the guide tubes while it may reduce the problem of clogged nozzle, might just be hiding the symptoms of another problem.

\n\n

I've heard, but never tried adding a dab of Canola oil to the tip of your filament prior to feeding it into your extruder. Specifically for Makerbots.

\n\n

Link my source:\nhttps://groups.google.com/forum/#!topic/makerbot/Wn-MKC1Ybm0%5B1-25%5D

\n" }, { "Id": "771", "CreationDate": "2016-03-16T10:04:42.047", "Body": "

As my first project, I'm trying to design a holder for glass vials, for a scientific application. The photo below shows the latest design iteration, and also shows the problem with it:

\n\n

\"vial

\n\n

As you can see, one of the clips that's meant to hold the vial in place has broken off. This happened after inserting the vial once or twice.

\n\n

I think I understand the reason for this. It's because I'm printing in the orientation shown below, in order to avoid the need for supports. (This will be more important later, when I scale it up to an array of many holders.) This means that it's relying on the strength in the z dimension, which is much weaker than in the other two directions, because it relies on the cohesion between the layers.

\n\n

\"enter

\n\n

My question is whether I can do anything about this by changing the design of the clips, without abandoning my hope of support-free printing. I don't care at all what the design looks like, except that as much of the vial as possible needs to be visible from the front. I'm printing in ABS.

\n\n

I have tried varying the thickness of the clips. Thinner ones don't break as easily, but they are a bit floppy and aren't very good at keeping the vial vertical. The ones shown are the thickest I've tried - I'm afraid that if they're too thick they won't bend at all. (The vial is supposed to be inserted from the front rather than the top.)

\n\n

Edit just as an update, here's what it looks like using John Biddle's suggestion, which works perfectly:

\n\n

\"enter

\n", "Title": "Designing clips that won't break", "Tags": "|3d-design|print-strength|", "Answer": "

Come on. Wider clips will just survive a bit longer.\nThe real cause is the orientation (plane) of printing.

\n\n

Continues filament layer will always be more durable than few layers sticked together.

\n\n

So the question is why not to change printing plane?\nYou can use better design which utilizes support without using \"support material\"\n\"enter

\n" }, { "Id": "796", "CreationDate": "2016-03-19T16:25:56.870", "Body": "

I am new to 3d printing and had bought my first 3d printer a couple of months ago. I have it all put together and the software uploaded to the Arduino. However, I need some help with configuration and calibration of the printer. I understand that there is a configuration file that can be changed and uploaded again the create those changes. I also have worked with Arduino before so I know the IDE.

\n\n

When I turn the printer on everything works, as far as I know. I did some tests to see if anything was wrong, but I couldn't find anything. However, when I hit home for all of the x, y and z axis to go to their starting marks, it tries to go past the rails and comes apart. From what I understand this is a configuration issue.

\n\n

I tried to contact HE3D about this, but received no answer, so you guys are my last hope. If you need any more information to help me, let me know and i'll try getting it to you asap. (I also have read the documentation on Repetier and the installation and configuration section but my brain for some reason is not clicking into gear.)

\n\n

Thanks for the help in advanced!!! Here are the specs:\nhttps://www.3dprintersonlinestore.com/reprap-prusa-xi3

\n", "Title": "HE3D-XI3 3D printer Repetier software config help!", "Tags": "|ramps-1.4|software|prusa-i3|", "Answer": "

Based on some troubleshooting in the question comments, it appears that the issue here is the motors running the wrong way. This can be fixed EITHER by inverting the motor directions in firmware, OR by simply flipping the orientation of the motor plugs where they connect to the controller board. Flipping the plugs is an easy and quick way to reverse the phase of a stepper and thus reverse its direction of motion.

\n" }, { "Id": "797", "CreationDate": "2016-03-20T00:06:31.977", "Body": "

I was watching a couple of videos about installing a Thermistor on the heat-bed and extruder of my Prusa i3; both videos mentioned that if I use either Sodium Silicate (Muffler & Tail Pipe Sealer) or Heat Resistant Silicone Glue to attach my Thermistors, I would be able to get a more accurate temperature reading.

\n\n

Is there a particular kind or specification that is best for use in their the Heatbed or the Extruder?

\n", "Title": "What kind of silicone glue should I use to attach my thermsistors?", "Tags": "|extruder|heated-bed|prusa-i3|thermistor|", "Answer": "

The main requirement here is temperature resistance. A 3D printer extruder is very, very hot, and will easily exceed the ratings of many typical silicone glues in normal service or during a runaway event. (You should always plan for an eventual thermal runaway. They're alarmingly common with cheap kit printers.) Even high-temp glues will not survive the temperatures an extruder can reach at full power if the firmware locks up.

\n\n

Muffler putty is popular for fixing thermistors because it's one of the few widely-available ways of attaching a thermistor that will easily withstand the heat. In fact, it will survive after the aluminum hot block melts. (Yes, that can happen.) But muffler putty is fairly brittle, and has a different coefficient of thermal expansion from aluminum, so there is some risk of the thermistor detaching over time. I really don't recommend puttying thermistors used in extruders -- many people do, but it's less robust and less secure for long-term use than the proper hardware fixing methods found in modern hot block designs like the E3Dv6. The temperature sensing is sufficiently fast and accurate simply by putting the thermistor into a pocket in the hot block, without any kind of potting around the glass bead.

\n\n

High-temp RTV glues are very suitable for heated build plates. Pretty much any RTV (such as from your local automotive repair store) is fine if you just want to attach a thermistor to a plate. However, silicone heaters will require silicone adhesives. I've had good success with Dow Corning 736.

\n" }, { "Id": "801", "CreationDate": "2016-03-20T18:22:24.270", "Body": "

I've been curious about the various UV/Laser printers in (or coming into) market that use liquid resin. I've seen the samples of the Pegasus Touch, Form1, and the Carbon3D as examples. I like the specifications of the quality that machines can put out. However, in my experience with FDM printing, there almost always seems to be something not quite right about the print.

\n\n

So, what are some major maintenance considerations for these types of 3D printing? Also, specifically, are supports and overhangs as much an issue in these types of printers as with FDM/FFF?

\n\n

Here are some things I consider major maintenance considerations in FDM:

\n\n\n\n

I'm not necessarily looking for printer recommendations, more so technical insight on the technology.

\n", "Title": "Cons to UV printing", "Tags": "|resin|maintenance|dlp|uv-printer|", "Answer": "

Despite how many vendors make it appear, resin-curing SLA/DLP printers are industrial or commercial tools that are really not suitable for home desktop use. Here are the major downsides:

\n\n\n\n

These are some pretty significant \"user experience\" downsides compared to a consumer desktop FDM printer. It's more hazard, more work, and more cost than FDM. SLA/DLP is primarily advantageous where high resolution or high print speeds are required.

\n" }, { "Id": "813", "CreationDate": "2016-03-21T04:49:04.347", "Body": "

This question is a bit of an edge case for what 3D Printing SE covers, but it has to do with topics most closely related to 3D printing.

\n\n
\n\n

I've found a plethora of ways to convert SVG (vector graphics) into G-code, but I can't seem to find any way to take a series of G-code movements and convert them into lines as a vector graphic.

\n\n

Why would I want to do this, you ask?

\n\n

I have a Silhouette, which can cut paper, or draw on paper, depending on if you insert a knife tool or a pen tool.

\n\n

I have a polar draw bot (Makelangelo) which doesn't want to behave... the motors keep losing steps when the number of steps/second is low, and thus positioning gets off.

\n\n

I want to use the drawing algorithms in the Makelangelo software to create drawings using my Silhouette, but the Silhouette wants vector graphics, not G-code.

\n\n

The simplest way in my mind to do this is to convert G-code generated in the Makelangelo software, convert it to a standard vector format (SVG), and import it into Silhouette's software suite.

\n\n

Alternately, if there's a way to send G-code directly to my Silhouette and have it work, that'd be a much better solution.

\n", "Title": "Convert G-code to SVG", "Tags": "|g-code|file-formats|", "Answer": "

There are a number of programs available which will convert g-code to DXF, a common drawing format. If your device does not support DXF directly, there are a number of programs to convert DXF to SVG. I would paste links but a quick search with your favorite search engine should give you useful results. The better conversion programs will allow you to eliminate movement g-code entries, which prevents connections between lines. I did not paste links, as my research shows such variety as to be overwhelming.

\n" }, { "Id": "826", "CreationDate": "2016-03-22T00:19:31.560", "Body": "

This is my first time calibrating my printer and I'm not sure where to start. The printer is running Repetier v0.91 firmware that came on the SD card that came with the printer.

\n\n

Anyway, the question I have is where do I start calibrating and how would I go about it? I have read on Google many ways and it seems there are different opinions. I am reading this at the moment: RepRap Wiki - Calibration.

\n", "Title": "Beginner in calibrating HE3D Prusa XI3", "Tags": "|prusa-i3|calibration|", "Answer": "

There's a great tutorial here on youtube.

\n\n

And personally my favourite is this tutorial.

\n" }, { "Id": "857", "CreationDate": "2016-03-24T02:32:29.347", "Body": "

I have adjusted my z axis end stop via the paper test. However when I press to home all the axis the z axis hits the print bed moves it down slightly then goes to the postion I describe. Is this how its supose to be if not what do I need to do?

\n", "Title": "Adjusting the Z end stop?", "Tags": "|calibration|", "Answer": "

Yes, this is normal. Your printer is, at first, probably going downward with some speed. When it hits the end-stop it has some momentum and doesn't stop right away. So once it hits the end-stop, it has to physically stop the print head and move upward again slightly to get to the right position.

\n" }, { "Id": "864", "CreationDate": "2016-03-25T13:09:16.460", "Body": "

I purchased the following wire for my MK2a heated bed:

\n\n

McMaster Carr\n1 8209K11 840 Degree F Braided Oven Wire 20 Gauge, Tan, 10 ft. Length 1

\n\n

Will this be acceptable? I'd like to print with ABS, which I know that the heatbed has to be hotter than when printing with PLA.

\n", "Title": "Can I use 20 gauge wire for my Mk2a Heatbed?", "Tags": "|heated-bed|mk2a|wire-type|", "Answer": "

Your wire should be rated for at least the necessary 11 amps which the MK2a heatbed is supposed to take.

\n\n

You can check this by measuring the resistivity of your wire:

\n\n
\n

Always test the heatbed wiring for resistance. Remember, at 10A, 0.1Ohms means 1V voltage drop means 10W dissipated by the wiring!

\n
\n\n

as taken from http://forums.reprap.org/read.php?392,493405 (useful read for seeing what CAN actually happen)

\n\n

Did you read through the following links? They might clear up some questions:
\nhttp://reprap.org/wiki/PCB_Heatbed
\nhttp://reprap.org/wiki/Heatbeds_-_A_beginner's_guide

\n\n

I personally would use 1.5 mm\u00b2 cables, which translates to about 15 gauge wire.

\n\n

*edit: This is a nice table to check:
\nhttp://www.powerstream.com/Wire_Size.htm

\n" }, { "Id": "865", "CreationDate": "2016-03-25T13:20:07.487", "Body": "

I saw a video in which someone used GC Electronics Type 44 Non-Contaminating Heat Sink Compound (Part No. 10-8118) to hold the resistor for heating up the J-Head extruder head in place.

\n\n

Is that a good way to attach it so it heats up the aluminum block?

\n", "Title": "Attaching the resistor for the J-Head Extruder with Type 44 Non-Contaminating Heat Sink Compound?", "Tags": "|extruder|", "Answer": "

Nope, not adequate. When heating at maximum power, you should expect the resistor to be 50-100C hotter than the hot block. (It must be hotter than the block in order to transfer heat to the block.) GC 10-8118 is only rated for 200C, but the service conditions at the power resistor may run up to 300-400C depending on power output and desired hot end temp.

\n\n

For that matter, you may want to question whether the power resistor itself is rated for the >300C temps it will see in normal service. It's probably not. Most modern hot end designs use steel-jacketed heater cartridges instead of power resistors because they can handle higher temps and are, you know, actually designed to be used as heaters. The power resistors used in older hot end designs are generally being operated way past their design ratings. It's kind of amazing they work at all.

\n" }, { "Id": "866", "CreationDate": "2016-03-25T13:44:43.030", "Body": "

I ordered an OEM Power Supply and I intend to hook up with this rocker switch I believe it's like the one specified in the Power Supply Documentation which reads:

\n\n
\n

In the US, IEC320 C14 plug connections are common, but use your\n preferred standard.

\n
\n\n

I've also seen where this was attached in this thingiverse project.

\n\n

Inside the switch I've also added an F4AL250V fuse.

\n\n

Now what's confusing me is, that the this rocker switch on the back reads 10A 250V~ so I'm wondering if it's safe to use.

\n\n

The switch will be connected to the house mains via a plug and wired just like in the this thingiverse project, so it can be the input into my OEM Power Supply via the black (hot - L), white (Common - N) and Green (Ground) wires.

\n\n

For the connecting wires I stripped a black PC Tower wire to get at the black, white, and ground wires inside it.

\n\n

http://reprap.org/wiki/Power_Supply#OEM_type_PSU

\n", "Title": "Safely Hooking a Switch up to an OEM Power Supply?", "Tags": "|prusa-i3-rework|switching-power-supply|wiring|wire-type|", "Answer": "

The power supply that you posted is 12v 360w. This means that (in the USA with 120v power), it will draw 3 amps at 120v. Your switch is rated for 10A at 250V so it works.

\n\n

The switch should be placed to interrupt the HOT wire coming from the wall to your power supply.

\n\n

If you happen to be using 240v power, it is also safe as you would be drawing 1.5 amps at 240v which is still within the spec of the switch.

\n" }, { "Id": "870", "CreationDate": "2016-03-25T15:15:58.453", "Body": "

I don't want to blow out my multimeter or my electronics, so is it safe to test the DC output voltage with a multimeter on my OEM power supply?

\n", "Title": "Can I use a multimeter to test the outgoing voltage of this power supply?", "Tags": "|prusa-i3-rework|switching-power-supply|multi-meter|", "Answer": "

If your multimeter can handle the voltage, it should be safe. 12 V should be in the capability range of common multimeters, so I would tend to say yes, - but make sure that your multimeter can handle the voltage and be sure it is set to voltage mode in the appropriate range (if set to current measurement mode for example, it will not survive).

\n\n

Also take the typical precautions when measuring with a multimeter, as described in, for example, Safety equipment and precautions for DC circuit experimentation and development.

\n" }, { "Id": "871", "CreationDate": "2016-03-25T15:24:37.770", "Body": "

I obtained the following wire 22 Gauge Stranded Hookup Wire from Radio Shack:

\n\n
Type/Style: AWM/1007\nWire Gauge: 22 AWG\nInsulation Thickness: AVE. 0.42 mm\nRated Voltage: 300V\nTemperature Rating: 80\u00b0C / 176\u00b0F    \n
\n\n
\n

Use Limitation: Internal wiring of appliances; or where exposed to oil\n at a temperature not exceeding 60\u00b0C or 80\u00b0C, whichever is applicable..\n Tags may indicate the following: 600V Peak - For Electronic Use Only.

\n
\n\n

I'd like to be able to hook up RAMPs 1.4 to my switching power supply with this wire. Will this work okay?

\n", "Title": "Is 22-Guage Stranded Hookup Wire the correct wire to connect the Switching Powersupply to RAMPs 1.4?", "Tags": "|prusa-i3-rework|switching-power-supply|wire-type|", "Answer": "

22awg wiring is good for a max of 7A in this usage case. Your power supply can provide 30A. So it is definitely not large enough wire gauge for good wiring practice -- in the event of a short, you want the PSU's over-current protection to kick in before the wiring overheats. That would mean 14ga between the PSU and board. 16ga would probably cover your actual load requirements just fine, but we need to know a lot more about your printer to say that for sure.

\n\n

12v systems need heavy duty wiring, at least on the main supply lines. You can use 22ga for individual power consumers, such as fans, extruder heaters, etc. (No heatbeds.)

\n" }, { "Id": "877", "CreationDate": "2016-03-26T00:42:59.643", "Body": "

I have a monoprice printer, works great. .4mm nozzle. I bought a number of PLA solid colors and they all print reliably.\nI also bought 2 rolls of PLA translucent colors, and I have lots of trouble printing with them.\nWhat I eventually noticed was that when the translucent filaments extrude, they come out in a noticeably thinner stream.

\n\n

Not sure why, I've tried increasing the hot end temperature and printing slower, but nothing seems to help. The effect seems to be that not enough filament comes out from the nozzle for the speed the head is printing at.

\n\n

I use cura for the slicer, and I've tried changing the filament flow percentage and I've tried slowing down the print speed. Nothing seems to help, the nozzle ends up dragging the recently extruded filament around the bed until it all comes off.

\n\n

Has anybody else seen this, where translucent filament extrudes thinner than solid colors? Any suggestions what to try to make it stick and print reliably?

\n\n

Thanks.

\n", "Title": "How do I compensate in cura for thinner filament?", "Tags": "|filament|speed|", "Answer": "

In cura (if you are not in the quick print mode) you can dine the diameter of your filament. Setting the filament diameter to a smaller value will give you more plastic coming out of the nozzle. You can also set the flow percentage to more than 100%

\n\n

You should also try to use a skirt. Some nozzles need some time until the flow is like it should be.

\n\n

Another reason for this might be that the distance between bed and nozzle is too high. Try to level your bed so that the distance is less than before.

\n\n

If all this doesn't work then it will at lest give you new ideas of where to look.

\n\n

Best luck!

\n" }, { "Id": "882", "CreationDate": "2016-03-26T13:05:02.423", "Body": "

I want to print a model that I acquired in the LiDAR file format LAS.

\n\n

I don't have a printer but I'm planning to send a model to one of those shops who print for you.

\n\n

What do I need to do beforehand to make it print OK?

\n", "Title": "How to print LiDAR file format LAS", "Tags": "|scanning|file-formats|", "Answer": "

Because my research shows that LAS files are point cloud data, you would first have to convert the point cloud to a mesh. Point clouds are just that, data references to points in 3d space. A printing service works with files that represent planar surfaces, properly joined (watertight, aka manifold) to form a solid or solid surface model.

\n\n

The only resource I was able to find involves using Meshlab. There are two reasonably practical references for this process.

\n\n

The first is somewhat generic and provides the conceptual information:\nPoint Clouds to Mesh

\n\n

The second appears to be a more step-by-step method:

\n\n

Point Clouds to Mesh (2)

\n\n

I would expect that if you are successful in creating a mesh from your data, you would then want to use a program suitable to determine (and repair) any non-manifold portions of the model. There are many available, including Meshmixer, although the others have escaped my alleged mind at this moment.

\n" }, { "Id": "885", "CreationDate": "2016-03-26T15:27:18.303", "Body": "

I have done the calibration for the x, y, and z axis and everything works fine there. However when I went to do the calibration for extruder things got a little weird. The original number programmed on the board for the step per mm was 98 When I did my first measurements I used 120mm as the mark on the filament then extruded 100mm then remeasured the mark it was 37.66. Then by using the new_e_steps = old_e_steps * (100/(120-distance). I would use the new number and upload it to the printer which was 119.0187. After that I took another measurement, the new measurement was 61.27mm after marking 120mm then extruding 100mm of filament. Using the formula it came out to be 202.6540. Then the new measurement was some where around 80 some MM. It seams that the more I do the calibration the less accurate it gets. What am I doing wrong here?Triffid Hunters Calibration is the guide I have been using and this link is to the specifications to the printer HE3D Prusa XI3.

\n", "Title": "Prusa XI3 Extruder Calibration", "Tags": "|extruder|prusa-i3|calibration|", "Answer": "

I understand you marked at 120mm then tried to extrude 100mm and measured 37.66mm remaining. Take the 120mm - 37.66mm (remaining)= 82.34mm (that was extruded (so you were 17.66mm short of your 100mm).

\n\n

The formula I use is [New Setting=(Wanted Distance X old setting)/ Actual Distance].

\n\n

So [New Setting = (100 x Old Setting)/82.34.]

\n\n

Hope this helps

\n" }, { "Id": "894", "CreationDate": "2016-03-29T21:18:42.210", "Body": "

E3D mentions on their own wiki:

\n\n
\n

Excessively long retractions will cause issues by dragging soft filament into cold areas. [...] for bowden systems you might want to go up to 2 mm. Retraction beyond 2 mm is likely to cause issues.

\n
\n\n

I have retraction set to the recommended maximum of 2 mm, but I still get a lot of stringing and blobs. My printer is set up with a relatively long Bowden tube (500-600 mm). I wonder if I need to push my retraction setting slightly beyond 2 mm to take up some of the slack. Is the 2 mm a conservative rating (I guess they don't want dissatisfied customers with clogging problems) or is it really the maximum? Is there anything else I can do to improve retraction performance? (I already have a small coasting distance of 0.1 mm set.)

\n", "Title": "Can I increase retraction length on an E3D v6 beyond 2 mm if I have a really long Bowden tube?", "Tags": "|bowden|e3d-v6|", "Answer": "

To my experience, there is absolutely no problem in increasing retraction in E3D assemblies up to at least 5 mm. Typical retraction distance for my Bowden system is 3.5 mm (ABS). Clogging may occur after a series of retracts when thermal break doesn't have enough time to cool itself down.\nTo avoid clogging when there is a real need in long retracts (printing with filaments like PLA or PETG) you need to:

\n\n\n" }, { "Id": "899", "CreationDate": "2016-03-30T18:29:59.650", "Body": "

I am encountering a problem with this ID3 printer using ABS -- at some point during the print the print head displaces on the y-axis by 2-3 centimeters. I cannot pinpoint how or why it is doing this. It has displaced in the positive Y direction and in the negative Y direction on separate runs of the same piece (which is just a poker chip I found on Thingiverse).

\n\n

Is this a software issue (Simplify3D) or a hardware issue? Can anybody suggest a fix?

\n\n

See the following photos:

\n\n

\"3D

\n\n

\"Close

\n", "Title": "Why is my print displacing along the y-axis by 2-3 cm?", "Tags": "|abs|print-quality|", "Answer": "

In my case the solution was to use Slic3r instead of Cura. Cura has some issues.

\n" }, { "Id": "904", "CreationDate": "2016-03-31T07:14:05.890", "Body": "

I have a HE3D prusa XI3 printer with an MK8 extruder. While printing or testing, the extruder makes a clicking noise as if the filament isnt getting hot enough to extrude. I began my investigation and read about many reasons why this could be happening. Eliminating most of them, I finally took off the fan on the extruder and looked at it while it was trying to extrude. I noticed the teflon tubing and tried to push it back into the throat. Then the extruder worked fine. I put everything back together and began testing again. The clicking came back, - I then took the fan and heatsink back off and there it was again. Pushing back down and this time extruding some material and noticing the tubing slowly coming up. Is there anyway to fix this without upgrading to a better hot end?

\n", "Title": "Teflon tubing in nozzle throat coming out?", "Tags": "|extruder|prusa-i3|hotend|nozzle|", "Answer": "

It looks like you got too tight nozzle, too cold hotend or extruding speed is too high (all could be in the same time of course).\nAs filament goes into the teflon tube (coldend) and then into hotend it doesn't have enough time to go out throught the nozzle. So it causes that melted filament accumulates in hotend and pushes out teflon tube.

\n\n

Check application and arduino settings for

\n\n\n" }, { "Id": "907", "CreationDate": "2016-03-31T07:51:04.263", "Body": "

Recently, the ticking sounds started to come from feeder of my UM2. Inspecting it I have noticed that once in a while stepper motor jumps back for few steps. I have an idea of what can be the reason, but I just want to hear what you can think of. That is the video of feeder during the printing :)\nhttps://youtu.be/z6CzudMOeD0 First tick at 10sec

\n", "Title": "Ultimaker 2 ticking sound coming from feeder", "Tags": "|stepper|ultimaker-2|", "Answer": "

Oh yeah that is way too low for the ultimaker.

\n\n

That temp is a general \"this is what PLA melts at\" so if you took plastic, heated it up to that temp it would melt.

\n\n

However, you don't heat all the plastic at once. Your hotend it at that temp and if you let it sit it would heat up (fairly quickly). Printers like the ultimaker are fast, and well designed. They have a very small heat melt zone. So the plastic only has a fraction of time to heat up.

\n\n

Solution? Higher temps!

\n\n

(do not do this example. Not liable if you hurt yourself) You know how you can take a lighter and almost slowly wave your finger thought it without getting burned? You know if you let it stay even a full second you would regret it. Same thing here. It is going too fast to heat up.

\n" }, { "Id": "913", "CreationDate": "2016-03-31T15:23:27.067", "Body": "

I'm wondering if anyone has any experience with Makerbot smart extruders and knows whether or not the nozzle size can be changed. I'm trying to figure out if I can use different nozzle diameters (0.25mm, 0.5mm, 0.8mm, etc.).

\n\n

Makerbot doesn't sell any replacement nozzles on their site, so I suppose I'm just trying to figure out if the nozzles on the makerbot use the same diameter/thread pitch as other nozzles, like the ones from E3D or if I'd have to mill my own, which is way more trouble than it's worth.

\n\n

Any research on the subject is pretty sparse, and the best resource I found is a blog post from 2011 when they still allowed their customers to do such things themselves. I've searched around their Google Group but got nowhere. Beside that, there isn't really any information from the last three years or so on the Makerbot machines.

\n", "Title": "Are Makerbot Smart Extruder nozzles swappable?", "Tags": "|makerbot|nozzle|", "Answer": "

Yes, you can change the nozzles with the 5th gen line printers, although it's not recommended or supported by Makerbot. You're on your own if something goes wrong with an aftermarket nozzle. Here's a swap video from Fargo3D: https://youtu.be/vL80bslk9vw

\n\n

I would recommend \"mk8\" Makerbot Replicator 1/2/2x style nozzles, since these will be similar dimensions to the original nozzle. Ebay has lots of cheap ones, or P3-d and Micro Swiss are popular options for premium Makerbot mk8 style nozzles. But you should be able to use any standard M6 male thread, ~2mm ID nozzle (such as from E3D, because the Smart Extruder Z homing routine will compensate for nozzle length.

\n\n

When you change the nozzle, you're also going to need to change the slicer settings. Do not use default settings with a different nozzle. Smaller nozzles will require significantly lower print speeds, and larger nozzles will require wider extrusion width. You can create a custom profile in Makerbot Desktop to do this, or use a 5th-gen-compatible slicer like Simplify3D.

\n" }, { "Id": "917", "CreationDate": "2016-04-01T23:18:49.420", "Body": "

I'm writing a paper about a plotter (I use a Prusa i3 with a custom \"extruder\" aka pen). And I'm supposed to write about the algorithms I/the plotter use(s). I tried reverse engineering, but since I'm not very experienced with this technique I wasn't successful. \nSo I'm looking for the algorithm that Marlin uses for its G2/G3 commands.

\n", "Title": "Which algorithm does Marlin use to create arcs?", "Tags": "|prusa-i3|reprap|g-code|algorithm|", "Answer": "

The algorithm is based on the \"Vector rotation by transformation matrix\", and this solution is based on a solution from Jens Geisler. The formula for clockwise rotation is:

\n\n

\"Vector

\n\n

More information about the vector rotation can be found on Wikipedia.

\n" }, { "Id": "929", "CreationDate": "2016-04-03T18:12:34.253", "Body": "

new to 3D printers and to this community :)

\n\n

Would a 1/4\" brass rod with bronze bunting bearings for sliders and SAE 30 oil suffice for X and Y axes with moving extruder instead of print surface?

\n\n

I'm gathering cheap/scavenged materials for a first time build, as a learning experience, and it doesn't need to work very long.

\n\n

I just have no idea yet how heavy the extruders are, generally. I'm thinking of threading the rod so I can pull it tight for straightness under load.

\n", "Title": "Brass rails, bunting bearings ok for x,y axes?", "Tags": "|extruder|y-axis|", "Answer": "

Extruders tend to be quite light. An E3Dv6 complete with fan, shroud and wires is under 100g. Unless you use a Bowden extruder, you also have to add ~300g for a stepper motor. In any case, it will be quite light.

\n\n

Sintered bronze bushings can definitely be used in a 3D printer; they're used in the (quite popular) Ultimaker 3D printers. I can't tell whether your particular bushings will work well, but I think the most important consideration is that there isn't too much play between the rod and bushing (but I'm assuming you took care of that). 3D printers don't place high loads onto the motion system.

\n\n

\"Proper\" LMxUU bearings aren't very expensive either (reasonable ones can be had for less than $2) so if the bushings don't work out, you could always try those.

\n" }, { "Id": "931", "CreationDate": "2016-04-03T19:47:40.877", "Body": "

I have a long 8 mm smooth steel rod of about 55 cm long. This rods bend easily due to the length. If I replace them with 8mm solid carbon fiber rods, will the bending reduce? Will the bearings wear off the carbon fiber rod? I couldn't find too much information about this.

\n", "Title": "Carbon fiber instead of smooth steel rods?", "Tags": "|printer-building|carbon|", "Answer": "

I agree that steel is the best material for rails but the info provided on carbon fiber is just wrong.

\n\n

When making a carbon fiber part, whether a tube or any other shape, you lay the fabric in the orientation you need for the direction of the forces. If multi-directional strength and stiffness is needed then the fibers are laid in multiple directions. Nobody who knows what they are doing lays all the cf in one direction...

\n\n

In general, carbon fiber is stronger than steel by both the strength to weight ratio and by volume. I say \"in general\" because parts need to be designed in the right way. A poorly designed carbon fiber part or a low quality steel can easily be weak.

\n\n

There is a great video on YouTube showing a strength and stiffness comparison of a carbon fiber drive shaft vs the same part made in steel. The steel drive shaft bent and snapped with 3 times less force while the cf drive shaft had no visible bending until it snapped.

\n\n

The main reason why steel is a better linear rail material is surface hardness. Linear rails have bearings traveling up and down continuously. The rails need to be hardened to prevent them wearing out too quickly. The surface hardness (how easy it is to scratch or penetrate) of cf is far lower than steel. There is no comparison.

\n\n

Cf is great material for the frame but not the rails. For stiffer rails, I suggest buying some square THK rails. 15mm the rails will be plenty stiff enough for most 3d printers.

\n" }, { "Id": "936", "CreationDate": "2016-04-05T08:00:56.557", "Body": "

My printer will feature LM8UU bearings/threaded rods for the z-axis and bronze sinter bushings on the x- and y-Axis.

\n\n

As also, but not only, written here http://reprap.org/wiki/Lubrication, I know that one should:
\n - use machine oil for sinter bearings, if anything at all,
\n - grease on the 'more fluid' side for the linear bearings so that the lubricant stays eqally with balls on the upper and lower side
\n - and probably PTFE grease for the threaded rods (as for example provided by the Ultimaker UM2)

\n\n

Is there a way to unify this or at least only use two lubricants?
\nI do not have the slightest idea about lubricants, I would not know what to actually buy if the combination would e.g. be machine oil and low viscosity grease. Do you have specific recommendations of what to avoid?

\n", "Title": "Is there a lubricant that can be used for linear bearings, bronze sinter bushings and threaded rods alike?", "Tags": "|reprap|life-expectancy|mechanics|", "Answer": "

As for me... I used the oil I use to lubricate my Trombone slide!

\n" }, { "Id": "937", "CreationDate": "2016-04-05T16:25:04.363", "Body": "

I began printing the parts for the Ultrascope DIY telescope designed by the Open Space Agency. See http://www.openspaceagency.com/ultrascope.

\n\n

All of the STL files for the 3D printable parts are canted 45 degrees. Brackets, tubes, everything I have seen so far. Is there a reason for this? I printed one part last night and simply rotated the part so it would lay flat because I didn't want to deal with supports. I am relatively new to 3D printing -- Am I missing something I should know? Is this a thing?

\n", "Title": "Why are the STL files for the Ultrascope telescope at 45 degree angle?", "Tags": "|3d-design|", "Answer": "

Layer lines are failure lines.

\n

Sometimes I choose to print a part in a weird or sub-optimal orientation just to minimise the load across layer lines when in use.

\n

That may mean more support material and longer print time, but a completed part that fails is no use at all.

\n" }, { "Id": "941", "CreationDate": "2016-04-06T13:09:23.440", "Body": "

I have an idea for a 3D printed project, but I'm a total noob in this area and need someone to reality check it for me.

\n\n

Basically what I want to do is a tricopter frame made of a tetrahedral honeycomb.

\n\n

The whole model would be within 30 * 30 * 10 cm, the honeycomb edges would be approximately 1mm thick and about 15mm long and it would be printed out of nylon with SLS.

\n\n

I have found Shapeways' design guidelines and my idea seems to fit it, but still it feels slightly more extreme use than what they had in mind.

\n\n

Is this doable? Does anyone else use a similar method? Is there some software that I can use to generate honeycombs like this, or do I have to write it myself?

\n\n

Edit:

\n\n

This is how two layers of the honeycomb look like:\n\"honeycomb\"

\n\n

In the actual model there would be several layer of this on top of each other and the shape would be kind of carved out of the honeycomb (+ some finishing to avoid spiky surface)

\n", "Title": "Tetrahedral honeycomb?", "Tags": "|sls|", "Answer": "
\n

Is this doable?

\n
\n\n

It has been done, therefore it is doable.

\n\n

I agree with fred_dot_u that OpenSCAD is a good system for programmatically generating highly repetitive 3d procedural content like this space truss 3d structure.

\n\n

p.s.: A few links to people 3D printing various space trusses:

\n\n\n" }, { "Id": "946", "CreationDate": "2016-04-07T17:01:44.793", "Body": "

I have problems with setting up Marlin for my new printer. Each time I rotate the dial, axis move to the same amount. It doesn't matter how much I moved it, to 1mm or to 20 mm (according to screen), axis move to the same really small step. It happens with all of axis. I tried to change step per unit to much higher value and it still does the same. Maybe you have some ideas guys? :)

\n", "Title": "Marlin move axis issue", "Tags": "|marlin|", "Answer": "

Update. Jumpers were inserted to 1/16, but stepper chip I have (9488) could not handle that many micro steps. I changed it to 1/8 and now it works good.

\n" }, { "Id": "951", "CreationDate": "2016-04-08T08:23:49.213", "Body": "

I am new to 3D printing and need to know if I use steel in printing, do I get the same strength (compression and shear) as steel profiles manufactured in a factory?

\n", "Title": "Strength of 3D printed Objects", "Tags": "|3d-models|", "Answer": "

In terms of FFD plastics:

\n\n

Using metal infused filaments for FFD printing; definitely not. The properties mixed into the filament will have some affect on the final piece, stronger, heavier, etc for BronzeFill, but still the majority of the print material will be plastic.

\n\n

However, you can use PLA plastics (and new Moldlay wax like filament) for lost wax casting so all of the plastic is replaced with the molten metal.
\nIn this case, yes, all the plastic or filament is replaced with the final casting metal and it's at strong as any other cast item.

\n" }, { "Id": "953", "CreationDate": "2016-04-08T19:31:59.540", "Body": "

I've been asked to prepare a 3D model for 3D printing in sandstone. I've been told that it needs to be 3\" tall and the walls have to be at least 2 mm thick. It's an absolute pain in the neck having to make sure everything is the right thickness. So, I was wondering, can the whole model just be printed as a solid object, with 100% infill, and does that work for sandstone?

\n", "Title": "Can a sandstone 3D print be printed as a solid object?", "Tags": "|print-material|", "Answer": "

To add to the selected answer, again, yes you can, however making all of the walls AT LEAST 2mm thick shouldn't be too hard, depending on your model and modelling app.

\n\n

If your modelling app doesn't have a good shell function (or if it's too finicky on your model) you could easily define SOME negative space within the larger volumes of your model in order to save costs without having to actually make perfectly consistent wall thicknesses. Just make sure you leave a hole through which the excess material can escape from the interior void.

\n\n

Shapeways has a good set of guidelines for their full color sandstone prints, which work about the same as any: http://www.shapeways.com/materials/full-color-sandstone\nHowever, you'll want to double check exact measurements with your printing service.

\n" }, { "Id": "957", "CreationDate": "2016-04-09T10:15:56.380", "Body": "

Is there a way to make all your prints seamless?? I know there was this program that printed a vase constantly changing the z axis making it seamless. Why cant this be done with regular prints?

\n", "Title": "Seamless prints?", "Tags": "|prusa-i3|3d-design|slic3r|slicing|", "Answer": "

With Cura (a 3d printing software) you can go to the Expert tab and click Open Settings and you will see under an area called Black Magic, an option to Spiralize the outer contour.

\n" }, { "Id": "962", "CreationDate": "2016-04-10T10:31:22.693", "Body": "

During printing, my printer occasionally makes some mystery moves: it will very slowly move either the X or Y axis all the way to the left/front, before very slowly moving back to its original position and resuming the print as normal. I've checked my G-code files, and the moves are definitely not part of the G-code. What could be causing this?

\n\n

I'm printing from an SD card on a Cartesian printer.

\n", "Title": "Printer randomly moves to home during printing, then resumes as normal", "Tags": "|g-code|marlin|", "Answer": "

Beides a corrupt SD card that stores worng bits, leading to absurd commands, it's also possible that ithers parts in the Creatin of the file are compromised:

\n\n

This can for example happen if the card is removed during writing - but in this case it should be mostly an incomplete file on importing into an interpreter.

\n\n

More a random distribution of bad commands would appear if the writing process in itself is faulty, for example if the SD Port is defect or the adapter hast a error. Another way that writing can fail is if the drivers for the SD card adapter/port are corrupted.

\n\n

To detect a bad file or corrupted card is possible by re-importing the G-Code into a slicer (for example CURA allows this) and look at the tool paths. If any port does this with any card, software is to blame: See if it persists after a driver update (rare!) and a reinstall of the slicer. If it fails in one port but works in a different the port or adapter might bei at fault and might need replacement. If it is endemic to one card, this one is corrupt and to be thrown out. If it is endemic to a single file, overwrite it with a new one - sometimes writing fails for reasons that are almost impossible to understand.

\n\n

If the file&card are fine but read wrongly at the printer, then the card reader in the printer or the board are to blame.

\n" }, { "Id": "964", "CreationDate": "2016-04-11T03:30:22.983", "Body": "

All of Carbon3D's marketing and demonstrations show their CLIP technology producing contiguous and bridge-free parts. Has anyone seen examples of their M1 printing something like a buckyball within a buckyball or a figurine or some other shape that requires support material to print?

\n\n

For example, how would the Carbon3D print a shape like this:

\n\n
 vvvvvBUILDPLATEvvvvv\n ====================\n  XXX\n  XXX            BBB <----- columns A and B require support material\n  XXX    AAA     BBB        in order to connect them to the build\n  XXX    AAA     BBB        plate.\n  XXXXXXXXXXXXXXXXXX\n\n\n|~~~~~~~~~~~~~~~~~~~~|\n|~~~~~~~~RESIN~~~~~~~|\n|~~~~~~~~~~~~~~~~~~~~|\n
\n\n

In a similar vein, how do they handles enclosed volumes? I doubt anyone wants a sphere filled with still-liquid resin...

\n\n

Again, I've looked around, I can find no material addressing these questions, so if anyone can shed some light, I'd be glad.

\n", "Title": "How does Carbon3D's CLIP technology handle bridges and support material?", "Tags": "|support-structures|mechanics|clip|", "Answer": "

On Carbon3D's homepage they show a part that appears to have supports in it.

\n\n

\"image

\n\n

Per a conversation with Carbon3D's support they confirm that their slicing software will generate supports based on the overhang angle geometry, and in the case of a buckyball within a buckyball, there would be supports generated to create the buckyball, and to support the interior buckyball joining the two together that would have to be removed in post processing.

\n\n

Enclosed volumes need to have a drain hole, and you would have to avoid vacuum forming shapes such as an upside down cup in which atmospheric pressure would keep the interior of the cup filled with resin until the reservoir runs out or the vacuum is able to be broken releasing the excess resin. In the case of a cup you would change the orientation. I am not sure how one would handle trying to print a solid sphere with no holes to avoid this condition.

\n" }, { "Id": "970", "CreationDate": "2016-04-12T07:13:43.367", "Body": "

I have a 3D printer that is going crazy with x-axis shift, and I need a tension gauge belt to measure the tension.

\n\n

I've never used one before, and looking online, I can't tell which one would be the right fit.

\n\n

Any ideas? What things should I look for?

\n", "Title": "Which belt tension gauge do I need?", "Tags": "|hardware|belt|", "Answer": "

Added for reference. To my understanding, especially if you run multiple identical printers, having the same belt tension on all of them is essential to getting accurate and consistent results. The effects of overly loose or overly tight belts have been demonstrated in a YouTube video by CHEP.

\n

As mentioned in the comments above, a frequency analyser on a smartphone can be used, a web-based analyser is available at belt.connect.prusa3d.com. Additionally, the article Adjusting belt tension (MK3/MK3S/MK3S+/MK4) explains the concepts and steps that can be taken for belt tensioning.

\n

\"Photo

\n

Also, a simple belt tension measurement tool called Tension Meter for the GT2 belts of i3 MK3S+ or Prusa MINI+ that can easily be printed has been published by Prusament.

\n

\"Tension

\n

There is a similar printable model called True Open source tension meter for GT2 belts by ChipCE.

\n

\"True

\n" }, { "Id": "974", "CreationDate": "2016-04-12T20:56:20.047", "Body": "

About two years ago I purchased a Boots Industries v.2.5 3D printer in a delta configuration. The wiki is still up at this location where you can see what the printer looked like.

\n\n

The printer is prone to break and slip because the design uses thin wire to move the carriages up and down. I'd like to convert my printer over to using a belt like the one on this page, but the conversion kit is no longer available. I believe I can 3D print the carriages, but would anyone be able to point me toward the resources to convert my printer over to using a toothed belt?

\n", "Title": "Looking for information on modifying Boots Industries 3D printers", "Tags": "|delta|belt|", "Answer": "

Contact Brian or Lykle. We all have Boots Industries printers and have heavily modified it to belt drive and magnetic effectors. The print quality has improved drastically. The old Boots support group has since moved on to Slack and we are in constant contact everyday.

\n\n

https://biv25.slack.com/messages/@slackbot/

\n\n

https://www.thingiverse.com/thing:1843195

\n" }, { "Id": "975", "CreationDate": "2016-04-13T01:05:20.973", "Body": "

I'm still looking at wires for my Prusa i3, to go between the power supply and RAMPS 1.4, and the power supply and the MK2a Heatbed.

\n\n

I also recently found a 400 ft. Wire Storehouse that I bought from Harbor Freight which has wire sizes in it from AWG 10 through AWG 22 (and additionally speaker wire, Zip Cord and Bell).

\n\n

I also bought some reading material, I picked up Wiring Simplified 44th edition, and in it on page 28, Table 4-1, there is a table with information about the Ampacity of copper wires including their maximum temp (C), and maximum carrying current (Amps) based on their insulation types.

\n\n

Unfortunately, the 400 ft. Wire Storehouse does not provide any information in regards to the insulation type or quality and this makes it difficult for one to choose the correct wire based on the specification in the table.

\n\n

Given that the thing only cost $30 for 400 Ft. of wire, it would lead me to believe that the cheapest grade of insulation was used; as I understand it, the TW type wire.

\n\n

I also read a forum somewhere in which people were complaining about the cheapness of the wire in this kit, stating that one ought to wear gloves when working with it as there is probably lead in the insulation as well as the wire.

\n\n

The largest copper wire I have found in the table that I have (AWG 10) says that it is rated at 30 AMP regardless of which type of insulation it has, should I be using the speakerwire instead? That isn't listed in the table. Also it should be noted that though the ratings for the Ampacity are 30 AMPs, the max temperatures are different; with the TW being at 60 C.

\n\n

As far as I can tell if I use the AWG 10 (TW?) to connect everything it won't matter, but I just thought I'd check here to be sure first since my power supply is rated at 30 AMPs and that's probably the same as the wire....

\n", "Title": "Do the TW, THW and THHN or THWN wire insulation types matter in terms of powering RAMPS 1.4 or the MK2a Heat Bed?", "Tags": "|ramps-1.4|switching-power-supply|wire-type|mk2a|", "Answer": "

No "mystery meat" wire in AC electrical

\n

That kit is random Chinese "no-name" hookup wire and cannot be used for AC power. It's fine for low voltage hobbyist tinkering, which is what it's sold for. If it was better than that, they'd say.

\n

But who knows? Maybe the boat didn't arrive and they needed some 12 AWG to finish some kits, so they ran to the electrical supply and bought bona-fide Southwire THHN. If so, the wire would be labeled "UL Listed" and "THHN/THWN-2". Then you can use it for AC power.

\n

Without "UL Listed" or "UR Recognized" or other mark from a credible NRTL, any other labeling is lies. CE is not an NRTL.

\n

Proper THHN/THWN-2 is available locally, dirt cheap.

\n

Competent hardware stores and lumberyards will cheerfully sell it by the foot. So like 6 feet for a dollar-ish.

\n

(However they are often not located in strip malls, so some may need to rethink their shopping mindset if they want value).

\n

Any good tinkerer/hacker ought to know the location of every good family-owned hardware store, real lumberyard, and electrical supply houses, bonus points for HVAC supply.

\n

TW, THW and THWN-2 are NOT "good, better, best".

\n

More like the Ford Edsel, Thunderbird and Escape: "obsolete, obsolete, and currently sold product".

\n

Why not keep making TW, THW and THWN? No money savings there. It would cost more to keep 4 separate production lines than to just make everything dual-rated THHN/THWN-2. So that's what they do.

\n
\n

it would lead me to believe that the cheapest grade of insulation was used; as I understand it, the TW type wire.

\n
\n

No, wiring doesn't work that way. You can't even assume it's a "T" family - there are also XHHW and RHH/RHW and the "S" family cordage, just in NEC Chapter 3 wiring methods. And many other families not approved for AC power.

\n
\n

Unfortunately, the 400 ft. Wire Storehouse does not provide any information in regards to the insulation type or quality and this makes it difficult for one to choose the correct wire based on the specification in the table.

\n
\n

Doesn't matter what the ad copy says, it matters what the markings on the wire say. Wire that is unmarked is literally nothing.

\n

Wire manufacturers know this. Chinese wire manufacturers don't care. But that Harbor Freight pack never claimed to be anything else, did it?

\n" }, { "Id": "978", "CreationDate": "2016-04-13T07:30:04.903", "Body": "

I have a really bent heatbed PCB, the middle is elevated about 3 mm with respect to all edges.

\n\n

I have found this thread Warped PCBs, where a heating method is applied by baking a PCB in the oven, as described here: 3.2 Bow and Twist Repair.

\n\n

Can this help straightening out a Prusa heatbed PCB? If so, can I apply the heat by the heatbed itself, or do I need to utilize an oven? Will the pressure from the strongly clamped glass plate be enough or will the glass break at these temperatures (given that the heatbed can reach them).

\n", "Title": "Can a heatbed PCB (Prusa Mk2) be straightened out?", "Tags": "|electronics|heated-bed|repair|", "Answer": "

Easiest way would be to place a glass sheet on top of the bed and run a calibration... Although, that way you may lose a few millimeters of print height.

\n" }, { "Id": "979", "CreationDate": "2016-04-13T14:10:00.600", "Body": "

Before you put duplicate from this Which are the food-safe materials and how do I recognize them? please read

\n\n

I need to know if this 3D Ink\u2122 (PLA Filament) is food safe\"this

\n", "Title": "Micro 3D filament food safe", "Tags": "|filament|food|", "Answer": "

In general no filament is safe as the printing process leaves \"nooks and cranies\" between the layers where germs can gain a foothold.

\n\n

For food safe, I would advise:

\n\n\n\n

At that point it it should be dishwasher safe and food safe.

\n" }, { "Id": "984", "CreationDate": "2016-04-13T19:59:29.573", "Body": "

Is there any well-known test set of 3D models that I can print out which will clearly show up the problems I need to work out in calibrating my 3D printer? Ideally, it would be either one or multiple models which shows the point at which overhanging starts to fail, points at which stringing occurs, the accuracy of one layer over another, and maybe edges which are supposed to be a particular length.

\n", "Title": "3D printer calibration test STLs?", "Tags": "|3d-models|calibration|", "Answer": "

Yes.

\n\n

There are more test models one can download than would be possible to list here. A search on Thingiverse results in pages and pages of useful models. A common model for testing is called the 3d Benchy, although it is not particularly parametric. Overhangs, retraction, layer alignment and other aspects of your printer are tested with this model.

\n\n

Another option is to purchase a set of test models from 3dkitbash.com:

\n\n

QuickPrintTests

\n\n

The models lists as such:

\n\n
#3dk01 - Voids\n#3dk02 - Spikes\n#3dk03 - Flex\n#3dk04 - Edge\n#3dk05 - Text\n#3dk06 - Bridge\n#3dk07 - Dissolve\n#3dk08 - Ball Joint\n#3dk09 - Hinge\n#3dk10 - Columns\n
\n\n

which are more specifically defined for testing your printer. I would expect that Thingiverse and other model-sharing sites would have similar designs available to meet your specific requirements.

\n" }, { "Id": "991", "CreationDate": "2016-04-14T04:23:04.040", "Body": "

On my Switching Power Supply there is a little orange trimpot that's marked VR1; what does that do, and does it work similar to the ones on the stepper drivers?

\n", "Title": "What does the trimpot on my Switching Power Supply do?", "Tags": "|printer-building|switching-power-supply|", "Answer": "

No doubt it's just a final tuning potentiometer. Even on the pictures you linked it's described as V adj which stands for voltage adjustment.\nIt's a way to tune your power supply's output as it can vary depending on temperature/humidity/wall-plug voltage/etc.

\n\n

It's usually set properly and doesn't need to be touched. But you can connect a multimeter to check if your power supply gives the voltage you need.

\n\n

If you are not familiar with the subject but still want to do it yourself, it's good to set your multimeter to the highest available range and make sure that AC/DC is properly chosen. Now you can measure voltage. You can set the range closer to the expected values then. Usually multimeters have a range around 20 or 30V DC which is propbably what you expect from your power supply. In such a range you should see something like 12.000. If it varies from 12.000 you can then precisely adjust it to get as close as possible to this value. But watch out... cheap power supplies can cheat you. When the are have a load on the output (your 3D printer for example) they can reduce voltage.

\n" }, { "Id": "992", "CreationDate": "2016-04-14T04:44:40.390", "Body": "

I have a solidoodle 4 and have blown the power supply . Long and the short of it is I am trying to modify a Xbox 1 power supply to fulfill the need. I am measuring he voltage at the output and its 14.04 volt.

\n\n

Question is this, is the operational voltage range on a solidoodle able to operate on 14 volt ?

\n", "Title": "Solidoodle 4 voltage requirements", "Tags": "|printer-building|desktop-printer|electronics|", "Answer": "

I have not used or seen a solidoodle 4, but as found on the internet, the controller board used in the printer is a version of the Sanguinololu, and the reprap wiki says that

\n\n
\n

The board is designed to be flexible in its power source, working with\n a 12V/5V ATX power supply or any 7V-35V power source via the on-board\n voltage regulator.

\n
\n\n

if you're using a board not older than V0.7 http://reprap.org/wiki/Sanguinololu

\n\n

However, I have also found images that show the Solidoodle with this board:
\nhttp://www.reprap.org/wiki/SAV_MKI
\nwhich can be used with 11-15V. This however might be for older versions of the printer.

\n\n

There are some things to keep in mind: if there is no voltage regulator on the board, make sure that all parts can take the voltage without overheating at some points.

\n" }, { "Id": "1006", "CreationDate": "2016-04-15T12:29:19.757", "Body": "

Sorry if this isn't the right place to ask, but it seemed like the most relevant stackexchange.

\n\n

I want to print a connection from a motor to an omniwheel, the axle of the wheel is listed as 8mm, but the one of the motor only say \"3h5\" which isn't really helpful for me, and neither was Google.

\n\n

Side-question: Would pla be sturdy enough or do I need to use abs?

\n", "Title": "What does a \"3 h5\" mean in reference to the axle of a motor?", "Tags": "|abs|pla|3d-models|print-preparation|motor|", "Answer": "

I think it's a reference to a tolerance. Look at this ISO table. This States that the nominal diameter of 3mm is held to a tolerance of +-0.004mm.

\n\n

@DarthPixel provided some great links identifying the term interference fit (or press fit as I've heard locally) as described here. Also, here is a better link providing examples of how the tolerance works and more legible tables to reference the correct tolerance. Keep in mind that the units in the tables is in nanometers.

\n" }, { "Id": "1012", "CreationDate": "2016-04-18T01:39:11.703", "Body": "

I just recently purchased a new MightyBoard (Rev G) for my Replicator that requires a new power supply.

\n\n

The power supply for the Rev E was a 24 V/9.2 A which was necessary for the dual extruders and heated bed. I know I need a 24 V power supply, but should I be concerned about the amperage? What will a higher (or lower) rating affect on my machine?

\n", "Title": "Power Supply Safety", "Tags": "|electronics|switching-power-supply|mightyboard|", "Answer": "

A short, figurative answer from the electronical point of view:

\n\n

A power supply (an an analogy you can view it as a water pump) as used by 3D printers is usually supplying a fixed voltage (a constant pressure going into your pipe system), in your case 24V.

\n\n

The given amperage/current (the amount of water that actually flows) that is actually utilized at a given point in time is determined by whatever you hook up to your power supply (the system of tubes or pipes - imagine a valve like your tap/faucet). Now the amperage rating of your power supply gives how much current you can run through your electrical system (as is the flow of your tap/faucet by the pump if you had a limitless big, imigainative one).

\n\n

The power that your supply can deliver is the product of voltage and amperage: P(power) = U(voltage)*I(current).

\n\n

What you need to make sure is that a) you deliver the correct voltage, because this is what your circuitry needs to be specifically designed for (image the pressure of your pump being too low or to high, - you either won't recieve any water in the second floor, or your tubing can't stand the pressure) and b) that you can supply at least the needed power (otherwise you get a problem once you open up all the taps/faucets in your house, because they don't supply as much water as demanded). If your power supply can give more current, that's fine, it might just not be used. And as mentioned by Thetravellingfool already, keep a certain plus for losses and as a reserve, because no pump likes to run constantly at it's limits either ;)

\n" }, { "Id": "1032", "CreationDate": "2016-04-21T02:12:52.817", "Body": "

I was working on my printer when something metallic came into contact with the pcb. I smelled smoke and quickly unplugged the printer. Anyway, this is the result and, of course, the heat bed won't heat.

\n\n

Can this be salvaged or should I toss it and buy a new one?

\n\n

\"shorted\"

\n\n

update the heat bed was not hot at the time. I had the heat bed unscrewed from the chassis but had forgotten to unplug the printer. I am not exactly sure how it shorted but I think it shorted between the power lead connection and the thermistor.

\n", "Title": "Can a short-circuited heat bed be salvaged?", "Tags": "|heated-bed|", "Answer": "

Yes, it should be possible to fix, although you might choose to replace anyway on the basis that the repaired bed might give you concerns about how long it will be before your repair fails.

\n\n

You will need to carefully remove the protective layer to expose the heating element (assuming you can identify where it is likely to have broken). Then carefully solder across the break (maybe with a short fragment of wire).

\n\n

After making the repair, you should cover the exposed track. This provides both electrical and thermal insulation. In the absence of any suitable high-temperature paintable covering, you could try using kapton tape.

\n" }, { "Id": "1051", "CreationDate": "2016-04-27T04:15:08.270", "Body": "

I'd like to find an opensource software that communicates directly with the RAMPS board (or any other 3D printer driver) without using the arduino...

\n\n

It doesn't matter if it runs on Windows or linux as long as it's opensource. Also, I'm not worried about how this communication is done (USB, serial port, parallel port)...

\n\n

Has anyone ever heard about projects like this?

\n", "Title": "Is there any opensource software that controls the RAMPS RepRap board directly from the computer?", "Tags": "|ramps-1.4|software|open-source|", "Answer": "

The key question is not really if there is such software, but rather if such software is possible, or if it would work very well.

\n\n

While a printer is more than just stepper motors, those are one of the trickiest parts, so it makes sense to look primarily at that.

\n\n

Back in the old days of personal computing, it was not uncommon to generate stepper motor signals from the CPU of a personal computer, using individually settable bits on either a special purpose interface (that is in fact basically how the head in a floppy drive was typically moved) or borrowing another available interface such as a parallel printer port.

\n\n

But then two things happened: computers got faster, but more isolated from the world, and operating systems in common use became much more strict in what they permitted.

\n\n

To move a high resolution stepper at a decent speed, you need to generate either step pulses or winding activation signals at a fairly high rate. And to accelerate and decelerate a motor under load, you need to finely vary their timing. Back when I/O ports hung directly off processor buses, and operating systems could not prevent programs from speeding up the system hardware timer in order to run a stepper routine rapidly, this worked to a degree. But today:

\n\n\n\n

While there are ways around these issues, they tend to require atypical hardware and deep changes to the operating system installation - making them neither inexpensive nor easy to setup for end users.

\n\n

Instead, it is generally simpler and more cost effective (not even $10 these days) to put an embedded processor on an external circuit board, and have it act as a delegate to execute precise-timing tasks on behalf of the host processor. Somewhat extending from the idea of industrial CNC machines that originally read punched paper tape, and were later updated with a scheme where an ordinary computer \"drip feeds\" G-code commands over a serial port, modern 3D Printers tend to deliver G-code (or other) command data a little bit in advance of when it is needed, so that the latency of a USB or serial connection doesn't really matter. Normally enough data is buffered on the printer for it to keep running, but even if not it would only pause briefly between the complete moves which are transmitted, not experience motor stuttering as it would if the USB were trying to deliver each individual step pulse.

\n\n

As for why an Arduino - probably mostly the history of who built the machines which kicked off the enthusiast printing trend. If someone from an industrial background were tasked with building something like an FDM printer or a machine with similar motion needs in isolation today, chances are they would end up with an ARM processor that would be a bit faster, more flexible, and with more resources, and likely cost a little less. But in actual history, the early affordable machines were built by those in maker community, who were already familiar with the availability of the Arduino, and willing to put some cleverness into getting good motion out of its limitations. RAMPS in particular seems designed to be a coarse-pitch through-hole bridge that a hobbyist could build themselves, and then buy the slightly trickier to work with surface mount processor and motor drive chips preassembled in the form of an Arduino Mega and stepper drive modules. That even fairly propriety machines maintain these basic parts choices is probably an indication of the utility of not \"reinventing the wheel\" - if you want to develop a printer, you can start from available components and customize them only one by one as you choose, rather than not being able to run your development prototype until you get a working circuit board designed and fabbed, a working software base developed, etc.

\n" }, { "Id": "1053", "CreationDate": "2016-04-27T13:12:32.580", "Body": "

In Google Sketchup, I have text on a surface that I'd like to push / pull so that the text is etched into, rather than placed upon the surface. I can't seem to figure this out, the text disappears when I try to push it into the surface and I'm not quite sure what else to try. Any advice?

\n", "Title": "Pushing Text into surface - Google Sketchup", "Tags": "|3d-design|", "Answer": "

Definitely you have to perform substract operation.\nTake a look here

\n\n

Extrude text and then substract it from the object to etch in.

\n" }, { "Id": "1058", "CreationDate": "2016-04-28T12:15:26.643", "Body": "

The wiki page for the RAMBo board mentions:

\n\n
\n

Step and Direction pins are on their own ports for synchronous movement capability

\n
\n\n

What does this mean, and how does it affect printing? Do I need special firmware to take advantage of this?

\n", "Title": "What does it mean for step and dir \"to be on their own ports\"?", "Tags": "|electronics|firmware|", "Answer": "

A \"port\" for the AVR microcontroller is a set of eight IO pins that are controlled together at a hardware level. The underlying machine code can write an entire byte to set the status of all 8 pins at once. So in principle, all the step pins on the RAMBO board can be triggered exactly simultaneously.

\n\n

The Arduino IDE hides this behavior from firmwares by abstracting the port's byte handling as a function call for each individual pin. That means programmers only have to know the Arduino PIN number, not which port and position on the processor is being used. And then the modern firmwares typically used by RAMBO do a further layer of abstraction to make it easy for the same firmware to be used by different boards, by changing pin assigned names in a config file (usually pins.h) and not assuming any specific port selection was made during board design.

\n\n

There's no guarantee for open-source firmware that a particular pin selection was made in hardware. Lots of 3D printer controller boards even have pretty dumb pin assignments, like using hardware-PWM-capable pins for stepper signals rather than PWM'd heaters. Marlin and Repetier have chosen flexibility over performance optimization in this regard. They pretty much let the board designer use any pin for anything. When they have to do stuff like run heater PWM control or fire a bunch of step pulses as fast as possible, they emulate that in software rather than taking advantage of specific hardware that isn't always available.

\n\n

In this specific case, there may not actually be all that much performance gain/loss. Writing to an output pin is pretty fast. The amount of time difference between firing a few step pulses in sync or firing them sequentially is on the order of a few microseconds. That won't make any difference to the motion fidelity of the printer's physical drivetrains. Somebody involved in the RAMBO design just thought it could be useful, and put a reference to it on the Wiki page (in the very first upload!) and it's never been clarified or removed from the Wiki page in the years since.

\n" }, { "Id": "1063", "CreationDate": "2016-04-30T22:06:39.797", "Body": "

I have a flashforge creator dual.

\n\n

From time to time while printing a model I notice when there are abrupt changes in direction, that a few \"Grains\" of old filament fall out from the head. Generally they end up embedded in the print which is not a big deal, but sometimes they get into a visible portion.

\n\n

I can mitigate this somewhat by disassembling down to the gears, and blowing out all of the old filament.

\n\n

My question is two fold
\n\"What is causing these chips to accumulate near the gears?\"

\n\n

\"Is there something I can do to resolve this issue?\"

\n", "Title": "Print contaminated by chips of previous filament colors", "Tags": "|print-quality|", "Answer": "

My bet is your extruder knurls are scratching the filament. If it happens for specific filament then probably it doesn't keep its diameter. It means you can avoid these grains by using better filament.

\n\n

If it happens always then you could do 3 things

\n\n
    \n
  1. reduce stress on the spring which pushes filament to extruder gear
  2. \n
  3. make extruder gear less sharp
  4. \n
  5. eventually change the great itself
  6. \n
\n" }, { "Id": "1065", "CreationDate": "2016-05-01T06:42:46.247", "Body": "

Hey so theres a product we have been prototyping. \nWe cant do FDM. The item has a ball valve. \nand the ball ~ 1.6-1.9mm \nI cant seem to prototype it. I have tried SLS, SLA, Polyjet. \nAnybody knows what could be wrong, or what should be the dimension i should be using?\nI thought I would post this at engineering section, but since I have to prototype I thought its best if its in 3D Printing section

\n\n

Edit:

\n\n

\"Ball

\n\n

Update:

\n\n

\"Figure

\n\n

So heres a figure showing the direction of air flow.

\n\n

Update: \nThanks both answers are helpful. though not a precise answer. Then again, there is no correct answer. \nThanks a bunch. Lets see how the coming months are. :p

\n", "Title": "Ball valve to be prototyped", "Tags": "|3d-design|rapid-prototyping|", "Answer": "

Hey why to make it soooo sophisticated and poor?\nAir valve cannot be designed with 2 stiff elements - it will never work.

\n\n

Make the air your friend but not the enemy. Use old good rubber (silicone) \"flake\" instead. Especially when you have such small design.

\n\n

Take a look on the picture.

\n\n

\"enter

\n\n

Here you have black element which is rubber or silicone. It's sticked on one side. When air goes from the pipe (goes up) then rubber is slack so the air can flow. When you suck then rubber seals the pipe.

\n\n

[edit]

\n\n

You can find such solution in for example bike pumps, inflatable matterss pumps also gas masks and so on. It's pretty fine for low pressure.

\n\n

Of course ball valves are also widely use but then such device has to have \"rubber\" ball nest so ball lies on the rubber ring to seal it. There has to be a spring to tight the ball to the ring. Without such spring turning the device upside down would cause the valve would not work at all.\nBall solutions are intended to higher pressure.

\n\n

Please note the spring which tights a ball creates kinda threshold on the pressure level which means that pressure has to exceed a spring force to reduce a pressure. This doesn't occure in rubber valve.

\n\n

I don't know what device you are trying to prototype but I'm pretty sure a rubber valve will be just fine.

\n\n

Please also note that sealing surface is very low in ball valve solution but in rubber valve it can be almost unlimited so it can seal quite well even when the surface is not perfect.

\n" }, { "Id": "1076", "CreationDate": "2016-05-02T14:25:23.397", "Body": "

I've been interested in 3D printing for the past month however, I have noticed that it's sort of a \"reserved\" topic. Meaning that everyone who talks about it, has already some basic knowledge about the topic. What are some good resources for someone who wants to start learning from zero? My main goal is to acquire enough knowledge in order to build my own 3D printer.

\n", "Title": "What is a good book to read about 3D printers?", "Tags": "|printer-building|", "Answer": "

If you really want to learn about three dimensional printing then \"Mastering 3D Printing\" by Joan Horvath, published by Apress, would be a good place to start.

\n\n

It does not provide you the in depth knowledge about all the types of printers or materials but it is good enough to get a newbie like me or you get of the ground.

\n" }, { "Id": "1083", "CreationDate": "2016-05-03T16:21:48.817", "Body": "

I've printed mostly ABS in the past and encountered delamination between layers many times. I've ensured the following conditions regularly:

\n\n\n\n

What are some other variables to consider to help prevent delamination between layers?

\n", "Title": "Layer delamination", "Tags": "|fdm|makerbot|abs|print-quality|delamination|", "Answer": "

wall size and filling are also parameters.\nIf wall size is too thin delamination is more visible

\n" }, { "Id": "1085", "CreationDate": "2016-05-03T18:08:00.367", "Body": "

I'm using brand new PLA filament and am getting frequent clogs in my extruder.

\n\n

I've had this problem with 2 different filaments from 2 different vendors.

\n\n

It will be print just fine, then clog up. It doesn't ever seem to go more than 5 minutes before clogging. When it clogs, and I pull out the filament, it is always twisted in a spiral (helix) shape like a corkscrew. I have put a picture of 2 clogs below.

\n\n

I have tried using temperature of 220 all the way down to 180 in increments of 5 degrees and seem to get the same result. it prints the base layer GREAT on the 70 degree heated bed. No warping or peeling off. However, after a few layers, it clogs up and stops extruding.

\n\n

I am using an HIC PRUSA I3 printer with a single extruder head. I've only had the printer for a couple weeks. It had been printing fine with ABS, but the ABS would peel up from the heated bed, so somebody suggested that I use PLA and hairspray. Hairspray is AWESOME !! It sticks really well and removes easily as well (once the bed cools a bit).

\n\n

Please let me know if you've had the same problem with the extruder just clogging up and twisting the filament into a corkscrew shape.

\n\n

By the way, pay no attention to the black marks on the green filament below. That's just me marking every half inch or so with a sharpie marker to see if it's still being extruded.

\n\n

\"Filament

\n\n

I think I figured out the problem. Now, to figure out a solution...\nTake a look at the image below. There is a 1 inch tube that goes from the heat element to the heat sink. This 1 inch of tubing is REALLY hot and larger than 1.75mm. So, the filament goes through that tube on its way to the head and gets soft in the tube because the tube is so hot all the way down to the head. When the filament gets hot, it melts and bends and curls which makes it NOT push itself into the heated tip and out onto whatever I'm printing.

\n\n

The solution would be to find some way to cool this 1 inch shaft between the heat sink and the heated head so that the filament inside of it won't melt.

\n\n

Any ideas???

\n\n

\"Extruder

\n\n

Here is a picture of my heat tip. Note the shaft has about 1 inch sticking out of the heater. The top of that (above the white arrow) is inside the heat sink. But 3/4 inches of it are bare and uncovered. There is also no teflon tube inside the metal throat.

\n\n

\"enter

\n\n

\"Another

\n", "Title": "Why does my PLA filament form a spiral shape and clog my extruder?", "Tags": "|filament|extruder|prusa-i3|pla|", "Answer": "

I've found that this happens on my prints on the first layer when it's a really big layer. If you have the initial fan speed on something really low, and it prints for a long time (ie giant first layer) it will twist up the filament. Thanks for the answers, I was confused too.

\n" }, { "Id": "1089", "CreationDate": "2016-05-04T02:47:04.393", "Body": "

I found a nice model for a ship from the game \"Eve\". It doesn't have a flat bottom, so it needs support material. But Slic3r generates several dozen tiny support pillars, and one by one they break loose from the build plate. As they get tall, the leverage of course increases, and since they're so tiny they don't have much area in contact with the bed, so they separate. Eventually the whole model broke free, turned a bit, and since I had gone to dinner it kept printing... see below. I think the main problem is the poor support material geometry, but the controls in Slic3r for support material don't seem to give many options.

\n\n

How can I get better support material layout? Oh, this is PLA, by the way.

\n\n

\"Rifter

\n\n

(appending to question to be able to put in the picture)

\n\n

I tried a bunch of things, and the MeshMixer support, plus fiddling with several settings, got it to come out pretty well. Thanks to all, esp. @Tormod!

\n\n

\"enter

\n", "Title": "Getting better support than Slic3r generates", "Tags": "|support-structures|support-material|", "Answer": "

In my point of view it seems that the bed offset not optimal.\nI see that you are using the blue tape, which is better than any other tape. As a next step I would recommend to try different types of glue.

\n\n

I agree with Tormod Haugene and can also recommend Cura. You can also experiment there with a lot of parameters and after a time you will get a feeling how to print your things successfully.\nJust take a look at the download page at Ultimaker for the application and if there are problems with the software you can feel free to report issues at GitHub.

\n\n

Regards

\n" }, { "Id": "1098", "CreationDate": "2016-05-04T13:19:59.970", "Body": "
\n

You may want to use this code to test all the electronics before\n installing any of the suggested firmwares.

\n
\n\n

I'm planning on:

\n\n
    \n
  1. Flashing the test firmware onto the Arduino Mega 2560, then unplugging it from the USB.
  2. \n
  3. Connecting RAMPS 1.4
  4. \n
  5. Connecting all 3 jumpers under the X-Axis (leaving the other jumpers disconnected).
  6. \n
  7. Connecting a stepper driver to the X-Axis on the board.
  8. \n
  9. Turning the trimpot down all the way, and then back up 1/4th of the way.
  10. \n
  11. Plugging in 1 NEMA 17 motor to the X-Axis.
  12. \n
  13. Connecting 5A DC input into RAMPS 1.4 (not plugged in).
  14. \n
  15. Finally plugging it in and seeing if the motor moves for 5 seconds.
  16. \n
\n\n

Now my question is, if I'm going to do this to test out a single NEMA 17 motor, do I need to comment out the rest of the test code before loading the firmware?

\n", "Title": "Should I comment out the code for what I am not testing when loading the test firmware for Ramps 1.4?", "Tags": "|ramps-1.4|firmware|", "Answer": "

There's no reason to comment anything out. It will work just fine without any modification, even if you're just testing a single motor. That said, I don't see the value of this \"test firmware\" over just installing Marlin.

\n" }, { "Id": "1102", "CreationDate": "2016-05-04T15:35:03.997", "Body": "

Has anyone tried painting a PLA (or other) print with stone-look spray paints? In particular, how does the result look compared to printing with stone-fill filaments? It looks a lot cheaper, but will it look inferior?

\n\n

One such paint: http://www.krylon.com/press-room/press-releases/2013/04/add-a-touch-of-nature-to-your-next-project-with-new-natural-stone-textured-finish/?

\n\n

One such filament: https://www.matterhackers.com/store/3d-printer-filament?t=LAYBRICK

\n", "Title": "Stone-look surface via painting?", "Tags": "|filament|post-processing|filament-quality|", "Answer": "

It's totally possible to achieve but the result vastly depends on your painting skills and your spray paint quality.

\n\n

Your can look at this page for a concrete example.

\n\n

How it compares to stone filament is fairly subjective though. Painting a 3D print usually breaks down in 3 steps :

\n\n\n\n

If any of these steps are neglected, the final result won't look as good as using stone filament. However, if the quality of you stone filament is really bad, painting will offer a better result.

\n" }, { "Id": "1113", "CreationDate": "2016-05-05T20:20:01.943", "Body": "

I am very close to buying a 3D printer and have started to do some preliminary design work from the things I'd like to make, but I have a question: Which corner of the print bed corresponds to the origin (0,0,0) in slicer software? Is this the same across slicers and printers?

\n\n

The reason I ask this is because of the difficulty some have in removing items from the bed. It seems to me like I'd want to print small items closer to the front of the printer to make access easier, but it looks like most slicer hosts only show a box representing the build volume with no real indication of what's \"front\".

\n", "Title": "Slicer/Printer Origin", "Tags": "|software|slicing|", "Answer": "

On Cartesian printers, the origin is usually the front left corner.

\n

On Delta printers, the origin is in the center.

\n

If you are having trouble getting prints off the build plate, maybe consider a removable flexible build surface, such as the Easy Peelzy.

\n" }, { "Id": "1117", "CreationDate": "2016-05-06T00:02:50.243", "Body": "

i have seen formbox http://golem13.fr/formbox/ and it uses heat and many materials like PVC. Is there any way to do this with paper or paper derivatives(like egg carton)? If the answer is yes, how can i do that?

\n", "Title": "Alternative 3d molding techniques at home", "Tags": "|3d-models|material|", "Answer": "

Forming kinda blister out of paper is possible but not the way it's formed out of plastic. Paper is not elastic, it's bendable but not strechable. It's the issue of how cellulose fibers are formed and how are they entangled.

\n\n

Anyway - you can create a form out of wet mixed paper pulp. Wrap it around your model and then wait until it's dry. It will form amazingly stiff form. You can do it also out of wet handkerchiefs.

\n\n

You could build a vacuum machine as in the formbox to squeeze the paper pulp the get water out of the material but the cover should be done out of light silicone film so it would be treated by vacuum.

\n" }, { "Id": "1120", "CreationDate": "2016-05-06T18:56:01.593", "Body": "

I'm trying to set up OctoPrint on my Linux Ubuntu 16.04 desktop to work with my Micro3D printer. The printer works fine on Mac and Windows:\n\"enter

\n\n

I've installed OctoPrint with the M3D Fio Plugin Manager. I can see the files in ~/.octoprint/ fine and I can even turn the printer fan on/off, move the extruder, etc. through OctoPrint:\n\"enter

\n\n

But when I press the blue \"Print button\", there is no response. Also the files in the \"upload\" section are greyed out. Must I download a slicing program or something? I'm new with 3D printing in general but not with Linux. Grateful for help!

\n", "Title": "3D printing using OctoPrint on Linux Ubuntu desktop", "Tags": "|desktop-printer|octoprint|", "Answer": "

For me it sounds like you've missed to install the CuraEngine for slicing, but I'm only guessing, as I'm not using OctoPrint at all.

\n\n

Instead I'm using Cura directly and save gcode to a SD or use USB printing for quick/small prints.

\n\n

Regrads ;)

\n" }, { "Id": "1127", "CreationDate": "2016-05-08T14:30:46.043", "Body": "

I've noticed that some of my prints (mostly square-ish objects) are coming out with gaps between the outside shells and the inner parts. This gap is visible even in the 2D sliced preview of the layers so I think it must have something to do with slicing settings, but I'm at a loss for what I need to change to fix it.

\n\n

See below for images of the issue. On the orange piece near the right hand side you can see light shining through the gap. The black cube has it at the top, though it's not deep enough to let light through like with the orange one. There is also an image of the layer preview which shows the same gap.

\n\n

I have a Rostock Max v2 (stock hot end). I am using Matter control using mostly stock settings, I've tweaked around layer height, speed, and temp but I don't think those are the cause.

\n\n

.

\n\n

Images of the issue:

\n\n

\"enter\n\"enter\n\"enter

\n", "Title": "What setting do I need to change to get rid of this gap between the shells and the inside?", "Tags": "|software|slicing|mattercontrol|", "Answer": "

It's generally called infill overlap. \nIn terms of first layer it's good to set extrude rate to 120% or even more so infill will overlap perimeters and itself. There is also the issue of \"overlapping\" layers which is not managed by any specific parameter. it's the issue of layer thickness and HE temperature.

\n\n

Unfortunately Matter Control has kinda bug or at least an issue with extrude rate. If you set higher extrude rate then overlap will be automatically reduced in some way. So it's good to increase extrude rate manually during printing and then set it back to normal. this will cheat slicer.

\n" }, { "Id": "1128", "CreationDate": "2016-05-08T14:50:39.170", "Body": "

I have a 30ish year old travel mug from Sheetz that I intend to model and print using my OpenScad skills. The fact that it is an insulated mug means that the inside is hollow to an extent.

\n\n

What reccomendations do you have toward turning this into a foodsafe, leakproof success?

\n", "Title": "Travel Mug Woes", "Tags": "|food|", "Answer": "

You cannot make a safe coffee cup using home FDM printers. There are no printable thermoplastics available to consumers/hobbyists that will reliably AND safely contain coffee/tea temperature beverages.

\n\n

Even though some plastics may appear mechanically suitable at first sight, there are long-term issues with using thermoplastics for elevated temperature service. The material becomes weaker and less dimensionally stable under load as the temperature approaches the glass point of the polymer. For example, here is some Stratasys test data for ABS strength versus temperature:

\n\n

\"enter

\n\n

Even if the plastic survives a few uses, it may creep and warp over time. And that might be fine, depending on your needs! People have printed working coffee cups out of ABS before. For some definition of \"working\" you can make print a working coffee mug. It's just not going to work at the level of longevity and reliability that people typically expect from hot beverage containers. (Imagine if the handle breaks and pours scalding liquid in your lap.) PLA in particular tends to give out abruptly due to its unusual ability to creep to ultimate failure with very low elongations.

\n\n

Then there is the question of whether ANY FDM printed part is ever safe for repeated food service use. One-time uses (such as throw-away cookie cutters) might be perfectly fine, but repeated use introduces additional problems because FDM parts are porous. They contain a microscopic void/grain structure that is perfect for trapping food debris and sheltering bacteria. Every FDM part is similar in that regard to an old, scratched, wooden cutting board. They are difficult to effectively sanitize. A high-temp dishwasher cycle will do the job, but that imposes significant thermal stress on thermoplastics.

\n\n

Here's an example micrograph I took of voids in a printed part (strands all aligned the same way to highlight the shape):

\n\n

\"enter

\n\n

These voids will also allow liquids to leak unless careful print settings are used. Over-extruding with multiple perimeters, or doing some kind of post-print sealing can make water-tight parts. It's not hard, just takes some experimenting.

\n\n

There is also considerable anecdotal evidence that much of the cheap filament from Asia is full of nasty, toxic substances that may leach into hot beverages. Heavy metals like lead, strontrium, and cadmium are often found in thermoplastics as processing aids, pigments, or simply from unscrupulous companies disposing of industrial ashes by mixing them into dark-colored filaments for export. US-made and EU-made filaments seem to be better in this regard.

\n\n

Now that the basic issues are out of the way... Here is the breakdown of specific materials you might want to try:

\n\n\n\n

In summary, you can make a working coffee mug via FDM printing, but it isn't a great idea. Your best bet for 3d printing a coffee mug is ordering ceramic printed from an online service bureau.

\n" }, { "Id": "1135", "CreationDate": "2016-05-09T16:28:22.077", "Body": "

I need help opening a .max file a friend sent me. The thing is, I don't have Alias Maya. Well, a copy of Alias Maya that works anyway.

\n

Are there any other file openers that I can use? Or do I have to get Maya to open the file? Would Alias Maya open the file? I don't want to get download 1.3 GB and discover that there is 15 MB file opener.

\n

Is there a file converter maybe? That converts .max to, maybe, .obj?

\n", "Title": "How to open .max files", "Tags": "|3d-models|file-formats|123d-catch|", "Answer": "

You could try import it with blender. It's a free software which should be able to import the files.

\n\n

First thing is to go the the File->User Preferences->Add Ons tab. In the Import-Export section, enable the 3DS Add-on.

\n\n

After that you can export it to the desired format. Before exporting you must select the model/mesh you want to export. Otherwise nothing will be exported. The obj exporter is enabled by default in blender.

\n" }, { "Id": "1136", "CreationDate": "2016-05-09T17:46:43.240", "Body": "

So, have a plastic car part I want to duplicate because the driver side part is broken (I have the passenger side part) and It isn't sold anymore. It's a small piece that would be an excellent candidate for a 3D printed replacement. I know there are companies I can send a 3D model to that will happily print it for me, but I also need to create a 3D model from the part. Is there a company or service that I could mail the part to that would return it along with a 3D model of it?

\n", "Title": "Are there any services that offer 3D scanning?", "Tags": "|3d-design|3d-models|", "Answer": "

The right Google (or other) search should do the trick. I've provided 3D Printing services via 3D Hubs and MakeXYZ and some people do provide 3D scanning services. If you can't find 3D Scanning, you could try talking to a local machine shop. They might have the tools to be able to reverse-engineer the object, or know another place that can.

\n" }, { "Id": "1143", "CreationDate": "2016-05-10T15:52:03.190", "Body": "

I know there is a 3d printing technique, where sheet of papers are cut and glued. Like the ones Staples has in their stores. But this sort of paper isnt recyclable, because of the heavy use of glue.

\n\n

Is anywhere filament available, which is made of cellulose or other recyclable and bio-degradable material? Or do you know if research is going on in this area?

\n\n

edit: added bio-degradable

\n", "Title": "Print paper or cellulose - any recyclable filament?", "Tags": "|filament|recycling|", "Answer": "

Filament made of Polylactic acid (PLA) is usually made of biological materials (such as corn), and can therefore be considered bio-degradable in most cases.

\n\n

Whether the filament is 100% bio-degradable (and non-toxic for the surroundings) will depend on the specific formula used by each individual filament manufacturer. (Many manufacturers include various additives to achieve particular effects, such as glow-in-the-dark, metallic finish or extra strengh.)

\n\n

Woodfill PLA-like filament might be of extra interest to you, not only because it typically is bio-degradable, but also because it will give you the look and feel of being bio-degradable.

\n\n

Hope that helps!

\n\n

PS: there are multiple other filament types that are either recyclable, bio-degradable, or both, although PLA might be the most commonly available of them all.

\n" }, { "Id": "1144", "CreationDate": "2016-05-11T06:45:35.900", "Body": "

I have moved forward with the whole .max thing. It's on the back burner. But not as critical now. I have found a Sketchup model of the model I want. But it is ridiculously small (0.17\u00a0m long by 0.10\u00a0m wide!).

\n

How do I enlarge it to fit my dimensions as a whole?

\n", "Title": "How to enlarge a Sketchup Model", "Tags": "|3d-models|resolution|support-structures|", "Answer": "

Use Cura middle button in bottom\n\"enter

\n" }, { "Id": "1150", "CreationDate": "2016-05-11T20:46:53.420", "Body": "

I am trying to make a structured light 3D scanner using single camera, light projector and a turntable.

\n\n

After days on Google I did not find any reliable open source project which I can get to work. SLStudio really seemed a good choice but did not compile properly.

\n\n

I was wondering if anyone knows a good open source which they have used with the same kind of setup? Any help would be really appreciated. I am blocked here.

\n", "Title": "Open Source 3D scanning", "Tags": "|3d-design|open-source|scanning|", "Answer": "

You can use a Kinect sensor for Xbox 360 OR Kinect sensor for Xbox One. That sensor allows you to get a true 3D surface with its SDK. You can connect these devices directly to your PC using USB (I have one).

\n\n

Even the Xbox One model, in its SDK, has an example that allows you export your captured mesh as STL files.

\n" }, { "Id": "1151", "CreationDate": "2016-05-11T22:38:09.423", "Body": "

After printing successfully for a while I received an error saying:

\n\n

Tool 0 Failure!
\nTemp limit reached
\nShutdown or restart.

\n\n

The front panel is not responsive and doesn't allow the printer to print at all. If I quickly go to monitor mode it shows tool 0 temperature in the ~700\u00b0C range; which made me think it was the thermocouple.On the motherboard I unplugged the thermocouple and the same error occurred, I then swapped the other thermocouple and put it in its place and got the same error except the tool 1 temperature showed \"NC\".

\n\n

I had recently taken apart the extruders to change out the nozzles and thought I had possibly smashed the wires when putting it together but now I don't think that's the case.

\n\n

I'm on the phone with FlashForge's tech support but wanted to see if someone else has had the same experience for cross reference.

\n\n

What's the underlying issue? Is there a quick solution to this that can get me printing right away (I'm under a deadline)? What's the end solution?

\n", "Title": "FlashForge Creator Pro Tool Failure! Temp limit reached", "Tags": "|electronics|heat-management|", "Answer": "

Oh yeah my printer had that. Specifically 2 Flash Forge Creator Pros, Replace the thermal couple. Done. Really fragile thermal couples they use.

\n\n

Edit I see you tried shorting the TC. Wouldn't shorting the TC, produce the same error?

\n" }, { "Id": "1156", "CreationDate": "2016-05-12T15:44:06.400", "Body": "

I have a Flashforge Creator Dual.

\n\n

One corner of my print bed is warped down. I am thinking about having a steel print bed made so it would tend to stay flat.

\n\n

Has anyone tried this?

\n", "Title": "Would a steel, instead of an aluminium, plate be reasonable?", "Tags": "|heated-bed|", "Answer": "

Whether you should use steel or aluminum depends on the construction of your print bed stack. Either will work, but there are trade-offs involved.

\n\n

Various considerations that may come into play:

\n\n\n\n

So, it's really a holistic design decision. Aluminum is far more common because of its thermal properties and lower weight, but steel print beds (particularly with permanent adhesion coatings) are often used too.

\n" }, { "Id": "1160", "CreationDate": "2016-05-13T09:25:18.297", "Body": "

I use Prusa i3 with one extruder for some years and I would like to print from one material in two colors or from different materials for one model. Therefore I'm lookig for new printer with dual extruder.

\n\n

Is there some way how to measure and/or compare quality of printers with dual extruder on the market?

\n\n

For example to create 3d model - ask the seller(s) to print it - and then compare? - what details to focus on?

\n", "Title": "How to choose printer with dual extruder?", "Tags": "|dual-nozzle|", "Answer": "

\"Make\" Magazine compares 3-D printers in a way that is as scientific as possible to help determine the strengths/weaknesses of each 3-D printer compared to others. The link to the November 2015 comparison test is available here: http://makezine.com/comparison/3dprinters/ I was unable to find the ability to compare among dual-extruders, however.

\n" }, { "Id": "1161", "CreationDate": "2016-05-13T13:12:06.020", "Body": "

I recently started to use Simplify3D! It is great software, but I have a problem with adjustment of amount of plastic extruded. I have a slight under extrusion after I adjusted amount of plastic extruded using perimeter test (Printing a square with wall thickness of one layer, measuring, adjusting extrusion multiplier). I suggest that there is under extrusion by looking at top layer (there is distance between layers) and there are gaps around holes as well. I'm using Ultimaker two and colorFabb filament. I can resolve problems by increasing extrusion multiplier, but then I have incorrect layer width :/\nI have also attached picture. http://postimg.org/image/keghmu075/ On a right side, there is a part with correct extrusion multiplier (under extrusion). \nMaybe you guys have some ideas?

\n", "Title": "Extrusion adjustment", "Tags": "|software|slicing|extrusion|simplify3d|", "Answer": "

SHORT ANSWER

\n\n

You're not supposed to do the single-wall perimeter thickness test to calibrate Simplify3D. That screws up the extrusion volume. The correct volume calibration procedure for S3D is:

\n\n
    \n
  1. Measure actual average filament diameter and input that
  2. \n
  3. Print a 100% infill calibration cubes
  4. \n
  5. If the print is over-extruded (top or sides bulging), decrease Extrusion Multiplier by about 0.05 and try again. If the print is under-extruded or looks clean, increase Extrusion Multiplier by about 0.05 and try again.
  6. \n
  7. Repeat as needed to dial it in. The correct value of Extrusion Multiplier is the largest value that does not produce over-extruded prints. This produces minimum void volume and strong parts.
  8. \n
  9. In the future, that specific material+extruder combination will always have the same extrusion multiplier. You only need to measure and input filament diameter and you will always get accurate volume output. (And if you use high-quality filament with consistent diameter, you don't even really need to measure the diameter.)
  10. \n
\n\n

If you really want to measure perimeters, you can do 3 perimeters and divide the measurement by 3. That takes into account most of the overlap and will be much closer to accurate than a single-perimeter measurement.

\n\n

COMPLETE ANSWER

\n\n

The goal here is to fill the print volume as completely as possible (at least in the \"solid\" parts of the print like perimeters, roofs, and floors). Empty space between strands doesn't contribute to part strength. In fact, voids act as failure initiation sites by concentrating stresses. Because extruded strands come out with rounded edges, they have to be overlapped to squeeze plastic into gaps and minimize \"corner voids.\" That looks like this:

\n\n

\"enter

\n\n

Where the bulges overlap, the excess plastic gets pushed into the corners to mostly fill the voids. It's very difficult to get 100% packing density, but you can get pretty close if you calibrate volume correctly.

\n\n

\"enter

\n\n

You DO NOT want to space the strands without overlap. That makes very weak prints and looks like this:

\n\n

\"enter

\n\n

To get the right amount of strand overlap, the slicer has to do some math and make some assumptions about how you're calibrating it. This is not uniform between software packages. So it's important to understand what \"extrusion width\" means to different slicers. For a couple prominent examples:

\n\n\n\n

\"enter

\n\n

Both of these techniques can produce the exact same output if calibrated properly, but they require different calibration techniques because they calculate the plastic volume and adjacent strand spacing different ways.

\n\n

It's important to understand that S3D spaces its strands 1x extrusion width apart. When you use the measured perimeter thickness to calibrate Simplify3D for extrusion width setting = measured thickness, you get under-extruded prints with no strand overlap. S3D's strands must measure wider than the \"width\" setting to get the correct overlap.

\n\n

In practice, S3D's code is smart enough to know how this affects print dimenions, and will adjust perimeter positions to get accurate overall dimensions. But single-wall test boxes will be thicker than expected.

\n\n

The downside to the way Slic3r calculates volume is that it is only accurate for strands that are shaped like ovals. And that is only an accurate assumption when [extrusion width > nozzle width + layer height]. The strand must be wide enough for molten plastic to flow sideways and develop the circular bulge cross-section. So people almost always print with wider strands in Slic3r than is strictly necessary. The volume calculations don't work all that well with thin strands. Whereas S3D's volume calculation method works fine with pretty much any extrusion width greater than layer height and greater than nozzle diameter.

\n\n

There are pros and cons to both systems. You just need to understand the correct calibration technique for each.

\n" }, { "Id": "1164", "CreationDate": "2016-05-13T17:06:02.507", "Body": "

This is related to Cons to UV printing and focused on Ryan Carlyle's answer, mentioning:

\n\n
\n

\"...resin-curing SLA/DLP printers are industrial or commercial tools that are really not suitable for home desktop use.\"

\n
\n\n

So, my general question is what are the pros to using a DLP printer in an manufacturing environment?

\n\n

My experience with 3D printing in manufacturing has shown me the necessity of understanding material strengths as well as how to utilize the 3D printing technology (FDM mostly) to produce a structurally strong part. Most of what I've made has been fixtures with small to moderate forces applied to them.

\n\n

In short, are there any technological advantages to using a DLP printer in a manufacturing environment?

\n", "Title": "Pros to UV printing", "Tags": "|resin|sla|dlp|uv-printer|", "Answer": "

When using a DLP 3D printer, a projector (or other UV light source) is shining on a layer of resin. As the light source shines on a whole layer at a time, it is able to print at a rather constant linear vertical rate. This rate is normally around 2.5\u00a0cm (1\u00a0inch)/hour

\n

When comparing this to a standard FDM printer or a single laser system (such as the Pegasus Touch. The printer takes more time on more complicated layers. If a you were able to print the same 2.5\u00a0cm (1\u00a0inch), at 0.3\u00a0mm layer height, it would be a total of ~85 layers. This would allow for ~42 seconds per layer. Most layers would take longer from this (from personal experience).

\n

With the constant layer rate, multiple objects can also be printed at once because if the printer has to print the layer for the first object, it is not taking more time for it to cure resin in another location on the print bed.

\n

In addition to this, DLP printers can have much better resolutions than FDM printers. A DLP printer such as the Titan 1 has a 37\u00a0\u03bcm XY resolution while a Prusa i3 has a resolution closer to 300 or 400\u00a0\u03bcm.

\n" }, { "Id": "1165", "CreationDate": "2016-05-14T02:39:17.017", "Body": "

I have a 3D model that I'd like to print. This is my first project, so I'm trying to decide if I'm getting in over my head. Here's the model rendered with Blender:

\n\n

\"enter

\n\n

If I throw it at a commercial printing shoppe, is it going to be fairly straight forward? Or is it unlikely to be able to get the colours fitting together like this to work well?

\n", "Title": "Is this model feasible to print?", "Tags": "|3d-models|3d-design|services|", "Answer": "

The simplest and cheapest method to have this object printed would be to have individual STL files for each part and give these to the print shop with the color of each piece.

\n\n

Printing as a single piece would be much more expensive as more advance methods would be needed as mentioned in the other answers.

\n" }, { "Id": "1167", "CreationDate": "2016-05-14T03:31:20.547", "Body": "

I printed a big base for a model, but the corners of the bottom bent up, making the whole base rock when set on a table. Is there a quick fix for a makerbot2 without a heated plate?

\n", "Title": "How could I keep the material on the print plate from bending up on a makerbot 2?", "Tags": "|makerbot|print-quality|warping|", "Answer": "

Try printing with a raft, and adding \"helper discs\" to expand the size of the raft. You might also try lowering your extruder temperature a bit to reduce the cooling temperature differential.

\n" }, { "Id": "1180", "CreationDate": "2016-05-16T02:05:59.790", "Body": "

Do 3D printers prefer particular topology? In case I'm not using the word quite right, I'm specifically wondering if I should make the faces in my models:

\n\n
    \n
  1. triangles
  2. \n
  3. quads
  4. \n
  5. n-gons..
  6. \n
\n", "Title": "Do 3D Printers Prefer Particular Topology?", "Tags": "|3d-models|", "Answer": "

The most common file format in 3D printing is STL. This file format is using triangles only so when you export an object from your CAD application to STL then exporter has to transform all n-gon faces into triangles.

\n\n

Once the file is created then it is usually imported by slicer application (or module) which performs slicing using polynomial calculation to find intersection with next surfaces. Here is good example of such intersection finding in javascript.

\n\n

So answering your question:

\n\n\n" }, { "Id": "1187", "CreationDate": "2016-05-16T12:11:49.243", "Body": "

Will an inconsistent vertex density likely introduce artifacts in a print? Or is it OK as long as it's a smooth surface?

\n\n

\"enter

\n", "Title": "Is inconsistent vertex density a bad thing?", "Tags": "|3d-models|3d-design|slicing|", "Answer": "

For the most part, the exact level of vertex density doesn't matter too much. But it does depend on the slicer and settings. Some slicers (like Slic3r) will auto-decimate toolpaths to ensure that the rate of motion commands isn't too difficult for the old, slow 8bit processors in most consumer/hobbyist 3d printers. Having a large number of very small motion commands can bog down these motion controllers and cause pause-stuttering that creates little zits on the print. Most slicers simply reduce model detail level to safeguard the motion controller. Because the contours are decimated to a minimum motion segment length, very small model triangles are irrelevant to printer performance. At the most, they might add a marginal amount of slicing time.

\n\n

On the other hand, some slicers (like Simplify3d) assume you have the correct level of detail you want in your model, and will pretty faithfully reproduce the model file's contours in the sliced toolpath. If your entire model is very high mesh density / poly count in general, or if your high-density mesh regions are crossed by a layer slice, this can produce a series of very, very short motion commands. Sometimes the motion commands are even smaller than the motion resolution of the printer, and simply take up processor time (to evaluate and drop from the queue) with no benefit to print fidelity.

\n\n

In a more general sense, high-poly models are dramatically more difficult for the 3d printer to reproduce accurately. There are two big issues:

\n\n\n\n

These are primarily issues with high vertex density versus low vertex density, not variable density. Small pockets of high detail are usually not problematic, as long as they are small enough that the motion planner queue (say, 16 segments) doesn't get filled with too many very small movements. A few small segments in a row is ok, but a few dozen is not.

\n\n

These are limitations baked into the algorithms used by today's slicers and motion controllers. In the future, they may not be so problematic.

\n" }, { "Id": "1196", "CreationDate": "2016-05-18T06:49:31.870", "Body": "

I've built a a PrintrBot inspired printer with a moving printbed for the X-Axis movement. It's controlled by a RAMPS board running Marlin firmware. And there is a problem with the X-Axis positioning.\n\"Four

\n\n

As can be seen in the image the X-Axis is not really stable. The blocks should be 10x10x10 mm, and the top and bottom are perfect. The waving does not appear in the Y-direction, it's only in the direction of the bed movement.
\nEach block was printed at different speed, from medium to dead-slow. I limited the acceleration and in the last block I also limited the non-printing speed so the bed would never experience a higher acceleration than with the printing itself. But the wave effect remains. And it's also remarkably repeatable so it looks more like a synchronisation error somewhere. Does anyone has a clue, or experienced a similar effect ?

\n", "Title": "'Sine wave' deviation in X-Axis on moving bed printer.", "Tags": "|print-quality|marlin|", "Answer": "

This is the issue of your z-axis rather than x- or y-axis because printing speed doesn't have any influence on the results.

\n\n

I bet:

\n\n\n\n

All above suggestions concern z-axis.

\n\n

[edit] because it's PrintBot its z-axis has the following disadvantage

\n\n

\"enter

\n" }, { "Id": "1205", "CreationDate": "2016-05-20T14:36:50.827", "Body": "

I have a FlashForge CreatorX (MakerBot clone) that's been working fine for about 15 months. Int he past month, I started noticing \"thin\" layers in some of my ABS prints. I finally tracked the issue down to the extruder gear grinding the filament (after a while, enough filament had ground off that the gear teeth were filled with plastic). I cleaned the gear twice before giving up on ABS and switching to PLA. Everything seemed ok until the PLA started doing the same thing during an overnight print (thin layers on prints, audible skipping during filament feed).

\n\n

Raising the temperature on the extruder seems to fix the problem, but I'm now extruding PLA at 242\u00a0\u00b0C, much higher than I used to need. I worry that continued printing at this temperature will increase wear on the thermistor and increase my chances of clogging PLA in the gear (an issue I've had twice before).

\n\n

Are there any common issues that cause this issue of filament not feeding properly? I've tried cleaning out the extruder head with wire and there don't seem to be any clogs\u2026

\n", "Title": "Increased issues with filament grinding", "Tags": "|filament|", "Answer": "

After disassembling the extruder mechanism, I tracked the issue down to a clogged extruder\u2026 though not in the way I'd expected. A (previously?) common method to drop filament down to the hot end and create a heat-break between the heater and the extruder gear was the use of a PTFE (Teflon) sleeve. Over time, this sleeve can become clogged by filament that has melted and degraded/carbonized, sticking to the \"nonstick\" walls. This creates a bottleneck for the new filament, preventing smooth extrusion.

\n\n

\"Carbonized\n\"Location

\n\n

My printer came with two replacement tubes. Swapping those in for the old, clogged tubes resulted in instantly improved printing\u2026 sorta like getting a new printer! Print temperatures are back to normal and everything operates fine. I was also able to drill out the carbonized filament from the old sleeve, though I doubt I'll reuse it. I found replacement sleeves on Amazon here and an alternative, all-metal extruder assembly manufactured by Micro-Swiss. I'm not sure if you'd run into clogging issues on the all-metal one, so perhaps staying with the PTFE tubes and replacing them occasionally is a better way to go.

\n" }, { "Id": "1211", "CreationDate": "2016-05-21T21:44:29.903", "Body": "

What is the least expensive 3D printer available today? I am looking for something suitable for general use in a home office.

\n", "Title": "What is the least expensive 3D printer?", "Tags": "|desktop-printer|", "Answer": "

You can get an A8 3D printer on Gearbest at 149$, It's a version of an i3, with easy assemble and pre configured, and it have a suprising good quality.

\n\n

I thinks is the best price/quality that you can find at the moment. It prints with a good quality by default and you can upgrade it if you need it without spending hundreds of dollars.

\n\n

http://www.gearbest.com/3d-printers-3d-printer-kits/pp_337314.html

\n" }, { "Id": "1234", "CreationDate": "2016-05-29T10:44:52.647", "Body": "

I am attempting to construct model tank tracks with accompanying wheels and sprockets. All parts will be printed in PLA. The tracks will be driven by electric motors.

\n\n

What would be a suitable grease for this project to minimize friction without damaging the plastics.

\n", "Title": "Grease for PLA sprocket", "Tags": "|pla|3d-models|", "Answer": "

I have been using sewing machine oil for my 3D printed extruder gear for a couple of years, without any noticeable wear so far.

\n\n

In general, I believe any kind of low viscosity oil would do, but then again, it also depends on the environment in which you plan to use the model. If you plan on using it outside, for instance, you probably should get some grease that does not attract dirt, as the dirt itself will grind down the gears quickly.

\n" }, { "Id": "1242", "CreationDate": "2016-05-31T16:52:13.810", "Body": "

Should I consider health impact of ABS or PLA when printing cookie cutter similar to image below?

\n\n

How safe it is to use ABS or PLA for kitchenware in general?

\n\n

\"enter

\n", "Title": "Should I consider health impact of ABS or PLA when printing cookie cutter?", "Tags": "|pla|abs|health|", "Answer": "

If it's one-time-use, both ABS and PLA are perfectly safe for use as a cookie cutter.

\n\n

The \"food safety\" of 3D printed parts is fairly controversial. In fact, whether any particular material is approved by regulators (such as the US FDA) for food contact is much more complex than most people realize. Materials can be accepted for some uses and not others. But there are two major considerations:

\n\n
    \n
  1. Does the material leach into the food, or does the food leach into the material?
  2. \n
  3. Can the material be adequately cleaned and sterilized between repeated food exposures?
  4. \n
\n\n

Whether leaching is a concern will depend on the kind of contact. For example, nylon will absorb acids but is often used for potable water service. ABS is fine for cold foods but not hot foods. PLA (injection grade) is often used for disposable plastic forks and cold beverage cups, but rarely reusable containers.

\n\n

There is some anecdotal evidence that lots of 3D printing filaments, particularly cheap Asian filaments, contain toxic chemicals in the pigments and additives. Heavy metals like lead, cadmium, strontium, and all sorts of nasty organic chemicals have been found. You probably WON'T get any meaningful leaching of these chemicals in the brief exposure with cookie dough, but to be safe I would only use reputable US and EU filament manufacturers for food contact. I would also stick to filament colors that use non-toxic pigments (eg plain white is almost always safe titanium dioxide) or no pigments at all (\"natural\" color).

\n\n

When it comes to cleaning a cookie cutter for multiple uses, ABS is probably a much better choice than PLA, because ABS can withstand fairly hot water without losing shape. PLA probably won't survive a cycle through the dishwasher. PETG is perhaps a better choice than ABS for longer contact with wet foods, but may or may not survive the dishwasher.

\n\n

However, the porous structure of 3D printed parts is a serious problem for cleanliness. The tiny grain structure and voids between extruded strands can act as a safe haven for bacteria between uses. This is particularly an issue with cookie dough containing raw egg. A porous surface for raw-food handling would not be acceptable in a commercial kitchen.

\n\n

Now, the odds of actually transmitting salmonella via poorly-cleaned cookie cutter may be fairly low. Lots of people cut meat at home on wooden cutting boards (which are also porous) and never get sick. Properly cooking the food immediately after handling will go a long way towards reducing the odds of harmful cross-contamination. You'll have to decide for yourself whether the risk profile is worth it. One-time use is the conservative choice.

\n" }, { "Id": "1244", "CreationDate": "2016-05-31T20:03:01.093", "Body": "

I've been having a hard time lately getting the raft off of my ABS prints.

\n\n

Is that a symptom of either a nozzle or bed that are too hot? Or is there some other factor I should be looking in to?

\n\n

I have an UP mini that I've modified both the nozzle and bed to customize the temperatures on.

\n\n

Bed gets heated to 100\u02daC and nozzle is either 266\u02daC for UP ABS filament or 236\u02daC for off-brand ABS filament.

\n", "Title": "Raft hard to remove?", "Tags": "|abs|fdm|rafts|fff|", "Answer": "

I haven't got a printer with a heated bed so have only tested this on PLA but I have found editing the G code so the printer cools the nozzle down and then heats it up again gives the raft enough time to cool down so that it peels off easier when the print is finished.

\n" }, { "Id": "1245", "CreationDate": "2016-06-01T00:30:14.640", "Body": "

Would I damage anything running 12\u00a0V on a 24\u00a0V cartridge heater? I know it wont reach a max temperature 300\u00a0\u00b0C? I am using the E3D hotend set up.

\n", "Title": "Running 12 V on a 24 V heater cartridge?", "Tags": "|electronics|heat|", "Answer": "

Applying 12\u00a0V to a 24\u00a0V heater cartridge won't damage anything, but you may have severe issues reaching and maintaining your target temp. A standard E3D heater cartridge is 40\u00a0W. When you run a 24\u00a0V cartridge on 12\u00a0V, you only get 10\u00a0W of heater power. Here are some rough estimates on where your hot block heat goes:

\n\n

I've never heard of anyone using less than about 16\u00a0W to print. (Two 8\u00a0W power resistors.) 20\u00a0W is the lowest vaguely normal hot end power. I think you'll struggle to print.

\n" }, { "Id": "1251", "CreationDate": "2016-06-02T15:05:32.857", "Body": "

From what I've been able to find out, online sources recommend around 205\u00baC for PLA and around 240\u00baC for ABS. But these are only guidelines, of course. Optimal printing temperature can be different depending on the printer, the filament, the model and other slicer settings.

\n\n

For example, I've had success printing black PLA at 190\u00baC, but silver PLA of the same brand is giving me trouble. I'm having a hard time figuring out the general rules. So I would like to see a general guide for this, based on (at least) the following questions:

\n\n
    \n
  1. Which known factors before a print can help determine the right extrusion temperature? Obvious example: ABS vs PLA

  2. \n
  3. What can happen during or after a print when the temperature is too low?

  4. \n
  5. What can happen during or after a print when the temperature is too high?

  6. \n
\n\n

An answer to the first question could take the form of a lookup table, or similar. The second and third could help someone adjust their temperature based on the symptoms of a failed print.

\n\n

I understand that the failure or success of a print can depend on many more factors than extrusion temperature, but I didn't want to make this question too general. I may later ask the same question for other settings (e.g., print speed). However, do let me know if this question should be expanded or improved to make it more useful.

\n", "Title": "How to choose an extrusion temperature?", "Tags": "|print-quality|extruder|hotend|knowledgebase|", "Answer": "

Printing temperature basics

\n

Manufacturers generally specify a somewhat wide range of printing temperatures, and what temperature you should actually need can only be determined by trial and error:

\n
    \n
  1. The thermistor in your hotend is not 100\u00a0% accurate and may have an offset of a few degrees compared to its actual temperature.

    \n
  2. \n
  3. Your hotend has a small temperature gradient, the place where the plastic is melted may have a higher/lower temperature compared to the temperature of your thermistor.

    \n
  4. \n
\n

2 is further exacerbated by

\n
    \n
  1. As you print faster, you need more heat. The cold filament rapidly moving through your hotend will cool it down locally, meaning that the temperature will be cooler than what the thermistor measures. Faster prints equal bumps in the temperature up to 10\u00a0\u00b0C, and for a really slow print you might turn it down 10\u00a0\u00b0C from where you normally are.

    \n
  2. \n
  3. This is a minor issue, but different colors of the same brand and material might work better at different temperatures. The pigments used can affect the melting point somewhat. Different brands also might have different temperatures.

    \n
  4. \n
\n

Some symptoms may give you a guide as to how to adjust your temperature:

\n

Printing too hot

\n\n

Printing too cool

\n\n

Furthermore, hot prints can sometimes have a more glossy finish than colder prints.

\n" }, { "Id": "1258", "CreationDate": "2016-06-03T17:56:37.480", "Body": "

I've been working on my own DIY 3-D printer recently, and I've been trying to experiment a little with different materials for the body. Someone suggested using HDPE (high-density polyethylene), since it works well on a CNC machine, which would make creating a number of prototypes easy. I know HDPE can also be used for filament, but I've never tried it before. Does anybody have any input on HDPE, or other potential materials for the body? I'm trying to avoid using wood, as I've had some poor experiences with it.

\n", "Title": "HDPE as the body of a 3-D Printer?", "Tags": "|diy-3d-printer|printer-building|hdpe|", "Answer": "

I've tried this material (8 mm thickness) for a Prusa i3 clone of my own design but needed to abandon using it as it doesn't allow to be cut by laser easily on my friends laser cutting machine (not a hobby laser cutting machine, it is his business).

\n\n

HDPE requires lower speed than cutting Plexiglas or Acrylic resulting in more heat input and a small top cut and a large bottom cut width leaving the edges far from being straight.

\n" }, { "Id": "1262", "CreationDate": "2016-06-05T14:24:14.600", "Body": "

For a large scale project, I need a slicer which gives me the slices as image (BMP or vector). \nI have a 3d model and want to assemble it manually using large printouts on cardboard. For this I need my 3d model somehow sliced into 2mm layers, get a silhouette of the outline for every layer and print it in cardboard. Later all cardboard layer will be cutted manually and glued in the right order.

\n\n

Does somebody know a slicer, which gives me as output images for every slice?\nOr any different idea how to geht my large cardboard 3d model?

\n", "Title": "Slicer that exports vectors or pics for every slice", "Tags": "|3d-models|slicing|", "Answer": "

Your approach is one of many and you may find that Slic3r will do as you require.

\n\n

There's a direct reference to generating SVG files (vector) at this link:\nhttp://manual.slic3r.org/advanced/svg-output

\n\n

You may have to create a printer profile based on irrational figures when compared to conventional 3d printers. Your 2mm \"layer height\" should be matched in the printer settings to an appropriate nozzle diameter for the desired result.

\n\n

You may find also that Simplify3d will create such files, but it's not a free program.

\n\n

An alternative to 3d printer slicing software would be to use OpenSCAD to import your model (or create it directly in code) and use the Projection command to generate the individual layers. I've done this and found it works well, but the process to learning the necessary code was time consuming, as I'm not a programmer.

\n\n

You would also want to incorporate some form of registration in your model, cylindrical holes or external tabs to enable you to properly align each cardboard layer.

\n\n

If you have a makerspace nearby or know someone with a suitably sized laser cutter, you may also save some labor cutting each panel, as the laser will accept the vector files easily.

\n" }, { "Id": "1266", "CreationDate": "2016-06-06T04:59:03.687", "Body": "

I am planning on getting a 3D printer soon and I was just wondering, what do you do with 3D prints that either failed or were prototypes that you no longer want?
\nI tried looking online but the closest I got was effects on environment, turning prints back into filament, or restarting a failed print half-way, none of which were the information I was looking for.

\n\n

The solution should be somewhat eco and just keeping unwanted prints in a box somewhere isn't a very good solution either.
\nFinishing off failed prints isn't totally applicable because it wouldn't apply to prototype prints that you don't want to keep.

\n\n

Is there anywhere to send failed 3D prints for professional recycling, or are there any recommendations for properly disposing 3D prints?

\n\n

In case this is of any use, the printer I am planning to get uses filament that can be made of PLA, ABS, Nylon, and possibly other materials. The printer is this one specifically (https://www.kickstarter.com/projects/101hero/101hero-the-world-first-us49-3d-printer) from KickStarter.

\n", "Title": "What to do with failed/unwanted 3D prints?", "Tags": "|filament|pla|abs|recycling|", "Answer": "

If you have a few friends you could pitch in and use a recycling service like Terracycle where you can send your failed prints, rafts and scraps. No affiliation with Terracycle, it's just a service that I found and am considering.

\n" }, { "Id": "1271", "CreationDate": "2016-06-06T13:33:48.803", "Body": "

I have an object that I want to print in 3D. But I have a few questions about it. What are the things that I have to watch out for when 3D printing?

\n\n

I know how to change the metric size etc. Some people said that it's best to set the thickness to a low amount and not make the object solid (to leave the inside empty/hollow) in order to save money when printing.

\n\n

Is this true or does it not matter?

\n\n

Also what if I want two parts of an object to be separate colours or materials? Do I have to change this in Blender?

\n\n

Any advice and information would be helpful, thanks.

\n\n

\"enter

\n\n

\"enter

\n", "Title": "3D printing with blender", "Tags": "|3d-models|3d-design|software|blender|", "Answer": "
\n

What are the things that i have to watch out for when 3d printing?

\n
\n\n

non manifold geometry : geometry that can not exist in the real world.

\n\n

It's a good idea to check if the dimensions of your mesh are correct before exporting:

\n\n

\"enter\n


\nTurning on mesh analysis allows visual inspection of problems such as intersecting faces, sharp edges ,edges with thickness below a threshold and other criterias.
\nthe inspection tool will color the faces with those problems.

\n\n

\"enter\n


\nAnd lastly you can make selections by traits, such as loose geometry, interior faces or unconnected vertices.

\n\n

\"enter

\n\n

note that those are just tools to help you find the problems , none of them will fix the mesh for 3d printing.

\n" }, { "Id": "1275", "CreationDate": "2016-06-07T15:06:01.907", "Body": "

I just recently upgraded my Printrbot Simple Metal with a heated bed (and longer x-axis). I looked up some tutorials, and all of them placed the thermistor as in the picture on the left below, so I did too. However, the design of the heat plate seems to strongly suggest thermistor placement as in the picture on the right, inside the small hole near the center.

\n\n

\"enter \"enter

\n\n

I can certainly see the upside of that. The reported temperature may overshoot the average bed temperature (the reasoning used in the tutorials I read), but most of my prints are built in the center of the bed, and the center placement would surely make the temperature control system more responsive.

\n\n

What are the pros and cons of these placements? And what would be the proper technique for putting the sensor in the center? Should I still use kapton tape? Do I need to make sure the thermistor makes physical contact with the aluminum of the print bed?

\n", "Title": "Heated bed thermistor placement", "Tags": "|heated-bed|thermistor|", "Answer": "

It doesn't make a great deal of difference where you place the thermistor; do whatever is most convenient for you. The temperature of the heated bed is not very critical, and some inaccuracy in the measurement is perfectly fine. The point is to keep the print from cooling down too fast, and whether the bed is at 95C or 93C makes little difference.

\n\n

The pros and cons of thermistor placement are exactly as you describe: in either case the center of the bed will be hotter than the edges. Placing the thermistor in the center the temperature of the center will equal the setpoint, while the edges are cooler than the setpoint. Placing the thermistor at the edge the edge will be at the setpoint while the center will be hotter than the setpoint. Placing the sensor at the center you would need to use a slightly lower setpoint to get the same result compared to having the sensor at the edge.

\n\n

It is possible the holes in the PCB are meant for if the PCB is used with a different printer. In any case, soldering the thermistor to the pads and placing it in the hole is acceptable, kapton tape wouldn't be required then (but can't hurt either). It would be best if the thermistor made good contact with the heated bed surface, using (non-conductive!) thermal paste would be convenient for this (but not strictly required).

\n" }, { "Id": "1277", "CreationDate": "2016-06-07T20:00:05.497", "Body": "

It occurs to me that I'm never really thinking about layer height when I calibrate my z-probe offset. This is obviously an oversight, because I'll want my nozzle to start higher for 0.3mm layers than for 0.1mm layers.

\n\n

After this realization comes the question: Do I need to recalibrate every time I switch to a different layer height? Or can slicers store a reference layer height + probe offset and just deduce the proper starting height for every print? Or is it smartest to keep the 1st layer height constant throughout my prints to spare myself this frustration?

\n\n

If it matters, my printer is the Printrbot Simple Metal, and my goto slicer is Cura.

\n", "Title": "Do I need to recalibrate z-probe offset every time I switch to a different layer height?", "Tags": "|calibration|z-axis|z-probe|layer-height|", "Answer": "

All modern slicers adjust the nozzle position for the first layer in accordance with your chosen layer height. You can see this in your gcode if you slice files with different layer heights. Before you add special slicer settings and offsets, if you print 0.1mm layers, the nozzle will start at Z=0.1mm, and if you print 0.3mm layers, the nozzle will start at Z=0.3mm.

\n\n

There are two reasons this is more complex and less reliable than it seems:

\n\n

Different slicers assume different initial tramming gaps. And your actual tramming gap may not match that assumption. If the slicer thinks your nozzle is leveled at Z=0 with a real physical gap of 0.1mm to start with, that means gcode Z=0.1mm is actually a 0.2mm gap that must be filled with plastic. So the slicer must compensate by starting lower than the nominal layer height.

\n\n

So what works perfectly for one slicer won't necessarily work correctly for another slicer. And if you tram with a thinner object than the slicer expects (say a post-it note instead of business card) then your first layer will be off. This is why I personally prefer physical build plate leveling mid-print using screws while watching the strands go down. It bypasses all the assumptions about tramming gaps and just gives you the correct result. (Or you can do the same thing with babystepping in firmware that supports that.)

\n\n

The other issue is that people use lots of weird, ad hoc slicer tricks to get their first layer to stick. Things like printing the first layer much hotter, or at half speed, or squashed way down and over-extruded, or at 60% layer height, or at 200% extrusion width, will all affect the extrusion volume calibration and the space-filling behavior of the molten plastic flowing onto the bed. The slicer doesn't really have the ability to understand \"your\" first-layer adhesion recipe.

\n\n

The combination of incorrect/unknown tramming gap and person-specific first-layer settings is why the slicer can't always get the first layer height and extrusion volume correct across all layer heights. Within some fairly reasonable assumptions, the slicer is smart enough to always correctly relate extruder flow and nozzle position so it fills the space between the nozzle and whatever surface you're printing onto. But if you break those assumptions, it may perform differently for different settings.

\n" }, { "Id": "1281", "CreationDate": "2016-06-07T23:22:16.030", "Body": "

First, a little background. A couple of years ago, I was researching making my own candy, and I came across this page: Lego brick shaped gummy candies, describing how to use real Lego bricks as a positive to make silicone molds for Lego brick shaped candy. Now that I have a 3D printer, and inspired by the usual description of ABS filament (\"It's the same plastic used to make Lego bricks\") it occurs to me that I can now make any positive I want.

\n\n

The question is, would that be safe? I know the filament I'm using is not food safe, but if I create positives for a food safe silicone mold, would toxins leech into the mold? And if so, is there a barrier I can use to prevent this, such as some kind of coating?

\n", "Title": "Creating positives for a food-safe silicone mold: safe?", "Tags": "|safety|food|", "Answer": "

In principle, ABS is safe for contact with (cold or room-temperature) food. The two main concerns specific to 3D printing are, assuming you start with a filament that is not itself contaminated:

\n\n
    \n
  1. Pores and holes in the printed part which may harbor bacteria

  2. \n
  3. Impurities introduced into the plastic during the printing process

  4. \n
\n\n

I doubt that the silicone mold will capture the pores and holes with sufficient detail to be of any concern (it certainly won't capture the internal structure, only the surface).

\n\n

That leaves us with 2. It has been noted that brass nozzles contain trace amounts of lead. This lead can contaminate the printed part, which may in turn contaminate your mold, which may in turn contaminate your food. I don't think this is of realistic concern, since we're looking at trace amounts of trace amounts of lead. The nozzle might also have burnt plastic stuck to it (which might be carcinogenic) so you should make sure to do the print with a very clean nozzle and at a temperature that is not too high.

\n\n

ABS is food safe for contact with cold or room-temperature food. It is however not food safe for contact with hot food, because at higher temperatures the food may leach certain chemicals out of the plastic. Your application is one of low temperature, but silicone is not food and might perhaps leach some contaminants out of the plastic, regardless of temperature. However, this concern is not specific to 3D printing, as it applies to the method of making moulds out of Lego bricks as well. Therefore, making moulds from 3D printed positives does not appear to be different in a food safety perspective from making them out of LEGO blocks.

\n" }, { "Id": "1286", "CreationDate": "2016-06-09T06:56:11.857", "Body": "

I need to transport my FDM 3D Printer because I am moving.

\n\n

What are the precautions that one should take?
Should I dismount the motors and axes?
I would definitively unplug the electronics as far as reasonable and fix the motors to the frame so they don't slide during transport.
Should I have a housing to avoid dust and other mechanical issues?

\n", "Title": "How to transport a 3D printer? Dismount needed?", "Tags": "|safety|mechanics|", "Answer": "

You just need to take basic security actions. like fixing all movable parts simple as that

\n" }, { "Id": "1292", "CreationDate": "2016-06-10T04:09:11.657", "Body": "

I have a home built RepRap with all sides open..

\n\n

Would there be any advantage to enclosing the print area in acrylic?

\n", "Title": "Should I enclose my 3D Printer?", "Tags": "|reprap|cooling|knowledgebase|enclosure|", "Answer": "

Consider the environmental conditions where your printer is.

\n

I have my printer in a garage, where the door is sometimes open, closed, or slightly ajar. This leads to gusts of wind, causing lifting and ruining multiple prints.

\n

I surrounded the printer on three sides with walls made from the original packing foam, and my lifting problems have utterly vanished.

\n

The top and front side are open, so I have a "four sided" box. Heat does not build up, but the variable wind gusts are minimised. Access is slightly reduced, but not a lot.

\n

Ender 3 V2, printing the "Ender" brand of PLA from Creality. Bed is at 55\u00a0\u00b0C and print head is 200\u00a0\u00b0C.

\n" }, { "Id": "1309", "CreationDate": "2016-06-10T18:35:07.770", "Body": "

I've been trying to build CuraEngine on Ubuntu following the Github instructions.

\n\n

The problem is that it lists some requirements:

\n\n\n\n

I'm not sure where should I install Protobuf and libArcus. After several tries and locations I've received several errors during the build process.

\n\n

Does anyone have a more detailed guide on how to build this engine?

\n", "Title": "How to build CuraEngine?", "Tags": "|software|slicing|ultimaker-cura|", "Answer": "

I'm assuming you actually want to build Cura, rather than simply install Cura. If you instead want to install Cura, you can try sudo apt-get install cura-engine

\n\n

The following instructions were tested on my own Debian 8 (Jessie) distribution; they should be mostly, if not entirely, the same, for Ubuntu. Note that I did not follow the exact steps as described on the github/Ultimaker/CuraEngine README.

\n\n

Before we begin, let's make a build directory and do everything in there.

\n\n
mkdir ~/Downloads/curabuild\ncd ~/Downloads/curabuild\n
\n\n

The instructions will be broken into

\n\n

1. Install dependencies

\n\n

Some or all of these may already be installed on your computer. To be sure, we install them anyways:

\n\n
sudo apt-get install git curl libtool dh-autoreconf cmake python3-setuptools python3-dev python3-sip sip-dev\n
\n\n

2. Install protobuf

\n\n
    \n
  1. Clone and enter the protobuf git repository:

    \n\n
    git clone https://github.com/google/protobuf\ncd protobuf\n
  2. \n
  3. Build and install for C++:

    \n\n
    ./autogen.sh\n./configure\nmake # this will take some time\nsudo make install\n
  4. \n
  5. Install for Python 3:

    \n\n
    cd python\nsudo python3 setup.py install\n
  6. \n
\n\n

3. Install libArcus

\n\n
    \n
  1. Clone and enter the libArcus repository:

    \n\n
    cd ../..\ngit clone https://github.com/Ultimaker/libArcus\ncd libArcus\n
  2. \n
  3. Build and install

    \n\n
    cmake .\nmake\nsudo make install\n
  4. \n
\n\n

4. Install CuraEngine

\n\n
    \n
  1. Clone and enter repository:

    \n\n
    cd ..\ngit clone https://github.com/Ultimaker/CuraEngine\ncd CuraEngine\n
  2. \n
  3. Build and install

    \n\n
    cmake .\nmake # grab a cup of coffee\nsudo make install\n
  4. \n
\n\n

5. Celebrate!

\n\n

If all went well, you're done! You can now use the Cura engine via CuraEngine. Enjoy.

\n" }, { "Id": "1310", "CreationDate": "2016-06-11T12:19:06.190", "Body": "

Background:

\n\n

I am using the Flux Delta 3d Printer, and it arrived with a glue stick. Now on their forums it said to apply 2 layers of glue and let it dry, there's supposed to be a link there on how long it should dry. However that link is broken.

\n\n

My question:

\n\n

On a not heated bed, how much glue-stick should I apply and for how long should I let it dry?

\n", "Title": "Not Heated Build Plate Glue Amount and Dry Duration", "Tags": "|desktop-printer|build-plate|", "Answer": "

As a fellow Flux Delta owner, I can answer this question with certainty. Two layers is sufficient. Be sure to consider the size of the model being printed and the additional area covered if you are using skirts or rafts.

\n\n

If you apply the glue stick at the moment you turn on the printer, it will be dry enough to work properly. The printer \"uses time\" to initialize and to calibrate which provides enough drying time. You will also \"use time\" to load the software and import the model.

\n\n

It is not necessary to remove the glue from the plate after a print. For succeeding model printing, ensure that you've filled in the areas missing glue from removing the previous model.

\n\n

You will want to remove the glue (hot water works good, but the build plate gets hot!) when you feel that the glue build-up changes the texture of the bottom layers.

\n\n

You may have already learned from the forums that you print on the non-grid side of the build plate.

\n" }, { "Id": "1312", "CreationDate": "2016-06-11T15:31:38.163", "Body": "

I am printing on a non-heated bed right now, but the question also applies to heated building plates.

\n\n

How often should you replace the glue layers that's supposed to be applied before printing? Some say you can do up to a few prints, such as in this forum, while others say to replace it every print. What is the correct approach?

\n", "Title": "How often should I replace the glue layers on the bed?", "Tags": "|desktop-printer|heated-bed|build-plate|", "Answer": "

Depends on the glue and on your tolerance for messy undersides on your prints. It's fairly common for some of the glue to come off with the print. Or you may have marks from scrapers or rafts. Do you want to touch up that spot and have some artifacts on the bottom of the next print, or clean and redo the bed to get everything flat?

\n\n

Gluestick is pretty easy to wash and reapply. It can also be freshened up with a gentle spray of water, smeared flat, and redried, or more gluestick added on top. It will really come down to your preferred workflow.

\n" }, { "Id": "1316", "CreationDate": "2016-06-12T01:30:25.640", "Body": "

I use OctoPrint on an Ubuntu system with a M3D printer.

\n\n

Midway through a recent print, the filament just stopped extruding although the motor-functions of the printer were proceeding fine. Since then, every print I attempt has trouble extruding proper amounts of filament. It's always not enough. The output is stringy and not cohesive.

\n\n

I'm thinking there may simply be a clog in the extruder and wondering the safest way to remove it.

\n\n

The weird thing, though, is that when I use manual control and extrude at, say, 220C, the filament comes out fine.

\n\n

You can see the raft definitely isn't printing right. Way too little output:

\n\n

\"enter

\n", "Title": "M3D filament not extruding well (anymore)", "Tags": "|filament|extruder|octoprint|", "Answer": "

You might have a case of clogged nozzle. You can check this easily by lifting the Z axis and running the extrusion motor. If it's grinding on filament or you notice extruded plastic is curling or going out slow, it's probably partial or total clog.

\n\n

I have had the latter variant happen to myself recently, with the same symptoms as yours. Fixing it involved disassembly of the hotend and cleaning the clog manually using jet lighter (to heat up the nozzle) and guitar string to pull out clogged PLA.

\n\n

You might have some other options available:

\n\n
    \n
  1. Try pushing the filament through hotend by hand, it might clear the clog.
  2. \n
  3. Other variant I've heard of is heating up the extruder to 130-150\u00b0C (PLA) and just yanking out the filament, and hopefully clog along with it. I wouldn't recommend this if you don't know what you're doing, as it might damage the printer.
  4. \n
\n\n

In my case things that were causing the clog were the big retraction distance and temperature being set too high.

\n" }, { "Id": "1334", "CreationDate": "2016-06-13T18:08:39.310", "Body": "

I have this GoPro mount for a quadcopter as STL file. It looks as follows.

\n\n

\"enter

\n\n

How do I modify it so that it is wider and longer by a few millimeters but the screw holes stay the exact same size? Additionally the angle of the upper surface must stay the same. Please suggest the easiest solution for someone like me who as no idea about CAD software.

\n\n

The perfect solution for me would be to just import it in Tinkercad and then modify it however I have no idea what the steps are.

\n", "Title": "What is the easiest way to modify/resize an object inside a STL file?", "Tags": "|3d-design|software|3d-models|", "Answer": "

I like to use Slic3r for simplest slicing and rearranging operations.

\n" }, { "Id": "1355", "CreationDate": "2016-06-16T16:00:37.283", "Body": "

Is it possible to print toothbrush bristles using a common FDM 3D printer? I am particularly interested in the width of bristles, closeness together of each bristle, and the flexibility of each particular bristle.

\n", "Title": "Can toothbrush bristles be printed?", "Tags": "|filament|3d-models|fdm|", "Answer": "

I have had a go at doing something for a christmass tree using a drop loop technique. You could use the same method or somthing similar to try and create something that looks like toothbrush bristles, but I don't think you would want to try cleaning your teeth with it.

\n" }, { "Id": "1357", "CreationDate": "2016-06-16T19:58:12.030", "Body": "

Since iBox Nano is the smallest public-production-available 3d Resin printer (and the cheapest so far), I assume it has a huge size limitation. So far I've only seen pictures of its outputs that are miniature things. I've never tried it nor have I seen it in action in person so I'd like to be sure.

\n\n

For example, my 3D models are of the size of beads to figurines to a standard sized pencil cup holder.

\n\n

I want to know in inches or millimeters the dimension (width, length, height) of the biggest possible object the iBox Nano can print.

\n", "Title": "What is the biggest size of an object that iBox Nano can print?", "Tags": "|ibox-nano|", "Answer": "

On their website, I found the following picture, which states a build area of 40 mm x 20 mm x 90 mm (1.57\" x 0,79\" x 3.54\").

\n\n

\"iBox

\n" }, { "Id": "1359", "CreationDate": "2016-06-16T20:03:39.133", "Body": "

How to check leveling of printer bed ? \nWhat should be ideal values on x,y and z to ensure printer bed is flat and there is no angle between plane of nozzle and plane of printer bed ?

\n", "Title": "CubePro 3D Printer", "Tags": "|calibration|", "Answer": "

The CubePro printer appears to have a comprehensive leveling system built into the menu. The direct instructions for this process can be found here:\nCubePro Calibration

\n\n

I would suggest that \"ideal values\" are those created by the calibration process that confirms the necessary settings for a good print.

\n" }, { "Id": "1361", "CreationDate": "2016-06-17T00:00:34.130", "Body": "

What are the differences, and pros & cons, between 3D printers with varying layouts for moving head vs. moving build plate?

\n\n

Example layouts would include:

\n\n\n\n

In particular, what are their respective strengths, weaknesses, specializations, maintenance considerations, etc.?

\n", "Title": "Differences between moving build plate vs. extruder", "Tags": "|print-quality|mechanics|build-plate|", "Answer": "

I think this is simple, breaking it into simple parts... It is much easier to calibrate a machine where each axis does one thing. For instance, the Ultimaker; one of the few XY-on-the-same-axis printers... It has such a crazy complicated pulley system. If one thing gets out of whack, everything will start binding. Thankfully this doesn't happen as they did a good job, but it comes at a price. All the double pulleys and double rods - that costs money.

\n\n

However if you break it into smaller bits then less will go wrong.

\n\n

When I upgraded my MendelMax 1.5 to use a PBC linear solid slide for the X axis, it instantly solved all my issues with the rods binding (two rods into one big slide).

\n\n

To quote my coworker,

\n\n
\n

It's like balancing a stool. Is it easier to balance a 4 legged stool or a 3?

\n
\n\n

Following from that, there is probably a good hour's worth of mechanical engineering information that can be said in regards to the linear motion itself. I would rather calibrate one axis at at time, and not worry about weird behavior caused by a XYZ head.

\n" }, { "Id": "1371", "CreationDate": "2016-06-17T19:28:37.910", "Body": "

I am using Slic3r to generate the GCode for my Marlin-based printer. For some reason with increasing height my print starts to get messed up. On another part it starts to act like this when there are small parts. Is this related to my Slic3r settings, maybe to much filament being extruded or is this due to something else?

\n\n

Any help is highly appreciated and I can provide more pictures of messed up parts or slic3r config if necessary.

\n\n

\"The

\n\n

\"Another

\n", "Title": "Why is my print getting messed up mid-print?", "Tags": "|filament|pla|print-quality|slic3r|marlin|", "Answer": "

This can happen if your filament is having to be pulled off the spool and slipping in the extruder. Check if slippage is happening at the extruder, perhaps see if results change by providing some hand powered help. I have seen this with some glossy PLA I have used.

\n" }, { "Id": "1373", "CreationDate": "2016-06-17T21:47:42.837", "Body": "

I'm using a CraftBot original to print PLA, but some of the filament has become stuck in the teeth of the extruder gear on its way into the hot end. I'm having issues with the gear becoming stuck and \"clicking\" instead of turning, and I suspect it is because of the clogged teeth. I haven't changed materials in a long time. I've cleared several print head clogs, but each time the gear gets back around to the one spot, it seems to get stuck again.

\n\n

Are there any good ways to clean this gear? I was thinking of putting it in a toaster oven and trying to melt the PLA off it, ideally without setting my house on fire in the process. Anyone have better ideas?

\n", "Title": "Good methods to clean extruder gear (hobbed) from filament pieces?", "Tags": "|filament|extruder|drive-gear|", "Answer": "

If your gear skips at the same place each time I have found on my machine that the gear does not fit the motor shaft properly and has a larger gap between the gear and the idler roller.

\n\n

Check to see if there is a thicker buildup on one side of the gear than the other.

\n\n

On my machine what I had to do was to use a spare motor cable and watch the gear as it turned, and at one point on the gear it would slip and shave off a small amount of filament.

\n\n

If that turns out to be your problem use a good caliper to measure your motor shaft, and the inner diameter of the gear.

\n\n

Sometimes the minimum shaft diameter and the maximum bore diameter of the gear are just enough to allow that small variance in the outer rotation diameter of the gear.

\n\n

Sometimes just replacing the gear will solve the problem, and sometimes it will require a new motor and/or gear.

\n\n

As an alternative you can clean the gear using Acetone and a toothbrush, but that will be required often and you will have buildup in the chamber below the gear which can also cause feed problems, and you must make sure that all Acetone has dried before using the extruder again.

\n\n

I suggest starting with changing gears since they are the cheapest and using the Caliper to find the closet fit possible.

\n\n

You can get gears on e-bay at very reasonable prices. and I bought several to get the best fit possible.

\n\n

Good luck and consider if you want to make a more permanent repair or temporary repair when making your choice. I chose the more permanent one because the slippage was affecting some of my prints.

\n" }, { "Id": "1379", "CreationDate": "2016-06-18T04:11:16.817", "Body": "

I noticed 3Dsystems has Multi Jet 3D printing where wax is used provide support and give users with high resolution 3D printed objects. I was wondering if there were cheaper and smaller Multi Jet 3D printing?

\n\n

Can Form 1+ from FormLabs be considered to do the same job with resin?

\n", "Title": "What are some cheaper versions of Multi Jet 3D printing?", "Tags": "|desktop-printer|", "Answer": "

Objet by Stratasys is a comparable technology (they call it PolyJet). SolidScape is also somewhat comparable (they also use jetting).

\n\n

The FormLabs printers are not comparable. They use a completely different process, which can only print in one material. With jetting, you can mix different materials in the same part.

\n" }, { "Id": "1383", "CreationDate": "2016-06-18T09:54:45.630", "Body": "

Is there any simple way of creating tappered thread in OpenSCAD? I need something like 10 mm in diameter at the end, 9 at the top and the height of 10 mm.

\n", "Title": "How to create tappered thread in OpenSCAD?", "Tags": "|3d-design|", "Answer": "

I have contacted Dan Kirshner (the author of openscad threads library) and he has updated the library. Now it supports tapered threads. Thanks, Dan!

\n\n

Thread-drawing modules for OpenSCAD

\n" }, { "Id": "1389", "CreationDate": "2016-06-19T18:21:47.690", "Body": "

I have an option to purchase a hobby multi-purpose device (lathe/mill/drill/grinder/cutter) which is manually controlled, but easily converted for driving by stepper motors (all 3 axis are controlled by turning knobs that can be replaced by gears, with convenient mount to couple each to a stepper motor). I have the right motors and can easily obtain drivers for them.

\n\n

\"pic

\n\n

That is the way to overcome the worst problem of converting Prusa to CNC: the flimsy mechanics not able to withstand stress and vibrations of machining. Then I can connect the drivers to the 3 axis of Prusa's electronics, optionally connect some driver of the spindle to the extruder output (or just control it manually), and it seems the hardware side of the device is done.

\n\n

The problem is the rest - adapting the software. RepRap family of 3D printers being open source means their software and hardware can be adapted. It's only a matter of how hard it is.

\n\n

Does anyone have any experience in that direction? What would such conversion involve? Just recalibration to the new gear/leadscrew ratios, or something more involved, like editing the sources to get rid of all the temperature safeguards and the likes?

\n", "Title": "How much work would be converting Prusa firmware/software for CNC use?", "Tags": "|electronics|mechanics|", "Answer": "

None, if I choose the right control board.

\n\n

The Smoothieboard supports CNC \"out of the box\"; it requires initial configuration, which, while somewhat different, is actually easier than for a 3D printer. Boards supporting Grbl or Teacup will be compatible for XYZ too, but may require some tinkering if you want to control the spindle.

\n\n

You can use the same CAD software, but the set of CAM tools will need to be considerably different.

\n" }, { "Id": "1395", "CreationDate": "2016-06-20T20:13:11.347", "Body": "

I am trying to print model for Prosthetic Hand \n(File Here: http://www.thingiverse.com/thing:596966) using Tinkerine 3D Printer. It went smooth for first hour and after that it started messing up(attached image).\nI checked the leveling of plate and it was fine. Also, the nozzle didn't get clogged anywhere. Can anyone suggest, what could be the reason for this or what should be inspected in order to overcome this?

\n\n

Expected Result\n\"enter

\n\n

Actual Result\n\"enter

\n", "Title": "Tinkerine 3D Printer", "Tags": "|print-quality|slicing|g-code|", "Answer": "

The issue occurring here is similar to a question asked over here. It looks as though this is a result of a hardware fault related to the stepper drivers. @DarthPixel recommended placing heatsinks on the stepper driver(s) and/or properly placed fans.

\n\n

\"A\"A

\n\n

Depending on the design of your machine, some stepper drivers are going to be worked harder than others. For example, a MakerBot Replicator will work the X-Axis stepper driver more than the Z-Axis stepper driver. Therefore, you have a higher chance of overheating your X-Axis stepper driver.(Images are of A4988 Stepper Driver, not directly compatible with MakerBot Replicator)

\n\n

Alternatively, I personally have seen the issue repeated by manually triggering one of the limit switches. The effects seemed to take effect on the next layer.

\n" }, { "Id": "1399", "CreationDate": "2016-06-21T19:54:41.557", "Body": "

I've been printing small quantities from a PLA filament spool on a Craftbot printer for about two months now. Recently the printed objects have been coming out very brittle. Some structures that printed fine two months ago are now difficult to re-print. The print head gets clogged easily, and when the object does print, it's quite brittle and 1/4\" to 1/8\" rods will easily snap off if not handled gently.

\n\n

I'll admit to not following the precautions for storage of PLA. It's much easier to just leave the filament installed rather than trying to remove it after each print, so this one spool has just been sitting on the back of the printer for all these weeks now. I'm sure it's been humid some of the days, we've had some rain here.

\n\n

Has the spool of PLA been damaged just by leaving it exposed to room air for two months? Could that be the sole cause of the brittle prints, or are there other possible causes? Is there any way to fix the spool or future prints from this spool, or do I have to scrap it and get a new spool?

\n", "Title": "Any fix for PLA prints getting more brittle over time?", "Tags": "|filament|pla|print-quality|filament-quality|", "Answer": "

Increase nozzle temperature. When the filament is new it will print easier, requiring less heat to print well. So if you didn't store your filament properly to begin with, increasing print temperature will make it jam less and increase layer bonding.

\n\n

The reason for this is because the moisture that accumulates in the filament will absorb heat and evaporate when printed, meaning that the filament itself isn't getting the same amount of heating as it used to.

\n\n

That being said, the storage suggestions mentioned by tbm should be your first priority. I personally put my filament in Zip Lock plastic bags and store these in a dry location not exposed to sun or temperature changes.

\n" }, { "Id": "1403", "CreationDate": "2016-06-22T10:39:23.163", "Body": "

So I have a self build Mendel Reprap style 3d printer.

\n\n

I've not used it in sometime after moving house but I'm looking to use it again. What should I pay attention to before calibrating and running it again?

\n", "Title": "What should I pay attention to after taking a 3d printer from extended storage", "Tags": "|maintenance|", "Answer": "

After storage you can check the following things before taking the machine into operation again:

\n\n\n\n

Happy calibrating after that ;)

\n\n

*edit:\nAs Tom already pointe out, most of these things are unlikely, due to the usually rigid construction of the printer. These are merely more or less likely possibilities of things that can go bad or worse in storage.\n*/edit

\n" }, { "Id": "1404", "CreationDate": "2016-06-22T12:35:46.923", "Body": "

Inspired by another question and due to the fact that some of my filament will face the same problem when I will use them again, I wanted to know if there are proven recipies to get rid of water that has ben incorporated to PLA filament from humidity? One knows the filament had too much exposure to humidity when hearing tiny puffs during extrusion and/or more brittle prints.

\n\n

I know about suggestions to heat the water out of the filament at a temperature well below the glass transition temperature, but can someone provide first hand knowledge or even evidence?

\n", "Title": "Can aged PLA be refurbished?", "Tags": "|filament|pla|repair|", "Answer": "

I have turned unusable PLA into filament that printed well by putting it in a dry oven for a couple of hours. I've used 90 degrees C (194 degrees F), which may be too hot as it distorts round filament to be flat sided as the filament softens and relieves stresses. It printed well. Perhaps 70 degrees C (158 degrees F) would less deform the filament.

\n\n

Today I have a large plastic bin with rolls of filament and three color-changing rechargeable drying cartridges. As soon as they look other than absolutely dry, I plug them in to recharge. It may be insufficient for PETG and Nylon -- I don't yet have much experience.

\n" }, { "Id": "1410", "CreationDate": "2016-06-23T04:24:21.237", "Body": "

I have a 3D Printer with 0.4 mm (400 micron) nozzle length and the printer guide suggests that it can print up to a layer thickness of 60 micron and has a accuracy up to 100 micron.

\n\n

What I'm trying to understand is that, how does the 0.4 mm nozzle output a filament size lesser than it? How does that process work? Does it reduce the speed at which the filament flows or something?

\n", "Title": "How does the sizes of nozzle diameter and the Z-axis layer resolution work?", "Tags": "|layer-height|", "Answer": "

Yes, they do reduce the speed. Basically, the speed at which you feed filament determines how large of a volume comes out, and the distance you move the extruder means this volume gets distributed over this distance. However, this is not the whole story.

\n\n

The size of your nozzle determines only two things: the maximum layer height and the minimum feature size. It does not affect minimum layer height (and obviously doesn't affect maximum feature size) although there are some practical constraints (you wouldn't want to print 0.1 mm layers with a 5 mm nozzle).

\n\n

The nozzle squishes the filament down as it is extruded; the filament is basically forced into the gap between the nozzle and the previous layer. There's no limit to how thin of a layer you can print (the 60 micron figure is just something the marketing department made up, you can likely print even thinner layers). So long as you can position your Z-axis with sufficient accuracy (which is usually no problem) you can print arbitrarily thin layers.

\n\n

On the other hand, the nozzle size does determine the maximum layer height. Your need the filament to be squished a bit, or else it won't stick to the previous layer properly. You can't print layers thicker than your nozzle size, and it is generally recommended to print with a layer size that is at most 80 % of your nozzle size (e.g. with a 0.4 mm nozzle you shouldn't attempt to print layers thicker than 0.32 mm) but this is just a guideline.

\n\n

Your extrusion width is the width of the line deposited by your nozzle. This is generally (due to the mentioned squishing) a bit larger than your nozzle size. With a 0.4 mm nozzle you should set your extrusion width to something slightly larger, like 0.5 mm. It is technically possible to print with an extrusion width that is the same as (or even smaller) than your nozzle size, but this results in very weak prints: as mentioned before, you want the plastic to get squished slightly.

\n\n

They claim an accuracy up to 100 micron. This doesn't mean that you can print features as small as 100 micron (since you can't, due to the extrusion width being much larger). What this means, is for example, that if you print a 10 mm cube, you should expect its real size to be between 9.9 mm and 10.1 mm. Such a cube does not have any features that are smaller than your minimum feature size, but its outer walls can be positioned with greater precision than this minimum feature size.

\n\n

I should caution you that the 100 micron figure is \"up to\" and in practice you will have a hard time achieving this.

\n" }, { "Id": "1442", "CreationDate": "2016-06-28T17:06:18.597", "Body": "

I created a new project in Materialise Magics, added a few parts (different STLs), moved them around in certain positions and now I want to export this project into another STL, containing my recent work.

\n\n

The export menu seems to be all grey, like this function is not available. Do I have to do some repairing first or something similar? Or Magics needs other software in order to export a Magics Project to a STL file?

\n", "Title": "How can I export a Materialise Magics Project as a STL file?", "Tags": "|3d-models|file-formats|materialise-magics|", "Answer": "

I gather from your post that you are trying to export a build file containing multiple individual pieces, possibly to be printed. I suggest using the 'Merge' tool to join each of the individual STL's into one file. You can then save it as as single part or export it to a printer. This information comes from my experience as an Additive manufacturing technician. The following link is a Github repo containing a workflow on this process:

\n\n

Magic's Build File Creation

\n" }, { "Id": "1460", "CreationDate": "2016-06-30T16:38:23.227", "Body": "

I homebuilt a delta 3D printer (like Kossel mini) with a Z probe near the hotend with manual deploy and RAMPS 1.4 board:

\n\n\n\n

Then I tested it inside Repetier-Host and all seems to work well (homing, moving, extruding) except for the Autobed leveling method.

\n\n

Specifically, it always starts measurement of 3 Points (with 3 probe repetition each) even if I change the type of measurement.\nIt never does a grid measurement or a 2 points mirror measurement.

\n\n

I also tried to re-upload the firmware with EEPROM_MODE to 0 but didn't work.

\n\n

Does anyone have the same issue, or can explain to me why this happens?

\n", "Title": "Bed leveling method not working with Repetier firmware 0.92.9?", "Tags": "|delta|repetier|kossel|z-probe|repetier-host|", "Answer": "

As a guess, you are using the wrong command. If set for n x n grid, you get 3 point measurement with G29, but that is not auto leveling, it is just setting Z height - based on average height at 3 points.

\n\n

What you need is G32 S2 with S2 to store result in EEPROM (and therefore you should have EEPROM enabled with auto leveling. It is also used to store endstop offsets).

\n" }, { "Id": "1465", "CreationDate": "2016-07-01T10:11:45.450", "Body": "

Before the question, here is my setup;

\n\n\n\n

My main task is to convert my 3D printer into a chocolate printer.\nI have replaced the filament extruder with a chocolate extruder. And it is there that my issue began. Because, the new extruder is stopping slightly off the y-axis when homing. It is going out of the standard 20x20 cm bed. The other X and Z axes are OK.

\n\n

So, I have played with the #defines explained below, but I couldn't even make any single mm difference by homing. They are all ignored when the printer is homing. It goes and rests on the hardware end-stops and stops there eventually.

\n\n

All I want 10 mm offset for Y axis.

\n\n

Started with this;

\n\n
// Travel limits after homing\n#define X_MAX_POS 200\n#define X_MIN_POS 0\n#define Y_MAX_POS 190  <<<< (tested with 190 and 210)\n#define Y_MIN_POS 0\n#define Z_MAX_POS 200\n#define Z_MIN_POS 0\n
\n\n

and this;

\n\n
// The position of the homing switches\n#define MANUAL_HOME_POSITIONS  // If defined, MANUAL_*_HOME_POS below will be used\n\n//Manual homing switch locations:\n// For deltabots this means top and center of the cartesian print volume.\n#define MANUAL_X_HOME_POS 0\n#define MANUAL_Y_HOME_POS 10 <<< (tested with 10 or -10)\n#define MANUAL_Z_HOME_POS 0\n
\n\n

I have also played with the slicer tool (Repetier) settings where homing related values are mentioned but no joy there as well.

\n\n

Any input highly appreciated.

\n", "Title": "How to set a new homing position using software and/or slicer without changing hardware end-stop?", "Tags": "|firmware|chocolate|homing|", "Answer": "

Now I've finally had time to look into this, since I knew it somehow existed, but wasn't sure how it worked:

\n\n

Use the M206 G-code command in Marlin, Sprinter, Smoothie, or RepRap Firmware to offset the 0,0,0 coordinate of your printbed relative to the endstops.

\n\n

The reprap.org wiki page says:

\n\n
\n

The values specified are added to the endstop position when the axes\n are referenced. The same can be achieved with a G92 right after homing\n (G28, G161).

\n \n

With Marlin firmware, this value can be saved to EEPROM using the M500\n command.

\n \n

A similar command is G10, aligning these two is subject to discussion.

\n \n

With Marlin 1.0.0 RC2 a negative value for z lifts(!) your printhead.

\n
\n\n

We see, this basically is the same suggested by @LarsPoetter, but it comes with the great advantage that it can be saved to EEPROM, hence you don't need to add it every time or into every different sliccer (if I understand it correctly, - I haven't yet tried it myself)

\n\n

Let us know if this works for a permanent solution.

\n" }, { "Id": "1468", "CreationDate": "2016-07-02T07:41:12.473", "Body": "

Is a 100 micron layer thickness object stronger than 300 micron layer thickness 3D printed object? Are there any rules to follow?

\n\n

Filament type - PLA

\n", "Title": "Does the layer thickness have any effect of the strength of the 3D printed object?", "Tags": "|pla|print-strength|", "Answer": "

3D Matter has published an excellent article on the subject. They find that thicker layers result in a stronger part, with 0.3mm layers giving a part that is around 24% stronger than the same part printed with 0.1mm layers.

\n\n

One small issue with this study is that it did not look at the effects of temperature. Raising the temperature generally results in stronger parts because the layers will fuse better. It is possible that you could make a 0.1mm part just as strong as a 0.3mm part by raising the printing temperature.

\n\n

Another consideration for inter-layer bonding is how much the next layer is \"squished\" onto the previous one. Using a wider extrusion width will improve strength.

\n\n

The main issue with the strength of FDM parts is that they tend to break much easier along the layers, much like how wood is much stronger across the grain. You have to take this into account when making your design, and ensure that features that will be subjected to stress/strain are printed in the XY-plane.

\n" }, { "Id": "1474", "CreationDate": "2016-07-04T03:51:53.017", "Body": "

I get nothing but black bars and \"Unknown USB Device\" from Windows 10. This is after a failed flash that was otherwise going directly according to plan.

\n\n

This Davinci has been nothing but frustration for me, and I'm really tired of fruitless Google-Fu.

\n\n

Much of what I see pretty much says it's bricked; attempting to jump the jp1 jumper any further does me no good.

\n\n

Is there anything I can do to actually move forward, or are we in soldering-iron territory now?

\n", "Title": "Trying to flash Davinci XYZ 1.0, nothing but black bars", "Tags": "|repetier|", "Answer": "

I had this occur on my MakerBot Replicator Dual after I tried repairing a blown voltage regulator. If you refer to the previous link to my SE question, pay attention to the comments of Ryan's answer where it is explained that (in my case at least) the main processor was damaged and therefore couldn't properly load the firmware. The machine was able to load just enough to initialize the chip on the LCD screen, but since the main boards processor was blown, couldn't load the firmware to send commands to the screen and continue the boot sequence.

\n\n

I'm not certain that you're encountering the same EXACT issue, but you have all of the same side effects that I did. Basically, if you're not pro with electronics, your board is bricked. Otherwise, you might be able to salvage the board by replacing the processor, but there's no guarantee that something else isn't wrong or that the new processor was installed correctly.

\n" }, { "Id": "1477", "CreationDate": "2016-07-04T21:53:18.823", "Body": "

I would like to obtain a 3D model of my insoles. I tried to scan it with 123D catch but finding reference points is difficult because the insole is black.

\n\n

How could I improve the scanning? I attach a photo of the insole to show its not easy shape.

\n\n

I thought about placing a grid of white spots on the surface, but will it be enough?

\n\n

Also, is it recommended to keep the camera still and rotate the object (I have a rotating stand) or should I move the camera? in the second case, the accuracy of the photos will be lower, while rotating 5-10 degrees the stand every time is extremely easy.

\n\n

As background should I use a checkerboard or something else not repetitive?

\n\n

I noticed that the insole has basically three different \"levels\", joined smoothly with each other. These \"planes\" are sloped the same way, so that if I change the pitch of the insole by about 10 degrees, they will be horizontal. Would it improve the accuracy?

\n\n

Edit: the insole is perfectly new, newer worn before.

\n\n

\"enter

\n\n

Update

\n\n

I dotted the insole with white paint (the white eraser paint used in office and school) and I took a series of photos with the insole in top of a tripod, laying on a flat white cardboard support. 55 photos in total at 3 different elevations (side, medium, high). I also did a test with fixed camera and rotating object.

\n\n

Photoscan did not work well in general. Keeping the object fixed produces the best results, but poor.

\n\n

123D Catch did a much better job! again with fixed object and moving camera. Still, treating the resulting mesh was difficult and the accuracy could be improved.

\n\n

Autodesk Remake 2017 worked VERY well, as you can see in the attached screenshot, and it allows the editing of the mesh to remove useless parts of the model. I haven't tried yet to process the photos with the camera fixed and the rotating object. I did the processing locally (it's slow!) with maximum details and resolution.

\n\n

\"enter

\n", "Title": "How to optimally make a 3D model of an object using photos?", "Tags": "|scanning|", "Answer": "

I don't have enough reputation to comment.

\n\n

This is a short pointer to another webpage: https://www.sculpteo.com/blog/2016/01/20/turning-a-picture-into-a-3d-model/
\nIt may be helpful.

\n" }, { "Id": "1478", "CreationDate": "2016-07-04T22:48:21.577", "Body": "

I have Sainsmart Mechanical End Stops, and I'm building a Prusa i3 Rework.

\n\n

I've recently gotten the x-axis to move, but it needs to stop when it gets to the end and it does not do that; I end up having to pull the plug.

\n\n

From what I understand, there are two ways to wire ends stops in general, but you have to change the settings in the firmware to make it work right.

\n\n

There are 3 connectors on the board:

\n\n\n\n

My Sainsmart Mechanical End Stops have 4 connectors!!!

\n\n

According to their website, left to right with the white connector facing you, they are as follows:

\n\n

\"enter

\n\n
    \n
  1. S
  2. \n
  3. Normally Closed (NC)
  4. \n
  5. Normally Open (NO)
  6. \n
  7. VCC
  8. \n
\n\n

Now it seems to me that if I take those connections and connect them as follows it should work...(ramps on the left, sainsmart end stop on the right):

\n\n
    \n
  1. S (signal) -> S
  2. \n
  3. - (ground) -> Normally Open (NO)
  4. \n
  5. + (VCC) -> VCC
  6. \n
  7. nothing -> Normally Closed (NC)
  8. \n
\n\n

...but! Screwing this up by either using the wrong firmware of selecting the wrong wire frys your board, so I thought I'd ask here. Sources please.

\n", "Title": "Connecting Sain Smart Mechanical End Stops to Ramps 1.4?", "Tags": "|ramps-1.4|prusa-i3-rework|marlin|firmware|endstop|", "Answer": "

The multiple connections are redundant. Your picture indicates the wires labelled as \"SIGNAL, GND, GND, VCC\". This is correct if the board is a \"standard\" mechanical endstop v1.2. The two middle pins are both ground, but you only need to use one of them.

\n\n

You can simply connect signal to signal and ground to ground. That is sufficient for the endstop to work. You only need to connect VVC to VVC if you want the LED that's on the board to function.

\n" }, { "Id": "1480", "CreationDate": "2016-07-04T23:50:08.490", "Body": "

I've been looking into this, but:

\n\n
    \n
  1. I'm not certain how to configure my multimeter;
  2. \n
  3. I don't know how to keep the voltage going, and;
  4. \n
  5. I don't know how to keep the multimeter connected to the VMOT?
  6. \n
\n\n

I'm told you're supposed to aim for about 1 A.

\n", "Title": "How do you make sure you have the right voltage on the trimpots on an A4988 stepper driver?", "Tags": "|prusa-i3-rework|multi-meter|stepper-driver|", "Answer": "

I want to add some points and clarifications to the answer that @darthpixel already has given. Most information you need is in there, I want to give some more practical advice, since that is what I understand you're question is asking for. I'll start with some points on the more theoretical side, though:

\n\n

Now to the practical side and the application of darthpixel's answer and the above:

\n

You want to measure the reference voltage to limit the current that produces the torque, but also heats up the motor - let darthpixel's advice be your guide: if you can't touch it because it is too hot, then there is too much current, i.e. Vref is too high). To do so:

\n
    \n
  1. Set your multimeter to volts, range can be autorange or something bigger than 2V.
  2. \n
  3. Connect one lead of your multimeter to the ground of your Prusa i3 controller board's power input (I use the screw that fastens the ground input of the RAMPS). The other lead goes directly to the center of the trimpot on the Stepstick. I took the best of my paint skills to create an image showing the process: \"measure
  4. \n
  5. Note the value you read (12V power has to be on)
  6. \n
  7. Use an insulated screwdriver and turn the trimpot slightly.
  8. \n
  9. Get a new reading by repeating the measurement process.
  10. \n
  11. Repeat the whole process until you get the desired Vref.
  12. \n
\n

Warning: While I've had no problems turning the trimpot while everything was switched on (with my DRV8825 drivers), you should switch the power supply off when doing so.

\n

The described process allows only for a stepwise and rather slow setting of the Vref, but this is the easiest way I've found. I have read of people that use a crocodile clamp to attach the multimeter to the screwdriver for a readout while turning.

\n

If you don't have any idea whether you need more or less current on the motor at the moment, check your resistors on the board and calculate the Vref you should need (see darthpixel's answer for the formula). I would however just do what darthpixel already suggested: figure out the trimpot position by ears and touch: klicking motor: go to higher Vref. Can't touch the motor for more than some seconds: go to lower Vref. It might be a lengthy process, but in the end you'd need to do it anyway to get the best out of the printer!

\n" }, { "Id": "1503", "CreationDate": "2016-07-12T08:08:41.043", "Body": "

At the moment, I am thinking about print fans that cool the plastic when printing. I am not asking about the design of the fanducts, which might be a whole book on its own. I would like to know how to find out the best application of print cooling for a given PLA filament, - that is fan speeds and setup in a slicer of your choice (to learn what the different options are).

\n", "Title": "How to utilize/calibrate print fans for PLA?", "Tags": "|print-quality|heat-management|print-fan|", "Answer": "

I use PLA and both my fans stopped working. I am not techy and couldn't figure out how to fix them so I set up a house fan that blew directly at the motor keeping it cool and it works as good if not better than the fans that came with the 3d printer. You cannot let your motors over heat because your filament will get curled up insize the nozzle.So try that before buying 3d printer fans.

\n" }, { "Id": "1505", "CreationDate": "2016-07-12T10:31:05.890", "Body": "

I am using a BAUDrate of 115200 since I cannot make a connection to my printer with the advised 250000 rate. Are there any downsides or limits I reach earlier given by the lower BAUDrate?

\n", "Title": "What is the influence of a lower BAUDrate?", "Tags": "|electronics|", "Answer": "

If a GCODE line is about 30 characters (which is about the norm), you can send at most 420 of them per second at 115200 bps.

\n

If each GCODE prints 0.15 mm (for example in arcs), you would be limited to 63 mm/s in arcs/circles (straight lines are not an issue).

\n

If that's limiting for you, then you should find a way to set 250 kbps.

\n

You don't mention the firmware you use, but if you use Klipper, which compresses data over the serial line, connection speed is not an issue.

\n" }, { "Id": "1511", "CreationDate": "2016-07-15T19:49:40.867", "Body": "

I tried printing from an STL file on the QIDI TECHNOLOGY 3DP-QDA16-01 Dual Extruder Desktop 3D Printer QIDI TECH I. They recommend using the MakerBot software with the Replicator(Dual) profile. The 3D print that I tried is messed up.

\n\n

The material used is PLA.

\n\n

Could you help me determine the cause? The following pictures give more details:\n(All settings which are not shown are Makerware default settings)

\n\n

1 The 3D model

\n\n

\"The

\n\n

2 The printed sample

\n\n

\"The

\n\n

3 The Settings

\n\n

\"Settings\"

\n\n

\"Settings\"

\n", "Title": "Irregular 3D printed part", "Tags": "|pla|print-quality|makerbot|makerware|", "Answer": "

I agree with RyanCarlyle on this:

\n\n
\n

Your print isn't cooling fast enough.

\n
\n\n

Please regard this answer of mine that covers a few options to help with the issue you're encountering. Pay particular attention to the layer times, and active cooling.

\n\n

The essential take-away from my answer (linked above) is this:

\n\n\n" }, { "Id": "1519", "CreationDate": "2016-07-17T20:26:44.100", "Body": "

My first and only 3D printer is a Printrbot Simple Metal, which has a hotend that doesn't expose any of its internal parts. Easy for beginners, I suppose: \"The hotend is that tube that heats up the plastic and deposits it on the print bed.\"

\n\n

\"Printrbot \"More

\n\n

But I've been trying to learn more, and many hotends out there don't look quite as simple. My Printrbot hotend probably isn't as simple as it looks, either.

\n\n

What are the parts that make up a hotend, and what do they do?

\n\n

(PS: This is a general question, not specifically about the two example hotends above.)

\n", "Title": "What are the parts that make up a hotend, and what do they do?", "Tags": "|hotend|", "Answer": "

This varies by hotend design. The following is a list of components which you might find in a typical hotend, but note that different designs may integrate these components to some extent. For instance, on the J-head the heat block, nozzle and heatbreak are all one and the same component whereas on the E3D hotends these are all separate parts.

\n\n\n\n

The ubis hotend you mentioned is a bit simpler than this, and simply uses a big chunk of PEEK in place of the heat break/sink. PEEK has very low thermal conductivity and thus passive cooling is sufficient. However, PEEK limits the temperatures at which you can print.

\n\n

Here is an illustration outlining these components on an E3D V6 hotend:

\n\n

\"enter

\n\n

Note that in this image the Teflon liner only goes into the heat sink, and not into the heat break or block. This means the maximum temperature is not limited by the Teflon, but if it did go all the way in (as is the case with, for instance, the Lite6) then it would be.

\n" }, { "Id": "1520", "CreationDate": "2016-07-18T02:43:22.077", "Body": "

I have a Monoprice Maker Select, which is a rebranded Wanhao Duplicator I3 V2 (Prusa clone). I've found that the heated bed temperature values on the LCD are incorrect. The heater works, and the controller maintains the bed temp just fine, but the temperature reported isn't the true temperature. When using the PLA preset, with the bed set for 60\u00b0C, once the temperature stabilizes at 60\u00b0C on the display, I can measure it with a laser thermometer and get a consistent 54\u00b0C across the buildplate. With the ABS preset of 90\u00b0C, I read 80\u00b0C. And with the bed set for the maximum value of 120\u00b0C, I'm only reading 102-104\u00b0C.

\n\n

I've checked these values with the bare aluminum build plate, and I've allowed the temps to stabilize for at least 10 minutes to ensure that I have consistent readings. I believe the the firmware is using the wrong temperature curve for the thermistor in my device.

\n\n

I had hoped that there was a simple scaling constant that I could adjust, but that doesn't seem to be the case. I've researched enough to learn that my device is running Repetier 0.91 firmware, and this page on temperature control on the Repetier wiki says that each type of thermistor needs a custom voltage->temp lookup table, and talks about building a custom table in \"configuration.h\". I've tried to follow this line of research but I feel like I'm going around in circles. It sounds like I could build a custom version of the firmware to install on my printer, and in doing so I could potentially specify a better conversion table. This gives me pause for several reasons:

\n\n
    \n
  1. I haven't found any online reference to other people installing a custom Repetier build on a Wanhao I3.
  2. \n
  3. I have no idea what values to use for the temp conversion table.
  4. \n
  5. I can't even find anyone else reporting my issue.
  6. \n
  7. I'm fairly new to this; I rather not brick my printer, and I haven't found any good guides to installing firmware on the device either.
  8. \n
  9. I'm not convinced this is the right option. Should I be looking at replacing the thermistor instead? And if so, what's the correct part?
  10. \n
\n\n

I've also looked into the Melzi board inside the printer hoping for a variable resistor to tweak the thermistor voltage divider, but no such luck, at least in the schematics.

\n\n

I could just live with it; I've been using a 67\u00b0C setting to achieve a true 60\u00b0C bed temp for PLA, but I'm starting to work with ABS and I'd like the option to get above 100\u00b0C bed temp. Am I on the right track? Suggestions? I'm still pretty new to the device and 3D printing in general, so I may have overlooked something obvious.

\n\n

Update: additional question in light of Tom's answer: what is the expected max actual bed temp achievable on a Wanhao/Monoprice I3? I'm measuring 100\u00b0C with an LCD reading of 120\u00b0C, but I'd like to get to 110\u00b0C if possible.

\n", "Title": "Adjusting Bed Temp values on Monoprice/Wanhao I3 (Repetier Firmware)", "Tags": "|prusa-i3|heated-bed|repetier|", "Answer": "

It is completely normal for the surface of the bed to be cooler than the indicated temperature. The thermistor goes on the underside of the bed, near the heating traces. The top of the bed (which is further away from the heating traces) will naturally be cooler.

\n\n

It would be possible, though a lot of work, to build a custom thermistor table that more accurately reflects the surface of the bed, though that means your thermistor table will no longer reflect the characteristics of your thermistor, but instead reflect the particular circumstances under which you build your custom table (which would be affected by ambient temperature, any drafts, ...).

\n\n

The accuracy of an infrared laser thermometer depends on the characteristics of the surface you're measuring for, so unless you adjusted the thermometer specifically to measure the aluminium surface of the bed, it's possible your thermometer is off as well.

\n\n

The exact temperature of your bed doesn't matter anyways (you just need it approximately in the right ballpark) so I would suggest to just live with it. The 60C/90C figures are not supposed to be for the surface of the bed. They're meant to reflect the temperature reported by the printer. That said, 90C is a bit low for printing ABS (but the bed/power supply on your printer might not be powerful enough to get any higher).

\n" }, { "Id": "1526", "CreationDate": "2016-07-19T13:45:51.487", "Body": "

Can anyone tell me where to connect a cooling fan on the Sanguinololu v1.3a board? A few of the information pages mention three PWM outputs: Extruder, bed heater and fan, but none of the wiring diagrams that I can find, actually show the connections for the fan.

\n", "Title": "Sanguinololu 1.3a fan connection", "Tags": "|cooling|fans|", "Answer": "

It appears that there isn't an \"out-of-the-box\" solution to your request, but luck is with you. Someone with your board and objective has posted what appears to be a reasonable modification:

\n\n

http://www.instructables.com/id/Add-a-Cooling-Fan-to-your-Rep-Rap-Sanguinololu/

\n" }, { "Id": "1535", "CreationDate": "2016-07-21T17:22:27.533", "Body": "

The $200 Monoprice Select is inexpensive but also cheap, so experiences I've seen appear to involve lots of repairs. In spite of that, I've bought one, as I'm able to fix things (sometimes) and the first few prints have been pretty good quality. ( I hear this unit is also sold as a Wanhao I3 )

\n\n

First repair was that the control knob only worked in one direction (fix: bad encoder-PCB connector cable. Repaired)

\n\n

Upon trying to change filament, the feeder fitting that enters the hot-end heatsink has come off. This fitting has a thread, but strangely the heatsink does not seem to. As I tried to unscrew it, to check for a feed blockage, it just let go, and I see no part that could have held it in place. (See pic)

\n\n

I'm guessing that it was just epoxy-glued to the heatsink. Sounds kludgy but possible. Does anyone know?

\n\n

Can any other owner verify how theirs is attached?

\n\n

[ Also, posted as a caution for someone else who tries to do this. :( ]

\n\n

Thanks in advance. \n\"broken

\n", "Title": "Monoprice Select Mini - Detached Filament Feeder fitting", "Tags": "|monoprice-select-mini|repair|", "Answer": "

Solution - there is a set-screw and a square nut in that T-slot which holds the feeder fitting in place on the heat sink.

\n\n

One may well think (as I did) that the fitting is threaded into the heatsink, since it does have a big hex-head for a wrench. A small turn an the set screw lets go, the nut sliding down the slot with it. Thus when you inspect it, you don't realize they are involved 'cuz they aren't around.

\n\n

Simply slide back up and you're in business.

\n\n

Mystery solved - thx to Mark on Twitter. :)

\n" }, { "Id": "1546", "CreationDate": "2016-07-23T02:32:45.660", "Body": "

I would like to use my old Xbox 360 Kinect as a scanner for 3D modeling and (hopefully) printing a few busts of friends/family members; however, my efforts have failed in each direction that I have taken. Has anyone had success with this, and if so, how do I fix the issues that I am facing?

\n\n

What I have done so far and the problems that I have had:

\n\n
    \n
  1. I have purchased a cable to supply 12V to the Kinect while also allowing USB to a PC.
  2. \n
  3. I have installed the Kinect SDK as well as the Kinect Developers Toolkit on my Windows 10 Ultrabook.
  4. \n
  5. I installed the Microsoft \"3D Scan\" app and the \"Kinect Readiness\" app (not the name, unsure of it at this point) provided by Microsoft.
  6. \n
\n\n

**Issue: ** Neither Microsoft app detects the Kinect when cable is plugged in. The Kinect's light slowly flashes green. Also, my cursor freezes when the Kinect is plugged in.

\n\n
    \n
  1. I checked the Hardware profiles and found a notification that I do not have the drivers for the Kinect motor and that no suitable drivers could be found. However, no other details about the Kinect showed up.
  2. \n
  3. I uninstalled the SDK and developer toolkits and installed v1.7, ran them in compatibility mode (Windows 8) and same issues arose.
  4. \n
  5. I then began looking into using a different program, found ReconstructMe. This too failed to detect the Kinect even when it was manually selected as the scanner.
  6. \n
\n\n

Any ideas? I also have a Linux Mint laptop that I will happily use for these efforts if I knew what software to capture the 3D scan with. It seems that there are several driver options out there, just unsure of what to use besides that. It seems that Blender can be used for some motion capture with Kinect; however, I am unsure of how this relates to my goals.

\n", "Title": "Using Kinect 360 as 3D Scanner", "Tags": "|3d-models|scanning|", "Answer": "

I tried the same setting old kinect (but for PC - actually with power supply and usb-cable) plug to my surface windows10.

\n\n

After some troubles, it works now ;-)

\n\n

The mentioned MS 3D scan and kinect readiness wont work. It is supposed for the new kinect2 model, because of the new driver. Best you deinstall both. Even the drivers and SDKs.

\n\n

You need Kinect for Windows v1.8 for the old kinect. https://www.microsoft.com/en-us/download/details.aspx?id=40278

\n\n

RecFusion is a software for using a single or multiple depth cams for getting 3d models. It works quite good with my kinect. It costs 99euros. http://recfusion.net/index.php/en/download

\n" }, { "Id": "1555", "CreationDate": "2016-07-24T21:40:34.267", "Body": "

I want to access the bed and nozzle temperature sensor data of a 3D printer via serial connection. Could someone please guide me in the right direction?

\n

Edit: Response from printer:

\n
Connecting to printer...\nConnection response from printer:\n6R7\ufffdP)\ufffd\ufffdh>L\ufffdJO\ufffd \ufffd \ufffd\ufffdV\ufffd\\\ufffd\ufffd`\ufffdr\ufffd\ufffdT \ufffd\ufffd SK\ufffd<\u02aa\ufffd \ufffd>\ufffdVw^$\ufffd\ufffd\ufffd|\ufffd\ufffd\ufffdR n\ufffdI, o\ufffd!\ufffd\ufffd\ufffd\ufffdH>\ufffd\n                                                                          mx\ufffd}M#\n                                                                                 \ufffdLY\u0523Ph\ufffd\ufffd\ufffd^@ \ufffd\ufffdrp \ufffd TP\ufffdj\ufffd~1\ufffd\ufffd V6     6 \ufffd f\ufffd 6\ufffdk#\ufffd\ufffd\ufffd&\ufffdH\".\ufffd\ufffdk K2\ufffd\ufffd\ufffdek\ufffd(\ufffd\ufffd&\ufffd\ufffd^K\ufffdk2\ufffd\ufffd\ufffd\ufffdUl\ufffd(\ufffd \ufffd\ufffd\ufffd~N\ufffd\u0288*.\ufffd>Z\ufffd\ufffdk#d$\ufffd\ufffd\ufffd\ufffd\ufffd(\ufffd \ufffd\ufffd\n\ufffd\ufffd(\ufffd \ufffd\ufffd\ufffd v\ufffd\ufffd\n\ufffd\ufffd\ufffd\nAsking for temperatures (M105)...\nTemperature response from printer:\n^CTraceback (most recent call last):\n  File "printer_files/serialaccess.py", line 15, in <module>\n    response = ser_printer.readline()\n  File "/usr/lib/python2.7/dist-packages/serial/serialposix.py", line 446, in read\n    ready,_,_ = select.select([self.fd],[],[], self._timeout)\nKeyboardInterrupt\n
\n

After the code asks the printer for temperature values, there is no output on the terminal anymore. I waited for a while and then killed it using control+c. It is clear that the printer is responding to the connection response but I am not sure why it is not returning the temperature values.

\n", "Title": "Access Temperature sensor data of 3D printer via Serial connection", "Tags": "|extruder|heated-bed|nozzle|", "Answer": "

Assuming your printer accepts conventional G-code flavor, extruder and bed temperature can be retrieved by sending M105 through the serial port. The printer will respond with ok T:XXX.X B:XXX.X where T is the nozzle temperature and B is the bed temperature.

\n\n

You can read more about the specific G-code in the RepRap wiki.

\n\n

EDIT:\nI am editing the answer to include information for connecting to the printer using a simple python script.

\n\n

The following script, first opens a serial connection to the printer. For Marlin firmware, when you first connect to the printer, it needs some time to initialize and respond back. That is why some delay is needed before reading the response. After printing the response, the script sends the M105 command, waits for 100ms, then reads the response of the serial buffer and prints it on the screen.

\n\n

Note /dev/ttyUSB0 is the serial port name and in your case it may be different. Also 250000 is the baud rate of the connection; 250000 is the default value of my printer so you need to replace this with the baud rate that your printer uses.

\n\n
import serial\nimport time\n\nser_printer = serial.Serial('/dev/ttyUSB0', 250000)\nprint \"Connecting to printer...\"\ntime.sleep(30)  # Allow time for response\nbuffer_bytes = ser_printer.inWaiting()\nresponse = ser_printer.read(buffer_bytes)  # Read data in the buffer\nprint \"Connection response from printer:\"\nprint response\nprint \"Asking for temperatures (M105)...\"\nser_printer.write('M105\\n')\ntime.sleep(0.1)  # Allow time for response\nprint \"Temperature response from printer:\"\nresponse = ser_printer.readline()\nprint response\n
\n" }, { "Id": "1559", "CreationDate": "2016-07-26T11:56:13.410", "Body": "

A 3D printer can either print layer by layer or carve an object layer by layer to obtain an object. I heard somewhere that 3D printing technology isn't that accurate for printing minute details like fingerprints and iris patterns. Printing an iris pattern using a 3D printer would be a nice test to find this out. Can it print the iris pattern accurately? If not, then to what extent would be the accuracy of the 3D printed model of iris? Many commercial iris scanners can be easily fooled by high quality images of iris. Can a 3D printer print the these minute iris patterns with as much detail?

\n", "Title": "Are 3D printers efficient for printing a detailed iris to bypass iris identification system?", "Tags": "|efficiency|", "Answer": "

I take lectures in university and was asked to read a review paper on 3D printed organs by Anthony Atala (the most famous paper in printed organs research). The paper discussed about using several techniques to print the tissue we need at functional resolution. The review also cites detailed procedures to 3D print lung and skin tissues.

\n\n

Coming back to your question, we have reached a point in time where we can scan a real Iris and print them! Yes. And people use this technique called self assembly to achieve this (Other design approaches like Biomimicry and MiniTissue assembly is also being used right now). To do this we first extract cells from the donor. Or We do a functional high res scan of the extracted cell/part that we want to replicate. This is done via FMT-CT-Fluorescence Imaging, etc. And we cultivate the cells in bio incubators (we can also print cells btw - If the exact environment and operational conditions are maintained, we can print cell-replicas that will later self assemble to form the Iris with the same resolution and functional properties as that of the real one). The cultivated/printed cell is used as the tissue forming material. Forming is done by the cells themselves and is thus christened Self assembly.

\n\n

Although this may sound futuristic, Autonomous Self Assembly is something that's already being done in Labs! The method works by studying embryonic organ development. For instance, Early stage cellular components of a developing tissue makes their own ECM. Like mentioned before, if we use proper signalling, and environmental manipulation, we can create autonomous organization and patterning to make something we want. Advantage of this method is that we can work without scaffolds. This method relies on cell as primary driver of histogenesis. Knowledge of how an embryo grows into tissue (embroyo tissue genesis and organogenesis) is applied to achieve \"real\" cell dimensions/properties.

\n\n

I would recommend you to read the paper for detailed information. The method I mentioned in this post is only one among three other methods that are being used currently in this domain.

\n\n

So yeah. My answer is yes. You can print an Iris and fool the system.

\n\n

--\nUpdates

\n\n\n" }, { "Id": "1560", "CreationDate": "2016-07-26T17:40:49.773", "Body": "

There was a contest to develop 3D printable files for the International Space Station's 3D printer. The winner got a 3D printer ... runners up got Fluke DVOM's and all entrants got a t-shirt.

\n\n

ISS 3D Print Contest

\n\n

They offer 3 materials: ABS, HDPE, and PEI+PC ... I'm not familiar with the last one. Anyone know?

\n\n

If found this material on Matweb: PEI+PC Alloy

\n\n

These links are thought to last a very long time. I hope many of you decide to upload a project into contest site and compete for the grand prize ... A sweet John Fluke DVOM. If nothing else a free awesome T-shirt.

\n", "Title": "What is PEI+PC 3D Print material? ISS 3D Print Contest", "Tags": "|abs|hdpe|", "Answer": "

Ultem 9085, the most common ultem resin used for AM, is a blend of PEI-PC, as seen here https://www.sabic-ip.com/gepapp/Plastics/servlet/ProductsAndServices/Product/series?sltPrdline=ULTEM&sltPrdseries=Aerospace%20and%20Transportation&search=Search#searchresults.

\n\n

Ultem is a trade name for PEI alloys made by Sabic and 9085, used in filaments made by both Stratasys and 3dXtech as the two most visible suppliers are both made with this same alloy. It is used for high temperature resistance and strength and needs to be printed at upwards of 300C in a contained environment.

\n\n

Source-Intern at Made in Space.

\n" }, { "Id": "1562", "CreationDate": "2016-07-26T20:08:08.337", "Body": "

Ryan Carlyle seems to keep suggesting in my questions here, that using a 24 V power supply is safer than using a 12 V one.

\n\n

Early on, I ordered the wrong power supply, obtaining a 24 V one instead of a 12 V; I found that it was incompatible with my RAMPs 1.4 board... unless of course you pull off all the components and replacement with 24 V compatible ones.

\n\n

Now granted my board needs to be replaced if I am to use such a power supply, but are there any other components that need to be replaced if I'm going to use 24 V power, or am I safe with what I am using?

\n", "Title": "Do I have to buy all new components if I were to get a RAMPS 1.4 that supports 24 V power?", "Tags": "|switching-power-supply|vitamins|", "Answer": "

You need all components that are supplied voltage by the RAMPS board to be able to deal with a 24V input. Some of your parts are most likely compatible, as the stepper drivers. Others get the 5V from the Arduino, as the endstops. Some will most likely need replacement, as

\n\n\n" }, { "Id": "1577", "CreationDate": "2016-07-31T16:15:33.237", "Body": "

I have a prusa i3 and have been trying to calibrate it. I have been trying to upload the new steps per mm and I get to done uploading. However when I go into Repetier host and type M503 to get the printers settings it still shows the steps as 100 for the y axis, it needs to be 96.1810. I am using ramps 1.4 I think, with Adrunio mega 2560. If I am remembering this correctly isn't there something you have to do when uploading a new sketch like hold the rest button or something?

\n", "Title": "Uploading Repetier firmware to Arduino?", "Tags": "|prusa-i3|firmware|arduino-mega-2650|", "Answer": "

If your firmware doesn't store values you modified after flashing, for example using Repetier's option \"Firmware EEPROM configuration\", you probably didn't define EEPROM setting in Marlin.

\n\n

To do this you have to uncomment the option in Marlin's Configuration.h:

\n\n
// EEPROM\n// The microcontroller can store settings in the EEPROM, e.g. max velocity...\n// M500 - stores parameters in EEPROM\n// M501 - reads parameters from EEPROM (if you need reset them after you changed them temporarily).\n// M502 - reverts to the default \"factory settings\".  You still need to store them in EEPROM afterwards if you want to.\n//define this to enable EEPROM support\n  #define EEPROM_SETTINGS\n\n#if ENABLED(EEPROM_SETTINGS)\n// To disable EEPROM Serial responses and decrease program space by ~1700 byte: comment this out:\n#define EEPROM_CHITCHAT // Please keep turned on if you can.\n#endif\n
\n\n

Check that the lines #define EEPROM_SETTINGS and #define EEPROM_CHITCHAT are both uncommented (i.e. defined)

\n" }, { "Id": "1579", "CreationDate": "2016-07-31T19:53:18.623", "Body": "

I cloned the Marlin repository and changed all my settings in configuration.h to match my old settings. Now when the RC branch gets updated, is there a way to upstream pull the changes without losing my settings?

\n", "Title": "Cloning the Marlin git repo while keeping my configuration.h settings?", "Tags": "|marlin|reprap|", "Answer": "

Based on this answer, you need the following procedure:

\n\n
    \n
  1. Stash your local changes using git stash
  2. \n
  3. Pull from remote repository using git pull
  4. \n
  5. Merge your stashed configuration file using git stash pop
  6. \n
\n\n

Of course, if there are changes in the configuration.h file on the remote repository and cause conflicts, you will need to resolve them, but it should be straight forward.

\n" }, { "Id": "1581", "CreationDate": "2016-08-02T17:30:44.143", "Body": "

I use a Micro3D printer, running on OctoPi (yay!) (although this question should be relevant to any 3D printer that offers these features) and have options for raft and wave bonding.

\n\n

Are there best case scenarios for when it is appropriate to use (or not use) either? Can/should they ever both be used at the same time?

\n", "Title": "When to use Wave Bonding vs Raft on a 3D print?", "Tags": "|rafts|", "Answer": "

After additional research, it seems that using both at the same time is ill-advised (more like pointless).

\n\n

Wave bonding is best suited for larger prints, primarily to prevent warping of the initial layer.

\n\n

Rafts appear to be recommended regardless, other than for advanced users.

\n" }, { "Id": "1582", "CreationDate": "2016-08-02T18:15:54.343", "Body": "

I often have trouble with prints being especially difficult to remove from the build plate on my Micro3D printer. My wife suggested using a hair dryer on the underside of the plate. I was initially appalled at the idea, but now I think she may be on to something. Is this safe? Good idea? Bad idea? Heresay?

\n", "Title": "Is using a hair dryer on my M3D build plate safe?", "Tags": "|build-plate|", "Answer": "

cold wind can help to acceleratory cool down and dry the build plates to make it easy to part them.

\n" }, { "Id": "1588", "CreationDate": "2016-08-03T21:47:25.763", "Body": "

I just received my Wanhao Duplicator i3 PLUS. Everything prints and functions fine via SD card and the display but when connecting to my Mac computer the printer seems to restart.

\n\n\n\n

Any ideas what it could be?

\n", "Title": "Wanhao i3 PLUS restarts when connecting USB", "Tags": "|arduino-mega-2650|wanhao|", "Answer": "

That is normal on most printers. They restart if you connect.

\n\n

As you have an SD card slot I would recommend using it rather than your computer. If your connected computer goes to sleep it would also reset the connection after wakeup.

\n" }, { "Id": "2593", "CreationDate": "2016-08-04T15:17:32.630", "Body": "

\"enter

\n\n

By what process does the Prusa i3 determine it's home position?

\n\n

I have a feeling that it works like this, but I'm not entirely sure about it:

\n\n
    \n
  1. If none of the end stops are activated; assume you are somewhere between Max-X, Max-Y, Max-Z and the point 0,0,0.
  2. \n
  3. do\n\n
      \n
    1. For axis in Axies\n\n
        \n
      1. If axis endstop not signaling\n\n
          \n
        1. subtract 100 steps from axis.
        2. \n
      2. \n
    2. \n
  4. \n
  5. while endstops are not signaling, or if point 0,0,0 not reached;
  6. \n
\n", "Title": "How does the home position work on a Prusa i3 RepRap Printer?", "Tags": "|prusa-i3|reprap|", "Answer": "

You have the endstop which sends a on or off to the controller board.

\n\n

The boards firmware knows if it is a close or open switch.

\n\n

Lets assume pressed is on and unpressed is off.

\n\n

When the axis is at home it will read as ON. The firmware then will not allow the axis to move in the direction you have designated is the Home direction. Now this is probably where you got confused. You in fact are the one to open a code file, and say what direction is home. When your turn it on the first time, chances are you have done it wrong on at least one. Then you will frantically go and manually press it yourself as your printers hotend crashes HARD into the bed.. All of us have been there !

\n\n

Generally Prusas use ramps 1.4

\n\n

http://reprap.org/wiki/RAMPS_1.4

\n\n

Your code is your firmware. Usually you just edit a header file (single file). These files are heavily documented, depending which you use. Your seller might provider you with a preconfigured header file.

\n\n

Will note that yes, if the endstop slips or is not position right, the printer will crash into it self. Usually no damage, but it is a very unpleasant thing to happen.

\n\n

Also worth noting that some firmware will force you to home before you move. Others will let you crash the printer on the opposite side. Just a motor, will do whatever you tell it to.

\n\n

In short. You tell it how the endstops work and what direction is home on a printer.

\n" }, { "Id": "2607", "CreationDate": "2016-08-06T23:15:36.037", "Body": "

I own a delta 3D printer. The problem is that, at the beginning of a print the extruder outputs dirty filament. I want a clean filament flow at the start of my prints!

\n\n

How can I make the hotend exit the print surface (glass plate) by 10mm, extrude the bad filament and go back to printing again? Can this be done with G-code?

\n\n

My Z high is 190 mm and the glass plate diameter is 120 mm. I'm using Marlin + Ramps 1.4.

\n\n

I'm using Repetier-Host and CuraEngine as Slicer, but I really would like a G-code that can work on multiple environments like Cura and Repetier. I just want to add it to the start G-code and print!

\n", "Title": "Clean or wipe move for delta 3D printers", "Tags": "|print-quality|marlin|delta|", "Answer": "

Slic3r, and all other slicers that I saw, have an option called Skirt that enables your printer to print a number of lines around your object before it starts printing the object itself.

\n\n

This should ensure that both the nozzle is filled with filament and the printer got rid of any burned / dirty filament when it starts printing your object.

\n\n

If you use Slic3r as standalone, you have to activate expert mode in the preferences. If you use it through Repetier-host it should already be in expert mode.

\n\n

You can see all available options in the Slic3r documentation for Skirt.

\n" }, { "Id": "2608", "CreationDate": "2016-08-07T02:49:14.600", "Body": "

Are there any 3D printing filaments or specially designed desktop systems which can print soft artificial skin for human-robot? For example made of soft silicone or something similar?

\n", "Title": "Possible ways to print soft skin for human robot?", "Tags": "|filament|", "Answer": "

Oh that is a tricky tricky question!

\n\n

First you will probably need a good scan of the robot. You will have the cad of the robot but chances are scanning will make life easier.

\n\n

http://www.3ders.org/articles/20151201-kinect-easy-3d-printing-tool-with-release-of-3d-scan-app-for-windows.html

\n\n

Also see Reprap for other options

\n\n

http://reprap.org/wiki/3D_scanning

\n\n

Following... it is not going to be that simple. If it was my project I would\nPrint a mold out of ABS. ABS reacts to acetone and you can vapor treat it.

\n\n

http://www.logarithmic.net/pfh/blog/01366106156

\n\n

\"Example

\n\n

That said you might burn your house down. Also it is inconsistent. But would give you a smooth finish.

\n\n

Next you can do just normal mold printing and be okay with the ribbing. Then you pour your silicon material

\n\n

\"Example

\n\n

Next you can well just print with silicon. The ninjaflex guys have a ton of different types of flexible materials and new ones coming out.

\n\n

https://ninjatek.com/products/filaments/ninjaflex/

\n\n

\"Filaflex

\n\n

On that topic I will mention a local company that I have no direct affiliation with. I just see them at the hacker space. They make a extruder just for flexibles.

\n\n

https://flexionextruder.com/

\n\n

Other areas to look at. More for making metal but it may be of use.

\n\n

http://www.instructables.com/id/From-3d-printed-part-to-metal-the-lost-plaabs-me/

\n\n

Last is again the mold but you make the molds on a expensive form 1. Or you make the skin on the form1 as they can do flexibles. Small build area. Material is around 100 a ltr or more

\n\n

http://formlabs.com/products/3d-printers/form-1-plus/

\n\n

Good luck!

\n\n

Also check out this facebook group (no affiliation)

\n\n

https://www.facebook.com/groups/3DPrintProps/

\n\n

If anyone has done this it is those guys. Oh Wait No I take that back Look into these fellows. They have a very active group. (google group) not to mention that they are also directly doing what you are.

\n\n

http://inmoov.fr/

\n\n

\"InMoov\n\"InMoov

\n\n

Just don't be like this guy and make it look like a movie star... \nhttp://www.mirror.co.uk/news/weird-news/man-builds-scarlet-johansson-robot-7667715

\n" }, { "Id": "2615", "CreationDate": "2016-08-07T23:54:39.570", "Body": "

I am new to 3D printing and I am assembling/calibrating an Ultimaker Original+ for my college.

\n\n

I am done with all the assembling and the printer works fine except for one thing.

\n\n

The print bed moves way up. It touches the extruder and pushes it upwards. So, the extruder cannot extrude any material.

\n\n

I tried to set the Z axis value before string the print but as soon as I start the print the platform just moves up and blocks the extruder.

\n\n

Please let me know if you need any more information from my side.

\n", "Title": "Print bed moves way up for printing", "Tags": "|z-axis|ultimaker-original|", "Answer": "

It is normal for an Ultimaker print bed to move up. But it should be stopped before the nozzle.

\n\n

As it doesn't stop the end stop doesn't work. So either the end stop is not connected to the right connector or it is triggering too late.

\n\n

The Z end stop must be screwed at the right hight. If you mount it too far up it will not trigger. The bed assembly has a small wooden peace that sticks up. The idea is that that wooden peace pushes the end stop at the right hight.

\n\n

So instead of configuring the hight in the firmware you need to mount the Z end stop at the exact right hight. You can make the fine adjustments(Bed Leveling) with the screws that hold the bed.

\n\n

You can test the end stop by manually triggering while the bed moves up. It needs to stop immediately once you trigger the end stop. If the bed does not stop, then check the connection or exchange the switch.

\n\n

It can happen that the switch itself malfunctions if you screwed it in too tightly. So loosening up the screws a bit is a final test.

\n" }, { "Id": "2619", "CreationDate": "2016-08-09T04:55:36.843", "Body": "

Background:

\n\n

I just successfully installed a RepRapDiscount Full Graphic Display on my Prusa i3; and in doing so, I've upgraded all of the firmware to the latest version of Marlin, 1.1.0-RC7. With my previous version of Marlin, 1.0.2-1, everything was working perfectly. I've transferred every parameter over from my older version to my newer one. There were some aspects of code that were written differently, but essentially everything works except the Z-Axis.

\n\n

Problem:

\n\n

The one issue that I seem to be having is with any kind of movement with the Z-Axis. When it moves, whether from the LCD or Repetier, it appears to accelerate as it is supposed to, but then gets stuck/skips a bunch of steps, and then decelerates slowly as it's supposed to. Regardless of the distance I instruct it to move, it will accelerate normally for the same amount of time/distance/steps, go into this unholy mode for a period of time/distance/steps relative to what it was instructed, and then smoothly decelerates for the exact same amount of time every time. For example, if it is supposed to move 10000 steps, it will move 100 steps smoothly, then skip for 9800 steps, and finish with another 100 steps smoothly. If, for example, it needs to move 5000 steps, it will also move 100 steps smoothly, then skip for 4800 steps, and finish smoothly the last 100 steps. Keep in mind, these numbers are completely drawn from space; they are not accurate by any stretch of the imagination. I'm just trying to paint as clear of a picture as possible.

\n\n

Troubleshooting:

\n\n

At first I checked to make sure everything is mechanically sound.

\n\n\n\n

I then wanted to narrow it down to being the firmware, and not electrical. For the record, maybe I'm missing something obvious, so please let me know.

\n\n\n\n

Moving onto the firmware.

\n\n

I've played around with the default settings section in Configuration.h with no avail. Regardless of how low, or high, I go with any of the max acceleration settings, I cannot seem to fix the problem. It accelerates slower, but the overall effect is the same.

\n\n
#define DEFAULT_AXIS_STEPS_PER_UNIT   {80,80,3840,90}\n#define DEFAULT_MAX_FEEDRATE          {500, 500, 5, 25}\n#define DEFAULT_MAX_ACCELERATION      {2000,2000,10,2000} \n#define DEFAULT_ACCELERATION          1000\n#define DEFAULT_RETRACT_ACCELERATION  1000\n#define DEFAULT_TRAVEL_ACCELERATION   1000\n#define DEFAULT_XYJERK                20.0\n#define DEFAULT_ZJERK                 0.4\n#define DEFAULT_EJERK                 5.0\n
\n\n

Theory:

\n\n

At this point, if I had to guess, it has something to do with the maximum speed setting, or the microstepping. These are some of the settings that I have on the new firmware.

\n\n
#if ENABLED(ULTIPANEL)\n#define MANUAL_FEEDRATE {50*60, 50*60, 4*60, 60} // Feedrates for manual moves along X, Y, Z, E from panel\n#define ULTIPANEL_FEEDMULTIPLY  // Comment to disable setting feedrate multiplier via encoder\n#endif\n\n// minimum time in microseconds that a movement needs to take if the buffer is emptied.\n#define DEFAULT_MINSEGMENTTIME        20000\n\n// If defined the movements slow down when the look ahead buffer is only half full\n#define SLOWDOWN\n\n// Frequency limit\n// See nophead's blog for more info\n// Not working O\n//#define XY_FREQUENCY_LIMIT  15\n\n// Minimum planner junction speed. Sets the default minimum speed the planner plans for at the end\n// of the buffer and all stops. This should not be much greater than zero and should only be changed\n// if unwanted behavior is observed on a user's machine when running at very slow speeds.\n#define MINIMUM_PLANNER_SPEED 0.05// (mm/sec)\n\n// Microstep setting (Only functional when stepper driver microstep pins are connected to MCU.\n#define MICROSTEP_MODES {16,16,16,16,16} // [1,2,4,8,16]\n\n// Motor Current setting (Only functional when motor driver current ref pins are connected to a digital trimpot on supported boards)\n#define DIGIPOT_MOTOR_CURRENT {135,135,135,135,135} // Values 0-255 (RAMBO 135 = ~0.75A, 185 = ~1A)\n\n// Motor Current controlled via PWM (Overridable on supported boards with PWM-driven motor driver current)\n//#define PWM_MOTOR_CURRENT {1300, 1300, 1250} // Values in milliamps\n\n// uncomment to enable an I2C based DIGIPOT like on the Azteeg X3 Pro\n//#define DIGIPOT_I2C\n// Number of channels available for I2C digipot, For Azteeg X3 Pro we have 8\n#define DIGIPOT_I2C_NUM_CHANNELS 8\n// actual motor currents in Amps, need as many here as DIGIPOT_I2C_NUM_CHANNELS\n#define DIGIPOT_I2C_MOTOR_CURRENTS {1.0, 1.0, 1.0, 1.0, 1.0, 1.0, 1.0, 1.0}\n
\n\n

Solution:

\n\n

If anybody could help me, or even point me in the right direction, I would really appreciate it. I'm getting desperate with this new firmware version. It has all these conditional tabs that didn't exist in my other one, and I can't quite figure it out.

\n", "Title": "Z Axis getting stuck with Marlin 1.1.0-RC7", "Tags": "|prusa-i3|marlin|firmware|z-axis|", "Answer": "

The problem is likely in the MAX_FEEDRATE. Initially the moves are smooth, indicating that acceleration is not a problem. However, you have your maximum speed set to 5mm/s, which, for m5 threaded rod (with 0.5mm pitch), translates to 10 revolusions/second for the threaded rod. That is quite fast, and the stepper probably can't keep up. Try reducing the feedrate.

\n\n

The firmware microstepping and current settings have absolutely nothing to do with it, since these are not supported on your setup (which has physical potentiometers and microstepping jumpers).

\n" }, { "Id": "2621", "CreationDate": "2016-08-09T15:33:03.320", "Body": "

Related to an issue I had in this question, where the PTFE tube feeding my filament to the metal tip of the extruder clogged and became discolored: what are the advantages and disadvantages of changing out my extruder (Mk10 on a FlashForge Creator X) for an all-metal solution like the one advertised here (by Micro-Swiss).

\n\n

I understand that the conversion would allow me to print higher-temperature materials (like nylon), but I'm also trying to figure out the trade-offs with regard to printing PLA/ABS parts.

\n", "Title": "What are the advantages and disadvantages of an all-metal hot end compared to one with a PTFE heat break?", "Tags": "|extruder|fdm|hotend|", "Answer": "

My Tevo Tarantula had an all metal hotend included and I never ran into any issues only printing PLA and PETG, most of the time I was even able to pull out the filament while the printer was cold.

\n\n

After I upgraded to an E3Dv6 clone with PTFE lined heatbreak I started to have issues because of the filament getting stuck where the Bowden tube and the heatbreak connected, so I recently replaced it with an all metal heatbreak again and the issues instantly went away.

\n" }, { "Id": "2629", "CreationDate": "2016-08-11T10:27:03.247", "Body": "

I'm currently using FSL3d's RetinaCreate to prepare 3d-files for printing for research purposes. In this, I am relying on their Optimal Rotation feature which rotates the object by, as far as I can tell, judging models on their stability with regards to the printing direction. After rotating the model, supportstructures are generated to keep the model in place while printing without the bits and pieces floating about and to keep structural stability.

\n\n

My question is, as I've neither found a manual nor any source-code (and admittedly I am terrrrrible at maths), how can I describe the kind of algorithm/logic used by the software here? This is essential for me, as I need to describe this in my research. Cheers, Jesse

\n", "Title": "RetinaCreate Optimal Rotation", "Tags": "|3d-models|software|rapid-prototyping|", "Answer": "

Optimal rotation can be considered from different perspectives. For example

\n\n\n\n

But have a look on Meshmixer as Ryan Carlyle suggested. In this app you can see 3 kinds of auto orientation procedures. Additionally Meshmixer can create angle support and tree-like support.

\n\n

In terms of mathematics.

\n\n

I cannot suggest any sources but you can imagine the way it calculates such optimal rotation. Let's say it will be dead simple explanation of the second approach:

\n\n

Let's get all faces (facets) and let's project their center points on the \"floor\" (heatbed) or \"ceiling\". If imagined projection line slices any other faces then rotation is not optimal. App can calculate \"a weight\" of such orientation and then try to find the smallest weight of all possible orientations.

\n\n

Of course it's not that simple. App has to manage concavity, which face requires support at all, and many other aspects but in general you can imagine the algorithm.

\n\n

edit

\n\n\n" }, { "Id": "2637", "CreationDate": "2016-08-12T06:42:27.340", "Body": "

When using a heated bed with your printer, I have seen claims of running temperatures of 90c throughout the print.

\n\n

That seems like a fairly high power use to keep a large slab of, say, aluminium at 90c for long print times (ie multiple hours).

\n\n

Is there a common 'sweet spot' for operating temperature?

\n\n

Does it depend on material?

\n\n

Is a heated bed required?

\n", "Title": "Heated bed - what are the benefits? Why use one?", "Tags": "|heated-bed|", "Answer": "

Heatbeds have two purposes:

\n\n
    \n
  1. Increase surface energy of the print bed to improve bonding strength of the first layer (particularly important when using surfaces like PEI or Kapton)
  2. \n
  3. Keep the bottom few millimeters of the print hot enough to provide a warp-free foundation for the rest of the print.
  4. \n
\n\n

The bit about surface energy is straightforward. Most materials are stickier when hot than cold. In comparison, pure mechanical-bonding bed surfaces like fibrous painter's tape and perfboard don't particularly benefit from bed heat.

\n\n

Warping is a bit more complicated. The basic cause of warping is when the previous layer is allowed to cool and thermally contract before the next layer is deposited. When you stick hot, expanded material on top of cold, contracted material, large shear stresses are generated when the fresh material cools and contracts. Those inter-layer shear stresses then accumulate over many layers into large-scale bending stresses that try to lift the edges of the print off the bed.

\n\n

So, to prevent warping, we should minimize the amount that the previous layer is allowed to cool before the next layer goes down. But we DO need it to cool solid so the print doesn't sag in a mushy mess. This is a balancing act: cooling the plastic solid without over-cooling it. The optimal temperature for the print is right around the glass point of the plastic: this is the temperature at which the plastic becomes fully solid and thermal contraction stresses start to accumulate.

\n\n

The extruder pumps more heat into the print as it deposits molten plastic and radiates a little bit of heat. So we want to set the heatbed temp a little bit below the glass point to ensure the print is able to cool solid. Now, this gets a bit difficult, because everyone's print bed temperature sensor is different. What matters is bed surface temp. Many people have to set their bed temp quite a bit higher than the actual surface temp. It's just something you have to calibrate via print results. The exact filament glass point (Tg) also depends on the blend.

\n\n\n\n

There are other schools of thought, for example printing the first layer onto a surface much hotter than Tg for good adhesion, and then dropping the bed temp to a value somewhat below Tg to allow the print to solidify. That works fine too.

\n\n

But, with all that said, it's important to understand that the heatbed only keeps the bottom of the print warm. A centimeter up from the build plate, the print is typically much closer to ambient temp than it is to the bed temp. Heated build chambers are thus much more effective for large prints. But heatbeds are still quite effective, because they allow building a strong, warp-free foundation that resists warping stresses induced by the cooler zones higher up in the print.

\n" }, { "Id": "2654", "CreationDate": "2016-08-18T04:34:05.640", "Body": "

I have a object to print for which I want the base to be printed very rapidly because it's just a cube but as the print reached around 70\u00a0% a complex circular structure needs to be printed at a slower speed. Is there any way I could control the speed at the given percentage of job done?

\n

I want the cube to be printed at 50\u00a0mm/s and the complex circular structure at 40\u00a0mm/s.

\n

Printer Type - FDM

\n", "Title": "How to 3D print an object with variable printing speed?", "Tags": "|fdm|speed|", "Answer": "

Cura has a plugin called "Tweak at Z" that lets you change the speed at a specific layer/height, I used it when printing an object that's basically a curved box for 100\u00a0mm and then has tiny features in the last 10\u00a0mm and it worked very well.

\n" }, { "Id": "2658", "CreationDate": "2016-08-19T09:09:26.570", "Body": "

I'm currently attempting to make a repstrap using paper printed parts, like this guy : http://www.mariolukas.de/2012/05/repstrap-3d-drucker-aus-computerschrott-teil-1/\nI replaced the DC motor in a paper printer carriage assembly

\n\n

with a stepper motor (NEMA17). But there was not enough space to fit the axis of the nema 17 at the exact spot of the older DC motor axis, in short, the axis are not in the same place. The question is : if the axis is not in the exact same spot, will it affect the movement of the carriage or not at all ?\"new

\n\n

I supposed it would but i'm not sure since the carriage is limited in movement by the rails and that we still move the belt around.

\n", "Title": "Belt driven axis question", "Tags": "|reprap|belt|", "Answer": "

A rotation of the stepper motor 90 degrees clockwise or counter-clockwise would move it more to the center. You would only need lengthen or shorten the belt.

\n" }, { "Id": "2665", "CreationDate": "2016-08-21T14:31:02.263", "Body": "

My goal is to 3D print a 5 liter miniature barrel with a side stand, similar to this wooden one on Amazon. I want it to have a removable top so that a boxed wine bladder may be put inside, and there should be a hole on the top as well so that the spigot may stick out and be used. I have no experience with 3D modeling or printing, but I have access to a public 3D printer at my local library. I know you can print parts individually (ex. curved wood-colored sides with staves and holes to interlock and make up the body of the barrel, the metal-colored hoops to go around the barrel). I don't know what software to use, though. I was thinking of starting to learn Blender? Would that be effective for this project?

\n", "Title": "What is the best 3D modeling software for a beginner on a 3D printed mini barrel project?", "Tags": "|3d-design|software|blender|", "Answer": "

If you have any programming background at all, consider OpenScad. It is a functional type of programming language that lets you do a lot of things quite easily without art skills. OpenScad skills are useful for building customizable things on Thingiverse.

\n\n

Here is a place to start

\n" }, { "Id": "2667", "CreationDate": "2016-08-22T13:51:44.837", "Body": "

Metal powders are the fastest-growing segment within the 3D printing materials market, and 3D printing with metal offers a range of highly-sought out characteristics, including immense strength, reduced weight, biocompatibility and corrosion or thermal resistance, making it ideal for high-demand industries such as aerospace, medical, etc.

\n\n

Conventional methods require focusing a very intense energy source, such as a laser or electron beam, across a bed of metal powder, fusing the powder particles together in a pre-determined pattern to create the final 3D structure.

\n\n

While this method does allow for incredibly strong metal 3D structures to be produced, it has its drawbacks, mainly:

\n\n\n\n

If we ignore the cost, why can't we do hollow or enclosed architectures to be printed with this technique?

\n\n

Source: A closer look at the 12 biggest 3D printing tech innovations of the first half of 2016.

\n", "Title": "Why can't powder-based 3D printing techniques create enclosed or hollow structures?", "Tags": "|print-material|printing-powder|metal-printing|", "Answer": "

It's not hollow or enclosed structures that are a problem. It's structures that are hollow and enclosed. Think about it. The machine lays down a thin layer of powder, and then a laser fuses some of that powder together to make a shape. Then, it repeats the process for the next layer. If you try to build, for example, a hollow ball, the ball gets created in the middle of the powder, meaning that there will be powder around the outside of the ball and in the inside of the ball, and with the ball completely enclosed, there is no way for the powder that ends up inside the ball to get out. That's why hollow objects printed in this manner must have a hole somewhere to let out the powder contained in the object.

\n\n

This is not only true of metal powder, but any 3D printing process that uses a bed of powder or a volume of resin and fuses some of it into a shape. Fused deposition modelling (FDM), the process used by most consumer-level 3D printers, build models in thin air, so only air is trapped inside the printed objects, making the printing of hollow, enclosed objects less of a problem. Of course, one has to deal with gravity. A hollow object might collapse before the filament hardens enough to support its weight.

\n" }, { "Id": "2670", "CreationDate": "2016-08-23T05:58:00.500", "Body": "

This came up in one of my groups today. That we could not color bend, or mix 3d printing filaments. I have researched but I am not finding anything talking about Plastic mixing in an extruder.

\n\n

Why is it that we cannot take say a Diamond hotend, or a hotend with 5+ inputs, and mix any color we want? (assuming all the same type, ABSm, PLA). I think it would be interesting to at the least get a gradient effect on prints.

\n\n

The best I have seen is natural plastic and a marker system. Or a powder / advanced / out of hobbyist price range process that sprays ink. The only Color Bending I know of is with Recycled plastic that uses multi color. Not quite what I am looking for.

\n\n

Thanks!

\n", "Title": "What is stopping us from mixing 3D filament colors in an Extruder?", "Tags": "|filament|hotend|color|", "Answer": "

If you clearly observe the geometry of the print it has printed in different colours .the part is designed in such a way that it has the small height gap between the each colour but we cannot seen this gap in one projection plane..\nWe can print only in direct drive extrusion .by changing the filament of different colours without pausing the print by immediately changing the material . It is printed in single nozzle. The above image clears your doubt.it is designed with different heights.\"enter

\n" }, { "Id": "2673", "CreationDate": "2016-08-23T08:51:20.780", "Body": "

The turntable support part in my microwave has broken. It is a three armed part, with small wheels at the end of each arm.

\n\n

\"support

\n\n

I'm confident I could print a replacement, and reuse the existing wheels (since they can be removed).

\n\n

The heat of the food would be unlikely to conduct through to the PLA, but I'm concerned that it might get heated up by the microwave radiation. I can't find any clear information online about whether PLA absorbs microwaves, or if it is in any other way unsuitable for this.

\n\n

Will this be a disaster, or should I give it a go?

\n", "Title": "Can I repair my microwave with a PLA part?", "Tags": "|pla|food|", "Answer": "

There are Food Safe PLA filaments. These PLA filaments share similar qualities with ABS, in regards to temperature extremes et al. There are several on the market. (Eg. https://shop.germanreprap.com/en/pp-plastic-600g-3mm-black and http://www.formfutura.com/hdglass/ -- I've not used the HD glass personally but have been told it works well)

\n\n

When shopping for food safe PLA filaments check for the Material Safety Data Sheet (MSDS) that comes with the filament. It will contain all the normal chemical warnings, some composition information, etc. but will also indicate if the material is Food and Drug Administration(FDA) approved.

\n\n

PLA, food safe or not, can also have bacterialogical issues--the material is pourous enough to allow for bacterial growth. I would suggest using a polyurethane to seal the printed part to limit bacterial growth.

\n\n

You will need to check what sort of extruder you have on your pinter. Many brass extruders also contain lead which can leach into the printed product. Switch to a stainless steel print head to avoid this issue.

\n\n

General care--warm soapy water and handwashing.

\n\n

I would not, as yet, recommend any printed product as a vessel for food--cup/bowl. This element is still somewhat new and I don't personally trust the materials for extended contact with food. (Knives/spoons et al. have much shorter contact with food vs. a plastic mug of hot coffee).

\n\n

For your intended application, the piece in question would not come into direct contact with food to be consumed. As such, you should be fine with the correct filament.

\n" }, { "Id": "2676", "CreationDate": "2016-08-23T14:44:32.567", "Body": "

For a while now, I have been thinking about designing things such as small bedside tables, game/dvd/bluray racks for 3d printing. I've always thought that making them modular would be a good way to go about doing this as well.

\n\n

Modular design would help to create an end result that is vastly larger than the print volume of my 3d printer. I might even be able to recycle models for use in other projects. However, I'm not sure of what I need to think about if I decide to go ahead with these ideas I have floating around in my head.

\n\n

I'm assuming that certain joints (dovetail, etc), tolerances for different types of plastic due to shrinkage, and print settings (% infill, in particular) would be important to have thought about and evaluated to some extent, but I'm not sure about what else I might be missing.

\n\n

So my question is to anyone who has designed anything to be modularly printed. Have you really had to think carefully about the engineering side of the print? Or am I simply overthinking this? Should I just design what I want and give it reasonable infill, walls and whatnot, and just go for a trial and error approach? I'm sure there is a method to this madness, but is a concrete understanding of this type of engineering absolutely paramount when it comes to this sort of stuff?

\n\n
\n\n

EDIT: Although I've marked darth pixel's answer as accepted, I'm still going to follow JKEngineer's advise and check out that book as well since I feel as though proper engineering techniques alongside a good mentality towards how I would tackle the problem (as outlined in darth pixel's answer) would prove to yield better results in the long run.

\n", "Title": "What should I think about if I want to design something modular?", "Tags": "|3d-models|quality|print-preparation|print-strength|", "Answer": "

A book you would benefit from reading is \"Functional Design for 3D Printing...Designing 3D Printed things for everyday use - 2nd Edition\" by Clifford Smyth.

\n\n

It deals with FDM printing only. It deals with considerations of orientation of the parts being printed to address required strength in the 3 directions (x, y, z), tolerances, and designing parts in such a way that they can be assembled, have the strength needed, have flexibility, etc. In some instances he shows how to split a single functional part into multiple parts so that, when assembled, it actually performs as required.

\n\n

It's available from Amazon at Book on Amazon. I received it as a present and have no commercial interest in it.

\n\n

Here's a review: Book Review on 3D Printing for Beginners

\n" }, { "Id": "2689", "CreationDate": "2016-08-26T18:59:13.000", "Body": "

I have printed a MPCNC machine. It has a print area of about 30\" x 30\" and up to 11\" tall. (yes, those numbers are correct).

\n\n

I found a perfect piece of glass at a garage sale for $5.00 to use as my print bed.

\n\n

My problem now is how to heat the glass? I was wondering if there is some sort of tape that would perhaps mimic what is on the rear window of a car, but I couldn't find it anywhere.

\n\n

Any ideas or links to something that can get me some progress on my search would be greatly appreciated.

\n", "Title": "How do you heat a large glass print bed?", "Tags": "|heated-bed|glass-bed|", "Answer": "

I am heating my 1 meter by 1 meter plate (yes those numbers are correct) (in progress long term project) of tempered glass with a silicone heater bed I bought as overstock on ebay.\nSilicone Heating Mat

\n\n

\"Silicon

\n\n

From Reprap Wiki

\n\n
\n

Silicone Heater Pad in sizes\n silicone heater pad Pros: Fast heating Reliable Most use mains voltage\n - DOESN'T require any amps from the 12 volt supply Very low height Easy to install (adhesive backed) Can be relatively inexpensive 12 V\n types can be run directly from the controller MOSFET without a relay\n Integrated thermistor Long life span Cons: Can be expensive - silicon\n mat (50\u20ac) plus solid state relay (16\u20ac). Mains voltage, but can be\n purchased for 12 V. Need GROUND line to aluminum bed for safety.\n Relatively high temperatures possible (safety problem if thermistor\n dies/falls off).

\n
\n\n

I also talked to a few alibaba companies and they will happily make you a custom order. Just be wary working with them. You will also need to start another question about the particular parts you need. Actually I think I have a second one I bought from Aliexpress about that size. So no custom orders needed. Ran about 100 USD. I will note I used a MIC6 aluminum plate, custom cut at a local shop. That will have a more even heat profile than glass.

\n\n

I will note that I have to run mine from 220 VAC not 110 VAC power lines.. But the actual electrical costs are pretty low!

\n" }, { "Id": "2693", "CreationDate": "2016-08-27T13:46:01.610", "Body": "

I recently bought a Geeetech Prusa i3 x. After two full days assembling I can finaly try to print something. Having put in the filament into the extruder no filament actually came out of the nozzle. I opened up my extruder and the filament seems to be stuck in the nozzle.

\n\n

\"Filament

\n\n

Note that I have tried heating the nozzle up and both pushed and pulled but no movement.\nDoes anyone know what to do?

\n\n

Thanks,\nMerijn

\n", "Title": "Filament is stuck in nozzle", "Tags": "|filament|nozzle|", "Answer": "

There are a number of options you could try. If heating up the hotend does not work, you'll have to disassemble it. Remove the nozzle and the heatbreak (the threaded part). To disassemble the nozzle, you will need to heat it up and use pliers or wrenches to unscrew the parts.

\n\n\n\n

Do not try to disassemble it while cold, the expansion and contraction of metal with heat makes this impossible. When reassembling, be sure to heat up the nozzle before giving it a final tightening (again, this is to make sure that when the nozzle expands as it is heated, it makes a tight seal).

\n" }, { "Id": "2697", "CreationDate": "2016-08-28T00:32:36.523", "Body": "

I am an absolute beginner when it comes to 3d printing. I want to get into the hobby by designing aero automotive parts such as fender flares, custom gauge and switch pods, lips etc. I've never taken a CAD course but I would say I'm proficient enough with computers as I work as a developer. My uncle is an architect and it seems like autocad might be something good to go with. What are some other good alternatives that allow accurate modeling down to millimeters and possibly breaking larger objects into smaller 3d printable pieces to mash together?

\n", "Title": "What is a good software for designing car parts", "Tags": "|3d-design|", "Answer": "

You've avoided a number of attributes of a poor question by specifying your objective in detail, while also providing some indication of your experience. AutoCAD is not well known as being 3d printer friendly, although a skilled AutoCAD design user may be able to create acceptable models.

\n\n

One could consider more organic modeling software such as Blender, although it works in reverse of my comment above. It's great for free-form model building but not so much for engineering type construction. Again, a competent Blender user can avail himself of the parametric aspects of the software, but it's more work than one would necessarily desire to engage.

\n\n

SolidWorks has a number of followers in the 3d modeling community. My experience with the software has shown me that it can be learned fairly quickly and is powerful enough to allow one to explode creatively over time.

\n\n

The structure of Solidworks workflow fits in directly with your description, both to the reference to accurate modeling as well as the break-apart aspect. The construction of a model in SW is done via sketches and various actions performed on resultant objects created from those sketches. It's not free software, rather pricey, but it may also save you so much work as to pay for itself over time.

\n\n

There are other options, of course, some of which are free to students and hobbyists. I did a quick Google search for \"free parametric 3d modeling software\" which resulted in this link:

\n\n

https://www.matterhackers.com/articles/finding-the-right-3d-modeling-software-for-you

\n\n

You will find many suitable programs in the list generated from that link, as well as many that are unsuitable for the reasons mentioned previously.

\n\n

Consider also to view YouTube videos of your candidate selections. Most of the programs referenced will have tutorial videos which can give you a clearer idea of how well it fits your needs and perhaps how easily it can be understood.

\n" }, { "Id": "2710", "CreationDate": "2016-08-30T19:43:39.117", "Body": "

SOLVED: I replaced the leads coming from the fan motor and it is working just fine. Thanks for the input. If anyone else has this model, I would suggest printing and installing a wire clip in order to prevent the issue. Here is the one that I am using now: http://www.thingiverse.com/thing:620627

\n\n

I am running a Printrbot Simple Metal with a heated bed and have not been able to get my fan to run recently. The fan used to run just fine (for the past 3 months in fact--since purchase). I cannot be certain, but this may be related to the cord getting snagged by the hot-end last week (cables have begun to sag over time). Nothing melted or damaged except for the hard-plastic coil used to bundle all cables together, and that appears to just be minor, cosmetic damage.

\n\n

I have checked Cura settings, ensuring that fan operation is selected (starting at 0.6 mm) and that it is being told to run at 100%.

\n\n

I have also sent 'M106 S255' from Proterface UI (this, from what I have researched, should turn the fan on at 100%). This command had no results.

\n\n

The firmware is stock (Marlin), all exposed cables seem intact and without damage, and the plug to the control board seems to be fine. Maybe I could apply current to it directly and see if it runs?

\n\n

EDIT1: I put 2 6V lantern batteries in series (voltmeter shows 12.3V) and hooked it to the fan cable; the fan did not budge (even with encouragement in the form of a slight push from the hand). Is there a particular spot in the cable that is vulnerable to failing if pulled on?

\n\n

Any thoughts, suggestions, or things to look into?

\n\n

EDIT2: I have disassembled the cable bundle leading to the fan and hot-end. I have found a connection point where the fan plugs into an extension cable which goes to the control board. I have applied current here and the fan doesn't move. I tested the current on another 12V fan laying around and that fan runs just fine. So, it seems that the fan has gone bad...somehow. I am still quite confused on how this would happen. Is it possible for a DC fan to \"overheat\" due to poor airflow? I have a shroud on the fan (http://d17kynu4zpq5hy.cloudfront.net/igi/printrbot/CnoDPS5D1CZ5EgBR.standard) and there has always been a little back pressure pushing wind out of the back of the fan.

\n\n

EDIT3: So I have resorted to tearing into the fan and have applied 12V to the wires which are soldered directly to the fan and the fan doesn't work. I then applied the current directly to the solder points and the fan runs. Is there a reason why a set of wires, which look perfectly fine from the outside, would fail to run the fan?

\n", "Title": "Fan Fails to Start", "Tags": "|print-fan|", "Answer": "
\n

Is there a reason why a set of wires, which look perfectly fine from the outside, would fail to run the fan?

\n
\n\n

The internal conductor is broken. This can happen over time due to metal fatigue from the constant movement, and the cables getting snagged may indeed have something to do with it. You could try desoldering the current wires from the fan and replacing them with fresh ones, or getting a new fan altogether.

\n" }, { "Id": "2723", "CreationDate": "2016-09-02T10:30:22.413", "Body": "

Can the flexible TPU filament be used in the case of printing a dental fixture?

\n\n

Filament - SainSmart 1.75mm 1kg/2.2lb Flexible (TPU) Series Filament for 3D Printers RepRap.

\n", "Title": "TPU filament usage for dental application?", "Tags": "|3d-models|", "Answer": "

There are a Ton of issues with using this type of material. First off there is no way anything that comes out of your FDM printer will be sanitary enough to meet the requirements for medical applications. Your nozzle might even be contain Lead. Last you would need this printer to be cleaned to clean room standards. As well as stored and operated in one..

\n\n

Second I have not heard of a flexible material that meets FDA requirements. (which is not the same as does not) You have to worry about even the additives they add for color. Or straight up contamination of the material.

\n\n

I do want to mention that there are people who use SLA printers for dental items. I cannot say how SLA would meet food or medical grade requirements. My guess is a lack of regulation.

\n\n

So a bad idea I would say...

\n" }, { "Id": "2725", "CreationDate": "2016-09-03T08:24:23.257", "Body": "

Is there any forum where expert designers can help beginners like me?

\n\n

I am starting to design a panel for my car, and I am having some difficulties with finding a modeling program and using it.

\n", "Title": "3D printing forum community", "Tags": "|3d-design|", "Answer": "

See DesignSpark - DS Mechanical is good, free and fun to use.

\n" }, { "Id": "2729", "CreationDate": "2016-09-03T09:40:16.417", "Body": "

I am designing an part for my car. It is a panel for the rear door. I want to add it some cases for tools, but before going on, could you tell me:

\n\n

It is a large flat panel, around 100cm by 35cm.

\n\n

Is it going to be suitable for printing in that ABS material? Is it too big?

\n\n

\"enter

\n", "Title": "Is this very large flat panel printable in ABS?", "Tags": "|replacement-parts|", "Answer": "

ABS would be fine, as a material, for the application.

\n\n

The problem, given the scale, would be the printing device used. I would suggest that you have the piece cut to spec for you using a polycarbonate.

\n\n

If you have access to a workshop you could do so yourself if you set up proper fencing using a larger sized bandsaw(with a very fine toothed blade), had rollers to help support the size of your piece(as it would not fit on the tool's platform) and likely need a second set of hands to help control the outfeed. You would also need to drill access for the cut out then cut the perimetre.

\n\n

You could also consider printing(ABS) in smaller sections then fusing each section together edge to edge(solvent welding of ABS is very strong and permanent), if you were really dead set on using a typical 3D printer for this. It would, in theory, be possible, but it would be very tedious. The adhesives, and cleaner, would be available from any plumbing supply and/or (home depot, lowes, walmart et al.). If you go this route be sure to have ample ventilation and plan your work space well in advance. I would not recommend this option but it is \"possible\".

\n\n

Given the scale of the piece though, and for sake of accuracy, going to a shop with CNC equipment and using polycarbonate would be your best bet.

\n" }, { "Id": "2736", "CreationDate": "2016-09-05T12:08:17.990", "Body": "

Good morning everyone,

\n\n

I am developing a consulting job in a clinic of dental CT scans.

\n\n

This work involves the development of administrative software, and preparing a routine for conversion of tomographic files in DICOM format to STL format. The files in STL format will be used for both visualization and analysis of 3D models, such as printing in 3D printers.

\n\n

Our problem is just the conversion DICOM to STL.

\n\n

Has anyone come across this kind of situation? We did not find any documentation or tool for this purpose in our searches and we are really with a gande urgency in the solution.

\n\n

Advance grateful for any assistance.

\n", "Title": "Conversion of DICOM files to STL files", "Tags": "|3d-models|file-formats|", "Answer": "

This software may be of some use to you.

\n

Whilst looking into software/firmware for a Frankensbox FX-800, I came across this document For medical use - Dedicated series 3D printer, on the Dedibot website, which on page 4 states the following:

\n
\n

\"MIR

\n
\n

The relevant part (in text form) is this:

\n
\n

It can be used to read DICOM format CT/MRI/Micro CT/Micro MRI/Industrial CT images and non-DICOM common image formats BMP/TIFF and so on.

\n
\n" }, { "Id": "2750", "CreationDate": "2016-09-12T08:07:07.420", "Body": "

I have a file I want to print, but the problem is that the dimensions that came with the model are just too big to print. I used the Tape Measure Tool on SketchUp, and it came in at a whooping 5.56 meters. Is there a way to split the big model into smaller, printable sized models that can be reassembled after printing?

\n\n

Thanks Again

\n", "Title": "How to split a model in SketchUp for printing", "Tags": "|3d-models|resolution|", "Answer": "

SketchUp is likely to not work well for you if your objective is to create a printable STL file in pieces. You would want to determine initially that the un-segmented model is manifold and 3d printer ready. Once that is determined, consider using an alternate program for your chop-up actions.

\n\n

You do not specify the size of the printer you intend to use, but that is obviously a factor in the segmentation of the model.

\n\n

I have some experience with Meshmixer, which has an edit feature named \"plane cut\" which will do as you require. The plane is placed on the model and the options are selected to keep both pieces after the slice is performed.

\n\n

For a huge model of the type you describe, one would hope you have access to a printer with a one meter print bed or larger, although such printers are rare and of course, expensive.

\n\n

My printer has a 200 x 300 mm build plate, up to 200 mm high, which would entail substantial segmenting.

\n\n

If your model is mostly a shell, your segments would have to have wall thickness. It would be best accomplished, again in Meshmixer, by using the hollow or shell features. Those are a bit more challenging to understand, but there are plenty of videos and tutorials specific to creating hollow shells from a solid model. Once the shell is created, segments would be managed easily enough.

\n\n

I suggested that your segments would \"have to have\" wall thickness, but it isn't a requirement. Solid segments would be like bricks and you may have a large number of rectangular solids that compose your model. Lego models are difficult or impossible to create with only an outer shell, so the interiors are often solid bricks, but 3d printers are a bit more flexible. Not using solid interiors would save money, of course.

\n\n

I envision a jigsaw puzzle in three dimensions. Taking a nearly six meter dimension and chopping it into 20 slices multiplied by width and height means a carton of 3d puzzle pieces.

\n" }, { "Id": "2774", "CreationDate": "2016-09-14T22:37:01.057", "Body": "

I want to print people's bodies scanned (or get 'minimes', see the Figure 1). And I watched that the printer 'project 660' is very good to do this (Figure 1). But, the price of that printer is really expensive, roughly $60'000. And If you want to get a 'minime' printed by that printer, the cost is $230 for a model of 10cm. For me that price is expensive too.

\n\n

Does anyone know what printer I could use in order to print 'minimes' with:

\n\n\n\n

Thanks.

\n\n

Figure 1: Models printed

\n\n

\"Mimes

\n", "Title": "Which is the best 3D printer to print bodies scanned?", "Tags": "|3d-models|print-quality|", "Answer": "

There are several options for printers that can produce high-quality 'minimes' at a lower cost. One option is the Formlabs Form 3, which has a maximum build volume of 14.5 x 14.5 x 18.5\u00a0cm and can print in color. Another option is the Prusa i3 MK3S, which has a build volume of 25 x 21 x 20\u00a0cm and can also print in color. Both of these printers have good resolution and can handle complex models. Another option is the Creality CR-10, which has a larger build volume of 30 x 30 x 40\u00a0cm and can print in color. However, it may have a slightly lower resolution compared to the other two options. These printers range in price from \\$500 to \\$1000, making them more affordable than the 'project 660'.

\n" }, { "Id": "2776", "CreationDate": "2016-09-15T11:24:29.093", "Body": "

My 12V DC 30A Power Supply 360W Power Supply is really cheap, and it's worked well for setting up the motors; but now that I'm on to the heated bed, which uses considerably more Ampage than that of just the motors, I'll confess, I'm getting frightened to continue using it; if the summer was a bit longer, maybe it wouldn't bother me, but we're getting into the cold months, and now I'm afraid of ending up using too much ampage just trying to heat the bed in the winter months...(and I don't mean my bed). Is there anything I should look out for in terms of using the either the cheap power supply I already have, or are there certain specs on a new not-so-cheap power supply that I ought to be using instead?

\n", "Title": "Frightened of Cheap Chinese Power Supply as I hook up the heated bed", "Tags": "|prusa-i3-rework|switching-power-supply|", "Answer": "

I've used a similar cheap psu before. It'll work without blowing up but my heatbed struggled to get up to 60c, swapped psus with one I had lying around from a desktop and there was a huge difference.

\n" }, { "Id": "2778", "CreationDate": "2016-09-15T15:15:39.493", "Body": "

I'm building a 3D printer from scratch (My first one, so yeah, tall order), so I'm buying all the parts separately, now one of the things I bought was a 300x300 12v 25A Silicon heaterbed (Yeah, I know, should have gone for a higher voltage, but that's the only one I could get at the time)

\n\n

I have a RAMPS 1.4 board in all it's crappy MOSFET glory, so I tried giving my math skills a shot in trying to find a decent MOSFET that won't explode, though I'm still having issues deciding. And due to where I live, getting anything very specific is not the easiest thing to do.

\n\n

So can anyone recommend a MOSFET that I can use to switch 25A and mention if it will need a heatsink?

\n\n

I live in South Africa, So one place that I can get them from is from RABTRON Electronincs

\n\n

There are a couple other places, but I'd prefer to get them here if I can.

\n", "Title": "12v 25A 300x300 Heater bed Mosfet", "Tags": "|heated-bed|", "Answer": "

I have looked over the 20 or so most expensive MOSFETs in that shop, and none are suitable.

\n\n

It might be easier to go with a DC Solid State Relay rather than a bare MOSFET.

\n\n

If you do want to go with a bare MOSFET instead, a TO-220 package MOSFET can dissipate around 1W without a heatsink, and around 3-4W with one. For 25A of current, that means the resistance of the MOSFET can be at most 1.6m\u2126 (without heatsink) or 5.6m\u2126 (with heatsink).

\n\n

The IRLB3034 might be suitable. It is readily available on ebay (for under $1 shipped). 195A, 40V and an Rds(on) of only 1.4m\u2126. A caveat is that this resistance is specified at a gate voltage of 10V, whereas it is slightly higher at lower gate voltages (1.6m\u2126-2.0m\u2126 at 4.5V). This is on the upper edge of what is acceptable without a heatsink, but should still work (junction to ambient resistance is 62C/W, so in the worst case of 2.0m\u2126 the temperature would get to 77C above ambient - the MOSFET is rated for operation up to 175C). With a heatsink it will definitely work well.

\n\n

Note that the gate charge is much higher (10x) than that of the MOSFET used on RAMPs so you should use a low switching frequency.

\n" }, { "Id": "2789", "CreationDate": "2016-09-17T19:17:52.940", "Body": "

I have an M3D Micro 3D printer that printed fine for a couple of weeks and then was plagued with issues afterward. I've done the fixes from the forum to get proper heating and cooling of the nozzle (I've added aluminum foil around the nozzle to make sure the hotend is fit snug against the nozzle and I've added an external fan, powered externally, to compensate for heat creep).

\n\n

This works very well for short prints and it usually finishes successfully. When I do a longer print it always stops midway and usually at the same exact point.

\n\n

I tried printing at 200 \u00b0C with black PLA and then again at 215 \u00b0C with the same filament and it stops at the same exact point. I also tried M3D brand white filament. I am using CURA slicer with Octoprint GCODE sender and M3D Fio.

\n\n

I know it is not clogged because if I stop the print and press extrude without letting it cool down, it extrudes fine.

\n\n

What is causing my printer to stop printing?

\n", "Title": "M3D Micro printer stops extruding but is not clogged", "Tags": "|pla|ultimaker-cura|octoprint|", "Answer": "

I'm not sure how similar the two systems are, but I use a Stratasys uPrint SE Plus and I've run into a similar problem.

\n\n

There are two rollers in the head that pull the filament through to the extruder nozzle, and in one instance they appeared to have heated up, melted the filament enough to create two \"indentions\" on either side of the filament, making it such that the rollers had no purchase on the filament itself. There was never any clog, no material feed error, but it was still failing to print. Wound up having to replace the head altogether.

\n\n

Again, not sure how similar the extrusion mechanics are in the M3D, but suggest checking the components that actually advance the filament, and the filament itself.

\n" }, { "Id": "2790", "CreationDate": "2016-09-17T22:01:21.547", "Body": "

I've built the mechanics of my 3D printer myself, because I need to print parts that are really huge, (and for budget reasons). So, I already have the 3D movement functionality.

\n\n

But what I need now, is the printing mechanism itself. I've been reading a lot, but it became clear to me that things are more complicated than I thought.

\n\n

Let's skip mechanics and software, I'm just interested in how the print head works. Can somebody explain me that?

\n\n

To be honest, I was so naive that I thought that I just had to buy one part with one data wire (print/noprint) and the 5 V/GND wires. But it came to my intension that things are way more complicated.

\n\n

For example, these RepRap printers have some kind of air tube attached to the print head. I'm not sure what that's all about, is it cooling?

\n\n

Perhaps I'm always reading the wrong manuals (i.e. the more advanced ones). Can somebody enlighten me or point me to a good starting point?

\n", "Title": "What parts are strictly necessary for a 3D printer?", "Tags": "|printer-building|", "Answer": "

You will certainly find that the print functionality of a 3d printer is a bit more complex than you suggest. The mechanical portions include a means to push the filament into a heated nozzle as well as the software portion to regulate the speed of the filament movement. You haven't referenced the heater cartridge and temperature sensor, but you will discover that aspect soon enough.

\n\n

The \"air tube\" you think you've seen is likely called a bowden tube. Such designs permit lighter weight print heads, which is beneficial for speed, acceleration and precision, but has complications with respect to compression of the filament as well as retraction considerations. Non-bowden print heads will have the extruder motor as part of the moving assembly, with the drive wheels very close to the nozzle opening. This allows for flexible filament and more precise control of the filament feed.

\n\n

Either design has compromises, so one must determine priorities for the design.

\n\n

Cooling is also a factor. The heater cartridge is designed to heat the nozzle to a specific temperature for the type of filament used, but also requires a means to keep the heat from traveling to the portion of filament not in the nozzle. You'll discover terms such as heat break, referring to narrow threaded portion connecting the nozzle assembly to the heat sink. There will also be a cooling fan to blow air over the heat sink and very often a cooling fan to cool the filament as it exits the nozzle and attaches to the model being printed.

\n\n

You suggest to ignore the mechanics and software, but it's important to be aware of both when considering the principles of the print head assembly.

\n\n

Simplified, filament enters bowden tube then into heat sink, pushed by extruder motor (or) filament is pushed into heat sink by extruder motor. Filament travels through heat break, gets melted in heater block and exits nozzle. Sheesh, that's way too simple.

\n" }, { "Id": "2797", "CreationDate": "2016-09-18T23:08:15.470", "Body": "

I am making a part that needs to come apart, so I don't want to use glue or similar adhesives. I saw an article about printing snap-fit pieces, but it seems like you would need a lot of expertise to make it work well. Does anybody know of an easy way to non-permanently attach PLA parts, while still maintaining a sturdy connection?

\n\n

EDIT: (more detail about specific project)\nI need to attach plastic to plastic, in a pretty small surface area. This is a prop knife/sword in which the \"blade\" retracts into the handle. \"enter For the prototype I printed the handle in two halves lengthwise, which I do not want to do for the final product. I want to print as much of the handle in one piece as possible. Because I need to get the blade into the handle (and remove it for working on parts, painting, etc) I need part of the handle to detach. I am thinking that the front bit (examined more closely below) would be the best place to detach. \"enter As the wall thickness is about 1/4 in, there is not a whole lot of room to work with. However, I remain hopeful that there is a way to attach it such that it will remain securely together and retain the \"blade\" properly.

\n\n

Thanks for the help so far.

\n", "Title": "How to create attachable/detachable printed parts", "Tags": "|pla|", "Answer": "

This question may yet fall into the too-broad category, but I'll give it a shot.

\n\n

You don't want to use glue or adhesives, but does that exclude bolted or screwed fasteners? Threaded inserts are metal \"nuts\" with knurled grip extrusions that enable one to use a matching bolt. The insert is heated with a soldering iron or similar source and embedded into the part. It's especially useful when the part thickness provides sufficient strength and the threaded insert is on the surface of the part opposite the bolt head.

\n\n

Another option you have is to design into the part a \"conventional\" joint such as a dovetail or slot with the matching dovetail or T shape on the adjoining part. This requires access to the end of the part to enable the sliding action of joining them together, of course. Some testing of prints should be performed to ensure proper mating.

\n\n

Directly related to that method is pins and holes. With suitable testing, one would create a hole on both parts and print a pin to match. If one is well versed in the software of choice, one hole with a built-in pin on the adjoining part would work too.

\n\n

Yet another extension is provided free from 3DKitbash:\nhttp://www.3dkitbash.com/free3dmodels/\nin the form of friction fit pins. I've constructed one of their models using this design and find the straight-line pins eventually work free. The pin model with \"mouse ears\" allows for a bit stronger gripping due to the compression of the shape on insertion.

\n\n

The friction fit pins would be considered to be the snap-fit parts you want to avoid, but the level of expertise should not be beyond that of a model designer.

\n\n

Trial and error works wonders when it comes to putting these things together. I printed an Overwatch logo for a fellow makerspace member. The part used a pin on the insert and a hole on the logo. I was astonished when the two different colored parts slid together quite snugly. I have not calibrated my printer and it was indeed a snap together build. On the flip side, when I pried the two pieces apart (low infill) the pin remained in the hole, snapping cleanly off the insert. Should I be asked to do this again, I would have much more infill on the pins.

\n\n

For clarification, consider also to note what software you are using for your design. Some programs lend well to creating connections, others not so much.

\n" }, { "Id": "2805", "CreationDate": "2016-09-20T15:49:55.283", "Body": "

I recently finished building my first printer. The only problem that I'm having is that the hotend is not getting hot enough to start printing with PLA (180 to 230 degrees celsius), the hotend getting hotter stops at 170 degrees. Please help I've been stuck on this problem for days. Thanks in advance.

\n", "Title": "Prusa i3 hotend not getting to the temperature that I want (180 to 230 celsius)", "Tags": "|pla|prusa-i3|hotend|", "Answer": "

There are software limits that might be set low by default depending on the software you are using. I know there is with Marlin.

\n" }, { "Id": "2807", "CreationDate": "2016-09-20T20:02:48.670", "Body": "

I'm planning to print a certain sample holder which is going to be placed on a CD-Rom BLDC motor and spun at about 1000-1500 RPM. The holder would essentially be a 2-3 mm thick 100 mm square platform with 1-2 mm thick hooks rising up from outer edges of the platform to hold the sample. The bottom of this platform would also contain a 15 mm diameter crevice designed to latch onto the disc holder attached to the motor. How durable would this printed holder be under these conditions, if it was made of ABS plastic, for example?

\n", "Title": "Typical plastic strength at 1500 RPM", "Tags": "|abs|print-strength|stability|", "Answer": "

Really the only thing that would matter for this project is the amount of torque the motor has available and subsequently how heavy your setup is that is connected to the motor. A part that size may just be too heavy for a CD-ROM motor if you intend on adding more parts.

\n\n

However, to answer your question, ABS should be able to endure the stress. I recommend paying attention to how the hooks are printed. You'll want to make sure that the hooks are printed in profile, meaning that the profile of the hooks should be printed with each layer. This will help provide structural integrity to, what sounds like, the most stressful area of the part (the outer edges of a spinning device and a clamping feature).

\n\n

Something to keep in mind for projects like this is that most of the time the design will likely be the cause of failure, not the capabilities of the material.

\n" }, { "Id": "2809", "CreationDate": "2016-09-21T13:02:56.493", "Body": "

I have built a RepRap Prusa i2 a while back. It worked for a while, but then I moved the printer from place to place and after a while I noticed that the printer's bed is not heating anymore. The thermistor shows the temperature (room temperature), but the bed is not heating. \nThe wires are connected and the heated bed has a resistance.

\n\n

Did anyone else confronted with this? Thanks!

\n", "Title": "Heated bed is not heating anymore", "Tags": "|heated-bed|", "Answer": "

I have a Prusa i3 and the same case happened to me. However, only the power cable to the mainboard was bad.

\n\n

I cut it in order to solder a new one to the 4 pin connector but inside the cable sleeving, there wasn't any copper, only grey powder. With a new connector, and cables, it is working very well now.

\n" }, { "Id": "2810", "CreationDate": "2016-09-21T14:59:53.437", "Body": "

So, I'm having this problem where almost anything I print with a section(s) that is not directly connected to something below it breaks when I try to pull the small filaments meant to hold it up during printing off. For example, I 3D-printed a Rayquaza(this one) from Pokemon for my little brother, and as I was carefully pulling the filament from under the mouth, the whole head just snapped off. Does someone have a recommendation as to a way to get the small filament off without breaking the object? Would a solution just be to print it bigger and see if it holds up better, or is there something else I can do? Thanks.

\n", "Title": "How to stop objects with \"floating\" parts from breaking", "Tags": "|filament|", "Answer": "

To increase the strength of small parts I do:

\n\n\n" }, { "Id": "2817", "CreationDate": "2016-09-25T15:06:21.483", "Body": "

I'm trying to find the correct way to read and understand the dimensions of a 3D printer. For example, if I read the following dimensions for the Robo 3D R1 Plus as 10x9x8 Inch - I want to know what 10 stands for (does it mean print height?), 9 (depth?), and 8 (width?); etc. I'm not sure if I'm assuming correctly.

\n\n

I know this should be simple, but I'm not sure where to reference the proper ordering, respective to each aforementioned dimension.

\n\n



\nUPDATE\n

\n\n

While looking at Makerbot's Replicator technical specifications, I see them listing their dimensions as follows:

\n\n

Build Volume

\n\n
29.5 L X 19.5 W X 16.5 H CM\n[11.6 X 7.6 X 6.5 IN]\n
\n\n

So, if I'm reading this correctly, can I infer that 11.6 IN is referring to the length/depth, 7.6 IN is referring to the width, and 6.5 IN is referring to the height. Would this be the universal/standard way of referring to build specifications for all 3D printers?

\n", "Title": "How to properly read dimensions for a 3D printer?", "Tags": "|diy-3d-printer|printer-building|desktop-printer|", "Answer": "

Having the dimensions of your 3D printer ordered in Length x Width x Height is a common way, but nobody actually decided it.\nI think it has something to do with the 3rd dimension being the \"new\" dimension for printing things, that's why it's appended at the end.\nAdditionally, the first axis on a 2D-coordinate system usually is the horizontal one, so writing Length x Width makes sense for me.\nIf you are unsure you can always test your printer (if you have one) by writing your own G-Code that moves the printhead on a defined axis.

\n" }, { "Id": "2822", "CreationDate": "2016-09-29T14:11:18.670", "Body": "

My wife wants me to use an FFM 3d printer to make custom stamps for her to use on paper (scrap books, letters, etc.). She is convinced, however, that they will be too rigid to make good stamps. A quick google search showed ones made from PLA and ABS. Logically, though, a TPU or similar would address her concerns. A good quality stamp needs to hold ink and make good, even contact with the paper. It would probably need to be able to be sanded or smoothed in some way.

\n\n

I am supposed to receive my printer next week or so and am trying to get some filaments, STL files, and accessories I will need ready in advance so I can rapidly learn how to use it.

\n", "Title": "What type filament should I use to produce stamps?", "Tags": "|filament|", "Answer": "

I see three options...

\n\n

1. Print with a flexible filament:

\n\n\n\n

2. Print with any hard filament, but use a rubber mat under the paper.

\n\n\n\n

3. Print your stamp shape, then use it to make a rubber stamp.

\n\n\n" }, { "Id": "2823", "CreationDate": "2016-09-29T20:54:17.693", "Body": "

I know I should use different nozzles for filaments that use vastly different temperatures but what about filament types that use similar temperatures?

\n\n

For example, PLA and PLA composites like ColorFabb Woodfill filament. Should I use a separate nozzle for that?

\n", "Title": "Should I use different nozzles for different materials?", "Tags": "|nozzle|", "Answer": "

It depends on the size of the nozzle you are using. If you are using a small nozzle, e.g. 0.2 mm, normally, you should increase the nozzle diameter (filament manufacturers often refer to about 0.5 mm nozzle diameters). Wood/cork, or whatever particle filled filament requires a somewhat larger diameter to prevent clogging.

\n\n
\n\n

In addition to Tom's answer, note that there are stainless steel nozzles and hardened steel nozzles. The latter is harder than the prior and should be used for abrasive filaments, like filament filled with metal particles or some sort of abrasive fibres like glass or carbon fibre. Note that the addition of fibres or particles may require you to increase the nozzle diameter as fibres or particles may cause clogging. A last nozzle discussed is the nozzle with an embedded industrial Ruby, a design of Anders Olsson, a research engineer at Uppsala University, more information is found on the Olsson Ruby website.

\n\n

From the Olsson Ruby website, the following interesting image is shown, comparing the wear resistance of the various nozzle materials up to 0.5 mm (from left to right; brass, stainless steel and hardened steel by printing respectively 0.3 kg, 1.0 kg and 4 kg of carbon fibre filled filament):

\n\n

\"Nozzle

\n" }, { "Id": "2831", "CreationDate": "2016-10-03T05:21:07.127", "Body": "

I was very excited to get my 3d printer but sadly I didn't realise I'd installed the z axis upside down. There's a plastic coupler that goes into the hole for the polished rods it broke then released 1 -2mm sized ball bearing all over my floor.

\n\n

Is it possible to just replace that coupler thingy?

\n\n

Or if I 3d print a replacement will it still need that coupler thingy?

\n\n

\"enter

\n\n

\"enter

\n\n

The first picture is the one that spilled it's guts all over my kitchen floor.

\n\n

The last image is an example of how it used to look like but on the left hand side. I know the image looks like it's the right but you can see a bit of the horizontal rods if look closely

\n", "Title": "Hictop prusa I3 clone - broken z axis right bracket bearing", "Tags": "|diy-3d-printer|", "Answer": "

That \"plastic coupler thingy\" is actually a linear bearing, and it consists of a metal shell, and the metal balls roll between the smooth rod and this shell. The plastic part has a channel that guides the balls in a circle.

\n\n

It would be best to replace the entire bearing (metal part+balls+plastic liner). It's probably either an LM8UU (most likely) or LM8LUU bearing.

\n\n

3D Printing a new part will not give good results.

\n" }, { "Id": "2832", "CreationDate": "2016-10-03T14:11:16.033", "Body": "

I am wondering if making an hermetic box is feasible using 3D printer. The box would be a cube with a front face removable, with screw and sealing joint to close it.
\nI searched for different materials, however, none talks about hermiticity. (However, I found a product that seems to improve water resistance of 3D printed items here, which might be a starting point)

\n\n

Does anyone have experienced to make hermetic things ? I am specially interested in carbon fiber reinforced materials.

\n", "Title": "Is it possible to make a hermetic sealed 3D Printed case?", "Tags": "|3d-design|filament-choice|water-resistance|", "Answer": "

A few thoughts that might help...

\n\n

Material:

\n\n\n\n

Print Method:

\n\n\n\n

Sealants: This is probably your best bet.

\n\n\n\n

Design:

\n\n\n\n

I hope this helps. :-)

\n" }, { "Id": "2833", "CreationDate": "2016-10-03T15:01:39.083", "Body": "

I'm trying to print with Laybrick and for the most part it is going. The problem lies with the top layer and gaps appearing. I've tried increasing the number of top layers but the gaps still appear. Any ideas what else I can try?

\n\n

I'm using Simplify3d.\n\"Image\"

\n", "Title": "Laybrick - Gaps on top layer", "Tags": "|simplify3d|", "Answer": "

The solution was a combination of several items. The primary one was slowing down the top layer of the print significantly. I was using 3200 mm/min for the print and used the option 'solid fill underspeed' to slow down the top layers to 40%. I increased the top layers to 7. I also increased the infill, to ensure there was support in the tiny top pieces. I also decreased the minimum infill length to 0 to ensure the infill went in tiny places. Finally, I used the 'Concentric' external fill pattern.

\n\n

At some rotations of the model, I had slight gaps in the corners of the model, increasing the number of outline layers fixed it.

\n\n

I also decreased the temp to 160 compared to the initial print above.

\n\n

\"Castle

\n" }, { "Id": "2843", "CreationDate": "2016-10-05T15:20:36.943", "Body": "

I've a friend who is expecting. There are several adorable weapon themed rattles on thingiverse. I am, however, concerned about safety associated with such a product in the hands of a baby who will gnaw on it. To me the safety concerns here are much larger than for most food handling applications.

\n\n

What steps should be taken to ensure such a print is safe for use?

\n\n

This includes: filament selection, pea material selection, wall thickness, smoothing, construction, etc.

\n\n

My current thoughts are as follows: \"food grade PLA\", dried peas, sanding, and single piece construction

\n", "Title": "3D printing related safety precautions for a baby rattle", "Tags": "|print-material|filament-choice|", "Answer": "

Also consider the nozzle on your printer. Most nozzles are made of brass, which is not considered food safe due to the presence of small amounts of lead. Stainless steel nozzles are available which will not perform quite as well as brass but are food safe.

\n" }, { "Id": "2845", "CreationDate": "2016-10-05T19:58:12.607", "Body": "

I 3d-printed a key.\n\"enter\n\"enter\nWhen I put it in the lock the pins move but, when I go to turn it the key rips and the lock doesn't turn. I compared it to the real key and it's identical. Is there something I can do to make it work and not rip? Is there some filament that I can print it out of? I used ABS.

\n", "Title": "3d Key not strong enough", "Tags": "|filament|", "Answer": "

I must admit, I've never printed a key...but I think I can help anyway:

\n\n

Print method:\nConsider printing on side, solid concentric infill. Or, if you can't manipulate your infill pattern, just increase the perimeter so you get the same effect, several continuous perimeter layers around the outline of the key.

\n\n

Print material:

\n\n\n" }, { "Id": "2848", "CreationDate": "2016-10-06T16:05:39.450", "Body": "

I have a model I would like to print, specifically it is a free-for-personal-use model of a car, complete with interior.

\n\n

I would like to print it, but it is unprintable. I have tried to fix it in netfabb, which works OK. But now I realised I have another problem - the inside of the car is hollow, and has a full interior. I just want the shell of the car and a solid block (complete with opaque 'windows').

\n\n

How would I take a detailed model and somehow extract a solid 3D printable model from it?

\n", "Title": "Take a detailed model and make it suitable for 3D printing?", "Tags": "|3d-models|", "Answer": "

I would recommend \"The Maker's Muse\" youtube video channel.

\n\n

In particular this video Fixing impossible STL's with Meshmixer 3.1.118 BETA might solve your problem.

\n\n

Few other videos which might help:

\n\n\n" }, { "Id": "2849", "CreationDate": "2016-10-06T18:05:35.457", "Body": "

What is the 3D printing technology that allow to have minimal loss in precious metals when 3D printing jewels?

\n", "Title": "Can I print jewelry without any loss of the precious metal?", "Tags": "|metal-parts|", "Answer": "

The jewelry industry typically uses printers that print in wax, and transform the models into precious metal by lost wax casting. Statasys offers wax printing in their Solidscape line, 3D systems offers ProJet. With this process there is basically no waste, since you can remelt the casting sprues.

\n\n

I am not aware of printers that print directly in precious metal. It might technically be possible with DMLS but you have to fill up the entire machine with metal powder, which is cost prohibitive (and potentially more wasteful).

\n" }, { "Id": "2854", "CreationDate": "2016-10-07T06:11:42.397", "Body": "

On my Reprap-like 3D printer, I routed all the wires to a spot near the base; for the motors, endstops, thermistors, etc, I plugged them all into a DB25 breakout board, and that's working great.

\n\n

For my Extruder (12v), and my heat bed (16v), I'm using a 4-pin molex connector -- like we used to use for old hard drives and such, and it's working, but it gets a bit hot, and my print bed is having trouble reaching temperatures that it didn't have trouble with before. -- None of the wiring gets hot at any other point, and none of the other connectors get hot. -- The only thing that gets hot is this one molex connector.

\n\n

I believe the heat is caused by passing too much current through that molex connector. -- I'm curious to know what other connectors might be better suited to this task. Can you recommend something that's worked well for you, or others?

\n\n

I prefer not to double or triple up this connector if it can be avoided, so that I can just have one physical connector to disconnect there, and so that I don't end up mixing them up. -- also, being able to easily disconnect it is important (it's hard to work on the reprap if I can't spin it around or turn it over, etc., that's why I'm using connectors for everything).

\n", "Title": "Connector Suggestion for Extruder and Heatbed", "Tags": "|reprap|wiring|", "Answer": "

I've had great luck with \"G16 aviation connectors\" -- easy to find with a Web search, and they come in any number of pins from 2-10 (the 10s are a little harder to find). Round (so easy to panel-mount), metal barrels, locking ring so they don't come loose. If you get ones with a couple extra pins, you can double up on pins without having to double up on actual connectors. I used a 4-pin for the hot bed (2 power, 2 thermistor); 10-pin for the extruders (4 motor, 2 heater, 2 thermistor, 2 fan), and 8-pin for each axis (4 motor, 4 limit).

\n" }, { "Id": "2862", "CreationDate": "2016-10-08T05:46:41.747", "Body": "

This is something I've ignored for awhile, but it keeps cropping up, and occasionally, if the filament snags real good, it can mess up a great print half way through, etc.

\n\n

Basically I have my filament spools hanging on a PVC pipe mounted directly above the printer, and the printer just sucks it in from the spool, but occasionally it becomes knotted at the spool, or becomes otherwise tangled.

\n\n

I'm thinking even if I completely unwrapped and rewrapped all of my spools so there was no trace of a tangle, what's to stop it from happening again? -- What's a good strategy for managing this kind of issue?

\n\n

Edit: From some of the answers below, it seems that maybe the travel of my X carriage back and forth could be why my spools keep getting tangled. -- It looks like many people have their spools at about 90 degrees from mine (rotated relative to the Z axis of the printer), so that the travel of the X carriage won't have that affect. -- Also guides, etc are probably a good idea.

\n\n

Here's my current setup for reference (filament removed):\n\"enter

\n", "Title": "Keeping my filament spools from getting knotted?", "Tags": "|filament|", "Answer": "

I used this solution, it makes a lot of sense, because I store the filament back in a box and the tip endup getting knotted.

\n

\r\n \r\n

\n" }, { "Id": "2864", "CreationDate": "2016-10-08T10:04:48.197", "Body": "

Repetier host 1.6.2.\nUsed Slic3r and CuraEngine in RH1.6.2 to slice my prints.

\n

OK, I have a problem. Z offset does not work. Well...

\n
    \n
  1. Flashed EEPROM clear.
  2. \n
  3. Enabled EEPROM and CHIT CHAT within the firmware.
  4. \n
  5. I've set -0.4 in Marlin firmware 1.0.2-1 stable and flashed it.
  6. \n
\n

results in

\n
    G28; home axis = works\n    G29; auto level = works\n    G28; it goes to home and then sets Z to 0.4\n    G1 Z0; brings nozzle down from 0.4 to 0, so this works also.\n
\n

But when I slice with Repetier host 1.6.2 with the latest slicer/curaengine, it does not go down 0.4 mm before starting to print.\nI've set the first layer to 0.2\u00a0mm, but when it starts printing the first layer, it goes from 0.4\u00a0mm (after home Z is on 0.4\u00a0mm) to 0.6\u00a0mm instead of 0.2\u00a0mm! ...

\n

How do I fix this?

\n

Gcode start:

\n
    ;Generated with Cura_SteamEngine 15.01\n    ; Default start code\n    G28 ; Home extruder\n    G29 ; Auto level\n    G1 Z15 F100\n    M107 ; Turn off fan\n    G90 ; Absolute positioning\n    M82 ; Extruder in absolute mode\n    M190 S35\n    ; Activate all used extruder\n    M104 T0 S230\n    G92 E0 ; Reset extruder position\n    ; Wait for all used extruders to reach temperature\n    M109 T0 S230\n    ;Layer count: 226\n
\n

It's definitely Repetier or its slicers. I'm using Cura 15.04.6 and it also calculates the offset set in Marlin when printing! I'm trying to use Cura 2.1 because it's newer, but I do not get the print USB option in Cura 2.1 :/

\n

I just don't get it, it was printing very nice the first 4 layers, and suddenly it moved up more than 0.5\u00a0mm and continued printing there, in the air.

\n

Yesterday's print:

\n

\"Photo

\n

\"Photo

\n

Today's print:

\n

\"Photo

\n", "Title": "Z offset incorrect", "Tags": "|prusa-i3|z-probe|", "Answer": "

Your question is a little hard to pin down, but I'll try to help anyway. :-)

\n\n

Z-Offset:

\n\n\n\n

Print Quality and Skipped layer issues:

\n\n

From your pictures, you do have some loose or wobbly belts, pulleys, rails, or other mechanical components causing excessive play in your x and y axis...very apparent with the way your layers don't align well resulting in the very visible lines. You can address this in two ways:

\n\n\n\n

You mentioned skipped layers. I see that in your tower. This is most likely caused by:

\n\n\n\n

However, it could also be caused by issues with your filament feeding or your hot end. Clogs, snags, etc. If this were the case, your previews in Cura would all be fine. To solve this you could:

\n\n\n\n

I hope this helps. :-)

\n" }, { "Id": "2867", "CreationDate": "2016-10-09T05:01:43.607", "Body": "

I've recently upgraded my Replicator 2 with a heatbed and the more powerful power brick (9.2A). The problem I'm having right now is that the Replicator restarts right after the preheat of the bedplate finishes. It does not matter which temperature I set as threshold - it restarts either way.

\n\n

So it restarts even in very low temperatures on the HBP. Tried upgrading the firmware and even switch firmware to Sailfish, but it's still the same issue. I would really love if someone could help me with this.

\n\n

Edit: I've checked my cables and it seems okay. Themistor is connected via pin 1 and 3 and the power should be fine.

\n\n

Edit2: Found the problem. It was a faulty hotend that caused all of this. It works to print with , but togheter with the heated bed it makes the Replicator to restart.

\n", "Title": "Makerbot Replicator 2 restarts after preheating the hbp", "Tags": "|heated-bed|makerbot|", "Answer": "

Clues so far:

\n\n\n\n

My Assumptions

\n\n\n\n

Potential Problems: Listed starting with the easiest to fix...

\n\n\n\n

I hope this helps. For further reference, read this thread as I believe it has some relevant info. :-)

\n" }, { "Id": "2880", "CreationDate": "2016-10-12T06:31:51.507", "Body": "

I'm looking for any idea of what could cause this problem. I'm printing (1.75\u00a0mm PLA @ 220\u00a0\u00b0C) a 14 x 14\u00a0cm box, sliced with Simplify3D. Relevant settings are 3 bottom layers, 3 top layers, 3 outline/perimeter shells, and 15\u00a0% orthogonal infill every other layer.

\n

The first 3 layers print fine. Here's the first (bottom) layer after removing the print and turning it over:\n\"bottom

\n

The infill (layers 4-10) also prints beautifully (see left side of photo below).

\n

But the moment it starts printing the next layer (layer 11, the top layer of the bottom of the box), which should be solid fill exactly like layers 1-3, it starts underextruding and generally looking like crap:

\n

\"Not

\n

The first time this happened, I figured the nozzle got clogged or the extruder gear started slipping. This is not the freshest PLA, so maybe it was a PLA quality problem. But the same thing happened at the same layer when I tried it again. And again. And again. As soon as I cancel the print I can have it extrude 5\u00a0cm of filament and it's fine, no clogs...and if I immediately start another print it again perfectly prints layers 1-10. So it doesn't seem to be the extruder, the nozzle, or PLA quality. I can't imagine the "stress" of laying 7 layers of infill could screw up the next layer...

\n

I just can't figure out how layers 2 and 3 could be basically perfect, but layer 11 is consistently a disaster, when they should be almost the exact same G-code (only a milimeter apart). I looked at the G-code and it's basically identical for layers 3 and 11, including same feedrate (G1 F2250).

\n

This is on a DeltaMaker printer with a new E3D Lite6 hot end. Can add more details if needed, but basically I'm just looking for an idea of what could cause this.

\n

Update: Just tried a different roll of PLA and got the same results.

\n", "Title": "Why does print fall apart at beginning of top layer?", "Tags": "|print-quality|simplify3d|deltamaker|", "Answer": "

Two things. First for a first layer from infill this is not unexpect. This is why you do 3-4 shells with such a sparse infill.

\n\n

Most of the time we also have the first layer at a slower speed. Often we will get to this point and the difference in flow can be contributed to the temp to speed ratio. Try increasing your temp, or decreasing your solid layer speeds.

\n\n

I recommend starting with a basic calibration STL. You can find them on thingiverse. Start with single wall, upgrade to a square, hollow square, etc.

\n\n

Also increasing either number of top shells and or infill percent will also help.

\n\n

3D printing is all about balancing 20 settings. I think of it like a performer spinning multiple plates. That said all about baby steps.

\n\n

But to answer your question. Ignoring that this is exactly what I would expect even from a well calibrated printer on the first layer post infill.\nlayer 1-3 is that the first layer is very slow. The extruder has built up a backlog of melted plastic. Assuming you are running too cold or too slow for some different types of printing (infill, solid, bridging, solid infill) you will slowly run out. Then suddenly you hit this solid layer and it needs a LOT fast. But it can only melt plastic 90% of what you need. Then less will come out. A good sign this is happening is if there is a lot of plastic shavings by the geared extruder. (if it can't melt then it stalls and the extruder shaves the plastic)

\n" }, { "Id": "2892", "CreationDate": "2016-10-14T00:54:25.423", "Body": "

I have a QIDI Tech 1. It has a heated bed, and a cooling fan attachment. Whenever I print without a raft, the first inch or two of material laid down does not adhere to the bed, but the rest of the first layer is flawless.

\n\n

I have tried speeding up and slowing down the first layer walls, but the problem remains. It also seemed to get a little worse when slower. I also tried not turning on the cooling fan for a bit to see if maybe the material was cooling too quickly, but that had zero effect on it.

\n\n

I'd like to avoid using tape and other methods since the rest of the print is perfect, and the bed already has a material on it to aid adhesion.

\n\n

What else can I try to prevent the dragging for the start of the print?

\n", "Title": "Poor adhesion only for first couple of inches", "Tags": "|pla|adhesion|", "Answer": "

I'd recommend using the \"skirt\" function if you're not already.

\n\n

The idea is to print a few perimeter layers around where your part will be, but not actually touching your part.

\n\n

Most slicers support this and you can choose how much skirt you want to print. This addresses the issue you mention, and it purges old filament that has spent too much time in the nozzle. As an added bonus, it gives you a good indication that your print location, print height, and first layer adhesion are all good.

\n" }, { "Id": "2900", "CreationDate": "2016-10-16T19:29:07.013", "Body": "

can PLA be used to print out containers or other parts that are in direct contact with gasoline, diesel or other hard chemical substances? Will it start degrading when in contact with said chemicals? Should I use ABS for this?

\n", "Title": "is PLA gasoline, diesel or chemical resistant?", "Tags": "|pla|abs|print-material|", "Answer": "

I have used both ABS and PLA for diesel gas caps and oil plugs. As well have made a gasoline funnel out of both. Both have done well for me.
\nI have attach a link to a plastics resistance chart if it is any help. Unfortunately it does not show testing for ABS for all.

\n\n

I prefer the ABS only do to the fact some items I have made sit in the desert sun.

\n\n

Plastic resistance chart

\n" }, { "Id": "2903", "CreationDate": "2016-10-17T05:36:23.760", "Body": "

I am planning on building a large enclosed (Cartesian XY-Head) 3d printer. I want to keep the inital build time minimal and it is very likely I would need to build the ~50x50cm heatbed from scratch, because it doesn't exist in ebay.\nThe enclosure itself is not separately heated, but depends on waste heat from the printing process. The enclosure will eventually be vented outdoors with a 12V CPU vent via ducting (air flow unknown).

\n\n

Do I even need a heated print bed, when I am planning to only print PLA?

\n", "Title": "Do I need a heated bed for printing PLA in an enclosed printer?", "Tags": "|pla|build-plate|heated-bed|enclosure|", "Answer": "

The answer is \"No\" you don't need a heated bed for PLA but it does make the base layer a little easier to lay down and also helps with print removal post print.

\n\n

PLA is a very easy filament to work with and the majority of PLA printers don't come with heated beds and suggest blue tape and/or elmers glue.

\n\n

You may find that if you are purchasing very cheap/poorly extruded PLA, that things don't stick as well and a heated bed can alleviate some of these issues.

\n" }, { "Id": "2904", "CreationDate": "2016-10-17T06:18:24.220", "Body": "

There is this great hotend called a diamond hotend, which can be used to print in 3 colors and mix them into hundreds? of colors. This can for example be used with Red, Green and Blue filament to mix a RGB palette. They don't have to be these colors, but I believe RGB would give the maximum range of colors when constrained to 3.

\n\n

However true RGB in physical printing would use separate colored voxels to create the appearance of a color, just like monitors display colors. As far as I know only HP Jet Fusion 3D printer uses this process, but it uses a process vastly different from normal diy 3D FDM printers.

\n\n

CMYK is mixed physically like you would mix watercolors together to make new colors. It is used for printing on paper by all laserjet and inkjet printers (and in printing presses).\nSo that means even the 3 input diamond hotend is actually mixed like CMYK. Repetier firmware v92.9 has this built in with support up to 16 inputs for a nozzle, but Marlin firmware v1.0.x only supports 4 inputs per nozzle at this time.

\n\n

Using RGB for the 3 inputs of a hotend, means the printing color palette lacks White and also it seems that CMYK would give a bigger range of colors. That brings our tally to 4 inputs. It still needs a white filament to print white, so that means 5 inputs. And while we are at it, probably a 6th input would be useful: like for printing black infill (to save using CMYK to mix into black) or for using transparent filament or elastic filament.

\n\n

So why isn't there a nozzle with 5-6 inputs already? Could it be done? Are there such hotends already?

\n\n

P.S These are just theoretical assumptions. I just discovered 3D printing and I am in the planning phase of building my first 3D printer, so I am a total n00b in this. Please correct any assumptions I got wrong.

\n", "Title": "true color printing with CMYK+White (not 3-in-1 diamond head RGB)", "Tags": "|printer-building|hotend|nozzle|extrusion|color|", "Answer": "

So whilst this is not quite what you were talking about I think this is the closest to the effect you\u2019re looking for that I\u2019ve seen.

\n\n

https://www.xyzprinting.com/en-US/product/da-vinci-color

\n\n

This da Vinci printer uses a CMYK inkjet to colour a white filament as its being extruded. This allows for very quick changes in colour as well a nearly infinite combination of colours.

\n\n

This means it only needs a single extruder and hotend but obviously the hotend needs to be built to allow the injection of pigment from the inkjet.

\n" }, { "Id": "2905", "CreationDate": "2016-10-17T09:15:49.573", "Body": "

I have wired in a new extruder with its own cooling fan and I tried to change the Marlin firmware to switch it on automatically above a specific temperature (50 \u00b0C).\nI did noting in the Configuration.h\nI changed a line in the Configuration_adv.h

\n\n

from

\n\n
#define EXTRUDER_0_AUTO_FAN_PIN   -1\n
\n\n

to

\n\n
#define EXTRUDER_0_AUTO_FAN_PIN   8\n
\n\n

as described in this article.\nBut the fan is not starting when I bring the temperature above 50 \u00b0C (manually). However it starts if I switch the fan on manually from Repetier.\nAm I missing something? How do I control the fan automatically on FAN0 output when the temperature reached the set limit?

\n", "Title": "Marlin (on RUMBA board) switch extruder fan automatically", "Tags": "|marlin|cooling|fans|", "Answer": "

The RUMBA board has two fan outputs. The primary fan output is switched by pin 7, the secondary one by pin 8. The reason your fan is not working when you set #define EXTRUDER_0_AUTO_FAN_PIN 8 is because it's switching the secondary fan. If you switch the wires over to the secondary fan output it will work correctly.

\n\n

If you do want to use the primary fan output then you can achieve this by setting #define EXTRUDER_0_AUTO_FAN_PIN 7 but you will need to search for the #define FAN 8 in the appropriate pins.h file and change 8 to something else (-1 or 7, for example).

\n\n

I would recommend the first solution, because you should have a controllable print cooling fan, and an automatic extruder cooling fan.

\n" }, { "Id": "2909", "CreationDate": "2016-10-17T15:24:18.113", "Body": "

If the hot-end is at ~0 on the Z axis and I go to level it, as it approaches X = 0 it begins scraping along the bed, then when moving back to probe the center of the bed (after homing X to 0) it will scrape the bed again.

\n\n

I want to add a glass plate but am worried this aggressive homing will smash the glass. How can I fix it? The printer auto-levels and runs Marlin.

\n", "Title": "When leveling hotend always slams into bed?", "Tags": "|marlin|", "Answer": "

You can always add to the starting code as told in an answer above. Also for the scrapping, you might want to set Z_HOMING_HEIGHT to prevent further damage to the hotend. If the Z axis is below the homing height, on homing x or y will first cause z axis to travel to the homing height and then proceed with homing of x or y axis.

\n" }, { "Id": "2913", "CreationDate": "2016-10-17T20:24:48.030", "Body": "

I'm working on designing a 3d printer with a team. The team is using marlin open source software and I started half way into project trying to make sense of things.

\n\n

Apparently there is an issue with the wheels moving the print head and it always moves a few mm short in the of the desired position. Can some one tell me which part of the marlin code interprets the G-code file and controls movement? I'm thinking of increasing the distance in the x and y axis to correct this (seemingly) minor problem.

\n", "Title": "Editing arc in Marlin for 3d printing", "Tags": "|marlin|g-code|", "Answer": "

Since your team is designing a printer, you'll have to do some basic calculations to figure out your x and y axis steps per millimeter and then set them in your firmware.

\n\n

For a decent primer in the basic idea, this video is a good start. For another decent getting started summary check this blog post

\n\n

Also, since you're using Marlin, become very familiar with the Marlin Firmware and how to configure it. The setting you want to change exists in the configuration.h file. Documentation is included in the file, but you can find good information here in the Marlin Configuration instructions within the Marlin Firmware Github repo.

\n\n

I hope this helps! :-)

\n" }, { "Id": "2916", "CreationDate": "2016-10-18T11:21:17.437", "Body": "

I have my nozzle close to the substrate that I am printing on, so that a piece of paper can just about slide underneath it freely, without catching.

\n\n

Is this the right way to do it?

\n", "Title": "How close should the nozzle be to the substrate?", "Tags": "|nozzle|", "Answer": "

This photo isn't exact, but may help

\n\n

Edit: Whoops! Forgot to include source. This is from the Wanhao User Manual/Build Guide. I can't quite find the webpage at the moment.

\n\n

\"enter

\n" }, { "Id": "2921", "CreationDate": "2016-10-18T22:32:58.637", "Body": "

What are the methods to auto eject parts (into a collection area/box/basket) in order for the 3D printer to continue printing?

\n\n

For some reason this feature isn't common (yet?). Is there a hidden reason why?

\n\n

Will using the print head to ram the part off the build plate into a basket nearby cause the print head to misalign (if using belts).

\n\n

I am planning to use a Cartesian XY-Head type (like CoreXY) printer, where the build plate moves along the Z axis and XY axes are on the ceiling of the printer using belts to move the print head.

\n", "Title": "Printed part auto-eject (automatic part removal)", "Tags": "|printer-building|build-plate|", "Answer": "

Here's a wild idea. Since you're planning to have a bed which only moves in the Z-axis, build a roller system which feeds a thin, flexible layer of some material (unobtanium, I fear) to cover the build plate. The feed rollers have a weak spring to provide some tension to keep this layer flat but not overly stretched while building the part. When the part is done, lower the bed to zero while allowing the roller to retract excess parts of the sheet (rollers on both sides of the build plate will have to do this). At zero, lock down the rollers and move the bed to some negative Z position, causing the overlay sheet to pop free of the bed and ideally pop the printed parts loose as well. \"demo

\n\n

Hey, it \"could\" work!

\n" }, { "Id": "2929", "CreationDate": "2016-10-19T07:51:10.950", "Body": "

With FFF printers able to manage only a small handful different colours (by using different filaments & extruders), how is it some SLS prints are able to be produced in such a broad range of colours?

\n\n

Are they sprayed post-production?

\n", "Title": "How are SLS printers able to print multiple colours?", "Tags": "|sls|color|", "Answer": "

The Z-corp/3D systems printers lay down what is essentially ink in each layer (only around the perimeters) much like an inkjet printer, dying the powder as the parts are made. This means they can make almost any color at any point in the model. The down size is these models are pretty fragile, at least the last ones that I have handled. This can be helped by dipping them in cyanoacrylate and letting them dry.

\n\n

As pointed out in the comments, this is not an SLS process, but looks very similar. The printer lays down a binder (clear or colored) on each layer, and is why these models are much more fragile than SLS models, which are very strong.

\n\n

Here is an example of some prints: http://mcad3dprintingandprototyping.blogspot.com/

\n" }, { "Id": "2931", "CreationDate": "2016-10-19T09:02:57.463", "Body": "

I did calibrate the extruder to extrude exactly 50/100 mm and it is fine.\nI have replaced the old (prehistorical) extruder that was giving me the problem with a new one.\nThe issue does not go away. It is severely under extruded.\nThe nozzle is a 0.4 mm, if I extrude manually the extrusion is nice and clean but when printing its a mess.\nI have the following setting in Slic3:

\n\n\n\n

In Marlin I have the following setting for the extruder:

\n\n\n\n

How can I solve this problem?

\n\n

\"enter

\n\n

This is a 20 mm cube I stopped after 15 layers!

\n\n

Here is another 20 mm cube, the dimensions are perfect but is absolutely a mess.\n\"enter

\n", "Title": "I am experiencing some severe under extrusion", "Tags": "|print-quality|extrusion|", "Answer": "

Short addition to the already great answers:

\n\n

Check whether the gear wheel on your extruder motor axle is spinning with the motor. It can seem to be quite fixed, but when under load the motor is turning nicely but the gear slips on its axle.

\n" }, { "Id": "2935", "CreationDate": "2016-10-19T18:31:17.103", "Body": "

The local stores have started to stop selling filaments in the smaller 0.5 kg spools, and it's getting harder to maintain an assortment without buying lots of big spools.

\n\n

I mainly just want a bunch of different colors (red, black, blue, green, etc.), ideally as a bunch of small spool \"samples\" or like a \"sampler pack\". -- Even if they're at .25 kg spool sizes... -- I don't want to spend a ton of money on buying and storing a ton of 1.0kg - 2.0kg spools.

\n\n

I prefer to print in PLA, but ABS is fine. I can only print 1.75mm filament.

\n\n

The seller should to be able to ship to western US, and have filament of decent quality that's not going to ruin my printer.

\n\n

What do you recommend?

\n", "Title": "Good sources of filament \"sampler\" pack?", "Tags": "|filament|", "Answer": "

Amazon, of course. I found a pack of 20 colors, maybe 50g each or so, 1.75 PLA. (that link is direct to this product).

\n\n

edit:

\n\n

Well, dang, I blinked at that particular item is off the list. Here are two other multicolor packages currently available (2PM EDT 20 Oct 2016)

\n\n

one , and ... two

\n" }, { "Id": "2943", "CreationDate": "2016-10-20T22:15:52.923", "Body": "

First off - I'm rather new to this and I might be on the wrong track altogether...

\n\n

I'm printing PLA using a simple DIY XYZ printer with a direct-driven extruder, 0.3 mm nozzle, 0.2 mm layer heights, using Slic3r. I've tried to calibrate the bed as level as I can. I believe the distance to be OK because the skirt prints in straight lines with no wobble.

\n\n

While the overall results aren't that bad, I'm trying to improve the quality. I'm not happy with the first layer: While it sticks to the bed nicely, the printed strands are too far apart - there's a very noticeable gap between them that I can actually see the second layer through. There appears to be a problem with the second layer as well which always ends up too small (recessed). That's only the second layer, though - from the third layer on, that problem disappears.

\n\n

What I've tried so far:

\n\n\n\n

Now - is this the right way to fix this or am I just patching over an entirely different problem that I just failed to diagnose properly? If it is the right way, how can I tell Slic3r to either change the flow rate / extrusion multiplier just for the first layer or increase the extrusion width without spacing the strands further apart?

\n", "Title": "First (bottom) layer has gaps", "Tags": "|print-quality|slic3r|", "Answer": "

You can, in most slicers, set a separate extrusion multiplier for the first layer. This doesn't appear to be possible in slic3r, but there are a couple of ways to work around this:

\n\n\n" }, { "Id": "2945", "CreationDate": "2016-10-21T07:13:56.203", "Body": "

I had a problem with my Z limit switch bracket falling just short of the bottom edge of the z stage.

\n\n

I'm trying to make the bracket thicker so it's pushed more towards the left.

\n\n

1) how do I measure the thickness of the bracket in the stl\n2) HOw would I make it thicker if it is indeed too thin

\n\n

\"enter

\n\n

It's might to be mounted standing up right.

\n\n

I want to make it thicker x axis (if it's stood up)

\n", "Title": "stl files -measuring stl files", "Tags": "|3d-models|", "Answer": "

If you have the original CAD file this would be far easier to modify and maintain then an STL.

\n\n

MeshLab also has the capability to measure the view on the screen as well as bounding box measurements. Bounding box measurements should be possible in any pre-processor you use for slicing STL files though.

\n\n

MeshLab can also be used to scale, merge or translate your STL if desired. however I would also recommend mesh mixer as it is more user-friendly.

\n\n

The last option if you have any coding experience, an STL file can be easily read and then you could select the desired vertices and measure/move them and then rewrite out the STL file. There are libraries for most major coding languages for dealing with STL files.

\n" }, { "Id": "2951", "CreationDate": "2016-10-22T09:17:56.033", "Body": "

I am trying to use my M3D printer that doesn't work with my Linux computer. I have been told that OctoPi would make it work.

\n\n

My current setup is:

\n\n\n\n

I don't know anything about OctoPi, but it seems to be created to use as a print server connected by WiFi.

\n\n

My question is: how can I use it locally, directly on my Raspberry?

\n\n

NOTE: I can only use my Raspberry in command line for the moment, but if required I can install lightdm to display stuff.

\n", "Title": "Can I use OctoPi locally?", "Tags": "|octoprint|raspberry-pi|", "Answer": "

To use it locally you need to be able to view the desktop.
\n
sudo raspi-config to pull up the config menu and enable boot to desktop.

\nIf there is no browser installed already you'll need to install one.\n

sudo apt-get update
\nsudo apt-get install epiphany-browser

\n\n

Open the browser and navigate to http://localhost/

\n\n

I recommend configuring access control when it prompts you. After you configure it, click login in the top right and login to use octopi.\n

\nALTERNATIVE\n
\noctocmd is a command line interface for octoprint. If you would rather not boot into the desktop then this is the way to go.

\n" }, { "Id": "2953", "CreationDate": "2016-10-22T11:58:29.533", "Body": "

I have a question. Home printers are controlled via open-source software such as Pronterface. These printers print mostly in plastic, but how to control the \"metal\" printer. The principle of the printing here is a little different. Is it possible to use, for example, Pronterface for this task or the completely different software is needed?

\n", "Title": "How to control 3d printer printing in metal?", "Tags": "|metal-printing|", "Answer": "

Pronterface would control the printer, but you would need a slicer that could give you g-code that works with said metal printer. Being it would be using metal, most of those are powder based, using a laser to sinter. The slicers for FDM based machines would not create the correct g-code for the application. I know there has been some effort around powder based printers (both metal and plastic) but I do not know of any software that has resulted from these efforts.

\n\n

This wiki index for powder printers may be of some help to find out what software they are working on, most likely a custom solution.\nhttp://reprap.org/wiki/Powder

\n" }, { "Id": "2964", "CreationDate": "2016-10-23T08:16:41.523", "Body": "

I am rebuilding my printer, a hold Prusa/Mendel that has been boxed for years. I have changed the board ( I am using a Rumba) and the drivers (I am using Pololu 8225) with heatsinks and 1/16 microsteps.

\n\n

5 motors type 42BYGHW811, rated: 2,5 A, R 1,25 Ohm.\nI have set the stepper voltage at 0,8 V for XYE which gives a current of 0,64 A and they seems to work without problems.

\n\n

To get the same current on Z, where I have 2 motors in parallel, I should set it at 0,4 V but the motors do not turn at such low voltage. They just make noise, no turn. I have M8 rods nicely lubricated on such axis and I can turn them easily by fingers. If I go up to 1 V the motors turns but then the driver gets very hot and I loose steps. May be I should play with the acceleration too?

\n\n

I am not sure if the above settings are correct. Your hints from your actual settings and experiences will be appreciated.

\n", "Title": "How should I power these stepper motors", "Tags": "|stepper|stepper-driver|", "Answer": "

If the motors are in parallel, then setting it at 0.4V means each motor will only get a quarter of the current a single motor would get at 0.8V. If you set it to 0.8V each motor will get half. I think 0.8V (same as for the other motors) would be an appropriate setting. Stepper drivers are constant-current, not constant-voltage.

\n\n

Are the steps/mm for your Z-axis correct? Perhaps it's just trying to move too quickly.

\n" }, { "Id": "2969", "CreationDate": "2016-10-23T22:20:26.973", "Body": "

I seem to be observing a couple problems w/ Cura2.3 OSX . It seems to be placing the sliced data (gcode) over to negative X by a cm or so. Further, the X- and Y-axes displayed in Cura definitely map to Y and X when the printer starts up. I've verified that I have the printer's X and Y end stops and motors connected to the correct outputs. Now, I can just rotate my parts before slicing, but I'd rather have things be \"correct\" to begin with. \nI've read the relevant info at question 1323, but I'd rather if possible adjust the Printer Settings for my defined printer in the Cura Preferences sub-menu. I couldn't find any description of what the \"printer head\" X and Y offset values mean there, nor what the \"Z-gantry\" value does either.\nI'm running from the SD card; no USB connection.

\n", "Title": "How to map Cura printspace to Prusa I3 printspace", "Tags": "|prusa-i3|ultimaker-cura|axis|", "Answer": "

Well, in my particular case, it helps to have the X and Y control (motor and endstop switch) plugged in to the correct ports :-( . That took care of the X vs. Y orientation problem.

\n\n

If you command \"Home All\" either via the LCD panel or over USB, the nozzle should sit in the front left corner of the bed as you face the machine (\"stage right\" for you thespians). \nThe link Chris T provided, ultimaker, provides helpful background information as well.

\n\n

Next, if there are small offsets, find control can be set with a script similar to the following (leave out the auto-levelling command if you don't have that),\nfrom printerbot talk forum .

\n\n
M104 S{print_temperature}\nG21 ;metric values\nG90 ;absolute positioning\nM82 ;set extruder to absolute mode\nM107 ;start with the fan off\nG28 X0 Y0 ;move X/Y to min endstops\nG28 Z0 ;move Z to min endstops\nM109 S{print_temperature} ;wait on temperature line\nG29 ; Auto bed levelling\n;G28 X0 Y0 ;move X/Y to min endstops\nG92 X132.4 Y20; Reset actual position after G29 bug\nG92 E0 ;zero the extruded length\nG1 F{travel_speed}\n
\n\n

And finally, there's a calibration file at thingiverse which prints large squares so you can see the exact offset. I hope to get that file modded with a couple markers (indents, e.g.) to verify orientation as well as location.

\n" }, { "Id": "2970", "CreationDate": "2016-10-24T13:59:18.637", "Body": "

When I design parts that must fit in each other, I usually keep in mind that prints are 0.2-0.5 mm wider than expected (depending on the material), and I size the parts accordingly.

\n

However, sometimes, I design the whole part and then slice it with planes or lines. In these cases, I need to push/pull each contact surface manually by the same amount (0.2-0.5 mm), which is a time-consuming task. Another option is to use a cutter to remove the outer surface layer (basically, I remove the rigged surface, making it smooth again). Still, it's unsafe and even more time-consuming.

\n

How can I quickly generate said controlled gaps on the contact surfaces between two objects but not on the rest of the object? To make it everywhere, I could use shells, I guess, but I don't need that on the whole object.

\n", "Title": "How to add tolerances/gaps in Autodesk 123D Design?", "Tags": "|3d-design|print-preparation|autodesk|", "Answer": "

AFAICT Push/Pull faces is the closest thing to what you're looking for. eg:

\r\n \r\n

\n

In OnShape there is an "offset face" command that allows applying such an offset to multiple faces at once: How to add tolerances/gaps in OnShape?

\n

Unfortunately I'm not aware of a way to do exactly the same thing in 123D Design, but I'm far less proficient with that tool (as Alucard Pawpad noted, though, it is possible to just select multiple faces at the same time and manipulate them all together, which is somewhat similar)

\n" }, { "Id": "2972", "CreationDate": "2016-10-24T19:55:19.650", "Body": "

A lot of consumer desktop FDM printers come with a 0.4\u00a0mm nozzle. I'm looking to print fine details objects and I was considering trying to use a smaller size nozzle. But before I do so I would like to establish a list of downsides and unwanted consequences.

\n", "Title": "What are the downsides and aftereffects of using a smaller nozzle?", "Tags": "|fdm|desktop-printer|nozzle|", "Answer": "

In addition to the answer of tbm0115; special care has to be given to the strength and stiffness of the model. Lines are thinner and thus when using the same amount of walls and infill percentage, the walls will be less stiff. This requires a higher percentage of infill or more walls to counteract this phenomenon. Obviously, this will cause the print to take longer.

\n" }, { "Id": "2979", "CreationDate": "2016-10-25T05:59:09.757", "Body": "

Are there any 3D printing services or something similar to 3D print or injection mold light reflectors?

\n\n

I'm trying to find something that is similar to PCB printing that allows you to upload a 3D design of a reflector and they will produce this reflector and coat it with mirror surface.

\n", "Title": "Affordable 3D printing of reflector", "Tags": "|material|services|", "Answer": "

You can make flat reflectors with glass panels or acrylic panels and apply window mirror film. Ordinary window glass might be less expensive than acrylic and many places will cut to size, but unusual shapes might not be an option. The mirror film is relatively easy to apply and the packages have instructions included.

\n\n

For the hologram pyramid you referenced, one could 3d print the bracket at the base. If your design is different, yet similar, that's an option for relatively easy construction.

\n\n

100-200 pieces is really too small of a quantity to justify expensive injection molding. If the part is not too complex, one could create the model with 3d printing, then build a silicone mold around it, followed by pouring polyester or epoxy casting resin to make larger quantities. A better idea of the shape would be useful, but the information I've provided may head you in a practical direction.

\n" }, { "Id": "2983", "CreationDate": "2016-10-25T17:14:38.847", "Body": "

The quality of the filament is frequently mentioned either as the source of an issue but also as the solution to a problem.

\n\n

I know that the way you use,store and protect your filament will greatly impact it's quality, especially overtime, but here I'm interested to understand what makes a brand new, out of the air-tight package, good or bad? What to look for when you buy filament (specs,chemical,weight,...)? What are some brands that are notoriously know to make \"high quality\" filament?

\n", "Title": "What makes a good PLA filament?", "Tags": "|filament|pla|", "Answer": "

General characteristics of a \"quality\" filament:

\n\n\n\n

I hope this helps! :-)

\n" }, { "Id": "2986", "CreationDate": "2016-10-26T19:04:33.863", "Body": "

Has anyone succeeded in installing the auto bed levelling on a Rumba board with Marlin firmware?

\n\n

I have the last stable version 1.1.0 RC6.

\n\n

I would appreciate some direction especially about:

\n\n\n\n

I have only installed the hardware for now (5 V servo) connected to Ext. 3 (EXP3):

\n\n\n\n

\"Servo

\n\n

\"RUMBA

\n\n

but I cannot move it with the G-code command M280 P0 S180. I have no idea where to put my hands on firmware to get this going. However my ultimate goal is to set the ABL.

\n", "Title": "Auto bed levelling with Rumba board - Servo not deploying", "Tags": "|calibration|z-probe|", "Answer": "

For future reference.

\n\n

My issue about the servo not moving was caused by a wiring mistake.\nThe Exp. 3 has 14 pins has per this diagram.

\n\n

\"enter

\n\n

However when phisically looking at the board, what you see is this:\n\"enter

\n\n

I took the first 2 pins on the right of such connector and the 3rd one of the first row thinking that I was connecting pins 2-4-5 of Exp. 3.\nI was wrong, because the first 2 (1-2) pins are not part of Exp. 3.

\n\n

The right way to connect the servo is as following:\n\"enter

\n\n

Then use PWM1 (pin 5 Ext.3)\nI decided to leave trace of this issue and the relevant solution for someone that may experience the same issue.

\n" }, { "Id": "2996", "CreationDate": "2016-10-29T17:26:57.503", "Body": "

I have an old Solidoodle 2 that I bought broken from a garage sale that I am converting to use RAMPS 1.4 with Marlin Firmware. All the motors work correctly, I am just having issues getting the endstops to work.

\n\n

I am using a regular limit switch with NC going to the signal pin and the other to ground. I have this switch plugged into first header column for X-min. My endstop configuration is currently:

\n\n
//=========================================================================== \n//============================== Endstop Settings ===========================\n//===========================================================================\n\n// @section homing\n\n// Specify here all the endstop connectors that are connected to any endstop or probe.\n// Almost all printers will be using one per axis. Probes will use one or more of the\n// extra connectors. Leave undefined any used for non-endstop and non-probe purposes.\n#define USE_XMIN_PLUG true\n#define USE_YMIN_PLUG true\n#define USE_ZMIN_PLUG true\n//#define USE_XMAX_PLUG false\n//#define USE_YMAX_PLUG false\n//#define USE_ZMAX_PLUG false\n\n// coarse Endstop Settings\n#define ENDSTOPPULLUPS // Comment this out (using // at the start of the line) to disable the endstop pullup resistors\n\n#if DISABLED(ENDSTOPPULLUPS)\n  // fine endstop settings: Individual pullups. will be ignored if ENDSTOPPULLUPS is defined\n  //#define ENDSTOPPULLUP_XMAX\n  //#define ENDSTOPPULLUP_YMAX\n  //#define ENDSTOPPULLUP_ZMAX\n  //#define ENDSTOPPULLUP_XMIN\n  //#define ENDSTOPPULLUP_YMIN\n  //#define ENDSTOPPULLUP_ZMIN\n  //#define ENDSTOPPULLUP_ZMIN_PROBE\n#endif\n\n// Mechanical endstop with COM to ground and NC to Signal uses \"false\" here (most common setup).\n#define X_MIN_ENDSTOP_INVERTING true // set to true to invert the logic of the endstop.\n#define Y_MIN_ENDSTOP_INVERTING false // set to true to invert the logic of the endstop.\n#define Z_MIN_ENDSTOP_INVERTING false // set to true to invert the logic of the endstop.\n#define X_MAX_ENDSTOP_INVERTING false // set to true to invert the logic of the endstop.\n#define Y_MAX_ENDSTOP_INVERTING false // set to true to invert the logic of the endstop.\n#define Z_MAX_ENDSTOP_INVERTING false // set to true to invert the logic of the endstop.\n#define Z_MIN_PROBE_ENDSTOP_INVERTING false // set to true to invert the logic of the endstop.\n
\n\n

I have X-min enabled and inverted. When I send an M119 (endstop status code) I recieve:

\n\n
Send: M119\nRecv: Reporting endstop status\nRecv: x_min: open\nRecv: y_min: TRIGGERED\nRecv: z_min: TRIGGERED\n
\n\n

And then when I press down the X endstop with my hand I get:

\n\n
Send: M119\nRecv: Reporting endstop status\nRecv: x_min: open\nRecv: y_min: TRIGGERED\nRecv: z_min: TRIGGERED\n
\n\n

-No change. There is no mechanical failure with the switches, I've tested it with a continuity tester. I have even shorted the signal and ground pins on the Ramps board with a jumper wire and I still haven't seen any change.

\n\n

Where is the fault at?

\n", "Title": "Endstops will not trigger using Marlin Firmware", "Tags": "|marlin|ramps-1.4|endstop|", "Answer": "

After fighting with this for a while, I found out on the end stop pins, if coming from another setup like mine, you need to switch the ground pin to the center of the connector and the 5 volt pin to the pin it is reading, for example 1.29 pin on skr 1.4.

\n

Once you do that then it will read the status of the pin.

\n" }, { "Id": "2999", "CreationDate": "2016-10-30T06:40:38.223", "Body": "

I'm using the Marlin firmware (1.1.0-RC7 - 31 July 2016) for a 3d printer. Currently the printing is not perfect due to slight inaccuracies in movements along the x and y axis. I'm trying to change the feedrate for speed along the xy axis whilst the printer is in operation to make sure the printer stops on time and prints accurately.

\n\n

I have some code for controlling the feedrate but the problem is that I'm not sure where I am supposed make these adjustments. In the configuration.h file I see this code: (lines 742 and 753 )

\n\n
/*line 742*/ #define HOMING_FEEDRATE_XY (50*60) \n/*line 753*/ #define DEFAULT_MAX_FEEDRATE {300, 300, 5, 25}  // (mm/sec)\n
\n\n

I'm probably misunderstanding something but it seems like this sets the feedrate to a default value which is the same as the maximum.

\n\n

If the feedrate changes during printing I'm guessing it would be done in Marlin_main.cpp but I'm not sure which part it actually changes. Can someone point me in the right direction here?

\n", "Title": "Marlin Adjusting feedrate", "Tags": "|marlin|speed|", "Answer": "

It seems like you might be wanting to look at the steps per mm line.

\n\n
#define DEFAULT_AXIS_STEPS_PER_UNIT\n
\n\n

This line is where you calibrate the number of steps per mm. To calibrate, mark a point along an axis, then move that axis something like 100 mm. If it didn't move 100 mm exactly, make a change

\n\n
(how far it moved)       (how far it should have moved)\n------------------   =   ------------------------------\n(current steps/mm)         (new steps/mm) --> find this\n
\n" }, { "Id": "3012", "CreationDate": "2016-11-02T16:52:21.587", "Body": "

I've posted an issue on GitHub and I have been advised to load RCBugFix.\nI have never used it and I do not know what is about.

\n\n

What is it?\nHow does it work?

\n", "Title": "RCBugFix what is it?", "Tags": "|marlin|", "Answer": "

It refers to a specific branch of the current project, named RCBugFix. It's basically the most up-to-date, bleeding edge version of Marlin that contains all of the most recent fixes (it is more up-to-date than RC). However, as it's so up-to-date, it's also not thoroughly tested and possibly quite unstable.

\n\n
\n

The latest Release Candidate lives in the \"RC\" branch. Bugs that we find in the current Release Candidate are patched in the \"RCBugFix\" branch, so during beta testing this is where you can always find the latest code on its way towards release.

\n
\n\n

Use with caution.

\n" }, { "Id": "3015", "CreationDate": "2016-11-03T21:36:24.987", "Body": "

I have been studying the differences between version 2.x and version 4 of the P3Steel frames - in particular the AC08 bracket at the top of the frame which secures the top of the smooth bars and threaded rods of the Z axis on both the left and right sides. Here is the laser cut parts, for version 1.x, showing the part labelled as AC08:

\n\n

\"Laser

\n\n

Version 1.x/2.x has the AC08 bracket with two holes, one for the threaded rod and one for the top of the smooth bar (from the lasercut image):

\n\n

\"P3Steel

\n\n

However, in version 4, the corresponding top Z axis bracket only has one hole for the smooth bar and just an indentation for the bearing which holds the top of the threaded rod (from Twitter):

\n\n

\"P3Steel

\n\n

Here is a close up of the diagram from the google docs repository, listed in the v4 section on the RepRap Wiki page for the P3Steel, which shows the bearing assembly just apparently \"resting\" against the indentation:

\n\n

\"Close

\n\n

Here is the bracket shown with the bar and threaded rod (again, from google docs):

\n\n

\"P3Steel

\n\n

Does anyone know why the top of the threaded rod is not secured by a hole, as it was in version 1.x/2.x? It just does not look particularly well secured.

\n\n

Under the list of version 4 changes, see 2. Frame versions, it is mentioned:

\n\n
\n

The extruder no longer hits the Z axis top bracket

\n
\n\n

Is this the reason why the change has been made?

\n", "Title": "Z axis top brackets, of P3Steel, differ between v1.x/2.x and v4", "Tags": "|prusa-i3|p3steel|", "Answer": "

I've just seen this right now, I'm Alvaro Rey, the designer of the p3steel v4 mod. The change was made, because with previous versions with the extruder homed, if you go up in the Z axis, the extruder motor could hit the Z axis top plate. So, in order to avoid that, I just changed the design.

\n\n

The bearing in the z axis is not necessary but some people prefer to use it, in order to avoid wobble in the threaded rod.

\n\n

Anyways, I designed a printer part to fix the bearing in the Z top plate.

\n" }, { "Id": "3023", "CreationDate": "2016-11-07T20:37:56.777", "Body": "

I had my printer printing fine when using the stock trigger switch as I used it to print the green bracket you see in the picture.

\n\n

\"Photo

\n\n

My problem now is when I do a print with the sensor, it moves to 0,0 position. However in this position the sensor is hanging off the bed hence there is nothing for it detect so it crashes into the bed.

\n\n

As far as I can tell the nozzle is homing in the right place.

\n\n

How do I tell Marlin the new minimum position it needs to be in so it doesn't crash into the bed?

\n", "Title": "How to offset my probe so it's not hanging off the bed at 0, 0 position when printing", "Tags": "|diy-3d-printer|", "Answer": "

It is not a problem that the sensor is not above the build plate during printing as long as it is above the build plate during the auto bed levelling sequence.

\n\n

Homing does not necessarily need to be the (0,0) coordinate. Usually, a printer homes on the endstop switches, from that coordinate an offset is defined in the firmware to move to the origin. This implies that (depending on the position of the sensor), the sensor may be outside the bed area when the nozzle is at the origin (0, 0)). Therefore, similarly, you need to tell the printer the location of the Z sensor with respect to the nozzle position in order for the printer to keep the sensor on the bed when levelling by setting boundaries for the sensor to reach.

\n\n
\n\n

E.g. for Marlin firmware the offset from homing to the bed origin is defined for an Anet A8 by:

\n\n
#define X_MIN_POS -33\n#define Y_MIN_POS -10\n
\n\n

The values you should use need to correspond to the actual offset from the homing point to the origin of the bed (0,0).

\n\n

When using an auto bed leveling sensor like you are using you should consider this remark:

\n\n
\n

If using a Probe for Z Homing, enable Z_SAFE_HOMING also!

\n
\n\n

Un-comment the proper line in the configuration file to read:

\n\n
#define Z_SAFE_HOMING\n
\n\n

This will make the printer aware of the sensor, and home Z in the middle of the bed (default behavior, but can be changed), so that your sensor is never off the bed when probing the bed for Z homing.

\n\n

Furthermore, you need to set the offset values of the center of your sensor to the nozzle center:

\n\n
 *   Z Probe to nozzle (X,Y) offset, relative to (0, 0).\n *   X and Y offsets must be integers.\n *\n *   In the following example the X and Y offsets are both positive:\n *   #define X_PROBE_OFFSET_FROM_EXTRUDER 10\n *   #define Y_PROBE_OFFSET_FROM_EXTRUDER 10\n *\n *      +-- BACK ---+\n *      |           |\n *    L |    (+) P  | R <-- probe (20,20)\n *    E |           | I\n *    F | (-) N (+) | G <-- nozzle (10,10)\n *    T |           | H\n *      |    (-)    | T\n *      |           |\n *      O-- FRONT --+\n *    (0,0)\n */\n#define X_PROBE_OFFSET_FROM_EXTRUDER XXX   // X offset: -left  +right  [of the nozzle]\n#define Y_PROBE_OFFSET_FROM_EXTRUDER YYY   // Y offset: -front +behind [the nozzle]\n#define Z_PROBE_OFFSET_FROM_EXTRUDER 0   // Z offset: -below +above  [the nozzle]\n
\n\n

Where XXX and YYY are your actual values.

\n\n

And set the boundary of the probing section:

\n\n
// Set the boundaries for probing (where the probe can reach).\n#define LEFT_PROBE_BED_POSITION 15\n#define RIGHT_PROBE_BED_POSITION 190\n#define FRONT_PROBE_BED_POSITION 15\n#define BACK_PROBE_BED_POSITION 170\n
\n\n

Note that the values should match your bed size!

\n\n

And:

\n\n
// The Z probe minimum outer margin (to validate G29 parameters).\n#define MIN_PROBE_EDGE 10\n
\n\n

Details on setting the boundaries of the bed to keep the sensor on the bed is described in question \"How to set Z-probe boundary limits in firmware when using automatic bed leveling?\".

\n" }, { "Id": "3024", "CreationDate": "2016-11-07T22:46:36.773", "Body": "

I'm looking to make 20-30 IoT devices, with the pi zero.\nThe price point is perfect, the only downside is that it doesn't have an Ethernet jack and the power adapter is not included.

\n\n

I found a mod that deals with both of these issues, however it would involve a lot of soldering. The mod takes a USB to Ethernet adapter and 48 to 5 volt converter and allows the device to be both powered and connected to the internet with just an Ethernet cable.

\n\n

With 3D printing or Milling is it possible to make something that snap fits the components together and mitigate a lot of soldering?\nIs it possible to print or mill something that will do the 48 to 5 volt conversion?

\n\n

Components:

\n\n
    \n
  1. Two male micro USB heads
  2. \n
  3. 48 to 5 volt converter
  4. \n
  5. Ethernet adapter board
  6. \n
  7. Pi zero
  8. \n
\n\n

\"enter

\n", "Title": "Power Over Ethernet (Pi Zero Mod)", "Tags": "|raspberry-pi|", "Answer": "

3D Print A Solderless Circuit Board

\n\n

I think this was the solution I was looking for, unless someone else knows a method that's cheaper, faster, safer, etc.

\n" }, { "Id": "3028", "CreationDate": "2016-11-08T22:00:24.473", "Body": "

I've tried looking online but haven't quite found the answer to what I'm confused about, exactly. Consider the following 3 lines of G-code:

\n\n
G1 X129.000 Y126.322 E7.90758\n\nG1 X128.349 Y125.707 F7800.000\n\nG1 X128.476 Y125.548 E7.92045 F1800.000\n
\n\n

Does the absence of an E command in the second line mean that no material is to be extruded from line 1 to line 2? As I understand it, the E parameter controls how much material has been extruded up to that line.

\n\n

The reason why I am asking is because I'm trying to understand on the code level how to identify when a printer will travel without extruding any material, in the case of moving in the Z direction, or if you are printing multiple disconnected parts in one print. Since none of the Z lines seem to have any E commands, I'm inclined to believe that the absence of an E command means that when moving to the given position from the previous line, no material will be extruded. I've tried quite a few different queries online to try to discover if this is the case, but haven't found a simple answer, so I was hoping someone on this board could enlighten me.

\n", "Title": "How do the E commands in G-code work, exactly?", "Tags": "|g-code|", "Answer": "

Yes, the absense of an EX.XXX (or an EX.XXX with the same value as the previous one) means nothing will be extruded during the move. The extruder is treated as an imaginary fourth axis and works exactly the same as any other axis: if in a G1/G0 no new coordinate for it is specified, it retains its original position.

\n" }, { "Id": "3043", "CreationDate": "2016-11-14T07:45:04.707", "Body": "

When 3D-printing on an 20*20cm, I've heard that the quality of the printings get worse if I fill out the board... Is it true? Should I keep it to small amounts at the time or doesn't it matter?

\n", "Title": "Does the amount of figures on the board matter?", "Tags": "|print-quality|", "Answer": "

Just to add to the other answers already here, I've also had problems with layer adhesion when filling up the print bed on an extrusion printer. The longer the print head spends working on details around a single layer, the more chance the layer has to cool off before the next layer gets started. I've started to suspect that layers that take longer to print end up with less adhesion to the next layer.

\n" }, { "Id": "3047", "CreationDate": "2016-11-15T01:33:13.743", "Body": "

I cleaned up my Flashforge Creator Dual tonight, and loaded some transparent ABS prepping for a print. The filament extruded fine, then started to wiggle, then became fine again. Hot end is 0.4 mm and was heated to 230C. What sort of steps should I take to troubleshoot the issue? Has anyone seen this before?

\n\n

\"Photo

\n", "Title": "While filament loading extruded filament wiggles", "Tags": "|filament|troubleshooting|", "Answer": "

It might be similar to this issue (where teflon tube wasn't installed what caused softening filament inside heatsink)

\n\n

Please check if your extruder is equipped with proper pipe inside (inner pipe diameter).

\n\n

But the most probable cause is that a pipe had slide out so filament has enough room to get soft and to form such shape going out. Please take a look also here.

\n\n

Here is a situation and its evolution.\n\"enter\nfig. A:\n1. filament\n2. teflon tube (well positioned)\n3. heatsink\n4. nozzle\n5. extruded filament

\n\n

fig. B:\n6. slided out teflon tube\n7. empty space (heat)

\n\n

fig. C:\n8. bent filament

\n\n

fig. D:\n9. teflon tube\n10. no extrusion

\n\n

How it happens?

\n\n

Teflon tube is usually well positioned so everything is fine in most cases. Sometimes because of poor filament (not-constant diameter) or because of bends on filament, teflon tube is slided out os its place while retraction. It goes on and on, up and up (fig.B)

\n\n

Once tube is extended that much so filament starts to bend in empty (heat) space (fig.C) It increases the effect of sliding out teflon tube just because of bend (it works as a hook). But it can still work. Teflon tube can even be pressed back inside the nest.

\n\n

Now because the empty space is heat then filament starts to melt (for example when there is no extrusion for a second) it becomes soft. When filament is soft enough it doesn't extrude as the force to press it throught the nozzle is too high for such soft material so filement definitely stops extruding itself. (fig.D) This situation cannot be fixed automatically. There is a need of user intervention.

\n\n

Now you know why some extruders has a rim (or collar) on the top of heat barier tube. It secures a teflon tube from sliding out.

\n" }, { "Id": "3050", "CreationDate": "2016-11-15T12:51:23.607", "Body": "

I've recently designed a non-self-aligning caged deep-groove ball bearing. Now I'd love to get one 3D printed.

\n\n

However, assembling those can be tricky and I highly doubt it's even plausible to print them. All the components themselves can be printed without a problem, but I'm not sure whether I'll be able to put them all together in the end.

\n\n

What are my options?

\n\n

FDM printers are probably out, although it would be great if I can find a way to use those. Would an SLA or perhaps an SLS printer be able to pull it off?

\n\n

Of course the thing still has to work (move) in the end.

\n\n

\"Caged

\n", "Title": "Printing a non-self-aligning caged deep-groove ball bearing", "Tags": "|3d-models|3d-design|mechanics|sls|", "Answer": "

How about using a dual nozzle FDM printer with ABS and HIPS?

\n\n

You can use HIPS to print any support materials or any spacers that you need. The HIPS can then be dissolved using Limonene. Limonene will not do much of anything to ABS.

\n\n

Similar ball bearings have been printed like this before. I am not aware of any reason this would not work for you. Other types of ball bearings have been printed in FDM with no support material.

\n" }, { "Id": "3059", "CreationDate": "2016-11-18T15:22:54.970", "Body": "

My newly built ANET A8 has dual Z-axis steppers (and two threaded rods), but only an end-stop on one side. Am I missing some detail for ensuring proper alignment of Z as the extruder moves along the Z-axis? I notice there are mountings for a 2nd end-stop, but nowhere to attach this on the MCU board.

\n\n

It seems that even after I check the leveling, there is scope for the two steppers to get out-of sync once the servos are turned off - is this something to worry about? I realise that during a print run, if I start with good alignment this should stay good.

\n\n

I've only managed about 10 minutes of printing so far, and everything seems to work, but this is my first printer, so obviously I might have missed a vital step in the instructions quite easily.

\n", "Title": "Alignment of dual Z-axis steppers", "Tags": "|prusa-i3|z-axis|", "Answer": "

All the Prusa-based designs I've seen have only one end stop. While you are correct that it's theoretically possible for the two Z-drives to get out of sync, it's very unlikely in practice (barring serious friction, binding, etc.).

\n\n

But even if it happens, remember that the endstop microswitch is only used to keep the extruder assembly from crashing into the print bed. The stepper motors do not have shaft encoders, or any other position sensing mech, so if they were to get out of sync, there'd be no way to know this.

\n\n

The reason there's provision for a stop mount, on both sides, is simply to make the physical parts of the frame the same.

\n\n

That said, it is important to check the extruder support assembly to verify it's level as you build the printer - \"level\" meaning both supports are the same distance from the screw-drive shaft couplers.

\n" }, { "Id": "3065", "CreationDate": "2016-11-21T10:47:13.383", "Body": "

I've actually solved this, but I think its still a useful question which I don't think is easy to answer with existing questions.

\n\n

As soon as I'd built my ANET-A8 (Prusa i3 DIY kit), I found I was having problems with the extruder crashing into the bed. Although I thought I'd adjusted the bed leveling OK, the calibration seemed to keep getting messed up.

\n\n

I tracked this down to two factors. First, I was winding the extruder head up some distance before loading the filament and starting a print. Second, at roughly half-way up the axis, the right-hand thread seemed to be getting stuck (more often when moving up than down).

\n\n

What wasn't clear (and not mentioned in the building instructions) was what might cause this problem.

\n", "Title": "Z-axis steppers and bed alignment problems", "Tags": "|prusa-i3|z-axis|", "Answer": "

From http://3daddict.com/anet-3d-printer-common-mistakes-fix/

\n\n

The motor shaft and the threading rod must have space between them in the flexible coupler.

\n\n

That means unlocking the coupler from the motor shaft and moving it up a bit, in the end the threaded rod nearly touches the printer top hole instead of having like a 1cm gap.\nThis will allow the coupler to flex, and thus should handle small misalignments of the brass nut.

\n\n

That's about point 2 of @darth-pixel answer, but before locking the screws, make sure to have empty space between rod and motor shaft

\n" }, { "Id": "3068", "CreationDate": "2016-11-21T15:14:08.900", "Body": "

5AxisMaker has a 5 axis CNC/3D printer combo machine. I understand what the benefits of 5 axis are for CNC machines, but are there any benefits for 3D printing. In this video they show the printer printing on an angle, but this could have been done with just linear layers.

\n\n

Would there be any cases where a 5 axis printer would preform better than a 3 axis printer?

\n", "Title": "Does a 5 axis 3D printer have any benefits over a regular 3 axis printer?", "Tags": "|axis|", "Answer": "

Slicing prints for printing in 5 Axis is not going to be simple with the current software. If you look at the web site for the 5AxisMaker you can see they are using Algorithmic modeling software (Grasshopper). You would probably need to buy this as well and then spend a decent amount of time learning how to use this software as well if you were going to try and take advantage of the 5th Axis for 3D printing.

\n\n

I have only seen Grasshopper used for 3D printing and 5 Axis used in research papers.

\n" }, { "Id": "3073", "CreationDate": "2016-11-21T17:56:01.737", "Body": "

We are custom building a Cartesian 3D printer to be used in a production 3D printing environment, with the following requirements:

\n\n\n\n

Question: How to select firmware?

\n\n

Are there notable differences between the primary firmware options (Marlin/Repetier) for this setup? From the google code groups, I understand the Repetier has a much cleaner code base, and apparently gives smoother physical performance. Yet, from what I can gather, 95% of the community uses Marlin - is that correct?

\n\n

Given that this printer will have the Pi3 to control \"higher functions\", is it worth considering a compact firmware like Teacup?

\n\n

Edit 12/April/2017:

\n\n

For others looking: After much review, Klipper was selected as the most forward firmware - all kinematic calculations are done on the host computer, instead of on the microprocessor, resulting is significantly faster/smoother stepper movement.

\n", "Title": "Firmware Choice: Marlin vs Repetier vs Other", "Tags": "|firmware|", "Answer": "
\n

Given that this printer will have the Pi3 to control \"higher functions\", is it worth considering a compact firmware like Teacup?

\n
\n\n

A few days ago I came across klipper.

\n\n

It seems to do exactly that functionality split you mention

\n" }, { "Id": "3084", "CreationDate": "2016-11-24T07:15:49.703", "Body": "

What would cause this effect, and how can it be avoided?

\n\n

This is a PLA print, and it should look like this. I tried 215 and 225\u00b0C and both had the same effect. An earlier similar print at 220\u00b0C was not as bad but it still had some catching- it seems hit and miss and not strongly related to extrusion temperature.

\n\n

Outside:

\n\n

\"enter

\n\n

Inside:

\n\n

\"enter

\n\n

I'm using a QIDI dual extruder printer with Makerware 2.2.2.89 software and these parameters:

\n\n

\"enter

\n\n

\"enter

\n\n

Speed is 60/80mm/s.

\n", "Title": "What causes the print heat to 'catch' on the part?", "Tags": "|fdm|reprap|", "Answer": "

I think that you should first verify that you have the latest firmware and a newer version of MakerWare. I experienced similar issues around that version of MakerWare and remember a lot of print errors came with it.

\n\n

I believe the MakerBot Dual firmware is somewhere around 7.? and is no longer in development.

\n\n

MakerWare is also no longer in development as a standalone application and seems to have been ported over to the MakerBot Desktop. However I've personally found v2.4.1 to be substantially more stable than v2.2.

\n\n

I have not tested it, but supposedly the new MakerBot Desktop (v3.10) is compatible with the Replicator Dual \"Original\". I had tried an earlier version of MakerBot Desktop and reverted back to MakerWare 2.4 because I ran into issues with connectivity. However, I'm not certain it was an issue with software so much as the exploding voltage regulator...

\n\n

It might be best to give the new software a shot and/or try v2.4 of MakerWare, if you can find it.

\n\n

Here's the link to the latest MakerBot Desktop

\n\n

Here's the release notes for MakerWare/MakerBot Desktop v1.0-latest (v3.10+)

\n" }, { "Id": "3089", "CreationDate": "2016-11-24T17:07:25.063", "Body": "

I'm seeing the following pattern on my Prusa clone:

\n\n

\"Photo

\n\n

The first layer prints OK, layer adhesion to the bed is good. The second layer shows the weird gaps. On larger pieces, or with an increased feed rate, it sometimes gets so bad that parts of the second layer detach and are dragged across the print. I get the impression that this effect is worse when printing in the \"top left to bottom right\" direction, but not as bad when printing in the \"top right to bottom left\" direction.

\n\n

This occurs with multiple filaments and materials. I've already tried to manipulate the flow rate, temperatures and print speed, but gotten little improvement if any.

\n\n

What might be causing this issue?

\n", "Title": "Second layer: gaps, poor adhesion - why?", "Tags": "|print-quality|adhesion|", "Answer": "

Your first layer is horribly misscalibrated as well.

\n\n

You really should do all calibrations over. Including your firmware when your result is this bad.

\n\n

Following. Let's just assume your firmware is set right. And that your flow is right as well. The second layer speaks loudly of overcooked dribbling plastic. You might have a jam. Maybe the factory left some metal shavings in the hot end. I've had it happen. Try taking apart the extruded. Make sure there are no obstructions. Clean the tip with a micro hand drill bit.

\n\n

After that we will need more info. Post your settings. Your might have underflow (you set filiment size higher than it is) and have temperature so hot your are literally boiling the plastic and it is all dripping sparatically instead of controlled.

\n\n

Having temp too high can increase carbonization build ups and creats Jams. Did you ever have a good print ? My money is on a partial obstruction.

\n" }, { "Id": "3091", "CreationDate": "2016-11-24T20:48:30.587", "Body": "

I have a hypothetical question. From which parts can I build a metal-based 3D printer and can I buy those parts somewhere?

\n", "Title": "How to build a metal-based 3D printer?", "Tags": "|printer-building|metal-printing|", "Answer": "

I think this depends on what you are trying to accomplish with the 3D printer.

\n\n

I have seen people online build metal 3D printers from a robot arm and a welder this would probably be the simplest design and build but robot arms that can weld are expensive.

\n\n

I have also seen that someone at MIT build a glass 3D printer by building a small kiln with a hole at the bottom and moving it around like a normal extrusion printer. This method might work for a metal printer as well if you could get the temperatures right. This might be cheap enough a hobbyist could do it as well because you just need the same parts as a normal FDM 3d printer just able to more more weight. \nBut with the same parts you could also do lost pla casting and that would be a simpler approach with a better end product.

\n\n

But probably what you would be really interested in building would be a laser sintering 3D printer. Where a layer of powder is put down and then a laser melts part of the powered to join it to the model. For this the main thing you need would be the laser. I don't know a lot about how these printer work but the laser would need to be able to melt metal so I would think it would need to be very powerful and that would make it very expensive.

\n" }, { "Id": "3094", "CreationDate": "2016-11-26T16:55:09.137", "Body": "

I am completely new to 3D printing. I need to build a calibration plate, which I was told can be built using vero back plastic and a 3d printer. But I am afraid I need to know more if I give this to someone for fabrication. In particular, I am wondering how to get the white dots on the surfaces. My question is probably ill-posed, but I am trying to get as much info as I can before I consult any 3d printing vendors. Thanks

\n\n

\"Calibration

\n", "Title": "How to build this part using 3d printing", "Tags": "|3d-models|", "Answer": "

What are the characteristics of the white dots? (That is, are the dots small raised bumps, do they denote where a hole will be drilled, are they integral to a piece-to-piece connection, etc...)

\n\n

If these are small bumps that need to be added to the top of the black surfaces, your most time-effective solution is probably print the black component first (with placement references for where the white dots are to go), then switch materials and print the dots, and attach them to the black component.

\n\n

Otherwise, you'll need a machine capable of printing in two materials simultaneously(ish). See some of the newer Stratasys machines if you have a good budget ha!

\n" }, { "Id": "3097", "CreationDate": "2016-11-27T13:57:52.213", "Body": "

I have purchased a P3Steel v.4 kit that, rather unfortunately, comes with a steel Y plate.

\n

\"P3Steel

\n

I understand that there may be interia, as well as stepper motor wear, issues related to the weight of the steel plate. Therefore I would like to substitute the steel plate for another material. I had considered aluminium, as recommended by the RepRap wiki - P3Steel/Frame Versions/ Version 4.0 :

\n
\n

We recomended use aluminum beds for y axis.

\n
\n

However, it was suggested, by a vendor, that I use laser cut 6\u00a0mm thick MDF, principally for reasons of economics and availablity, over aluminium.

\n

I have subsequently found a supplier of 3\u00a0mm thick aluminium 200 mm x 300 mm Y axis plates, so availability is no longer an issue, and the slightly higher cost is not really an issue for me. However, I was wondering whether there would be significantly less interia if using an MDF Y axis plate, than with an aluminium plate.

\n

I assume that the masses/densities of aluminium and MDF are comparable, and a magnitude less than that of steel.

\n

I have found the densities of aluminium and steel:

\n
\n

Aluminium 2.7x103 kg/m3 (167 lb/ft3)

\n

Steel 7.82x103 kg/m3 (488 lb/ft3)

\n
\n

[Source: Densities of Solids - Engineering ToolBox]

\n

The density of MDF is given as 700\u2013720 kg/m3 (43.7-44.95 lb/ft3), which is a magnitude less than that of aluminium.\n[Source: Medium-density fibreboard]

\n

There is the issue that MDF can eventually warp, whereas aluminium supposedly does not, although the OP of Wanhao duplicator i3 print bed support warped shows that warpage is possible with aluminium plate - see Is the weather a problem for MDF frames? and Would a steel, instead of an aluminium, plate be reasonable?1.

\n
\n

TL;DR

\n

Considering the benefits of weight/inertia, stiffness and lack of deformation/warping:

\n\n
\n

Notes

\n

1 I found this question after having written up my question, so I admit that there is a risk of duplicity of the answers. However, I am concentrating more of the comparison of aluminium and MDF in this post, rather than just steel versus aluminium. Also, my question deals with the Y axis heatbed support, rather than the heatbed itself.

\n", "Title": "Steel versus MDF/Aluminium Y axis plate?", "Tags": "|heated-bed|y-axis|", "Answer": "

I have not used MDF for building a printer before; but, I have used it for other projects. It has the advantage of being very flat (initially); but, it has a LOT of issues with moisture. It is basically just a compressed slurry or water-based glue and sawdust. If you expose it to humidity or water it will swell like a sponge. I would not consider it for anything that requires a dimensional stability. For that, Aluminum is your better bet.

\n\n

Regarding the material properties of AL vs MDF, here is a good comparison:\n\"AL

\n\n

Note that while MDF has about 1/4 the density of AL, it has a MUCH lower Elastic Modulus (1/17 of AL) For the same thickness, it MDF is MUCH easier to bend than AL. Also note the strength to weight ratio of AL is also better.

\n\n

Even at twice the thickness, my calculations indicate that, for the same load, 6mm MDF would deflect about twice as far as 3mm AL. Also when AL exists its elastic region it becomes plastic (bends) where MDF breaks.

\n\n

Another aspect to consider is flammability. There a lot of heat sources around an FDM printer and if you are planning on a heated bed, there is one right there under the bed. Where MDF is hard to ignite, it is flammable and does not respond to heat well. On the other hand, AL can handle temperatures over 1000 degC and is a great thermal conductor for a bed heater.

\n\n

I would definitely choose AL over MDF for you printer bed.

\n\n

Another option to consider it is using a bare PCB (like FR4). The material is really strong (it is fiberglass), is relatively inexpensive, and is fire resistant (hence the\"FR\" in the name). Some commercial printers use FR4 for their print bed. One disadvantage is that is can sometimes develop a curl and there is really no way to get it flat again.

\n" }, { "Id": "3098", "CreationDate": "2016-11-27T21:38:22.680", "Body": "

Is there actually a way to determine whether the sheet of glass that one has received from a Chinese supplier on eBay is actually a sheet of Borosilicate glass, and not just a piece of normal glass intended for glazing purposes?

\n\n

Are there any laminations, or markings/features, visible under a certain light or if the sheet is viewed at a particular angle?

\n\n

I ask because most of the 200 x 300 mm sheets on eBay are priced at around \u00a316 - \u00a326, but there are a few priced at just \u00a310, which still claim to be toughened glass, although they do not mention the word Borosilicate, such as this one, 300x200x3mm 3D Printer Heated Bed Toughened Glass Clear Build Plate UK STOCK.

\n\n

To paraphrase, \"Usually if things seem to good to be true, then they are probably fake.\" However, I wondered whether it was worth a punt as it is only \u00a310, and then if I would be able to verify its veracity once it was in my hands.

\n", "Title": "How can I determine whether my borosilicate glass is fake or not?", "Tags": "|build-plate|", "Answer": "

Some more details about a suggestion made to the answer on How can I determine whether my borosilicate glass is fake or not?.

\n

Submerging the glass in mineral oil should make it disappear if it is borosilicate glass, as demonstrated below. This phenomenon is known as the "index matching" effect. The index of refraction of borosilicate glass is n=1.47, and that of mineral oil is also around n=1.47, depending on the type of mineral oil used. However, as mentioned in this comment, other types of glass might have a similar or close refraction index, making it hard to distinguish borosilicate from non-borosilicate using this test alone.

\n
\n

The borosilicate glass and the oil have roughly the same index of refraction. When this beaker is immersed into a container of oil, the beaker becomes invisible.

\n
\n

\"An

\n

Source: The Disappearing Beaker Experiment

\n" }, { "Id": "3101", "CreationDate": "2016-11-29T01:19:32.740", "Body": "

I switched out my 0.4 mm Ultimaker 2+ Nozzle the other day for the 0.25 mm and started using a ColorFabb Copperfill 2.85 mm filament. I am having some issues with it clogging and sticking to the bed for larger prints:

\n\n

\"Snapshot

\n\n

Current settings are:

\n\n\n\n

I thought it might be a calibration issue since I could see it skipping certain sections and slowly collecting excess material on the outer nozzle, so I recalibrated the bed but I still ended up with a blocked nozzle after about two hours of running. Not too sure if there are other settings I need to take care of. Any suggestions would be great!

\n", "Title": "Recommended settings for ColorFabb Copperfill Filaments on a 0.25 mm Nozzle?", "Tags": "|pla|ultimaker-2|filled-pla|", "Answer": "

It looks like a problem with feeding filament. It could be good to know if you did try such big printouts before but even if - from your picture it looks like on far end there is too less filament but on near end it's too much of it (because of 110%). And near the big circle it looks better even on far end. And there are \"waves\" close to big circle.

\n\n

How is that possible?
\nAssuming that your extruder works well it might be caused by bending bowden tube. If your filament is not slick enough - in some settings (positions/arrangements) of bowden it might create enough friction so it doesn't flow with proper rate. It might be also caused if your bowden tube was bent too much (broken).

\n\n

To be sure please try to print 4 smaller (5 cm in diam) circles on bed's corners and the one in the center. Just one layer of course.

\n\n

How to fix the issue?
\nIn my opinion you could try to slow down whole printout. Try to print such big circle with 30% of regular speed. I'm pretty sure you'll get proper results.

\n\n

Try to check if filament goes smoothly thought the tube. Check if it has proper inner diameter. You can try to clean up a tube inside if it's not perfect clean or change a tube to new one. I know it was probably ok for recent filaments but maybe this one is more \"demanding\" ;)

\n" }, { "Id": "3103", "CreationDate": "2016-11-29T13:20:34.850", "Body": "

TL;DR

\n\n\n\n

I would have also asked whether aluminium PCB heatbeds are worth the additional cost, over standard PCB heatbeds, but for the slightly less common dimensions of 200 mm x 300 mm, the costs, of an aluminium PCB heat bed, are roughly the same as standard PCB.

\n\n

Some thermal imaging pictures, comparing standard PCBs with aluminium PCBs, would be appreciated.

\n\n
\n\n

I am keen to buy an aluminium PCB heatbed for my P3Steel v.4 printer, which has a larger build area of 200 mm x 300 mm. So, after some searching, I found this MK2A: MK2A 300x200mm Aluminum Heated Bed Hot Bed for RepRap 3D Printer 12V + Wiring + NTC 3950 Thermistor - Upgrade you Prusa i3 for LARGE Printsize, or this cheaper MK3, Anycubic Dual Power MK3 Heatbed 300*200*3mm Aluminum Heat Bed 12V/24V Perfect for Prusa i3 3D Printer.

\n\n

Note: this is not a shopping question, I am merely providing a background to my questions.

\n\n

\"Mk3

\n\n

While the RepRap wiki is full of information, the information can be rather piecemeal and incomplete. Cases in point being the two pages on heatbeds:

\n\n\n\n

As stated in the former link, on the section on Metal print beds, PCB based heatbeds can suffer from hotspots, and that is why, apparently, it is sometimes preferrable to use a subsequent aluminium build/print bed on top of that, en lieu of glass plate, in order to even out the hotspots, as the aluminium's greater thermal conductivity helps smooth out the hotspots, by distributing the heat more efficiently than a glass plate can. However, it doesn't describe the heating effect of just a single aluminium PCB heatbed.

\n\n

The section, in the latter link, on Aluminium heatbeds, where the heating circuit is printed directly onto the aluminium, rather than a standard PCB (on a non-conductive substrate), makes mention that the print can be made directly on to the aluminium, without the need of an additional glass plate, resulting in a lighter print plate, and hence faster print speed. However, yet again, it is not particularly clear, or explicit, when it comes to heat distribution.

\n", "Title": "Do aluminium PCB heatbeds reduce hotspots?", "Tags": "|heated-bed|", "Answer": "

A thermal image of my aluminum heated bed does not show hotspots, although the edges are cooler than the majority of the center.

\n\n

\"Hyrel

\n" }, { "Id": "3107", "CreationDate": "2016-11-30T17:51:16.927", "Body": "

I am new to 3D printing. I thought that the black surface of my print bed was supposed to keep things stuck without additional tape, spray, etc. However, every time I print something, this same corner, and only this corner, detaches.

\n\n

Any advice on how to fix this?

\n\n\n\n

\"Photo

\n", "Title": "One corner never sticks", "Tags": "|adhesion|", "Answer": "

This is very likely caused by an uneven bed. It's probably lower in that particular corner: the nozzle is further from the bed, so the filament isn't squished down as much and therefore releases more easily from the bed.

\n\n

Your bed should have 3 or 4 screws that you can adjust, you should loosen the screw in the corner that is giving you trouble.

\n" }, { "Id": "3111", "CreationDate": "2016-12-01T11:53:29.057", "Body": "

I have been looking at cork sheet insulation for my 200 mm x 300 mm aluminium PCB heatbed, by I am not entirely sure how thick it should be. There seems to be a trade off between losing a few millimeters of print height, and providing adequate installation.

\n\n

I have seen 10 mm thick table mats, and then 5/3/2 mm thick cork insulation tiles. On some forums people say they use two 2 mm sheets beneath the aluminium heater and then another 1.5 mm aluminium plate under those, to hold it altogether (source: Re: Is a cork board necessary under the heated bed?).

\n\n

Hopefully this does not come across as a how-long-is-a-piece-of-string type question. I am just trying to get the right amount in the first place whilst spending as little as possible, and keeping the mass/volume and height down. If someone else has already gone through the empirical adjustments themselves, then it might save me the time and expense of having to test various configurations.

\n\n\n", "Title": "How much insulation do I really need?", "Tags": "|heated-bed|heat-management|", "Answer": "

Since 50% of the bed is uninsulated, you're definately into diminishing returns as soon as you start adding any insulation.
\nWith that area, I think you are looking at 1.2W per kelvin for a 2mm thickness.

\n\n

I'm guessing a bit with these powers, but roughly, maybe from 100 W un-insulated, 75 with 2mm, 60W with 4mm. You can get a reasonably accurate measure of the power by looking at the duty cycle of the heating element.

\n\n

Actually, its not clear if your primary goal is to reduce energy/maintain a very high temperature, or speed the initial heating. You can place a temporary sheet of cork on top of the bed (preferably extending over the edges to prevent convection) and this will significantly improve heat-up times.

\n" }, { "Id": "3112", "CreationDate": "2016-12-01T14:32:21.223", "Body": "

Should it be possible to directly send G-code to the printer serial connection using pipes under Linux?

\n\n

Example:

\n\n
echo M106 > /dev/ttyUSB0\n
\n\n

My controller runs at 250000 baud, I have tried setting the TTY baud rate to 250 kBd with:

\n\n
stty -F /dev/ttyUSB0 250000\n
\n\n

But, unfortunately, this particular baud rate appears to be unsupported under Ubuntu, giving the error:

\n\n
stty: invalid argument \u2018250000\u2019\n
\n", "Title": "How to directly send G-code to printer from a Linux terminal?", "Tags": "|firmware|arduino-mega-2650|", "Answer": "

The accepted answer didn't work for me. Everything received on the serial port was repeated back to the serial port, sending the printer into a confusing loop. There is more information about that in this answer: https://unix.stackexchange.com/questions/42964/unexpected-results-testing-serial-loopback-using-echo-and-cat

\n

The key is to use a command like this to prevent certain line termination characters from repeating themselves: stty 115200 -F /dev/ttyUSB0 -echo -onlcr , where 115200 is your baud rate.

\n

Here is what I did:

\n
# 1. plug in printer\nsudo chmod +777 /dev/ttyUSB0 # 2. allow access to printer USB permissions, add user to dialout or tty is better\nstty 115200 -F /dev/ttyUSB0 -echo -onlcr # 3. set serial port baud rate, might be 250000 for you\ncat -v < /dev/ttyUSB0 # 4. get printer output \n\n
\n

Then in a new terminal:

\n
   echo "M119" >> /dev/ttyUSB0\n\n
\n

This will give you a real console.

\n" }, { "Id": "3113", "CreationDate": "2016-12-01T14:50:28.943", "Body": "

I have a large resistor that goes in my J-Head extruder. It's grey, and it came with the extruder. I'm uncertain as to what grade of wire I need to solder to it. It being one of the elements of the system that heats the hottest, I would think that it would be important to find out what sort of wire is the correct kind to use on it, since heating elements require a lot of electricity.

\n\n

Also does the solder type I use matter?

\n", "Title": "What grade of wire is required for the heating element on a J-Head extruder?", "Tags": "|extruder|prusa-i3-rework|", "Answer": "

There are a number of things to consider:

\n\n\n\n

Finally, resistors have fallen out of favor compared to ceramic heating elements. Since they are inexpensive and solve all of the above problems (the leads are already attached and appropriately insulated) consider using a ceramic heater instead.

\n" }, { "Id": "3118", "CreationDate": "2016-12-01T20:38:41.543", "Body": "

Doing some low level monitoring, the following sequence is sent to the printer:

\n\n
N0 M106*36 \nN1 G28*18 \nN2 M107*39\n
\n\n

This sequence just turns the fan on, homes the printer, then turns the fan off.

\n\n

Question: What is the purpose of the asterix/star/(*) and the two succeeding digits on each line?

\n", "Title": "What is the purpose of asterix (*) in Gcode?", "Tags": "|g-code|", "Answer": "

That is a checksum. It's added by the host software to the G-code, to allow some basic verification by the firmware that the G-code was transmitted unchanged. It doesn't change the meaning of the G-code, and what your sequence actually represents is just M106, G28, M107. The N0,N1,N2,... are line numbers, and the combination of line numbers and checksums is used to request a re-send of any lines that were corrupted during transmission.

\n" }, { "Id": "3119", "CreationDate": "2016-12-02T03:32:53.420", "Body": "

Is it possible to 3D print an axial turbine 2 - 4 inches (50 - 100 mm) in radius, capable withstanding temperatures about 800 - 1000\u00b0C and rotation speeds of 100 - 120 x 103 rpm?

\n\n

How expensive is that? Is it cheaper to mill such a turbine from a whole piece of alloy?

\n\n

What technologies and materials should be used?

\n\n

Are Inconel alloys suitable for 3D printing?

\n\n

Are there any titanium alloys suitable for this task? I've read titanium is rarely used in rapidly rotating parts due to its ability to ignite if mechanical failure occurs and rotating blades touch the casing. Do titanium alloys still have this drawback?

\n\n

Is it possible to make disk of titanium and blades of Inconel, and have them welded (considering heat expansion)?

\n\n

How blades or blisks can be ceramically coated?

\n\n

Thank you!

\n", "Title": "3D printing and coating of heat-resistant objects, such as a turbine blisks or blades", "Tags": "|material|metal-printing|applications|", "Answer": "

You ask some very interesting questions! Firstly, when researching topics such as this, you will have far more luck using 'additive manufacturing' as a search term rather than '3D printing'. In the professional industrial environment, '3D printing' is not a term that is really used to describe the manufacturing you are talking about.

\n\n

Selective laser melting is the additive manufacturing process most suited to metallic aerospace parts. Inconel alloys can be processed (e.g. IN718 being one of the easiest) along with titanium (almost exclusively Ti6Al4V). As for manufacturing turbine blades and similar parts, you might find this interesting: Additive Manufacturing - Breakthrough with 3D printed Gas Turbine Blades.

\n\n

Titanium is not typically used in high-temperature sections of gas turbines, but will be used in larger, cooler components such as fan blades, where it's strength to weigh ratio is a benefit (less mass to rotate = better fuel efficiency).

\n\n

Coating of high-temperature nickel superalloy components is usually performed with electron beam physical vapor deposition (EBPVD) or thermal spray such as high-velocity oxy-fuel (HVOF); each process has certain characteristics that dictate when/where is it used.

\n\n

This is only really a surface depth answer to your questions, but it would be impossible to answer fully here!

\n\n

(My experience: PhD student using selective laser melting with aerospace alloys)

\n" }, { "Id": "3121", "CreationDate": "2016-12-02T10:17:43.533", "Body": "

I've just tried to make some small labels with embossed lettering. Base is 1.5 mm, with letters and a perimeter protruding by another 1 mm. After printing, I realised that Cura is not giving me a top layer, all I get is an inner and outer wall for the perimeter and the lettering:

\n\n

\"Screenshot

\n\n

The base is OK, with 4 layers, but there is no infill at the top. Presumably there is a setting that I've missed (I can appreciate that maybe this is a detail to the slicing which is model dependant). Layers are 0.2 mm, top and bottom set to 4 layers.

\n\n

I had walls set to 1.6 mm, (4 layers) and the wall features here are 1 mm (i.e. 2.5x the nozzle). The features are solid so far as I know, not drawn as a hollow.

\n\n

I found a relevant bug for Cura: https://github.com/Ultimaker/Cura/issues/1303 (and I also understand a workaround now, just need to write up an answer).

\n", "Title": "Missing top layers in Cura", "Tags": "|slicing|ultimaker-cura|", "Answer": "

Try Cura 2.4.0-beta2: there is option named \"Fill Gaps Between Walls\" under \"Shell\" settings, set it to \"everywhere\". In stable Cura 2.3.1 this option is missing.

\n" }, { "Id": "3122", "CreationDate": "2016-12-02T11:05:10.800", "Body": "

A comment to my question How much insulation do I really need? has made me paranoid about using cork as a heatbed insulator:

\n\n
\n

Why worry -- other than the fact that it's flammable. All that's needed to start a fire is having the thermistor fail or come loose. I'd use a flameproof material if you're going to insulate.

\n
\n\n

The flash point (ignition temperature) of cork is, apparently, 300 - 320\u00b0C1, which is not, as far as I can tell, a temperature that the heatbed reaches, so, in theory, cork should be safe to use as an insulator. In fact temperatures of around 300C are used in the manufacturing process of some cork products2:

\n\n
\n

For insulation applications, agglomerates of granules of\n cork, known as black agglomerates, are employed. They\n are manufactured in a closed autoclave at high\n temperature (approximately 300uC) and pressure\n (around 40 kPa) without the use of adhesive

\n
\n\n

In addtion, according to Why should we use cork?

\n\n
\n

Does cork burn?

\n \n

Cork is a slow combustion material. That is to say, yes it burns but very slowly and it doesn't produce flame so it doesn't spread. Also, when burning, the smoke that it releases is not toxic.

\n
\n\n

However, I am not sure if all cork is equal, or whether the thickness of the cork can affect the safety. To give a definite figure, I was thinking of using 2 mm - 5 mm thick cork sheeting.

\n\n

Has anyone experienced, or know of, any burning (or scorching) of cork, when used as a backing insulator to a heatbed, in particular, an aluminium PCB MK3 heatbed?

\n\n
\n\n

1 Source: What is the the ignition temperature of cork? | ChaCha

\n\n

2 Source: Cork: properties, capabilities and applications

\n", "Title": "Has anyone experienced scorching or burning of cork heatbed insulation?", "Tags": "|heated-bed|", "Answer": "

The real fire risks with 3d printers are electrical in nature. Lets say your wires come undone and happen to come next to the other lead. If it archs and happened to be in just the right position. Pretty unlikely. Once I did a bad solder job and when I was working on my printer. The wires literally burst into flame in my lap. No damage done (other than needing a new board). That said we often see people who have their boards catch fire in the flashforge owner groups. Weak solder joints, and over all bad quality.

\n\n

Do you really have something to worry about with Cork? No. A series of unfortunate events would have to happen, and more likely your board will cause a fire. That said I would look into adding a layer of aluminum and reflect some of that heat back up.

\n\n

If you are that worried I would just remove the sheet. Unless you are having issues calibrating you heated build plate / PIDs there is no reason to use insulation. I only use it on printers that have a hard time hitting ABS temps of 100c. Even then I only use tin foil and try to trap the air underneath.

\n\n

Just a side note. If you are having heated build plate issues maybe what you really need is a heat chamber.

\n" }, { "Id": "3134", "CreationDate": "2016-12-04T00:02:37.340", "Body": "

I bought a self-made Reprap Prusa Mendel 3 printer, modified to be built from cheaper materials, and immediately after the deal I got various problems. I fixed the majority of them, but don't know what the reason of the strange extruder behavior is: The stepper motor is not rotating while extruding filament, it's moving back and forth with small steps instead. I made some footage, Reprap Prusa Mendel 3d printer extruder problem, hoping that someone has met a similar problem before.

\n\n

If so, please tell me what to do to make it work as it should.

\n\n

The obvious option while encountering this problem was to slacken the bolt which holds this parts together (in the left bottom corner of the video), but that did not help.

\n\n

Any ideas are very welcomed. Thanks for your time. I hope this is an appropriate kind of question here.

\n", "Title": "Extruder stepper motor problem, what can be wrong?", "Tags": "|extruder|stepper|", "Answer": "

in my case there were random shifting and I found the motor is getting very warm .. so when I lowered the driver voltage it didn't get as warm and the skipping and shifting just stopped.

\n" }, { "Id": "3137", "CreationDate": "2016-12-04T21:19:34.110", "Body": "

I am using an M3D printer and loaded an STL design with holes in the middle:

\n\n

\"Screenshot

\n\n

However, the output is an object without holes (so I stopped the printing):

\n\n

\"Photo

\n\n

What can be done to be able to print with holes?

\n", "Title": "Printing objects with holes using M3D Printer", "Tags": "|slicing|", "Answer": "

Have you tried letting it print a few more layers? It is very common that printers use the first few layers to create a raft, which will make the model adhere better to the bed.

\n\n

I believe this illustration from Simplify3D displays this well:

\n\n

\"raft

\n" }, { "Id": "3151", "CreationDate": "2016-12-08T09:14:27.373", "Body": "

Does anyone know where I can get a free 3D design (STEP or STL) of an M4 Screw and nut? I have found only an M3 on Thingiverse: M3 Bolt by Kaleta.

\n", "Title": "M4 screw and nut for 3D printing?", "Tags": "|3d-models|", "Answer": "

You could also apply a scaling factor in your\nslicer to make a downloaded object larger or smaller. An M3 model scaled at +1.333 would produce a working thread with a nominal 4 mm OD. As noted in the comments below, a different scale factor might be needed in the axial direction if exactly matching the pitch of an M4 thread is required.

\n" }, { "Id": "3159", "CreationDate": "2016-12-08T20:56:15.760", "Body": "

I just got my HICTOP Prusa i3 Aluminium frame printer and finished putting it together. After setting it to preheat, I realized the bed was not level - I adjusted it and accidentally caused the head to crash into the bed.

\n\n

It sounded like something popped, and the printer restarted. It is now showing a MINTEMP error message, and both bed and hotend temperatures read 0\u00b0C.

\n\n

The only visible damage is that a thin layer came off the printbed - After testing with a multimeter, I realized that the metal under this layer is connected to power. As both the bed and hotend were heating, I believe I may have caused a short.

\n\n

Replacing the thermistor on the hotend with a spare did not solve the issue. I tested the thermistor on the printbed with a multimeter, and it seems to be working correctly (resistance value around ~200 kΩ at room temperature).

\n\n

Which leads me to believe that I may have damaged the control board. What do I do?

\n\n

Note: This is not a firmware problem as the printer was getting correct temperature readings before that. Also, I have a second extruder thermistor port that is unused. If I did damage my board, could I edit pins.h and configuration.h to use this port instead, and completely disconnect the heated printbed?

\n", "Title": "MINTEMP error after crash", "Tags": "|reprap|hotend|thermistor|", "Answer": "

After replacing the control board, it now prints flawlessly. Apparently,I really did damage the control board...

\n" }, { "Id": "3160", "CreationDate": "2016-12-09T00:31:53.750", "Body": "

I just want to ask if anyone has successfully printed a screw (M3 or M4). Is the printed output usable as a screw? What printer is capable of printing screws? I am using an M3D printer - is there a configuration to successfully print a screw that is usable?

\n\n

Can anyone share a picture of the best 3D printed screw?

\n", "Title": "Printing screws - is the output usable? (M3 or M4)", "Tags": "|fdm|quality|", "Answer": "

Vertically printed threaded bolts generally work better than you might expect, because (1) the layers are nearly parallel to the direction in which they have to slide, so the bolt can turn smoothly, and (2) when a typical bolt is sliced, the resulting layers are just circles, which are about the least problematic shape you could print.

\n

Also note that, especially if you're printing a bolt to use with a metal nut (or vice versa), then as long as the thread is close enough, it will end up being crushed / sheared into the right profile; it's a bit like how you can drive a screw into timber without having to tap a thread.

\n

In principle, the pitch of a printed bolt could be very fine; as long as the layer height is less than half the pitch (i.e. the Nyquist limit), and the spacing is correct, then the threads will engage. So if you can print 0.1mm layers, you have enough detail for M1 or smaller. Of course you're not going to print a bolt that small, but the point is, the limit is not resolution, it's the strength of the print.

\n

The horizontally-printed bolt in @R.. GitHub STOP HELPING ICE's answer is impressive \u2013\u00a0if you can print that, then that's the answer. But even if you can't, you could definitely print a horizontal 3mm diameter cylinder, and then use a die to cut the thread on the finished part. It's not a "pure" printed solution, but you can get a cheap tap-and-die set from any auto parts store for less than the cost of a spool of filament.

\n" }, { "Id": "3165", "CreationDate": "2016-12-11T18:41:40.657", "Body": "

I am planning a mechanical 40% keyboard build and are coincidentally on the home stretch of a homemade CNC project.

\n\n

The only thing the CNC needs to do for the keyboard project is to drill 7*48 holes. So what I need to do now is layout those holes in SVG. Therein lies the question. What resolution should I use for the SVG? I want to space the center of the keyboard switches 19 mm apart. An online pixel to mm converter suggested that 72 pixels is exactly 19.05 mm (which actually is what Cherry MX says should be their spacing).

\n\n

Now, I do understand that this really doesn't matter, but I am curious as I am new on CNCs and was suspecting that there is a number that will \"just work\".

\n\n

EDIT:\nFor example, if I where to print the template (SVG) on a regular printer, what pixel to mm ratio should I use so that it would come out the size I want?

\n", "Title": "What resolution (pixels/mm) should I use in SVG for CNC?", "Tags": "|resolution|", "Answer": "

I found one of those printer things that puts ink on dead trees and tested to print a simple SVG file.

\n\n
<svg xmlns=\"http://www.w3.org/2000/svg\"\n     width=\"400px\" height=\"800px\">\n  <rect x=\"10\" y=\"10\" width=\"72\" height=\"72\" fill=\"#999999\" />\n  <rect x=\"10\" y=\"100\" width=\"378\" height=\"378\" fill=\"#999999\" />\n</svg>\n
\n\n

As I suspected 72 pixels came out pretty much exactly 19mm. (72/19.05)*100~=378 came out 100mm.

\n\n

Given this I am going to assume that 72/19.05 is the de facto best pixel to mm ratio to use for CNC projects.

\n\n

EDIT:

\n\n

Found this documentation: http://w3.org/TR/SVG/coords.html#Units

\n\n
<svg xmlns=\"http://www.w3.org/2000/svg\"\n     width=\"400px\" height=\"800px\">\n  <rect x=\"10\" y=\"10\" width=\"19.05mm\" height=\"19.05mm\" fill=\"#999999\" />\n  <rect x=\"10\" y=\"100\" width=\"100mm\" height=\"100mm\" fill=\"#999999\" />\n</svg> \n
\n\n

Much simpler to use mm as units right away

\n" }, { "Id": "3178", "CreationDate": "2016-12-12T22:13:03.630", "Body": "

3D printing can be used to make injection molds of unimaginable complexity but which kind of 3D printing process is suitable when?

\n\n

Suppose that a part is to be made using injection molding in large quantity for an extended period of time, what Additive Manufacturing (AM) process will be the best, such that the mold does not give way too soon?

\n\n

Also, suppose that the part to be made is custom and only has to be made in small quantities - that is to say that the injection mold will be used limited number of items and then thrown away - which is the best AM technique then? Best in the sense of economic feasibility, lower cost, lower capital investment etc.?

\n", "Title": "3D printed mold techniques for long and short term usage", "Tags": "|print-quality|print-material|molds|", "Answer": "

Injection molds generally require tooling and are not usually cast from 3D prints. Depending on the size of your part - costs can easily skyrocket to over $60,000. The price per part is sometimes lower than other methods. Other possible methods (depending on the part) are Thermo-Forming, Vacu-Forming, and different Casting methods - sandcasting (investment casting), flexible casting (rubber, urethane, and alginate), and hard casting (printing mold negative). Some have immediate use of 3D Printed parts...others have work flows in place to convert your files for use. You can print a mold negative, prep it, use mold release, and cast shapes from that mold... certainly the casting material would be flexible or the mold would need the proper relief- But it is certainly possible.

\n" }, { "Id": "3185", "CreationDate": "2016-12-13T10:36:31.993", "Body": "

I have Prusa i3 derivative with MK8 extruder and Marlin 1.1RC8 as firmware. I already reduced the default speeds as well as the accelerations. But sometimes when trying to print with BQ PLA filament (220\u00b0C), mostly during filling areas, my extruder clicks. The below screenshot of Slic3rs Layers view shows the clicking \"lines\".

\n\n

\"enter

\n\n

What settings I need to change to avoid the overextrusion in this case?

\n", "Title": "Extruder clicks", "Tags": "|extruder|", "Answer": "

I had this issue and it turned out to be the extruder cog rubbing on the inside of the hole in the heatsink, causing the stepper to slip.

\n" }, { "Id": "3189", "CreationDate": "2016-12-13T16:10:42.753", "Body": "

I'm looking to buy my first 3D Printer, on a tight budget of $250. Unfortunately, this printer that I found on Amazon comes with all the bells and whistles, except for a heated bed.

\n\n

I want to know if this would affect printing severely, as I have read that the plastic/ filament cools down rather quickly, and that it results in Printer \"fails\".

\n\n

I'm actually a bit nervous with this buy, as I don't want to spend $250 on a printer that produces 90% print fails.

\n\n

An example of the printer I'm referring to is the Cube 3 Printer, Grey\nby 3D Systems.

\n", "Title": "Non-Heated Beds affect on 3D Prints", "Tags": "|heated-bed|", "Answer": "

At the risk of an opinionated answer...

\n\n

I would say that if money is tight, then get the correct printer in the first instance, en lieu of purchasing one that ends up giving you unsatisfactory prints, and that you then have to spend even more money on, in order to get prints of an acceptable quality.

\n\n

A heatbed, while not essential, is, IMHO, certainly desirable.

\n\n

Just a thought - Instead of purchasing a ready built printer on Amazon, have you considered building your own RepRap? If you look at a Bill Of Materials (BOM), and then purchase the parts individually from China, then not only will you end up with a good/better spec printer, for the same price, but you would also understand the mechanics a lot better. A good, and thorough, understanding the printer is essential for repairs, when (not if) the printer goes wrong (and believe me, it will). For a little over \u00a3200 I have managed to source all of the parts separately for a P3Steel v4 printer, with Kysan stepper motors, and an aluminium 200 mm x 300 mm heatbed.

\n\n

Note that I knew nothing about 3D printing, three months ago, so you do not need to be an expert, just do plenty of research beforehand.

\n\n

FWIW, my aluminium MK3 heatbed was \u00a319.99.

\n" }, { "Id": "3190", "CreationDate": "2016-12-13T17:08:38.697", "Body": "

I wanted to extend the E3D V6's extruder heater and the thermistor wires. Is it ok to solder extension wires or does it have to be crimp only! Also are regular wires ok or does it have to be some special cable?

\n", "Title": "Extending extruder heater and thermistor wires", "Tags": "|hotend|", "Answer": "

Soldering is fine, both for the thermistor wires as for the heater.

\n\n

The wires don't need to be particularly special, though there are some things you should keep in mind:

\n\n\n" }, { "Id": "3197", "CreationDate": "2016-12-14T07:37:37.493", "Body": "

Are there commercially available molecular 3D printers that can create catalytic structures like Zeolites, if not are there any research efforts underway in such field?

\n", "Title": "3D Molecular Printers", "Tags": "|print-material|", "Answer": "

The link I mentioned from popular mechanics is the state of molecular 3d printing. So yes the printer does exist and can do some really impressive things. However as the article states it is limited to what it can create right now. The technology sounds really interesting. However it might be a bit out of our knowledge as it doesn't seem to be widely available passed the scientific academia world.

\n\n

This link here talks about reactionware for Chemical Syntheses as well as a TED talk about it

\n" }, { "Id": "3198", "CreationDate": "2016-12-14T08:32:42.480", "Body": "

Just started doing some design. First project was a replacement handle for my angle grinder, so basically a hollow cylinder. I want to reduce the amount of material used in the print. I could sit here and punch holes through the handle with a smaller cylinder, or some other shape etc., but is there an easier way to do this?

\n\n

It must be a pretty common requirement, just like in the movies where the spies look at a photo and tell the tech to 'enhance ... enhance'. Ideally you could select a surface and overlay some sort of pattern to remove material.

\n", "Title": "Reducing amount of material", "Tags": "|material|", "Answer": "

\"Just like the movies\"-type tech typically means $$$.

\n\n

For those who do not own (legally or otherwise) expensive CAD software, it may be difficult to find an out-of-the-box solution. That's not to say that it can't be done.

\n\n

A close, readily available, solution would be to use a series of common CAD tools such as Shell and Scale. Between these functions, you should be able to scale a copy of your main solid object, then shell the new inner object. When sliced, the object should have a smaller \"inner wall\" and a hollow center. I would recommend this over 0% infill as a shell will not provide enough strength for the intended use you mentioned.

\n\n

There is a great article on My3DMatter.com that provides details and a very interesting set of charts, outlining cost, speed, strength, and quality distributions at varying layer heights and infills.

\n\n

Please Note: These results are going to differ between different material types, vendors, part shape, and infill pattern. These results were found printing from a MakerBot Replicator and presumably MakerBot-branded filament.

\n\n

\"Infill

\n" }, { "Id": "3199", "CreationDate": "2016-12-14T09:37:34.053", "Body": "

Ironing out all my worries before buying my first 3D printer.

\n\n

I'm looking at getting the da Vinci miniMaker 3D printer as my first 3D printer, but it doesn't come with a heated bed.

\n\n

On my previous question about heating beds effect on a print, I was told that I need to use a raft to compensate for the absence of a heated bed. What I want to know is, does the software that comes with the printer allow the creation of rafts whilst/prior to printing? Or will it create rafts if need be?

\n", "Title": "Does XYZPrinting software allow the creation of rafts when printing?", "Tags": "|3d-models|software|", "Answer": "

Ok. After watching some YouTube Videos, I would take it that the abovementioned software does include the option to create rafts when printing a model.

\n" }, { "Id": "3207", "CreationDate": "2016-12-15T14:42:18.353", "Body": "

I'm in the process of building the D-Bot core XY printer, and I was hoping to know if the Z-axis 'zero' is near the hot end or near the bottom of the printer furthest away from the hotend? In this printer the Z-platform moves up and down and the nozzle stays at the same height.

\n", "Title": "Is z-axis zero near the hotend?", "Tags": "|z-axis|", "Answer": "

Generally, Z-axis zero is when the hot end and the build plate are close to each other. Some printers keep the hot end up at the top, and bring the build plate up to reach it. Other printers keep the build plate at the bottom, and lower the hot end down to reach it. As such, \"Z axis zero\" doesn't specify top or bottom of the physical printer, it just specifies that the hot end is right up against the build plate, ready to print its first layer. In the printer you describe, where the hot end remains at the top, that's where Z zero is.

\n\n

In terms of the model being printed, Z axis zero is always the bottom of the model, the first layer to be deposited on the build plate. The numbers increase from there, either lifting the hot end up, or pushing the build plate down, depending on the design of the printer.

\n" }, { "Id": "3213", "CreationDate": "2016-12-16T18:28:38.123", "Body": "

When printing the first layer, the infill overlaps on just one side of my print. Thereupon there's a rough, and a lot higher, surface on the first few millimeters after the wall.

\n\n\n\n

I have tried to troubleshoot it, but I just found information about a problem when the infill isn't close enough to the wall everywhere.

\n\n

However, for me, the problem is the contrary and just on one side.

\n\n

\"Photo

\n", "Title": "First layer overlap on one side", "Tags": "|pla|layer-height|", "Answer": "

This may be a result of an unlevel build plate (OP did not specify if calibration was done at time of writing).

\n\n

If the area that is overlapping is higher (closer to the nozzle), the filament will be pushed down and around the nozzle as it extrudes in that area. This will result in excess filament overlapping unto other strands on the layer.

\n\n

Please excuse my lack of artistic skills in paint, but the image below should illustrate what can happen when your build plate is unlevel:

\n\n

\"enter

\n\n

Basically what it's trying to illustrate is that if the nozzle is closer than the expected layer height, the machine will continue to flow as if the nozzle is layer height away. This typically results in a larger layer width because the nozzle is essentially pushing material out of the way.

\n\n

You will notice that as you go further to the right in the drawing, that your layer may begin to \"thin out\" because if the nozzle more than a layer height away, the filament \"stretches\" until it settles on the build plate, resulting in a thinner layer width.

\n\n

Ideally, your nozzle will be parallel to your build plate at all points along the build space and the \"Standoff Distance\" will be equal to your layer height. So, you should see the top of your bead of filament at the same height as the bottom of your nozzle.

\n" }, { "Id": "3215", "CreationDate": "2016-12-17T10:39:48.277", "Body": "

When trying to print parts that should contain certain sized holes, e.g. for screws, how to achieve that they are sized correctly?

\n\n

Is it possible to calibrate the printer perfectly, so it prints holes correctly sizes in all common sizes (e.g. starting at 2mm diameter)? Or is it better to design the holes larger or print prototypes and increase the sizes according to the real prints?

\n", "Title": "Designing/Printing objects with sized holes", "Tags": "|calibration|", "Answer": "

I've had this issue too and I've pretty much solved it by making all holes polygons instead of round.

\n\n

The general rule of thumb I follow is to use a polygon with 4 times as many sides as the hole diameter in millimeters, e.g. for a hole with a 3 mm diameter I'd use a 12 sided polygon.

\n\n

Then it's a matter of choosing between a circumscribed (circle fits inside polygon) or an inscribed (polygon fits inside circle) polygon depending on what you're using the hole for.

\n" }, { "Id": "3219", "CreationDate": "2016-12-17T20:00:27.670", "Body": "

Is it possible to use the onboard camera on the 5th+ generation MakerBot printers to stream or save to a file?

\n\n

I'd like to try and use the stream for customers on my 3DHubs account and they recommend setting up a YouTube stream. However, even if I'm able to stream to VLC Player or something, I'd like to get the video/images somehow.

\n", "Title": "MakerBot Camera Stream", "Tags": "|makerbot|services|", "Answer": "

I own the gen 5, not the gen 5+. As far as I can tell, the camera has not gotten much of an upgrade and the software is also just as limited. That said to be sure I verified on several sites such as this reddit.

\n\n

There are some that claim that the software is open source. I do not think this is true given the parent company. Following even if it was you do not want to use their camera. It is shocking how cheap they went on this ultra premium printer. Which despite printing very well has not been received well. The main issue is that the camera is 320 by 240! That alone is enough to derail your quest I am afraid.

\n\n

Take a look at this raspberry pi project on building your own camera set up. Though really any wireless camera setup should do. Using a Pi has the stand alone advantage.

\n" }, { "Id": "3225", "CreationDate": "2016-12-19T22:10:02.343", "Body": "

To learn about printing, and to print the parts for a printer of my own design later on, I've ordered an Anet A8.

\n\n

I just saw this video, Anet A8 3D printer Review. all you need to know. This guy recommends using an external MOSFET. I was wondering if using a relay instead would also make it safe?

\n", "Title": "Anet board - MOSFET replacement?", "Tags": "|electronics|anet-a8|", "Answer": "

There are two areas of this design which seem to pose a risk of connectors overheating, the hot end and the heated bed. The heated bed runs at about 6 amps, 12 Volts. The hot end runs at a lower current, but can itself overheat if the thermistor stops providing feedback.

\n\n

The problem in this design with the heated bed is mainly the connectors - they work OK when they are good, but can easily come a bit loose. This has two side effects. The bed heats less well, and the connector itself heats instead. Replacing the 2mm plug connector, and 2mm molex with direct soldered connections will help (the on-board mosfet seems OK with the current, unless you increase the power rail voltage).

\n\n

A mosfet is a good solution for high current switching, the modules available have good screw terminals which should be OK for maybe 25A (well in excess of what you need). Mosfets are efficient when switched fully on, so there won't be much heating.

\n\n

You would also use a relay (rated for at least 10A), but this will be noisy since the heater is switched repeatedly to maintain the target temperature. A relay also takes more power itself and will ear out if repeatedly switched under load.

\n\n

Bottom line, if you have a good relay already, it is OK as a temporary (safe) solution. Longer-term a mosfet is better. Pay attention to the connectors at power supply, board (in and out) and heated bed. Using an additional mosfet for the hotend is maybe overkill.

\n" }, { "Id": "3238", "CreationDate": "2016-12-20T21:34:28.330", "Body": "

I am planning on buying a cheap 3D printer to get into 3D printing, but the printer I'm planning to buy only takes 1.75\u00a0mm filament, I was wondering if it might be possible to change the hotend of that printer or something to take in 3.0\u00a0mm filament, the reason I want to use 3.0\u00a0mm filament is because it is cheaper than 1.75\u00a0mm filament.

\n", "Title": "Change 1.75 mm 3D printer to 3.0 mm printer", "Tags": "|filament|hotend|nozzle|", "Answer": "

It can be quite straightforward to convert from 1.75\u00a0mm to 2.85 or 3.0\u00a0mm, but it requires to buy hardware specifically for larger diameter filament. Nowadays, in 2020, Ultimaker is still using 2.85\u00a0mm filament which is not more expensive than the smaller diameter (at least not for the premium brands I'm using). I converted to the larger diameter filament years ago to be able to exchange filament with an Ultimaker 3E I maintain and manage.

\n

To convert to a larger diameter, you need to be aware of

\n\n

Thingiverse has always been a great starting point for me for larger diameter filament extruders; my own custom extruders are based on such designs. It is perfectly possible to convert to a different filament diameter, but, it requires some money, time, elbow grease and engineering from your side.

\n" }, { "Id": "3244", "CreationDate": "2016-12-21T12:56:42.090", "Body": "

I'm looking at getting this printer, the da Vinci 1.0w 3D Printer, very soon as my first printer. Since this is an enclosed printer, from what I can tell from the pictures, and given that it is a PLA Printer (I'm assuming that is the filament it prints with), is a heated bed necessary? Especially since this printer doesn't have one? Or should I look at a ABS printer instead?

\n\n

I plan on printing 1:1 scale props.

\n", "Title": "Is a heated bed necessary if printing with PLA?", "Tags": "|filament|pla|heated-bed|enclosure|", "Answer": "

There is problem with sticking to bed without preheat, PLA is possible to print on Kapton tape with no-heated bed. Or there are other tapes dedicated for printing. da Vinci 1.0w is good for small models with PLA. ABS is not possible with no heated bed.

\n" }, { "Id": "3263", "CreationDate": "2016-12-23T17:07:50.837", "Body": "

tried searching but couldn't find anything.\nI do not have a 3d printer so can't really experiment on my own, which means that when I am going to order a 3d print I want to get it as good as possible. So, my question:

\n\n

Do quality of geometry matters when 3d printing? Will 3d printer only print quads, or ngons are fine? Are there shapes to avoid?

\n\n

Cheers :)\nM.

\n", "Title": "Mesh/Geometry quality", "Tags": "|print-quality|3d-models|3d-design|cad|", "Answer": "

If you're talking about the shape of the end result, rather than the constituent elements of the model - the answer is no, there is no simple geometric restriction. Have a look at 3D benchy for an example of how print quality can be affected by different aspects.

\n\n

One obvious issue is overhangs, so the orientation of the part is important for printing. A flat circle will (on a cartesian printer) come out smooth as X any Y move in sync, and have good support. A vertical circle will have steps introduced by the slicing which quantises X and Y from layer to latyer.

\n\n

Sharp, un-supported corners may be the weakest aspect to resolve in the print - extruded filament tends to shrink as it cools, but as far as I know the errors like this can be reduced by printing more slowly (and reducing the dynamic flexing of the frame too).

\n\n

Talking about a top surface of an extruded n-gon, if you look at the slicing output, you'll see infill in the bulk, with a layer filling only the top 3 or 4 layers. There are several infill patern options, but yes, there is scope for a scenario where the top layer needs to bridge a long way. However, since the alternate layers fill in orthogonal directions this should be a minimal effect. Tweaking the infill percentage, or chamfering the corners can fix these small defects.

\n" }, { "Id": "3265", "CreationDate": "2016-12-24T03:13:41.090", "Body": "

I'm trying to design a camera handle, which will be around 8\" long and will have a brass camera thread insert in the end, where the camera will be mounted. (That way, I don't have to screw the camera thread into plastic which will wear out faster.)

\n\n

If I print the handle normally, the end of the handle won't be solid so I can't solidly put that brass fitting in. If I set the fill in Cura to 100%, the print will take a very long time and will be unnecessarily solid. I only need a centimeter or two at the end to be solid.

\n\n

Is there a way to get one particular wall in Cura to be very thick (1-2cm) without affecting the other walls? Is there some other way to get a solid chunk in the end of the part?

\n", "Title": "Thick wall or solid part of an otherwise medium-fill part", "Tags": "|ultimaker-cura|infill|", "Answer": "

I believe the solution is to use more walls in Cura.

\n\n

Here is a 50mm cylinder with a 20mm hole. I specified 10 walls in Cura.

\n\n

This should give extra plastic for the screw to bit into, but not take all day to print!

\n\n

\"enter

\n" }, { "Id": "3275", "CreationDate": "2016-12-25T21:04:16.493", "Body": "

I have just got a Monoprice Maker Select 3d printer (which is really just a Wanhao Duplicator i3).

\n\n

I want to print a model that came on the included SD card (1.gcode) with ABS filament. I first press \"Preheat ABS\" and wait for the display to show the correct temperatures, 245\u00b0C for the extruder and 90\u00b0C for the print bed. Then, I mount the SD card and select the file to print.

\n\n

However, once I do that, the \"goal\" temperatures change to 195\u00b0C for the extruder and 50\u00b0C for the print bed, and the temperatures reported by the printer gradually go down as the printer waits for it to match the \"goal\" temperature. This results in a failed print, with the filament sticking to the extruder in a clump.

\n\n

What is going on here? I am trying PLA with the same file, and it seems to work fine, although the temperature change still occurs.

\n", "Title": "Printer changing temperature after selecting file", "Tags": "|abs|monoprice-maker-select|", "Answer": "

If you want to print the demo files with ABS rather than PLA, as the files are designed to do, I recommend loading the file into CURA and saving out a new gcode file with the adjusted temperatures. You can load the file from gcode, which will give you the basic settings that are included in the PLA file. I would also recommend slowing down the first layer a bit as adhesion of the initial layer is much more important for ABS than PLA, as ABS tends to warp more. Also you should reduce, if not turn off your cooling fans. Then once all the settings are where you need them to be for ABS, just export out a new file and probably change the name to include _ABS so you know the difference when you're mounting the card.

\n" }, { "Id": "3278", "CreationDate": "2016-12-26T09:46:12.370", "Body": "

This is my problem:

\n

I'm assembling a 3D printer with the RAMPS 1.4 board and Arduino Mega. I have assembled the structure and the electronics (set drivers, placed the jumpers, connected stepper motors...) and have uploaded Marlin firmware (configuring: thermistor, endstops...) on the Arduino Mega.

\n

I've tried to connect, via USB, to the computer and using the Repetier software I have commanded the printer which did do some movement. The printer worked perfectly. After a few tests, however, I've noticed that the Arduino was restarting several times and at one point I saw a component on the Arduino board burning. Searching the internet I saw that the burned component was the voltage regulator.

\n

I heard also about unplugging the screen because it consumes a lot of current, that passes through the voltage regulator, thereby heating it up. I then proceeded to buy another Arduino Mega. I also checked the voltage of my 12V 360W power supply and it is correct. I always powered the Arduino with USB and RAMPS 1.4 was connected to the power supply from the two terminals.

\n

\"Power

\n

When the new Arduino arrived, I connected the whole (without connecting the screen) and tried. The printer worked very well until it started giving the same problem as the first Arduino board. Someone can tell me if I have done something wrong, or is it the RAMPS board that does not work properly?

\n

EDIT:

\n

I read that the endstops can cause this problem. I have these endstops: 1 PZ di Alta Qualit\u00e0 Finecorsa Meccanico Per rampe Reprap 1.4 stampante 3D Con imballaggio indipendente kit fai da te and I connected the black wire to GND, red wire to 5V and the green wire to SIGNAL.

\n", "Title": "Arduino Mega voltage regulator overheats with RAMPS board", "Tags": "|reprap|ramps-1.4|", "Answer": "

I ordered an authentic Arduino Mega today after the voltage regulator on my Chinese clone Mega went up in smoke. But I couldn't wait two whole days so I dug around and found an old 7805 (less than US$1 new btw) and soldered it in. It worked! Just one more possible solution. Actually that's two if you count buying a real Arduino.\"7805

\n" }, { "Id": "3283", "CreationDate": "2016-12-27T17:56:50.160", "Body": "

I have never much cared about self intersecting meshes when slicing with Cura. Geometry like the one below are often practical. I for instance add lots of rivets that self intersect with the base geometry:

\n\n

\"self

\n\n

Now I have switched printer, and am using Simplify3D instead. All of a sudden, I get lots of problems with these models. The intersection of the objects become hollow. Simplify3D has a setting to join the outer solid shell but it also fills holes (such as the center hole of a gear).

\n\n

I make models to sell so this is a big deal for me.

\n\n

EDIT: Also, they printed perfectly fine in formlabs \"slicer\".

\n", "Title": "Are self intersecting meshes dirty from a 3D printing perspective?", "Tags": "|3d-models|3d-design|ultimaker-cura|simplify3d|", "Answer": "

You definitely need to get rid of the self-intersecting meshes in order to be able to 3D print your object. I would personally recommend you try MakePrintable: automatically clean these type of meshes.

\n\n

Makeprintable is a cloud service (your model is uploaded to their servers and processed there) currently in beta. You need to sign up but it is free (and they state that they will keep it free).

\n" }, { "Id": "3285", "CreationDate": "2016-12-27T22:12:26.510", "Body": "

I just got a Hictop Prusa i3 printer and I have it fully assembled. When I tested out the motors to check them the Z-axis motors were not moving and it was making a grinding sound. I have lifted the screw rods out of the coupling to see if the motor would move the coupling would move and it did. I can move the screw rods manually and it works. How can I make the Z-axis work?

\n\n

Thank you in Advance!

\n\n

Edit

\n\n

Here is the vidoe of the problem https://www.dropbox.com/s/93g0pg0qfhq965d/IMG_0369.MOV?dl=0

\n\n

\"enter

\n", "Title": "Prusa i3 Z axis not moving up", "Tags": "|prusa-i3|printer-building|z-axis|motor|", "Answer": "

Welcome to the group! A video would help. Or at least some photos.

\n\n

Are both the motors connected? If you remove them from the coupling do they move? Likely it is binding (too much friction, not level etc) or you need to adjust your voltage controller. I am going to say 80% confident you need to play with the voltage.

\n\n

Chances are it's just the voltage. I do not know what electronics you have but if they are RAMPS 1.4 then you are looking for these

\n\n

\"enter

\n\n

Also here is the wiki on the 1.4. Just turn the screw gently. One direction will give it more power. The other less. When it has too much power your motors will start making a thud noise.

\n\n

EDIT post adding the video

\n\n

Oh yeah that is binding. If you wouldn't mind putting the video on youtube to that the video can help people for years to come? Dropbox is a bit volitle.

\n\n

I would also in addition to my advice in your other question take a bubble leveler to all the rods. It could be a distortion on your camera but it looks like the rod is a bit bent. Also in my Prusa (original) I used to have to make the motor mounts lose, as it did not fit all that well and had binding issues. Try making the screws loose enough so that they float and can move around a lot. The lead screws are less important to the overall stability.

\n" }, { "Id": "3288", "CreationDate": "2016-12-28T07:53:40.060", "Body": "

I'm new with my 3D printer, I just print two different pawn pieces from thingverse. I just used Cura to convert the files to be readable for the printer. Is my problem with the pieces has to do with the configuration from the Cura software? or with my printer itself?

\n\n

\"Print

\n\n

Update:

\n\n

I just printed a baymax that came with the SD printer. And it looks awesome. I think the problem is with the configuration from Cura.

\n\n

\"Print

\n", "Title": "3D print Configuration Cura - Anet A8", "Tags": "|print-quality|fdm|anet-a8|", "Answer": "

Underextrusion (as this looks like) could be a extruder problem. The extruders main part is the small gear that pushes the filament through the hotend. There could be lots of reasons for it to fail:

\n\n\n\n

If you don't have a geared extruder you can print one. There are plenty of them on thingiverse. Increase the temp and lower the speed to get this vital part in good quality.

\n\n

Or you can buy a geared extruder. (I use the titan extruder from e3d)\nOr you can buy a geared motor.

\n\n

How to detect an extrusion problem:

\n\n

It is quite easy to hear if you have extrusion problems (at least some of them). The Extruder will make a sound like \"drrrr\" with regular intervalls as it tries to push through the filament. Almost like a clock ticking but slower. This is the gear slipping against the filament.

\n\n

If you don't have an extrusion problem:\nExperiment with upping the Material flow rate in your slicer (in your case Cura). Try something like 300%. Stand watch and see what happens.

\n\n

\"enter

\n\n

If this almost fixes the problem, you must calibrate the printers extrusion. It is possible that the already generated Gcode (found on the sd-card) compensates for this.

\n" }, { "Id": "3291", "CreationDate": "2016-12-28T14:14:56.003", "Body": "

Hello is there a way to prevent bend on print with M3D printer?

\n\n

\"enter

\n", "Title": "How to prevent bend (or warping) with M3D printer?", "Tags": "|warping|", "Answer": "

Try using an adhesive before you print. This could be http://airwolf3d.com/shop/wolfbite-prevents-3d-printed-parts-from-warping ... What is the temperature of the room you are printing in like?

\n" }, { "Id": "3294", "CreationDate": "2016-12-28T21:14:16.630", "Body": "

I recently had a problem with the z-axis of my printer. To resolve the issue with the Z axis not moving I remove the left polished rod. Am I able to use the printer with just one smooth rod?

\n", "Title": "Is there a problem with removing a polished rod from the Z-axis?", "Tags": "|z-axis|", "Answer": "

It may help someone I am just noting that all your comments are valid. I was reading through forums to find a solution to the binding problem. After calibrating, levelling, and adjusting, what I found out was rather simple. The two motors came out of phase (not sure how).

\n\n

Here are the steps I took.

\n\n
    \n
  1. I unplugged the motor on the left and the other motor was driving the Z axis just fine.
  2. \n
  3. I unplugged the motor on the right and the other motor was driving the Z axis just fine.
  4. \n
\n\n

I started thinking in terms of my power supply providing enough current to drive both motors and then I rebooted the controller. When the system came back both motors were driving in synch.

\n" }, { "Id": "3302", "CreationDate": "2017-01-01T14:49:33.857", "Body": "

I want to create parts for a 3D printer using OpenSCAD. Having some STL files from the vendor, but nothing else (no technical drawing, no CAD files).

\n\n

Does anybody knows a free tool, that allows me to

\n\n\n\n

I very much like the way Blender allows to work with meshes, especially select vertices or planes, but unfortunately haven't found a way to measure with Blender.

\n", "Title": "Tool to measure in STL files", "Tags": "|3d-models|software|blender|", "Answer": "

I recommend Ultimaker Cura 4.3 for dimensioning.

\n

You can access it here.

\n

\"Screenshot

\n

As can be seen, detailed measurements can be made over the specified coordinates.

\n

\"Screenshot

\n

However, you can make an accurate dimensioning by using the Render program. I recommend doing this before working on the STL file.

\n

I wish you good work.

\n" }, { "Id": "3304", "CreationDate": "2017-01-01T22:41:58.273", "Body": "
Nozzle diameter = .4\nExtrusion multiplier = 1\nExtrusion Width = .45    <-- I feel like this could be reduced to fix it?\n\nLayer Height = .3\n
\n\n

I'm using Simplify3D.

\n\n

\"enter\n\"enter

\n", "Title": "What causes these round corners", "Tags": "|extrusion|", "Answer": "

Enable "classic jerk" in the firmware and put it to 12 on both X and Y.

\n

That helped me get perfect prints.

\n" }, { "Id": "3312", "CreationDate": "2017-01-03T21:21:46.437", "Body": "

I have a derivative of the P3Steel v4, which is not the Irobri design but another version (I'm not entirely sure which), from Poland: Frame Prusa I3 P3Steel v4.0 +RODS.

\n\n

I have assembled the frame, following this guide, manual_p3steel_xl, only to be left with one part, which is the long thin rod, which has a small hole, only in one end (the hole is marked by the big arrow in the image below):

\n\n

\"P3Steel

\n\n

This part obviously goes into the slots at the bottom of the Z axis arms, as shown by the curved arrows.

\n\n

However:

\n\n\n\n

I have looked at a number of other photos and videos for the P3Steel frame, and none of them clarify which way around, or how, the rod should be fitted. The bar is either omitted completely, or the image is not clear enough to show the correct orientation.

\n\n

The guide does not show this part being used, although it clearly shows the slots for it in the Z axis arms, in a number of the pictures, such as here:

\n\n

\"Screenshot

\n", "Title": "Orientation of long thin rod on P3Steel v4", "Tags": "|printer-building|p3steel|", "Answer": "

This is half of an answer. I have finally received an answer from my supplier, which says:

\n\n
\n

Long thin rod is mounted as in the photo. I do not know what is a\n small hole.

\n
\n\n

So the small hole is still a mystery.

\n\n

Here is the photo that was included with the message (which confirms my placement, just not the orientation):

\n\n

\"Long

\n\n
\n\n

Update

\n\n

As tjb1 points out in his comment, the hole can be clearly seen on the right hand side of the bar, in the eBay photo:

\n\n

\"Photo

\n" }, { "Id": "3324", "CreationDate": "2017-01-05T02:22:18.017", "Body": "

I'm new to 3d printing, so I might be missing something obvious. If so, please let me know.

\n\n

I printed my model successfully yesterday, but today I'm trying to print the same model and the first layer refuses to adhere, which means at best several layers after that are messed up if it manages to recover, but usually it just means I have to cancel and start again.

\n\n

It will print one horizontal line (across the x axis), then when it tries to vertical line (across the y axis) the horizontal line doesn't adhere and gets dragged along with the print head and everything is screwed up.

\n\n

I've tried leveling the bed over and over again. (I use a sheet of paper and try to slip it between the bed and printhead. I adjust the bed so that I feel a bit of resistance as I push and pull the paper under the printhead.)

\n\n

I've tried increasing the preheat on the printhead and on the bed. I'm using black PLA 1.75mm that says it has a print temp of 205-225. I've tried printing at 205, 210, 215, 220, and 225. I've tried a bed temp of 50, 55, 60, 65, and 70.

\n\n

I've tried setting the print speed multiplier to 0.5 to give it time to adhere, but no changes.

\n\n

I'm trying to print something a wireframe cube that is at the extent of my printable size, so I don't know of a way to use a raft or a brim to help adhesion.

\n\n

This is what my model looks like:

\n\n

\"my

\n\n

When I printed a good one yesterday, here is what the first two lines looked like:

\n\n

\"good

\n\n

When I print today, even after multiple attempts to level the bed, this is what the first layer tends to look like:

\n\n

\"enter

\n\n

I'm using a Monoprice 15365. I created my model in SketchUp, then exported as STL, which I imported into Cura 2.3.1. Then I used Cura to export a gcode file to an SD card. I put the SD card into my 3d printer and printed from there.

\n\n

Any advice is welcome. I don't know if the problems I'm having are because the bed is too low or too high or too hot or too cold or if the printhead is too hot or too cold... nothing I've tried seems to change the results.

\n", "Title": "Can't get print to stick, no matter what I try", "Tags": "|build-plate|adhesion|", "Answer": "

Eureka! All of the other answers and suggestions on this post were all very helpful, and would likely be the right answers 90+% of the time, but they didn't give me any relief. I spent a bunch of time watching YouTube videos about 3D printing and I started to notice a difference in size of the lines of plastic being laid down on the build plate.

\n\n

In the videos, the lines of plastic looked bigger. Mine were very fine lines. That lead me down a series of different search terms until I found some other videos that talked about nozzle cleaning.

\n\n

I noticed that one of the symptoms they mentioned was that when the plastic would come out of the nozzle that it would curve around. That's a sign that it's partially blocked and causing it to bend and curve. After the people in the YouTube videos cleaned their nozzle, the plastic would then extrude and drop straight down.

\n\n

If my nozzle was partially blocked and only about half the right amount of plastic was coming out that would make sense since there wouldn't be enough surface area of the plastic to make a good connection to the build surface.

\n\n

I acquired some 0.4mm drill bits from Amazon and followed the instructions on this YouTube video.

\n\n

Sure enough, when using the 0.4mm drill bits, I could tell the nozzle was clogged. After clearing it out, the plastic started extruding in a straight line down, just like the video. I was stoked!

\n\n

I tried another print and it was NIGHT AND DAY. I was finally getting nice plump lines that stuck to the build plate. I still had an issue or two on certain parts of my build plate, but I think that's because in all of my tries I had damaged the masking tape. I put down new blue tape and started up my print and it's BEAUTIFUL.

\n\n

Thank you all very much for your help. Hopefully, this will be helpful to someone else as well.

\n" }, { "Id": "3332", "CreationDate": "2017-01-05T18:57:51.187", "Body": "

Here is the X-axis of the P3Steel:

\n

\"X-axis

\n

The X-axis idler end of a P3Steel printer, employs an 8 mm diameter rod for the axle on which a 608zz bearing is mounted for the GT2 belt. This 8 mm rod is approximately 20 - 24 mm in length, with grooves at either end, for circlips.

\n

A photo of the assembled idler, with the rod and circlips highlighted:

\n

\"X-axis

\n

My question is: Does this part need to be custom made?

\n

The short rod didn't come with the frame kit that I purchased (nor was it listed in the parts list, or shown in the photo of the parts - so it not as if it was omitted with my order). I have searched on eBay for it, using various search terms, and I am unable to find one. As I was not trained in mechanical engineering, I am not sure if this part has a special name, or is it just called a "smooth rod, with grooved ends"? I have also done a fair amount of googling, and although I have found some people who have constructed this particular frame, no one makes mention of this axle, nor any difficulties in sourcing it.

\n

I have contacted the supplier of the frame, Frame Prusa I3 P3Steel v4.0 +RODS, and I am awaiting a reply.

\n

This missing part is holding up my build progress - I already have the 608zz bearing and circlips.

\n
\n

Additional images

\n

This image shows the "exploded view" and the 8mm rod can be clearly seen:

\n

\"Exploded

\n

Here are images of the assembled idler, showing the bearing inside -

\n

Front view:

\n

\"Front

\n

Side view:

\n

\"Side

\n

This images are a little blurry, as they are screen shots taken from the video, I3 Steel CORDOBESA con extrusor/with extruder.

\n", "Title": "Is the 8mm x 20mm bearing axle for the X-axis idler (of a P3Steel) a custom part?", "Tags": "|printer-building|p3steel|", "Answer": "

You could look up a Clevis pin with one groove.
\nYou could look up a Clevis pin with a hole for a split pin.
\nPerhaps a Shoulder screw with a ground shank and a low profile head.
\nUse a plain rod with Dome caps if you will not need to remove often.
\nIf available an internal threaded Standoff would work.
\nA Slotted spring pin may work if the hole dimensions are suitable.

\n\n

EDIT:
\nYou could also cut the grooves yourself pretty easily. Cut a section of 8mm rod to length and mount it into a drill chuck so it stick out 2-3mm 1/8\". Hold a hacksaw at the edge of the chuck and run the drill for a minute with gentle pressure, try hacksaw on other side or reverse direction if nothing is happening.
\nA hardened rod will cut better with a Dremel type cut-off disk

\n\n

\"enter

\n" }, { "Id": "3346", "CreationDate": "2017-01-06T19:24:53.300", "Body": "

I'm experiencing an issue with my Makerbot Replicator 2X (2000+ hours printing). The dimensions of my final part are smaller than what is initially modeled. My parts have been coming out roughly 0.3mm smaller than what is initially modeled. Is this a common issue or does this sound more like I may have a hardware issue?

\n\n

I've confirmed that all my belts are tensioned properly and lubricated correctly. I haven't seen a calibration step that would correct for this issue short of manually scaling my parts to correct for the offset. The offset appears to only occur in the XY axis, never seems to happen in Z though

\n\n

Example: \nIn the part modeled I had the width designed to be 35 mm (face to face, parallel walls). It doesn't seem to matter what size of part I have the final dimensions always seem to be off by about the same amount.

\n\n

\"35mm

\n", "Title": "Dimensions off on final part", "Tags": "|makerbot|calibration|", "Answer": "

Yup that is what happens. It is simply the plastic cooling and shrinking. It will happen on just about any printer. 0.3mm on a what 40mm part. That is 99.3% on target.

\n\n

There are some great blog links about it, \nand here is a Stack overflow where I talk about it more in detail.

\n\n

The only mitigations I can think of is\n1) use a hear chamber.\n2) use a SLA 3d printer.

\n\n

I wouldn't worry about it. Just make sure your designs have good tolerances.

\n" }, { "Id": "3351", "CreationDate": "2017-01-07T17:25:53.840", "Body": "

I bought an Anet A8 over christmas. When I bought my printer from Gearbest, I also bought the inductive sensor that they sell.

\n\n

It doesn't seem to work, and I think it might be broken. However, I have no idea how I can test if it is broken. This is what the wiring looks like and how it's connected to the board

\n\n

I'm not sure if I need to modify the wiring in order to hook it up to the A8's motherboard. I have no idea what the pinout is on the A8's mobo, and it's not indicated either. Most guides deal with using RAMPS when it comes to installing such a sensor. I'd like to know how to connect this to my board, if possible.

\n\n

The printer works fine otherwise, so I don't think the board is broken or anything. For now I've resumed using the normal endstop, but I would like to enable bed levelling by installing this sensor. (or a different one if necessary)

\n\n

How do I get this working?

\n", "Title": "Connecting an inductive sensor to the Anet A8", "Tags": "|electronics|anet-a8|", "Answer": "

I'm guessing you have the Anet sensor? If so and you are using the stock firmware that came with your A8 - it's not going to recognise the sensor but you can download and flash the Anet A8L firmware that is designed to run with the sensor from the Anet site - I stumbled on this while searching for stock firmware.

\n\n

I hope this helps you out.

\n" }, { "Id": "3353", "CreationDate": "2017-01-07T21:09:20.083", "Body": "

I would like to calculate the amount of steps to turn the motor to have 1 mm of the filament.

\n\n

I did the following (using approximate values):

\n\n

The gear on the outside of the teeth as a diameter of 11mm.\nTherefore the circumference is 34.56mm. Divided into one degree i get 0.096mm / degree. The stepper does 1.8 degree per step which results in 0.1728mm per step.

\n\n

To get 1mm of filament pushed into the extruder I will have to do 5.787 steps.

\n\n

The same extruder is in a Geeetech i3. The firmware there is configured to do 93 steps per millimeter.

\n\n

I don't have any experience in working with stepper motors but in theory, this would be my approach with a huge difference to an existing firmware.

\n\n

Where is my mistake?

\n", "Title": "Calculating extruder motor steps for 1 mm", "Tags": "|filament|extruder|", "Answer": "

Microstepping. Your board likely has 16x microstepping, so that each step is divided up into 16 parts. Your 5.787 figure, when multiplied by 16, comes out to 92.6 steps/mm - matching the value in your firmware.

\n" }, { "Id": "3356", "CreationDate": "2017-01-08T18:50:41.707", "Body": "

I received a roll of Black filament as a present, and struggled to print with it, experiencing extreme under-extrusion (and slippage of the hobbed gear). Higher temperatures seemed to work better, particularly in freeing up the extrude rate.

\n\n

The filament is advertised as PLA, but without any temperature recommendations that I can find on the vendor's website (or the amazon listing). Amazon reviews have a fairly wide range of temperatures, but the listing aggregates several colours. There is a suggestion that the filament may be PETG, or ABS.

\n\n

I have tried using acetone, and it dissolves roughly the same as some ABS I have, my PLA shows some weakening but doesn't dissolve.

\n\n

Below about 215\u00b0C, extrusion is borderline non-existent. I printed at 225\u00b0C with a fan, and it extruded kind of OK, but adhesion was poor. 240\u00b0 without a fan seems a bit on the hot side (see photo). Even 255\u00b0C seems to extrude OK without burning. There is no smell I can observe (but I have a cold). It burns with a small blue flame, and minimal smell (kind of like natural gas).

\n\n

One factor against it being ABS is that with the bed at 60\u00b0C, printing both a benchy and the heat tower below, there was no warping. Bed adhesion remained good.

\n\n

Is there anything I can do to better identify this filament and decide the best combination of fan/temperature without wasting too much time?

\n\n

255\u00b0 to 225\u00b0 heat tower, seems different to the ABS i've used:\n\"enter

\n", "Title": "Identifying PLA/PETG or ABS", "Tags": "|filament|part-identification|", "Answer": "

There is a trick. However I would like to mention that chances are that the random spool you pickup from most stores, especially no name brands, are either completely lying to you about its material or is mixed / cut with cheaper fillers. All about finding a good brand. Course rarely does one actually need pure PLA, a mixed material will often perform as well without you even noticing. Aside from the temp difference. Not to say it is right.

\n\n

Now the trick is this. While I don't advise ever breathing in potentially carcinogenic plastic smoke. You take a lighter. Burn it. If it smells sweet it is PLA.

\n\n

From this link from digital trends about PLA VS ABS you can see why it has this smell.

\n\n
\n

The thermoplastic is also more pleasant on the nose, as the sugar-based material smells slightly sweet when heated opposed to the harsh smell often associated with ABS. However, while PLA might seem like a better overall choice at first glance, it features a far lower melting point than ABS.

\n
\n\n

Also not responsible if you burn yourself or your property. At your own risk.

\n\n

Just warming it with lighter might work too. But just burning a small section. It will be real clear.

\n\n

Or you could just drop a bit into aceton and see if it melts. PLA will mostly just look like crud after being treated is aceton. ABS is 100% dissolve.

\n\n

We see from these forums that PETG does not dissolve via Aceton. So you can test you material. If it does disolve it is ABS. If it smells sweet when burned it is PLA. If it melts at 240 and does not dissolve it is likely PETG.

\n\n\n\n

We are a bit lost on a test for PETG. However we do know its melting point. Which is 240-260c. I would say it is likely you have ABS

\n\n
\n\n

Once we know the material we can get to the meat of your question.

\n\n

Best way to proceed is to start with simple calibration prints. The thin wall. Cube, and part fitting. Once those pass you are good to go. Other than buying simplify3d I do not know of any short cuts. Back when I used slic3r I kept a spread sheet. I would change a temp by 5 degrees and give it a quality score. Repeat with speeds. Far as fan, I would expect that it wont effect much. Maybe need to calibrate your PIDs.

\n\n

Once you have it dialed in, all you will need to do is calibrate the temp when switching materials. Remember you need to calibrate even between different colors (okay its best to.. I often skip) I usually do the temp adjustments live on the printer. Increase and degrease a few degrees. For me the goal is to have the plastic as cool as possible. That allows for highest quality prints (ie less oozing and such)

\n" }, { "Id": "3361", "CreationDate": "2017-01-09T03:42:51.423", "Body": "

The right side of my prints always have the standard ribbing effect given by a bent z rod, however I have replaced the rod and there was no change. Here are pictures of one of my more recent prints:\n\"\"\nand the left side of that same print:\n\"\"\nAs you can see the left is almost unaffected the right is the only side with the error. Anyone have any ideas about this?

\n", "Title": "3d prints come up with ribbing on one side", "Tags": "|print-quality|prusa-i3|z-axis|", "Answer": "

Hmmmm In my experience when a printer has that bad of wobble it is still the Z axis. Your rods might be good, but are the two Z axis the exact correct distance and equal distance on the printer frame? Imagine that you had one a few millimeters too far out. How might the rods act? Mayhaps, over the course of several layers, they would slowly drift back and forth? Creating a period effect on your print? What do the top of the rods do over time? Do they move a lot if you go from height 0 to 100mm on z?

\n

I have a few ideas. What happens if you loosen you Z motors so they can move around their mount? I had to do that on my Prusa back in the day. I think for that machine the Belt was actually too tight causing the issue. Current printers are usually better designed so its unlikely that is your issue. Also measure the distance between the top of the rods when the carriage is at 0, and at max. You should see very little variance.

\n

If it was on both sides I would say it was bad PID calibration. IE this simplify 3d link

\n

Though you can see on this forum the fellow had a bad coupler. So still mechanical.

\n

Also could be back lash / belt lash.. but unlikely

\n" }, { "Id": "3363", "CreationDate": "2017-01-09T16:06:26.000", "Body": "

I've build a 3D printer from sourced parts and mounted the hotend cooler to blow air over the heatsink.

\n\n

Talking to a friend, he said it's better to reverse the airflow over the heatsink, but couldn't give me an argument other than everywhere he saw it was like this: all coolers are mount to suck the hot air away from the heatsink.

\n\n

Is it one way better than the other way ? And if so, why ?

\n", "Title": "Blow vs suck air over hotend's heatsink", "Tags": "|hotend|cooling|", "Answer": "

Getting hot air from the radiator is definitely wrong idea because of few reasons:

\n\n
    \n
  1. hot air can damage your fan (as they are usually not heatproof). Cold air cools the fan
  2. \n
  3. cold air is denser so fan can suck more cold air than hot air so cooling is more efficient (fig A)
  4. \n
  5. in terms of plug of radiator (with dust) it's much better to try to push cold air into plugged fan (fig C) than to suck the air from it (fig B). When flow is decreased there is higher possibility to damage the fan as it doesn't cool itself
  6. \n
\n\n

Of course we are talking about 3D printers with small plastic fans. Please notice (almost) all fans construction: they are designed to get air from the top of the fan and push it to the bottom (this construction prevents damaging/plugging fan motor by dust)... and their mounting wholes (or clips) are usually... naturally suggesting to direct air to the heatsink)

\n\n

\"enter

\n" }, { "Id": "3366", "CreationDate": "2017-01-09T16:25:36.013", "Body": "

In this question I was told that I should use silver solder to connect the heating element to the power supply.

\n\n

(I was also told that a ceramic extruder head was the way to go, but I'm working with what I have)

\n\n

I bought two types of silver solider from Radio Shack:

\n\n\n\n

Is there any reason I should use one of these over the other to power the heading element of the J-Head extruder?

\n", "Title": "Silver Solder specifications for hot end heating element", "Tags": "|extruder|prusa-i3-rework|", "Answer": "

I can't comment yet, but for those wondering, the issue with tinning the wires is when you are clamping them. Tinning them actually increases the resistance between the wires and terminal, due to making them harder and not getting squished out to make more contact with the terminal. This increased resistance means increased heat, and enough heat means fire.

\n\n
\n

The reason for the prohibition is that when you fully tin a multistrand wire fully, the solder wicks between the strands of copper and forms a solid block, part of whose volume is metallic solder. When you clamp the solder and copper bundle you tighten the screw or clamp against the solder block, and in time the solder metal \"creeps\" under the compressive forces and the join loses tension. The wire can then either pull out or cause a high resistance connection with heating.

\n
\n\n

Source

\n" }, { "Id": "3370", "CreationDate": "2017-01-09T20:30:13.633", "Body": "

Is there an FDM 3D printing material that looks similar to a cardboard box? I guess, and understand, it would not be possible to 3D print a cardboard box but would there be something similar?

\n", "Title": "3D printing a cardboard box", "Tags": "|material|print-material|", "Answer": "

The closest thing you will find is Woodfill plastic. As time goes on we are seeing more and more PLA that is infused with other materials. Woodfill will look and feel like waxy wood. We are pretty used to seeing fiber boards and the like so this will not come off as odd. However it is not quite cardboard, except that it is also a wood product. It will also smell of wood.

\n\n

Here is a close up of a beyond exceptional wood fill printed piece. There is also other types of wood fill including bamboo.

\n\n

\"woodfill\"

\n\n

I mean if you want Cardboard.. Why not a laser cutter? While you can get a MUCH cheaper laser cutter, the glowforge comes to mind.

\n\n

\"enter

\n" }, { "Id": "3372", "CreationDate": "2017-01-09T23:41:19.193", "Body": "

I decided to fire up my 3D Printer again after a while. I have new PCs, so I had to download Slic3r fresh, so I got the latest version (1.2.9). Then I went and found my old config file. It's really old, from 2013!

\n

And Slic3r did not load it when I told it to, and consequently, when Octoprint was telling my Printrbot LC (custom) to print my things off Thingiverse, it was not in the bed, and the x axis skipped a few nubs, as would happen when you tell it it's too far away.

\n

I recall having to double and triple check Slic3r to make sure that it actually loaded the configuration I spent time crafting and honing. However, it's obviously been 3-4 years since I last played with this stuff, and this is a very low level feature that should have been fixed at Day 0 or 1.

\n

I presume that I'm doing something wrong. Do I need to stand on 1 leg and hold my mouth just right? I don't even know how I could screw it up...

\n

Update

\n

I changed from "Simple" mode to "Advanced" mode, and the config imported fine. Though, I don't think it was my most recent config. C'est la software!

\n", "Title": "Why do I have problems trying to get Slic3r to load a config file?", "Tags": "|slicing|slic3r|octoprint|", "Answer": "

I changed from \"Simple\" mode to \"Advanced\" mode, and the config imported fine. Though, I don't think it was my most recent config. C'est la software!

\n\n

Also, I switched to Cura after this.

\n" }, { "Id": "3375", "CreationDate": "2017-01-10T06:23:03.147", "Body": "

Under FreeCAD, I sketched on the face of a solid as follows :

\n\n

\"sketch\"

\n\n

My problem is, when I use \"Pad a selected sketch\", the solid created comprises also the original solid.

\n\n

\"created

\n\n

It prevents me, for instance, to get the difference with the first one.

\n\n

Any idea what may be causing this? How to prevent it?

\n\n

MyFreeCAD config:

\n\n
OS: Ubuntu 16.04.1 LTS\nWord size of OS: 64-bit\nWord size of FreeCAD: 64-bit\nVersion: 0.15.4671 (Git)\nBranch: releases/FreeCAD-0-15\nHash: 244b3aef360841646cbfe80a1b225c8b39c8380c\nPython version: 2.7.11\nQt version: 4.8.7\nCoin version: 4.0.0a\nOCC version: 6.8.0.oce-0.17\n
\n", "Title": "Padded Sketch comprises too many solids (FreeCAD)", "Tags": "|3d-models|3d-design|", "Answer": "

Ok, I got help on this forum thread

\n\n

Using Part Extrude instead of Pad a selected sketch on the sketch will create an independent solid.

\n\n

It also turns out that, instead of creating the second solid to do a boolean difference, the good way to \"cut\" my solid is using the PartDesign Pocket tool.

\n" }, { "Id": "3382", "CreationDate": "2017-01-11T20:49:52.380", "Body": "

I'm using a Flashforge Pro and attempting to print a wheel about 6mm thick to serve as a platform. In other words, the wheel doesn't have to be solid, but spokes won't do the job. I've experimented with different temperatures, but, because of ABS' thermal expansion, I don't think that will solve the problem. Also tried putting lots of 2mm holes in the wheel. I've considered other designs for the interior, but doubt that would be a solution. Has anyone tried using different print paths, i.e. actually altering the path that the slicer suggests? (grasping at straws)\nThanks for your suggestions.

\n", "Title": "How to print a 6\" wheel in ABS?", "Tags": "|heated-bed|abs|simplify3d|warping|", "Answer": "

I had pretty much this exact situation when printing a disk for a rocket avionics bay. It seemed to come down to getting the basics of bed adhesion: Heat bed, ABS temp, bed composition, and a \"primer\" layer. I found the FF Creator Pro to work well with a bed at 110, filament at 230, printing on a glass plate with ABS slurry. Then adding the brim 6 orbits wide held it down very well. I found that printing on the stock FF blue plastic was inconsistent with adhesion. Also, keep the door closed while printing and for something this size don't run the cooling fan (if you're printing on the left nozzle).

\n\n

And, of course, the right infill helps with the thermal contraction and strength. I found through testing that the rectilinear patter in S3D gave the best structural support in multiple dimensions. I also end up typically printing infill at about 15% to ensure a good surface on the upper face. Any lower and I found a lot of sagging in the top finish.

\n\n

If you're using this as a platform, then the layer size matters a lot also. You don't need .10mm layers, as .25mm or .20mm layers would probably work better. Then make sure you print enough top layers to get a good finish. At .20mm I usually print 4 top layers with a 15% infill.

\n" }, { "Id": "3383", "CreationDate": "2017-01-11T20:56:10.890", "Body": "

Am seeing the following behavior in Marlin:

\n\n

When moving the printhead, if an end stop is hit, a position report (M114) erroneously reports the print head position as where it was trying to go, as opposed to where it's position actually is.

\n\n

For example (using pronsole):

\n\n
> M114\nX:1.00 Y:1.00 Z:1.00 E:0.00 Count A: 142 B:0 Z:1260  \n> G0 Z100\nendstops hit:  Z:16.88\n> M114\nX:1.00 Y:1.00 Z:100.00 E:0.00 Count A: 142 B:0 Z:21263\n
\n\n

In this example, the Z endstop was hit at (z:16.88), but the printer appears to think it is at the target location (z:100).

\n\n

The \"Count\" Z appears to remain accurate however.

\n\n

Question: Is this a bug in Marlin, or is this correct behavior and I am misunderstanding something?

\n", "Title": "Marlin, end stops, and position report - possible bug?", "Tags": "|marlin|firmware|motor|", "Answer": "

Some info on the subject: few stepper motors have any way of knowing their exact position during operation, which means Marlin will have to assume the steppers always are in the right place.

\n\n

In other words, all g-code commands are executed relative to their current position, not with regards to the real positions. So if you forcefully move the printhead during print, the printer will just continue printing the same patterns in mid-air (this is basically what is called layer shifting).

\n\n

The only way the printer can know the real position of the steppers is when the endpoint switches are triggered. This is why the homing action is done to before prints in order to reset the internal positional bookkeeping of the steppers in Marlin. So if the endpoints are triggered without the printer actually being at the endstops, you will seriously confuse Marlin.

\n\n

I agree that is sounds weird that Marlin does not reset the positions of it's steppers when an endstop is triggered (during print?), but then again, if it happens accidentally (and not as part of a re-calibration procedure), I believe the default actions should be to abort all operation since this would indicate the printer being out of control. Perhaps there simply is no routine to handle movement after the program has been aborted - a situation where probably nothing would be right anyway..

\n" }, { "Id": "3388", "CreationDate": "2017-01-12T23:24:57.273", "Body": "

How are professional or regular products designed on the computer for manufacturing?

\n\n

Do they do it by making a 3D model and then getting it made by manufacturers/plastic factories, or the process is different?

\n\n

If it's by making 3D models then what kind of programs do they use - Regular 3D software like blender?

\n\n

I also understand that you can get a 3D model printed by 3d printing services, but are they the same as mass manufacturing companies, by that they make the product by 3D models.

\n", "Title": "How are things like a USB casing designed for 3D printing?", "Tags": "|3d-models|3d-design|", "Answer": "

3D printing provides a faster method for prototyping and have always been labeled as prototyping machines. Until recently, it has been rare to see 3D printers used for \"mass manufacturing\".

\n\n

Yes, most mass-produced products start the manufacturing process with a 3D model. 3D models can be created in many different applications such as Solidworks, AutoCAD, Unigraphics, Blender, even Sketchup just to name a few.

\n\n

In product development, the 3D model will then go through prototyping. Rapid prototyping can be done using a 3D printer by utilizing cheap materials and almost no labor cost.

\n\n

Here are a few costs that can be associated with the different prototyping methods.

\n\n

Traditional Prototyping

\n\n

(Typically involving \"traditional\", subtractive manufacturing methods such as CNC mills, lathes, routers, etc.)

\n\n\n\n

Rapid Prototyping

\n\n

(Typically involving a 3D printer or other additive manufacturing methods)

\n\n\n\n

Once a prototype is produced, the designer will adjust the 3D model accordingly based on results of the prototype. This process will be repeated until the prototype is adequate for the purpose of the end product.

\n\n

When the product design is ready for mass production, it will go through traditional manufacturing methods such as:

\n\n\n" }, { "Id": "3402", "CreationDate": "2017-01-15T14:01:40.907", "Body": "

We are trying to repair an Ultimaker Original+. One problem is a missing resistor isolation. The Ultimaker Original+ prints with up to 260 Degree Celcius.

\n\n

Which kind of isolation products are suitable to resist the heat and are fitting on the thin wires of the thermistor?

\n", "Title": "New thermistor isolation, how?", "Tags": "|thermistor|ultimaker-original|", "Answer": "

There are many materials you can use one that came to my mind is this High Temperature & Pressure Sealant and also the best way to apply it so it can be removed easy is this . take a kitchen food very thing nylon cover you know the very thin elastic one or cover the parts with some type of release agent (dont use something that inhibit silicone like sulfur or sulfur containing staff) and apply it over it then when is solid cut it and release it it would be like a glove i hope ;)There are silicone that are really high temp 1000 f to 2000 f good luck :)

\n" }, { "Id": "3406", "CreationDate": "2017-01-15T20:22:27.473", "Body": "

I'm creating a reverse Bowden setup to guide my filament from spool to extruder, through a path which contains two couplers in the middle as follows:

\n\n

[spool] --- |#= --- =#| --- [extruder]

\n\n

So I have to connect a tube to the back of a coupler (---=#|) and not just to the front (---|#=). That's the end that contains the screw thread, and it's not designed to take a tube. I can't manage to make it a smooth transition. When I try to push my 1.75 mm filament through, it will often get stuck there. After it's through, the extruder seems to have no problem with it anymore.

\n\n

Is there a trick to making this a smooth transition?

\n", "Title": "Smooth transition between a PTFE tube and the back of a push-fit coupler", "Tags": "|bowden|ptfe-tube|", "Answer": "

An alternative to chamfering the connector is buying a different type of connectors with a larger bore hole all the way through the connector and let the tube pass all the way through. These are used in my similar spool to extruder setup (reversed Bowden setup in OP's terminology).

\n\n

\"Large

\n" }, { "Id": "3410", "CreationDate": "2017-01-16T21:09:56.540", "Body": "

I have read a few times1 that the Arduino Mega can struggle to perform the tasks required for 3D printer control, as the AVR chip is working at its limits, and this is why some manufacturers have moved away from the ATmega2560, to make custom (and integrated) controller boards using ARM processors.

\n\n

Having just seen the latest answer to this question, Multithreading with the Arduino, on the Arduino SE site, I wondered if anyone had used a Shield Buddy in conjunction with the standard RAMPS 1.4 board?

\n\n

\"Shield

\n\n

It is pin for pin compatible with the Arduino Mega (and Arduino IDE compatible - once the appropriate add-ons have been installed), but it has a much faster three core processor. Obviously only one core would be used, leaving the other two idling, but even so the performance is apparently much better.

\n\n

See Bringing Multicore To The Arduino World With ShieldBuddy TC275.

\n\n

If anyone has experience of using this board, I would like to know whether it was successful or not? I don't see why it should not, although the Marlin firmware would need recompiling, for the Aurix TC275 processor. Would any improvement be seen? Is it worth paying the high price tag of \u00a389?

\n\n
\n\n

1 One of the places was 3D Printering: Electronics boards:

\n\n
\n

While they work for what they\u2019re intended to do, there are a few\n limitations. Arcs and circles are a little weird to program, and using\n these boards for something other than a cartesian 3D printer \u2013 a CNC\n machine, or a laser cutter, for example \u2013 is a bit out of the\n ordinary.

\n
\n", "Title": "Has anyone used a Shield Buddy (en lieu of a Arduino Mega2560) as the RAMPS 1.4 host board?", "Tags": "|ramps-1.4|arduino-mega-2650|hardware|", "Answer": "

It's not quite as simple as you would suggest.

\n\n

You can't just recompile Marlin for another device. You'd need to rewrite large parts of it. It may be compatible with the Arduino IDE, but that doesn't mean you can just run firmware intended for the AtMega2560 on it. All the timings (e.g. those of the pulses sent to the stepper motors) would be off, if you managed to get it to compile at all.

\n\n

As some examples, here are some pieces of code from Marlin that would be broken, as they're written directly in AVR assembly:

\n\n
#define DELAY_1_NOP  __asm__(\"nop\\n\\t\")\n#define DELAY_2_NOP  __asm__(\"nop\\n\\t\" \"nop\\n\\t\")\n\n[...]\n\n// ensure 100ns delay - a bit extra is fine\nasm(\"nop\");//50ns on 20Mhz, 62.5ns on 16Mhz\nasm(\"nop\");//50ns on 20Mhz, 62.5ns on 16Mhz\n\n[...]\n\n#define MultiU24X32toH16(intRes, longIn1, longIn2) \\\nasm volatile ( \\\n             \"clr r26 \\n\\t\" \\\n             \"mul %A1, %B2 \\n\\t\" \\\n             \"mov r27, r1 \\n\\t\" \\\n             \"mul %B1, %C2 \\n\\t\" \\\n             \"movw %A0, r0 \\n\\t\" \\\n             \"mul %C1, %C2 \\n\\t\" \\\n             \"add %B0, r0 \\n\\t\" \\\n             \"mul %C1, %B2 \\n\\t\" \\\n             \"add %A0, r0 \\n\\t\" \\\n             \"adc %B0, r1 \\n\\t\" \\\n
\n" }, { "Id": "3419", "CreationDate": "2017-01-18T00:44:39.150", "Body": "

I'm (surprisingly) having a problem getting my PEI substrate to stick to the heated bed surface. The ABS item being printed stuck great to the PEI surface. I've not seen anyone else post on this problem, but perhaps someone has seen it.

\n\n

Note, this is the PRINT SURFACE warping, not the item being printed. That part went great with no delamination even on a large flat bottom.

\n\n

I'm using 3M 468 adhesive to attach a PEI plastic sheet directly to the heated surface of my Flashforge Creator Pro printer. The heated bed is anodized black and the adhesive is a sheet cut to the requisite 6\" X 9\". It seemed fine and looked pretty good with few air bubbles when first attached, but the first print caused the edges of the PEI plastic to warp up from the corners of the bed.

\n\n

It appears that the weak link is that the PEI detached from the adhesive. While this was not uniform, removal of the PEI sheet showed that 95% of the adhesive was still attached to the print bed.

\n\n

The PEI has one glossy side and one matte side, and I chose to apply the adhesive to the matte side to get the glossy surface. Does this make a difference?

\n\n

I applied the adhesive to the PEI surface just as it was after removing the protective plastic coating. I figured this was a clean as it was ever going to get.

\n\n

I'd definitely appreciate some ideas on this.

\n", "Title": "Bad PEI plate adhesion to heated bed", "Tags": "|heated-bed|", "Answer": "

Perhaps this is due to the aluminum build plate warping/crowning as it heats? I use a PEI print surface on my Monoprice Maker Select (Wanhao Di3), which has an 8.5\"x8.5\" build plate, and I've had no issues with the PEI coming unstuck. However, instead of attaching mine directly to the aluminum heated build plate, I attached it (using 3M 468) to a piece of Borosilicate glass, which is then attached to my aluminum build plate using silicone thermal heatsink pads. The rational for using a Borosilicate glass print surface is that a heated metal build surface is going to warp or move some, as the heat is coming from one side; the glass will not warp and so you have a completely flat surface on which to print (and the heatsink pads help make up the difference in surface geometry). I used this 3dprinterwiki article as the basis for my glass bed mod, and applied the PEI/adhesive using another source (which I've forgotten, possibly the RepRap wiki). While the first link is Wanhao Di3 specific, it covers the idea. Make sure you tweak your z-endstops if needed on your particular machine, as adding the glass plat raises the bed height by several mm.

\n\n

Please note that I'm theorizing - I did not ever try applying my PEI directly to my aluminum build plate, so I have no direct comparison. But the use of glass to eliminate build plate warping/crowning is fairly well documented. See also this 3DPSE Answer.

\n" }, { "Id": "3420", "CreationDate": "2017-01-18T05:40:20.393", "Body": "

I've got a Monoprice Mini Select (15365) and it takes FOREVER for me to manually spin the dial to get the printhead to raise all the way up so that I can perform maintenance (clear blockages in the nozzle or apply new tape to the bed, etc).

\n\n

So, I was thinking about writing a snippet of gcode that I could just run which would contain the commands necessary to do that for me. I'm a n00b to 3D printing, but I'm an old programmer so I figured it would be too hard. However, before running this code on my printer, I wanted to get some experts to double check me to make sure I'm not going to hurt anything. :)

\n\n

Here is what I have, please let me know if I've done anything wrong or if you have any suggestions.

\n\n
; Move print head to center and top to prepare for cleaning/maintenance\n\nM107    ; fan off\nG28 ; home all axes\nG21 ; set units to millimeters\nG90 ; use absolute coordinates\n\n; full dimensions of the print area are 120mm x 120mm x 120mm\nG0 X60 Y60 Z119 ; move to center X,Y and just below the max height\n\nM84     ; disable motors\n
\n\n

I grabbed a few lines from gcode generated by Slic3r and used the gcode wiki entry to understand each of the commands and fill in the extra ones I needed.

\n", "Title": "gcode to move the printhead up for maintenance", "Tags": "|g-code|", "Answer": "

It really depends on whether you currently have something half printed on the bed when you need to do maintenance. \nFor example, you may have a blockage mid-print or need to reprime the nozzle.

\n\n

So with that in mind, personally I would separate the line that does the move into two different lines.

\n\n

Move vertically first, then in X Y. Otherwise, the print head may move diagonally and hit your object. This is because the head will move all three axes at once.

\n\n

Otherwise looks good.

\n\n

Edit: Another suggestion is that if you use repetier host to control the printer it will be easier to return to the print in-progress automatically.

\n\n

Also take a look at the S parameter, for moves it can control how fast the move is.

\n" }, { "Id": "3428", "CreationDate": "2017-01-18T22:38:32.910", "Body": "

I'm very new to 3D printing, and I am very interested in printing electronics. I want to be able to print out circuits, so I've been trying to find a conductive filament. What sorts of conductive filaments are available to the consumer (me) and are of acceptable quality

\n\n

Edited for clarity: I am not seeking a product recommendation, but rather am looking to know what sort of variety I can expect and more specifically which filaments are useful for printing electronics.

\n", "Title": "What are some conductive filaments that can be used in printing electronics?", "Tags": "|filament|reprap|electronics|", "Answer": "

At this point conductive filaments are a very new thing. All the filiments on the market are PLA based that have been infused with Carbon or Graphine

\n\n

However I worry you over estimate what you can do with this. You will at most be able to 3d print a simple wire and power a LED. You will not be able to run a microcontroller, and honestly I would expect a complicated print to have too much or too little resistance, maybe burst into flames.

\n\n

\"enter

\n\n

You really should look into the Voxle 8 and other printers. The printer is really the driving factor. Using a conductive filament will not be enough. Note this printer uses a syringe not PLA plastic.

\n\n

Here is a Voxle8 print.\n\"Voxlel8\"

\n" }, { "Id": "3431", "CreationDate": "2017-01-19T14:11:02.343", "Body": "

I've noticed this on almost ever print I've ever had. On the initial first line that clears the extruder nozzle tiny little bubbles/craters seem to form on the line. While I don't think these are causing any issue with my prints I'm curious to know the reason why they form at all.

\n\n

Is this due to water absorption in my filament that turns to steam, which then bursts through the molten plastic? Is it due to air bubbles in the filament that are cause by the manufacturing process of the filament? Or is this more an indication that my nozzle is damaged or clogged in some way?

\n\n

\"Bubbles

\n\n

This image was made using ABS plastic and a heated build plate. I've noticed these same 'bubbles' appearing using PLA, and Nylon.

\n\n

Edit: Nozzle temperature 240\u00b0C, build plate temperature 150\u00b0C, Nozzle diameter 0.4 mm, filament diameter (measured 1.75 mm) retraction distance 1.7 mm. Using the Makerbot Desktop Slicer.

\n\n

The first line that my printer extrudes, where I'm seeing these 'bubbles' is a nearly full line. Makerbot starts from the right side of the image, extruding to the left.

\n\n

\"Single

\n", "Title": "What causes bubbles in extruded filament?", "Tags": "|filament|print-quality|makerbot|", "Answer": "

The temperature is too high or your keep your filament in open for days. So that filament observe water from air do the following steps:

\n\n
    \n
  1. pla 190-220\n abs 220-240

  2. \n
  3. If temperature in range then bake your filament at 50-60 temp.

  4. \n
\n" }, { "Id": "3443", "CreationDate": "2017-01-20T06:14:55.227", "Body": "

TL;DR - What needs to be done to the following model to ensure a successful print?

\n\n

As part of a father-son project we're trying to make an enclosure for a digital clock. The idea is to make a 3D printed model of a rocket with a hollow body to hold the electronics. The main body consists of two halves that will press-fit together with some integrated mounting locations for the display, micro-controller, etc.

\n\n

Since this is our first 3D printing project, I would like to know what can be done to the following model to help ensure a successful first print. We have not chosen a material or supplier yet, but I suspect we'll go with PLA since it seems to be quite common, inexpensive, available in lots of colours, etc.

\n\n

\"Main

\n\n

The model is not yet complete - there are several details missing, like additional mounting locations - but I was hoping to make corrections to what we already have before too much further, just in case it requires a complete restart.

\n\n

Here are some possibly relevant details and goals:

\n\n\n\n

There aren't a lot of hard requirements as this is just a learning project, so we're willing to change almost anything about the design, as long as we can fit in the existing clock parts.

\n\n

The Fusion 360 model is available to view online if it helps: Rocket Concept 3

\n", "Title": "How can I make this rounded enclosure feasible to print?", "Tags": "|3d-models|", "Answer": "

With a couple of minor adjustments, that would print just fine standing up as it is.

\n\n

Alternatively you could lay it down with the open side facing upwards and use a few supports (my least favourite method), or simply make a flat spot on the back to lay on the printer bed. Which is probably what I would do.

\n\n

As far as shell thickness goes - 2.5 mm is pretty heavy duty. You also need to bear in mind that it's best to have a thickness as a factor of the thickness of your bead or your layer height (depends on printing orientation) - assuming you are going to print this yourself. So usually that would either be divisible by the bead width or by the layer height. If you want translucent - easy enough use translucent or clear filament. I'd probably go for 1.2 mm or 1.6 mm wall thickness.

\n\n

I made an angel at Christmas with clear PLA and quite a thick wall (wasn't my model and I didn't bother measuring - but at least 2 mm) and an LED candle lit it up really well.

\n\n

It's much easier doing this kind of thing after buying the printer. that way you learn much quicker what works and what doesn't.

\n" }, { "Id": "3450", "CreationDate": "2017-01-21T01:50:51.590", "Body": "

I am trying to print an earbud holder with my Prusa i3. About an hour though the 3-hour print I left the printer for about 20 minutes and when I came back the printer stopped printing. The printer didn't display any errors on the LCD. The extruder and printing bed all cooled down but the z-axis didn't raise so the nozzle was stuck to the print. What could cause the printer to stop printing?

\n", "Title": "Prusa i3 Stopping midprint", "Tags": "|filament|prusa-i3|", "Answer": "

There's really no telling why that happened if you weren't there to observe it.

\n\n

A possibility is a temporary power outage, which would stop and reset the printer without any trace of it having happened. Even if there was not a power outage, maybe there was a temporary dip in power that caused the power supply to be unable to supply the required voltage (or perhaps the power supply was of poor quality to begin with and suffered some issue that caused the voltage to drop).

\n\n

Another possibility is that you were printing via a computer, and the computer rebooted during the print. If this is not the case and you were printing via an SD card, maybe the SD card became corrupted and the printer read some invalid G-code and reset itself (though this latter case would probably come with some indication of a fault on the LCD).

\n" }, { "Id": "3465", "CreationDate": "2017-01-25T20:09:41.100", "Body": "

There is an option in Slic3r to disable cooling for layers that take more than n seconds.

\n\n

What would be the disadvantages of having cooling on big layers ?

\n", "Title": "Cooling for big layers", "Tags": "|slicing|slic3r|cooling|", "Answer": "

Warping. Especially with materials like ABS, you want the plastic to cool down as gradually (and slowly) as possible, to prevent the print from warping as the cooling plastic contracts. On small layers, cooling is usually mandatory: with really small layers, you just end up with a big glob of molten plastic if the previous layer hasn't solidified enough before the next layer is put on top.

\n\n

You want just enough cooling that the plastic holds its shape, but no more than that. On a large layer, the plastic cools enough naturally without help from a fan.

\n" }, { "Id": "3470", "CreationDate": "2017-01-26T10:30:00.143", "Body": "

I have a Tronxy P802M (very similar to the Anet A8, but using a Melzi2.0V5 board) that seems to work fine (I just finished building, and axes movement and the integrated display work) but when I try to connect to the printer from my Simplify3D on Windows 10, I get the following:

\n\n
[...]\n  Connected to machine!\nSENT: T0\nREAD: ok 0\nREAD: wait\nSENT: M105\nREAD: ok 0\nREAD: T:24.44 /0 B:23.33 /0 B@:0 @:0\n  Connection failed.\n
\n\n

My other printers all connect fine.

\n", "Title": "I can not really connect successfully to my printer via USB", "Tags": "|software|electronics|simplify3d|usb|tronxy-p802|", "Answer": "

Try changing your port in the control menu. By default, mine is set to COM 1 and I have to change it to COM 3.

\n" }, { "Id": "3471", "CreationDate": "2017-01-26T16:32:33.657", "Body": "

I received a Monoprice Maker Select v2 (I3 V2 clone) from my wife for Christmas. The sample .gcode files that came with the printer generally print great with no noticeable defects.

\n\n

However, when I try to print miniatures for use with table-top gaming (D&D, primarily), I tend to get a lot of oozing and stringing. On top of that, bridge supports don't cool in time and tend to get fudged by the print nozzle, which results in oddities like arms being only half printed, sticking to the nozzle, and getting relocated to some other part of the print. If I use full grid supports with the most modest fill settings (8%) they end up being stronger than the miniature and are a real pain to remove.

\n\n

My printer is calibrated, as level as can be (the desk it's on is slightly warped but I've got the printer in the center of the warp; there is no wobble or lean) and squared. The build plate is calibrated and set to the right height.

\n\n

I'm using the version of Cura that came on the SD card with the printer (honestly not sure which one and I'm not at home to check). I've fiddled with print speed, extruder temp (ranging from 185 to 210), layer cooling, retraction settings, and tried switching to Slic3r (didn't go over well -- couldn't even get past the first layer).

\n\n

I'm using Hatchbox silver PLA filament (1.75mm +/- 0.05mm). I don't have another filament I can test with to compare performance.

\n\n

Even when I import the profile settings from one of the sample .gcode files I tend to end up with blobbing, pulling, and stringing all over the miniatures, in addition to missing or deformed parts. Notably, arms and hands -- most often overhangs -- tend to stick to the hotend and get repositioned, sending the whole thing out of whack.

\n\n

I end up with similar problems when I use the Novice mode settings in Cura (Normal Quality, High Quality, etc).

\n\n

Is there something I'm missing that I can do to improve the quality of small, detailed prints, or is the I3, as an entry-level printer, simply not up to the task? I'm especially interested in answers from users who have experience printing miniatures and their experiences in tuning for that type of print job.

\n\n

A few things I've tried test printing:

\n\n

Printer calibration test model v1

\n\n

Printer calibration test model v3

\n\n

Both of these came out very stringy, with lots of blobbing and layer pulling, especially towards the upper portions of the taller elements.

\n\n

I have also tried printing these two models:

\n\n

Knight with sword - high detail

\n\n

Elf monk - high detail

\n\n

All test prints of these two models had at least one missing hand due to it getting stuck to the hotend and pulling off the part, and both demonstrated lots of minor blobbing and pulling on otherwise smooth surfaces, with loss of detail in the more finely detailed areas. Here's a sample of the output for the knight model. Please ignore the fact that he has been dismembered, that was mostly my fiddling with it post-printing (I guess I don't know my own strength), though the left hand was not well-attached and fell off rather easily. He was printed with both hands and feet/base intact, but you can see the blobbing and pulling pretty well.

\n\n

\"enter

\n", "Title": "Why are my prints so defective?", "Tags": "|prusa-i3|print-quality|slicing|", "Answer": "

I've not done much miniatures printing, but I have the same printer and I happen to have the exact same filament loaded. Also, I've been doing a lot of tuning lately, including this z-brace mod which has improved my overall print quality, so I thought I'd take a pass at printing the Knight from your photos and sharing my findings.

\n

First, I've posted a series of pictures to show my findings. Overall, I'd say my print quality was better than what you showed in your photos, but still isn't good enough. I sliced with Cura 15.04.6, and printed from SD card. Here are my (Full) settings:

\n
Layer height (mm)         : 0.1\nShell thickness (mm)      : 0.5\nEnable retraction         : Yes\nBottom/Top thickness (mm) : 0.3\nFill Density (%)          : 20\nPrint Speed (mm/s)        : 20\nPrinting Temperature (C)  : 210\nBed Temperature (C)       : 67\nSupport Type              : Everywhere\nPlatform Adhesion         : None\nSkirts                    : 3\nFilament Diameter (mm)    : 1.75\nFilament Flow (%)         : 100.0\nNozzle size (mm)          : 0.5\n\nRetract Speed (mm/s)      : 40.0\nRetract Distance (mm)     : 7\nInitial Layer Thick (mm)  : 0.2\nInitial Later width (%)   : 100\nCut off object bottom (mm): 0.0\nTravel Speed (mm/s)       : 100\nBottom Layer Speed (mm/s) : 20\nInfill speed (mm/s)       : 50\nTop/bottom speed (mm/s)   : 20\nOuter Shell speed (mm/s)  : 20\nInner Shell speed (mm/s)  : 20\nMin. Layer Time (sec)     : 10\nEnable cooling fan        : Yes\n
\n

I do most of my printing with a later height of 0.2mm, but for a detailed mini, 0.1mm is probably the largest that will look good (and probably the smallest possible on this printer. I normally set most of my speeds to 50 mm/s, with first layer at 20 mm/s; for this I slowed it all to 20 mm/s due to the fine details, and I think it helped.

\n

Temps of 67\u02daC bed and 210\u02daC extruder are what I've found to work best on my machine for PLA, after much experimentation, but your machine may vary; I'm not sure how accurate the temperature measurements are on these machines. 67\u02daC gives me an observed bed temp of 60\u02daC, but that's at the top surface - I have PEI atop Borosilicate glass, adhered to the bare aluminum bed with silicone-based heat transfer pad.

\n

I think I miscalculated the top/bottom heights and infill. I'm not used to printing at 0.1mm layer height, but 3 top layers over 20% infill is clearly not enough - see the closeup of the mini's base in my linked gallery. Next print, I'll either try 0.6mm top/bottom, or much higher infill.

\n

The supports came off easily; I used a pair of sidecutters from my electronics bench. A little more cleanup with a sharp hobby knife, combined with a better base top layer would probably produce an acceptable result.

\n

There were two major flaws. The first are the little blobs on many layers; see for example the inseam area on the picture of the knight's back. Ed Nisley at Softsolder.com calls these "Reveral Zits", and I think the name is apt. These happen when the print head needs to quickly reverse direction or stop-move-print, but filament continues to extrude. I use fairly aggressive retraction settings, and I think my print shows smaller zits than yours, but still far too many. Ed has explored this topic in some depth; it's possible my extruder stepper isn't keeping up with my settings due to mechanical limits. This is an area I want to pursue, but I don't have time at the moment. I plan to read Ed's work and try some experiments on my machine to see if I can get better results; I will update this answer if/when I do. As it stands, most of them are quite small, and could probably be cleaned up with a knife; the worst are those around unsupported areas, such as the back of the shoulder guard.

\n

The second major flaw is the helmet. It's just... bad. I'm not sure the printer has much hope of nailing those horns, but overall the head is just bad. I'm not sure what can be done there.

\n

To summarize: @disc0ninja's advice on Bed Level and Print speed are certainly the right place to start; You might want to try my Cura settings to see if you get similar results. Also, the Z-brace mod I linked to above has made a big difference for me; I rarely have to adjust my leveling anymore. I also plan to try slicing with Slic3r, which I haven't used previously, but have been looking into. You mentioned you couldn't print with Slic3r, was that USB or SD Card? I'd suggest trying via SD if it failed during USB printing.

\n

Update 30 Jan 2017: It took a little doing, but I managed to slice and print this model via slic3r. I had some issues with the original STL in slic3er, which I ended up fixing with a free trial at makeprintable.com. I spent a lot of time fiddling with slic3r; it has a lot more knobs to turn than Cura, and I make no claims of having the best settings for this print. There are so many settings that rather than transcribe them here, I've captured them in my pictures of the slic3er print.

\n

\"Layers

\n

\"Infill\"

\n

\"Skirt

\n

\"Support

\n

\"Speed\"

\n

\"Multiple

\n

\"Advanced\"

\n

\"Cooling\"

\n

\"Filament\"

\n

Overall, I feel like the quality is higher. The "reversal zits" are hardly noticeable; but the big remaining problem is one I didn't fully diagnose in the original Cura print - lack of support for areas such as the shoulders. Slic3r added more support than Cura, but it's also harder to separate from the base. The head isn't great, but much better than the Cura print; I don't think my photos show it as well as it looks. This print has convinced me that there's plenty of quality still to be wrung from this printer; I hope to make time to do some more slic3r prints of this model while tweaking the params to see what's possible. If I make any big leaps in quality I will update this answer.

\n" }, { "Id": "3472", "CreationDate": "2017-01-26T17:13:26.313", "Body": "

In this answer user Barafu says,

\n\n
\n

Yet I manage to keep my tolerances +- 0.05 mm which is enough for everything but miniature printing.

\n
\n\n

I have asked for clarification on that answer regarding what is meant by \"miniature printing\" but in the meantime, I want to ask the general question.

\n\n

What impact does dimensional accuracy of filament have on final print quality, and why? Does it vary between different filament types?

\n", "Title": "How important is the dimensional accuracy of filament relative to the detail of a print?", "Tags": "|filament|filament-quality|dimensional-accuracy|", "Answer": "

Put it simply: they say 1.75 mm with a ± error of 0.05 mm. Which means your flow may vary ±2.9 % while you are printing creating blobs and such.

\n\n

The smaller the filament tolerance, the more expensive the production costs.

\n" }, { "Id": "3483", "CreationDate": "2017-01-29T02:11:14.410", "Body": "

I am running the SeeMeCNC customized version of Repetier on a Rostock Max V2. I have experienced this problem with both the stock configuration along with an E3D The Chimera. Sometimes, when I am printing and connect a mains device, my printer will fail in some way. The first time this happened I was printing with ABS, so I connected my fume sucker (A ~25-50W fan) to an outlet. However, when I connected it, the LCD showed garbled characters and my software (MatterControl) reported an error. Restarting the printer returned it to the previous state. I had a ~15 foot (coiled) extension cord connected to a 3-way splitter.

\n\n

Today, I connected a hot glue gun, and MatterControl reported that Repetier had switched to \"Printer set into dry run mode until restart!\" (no heat or extrusions). Looking at the firmware, this seems to be caused by a thermistor failure. I had everything plugged into the same setup as before, except for a surge protector at the end. The circuit that it is connected to has a 20A circuit breaker, with probably ~3A of it constantly used.

\n\n

My theory for this is that an additional device causes a slight fluctuation in the thermistor wires, causing an error to occur. However, the Rostock Max is designed so that wires are inside of aluminum tubes, which I think would provide EMF protection. The power supply is a generic 12V 30A power supply that was included in the kit, usually seen in LED strips. Both of these occurred while printing. I have ordered a UPS for the printer: would its under/over-voltage detection help? Are there any other solutions for this?

\n", "Title": "Print failing when connecting mains loads", "Tags": "|electronics|switching-power-supply|repetier|", "Answer": "

I have had the same Issue with my Maker select prusa printer, the way I found that best combats this is to attach the printer to a UPS and avoid attaching any High draw devices from the same circuit. Every Time i'd switch something on, My TV, my lights, etc. The same would happen. Hope this helps!

\n" }, { "Id": "3486", "CreationDate": "2017-01-29T13:38:04.857", "Body": "

I want to print a text inside a box. I mean the text should be cut out (see through). Example:\n\"enter

\n\n

As you can see the parts in \"o\" and \"e\" cant float.. it needs some kind of a support. Also the top triangle in the M can't be printed without a support.\nHere is something I found, I think this is what I need:

\n\n

\"enter

\n\n

Can you recommend me a font or some app to make these fonts printable the way I need it?

\n\n

The software I use is Simplify3D and all the free ones like cura.

\n", "Title": "Font for cutting?", "Tags": "|3d-models|3d-design|", "Answer": "

I did a quick search using The Google with the terms \"stencil font.\" There were too many results to list here and the first link I clicked on provided some seriously ugly fonts. Stencil fonts by definition will result in supported center pieces. Windows has a native font named, surprisingly enough, Stencil which does the same, but it's not quite as imaginative as the ones found using The Google.

\n\n

Here's an example from Font Squirell:

\n\n

Octin Prison

\n\n

\"Octin

\n\n

How about something a bit more bizarre from the same location:

\n\n

VanBerger Font

\n\n

\"VanBerger\"

\n" }, { "Id": "3487", "CreationDate": "2017-01-29T14:39:53.090", "Body": "

I have a Wanhao Duplicator i3 v2 (A.K.A. Maker Select, Cocoon Create).

\n\n

My extruder got clogged: I noticed that it wasn't extruding, so I stopped my print, removed the fan, heatsink and the extruder motor and I saw that the clog is at the heater block level.

\n\n

This is a what I have:

\n\n

\"Blocked

\n\n

\"Blocked

\n\n

As you can see, there is some PLA clogged at the very beginning of the 1.75mm hole in the extruder block.

\n\n

I can see there is something white inside (a stone? something that is not PLA?). I tried to heat up the extruder to 250\u00b0C and pushing with an hex tool but the block did not move.

\n\n

I solved a lot of clogged nozzle issues with the cold pull method, it always worked like a charm, this time I tried too but as a result the PLA filament broke and the \"stone\" is still there.

\n\n

Does anyone know if there is a way to unclog the extruder in this situation?

\n\n

Do I have to change it?

\n", "Title": "How to unclog a clogged extruder?", "Tags": "|filament|extruder|pla|", "Answer": "

This may not help, but I have a printrbot and when it clogs I heat up the extroder to 210C(for pla) or whatever the normal temperature is. Then manually push your filament through the hole till it's as far as it can go, then drop the hotend temp to 100 degrees C. When it is at 100C pull the whole filament out and hopefully your clog will come out too. After that just cut off the end of the filiment that has the clog on it. And you're good to go.

\n" }, { "Id": "3492", "CreationDate": "2017-01-30T13:42:55.383", "Body": "

I have seen many people saying on this site and many other 3D printing websites that 24 V systems are safer, compared to 12 V systems. By safer, I am talking in terms of fires or other electrical and component failures.

\n\n

Why would a 24 V system cause less danger? I would think that 12 V would be safer because it is very common (automotive) and many parts have been around for a while that use it. Although there are an increasingly amount of boards that support 24 V, many don't or need fuses or other parts that do support 24 V.

\n\n

Also, many parts that I have used are rated for 12 - 24 V. A 12 V power supply can go a bit over fairly comfortably. A 24 V power supply can't without partially going over the rating.

\n\n

If I had to build a printer designed with safety as a main priority, what voltage would be best?

\n", "Title": "Why would one choose 12 V from 24 V, from a safety standpoint?", "Tags": "|electronics|switching-power-supply|", "Answer": "

The most important \"safety\" advantage when using 24V (compared to 12V) is that to get the same power, you only need half the current. A 192W heated bed would need 16A at 12V, but only 8A at 24V.

\n\n

Since one of the most common safety issues is underrated screw terminals being used for the heated bed (just search for \"3d printer fire\"; you'll find quite a few pictures of charred plastic around screw terminals). For example, the screw terminals on RAMPs board are only rated for up to 12A. That would be okay at 24V, but well over the limit at 12V.

\n\n

Since the wire gauge is dependent on current, you can also use somewhat thinner wires with a 24V system (or equivalently: wires that would melt in a 12V setup won't in a 24V setup). The power dissipated in a wire scales quadratically with current, so the same wire being used in a 24V setup would only waste a quarter of the heat of that wire in a 12V setup. There is also less strain on switching devices (such as MOSFETs or relays). The same applies here: power loss is quadratic with current.

\n" }, { "Id": "3506", "CreationDate": "2017-02-01T16:11:42.370", "Body": "

I am calibrating my Kossel XL with a new Marlin firmware changing the MANUAL_Z_HOME_POS since I have modified my print bed.

\n\n

I change the value in the Marlin code and upload it through Arduino (the editor on Windows 10). It has worked many times before with the exact same version of Arduino.

\n\n

When I then use Pronterface and issue G28 (home all) followed by G1 Z30 (just to be on the safe side) it shows that Marlin wasn't uploaded at all. It is a the same height as before. It's a 22.2 mm difference so it is easy to see.

\n\n\n\n

What can be the cause of this?

\n", "Title": "How come firmware isn't uploaded?", "Tags": "|firmware|", "Answer": "

Are you sure that the firmware is not being uploaded?\nYou can debug this changing the STRING_VERSION on Configurations.h.\nIf the version is not being changed, I would suggest trying to use another computer to upload the firmware or even another board.

\n" }, { "Id": "3507", "CreationDate": "2017-02-01T16:43:29.490", "Body": "

I am planing on printing something that will make contact with PCB boards. The print will be most likely to be in PLA. I don't want to fry the PCB board so I want to know if 3D printed PLA objects are conductive.

\n\n

I googled and found out about special non-conductive PLA and conductive PLA. But what about the conductivity of normal PLA?

\n", "Title": "Is PLA filament conductive?", "Tags": "|filament|pla|material|", "Answer": "

PLA itself falls in the category of non-conductors, with a resistivity ($\\rho=R\\frac A l=\\frac 1 \\sigma$) in the order of $10^{16}\\ \\Omega \\text m$ (see here), similar to other plastics. Following image gives an idea of the values of resistivity for usual conductors and isolators, insulating materials have resistivity greater than $10^9\\ \\Omega\\text m$, conductors have it smaller than $100\\ \\Omega \\text m$ (copper is about $10^{-8} \\ \\Omega \\text m$):

\n

\"enter

\n

However, PLA can be mixed with some amounts of a conductor as graphite, making the filament a conductive material (not as good as, by example, copper). This filament can be used to create some kinds of electric circuits.

\n

Other additions (coloring, ...) could also change slightly its properties.

\n

Obviously, if PLA reaches glass transition temperature or melting point, holes can appear in the surface, breaking insulation.

\n" }, { "Id": "3510", "CreationDate": "2017-02-02T09:03:13.320", "Body": "

I intend and would like to print a transparent hemisphere. I have a Taulman T-Glase clear 1.75 mm filament and I have XTC-3D.

\n\n

You can read this interesting page, Hacking t-glase to look more like glass!

\n\n

First question, what kind of printing settings should I use? Should I go for a low infill percentage or a high one? Should I go for line or hexagon? I would say 5% line but perhaps there is a better setting.

\n\n

Then, how should I use XTC-3D as mentioned on the above link? I'm a little bit confused how it can make the part more transparent.

\n", "Title": "FDM: Clear=transparent part", "Tags": "|filament|fdm|smoothing|", "Answer": "

You will not get perfectly clear prints with FDM because of the mechanics of FDM printing there will always be tiny gaps. But, you can get pretty close as demonstrated by a ColorFabb tutorial.

\n\n

A clear filament and printing an object at an elevated temperature (more than normal) and enough material to close the gaps (e.g. 110%) should be able to produce transparent prints, for example:

\n\n

\"Transparent

\n" }, { "Id": "3513", "CreationDate": "2017-02-02T15:38:48.467", "Body": "

I probably have to find out all the answers myself, but I could really use somebody else's experience.

\n\n\n", "Title": "Is it possible to 3D print a front bumper grille for a Honda Accord CL7?", "Tags": "|print-material|", "Answer": "

Your best bet for the material would be one of the high temperature filaments like the ColorFabb HT ones, they can take quite a punch when it comes to heat.

\n\n

As for the model, you could try to source one of the grills that is at least as good in shape as possible and get it scanned, there are people offering this as a service for example over at 3dhubs.com

\n\n

Glue is really depending on the part, I would suggest creating a click-together type of system so the \"glue\" is only stabilizing it and not beeing a structural component. You can also get hold of a 3D Pen and weld the parts together with the same material used to print.

\n" }, { "Id": "3514", "CreationDate": "2017-02-02T17:14:01.587", "Body": "

Last night there was an error in printing that caused the printer to stop printing, but kept the hotend on. This morning I discovered that it had fused to what it was printing after remaining on all night - it took some time to remove.

\n

Prints afterwards showed signs of under extrusion. Now when I tell it to extrude a millimeter, it no longer extrudes a thin strand, but instead exactly a millimeter of filament comes out the same diameter as it went in. Cleaning the hotend has done nothing, cleaning the extruder has done nothing.

\n

I'm left with two conclusions, either:

\n\n", "Title": "3D printer extruding too thick", "Tags": "|extruder|extrusion|", "Answer": "

It also can simply be the PLA. I have had PLA that runs thick no matter what setting I use. I know it's the PLA because when I replace it then it goes back to normal. I have tried temp ranges from 200-220\u00a0\u00b0C; all with the same results. I even dried the PLA for 9 hours and still did it.

\n

Sometimes the PLA just sucks aka a bad batch.

\n" }, { "Id": "3518", "CreationDate": "2017-02-02T20:05:22.480", "Body": "

Will my Duplicator i3 be able to print this hole in the vertical wall without infill?\nThe wall is 7mm the diameter of the hole is 24mm, the shell thickness is 0.8mm.

\n\n

\"Image

\n", "Title": "Will my Duplicator i3 be able to print this hole in the vertical wall without infill?", "Tags": "|pla|print-quality|infill|", "Answer": "

The answer is yes. However I notice that some knowledge is missing due your comments.

\n\n

The walls of the part is formed by 1 or several lines, this lines are called shell.

\n\n

The Infill is the part that fills all within the walls or shell; you can set the infill by 5% to 100% depending in how strong you need the printed part or set 0 to get an empty shell.

\n\n

The printer can be able to print any part, but some areas will need supports, this suports is a kind of outerfill to support areas that could overhang basically walls with 45 degrees or less, this support can be easily removed from the final part

\n" }, { "Id": "3519", "CreationDate": "2017-02-03T14:02:49.840", "Body": "

I'm not really sure if I'm asking the right question here, but I just made a noob mistake of buying 3\u00a0mm filaments instead of 1.75\u00a0mm. I have a Makerbot Replicator 2 which I've been using and so far it is pulling in 1.75 mm quite well.

\n

Is there any way I can still make use the 3\u00a0mm filaments, or do I need to use the filaments on different models? If it is the latter, which particular model is able to pull in 3\u00a0mm filament well?

\n", "Title": "How to Make Use of 3 mm Filaments?", "Tags": "|filament|extruder|", "Answer": "

Any modern and decent drive gear (single or dual) is shaped round (concave) to better grip the filament.

\n

\"enter

\n

Some are flat, but only the cheapest and least performing ones.

\n

\"enter

\n

(from Reprap wiki)

\n

The concave part designed for 1.75 mm filament will be too small to accept 3 mm filament, it will grip it so weakly that it will skip or skid very easily, especially given the increased grip required for 3 mm filament.

\n

1.75 <-> 3 mm is a complete overhaul of anything related to the extrusion path. I did it so I know...

\n" }, { "Id": "3527", "CreationDate": "2017-02-05T06:20:27.020", "Body": "

Many 3D printers employ a GT2 band for the y-axis plate and the printhead in the x-axis carriage.

\n

However, there seems to be the obvious disadvantages of:

\n\n

Is there any particular reason why GT2 is used over a straight forward rack and pinion system?

\n

\"Rack

\n

In particular, for the y-axis plate, as a rack would appear to me, be:

\n\n

If the rack is made from aluminium, surely weight can not be a major factor... or can it?

\n

Is cost a factor? A pinion seems to cost around \\$10-\\$15 (650 B\\$, here in BKK, Thailand), which is obviously more than a reel of GT2 and a couple of GT2 pulleys.

\n

Would the rigidity of the rack be less forgiving of an imperfectly aligned axis, something which the GT2 band and pulleys combination would not be so affected by?

\n
\n

TL;DR

\n

Which of these factors cause designers to use GT2 en lieu of a rack?

\n\n
\n

1 Maybe constant adjustment is not required on an everyday basis, but the tension would still need to be checked now and again, whereas a pinion would not have this requirement.

\n", "Title": "Advantages of GT2 over a rack", "Tags": "|printer-building|", "Answer": "

The answer so others can understand is that it's cheap and simple to run with a belt. Much of the 3D printers these days either stem from the sintering systems that were developed years ago and things that rose out of the RepRap community where a Lead Screw that had the precision needed to do proper 3D printing at decent speeds was out of reach for most of the community.

\n

It's a bit of, "we've always used belts," and then people come up with all sorts of stories to validate the dubious take to begin with.

\n

At one point in time (to be specific...around a couple of years BEFORE the question was asked here...) the costs of a lead/belt-screw system for anything other than a Z axis solution for a Cartesian or a Core-XY was prohibitive for a hobbyist or a system bought off shelf or build by them.

\n

It was too hard and expensive to get straight enough parts with precision to actually make anything other than small toy systems with those. We wouldn't get into racks as they were even MORE expensive. With the precisions we were previously working on and with in the community at the time this question was originally posed, it was something that only made sense for select solutions.

\n

As you scale larger (Hey, we're now professionally making HUGE parts with this stuff) it makes much, much more sense. For certain classes of manufacturing...it actually makes sense with certain filaments to make short run parts for almost ANYTHING, including things in the automotive space. At that juncture, prototyping, fixing "impossible" to repair situations on older cars, etc. makes some sense to get more precision or vastly larger size. Desireable becomes the ability to make printers with volumes up to 2 meters in size. Professional space? Maybe. But telling people that a pinion isn't making any sense? Heh...hardly. Same goes for lead/ball-screws. Several have made incredible printers fairly cheaply that can print seemingly impossible print runs (Hung at a 45 degree angle in mid air and print...) with some of this stuff. Is it cheaper? No. Does it make sense? Possibly. Especially if one wants to DIY a massive print volume printer (waves hand). I want a meter and a half cubed on a side print volume that I can enclose to crank out ASA prints of Smart body panels, for example... You're not getting that with belts- too stretchy, even with PU steel core belts.

\n

Frame it in from a relative cost perspective (It's only going to be about 25% more costly on a DIY or commercial product with a speed upside and precision that might make it WORTH that...but understand that an Ender 3 designed that way will run 350 dollars instead of the 200 or thereabouts it currently runs) and you're going to be more realistic and honest on the question's answer. Saying it isn't practical...even from when the post originated isn't being exactly forthright in this day and age- and disregards that a person MIGHT just be willing to spend the more money for the precision, etc.

\n" }, { "Id": "3532", "CreationDate": "2017-02-05T14:27:13.000", "Body": "

I have a Prusa i3 made by Geeetech. My 3D prints keep suffering from warping when printing with PLA.

\n\n

Whenever I print something with a base at about 10 cm x 10 cm, at least one corner of the print would warp up. I've read numerous articles about warping and tried all sorts of methods. My printer's bed is level, and heated to 60\u00b0C. My bed is made from clean glass. I've tried all sorts of adhesives. I tried blue tape, and used hair spray.

\n\n

The only way for me to combat this is gluing the base to blue tape with 502 Glue. I used brim and the whole brim just warps up. I sometimes leave the model printing over night. For the first few hours it's perfectly flat. When I go back to it the next morning I'd find one corner warped up. This is very dysfunctional to my prints.

\n\n

Is there a reliable way to stop this warping from happening?

\n", "Title": "3D prints keep suffering from warping", "Tags": "|pla|warping|", "Answer": "

It might be useful you confirm again the bed levelling. I had warp on big pieces despite I thought my bed was properly leveled but in fact, for some reasons, the bed was too far from the head on one corner. If you face the same issue, you should see the bottom part of your print is showing the filaments not well melted to each others on the 1st layer. I really make sure the nozzle is gripping my paper sheet (almost scratching it when I move the paper).

\n" }, { "Id": "3545", "CreationDate": "2017-02-07T14:13:32.897", "Body": "

I'm still putting aside money for buying my first 3D printer and I'm designing all the things I have to print.

\n\n

As I have a lot of things to print I would like to print as much of them as possible in a single print.

\n\n

I have some arcs of circle (between 90\u00b0 and 320\u00b0 and 10/30 cm of diameter) and I'm going to slice them so I'll be able to print many of them, vertically, in a single print. I read in the internet that I cannot print over 45\u00b0... but starting from where?

\n\n

The image below shows how I would like to print my (orange) things

\n\n

\"Illustration

\n\n

I suppose I can print my things without supports because from Y-start to Y-end they are <= 45\u00b0 (as the green line shows) is that right?

\n\n

The red line, instead, shows a case where the angle, starting from a (Y: 50%) point, is higher than 45\u00b0.

\n\n

So the question is: Can I print my things in such way?

\n", "Title": "Print circle arcs without supports", "Tags": "|3d-design|support-structures|", "Answer": "

There are a lot of variables here...

\n\n

If you did at ..

\n\n
    \n
  1. higher resolution.
  2. \n
  3. Calibrate your machine extremely well. Trial and error. Minimum temps and speed.
  4. \n
  5. If you use a fan.
  6. \n
\n\n

IE bad calibration.\n\"enter\nGood calibration at a slower speed and lower temp.

\n\n

\"enter

\n\n

What is going to happen is you will get a lot of junk, lines, loops, stringers that you can later remove with clippers. See the first photo for an extreme case. It still printed.

\n\n

Also if you use the right material as well. Some material, will do bridging better, IE ABS has a longer molecular chain than PLA so it tends to do overhangs / bridges better.

\n\n

Images from ultimaker

\n" }, { "Id": "3550", "CreationDate": "2017-02-08T05:11:22.170", "Body": "

I have seen a few guides mentioning w/t ratio but I can't find anything which defines it.

\n\n

I am using Simplify3D with a Wanhao i3.

\n", "Title": "What is a w/t ratio?", "Tags": "|simplify3d|", "Answer": "

It appears to refer to calibration factor called Width over Thickness.

\n\n

From Calibrate your 3D printer to print parts to fit

\n\n
\n

Here is the list of necessary variables for this calibration to work,

\n \n \n
\n" }, { "Id": "3552", "CreationDate": "2017-02-08T22:25:01.630", "Body": "

I have just built a Prusa i3 MK2 printer. One of its features is automatic XYZ axes calibration done using inductive probe which is located next to the nozzle.

\n\n

In my case, this calibration procedure failed with the message:

\n\n
\n

XYZ calibration failed. Please consult the manual.

\n
\n\n

The manual on this problem reads:

\n\n
\n
    \n
  1. XYZ calibration failed. Bed calibration point was not found.
    \n Calibration routine did not find a bed sensor point. The printer stops close to the bed\n point, which it failed to detect. Please verify, that the printer is assembled correctly,\n that all axes move freely, the pulleys do not slip and the print nozzle is clean. If\n everything looks good, re-run the X/Y calibration and verify with a sheet of paper\n between the nozzle and the print bed that the print nozzle does not touch the print\n bed during the calibration routine. If you feel a friction of the nozzle against the sheet\n of paper and the nozzle is clean, you need to screw the PINDA probe slightly lower\n and re-run the X/Y calibration.
  2. \n
  3. XYZ calibration failed. Please consult the manual.
    \n The calibration points were found in positions far from what should be expected for a\n properly assembled printer. Please follow the instructions of case 1).
  4. \n
\n
\n\n

My nozzle is new, therefore clean and I have already checked that all axes move without any problems. The manual does not seem to offer any additional advice.

\n\n

I tried to put the PINDA probe lower. It had helped the calibration, but at the same time the probe was lower than the nozzle, which means I couldn't print with it being on the extruder.

\n\n

What else can I check? How to find the issue? This is my first printer, so I'm totally clueless.

\n", "Title": "Prusa's P.I.N.D.A. XYZ axes calibration fails with generic message \"Consult the manual\"", "Tags": "|prusa-i3|calibration|", "Answer": "

One thing, and the one that eventually solved my problem, is to update the firmware. Firmware update instructions: Upgrading firmware - Prusa3D

\n\n

The only thing you might get stuck on is selecting the right COM port.

\n\n
    \n
  1. Go to the device manager (run devmgmt.msc)\n \"image
  2. \n
  3. In COM and LPT section, you should see port for your printer by name. Select that port number in the firmware update tool.
  4. \n
\n\n

In my case, I had to run the calibration twice before it worked. I had no problems since then.

\n" }, { "Id": "3557", "CreationDate": "2017-02-10T04:02:14.437", "Body": "

We have a toy with some broken parts, an Executivity Gear Master. I don't think it's made anymore. Some tiny parts were easy to break and we'd like to 3d print some replacement parts. We don't have CAD or any other 3D drawings file, just a few of the unbroken parts. What's the best way to get some of these printed? Do I have to turn this into a 3D file first? (Is there a quick way to do that from the part itself?) Or is there a way to do it where I just need the part, rather like getting a spare key cut from a pre-existing key being used as the template?

\n\n

Here's a photo of the part I need to print. Placed next to a quarter for size comparison:

\n\n

\"enter

\n", "Title": "How do you 3d print from an existing part when you have no CAD drawings?", "Tags": "|3d-design|", "Answer": "

Unfortunately, There's no easy way to go about this. The easiest solution is to model the part from scratch. A pair of (digital) calipers is an invaluable tool for doing that.

\n\n

Given that the part is rather small and (presumably) needs to fit with something, you need very precise measurements. 3D scanning or photogrammetry (reconstructing the models from 2D photographs) are not suitable for this, as they generally result in a model that needs clean up and can't be printed directly (which would be more work than modelling from scratch) and moreover the dimensions won't be accurate enough.

\n" }, { "Id": "3564", "CreationDate": "2017-02-10T21:50:45.500", "Body": "

It's my first encounter with 123D Design, and first time playing around STL/3D printing in general. I can't find much information about the grid. But I noticed, while trying to move it, you can actually raise it from the grid. Does this affect how it will be printed? Does the print have to be snapped to the ground for it to print properly? Or is the grid relative, used for approximation?

\n\n

\"Screenshot

\n", "Title": "Snapping to Ground - 123D Design", "Tags": "|3d-design|", "Answer": "

The position matters a lot if you use the default STL file without doing anything in the slicer. I've used 123D extensively and when I've inadvertently left a gap between the part and bed (and not snapped it to the bed in the slicer), the printer tries to print in mid air. Use \"d\" to \"drop\" the object to the grid. Different slicers have similar commands that will attach a face to the bed. I use Simplify3D.

\n" }, { "Id": "3569", "CreationDate": "2017-02-11T04:25:59.960", "Body": "

My heatbed won't let go off the prints until it considerably cools down. The cooling process takes decent amount of time.

\n\n

I was considering putting the M140 S0 (sets heatbed temp to 0) somewhere near the end of the printing process, so that when the printing is done the cooling is already in process.

\n\n

Is that a bad idea? I am asking because if it was 100% perfect idea, slicer would probably do it already.

\n", "Title": "Turning off heatbed for the last few layers. Good idea or not?", "Tags": "|heated-bed|slicing|g-code|adhesion|", "Answer": "

It's not a bad idea, and you should try it. But only on prints with some height, because:

\n\n

The goal of the heated bed is to ensure adhesion for the first few layers. Without the heat on the bottom side of the layer, the layers above will pull those layers with it as they cool, causing the warp that you see. When your bed is warmer than the layers above, those first layers stay with the warmth. This continues as the layers above are pulled into adhering to the bottom layer instead of going rogue.

\n\n

Print some objects that are taller than 10 layers, and see. When you succeed, get scientific and dial down the amount of layers until you see warp. Then you'll know which object height you can employ this.

\n" }, { "Id": "3586", "CreationDate": "2017-02-13T19:41:31.437", "Body": "

If I wanted to add my own custom M Codes in Marlin - in which source code file would I do that?

\n", "Title": "Adding custom M Codes to Marlin", "Tags": "|marlin|", "Answer": "

In the file Marlin_main.cpp on line 7131 there is a switch case:

\n\n

(To turn on line numbers go to File>Preferences and click Display line numbers.)

\n\n
case 'M': switch (codenum) {\n  #if ENABLED(ULTIPANEL)\n    case 0: // M0 - Unconditional stop - Wait for user button press on LCD\n    case 1: // M1 - Conditional stop - Wait for user button press on LCD\n      gcode_M0_M1();\n      break;\n  #endif // ULTIPANEL\n\n  case 17:\n    gcode_M17();\n    break;\n  etc.....\n
\n\n

Adding another case with an unused number such as 5 and then the code you want followed by a break should do the trick. Ex:

\n\n
case 5:\n   doABunchofCoolStuff();\n   myservo.write(thebestposition);\n   break;\n
\n\n

-AC

\n" }, { "Id": "3591", "CreationDate": "2017-02-14T11:25:55.430", "Body": "

Using a 3D pen I printed a small box. However, I was doing it on plain paper and of course the paper didn't come off the plastic very well. It didn't matter for that specific case, but if I want to print something else, which non-sticky surface would you recommend? Is there any way to use transparent surface (so that I can put a paper with picture as a guide under it)?

\n", "Title": "I am using a stereo 3D pen. What surface should I use?", "Tags": "|build-surface|", "Answer": "

I use parchment paper, the kind that you buy on sheets for baking.

\n

It come off nicely but there are adhesion problems if you want to do sharp corners.

\n

I prefer using ordinary printer paper for that. You can simply soak any scraps of paper off in a sink.

\n" }, { "Id": "3596", "CreationDate": "2017-02-15T02:17:27.410", "Body": "

I have a Prusa i3 that homes properly when you use the menu. After adjusting the bed and homing several times, it keeps the head at a reliable distance. However, when I try to print from an SD card, the printer buries the hot end in the heat bed.

\n

What needs to be adjusted to fix this issue?

\n

G-code from one of the 2 files

\n
M190 S50.000000\nM109 S200.000000\n;Sliced at: Mon 13-06-2016 15:39:25\n;Basic settings: Layer height: 0.2 Walls: 1.2 Fill: 20\n;Print time: 3 hours 47 minutes\n;Filament used: 11.435m 34.0g\n;Filament cost: None\n;M190 S50 ;Uncomment to add your own bed temperature line\n;M109 S200 ;Uncomment to add your own temperature line\nG21        ;metric values\nG90        ;absolute positioning\nM82        ;set extruder to absolute mode\nM107       ;start with the fan off\nG28 X0 Y0  ;move X/Y to min endstops\nG28 Z0     ;move Z to min endstops\nG1 Z15.0 F3000 ;move the platform down 15mm\nG92 E0                  ;zero the extruded length\nG1 F200 E3              ;extrude 3mm of feed stock\nG92 E0                  ;zero the extruded length again\nG1 F3000\n;Put printing message on LCD screen\nM117 Printing...\n\n;Layer count: 446\n;LAYER:0\nM107\n;LAYER:1\nM106 S127\nG1 F2400 E-4.50000\nG0 F3000 X81.474 Y58.603 Z0.300\n;TYPE:WALL-INNER\nG1 F2400 E0.00000\nG1 F2220 X83.063 Y56.456 E0.06663\nG1 X84.277 Y55.080 E0.11240\nG1 X84.836 Y54.444 E0.13353\nG1 X85.799 Y53.476 E0.16759\nG1 X86.755 Y52.587 E0.20015\nG1 X87.179 Y52.244 E0.21376\nG1 X88.836 Y50.911 E0.26680\nG1 X89.285 Y50.610 E0.28029\n
\n", "Title": "Printer homes to Z stop-switch, but presses on heat bed during printing", "Tags": "|prusa-i3|calibration|z-axis|", "Answer": "

I had the same issue: Z-axis correctly homing manually but not when printing.

\n

What worked for me is disabling the following line in the Marlin configuration file:

\n
#define Z_MIN_PROBE_USES_Z_MIN_ENDSTOP_PIN\n
\n" }, { "Id": "3598", "CreationDate": "2017-02-15T13:08:29.087", "Body": "

So they say PLA is biodegradable. But I still don't know how much biodegradable.

\n

I live in a flat and we have composting trash can in the courtyard. However if I throw PLA in there and it does not decompose, neighbors are gonna be pissed since many of them are intending to use the soil for their balcony flowers.

\n

So can I really compost PLA, or does "biodegradable" just mean it falls apart eventually, eg. in years? And if it decomposes, is it safe to use for plants?

\n", "Title": "Can I really throw failed PLA prints on compost?", "Tags": "|pla|recycling|waste|", "Answer": "

TL;DR - No... well, yes you can, but it won't decompose in your lifetime.

\n
\n

Addressing this question fully - from the point of composting, rather than landfill:

\n\n
\n

In addition, PLA can only be recycled at plants that have separate facilities that deal with PLA (as bio-plastics interfere with the recycling of other plastics if combined), and (as an aside) the fumes from PLA are not as harmless as many people think, due to the (up to 40%) additives.

\n

So, PLA is not such a "happy" plastic as many people are led to believe by the marketing folks...

\n" }, { "Id": "3605", "CreationDate": "2017-02-15T16:44:09.693", "Body": "

Is there anything to the process of 3D printing that might make the dice unfair/land on a specific side more frequently? Or will any imperfections be negligible, thus making the dice fair? I'm more concerned about making a d20 than anything else, so focus on that shape if need be.

\n\n
\n\n

I am using a MakerBot Replicator+ with FDM, so take this into consideration if it affects your answer, and if you believe a different printing process would have an advantage over this don't hesitate to include it.

\n", "Title": "Will 3D Printed Dice Be Fair?", "Tags": "|3d-models|3d-design|", "Answer": "

I have tried to make an unfair die by putting 100% infill in the conner opposite to 6, 5 and 4.

\n\n

When you float it, it always comes with expected side up. So with floating you really can check if somebody has tried to make it unfair.

\n\n

The real-life tests have unfortunately shown my complete failure :)

\n\n

After 104 throws the average is 3.49. Even less than \"fair\" 3.5!

\n\n

It looks like the weight of plastic is too low to compete with hitting and rotation. The next try could be to put something metallic into it during printing.

\n" }, { "Id": "3617", "CreationDate": "2017-02-18T04:44:33.427", "Body": "

This has happened a few times whenever I print with the white PLA (Hatchbox 1.75 mm). It seems that the print job produces smooth sidewalls for most of the part but at a certain point and above, the walls become rough as if the alignment is off or something. I've attached a picture to show the issue. Does anybody have any insight?

\n\n

\"Problem

\n\n

I'm using a Printrbot Simple Metal and some of the print settings are listed below:

\n\n\n\n

I've noticed in the Slic3r rendering that the affected area seems to be the point where the infill switches from 25% to solid as seen in the included picture. If this is the problem, is there any way to enforce solid infill a few layers below this to eliminate the awkward transition, or specify a smaller extrusion width at this point?

\n\n

\"Screenshot

\n", "Title": "White PLA prints frequently have rough sidewalls near top of the print", "Tags": "|print-quality|fdm|desktop-printer|", "Answer": "

If you want to change settings on some area of your part check out modifier meshes in Slic3r.

\n\n

It looks like to0 much heat is delivered when solid infill starts and some melting occurs. See 3D Printing Problems #22: Extrusion Temperature Too High.

\n\n

You can try:

\n\n\n" }, { "Id": "3631", "CreationDate": "2017-02-20T23:19:10.480", "Body": "

I have been trying to print an object that is 4 inches tall. About at 3 inches it falls off the bed. I am using tape on the heated bed and right before the print I am wiping the bed with rubbing alcohol. After the first time I tried hot gluing it to the bed when it was mid way through so that it wouldn't fall off but that didn't work. I am printing at 185\u00b0C and the bed is 55\u00b0C. I am using PLA to print. Should I increase the temperature of the bed or is there something else that is wrong?

\n", "Title": "Prints falling off the bed towards the end of the print", "Tags": "|pla|prusa-i3|heated-bed|", "Answer": "

Check your Z-calibration. I have a similar problem that I'm about to investigate. I have a feeling the Z-axis pulses per mm may need to decrease ever so slightly. The cumulative error may lead to the extruder poking at finished layers as the part gets taller.

\n" }, { "Id": "3633", "CreationDate": "2017-02-21T08:21:29.327", "Body": "

I'm relatively new to 3d printing, and wanted to get a few things understood.\nFirstly, I am unclear on how Hexagonal infill is stronger than, say, diamond pattern.

\n\n

Can anyone explain how the different shape causes the structure to be stronger? I saw a few places that hex is stronger; usually, more vertices means 'weaker' shape (i.e. a triangle is stronger than a square), so how does that work with hex vs diamond?

\n\n

Also, in small objects, where the printer makes only a single dot as the infill (a dot instead of a line in larger objects), does the infill strengthen the object at all?

\n\n

EDIT: I am trying to understand the effect of the infill pattern on the strength of the print, regardless of print time.

\n", "Title": "Infill pattern comparison", "Tags": "|pla|makerbot|infill|", "Answer": "

Hex grids are used for different reasons than triangular grids (such as you often see on bridges and roof systems). Triangles are especially good at being rigid, while hex grids are very material-efficient for a given strength. The second reason ($) is typically more important for 3D printing.

\n\n

Triangles do have fewer vertices than squares, but it's not always true that \"fewer vertices\" means stronger. Vertices are one kind of weak point. But in a triangle, vertex \"angle-holding\" failures simply don't matter. You can fasten 3 bars together with hinges or other joints that have little resistance to changing angle, and the triangle is still rigid.

\n\n

In contrast, rectangular grids can (and do -- https://www.youtube.com/watch?v=5t9MpNTSbYg) completely collapse if their vertices aren't rigid enough. That combines badly with the fact that vertices concentrate forces, so have to be much stronger than sides in comparable settings.

\n\n

A triangle cannot change without changing multiple things -- at least 2 angles and a side, or all three sides. Intuitively, the sides and vertices of a triangle work together for strength. This advantage of triangles doesn't transfer to hexagons, but hexagons have two other advantages: First, if you want to fill a space with a repeating shape, hexagons use less material than other shapes. And second, hexagons keep all the individual \"walls\" shorter compared to others shapes, which makes them less prone to bend.

\n\n

The material efficiency was proven by Thomas Hales in 1998, according to http://www.slate.com/articles/health_and_science/science/2015/07/hexagons_are_the_most_scientifically_efficient_packing_shape_as_bee_honeycomb.html. His paper \"The Honeycomb Structure\" is available at https://www.communitycommons.org/wp-content/uploads/bp-attachments/14268/honey.pdf

\n" }, { "Id": "3637", "CreationDate": "2017-02-21T18:10:28.567", "Body": "

I recently purchased an FT-5 from Folgertech and finished assembly about a week ago. I have yet to accomplish a successful print. It seems like either the file is not being sliced correctly, or that it is possibly reading the G-code incorrectly. It will lay down the first perimeter layer of the part and then shift down the build plate a few mm in the y-direction, extrude another perimeter layer and then repeat. I have tried printing from an SD and through USB using Cura as the slicing software, with the same results.

\n\n\n\n

All of the mechanical movements and offsets seem to be correct. The printer connects to Pronterface without issue. It also homes fine from the machine and USB and moves where it should during these processes. Again, it seems to be that the issue is when it tries to interpret the G-code. I have tried multiple G-code files from different sources as well, still with the same luck.

\n\n

If anyone has experience with similar issues or can offer any help, it would be greatly appreciated. I have no clue what could be going on!

\n\n

In red filament is my attempt to print the 5 mm calibration cube this morning, an example of how this calibration cube should look like is seen in the picture with the white calibration cubes.

\n\n

\"5 \"5

\n", "Title": "Folgertech FT-5 : Does not seem to be reading G-Code correctly", "Tags": "|g-code|printer-building|electronics|calibration|folgertechft-5|", "Answer": "

The image shows that somehow steps in Y direction (under the assumption that you have taken the photograph from the front) are missed.

\n\n

Missing steps can be caused by too high settings for acceleration and jerk, too much friction on the undercarriage or the belt assembly, undercarriage catching something at a certain point, too less current through the stepper.

\n\n

You could increase the torque on the stepper by increasing the Vref of your stepper driver is you have separate stepper driver modules, or a adjustment screw on the printer controller board. Furthermore reduce the acceleration and jerk settings and check the undercarriage for stick-slip movement by disconnecting the belt, then reconnect the belt and check for friction of the pulleys.

\n" }, { "Id": "3659", "CreationDate": "2017-02-27T15:52:54.077", "Body": "

I have a Wanhao Duplicator i3.

\n\n

I have done many excellent prints with this printer, and have first hand experience that it can do a pretty much flawless print.

\n\n

But... Recently, I am experiencing weird results. My \"flow\" seems uneven. When laying down the first layer of the raft, I can see it looks like it \"beads\" in some places. Thin lines with little \"beads\" here and there (which seem to be in a constant pattern) i.e. -------()-----()-----()------()------ etc.

\n\n

I initially thought, wet filament... But drying the filament had no noticeable effect. Further more, all my prints seem to be horribly laminated at a certain \"height\" of the print. This is really strange as it will print perfectly and only at a certain height, mess up about 5mm of layers (height wise), and then print great again. This seems to be a constant now.

\n\n

I dont understand how/why this could happen, as the whole z-axis is on a linear spiral shaft. Unless there is a gcode issue somewhere that I am not aware off. I am using CURA as my slicer and I feel that even if there was an isnturuction hidden somewhere at a certain height, it would possibly effect a single layer, and not 5mm worth of layers.

\n\n

Any ideas?

\n\n

I have tried:

\n\n

Different filament\nPushing flow % to 105 and 110% respectively\nCleaned extruder gears and print head\nOiled z-axis shafts

\n\n

What baffles me, is the weird delamination (or rather lack of lamination) at a certain height. I have not measured this height exactly, but from guestimating, it looks like roughly the same height on every print where the issue is visible (about 3/4 up in the attached image). My support structures are also VERY messy, whilst they were very precise and perfect previously.

\n\n

\"enter

\n\n

Thanks

\n", "Title": "Print is not properly laminated at certain height", "Tags": "|filament|prusa-i3|print-quality|delamination|wanhao|", "Answer": "

Thanks to all the replies. Whilst all of them may be valid in some scenarios, my case seemed to have been a combination of things, that relates to most of the replies here.

\n\n

I had my spool holder on the side of the machine, and I noticed some friction as the filament feeded over the \"arm\" on top of the printer into the extruder. I sat and looked at this for a while, and it seemed apparent that as the extruder moves up, the angle of the filament over the \"guide arm\" on top changes to a more severe bend. This may have been why it always battles to extrude nicely at a certain height, and then maybe recoveres after that to an extent. (SO yes, under extrusion may have been the issue)

\n\n

I have now moved the filament to on top of the printer to feed straight down. I have also slowed down the travel to about half of what it was (40 now, was 60). I also dropped the infill speed a bit as that was also a mess most of the times.

\n\n

So far I have had excellent results! I am not sure if its the travel speed drop or the extruder feed that helped the most, but I am a happy chappie again. (So far).

\n\n

Thanks for all the input. It really made me re-look at all of this from a different view.

\n" }, { "Id": "3661", "CreationDate": "2017-02-27T23:39:34.063", "Body": "

I am wondering - of course if the 3D printer's bed big enough - printing multiple copies of the same print could save me significant amount of time in a small production line, excluding minor wastage such as setup time, post-processing time, etc.

\n\n

\"enter

\n\n

e.g. if my foo print takes 10 hours, printing 2x copies at the same time would take 2x times more, increasing linearly or it would be significantly less?

\n", "Title": "Would 3D printing multiple copies at once saves time?", "Tags": "|diy-3d-printer|", "Answer": "

It will save you and the machine time if done properly.

\n

Simplify 3D has a setting that lets you do multiple layers at once; as most slicers do. Therefore if the part is 10cm tall, then you can set it to move to the next one after 10 cm. The machine would then build each one in turn on the build plate in sequence. You won't have to remove the parts very often, and the machine won't have to take the time to heat up after each print. If done incorrectly, as in moving from one part to another for each layer, then yes it will require a large number of slow retraction and Z-axis moves, costing you more overall time.

\n" }, { "Id": "3662", "CreationDate": "2017-02-27T23:49:41.503", "Body": "

I'm familiar with 3D printing, and the G-code concept. I'm also comfortable with programming.

\n\n

Can anyone give me the G-code (or probably the M-code, actually) to read the bed temperature?

\n\n

Is there an equivalent of M105 (Get Extruder Temperature)?

\n", "Title": "G-code (M-code?) for Get Bed Temperature", "Tags": "|heated-bed|g-code|", "Answer": "

M105 should give you the bed temperature.

\n\n

For future reference you can find a general list of G/M codes here - RepRap Wiki - G-code.

\n\n

Most firmware files include a list, Marlin has it in Marlin_main. I have no idea how often the list is updated but they don't change often.

\n" }, { "Id": "3670", "CreationDate": "2017-02-28T13:20:35.137", "Body": "

I am doing a quad-copter now.

\n\n

I am not sure what color is the best to use for outdoor durability, especially in sunlight.

\n\n

They say PETg is the most durable material amongst cheap ones.

\n\n

But what color to choose: white or transparent? Or it does not matter?

\n", "Title": "What is the best color of PETg to use for outdoor durability?", "Tags": "|fdm|print-material|color|pet|", "Answer": "

PETG might not be the best for long term UV exposure.

\n\n

According to Midland Plastics, PETG doesn't hold up well in outdoor applications, and is susceptible to UV degradation. Unless you have an outdoor or UV rated PETG product which will have additional chemicals added to it, it may breakdown more quickly than you would expect. (the link I used was : http://www.midlandplastics.com/srtd_petg.htm)

\n\n

Polycarbonate on the other hand does hold up better in outdoor applications and is better suited for long term exposure to UV light. Most quads and RC products you see that have a nice slick looking shell on them, are made from Polycarbonate. That stuff is usually vacuum formed sheets. It does still breakdown but it is a much stronger material. The downside is that PC requires a higher nozzle temperature, than PETG. (The reprap wiki has more info: http://reprap.org/wiki/Polycarbonate)

\n\n

That being said PETG is not going to dissolve in sunlight. It will likely last upto a year if not longer depending on usage. If this is your first quad you're making, then odds are you'll have a crash due to hardware failure before excessive UV degradation (at least thats been my experience with prototype aircraft).

\n\n

Good luck!

\n" }, { "Id": "3683", "CreationDate": "2017-03-05T20:45:52.083", "Body": "

When I am printing objects that have a lot of surface area on the first layer parts of that layer will rise up causing there to be gaps in the first layer. Here are two pictures. The first one was printing with a raft and the second one was without a raft.\"Raft\"\"No I am printing on 3M Painters tape, extruder temperature at 200, bed temperature at 60. I am using Hatchbox PLA Filiment.

\n", "Title": "Lines of PLA not sticking to the bed", "Tags": "|pla|prusa-i3|print-quality|adhesion|rafts|", "Answer": "

I am having the same issue. I saw one similar post describe this as \"baconing\" and said it comes from over extrusion. Looking at the filament coming out I could imagine that if there is too much filament for the given travel speed, then it would buckle up. I tried decreasing the extrusion percentage. The Makerbot slicing program doesn't have a setting for that so I did multiple test prints and raised the filament diameter from the default 1.77 to 1.84 in the end, which is about at 20% decrease in the extruder output. I also slowed the prints speed. Both factors seemed to improve the quality, so over extrusion and speed both seem to be contributing factors, but the problem still hasn't gone away completely. It's happening with white Makerbot brand filament on large layers but not with green Makerbot brand and small layer heights (I haven't yet tried green coarse or white fine). I measured the white filament diameter with calipers and it is the proper 1.75mm, I'm just forcing it to extrude less by putting 1.84mm into the slicer settings.

\n" }, { "Id": "3688", "CreationDate": "2017-03-07T08:15:34.147", "Body": "

I have access to a MakerBot Replicator 2X which I use to try to print ABS + dissoluble (both are MakerBot's original filaments). It is really a pain and the filaments most often gets clogged (80% of the prints have to be thrown away). I a supposed to use the printer in a professional context, but at the moment it is really problematic and I feel pressure going up...

\n\n

I have initially tried the default parameters provided by the machine for known filaments (ABS: 230\u00b0C / dissoluble: 250\u00b0C / plate: 110\u00b0C + 0.1mm layers). As the nozzle got clogged I have made many other attempts with varying parameters and up to (250\u00b0C / 270\u00b0C / 135\u00b0C), which slightly improves things but is far from being really usable.

\n\n

Any idea of where this comes from?\n - ABS being notoriously difficult to print?\n - The Replicator 2x being old tech?\n - A parameters problem?

\n\n

Any advice on what I should do to improve the situation?

\n", "Title": "MakerBot replicator 2x glitches", "Tags": "|filament|makerbot|replicator-dual|", "Answer": "

I don't have a profile that has settings for the dissolvable filament anymore, but this is one I use for thin layers (second extruder at 232C my first extruder isn't working so just ignore that one).

\n\n

You may want to try printing small simple objects with each extruder independently first to confirm that you have good settings, then try both together after you know you have good settings. ABS is a pain but mostly for warping and sticking to the build plate. The dissolvable filament I believe is PLA if you're using Makerbot materials.

\n\n
{\n       \"_attached_extruders\" : [ \"mk8\", \"mk8\" ],\n       \"_bot\" : \"replicator2x\",\n       \"_extruders\" : [ 0 ],\n       \"_materials\" : [ \"abs\", \"abs\" ],\n       \"adjacentFillLeakyConnections\" : false,\n       \"adjacentFillLeakyDistanceRatio\" : 0,\n       \"anchorExtrusionAmount\" : 5.0,\n       \"anchorExtrusionSpeed\" : 2.0,\n       \"anchorWidth\" : 2.0,\n       \"backlashEpsilon\" : 0.050,\n       \"backlashFeedback\" : 0.90,\n       \"backlashX\" : 0.0,\n       \"backlashY\" : 0.090,\n       \"bedZOffset\" : 0.0,\n       \"bridgeAnchorMinimumLength\" : 0.80,\n       \"bridgeAnchorWidth\" : 0.80,\n       \"bridgeMaximumLength\" : 80.0,\n       \"bridgeSpacingMultiplier\" : 1.0,\n       \"coarseness\" : 9.999999747378752e-005,\n       \"commentClose\" : \"\",\n       \"commentOpen\" : \";\",\n       \"computeVolumeLike2_1_0\" : false,\n       \"defaultExtruder\" : 0,\n       \"defaultRaftMaterial\" : 0,\n       \"defaultSupportMaterial\" : 0,\n       \"description\" : \"External Definition\",\n       \"doAnchor\" : true,\n       \"doBacklashCompensation\" : false,\n       \"doBreakawaySupport\" : true,\n       \"doBridging\" : true,\n       \"doDynamicSpeed\" : false,\n       \"doDynamicSpeedGradually\" : true,\n       \"doDynamicSpeedInteriorShells\" : false,\n       \"doDynamicSpeedOutermostShell\" : true,\n       \"doExponentialDeceleration\" : false,\n       \"doExternalSpurs\" : true,\n       \"doFixedLayerStart\" : false,\n       \"doFixedShellStart\" : true,\n       \"doInfills\" : true,\n       \"doInsets\" : true,\n       \"doInternalSpurs\" : false,\n       \"doMixedRaft\" : false,\n       \"doMixedSupport\" : false,\n       \"doOutlines\" : true,\n       \"doPrintLayerMessages\" : false,\n       \"doPrintProgress\" : true,\n       \"doPurgeWall\" : false,\n       \"doRaft\" : true,\n       \"doSplitLongMoves\" : true,\n       \"doSupport\" : true,\n       \"doSupportUnderBridges\" : false,\n       \"endGcode\" : \"\",\n       \"exponentialDecelerationMinSpeed\" : 0.0,\n       \"extruderProfiles\" : [\n          {\n             \"bridgesExtrusionProfile\" : \"bridges\",\n             \"feedDiameter\" : 1.820000052452087,\n             \"feedstockMultiplier\" : 0.9300000000000001,\n             \"firstLayerExtrusionProfile\" : \"firstLayer\",\n             \"firstLayerRaftExtrusionProfile\" : \"firstLayerRaft\",\n             \"floorSurfaceFillsExtrusionProfile\" : \"floorSurfaceFills\",\n             \"infillsExtrusionProfile\" : \"infill\",\n             \"insetsExtrusionProfile\" : \"insets\",\n             \"layerHeight\" : 0.20,\n             \"maxSparseFillThickness\" : 0.20,\n             \"nozzleDiameter\" : 0.40,\n             \"outlinesExtrusionProfile\" : \"outlines\",\n             \"raftBaseExtrusionProfile\" : \"raftBase\",\n             \"raftExtrusionProfile\" : \"raft\",\n             \"restartExtraDistance\" : 0.0,\n             \"restartExtraDistance2\" : 0,\n             \"restartExtraRate\" : 25.0,\n             \"restartExtraRate2\" : -1,\n             \"restartRate\" : 25.0,\n             \"restartRate2\" : 25,\n             \"retractDistance\" : 1.700000047683716,\n             \"retractDistance2\" : 0,\n             \"retractRate\" : 25.0,\n             \"retractRate2\" : 50,\n             \"roofSurfaceFillsExtrusionProfile\" : \"roofSurfaceFills\",\n             \"sparseRoofSurfaceFillsExtrusionProfile\" : \"sparseRoofSurfaceFills\",\n             \"toolchangeRestartDistance\" : 18.50,\n             \"toolchangeRestartRate\" : 6.0,\n             \"toolchangeRetractDistance\" : 19.0,\n             \"toolchangeRetractRate\" : 6.0\n          },\n          {\n             \"bridgesExtrusionProfile\" : \"bridges\",\n             \"feedDiameter\" : 1.769999980926514,\n             \"feedstockMultiplier\" : 0.9300000000000001,\n             \"firstLayerExtrusionProfile\" : \"firstLayer\",\n             \"firstLayerRaftExtrusionProfile\" : \"firstLayerRaft\",\n             \"floorSurfaceFillsExtrusionProfile\" : \"floorSurfaceFills\",\n             \"infillsExtrusionProfile\" : \"infill\",\n             \"insetsExtrusionProfile\" : \"insets\",\n             \"layerHeight\" : 0.20,\n             \"maxSparseFillThickness\" : 0.20,\n             \"nozzleDiameter\" : 0.40,\n             \"outlinesExtrusionProfile\" : \"outlines\",\n             \"raftBaseExtrusionProfile\" : \"raftBase\",\n             \"raftExtrusionProfile\" : \"raft\",\n             \"restartExtraDistance\" : 0.0,\n             \"restartExtraDistance2\" : 0,\n             \"restartExtraRate\" : 25.0,\n             \"restartExtraRate2\" : -1,\n             \"restartRate\" : 25.0,\n             \"restartRate2\" : 25,\n             \"retractDistance\" : 1.399999976158142,\n             \"retractDistance2\" : 0,\n             \"retractRate\" : 25.0,\n             \"retractRate2\" : 50,\n             \"roofSurfaceFillsExtrusionProfile\" : \"roofSurfaceFills\",\n             \"sparseRoofSurfaceFillsExtrusionProfile\" : \"sparseRoofSurfaceFills\",\n             \"toolchangeRestartDistance\" : 18.50,\n             \"toolchangeRestartRate\" : 6.0,\n             \"toolchangeRetractDistance\" : 19.0,\n             \"toolchangeRetractRate\" : 6.0\n          }\n       ],\n       \"extruderTemp0\" : 228,\n       \"extruderTemp1\" : 232,\n       \"extrusionProfiles\" : {\n          \"bridges\" : {\n             \"fanSpeed\" : 0.50,\n             \"feedrate\" : 40.0\n          },\n          \"firstLayer\" : {\n             \"fanSpeed\" : 0.50,\n             \"feedrate\" : 10.0\n          },\n          \"firstLayerRaft\" : {\n             \"fanSpeed\" : 0.50,\n             \"feedrate\" : 50.0\n          },\n          \"floorSurfaceFills\" : {\n             \"fanSpeed\" : 0.50,\n             \"feedrate\" : 40.0\n          },\n          \"infill\" : {\n             \"fanSpeed\" : 0.50,\n             \"feedrate\" : 40.0\n          },\n          \"insets\" : {\n             \"fanSpeed\" : 0.50,\n             \"feedrate\" : 40.0\n          },\n          \"outlines\" : {\n             \"fanSpeed\" : 0.50,\n             \"feedrate\" : 10.0\n          },\n          \"raft\" : {\n             \"fanSpeed\" : 0.50,\n             \"feedrate\" : 90.0\n          },\n          \"raftBase\" : {\n             \"fanSpeed\" : 0.50,\n             \"feedrate\" : 10.0\n          },\n          \"roofSurfaceFills\" : {\n             \"fanSpeed\" : 0.50,\n             \"feedrate\" : 90.0\n          },\n          \"sparseRoofSurfaceFills\" : {\n             \"fanSpeed\" : 0.50,\n             \"feedrate\" : 90.0\n          }\n       },\n       \"fixedLayerStartX\" : 0.0,\n       \"fixedLayerStartY\" : 0.0,\n       \"fixedShellStartDirection\" : 215.0,\n       \"floorSolidThickness\" : 0,\n       \"floorSurfaceThickness\" : 0,\n       \"floorThickness\" : 1.0,\n       \"gridSpacingMultiplier\" : 1.0,\n       \"infillDensity\" : 0.3000000119209290,\n       \"infillOrientationInterval\" : 90,\n       \"infillOrientationOffset\" : 0,\n       \"infillOrientationRange\" : 90,\n       \"infillShellSpacingMultiplier\" : 0.70,\n       \"insetDistanceMultiplier\" : 1.0,\n       \"jsonToolpathOutput\" : false,\n       \"layerHeight\" : 0.1199999973177910,\n       \"leakyConnectionsAdjacentDistance\" : 0.0,\n       \"maxConnectionLength\" : 10.0,\n       \"maxSparseFillThickness\" : 0.1000000014901161,\n       \"maxSpurWidth\" : 0.50,\n       \"minLayerDuration\" : 5.0,\n       \"minLayerHeight\" : 0.010,\n       \"minRaftBaseGap\" : 0.0,\n       \"minSpeedMultiplier\" : 0.30,\n       \"minSpurLength\" : 0.40,\n       \"minSpurWidth\" : 0.120,\n       \"minThickInfillImprovement\" : 1.0,\n       \"modelFillProfiles\" : {},\n       \"numberOfShells\" : 2,\n       \"platformTemp\" : 110,\n       \"purgeBucketSide\" : 4.0,\n       \"purgeWallBaseFilamentWidth\" : 2.0,\n       \"purgeWallBasePatternLength\" : 10.0,\n       \"purgeWallBasePatternWidth\" : 8.0,\n       \"purgeWallModelOffset\" : 2.0,\n       \"purgeWallPatternWidth\" : 2.0,\n       \"purgeWallSpacing\" : 1.0,\n       \"purgeWallWidth\" : 0.50,\n       \"purgeWallXLength\" : 30.0,\n       \"raftAligned\" : true,\n       \"raftBaseAngle\" : 0.0,\n       \"raftBaseDensity\" : 0.6999999880790710,\n       \"raftBaseLayers\" : 1,\n       \"raftBaseRunGapRatio\" : 0.8000000119209290,\n       \"raftBaseRunLength\" : 15.0,\n       \"raftBaseThickness\" : 0.3000000119209290,\n       \"raftBaseWidth\" : 2.50,\n       \"raftExtraOffset\" : 0.0,\n       \"raftFillProfiles\" : {},\n       \"raftInterfaceAngle\" : 45.0,\n       \"raftInterfaceDensity\" : 0.3000000119209290,\n       \"raftInterfaceLayers\" : 1,\n       \"raftInterfaceThickness\" : 0.2700000107288361,\n       \"raftInterfaceWidth\" : 0.4000000059604645,\n       \"raftModelSpacing\" : 0.3499999940395355,\n       \"raftOutset\" : 4.0,\n       \"raftSurfaceAngle\" : 0.0,\n       \"raftSurfaceLayers\" : 3,\n       \"raftSurfaceShellSpacingMultiplier\" : 0.6999999880790710,\n       \"raftSurfaceShells\" : 2,\n       \"raftSurfaceThickness\" : 0.1400000005960465,\n       \"roofAnchorMargin\" : 0.40,\n       \"roofSolidThickness\" : 0,\n       \"roofSurfaceThickness\" : 0,\n       \"roofThickness\" : 1.0,\n       \"shellsLeakyConnections\" : false,\n       \"solidFillOrientationInterval\" : 90,\n       \"solidFillOrientationOffset\" : -45,\n       \"solidFillOrientationRange\" : 90,\n       \"sparseInfillPattern\" : \"hexagonal\",\n       \"splitMinimumDistance\" : 0.40,\n       \"spurOverlap\" : 0.0010,\n       \"startGcode\" : \"\",\n       \"startPosition\" : {\n          \"x\" : -112,\n          \"y\" : -73.0,\n          \"z\" : 0\n       },\n       \"supportAligned\" : false,\n       \"supportAngle\" : 30.0,\n       \"supportDensity\" : 0.2000000029802322,\n       \"supportExcessive\" : false,\n       \"supportExtraDistance\" : 0.50,\n       \"supportFillProfiles\" : {},\n       \"supportLayerHeight\" : 0.2000000029802322,\n       \"supportLeakyConnections\" : false,\n       \"supportModelSpacing\" : 0.2000000029802322,\n       \"supportRoofModelSpacing\" : 0.4000000059604645,\n       \"thickLayerThreshold\" : 0,\n       \"thickLayerVolumeMultiplier\" : 1,\n       \"travelSpeedXY\" : 150.0,\n       \"travelSpeedZ\" : 23.0,\n       \"version\" : \"3.9.4\"\n    }\n
\n" }, { "Id": "3696", "CreationDate": "2017-03-08T04:06:17.447", "Body": "

I have been playing around with creating scripts to generate some custom G-code for a Malyan M180 and I am having trouble understand what commands to use to switch nozzles. Sometimes I can get the nozzles to switch and it doesn't recenter but sometimes it does.\nI have been using:

\n
G54\nM108 T0; switch to left\n
\n

and

\n
G55\nM108 T1; switch to right\n
\n

Has anyone else looked into this and have any idea what commands or sequence of commands should be used to change nozzles?

\n", "Title": "What G-code commands tell the printer to change nozzle on a dual extruder printer?", "Tags": "|g-code|dual-nozzle|", "Answer": "

As @tbm0115 has mentioned - it differs. Your M108 T{index of tool} command is perfectly fine on some FlashForge printers, a company known for its different sorts of proprietary code. It is indeed used for tool changing here.

\n

While on the common Marlin firmware (e.g. used on the popular Creality Ender printers), M108 does a completely different thing-namely acting as a break sequence if the G-code parser is in a closed-loop-condition (after certain commands like M109 that wait until a certain action takes place).

\n" }, { "Id": "3702", "CreationDate": "2017-03-09T18:04:22.330", "Body": "

I'm building an automatic warehouse system using three NEMA 17 stepper motor.

\n\n

My problem is to move the motors with precision, since I do not have any kind of encoder on the motor and so I cannot know the position of the axes. I thought that the system could be similar to a 3D printer, since neither 3d printers have encoder on the motor.

\n\n

Where can I find a sketch for Arduino of a 3D printer, to understand how they work? How do they move with such precision without any kind of sensor?

\n", "Title": "Arduino 3D printer sketch", "Tags": "|diy-3d-printer|arduino-mega-2650|", "Answer": "

A stepper motor is different from a typical electric motor. Instead of being \"on\" or \"off\", or running a some approximate variable speed, the controller sends a series of pulses to drive the motor one step at a time. This can be driven forward or backwards.

\n\n

Based on how the motor and machine is designed, the machine will move a tiny amount. If for example, one step moves , say 0.1mm. If you send 150 pulses in the forward direction, the machine will move 15.0mm If you then send another 8 pulses in the reverse direction, the machine will be 14.2mm from the starting point.

\n\n

To know the absolute position, you need to establish the starting point. Most printers have limit switches at their \"home\" position. To initialize the position, drive the motor until it hits the switch, and then set position as zero.

\n\n

Then count the pulses up and down as you send them, you will will keep track of the machine's position.

\n" }, { "Id": "3703", "CreationDate": "2017-03-09T19:43:20.960", "Body": "

I am a totally beginner at 3D printing and I have this question: I see many 3D printers (at amazon) with Z precision as low as 0.1mm! To me that's amazing but what does not amaze me is this: when I see the printed objects of those 3D printers you can easily with naked eye see the \"vertical steps\". How?

\n\n

A precision of 0.1mm should be really almost impossible to see. If a person printed using 0.1mm precision how can I see the vertical steps?

\n\n

I know there are some smoothin techniques to make the surface better but that shouldnt even be needed if the printer actually printed at 0.1mm in the first case.

\n", "Title": "Ugly 3D printing with high precision", "Tags": "|3d-models|", "Answer": "

The 0.1mm refers to the thickness of each layer. However, it does not say anything regarding:

\n\n\n\n

No matter how fine the layers (and these printers that you refer to can definitely print 0.1mm layers just fine); if they're not well-aligned with each other, or the filament extrudes inconsistently, you're going to see the layer lines. It takes a rigid printer, with low-play bearings, a well-calibrated extruder and filament with a consistent diameter to get smooth-looking prints (but you will always see some layering, especially if you look up close). Also, since filament is extruded in a round shape, the sides of the object are not perfectly flat, but consists of many small arcs, which makes it easier to see the layer lines.

\n" }, { "Id": "3710", "CreationDate": "2017-03-10T23:27:48.510", "Body": "

I've heard that using hairspray is useful for keeping the 3D objects from peeling off of the bed, but every example I have seen where someone uses hairspray, they use it on a glass bed.

\n\n

Is it okay to use it on a metal bed as well?

\n", "Title": "Should you use hairspray on a metal bed 3D printer?", "Tags": "|pla|heated-bed|adhesion|", "Answer": "

I have a home-brew printer with a 9.5\u00a0mm (3/8"( thick solid aluminum heated bed. The bed is bare, with no additional tape, plastic film, or glass. For PLA I use purple Elmers glue stick or Aqua Net hairspray directly on the aluminum. For ABS I use acetone/ABS solution.

\n

Yes, it is OK to use hairspray on an aluminum metal bed.

\n" }, { "Id": "3720", "CreationDate": "2017-03-12T01:49:56.653", "Body": "

I'm looking for a post processing method for increasing the functional strength of a 3d printed part originally made by FDM. I've tried printing my part with solid infill but the layer separation is still the primary failure point. I'm looking for a way to get something closer to a cast or injection molded part. Obviously less strength but there is a pretty big gap in material properties.

\n\n

The only method I've thought of that might work is drilling a small hole, or series of holes in my part to inject an epoxy into the part. Haven't tried it.

\n\n

I'm open to any possible ideas or advice if someone has tried something like this. Not sure if this is necessarily the best place but thought it's a good place to start.

\n", "Title": "Post processing FDM for strength", "Tags": "|fdm|post-processing|", "Answer": "

Recognizing that the posting party feels that FDM constructed parts are of insufficient strength for his purpose and allowing for proper layer bonding, one can understand that the model can be perfectly constructed and not reach the strength objective.

\n\n

Filling a model with an epoxy or a casting resin will provide additional strength. Testing smaller, non-critical models is recommended to determine the level of increase. The design has to be re-engineered to provide for resin/epoxy flow within the model. Some epoxy and resin formulations generate heat when curing and may soften the model. The solution in such cases is to mix and pour small amounts, allowing for a pause between pours.

\n\n

An alternative to filling a model is to reduce the perimeter (if applicable) and apply a reinforcing layer. I've constructed satisfactorily printed models with insufficient strength for my purposes, but then applied fiberglass cloth and epoxy resin to the outside. A single layer provided the necessary strength in my case.

\n\n

One could apply sufficient layers to provide all the necessary strength, effectively turning the model into a positive mold. This is commonly done with amateur-built aircraft of the Burt Rutan design style. More recently, an article appeared on the internet of a model being printed with wash-away support material only, covered in carbon fiber. The wash-away was washed away and the wing structure became the product. For your application, it may not be necessary or practical to remove the inner model.

\n\n

Just as with the injection concept, one must re-engineer the model to allow for this type of reinforcement. Edges will have to be radiused or the sharp termination of reinforcement layer will become a weak point. Tight inside angles will have a similar problem.

\n\n

Fiberglass cloth comes in various weights, measured in ounces per square foot (US). The lighter cloth is more capable of \"turning corners\" and fitting into tight angles.

\n" }, { "Id": "3728", "CreationDate": "2017-03-12T12:07:23.187", "Body": "

Is there any way to test what kind your filament is? There are no labels on the spools and I don't know whether they are ABS or PLA.

\n\n

I got the plastic with the printer, which is no longer sold (Solidoodle 2). Since I bought it on eBay that is probably why it has unprofessional filament. The plastic filament came with the printer which is now off sale (Solidoodle v2).

\n\n

I set my extruder to 210 \u00b0C and bed to 50 \u00b0C and it printed fine (with tons of hairspray and painters tape).

\n\n

I figured out where I got it. I got it from Solidoodle (who have gone out of business) when I bought the Solidoodle 2 right after it came out.

\n\n

I bought PLA and ABS so it has to be one of the two. Any other ways without having to burn and smell plastic? I just have the roll with no numbers, works or anything on it. And how to I smell without breathing in the fumes?

\n", "Title": "Identifying the plastic type of an unknown spool of filament or recyclable waste", "Tags": "|filament|part-identification|", "Answer": "

I know this question is old, but the existing answers are all overly general relative to the question. If someone is in OP's same sitution, the \"prints at 210/50\" data point pretty much guarantees it's PLA. No subjective \"smell tests\" or playing with fire needed.

\n\n

The comments indicated some doubt that it's PLA due to lack of \"sweet\" odor, but PLA has very little odor at all, and whether you'd describe it as \"sweet\" probably varies by individual.

\n\n

Aside from \"PLA\" (usually blends of PLA of smallish amounts of proprietary ingredients), there's virtually nothing on the filament market that prints at these temperatures, especially not without adhesion and warping problems. TPU/TPE can also print at these temperatures, but you'd usually know if that's what you had just from the flexibility. These properties of PLA are largely why PLA is so popular.

\n" }, { "Id": "3732", "CreationDate": "2017-03-12T20:45:20.317", "Body": "

I have an stl with multiple parts that I want to split up. Cura 15 had an option to \"split object into parts\" but I can't find that in cura 2.4. Did it get removed?

\n", "Title": "Cura 2.4 missing \"split object into parts\"", "Tags": "|slicing|ultimaker-cura|", "Answer": "

Nope, not in my version 2.4+

\n\n

Seems like something so useful and silly to have to get yet another program to make it work. Like that is somehow faster? I have no problem with slow.

\n\n

Can't believe it would be slower than hopping into another program, opening file, dividing file, saving stl file, then reloading into Cura again... Cura slicing it would need to be awfully slow for that to be faster.

\n" }, { "Id": "3742", "CreationDate": "2017-03-14T20:53:52.453", "Body": "

I have an STL file and I would like to know how many grams would this print consume. Is there a software so I can get it or an online link that can say me that?

\n", "Title": "How many grams will be used in a print", "Tags": "|filament|abs|", "Answer": "

Slic3r will calculate the weight after you have exported the G-Code.

\n\n

Normally, you'll only see the \"Info\" box. But after saving as G-Code, there's a \"Sliced Info\" which mentions the weight according to the settings you made (filament type and infill).

\n\n

That's how it looks like:

\n\n

\"enter

\n" }, { "Id": "3750", "CreationDate": "2017-03-16T14:57:18.373", "Body": "

I am wanting to buy a 3D printer to add to my shop.

\n\n

I am an engineer and enjoy making/building things so the kit idea sounds fun and economical.

\n\n

I see Tronxy has two different styles for their larger printers:

\n\n\n\n

As far as I can see, both printers have the same basic resolution, accuracy, and material specs.

\n\n
    \n
  1. What are the advantages/disadvantages/differences between the\nP802 and X3?
  2. \n
  3. How important is auto-leveling?
  4. \n
\n", "Title": "Tronxy printers", "Tags": "|print-quality|hardware|resolution|tronxy-x3|tronxy-p802|", "Answer": "

I ended up buying a TronXY X3 and have had it several months.

\n\n

I have seen several videos on the P802 and the X3 so I believe I can answer this question fairly.

\n\n

Here is what the two printers look like

\n\n

P802 \"P802\"

\n\n

X3 \"X3\"

\n\n

Here is a comparison of the features\n\"enter

\n\n

Here are my overall impressions:

\n\n\n" }, { "Id": "3754", "CreationDate": "2017-03-17T01:43:29.620", "Body": "

I have some files for 3d printing that I want to do structural analysis because I plan to apply loads on them. I know there is a Solidworks analysis module for this purpose called Simulation, but according to what I have read about this software it is not very effective for analyzing parts that have been created by layers such as 3d prints.

\n\n

Does anyone work with this and could describe important features to make a good structural analysis?

\n", "Title": "Structural Analysis", "Tags": "|support-structures|", "Answer": "

For running an analysis on FDM printed parts Solidworks Sim would be extremely difficult to use. It's not really designed for running this type of analysis and is at the fairly low end of FEA analysis packages. An FDM analysis should be treated almost like a composite structure and you'd have better luck running something like ANSYS, or NASTRAN but those are by no means the only two. To get good (and accurate) results from FEA you'll need to model each layer, the bonding between each and the internal structure.

\n\n

I've never done it myself but i've looked into doing it. The mesh required for even a fairly simple part looks pretty massive. Certainly possible and certainly doable, but you'd need to weigh your expectations against how much time it would take to model everything, set up the mesh and then run the analysis. In my experience, given that most FDM prints are pretty quick to build, it always seemed to be more efficient to build a few parts load them up and watch how they fail in a structural test. 20 hours of printing and then an additional 10-15 of setting up and running the tests were faster than running a proper FEA.

\n\n

For a quick and dirty FEA, i'd use the solid model of my part and only consider loads that acted normal to the Z-Axis of the print. Any loads that act in shear apply a huge FOS. The closer to pure shear the larger the FOS you'd want to have. And any thin, tall parts i'd assume can take no loading outside of pure compression.

\n\n

Edit:\nJust noticed that this question had a tag on Heat-management

\n\n

As far as a Heat loads go, it'll depend heavily on the infill type and density you have on your model. Everything from above will still apply, but the loads would then be thermal loads instead of structural. Particularly with FEM at best your first analysis will only be within about 50% of true values and you will then need to update your thermal model with the results from your first verification test.

\n\n

Depending on the accuracy required from the analysis you could get away with modelling a hollow part with the walls of the part being the thickness of the printed shells. Higher accuracy analysis you'll have to model all layers as they are printed. If you're just looking for a 'will it melt' analysis, thin walls will work fairly accurately assuming you're confident in your material properties.

\n" }, { "Id": "3755", "CreationDate": "2017-03-17T06:48:39.623", "Body": "

I am about to purchase a TronXY X3 or P802; but, my PC is running Windows 10.

\n\n

The spec sheet for the printers does not list anything above Windows 7.\nIs anyone using either of these printers with Windows 10?

\n", "Title": "TronXY Windows 10 driver", "Tags": "|prusa-i3|software|electronics|tronxy-x3|tronxy-p802|", "Answer": "

For Windows 10 driver, go directly to the Microsoft update catalogue.\nhttps://www.catalog.update.microsoft.com/Search.aspx?q=USB%5CVID_1A86%26PID_7523

\n

Download the .cab file, extract the contents. Using Device Manager update the driver for the partially installed device. After the update TronXY-2 now appears as a COM port.

\n" }, { "Id": "3771", "CreationDate": "2017-03-19T14:24:01.617", "Body": "

Is there a commercially available fume and nano particle extractor for a 3D printer, like the Ultimaker3 extended? I'm looking for a safe solution, to use at home, for around $800.

\n", "Title": "Commercially available 3D printer fume and UFP extractor", "Tags": "|diy-3d-printer|safety|enclosure|", "Answer": "

There are two main options you could consider. The first being a diy filter and the second being a commercially available fume extractor.

\n\n
\n\n

Option 1

\n\n

You could make a filter yourself. On Thingiverse, there are many ideas that you could implement. A few examples are these :

\n\n

https://www.thingiverse.com/thing:1992079

\n\n

https://www.thingiverse.com/thing:2285882

\n\n

https://www.thingiverse.com/thing:2105113

\n\n
\n\n

Option 2

\n\n

Buy a commercially available fume filter. An example that would be well below your budget ($178) would be the Zimpure (https://www.zimple3d.com/zimpure/). This is a device that will vacuum the fumes right by the nozzle and filter them. Additionally, you could get this filter from Matterhackers : MatterHackers. This could be an option if you are willing to go a little bit above your budget. If you would rather have a filter for 30 bucks, you can also look at this filter that attaches to your enclosure : https://www.3dupfitters.com/products/fan-and-charcoal-air-filter.

\n\n

Lastly, if you wanted to go all out, you could also get this enclosure : https://www.3dprintclean.com/.

\n\n
\n\n

I hope any of these solutions named might be beneficial to you :). For even more options, visit this post : What air filtration options exist for enclosures?

\n" }, { "Id": "3776", "CreationDate": "2017-03-20T00:12:31.033", "Body": "

I have built a few parts that have printed 'pins' on them (2mm diameter 1.5 cm length). Due to the orientation that the build requires the pins have to have support material on them. (The part has to be printed with the pins parallel to the build plate for strength reasons and the orientation can't change).

\n\n

The problems i'm having is that the surface of the pins are quite rough. The pins are designed to press-fit into a soft rubbery tube, and the pin surface roughness is cutting into the softer tube. The roughness is due to both the layer edges of the print, and that the support material leaving some 'burrs'. I'd like to smooth the outside of my pins with some type of post processing technique. Ideally I'd like to keep the pins as close to dimensionally accurate as possible, but I realize they won't be perfect.

\n\n

The two methods I've considered are sanding the pins by hand and placing my part in an acetone vapor chamber for a while.

\n\n

Sanding, I think, is the best option of the two so far but it's a bit tedious, and is quite difficult to do, due to the size of the pins, their location and my fingers are pretty big relative to the space I'm working in.

\n\n

I don't like the idea of using acetone because the few times I've tried doing this in the past, my parts always came out warped or misshapen I think due to the relief of stresses. (or I did it wrong, both seem plausible).

\n\n

If anyone has a third option (or more) I'd be glad to hear. Or if there is another approach altogether that I haven't considered that can be done on an FDM style printer that would also be appreciated.

\n", "Title": "FDM, ABS Post-processing methods", "Tags": "|abs|post-processing|", "Answer": "

Your objective is quite a challenge, considering the limitations within which you have to operate. Small diameter of an appreciable length is daunting indeed.

\n\n

I would consider to use a regulated heat source such as a soldering station, one with removable tips. After electing an appropriate tip to handle the dimensions, a hole drilled into the tip to accept the pin diameter and length would be the next step.

\n\n

There would be consideration needed for the plastic material pushed ahead of the soldering tip, as well as angular stability while pushing the soldering iron onto the pins.

\n\n

To improve these circumstances, a sharp edge ground at the end of the soldering iron tip would give a better shearing of the excess plastic.

\n\n

The temperature control would be critical to prevent destruction of the pin within the tip and to provide optimum removal via the cutting end.

\n\n

Directly related to the above, but without the heat would be the equivalent tool to be used in a rotary grinder such as a Dremel. The bore of the custom-made cutter would match the pin and the sharp cutting end could also have serrations such as those found on hole-saws. Low speeds and a steady hand are required!

\n\n

If either of the above suggestions do not remove sufficient plastic roughness, one would make the pins of a larger diameter and ensure that excess is removed to specifications.

\n\n

EDIT: Additional thoughts. Hobby stores and online equivalents will sell very small diameter tubing. It might be an easy matter to find 2 mm inside diameter tubing of brass or even stainless steel. A few passes on a piece of sandpaper while the tubing is chucked in an electric hand drill and you have a sharp cutting edge.

\n" }, { "Id": "3782", "CreationDate": "2017-03-22T20:02:09.020", "Body": "

My printer just stopped working. It seems like it's working perfectly, but it gives me an error whenever I move the Z-Axis (Error: Printer set into dry run mode until restart) then, the bed temperature appears to be at 293\u00baC when it's actually at 95\u00baC.

\n\n

I suspect that the bed wires short-circuited since they got stuck behind the bed when the printer was moving Y-Axis to home (the wires blocked the movement). Since then, the error appeared.

\n\n

I suspect that the controller board could have been harmed by the supposed short-circuit, but despite the error, I can move perfectly any axis; I can heat the bed and the extruder; I can extrude filament; it is just the error which doesn't let me print.

\n\n

Printer:\nAnet A8 Desktop 3D Printer Prusa i3

\n\n

Software:\nMatterHackers: MatterControl 1.7

\n\n

Update: The bed temperature is stuck at ~95\u00baC, sometimes ~250\u00baC, even if I disconnect the sensor wires and restart the printer, so apparently, it's failing to read the bed temperature.

\n", "Title": "Error: Dry mode when moving Z-Axis", "Tags": "|heated-bed|", "Answer": "

I bought a new board controller and it's working perfectly

\n" }, { "Id": "3784", "CreationDate": "2017-03-23T10:47:35.790", "Body": "

My mono mendel 0,0 position is on the left of the bed, when the nozzle is at the position of the green dot in this image.

\n\n

\"Mendel

\n\n

I would like to move that point to the left a bit, in order to print a slightly larger part (the bed has about a centimeter more space to the left). I know I can just allow negative values but I'd rather not to, for safety reasons.

\n\n

However I cannot find where the offset of that point from the endstops (home position) is declared. There is no offset set in either pronterface or Slic3r, and unless my board came preloaded with different firmware than the one reprappro publishes, the *_HOME_POS variables in their Marlin repository are all equal to 0.

\n\n
// The position of the homing switches. Use MAX_LENGTH * -0.5 if the center should be 0, 0, 0\n#define X_HOME_POS 0\n#define Y_HOME_POS 0\n#define Z_HOME_POS 0\n
\n\n

https://github.com/reprappro/Marlin/blob/master/Marlin/Configuration.h#L290

\n\n

Any ideas how I should go about it?

\n", "Title": "Resetting the zero position of the X axis on a RepRapPro Mendel Mono", "Tags": "|marlin|g-code|", "Answer": "

I think the cleanest option is to move the X-endstop back about a centimeter to match, and then increase the size of the bed in both Slic3r's and the printer's settings.

\n\n

You could also try using M206 to set a persistent offset after homing.

\n" }, { "Id": "3786", "CreationDate": "2017-03-24T06:27:41.337", "Body": "

I'm having a similar issue to this one: I have retrofitted my printer with a BuildTak sticker. I'm printing PLA at 60\u00b0C bed temperature - currently using a 0.3 mm nozzle, but I had similar issues with the 0.4mm nozzle before.

\n\n

If I bring the nozzle close enough to the print surface to get a reasonable first layer, I need excessive force to remove the parts afterwards - so much force that I already have damaged the BuildTak in one or two places. If I lower the bed (the physical bed using the screws, not the temperature!) so that the parts come off easier, the first layer is very uneven and it takes 5-6 layers until the layers start to stack onto one another the way they are supposed to. What can I do to influence the adhesion of the printed parts to the BuildTak surface - especially to lower it in a controlled manner without changing the nozzle height?

\n", "Title": "Lower BuildTak adhesion", "Tags": "|pla|adhesion|", "Answer": "

These are more like backups for when the part gets stuck, but you could try spraying that part where it meets the bed with liquid computer duster. The shrinkage from cooling usually helps to release the part from the bed. You could also try heating the bed very hot to save the bed at the expense of the part.

\n" }, { "Id": "3793", "CreationDate": "2017-03-26T16:44:21.083", "Body": "

I have a makerbot, and am using their slicer program. The part I am printing has an extremely coarse thread, 5 threads per inch with a 2 mm thread height. The thread face is at a 30 degree angle (15 degrees of overhang)

\n\n

I have printed external threads like this without issue and without support material before for various gardening tools (a flow straightener for example). I'm now trying to print an internal threaded part however the rest of the part needs to have support. I've played with the settings in the makerbot software but can't find a combination that will acomplish my goal.

\n\n

I'm looking for an idea, a setting, or an alternate way of printing the threads without support, but still have supports for the other sections of the part that I know do need them.

\n\n

Perhaps there is an alternate software that can do something like this, or is best practice to modify the part to have weak structural elements to support the extremely overhanging sections that I'll then break off afterwards and print the part with no supports?

\n", "Title": "Printing coarse threads", "Tags": "|3d-models|makerbot|support-structures|", "Answer": "

Try using simplify 3d. It's a very powerful slicing software compatible with almost every 3d printer. It does cost 150 dollars, but that would be the only drawback to using that software. You can manually place supports around your model without having he software mess it up. I think this is exactly what you're looking for.

\n" }, { "Id": "3795", "CreationDate": "2017-03-26T22:28:04.593", "Body": "

I replaced the controller board in my Monoprice Select Mini with an Arduino/Ramps setup and compiled an uploaded Marlin 1.1.0-RC8 to run it. I've got most everything calibrated and working with one exception.

\n\n

I tell the printer to home for xyz and then level my bed with a piece of paper then start a print and the nozzle consistently drops too far down into the bed and nothing can extrude. If I adjust the bed and put 1-2mm gap between the nozzle and bed then it prints fine.

\n\n

I can't find anything in Marlin to adjust for this and I'm kind of stumped. I'm printing the original cat gcode that came with the printer that should just work fine as it always has and shouldn't have anything that a slicer would put in there to screw things up.

\n\n

Can anyone point me in the right direction?

\n\n

This is the output of M503

\n\n
Send: M503\nRecv: echo:Steps per unit:\nRecv: echo:  M92 X93.00 Y93.00 Z1097.50 E99.00\nRecv: echo:Maximum feedrates (mm/s):\nRecv: echo:  M203 X300.00 Y300.00 Z5.00 E25.00\nRecv: echo:Maximum Acceleration (mm/s2):\nRecv: echo:  M201 X3000 Y3000 Z100 E10000\nRecv: echo:Accelerations: P=printing, R=retract and T=travel\nRecv: echo:  M204 P3000.00 R3000.00 T3000.00\nRecv: echo:Advanced variables: S=Min feedrate (mm/s), T=Min travel feedrate (mm/s), B=minimum segment time (ms), X=maximum XY jerk (mm/s),  Z=maximum Z jerk (mm/s),  E=maximum E jerk (mm/s)\nRecv: echo:  M205 S0.00 T0.00 B20000 X20.00 Y20.00 Z0.40 E5.00\nRecv: echo:Home offset (mm)\nRecv: echo:  M206 X0.00 Y0.00 Z0.00\nRecv: echo:Material heatup parameters:\nRecv: echo:  M145 S0 H180 B70 F255\nRecv:   M145 S1 H240 B110 F0\nRecv: echo:PID settings:\nRecv: echo:  M301 P26.15 I2.74 D62.35\nRecv: echo:  M304 P231.09 I45.21 D295.34\nRecv: echo:Filament settings: Disabled\nRecv: echo:  M200 D3.00\nRecv: echo:  M200 D0\nRecv: ok\n
\n\n

I also wanted to test whether the nozzle is actually moving closer. I did a G28 and manually put a piece of paper's width gap between the nozzle and the bed at all points. Then I started a print with no filament but with the sheet between the nozzle and bed. When it got to temp it homed all the axis and moved the nozzle to the first position. I would expect the paper to maintain the same gap but it tightly presses into the paper. The first gcode move that includes the Z before extruding is :

\n\n
G0 F3600 X42.228 Y46.985 Z0.3\n
\n\n

Which you would expect would put even more space between the nozzle and bed for the first .3 layer of filament but it isn't.

\n\n

I printed directly from the SD Card to hopefully rule out Octoprint running gcode before the print so I really think the culprit is Marlin at this point.

\n", "Title": "Z axis at print is lower than when I home", "Tags": "|marlin|ramps-1.4|arduino-mega-2650|", "Answer": "

I found the issue. When the hotend and bed are at temps for PLA everything works fine but at temps for ABS the Z offset would get all messed up. After a bunch of testing I was able to track it down to a single gcode statment G1 Z15.0 F6000 \nAt the higher temps my Z stepper skips steps at that feed rate.

\n\n

The \"Custom FDM printer\" machine settings I used for my printer in Cura had that statement in it and so did the cat gcode that was on the card from the factory. I changed the feed rate to 200 and was able to print in ABS.

\n\n

I was surprised Marlin didn't have a 'limit the feedrate for Z to this number' setting.

\n" }, { "Id": "3809", "CreationDate": "2017-03-31T21:18:46.847", "Body": "

As the title suggests, I am looking to remove nylon that has adhered strongly to my build platform. Mechanical methods (ex. scraping) haven't yielded results, so I was hoping for a solvent or something that would remove the adhered nylon.

\n\n

Also, the parts themselves remove well enough, but some areas have a bit of nylon that simply persist.

\n", "Title": "How to remove adhered nylon from build platform?", "Tags": "|build-plate|", "Answer": "

One important and missing aspect of your question is what is your build platform? Glass is likely to give you the best results, while aluminum has some risks, depending on your choice of solvents.

\n\n

My hasty research at first showed Glacial Acetic Acid to be a candidate, but as a very strong acid, it will require some care in use.

\n\n

I found a pdf document with a comprehensive chart of possible solvents for nylon. The above noted solvent (100% acetic acid) lists with an Unsatisfactory result. I read this at first as being unsatisfactory for dissolving, but the correct interpretation is that nylon is unsatisfactory for being resistant to this solvent. Additional notes show nylon will dissolve in this solvent. I have a bottle of white vinegar (acetic acid) but the panel reads as five percent concentration, almost certainly ineffective for your purposes.

\n\n

There are other solvents listed as unsatisfactory, which points them in the right direction for your purposes. A couple of them are in the chlorine family and no reference is made regarding concentration. Chlorine of even weak concentration will attack aluminum very quickly.

\n\n

Despite the weak concentration of the white vinegar, I've dropped a segment of 3 mm nylon to see what happens overnight and may report via an edit here later.

\n" }, { "Id": "3814", "CreationDate": "2017-04-01T18:54:27.047", "Body": "

Lately I'm having some issues with white PLA.

\n\n

It sticks nicely to the print bed, but the very last layer of the print comes out really ugly.

\n\n

I'm not a native english speaker, I don't know what is the proper term to describe what I see, so I'm attaching two pictures instead:

\n\n

\"enter\n\"enter

\n\n

As you can see, the surface has wrinkles. If you touch it, it feels like a jaggled surface. What is causing this? I've never had this kind of issue before with the same PLA roll, what can be the issue?

\n\n

I have a prusa-style printer, 45\u00b0C printbed, 225\u00b0C hotend, 10% hexagonal infill, 30mm/s print speed. The object stays on the surface without warping, I can tell for sure that it isn't detaching from the bed and coming up towards the extruder.

\n\n

It happens only for infill, perimeters are just perfect.

\n\n

What can be the issue here?

\n", "Title": "Why 3d print has an ugly surface?", "Tags": "|pla|", "Answer": "

This looks like the air trapped inside the print is expanding, causing wavy surfaces. This can be solved by lowering nozzle temperatures, and turning on the fan.

\n" }, { "Id": "3820", "CreationDate": "2017-04-03T14:15:28.790", "Body": "

I'm replacing my standard MK3 Aluminium printing bed with a custom sized bed paired with a silicone heater. I came across a few on Amazon and they all have grades such as 6061, 3003, 5052 etc. I know these mean the composition of the Al alloy and their end purpose but I'm not sure what they mean in terms of being used for a 3D printer. Can anyone here advise me on this?

\n", "Title": "What kind of aluminium grade for a heated 3D printer bed?", "Tags": "|heated-bed|", "Answer": "

Various grades and alloys of aluminum will have characteristics related to ease of welding, resistance to corrosion, malleability, and other aspects. For a heated bed on a 3D printer, you'd really want to have something resistant to warping and something that can be assured to be planar across the surface, that is, flat.

\n\n

The terminology you'd want to use for your search is \"MIC 6 Cast Aluminum Tooling Plate\" and the results are many.

\n\n

From a rather comprehensive web site:

\n\n

Flatness tolerance is maintained within .015\" on 1/4\"-5/8\" thickness and .005\" on 3/4\"- 4\" thickness. Thermal Cycling can be performed up to 800\u00b0 F under controlled conditions.

\n\n

You would want to confirm from the seller that the surface has been prepared, as some sites appear to sell un-finished tooling plate, but I've not been able to clarify that. Most appear to provide either no specifications regarding flatness or give a figure such as that above.

\n" }, { "Id": "3823", "CreationDate": "2017-04-04T03:13:47.447", "Body": "

I am facing a problem with my 3D print. Whenever I am printing any object, the print from the top is shifting to the right hand side - it is symmetric at the bottom but not at the top.

\n\n

Checkout the photo below:

\n\n

\"101hero

\n", "Title": "101Hero 3D Printer - Printed object is tilted", "Tags": "|print-quality|3d-models|3d-design|ultimaker-cura|101-hero|", "Answer": "

Make sure you are using cura 15.02.1, also download the configuration file from the 101hero website, and upload it to cura using the\" open profile\" button. Make sure your extruder temp is set to 198. I use a 113% flow rate on my 101hero.

\n" }, { "Id": "3826", "CreationDate": "2017-04-04T16:42:23.250", "Body": "

I've never gotten a 3D printer, nor do I know very much about them, but I'm thinking of buying a delta 3D printer kit for around $450. When I looked online of some disadvantages of Delta printers I found that they typically don't have Bowden extruders. I'm wondering what are the advantages of a Bowden extruder and should I spend more money in a 3D printer just to get one.

\n", "Title": "Delta 3D printer extruder?", "Tags": "|extruder|delta|", "Answer": "

The only advantage of Bowden extruder is a reduced mass of moving parts attached to the effector.

\n\n

All other differences from direct extruder are to the Bowden's disadvantages. Generally speaking, Bowden-type extruder has much worse control of filament extrusion than direct one.

\n\n

Since most of the modern consumer-grade printers suffer from low rigidity, reducing mass seems as a good trade-off to the printers' manufacturers.

\n\n

Unfortunately, for the Delta printers trading speed for quality may not be so effective. Delta construction is used with the only goal to achieve highest possible speed of printing. As a contrary, Bowden extruder becomes less precise with the speed increase. This makes it unreasonable to increase print speed at some point when printing quality degrades below acceptable.

\n\n

As a result, at this moment of time, Delta printers with Bowden extruder is a clear sign of under-performing device.

\n\n

To answer your question: there is no reason to pay extra for the Delta printer with Bowden extruder, although I doubt that a printer with similar characteristics and a direct extruder would be cheaper.

\n" }, { "Id": "3837", "CreationDate": "2017-04-10T13:07:30.243", "Body": "

First of all I apologize if this stack exchange site isn't the proper spot for CNC questions but I do not see any other ones that are close. If the community feels this is too far off topic then just let me know and I will remove this question. Even though this is for a CNC machine, the same issue could exist for a 3D printer. I do know this is very specific to the TAIG CNC.

\n\n

Currently I have the TAIG CNC machine. Recently it has been getting bound when going up in the positive Z direction. It will go down no problem but when going up, at certain points the motor will make a high pitch noise like it normally does when it hits a Z extreme. At these points if I manually try to crank (with the controller off) it is very hard to move up. This indicates that it is a mechanical problem.

\n\n

I took apart the Z axis and understand that there is a jib inside. I do understand how the jib works, tightening and loosening the top and bottom screws to move the wedge. I have tried with the wedge all the way up (biggest gap) and all the way down, along with in between in small increments. It will go up when the jib is all the way up but if I press down with my hand barely then it gets bound so I am thinking it is something other than the jib? I am not sure which other parts I can adjust or would get stuck. I did follow the directions on youtube of setting the brass insert that is attached to the threaded rod, where I tightened it to the point of where the motor couldn't move and then backed off a little bit. Has anyone had binding troubles like this?

\n\n

I am a pretty new member on this forum so I apologize if I missed anything but feel free to ask any questions too.

\n", "Title": "TAIG CNC - Z axis getting bound", "Tags": "|mechanics|", "Answer": "

So inside the Z axis is a brass component that is connected to the threaded rod (lead screw). This brass component has 3 screws and depending how tight / loose, determines how much it will bind up. You of course don't want it too loose otherwise the enclosure will \"jump\" off of the track at times. In my case, the two screws were too tight. The way I determined how tight to go was taking the entire Z track off of the back, tightening both screws just to the point of binding and then backing off 1/2 turn on both.

\n" }, { "Id": "3838", "CreationDate": "2017-04-10T19:01:24.930", "Body": "

Why does the Ultimaker 3D Printer has a Heater + Heater transfer plate (aluminium) + Glass?

\n\n

I wonder why a glass plate, and if is possible to remove the glass and print directly in the aluminium plate adjusting the heating.

\n\n

Link to the ultimaker.

\n\n

Pictures:

\n\n

\"Ultimaker

\n\n

\"PCB

\n", "Title": "Why does the Ultimaker 3D Printer has a Heater + Heater transfer plate (aluminium) + Glass?", "Tags": "|heated-bed|ultimaker-3|", "Answer": "

According to this page, heat transfers more evenly across an aluminum build plate than with just glass. But as Kevin pointed PLA adheres better to glass because it doesn't flex as much as aluminum under heat.

\n\n

The link above shows that aluminum has a much higher thermal conductivity at 205 (W/(mK)) vs glass at 105 (W/(mK)) at 25 °C (77 °F).

\n\n

Because of this conductivity difference, you may find that it takes longer to heat the glass plate, but it should heat more evenly.

\n" }, { "Id": "3841", "CreationDate": "2017-04-11T10:05:07.007", "Body": "

I'm considering removing the driver motor from the extruder assembly, and placing it on a stationary mount point instead, and then using a flex-shaft type connector from the motor to the extruder assembly to actually drive the extruder. The motivation for this change is to reduce the overall weight of the extruder driver and hot end assembly, allowing for quicker movement of the carriage on it's associated axis(one of X,Y).

\n\n

Would it be better to run a pair of drivers (one for each direction) to manage reversing the filament pressure or would it be better to use just one driver and reverse the motor as usual?

\n", "Title": "Weight Reducing Design Change for Extruder Driver", "Tags": "|extruder|fdm|extrusion|extruder-driver|", "Answer": "

Very cool idea,

\n\n

One motor would definitely be more than capable of producing the required torques even through a flexshaft connector. For any normal sized 3D-printer the torques required, and the speeds you'll need for rapid response are well within the capabilities of any off-the-shelf stepper motor.

\n\n

Just a note on the idea though, with a normal, 'rigid', connection there is essentially no winding or unwinding, and only the backlash between the gears and the filament to consider, and that is effectively zero. With a flex-shaft though, the stack-up of twists and flexing will be much greater. The system will require more rotations at the source to effect the same amount of torque at the end effector as the flex shaft flexes and bends under the load. For tiny torques with short flex shafts, this wont be an issue as filament pressure is pretty minimal. But if you scale up this project or start working at much higher speeds, you may run into some issues with this design.

\n\n

I have no idea how big or fast you'd need to be working at for this to begin to become a problem. I'm imagining pretty big though. Just something you might want to keep in mind if you try turning this into a huge, super fast 3D-printer.

\n" }, { "Id": "3842", "CreationDate": "2017-04-11T10:41:05.527", "Body": "

In a discussion about motors with a friend who used to work in the robotics industry, he told me that he despised stepper motor systems, as every stepper based system he had worked on required a bunch of hacky software fixes to make the system perform to the required level.

\n\n

He said that servo motor based systems had their own foibles, but at least they could generally be tuned out and you always knew that if the encoder said you were in a given place then you would be (to within the constraints of the backlash compensation).

\n\n

Because of this, I was wondering if there were any options for using brushless DC motors + encoders + drive electronics instead of steppers + drive electronics.

\n", "Title": "Reprap variants with servo motors rather than stepper motors?", "Tags": "|stepper|servo|", "Answer": "

You can get \"stepper replacement\" servo drives that supposedly put all the needed control in the drive, and accept ordinary stepper inputs. Those should make the servo-drive a \"drop-in\" option on anything that uses stepper drives.

\n\n

That said, I've seen an affordable CNC router system based on steppers turned into a much more expensive system based on servos, and I still have the \"obsolete\" stepper version, as the servo based system is MUCH more expensive and I could not keep up with the \"upgrades\" and never would have bought it at the price it now goes for. I'm in fact considering changing that to a reprap control system (still as a router, not as a printer, as I currently think.)

\n\n

Given some practical limits to printhead speed in additive 3D printing based on the material solidifying, there may not be a lot of benefit in the considerable added expense of servos. They can move faster, but how much of that will translate to actually printing faster? How fast can you melt and pump plastic and have it stay where you put it?

\n" }, { "Id": "3853", "CreationDate": "2017-04-13T14:23:07.440", "Body": "

The backstory: I'm installing a pigeon net in my home. Because of the shape of the opening I'm installing the net in and the material on the sides it's difficult to anchor the net using the normal means but I can print clips that will hold the net in place.

\n\n

The clips will be outside and will be exposed to the weather and direct sunlight, the weather here is relatively hot (up to 30C) with a lot of sun most of the year and rain in the winter.

\n\n

I only have PLA, ABS and PETG available, anything else will take too long to arrive.

\n\n

I don't care about the parts changing color and mostly I don't care about them deforming a little bit - only about breaking.

\n\n

If the parts have to be replaced after a year I'm ok with it, less then that will be annoying, longer will be better.

\n\n

So, under those conditions, which of the 3 materials is more durable?

\n", "Title": "Which is more durable to sunlight/weather - PLA, ABS or PETG", "Tags": "|print-material|material|outdoors|", "Answer": "

I might be late to the party, but I have a suggestion. Why not use ASA filament, it's the kind of plastic used in car cup holders, lawn rakes and sprinkler heads, it is both heat and cold resistant, as well as solar stable and weatherproof. As far as printing with it it is similar to printing in ABS as far as I am aware, I might be wrong though. I personally am surprised how often this kind of question pops up and literally No One mentions ASA filament. I think I might buy some because I like the idea of printing something that can be left outside.

\n" }, { "Id": "3854", "CreationDate": "2017-04-13T14:41:27.853", "Body": "

I know that for printing some materials like the carbon-fiber filled PLA it's recommenced to use a steel nozzle since it'll wear-out a standard brass nozzle quickly. Is there any reason to prefer using a brass nozzle for 'simple' plastic-only filaments instead of just printing everything with steel?

\n", "Title": "Is there any reason *not* to use a steel nozzle for printing all materials?", "Tags": "|filament|print-quality|nozzle|", "Answer": "

Thermal conductivity of brass is approximately twice as better than steel (not stainless). Given that the size of nozzle is relatively small, it should be able to transfer enough heat for a medium-speed prints at least.\nI have printed PLA and ABS using "steel" nozzle (brand of steel unknown) at the speed around 80\u00a0mm/s without any visible differences compared to brass.

\n

However, an ideal nozzle (I see) is made of tempered bronze with polished inner channels. Bronze is known to have better friction coefficient and tempered one should be enough to resist carbon-type filaments more efficiently.

\n" }, { "Id": "3861", "CreationDate": "2017-04-15T19:48:06.630", "Body": "

I'm looking for some advice, rules of thumb, or models for how to design parts for layer bonding strength.

\n\n

As an example part, lets assume a circular beam in bending (tall cylinder). Depending on how the part is setup on the build platform (tube axis aligned parallel to Z, or setup to lay in the X-Y plane) the layers and internal structure will then be different depending on the orientation chosen at the time of building. Does/has anyone done any investigation to what roles these factors will play on simple shapes like a solid cylinder?

\n\n

Ideally, with a CNC'd, injection molded part the total strength of a solid part is quite easy to model (mathematically) for strength and flexibility. However, with a 3D printed part the orientation makes the isotropic material properties effectively anisotropic, as layers don't bond perfectly, different cooling rates, and a host of other reasons.

\n\n

Ideally I'd love to find a 'simplified' model that can be applied to hand-calculations to come up with rough approximations to part strength and bending. My guess is that these parts would be modeled something similar to an ABD matrix, similar to composite materials, but that's just a guess.

\n", "Title": "FDM layer bonding strength", "Tags": "|3d-models|3d-design|print-strength|", "Answer": "

I have not been able to find a simple model for FDM part strength. FDM parts are pretty complicated as they have a LOT more things that affect their strength than just layer adhesion. Since any \"solid\" part will have infill, the part can't really be modeled as a laminant. There are so many settings you can play with in the slicer that effects part strength, the model would have to have tens (if not hundreds) of parameters. Also, you would likely have to establish them for your own printer since lots of things can affect them (like room temp, ventilation, humidity, material, material storage, ...)

\n\n

One empirical example discussing the affects of FDM choices on part strength is a video done by Angus at Marker's Muse. In the video he discussed how orientation and wall thickness affects strength. Angus is not a math/ME guy (by his own admission). FEA or other modeling would not be his approach; but, you can get some insight from his experience.

\n\n

On the modeling front, I have seen one company that did create an FEA model to test whether their product would be strong enough if they produced it using FDM. Here is the publication they wrote showing their analysis approach. It will give you some insight on how they approached what I think you want to do.

\n\n

Note: They do offer a service where you can pay to have your part analyzed for a considerable amount of money. I have no idea how much money; but, based on jobs I have quoted in the past, I would expect it would be a least a few hundreds dollars.

\n\n

As for me, I really enjoy basing my decisions on models and understanding how a part/process works. In this case; however, an empirical/experimental approach might be more cost effective. If you really want to take an analytical approach (and have the time to spend doing it), I would recommend choosing several key parameters and run an analysis using DOE (Design of Experiments).

\n\n

If you come up with something, I would love to see it posted here.

\n\n

Good luck and happy printing.

\n" }, { "Id": "3863", "CreationDate": "2017-04-16T15:19:56.283", "Body": "

The overall goal is to create a print of the liberty bell. No clapper is needed. The support beam is not needed. I am just looking to mimic the outside shape of the bell. After an application of Google-fu I found this image. It has been cleaned up and cropped a bit.

\n\n

\"enter

\n\n

In OpenScad the surface function was applied to the png to generate a height model. I subtracted out a few cubes, and eventually got it down to a layer of about .1 height.

\n\n

My Evil (pun intended) plan is to get it down to a 2d model. Then by shifting a copy of model from itself, and subtracting the two. That should leave just the curve. Then rotate_extrude the curve to generate the actual bell at the desired thickness.

\n\n

I see the problem coming. If the model is layered down to zero height it will vanish. Leaving me nothing to rotate.

\n\n

So on to the question.

\n\n

Is there any tool or technique to turn the image into a 2d openscad object or failing that is there another way to approach the problem?

\n\n

\"\"

\n\n

The direction that fred pointed me at is viable. I didn't use the actual image shown here, but a manually generated one from OpenScad. As a proof of concept the technique works well.

\n", "Title": "Is there a technique for turning an image into a 2d model?", "Tags": "|2d|openscad|", "Answer": "

You could have done that with OpenSCAD, there are instructions for extruding.

\n\n

Personally, I'd just do a 1 altitude copy with \nImage 2 STL 2 Heightmap Mesh Converter Generator, then rotate in OpenSCAD. It's called rotate_extrude() and is used for seashells.

\n\n

In MeshLab you can use Laplacian Smooth to depixelate/antialias it.

\n" }, { "Id": "3874", "CreationDate": "2017-04-17T15:16:58.367", "Body": "

What does the pink color in Slic3r preview mean? Yellow is my model, green is support, and pink is..?

\n\n

If the pink color is some kind of warning, how do I fix it?

\n\n

\"Illustration

\n", "Title": "What do the pink colors in Slic3r preview signify? If it signifies a problem, how do I fix it?", "Tags": "|slic3r|", "Answer": "

In slic3r preview, salmon (pink) represents infill, yellow represents perimeters, and green represents support material, including skirt and brim.

\n" }, { "Id": "3884", "CreationDate": "2017-04-18T23:28:24.900", "Body": "

I apologize if this is in the wrong section, but it seems relevant.

\n\n

I am an absolute noob at 3D printing, but recently my need for a specific, small part intersected with access to a 3D printer at work (Zortrax M200), so I figured now was a good opportunity to get into printing.

\n\n

I am designing a clamp for smaller can capacitors that will hold them upright from a mounting surface. I would like the capacitors to snap into the clamp, but I am struggling to find information on how wide the opening to the clamp should be. It clearly needs to be smaller than the diameter of the clamp, but I'm not sure how much smaller it can be without snapping the clamp arms.

\n\n

Is there a rule of thumb or general guideline for this type of geometry? I am using Z-ULTRAT, which is some proprietary filament made by Zortrax. It seems to be similar to ABS in terms of strength properties, but stiffer.

\n\n

I don't have an image at the moment, but I can provide one tomorrow when I have access to the part files. The best analog I can think of is a LEGO figure's hands.

\n\n

Thanks.

\n\n

Edit: Per requests for additional information:

\n\n\n\n

Edit 2:

\n\n

Based on the link in the comments below, I came up with the following design:\n\"Clamp\"

\n\n

The little nubbins in the back fit into the \"neck\" at the bottom of the capacitor to provide some positive lock. 220 degrees of surround with 4mm thick arms - I think this should work.

\n", "Title": "Snap-in clamp geometry", "Tags": "|3d-design|print-strength|", "Answer": "

Based on the links and insights provided, I was able to adjust my design to that shown in Edit 2. I opted to instead have it professionally printed via Sculpteo for a very reasonable price, and can say that the design works as intended.

\n" }, { "Id": "3886", "CreationDate": "2017-04-19T13:05:37.113", "Body": "

I am creating a 3D printed part which will act as an attachment between a plate (actually a strain gauge) with two M2.5 bolts and a 2kg load with a single M4 bolt. I wouldn't be happy with tapping a screw thread into the print itself so I want to try embedding the nuts into the print itself - so I'll pause the print at the right layer and then insert the nut and, hopefully, the print will resume.

\n\n

I'm wondering is it worth also including washers? It's a major pain because, while a nut can be captive in a hex space, a washer is circular. So it needs to be glued down on insertion in case it rotates while the printer is printing on top of it.

\n\n

For a 2kg load I don't believe the nut will attempt to pull through the part but there isn't a whole lot of support between the bore hole and the edge of an M2.5 nut though, only about 1mm.

\n\n

Edit: I'll add an image of my initial design. The strain gauge is one of a set pulled out of a weighing scales (from Lidl, \u20ac5.99. You can buy each separately on the internet for 20 euro, so I saved 74 euro!). The gauge itself is mounted on the narrow \"neck\" of the gauge. The whole block will drop into a holder that grabs the outer edge of the gauge.

\n\n

\"enter

\n", "Title": "Embedding nuts and washers into print", "Tags": "|abs|", "Answer": "

I've had good luck with just leaving a hex-shaped hole in the print, and press-fitting in the nut afterwards. You'll want the axis of the bolt to be along the z-axis, and to use a fair amount of infill and shells. I usually use PLA, so YMMV slightly.

\n\n

You may also want to consider your strain gauge setup carefully. If you're doing something like making a hook out of the ABS and the plate is acting as a lever arm, you need to make sure the lever arm length is the same every time, or your calibration will be wrong.

\n" }, { "Id": "3887", "CreationDate": "2017-04-19T13:36:54.377", "Body": "

I had this idea for bulky parts for just printing the shell (PLA) then pouring in some kind of filler in to make up the bulk/strength.

\n\n

Printing bulk .2mm at a time line by line is slow and subject to warping!

\n\n

So I though precision print a shell and fill it with 'something' - has this been done by anyone? What is a good something to use?

\n\n

Yours hopefully!

\n", "Title": "Printing shells and pouring filler", "Tags": "|filament|quality|warping|", "Answer": "

I've looked into doing something similar to this before and love the idea, never had the chance to follow through on it yet. This is sort of a hybrid method between cast molding and 3D printing.

\n\n

The accepted answer to a question I had a while back had some very good points by fred_dot_u

\n\n

Post processing FDM for strengrh

\n\n

Short version, Epoxy is a good option but you may have to consider heat generated from it. Urethane is another really good option (cast urethane is a pretty standard process). Chem-Eng isn't my area of expertise but there is a huge range of material options out there that can be mixed as two parts. I think there are enough options out there right now that you can choose your material properties you need and then select the material from there.

\n\n

You could also consider going the chopped-fiber composite route. (carbon, glass, etc) and then combine with whatever the appropriate resin is for those materials.

\n\n

I see the most difficult part of this is getting the shells to print properly. When I had looked into doing this, I considered modeling my part then hollowing it out completely. Then going back into the hollow part and designing in minimal internal structures for the purpose of supporting the thin-walled shell model. Printing that, and then drilling and filling the part after the fact. This approach I see as being a good option however the location of the drill points would be critical otherwise you could get voids as your fill material is injected in. And, the additional modeling time wouldn't be insignificant, however the saving I expected would come from having a ridiculously strong part, with complex geometry and be significantly cheaper than even a cast-urethane part.

\n\n

If you get some good results, please post a link to them! This is a huge interest of mine!

\n" }, { "Id": "3889", "CreationDate": "2017-04-19T15:29:40.200", "Body": "

So when my Prusa I3 is heating up, a small bit of filament is coming out of the nozzle, forming a small string at.

\n\n

When the printing starts, this results in a small plastic ball on the bed, impacting the overall printing process.

\n\n

How can I avoid this?

\n", "Title": "How can I avoid that a small bit of filament sticks out of the nozzle during heating?", "Tags": "|filament|prusa-i3|", "Answer": "

I just go and pull it off the print head with pliers when it's about to start and sometimes shortly after it starts printing, but its probably not the safest way to deal with the problem. Printing a skirt, brim, or raft should help to fix the problem.

\n" }, { "Id": "3900", "CreationDate": "2017-04-21T14:31:08.510", "Body": "

I changed my original HICTOP controller for one which actually has some fuses.

\n\n

I tried to print a part last night - the resultant print looks significantly bigger.

\n\n

Do you need to recalibrate the stepper per unit after you've changed your controller?

\n", "Title": "Parts printing out bigger after changing controller", "Tags": "|diy-3d-printer|", "Answer": "

A quick search for your specific controller brought up the following link:\nhictop wiki page\nin which a reference is made that some printers were shipped with incorrect firmware, specifically in error by a factor of six. If your prints are about six times larger, this may apply to you. The same page contains a link to the firmware.

\n\n

You should also be able to \"look inside\" the firmware using any number of programs providing terminal access to the controller. Pronterface is one, Simplify3D (not free) is another. I'm fond of using OctoPrint, but it requires some \"gymnastics\" to install.

\n\n

Once you've installed your selected software, open the terminal feature and type M503 to display the current settings. You may have to turn off some automatic terminal display features, as some controllers will repeatedly send data to the screen, causing inconvenient scrolling.

\n\n

I attempted to locate the correct code sequence for your printer, with limited success. Consider to scan over the wiki page for the appropriate information and correlate it to your specific controller and printer:\nFirmware reprap codes

\n\n

You'd want to confirm a specific M-code to change the stepper figure to match that which is for your printer. I've done my extruder steppers recently, which is M92 for my controller. My quick scan of the codes in the above link shows M350 might be the one for you, but don't take my word on it, get a confirmation from another source specific to your printer.

\n" }, { "Id": "3904", "CreationDate": "2017-04-21T21:01:33.690", "Body": "

What are the \"magic numbers\" people refer to regarding print resolution on the Monoprice Select Mini?

\n", "Title": "What are the \"magic numbers\" on a Monoprice Select Mini?", "Tags": "|monoprice-select-mini|", "Answer": "

This is an issue with homing. It is not synced to the stepper motor so it may occur on something other than a full or half or even quarter step. That said, it always does this so moving up from there, the first little bit may be off but I would expect once the motor runs freely, the error will go away. Also keep in mind with microstepping, you can have 1/16 or 1/32 steps as well. Using the magic number ensures the new position you request is within the limits of the stepper/controller to reach. Its like rounding numbers. if you don't, as you increment, the numbers will always be consistant. If you round, the numbers will dither around.\nAt least thats my $45 (inflation) and stickin to it.\nGood luck everyone and have fun.

\n" }, { "Id": "3909", "CreationDate": "2017-04-23T17:10:04.950", "Body": "

I've done some research on this matter however I cannot find any clear answers.

\n\n

How does a SolidWorks CAD file get 'converted' into a file format suitable for 3D printing, in detail?

\n", "Title": "How does a SolidWorks file get converted to be 3D printed?", "Tags": "|print-quality|file-formats|", "Answer": "

Any and all CAD files can be converted to STL (Standard Triangle Language). In this file format, surfaces are parsed to be combinations of triangles. For example a flat rectangular surface gets represented as two triangles in the same plane.

\n\n

As the complexity of the solid increases more triangles are required to adequately approximate the surface. Flat surfaced models are simpler to convert and will have smaller file sizes whereas curved solids will be more complex and larger file sizes (for the most part). The dimensions of your part do not translate to the STL file technically. The STL version of your part is based on unit vectors and a scaling factor is included in the file. So none of the vertices of the STL file are dimensioned, but the file is then scaled appropriately when opened.

\n\n

If you think of something as simple as a large diameter cylinder, you would need a relatively large number of triangles to approximate the curved surface. With a low number of triangles the curved surface would come out quite blocky, however increasing the resolution and the number of triangles used the blocky surfaces begin to approximate a curved surface. In theory if you had infinite resolution your curved surface would be exactly represented. The resolution of the file is something that you can choose when you save the file in STL format. The higher the resolution the more triangles will be required and the file size will also increase.

\n\n

The specific algorithm on how to complete this task will be proprietary to each CAD software, however if you're just curious on the math involved or the general process of converting them, I did a quick google search for \"STL format algorithm\" and found several useful links on the first page that could be useful.

\n\n

Here are a few links to a few sites with good information:

\n\n\n" }, { "Id": "3910", "CreationDate": "2017-04-23T17:30:35.137", "Body": "

I'm having an issue where the first layer of my support structures isn't sticking on the edges and causing the print to (eventually) fail. Any ideas on how I can fix it?

\n\n

I'm using Slic3r.

\n\n

\"Photograph

\n", "Title": "Having trouble getting first layer support to stick", "Tags": "|slic3r|support-structures|", "Answer": "

I had a similar issue where slic3r generated supports that attach directly to the bed, but didn't put enough material around the thin supports on the first layer, so they didn't stick to the bed.

\n\n

To solve this I added a brim, see \"skirt and brim\" under \"print settings\". This puts more material around both the part and also the supports on the first layer.

\n\n

I didn't need the brim round the whole part as it was sticking to the bed just fine anyway, but this was the only way I could find to give the thin supports the wide base they need to stick to the bed. I used a 5mm brim, but I am sure it would work with less than this.

\n" }, { "Id": "3913", "CreationDate": "2017-04-24T07:32:44.170", "Body": "

I've been using a 3D printer - Makerbot Replicator 2, since almost a year now. I'm quite good at operating and solving somewhat difficult problems with it. \nHowever I'm looking forward to build a 3D printer of my own, during the Summer break of 40 days. \nWhere's the best way to look for a start (except for Instructables) and build it in my given time? \nAnd how should I proceed? \nAlso what will be its approximate cost?

\n\n

PS: I don't know coding but I'm a really proficient designer and a good knowledge of electronics too.

\n", "Title": "Best way to start building a 3D printer of my own", "Tags": "|diy-3d-printer|", "Answer": "

Building a printer isn't that difficult due to RepRap (meant for rapid prototyping) but there are many factors that decide the price, difficulty, and abilities of a custom build.

\n\n

Using a Melzi board (a glorified Arduino Mega with motor drivers) helps a great deal and Repetier firmware is built to integrate many different types of hardware so those take a lot out of the engineering part. While you might not have to learn C++ to configure the firmware, it helps a great deal. There are plenty of sites that walk you through everything.

\n\n

After that, lead screws, GT2 belts, steppers, sensors, extruder, wires and the frame can be found very reliably from dozens of sites online.

\n\n

Cartesian style printers are much easier to build than delta type printers. There are countless numbers of custom builds that people document and put online. Check out as many as you want and draw ideas and inspiration from what they do. Listen for problems and solutions.

\n\n

And one of the most important things: there is a difference between cheap and inexpensive. Make sure you understand that. You don't want to burn your house down just to save a few pennies.

\n" }, { "Id": "3915", "CreationDate": "2017-04-24T12:35:13.747", "Body": "

I have an MK8Makerbot(?) extruder on my HICTOP branded printer.

\n\n

The problem with the E3D hotend is that the heatbreak is shorter and doesn't protrude through the top of the heatsink. Therefore I can't mount it to the motor mount.

\n\n

\"Photograph

\n\n

Any ideas?

\n", "Title": "E3D-v6 hotend on MK8 extruder", "Tags": "|e3d-v6|makerbot-mk8|", "Answer": "

For reference, I have the same printer and am speaking from my experiences.

\n\n

You have two routes you can take with this, but both are effectively the same result - you need to replace the X carriage.

\n\n

If you want to re-use your existing extruder components you can pull the MK8 extruder off the X carriage and use it as the extruder to drive a bowden configuration, then all you need is a length of PTFE tube and the couplers between the E3d and the MK8 to hold the tube. This is probably the best solution as it takes a lot of weight off of the X-carriage resulting in significantly better print quality, the only downside is that printing with flexible filaments is a lot more difficult with a bowden configuration, if you intended to do a lot of that.

\n\n

The other option (and the one I took) is to print an entire new x-carriage/extruder assembly. Because the Anet A8 is based off the Prusa and the Z/X carriage assemblies are basically the same as the original Prusa I3 you can just take any design for the Prusa and fit it onto the Anet, I did this with a gregs wade extruder & E3D hotend mount I found on thingiverse.

\n\n

I didnt look too hard, but if you are adament on wanting to keep the MK8 extruder on the X-carriage and couple it direct-driven to the V6, you can probably find some x-carriage that lines the two up for this purpose (or you could even design one yourself if you are interested in doing so). I didn't look in to this route myself, so cannot give much guidance.

\n" }, { "Id": "3955", "CreationDate": "2017-04-27T10:03:51.680", "Body": "

I have a 3D printed object that I'd like to print on. Adhesive stickers are an option, but as the surface is rounded it's difficult to get a good film to stick well.

\n\n

Is there any system to print on a 3D object (e.g. ink jet). I need at least 300dpi. Black in the first instance, but color would be nice for future projects.

\n", "Title": "Is there a way to print on a 3D printed surface?", "Tags": "|surface|color|graphics|", "Answer": "

One option is to hire a painter to paint the image onto the surface.\nWith a skilled painter, you could have a very photorealistic finish. Not sure if this would cost more or less than other routes.

\n" }, { "Id": "3965", "CreationDate": "2017-04-28T18:30:24.857", "Body": "

TL;DR

\n

How do I upscale a Wilson II? What lengths of aluminium1 do I need in order to achieve a particular (increased/reduced) build volume?

\n
\n

The design of the Wilson II is scalable (source: RepRapWiki - Category:Wilson):

\n
\n

...the design has a parametric build area, meaning it is relatively easy to scale the X, Y, and Z axis within reason.

\n
\n

What does parametric mean exactly, in this scenario? How does one scale up from 200x300x2001? Also, how would that affect the Repetier/Marlin firmware?

\n

Is it simply a matter of maintaining the ratios of the lengths of the X, Y and Z axes constant, or can the ratios be ignored? Is there a formula, or set of formulae, for this?

\n

Has anyone gone beyond the 200x300x200 build volume? I have seen the Scalar M and XL series printers (with the XL having a print volume of 400x300x300) which, while they are not based on the Wilson, also boast of scalability:

\n
\n

Scalar Family 3D printers are "scalable" printers.\nReviewing the idea of a reprap printer, a printer that can auto replicate and scale, we wanted to propose a 3D printer with plastic parts for you to print, and with a way to "scale" easily.

\n
\n

Can one (within reason) arbitrarily section various (supersized) length for the three axes and then modify the firmware accordingly, or is there a set of rules which govern the relationship between the lengths of the three axes?

\n
\n

A simplistic view

\n

As an example, the lengths (in mm) of the 2020 aluminium corresponding to the build volume of 200x300x200 are 330, 500, 400 for the X, Y, and Z axes, respectively.

\n

Obviously, there are some constants to consider for the stepper housings, and idlers, for example. So, assuming that for X, Y and Z axes respectively, the constants are:

\n\n

If I wanted a build volume of, let's say, 400x500x300 (XYZ), would the new XYZ lengths of 2020 aluminium simply become (by adding the respective constants):

\n\n

or is there more to it than that?

\n
\n

1 I appreciate that 3030, en lieu of 2020, extruded aluminium may be required to maintain rigidity for larger print volumes

\n", "Title": "For a larger build volume, what lengths of 2020 aluminium do I need?", "Tags": "|printer-building|diy-3d-printer|wilson|", "Answer": "

Long story short: ratios can be ignored. You only need to let your firmware know actual size, if it needs so. However, using golden ratio seems quite reasonable.

\n\n

When talking about rigidity, keeping it will become a real issue at some point and may not be always effectively resolved by merely changing 2020 to 3030. Using 4020 or double 2020 could be better solution to compensate the most common forces during printing.

\n" }, { "Id": "3968", "CreationDate": "2017-04-30T13:12:33.240", "Body": "

TL;DR

\n

Is ABS a better material to use for structural parts of a 3D printer, as opposed to PLA?

\n
\n

I have been looking at various suppliers of printed parts for a Wilson II, on eBay.

\n

One supplier supplies all of the parts printed in ABS, whereas another supplier provides the structural parts, which form part of the frame, in PLA, and the remaining (non-structural) parts in ABS. The second supplier states the following:

\n
\n

This kit is printed in two materials, the structural parts are printed on high-quality PLA to provide the necessary rigidity, and the others in premium ABS.

\n
\n

MJRice, who developed the printer, also supplies the printed parts, which are made of PLA.

\n

Is PLA really a better choice for structural components? A quick google lead me to ABS or PLA: Which 3D printing filament should you use?

\n
\n

ABS is going to give your projects better structural integrity and will be more suited to mechanical use given the material can better withstand the elements, but it will also require specific types of printers and printing surfaces. On the flip side, PLA will give you more precise prints and better aesthetic quality, as well as more flexibility with printing conditions if you can do without the strength and resilience of ABS.

\n
\n

Another link, What's stronger? PLA or ABS?, also implies that ABS is stronger than PLA:

\n
\n

The strongest ABS is stronger than the strongest PLA

\n
\n

and

\n
\n

ABS is much less brittle than PLA

\n
\n

although warpage could be an issue...

\n
\n

I choose ABS for it's strength over PLA. I would love to avoid the warpage/shrinkage problems of ABS.

\n
\n

Heat seems to affect PLA more than ABS (obviously, due to the lower melting point),

\n
\n

FWIW, I know a guy who had a reprap using PLA-printed parts. He was taking his machine around to do demos and he left it in the car mid-day once. It melted the parts enough that prints were coming out very poorly even after his best effort at recalibrating the machine.

\n
\n

and as I am in Thailand, my room gets up to 50\u00b0C some days (it is unbearable), I wonder if that would be cause for concern?

\n

So, from the above nuggets of information, wouldn't ABS be a better choice? I am surely missing something, as both the second supplier, and MJRice, use PLA.

\n", "Title": "ABS or PLA for structural printed parts?", "Tags": "|pla|abs|printer-building|diy-3d-printer|", "Answer": "

Actually, Both PLA and ABS could use for structural parts. PLA has high strength and is brittleness. ABS has better thermal resistance and durable. PLA and ABS are the main types of 3D printing material, each has cons and pros.

\n

If you want to choose which type, you need to know the detailed application.

\n" }, { "Id": "3972", "CreationDate": "2017-05-01T10:57:16.473", "Body": "

After I updated the firmware on my Prusa i3, the Bed won't switch off anymore. It worked perfectly before the update, but now, the moment I power up my printer, the LED on the bed turns on and it starts heating up. The manual control in Repetier Host doesn't turn if off or on and I even tried g-codes M140 S0 as well as M0, but it does not switch it off.

\n\n

Edit:\nI have an Arduino Mega2560 with a RAMPS shield. The Marlin firmware came pre-configured from the store I bought the kit from.

\n", "Title": "Bed won't switch off after Marlin firmware update", "Tags": "|prusa-i3|heated-bed|marlin|", "Answer": "

This was actually a coincidental situation where I somehow damaged the RAMPS board around the same time I reloaded the firmware. Replaced the board and it works correctly now

\n" }, { "Id": "3983", "CreationDate": "2017-05-03T14:23:30.573", "Body": "

So here How to achieve dimensional accuracy of printed parts the Cartesian printer approach to accurate sizing is covered. And deltas get a footnote of being a whole other ball of wax - so let's look at that wax ball.

\n\n

I'm currently resurrecting, or erecting, depending how one looks at it, a poorly documented Chinese printer (purchased in China by a Chinese student and abandoned in the US after graduation) that appears to be based off a Rostock mini. Z 204mm, X Y 100mm radius or 200 mm diameter, Bowden tube.

\n\n

\"enter

\n\n

It appears to be equipped with an absurdly small nozzle (lacking a good way to check that precisely yet, feeding suggests possibly 0.1mm, and no, the vertical resolution is not nearly enough to make that in any way reasonable), which is obviously fixable if the rest of it can be made to work.

\n\n

Yesterday we got it to the point of (very tediously with tiny nozzle) spitting out a test cube, which was 17 by 17 by 20 mm - undersized in XY, accurate in Z. The last bit makes me pretty sure the steps/mm are right on the steppers, but obviously something is off in the geometry (measured, no documentation for this exact printer can be found, at least by non-Chinese-speaking/reading me - it being utterly un-branded does not help.)

\n\n

A second test cube was produced with scaling set to 20/17 (1.176) in the XY directions, and that seems to be accurate at least to non-precision measurements. I'm now contemplating \"what likely needs tweaked, and in what direction\" for the delta geometry - I'd say the rod eye-to-eye measurement (85mm, IIRC) is fairly decent, the offset from rod mount at carriage to rod center is not too terrible, and I have low confidence in the rod mount at printhead to nozzle - so that's the one I suspect most.

\n\n

tl, dr:But I lack an intuitive understanding of how each of those parameters (offset at carriage, offset at printhead, and arm length) would affect the printed size. I'll come back and edit in what we currently think each is. Rather than depending on scaling in the slicer I'd like to seek the \"right numbers\" but when at the limits of available measurement precision, having an idea how and in what direction incorrect offsets or arm length affect the print size would be good, rather than just blindly changing numbers and hoping.

\n\n

I also have some other \"print quality\" issues and a bed leveling issue which will be other questions when I can sort that out.

\n", "Title": "Scaling/size issues in delta printers", "Tags": "|delta|", "Answer": "

Issues in X-Y size on a Delta are usually the result of an incorrect diagonal rod value in the firmware. This should be easy to fix assuming the rods were built together.

\n\n

The formula for this is (20 in this case is your test print X-Y):

\n\n

New DELTA_DIAGONAL_ROD = 20 / measured_length * Original DELTA_DIAGONAL_ROD

\n\n

I found this information (here), this site also has more information on setting the radius and other delta specific issues.

\n\n

If you encounter issues with the size of the rods you should build new ones using a jig. There are several (here's one) I found on Thingiverse but they require printed parts and they do all the rods horizontally which I think would be tough to keep equal. I would use something similar to this picture but with a longer rod/bolt so you can fit all rods on the same time. When I did my Kossel Mini I used a piece of the extrusion with bolts coming off of it to keep the rods consistent.

\n\n

\"enter

\n\n

For the other issues you mention you should open another question specific to each issue.

\n" }, { "Id": "3985", "CreationDate": "2017-05-03T23:03:52.200", "Body": "

I've been using a printer for several years now and I've never replaced any of my extruder hardware (nozzles, steppers, gears etc.). I was wondering how long you should expect your extruder hardware to last? Is it indefinite, or are there normal life expectancies on them?

\n\n

Also, what signs should you look for, in your prints, that would suggest that your nozzle is starting to wear out?

\n\n

I'm more interested in the parts that contact your filament directly - I'm painfully familiar with replacing gantry parts (X-Y belts and pulleys).

\n", "Title": "How long is an extruder's/nozzle's life?", "Tags": "|hardware|replacement-parts|life-expectancy|", "Answer": "

This is highly dependent upon the type and quality of filament you use.

\n\n

Exotic materials such as wood, metalic, or otherwise infused plastics can potentially wear out the inside of the nozzle and at the very least leave deposits of composite material in the nozzle. This can lead to more frequent or irreversible clogs. You may also notice your prints becoming \"stringy-er\" (sorry for lack of vocabulary at the moment) as the volume of your nozzle reduces with more deposits sitting in the basin of the nozzle, therefore leading to lighter extrusions against what your slicer is expecting.

\n\n

Lower quality materials (common ABS/PLA) can also have an effect on your nozzles life span. So can good quality filament, don't get me wrong. However, lower quality materials have a tendency to go through much more drastic material changes during the printing process than better quality filaments. For instance, a lower quality PLA may not be 100% PLA. It could have 10% Nylon, 5% ABS, 0.1% operator's BandAid, and 83% PLA. Well, the 17% of the filament that isn't PLA has the potential to clog the nozzle just as easily as the exotic filaments. This is why it's so important to verify the quality of your source of filaments.

\n\n

Some signs that your nozzle may be wearing:

\n\n\n" }, { "Id": "3986", "CreationDate": "2017-05-03T23:59:04.813", "Body": "

I'm building a new printer after having an Anet A8 for about two months and getting a feel for what I want. I'm planning to buy a P3Steel frame; but can't decide which version to get.

\n\n

There are one or two prints I would like to make that are larger than 20 cm in length; but I could always do these in multiple pieces.

\n\n

What I'm wondering is how serious are the trade-offs I would be making for a larger bed? I would think I would need thicker linear rods for the 510 mm length y-axis; and the bed would also have a large amount of inertia so that could slow down printing. Does anyone have any experience with this size bed in a moving y-carriage scenario? Also how thick of rods should I get? Orballo printing is saying 8 mm is fine but the research I've done implies that you shouldn't really go over 200-250 mm unsupported for that width.

\n", "Title": "P3Steel v4 w/ 20x30 cm bed, or 2.5.1 w/ 20x20", "Tags": "|prusa-i3|mechanics|p3steel|", "Answer": "

Go for the P3Steel v4 (20x30). The extra print area is worth it. There is a Polish supplier, Printo3D, on eBay that has the cheapest frame, and parts - cheaper than the Spanish supplier. That is where I got mine from. See Frame Prusa I3 P3Steel v 4.0, 300mm x 200 mm, which costs around \u00a380.

\n

This kit uses 10\u00a0mm smooth rods for the Y axis:

\n
\n

Smooth stainless steel rods:

\n\n

Threaded stainless steel rods:

\n\n
\n

With respect to the Y-axis carriage, the steel carriage does add a lot of weight/inertia, you are correct. This may or may not be an issue, depending on your steppers motors that you choose1, and their torque. That said, the 3\u00a0mm steel print bed/Y-axis carriage, is ridiculously heavy, and it would be most wise to substitute it for an aluminium, plywood, or some other lightweight solution.

\n

Apart from that the 3\u00a0mm steel frame is fine and as solid as a rock. There are a number of aluminium 20x30 print beds/Y-axis carriages available on eBay and Amazon. A thorough search should reveal a few. There are also composite Y-axis carriages, I found a supplier in the Ukraine, tehnologika_net, who, last year, had a number of different types at a reasonable cost - in fact they were the cheapest that I found.

\n

As an aside, I built mine sourcing all of the parts separately. It was a bit of a task, but an educative one. The process certainly made me understand the ins and outs a lot better than purchasing a ready built, or complete kit, 3D printer.

\n

I have written up some blogs regarding the kit that I purchased, see P3Steel from Poland \u2013 A tale of despair, dismay and woe. Ignore the depressing title, it really isn't that bad.

\n

See also Heatbeds. At the bottom there are some links to various alternative Y-axis carriages. However, some of the links/items may no longer be available.

\n

You may also find this question of mine useful, Z axis top brackets, of P3Steel, differ between v1.x/2.x and v4.

\n
\n

There are a number of modifications to the standard P3Steel, that may well be worth considering. In particular, you should note the Toolsen Edition MK2, see P3steel toolson edition MK2 (in German), and P3steel - toolson edition. I have written about these, and more, see P3Steel version 4 modifications. In summary, these are:

\n\n
\n

1 I got the Rattm 17HS8401 steppers. See RepRapWiki - Nema17. The recommended steppers are high torque:

\n\n
\n

However, motors close to NEMA 17 size, with approximately the\nfollowing specifications, can also work:

\n\n
\n" }, { "Id": "3989", "CreationDate": "2017-05-04T07:17:47.683", "Body": "

I recently ordered some spare E3D 0.4 mm nozzles. However, there was a mix up at the factory and they delivered 0.2 mm nozzles instead.

\n\n

I usually print at 0.3 mm using a 0.4 mm nozzle. Can I still use the 0.2 mm nozzles to print at a resolution of 0.3 mm?

\n", "Title": "Can you print at 0.3 mm with a 0.2 mm nozzle?", "Tags": "|layer-height|", "Answer": "

To get the best results, the plastic coming out of the nozzle needs to be squished/ironed down by the nozzle. If you are using a higher layer height than nozzle size, this does not happen. Instead, you are taking a 0.2mm diameter string of plastic and folding it back onto itself to create a thicker 0.3mm bead.

\n\n

Triffid_Hunter's calibration guide recommends using a layer height that is not higher than 80% of your nozzle size, so with a 0.4mm nozzle you should not print layers thicker than 0.32mm, and with a 0.2mm nozzle you should not print layers thicker than 0.16mm.

\n\n

While it is not impossible to print thicker layers with a smaller nozzle, the results won't be as good. Moreover, if you are used to working with a 0.4mm nozzle then swapping to a 0.2mm nozzle may take some getting used to, as printing with smaller nozzles is more difficult (for example, the smaller the nozzle, the higher the likelihood of clogs).

\n" }, { "Id": "3994", "CreationDate": "2017-05-04T18:18:25.673", "Body": "

I have recently purchased an Anet A8 but have been wondering why the prints look squished and have tiny balls on them.

\n\n

I am using 1.75 mm cheap PLA bought from eBay and have also had problems of filament oozing out of the print block.

\n\n

\"Photos]1

\n", "Title": "My Anet A8 prints have been looking squished", "Tags": "|filament|anet-a8|", "Answer": "

First, I would check for bed adhesion issues. I would then check the hot-end, when it is warmed up, for any loose parts. I have fixed such issues with a piece of PTFE tape.

\n\n

I would also check the filament cooler parts, or proximity sensor (if you have one), for possible drag on the printed sections during printing.

\n" }, { "Id": "3996", "CreationDate": "2017-05-05T01:36:50.657", "Body": "

As mentioned in my previous question, I have some bed-levelling issues with my printer. Following the link in @tjb1's answer I figured they were all about to be solved - just follow the steps and bim, bam, flat printing aligned with the plane of the bed. I was wrong. Firmware is Marlin 1.1.0-RC8

\n\n

From minow.blogspot.com:

\n\n
\n

For Marlin in Marlin.ino.Marlin.pde

\n \n

DELTA_RADIUS is DELTA_SMOOTH_ROD_OFFSET - DELTA_EFFECTOR_OFFSET\n -DELTA_CARRIAGE_OFFSET\n To correct the problem, the best course is to change one of the variables set earlier to force the value of DELTA_RADIUS to\n increase(to lower the extruder nozzle) or to decrease (to raise the\n nozzle).

\n \n

To lower the extruder nozzle increase DELTA_RADIUS by increasing\n DELTA_SMOOTH_ROD_OFFSET(Marlin) or PRINTER_RADIUS(Repetier).

\n \n

To raise the extruder nozzle, decrease DELTA_RADIUS by decreasing \n DELTA_SMOOTH_ROD_OFFSET(Marlin) or PRINTER_RADIUS(Repetier). For your\n first cycle, if you are above the build surface, increase DELTA_RADIUS\n by (about) the same amount. If the extruder nozzle hits the build\n surface, decrease DELTA_RADIUS.

\n \n

Next, repeat the ABC tower calibration process. Changing the\n DELTA_RADIUS changes the tower calibrations, so you must cycle through\n the process of adjusting the ABC buttons/scripts again. And when the\n ABC towers are set, test the center D button/script.

\n \n

You may have to repeat this a few times with smaller changes to\n DELTA_RADIUS, but eventually, the drag on a piece of paper should be\n the same at all four locations. Now the printer knows how to print\n flat at a given Z height.

\n
\n\n

So I did that. It's about 5-6mm high in the center of the bed. On point in front of the 3 towers. And it stays pretty much there (less than one 'precision' business card movement, if any) when DELTA_SMOOTH_ROD_OFFSET is increased, increased more, or decreased, for that matter - we started wondering if there was a sign error issue and tried the other way.

\n\n

There is mention on the page of \"turning off EEPROM\" lest values be over-ridden by those in EEPROM, but the text appears to suggest that it's a Repetier-firmware-specific problem. So I was not expecting to have it in Marlin. Does it also apply to Marlin, or:

\n\n

TL;DR mode: why would changes that should affect the \"DELTA_RADIUS\" to get the printer on plane with the bed be ineffective?

\n\n

\"delta]3

\n", "Title": "Delta printer not responding to changes in DELTA_RADIUS", "Tags": "|delta|", "Answer": "

OK, so a little more poking around finds that it does seem to be loading values from EEPROM and ignoring the configuration files. So that's not Repetier-firmware-specific.

\n\n

...and I found M665 in G-code which lets me just set a value for it, then M500 saves it to said EEPROM. And it's making a difference (so much of one that I may need to reset Zmax before I can actually adjust it all out as the endpoints on the bed are now off, but they are all 4 much closer to being off by the same amount!)

\n" }, { "Id": "4000", "CreationDate": "2017-05-05T11:53:33.177", "Body": "

I\u00b4m currently working on a custom delta printer running Marlin 1.0.2 firmware. To control it I\u00b4m using the Repetier Host software with the onboard Cura software to slice my objects. The printing works fine but it takes the nozzle nearly 5 minutes to move from the homing position at 250 mm to the starting position of the print near z=0.

\n\n

I already tried to increase the travel speed but nothing changed. Can you guys please tell me how to increase the speed of my nozzle to move down to my starting point of the print?

\n", "Title": "Nozzle moves very slowly from home to start position", "Tags": "|marlin|ultimaker-cura|delta|speed|", "Answer": "
    \n
  1. I would guess (without G code) to change start G code. You should find it in Cura, the feed rate should be low there.
  2. \n
  3. Try other slicer and print from SD card.
  4. \n
  5. Try using other software. Try Cura it self if SD card printing is not available.
  6. \n
\n\n

Picture is just an example what to look in Repertier or Cura.\n\"enter

\n" }, { "Id": "4010", "CreationDate": "2017-05-07T20:15:46.773", "Body": "

Background: I have many years of experience with AVR and ARM Cortex PLCs and I feel very comfortable using them in projects but I am not an EE or SE. So, if I make a mistake or misunderstand something, keep that in mind. Thanks.

\n\n

I cracked open the case on my Maker Select v2.1 and I noticed that I have a 2x3 header that is labeled ISP. This is fairly standard for the AVR PLCs and the Melzi board that I have uses the ATMEGA1284P.

\n\n

I have asked at numerous forums and nobody seems to really know the answer (the downside to RepRap--people use but don't fully understand).

\n\n

I'm wondering if I can tweak the Repetier firmware (I'm using the stock version 0.91) to use the ISP headers as a standard SPI bus. It has the SCK, MOSI, and MISO pins but it doesn't have the SS pin.

\n\n

What I want to do is put a port expander on the SPI bus and break out additional usable GPIOs.

\n\n

So, is there a hardware limitation or any other reason why I wouldn't be able to repurpose the ISP headers into an SPI bus?

\n", "Title": "Can I repurpose the ISP pins in the lower right corner of the Melzi board?", "Tags": "|reprap|repetier|", "Answer": "

In theory, you can; but, you may need those pins to attach an external programmer to bring your system back to life when playing around with the code.

\n\n

Here are a couple of articles that could help if you still want to pursue that path:

\n\n\n\n

Another option (the one I chose) is to just buy a RAMPS board set. I got one on ebay for $19 and it have lots of more options for IO. That way you can play and still go back and plug in your stock Melzi and print whenever you need it.

\n\n

I got all this for $40 - boards, display, cables, power supply, and even shipping\n\"enter

\n\n

Here is a really nice detailed description of converting a Duplicator i3 from Melzi to RAMPS. The process would likely be very similar for your printer.

\n\n

The biggest challenge will likely be setting up the firmware

\n\n

BTW, what printer did you get?

\n" }, { "Id": "4012", "CreationDate": "2017-05-08T14:28:36.757", "Body": "

TL;DR - For a given Kossel frame size (w.r.t. the vertical and horizontal frame lengths of the aluminium extrusion), what would the length of the carbon fibre rods be?

\n\n
\n\n

A case in point, from RepRapWiki - Kossel, there is an intriguing note about a scaled down Kossel:

\n\n
\n

Optionally scale down to a Traveling RepRap that fits within IATA hand\n luggage size limit (see transportation):

\n \n \n
\n\n

However, there is no mention of the length of the carbon fibre rods (carbon tubes).

\n\n

Now, as per my previous question, For a larger build volume, what lengths of 2020 aluminium do I need?, is there a formula or ratio by which one needs to abide? Whereas in my previous question, the answer was along the lines of: Not really, you can use any lengths, within reason, and account for it later in the firmware, I would imagine that the Delta aspect of the printer is somewhat more exacting.

\n\n

I have tried googling for further information on this Travelling Kossel, but found nothing, except for the information of RepRapWiki.

\n\n

Looking at the corresponding lengths (vertical/horizontal) of the aluminium versus those of the carbon fibre rods for the Mini and XL:

\n\n\n\n

I really can not see what the (trigonometric) relationship is, and therefore can not deduce the lengths of the carbon rods for the Travelling Kossel.

\n\n

Unless it is simply that the carbon rods are 60 mm shorter than the aluminium horizontals? Is it really as simple as that, or is this just a coincidence? In which case, would the carbon rods be (210 - 60 =) 150 mm?

\n\n

By extension, imagine if you wanted to build a Kossel XXXL, with a horizontal aluminium extrusion length of, let's say, 1000 mm, would the length of the carbon rods be 940 mm?

\n\n

Any ideas?

\n", "Title": "How long are the carbon fibre rods for a Travelling Kossel?", "Tags": "|printer-building|diy-3d-printer|kossel|", "Answer": "

To compliment tjb1's answer, The link seems to go to a spreadsheet that doesn't have any formulae in it. However, I managed to find a copy, from Google groups: How to calculate Delta Dimensions for new build.

\n

However, another source of the Document is Kossel frame calculator (delt...@googlegroups.com). Right at the top is a posting of Johan's (who is the designer of the Kossel) spreadsheet, which is essentially the same as before, but it is contained within an interesting message thread.

\n

There are a few more calculators, that a quick google search will throw up, which I have listed in a short blog, Kossel - The Ratio:

\n\n

I have an enhanced version that allows additional parameters to be modified, see Github: Greenonline/Kossel/Spreadsheets.

\n
\n

Design Process by David Crocker

\n

Apparently, the ratio is not particularly vital. I found a design process in the comments section of More upgrades to the large Delta 3D printer:

\n
\n

The design process goes something like this:

\n
    \n
  1. Estimate the size of effector you will need.
  2. \n
  3. Given that effector size, work out how close to the towers the nozzle will be able to get. From that and the desired printing radius,\nwork out what radius the towers are on.
  4. \n
  5. That fixes the lengths of the horizontal extrusions. Choose the diagonal rod length so that when at the edge of the bed opposite a\ntower, the rods to that tower are at 20 degrees or a little more to\nthe horizontal.
  6. \n
  7. Given that diagonal rod length, choose the rod separation. I suggest about 1/6 of the rod length (this is larger than on my delta).
  8. \n
  9. If that rod separation means you need a larger effector, repeat from step 1.
  10. \n
  11. Choose the tower height to give the required print height at the edges of the bed, when one pair of rods may be more or less vertical.
  12. \n
\n
\n

When I queried David about his use of the 0.8 ratio, again in the comments section, on Building a large delta 3D printer, this was his reply:

\n
\n

The notion that there is a single ratio of diagonal rod to horizontal\nextrusion length that is right for all delta printers is misguided. It\ndepends on the geometry of the corners that join the horizontals to\nthe towers, the size of the effector, and the carriage design. What\nmatters is that the rods are at no less than 20 degrees (preferably 25\nor more) to the horizontal when the nozzle is at the edge of the bed\nopposite a tower.

\n
\n" }, { "Id": "4022", "CreationDate": "2017-05-09T14:06:03.460", "Body": "

I'm using M80 and M81 G-codes to power on/off power supply.

\n\n

Is there a G-code to know the actual state of the power supply?

\n", "Title": "Is there a G-code to get power supply state?", "Tags": "|marlin|reprap|g-code|switching-power-supply|", "Answer": "

Thanks! I confirmed that there isn't such a G-code.\nI sent a pull request to make this posible:\nhttps://github.com/MarlinFirmware/Marlin/pull/6671

\n\n

Now, \"M80 S\" reports the current state of the power supply.

\n" }, { "Id": "4026", "CreationDate": "2017-05-09T16:58:42.367", "Body": "

I have had many problems with my heat shrink for the thermostat on my Anet A8 melting from the heat block.

\n\n

Is there a way I can insulate my wires from heat but still have enough room to put the thermistor into the block?

\n", "Title": "How can I insulate my thermistor?", "Tags": "|heat-management|thermistor|", "Answer": "

Ok, so I think I have found the answer. You can get cotton wraps for print heads with some Kapton tape on them, see 10Pcs 3mm Thickness 3D Printer Heating Cotton Hotend Nozzle Heat Insulation. I think this will work best as long as it doesn't catch on fire.

\n\n

Please let me know if there is a better alternative but for now I will use these.

\n" }, { "Id": "4031", "CreationDate": "2017-05-10T09:31:33.463", "Body": "

By "better" I mean "more precise"...

\n

With respect to a RepRap P3Steel or Wilson II, I am getting some 330\u00a0mm T8 leadscrews for the Z-axis movement.

\n

There seems to be a choice between a lead of 1, 2 or 8\u00a0mm - the pitch is 1\u00a0mm in the first case and 2\u00a0mm in the last two cases1. However, there is precious little information about the advantages and disadvantages of each lead size in RepRapWiki - Threaded Rod - Leadscrew.

\n

It seems to me that using a leadscrew with a 1 or 2\u00a0mm lead could result in a more precise Z-axis movement, as one rotation of the stepper results in a smaller increment in height. Therefore the layer thicknesses could be smaller.

\n

However, is the minimum layer thickness not, also, dictated by the thickness of the filament, horizontal speed of the print head, nozzle size, etc.? Thus, at some point there would be no need for a super fine vertical resolution from the leadscrew, as it may be constrained by other limiting factors.

\n

Of course, conversely, the use of a 2\u00a0mm lead would mean that the stepper would need to "work" four times as hard, than when using 8\u00a0mm, in order to raise the print head the same distance, as well as making the movement, during a "home", take four times as long (or, if you will, four times slower). In the case of a 1\u00a0mm lead that would become eight times...

\n
\n

TL;DR

\n

Is it worth getting a leadscrew with a 2\u00a0mm lead, or is 8\u00a0mm sufficient?

\n

By extension, would a 1\u00a0mm lead be even better, or just overkill?

\n
\n

Footnote

\n

1 Nomenclature:

\n\n

So, for a leadscrew, with a pitch of 2 mm, if there is only one start to the screw then the lead is the same as the pitch. However, if there are four starts to the screw, then the lead will be 8 mm. If there are two starts to the screw, then the lead will be 4 mm. And so on.

\n

For more information, see Wikipedia - Lead, pitch and starts.

\n", "Title": "Would using a leadscrew with 1 or 2 mm lead, en lieu of 8 mm, result in a better printer?", "Tags": "|diy-3d-printer|printer-building|lead-screw|", "Answer": "

Longer lead

\n\n

Pros:

\n\n
    \n
  1. Faster movement (mostly benefits homing)
  2. \n
  3. More standard
  4. \n
  5. Weak motor friendly
  6. \n
  7. Might be slightly more stable (sideways) than shorter leads because of more starts creating more contact with nut (?)
  8. \n
\n\n

Cons

\n\n
    \n
  1. Backlash
  2. \n
  3. Less resolution (unless using 0.9 degree motor but see note1) although this is ignorable since it's still very high. It's already around 5x higher than xy axes using the longest leads2.
  4. \n
\n\n

Shorter leads

\n\n

Pros

\n\n
    \n
  1. More accuracy if you turn off microstepping. it can give you more resolution, but it's useless resolution unless you print with < 0.01 mm layer heights3. Turning off microstepping trades away this useless resolution for increased accuracy. But this comes at the cost of (much) louder drivers.
  2. \n
  3. Less backlash
  4. \n
\n\n

Cons

\n\n
    \n
  1. Stronger motors maybe needed
  2. \n
  3. Slower homing
  4. \n
\n\n

The reason why companies prefer longer leads is because shorter leads simply add useless resolution and slows down movement.

\n\n

Footnotes

\n\n

1 It makes more sense to use higher degree motors to drive shorter leads than vice versa because of higher torque.

\n\n

2 0.0025 mm vs 0.0125 mm

\n\n

3 or using a probe to autolevel, which causes the z axis to go up and down in very small increments

\n" }, { "Id": "4051", "CreationDate": "2017-05-13T01:29:36.787", "Body": "

I have an STL file that, when I load it in Slic3r, looks fine, but somewhere in the actual slicing process, something goes wrong and parts of the model disappear.

\n\n

\"Render

\n\n

That's the render on the left, and the slicing preview on the right. That big hole is the most obvious fault, but there are more little notches elsewhere on the model; you can see a couple on the bottom edge there. It looks like the program is ignoring some of the model's faces entirely. What's all that about?

\n\n

The model passes every manifoldiness test I've thrown at it: Slic3r's, Meshmixer's, Blender's, and a couple of online services I've forgotten.The walls that go missing are pretty thin, but they're at least a millimeter at their narrowest point, so I think it should be thick enough for the printer to handle.

\n\n

Here's a link to the STL file in question, on FileHosting.org, FlashHovering.stl.

\n", "Title": "Parts of my (apparently manifold) model aren't appearing in my Slic3r preview", "Tags": "|slicing|slic3r|", "Answer": "

I fixed it!

\n\n

I had my extruder diameter set to 0.5 mm, and the narrowest point of the model was just a hair under a full millimeter, so I guess the program took it to mean that it should only put down one line of material. I made the walls a little thicker and now it looks fine.

\n" }, { "Id": "4071", "CreationDate": "2017-05-16T04:02:28.803", "Body": "

I have a solid of revolution defined by two equations, and I want to generate a STL file for printing from the difference of the two equations, revolved around x=0. I can get a good visualization when I query this on Wolfram Alpha, but I cannot figure out how to download an STL of this. I know there is a way to do this via Wolfram's Development Program but I'm not sure how or if that is the best way to do this. Solutions do not have to involve WA.

\n", "Title": "How to generate a STL from a rotational solid of two equations?", "Tags": "|3d-models|", "Answer": "

The best free solution I could find was to graph this (in Desmos), screenshot it, and convert it into a SVG, convert that into an OpenSCAD file, and then use that to make the STL. A similar process is shown on someone's blog here.

\n" }, { "Id": "4076", "CreationDate": "2017-05-16T14:39:16.857", "Body": "

I'm trying to connect two 3D printed parts (ABS) together with a threaded connection. Thus, I need to fix a threaded metal nut (M4) inside a corresponding slot which I've implemented in the design.

\n\n

Right now I apply a conventional super glue on the nut and press it inside the print, wait for a couple of hours and then use it.

\n\n

Problem is that the nut keeps falling out of the print when I apply a more tension to the bolt. Can you please advise me on how to make this method work. Maybe some of you aware of special glue for that purposes?

\n", "Title": "Best practices to fix a threaded nut inside a print", "Tags": "|3d-models|filament|metal-parts|", "Answer": "

Instead of super glue, you could try a two part epoxy resin (any brand should do, i.e. Bison Kombi Power or JB Weld). This may take longer to dry, than the super glue, but should be much stronger, and deal with torsion forces better1.

\n\n

Or, you could try red (not blue) Loctite. However, the epoxy resin would probably be stronger.

\n\n
\n\n

1 This is admittedly an opinionated reference, but from Epoxy or super glue, which is stronger?

\n\n
\n

depending on the surface, [epoxy] can be superior. Epoxies are generally a must if the surfaces are porous

\n
\n\n

and

\n\n
\n

Epoxy is superior to CA. CA is nearly always misapplied. Epoxy has a higher tolerance to misapplication thus is much less likely to fail under such circumstances. I've used Belzona epoxies to repair parts on various aircraft and spacecraft where welding would be inappropriate! Belzona epoxy is without a doubt the best there is. Too bad you cannot purchase it in Home Depot.

\n \n

The closest thing you can get though is PC-7.

\n
\n" }, { "Id": "4078", "CreationDate": "2017-05-16T14:54:42.140", "Body": "

I'm now currently looking at getting my next 3D printer, after my success with the Prusa i3 Reprap Build, and I have had my eyes on the Da Vinci line of printers for some time now.

\n\n

What I want to know is, does this line of 3D printers include a heated bed? Or does it come with a stock aluminum bed?

\n", "Title": "Does the Da Vinci line of 3D printers come with a heated bed?", "Tags": "|heated-bed|hardware|", "Answer": "

Yes. The Da Vinci Duo 2 (most common one) has a heated bed.

\n" }, { "Id": "4084", "CreationDate": "2017-05-17T05:54:56.123", "Body": "

3D printing should be relatively safe, however, the inherent nature of 3D printers, with all of the heated parts, constitutes a fire risk. A well designed 3D printer should be designed to be as safe as possible, especially one used in the home... Yes, the recommendation is, when printing, to watch the 3D printer at all times and never leave a print unattended. However, with some print times lasting hours and days, this is not always feasible, nor practical. So, some inbuilt safety features should be included, to at least mitigate the risk of fire, to some extent.

\n\n\n\n

\"Common

\n\n

Has anyone added thermal fuses to their 3D printers? Or has anyone examined where the thermal fuses are placed in commercial 3D printer designs, if used at all?

\n\n
\n\n

Background

\n\n

I have recently found myself having to repair rice cookers and fans in Thailand. In those, it is very often the thermal fuse (axial thermal fuses for the rice cookers and the square \"radial\" types for fans) that requires replacing, as they have blown before the device got hot enough to start a fire. This got me thinking about their use in a 3D printer.

\n\n
\n\n

Footnotes

\n\n

1 We are not talking about the standard, replaceable, thermo-fuse,or fuse, which blow upon a current surge, short-circuit, etc. These are thermal fuses that contain metal connector within them that melts (permanently) at a specific temperature (typically ~135\u00b0C), thereby breaking the circuit.

\n\n

2 Nor am I referring to resettable fuses (AKA PPTC, multifuse, polyfuse or polyswitch)

\n\n

3 Would a thermal fuse be preferable to thermal cut offs, in the case of fire?

\n\n

4 The thermal fuses used in rice cookers are the axial type, and in the motors of fans are the radial type.

\n", "Title": "Is it a good idea to include thermal fuses in a DIY 3D printer design?", "Tags": "|printer-building|diy-3d-printer|", "Answer": "

I'm going to set this up for my printer: 3D Printer Safety Shutdown - Smoke Detector

\n\n
\n

\"3D

\n \n

\"3D

\n
\n\n

Looks like a great solution for preventing fires.

\n" }, { "Id": "4106", "CreationDate": "2017-05-19T14:16:04.457", "Body": "

The surfaces of my printed parts using ABS plastic look rough and uneven.

\n\n

What methods can I use to achieve a smoother finish for my for 3D-printed objects?

\n", "Title": "How do I give 3D-printed parts in ABS a shiny smooth finish?", "Tags": "|abs|post-processing|surface|", "Answer": "

Acetone can be used to smooth ABS.

\n\n

\"enter\nThe left has been smoothed by an acetone bath and the right is the original model. (Image credit: Andrew Sink)

\n\n

Take note: acetone is dangerous to breathe in, so work in a well ventilated area. It is also flammable.

\n\n

Warm acetone vapour bath using stove

\n\n

This method involves suspending the model in a glass jar of boiling acetone that is heated up by a pot of boiling water.

\n\n\n\n

See here for a step-by-step guide with pictures.

\n\n

Warm acetone vapour bath using printer bed

\n\n

This method elevating the model in a glass jar of acetone that is heated up by the printer's heatbed.

\n\n\n\n

See here for an example.

\n" }, { "Id": "4117", "CreationDate": "2017-05-20T20:46:47.597", "Body": "

Acetone is banned in my country, and I cannot get it.

\n\n

What substitute could I use?

\n", "Title": "I need an acetone substitute", "Tags": "|filament|prusa-i3|abs|quality|acetone|", "Answer": "

I have used DCM (dichloromethane, or ethylene dichloride) to dissolve ABS (or cold-weld/glue parts together).

\n\n

From ABS wikipedia page:

\n\n
\n

ABS polymers are resistant to aqueous acids, alkalis, concentrated\n hydrochloric and phosphoric acids, alcohols and animal, vegetable and\n mineral oils, but they are swollen by glacial acetic acid, carbon\n tetrachloride and aromatic hydrocarbons and are attacked by\n concentrated sulfuric and nitric acids. They are soluble in esters,\n ketones, ethylene dichloride and acetone.

\n
\n" }, { "Id": "4120", "CreationDate": "2017-05-21T05:21:40.173", "Body": "

Sometimes supports are very difficult to remove (physically) when I print with ABS. The image below, from Thingiverse - MOF-5 unit cell, is after significant effort to remove the yellow ABS supports from a black ABS model.

\n\n

\"Yellow

\n\n

I've learned about polyvinylalcohol (PVA) as a 3D-printing filament which is soluble in water. It seems like a great option for dual-extruder printers, where you can print the model with ABS or PLA and the supports with PVA, then throw the whole thing in water and let the supports dissolve.

\n\n

I've considered other options since PVA is ~4x more expensive than ABS.

\n\n

Doing a PLA print with ABS supports, followed by dissolution of ABS with acetone, is my best idea currently.

\n\n

Has anyone found success with another option?

\n", "Title": "Best method to make dissolvable supports?", "Tags": "|filament|pla|abs|support-structures|support-material|", "Answer": "

Not sure if frame challenge is on-topic here, but have you tried using different support options in your slicer? In Cura, I've found the experimental support tree mode both reduces the material usage a lot and gives supports that are easier to remove, at least for geometric (vs \"organic\", where it's sometimes good and sometimes bad) models. Turning off walls for normal (non-tree) support (infill and roofing only) and/or using a thinner line width than your nozzle can also help with removal but whether it provides sufficient support depends a lot on your model's geometry.

\n" }, { "Id": "4127", "CreationDate": "2017-05-23T09:11:51.743", "Body": "

The organization I am working for plans to pull down and rebuild our office building. As some people are a bit sentimental about the old building, we think about making a 3D model of the old building.

\n\n

The old house was built in the 60s or 70s, when everything was done by hand. So we have some really detailed hand-drawn plans of the building. We also have a nice and detailed (physical) 3D model of the house. However, what we don't have yet is any material that can be processed in modern 3D modeling software.

\n\n

So this is what my question is about:\nWhat is the best (easiest, with reasonable degree of details) way to make a 3D model of the old building?

\n\n

At the moment I think of two different paths:

\n\n
    \n
  1. Transferring the hand drawn plans to a 3D modeling software. I think, Sketchup might be suitable, but I am open for suggestions. It should be something easy to start with as I would like to motivate some colleagues to join the project.
  2. \n
  3. Using some kind of photogrammetry software to transfer the (hand crafted) 3D model of the building to an \"electronic\" 3D model. I have no experience with this method, however if it works it might be much easier to do.
  4. \n
\n\n

Can you help me to decide which path to choose and give me some tips on how to do it exactly?

\n", "Title": "Best practise of re-modeling a building", "Tags": "|3d-models|", "Answer": "

That are certainly trade-offs between the two options.

\n\n

I think main trade-off between the two options you described would be visual details vs. configurability.

\n\n

With a scanned design you have the ability to capture all the visual details that are in your hand crafted model. Those same details might require more effort than you are willing to put into a CAD design (like window and door detail and other architectural accents).

\n\n

The big disadvantage of a scanned model it that it would not be parametric so you can't fiddle with design details - like adjusting the width or spacing of the windows, the position of a door or external approach (stairs, etc). Also you could play with the external skins/finishes and play with how wood looked as compared to concrete of brick.

\n\n

Another advantage of a CAD model would be that you would not be able to break out components and print them separately because of print volume issues or to be able to print in different colors.

\n\n

My personal preference would be to create a 3D Model using a CAD package. The limitation on this would be that it might not have the same level of detail; but it would be much more flexible. You could probably create the base design pretty easily either from the architectural drawings or by deconstructing the hand crafted model. The model was probably built from pieces much the same way a 3D CAD model is created.

\n\n

Here is a good example of how to do that. In the example, they create a pretty detailed house design in Sketchup in 35 min by starting with scans of the architectural drawings.
\n\"house

\n\n

There are quite a few other similar videos so you should look for the one that fits you best.

\n" }, { "Id": "4131", "CreationDate": "2017-05-24T04:29:47.617", "Body": "

Today I received my Monoprice Maker Ultimate 3D Printer. It is a rebranded Wanhao Duplicator 6 for reference. I am using the default settings for a Wanhao Duplicator 6 in Simplify3D.

\n\n

Here is the Start G-Code that Simplify3D has setup for me via the Configuration Assistant:

\n\n
G28 ; home all axes\nG92 E0 ; zero the extruded length\nG1 Z10 ; lower\nG1 E20 F200 ; purge nozzle quickly\nG1 E10 F60 ; purge nozzle slowly\nG92 E0 ; zero the extruded length again\nG1 E-1.5 F400 ; retract\nG1 X170 Z0 F9000 ; pull away filament\nG1 X180 F9000 ; wipe\nG1 Y20 F9000 ; wipe\nG1 E0 ; feed filament back\n
\n\n

The problem is that right before a print, the extruder squeezes out a bunch of filament making a nice little spiral tower. It is a waste of filament. I suspect it is all the purging in the Start Code that is doing it, but I don't know what I should change because I don't know what is necessary, so I am coming here to ask the question before I start experimenting. Has anyone had this problem? Does anyone know the solution?

\n\n

Update: I tried printing one of the models that came on the SD card with the printer (I think it was created with Cura) and the Start G-code is different.

\n\n
G21        ;metric values\nG90        ;absolute positioning\nM82        ;set extruder to absolute mode\nM107       ;start with the fan off\nG28 X0 Y0  ;move X/Y to min endstops\nG28 Z0     ;move Z to min endstops\nG1 Z15.0 F4800 ;move the platform down 15mm\nG92 E0                  ;zero the extruded length\nG1 F200 E3              ;extrude 3mm of feed stock\nG92 E0                  ;zero the extruded length again\nG1 F4800\n;Put printing message on LCD screen\nM117 Printing...\n
\n\n

It also uses absolute positioning. It got going without purging a bunch of unnecessary filament on that print. Now I am not sure how to combine these two to get the good working starting G-code. Any ideas?

\n", "Title": "Monoprice Maker Ultimate Extrudes Too Much Filament At Start", "Tags": "|g-code|extrusion|simplify3d|wanhao|monoprice-maker-ultimate|", "Answer": "

Changing the Simplify3D start script to this will change the nozzle purge to the same length as what was on your SD card.

\n\n
    G28 ; home all axes\n    G92 E0 ; zero the extruded length\n    G1 Z10 ; lower\n    G1 E20 F200 ; purge nozzle quickly<---------Change E20 to E3, E is the extrusion length\n    G1 E10 F60 ; purge nozzle slowly <----------Remove this line\n    G92 E0 ; zero the extruded length again\n    G1 E-1.5 F400 ; retract\n    G1 X170 Z0 F9000 ; pull away filament\n    G1 X180 F9000 ; wipe\n    G1 Y20 F9000 ; wipe\n    G1 E0 ; feed filament back\n
\n\n

The rest of it is just moves to try to clean the nozzle.

\n" }, { "Id": "4133", "CreationDate": "2017-05-24T13:52:19.213", "Body": "

I am having trouble printing a hollow object using Slic3r. On flat slopes on top, there are gaps that I cannot get fixed. The perimeters of successive layers just don't cover each other.

\n\n

Cura however adds filament to cover the gaps.

\n\n

The bottom left bunny is sliced with Slic3r 1.2.9.99.\nThe top right bunny is sliced with Cura 2.5. Take a closer look at the forehead and the back of the bottom left bunny.

\n\n

I have \"extra perimeters if needed\" turned on. But turning it off makes no difference. What am I missing?

\n\n

So far only adding infill and increase the solid top layer count helps to get a closed surface. But then everything gets stiffer. The bunnies are printed with nylon so they are a bit squishy.

\n\n

\"Cura

\n\n

The printer settings in both Cura and Slic3r are:

\n\n\n", "Title": "Slic3r settings for flat slopes on top", "Tags": "|slic3r|slicing|ultimaker-cura|", "Answer": "

This seems to be a recurring problem with Slic3r.

\n\n

Slic3r appears to have problems with perimeters that are not attached to infill. I suspect that it is getting confused on what is the inside and what is the outside. I know that seems a bit silly; but as you slice an object with indentations (like the bunny's face) then the perimeter can cease to be a simple closed shape and it gets confused. If you use a viewer to step through the gcode layers around the place it starts having problems you may be able to see what is going wrong.

\n\n

Here are a couple of examples of why I say this is a recurring problem with slic3r. I also recall seeing a video that showed the problem but I can't remember where. That was one of the reasons I don't use slic3r.

\n\n\n\n

Here are three options that may work

\n\n
    \n
  1. Use a different slicer for this specific condition. Every product is going to have vulnerabilities - this may be one of slic3r's.
  2. \n
  3. Increase the perimeter and top and bottom layer thicknesses. Making them thick enough it will bridge the problem areas. Use a gcode viewer to inspect that area to see if it fixed the problem. That way you don't waste material on another fail. It sounds like you may have already tried this but you didn't like that it made the model stiffer.
  4. \n
  5. Repair the STL file using an application like Meshmixer. Maybe you will have to get the file close then tweak it where it doesn't. Here is good article from PinShape repairing and STL file.
  6. \n
\n\n

Good luck, hope this helps.

\n" }, { "Id": "4135", "CreationDate": "2017-05-24T16:37:21.587", "Body": "

What would be the end g-code so that I can get my Monoprice Ultimate (Wanhao D6) to lower the bed to midway on the frame or something easier to remove the print? The default end position is very high up at the top.

\n", "Title": "How do I get my Wanhao Duplicator 6 / Maker Ultimate to lower bed at end of print?", "Tags": "|g-code|", "Answer": "

In your ending script add the line below and change Z200.0 to your max Z height. This will rapid to the given value using absolute coordinates. I added G90 in case G91 was previously set so it did not do a relative move. I'm not positive if G91 is modal in printer firmwares or not but the G90 should not hurt anything.

\n\n
G90 G0 Z200.0\n
\n" }, { "Id": "4144", "CreationDate": "2017-05-27T15:59:45.830", "Body": "

I've got the following PLA filament that is not feeding correctly into our Ultimaker 2+

\n\n

It starts to feed and then all of the sudden, the wire 'eats' (read breaks, but not entirely) the plastic filament as you can see on the picture below:

\n\n

\"Photo

\n\n

Any hints are more than welcomed.

\n", "Title": "PLA filament not feeding correctly with Ultimaker 2+", "Tags": "|filament|ultimaker-2|", "Answer": "

It looks like your nozzle is jammed, since the filament is being chewed. You can fix this by heating the extruder up to around 220 C ( or just about 15 degrees above the recommended printing temp for your filament), then attempting to push some filament through the extruder. If this doesn't work, use a small wire and push it through the nozzle a few times, then run some filament through it.

\n" }, { "Id": "4150", "CreationDate": "2017-05-28T14:05:55.497", "Body": "

I had to pause an ongoing print (because of complicated reasons) and that caused the printer to reboot (I don't know why).

\n\n

When it rebooted, the Z extents were screwed up. I know this because when I gave the print again, the nozzle hit the print bed and started grinding.

\n\n

The printer has an auto bed levelling function that runs before every print.

\n\n

It takes three points on the bed where the sensor needle comes down to contact in a vertical to-and-fro motion. When that runs now, the head starts its vertical to-and-fro motion from a point considerably higher than where it used to start from earlier. The sensor needle does come down to touch the bed but it takes a longer time per point since the distance travelled is more now. (Hope this is clear)

\n\n

To print again, I compensated for the Z-direction by giving a global offset (2.65 mm) while generating G-code. But I realised later that X and Y dimensions were incorrect (larger by 4.6% to be precise). I tested this by printing cubes of 10, 20, 30 and 100 mm. The Z dimensions are perfect though.

\n\n

Also, there have been weird \"lags\" while printing larger objects. The print head stutters and this is causing sharp vertexes in between curves and some little blotching of material.

\n\n

The printer runs on Repetier 0.92.6 and I've been reading about it but its honestly intimidating to me. Please let me know how to figure out what to change and by how much.

\n\n

Edit: I agree this is quite similar to scaling issues thread here: Scaling/size issues in delta printers

\n\n

I just want to know what could've caused this problem, whether there are any other possible solutions, and how to get started with Repetier framework.

\n\n

Edit2: Attached image is the bottom surface of the 100 mm cube. You might notice slight charring on the bottom right corner. The masking sheet I pasted on the print bed was charred underneath. Also, it's evident that the bottom surface is not as compact at the corners as it is in the center.

\n\n

\"100mm

\n\n

What could be the possible reasons for that?

\n", "Title": "Delta printer printing incorrect dimensions in X and Y directions. Z dimensions are correct", "Tags": "|delta|repetier|", "Answer": "

As detailed at minow.blogspot.com (which I found very helpful for the most part - it was the first link in the answer to my related question) you need to check/adjust the zero in front of each tower, check the zero in the center, tweak the radius, and repeat (checking the zeros at the edge of the bed in front of each tower again) until it's correct.

\n\n

After that you adjust the rod length to correct XY size (and recheck all 4 zeros, thus the radius - it makes for a less than exciting day, but is worth it in the long run.)

\n\n

You either need to turn off EEPROM until you have the right settings, or save the setting to EEPROM each time - depends how worried you are about using up EEPROM write cycles, I guess.

\n\n

You should also turn off auto-bed-levelling while getting the basic calibration correct, per the same blog instructions.

\n\n

My fun with figuring out what my EEPROM was doing behind my back is here:\nDelta printer not responding to changes in DELTA_RADIUS

\n" }, { "Id": "4154", "CreationDate": "2017-05-29T17:40:26.627", "Body": "

I'm a new one for this community.\nThis also not directly related with 3D printing.\nI searched about this and I couldn't find good answer.

\n\n

One of my friends told me CNC machining centers (Milling) mostly use servo motors and CNC laser cutter and plasma cutters use stepper motors mostly.

\n\n

Position controlling is more accurate in servo motors than steppers.

\n\n

I think position controlling is more important in laser and plasma cutters than CNC machining centers, but laser and plasma cutters use stepper motors.

\n\n

Why do laser and plasma cutters use steppers without using servo motors?

\n\n

P.S.\nThis question has more area than 3D printing and CNC routing.And also, This question asked for more reason for why use steppers in laser cutters,plasma cutters and CNC router.SO, this is not a duplicate of this one.

\n", "Title": "Servo vs Stepper", "Tags": "|stepper|servo|", "Answer": "

The basic difference between a traditional stepper and a servo-based system is the type of motor and how it is controlled. Steppers typically use 50 to 100 pole brushless motors while typical servo motors have only 4 to 12 poles.

\n" }, { "Id": "4161", "CreationDate": "2017-05-30T10:43:50.183", "Body": "

I'm in the process of tweaking the firmware (Repetier 0.92.6) because of some problems mentioned here: Delta printer printing incorrect dimensions in X and Y directions. Z dimensions are correct

\n\n

But I found out that the printer manufacturer has locked the firmware (probably) and I cannot edit it. So I have to burn a new one (I guess? Correct me if I'm wrong)

\n\n

Printer name: Dimension Dual Delta

\n\n

Manufacturer: J Group Robotics

\n\n

Website: http://www.jgrouprobotics.com/dimension-dual-delta

\n\n

I have the slicer (Simplify3D) configured with the FFF profile as my printer was working completely fine up until a few weeks ago. I just want to know that if I change the firmware, would I have to change/update the profile as well? If yes, how do I do that?

\n\n
\n\n

Here is a screenshot of Repetier host. The \"JGR\" continues indefinitely:

\n\n

\"Screenshot

\n\n

Here is the actual log text

\n\n
22:56:00.216 : OpenGL version:4.2.0\n22:56:00.218 : OpenGL renderer:GeForce GT 540M/PCIe/SSE2\n22:56:00.218 : Using fast VBOs for rendering is possible\n22:56:06.634 : Printer reset detected - initalizing\n22:56:06.661 : start\n22:56:06.662 : Info:External Reset\n22:56:06.662 : Info:Autoleveling enabled\n22:56:06.662 : Transformation matrix: 0.999994 0.000010 0.003587 0.000000 0.999996 -0.002727 -0.003587 0.002727 0.999990\n22:56:06.865 : N1 M110*34\n22:56:06.865 : N2 M115*36\n22:56:06.865 : N3 M105*36\n22:56:06.865 : N4 M114*35\n22:56:06.865 : N5 M111 S6*98\n22:56:06.869 : N6 T0*60\n22:56:06.870 : N7 M20*22\n22:56:06.871 : N8 M80*19\n22:56:06.872 : N9 M105*46\n22:56:08.706 : Free RAM:1002\n22:56:08.706 : Autoretract:0\n22:56:08.706 : X:0.00 Y:0.00 Z:0.000 E:0.0000\n22:56:08.713 : N10 M220 S100*80\n22:56:08.713 : N11 M221 S100*80\n22:56:08.714 : N12 M111 S6*84\n22:56:08.714 : FIRMWARE_NAME:Repetier_0.92.6 FIRMWARE_URL:https://github.com/repetier/Repetier-Firmware/ PROTOCOL_VERSION:1.0 MACHINE_TYPE:Delta EXTRUDER_COUNT:2 REPETIER_PROTOCOL:3\n22:56:08.714 : Printed filament:32.75m Printing time:0 days 6 hours 0 min\n22:56:08.714 : PrinterMode:FFF\n22:56:08.714 : N13 T0*8\n22:56:08.745 : X:0.00 Y:0.00 Z:0.000 E:0.0000\n22:56:08.745 : Begin file list\n22:56:08.746 : End file list\n22:56:09.726 : JGR\n22:56:09.960 : N14 M105*18\n22:56:10.972 : JGR\n22:56:11.979 : JGR\n22:56:12.969 : JGR\n22:56:13.016 : N15 M105*19\n22:56:14.028 : JGR\n22:56:15.035 : JGR\n22:56:16.025 : JGR\n22:56:16.072 : N16 M105*16\n22:56:17.085 : JGR\n22:56:18.091 : JGR\n22:56:19.081 : JGR\n22:56:19.134 : N17 M105*17\n22:56:20.157 : JGR\n22:56:21.163 : JGR\n22:56:22.154 : JGR\n22:56:22.189 : N18 M105*30\n22:56:23.202 : JGR\n22:56:24.208 : JGR\n22:56:25.199 : JGR\n22:56:25.245 : N19 M105*31\n22:56:26.253 : JGR\n22:56:27.261 : JGR\n22:56:28.267 : JGR\n22:56:28.307 : N20 M105*21\n22:56:29.319 : JGR\n22:56:30.326 : JGR\n22:56:31.316 : JGR\n22:56:31.363 : N21 M105*20\n22:56:32.376 : JGR\n22:56:33.382 : JGR\n22:56:34.372 : JGR\n
\n\n
\n\n

Edit: Since it is advisable to get a new controller board and keep the current one as it is, I'm attaching a picture of what my printer is controlled by.

\n\n

\"enter

\n\n

The black box on the green board reads HRD 12008. Google says its a DC-DC SMPS module but mine has some extra connectors than what I could find online.

\n\n

The black board is a MKS Base v1.3

\n\n

The other box is a solid state relay.

\n\n

This is more intimidating than I thought it would be. How do I proceed now? If I get a brand new controller board to play with, which should it be? Also, I can't seem to locate any RAMPS shield.

\n", "Title": "Help with Repetier firmware (probably locked and licensed)", "Tags": "|fdm|repetier|", "Answer": "

The firmware is not really locked by the OEM1, you just need to upload (or, as you say burn) a new version. This is not unusual2.

\n

Repetier is just like any other firmware that uses Arduino boards. While it may seem intimidating at first, it really is quite easy to regularly upload new firmware to the Arduino Mega 2560 board3. If you are not familiar with Arduino programming at all, then there are a good number of Repetier oriented guides out there, for example, from Repetier's own web site, Repetier-Firmware Documentation.

\n

If are you familiar with Arduino programming, then it really is the same as uploading the Blink sketches, but the base code is just bigger, that is all.

\n

There is too much to detail in a simple answer here and, as I stated above, there are a lot of online guides. However, en bref:

\n
    \n
  1. Download and install the Arduino IDE to your Mac/Wintel/Linux PC;
  2. \n
  3. Download the latest Repetier firmware to your Mac/Wintel/Linux PC4;
  4. \n
  5. Connect the Arduino Mega 2560 to your PC using a USB cable;
  6. \n
  7. Change the settings, i.e. Boards and Port settings, in the Arduino IDE, so that your PC can "see" the Arduino Mega 2560;
  8. \n
  9. Open the Reptier firmware, that you have just downloaded, in the Arduino IDE;
  10. \n
  11. Configure the Repetier firmware source code, i.e. make some edits in the Configuration.h file5, with the tweaks that you require (which you mention in your question)6;
  12. \n
  13. Compile the firmware using the Arduino IDE. and (finally);
  14. \n
  15. Upload the firmware to the Arduino Mega 2560.
  16. \n
\n

That is it. Once you have done it a few times, it will become second nature.

\n

Watch a few online youtube videos, as well, before you start, so that you have a better idea of what to do. For example:

\n\n

With respect to the Slicer's profile settings... I am not sure about that at all.

\n

The reason that you have no RAMPS board is that, on your MKS board, the RAMPS and the Arduino are merged into one. Any new controller would work, as they all run the same firmware. There are many to choose from. As you are just experimenting (and reverse engineering) at this stage, I personally would buy a cheap Chinese Arduino Mega 2560 (\u20ac6) and RAMPS (\u20ac4) board on eBay, for a total of only \u20ac10 (if you buy them separately - don't get a combination "deal" as they are generally more expensive - although compare the prices first, you may find a bargain), to test your settings. Once you are happy with those settings, then you can spend more money on a better board, should you feel that you need it.

\n
\n

Footnotes

\n

1 If you have to get around the licensing, then it sounds like you could/should recompile the firmware, or use some other firmware, i.e. Marlin. However, wait to see what other people suggest.

\n

2 There really isn't much risk in uploading new firmware. The only risk, which I guess Mark is referring to, is that you may/will not have the correct firmware settings for your make of printer, and therefore the printer may not work correctly, until you get the firmware settings correct. So, in that respect, it is a good idea to buy a secondary Arduino Mega and RAMPS1.4 board to play with, for just $10 (Chinese clones work just fine). But, as you say that your board already has bad settings, and you do not wish to pay for more licensing, then you haven't much to lose by uploading new firmware to it, except the warranty. So, if I were in your position, and I did not want to pay the licensing, I would set aside the supplied controller (for possible future use/reference), and use a Arduino Mega 2560/RAMPS 1.4 combination to control the printer. That said, if your printer is still under warranty, can't you ask the manufacturer to supply you a new controller with the correct settings?

\n

3 There is no way of extracting the code from an Arduino and then modifying it. You need the original source code files. See Is there any way to download a sketch from an Arduino? OK, you can obtain the machine code, but unless you are an expert, or guru, modifying it is impossible.

\n

4 The installation order, of the Arduino IDE and Repetier, will not matter. You probably need to rebuild the firmware, I would guess, but I am not 100% sure. It might be a good idea to contact the vendor to see what they recommend, before overwriting their code (assuming that the control board came pre-programmed)

\n

5 I found this video #18: Calibration extremely useful, for configuring Marlin, and understanding the firmware settings in the Configuration.h file.

\n

6 You may find this link useful for your "tweaks": Repetier-Firmware configuration tool for version 0.92.9 version

\n" }, { "Id": "4164", "CreationDate": "2017-05-30T21:21:01.550", "Body": "

I can't seem to understand the margins that Cura uses or how to fix them for my machine. Look at this:

\n\n

\"Screenshot

\n\n

I can't seem to tweak the stuff under machine settings to anything that does this better:

\n\n

\"Screenshot

\n\n

Maybe a bit of dup of Cura not allowing full print area to used, but I think my case is a lot worse and can't be explained by skirts.

\n", "Title": "Cura grey large grey area, almost nothing fits\u2026", "Tags": "|prusa-i3|ultimaker-cura|", "Answer": "

I had mistakenly left the setting \"one at a time\" in print sequence. This meaning the printhead needs a lot of room since it will go back and forth in z.

\n\n

Cura allocates this extra space even if there's only one object.

\n" }, { "Id": "4173", "CreationDate": "2017-05-31T18:49:09.567", "Body": "

TL;DR

\n

I need a complete list of fasteners (bolts, nuts and washers) for the Sintron Kossel.

\n
\n

Has anyone bought this kit, and knows the answer, or knows where this is documented?

\n

A complete list would be great, but in particular, I need the dimensions (M2.5/3/4/5? and length) and quantity of the bolts (and nuts) which:

\n\n

I have:

\n\n

Note that I am only interested in the fasteners specifically for the Sintron Kossel and not the Kossel in general. The Sintron uses the PCB mounted mechanical switches (with four holes), and not the simple barebones mechanical switches (with two holes), and uses 2020 aluminium1, not 1515 OpenBeam:

\n

\"Endstops

\n

Why? Because I purchased only the 3D printed parts from Sintron, and I am sourcing the rest of the parts cheaply, either locally or from Chinese suppliers on eBay.

\n
\n

1 In my particular case, there is an additional complication. The profile of the non-standard/non-"European" 2020 extrusion that I am using, does not take T-Nuts, only M5 hexagonal nuts (it is unbelievably difficult to find the standard 2020 extrusion in Bangkok). Combine that with the fact that the Sintron kit uses only M3 nuts and not M5 (I only discovered this after purchasing the printed parts kit (see the email from Sintron, in my answer below) - not through lack of research, but because there is no available list of fasteners), then I have the problem of finding a nut to use (see Fat hexagonal M3 nuts, with outer diameter greater than 6 mm).

\n

Here is a photo

\n

\"Four

\n", "Title": "Complete list of fasteners for the Sintron Kossel", "Tags": "|printer-building|diy-3d-printer|kossel|", "Answer": "

For the M3 t-nuts, could you use this printed solution.\nYou are just attaching a PCB; so, it you wouldn't need that much strength.

\n\n

Another excuse to use your 3D printer. ;)

\n" }, { "Id": "4182", "CreationDate": "2017-06-03T04:59:14.227", "Body": "

I'm very new to 3D printing, I purchased a Monoprice Maker Select Plus last week and I'm just getting started with printing my own models.

\n\n

I can create fairly simple models in Sketchup without a problem, export in .obj, then using Slic3r, covert the STL into G-code. This method worked fairly well for some simple cubic bearing holders I was printing, however when I tried to slice a complex shape (a wheel), the slicer shows an incomplete version of the .obj file, missing faces and transparent from some angles.

\n\n

\"the

\n\n

Here is now what the wheel looked like on the slicer

\n\n

\"preview

\n\n

There is obviously something I'm doing wrong in Sketchup, maybe I have to put it through some filters in MeshLab? Or I'm not constructing the model properly?

\n", "Title": "Slic3r not interpreting STL correctly", "Tags": "|3d-models|3d-design|software|slic3r|slicing|", "Answer": "

Sketchup is notorious for creating non-manifold models for 3D printing. The first recommendation would be to consider using a different resource for your models. I'm not a big fan of TinkerCAD, but it is similar in operation to many features of Sketchup. I've used it on a couple occasions where it was the fastest simplest way to create a particular model.

\n\n

There are other programs that are more complex in the learning curve. Meshmixer is good for more organic shapes, although it will perform reasonably well for engineering-type models such as yours.

\n\n

Blender is a great piece of free software with an extremely steep learning curve, but again, better suited to organics.

\n\n

I'm quite a fan of OpenSCAD which is script/text based and quite well suited to creating engineering-type models, especially if you need parametric modeling. The model you have in your pictures would be an easy build, perhaps twenty minutes creation time, a few dozen lines of text. I see two cylinders, some boolean subtraction and some radial primitives. If the code is clean (easy to do), the result is a manifold object.

\n\n

On the even more challenging level, there are programs such as SolidWorks but the learning curve there is steep too. In the same ballpark would be Autodesk Fusion 360, for which a free hobbyist version exists.

\n\n

Of course, with all of the above, there are YouTube tutorials to assist your learning process.

\n\n

More to the point, one can use an online 3D model repair service. I've attempted to use a couple with mixed results.

\n\n

Netfabb is a commonly recommended online repair service. I've not used that one, or if I have, the results were poor. A bad model (your example) will likely fail with most services.

\n\n

I've used MakePrintable in the past as well. Again, a bad model will fail and Sketchup is the culprit here.

\n\n

You can use Meshmixer in Analysis mode to find the errors and use Auto Repair All, but the results aren't going to be pretty!

\n" }, { "Id": "4185", "CreationDate": "2017-06-03T22:13:06.413", "Body": "

I have this wing design that I want to print:

\n\n

\"wing\"

\n\n

You can notice that its walls have a thickness, which is 1.0 mm.\nI want to print it so that the perimeters are inside that thickness.\nHere is the wing sliced with 1 perimeter and 0% infill:

\n\n

\"wing

\n\n

You can notice a gap between the perimeters. That gap is what I want filled and not the perimeters in the image. With 0 perimeters and 100% infill I got this zig-zagged line:

\n\n

\"Zigzag

\n\n

I want the gap to be filled; but, I don't want it to be zig-zagged like in the image. I want it to be smooth like the perimeters in the other image.

\n\n

Basically I want a smooth infill that goes around the curves just like the perimeters. The reason why I don't want to print both the perimeters and the infill is because I want to save as much weight a possible as this is a wing of a model plane that must fly, so the lighter it is the more efficient it will be.

\n\n

Any ideas how I can slice this?

\n\n

The images are screenshots in Slic3r, but I can use Cura as well.\nThis is just a test slice. The wing model is not finished yet.

\n", "Title": "How to slice this wing so it has a smooth infill like perimeters", "Tags": "|slicing|infill|", "Answer": "

I found a great solution!

\n\n

In Cura, there is a setting under Shell called Horizontal Expansion. What this does is it controls the distance between the two perimeters. A negative value in this field will make them come closer together, thus removing the gap between them.

\n\n

\"Horizontal

\n\n

I found that -0.1 is the perfect value for 1 mm thick walls like the ones in my design. So I set it to -0.1, then set the infill to 100% because in some spots the thickness is a little bit more than 1 mm, so the infill closes the gaps in those places. Here is the result:

\n\n

\"Image

\n\n

You can see the perimeters are now sticking to each other and there is no gap between them. The result estimated weight is 71 g, which is a quite good reduction from the estimated 92 g when not using horizontal expansion.

\n" }, { "Id": "4203", "CreationDate": "2017-06-08T15:04:33.720", "Body": "

Has anyone used one of these aluminium cubes?

\n\n

\"Aluminium

\n\n

It is used to mount the fan for the hotend, when used on a delta effector, like so:

\n\n

\"Aluminium

\n\n

Are they a good idea?

\n\n

Without actually buying one and trying it, compared to the traditional fan mount, I imagine the pros are

\n\n\n\n

and that is it, I could think of only one tenuous advantage...

\n\n

However, I would imagine that the cons are:

\n\n\n\n

Therefore, as the cons appears to outweigh the pros, are they worth upgrading to from, say, a typical plastic fan mount:

\n\n

\"Plastic

\n\n

Does anyone have first hand experience? Are there any other additional benefits, and/or is heat dissipation that much better?

\n\n
\n\n

1 Of course, the lightest solution is with the fan mounted, such that it hangs off the top of the effector, but with no air flow guide and hence less effective heat dissipation.

\n", "Title": "Aluminium cube - fan and hotend mount - is it worth it?", "Tags": "|hotend|delta|fans|", "Answer": "

I think its only advantage is that it serves to move money from the folks who buy it to the folks that sell it, and that's an advantage purely for the folks that sell it.

\n\n

\"Additional heatsink surface area\" is quite doubtful - it would have poor coupling to the actual heatsink. If the actual heatsink is correctly designed, there's no need for additional area, anyway. This design clearly does limit airflow to the top and bottom fins - the bottom one, in particular, is not a good place to limit airflow for best function, and yet it's severely occluded by the design of this block.

\n\n

Save your money.

\n\n

Edit - I think the plastic one shown is also less than ideal - the close coupling to the edges of the heatsink reduces the effectiveness of the heatsink, which would otherwise be dumping heat into the airflow past those edges. A better design (IMHO, gut engineering) would be spaced off the ends/edges of the fins about as much as the space between fins, not touching them. Axial fans perform poorly with backpressure, and that would also reduce backpressure.

\n" }, { "Id": "4205", "CreationDate": "2017-06-09T05:33:24.357", "Body": "

I just printed my first thing, now there's very little filament left in my Monoprice Select Mini v2.

\n\n

What do I do about it? Yank it out? Try to print out what's left of it and hope it all extrudes safely?

\n\n

I'm not sure what my next move here is. Is doing either, dangerous to the safety of my machine? I haven't seen any guides or information on this and I'm afraid I'm going to break something.

\n", "Title": "What do I do when I have a little filament left?", "Tags": "|filament|", "Answer": "

Heat the extruder up first, then remove the filament. You can remove the filament either by reversing the extruder using a command such as G1 E-100 F200, by using your printer's controls/LCD (if it has one) or simply pull the filament out by hand. To this end, most extruders have a lever that you can push to disengage the drive gear to make it easier to pull the filament out.

\n\n

You can just heat the extruder up to printing temperature, but a perhaps better approach is a cold-pull, where you heat the extruder up just barely enough to be able to remove the filament (to, say, 120-180\u00b0C) and then pull the filament out. This has the advantage of removing more of the old filament and perhaps also taking some debris that may be stuck in the nozzle with it.

\n\n

Trying to print it all out won't work because once the last bit of filament goes past the drive gear, you won't be able to extrude the rest of it. Feeding in a new bit of filament might push the old filament out, but it could just as easily get jammed.

\n" }, { "Id": "4208", "CreationDate": "2017-06-09T16:12:34.507", "Body": "

I'm able to connect to Arduino+RAMPS through USB. I can even check end stops with M119. However, if I plug it to the power supply, my MacBook goes crazy and restart itself! Things I have tried:

\n\n\n\n

I'm at a point I started to think my MacBook's USB ports are messed up. It is a 2008 model, do you think that would be right to blame?

\n\n

Update

\n\n

Tried a new and a very short USB cable: It worked for the first time and I'm able to home all axes, but laptop's trackpad gets very hard to control for some reason. I can only use an external mouse.

\n", "Title": "Printrun doesn't connect when 12 V is on", "Tags": "|ramps-1.4|arduino-mega-2650|pronterface|grounding|", "Answer": "

It sounds a bit like you are having (additional) grounding issues1. Are your MacBook and the RAMPS using the same ground, i.e. are they using the same mains circuit?

\n

If you are using the same power strip, then the next thing to check would be that the PSU on the printer correctly grounded (at the bottom of the unit).

\n

From David Crocker's superlative blog, see Upgrading the large delta printer - Power supply output grounding:

\n
\n

Power supply output grounding

\n

While taking some measurements on my machine, I discovered that the\nnegative side of the output of the 12V LED power supply I am using was\nnot connected to ground inside the power supply as I expected. This\npresents a safety hazard. So I added an additional wire from ground to\nthe spare negative output terminal of the power supply.

\n
\n

\"Correctly

\n
\n

1 Why do I think this? Three reasons:

\n
    \n
  1. Because grounding issues is one of the most common, and over-looked, problems, in circuit design;
  2. \n
  3. I have often had peripheral devices, that use a separate additional power supply, cause a PC laptop to go "haywire" (or worse, die) once connected via RS232/USB (especially charge sensitive devices on the laptop, such as the track pad), due to different, or floating, grounds, and;
  4. \n
  5. I had recently read the blog and made special note to check all of my power supplies as they are all terrible, cheap, Chinese knockoffs, and grounding circuits in Thailand are usually non-existant, or, at best, poorly wired up.
  6. \n
\n" }, { "Id": "4209", "CreationDate": "2017-06-10T01:56:58.333", "Body": "

TL;DR

\n

Is it possible to build a hotend, using separate parts, sourced on eBay, from China, and still achieve a high quality print?

\n
\n

In my opinion, the E3D V6 hotend is rather expensive, at $/\u00a370, especially to those building to a tight budget. Now, obviously/presumably, there is some quite rigorous QA and testing of the build, and they have to pay for their research costs and initial machining set ups and this is reflected in the high cost. Also, the expression, you get what you pay for, springs to mind.

\n

However, I wondered if I sourced the individual parts from China, for a dollar apiece, that is to say the:

\n\n

could I too, end up with a hotend, that performs as well as, or even out performs, an E3D hotend, or am I living in cloud cuckoo land?

\n

After all, let's not forget that most, if not, all of the parts used in a E3D hotend probably come from China anyway, these days, as most production facilites have moved from the US/EU to the cheaper manufacturing bases and machining shops, in SE Asia.

\n

I have read so many times that people were experiencing extrusion problems, shoddy prints, etc, from their cheap printer, until they finally shelled out the cash and upgraded to a genuine E3D hotend. After that the prints were much improved. I have read this countless times, in blogs, threads, etc., so I presume that it is not possible.

\n

I realise that if the parts are not finished correctly, and end up with burring on the inside of the throat, nozzle, etc., then the filament will not extrude correctly. See tbm0115's answer to How long is an extruder's/nozzle's life? However, assuming that the parts that I receive are machined, and finished, correctly then there should not be a problem, right?

\n

Has anyone actually done this? Is tinkering with hotends a done thing? Does anyone have any good pointers and/or tips?

\n

NOTE: This question is not really after opinionated answers, in order to stay within SE rules - although opinions are always welcome - but I just want to know if anyone has first hand (positive or negative) experience of this.

\n", "Title": "Can cheap hotend parts sourced from China actually produce good prints?", "Tags": "|printer-building|hotend|diy-3d-printer|", "Answer": "

Sorry for digging up the topic, but I'd like to share my experience with cheap Chinese parts obtained from those obvious auction sites.

\n\n
\n\n

For my Hypercube Evolution I have ordered all parts from Chinese suppliers except for the Aluminium extrusion profiles. This includes belts, screws, nuts, electric cables, printer board, steppers and drivers, hotend parts, you name it.

\n\n

I do not own, or have ever owned, an official E3D hotend, so I cannot compare the clone E3D hotends to the official directly. However, I do own an Ultimaker 3 Extended (UM3E) that I manage for a group of printing enthusiasts to promote building your own printer and produce spare parts that are not sold commercially or sold at ridiculous prices. The quality of the products produced by the UM3E are very good, as expected from this quality brand, so I can compare the products from my own printer with the ones from the UM3E.

\n\n

It is often said by others that the prints I have created with my HyperCube Evolution are at least as good as the products produced on the UM3E, or sometimes even better. In that respect, it is very hard to see any differences in the products. The UM3E is known for the simplicity of use and it's relative low need to tinker to get or keep it working, but my own creation does not require a lot of tinkering either. E.g. I have never had any clogs (several kilometers of 2.85 mm filament printed) while I use a relative high retraction length (the same as on the UM3E as default in Cura). In this respect I have never had the need to upgrade to an official E3D hotend, and most likely will never do need to.

\n\n

So to answer your question if it is possible to build a hotend, using separate parts, sourced from China, to achieve a high quality print (while maintaining a low need for tinkering), I can vouch that it is indeed possible.

\n\n
\n\n

Sidemark:\nNote that if you source your materials locally, e.g. local webshops, you probably also end up with imported Chinese parts! (usually at much higher costs; but at least they arrive sooner!)

\n" }, { "Id": "4213", "CreationDate": "2017-06-10T08:49:15.687", "Body": "

So my printer's calibration got screwed after a sudden reboot and since the current firmware (0.92.6) is licensed, I cannot edit it. So I'm building a new one.

\n\n

These are the controller boards of my printer:

\n\n

1. Main controller board:

\n\n

\"Main

\n\n

The black board is a MKS Base v1.3

\n\n

The other green board beside it has a DC to DC SMPS (HRD12008) on it, along with Z-min, X-min, T0, Fan-, D11 and a flat ribbon wire connector (can be seen in the picture).

\n\n

The other box with an acrylic top is a solid state relay.

\n\n
\n\n

2. Cooling fan and Z-probe PCB:

\n\n

\"Hot-end

\n\n
\n\n

The brand on all these boards is \"D-force\". I've traced the printer down to a Taiwanese manufacturer: D-FORCE.TW - 3D Printer.

\n\n

There are a few parts available:

\n\n\n\n

I haven't been able to find the other green board (with the HRD12008 SMPS on it), yet.

\n\n

I've downloaded a new firmware from Repetier website. It can be found here, Repetier-Firmware-2017-06-09.

\n\n

Now, while compiling the firmware, I need to specify the Z-probe pin. I'm unable to do so because it's not plugged into the main board.

\n\n

Here is a close-up of the Z-probe & the servo motor that pins it down.

\n\n

\"Z-probe

\n\n

Any help would be appreciated here. On the web these printers are known as \"D-force printers\".

\n\n

Edit1: I found this on the manufacturer's Facebook group:

\n\n

\"MKS

\n\n

Please note that this is v1.1 and my printer has v1.3 although I haven't found any difference between the two, yet. I'll update on that if I find any. Hope this helps.

\n\n

Edit2: The Y-min pin on the main controller board is empty. The X-min and Z-min are connected to the X-min and Z-min on the additional green board respectively. Also, there is a T0 on the additional board which is connected to A13 of the main controller.

\n", "Title": "Building new Repetier firmware for a delta printer (D-force)", "Tags": "|delta|repetier|", "Answer": "

You, obviously, do actually have a Z-probe, but in case another user does not, or you decide not to use yours, I will cover both cases (with and without a Z-probe).

\n\n

Z-probe not present

\n\n

If not, then in configuration.h set the FEATURE_Z_PROBE to false, like this:

\n\n
#define FEATURE_Z_PROBE false\n
\n\n

or

\n\n
#define FEATURE_Z_PROBE 0. \n
\n\n

Then you also need to set

\n\n
#define DISTORTION_CORRECTION 0\n
\n\n

as this requires the FEATURE_Z_PROBE if otherwise set to 1 or true. However, thereafter, upon recompilation, I then get a lot of other errors... but these seem to be more to do with selecting the correct board, #define MOTHERBOARD, in pins.h, than the disabling of the Z-probe. This error:

\n\n
fastio.h:29: error: 'DIOORIG_FAN2_PIN_WPORT' was not declared in this scope\n
\n\n

is due to, from Can't compile firmware:

\n\n
\n

You are using ORIG_FAN2_PIN in your configuration but apparently your boards pin definition does not define that pin. In that case check board description and use the digital pin number instead for that function.

\n
\n\n

Actually that is not entirely true1.

\n\n

NOTE: It might be easier to just define Z_PROBE_PIN to something other than -1, which is what it is defaulting to, as set in configuration.h:

\n\n
#define Z_PROBE_PIN -1\n
\n\n

Z-probe present

\n\n

If you do have a Z-probe, then trace the wires to see where they go on the controller board, determine that pin number on the board and then enter that pin number into configuration.h. For example, if it is connected to pin 63, then:

\n\n
#define Z_PROBE_PIN 63\n
\n\n

and leave DISTORTION_CORRECTION untouched.

\n\n

Where to plug the Z-probe in

\n\n

If your Z-probe has become unplugged, then you need to choose a pin to connect it to. This is easy... you simply connect it to any free pin. Now, you probably only have three end stops, at the top of the delta frame. These are the MAX end stops. So, you can simply connect the Z-probe to any of the three MIN end stop pins, on the board. Obviously, you will leave the three #defines for the MIN end stop, in Configuration.h, un-configured, and then specify the pin, that you did use, in

\n\n
#define Z_PROBE_PIN <your pin>\n
\n\n

See Is there a complete step by step manual for building Z probe - comment 1950

\n\n
\n

Put the zprobe on any free pin you have but do NOT set it in endstops. It is not a endstop it is a zprobe.

\n \n

Then set it in zprobe section with pullup enabled so it is drawn to high internally until you connect the pin with gnd. Then G31 should also be consistent. I guess you missed the pullup part making it a random result.

\n
\n\n

and Is there a complete step by step manual for building Z probe - Comment_1952

\n\n
\n

Not sure why this is a problem. You have 6 endstop pins and need 3 for\n endstops + 1 for zprobe = 4 pins. Still 2 endstop pins free.

\n \n

When I said not in endstops I meant not in the endstop configuration.\n There you only put switches that work as endstop.

\n
\n\n

Highlighting the connections

\n\n

The green PCB, for the fan and Z-probe, does not appear to have any logic on it (I can't see any), so it is just a collection of tracks (i.e. wires). As there are no logic chips on it, you can trace the circuit from the Z-probe to the controller (either visually, or using a multimeter set on continuity check). I assume that the PCB is connected to the controller board somewhere/somehow... Via the 10 wire grey ribbon cable that goes to the second green PCB next to the main board, next to LED3, as shown in the first photo? This ribbon cable transfers signals to/from the Z-probe/fan PCB to the secondary PCB, which appears to be a power related PCB (i.e. heaters). Somehow the secondary, power, PCB then connects to the main controller board.

\n\n

One of the three unused MINIMUM end stop pins should be used for the Z-probe. This is assuming that:

\n\n\n\n

If you have six end stops, (one at the top and one at the bottom), then that changes things drastically. Assuming that you only have three, then the three unused MINIMUM pins will be used for some other purposes (Z-probe, servo arm, something else?).

\n\n

You need to confirm that you only have end stops at the top of the printer and not the bottom, i.e. only MAX_ENDSTOP (or X+, Y+ and Z+) pins. If so, then you should have three (sets of) (MIN_ENDSTOP) pins with nothing connected to them (in theory). So, one of those three MIN_ENDSTOP pins will have the Z-probe connected to it. If you are having trouble locating the MIN_ENDSTOP pins, trace where the 3 physical (MAX) end stops are connected to, those will be the MAX_ENDSTOP pins. Then from that, the MIN_ENDSTOP pins should be relatively easy to identify. Hopefully that makes sense.

\n\n

Looking at the schema then it becomes obvious. You see X+, X-, Y+, Y-, Z+ and Z-. The physical endstops should be connected to the three + connectors. The three - connectors: two of them are hopefully not connected. The one that is should be the Z-probe. Looking at the photo, unfortunately, it seems as if all six (or just five?) are populated.

\n\n

Personally, to save time, I would get a multimeter, unplug the X, Y, Z +/- connectors (actually only the - connectors) in turn and test each one, to see which is connected to the Z-probe.

\n\n

To check the correct pin is used, you really only have two (sensible) options: either visually, or; multimeter. Multimeter would be best, and the most certain. However, there are two other options open to you (one risky, and one laborious):

\n\n\n\n

Schema

\n\n

\"Schema

\n\n

Photo of control board

\n\n

\"Photo

\n\n
\n\n

It is probably a good idea to read this, lengthly, article: Repetier - Z-probing.

\n\n
\n\n

Footnotes

\n\n

1 I have finally managed to get Repetier.ino to compile. The problem lay with the Configuration.h file that came with the googledrive download, that you provided. Trying to either manually configure Configuration.h, or pre-loading it into Repetier's configuration tool, always resulted in the same compilation error, see DIOORIG_FAN2_PIN_WPORT error. The simplest solution is to create a Configuration.h file from new, using the Configuration tool. When you do that, it will compile ok.

\n\n

Or, if you really want to do it manually, either:

\n\n
    \n
  1. Set NUM_EXTRUDER to 1 and not 2 as the FAN2 compile error comes from the second extruder's set of #defines, or;
  2. \n
  3. More precisely, if you do have two extruders, then change the line #define EXT1_EXTRUDER_COOLER_PIN ORIG_FAN2_PIN and specify a pin, i.e. 65, en lieu of using ORIG_FAN2_PIN.
  4. \n
\n" }, { "Id": "4224", "CreationDate": "2017-06-13T07:24:13.320", "Body": "

I have a Tronxy X3 (i3 Clone) running Repetier firmware on a Melzi board.

\n\n

I would like to share the printer over my home network so that:

\n\n
    \n
  1. Both my boys and I can use the printer. (We have separate Windows 10 PCs)
  2. \n
  3. I can initiate a print from my computer upstairs
  4. \n
  5. I can monitor the print progress remotely
  6. \n
\n\n

I have (and could use)

\n\n\n\n

What I want to know:

\n\n\n", "Title": "Sharing a printer over a network", "Tags": "|software|reprap|tronxy-x3|", "Answer": "

If you set up OctoPrint or Klipper, you can then use a tool like OctoEverywhere to get remote access and share it. OctoEverywhere has a feature called Shared Connections that lets you share printer access with anyone over the internet.

\n" }, { "Id": "4225", "CreationDate": "2017-06-13T08:11:08.787", "Body": "

Sometimes I notice that if I manually command a single axis movement (typically Z, when I want better access to the extruder), I observe that several channels move together (and they maybe move slower than I expect). After one 'coupled' movement, subsequent commands have the result I'm expecting.

\n\n

What happens is as well as the Z-axis moving up, the bed moves forward, and the extruder moves to the right. I have no auto-leveling or anything else non-standard on this printer (dual Z steppers, X, Y, extruder, bed, extruder heat).

\n\n

It even happens if I simply extrude some of the time (e.g. changing filament after warming up, retract gave me some X movement)

\n\n

Printer is an ANET-A8, I'm using mainly OctoPrint, but I think I've also observed this with other PC software manual controls.

\n\n

It doesn't happen often enough for me to have identified any pattern - maybe it only happens if I've not homed first after turning the printer on, but I suspect not.

\n\n

It's not so much of a problem, as just a question for interest. Also not sure how to tag.

\n", "Title": "Lack of isolation between axes", "Tags": "|software|", "Answer": "

My best guess based on the answers and comments so far is that there is a minor firmware bug and some poorly initialised state. This occurs at start-up, or after a print has finished, but only once (till presumably the idle state is reached again).

\n\n

See the image below, captured after the power went off during a print. Heated the extruder, and did Z -10, got this. Both x and Y moved, but in about a 10:1 ratio. The length of the track is about 10mm. Interestingly, during this movement, there was no Z movement.

\n\n

\"Movement

\n" }, { "Id": "4228", "CreationDate": "2017-06-13T15:46:32.617", "Body": "

I am writing a G-code sender for Android, but I have hit a snag.\nHow can I prevent the phone from discharging while it sending G-code to the printer?

\n\n

I have several Android phones and tablets and they have a micro USB OTG connector, which I can connect to my 3D printer using an adapter.\nBut it looks like OTG has this limitation - it can either accept power when in Device mode, or it can provide power in Host mode. So, while in Host mode the phone can't be charged, so I can't do long prints, which seems like an unreasonable limitation.

\n\n

Any ideas on how to solve this?

\n", "Title": "How can I charge an Android phone while it's controlling my 3D printer?", "Tags": "|g-code|", "Answer": "

See elsewhere on StackExchange:

\n\n\n\n

You certainly can charge whilst in host mode, but some re-wiring will be required.

\n\n

From the SE.Electronics link

\n\n
\n

Apparently it is possible to charge the Host-Device! -->\n http://en.wikipedia.org/wiki/USB_On-The-Go

\n \n

Under \"OTG Micro Plugs\" it says that a USB OTG cable with a 36.5 k\u03a9\n resistor between Pin 4 (I suppose its pin 4) and Pin 5 allows you to\n connect a B-Device (Slave) and (!) a Charger to the Smartphone/Tablet.\n The Phone and the B-Device can be supplied by the external power\n source.

\n \n

USB-Power Specifications:\n http://www.usb.org/developers/devclass_docs/batt_charging_1_1.zip

\n
\n\n

and

\n\n
\n

I just sifted through the docs because I didn't trust Wikipedia's implicit data on which of the three resistances I should use. Here you can find the official Battery Charging v1.2 Spec and Adopters Agreement: usb.org/developers/docs/devclass_docs In this case you want 124k\u03a9, because you want the OTG device (=tablet) to be the A device (=host).

\n
\n\n

Here are the schematics:

\n\n

\"Post

\n" }, { "Id": "4235", "CreationDate": "2017-06-14T15:56:58.717", "Body": "

From my understanding, both PLA and ABS can be treated with acetone to make them smoother. So when they are treated with acetone, which is smother PLA or ABS or are they about the same?

\n\n

When I search online, all I find is how to treat the objects.

\n", "Title": "What's smoother? Acetone treated PLA or ABS", "Tags": "|pla|abs|smoothing|vapor-smoothing|", "Answer": "

ABS filaments will smooth well with using acetone, it's been used for a while now.

\n\n

For PLA filaments it's a different story, pure PLA will not smooth out in acetone and it will likely only cause structural failure of the product.

\n\n

However most PLA filaments aren't pure PLA, they contain additives including ABS that react differently when exposed to acetone and the reaction will really depend on the manufacturer of the filament, only a few PLA filaments are known to smooth like ABS when in an acetone vapor bath, it is the case for ColorFabb PLA filaments and it is absolutely not the case for bq PLA filaments that only soften and break when exposed to acetone.

\n\n

You'll have to make small tests with different brands of filament to see which smooth well and which don't but if you want to go fast either go with ABS or use other smoothing techniques such as sanding + 3D print Smooth On epoxy.

\n" }, { "Id": "4236", "CreationDate": "2017-06-14T22:23:42.443", "Body": "

I am at my wits end with this problem. I start a print and the skirt goes down fine, then the outline of the parts go down fine (usually) and then when it goes to fill in the first layer, it will always get stuck to the hotend at some point and rip apart the layer.

\n\n

Any ideas on how to solve this?

\n\n\n\n

Maker Select V2.1, using Cura to slice.

\n", "Title": "First layer eventually sticks to the hotend and tears", "Tags": "|ultimaker-cura|", "Answer": "

Check if you have fan turned on for 1st layers. Cooling fan must be turned off for initial layers. That's a mistake I've made.

\n" }, { "Id": "4247", "CreationDate": "2017-06-17T14:25:58.920", "Body": "

I've been 3D printing for almost 4 years. Now that I have an M180, my RepRapPro Huxley is collecting dust as I only use it for flexible prints and ABS prints as it handles them better than the M180 does.

\n\n

I was wondering, since everything is open source in it and hardware-wise most open style Y carriage bed 3D printers are the same, would it be possible to make a 500 * 500 * 300 printer using the hardware from this printer?

\n\n

Since the carriage system isn't that good, I would replace it with solid machined aluminium rails and the heat bed would be made out of four Prusa I3 HB, which would only require me to add a PC PSU to power the whole thing and some MOSFETs to handle the load. I would be salvaging the board, the steppers, the endstops and extruder assembly basically.

\n\n

Would such a setup be worth it?

\n\n

Would the steppers have to handle more load?

\n", "Title": "Upscaling a reprap", "Tags": "|reprap|diy-3d-printer|", "Answer": "

To your three questions:

\n\n
    \n
  1. \".. would it be possible to make a 500 * 500 * 300 printer using the hardware from this printer?\" - YES
  2. \n
  3. \"Would such a setup be worth it?\" - Only you could determine that. It would quite a bit of work; but, if you enjoy making things it would likely be a fun accomplishment.
  4. \n
  5. \"Would the steppers have to handle more load?\" - X and Y axis don't fight gravity so the the only force you fight is friction and accel/decel. If you keep with a low friction design then you will just need to limit the accel and decel to what the motors can handle. Regarding Z, this depends on your design, if you stick with the standard lead screw approach then the motor power required really isn't affected by the height.
  6. \n
\n\n

Two thing to note about larger printers:

\n\n
    \n
  1. It takes a LONG time to make a large object. Make sure you consider this in the design as you probably won't be able to be there the whole time. Design with fire prevention in mind.
  2. \n
  3. Tall lead screws increase the risk of Z-axis wobble. Make sure your consider this. You will need VERY strait lead screws. If you are going to capture both ends of the lead screw, use flex couplings.
  4. \n
\n" }, { "Id": "4248", "CreationDate": "2017-06-17T19:03:46.070", "Body": "

First, X and Y axis works perfectly, I measured well. I have plugged all jumpers, and use a very standard ramps 1.4 setup with standard Nema 17 200rev 1.8 motors and A4988 drivers.

\n\n

For Z axis, I have leadscrew with 8mm diameter and 2mm pitch.

\n\n

I calculated it should be 1600 steps per mm.\nProblem is when I call G1 Z5 it goes to 2 cm. So I lowered my setting to 400 steps per mm and problem solved.

\n\n

I don't understand what I'm missing here. I have used this calculator to find 1600.

\n\n

Where I'm wrong on the math then?

\n", "Title": "Why do I have to lower my Z axis steps per mm?", "Tags": "|prusa-i3|marlin|z-axis|", "Answer": "

Your leadscrew probably is a 2 mm pitch, 4-start leadscrew. This means that there are actually 4 separate grooves on the leadscrew, each with a pitch of 8 mm. Confusingly, this makes the total pitch 2 mm, since the distance from one groove to the next is 8mm divided by 4 grooves. However, one revolution of the leadscrew will still move the nut by 8 mm. Thus, in the Prusa calculator, you should enter a pitch of 8 mm/revolution. Since you entered a pitch of 2 mm, you ended up with a figure that is 4 times too large.

\n\n

The reason leadscrews are made like this is that if you just had a single groove with an 8 mm pitch, the nut would need to be made very long to enable it to make contact with a sufficiently long portion of thread. By increasing the number of grooves, you can get away with a shorter nut. With lower pitch (lead-)screws you don't need multiple starts, since the lower pitch means the same length of nut is in contact with more thread.

\n\n
\n\n

For the relative advantages and disadvantages of the various leads (8/4/2 mm), as opposed to pitches, see also Would using a leadscrew with 1 or 2 mm lead, en lieu of 8 mm, result in a better printer?

\n" }, { "Id": "4263", "CreationDate": "2017-06-19T16:24:45.330", "Body": "

I configured Marlin 1.1.3 for auto-leveling with a fix mounted sensor connected to the Z end-stop pin.

\n\n

I have the following settings in my config:

\n\n
#define X_PROBE_OFFSET_FROM_EXTRUDER 25  \n#define Y_PROBE_OFFSET_FROM_EXTRUDER 20  \n#define Z_PROBE_OFFSET_FROM_EXTRUDER 0   // Z offset: -below +above  [the nozzle]\n\n#define Z_CLEARANCE_DEPLOY_PROBE   10 // Z Clearance for Deploy/Stow\n#define Z_CLEARANCE_BETWEEN_PROBES  5 // Z Clearance between probe points\n\n#define AUTO_BED_LEVELING_BILINEAR\n\n#define GRID_MAX_POINTS_X 7\n#define GRID_MAX_POINTS_Y GRID_MAX_POINTS_X\n\n// The Z probe minimum outer margin (to validate G29 parameters).\n#define MIN_PROBE_EDGE 10\n\n// Set the boundaries for probing (where the probe can reach).\n#define LEFT_PROBE_BED_POSITION 25\n#define RIGHT_PROBE_BED_POSITION 150\n#define FRONT_PROBE_BED_POSITION 30\n#define BACK_PROBE_BED_POSITION 180\n
\n\n

I enter M111 S38 to enable LEVELING+INFO+ERROR debugging.\nThen I enter G28 to home all axes and then enter G29 to start auto-leveling.

\n\n

The auto-leveling starts successfully and after finishing I see the scan grid in console:

\n\n
12:41:35.983 : Bilinear Leveling Grid:\n12:41:35.983 : 0      1      2      3      4      5      6\n12:41:35.983 : 0 -3.127 -3.405 -3.405 -3.292 -3.595 -3.487 -3.537\n12:41:35.983 : 1 -3.110 -3.367 -3.337 -3.220 -3.470 -3.350 -3.365\n12:41:35.983 : 2 -3.138 -3.367 -3.330 -3.215 -3.442 -3.345 -3.385\n12:41:35.984 : 3 -3.013 -3.225 -3.182 -3.047 -3.225 -3.132 -3.150\n12:41:35.984 : 4 -2.970 -3.165 -3.097 -2.972 -3.160 -3.045 -3.065\n12:41:35.984 : 5 -2.875 -3.075 -3.005 -2.847 -2.990 -2.872 -2.875\n12:41:35.984 : 6 -2.680 -2.845 -2.755 -2.615 -2.753 -2.617 -2.622\n12:41:35.985 : G29 uncorrected Z:10.00\n12:41:35.985 : corrected Z:12.85\n12:41:35.985 : <<< gcode_G29\n12:41:35.985 : X:120.00 Y:160.00 Z:12.85 E:0.00 Count X:9600 Y:12800 Z:4000\n12:41:35.985 : current_position=(120.00, 160.00, 12.85) : sync_plan_position\n
\n\n

So the auto-leveling scanning seems to be successful.

\n\n

Here is a visual of the leveling grid (but upside down to make it easier to view):

\n\n

\"Inverted

\n\n

Naturally I DO NOT enter G28 after the scanning.

\n\n

I enter G0 Z1 to down nozzle almost to table. But when I enter for example G0 X25 Y30 and look at level and then I enter G0 X150 Y150 I see the big difference between nozzle levels relatively to table. So it seems that height compensation does not work. I expect that Z axis would lift up or down depending on auto-leveling results but Z motor do not work when I move X/Y.

\n\n

By the way I tried 3 point autoleveling. It was pretty rough but Z axis corrected it's level when I moved axes using G0 commands.\nAnd to avoid questions \"why my table is inclined so much?\" I deliberately inclined the table to be sure that auto leveling works. By the way, I tried different Z_PROBE_OFFSET_FROM_EXTRUDER so the current 0 setting is not actual but situation is the same with any value of this option.

\n\n

What I am doing wrong?

\n", "Title": "Auto leveling with Marlin and RAMPS 1.4 does not work", "Tags": "|marlin|bed-leveling|", "Answer": "

In additional to Sergey's answer M420 S1 should be put into custom start G-code. By default Marlin disables autobed after G28 unless specifically configured otherwise

\n
/**\n* Normally G28 leaves leveling disabled on completion. Enable\n* this option to have G28 restore the prior leveling state.\n*/\n#define RESTORE_LEVELING_AFTER_G28\n
\n" }, { "Id": "4267", "CreationDate": "2017-06-21T06:21:13.097", "Body": "

I have a Monoprice architect which is a barebones clone of the FlashForge Creator Pro, or Replicator 1 Dual. I have upgraded the power supply and added a heated bed and, after getting fed up with MakerBot software, I've started using Cura to slice then post process with GPX.

\n\n

I did a lot of searching and finally found someone who posted their start and end G-code for this particular printer. The only catch is that his code only works on version 15.04.

\n\n

Don't get me wrong, 15.04 is a huge upgrade compared to MakerWare. But, I would really like to start using a newer version like 2.5 or anything relatively new.

\n\n

Here is the start code I found. I have tried it in 2.5 with error in post processing. Any help is appreciated!!

\n\n
; -- START GCODE --\nM136 ; start build\nM73 P0\nG90 ; absolute coordinates\n;\n; set temperatures and assert Vref\nM140 S{print_bed_temperature}\nM104 S{print_temperature} T0\nG130 X118 Y118 A118 B118 ; set stepper motor Vref to defaults\n; let the Z stepper vref stay at eeprom level (probably 40)\n;\n; home and recall eeprom home position\nT0 ; home on the right nozzle\nG28 X Y Z ; home all axes at homing speed\nG92 X0 Y0 Z0 A0 B0 ; set all coords to 0 for now\nG1 Z5 F500 ; move Z 5mm away so we can carefully hit the limit switch\nG161 Z F100 ; home Z slowly\nM132 X Y Z ; recall stored home offsets for XYZ axes\n;\n; wait for heat up\nG1 X110 Y-72 Z30 F3300 ; move to waiting position\nM116 ; wait for temps\n;\n; purge and wipe\nG92 E0 ; set current extruder position as 0 so that E15 below makes sense\nG1 X110 Y-70 Z0.2 F2400.0 ; move to just on the bed\nG1 X110 Y70 E15 F1200.000 ; extrude a line of filament along the right edge of the bed\nG92 E0 ; set E to 0 again because the slicer's next extrusion is relative to this 0\n;\n; Sliced at: {day} {date} {time}\n; Basic settings: Layer height: {layer_height} Walls: {wall_thickness} Fill: {fill_density}\n; Print time: {print_time}\n; Filament used: {filament_amount}m {filament_weight}g\n; Filament cost: {filament_cost}\n; -- end of START GCODE --\n
\n", "Title": "Using a MakerBot Replicator 1 dual (or clone like FlashForge Creator) with Cura", "Tags": "|software|makerbot|ultimaker-cura|g-code|replicator-dual|", "Answer": "

Thanks for everyone's help. So after playing around with the G-Code and trying to set the temperatures manually, I came across a post by a user who said that Cura will automatically include code for the temperatures regardless of your g-code. So I deleted the m140 and m104 lines and now it works great!

\n\n

I am also going to take a second to rant about how difficult it is to find information on this g-code business. It seems that the language, which should be standardized, is different depending on the slicer that you use. What is the deal?

\n" }, { "Id": "4280", "CreationDate": "2017-06-23T20:17:19.010", "Body": "

I would also be curious on how I could get the best results with these printers. I am in a situation where I have cheap access to a:

\n\n

Fortus 250mc, which prints in ABS and has a minimum resolution of 0.007mm (but that can be raised as high as 0.013mm to print faster), and has a \"Soluble Support System\" which enables printing overhangs.

\n\n

Form 2, which prints with Photosensitive Resin, has a resolution of .05mm, and can also print overhangs.

\n\n

Stratasys Dimension SST1200es, prints in \"ABSplus\" (whatever that is), has a resolution of .254 mm, and can indeed print overhangs. As far as I can tell the main advantage of this one over the Fortus is that the supports can be immediately removed after printing and don't need to be dissolved first.

\n\n

I am okay with processing or finishing the miniatures later so long as it's not too time consuming (such as by running an acetone vapor bath), but I'm curious when I would use each printer and how I could get the most out of both.

\n\n

P.S. Also the Stratasys is more expensive for me to print on than any of the others so please keep that in mind when answering :)

\n", "Title": "Which 3D printer should I use to make custom miniatures for a tabletop game?", "Tags": "|print-quality|abs|post-processing|", "Answer": "

The Form 2 will definitely give the best results for your application.

\n\n

Both the Fortus and the Dimension use FDM, which builds the model up using a bead of molten plastic. Because this bead is typically 0.5mm in diameter, this strongly limits the details you can print.

\n\n

The Form 2 uses liquid resin, that is cured by scanning it with a laser. This process is called SLA. The laser produces a 0.14mm dot, and so you can produce much finer details. The Form 2 can also use much thinner layers (down to 0.025mm v.s. the 0.178mm of the Fortus) so the models will be much smoother.

\n\n

To make small miniatures SLA is much more suitable than FDM.

\n" }, { "Id": "4283", "CreationDate": "2017-06-24T17:49:55.637", "Body": "

I was thinking of getting the MonoPrice Mini 3D v2 printer. I know the bed is kind of small.

\n\n

How would I know what I could print from Thingiverse? They don't really state what the bed size should be.

\n\n

Does the software fix this for you? Do you scale it?

\n", "Title": "3D printing bed size and selecting models", "Tags": "|diy-3d-printer|", "Answer": "
    \n
  1. Yes, objects in thingiverse usually doesn't include their size and this can require extra effort to obtain.
  2. \n
  3. There are several options to scale objects to fit your printer. You can often do that in the slicer or using a separate program like meshmixer.
  4. \n
  5. The bed size is not the only (or maybe even primary) limitation you will need to consider when printing an object. You also need to think of your max height, nozzle size and for your choice of material your will need to consider things like you max extruder and bed temperatures (or even have a heated bed), nozzle type, etc.
  6. \n
  7. You should be able to print just about anything on thingiverse; but, the limitations of your printer will affect HOW you will have to print it and whether you will be happy with the final product.
  8. \n
\n\n

I have heard of plenty of people the have smaller printers and it was the right place for them to start. Having a smaller printer that performs well for you will be a much better experience than a larger printer that performs poorly.

\n\n

Whether the Monoprice Mini 3D v2 printer is the best printer is something only you can decide. Read and watch the reviews, they can be a great source of \"perspective\". I know we all have limited budgets and there are certainly way more things you can print with a printer than you can without one.

\n" }, { "Id": "4286", "CreationDate": "2017-06-25T02:40:20.267", "Body": "

I have been getting clogs and believe that it may be due to a damaged PTFE tube inside my hot end. I have a replacement (it came with my printer), but I can't seem to fit the tube into the nozzle.

\n\n

I also tried to turn the original PTFE tube around, and I can't get the other end to fit into the nozzle either.

\n\n

Is there a trick to it?

\n", "Title": "How do I get a replacement PTFE tube to fit inside my nozzle?", "Tags": "|hotend|nozzle|ptfe-tube|", "Answer": "

For the sake of the few pennies, I'd buy the threaded tube with the PTFE already in from any internet auction site.

\n" }, { "Id": "4288", "CreationDate": "2017-06-26T05:21:55.250", "Body": "

I printed Planetary Gears and the top looks great\"top\"\nbut the bottom doesn't\"bottom\"

\n\n

I am printing on a TronXY X3 (Prusa i3 metal frame clone) using eSun PLA+ and sliced using Cura 2.4. I print on glass and do manual leveling (sheet of paper to set gap).

\n\n

What could be causing this?

\n\n

It almost looks like a raft; but, I selected to print with a Brim not a Raft.

\n\n

I have seen this on some other prints so I suspect it is a slicer setting.\nNote: Bed adhesion seemed great. First adhered well and part popped off with very little effort.

\n", "Title": "Why is the bottom of my part not smooth", "Tags": "|print-quality|ultimaker-cura|slicing|tronxy-x3|", "Answer": "

In my case, the build plate adhesion type was set to raft in the cura software settings which is why a base will be printed first before printing the object on it. Hence, the bottom of the printed object will look rough because it needs to be easily separable from the initially printed base. Build plate adhesion type should be set to brim, skirt or none, the bottom print should be smooth. That was what worked for me.

\n" }, { "Id": "4291", "CreationDate": "2017-06-27T08:46:16.010", "Body": "

I have an STL file from thingiverse. The model is of a rectangular lid with an engraving. I would like to print it using two different colors, so that the engraving would be in a different color than the lid base. In the model description, the creator explained that he simply switched the material mid printing.

\n\n

However, I have a two-extruder printer, and I'd like to utilize it for this printing. What's the easiest way (tool) to select a part of the model and define that it should be printed using a different color?

\n", "Title": "Convert an STL model to a two-extruder model", "Tags": "|3d-models|", "Answer": "

After a bit tinkering, I finally managed to split the model to two STL files, then used Cura to print them each in its own color. I decided to post here the steps I followed, as simply as I could, hoping that it would help other beginners.

\n\n

The procedure was quite simple, even though it took a while for me to figure it out:

\n\n

Following the advice of this video, I used MeshMixer to edit the STL file I downloaded from thingiverse. I:

\n\n
    \n
  1. Selected the areas I wanted to separate using the select tool (I had to change the tool \"radius\" so that it won't keep selecting unwanted faces of the model).
  2. \n
  3. In the same tool, I used the \"Separate\" command to create two objects from the one I had before. This showed the \"object explorer\" window.
  4. \n
  5. I clicked each of the parts in the object explorer window and exported them separately to STL files.
  6. \n
\n\n

Then, using Cura, I followed this site, and performed the following:

\n\n
    \n
  1. Loaded both models to Cura (order doesn't matter).
  2. \n
  3. Picked Print Core 1 for the first part and Core 2 for the second part.
  4. \n
  5. Selected both models (using the Shift key), then right-clicked and chose \"Merge Models\"
  6. \n
  7. Profit!
  8. \n
\n" }, { "Id": "4305", "CreationDate": "2017-06-29T13:09:18.130", "Body": "

I made a pressing stamp in AutoCAD 2013 for stamping letters, but I'm having difficulties to save the file. I can save it anyhow in any format, but when it comes to .STL, some parts just doesn't show.

\n\n

I made a platform on bottom (25x25x3 mm cuboid) on which I put solids as they represent my logo. But when I export, I select all, but it saves me just 3 or 4 parts from 7 total. On many occasions I get saved different parts, but never all of them or more than 4.

\n\n

I am also getting the error while saving, which says: \"the boolean operation on solids failed\" and \"Failure in face-face intersection merging algorithm.\"

\n\n

Anything I try to do (even copy to another file and do there, doesn't help at all. But the irony is, that I just created a simple text stamp, for which I had no difficulties at all to save to .STL.

\n", "Title": "Unable to export all objects to a STL in AutoCAD", "Tags": "|ultimaker-cura|", "Answer": "

Based on your error messages, it's likely you have created a non-manifold 3D object. As a simple example, let's start with a cube as suggested in your question. To keep things simple, you want to add a cylinder to the cube.

\n\n

In other programs, one would create a sketch on the surface of that cube and extrude the circle into a cylinder. Because the circle is constructed on the surface, it become an integrated part of the model.

\n\n

If you had created the cylinder independently, which can be done in AutoCAD and placed it on the surface of the cube, the resulting model would appear to be the same as the one created above.

\n\n

The placement method may have resulted in the end of the cylinder \"penetrating\" the surface of the cube. Such a model may generate the face-face error you've received.

\n\n

The quantity reference you've provided in your question indicates that the problem is contained in more than one intersection.

\n\n

If you have patience, you can return to the original model and attempt to locate piercings or penetrations and adjust the locations to be joined properly. Selecting a wireframe view may assist you in those efforts.

\n\n

Another option would be to import your .STL file into a program such as Meshmixer to attempt an automatic repair. Under Analysis/Inspector, errors will appear with flags which can be clicked to perform a repair.

\n\n

Severe errors in construction, such as the one you describe, are more likely to destroy the model when using this method, however.

\n" }, { "Id": "4307", "CreationDate": "2017-06-29T20:10:08.590", "Body": "

Building a 3-D printer is obviously a huge undertaking.

\n\n

Does anyone know of any reasonably cheap guides to build my own 3-D printer?

\n", "Title": "Building a 3-D printer", "Tags": "|diy-3d-printer|printer-building|", "Answer": "

Have you built a 3D printer before, or have much experience in electronics? Building your own printer is usually more expensive than purchasing a higher end low-grade consumer printer.

\n\n

There are a few routes you can go.

\n\n

Creality Ender 3 \nIt's a good starting 3D printer around 200ish. Just going to throw that out there first - as that is probably a far better route to think about when thinking about cost. It's decent out of the box. Mix it with Amazon Basics filament (roughly 20 a roll), cheap chinese filament (roughly 10-15 a roll), or if you live near a microcenter - Inland Filament (roughly 15 a roll) and you're set with minimal effort.

\n\n

Now... to answer your question - what TYPE of printer?

\n\n

FDM: That the plastic in the rolls - and by far the cheapest type of 3D printer (and the most common)

\n\n

SLA: That's the Resin Printers. This is far better quality (normally), but much harder to use, not really for those just getting into 3D printing. They are also far more costly. If you have the ability - this is the ones that start getting cheaper to make - but making a large resin printer is not easy at all.

\n\n

Bio Printers: I do not suggest making this. In fact, you probably can't even easily buy them. From your scope of what you are looking for - doesn't seem much like what you want either. This is the type to make molds for cataracts, organs, other medical needs.

\n\n

There are other various types of 3D printers, a lot of them actually, but these are three big ones. As I said, I'm going into a general overlook - if you want more info, I'd be glad to give you more in any of the topics.

\n\n
\n\n

SLA / DLP (both very similar)

\n\n

I'm going to assume you want to look between either SLA or FDM. SLA (Resin) can be extremely pricey, and the resin even moreso. They work by curing Resin via light.

\n\n

The -cheapest- way to build an SLA printer is by a projector! If you happen to have one already, then this route can cost you 20ish +-, not including Resin.

\n\n

What you need: Projector(50-100+ or free), old DVD drive/ cd drive from a pc (5 from goodwill, free for most people who have old systems laying around), Nema 17 Motor(brand new 15ish - shopping around ebay I have gotten 5 for 15 before), Threaded rod (home depot - 3-5), Arduino (10ish), a glass vat (dollar store), and some form of a base - either a cleaned off blank circuit board - pegboard, or a few various other things that can be used, various screws, mounts, belts ect. You also need resin. Monoprice has some of the better priced resins at like 40-50 per vile. Amazon has some cheaper ones too. As you can see, the price is already starting to go up.

\n\n

Making a SLA printer, the basic idea is that you are hooking up the projector to be controlled by the Arduino. The Arduino you are loading open source (freely available code online) for SLA printers. The Arduino will control the projector itself, using the light to cure the resin as the build plate moves. The threaded rod you attach to the Nema 17 motor to move the build plate up and down. The plate will need to dunk the bottom side into the glass vat - the projector actually cures your print upside down - connecting to the build plate! As the build plate moves up, it will pull your print up with it to. After building, you will need to finish curing your print - you take it and cure it in UV light, or just set it outside in sunlight for a few hours. SLA can get some super high quality prints.

\n\n

Check out this tutorial. http://www.buildyourownsla.com/forum/viewtopic.php?f=8&t=2768

\n\n
\n\n

FDM

\n\n

This is the type I'm assuming you are thinking about - the type where the extruder (print head) moves, and the build plate sometimes moves, and is sometimes heated. You print with rolls of plastic.

\n\n

There are a lot of open source designs out there, one of the most popular designs is the i3 style. This is the style of the Ender 3 that I recommended above. It might not be the -best- design, but it is certainly popular! The \"best\" is highly subjective though.

\n\n

For this, you need to have some sort of building material. You can use anything from Legos, plywood, plexiglass, metal, ect. Although you can use just about anything to build with, metal via 2020 rods (50ish+) are going to be some of your better, more stable options. You need a build plate. You can go cheap and use an unheated metal plate, mirror, glass, table... really anything flat - plus the addition of some blue painters tape for adhesion. You need some way to have endstops - either endstop switches - which cost a few bucks a piece (let's say 10 for 4 switches), you will need a good handful of screws, bolts, nuts, t-slot nuts, brackets - you get the picture. This can be cheap to pricey quick. (let's say 20ish for random number). You will need (if building i3 style) two threaded rods (10) or if you are going xyz style 4 (20). You will need Nema 17 motors - i3 style 3-4 motors(15-80) - xyz 2-5 motors (10-100), an arduino board (can go cheap from 20 all the way to amazing boards such as the duet wifi 2 which is around 200), a PSU (get a good one here 12v at least 40amps - switchable psu), you need some mosfet boards 1 for your extruder and one for your bed if you are using a heated buildplate. Speaking of heated buildplates, if you use one don't make your own. It's safer to buy one and they can run cheap. You will also need a lot of wire, solder, shrink wrap... I'm sure you're getting the picture on that there. Basic microcontroller and electronics work tools and accessory parts there including various sensors and whatnot of the like. Extruder gear, you can build your own - but until you are really familiar with it, just go buy one. You can pick up an entire extruder assembly, off brand knock off for around 20.

\n\n

The probably hands down best way to learn to build your own 3D printer is to get your hands dirty with a pre-built or a kit, start taking it apart, and really learning how they work rather than just following a guide to build your own. There is so many resources out there that are only 'half complete' or 'half correct' that without foundation knowledge - it becomes pretty easy to burn your house down. Not to mention, building your own is almost always more expensive than a kit.

\n" }, { "Id": "4308", "CreationDate": "2017-06-29T20:54:06.170", "Body": "

I was adjusting the position of our projector to allow for bigger prints to be generated. I encountered a problem that at larger distances, our prints are not seeming to adhere as desired.

\n\n

Does the inverse square law of light apply to DLP projectors?

\n\n

I am aware that the inverse-square law only is 100% applicable when you have a point as your light source. But I assume close to the same holds constant for other real world light sources.

\n\n

Unless DLP projectors compensate for distance with their power output. Is this the case?

\n\n
\n\n

Cross-Post: @ Physics.SE

\n", "Title": "Does the Inverse-Square law apply with SLA Printing?", "Tags": "|print-quality|sla|", "Answer": "

Yes, of course it does. The same amount of light is being spread across a wider area, so there's less light per area. Once you're past a few 10's of filament diameters, a point source is a highly accurate representation of most light bulbs. Even more so when there's a lens setup that causes the light to go through a point focus.

\n" }, { "Id": "4310", "CreationDate": "2017-06-29T22:37:19.573", "Body": "

I know nothing about 3D printing and I was wondering if it is a good candidate for what I want to make.

\n\n

I want to make a custom game cartridge which looks like this:\n\"enter

\n\n

Basically it's like a SD card in a custom shell. Now I can produce the inside as a thin PCB (0.6mm-1mm). But I was wondering what the best (and cheapest) way to prototype (and maybe make a small run production) the outer shell would be. The entire cart is about 2mm thick, so each half of the shell would be at most ~0.6mm thick.

\n\n

Is this something I can do with a typical 3D printer? How would I \"attach\" the two halves together?

\n", "Title": "3D printing for storage card shell?", "Tags": "|rapid-prototyping|", "Answer": "

I print such parts by embedding the PCB directly in the print during the printing process: after the bottom part has been printed, there's a print pause gcode, the printer stops and beeps, I insert the PCB into a recess in the print, then the printer resumes when I press a button. I attach the bottom of the PCB to the print with a drop of a quick setting superglue so that it won't have a chance of getting loose and crashing into the nozzle as the next layer is printed. I use a 0.2mm nozzle to print SD card shells directly over PCB without any issue.

\n" }, { "Id": "4315", "CreationDate": "2017-06-30T06:35:09.927", "Body": "

As far as I know resin trays have a Teflon coat that allows prints to stick to the build plate easier than the resin tray but this Teflon coat wears over time.

\n\n

I am new to the SLA scene and am currently troubleshooting a Draken Facture and trying to hone in my setting but my print keep sticking to the bottom of the resin vat.

\n\n

How often should these trays be swapped out to allow for smooth printing?

\n", "Title": "What is the lifespan of a SLA Resin Tray?", "Tags": "|maintenance|sla|", "Answer": "

It depends on tray and resin type you are using.

\n\n

PDMS

\n\n

If you are using PDMS (eg. sylgard 184) coating for your tray. (B9 and similar printers using this type of tray).

\n\n

Life of tray PDMS coating depends mainly on:

\n\n\n\n

You could get 2 3 prints up to 15 20 prints.

\n\n

It is suitable for printing delicate pieces.

\n\n

FEP

\n\n

Large number of manufacturers using FEP.

\n\n

Life of tray largely depends on.

\n\n\n\n

Usually you could use same tray for couple of hundreds of prints. \nOptical quality is not comparable to other alternatives but without human error factor you could get almost unlimited prints. Delicate pieces require tough resin.

\n\n

Teflon

\n\n

It is halfway between PDMS and FEP.

\n\n

There are other tray alternatives:

\n\n\n" }, { "Id": "4330", "CreationDate": "2017-07-02T20:18:50.810", "Body": "

My Anet A8 frame are broken. I find frame project AM8 - Metal Frame for Anet A8. I like it but I can't find aluminum extrusion needed, like this: MiSUMi - Aluminum Extrusion - 5 series, Base 20, 20mm x 40mm.

\n\n

Maybe somebody knows where I can buy it in Ukraine? Or maybe another frame options?

\n", "Title": "Anet A8 frame replacement", "Tags": "|diy-3d-printer|anet-a8|hardware|", "Answer": "

Not precisely cheap, but if you look on aliExpress for AM8 3D Printer Extrusion Metal Frame you will find kits build precisely for moving Anet A8 to aluminium frame

\n" }, { "Id": "4332", "CreationDate": "2017-07-03T15:49:00.163", "Body": "

I've gone on quite a few sites (thingiverse, grabcad, etc) in search for a coiled tube, but I have yet to find anything suitable. There are a few coils (\"springs\") but no coiled tubes (i.e. the springs are hollow). Maybe my searching hasn't been good enough! But I was wondering if this is a limitation to 3D printing models?

\n", "Title": "Is it possible in any 3D printing software to create a coiled tube?", "Tags": "|3d-design|", "Answer": "

I managed to do it in the end using AutoDesk 123D.

\n\n

\"enter

\n\n

Did it by making two coils of differing radius, then subtracting the smaller from the larger. I made each coil using the instructions found here:

\n\n

\r\n \r\n

\n\n

So there we go...

\n\n

Printed it out just now without supports - in the orientation shown - and it came out fine. Used a brim, though (don't want it rolling away!)

\n" }, { "Id": "4337", "CreationDate": "2017-07-04T16:20:08.520", "Body": "

I just designed a robot in blender, and whenever I print it, it goes horribly wrong. The first print was laying down, I had supports on, Then it started to print the arm, starting with the bottom of a sphere, and the supports caused it to fly off. The second time, I printed it standing up, with supports, and the arm didn't even print, PLA got all over my build plate. How would I print this successfully? Sideways like this? \"maybe?\"

\n\n

I uploaded the model to Thingiverse last night, Here's the link: https://www.thingiverse.com/thing:2417504

\n", "Title": "Print not coming out good", "Tags": "|3d-models|ultimaker-cura|", "Answer": "

Based on your description of the print failures, it appears that the problem is not specific to the model. If printing supports is causing parts of the model to \"fly off,\" there may be a temperature problem or a speed problem or a combination of the two. If you have a new filament type, it may be necessary to perform test model prints to ensure you are using good settings for the filament. If you are using old filament, it may be necessary to print test models with no complex shapes to ensure that your filament will still function properly.

\n\n

You say that \"PLA got all over your build plate.\" This is another indication of a problem not related to the model position or model design or orientation.

\n" }, { "Id": "4345", "CreationDate": "2017-07-06T05:48:16.157", "Body": "

I have a question. I have a dual extruder printer and I want to try using HIPS for printing. Can I use PLA as my main material and HIPS for supports?

\n\n

Searching online I found combinations of ABS + HIPS mentioned. No where is it mentioned that It can be used.

\n", "Title": "Using HIPS as support for PLA printing?", "Tags": "|pla|support-material|", "Answer": "

You can as PLA is not affected by Limonene (the chemical used to dissolve the HIPS) but it is not recommended as the print temperatures for each material (~180C for PLA and ~230C for HIPS) are quite far apart and the PLA may not stick to the HIPS.

\n\n

A better choice would be PVA which prints ~185C and dissolves in water but this material has its own set of fun issues.

\n\n

*Temperatures are estimates, different vendors may have different temperatures.

\n" }, { "Id": "4353", "CreationDate": "2017-07-11T01:08:27.473", "Body": "

I have been working with our SLA printer (Facture Draken) for a couple weeks now printing in makerjuice waxcast. . I have had some successful prints, but the majority (80%) end up as pancakes stuck to the bottom of the resin tray. Some others break in half mid print.

\n\n

I have experimented with laying my models (round circular diks) flat on the build plate, but most of the times I place them on supports which stem from a square baseplate.

\n\n

It seems clear to me that there is allot of force being put on the models while curing on the bottom of the resin tray. I have noticed that printing objects with a larger projection area are more likely to stick in the vat. I assume this is because of the increased contact with the tray. Usually it seems ~10 layers are built before my baseplate breaks away.

\n\n

Things I have tried

\n\n\n\n

Anyone with experience got more suggestions on how to continue troubleshooting?

\n", "Title": "SLA prints not sticking to build plate", "Tags": "|build-plate|sla|troubleshooting|", "Answer": "

I have a plain smooth aluminium build plate and it seemed that nearly all problems with adhesion which I had were caused by the plate not being cleaned well enough with IPA and a paper towel, after removing (allegedly) the rests of the previous model with a metal scraper. The new model was peeling off only in places where the old model was previously attached. Now I clean the plate thoroughly, tightly pressing the towel, and there are no more peel-off problems with any typical resin. I have even reduced the typical exposure times of bottom layers because otherwise the model might be attached too strongly.

\n

Then, there are special resins which can be more difficult to work with. I have one resin for ultra-resistant prints which seems very sensitive to the plate alignment or other inaccuracies at lower temperatures. If I use that resin, I warm the build plate up to about 35\u00a0\u00b0C (95\u00a0\u00b0F) with a hairdryer and unless there is not a large alignment problem, the printouts stick very well. The built plate is, however, trapezoidal and rather massive, so that it keeps the temperature for a necessary period of time.

\n

The sensitivity to plate alignment leads to another subject: check the plate angles if you didn't do so. The angles should be well adjusted by allowing the plate to rest slightly pressed to the calibration pad with the plate's screw loose before tightening it again and adjusting the height.

\n" }, { "Id": "4365", "CreationDate": "2017-07-12T19:45:26.720", "Body": "

So I was trying to print 3DBenchy on my new Wanhao i3+. PLA plastic, basic high quality settings in Cura (I guess 0.06 mm layer, 40 mm/s speed, 50c bed temp, 195c extruder temp). I didn't use any tape, just regular wanhao printer mat, which was installed on printer bed from the box.

\n\n

At some moment when I looked I found things fouled up: object came off from the mat and sticked to the extruder and was moving with it. \nIt printed like 3-4 mm of the object (about 50 layers) before this happened.

\n\n

I would like to know what are the main reasons this can happen? And what can I do to prevent this? I think it's very sad when you lose many hour of work.

\n", "Title": "Object got unstuck from printing mat", "Tags": "|pla|print-quality|troubleshooting|", "Answer": "

You have to secure your print to the build plate.

\n\n

One thing that I always do is to use a brim in cura. prints a single layer which extends in all direction away from your object, adding extra adhesion between your object and the print bed.

\n\n

You can also use a slower extrusion speed for your initial layer but I would use this after trouble shooting everything else since you will be adding time to each print.

\n\n

If you are using a heated bed make sure that the temperature is set for the correct material. For PLA i think that is 40C-60C.

\n\n

We put down a layer of craft/school elmers stick glue before most prints. This helps a bit with adhering to the build plate, and also seems to allow the prints to pop off a bit easier.

\n\n

Also be sure to double check you Z calibration and your bed level. If there are inconsistencies and your nozzel moves to far away from you bed, you will be extruding in air and not getting and stickage at all.

\n" }, { "Id": "4380", "CreationDate": "2017-07-16T02:05:36.300", "Body": "

I have a TronXY printer (i3 Clone). It has a 220x220 mm heated aluminum bed and I print with a Borosilicate glass plate.

\n\n

I have a slightly longer print (245 mm) I would like to do and I think I could adjust to settings and end stop to stretch the y-dimension travel and I have found a 229x257 mm plate. This would extend over the edge of the aluminum bed.

\n\n

Will the thermal conduction and mass of the glass plate be sufficient to still keep the bed warm enough?

\n", "Title": "Extending TronXY print bed", "Tags": "|heated-bed|tronxy-x3|", "Answer": "

After Fred's answer, I realized there was a way to test out how much the temperature would drop by offsetting my current glass base to extend past the exge then using a FLIR IR Camera to see what the effect would be.

\n\n

Here are the results:

\n\n

The the glass plate does seem act a bit as an insulator causing the print surface to be about 1\u00b0C cooler. Here are two photos showing that.

\n\n

\"enter\n\"enter

\n\n

As Fred proposed, where the glass extends past the base, the temperature drops pretty quickly. It is still warmer than ambient; but is definitely not \"Hot\".

\n\n

\"enter

\n" }, { "Id": "4389", "CreationDate": "2017-07-17T00:55:10.760", "Body": "

I've got a webcam (Logitech ProductID_2470, according to my Mac) attached to my Octopi running on a RPi 3B.

\n\n

I am able to get the video on the Octoprint display page, but the lag is huge: over 50 seconds.

\n\n

How can I diagnose the problem and reduce this lag time?

\n", "Title": "Octoprint: Reducing Camera Lag Time?", "Tags": "|octoprint|", "Answer": "

This was due to a slow Wifi connection. It was fast enough for the low bandwidth of printer commands and status reports, but the video feed was overwhelming the connection.

\n\n

Adding a Wifi repeater increased the bandwidth and fixed the issue.

\n" }, { "Id": "4411", "CreationDate": "2017-07-22T18:26:18.357", "Body": "

How do I speed up prints for the Monoprice Select IIIP Plus printer?

\n\n

The manual shows [Cura] examples of:

\n\n\n\n

However, this doesn\u2019t line up with their advertisements online of a 150mm/s printing speed.

\n\n

Are there better settings to use, especially ones which can speed up printing time? Or are there any other measures which I can take in order to reduce printing time in general?

\n", "Title": "Monoprice Select IIIP Plus - Best speed settings and speeding up prints", "Tags": "|ultimaker-cura|speed|", "Answer": "

I was using Cura's default settings for a Prusa I3 on my MonoPrice Select V2 (model #13860), and got horrible results frequently. Then I used the settings you list, and got very nice results. Compare the below images for the bottom layer of 4 benchys, with adhesion brim.

\n\n

I'm using PLA, 0.4mm nozzle, 60C for bed, 200C for extruder, 1.75mm filament from Hatchbox.

\n\n

Default Cura Settings\n\"Using

\n\n

Listed Settings\n\"Using

\n\n

Settings breakdown

\n\n
Setting            Cura default    Recommended\nPrint Speed        60              50\nOuter Wall Speed   30              15\nInner Wall Speed   60              30\nTop/Bottom Speed   30              20\nTravel Speed       120             80\n
\n\n

Symptoms of my printer being told to print too fast:

\n\n

Material would not adhere properly to the print bed, and would start making clumps. These would rest either on the bed, or on the nozzle itself. The ones on the bed would grow taller than layer height, making the next pass of the nozzle bump against it, further depositing material on it. The clumps on the nozzle would drop at other points, leaving stringing filament all over, and further compromising the next pass of the head.

\n\n

This might seem like a bed adhesion problem, but the prints were very well attached to the bed; I had to apply a lot of force to remove a 15*15cm square from the bed.

\n\n

Increasing the hot-end temperature seemed to help; at some point we were printing at 230C, well beyond Hatchbox' extrusion temperature range (range is listed as 180C-210C for this PLA batch).

\n\n

Ultimately, reducing print speed to the settings listed helped us increase print quality back to acceptable levels.

\n" }, { "Id": "4414", "CreationDate": "2017-07-24T00:42:20.090", "Body": "

I have some older resin that got a bit thick. I was thinking I could use some paint thinner to thin it a bit and restore it. My thoughts are the paint thinner is made of terpene oils so should be compatible with the resin.

\n\n

To test, I went ahead and mixed a half capful of paint thinner with about a cup of resin. I mixed it thoroughly and it seemed to mix well and restored proper viscosity. I then filtered it through a paper coffee filter twice. The results were very nice and smooth resin. I did a UV hardening test with a few drops on a strip and into the UV lamp. It still hardens.

\n\n

Any thoughts, suggestions or OMGs! before I try this in my printer? It's a Pegasus Touch SLA.

\n", "Title": "How to thin UV sensitive resin", "Tags": "|sla|uv-printer|", "Answer": "

You've covered a couple of concerns \"outside the box,\" but the real tests will come when you pour some of the resin into the vat.

\n\n

If you don't get a reaction with the release layer, score one for your side. The release layer is a silicone kind of compound, which typically is un-reactive to most substances, but that's not to say the thinner qualifies in that respect.

\n\n

To be cautious, you could dab a bit on the corner of the vat, especially out of the usual scan area or off the build plate contact surface. If it sits without clouding the release layer, the next test would be a layer test.

\n\n

You could even remove the build plate, pour the modified resin in the vat and run a model comprised of only one layer, perhaps two. You'd be able to carefully pull the cured resin off the release layer and make a preliminary determination.

\n\n

If all is well by this point, it's time to create a model, yes?

\n\n

I hope you'll report back on your results. My Pegasus is still in the box from Kickstarter days. I'm sure my resin is well beyond the \"sell-by\" date!

\n" }, { "Id": "4416", "CreationDate": "2017-07-25T02:53:01.673", "Body": "

Cura is generating temperature commands in the wrong order. I need to swap lines 6 and 7 below so that both the printhead and bed can begin to heat at the same time.

\n\n

Is this code generation configurable in Cura?

\n\n
5 ;Generated with Cura_SteamEngine 2.5.0\n6 M190 S60    ; set bed and block\n7 M104 S215   ; set printhead, don't block\n8 M109 S215   ; set printhead, block\n
\n", "Title": "How do I modify Cura's code generation?", "Tags": "|ultimaker-cura|g-code|", "Answer": "

You can do so by placing this at the top of the start code under machine settings (tested on 2.6.1).

\n\n
M104 S{material_print_temperature}\nM190 S{material_bed_temperature}\n
\n" }, { "Id": "4427", "CreationDate": "2017-07-26T21:56:24.457", "Body": "

I'm planning my first project. I want to print an iPhone case with a flush letter on the back, but I want the letter in a different color (single extruder printer). The thing is the two colors will be on the same level. Do I create two separate models, import them to a single model, mesh the parts and somehow specify which one to start on, then just change the filament when it's done with the first section? Will the letter be loose fitting in the cut out? Or will I need to print them separately and glue them together?

\n\n

What is a good slicing software to accomplish this?

\n", "Title": "How to print a two color part with only one extruder", "Tags": "|3d-models|filament|software|slicing|multi-material|", "Answer": "

What you ask may be easier than you think. The slicing software is unimportant, generally speaking. Consider especially that your requirements fit the solution spot on.

\n\n

Take a look at this thingiverse snowman.

\n\n

\"colorful

\n\n

Each of the colors is a separate model. Each of the colors is only one layer thick (0.200 mm). Each of the colors is placed with zero overlap to the adjacent colors, as the extrusion multiplier will allow enough \"squish\" to cause a bond to its neighbor.

\n\n

You would create a model with the necessary shape for your lettering and insert the appropriate color for them. Print that model but leave it on the bed. If the bed is heated, all the better, but don't let it cool to the point of release. If it's not heated, the adhesive should hold it until project completion.

\n\n

For the main background color, your model would be normal but with a single layer subtracted at the base. The subtraction would have to be the shape of the lettering. Consider that you're creating indented or recessed lettering. Use appropriate modeling techniques to perform this action.

\n\n

Keep in mind that you'll want the lettering to be mirrored, as you're printing it \"upside down\" on the build plate.

\n\n

Swap out the filament to the main color and print.

\n\n

You are color limited only by your determination and patience, as you have to swap out filament and also have to create appropriate models with recesses to take the first layer, of which you'll also have to create models.

\n\n

I've created at least one of the snowman pictured and it came out quite good. The layman will not be as astonished as someone familiar with 3D modeling and printing, however. You get a better appreciation when you know what goes into the project to get this result.

\n\n

EDIT to add more info, rather than fill the comments:

\n\n

The above method works only because the inlay is a single layer deep. If your slicer supports changing layer thickness during a print, you could print the first layer at the max for your nozzle, say 0.3 mm then set layer 2 and upwards to the resolution you desire for the majority of your print. I use Simplify3D which supports such actions. I'm not sure of the others.

\n\n

Relevant to the questions posed in the comments, your modeling software may be a factor in this process. The modeling software will create/assign a pivot point or some other name to a location of reference in the model. The slicer will use that to position the model on the bed.

\n\n

Here's a concept I would use to perform this task. Start with your single layer lettering, ensuring that the thickness of the letters will match your first layer height. Save the model as an STL file. Leave the lettering on the screen in your modeling software.

\n\n

Create or import the case file. Position it as you wish relative to the letters. Of course, both models will be flat on the build plane or all bets are off. Use the lettering in a boolean subtraction to remove the shapes from the phone case model.

\n\n

If the above does not remove the lettering model, delete that and you have the recess. Save this model in STL.

\n\n

When you load the models in the slicer, they should drop into the same location relative to each other.

\n\n

When you print the letters, they will be on layer one. When you print the remaining model, the cut-out portion will be on layer one, while the rest is placed atop the two color portion. The print head will move up one layer to adjust for the extra depth.

\n\n

If you require the letters to be thicker than a single layer, one of the options you have is also in your question. Create the letters in the thickness you wish, use them to create the recess in the case and print separately. In so doing, you will be counting on the printer's ability to make clean bridges. The letters may fit cleanly, may have some play or slop, or be too large to fit easily. It would behoove you to make a test piece or two, perhaps a simple slab with one or two letters to see how that concept works in the real world. I was pleasantly surprised to see how well the public library's genuine Prusa i3 MK2S performed a challenging bridging operation.

\n\n

I apologize for overlooking the reference to first project. With that in mind, I would consider that this is a slightly advanced project. Your modeling software and modeling software skills will obviously come into play here and is a separate subject/question/topic for discussion.

\n" }, { "Id": "4429", "CreationDate": "2017-07-27T18:52:11.410", "Body": "

This is my first time building a 3D printer (a \"Creality Ender-4\").

\n\n

Everything is going fine except the \"extruder kit\" part that does not have enough space to attach on the frame. Should I drill it to have a longer hole so it can be attached to the frame? I just want another set of eyes to look at it to make sure I'm not crazy.

\n\n

\"Extruder

\n\n

\"Extruder\n\"Extruder

\n", "Title": "Creality 3D Ender-4, a \"extruder kit\" does not fit the frame", "Tags": "|diy-3d-printer|printer-building|creality-ender-4|", "Answer": "

Dude, you're absolutely not crazy, I almost put the entire thing back in the box when I realized I'd have to void the warranty just to finish putting together my printer but I wound up boring out the holes a bit closer to the edges of the extruder and assembling it all the same, it works wonderfully now that I have it attached.

\n" }, { "Id": "4432", "CreationDate": "2017-07-28T09:23:58.777", "Body": "

I'm planning to make an LCD Resin printer. I'm still learning and I found out that I need daylight resin (which hardens when white light comes from the LCD).

\n\n

If I search for \"Daylight resin\" on Google, the only \"useful\" result I find is for Photocentric.

\n\n

\"Photocentric

\n\n

Photocentric sells Daylight resins and the prices seem to be good too; however, I'm trying to search for alternatives. I'm not interested in something cheaper, I'd like to choose a resin which has more colors. Photocentric's hard resin only comes in cream/green/gray colors. I would need at least white/black/transparent resin.

\n\n

Is it the only option we have? Are there any other manufacturer out there which produces Daylight resin/LCD-hardening resin with a variety of colors?

\n", "Title": "Which choices do we have on Daylight Resins?", "Tags": "|diy-3d-printer|sla|", "Answer": "

Daylight resin choice is extremely limited. And would not get much better at-least in short term. Majority of new LCD based 3D printers are using UV LED.

\n\n

Checkout Wanhao D7, Phrozen3d and other newly released 3D printers.

\n\n

I strongly advice you to checkout Ionel Ciobanuc progress on his printer.

\n\n

He starts by making the first LCD based 3D printer actually works! to modify his printer to make it another UV LCD 3D Printer.

\n\n

From the progress it is very clear why everyone adding UV LED to their LCD 3D printers.

\n" }, { "Id": "4449", "CreationDate": "2017-08-02T00:26:23.020", "Body": "

I have almost 0 experience in 3d modeling and I'm looking for a simple way to color a model.

\n\n

I want to replicate this paint job of a model.

\n\n

I've tried Blender, tried Mudbox and I don't have the money for Mari which is the software being used in the video.

\n\n

The issue with both is that it seems to take forever to try and paint them. It's a high-poly model with a bunch of vertices. Both programs seize up and hang and I'm forced to quit them.

\n\n

How can I simply and efficiently color the model to get it ready for sandstone color printing?

\n", "Title": "How to \"paint\" a mesh on a stl/obj", "Tags": "|3d-design|software|", "Answer": "

If you've looked into using Blender, you would have discovered that that program is free. It is also one of the better suited programs available for the task you've selected.

\n\n

It's possible that your search terms may not have provided the best results for your objective. The task you describe is known as UV mapping, also known as adding texture to mesh, adding texture to a model. The generic term \"texture\" can be confused, as many people would consider texture to be the feel of a surface. Rough, smooth, slippery, etc., do not apply here.

\n\n

Texture in the 3D modeling world applies to a surface description specific to visibility. In your task, texture refers directly to color.

\n\n

Consider to search using the above terms or follow this lead:\nAdd texture to model Blender

\n\n

Doing so provided a few quite well written links. One of the first to appear is an Instructable.

\n\n

The above Instructable also links to a video, which is also a good location for learning basics for Blender and texturing models.

\n\n

The above search also results in finding an SE entry directly related to Blender:\nStackExchange Blender texture Q/A

\n\n

Consider that you can create a flat image using your preferred graphics editor, placing shapes and colors as needed and then \"project\" or paste or plaster or project that image on the surface of the model in a controlled manner. This would be easier and faster than attempting to color manually individual triangles of the model's mesh. The SE link summarizes this process, although additional research is indicated.

\n\n

Practice with Blender on simplified models and you should be able to grasp the necessary skills in short order.

\n" }, { "Id": "4457", "CreationDate": "2017-08-04T06:41:29.940", "Body": "

I have an ANET A2 Prusa - which I've setup and performed a few prints on and they have various problems with the quality. I'm after some specific experience on what the flow of filament should look like or if my decription triggers someone

\n\n

I've been adjusting settings - In particular the temperature - as the filament seemed too fluid as I could easily cause a large spurt of molten plastic by manually pushing the filament with very little effort.

\n\n

So I reduced the head temperature to 195 and all seemed better

\n\n

However after a time - I noticed on a longer print that the feeding was sometimes failing with the filament jerking back as the feeder slipped off it

\n\n

The stepper did not appear to slip back just the gear skipped on the filament\nIt appeared to cause a problem in the print with a few of the lines being missing before it started extruding normally again

\n\n

I increased the temperature back to 200 - however the issue continued intermittently however I left it and when I returned the head had become blocked with the final part of the succesful print consisting of very thin hair like extrusions and eventually stopping completely

\n\n

The head is flooded and I need to clean it out

\n\n

So my questions are - how runny should the filament look when the head is at the correct temperatures, is the extremely runny filament I saw at 200 obviously too hot - or is that normal or at least have people seen it looking like that when successfully printing?

\n\n

Initially , before I reduced the temperature, I doubled the skirt and that seem to make a good enough print

\n\n

Does anyone have experience of why it can seem to be printing but then slowly start failing until the head becomes blocked?

\n", "Title": "Filament not feeding then eventually blocked", "Tags": "|filament|extruder|print-material|", "Answer": "

From my experience with few printer, Sometime it's happen because of the filament. \nSolution-\nIf your printer have unload function or if you don't have that funstion select a printerable file and change temperature to 205-210 C and try to use long and thin hex key like this to push the filament out.\n-Make sure you extruder motor work accordingly to feed the nozzle.

\n\n

\"

\n" }, { "Id": "4460", "CreationDate": "2017-08-04T14:34:58.773", "Body": "

I play a berimbau for Capoeira. One of the most fragile (and most expensive) bits is the caba\u00e7a, a hollow gourd used as a resonator.

\n\n

\"caba\u00e7as\"

\n\n

I'm not very familiar with the qualities of the resin used for 3d printing. If I were to take this to our local Maker Lab and have them scan and print a copy, how likely is it that it would work? My fear is that the plastic would be too sound deadening.

\n\n

If you want a less exotic parallel, imagine the body of a guitar. That's a resonating chamber.

\n", "Title": "Is it possible to print a resonator for a musical instrument?", "Tags": "|print-material|quality|", "Answer": "

Following up on this, the answer is, yes, this works quite well. I printed this caba\u00e7a model from Thingiverse using PLA on a Lulzbot Mini and put it on my berimbau today to test it out. I can't make a direct comparison because the printed resonator is smaller than the gourd one that I own, but the sound is good. I am not certain whether it is actually cheaper (it used a fair amount of PLA because it's probably about a half-inch thick, and it took a few tries to get a good print due to it taking about 9 hours and needing to be monitored for the filament breaking), but it may be more accessible for people who can't ship a gourd in from Brazil. It is definitely more durable.

\n\n

\"Exterior (click to enlarge)

\n\n

\"Interior (click to enlarge)

\n" }, { "Id": "4471", "CreationDate": "2017-08-08T18:59:07.597", "Body": "

I have a mesh of a bowl that has the perfect shape of half a sphere. I want to easily convert it to the containing sphere solid and a box solid that will be subtracted from it.

\n\n

\"Conversion

\n\n

Googling mesh to solid shows that in various tools such as 3ds Max, Fusion, etc., manual approximation of where the sphere might go is created manually by visually comparing to the mesh or the cross section when creating the solid but I am looking for the minimum enclosing sphere and box to be generated/calculated by the software.

\n\n

Source file format is of course not an issue, it can be any known mesh file.

\n", "Title": "Converting a mesh to constructive solid geometry", "Tags": "|3d-models|cad|", "Answer": "

The answers by @markshancock and @fred-dot-u describe the process of creating a CAD primitive sphere and subtracting a cube; this is a trivial task in most any 3D CAD program. You could manually measure the mesh to determine its dimensions, and frankly, I would recommend this approach. (If you have a large task like converting 100,000 half-sphere meshes, I'd re-think the approach and go to the source that generated the meshes in the first place!)

\n\n

However, these don't address converting the mesh to a primitive, which is normally not possible for arbitrary meshes (therefore a CAD program will not have such a function), and much more difficult than simply measuring and re-creating.

\n\n

You assert that your mesh file \"has the perfect shape of half a sphere\". A mesh is basically on a collection of vertices, joined by edges and faces. The vertices are defined by numeric coordinates, and in a computer will always have some rounding errors, and the linear edges and planar faces are most definitely not round. Therefore, it may be incredibly close, but it will most definitely not be perfect.

\n\n

In this very particular case, the diameter of the sphere is the same as the diameter of the circular face of your half-sphere. You could plausibly write a simple program that would locate the maximum distance between any two vertices in your mesh and use that for the diameter. The midpoint of the line between those two points would be the center of the sphere. This should produce a good result but may not be \"exact\" depending on the details of the mesh.

\n\n

I won't try to explain how to create the actual program as that is not in the scope of 3D printing.

\n" }, { "Id": "4472", "CreationDate": "2017-08-08T20:51:19.497", "Body": "

I am curious about the algorithm/principles behind the estimates that the slicing softwares provide. Is there a standard technique behind this and how accurate is it ?

\n", "Title": "How is the print time of an object to be printed estimated?", "Tags": "|slicing|g-code|", "Answer": "

Much of the software used in 3D printing is open-source, and so are some slicers. Cura, for instance, does (or did, this source code is from an older branch) its print time estimation in gcodeInterpreter.py.

\n\n

The relevant portion of the source code is (simplified and with many lines removed for clarity):

\n\n
    totalMoveTimeMinute = 0.0\n    pos = util3d.Vector3()\n\n    for line in gcodeFile:\n        G = self.getCodeInt(line, 'G')\n        if G is not None:\n            if G == 0 or G == 1:    #Move\n                x = self.getCodeFloat(line, 'X')\n                y = self.getCodeFloat(line, 'Y')\n                z = self.getCodeFloat(line, 'Z')\n                e = self.getCodeFloat(line, 'E')\n                f = self.getCodeFloat(line, 'F')\n                oldPos = pos.copy()\n                pos.x = x\n                pos.y = y\n                pos.z = z\n                feedrate = f\n                currentE = e\n\n                totalMoveTimeMinute += (oldPos - pos).vsize() / feedRate\n
\n\n

As you can see, (this version of) Cura simply:

\n\n\n\n

and does not take into account:

\n\n\n\n

The accuracy of this estimate can be arbitrarily bad if the feedrate is set to some unrealistic value.

\n\n

Newer versions of Cura use a much more advanced time estimate method, and it can be found in timeEstimate.cpp. It is much more complicated, and actually takes jerk/acceleration/deceleration into account. It is much more accurate.

\n\n

We know exactly how 3D (open source) 3D printer firmwares work, so estimating print time is as easy as simulating execution of the G-code by your given firmware. There is no reason you can't get a really good estimation (if you take into account all of the intricacies of your given firmware's acceleration/deceleration techniques) but writing the code for it is rather involved.

\n" }, { "Id": "4484", "CreationDate": "2017-08-13T01:31:20.750", "Body": "

On my Mac I've got two versions of Cura installed, in /Applications/Cura250 and /Applications/Cura262.

\n\n

How can I copy my printer and profile settings from Cura 2.5 to Cura 2.6?

\n", "Title": "How do I upgrade (copy settings) from Cura 2.5 to Cura 2.6?", "Tags": "|ultimaker-cura|", "Answer": "

From the Ultimaker Forum:

\n\n\n" }, { "Id": "4487", "CreationDate": "2017-08-13T15:21:25.787", "Body": "

For personal usage, indoor, I'm doing some experiments with following lamp (v0.1):

\n\n

\"enter

\n\n

Lamp is a led bulb enclosed in a methacrylate tube and with a 3D printed finish at the top using PLA (my first 3d print ;-).

\n\n

In some www pages (by example, here) I've read that the led buld radiator can reach 90\u00ba C. Experimentally, the methacrylate and the PLA feels only slightly hot, I suppose around 40\u00baC.

\n\n

According to www info, PLA has a melting point of 150\u00baC, far from this usage, but a continuous operative temperature of only 40\u00baC ( !? this made 3d printing unable in a country as my own one, where ambient in summer is around 40\u00baC).

\n\n

So, my question, is PLA a valid material for this application? If not, some other one better?

\n\n

Thanks a lot.

\n", "Title": "PLA continuous operative temperature", "Tags": "|pla|print-material|", "Answer": "

EvilTeach's answer is correct, ABS is a more reliable plastic for any kind of work which may get above what feels \"hot to the touch.\"

\n\n

Just to elaborate on the why: the property you're looking for in the thermoplastic (which will determine the continuous operating temperature) is glass transition temperature. This is the point at which the plastic begins to flow, and becomes deformable as EvilTeach described. PLA reaches this state at around 60 \u00b0C, whereas ABS is around 105 \u00b0C, just suiting your specifications. To go a bit further, polycarbonate offers a glass transition temperature of around 150 \u00b0C, and Ultem at 217 \u00b0C. So there's a thermoplastic for everyone, you just need to know what you're looking for!

\n" }, { "Id": "4489", "CreationDate": "2017-08-14T00:36:39.433", "Body": "

I just got a Monoprice MP Mini Delta from an Indiegogo Campaign, but it came with no printed docs.

\n\n

How do I get started?

\n", "Title": "Monoprice MP Mini Delta - How to get started?", "Tags": "|monoprice-mini-delta|", "Answer": "

Updated Manual

\n\n

Resources and sample files

\n\n

Common Problems and Solutions

\n

The following are some problems I had and their solution

\n

Can't print custom models

\n

If you find you can print prebuilt gcodes fine (cat,viking,toothless), but can't seem to print any other models without the print head ramming into the bottom of the printer and going off to the side, you probably haven't setup the autoleveling gcode that's required. You can add something like the following line to your software, in a "startup" gcode section. This is detailed in the manual (just search for G29), and more info can be found here

\n
; auto-levels the build plate with a overall vertical \n; offset of 0.3mm with a center offset of -0.8mm\nG29 C-0.8 Z0.3 \n
\n

First layer doesnt stick

\n

If you find your first layer isn't sticking, or it seems like the plastic is balling up, you need to adjust your startup gcode line mentioned above , so it has a lower Z offset, like Z0.25 for example

\n
G29 Z[offset] ; raises\nG29 Z-[offset] ; lowers\n
\nNotes\n\n" }, { "Id": "4496", "CreationDate": "2017-08-15T09:28:01.347", "Body": "

I am experiencing a minor layer separation when printing a body for a tipping-bucket rain gauge, which is basically a hollow tube with thin walls (3 mm). I am using Prusa I3 MK2 and a Fillamentum ABS white plastic. The model has been sliced in Siplify 3D with the following settings:

\n\n\n\n

Is it possible to prevent the irregular and layer separation by adjusting some of the settings, and not significantly increasing the printing time, which is already 13 hours?

\n\n

PICTURES:

\n\n

\"enter

\n\n

\"enter

\n", "Title": "How to prevent layer separation, when printing tall vase-like (tube) objects?", "Tags": "|prusa-i3|abs|simplify3d|", "Answer": "

increasing the temperature by 5 to 10\u00b0 as well as, increasing the layer thickness to \"0.3\" or \"0.35\" may assist ,additionally increasing the flow rate of the filament, by a small margin at most 10%. These options should improve the quality of the print as they improve the layer adhesion, however none of these options will completely solve the problem.

\n\n

alternatively you can repair the current print. In order to do this you will need to use something cot such as a soldering iron with a variable temperature, to melt and rejoined the layers. similarly you can use a 3d pen to apply additional plastic across the sections of separation in order to repair it.

\n" }, { "Id": "4508", "CreationDate": "2017-08-18T22:17:44.267", "Body": "

I am wondering if anyone can help me achieve a good 3d print using polypropylene. I am trying to print custom insoles for shoes and I'm getting some bad warping/lifting (see image).

\n\n

\"example

\n\n

Some details about the print and process:

\n\n\n\n

The print takes about 5 hours and seems to stay down for an hour or two before starting to warp.

\n\n

Does anyone have any suggestions of things I should try?

\n", "Title": "Help preventing warping while 3d printing with polypropylene", "Tags": "|filament|print-quality|warping|pp|", "Answer": "

One thing that seemed to help is making sure the heated bed was 100C. I was able to do this on my Prusa by turning off the front fan (for the whole print). If I have both the fan on and the heated bed at 100C I get the \"Heatbed Thermal Runaway\" error.

\n\n

This produced a print that was better but still a bit warped.

\n" }, { "Id": "4509", "CreationDate": "2017-08-19T13:27:50.473", "Body": "

before a mesh can be printed it needs to be 'repaired'. According to most tutorials that is including making the shape close and removing inner shapes. Reading through B-REP in Wikipedia there is no layman term or explanation how it differs form the traditional polygonal mesh create for example from scanning or exporting from CAD. Is B-REP a mesh that only represents a boundary and therefore ready to be printed?

\n", "Title": "Is B-REP a ready to print mesh", "Tags": "|3d-models|cad|repair|", "Answer": "

a) about

\n\n
\n

Is B-REP a mesh that only represents a boundary [..] ?

\n
\n\n

B-Rep contains more information, see here:

\n\n
\n

There are two types of information in a B-rep: topological and\n geometric. Topological information provide the relationships among\n vertices, edges and faces similar to that used in a wireframe model.\n In addition to connectivity, topological information also include\n orientation of edges and faces. Geometric information are usually\n equations of the edges and faces.

\n
\n\n

b) about

\n\n
\n

and therefore ready to be printed

\n
\n\n

it is need not only check that the model is correct (no lack of a face, no wrong face normals, ...) but also that is valid for a 3d printer (no \"floating\" parts, addition of supports, ...).

\n\n

Finally, recall the model is not what drives the printer, the printer is controlled by the machine instructions, usually gcode. Thus, the usual evolution is: parts description (.scad, ...), model format (.stl, ...) and machine instructions (.gcode).

\n" }, { "Id": "4530", "CreationDate": "2017-08-22T19:32:11.643", "Body": "

I print my ABS at:

\n\n\n\n

When printing ABS, I place an aluminum foil lined cardboard box over my printer to help keep the ambient temps up for less warping and stronger prints. I've never actually measured the temperature inside, but the cardboard box insulates very well.

\n\n

I get this weird kind of tearing in my prints, I'm not sure if it's from too large of gaps in my infill, too fast print speeds, or not enough top layers.

\n\n

\"Torn

\n\n

Another guess is some kind of drooping because of the high ambient temps.

\n\n

The tearing only occurs on large top layer surfaces.

\n", "Title": "Weird ripping and warping of ABS print", "Tags": "|print-quality|abs|desktop-printer|heat-management|", "Answer": "

First of all, don't specify the extrusion width, the slicing software does the calculation for the optimal value by itself: if you watch this video about extrusion width and you cross the data with this tweet you will see that the standard value is already optimal.

\n\n

Second, if you put the printer in a box for insulation you should redo the temperature calibration tower to find out which temperature produces the best results.

\n\n

Once you do both you'll find the problem will disappear.

\n" }, { "Id": "4533", "CreationDate": "2017-08-22T23:31:18.417", "Body": "

I'm interested in printing small machine parts (gears, linkages, structural components) so I'm looking for accuracy and mechanical strength over speed and volume.

\n\n

I'm also somewhat concerned about harmful emissions so would like a solution with some sort of filtration, whether it's built into the machine or something added. I'm thinking I will run the machine in an unventilated garage, which is quite warm and humid during the summer in Texas.

\n\n

My price range is \\$1500-\\$2000 USD. I've looked at several options but I didn't really come across any scenarios like I've described and would like some advice from the experts before committing.

\n\n

Anyone in a similar boat have any suggestions?

\n", "Title": "Printer/Material/Setup recommendation for printing mechanical parts", "Tags": "|print-quality|print-material|ventilation|", "Answer": "

There is more than one question here. Which touches three topics: the choice of appropriate 3D printing technique, the choice of materials, and control of any hazardous, noxious, or annoying out-gassing.

\n\n

It is important to understand the limits of the 3D printing process you will choose. Each process has its own limits on repeatability and resolution of small features, such as gear teeth. A 10 pitch (teeth-per-inch) gear could print acceptably well with a low-cost printer, but the size of a 10-tooth gear would be 1/2\" diameter. This could be a small part, or a huge part, depending on the use. To print an 1/8\" diameter 10-tooth gear would require a 100 pitch gear likely would not, and would call for an optical process (laser sintering (SLS) or photo-polymerization (like the FormLabs Form 2)) or a dot-jetting process (such as the StrataSys Objet Connex machines). These are not in the budget you've suggested, but are available through service bureaus.

\n\n

Answers here another community question give some hints for using a plastic-extrusion machine, and information about SLS.

\n\n

Each process has different environmental requirements, and impacts, but first should be to understand the requirements of the parts, based on those choose a process and material, and then mitigate the environmental factors.

\n" }, { "Id": "4536", "CreationDate": "2017-08-24T00:37:58.567", "Body": "

Today I installed an inductive sensor (4 mm sensing distance, NPN) on my printer to perform auto-bed leveling. The sensor works at 12 V and the board at 5 V, so I used a voltage divider (as suggested in many places online) using a 10kOhm and a 15kOhm resistor.

\n\n

In testing the sensor, I noticed that the sensor's LED turns on, but the printer (Anet A8) doesn't recognize the fact that the sensor is triggering. I'm running the Skynet3D firmware, though I still have not switched to the version with auto bed-leveling. Regardless, the inductive sensor should still act as a limit switch, yet the Z-Axis motor does not stop when the sensor triggers. Am I doing something wrong, or missing a step?

\n\n

I have also measured the voltage across the leads that connect to the board and the voltage is slightly over 5 V when the sensor is not triggered, and lowers to around 2.5 V when the sensor triggers. I get the feeling it should be closer to 0 V.

\n\n

Thank you very much for any help.

\n", "Title": "Auto Bed Levelling - Printer not detecting, but sensor is triggering", "Tags": "|electronics|", "Answer": "

Perhaps the culprit is a pull-up resistor on the board. Normally, endstops on 3D printers use the microcontroller's internal pullups. These have a resistance of around 50k\u03a9, which is far too high to be a problem. However, if lower value pull up resistors are used on your main board, this could cause a problem.

\n\n

The resistor of your voltage divider form, when the output of the sensor is low, a parallel pair of resistors to ground, with an effective resistance equal to 1/(1/10+1/15) = 6k\u03a9. If there was (let's say) a 4.7k pull-up resistor on the board, you'd expect to see around 2.8V on the output (because the pull-up resistor, together with the two resistors of your voltage divider, forms another voltage divider).

\n\n

I don't have the Anet A8 main board myself, but on pictures I do see a set of 6 resistors suspiciously close to the thermistor and endstop connectors.

\n\n

You could verify my suspicions by unplugging the endstop, powering down the electronics and then measuring the resistance between the endstop signal and 5V pins.

\n\n

Possible solutions:

\n\n\n\n

With this last solution, make sure the reverse leakage current of the diode is not too high. If it has a reverse current of (let's say) 50uA, then 50uA flowing through the (supposed 4.7k) pull-up to ground would raise the voltage at the signal pin to 5.002V. This is unlikely to be a problem, but with higher value resistors or higher leakage you'd see the voltage raise higher above 5V (which the microcontroller won't like).

\n" }, { "Id": "4552", "CreationDate": "2017-08-26T13:50:56.717", "Body": "

I have been happily printing with my Original Prusa i3 MK2S for a few weeks now. I have been thinking about replacing the PINDA probe with a BLTouch tactile sensor.

\n\n

If I do so, how will I be able to do the XYZ calibration? I believe that the tactile sensor would only be able to do Z calibration. If I need to redo the XYZ calibration in the future for some reason, is it possible to do manually? Or would I need to remount an extruder with a PINDA probe temporarily?

\n", "Title": "XYZ calibration without PINDA probe", "Tags": "|prusa-i3|calibration|z-probe|", "Answer": "

If you want to replace the PINDA probe (whatever reason for) then you can go with the BLTouch Sensor but only for certain operations.

\n\n

It will be good for leveling the bed before printing because here, only the bed level is important and not the skew of the bed.

\n\n

If it comes to calibration of the skew itself, the BLTouch Sensor will not be usable because the PINDA Probe detects the boundaries of the copper circles on the print bed. The BLTouch cannot detect these copper areas.

\n\n

The PINDA Probe is a proximity sensor.

\n\n

If you want to do a recalibration you have to mount the PINDA again.

\n\n

Therefore, it is possible but not recommended.

\n" }, { "Id": "4553", "CreationDate": "2017-08-26T14:39:26.787", "Body": "

I am thinking about rewiring the extruder heater cartridge. What is the connector on the Rambo Mini? I'm thinking about putting a similar connector close to the extruder to make replacement easy. I tried looking through the schematic but could find the part number for the connector (I believe on page 4?).

\n\n

\"enter

\n", "Title": "Heater cartridge connector for Original Prusa i3 MK2S", "Tags": "|prusa-i3|electronics|prusa-i3-rework|wire-type|", "Answer": "

This is referred to as a Terminal Block Connector. More specifically this is a 2-position pluggable terminal block connector commonly manufactured by Phoenix Contact and others.

\n\n

Newark.com Sale Page: Pluggable Terminal Block, 5.08 mm, 2 Positions, 24 AWG, 12 AWG, 2.5 mm\u00b2, Screw

\n\n

Larger Picture: 2-Position Terminal Block Connectors

\n" }, { "Id": "4556", "CreationDate": "2017-08-27T10:16:41.047", "Body": "

can anyone explain in the simplest terms please what is the difference between a point cloud and a voxel mesh?

\n", "Title": "Layman term explanation of the difference between voxel and point cloud", "Tags": "|file-formats|scanning|", "Answer": "

A point cloud is often derived by sampling. Each point represents an observation. Sometimes, a point cloud is turned into a surface by fitting triangles to the points in the form of an STL file.

\n\n

A raster is a 2D grid of pixels. It divides the area of an image into constant-sized little squares. Each of these squares has a value.

\n\n

A 3D raster is made of voxels. It divides 3-space into constant-sized little cubes. Each of these cubes has a value.

\n\n

Pixels and voxels are rendering techniques. A point cloud is a sampling technique.

\n\n

The Wikipedia article, https://en.wikipedia.org/wiki/Voxel, is helpful.

\n\n

In a real system the pixels may not be square or the voxels not strictly cubic, but in every system I've worked with, they do form a regular tiling of the plane for pixels, and fill 3d space for voxels.

\n" }, { "Id": "4575", "CreationDate": "2017-08-31T23:17:18.843", "Body": "

I'm getting this printing where it's not laying the plastic down very well. What could be causing this? I've printed with these settings before, and it turned out just fine. If you need any other info to properly diagnose this, let me know.

\n\n

I'm using a Robo3D R1+

\n\n

[\"Bad

\n", "Title": "What could be causing this spotty extrusion?", "Tags": "|print-quality|", "Answer": "

I had this problem also. After a few hours of troubleshooting and changing the extruder temp, printing speed, layer height, and infill density. I realized that I had set the PLA diameter at 2.85 mm when it was 1.75 mm. Changing that fixed all my problems.

\n" }, { "Id": "4590", "CreationDate": "2017-09-06T08:25:23.983", "Body": "

I am having some problems with bed adhesion and curling of the edges. Also my printer causes a lot of stringing when creating the first layer. This can cause the print to let go of the bed and then it just drags around with the nozzle.

\n

I have a Anet A8 and I tried multiple different amounts of force pressing down on the paper. However, what is the desired amount of force pushing down on the paper? Is it supposed to move freely, or is it supposed to be quite hard to move around under the nozzle?

\n

I'm using basic filament from Gearbest: Gearbest Filament.

\n

\"Perfect

\n", "Title": "Correct amount of force on piece of paper when leveling bed", "Tags": "|print-quality|heated-bed|extrusion|adhesion|", "Answer": "

Actual task in your case is to find correct nozzle height for the first layer, so the force for pulling out the paper is not important. It just has to be the same every time you make calibration. Because of the different force you apply, different paper thickness and other factors, found position can only be treated as 'relative'.

\n\n

To complete the calibration you will have to make several test prints with different initial heights, starting from the lowest, adding 0.1-0.2mm (depending on the nozzle diameter) before you find the right value.

\n\n

Please note that 1) initial layer height is not the only factor, affecting adhesion 2) some slicer programs may add not the same amount of height that you specify

\n" }, { "Id": "4593", "CreationDate": "2017-09-07T07:21:49.740", "Body": "

My printer is kossel (delta 3d printer).

\n\n

I have a probe far away from the nozzle (offset -x45 y17),every time I run G29, the result is unacceptable, the z distance between points is over 0.5mm, it's not because of the probe's accuracy, I test repeatedly the result is the same.

\n\n

I cost much time level the bed, finally I realize maybe it's because not only my bed, but also my z max endstops are not horizontal.

\n\n

My question is

\n\n
    \n
  1. how to level the z endstops? (now I think both bed and z endstops are not horizontal)
  2. \n
  3. is my analyzation correct? any other possible?
  4. \n
\n", "Title": "how to level kossel's endstops?", "Tags": "|kossel|endstop|", "Answer": "

First, you should make sure that printing surface is adjusted correctly: it must be perpendicular to all three delta columns, all other adjustable parts must be checked and adjusted if necessary (depending on actual printer design).

\n\n

Assuming that upper end-stops can be adjusted as well.

\n\n\n\n

This procedure should make your endstops properly adjusted relative to the printing bed.

\n" }, { "Id": "4594", "CreationDate": "2017-09-07T13:53:14.260", "Body": "

I am currently looking to buy a 3D printer. I've done some research about which technologies and materials they use. I've seen some models seems good so far but my concern is 'Are they mechanically solid?'. I don't want it to become a scrap after some usage. Since there are a lot of enthusiasts here that use 3D printer regularly, I would like to get use of your experience with 3D printers and their producer brands.

\n\n

I am not buying it for hobby. I'll mostly use it to make cases for circuit boards.

\n", "Title": "What should I consider before buying a 3D printer?", "Tags": "|print-quality|desktop-printer|hardware|", "Answer": "

Not surprisingly, your question is a difficult one to pin down in a precise manner. I'm going to pull one line from your post as the focus for my answer.

\n\n
\n

I would like to get use of your experience with 3D printers and their producer brands

\n
\n\n

I would suggest that you isolate a few models (or more) that hold your interest. A good example of a popular, quality printer is the Prusa i3 MK2s, although some would consider it to be expensive. It's available in kit form, as well as fully assembled, another topic entirely.

\n\n

Find the forums specific to the printer models you've selected and read as many posts as you can. Look for user postings describing problems and suggested solutions. Look for user postings describing modifications which implies a problem that had to be solved by the user or community before the printer performed satisfactorily.

\n\n

I've assisted in the building of a Prusa i3 MK2s recently and am currently assisting in a home-spun design based on a number of other generic printers. The owner of this model is incorporating as many modifications as he can find. That tells me that this model, although not yet completed, is likely to be a handful to tune and operate to his/my satisfaction.

\n\n

On the flip side, everything I've read about the Prusa told me that for the public library for which it was destined, it would be the lowest level of trouble for any model I researched. I was asked to make a recommendation and performed the suggested research via all the forums I could find.

\n\n

The above sounds like a direct recommendation, and it could be, but you have not provided too many specifics. The Prusa i3 MK2s (genuine, not clone) would likely print well for your circuit board cases, but there certainly would be other models to do the same job well enough.

\n\n

Watch out for print bed size, based on your board size requirements. Heated beds usually mean more money, but greater variety of materials can be used as a result.

\n\n

If you plan to limit your use specifically to circuit board cases, you may not have to be concerned about \"expandability\" from a feature standpoint. Single extruder models are fine for almost all 3D printing, but it's handy sometimes to have dual nozzle capacity. That's another plus for the Prusa i3 MK2s with an add-on kit, but not a big deal for case building, I think.

\n\n

Look in the various forums that provide general support for 3D printing (such as this one) for other posts from people who use Brand X 3D printer and have had a problem with, for example, bed adhesion, filament quality, layer shifts, nozzle clogging, just about anything.

\n\n

As with many things, high quality usually means higher price. The library Prusa has been zero problems for me to support on a volunteer basis, hence my recommendation. The makerspace also enjoyed the building process by buying the kit, saving US$200 in the process and learning more about the printer.

\n\n

Keep an eye out for users who have experienced broken components or typically troublesome components. You would not want to buy a printer model that ships with a junky hot-end assembly if the suggested modification is to upgrade to an E3D V6 to make it work properly. A user who posts about a continuing belt breakage (rare!) could have a user-induced problem or a printer design problem, which means you'll want to cull out some of the information you see.

\n\n

Good luck, it's a tough decision, to be sure.

\n" }, { "Id": "4597", "CreationDate": "2017-09-07T19:45:51.667", "Body": "

I've recently purchased Anet A8. First print went well, but now, the hot end temperature stays at ~230 \u00b0C. When I touch it, it is still cold. I tried the thermistor, and it correctly changes resistance when I blow on it (from 100 kΩ to ~70 kΩ). The voltage in connector is about 300 mV.

\n\n

Does anybody know what could be wrong?

\n", "Title": "Anet A8 - hot end temperature still same", "Tags": "|printer-building|hotend|anet-a8|", "Answer": "

In my case, I had a bad resistor on my board. R41 had only 2.2 kΩ instead of 4.7. I replaced it and everything works now.

\n" }, { "Id": "4609", "CreationDate": "2017-09-12T01:14:27.913", "Body": "

I like using \"Vase Mode\" (or single outline corkscrew printing mode) for quick nonfunctional prints, but it tends to leave gaps in horizontal or near horizontal surfaces. I understand why it does this, but is there a way (beyond printing with no infill*) to get a little more horizontal coverage out of it?

\n\n

*Is the answer to this literally just \"Try to print without infill?\"

\n", "Title": "Is there a way to get more horizontal coverage out of \"Vase Mode\" in Simplify3D?", "Tags": "|print-quality|simplify3d|", "Answer": "

Version 4 allows you to have multi-process vase mode prints -- do the vertical surfaces in vase mode, then switch over to normal mode for the horizontals. This lets you have the best of both worlds in the same print. :)

\n" }, { "Id": "4615", "CreationDate": "2017-09-12T16:15:10.160", "Body": "

Here's the model I'm trying to print, Truncated Icosahedron

\n

\"Truncated

\n

But it must be 12" in diameter and gray in color rgb = #444444

\n

I also want two of the hexagons to be yellow in color: rgb = #FFFF00

\n

Similar to this photo:

\n

\"Coloured

\n

How to do this? I don't seem to be able to edit the shape on shapeways.

\n
\n

UPDATE

\n

Got it.

\n

skeleton bucky ball

\n", "Title": "How can I make this model?", "Tags": "|3d-models|", "Answer": "

Without any restrictions to your methodology, your objective is easily accomplished. Thingiverse has a truncated icosahedron model available for download and printing.

\n\n

\"icosahedron\"

\n\n

Because the creator has included in the download the OpenSCAD source files, you could edit it to meet your bed limitations. I've downloaded the source, opened it and found the model is a single hexagon, until you change a parameter from 6 to 5 to get a pentagon. I suspect the assembly is up to the builder to figure out.

\n\n

It appears that the default is for a 50 mm diameter sphere. Change the 50 in the code to 300 or so to reach your 12\" desired diameter.

\n\n

An alternative to friction welding would be just about any decent epoxy or even a 3D printing pen such as the 3Doodler.

\n" }, { "Id": "4619", "CreationDate": "2017-09-13T15:08:12.017", "Body": "

I am having issue with my new printer it is not printing circles correctly although I have change firmware and stepping of motor\nadvise if anyone know the solution.\n\"enter

\n", "Title": "3d printer not printing circles correctly", "Tags": "|print-quality|marlin|desktop-printer|diy-3d-printer|", "Answer": "

Possible causes for the printer not printing correct dimensions:

\n\n\n\n

Looking at the picture, I would go for the first case, because distortion looks regular. Try checking microstep settings on your board, and settings in the firmware.

\n" }, { "Id": "4632", "CreationDate": "2017-09-16T14:40:53.563", "Body": "

I've been trying to get something decent printed for days but nothing works! I have a Tarantula Tevo i3 MKS Base V1.4 and have done a lot of trial & plenty of error. Still I am puzzled to get good prints.

\n\n
    \n
  1. What is the stock firmware for a single extruder regular/large bed firmware & how to configure a large bed (if needed to be configured)?
  2. \n
  3. Which is the auto bed leveling firmware?
  4. \n
\n\n

I need help sorting out what's out there. I did not manage to configure a large bed with a single extruder. But did manage to restore firmware with Marlin-2.0.x

\n\n
    \n
  1. So the Tevo 3D Printing Store firmware link directs to a dropbox - only dual extruders - both regular & large bed,
  2. \n
  3. There is JimBrown's GitHub MarlinTarantula - Optimized firmware for RepRap 3D printers based on the Arduino platform,
  4. \n
  5. JoelLisenby's GitHub TEVO-Tarantula-I3-Marlin-Firmware.
  6. \n
\n\n

I followed this, YouTube - Setting Up Auto Bed Leveling (Tevo Tarantula), for setting up the auto bed leveling sensor but it just got me messed-up even more, see Tevo Tarantula incorrectly auto leveling of bed. I'm just now in the process of trying to manually level the bed and I broke the hot end holder plastic plate...

\n\n
\n\n

EDIT:\nThe sensor I'm using is SN04-N Inductive Proximity Sensor - 5mm

\n", "Title": "Tevo Tarantula I3 firmware", "Tags": "|marlin|heated-bed|firmware|tevo-tarantula|", "Answer": "

Tevo Tarantula i3 owners on FaceBook is where I got marlin 2.0. Other things I tried didn't work, but this is working fine if you just want to add auto bed leveling using the standard SN04 sensor.

\n" }, { "Id": "4637", "CreationDate": "2017-09-18T17:57:17.290", "Body": "

I have make a little test with 4 dots aligned with A tower, B and C tower. Distance W and S are the same in the stl but not in the print. I have tried diferent values of diagonal root but S always is smaller than W, and all S are equal (more or less 38.20mm) and all W are equal (more or less 40.80). I expect that W and S will be 40mm. How can fix this problem?

\n\n

\"enter

\n\n

Update:

\n\n

Here is the stl I use: https://www.dropbox.com/s/2vwjbo387cmk5qa/DeltaCalibration%20v15.stl?dl=0

\n\n

Update:\nI have replaced the steper motor in tower B but same result.

\n", "Title": "Delta printer distortion", "Tags": "|print-quality|calibration|delta|repetier|", "Answer": "

I bet your towers are not standing straight (vertical) or\nyour bed is not clearly horizontal

\n

I've recreated your picture with some assumptions (for example that your SW calculates properly and your steppers and motors act well).

\n

Take a look here:\n\"enter

\n

If you deliver your printer dimensions

\n\n

I can calculate what the inclination angle on all towers is, but, I suppose it's not really important.

\n

The important thing is to set them straight/vertical (perpendicular to the bed).

\n

We can see from the picture that tower A is the most inclined to the center or\nthe bed highest point is next to tower A (and I bet one of those or both cause the issue).

\n

As an example, I've made some calculations based on imagined assuptions of the tower height\nhere are details:

\n
    towerH | inclination\n-----------+----------------\n    300 mm | 0.11\u00b0\n    400 mm | 0.08\u00b0\n    500 mm | 0.06\u00b0\n
\n

It seems to be quite small but in fact your differences in dimensions are also small!

\n

The inclination of tower B is bigger as there is bigger difference in dimensions, so maybe the issue is more in bed "horizontality".

\n

It would be good if you would check and measure these parameters.

\n

As for the explanation why bed the inclination causes dimension distortion:

\n

\"enter

\n

This is exaggerated but it's just to show the issue.

\n" }, { "Id": "4644", "CreationDate": "2017-09-22T08:38:02.017", "Body": "

I have an STL-file that Slic3r thinks has errors. They are not visible in the 3D view. I have had them anazlyzed in both Blender and netfabb. Both of these programs say that the model is good. I don't want to leave this to chance. Since I sell STL-files I need the STL-file perfect.

\n\n

Is there any way I can find out what the problem is. I encounter this from time to time. Often I can go back into blender and find the error by analyzing the mesh. But not always. It would be very helpful to have slic3r tell me what it repaired.

\n", "Title": "How can I see what errors Slic3r have repaird?", "Tags": "|3d-models|slic3r|", "Answer": "

Slic3r uses ADMesh internally to validate and fix mesh. You could try to use ADMesh directly to see a limited information about what was changed. Note that Slic3r bundles it's own copy of ADMesh and depending on your Slic3r version and edition, the behavior of it's ADMesh might slightly differ from the standalone one. (For example Slic3r Prusa Editon patches it's own ADMesh very heavily.)

\n\n

Using ADMesh CLI:

\n\n
$ admesh cube_bad.stl \nADMesh version 0.98.2, Copyright (C) 1995, 1996 Anthony D. Martin\nADMesh comes with NO WARRANTY.  This is free software, and you are welcome to\nredistribute it under certain conditions.  See the file COPYING for details.\nOpening cube_bad.stl\nChecking exact...\nChecking nearby. Tolerance= 1.000000 Iteration=1 of 2...  Fixed 0 edges.\nChecking nearby. Tolerance= 1.000173 Iteration=2 of 2...  Fixed 0 edges.\nRemoving unconnected facets...\nFilling holes...\nChecking normal directions...\nChecking normal values...\nCalculating volume...\nVerifying neighbors...\n\n================= Results produced by ADMesh version 0.98.2 ================\nInput file         : cube_bad.stl\nFile type          : ASCII STL file\nHeader             : solid cube (repaired)\n============== Size ==============\nMin X =  0.000000, Max X =  1.000000\nMin Y =  0.000000, Max Y =  1.000000\nMin Z =  0.000000, Max Z =  1.000000\n========= Facet Status ========== Original ============ Final ====\nNumber of facets                 :    12                  12\nFacets with 1 disconnected edge  :     3                   0\nFacets with 2 disconnected edges :     0                   0\nFacets with 3 disconnected edges :     1                   0\nTotal disconnected facets        :     4                   0\n=== Processing Statistics ===     ===== Other Statistics =====\nNumber of parts       :     1        Volume   :  1.000000\nDegenerate facets     :     0\nEdges fixed           :     0\nFacets removed        :     1\nFacets added          :     1\nFacets reversed       :     2\nBackwards edges       :     0\nNormals fixed         :     2\n
\n\n

The statistics should give you some idea about what happened.

\n\n

Using ADMeshGUI:

\n\n

Find ADMeshGUI at github.com/admesh/ADMeshGUI.

\n\n

Open the file and click the REPAIR button in bottom right. See the changes.

\n\n

\"ADMeshGUI\"

\n" }, { "Id": "4664", "CreationDate": "2017-09-27T21:50:05.010", "Body": "

I have had a 3d printer for a while now, and I have a lot of the quality settings dialed in pretty well, but one thing that constantly bugs me is removing the raft from my finished prints. I am using Repetier and I have set the air gap to 0.2 mm. That led to much better results than the default 0, which were impossible to remove at all, but it is still not great. Are there any settings I should look at changing to get easier to remove rafts? Does the filament affect this? I am printing in Hatchbox PLA at high temps. I have a heated bed, and reducing the temp on that did seem to help. Maybe it keeps the layers on the raft from fusing with the layers on the part? Any help is greatly appreciated. Thanks.

\n", "Title": "How should I go about printing easily removable rafts?", "Tags": "|repetier|post-processing|rafts|repetier-host|", "Answer": "

I have been able to get rafts that peel off by editing the g-code after the raft has finished and adding an M104 set temperature command telling my printer to cool the nozzle down to about 40 degrees and then another M104 command to tell the nozzle to heat back up again. This gives the raft enough time to cool and then the raft doesn't weld to the rest of the print. If you are using a heated bed I would suggest trying the M190 command and turning that off after the raft and then back on after the first layer as well.

\n" }, { "Id": "4666", "CreationDate": "2017-09-28T21:44:19.737", "Body": "

Apologies, I'm a EE designer and software guy. We've been CNC'ing prototypes, and my office just bought a very cheap 3D printer.

\n\n

I'm using Cura as recomended, and wanted to print a piece that has features on both sides.

\n\n

Here is a screenshot of each side.

\n\n

So if you laid one side flat, you see how there is a subtractive portion underneath it?

\n\n

Is there a way to 3D print an object like this, and keep the details on each side?\"enter

\n\n

UPDATE\nI copied some Cura settings from guys and basically tipped this thing to a 45 degree. Here are the results. Pretty good! The finish has some zits and pops, but the surface details are quite accurate enough to fit a PCB board in there with confidence.

\n\n

\"enter

\n", "Title": "Printed 2 Sided Object?", "Tags": "|print-quality|", "Answer": "

You can print it laying flat on the bed with supports. It will be very laborious to clean up, but it will probably work. supports usually are thin enough to strip away, but they leave marks which you need to cut off to get a clean result.

\n\n

You can also buy a printer with dual extruders and then use dissolvable supports. That would probably be easier to clean up and provide a cleaner result.

\n\n

If you're using PLA to print, you might get away with turning up the part cooling fan to the maximum level and hope for the best. Overhangs are very much prone to dropping though. I'm not sure if a stock MP Select mini will get the job done. You might want to consider an upgrade to the part cooling fan. You can look for fan duct related upgrades which allow you to mount bigger fans on Thingiverse

\n\n

With PETG you can possibly get away with longer overhangs, as it cools quicker. However, PETG is much harder to print well, as it's much more prone to stringing, which can cause issues like artifacts and clogging.

\n\n

All in all I'd start out with getting a good benchy before you start on engineering projects. This will make sure that you have your printer calibrated to perform for your chosen filament. Every brand and type has their own quirks and differences, so you'll have to fine tune your settings to get the best result.

\n\n

A CNC-like finish will not be easy to achieve. A CNC type finish will be unachievable without extensive post processing if you're using supports. In any case, getting a good finish requires some practice with finding the right settings for your filament.

\n\n

Learn how to print and to get your printer dialled in to achieve the required result, which, in the case of it having to be CNC-like, is quite high-end. The fact you're laying down filament means that you'll be able to see lines, even at top quality. Maybe this guide will help you get on your way.

\n\n

Getting a $1000 printer mainly inproves reliability of the print. The quality of the print is in the skill of dialling in the right settings in your slicer for a given filament choice, plus recognising the type of supports necessary. This takes practice to achieve.

\n" }, { "Id": "4681", "CreationDate": "2017-10-01T01:36:14.353", "Body": "

I am new to Fusion 360 and I think I'm going straight to something complicated. Is there a way to make a nose cone for a model rocket? What tools would one you to accomplish this?

\n", "Title": "Make a nose cone in Fusion 360", "Tags": "|3d-design|", "Answer": "

Try model8ng the rocket into the workspace, and then you can extruder a nose cone from the top using a 30 degree angle

\n" }, { "Id": "4692", "CreationDate": "2017-10-03T04:11:02.003", "Body": "

I'm having a frustrating problem with my recent built custom 3D printer\nso every single print I made, from the third layer, the nozzle is "rubbing" on the already placed filament. This results in a complete mess, melting the previous layers with the nozzle / new extrusions coming, getting malformed and out of precision forms, if I leave this happening, my Y-axis motor (bed) start to jump steps (by the force of nozzle friction to cold material). To stop that symptom, at the beginning of this, I have to raise the Z-axis by hand turning about 1/8 rotation of T8 fuse. By doing this, every rest of my printing runs peaceful, nicely, and beautiful.

\n

I'm using Marlin firmware, the most recent version, and Ultimaker Cura. My nozzle size is 0.5 and I'm using Ultimaker Cura's Fine Preset (0.1\u00a0mm height)

\n

My stepper motors axis are very well calibrated (X, Y, Z and Extruder). I tried:

\n\n

but nothing seems to solve my problems.

\n

Due to my lack of experience, I don't know what I could try to solve this frustrating issue. I already check and rechecked my mechanical structure and everything was fine solid and very well balanced and square.

\n

\"Photo

\n

\"Photo

\n

From comment:

\n\n

I'm thinking about over extrusion but I have fine tuned my stepper, checked and rechecked for it and seems normal

\n", "Title": "Nozzle rubs on previous layers", "Tags": "|print-quality|diy-3d-printer|calibration|z-axis|layer-height|", "Answer": "

Taken from a rejected suggested edit. If the author (Gareth) posts their own answer, this can be deleted, or flagged for deletion

\n
\n

My Ender 3 is not warped in any way but there are several issues I needed to address:

\n
    \n
  1. Extrusion:

    \n

    Check your extruder and Z eSteps for accuracy, as detailed in numerous places.\nTest layer width: print a cube in vase mode (1 shell thick) and measure wall thickness. Adjust extrusion multiplier accordingly.

    \n
  2. \n
  3. Bed Level:

    \n

    I was leveling with the '1 sheet of paper' method. I started using two sheets of paper. To compensate for reduced bed adhesion I use hairspray.

    \n
  4. \n
  5. Mechanical 1:

    \n

    I found my X carriage was slightly loose: the hotend was pulled upwards or rode upwards during some moves then grinding on subsequent layers.

    \n

    Test: Grab the rail with your left hand then push with your thumb against the top left roller. If the roller rides up the groove then the carriage is loose.

    \n

    Fix: The carriage is held firm by an eccentric on the bottom roller. Loosen the bottom wheel bolt, adjust the eccentric nut until the carriage is firm on the rail. Then back off slightly until motion is free. This is best done with the belt loose or disconnected.

    \n
  6. \n
  7. Mechanical 2: X rail sags on the right.

    \n

    I found the X rail assembly was not even height across the width. Again, the eccentric roller was loose. Test: Measure rail height on both sides.

    \n

    Fix: Adjust inside eccentric roller as above.

    \n
  8. \n
  9. Mechanical 3: Tighten belts.

    \n
  10. \n
\n

Obviously, this is brief. Check your manual or YouTube for anything you're unsure of.

\n" }, { "Id": "4694", "CreationDate": "2017-10-03T15:26:56.933", "Body": "

I ordered a MOSFET module on eBay for my 3D printer and I did not get the one in the picture but I got this:

\n\n

\"The

\n\n

If there is any way I could use this on my Anet A8 could someone tell me how, as it only has one input connector, DC in, and 12 V out.

\n", "Title": "MOSFET problems!", "Tags": "|heated-bed|diy-3d-printer|", "Answer": "

No, this module is completely useless for your intended purpose.

\n\n

The load side is marked with \"5A 5-220VDC\". This means it can only switch up to 5A, maximum. The heated bed draws more than twice this current.

\n" }, { "Id": "4706", "CreationDate": "2017-10-05T14:04:49.993", "Body": "

I would like to use nail polish to paint and smooth a PLA model. Could nail polish damage a model made out of pure PLA?

\n", "Title": "Could nail polish damage pure PLA?", "Tags": "|pla|color|smoothing|", "Answer": "

Nail polish will not damage your PLA model.

\n\n

1) Let's talk about \"gel\" nail polish. Gel polish self-levels, does not drip, is cheap, is much more health-friendly than many finishing methods, and only takes 30 seconds under a suitable strength UV/LED lamp to dry, providing a thick smooth coat (clear, color, glitter!, anything) that makes any layer lines disappear.

\n\n

2) The FDA regulates nail polish in its \"cosmetics\" category, so, not only are the ingredients known, they have to be printed on the bottle.

\n\n

3) Gel polish consists of various methacrylate monomers which undergo a radical (in the chemistry sense, although it is pretty darn cool too) polymerization process that is UV-activable. Filament is primarily polyactide, which is polymerized at room temperature. Polymerized polyactide reacts with methylacrylate at the ends of the long strands (not in the middle, which is what you would describe as \"damage\"). Methylacrylate actually makes the PLA ends less reactive (read: it makes it stronger). As an industrial material, this compound is known as \"supertoughened\" PLA.

\n\n

Hope that clears things up a bit. As an aside, if you're not familiar with the different families of nail polishes, and you want to try this, you can tell if the polish you're looking at is \"gel\" because the bottle will be opaque (keeps UV out).

\n" }, { "Id": "4735", "CreationDate": "2017-10-14T10:44:39.993", "Body": "

A while ago, I created some simple command buttons in Octoprint to help with bed levelling, by hopping to particular points on the bed. I realised that what I actually want to do is move Z up 10mm, move, and then home Z though, to avoid ploughing the nozzle along the bed if the level is badly off.

\n\n

So I did this:

\n\n
  - commands:\n    - G91\n    - G1 Z-10\n    - G90\n    - G1 X30 Y160 F9000\n    - G28 Z\n    name: Back Left\n    type: command\n
\n\n

Which should be, switch to relative move, Move -10mm Z, switch back to absolute, go to the specified point and then home Z.

\n\n

Except there's no Z move. I get the feeling that something might be optimising the moves together into one, or something similar... how can I get my printer to move up, then across, then down?

\n\n

(printer is an quite modified Anet A8 i3 clone - I think the board is Melzi-based?)

\n", "Title": "gcode commands in octoprint: Z moves ignored?", "Tags": "|g-code|octoprint|", "Answer": "
\n

what I actually want to do is move Z up 10mm

\n
\n\n

The command G1 Z-10 tells the printer to decrease the Z-axis position, i.e., move the nozzle closer to the bed. You should use G1 Z10 instead.

\n" }, { "Id": "4745", "CreationDate": "2017-10-16T14:39:31.857", "Body": "

I'm having a Prusa i3 derivative printer with a capacitive sensor for the z-axis. It switches a tiny bit before the nozzle hits the print bed and hence needs a z-offset to be configured.

\n\n

In Slic3r I have configured the z-offset to -0.1 on the General page of the Printer Settings, but currently I'm evaluating Cura and can't find such a setting. Slic3r seems to apply this setting directly to the generated z-values in the g-code, so it does not use a short version at the beginning of the g-code. My current (except of the auto-bed-leveling part default) g-code:

\n\n
G28 ;Home\nG29 ; auto-bed-leveling\nG1 Z15.0 F6000 ;Move the platform down 15mm\nG92 E0\nG1 F200 E3\nG92 E0\n
\n\n

Is there a way to configure Cura, e.g. using the Start Gcode options, to apply the z-offset?

\n", "Title": "Cura: set z-offset", "Tags": "|ultimaker-cura|g-code|", "Answer": "

I built my printer five years ago with a similar issue and I strongly suggest that the physical 0 will be the actual 0 instead of tweeking it into the system. Especially if you are exploring and testing new methods, i.e. Cura vs Slic3r settings. In Marlin, you can insert a z adjustment for the endstops but it can create future complications if you decide to upgrade the machine.

\n\n

Keep it as simple as possible, if a sensor is misplaced don't try to reverse calculate it, the pressure of the motors will nudge it and after a while you'll have the same problem...

\n" }, { "Id": "4747", "CreationDate": "2017-10-16T15:28:02.170", "Body": "

I want to draw a corner piece for a Kossel delta 3D printer in Fusion 360 for 2040 aluminum extrusions like in the picture below, but I can't find a way to actually start. I draw a 3-sided polygon and a 20x40\u00a0mm rectangle but don't know where to go from there. Do you have any suggestions?

\n

\"2D

\n", "Title": "How to draw a Kossel delta corner piece in Fusion 360?", "Tags": "|3d-design|kossel|", "Answer": "

I love me a challenge so I just hacked this together: (a half waiting for mirror)\n\"enter\nAs for strategy, I simply used the line tool to do a basic shape and then went to town with constraints and dimensions to see what I end up with. Can't fail.

\n\n

Clearly, you're missing information to constraint the sketch fully, but if you have the extrusions and the machine, you could just print a few tests to guess the missing values.

\n\n

(Like precise position of the circles, of the 20x40 rectangle...)

\n\n

You can download/ view my fusion model here: \nhttp://a360.co/2gJsdxB

\n\n

Good luck.

\n" }, { "Id": "4750", "CreationDate": "2017-10-17T07:21:56.447", "Body": "

Is it possible to use Cura Engine (the command line tool) to generate a gcode with support?

\n\n

Here is what I do now:

\n\n
CuraEngine slice -j /usr/share/cura/resources/definitions/ultimaker2.def.json -l my.stl -o my.gcode\n
\n", "Title": "Adding support when generating g-code from Cura Engine (command line)?", "Tags": "|ultimaker-cura|g-code|support-structures|", "Answer": "

I found this in the fdrmprinter.def.json which you could might make a copy of and use instead for the -j option:

\n\n
\"support\":\n    {\n        \"label\": \"Support\",\n        \"type\": \"category\",\n        \"icon\": \"category_support\",\n        \"description\": \"Support\",\n        \"children\":\n        {\n            \"support_enable\":\n            {\n                \"label\": \"Enable Support\",\n                \"description\": \"Enable support structures. These structures support$\n                \"type\": \"bool\",\n                \"default_value\": false,\n                \"settable_per_mesh\": true,\n                \"settable_per_extruder\": false\n            }\n        }\n    }\n
\n\n

Change the \"default_value\" under \"support_enable\" to true and that could work.

\n" }, { "Id": "4751", "CreationDate": "2017-10-17T19:58:04.243", "Body": "

I am building a Prusa i3 MK2 Clone and I am caught on this one. I have replaced the extruder with a MK8 adapted by myself to fit with most of the previous hardware from the E3D style extruder.

\n\n

\"Here's

\n\n

My current problem is that the nozzle appears to be dragging on the top of the layers. I can't tell if it's over extrusion or a layer height issue or what. I am testing by printing the 20mm calibration cube. The dimensions appear to be spot-on, but the layers start building up too much filament such that it just gets re-melted and shuffled around:

\n\n

\"just

\n\n

Anyone have a clue what this could be? Software, Hardware, Slic3r Settings?

\n\n

I have:\n0.5mm Nozzle Diameter.\nMK8 Direct Drive extruder.

\n\n

Anything else you want to know?

\n\n

Thanks!

\n\n

--- UPDATE ---

\n\n

Thanks everyone for your help. Turns out it was seriously over-extruding, like 3x what it should have been! I ran through this instructable: http://www.instructables.com/id/How-to-calibrate-the-Extruder-on-your-3d-Printer/ Which helped me with what I needed to do to fix the issue, but just re-flashed Marlin on the printer with the updated setting.

\n\n

Here's the result! WAY better, I still have some z-wobbling issues but i think the frame just needs more support than it has now and that should fix up the slightly jagged corners.

\n\n

\"Steps

\n", "Title": "Print Nozzle Dragging and Smearing Filament while Printing", "Tags": "|prusa-i3|pla|extruder|slic3r|prusa-i3-rework|", "Answer": "

You stated your print of a 20mm cube came out spot on so I will rule out any z-axis configuration issues. It clearly seems like you are extruding too much plastic.

\n\n

I use slic3r myself with repetier so I am familiar with the settings.

\n\n

Print Settings - Advanced

\n\n

All the values should be 0 except first layer sometimes is 100%-200% depending on your personal setup. Increasing these values may cause more filament to leave the extruder.

\n\n

Filament Settings - Diameter

\n\n

This should be set to 1.75 or 3 depending on your filament size. If you accidentally have it set to 1.75 when your filament is 3mm, it would probably be over-extruding since it pushing almost double the plastic with the same length.

\n\n

Filament Settings - Extrusion multiplier

\n\n

This should be set to 1 or slowly adjust it 0.05 increments to reduce overall amount of plastic leaving the nozzle.

\n\n

Printer Settings - Nozzle diameter

\n\n

This should be set to 0.5 since your nozzle is 0.5mm. Make sure your nozzle truly is this value.

\n\n

EDITED 10/19/17 - OP has solved problem and his own answer made me realize I made mistake on how to measure and correct extrusion steps/mm. Ignore this below and see what OP wrote in their edit.

\n\n

If all these Slic3r configurations fail or are correct and do not solve your problem, I think you may want to look into your motor step settings for the extruder. It may be too high pushing more filament than it should. To correct this, you need to do the length test where you will extrude 10mm or 50mm and measure the string of plastic. There might be some stretch so a 10mm may measure as 11mm but as long as your close it should be good. If you extrude 10mm and you measure 25mm, you need to recalculate your steps per mm.

\n\n

Also for your first layer, make sure it is not being smushed to the build plate. You want it to be flattened a little to provide good adhesion but not too close to the build plate where it would constrict the nozzle.

\n\n

Hope this helps, and good luck on your prints. May you never have to deal with thermal runaway error :)

\n" }, { "Id": "4762", "CreationDate": "2017-10-18T20:04:26.710", "Body": "

I've printed a 2x2x2 cm test cube with Slic3r (left) and Ultimaker Cura (right) and my Prusa i3 derivative machine with tight belts. The print settings should be quite equal (0.15 mm layer height, 40 mm/s outer wall speed, default accelerations/jerks). Though the top surface of the Ultimaker Cura-cube looks much better than the Slic3r-cube,

\n\n

\"Left:

\n\n

the latter has much flatter vertical walls than the first.

\n\n

\"Left:\n\"Left:

\n\n

The front wall of the Ultimaker Cura-cube was printed from left to right which could explain the wavy result as some kind of vibration caused by the sharp y-stop at the left front corner.

\n\n

But what setting could have influenced this? Should I try to manual setting of lower accelerations in Ultimaker Cura?

\n", "Title": "Wavy walls with Ultimaker Cura", "Tags": "|ultimaker-cura|", "Answer": "

It is difficult to see with the lighting and the shiny green, but try printing a more complex object, like the usual XYZ cube. Does it look like this one? which means big waves after a change of direction, but then getting smaller?

\n

\"enter

\n

If so, it is "ringing" or "ghosting", which means that the acceleration you set is too high for the mechanics of your printer and the machine vibrates too much.

\n

Does it look like this one? Which means, exactly aligned vertically, and everywhere, not only after changes of direction?

\n

\"enter

\n

Then it is NOT ringing, it is due

\n\n

I also had it and it was the belt, too tight. Loosen it, it can be much looser than you think before you get issues.

\n

Check How to fix evenly spaced vertical print pattern to get more information about the second issue.

\n

There is another potential cause for wavy walls, however I'm mentioning it for completeness, it is not the case of the question.

\n

If the waves repeat horizontally, but are not aligned vertically, and the extruder is a direct drive, it could be caused by the issue explained in this video, which shows that the dual gear of direct drives introduce periodic changes in flow, which are visible sometimes.

\n

\r\n \r\n

\n" }, { "Id": "4764", "CreationDate": "2017-10-18T22:13:55.230", "Body": "

Currently, 3D Builder is telling me \"one or more objects are invalidly defined. Click here to repair.\" What is this and what does it mean? When I click it, it totally messes up my model (I can't tell what it even did due to the orthogonal camera). Model stats:

\n\n\n\n

Screenshots:

\n\n

This is a picture of the model before repairing:\n\"This

\n\n

This is a picture of the model after repairing:\n\"This

\n\n

(Just in case you wonder. I actually have to use STL for this project)

\n", "Title": "3D Builder is Saying \"One or more objects is invalidly defined\"", "Tags": "|3d-design|", "Answer": "

SketchUp is notorious for creating non-manifold (non-3D-printable) models. You would be much better off to learn to use a different, better suited program for your purposes.

\n\n

Disregarding that aspect for the moment, your model prior to repair appears to be a general rectangular prismatic shape with dividers. Unfortunately, those dividers also appear to have zero wall thickness. One must create objects with thickness (minimum should be no lower than your nozzle diameter) in order to have that portion addressed.

\n\n

Other views of the object may confirm this.

\n\n

Back to the first reference, even a program as simple as TinkerCAD will give you SketchUp simplicity with a much smaller chance of this sort of failure. For more advanced model creation with parameters, Fusion 360 offers hobbyist free license. I'm also quite fond of OpenSCAD ( also free ) for parameter-based model creation.

\n" }, { "Id": "4770", "CreationDate": "2017-10-19T10:34:00.163", "Body": "

My processor is running at 4.70 GHz and has 8 cores. I also have 32GB RAM. I have a USB 3.0 and a USB 3.1 xHCI hub, and Nvidia 1700 graphics card.

\n\n

Can I use the Kinect v2 and the adapter to scan rooms, and objects, quickly, without trouble with the USB?

\n", "Title": "Scanning 3D with a Kinect v2 on AMD Ryzen 7?", "Tags": "|scanning|", "Answer": "

In short with the hardware you have it should be no problem. In actuality your specific setup, how you move the scanner, how stable is the scanner, what software you are using, and how you've calibrated the kinect will most likely have the largest impact on the quality, and speed in which you can scan.

\n" }, { "Id": "4771", "CreationDate": "2017-10-19T16:57:45.853", "Body": "

So there is a company called impossible objects that use CBAM technology witch the description can be found here http://impossible-objects.com/technology/ I am pretty sure that they are the only ones using this process, I want to know why. CBAM makes stronger material out of carbon fiber in a very sophisticated process stronger than FDM(Fused Deposition Modeling). This is interesting and I want to know why. I understand the process but not the pros and cons.

\n", "Title": "How is FDM Better than CBAM?", "Tags": "|fdm|carbon|", "Answer": "

Well the CBAM stuff is SUPER new, requires a lot more work done by hand (currently), and is going to be WAY expensive compared to widely used FDM machines.

\n\n

Seems to me like a carbon fiber mixed filament is a cheaper way to go, with a possible annealing process to get the tensile strength up post-print.

\n\n

Try locating a CBAM printer for $200! Maybe if the process becomes quicker, and the machines/materials cheaper to purchase, we could be using this method more in the future. I feel that we will likely advance the materials we have with FDM much faster than this other process would catch on and become affordable.

\n" }, { "Id": "4774", "CreationDate": "2017-10-20T02:19:53.487", "Body": "

My first layer is suddenly not sticking for every new print I'm trying. When it lays down the first layer it has this weird jaggedness to it, doesn't firmly stick to the plate, and gets dragged by the nozzle. I've printed previously successful prints without an issue, but any new print I export from Cura is having this issue. I'm using the same profile that I do for the previously successful ones.

\n\n

My relevant print settings are:

\n\n\n\n

\"Problem

\n", "Title": "Prusa i3 - First layer does not stick and looks jagged", "Tags": "|prusa-i3|ultimaker-cura|", "Answer": "

Your nozzle is too far from the build plate. You should make some changes to bring the nozzle closer to the build plate for the first layer. There are a number of possibilities for doing this:

\n\n
    \n
  1. Adjust the Z-axis endstop

  2. \n
  3. Loosen the bed leveling screws

  4. \n
  5. Add an offset in G-code

  6. \n
\n\n

This will squish the plastic down more, enabling it to stick to the build plate.

\n\n

With solution (3), you have to be careful that you don't crash into the endstop. This solution works best if your printer homes towards max (and not min) or if you have non-mechanical (hall or optical) endstops.

\n" }, { "Id": "4778", "CreationDate": "2017-10-21T11:01:56.663", "Body": "

I recently switched to a RAMPS 1.4 on an Arduino Mega 2560.\nEver since I have extruder temperature swings a couple of minutes into the print, but it looks like a problem reading the temp rather than actual fluctuations in the temperature (as can be seen in the attached pic).\nAlso, I've noticed that the MOSFET is getting really hot when I heat the heated bed.

\n\n

What is the problem and how can I fix it?

\n\n

\"enter

\n", "Title": "Temperature problems after switching to RAMPS 1.4", "Tags": "|ramps-1.4|", "Answer": "

\"enter\nBy my mistake 12v in heatbed Vout plug and then the temperature shows(ramps 1.6)\nPlase help me

\n" }, { "Id": "4781", "CreationDate": "2017-10-23T03:18:07.403", "Body": "

Could I 3D model and print a working airbrush in\nPLA?

\n", "Title": "Could I 3D print an airbrush?", "Tags": "|3d-models|pla|3d-design|", "Answer": "

The problem with this I see is that the PLA takes and holds paint super well. I have painted it with acrylic a lot and it works great for models you want painted after printing.

\n\n

I know next to nothing about airbrushing, but it seems to me like keeping a printed airbrush clean for re-use would be a big pain. It looks like they make the pro airbrushes with some kind of stainless steel most likely for the non-sticking purposes of re-use, which PLA just won't have.

\n\n

That being said, don't be afraid to model it and try! I would make a suggestion that you switch to ABS and then do a vapor bath on the result to smooth out the material and possibly make it easier to clean excess paint out of.

\n" }, { "Id": "4784", "CreationDate": "2017-10-23T18:13:46.473", "Body": "

So I've seen some very good design software, but almost all of it is very expensive. I'm just wondering if there's a good cheap design software out there.

\n", "Title": "Good Designing Software for cheap", "Tags": "|software|", "Answer": "

OnShape is free for students and the public and is very similar to Fusion 360. It works online on almost any device and was built by some of the engineers from Solidworks. Free accounts can not have any private documents so anyone can see your design. If you know how to use Solidworks/Inventor you will be able to pickup Fusion 360/OnShape very quickly.

\n\n

OpenSCAD is also free but works through its own programming language. It requires a strong math background and some understanding of programming to use as it has no GUI tools.

\n" }, { "Id": "4792", "CreationDate": "2017-10-25T06:50:39.283", "Body": "

How can I center a model at the middle of the printing area of the printer when creating a g-code with CuraEngine.

\n\n

Are there any parameters I can add to ultimaker2.def.json to achieve this?\nThanks.

\n", "Title": "Centering model with CuraEngine when creating g-code?", "Tags": "|ultimaker-cura|g-code|", "Answer": "

Found a solution.\nThis need to be applied under \"settings\"

\n\n
\"command_line_settings\": {\n    \"label\": \"Command Line Settings\",\n    \"description\": \"Settings which are only used if CuraEngine isn't called from the Cura frontend.\",\n    \"type\": \"category\",\n    \"enabled\": true,\n    \"children\": {\n        \"center_object\": {\n        \"description\": \"Whether to center the object on the middle of the build platform (0,0), instead of using the coordinate system in which the object was saved.\",\n            \"type\": \"bool\",\n            \"label\": \"Center object\",\n            \"default_value\": true,\n            \"enabled\": true\n        }\n    }\n}\n
\n" }, { "Id": "4804", "CreationDate": "2017-10-25T18:31:24.267", "Body": "

I am re-writing this question because, well, it needs to be updated.

\n\n

I have the Anet A6, but in a general sense of things, what kind of threads can I produce before it no longer works?

\n", "Title": "Printing threads", "Tags": "|print-quality|", "Answer": "

I don't have a printer like yours but it should not matter.

\n

Once you follow the calibration steps listed at https://github.com/AndrewEllis93/Print-Tuning-Guide/blob/main/articles/extrusion_multiplier.md and you set the slicer to 0.1 mm, you should be able to print working threads at the first attempt.

\n

I printed in PLA a thread (both screw and nut) with a 1.5 mm pitch and it worked immediately (it was a bit hard to turn, it got better after using it few times).

\n

A G 1/2 thread (pipe thread) in ASA (only nut) also worked immediately.

\n

I had more issues with deep threads: M30x1.5 worked immediately, but I couldn't get M30x3 mm to work. Basically printing threads with pitch from 0.8 to 1.5 mm should be fine if the axis is vertical.

\n

0.1 mm layer height is important to keep overhangs small, see video

\n

\r\n \r\n

\n

The nozzle is not so important, you can use 0.4-0.6 without issues. I used 0.4 mm.

\n" }, { "Id": "4807", "CreationDate": "2017-10-25T21:35:22.117", "Body": "

Cura is slicing my object (a gear with text) with a partial top layer, and I believe it has to do with layer height and rounding issues. I have been able to solve this at specific heights 10-14mm using a layer height of 0.25 mm, however other heights will cause this issue to arise again. There must be a solution to this that doesn't require the user to constantly be adjusting layer height and object height in order to resolve what I perceive to be a rounding issue.

\n\n

Here is the model:\nhttps://filebin.ca/3f2g2s0dklzR/hackEly_v1.0_joined_text_2.stl

\n\n

Here are my settings:\n\"Settings\n\"Settings

\n\n

Here is the model at a height of 10 mm:\nNote the diagonal line, this is the line between two layers.\n\"Object

\n\n

Here is the model at a height of 13 mm:\nNote the diagonal line again, and how it is farther to the right than the 10mm height view.\n\"enter

\n\n

I don't believe that this is not a non-manifold issue. I have also tested this using a gear without text.

\n\n

Like I mentioned, I can get rid of that layer division at certain layer heights and object heights (e.g. 0.25mm layer height), but I would like to find out how to get rid of this issue regardless of object/layer height. Thank you for your help.

\n", "Title": "Layer height issues creating partial top layer", "Tags": "|3d-models|ultimaker-cura|slicing|", "Answer": "

Even though you've selected an answer, here's another viewpoint. Ignoring the brim that I've added by default via my slicer, you can see the first few layers are not complete. This indicates a model that's a fraction of a degree off-plane.

\n\n

\"slanted

\n\n

Any number of methods can be used to correct this. Meshmixer has a great \"align\" function in the edit menu which will drop it on the bed. My slicer, S3D will work in that respect with \"Place surface on bed\" or some such terms.

\n\n

The image below represents the result of using Align in Meshmixer, Inspector (repair all) and export. The STL file imported to S3D presented exactly the same image on the first layer as in the image above. When I used \"Place surface on bed and selected the horizontal cross-member as the reference surface, the first layer result is this:

\n\n

\"first

\n\n

The slicer completed the imaging with this result, clearly with no diagonal artifacts:

\n\n

\"last

\n" }, { "Id": "4809", "CreationDate": "2017-10-25T22:57:43.777", "Body": "

I have built a 3D printer from salvaged/purchased parts. I am using an Arduino Uno and three easy driver stepper drivers with 3 CD-ROMs drives and a PC power unit. I ordered a 3D pen and have it mounted with a transistor to switch it on/off. Everything works but when I try to run code that I got from makercam.com it seems like it wants to fill in the shape rather than build up.

\n\n

I followed this tutorial YouTube - How To Make A Cheap 3D Printer and after modifying the G-code I am unable to get any successful prints.

\n\n

\"Bad

\n", "Title": "How to modify G-code or a better place to get it", "Tags": "|g-code|", "Answer": "

I would typically shy away from downloading gcode and printing it directly. Always slice it yourself since, inevitably, every printer is different. What happens if the gcode is setup for ABS (higher temps) and you are printing with PLA? If it's setup for a build platform larger than yours and you just hit endstops?

\n\n

Definitely go with Aaron's idea, download a model (Thingiverse is great, but there are SO many sites where you can get files), configure a slicer for your print settings (I prefer slic3r to Cura, but both are great), slice it and print away.

\n" }, { "Id": "4811", "CreationDate": "2017-10-26T01:27:47.787", "Body": "

Ok so i have built a 3d printer from old cd rom drives. Before installing the 3d pen(extruder/hotend) i used www.makercam.com to export i file containing a star. With a marker attached the machone successfully drew a star. Now i have the 3d pen but using makercam i am not having any luck. Is there a better way to generate files for my new toy?

\n", "Title": "What is the best free source for generating g-code", "Tags": "|diy-3d-printer|g-code|", "Answer": "

The g code is generated by a program like slic3r, skeinforge or cura. These programs import a CAD model slilce it into layers and output the gcode required to print each layer.

\n" }, { "Id": "4819", "CreationDate": "2017-10-28T00:46:40.167", "Body": "

I have a 3D printer that I built using CD-ROM drives. It's all set and ready to go but when I generate .stl or .obj files my G-code sender program cannot load it. I have found that Slic3r will export the .stl into G-code but it has the option of changing the G-code flavour, or firmware.

\n\n

My machine is running from an Arduino Uno with Grbl v0.8.

\n\n

So the question is, which firmware setting would be appropriate for my machine that would require the least amount of editing before I can print?

\n", "Title": "Which firmware to use on printer settings for DIY printer", "Tags": "|diy-3d-printer|slic3r|", "Answer": "

Slice your model... then edit the code in Notepad... where you will see that you will need to replace all E (extrude commands) with M3 for Extrude on and/or M5 for Extrude off.

\n\n

You must look at the code and figure out where:

\n\n\n\n

I did this a year ago and got mine to work perfectly but it takes time to edit the code. Be patient and keep at it .

\n" }, { "Id": "4824", "CreationDate": "2017-10-29T01:00:22.167", "Body": "

I am writing some G-code for my DIY 3D printer. From what I understand, G4 is dwell and its expressed in milliseconds. So my extruder takes about 30 seconds to heat up. Do I just type

\n\n
G04 30000\n
\n", "Title": "How to express dwell time in G-code", "Tags": "|g-code|", "Answer": "

Depending on your G-code flavor you may be able to use M109 (heat and wait) instead. If supported M109 will wait until the target temperature is achieved.

\n" }, { "Id": "4826", "CreationDate": "2017-10-29T01:55:39.243", "Body": "

On the reprap wiki it says using Znnn it sets a new axis position. But then it says "No physical motion will occur". What would the line G92 E0 be used for?

\n", "Title": "What is G92 used for in G-code", "Tags": "|g-code|", "Answer": "

To supplement the accepted answer, and answer a question in the answer's comments (which should not be there), consider the E value as another axis - the axis of the filament.

\n

If you executed:

\n
G92 E0     ; Reset the extruder's origin\nG1 F200 E3 ; Extrude 3 millimetres of filament at a rate of 200 units per second\n
\n

and then went on to printing, the first filament move would have to take you from E3 to whatever E value the next move specified.

\n

If the next move assumed starting at E0, you'd already be 3 mm further along, and the first move would probably be a retract, so for example, if the next printing move was:

\n
G1 Xnnn Ynnn E0.5 ; Extrude 0.5 millimetres of filament\n
\n

then instead of extruding 0.5 mm, you would actually retract 2.5 mm, to get from 3.0 to 0.5. Just like moving in a negative direction on any other axis.

\n" }, { "Id": "4831", "CreationDate": "2017-10-29T10:40:33.143", "Body": "

Recently, I changed my RAMPS 1.4 card (because there was a problem with the power output).So I bought this new one, and an Arduino Mega.I didn'T change the code (Marlin 1.3.3).I used the same stepsticks (a4988).I plugged my motors to the Ramps card. When I give them the command to turn they started to turn but didn't stop.Stepsticks got hot.After some time I tried it again and they just vibrated.The stepsticks got insanely hot after just 5-6 seconds.I used a different stepstick RAMPS card and Mega, they worked perfectly.But when I plugged the stepstick to the new card, the motors did te same thing again. What can be causing this and how can I fix it? \n(I've measured the voltage input, it is 12V and stable)

\n", "Title": "Step motors vibrate, don't move at all (Prusa I3)", "Tags": "|prusa-i3|ramps-1.4|stepper|stepper-driver|", "Answer": "

Usually if they get too hot you have to adjust the amperage by turning the little potentiometer on the A4988 (turn left until you can move the motor by hand, turn right until you cant, add a little bit like 1/8 to 1/4 of a turn).

\n\n

But this:

\n\n
\n

I used a different stepstick RAMPS card and Mega, they worked perfectly.

\n
\n\n

it is not completely clear, so:

\n\n\n\n

HTH

\n" }, { "Id": "4835", "CreationDate": "2017-10-29T20:01:20.977", "Body": "

I recently got started in 3D printing but here's an issue I can't seem to find a solution for (I don't know what exactly to look for).

\n\n

Here's what I did:

\n\n\n\n

Is the software making a mistake during the slicing? Or is my mesh screwed up?

\n\n

I have a FlashForge Finder and using the software that came with it: FlashPrint.

\n\n

Edit: I uploaded everything to Thingiverse for those who are interested.

\n", "Title": "Slicing adds holes/cracks to my object", "Tags": "|3d-models|3d-design|slicing|", "Answer": "

here is just a addenum to Tom van der Zanden's answer

\n\n

this is (an example of) what you may design - nice object with virtual outline, and virtual fill\nso this is what you see (and what you potentially expect)

\n\n

\"enter

\n\n

but here is what you get (and probably not really expect)

\n\n

\"enter

\n\n

red parts are the areas which are not covered by fill because nozzle cannot reach there

\n\n

solution is

\n\n\n\n

and of course you can use all 3 options together to get best results ;)

\n\n

EDIT\nhere is simple explanation why smarter app could do the thing\n\"enter

\n\n

and here goes the difference

\n\n

\"enter

\n\n

green parts are new covered areas

\n\n

not much but somthing extra

\n" }, { "Id": "4843", "CreationDate": "2017-10-30T14:36:29.160", "Body": "

With some of the items I am designing I would love to put them through a simulation. Like crushing, for example. I want to know how well my object can handle any situation I put it through so I can make changes before I print out the object, only to find out there was a weak spot.

\n

I have been trying to learn ParaView but it is a little complicated off the bat, I would like something easy to use.

\n

If anybody knows of a program that would be fantastic!

\n", "Title": "A free simulation program", "Tags": "|3d-design|", "Answer": "

Fusion 360 will do finite element analysis (simulation), although I haven't used it. Whether it will perform the type of analysis that you are looking for, I do not know. Check Autodesk's tutorials.

\n\n

One problem that you will encounter is that items printed using FDM technology are highly anisotropic. In other words, they have a grain, and are stronger/weaker in some directions than in others. I do not think that Fusion 360 can take account of that in its analysis.

\n\n

Autodesk: Fusion 360

\n" }, { "Id": "4845", "CreationDate": "2017-10-30T16:55:58.880", "Body": "

I planning on getting a resin 3d printer kit, and I don't want to take any risks building it myself. Where, or who, could I hire a professional capable of constructing a 3D printer kit? They don't necessarily have to specialize in constructing 3d printers, I just need someone qualified with the mechanical and technical skills for the job.

\n", "Title": "Where or who could I hire to assemble a 3d printer that came in a kit for me?", "Tags": "|sla|dlp|services|", "Answer": "

I agree that a local makerspace is a good option, but I will say this...

\n\n

Buying a kit and building it yourself, even with some frustration and learning curves, will help immensely with your ability to calibrate and troubleshoot issues later. If someone else builds it for you, chances are if something goes wrong, you'll need to seek out that person for help fixing things.

\n\n

I just recently bought my first 3D printer (FDM) as a kit. Building it was a slight learning curve - I'm a technophile and have built things with Arduino before, but never a printer and the specifics that come along with it. Getting it to actually print reliably took me about two weeks. However, I'm MUCH more confident now to take care of issues as they arise, and keep it running smoothly. The value of this experience can not be overstated.

\n" }, { "Id": "4849", "CreationDate": "2017-10-30T16:47:28.023", "Body": "

As 3D printers become more and more reliable, their prints get better and better. But FDM printers do have their problems too: you print tiny ovals that smooch together at the edges, and infill makes it awkward at times. So, how do I make a 3D-printed die fair (as in: not favoring one side too much)?

\n", "Title": "How do I 3D-print fair dice?", "Tags": "|print-quality|3d-design|", "Answer": "

Honestly, I wouldn't.

\n\n

You can find dice templates at places like thingiverse, but with my (admittedly limited) experience of affordable 3D printers, I would be highly skeptical that the machine tolerances are up to snuff for producing a fair die.

\n\n

See this discussion from the 3d printing exchange, especially these two answers.

\n\n

If you're really bound and determined to do this (we all have our geeky little passion projects!) I'd advise trying to find a template which the authors claim to have tested with chi-square, then find out what (type of) device they used. If you can replicate that, go for it... and then do the chi-square test yourself. It's ridiculously easy to set something up wrong, or forget something (like the possible effects of internal orientation.)

\n" }, { "Id": "4868", "CreationDate": "2017-11-02T12:37:12.347", "Body": "

I have an issue with implementing a G-code to print a large 3D object using the Creality CR-10 printer. The print went well for over 30 hours but the filament broke off during the print and stopped at a height of around 172 mm of a total height of 256 mm. 1753 layers total , stopped at layer 1170.

\n\n

I want to restart the print so the extruder begins printing at a height of 172 mm rather that from the start.

\n\n

I have tried several sites online and had removed the previous 1169 layers and also changed the start position of the extruder to the code below:

\n\n
G1 X20 Y20 Z385; Center extruder above bed\n
\n\n

The problem I have is that when I start the print, the extruder moves to the middle of the plate and begins moving upwards but I need the extruder to stay near the edge of the build plate and move upwards past 172 mm and then begin printing as it would have for layer 1169 and continue the print. At the moment the extruder moves upwards and towards the middle of the plate and will impact with the existing printed material (see picture attached) and this stops it from correctly positioning.

\n\n

\"Existing

\n\n

Can anyone help with the start code for the G-code that I could use ? I just need the x,y,z to home and then for the extruder to stay near the edge of the build plate but move past 172 mm in the z direction before then beginning to print. Any advice would really be appreciated , first question on this platform so apologies if the detail isn't there. If you need anything more please don't hesitate to get in touch.

\n", "Title": "How to edit G-code created in Cura to begin printing at a specified layer height?", "Tags": "|print-quality|3d-models|", "Answer": "

Well this is a little complicated since nozzle height could collides with the part if the line 1169 has had started to be printed (part of the line), so the overlap will be the main problem, however adding the initial parameters to heat the extruder and getting the X0 and Y0 with G1 X0 y0 Z385 I recommend to increase Z by 3 or 5 mm, so the extruder will travel from a higher position to avoid the crash going for example, from Z390 to Z385.

\n" }, { "Id": "4873", "CreationDate": "2017-11-03T13:12:05.397", "Body": "

I've seen several Q&As on recycling and reusing plastic from failed prints, but what have you done with the last few meters of filament? I've been keeping the ends of PLA (or sections I needed to cut) to maybe use for friction welding pieces together, but I only need so much for that. I've considered just shoving the last bit in the tube and using a new roll to push it through (so long as retractions aren't necessary).

\n\n

This is especially a concern for more exotic filaments where friction welding isn't useful and the price is higher for that 1m section (something like Copperfill).

\n\n

Note: I have a Bowden extruder. I imagine this is less of an issue with DD extruders.

\n", "Title": "Using the end of a spool", "Tags": "|filament|pla|recycling|", "Answer": "

The welding option is only appropriate if you have the tool needed for it, the lighter welding is really hard to do and if your printer allows it you could just watch for the moment the spool runs out and push the new filament as the last of the previous one gets extruded, that's what I used to do on my bowden extruder reprap and apart from a really negligible retraction problem on one layer it just works

\n\n

Or you could go the cool way:

\n\n\n" }, { "Id": "4878", "CreationDate": "2017-11-04T02:49:45.947", "Body": "

My Monoprice maker select recently fried its melzi board. So I went out and purchased a ramps 1.4 kit. I've wired it all up and Flash the firmware with marlin(most recent build). When it boots up it boots up fine I go to prepare. And then I go to move axis I can move the X Y and Z axis and the access is moving in the correct direction. I go to auto home and the X and Y axis home properly but the z-axis does not move at all. Even though I can move it manually. Does anyone have any suggestions. I checked the end stop for the z-axis even though I know it probably has nothing to do with it and it has continuity when the end stop switch is depressed.

\n", "Title": "Z motor not moving during auto home", "Tags": "|marlin|z-axis|ramps-1.4|monoprice-maker-select|", "Answer": "

Alright, I have figured everything out with the help of #reprap IRC community.

\n\n

Issue #1 - Z axis not moving during zero. Just as tjb1 suggested the issue was that it thought it was hitting the endstop, I needed to invert the logic of the endstop within the configuration.h

\n\n
const bool Z_MAX_ENDSTOP_INVERTING = true;\n
\n\n

Issue #2 - No Heated bed controls. The issue resided with the configuration of the heated bed within the configuration.h. The bed was not defined correctly(I do not have the solution for this one as I found a pre-configured configuration.h for the Monoprice maker Select and after loading the firmware it worked

\n\n

Issue #3 - The extruder motor would not move. This was the biggest issue and came down to it being a cheap RAMPS/Arduino. E0's pinouts were not working properly so I took off the driver and wiring from E0 and put it on E1 then altered the pins_RAMPS.h and swapped the values for E1 and E0 pinouts. after reloading the firmware the extruder then moved fine.

\n\n
#define E0_STEP_PIN        36\n#define E0_DIR_PIN         34\n#define E0_ENABLE_PIN      30\n\n#define E1_STEP_PIN        26\n#define E1_DIR_PIN         28\n#define E1_ENABLE_PIN      24\n
\n" }, { "Id": "4894", "CreationDate": "2017-11-06T18:55:24.293", "Body": "

octoprint seems to have a setting that involves googles DNS server.

\n\n

my question is is that server used to access octoprint over the internet via printoid \"android app\" or via browser? if not then what does it mean?\n\"enter

\n", "Title": "Octoprint \u201cusing Google's DNS server\u201d", "Tags": "|octoprint|raspberry-pi|", "Answer": "

That's just Google's DNS (name resolution) server. 8.8.8.8 and 8.8.4.4 are the Google values. Your ISP will also have a DNS server that you could use, which is what it says to use Google's \"if you don't know what to set here\".

\n\n

Using the Google values are your best bet since they'll pretty much absolutely always be online and available.

\n" }, { "Id": "4907", "CreationDate": "2017-11-07T20:11:57.840", "Body": "

I was looking at the prusa i3 3d(the original i3, not the mk2) on sale and I wanted to know how I could tell if it(and printers like it) had a replaceable nozzle, since I wanted to try out different nozzle sizes, and I need to be able to use a .1 mm nozzle for micro armor 3D printing(8mm height miniatures, I'm not too picky about their appearance) so anyways, does anyone know how to tell if a printer has a replaceable nozzle?

\n", "Title": "How to tell if A 3d printing kit has a replaceable nozzle", "Tags": "|prusa-i3|nozzle|", "Answer": "

Prusa has a replacable nozzle. Usually all printer does have an replacable nozzle. Since this is a part of a printer which could get jammed during a printer life.

\n\n

As written here: https://www.prusaprinters.org/prusa-i3-is-now-1-75-mm/ The Prusa i3 uses an E3D Hotend. This Hotend has many diffenez nozzles. You can search on E**ay for E3D V6 0.1mm nozzle and you will find a lot of offers.

\n" }, { "Id": "4908", "CreationDate": "2017-11-07T21:18:06.827", "Body": "

I just got an Anet A8 and I'm pretty happy with it except for one small detail:

\n\n

\"First

\n\n

All my prints start like this. I tried to :

\n\n\n\n

I\u2019m kinda new to this and i did not find a correct answer to my problem so here\u2019s my Cura configuration:

\n\n

\"Cura

\n", "Title": "Anet A8 - first couple of layer are trash", "Tags": "|print-quality|anet-a8|layer-height|", "Answer": "

I think your issue is bed leveling.

\n\n

I recently got my Anet A8 and the biggest kill for my print quality is bed leveling. If its too far from the nozzle, I get something like your picture. You might want to try the paper test where you manually move the nozzle to each corner of the bed and adjust the bed till the paper cannot move freely between the bed and nozzle.

\n" }, { "Id": "4917", "CreationDate": "2017-11-08T12:28:51.957", "Body": "

I am using a STRATASYS Fortus 250mc to 3D print some parts. I have read a lot on internet and also on some scientific literature that Acetone dissolves ABS and cold/hot vapor has been successfully used to smoothen the surfaces. But it doesn't work for me.

\n\n

I followed the instructions: soaking some papers with Acetone and putting the parts in a closed container with a fan for some time. But it had no effects. Then I tried dipping the parts in Acetone and I observed that they do not react to the substance at all!

\n\n

Has anybody ever had a similar experience? Maybe ABSPlus-p430 is not soluble with Acetone? in that case what is a solvent for it?

\n\n

I asked the same question here on Reddit.

\n", "Title": "Acetone smoothing not working on ABSPlus-P430", "Tags": "|abs|smoothing|vapor-smoothing|acetone|", "Answer": "

Make sure that your acetone is actually acetone. According to the MSDS, ABSPlus-P430 is indeed \"ABS resin\" so acetone smoothing should work. Some companies sell confusingly-labeled products that might be mistaken for acetone, but are actually \"eco-friendly\" alternatives such as 2-butoxyethanol. While these alternatives work for some of the uses for which acetone is normally used (such as cleaning or degreasing), they don't work for smoothing ABS.

\n" }, { "Id": "4919", "CreationDate": "2017-11-08T14:56:14.000", "Body": "

We need a 3D printer, that can print with plastic suitable for medical applications. We have about $5000 for the printer. What printers can we choose? I am not a specialist in 3D printing, so please answer in detail.

\n\n

We need to print breathing tubes with cuffs. The tubes should be flexible, but rigid enough in order not to collapse. The cuff material should be thin and collapsible, but very strong. Breathing tubes are inserted through the mouth and stay in contact with mucous membrane for many hours.

\n", "Title": "Inexpensive 3D printer for medical applications", "Tags": "|material|medical|", "Answer": "

Find another medical facility that is doing it, and ask what equipment and filament they use. They may also have a source for medical related models.

\n\n

I would expect medical stuff to be regulated by the FDA, so there are probably limitations on what you can get.

\n" }, { "Id": "4924", "CreationDate": "2017-11-09T16:37:45.607", "Body": "

We are using a Prusa i3 MK2 printer in a structural design firm to print the buildings we design as well as print individual panels and components. Projects can have 100s of pieces that need to be printed quickly, but the printer cools of quickly after the print finishes. When we go to start the next print we have to wait 10-15 minutes for the printer to warm up again.

\n\n

We tried preheating it using the built in function hoping that it would keep the heaters on after the print is complete but had no success.

\n\n

Is there any way to set the printer to stay warm after the print to be able to quickly print back to back?

\n", "Title": "Always keep printer hot and ready to print", "Tags": "|prusa-i3|", "Answer": "

Just set values in your end code for your slicer. Set the bed to the temp you use, set the nozzle to roughly the Tg temp of the filament you use. Typically the bed heatup time is the worst offender here. I wouldn't keep the nozzle at extruding temps, though.

\n" }, { "Id": "4927", "CreationDate": "2017-11-09T17:33:13.327", "Body": "

I have built a 3D printer out of parts from my tip, a ramps board, arduino mega, and a hot end. However, after doing a test extrusion some plastic was spewed out and then the print jammed un-expectantly and my homemade extruder no longer had the strength to push the filament.

\n\n

I took the filament out and what I found was that the filament going into the extruder had formed a cylinder at the end. Then after pushing the filament through by hand to eliminate the possibility of my weak extruder, I found that the filament was expanding coming out of the filament, and then cooling down unable to go through. I cut the filament, removing the bloated end, pushed it back into the extruder and then again after 30 seconds the same problem occurred. \nAfter researching, I came to the conclusion that maybe there is a gap allowing filament to go out of the heating area, expand and then cool down, or even my wooden direct to bowden adapter is stopping the filament cooling causing it to clog. However I am not sure.

\n\n

I am becoming frustrated as I am doing this for a school project, I only have two weeks left to finish and everything seems to be failing. Any help would be much appreciated.

\n\n

For reference this is my hotend: link to amazon\nI brought the cheapest one available on amazon, so it has no fan and no way to connect a bowden tube. I have created a basic adapter between the thread and a bowden tube holder, out of wood. It's not good but it does the job.

\n\n

Here is a picture of my hot end and what the filament looks like after I removed it. There appears to be a spiral shape on some of them.

\n\n

\"enter\n\"enter

\n", "Title": "Hot end jamming very quickly", "Tags": "|diy-3d-printer|hotend|extrusion|nozzle|", "Answer": "

Hello it's seems that your hotend's barrel get's too hot when your ar printing, have you try to make a heat sink on the barrell, with some washers and nuts? That worked for me, like this\"enter

\n\n

I Took that idea from this video https://www.youtube.com/watch?v=UvICpdVONXM

\n\n

Also make sure that the hotend's aluminum block is properly insulated, you could do this with some Kapton Tape.

\n" }, { "Id": "4934", "CreationDate": "2017-11-10T16:45:53.553", "Body": "

I'm struggling to connect a K8200 printer to a rpi3 running repetier-server 0.86.2 armhf.

\n\n

Everything works fine on a PC with the 0.86.2 windows version, but on the Rpi i'm stuck on the second step of the printer setup :

\n\n

I define my firmware, Marlin, my baudRate at 250000, then I tried every port that seems involved (/dev/serial/by-id/usb-FTDI_FT231X_USB_UART_DA00DDXD-if00-port0, /dev/serial/by-path/platform-3f980000.usb-usb-0:1.2:1.0-port0, /dev/tty/USB0, /dev/tty/AMA0, )\nbut whatever I may try, I'm stuck on \"En attente de connexion\", that forbids me to go to step 3, as to see my printer connected.

\n\n

I tried to use the reset button on the printer board, as unplugging-plugging the USB, with no more luck.\n/var/lib/Repetier-Server/logs/server.log don't shown any error.

\n\n

Note : CartesianVirtual port still works fine...

\n\n

What else can I do ? Is there other relevant logs anywhere ?

\n\n

I'm unsure if this question should have been adressed to rpi, 3d printing or linux community, I'm sorry if I offend anyone :)

\n", "Title": "Can't connect printer to repetier-server on RPI3", "Tags": "|marlin|repetier|", "Answer": "

Thanks to the repetier team on git hub, we found that the problem came from a permission issue on /dev/ttyUSB0, as the user repetierserver couldn't access it.

\n\n

Other people reported that these commands solves the problem:
\nsudo adduser repetierserver dialout
\nsudo adduser repetierserver tty
\nbut in my case it seems I need to run :\nsudo chmod 777 /dev/ttyUSB0\nin order to solve this.

\n" }, { "Id": "4935", "CreationDate": "2017-11-10T18:34:40.830", "Body": "

My Y-axis on my Monoprice maker select running ramps 1.4 and Marlin makes a thud noise when moving every now and then. I have noticed a 1 to 2 mm shift in the print when this occurs. I tighten the belts and alas no joy. I ordered new bearings which should be coming Monday. I also tried slowing the print down and still get the same result. Anyone have any ideas.

\n", "Title": "Y-axis slipping causing failed prints", "Tags": "|marlin|ramps-1.4|y-axis|", "Answer": "

The issue was the X-axis top smooth rod came out. this caused the extruder to bang around and shift on the Y-axis when it shifted forward. thank everyone for their help.

\n" }, { "Id": "4941", "CreationDate": "2017-11-11T01:24:13.490", "Body": "

Recently my printer has been creating double images along the X-axis, I just printed a calibration cube after checking all of my rods to make sure nothing was catching and here was my result:\n\"Calibration\nIt looks like there is another X to the right of the X that is supposed to be, would this be a retraction problem, or could it be a problem with the filament because I printed a calibration cube with another filament a couple of weeks ago and there was no double X.

\n", "Title": "Double Image Along X Axis", "Tags": "|prusa-i3|pla|ramps-1.4|calibration|", "Answer": "

There are several problems, but lets start with a method for discovering them.

\n\n

The first thing to do is look for any looseness or slop in the mechanism. Try wiggling the hot end, looking for any free movement. Try moving along the axes that move. You haven't specified the machine, so I can't suggest the specific points to try moving. If the bed moves, try wiggling the bed. If the extruder moves, try wiggling the extruder on each movement axis. Nothing should move at all until the motors skip.

\n\n

If the mechanism is fully tight (and from this picture, I expect it to have some \"wiggle room\"), you could follow by slowing the machine, including reducing the acceleration settings. If you find something moving, you'll need to investigate why -- what is allowing the movement to happen. On a belt-driven machine, the belts can be loose, but one can also get movement from loose set screws, motor mounts, hot-ends which are coming apart, or loose thrust bearings or nuts on threaded shafts. Applying force to the parts that move during printing and watching for movement is the best diagnostic.

\n\n

In the future, when asking questions it can be helpful to include the type of machine, any modifications you've made, and information about the printing conditions.

\n" }, { "Id": "4944", "CreationDate": "2017-11-11T11:22:26.400", "Body": "

I have a problem with Z direction. As you know it has two motors. And my problem is that one sometimes goes to the different direction (so basically one is screwing and another one unscrewing). It is like random, sometimes it is ok, sometimes this one goes to the different direction. Could you help me, please? Wiring is like in building tutorial. The tutorial is here http://reprap.org/wiki/ToyREP_Build_Manual . I am using Marlin and Pronterface. Code I took here http://reprap.org/wiki/ToyREP_User_Manual

\n", "Title": "Z motors work synchronously ToyRep", "Tags": "|ramps-1.4|reprap|z-axis|", "Answer": "

The problem was in the wiring of the motor. According to different sources it says to check wiring, everything seemed ok, but I've found one wire was damaged.

\n" }, { "Id": "4948", "CreationDate": "2017-11-12T14:23:07.923", "Body": "

I use the Cura; nylon default(recommended) settings for PETG. Upon my filament's box is written 200-240 for nozzle and 75-85 for bed. So I tried 245/80 - 220/80 - 220/60 - 245/60.

\n\n

But I got same result as you can see in the pics:

\n\n

\"enter

\n\n

\"enter

\n\n

My printer is PrusaI3 (actually a chinese kit that I assembled it myself). Also I had good print results with PLA before this. I was using 190/50 for PLA).

\n", "Title": "It seems my filament is burnt when I use PETG", "Tags": "|prusa-i3|", "Answer": "

It looks like you have a clogged nozzle/hot end. I use 225\u00b0C @ 40mm/s for PETG with 235\u00b0C @ 30mm/s for the first layer.

\n\n

If you print at 245\u00b0C, you risk burning the PTFE liner in your hot end. You really need an all-metal hot end to print at such temperatures.

\n\n

It is possible that you have already damaged the PTFE liner, and this may be the cause of your problem. However, I would check for a clogged nozzle first.

\n" }, { "Id": "4950", "CreationDate": "2017-11-12T17:15:32.713", "Body": "

I have a PrusaI3(Chinese kit that I assembled myself). my strange problem is that: when I try to leveling the bed, upper-left and bottom-right corners of the bed, are more far than the nozzle, respect to upper-right and bottom-left.

\n\n

I don't know why, but I can't level them with the screws(because upper-left and bottom-right screws goes to be free sooner than their opposite side screws!!). I thought maybe the heated bed is not flat but I use a glass upon it! and it's not possible both of them are not flat!

\n\n

EDIT: I can remember the last time I used my printer, there was a knocking sound when Y-AXIS was moving about the half of it's way. I very tried to find the cause of knocking sound but I couldn't. now, I have opened the heating bed and there is no sound when I move Y-Axis by hand. \nI hope you can understand me and help me too!

\n\n

EDIT2: I did measure the rods and bed corners, I found that this corner(pointed by finger) is about 2 millimeters lower than other 3 corners! (It seems the bed part is not flat).

\n\n

\"enter

\n\n

How can I fix this?

\n", "Title": "Strange leveling problem by PrusaI3", "Tags": "|heated-bed|", "Answer": "

I could solve my problem by this strange system for both opposite sides but I couldn't find the really reson of this problem!:

\n\n

\"enter

\n" }, { "Id": "4956", "CreationDate": "2017-11-13T16:09:55.073", "Body": "

I want to paint my ABS parts. But I foresee a problem that will arise from 0.16mm layer - those layers will be perfectly visible, even with primer. Sanding them wouldn't be a good idea, because walls are just 0.8-1mm thick, and probably acetone bath isn't right thing too. I want to use some sort of cement or paste, and cover parts with it using some paint scraper or knife before painting. Do cements suitable for this task exist? Would they stick to ABS?

\n", "Title": "Cement or another solid pore filler suitable for ABS?", "Tags": "|abs|post-processing|", "Answer": "

Consider these moments:

\n\n\n\n

Hope this helps.

\n" }, { "Id": "4965", "CreationDate": "2017-11-14T10:25:01.993", "Body": "

I have a Prusa i3 printer with mk8 extruder. I am using PETG with (200-240/75-85) degrees recommended temp on it's box. but when I do printing i hear some knocking sound that means there is a problem in extrusion. last time I did increase my temp up to 270 degrees!! maybe this sound solved! but it damaged my extruder's screw's inner Teflon and I forced to change it!

\n\n

Now I have a new nozzle and extruder's screw, but again I can hear this sound during the printing! I thought maybe it's because of very close distance between the nozzle and the bed, but when I increase the distance, the filament starts to warping!

\n\n

My nozzle is new and it can't be for clogging. so I decreased the print speed from it's default 60mm(in Cura software) to 50, but didn't solve this sound. Also when I use pronterface and do extrusion manually, it works nice with 200 degrees! but I can hear the sound in 190 degrees. But when I do printing I hear the sound even in 200 degrees! Is this mean I must increase the temperature for printing? how much?

\n\n

By knocking sound I mean the sound you can hear when you have a clogged nozzle and the extruder's stepper motor can't push the filament into it and rotates with knocking sound!

\n", "Title": "What is the meaning of extruder's knocking sound?", "Tags": "|diy-3d-printer|print-material|", "Answer": "

It sounds like your filament is not hot enough to be extruded, which will cause the stepper motor to 'skip' (and produce a knocking or clicking sound). 200\u00b0C sounds rather low for PETG, and I do not get good extrusion below 215\u00b0C. I normally print at 225\u00b0C, with the first layer at 235\u00b0C. I also print slower than I do for PLA -- 40mm/s and 30mm/s for the first layer.

\n\n

Note that thermistors can be inaccurate, and may report temperatures several degrees lower or higher than are actually being achieved.

\n" }, { "Id": "4973", "CreationDate": "2017-11-14T18:28:58.097", "Body": "

Here is a video about 30MB, that shows my printer and it's sound during printing: https://drive.google.com/file/d/1wj5_Uwv4X8SoLaouxBi1FHFi4x4Uoegd/view?usp=drivesdk

\n\n

I guess the reason of this sound is the bed. Because I couldn't level my bed with screws and upper=left and bottom-right corners of the bed were about 1.5mm lower than the opposite sides. So I added two nuts behind the springs of these two sides! I don't know really if it could help but know I can print almost good.\n\"enter

\n\n

So i think the knocking sound is because of these nuts and my bed carriage is not 100% flat(although it seems flat by eyes). Or maybe my rods aren't parallel(although they seems parallel by simple ruler measurement).

\n\n

By the way, I like to know does anyone experimented same problem? or can guess the real problem or suggest a way to solve it?

\n\n

And IMPORTANT question: May this problem break my heated-bed? Or causes new problems in future?

\n", "Title": "Knocking/Clicking sound when Y-Carriage passas trough the middle of rods", "Tags": "|diy-3d-printer|print-material|", "Answer": "

I don't think the sound is coming from nuts and springs.

\n\n

I can think of 3 possible sources.

\n\n

Y axis rod bearings

\n\n

Those four bearings that are mounted onto your bed frame may be binding through rough spots on the smooth rods. Usually printer kits don't give you the best quality stuff and those rods may not be perfectly the same diameter throughout its length or may be bent so slightly. Your best bet to test if this is the problem is to remove the belt from your bed and slide the bed back and forth and see if you can recreate the sound trying different speeds and pressure while doing so.

\n\n

Y axis pulley/idler

\n\n

I think it's probably this because I have heard a similar noise and this part has failed on me. Usually these are either a plastic pulley, two bearings, two bearings inserted into a plastic pulley, or one larger bearing inserted into a plastic pulley. If your printer uses a bearing here, I recommend taking that part off and inspecting that bearing. I've had mine destroyed and the little balls went everywhere. You can test this by just trying to hear for it. Turn off the printer motors and move the bed manually, see if it sounds like it is coming from the idler.

\n\n

Nozzle hitting print

\n\n

I doubt this but sometimes when prints are over extruded or curl up, the nozzle hits the print as it passes over so maybe your hearing individual collisions clicking. Again I really doubt this.

\n\n
\n\n

Will this break your heated bed? Probably not. Will it cause problems in the future? If it is a fault with the bearings, most likely they will fail eventually but nothing else should get damaged in the process. But no worries, parts are cheap and readily available online or even at some hardware stores.

\n\n
\n\n

Note : My mechanic taught me this when trying to figure out which bearing was making noise in a car. Take a long screw driver, preferably with a wooden handle, and place the tip on the part you think is making noise and place the handle to your ear. Usually this amplifies the noise when your making contact with the faulty part. Using this I was able to figure out which bearing was squeaking among the half dozen points where the belt would spin. Not sure how well this works for a printer and be careful not to have the mechanical moving parts(such as your printer bed) hit the screwdriver into your face.

\n" }, { "Id": "4974", "CreationDate": "2017-11-14T18:49:29.200", "Body": "

Is this a leveling problem? or it's because the bed temperature isn't equal in everywhere?

\n\n

If it's because of leveling, it means the distance between nozzle and bed in this not-sticked corners are lot and should be decrease?

\n\n

EDIT: material is ABS, and it's recommended temp is 190-230/80-100. But I use Cura high quality ABS default settings with 225/80. (But I had this problem with PLA with 190/60 degrees too)

\n\n

\"enter

\n\n

\"enter

\n\n

\"enter

\n\n

\"enter

\n\n

EDIT2: I have a cooling fan for my Arduino/ramps board. But there was a wire between it's blades and it didn't work. today I saw it and tried to fix but it's plastic duct broke. So I tried to put it by hand as how it's air can cool both the Electronics boars and the bed(you can see my fan in one of the pics in left side). And I got a nice printing results for the same part printing:

\n\n

\"enter\n\"enter\n\"enter

\n\n

So the cooling fan for printing part is very important to stick print to bed and I will print a duct for extra fan very soon!

\n", "Title": "Why some corners aren't sticked well?", "Tags": "|heated-bed|nozzle|", "Answer": "

You can clearly print but there are also some problems it seems.

\n\n

To see if you're about okay when it comes to leveling, I recommend to activate the 'skirt' in your slicer soft. Slic3r has it and I think most others too, it just makes the printer do a big one-layer all-around 'perimeter' around what is going to be printed before starting. You'll see quickly if there is not enough or too much space between the bed and the nozzle. You might catch other problems too (I love it because when the hotend drips a bit when leveling it get's smeared out far away from where the print is going to happen).

\n\n

Your print looks like it is not optimal (overextrusion? no print-cooling? ...) so start with slowing down, first layer 20-25mm/s (Look at it when it's printed, it's very telling!). Then for the following ones, don't go over 40-50mm/s for starters.

\n\n

Last but not least, you are printing ABS (but you said you had the problem with PLA too, hence my post), I have never printed ABS but for what I have read, you'll need a heatbed and a heated chamber(enclosed printer) to not have this kind of problems.

\n\n

But for PLA especially; If you have a heat bed, don't put on too much heat when printing, I went from 60\u00b0C to 50\u00b0C, first layer, 30\u00b0 then, and my prints now stick like they are melded onto my plate instead of skidding around.

\n" }, { "Id": "4975", "CreationDate": "2017-11-14T19:34:29.083", "Body": "

I'm printing a cylindrical piece but at any moment it breaks down. I tried it two times, both have break down in different place. I'm using a 1.75 mm PLA filament in my Anet A8. I'm using Cura 2.6.2 to export to a .gcode file.

\n\n

This is the original model:

\n\n

\"Original

\n\n

And this is the result:

\n\n

\"Printed

\n\n

Here is the G-code file.

\n", "Title": "Printing starts well but then it breaks down, Anet A8", "Tags": "|pla|print-quality|ultimaker-cura|anet-a8|", "Answer": "

The answer by fred_dot_u is fine for small prints with relative low cost or batch producing, where a single part is needed several times. But if you print something big or unique it's not cost effective. Slowing down the complete print is also not very time efficient.

\n\n

In Cura there an option called Mininum layer time, which addresses this problem:

\n\n
Cooling -> Minimum Layer Time / Minimum Speed\n
\n\n

This means that it will only slow down when the print distance for a particular layer is short. Giving the material some extra time to harden. Other layers will be printed at normal speed. Increased print time will be limited this way.

\n\n

For your situation I would advise you to increase the part cooling, if possible, in conjunction with configuring the Minimum Layer Time option mentioned above.

\n" }, { "Id": "4980", "CreationDate": "2017-11-15T11:26:41.307", "Body": "

I'm fairly new to 3D printing but I'm getting the steam up and I chug out pieces without much hassle. One thing though is that I have the impression that sometimes, parts are easily broken, and I suspect bad adhesion between layers.

\n\n

I'm printing eSun PLA at 208 \u00b0C (212 \u00b0C first layer), bed at 50 \u00b0C first layer then 30 \u00b0C.

\n\n

When I print the Benchy boat, it feels extremely sturdy (even the small chimney is unbreakable by hand), but if I make, say, a 50 mm diameter cylinder with 4 mm wall thickness, It breaks fairly easy along some layer. I don't see any specific error, it \"looks\" okay.

\n\n

So my question is:

\n\n

How do you assure that you print layers that bond enough?

\n\n

[Edit] I already know that might actually cause bad bonding (low heat, underextrusion, ...) but I'm looking for a way to see when it happens.

\n\n

Here is an image from when I stopped mid print (you can see the little ooze string just at the start at the crack) for checking dimensions. The next layer didn't bond well because, I guess, the already printed part had cooled down when I un-paused the print (say 1 minute later) or maybe the z-axis went off a bit when I touched the build plate.

\n\n

\"Sorry

\n\n

Is the only way of knowing layers bonded correctly, to try to break the part apart?

\n", "Title": "How do you assure that you print layers that bond correctly?", "Tags": "|pla|print-strength|", "Answer": "

(Note: I realize this question is old, but I came across it looking for information on related topics and it didn't seem to have any sufficiently good answers yet.)

\n\n

To me, this looks exactly like what I'd expect printing with layers almost as thick as the nozzle diameter and insufficient hotend temperature or excessive speed. I've had problems like this at sizes 0.3 to 1/3 mm using an 0.4 mm nozzle even at my usual PLA temperature of 210, and can imagine it would be much worse at lower temperatures. If you didn't see the problem on benchy with a smaller cylinder, it's probably either because acceleration limits (which will be dominant on small details) prevented reaching high speeds, or (less likely in my opinion) because the deposited material had not had as much time to cool before the next layer was reached.

\n\n

In order for layers to bond the newly deposited material needs to be pressed against the material already present, while melted, with sufficient heat transferred into the already-deposited material to make it amenable to bonding (naively I would guess this means it needs to reach around the glass transition temp, but that may be wrong). With layer thickness near the nozzle diameter, the nozzle will be putting less pressure on the existing material, making this harder. Also, if there has been underextrusion anywhere in the supporting wall below, the already-deposited material can simply compress downward into the gaps when new material is extruded against it, rather than them getting pressed together.

\n\n

For 0.3 mm layers, I'd go with the highest temperature the filament manufacturer rates it for, or even slightly higher if you have reason to believe the material is okay at higher temperatures. Some people say lowering the fan speed or turning it off is an option, but I've had really bad results with precision and/or stringing whenever I try that.

\n\n

Significantly lowering the print speed is also an option. That will greatly reduce the chance of underextrusion, and results in more heat transfer from the nozzle into already-deposited material. However lowering the speed generally defeats the purpose of 0.3 mm layers; you could instead just drop down to 0.25 mm or 0.2 mm and get much better quality at the same total print time.

\n" }, { "Id": "4982", "CreationDate": "2017-11-15T12:06:11.210", "Body": "

Question

\n\n

What is PLA+? How is it different than PLA? I'm looking for science, composition, formula, safety concerns (or lack thereof), etc.

\n\n

Background

\n\n

I picked up a roll of PLA+ at Microcenter (their in-house Inland brand) because it was on clearance. I didn't even notice the \"+\" until I decided to try that color, and then I noticed it on the sticker. It prints well, feels like ABS, smells like PLA when printing, and I can use PLA temps on my printer. It sands better than PLA, and if I I hadn't noticed the PLA+ sticker, and the smell, I would think it was ABS. It will break its line into my printer like PLA does; ABS doesn't break if left alone. However, PLA+ lasts longer than regular PLA before breaking.

\n\n

Getting info from the Internet

\n\n

Aside from a few discussions on reddit (review, commercial introduction), I can't find anything about it.

\n\n

Getting info from the Manufacturer

\n\n

I went back to Microcenter and the guy that was there working the 3d printing section did not know what I was talking about.

\n\n

I went to Microcenter another time and the guy in the filament area said that all of their PLA filament was now PLA+, and that the + meant it was to be used at a higher temperature. The boxes are labeled with 205 - 225\u00b0C. It seems that all the inland brand PLA I have is PLA+, save for the first roll I bought. It does not have any kind of temperature markings on it.

\n\n

Flash forward 1.5+ years from the original question

\n\n

This question got some recent attention, so I looked to find the answer again. I found this article, which is a hot pile of !usefulness, giving no data, lots of opinion, and probably some direct insights from someone's marketing department.

\n\n

These guys say it's good stuff, but nothing about the chemical or compositional difference between the two. When I find people talking about the difference (like on reddit), those are the details usually mentioned, which are vague, anecdotal, and opinionated, and could be clever marketing (could be, not guaranteed to be). One man's shiny is another man's matte for example.

\n\n

Monoprice confirmed what I already did by reading the label and printing with it, but does mention TPU, which might be Thermoplastic polyurethane. No quantity or proportion or anything, and since they're the only manufacturer/reseller to officially say this, I consider it unconfirmed. One of the answers below says that PLA+ probably includes TPU or something like it, but that's conjecture or opinion by their own admission.

\n\n
\n

PLA+ is a variation of PLA that has added material in order to make\n the filament less brittle, have a smoother surface finish, and less\n likely to absorb moisture. Typically, TPU is added into the filament\n in order to achieve this property. PLA+ will have the feel and\n functionality of ABS without the smell. If you didn\u2019t know better, you\n would think it was ABS. We suggest printing with PLA+ at 205 to 210\n degrees Celsius and with a bed temperature of 45 degrees Celsius. PLA+\n responds very well to blue painter\u2019s tape and a glue stick to hold\n properly and not peel up when printing.

\n
\n\n

These people also ask what it is, but they're doing science about it circa 2014 to try to figure it out.

\n\n

I'm not sure what to think of this manufacturer/seller's description. It sounds like they are implying that PLA has a branding problem, so they added a plus to it for a new formula to fix their branding.

\n\n
\n

PLA Plus is an enhanced version of our PLA that's less brittle and\n more durable. \u2018Enhanced\u2019 PLAs have a bad reputation, some are no\n better than PLA, some perform worse in some conditions. We\u2019ve taken a\n different approach: our \u2018regular\u2019 PLA is regarded as the strongest\n pure PLA in the industry, it\u2019s hard to improve on the best. But\n sometimes you need something a little more durable. Enter our\n specially formulated PLA Plus. Prints like PLA, but with better\n durability. Its available with their brighter color options!

\n
\n\n

Final thoughts

\n\n

I find a lot of articles/posts talking about \"eSun PLA+\" specifically. I'm starting to think that this might be the OEM and that other companies are selling it with their own branding, but that all PLA+ comes from the same place. I found their product page, and it says this, which mentions nothing about the formula:

\n\n
\n

Characteristics\uff1a

\n \n \n
\n", "Title": "What is PLA+? How is it different from PLA?", "Tags": "|filament|pla|filament-choice|pla+|", "Answer": "

Adding this as a new answer since it doesn't seem to be covered in existing ones:

\n

Despite "PLA+" being a marketing term without a specific definition, I've found that many (most?) filament vendors don't seem to be doing their own secret-sauce blending to make it, and most premium filament vendors who do document the source polymers they use are using roughly the same thing - most often one of the Natureworks Ingeo 3D series of polymers, obtained in pellet source form, often 3D850 or 3D870. As such, if you can verify that this is what you filament marketed as "PLA+" is made from (or just search out one that's well-documented) you can use the data sheets for the polymer to find out more about how it's formulated.

\n" }, { "Id": "4985", "CreationDate": "2017-11-16T01:48:51.287", "Body": "

I'm relatively new to Cura, having just installed it a few weeks ago. I have 3.0.3 and I'm running on Windows 7 64 bit. I noticed the auto slicing setting while poking around the settings. Auto slice almost always works.

\n\n

A couple times now, Cura won't auto slice. I can't seem to find the slice button. How do I manually slice? I found an article on how to disable auto slicing, but it doesn't show how to manually slice.

\n\n

Where the button \"save to file\" is is greyed out and just says \"Print with Octoprint\".

\n\n

Closing Cura and re-opening it will fix the issue.

\n", "Title": "Where is the \"slice\" button in Cura?", "Tags": "|ultimaker-cura|", "Answer": "

If auto-slicing is disabled, the button in the bottom-right corner of the window toggles between \"Prepare\" and \"Save to File\" depending on whether the model needs to be re-sliced or not. While slicing is in progress, it changes to \"Cancel\".

\n\n

If auto-slicing is enabled, the button always reads \"Save to File\", and is greyed-out when slicing is being performed (it does not change to \"Cancel\").

\n" }, { "Id": "4999", "CreationDate": "2017-11-19T02:07:12.230", "Body": "

I just backed a 3d printer on kickstarter, called \"Mooz\", and I was wondering about some of the possible applications of it. I realized it would be cool/helpful to be make cookie models, and bake cookies to that shape. The cookies can be baked as low as around 250 \u00b0F, So is there any food-safe and heat resistant filament that can withstand those temperatures? (I know many filaments are printed above those temps anyways, but in my case I don't want them to even warp.) Google has failed me on this one, so I hope someone here can answer it for me!\n-Thanks in advance :)

\n\n

P.S. My 3d printer will have a heated bed, so it can print more types of filaments. The highest bed temp is 100 \u00b0C, and the highest hot end temp is 250 \u00b0C. I don't really mind using some sealant, but I'd rather not use something toxic like ABS.

\n\n

P.P.S I'm a newbie to this stuff, so I apologize in advance if I asked a really stupid question.

\n", "Title": "Oven/Food safe Filament?", "Tags": "|filament|food|", "Answer": "

Don't try to bake cookies inside a plastic mold; the plastic will smoke even if not melt. If you need to bake cookies use a cookie cutter made of plastic (your own design or copy) obviously printed on your new printer then bake normally. here is a link from thingeverse to get a cookie cutter .stl file

\n\n

:) enjoy cutting cookies these holidays

\n\n

Here is another link from hacks from one person is making the same to show his idea.

\n" }, { "Id": "5001", "CreationDate": "2017-11-19T09:17:31.420", "Body": "

I know you should, like an infant, never leave your printer without surveillance.

\n\n

But sometimes we all do, trusting our double thermistors and heat runaway configurations. But electronics fry and who says there is no danger even after the print job has finished and it's cooling down, still hooked up?

\n\n

I have searched but smoke/fire detectors come in a wide range of varieties:

\n\n

They can be battery powered or hardwired, they detect different things: carbon monoxide, heat, smoke..., they thus also have different detectors like photoelectric sensors, ionization sensors or both. We also print with different materials...

\n\n

So what's the best safeguard for my 2 year old (CoreXY ^^)?

\n", "Title": "What type of fire alarm/smoke alarm should my printer have?", "Tags": "|electronics|", "Answer": "

According to http://www.fireservice.co.uk/safety/smoke-alarms/ you have these two choices:

\n\n
\n

Ionisation: These are the cheapest and cost very little to purchase.\n They are very sensitive to small particles of smoke produced by fast\n flaming fires, such as paper and wood, and will detect this type of\n fire before the smoke gets too thick. They are marginally less\n sensitive to slow burning and smouldering fires which give off larger\n quantities of smoke before flaming occurs. They can also be too\n over-sensitive near kitchens.

\n \n

Optical: These are more expensive but more effective at detecting\n larger particles of smoke produced by slow-burning fires, such as\n smouldering foam-filled upholstery and overheated PVC wiring. They are\n marginally less sensitive to fast flaming fires. Optical alarms can be\n installed near (not in) kitchens, as they are less likely than\n ionisation alarms to go off when toast is burned.

\n
\n\n

Although 'overheated PVC' might tend to point to the optical sensor being best, any fault on a printer is likely to be quite different to a domestic installation fault (I think you're less likely to detect the early warning from a damaged connector since the fault will be physically small).

\n\n

I have an ionisation sensor next to/above the printer, and the only time I had a false-alarm was when I temporarily placed it on the desk.

\n" }, { "Id": "5009", "CreationDate": "2017-11-21T11:07:05.870", "Body": "

I have this prusa i3 printer(I don't know what model is it):

\n\n

\"enter

\n\n

My bed part is broken. And I am looking for the cads to laser cut them! this is my bed pic:

\n\n

\"enter

\n", "Title": "I am looking for laser cut cads for my prusa i3?", "Tags": "|prusa-i3|cad|", "Answer": "

This is not exactly my part, but the holes place are true. I think if someone measures the bed dimensions( for me it's 21.9mm * 21.9mm) and orders a plate in this size, he can also make holes later with drill(I did it before for another part of my plexi to attach the power supply).

\n\n

https://www.thingiverse.com/thing:1310778

\n\n

\"enter

\n" }, { "Id": "5013", "CreationDate": "2017-11-22T02:13:33.930", "Body": "

I recently backed a 3D printer on Kickstarter, and I would like to 3D print parts for high-temperature applications. So I have two questions;

\n\n
    \n
  1. What's the highest temperature polycarbonate can be safely heated to without warping or releasing toxins?

  2. \n
  3. If there's a filament with better temp-resistance I can print could you tell me? (If you know the highest temperature it could reach safely, that would be helpful too.)

  4. \n
\n\n

My 3D printer will have a heated bed up to 100\u00b0C, and an extruder temp of up to 250\u00b0C.

\n", "Title": "What are the safest temperatures to heat different filaments to?", "Tags": "|filament|health|", "Answer": "

All plastics have two temperatures to consider for operation and evaluation:

\n\n
    \n
  1. Melting point = the temperature that plastics starts to get soft; this can be considered for maximum temperature operation.

  2. \n
  3. Flow temperature = normally used for molding process and this have a wide range of temperatures depending on PPM\u00b4s and mixtures to meet a purpose like coffee cups. Here a link for filament temperature guides

  4. \n
\n\n

Polycarbonates are the most plastic used for industrial and kitchen appliances. Due higher temperatures support and hardness, so I think is not possible to use polycarbonates in 3D printers due its temperatures are from 250\u00b0C to 320\u00b0C.

\n" }, { "Id": "5030", "CreationDate": "2017-11-26T18:29:55.927", "Body": "

I haven't purchased a 3D printer yet, I've been working on the software and electronics for my project and now I'm about ready to get a printer, haven't decide on which yet.

\n\n

Recommendations welcome.

\n\n

My project will put 3 rows of Neopixels on the windscreen of my car, attached to the top inside. I intend to print the container in sections that will slow together.

\n\n

I want to allow for the curvature of the windscreen in the design of the container and this is my question, I want to design a container that will hold 3 rows of Neopixel strips, thats the easy part, I want the container to fit neatly against the windscreen with no gaps, how do I work out the curve of the windscreen so I can put this onto the enclosure ready for printing?

\n\n

The enclose will be as wide as the read windscreen and split into printable sections that will slot together for installation.

\n\n

The car this is going into is an Audi A5 2012.

\n", "Title": "Project help, printing a Neopixel enclosure to secure to a car rear windscreen", "Tags": "|enclosure|", "Answer": "

There's a handy tool for replicating curves that use a series of plastic or metal fingers in a sliding mount. One presses all the fingers to one side, then presses the bracket to the desired surface until all fingers are in contact. The opposite side now represents the measured curve and can be traced into a modeling program.

\n\n

For your purposes, that is unlikely to work, as the tool is perhaps six inches long, far too short to span your windscreen.

\n\n

Consider the following:

\n\n

Attach a string to each edge of the windscreen or wedge a thin rod from one side to the other. This provides you with a straight line reference. Measure as nearly as possible to perpendicular from the reference string to the windscreen surface. I have a laser measuring device that has 1/16\" or 1 mm accuracy above certain distance, which would not work well in this case as you approach zero at the edges.

\n\n

Start in the center, record your measurement, and move an appropriate distance outward. I suggest you need only approximately 20-30 mm spacing as you are collecting points on a curve, which are likely to be relatively uniform and easily reproduced in software.

\n\n

Your graph now has zero, zero on the left, and say 750 mm, 40 mm on the right, with matching numbers in the middle. On real paper, turn it into a graph, or use a suitable vector editing software such as Inkscape and duplicate it there.

\n\n

Depending on the shape of your window, you may have to replicate the process at different heights in order to get a proper representation of the curve of the glass.

\n\n

Once you have the dots, run a Bezier curve through the dots. The resulting like can be extruded or otherwise converted into the segments necessary for your 3D printing project.

\n" }, { "Id": "5034", "CreationDate": "2017-11-27T12:20:48.767", "Body": "

I'm using an Anet A8 with Marlin 1.1.6.

\n\n

I've read countless guides on PID auto-tuning and never saw that the fan needed to be on, but when I look at the Marlin source code's ANet A8 configuration (here: example_configurations\\ANet\\A8\\Configuration.h) it states:

\n\n
  ...\n\n  // ANET A8 Standard Extruder at 210 Degree Celsius and 100% Fan\n  //(measured after M106 S255 with M303 E0 S210 C8)\n  #define  DEFAULT_Kp 21.0\n  #define  DEFAULT_Ki 1.25\n  #define  DEFAULT_Kd 86.0\n\n#endif // PIDTEMP\n
\n\n

So, should activate the fan before running the auto-test if I wish to update the PID values in Marlin for my Anet A8?

\n", "Title": "Should I be doing PID auto-tuning with my fan at 100% (Anet A8)", "Tags": "|marlin|anet-a8|cooling|", "Answer": "

You should run the fan at what you expect it to be at the majority of the time it is printing. If you tune at 100% fan and never use a fan then it will be too aggressive, if you tune at 0% fan and use the fan then you will struggle to maintain/reach temperature.

\n" }, { "Id": "5035", "CreationDate": "2017-11-27T13:36:44.387", "Body": "

I am modelling a few cut templates to be used on an hexagonal grid (honeycomb) material using OpenSCAD. Basically, from a reference cell, I need to select all cells that are within a given range and given angle.

\n\n

I implemented this by creating an in memory grid that covers an area larger than what I need (extra range, 360 degrees), and then testing each cell for both the distance and angle requirements, extruding only those that test positive for both conditions.

\n\n

Everything works as expected...

\n\n

\"Range

\n\n

...but now I would also like to add the possibility to have the outer contour of the template without having each individual cell within it (so, a single thin line going around the whole \"pizza slice\" above).

\n\n

I'm pretty new to OpenSCAD: what would be the best approach here?\n(I'm happy even with a solution that requires to re-implement what done until now).

\n", "Title": "How to remove internal part of a hex grid", "Tags": "|3d-design|openscad|", "Answer": "

I ended up finding a reasonable solution myself:

\n\n

\"enter

\n\n

Basically, I diffed two identical, non-hollow geometries, in which the first one had the cells larger than they needed to be (so overlapping with others), and the second one had them exactly of the right dimension:

\n\n
difference() {\n  base_geometry(range, angle, infill, extra_padding = 2);\n  base_geometry(range, angle, infill, extra_padding = 0);\n}\n
\n\n

This way the only portion of the solid remaining was the extra_padding on the outer edges of the geometry.

\n" }, { "Id": "5037", "CreationDate": "2017-11-27T14:29:51.583", "Body": "

I made a scan and got a cloud point file, but the cloud point is not that good, but I need to print it in a 3d printer as soon as possible, just to see how it would look.

\n\n

Basically it's missing some points that were not scanned properly due to it being transparent at those points. I'd like to add those points manually (using meshlab preferably) and create a surface with them .. something that makes this file printable.

\n\n

I know I could edit this file manually since it is an ascii file with the values of xyz .. but that is a lot of trouble and certainly there is a faster way to do this with a free tool.

\n", "Title": "Adding points to a point cloud", "Tags": "|3d-models|scanning|", "Answer": "

A good free tool for editing meshes would be Blender. Linux, Windows, Mac are supported. It has a serious learning curve, and is somewhat counter-intuitive in use (right click object to select) unless you customize it.

\n\n

Plenty of online video tutorials to learn the basics, though, and if you have a membership for Lynda.com, those videos are the best (opinion).

\n\n

I've used it to pull edges and vertices together, to chop out pieces that weren't supposed to be there, etc. The flexibility and power of the program is amazing, considering the price.

\n\n

Many artists use the program for creating 3D animations and full length movies. Amazing talent, well beyond my capabilities, but it shows what the software can do.

\n\n

The website also has links to tutorials and the manual, which is also a work of art.

\n\n

\"blender

\n\n

If you can devote the time to learn the program, anything 3D modeling related will be within reach.

\n" }, { "Id": "5044", "CreationDate": "2017-11-28T18:09:44.930", "Body": "

I have created a budget 3d printer using parts of old computers, specifically I used the CD drive mechanisms for the x,y and z axis. This printer is similar to the curiosity E-Waste printer, as seen here: Link

\n

After quite a bit of time working on it I have got to the point where my printer should be able to print, but unfortunately, I have encountered an issue.\nThe layers in which the printer prints fluctuate in height, and so when the nozzle travels over the lower layers to do the next layer, it hits a bump, misses some steps and becomes mis-aligned. I have never owned a 3d printer before, but I think that it is likely that the problem I have which causes the prints to not be flat is likely a common 3d printer problem. However, with commercial 3D printers, these bumps would probably only cause a drop in the quality of the print and not a complete fail; it is only due to how weak my motors are that the nozzle will miss a step with even the slightest force against it.

\n

I have thought about why the layers may not be flat and I have come up with a few possibilities that I think are the problem. I do not know if these are in fact the source of the issue, but these are the things that I have considered so far.

\n
    \n
  1. The nozzle height when starting the print. If I lower it then the layer height will be smaller, and so there is less chance for it to be higher in different places. However upon testing this I found that, due to the reduced height upon the first layer the nozzle would get stuck.
  2. \n
  3. When the nozzle changes direction, for example at the edges of the print, there is significant vibration and it is possible that this cause the filament to overlap and not be flat. The vibration is due to the nature of my printer and would be very hard to reduce. A solution could be some kind of software fix, but I do not know enough about this to do so.
  4. \n
  5. Maybe the speed at which I am printing is not good. I'm printing currently at an incredibly slow 40mm/s.I have tried different speeds but at low speeds there is over extrusion and at high speeds, the vibration is too much.
  6. \n
  7. I have tried experimenting with flow rate (100%,75%,and 50%) and at the moment I am using 50%. This is because my extruder motor is a bit weak and often misses steps, also the filament requires considerable force to push it through. Is this normal?. My idea is that maybe there is too much filament being extruded and so if I lower the flow rate the print will not create bumps.
  8. \n
  9. The filament curls slightly when it comes out of the nozzle and I have tried to fix this by cleaning it and printing faster but it is only reduced. I'm printing at 200\u00b0 currently; Will higher or lower print temperatures cause the filament to curl less and be more easy to push through the hot end?
  10. \n
\n

As you can see there are a lot of factors in which I need to consider if I want my printer to be able to print correctly. And unfortunately, with my printer it is either perfect or a failed print; there is no in between. Due to the weak motors everything has to be exactly right, or the nozzle will get stuck.

\n

I am not sure how to add videos here so I will instead. add some photos of the prints, and three photos detailing how the prints fail.

\n

Below is an image containing the 6 most recent prints in which I played around with the different factors mentioned above. The top middle one is the most recent one. The object i am trying to print is a simple cuboid.\n\"The

\n

This is what the first layer of my most recent print looked like. There is some kind of gap in the middle for some reason, but I don't think that is the source of the error. The print appears to have adhered well, at this point and it is relatively flat.\n\"first

\n

This is what the second layer of the print looked like, notice how the lines are wobbly and not very neat. There is also a small area of higher height in the bottom left corner which inevitably causes the nozzle to get stuck.\n\"enter

\n

This is the print after the nozzle hit the now cooled down blob, missed some steps and then was misaligned. It then goes on to print the next layer in the wrong place and gets stuck very badly.\n\"enter

\n

Thank you, for reading this. Any help would be much appreciated

\n", "Title": "How can I prevent my nozzle catching on my print?", "Tags": "|print-quality|extruder|diy-3d-printer|fdm|", "Answer": "

A printer must be working very well to not have the problem with the head hitting against small verticle protuberances and losing control of the X and Y positions.

\n\n

Some factors I've found that make bigger bumps. If you have any of these, fixing them will make the problem better. From observation, it is ultimately the power of the motors, the ability fo the nozzle to melt through small bumps and the resilience of the mechanism (not something that is otherwise helpful) that let decent prints proceed.

\n\n
    \n
  1. The first layer may not be the height you expect. First layers are\ntypically 100% filled. If you are extruding too much plastic for\nthe layer height, it has to go into a bump. To fix this, check your\nvertical height. Measure with calipers the height of the flat\nportions of a 1-layer print, and compare that with your slicer\nexpectation. Adjust the vertical offset to make the actual meet the\nintended.
  2. \n
  3. You may be extruding too much plastic. Depending on your\nslicer, you can adjust this by tweaking. I user slic3r, and can\nadjust the filament diameter. Overstating the diameter causes\nunder-extrusion. Understating the diameter will cause over\nextrusion.
  4. \n
  5. Sometimes the bumps come mostly at changes of direction. \nThis is possibly caused by several things, but I'd look first for\nbacklash. The mechanism should be tight, and not move under\nmoderate side-to-side or front-to-back force. If it jiggles, it\nwill can cause problems. Wiggle it. Look at what moves. Should it\nmove? If belt driven, the belts should be tight. If driven with a\nlead screw, the nut should not move and the screw should not move. \nIf they do, fix it somehow.
  6. \n
  7. If you are running the motors near the maximum speed and/or\nacceleration, their torque will be lower than if they are running\nslower. Try slowing everything down by a factor of two and see how\nthe problem changes.
  8. \n
  9. It might help to print at a higher temperature, so the nozzle can\nmelt through the bumps with less force. This could cause other\nproblems, but if you are marginally cool now, warmer might work\nbetter.
  10. \n
\n" }, { "Id": "5049", "CreationDate": "2017-11-29T05:59:45.217", "Body": "

I expected that the movement of an axis would start slowly, become faster and then run at a constant speed when reaching the maximum speed, then slow down slowly and arrive at the target point at the lowest speed.

\n\n

At my DIY machine, however, I achieve a constant, very low speed at the beginning, then jumpy change to the maximum speed, finally a also jumpy change to the low speed, which also happens at the beginning.

\n\n

I've been working on the Marlin settings for days, but I haven't had the slightest success.

\n\n

Why don't I get any rising and falling ramps, why the sudden change?

\n\n

These are my current Marlin settings:

\n\n
G21    ; Units in mm\nM149 C ; Units in Celsius \nFilament settings: Disabled\nM200 D1.75\nM200 T1 D1.75\nM200 D0\n\nSteps per unit:\nM92 X800.00 Y640.00 Z800.00 E500.00\n\nMaximum feedrates (units/s):\nM203 X200.00 Y200.00 Z12.00 E25.00\n\nMaximum Acceleration (units/s2):\nM201 X5000 Y5000 Z1000 E10000\n\nAcceleration (units/s2):\nP<print_accel> R<retract_accel> T<travel_accel>\nM204 P3000.00 R3000.00 T3000.00\n\nAdvanced: S<min_feedrate> T<min_travel_feedrate> B<min_segment_time_ms> X<max_xy_jerk> Z<max_z_jerk> E<max_e_jerk>\nM205 S0.00 T1000.00 B20000 X10.00 Y10.00 Z1.00 E5.00\n
\n\n

What's the secret of a beautiful ramp?

\n", "Title": "Acceleration with Marlin", "Tags": "|marlin|acceleration|", "Answer": "

Your steps/mm settings are very high. Assuming you are running an ATMEGA based controller, like RAMPS, you will only be able to move at very slow feed rates (<20mm/s). There are also many hardware factors that influence your maximum speed at a given steps/mm (which is typically referred to a your maximum step rate):

\n\n\n\n

If you aren't sure how to set/select/tune the things above it's best to just mimic what is done on common printers like the Prusa i3, which have robust designs.

\n\n

In more direct regard to your Marlin settings, your acceleration and feedrate values are very high for an untested printer. It's best to start with conservative values (Accel ~500mm/s^2, Feedrate <10mm/s) and work your way up 10% at a time until you start having issues, then back off ~20% from there.

\n" }, { "Id": "5053", "CreationDate": "2017-11-29T16:07:25.517", "Body": "

I\u2019m trying to have this bracket printed, but I don\u2019t know what settings I should use.

\n\n

\"Google

\n\n

The project details say 50% for infill, but is there a reason why I wouldn\u2019t get 100% for making it sturdier? I imagine the developer used 50% because he used his own printer and wanted to preserve more material. The 3D printer service I\u2019m using doesn\u2019t charge more for 100%.

\n\n

But I\u2019m more concerned about the material I should select. Should I select PLA or ABS? The developer didn\u2019t specify this.

\n", "Title": "What infill settings and material type do I need?", "Tags": "|pla|abs|", "Answer": "

You should use PLA at 215 \u00b0C (for better layer adhesion) with 35 percent infill and a shell thickness of 2 mm. Shell thickness is what's crucial here, not necessarily infill. The thicker the shell, the stronger the part. This has a much bigger influence on strength than infill does.

\n" }, { "Id": "5062", "CreationDate": "2017-12-01T21:18:41.577", "Body": "

I have been working at converting game files into 3d files that can be printed, but many of the models have very thin or walls. I was wondering if there was a way to increase the thickness of the walls using meshmixer or meshlab.

\n", "Title": "How to make walls thicker using meshmixer or meshlab", "Tags": "|3d-models|", "Answer": "

I figured out how to do it by using the make solid tool.

\n\n

Edit: I selected the whole model using control and selected make solid. Set the mode from fast to accurate. I set the solid accuracy and mesh density to anywhere from 300 to 500. Then I slowly increase offset distance until the holes are sealed. I leave the minimum thickness at 0 because it doesn't appear to help much. When I'm satisfied, I click accept. I usually use the reduce feature to make the file smaller, but it isn't required. Sorry about leaving an unhelpful answer. Hope this helps people.

\n" }, { "Id": "5066", "CreationDate": "2017-12-02T19:39:42.413", "Body": "

I recently bought an Anet A8 (https://pevly.com/anet-a8-3d-printer-review/). I've managed to get everything up and running, leveled the board, but am now running into a problem.

\n\n

At the start of the print, the printer moves to 0,0,0, bumps into the switches a couple times (I assume to calibrate or so?), and then starts \"printing\". But the nozzle \"randomly\" moves to either an X of 0 or an Y of 0 before returning to the printing position. This movement seems to pull off any basis the printer managed to lay down, which then forms a nice \"ball\" on the nozzle, to which the rest gets stuck. (I'm still having some other issues with getting the filament to stick to the bed, but there's plenty I still have to try out for that.)

\n\n

During one attempt of printing a very simple small cube, I carefully pulled the filament \"ball\" from the nozzle while it did one of those movements to X 0, and afterwards it managed to lay down the bottom layer perfectly fine. This causes me to believe those movements are the biggest problem I'm facing right now.

\n\n

After it did the first layer, it moved up a bit, moved to X 0, back to the model, and got stuck on a piece of plastic that was standing upwards.

\n\n

These movements seem to happen at around the same phase in the print, and happen quite consistently. Is this normal behavior? If so, how do I make sure the filament does not get pulled off during these weird movements? If not, how do I get rid of them?

\n\n

(No, not a duplicate of Printer randomly moves to home during printing, then resumes as normal as I print directly from PC.)

\n\n
\n\n

Edit to add more information:

\n\n

I use Cura 3.0.4 for printing, the stock Anet A8 firmware, and am attempting to print the cube model that comes with Windows 10. (Yes, I've tried different models, same result.)

\n\n

I seem to have more issues, in the video it's visible that the feeding does not seem to work too great, but I think the random movements are the most clear and biggest problem right now, so I should tackle that first.

\n\n

In Cura I've used the Pruisa I3 printer, with the following G-codes:

\n\n
G21 ;metric values\nG90 ;absolute positioning\nM82 ;set extruder to absolute mode\nM107 ;start with the fan off\nG28 X0 Y0 ;move X/Y to min endstops\nG28 Z0 ;move Z to min endstops\nG29\nG1 Z15.0 F9000 ;move the platform down 15mm\nG92 E0 ;zero the extruded length\nG1 F200 E3 ;extrude 3mm of feed stock\nG92 E0 ;zero the extruded length again\nG1 F9000\nM117 Printing...\n
\n\n

and end

\n\n
M104 S0 ;extruder heater off\nM140 S0 ;heated bed heater off (if you have it)\nG91 ;relative positioning\nG1 E-1 F300  ;retract the filament a bit before lifting the nozzle, to release some of the pressure\nG1 Z+0.5 E-5 X-20 Y-20 F9000 ;move Z up a bit and retract filament even more\nG28 X0 Y0 ;move X/Y to min endstops, so the head is out of the way\nM84 ;steppers off\nG90 ;absolute positioning\n
\n\n

(Yes, I added in the G29 in the start code manually, as I bought the official auto-leveling sensor. I'm not sure if it works though, but I read somewhere that I might need a different version of the firmware to support it properly.)

\n\n

And here's a video showing what my printer does do exactly. It started printing from the center in this case, it seems to randomly either move to the middle or to 0,0,0 when I abort the print.

\n", "Title": "Nozzle always \"randomly\" moves to 0 during print, then resumes", "Tags": "|pla|extruder|heated-bed|ultimaker-cura|", "Answer": "

Formating SD card fixes issue for me.\nI think that writing many times to the same block of SD card corrupt some block of flash SD card memory.

\n" }, { "Id": "5068", "CreationDate": "2017-12-02T21:52:21.840", "Body": "

I drew a fairly simple model in Google SketchUp. I exported it as an STL. I imported it into Cura and exported as gcode. Then I printed the model.

\n\n

All of the bottom layers of the model cover the entire space instead of leaving the two open gaps that should exist. I don't know why it's happening. Do I have some weird setting in Cura?

\n\n

The section that is filled, but shouldn't be, isn't a raft. I printed without a raft because my model goes to the max extent that my printer can print and I don't have room for a raft around the edges.

\n\n

Here is what it looks like in SketchUp:

\n\n

\"SketchUp

\n\n

Here is what it looks like in Cura. The two empty areas are shaded darker than the other empty areas of the build plate. Is that an indication of my problem?

\n\n

\"Cura

\n\n

Here is what was printed. The empty areas are solid and filled in as if the center piece extended to the inner edges of that area. (The jacked up corners are due to the model warping a bit and no longer being in the right position for subsequent layers.)

\n\n

\"enter

\n\n

Here is a video of the layers according to Cura, and Cura clearly knows the gaps should be there:

\n\n

https://youtu.be/r0a9gGFerHI

\n", "Title": "Empty space in model is getting filled", "Tags": "|3d-models|ultimaker-cura|", "Answer": "

You have a problem with your STL file. Check the orientation of the faces of your 3D model, if that is not the problem, search for a software to fix your stl file. Just a tip, give Fusion360 a try, it is the best!

\n" }, { "Id": "5087", "CreationDate": "2017-12-08T02:11:42.683", "Body": "

I have seen that the new Prusa Mk3 design has a optical mouse sensor that can be used to detect if the filament jams. Optical mouse sensors are just a really low resolution camera with no color.\nI am interested in finding out if it is possible to get access to the image data coming from that sensor? \nCould I add a rgb led and interpolate the filament color by comparing pixel intensity under different lighting conditions?\nAnd I was interested in seeing if the image data could be used to measure the filament width?

\n", "Title": "Prusa Mk3 filament detection sensor, can you access the image data?", "Tags": "|prusa-i3|firmware|", "Answer": "
\n

I am interested in finding out if it is possible to get access to the image data coming from that sensor? And I was interested in seeing if the image data could be used to measure the filament width?

\n
\n

No. The sensor is the PAT9125EL. The only output it provides is the movement in the X and Y directions. There is no way to get image data out of it.

\n
\n

Could I add a rgb led and interpolate the filament color by comparing pixel intensity under different lighting conditions

\n
\n

No. The sensor uses laser light of a specific wavelength. It's likely not sensitive to any other wavelengths. On a positive note, there is an "average frame brightness" register that can be read from the chip, but I don't think it's likely to work with RGB LEDs.

\n" }, { "Id": "5093", "CreationDate": "2017-12-09T14:38:18.940", "Body": "

I have just received my QIDITECH printer, I've have ABS, PLA and two reals of PETG. I printed a sample using the clear PETG and this worked find, I didn't have to change any of the default settings.

\n\n

Today I've tried three times to print a simple model using the black real of PETG. I've created which is essentially just a rectangle, but after a few minutes the extruder starts to drag around filament.

\n\n

What can I do? The filament is loaded ok, and it starts out ok, but very quickly messes up.

\n", "Title": "QIDITECH Dual Extruder and PETG", "Tags": "|filament|petg|", "Answer": "

It was a while back I created this post, since then I've have numerous problems / learning curves and now I'm very pleased to say I have a printer that prints perfectly and is set-up correctly.

\n\n

I must say that the support from QidiTech is first class and during my journey I was sent a new extruder assembly, new extruders and also new print matts, all completely free and covered by the warranty.

\n\n

I have now printed numerous models successfully.

\n" }, { "Id": "5094", "CreationDate": "2017-12-09T19:35:44.960", "Body": "

I've printed the same model at least twice and both times it was impossible to separate from the base material it was printed on.

\n\n

I'm using a QIDITECH dual extruder, when I start the print it uses the right side extruder to print a base layer, which is not part of the original model, it seems MakerBot issues this instruction to the printer.

\n\n

Once the initial layer has been printed it then prints the model.

\n\n

Both extruders are loaded with PETG. In the left side I am printed with a transparent material. The model I am trying to print is a flat rectangle 2 mm thick.

\n\n

The problem is that once it has completed printing I'm finding it impossible to separate from the initial layer it put down.

\n\n

How do I solve this problem?

\n", "Title": "Model stuck on base", "Tags": "|3d-models|", "Answer": "

The \"base\" you are referring to is normally called a \"raft\". It's not very common to print with a raft these days (it was originally used to help with bed adhesion, but the current-day standard is to use a heated bed if available in combination with some surface preparation (hair spray, PEI sheets, etc...) to aid adhesion); you might consider just disabling the raft.

\n\n

Alternatively, you could consider (since you have a dual extruder) to print the raft in a different material that is more easily removed from the main material. There exist materials that are specifically formulated to break away easily from your print, and another option is to use a (water-)soluble material for the raft and any supports.

\n" }, { "Id": "5099", "CreationDate": "2017-12-10T20:44:48.570", "Body": "

I have created a design with fusion 360. I tried to print it with my M3D Pro printer but it seems the designs have a flaw.

\n\n

If I try to print the bottom part, the second layer is wrong. It seems to have moved to the left by a few centimeters.

\n\n

Here is the link to the .stl files and pictures of how the first few layers turned out.

\n\n

https://seafile.fmk.me/d/09e43aa7fc8e416ab187/

\n\n

\"broken\n\"broken\n\"bottom.stl\"\n\"top.stl\"

\n\n

The bottom.stl file can be loaded and viewed in the m3d software but leads to faulty print after the first layer. The top.stl cannot even be viewed in the M3D software.

\n\n

Other 3D files from thingiverse etc. can be printed without a problem.\nDo I need to enable any special features to be able to print those files correctly?

\n\n

Yours, Felix

\n", "Title": "Fusion 360 M3D Slicer", "Tags": "|3d-models|3d-design|software|slicing|", "Answer": "

Apparently my x belt and/or my x motor is broken. Getting replacement parts fixing the problem!

\n" }, { "Id": "5103", "CreationDate": "2017-12-11T16:15:24.397", "Body": "

I am building a 3d Printer here:

\n\n\n\n

Problem is, I can't get V-slots or T-slots easily where I live.\nI can get Steel Rods or Seamless pipes, and Bearings & Drylin bushings.

\n\n

What aluminium profiles can I use to construct a rigid frame and accurate movement?

\n", "Title": "What else can I use for a 3D Printer frame?", "Tags": "|diy-3d-printer|printer-building|", "Answer": "

Were I to build a frame now, I might use 1\" square steel tubing members, welded together. You could also use aluminum, but for the same cross-section, the steel will be stronger. It is also easier to weld. If welding is not available, you are limited to shapes which can be bolted together. This is not impossible, but it requires more thoughtful design.

\n\n

For the guides for moving or sliding elements, stainless steel drill rod is good. Because it is round, it doesn't constrain rotating motion, so you might need two drill rods, spaced as far apart as your design allows. The twisting force will be harder to manage with rods than it would be with a solid element. If the parts are moving during printing, you could consider stainless steel tube or aluminum rod to keep down the weight.

\n\n

If you have the machining ability, instead of rods you could use a T cross-section (steel or aluminum) with wheels rolling on the two outside elements of the T. Machine is likely needed to adjust the profile of the edge to more closely match the wheels (which could be /V\\ shaped), and to make the separation be accurate enough that you wouldn't need a spring element to clamp to the T.

\n\n

There are many materials and forms that could work. You are limited only by your fabrication ability and access to materials.

\n" }, { "Id": "5110", "CreationDate": "2017-12-12T16:40:41.067", "Body": "

I want to print worm gears for my robot arm. I would like to design it in FreeCAD. Normally, I would use involute gear for regular gears. However, this tool cannot be used for worm gears. I can't find any add-ons for this.

\n\n

Is it possible to produce a worm gear automatically? If not, how can I make it manually?

\n", "Title": "How to design a worm-gear in FreeCAD?", "Tags": "|3d-models|cad|freecad|", "Answer": "

As far as I know there is not a workbench capable of producing the design you want with a single click \"new worm gear\". But it is a rather simple affair to create the part you want from scratch.

\n\n

What you want to do is to sweep a sketch along a helix. It is a very similar process to the one you would follow to create a thread documented in the official tutorial (it's the \"method #3\" on that page). This is how it should look like:

\n\n

\"enter

\n\n

Since helices are subject to a few limitation in FreeCAD, I recommend to read the section called \"tricks to success\" and the following tips, as it is very likely you will incur in problems otherwise.

\n" }, { "Id": "5119", "CreationDate": "2017-12-13T22:10:49.067", "Body": "

I'm considering getting a prusa i3 after my da vinci jr. fiasco, with that said, how easily can I remove/replace the nozzle in order to clean it?

\n", "Title": "How easily can I remove/replace the nozzle of the prusa i3?", "Tags": "|prusa-i3|nozzle|", "Answer": "

Assuming you have a typical setup: nozzle threaded into heater block, use a socket wrench on the nozzle and any wrench or vise-grip on the block, and unscrew the nozzle. Now, if your system is fully clogged, you may want to heat the head to melt all the residual gunk (which otherwise will strongly resist your unscrewing force). Obviously take care not to burn yourself if removing the nozzle while hot.

\n\n

Alternatively, don't bother removing the nozzle. Raise the extruder as high (Z-axis) as you can, heat, and use a properly-sized stiff wire to ream out the nozzle. You can use the back end of a microdrill bit. Hot or cold, you can use the drill itself to ream, but be very cautious as you may widen the nozzle itself doing this.

\n\n

I should add that you may well have gunk in the feeder tube, and that's considerably wider diameter. Remove the nozzle and gently drill out the feeder with a matching drill.

\n" }, { "Id": "5128", "CreationDate": "2017-12-15T12:41:20.943", "Body": "

I'm using PETG, the thickness of the filament is 1.75mm, in MakerBot there is a setting for \"Layer Height\", the default is 0.1mm, I've had lots of messed up prints with this material and I'm thinking it could be this setting that needs adjustment.

\n\n

What layer thickness should be used?

\n\n

Found this: http://forum.makergear.com/viewtopic.php?f=11&t=2593

\n", "Title": "Makerbot layer thickness", "Tags": "|replicating-printers|", "Answer": "

Unlike PLA, PETG does not like to be \"squidged\" down, it likes to be laid down. If you use too small a layer height, there is the danger of the filament sticking to the nozzle, rather than the bed (or filament already laid down). Try using a larger layer height -- between 0.2mm and 0.3mm, if you have a 0.4mm nozzle.

\n" }, { "Id": "5131", "CreationDate": "2017-12-15T20:50:23.290", "Body": "

I'm very new to 3D printing and I've had numerous failed prints. Sometimes, the print is good but while trying to get it off the base I end up bending or breaking the print.

\n\n

My question is: How to I print and make it easier to release from the plate?

\n\n

I'm printing with PETG, PLA and ABS. At the moment, it's PETG that is giving me trouble.

\n\n

My printer is a QIDITECH Dual Extruder.

\n\n

I found this:

\n\n

https://all3dp.com/1/remove-3d-print-from-bed-stuck-glass/

\n\n

https://www.youtube.com/watch?v=V6fudqMEGyI

\n\n

I haven't tried any of the proposed solutions yet.

\n", "Title": "Tips for printing and being able to get of base easily", "Tags": "|post-processing|replicating-printers|", "Answer": "

I use multiple layers of Elmer's white glue on the glass bed (five layers I think). The glue is PVA which provides a textured surface onto which the PETG adheres. It sticks better when the bed is hot than when the bed is cold. I use a bed temp of 58 degrees on my Anet A8. I have found out that the temp reported by the Anet is lower than the temperature that I measured the surface of the glass to be with my temperature probe. The PETG removes some of the layers of PVA when it comes off. For this reason I'm constantly needing to add more glue to the area that was printed, until the surface is so uneven that I have to remove the glass plate, clean it and start over again. If you take this advice, never flip the glass plate over onto the bare aluminum with the glue on it, as it will stick very firmly!

\n\n

My colleague in work who has the same Anet A8 uses a glue stick in place of white glue. He also has a good experience printing PETG in this manner. I have tried his glue stick method, and found it difficult to clean when it's time to clean the glass. Now I live by my white glue method.

\n\n

To apply the glue I use a foam artist's brush to make the layer nice and even. I let each layer dry before I apply the subsequent layers. After each use I quickly wash the brush because the glue will harden in the foam and cause the brush to wear out.

\n\n

I also use this method with PLA, and TPU.

\n" }, { "Id": "5139", "CreationDate": "2017-12-17T09:01:34.023", "Body": "

I am writing my own slicer and wonder if there is a mathematical proof that proves that the intersection of the slicing plane with the STL file will only produce closed-loop polygons for every given slicing plane?

\n\n

Thanks!

\n", "Title": "Proof that Slicing Plane/STL intersection will only produce Closed-Loop Polygons?", "Tags": "|software|", "Answer": "

I am also trying to write my own slicer and so I would just like to offer some additional information.

\n\n

What you will find is that .stl files just store triangles and don't validate that the triangles assemble to create a printable model so a lot if not most .stl files are going to have data that isn't exactly what you want it to be. So you will want to have additional steps before you get to slicing, to fix up the data you get from the stl file.

\n\n

A lot of stl models are designed for 3d graphics where intersecting other triangles and not being manifold isn't an issue. So you will probably want to write some code to test your models before trying to slice them.

\n\n

For example if you just sliced the layers of the Utah Teapot\n \"Teapot\" and don't repair the model for 3D printing first you will get two different types of errors. The handle of the teapot does not actually attach to the teapot and is a separate model. And the spout of the teapot is a separate model that intersects the teapot. Both the handle and spout are also open at the ends and not water tight.

\n" }, { "Id": "5145", "CreationDate": "2017-12-18T19:54:47.410", "Body": "

I am using Cura to slice my prints, and despite turning the flow rate to the minimum value of 5%, my prints are still hugely over-extruding.

\n

I have calibrated the extruder stepper perfectly, using Pronterface, so I do not understand why this is occurring. I have also timed how long it took to extrude a certain length and compared it to the length of time it was meant to extrude and it was exactly the same. Therefore, I have concluded it is not a problem with the calibration of the stepper.

\n

So, I think there is a problem with the settings on Cura. Originally, I had the flow rate at 100% and this was really, really terrible. Then I turned it down as far as possible and the print got better but there was still over-extrusion. I can't down it down any further.

\n

I can not figure out what the problem could possibly be and as you would imagine it is very, very frustrating.

\n

Here is the print profile:

\n

\"Screenshot

\n

So the printer is not of any model, as it is a homemade CD drive 3D printer. It shares many similarities with the Curiosity3D printer, so if you want more information on how it works, then their website will be of much value.

\n

The extruder is a Bowden-style one. It uses a cheap E3D hotend and a RepRap extruder kit as the motor.

\n

Here are the Machine settings:

\n

\"Screenshot

\n

This is a photo of two failed prints. On the left is a G and on the right is a heart.

\n

\"Photo

\n

This is what it was the G was meant to look like:

\n

\"Pronterface

\n

So here is my configuration.h file which I previously modified for my 3D printer. The filament I use is "Robox PLA SmartReel Cornflower Blue".

\n", "Title": "Why is my 3D printer over extruding when I have set the flow rate very low?", "Tags": "|print-quality|ultimaker-cura|diy-3d-printer|extrusion|", "Answer": "

I've been building 3D printers for a few years and working in the computer science field for over 2 decades. Here's a simple trick I use for adjusting your extruder steps.

\n
    \n
  1. Tweak your flow rate till it's where it should be. Then mark down\nthe percentage it's at.

    \n
  2. \n
  3. Go into your configuration file and use that percentage to adjust the number of steps. So if $x$ is the number of steps and our flowrate is at 40\u00a0% to be extruding normal then:

    \n

    $x * 0.4 =$ new step count

    \n
  4. \n
  5. Save the file and compile.

    \n
  6. \n
\n" }, { "Id": "5157", "CreationDate": "2017-12-20T18:55:06.917", "Body": "

I want to draw a cube with cone using Onshape.

\n\n

I need the cone to \"shave\" the corners of the cube, till it gets to the middle point.

\n\n

This is diagram I made:

\n\n

\"Sketch\"

\n\n

What is the best way to do it?

\n", "Title": "How to draw cube with cone (Dreidel like), using Onshape?", "Tags": "|3d-models|3d-design|", "Answer": "

After rejecting the intense interrogation process for signing up with Onshape, I've settled for a general description based on common practices in 3D modeling.

\n\n

You would create the rectangular solid representing the cube, as well as another rectangular (or cylindrical) solid slightly larger than the first one.

\n\n

\"three

\n\n

Create also a cone shape with the dimensions appropriate to the segment you wish to have remaining on the first solid.

\n\n

Using Boolean operator or the equivalent, subtract/difference the cone shape from the second solid, resulting in a \"pencil point\" shape of empty space in the second solid.

\n\n

\"cone

\n\n

The skirt of the cone is barely visible in the image above and is transparent, the TinkerCAD\u2122 version of subtraction.

\n\n

Once grouped in TinkerCAD\u2122, the entire cube becomes a subtractive body.

\n\n

\"cutting

\n\n

Place it in the appropriate position over the first solid and perform a second subtraction/difference or Boolean equivalent.

\n\n

\"adjusted

\n\n

As one may note, the original cube was not tall enough and had to be stretched to provide the desired result. Not visible in this image is that the subtractive shape was not centered on the cube and resulted in an off-center final solid. Easily corrected with alignment tools, but an important aspect when performing Boolean operations. Control-Z is your friend.

\n\n

\"final

\n\n

The above steps work with OpenSCAD\u2122 as well as Fusion360\u2122 and even within TinkerCAD\u2122 but the specific steps/sequence varies from one program to the other, of course. This example was performed in TinkerCAD\u2122 because it was the fastest method. Fusion360\u2122 would have stretched my abilities to accomplish the same result, due to my minimal experience with that platform.

\n\n

I am not surprised to discover and easily locate an exact tutorial for OnShape\u2122 that covers the boolean subtraction process. The images are complex within the tutorial, but the process and concepts are identical. The video accompanying the tutorial is also well done and explains clearly the steps involved.

\n" }, { "Id": "5160", "CreationDate": "2017-12-21T12:07:04.233", "Body": "

Most of time my prints fail due to lack of support and \"base\" for the 3D model. I'd like to know if there is any software that can automatically check my model, and add support and a \"base\" to it, if required.

\n", "Title": "How to automatically add support and \"base\" to the model I am printing?", "Tags": "|3d-design|software|", "Answer": "

This program type you seek for is called a slicer. However, you got to manually choose support structure and rafting - the tricky part is getting the configuration right for your build. To my knowledge there are no slicers that decide automatically to raft unless you configure to raft always and you have to enable support.

\n" }, { "Id": "5161", "CreationDate": "2017-12-21T12:43:04.930", "Body": "

I upgraded from Cura 2.7 to 3.1.0 and I'm getting horrible under extrusion, I'm sure this is the software because I rolled back to 2.7 and everything is working fine again.

\n\n

My printing is a Robo3D R1+ using the \"custom FDM printer\" profile.

\n\n

Is there any new setting or a setting that isn't migrated properly that causes this?

\n", "Title": "Under extrusion with Cura 3.1", "Tags": "|ultimaker-cura|", "Answer": "

I hit this issue again in January 2019 with CuraEngine 3.6+git. Since support for multiple extruders was added, it now auto-loads a per-extruder settings file that overrides the main settings file and the command line, so -s material_diameter=1.75 on the command line did not help. Given $foo.def.json, it seems to construct the filename %$foo_extruder_0.def.json, and if that does not exist, loads fdmextruder.def.json, which sets material_width to a default of 2.85.

\n\n

I was able to solve the problem by either patching fdmextruder.def.json, or making a copy of it named to match my printer's settings file, with the 2.85 changed to 1.75.

\n\n

Update: There should already be extruder files for supported printers, but they're in a different directory, so they won't be found when invoking CuraEngine from the command line unless you've set the CURA_ENGINE_SEARCH_PATH environment variable to include the path they're in. Alternatively you can copy them to the same directory as the main printer json files.

\n" }, { "Id": "5170", "CreationDate": "2017-12-22T22:12:51.127", "Body": "

I have built a couple of 3d printers now and I'm having a little trouble with one I'm currently on. I frankensteined this one with two of my other printers that I had built. I took the Arduino mega and ramps 1.4 out of the older one and wired it up to the newer one. When I plug the printer in and try to move the stepper motors they will go in the positive direction and skip and make loud noises when going the other direction. I have taken and separated out all the wires to make sure it wasn't a stray signal. I have also tried turning the voltage up and down on the drivers with no luck. I also swapped cables in case one might have been broken. The next step when I get home I'm going to try is connecting the old steppers and see if they work. After that I am pretty much at a loss. I already search Google and found a couple of things I could try, but figured maybe someone here might have an answer to why this might be happening. Steppers motors from the old printer are the same as the new one the only difference is manufactures.

\n\n

Update:\nThe printers are:\nHE3d Prusa XI3\nMax Micron Foldbot

\n\n

I'm just using the LCD, Ramps 1.4 and Arduino Mega 2560 R2 from the He3d and putting it on the Foldbot. With some other features but those shouldn't matter as they would work on both printers. The board from the Foldbot is the Arduino Mega 2560 built into the Ramps board MXP_PRO_V3.0. I'll try and get some pictures. Also switching steppers didn't work. Going to try to get X,Y,Z axis working separately see if i can find the problem that way.

\n\n

Update: Ok so I attached the old axis up and did a test with the endstops. Basically I think the new endstops must be the opposite of what the other printer was cause when i hold the end stop down the motor rotates in both directions and when i let go then it rotates only in a positive direction. I will see if this is part of the issue. I also believe that one of the motors is bad and that's why it wasn't making any sense when i first diagnose it.

\n", "Title": "3D printer stepper motors only going one direction and skipping in the other", "Tags": "|prusa-i3|diy-3d-printer|reprap|repetier|prusa-i3-rework|", "Answer": "

Had a similar issue with one of the stepper motors on a Flashforge Creator Pro.

\n\n

Just finished a print as the head was returning to the home position and sounded like it was jammed up, inspected in jog mode and it would move left but not right then neither direction, disconnected, check for free movement and all was good found this thread and was thinking it might be a bad motor but turned out to be just a bad wire going to the motor.

\n" }, { "Id": "5171", "CreationDate": "2017-12-24T09:42:04.190", "Body": "

I just bought new TEVO Tarantula and tried to print xyz cube. I found that my cube's layer was shifted as showed in picture. How can I fix this ?

\n\n

\"enter

\n", "Title": "XYZ cube's layer is shifted, How can I check my TEVO Tarantula axis?", "Tags": "|heated-bed|calibration|axis|", "Answer": "\n\n

To adjust the current on your stepper driver, this is a classic scheme:

\n\n

With power on:

\n\n

1) Turn the potentiometer on the driver counter clockwise until you can move the motor by hand.

\n\n

2) Turn clockwise until you can't turn the motor by hand

\n\n

3) Continue clockwise until you can turn the motor again (as soon as you feel it can't hold really hard anymore, stop).

\n\n

4) Turn 1/4 of a turn (or maybe a bit less) counter clock wise.

\n\n

Done!

\n\n

If the current is too low, it will skip steps, especially when traveling / accelerating fast/hard.

\n\n

If the current is too high, the motor will be, really, hot. A bit hot is okay, like 50\u00b0C can be considered a really safe maximum (it all depends of course, some motors can run at 80+ others could bake your electronics because it's placed badly etc.).

\n\n

If this doesn't cut it, you want to check your drivers; If you can, switch X and Y driver:

\n\n\n\n

I also encourage you to post more information about print speeds, temperature and so on. It seems like it's a Prusa i3 clone and there are a lot of people having them and they can probably help better if this didn't cut it so please post all data available!

\n" }, { "Id": "5176", "CreationDate": "2017-12-26T11:29:35.203", "Body": "

I work for a company that makes items from plastics.
\nMany or our current runs are between 500 and 5000 copies, but knowing the company, if we find a good method to do smaller runs, they are willing to see if it is a good commercial option.

\n\n

At the moment we do use several different methods but the technical people are not yet looking into 3D printing.

\n\n

While I am not sure printing is the right option just yet, I would be surprised if it will not be in the future.

\n\n

At this time I am interested in finding information to convince the tech people to look into the abilities of printers and what would impress them to look further would be printers that can produce in short times or at multiple stations so the overall run will be relatively short term.
\nOur current items are mostly simple in shape, (disks with relief print) and small in size (no bigger than a 2 pound coin).

\n\n

Do you know an online magazine where the tech people can look or can you suggest a (few) printer(s) to look at right now?
\nLinks to online general information or names to search for will be great.

\n\n

Knowing our current bunch of tech people they will likely prefer commercial available printers but proven 'home build technology' might be useful as well.

\n", "Title": "Where to find information on 'fast' or 'professional' 3D printers?", "Tags": "|rapid-prototyping|", "Answer": "

3D printing may absolutely be a viable technology for what you are trying to achieve. The term you should search online is printing farm or 3D printing farm. A typical farm looks like this.

\n\n

The reason you normally want to set up a farm is that - despite 3D printing being often associated to the expression \"rapid prototyping\" - 3D printing is anything but fast, and operating several machines is an easy way to increase the throughput. (On a side note: the term \"rapid\" in \"rapid prototyping\" refers to the fact that there is little to no overhead between the design and production phases, as opposed to the need to create a mold, or send out technical drawings to a machining shop, for example).

\n\n

The right technology to be used (i.e.: what types of 3d printing you would need in your farm) is entirely dependent on the requirements and characteristics of the printed parts. There are so many different 3D printing technologies, and each technology has so many variables attached to it that it would be silly for me anybody else to state with certainty which one would be best for you (your \"tech people\" will likely spend a lot of time evaluating their choices), but to give you a sense of the complexity of the problem, I could mention that FDM/FFF printers are very cost-effective, quite slow, can print in a variety of materials, have limited resolution, suffer from wear, produce anisotropic parts while SLA printers have incredible resolution, can't print large parts, struggle with solid objects, emit toxic fumes, are expensive to buy and operate, etc...

\n\n

Be advised that the list of 3D printing technologies is not limited to those two (especially when it comes to industrial settings): FDM and SLA are the most known technologies as there are several consumer-grade printers using them, but DLP (Digital Light Processing), SLS (Selective Laser Sintering), SLM (Selective Laser Melting), EBM (Electronic Beam Melting), LOM (Laminated Object Manufacturing), BJ (Binder Jetting), MJ (Material Jetting) and others are also available... each with its own pros and cons.

\n\n

When it comes to source of inspiration and information, I have to disagree with the suggestion made by another responder that Make Magazine would be a good resource for forming an opinion. Make Magazine is a magazine targeting hobbyist, and as such it pitches and explores technologies that are geared towards enabling creativity. What you should be after is information on 3D printing in a commercial setting / on an industrial scale, as the requirements of a hobbyist printing their own drone are very (very!) different from those of a company needing to meet quickly, reliably and consistently a customer's specifications.

\n\n

3D printing technologies evolve continuously and quickly, so - if you are after printed material - it is essential for you to get hold of something published recently. Off the top of my head, The 3D Printing Handbook: Technologies, design and applications that came out a couple of months ago seems to be an excellent match for your current needs of forming an opinion / acquiring information (the link is to Amazon, and allows you to browse its index online). Keep in mind it was put together by 3D HUBS the largest network of manufacturing services in the world... so it is not some random guys' opinion!

\n\n

A couple of additional considerations that I would also keep in mind:

\n\n\n\n

Hope this helps! :)

\n" }, { "Id": "5183", "CreationDate": "2017-12-28T09:06:54.543", "Body": "

I'm trying to print a cylinder with Flashprint. Problem is, that the .stl file i created (with OpenSCAD) is totally ruined once loaded into Flashprint. With ruined I mean the round outline is now with spikes and steps everywhere. And by loaded I mean directly after loading, not even creating the .gx files. The output of OpenSCAD looks fine, also in other stl viewers I tried.

\n\n

I figured out the problem occurs more when using certain number of fragments ($fn). If its very small, loading works better. But for numbers that make it acutally look like a round circle (eg 50), Flashprint is unable to load them correctly it seems.

\n\n

It's obvious that its no slicing or printing problem, but simply a loading one, since the shown object (which does not look any round) also prints the way it is shown in flashprint before slicing.

\n\n

Does someone know a way to load a cylinder with flashprint correctly, or do I have to switch to different software? I already installed some but am unfamiliar with the printer settings which I do not know in detail. I really though a cylinder should'nt be too complex for any 3D software.

\n\n

Here is the OpenSCAD Code for Cylinder, just create .stl and load into Flashprint to reproduce the Error:

\n\n
rotate_extrude($fn=70)\n    translate([0, 0])\n        square([20,2]);      \n
\n\n

See also this image (tried native cylinder code, problem remains the same however):\n\"\"

\n", "Title": "OpenSCAD to Flashprint ruins model based on number of fragments", "Tags": "|3d-models|software|resolution|", "Answer": "

While my first try with slic3r was really bad, due the required manual setup and my lacking experience (which was the reason I really wanted to stick with Flashprint), I actually found a solution to my problem. Its more a workarround but im perfectly happy with it for now.

\n\n

I just load the OpenScad .stl with slic3r, use it's \"Export STL...\" feature, and load the resulting stl with Flashprint. Looks like a charm.

\n\n

So the problem is really the OpenSCAD specific .stl files being misinterpreted by Flashprint.

\n\n

Hopefully this helps some of you as well and save you some trouble.

\n" }, { "Id": "5191", "CreationDate": "2017-12-30T22:28:28.170", "Body": "

I'm getting a Prusa i3 with a replaceable nozzle.

\n\n

If I have a nozzle clog, can I easily get rid of it by simply:

\n\n\n\n

Or can the clog get so bad that the whole extruder has to be cleaned, and if so how could I clean it?

\n", "Title": "If I have a nozzle clog, can I easily get rid of it by simply replacing the nozzle?", "Tags": "|extruder|nozzle|", "Answer": "

The nozzle on a Prusa i3, if a genuine Prusa, would be part of the E3Dv6 assembly and is removable. It's wise to heat the nozzle to 150\u00b0C or higher, and handle with the care necessary for something that hot. When loosened, be prepared for it to fall onto the surface below. A catching tin would be a good idea, at the very least.

\n\n

Once removed, you can test that it's the nozzle that is clogged by pushing or extruding filament via the control panel to confirm that the rest of the system is clear.

\n\n

Consider also to perform a nylon cleaning of the nozzle before you perform the above disassembly. I've performed this process more than a few times and disagree with one aspect of the process. The process describes heating the nozzle to 250\u00b0C and forcing nylon filament into the hot end. I suggest doing so manually, not via the extruder controls, as it give you better feedback and control.

\n\n

Once you've forced as much through as you can, allow the hot end to cool. Not included in the steps reference above is a part of the process that is semi-automated on my printer. Heat the nozzle to about 140-150\u00b0C and carefully and slowly pull the filament back out of the hot end. The above link suggests yanking it sharply. The article also includes a caution that doing so is brutal and could damage your machine. Carefully and slowly will not damage your machine. I have to use pliers to get sufficient grip on the nylon but that's a trivial aspect.

\n\n

The nylon will come out with debris and previous color filament. Clip off the debris and perform the task again. Repeat until the nylon is clean and you have good flow through the nozzle.

\n\n

\"nylon

\n\n

Acetone will not dissolve PLA filament. Extreme heat may carbonize the material in the nozzle and render it useless. The nylon cleaning method is nearly universally successful unless you have a physically damaged nozze.

\n\n

There are kits to be purchased which are described as nylon cleaning kits, and include a small length of nylon filament and sometimes a cleaning drill. Using such a drill may damage the nozzle and is contraindicated. If you can afford to purchase a half-kilogram or full kilogram of nylon in the diameter specific to your printer, it will last quite some time. You can share segments with friends and/or members of your local makerspace and improve circumstances for many.

\n\n

Nylon absorbs moisture more quickly than any other filament, making storage for extended periods challenging if you intend to print with nylon, but for cleaning, it matters little that there's moisture boiling out of it as you clean your nozzle. I've found the little bubbles in the extruded nylon to be a sign that I'm getting clean nylon through the nozzle and can expect good results.

\n\n

In the worst case I've had thus far, I've had to push nylon four times to clear a clogged nozzle. I had some overheated PVA support material jam the nozzle and once cleared, everything flowed properly.

\n" }, { "Id": "5197", "CreationDate": "2018-01-01T12:48:08.313", "Body": "

So, some background. I bought a e3d Lite6. When assembling the hotend, the PTFE tube does not even go in 1/4th of the way. I found out that the issue was that the PTFE tube was too wide. It measured 4.26mm (outer diameter) and the heat break is only 4.1mm (inner diameter). How should I make my PTFE tube smaller?

\n", "Title": "How should I make my PTFE tube smaller?", "Tags": "|hotend|ptfe-tube|", "Answer": "

I would advise buying a new tube for a few reasons:

\n\n
    \n
  1. PTFE tubes should be either 4 or 6 mm external diameter (for 1.75 or 3 mm filament respectively). Chances are that if your external diameter is that much off from the nominal value, the inner diameter will also be inaccurate, and this could cause a lot of problems during printing, as the filament will likely bend slightly and unpredictably within the tube and the pressure in the nozzle will be unstable, as well as retraction may work inconsistently.

  2. \n
  3. A non all-metal extruder is designed with the assumption that the inner and external sections of the PTFE tube are concentric. This may or may not be the case for your tube, but it is likely that any hand-made modification to the tube will cause the above not to be true.

  4. \n
  5. PTFE tubes are pretty cheap (around 2\u20ac per metre or less, when bought online).

  6. \n
\n\n

That said, should you still want to manually modify your tube, I would proceed like this (beware: untested!):

\n\n
    \n
  1. Insert the end of the tube onto a drill bit that fits snugly into it.
  2. \n
  3. Insert the drill bit into a drill press or a drill held in a vice or otherwise immobilised.
  4. \n
  5. Let the drill spin the bit/tube at a moderate speed
  6. \n
  7. Move a piece of fine grit paper up and down the portion of the tube on the bit, trying to apply consistent pressure.
  8. \n
  9. Check your progress with a caliper or a micrometer often, and reduce increase the grit while approaching the desired result.
  10. \n
\n\n

Best luck! :)

\n" }, { "Id": "5198", "CreationDate": "2018-01-01T20:52:19.030", "Body": "

I was thinking about what would happen when flexible filament is frozen. Would it become brittle or still be rather tough*. A situation I think of would be a ice tray in the freezer. It is nice to have some flex to get the ice out, but PLA and other filaments wouldn't work, but will flexible filament work?

\n\n

*when I say 'tough' I mean having similar properties when unfrozen.

\n", "Title": "Flexible filament frozen", "Tags": "|filament|", "Answer": "

The behaviour of \"frozen filament\" will entirely depend from the specific formulation of it.

\n\n

The term \"flexible filament\" encompasses a variety of different polymers as for example: thermoplastic elastomers like TPE and TPU (e.g.: ninjaflex), copolymers (e.g.: bendlay), copolyesters (e.g.: Ngen Flex), polycaprolactones (e.g.: PCL), etc...

\n\n

Even in those broad classes of chemicals, the amount, type and quality of additives will affect the physical properties of the filament a lot. In fact rigid.ink even produces a flexible PLA that proves the point of additives radically affecting the properties of the main material.

\n\n

In general, all materials lose elasticity at lower temperatures (a Space Shuttle came down because engineers failed to account for this). Polymers that do not contain water are unlikely to crystallise though, so I would expect it to become stiffer but not to fail catastrophically at 0\u00b0C.

\n" }, { "Id": "5200", "CreationDate": "2018-01-01T23:09:19.817", "Body": "

I have a prusa 13 that's shipping in the mail, and I intend to make good use of it, one also own a da vinci jr. and the one time it got so clogged that the extruder itself was filled with pla, with that said I replace the extruder, for the da vinci, but besides that, as for my a prusa, what should I do if the extruder, not the nozzle gets clogged that badly?

\n", "Title": "What do you do If the whole extruder is clogged?", "Tags": "|extruder|", "Answer": "

If you are good at being patient and extremely careful, you can clean out the feeder tube by using a small drill, manually operated. Use a bit with diameter smaller than the filament in use.

\n\n

I recommend first removing the nozzle. Then,from the top, hold the drive gear open and gently work the drill bit down, removing often to clean off debris. If the bit is long enough, sooner or later it'll project out the hole where the nozzle fits. If not, gently work from the bottom up.

\n\n

Since you have the nozzle out, it's easy to see if there's gunk in the upper chamber of the nozzle; either drill that out or use heat-methods to liquify, and clean out with any soft tool.

\n\n

Usually any clog in the narrow orifice can now be removed by re-assembling, heating the extruder head to operational temps, and pushing the remaining gunk out simply by guiding fresh filament all the way in. I do not recommend trying to use a micro-drill bit as that can easily damage or open up the orifice. If you can get a piece of stiff wire (e.g. 0.375 for a 0.4 nozzle), try that once the head is at operational temperature.

\n" }, { "Id": "5215", "CreationDate": "2018-01-03T18:39:38.413", "Body": "

I have just received a 3D printer for Christmas (Robo R2). I am confused by the sheer amount of settings that I can tweak and I'm hesitant to do so until I know more about them. I was wondering if anybody has any recommendations for literature on:

\n\n\n\n

Books are preferred but websites are acceptable as well.

\n", "Title": "3D printing references for beginners", "Tags": "|knowledgebase|literature|", "Answer": "

I have found that \"Troubleshooting and Maintaining your 3d Printer\" by Charles Bell has a good overview of:

\n\n\n" }, { "Id": "5220", "CreationDate": "2018-01-04T13:32:02.013", "Body": "

I have successfully assembled my custom-built 3D printer and configured Marlin for two extruders and one heated bed. Here is a picture of the printer.

\n

\"enter

\n

My heated bed runs on a linear axis with ball bearings. When the printer has been running for an hour or so these parts get really hot and I am afraid that the plastic parts will melt if I print any longer or with higher temperatures. So I decided to add fans below the heated bed to keep them cool.

\n

A known problem when using two extruders and a heated bed is that all three power outputs D8, D9, D10 are in use (in my setup D8 belongs to the first extruder, D9 to the bed, and D10 to the second extruder). If you want to have software-controlled fans on top of that, you need to use a workaround. I bought the RRD fan extender which does exactly what I need. You plug it into the RAMPS 1.4 board and get two new outputs D6 and D11.

\n

\"Picture

\n

Previously I had configured the firmware as follows:

\n
#define E0_AUTO_FAN_PIN 11\n#define E1_AUTO_FAN_PIN 6\n
\n

This automatically enabled the fan of the left extruder E0 when its hotend exceeds 50\u00a0\u00b0C. The same goes for the right extruder E1. The fans are plugged into the fan extender's outputs D6 and D11. It all worked fine.

\n

Now to add fans to the heated bed, I have modified the firmware so that D11 controls both extruder fans. As long as at least one extruder is hot, both fans are running. For that purpose, I connected both extruder fans in parallel to D11 and modified the firmware as follows:

\n
#define E0_AUTO_FAN_PIN 11\n#define E1_AUTO_FAN_PIN 11\n
\n

This part works fine and was quite easy to achieve. What I would like to do next is connect the other pin, D6, to the temperature sensor of the heated bed so that the fans underneath it become active when the bed is at 50\u00a0\u00b0C or more.

\n

I made several attempts to trick the firmware into believing that there are three hotends, registering the heated bed as E2.

\n
#define E2_AUTO_FAN_PIN 6\n
\n

I manually defined the temperature sensor of the bed for E2 and commented out some sanity checks and conditionals to enable some parts of the firmware that control the auto-fans. While I get the code to compile, the printer usually halts immediately after it is turned on or as soon as an extruder or the bed is activated. The error messages are not very helpful ("killed, please restart" etc.).

\n

Does anybody know a good way how to achieve my goal? Any help would be appreciated. Thank you in advance.

\n", "Title": "How to configure Marlin to enable auto-fans with dual extruder", "Tags": "|marlin|ramps-1.4|dual-nozzle|print-fan|", "Answer": "

After trying many different things, I found out that the solution is quite simple and requires only a few lines of code. I'll answer my own question in the hope that this will help someone.

\n

First, I defined a few constants (macros actually). To keep my own additions separate, I created a new file for them called myconfig.h:

\n
#define MY_BED_TEMP_THRESHOLD 50\n#define MY_BED_AUTO_FAN_PIN 6\n#define MY_BED_AUTO_FAN_SPEED 255\n
\n

The pin constant corresponds to D6 which is the green marked output of the RRD Fan Extender where I connected the fans under my bed.

\n

\"enter

\n

Second, in the file temperature.cpp of the Marlin Firmware, I included my file and added four lines of code:

\n
#include "myconfig.h"\n\n... \n\n#if HAS_AUTO_FAN\n\n  void Temperature::checkExtruderAutoFans() {\n  \n    ...\n\n    HOTEND_LOOP() {\n      if (current_temperature[e] > EXTRUDER_AUTO_FAN_TEMPERATURE)\n        SBI(fanState, fanBit[e]);\n    }\n\n\n    // --- start of my code ----------\n    if (current_temperature_bed > MY_BED_TEMP_THRESHOLD)\n      digitalWrite(MY_BED_AUTO_FAN_PIN, MY_BED_AUTO_FAN_SPEED);\n    else\n      digitalWrite(MY_BED_AUTO_FAN_PIN, 0); \n    // --- end of my code ------------\n\n    ...\n\n#endif // HAS_AUTO_FAN    \n...\n
\n

Now my fans automatically turn on when the bed temperature is above 50\u00a0\u00b0C, and turn off again when the bed has cooled below that temperature.

\n" }, { "Id": "5221", "CreationDate": "2018-01-04T23:16:56.747", "Body": "

I am designing my own shield for the MEGA2650 that I will use to control my printer.

\n\n

When defining pins in the marlin firmware, the pin numbers are the pin numbers of the RAMPS shield and not the Arduino MEGA itself.

\n\n

How can I define pins so that I could write #define HEATER1PIN 5 and it will make heater1pin be on Arduino MEGA's pin 5 and NOT RAMPS's pin 5

\n", "Title": "How do I use MEGA pin number designations rather than RAMPS pin numbers in marlin", "Tags": "|marlin|reprap|firmware|arduino-mega-2650|", "Answer": "

The pin mapping that Marlin uses is defined in the file fastio_1280.h:

\n\n
/**\n * Pin mapping for the 1280 and 2560\n *\n *   1280     22 23 24 25 26 27 28 29 53 52 51 50 10 11 12 13 37 36 35 34 33 32 31 30 21 20 19 18 81 82 83 38 00 01 78 05 02 03 79 80 54 55 56 57 58 59 60 61 41 40 39 71 70 04 17 16 84 06 07 08 09 85 15 14 72 73 75 76 77 74 62 63 64 65 66 67 68 69 49 48 47 46 45 44 43 42\n *   Port     A0 A1 A2 A3 A4 A5 A6 A7 B0 B1 B2 B3 B4 B5 B6 B7 C0 C1 C2 C3 C4 C5 C6 C7 D0 D1 D2 D3 D4 D5 D6 D7 E0 E1 E2 E3 E4 E5 E6 E7 F0 F1 F2 F3 F4 F5 F6 F7 G0 G1 G2 G3 G4 G5 H0 H1 H2 H3 H4 H5 H6 H7 J0 J1 J2 J3 J4 J5 J6 J7 K0 K1 K2 K3 K4 K5 K6 K7 L0 L1 L2 L3 L4 L5 L6 L7\n *   Marlin   00 01 02 03 04 05 06 07 08 09 10 11 12 13 14 15 16 17 18 19 20 21 22 23 24 25 26 27 28 29 30 31 32 33 34 35 36 37 38 39 40 41 42 43 44 45 46 47 48 49 50 51 52 53 54 55 56 57 58 59 60 61 62 63 64 65 66 67 68 69 70 71 72 73 74 75 76 77 78 79 80 81 82 83 84 85\n*/\n
\n\n

Confusingly, the line labeled \"1280\" refers to the pin numbers as used by Marlin. The line labeled with \"port\" gives the corresponding port/pin combination as it applies to the AtMega2560 chip itself. To translate between the port numbers given above, and the Arduino Mega pin numbers, you can use the following pinout diagram:

\n\n

\"Arduino\nLicensed under CC-BY-SA, www.bq.com

\n\n

For instance, according to the pin mapping in fastio_1280.h, Marlin pin 34 is \"port\" C3. This corresponds to PC3 in the pinout diagram, which is Arduino Mega A11. Conversely, if we want to find out what PWM pin 9 is in Marlin, we can look at the diagram to find out it corresponds to PH6, which is Marlin pin 09.

\n\n

If you want to change this numbering, then unfortunately, you have to make quite a few modifications to fastio_1280.h. For instance, the \"reason\" pin 5 corresponds to the pin it does, is the following set of definitions:

\n\n
#define DIO5_PIN    PINE3\n#define DIO5_RPORT  PINE\n#define DIO5_WPORT  PORTE\n#define DIO5_DDR    DDRE\n#define DIO5_PWM    &OCR3AL\n
\n\n

If you wanted to point pin 5 towards another pin, you'd need to change all of these lines to point to the correct registers and pin numbers.

\n" }, { "Id": "5224", "CreationDate": "2018-01-05T14:54:12.177", "Body": "

I just finished building my Anet A6 and I was working on inserting some filament into the extruder. It was very difficult to get the filament to go in the hole (past the gears to go down to the hot-end). What can I do that will make it easier to get the filament into the hole (I tried cutting the tip at a angle)?

\n", "Title": "Problems with feeding the filament into the bowden tube", "Tags": "|extruder|anet-a6|", "Answer": "

What you are after is a small common mod called... filament guide (as your question title!).

\n\n

The first one to pop up in my google search was this one: https://www.thingiverse.com/make:346736 which in turn is a make of this model: https://www.thingiverse.com/thing:2242903

\n\n

Also, a couple of tricks that help on my printers (YMMV):

\n\n
    \n
  1. manually straighten the first few cm filament before inserting it into the extruder (e.g.: remove the natural bend that is there because the filament came off a round spool by bending it in the opposite direction)

  2. \n
  3. when the filament is past the gears/cogs, while still keeping the cogs \"open\" (i.e.: not yet clamping the filament), twist/roll the filament between your finger.

  4. \n
  5. sharpen the tip of your filament with a pencil sharpener. This make so that the tip of it is at the very centre of the hole, rather than at its edge.

  6. \n
\n" }, { "Id": "5226", "CreationDate": "2018-01-05T20:34:40.817", "Body": "

Since i updated Cura from 2.7. to 3.1. I have a really hard time at configuration and especially removing the support interfaces. The update automatically uses the settings from the old version, but as I said: it doesn't work anymore.

\n\n

I even tried to create a test print with 4 copies of a test object with different settings, but there where no differences in the slicing for them. Every copy and the support interface that belongs to it was the same.

\n\n

Did anybody have an idea?

\n\n

Printer Anet A8 with PLA

\n", "Title": "Problems with support interface since cura 3.1", "Tags": "|ultimaker-cura|support-structures|", "Answer": "

In Cura 3.1 to 3.2.1 (date today is 2018-Feb-24): The Support Z Distance only changes the Support Bottom Distance and NOT the Support Top Distance. By activating the two sub-options you can configure this again.

\n" }, { "Id": "5234", "CreationDate": "2018-01-07T16:24:26.623", "Body": "

The bottom of my prints warp/curve upwards, most often at the corners. This is a very slight curve, only about 1-2\u00a0mm.

\n\n

What slicer settings might be the cause of this phenomena?\nOr could it be 3D printer settings?

\n", "Title": "Warping of bottom of print", "Tags": "|warping|", "Answer": "

The first thing to understand is what causes warping. Warping is caused by the thermal contraction of the plastic when it cools down.

\n\n

Simplifying things a fair bit, you can visualise the process like this:

\n\n
    \n
  1. hot, expanded plastic gets deposited on cooler, shrunk layers,
  2. \n
  3. when the hot plastic cools down, it shrinks and pulls the upper part of the layer below inwards
  4. \n
  5. at this point, the layer below has a differential in the compression between its upper and lower parts, and curls up
  6. \n
  7. the problem is exacerbated at the very first layer (the one touching the bed) as this is \"locked\" to a rigid body (the bed) and cannot shrink, while subsequent layers are only attached to the somewhat flexible plastic beneath, and thus can contract.
  8. \n
\n\n

\"diagram

\n\n

Also notice that the larger the part being printed, the stronger is the force trying to curl-up your print.

\n\n

Once one understands all of this, then it is possible to appreciate the many ways the problem can be mitigated.

\n\n

Here are the common ones:

\n\n

USING A MATERIAL WITH LOW SHRINKAGE COEFFICIENT

\n\n

This translates in smaller tensions and thus less force \"pulling up\" the corners of your print. Historically, 3D printing started with ABS because this material was one of the very few, relatively safe ones to source. Nowadays there are materials like PETG which have similar mechanical properties to ABS but are much easier and forgiving to print with, so - unless you need ABS for some very specific reason (e.g.: acetone smoothing) consider never printing with it.

\n\n

DECREASING THE THERMAL DIFFERENCE BETWEEN MOLTEN AND SOLID STATE

\n\n

Concretely, this means lowering the \"gap\" between the ~200\u00b0C of the nozzle and the ~20\u00b0C of room temperature by using a heated bed and - possibly - an enclosure.

\n\n

The heated bed not only drastically diminish the shrinkage of the first layer, but because heat radiates, and hot air goes upwards, the entire bottom of the print has shrinkage mitigated.

\n\n

An enclosure just increase the benefit of the heating bed, by reflecting IR radiation back towards the print and preventing hot air to escape. A heated enclosure just improve things even further.

\n\n

Some slicers offer a \"shroud\" option, that encloses the entire print in an enclosed, sacrificial structure, that tries to emulate the benefits of a proper printer enclosure.

\n\n

INCREASING ADHESION WITH THE PRINTING BED

\n\n

That is the \"brutal force\" approach: if you face a strong \"curl up\" force, oppose it with a strong \"anchor down\" one.

\n\n

The increase in adhesion can be achieved in a number of ways:

\n\n\n\n

REDUCING THE CURL-UP FORCE

\n\n

This is typically achieved during design. Designing is a vast field and it would be impossible to cover all the possible mitigating strategies one could use, but here are some of the most common ones:

\n\n\n\n

\"I

\n\n\n\n

\"warping

\n\n

Of course all of the above strategies can/should be combined, when possible. Even if not warped, a part with a lot of internal tension will perform less predictably and possibly worse than a part where such tensions are lower.

\n" }, { "Id": "5239", "CreationDate": "2018-01-07T21:54:45.857", "Body": "

I'm new to 3d printing and I recently bought a Geeetech Prusa i3 Pro B along with a BL Touch (3D touch) sensor to do auto bed leveling.

\n\n

I printed the sensor holder, mounted it and the sensor to the extruder carriage and uploaded a configured Marlin firmware to the printer. It worked great for about 6 times and now, it crashes on the bed on the two points on the right of the bed.

\n\n

I tried manually leveling the bed thinking that the nozzle was touching the bed plate before the sensor's needle could, but that's not the case. Even with the bed manually leveled, when using G29 (Auto Bed Leveling code), the Z-axis carriage crashes into the bed.

\n\n

What can I do/check?

\n", "Title": "Auto bed leveling with BL Touch sensor crashing to bed", "Tags": "|prusa-i3|calibration|bed-leveling|z-probe|", "Answer": "

As it turns out, my Z-axis carriage mount to the X-axis was bent, causing the nozzle to touch the bed before the proble could get a chance to detect anything. Manually bending it back to the correct angle solved the problem.

\n" }, { "Id": "5241", "CreationDate": "2018-01-07T22:57:58.193", "Body": "

Whenever a 3D printer that uses linear rails is announced (case in point: the cetus), the Internet (well... at least that corner of it dealing with 3D printing) gets abuzz with excitement.

\n\n

I researched a bit the topic myself, and while I understand that linear rails can be produced to a fantastic degree of precision for super-heavy machinery, it escapes my comprehension why they are considered superior \"by default\", relative to the classic linear bearings on a shaft.

\n\n

3D printing is a lightweight application, and the motion of at least 2 axis does not happen against a solid surface (where you could bolt a linear rail every few cm) but suspended between the 2 ends of the axis. Furthermore, the internals of the bearings used on linear rails are substantially identical to those used on a shaft.

\n\n

The cetus site says under the heading \"Quality Linear Rails\":

\n\n
\n

Self-lubricated | Maintenance Free | High Precision | Long Lifespan | Quiet

\n
\n\n

but this - in my experience - can be said of \"Quality Linear Bearings on a Shaft\" as well, and in some cases even bushing deliver to a high standard on 3D printers.

\n\n

So, what am I missing?

\n", "Title": "Why all the excitement about linear rails?", "Tags": "|hardware|axis|linear-motion|", "Answer": "

Linear rails will always produce a high degree of accuracy and stability and more so than round rod with PTFE bushings and/or bearings.

\n\n

One may argue the fact however even as a product developer and one who is involved with the mechanicals and development of machinery on a day to day basis comparing the two we see \u201csignificant\u201d improvements using rails over rods and if properly utilized on a Z axis print bed you will have a bed free of leveling issues that can smoothly and accurately operate with one driver versus two.

\n\n

I will further add that getting rods aligned perfectly is a difficult task for the average person and even a slight twist or angular position can affect final print quality. I\u2019ve seen many linear rods that appear visually straight and when chucked into a lathe spindle with 0.0000 accuracy there will be 0.005 or more in runout. In fact I\u2019ve yet to see one perfecting concentric motion that is longer than 6 inches. This tells me that they cannot plausibly be as accurate and that while they may function they will never function with a high degree of accuracy.

\n\n

Do we need higher accuracy in 3D printer axis? Sure we can have quality control boards that compensate to some degree however the mechanics of the machine are utmost important before you choose the quality of board and software. Why install a \\$300 motion control on a cheap linear rod printer if you\u2019re not going to see the full benefits?

\n\n

With technology further advancing into 64 bit and eventually 128 bit and higher degrees of precisional accuracy 3D printing is turning a page and will if not already be capable of micro resolutional accuracy and can only do so if all the components function properly together.

\n\n

So sure, your rod guided printer works. However, it will never work as well as my linear rail guided printer with ballscrews and servos. You can have your layered fuzzy prints. I will keep my smooth finished injection molded looking parts that are made from materials a typical desktop cannot even print. So to argue it\u2019s not needed is arguing that high quality isn\u2019t accepted in a lower price point market.

\n\n

One other addition here. Ask yourself how level and square is your printer? I\u2019m not talking about using a carpenter's level for checking your machine they can be inaccurate up to a 1/4\u201d per 10 feet. When you can dial your printers bed down to 0.00005 or less every direction and your structure is just as accurate than you know what a quality printer and print looks like.

\n\n

I guarantee no printer priced \\$300-\\$1000 comes even close to that degree of accuracy. The average consumer is so drawn into the technology of a final print itself they overlook the precisional aspects involved and learn to settle for less. Then you wouldn\u2019t expect your \\$500 printer to compete with my \\$10,000 printer either.

\n\n

Bottom line you get what you pay for.

\n" }, { "Id": "5245", "CreationDate": "2018-01-09T13:23:01.660", "Body": "

After a long time tweaking my new 3D printer I solved all the unexpected errors and I can print succesful pieces but I am facing a \"problem\", they are over sized, I found this problem trying to print a Raspberry Pi Case and an smartphone case, the printed pieces are bigger than the objects.

\n\n

Here some related information

\n\n

Printer: Geeetech Prusa i3 Pro B

\n\n

Firmware: My GitHub (Marlin)

\n\n

Software: Repetier Host with Slic3r

\n\n

Does somebody know how could I solve it?\nThank you very much

\n", "Title": "Oversized pieces", "Tags": "|3d-models|marlin|stepper-driver|", "Answer": "

I finally solved it by manually adjusting the steps per unit in the printer firmware.

\n\n

The process is easy just move the extruder to one position and draw two marks, one at the extruder position and another one 10 cm in the X axis from the extruder position then with Repetier or with the the printer controls move the X axis 100 mm, if the extruder stops after the mark reduce the steps per unit or increase them if the extruder stops before the mark, do until the movement matches both marks. Repeat for the Y axis.

\n\n

You can see the changes I made to the firmware in my GitHub

\n" }, { "Id": "5246", "CreationDate": "2018-01-09T13:40:35.500", "Body": "

I got an Anet A8 for Christmas.

\n\n

I am trying to print a calibration tower and the z axis stops lifting at approximately 100mm.

\n\n

I checked that the gantry is able to travel along the full length of the threaded rods, and while resetting the bed height I noticed that the LCD display showed the extruder height above the bed. It reads 0 at home (good) but goes back to zero when I manually turn the rods so that the gantry goes above the heigh at which the print stopped.

\n\n

Any thoughts on what to check? I am double checking all the mechanical bits (connection to stepper coupling, ease of turning, etc...).

\n\n

My slicer is Cura 3.1.

\n", "Title": "Z axis stopped part way through print", "Tags": "|marlin|ultimaker-cura|z-axis|anet-a8|", "Answer": "

I found two issues:

\n\n\n" }, { "Id": "5247", "CreationDate": "2018-01-09T16:21:06.117", "Body": "

I have a Monoprice Select Mini v2 and it came with a 256 MB SD card. I have a bunch of 16 GB cards. I have made sure that the new SD card has a FAT32 filesystem. I copy the gcode file onto this card and when I put it in the printer, it can't find any files!

\n\n

And yes, the file is at the root level of the filesystem and it uses the proper naming convention. The file works on the old card.

\n\n

Since the old card still works, this isn't an emergency, but I want to have a backup and I don't have any other cards that small.

\n", "Title": "What is the largest microSD card that a Monoprice Select Mini can read?", "Tags": "|firmware|hardware|monoprice-select-mini|microsd|", "Answer": "
    \n
  1. Get a copy of Partition Wizard (Free)
  2. \n
  3. Take your 16G or 32G or ...? SD card and delete the existing partition.
  4. \n
  5. Create a 2G partition and format it as FAT.
  6. \n
\n\n

Yes, you are wasting a lot of the SD card space but given the price of these things these days, who cares. And it will work.

\n" }, { "Id": "5251", "CreationDate": "2018-01-10T01:29:52.727", "Body": "

I have really strange problem. Thing is that my print (first layer) started ok, not good nor perfect but ok and everything was going well but then all of a sudden, near the end of a print, quality drops drastically. I'm not really sure but I think this happened because of under extrusion. I'm not so good with English so here are pictures of finished print and some more details.

\n\n

\"enter

\n\n

\"enter

\n\n

I'm using custom build Delta printer with RepRap and Repetier firmware, CURA for slicing and Repetier-host for printing.

\n\n

Slicing parameters in CURA are:
\n- ABS 250 °C hotend and 70 °C heatbed
\n- Layer height 0.2 mm (initial layer 0.18)
\n- Printing speed 50 mm/s (30 mm/s outer walls)
\n- Infill 40 %
\n- Extrusion multiplayer 0.96 (96 %)

\n\n

Do anyone have any ideas? What this can be? How can I fix this?

\n", "Title": "Under extrusion towards the end of the print", "Tags": "|print-quality|reprap|extrusion|repetier|delta|", "Answer": "

I had a similar problem with PLA after upgrading to all metal hotend. My printer was in a cabinet and after some time in print had the same problem (heat creep). In the end, it was resolved by turning bed off after first 3 layers and turning on a fan to keep the cabinet cool.

\n" }, { "Id": "5257", "CreationDate": "2018-01-10T16:00:19.173", "Body": "

I am still at calibration stage and need some info from the PCB. I connected the USB and ran Repetier. The PCB wants to talk at a higher baud rate than my serial port says it can do. I tried setting the serial port to its highest setting 125k and reduced the PCB baud in Repetier setting to 125k. No joy. PC port reverts to 9600 every time I check it. Thoughts?

\n\n

PC running Windows 7 Home Premium 32bit.

\n", "Title": "Connecting Anet A8 to PC issue", "Tags": "|anet-a8|", "Answer": "

My A8 just started doing this.

\n\n

I shut down repetier server, and voila, all is good! Since I will go do OctoPi eventually, I just uninstalled RS.

\n" }, { "Id": "5263", "CreationDate": "2018-01-11T05:47:28.070", "Body": "

I'm relatively new in the field of 3D printing and design. By now I've created and printed some technical objects with TinkerCAD, but now I've a task, which I don't know how to solve.

\n\n

I have the following model as STL-file:\n\"enter

\n\n

Now I want to \"adjust\" the arms of the model, as shown in the picture. I want the arms to hang besides the body.

\n\n

I know that I could cut and rotate the arms and then merge them again with TinkerCAD but the outcome dosn't look good and the workflow feels wrong.

\n\n

So what is the right tool/way to get this task done?

\n\n

*Disclaimer: I'm not Denis Almaral, but he released this model unter CC license. So I kept his name on the image to credit him, as requested via CC.

\n", "Title": "How to modify 3D model of body?", "Tags": "|3d-models|3d-design|cad|stl|", "Answer": "

The correct/good method to achieve this is called \"rigging\", but it is not an easy feat (as pointed out by others), as it requires plenty of knowledge about the software being used to edit the model, and a good understanding of the theory behind it.

\n\n

Skeletal animation requires the designer to set up a skeleton (also called \"rig\", hence the slang term \"rigging\") for the mesh and define the variables controlling the motion range of the joints and and the geometry and deformation of the mesh.

\n\n

It's quite some job to perform, so - unless this assignment is the first in a series of assignments having to do mesh movement for this model, or you have a keen interest in the topic - I would suggest you to take a shortcut and edit the mesh directly in a \"one off\" not reusable fashion.

\n\n

(Rigging would conversely allow you to create a \"rag doll\" or at least a \"rag arm\" to freely and intuitively move around as you please).

\n\n

The tool I would use for either task is blender. If you want to take the shortcut, maybe a less complex software like meshmixer could also do the job (I don't have direct experience with it, though, so I'm not 100% sure).

\n\n

The blender foundation have a nice series of videos on the topic, called \"humane rigging\".

\n" }, { "Id": "5268", "CreationDate": "2018-01-11T22:11:07.633", "Body": "

As first layer is so important, I am looking for an easy way to generate the gcode to print just the first layer.\nI see that with Slic3r you can cut from a Z\nBut for test purposes I prefer just selecting a number of layers to be generated so I can easily generate different \"first layer(s) tests\" with different first layer(s) settings (width, height, speed, flow....)\nThe only way I achieve it is editing the gcode.\nAny help?\nThanks

\n", "Title": "How to generate gcode only for first layer?", "Tags": "|slic3r|", "Answer": "

You could just insert an M30 at the end of layer 1, and the job will end there.

\n" }, { "Id": "5270", "CreationDate": "2018-01-12T00:56:30.123", "Body": "

Disclaimer: I have read about this elsewhere but haven't found a solution.

\n\n

Printer: Printrbot Simple Metal with heated bed

\n\n

Slicer: Cura 15.04.6 (also tried Cura 3.1)

\n\n

Printing software: Whatever Cura 15.04.6 comes with when printing from USB

\n\n

The print goes well for the first couple layers then just stops. It stays heated, fan keeps blowing, but print has failed. Here is the error:

\n\n

...\nSend: N19517G1 X47.047 Y59.035 E1574.00486*126\nRecv: ok\nSend: N19518G1 X68.604 Y74.097 E1574.44219*122\nSerial timeout while writing to serial port, trying again.\nUnexpected error while writing serial port: SerialException: 'WriteFile failed ([Error 22] The device does not recognize the command.)' @ machineCom.py:_sendCommand:565\nChanging monitoring state from 'Printing' to 'Error: SerialException: 'WriteFile failed ...'\nConnection closed, closing down monitor

\n\n

I have gotten this using Cura 3.1 to slice as well as Cura 15.04.6 (though the error above was using the Cura 15.04.6). I've tried using a different interface for printing, but nothing helps.

\n\n

Any ideas on solving this? I feel like I am missing a small but simple detail here to making it work.

\n\n

[Edit] Added printing info at top to make it a little more clear what I was using.

\n", "Title": "SerialException: 'WriteFile failed ([Error 22]...' Printrbot Simple Metal using Cura 15.04.6", "Tags": "|ultimaker-cura|g-code|", "Answer": "

From the look of it, and given that you already two different slicers, it look like this may be a hardware issue. I have had very similar error messages with my cheap Chinese printer and this is ultimately why I ended up always printing via the SD card and stop worrying.

\n\n

That said, the error is about the serial connection over your USB cable. I was about to write a list of suggestions, but I found out that your manufacturer already has a troubleshooting guide for your printer here.

\n\n

Adding my own suggestions to those of Printrbot, this is the final troubleshooting guide (the linked page has detailed instructions for each step but 6 and 7):

\n\n
    \n
  1. Cycle power
  2. \n
  3. Update your Operating System.
  4. \n
  5. [Windows users only] run a VCP (Virtual Com Port) driver wizard
  6. \n
  7. Check physical connections and swap power and usb cables.
  8. \n
  9. Use a standard 2.0 USB input rather than USB 3.0, if applicable
  10. \n
  11. Make sure that your printer is away from any potential source of EMI (Electro-Magnetic Interference). Microwaves ovens, many types of energy-saving lamps and power bricks/adapters are known offenders.
  12. \n
  13. If possible, reduce the serial speed of your connection (revert if this does not solve the issue, as it may effect print speed and quality).
  14. \n
  15. Flash your Printrboard
  16. \n
\n\n

To clarify what these tips are all about:

\n\n\n" }, { "Id": "5276", "CreationDate": "2018-01-13T02:59:25.773", "Body": "

Here is my understanding of Coasting: Coasting stops extruding early in a move so that the string itself will finish the layer.

\n\n

Here is my understanding of Combing: Combing reduces the need to retract during travel moves by making sure that the nozzle oozes where you want it to on the way to the next point.

\n\n

I'm curious as to what types of prints these are good for, and also what types of prints these would be bad for.

\n\n

So for instance, Coasting is good for prints that have a high propensity to exhibit stringing, but what types of prints would I want coasting to be deselected for?

\n\n

Similarly for combing, although I know neither the pros nor cons other than it reduces the number of retractions (decreases wear on extruder?)

\n\n

In short, basically I'm looking for the pros and cons of both of these settings. Also if my understanding of the settings themselves is incorrect please let me know. Any advice would be much appreciated.

\n", "Title": "Combing and Coasting", "Tags": "|slicing|", "Answer": "

Coasting is good for filaments that ooze. The stop/up/start time at the end of a layer can be long enough that a visible seam appears if the layer starts in the same place. Transparent filaments also suffer from the velocity effect at layer shifts (more transparent when extruded slower).\nThe disadvantage is that it becomes another parameter to tune per filament.

\n\n

Combing is most useful where a part has internal spaces, but might result in longer travel.

\n" }, { "Id": "5279", "CreationDate": "2018-01-13T17:16:51.197", "Body": "

So I have an MKS Sbase v1.3 that I am trying to use in a delta printer. I am trying to use a PNP inductive sensor with it and it doesn't work. I have tried using a voltage divider with it already (10k ohm and 15k ohm) but it doesn't give any signal when the probe is triggered. Can anyone help me?

\n\n

P.S Yes, I do know that the board has physical pullups. I am thinking of using NPN MOSFETS but I don't know how I should wire them.

\n", "Title": "MKS Sbase Probing", "Tags": "|z-probe|wiring|", "Answer": "

After testing my z-min endstop pins, I found out that I could trigger the probe manually by using a jumper wire between Signal and Ground. I then used a NPN MOSFET to connect the sensor to the endstop by connecting the black wire of the probe to the base, the signal wire of the endstop pins to the collector, and the blue wire from the probe and the ground pin of the endstop connector to the emitter of my MOSFET. Note that my mosfet is a TIP120.

\n" }, { "Id": "5285", "CreationDate": "2018-01-15T05:50:11.280", "Body": "

I'm attempting to model and print a globe. The semi-circular arm that holds the globe has an outer diameter of 98mm and inner diameter of 92mm, so the arm is fairly thin. The arm has a small hole on either end that fits around a protrusion at each pole of the globe to hold it in place. The hole extends into the arm only a few millimeters, so it doesn't go all the way through.

\n\n

I printed the arm in ABS and it ended up being a bit too flexible to hold the globe securely. Obviously I could do things like making the arm thicker or extending the globe's protrusions all the way through the arm, but I'd prefer not to if I don't have to (you know, artistic integrity or whatever).

\n\n

Would printing in PLA result in a more rigid part? I've tried to do some googling on this, but couldn't really find a definitive answer. Most comparisons focus on strength which I assume doesn't necessarily correlate to flexibility.

\n\n

I'd also welcome any other suggestions for making the part more rigid.

\n", "Title": "Increasing rigidity of curved, long, thin parts", "Tags": "|pla|abs|", "Answer": "

Print thinner layers. Also, you did not mention the other dimension of your arm. If you view the globe with the arm to the left, then you have said the thickness in the X direction (left to right) is 6mm. But what about in the Z axis (away from you)? You could make it thicker in that direction to improve its rigidity.

\n" }, { "Id": "5292", "CreationDate": "2018-01-16T14:39:45.943", "Body": "

At first few layers, each layer has offset to -X/-Y direction base on previous layer. But above ~2mm, it is vertical.

\n\n

I've checked the printer build that there is no noticeable error in towers. Tried both manual calibration and Marlin auto calibration G33 with z probe.

\n\n

No idea what's the cause and don't know which part I need to check with.

\n\n

\"enter\"enter

\n", "Title": "Delta printer misalignment at first few layers", "Tags": "|delta|kossel|", "Answer": "

Finally I found out that it was caused by not enough torque output by B tower stepper motor. Increasing current didn't work. I replaced the stepper motor and it prints perfectly.

\n" }, { "Id": "5294", "CreationDate": "2018-01-17T07:15:53.417", "Body": "

So I am trying to get the XSD-Schema from this object. When I open the File I just get something like this code (snippet):

\n\n
\u00b8\u20202\u00a1Q\u00b72\u00bay\u0192eC\u00e32\u00ef\u2026w \u00ef\u00c0|\u00bc\u00f0A\u00f8\u00e4[0\u0178 |>\u201a|\u00f3\u2018\u00e52\u00b2\u00baF\u0192\u00bc\u00c6\u00f21\u00f9\u00e0\u00e5cj@\u00de`\u00f9\u00d0\u00b8\u00cc{\u00e1\u00c8;0/|\u00be \n \u00f9\u00cc'\u0178\u201e \u00c1|d\u00bd\u00ac\u00ac\u00af\u00d5 \u00af\u00b1|l\u00be\u2026\u00ad\u0152o@\u00de`\u00f9\u00d0\u00b8\u00cc{\u00e1\u00c8;0/|\u00be\u00f9\u00cc'\u0178\u201e \u00c1|d\u00bd\u0153\u00ac\u00af\u00d3 \u00af\u00b1|h\\\u00e6\u00ad\u0152o@\u00de`\u00f9\u00b8|\n \u00dfBs\u00a65\u2013\u0152~\u00f4\u00e8\u00bb\u00ad\u00a3(\u2122c\u00b4\u201c\u00c7\u00a3[yp1:\u00e6'\u00c3\u2030c4J\u00f3\u00a0U\u00e2\u02dc\u00cd\u00c7<h\u20148^'\u00d0\u00af\u00c9\n
\n\n

What is this? How can I convert it back? I need to edit the structure manually.\n
Thank you in advance.

\n", "Title": "How can I read this 3mf-File", "Tags": "|3d-models|", "Answer": "

Three likely culprit:

\n\n\n\n

As for the \"wrong encoding\" option... Oversimplifying a bit, the story goes like this:

\n\n\n\n

Back to your question: despite unicode having been around for a few decades now, legacy software and sloppy programming are a thing, and there are still systems that do not use unicode internally but some legacy \"special purpose\" encoding.

\n\n

Unfortunately, it is impossible to say with certainty how a file was encoded, so occasionally you may find yourself opening a file and starting decoding it according to a \"conversion table\" that is not the one used by the author of the file itself. This is what it looks like is happening to you.

\n\n

Onward to what you can try to do to fix this...

\n\n

First of all: as attentive readers may have already inferred, you will need the actual file for this. In fact if you cut-and-paste its \"content\" from an editor what you are really doing is cutting-and-pasting the decoding your computer did of the byte values, and not the byte values themselves.

\n\n

Then your best bets are:

\n\n\n" }, { "Id": "5297", "CreationDate": "2018-01-17T16:30:23.570", "Body": "

I have an odd-shaped object (I'll call it \"A\") that I have imported into a TinkerCAD drawing. Its dimensions do not align to any specific size, and therefore I can't really use the grid lock to help.

\n\n

However, I need to get some other objects (called \"B+\") a specific and precise distance from the outside edge of \"A\". I was trying to get the ruler to lock to the edge of \"A\", but I can only get close.

\n\n

Is there a way to do this so that I can get \"B\" perfectly positioned from the edge of \"A\"?

\n", "Title": "TinkerCAD Align to Odd Shaped Object", "Tags": "|3d-design|", "Answer": "

Consider to construct an object of any suitable shape that will properly align with the edge of \"A.\"

\n\n

The attached image is certainly an odd-shaped object (from a previous test project).

\n\n

\"odd

\n\n

Add a regular/symmetrical object such as a cube and size it appropriately for your purpose. It would not have to be a cube, although such a shape provides for convenient alignment in this case:

\n\n

\"convenient

\n\n

Use the alignment tools feature in Tinkercad to align the desired edge(s). In this case, alignment is common right side, centered top to bottom.

\n\n

\"unaligned

\n\n

After clicking on the alignment button, lower right corner:

\n\n

\"aligned

\n\n

Group the two objects using the Group feature.

\n\n

I created a sample alignment reference in this image and placed it in contact with the group.

\n\n

\"second

\n\n

Move the group or reference item the desired distance:

\n\n

\"moved

\n\n

Select the group and ungroup it.\nSelect the initial alignment item and delete it.

\n\n

\"delete

\n\n

Objective completed.

\n\n

\"exercise

\n" }, { "Id": "5304", "CreationDate": "2018-01-18T17:34:36.383", "Body": "

As long as there's a MOSFET in between, can I supply 24 V bed/extruder heat from an ATX power supply using -12 V and +12 V and ignoring the common rail?

\n\n

I assume so, I can't think of a reason why not, but want to check.

\n", "Title": "24 V heated bed from -12 V and +12 V ATX power supply?", "Tags": "|heated-bed|switching-power-supply|", "Answer": "

No, this is not possible with most ATX power supplies. While in principle you can get a 24V supply by combining the +12V and -12V supplies, the rails are not symmetric, and the negative 12V supply is usually designed for a much lower load than the positive supply. In the example in the following picture, there are two positive 12V rails, capable of sourcing respectively 12A and 15A each, but the -12V rail is only good for sinking 0.5A. If you tried doing what you're proposing, using this supply, you'd be limited to only 0.5A from your \"24V\" supply.

\n\n

\"ATX

\n\n

It is very unlikely that you'll find an ATX power supply with a negative 12V rail capable of sinking significant current since computers don't need such large amounts of current from their negative supplies.

\n\n

There are also issues with using a MOSFET as you describe. I assume that by MOSFET you are referring to a complete board with various support components, and not just a bare MOSFET by itself. Usually these boards have optocouplers and thus they will probably work correctly, but using just a MOSFET by itself this would not work, as the gate needs to be driven below GND (namely: to below -12V plus the gate threshold) to turn the MOSFET off.

\n" }, { "Id": "5308", "CreationDate": "2018-01-20T06:40:23.790", "Body": "

I have been searching around the internet for the last 3 days trying to figure this out. My Z axis motors for a pursa-i3 3d printer are not working correctly. I have marlin firmware and using repetier host. I send a command to move the z axis and I get it to move, however I if I send the same command again the motors will sometimes spin the other way. feel like They almost randomly choose which direction they turn.

\n\n

As I said I have been trouble shooting this for a while now. What I am suspecting is the firmware feedrates and acceleration or some setting is not correct.

\n\n

Here is my code:

\n\n
//// MOVEMENT SETTINGS\n#define NUM_AXIS 4 // The axis order in all axis related arrays is X, Y, Z, E\n#define HOMING_FEEDRATE {50*60, 50*60, 2*60, 0}  // set the homing speeds (mm/min)\n\n#define DEFAULT_AXIS_STEPS_PER_UNIT   {80,80,4000,590}  \n#define DEFAULT_MAX_FEEDRATE          {300, 300, 3, 45}    // (mm/sec)\n#define DEFAULT_MAX_ACCELERATION      {1000,1000,50,500}    // X, Y, Z, E maximum start speed for accelerated moves. E default values are good for Skeinforge 40+, for older versions raise them a lot.\n\n#define DEFAULT_ACCELERATION          1000    // X, Y, Z and E max acceleration in mm/s^2 for printing moves\n#define DEFAULT_RETRACT_ACCELERATION  1000   // X, Y, Z and E max acceleration in mm/s^2 for retracts\n\n#define DEFAULT_XYJERK                10   // (mm/sec)\n#define DEFAULT_ZJERK                 0.3     // (mm/sec)\n#define DEFAULT_EJERK                 5.0    // (mm/sec)\n
\n\n

I tried swapping the drivers around and the motors will work perfectly on another axis so I don't suspect it to be a driver issue. I have been turning the pots ontop of the drivers to make them work but can't make them to go the same direction i want them to.

\n\n

I've checked the wires and I almost sure they are wired up correctly. (could be wrong but have checked it over with a multimeter.)

\n\n

I am new to this and it's my first time building one of these would appreciate any help I can get and and maybe I have over looked something I have tested.\nJust really want the axis to move in the direction That I say it to move in.

\n\n
\n\n

More details about my setup is: A Robocraze 3D Printer Controller Board RAMPS 1.4 using A4988 stepper motors drivers and my motors are the nema 17 stepper motors. I currently have the two z axis motors wired in parallel but have tried before using series, however the problem of being unable to control the direction of the Z motors still arises (can easily switch back to series).

\n\n

currently trying with no load just to get the motors turn in the correct direction when I send a G-code command. I am using Repetier host on ubuntu 14.04.5.

\n\n

I have also check the endstops and they are working perfectly, so they ain't a problem (I don't think :p)

\n\n

Thank you,\nBobby

\n", "Title": "Z axis Stepper motors not working correctly", "Tags": "|diy-3d-printer|reprap|ramps-1.4|z-axis|", "Answer": "

So after 5 days of trouble shooting, Bob-the-Kuhn over on the marlin github forum solved it for, anyone else who faces the same issue can head over to github for my solutions.

\n

https://github.com/MarlinFirmware/Marlin/issues/9287#issuecomment-359428147

\n

Conversation from link:

\n
\n

Bob-the-Kun:\nProblem does not follow the driver. Problem does not follow the\nsteppers.

\n

I'm thinking that the Z socket has a problem. Sounds like an open/poor\ncontact.

\n

Try bending the DIRECTION lead on the Z driver a little and see if the\nproblem disappears. It's one of the corner pins. Sometimes it's called\nDIR. If your driver's pins aren't labeled then bend all four corner\npins a little.

\n

Another option is to move the Z function to the E1 socket. Replace\nyour pins_RAMPS.h file with this one. pins_RAMPS.zip

\n

FYI - if this really is a hardware problem then it's the second RAMPS\nhardware problem within a week. Most unusual.

\n

Post reply:

\n

Yes the socket I am now assuming is just broken (not sure what exactly\nbut possibly one of the connections), After using for pins_RAMPS file\nand changed the motors back to series and connected to the E1 slot I\nsuccessful got the printer to work!!! Thank you very much Z axis is\nworking as I would expect! I am now calibrating the printer as it\ndefinitely needs it.

\n
\n" }, { "Id": "5312", "CreationDate": "2018-01-20T13:16:53.573", "Body": "

I've have observed that when the printer is finishing a layer, the flow of plastic through the nozzle starts fading out as it comes closer to the point of layer change.

\n\n

As an example, let's say that I'm printing the first layer of a cube. The nozzle first prints the perimeters ok. Then it begins to print the inner part, beginning from one corner and finishing on the opposite one. As the nozzle comes closer to the finishing corner, the flow of plastic diminishes, resulting in the lines of the filament to touching each other.

\n\n

Maybe it's not a big deal, but it's annoying because it's stopping the part of having a very nice first layer and finish.

\n\n

My setup is:

\n\n\n\n

I tried disabling all \"retract\" settings but the issue persists.

\n\n

I'm beginning to think that this could be a software bug (Slic3r), but before I submit it to GitHub, I'd like to be sure.

\n\n

Any opinion is welcome!

\n", "Title": "Nozzle reducing flow as it comes close to finish layer", "Tags": "|print-quality|marlin|slic3r|pet|", "Answer": "

As mac's answer says, this is coasting, but contrary to that answer, coasting is not good. At one time (that answer was written way back in 2018 when the state of software was much worse than it is now), it was an idea a number of people accepted as reasonable, but it was operating at the wrong layer, and was a very bad hack, necessarily extruding less material than needed to accurately construct the part being printed. Leading to exactly what OP saw.

\n

Nowadays, printer firmware has a feature called Linear Advance (Marlin) or Pressure Advance (Klipper) that can be calibrated with test prints and compensates for the increased backpressure when extruding fast - or more importantly, seen from the other direction, the reduced backpressure when slowing down at the end of an extrusion path, that results in continued oozing after the extrusion was supposed to end. This feature eliminates the need for hacks that underextrude and produces very accurate extrusion regardless of changes in speed or flow.

\n

Anyone experiencing problems like OP hit should check that their profile does not have coasting (and hacks meant to compensate in the other direction for problems it causes, like "extra prime") enabled in the slicer.

\n" }, { "Id": "5317", "CreationDate": "2018-01-21T03:42:15.537", "Body": "

I have printed 1 voronoi piece before. I was able to get the supports that were inside the structure out with tweezers because the holes were big enough. This one (pic below) I do not think I can do that with. I looked at the proposed support structure preview in my slicer (I use Cura) and it wants to print them inside the torso here as well. Is there a way to force the slicer to not print supports within a hollow piece? I use cura but I can use another slicer if need be. Or any other recommendations for a successful print here would be fine as well.

\n\n

\"enter

\n", "Title": "Supports for dense Voronoi pieces", "Tags": "|slicing|support-structures|", "Answer": "

One feature of Cura is the ability to set supports to build plate only. This would remove any supports internal to the model, as your model has a base that is not considered the build plate or raft. In the image provided, you would have some complications, I believe, as there would be no supports on the outside of the model if it was directly over the base.

\n\n

\"cura

\n\n

As you are not eliminating other options, you can also turn off supports in Cura, load your model into Meshmixer and make use of their support feature, which allows you to adjust placement, size and other parameters in order to get a good result.

\n\n

The flexibility is limited only by your imagination:

\n\n

\"meshmixer

\n" }, { "Id": "5318", "CreationDate": "2018-01-21T06:25:47.097", "Body": "

I am trying to print a model that requires support material to be on the first layer, what is the best way to accomplish this? I am currently using Cura. It seems that the support material is too flaky. I have the infill for the support at around 8%

\n", "Title": "What is the best way to print a model with support material in first layer?", "Tags": "|adhesion|support-structures|", "Answer": "

I had the same problem printing a miniature just recently. As always, settings are somewhat dependent from the object you want to print, but here are some suggestions:

\n\n\n\n

For reference, here's a screenshot of my settings as I tweaked them for that miniature (printed a 0.1mm layer height).

\n\n

\"complete

\n\n

(\"Support line distance 1mm\" is the same as \"Support density 15%\", or at least it is the same with my nozzle size)

\n\n

If your problem was not only with the \"flakiness\" of the material, but also with it adhesion to the bed, then prepping your printing bed somehow (with a bit of painter tape, glue, etc... can help. Alternatively using a brim or a raft (as also suggested by another responder) could also help.

\n\n

EDIT: all the above still standing true, it turns out the OP had a hardware problem as well, his timing belt being loose (see comments).

\n" }, { "Id": "5338", "CreationDate": "2018-01-24T16:43:57.137", "Body": "

I have been playing around with the bed for a while and I still can't get the first layer to stick properly. I think I am getting the bed leveled good, I use the paper method. But yet I find myself with the filament lifting off and getting pulled away with the nozzle or some balling in rare situations.

\n\n

I use masking tape, which I found for $3 at Dollar general. I heard something about shine being a factor, but why is that important? And is more expensive masking tape worth it? I don't want to buy it because I would hate for it to tear when I take off a print.

\n\n

I also use some glue stick, which seems to sometimes not help, almost like it doesn't allow the filament to stick. I don't think I was adding too much, maybe I was. But is glue stick needed? Does it really make that much of a difference?

\n\n

So other than that I don't know what to try. I can't get the first layer to work properly. Maybe it is my speed, what is a good speed to print at for the first layer, I am doing 60 mm/s, just like the rest of my print.

\n\n

Any advice from there with first layers would be great. I am tired of wasting time and filament over failed first layers. But when I do get the first layer done, the second+ layers all work fine, no issues.

\n\n

Here are the specs of my printer, that should be helpful for people:

\n\n\n\n

I think that covers everything, Let me know if I need to add anything else.

\n", "Title": "Adhesion problems with masking tape", "Tags": "|adhesion|masking-tape|", "Answer": "

I'm Using Masking tape with no problems, your parameters are OK and just need to sand the surface before sending to print, use sand paper #120 or #150. The first layer height is 0.18\u00a0mm to 0.25\u00a0mm. The masking tape can be used for a lot of prints, you can change it every week to keep a good adhesion.

\n

I found that not all brands works fine, for example TUK has a good adhesion and can be used for several prints. 3M has an stronger adhesion but is needed to sand on 100\u00a0% area and can be used only for one printing. Jevelin has a good adhesion but, if the heat bed is above 40\u00a0\u00b0C this masking tape starts to peel off. and the other ones might have the same issue near to 50\u00a0\u00b0C or more.

\n" }, { "Id": "5346", "CreationDate": "2018-01-25T04:12:41.133", "Body": "

\"enter

\n\n

This is what is happening to my motor. Any suggestions would help. \n1. I have tried adjusting the trimpot. \n2. Rewire the connector to match the one on the motherboard.\n3. Anything else I found on the internet.

\n", "Title": "Anet A8 extruder motor moves back and forth", "Tags": "|extruder|stepper|anet-a8|", "Answer": "

The Anet A8 stepper motor 4 lead cable need to be correct in colour arrangement:

\n\n\n\n

I bought DIY RepRap Anet A8 3d printer which came with one of the red lead end not attached, so in order to complete the build I used another lead from the Kossel delta 3D printer which has a different colour arrangement from the Anet A8 and made the motor rock back and forth as in your video.

\n\n

It seems each colour lead carries a specific current and there are many different 4 colour cables out there.

\n" }, { "Id": "5351", "CreationDate": "2018-01-26T20:34:01.747", "Body": "

I would like to ask this in more of a general sense than anything, just for people to make note.

\n\n

I am printing out things for people and some files have some edges hanging out the side. I always worry, since it is printing in mid air, that it would screw up the print. But I was able to go, maybe 1mm(I am not to good with metric when it comes to guessing). My question is, how far at 90* from a wall can a printer pull off before it is necessary to have support? This would help me when slicing up files.

\n", "Title": "Over-hangs in prints", "Tags": "|slicing|support-structures|", "Answer": "

To add to what mac described, there are small detail effects which sometimes change the behaviour of an overhang (and not necessarily in a predictable manner).

\n\n

Concave overhangs might sag (rather than turn out like a bridge).\nConvex overhangs might pull tight.\nCooling plastic tends to shrink,so might sag less.

\n\n

I'm often surprised how well overhangs turn out, but equally often see failures which I don't anticipate.

\n" }, { "Id": "5355", "CreationDate": "2018-01-27T14:19:34.450", "Body": "

I tried to print 30mm calibration cube with my TEVO Tarantula. I found that a few bottom layers was missing as shown in picture. I noticed that a few bottom layers looks thin during printing. Anyone know how can I solve this ?

\n\n

\"enter

\n\n

\"enter

\n", "Title": "Z height incorrect on calibration cube", "Tags": "|calibration|z-axis|axis|", "Answer": "
\n

Anyone know how can I solve this ?

\n
\n\n

In order to know how to fix it, you must first diagnose what the problem is.

\n\n

Two things you should verify in Slic3r:

\n\n
    \n
  1. Check that under \"printer settings\" the Z offset is really what you want it to be (if you don't know what that is, you want it at zero).
  2. \n
  3. Check that in the preview (Plater plane, the \"preview\" tab is on the bottom), you can actually see the bottom layers being generated by the slicer (you can verify this by moving up and down the slider on the right of the main window).
  4. \n
\n\n

If both of these checks are positive, then the problem is probably not with the slicer but with the printer or printer setup. The ultimate proof of this would be to try another slicer like for example cura.

\n\n

If the problem is with the printer, the only obvious one I can think of, is that the bed is too high, the nozzle touches is, and the plastic cannot be extruded for the first few layers.

\n\n

You can confirm or refuse this hypothesis by simply observing the printer while in operation. If it is the case:

\n\n\n\n

If you realise this is the problem, stop the print immediately: this is the typical situation in which you could generate a clog in the cold end of your extruder, which is not a permanent damage, but fixing it is a somewhat complex and tedious procedure.

\n\n

The good news is that if this is the problem, the solution is trivial: you should simply set the nozzle height correctly. How to do this changes from printer to printer and you should refer to the user manual of your printer. Here is a video showing a few of the most common method to achieve that.

\n" }, { "Id": "5357", "CreationDate": "2018-01-27T21:48:56.823", "Body": "

I am using Repetier Host and Slic3r Prusa 1.36.2 - (the website says 1.2.9 is the latest version - however I don't believe this is the problem) . The printer is a Anet A3

\n\n

With my current setup I am printing mirrored - I can mirror the objects in Repetier before slicing , however then the preview picture is wrong, and worse I have to remember.

\n\n

I have a Anet A3, the connectors for the motors are polarized and if I swap the X and Y cables when I home the X and Y motors move in the wrong direction.

\n\n

I believe the problem is the coordinates and I don't fully understand the Repetier coordinates. This is how I believe it is setup (these are Slic3r screen shots ) \n\"Current

\n\n

Which I will call \"Front Left\" however my home position is Front Right - ie I believe it should look like this

\n\n

\"correct

\n\n

However these two screen shots are from the slicer - and these settings do not get used. In the Repetier they use a different system for setup. I tried these settings shown in Repetier

\n\n

\"enter

\n\n

Then everything printed in a line along the left most edge. That is the head was moved to the very right , never moved again then the platform just moved

\n\n

Can someone explain what settings I should be using for Repetier for a platform setup as I have.

\n\n

If I've misunderstood the Slic3r/Repetier interface then you can point that out as well.

\n", "Title": "How do Repetier coordinates setup work", "Tags": "|slic3r|repetier-host|", "Answer": "

I'm not sure exactly sure if you have one problem or two distinctive ones. However a few notes:

\n\n

The latest version of Slic3r PE can always be fetched on their GitHub release page. At the time of writing that would be version 1.38.5, but I concur with you that that is unlikely to be the problem.

\n\n

Mirrored objects are a clear indication that one of the Cartesian axis is swapped (not that it is translated). Any of the axis can be swapped, but since the peculiarity of your printer is that the homing point is at the front RIGHT, I would guess the axis that got messed up is the X one.

\n\n

I have no way to test the following, but if I got the meaning of your first two pictures, then I guess the correct settings in Repetier may probably be these:

\n\n

\"enter

\n\n

If even that fail, check the handedness of the coordinate system of the software generating the model you are trying to slice. Modelling software like Blender and others are typically right-handed, but many tools for processing those model for on-screen application (movies, videogames) are left-handed. That means that even if the model display correctly in the native editor, it may be rendered mirrored in another software.

\n" }, { "Id": "5366", "CreationDate": "2018-01-29T03:03:15.337", "Body": "

I just noticed, and I KNOW it was not a problem before. But recently the fan that blows over the nozzle has been acting funny. I have it set to start blowing after 3 layers, and then turn on at 100%. I decided to watch so I can give more information and what I found...

\n\n

After 3 layers, it does seem to start, very slow, barely moves. After a little bit it stops moving all together. After a short amount of time (I was elsewhere for a min) it got really warm. So I think it is putting current through the fan. I gave it a small turn and it started to speed up like it should have. And this is every time.

\n\n

Do I need to replace the fan or something else wrong with this? What kind of fan do I replace this with?

\n", "Title": "Anet A6 fan issues", "Tags": "|anet-a6|", "Answer": "

It is impossible to say with certainty without a tester, but my educated guess is that some of the induction coils in the motor are broken.

\n\n

Electrical motors that need to spin in a defined direction have multiple sets of coils (thing \"magnets\"), so that at any given moment the rotor will be pulled (more strongly) in one direction than the opposite one, thus determining the direction the motor will spin.

\n\n

From the description of your symptoms, it seems that one or more of these coils have broken, and now the fan can stall in one specific point, as it misses the \"pulling\" force that should move the motor past that point.

\n\n

If you spin the fan manually, the momentum of the fan will allow the blades to move past that static equilibrium point and reach the next position where the working set(s) of coils will begin to pull it again.

\n\n

While theoretically it is possible to open the motor, do some testing and fixing this, to all practical effects is much more effective to just replace the part (which is a regular fan for computer CPUs).

\n\n

I don't own an Anet A6, but from a quick googling it seems the correct one would be a 40mm one rated for 5v (albeit I found a link suggesting a 12v instead). My suggestion is to simply look at the specifications of the fan you have now. The specifications are normally on a sticker at the very centre of it (the side with the sticker may be facing the extruder, so you may need to unscrew the fan first).

\n" }, { "Id": "5372", "CreationDate": "2018-01-30T19:38:08.313", "Body": "

I recently changed filament and for starters it worked perfectly well, but quickly I got some problems with my first layer. It's like droplets are forming on the surface (finished item + 3 undersides):

\n\n

\"enter

\n\n

The final part is (for me) okay but I'd like to get rid of those pesky 'drops'.

\n\n

As the defect is quite stable and quite recognizable and it happens all the time, I bet somebody more experienced than me knows what's going on!

\n\n

Info:

\n\n

I'm using a 1 week old eSun filament, a E3D V6 Hotend 1.75mm on my scratch CoreXY printer, short Bowden direct drive, Repetier, Marlin, Linear advance (M900 K60 so quite low).

\n\n

The print is done using a 0.4mm diameter nozzle at 0.2mm height at:

\n\n

50\u00b0C(122\u00b0F) heat-bed and 214\u00b0C(417\u00b0C) hot-end for first layer, then lower a bit to

\n\n

40\u00b0C(104\u00b0F) heat-bed and 211\u00b0C(412\u00b0F) hot-end.

\n\n

I use Blue-painter tape.

\n\n

Worked perfectly well with my black eSun and my white eSun.

\n\n

What's my problem?

\n\n

Thank you all!

\n\n

[Edit]

\n\n

Tried about everything (more temp, less temp, fan on item, more / less bed temp, underextrude, moving around z delta, removing linear advance, radically lowering speed) but the only thing that worked about okay (problem was quite reduced but didn't go away totally) was to bump up the layer thickness to 0.3mm

\n\n

[Edit 2] Finally it seems it was, as it usually is, a conjuncture of several problems that I will list here:

\n\n\n\n

I fixed this by installing a borosilicate glass bed on top of the heat bed which ensures a perfect flatness.

\n\n

I had to change my inductive sensor to an 8mm one (inste\u00e9ad of the 4mm I had previously added) to make this function. I think it is not as good as it can be as it detects the heatbed under the glass (which is not perfectly flat), but it's enough for now. I'm thinking about how to make the glass conductive, like aluminium or something, but that's for another day/post.

\n\n

I also put blue tape on the glass because the first layer was tricky. This might be because the inductive sensor doesn't do its job that well (see above).

\n\n

Heatbed temperature for first layer: 70\u00b0C

\n\n

Extruder temperature anywhere from 190\u00b0C to 220\u00b0C works fine, that was not the problem, or it is no longer a problem.

\n\n

I also lowered the first layer speed to 20mm/s because the first layer didn't stick easily directly onto the glass, with blue painters tape it sticks even too much so I'll try higher speeds. It did mitigate problems though so it might be a path to try if you run into similar problems.

\n\n

And now it seems that it works even amazingly well, I just have to print something really big to see!

\n\n

---------------Old:

\n\n

Moving z up made spaghetti :-) and different temperatures made quite extreme warping:

\n\n

\"enter\n\"enter

\n", "Title": "What is causing 'droplets' on first layer?", "Tags": "|print-quality|filament-quality|", "Answer": "

First of all, thanks for all the help, it really is a nice community!

\n\n

But none of them actually helped and I finally find out what to actually do to fix the problem so here it goes:

\n\n\n\n

So, what did the trick?

\n\n

I figured that it was extreme warping, so heating up the heatbed to 80\u00b0C/176\u00b0F and the first layer comes down really good, for the rest of the print I use normal temperatures and it works okay. For overhangs I bump some degrees and point a small squirrel cage fan on the overhang.

\n\n

I think the filament is damaged or that this particular color might be a bit different than other colors, but well, now it works!

\n\n

[Edit] From a recent firmware upgrade the heatbed PID was disabled, re-enabling it really made it simpler too.

\n" }, { "Id": "5382", "CreationDate": "2018-02-02T19:45:53.627", "Body": "

Model of the printer is unknown, got it as present, probably something generic cartesian on arduino mega and ramps boards stitched together and with marlin firmware.

\n\n

I've used accepted answer from here to try moving this thing from terminal.\nHow to directly send G-code to printer from a Linux terminal?

\n\n

My first attempt to get access to low-level printer interface looked like that:

\n\n
./baud.py <> /dev/ttyACM0 250000\ntail -f /dev/ttyACM0 &\ncat > /dev/ttyACM0\n
\n\n

First it was fine: i've entered g-code, printer executed it and returned an ok message into my terminal.

\n\n

Then i've turned the printer off and on again and repeated the whole process, but now tail -f didn't output anything and printer LCD displayed garbage in the status line after I ran the command.

\n\n

I've also noticed that printer controller reboots every time the serial port is accessed, not sure if it happened in the first time when everything worked well.

\n\n

The output of cat /dev/ttyACM0 after baud setting is a bit weird too - and there's garbage in the status line instead of standard \"%printername% ready\" as well:

\n\n
start\necho:Marlin1.0.0\necho: Last Updated: May 20 2017 18:12:04 | Author: (none, default config)\nCompiled: May 20 2017\necho: Free Memory: 3763  PlannerBufferBytes: 1232\necho:Hardcoded Default Settings Loaded\necho:Steps per unit:\necho:  M92 X80.00 Y80.00 Z3200.00 E97.94\necho:Maximum feedrates (mm/s):\necho:  M203 X50.00 Y50.00 Z2.50 E25.00\necho:Maximum Acceleration (mm/s2):\necho:  M201 X750 Y750 Z100 E10000\necho:Acceleration: S=acceleration, T=retract acceleration\necho:  M204 S500.00 T500.00\necho:Advanced variables: S=Min feedrate (mm/s), T=Min travel feedrate (mm/s), B=minimum segment time (ms), X=maximum XY jerk (mm/s),  Z=maximum Z jerk (mm/s),  E=maximum E jerk (mm/s)\necho:  M205 S0.00 T0.00 B20000 X20.00 Z1.00 E5.00\necho:Home offset (mm):\necho:  M206 X0.00 Y0.00 Z0.00\necho:PID settings:\necho:   M301 P22.20 I1.08 D114.00\necho:SD init fail\necho:Unknown command: \"starto\"\nok\necho:Unknown command: \"SD init failstartuthor\"\nok\necho:Unknown command: \" (none, default config)50.00 Z2.50 E2rBy00.00 Y0.00 Z0.00echo\"\nok\necho:Unknown command: \"Unknown command\"\nok\necho:Unknown command: \" \"starto\"own comm\"\nok\necho:Unknown command: \"aximum XY jerk (mm/s),  Z=maximum Z jerk (mm/s),  E=maximum E jerk (mm/s)echo\"\nok\necho:Unknown command: \"PID settings\"\nok\necho:Unknown command: \"okechecho\"\nok\n
\n\n

The \"SD init fail\" line and everything after it appears when sensor data appears on the LCD, there's a delay before that during which the LCD is empty.

\n\n

If you send commands to printer using something like echo \"G0 X10\" > /dev/ttyACM0, it executes them only on next serial port accessing (and therefore reboot) - or doesn't execute at all.

\n\n

The interesting part is that Cura \"Monitor\" tab can actually manipulate the caret and the Cura itself can print things in general - but i want to be able to do it manually.

\n", "Title": "Issues with direct g-code transmission via serial port", "Tags": "|marlin|g-code|", "Answer": "

That's weird but i've got some code from here: https://stackoverflow.com/questions/6947413/how-to-open-read-and-write-from-serial-port-in-c , changed it a little, removed few lines, ran it - and my port started working just fine. Here's the complete code (for Linux of course).

\n\n

I guess that was the c_lflag line that solved the problem.

\n\n
#include <asm/termios.h>\n#include <stropts.h>\n#include <unistd.h>\n#include <fcntl.h>\n#include <string.h>\n\nint set_interface_attribs (int fd) {\n    struct termios2 tty;\n    memset (&tty, 0, sizeof tty);\n    ioctl(fd, TCGETS2, &tty);\n    tty.c_cflag&=~CBAUD;\n    tty.c_cflag|=BOTHER;\n    tty.c_ispeed=tty.c_ospeed=250000;\n    tty.c_cflag = (tty.c_cflag & ~CSIZE) | CS8;\n    tty.c_iflag &= ~IGNBRK;\n    tty.c_lflag = 0;\n    tty.c_oflag = 0;\n    tty.c_cc[VMIN]  = 0;\n    tty.c_cc[VTIME] = 5;\n    tty.c_iflag &= ~(IXON | IXOFF | IXANY);\n    tty.c_cflag |= (CLOCAL | CREAD);\n    tty.c_cflag &= ~(PARENB | PARODD);\n    tty.c_cflag &= ~CSTOPB;\n    tty.c_cflag &= ~CRTSCTS;\n    ioctl(fd, TCSETS2, &tty);\n    return 0;\n}\n\nint main() {\n    int fd=open(\"/dev/ttyACM0\",O_RDWR|O_NOCTTY|O_SYNC);\n    set_interface_attribs(fd);\n    close(fd);\n    return 0;\n}\n
\n" }, { "Id": "5387", "CreationDate": "2018-02-03T16:28:11.317", "Body": "

Firstly, I just bought a G2S mini pro from Geeetech and it is my first 3D printer (therefore I'm 100% newbie in this area).

\n\n

The first extruder seems to be ok, the temperature varies from 23\u00b0C to 201\u00b0C and if you touch it you can see that it is hot. However, for the second extruder, the temperature is shown with more than 400\u00b0C:

\n\n

\"Second

\n\n

The cables are connected correctly and the firmware is updated.

\n\n

Do I have to return the printer? Or is there another solution?

\n", "Title": "Wrong temperature on extruder 2", "Tags": "|extruder|", "Answer": "

If you have access to an IR thermometer, it would be interesting to verify what the actual temperature is. I doubt the hot end actually reaches that temperature, but:

\n\n\n\n

Either way: thermistors and cables are cheap to replace, while problems with the firmware may be fixed only if you have access to the code.

\n\n

If you just bought the printer in a physical store, I would simply swap it with another unit, rather than fiddling with it, though.

\n" }, { "Id": "5389", "CreationDate": "2018-02-03T19:58:11.527", "Body": "

I am printing some minion chess pieces for my teacher at school and on every model I have found something called \"ghosting\", or at least I heard that is what it is called. For example there is a strap on the model for the pants. And going left and right there is very shallow \"straps\" or something happening.

\n\n

I am interested to know what causes this to happen and how to fix it. It is not super bad, but would be nice to fix.

\n", "Title": "What causes \"ghosting\"?", "Tags": "|print-quality|ghosting|knowledgebase|", "Answer": "

To expand on the #3 solution: reducing the elasticity of the system, if you have your filament mounted on the top of your printer, placing the filament elsewhere on a separate spool holder can also reduce ghosting. Tightening the belts and moving my filament got rid of ghosting for me on my MP Maker Select.

\n" }, { "Id": "5390", "CreationDate": "2018-02-03T20:04:02.017", "Body": "

I have a geeetech g2s pro and when I request to print something, the extruder motor doesn't move (it is connected on the board), therefore, the printer prints some kind of imaginary object. I cannot ask Repetir host to extrude it as prevents moving when the extruder is cold, and when it is hot all the repetier host does is moving the header up. Is there a way a could test these motors without needing a hot extruder?

\n\n

Here you have a picture of it, maybe it is upside down.\"motor\"

\n", "Title": "Extruder driver not working", "Tags": "|motor|extruder-driver|", "Answer": "

Found the solution, I had to swap the cables on the board, now they are working.

\n" }, { "Id": "5395", "CreationDate": "2018-02-04T03:42:27.450", "Body": "

I'm new to 3D printing and I recently got a LulzBot Taz 6 printer.

\n\n

I'm interested in doing dual extrusion printing (especially with dissolvable supports), but I'm also interested in doing single extrusion prints. In particular, I would like to experiment with printing flexible filament.

\n\n

The LulzBot TAZ Dual Extruder v3 Tool Head says that it can handle all the filament types that I am interested in.

\n\n

Here is the question: can I use a dual extruder (in particular, the LulzBot TAZ Dual Extruder v3 Tool Head) for general purpose printing (read single extrusion) or do I need to use a single extrusion head for single extrusion print jobs.

\n\n

If you can use a dual for single use, why buy single extruders (I've been looking at LulzBot TAZ Aerostruder Tool Head for flexy prints) if you can get the same performance out of a dual head?

\n\n

If you should use a single extruder for non-dual prints, please explain the shortcomings of using a dual print head for general purpose use.

\n", "Title": "Is a dual extruder a reasonable choice for all-purpose printing", "Tags": "|dual-nozzle|lulzbot|", "Answer": "

This may be too late to help, but I wanted to describe my own experience with the Lulzbot Taz Dual Extruder v3. It was...not good.

\n\n

In terms of your question, I spent a fair amount of time trying to do exactly this. It is possible, but there are some major caveats:

\n\n\n\n

As such, I personally gave up and swapped my old single extruder back in.

\n\n

This isn't meant to contradict the other answer. In theory it should be workable. But in practice, with the device you mention, it's not.

\n" }, { "Id": "5402", "CreationDate": "2018-02-05T12:13:25.300", "Body": "

I saw an extruder mod on Amazon \"EAONE 2 Pcs PTFE Teflon Tube (2 Meters) with 4 Pcs PC4-M6 Fittings for 3D Printer 1.75mm Filament (2.0mm ID/4.0mm OD)\"\nAnybody know how this is fitted? Is it simply tapping the feed hole on the top?

\n", "Title": "Anet A8 Hot End Spares Quey", "Tags": "|extruder|anet-a8|ptfe-tube|", "Answer": "

that item is for a bowden setup. (the a8 does not come with a bowden setup, but can be modded to have one.)\nwhen installing the bowden setup on a a8, the first thing you will need to do is move the motor to somewhere else, there are tons of choices on thingiverse.\nthen the PC4-M6 fitting will screw into the black aluminum piece that the throat used to screw into. \nand the other end will go into a customised hot end. again tons of choices

\n\n

now this is oversimplified, because installing a bowden extruder on a anet a8 is a lot more complicated than explained above due to the sheer number of options.\njust search anet a8 bowden and you will have lots of choices.

\n\n

but it works as mac said, the PTFE tube slides into self locking fittings, the PC4-M6.\nto release simply push the top of the fitting down and pull out the tube.

\n\n

source, my own anet a8 modded to have a bowden setup.

\n" }, { "Id": "5405", "CreationDate": "2018-02-05T17:38:06.503", "Body": "

I'm using a Prusa clone with Ramps 1.4, Marlin 1.1.5 and an inductive proximity sensor Z-probe. Due to the design of the printer, the X and Y end-stops and the Z-probe are not exactly aligned with the heated bed, so I must use Marlin's Home Offset feature to align the coordinate space to the print area, using the G-code command M206 X-18.5 Y-2.5 Z1.1.

\n\n

I am now attempting to use Marlin's Unified Bed Leveling feature to compensate for a slight but significant curvature of the bed.

\n\n

Unfortunately, it appears that the Mesh Validation Pattern command (G26) is not taking the Home Offset into account when printing a test pattern. The printed pattern hangs off the left side of the bed and the center printed circle is clearly misaligned with the center of the print bed by the bed offset amount.

\n\n

Does this suggest that my firmware is misconfigured? Or is there an alternative method for aligning the mesh validation test pattern with an offset print bed?

\n", "Title": "Marlin's bed leveling Mesh Validation Pattern ignoring home offset", "Tags": "|prusa-i3|marlin|calibration|z-probe|bed-leveling|", "Answer": "

A Marlin developer confirmed that bed-leveling and probing are now performed entirely in the machine coordinate space.

\n\n

To clarify further, if one has a non-zero home offset enabled, the following commands refer to different physical locations despite the supplied parameters being the same:

\n\n
M206  X10  Y10 ;  Set a home offset in X and Y\nG30  X100 Y100 ;  Probe (100, 100) in machine space\nG0   X100 Y100 ;  Move to (100, 100) in print space, ie. (90, 90) in machine coordinate space\n
\n\n

I was able to correctly align the bed-leveling grid by altering header files in the source code. I defined the XY offset using two new macros BED_OFFSET_X and BED_OFFSET_Y in Configuraion.h and altered the following lines in Conditionals_post.h.

\n\n
#define X_CENTER ((X_BED_SIZE) / 2) + BED_OFFSET_X\n#define Y_CENTER ((Y_BED_SIZE) / 2) + BED_OFFSET_Y\n
\n\n

The mesh is now physically aligned with the bed and I have not observed any adverse effects.

\n" }, { "Id": "5409", "CreationDate": "2018-02-06T04:36:59.403", "Body": "

I have an STL that has a flat surface which should contact the build place when 3D printing. The model has an unknown rotation. How can I rotate the 3D model to be flush with the build plate?

\n\n

I am very comfortable with OpenSCAD, and I can use it to rotate the STL if I know the correct angle. Is there a tool that will let me analyze a single face (triangle) of a model to determine it's normal? If I can even get the coordinates from 3 points from the desired face I can calculate the normal and use that to calculate the rotation vector.

\n", "Title": "How can I rotate a 3D model to be parallel with the build plate?", "Tags": "|3d-models|software|file-formats|analysis|stl|", "Answer": "

Some tools like Cura or Repetier Host have slicers that analyses and tell you if it is ok to print or not. Both of them allow rotations.

\n" }, { "Id": "5413", "CreationDate": "2018-02-06T12:59:13.713", "Body": "

I had been printing with ABS and took the advice to alter the fan so I can see the filament when I am loading it into the cold end. It was tricky but doable. I am now trying with PLA and getting it to line up with the whole is a nightmare. Can the driving cog and guide wheel be moved? A couple of mm would stop the driving cog pushing the filament off line.

\n", "Title": "Anet A8 hard to insert filament", "Tags": "|pla|extruder|anet-a8|", "Answer": "

A lot of people complain on the filament insertion of the Anet A8. Personally, I have no problems at all. I cut the filament under a sharp angle and pre-bend the filament (not completely straight) and push it in the hole, it works every time without having to disassemble the extruder fan. Note that if you have the throat screwed in too much that it sticks out of the aluminum throat holder of the extruder (so not the part where the brass nozzle screws in) it is way more difficult to insert. I kept the top of the throat more or less flush with the block.

\n

Other techniques to insert filament include:

\n\n" }, { "Id": "5414", "CreationDate": "2018-02-06T13:48:19.880", "Body": "

I have a homemade 3D printer running on ramps 1.4\nWhen i start a new print and the hotend reaches melting point of PLA, the PLA start coming out of the hotend.
\nThis goes on for as long as the temperature is kept above melting point, without moving the extrusion gear.

\n\n

The extruder is a bowden type.
\nHotend is a J-head.
\nI am currently using simplify if that makes any difference regarding configuration.

\n\n

Any ideas what to do to prevent this from happening?

\n", "Title": "Filament starts extruding as soon as hotend reaches melting point", "Tags": "|pla|ramps-1.4|extrusion|simplify3d|", "Answer": "
\n

Any ideas what to do to prevent this from happening?

\n
\n\n

You cannot prevent it entirely, but you can probably mitigate the problem by depressing the lever that squashes the filament against the hobbed gear of the extruder before starting to heat the nozzle.

\n\n

In bowden extruders, the long portion of filament between the stepper motor and the nozzle is subject to compression during the print. Because of the hysteresis in the filament, and of the slack between filament and PTFE tube, this filament acts like a slow-releasing compression spring. When the nozzle cools down, the potential energy stored in the filament is \"frozen\" in place.

\n\n

By depressing the lever, you allow the spring to extend \"backward\" towards the spool, rather than \"forward\" through the nozzle.

\n\n

Some oozing is still bound to happen because of gravity and - as highlighted by others - thermal expansion, but it should be significantly less.

\n\n

If you adopt the lever trick, remember to print with a skirt, as you will want the printer to recreate that \"compression\" in the filament before the model proper begins.

\n\n

Another way to address the issue would be to add a little bit of retraction in the closing stanza of your GCODE (the part where you also tell the printer to unpower the steppers and stop heating). This will prevent any \"compression\" to be \"frozen\" in the first place.

\n\n

This anwer is based on the assumption that the stepper motor is not actively spinning (i.e.: yours is not a hardware/firmware issue).

\n" }, { "Id": "5430", "CreationDate": "2018-02-07T23:27:02.397", "Body": "

I read that the best way of removing ABS was to let the temperature at the hot end to drop to around 190deg c then a sharp pull. This worked really well. I am trying to print with PLA but no matter what temperature I drop the hot end to I get left with a length of PLA in the feeder tube. OK I can heat the hot end and poke the excess down with a wire but that is a pain. I think the technique is right but the temperature is wrong. Any help great fully appreciated.

\n", "Title": "Removing pla from extruder", "Tags": "|pla|extruder|anet-a8|", "Answer": "

Looks like I had a gap between the hot end and the screwed rod. A gap filled with cool pla. A sort of washer for want of a better description. Will let you know if heating above temp is the answer

\n" }, { "Id": "5432", "CreationDate": "2018-02-08T03:09:25.973", "Body": "

Some filaments suggest \"reducing cross-sectional area\" of the print. Is this referring to the vertical plane or horizontal plane? In other words, if I were to print a rectangular prism, would I want the long side of it printed in the vertical direction or parallel to the print bed?

\n", "Title": "reducing cross-sectional area", "Tags": "|print-quality|", "Answer": "

My guess is, based on the fact that this is ABS and the same paragraph recommends an heated bed, that they mean you should avoid layers with large continuous areas because those can be problematic with ABS if your heat is not very well controlled.

\n\n

Basically while you are printing the upper part the lower part is cooling, and ABS shrinks as it cools, short strands shrink by a little and apply little force to the print while long strands shrink by a lot (same percentage, but more length - because they are longer to begin with) apply a lot of force and pull the edges of the print of the bed

\n\n

so they suggest the shorter sides be on the X and Y and the longer side be on the Z axis.

\n\n

Note that this will make materials with high shrinkage and low bed adhesion like ABS easier to print - this will not make the resulting part stronger or in any way better

\n" }, { "Id": "5434", "CreationDate": "2018-02-08T06:55:21.307", "Body": "

I made and assembled my own 3D printer two years ago and I notice that some extruders are MK6 and MK(*whatever). Some of them has the block fuser horizontal and others vertical, then uses a nozzle longer.

\n\n

The block heater that I made was an aluminum block 20x20x10 mm using the M6 short nozzle. Then adapted the radiator to upgrade to Jhead heater.

\n\n

So I wanted to know to which group belongs my printer, as base is a prusa clone with direct extrusion.

\n", "Title": "Which is the difference between MK6 and MK8 and even MK10?", "Tags": "|extruder|hotend|nozzle|", "Answer": "

All credits for the following go to user \"vermon\" who posted in this thread.

\n\n

The following is a heavily amended version from his longer answer there.

\n\n
\n\n

Makerbot did start its hotend series using MK as iteration designator (for Mark). The first commercially available version was the [Makerbot] MK4 on the Cupcake circa 2009. MK4 was hand built with nichrome wire heaters and prone to all kinds of failures. They only worked with ABS and 3 mm filament.

\n\n

Following here, Makerbot was skipped as a manufacturer and that it is the hotend we talk about. The correct nomenclature would be Makerbot Hotend MK#

\n\n

MK5 was a complete redesign and had huge nozzles and a thick PTFE liner. It used 2 large power resistors in parallel for heating and was also prone to failure.

\n\n

MK6 was the first hotend sold as a kit that I know of (however Repraps had also started using them probably before).

\n\n

Mk6+ was the first cartridge heater hotend sold in kit form. The MK5/MK6 heater block was stainless steel, where MK6+ upgrade kit was an aluminum block that was slightly smaller, a lot lighter, and had the now standard 1/4 inch heater cartridge.

\n\n

MK7 was the first hotend you folks would think looks familiar. It was the first move to 1.75 mm filament dedicated extruders. While a MK6+kit had parts to adapt to 1.75 mm filament, it never worked well at all and was really a failure.

\n\n

Again, this is all Thing-O-Matic and Cupcake era. Other than a few clones, there really was no third party market in the US at this time. You either had a MakerBot or some other Reprap based kit. Wanhao, FlashForge- they didn't even exist and if they did, weren't talked about like now.

\n\n

MK8 was the all new dual extruder hotend setup on the MakerBot Replicator. The cooling bar was thicker than the MK7 but the same all metal thermal barrier and supposedly, the MK8 has slightly different nozzle geometry internally and externally.

\n\n

MK9 was a MK8 hotend, cooling bar, thermal barrier, and nozzle, but the new feeder with spring lever we know on all current models. This is why it's confusing, MK9 was a feeder upgrade, not a hotend change.

\n\n

MK10 was a complete change of the hotend. MK10 uses smooth OD thermal barriers with a larger 4 mm OD 2 mm ID PTFE liner. MK10 also uses M7 threads, vs the M6 of all previous models. This is because a 4 mm PTFE liner is barely enough metal to make the outer tube with M6 threads. MK10 is completely incompatible with all previous hotend parts. Every part is different. MK10 still uses MK9 feeder parts.

\n\n

MK11 on the D6 is just an MK10 nozzle and thermal barrier, but a different heater block and the cooling bar is part of the D6 central cross. Again, the only real difference is MK11 is a different heater block, and that's to adapt a different and longer heater cartridge and slide in the thermocouple. MK11 still uses MK9 plastic feeder parts.

\n\n
\n\n
\n

I wanted to know to which group belongs my printer, as base is a Prusa clone with direct extrusion.

\n
\n\n

This question seem difficult to answer concretely, as the number in the series seem to be assigned based on a combination of heating block technology and format, the feeder geometry and the nozzle shape and thread that was adopted on the MakerBot printers. Your actual combination of those three may not exist in the MakerBot universe.

\n\n

I suggest you read the full, unabridged version of the post linked above though, as more details relevant to your inquiry may be disclosed there.

\n" }, { "Id": "5445", "CreationDate": "2018-02-09T07:18:03.403", "Body": "

I'd like to mirror a triangular prism I made. However, whenever I select it I am only able to select the faces or sides.

\n\n

Is there a way to select the whole 3D object to move or mirror it?

\n", "Title": "In Fusion 360 how do I select an entire 3D object, not just its sides or faces?", "Tags": "|software|cad|fusion360|", "Answer": "

In the top left of your workspace (still inside the editor) you will have a dropdown, in that dropdown there is a folder called bodies, open that and select the body you want. You can then move the selected body.

\n" }, { "Id": "5447", "CreationDate": "2018-02-09T10:44:18.653", "Body": "

When using G1 command in G-code, what is the difference between Z- axis and E- axis?

\n\n

I see all E, F and Z in

\n\n
G1 Z0.350 F7800.000\nG1 E-2.00000 F2400.00000\nG92 E0\nG1 X96.753 Y95.367 F7800.000\nG1 E2.00000 F2400.00000\n
\n", "Title": "E axis vs Z axis", "Tags": "|g-code|", "Answer": "

G-CODE can be confusing as historically it was developed for machining tools rather than FDM printers, and thus:

\n\n\n\n

Typically, Cartesian printers use 4 \"axis\": X, Y and Z for moving the printhead in space and E for \"extruder\". The \"extruder axis\" is not in fact at all an axis in the geometrical sense of the word: it refers to the amount of filament to be moved into (extruded) or out of (retracted) the printing head. The reason why it is considered an \"axis\" is that it is used in conjunction with the codes G0 and G1 which are for movement.

\n\n

Confusingly, the letter E is used for the precision feedrate of lathes but in the 3D printing world we rather use F to that purpose. As a non-native English speaker, I was further confused, because for the longest time I thought \"feedrate\" was referring to the amount of filament fed to the printing head, while in fact it is the speed at which the printing head moves (in mm/minute).

\n\n

So to summarise, your example code \"translated\" would look like this:

\n\n
G1 Z0.350 F7800.000           ; move up 350 microns at 7.8 m/min\nG1 E-2.00000 F2400.00000      ; retract 2mm of filament at 2.4 m/min\nG92 E0                        ; reset the extruder position\nG1 X96.753 Y95.367 F7800.000  ; move to X,Y (without changing Z) at 7.8 m/min\n
\n\n

Should you wish to dig deeper into the topic, the RepRap wiki page on gcode is rather comprehensive. Mind that several manufacturers add their own \"special codes\" to the common ones....

\n" }, { "Id": "5451", "CreationDate": "2018-02-10T00:17:25.573", "Body": "

I realize this issue (warping) has been repeatedly addressed on this site. I've just graduated to high-temp filaments (PC in particular). I don't know much of the physics of this. I'm wondering whether the degree to which the filament contracts is proportional to the amount that it cools. If the answer is yes, then wouldn't it suggest that a lower printing temperature might reduce warping-as the temperature interval over which the filament cools is smaller? Or perhaps the difference is negligible?

\n\n

Also, I see a lot of emphasis placed on good first layer adhesion. Is this still an issue if you are printing on a raft?

\n", "Title": "Will lowering print temperature help warping?", "Tags": "|warping|", "Answer": "

The answer already provided by @fred_dot_u at the time of writing is good, so I won't rehash what they already said, but will try to answer your questions from another angle:

\n\n
\n

I'm wondering whether the degree to which the filament contracts is proportional to the amount that it cools.

\n
\n\n

Yes. This is generally true for any solid material. This property is linked to the fundamental nature of matter in the way we understand it today.

\n\n
\n

If the answer is yes, then wouldn't it suggest that a lower printing temperature might reduce warping-as the temperature interval over which the filament cools is smaller?

\n
\n\n

Shrinkage is the root cause of 3D prints warping, however warping itself happens because of the differential in temperature between layers: when a hot layer is extruded on top of a cold one and begins to contract, it will apply a compressing force on the layer underneath, bending it.

\n\n

In fact - if warping were a function of shrinkage - an enclosure would do no good: sooner or later the print would cool down to room temperature, and would warp. The reason why - contrarily - an enclosure works, is that it limits the differential in temperature between layers (which causes warping) and lets the entire print too cool uniformly and slowly afterwards.

\n\n

So, would...

\n\n
\n

...a lower printing temperature reduce warping? [...] Or perhaps the difference is negligible?

\n
\n\n

Nothing beats real world data, especially when the issue has so many variables that are difficult to account for, as in 3D printing, so I would invite you to simply try to print the same model twice, only changing the printing temperature (and of course making sure the environmental temperature is the same), and see if it does.

\n\n

From a theoretical standpoint, I could argue both ways.

\n\n

On one end, I could argue that this is nothing different than using an enclosure set at a slightly warmer temperature than the environmental one, so it will reduce warping (even if by not much).

\n\n

On the other hand, I could argue that until the filament is solid enough, it won't be able to \"pull\" the layer below, so it doesn't matter if the extrusion temperature (fluid state) is 230\u00b0C or 210\u00b0C, if until 190\u00b0C the filament won't begin to \"pull\". So warping will be identical.

\n\n
\n

Also, I see a lot of emphasis placed on good first layer adhesion. Is this still an issue if you are printing on a raft?

\n
\n\n

Yes, as also the raft has a first layer that needs to adhere to the build plate. Rafts typically have a discontinuous and over-extruded first layer over a large area, which definitively helps with adhesion, but you still have to make sure the raft sticks well. In my experience it is far easier for a raft to come off the build-plate than for the model to come off the raft. YMMV though, as the material of the build-plate, and the slicer can dramatically affect this.

\n" }, { "Id": "5454", "CreationDate": "2018-02-10T07:55:45.890", "Body": "

I was wondering if is possible to send printing using the software MACH3 for CNC since this software can handle up to 3 axes: X,Y,Z,A,B and C.\nOf course the CNC needs an extra control for temperature which I already have it; My CNC handle a Z axis with 150mm height, so I could print higher parts than normal CNC.

\n", "Title": "is possible to send printing with mach3?", "Tags": "|printer-building|cnc|", "Answer": "

Well, few years ago I could set the software Mach3 for printing doing the next:

\n\n
    \n
  1. Set a profile for 3D printing for the faster movements that your CNC can support; for example some CNC uses standard threads, others ACME threads and other GT2 belts like the 3D printers. If we try to use a feed rate too high the motors will shake. For this profile is not needed the torque for machining or routing, so we will sacrifice force to gain speed.
  2. \n
  3. Create the proper gcode. Mostly Gcode maker generate the code to send pulses to Extruder E but Mach3 takes as invalid E instructions, so the extruder will never move. To avoid this there are 2 modes:
  4. \n
\n\n

a) Find and replace all E instruction with A to enable Motor A or

\n\n

b) Create the Gcode with an interpreter to output A instead E\n like Slicer 0.71 as you can see in the image below. I haven't tried with other software

\n\n

\"enter

\n\n

3.For controlling the extruder temperature is possible with any other controller like REX-C100/C400 any PIC or Arduino. The accuracy will depend on your design.

\n" }, { "Id": "5461", "CreationDate": "2018-02-10T17:14:35.423", "Body": "

That's my first 3d printer. I'm using Repetier Host as the brand recomends, and set all the configuration as the recommended one. I decided to print one STL file but the result is not the best one.\nThat's what I was trying to print:\n\"handle\"\nand that's what I've got.\n\"cr*p\nHere you have a video of the impression. it is a G2S pro rostock mini

\n", "Title": "Filament is not stuck", "Tags": "|troubleshooting|", "Answer": "

I would suggest buying Buildtak, which is a high-quality adhesion surface. This surface is almost guaranteed to make your prints stick to your bed. If Buildtak is outside of your budget, I would suggest using a relatively high grade painters tape combined with either purple gluestick or hairspray applied to your bed before each print.

\n" }, { "Id": "5465", "CreationDate": "2018-02-11T19:38:49.110", "Body": "

I was wondering if adding (an) extra fan(s) (not connected to the printer, but blowing on the print area) could improve the quality of PLA based prints(printing at 210 C). The printer already has a built in fan with a fan shroud that directs air to the hotend, but is it beneficial to add an extra fan in order to get better results on overhangs, fine details, etc, or does extra cooling negatively/not affect print quality?

\n", "Title": "Is it advisable to use additional external fan(s) for printing PLA?", "Tags": "|pla|quality|cooling|print-fan|", "Answer": "

Fan blow at hot end is necessary because hotend needs cooling.

\n\n

For PLA it will yield better result with a seperate controllable fan direct airflow across the print head, but just like everything with 3D printing, you will need to test out every possible configuration to get the best for your setup.

\n" }, { "Id": "5468", "CreationDate": "2018-02-12T02:44:38.083", "Body": "

I am getting started with GCode, and I don't know all the commands. Is there a way to smooth a path between two points? I wrote a program which makes GCode from a silhouette. Problem is, when it looks at the individual pixels, its path becomes a bit choppy. Is there a command that will make a more natural path. Attached is a close up picture of the path simulation.

\n\n

\"enter

\n", "Title": "Make smooth curves in GCode", "Tags": "|g-code|cad|", "Answer": "

Curves are normally approximated with straight-line segments. However, some firmwares (e.g. Marlin) support arc commands. See:

\n\n

Marlin: Controlled Arc Move
\nRepRap: Controlled Arc Move

\n\n

Note that the RepRap Wiki is not always accurate in its list of supported commands.

\n" }, { "Id": "5474", "CreationDate": "2018-02-12T09:25:07.363", "Body": "

I had a problem with my nozzle on my homemade printer. The problem was that the nozzle with a 0.4\u00a0mm diameter wasn't putting out any plastic. I replaced the nozzle with a 0.2\u00a0mm one and now the bottom of my model is looking like this: \"waves\"\nI tried to change flow,temperatures and speed.\nBut nothing helped it keeps making this waves. At the old 0.4\u00a0mm nozzle there everything was okay.

\n", "Title": "The bottom of the printed objects has waves after replacing the nozzle", "Tags": "|print-quality|nozzle|", "Answer": "

The layer thickness should not be more that 75% of your nozzle diameter: Wavy lines on 1st layer only

\n" }, { "Id": "5479", "CreationDate": "2018-02-12T16:36:20.597", "Body": "

I printed thingiverse minion Dave and after 2 failed attempts (1st tore of from bed, 2nd had jitter at about 50%) I slowed the print speed down. The final result was amazing. However the reported print time, 3hrs, was in reality 7hrs. The speed reduction was small and I would not have expected such a dramatic change in time. Did I miss a step somewhere?

\n", "Title": "Cura 3 speed selection", "Tags": "|ultimaker-cura|anet-a8|", "Answer": "

No you didn't do anything wrong.

\n\n

The problem with time estimates in slicers is that they don't know anything about the printer's firmware and physical limitations, so they estimate time based on the (whoefully wrong) assumption that the printer will do only and exactly what told to.

\n\n

So, if the gcode says \"move 100mm at 1m/s\" the slicer will assume your printer will take 100ms to perform that action. In reality though, your printer will take some time to accelerate, and it may actually be unable to reach the target speed of 1m/s altoghether, taking a lot longer to perform the full action.

\n\n

Typically, Cura estimates are OK for makerbots, and Slic3r PE for Prusa printers, as the software is mainly developed towards those machines, but for most other printers the estimates will be sensibly off.

\n\n

You may also be interested in trying to use octoprint to feed the GCODE to the printer: octoprint monitors the actual elapsed time against the gcode and try to adjust the estimate of the remaining time accordingly, also telling the operator how reliable that estimate is.

\n" }, { "Id": "5481", "CreationDate": "2018-02-12T20:21:28.503", "Body": "

Quick thing: Please tell me if I misuse any of the terminology

\n\n

On a replicator+, I have been printing successfully for a while, when suddenly the raft started to warp. I was doing a bunch of models that covered the whole tray, so I shrunk to just a small area, but It still warped I read up on how to fix, but most covered how to fix warping in the model itself, not the rafting. Some said to lower the temp, would that work? smart extruder at default settings, 215 C. The printer does not have a heated base, nor have I treated it with anything, and I am using it with the stock program (makerbot print). Otherwise, I am using it as it came out of the box.

\n\n

\"bad\"

\n", "Title": "Raft warping (Makerbot Replicator+)", "Tags": "|makerbot|warping|rafts|", "Answer": "

I'm not sure I am reading your post correctly, but if you are doing a batch of small prints, I would recommend to space them enough so as each of them has its own mini-raft, rather than all of them sharing the same large one.

\n\n

If you are using cura, you can tweak how much the raft goes past the footprint of the part. Unless you are printing very small parts, you don't need that to be a lot.

\n\n

In general, you should think to a raft as a print in and by itself: the larger it is, the more prone to warping, although the way filament is layered with gaps makes the raft bend and warp a lot less than a regular print of the same size.

\n" }, { "Id": "5483", "CreationDate": "2018-02-13T11:28:51.037", "Body": "

I'm trying to get a clean set of measurements of the Anet A8 mainboard and determine the exact positioning of the holes in both the board and frame.

\n\n

The frame and the board are cut to accommodate an M3 bolt.

\n\n

Here are the measurements I have currently:

\n\n

Mainboard

\n\n\n\n

Frame (main board holes)

\n\n\n\n

I was hoping to just find a PCB layout for this board with the measurements and offsets but have found nothing.

\n\n

I am attempting to provide a mount adapter frame for a PSU with a 50 mm x 150 mm mounting footprint.

\n", "Title": "Anet A8 mainboard dimensions and hole offsets", "Tags": "|anet-a8|", "Answer": "

Why don't you download one of the many casings that are found on Thingiverse that house the Anet electronics board and take measurements from those cases.

\n\n

I measured it from my own casing I designed a while ago:\nHole distances are 86.3 mm and 91.3 mm (center to center)\n\"Anet

\n" }, { "Id": "5487", "CreationDate": "2018-02-13T14:34:39.763", "Body": "

I have printed off some upgrades for my Anet A6 for tensioning the belt on the Y and X axis. I was wondering how much deflection the belt should have. I would think being to tight will stress the machine.

\n", "Title": "Belt Tensioning", "Tags": "|anet-a6|belt|", "Answer": "

I wanted to know the proper deflection I should have on the belt. What I have read is for every inch of belt (25.4mm) it should deflect 1/64 of a inch (.4mm).

\n\n

I found this information on this Website.

\n" }, { "Id": "5490", "CreationDate": "2018-02-13T23:09:20.920", "Body": "

I recently rebuilt my A8 printer to have a metal frame and replaced the board with a GT2560 reva+. I'm having problems with the temp readings from both my hotend and hotbed. Using marlin 1.1x and Its possible I have something configured incorrectly. I can get successful prints with good quality but sometimes I get thermal runaway during a print.

\n\n

Both cold and at temp (200c) have fairly large temp spikes. I didn't have this problem with my old board. I had much smoother temp curves. I'm using the default PID values for Ultimaker Kp 22.2, Ki 1.08, Kd 144.

\n\n

I have tried to use PID autotune but get drastically different values every time I run it and using the results gave worse results (sometimes giving a heating error during the autotune). I checked all the wiring and nothing looks frayed or damaged.

\n\n

First pic is cold temps. Second is trying to do a PID tune. Third and fourth are during a print.

\n\n

\"Temps

\n\n

\"PID

\n\n

\"During

\n\n

\"During

\n", "Title": "Spiky hotend and bed temps", "Tags": "|hotend|", "Answer": "

This really looks like a bad connection, so try with your old board to see if the broken connection is on the board, or it's the temp sensor or the wires.

\n\n

I had exactly this problem with a bad connection from the temp sensor.

\n\n

If you didn't break anything, it's the board that's fried / badly connected.

\n" }, { "Id": "5495", "CreationDate": "2018-02-14T18:34:08.173", "Body": "

Can anyone explain to me why I'm getting regular over-extrusion patterns on this extrusion calibration cube?

\n\n

Just upgraded the extruder from stock to Flexion HT on my Balco Touch (Wanhao i3 plus) and now I'm getting a strange pattern on my prints.

\n\n

The photos below show me trying to calibrate the extrusion multiplier after completing the extruder install. The first is with a factor of 1 and the second with an adjusted factor of 0.86 (which I suspect is the reason for the change in pattern?).

\n\n

I guess this is some kind of mechanical issue causing the over-extrusion? Any ideas?

\n\n

\"first

\n\n

Note: during the second print (below) I was playing with the Flexion extruder's Cam set screw hence why it stopped extruding all together. I tried it set tight and loose and other than when really tight and stopping the extrusion there was no real difference in the pattern when extruding.

\n\n

\"second

\n\n

Help..!

\n", "Title": "Regular over-extrusion..?", "Tags": "|marlin|extrusion|wanhao|", "Answer": "

Mine is a shot in the dark, but the fact the overextrusion is so regular let me think it is due to something rotating going around in cycles.

\n\n

The fact that the pattern changes with the flow parameter make me think the culprit is the stepper motor pushing the filament (as different flow means different number of rotations for the same lenght of printed wall).

\n\n

A possibility could be an irregularity in the cobbed wheel biting it the filament: a longer tooth would push more filament into the hot end, for example.

\n\n

Another cause could be a problem in the actual stepper motor, in which a failure in the coils causes it to move to steps instead of one, at some point.

\n\n

There are possibly other suspects in firmware settings and stepper dirivers, but that's not the area of 3D printing I am most expert in.

\n\n

Much more unlikely, it could be a defect in the filament, but that's easy to test: just swap it for another one!

\n" }, { "Id": "5511", "CreationDate": "2018-02-18T19:08:54.457", "Body": "

I have a problem with my 3d printer. When I am printing, the printer seems to drip. \nI am very confused because I never saw this before and I do not know what to do.

\n\n

I attach a picture of it.

\n\n

Why does this happen?

\n\n

Thank you very much for yours ideas.

\n\n

DATA:

\n\n\n\n

I Have enable retraction and I tried to disable it without changes...

\n\n

\"Image

\n", "Title": "3d printer drip problem", "Tags": "|prusa-i3|pla|extruder|calibration|hotend|", "Answer": "

I have two theories to offer.

\n\n

It is really dripping. If this is the case, the likely culprit is a loose part in the hot-end, namely the nozzle/heating block coupling, or the heating block/heat break one. If this is the case, you should be able to see it by visually inspecting the part. The solution in this case is to fully disassemble the parts, clean them thoroughly of any remaining plastic, and reassembling them while hot. This is essential, as parts that are assembled cold have the tendency to come loose when heated (due to thermal expansion).

\n\n

It is overextruding, and the nozzle collect and drag around the excessive plastic, until the blob is big enogh to detach and remain on the bed. If this is the case, then you should calibrate your extruder, and the nozzle height.

\n" }, { "Id": "5524", "CreationDate": "2018-02-21T12:15:51.290", "Body": "

I am trying to use a 3D850 PLA filament which is supposed to work without any issues with any PLA printer. As a printer, I use Dagoma NEVA which is supposed to work with any PLA filament. I can print with the filament without any issue, but I encounter a problem when I need to swap the filament. Somehow 3D850 sticks within the nozzle and even when it's heated I have to push really hard with another filament to push the current filament out (usually when the nozzle is heated up I can just easily push a filament inside for it to come out of the nozzle).

\n\n

Is there any special behavior of 3D850 that may cause it to stick inside of the nozzle?

\n", "Title": "3D850 stuck in the nozzle", "Tags": "|filament|pla|nozzle|", "Answer": "

My guess is that particular brand of PLA is changing state inside the nozzle as it cools. Quoting from the manufacturer's page,

\n\n
\n

Materio3D PLA uses the NatureWorks Ingeo 3D850 polymer, specially\n engineered for 3D printing. It is tougher and stronger than standard\n PLA and can be annealed for improved heat resistance and toughness!

\n
\n\n

If the residual material in the nozzle cools slowly enough (at the end of a print) to anneal, then by design it won't re-melt at the same temperature as the raw filament material. I would recommend changing your gcode so that the extruder hotend is held at temperature after a print completes, and making sure to clear the nozzle with an alternate type of filament before allowing the nozzle to cool.

\n\n

quoting from another page,

\n\n
\n

To achieve a heat treat on a printed part, submerge in water (or bake\n in oven) at 200F for up to 30 minutes.

\n
\n\n

notice that's Fahrenheit, well below extruder temperatures.

\n" }, { "Id": "5527", "CreationDate": "2018-02-21T23:02:18.833", "Body": "

Is the firts time that I saw this movement after the printing has finishig and causes the nozzle crashes to the printed part and I noticed due the part is 14x8 and the nozzle is to near and below to the border of the shape. I supposed that some scripts has changed but, seems to be everything ok.

\n\n

this is the end script:

\n\n
G92 E0\nG1 E-1.5000 F1800\n; layer end\nM104 S0 ; turn off extruder\nM140 S0 ; turn off bed\nG28 X0  ; home X axis\nM84 ; disable motors\n; Build Summary\n;   Build time: 3 hours 9 minutes\n;   Filament length: 12689.1 mm (12.69 m)\n;   Plastic volume: 30520.78 mm^3 (30.52 cc)\n;   Plastic weight: 38.15 g (0.08 lb)\n
\n\n

Z axis moves down 4mm after finishing going to X0, why? I don't want the nozzle crashes the part on going to zero.

\n", "Title": "Any code to move up Z axis after finishing the printing?", "Tags": "|g-code|z-axis|", "Answer": "

There is something fundamentally wrong with your z-motor drive if both screws are dropping the gantry at all.

\n\n

Try a couple experiments.
\n Write a quick script to move the head somewhere up high, then terminate without the X-homing command. See if the z-axis moves. Ditto for X-home but not motor disable.

\n\n

Write any script, and while the gantry is up high, pull power and see what happens.

\n\n

That will at least help narrow down the list of possible problems. In the meantime, please post your printer, the motors, and the driver board/firmware in use.

\n" }, { "Id": "5529", "CreationDate": "2018-02-22T11:20:33.190", "Body": "

I've been asked to print a set of this:\n\"enter

\n\n

https://www.thingiverse.com/thing:2200026

\n\n

I used Slicer with my default settings for my FLSun printer which includes support and 0.2mm layer height and that rail was printed as whole block with some excess which would allow me to remove the support, but that didn't work because the support simply melted with the top part of the rail.

\n\n

Tried again with no support and the first layer of the top part of the rail simply felt because there was nothing there to support it.

\n\n

Is there any configuration that would allow me to print this bridge overhang correctly? Maybe printing the rail diagonally?!

\n\n

Any help will be appreciated.

\n", "Title": "Is there any setting that could allow me to print this overhang without support?", "Tags": "|print-quality|support-structures|", "Answer": "

First of all a minor correction: the feature you are trying to print is not a bridge (which is an unsupported length of filament between two parts of the print) but rather a overhang (which is an unsupported length of filament supported only at one end).

\n\n

The reason why your overhang is impossible to print without support is that slicers create a shell first or - in other words - trace the perimeter of the layer you are printing. This feature, combined with the fact that your overhang is perpendicular to the part it originates from, causes the printer to try to extrude into thin air the perimeter of your rail, like this (the yellow lines are the shell, the red ones the infill):

\n\n

\"enter

\n\n

In the image above, also the infill is extruded into thin air, but this is actually something that you can change in most slicers (look for \"infill pattern orientation\" or something similar).

\n\n

One option you have is to tweak the support parameters to make it unobtrusive and easy to remove, like for example this:

\n\n

\"enter

\n\n

(The above was done in Cura, with infill pattern \"lines\", spaced 5mm apart and no support interface or other additional structures).

\n\n

Since your parts won't be subject to a great deal of mechanical stress another solution could be to change their printing orientation. In the following example, the overhangs do actually become bridges and as such should print without problems:

\n\n

\"enter

\n\n

Just consider it will take considerably longer to print.

\n" }, { "Id": "5535", "CreationDate": "2018-02-23T04:57:20.143", "Body": "

I just got my first 3D printer today, QIDI X-ONE[2], and so far so good with the setup and getting my 1st print. I wanted to power off the printer, but I don't see any instructions on how to properly power off the machine.

\n\n

Does anyone know how long I should wait, or what the minimum temperature would be safe to power down the machine?

\n", "Title": "Proper way to power down a FDM printer", "Tags": "|cooling|heat-management|", "Answer": "

According to the M109 G-code the documentation, the printer will wait to reach temperature given with S only when heating. With R, the printer will wait also when cooling down.

\n\n

So if the answer of mac fails to work, try:

\n\n
M109 R60 ; wait for nozzle temp to drop to 60 \u00b0C\n
\n" }, { "Id": "5537", "CreationDate": "2018-02-23T11:09:20.070", "Body": "

What is the power consumption of your heatbed (size) and hotend (model)?

\n\n

I want to verify that it is possible to use a battery to power them.

\n", "Title": "What is the power consumption of the heatbed and hotend?", "Tags": "|heated-bed|hotend|switching-power-supply|", "Answer": "

To answer the underlying (X-Y) question, yes it is possible to power a small 3D printer from a battery pack. This Article describes a printer built by Naomi Wu, mounted on a frame to carry around whilst printing, as a 'novel' style of sponsored video. The printer here is a BIQU Delta printer, and the power supply is 2x 3Ah batteries (guessing this is @12V, but it's not clear). Presumably there is no heated bed, but still the run-time will be quite limited.

\n\n

The important part for working out battery life is the duty cycle of the hot-end, not the load required to get it up to temperature. This probably comes to something like 15-30 watts on average, provided you can live without a heated bed.

\n\n

Of course, if you have 10-15v batteries, the printer will probably run off these directly, no need to waste energy converting up to 110/220V and back again.

\n" }, { "Id": "5552", "CreationDate": "2018-02-25T03:00:46.800", "Body": "

I don't know how to word this right now but I am going to try my best... moving on!

\n\n

So I was playing with my slicing software (Craftware) and I was looking at two infill patterns, Square and Parallel. When looking at strength, print quality, and time does Square really benefit a whole lot over parallel? I ask because Parallel does not use near the filament and has less contact points on the model. I provided pictures of both to give a visual. Can I use parallel and still get super strong and good looking parts?

\n\n

\"Parallel\"\nParallel infill pattern

\n\n

\"Square\"\nSquare infill pattern

\n\n

If I didn't include something or not making sense ask in the comments and I will answer/edit as needed. Thank you!

\n", "Title": "Square and parallel infill patterns", "Tags": "|infill|", "Answer": "
\n

... does Square really benefit a whole lot over parallel?

\n
\n\n

That would depend by the definition of \"a whole lot\", of course! :)

\n\n

Strength

\n\n

Generally speaking, the variable you want to operate to tweak the overall sturdiness of your part is not the type of infill, but rather its density. According to the literature I have access to, sturdiness grow fast until about 60 % infill, then the gains become progressively more marginal.

\n\n

Different types of infill modify the way the part reacts to stresses. The two you showcased, for example, are strongly anisotropic: a part with those infill will resist a lot more to stresses along the Z axis than along either the X or Y ones.

\n\n

Also, relative to the squared infill pattern, my experience is that parts with a parallel infill pattern typically have a bit more give when squashed.

\n\n

In case you wonder: cubic, cubic subdivision (and solid) infills are those regarded as offering the most equal resistance along all axis.

\n\n

However, the way the infill will affect rigidity and sturdiness of a part is as much dependent from the type of filament and the shape of the part as much as from the infill settings, so - if you are after precise numbers - you should most definitely run some tests.

\n\n

Time

\n\n

That depends entirely from the part shape and size.

\n\n

Typically, a printer will spend most of its time printing the shell. This is due both to the settings (you want to go slow on the shell to increase quality) and to physics, the shell typically requiring many more changes of direction (and thus accelerations and decelerations) than the infill.

\n\n

So, even if the parallel infill requires about half the work of the squared one (50 % faster), in a small print with a sparse infill, the total time spent doing infill may be 10 %, thus the net gain would be only 5 % of the total printing time.

\n\n

Quality

\n\n

This is very printer-dependent. For me, when printing with two shells, I can't tell the difference on any infill. For the top layer, the quality is affected more by infill density than type (with the filament sagging slightly between walls if the infill is very sparse).

\n" }, { "Id": "5554", "CreationDate": "2018-02-25T08:58:58.680", "Body": "

When going through printer reviews, I often see the presence of a MOSFET listed as an important safety feature.

\n\n

I understand the MOSFET to be a transistor, but I haven't understood how it is used in the circuitry of a printer, and - above all - why it increases the overall operating safety of the printer (nor what the less safe option would be).

\n", "Title": "MOSFET as a safety feature", "Tags": "|electronics|switching-power-supply|safety|", "Answer": "

In the world of (cheap) printers, \"MOSFET\" has taken on a meaning of its own.

\n\n

For a long time, 3D printers have had MOSFETs on board of their motherboards to switch the heated bed. In the past two years or so, we've seen a surge of (mainly) Chinese printers where the on-board MOSFETs (or, more often, the terminal blocks) weren't rated for the high current for the heated bed and would melt down or catch fire.

\n\n

People then started recommending fixes to these issues, such as using a relay, soldering wires directly to the board (to bypass the terminal blocks) or using an external (better) MOSFET. Eventually, manufacturers caught on and started offering \"MOSFET boards\" which basically consist of a high-power MOSFET, high-current rated terminal blocks and (often) an optocoupler to isolate the drive signal from the bed power (but this doesn't really do anything; there's no need for this isolation). These boards are meant to be used to switch the high current to the bed directly, without it passing through the main board (and instead, the bed output of the main board is used as a control signal for the MOSFET board).

\n\n

People often refer to these pre-made \"MOSFET boards\" simply as \"MOSFETs\" but there is more to them than just a single transistor.

\n\n
\n

why it increases the overall operating safety of the printer

\n
\n\n

Using an external MOSFET board does not increase the safety of the printer at all, unless the main board of the printer is badly designed. There's no reason the MOSFET needs to be external and can't be integrated on the main board. Essentially, any company that is offering a printer \"with a MOSFET (board)\" is saying that their main board is poorly designed and that they've included a band-aid fix. Or, perhaps, because installing an external MOSFET is such a common \"mod\" nowadays, they're advertising an external MOSFET because it is what people want to see (even if the main board might have a perfectly capable MOSFET already...).

\n" }, { "Id": "5558", "CreationDate": "2018-02-26T10:12:59.667", "Body": "

I am having problems with my tevo tarantula large bed 12Volt power supply, I am getting the thermal protection message when heating my bed with target temperature set to 115 degrees.\nThe process slows down after reaching 90.\nI changed merlin settings to trigger thermal shutdown after 5minutes/2degrees and added a cover to the printer, so getting now 103 degrees (usually shutdown was at 100/101).

\n\n

link to a video showing panel: https://photos.app.goo.gl/jiW9NE7wEB4H0mOy1 \n\"printer

\n", "Title": "How to increase bed temperature over 103 degrees", "Tags": "|heated-bed|", "Answer": "

Some simple steps that may help to reach higher temperature:

\n\n
    \n
  1. Check if you can use heated bed insulation of any kind - the most common approach is to use cork sheet
  2. \n
  3. Protect heated bed and printer from any possible air movements - arrange a set of walls around printer or just simply put it in the cardboard box large enough
  4. \n
  5. Cover heated bed with cloth or cork sheet until it reaches desired temperature
  6. \n
  7. Replace wires from heated bed to the power supply with thicker ones (2.mm2 / AWG14 should be fine)
  8. \n
\n" }, { "Id": "5571", "CreationDate": "2018-02-27T18:25:22.440", "Body": "

I'd like to secure bushings in 3D printed parts. They would be used for a drilling jig. I've seen heat-set inserts, but they've all had threaded interiors. The best idea I can come up with is printing the plastic hole several thousandths of an inch larger than the bushing and applying a lot of epoxy to the OD of the bearing and inserting it. Is there a better way?

\n\n

As an aside, I had a hard time finding proper tags.

\n", "Title": "What's a good way to secure bushings in a 3D printed part?", "Tags": "|3d-design|hardware|metal-parts|", "Answer": "

If the outside of the bushing will bond well to the epoxy, your method is simple and likely to be effective.

\n\n

You could use the same method used for threaded inserts by roughing the exterior of the bushing, heating it and forcing it into a correctly sized hole printed in the model.

\n\n

The threaded inserts are knurled or otherwise textured to provide stronger bond with the melted plastic. Knurling your bushing is more complex than a good epoxy bond.

\n\n

I've discovered a product called \"castable bushings\" which would appear to meet the above requirements for heated inserts:

\n\n

\"knurled

\n\n

The product listing indicates that these are used for drilling jigs.

\n\n

It might be less messy but more expensive to purchase and use these as opposed to epoxy and a bit of sanding.

\n" }, { "Id": "5582", "CreationDate": "2018-03-03T12:16:30.540", "Body": "

This print failed a couple of hours in. I was wondering if the nature of the print surface, with lots of retracts (similarly the previous print which was OK) might have contributed to the clog, or if it's just bad luck? To be clear, the surface has lots of short dead-end, not just a wiggly perimeter. \n\"enter\nThe filament seemed to have stopped moving, and was cut through by the drive gear.

\n\n

This was a genuine Titan Aero extruder, 0.4mm nozzle, 215C (on an Anet a8 printer)

\n", "Title": "Do more retracts lead to extruder jams", "Tags": "|print-quality|extruder|", "Answer": "

In addition to the retraction distance, in the case of this model, I was seeing some places where there were a lot of very close retractions. I think these were increasing the risk of damaged filament, so I reduced the 'maximum retraction count' which at 50 over a 5mm length seemed fairly high.

\n" }, { "Id": "5588", "CreationDate": "2018-03-04T19:36:09.447", "Body": "

This is an issue that's started happening recently. This is a Prusa I3 using a 1.75mm pla filament

\n\n

The settings for this print are as follows

\n\n
Layer height: 0.2mm\nLine width: 0.4\nInfill: 60%\nInfill pattern: Triangles\nNozzle temp: 190C\nPlate temp: 60C\nRetraction distance: 8.5mm\nRetraction speed: 60mm/s\nPrint speed: 50mm/s\nCooling fan speed: 20%\n
\n\n

This happened on a previous print, but not as much. The only difference is the other print was printed 10mm/s faster. I'm thinking it has to do with too much time in a certain area since it seems to happen in pretty small areas that don't required infill and print as two walls back to back

\n\n

\"Sample\"

\n", "Title": "Prusa I3 - Burn marks on print", "Tags": "|prusa-i3|pla|nozzle|", "Answer": "

I have seen this after I reassemble the extruder heat block and then leave it too loose during the print.

\n\n

Is your extruder above the nozzle coated in a brown or black film?

\n\n

What happens is the liquid plastic oozes through the loose connection, travels around the outside of the block, where it is heated longer and hotter than normal turning it black/brown. Then it drips, or flakes, into the print where it is spread around giving it a diffused look that you see.

\n\n

The fix is to tighten the heating block. Do this while it is at operating temperature (be careful!) because it fits differently when cold.

\n\n

Another cause: I made a tiny sharpie mark on my filament. I did not realize how concentrated it would come out in the print. Perhaps you have ink or adhesive or something on your filament that you can't see that turns brown when heated.

\n" }, { "Id": "5589", "CreationDate": "2018-03-05T12:49:58.393", "Body": "

I am building a Prusa i3 MK2S (Dolly). I find it very difficult to find M5 rods for the Z axis, I have an M8 lead screw with 8 mm lead, So I thought since M8 is widely available online I could just use them instead of M5. What do you think about it? What should I keep in mind?

\n", "Title": "Lead screws - M8 instead of M5?", "Tags": "|prusa-i3|diy-3d-printer|z-axis|printer-building|lead-screw|", "Answer": "

You can use the screws and rods you can find easily, of course some dimensions will change and is needed to adjust parameters.

\n\n

I used in my first 3D printer and standard screw of 9.5mm (3/8) the main problem was the coupling available in Mexico that werer none on that time.

\n\n

You can buy the bearings on Ali Express, they can send from there to Mexico, I guess they can deliver on your country.

\n\n

The bearings has these sizes:
\nwhere is Number ID, inner diameter, outter diameter, width.

\n\n
608ZZ 8x22x7  \n607ZZ 7x19x6\n6000  10x26x8\n
\n\n

Don't try to use 6000 :D is a little complicated to reduce the size for a proper coupling if you don't have a lathe. My 3rd Printer will use this kind of thread because is easily to find with no warpage and now I will use a lathe to reduce the coupling area to 8mm or 5mm since I have on hand the couplers.

\n" }, { "Id": "5596", "CreationDate": "2018-03-06T20:12:39.740", "Body": "

I'm getting seemingly random lines scattered across the top surface of my prints:

\n\n

\"lines1\"\n\"lines2\"

\n\n

Printer: Anycubic i3 mega
\nSlicer: Cura 3.2.1
\nPrinter chosen in Cura: Prusa i3, Gcode flavor changed to RepRap
\nCura Profile: Fine, \"Outer before inner walls\" enabled

\n\n

What might be the reason?

\n", "Title": "Seemingly random lines on the surface", "Tags": "|ultimaker-cura|", "Answer": "

It seems like the issue may only be that your nozzle has developed a bit of oozing. The g code likes to repeat the same patterns for at least a few layers every time it changes patterns and if it does the same pattern on the top 3-4 layers than by the time it reaches the last layer on the top that pattern will actually be the slightest bit raised above the rest of the layers because of the ooze building up along the same pattern for how many ever layers it followed the same pattern. Your nozzle is than rubbing on that raised pattern when it completes the last layer. This explains the randomness like OP said because the g code may not repeat the same pattern for 3 or more layers at the surface but without reading the gcode it would seemingly be random when and where they occurred. And truthfully if its a small oozing you could reprint this and it could be gone if you changed almost any setting in cura because the g code would be recalculated differently and new patterns would be chosen but so long as the new gcode did not have the same pattern repeat itself on the top 3 or so layers you would not have these raised patterns for the nozzle to rub on when finishing the top last layer.

\n" }, { "Id": "5602", "CreationDate": "2018-03-07T19:55:51.927", "Body": "

I have the Anet A6 and I was wondering if it is possible to add a filament run-out sensor. I would feel better having something to stop the filament before it runs through and I have to take apart the extruder.

\n", "Title": "Adding a filament sensor", "Tags": "|anet-a6|", "Answer": "

With marlin firmware:

\n\n

source here: filiament sensor config

\n\n
//#define FILAMENT_RUNOUT_SENSOR\n#if ENABLED(FILAMENT_RUNOUT_SENSOR)\n  #define FIL_RUNOUT_INVERTING false // set to true to invert the logic of the sensor.\n  #define ENDSTOPPULLUP_FIL_RUNOUT // Uncomment to use internal pullup for filament runout pins if the sensor is defined.\n  #define FILAMENT_RUNOUT_SCRIPT \"M600\"\n#endif\n
\n\n
\n

With this feature, a mechanical or opto endstop switch is used to\n check for the presence of filament in the feeder (usually the switch\n is closed when filament is present). If the filament runs out, Marlin\n will run the specified GCode script (by default \u201cM600\u201d). RAMPS-based\n boards use SERVO3_PIN. For other boards you may need to define\n FIL_RUNOUT_PIN.

\n
\n\n

and this is a nice piece of hardware: run out sensor

\n" }, { "Id": "5603", "CreationDate": "2018-03-08T02:16:02.417", "Body": "

One of the CAD programs I use is called TinkerCAD, which lets you export your design in either STL or OBJ form. What is the difference between these two file types? And which one is better to use?

\n", "Title": "What is the difference between STL and OBJ files and which one should I use?", "Tags": "|cad|stl|obj|", "Answer": "

While the STL-format can only describe your object aproximatively by those well known triangles, OBJ-files can describe parts of your object parametrically by curves. This can lead to a higher precision and be a huge advance with regard to scalability.\nWhich data format to choose depends, as always, on the application and the processes it contains. E.g. if you want to print large objects, you might consider the OBJ-format if your slicer can translate it to usable g-code.

\n\n

You might want to read this article on All3DP to get a brief overview.

\n" }, { "Id": "5604", "CreationDate": "2018-03-08T04:29:31.003", "Body": "

Machine specs: Prusa MK3, firmware version 3.1.2.

\n\n

Facts: I have designed a new Z-axis frame for my printer, so I can print models up to 360mm high in stead of the standard 210mm. The plan has worked and the printer is functioning normally with a new, taller z frame.

\n\n

However... To calibrate, the Z lead screws carry the X carriage all the way up and bump it against the top frame mounts, to make sure the X carriage is level. On the taller frame, the X carriage stops its ascent at 220mm and descends back toward the bed. Because the X carriage goes up farther than it was 'supposed to' and didn't bump into anything, the calibration fails. When the X carriage bumps into something at 210mm (like my fingers) and descends from the 'normal' frame height, the printer calibrates the bed levelling normally.

\n\n

The Prusa MK3 is so 'smart', it still thinks the frame will only ever be 210mm high. How do I tell it that when calibrating, the X axis must rise to 360mm instead of 210mm? The calibration is an integral part of the firmware... is there any way I can edit it?
\nI've looked at the .hex firmware file, this is the first line of a huge text file -> :100000000C947D320C94AE320C94AE320C94AE3221 It means nothing to me, but I'm guessing there's a way. After all, a cap height of 210mm had to be written in there somewhere to begin with...

\n\n

Any suggestions would be appreciated, \nfairfarren.

\n", "Title": "How to re-program Prusa firmware to accept a taller Z axis?", "Tags": "|calibration|firmware|axis|", "Answer": "

A .hex file is of no use to you, because it consists of compiled firmware which is very difficult to edit. You need to go to Prusa's GitHub and download the source code. Then, find the header file for your model of printer, and change Z_max_pos to the correct value. Finally, you need to compile and upload the firmware to your printer following the build instructions (see README.md). You will need to have the Arduino IDE installed to do this.

\n" }, { "Id": "5609", "CreationDate": "2018-03-08T14:13:23.063", "Body": "

When printing from Cura on my laptop via USB, my extruder motor does not turn to extrude filament. However, when I use the \"move axis\" command in the firmware (on the little screen on the printer), it seems to work fine, extruding plastic smoothly. The nozzle temperature is where it should be (230 degrees or so for ABS) while printing and I get no error messages in Cura.

\n\n

What would cause the extruder to not function when printing even though it seems to be fine electrically, mechanically, and in firmware? Is there g-code from Cura that could be incorrect?

\n\n

I'm not sure where to look for problems.

\n\n

This is my first 3D printer. It is a Alunar M508 (a Prusa clone) that I got for free in a partially disassembled state. The firmware seemed to be screwed up when I first started using it. The firmware that came on the SD card with the printer would not compile in my Arduino software, so I used a Marlin firmware from here:\nhttps://github.com/camalot/alunar-prusa-i3-marlin-i3-firmware

\n\n

I had to make one change to the configuration file to reverse the X home direction, but otherwise I have not changed the firmware settings or anything. The new firmware does not recognize the SD card slot at all, but that's a different story.

\n\n

I'm using Cura to print via USB. The printer is set up as a Prusa I3, using the \"Marlin\" GCode flavor. Nozzle size: .4mm, Compatible Material diameter: 1.75mm.

\n\n

Start Gcode:

\n\n
G21 ;metric values\nG90 ;absolute positioning\nM82 ;set extruder to absolute mode\nM107 ;start with the fan off\nG28 X0 Y0 ;move X/Y to min endstops\nG28 Z0 ;move Z to min endstops\nG1 Z15.0 F9000 ;move the platform down 15mm\nG92 E0 ;zero the extruded length\nG1 F200 E3 ;extrude 3mm of feed stock\nG92 E0 ;zero the extruded length again\nG1 F9000\n;Put printing message on LCD screen\nM117 Printing...\n
\n\n

End Gcode:

\n\n
M104 S0 ;extruder heater off\nM140 S0 ;heated bed heater off (if you have it)\nG91 ;relative positioning\nG1 E-1 F300  ;retract the filament a bit before lifting the nozzle, to release some of the pressure\nG1 Z+0.5 E-5 X-20 Y-20 F9000 ;move Z up a bit and retract filament even more\nG28 X0 Y0 ;move X/Y to min endstops, so the head is out of the way\nM84 ;steppers off\nG90 ;absolute positioning\n
\n\n

Edit to Add some Gcode from the beginning of the test print:

\n\n
;LAYER_COUNT:127\n;LAYER:0\nM107\nG0 F3600 X86.923 Y86.649 Z0.3\n;TYPE:SKIRT\nG1 F1800 X87.522 Y86.134 E0.03941\nG1 X88.17 Y85.683 E0.0788\nG1 X88.861 Y85.299 E0.11824\nG1 X89.586 Y84.987 E0.15762\nG1 X90.339 Y84.75 E0.197\nG1 X91.113 Y84.591 E0.23642\nG1 X91.898 Y84.51 E0.27579\nG1 X92.3 Y84.5 E0.29586\nG1 X117.7 Y84.5 E1.56307\nG1 X118.489 Y84.54 E1.60248\nG1 X119.269 Y84.659 E1.64185\nG1 X120.034 Y84.857 E1.68127\nG1 X120.774 Y85.131 E1.72064\nG1 X121.483 Y85.479 E1.76004\nG1 X122.154 Y85.897 E1.79948\nG1 X122.778 Y86.38 E1.83885\n
\n", "Title": "Extruder motor does not turn", "Tags": "|extruder|g-code|", "Answer": "

It turns out the problem was a firmware issue. Using the gcode below (from David Lotts in the comments), I was able to determine that the extruder was running backwards. A flag in the firmware reversed it, solving the problem.

\n\n
M82 ;set extruder to absolute mode \nG92 E0 ;zero the extruded length \nG1 F200 E20 ;extrude 20mm of feed stock\n
\n" }, { "Id": "5610", "CreationDate": "2018-03-08T14:56:52.343", "Body": "

I am experiencing communication drops when printing from mp raspberryPI (using octoprint, repetitier server). Moreover when printing from Cura (on Windows), the programs freeze (have to kill the window) and sometimes stops the print job.\nI was changing the serial0 speed in Marlin from 225k to 9600 and still no luck.\nMainboard MKS 1.4 all in one.

\n\n

please let me know if you need any more details

\n", "Title": "Communication is dropping when printing on my Tevo Tarantula", "Tags": "|marlin|firmware|", "Answer": "

As per long article Marlin firmware it looks like my system is a victim of closed ground loop via USB connection. It is now clear to me that printing from laptop (connected to different socket) was a ground loop separated case and therefore the connection was stable. Connecting my raspberryPi from same socket using long unshielded cable is the main reason of connection drops.

\n\n

Will try to run raspberryPi with short shielded cable and also will test raspberryPi powered from power bank.

\n" }, { "Id": "5614", "CreationDate": "2018-03-10T19:05:08.900", "Body": "

I am designing a fan mount for the CR-10S printer to hold the E3D All-metal v6 HotEnd and BLTouch sensor.

\n\n

The CR-10S printer has two fans.

\n\n

One on the front:

\n\n

\"enter

\n\n

and another one on the right:

\n\n

\"enter

\n\n

I need to know what the two fans are actually cooling in order to make a perfect mount.

\n\n

When powered on, the front fan seems to be on 100 percent of the time. The right fan can only be on via gcode when printing.

\n\n

I removed the fan mount to study it and it looks like the front fan is cooling the hotend heat sink.

\n\n

\"enter

\n\n

The right fan is attached to the mount and there no hole there to let the air from the fan go through. It seems to be cooling the metal it is bolted to and that doesn't make sense.

\n\n

Am I correct about the front fan? What's the right fan cooling?

\n", "Title": "Understand what CR-10S fans are cooling", "Tags": "|cooling|print-fan|fans|", "Answer": "

The function of the front fan is keeping the cold end well... cold. :) It should be spinning as long as the printer is turned on.

\n\n

The right fan is known as \"part fan\" and its function is to cool down the plastic that has been just extruded, the idea being to solidify it as soon as possible. As you noticed it is controlled by the gcode and it can be turned on, off and even made spin at any speed in-between still and full throttle. It normally stops spinning at the end of the print (with some plastic like ABS it may be set not to spin at all even during the print).

\n\n

If you take a closer look at your CR-10, you will see that the receptacle the air is blown into by the part fan has no bottom: that slit is where the air passes through, before being deflected 90\u00b0 towards the nozzle.

\n\n

On the CR-10 the standard deflector for the part fan does not do a good job (the air is not blown where it should) and replacing it with a custom part is one of the most common upgrades performed on the printer.

\n\n

Personally on my CR-10 I used a custom mount with an integrated \"fang\" that blew the air from both sides and worked really well. It also had the advantage of reusing the hardware from the original assembly (= you don't need to buy anything, just print the part).

\n" }, { "Id": "5618", "CreationDate": "2018-03-11T11:32:40.657", "Body": "

I have this fan model, it is a SUNON model number MF50151VX-B00U-A99\nand it is a blower type.

\n\n

\"enter

\n\n

When the head is moving in the X-axis it makes noise.
\nI think this type of fans is not suitable for rapid movement and rapid changes in directions.

\n\n

I think the noise is coming from the axial of the fan because I think there is a clearance in the axial for moving up and down.

\n\n

When I put my finger on the fan body(the rotating part) the noise stop!

\n\n

My question, What type of fans suitable for rapid movement and rapid direction change? and if this is not the problem what is the problem in my situation?

\n\n

I have tried searching but I can really find a direct answer!

\n", "Title": "Cooling fan noise when head is moving in the X axis", "Tags": "|diy-3d-printer|cooling|mechanics|print-fan|", "Answer": "

MagLev fans with Vapo bearing are (like the name suggests) magnetic levitation bearings - the rotor "floats" in the air, it's not press-fot like in ball bearings, therefore it has movement in perpendicular to the axis.

\n

MagLev fans are great at low acceleration and low jerk (or in stationary applications) but their bearings will bang on sudden movement changes. Just take a MagLev (with Vapo bearing - VX, V1, V2..), there actually are Dual Ball Bearing versions of MagLev fans (BX, B1, B2..) which are way better suited in moving applications.

\n" }, { "Id": "5621", "CreationDate": "2018-03-12T05:31:51.617", "Body": "

About two days ago, I started seeing that my hotend was heating up erratically. I first noticed this while printing a part in PETG and the temp jumped to 260 \u00b0C. I shut down the printer at that time and first started checking the hardware. I noticed that the E3D V6 thermistor had been tightened too much. I disassembled the entire hotend, cleaned everything and then reassembled everything. I thought to retune the hotend and when I tried tuning it at 240 \u00b0C.

\n\n

This is where the strange behavior occurs. The hotend steadily climbed up till about 200 \u00b0C. After that it just went nuts. I started seeing unreal temps such as 646 \u00b0C and such. At this point I thought the MEGA might be at fault. I replaced it and the hotend (an E3D V6 clone). This had the screw on glass thermistor. Again the same erratic behavior and unreal temp readings.

\n\n

What could be wrong here? What am I missing? Can this be the heater cartridge?

\n", "Title": "RAMPS won't PID tune and shows unreal temps", "Tags": "|ramps-1.4|hotend|thermistor|", "Answer": "

One of the thermistor wires had come loose from the crimp ferrules. I re-crimped it and it works fine now.

\n" }, { "Id": "5625", "CreationDate": "2018-03-12T20:47:31.797", "Body": "

So like I sayed in the title, Why can't the Anet A6 do .05 layer height? I found that some printers can do .05 layer heights, but the Anet A6 can't? I am interested to know if it is the stepper motors or the threaded rods or something. Maybe this is something I can do a small \"test print\" on?

\n", "Title": "Why the Anet A6 can't do .05 layer height", "Tags": "|anet-a6|", "Answer": "

I have an Anet A6 and once you get your settings dialled in, bed level accurate and belt tensioner correct you can do 0.05 mm very well (although I for some reason went with 0.06 mm and never tried the extra 0.01 mm lower but it shouldn't have an issue). Note: I did this using all the stock parts so 0.4 mm brass nozzle.

\n\n

I highly recommend you print and install a front Y axis frame brace and a Y axis belt tensioning mod as well as an X axis belt tensioner. It will allow you to print at these layer heights much more accurately if you can adjust the tension correctly.

\n\n

The kit out of the box makes it almost impossible to get good/correct tension on the belts.

\n" }, { "Id": "5632", "CreationDate": "2018-03-14T20:41:20.770", "Body": "

I often tried to print sth in the prusa i3-mk3 but in most time he prints at the middle of the object more right or left. Can someone please tell me why that comes?

\n", "Title": "Why does the prusa i3 sometimes put the layer more right or left as they should be?", "Tags": "|prusa-i3|", "Answer": "

I'm not sure if I read your question correctly, but if I do, what you are referring to is called \"layer shifting\" and looks like this:

\n\n

\"enter

\n\n

This happens when the stepper motors fail to perform a step upon receiving the signal from the firmware. Since stepper motors do not have any way to know their actual position (differently, for example, than servos) they will keep on printing as if nothing happened, and thus the subsequent layers will be shifted of the amount of steps they missed to perform.

\n\n

Moving on to why this happens in sth... I don't own a MK3, but I take it that sth is short for stealth mode, the new silent mode that has been widely showcased in reviews and articles on the printer.

\n\n

That mode of operation is made possible by the Trinamic TMC2130 stepper drivers. Normally these drivers monitor the power consumption of the steppers and are capable of deduce a missed step by sudden changes in that. If they do, the MK3 will actually re-home the X and Y axis to fix the problem and resume printing normally.

\n\n

However when operating in their StealthChop mode, the TMC drivers provide less energy to the motor themselves (to keep them operating quietly) and - more importantly - are unable to detect missed steps. Less power will make much more likely that any sort of resistance to the print head movement will cause missed steps, the absence of detection will cause the printer to not even notice and cause layer shifting.

\n\n

Again, I don't own a MK3 and I have no direct experience with it, but I would suggest updating the firmware to the very last version: I read in an article some weeks ago that a recent version addressed exactly your problem, by progressively increasing the power output along the Z axis (as the likelihood of layer shifting increases with the height of the print).

\n" }, { "Id": "5637", "CreationDate": "2018-03-15T21:47:08.043", "Body": "

We have a Prusa i3 MK2 and we've changed for a print with another colour material but when I pressed \"load filament\", it didn't make anything.

\n\n

Often after five times doing that the printer works. I don't have any pictures but the filament is in the hole and the printer holds it. However, it doesn't push it into the extruder and the remains of the old filament do not come out, so I wonder why the motors just stop and don't work.

\n\n

Sometimes there are no problems - sometimes more and sometimes less. I hope someone has the reason for this problem.

\n", "Title": "Prusa i3 MK2 doesn't accept the printing material", "Tags": "|extruder|infill|", "Answer": "

I would agree with PR90, cutting the filament tip at a 45% angle before inserting might help. If the new material has required a different temperature to the previous, then you could have burnt the old material clogging the nozzle (take nozzle off by unscrewing, if it a brass hex nozzle) - if the new material requires a lower temperature, then it will not melt the materiel old pushing it out.

\n\n

See if you can go into Move Axis (manually) and then go to 0.01 mm then Extrude and see if that works.

\n\n

If you hear a slipping or backlash sound coming from the extruder stepper motor, look at motor gear and check for clogging.

\n\n

Please check the heat requirements of the new and old materials

\n" }, { "Id": "5638", "CreationDate": "2018-03-16T06:42:56.157", "Body": "

My problem:

\n

I need to connect to my 3D printer via the internet from an Android app (using IP address and API key).

\n

My 3D printer is based on Marlin firmware latest and connected to a Raspberry Pi running OctoPi. I don't want to use a router and port forward it, because the Raspberry will be connected to internet via a Wi-Fi hotspot.

\n

My question:

\n

Is it possible to control the printer using a cloud platform or a server (another Raspberry Pi) if my OctoPi Raspberry Pi can access the internet? If it is possible, how can one do it?

\n

Any suggestions or solutions are welcome.

\n", "Title": "Connect to 3D printer via internet without router", "Tags": "|software|printer-building|", "Answer": "

There are a few solutions, but each one needs to match same criteria: an open network socket visible outside the local network.

\n\n

So from TCP/IP protocol theory, we don't even need to have an IP address to send a packet to another computer as we can use layer One which is just the MAC address to identify the receiver. But things become more complicated when we have to provide a link to two separate endpoints.

\n\n

Basic requirements:

\n\n
    \n
  1. We need to know public accessible IP address and port that server is exposed (listening) on
  2. \n
  3. We can have a DNS entry that will be translated to public IP, but that assume that public IP is same all the time (or we have a dynDNS service)
  4. \n
\n\n

Solutions:

\n\n
    \n
  1. Have the RPi exposed with public IP address and routed properly;
  2. \n
  3. Create a VPN bridge from RPi to your home/cloud network and connect Android to the same VPN;
  4. \n
  5. Do a port forwarding to RPi from public IP and expose that in the firewall.
  6. \n
\n\n

So at this point, if you don't have admin access to network devices then only solution '2' is viable, IMHO.

\n" }, { "Id": "5639", "CreationDate": "2018-03-16T09:25:15.337", "Body": "

I have an STL file that I want to add a tribal tattoo to with the end goal of getting it printed in colour sandstone from Shapeways, or in full colour on an Mcor Arke at some point, but I'm not really sure where to go.

\n\n

I've looked at Microsoft Paint 3D. It's ok, but I cannot get the paint job symmetric. It outputs the .3mf format which the Arke will eventually handle with their Orange software, but it won't export into anything Shapeways will accept.

\n\n

I've looked at UV unwrapping in Blender, but it seems I have to create the image in another graphics program and then import it... but I can't really work out where the images will be applied to the object, and then when I'm done it seems I cannot export the bits I'm expecting. On the plus side I have the mirror function so I can just paint one side and it's symmetrical.

\n\n

I've looked briefly at Maya and 3Ds Max, but they are eyewateringly expensive, way out of my league.

\n\n

Blender with some form of layers for the image might be really useful so I can make the tattoo in bits and then apply them to parts. I've kind of seen how you can split the model into panels but my brain is struggling to get round this as it's really quite hit and miss... and exporting seems to be a problem, although DAE and the texture will go to Shapeways, assuming the DAE has the texture layout embedded. FBX seems to go into Paint 3D and then I can export it to 3MF for the Arke.

\n\n

Has anyone got any helping pointers for this confused noob?

\n\n

Thanks

\n", "Title": "Painting an STL file", "Tags": "|3d-design|", "Answer": "

It is indeed more related to the blender forum than the 3d printing one.

\n\n

You can start by looking at this post from blender.stackexchange.com which explain how to unwrap and paint on a 3d object. You can also paint the texture on a 2d software like Gimp and then put it in the UV Image editor of Blender. Then you should be able to export your project (with the texture) in a suitable format for 3d print.

\n" }, { "Id": "5641", "CreationDate": "2018-03-17T08:28:28.860", "Body": "

I bought a new printer, and unfortunately the left nut for the lead screw isn't tapped.

\n\n

Does anyone know the thread size, w.r.t. both pitch and lead, of the lead screws?

\n", "Title": "Does anyone know the thread size (pitch and lead) of the Anet A8's lead screw?", "Tags": "|prusa-i3|z-axis|hardware|lead-screw|anet-a8|", "Answer": "

The pitch no matter the screw is the distance between the threads. The lead can be different based on a one or two start threads.

\n" }, { "Id": "5646", "CreationDate": "2018-03-17T20:57:46.850", "Body": "

I have an NEMA 17 stepper motor 17HS4401 and a RAMPS 1.4 driver unit.

\n\n

The motor pinouts are:

\n\n
RED   A+\nBLUE  A-\nGREEN B+\nBLACK B-\n
\n\n

while the RAMPS board has the following:

\n\n
2B 2A 1A 1B\n
\n\n

Could someone point me in the right direction? How can I connect the driver to the motor? I have a feeling I already messed things up.

\n", "Title": "How to connect a NEMA 17 stepper to RAMPS 1.4?", "Tags": "|ramps-1.4|stepper|wiring|nema-17|", "Answer": "

There are very many combinations that will work. A bipolar stepper has 2 coils. Swapping the coils, or reversing the polarity of a coil, will simply cause the stepper to rotate in the opposite direction.

\n\n

On RAMPS, one coil should connect to 1A and 1B, whereas the other coil should connect to 2A and 2B.

\n\n

On the stepper, one coil is A+ and A-, the other coil is B+ and B-.

\n\n

A possible wiring is A+ to 1A, A- to 1B and B+ to 2A and B- to 2B, but there are 15 other possibilities. The only \"wrong\" way is one that connects a coil of the stepper to two different coil outputs on the RAMPS (so connecting, for instance, A+ to 1A and A- to 2A wouldn't work).

\n" }, { "Id": "5649", "CreationDate": "2018-03-18T21:41:31.840", "Body": "

I just finish building a Graber i3 printer from mixed parts (a MDF clone of Prusa i3 MK2). For the hotend, I've bought the Greetech MK8 extruder. However, the temperature sensor is driving me up the wall.

\n\n

I've already checked the wires and connections, but the darn thing keeps showing a steady 500 degrees Celsius. In Marlin, it shows it right up (I'm using the 1st option for a 100k thermistor), in Repetier Firmware (using the same one), it shows 0.00\u00b0C, until I tell the printer to heat up, when it shoots to 500\u00b0C just like before with Marlin. The documentation for this extruder only lists it as a \"100K NTC Thermistor\", so I tried to select one of the NTC options on the list for both Marlin and Repetier and the temp sensor reads steady 3.600\u00b0C!!!

\n\n

Measuring it with the multimeter, it shows around 60k, it's 31\u00b0C outside.

\n\n

Is it broken or am I selecting the wrong thermistor type?

\n\n

Documentation for MK8

\n", "Title": "MK8 extruder showing 500+ \u00b0C on temperature sensor?", "Tags": "|prusa-i3|extruder|thermistor|", "Answer": "

If your printer is a cheap one that uses a ribbon cable or other combined cables, verify your thermal sensor's polarity. I know, I know - it's a resistor (in most cases) and that means there's no such thing as polarity. Bear with me - I just helped a friend diagnose his printer (Geeetech, for the record) for showing 500C on one sensor and 'def' on the other, after replacing the mainboard with a different/more capable model. Turns out, they SHARE PINS, which suddenly makes polarity important. Swap the pins in the connector, and it might just work. First time I've run into this. We swapped only the Ext0 sensor and left the bed disconnected - Ext0 read normal, so we swapped both, and everything's been perfect since.

\n" }, { "Id": "5660", "CreationDate": "2018-03-21T11:40:05.433", "Body": "

I am building a dolly and I am confused as to which proximity sensor to use.

\n\n

Should I go for M8 or M12 and 5 V or 6-36 V?

\n\n

What should be the best detecting distance? Should it be 2/4/8 mm. Which one should I select?

\n", "Title": "Which proximity sensor to use?", "Tags": "|prusa-i3|diy-3d-printer|z-axis|z-probe|", "Answer": "

Duet3D has a very nice overview. It totally depends on your demands. e.g., accuracy, different build surfaces, tolerance. Personly I realy like the piezo. It actually uses your nozzle to 'touch' the bed. So 0 height is absoluty 0. This eliminates the need for Z-offset.

\n" }, { "Id": "5666", "CreationDate": "2018-03-21T14:30:39.123", "Body": "

Harking back to the days of \"singing disk drives,\" I am wondering if anyone's written music to be performed on a 3D printer. Most of us have noticed in passing that the servo motors for X and Y drive generate a different pitch depending on motion speed. With some care and experimentation, one could write g-code to produce not only a tone but even a 2-tone chord. So -- has this been done? Does anyone want to do so? (Note that there's no need to simultaneously produce a print, but that would be even classier).

\n", "Title": "Music from the printer?", "Tags": "|g-code|", "Answer": "

I actually did this, on my Malyan M150.

\n\n

After not that much experimentation at all it turned out that the frequency of the produced tone scales linearly with the speed of the motors. Due to the nature of the way humans perceive music, the actual frequencies do not matter at all, just the relative difference in frequency between the tones. This means that it is actually really easy to let a 3D printer play a tune, you just have to make sure that when the next tone needs to have a frequency that is for example 20% higher than the last one, the motor needs to move 20% faster, no need to actually measure at which speeds the printer produces which tones.

\n\n

One of the main issues turned out to be the maximum speed of the motors, which wasn't high enough for some of the higher notes, but this issue could be solved by multiplying all the speeds by a number lower than 1, thereby lowering the required speeds while still maintaining the original melody. Playing two tones at the same time proved to be a bit too difficult for my not that great programming skills, while not being worth the effort because somehow there was quite a large difference in volume between the motors.

\n\n

In the end, I made a MATLAB script that converts MIDI files to G-code with the following result: 3D-printer playing popcorn song

\n" }, { "Id": "5669", "CreationDate": "2018-03-21T23:50:50.940", "Body": "

I'm new at 3D Printing. I made a model of what I want to print using blender, but it's really small with a little system that must be very precise to work.

\n\n

What I have is like a hollow cylinder with a diameter of 11mm, and a thickness of 1mm. This goes inside a counter part, that is carve with the cylinder shape, but with a little extra gap so it can rotate like an axis. Maybe this picture explains better:

\n\n

\"enter

\n\n

And this should be the front view of the counter part:

\n\n

\"enter

\n\n

So, I need to know if it's possible to print that cylinder hard enough to work as an axis. And what should be the gap size between the cylinder and the counter part's hole to rotate properly? If it's is like 0.05mm, can I print that level of detail with a 3D printer too?

\n\n

What hardware and material should I use to do this?

\n\n

Thanks in advance.\nSorry for bad english, I hope you understand!

\n\n

EDIT:

\n\n

\"enter

\n\n

(6mm is the depth of the hole)

\n", "Title": "Is possible print this level of detail?", "Tags": "|print-quality|print-material|", "Answer": "

It is definitively possible to do what you want, but your questions are samewhat problematic:

\n\n
\n

So, I need to know if it's possible to print that cylinder hard enough to work as an axis.

\n
\n\n

\"hard enough\" is a mysterious quantity. What is the intended application? The load of the axis, the rotation speed, the medium in which the part will be in, its operating temperature... they all affect the answer.

\n\n
\n

And what should be the gap size between the cylinder and the counter part's hole to rotate properly?

\n
\n\n

Reading at the question and the comments, I think you may have the wrong representation model in your mind. There are four different concepts at work here:

\n\n\n\n

So: what you want to achieve for the object to rotate is to have at least some clearance once you have the parts printed. Therefore, you want to design your part with an allowance which is at least as much as the accuracy.

\n\n

Note that a machine cannot produce parts with a tighter tolerance than its accuracy. So you must design your part with a tolerance equal or greater than your printer accuracy.

\n\n

The correct number will therefore be entirely dependant from the specific printer you will be using. You can find out the specific accuracy of a printer by printing a tolerance test (I know, I know... why isn't it called \"accuracy test\"?)

\n\n

See this unrelated answer - from wich I took the above definitions - for learning more about the above and a concrete example.

\n\n
\n

If it's is like 0.05mm, can I print that level of detail with a 3D printer too?

\n
\n\n

I hope it is now clear why this question makes no sense: clearance is a variable which depends from accuracy (and the application), not the other way around.

\n\n
\n

I can't add so much gap because I have really limited space

\n
\n\n

This comment too is incorrect: the \"gap\" (clearance) can be very very small. You have to have the correct allowance in your design, and allowance will not intrinsically make a part larger.

\n\n
\n

What hardware and material should I use to do this?

\n
\n\n

Again: this is entirely dependent from your application (load, operating temperature, orientation, speed...)

\n\n

A consumer-grade FDM printer (easy accessible, cheap and cheap to operate) will allow you to print a rotating part, a SLA/DLP printer (less common, toxic resins, more expensive to operate) will allow to print the same part with different materials and tighter tolerances...

\n\n
\n

I don't worry about breaking, but it cannot be flexible

\n
\n\n

Again: without an explanaton of the intended use (or the numbers associated to it) it's impossible to answer this comment conclusively. Resins tend to harden to more rigid solids, but you have thrown around tolerances as small as 0.05mm in your writing, and over 12mm of axis, that is a deviation of less than 0.5% from \"perfectly straight\". I'm hard pressed to think you will find a printable material with such a rigidity.

\n" }, { "Id": "5674", "CreationDate": "2018-03-23T01:26:52.520", "Body": "

Is there any commonly printed plastic which I can buy that might be transparent to UV light?

\n\n

I wish to print a mould, then pour in my plastic which requires a UV light to activate the curing process.

\n", "Title": "Transparent Plastic", "Tags": "|print-material|", "Answer": "

Have you tried printing with Form labs clear resin ?

\n\n

( This is assuming you have access to a desktop SLA like the Form2+ or Form 1 )

\n\n

The printed part itself gets hardened after exposure to uv light and actually can increase the life time of your mould .

\n\n

I have been using the Form labs Form 2+ a lot recently for small moulds and casting them with Polyurethane . They work amazing and the precision of the mould is high due to 0.05mm layer resolution that is achievable .

\n\n

Once hardened the UV light easily passes through it and does not degrade the part at all .

\n\n

Here is the material data sheet \nhttps://formlabs.com/media/upload/Clear-DataSheet.pdf

\n\n

You can easily get a clear resin print done through a local 3D printing service bureau that you can find using 3D hubs .

\n\n

Prices for printing on a formlabs are comparable to FDM 3D printing , it really just depends on your supplier .

\n\n

If the above doesn't cut it then ,

\n\n

Use DSM Somos\u00ae WaterClear Ultra 10122 , please check below link and corresponding data sheet for technical specifications .

\n\n

https://www.dsm.com/products/somos/en_US/products/offerings-somos-water-clear.html

\n\n

This can only be printed on a 3Ds systems viper , Envision tech preform series or other industrial 3D printers .

\n\n

Again you can head to 3D hubs or call up the closest industrial 3D printing service bureau and ask them for the above material . They should be able to hook you up easily .

\n" }, { "Id": "5676", "CreationDate": "2018-03-23T09:54:50.250", "Body": "

I want to upgrade my Anet A8's motherboard (the stock one is broken anyway, see: Anet A8 reading 739°C from the extruder thermistor!) to the RAMPS 1.4, without having to get a new LCD/microSD module. I am thinking of getting both a smart adapter and sdramps, then maybe plug the sdramps and the stock Anet A8 LCD into the smart adapter, but I am not sure whether the sdramps module has a different pinout than the RAMPS smart controller, which is what the smart adapter is made for.

\n\n

Can someone confirm whether they have the same pinouts or not, or give me some alternatives to my idea?

\n", "Title": "Plug sdramps module and Anet A8 LCD together into RAMPS smart adapter", "Tags": "|ramps-1.4|anet-a8|wiring|microsd|sd|", "Answer": "

The Anet A8 uses a simple LCD with 5 buttons, the problem is that the buttons do not use digital pin signals, instead analog pins are used. Based on the pressed button, with corresponding resistance the firmware knows which button is pressed. The RAMPS smart adapter uses digital pins, so the Anet LCD is not compatible with the smart adapter, you may wan't to opt for a cheap Chinese 12864 LCD smart controller, they are about $8, they are compatible.

\n\n
\n\n

The topic Connecting (Anet A8) 2004 display to MKS GEN L shows you to hook up the Anet A8 LCD to a RAMPS board. You do not use the smart adapter, instead you use AUX2. You only have to change 2 wires (interchange them on AUX2) and you will be able to use the display. The reason you cannot use the smart adapter is that the 5 button display requires analog pins, which are not available on the smart adapter, but are available on the AUX2 header.

\n" }, { "Id": "5695", "CreationDate": "2018-03-26T16:06:20.553", "Body": "

https://photos.app.goo.gl/O6yPf3sDeV1yhS0C2

\n\n

I tried to illustrate my problem in the videos above, two of them show the weird movement and the other shows me clicking on the home button repeatedly.

\n\n

Some Info:

\n\n\n", "Title": "Weird Movement and not homing", "Tags": "|marlin|ramps-1.4|motor|stepper-driver|homing|", "Answer": "

The problem was the logic of the endstops that were reversed

\n" }, { "Id": "5706", "CreationDate": "2018-03-27T15:30:40.190", "Body": "

I have this clogging problem on my 3D printer that I am having a really hard time to solve. I am a hobbiest so in no way am I a professional with regards to 3D printers. I also know that this is a popular issue and I have tried a lot of the proposed solutions (going through a lot of PLA filament and failed prints) to no avail. I will try to describe the problem and what I have tried to solve it.

\n\n

First the issue:\"Removed

\n\n

This shows how the filament clogs. The next image shows the assembled printer head for reference.: \"Assembled

\n\n

This is how the print looks when it fails:\"Failed

\n\n

I have tried the following:\n- Adjusting the temperate between 190 up to 220 for the PLA filament. Everything trying a print with it.\n- Doing a thorough cleaning of the nozzle (Acetone soak, wire brush, guitar string etc.)\n- Checking the Teflon pipe inside the tube if it is clear and ensuring that the edges allow the filament to pass through.\n- Check if the cooling fans work well to ensure cooling of the whole assembly (they work, they aren't jammed etc.)\n- Dissambling the printer head and putting it back together.

\n\n

I have printed with the printer before without a problem. I printed PETG a few times to get that working and the results were decent. When I returned to PLA this issue started and no matter what I change on the profile it won't work. I have tried adjusting feedrate, flow rate, retraction amount and speed, temperature, fan speeds to name but a few things.

\n\n

I have a Cura profile I can upload if that will help. Any advise would be much appreciated. I have been struggling with this for a while now and really need some options.

\n", "Title": "Filament clogging just above feeding tube - Duplicator 6", "Tags": "|pla|", "Answer": "

that looks like heat creep.
\nheat creep is the result of in too much heat in the hot end creeping its way into the cold end. and results in a gradual clogs mid print.

\n\n

to combat heat creep you need to sink that heat elsewhere and dissipate it.
\nthis is where a heatsink and fan come into play.
\nthe heatsink should be in direct contact with the throat mount/clamp, giving the heat more space to dissipate, and that dissipation is aided by the fan.

\n" }, { "Id": "5724", "CreationDate": "2018-03-29T14:10:50.857", "Body": "

I have an Anet A8 printer for about 4 months, set up pretty well (or so I thought) and printing a number of models pretty well. I made a large 3\" x 6\" box with a sliding lid yesterday and when it was done there was a gap on one side when the lid was slid on. I checked the parts and it turns out they are not square - which means the X and Y axes are not square to each other.

\n\n

I'm wondering how to adjust this - I'm thinking that extending the distance between the back of the frame and the front by adjusting the threaded rods that separate them to a wider distance on the side where the angle is obtuse. Obviously one of the first things I'll check is that the distance between the front and back is the same (I can't imagine why I never checked that before, come to think of it).

\n\n

Does this sound like a sound plan?

\n", "Title": "Anet A8 - X and Y axis not square - how to fix?", "Tags": "|calibration|anet-a8|y-axis|", "Answer": "

To correct x-y axis alignment to 90 degrees: Loosen the nuts on the two threaded rods at one end and in the middle. Lower the Z-axis to its lowest level. Place a square block of wood or metal so that it rests on the table and is snug against the X-axis rods. Slide the table so that either its rear or front edge is under the edge of the block. There you can clearly see any misalignment by comparing the edge of the block and the edge of the table. Now the 'fun' part: rack the entire assembly, pulling the diagonal corners to square the table and block. You may have to tape the assembly down to keep it in place while you spin the threaded rod nuts back into place....gently,then carefully snug the nuts while watching the table and block to insure they stay put!

\n" }, { "Id": "5729", "CreationDate": "2018-03-29T20:17:44.827", "Body": "

I recently bought an Anet A6 and decided to assemble it today. When I plugged in the power, I don't get any sign of life from the printer. No lights, no bleeps, no fans which are starting to blow.

\n\n

I checked the motherboard and it receives power from the adapter (I live in Europe, the Netherlands, so the voltage is 220V) but nothing happens.

\n\n

Can someone help me out?

\n\n

I added some pictures of the wiring for references. Display cables are correctly set (J3 to J3 and LCD to LCD).

\n\n

\"enter

\n\n

\"enter

\n", "Title": "Anet A6 - First time assembled, won't turn on", "Tags": "|troubleshooting|wiring|anet-a6|", "Answer": "

Welcome to 3d Printing!\nas this is a first day of the printer there could be a list of items...

\n\n

First thing first: disconnect all peripherals, so we can test mainboard.

\n\n

Before connecting PSU we can connect mainboard via usb and see if it starts (I will probably use a power bank in that case to avoid burning USB (worse case scenario)).\nThen you can use cura or another host program to connect to the printer and validate firmware. If the printer is not recognized we can reflash the firmware using provided app or ArduinoIDE .

\n\n

If all looks good we can check PSU.

\n\n
    \n
  1. please check output voltage from power supply unit (shall be 12V) (on the terminals, there is a small voltage regulator on the lefthand side)
  2. \n
  3. then double check polarity
  4. \n
  5. connect power to mainboard
  6. \n
  7. check if we have same voltage on mainboard terminals
  8. \n
  9. then check if we have 12v on the fan terminal (40 * 10 fan shall run all the time on the hotend)
  10. \n
\n\n

if that is done and we have power provided to the mainboard then we need to check for 5V that powers the arduino mega2560 (that could indicate a faulty power regulator)

\n\n

Select one of the available sockets and measure voltage on VCC and GND pins. If we have 5V then we can connect pheriperials one by one, always switching off PSU and disconnecting USB.

\n\n

If one of the underlying components is faulty then you will see straight after powering the mainboard.

\n\n

\"reprap

\n\n

image source

\n" }, { "Id": "5736", "CreationDate": "2018-03-31T22:48:49.780", "Body": "

I have a simple printer bot metal with a heated bed, the heated bed I am not using. I am using conductive pla by protopasta

\n\n

The conductive pla is not that strong, so when I take my pieces off the board, sometimes they break. The only time it appears to be invincibly strong is when it sticks to the bed plate! I cannot get the skirt off the bed plate, no matter what I try

\n\n\n\n

What\u2019s a good way to remove a conductive pla skirt from one of the beds? The skirt is the initial outline a printer lays down, it is very thin

\n", "Title": "Removing protopasta conductive PLA skirt from a bed", "Tags": "|adhesion|", "Answer": "

I think the setting having the most significant effect would be to increase the Z-height by 0.05-0.1mm so that the skirt is not \"sandwiched\" against the bed like typical first layers in FDM. This should reduce the adhesion enough for you to remove the skirt from the bed without damage.

\n\n

Alternatively, you could also increase the skirt extrusion width if possible (more material=stronger) and/or print another layer of skirt on top (skirt layers=2).

\n" }, { "Id": "5743", "CreationDate": "2018-04-02T21:02:54.000", "Body": "

Ultimaker Cura offers a platform support type of \u201ctouching buildplate\u201d which enables the printer to only make a raft for parts of the object that should be touching the build plate. It also offers \u201ceverywhere\u201d for any object that might be hanging over the build plate.

\n\n

I have a need to only offer support for overhangs up to a certain z height, such any overhang located at a z-point of 4 mm or below. Is there a software that will enable this, either as a setting/addition to Ultimaker Cura or just a G-code export for Pronterface?

\n", "Title": "Platform support up to a certain Z height in Ultimaker Cura/G-code", "Tags": "|ultimaker-cura|g-code|support-structures|", "Answer": "

As an alternative, look into 3D modelling tools which allow you to add your own support towers to the model itself. Do that, then set your slicing tool to \"no support\" or equivalent.

\n" }, { "Id": "5748", "CreationDate": "2018-04-03T18:48:05.973", "Body": "

For example, to make a DIY cartesian 3d printer you could use/do the following:

\n\n\n\n

If you want to make a DYI delta 3d printer, which point of this whole process needs to be altered in order for the delta printer to work properly? Is there an existing open source software for delta printers/cncs?

\n\n

EDIT: This question could be asked about any kind of non-cartesian 3d printer, including Delta, SCARA, Polar, etc.

\n", "Title": "At which point does a delta 3d printer change the G-code in order to work in its non-cartesian design?", "Tags": "|diy-3d-printer|delta|open-source|", "Answer": "

Every 3D printer or machine tool that is commanded through G-code must interpret the G-code in terms of the particular mechanism. Even a Cartesian machine in which there is a clear X, Y, and Z axis, each with independent actuators, interprets the G-code and adjusts for the scale factors, considers the current kinetic energy and the implicit changes in the kinetic energy, and constructs a move plan to implement the G-code. This involves considering the velocity limits, the acceleration, the jerk, and possible higher derivatives. This plan is passed to the motor drivers, and the mechanism responds.

\n\n

A delta mechanism is really the same. The difference is that there is not a distinct X, Y, and Z axis, even though the commands in the G-code are given in Cartesian coordinates.

\n\n

My second 3D printer is one I designed using standard hot ends and extruders. I'm using the reprap firmware, and haven't adequately studied the kinematics.

\n\n

My first milling machine, however, was a delta machine with 3 additional degrees of freedom -- a machine style generically called a parallel-kinematics inverted Stewart platform. In my kinematics code, I plan a movement by breaking the Cartesian command into small enough segments that the non-linearity of the 6-axis movement space never exceeds the tiny error of the actuators. I developed a CPU-intensive but effective calibration system that estimates the errors that I introduced when building it, and so the mechanical performance is good enough. The machine itself is a 5'x6'x6' frame of welded steel, so it is pretty dimensionally stable.

\n\n

A delta 3D printer is simpler because there is no control over the roll, pitch, and yaw of the hot end. Unfortunately, not being able to control also means that you are subject to whatever errors are introduced in the construction.

\n\n

\"Bed-leveling\" of a delta printer consists of estimating some of the machine-unique parameters and compensating for their effects: cup, bowl, ripple, and tilt. Applying these adjustments is done in the kinematics code as a further modification of the G-code Cartesian parameters to the leg-space delta mechanism motions.

\n\n

TL;DR

\n\n

The G-code is not modified, but the parameters expressed in the G-code are adjusted and interpreted in light of the machine kinematics so that the intention of the G-code can be faithfully followed.

\n" }, { "Id": "5757", "CreationDate": "2018-04-05T17:17:46.723", "Body": "

How do I know what nozzle to get for my Anet A6 printer? I want to get some hardened nozzles because I would love to print with some glow in the dark filament, but I know that eats up brass nozzles fast. But there is so many thread differences so I don't know which one to get, or even what thread the Anet A6 is. Could I have some help finding the thread type and what hardened nozzles would be recommended?

\n", "Title": "Threads for nozzle (Anet A6 extruder)", "Tags": "|nozzle|anet-a6|", "Answer": "

look for MK8 or V6 nozzles that are for 1.75mm filament.

\n\n

but to answer your question the exact threading is M6 x 1mm thread pitch

\n\n

both MK8 and e3d v6 nozzles use m6 x 1mm pitch
\nSource: manual measurement of both my brass mk8 and stainless steel v6 nozzle

\n" }, { "Id": "5760", "CreationDate": "2018-04-06T02:47:24.460", "Body": "

I hear that heated beds can help with removing finished prints, but not all printers have them!

\n\n\n", "Title": "Why is a heated bed important?", "Tags": "|heated-bed|", "Answer": "

There is no simple answer. I will only cover the three main filaments: PLA, ABS, and PETG.

\n\n\n\n

Now, for print removal, a heated bed can help as well:

\n\n\n" }, { "Id": "5762", "CreationDate": "2018-04-06T05:18:29.500", "Body": "

I want to run two stepper motors using Arduino Mega 2560 and RAMPS 1.4. But, somehow I burned the Arduino while I was connecting the motor and plugging the board to the computer.

\n

Can I plug external power supply and USB connection to the computer at the same time?

\n

Additional info from comments

\n

I am using RAMPS 1.4 with Polulo red stepper drivers and Arduino Mega R3. I am only using two Nema 17 stepper motors and not using any hotbed or anything else other than the two stepper motors for the x and y axes. I am feeding 11.9\u00a0V supply to the RAMPS board. When powered on, can I plug the USB to my computer?

\n

Suppose I have connected two motors on the X and Y outputs of the RAMPS, and the external power supply is on, then can I connect my Arduino to my PC using USB? Will it burn my board or not?

\n

My only trouble is that when the board is on load, connecting the USB to computer motors would take power by Arduino pins instead of the RAMPS external supply.

\n", "Title": "Arduino and Ramps powering up", "Tags": "|diy-3d-printer|ramps-1.4|power-supply|", "Answer": "

I had a similar issue building my Prusa i3 Mk2s clone. I was constantly hooking & unhooking my Arduino from my PC while the printer was plugged into the mains. At some point it had enough & it released some "magic smoke". Later on I found out this was the power regulator on the Arduino.

\n

You shouldn't need to plug in the USB and the external power to the Arduino at the same time, but if you did, it is smart enough to only use one.

\n

That being said, by default the RAMPS passes 12V straight to the Arduino in order to power it (the Vin pin).

\n" }, { "Id": "5765", "CreationDate": "2018-04-06T11:45:29.320", "Body": "

I'm building a 40x40x40cm corexy and I am quite impatient so I want the heated to reach the target temperature as fast as possible, so I ordered a Keenovo silicone heater\nIt is a 220VAC 1200Watt bed, so I really want to make sure that it is safe to use.\nI also bought a Crydom D2450 SSR.

\n\n

Could someone tell me if the wiring in the diagram I made below is safe?

\n\n

Do I need to put a fuse or some other kind of safety?

\n\n

\"wiring\"

\n", "Title": "Mains powered heatbed safety", "Tags": "|heated-bed|fdm|safety|wiring|", "Answer": "

The diagram you show is in compliance with the manufacturers' specifications so far as connections go. Make sure any wire you supply is rated for the voltage and current intended for use.
\nIf there is not a safety cover (typically clear plastic) over the junctions where you'll be attaching wires, you should add a cover, or alternatively put the entire relay inside a UL-rated box with stress-release feedthrus for all the wires.
\nIt never hurts to add a fuse or breaker in the source hot feed. I'd recommend slow-blow. NEVER fuse the return side, since a fault or blown fuse here will leave everything live and floating

\n\n

That said, this is more of an electronics question than a 3D printer question, so you might want to wander over to electronics.stackexchange for information on general design safety for high-current systems like this.

\n" }, { "Id": "5768", "CreationDate": "2018-04-06T14:59:52.603", "Body": "

With hot plastic being laid down layer after layer, I am worried about fumes. Should I only print in a well ventilated work space? Should I add additional ventilation?

\n", "Title": "Should I print in a well ventilated area?", "Tags": "|ventilation|", "Answer": "

The short answer is: yes, it is always a good idea to print in a well-ventilated area. The longer answer can be articulated as follows:

\n

Definition of "fumes"

\n

"Fumes" is a fuzzy word that from a chemical/physical perspective includes at least three different things:

\n\n

Interactions with the human body

\n

Each of the above has a different way of interacting with the human body. The list of possible interactions is huge, and out-of-scope for this answer, but just to mention a few obvious ones:

\n\n

Composition of filaments

\n

Modern filaments are a combination of different substances: the basic plastic (PLA, ABS, PETG...) that gives the name to the filament is almost always mixed with other plastics and additives that change its physical characteristics.

\n

In some cases, the filament is host to particles of other materials (like wood, metals or phosphorescent compounds).

\n

Each of the different materials have different transition and critical and flash points (the temperatures at which they will become vapour and ignite respectively), and different physical properties which in turn will affect differently the size of the particles in the aerosol coming out of the printer.

\n

Conclusion

\n

The bottom-line is that it is close to impossible to have a complete understanding of how a given "fume" affects human health.

\n

Typically the safety of a substance is tested in a lab by directly observing its effect on cells, or by performing epidemiological studies in a population, if the exposure data to a given substance is known.

\n

When people comment on PLA being "safe" for example, they typically refer to studies that tested inert, cold, chemically pure PLA. But the fumes of a PLA filament will probably not be inert, nor cold, nor be exclusively PLA.

\n

Additionally, it has to be observed that it is much easier to rule a filament harmful than safe: for it to be considered harmful it is sufficient to know that one of its components is harmful (for ABS that is typically studies showing the adverse affect of ABS aerosols on health). For it to be deemed safe, one must know that all if its components are safe, but most filament do not go through the rigorous testing required to ascertain that.

\n

In conclusion, it is always a good idea to get rid of the fumes from 3D printing regardless of the type filament being used. The ideal solution is a printing enclosure maintaining negative pressure, but an enclosure with air filtering or a well ventilated room are also good options (ventilation can have adverse effects on printing quality though, due to drafts and their cooling effect).

\n" }, { "Id": "5775", "CreationDate": "2018-04-07T17:56:22.463", "Body": "

I am currently printing the QTrainer aircraft from 3DLabPrint on my Anet A8, via the supplied i3-compatible gcode files.

\n\n

So far, everything has printed fine, but the main undercarriage part tries to print off the left hand side of the bed by about 5mm.

\n\n

I have tried manually adding an M206 X-10 line to the gcode file, just after G28 ; home all, however this has not had any effect:

\n\n

G90\nM83\nM106 S0\nM140 S65\nM190 S65\nM104 S220 T0\nM109 S220 T0\nG28 ; home all\nM206 X-10 ; added by me\nG1 E-0.8000 F1800\nG1 Z0.200 F1002\n; layer 1, Z = 0.2\nT0\n; tool H0.200 W0.450\n; skirt\nG1 X9.382 Y10.533 F6000\nG1 E0.8000 F540\nG1 X10.536 Y10.537 E0.0388 F1440\n...\n

\n\n

However this does not work and I'm not clear on whether I am using this gcode command correctly, or whether my printer simply doesn't support that command.

\n\n

Is there another way I can apply an X-offset to the gcode file (or printer) to move it a few cm to the right of the print area?

\n\n

As I do not own Simplify3D and cannot currently afford it, re-slicing the STL files is unfortunately not an option.

\n\n

Unfortunately there is no X-adjustment on the X limit switch (although I suppose I could temporarily tape something to the print head, so that it activates the switch sooner?).

\n\n

My Anet A8 currently runs stock firmware, hooked up to OctoPrint and I have access to a Macintosh desktop computer.

\n", "Title": "Apply X-offset to gcode (re-slicing not possible)", "Tags": "|g-code|anet-a8|", "Answer": "

You can use G92 to change the coordinates the machine \"thinks\" it's at. If just after homing, you apply

\n\n
G92 X-10\n
\n\n

the machine now thinks it's at X = -10, while in reality it's at X = 0. This results in everything thereafter behaving as if it is shifted in the positive X direction by 10 mm (since if you then commanded a move to X = 1 the machine would move to what is physically X = 11).

\n" }, { "Id": "5778", "CreationDate": "2018-04-07T22:06:33.297", "Body": "

Recently I noticed that Cura always is giving me less printing time than the print itself takes on my TEVO Tarantula with Marlin firmware.

\n\n

The time difference is about 15 %, at requested 50 mm/s printing speed.

\n\n

How could I verify real printing speed?

\n", "Title": "How to determine real printing speed (TEVO Tarantula/Cura)?", "Tags": "|ultimaker-cura|slicing|tevo-tarantula|", "Answer": "

I use a Anet A8 which has a system acceleration and jerk of 400 and 20 \nWhen i slice with Cura, then I use a gcode without acceleration control, because the printer does this by its own and a combination with the gcode acceleration/jerk control gets me some strange behavior (e.g. sometimes some ultra slow movement in fine detail sections, or random nearly stopped movement) \nBut to get a realistic approximation for the print time, i enable the acceleration/jerk control in cura and then I get a really good result, that is really close to the real values.

\n\n

And to be more specific to your question: It is e.g. 50mm/s >>maximum<< speed you can set, not a exactly and absolut speed.\nIf you know your printer you can also experiment with the following approach: Set the max speed really high, so that the max material volume per time is the limiting factor and set the acceleration and jerk as low as needed for good results. So that your printer will accelerate on long lines as long as it can up to the point it has to deaccelerate for the next corner.

\n" }, { "Id": "5782", "CreationDate": "2018-04-09T06:13:29.317", "Body": "

I have a RAMPS 1.4 with Arduino Mega 2560 with 2 steppers connected to an external power supply using Polulo stepper drivers. The steppers are for X and Y axis movement. This external power supply is set to 12 volts and power is applied. I have configured the marlin firmware also.

\n\n

Can I connect my setup to my PC using USB when the external power is also on?\nIf i connect the USB while the external power is off, do the motors take power from USB? Will this will burn my board?

\n", "Title": "Can I use external stepper motor power and USB connection", "Tags": "|ramps-1.4|stepper-driver|usb|", "Answer": "

The motors are powered from 12/24V external power supply, so the motors will not be powered. \nthe ramps Schematics shows the connection diagram and the power is provided via the VMot pin/12V.

\n\n

\"Stepper

\n" }, { "Id": "5795", "CreationDate": "2018-04-11T10:22:31.987", "Body": "

I am planning to buy 4 hotends (e3d clones I assume), but I am wondering if the Bowden (long distance) be able to be used as Wade (direct extruder)?. When the pneumatic fitting is removed there is a very similar filament drive to the throat/heat-brak and the collar size is same as wade.\nAs per attached pictures, I think that could be possible.\n\"front

\n\n

\"hotend\"

\n", "Title": "Can I use Bowden hotend instead of Wade for a direct extrusion?", "Tags": "|extruder|bowden|wade|", "Answer": "

You can use any of them, however the adaptor will be different to make it fit on your printer. I\u00b4m using the J-head on direct drive by removing the pneumatic connector; the distance from top to filament hole is close to 4mm, but does not affect the feeding. If the cylinder hasn't the pneumatic connector is possible to print a cap to mount the connector and make it remote drive

\n" }, { "Id": "5796", "CreationDate": "2018-04-11T11:44:13.350", "Body": "

I have a Cartesian style 3D printer (FLSUN Cube), with two Z-axis stepper motors (1.8\u00b0, 200 steps/revolution) with T8x8 lead screw (8mm / 2mm pitch with 4 starts), and currently they are using 1/16 microstepping.

\n\n

As I know, microstepping is an unstable state and can affect accuracy of the Z-axis. But many of modern 3D printers (like CR-10, Prusa i3 Mk2s and others) use similar microstepping for the Z-axis.

\n\n\n", "Title": "Full steps mode for Z axis stepper controller", "Tags": "|z-axis|stepper|microstepping|", "Answer": "

Focussing on the questions at hand:

\n\n
\n

Is it possible to use full steps?

\n
\n\n

This depends on your printer board. Many boards use dip switches to select the (micro) stepping mode of the stepper driver. E.g. a RUMBA board has dip switches located underneath the stepper driver boards (e.g. DRV8825 or A4988). Sometimes you also see jumper caps. The answer is both yes as is no as it depends on the board you are using.

\n\n
\n

Does it improve accuracy if I use compatible layer height (like 0.04mm,\n 0.08mm, 0.16mm, etc) for my printer with 0.04mm per full step on Z axis?

\n
\n\n

Increasing the number of microsteps results in reduced incremental torque (for full step this is 100%, for 16 micro steps this drops to about 10%; this implies that a micro step requested by the controller may not effectively lead to an actual step as it cannot overcome the torque to turn the shaft). So highly loaded steppers could result in positioning errors. Resolution increases but accuracy will actually suffer. Furthermore, Few, if any, stepper motors have a pure sinusoidal torque vs. shaft position and all have higher order harmonics that in fact distort the curve and affect accuracy. according to this source. On the other hand micro stepping makes rotation go smoother (major advantage, see source). Interesting literature (must read) is this test and this paper.

\n\n

The answer to this question also depends on the situation; when you load the steppers very highly, using micro stepping may result in more inaccurate movement compared to full stepping. When you use the lead screws native resolution for your setup, only when the stepper is actually at the full step position, you would benefit as in this position it will not dwell to the next full step as it is already in the stable position.

\n\n
\n\n

As a side remark I've added the correct calculation of the native resolution of your lead screws. From your question I deduce that you have Tr8x8(p2) lead screws. \"Tr\" for trapezoidal thread, followed by the nominal diameter in mm. The digit after the \"x\" tells you how much the nut advances per revolution, this is called the lead of the screw. The value between the brackets \"p2\" denotes the pitch. This means that the screw has 8 (lead)/2 (pitch) = 4 starts. So with every revolution of the stepper (200 steps) the nut advances 8 mm which translates to 8/200 = 0.04 mm per step of 1.8\u00b0.

\n" }, { "Id": "5802", "CreationDate": "2018-04-12T17:56:16.760", "Body": "

I noticed a knocking noise one day on my QIDI and things seemed fine at first glance, then 7 hours later my print only had 3 layers. So I figured it was a jam, and I cleaned it out but every now and then I heard the knock again which cause incomplete prints. When I went back to remove the front facing cooling fan I noticed one of the screws that go through the fan to the stepper motor wasn't secure. This was causing the filament guide to rock out of place periodically until it got worse and worse.

\n\n

I've been trying for hours but the screw just won't \"catch\" on the stepper motor that feeds the filament into the extruder. The screws aren't stripped because I can take the motor off and screw them directly into it with no problem. It's as if they aren't long enough anymore or that they're bent.

\n\n

Has anyone else had this problem & found a solution that works consistently?

\n", "Title": "QIDI x-one2 : Fan screws not holding filament feeder stepper motor in place", "Tags": "|extruder|motor|", "Answer": "

I just had this same problem and my solution was to push the motor as close as I could up to where the screw goes, because they weren\u2019t lined up. I hope that helped.

\n" }, { "Id": "5804", "CreationDate": "2018-04-12T21:40:23.060", "Body": "

I try to configure my 3D Printer properly.

\n\n

I printed this boat model https://www.thingiverse.com/thing:2763854 (with a scaling of 0.6):The quality of the print looks nice but at the last roof the entire print slipped away. (e.g. no adhesion between bed and print)

\n\n

Here are some printing configuration

\n\n\n\n

\"enter\n\"enter\n\"enter

\n\n

Is this common that the print can slip away?\nIs e.g. the raft bed adhesion a better option?\nI have read that glue could prevent the print to slip away. What kind of glue I should use?\nCould a hotter/colder heating bed prevent the print to slip away?

\n\n

Thanks in advance

\n\n

Edit:\nAdded Pictures after extruder calibration (https://www.youtube.com/watch?v=fb7lit5ZRq4):\n\"enter\n\"enter

\n", "Title": "Print slipped away", "Tags": "|prusa-i3|heated-bed|adhesion|", "Answer": "

If your printer is a genuine Prusa i3, MK2 or MK2.5, it will have a PEI bed which requires no additional adhesive for a model to remain properly attached during the print.

\n\n

The PEI should be cleaned with denatured alcohol prior to a print. If your bed has had many prints in one location, the PEI will lose adhesion. I've recently assisted a Prusa owner to change the PEI sheet which improved print adhesion greatly. The strange aspect of that is the problem was getting the print to release, not that it would not remain attached. The user now knows to move the print location around the bed and not always in the center.

\n\n

In your case, all the parameters look good, although 60mm per second is rather fast, that should not have a big effect on adhesion.

\n\n

Prusa printers have the ability to perform a live-z adjustment. The first layer being printed should have a slightly squished or squashed appearance. When you begin the print, use a skirt or brim to force the printer to outline the model. During that period, use the live-z adjustment to flatten the first layer, but use caution to not drive the nozzle into the print surface.

\n\n

For testing purposes, consider to create a model of only 1 mm thick using the same settings. This will give you faster results on testing the live-z and getting a good bond to the bed.

\n\n

I use 70\u00b0C for printing PLA, but the measured temperature on my glass bed is 60\u00b0C which leads me to believe that you have a good number. The printing temperature is reasonable too.

\n\n

10mm brim is wide and further indicates that your adhesion may be related to first-layer nozzle height.

\n\n

Post-photo edit: I can see in the image provided that the first layer has large air gaps. This is directly related to either the extrusion factor or the z-height. The remainder of the model looks too good to be extrusion factor, returning my first assessment of z-height problems as more likely.

\n\n

Yet another edit:

\n\n

As best as I can determine from your printer name, you do not have a genuine Prusa i3, but rather a Geeetech prusa clone i3. As such, many things are different. My suggestion about z-height is no longer valid, as the firmware is likely to be different for your printer.

\n\n

I'm not a cura user, but my quick research shows that it's probably called flow percentage. Other posts on the 'net suggest that below 100% would be underextrusion. I've found a video that shows how to make your extruder match the 100% flow setting in the software. It's smarter to have the firmware represent accurate figures and the video will allow you to perform this calibration.

\n\n

The z-height for your printer is related to print bed calibration. I was unable to locate anything useful to me with respect to that aspect of your printer. It appears that the bed has four leveling screws, although I'm not certain of that. If your bed leveling involves adjusting those screws, it is possible that all four (or three?) of them are too tight and have to be released a bit.

\n\n

If you have bed calibration instructions, consider to perform them again, especially if you are going to adjust the extruder settings as per the previous video.

\n" }, { "Id": "5817", "CreationDate": "2018-04-15T09:20:36.067", "Body": "

My model looks like this, does anyone know what's going wrong?

\n

\"Bad

\n

I'm thinking that if the small object looks like that, it's probably because of the slicer? And how about the bottom part of the cube... ummmm... I really don't know

\n

I'm using an M3D printer, its own slicer and PLA at 205\u00b0C

\n

Settings:

\n\n

Update:

\n

The smaller object is using the same file but resized.

\n

\"Side\"

\n

\"Top\"

\n

\"Inside

\n

\"Side

\n

\"20x20x20

\n

Here's the calibration cube. After I've changed the filament flow to 98%, it seems that the situation became better. But, it might also be the problem of the slicer as the two object isn't the same.

\n

\"Calibration

\n

Update :

\n

Here are the settings

\n

\"setting01\"\n\"setting02\"\n\"setting03\"\n\"setting04\"\n\"setting06\"\n\"setting05\"

\n

STL Files

\n\n", "Title": "Print quality problem with M3D FDM printer", "Tags": "|print-quality|3d-models|fdm|m3d|", "Answer": "

It appears that there are several issues with the prints.

\n\n\n\n

To determine to improve the print quality you first should look at the bed leveling issue, second calibrate the extruder (see if you ask it to spit out 100 mm of filament it actually is 100 mm rather than randomly lowering the extrusion flow parameter) before you print some calibration cubes at reduced temperature and or increased part cooling flow.

\n" }, { "Id": "5821", "CreationDate": "2018-04-17T03:03:47.340", "Body": "

I am trying to 3D print some shells for a project, and would like them to be about 5 cm long.

\n\n

I created the shells in Blender and converted them to .stl files which the printer requires, but it tries to print them as 0.02 mm long. I am not printing them myself, so I have to rescale the files without access to the actual printer. I have tried scaling the object both in Blender, and when exporting them to .stl files, but can't figure out how to tell what size the printer will interpret the shells as.

\n\n

Is there a way to make sure I have the objects scaled properly before printing them?

\n", "Title": "Printing .stl file using ObJetPro 3D Printer", "Tags": "|software|scaling|", "Answer": "

the best way is to install slicer on your pc and open the .stl file.\nMost slicers will tell you the size of an object and there is an option to rescale resize.

\n\n

Using Slic3r you will have object details in the corner\n\"enter

\n\n

Using CURA you will have a detailed view in scale mode \n\"enter

\n\n

Also you can use online services CLICK\n\"an

\n" }, { "Id": "5822", "CreationDate": "2018-04-17T03:29:28.043", "Body": "

Are these vertical lines described as \"banding\"?

\n\n

Would the most likely culprit be the extruder?

\n\n

FWIW, this was printed in \"vase mode\".

\n\n

\"Vase

\n", "Title": "Is this banding?", "Tags": "|troubleshooting|", "Answer": "

Ok, so it turns out that this was caused by a problem in my stepper motor drivers, which results in missed steps, as described on this guy's blog

\n\n

The fix was to hook up these \"TL Smoother\" modules (I got these particular ones from amazon but there are a bazillion clones)

\n\n

After hooking up TL Smoother, this is what the print looks like:

\n\n

\"enter

\n" }, { "Id": "5829", "CreationDate": "2018-04-18T13:34:18.830", "Body": "

When I wanted to take out the filament from the extruder, it didn't wanna come out, it was stuck (even when I heated up the nozzle). So I unscrewed the nozzle and it looks like the filament is all pressed together at the top of the nozzle (see picture). What does this mean? I am heaving issues extruding filament from the nozzle, the filament barely comes out and the extruder motor is clicking. Can someone help, please.\"enter

\n", "Title": "Why is my filament pressed together at the nozzle", "Tags": "|filament|nozzle|", "Answer": "

As per attached picture I can see that the issue source could be:

\n\n
    \n
  1. the ptf tube is not inserted to the end of heat-break, or it is not straight-cut at the end - see this video for help

  2. \n
  3. the cooler on the hotend is not working properly/not installed and heat goes up to the throttle and melts the material

  4. \n
  5. an object in the nozzle that blocks the flow (usually a ptf tube particle that probably melted) - clean/replace the nozzle

  6. \n
\n" }, { "Id": "5848", "CreationDate": "2018-04-20T08:25:09.537", "Body": "

When trying to upload firmware I get the following errors:

\n\n
    \n
  1. timeout - cannot sync, or
  2. \n
  3. port is in use
  4. \n
\n\n

What can be the possible sources of these errors?

\n", "Title": "How to upload firmware to reprap printer?", "Tags": "|firmware|", "Answer": "

There are mainly three reasons for that:

\n\n
    \n
  1. Arduino studio settings should be: \n\n
  2. \n
\n\n

\"Arduino

\n\n
    \n
  1. Please close all slicer's instances (Cura, Slic3r, Repetiter) and host servers and other software that communicate with the printer as they lock the COM port;

  2. \n
  3. Please check that the appropriate usb2serial drivers are installed and working - the best way is to start the serial monitor from the Arduino Studio Tools menu and see if there are any.

  4. \n
\n" }, { "Id": "5850", "CreationDate": "2018-04-20T09:18:40.610", "Body": "

I have a 15x15 cm heating resistor from my current printer (printing area: 12x12 cm).

\n\n

I would like to switch to a glass bed and to rework my printer to increase the printing area to 20 cm (22x22 cm glass plate).

\n\n

Would it be possible to use the old heating resistor placed only in the centre? this way I would have a smaller heated bed for ABS and a bigger one for PLA.

\n\n

Would the glass crack due to non uniform heating? This is because glass has a conductivity of less than 5 W/mK, therefore the hot area will stay hot and basically never really spread the heat to the surrounding area. So the frame will be cold and the center hot, causing stresses.

\n\n

Related: https://engineering.stackexchange.com/questions/31842/how-much-an-unevenly-heated-glass-plate-bows

\n", "Title": "Can I have a glass bed with only the center heated?", "Tags": "|heated-bed|glass-bed|", "Answer": "

I purchased some very, very cheap thin glass from walmart (or home depot, cant remember for sure). Probably the thinnest kind available. Went through dozens of prints heating it up to 85C only at the center and never experienced issues with cracking. Personally I believe you\u2019ll be fine. I can look for specifics if you need them.

\n" }, { "Id": "5856", "CreationDate": "2018-04-21T12:31:20.670", "Body": "

I recently bought the MGN12H linear guide and rail from robotdigg, which is supposed to be the copy of the Hiwin.\nI have noticed that the motion is very smooth when moving fast, but if I move it very slowly, it gets stuck and the motion is not smooth at all. I think this will cause problems on my prints once I assamble the printer.\nAm I missing something? Should I put some king of oil on the balls?

\n", "Title": "Robotdigg linear guide not smooth motion", "Tags": "|diy-3d-printer|linear-motion|", "Answer": "

Difficult to say with the given data, but here are a couple of ideas:

\n

First theory:

\n
\n

The belt is too thin/flexible for the linear guide.

\n
\n

Linear guides remove the backslash by adding quite some pressure in the ball system: The block is slightly too small (compared to the rail) and thus it compress the balls and rail. The backslash is avoided until you reach this force.

\n

This pressure require a little bit of force to move the block along the rail, so if your belt is too thin or flexible, the belt will start to move until the elasticity of the belt reach the force required to move the block, then the block move a little bit and stop again.

\n

In a fast movement, your block never get to stop, while in a slow movement, it has time to start and stop. Maybe what you feel like "not a smooth movement"

\n

If you identify this case, I suggest you the following solutions:

\n\n

Second theory: (Thanks to FernandoBaltazar)

\n
\n

You got some dust, rust inside

\n
\n

Remove the belt and move the block with the hand, if you are able to feel the movement is not smooth, then it's probably this case.

\n\n" }, { "Id": "5857", "CreationDate": "2018-04-21T18:01:35.723", "Body": "

How does Z offset (M851) work with an auto leveling sensor? Does it add the Z offset to the offset of the G29 mesh? or the G29 value replaces the M851?

\n

My printer is an Anet A8 with Marlin firmware, I was having issues with the autoleveling sensor and reset the Z offset to 0 and let G29 get the mesh offsets and its working good now.

\n

I was looking through Marlin G-code page but couldn't figure out how G29 affects M851 or vice-versa.

\n

My setup with level issues:

\n
M851 Z0\nG28 \nM211 S0 ;turned endstops off and got a paper to find the zoffset\nM851 Z-0.59\nM500\nM211 S1\n
\n

And G29 before printing.

\n", "Title": "Z Offset on autoleveling sensor setup", "Tags": "|bed-leveling|", "Answer": "

You must use the sequence

\n\n
G28\nG29\n
\n\n

If you do G28 after G29 it will reset bed leveling. I guess you don't want that.

\n\n

On my printer deployed z-probe falls 2.3 mm lower than nozzle. In printer settings I've therefore stored static Z-offset of -2.3 mm, so after G28 and G29 I can be sure that when ever I tell Z-axis to lower to zero, it can go down -12.3 mm, coming to stop at around 0.1 mm above the bed surface (so just a single sheet of paper fits between the nozzle and the bed).

\n\n

That said, I'm not worrying about sending the Z-offset via print commands, as restart restores the aforementioned -2.3 mm offset.

\n" }, { "Id": "5859", "CreationDate": "2018-04-22T18:45:17.527", "Body": "

The light of the build area of a DLP UV printer isn't evenly spread. The source of light is a 9 LED UV array. The light mainly falls of to the sides. This results in objects curing too much in the center, or not enough on the outer sides of the build area. One can compensate for this, making the LCD build area have a homogeneous intensity of light, using a mask. But it needs to be specifically created for each physically unique printer. Having a homogeneous illuminated build area results in better quality prints.

\n\n

How can I create such an mask accurately for a UV 3D resin printer that uses DLP technique, like for example for a Wanhao Duplicator D7?

\n", "Title": "How to create an accurate mask for a UV 3D resin DLP printer", "Tags": "|print-quality|dlp|uv-printer|", "Answer": "

I've created software to do just that. See the project on Github: CreateMask. It has a wiki page that explains what to do.

\n\n

To summarize: you measure the LCD build area using a multi-meter and a light dependent resistor. You do this with low and high intensity masks. You feed the numbers in CSV files to the software, and the software will generate a mask for you by polynomial curve fitting. See this page that explains in detail how the mask is created, if you are interested.

\n\n

You can download a release here.

\n" }, { "Id": "5863", "CreationDate": "2018-04-23T18:33:31.267", "Body": "

I have the Monoprice Select 3D Printer (i3 clone). I'm trying to print ABS. I set the temperature to 230\u00b0C for the extruder and 110\u00b0C for the bed. I can see the printer warming up to those temperatures and then when it gets there the bed and extruder set temperatures reset to 0, and the print does not start.

\n\n

I pulled the ABS out and went back to PLA and printed a part without any issues.

\n\n

Any thoughts on what the issue might be?

\n", "Title": "Monoprice i3 clone heat shuts off printing abs", "Tags": "|extruder|heated-bed|abs|", "Answer": "

this could be connected to a thermal shutdown protection.\nMost firmwares requires a decent increase in temperature every few seconds.

\n\n

if you use Marlin then in the source code you can extend thermal shutdown timings.

\n\n

what to check:\ncheck every single connector to eliminate any fiddling or play - risk of fire !!!

\n\n

if any of wires goes hot - double check connections, replace wires/connectors

\n\n
    \n
  1. note temperature change (how many seconds/degree) on the hotbead
  2. \n
  3. then increase voltage on psu (probably you have a led psu - so there is a small regulator next to the connectors)
  4. \n
  5. validate if that help
  6. \n
\n" }, { "Id": "5865", "CreationDate": "2018-04-24T02:51:26.850", "Body": "

On nearly all of my prints, my first few layers are flared out slightly. I'm using painter's tape for the bed, and the bed temperature is at 60 °C while the extruder is at 205 °C. This seems to happen regardless of print quality settings.

\n\n

Here's one example below:

\n\n

\"enter

\n\n

That extruded rectangle shape is 19.50 x 19.30 mm on the outside on the straight part. On the flared part, it measures 19.92 x 19.70 mm.

\n\n

Is there a way to fix this? If so, how can I improve it?

\n", "Title": "How can I stop the flare out of the bottom layers?", "Tags": "|prusa-i3|pla|print-quality|anet-a8|simplify3d|", "Answer": "

The picture is not very clear so this can be related to either:

\n\n\n\n

The general remedy for these problems is to level the bed (e.g. nozzle can be too close to the bed). \"Elephant foot\" can be reduced by reducing the heat bed temperature, re-positioning the cooling nozzle and/or start cooling at a lower height. Please experiment with the print parameters by printing XYZ 20 mm test cubes till you find the correct settings.

\n" }, { "Id": "5866", "CreationDate": "2018-04-24T05:03:01.530", "Body": "

I just bought a FLSUN QQ and the instructions say I need to change the voltage from 220 to 110. Sadly the instructions don't say how to do that. Can anyone tell me how to do it?

\n", "Title": "How to change the voltage on a FLSUN QQ", "Tags": "|delta|flsun-qq|power-supply|", "Answer": "

I don't know if this is the case with all FLSUN QQ printers, but mine was indeed set to 220v as the instructions/user manual indicated. First I needed to remove the bottom of the printer. Then there was a switch on the side of the power supply with the following label:

\n\n

\"voltage

\n\n

I know the picture is terrible, but the lighting inside the base of the printer was less than optimal. It should be pretty easy to find once you know what you're looking for.

\n" }, { "Id": "5868", "CreationDate": "2018-04-24T16:40:29.547", "Body": "

Not asking for a product recommendation, unless there's only one product out there like this...

\n\n

I have a need for a flexible filament that retains its deformation, kind of like silly putty. Are there any filaments out there that, after you bend or stretch them, they keep their shape?

\n\n

I'm sure to some degree flexible filaments do this -- Which filament might be best? Commenters -- what's the scientific term for this?

\n\n

Actually, metals exhibit this type of flexibility. Do any of the 3D printable metal filaments flex, bend, and keep their shape?

\n", "Title": "Any family of plastics / filaments that bend and keep their shape?", "Tags": "|fdm|print-material|lulzbot|", "Answer": "

To have a material be \"flexible\" and retain the shape after the \"flexing\", you need to apply a stress in excess of the elastic deformation stress. You will want a material that has the elastic deformation limit that is much lower than the ultimate tensile strength. Unfortunately, for plastics, this is difficult to find.

\n\n

Most plastics are made up of tightly bound, long-chain molecules. Flexing beyond the elastic limit requires breaking these bonds, which introduces weak points in the plastic. You can see this by bending almost any plastic. Either it will snap in two, or you will see a light or white colored line along the fold. The lighter color comes from light scattering from the broken bonds.

\n\n

\"Flexible\" structural plastics such as ABS are a copolymer (more than one type of molecule) in which one of the polymers is rubbery. The rubbery bits provide places within the bulk material where the stress can create strain that doesn't require breaking bonds.

\n\n

I doubt that any 3D-printer FDM compatible filaments will satisfy your needs. A filament must keep its shape during printing, if for no other reason than to permit the extruder drive mechanism to apply pressure to the filament. There are 3D-printable filaments, but they are very elastic and return to their shape when the stress is released.

\n\n

Some metals can respond to stress with by flowing rather than by elastic strain. Lead comes to mind. Someone suggested copper. There are several degrees of hardness of copper depending on the alloy and the annealing schedule after it was formed. Copper also work-hardens, becoming harder and more likely break under stress the more the copper flows under stress.

\n" }, { "Id": "5869", "CreationDate": "2018-04-24T18:25:08.703", "Body": "

So I bought a Lulzbot Mini a couple months ago and finally downloaded Ultimaker's version of Cura... Boy... have I been missing out...

\n\n

One feature Ultimaker Cura implemented that I've been looking for is a \"pause at z-height\" feature (\"post-processing tool\"). I'm building prototypes of an electronics device, and creating two pieces that snap together looks a lot worse than printing a single piece. If I could pause my print, insert my electronics, and continue printing, my device would look a lot more professional (even if it took longer to make).

\n\n

My one concern is the lithium ion battery. Right now I'm printing in TPU. With a heated bed of 40 degrees Celsius, and a heated extruder at 240 degrees Celsius, there seems to be a significant risk that the lithium ion battery reaches a temperature above 60 degrees Celsius (damaging the cell, causing a potential explosion). Granted, I am not sure what \"60 degrees Celsius\" actually means. It could mean only one part of the packaging needs to reach this temperature, or it could mean the entire LiPo's internal temperature would need to reach this. In either case, the numbers don't look good.

\n\n

On the other hand, the heated bed surely doesn't need to remain heated beyond the first few layers? Additionally, I can create a \"roof\" for the LiIon battery that I can slip it under, providing some insulative TPU before the rest of the device. I think the print would happen safely like this, but obviously, an explosion would be really really bad. Like it would probably burn my house down, and I would be asleep when it happened.

\n\n

Does anyone have any experience doing this?\nIs there a way to turn off the heated bed mid-print? I guess I can insert a g-code line during the pause? Will this affect the remainder of the print you think?\nAm I being paranoid? Can the extruder actually pass heat through a 1\u20132\u00a0mm of insulation and cause an explosion?\nAnyone know how heat travels from the initial, liquid print material through the rest of the structure?

\n\n

Any more advice or things I should consider before pursuing this?

\n\n

A more specific pause type might be helpful, if anyone knows of any.

\n", "Title": "Danger in 3D printing over a lithium ion battery", "Tags": "|ultimaker-cura|lulzbot|", "Answer": "

Although the bed may only reach 60\u00b0, the extrusion above the battery will be at closer to 200\u00b0C. Granted there will be a fairly low heat flux, but it will still cause ageing of the battery. At a minimum, some insulation or packing above the battery would seem like a sensible move. This could be printed material, and you could print over the top if you really want to seal the part.

\n\n

Also remember that as LiPo cells age, they generate gas (this is designed to be trapped in the pouch) and swell. You should try and avoid any risk of puncture if this happens.

\n" }, { "Id": "5878", "CreationDate": "2018-04-26T00:15:37.847", "Body": "

I made my printer to have a left handed coordinate system (it homes to the back left corner of the heatbed), I did this because it happens to make working on the printer easier for me. This, however, causes the objects to be printed mirrored. In Slic3r I manually mirror objects every time I load a new objects. From time to time I forget doing this which is really annoying. I was wondering if there is a setting in Slic3r to automatically do this. Something like mirroring on import, or changing the axis in Slic3r itself.

\n", "Title": "Change Slic3r settings for left handed coordinate system", "Tags": "|slic3r|axis|", "Answer": "

The direction of the end stop is set in the firmware of the printer. Even with different setup end stops, you should be able to get a correct coordinate system without mirroring axes in slicers. This would be the preferred method to fix your problem!

\n\n

E.g. my Ultimaker 3 Extended homes the Z on Z max having the platform at the bottom of the machine, a calibrated offset determines the actual Z=0.

\n\n

Not knowing which firmware you are using, in e.g. Marlin Firmware this is set by code lines in the file Configuration.h:

\n\n
// Direction of endstops when homing; 1=MAX, -1=MIN\n// :[-1,1]\n#define X_HOME_DIR -1\n#define Y_HOME_DIR -1\n#define Z_HOME_DIR -1\n
\n\n

Your end stop triggers at the maximum of the Y axis, so you need to configure it as a MAX end stop, i.e. use the Y_MAX pins by defining (search for the Endstop Settings section, note to also disable the YMIN endstop):

\n\n
//#define USE_YMIN_PLUG // This disables the YMIN endstop\n#define USE_YMAX_PLUG   // This enables the YMAX endstop\n
\n\n

and change the homing direction (Y_HOME_DIR) to 1:

\n\n
#define Y_HOME_DIR 1 // This tells the printer where the endstop is located: positive for YMAX direction\n
\n\n

Otherwise when used at Y_MIN endstop and the homing direction set to -1, the axis is reversed as you experienced.

\n" }, { "Id": "5892", "CreationDate": "2018-04-28T17:46:41.677", "Body": "

If I have a stl file is there a good way to estimate how long it will take to print on a given printer? I know there are a lot of things that go into print speed, such as the speed of the printer, the size of the print, etc. I was wondering if anyone knows of perhaps an application that you could enter your printer's specifications and the stl that you want to print, and it could calculate the print time?

\n", "Title": "How long will my print take?", "Tags": "|software|speed|", "Answer": "

I use Repetier software with the Cura slicer and the print time estimate tends to be about 10 % longer than the actual print.

\n" }, { "Id": "5905", "CreationDate": "2018-05-02T23:36:12.267", "Body": "

Currently I am using a 12 volts, 20 amperes power supply (Model S-240-12)

\n\n

\"enter

\n\n

The stepper motors and the extruder need 5 amp, and the heated bed build plate needs 11 amp.

\n\n

\"enter

\n\n

Technically you only need to use a 12 Volts, 16 amperes power supply, but I understand that you use the one with 20 amp because pulse currents from extruders and stepper motors can be stressful to supplies loaded to the max, so for reliability and performance, it is better to use a supply rated for 25% more than you need

\n\n

In the place where I buy the spare parts for my 3d printer they also sell 12 V power supplies capable of delivering 25 amp and 30 amp and they told me that if I use those ones you are going to be able to speed up the heating of the heated bed. Is that true? I understand that the heated bed is only going to take the 11 amp that it needs so is not going to make any difference to use power supplies capable of delivering more current

\n", "Title": "Speeding up the heating of the heated bed", "Tags": "|heated-bed|ramps-1.4|switching-power-supply|", "Answer": "

A more powerful PSU only would solve the problem in two cases: Either your PSU is anemic and underpowered in the first place, or you want you'd separate the bed's power supply from the rest of the machine - by using a higher Voltage for the bed. This would however need you to regulate the heating by having the board control not the bed directly but, control a (Normally Open for safety!) MOSFET, which in turn throttles the power to the bed.

\n

In that case, you can use the resistance R of the bed with whatever voltage your alternate PSU provides to get the power that is turned into heat from the bed using $P_\\text{bed}=\\frac{U_\\text{bed}^2}{R_\\text{bed}}$. Our MOSFET can regulate the power that is turned into heat in the bed as it is a Variable Resistance: The total potential differential stays the same, but the voltage available to the heated bed is governed by the resistance of the bed and the MOSFET's resistance. Since the two are in line, they have the same Current flowing through them:

\n

$$U_\\text{supply}=U_\\text{bed}+U_\\text{MOSFET}=I_\\text{total}\\times(R_\\text{bed}+R_\\text{MOSFET})$$

\n

That results in what is commonly called a Voltage Divider: the voltage that is available for the bed comes from a derivate of that: $$U_\\text{bed}=U_\\text{supply}(\\frac{R_\\text{bed}}{R_\\text{MOSFET}+R_\\text{bed}})$$

\n

Why the hazzle?

\n

Often, a board also might have a potentiometer for each power exit, and these are generally nothing else but variable resistances - and give us the same effect as a MOSFET for controlling the voltage available to a bed. If available, turning the Bed-Potentiometer a tiny bit will provide just a little higher voltage to the bed and allow slightly faster heating.

\n" }, { "Id": "5913", "CreationDate": "2018-05-04T09:36:39.083", "Body": "

While this may seem like terribly noobish question, I'm sure one day someone will have same problem.

\n\n

I own a ZONESTAR P802QSU (Bowden extruder) and all of sudden my extruder motor stopped working. What I did:

\n\n
    \n
  1. I check whether or not motor is moving freely - I can rotate it without any problems manually.
  2. \n
  3. I checked out cable and plugs - everything seems OK.
  4. \n
  5. I checked whether motor is working after plugging it into another slot in motherboard - motor works fine.
  6. \n
  7. I checked if another motor works after plugging it into same slot on mainboard - and it does not.
  8. \n
\n\n

It would seem that there is something wrong with E0-mot driver module, or with socket. I'm however totally at loss about what can I do with any of those. I do not have any electronic equipment except multimeter, and voltage on motor seems fine(11.3V between red and black, I guess V+ and ground). It sometimes seemed to get lower, but I'm almost sure it is my trembling hands.

\n\n

After some googling, I started checking A4988 stepper driver with my multimeter. VDD was ~5V and VMOT around 12V, so it seems to work at least in this way.

\n\n

What can I do, to see what is broken? Visually nothing looks like it burned out, but I'm fully aware it may not be visible.

\n", "Title": "Zonestar Extruder motor does not work - what can I check?", "Tags": "|extruder|", "Answer": "

As this is not an obvious case, most printers' firmware has a cold extrusion prevention. That is usually set at 170C, so the nozzle need to be over 170 to allow stepper to move.\nI experienced that when I was assembling my TT and doing a cold run, to validate all mechanical movements and clearance.

\n\n

In Marlin firmware, the extruder stepper is switched off after extrusion, so we can freely move it, but not other motors and that could give us a thought that something is bad with the driver.

\n" }, { "Id": "5921", "CreationDate": "2018-05-06T14:30:03.967", "Body": "

My Printrbot simple metal's extruder is jammed and I need to heat it up to unjam it. Unfortunately, the printer does't want to connect to my laptop regardless of the program I'm using (Repetier-Host or Cura 15).

\n\n

Is there a way to use a micro SD card to heat up the printer hotend but not print anything?

\n", "Title": "How to heat up a Printrbot's hotend using an SD card", "Tags": "|extruder|hotend|microsd|sd|", "Answer": "

I heated it up and was able to extract some plastic that had gotten jammed in the hot ends opening.

\n\n

I heated up the hotend by commenting out all the other lines of code in Cura's start and end G-code tabs (must have missed some because the printer moved. I would just unplug the printer when it stopped moving). I used an SD card but it would probably work with USB too. I tried to force the filament in to force the jam out of the hotend, but that did nothing.

\n\n

I took apart the extruder assembly and discovered that a section of filament that was too wide got stuck in the hotend's entrance. I pulled the filament out using me multitool and put the extruder back together. Hope this helps others with a similar problem.

\n" }, { "Id": "5929", "CreationDate": "2018-05-07T09:49:16.743", "Body": "

I have excitedly decided to get my feet wet in the 3D printing world, and being that budget is fairly small got myself an ANet A8 3D printer. I have it all assembled and am able to 'print' however I don't really have any luck getting a successful print to actually occur.

\n\n

At the moment I am printing with PLA (start easy(er) is my logic) and had a few issues with getting the print to adhere to the print bed so found a plastic-like print surface that seems to adhere fairly well but I am still having issues :(.

\n\n

When the first layers are being deposited onto the bed (extruded at 215, to a bed temperature of 60) they appear to adhere fairly well to the surface, and the layers start to be built up. However, after a random period of time, the layers lost adhesion to the print bed and start to move around the bed which stuffs up all the prints.

\n\n

in terms of the extruder, the range for the PLA is 190 - 215. I tried lower tempertures but it did not really work in terms of properly melting the filiment unless it was around the 215 range.

\n\n

To give you an example of how the prints come out, i have attached two images (front and back) of a set of prints. I stopped these printing as they lost grip on the bed.

\n\n

Bottom Layer (on the bed)\n\"Bottom

\n\n

Top Layer\n\"Top

\n\n
\n\n

Thanks heaps for helping a newbie out. I had tried changing speeds (faster and slower), changing temperatures and so on but i have no real idea what i am doing and was hoping someone far more knowledgable than me could give me some idea on where i may be going wrong and how to fix the issue.

\n\n
\n\n

My Cura profile;

\n\n
[profile]\nlayer_height = 0.2\nwall_thickness = 1.6\nretraction_enable = True\nsolid_layer_thickness = 1.6\nfill_density = 20\nnozzle_size = 0.4\nprint_speed = 30\nprint_temperature = 215\nprint_temperature2 = 0\nprint_temperature3 = 0\nprint_temperature4 = 0\nprint_bed_temperature = 60\nsupport = None\nplatform_adhesion = None\nsupport_dual_extrusion = Both\nwipe_tower = False\nwipe_tower_volume = 15\nooze_shield = False\nfilament_diameter = 1.75\nfilament_diameter2 = 0\nfilament_diameter3 = 0\nfilament_diameter4 = 0\nfilament_flow = 100\nretraction_speed = 45\nretraction_amount = 4.5\nretraction_dual_amount = 16.5\nretraction_min_travel = 1.5\nretraction_combing = True\nretraction_minimal_extrusion = 0.02\nretraction_hop = 0.0\nbottom_thickness = 0.2\nlayer0_width_factor = 100\nobject_sink = 0.0\noverlap_dual = 0.15\ntravel_speed = 50\nbottom_layer_speed = 20\ninfill_speed = 0\ninset0_speed = 0.0\ninsetx_speed = 0.0\ncool_min_layer_time = 5\nfan_enabled = True\nskirt_line_count = 1\nskirt_gap = 3.0\nskirt_minimal_length = 150.0\nfan_full_height = 0.5\nfan_speed = 30\nfan_speed_max = 70\ncool_min_feedrate = 10\ncool_head_lift = False\nsolid_top = True\nsolid_bottom = True\nfill_overlap = 15\nsupport_type = Lines\nsupport_angle = 60\nsupport_fill_rate = 10\nsupport_xy_distance = 1.2\nsupport_z_distance = 0.45\nspiralize = False\nsimple_mode = False\nbrim_line_count = 4\nraft_margin = 5\nraft_line_spacing = 1.0\nraft_base_thickness = 0.3\nraft_base_linewidth = 0.7\nraft_interface_thickness = 0.2\nraft_interface_linewidth = 0.2\nraft_airgap = 0.22\nraft_surface_layers = 2\nfix_horrible_union_all_type_a = True\nfix_horrible_union_all_type_b = False\nfix_horrible_use_open_bits = False\nfix_horrible_extensive_stitching = False\nplugin_config = (lp1\n    .\nobject_center_x = -1\nobject_center_y = -1\n
\n", "Title": "Stuck getting ANet A8 Printer to build layers", "Tags": "|extrusion|anet-a8|", "Answer": "

Your nozzle is way too far from the bed for the first layer. You should adjust the bed leveling screws to bring the nozzle closer to the bed, to the point where a piece of paper just barely fits between the nozzle and the bed. You should closely watch the first layer as it is being put down, the plastic should get squished slightly and there should be no gaps between adjacent lines.

\n" }, { "Id": "5931", "CreationDate": "2018-05-07T12:01:12.850", "Body": "

Having printed exclusively in PLA for the past year, I'm now looking at trying out PETG in order to print a set of spare parts for my new Prusa i3 mk3.

\n\n

I have bought a roll of PETG from Rigid.ink and their recommendation is to leave an extra 0.2mm gap between the nozzle and bed.

\n\n

Can I achieve this extra Z offset in gcode using Slic3r PE, or would I need to do it using Live Z Adjust on the printer? How would I go about doing this if so?

\n\n

Or is there some other way of doing it, so whenever I select my PETG filament profile in Slic3r, the extra Z-offset gets added?

\n", "Title": "Adding extra Z-offset to bed for PETG in Slic3r", "Tags": "|slicing|slic3r|bed-leveling|pet|", "Answer": "

An extra gap of 0.2 mm on top of your existing gap seems rather illogical as the filament is not squished at the build plate anymore. Furthermore 0.2 mm plus something you already have will soon result in a gap over 75% of your nozzle diameter (including your first layer; note that 75% is considered to be the max for good adhesion). The filament will then drop down to the plate... Possible effects/results can be seen in this recent question.

\n\n

Having printed about 8 kg's of PETG, I have never had to add an additional gap. I have printed on various machines; Prusa clone, Ultimaker 3E and self build CoreXY. I use a 0.2 to 0.25 mm first layer thickness (for a 0.4 mm nozzle) while printing on glass or directly onto the aluminium bed using a PVA based spray glue (3DLAC). Beds are always leveled with the nozzle to bed distance of a sheet A4 printing paper. I have zero adhesion problems.

\n\n

I would recommend you to try print as you would normally do and, when that does not work, increase the first layer in your slicer slightly. Printing temperature/speed towers is also recommended to find the best settings for your brand of PETG.

\n\n

Now to address your specific question how to change the offset in your slicer; an additional offset can be added into your start code script by adding a move to a certain height and redefining the zero Z level. This should be typically done after auto leveling of the bed (if used). So before the first layer is being printed you could add:

\n\n
; start code contains homing, pre-heating, auto-leveling, nozzle priming, etc.\nG1 Z0.2 F500 ; Go to the level of 0.2 mm + your paper thickness\nG92 Z0       ; This redefines the zero Z level\n
\n\n

You can create custom startcode profiles in Slic3r (requires setting the Printer Settings) and store them with a specific name so you can have different printer profiles.

\n" }, { "Id": "5942", "CreationDate": "2018-05-08T13:23:53.930", "Body": "

I am a complete noob when it comes to the 3d printing world. I just finished assembling my printer and I plug it into my computer with the included usb cable and nothing happens. My computer does recognize the printer being plugged in but it just says \"unrecognized device in com 4\". Nothing else past that. Somebody please help me with the following steps that need to be taken to get my CPU talking with my printer.

\n", "Title": "Help installing my Anet A8 printer to my computer", "Tags": "|anet-a8|usb|", "Answer": "

Most of the latest Windows 10 systems with high speed USB (USB3.0) fail to recognize USB to Serial connector (which is here: CH430).

\n\n

Try with a decent self powered USB 2.0 hub that has been seen to fix a majority of USB 3.0 port issues, or try by disabling driver signature enforcement on Win10.

\n" }, { "Id": "5952", "CreationDate": "2018-05-10T08:08:03.047", "Body": "

My main application for my 3D printer (Zortrax M200 Plus) is making 28 mm scale miniatures for role-playing games. Basically people and animals at 1:60 scale, which means that things like arms, legs, or weapons are only a few millimeters thick. If I use the automatically generated supports of the Z-Suite software, the supports end up being thicker than the model parts, and are impossible to remove.

\n\n

I had a bit more luck creating support structures with Meshmixer, but am not totally happy with those. So I am looking for other software to edit .STL files to add supports automatically, preferably with an option to edit those support structures easily afterwards. Any ideas?

\n\n

Note that Zortrax printers only work with proprietary Z-Suite software, so the software that adds the support also needs to be able to export the model with the supports into an STL file, not just gcode.

\n", "Title": "Software for adding supports to 3D printed miniatures in STL file", "Tags": "|software|support-structures|", "Answer": "

I see that you've already tried Meshmixer and didn't find it helpful, but I wanted to call out an article and accompanying video that I recently found which helped me understand Meshmixer's support generation feature a bit better. It isn't magic, but it is pretty flexible and you can customize them. Plus, you can export them either as a separate file (to be imported via Slic3r's Load Part for example), or as part of the primary object STL file (though you loose the ability to set different print settings for the supports). Much of my printer's time is also devoted to 28mm figurines and I've had varied success with them. There are some models whose detail is too fine and which require too much support to be worth it considering the cleanup - I have a bucket-of-shame that's full of them. I just ordered an upgrade for my printer to allow me to print with multiple filament and I'll be seeing if soluble support material is helpful for those small details. Barring that, I've found that some prints do better with Meshmixer's supports while others do better with simplify3d supports, while others still do better with slic3r supports.

\n\n

Summarizing the article on custom Meshmixer supports:

\n\n
\n
    \n
  1. Open your model in Meshmixer
  2. \n
  3. From the top menu select View \u2013 Show Printer Bed
  4. \n
  5. Select Edit \u2013 Transform and move the model to the middle of the print bed\n \n
      \n
    • This step is important because Meshmixer won\u2019t generate any supports outside of the print area
    • \n
  6. \n
  7. If you want to print the model on a different scale, scale the model now, again by using the Edit \u2013 Transform. It\u2019s better to scale\n the model now, because an additional change of scale later in slicer\n would also affect the generated supports, resulting in either too thin\n and weak supports or too thick and hard to remove supports.\n \n
      \n
    • Change the Scale X (Scale Y and Scale Z) to the desired value (1 = 100%, 1.5 = 150% etc.)
    • \n
  8. \n
  9. Select Analysis \u2013 Overhangs\n \n
      \n
    • You can now adjust the Angle Thresh and see a live preview of areas of the model that should be supported
    • \n
  10. \n
  11. Click on Generate Support to see a preview of the support structure\n \n
      \n
    • Every time you make changes to the support settings you\u2019ll have to click on Remove Support and Generate Support to refresh the view
    • \n
  12. \n
\n
\n\n

(The video in the article goes into greater detail on the settings available in step 6.)

\n\n
\n
    \n
  1. Adding and removing supports manually\n \n
      \n
    • You can create a new support by left-clicking and dragging from an overhang to the ground or from an existing support to the ground
    • \n
    • Hold down the Shift key to ignore intersections of the support strut or any other warning and force Meshmixer to generate the new\n support (use wisely)
    • \n
    • You can also click on an existing support to generate a new strut going down to the build plate
    • \n
    • CTRL + Left click on an existing support to remove it
    • \n
  2. \n
  3. When you\u2019re happy with the support structure you can export the model and the support structure together as STL by simply clicking\n Done and clicking on the Export button in the left menu
  4. \n
  5. Alternatively, you can select Convert to Solid to create a separate mesh from the support structure. This will let you set different\n settings in Slic3r for the supports and for the model itself\n \n
      \n
    1. After choosing Convert to Solid choose Edit \u2013 Separate shells
    2. \n
    3. Export both the model and the supports as individual STL files
    4. \n
    5. In Slic3r first load the STL with the model
    6. \n
    7. Double-click on the model and choose Load part\u2026, select the supports STL file
    8. \n
    9. When the STL loads, you can overwrite some of the settings by clicking on the green plus icon
    10. \n
  6. \n
\n
\n" }, { "Id": "5954", "CreationDate": "2018-05-10T14:06:30.123", "Body": "

My print popped out from the bed and glued to the nozzle. As the printer was printing next hour or so, a lot of pla was extruded and formed on the nozzle.

\n\n

I'm wondering what will be the best way to remove pla from the nozzle without overheating wires?

\n\n

a remark: was trying to heat the nozzle over 180, but I am getting a thermal runout. The pla is hard, I don't want to broke the throat.

\n\n

\"enter

\n", "Title": "What will be the best way to remove pla from a volcano nozzle?", "Tags": "|nozzle|", "Answer": "

I would suggests to use a heat gun and carefully warm up the PLA to be soft and remove it carefully like a big piece of Play-Doh. \nUsing the extruder to heat up the PLA don't sound like the best idea, for me, because the inner PLA will be fluid and it could be a bigger mess than it is up to now.

\n" }, { "Id": "5959", "CreationDate": "2018-05-11T17:53:10.460", "Body": "

I would like to print a model of my brain (figure 1), it requires supports (figure 2). The supports also get generated between the brain folds/wrinkles of the brain by Slic3r. I only want the supports to be generated between the bed an the object, not within the object itself. I thought of writing a script to edit the g-code, is there an easier way out?

\n\n

Figure 1:\n\"brain\nFigure 2:\n\"brain

\n", "Title": "Only generate supports between object and bed (for a brain model)", "Tags": "|slicing|slic3r|support-structures|", "Answer": "

Within Slic3r, there are settings for support, which can be enabled to create support only from the build plate. This would still generate supports under the brain, but not within the hollow sections.

\n\n

\"slic3r

\n" }, { "Id": "5967", "CreationDate": "2018-05-14T11:56:22.840", "Body": "

Suppose I'm given a shape with a Voronoi pattern, such as this lamp. I'd like to be able to add a \"skin\" which covers the complete shape on the inner surface so I could print it with translucent material to produce a more-or-less uniform glow. Here's an example of what I'd like to end up with: tea lamp shade. Any ideas on how to reverse-engineer a Voronoi object?

\n", "Title": "How to add \"skin\" to a Voronoi shape?", "Tags": "|cad|voronoi|", "Answer": "

I posted this to an Autodesk forum, and a fellow named \"MagWeb\" proposed the following solution. I have not tried it yet.

\n\n
\n

A possible workflow depends much on the overall shape of the voronoi\n object:\n If it's convex all over (like an egg) or convex and planar (like a\n cylinder) e.g:

\n \n \n \n

Harder but doable with a different workflow on an voronoi object\n owning concave regions...

\n
\n\n

edit

\n\n

Having run some trials, I can confirm this works for simple convex objects. If there are concavities, most likely the source shape needs to be chopped into sections each of which can be treated as convex.
\n I played around with Meshmixer's \"apply pattern\" functions with limited success. I could get a form-fitting pattern shape but with a rather uneven surface. Some fine-tuning of the pattern parameters may help. Note that the new shape tends to be as thick as the original voronoi object, so it may well be better to do something like the following:
\n1) create a duplicate of the original
\n2) shrink the duplicate by a few percent
\n3) align the two objects to a common origin and take the boolean difference to create a thin-wall version of the original.
\n4) Build the pattern object based on that thinwall object.

\n\n

edit number 2

\n\n

I succeeded! For those who care, I took an open Voronoi glasses case and put a skin inside to protect your glasses. See this Thingiverse item

\n" }, { "Id": "5968", "CreationDate": "2018-05-14T18:57:30.283", "Body": "

So I recently installed a genuine E3D V6 (direct) on my AM8 (Anet a8 with upgraded frame). I have already been able to fix most of the print quality problems. But the last one is a hard one for me.

\n\n

It prints super fine up until a certain height. Then the extruder starts slipping and starts making weird noises like its clogged but I will be able to push through filament properly after cancelling the print. I thought it was heat creep but when touching the spiral heat break it is just a tad warm. I can comfortably put my finger on it and keep it there.

\n\n

I also tried two different brands of filament and it seems to happen roughly at the same spot.

\n\n

I tried PID tuning (the temps are literally perfect and super straight with no fluctuations), I tried cleaning the extruder gear and checked if it was worn or anything. Then I tightened everything on the extruder, making sure everything was nice and snug.

\n\n

I am not exactly a newbie when it comes to fixing my own printer and troubleshooting the problems but this one I have never seen or had before.

\n\n

Do you guys have any tips for me to point me in the right direction?

\n\n

Running Marlin 1.1.8. I have a BLTouch on it and an AnyCubic UltraBase.

\n", "Title": "Extruder prints fine up until further down the print", "Tags": "|extruder|anet-a8|", "Answer": "

I used a different heat break with a PFTE lining inside. After that it ran smoothly.

\n\n

Probably what was happening was that my filament was getting too hot and started to stick on the full metal heat break.

\n" }, { "Id": "5971", "CreationDate": "2018-05-15T10:53:34.563", "Body": "

I have a spool of translucent PLA filament that doesn't work well with the filament sensor on my Prusa i3 MK3. The translucency trips up the sensor, making it think the filament ran out. I thought I'd create a filament profile in Slic3r and disable the sensor in the \"Start G-code\" block that gets inserted at the beginning of the exported gcode file.

\n\n

I've got the following code:

\n\n
M900 K{if printer_notes=~/.*PRINTER_HAS_BOWDEN.*/}200{else}30{endif}; Filament gcode\nM406 ; Disable filament sensor\nM117 Filament sensor OFF\n
\n\n

The first line is provided by Prusa's default PLA profile. The second line should disable the sensor, and the third line should print the \"Filament sensor OFF\" message. If I look in the gcode, it's there:

\n\n
G92 E0.0\nM221 S95\nM900 K30; Filament gcode\nM406 ; Disable filament sensor\nM117 Filament sensor OFF\nG21 ; set units to millimeters\nG90 ; use absolute coordinates\nM83 ; use relative distances for extrusion\n;BEFORE_LAYER_CHANGE\n
\n\n

But if I print this gcode file, I see no message, and when checking the sensor in the \"Tune\" menu while printing, the sensor is still on.

\n\n

I thought I might have a problem with line endings, but looking at the file in a hex editor, all the lines seem to end with a 0A line feed character, including mine.

\n\n

Why isn't my printer doing anything with the M406 and M117 messages? Full gcode file here.

\n", "Title": "What is wrong with my \"disable sensor\" gcode?", "Tags": "|prusa-i3|g-code|slic3r|", "Answer": "

When you download the latest version of your firmware you find the implementation of all the codes in Marlin_main.cpp. Here you see that the M406 is not implemented! Hence it does not work.

\n" }, { "Id": "5986", "CreationDate": "2018-05-18T14:20:01.363", "Body": "

I had few printing troubles for first layer yesterday (will certainly ask there as soon as I can take photos) but one of the things I've noticed is that the model cooling fan (the fan that is supposed to blow air on the printed model, not the fan that regulates the noozle temperature) is turned off for printing the first layer.

\n\n

It's a Dagoma DiscoEasy200, printing PLA on a non-heating bed (blue tape).

\n\n

Wandering down stackexchange, I also read turn off the part fan for the first layer from https://3dprinting.stackexchange.com/a/5494/10836

\n\n

So my question is:

\n\n

what's the reason behind turning off the part/model-fan for the first layer?

\n", "Title": "Why turning off model cooling for first layer?", "Tags": "|print-quality|print-fan|", "Answer": "

Filament expands as it gets hot. Cooling the filament will make it shrink, so cooling the filament deposited on the bed can lead to adhesion problems and warping of your products. This is exactly the reason why you use a heated bed (the delta temperature is smaller). So keep the cooling off for the first layers and you'll be fine.

\n\n
\n\n

Additionally (having answered the question for not using cooling air for the first layer), I would like to add that sometimes you won't need any cooling at all (for other layers), or very little cooling.

\n\n

Please do note that some filaments do not like to be cooled down too fast; this may lead to weaker layer bonding or other issues. E.g. POM filament will harden very quickly when cooled causing the nozzle to bump on the quickly hardened filament knocking the product over. Also I found that too much cooling air when using PETG leads to \"string-cheese\" products caused by insufficient layer bonding. ABS is reported to be even more sensitive for cooling (shrinkage during cooling is higher than of PLA or PETG filaments); a frequent advice is not to use filament cooling when using ABS filament.

\n" }, { "Id": "5996", "CreationDate": "2018-05-20T11:34:41.073", "Body": "

I noticed that one edge (which is also the starting point of the print) is always bigger than the other three. Additionally, the walls do not have the same thickness as well. The wall thickness (starting from the thick edge) starts very thin and gets thicker till reaching the thick edge again. Does anyone have a clue whats the issue?

\n\n

\"enter \n\"enter\n\"enter

\n", "Title": "Uneven wall thickness with test cube", "Tags": "|prusa-i3|pla|", "Answer": "

This happens when you have poor retraction / oozing.

\n\n

Signs of too high temp and poor retraction

\n\n
    \n
  1. Thick blob at start point, meaning moltin plastic is oozing out as you change Z position
  2. \n
  3. As you see at the end of the line less material, you prematurely oozed out liquid plastic. As the melt zone is depeleated you get thinner lines.
  4. \n
\n\n

Solutions

\n\n
    \n
  1. Increase retraction distance. This will remove the plastic from the melt zone and prevent some of the oozing.
  2. \n
  3. Speed up Z axis lower and raising. Or decrease distances
  4. \n
  5. Lower temps. Start with 3 cel increments. You want to go as low as you can without causing print defects.
  6. \n
\n\n

You will likely need to do 1 and 3. 2 you really shouldn't need after you fix your temps.

\n" }, { "Id": "5997", "CreationDate": "2018-05-20T15:36:15.710", "Body": "

I'm trying to build a headrest for my Sayl office chair. For that, I'm designing a 3d-printed part that's going to fit on one of the existing rods of the chair.

\n\n

Check out this picture:

\n\n

\"\"

\n\n

How would you go about in getting the exact measurements of that white rod? I tried a caliper, and I'm able to get the width and depth, and I can just assume that the rod's profile is a perfect ellipse, which is probably a close estimate. But say that I want to get a more precise measurement. Is there any technique to do that?

\n", "Title": "Getting precise measurement of a complex rod", "Tags": "|3d-models|3d-design|scanning|", "Answer": "

In an earlier comment you stated that you cannot take it apart. So without taking it apart, you could try to determine the profile the old-fashion way with a piece of cardboard and a short pencil, just cut the rough shape of the rod and place it onto the rod, then take the short pencil and draw the profile onto the cardboard with the pencil parallel to the rod. Measuring the distance from the pencil center to the pencil radius will give you the profile of the rod with that off-set. This technique, or the technique used to create notches in logs for log cabins may be used to find the profile at various sections which have to be entered in a 3D CAD model program and splined/lofted to get the surface of the rod.

\n\n

Alternatively you could use a profile shape tool carpenters use: \"enter

\n" }, { "Id": "5999", "CreationDate": "2018-05-20T23:14:38.813", "Body": "

I have a problem with my first layer height. But lets start at the beginning:

\n\n

I own a Tevo Tarantula Dual extruder printer with a endstop switch for Z axis. \nNo Auto bed level. 0.4mm nozzle.\nEverything worked as it should. I home extruder with G28 command and level the bed using a piece of paper or a thickness gauge 0.1mm. In the slicer I set Initial layer height 0.2mm for layer height 0.1mm. The fisrt Layer looked perfect and sticked to the bed fine.\na few days ago it stopped in the middle of a print, nozzle clogged.\nNo problem i changed the nozzle to a spare one i had. It is not the first time i took it apart.

\n\n

Here starts my problem:\nAfter everything was build together again i home the extruder with G28, level the bed and start a print with the same settings i usualy used to print.\nBut either the first layer didn't stick to the bed at all or looks terrible.\ni tried to level the bed again and also tried to get it to work with playing with the settings. But with no satisfactorily results.

\n\n

Here is is the part my Question starts:\nI noticed that the first layer is actually not at the same Z=0 if i use G28 to home and the Z0 it shows when i print a part.\nFor example: \nSet G28 Level bed with 0.1mm thickness gauge.\nThen start printing with layer height 0.1mm and initial layer height setting 0.2mm

\n\n

The result should be 0.1+0.1+0.2 =0.4 real layer height (if i also count the 0.1 layer heigth(not sure if this is correct)) the printer shows here a Z of 0.2\nBut if i meassure it, it is around 0.7mm thats way to much.

\n\n

\"hope

\n\n

I am confused why this is the case after i changed the nozzle. Considering i leveled the bed the exact same way i did before and used the same slicer.\nThe new nozzle is a little bit shorter but i made sure both nozzle of the dual extruder have the same height. \nI do not understand why this happens. Is there a way to reset the Z? \nI tried the M851 code but as far as i understand it, it i used for Auto-Bed leveling. It didn't work anyway, says unknown command.

\n\n

I could work around the problem by adjust the endstop until it somehow works, but then if i want to level the bed and use the G28 code the nozzle would crash into the bed and that is not a satisfactory solution.\nWould it suffice to set a z-offset in the firmware? I don't have much experience on that area.

\n\n
\n

;FLAVOR:Marlin
\n ;TIME:737
\n ;Filament used: 0.351177m
\n ;Layer height: 0.1
\n ;Generated with Cura_SteamEngine 3.2.0
\n M190 S47
\n M104 S197
\n M109 S197
\n M82 ;absolute extrusion mode
\n G21 ;metric values
\n G90 ;absolute positioning
\n M82 ;set extruder to absolute mode
\n M107 ;start with the fan off
\n G28 X0 Y0 ;move X/Y to min endstops
\n G28 Z0 ;move Z to min endstops
\n G1 Z15.0 F9000 ;move the platform down 15mm
\n G92 E0 ;zero the extruded length
\n G1 F200 E3 ;extrude 3mm of feed stock
\n G92 E0 ;zero the extruded length again
\n G1 F9000
\n ;Put printing message on LCD screen
\n M117 Printing...
\n ;LAYER_COUNT:9
\n ;LAYER:0
\n M107
\n M204 S2200
\n G0 F3000 X75.185 Y64.596 Z0.2
\n M204 S1750
\n ;TYPE:SKIRT \n G1 F1650 X76.267 Y63.316 E0.06132

\n
\n", "Title": "First layer height problem after changing nozzle", "Tags": "|z-axis|calibration|layer-height|", "Answer": "

this is a common case in my TT....\nPlease check the y-carriage wheels - on the 2040 and 2020 profile.\nMostly you shall find a little wobble on one of the ends, that introduces instability in leveling, so you could level it, home it and another level is needed. \nIn my case, I decided to upgrade for dual-z drive - please see this

\n\n

Another interesting Z upgrade:\noption two

\n\n

We can observe z-wobble by watching the x-axis on the left side (no lead screw), it will have a little delay in response when moving up/down comparing to the other side.

\n" }, { "Id": "6018", "CreationDate": "2018-05-24T13:43:25.600", "Body": "

I've just purchased an Alladinbox SkyCube 3D that I want to use to print board game miniatures and other fun stuff.

\n\n

However, the instructions do not give the settings I need to put into software like Ultimaker Cura, and this is where I need some help so that I can generate the G-code files from models I download from MyMiniFactory.

\n\n

I know it uses PLA and the extrusion temperature should be 210\u00b0C. However, I need help with the other settings.

\n\n

Can someone please point me in the right direction? Is there a better software I could be using? Where can I find settings?

\n\n

Thanks.

\n\n

UPDATE: For those wondering \"what\" settings, I would probably start with the printer and extruder specifications. The following is my best guess.

\n\n

\"Machine

\n\n

and

\n\n

\"Machine

\n\n

I'm basing these settings on the device specs on this page:\nLink to Alladinbox specs

\n", "Title": "Cura printer settings for Alladinbox SkyCube 3D", "Tags": "|ultimaker-cura|", "Answer": "

Okay, after some research and experimentation, I've come up with some settings that seem to work.

\n\n

Firstly, some specs about the Alladinbox SkyCube 3D:

\n\n\n\n

Note: There is NO cooling fan present and the base is NOT heated.

\n\n

Now for the actual printing settings...

\n\n

I'm using the PLA supplied with the printer so far, and it works well at a nozzle temperature of 210C.

\n\n

I'm printing at 0.2 mm per layer, which seems to afford to a good level of detail. I've printed a scanned Greco-Roman basin, and the details are very nice indeed.

\n\n

So far I'm using a 20% line filling, and this seems to give the structure a good solidity and strength. I'm also operating the nozzle at its maximum speed of 70 mm/s and it seems to work just fine.

\n\n

Obviously, I'm still experiments, and different materials may require different settings, but overall I'm very happy. I hope this post helps someone.

\n" }, { "Id": "6022", "CreationDate": "2018-05-24T15:12:23.967", "Body": "

I want to print a structure that I can embed in a resin and later dissolve. I know that some fancy 3D printing systems have raft materials etc., that can be printed and later removed easily.

\n\n

Can any one suggest a 3D printing material that can be dissolved in say water or another readily available solvent?

\n", "Title": "3D printing template material", "Tags": "|support-material|", "Answer": "

Wash-away filament used for support in PLA printing is typically PVA, which is completely water soluble and may serve your purpose. It is easily 3D printed as the primary filament and attaches well to the build plate.

\n\n

Many 3D printer filament suppliers will carry this type of support material. It is important to keep it in a sealed bag with desiccant as it will absorb moisture from the air, rendering it useless for printing.

\n\n

One such resource is MatterHackers which prices a half-kilogram at US$45. The link provides suitably appropriate information:

\n\n
\n

PVA (Polyvinyl Alcohol) is a water-soluble material that is often used\n as a support material, but can also be used to print independently.\n PVA supports are useful for complex designs where removing support\n material manually is difficult or impossible, but leaving the part in\n a water bath overnight will completely dissolve this material.

\n
\n\n

\"PVA

\n" }, { "Id": "6026", "CreationDate": "2018-05-25T00:23:27.163", "Body": "

Ok so I am trying to print a new fusion 360 file that I converted into an STL and then into gcode using cura and I got the gcode loaded onto the SD card and mounted into my Monoprice 3D printer, Maker Select 3D Printer v2, and tried to do a print. Now here is the strange part. When I choose \"print file\" and then select a gcode to print it takes me back to the main screen and from there the 3D printers screen displays \"Printing...0%\" for a few seconds. After this it just goes back to displaying \"Stepper Disabled.\" And if it is not stepper disabled it is just a blank screen. I tried to mount and print multiple gcodes just to make sure that it was not the softwares fault and low and behold I was running into the same issue.

\n\n

Now before using today all of the gcode was printing just fine, however for some reason today it decided to give me this issue.

\n", "Title": "Problem with 3D printer not accepting new print gcodes", "Tags": "|g-code|", "Answer": "

Determined Solution

\n\n

Ok so after an hour or so of searching and trying to figure this out, I found that the best to deal with this is by reformating the SD Card once the problem starts.

\n\n

Now I had to make sure that I made a backup of all the files so that I could recover them once the process was complete.

\n\n

Also please note that the problem may have been with one of the files on the SD Card so backing up the SD Card and then restoring the files from your place of storage for the SD card may or may not work.

\n\n

Also, this problem and solution were done on a Monoprice 3D Printer V2 so I am not sure how this would work with any other 3D printer.

\n" }, { "Id": "6027", "CreationDate": "2018-05-25T01:19:10.300", "Body": "

I want to render a visual of an mesh I created with a real support structure that a 3d printer would print. I haven't had any luck in finding a way to do this. Is there a program (ideally free) that can generate the proper support structure and export it into a mesh format, like .stl or .obj ?

\n", "Title": "How can I generate and export the support structure for an object as a mesh?", "Tags": "|3d-models|support-structures|", "Answer": "

You could try Meshmixer its free and the supports generated with it are embedded into the 3D model.

\n\n

Here you can find a reference on Meshmixer supports. Meshmixer is well known for making custom supports for complex 3D models.

\n" }, { "Id": "6030", "CreationDate": "2018-05-25T08:11:14.310", "Body": "

Was anyone able to connect the default Anet A8 (2004) display to a MKS GEN L?

\n\n

I've tried direct connection with re-wiring, and see white squares there, but I'm only able to see white squares on the screen as on screenshot:

\n\n

\"White

\n\n

Wiring:

\n\n

\"Wiring\"

\n\n

The code that is currently used for the display :

\n\n
    \n
  1. Config.h uses #define ZONESTAR_LCD. \nAlso #define MOTHERBOARD BOARD_MKS_GEN_L

  2. \n
  3. Config_adv.h uses #define LCD_PROGRESS_BAR

  4. \n
  5. Pins were left as they were in config file.

  6. \n
\n\n

Also I've tried replacing them with this numbering (which comes from MKS spec):

\n\n
35  16  25  29  5V\n37  17  23  27  GND\n
\n", "Title": "Connecting (Anet A8) 2004 display to MKS GEN L", "Tags": "|anet-a8|", "Answer": "

I've got the Anet A8 display working on a RAMPS 1.6 (which basically works exactly the same as a RAMPS 1.4 board). Note that at first connection I got exactly the same display when I connected the LCD to the EXP1 and EXP 2 headers using the \"smart adapter\".

\n\n

To get the Anet A8 display working on a MKS GEN L (which is basically a RAMPS board as the pin layout of the RAMPS configuration are used in the firmware of the MKS GEN L) you need to forget about the EXP1 and EXP2 headers and the \"smart adapter\" for the Anet A8 display. Instead you need to connect the Anet A8 display to the AUX2 header. The only thing you would have to do (from the hardware side) is to switch the cables that are connected to the pin 1 and pin 2. I used Dupont connectors and cables. Pin 1 on AUX2 is VCC and pin 2 on AUX2 is GND, this is reversed on the Anet A8 display.

\n\n

From the software side you need to set the following constants:

\n\n
#define ULTRA_LCD   // Character based\n
\n\n

and

\n\n
#define ZONESTAR_LCD\n
\n\n

\"Anet

\n\n

Please note that in the photo you still see the \"smart adapter\" with flat ribbon cables, but they are not connected to a screen, the reason for them being there is I also tested the full graphic discount controller in a previous session. The Anet A8 display is connected to the flat ribbon cable on AUX2 just above the top-left most stepper driver.

\n\n
\n\n

UPDATE:\nAt the time of writing and the use of an older Marlin version, this required to set #define ULTRA_LCD. In Marlin 1.1.9 you do not need to set #define ULTRA_LCD explicitly, this is now intrinsically set by #define ZONESTAR_LCD. To elaborate on this, the #define ZONESTAR_LCD sets the constant #define ULTIPANEL which sets constant #define ULTRA_LCD in Conditionals_LCD.h.

\n" }, { "Id": "6046", "CreationDate": "2018-05-28T04:36:45.853", "Body": "

I need to add some simple image renders of STL files to a document. I currently open the STL files in Preview or one of the slicers and grab a screen shot.

\n\n

Is there an easier or automatic way to generate PNG images from STL files on a Mac?

\n", "Title": "How can I create PNG image files from STL files?", "Tags": "|slicing|stl|", "Answer": "

You can use OpenSCAD, as stated in the accepted answer. \nHere is a version of that script that works for Windows for anyone who needs it, as I did.

\n\n
# Change height and width to the desired output image dimensions, in pixels.\n# The path to openscad.exe may also have to be adjusted based on your installation.\n\nheight=1080\nwidth=1080\n\nfor i in *.stl; do\n  T=__tmp__$i\n  b=`basename \"$i\"`\n  echo import\\(\\\"./$i\\\"\\)\\; > \"$T\"\n  C:/'Program Files'/OpenSCAD/openscad.exe -o \"$b\".png --autocenter --viewall --imgsize=$width,$height \"$T\"\n  rm \"$T\"\ndone\n
\n" }, { "Id": "6055", "CreationDate": "2018-05-29T01:40:38.827", "Body": "

In cura one of the options under \"build plate adhesion\" is \"skirt\", which seems to simply print a loop around, but not touching, my print. How is this supposed to help my prints stick to the bed?

\n", "Title": "How do skirts aid print bed adhesion?", "Tags": "|heated-bed|adhesion|", "Answer": "

To use skirts as a bed adhesion tool

\n\n
    \n
  1. Make the value of Skirt Offset from part to 0
  2. \n
  3. Set the number of Skirt outlines to 10 or 15 ,so that it forms an extended outer perimeter of the first layer
  4. \n
\n\n

notes:

\n\n\n" }, { "Id": "6058", "CreationDate": "2018-05-29T10:56:45.593", "Body": "

The printer stopped heating the extruder in the middle of the print and I noticed it getting cooled down. After a few seconds, I heard a ticking noise near the filament (which could probably mean the filament was being pushed into a cold extruder and was skipping. Now, when I try to heat the extruder, it won't heat up. What could've gone broken?

\n", "Title": "German RepRep Neo can't heat the extruder", "Tags": "|extruder|heat-management|", "Answer": "

first, check the breakers \n\"enter\npic source

\n\n

secondary stuff

\n\n

please see a list of item to check, that could point you to the source:

\n\n
    \n
  1. heater cable cut/melted/loose => the best way will be to check the resistance (it shall be around 3.3 Ohms) from the heater cable terminals on the mainboard, if that fails it could be a wire on heater itself.
  2. \n
  3. with the heating switch on - measure the voltage on terminals - shall be 12V, if not - that could suggest a mainboard failure or mosfet failure (in case of mosfet you could measure if there is an input signal given on the middle pin)
  4. \n
  5. there also could be a breaker that was engaged due to short circuit on the wires (the main board layout is not known to me and it is not clear form manual I found online)
  6. \n
\n" }, { "Id": "6060", "CreationDate": "2018-05-29T12:48:31.387", "Body": "

After lots of hassle, I finally made the printer work.

\n\n

I also got myself one bed leveling sensor from eBay, LJ18A3-8-Z/BX 3D Printer Inductive Sensor Bed Auto Level +Plug For Anet A8 A2 A6.

\n\n

I'm not sure how to setup it. Should I print some holders for it first? Where are they?

\n\n

Should I somehow update the firmware?

\n", "Title": "Anet A6 install bed leveling sensor", "Tags": "|anet-a6|", "Answer": "

You need to do two or three things:

\n\n
    \n
  1. Print a holder or bracket for the probe, if your probe did not come with one, and looking at the eBay item listing, it doesn't appear to include a bracket. There are a number of designs out there, take a look at thingiverse, for example:\n\n
  2. \n
  3. Change the firmware - SkyNet 3D, Marlin is a good choice, or Anet's own
  4. \n
  5. You may need to change the Z-stops
  6. \n
\n\n

There is a step by step guide here, Autolevel for the A8 Anet 3D Printer. It is for the A8 but most, if not all, of the steps will apply to the A6

\n" }, { "Id": "6067", "CreationDate": "2018-05-30T13:03:56.937", "Body": "

I have been having an issue with certain portions of the walls of certain prints becoming separated from the rest of the model. It happens with smooth vertical edges.

\n\n

Specifically, this model, ID Badge Holder, on the edges where the lanyard would be attached. And on several places on this model, Cat Necklace.

\n\n

Here is the resulting issue:

\n\n

\"Detached

\n\n

I am quite new to 3D printing, so I'm sure this is an easy fix, I just don't know about it yet.

\n\n

I am printing with a Monoprice Select Mini V2, using Hatchbox PLA, and the default slicing settings inside Cura. I don't have problems with any of the D&D figures I've printed, or some of the other thicker square pieces I've printed. I know I've got kind of a bargain printer; if it's just a quality issue I have to learn to live with, no problem. But if an expert knows of some slicer settings to tweak for these kinds of prints with flat vertical walls, I'd love to give it a try.

\n", "Title": "What is causing the walls of my prints to separate?", "Tags": "|print-quality|monoprice-select-mini|", "Answer": "

Yup I can tell at a glance it is under extrusion. I bet the issue is temp.

\n\n

Test your steps per MM for E. See https://reprap.org/wiki/Triffid_Hunter%27s_Calibration_Guide

\n\n
\n

Measure Required tools: vernier caliper with depth gauge, or similar\n tool that can precisely measure 100mm. Your hob effective diameter is\n unlikely to be exactly 7mm.

\n \n

Remove the hot-end from extruder so you don't waste filament. Feed in\n some filament. Using the extruder body as a reference point, mark the\n filament at 120mm. Tell the printer to feed 100mm of filament. Measure\n the distance from the extruder body to the mark you made. It will be\n over 20mm if it moved too little, under if it moved too far.\n new_e_steps = old_e_steps * (100 / distance_actually_moved) \u2026 or,\n old_e_steps * (100 / (120 - distance_to_mark)) Set this value in your\n firmware. You may need to re-flash your board. Sprinter/Marlin\n supports M92 Ennn to set this value temporarily. Repeat from Step 3\n until you get between 96-104mm. Then continue with this guide. You'll\n dial it in perfectly later on. Don't flash firmware yet. There's a\n further refinement to this value below. Why? The back-pressure from\n the hot-end alters how much plastic each hob revolution pushes, and\n you'll probably end up tightening your idler more which reduces the\n hob effective diameter. Re-attach hot end.

\n
\n\n

If that comes out correct, your Temps are too low.

\n\n

To fix this, you will need to play around and increase your temps by 2-3 degrees until the infill comes out correctly. Do not go over the minimum needed else you will have other issues.

\n\n

Also check for obstructions such as carbon buildup in the nozzle. In addition to the tightness of your filament tensioner on your extruder drive if you have one.

\n\n

I will say, I had this issue on my Ultimaker 2. An apparent design flaw caused the filament drive to lose power over time or possibly wear on the tube causing extra drag (bowden tube only issues). I corrected it by increasing the extrusion multiplier in simplify3d. However that is a bandaid and the real issue should be addressed. In my case in the end I replaced the drive with a bondtech.

\n" }, { "Id": "6078", "CreationDate": "2018-06-01T11:00:48.853", "Body": "

I'm running a RepRap based ORDBOT Hadron on Repetier firmware version 1.0 that I built from a kit. I've slowly worked out the kinks but this one is an utter doozy.

\n\n

Basically I occasionally get shifting in the y-axis - the print bed moves on this axis - but only in the positive y direction. Never in the negative y direction. I'm not sure if I'm using those terms correctly, but the bed shifts forwards (negative y), so subsequent print moves are displaced in the positive y direction relative to the rest of the print. I'm calling that a \"positive y direction shift\".

\n\n

The offending y-axis is belt-driven by a single NEMA 17 stepper motor. The belt is tight (not too tight I don't think) and well-aligned. It would be difficult for it to become unaligned or lose tension since it runs along a piece of extruded aluminium that is very rigid.

\n\n

It took a long time to notice the pattern. Some prints don't have the problem, and some prints I simply cannot finish no matter what I do. Finally I found a model that reliably reproduces the problem, on the same move, on about 50% or so of its layers.

\n\n

This piece is supposed to be straight up & down. Wild.

\n\n

\"This

\n\n

The moves that cause the shift are on this red line.

\n\n

\"The

\n\n

The issue seems to only occur on curved moves with a radius of approximately 2-3 mm that point their convex side towards the y-negative direction. Larger or smaller radius moves don't cause the problem. In fact sharp turns don't cause the problem either. Only 2-3 mm radius moves with their convex side towards y-negative produce the issue. No other kind of move causes the problem.

\n\n

I think the offending move is highlighted here in red, but it might be one of the two moves either side of it. I haven't been able to narrow it down.

\n\n

\"Cross-section

\n\n

Also note that there is no opportunity on this part of the G-code for the hotend to snag on the model, and I see no evidence of this when it happens. If it were, I imagine a small model like this would simply dislodge, rather than jamming the y-axis.

\n\n

I've tried lowering the y-axis acceleration, to the point where you can hear the y-axis spooling up and down as it slowly accelerates to and fro, and the problem remains. What is especially baffling is that if I leave the y-axis acceleration at 300 mm/s2, the shifting never happens in the negative y-direction, only in the positive. And even if I lower it to 50 mm/s2, the shifting still happens towards positive y. So somehow this problem is independent of y-axis acceleration as set in the firmware.

\n\n

One thing I have noticed, is that even if you can visibly see how slowly the y-axis accelerates, when the problem occurs, the y-axis seems to launch itself into overdrive and whip around that corner as fast as possible, to the point that it overwhelms itself. I'm almost certain the moves that cause the skip are breaking the acceleration limit, but I have no idea what to do about it. It seems like a bug in the firmware, like instead of reducing the acceleration it's increasing the acceleration.

\n\n

I would guess that somewhere in the code there should be a mathf.abs() around a term, so it slows down the move whether it's positive or negative, but that's pure speculation.

\n\n

The above paragraphs no longer appear to be true. I changed the y acceleration limit to 50 mm/s2 and the piece printed perfectly. It's possible the firmware update made a difference. I've also enabled EEPROM, so that may have changed something as well. It's also possible that by re-compiling the firmware every time I made a change in the past, I made an error that misled me about the problem. I will try to reproduce the problem and post an answer about it if I manage to, otherwise I may just close the question.

\n\n

I'm hesitant to say the firmware is the problem because a) I don't know enough to confirm it and b) it makes the solution super difficult: either wait for a solution from the developers, or write it myself. Whilst I could find & write the solution, it would take a lot of work and I'm hoping it's simpler than that.

\n\n

I've recently upgraded the Repetier firmware from 0.92 to 1.0, and the problem has remained. This also has happened when controlling the printer from Repetier Host, Repetier Server and Octopi, so I'm confident it's not the controller. I'm also using Slic3r.

\n\n

Here are some photos of the y-axis belt as requested:

\n\n

Motor:\n\"Motor\"

\n\n

Idler:\n\"Idler\"

\n", "Title": "Perplexing Y-axis shifting problem - only in positive y direction, and only on moves of a very specific radius", "Tags": "|reprap|firmware|repetier|troubleshooting|", "Answer": "

You see this for a few reason.\nFirst you are going too fast and you are getting belt shift from the whip lash. You can mitigate that by going slower and adjusting your Jerk settings to lower. Though usually this is not a consistent wall. Usually you see this.

\n\n

That said it is likely you have not adjusted the current to your stepper motors correctly. I don't know if your system has pololus, but you will want to adjust your current carefully. If you hear thudding from your stepper or the stepper cannot move, you have not done this correctly. Note I've fried many boards adjusting these, make sure to do it with the board unplugged or with a ceramic screw driver. Here is a more complete guide.

\n\n

A last option is your system has too much friction as Ultimaker points out in their trouble shooting guide. You said your belts are very tight. I wonder if you have them So tight you are actually creating binding. Check to make sure the belts are not rubbing in any way.

\n\n
\n

Leaning: A leaning print is usually caused by friction causing the\n print head to move a shorter distance than expected. Make sure that\n the short belts that connect the stepper motors to the axes do not rub\n up against the main body of the printer. Similarly make sure that the\n pulleys on the stepper motors that the belts ride over are not\n touching the side of the printer. If they are you must move the pulley\n closer to the stepper motor.

\n
\n\n

My bet is it's current.

\n" }, { "Id": "6085", "CreationDate": "2018-06-04T07:42:42.550", "Body": "

I have various G-code files stored at SDCard attached to my Prusa i3 MK2S printer. They are either for ABS, PLA or SBS. The more files I had on the SD card, the more I was running into trouble of finding for which material is which G-code.

\n\n

To tackle the situation I created folders ABS, PLA and SBS and put new files into those directories.

\n\n
\n\n

Is there any better method of finding, backwards from G-code, which material settings were intended to be used?

\n\n

If so, is there a way to read temperature settings from G-code by Prusa i3 firmware without printing the actual model?

\n", "Title": "How to sort G-code on SD card based on material (ABS|PLA|SBS)?", "Tags": "|prusa-i3|g-code|microsd|sd|", "Answer": "

Good question as I also ran into some files not remembering for which purpose or which material I printed these. The online G-code visualizers do not display the temperature or the correct filament width, so basically that won't help you (unless you modify the open source programs...). Reading from firmware would not be possible without adding a new feature, as far as I know, this is not implemented in Marlin Firmware or in Prusa firmware (which is based on Marlin).

\n\n

When using a printer server like OctoPrint, you can store your G-code files in a folder structure that you can define yourself; so basically create a directory structure based on the material type, brand, etc.

\n\n

However, it is relatively easy to write a small program in Python to read a file and interpret the lines (even if you're not a programmer). There are lots of tutorials and examples to find to open files read the file e.g. line by line and detect strings to identify the settings for G-codes M104/M109 (hot end temperature) and M140/M190 (bed temperature). Usually you slice your models with a specific set of bed and hot end temperature (which you know), so basically you can find out which material was meant to be printed with the G-code file. You could even let the program sort the files for you by moving them into separate folders.

\n\n

I'll add it to my list of things I'd like to do some rainy day! :)

\n\n

Note that some printers use different, or even custom G-codes to control bed temperature; e.g. when reading Ultimaker 3 G-code files you cannot see the bed temperature!

\n" }, { "Id": "6088", "CreationDate": "2018-06-04T13:14:29.050", "Body": "

I'm having an issue with my very first printer, a Monoprice Select Mini V2. After doing about 20 successful prints on it, the Z axis is suddenly acting very odd.

\n\n

When I 'home' the Z axis and move the nozzle on the X and Y, I can do the paper test on all four corners of the bed without issue - the leveling is near-perfect. But when I start a print job, the nozzle is much lower - by at least 1-2 mm.

\n\n

This causes the print head to grind against the print bed, which I unfortunately need to replace as it's pretty much destroyed. The nozzle is so much lower at the start of a print job than it is at the home position that it ground a permanent line on the bed. It's trying to go so low, there's enough pressure on the nozzle to not let any filament escape - leaving a bad gouge like I dragged a screwdriver across the print surface. Not good.

\n\n

It's not the Z-axis limiter switch. I confirmed that is both working and secured tightly to the printer body. When homing the Z axis, I can hear the switch click and the printer stops at that position correctly. It's only when I start a print job that it ends up lower, almost as if it's ignoring the switch.

\n\n

I also eliminated my slicer software from the equation by printing something I had printed successfully just a couple hours prior - without reslicing or modifying the GCODE file at all. I'm at the point now where I can't print anything that I could before, without having this problem. My heat and speed settings remained untouched.

\n\n

How can I solve this? The issue popped up just after doing a successful print. What gives? I've heard of the opposite problem (Z-Axis 'too high'), especially after changing nozzles, but not 'too low', and I've never seen it where the print job actually starts lower than the true zero position. Help!

\n", "Title": "Z-Axis zero/home position is higher than the actual print starting position", "Tags": "|z-axis|bed-leveling|monoprice-select-mini|", "Answer": "

I just had this issue with my Monoprice Select Mini V2. It seems there's some bug in the software if you already have the 3D printer at its lowest Z-elevation before starting the home calibration. To make sure it prints correctly, move the head up using the manual controls, then hit home. It should then go to the correct default location. I then adjusted my print height to that default home location and it seems to be working again.

\n

Hope this helps other people still having difficultly with this is

\n" }, { "Id": "6090", "CreationDate": "2018-06-04T17:12:49.220", "Body": "

We wanted to test the post process properties of PVA but did not want to print anything. We unloaded the ABS that was in the printer and then started loading some PVA. After the PVA was loaded we let it keep going through the loading process until it pushed all the PVA out and we had our test sample. We then stopped the loading process and started the loading process again to put the ABS back so we could continue with normal printing procedures. Now, the ABS will not load into the right side and started making a clicking/ticking noise. Our printer is a Flashforge Dreamer. What is wrong with my printer and how can I fix it?

\n", "Title": "3D printer filament will not load after running the printer out of filament", "Tags": "|extruder|filament|diy-3d-printer|abs|", "Answer": "

PVA is nasty filament to print (from experience). I use it in the Ultimaker 3 in a separate designed core (BB) and even with that core the filament frequently cooks up and carbonizes clogging the nozzle resulting in grinding of the filament at the feeder (it also attracts water really easy, so keep it in the bag with desiccant bags).

\n\n

To clean the inside of the nozzle, a few techniques exist to remove blockage. E.g. by performing a cold pull or using the atomic method. Both techniques rely on the mechanism to insert the (cleaning) filament when it's hot and remove it quickly at a lower temperature. E.g. see here or here.

\n\n

The trick is that you pull out all the goo inside the nozzle, you do that by raising the temperature and sticking some (cleaning or high temperature) filament (your ABS will do fine) to the goo. When the temperature lowers, you quickly pull and hope that some of the goo sticks to you cleaning filament. Cut of the tip and repeat until no goo or burnt particles come out and you can push the cleaning filament through the nozzle. You do not need the automatic feeding, it's all manual labor, just raise temperature through the menu and press the lever of the extruder to load the cleaning filament, lower temp and pull back after cooling a bit while pressing the extruder lever.

\n" }, { "Id": "6096", "CreationDate": "2018-06-05T01:22:33.147", "Body": "

2 days into a 5-day build, I came home from work and found the build ruined because the build plate had slipped.

\n\n

I'm using a Raise 3D N2 Plus printer, with the standard glass build plate that comes with it, attached via 4 clips: two stationary ones at the back, and two standard binder clips at the front, which shipped with the printer, which hold the glass build plate plate to the heated surface beneath.

\n\n

The left-side clip had come off of the heated surface, remaining clipped to the top and bottom of the glass plate, and the whole thing slipped an inch or so. I immediately canceled the build, and I can start another one, but before I do I'd like to know how this happened and what I can do to prevent it from happening again.

\n\n

What typically causes the plate clips to come free? Is there anything I can do about it? Will adding more clips around the edges help? I'd really prefer not to ruin more builds if I can help it...

\n", "Title": "What can cause the build plate to slip?", "Tags": "|build-plate|", "Answer": "

Before two month ago I tested double-sided tape between bed and glass plate. It's awesome and really works perfectly. No more clips, so I'm able to print to the complete area. And no more slipping. Don't use that much tape, just a little bit at the edges should be totally fine.

\n" }, { "Id": "6100", "CreationDate": "2018-06-05T09:35:19.560", "Body": "

How much should I subdivide a surface in preparation for 3D printing?

\n\n

For computer graphics, I know that I need to balance the smoothness and the rendering time, but for printing, I'd like it to be completely smooth. Are there any reasons why I shouldn't subdivide a lot (e.g. 7x) in preparation for 3D printing?

\n", "Title": "Subdivision surface for STL files", "Tags": "|3d-models|surface|", "Answer": "

Subdividing an existing mesh further won't do anything because you're not adding additional detail, just representing the same thing with more triangles. Subdividing as \"preparation\" doesn't make much sense. You should make sure the mesh is created with sufficient detail while modelling.

\n\n

A mesh created for 3d printing should generally have a lot more triangles than one created for use in rendering, but within reason. It doesn't make sense to make the mesh (much) more detailed than the printer can print, and similarly having lots of triangles can make the slicer slow or unreliable. As a very rough guideline, I would say that 10.000-100.000 triangles per model is reasonable (but this obviously depends on the size and level of detail).

\n\n

Some slicers may output G-code that will have segments corresponding to each and every triangle in the model, even if these segments are very tiny. This may cause the printer to slow down a lot while printing, but most slicers take care of this by merging small segments into larger ones. Depending on your slicer, you might have to watch out for not having the triangles be so tiny that the number of segments created becomes a problem.

\n" }, { "Id": "6102", "CreationDate": "2018-06-05T13:37:43.077", "Body": "

Has anyone configured BLTouch with Marlin firmware?

\n\n

I could only find videos about older firmware version. I followed this document. To avoid causing any damage to the printer, I removed all connections, took the board out and connected a stepper to Z-axis terminal and BLTouch to Z-min and servo 1. All other axes are not connected. X-min and Y-min end stops were shorted using jumper (Mine is NC configuration).

\n\n

After updating the firmware, I can move the X and Y steppers, but not Z stepper. There's a blue light glowing inside BLTouch, which turns off if I remove Servo connections.

\n\n

The M119 command shows all end stops are open.

\n\n

Do I have to change pull up settings of end stop?

\n\n

I cuurently have:

\n\n\n\n

Here's my configuration file

\n\n

\"enter

\n", "Title": "Configuring BLTouch with Marlin firmware", "Tags": "|marlin|ramps-1.4|bltouch|", "Answer": "

Edit: The answer below reflected the original question and upload of the configuration that was incorrectly configured. The value of this answer can be found in checking your configuration thoroughly before you post a question. It also answers the question whether pull-up resistors need to be set for the end stops.

\n\n
\n\n

Basically you need to follow the instructions of the manual/sheet you mention in your question. Be sure to get all the changes correct in your configuration.h file!

\n\n

E.g. in your configuration

\n\n
//#define NUM_SERVOS 3 // Servo index starts with 0 for M280 command\n
\n\n

should be

\n\n
#define NUM_SERVOS 3 // Servo index starts with 0 for M280 command\n
\n\n

So no servo was defined (the servo pulls up the pin of the BLTouch sensor)! Also your BLTouch delay is 375 while 100 is prescribed. Please check that you managed to get all the proposed changes in the configuration file (as I stopped comparing for you after finding 2 errors in your configuration!) as I believe that you have not updated your configuration correctly as described. There should be no differences in setup for Marlin 1.1.6 or 1.1.8.

\n\n

To answer your question: No, you do not need to change pull-up settings.

\n" }, { "Id": "6111", "CreationDate": "2018-06-07T07:27:17.280", "Body": "

IANAE (I am not an engineer), but I've been wondering: wouldn't it make more sense to limit the length of the melt-zone and hot-end as much as possible? I've been having a lot of problems with nylon causing jams inside the heat-block, this being the region where the filament is no longer rigid, but becomes squishy then liquid as it is heated.

\n\n

In my extruder (E3d v6) there is a cold section, then a heat-break then about 20mm of hot-end.

\n\n

\"Extruder

\n\n

If the heating of the filament took place at the last possible moment, say the last couple of mm before the orifice, wouldn't this not only alleviate jamming problems caused by soft filament deforming, but also allow for much more precision with extrusion - less hysteresis due to the reduced volume of semi-liquid material.

\n\n

\"Extruder

\n\n

I'd imagine that this would work by having a heated nozzle tip, say a nichrome ring around the nozzle orifice, and as the filament hits the inside of the bottom of the nozzle it melts and is forced out the hole.

\n", "Title": "Would a really short melt-zone be beneficial", "Tags": "|hotend|extrusion|development|", "Answer": "

There is a trade-off between the length of the melt zone and the speed at which you can print. The filament itself is somewhat of an insulator, so as the outside of the filament is heated up by being in contact with the melt zone, the inside stays cold. Therefore, the filament needs a certain amount of time inside the melt zone for the inside to fully melt. If you have a shorter melt zone, you need to print more slowly to give the filament enough time to melt. This is precisely the reason why the E3D volcano exists. It has a longer melt zone, so you can print more quickly.

\n\n

You can partially compensate for a shorter melt zone by heating the nozzle up more, but there is an upper limit to how hot you can go (you don't want the outside of the filament to burn before the inside is melted).

\n" }, { "Id": "6119", "CreationDate": "2018-06-07T19:31:09.973", "Body": "

I'm in the process of building my own head unit / stereo prototype for a car, which will have a 3D-printed enclosure.

\n\n

My concern is that cars can get quite hot in the sun, and even more so if you live in hot climates. Some estimations put the interior of cars getting up to 50-60 \u00b0C, sometimes even in only 20 \u00b0C weather due to the 'greenhouse' effect created in the car. I live in a fairly temperate climate, but the summers can still get up to 20-29 \u00b0C (80-85 \u00b0F), and my car might get up to 60 \u00b0C/150 \u00b0F on a hot day.

\n\n

The part won't be exposed directly to the sun, but will obviously be exposed to heat both from the interior of the car when in the sun, and potentially from the engine radiating heat through the firewall, though the latter factor will differ from car to car.

\n\n

Should I be concerned using PLA for my part? If not, what material, if any, would be better suited for these possible temperatures (other than metal)?

\n", "Title": "Can you put PLA parts in your car (in the sun)?", "Tags": "|pla|filament-choice|stability|", "Answer": "

Just for completeness sake, one can print a casting mould and then cast UV resilient resins from the mould.

\n

The mould would be made of PLA. Because it is quite brittle, you would need to make a multi-part mould if you want it to be re-usable. Otherwise, if you are using something heat-resistant then you can melt or break the mould once you are done.

\n" }, { "Id": "6130", "CreationDate": "2018-06-09T10:47:22.053", "Body": "

I changed the filament (PLA) in my Wanhao Duplicator I3+. I ended the first try to print when I saw that the 'lines' were too thin. I re-sliced with a higher temperature (195\u00b0C instead of 190\u00b0C). Now the print started without a problem but after about 25% no more filament came out of the extruder. What can be the reason and how can I resolve it?

\n\n

The filament is from Vertex, grey. The object that I use to test is a 20 mm hollow cube from Thingiverse that I have used for the previous filament too.

\n\n

I am not sure that this is a clogging problem since the print starts with no problems. It just stopped after 25%. When I started another print I was able to finish by increasing the temperature.

\n", "Title": "Extrusion stops during print", "Tags": "|filament|pla|wanhao|", "Answer": "

I redid the print in order to reply to some questions posed in the answer of @kdtop. The print started but the output was not consistent and sometimes stopped. The temperature is 195\u00b0C and sometimes 'drop' to 194\u00b0C. First I pushed the new real so that the extruder did not need to pull so much. When this did not solve the problem I changed the temperature to 200\u00b0C. Now the output became consistent and my print finished. It was not as good as the one that I did with my previous filament. The top was not as neatly closed. Only the last 2 layers covered more or less for 100% the surface (perhaps 200\u00b0C is too high for this?).

\n\n

For me the solution is to higher the temperature to 200\u00b0C (or perhaps 205\u00b0C).

\n" }, { "Id": "6172", "CreationDate": "2018-06-13T16:29:46.320", "Body": "

I have a Monoprice Maker Select v2. It is the kind where the moving plate (heat bed) provides the Y-axis and the moving extruder provides the X-axis. It has a fairly rigid sheet metal frame. In addition, I added steel rods has Z-braces.

\n\n

I see some pretty obvious ghosting. This happens for an inch or so right after every sharp turn. Clearly, vibration is to be blamed. I found two simple techniques that improves the situation:

\n\n
    \n
  1. soft floor mats under the feet (allowing the machine to move freely)
  2. \n
  3. bolt the machine to the desk (preventing the machine from moving)
  4. \n
\n\n

Surprisingly, these two opposites provided exact same level of improvement for ghosting. My question is: which approach is better? Moreover, to further improvement, should I use...

\n\n\n", "Title": "Fix ghosting problem (damping versus bolting printer to a desk)", "Tags": "|ghosting|knowledgebase|", "Answer": "

I got good results by mounting my printer on a concrete paving slab (obviously cleaned and sealed) and placing that on a foam mat. The combination of high mass and flexible mount has substantially reduced ghosting in my prints. Not my idea; I found it on a video on YouTube (linked below). Honestly not sure if this is the best solution from an engineering perspective (I like the idea suggested above of trying to find the resonant frequency) but it's easy, fast and cheap. And as a bonus, it has substantially reduced the noise level of my setup.

\n

\r\n \r\n

\n" }, { "Id": "6183", "CreationDate": "2018-06-16T14:01:06.227", "Body": "

Has anyone run into problems printing with Proto-pasta's HTPLA on a Prusa MK3 (or any printer really)?

\n

I picked up some samples from PP, specifically their glitter filament and a couple of the premium HTPLA filaments. I printed one small piece with the glitter filament the first night I got the package, but I haven't been able to get any of the filaments to print since.

\n

Here's a video I shot outlining the issue's I'm running into.

\n

The problem seems to be that the filaments won't extrude from the nozzle and this problem applies to all of the samples I got. If I unload the HTPLA and print with any of the other filaments I have been printing with (various inland filaments) they all print out fine.

\n

I thought maybe I had a clogged nozzle so I did a couple of cold pulls with a different filament. The pulls came out clean each time. I tried loading the HTPLA back in and while the filament extrudes fine during the MK3's loading process it still doesn't extrude when trying to run an actual print.

\n

I've read through Proto-pastas page on avoiding clogs with composite PLAs and adjusted my slicer settings accordingly but still can't seem to get a print out.

\n

I'm printing with:

\n\n

Any ideas??

\n

UPDATE

\n

Over the weekend I talked a bit with proto-pasta and did some tests on my own. Proto-pasta suggested disabling the filament sensor on my printer since the additives in some exotic filaments can falsely trigger the sensor.

\n

I went to try this, but my sensor was already off.

\n

I tried switching back to a known good filament and found that I couldn't get anything to extrude anymore. After researching a bit online I suspected that there may be an issue with a clogged nozzle, so I did a couple of cold pulls and then tried one of my known good filaments again. This time I was able to extrude and print out a full test without issue.

\n

After that I switched to one of the HTPLAs and it started printing. I thought all was good, but during the course of the test print the nozzle clogged again and stoped extruding.

\n

So I'm close, but not quite there. I'm wondering if cold pulling again to unclog the nozzle and then walking through some of the settings changes outlined in proto-pasta's page on avoiding clogs will help. If I spent the entire time walking through that page with an already clogged nozzle it would make sense that nothing worked. That page is more a list of preventative measures rather than reactive.

\n

I'm going to dig in more tonight. I'll post back with an update.

\n

Reply from Proto-pasta

\n

I got a reply from Proto-pasta for some questions I asked regarding the nozzle dimensions I should use with their filament. The 0.4 nozzle borehole is not too small for the glitter, though the larger diameter will help it sparkle more.

\n
\n

Thank you for following up. Yes cold pulls that leave behind material can cause all sorts of trouble. It happens to me on the Lulzbot quite often, probably my biggest gripe about that machine. Anyways, on to your questions:

\n\n
\n", "Title": "Issues with Proto-pasta filament extruding on prusa mk3", "Tags": "|filament|prusa-i3|filled-pla|", "Answer": "

tl;dr

\n\n

I had a dragging idler pulley that was causing extrusion issues. Opening the idler pulley door and working it a bit with my finger resolve the issue.

\n\n

The detailed explanation

\n\n

So after two weeks or so of troubleshooting, banging my head, taking a break, and coming back to it I'm finally getting good prints again.

\n\n

I do believe the original problem started with the clogged nozzle due to the bore hole diameter being too small for the glitter filament as mentioned by Axel in the post comments (a bit rude about it, but still probably true), but there were other issues that I ran into.

\n\n

Troubleshooting steps

\n\n

General cleaning

\n\n

After I switched back to inland filament (which had been working fine for months) I was still having extrusion issues. At this point it was due to the clog (I suspect).

\n\n

\"enter

\n\n

I tried several steps to unclog the nozzle including the steps suggested by prusa support and cleaning techniques suggested by Tom amongst other places.

\n\n

I also picked up a couple of pencil brushes on amazon including a small brass brush that's perfect for cleaning off the extruder pulley teeth.

\n\n

Cold pulls

\n\n

During these steps, I think I introduces the second problem without being aware of it. I perform a bunch of cold pulls (explained by Tom in his video and other videos and posts). During one of these cold pulls I forgot to loosen the MK3's idler pulley door before tugging. Once I noticed it I loosened the door, but I think that tug may have affected the idler pully getting it a bit gritty (I don't know how it would have done that, but it's my current guess).

\n\n

At this point I was still getting extrusion issues and didn't think about the idler pulley being the issue, so thinking that the nozzle was still the issue I ordered a second E3D nozzle and some Ethyl Acetate which dissolves PLA.

\n\n

Notes on Nozzles and Ethyl Acetate

\n\n

E3D nozzles

\n\n

A couple of helpful points I learned while researching these purchases.

\n\n

There are a lot of cheap nozzles that say they work on the E3D, but be sure to read the comments from whatever nozzle you're looking at because apparently a lot of them either don't fit or don't print well.

\n\n

I ended up purchasing a replacement nozzle sold directly from E3D to be safe.

\n\n

Ethyl acetate

\n\n

As far as the Ethyl Acetate, I was having a hard time finding it online when looking for it as a solvent without having to purchase the chemical in a giant jar. I didn't want to buy a jug, I only needed a bit.

\n\n

After doing a bit more research I found out that it's the same chemical that entomologists use to kill insects for preservation and when you look for the chemical from that context you can find it sold in much smaller quantities. I was able to find a small jar of ethyl acetate on amazon for 7 dollars. Smaller, safer, and cheaper.

\n\n

Finding the actual problem

\n\n

Once the nozzle and ethyl acetate came in I pulled off the old nozzle, dropped it into the ethyl acetate to soak, put the new nozzle on (per the instructions from prusa's website), and reaccelerated the printer just to be safe. I tried printing a test block and it seemed better, so I then tried a larger print and while the printed model seemed better, it was still obviously having extrusion issues and crumpled in my hand.

\n\n

With the nozzle ruled out, I started looking elsewhere on the printer. After watching a video from the 3D Printing Nerd on extrusion issues he was having on his MK3 I decided to check the extruder pulleys. When I tried to open my extruder idler pulley door I noticed I couldn't actually get it open; it seemed to be stuck on the bottom. I tried to free the bottom of the door, but couldn't without forcing it past my comfort level. Instead, I unscrewed the top of the pulley door to remove it.

\n\n

\"enter

\n\n

From here I noticed two things, I didn't get the pulley teeth as clean as I though I had before and the idler pulley's mounting rod was extended just a bit (which is what was causing it to stick at the bottom). I adjusted the rod and put the door back on. It was printing better but still had extrusion issues.

\n\n

I then watched the pulley during the filament loading and notices that while the extruder stepper motor was turning smooth, the idler pulley was stuttering a bit while turning. I imagine those little stops and starts were what was causing the extrusion issue.

\n\n

I opened the idler door back up and worked the idler pulley a bit with my finger. It started to spin free (maybe there was gunk in it? maybe it was rubbing a bit against the mount? \u00af\\_(\u30c4)_/\u00af), so I closed the door and screwed it back in tight.

\n\n

Ran a test print -> perfect.

\n\n

\"enter

\n\n

I've run several prints increasing in complexity and it seems like I'm back to printing well.

\n\n

\"enter

\n\n

How I introduced my own problem

\n\n

As I noted above in the explanation, I think I introduced the problem during a cold pull where I forgot to loosen the idler pulley door. I know this explanation is super verbose, but when looking around, even after fixing the printer, I haven't really seen many people noting that on the MK3 you should make sure the door isn't screwed down when performing the cold pull. It's def a common sense move, but I don't see it mentioned as a potential for introducing a problem.

\n\n

Also I didn't see a dragging idler pulley as a possible cause of extrusion issue so I thought I'd note it here as well.

\n\n

Hopefully if anyone else has a similar problem on the MK3 this will be a source of help.

\n" }, { "Id": "6187", "CreationDate": "2018-06-17T11:49:21.653", "Body": "

I am building an enclosure for my 3D printer (Anycubic i3 Mega) and I'm wondering about heat and ventilation for my machine. My enclosure is is build from five 50x50x50 cm Plexiglass frames glued together. Currently I'm using only PLA for printing.

\n\n\n", "Title": "Enclosure Ventilation", "Tags": "|pla|abs|ventilation|enclosure|", "Answer": "

When printing PLA you do not require an enclosure! PLA does not shrink as much as e.g. ABS. When printing PLA you should definitely ventilate your casing. I guess your steppers are also located in the enclosure, so you should be careful of not overheating the enclosure. Not only the steppers, but also think of the cold end cooling, too much heat in the enclosure means that the cold end cannot cool enough to prevent the filament to melt prematurely; this can lead to clogs. Also note that the printer electronics board may become too hot too (too hot stepper drivers will cause steppers to miss steps).

\n\n
\n

I once tried a towel over a Ultimaker 3 Extended to print a difficult filament to keep some heat in the enclosure; I encountered the above\n problems when I was young(er) and inexperienced :) We learn by trial and error some times!

\n
\n\n

However, ABS, requires control over the temperature during printing. Draft or uneven temperatures may lead to problems related to layer separation or heat bed separation. Many of the high-end box printers have doors available to enclose the front, this results in an enclosure with an open top (sometimes even for the top there are covers available). The heat of the bed then heats up the enclosure. A constant temperature of 45\u00b0C should not be too high to cause problems. When boxing it up completely, you could use a temperature sensor to monitor the enclosure temperature and schedule a fan to ventilate when it gets too hot.

\n" }, { "Id": "6201", "CreationDate": "2018-06-19T11:53:31.143", "Body": "

I am building a AlfaMendel 3D printer (edited : The original question named it to be Prusa Mendel I2). I printed the extruder drive using PLA material from the STL available in the package but I'm not able to insert the hobbed drive as well as the filament in it. The dimensions of hobbed drive is correct.

\n\n

Is the problem related to shrinking of print material while printing the extruder drive or something else.

\n\n

I have included few pics of the extruder drive and hobbed drive.

\n\n

\"Filament

\n\n

\"Extruder

\n\n

\"Close

\n\n

\"Hobbed

\n\n

This image shows the maximum possible insertion of hobbed drive in the extruder drive.

\n\n

\"Trying

\n", "Title": "Not able to insert hobbed drive and filament in the extruder drive", "Tags": "|prusa-i3|reprap|", "Answer": "

Your parts are not in the Prusa Mendel i2 as found here or here.

\n\n

Also note, quote:

\n\n
\n

Parts included in Github repository have the wrong dimensions for the\n nut traps! Download the correct version from [Thingiverse Greg's\n Hinged Accesible Extruder]

\n
\n\n

Are you sure you have the correct parts for the original Prusa Mendel i2? Maybe you have a different version? The original extruder mount uses an M8 hobbed bolt, not an MK8 hobbed extruder gear. The first is 8 mm in diameter, the latter is 9 mm in diameter.

\n\n
\n\n

Edit:\nAfter you updated your post to hint to the correct printer, it appears that my answer still holds. You did use the incorrect extruder gear. The 9 mm gear doesn't fit in the extruder part as it requires an 8 mm diameter extruder gear.

\n\n

It might be difficult to find a 8 mm hobbed gear with a 3 mm shaft diameter, they are usually 5 mm (as in the stepper shaft diameter) or 8 mm (as in the diameter of 8 mm bolts that are frequently used for extruder setups). You could redesign the part (load STL in a 3D CAD program and make the changes). By the way, PLA should not shrink so much. As can be seen in the photo's, it looks like your current printer does not print very accurately, that may also contribute.

\n" }, { "Id": "6204", "CreationDate": "2018-06-20T00:03:11.560", "Body": "

I have just assembled the Creality3D Ender-4 kit a couple of days ago.\nI completed some rewiring and everything seems to be wired correctly, I'm able to auto-home successfully, but this is the problem I'm having:

\n\n

I tried printing a calibration cube. \nThe first problem I noticed is that the printing is starting on one corner instead of the center as specified on the Printer Settings in CURA (check settings below).

\n\n

Second problem is that I'm getting significant distortion. The movement of the printer seems to be fine, no jerks of weird sounds. I tried all I know but I honestly don't know how to proceed with this. I have a couple of hours of experience in 3D Printing so I'm completely lost.

\n\n

This is the result I got (Model printed with Raft for adhesion):

\n\n

\"Model

\n\n

These are my CURA settings:

\n\n

Printer

\n\n

\"CURA

\n\n

Extruder

\n\n

\"CURA

\n", "Title": "How to calibrate Ender-4", "Tags": "|calibration|creality-ender-4|", "Answer": "

Fix was actually pretty simple.

\n\n

I just removed the printer that I had setup on CURA following the manual provided by Creality (found on the provided SD Card) and started from scratch.

\n\n

The one that worked was actually setting up the printer as a Creality CR-10 and then just changing the XYZ dimensions. This setup has the Origin at Center option unchecked as suggested in the comments. That's all it took...

\n\n

\"enter

\n\n

New results:\n\"enter

\n" }, { "Id": "6206", "CreationDate": "2018-06-20T18:27:14.750", "Body": "

I'm about to build myself a 3D-printer.

\n\n

After a long search for a brain for this one I came across the GT2560 from Geeetech, because it leaves a solid impression on me.\nThe manufacturer himself has a quite extensive wiki page for this board.

\n\n

My idea now was to use an ATX power supply.

\n\n

The PSU is a be quiet! BN257. On 12 V (combined) it can deliver 324 W. On 12V1 there is 20 A, on 12V2 16 A.

\n\n

Would that be enough or would you rather use a 24 V power supply?

\n\n

Since I have read some reports about the Anet A8 and its danger of overloading, I am a bit uncertain, even if the GT2560 looks a lot better than the A8.

\n\n

Are the connections to which the heating bed and the extruder are connected sufficiently large to withstand 10 A and 15 A respectively, or should I solder the cables directly to the board from the very first?

\n\n

According to the information I received from Dr. Search Engine, MOSFETs of type STB55NF06 are used. According to the data sheet, these can withstand loads up to 60 V/55 A. I don't necessarily need to replace them with external ones, do I?

\n", "Title": "Geeetech GT2560: Electronic safety and power source", "Tags": "|printer-building|electronics|", "Answer": "

Whether 350 Watt is enough depends on the amount of Ampere can be generated over the 12 V lines. Computer power supplies add up the power of all the voltages! A nameless or cheap 350 Watt PSU may not deliver enough power (in the sense that they may never reach the given power), you should check that out. The high-end PSU's are usually well fabricated and can usually deliver more power (but should not be taken for granted!).

\n\n\n\n

This totals to about 200 Watt, which your PSU should be able to generate without a problem.

\n\n

Those green connectors plug in and out of the board don't they, usually with those small pins, transporting up to 10 Amperes is not recommended, you should look up the ratings of those connectors.

\n" }, { "Id": "6214", "CreationDate": "2018-06-23T14:41:38.567", "Body": "

I have an old laser engraver that runs from Windows 98SE and DOS-6. Yup, still works fine but a pain as I regularly have to open it up and unplug and re-seat all the ribbon connectors. Fifteen of them and I tire of that fast.

\n\n

I am thinking of pulling out all the electronics and installing a TB6600 stepper drivers and Arduino to run G-code. My order of desire is based on price, RAMPS, Smoothie and the AWC708C. I'd actually like to stay away from the AWC as it is closed source and I would be stuck with whatever it has.

\n\n

With the existing NEMA 17 steppers I can get rapids to 400 mm/s. Will a RAMPS system drive this fast enough? It doesn't have to be that fast, but close to and certainly not below about 200 mm/sec.

\n\n

I have been searching but cannot find any actual figures achieved. If not then I guess my second option is the smoothie.

\n", "Title": "Laser Engraver with Smoothie, RAMPS 1.4 or AWC708C?", "Tags": "|g-code|ramps-1.4|laser|", "Answer": "

Marlin supports a stepping frequency of up to 40kHz. For a pretty typical setup with 100 steps/mm, this translates to 400 mm/s - obviously, if you use higher microstepping settings or use finer pitched belts you will get a lower maximum speed.

\n" }, { "Id": "6217", "CreationDate": "2018-06-24T11:40:27.047", "Body": "

I was reading this research paper titled Sub-modeling Finite Element Analysis of 3D Printed Structures. In this, firstly, the author is trying to create a sketch for Engineering analysis using the 3D printing path coordinates and integrating it to a CAD software like Autodesk Inventor.

\n\n

It says,

\n\n
\n

By analyzing the corresponding G-code for the desired structure,\n important information can be extracted, such as the coordinated of the\n 3D printing path, key points, paths of printing and non-printing\n paths.

\n \n

...

\n \n

The coordinate of the 3D printing path can then be imported into CAD software to obtain the corresponding sketch and consequently a solid\n body for each layer. Most of commercial CAD software packages are\n capable of this task. For this Purpose we have chosen Autodesk\n Inventor.

\n
\n\n

Paper is attached here: Research Paper in subject

\n\n

Can anyone help me out with how this can be done?

\n", "Title": "Linking the 3D printing path coordinates to CAD for modelling", "Tags": "|3d-models|3d-design|g-code|fdm|", "Answer": "

The wording in the paper is quite verbose and somewhat unclear. All it says is they read the G-code file and somehow turn it into a 3D model.

\n\n

A g-code file is just a list of linear moves. Here is an example snippet I took from a random file (keep in mind a typical file would consist of thousands of such lines):

\n\n
G1 X140.621 Y114.840 E0.0065\nG1 X140.804 Y114.765 E0.0129\nG1 X141.016 Y114.737 E0.0199\nG1 X158.984 Y114.737 E0.6070\nG1 X159.196 Y114.765 E0.6140\n
\n\n

Each move is relative to the previous, so the second line of the code (for example) tells the printer to move to X=140.804 and Y=114.765 from the previous position (X=140.621, Y=114.840) while extruding an amount of material equal to 0.0129-0.0065=0.0064 mm of filament.

\n\n

It appears that the authors have developed a toolchain to turn a G-code file into a 3D model, translating every extrusion segment into a part of a solid body (from the pictures, it appears that for a given move segment, they create an ellipsoidal extrusion and merge all of these together into a single solid body) - see Figure 9 in the paper.

\n" }, { "Id": "6223", "CreationDate": "2018-06-24T23:48:25.937", "Body": "

I have configured slic3r's OctoPrint Upload section with the with hostname (\"octopi\") and port (80) of my OctoPrint server. The Test button indicates a successful connection.

\n\n

But when I use \"Send to Printer\" I get an HTTP error 302.

\n\n

How can I fix this?\n\"Error

\n", "Title": "Why does slic3r get an error \"302 moved temporarily\" when uploading to OctoPrint?", "Tags": "|slic3r|octoprint|", "Answer": "

Specific answer: use \"octopi.local\" rather than \"octopi\", since that will properly resolve to the correct Octopi IP Address.

\n\n
\n\n

More generally, investigating the network traffic on my network (AT&T Fiber Home) revealed these facts regarding the octopi server:

\n\n\n\n

And these facts regarding the slic3r configuration:

\n\n\n\n

So in this particular case, entering \"octopi.local\" instead of \"octopi\" corrected the error. If you receive a 302 error, you will need to research and verify the exact IP address or host name for the target Octopi system.

\n\n

A future version of slic3r should probably report an error on the \"test\" button if it doesn't (a) get a proper 200 return code and (b) return the proper version information as slic3r is requesting.

\n" }, { "Id": "6225", "CreationDate": "2018-06-25T00:34:15.073", "Body": "

I'm trying to get my printer to auto level its bed, with Marlin firmware using the following:

\n\n\n\n

I go into the menu on my LCD, and go to the prepare screen. The screen has the following options:

\n\n\n\n

When I select the \"Auto home\" function, the printer uses the stops at the top of the delta to zero itself.
\nWhen I select the \"Level bed\" function, the printer once again uses the end stops to level itself.
\nIf I turn the Bed leveling to ON, then the Level Bed function will do the exact same thing.

\n\n

My auto-leveling device is connected to the Z-axis end-stop, and looks like so:

\n\n

\"Auto-leveling

\n\n

I have the #define AUTO_BED_LEVELING_LINEAR line in my config.h, as well as the #define Z_MIN_PROBE_USES_Z_MIN_ENDSTOP_PIN line. I'm not sure why the printer doesn't actually lower the head to the plate and press the endstop.

\n\n

Any clues as to how to make the printer level the bed and not home the XY axis?

\n", "Title": "Marlin Firmware Auto Leveling only re-centers the XY-axis (Delta printer)", "Tags": "|marlin|delta|bed-leveling|", "Answer": "

You are correct to define the constants AUTO_BED_LEVELING_LINEAR and Z_MIN_PROBE_USES_Z_MIN_ENDSTOP_PIN, be sure to define the following also:

\n\n
#define Z_MAX_POS MANUAL_Z_HOME_POS\n
\n\n

and

\n\n
#define MANUAL_Z_HOME_POS 300 // Distance between the nozzle to printbed after homing\n
\n\n\n\n

Please look into the configuration files of this Kossel Linear Plus and compare them with your own configuration files, you may have missed something.

\n" }, { "Id": "6230", "CreationDate": "2018-06-25T16:19:52.930", "Body": "

I have been given a proprietary 3D file of a real world item from an item manufacturer with the obligations not to hand it out to anyone to produce a print in a low single digit percentge scale of the object. The object model I was handed was given in STL format, and contains even the tiny geometry of the internals. The first set of files given did come with walls that became too thin to be printable in some areas, and I had to ask them to thicken these parts, which they did.

\n\n

Now, these files still contain tons of redundant internal geometry), like bolts that would be M20 or something IRL. The technical design is actually produced in a 1:1 scale - and thus much of the item's complexity is retained in scaling and slicing, because I was provided with neat, nested shells that don't intersect at all. A tiny piece of internal geometry after slicing for illustration without giving much about the actual object (which is under DND).

\n\n

\"enter

\n\n

ALL of this internal is superflous, waste of material and additional print time.

\n\n

The STL of this item contains (according to Meshmixer) about 40 shells, most of them representing single bolts, while the really relevant parts are all in the shell 1. The really problematic part is though, that even ditching these 39 superfous shells, the space reserved for them and a lot of internal geometry is left behind when I try to get rid of the internal geometry.

\n\n

Saturday I tried for hours to simplify the model first and then stitch away tons of the internal geometry with blender, but while this did reduce print time greatly for the internal cavities were gone, the simplification did mess up other parts of the model to a degree that wasn't nice anymore and it messed up the scaling. Also, it was very time consuming - 6 hours or more - which makes this a very bad time-effect ratio. The print with the retaining 2 % of the vertices was... ok, but not as nice as with the half million vertices from the original file, mainly because I had to simplify the model first to even have a chance to see into the model to get the internal cavity vertices grabbed and removed/merged, before fixing any holes left from the mercyless treatment by Meshmixer's auto-repair feature.

\n\n

Is there a way to analyse a model for internal structure and remove them for slicing without having to remodel a 500k vertex / 50 MB object in its full? It would be best if such a way was somewhat automated.

\n\n

My setup usually uses Cura as a slicing engine, my modeling softwares of choice are - in this order - Fusion360 or DesignSpark Mechanical. If nothing else helps and I have to attack singe vertices I do my way around Blenderauto-updated via Steam. Meshmixer I usually use only to fix up models before slicing.

\n", "Title": "Easy way to refine a 3D-model for 3D printing by removing internal geometry", "Tags": "|3d-models|slicing|", "Answer": "

About 10 minutes after writing the question, it suddenly dawned on me, that I was not using Meshmixer to its full potential, and especially not a simple property of Cura:

\n\n\n\n

Now, these two parts can be used to get rid or complex internal geometry by intentionally setting it up an intersecting shell. Usually intersecting shells are a no-no for good 3D design, but by having something - for example a cylinder - intersect just the internal geometry and enclosing it, these parts vanish in the slicing, if the slicer is set to fix intersecting shells by ignoring internal geometry. Cura does so with a simple setting, one that seems to be on by default in the 3.3.1 distribution.

\n\n

So the odd solution to how to reduce a model's internal geometry without impacting the outer look can be this at times:

\n\n\n\n

To illustrate, almost the same area of the object before and after the added cylinder:

\n\n

\"enter \"enter

\n\n

As one can see, the structure became much simpler, as the cylinder cuts away all the internal structure (the 'spokes' and 'axle' one might want to identify from the left picture, but that would be a misidentification) is gone. Much less internal geometry is retained and instead it is now filled with a cylindrical space of nice and fast(er) to print infill, here \"Quarter Cubic\".

\n\n

While this is just a partly automated solution - demanding the manual addition of the intentional intersecting shell - I am yet to be taught about a fully automated way.

\n" }, { "Id": "6244", "CreationDate": "2018-06-28T00:14:49.520", "Body": "

I'm trying to print a model that looks like this:

\n\n

\"enter

\n\n

at a size of approximately 10\"x7\"x7\". I've tried 3 times now, with various different infill and shell settings, and had it fail all 3 times in the same way at the same point: right at the point where the central column begins to angle back (which is a bit higher than where the two side columns angle back--those angles print just fine) the whole thing ends up detached somehow and I end up with messes like this:

\n\n

\"enter\n\"enter

\n\n

How can I figure out what's going wrong with this print? I've already wasted a non-trivial amount of time and filament on this, and it's starting to drive me up the wall. Somehow, I've got the wrong settings to make this geometry print, but I can't seem to find any settings that actually work.

\n\n

Using the Raise3D N2 Plus dual-extruder printer.

\n\n

EDIT: WRT questions raised in the comments:

\n\n

The extruder isn't clogging. It's continuing to print; it's just that at the point where the central column angles back, the whole thing separates and I end up with a distinct layer that prints straight up and down rather than angling inward.

\n\n

I'm using PLA at 215\u00b0C and 3 wall layers.

\n\n

This problem seems to be specific to this model; I've been able to print other large things before without problems.

\n\n

Never heard of MeshMixer, but the reviews on the download page make it appear to be a very buggy and wouldn't work at all, at least in the current version, and it requires a registration to download even though it's nominally free, so I think I'll pass on this one.

\n", "Title": "What is wrong with this angle?", "Tags": "|software|", "Answer": "

Hex infill patterns are normally chosen for strength, as the honeycomb resists force in many directions. However, hex infill patterns are slow to print and the older, simpler fill patterns print faster and provide sufficient support for solid architectural models.

\n\n

Models with shallow roof angles of less than 45 degrees are challenging to print and often result in \"air prints\" where unsupported filament cascades into a sorry tangle of sadness. Shallow roofs are challenging because each horizontal filament overlaps very little with the preceding adjacent filament. This is where infill proves critical, since the infill supports these filament bridges as they cross each infill line segment.

\n\n

Slicing software has only recently introduced the hex fill pattern because the code to print a hex infill is VERY complicated. You can see this in the picture as all those fine lines of retracted filament scattered throughout the hex infill. Earlier, simpler fills such as diagonal lines provide simpler longer paths for the slicer to implement. Bridging works best at speed over many supporting points. Excessive retraction causes the extruder to \"stutter\", and makes bridging difficult.

\n\n

It may therefore help to choose a simpler fill pattern for printing this model and reserve hex infill for simpler models requiring utmost strength.

\n" }, { "Id": "6247", "CreationDate": "2018-06-28T13:24:45.037", "Body": "\n\n
\n\n

With respect to this question: Anet A8 reading 739°C from the extruder thermistor!

\n\n

Having read the issue, it seems that this is a common problem for the ANET3D board. I'd like to ask two questions:

\n\n
    \n
  1. Is it possible to sever connections to the ATMEGA and use the hardware itself while controlled with the Raspberry Pi or some other dev board?

  2. \n
  3. While waiting for a new board (seems to be the only cost/time effective choice), is there any way to use the other temperature circuit (likely having to do some work in Arduino IDE)?

  4. \n
\n\n

My hot end decided to set itself to 265\u00b0C while my bed is reading the new thermistor on the new Hot End just fine. I need to get a few prints made while waiting on funds to replace the board for full functionality.

\n\n

I figure #1 is too much to hope for, but I have to ask. So, if you swap the heater circuits, what do you have to modify in configuration.h to convince the firmware to accept the modified input?

\n", "Title": "How to use an Anet A8 control board with a damaged AVR IC?", "Tags": "|extruder|anet-a8|", "Answer": "

This post was taken from Tooniis's comment.

\n\n
\n\n

I've been doing #2 for a week now. I swapped ports of the nozzle and hotbed. Now I have a functional nozzle but the bed cannot be heated. The new hardware should arrive soon though. As for #1, it would be very hard to do since the ATmega chip is an SMD package.

\n\n

In sanguino.h there are two lines which define the pins for the hotbed thermistor and the nozzle thermistor. One of them is 6 and the other is 7, and I just swapped those two. The lines are next to each other.

\n" }, { "Id": "6260", "CreationDate": "2018-06-29T10:25:09.957", "Body": "

I just re-ran all basic calibration steps from the Original Prusa i3 MK2 Manual.

\n\n

Now, when doing the first layer calibration, lines that are running in positive X direction are ok, while those running in negative X direction are severely squished.

\n\n

\"Print\n(The \"waviness\" of my print bed is an artifact of the camera lens distortion of my smartphone)

\n\n

I already did Bed level correction, so each line is exactly the same width over its entire distance and tried to raise the live-adjust Z, but that leads to the thin lines not adhering at all. My printer is 100% stock, I modified nothing about it.

\n\n

What can I do to troubleshoot this further?

\n", "Title": "Line width changes depending on the direction the print head moves", "Tags": "|prusa-i3|print-quality|fdm|calibration|troubleshooting|", "Answer": "

Due to the advice from @0scar i formulate my answer in the comments as a proper answer ;)

\n\n

Check the nozzle, if it is in shape and good condition.\nI scratched my nozzle over the metal heat bed by accident...it becomes nasty... and produced inconsistent line widths. Since the metal of most nozzles is brass it is easy to machine...and easy to deform. And since you will need several nozzles over time get some new ones and try with them.

\n\n

Also not every new nozzle will be perfect, so defect products happen, especially if they are cheap and have really tight measurements.

\n" }, { "Id": "6266", "CreationDate": "2018-07-01T02:09:53.910", "Body": "

How can I detect if a generated STL model will need to be printed with supports?

\n\n

Context: I have a pipeline which parametrically generates OpenSCAD models, generates the STL, and sends them to the printer. I would like to (a) automatically detect the cases where supports are necessary so that I can specify the appropriate flags to the slicer, and (b) attempt to rotate the generated model so that supports won't be necessary.

\n", "Title": "How can I tell if an STL model will need supports?", "Tags": "|support-structures|stl|algorithm|", "Answer": "

You could try to:

\n\n
    \n
  1. Slice with slicer (Cura in my case) with support enabled.

  2. \n
  3. Search for text: TYPE:SUPPORT

    \n\n

    G0 F1800 X237.873 Y184.24
    \nG0 X233.869 Y183.237
    \n;TYPE:SUPPORT
    \nG1 F1500 E562.81355
    \nG1 F900 X233.579 Y183.939 E562.91577
    \nG1 X233.368 Y184.67 E563.01816

  4. \n
  5. If it exists, then try to call it again:

  6. \n
  7. Use auto-orientation plugin to validate if there is a better (no support) model position.
  8. \n
\n\n

As an alternative, you could scan the mesh and looks for an angle greater than 45 or 50 degrees.

\n" }, { "Id": "6267", "CreationDate": "2018-07-01T02:19:17.693", "Body": "

When generating STL files with slic3r I would like to automatically prefix the filename with strings such as PLA or PET before uploading to my printer. How can I specify this in the filament settings?

\n", "Title": "Slic3r: How can I automatically modify the STL filename?", "Tags": "|slicing|slic3r|", "Answer": "

The filename is under the \"Print settings\" tab. Under output options, you can change the name template. You can use words like: [layer_height] to make the name dynamic.

\n\n

The variable you are looking for here is: [filament_preset]\nBut the full name, with space and everything, is used.

\n\n

Here is a screenshot:\n\"output

\n\n

Here is a link to fuller docs:\nhttps://github.com/slic3r/Slic3r/wiki/FAQ#how-can-i-specify-a-custom-filename-format-for-output-g-code-files

\n\n

New version of Slic3r

\n\n

It lets you edit the filename when upload it. You will see what will be upload it and give it any name. But this is a manual process. Just updated my answer for the sake of completeness.

\n" }, { "Id": "6269", "CreationDate": "2018-07-01T20:19:23.797", "Body": "

I recently had a series of issues with my prusa MK3 that kicked off when I tried to print with Proto-pasta's glitter filament using a 0.4 mm nozzle. The glitter clogged up and cleaning it out was pretty difficult.

\n

A different user here pointed out that 0.4 mm was too small for glitter filament. When I initially read (and, after the clogging fiasco, re-read) the Proto-pasta website I didn't find any information suggesting that a larger nozzle borehole diameter was necessary.

\n

I have the glitter and metalic green filaments. Do I need a nozzle with a larger diameter to print these or not? I emailed Proto-pasta about it the other day but have yet to hear back.

\n

Update

\n

Here's the reply I got from proto-pasta on these questions:

\n
\n

Thank you for following up. Yes cold pulls that leave behind material can cause all sorts of trouble. It happens to me on the Lulzbot quite often, probably my biggest gripe about that machine. Anyways, on to your questions:

\n\n
\n", "Title": "What nozzle size should I use for filament containing glitter?", "Tags": "|filament|filled-pla|", "Answer": "

Fact is that you encounter clogs with this filament, so trying a larger diameter nozzle is an option to solve this. Nozzles are very cheaply found in various sizes, so buy a few and experiment. Commonly found larger nozzle sizes are 0.5, 0.6 and 0.8 mm nozzles, even larger nozzles exist, like e.g. 1.0 mm or even larger, but keep in mind that the hot end needs to keep up heating of the extruded filament, so deposition speed may need to be reduced for larger nozzle diameters.

\n\n

Alternatively, you could fight clogs the usual way by playing with temperature, layer height, retract settings, oiling filament, purging nozzle from previous filament or cooked up residue, etc, etc.

\n\n

Quoting someone's experience:

\n\n
\n

As far as differences, here has been my experience:

\n \n \n
\n\n
\n\n

Edit: According to Proto-pasta, concerning filament with glitter, a 0.4 mm nozzle should not be a problem as the glitter particles are smaller, the print layer thickness definitely can be a problem. However, they state that a larger diameter nozzle will result in more sparkle as the glitter is not laid too flat.

\n" }, { "Id": "6277", "CreationDate": "2018-07-03T19:32:30.193", "Body": "

My G29 command reports

\n\n
+0.178 +0.281 +0.830\n-0.614 -0.012 +0.371\n-1.208 -0.849 -0.351\n
\n\n

So should I tighten up the screw of the bed, close to 0,0 position or loosen it?

\n\n

I have a feeling that when I loosen it, it gets away from zero and I expect the opposite to happen.

\n\n

For bed leveling i use a capacitive probe and after playing around with the screws here is the result

\n\n
+0.406 +0.127 +0.411\n-0.161 -0.007 -0.041\n-0.572 -0.652 -0.668\n
\n\n

Finally the leveling process was found here\nBut the question remains. The value -0.572 corresponds close to 0,0 ?

\n", "Title": "BiLinear bed leveling", "Tags": "|marlin|bed-leveling|", "Answer": "

An additional suggestion: after levelling with sensors or mechanically as in Oscar's great answer, verify both the level and the Z-zero by printing a single-layer pattern such as a few concentric squares or circles. Inspect to verify that no location is too high (material fails to extrude) or too low (material doesn't adhere, or appears much thicker than elsewhere).

\n\n

By doing this you can avoid setting up a multi-hour print only to discover a 'bad corner' or some such.

\n" }, { "Id": "6279", "CreationDate": "2018-07-04T16:54:07.093", "Body": "

I just received an old 3D printer from one of my school teachers. I have no idea whatsoever as to which brand it is, no instruction manual attached to it, or any other info about it.

\n\n

How can I find some information about it?

\n\n

Some links would be very useful. Remember when giving advice that I know nothing about 3D printers.

\n\n

This is the printer:

\n\n

Backside\n\"Printer

\n\n

Front\n\"Printer

\n\n

The X-axis stepper\n\"The

\n\n

The electronics board\n\"The

\n", "Title": "Very old 3D Printer, which brand or type is this?", "Tags": "|troubleshooting|", "Answer": "

Here are some further details on:

\n\n\n\n

Control board

\n\n

The electronics board has this marking:

\n\n

\"EJE

\n\n
EJE Electronics Gubbels Engineering\n
\n\n

The serial number is 0070-003

\n\n

According to this site Xinchejian First Mendel V2 Reprap the board is:

\n\n
\n

Gen6 Electronics, with AT Mega 644p processor (PCB EJE Electronics, Gubbels Engineering - mendel-parts.com)

\n
\n\n

It seems to be this board (the serial numbers match):

\n\n

\"Gen6_PCB_final_batch\"

\n\n

Information on this board can be found here: Generation 6 Electronics:

\n\n
\n

Generation 6, or Gen6, can be described as a dedicated, Plug-&-Play, single board solution for FFF/FDM 3D Printers. It is designed to be professionally manufactured, with many small surface mount components, as opposed to Generation 7 which is designed to be printed on a Mendel(among other design goals).

\n \n

Another major difference with Gen6 electronics is that they use Texas Instruments DRV8811 chips to drive the stepper motors. This means they require firmware modifications from the normal Polulu-based electronics which use Allegro A4983 chips.

\n
\n\n

It goes on to say:

\n\n
\n

Benefits of this Design

\n \n \n \n

Hardware Features

\n \n \n \n

Specifications

\n \n \n
\n\n

There is a lot more information on that page, including information about the power supply, USB, End stops, Heaters, Motors, Firmware, etc. I suggest that you read it fully, in order to understand the board's functionality.

\n\n

Stepper Motors

\n\n

The stepper's model number can be seen here:

\n\n

\"Stepper

\n\n

The stepper's model number is SY42STH47-1683B,

\n\n

which is a NEMA-17 High Torque Hybrid Stepper Motor, with these specifications:

\n\n\n\n

This would be, at least, part of the datasheet:

\n\n

\"Stepper

\n\n

Here are a couple of links, should you need replacements (~\u20ac17):

\n\n\n\n

This stepper motor is also listed on the RepRapWiki - NEMA 17 page:

\n\n
Model           Holding Torque  Rated voltage       Shaft   Step angle  Motor length    Rated current   Inductance\nSY42STH47-1684B   43.1 N\u00b7cm        2.8 V       \u00d8 5 mm double    1.8\u00b0        48 mm            n/a            n/a\n
\n" }, { "Id": "6281", "CreationDate": "2018-07-04T21:32:04.370", "Body": "

I have an Anet A8, and recently updated the firmware to Marlin 1.1.8.\nMy only change was replacing the configuration files by the Anet A8 sample files built in with the firmware zip file.

\n\n

Everything works fine but, sometimes, right after finishing a printing (while doing the \"home all\"), the LCD shows some weird data, like this:

\n\n

\"LCD

\n\n

The same happens when push the \"confirm\", after the mesh bed leveling.
\nI'd like to know why it happens. Is it a Marlin bug?

\n", "Title": "Anet A8 LCD shows garbage data after done printing", "Tags": "|marlin|anet-a8|", "Answer": "

I had this issue for a long time and finally I tried a super-simple hack. Attach a testlead from bed to PSU (which is grounded). Problem solved!

\n\n

In other words: The Marlin firmware may be more \"sensitive\" than the original stock firmware, but it is not really a firmware issue.

\n\n

\"enter

\n" }, { "Id": "6289", "CreationDate": "2018-07-05T17:35:22.243", "Body": "

I'm working on building a tool to generate G-code (a simpler slicer), and I'm trying to calculate how much filament should be extruded per movement.

\n\n

Is there a standard calculation for this? Something like:

\n\n
layer height * flow % * extruder diameter * distance \n
\n\n

How does Ultimaker Cura calculate this?

\n", "Title": "How is the E argument calculated for a given G1 command?", "Tags": "|ultimaker-cura|extrusion|", "Answer": "

Basically, all movements are (small) straight lines, the volume of a straight line is easily calculated as you already guessed.

\n

To calculate the volume to be extruded you multiply the following parameters:

\n\n

With this volume you can calculate how much filament you need to extrude. To get the length (thus the length defined by the E parameter), divide the obtained volume by surface area of your used filament by:

\n\n

To sum up, the value of E is given by:

\n

$$ E_{value} = \\frac{h \\times {SF} \\times d_{n} \\times l}{\\frac{\\pi}{4}d_{f}^2} = \\frac{4 \\times h \\times {SF} \\times d_{n} \\times l}{\\pi \\times d_{f}^2} $$

\n" }, { "Id": "6298", "CreationDate": "2018-07-06T11:30:53.763", "Body": "

In the creation of the process of printing my own bearings to reduce noise, minimize play/tolerances and size the bearings to the actual size of the application, I obtained a spool of Polyoxymethylene (POM) or also known as Acetal or Delrin.

\n\n

\"Spool

\n\n

This polymer is a thermoplastic polymer that is frequently used in engineering precision parts that require high stiffness, low friction and dimensional stability. It has been chosen for these material properties to be used as bearing material for linear guide rails.

\n\n

This image shows an example of the application of customized igus® bearing that is as long as the housing part it is going to fit in:\n\"customized

\n\n

Prints frequently get knocked over as it does not stick well.

\n\n

How do I get POM filament to stick to the build plate?

\n", "Title": "POM filament not sticking to the build plate?", "Tags": "|filament|heated-bed|adhesion|pom|", "Answer": "

Great material but very hard to print as it does not stick easy to the build plate as it has a low friction coefficient to grip onto the heated bed. Also, the material sets quite fast, once the filament leaves the nozzle, it soon hardens so you need to be careful with retraction and Z-hop (leaving small peaks that will be hit later by the nozzle knocking over your print).

\n\n

My experience with printing this material is based on printing with an Ultimaker 3 Extended (on glass) using a modified material profile (based on Nylon). It is printed at 240 \u00b0C (+5 \u00b0C for the first layer), no usage of the part cooling fan (if you do use cooling, the layers will not adhere well), a heated bed temperature of 80 \u00b0C, and a slow printing speed of 40 mm/s (20 mm/s for the first and second layer).

\n\n

Ultimaker 3 Extended with printed POM bearing:

\n\n

\"Ultimaker

\n\n

First thing I learned is to use quite large brim's (the image above shows a rather small one for this short bearing, for the longer bearings the brim size was more than doubled, e.g. 20 mm), brims enlarge the surface area so that there is more material that grabs hold of the heated bed. Also ensure to get it to stick long enough is heating the bed to 80 \u00b0C after smearing ABS juice (ABS dissolved in acetone) and spraying a PVA based spray over the dried ABS juice layer (3DLAC has been used, but other hairsprays or even gluesticks may work as well, as long as there is PVA in it). The temperature of 80 \u00b0C is chosen as this is the temperature where my PVA spray has the most tack/sticky-ness.

\n\n

Other solutions like heating the bed up to 110 \u00b0C using 3DLAC or specifically designed sprays for higher temperature like e.g. Dimafix did not work well for me.

\n\n

Note that complete infill may also give problems as filling out the whole surface area sometimes creates excess material that curls up, which is an easy target to be hit by the nozzle on the next layer.

\n\n

This image shows an example of using the printed customized igus\u00ae POM bearing that is as long as the mount height:

\n\n

\"Prusa

\n\n

Collection of POM printed bearings:

\n\n

\"Collection

\n\n

Another application of POM bearings:

\n\n

\"Bowden

\n\n

CoreXY hot end carriage

\n\n

\"Hypercube\"Hypercube

\n" }, { "Id": "6315", "CreationDate": "2018-07-07T08:49:43.057", "Body": "

I came across several issues which seem to have been lowered.\nFirstly, I changed from a 0.4 to a 0.5 mm nozzle. Because of the backpressure I was not able to print my PETG (Colorfabb XT filament) below 270\u00b0C which caused unresolvable oozing. After that I was able to extrude till 230\u00b0C.

\n\n

The left print below shows the result. I disassembled the hotend, there was no leak or whatsoever. Maybe it was too cold for printing. However, the temperature displayed was 250\u00b0C. Then I replaced the cheap aluminum heat block with a copper alloy based one. After that my PIDs did not work anymore. I had to greatly enhance the d-term, otherwise there was a big overshoot. Guess there was a serious heat conducting issue with the old BQ hotend.

\n\n

\"Prints

\n\n

Anyway, from there it became better. However, I noticed that I still have severe underextrusion after travel moves (second piece, first picture, first piece, second picture). I use Cura, so I activated retraction with the feature to prime after travel moves. I got the wall closed just after 0.35 mm\u00b3.

\n\n

\"Prints

\n\n

My Question: Is this underextrusion after travel moves normal for PETG/XT? I did not discover such behavior with PLA or ABS in the past.

\n\n

Current Site Advice: Despite the weight, copper heat blocks seem to be worth the upgrade.

\n", "Title": "Underextrusion after travel moves with PETG/Colorfab XT", "Tags": "|extrusion|colorfabb-xt|", "Answer": "

Zero retraction is just a special (worst) case of under-retraction. You'll pretty much always lose material to oozing if you don't retract before travel. At best this ends up hidden inside the print (but can still affect weight and weight balance); usually it'll also harm the surface.

\n\n

Ensure that retraction is set to always happen (not skipping short travel), and at least 5-6 mm for bowden extruders. Direct extruders can get away with less but I'm not sure exactly how much less. Less-rigid materials need more retraction to make up for compression of the filament between the extruder gear and the nozzle.

\n" }, { "Id": "6316", "CreationDate": "2018-07-07T09:31:29.617", "Body": "

I'm thinking of buying an Arduino WEMOS1 as I am tired with SD card fiddling.

\n\n

\"Arduino

\n\n

As the ESP8266 chip has 32 Mb (~4 MB) flash memory, I am hoping to be able to push most of the G-code files over the Wi-Fi.

\n\n

I was trying OctoPi but, having electronic noise issues, the transmission was not stable.

\n\n

Has anyone tried WEMOS with RAMPS? Is the serial communication stable between ESP and Arduino?

\n\n
\n\n

1 Product description is Mega +WiFi R3 ATmega2560+ESP8266 (32Mb memory), USB-TTL CH340G. Compatible for Arduino Mega, NodeMCU, WeMos ESP8266

\n", "Title": "Arduino WEMOS and RAMPS", "Tags": "|diy-3d-printer|ramps-1.4|", "Answer": "

After having looked into them, these boards certainly seem to be rather interesting.

\n\n

There appear to be two varieties of this board, as I have seen photos with either RobotDyn:

\n\n

\"RobotDyn

\n\n

rear (from Instructables)

\n\n

\"RobotDyn

\n\n

or WeMos logos on them.

\n\n

\"WeMos

\n\n

They may, or may not, be the same. There is also this one, with just a generic \"WiFi\" logo (from the eBay page below):

\n\n

\"Generic

\n\n

Memory Discrepancies

\n\n

However, there is certainly some discrepancy relating to the specification and amount of memory (unless there are a number of variants - which would certainly be possible):

\n\n\n\n

Looking at this article, Instructables - Arduino MEGA 2560 With WiFi Built-in - ESP8266, it isn't exactly the ESP8266-12:

\n\n
\n

In today's text, we discuss an Arduino that I consider extremely special, as it has an ESP8266 embedded in its board. It doesn\u2019t have the ESP12 soldered onto the board. Instead, it has the Espressif chip. So, on the board you have the built-in Tensilica chip with 4MB of memory, along with the ATmega2560, which is the traditional Arduino Mega.

\n
\n\n

So the ESP has 4 MB, but where does the 32 Mb come in to play?

\n\n

Well, looking at this RobotDyn memory table (taken from the Instructables page):

\n\n

\"RobotDyn

\n\n

The 4 MB of the ESP appears to have been immediately contradicted and upgraded to 8 MB! The ATmega2560 somehow has 32 Mb (which seems a bit of a lie - as there isn't an ATmega2560 known to man that has that much memory2), and is confused further by the corresponding text:

\n\n
\n

As we see in the table above, the ATmega has 32MB of memory, not counting the ESP memory. This is wonderful, since the traditional Mega Arduino has only 256kb of memory.

\n
\n\n

So MB or Mb? Is there an additional flash IC external to the ATmega2560? Looking at the schematic, taken from the Russian RobotDyn page, there doesn't appear to be:

\n\n

\"RobotDyn

\n\n

So this could be nonsense. Logic would dictate that the ATmega has 256 kB and the ESP has 4 MB... But I could be wrong..?

\n\n

Links from \u041a\u043e\u043d\u0442\u0440\u043e\u043b\u043b\u0435\u0440 RobotDyn Mega 2560 R3 + WiFi ESP8266b, flash \u043f\u0430\u043c\u044f\u0442\u044c 8Mbit, USB-TTL CH340G, MicroUSB:

\n\n\n\n
\n\n

Reliability of communication

\n\n

Regardless of the actual spec, to address your question of ESP/Arduino communication reliability:

\n\n

As the ESP8266 and the ATmega2560 are on the same board one would assume that the (serial) communication between the two would be faultless. However, some issues have been reported3.

\n\n

Nevertheless, from your question, it seems that you were having issues with the ESP to the OctoPi communication, rather than ESP to Arduino, so maybe this doesn't actually answer your question.

\n\n

Also, as to their compatibility with RAMPS, there is very little information available currently, and it would seem to be a case of try-it-and-see. The closest I got was this post on the thread, OnStep and RAMPS1.4, but it seems to be spam.

\n\n

However, it certainly sounds worth a gamble as it would be a nice combination of technologies. However, I would imagine that some modification of the firmware would be necessary.

\n\n
\n\n

Useful Notes

\n\n\n\n
\n

To upload the sketch to the board:

\n\n
5, 6, 7 ON\nRXD3 - TXD3   \n
\n \n

To run the sketch:

\n\n
5 and 6 ON\nRXD3 - TXD3\n
\n
\n\n\n\n
\n\n

Footnotes

\n\n

1 From WeMOS Mega + WiFi R3 ATmega2560 + ESP8266 USB-TTL For Arduino Mega NodeMCU

\n\n
\n

WeMOS Mega + WiFi R3 ATmega2560 + ESP8266 USB-TTL For Arduino Mega NodeMCU \u3000

\n \n

Features:

\n \n \n \n

Table DIP- Switch:

\n\n
Connection                                                          DIP\n                                           1        2        3         4       5         6        7\nATmega2560<->ESP8266                      ON       ON       OFF       OFF     OFF       OFF      OFF\nUSB <->ATmega2560                         OFF      OFF      ON        ON      OFF       OFF      OFF\nUSB<->ESP8266 (Update firmware or sketch) OFF      OFF      OFF       OFF     ON        ON       ON\nUSB<->ESP8266 (communication)             OFF      OFF      OFF       OFF     ON        ON       OFF\nAll independent                           OFF      OFF      OFF       OFF     OFF       OFF      OFF\n\nSpecial solution:\nUSB <->ATmega328<-> ESP8266\nConnection                                                          DIP                                     SWITCH 2\n                                           1        2        3         4       5         6        7\n\nUSB <-> ATmega2560<-> ESP8266             ON       ON       ON        ON      OFF       OFF      OFF         To RXD3/TXD3\n
\n \n \n \n

Size: 10.5x5.3cm/4.13x2.09inch

\n
\n\n

2 From the Atmel ATmega640/V-1280/V\n-1281/V-2560/V-2561/V datasheet

\n\n
\n

\"ATmega256

\n
\n\n

3 From Mega + WiFi R3 ATmega2560 + ESP8266 (8 Mb m\u00e9moire), there do appear to be some issues communicating between the ATmega and the ESP ICs, with a number of users reporting problems.

\n\n

However, a couple of solutions are provided:

\n\n

Solution 1

\n\n

From post #2

\n\n
\n
    \n
  1. Set the DIP switch to \"special\" mode: (1=on, 2=on, 3=on, 4=on, 5=off, 6=off, 7=off, 8=off) see the the description of the\n product.

  2. \n
  3. Set the switch in RXD3

  4. \n
  5. Run the below sketch, change the SSID and PASSWORD values. this is just to test that Arduino and ESP8266 are working together. The code\n originated from BISSA ORBOEA in Element14.
  6. \n
\n \n

Code:

\n\n
#include \"ESP8266.h\"\n\n#define SSID        \"YOURSID\"\n#define PASSWORD    \"YOURPASSWORD\"\n\nESP8266 wifi(Serial3,115200);\n\nvoid setup() {\n\n\n  Serial.begin(9600);              \n\n  Serial.println(\"Restart\");\n  wifi.restart();\n  delay(1000);\n\n  Serial.println(\"FW Version:\");\n  Serial.println(wifi.getVersion().c_str());\n\n    if (wifi.setOprToStation()) {\n        Serial.println(\"OprToStation ok\");    }\n        else {\n        Serial.println(\"OprToStation  err\");\n    }\n\n    if (wifi.joinAP(SSID, PASSWORD)) {\n        Serial.println(\"Join AP success\");\n\n        Serial.print(\"IP:\");\n        Serial.println( wifi.getLocalIP().c_str());      \n    } else {\n        Serial.println(\"Join AP failure\");\n    }\n}\n\n\nvoid loop()\n{\n}\n
\n
\n\n

Solution 2

\n\n

From post #6

\n\n
\n

First you need to write the code for each board according your\n application, but in your code you need to establish a commmincation\n between serial port of Atmega and ESP in both code, again in according\n you application. Finally, put the switches which make the intenal\n interconnection between both.

\n \n

For example, I made a code for ESP to connect in an Access Point and\n put on a TCP server. When a wireless client is connected in ESP and\n consequently in the TCP Server, all data send by client I redirect to\n the serial port. As serial port of ESP is connected with serial 3 of\n the Atmega, the code in Atmega pass to theat the data from serial 3.

\n
\n\n

Solution 3

\n\n

From post #11

\n\n
\n

Follow this if you have the same to my cases:

\n \n
    \n
  1. programm to the ESP(upload sketch) for wifi connection by configure the DIP to 5:on,6:on,7:on and other off.
  2. \n
  3. program to mega(upload sketch) in case you want to send communication to each other by configure Dip to 3:on,4:on, and other\n off.
  4. \n
  5. switch two RX0/TX0(in my case I use Serial.print() and Serial.readString() for communicate between Esp and Mega)
  6. \n
  7. after you already upload sketch to both Modules you can test communication between them by configure DIP to: 1:on,2:on,3:on,4:on.\n and then press Reset Button to restart the boards.

  8. \n
  9. In ESP you can get the string from Mega by if(Serial.available()) Serial.readString(); and In mega you can get data from ESP by :\n if(Serial.available())Serial.readString().

  10. \n
  11. After you got a string so you can split or do anything whatever you want.
  12. \n
  13. If you want send data to Firebase you must use the Firebase Library at the Esp side, in Mega it not support.
  14. \n
\n
\n" }, { "Id": "6321", "CreationDate": "2018-07-08T08:15:58.327", "Body": "

I have a Bowden extruder in my printer setup. Everything works great except this effect which must be related with retraction and is generated on Z layer change:

\n\n

\"Print

\n\n

The model itself, is printed in one layer perimeter / outline shell, with 0.25 mm layer height with the following settings:

\n\n

\"Printer

\n", "Title": "What do you call this effect in 3D printing and how can I remove it?", "Tags": "|print-quality|bowden|", "Answer": "

Your retraction-related setting "Extra Restart Distance" = 0.12 mm could be the cause of your issue, by using a too-high value.

\n

I don't know which slicer you use, but it seems that this setting is the equivalent of "Retraction Extra Prime Amount" in Cura, there explained as follows:

\n
\n

Some material can ooze away during a travel move, which can be compensated for here.

\n
\n

So if retraction happens on layer changes in your case, then this setting could be too high, leading to the observed zits.

\n

Indeed your value is quite high. It is equivalent to $\\pi \\times (\\frac{1.75 mm}{2})^2 \\times 0.12 mm = 0.29 mm^3$ (assuming 1.75\u00a0mm filament). Each of your zits is roughly a half cylinder of 0.6\u00a0mm in diameter and 0.35\u00a0mm in height, so $0.5 \\times \\pi \\times (\\frac{0.6 mm}{2})^2 \\times 0.35 mm = 0.05 mm^3$. It is easy to imagine that your 0.29\u00a0mm3 extra prime amount could be too much by 0.05\u00a0mm3.

\n" }, { "Id": "6326", "CreationDate": "2018-07-08T16:41:08.080", "Body": "

Why does this happen (circled in red), and how can I fix it? It is making my prints come out horrible.

\n\n

Not shown in the picture, but the option \"Coasting\" was Enabled:

\n\n\n\n
\n\n

\"Rendered

\n", "Title": "Gaps in Cura's Slicing for Monoprice Select Mini V2", "Tags": "|ultimaker-cura|slicing|monoprice-select-mini|", "Answer": "

If you enable the option \"Coasting\", the extruder will follow the extrusion path at the end of switching to the next layer or the next section, but will not deposit any material as it uses the build up pressure in the nozzle to deposit the final bits. This shows up in your G-code representation by empty (non-depositing) gaps (in reality, when fine-tuned correctly, will be filled).

\n\n

E.g. sliced without \"Coasting\" enabled:\n\"Print

\n\n

E.g. sliced with \"Coasting\" enabled:\n\"enter

\n" }, { "Id": "6329", "CreationDate": "2018-07-08T19:29:03.427", "Body": "

My first printer is Delta style Kossel clone and I have bad luck with Print In Place (PIP) models, especially with hinges. I suspect that my printer just can't achieve low enough tolerances to make the hinges work.

\n\n

Are there any tricks I can employ to get better prints for PIP models?

\n", "Title": "Print quality of Kossel clone for Print in Place models", "Tags": "|print-quality|delta|kossel|", "Answer": "

It is possible that your printer is over-extruding, and this will certainly give less satisfactory results when you try to print models which have internal clearances.

\n\n

It is also possible that you have some calibration issues which affect orthogonality, x/y/z dimensions or flatness in each axis.

\n\n

In terms of general improvements, slower and maybe cooler might help - but before jumping in with difficult models, how are your 'standard' print test parts coming out?

\n" }, { "Id": "6331", "CreationDate": "2018-07-08T21:07:59.630", "Body": "

Waiting for a heatbed to get up to 85\u02daC for a relatively small part got me wondering why beds aren't hardware/G-code configurable for what area is heated? I'm sure it would be an increase in parts costs and electronics, but it seems that being able to just heat an area a little larger than the part(s) being built would save in time and energy use.

\n", "Title": "Heatbed with zones?", "Tags": "|heated-bed|", "Answer": "

It makes more sense to insulate "unused" areas, since the heat conduction goes with the surface area. Heating the plate at some place (instead allover the area) makes no big difference (aluminium is a very good heat conductor, this is already mentioned). But insulation will do. I would propose a downside full insulation (5-10\u00a0mm) and some "windows-shaped" square silicone (other thin material?) mats to cover the unused areas (mostly on the outer side of the bed) on the top side. Beware of collisions and auto-leveling algorithms (starting very often at corners). Side effect: you may reach much higher temperatures at the uncovered areas (might be doubled if covering half of the surface).

\n" }, { "Id": "6336", "CreationDate": "2018-07-09T10:16:53.013", "Body": "

I am slicing with Cura and Slic3r and one important thing that I recently took my attention is that cura positions head in start point of the new layer and then lifts the nozzle. That caused my few printouts to fall as they collided with the nozzle.

\n\n

Slic3r behavior is different: it raises the nozzle in last printed point and then moves to a new layer starting point (which for me is more obvious)

\n\n

Is there a way to instruct Cura to lift nozzle before it goes to the starting point of a new layer? excluding Z-hops.

\n", "Title": "Cura slicer, enforce Z move before layer change", "Tags": "|ultimaker-cura|slicing|", "Answer": "

OK, after going via all the options I found that CURA has a combing mode which reduces retraction and offers another option which is Avoid Printed Parts When Traveling.

\n\n

That solves my problem.

\n" }, { "Id": "6338", "CreationDate": "2018-07-09T11:28:59.720", "Body": "

A member of our hackspace wants to get their 8- and 11- year old kids1 excited about 3D-printing and CNC cutting and makering in general. We have a weekly open training where people can design and print/CNC/laser an item of their choice like a dogboned box or a two-piece sword and hilt, or a name tag, or whatever. I need a few ideas prepared so we don't spend half our lesson on Thingiverse or in Fusion (more like a quarter)! I'm not a parent and my youngest friend is probably in their twenties.

\n\n

What we have:

\n\n\n\n

Ideas we've had: a minecraft creeper, done as a simple-ish box.

\n\n


\n1: the (girls) are not interested in my normal kids' goto, which is: swords, shields. They are interested in: minecraft, dragons, horses. _o_/

\n\n

edit: this is NOT an opinion-gathering post, though there may be more than one \"correct\" answer. We need specific applications of 3d printing for a young audience. This collection of answers will be useful to evangelize making to a whole new generation!

\n", "Title": "Interesting project for a child", "Tags": "|3d-design|cnc|", "Answer": "

This question is unfortunately, not a good fit for this site, as it stands, for as you say it is opinion based. However, it is great to see that you are getting kids into a relatively new technology (yes, I know it has been around for years, but it is still seen as new to big media and the general public).

\n\n

My answer doesn't provide you with any actual designs, as you asked for. However, just to add an idea or two that I have been thinking about recently, in order to engage kids:

\n\n\n\n

As for examples, there are 42 kids toys tagged on Thingiverse, such as:

\n\n\n\n

\"Grand

\n\n

Also to take from IronEagle's idea, some fidget spinners:

\n\n\n" }, { "Id": "6342", "CreationDate": "2018-07-09T15:42:07.727", "Body": "

I am pondering about buying a Creality Ender-3, and I am honestly confused about some reviews. Some claim it is running 24 V, one did claim it was 12 V, most don't mention it. Since I know about some issues with the clamps, if I get myself an Ender-3, I want to replace the hotend with a proper one from day one. So knowing its voltage is needed to order the right parts.

\n", "Title": "What voltage does the Creality Ender-3 run at?", "Tags": "|creality-ender-3|replacement-parts|", "Answer": "

The Ender 3 and the CR-10 printers use the same board which runs on both 12V and 24V. I have one of each and have to be careful swapping parts between the two.

\n" }, { "Id": "6355", "CreationDate": "2018-07-10T09:38:45.590", "Body": "

Ok, I did it, I ordered myself an Ender-3, a genuine 24V e3D hotend, inductive sensor and some better tubing/clamps to cope with the problem the CR10/Ender line has occasionally.

\n\n

But now I need to fix up my Cura for the machine coming in. The start is the CR10, and fixing the dimensions is easy.

\n\n

But now comes the tricky part: Start and End G-code. For my TronXY I never bothered with changing it away from the \"basic\" settings that a \"custom 3D printer\" on Marlin gave, but this time I want to know what I type in there. The basic code, after I dragged out the G-code handbook from the RepRap wiki to add the missing comments is:

\n\n
G28            ;Home\nG1 Z15.0 F6000 ;Move the Gantry up 15mm going fast\n;Prime the extruder\nG92 E0         ; reset extrusion distance\nG1 F200 E3     ; extrude 3mm of feed stock\nG92 E0         ; reset extrusion distance\n
\n\n

The RepRap Wiki suggests that there could be made so much more from this.

\n\n

I would love to swipe the nozzle before starting to print, making sure that the curled up filament from this first extrusion doesn't get squished against the nozzle and make a bad first layer.

\n\n

How does an example (commented) G-code for swiping the nozzle look like?

\n", "Title": "Writing G-code : swiping at start of print", "Tags": "|ultimaker-cura|g-code|", "Answer": "

I've experimented a lot with priming/swiping in the G-code prologue, as a result of realizing from this question and answer how important it is to get it right. The key points are:

\n\n\n\n

And with that said, here's what I use with my Ender 3:

\n\n
M106 ; Max fan to discourage ooze sticking to nozzle\nG28 ; Home all axes\nG92 E0 ; Reset Extruder\nG1 Z2.0 F3000 ; Move Z Axis up little to prevent scratching\nG1 X0.1 Y20 Z0.3 F5000.0 ; Move to start position\nG1 X0.1 Y200.0 Z0.3 F750.0 E18 ; Draw priming line\nG92 E0 ; Reset Extruder\nG1 Z2.0 F3000 ; Move Z Axis up little to prevent scratching\n
\n\n

I also undo all but 1mm of the filament retraction at the end of my epilogue (custom end) G-code, to leave the printer in a state comparable to having freshly loaded filament. If you don't undo the retraction here, the low extrusion rate in the prologue might not be enough to prime the next time you print, but increasing it would over-prime in the case where you're dealing with freshly loaded filament.

\n" }, { "Id": "6358", "CreationDate": "2018-07-10T13:49:35.493", "Body": "

I am getting a 24 V based Ender 3. From the factory, it has an aluminium bed. So I also put this LJ12 A3-4-Z/BX Inductive NPN NO 4\u00a0mm with 6-36 V operation current into the box together with a few other spare parts. Now, as I read up on these things something dawns on me: The normal input and output voltage of a simple switch is 5 V, as sensors are ran on 5 V on most boards (and in digital logics). The sensors run on 6 V plus though.

\n\n

I do not want to fry my machine by putting in 24 V into the sensor input: What do I have to do (besides making a mount)?

\n", "Title": "Inductive Sensor in 24\u00a0V machine?", "Tags": "|z-probe|creality-ender-3|inductive-sensor|", "Answer": "

Minimalist Solution

\n\n

The very simplest possible implementation is to use a single resistor(!) to protect the input pin, as described in this source. The AVR inputs are protected from over/under voltage by internal silicon diodes D1 and D2. The input resistor must be sized such that it will conduct no more than 1 mA when the internal clamping diode, D1, conducts at Vcc+0.5V, or 5.5V in this case. A 22K resistor could work in this case if we use our 24V supply for the sensor. Our current through the clamping diode is calculated by Ohm's Law as (24V - 5.5V)/22K = 0.84 mA.

\n\n

There is no reason, in this case, to cut so close to the bone though. We do not need particularly high speed detection for this application, so a 100K resistor would be a better choice, and limits diode D1 to 0.19 mA. This gives additional protection for voltage spikes.

\n\n

This solution works well until the internal diode gets fried by a spike or surge, so it is much better to add redundancy, and use a pair of external Schottky clamping diodes, which have a lower forward voltage drop and will conduct before the internal silicon diodes.

\n\n

So, my solution, (the one I plan to implement soon on my own Ender 3 Pro*), is from this source article and looks like this:

\n\n

\"Circuit

\n\n

Where R1, D3, and D4 are my external components as described, and C1 is omitted for simplicity. (If C1 is used, it forms a low-pass RC filter, so you'd need to size it appropriately. If we find that noise or 'bounce' is an issue, we can add C1 easily later.)

\n\n

In my opinion a single 100K resistor and two Schottky diodes are adequate protection for this circuit, and the 6-36V proximity sensor will work very well on the available 24V supply.

\n\n

*Based on the first comment to this proposed solution, I recognize the need to look carefully at the failsafe property here. Depending on whether the existing Z endstop is included in the loop, and how Marlin handles this too will determine whether this is a satisfactory solution.\nI'll leave my part of the discussion there for now, until I get closer to design and implementation on my own machine.

\n" }, { "Id": "6375", "CreationDate": "2018-07-11T20:25:53.417", "Body": "

When I print large prints close to (but not exceeding) the maximum dimensions of the heated build platform on my Anet A8, the brim or skirt or the print itself is printed outside the heated bed, while there is some space left at the opposite sites. It appears as if the print is not in the center.

\n\n\n", "Title": "How to center my prints on the build platform? (Re-calibrate homing offset)", "Tags": "|heated-bed|calibration|build-plate|knowledgebase|homing|", "Answer": "

As to why this happens, particularly with a budget printer the end-stop mountings may not be particularly precisely located, or the moving part may actuate the endstop slightly differently in each build. In my case, replacing the hot end (and thus the whole carriage) gave me an offset of some cm. With this upgrade, it was impossible to retain the stock calibration since the extruder dimensions are quite different.

\n\n

As to how to fix it, the easiest way for me was to modify the 'start g-code'. This fix only applies to the particular slicer, means that what I slice for my A8 won't be centred on the work maker-club Prusa, but doesn't need me to mess about with firmware or apply a per-model update.

\n\n

After the Z-home operation, I already have a pre-extrude step. Prior to this, I set the actual position of what I want my homed point to be, using G92

\n\n
M82          ; absolute extrusion mode\nG28          ; home all\nG92 X17 Y-12 ; re-define origin\nG92 E0.0     ; reset extruder distance position\n
\n" }, { "Id": "6394", "CreationDate": "2018-07-12T21:17:30.247", "Body": "

I have an Anet A8 which I want to convert to an aluminium frame printer like the AM8 (rebuild of Anet A8 parts):

\n\n

      \"Anet

\n\n

This should give a much more stiff frame with respect to the acrylic frame to aid in a better print quality.

\n\n

Since the bed (220 x 220 mm) has a slight warp, I want to use a larger build platform (200 mm x 300 mm) I have laying around. My plan is to use the 300 mm in the Y direction.

\n\n\n", "Title": "How to calculate linear Y rod length for a larger replacement bed?", "Tags": "|prusa-i3|diy-3d-printer|anet-a8|build-plate|y-axis|", "Answer": "

Basically, your setup is the following:

\n

\"Y

\n

The overhang of the bed, assuming the bearings are in the center, equals (300-105)/2 = 97.5 mm on each side. So the distance from the leftmost bearing face (when bed is at y = 0 mm) to the center of the Y rods assembly equals 300 - 97.5 = 202.5 mm. Knowing this distance for the other side of the center to the right when y = 300 mm is the same, the minimum length of the rod will therefore be 2 x 202.5 = 405 mm which equals the addition of the bed length and the outer bearing distance 300 + 105 = 405 mm.

\n

Please note this excludes extra length for e.g. a limit switch, and some extra space around the bed. Basically the extra length of the bed 300 - 220 = 80 mm needs to attributed to each side of the bed, so 40 mm on each side extra.

\n

For a 220 x 220 mm bed, the minimum length would be 2 x (220 - (220 - 105)/2) = 325 mm. When I measure the rods between the acrylic flanges it measures about 365 mm, about 40 mm extra for clearance around the bed (20 mm on each side).

\n

The overhang for the 300 mm bed is not that much, and a construction under the heated plate will be used to fasten the bed at the corners, as long as that construction is not too flimsy, the current distance will be alright. For other distances you should change the value of 105 in the formulas above. E.g. for an outer flange distance for the bearings of 120 mm, you would require rods of at least 420 mm.

\n" }, { "Id": "6398", "CreationDate": "2018-07-14T11:26:03.753", "Body": "

I have built a 3D printer from parts. It is using a standard 12V power supply, an Arduino Mega 2560 replica and a RAMPS 1.4 board. The hotend cooling fan is connected to the 12V-AUX pin (the one right next to the x axis stepper driver) on the RAMPS board so that it continuously receives power as long as the machine is turned on. The printer is controlled by the Marlin firmware.

\n\n

When I give power to the board, the cooling fan starts spinning with a lot of noise. It sounds like it is receiving a higher amount of current than it should. Moreover, the noise is not steady, but fluctuates slightly. After about 30 seconds, it gets slightly quieter; after about 40-50 seconds, there is a sudden break. From then on, the fan spins quietly and regularly.

\n\n

According to measurements with a multimeter, the power supply is keeping a constant and correct voltage of slightly over 12V right from the beginning. Hence, I suspect the issue is somewhere on the RAMPS board.

\n\n

I am worried about this being more than a problem of unpleasant noisiness. What could be the cause and is there more to worry about?

\n\n

You can listen to a recording of the sound: 3D printer hotend cooling fan noise

\n\n

Note that I am not talking about a fan for cooling the extruded plastic, but the one sitting on the extruder heatsink.

\n\n
\n\n

Shutting down the printer (even literally unplugging the power supply) and plugging it back in does not lead to the startup noise appearing again. Only when waiting for multiple minutes before reconnecting does it happen again.

\n\n

The voltage to the fan is measured to be constant. The current, on the other hand, correlates with the sounds heard. It starts at 110 - 115 mA. When the fan is quiet, it is at about 90 mA.

\n", "Title": "Cooling fan spins extremely fast, noisy on startup", "Tags": "|ramps-1.4|electronics|print-fan|", "Answer": "

Late to the party but still...

\n

The noise could be caused by the lubrication of the fan ...when you turn on the fan, the lubrication is seated down. As the fan spins, the lubrication gets all over the bearing thus centering the fan reducing the vibrations aka noise.

\n" }, { "Id": "6399", "CreationDate": "2018-07-14T18:47:23.570", "Body": "

I got myself the Ender 3. The Home position is about 1\u00a0mm left and 2\u00a0mm in front of the front left corner. Now, the hotend center axis is 11\u00a0mm from the carrier plate with the "Mk 10" cooler mounted right onto a pair of 5\u00a0mm pegs that are part of the plate. As I want to change to an e3D v6, which has a diameter of 22\u00a0mm, I will have to move out some distance (ca. 13\u00a0mm) to the current Z axis. This means, that any Y command will be off by this distance.

\n

How do I reconfigure the Home position to have an offset to the 0-positions gained from the limit switches?

\n

According to Repetier Host it runs on Marlin 1.0. Creality offers the firmware on their website as a .hex file - which is hard to edit.

\n", "Title": "Recalibrating Home-position", "Tags": "|creality-ender-3|marlin|homing|knowledgebase|", "Answer": "

Greenonline and I spent this evening hacking/reverse engineering the Ender3 to solve this. Greenonline also wrote about this in his blog.

\n

Step 1 - Get the Firmware

\n

To do this, there are basically two ways:

\n
    \n
  1. Find the a ready-to-use firmware
  2. \n
  3. Make your own
  4. \n
\n

Variant 1 would be to use the pre-supplied .hex files from Creality or some other manufacturer. Variant 2 means writing the settings yourself or using Repetier firmware to write the files for you. In the case of the Ender 3, the chip used sadly disqualified Repetier firmware, so writing it yourself is the only option.

\n

To write the firmware yourself or using Repetier firmware, you need the old settings. To get these, connect the printer via USB, connect it with a software like Repetier host and send M503 to get all the settings.

\n

Now, either hack those into Repetier firmware or use a blank Marlin.ino and the Arduino IDE to customise it. In the later case, you need to put the folder Marlin-?.?.X.ZIP/Marlin-?.?.X/Marlin into the Arduino folder in Documents (Windows) or Home (Mac). Now open the Marlin.ino in Arduino IDE. Set up the new firmware with these base settings or adjust as needed.

\n

Step 2 - Adjust

\n

Now that we have our Configuration files (in the end we pretty much skipped step 1 and grabbed a fitting (German!) one from Christian, a German maker also known as "Chaos-Drucker"), we need to first adjust them as needed to fit our needs and then compile them properly (next step!). As an alternative (English!) base to work with, the TH3D firmware is also available, provided by the Hooglands and team at TH3D.

\n

So, let's open your Marlin.ino with arduino and find the X_MIN_POS (CTRL+F > X_MIN_POS > Search all sketches).

\n

In the configurations we grabbed the sketch file Configuration.h contains lines 786-789 (might differ):

\n
// Travel limits (mm) after homing, corresponding to endstop positions.\n#define X_MIN_POS -10\n#define Y_MIN_POS -15\n#define Z_MIN_POS 0\n
\n

Adjust these till the wanted position is achieved - Repetier Host can help here to determine the right position.

\n

Step 3 - Compile

\n

The hardest step with the Ender3 was to get the code to actually compile, as it uses an ATmega1284P 16Bit which isn't supported by Arduino natively. While the MightyCore supports this chip, it doesn't compile with this hardware library. However, Christian did figure out that the Sanguino set to an ATmega 1284P 16bit does work. These belong as extracted folders into the Arduino/hardware folder, usually in your Documents (if you use Windows) or Home (on mac). Now, we did set Tools>Board:> Sanguino and Tools>Processor:>ATmega1284 or ATmega1284P (16 MHz)

\n

It was also mandatory to have the U8glib.h for Arduino installed. To do this, download the zip from github, then in the Arduino software Sketch>Include Library>Add .ZIP Library... and choose the whole zip repository to auto install.

\n

With these settings we were able to compile the settings.

\n

Step 4 - Getting it on the machine

\n

Most people would expect to read about bootloaders and flashing now, and indeed that is an option. But as I had just figured out how to install firmware from .hex and so I wondered: Can't we make .hex files from our settings? And yes, Arduino supports this.

\n

So, in Arduino IDE press Ctrl+Alt+S (or Sketch > Export Compiled Binary). This will result in two files:

\n\n

Of these two, the Marlin.ino.sanguino.hex is the interesting one for "direct flashing" via CURA. Use this file with the process starting at Step 2 here or use one of Greenonline's solutions to get the firmware flashed - no bootloader needed!

\n" }, { "Id": "6409", "CreationDate": "2018-07-16T13:43:05.273", "Body": "

I made a simple extrude on a sketch in Fusion 360:

\n\n

\"Fusion

\n\n

But what MakerBot software shows me in the preview is

\n\n

\"MakerBot

\n\n

Can someone explain how to make it print a smooth wall? Mesh of the model also seems to be okay. Configuration is in the high preset.

\n", "Title": "Why does MakerBot software shows holes in the wall of a 3d model?", "Tags": "|3d-models|makerbot|makerware|", "Answer": "

I guess you are referring to the corner of the part facing you in the first figure, and the sliced corner on the left in the second figure and not the white lines on the wall facing you. This latter effect is called Moir\u00e9 or moir\u00e9 pattern which is an optical illusion created by interference.

\n\n

This appears to be a slicer error or a rendering error, the walls are not finished. Furthermore from your sparse question it is noteworthy to say that you have sliced the model on a raft.

\n\n

Why not print the model, it will start a raft and when you come to the part it will clearly show whether it is a slicing or a rendering problem!

\n\n

If it is a rendering problem, keep continuing printing, if not, stop the print and download a most recent version of any other free available slicer tool. The most commonly used free slicer tools are Ultimaker Cura and Slic3r. Setup a printer profile and select and modify material and slicing properties to create a new G-code file for you to print. You can now compare the rendering of used new tool to your current rendering and print the model to see if the part now prints well.

\n" }, { "Id": "6410", "CreationDate": "2018-07-16T19:07:06.840", "Body": "

I am wondering if this piece of G-code is valid:

\n\n
G0 (Some comment (Its G0 command)) Y10 Z-5\n
\n\n

I have tested this on my Chinese CNC machine and it strips out the comment and works flawlessly.

\n\n

The machine processes this as G0Y10Z-5 which seems like the correct approach to me.

\n\n

I have however never seen such comment in real CNC practise.\nIt would be nice if anyone is able to test it out on their CNC/3D Printer.

\n\n

Many G-code simulators on the internet fail to process such a line in their parser so it makes me confused. I haven't found anything about it on RepRap Wiki or even Google.

\n", "Title": "Are parentheses allowed within a G-code comment?", "Tags": "|g-code|cnc|", "Answer": "

It looks like comments inside parentheses are not allowed in numerous parsers (ie. Marlin). This seems to be true for most of the 3D printers. Classical CNC milling machines use parentheses without problems.

\n

It should work on Prusa printers as stated in their Wiki. Unfortunately there are no words about nesting of the comments.

\n

I have however found a comment on cnczone.com forums regarding the nesting of comments within parentheses.

\n
\n

Printable characters and white space inside parentheses is a comment. A left parenthesis always starts a comment. The comment ends at the first right parenthesis found thereafter.

\n

Once a left parenthesis is placed on a line, a matching right parenthesis must appear before the end of the line.

\n

Comments may not be nested; it is an error if a left parenthesis is found after the start of a comment and before the end of the comment.

\n

Here is an example of a line containing a comment: G80 M5 (stop motion)

\n
\n

Source: cnczone.com

\n" }, { "Id": "6419", "CreationDate": "2018-07-17T23:22:59.773", "Body": "

I noticed that my version of Ultimaker Cura was out of date, so I tried to go to their website and download a new version. For whatever reason, I could not get the download link to work properly in Firefox with a slew of security extensions.

\n\n

Where are reputable mirrors for Cura?

\n\n

Download.Cnet.com and Sourceforge came up as sources, but years ago they both went to the darkside (SourceForge: HowToGeek, thecomputerpeeps). I don't see anything else that looks even remotely reputable in the search results.

\n\n

I did some analysis, and it appears that the drop down and form don't work correctly if you select \"I don't want to share any personal information\", leading me to think this is web page bug.

\n", "Title": "Where is a reputable place to download Ultimaker Cura?", "Tags": "|ultimaker-cura|", "Answer": "

The answer to your question is: "From the application developer itself!".

\n

Below is the explanation how you can (directly) download Ultimaker Cura from the most reputable source: Ultimaker.com

\n
\n

Regular download

\n

When clicking the link to download of the Cura application download button from the Ultimaker website you are presented with a pop-up GUI that asks you if you want to share where you are using the program for. When you select the required field and press download, the download starts. If this doesn't work, a direct download approach can be used.

\n

Direct download

\n

If the regular download method fails, or you do not get the pop-up GUI, you can use the direct link for Ultimaker Cura from the developer's website:

\n

https://download.ultimaker.com/cura/Ultimaker_Cura-4.1.0-win64.exe

\n

This link can be found at the download directory at the website:

\n

https://download.ultimaker.com/current/

\n

This web directory overview also shows all previous releases and installers for other platforms.

\n

As of version 3.6.0, the download does not appear to be in the list, this link will show an overview: https://ultimaker.com/en/products/ultimaker-cura-software/list\nDo note that the latter link will show the pop-up.

\n

As of version 4.1, the web directory overview link will serve a 404.

\n

Github

\n

The Ultimaker Cura Project has a Github, where they offer the current and previous editions under:

\n

https://github.com/Ultimaker/Cura/tags

\n" }, { "Id": "6427", "CreationDate": "2018-07-19T06:02:46.303", "Body": "

I made a few successful prints since I got my CR-10 two weeks ago and I didn't run into any major trouble. The printer is new.

\n\n

Today I set it to \"preheat\" mode while I was preparing the SD card with the settings being 210\u00b0C for the nozzle and 60\u00b0C for the bed. When I wanted to start the print I noticed that the temperature showed as \"actual temperature\" on the printer's screen showed 233\u00b0C and it was going up steadily while the \"requested\" temperature was still 210\u00b0C.

\n\n

Thiking it might be a mis-manipulation on my part I powered it down for a few minutes (I got scared by the high-temp) and then powered it back on. I then immediately requested the print to start. The CR-10 heated up to the proper value, started printing and kept heating the nozzle. I stopped it at 217\u00b0C.

\n\n

I looked for an answer on the internet but all I could find is people having trouble with the nozzle not heating at all ...

\n", "Title": "Nozzle heats up past the setpoint and increases", "Tags": "|nozzle|troubleshooting|creality-cr-10|", "Answer": "

A similar condition occurred in my 3D printer. I solved the same. I checked all my connection and I came to know that I connected the thermistor of the extruder in the wrong port. So just check the connection of your thermistor.

\n\n
\n\n

Actually my 3D printer circuit board frequently failed because of over current. I then added a multimeter in series with my power supply and the load, also a voltmeter across the voltage regulator. I then corrected every motor driver DRV8825 to a reference voltage of 0.6 V so that the maximum current supplied to each of the motor will be less than 1 A. All these made my circuit checked ok. Then I uploaded the G-code, but I couldn't print because my extruder gets heated about 280 \u00b0C and got reset and suddenly shut down the extruder supply. This continues to happen.

\n\n

Then I reinstalled the Marlin firmware and I also changed the port of my thermistor. Now my 3D printer is ok and prints nicely. I also faced another problem while I gave the print command - it showed that the extruder and bed had started heating but it actually was not heating. So I reinstalled the firmware again and this fixed it. Now my 3D printer works OK. You just try for these steps:

\n\n
    \n
  1. Check the connection.
  2. \n
  3. Reinstall the firmware.
  4. \n
  5. Try manually preheat the extruder, and just see whether it heats to infinity.
  6. \n
\n\n

These are my working experience please try for these, I think it will help you. Just don't leave it you will get the solution. Keep on trying.

\n" }, { "Id": "6447", "CreationDate": "2018-07-21T23:34:38.153", "Body": "

I'm trying to modify some parts on this thing. I'm kind of new to dealing with the modeling aspects of this.

\n\n

I am attempting to widen the part essentially 3mm. This would involve modifying the chain segments, and the anchor points.

\n\n

I have already attempted this with 3D Builder and the result was less than stellar. I split the part, cloned off about 3mm of the middle of the X-Chain (including the top & bottom bits). The intent was to break the part, translate one of the halves off 3mm, add back the 3mm into the gap.

\n\n

When I put all the parts back together and merged them in 3D Builder, the resultant model, while visually on screen appeared to be one piece, the slicing proved otherwise.

\n\n

Is there a way that I can do the above in FreeCAD? I'm learning this so tutorial links would be more than sufficient. But if there is someone who can explain this process to me in FreeCAD that would also be appreciated.

\n", "Title": "Split/edit part in FreeCAD?", "Tags": "|3d-models|stl|freecad|", "Answer": "

I found Dave G's answer to be helpful, however I could only use additive features (e.g. Pad) on the created object, and could not make subtractive features into the object (e.g. Pocket).

\n\n

Here's what worked for me using FreeCAD 0.19 (Instructions from this site):

\n\n
    \n
  1. Create new document
  2. \n
  3. Import STL (File --> Import)
  4. \n
  5. Part workbench --> Create shape from mesh
  6. \n
  7. Part workbench --> Shape builder...
  8. \n
  9. Create shape, select \"Solid from shell\". Make sure \"Refine shape\" box is checked. Press \"Create\" button. (You may need to switch to the \"Model\" tab in the Combo View and select your shape object.)
  10. \n
  11. Change to the PartDesign workbench. Select the newly-created \"Solid\" in the Combo/Tree View, then select PartDesign --> \"Create body\"
  12. \n
  13. A new Body will be created with your Solid as a BaseFeature that can work with Pad and Pocket features.
  14. \n
\n" }, { "Id": "6451", "CreationDate": "2018-07-23T01:06:44.170", "Body": "

I am using a Flashforge Creator Dual Extrusion 3D Printer with RepG (ReplicatorG), and tonight a print completely detached from the build plate.

\n\n

I reset everything and watched it closely. It is a simple abs print at 230/110 degrees.

\n\n

A few seconds after start of print, I noticed that the print bed temp had dropped to 109... and kept dropping.

\n\n

This is new behavior. I haven't started to troubleshoot yet. I am looking for suggestions on how to troubleshoot the issue.

\n\n

Edit 1:

\n\n

Tonight RepG would not connect to the printer. I reseated the USB cord \n and now it does.

\n\n

The test print is ABS at 230/110 from this open scad.

\n\n
difference()\n{\n    cylinder(d = 10, h = 2);\n    cylinder(d = 9,  h = 2);\n}\n
\n\n

P\u1d00\u1d1c\u029fs\u1d1b\u1d07\u02802/Oscar\n Once the print starts the print bed begins cooling. It is cooling before \n any filament is laid down.

\n\n

Edit 2:

\n\n

Examination of the gcode shows the root cause. It does contain

\n\n
M104 S230 T1 (set extruder temperature)\n
\n\n

It does not contain a

\n\n
M109 S110 (set build plate temperature)\n
\n\n

When I manually add the M109 to the gcode file, the model prints normally, and the build plate temperature holds for the duration of the print.

\n\n

The drop in temperature last week is due to the fact that I tend to preheat the printer before actually starting the print. Since there was no M109 the bed plate cooled to ambient temperature.

\n\n

There isn't a hardware issue here. Most likely it is a Rep G configuration error of some sort.

\n\n

Edit 3:

\n\n

I resolved the issue by a fresh install of the latest Rep G. I rebuilt the profiles from scratch with G Code to set the correct bed temp. Thank you for your suggestions.

\n", "Title": "Build plate cools during print", "Tags": "|heated-bed|troubleshooting|flashforge-creator|", "Answer": "

Preheating the bed is a commonly used approach to shorten the time to print. I'll explain; it actually does not shorten the physical print time itself, but before you start a print there is always stuff to do like inserting the SD-card or loading the model in the print server, while you do that, the bed will be heating, so time gained.

\n\n

When you load a model it apparently resets the temperatures, so preheating will only have an effect when you also actually set the bed temperature in the G-code file. This means that either your G-code file has some codes that influence the temperature, the connector is loose or loading a print over the program you use must reset the bed temperature, there aren't any other options.

\n\n

To comment on your G-code, M109 is used to set the extruder temperature and wait till it is reached, M190 is used to set the bed temperature and wait till it is reached. Usually M190 is used to set the bed temperature before the first layer is being printed; the printer waits until the temperature is reached before the printer continues to the next command (usually the heating of the hotend). If you have selected a different temperature for the rest of the print you will find the M140 G-code command prior to printing the second layer.

\n\n

I have tested on my Prusa i3 clone running a Marlin fork to print a model with all references to M190 and M140 removed. I pre-heated to 70 °C (for PETG) and loaded the model in OctoPrint print server and commenced the print. The printer retained the bed value and did not drop.

\n\n

Basically, it is needed to take a closer look at your G-code file. The answer must be in there. If there is nothing to find in the G-code file and electronic connections are fine, the answer must be found in ReplicatorG, this might send reset signals prior to sending the print, but that sounds really strange.

\n" }, { "Id": "6462", "CreationDate": "2018-07-24T18:19:00.927", "Body": "

I have a new Tevo Tornado, which I have completed two good prints with, a 20x20 test cube from the supplied SD and the spool holder also from the SD. I say this to note that the printer was capable of producing a good print.

\n\n

Print 3 was a design I created in Fusion and it printed badly, very disappointing holes missing. Stringing gaps between material just rubbish. I downloaded a simple print from Thingiverse just to see if it was my poor design skills or the printer and that came out just as poorly: lots of strings between details. Both of these were sliced in Cura. As that doesn't have a tornado driver, I downloaded one from the support group and the prints have not even started properly, see pictures for example.

\n\n

I might be going down rabbit holes here but this is what I have found and tried:

\n\n\n\n

Now I have added to the suspicion the z axis coupler see images below:

\n\n

\"z\"z

\n\n

\"z

\n\n

YouTube videos, that I have seen, show couplers that are not a spring - any thoughts? I would certainly appreciate the time anyone has to impart their knowledge.

\n\n

EDIT 1: (Additional information posted as comment, now moved to question)

\n\n

The print bed is brand new from Tevo the unit has only done a few prints most aborted, and I also thought perhaps a residue from the feeler gauges had contaminated the bed, but I have cleaned it with alcohol wipes and also tried putting prints on to unused parts of the bed. You are right the g-code from the test print was from an unknown slicer, no doubt tuned by the manufacturer the part I designed in Fusion was sliced by cura. I have since tried the original test piece and it fouls the extruder nozzle almost straight away.

\n\n

The main differences and there are not many between the set up parts, are that the Cura code does a G92 E0 G1 F1500 E-3.5 before starting layer 0 (both set z0,3). The test piece just does a G92 E0 G1 F7200 the feed rates are different the cura print sets M204 S500 and the test sample sets no acceleration. I assume there is a default in the Marlin firmware.. there is no doubt some globs of PLA stick like in dots between the strings, but the extrusion between direction changes do not kind of like join the dots where dots stick and joins don't.

\n\n

I am going to change the coupler because, well, I don't know what else to do. Replacing it with a better one can't help. Other things I have thought about - PLA temps - I have gone up the whole range according to the manufacturers bandwidth in 5 degree increments no difference I have also done some bed changes but neither hotter nor colder (it's 30\u00b0C and humid at the moment, so maybe a materials property issue, but then again no difference in conditions between a successful first test and all the messes). I am storing the PLA in a gel bead box to reduce humidity. still basically stumped!

\n\n

EDIT 2: (Additional information posted as comment, now moved to question)

\n\n

Thanks for your observation, it's a new bed and I clean down with alcohol wipes ( isopropyl) I don't think I have ever put a finger on the bed - very aware of that. While I don't know what I am talking about on the one hand I am semi convinced it is not the bed, anyway I am going to get a glass bed in part to deal with the protruding screw head issue.

\n", "Title": "Layer 1 won't adhere anymore", "Tags": "|z-axis|layer-height|", "Answer": "

What it may useful to improve your prints, are the following elements, according to my personal experience with the Tevo Tornado: \n1) level the bed when it's warm.\n2) print a cylinder that should measure 100 mm, afterwards measure it with a caliper: regulate the steps per mm according to the difference (many guides for calibration are available, for example: https://youtu.be/Wk3qYOB4E9I)\n3) print this object to be installed on the z axis and regulated with an M5 screw and nut: https://www.thingiverse.com/thing:2827664\n4) Print a test for temperatures, like this: https://www.thingiverse.com/thing:2131069\nI hope it helps. Happy printing.

\n" }, { "Id": "6483", "CreationDate": "2018-07-27T12:18:53.717", "Body": "

For my DIY coreXY printer I use 2.85 mm filament that is fed into an E3D clone hotend using a Bowden tube and a custom designed extruder with belt reduction centered around an Aluminium Bulldog extruder with an MK8 extruder gear. The reduction is necessary to create more torque to feed the filament correctly to prevent skipping and filament slipping and grinding.

\n\n

The prints show under-extrusion (e.g. shell lines and bottom layer lines not or barely touching).

\n\n

How can I calibrate my extruder to extrude just enough filament?

\n", "Title": "How do I calibrate the extruder of my printer?", "Tags": "|extruder|calibration|knowledgebase|", "Answer": "

Basic process

\n
\n

To calibrate the extruder you would need to verify that the\nrequested amount of filament is actually what is being moved by the extruder.

\n
\n
\n

Structural or temporary problem

\n

A first thing to check when under or over-extrusion is encountered for a fresh spool of filament on a normally good working printer is to check the diameter of the filament with a caliper (see image below) and change this value accordingly in the slicer software of your choice and make a test print. If the problem persists or is present from the start of your purchase or build, please proceed. Note that there are two ways to change the extrusion, a simple solution is changing the extrusion multiplier in the slicer software, the better one is to fix the extrusion rate in the firmware.\n\"Measure
\nPlease note that you should measure the filament diameter at several positions\n

\n

3D printer

\n

There is a difference between DIY and commercial printers. Usually a commercial printer of decent quality rarely needs to be adjusted (as this is the job of the manufacturer). But cheap, commercial clones of well known printers, may need adjusting. Depending on the ability to change the settings of the software/firmware (closed or open source, or type of firmware), the user may (or may not) be allowed to change the values through configuration files or G-code commands.

\n

For DIY printers, the builder is responsible for the printer as a whole and thus entrusted with the software/firmware setup of the printer controller board. The chosen printer firmware and its configuration should be done based on the printer layout and used hardware (also electronics like stepper drivers, remember the micro steps setting) which e.g. determines the amount of steps that are required to move an axis a certain amount of millimeters (steps/mm). This also applies to the extruder stepper. To start with a value of steps per mm, you could search the internet for your extruder type and recalculate the steps/mm roughly keeping your specifically used micro step value of the used stepper driver (which is set by dip switches or jumper caps an the printer board) in mind. With this basic setting you will be able to do a test.\n

\n

Calibrate extruder (hot or cold)

\n

When you have a printer, or build a printer and uploaded a rough setting for the extruder steps per mm, please test the amount of extruded filament. It is customary to mark the filament with respect to a certain reference point, then extrude e.g. 100\u00a0mm, and then check the distance the mark on the filament has traveled. The distance should be 100\u00a0mm, if not, you should change the value of the steps/mm in the firmware. To extrude 100\u00a0mm you will need to send instructions to your printer over USB connection using a terminal or graphical user interface typically found in freeware applications as Pronterface, Repetier-Host, OctoPrint, etc. or by creating .gcode text files with specific instructions and load the models through the menu of the printer from e.g. SD-card.

\n

The G-code G1 E100 F100 commands the extruder stepper to move 100\u00a0mm in 100\u00a0mm per minute (please lookup if these G-codes are supported by your firmware!). Be sure that you have preheated the nozzle prior to extruding, many firmware's have built in protection to disallow extrusion below a certain temperature of the nozzle. For SD-card printing, e.g. for PLA you should preheat the nozzle to e.g. 195\u00a0\u00b0C with M109 S195 and as such must be placed before the actual extrusion command.

\n

If the measured distance is different than the instructed length, a simple calculation will determine your follow up actions. E.g. if the instructed length of 100\u00a0mm is in reality 95\u00a0mm, the extruder should extrude (100 - 95) / 95) * 100 = 5.2 % more, or similarly said, the extrusion needs to be multiplied by 1.052. This could be applied in the slicer, but when this is a structural problem you should fix this in the firmware itself. This can be done by adjusting the printer configuration file and uploading new firmware (and settings) or for some printer firmware solutions use the same method as previously used to instruct to extrude filament. The G-code for setting the extruder steps in Marlin firmware is M92 if done with codes. If the original value (can be obtained with the command M503) is e.g. 400 steps per mm, the value should be changed to 100/95 * 400 = 421 steps/mm. The command M92 E421 will set the new value which can be saved to memory (so that it is available after a printer power off/on cycle) using M500.

\n

When the configuration file approach is used, the following line in the configuration:

\n
/**\n * Default Axis Steps Per Unit (steps/mm)\n * Override with M92\n *                                      X, Y, Z, E0 [, E1[, E2[, E3[, E4]]]]\n */\n#define DEFAULT_AXIS_STEPS_PER_UNIT   { 100, 100, 200, 400 }
\n

needs to be adjusted to:

\n
#define DEFAULT_AXIS_STEPS_PER_UNIT   { 100, 100, 200, 421 }
\n

You can do this a few times to fine tune the extrusion process.

\n

For people that have a Bowden setup and firmware that supports altering the minimum extrusion temperature, you could disconnect the Bowden tube to ignore the hotend heat up by temporarily disabling the cold extrusion limit (M302 will display the current value) with M302 S0. After calibration please remember to set it back, e.g. M302 S170. Do note that temperature and extrusion speed do influence the extrusion process, so you might want to fine tune the extrusion process at temperature.

\n

Now you printer extruder is tuned and should extrude the exact amount as instructed.\n

\n

Fine tuning

\n

Although the extruder may now be correctly tuned to extrude exactly the amount as instructed, the slicer may have a different view. Be sure to set the slicer extrusion multiplier to 1.0 when fine tuning the extruder/slicer combination. The ultimate fine tuning starts with printing an object with zero bottom and top layers and a single perimeter with a fine layer height (0.1 or 0.15\u00a0mm).\n\"Single
\nMeasure the thickness of the wall and adjust your slicer extrusion multiplier accordingly.

\n

Now your extruder and slicer should be optimally fine-tuned!

\n" }, { "Id": "6485", "CreationDate": "2018-07-27T12:53:41.203", "Body": "

I just completed assembly of the Anet A8 and powered it up.

\n\n

Powering up shows white blocks on LED and a series of clicking noise for about three seconds with flashing red LED. During power-up, the following happens:

\n\n\n\n

Video: https://photos.app.goo.gl/mxNNeK5o13a9C6TC8

\n\n

The display of the Anet A8 now shows white blocks.

\n\n

Any hints as to what could be wrong?

\n\n

Update 1:\nThe clicking sound is actually the blower powering up and then going off immediately after.

\n\n

Update 2:\nI just did a blind flash to Marlin 1.18 and the firmware deployed successfully. Now, the boot sequence is almost 4 secs longer (previously, it was 3) and the clicking sound of the blower fan slowly spinning is replaced by a high spin-up of the blower. but after that, we are back to a dead printer with all whites.

\n", "Title": "White blocks on LCD screen after Anet A8 power up", "Tags": "|marlin|anet-a8|", "Answer": "

This is a known and documented problem of this printer. I'm afraid your printer board is dead.

\n\n

If you have white squares on the display like this picture:\n\"enter

\n\n
\n

You need to check Anet A8's mainboard - click reset button and check\n the D1 indicator:\n \"enter

\n \n \n
\n\n

From the video you posted it can be seen that the D1 LED does not light up, it is off the whole time. Hence the main board is dead.

\n" }, { "Id": "6488", "CreationDate": "2018-07-27T23:27:12.887", "Body": "

Context

\n\n

A while ago I bought a kit for a 3D printer, along with filament etc. however, after some weeks I ran into some mechanical problems and because I had a lot of other things to do at the time I put the project on hold.

\n\n

Now, almost a year later, I thought I would continue the project. I've got all the mechanical pieces back to work right now, but I have some questions concerning the filament.

\n\n

Main question

\n\n

Firstly, I have a nearly full roll of PLA still, but it has been left in a dusty attic for almost a year. I read here that it could still be used if baked for a couple of hours at 50 degrees Celsius. Does anyone have experience with this? Can this be done in my regular oven that I also use for food without risk?

\n\n

Secondly, there is still some PLA stuck in the printhead. As this may have taken up moisture I fear it might expand and damage the printhead. Is this possible? How could I remove it?

\n", "Title": "Baking old PLA spool", "Tags": "|filament|extruder|pla|", "Answer": "

Oven drying PLA

\n\n

Heating filament below the transition temperature of the filament should not be unsafe when using that oven later for cooking food. You need the temperature to stay under the glass transition temperature of PLA so that the filament doesn't deform. Depending on the PLA filament, you should use the very lowest temperature your oven will be able to handle. Temperatures around 40 \u2103 to 50 \u2103 would be fine to leave the filament in for periods of 3 to 6 hours. Even at this temp your PLA will soften anyways at elevated temperatures, oven drying PLA not necessarily will give you good results. It is therefore of prime importance to store at least PLA (and nylon) in airtight containers with bags of desiccants.

\n\n

Stuck PLA

\n\n

Moisture will not travel that far into the hotend, so changes are minimal that expanding filament has already damaged your hotend. Just heat up the nozzle and push the filament in by hand, put off the heat and pull the filament out fast. Heat up again and insert some fresh filament. Note that moisture in filament breaks the large polymer molecules, so even after drying you are left with brittle filament. Personally this has caused few of my prints to fail as the filament string broke a couple of times during prints (just snapped before the Ultimaker 3 extruder feeder); this was experienced with 2.85 mm filament on small diameter spools (about 20 cm in diameter).

\n" }, { "Id": "6495", "CreationDate": "2018-07-29T15:48:39.010", "Body": "

I had a Z probe installed but the wires came out of the header so I am trying to use software endstops, but any time I G28 the nozzle will always ram into the bed. I am using Marlin Firmware. Which I am relatively new with. I'm used to having hardware endstops, but,

\n\n

I don't have a hardware endstop currently (no probe/no switch).

\n\n\n\n
\n    #define USE_XMIN_PLUG\n    #define USE_YMIN_PLUG\n    #define USE_ZMIN_PLUG\n    //#define USE_XMAX_PLUG\n    //#define USE_YMAX_PLUG\n    //#define USE_ZMAX_PLUG\n    #if DISABLED(ENDSTOPPULLUPS)\n      // fine endstop settings: Individual pullups. will be ignored if ENDSTOPPULLUPS is defined\n      //#define ENDSTOPPULLUP_XMAX\n      //#define ENDSTOPPULLUP_YMAX\n      //#define ENDSTOPPULLUP_ZMAX\n      //#define ENDSTOPPULLUP_XMIN\n      //#define ENDSTOPPULLUP_YMIN\n      //#define ENDSTOPPULLUP_ZMIN\n      //#define ENDSTOPPULLUP_ZMIN_PROBE\n    #endif\n    #define X_MIN_ENDSTOP_INVERTING true  // set to true to invert the logic of the endstop.\n    #define Y_MIN_ENDSTOP_INVERTING true  // set to true to invert the logic of the endstop.\n    #define Z_MIN_ENDSTOP_INVERTING true  // set to true to invert the logic of the endstop.\n    #define X_MAX_ENDSTOP_INVERTING false // set to true to invert the logic of the endstop.\n    #define Y_MAX_ENDSTOP_INVERTING false // set to true to invert the logic of the endstop.\n    #define Z_MAX_ENDSTOP_INVERTING false // set to true to invert the logic of the endstop.\n    #define Z_MIN_PROBE_ENDSTOP_INVERTING true  // set to true to invert the logic of the probe.\n\n    // Enable this feature if all enabled endstop pins are interrupt-capable.\n    // This will remove the need to poll the interrupt pins, saving many CPU cycles.\n    #define ENDSTOP_INTERRUPTS_FEATURE\n    #define PROBE_MANUALLY\n    #define X_PROBE_OFFSET_FROM_EXTRUDER 1   // X offset: -left  +right  [of the nozzle]\n    #define Y_PROBE_OFFSET_FROM_EXTRUDER -55 // Y offset: -front +behind [the nozzle]\n    #define Z_PROBE_OFFSET_FROM_EXTRUDER -3.4\n    #define MULTIPLE_PROBING 2\n    #define Z_CLEARANCE_DEPLOY_PROBE   0  // Z Clearance for Deploy/Stow\n    #define Z_CLEARANCE_BETWEEN_PROBES  3 // Z Clearance between probe points\n\n    // For M851 give a range for adjusting the Z probe offset\n    #define Z_PROBE_OFFSET_RANGE_MIN -20\n    #define Z_PROBE_OFFSET_RANGE_MAX 20\n    #define INVERT_Z_DIR true\n    #define Z_MIN_POS 0\n    #if ENABLED(MIN_SOFTWARE_ENDSTOPS)\n      //#define MIN_SOFTWARE_ENDSTOP_X\n      //#define MIN_SOFTWARE_ENDSTOP_Y\n      #define MIN_SOFTWARE_ENDSTOP_Z\n    #endif\n    #define AUTO_BED_LEVELING_BILINEAR\n    #define MANUAL_Z_HOME_POS 0\n\n    #define Z_SAFE_HOMING\n\n    #if ENABLED(Z_SAFE_HOMING)\n      #define Z_SAFE_HOMING_X_POINT ((X_BED_SIZE) / 2)    // X point for Z homing when homing all axes (G28).\n      #define Z_SAFE_HOMING_Y_POINT ((Y_BED_SIZE) / 2)    // Y point for Z homing when homing all axes (G28).\n    #endif\n
\n", "Title": "Anet A6 Z-axis min endstop not triggering", "Tags": "|marlin|anet-a6|", "Answer": "

The A6 3D Printer \nInstallation Instruction shows that there is an endstop, part 2-7-2, page 4, included in the kit:

\n\n

\"Limit

\n\n

It needs to be fitted as shown on page 8:

\n\n

\"Image

\n\n

Page 36 shows the correct length of the wires, which should prevent the wires being pulled out of the sensors/switches:

\n\n

\"Wire

\n\n

Page 38 shows where the Z-axis endstop is attached to the controller board:

\n\n

\"Controller

\n\n

To be fair, this assembly guide seems to be rather poor quality and after having looked through it a number of times I fail to see any actual instructions on how, or where exactly, to mount the Z-axis minimum endstop.

\n" }, { "Id": "6498", "CreationDate": "2018-07-29T21:26:42.103", "Body": "

Having just re-read Tom's excellent answer to my question Advantages of GT2 over a rack, as my comment ponders1:

\n\n

Why is a threaded rod, or lead screw, used for the Z-axis movement in a Prusa (and its derivatives) given the inherent problems relating to backlash?

\n\n

Indeed the majority of, if not all, delta printers generally use GT2 for the three vertical movements, presumably for this reason (of reduced backlash). Maybe cost and simplicity also play a part? I am purely speculating on these three reasons. The mass of the three vertical carriages and the associated carbon rods, would certainly seem to be a lot less than the mass of a cartesian X-axis gantry.

\n\n

Is it purely for reason of the mass of the X-axis gantry (especially if the extruder is mounted upon it - in a non-Bowden solution) that mechanical rods are used, as GT2 would not be able to lift the weight, without stretching (both over time and flexing slight upon each movement)? If so, then surely wire reinforced GT2 could be used?

\n\n

As a potential aside, would another consideration be to compare the inertia of a GT2 solution to that of a threaded rod, leadscrew solution2?

\n\n
\n\n

1 I have a nagging worry that I have already asked this somewhere else, but after having spent the best part of a day looking for it and failing, I am asking here.

\n\n

2 Although I am not entirely sure how that would be measured/compared, as one solution (leadscrew) is rotational, with a translation to linearal via the screw thread, and the other (GT2) is (effectively) fully rotational? I am going to ask about this particular point on SE.Mechanical Engineering, and will update this question as necessary.

\n", "Title": "Why not use GT2 en lieu of a Z-axis threaded rod or leadscrew?", "Tags": "|z-axis|printer-building|lead-screw|", "Answer": "

A lead screw can be fitted with a anti-backlash nut to get past your concern about backlash. Generally, backlash is not that of a problem considering the weight of the carriage (e.g. Prusa), or whole bed (e.g. CoreXY or Ultimaker). Certainly if you're not using Z-hop, the Z screw will only advance in a single direction!

\n\n

GT2 belts are always reinforced with fibres either glass, steel or some other fibre, all with their own (dis)advantages.

\n\n

As far as I have learned is that lead screws are far better capable of transporting higher loads at the cost of speed. Also, a leadscrew will have more resistance so that if you unpower the steppers, the gantry will not plummet to the bed.

\n" }, { "Id": "6511", "CreationDate": "2018-07-31T07:54:46.837", "Body": "

High performance polymers are becoming available for specific applications.

\n\n

One of such materials is PEEK (PolyEther Ether Ketone), a thermoplastic polymer in the polyaryletherketone (PAEK) family. PEEK competes with certain Aluminium alloys but is half the weight of Aluminium. For aerospace application this sounds very promising!

\n\n

NASA has shown that printing these types of polymers is feasible using low-cost, open source hardware.

\n\n

Does anybody know why the prices of PEEK are so high?

\n\n

Depending on the supplier/manufacturer you're looking at about 700 - 900 Euro per kg.

\n\n

\"Natural

\n", "Title": "Why is PEEK filament so expensive?", "Tags": "|filament|", "Answer": "

Patents still seem to play a role as well.

\n\n

I was curious about this question and did a bit of research: If you filter the U.S. patents after 2010 mentioning PEEK and its various applications and improvements, you still get roughly 10,000 patents. Those are not all primarily patents for PEEK, but still.

\n\n

Even Victrex PLC, formerly named Imperial Chemical Industries, who filed the first patent for PEEK back in 1978, had a patent published in 2016 for a new PEEK production procedure:

\n\n
\n

The process can be used to produce high quality, relatively light\n coloured polymers having a lower Tm than those produced using a\n standard PEEK process.

\n
\n" }, { "Id": "6513", "CreationDate": "2018-07-31T12:03:38.773", "Body": "

I'm one of the lucky (??) Kickstarter supporters who received a T3D resin printer. This uses a proprietary (so far as I can tell) resin which cures under exposure to visible, rather than the usual UV, light. The manual is full of dire warnings about touching the resin, but no MSDS.\nAnyone know if there is a datasheet on their resin, or more generally, for light-curing resins in general?
\n(see myt3d.com for limited info on the machine and the resin)

\n", "Title": "Toxicity of Light-curing resin?", "Tags": "|sla|resin|t3d|", "Answer": "

While you probably cannot find an MSDS on the resin for your printer (yet), they are out there for other resin based printers. Here's a generic MSDS on one for Objet RGD515:

\n

\"enter

\n

It's not a pretty thing.

\n

To back this up, Fabbaloo has two articles out which talk about the toxicity of liquid resin which is light cured. I'm not entirely positive your resin falls into this arena, but would assume since they state in the handling instructions it's toxic, it's probably right along the same lines as the other toxic resins. Please note, in the article it states there are resins out which aren't toxic. They suggest one should handle all resins of this type as though they are toxic unless you have specific documentation which states otherwise.

\n

In one of the articles (Nov 2015) quotes findings which were published by the University of California, Riverside. Their study focused on zebrafish embryos and the toxicity of two different 3D printed materials which are commonly used. The materials were Stratasys Dimension Elite (plastic extrusion) and Formlab\u2019s Form 1+ (photo-cured resin). These printed samples were placed in zebrafish tanks to observe their effects on fish embryos and compare to \u201ccontrol\u201d tanks without the samples.

\n

The University stated:

\n
\n

While the embryos exposed to parts from the plastic-melting printer had slightly decreased average survival rates compared to control embryos, the embryos exposed to parts from the liquid-resin printer had significantly decreased survival rates, with more than half of the embryos dead by day three and all dead by day seven. And of the few zebrafish embryos that hatched after exposure to parts from the liquid-resin printer, 100 percent of the hatchlings had developmental abnormalities.

\n
\n

Note: Here is the second Fabbaloo article from April 2016.

\n

Fomrlabs has quite an extensive list of MSDS sheets for their products. Their page states the following about their resin materials:

\n
\n

Formlabs Standard resins are designed to be similar or safer to handle as other household chemicals or adhesives. When measuring potential acute health effects of inhalation or ingestion, there are no known significant effects or critical hazards. Eye contact may cause eye irritation. Skin contact may cause an allergic skin reaction. The safety data sheets (SDS) are up to date for every resin product and follow the latest government guidelines. Always consult the SDS as the primary source of information to understand safety and handling of Formlabs materials.

\n

Use chemical-resistant gloves - such as nitrile or neoprene; do not use latex - when handling liquid resin, including when removing printed parts from the printer. Exposure to resin in the liquid form can cause mild skin irritation for some people. If you get any on your skin, make sure to wash thoroughly with soap and water before moving on to other activities. Do not use solvents to wash hands, face, or any body parts. Wear gloves and use alcohol, followed by soap and water, to clean tools after handling resin.

\n
\n

While this is not conclusive about your specific resin, like was stated, always treat the material as though it is toxic until documentation tells you otherwise. If all else fails, treat it as toxic and you can't go wrong.

\n" }, { "Id": "6522", "CreationDate": "2018-08-01T05:50:15.653", "Body": "

I\u00b4ve have read an article to change different pattern depending on amount of layers, but my question is if is possible to have different infill in the same part? For example:

\n\n\n\n

Probably someone has seen or reviewed another software to achieve this, or I'm fooling myself.

\n", "Title": "Different infill in the same part", "Tags": "|slicing|infill|", "Answer": "

Using Cura 4.8

\n

\"https://puu.sh/HuLkR/085842df24.png\"

\n

5% infill

\n

This is the Support Blocker button:

\n

\"Support

\n

Drop anywhere on bed, then resize and move like any model, make sure to turn off uniform scaling

\n

With support blocker selected, then click the Mesh Type button:

\n

\"Mesh

\n\n

\"https://puu.sh/HuLAu/926ffa4208.png\"

\n

Infill density has now been changed to 100% for selected part only.

\n" }, { "Id": "6524", "CreationDate": "2018-08-01T12:47:21.620", "Body": "

Following on from What level of voltage does the Creality Ender-3 run at?

\n\n

Dave's answer states that both 12 V and 24 V can be used on the controller board. It also goes on to say that the controller board is used in both configurations in two different printers:

\n\n\n\n

I would like to know how this dual voltage operation works:

\n\n\n\n

Also related:

\n\n\n", "Title": "What are the specifications of the dual voltage controller of the Ender 3/CR-10?", "Tags": "|electronics|creality-cr-10|creality-ender-3|", "Answer": "

I own the Ender 3, and it runs on 24V, as this photo of the power supply shows:\n\"Ender3

\n

From power supply to the board, it uses a 2-wire line connected with a XT60 plug/jack that is common on RC cars:\n\"XT60

\n

The board itself is a proprietary design and labeled as "V1.1.2". The Voltage in is the lowest input on the left:\n\"Creality3D

\n

The Cooling fan (blue-yellow wire), the hotend cooling fan, heater cartridge (white shielded), bed heater (left black-red), board cooling fan (middle red-black) run (in this setup) on 24V. The Logic connectors (black-black & White-white) run 5V logic. I could not figure out the voltage of the stepper motors (upper row).

\n

I have not figured out how the power management circut works, but it achieves this:

\n\n

The chip's caption can't be read on the photo, but it is labeled as "\u0394 ATMEL // ATMEGA1284P // AU 1726"

\n

For the pinout, a german maker did determine, that it the configuration of a Sanguino equipped with an Atmega1284P 16Bit works for compiling firmware and flashing via a bootloader.

\n" }, { "Id": "6525", "CreationDate": "2018-08-01T14:28:24.807", "Body": "

We know from this answer, 3d printed materials continue to outgas after printing and being cured. My question relates to this:

\n\n\n", "Title": "Out-gassing of printed material when heated", "Tags": "|print-material|safety|", "Answer": "

This is not an answer to your question, but it relates to outgassing so I am sharing it here.

\n\n

I have used the acetone vapor method of smoothing the surface of ABS prints. It works really well, and the surface becomes much smoother and glassy. I printed a large coffee mug (lets, for the moment, ignore food safety issues) and made it very smooth.

\n\n

After a couple of weeks to allow the acetone to fully evaporate, I poured a nice, hot cup of tea. Unfortunately, the acetone had not fully left the print, and the surface was immediately covered with dozens of bubbles as the acetone evaporated and pushed against the ABS.

\n\n

This effect was only because of the acetone. Another cup that had not been vapor smoothed worked perfectly and was unaffected by the hot water.

\n\n

I have not seen similar outgassing from PLA, ABS, Nylon, or PETG.

\n\n

If I may hazard an opinion about the substance of your question...

\n\n

I don't think that outgassing is the biggest contributor to the aging of prints and their properties changing. I suspect that (perhaps not in order) these are larger factors:

\n\n
    \n
  1. exposure to UV light. UV light breaks polymer bonds and reduces\nthe strength of plastic.
  2. \n
  3. absorption of water vapor, which can both expand the material which causes stress, and chemically break polymer bonds.
  4. \n
  5. long-term crystallization of the material
  6. \n
  7. fatigue from repeated sub-failure stress
  8. \n
\n\n

NASA used to have resources that spoke to outgassing rates related to suitability for space applications.

\n" }, { "Id": "6535", "CreationDate": "2018-08-02T12:25:20.660", "Body": "

I am using RAMPS 1.4 with Marlin. With Marlin, you can control a large number of fans with the M106 P<fan index> command. I'd like RAMPS to control some additional fans, independent of layer fan. Are there free pins on RAMPS 1.4 to do that? Otherwise, could I setup the RAMPS board to send a command to a separate Arduino? I also have Octoprint if that makes it easier.

\n", "Title": "Controlling more fans with RAMPS board", "Tags": "|marlin|ramps-1.4|octoprint|print-fan|", "Answer": "

From the sources of Marlin you can find how may fans you can use, even if you're not a coder it should be doable. From there you can find how to set the pins for the fans, provided that there are free unused and exposed (so that you do not have so solder directly to the microprocessor). First find how many fans are possible.

\n\n

Starting in Marlin_main.cpp and searching for 106 (or 106 on the Marlin GitHub website) will lead you to the sources of the execution of the M106 command. Below a snippet of M106 and M107 is shown. The reason for M107 to be there becomes clear later.

\n\n
\n#if FAN_COUNT > 0\n\n  /**\n   * M106: Set Fan Speed\n   *\n   *  S<int>   Speed between 0-255\n   *  P<index> Fan index, if more than one fan\n   *\n   * With EXTRA_FAN_SPEED enabled:\n   *\n   *  T<int>   Restore/Use/Set Temporary Speed:\n   *           1     = Restore previous speed after T2\n   *           2     = Use temporary speed set with T3-255\n   *           3-255 = Set the speed for use with T2\n   */\n  inline void gcode_M106() {\n    const uint8_t p = parser.byteval('P');\n    if (p < FAN_COUNT) {\n      #if ENABLED(EXTRA_FAN_SPEED)\n        const int16_t t = parser.intval('T');\n        if (t > 0) {\n          switch (t) {\n            case 1:\n              fanSpeeds[p] = old_fanSpeeds[p];\n              break;\n            case 2:\n              old_fanSpeeds[p] = fanSpeeds[p];\n              fanSpeeds[p] = new_fanSpeeds[p];\n              break;\n            default:\n              new_fanSpeeds[p] = MIN(t, 255);\n              break;\n          }\n          return;\n        }\n      #endif // EXTRA_FAN_SPEED\n      const uint16_t s = parser.ushortval('S', 255);\n      fanSpeeds[p] = MIN(s, 255U);\n    }\n  }\n\n  /**\n   * M107: Fan Off\n   */\n  inline void gcode_M107() {\n    const uint16_t p = parser.ushortval('P');\n    if (p < FAN_COUNT) fanSpeeds[p] = 0;\n  }\n\n#endif // FAN_COUNT > 0\n
\n\n

Looking at this code, the first thing (actually the first line, #if FAN_COUNT > 0, already hints to that) that catches your attention is constant FAN_COUNT. Apparently the software knows how much fans are defined! Interesting! How does it know that! This piece of code ends just after handling the M107 command (line #endif // FAN_COUNT > 0), hence it is displayed here.

\n\n

Continuing the search by finding how FAN_COUNT is set (using the search on GitHub for the Marlin repository) leads to file Conditionals_post.h. An interesting part is:

\n\n
\n/**\n * Up to 3 PWM fans\n */\n#if HAS_FAN2\n  #define FAN_COUNT 3\n#elif HAS_FAN1\n  #define FAN_COUNT 2\n#elif HAS_FAN0\n  #define FAN_COUNT 1\n#else\n  #define FAN_COUNT 0\n#endif\n
\n\n

So depending on HAS_FANx (where x denotes 0, 1 or 2) you can have multiple fans, up to 3 in total! Cool, learned something in the process. :)

\n\n

Let's now search for HAS_FANx; in the same file you will find:

\n\n
\n// Other fans\n#define HAS_FAN0 (PIN_EXISTS(FAN))\n#define HAS_FAN1 (PIN_EXISTS(FAN1) && CONTROLLER_FAN_PIN != FAN1_PIN && E0_AUTO_FAN_PIN != FAN1_PIN && E1_AUTO_FAN_PIN != FAN1_PIN && E2_AUTO_FAN_PIN != FAN1_PIN && E3_AUTO_FAN_PIN != FAN1_PIN)\n#define HAS_FAN2 (PIN_EXISTS(FAN2) && CONTROLLER_FAN_PIN != FAN2_PIN && E0_AUTO_FAN_PIN != FAN2_PIN && E1_AUTO_FAN_PIN != FAN2_PIN && E2_AUTO_FAN_PIN != FAN2_PIN && E3_AUTO_FAN_PIN != FAN2_PIN)\n
\n\n

Now we are getting into clear water! This codes hints to FAN1_PIN and FAN2_PIN. If you look into you pin layout file, e.g. pins_RAMPS.h you see that that are the pin constants that need to be defined, actually FAN1_PIN is e.g. set if you choose an \"EFF\" configuration (Hotend, Fan0, Fan1), e.g. #define FAN1_PIN RAMPS_D8_PIN.

\n\n

What is left for you to do is find an unused pin of your micro-controller and set the FAN2_PIN with #define FAN2_PIN <a free pin number>.

\n\n

E.g. pin 4 is not a used pin number (and an analog output pin). Don't forget that the pin you choose cannot directly run a fan, it should be used to schedule a MOSFET.

\n\n

Note that a dual external fan header \"Reprap Ramps1.4 RRD Fan Extender\" exists, this describes using the D6 and D11 pins. Further investigation led to a 3D.SE posting using this fan header: \"How to configure Marlin to enable auto-fans with dual extruder\".

\n" }, { "Id": "6541", "CreationDate": "2018-08-03T19:42:13.433", "Body": "

I have been having a problem with stringing on my 3D printer, so initially I increased retraction distance from 6.5 mm to 8 mm. Then I tried again, with the same issue, so I increased retraction speed and lowered the extruder temperature. Same problem. Over and over. After I did each of these, I have since decreased move speed, increased minimum layer time, put fan on full blast, and nothing has seemed to increase print quality.

\n\n

I realize this is a lot of alterations to be making, but I'm not sure what to do next. I am using 1.75 mm PLA and a Monoprice Select V2.

\n", "Title": "How to eliminate stringing and regulate flow rate?", "Tags": "|pla|monoprice-maker-select|", "Answer": "

Stringing can be fought by increasing the retraction length, but 6.5 mm (the default in Ultimaker Cura) is already pretty much. Please do note that too much retraction can lead to sucking air into the nozzle, which can lead to air pockets/holes.

\n\n

Basically there are 3 mechanisms that are in play:

\n\n\n\n

You have tried changing multiple parameters already. You may have better luck in reducing the temperature of the nozzle even further, start with a decrease of 5 °C. All that is left to do is increasing the travel speed.

\n\n

There are many stringing test prints to print to fine tune stringing.

\n" }, { "Id": "6546", "CreationDate": "2018-08-04T10:56:57.320", "Body": "

I know very little about the history of 3D printing, except that SLA came first (in the 1980's?), and FDM development was probably held back by patents.

\n\n

By 2016, very low price kit machines were available to hobbyists, in the <\u20ac300 price range, as price-reduced clones of designs which had already seen several iterations.

\n\n

Was this the start of the break-out of cheap FDM machines (as opposed to the >\u20ac2000 semi-professional lab budget prototyping class), or were the earlier iterations of these kit machines also suitable/adopted by hobbyists?

\n\n

I realise that early popularity would grow exponentially, but I'm thinking particularly at what point people could build a printer without needing to compile their own firmware, solder any boards, etc.

\n", "Title": "When did mass produced FDM printers become available?", "Tags": "|history|", "Answer": "

The Makerbot Replicator was released in 2012 and was the first 3D consumer-level printer that was sold only as a complete unit, not a kit. They were also well funded and had a famous Maker-Person as a founder. This seemed to coincide with wide-scale coverage of 3D printing in the media, and a big boom in the DIY side of the movement which is continuing to this day.

\n\n

The unit was still expensive by many standards (US$2500), but it did solve the bootstrap problem... you could just buy one at a store, instead of having to have a specific knowledge set in order to build one.

\n" }, { "Id": "6554", "CreationDate": "2018-08-05T00:21:28.250", "Body": "

I'm printing PLA with the Creality Ender 2 and my print comes out totally fine with the exception of the first layer.

\n\n

Here's the weird part though, if I place a raft or a brim below/around the print - the raft or brim will print perfectly cleanly. Even on a raft though, the very first layer of the actual print comes out ugly, as seen in the photo below, but the raft itself will be perfectly printed.

\n\n

Following that first ugly layer of the actual print, the rest of the print will be 100% clean. This happens consistently every time I print any object.

\n\n

The perfectly printed raft can be seen on the right and the ugly first layer that printed on top of that raft can be seen on the left. Be sure to click the image to see the details of what I mean.

\n\n

\"Ugly

\n", "Title": "First layer of print comes out poorly even when placed on a raft (which prints fine)", "Tags": "|print-quality|fdm|", "Answer": "

Turns out it had to do with the nature of the small holes that I was printing. I had to slow the speed of the initial layer down from 25mm/s to 15mm/s and also set Cura to 'optimize wall printing order' so that it didn't jump back and forth between holes constantly.

\n\n

I also sped up the travel speed to 50mm/s on the initial layer to minimize oozing (although I'm not sure this actually did anything). Print came out beautifully.

\n\n

Didn't even need the raft.

\n" }, { "Id": "6572", "CreationDate": "2018-08-06T20:53:31.277", "Body": "

I purchased a glass bed to use with my still-in-transit Ender 3. Since the bed came in before the printer, I pulled it out and used a flat edge ruler to see how flat the glass surface is.

\n\n

It appears the glass is slightly \"dished\" in the center from one side and \"raised\" in the center on the other side. I am wondering if this is a sign of a defective glass, or if it is likely that the glass may change shape slightly as it is heated?

\n\n

If it does change shape, should it be put with the dish side up or down?

\n\n

Logic would say that if it does change shape, the bottom would probably get hotter since it is against the heater while the top is slightly cooler, so in theory the bottom may expand more \"pulling\" the center down?

\n\n

I guess the bottom line is, should I send the glass back as defective or wait until the printer arrives and see what happens when I heat it up?

\n", "Title": "Will a glass bed warp as it is heated?", "Tags": "|heated-bed|bed-leveling|glass-bed|", "Answer": "

Yes, glass will warp. Think about it this way: the edges cannot be as hot as the centre if you use uniform heating, because they lose heat more quickly.

\n\n

If the edges are colder, they are also shorter than the hotter centre, which expands more.

\n\n

If the centre is longer than the edges, it will bend to accommodate the extra length.

\n\n

You can solve it by placing extra heating along the edges. Once you heat the edges more than the centre, the edges will pull apart the centre, which will result in a flat centre. However, the edges will deform as well, you need just a bit of stretch of the centre, but not so much that it pulls the edges back.

\n" }, { "Id": "6574", "CreationDate": "2018-08-07T03:32:37.460", "Body": "

Let's say I'm modeling a simple open-top rectangular box to slice via Cura for printing. Normally I would set up a solid box of the size I want in the modeler, and then subtract a slightly smaller box to create the final result.

\n\n\n", "Title": "Can I use a 0 mm top layer thickness in Ultimaker Cura to create an open box?", "Tags": "|ultimaker-cura|", "Answer": "

Yes that will work in Ultimaker Cura exactly as you say.

\n\n

So if you have a solid model of a cube, you select wall thickness, bottom thickness, 0 % infill, and no top layers or top layer size of 0 mm. You will then end up with a box without a top layer, but with walls reaching up to the height of your model.

\n" }, { "Id": "6577", "CreationDate": "2018-08-07T05:51:29.623", "Body": "

I've been watching this video 3D printing guides: Calibration and why you might be doing it wrong and at the start (at 0:30) three of the first Reprap printers are mentioned (and shown):

\n\n\n\n

Here is a screenshot:

\n\n

\"RepRap

\n\n

which, to me, looks like a Mendel.

\n\n

However, after having looked at:

\n\n\n\n

... I can not see the \"Cells\" printer listed or mentioned.

\n\n

Am I hearing it correctly?

\n\n

Or is it \"Sells\", in reference to Edward Sells, one of the four authors of RepRap \u2013 the replicating rapid prototyper? However, this paper only mentions/shows the Darwin and Mendel printers.

\n", "Title": "What printer is this?", "Tags": "|reprap|", "Answer": "

Yes, it is a \"Sells Mendel\" in reference to Edward Sells. As noted on the RepRap wiki:

\n\n
\n

Ed Sells, a student of Dr. Adrian Bowyer, designed and built the first Mendel printer, sometimes called a \"Sells Mendel\" to distinguish the original design from later tweaks.

\n
\n" }, { "Id": "6586", "CreationDate": "2018-08-07T14:58:32.433", "Body": "

Recently, I have heard people talking about masterspool, when talking about 3D printing filament.

\n\n\n", "Title": "What is a masterspool?", "Tags": "|filament|", "Answer": "

One key difference between a 'masterspool' and any old spare empty reel you have left over is that the masterspool is a 2-part design which can be fitted over an existing roll of filament without needing to un-spool and re-spool the whole roll.

\n\n

The idea being that filament can be packaged and secured for shipment (with cable ties or the like) without needing to include a whole spool every time. The end user can buy/print only as many spools as they need.

\n\n

This concept also allows for 'extra' features to be included in the spool (such as retention for loose ends) without these features being wasted on every discarded spool.

\n" }, { "Id": "6602", "CreationDate": "2018-08-08T18:04:20.210", "Body": "

Looking in the Cura interface, I can set any whole number 0-100 for the infill percentage. Does Cura have an algorithm to calculate a pattern for any of those possible values, or does it have a few patterns where it selects the closest one?

\n", "Title": "Does Cura have an algorithm for any infill percentage or does it just have specific patterns?", "Tags": "|ultimaker-cura|infill|", "Answer": "

Infill serves two main purposes. These don't seem to map particularly well to the available controls.

\n\n

Adding strength to the part The more plastic your part has inside, the stronger it will be. At least, that is the simplistic assumption. In fact, it seems that infill is not a particularly effective way of strengthening a part (compared for example with thicker walls and structural design features). What is apparent is that some infill patterns are stronger in certain directions, some are more isotropic, and some are just weak.

\n\n

Supporting upper layers Without infill, any top faces on a model will involve bridging, so there is a trade-off between infill density, the number of top layers, and the quality of the top surface of your print. To reflect this function, slicers allow you to incrementally increase the infill density as you approach a top surface. This is particularly useful in a model that has a large inner void which does not otherwise need to be filled.

\n\n

In addition to affecting the infill strength, adjusting the infill pattern can influence the points at which infill connects to the walls. For complex shapes this might affect how successful the print is. There are also other parameters which you can adjust (overlap, orientation).

\n\n

The 'best' infill settings are influenced by the requirements of infill, and the 'success' metric does not appear to have a sharp response that would be useful in performing an optimisation.

\n" }, { "Id": "6606", "CreationDate": "2018-08-08T20:32:58.603", "Body": "

I've just bought Anycubic i3 Mega printer and trying to level it.\nSo far I've printed test object and 2 others but looks like there are problems with leveling.

\n\n

I want to make image of 5 small one layer squares(one in each corner and one in center). Looking for recommendations of simple software/tutorials/approaches to do it. I tried zbrush but found that it kind of complicated.

\n", "Title": "Recommend simple software to create calibration print for table leveling", "Tags": "|3d-models|software|calibration|", "Answer": "

One of the most accessible modelling tools has to be tinkercad. Everything is done in the browser, and it even works (to an extent) on a tablet.

\n\n

I wouldn't recommend getting too attached to it, since it is fairly limited. As an introduction to modeling in 3D, and some of the spatial concepts that you will need to get used to, it works very well.

\n" }, { "Id": "6618", "CreationDate": "2018-08-09T19:22:11.633", "Body": "

I came across this suggestion on the klipper github, https://www.facebook.com/groups/Hypercube.Evo/permalink/192106034761003/. In order to reduce the stretching in the bowden tube you can add fiberglass packaging tape lengthwise along the tube. This would decrease the elasticity while still allowing the plastic filament to run through it. Allowing you to reduce the retraction length and have better control over the amount of plastic being extruded.

\n\n

Is there any reasons that this would not work or actually decrease the performance of the bowden tube?

\n", "Title": "Would adding fiberglass packaging tape to a bowden tube be beneficial?", "Tags": "|bowden|", "Answer": "

The Young modulus of PTFE is about 500 MPa and the surface of a PTFE tube for 1.75 mm filament is about 9.4 mm2.

\n\n

The Young modulus of PLA is variable, between 350 and 2800 MPa, let's say 2000 MPa. The filament surface is 2.4 mm2.

\n\n

The ratio is about 1 because the surface ratio is 4x and the Young's modulus ratio 1/4x. As result, the stretching of PTFE is about similar to the compression of PLA inside it (of course they sum, since they go in opposite directions). PLA is very stiff: other materials will compress much more and make the stretching of PTFE less relevant.

\n\n

However the filament is thinner than the tube, so it will curl a bit, which increases its contraction much more! It is likely that even for PLA the stretching of the PTFE is much less relevant than the compression of the filament.

\n\n

Anyway, fiberglass has a Young's modulus of about 80 GPa (150 times PTFE), but its thickness is what? 0.1 mm? That bring a surface of 2 mm2 at most (and I'm very optimistic, more likely much less than 1 mm2). Fiberglass helps to strengthen PTFE by a factor of surely less than 30, more likely 10 or less (will the glue hold or slide?).

\n\n

As result, PTFE, from a contribution to the overall stretching/compression of 50 % (in fact, much less due to curling), will be reduced to about 5 %. It is good, but this is an optimistic value.

\n" }, { "Id": "6624", "CreationDate": "2018-08-10T19:31:29.763", "Body": "

I just received my new Creality Ender 3. I was going through and checking/adjusting everything for alignment, and I noticed that when you "auto home" the print head, the nozzle stops off the front of the print bed by 5-10\u00a0mm.

\n

Is that normal?

\n

Is it perhaps by design to allow purging the nozzle without dumping on the bed?

\n

It doesn't appear that there is any way to adjust the Y stop switch without making modifications to it. It also didn't look like there was any easy way to move the bed either.

\n", "Title": "Ender 3 nozzle homes off the bed in the Y axis", "Tags": "|nozzle|y-axis|creality-ender-3|", "Answer": "

It is intentional for the head to start slightly off the build plate.

\n\n

If it did start on the build plate you could crash the nozzle when the bed is not levelled. Note the level varies with temperature and build plate type. If you switch from PLA to ABS etc you should relevel the bed.

\n\n

Having just had the innaccurate z-end stop switch cause a deep gouge across my stock build plate, I strongly recommend getting an ABL (auto bed leveller).

\n" }, { "Id": "6628", "CreationDate": "2018-08-11T12:10:56.980", "Body": "

Recently I bought a Prusa i3 pro W (from Geeetech) and I've been struggling with something that is ruining all my printings. I attach a photo with what was supposed to be little \"boxes\":

\n\n

\"Controller

\n\n

As you can see the Z distance is doing weird things, I think may can be fixed through this configurations, as it seems the Z axis is moving too much:

\n\n

\"enter

\n\n

On the other hand, the form the printer is doing the skirt is also bothering me, I've tried a lot of configurations and I think it's a problem of over extrusion, but I'm a newbie and I don't really know what more could I do...

\n\n

\"Close

\n\n

I'll really thank you for every answer because I've been a lot of days with this and I begin to feel really disappointed with all.. Thank you!

\n", "Title": "Problems with Z step", "Tags": "|z-axis|extrusion|", "Answer": "
    \n
  1. as per base of the pink printout it looks like the bed is far below the nozzle (level the bed)
  2. \n
  3. to check the Z steps/mm setting, the best way will be to home it, then from menu move Z by 50mm and check with a ruler or meter traveled distance.
  4. \n
  5. then using a formula (requestedMove/measuredMove) * currentSteps set new value in to Z steps/mm setting. After that repeat the exercise until you will get no difference.
  6. \n
\n" }, { "Id": "6629", "CreationDate": "2018-08-11T15:43:00.390", "Body": "

For developing a CNC system, I am reading the \"ISO 6983-1:2009\".

\n\n

In it, there are several references to what they call \"Detailed format classification\" which I struggle to understand:

\n\n
\n

A classification of the data in a block shall be used to specify the programming detail for a system and machine configuration. This is called the detailed format classification and is described in Annex C.

\n
\n\n

...

\n\n
\n

Zero omission shall be specified in the detailed format classification (see Annex C).

\n
\n\n

...

\n\n
\n

The number of digits shall be designated by the detailed format classification ( see Annex C).

\n
\n\n

...

\n\n

It seem this \"Detailed format classification\" is sort of a definition of the machine on which the program will run. But in this case, I do not understand why this is something part of the program and not some values of the control system.

\n\n

My question is: What exactly is this \"Detailed format classification\"?

\n\n\n\n

\"enter

\n", "Title": "What is the \"Detailed format classification\" of the ISO 6983 (G-Code spec)", "Tags": "|g-code|cnc|", "Answer": "

This is simply the formalised definition of the syntax, so that a parser can be written to interpret any legal G-code. Without this, there is ambiguity - not in the general operation, but in the bounds of what is 'legal' and what should be rejected.

\n\n

Taking the example X+053, this is not a position of 53 units, it is:

\n\n\n\n

This means that X can range from +99999.999 to -99999.999 - an interpretation which matches the verbiage at the bottom of the page.

\n\n

As hinted at in the text, it is possible to use various fixed precision number systems (think using integer milimetres rather than decimal metres), and also to anchor at the most significant bit, so 001 could represent 00100.000. It may help to remember that these specifications were designed in the days of dedicated hardware rather than general purpose computers, when the standard portable storage medium held ~600 kB, and relate to actual machines that had been developed long before that.

\n" }, { "Id": "6630", "CreationDate": "2018-08-11T16:04:26.237", "Body": "

I have recently got an MKS GEN L V1.0 board and 5 TMC2130s. (X, Y, two Z, E). I have wired it all up and uploaded Marlin 1.1.9 and after some quick troubleshooting I got the motors moving. Next day I tried it again and it gave me this error:

\n\n
\n

E0 driver error detected:\n overtemperature\n short to ground (coil A)\n short to ground (coil B)

\n
\n\n

with the following driver debug report: (notice that the extruder driver doesn't show up)

\n\n
\n
  X   Y   Z\nEnabled     false   false   false\nSet current 800 800 800\nRMS current 795 795 795\nMAX current 1121    1121    1121\nRun current 25/31   25/31   25/31\nHold current    12/31   12/31   12/31\nCS actual       12/31   12/31   12/31\nPWM scale       10  10  10\nvsense      1=.18   1=.18   1=.18\nstealthChop true    true    true\nmsteps      16  16  16\ntstep       1048575 1048575 1048575\npwm\nthreshold       0   0   0\n[mm/s]      -   -   -\nOT prewarn  false   false   false\nOT prewarn has been triggered   false   false   false\noff time        5   5   5\nblank time  24  24  24\nhysteresis\n-end        2   2   2\n-start      3   3   3\nStallguard thrs 8   8   0\nDRVSTATUS   X   Y   Z\nstallguard\nsg_result       0   0   0\nfsactive\nstst        X   X   X\nolb\nola\ns2gb\ns2ga\notpw\not\nDriver registers:   X = 0x80:0C:00:00\nY = 0x80:0C:00:00\nZ = 0x80:0C:00:00\n
\n
\n\n

I did some research and found that you should change the CS pin for that stepper and so I did. I tried almost every available pin on my board but no luck. The only change I got was when I wired it to pin 11 and got this:

\n\n
\n

E0 driver error detected:\n overtemperature\n short to ground (coil A)

\n
\n\n

I have the Watterott TMC2130 SilentStep Sticks and have examined the board which seemed correct. Is all wired correctly. Tomorrow I will try to switch the driver to one from another axis and try to switch the stepper to see if anything changes, but it seems to me that it shouldn't since it worked for some time.

\n", "Title": "Short to ground error using TMC2130 stepper drivers", "Tags": "|marlin|stepper-driver|tmc2130|", "Answer": "

I have fixed it. I tried uploading the bugfix version of Marlin and no luck. Then I noticed a loose jumper wire that connected the driver to the board and when re-installing it I noticed it didn't grip the pin that strongly and could be easily pulled off so I switched the wire and everything works now. Turns out that even jumper wires can have a bad connection. Hope this helps someone.

\n" }, { "Id": "6632", "CreationDate": "2018-08-11T23:18:20.737", "Body": "

Am wondering if anybody knows what type of sensor this is (from a build plate).

\n\n

\"enter

\n\n

Perhaps a thermocoupler? The colouring of the wires seems to suggest it is not directional?

\n\n

Would like to know details so that I can make use of it in my own projects.

\n", "Title": "Identify temperature sensor", "Tags": "|electronics|part-identification|", "Answer": "

It is quite likely not a thermocouple, but a thermistor:

\n\n\n\n

You can easily verify this with a multimeter. If it was a thermocouple, its resistance would be near zero. With a thermistor, you should see a decent amount of resistance (most 3D printing thermistors are 100k at room temperature).

\n" }, { "Id": "6654", "CreationDate": "2018-08-13T19:51:37.393", "Body": "

Creality does offer its firmware on creality3d.cn as .hex files. These are pretty good as backups as one can't alter and destroy them by accident.

\n\n

But... How do you install them?!

\n\n
\n\n

This is about installing firmware directly and without another microcontroller. To use another mictrocontroller is How to install new firmware via a Microcontroller?

\n", "Title": "How to install a .hex firmware?", "Tags": "|firmware|", "Answer": "

The Prusa i3 MK3 comes with Slic3r, Prusa Edition. It has a menu for flashing the firmware, which takes a HEX file as input.

\n\n

\"Slic3r

\n\n

You can then select the HEX file. It will auto-detect the printer, if connected via USB.

\n\n

\"Screenshot

\n" }, { "Id": "6657", "CreationDate": "2018-08-13T20:18:42.070", "Body": "

Because .hex files can be installed without any need of an Arduino (serving as an ISP) to flash the firmware on Creality machines I was wondering, if firmware compiled via some other means (for example the Repetier firmware or Arduino) could be exported as a .hex file.

\n", "Title": "Export firmware as a .hex", "Tags": "|firmware|repetier|arduino|", "Answer": "

Current versions of Arduino IDE

\n\n

Use the menu option in the Arduino IDE:

\n\n

Sketch \u25ba Export Compiled Binary

\n\n

See post #6 from the thread How to get hex file from arduino files ?:

\n\n
\n

Well, since it was bumped it's worth mentioning there is a new way to\n do this added in Arduino IDE 1.6.5:

\n \n \n
\n\n

Here is a screenshot for OS X:

\n\n
\n

\"Export

\n
\n\n

There is no need to fully compile, a simple verify will suffice. From post #10:

\n\n
\n

Reopen Arduino, and open a sketch of your choice (use the Examples->Digital->Blink if you have no other sketches). Hit the Verify button to compile the sketch. Now, navigate to the build.path folder. You should see a bunch of files including one with a .hex extension.

\n
\n\n
\n\n

Older versions of Arduino IDE

\n\n

If using an older version of the Arduino IDE, from post #12:

\n\n
\n

Try this if using an older version of Arduino

\n \n

Taking out HEX file from Arduino in 3 simple steps

\n
\n\n
    \n
  1. Open Arduino IDE
  2. \n
  3. File \u25ba Preferences menu
  4. \n
\n\n
\n

\"Arduino

\n \n

\"Arduino

\n
\n\n
    \n
  1. Find \"Show verbose output during \" and check the \"compilation\" box
  2. \n
  3. Locate the preferences.txt file and open it
  4. \n
  5. Add the following line (this is the path to the directory where the hexfile will be stored):

    \n\n
    build.path=C:\\Users\\<username>\\Desktop\\hexfile\n
  6. \n
  7. Change the following line

    \n\n
    export.delete_target_folder=true\n
    \n\n

    to

    \n\n
    export.delete_target_folder=false\n
  8. \n
  9. Quit the Arduino IDE and restart it.

  10. \n
  11. Now when you compile, the directory specified in build.path will contain the .hex file (along with other files which can be ignored)
  12. \n
\n" }, { "Id": "6659", "CreationDate": "2018-08-13T20:44:29.197", "Body": "

Up till now, I've tended to scale my first layer according to the print quality, so a 0.12 mm first layer for a 0.08 mm print, and 0.28 mm for a 0.2 mm print.

\n\n

After changing to a PLA which isn't sticking well, I'm wondering if the first layer is best determined by the printer/tolerance/material, rather than the overall print quality settings. Am I going to get more predictable results if I stick to a 0.12 mm first layer regardless? This is with a 0.4 mm nozzle on an Anet-A8.

\n", "Title": "Should I vary the first layer height", "Tags": "|print-quality|build-plate|", "Answer": "

Layer height in my cura settings means that head of extruder will be going up at 0.3mmm, and how i know that? Because i measure few different settings with height of bed.\nWhen I set bed to -0.2mm, ike everybody is proposing on internet, and i started printing first layer, I did stop it and measure with precise caliper thickness, it was 4.2mm, (human precision with leveling bed :P ) that was for me to much, because extruded material almost was not sticking to the bed,, and walls was not connected each other.\nI did test also with home position of nozzle and setting bed to touch nozzle, to 0 height. ( YEAH) And again I measure thickness of first layer. It was precise 0.3mm. Exactly as I want.\nThat means that or I have better software witch can take nozzle up with initial layer in cura settings?, or magical printer, but my printer is cheap and is buy'ed almost by everybody. \n That means that peoples who are teaching others how to set bed in a printer are totally wrong. Maybe no one measure thickens of first layer? \n Even seller from who I buy printer propose me use paper with thickens of 0.25mm to set level of the bed.

\n\n

Previously I have problems with first layer, because that was too high from bed, now when I set 0 position of nozzle exactly with bed, I don't have problems with thickens of layer, and printed element looks allot better from bottom.

\n" }, { "Id": "6661", "CreationDate": "2018-08-13T23:26:30.287", "Body": "

I would love to start a small engraving business without having to purchase expensive hardware.

\n\n

Using scrap parts at home, or parts from broken CD players, are there any ways to make a laser engraver at home? My cousin managed to make one of his own from scraps.

\n", "Title": "How to build a cheap (but safe) laser engraver with spare parts", "Tags": "|printer-building|laser|", "Answer": "

If you do not have knowledge about the electronics then consider buying a cheap etching machine instead.

\n\n

Build

\n\n

Take a look on https://hackaday.com/ and search for laser engravers.

\n\n

I have found following articles in few seconds:

\n\n\n\n

Cheap laser engravers

\n\n

And this one is about 2018 list of laser cutters/engravers: 15 Best Laser Cutters, Laser Engravers & AIO Machines of 2018. It lists machines around 100$

\n\n\n\n

Safety

\n\n

Laser could damage your eyes (imagine laser reflection or if the machine falls from table when it's working). Take a look on wikipedia.

\n\n

The best option is to use acrylic cover in same color as your laser, so you can observe the etching process and the laser beam is always blocked. For example take a look on orange cover of Formlabs Form 2 3d printer (it uses laser for printing).

\n\n

Another protection is to wear laser protection glasses. Again the glasses must have same color as your laser.

\n\n

Using laser for cutting or engraving means to burn material away. You should have good ventilation in the room.

\n\n

Buy a smoke detector if you use cheap electronics with higher power consumption.

\n" }, { "Id": "6664", "CreationDate": "2018-08-14T04:44:35.477", "Body": "

I got my Ender 3 a couple weeks ago. Within a couple days of test prints, I was able to get it working pretty well. Prints looked great. However, I installed a more permanent solution to my X-Gantry binding issues and now print quality is down again.

\n\n

\"Layer

\n\n

The bottom .25\" of the calibration part looks absolutely terrible, with hideous layer separation issues, while the upper .75\" looks flawless. I have little idea what could be causing this issue.

\n\n

\"Macro

\n\n

The only possible failure mode I can think of is that the bed isn't the right distance from the print head, but even fiddling with the knobs doesn't yield any better print quality than the print on the right of the first image. What can I do?

\n\n

I slice with Cura and can make my profile available if it would be useful.

\n", "Title": "Strange layer separation issues on Ender 3", "Tags": "|print-quality|ultimaker-cura|creality-ender-3|", "Answer": "

I had a real problem with what I was poor bed adhesion and layer separation on my 6month old Ender 3 pro. this problem started all at once, I checked the bed height several times and layer height, temperatures etc still the problem was there.\nThen I noticed the filament guide pinch roller was at a slight angle and on further inspection the plastic arm on removal was fractured so instead of holding the filament against the toothed drive it was metal to metal causing a slip on the nozzle feed.\nI ordered a replacement aluminium feed roller kit at \u00a36.99 the machine is working as it did when I first got it.\nSo it\u2019s worth removing the pinch arm and inspecting.

\n" }, { "Id": "6667", "CreationDate": "2018-08-14T12:21:52.117", "Body": "

I recently purchased some glow-in-the-dark PLA filament (just \"because\"). Being new to 3D printing, I didn't realize you shouldn't run this type of filament through a standard brass nozzle and should replace it using a \"wear resistant\" type nozzle. This is true of carbon fiber type filament as well.

\n\n

My questions are:

\n\n\n", "Title": "Wear resistant nozzles", "Tags": "|nozzle|", "Answer": "

E3D have a blog post on the subject of hardened nozzles. The common abrasive materials are carbon fibre, wood impregnated filament (or any other particulate fill), and some pigments. The blog describes that as little as 250g of the more abrasive filaments can wreck a brass nozzle.

\n\n

For wear resistance, different manufacturers will have different options. Stainless steel is a little harder than brass, and hardened steel is a big step forward. If you fancy spending more money, aluminium oxide (i.e. Ruby) is an option.

\n\n

The downside is printability and cost. Specifically thermal conductivity of brass (or copper) is very good when compared to steel. However, this may not be the limiting factor for your printing unless you're running a very large machine. The various materials may also give different friction/wetting performance which can affect jams.

\n\n

Depending on the printer and the type of hardened nozzle, it probably makes sense to avoid swapping between hardened and standard nozzles most of the time.

\n\n

Hardened nozzles cost between 3 and 20 times what you will pay for a branded brass nozzle (and arguably a bag of no-name brass nozzles can be treated as near enough free disposables in comparison).

\n" }, { "Id": "6671", "CreationDate": "2018-08-14T23:55:20.540", "Body": "

I have an Anet A8 that I just flashed with the latest Marlin firmware (1.1.9). I'm sending prints to my printer with Ultimaker Cura via OctoPrint.

\n\n

I have affixed a PEI surface to my bed that's about 10 mm smaller in each direction than the full size of the bed (that is, I lose about 5 mm on each side of the bed). This hasn't been a huge problem as usually I account for this myself and just make sure there's lots of room on each side of a print, but having upgraded my firmware, I'd like to take advantage of Marlin's bi-linear bed-levelling. Unfortunately, several of the points it chooses are outside the bounds of my PEI surface and so it's impossible to get a decent setting for those points.

\n\n

What's the \"correct\" way to configure Marlin to know that my bed size is actually smaller than it thinks it is? Do I need to additionally adjust settings in Ultimaker Cura and/or OctoPrint?

\n", "Title": "Setting custom x/y bed bounds for Anet A8/marlin", "Tags": "|marlin|anet-a8|firmware|", "Answer": "

As you just flashed your own Marlin, you probably have the marlin.ino and its associated files in the Arduino IDE set for your Processor and board and know how to work with them to some degree. This is just the short version where to find what you need to change the bed size, if the marlin.ino is based on the marlinfw-release.

\n

Changing "home"

\n

In Configuration.h there should be a set of lines that reads akin to this:

\n
// Travel limits (mm) after homing, corresponding to endstop positions.\n#define X_MIN_POS 0\n#define Y_MIN_POS 0\n#define Z_MIN_POS 0\n#define X_MAX_POS X_BED_SIZE\n#define Y_MAX_POS Y_BED_SIZE\n#define Z_MAX_POS 200\n
\n

Usually it is faster to use Ctrl+F and then typing in X_MIN_POS and adding the hook in front of Search all Sketch Tabs and then moving through all the finds till the right one pops up. This not only brings you right to the lines you want to alter, but also covers up the bases in case the distribution you work with did put the movement limits into a different location like Configruration_adv.h.

\n

Changing Bed Size

\n

Alter these minimum positions and make sure that the Maximum positions (usually just after them) to fit the maximum of the bed.

\n

If you reduced your bed size, you also might need to change X_BED_SIZE and Y_BED_SIZE, which (in the marlin.ino) is in the same Configuration.h - I found it to preceede the Travel limits:

\n
// The size of the print bed\n#define X_BED_SIZE 200\n#define Y_BED_SIZE 200\n
\n

Adjust these as needed to fit the new sizing.

\n

Hint: Getting the Values for Adjusting

\n

For finding the adjusted values, I have had good experiences using Repetier Host on Manual Control to move the nozzle (lifted 1mm to prevent any accidents) into the correct XY-position and taking this position as adjustments for the minimum positions. Finding your personal settings is a bit of a trial and error still, so take your time.

\n

Finishing up

\n

Now, recompile the marlin.ino (the checkmark in the top left) to check for errors and flash the new firmware it.

\n" }, { "Id": "6685", "CreationDate": "2018-08-16T08:25:40.037", "Body": "

While uploading .hex firmware directly onto boards that allow this via CURA, this is not always an option for all machines (for lack of a functioning USB-connectable port for example).

\n\n

On both my TronXY X1 board (Melzi 2.0 V5 with ATMEGA1284P) and the Creality Ender 3 (Creality 3D v.1.1.2 with \u0394 ATMEL ATMEGA1284P AU 1726) I spotted these 6 pins labeled ISP that are supposed to allow to connect an Arduino or similar to flash firmware. According to the RepRap Wiki, Melzi is a derivative of the Sanguinololu and both are Sanguino Compatible.

\n\n

After creating the new firmware in Arduino IDE and making sure it compiles for the correct chip in the board, how can I use an Arduino Uno or other microcontroller to flash the new firmware via the ISP to my printer?

\n\n

This is asking specifically about ATmega based controller boards, using the Arduino toolchain.

\n", "Title": "How to install new ATmega firmware via the ISP pins?", "Tags": "|firmware|", "Answer": "

Considering AVR based controller boards...

\n\n

You might not need an additional Arduino, to serve as an ISP. It really depends upon the board. There are basically three scenarios:

\n\n
    \n
  1. Controller board which is Arduino based - Arduino Mega 2560 with a RAMPS 1.4 - with a bootloader
  2. \n
  3. Controller board which is Arduino based - Arduino Mega 2560 with a RAMPS 1.4 - without a bootloader
  4. \n
  5. Controller board which is AVR based board (no separate Arduino board) - as is the case with an Ender-3 and the TronXY board1 - without a bootloader
  6. \n
\n\n

Option 1 is the easiest, and you simply upload with the USB cable, directly from the Arduino IDE.

\n\n

Options 2 and 3 is where you would require an In-circuit Serial Programmer (ISP). Such as a USBasp: USBasp - USB programmer for Atmel AVR controllers

\n\n

\"USBasp\"

\n\n

or you can use another Arduino, see Arduino as ISP and Arduino Bootloaders.

\n\n

An ISP device basically converts the USB to TTL signals (along with a serial protocol (JTAG or derivative)) which then connects to the ICSP2 interface on the controller board:

\n\n

\"ICSP

\n\n

Scenario 1

\n\n

As stated above, you simply connect the board to the PC/Mac using the USB cable and, within the Arduino IDE, you hit Upload - ensuring that you have the correct Board (i.e. Mega2560) and Port selected in the Tools menu.

\n\n

Scenario 2

\n\n

For the Mega2560 the pins the MOSI, MISO and SCK are broken out on to are:

\n\n
Arduino / Genuino Board    MOSI            MISO             SCK       Level\nMega1280 or Mega2560    51 or ICSP-4    50 or ICSP-1    52 or ICSP-3    5V\n
\n\n

To program the Mega2560 using an Uno, connect as follows:

\n\n

\"Uno

\n\n

or using the ISP connector (in this diagram the electrolytic capacitor3 is not shown):

\n\n

\"Uno

\n\n

Take note of the electrolytic capacitor3 on the programming device:

\n\n
\n

The 10 \u00b5F electrolytic capacitor connected to RESET and\n GND of the programming board is needed only for the boards that have\n an interface between the microcontroller and the computer's USB, like\n Mega, Uno, Mini, Nano. Boards like Leonardo, Esplora and Micro, with\n the USB directly managed by the microcontroller, don't need the\n capacitor.

\n
\n\n

or using the Mega's ICSP connector, from Atmega bootloader programmer (see original image):

\n\n

\"Uno

\n\n

Note D10 is used, not RESET4

\n\n

Scenario 3

\n\n

So, to use an Arduino Uno on your controller board, which is scenario 3 (an AVR based board, but not an Arduino board per se), you would need to connect the Uno to the ICSP of the controller board like this:

\n\n
ICSP     Uno\n\nMOSI     D11\nMISO     D12\nSCK      D13\nGND      GND\n5V       5V\nRESET    D10\n
\n\n

or, like this, from Chaos Drucker - CREALITY ENDER3 MIT MARLIN AUSSTATTEN (see original image):

\n\n

There is an error in the image and the red line should go to D10 and not D9 as shown

\n\n

\"Uno

\n\n

Note, again, that the RESET on the Arduino Uno's ISP connector is not used (and Pin 10 is used to connect to the controller's ICSP's RESET pin instead)4.

\n\n

Configure the Arduino IDE

\n\n

Then in the Arduino IDE you will need to select the Programmer to be used, in this case Uno as an ISP, like so:

\n\n

\"Uno

\n\n

Once you have connected correctly, and selected the programmer, then you should be able to hit compile and upload as usual.

\n\n
\n\n

Footnotes

\n\n

1 Which both use the same board - see this answer to What voltage does the Creality Ender-3 run at?

\n\n

2 In case you are wondering, see What's the difference between ICSP vs ISP?:

\n\n
\n

The difference between ISP and ICSP is a hyphen.

\n \n

ISP stands for In-circuit Serial Programming, and ICSP stands for In\n Circuit Serial Programming.

\n \n

Any arrangement that allows you to program a microcontroller while it\n is in a circuit using a serial protocol can be called ISP or ICSP.

\n
\n\n

See also Wikipedia - In-system programming.

\n\n

3 See When using an Arduino as an ISP, is the capacitor required or not?

\n\n

4 Why? From Can the Arduino Uno become an AVR programmer and use the ICSP header to program the target board:

\n\n
\n

First of all, as configured, the RESET pin can not be used as an output pin. In order to use it as an I/O pin, you would have to program the RSTDISBL fuse. But once you do that, as jippie said, it becomes near-impossible to reprogram your Arduino (if you have the socketed through hole variety of the MCU, you could pull it out and put it into a high voltage programmer; in all other cases, it's effectively impossible).

\n \n

So in theory, you can use the ICSP header the way you suggest. In practice, it's under almost any conceivable circumstance a terrible idea.

\n
\n" }, { "Id": "6706", "CreationDate": "2018-08-21T08:19:49.693", "Body": "

Most tutorials ask you to change the printer settings using the The command M92 E420 will set the new value which can be saved to memory (so that it is available after a printer power off/on cycle) using M500.

\n\n

But I just want to modify the Extrusion Multiplier in Slic3r's settings. It's default at for each filament, and I'd like to modify this by adding a profile for each roll I have after calibration.

\n\n

\"enter

\n\n

What's the best way to do this? If I measure extruding 100mm and checking how much is left, what is the formula used to get the new EM value?

\n", "Title": "How can you calibrate extrusion multiplier value in Slic3r WITHOUT changing printer settings in memory?", "Tags": "|prusa-i3|calibration|extrusion|", "Answer": "

Divide the amount overextruded by the desired amount. If you wanted 100mm but got 101mm, that's 1mm extra, or 1% over. Use an extrusion multiplier of 0.99 (1% under) to compensate - AND THEN DO ANOTHER TEST to confirm.

\n\n

This modifier will be used by Slic3r to generate E values in your gcode without flashing anything.

\n\n

I recommend saving this recipe with an intuitive name, like Acme Green PLA.

\n" }, { "Id": "6715", "CreationDate": "2018-08-23T02:06:23.387", "Body": "

So apparently the wiring in my home is... questionable. Very rarely, plugging or unplugging things will cause a power dip. This is almost always the result of turning on a fan or something, is only for a moment before coming back, but its long enough to cause my printer to reset. I want to add a backup battery supply using either the 12v cell pulled out of an old apc ups (before the suggestion of just plugging it into that... I did, somethings fried in it and it will ALSO power cycle everything connected to it every few hours) or an old car battery. Is there any reason I shouldn't use this circuit, which was originally intended for amateur radio equipment? \"battery

\n\n

Also is there anything else i should be considering?

\n", "Title": "Adding a battery backup power supply", "Tags": "|diy-3d-printer|printer-building|", "Answer": "

There are a couple of points to consider with this question. First, you will need to make sure the PSU is correctly adjusted to the float charge voltage of the 12V battery. SLA should be safe to float charge, but over voltage will damage it over time.

\n\n

One alternative solution (since powering the heaters will limit your run time) is to detect power failure, cut the heaters immediately, and save state to EEPROM in the firmware. This is the approach taken in the latest Prusa printers, and is a bit more complex, but might turn out to be more reliable.

\n\n

If the problem you are solving really is brown-outs, then a simpler solution might be to isolate the high current and control sides of the circuit. The MCU will be run from a regulated 5V or 3V3, so a large capacitor (with diode isolation) on that regulator's input would do the trick.

\n" }, { "Id": "6716", "CreationDate": "2018-08-23T06:02:36.050", "Body": "

I have an Ender 3 which I primarily use for printing with PLA. I haven't branched out to other materials yet. :)

\n\n

I've done some research into PLA fumes and airborne particulates which seemed to mention that PLA is mostly safe, but ABS is rather dangerous to print without proper ventilation. However, I understand that there isn't much research on the topic and that there haven't been many studies.

\n\n

I have been keeping my printer in my bedroom, far isolated from flammable materials, which I sometimes leave on to print while I'm asleep. Should I be concerned with my health safety with respect to airborne particulates emitted by printing with PLA?

\n\n

Other questions ask about ABS, but here, I'm asking specifically about PLA.

\n", "Title": "Serious safety concerns with living in same room as PLA 3D Printer?", "Tags": "|pla|safety|health|", "Answer": "

Having asthma, I'm very sensitive to air quality and when i first started 3D printing i quickly noticed a sore throat, feeling out of breath, headaches and well the smell.

\n\n

One thing many beginners overlook is material oozing out of the extruder in places other than the nozzle itself. This causes a dirty extruder head which 'burns' that material and thus creating a ton of air pollution and a nasty smell.\nAs a fix, i took my extruder head apart and reassembled it with PTFE tape on all the threads. The oozing and burned smell of material is now completely gone, my initial health issues have also gone away.

\n\n

This does not take away the fact that ultrafine particles are still being generated, but you will not notice this problem in an acute way. If you print daily use a fume extractor, your lungs cannot clean out this size of particles. The health effects will be similar as living near a busy street or highway.

\n\n

One last thing few people mention: cleaning up parts. When you file or sand down parts this creates a lot of very fine dust, and since plastics are electrostatic this becomes very hard to clean up. I now use an extraction fan whenever I'm cleaning up a printed part.

\n" }, { "Id": "6722", "CreationDate": "2018-08-23T10:59:49.767", "Body": "

I've seen many references to a FDM print being weakest in the Z axis, due to poor bonding between layers compared to the extruded walls.

\n\n

Thinking about optimising this for a specific material (excluding temperature and geometry), is there an optimum layer height? It seems obvious that too thick a layer will give less compression and maybe less heat transfer into the layer below (so 0.3 with a 0.4mm nozzle might be expected to be a bit weak). Is there a single break point (i.e. less than half the nozzle is good), or are super fine layers either good or bad?

\n\n

I'm specifically using PLA at the moment, in case different materials have different behaviour in this respect.

\n\n

I am not asking how to model the strength of layer bonds or how to take that into account when designing a part.

\n", "Title": "Relation between layer height and bond strength", "Tags": "|print-quality|slicing|", "Answer": "

Just adding another datapoint. CNCKitchen has a new video where he analyses this for cross layer and between layer tension. He also references some other research (which is rather inconclusive). He supports the coarse estimate of 'no more than 50% of the nozzle diameter, with performance also dropping at very low layer height.

\n\n

He found cross layer tension supported more than 2x the layer-to-layer tension (with a typical 3-wall, low infill pattern).

\n" }, { "Id": "6723", "CreationDate": "2018-08-23T12:08:25.053", "Body": "

I will be creating a small tube styled piece to use as a junction between two pieces of plastic. The idea is to reattach the two pieces and provide strength so they don't break apart again. I plan on using PLA.

\n

My question is, will superglue (cyanoacrylate) work best for this or are there better choices for attaching PLA to hard (injected molded) plastic? The big thing I'm wanting to make sure is if any of these glues will dissolve the PLA and whether some glues might bond better than others.

\n", "Title": "What glues for bonding printed PLA to injection-molded plastic?", "Tags": "|pla|post-processing|knowledgebase|glue|", "Answer": "

PLA Gloop is dedicated glue for PLA. It contains chloroform, so be careful.

\n\n

Additionally a PrusaPrinters guide: The Great Guide to Gluing and Assembling 3D Prints

\n" }, { "Id": "6726", "CreationDate": "2018-08-23T21:30:56.073", "Body": "

What are the 'headline' basic design rules for FDM?

\n\n

Which topics for design principles do you recommend to someone who has purchased an FDM machine and wants to understand what is practical? In other words, where is the place to start when you don't yet know what questions to ask?

\n\n

This question and its answer discuss design tools and the workflow but doesn't cover design rules.

\n\n

Design rules here meaning the principles of what is special about planning a design to be manufactured with an FDM process as opposed to traditional machining or a printing process like SLA, not the detailed/automated checks that would be applied to something like a PCB layout prior to sign-off.

\n", "Title": "What special considerations must be taken when designing parts for 3D printing?", "Tags": "|3d-design|knowledgebase|", "Answer": "

In a presentation(pdf) that I've used as an introduction to 3D printing, I listed out the following points:

\n\n\n\n

The other detail worth calling out is the precision of what can be printed with FDM, although the best case resolution is ~0.1mm, the overall accuracy will be closer to the nozzle diameter due to the way that extruded filament is squashed during the print.

\n" }, { "Id": "6727", "CreationDate": "2018-08-24T09:48:06.743", "Body": "

I recently upgraded my Ultimaker 1. The upgrade includes a different stepper motor for the Extruder, the same a Ultimaker 2+ would use. I plugged the new motor and nothing happened.

\n\n

To eliminate the problem I ordered a new PCB and stepper motor drivers, reassembled all electronics. For some reason the extruder does not move, and actually any other motor I plug into the extruder port doesn't either. I switched the driver, twice, but without a different result.

\n\n

Can anybody tell me what component could be faulty or how I can find the problem? Could it be the Arduino board, even when I use the Ulticontroller? Maybe remove one of the jumpers next to the driver?

\n", "Title": "Customized Ultimaker 1: Extruder motor does not move", "Tags": "|stepper-driver|motor|ultimaker-original|", "Answer": "

Did you heat the hotend before attempting to move the extruder? Most firmwares block cold extrusion. If you send the printer M302 it will allow the extruder motor to move without the hotend being above the temperature set in the firmware. Jumpers next to drivers are used to set microstepping, no need to adjust these unless you changed to a different type of driver or want to use different microstepping. Changing them usually requires changing the steps in firmware as well. Also, swapping drivers or motors while the driver is powered can destroy it.

\n" }, { "Id": "6739", "CreationDate": "2018-08-25T13:50:51.290", "Body": "

When I purchased my China made Anet A8 printer, it came with the ball bearing style linear bearings for the 8mm guide rods. While pulling parts out of the box and putting them together, I noticed several of the small ball bearings fell out of their respective holders. At the time, I really didn't know what to think of it (ie: were these just extra ball bearings falling out; were they actually needed). I put the printer together anyway and it seems to work okay.

\n\n

I have noticed while I've been printing, there's a lot of noise during travel of the pieces. I'm not exactly sure where the noise is coming from, but realize it has to be coming from one or more of the bearings. To hopefully fix the issue, I've purchased some Igus Drylin polymer bushings to replace the linear bearings:

\n\n

\"enter

\n\n

My questions are:

\n\n\n", "Title": "Replacing bearings with Drylin bushings", "Tags": "|maintenance|bearing|", "Answer": "

In addition to this answer and addressing Anything else I'm not thinking of to worry about? I would like to add that you should take care in installing the plastic bearings in suitable housings. It is best to lower the mass of the carriages by replacing the blocks and the bearings, there are a lot of printable bearing housings to find on the internet (e.g. Thingiverse). When installing plastic bearings, it is quite easy to get problems with bearings that bind, so take care when you install them and make sure the bearings do not bind (before you re-install the belt).

\n" }, { "Id": "6747", "CreationDate": "2018-08-27T13:17:40.447", "Body": "

I sadly have a big problem since a couple of weeks and I cant fix it myself. For over 20 prints, I had the same problems: At the start, everything goes well but at some point of time, my Creality CR-10 starts under extruding and after like 1 minute, NO filament is extruded anymore and the 3D printer moves over the printing bed without extruding anything. After that, I have to pull out the filament and the feeder always grinds into it.

\n\n

Some problem with the hotend (so the feeder works well but it cant push the filament through somehow - I cleaned the nozzle and couldn't find anything wrong there. When I push the filament through by hand it gets extruded but after a failed print, it is VERY hard to pull it back (because of the filament being slightly bigger at the nozzle - hard to remove! -

\n\n

That could be the problem:

\n\n

I just tried to remove the filament from the printer (another failed print) I had to use two tongs because the filament was so hard to pull back. I noticed that the diameter of the PLA close to the hotend was a lot bigger (way over 1.75 mm) For about 5 cm that's a very long distance - that's the reason why it's hard to pull back (and also push through?) But I don't know why that happens... If I get an answer for that, I think that I have solved my problem

\n\n

I already tried printing at 50 % which didn't work.

\n\n\n\n

And here some pictures of the failed prints:

\n\n

\"Failed

\n\n

\"Filament

\n\n

\"Sorry

\n\n

\"Failed

\n\n

I think I can exclude these:

\n\n

It could be: (the things I can imagine but don't have a solution for)

\n\n\n\n

The filament seems to run out of the hotend. Something is really broken here. I tried to lower the print temperature and don't have an issue (at least not that big) with pulling out the filament but instead, the feeder started grinding into the filament again.

\n\n

\"Filament

\n\n

I'm very happy for any answer and possible solutions.

\n", "Title": "Constant under extrusion and filament grinding", "Tags": "|filament|pla|extrusion|hotend|", "Answer": "

A 0.27\u00a0mm layer height is not correct for any printer, any nozzle/hotend, any file, and should never be used as a layer height variable for any FDM printer.

\n

Layer height should be a factor of the diameter of your nozzle size. i.e. Your nozzle is 0.4\u00a0mm, so depending on your printer, your range of resolutions/layer heights could start as low as 0.04\u00a0mm, but is monumentally more likely to begin at 0.08 or 0.12\u00a0mm, with a maximum value being 0.28\u00a0mm, and maybe 0.32\u00a0mm if you're doing some risky/experimental spiralized vase printing.

\n" }, { "Id": "6751", "CreationDate": "2018-08-27T13:50:32.257", "Body": "

I get the concept of automatic bed leveling...the printer moves around the bed and uses a sensor to identify high and low spots, then \"software compensates for differences\".

\n\n

But what exactly does \"compensates\" mean?

\n\n

It is extruding more material in the low spots to build them up and thinning out the high spots? Is it adding or removing layers? Is it shifting layers as it goes up to compensate tilt? Or...?

\n\n

In what ways will this affect the final outcome? Would it be valid to say that if you wanted an automatically leveled bed and dimensional accuracy you should always print to a raft?

\n", "Title": "What exactly does automatic bed leveling do?", "Tags": "|bed-leveling|rafts|", "Answer": "

Basically there are two problems to address: first - bed inclination and second - uneven bed surface.

\n\n

There are different algorithms dealing with one or both problems.\nMost specialists insist that bed should be flat and most people use glass as the top to achieve this. And this eliminates second problem and require to solve only the first one.

\n\n

To find bed inclination - autolevel algorithm usually do three points test and get transformation matrix. This matrix then used to transform XY motion to be parallel to the surface.

\n\n

For uneven bed surface more complex algorithms used. But usually bilinear mesh used that could be looked as the whole square surface subdivided into smaller ones with individual inclinations. Basically algorithm from XY coordinates determines the sub-square and then applies corresponded transformation for it's inclination.

\n\n

The transformation fade out with the Z going up and usually applied within first 10mm over the bed level.

\n\n

There is UBL (Universal Bed Leveling) algorithm that deals with two problems simultaneously. Only once it probes the whole bed with many points to produce two dimensional mesh to address second problem.\nAnd for each next auto level it does three points test to find the whole bed inclination. Then it combines measured inclination with the mesh information stored in memory. UBL assumes that surface do not change but only it's inclination could change.

\n\n

In all cases this is all about to guarantee that the first levels of filament will lay on the bed evenly and will stick to it.

\n" }, { "Id": "6755", "CreationDate": "2018-08-27T16:48:01.473", "Body": "

\"extruded

\n\n

Hey!

\n\n

Pretty new to all this. Wondering how you would approach this problem to have an extruded element on top of a flat surface. I want to avoid lots of support material (actually no support at all for a cleaner print and no work with sanding etc.) I intend to print this inverted in Z (i.e. text down) for the main structure.

\n\n

My only idea would be to print it separately and then glue it on. But maybe there is another solution that comes to your mind?

\n\n

Cheers!

\n", "Title": "How to print text upside down for minimal support when printing", "Tags": "|support-structures|", "Answer": "

One way that I have solved that issue is by model it in openscad and embed the letters in the surface. then with my dual headed printer, print the letters in a different color. No support material is needed in that case.

\n" }, { "Id": "6760", "CreationDate": "2018-08-27T21:02:10.920", "Body": "

Are there any specific type of FDM 3D printers that I should look for?

\n", "Title": "Do all 3D printers allow the printing of flexible material?", "Tags": "|diy-3d-printer|nozzle|", "Answer": "

No. Not all 3D printers can print flexible material.

\n

The first place you will have trouble is in the extruder itself. Flexible filament will find any way to escape from the confines of the rollers and the guide tube. Any opening will allow the filament to buckle and find a new path. It is common to find that the filament has filled any gaps inside the extruder, and wrapped itself tightly around the drive roller.

\n

@0scar describes this in his answer. If the extruder does not have tight tolerances, with 1/4mm or less clearance between the drive roller on all sides, AND if there isn't a tube mated right up into the place where the filament is pressed between the drive roller and the idler, your extruder will fail to print Ninjaflex (a very flexible filament).

\n

Oscar also mentions that Bowden feed doesn't work well. It is fundamentally the same buckling problem. There is always a gap around the filament inside the Bowden tube. A stiff filament will rub at some points, but a soft filament will compress and ripple, filling the entire lumen inside the tube, and greatly increasing the friction. Higher friction means more extruder pressure, so more buckling, more friction, and more failure.

\n

Even if you are lucky and don't experience the run-away friction problem, the flexible filament is more compressible. With a Bowden feed system, that compression must be preloaded at the beginning of each extrusion movement and relieved for each retraction. With direct drive, the filament also compresses, but the amount of filament is much less, so the compression is less, and the problems are less.

\n" }, { "Id": "6762", "CreationDate": "2018-08-27T22:30:49.547", "Body": "

I'm trying to slice this model: https://www.shapeways.com/product/VVNUVZ6JL/dread-warrior?optionId=66361748&li=user-profile

\n\n

\"Original\"

\n\n

I'm using a .4mm nozzle and .1mm layer height. When using slic3r some of the layers seem to just not get generated. You can see this on the horns. It also has some weird artifacts on the shield of the model.

\n\n

\"Slic3r\"

\n\n

I tried using Cura to slice the same model and while it does a better job for the horns it removes most of the face on the model.

\n\n

\"Cura\"

\n\n

Are there any slicer settings I can adjust to help fix these artifacts? Is there any other slicer out there that may fix the problems? Is this just a problem of needing to use a smaller nozzle and lower layer height?

\n", "Title": "Slicing problem - Slic3r prusa", "Tags": "|ultimaker-cura|slicing|slic3r|", "Answer": "

It's a nozzle size issue. The model contains a lot of details which is really challenging to print with a 0.4 mm nozzle.

\n\n

A big nozzle is just not able to print such fine details. You should switch to 0.25 mm or 0.15 mm. Just try to set a smaller nozzle in your slicer.

\n\n

There is a nice article on Prusa's blog showing the difference in printing details with 0.4mm and 0.25mm: EVERYTHING ABOUT NOZZLES WITH A DIFFERENT DIAMETER, especially chapters Examples of practical use and Printing miniatures.

\n\n

See the difference:

\n\n

\"Business\n\"Business\n\"Pirate\n\"Chest\"

\n" }, { "Id": "6764", "CreationDate": "2018-08-28T08:27:49.767", "Body": "

A couple weeks before, I bought a custom 3D printer that has an Ultimaker 2 motherboard in it. However, the dimensions of the printer is not same with Ultimaker 2 (X and Y same, a bit smaller on Z). The printer had tinkerfirmware installed in it. Today, I tried to print a premade .gcode file (Which was for another 3D printer I guess) and after pressing print, The machine told me the file will overwrite machine settings, and I pressed yes for it. After that, the dimensions of my 3D printer has changed in it's firmware. The bed is raising more than it should while starting calibrating, and not setting it's position precisely. (To make the 1mm gap, I had to move the bed down 4-5mm away from where it should be.) Now the question is, what can I do to fix this problem ? I also tried reinstalling original firmware which didn't really worked. (All the parts are orginal except the frame, which is a bit more smaller on height) How should I measure the height of printing area?

\n", "Title": "Custom 3D printer printing dimensions has changed because of a .gcode file", "Tags": "|firmware|desktop-printer|", "Answer": "

Firmware is stored in EEPROM, and may contain some default values, but does not affect the calibration values which are stored in EEPROM.

\n\n

You should be able to reset the calibration to 'factory defaults', this actually means to take the defaults in the firmware and store them in EEPROM.

\n\n

Use M502 to load firmware defaults into the current session.\nUse M500 to write the settings from the current session into EEPROM.

\n\n

It is unusual for a design in gcode to include modifications to the settings, but maybe it was done to change acceleration or something similar.

\n" }, { "Id": "6778", "CreationDate": "2018-08-29T16:45:04.900", "Body": "

When running Marlin Firmware, is it possible to run G-code scripts/series of commands automatically when you insert the SD card?

\n\n
\n\n

I'm running Marlin on a 3D printer board using an ATmega 2560 based board with a reprap discount LCD controller with an SD card slot. I would like to do this without the need to add another computer/board, so native from the board running the Marlin Firmware.

\n", "Title": "Can G-code scripts be run automatically on inserting an SD card when using Marlin Firmware?", "Tags": "|marlin|g-code|firmware|scripts|", "Answer": "

I don't know what you would like to achieve.

\n\n

As @typo mentioned there is similar functionality which triggers at printer startup: https://github.com/MarlinFirmware/Marlin/wiki/Autostart

\n\n

I found in Marlin's source code that you can enable that autostart feature to be triggered from menu. Just enable MENU_ADDAUTOSTART in Configuration_adv.h.

\n\n

If you really need a g-code that start after sd-card insert then you have to adjust the marlin source code on your own. I suggest to alter file cardreader.cpp. There you have to trigger the autostart procedure after a sd-card is inserted.

\n" }, { "Id": "6784", "CreationDate": "2018-08-30T16:30:23.963", "Body": "

I've heard I should store filament in sealed container, preferably with a desiccant.

\n\n

But let's say I let a spool get a little old on the printer, or I purchased a filament spool that was old or improperly packaged. How would I know? How would this impact prints (what kind of symptoms would I see)? What things could I do (perhaps in the slicer) to correct for this and prolong the life of marginal filament?

\n\n

And the corollary... in a typical environment, how long can filament be left out without suffering from the exposure?

\n\n

I'm thinking mainly about PLA, but responses for ABS and other materials are useful, too.

\n", "Title": "How do I know when filament is getting old, and what things can I do to correct for it?", "Tags": "|pla|filament-quality|storage|", "Answer": "

Moisture creeps into the filament when PLA is exposed for a long time to a humid environment. This is audible like popping bubbles when heated in the hotend and does worsen the print quality. The moisture causes a noticeable property as it breaks the long molecular chains of the thermoplastic material. Once this has happened, its effect is irreversible.

\n\n

The effects of this moisture breaking the long molecular chains results in a more brittle filament; it can even snap during feeding, leading to unpleasant results like unfinished prints. If the filament snaps/breaks when you bend it, instead of folding, your filament has taken up moisture.

\n" }, { "Id": "6793", "CreationDate": "2018-08-31T12:52:27.280", "Body": "

What about the strength parameters?\nWhere can I give CAD models to print them and receive them?\nPlease guide me.

\n", "Title": "Can 3d-printing be used to print drone frames which carry around 4 kilograms?", "Tags": "|print-quality|", "Answer": "

Yes, no problem! But this does not depend on the production process, it depends on the design.

\n\n

Not only the production process (adhesion in Z direction is usually weaker than X or Y) should be taken into account, but also the choice of material. Even carbon fibre doped filaments (nylon or PLA) exist (harder to print however, due to excessive wear of the nozzle by the carbon fibres).

\n\n

You can also consider using carbon fiber tubes, and only print the connectors/mounts to connect the tubes to the engine mounts and main body mount.

\n\n
\n\n

Basically, your question is not specific enough and boils down to a \"How do I design a drone frame\" or \"What are the main design issues for designing a drone frame\", which should be asked in an other forum. Once you have a design you could change your question and ask for specifics regarding printing the parts using FDM (e.g. orientation, positioning, materials, etc.).

\n" }, { "Id": "6794", "CreationDate": "2018-08-31T15:23:36.763", "Body": "

I had recently purchased an ender 3 and after setting it up and plugging it in, I received an electrical shock from the power supply. I live in the UK and so I was provided an EU to UK adapter which I used and I set the voltage to 230 V.

\n\n

Does anyone know why I was shocked and if there is any solution? Was it because EU to UK isn\u2019t grounded (or am I wrong)?

\n\n

Would an older 10 amp monitor power cable work better as it\u2019s grounded?

\n", "Title": "Electric shock from ender 3 printer PSU", "Tags": "|creality-ender-3|electronics|safety|", "Answer": "

Sorry for leaving this so late, I completely forgot about my account until I went through some old emails. In terms of the PSU shock issue, it was fixed through a replacement 13 A cable from an old monitor (so @Green Online and @Sean Houlihane's response worked best I think).

\n" }, { "Id": "6807", "CreationDate": "2018-09-02T09:07:26.637", "Body": "

I have printed a couple weeks perfectly fine with my Ender 3. Two weeks ago I changed the firmware but the settings were all fine and resulted in good prints.

\n\n

Now, suddenly during a print the extruder motor started to under-extrude.

\n\n

I thought \"hey, could be the file\" and used a test file that printed well two weeks ago: Under-extrusion, barely a line.

\n\n

I looked at the extruder, no filament grinding, no clicking from lost steps or moving against pressure. The Bowden tube is firmly seated though.

\n\n

I measured what is coming out: instead of 50 mm I ordered to push into the hotend got 28.7 mm. So I went and looked at the steps/mm, which is 93, exactly what it is also on the TronXY X1, which uses pretty much the same extruder setup but for a different style, same sized gear.

\n\n

I cleaned out the nozzle nevertheless, but that didn't change the results.

\n\n

What might be wrong and how can I fix it?!

\n\n

Edit:

\n\n\n", "Title": "Sudden underextrusion on Ender3", "Tags": "|extruder|print-quality|creality-ender-3|", "Answer": "

A close inspection of what happened when printing the first layer resulted in this:

\n\n\n\n

I have no idea why the print had failed due to under-extrusion during the print, but apparently, my immediate tests were flawed enough to not detect the first layer height resulting in getting almost no extrusion. This I mistook for massive under-extrusion, making me believe something else was at fault.

\n" }, { "Id": "6829", "CreationDate": "2018-09-04T12:55:05.430", "Body": "

I have tried to print a 3D model with Creatr HS from Leapfrog. This print failed twice. The 3D model is the same, .gcode the same. Surprisingly it failed at different stage. Please look at the pictures. You can see the place where extruder was when print stopped.

\n\n

\"Same

\n\n

I checked the filament and extruders, it is NOT a problem here. Extruders are not clogged, filament is not tangled nor blocked. Stepper motors stopped working, anyway the display shows that print progresses.

\n\n

I have created a ticket for this issue on Leapfrog support page, but had an answer that this printer is no longer supported.
\n Does anyone has an idea what could be the core of the problem?
\nEDIT
\nToday I have tried print again to observe and answer questions from the comments. Observations are:
\n- The process suddenly stops
\n- Hotend cools down (you can observe it on the picture below - I am using left extruder - as you can see it shows 24/0; 24 is actual hotend temp, 0 - wanted temperature )
\n- Display shows that print is progressing even faster than it would happen when everything is fine (maybe this is not important)

\n\n

\"46
\n\"49

\n\n

EDIT2

\n\n

I have opened the printer to see what is happening inside. The electronics is divided into 2 PCBs. One is an 'heavy job workhorse' (drivers and some logics), the other is a Olimex A20-Olinuxino-Micro. The pendrive with .gcode is connected to this board as well as the display and control joystick (actually encoder).\n\"Olimex\"
\nOne of the USB ports from Olinuxino is connected to 'workhorse' board. Next to the USB port here you can see Rx/Tx diodes.
\nObservations:
\nWhen the print is in active state, two Rx/Tx diodes located on workhorse PCB blinking occasionally (mainly when there is a print head direction change), what brings the suggestion, that the data comes from Olinuxino PCB. When the print stops, these two diodes are blinking rapidly. I am thinking that there is some problem with Olimex board (it is sending uncontrolled data or data is being send too quick).
\nBelow you can see image of drivers board with Rx/Tx diodes continuously on. There is also another Rx/Tx pair and these are also both on. \n\"Driver
\nI'm going to do some search on Olinuxino board/system.

\n\n

EDIT 3
\nFinding: During board and temperature check I have found that the fan cooling bottom Olimex pcb is dead. I have took it out, connect to +24V and... no spinning.
\n\"Dead\nI have replaced the fan to a new one. Also, I have created a 20mm box model for printing, sliced it and tried to print it. No success here. After 2 lines of print sudden stop and Rx/Tx blink. I have downloaded model of Marvin, sliced it and tried to print: all ok!
\nAlso I want to know if the data is continuously fetched from USB or loaded to Olinuxino and then used to print. So, started the Marvin print, removed USB from Creatr HS and yes, the print continues until the end. I am thinking that the data must be stored in Olimex memory and then fetched and fed to drivers PCB. Question is: where in Olimex the data is stored? Is lack of fan (and possibly overheat) damaged it slightly? If this is stored in onboard NAND memory it could cause a problem after a long time - it has 100 000 write/erase cycles.
\n\"NAND
\nNew board would be a solution, unfortunately Leapfrog ran out of them. Olimex is selling boards, will try to buy one. I hope that different revision will work.

\n", "Title": "Leapfrog Creatr HS stops mid print (extruders not clogged)", "Tags": "|print-quality|", "Answer": "

I have solved the problem. After some investigation I have decided to connect directly to the drivers board. It needs to be disconnected from Olimex board and instead connected to the PC, then select 250kbps speed, COM port can be found through Device manager. I have used Repetier-Host in not Easy Mode and all possible messages turned on to see all possible messages. Then I prepared 20mm box to print and tried to print. After two lines voila! Error message:

\n\n
12:35:32.401 : N187 M105*41\n12:35:32.841 : Error:0\n12:35:32.841 : : Extruder switched off. MINTEMP triggered !\n12:35:32.924 : Error:Printer stopped due to errors.  \nFix the error and use M999 to restart!. (Temperature is reset. Set it before restarting)  \n
\n\n

Now I went for quick search and learned that this can be caused by intermittent fault of thermistor or its wiring. \n\"Thermistor\n So, I disconnected thermistor from the PCB and checked its resistance: 170kohm. Now, when I moved the extruders on X axis there was a particular position when thermistor resistance went to infinity. There is a connector in head which went loose. After refitting the printer is behaving correctly.
\nSummary:
\nIt is a shame, that the display on this machine does not present any feedback from the drivers board. Also, the PC connectivity on the rear of this printer does not bring any clear information (rear USB is first connected to the Olimex PCB and then to the drivers PCB, some information from drivers PCB are lost). BTW: Leapfrog drivers board is an Arduino based Marlin firmware. Info sent to serial connection after connectivity established is:

\n\n
Leapfrog Firmware: 2.5  \nModel: CreatrHS \nPROTOCOL_VERSION: 1.0 \nFIRMWARE_NAME:Marlin V1; \nExtruder offset X: 15.23 Y: 0.02\n
\n" }, { "Id": "6832", "CreationDate": "2018-09-05T03:25:42.310", "Body": "

I am attempting to write a basic slicer for some objects I am working with. I need to write a custom slicer as the objects are not polygonal based (they are implicit objects) and therefore cannot be plugged into slic3r. I can easily obtain the perimeter/shell of the objects I am working with and have a few successful prints. What I am having trouble with is how to add infill. I think the biggest hurdle is simply my inability to frame the question properly. How do current software tackle this problem?

\n\n

I don't know of my current approach is feasible but if I have a collection of vectors that represent the path around the outside of the object and a collection of vectors that represent an arbitrarily large infill pattern is there a way to union the two paths together to from an outer path (the object shell) and an inner path that is the infill pattern cut out in the shape of the object?

\n\n

EDIT:

\n\n

Sorry for the lack of clarification. So lets say I cut out the infill pattern to match the inside of the object. How do I then intelligently connect all the broken infill segments together to form an efficient path that doesn't cross gaps or mess the object up in any way?

\n", "Title": "How are infill paths connect to form an efficient path that respect the object's geometry?", "Tags": "|slicing|infill|", "Answer": "

The answer to this is pretty much basic algebra: The software tackles the problem by using a set of functions that generate the infill pattern for ALL the build volume, then discard anything outside the shells. Which is determined by algebra:

\n

Basics

\n

Outline Function

\n

Assume the outline of the body is a function $O(l)$ that has a parameter $l$ for its length. This function can be calculated into XY coordinates, giving us $y\\mapsto O^{xy}(x)$, that is parameterized after $x$, and should give us the values of $y$ for a closed function $O(l)$.

\n

Infill Functions

\n

Now, let's generate a function for infill pattern. Let's make it easy for us and use a diagonals pattern: $I_n(x)=x+n\\times d$\nwhere $d$ is a fixed parameter for "distance to last line" and $n\\in\\mathbb Z$ is the number of the line with 0 passing the origin.

\n

Comparation: Outline=Infill

\n

Now basic algebra! Let the computer solve for each $n$ the term $O(x)=I_n(x)$. The result should be (in the best case) paired points, all on the linear function $I_n(x)$. Sort these points by their correlating $n$ value first, then the $x$ values.

\n

Dealing with the results

\n

Let's assume we have some banana shape and our solutions for n=0 are like this: $P_{i=1 \\to 4}=\\{\\{1,1\\},\\{2,2\\},\\{3,3\\},\\{4,4\\}\\}$

\n

Modeling starter

\n

On the most simple cases, we hope to only have paired results - the outline is closed and thus each line passing it has to cut it in multiples of two. Because we don't allow geometry to be below $\\{0,0\\}$, the line in this example will pass into the body at the first solution of these points and pass out of it at the second and so on. Generally: It moves in at odd and exits at even i. So our infill lines in the example need to connect $\\{1,1\\} \\to \\{2,2\\}$ and $\\{3,3\\} \\to \\{4,4\\}$.

\n

Enhancing the Modeling

\n

checking for tangents

\n

Now, we might have an odd number of points that solve O(x)=In(x) for a given n. Let's assume $P_{i=1 \\to 5}=\\{1,1\\},\\{2,2\\},\\{3,3\\},\\{4,4\\},\\{5,5\\}$.

\n

Now we need to be careful as one of these points is guaranteed to be a point in which $I_n(x)$ is a tangent at of $O(x)$. So, we need to know the first differential of $O(x)$ in the points, which is the tangent at $O(x)$. But we don't need to solve all the points: We know the first should enter and the last exit the body, so we need (for most cases) to only solve this for the points $P_i$ with $i=2 \\to i_{max-1}$. When $O'(x)=I_n(x)$, we got a tangent and remove this point from the list of points to connect with infill lines.

\n

Because we could have several tangents in a set of points, this check has to be done for all sets of points to eliminate these points.

\n

Also, I used the "usually" there by intent: there are cases where the first or last point is a tangent, and because it is easier to cose, we should run the elimination process over all $P_1 \\to P_{max}$!

\n

The new, reduced set of points will be a paired list: $Q_{i=1 \\to 4}=\\{1,1\\},\\{2,2\\},\\{4,4\\},\\{5,5\\}$. The Infil connects $Q_1 \\to Q_2$ and $Q_3\\to Q_4$.

\n

Turning Points into vectors

\n

Now, we have our points $Q_1$ and $Q_2$ (or any other pair of $n \\land n+1$, where n is an element of the odd numbers), both on $I_{n=0}(x)$. How to connect? Easy! $I{n=0}$ is a function, most likely a linear one. Along this line has to be our connecting line from $Q_1\\to Q_2$, so the movement we have to plot is the function of our pattern between the points. For a simple, linear pattern this would be:

\n

$L_1=\\frac{I(x)}{|I(x)|} \\times |\\vec{Q_2}-\\vec{Q_1}|+\\vec{Q1}$

\n

Optimisation

\n

Sorting properly

\n

Now, we have a set of Lines $L_n$, where, as established in the last paragraph, n is an odd number declaring it has the lower-end $Q_n$, and the upper-end $Q_{n+1}$. How do we sort these lines smartly so we have the least movement? Let's take a look at our lists:

\n\n

Shortest movement between prints?

\n

Now, let's do some math again: What is the closest $Q_{a}$ to the $Q_{n+1}$ we did end at after doing the $L_n$ movement? Well, first of all, we need to make sure we don't get back to already moved paths so let's make a new list $R_{i}$, which contains all the $Q_{i}$ we have not yet moved to.

\n

So what is the closest $R_{i}$ to the end point of the path $L_e$ we just moved? Well, easy! Solve $min|R_i-L_e|$ with i being all the odd numbers in the list of $R_{i}$ and $L_e$ the point where the printhead was sent to at the end of the last movement

\n

fewest direction changes?

\n

Always moving just the shortest distance might create a large number of direction changes. So it might be a good idea to keep the point-lists sorted by the parameter n of the function $l_n(x)$ that created the points in the first place, and run down that list from minimum n that generated points (which can be below 0) to the maximum n that generated points.

\n

optimizing direction changes & movement paths

\n

Now, we have 2 approaches that pretty much only follow the pattern. However, we might make our average movement paths more efficient by using a simple trick:

\n

Up to now, all our line functions $l_n(x)$ had the same vector and just a different starting point to one another. So all the starts were on one side of the body, all the ends on the opposite. With a very simple trick on the infill function we can generate a group of functions that alternate the sides of the end-points between each line, jsut by adding an inverse element:

\n

$L_n(x)=-1^n\\times l_n(x)$

\n

Now, after all the movements with the same $n$ are done, check for the closest starting point (which should be on the same side, but is not necessary the neighboring line), and go down that line fully, eradicating these points from the list of remaining points $R_{i}$. Once back on the side we started first at, we look for the closest unused point again, run down that line, rinse and repeat.

\n" }, { "Id": "6845", "CreationDate": "2018-09-06T04:59:31.737", "Body": "

When manually controlling the z-axis in reprap, it works fine. If I use the home button, it doesn't move. Instead, it stalls/freezes/hangs. It attempts to turn, but can't successfully make a full turn.

\n", "Title": "Weird stepper issue when setting up Marlin printer", "Tags": "|marlin|z-axis|", "Answer": "

If you run M503 you should be able to see your max speeds. They, along with your homing speeds, are probably far too high and causing skipped steps (no movement). You can lower them down temporarily with M203. http://marlinfw.org/docs/gcode/M203.html

\n" }, { "Id": "6849", "CreationDate": "2018-09-06T18:12:10.460", "Body": "

The title almost says it all.

\n\n

I was searching on websites like amazon.com and ebay.com for a replacement of the temperature sensor of my Ultimaker 1, but could only find replacements for Ultimaker 2 (PT100), but I know those aren't compatible.

\n\n

What specifications have replacement temperature sensors that work with the tc2 signal amplifier of the Ultimaker Original?

\n", "Title": "Ultimaker Original temperature sensor replacement?", "Tags": "|replacement-parts|ultimaker-original|", "Answer": "

You need a thermocouple, not a thermistor. Also, as you say the Ultimaker Original+, and the Ultimaker 2, (which both use version 2.x.x boards) use a PT1001.

\n

According to this post, on E3D Thermocouple mount on UMO+, a k-type thermocouple will do:

\n
\n

I am changing over to E3D V6 on my UMO, and have elected to used the k-type thermocouples sold by e3d with the standard/original UMO amplifier board.

\n
\n

Or to be more precise, from this post, on\nTHERMOCOUPLE SENSOR REPLACEMENT, it is a 3\u00a0mm k-type thermocouple:

\n
\n

I would ask the reseller to measure the thermocouple to be really 3mm. I never bough a thermocoupler so no idea. I know that some aliexpress sellers did sell me a pt100 with 3.05mm (it doesn't fit in) so is good idea to ask for a photo with a caliper before paying.

\n
\n

The thermocouple in question, on AliExpress was this, SWMAKER Thermoelement Typ K (TC) f\u00fcr Ultimaker Original 3D drucker 3mm K thermoelement typ f\u00fcr Ultimaker Original 3D drucker, ~$10

\n

Or this, considerably more expensive at \u20ac33, Ultimaker Original - Thermocouple Sensor (out of stock):

\n

\"Ultimaker

\n
\n

With respect to the firmware, from this post, on E3D Thermocouple mount on UMO+

\n
\n

Normally you won't have any modifications to do cause your UMO already works with a thermocouple.

\n
\n

So, assuming that you have an AD595 amplifier and not a MAX6675, ensure that you have

\n
#define TEMP_SENSOR_0 -1\n
\n

set in Configuration.h

\n
//===========================================================================\n//=============================Thermal Settings  ============================\n//===========================================================================\n//\n//--NORMAL IS 4.7kohm PULLUP!-- 1kohm pullup can be used on hotend sensor, using correct resistor and table\n//\n//// Temperature sensor settings:\n// -2 is thermocouple with MAX6675 (only for sensor 0)\n// -1 is thermocouple with AD595\n// 0 is not used\n// 1 is 100k thermistor - best choice for EPCOS 100k (4.7k pullup)\n// 2 is 200k thermistor - ATC Semitec 204GT-2 (4.7k pullup)\n// 3 is Mendel-parts thermistor (4.7k pullup)\n// 4 is 10k thermistor !! do not use it for a hotend. It gives bad resolution at high temp. !!\n// 5 is 100K thermistor - ATC Semitec 104GT-2 (Used in ParCan & J-Head) (4.7k pullup)\n// 6 is 100k EPCOS - Not as accurate as table 1 (created using a fluke thermocouple) (4.7k pullup)\n// 7 is 100k Honeywell thermistor 135-104LAG-J01 (4.7k pullup)\n// 71 is 100k Honeywell thermistor 135-104LAF-J01 (4.7k pullup)\n// 8 is 100k 0603 SMD Vishay NTCS0603E3104FXT (4.7k pullup)\n// 9 is 100k GE Sensing AL03006-58.2K-97-G1 (4.7k pullup)\n// 10 is 100k RS thermistor 198-961 (4.7k pullup)\n// 11 is 100k beta 3950 1% thermistor (4.7k pullup)\n// 12 is 100k 0603 SMD Vishay NTCS0603E3104FXT (4.7k pullup) (calibrated for Makibox hot bed)\n// 13 is 100k Hisens 3950  1% up to 300\u00b0C for hotend "Simple ONE " & "Hotend "All In ONE" \n// 20 is the PT100 circuit found in the Ultimainboard V2.x\n// 60 is 100k Maker's Tool Works Kapton Bed Thermistor beta=3950\n//\n//    1k ohm pullup tables - This is not normal, you would have to have changed out your 4.7k for 1k\n//                          (but gives greater accuracy and more stable PID)\n// 51 is 100k thermistor - EPCOS (1k pullup)\n// 52 is 200k thermistor - ATC Semitec 204GT-2 (1k pullup)\n// 55 is 100k thermistor - ATC Semitec 104GT-2 (Used in ParCan & J-Head) (1k pullup)\n//\n// 1047 is Pt1000 with 4k7 pullup\n// 1010 is Pt1000 with 1k pullup (non standard)\n// 147 is Pt100 with 4k7 pullup\n// 110 is Pt100 with 1k pullup (non standard)\n#define TEMP_SENSOR_0 -1\n
\n
\n

However, according to\nQuestions about thermocouple and thermistor, the UMO board is 1.5.7, and can use NTC100K thermistors and (maybe - but it isn't clear) a 4.7K pullup resistor:

\n
\n

Firstly we have to do the Challenge with a Sainsmart UM Kit with that UM 1.5.7 Board.

\n

We also have bought the print heads by E3D model V6 which have including ntc 100k thermistors which have already tables in marlin. As the V1.5.7. Board works with thermocouple, there was the question how to connect this. I got already an answere to just connect Signal and GND, to the board but I'd have to look for the resistors behind the connectors. Is it true, that I need a different resistor for each type of thermistor?

\n
\n

However, it is probably advisable to stick with the 3 mm k-type thermocouple, for simplicity's sake.

\n
\n

Footnote

\n

1 Apparently for these reasons, from this post, on\nTHERMOCOUPLE SENSOR REPLACEMENT:

\n
\n

The UMO has the 'old' UltiBoard 1.5x which is a shield for an Arduino Mega, with the Pololu / Allegro stepper drivers plugged on it (Like the RAMPS controllers).

\n

At that time RTDs where not very popular, and most printers came with thermistors. To improve the temperature feedback loop Ultimaker decided to use a thermocouple with an amplifier on top of the print head, which gives better results in particular at higher temperatures.

\n

If you want to use a thermsitor instead of the thermcopule, you just have to swap the connections, and add a resistor on the board (the place for the resistor is foreseen on the board).

\n

You can even go for an RTD using E3D's adapter plate.

\n

The UMO+ is using the UltiBoard 2.x which is the same as in the Ultimaker 2, so it is PT100...

\n
\n

Note: RTD = Resistance Temperature Detector (such as a Pt100). A thermistor is similar to an RTD, but contains a ceramic or polymer resistor en lieu of metal.

\n" }, { "Id": "6853", "CreationDate": "2018-09-06T22:12:11.863", "Body": "

I've got a bracelet concept that I've sketched up as a flat design. I'm trying to found a route by which I can extrude this into a 3d object (depth map?), curve it into a bracelet, then ultimately create a STL file out of it. I'm having trouble finding a way to do this that allows me to \"warp\" the flat object into a bracelet before I try to print.

\n\n

Is there a recommend technique for this? I'm not worried about representation of the picture; it's effectively meant to be an 'etched' pattern.

\n", "Title": "Route to transform 2d image (depth map) into a curved bracelet (and STL file)?", "Tags": "|3d-models|stl|", "Answer": "

This is the best and simplest way I've found to transform a flat design into a 3D object that you can then save as an STL file: Blender-Converting 2D Image to 3D Object

\n\n

This solution requires you to use Blender and Inkscape, both softwares are open-source and available for free at this time.

\n\n

As suggested below by @Greenonline, I will provide a step by step in case the video is removed from Youtube. I'm sorry that I cannot provide screenshots at the moment, but I'm on a trip and writing from an old laptop that cannot run either Blender or Inkscape.

\n\n
    \n
  1. Load your image in Inkscape.

    \n\n
  2. \n
  3. Select the image or the portion of it that you want to turn into a 3D object.

    \n\n
  4. \n
  5. Go to Path > Trace Bitmap, or use Shift+Alt+B, and a pop-up window will appear. Keep the default settings, just make sure that the 'Remove background' option, located at the bottom of the window, is checked, then click OK.

    \n\n
  6. \n
  7. Got to File > Save As and save your file as 'Plain svg'.

    \n\n
  8. \n
\n\n

And you're done with Inkscape, time to switch to Blender.

\n\n
    \n
  1. Go to File > Import > Scalable Vector Graphics (.svg) and import the picture you just created in Inkscape.

    \n\n
  2. \n
  3. Your picture has been loaded in Blender as a Curve object. Go to the 'Curve' tab and in the 'Geometry' sub-menu you can use the 'Extrude' slider to give depth to your picture.

  4. \n
  5. Finally, once you've extruded your curve to your satisfaction, go to Object > Convert To > Mesh from Curve/Meta/Surf/Text, or use Alt+C, and voila, you're done!

  6. \n
\n\n

You know have a nice 3D object made of polygons that you can modify at your leisure using Blender's tools.

\n" }, { "Id": "6859", "CreationDate": "2018-09-07T11:15:28.410", "Body": "

When exchanging the mainboard of my Ultimaker Original, I found the new (unoriginal) mainboard to have a condensator that is quite a little higher than on the original, which means the fan duct doesn't fit any more.

\n\n

I read somewhere that those A4988 stepper motor drivers don't have to be actively cooled, but a heatsink will be sufficient:

\n\n

\"A4988

\n\n

After testing the theory I found the heatsinks to be really hot, but not too hot to touch. Sadly I do not have any means of temperature measurement other than my fingers...

\n\n

How hot do they normally get, when should I abort printing and look for a different solution?

\n\n

Are there any good solutions other than the original cooler & cooling duct?

\n", "Title": "A4988 driver cooling specifications", "Tags": "|ultimaker-original|", "Answer": "

The answer to that question is not as simple as it looks as we don't see the hardware.\nMain issue is the current pointed by Himanshu, that generates most of the temperature in the silicon chip.

\n\n

I was using drivers without active cooling, but my case was open so there was enough heat dispersion.

\n\n

You could do a dry run - by creating a command file with G-code like below

\n\n
G28 \nG1 X200 Y200 F6000 \nG1 X0 Y0 G1 X200 Y200 \n... \n... copy and paste many times to have a 10 minutes run on the printer\n
\n\n

That will keep the stepper busy and you could observe radiator temperature and probably see/hear miss-steps.

\n\n

other solution could be a duct like this one

\n" }, { "Id": "6860", "CreationDate": "2018-09-07T11:23:42.370", "Body": "

I am using cura with M3D entry level printer.

\n\n

When I print things more than 6-7cm/2.5-3inches, at the end of the print, the hot end sink back into the print then gets stuck as it tries to return to idle position.

\n\n

Please see the attached photo. Has anyone had this problem? Thank you\"enter

\n\n

The last 20 lines of gcode is:

\n\n
G0 X50.83 Y49.982\nG1 F600 X50.541 Y49.975 E4399.62304\nG1 X50.268 Y49.985 E4399.62637\nG1 X49.999 Y49.999 E4399.63024\nG1 X49.73 Y50.012 E4399.6341\nG1 X49.457 Y50.023 E4399.63742\nG1 X49.168 Y50.016 E4399.63941\nG0 F7200 X49.28 Y49.87\n;TIME_ELAPSED:6301.614981\nG1 F1500 E4393.13941\nM107\nM104 S0\nM140 S0\n;Retract the filament\nG92 E1\nG1 E-1 F300\nG28 X0 Y0\nM84\nM82 ;absolute extrusion mode\nM104 S0\n;End of Gcode\n
\n", "Title": "3D printer ends print by sinking in to it and getting stuck", "Tags": "|print-quality|", "Answer": "

From the provided G-code:

\n\n
G28 X0 Y0\n
\n\n

The meaning of the G28 code is move printhead to home. In theory, the presence or absence of X, Y, and Z values are used to indicate which axes the printhead should move on, but as the RepRap wiki notes, some printers ignore this, typically interpreting any G28 code as \"move to 0,0,0\". In particular:

\n\n
\n

Because the behavior of G28 is unspecified, it is recommended not to automatically include G28 in your ending GCode. On a Cartesian this will result in damaging the printed object. If you need to move the carriage at the completion of a print, use G0 or G1.

\n
\n\n

You should probably replace the code with G1 X0 Y0 Z[value near the top of your printer]. You could simply replace G28 X0 Y0 with G1 X0 Y0, but that risks dragging the tip of the nozzle across the upper surface of the finished print.

\n" }, { "Id": "6869", "CreationDate": "2018-09-08T15:54:48.133", "Body": "

After noticing that my ender 3 couldn't print a 'perfect' circle I decided to use a calibration cube to see if that could be the issue. These were my results:

\n

\"X

\n

I did have some successful prints before noticing these problems but it would be great if anyone could help/find a solution.

\n

I think it may be because of the belts tension as I had accidentally over stretched it by putting it over the wheels instead of under: is it worth getting a new belt then?

\n

I use 20% infill, with triangle pattern in cura, using the standard 0.4mm nozzle.

\n

Edit

\n

After doing a different testprint and measuring, it revealed that both had an accurate 20mm Y and Z, but only an 18.5mm X. Might I have destroyed my belt when I mounted it overstrained?

\n

Edit2:

\n

After purchasing a new X belt, the calibration cube was much closer at 19 x 20 x 20. But still not at a good enough tolerance for my liking. Additionally, when printing a cylinder, two relatively flat edges were printed with it either side (perpendicular to the x-axis).

\n", "Title": "Ender 3 Calibration Cube Fail + Can't Print Circles", "Tags": "|print-quality|creality-ender-3|", "Answer": "

First, check that the calibration part you have is actually a cube. Maybe you downloaded a trick 'calibration cube' model. Cura will show he overall dimensions of any model which it is slicing.

\n\n

Ignore any print quality issues first off. A hollow calibration cube is mainly used to check the X/Y/Z motion and so long as you print at a sensible (slow) speed), the other parameters shouldn't have much effect (so long as the first layer is OK, and you get within 30% of the right extrusion).

\n\n

If the calibration cube is within +/- 0.5mm on each side, it's probably good. Under/over extrusion, caliper technique, etc, can explain this sort of variation, and you might want to use a bigger test part like this star on my thingiverse account once you think you're close.

\n\n

The 'steps per mm' setting is determined within the firmware, it is part of how your printer consumes G-code. For a delta printer, for example, there is some fancy maths to combine X/Y/Z into the motor movements. This means that you can't change steps/mm in Cura, you need to either change the firmware defaults and 'reset to factory settings', or modify the parameters in EEPROM by sending G-code.

\n\n

Seeing too small a calibration part (for a Cartesian printer) can easily be explained by problems in the printer's motion system:

\n\n\n\n

An annoying feature of a lot of these problems is that for any particular repetitive sequence of steps, you might see a very consistent result. Even thought to problem is something being badly adjusted, it gives the impression that the problem is calibration.

\n\n

The drive wheel is held in place with a grub screw, see this diagram from the Anet-A8 assembly diagram.

\n\n

\"Anet

\n" }, { "Id": "6878", "CreationDate": "2018-09-10T22:44:53.173", "Body": "

Really strange observation with my MP Select Mini V2 printing PLA. I've printed a dual-fan shroud so I can have a dedicated part-cooling fan. I've noticed that when I turn it on, the extruder starts stringing everywhere. Little tiny spider-web wisps. No amount of temperature change will make them go away (printing at 200 °C). But if I simply turn off the part-cooling fan, I get a perfect print. This doesn't make sense to me - isn't the part-cooling fan supposed to reduce stringing by cooling the filament faster?

\n\n

More info: Using Ultimaker Cura 3.4.1, retraction enabled at 3.5 mm and 40 mm/s, which works great when the part-cooling fan is OFF. But when I turn it on without changing any other settings, I get stringing everywhere.

\n\n

Please see picture below. The part on the left was printed using the part-cooling fan and the one on the right was printed without the part-cooling fan. No other changes. The part-cooling fan is pictured in upper left (bottom fan).

\n\n

\"Two

\n\n

Why does my part-cooling fan cause stringing?

\n\n

Note that I print at 0.175 mm layer height, or \"Normal\" for the MP Mini. The cooling fan can be controlled but if I turn it completely off then the cold end part of the nozzle won't get actively cooled. Using Cura I can slowly increase the fan over the first several levels as well, but I'm worried that if the cold end doesn't stay cold it will get clogged up with melted filament. I will try some different fan speed settings and report back - currently running at 100 % all the time.

\n\n
\n\n

I went back to the stock shroud with single fan to remove variables. The stock shroud has a small vent at the bottom where air blows over the part, but not much. I had the same problem - I had to put a piece of tape over the blow hole to get the part to print without strings. I also couldn't get the PLA to stick to the bed easily when the blow hole was open. I'm not using the heated bed in any cases - I've found I can print most PLA without heating the bed at all. I'm beginning to think maybe this PLA is just extremely temperature sensitive. The brand is 3D Solutech Printer Filament Real Black.

\n", "Title": "Why does my part-cooling fan cause stringing?", "Tags": "|extruder|print-quality|pla|cooling|", "Answer": "

Prior to a move the print cooling fan causes the filament to cool on the outside while the nozzle is still hot, when it then moves it causes a strings to form that will be cooled instantly. This means that the cooling you have is too much and should be reduced.

\n\n

This printer has a single fan to cool the cold-end and the print through a slot with the same fan. As this fan can be scheduled, it would be recommended to create a dual fan on the carriage where you have a dedicated fan cooling the cold end at a fixed voltage, while the print is cooled by a separate fan that can be scheduled through slicer settings.

\n" }, { "Id": "6879", "CreationDate": "2018-09-11T05:34:32.090", "Body": "

A corner of my model curled up, causing the head to crash into it. The Prusa MK3 detected the error, printed a message to that effect, and moved the print head to the home position.

\n\n

I was surprised when printing immediately resumed, and of course immediately went awry as it pulled the model off the print bed and dragged it around.

\n\n

My questions:

\n\n\n", "Title": "Does Octoprint stop printing on Crash Detection?", "Tags": "|prusa-i3|octoprint|", "Answer": "

You are describing precisely how the crash detection should work on the Prusa Mk3, so it is doing exactly what it should do. If you look into this video from Jozef Prusa you will see him explaining that in case a crash is detected (steps missed are registered because the Mk3 uses trinamic stepper drivers), the print head is homed and will immediately restart printing.

\n\n

Please do read this interesting post; it describes your similar problems: after detection it resumes. He noteworthy mentions that even a few lines of G-code went missing, this is probably caused by the fact that OctoPrint buffers a few commands or something that get lost because the Pruse Mk3 crash fail safe kicks in.

\n\n

In effect, OctoPrint is just a simple supplier of G-code commands and does not recognize the problem to act on it and will therefore continue sending new commands after the Prusa Mk3 crash has recovered (sending ok to OctoPrint for receiving new commands) from the crash and re-homed and got the temperatures back to normal levels.

\n\n

An existing plugin for OctoPrint may help you in this case. The Action Trigger Plugin is able to detect events, but have to be implemented in your firmware. E.g. the action for filament describes:

\n\n
\n

This trigger will pause the print and home the X and Y axis, giving\n the user the opportunity to change out the filament. The print needs\n to be resumed manually through the UI.

\n
\n\n

Maybe it can be configured to pick up the existing event and issue a pause. It could be worth looking into this.

\n\n
\n\n

EDIT
\nLooking closer to the ActionTrigger plugin I noticed that you cannot configure it and the \"manual\" is very terse. I guess this might involve some serious hacking...

\n" }, { "Id": "6883", "CreationDate": "2018-09-11T15:35:51.963", "Body": "

problem solved by resetting cura.

\n\n

I have problems like this: How can I fix this? I can't find the right setting.

\n\n

\"No

\n\n

Also, what are these yellow lines? Do you know how to remove them? They disappear when I disable to show the brim/raft/skirt.

\n\n

\"Yellow

\n\n

\"Yellow

\n", "Title": "(Cura setting) How can I fix this? No top layer", "Tags": "|ultimaker-cura|software|", "Answer": "

This problem appears when a face is inverted, so the slicer confuses and expect the other wall to close the object. So you need to reverse that face to show the face out and the back face inside the shape to be filled

\n\n

\"enter

\n\n

I will edit this answer with more examples

\n" }, { "Id": "6890", "CreationDate": "2018-09-12T00:02:34.023", "Body": "

Can the glTF format be used for 3D printing?

\n\n

If not, is there any tool can convert it to another format such as STL, OBJ, STEP, and IGES?

\n", "Title": "Can the glTF format be used for 3D printing?", "Tags": "|3d-design|", "Answer": "

No, gltf is not a format that slicers accept commonly, and indeed, it is not even intended to be reverseable in the 1.0 format version. This has changed a little for the 2.0 standard. Some programs that allow exporting into the sliceable stl and obj formats can also import gltf:

\n\n\n\n

You have to be careful though: formats like dae or stl are meant to transfer 3d objects usually without the loss of information or with just a minimal loss (stl, for example, does technically not contain a scale natively), while gltf is end users and does not contain all the information. This can lead to models getting distorted and destroyed on importing them into a rendering or modeling software.

\n" }, { "Id": "6896", "CreationDate": "2018-09-12T15:00:15.403", "Body": "

I'm looking for clamps to fix the glass on my heatbed. After some search I found that some people use clamps printed of PLA. Can I use PLA clamps for a heated heatbed (~60 \u00b0C)?

\n\n

I also tried foldback clips but they block my nozzle. The Anet A8 starts in the front left corner. When I start to print, the nozzle moves a little bit up on z, then up on y and right on x. At this first move it moves into the fold back clip. I'm looking for a way to fix the corners and not to fix the edge in the middle.

\n\n

What other clamps or clips can I use to fix the glass (~3 mm) on the heatbed (~2.5 mm)? I don't want to use glue.

\n\n

My printer is an Anet A8.

\n", "Title": "Clamps / clips for glass on heatbed", "Tags": "|heated-bed|anet-a8|glass-bed|", "Answer": "

\"enter

\n\n

I went to OfficeMax and got some Small clips. They are bigger than the Micro/Mini clips that ship with the printer and seem to fit just right on a glass plate.

\n\n

One of the Mini clips that shipped with the Ender 3 is shown to the left. The Small size that works best is obviously centered. The Medium in the back seem way too big.

\n\n

For an added bonus, they sell this sandpaper with a rubber back and it really increases traction. An oversize chunk of that worked really well. But it's optional.

\n" }, { "Id": "6902", "CreationDate": "2018-09-12T22:55:02.340", "Body": "

I have been having issues with a vase print using a FlashForge Creator Pro. \nI am using the default slicer for the printer(FlashPrint) because the Creator Pro only accepts .x3g rather than .g/.gcode.

\n\n

The issue is that the printer seems to be both under extruding and over extruding.

\n\n

\"Under

\n\n

I am printing ASA filament at 0.18 mm layer height at 240 °C with a heated bed at 110 °C. I am running the print with the fan on. I am also running at 80 mm/s print speed.

\n\n

I have tried both lowering and raising the temperature, changing the print speed from 80 mm/s to 60 mm/s, and changing the extrusion multiplier.

\n\n

At this point I am not really sure what to do in order to get this to print nicely, any advice would be welcome.

\n\n
\n\n

After a few more prints and the link that \"E-Doe\" posted I am beginning to think that the problem might be with the z-axis. My reason for thinking this is that the layers that bulge out do so consistently for the entire layer and then stop for a bit. I think I also ruled out temperature variation. I measured the temperature in the enclosure with a separate device and it stayed pretty constant the whole time.

\n\n

Not really sure how to fix that but I will call this question answered.

\n", "Title": "Unknown issue affecting print quality", "Tags": "|print-quality|troubleshooting|extrusion|asa|flashforge-creator|", "Answer": "

A very helpful page for troubleshooting common errors is: Print Quality Troubleshooting Guide - Lines on the Side of Print

\n\n

It seems like your problem is inconsistent extrusion or temperature variation. From the photo you posted I guess that you use a big diameter nozzle. Keep in mind that your extruder might not be well equipped to deliver such a large amount of plastic consistently. This most likely is a problem with the heating capacity.

\n\n

You can try to lower the speed even more to give your extruder more time to heat the plastic.

\n" }, { "Id": "6904", "CreationDate": "2018-09-13T05:57:50.193", "Body": "

I asked this question as a small question under an answer of another question (How bed leveling is achieved without table screws?), but have been asked to post this as a separate question to answer it with more detail conform SE policy.

\n\n
\n\n

One question with respect to automatic bed leveling (also known as ABL): If the bed is tilted and the auto leveling measured this and then the printer adjust the z axis over the position on the bed, will a x-y-point on the first layer be located exactly under the (in model) same x-y-point in a higher layer, or will it be shifted to? I mean, if the bed is 1 mm lower on one side over 10 cm then a top corner of a 10-cm-cube should be also 1 mm shifted, so that the cube still is real cubic and not sheared.

\n", "Title": "Does auto leveling result in sheared prints?", "Tags": "|bed-leveling|", "Answer": "

Using automatic bed leveling assists you in getting the print to stick better to the build platform as a result of the print head following the non-uniform geometry or tilt of the build plate.

\n

In, e.g. Marlin Firmware, the bed level correction is fading out over a predefined distance, this is determined by the constant #define ENABLE_LEVELING_FADE_HEIGHT in the printer firmware Configuration.h file and the height can be set with M420 Z<height> (see here).

\n
\n
M420 S1  ; Enable compensation using current grid/mesh\nM420 Z10 ; Gradually reduce compensation until Z=10\n
\n
\n

If Z equals zero, leveling compensation will be fully applied to all layers of the print object.

\n

Yes, a cube will not be perfectly cubic, that is why even with auto bed leveling you need to provide a bed as level as possible, it only should correct for very small deviations.

\n

When the print is smaller than the fade out height, you would indeed get a sheared print, if larger, then the top of the print would be level with respect to the reference plane (machine true; this is the level of the X-Y plane, basically the level of the hot end carriage). Note that that still can be skew if the printer is improperly calibrated (e.g. when using 2 Z steppers or misalignment of the height on either side).

\n" }, { "Id": "6914", "CreationDate": "2018-09-14T17:06:07.100", "Body": "

The pictures explain my problem. I have already tried to reduce the retraction but that showed no effect. Thank you for your help.

\n\n

The effect is a total stop of the print (no material is extruded).

\n\n\n\n

I recently added this new feeder aluminium block because the 3D printed stock version was bad quality.

\n\n

\"Filament

\n\n

\"Close

\n", "Title": "Filament isn't going into the Bowden tube, instead it goes \"into the room\"", "Tags": "|filament|extruder|creality-cr-10|", "Answer": "

You just have to cut the filament at a 45\u00b0 angle. Then push down the extruder and push the filament in. Then heat up the nozzle. Next, hand feed the filament until the filament starts oozing out.

\n" }, { "Id": "6922", "CreationDate": "2018-09-15T22:04:02.290", "Body": "

There's PLA filament clogged in my Bowden tube, is there a best practice for cleaning it out or do I need to replace the whole tube?

\n

Also, the couplings are totally stuck, so I guess those would need replacement too or are there ways to get stuck couplings off?

\n

Click here for a video.

\n", "Title": "Bowden tube clog", "Tags": "|pla|ptfe-tube|bowden|", "Answer": "

Most standard filaments soften at 200\u00a0\u00b0C, meaning a pot of near-boiling water should soften whatever is stuck, allowing you to either fuse another strand of filament to it or use it to push it out of the tube.

\n" }, { "Id": "6927", "CreationDate": "2018-09-16T20:13:21.860", "Body": "

I have DEM with 1 m resolution, how can I create 3D printing file (e.g. STL, OBJ and IGES etc.) from DEM?

\n", "Title": "How to create 3D printing file (e.g. STL, OBJ and IGES etc.) from DEM?", "Tags": "|software|", "Answer": "

This question has been answered on a sister site, GIS Stack Exchange. It's a rather comprehensive answer posted, which means it's impractical to summarize here.

\n\n

GIS answer

\n\n

The answer does indicate that a plug-in for QGIS software is needed in order to accomplish this objective. If you do not use/have the QGIS software, my answer is invalid. All three answers to the question linked above appear to point to the same program/plugin.

\n" }, { "Id": "6937", "CreationDate": "2018-09-18T00:34:45.077", "Body": "

I've been looking into the viability of manufacturing a replacement part for a kitchen blender that has a broken part. I found this page that talks about what makes a print food safe. One of the items mentioned was:

\n\n
\n

... a brass extruder may contain lead, and lead contamination can cause some nasty health problems. ...

\n
\n\n

I own an Ender 3, and I haven't replaced the nozzle yet. How can I tell if my printer is capable of creating food safe prints in its current state?

\n", "Title": "Ender 3 is capable of food safe printing?", "Tags": "|safety|food|creality-ender-3|", "Answer": "

Parts printed in pla are food safe, as pla is usually made from corn starch. However, most 3d printed parts would not be food safe for they have many crevices where bacteria can grow. Also, if you printed with materials other than pla with that nozzle, traces of the material (which would make it into the print.

\n" }, { "Id": "6938", "CreationDate": "2018-09-18T02:07:43.240", "Body": "

I am a newbie to 3D printing and ran into a weird infill line on my second 3D printing object on a new Qidi X-Pro machine (which works great). I've included a screenshot of the infill line, which is deliberately printed the full height of the object. I'm thinking this line has been deliberately inserted by the Qidi slicer for some reason, but I have no idea why. Do all slicers generate these kinds of lines? If so, why?

\n\n

\"Print

\n", "Title": "What is this weird support line doing in this print?", "Tags": "|infill|", "Answer": "

Here have a posible solution -> How to remove unwanted filament trails from sharp corners

\n\n

I think, that can be a combination of z-hopping and combing-mode in the slicer that you use.

\n" }, { "Id": "6945", "CreationDate": "2018-09-18T19:44:50.687", "Body": "

I am a complete newbie at 3D printing (although learning fast). I switched from the default Qidi slicer to the Cura slicer on a Qidi X-Pro machine (which is wonderful). Using: plate 50 °C, 0.4 mm nozzle 200 °C, 1.75 mm PLA, print speed 20 mm/s outer wall, 40 mm/s inner wall, travel speed 100 mm/s (the cura default, I think).

\n\n

On two different prints, the same far corner (front left) had initial infill problems. Does anyone know why, or how I can solve it? My working theory is that the nozzle shoots across the whole print at high speed to start again at that corner and sometimes the infill filament doesn't bite enough to stick.

\n\n

Eventually one of the infill layers gets a bite and the rest of the print is fine. But it's disturbing that it occurs on different prints. (I carefully level the plate before every print, so I don't think that's the issue.)

\n\n

\"Front

\n\n

\"Front

\n\n

UPDATE

\n\n

In another question here, turning the Cura combing setting off solved an issue. It probably solved the issue described here, too, or at least contributed to solving the problem described here.

\n\n

My theory is that with combing on (as it was for that question), the oozing pulls filament out of the nozzle during the travel, so there's not enough filament at the destination corner to bite and bond to the corner wall. Turning the combing setting off seems to have solved the missing infills in the corner. It also greatly improved the quality of the infill walls, too, which relates to the problem described in this question.

\n\n

UPDATE: Here's a paragraph from a forum posting here that explains how oozing caused by combing can result in \"underextrusion effects.\" That's what I think is happening in this post - the nozzle gets to the corner, but it has no filament to bite and bond with the wall. Credit to @0scar for giving me the forum link. Quote from the link:

\n\n
\n

Even on interior layers, combing as implemented in Cura is a bad idea,\n as it doesn't perform retraction, and that can result in the head\n oozing as it moves, especially as each time the head crosses a line of\n infill it can tend to pull our some plastic. You can also get some\n long moves (e.g., all the way around the curve of a 'C' shaped object\n when moving from one side of the opening to the other). This can\n result in the head being empty when it starts printing the perimeter\n again, with resultant under-extrusion effects.

\n
\n", "Title": "Infill failure at the far corner of two different prints", "Tags": "|infill|", "Answer": "

Now that you are using Ultimaker Cura, the default (probably hidden) parameter Z Seam Alignment is set at Sharpest Corner, so it will always start in a sharp corner. The default retraction settings are pretty high in Ultimaker Cura (I think about 6.5 mm as default Retraction Distance). What happens here is that the filament is not available for printing in time (this is dependent on the extruder type), so the distance should be less than the current value.

\n\n

It is highly recommended to print a retraction test to find what is best for your setup. For me the 6.5 mm works very well on my Ultimaker 3 Extended and my large custom CoreXY printer as they both use a Bowden extruder setup using 2.85 mm filament, this can be different for your printer. Choosing a different Z Seam Alignment option, e.g. random prevents the use of the same location at every height, but could result in a less aesthetic print.

\n\n

Furthermore, you could try to visualize the G-code in a viewer, this can be done in the Ultimaker Cura slicer itself, but online viewers are available. After slicing your product, change the combo drop down box from Solid view to Layer view to show the sliced object. Nowadays there are 2 sliders to interface with the model; the vertical sets the layer, the horizontal the progress of the layer. It is advised to play with this to understand how your print is actually printed.

\n\n
\n\n

\nAbout your settings, print speeds are not high (could be increased), travel speed is fine (is only for non-printing moves). The brim is looking good and the level of your bed is also good.\n

\n" }, { "Id": "6947", "CreationDate": "2018-09-18T20:13:46.643", "Body": "

I have a problem with my Anet A8. It is a little strange because it does not always happen.

\n\n

When I turn on my printer, it freezes on boot in the second splash screen.

\n\n

If I turn off and turn on several times, the printer works OK.

\n\n

The printer firmware is:

\n\n\n\n

It was modified by me but I promise that I did not touch the boot section.

\n\n

Could the Anet A8 motherboard be dying ? (before this issue it worked perfectly)

\n\n\n", "Title": "Anet A8 - SkyNet3D freezes randomly at boot", "Tags": "|marlin|anet-a8|skynet3d|", "Answer": "

The Anet A8 is a low end printer which works better after installing a better (read safer) than stock firmware (stock firmware does not have thermal runaway protection which is considered to be a hazard). It is good that you flashed another firmware that does have thermal runaway protection.

\n\n

The electronics of the Anet A8 are also questionable, the stock connectors are not rated for their task and wires could have been of better quality and thickness.

\n\n

Now that your older SkyNet3D firmware does not boot properly anymore, it would be time to check all the leads, possibly use new wires of proper gauge and flash a new version of a more recent firmware. SkyNet3D has been forked from Marlin Firmware and is no longer maintained as all functionality has been implemented for Anet printers in the Marlin sources.

\n\n

If your printer still is giving you problems then when booting, a new set of electronics (e.g. board and steppers) would help out.

\n" }, { "Id": "6959", "CreationDate": "2018-09-20T08:26:11.787", "Body": "

I'm using a 3DTouch sensor (from Geeetech) on a self-build CoreXY printer (HyperCube Evolution). The sensor worked perfectly until not being used for a month.

\n\n

The symptoms are that it fails to deploy the pin on the second probe deployment so basically it fails after the first probe is triggered by the raised platform.

\n\n

On homing Z (not only for G28 or \"home all\" but also for G28 Z to \"home Z axis\") and goes into some sort of a fail-safe mode where the LED keeps flashing. Sometimes it does deploy and finishes the homing instructions and continues to the G29 instruction (note that double probing #define MULTIPLE_PROBING 2 is enabled in the Marlin 1.1.8 firmware) and show similar symptoms as described above for the G28 command. Note that failing to deploy the probe sometimes leads to a rising build platform that does not stop, other times some fail safe kicks in and the bed lowers and aborts the print.

\n\n

To lift the fail-safe condition, a reset of the sensor is required (this can be done through the LCD or by command M280 P0 S160).

\n\n

What I have done to solve the problem is to isolate the wires from the 3D touch sensor as I was under the impression that this was related to electromagnetic interference (therefore when the sensor is probing, the bed and heater cartridge are not powered momentarily), but that did not help.

\n\n

Any suggestions and solutions to get it working again are appreciated.

\n", "Title": "3DTouch sensor fails after probe deployment and triggering", "Tags": "|marlin|z-probe|bed-leveling|3dtouch|", "Answer": "

After troubleshooting I have managed to fix the sensor problems.

\n

(Please take note of the update at the end of the answer below as the problem started again shortly after the so-called fix...)

\n

Digging into several posts I found this post that hints to a solution:

\n

\"Instructions

\n

After readjusting, the sensor worked and I was able to produce a print, but the next day it failed again... What I found is that the bed and extruder are NOT shut off while probing although that should have been enabled in Marlin 1.1.8. I assumed that this de-powering is only during the actual probe lowering, so very short. So I figured that this might be an EMI problem. In a last resort I updated to Marlin 1.1.9 which has the functionality to shut the power off for heater and bed and everything now seems to work perfectly. I now see that the de-powering is a bit longer and in fact the extruder temperature drops slightly, this has never been seen with the previous firmware version.

\n
\n

Update:
\nNot long after the "update"/"fix" the problem manifested itself again... Further troubleshooting revealed that it was a bad connection of the connectors of the sensor that got temporarily "fixed" as the cables had been fiddled with during the firmware update.

\n

I have to note that the quality of these non-genuine sensors is not close the the original BLTouch. The BLTouch has never failed me to date, while several 3DTouch sensors did...

\n" }, { "Id": "6964", "CreationDate": "2018-09-21T04:23:35.593", "Body": "

So I've got a couple problems, to begin I'm using the RAMPs 1.4, generic Arduino mega with Marlin, generic stepper motor drivers, a generic power supply. Nothing is special or even out of the ordinary. But for some reason when I plug the RAMPs in, with everything connected or everything unplugged my Arduino heats up like hell. I clocked it at around 60\u00b0c after a couple seconds of being powered on around where the main chip is. The voltage regulator seems fine because it doesn't heat up much, I've checked it around 40\u00b0c.

\n\n

Second problem is the stepper motor drivers keep blowing up for some reason. With everything plugged in I power on the board for the first time and hear a small pop and then a small amount of smoke comes from the extruder driver. I replaced it thinking it was just a bad chip and the same thing happens. All other chips are fine it's just the extruder that fries.

\n\n

When the board is plugged into usb it doesn't give me any readings what so ever on what the printer is doing but the Arduino works fine with anything else I throw at it.

\n\n

Things I've tried: \n- Lower the voltage on the power supply from 12.00v to 11.26 with no changes\n- try running it with everything plugged in and everything unplugged\n- tried buying a new Arduino but the exact same problems occur

\n\n

I've gotten kinda tired of thinking of ways to trouble shoot so I'll open it up to the community.\n Picture of a friend stepper driver

\n", "Title": "Reprap RAMPs over heating issues", "Tags": "|ramps-1.4|stepper-driver|heat-management|arduino|power-supply|", "Answer": "

This really sounds like there is a short somewhere on the RAMPS add-on board. It is advised to not use the RAMPS add-on shield to prevent damage to the Arduino or the shield.

\n\n

Personally, I would ditch this one and buy a new one. By the looks of the picture you are using a clone RAMPS, these are mass produced and the quality is not always the best (many components are skew). If you are very into electronics you could try to find the short, but it can well be that the traces on the board are somehow connected. Alternatively, you could buy a new board where RAMPS and Arduino are already combined, e.g. an MKS board.

\n" }, { "Id": "6965", "CreationDate": "2018-09-21T06:13:23.900", "Body": "

I recently became curious about the Line Width setting in Cura and why one might change it if they aren't using different size nozzle.

\n\n

Since I've gotten my Ender 3, I've always kept the line width equal to my nozzle size (0.4\u00a0mm). I've looked around a bit, and it seems like most people actually set their line widths to be higher, depending upon who you ask anywhere from 120 - 150\u00a0% nozzle diameter.

\n\n

Why is this? They mention that it helps with print adhesion, but why? Shouldn't a 0.4\u00a0mm nozzle create a line of plastic 0.4\u00a0mm wide, necessitating a line spacing of 0.4\u00a0mm?

\n", "Title": "Why is it conventional to set line width > nozzle diameter?", "Tags": "|ultimaker-cura|slicing|nozzle|knowledgebase|", "Answer": "

There are several yet unmentioned considerations:

\n\n

Given the same movement speed thicker lines fill up a layer quicker, because more volume is extruded per second. In some systems the extrusion flow is the limiting factor for speed, but around corners the print head needs to slow down. Thicker lines = less lines = less corners = less slow down = higher print speed.

\n\n

Thicker lines have less detail, though. A line of 0.6 mm cannot represent details smaller than that, so smaller line widths capture the input geometry better. Also corners will get rounded by the same distance, so thicker lines = rounder corners.

\n\n

Thicker lines create worse overhang. Thicker lines require more pressure from the nozzle and if the layer below is (partly) missing the back pressure from the previous layer is less, which results in overextension, which will then also more likely go downward instead of to the sides.

\n\n

The higher pressure can force lines into small crevices of the layer below, though. This was highlighted by Trish already.

\n\n

The model Cura uses for a single line is rectangular, while in actuality the printed lines are rounded on the sides. This makes the width of the full extend from side to side larger than computed, at the cost of the corners of the rectangular model. This means that the line width setting should be set slightly smaller than what you would want the lines to end up like.

\n" }, { "Id": "6968", "CreationDate": "2018-09-21T11:28:37.490", "Body": "

My question would like to address what would be better for adhesion and general quality and possibly strength: either increase the slicer line width option or increase the slicer extrusion multiplier for layer to layer adhesion?

\n\n
\n\n

This question has a relation to \"Why is it conventional to set line width > nozzle diameter?\". This answer on that question addresses a phenomenon called die swell to explain that the extrusion width is generally larger than the nozzle diameter even if the line width in the slicer is set to the nozzle diameter. Basically, increasing the line width increases the extrusion multiplier, so why not set that directly?

\n\n

\"Die

\n", "Title": "Slicer line width vs. extrusion multiplier for layer adhesion?", "Tags": "|slicing|extrusion|nozzle|knowledgebase|", "Answer": "

Generally speaking, both settings result in the same1: The feed rate of the filament gets adjusted. Either you set a general multiplier, or you demand a wider line which does make it set a higher multiplier hidden in the software.

\n

1 - if you don't look at any other factor that is!

\n

But...

\n

There is always a but, and this one is big:

\n

While we can work with the extrusion multiplier to get wider lines, wider lines do change another setting: it changes the distance between the lines too. Let me show you:

\n\n

2 - ignoring die swell effects

\n

The result is pretty clear: the theoretical lines do overlap, which will lead to the polymer goop getting squished and pressed around to bulge up. Luckily, that stuff usually ends in the infill on the lower layers, but it can result in very messy sides. You pretty much deliberately chose to massively overextrude!

\n

To say it blatantly: line width changes the overall geometry of the slicing to account for the extra material getting extruded, extrusion multiplier does not.

\n

What is the extrusion multiplier for THEN?

\n

Well, using the extrusion multiplier can be done for many reasons, but mainly 2 are interesting:

\n\n

It might not look too obvious at first, but I once had printed a model from one roll of white PLA that came from China. Then I got my fingers on some other PLA that was transparent from the same warehouse. I tried to print the exact same .gcode with it again and got a hyper fragile object that just could get squished, every time I used it with the same settings as the white PLA. I had to increase both the extrusion multiplier and change the temperature to get the same strength as with the other PLA.

\n" }, { "Id": "6977", "CreationDate": "2018-09-23T09:50:33.600", "Body": "

I'm looking into upgrading my heated bed and replacing it with a silicone bed. I know nearly nothing about electronics, so I would like someone more knowledgeable to confirm if what I have theorized is correct or I'm going to burn my house down trying it.

\n\n

Basically, I'm considering setting up a 200W@12V heater, and I am using this premium ramps, which reportedly has Toshiba TK72E12N1, S1X MOSFETs.

\n\n

I did some basic math following Tom's guide, if that is correct, I would be pulling 16 A current. Assuming I plug the heatbed directly to the ramps, that would mean the MOSFETs are losing approximately (considering 20 % more resistance than the datasheet says as per Tom's advice) about 1.2 W, which means the MOSFET would heat up about 100 \u00b0C above room temperature, so the total temp would be around 125 \u00b0C, which is below the graded 150 \u00b0C in the sheet.

\n\n

According to this math, I'm thinking I can just plug that heater to my RAMPS board without any issues. Should this cause any issues? Will it set a fire or melt down something? Anything else I should consider?

\n", "Title": "Upgrading to silicone heat bed, will it burn my house down?", "Tags": "|heated-bed|ramps-1.4|safety|cooling|", "Answer": "

The heated bed port on your board has a 11A fuse. It will not work for a heated bed requiring 16A of current, no matter how good the MOSFETs might be. Note that the terminal block might also not be rated for that much current. You'd have to check, because often it is not the MOSFET itself that catches fire but the wiring or terminal blocks.

\n\n

Also, keep in mind RAMPS only drives the gate with 5V. The value specified in the datasheet (for 10V) plus 20% is probably a bit optimistic. The datasheet doesn't specify the resistance at 5V, so you're guessing at what the resistance might be. Since you're already running the part quite close to its limits, and considering ambient temperature might rise above 25C, I'd be cautious. The designers of that board only intended it for use at 11A, probably with good reason...

\n" }, { "Id": "6986", "CreationDate": "2018-09-25T19:54:31.450", "Body": "

I've long since been aware that some people swear by Kapton tape as a bed adhesive, with MatterHackers claiming that Kapton tape is the recommended bed adhesive for ABS.

\n\n

What makes Kapton tape so useful for printing with ABS? Is it also useful for printing with other materials?

\n", "Title": "When is kapton tape useful as a bed adhesive?", "Tags": "|abs|adhesion|", "Answer": "

I tested Kapton tape for PLA, ABS, PETG and XT. I know it works for other materials as well. What you need to keep in mind is that many materials only stick to Kapton tape well if you use a heated bed.

\n\n

The advantage of Kapton tape in comparison to other materials is its heat endurance and mechanical stability. It protects the glass underneath, while you remove the print from the surface. Some filaments stick so well to glass that you may break chunks out of it.

\n\n

The major disadvantage of Kapton tape is the time to apply the tape to the print surface and its very smooth surface pattern which seems odd in comparison to the rest of the print.

\n" }, { "Id": "6990", "CreationDate": "2018-09-25T23:49:57.660", "Body": "

I'm a 2-week newbie at 3D printing, working on a new Qidi Xpro machine (that is solid and one that I like). So, I do not want to believe that this issue is caused by my printer itself. I'm hoping that my settings have something to do with it.

\n\n

The problem is all the filament lines (travel lines, I think) that start at a sharp corner and go somewhere else. Most often, the lines go to another sharp corner but sometimes can intersect the middle of a side. See the image below for many examples. I have drawn white lines parallel to the unwanted filament lines in case they are hard to spot.

\n\n

The unwanted lines also appear in the infill underneath the surface lines. They look like porous infill grid lines since they should probably not be there, and they do not get a full load of filament extrusion. You could easily say \"Oh, they are the result of a \"leaky\" nozzle with poor retraction, but I think it's more complicated than that. I have done a full load of retraction tests and calibrations to optimize retraction lengths and speeds to minimize hairs.

\n\n

Here is an image showing the problem:

\n\n

\"Unwanted

\n\n

\"An

\n\n

My Cura settings seem reasonable for PLA: bed 50C, nozzle 200C, print speed 40mm, travel speed 90mm (100-110 makes no difference), retraction length 8.5-9.5mm (makes no big difference), retraction speed 35mm, infill: density 20% (line, grid, makes no difference), z-seam set to random, retract at new layer = enabled.

\n\n

I want to believe that something in my settings is telling the machine to extrude a 1/2 or 1/3 amount of filament when it starts those travels from sharp corners to somewhere else. But, I have not been able to solve the problem.

\n\n

Everything else works fine in a print (IMHO) except for another problem that I described here. I thought I fixed that one, but I saw it occur on the image shown above (inside the infill, under the surface that is shown above).

\n\n

Does anyone have any ideas that I might try to solve the problem? Thank you.

\n\n

UPDATE: Here's an image with Zhop when retracted enabled at zhop height 1mm. This is an infill picture, so it cannot be directly compared to the surfaced original. But, the unwanted trails are all still there (although very thin). Maybe 2mm hop height will do it. I will run another test.

\n\n

\"With

\n\n

UPDATE:

\n\n

I ran a second test with zhop 2.0mm, but without success. The problem was still there. (Retraction on, 8.5mm, min distance 0.8mm). Here is a comparison picture. I think the gcode is definitely telling my printer to do what it does, because the problem is not just a random leak. I think I'll try a different slicer in hopes that it generates different gcode.

\n\n

\"Zhop

\n\n

UPDATE ON COMBING

\n\n

Oscar pointed me to the Cura \"combing\" setting in his comment below. He nailed teh problem perfectly. Combing means \"don't retract, and ooze as you please while you move in a straight line to the destination.\" That's exactly what I've been showing in my images.

\n\n

The default for Cura combing is enabled. As a newbie, I didn't know enough to turn it off. Worse yet, if combing is enabled (\"don't retract\"), it prevents \"Zhop When Retracted\" from zhopping. So, all my zhop experiments did exactly nothing and had zero effect because no zhopping was occurring. After I disabled combing, then for the first time in my life I actually saw what a zhop looked like. (And thus I can assert that zhopping was not happening with combining=enabled.)

\n\n

Here is an image of my new \"perfect\" prints thanks to Oscar's pointer on combing. The left two prints have combing off, zhop off. They are almost perfect, inside and out. The infill walls are generally solid and smooth (20% infill) and without the globs and gaps in the image on the right (with combing on).

\n\n

\"enter

\n\n

As a closing note, I think the combing setting was probably also responsible for the problem of missing infill grids in the corner, as described in this question. The problem doesn't happen with combing off. My theory is that with combing on (as it was for that question), the oozing pulls filament out of the nozzle during the travel, so there's not enough filament at the destination to bite and bond to the corner. So, turning combing off solved 3 problems for me: the awful surface lines, the unwanted trails in the infill, and the missing infills in the corner. It also greatly improved the quality of the infill walls, too.

\n", "Title": "How to remove unwanted filament trails from sharp corners", "Tags": "|pla|ultimaker-cura|retraction|", "Answer": "

You can Z-hop what you like, but if it is oozing it is oozing, you will always see the effects of that as it just drops down.

\n

Basically you have multiple issues, first the oozing, second the line markings on the top.

\n

First

\n

Oozing is fought by applying correct settings for e.g. print temperature, retraction, coasting, travel speed, as is explained in an answer on one of your own questions. Note that you have not explorer the coasting option as far as I understand from your question. Coasting means that you stop extruding filament prior to a move when the head is still printing. This is explained by the pressure build-up in the hotend; ideally you set the coasting length as such that all the material that is pressed out as a result of the pressure build-up is extruded just before the head moves/travels to another location. Specific calibration prints can be found to tune this for your printer.

\n

\"enter

\n

Second

\n

The markings on the top can be removed by setting the correct combing setting. Combing does not use retraction and uses straight moves, this saves a lot of time in printing, but it makes those ugly scars on bottom and top faces. You can set the option to not comb on those surfaces, and frankly, what happens on the inside stays in the inside, I would not worry about that. A good read with additional info on combing is this post.

\n" }, { "Id": "7002", "CreationDate": "2018-09-26T15:38:17.920", "Body": "

I have a Tronxy X5S and I am having issues calibrating my X and Y axis. When I print a 20 mm3 cube it comes out 19.9 mm x 20.4 mm x 20 mm. I have already made the belt tensions as even as I can get them but it did not change the calibration cube size.

\n\n

I have added this upgrade to my printer for the idlers and motor mounts:

\n\n

\"idlers

\n\n

I have also upgraded the hotend to a V6 clone, after this change is when I noticed the discrepancy in the calibration print.

\n\n

What else can cause the X and Y axis to be uneven in a CoreXY printer besides uneven tension in the belts?

\n\n

UPDATE:

\n\n

Its seems Oscar was correct in his assessment that my printer is not printing square. I printed this calibration square to measure the diagonals and I got: $ \\overline{AC} = 141.82 $, $ \\overline{BD} = 141.35 $. I believe the correct value for these diagonals should be $ \\sqrt{2} \\times 100 \\approx 141.42 $.

\n\n

\"calibration

\n\n

Oscar also mentioned that I can adjust my firmware to correct this but I would rather fix the problem than apply a band-aid. Does this indicate that I did not assemble the printer frame correctly?

\n", "Title": "CoreXY Calibration Issue", "Tags": "|calibration|corexy|", "Answer": "

Oscar's answer is the correct solution to this issue. My printer was printing skewed and there is a built in correction for this in Marlin. Unfortunately the X5S does not have a boot loader and I don't have an ISP programmer (yet), so I am unable to re-flash my board at the moment.

\n\n

In the mean time I created a software fix for this issue. I used a script to add a Affine Shear Transformation to the stl file. This adds a reverse skew to the model so the print comes out (closer to) square. Basically read all the points from the triangle mesh (STL file), apply XY shear transformation preserving Z, re-save the STL.

\n" }, { "Id": "7016", "CreationDate": "2018-09-27T23:54:55.047", "Body": "

I have a Monoprice Select Mini V2 and I\u2019ve recently been having issues with little strings that are thick and sturdy sticking off of models:

\n\n

\"Example

\n\n

I used Ultimaker Cura v3.4.1 to slice this model: Basic Retraction test using Hatchbox Black PLA filament

\n\n

And the following settings:

\n\n\n\n

I\u2019d appreciate any advice on how to fix this problem.

\n", "Title": "Monoprice Select Mini V2 retraction/stringing problem", "Tags": "|ultimaker-cura|retraction|monoprice-select-mini|", "Answer": "

What I ended up doing was using the settings from the cat.gcode that came with the printer. It has a retraction of 10 mm specified, which I thought was a bit much, but if it works, it works.

\n" }, { "Id": "7023", "CreationDate": "2018-09-29T16:11:30.780", "Body": "

I use blue 3M Painter's tape to help with bed adhesion on my heated bed. Are there any tell tale signs which would lead me to believe I need to change the tape out?

\n\n

For reference, I haven't been printing all that much, so the tape has been on there for some time. I took a look at it today and see there are some bubbles in the tape in the places where the last print I did was laid down. It appears the tape has lost some adhesion to the bed. I can run my fingers over it and it will re-stick to the bed, but is that good enough? (Obviously I'd re-clean the tape surface with isopropyl before I'd use it again, so not an issue there.)

\n\n

The last thing I want to do is waste time and materials printing when I can be more proactive, so the real question here is:

\n\n

What signs might the tape be giving me it's time to change it out?

\n", "Title": "Tell tale signs bed tape has given up the ghost", "Tags": "|adhesion|maintenance|", "Answer": "

Obviously, if the tape is torn or gouged in area that you intend to print on, you will need to replace it to have a good surface. If the damage is well clear of your first layer and will not risk snagging on printer mechanisms during movement, you may disregard it.

\n\n

I can't really tell you how much bubbling is acceptable. If your prints still come out in with an acceptable quality, then it's OK. If it causes defects that you don't accept, or failed prints due to a loss of adhesion, then it's not.

\n\n

When I started printing, I used painters tape, and would change it every few prints. To be honest, I was not very careful about protecting the tape when removing a print, and would often damage it. I also often found it easier to just peel the tape from the print bed as a way to remove a print. I don't even think I finished a single roll of tape before I eventually switched to glass/hairspray.

\n\n

I never used isopropyl or other things to clean it between prints. If you are concerned with cost and waste, I would think this would be even more than simply changing the tape frequently.

\n" }, { "Id": "7024", "CreationDate": "2018-09-29T20:44:29.700", "Body": "

I'm interested in 3D printing toy action figures, but what I don't quite know how the joints should be designed to allow flexibility.

\n\n

I could imagine maybe something like a screw and nut for the arms and legs that are separated with a hole in the middle that can join them, but I don't know how to get a wider range of motion on shoulders and hips.

\n", "Title": "What is the best method of printing joints for action figures?", "Tags": "|3d-design|mechanics|", "Answer": "

(NOTE: Converting comments to an answer.)

\n\n

An interference fit would probably work. There are a couple of styles I can think of which would do. Here are some ideas from thingiverse.com which may work for your needs.

\n\n

\"enter

\n\n

(image taken from this design by OgoSport)

\n\n

With this style of ball/socket, you'd have very good articulation. More than enough movement to allow for a posable action figure.

\n\n

Another idea from thingiverse.com is this:

\n\n

\"enter

\n\n

(image from this design by NEMO_from_saturn)

\n\n

The idea behind this style of joint is to insert it into each part (part fits over the end). This is premade and would be easy to utilize. The thing about this is, they are covered by the licensing involved with patterns found on thingiverse.

\n\n

As with anything printed or mechanical, it will wear over time. You won't ever get anything which doesn't wear. It's just a fact of how it works. Some printed filaments will have better wear properties than others, like ABS, NylonX (polyamide with chopped carbon fiber strands), nylon, PET (PETG/PETT), or polypropylene (this is not all inclusive). Each filament has its own printing properties and idiosyncrasies, so pay attention to what needs to be done to print with them. This doesn't mean all of the parts need to be printed out of this material, but rather if using a joint (like the second image), you could print it out of the better wear material and print the rest out of filament which is cheaper and is easier to work with.

\n" }, { "Id": "7025", "CreationDate": "2018-09-29T23:42:49.567", "Body": "

How can I vary the infill percentage for different layer heights of my model?

\n\n

Context: The bottom part of my model needs about 20% infill. The geometry of the top part of the model (mostly cones of various sizes) prints well with 0% infill and is of course a lot faster to print if I can specify this.

\n", "Title": "slic3r: Can I vary the infill percentage for different heights of my model?", "Tags": "|slicing|slic3r|infill|", "Answer": "

Let's assume the model is 75mm high; the bottom portion (which needs 20% infill) is 40mm, and the top portion (which needs 0% infill) is 35mm.

\n\n\n\n

As a result of this, the bottom part of your model (which is not enclosed in the slab) will print at the default fill setting of 20%, and the top part of your model (which is enclosed in the slab) will print with 0% fill.

\n\n

You can verify this by slicing and looking at the preview. You will see the base with your default infill, and the top with no infill. The slic3r docs mentions several other ways to customize parameters for your model.

\n" }, { "Id": "7027", "CreationDate": "2018-09-30T10:30:51.530", "Body": "

I bought my Anet A8 less than 2 weeks ago. The setup went fine. In the first 1 week, I printed a lot, and the quality was really good. Now, the quality of prints is really bad. The infill is terrible, like little individual sticks and there are strands of PLA attached to the object(see picture). My Cura settings are:

\n\n\n\n

Any help would greatly be appreciated.

\n\n

\"After

\n", "Title": "Anet A8 prints not good", "Tags": "|print-quality|diy-3d-printer|anet-a8|", "Answer": "

This effect you see is called stringing. Stringing can be reduced by changing any or a combination of the following slicer settings:

\n\n\n\n

In order to find the correct settings you will need to print so called calibration test prints. There are a lot to find on Thingiverse, just search for \"stringing\".

\n\n

Your print temperature may be a bit high, but some PLA filament brands require that, I don't know if that is the case with your filament, but you could start try lowering that by 5 °C a time.

\n" }, { "Id": "7037", "CreationDate": "2018-10-01T12:21:30.383", "Body": "

I've seen a few print time-lapse videos lately which use gyroid infill: wavy lines, which deform across layers so that the waves end up alternating between the two axes. Other than making the time-lapse videos look much cooler, what are the benefits of this infill style compared to the more common hatching or cross-hatching?

\n", "Title": "What are the advantages of gyroid infill?", "Tags": "|infill|", "Answer": "

About a decade ago, we looked at the 'Gyroid infill structure' (which we called sheet solid). We looked at it as a linear-elastic solid and as possible bone scaffold design:

\n\n

https://www.sciencedirect.com/science/article/pii/S0142961211006776

\n\n

What came out of that study that lots of the triply-periodic minimal surfaces (a broader class of structures that comprise the Gyroid) had interesting elastic moduli.

\n\n

One thing that's worth noting though is that the Gyroid is not isotropic. Rather, it has cubic symmetry, ie 3 rather than 2 linear elastic constants. But it comes fairly close to isotropic

\n" }, { "Id": "7038", "CreationDate": "2018-10-01T12:42:29.440", "Body": "

When printing several objects, I recently encountered a problem that arises when the structure in itself is relatively thin or the support towers have a small surface: the printhead would in motion tend to knock one or more over as it traveled or catch at them and create layer shift.

\n\n

How can I avoid collisions with the already printed parts of a layer?

\n", "Title": "How to avoid collisions with already printed objects?", "Tags": "|print-quality|slicing|", "Answer": "

For the Creality Ender 3, I had the same collision problem and after a while I found it was because of the X-axis not being level.

\n\n

I found right hand side was more than 3-4 mm below the left side when the Z-axis height exceeds around 8-10 cm. Below that, the two sides were even. I made some calibration by turning the eccentric nuts of the wheels and tighten them.

\n\n

If you use Creality Ender 3, that might be the problem. Take a ruler and check between base extrusions and X-axis left/right hand sides at different Z heights.

\n" }, { "Id": "7041", "CreationDate": "2018-10-01T17:30:20.100", "Body": "

I'm looking at scanning in a statue on my pond and making a print of it. I'm wondering if PETG will fade and how fast. I'm thinking of using white or maybe a bone color if possible.

\n\n

If PETG fades kinda fast, what material should I use? Keep in mind this will deal with all weather types, full sun, and it will have water coming out of it.

\n\n

I've been 3D printing for a bit, but I never made anything to stay outside for looks. I know PETG is pretty good at water, and I think it's stronger than ABS when it comes to weather. But I'm in NC, and I even lived in FL at a point. Shockingly, I found the sound in NC can bleach wood in a matter of days to weeks in the mid of summer where it might take all summer in FL. So I'm worried what I might make will get that yellow fade look that plastic does in the sun.

\n", "Title": "Does PETG fade in the sun?", "Tags": "|petg|", "Answer": "

tl dr: For the most part, yes it should be color fast in the sun. It should be good for outdoor use.

\n\n

This website claims the following:

\n\n
\n

Filament materials ... for outdoor use include ASA and PETG that are perfect for use in extreme conditions without changing shape or appearance. Printed pieces do not change colour either with the sun's UV rays and do not take on a yellowish appearance as does ABS, for example.

\n
\n\n

Please realize, this is partially an advertisement (and no, I have no affiliation). What it boils down to is, PETG and ASA are going to be better for outdoor use, but every brand of each is going to be a little different in how well it reacts to the elements. Without advertising on the site, you'd need to do a little investigating on your own, with the possibility of testing different brands to come up with what you believe is the best. Beyond that, it's just an opinion.

\n\n

I doubt any filament, no matter what it is made of, is going to be perfectly colorfast in the sun. The sun is a harsh beast. It will bleach a printed object over time. Some colors are going to be more susceptible than others, especially red. This is due to the color of the sun. Again, some filaments are going to be better than others, but I'd not expect any filament to be perfect.

\n" }, { "Id": "7070", "CreationDate": "2018-10-02T11:33:08.153", "Body": "

I just finished a CNC that uses Marlin Firmware. The dimensions of it are quite large (3200 x 2000 mm) and the table is not perfect. I'm thinking of adding a proximity sensor to store a mesh in the eeprom and thus compensate for any errors. I'm not sure how to do this. I would like to be able to execute an action from the display (full graphics) and have samples taken over the whole table, but I do not know how to tell the firmware where to take the samples, nor what the dimensions of it are.

\n", "Title": "How do I add auto bed levelling to my Marlin-based CNC?", "Tags": "|bed-leveling|cnc|", "Answer": "

Auto bed leveling requires some settings (constants) in the configuration of your Marlin firmware.

\n\n

It is recommended to read about the implementation of automatic bed leveling first. There are a few options to choose the kind of leveling, for 3D printers a commonly chosen option is AUTO_BED_LEVELING_BILINEAR which is the best option if you do not know if your bed is flat or not. If you are certain it is flat but tilted (e.g. when you have a milled bed or a glass plate in 3D printing) you could go for AUTO_BED_LEVELING_3POINT or AUTO_BED_LEVELING_LINEAR.

\n\n

What further is important is the sensor type you choose. Do you want a touch or a proximity sensor. The latter is your preference (as specified in your question), which is a little simpler as you do not need to configure for servos for deploying and stowing.

\n\n

You need to set:

\n\n
#define FIX_MOUNTED_PROBE\n
\n\n

for using a fixed proximity sensor.

\n\n

In the configuration file you also need to specify the position of the probe in relation to the nozzle (in your case tool center):

\n\n
#define X_PROBE_OFFSET_FROM_EXTRUDER 10  // X offset: -left  +right  [of the nozzle]\n#define Y_PROBE_OFFSET_FROM_EXTRUDER 10  // Y offset: -front +behind [the nozzle]\n
\n\n

and optionally:

\n\n
#define Z_PROBE_OFFSET_FROM_EXTRUDER 0   // Z offset: -below +above  [the nozzle]\n
\n\n

This latter is not necessary as you can always set the distance of the sensor trigger point to the nozzle/tool plane by G-code command M851 Z-x.xx

\n\n

You would also need to set the boundaries of the probe area to prevent the tool to hit unwanted positions (fill out values or leave the constants):

\n\n
// Set the boundaries for probing (where the probe can reach).\n#define LEFT_PROBE_BED_POSITION MIN_PROBE_EDGE\n#define RIGHT_PROBE_BED_POSITION (X_BED_SIZE - MIN_PROBE_EDGE)\n#define FRONT_PROBE_BED_POSITION MIN_PROBE_EDGE\n#define BACK_PROBE_BED_POSITION (Y_BED_SIZE - MIN_PROBE_EDGE)\n
\n\n

Once properly setup, command G29 runs the leveling of the bed, options are available for that command to store the bed mesh to EEPROM.

\n\n

Enabling LCD_BED_LEVELING constant in the configuration file will add a Bed Leveling sub-menu to the LCD. But you could also work with SD-card stored files that load these codes.

\n\n

This reference explains the automatic leveling in more detail, but there are many more detailed guides to follow from the internet. Be sure that you get a recent guide to that is easier to set in the latest firmware.

\n" }, { "Id": "7083", "CreationDate": "2018-10-03T11:48:53.890", "Body": "

I wanted to create a coin stacking device similar to this one:

\n\n

https://www.thingiverse.com/thing:499177

\n\n

But I wanted to only 3 coins, 5c, 2c, and 1c. Unfortunately the STL came in pretty messy after importing to Blender, which made it difficult to edit the model to have only 3 stacks. So I decided to create my own. I used a box and three cylinders with the diameter of the coins + 2mm so the coins can fall loose into the stack. The cylinders where used for boolean modify the box to create the stack gaps.

\n\n

\"enter

\n\n

Exporting the model as an obj and importing them to Cura showed it getting strange diagonal faces at the stack gaps and at the top.

\n\n

\"enter

\n\n

After slicing however the faces were gone and the test print came out pretty decent, even on low quality settings the measurement were quite exact.

\n\n

\"enter

\n\n

Still I would like to know why these additional faces appear and how to prevent that. I am not a blender expert, but I guess it has to do something with the topology/faces? I started creating additional edges in Blender for the isolated vertex created by the boolean modifier, but I don't know if that's actually necessary for a 3d printed model that's not going to be textured ect.

\n", "Title": "Strange walls/faces when importing self made model from Blender to Cura 3", "Tags": "|ultimaker-cura|3d-models|slicing|blender|", "Answer": "

Judging from your second screenshot, I'd say that Cura does not like n-gons very much.

\n\n

An n-gon is any polygon with more than 4 sides. Most software tries to convert these polygons into triangles, with solutions that may not be what you want. Obviously, if you triangulate your surfaces manually this problem becomes moot.

\n\n

Go to Modifiers, add the Triangulate modifier, click apply.

\n" }, { "Id": "7122", "CreationDate": "2018-10-06T05:27:46.417", "Body": "

I just got my first spool of PETG and tried to calibrate an Ultimaker Cura profile for it on my Ender 3 today with limited success.

\n\n

I've had severe issues with the filament not sticking to the build surface, instead balling up around the edges of the nozzle tip. (rather like this post) Sometimes it sticks alright, but that happens properly > 50 % of the time. I've never seen this behavior with my PLA materials.

\n\n

So far, I've only been trying to print this calibration part.

\n\n

Relevant Profile Params: (let me know if I need to post more, I think these are the relevant ones)

\n\n\n\n

I was initially going by advice from Thomas Sanlanderer's video on PETG, starting with 230 °C/70 °C, but when that really didn't work, I then tried the advice of a Reddit user (can't relocate thread) that said to try a lower bed temp. I then experimented with different bed temps in the 50's to mixed avail. Most other threads are talking about PETG + glass, which seems to work well with a PVA glue stick surface finish.

\n\n

I'm getting a glass build surface soon (as soon as GearBest can ship it from across the pond), and I hear that will help, but in the meantime, I want to find a way to make it work with the fake Buildtak. What can I do to try to make this work better? My thinking is that the issue is with the bed config (temp, surface, etc.) and not with the nozzle temp, but I could be wrong.

\n", "Title": "PETG filament doesn't stick reliably to fake Buildtak surface", "Tags": "|ultimaker-cura|extrusion|creality-ender-3|petg|", "Answer": "

I have printed kilometers of PETG and found the sweet-spot for my brand to be 240 °C for the hotend and 70 °C for the build plate (for my Ultimaker 3 that is, the extruder temp is 5 °C higher for my home build HyperCube Evolution). The reason for the 70 °C is that the glass temperature of PETG is around 70 °C. The PETG is flexible at that temperature such that there are no stresses because of shrinkage causing the PETG to keep attached to the heat bed surface (aluminium, glass, Buildtak, etc.). A little PVA based glue (stick) or spray (hair or specific print sprays) can even further improve the adhesion. A slow first layer also helps adhering better.

\n\n

Note that the hotend temperature should be calibrated to the speed you are printing. If you print faster, a higher hotend tempearture is required. To determine the sweet spot for your filament you can print typical calibration towers that can be found on e.g. Thingiverse. Note that you need to manually change the G-code file after slicing of the tower or use plugins of your slicer to change the temperature at a certain level.

\n\n

Furthermore, PETG does not like to be cooled by the print fan, so keep cooling fan rpm low to prevent layers not to bond (else you get a sort of string cheese print).

\n\n
\n\n

Edit:

\n\n

I use parametric stair case style calibration prints that include the slicer print settings that are to determine the best settings for temperature, print cooling, layer size and print speed.

\n\n

\"Heat\"Heat

\n" }, { "Id": "7127", "CreationDate": "2018-10-07T04:59:55.353", "Body": "

Other than the most obvious issue with the filament kinking in the tube, what other issues could arise when trying to print flexibles (i.e. TPU/TPE, Nylon, etc.) with a Bowden style extruder setup?

\n\n

Can the kinking issue be alleviated by a well-constrained filament path (proper ID) in a properly sized Bowden tube?

\n", "Title": "What are the limiting factors when trying to print flexible filaments with a Bowden extruder setup?", "Tags": "|bowden|nylon|flexible|tpu|tpe|", "Answer": "

To also validate the first answer, I have been printing with TPU95A for a number of years and it has been one of the most reliable materials to use. The strength of the product alongside the flex makes it a test and production based outcome with fewer issues. My printer for the most part of this experience was the Ultimaker 2+ that has a similar set up to the UM3 with a more traditional hot end.

\n\n

Cautions: Some older material profiles have the heatbed at 60c. I have always had better outcomes from room temp (no heat on the bed).

\n" }, { "Id": "7130", "CreationDate": "2018-10-07T08:08:17.160", "Body": "

I've been searching for a reassuring answer for a week now. What type of mini USB does the Creality Ender 3 need? I have studied about them and five pin seems to be the answer but I haven't found a reassuring answer.

\n\n

If anyone knows the answer would you please inform me?

\n", "Title": "Does the Ender 3 need an 8 pin mini USB or 5 pin, or no difference?", "Tags": "|creality-ender-3|", "Answer": "

The ender 3 follows the USB standard for a USB-2.0/2.0 revised 5 pin USB-Mini-B.

\n\n

To prove, a picture of it. Left, you can clearly count the 5 pins that conform to the standard.

\n\n

\"Ender-3

\n\n

as a side note: There is no 8 pin USB-Mini - Mini has the T-shape. In fact, there is not even a USB standard conform plug that has 8-pins! The standard allows only 4, 5, 9, 11 and 24 pins

\n" }, { "Id": "7131", "CreationDate": "2018-10-07T09:24:54.037", "Body": "

When exporting an STL from Fusion 360, one must select an STL refinement level to use for calculating the maximum triangle count.

\n\n

For FDM printing (0.05mm and above layer heights), where is the point of diminishing returns on STL refinement level when printing PLA and PETG on an Ender 3 with a 0.4mm nozzle? All mechanical components on the printer are stock.

\n\n

\"Refinement

\n", "Title": "What is the point of diminishing returns on the STL refinement level in Fusion 360 on an Ender 3?", "Tags": "|cad|stl|creality-ender-3|fusion360|", "Answer": "

If you print for example a thin wall cylinder vertically you cannot turn the refinement high enough to not see the flats on the surface. My triangle counts are 524 for Low, 828 for Medium, 2206 for High, and if you select custom and pull the surface deviation slider all the way to the left you get 6572 triangles. The Surface Deviation shows 0.004406 mm which you would think would be fine enough. (This is for a 12 inch diameter cylinder with a 4 inch height and a wall thickness of 2.4 mm. The numbers vary depending on dimensions.) However you can see this level of deviation in the surface of common FDM type 3D prints. For those who are Metric impaired this is a deviation of 0.00017 inches or about 2 ten thousandths. The situation I present is the best case for an FDM type 3D printer and for just about anything else this level of refinement would be overkill. This is partially some arbitrary limitation in Fusion 360 but it is more a characteristic of using STL files. For my purposes being able to specify a surface deviation two or three times more precise would probably eliminate the visible artifacts. Increasing the refinement to the maximum possible will slow Fusion 360 when it computes the STL file and it will slow the slicer down as well but other than this there is no downside to using maximum refinement.

\n" }, { "Id": "7140", "CreationDate": "2018-10-08T19:40:38.577", "Body": "

I am trying to create a mechanism with moving parts, and would like to see how it works (whether it even works) before printing it.

\n\n

For example, there's a servo with a bracket, and I would like to see how far can the bracket move before colliding with other objects.

\n\n

\"enter

\n\n

Unfortunately I cannot find any information on how to set pivot points and rotate objects around these points in FreeCAD. Is this even possible?

\n", "Title": "FreeCAD to design moving parts", "Tags": "|3d-design|mechanics|freecad|", "Answer": "

I would also like to take a look at the A2plus Workbench (Freecad Addon). There you can define constraints which can help with this problem. As far as I know, parts cannot be moved with the mouse pointer, but angle parameters can be entered.

\n\n

https://freecadweb.org/wiki/A2plus_Workbench

\n" }, { "Id": "7148", "CreationDate": "2018-10-09T14:48:35.313", "Body": "

I was recently told I should probably update my printer's firmware from the current version (ANET_A8_20160701V2.0) to Marlin (going to use v1.1.9). To do this, I dutifully downloaded both Pronterface and RepetierHost trying to connect with either one of them to my printer. I've checked the Device Mangler (Manager) and found the USB does connect and is recognized on COM5, yet neither software are connecting. The software sees COM5 as active as well, but I get back an error stating \"access is denied\" for some reason. What might I be doing wrong?

\n", "Title": "Issues with connecting Pronterface to Anet A8 printer", "Tags": "|anet-a8|pronterface|", "Answer": "

After some trial and error, I discovered you need to set the baud rate to 115200 in the printer settings. I went through and tried each of the baud settings available until it connected. Glad I started with Pronterface, as RepetierHost has a lot longer baud list :o) Setting the baud rate is not inuitive as to the issue, as getting back \"access denied\" to me implied something else was going on. After that, was able to run direct commands to the printer through the USB port without issues.

\n" }, { "Id": "7150", "CreationDate": "2018-10-09T16:50:52.423", "Body": "

Have a Monoprice maker select with a Micro Swiss hotend, and am using Cura IIIP.

\n\n

Edit: plus Janbex PLA 1.75mm filament.

\n\n

Trying to print a Pi camera mount, but am getting (I believe) under extrusion, as seen here.

\n\n

\"enter

\n\n

My Cura settings are these.

\n\n

\"enter

\n\n

Does anybody see any obvious problems? What should I try to change?

\n\n

Edit: have played around with different temperatures, and some other settings (eg the filament diameter), but lost track of the reasons. Was following various instructions on web pages.

\n", "Title": "Under extrusion, looking for specific information", "Tags": "|ultimaker-cura|extrusion|monoprice-maker-select|", "Answer": "

Your filament is 1.75 mm, but you specified 1.6 mm in the Filament option Diameter (mm).

\n\n

Furthermore, your layer thickness is very small (0.06 mm), why not try 0.20 mm first; a thicker layer will cause a higher extrusion flow. It could be that the print speed is too high for the low layer size.

\n\n

Your initial layer 0.3 mm is also high, the maximum for a 0.4 mm nozzle.

\n\n
\n\n

Why did you change so many parameters? The standard values work pretty well.

\n" }, { "Id": "7159", "CreationDate": "2018-10-10T16:09:05.737", "Body": "

Assume somebody has a Monoprice Maker Select and has changed from the original brass extruder gear to a D4 Plus one, which is slightly smaller (10.6 vs 10.9 mm outer diameter).

\n\n

This person might be wondering if he/she needs to compensate for this difference, and which setting that would be in Cura IIIP.

\n\n

(Have never calibrated the extruder and am still not sure it is absolutely necessary, since the difference in the gear size is only 3%.)

\n", "Title": "Compensating for smaller extruder gear", "Tags": "|ultimaker-cura|extruder|wanhao|monoprice-maker-select|", "Answer": "

This will affect your extrusion rate. The best place to change this would be in your E steps located in the firmware but you can also compensate for this by changing the flow percentage in Cura, a setting of 103% (10.9/10.6) should get you close.

\n" }, { "Id": "7186", "CreationDate": "2018-10-16T05:24:58.090", "Body": "

I was reading up about how to extrusion print an overhang greater than 45 degrees, when something weird stuck me.

\n\n

I see lots and lots of 3D extrusion printed bowls that have perfectly printed hemispherical surfaces. However, given that there is some deformation expected as part of the print process, how is it possible to get a perfect hemispherical bowl - where both inner and outer surfaces are perfectly hemispherical?

\n\n

Note: I am not talking about surface smoothness, but about the shape of the bowl itself. I understand that surface smoothing will make the bowl look great, but it will not fix a deformation in the shape of a print.

\n", "Title": "Hemispherical Bowl : how is it possible?", "Tags": "|print-quality|3d-design|slicing|extrusion|", "Answer": "

There are several factors playing together. For example orientation, printer & slicer settings and more.

\n

Reminder

\n

First of all, not all overhangs of greater than 45\u00b0 need support. Many printers manage up to 60\u00b0, even 70\u00b0 is not unheard of - with the right settings. Pretty much all printers manage tiny 90\u00b0 overhangs.

\n

U-Bowls (open side up)

\n

Let's look at bowls that shape like a U - the dome is at the bottom. With a flat contact surface only the next areas need support, and with PVA or other easily removable support structures on the rise, it is no problem to use support and leave no trace. With support, there is no sagging, so the bowl gets its "perfect" look even though there are overhangs and support used.

\n

n-bowls (open side down)

\n

The other way around - dome on top - is probably the more "smooth" one, using different tricks to get it set.

\n

The outer wall now sits on the infill like the inner we had before, perfect, but we used that for the inner wall before. But what about the inner one?! Well, we need no support for the sides till we reach, let's say 55\u00b0 angle. Can we go further without support? Yes... if we are tricksy!

\n\n
\n

1 What counts as narrow is printer and filament dependent. It is no problem to print small spherical hollows in a print if they are just small enough as the overhang then is just short enough so it can carry itself. At some point it gets too much though.

\n" }, { "Id": "7188", "CreationDate": "2018-10-16T06:26:43.323", "Body": "

I've been printing PLA on a Pegasus 12\" for 2 years. Print quality has been great but in the last few months the quality of the prints has deteriorated. After looking into it, it seems when the head moves away from the spool and puts the filament in tension (and pulls on the spool) the extruder cog slips on the filament. \nThat results in traces where no plastic gets extruded the furthest away the head moves from the spool (the spool is on the left in the picture below):

\n\n

\"See

\n\n

The cog slipping can be seen between 0:22 and 0:27 on the video below: the cog on the right spins continuously but the ball bearing on the left stops spinning for 5 seconds.\nhttps://photos.app.goo.gl/cTfySUgXDy1XKXGv8

\n\n

The end result is a part with multiple gaps, especially on the side farthest away from the spool.

\n\n

\"enter

\n\n

I've cleaned the teeth of the cog (removed plastic dust) to improve traction but that didn't solve the problem. I've also tried to clean up the extruder by inserting a wire from the hot end side, no improvement in print quality.

\n\n

Any ideas on how to fix that slipping problem?

\n\n

Edit with solution:

\n\n

To complement Oscar's answer below, here is what I've done that fixed the problem: it turns out the issue was coming from a lack of friction on the filament. To increase the friction, I've slipped a pair of 0.25mm thick pieces of plastic sheet between the bearing axis and the mount. See pic below.

\n\n

\"enter

\n\n

That increased the force on the filament enough to fully solve the slippage problem. The print quality is perfect now (see pic below).

\n\n

\"enter

\n\n

The plastic sheet trick is just a short term fix. I'm going to try swapping the drive gear for a slightly thicker one and contact MakerFarm to see if there is any long term modification they recommend.

\n", "Title": "Extruder drive cog slipping on filament", "Tags": "|print-quality|extruder|reprap|", "Answer": "

In addition to the answers from 0scar, you might check your hot end temp. I was having a similar problem with slippage and prints that would literally fall apart in my hand even if they looked ok.

\n\n

It turned out that the filament I was using which claimed to be PLA actually wanted a much hotter temperature than most PLA. Once I bumped it up to around 216-218 \u00b0C the slippage stopped and the prints came out beautiful. Might be a possibility if this is a new filament for you.

\n" }, { "Id": "7191", "CreationDate": "2018-10-16T14:07:10.670", "Body": "

I am interested in 3d printing from aluminum valve cover for car SAAB valve cover

\n\n

I am thinking of creating 3d scan and the placing order with i.materialize or similar online services.

\n\n

I am curios if this is feasible at all, since I am interested in 3d printing functional part not just a souvenir. Also since 3d part does not have smooth surface, will sand blasting help without affecting dimensions?

\n\n

Also what would be an approximate cost of 3d scanning and printing?

\n\n

I really do not know where to start and would appreciate any directions.

\n", "Title": "3d print aluminum valve cover for car", "Tags": "|3d-design|", "Answer": "

This is a very large part. My estimate for the cost would be a few thousand to get this printed in metal, if not tens of thousands (assuming it even fits the maximum build volume of the printer, which is only 440mm on its longest axis).

\n\n

3D scanning also isn't a very reliable way to reproduce parts, especially for something mechanical that needs to be precise. You can not just scan something and then print it like making a photocopy. A lot of manual design and reverse engineering work would be involved.

\n" }, { "Id": "7194", "CreationDate": "2018-10-17T14:05:17.083", "Body": "

I have an SLA printed part I want to put a brass threaded insert in using a soldering iron. The insert is slightly larger than the hole so I would assume the edge will melt and re-freeze around the teeth. Are there any issues with melting SLA or this idea in general?

\n", "Title": "Using heat-set inserts with SLA printed part", "Tags": "|sla|", "Answer": "

Is it possible to make the hole larger, put the brass insert with a pre-installed screw inside the hole, fill the area with resin and cure with UV lamp?

\n" }, { "Id": "7207", "CreationDate": "2018-10-18T20:50:47.107", "Body": "

How can I achieve keeping the motors active during pause to avoid moving their position during filament changes? I have changed the filament during some prints to change the color or to change the a newer spool, but sometimes the X axis is moved during the change. I'm now using some cloth clips to prevent moving during this change.

\n\n

To pause the 3D printer I'm using the LCD menu ->pause, then I go to Move axis X, then I move close to 0. This change is manually and random since I don't know when the old filament reel is going to finish. The printer use Marlin as firmware with Ramps 1.4

\n", "Title": "Hold torque during pause to change filament", "Tags": "|change-filament|", "Answer": "

Is ADVANCED_PAUSE_FEATURE enabled in your printers configuration_adv.h file?\nThere is a PAUSE_PARK_NO_STEPPER_TIMEOUT option included in there, which prevents the steppers from timing out during a pause, and may be more robust than a G-Code command if you plan to manually pause and resume the print instead of setting it up in the slicer.

\n\n

Alternatively, in the same file,

\n\n
#define DEFAULT_STEPPER_DEACTIVE_TIME 120\n#define DISABLE_INACTIVE_X true\n#define DISABLE_INACTIVE_Y true\n#define DISABLE_INACTIVE_Z true  // set to false if the nozzle will fall down on your printed part when print has finished.\n#define DISABLE_INACTIVE_E true\n
\n\n

can be found. You may want to increase the DEFAULT_STEPPER_DEACTIVE_TIME, or set

\n\n
#define DISABLE_INACTIVE_X true\n#define DISABLE_INACTIVE_Y true\n#define DISABLE_INACTIVE_Z true  // set to false if the nozzle will fall down on your printed part when print has finished.\n
\n\n

to false to keep X, Y and Z engaged while allowing movement of the extruder stepper only.

\n" }, { "Id": "7214", "CreationDate": "2018-10-19T22:46:51.807", "Body": "

I want some indication to all the workings of 3D-Printers and the basics of design.\nAll links regarding DIY 3D-printer are welcomed.

\n", "Title": "Is a 3-D printer made from 3 CD Drives practically somewhat decent, being DIY and less expensive?", "Tags": "|diy-3d-printer|e-waste|", "Answer": "

I would guess that a printer made in the following way would be both poor quality, and annoyingly frustrating, as well as requiring constant tinkering/re-adjustment. Also, if the chassis of the CD/DVD drive is incorporated into the design (as below) then the print volume is rather small, given the inherent limited movement of a CD/DVD mechanism.

\n\n

If the steppers are used without the CD/DVD chassis then it might be possible to increase the print volume but then you would need to spend extra on the rods and support structure for the X gantry, the Y-axis print bed, and the Z-axis movement. If you do that, then the steppers from the CD/DVD drive probably would not have sufficient torque to move the additional weight - by incorporating the CD/DVD chassis the inertia of the movement has been kept to an acceptable minimum.

\n\n

By using the CD/DVD drive's stepper motors you are only really saving the cost of buying three or four actual, more powerful and useful, stepper motors (as well as the cost of the additional rods and support).

\n\n

That said, it could be quite amusing to make, and show off, and also provide some satisfaction if you do manage to print a tiny frog, parts for another printer, or whatever, with it. However, I wouldn't expect it to print anything to any great accuracy/tolerance/precision/etc., but it would maybe give you something unusual to talk about at dinner parties.

\n\n
\n\n

An example

\n\n

There is this (IMHO unjustifiably1) popular (i.e. well linked-to) Instructables guide: EWaste 60$ 3DPrinter, which describes making a 3D printer from old desktop computer parts2.

\n\n
\n

$ 3DPrinter\">\"EWaste 3DPrinter\" title=\"EWaste 60$ 3DPrinter\">

\n
\n\n

According to E-waste printer looks nice, prints really, really small, it has an awesome print volume of 37 mm x 37 mm x 18 mm.

\n\n

Apparently, all you need is to salvage:

\n\n\n\n

Then purchase these standard 3D printing components:

\n\n\n\n

To make the frame, you need a 325 mm x 362 mm, 5 mm thick acrylic sheet, and use this template (missing files):

\n\n
\n

\"Laser

\n
\n\n

or this one (cnc-calisma-yalniz.dwg):

\n\n
\n

\"Laser

\n
\n\n

You will also need to 3D print these parts (the links to which have died):

\n\n
    \n
  1. Extruder idle
  2. \n
  3. Extruder body
  4. \n
  5. Hot end holder
  6. \n
\n\n

Once you've gathered all of the parts required, you can then try to work through the incomplete assembly steps of the Instructables guide.

\n\n
\n\n

Other links

\n\n\n\n
\n\n

Footnotes

\n\n

1 Most, if not all, of the links for the printer above are dead (this variant, mentioned above in Other links, is much more complete). However, with a bit of knowledge, and common sense, it should be possible to work through those omissions:

\n\n\n\n

2 Funnily enough, whilst searching around looking for information to fill in the gaps in the above answer, I found this question on 3DP.SE: Missing print steps in e-waste 3d printer

\n" }, { "Id": "7217", "CreationDate": "2018-10-21T14:21:50.823", "Body": "

I have a long print that keeps aborting. At some random point mid-print the printer says \"Click to resume...\". There is nothing in the G-code that asks for user confirmation. What could it be that triggers this? I noticed that sometimes (not every time) there is a blob of plastic in the way that should not be there.

\n\n

On one occasion, after the \"Click to resume...\", the LCD showed the message FY178.N16466 and again waited for a click.

\n\n

The printer is an Anet A8 with Marlin 1.1.9. Slicer is Cura. I am printing via USB from Cura directly.

\n\n

This is the error message:

\n\n

\""Click

\n", "Title": "What triggers Marlin's \"Click to resume...\"?", "Tags": "|firmware|troubleshooting|", "Answer": "

For your information, this problem has occurred for me with Cura 3.6.0 (yesterday and 2 days ago)

\n\n

I thought the problem was occurring with Marlin since 1.8.

\n\n

My Marlin version is 1.3 (... just discover that because of that issue!)

\n\n

As my printer works perfectly with 1.3, I'd prefer not to upgrade.

\n\n

I had the 'click to resume' problem twice.

\n\n

I've printed tons of ABS models without problem, two days ago, I've printed PLA models and had this issue. (70 \u00b0C 200 \u00b0C).

\n\n

For sure if the problem occurs again, I'll upgrade to Marlin 1.9 (or even 2.0 even if still in beta) because it seems that you have found the problem and already solved it; I was surprised to read that it was fixed in Cura 3.6 since I had the problem with that version.

\n\n

My printer is a Tevo Tarentula (modified, I've removed the pseudo bed leveling options because I prefer to level manually (no z move while printing)).

\n\n

One more information is that in parallel, I've decided to print the first layer at a very low speed (adhesion problems).

\n\n

I've changed from 30 mm/s (ABS with big adhesion problems) to 10 mm/s with PLA + Cura 3.6 and went into this bug (yes one could argue that I may print faster etc., but that's not the point here). Maybe I have the problem because I'm printing the 1st layer at a this slow speed (thus making the full buffer problem more critical).

\n\n

The bug does not occur each time I print, even when printing the same model with same parameters...

\n\n

@HuguesDug reported the same problem 14 days ago and\n@Leeb answered him that running with Marlin 1.9 solved the problem.

\n" }, { "Id": "7219", "CreationDate": "2018-10-21T18:39:42.187", "Body": "

I'm having a problem with Ultimaker Cura (v3.4.0) where it doesn't always tell the extruder to retract when performing a travel, which causes the hotend to leave a \"trail\" as it's moving and not start printing again right away when the travel is over.

\n\n

For example, the extruder does not retract when performing this travel (blue):

\n\n

\"enter

\n\n

And yes, I do have \"retract before outer wall\" enabled....

\n\n

\"enter\n\"enter

\n\n

Any ideas how to solve this?

\n", "Title": "Ultimaker Cura not always telling extruder to retract when traveling", "Tags": "|ultimaker-cura|slicing|retraction|", "Answer": "

You probably have the \"Combing Mode\" option under the \"Travel\" options enabled for each layer (All). Combing reduces print time by leaving out the retraction, but leads to ugly first and top layers by leaving \"scars\" on the surface. It can be disabled by excluding the bottom and top layer by changing the setting to Not in Skin or for all layers by choosing Off. The latter will increase printing time drastically!

\n\n
\n\n

Please note that as of version 3.5 of Ultimaker Cura the options of the combing setting have an additional setting. As of Ultimaker Cure 3.5.0 the options are named:

\n\n\n" }, { "Id": "7221", "CreationDate": "2018-10-22T00:34:40.537", "Body": "

I'm trying to print the following model in my Prusa printer using its Slic3r PE software.

\n\n

Ferment Air Lock

\n\n

\"Ferment

\n\n

In the comments of the model it says that the way to print is the with the filler opener downside to the printing board.

\n\n

This is how I have it in Slic3r:

\n\n

\"Ferment

\n\n

When I print it I get this horrible form and I have to stop in order to avoid waste of material.

\n\n

\"Bad

\n\n

When I look in the layers tab and then scroll up to see layer workflow I can see this:

\n\n

\"Layer

\n\n

So that means is trying to print like in the \"air\" something without any support.

\n\n

Any clue how to print this? I'm kind of new to 3d printing.

\n", "Title": "How to correctly print an object with supports", "Tags": "|prusa-i3|slic3r|support-structures|", "Answer": "

You need to put a check mark to enable supports in Slic3r as you cannot print in mid-air.

\n\n

This option is found under \"Print settings\" with header name \"Support settings\". Please look at the Slic3r manual for more options.

\n\n

\"Slic3r

\n" }, { "Id": "7228", "CreationDate": "2018-10-22T13:28:25.657", "Body": "

With respect to Himanshu's comment about applying a lubricant to PLA filament, and then having read the Reddit thread, Seasoning all-metal hot ends with oil?, pointed to in 0scar's answer, I was wondering if any studies have been done on the topic?

\n\n

If so, what is the general consensus? From the Reddit thread it seems as if the better quality branded hotends, such as E3D do not require it, whereas the older, or less well finished clones, may well benefit from such treatment.

\n\n

In addition, which oil is preferable, animal, vegetable or mineral? Is vaseline a good idea?

\n\n

Also, would the advice differ if ABS filament is used, or would the same conditions apply?

\n", "Title": "Is lubricating filament a good idea?", "Tags": "|filament|", "Answer": "

W.R.T Nach0z's answer:

\n
\n

Third, the process of seasoning cast iron works because the surface of the cast iron should be smooth for minimal food sticking, and any kind of rusting causes pits and porosity in the metal.

\n
\n

You season a cast iron pan to create a carbon layer on top of the metal. Water should not be left standing in a cast iron skillet. For my 3D printers I replace the Bowden tube once a year. It cures many problems for me. I suspect that over time the filament cause abrasion in the tube that increases friction. I also do a teardown and cleaning of the extruder in the same interval.

\n" }, { "Id": "7229", "CreationDate": "2018-10-22T14:56:41.200", "Body": "

I'm having issues with ripples on the first layer of big flat prints. The initial corner of a big flat print is fine, but then ripples begin to form as shown in the screenshot.

\n\n

I'm just a newbie, so I was thinking they might have something to do with heat or contraction or something. Normally, I use the default and print with no turbofan on the first layer. When I tried adding fan 20% or 50%, nothing much changed (slight differences in the ripple pattern and area, but that pattern varies anyway).

\n\n

I also wonder if one strip gets bent, then maybe the rest just follow the bends. As far as I know, my heating plate is working fine, has no serious hot spots, and I'm using a high-quality PLA+ filament. I also tried adjusting the print temperature from 205-220 (the range on the box is 205-230). Nothing seemed to help. I am running a default first layer thickness of 0.3 mm because that is supposed to help adhesion (and adhesion is fine).

\n\n

The ripples look worse than they feel. They feel fairly flat, only slightly rippled, even though they look terrible! (And I don't know what that weird row with blobs is in the top left of the picture. That only happened once; almost like junk was in the nozzle or the feed gears slipped or something).

\n\n

I'm running a Qidi Xpro machine, Sunlu PLA+ (wonderful) filaments, bed 50 C, print temp 205-215, print speed 30-40 mm/s on the first layer, and first layer thickness 0.3 mm (normal layer thickness is 0.2 mm). This machine has a direct drive with gears immediately above the nozzle.

\n\n

Does anyone know why this rippling effect occurs, and what I might to do to correct it? Thanks

\n\n

UPDATE: I'm adding this info here to respond to several comments concerning bed leveling, etc. (Thank you to those who made comments!)

\n\n

1) I'm sure that the bed is as level as I can make it because I always go through the cycle twice).

\n\n

2) Regarding clearance, if anything I worry that my clearance is too small since there is a fair amount of drag on my leveling card under the nozzle. So, there is definitely drag on all three level points, about midrange between the lightest drag and the heaviest drag that makes me think I'm filing off part of the nozzle.

\n\n

3) I do have two nozzles, so I suppose the problem could show up on one but not the other if the nozzles were screwed into the block to give different heights. But the ripple shows up on both nozzles, always in the middle of the build plate, always in the middle of a big flat print. Corners don't usually show ripple effects. I don't want to believe that my build plate dips in the middle on my new machine, either ... :-) Adhesion is fine on small prints in the middle of the plate.

\n\n

Here is a picture of the bottom of the piece. A careful examination shows an oscillation in the squished filament segments on a filament thread. Almost like the extruder was oscillating vertically in the z-axis at that frequency, or perhaps the filament squishyness was oscillating at that frequency. Looks almost like a weave pattern, since the squished parts alternate position on alternating lines.

\n\n

It's worth saying again that the piece feels pretty smooth on both the top and bottom sides, even though it looks awful. I don't know what to make of that.

\n\n

\"Ripples

\n", "Title": "What causes ripples on part of first layer?", "Tags": "|print-quality|pla|smoothing|", "Answer": "

The main problem is solved (first layer thickness vs leveled nozzle height).

\n

The following image shows the problem. I was running with a default 0.3 mm first layer (the tooltip setting says a slightly thicker layer helps with adhesion). The build plate was correctly leveled with "midrange" friction on the leveling card at the leveling points.

\n

Problem cause: The midrange leveling height put the nozzle too close to the plate and caused rippling. The first layer thickness was set to 0.3 mm and the thickness of the leveling card was 0.25 mm.

\n

The following image illustrates the problem (and one of the solutions). The bottom right of the image shows rippling. Not knowing what else to do with the "too close" or "too far" or "unlevel" tips in the comments, I just manually lowered the build plate knobs 1/4 turn while the print was in progress. The print began in the lower right. You can see the smooth area where I manually lowered the build plate. Then, to be sure, I raised the plate by restoring the 1/4 turn on the knobs. The rippling returned.

\n

\"Ripples

\n

To further explore the 0-90 degree suggestion provided by profesor79, I changed the slicer degree settings to 0-90 degrees and set the first layer thickness to 0.2 mm, which was equivalent to lowering the build plate knobs by 1/4 turn. I kept the same "midrange" friction settings when leveling. The result was a first-layer print with no rippling.

\n

\"No

\n

Closing Thoughts

\n

From this experience, I think:

\n
    \n
  1. 0.05 mm difference between a thickness of 0.3 mm on the first layer and a leveling-card nozzle height of 0.25 mm makes a rippling difference.

    \n
  2. \n
  3. Using mid-range friction vs light friction on the leveling card also makes a difference. You don't need much of a height difference to reach 0.05 mm. Maybe even less is required to cause a ripple.

    \n
  4. \n
  5. When printing with a first layer thickness of 0.2 mm, tolerances were tight and I discovered a spot on my build plate that had no adhesion because of a buildup of old adhesive. It left a 1/2-inch hole in the 0.2 mm-thick first layer. I also noticed just a hint of ripple in another place on the build plate, which (I think) indicates a tiny magnetic build plate thickness or warp issue of some kind. Hardly noticeable.

    \n
  6. \n
  7. I think I will go forward with a 0.3 mm layer thickness to "absorb" minor flatness inconsistencies in my plate. (I have a glass plate but I have never used it because the magnetic plate is vastly more convenient.) But, to compensate for rippling effects, I will also use a "very light" friction amount when leveling the plate to ensure that the nozzle doesn't get too close to the plate on the first layer.

    \n
  8. \n
  9. I found that manually adjusting the build plate height during a solid first layer print was a wonderful way to detect, see, and explore all the relationships between plate leveling, plate flatness, first layer thickness, and friction adjustments on the nozzle. It's very easy to immediately see, understand, and adjust all the related settings to get the best print possible from the machine.

    \n
  10. \n
\n
\n

Thank you again to everyone who contributed ideas to understanding the problem. It's hard to pick any particular answer because the solution involved multiple ideas, so I have added my own answer to share.

\n" }, { "Id": "7264", "CreationDate": "2018-10-23T19:53:08.303", "Body": "

I have what I thought would be a simple question.

\n\n

I don't have an auto leveling probe, I do my leveling manually with 4 screws and a piece of paper (I measured the thickness to 0.1 mm).

\n\n

For the longest time I would have trouble with the first layer, sometimes having to give the bed screws a quarter turn to bring the bed up a bit. I would see that the nozzle seemed quite far away from the bed. This went on for the longest time and I just chalked it up to the quality of my printer.

\n\n

I realized recently that when I level the bed, I am inserting a piece of paper in between the nozzle and the bed. Obviously, I should be taking the thickness into account as a 0.1mm thick piece of paper accounts for 50 % higher than the nozzle should be for a 0.2 mm first layer height.

\n\n

My question is, how do I set (either in Cura or directly in Marlin config) the z home offset to account for the 0.1 mm thickness of my calibration paper?

\n", "Title": "How to set the Z home offset", "Tags": "|marlin|ultimaker-cura|bed-leveling|", "Answer": "

Assuming you already know your Z-offset value, there are several ways to set Z-offset for your printer:

\n\n
   M851 Z-0.69;\n   M500;\n
\n\n
    #define NOZZLE_TO_PROBE_OFFSET {\n       -53.5,   // X-offset\n       8,       // Y-offset\n       -0.69 }  // Z-offset <-- set it here\n
\n
\n

I usually set it via printer's display, since it seems like the fastest and most practical way.

\n" }, { "Id": "7281", "CreationDate": "2018-10-24T08:07:14.670", "Body": "

I have a Tevo Tarantula, the problem is that PTFE is getting pulled back to the extruder gears and stopping filament extrusion, do you know how to solve this?

\n\n

Image:

\n\n

\"Extruder

\n\n

One hypothesis I have is that there is a cooling problem in the hot end while a retraction is made, causing the extruder motor to pull the PTFE in it.

\n\n

Suggestions appreciated!

\n", "Title": "PTFE pulled into extruder gears", "Tags": "|extruder|tevo-tarantula|bowden|", "Answer": "

You are probably right, I have a Tarantula as well, and this happened many times to me. The reason is mostly because the hotend fan gets too hot, stops working, then, the filament in the aluminium heat sink melts and sticks the filament inside the PTFE tube. Then, on the retraction, the PTFE is pulled into the gears just like on your picture. Also, the filament stops getting extruded a few moments later.

\n\n

One solution for that was to buy a new PTFE tube with a pneumatic connector that doesn't allow it to slide into the gears. However, the diameter of the screw thread (of the one that I bought) was too large, so I had to design a new static block for the extruder to fit it.

\n\n

I chose this type of 1mm PTFE teflon tubing from aliexpress for my replacement.

\n\n

Maybe you can find a pneumatic connector with the proper diameter.

\n\n

However the real solution is to check why the heat sink is getting too hot. I bought a few other fans and printed an additional support for them on the hotend and I am making sure the fan stays on all the time.

\n\n

I hope it helps!

\n" }, { "Id": "7288", "CreationDate": "2018-10-24T17:03:39.637", "Body": "

I have a part with hair strands, such as the Hairy Lion on Thingiverse, and I want to print it on a Stratasys F370. The thin strands are intended to print without support, bridging to the exterior wall. Although the bridges are quite long, it doesn't matter if they sag, because they will be heat-treated afterwards.

\n\n

\"Hairy

\n\n

But when I open the STL file in GrabCAD Print, the slicer gets over-protective of those long bridges, and inserts soluble support for every one. As well as being a huge waste of support material, it would take ages to dissolve out all of that support, and it's completely unnecessary. However, there are other parts of the model that overhang and do need support, so I can't just disable support completely.

\n\n

How can I avoid supporting these bridges?

\n", "Title": "Print \"hairs\" without support in GrabCAD Print", "Tags": "|support-structures|grabcad-print|", "Answer": "

There's no way to do this in GrabCAD Print alone, but you can achieve it using Insight, which you can reach from the Apps menu in Print. The steps are as follows:

\n\n
    \n
  1. Open your STL file in Insight.
  2. \n
  3. Go to Modelers \u2192 Setup\u2026 and set the printer, layer height, and material to the same as in Print. If you get these wrong, you won't find out until you get to the end and Print refuses to print the part.
  4. \n
  5. Use one of the red icons to slice your print, with or without the wall thickness filter.
  6. \n
  7. Use the red and green icon to generate support. You'll notice all of your hairs are now supported.
  8. \n
  9. Go to View \u2192 Display options\u2026. Set View only to Group curves, and then Group to view to All support. This filters out the (red) model curves, showing only the (green) support curves.
  10. \n
  11. Go to Edit \u2192 Delete\u2026. Then left-click and drag a box around the unnecessary supports. For my part, dragging the 3D view to be top-down made it easy to drag a box around just the hair supports while leaving the other supports untouched, but it'll depend on the shape of your part.
  12. \n
  13. In the right panel, make sure the delete mode is set to Curves, and click OK. Now all your supports should be gone. At this stage you can set the display options back how they were if you want to inspect your model further.
  14. \n
  15. Use the green circle icon to create toolpaths
  16. \n
  17. File \u2192 Save As\u2026 \u2192 Toolpath to save as a .cmb or .cmb.gz file.
  18. \n
  19. Go back to GrabCAD Print. (You can close Insight now if you want.)
  20. \n
  21. File \u2192 Import CMB and choose the file you just saved.
  22. \n
\n\n

CMB files show up very boxy on the tray view (and Print will display a warning about that), but if you go into slice preview mode, you should see it as it was in Insight, with just the right supports.

\n\n

There's one caveat with this method. The slicer generates the base as a single contour, so deleting the supports under the hairs doesn't remove that section of base, and AFAIK you can't make the slicer recompute just the base (so generating support again would put the original supports back). You could edit the base contour before step 8, or disable the base completely in the support options before step 4, but the amount of support material saved probably isn't worth the effort.

\n" }, { "Id": "7305", "CreationDate": "2018-10-26T03:17:41.230", "Body": "

I have some leftover resin (100 ml) that I`m not planing to use anymore. What is a safe way of disposing it that doesn't involve curing?

\n", "Title": "Safe way of disposing resin", "Tags": "|sla|resin|disposal|", "Answer": "

Resin is notoriously hard to handle, especially as exposure to air and light can and will cure it over time. The uncured resin is a hazardous material.

\n

Handling hazardous waste

\n

The rules for safe disposal can - generally speaking - be broken down to this:

\n\n

This means (for example for acids) that they are neutralized (to pH 7) and then handled as chemical waste.

\n

WHY?

\n

Why go all these lengths? Let me explain with an example from Germany: The city of Leverkusen is the main site of Bayer. They produce pharmaceutical and chemical products there, which includes a lot of chemical and hazardous waste. Bayer knows how to handle waste, they handled 541000 metric tons globally in 2015. Most of the following information comes from Germany, and is in German, but I do provide my sources.

\n

For the remains of pharmaceutical product production at Bayer in Leverkusen around 2007, the process, as I was told on a tour to the fabrication plant, was generally speaking this: The nasty stuff got neutralized and reacted in ways to make it inert, the resulting sludge got dried and incinerated to destroy most toxic compounds. The remains were bagged in thick plastic bags that were carefully stored with a catalogue of what was stored where in a dedicated chemical waste landfill with (iirc) 3 independent groundwater protection systems, covered with a thick plastic sheet, then with a several meters thick cover of carefully constructed layers of dirt, clay, cement, more plastic, gravel, and rock to protect the rhine and the ground-water. Today, this process is done by the Currenta in a similar way.

\n

In the past, the landfills were less secure: the old landfill that was started by Bayer in 1923 in Leverkusen. It was used by Bayer, the local population and (for some time) the IG Farben. It was finally closed in 1965, contains 65000000 tons of waste, of which approximately 15% is residues of chemical processing (~1 million tons). Nobody ever cataloged what had been stored where in these times, one just knew that it was filled south to north. It had to be pretty much re-engineered in the 1990s and brought it to match the (then) current landfill standard, for example with a 38m deep cement wall: Checks had found that nobody knew anymore where what materials were stored and that residues - among them possibly LOST - had found their way to the surface. These chemicals threatened to get into the Rhine and the water supply at some point. It also lead to a case of public domain and following demolition (for public health concerns) of a group of houses that had been erected on one part of the closed landfill.

\n

You see the length people go to keep you safe from chemical waste: LONG.

\n

Now, how do we fix the problem at hand both safely and effectively?

\n

Getting rid of small batches of resin

\n

The basic rules of handling hazardous waste (the two bulletin points) tell us we should take a two-step process:

\n
    \n
  1. find a temporary storage solution (e.g. storing in a safe container)
  2. \n
  3. decide on a process that can get rid of the stuff in a safe way.
  4. \n
\n

How could one fulfill the target number 2? I came up with three options:

\n\n

Your local waste disposal service might charge a fee for taking care of your resin, but your problems end when you hand it over with old paints or other chemical waste. Check out who provides these services and what laws, rules and regulations apply for small batches of resin and paints.

\n

A different kind of specialist that would take the resin could be a maker or artist that plans to use it in their own SLA/DLP printer.

\n

Making it inert could be done by curing with only sunlight, without the need of much special equipment or chemicals in a very well ventilated area and make sure nobody touches it: pour the resin into a (disposable) mold like a cardboard box or yogurt cup. If you can't have a well-ventilated place, storing the resin in sealed transparent plastic or glass container (plastic bottle or marmalade glasses) in sunlight can cure the resin very slowly over time. Note that it will take quite some extra time, as such containers do filter out some of the 400\u00a0nm light that commonly cures the resin.\nThe resulting resin chunk can be handled like any plastic block once thoroughly hardened through: Use it for other projects, as a paperweight or dispose of it through the normal waste.

\n

tl:dr;

\n\n" }, { "Id": "7312", "CreationDate": "2018-10-26T19:09:32.363", "Body": "

I'm 3D printing almost 2 years, and I expected to have better result after changing to complete smooth rods with new bearings.

\n\n

I have a problem with my Z axis giving me inconsistent prints; I already replaced the leadscrews, E3D clone with Bowden tube, I decided to replace all smooth rods on all axis and also the bearings. After I replaced all this, my prints are still bad, also I'm very dissapointed with the results after the replacements. I have checked all I could think of; PID tuning, belts, ....) but I'm running out of ideas.

\n\n

\"inconsistent

\n\n

Printing settings:

\n\n\n", "Title": "Z Axis inconsistency - Geeetech I3", "Tags": "|z-axis|geeetech|", "Answer": "

Yesterday I tried some improvements, first I replaced the Y belt pulley, and aligned the frame which I thought to be non-perpendicular and removed the spool holder (which was the culprit of my problem) Attached photo shows a print after these improvements. I have insulation on my hotend.

\n\n

I think this is the quality I could expect from this cheap printer. Maybe I could reprint X axis gantry, but I'm a little tired of doing something on my printer.

\n\n

\"Print

\n\n

Update on my prints

\n\n

\"enter

\n\n

\"enter

\n\n

\"enter

\n\n

\"enter

\n\n

I see some elephant foot, but i will fix this by lift nozzle a bit, but is there some more improvements (temp, retraction, z-hop, coasting, ....) ?

\n" }, { "Id": "7315", "CreationDate": "2018-10-27T11:01:33.287", "Body": "

I have been running my i3 MK3 for about 12 hours now and the motor on the extruder is fairly hot, not too hot to touch but I'd guess its about 60c on the outside. Is this within normal operating temperatures or should I let it cool down before starting more prints?

\n", "Title": "Is it ok for stepper motors to be hot?", "Tags": "|prusa-i3|motor|", "Answer": "

50 \u00b0C feels very warm, 60 \u00b0C feels hot, 80 \u00b0C is painful.

\n\n

I measured the temperature of the steppers on a Prusa mk3last week. They were all around 45..50 \u00b0C, except the extruder stepper which runs around 55..57 \u00b0C.

\n\n

The temperature is stable like that. Letting them cool down is useless, because they would quickly go back to these temperatures after starting the next print. Stepper motors tend to dissipate more power when they stopped or slow compared to when they run fast.

\n\n

I would not worry about it.

\n" }, { "Id": "7317", "CreationDate": "2018-10-27T16:37:48.750", "Body": "

I have had my Prusa i3 clone for about two years and it has always made a screeching sound when the z-axis is moving down. I have tried many solutions for this problem like greasing the z-rods, printing a new top part (the holder at the top z-rods end) with a bearing to prevent the threaded rod from swaying, and even hot glueing bearing to the stock top piece. However, none of this has worked. I am not sure if it is affecting the print quality but the sound is very annoying when it is doing a lot of hops and I would like to fix it. Here is a video (with sound) of the rod moving up and down.

\n\n

Video

\n", "Title": "Screeching Noise when moving the Z-Axis Down", "Tags": "|prusa-i3|troubleshooting|z-axis|", "Answer": "

I had this same problem. I attributed it to the linear bearings which ride on the 8mm travel rods. I did two things which ultimately eliminated the issue (since I did both at the same time, I'm not sure which solved the issue, but believe it's #1 below):

\n\n
    \n
  1. I replaced all of my ball type (stock) linear bearings with Igus Drylin linear bearings. I'm pretty sure the metal ball bearings were screeching along the 8mm metal rods. The Drylin bearings pretty much eliminated any noise on the rods. I replaced all of them for X, Y, & Z axis rods at the same time.

  2. \n
  3. I used white lithium grease (just a small amount) on the Z-axis screw rod. This helped everything travel better. It should also help it last longer by preventing wear.

  4. \n
\n\n

Since I've done this, I've not had any screeching anymore. You don't have to use Drylin linear bearings, as there are other printable options on Thingiverse.com.

\n" }, { "Id": "7332", "CreationDate": "2018-10-30T17:22:34.917", "Body": "

I got a Tevo Michelangelo as first 3D printer today and already got a problem with the bed leveling: The extruder nozzle is way below the build plate. (When I auto-home the axes, which I've done several times). I thought to just adjust the plate downwards, but it was incredibly slow, and there was almost no progress.

\n\n

Here's a picture, sorry for the bad quality:

\n\n

\"Extruder

\n\n

Should I continue adjusting it downwoards via the screws (It's getting harder), or is there some other way to solve this? Or am I just making a dumb mistake?

\n", "Title": "Tevo Michelangelo nozzle below build plate?", "Tags": "|nozzle|bed-leveling|build-plate|", "Answer": "

Just move the Z-endstop up a little higher, also make sure the bed leveling screws are not completely screwed in.

\n\n

So:

\n\n
    \n
  1. Move the head of the printer up.
  2. \n
  3. Move up the Z-endstop so that the nozzle is a little higher than the build platform.
  4. \n
  5. Home the printer.
  6. \n
  7. Disable the stepper motors and move the head over the bed to a certain position (e.g. a corner without disturbing the Z axis), once you get to a new position of the bed, raise the bed by unscrewing the screws until a piece of paper shows noticeable drag when pulled between the nozzle and build platform.
  8. \n
  9. Goto 3 until you have done all corners several times, also the center of the bed. It is advised to once in a while re-home the printer, this way accidental Z-axis movement is compensated again.
  10. \n
\n" }, { "Id": "7334", "CreationDate": "2018-10-30T20:46:41.930", "Body": "

As moving the endstop upwards reduces the range of the z-axis, I was wondering whether it reduces the maximum height of the printable object, by the distance the endstop was moved. Or is this somehow (to a certain extent) beeing counterbalanced?

\n\n

(Follow-up question of this question)

\n", "Title": "Does moving the z-endstop upwards affect the maximum height of the printable object?", "Tags": "|endstop|", "Answer": "

After adjusting the endstop check that the documented maximum height (150mm) is still achievable. Just move Z axis in the menu.

\n" }, { "Id": "7344", "CreationDate": "2018-11-01T15:39:29.283", "Body": "

A while back, I had worked on building a Yostwerks Sea Cruiser. Now I'm thinking of trying to create a 3D printable (or 3D millable) version of the plans.

\n

The original design uses cross-sections cut from a half-inch thick HDPE sheet. However, I am considering changing this out for one of the easier to print materials, but I cannot find any comparative information about stiffness of HDPE versus other 3D printing materials.

\n

Can I get a suggestion for a alternative to HDPE that is as stiff or stiffer as well as same density or lighter. The Kayak frame is expected to be compressing the cross-sections.

\n

An example set of cross-sections is as below:\n\"Kayak

\n

Please note that I have also considering changing the shape of the cross-sections to make stiffer yet lighter cross sections, but that will require some trade-off between portable sized cross-sections versus the stiffness of the cross-sections.

\n", "Title": "Good alternative to HDPE for FDM", "Tags": "|material|", "Answer": "

According to the chart on this page HDPE has a flexural modulus of between 0.75 and 1.575 Gpa; meanwhile PETG has a flexural modulus of 2.20 Gpa, meaning PETG is the stiffer material.

\n\n

HDPE, by definition, has a density greater than 0.941g/cc. PETG has a density of about 1.25g/cc.

\n\n

So while PETG may be denser (depending on the HDPE used), it is more rigid, while not being too rigid.

\n\n

PETG is quite printable using FDM. If you were printing at a higher temperature and high % infill, I could see it holding up well enough. I've printed heavy duty parts with it using a 0.8mm nozzle.

\n" }, { "Id": "7345", "CreationDate": "2018-11-02T00:18:26.663", "Body": "

I've been asking around on how to choose the right temperature for a given filament. I noticed that with two different brands of pla I have to print at different temperatures, and the manufacturer specifies a broad range of print temperatures.

\n\n

I keep hearing about heat towers, and have found various examples on thingiverse.

\n\n

My question is how do I actually print one? And then, how do I judge the results?

\n", "Title": "How does one use a heat tower?", "Tags": "|print-quality|filament|slicing|calibration|print-material|", "Answer": "

When you slice an STL of a heat tower, you need to tell the slicer that you need a different temperature at a certain level and maintain that new temperature until another change is requested.

\n\n

The way I usually do it is by using a post processing script in Ulltimaker Cura, but you can do it yourself quite easily by changing the G-code file manually.

\n\n

To get it to work in Ultimaker Cura is to:

\n\n\n\n

If you look into the generated G-code file (the snippet below is taken from a G-code file for an Ultimaker 3) you will see that the post processing script will add extra lines in your G-code file. E.g. the following snippet shows you 2 changes of temperature, one at 5 mm (extruder 1 @ 250 °C), the other at 10 mm (extruder 1 @ 245 °C):

\n\n
...\n;LAYER:48\nG0 X93.4 Y132.161 Z5\n;ChangeAtZ V5.1.1: executed at 5.00 mm\nM117 Printing... ch@  5.0\nM104 S250.000000 T0\n...\n...\n;LAYER:98\nG0 X93.4 Y132.35 Z10\n;ChangeAtZ V5.1.1: executed at 10.00 mm\nM117 Printing... ch@ 10.0\nM104 S245.000000 T0\n...\n
\n\n

The bottom line is that the extruder needs to be instructed to be heated or cooled to a new temperature. This is also something you could have inserted yourself manually by adding the codes M104 SXXX.000000 T0 at specific levels where XXX is the extruder temperature you need for that level.

\n\n

Basically this describes how to print the heat tower calibration prints, not the selection of the settings (e.g. extruder temperature) to use for your prints. When you created the print file you are ready to print the tower and observe the quality of printing. You can look at the print and visually inspect the temperature giving you the best aesthetic performance, the best overhang performance, the best \"filling\" performance, highest speed, etc. It is you that decides what is best for your application. Alternatively you could print some coupon tests and see which are structurally the best. Please note that additional towers for different settings like layer height, print part cooling and print speed to may be necessary to optimize the print process.

\n\n
\n\n

Sidemark, I prefer the use of parametric models (e.g. in OpenSCAD) over the STL models found on the internet, this way I can embed the exact printer setting in the tower for reference. See e.g. this answer.

\n\n

\"Heat\"Heat

\n" }, { "Id": "7355", "CreationDate": "2018-11-05T00:58:59.340", "Body": "

I am modifying some Slic3r config parameters and comparing the results. How can I have two instances (or equivalent: I would like to see two model windows with their associated configuration screens) of Slic3r at the same time? I'm on OS X, but if there is a generic (e.g. within Slic3r) solution that will be preferrable.

\n", "Title": "How can I run two instances (or equivalent) of Slic3r on OS X?", "Tags": "|slic3r|", "Answer": "

You could duplicate the application by clicking on the application and pressing Command (\u2318) + D, and then run both - although I'm not sure if that would mess with the preferences. I haven't fully tested it, but both instances opened up and worked, seemingly ok.

\n\n

\"Duplicate

\n\n

You could even rename them to something other than \"Slic3r\" and \"Slic3r copy\" to remind you which application is slicing which model, for example:

\n\n\n\n

However, Mark's answer seems much more elegant.

\n" }, { "Id": "7364", "CreationDate": "2018-11-06T00:34:34.183", "Body": "

I'm new to 3D printing and I recently got my first 3D printer, an Ender 3 Pro by Creality.

\n\n

I've tried to find information about the type of nozzles should I look for. I'm trying to find stainless steel nozzles but there are so many models (M7, M8, etc.) and I have no idea what nozzle type I should get.

\n\n

I've tried searching on Google but the only info I could find is that the extruder is an MK-10.

\n\n

I am also looking for a heating cartridge for the hot end, but I still cannot find no information about the size that I need (15 mm/20 mm/30 mm). Also looked for info about the heated bed so I know what kind of thermistor I need (I found two types and no clue which one to get).

\n\n

Where can I find some technical information about these?

\n", "Title": "What parts are suitable replacements for an Ender 3 Pro?", "Tags": "|extruder|hotend|creality-ender-3|replacement-parts|part-identification|", "Answer": "

The Ender 3 takes an M6 thread (metric 6 mm diameter). Measurement of stock nozzle shown.

\n\n

Most sellers will list compatible printers Ender 2, Ender 3, Ender 4, CR-10, CR-10S, CR-10 Mini, CR-10-S4, CR-10-S5, CR-8, CR-7. Will Also Fit Any Other MK10 Heater Blocks.

\n\n

I recently bought some titanium alloy nozzles as the brass one got closed over after crashing into the glass bed.

\n\n

\"Measurement

\n" }, { "Id": "7369", "CreationDate": "2018-11-06T21:39:45.397", "Body": "

After few good prints with my new Ender 3, I noticed that prints with longer shell straight lines (> 5 cm) have extrusion problems in the middle of those lines.

\n\n

I figured that this might be because the printing speed (70 mm/s) and filament temperature (210 \u00b0C for PLA). So I increased my filament temperature about 215 \u00b0C and lowered print speed to 60 mm/s. I am using Cura v3.5.1. For the next few smaller prints everything was perfect.

\n\n

Now I am printing a bigger 128 mm X 48 mm rectangular shape. In the middle of the 128mm line of the outer shell, I have signs of under extrusion. Looks like dents.

\n\n

I also notice that the head is really fast during this 128 mm travel. No way ~2 seconds would take 60 mm/s to travel 128 mm. It is more like 1 second or less.

\n\n

Why would my head travel at such speeds?

\n\n

I am attaching my speed settings.

\n\n

\"Speed

\n\n

I am attaching the image of the printed piece and the defect generated.

\n\n

\"The

\n", "Title": "Ender 3 seemingly speeds up too much on straight long shell lines", "Tags": "|ultimaker-cura|creality-ender-3|speed|", "Answer": "

Your travel speed is set to 120mm/s so it would make sense the 128mm travel takes ~1 second. You most likely have combing turned on so that it doesn't not need to retract on travels. This makes it ooze plastic as it travels and would mimic underextrustion.

\n" }, { "Id": "7372", "CreationDate": "2018-11-08T13:41:13.097", "Body": "

I started using rafts more often, to get better results with complex or fragile parts, but my Ender 3 consistently under-extrudes the initial outside line of the raft (for the first inch or two, where extrusion starts).

\n\n

It is often very thin and does not adhere. This often leads to problems with the following pattern of raft layer 1 curling up (ABS) as it does not meet the edge line (due to the 1-2 inch gap in the perimeter).

\n\n

Skirts avoid this problem by getting the flow going, but rafts only print a single outer line.

\n\n
    \n
  1. Is it possible to specify more than one outer line on a raft in Ultimaker Cura?
  2. \n
  3. Is it possible to add a skirt to a print that has a raft (or at least some initial printing to get the flow going)?
  4. \n
  5. Is it possible to add some initial G-code that will extrude a line, say from near the start position to the start of the print?
  6. \n
\n\n

\"example

\n", "Title": "Adding additional extrusion to a raft (raft with a skirt?)", "Tags": "|ultimaker-cura|extrusion|creality-ender-3|", "Answer": "

The latest update to Cura (3.6.0 at time of writing) added a printer configuration specifically for the Ender 3.

\n\n

The Start G-code script now includes the following:

\n\n
; Ender 3 Custom Start G-code\nG28 ; Home all axes\nG92 E0 ; Reset Extruder\nG1 Z2.0 F3000 ; Move Z Axis up little to prevent scratching of Heat Bed\nG1 X0.1 Y20 Z0.3 F5000.0 ; Move to start position\nG1 X0.1 Y200.0 Z0.3 F1500.0 E15 ; Draw the first line\nG1 X0.4 Y200.0 Z0.3 F5000.0 ; Move to side a little\nG1 X0.4 Y20 Z0.3 F1500.0 E30 ; Draw the second line\nG92 E0 ; Reset Extruder\nG1 Z2.0 F3000 ; Move Z Axis up little to prevent scratching of Heat Bed\n; End of custom start GCode\n
\n\n

This prints 2 initial lines, right along the left edge of the print bed. This solves the priming issue completely and results in a perfect initial border on the rafts.

\n\n

This would indicate this priming issue is a known problem with the Ender 3.

\n" }, { "Id": "7379", "CreationDate": "2018-11-09T07:50:22.720", "Body": "

I\u2019ve just received my first 3D printer, the Ender-3 and while\nassembling the unit I noticed that the nozzle is slanted and\nnot vertical on its holder unit.

\n\n

\"Ender-3

\n\n\n", "Title": "Creality Ender-3 slanted nozzle", "Tags": "|hotend|creality-ender-3|", "Answer": "

This is not normal, the heat break (which is the cylindrical part between the nozzle block and the upper cooling part assembly is probably bent in your example) should be perpendicular to the 2020 guide beam (X-axis) so that the nozzle exit is exactly perpendicular to the (heated) platform.

\n\n

You either received a faulty heat break, or too much force is exerted during assembly. You should contact the seller.

\n\n

You could disassemble (after you contacted the seller) to see if the heat break is bent or something else is incorrect. Once you have the heat break out, you can measure the dimensions and order a replacement locally (for speed). It is advisable to have some heat breaks, nozzles, and even a heater block, heater cartridges and thermistors available as spares, these are pretty cheap to obtain through the regular Chinese vendor sites or auction sites.

\n\n

If the heat break is bent, bending it back is not advised as this heat break easily tears open as it usually consists of thin walls.

\n" }, { "Id": "7382", "CreationDate": "2018-11-09T14:48:22.973", "Body": "

What are the most common 3D printing file formats, and which one is more effective or used more than others?

\n", "Title": "What are the most common 3D printing file formats?", "Tags": "|print-quality|file-formats|", "Answer": "

3D-Model Exchange Files

\n

The most common file formats to exchange models for 3D printing are STL, OBJ, FBX, COLLADA, 3DS, IGES, STEP, and VRML/X3D.

\n

Of these, STL, OBJ, AMF, and 3MF are the most popular formats according to All3DP. Pointing out which is the best is a subjective interpretation and not fit for SE sites. However, for many of the model sharing sites, like e.g. Thingiverse.com, many people share the STL file format.

\n\n

Actual Printing Files

\n

In the end, all these formats store the models of your products which eventually need to be transformed into 3D printer understandable instructions. These instructions are called G-code commands.

\n

Lately I also noted that Ultimaker Cura stores sliced models in GZ file format for my Ultimaker 3 Extended, which is a compressed archive from the GZIP compression utility containing the actual G-code file.

\n" }, { "Id": "7393", "CreationDate": "2018-11-10T21:29:03.850", "Body": "

I have been have a problem printing ABS with Cura on an Ender 3.

\n\n

I am getting good bed adhesion on the raft, but the object on top of the raft easily curls off it. See photo below.

\n\n

Settings:

\n\n\n\n

How do you control how strongly the raft adheres to the object in Cura?

\n\n

\"Curling

\n", "Title": "Getting a Cura Raft to stick to the object more firmly (for ABS)", "Tags": "|ultimaker-cura|adhesion|creality-ender-3|rafts|", "Answer": "

You should reduce the air gap option called \"Raft Air Gap\". Printing ABS optimally requires an enclosure of the printer, wind and draft cause temperature changes which could lead to layer separation.

\n" }, { "Id": "7395", "CreationDate": "2018-11-10T22:23:45.753", "Body": "

I have just installed the BLTouch on my Ender 3 running Marlin 1.1.9 and I want to set it up so the probing does the 9 points in a perimeter that uses most of the bed surface area. At the moment the probe just covers a small area.

\n\n

\"enter

\n\n

I read that to sort it out you have to change the section \" // Set the boundaries for probing (where the probe can reach).\" but I can't figure out what to set it as, my current settings are as follows:

\n\n
 #define LEFT_PROBE_BED_POSITION 30\n #define RIGHT_PROBE_BED_POSITION (X_BED_SIZE - 120)\n #define FRONT_PROBE_BED_POSITION 30\n #define BACK_PROBE_BED_POSITION (Y_BED_SIZE - 120)\n
\n\n

When I try to set it so something like:

\n\n
  #define LEFT_PROBE_BED_POSITION 30\n  #define RIGHT_PROBE_BED_POSITION (X_BED_SIZE - 35)\n  #define FRONT_PROBE_BED_POSITION 30\n  #define BACK_PROBE_BED_POSITION (Y_BED_SIZE - 35)\n
\n\n

I get an error reading :

\n\n
  RIGHT PROBE_BED POSITION is outside the probe region.\n
\n\n

Any help you can give me on what I am doing wrong will be greatly appreciated.

\n\n

The probe offset from the extruder are as follows:

\n\n
#define X_PROBE_OFFSET_FROM_EXTRUDER -41  // X offset: -left  +right  [of \nthe nozzle]\n#define Y_PROBE_OFFSET_FROM_EXTRUDER -5  // Y offset: -front +behind [the \nnozzle]\n#define Z_PROBE_OFFSET_FROM_EXTRUDER -1  // Z offset: -below +above  [the \nnozzle]\n
\n\n

For completeness, this is the G-code that I am using: CE3_square.gcode.

\n", "Title": "How to set the boundaries for BLTouch probing", "Tags": "|bed-leveling|creality-ender-3|bltouch|", "Answer": "

As explained in this answer it is you that needs to calculate where the head (read sensor) is supposed to go depending on the amount of space you have left on the carriage shafts.

\n

As can be seen from your configuration, your sensor is located on the left front (when facing the machine) at 41 mm left from the nozzle you are requesting the sensor to travel to a position that is outside the bed (35 mm), so you should at least subtract 41 mm from your X bed size.

\n

As you have not posted a bed size, an assumed X bed size of 220 mm would mean that the maximum probing distance for X (i.e. RIGHT_PROBE_BED_POSITION) of 220 - 41 = 179 mm

\n

In your case -41 mm would take it to the edge, so 30 mm extra would make it symmetrical:

\n
#define LEFT_PROBE_BED_POSITION 30\n#define RIGHT_PROBE_BED_POSITION (X_BED_SIZE - 41 - 30)\n#define FRONT_PROBE_BED_POSITION 30\n#define BACK_PROBE_BED_POSITION (Y_BED_SIZE - 5 - 30)\n
\n

Furthermore, it is way more convenient to set the Z distance between the probe and the nozzle using M851 Zxx.xx (where xx.xx denotes the offset as a negative value) than setting it with Z_PROBE_OFFSET_FROM_EXTRUDER.

\n" }, { "Id": "7397", "CreationDate": "2018-11-11T01:56:33.287", "Body": "

Can I use a 2D photo to get a 3D printing model that gives a good result?

\n", "Title": "Can I use a 2D photo to get a 3D printing model?", "Tags": "|3d-models|graphics|", "Answer": "

With just 1 2D photo, you can never get a 100% accurate 3D object. That is because you don't really know about the hidden parts. Think of a photo of a smiling mischievous child that is facing you, and you can't see that they are holding a sling shot behind their back. However, with multiple 2D photos, one can reconstruct a 3D object, provided all the parts are seen. This is called photogrammetry. You can search wikipedia and youtube for in-depth discussion about this. It is an entire science. There are applications that do this for you, but I can't recall the details right now.

\n" }, { "Id": "7405", "CreationDate": "2018-11-12T20:50:43.540", "Body": "

I am attempting to print an axial fan blade, and due to the size requirements, I have to angle it on its side (despite only being 7.44 inches in diameter...). I am using an Ultimaker 3, which I thought was able to print 215mm (or 8.46 inches). This leads me to two questions:

\n\n

Question 1: If my diameter is only 7.44 inches, why am I unable to lay the fan blade flat (see below - notice the dimensions are within the bounds)? \"Fan

\n\n

Question 2: What is so wrong with the surfaces in red that Cura cannot print, even with supports? I am using PVA as a support material, yet there is something wrong with my model in Cura. I checked the X-ray view and nothing seemed amiss. \"enter

\n", "Title": "Ultimaker Cura problems with build platform size and overhang support", "Tags": "|ultimaker-cura|support-structures|build-plate|", "Answer": "

Ultimaker Cura print bed size sizes (shrinks) when you enable skirt, brim or raft build adhesion options. Try to print without build plate adhesion option if you want to push printing to the limits of the build plate. Furthermore, you mention the use of PVA, using 2 cores, the priming print tower also needs space to be printed, this can be also limiting your build plate space. Note that you can define where to print the tower, you could try locating it in a corner where the fan does not reach.

\n\n

From your screenshot with the fan flat it appears that you have selected dual extrusion. Note that the bed size depends on the amount of cores you use, from Ultimaker:

\n\n
\n

Left or right nozzle:
215 x 215 x 300 mm
(8.5 x 8.5 x 11.8 inches)

\n \n

Dual extrusion:
197 x 215 x 300 mm
(7.8 x 8.5 x 11.8 inches)

\n
\n\n

As an example I have made a 7.44\" cylinder of 2\" height and put it onto the UM3E build plate to show the differences when you use dual extrusion.

\n\n

This image shows the cylinder when printed with a single core:\n\"Single

\n\n

This image shows the same cylinder, but now with enabled \"Extruder 2\" as the \"Support Extruder\" option:\n\"Dual

\n\n
\n\n

Having an Ultimaker 3E myself, printing PVA might not be very good choice as you need to print a lot of support material when printing upright. PVA cooks easily even in the B core, it also needs to be very dry, PVA takes up moisture very fast and becomes useless after that. At work we have much better experience with break-away support material.

\n\n

Another consideration why you should not print this upright is the orientation of the layers. It is generally known that the layer to layer bonding is less strong that the material itself, it at least provides a lot of notches from which cracks can initiate and grow. This means that if you want to print this, you should lay it flat, this way the centrifugal force is in line with the layer deposition direction.

\n\n
\n\n

With the correct layer height (read thin layers), you should be able to print this flat without support. I have printed similar structures like turbo turbine wheels with similar overhangs without support.

\n" }, { "Id": "7410", "CreationDate": "2018-11-13T21:34:48.807", "Body": "

I want to calibrate my printer for my new BLTouch (Ender 3 using Marlins 1.1.9), every time I adjust the values in the configuration.h and send it to the printer I end up having to wait for the nozzle and the bed to heat up, it also wastes plastic. \nIs there a way to take out the values in the code that make the extruder turn without causing any damage.\nthe slicer that I am using is Cura and the print that I want to trace is this calibration crosshair. Any help will be greatly appreciated.

\n", "Title": "How To Trace The Line Of A Print With The Nozzle (With No Plastic Printed)", "Tags": "|ultimaker-cura|calibration|bltouch|", "Answer": "

Sure. Comment out your E values and the temp set/wait lines:

\n\n

...\nM109 S245\nM190 S120\n...\nG1 X123.113 Y95.644 E12.81357 F250\nG1 X125.613 Y94.214 E13.06364 ; skirt\nG1 X128.409 Y93.523 E13.31370 ; skirt\nG1 X131.287 Y93.624 E13.56377 ; skirt\nG1 X133.471 Y94.259 E13.76129 F250 \nG1 X133.771 Y94.384 E13.78949 ; skirt\n...

\n\n

Becomes:

\n\n

...\n;M109 S245\n;M190 S120\n...\nG1 X123.113 Y95.644 F250 ; E12.81357 \nG1 X125.613 Y94.214 ; E13.06364 ; skirt\nG1 X128.409 Y93.523 ; E13.31370 ; skirt\nG1 X131.287 Y93.624 ; E13.56377 ; skirt\nG1 X133.471 Y94.259 ; E13.76129 F250 \nG1 X133.771 Y94.384 ; E13.78949 ; skirt\n...

\n\n

Now the printer won't wait for any temperatures, and won't dispense any material.

\n" }, { "Id": "7412", "CreationDate": "2018-11-14T01:37:58.177", "Body": "

When I adjust the probe offset to get the proper layer height and adhesion, one side is set right but the other doesn't work. If I decrease the probe offset (say -1.100 to -1.050) the right side seems to print better; if I increase the probe offset (-1.050 to -1.125) the left side will adhere to the plate properly.

\n\n

Before I added the BLTouch and upgraded the firmware to Marlin 1.1.9 I was able to print a perimeter square (with a little manual adjustment).

\n\n

My Printer is the Ender 3 and I am using a glass bed and the model is this centre crosshair

\n\n

\"enter\n\"enter\n\"enter\n\"enter

\n", "Title": "Automatic leveling probe offset; adjusting for one side ruins the other", "Tags": "|bed-leveling|creality-ender-3|bltouch|", "Answer": "

From the looks of your test print, this is unlikely all or only due to levelling unless the glass plate is abnormally uneven. You may have to check your extrusion (speed, temp, feed) parameters and/or adhesion (clean with aceton and/or prepare with hairspray or glue stck)

\n" }, { "Id": "7417", "CreationDate": "2018-11-14T15:40:07.400", "Body": "

I am trying to add the line

\n\n
\n

G4 P4000 G1 F4000 E-50

\n
\n\n

to pause and retract the print in order for the printed line to dry (printing in mid air vertically). Does the above code work or should i skip lines or does it not matter at all?

\n\n
\n

G4 P4000
\n G1 F4000 E-50

\n
\n\n

Would the following line do the exact same thing?

\n\n
\n

G4 P5000 E-50

\n
\n", "Title": "Is G-code read line by line?", "Tags": "|g-code|", "Answer": "

Yes, G-code is read line by line. G-code is a numerical control programming language. It basically instructs the machine sequentially line by line to do a specific task. The printer than executes the lines one by one until it reaches the end.

\n\n

If you instruct the printer to wait (G4 dwell), it will do the wait/dwell first and than will execute the next command to retract the filament. As such, your examples will not work if you want to retract the filament during the pause, you reversed the process if you want to achieve that.

\n\n

To add a pause (simple) for e.g. filament changing, you should instruct the head to go to a certain position, extract the filament, and now insert the pause/dwell command. Give yourself enough time to insert and prime the nozzle and go back to the last location to continue printing.

\n\n

You could insert something like (e.g. in between layer change, before G1 Zx.xx):

\n\n
...\nG1 X0 Y0 F2000 ; Relocate the print head\nG1 F4000 E-50 ; Retract filament\nG4 P40000 ; Wait for 40 seconds\nG92 E50 ; The new filament should continue at this value\n...\nG1 Zx.xx\n
\n\n

Depending on what happens after G1 Zx.xx, you may need to set the head back to the location prior to where it was before G1 X0 Y0 F2000.

\n\n

Do note that there are pausing scripts/plugins available for e.g. Ultimaker Cura, and there is also a filament changing command M600 that can be enabled for certain firmware (if this is your ultimate goal).

\n\n

Using a post processing plugin of Ultimaker Cura, a pausing script looks like:

\n\n
\n...\nG0 X137.692 Y105\n;TIME_ELAPSED:707.873599\n;TYPE:CUSTOM;added code by post processing\n;script: PauseAtHeight.py\n;current z: 5\n;current height: 5.0\nM83\nG1 F300 Z6\nG1 F9000 X190 Y190\nG1 F300 Z15\nM104 S0; standby temperature\nM0;Do the actual pause\nM109 S200; resume temperature\nG1 F300 Z6\nG1 F9000 X133.423 Y105\nG1 F9000\nM82\nG92 E911.50045\n;LAYER:24\nG0 X137.692 Y105 Z5\n...\n
\n\n

Note that G0 and G1 are \"move to\" location instructions (albeit through a different way, fast move and linear move respectively). If you look closely, you see that after the pause, the printer returns to the X-Y position where it left prior to the pause (X137.692 Y105).

\n\n
\n\n

Side note:
\nSome firmware flavors allow buffering, but each statement is executed sequentially.

\n" }, { "Id": "7432", "CreationDate": "2018-11-16T10:26:06.903", "Body": "

Given the Marlin Firmware what is the difference between the following lines of code:

\n\n
\n

G4 S20

\n
\n\n

and

\n\n
\n

G4 P2000

\n
\n", "Title": "G4 S20 vs. G4 P2000", "Tags": "|g-code|", "Answer": "

The answer is that it depends on the type of firmware you are using.

\n\n

Let us look at the documentation of G4 to find that G4 is valid for all the listed firmware types:\n\"enter

\n\n
\n

Pause the machine for a period of time.

\n
\n\n

Furthermore it states that:

\n\n
\n

Parameters

\n \n \n
\n\n

It clearly shows that the S parameter (which defines the pause in seconds) is only supported by a few firmware types. Do note that this documentation may not be up-to-date, so it is best to look into the source code or the users manual of the particular firmware you are using.

\n\n
\n\n

E.g. if you are using Marlin Firmware, G4 S20 will pause the machine for 20 seconds while G4 P2000 will pause the machine for 2000 milliseconds which is 2 seconds. This means that a different time is requested, to have 20 seconds waiting time you could use G4 P20000

\n\n

To answer your question what the actual difference between the 2 commands is:

\n\n\n" }, { "Id": "7436", "CreationDate": "2018-11-16T18:19:05.457", "Body": "

I'm connecting to a printer that's using Marlin 1.0.3 dev firmware through the pyserial Python package.

\n\n

I want to stream large gcode files into the printer and I guess that the buffer has a limited size. How do I know when the buffer is full, so I can wait until a command is processed to send a new one?

\n\n

I've tried reading a big file, sending all the commands and checking for the printer responses in another thread. I didn't get an error or something that indicates that the buffer is full. I've also didn't found anything about this in the official docs.

\n\n

It's my approach correct? Is there something I'm missing?

\n\n

Some of the code:

\n\n
def check_for_responses():\n    while(True):\n        response = printer_connection.serial.readline()\n        print(response)\n\n\nasync def stream_gcode():\n    await printer_connection.open_connection('COM7')\n    Thread(target=check_for_responses).start()\n    asyncio.sleep(2) # wait to initialize\n    file = open(\"test.gcode\", 'r')\n    for line in file:\n        command= printer_connection.format_command(line)\n        printer_connection.serial.write(command.encode())\n\nasyncio.new_event_loop().run_until_complete(stream_gcode())\n
\n", "Title": "Strategy to stream large gcode file from serial port", "Tags": "|marlin|", "Answer": "

Well, after some research I found something about Buffering on the RepRap documentation. I didn't see it before.

\n\n

Works like this:

\n\n
    \n
  1. You send a command. An \"ok\" response it's received immediatly, that means that the command was successsfuly added to the buffer.
  2. \n
  3. Then you can send another command. When the buffer is full, the \"ok\" response will be delayed until a new command it's received.
  4. \n
\n\n

Some commands, for example M190: Wait for bed temperature to reach target temp will cause that the \"ok\" response will be delayed until the command finish his execution. That means that the printer won't accept more commands in the meantime.

\n\n

I still have to check how many commands the buffer can save approximately, and if it's convenient to fill it entirely. I guess not, because let's say you want to pause the print. If you added a lot of commands to the buffer, you'll not able to pause it until all of them finished.

\n\n

This thread on the RepRap forum was also useful.

\n" }, { "Id": "7440", "CreationDate": "2018-11-17T18:09:04.323", "Body": "

Given a Marlin firmware and a line of G-code such as the following:

\n\n
\n

G1 F100 X50 Y50 Z0 E-10

\n
\n\n

What defines the speed at which the stepper motor associated with the E-value is retracting? It is my understanding that the Feed Rate defines the speed of the movement (in this case 100mm/m) but I am not clear how I could accelerate a retraction?

\n\n

The reason I am asking is that I am not seeing a swift removal of material as i retract. Could the slow feed rate be the issue? I am using a pellet printer (WASP 3MT) and generating G-code from polylines on Silkworm.

\n", "Title": "What defines the speed at which a motor is retracting and how can it be changed?", "Tags": "|marlin|g-code|", "Answer": "

It seems like you are particularly talking about your extruder, please correct me if I have misread.

\n\n

In the command G1 F100 X50 Y50 Z0 E-10:

\n\n\n\n

If you are not experiencing high enough retraction speed:

\n\n\n\n

Notes:

\n\n\n" }, { "Id": "7446", "CreationDate": "2018-11-18T20:46:12.157", "Body": "

I'm looking for the specific density of the GEL-LAY and LAYWOO 3D materials by manufacturer CC Products.

\n\n

It isn't noted on their website or on the spool or the box the spools came in. I've looked for hours on Google and various websites, from resellers to people who tested it, without being able to find it.

\n", "Title": "Density of GEL-LAY and LAYWOO 3D print material?", "Tags": "|filament|print-material|tpe|pva|", "Answer": "

Indeed, the properties of this filament are kept rather secret, so to find out what the density is, you need to either contact the filament supplier or the manufacturer for accessing the data sheet or calculate this yourself. The answer below expands on the \"calculate it yourself\".

\n\n

Density is defined as $\\rho = \\frac{m}{V}=\\frac{[kg]}{[m^3]}$. The use of this formula has been show in this answer. The drawback of that answer is that it is an approximation that relies on a uniform piece of filament that requires cutting off expensive filament and relies on assumptions rather than actual calculations. Furthermore, the weighing of a small piece of filament is much less accurate of a small piece than for a larger piece or the whole spool (for the same scale, so a decent kitchen scale might be usable when more weight is concerned). You could improve the density calculation by measuring the diameter at various sections and make a better approximation based on the average diameter, but still that would need you to unroll the spool and carefully measure a piece of filament (and cut it). The advantage of that answer is that it is far easier than my proposition.

\n\n

The method that is proposed here relies on a well known method to calculate the density of materials that is called hydrostatic weighing. Hydrostatic weighing uses the displacement of a fluid due to a submerged object to determine the density of the object. Any submerged object will displace the fluid surrounding it by it's own volume, as such you would need to measure the rise of the volume level to read the volume of the submerged product. This can be done accurately by using methods that include containers of known dimensions, known fluids and even an overflow method and weighing.

\n\n

If the filament comes on a spool, you would require an identical spool to prevent removing it from the spool. But, I read that it is sold in bundles, not on spools. Without a spool would make it even easier to calculate the density as you do not have to subtract the spool weight and volume, the answer continues as if you have it on a spool. This is purely necessary so that you would not need to cut off filament or unroll the whole spool. The suggestion below let's you measure the whole spool, so weight is measured more easily as there is a lot more.

\n\n

Theoretically, you could put the filament in a fluid which is known to not affect the filament properties (so not water for GEL-LAY!) in a bath of known dimensions. Once the spool and filament are completely submerged, you could measure the volume rise. If you do the same for the empty spool, you also know the volume of the spool alone. If you also are able to weigh the empty spool and the full spool (before you plunged them in the \"bathtub\"/container), you now know the volume and the weight of the filament, dividing the weight (full spool weight minus empty spool weight) by the volume (full spool submerged volume minus empty spool submerged volume) will give you the density.

\n\n

$$\\rho_{filament} = \\frac{(m_{full\\ spool}-m_{empty\\ spool})}{(V_{full\\ spool}-V_{empty\\ spool})}$$

\n\n


\nNow let your filament dry for a long, long time! :)

\n" }, { "Id": "7447", "CreationDate": "2018-11-18T20:50:05.237", "Body": "

POROLAY GEL-LAY is a kind of jelly-like material made by exotic filament producer Kay Parthy / CC Products.

\n\n

It is a mix of TPE and PVA that is supposed to have a jelly-like feeling. Once printed, you need to let your print soak in water for several days to remove all of the PVA, then what's left should have a jelly-like feeling since the PVA will have left many micro-cavities in the object.

\n\n

However, my initial tests end up producing prints that feel a bit like jelly when wet, but reverts to a sort of cardboard feeling once dry.

\n\n

Does this material needs to stay moist to keep the jelly feeling or not?

\n", "Title": "Does GEL-LAY needs to stay moist?", "Tags": "|filament|print-material|tpe|pva|", "Answer": "

No, GEL-LAY should not be kept moist to stay flexible. It is made from TPE and filled with small PVA particles. It is advised to print less than normal amount of walls and infill so that the PVA can be rinsed out more easily.

\n\n

If you look at the way the manufacturer rinses the material, you frequently see ultrasonic baths that improve the dissolving of the PVA. Once the PVA is removed, TPE is all that it left, which is elastic. If your print is still hard after drying it means that not all the PVA has dissolved yet, once dry it will become gluey/sticky again like PVA, hence the cardboard structure.

\n" }, { "Id": "7450", "CreationDate": "2018-11-19T09:28:33.463", "Body": "

Is there a difference between 3D printing and additive manufacturing if any then explain?

\n", "Title": "Is there a difference between 3D printing and additive manufacturing?", "Tags": "|terminology|", "Answer": "

I see several answers to that.

\n\n

A lot of persons say 3d printing while they really mean additive manufacturing. For them, the machine got a 3d part out of raw material as a 2d printer got them images out of sheets.

\n\n

Some persons think that 3d printing refers to lowpriced polymers additive manufacturing machines while additive manufacturing refers to industrial, expansive equipment like the so-called \"DMLS\"or \"SLM\".

\n\n

Others persons say that additive manufacturing processes that use 2d-printers heads (ink injectors, such as HP additive manufacturing machines) should be called 3d printers and 3d printing because it's so similar to 2d printers.

\n\n

I consider that \"3d printer\" should be avoided in general, because it is way too vague and referring to way too much different things. But it is up to anyone using these words, I just think that a serious person wouldn't use \"3d printer\" given all the mismatch that it could generate. I think we should use the exact process' name instead (like FDM for low-priced additive manufacturing polymer machines), or if you refer to the overall technology, use additive manufacturing.

\n" }, { "Id": "7455", "CreationDate": "2018-11-19T22:10:06.197", "Body": "

In a previous question, I have modified the Z-axis-height of my Prusa Mk3 from 250\u00a0mm to 350\u00a0mm.

\n

Prior to the MMU2 upgrade, I changed the code in my printer to accept the new height the following way: I changed Arduino \u2192 Configuration_prusa.h \u2192 Axis settings \u2192 line 54 to #define Z_MAX_POS 210 (it worked perfectly).

\n

However, the layout is different for the new firmware v. 3.4.1, and I have no idea where the Z axis settings have gone, if they are even still there.\nCan anyone point me in the right direction to find the Z MAX POS settings, please?

\n

Obviously, I'm not very good with coding, but it looks like the entire code layout has changed for the v. 3.4.1

\n

Firmware

\n

Prusa MK3 (firmware 3.4.1) MMU2 (firmware 1.0.1). The source code on Github is found here.

\n", "Title": "How to re-program NEW Prusa MK3 MMU2 to accept taller Z axis", "Tags": "|prusa-i3|firmware|z-axis|prusa-mmu|", "Answer": "

Latest firmware: Prusa-Firmware-3.5.0\\Prusa-Firmware-3.5.0\nArduino 1.6.9, RepRap Arduino-compatible Mother Board RAMBo by Ultimachine latest version 1.0.1, Arduino AVR Boards Built-in by Arduino latest version v.1.6.23.

\n\n

Copy the 1_75mm_MK3-EINSy10a-E3Dv6full file from\nPrusa-Firmware-3.5.0\\Prusa-Firmware-3.5.0\\Firmware\\variants into \nPrusa-Firmware-3.5.0\\Prusa-Firmware-3.5.0\\Firmware and rename it Configuration_prusa.h.
\nOpen Firmware Arduino file. Configuration_prusa.h shows in the tabs next to Configuration_adv.h. Change //Travel limits after homing #define Z_MAX_POS from 210 to 350. Verify \u2192 \nSketch uses 225,482 bytes (87%) of program storage space. Maximum is 258,048 bytes.\nGlobal variables use 6,091 bytes of dynamic memory.

\n" }, { "Id": "7462", "CreationDate": "2018-11-21T05:47:39.893", "Body": "

I have a CR-10S 500 and want to change a capacitor on it to improve and solve temperature issues. This capacitor that needs to be changed should be labeled as \"C4\" as mentioned on this post but it's not present on my motherboard. The goal is to replace 100uF capacitor with 220uF 16V capacitor.

\n\n

This is what the motherboard should look like:

\n\n

\"enter

\n\n

but mine looks different. Below is what it looks like:

\n\n

\"enter

\n\n

There is no version number on my motherboard and there is no \"C4\" capacitor. Also, the component that's labeled \"330\" on the original board is labeled \"470\" on my motherboard. It's hard to decide which capacitor to change. On my motherboard, there two capacitors instead of one, closer to the location of the capacitor that needs to be replaced. They are labeled \"C42\" and \"C35\" instead of \"C4\". I do not want to replace the wrong one since it's risky enough to replace just one capacitor on these boards.

\n\n

Anyone know my motherboard type or version? Also, which capacitor to change?

\n\n

EDIT:

\n\n

Here is a better or zoomed in section of the place:

\n\n

\"enter

\n\n

As Trish requested in the comment, below is also the back side of the motherboard.

\n\n

\"enter

\n\n

theSealion might be right in his answer and I did a test and it all points to \"C31\" as he suggested but the problem is that the capacitor is different from the one it is supposed to be replaced with like on other CR-10 boards. I am not entirely sure if this is the capacitor since the type of capacitor are different from the one I was suggested to use. Below is the new capacitor:

\n\n

\"enter

\n\n

My current idea is to remove the tiny \"C31\", solver wire to pin 4 of LM2596 and to the new capacitor but I do not want to remove the \"C31\" because I don't want anything to go bad.

\n\n

Can I add the new capacitor parallel to the existing \"C31\" without removing the \"C31\"?

\n", "Title": "Which capacitor to change on CR-10S S500 motherboard?", "Tags": "|heat-management|creality-cr-10|power-supply|", "Answer": "

An important note on Voltage Rating

\n

Just to add to the existing answers.

\n

If you are planning on upgrading the capacitors to some with larger capacitance, then assuming that they are for the supply regulation/smoothing then upping the capacitance shouldn't be (too much of) an issue. If they are used for timing, i.e. in an RC circuit (which seems unlikely in this particular case), then the capacitance should/must be kept the same.

\n

However, the main issue that I see is that the existing capacitors are rating at 35\u00a0V, and you are planning on adding lower rating capacitors, i.e. 16\u00a0V. This is asking for trouble and at best an early failure, or blown capacitor.

\n

The voltage rating should be between 1.5-2.0 times the voltage of the circuit, or better still (possible overkill), double the circuit voltage and then use the next voltage rating.

\n

So, for a 12\u00a0V circuit, use a 25\u00a0V rated capacitor, or 35\u00a0V. The 16\u00a0V is a bit of a no-no, and besides you don't want to mix voltage ratings for the same circuit, not a good idea, unless you actually want a common point of failure.

\n

See sources: Selecting voltage rate for capacitors and High Voltage Capacitor, in a low voltage system?

\n" }, { "Id": "7464", "CreationDate": "2018-11-21T11:53:53.633", "Body": "

Working on getting my Anet A8 upgraded with some extra toys and although the sensor works, I can't get it to home correctly. I'm looking for some assistance to home the X in the center.

\n\n

The probe is working and showing results, but when homing, it does not home to center (SAFE_HOME is turned on, Y homes to center but X is about 3/4 from endstop) and the Z offset always reports as 0 when using M851. There is a clear gap of about 10 mm between the bed and the nozzle after G29 is ran.

\n\n

I'm using the following mount: \n\"View

\n\n

Bed is anycubic ultrabase 220*220

\n\n

I have been using a combination of E3D configuration documents,\u00a0https://www.youtube.com/watch?v=WWDkZtWwd6I\u00a0and\u00a0https://www.youtube.com/watch?v=y_1Kg45APko\u00a0to try and get it working.

\n\n

I've posted my Configuration.h here to see if anyone can spot what I've done wrong and possible explain it to me so I can learn.

\n", "Title": "Anet A8 with 3DTouch does not home X in the center", "Tags": "|marlin|anet-a8|bltouch|", "Answer": "

Your sensor offset for the X direction is incorrect.

\n\n

#define X_PROBE_OFFSET_FROM_EXTRUDER -28 // X offset: -left +right [of the nozzle]

\n\n

should be

\n\n

#define X_PROBE_OFFSET_FROM_EXTRUDER 28 // X offset: -left +right [of the nozzle]

\n\n

The hints in the comment already suggest that (left is negative, right is positive), your touch sensor clearly is located on the right side, hence +28 mm.

\n\n

See also this hint in the configuration file:

\n\n
\n *      +-- BACK ---+\n *      |           |\n *    L |    (+) P  | R -- probe (20,20)\n *    E |           | I\n *    F | (-) N (+) | G -- nozzle (10,10)\n *    T |           | H\n *      |    (-)    | T\n *      |           |\n *      O-- FRONT --+\n *    (0,0)\n
\n\n

Also, #define Z_PROBE_OFFSET_FROM_EXTRUDER -1.05 is not really necessary. Many prefer this to be 0 and set the value later by using M815 Z-1.05 (sent over USB or \"printed\" from .gcode file over SD card)

\n\n

You also have not set the boundaries for the sensor to reach, I quote:

\n\n
// Set the boundaries for probing (where the probe can reach).\n//#define LEFT_PROBE_BED_POSITION MIN_PROBE_EDGE\n//#define RIGHT_PROBE_BED_POSITION (X_BED_SIZE - MIN_PROBE_EDGE)\n//#define FRONT_PROBE_BED_POSITION MIN_PROBE_EDGE\n//#define BACK_PROBE_BED_POSITION (Y_BED_SIZE - MIN_PROBE_EDGE)\n
\n\n

This can be found in How to set the boundaries for BLTouch probing.

\n\n

After a G29, the head is raised as instructed by the configuration file to employ clearance.

\n\n

An M851 will report the actual value it is set to, if you have not set it (by using M851 Z-x.xx, or set it through the display using the menu) then it will report 0, that is correct.

\n" }, { "Id": "7466", "CreationDate": "2018-11-21T21:45:44.417", "Body": "

I am confused about installing BLTouch on CR-10S.

\n\n

This is my motherboard:

\n\n

\"enter

\n\n

Below is a screenshot for the wiring of the BLTouch sensor:

\n\n

\"enter

\n\n

See the circled sections in yellow.

\n\n

Notice that the Z- socket is also used (black and white BLTouch wire) and that's normally where the endstop limit switch sensor is plugged in.

\n\n

If I unplug the limit switch wire and simply plug the BLTouch wire to this:

\n\n

1. How can the printer determine when it has reached home?

\n\n

2. How does it know when to stop when it hits home?

\n\n

3. I thought BLTouch is used to do some leveling on the surface... Does BLTouch have a sensor to stop the printer when it hits some?

\n\n

4. Any way to install BLTouch without removing the Z limit switch sensor?

\n\n

Sorry for the questions. I am just confused about BLTouch using pins of another sensor that's also required.

\n", "Title": "Is endstop / z-probe wire required for BLTouch installation on CR-10S", "Tags": "|creality-cr-10|bltouch|", "Answer": "

A touch sensor is also a height limiting switch, so you sacrifice the Z- limit switch in favor of the touch sensor (or an inductive sensor). Furthermore, all your concerns are taken care of by the firmware. You configure the firmware so that the nozzle homes Z in the center of the build plate after X and Y are homed first.

\n\n

It is advised to look up some videos of working touch sensors, then you will get an idea what actually happens. You could start with the inventor of the sensor.

\n" }, { "Id": "7473", "CreationDate": "2018-11-22T22:23:25.560", "Body": "

Just received my second Ender 3 (see my other questions for the problems that first one developed). Figured it best to have a spare while I upgrade my first one with an ABL.

\n\n

I assembled it as normal, without any issues, but on applying power the LCD screen just flashes on and off continuously along with the blue LED on the main board. See photo.

\n\n

When I power it off, the Ender 3 logo appears briefly before it goes black. I am in the U.K. and running 240v setting. Is this likely the power supply or the motherboard? I already tried another screen and same problem.

\n\n

\"enter

\n", "Title": "Ender 3 Won\u2019t Start - flashing screen and blue LED on main board", "Tags": "|electronics|creality-ender-3|power-supply|", "Answer": "

Just after I posted this I tried swapping various components with my other Ender 3.

\n\n

It turns out a faulty power supply will produce this effect. When I swapped out the power supply it started normally.

\n" }, { "Id": "7477", "CreationDate": "2018-11-23T13:30:01.763", "Body": "

In the documentation of some commands of the Marlin firmware (like M112 - Emergency Stop), it says that the EMERGENCY_PARSER should be enabled to execute them instantaneously.

\n\n

The thing is, I didn't find any information there of how to enable that EMERGENCY_PARSER or how it works.

\n\n

Any help will be appreciated.

\n", "Title": "How to enable EMERGENCY_PARSER in Marlin firmware?", "Tags": "|marlin|", "Answer": "

The constant EMERGENCY_PARSER is located in the advanced printer configuration file Marlin/Configuration_adv.h:

\n\n
\n// Enable an emergency-command parser to intercept certain commands as they\n// enter the serial receive buffer, so they cannot be blocked.\n// Currently handles M108, M112, M410\n// Does not work on boards using AT90USB (USBCON) processors!\n//#define EMERGENCY_PARSER\n
\n\n

To enable the EMERGENCY_PARSER, you need to remove the // before #define EMERGENCY_PARSER and recompile the sources.

\n\n

Normally your printer will execute a command until it is ready to accept a next instruction. Without the EMERGENCY_PARSER set, the printer finishes the instruction that it is executing at the moment, if set, the execution is interrupted and immediately sent and thus not waiting for a clear space in the buffer.

\n" }, { "Id": "7484", "CreationDate": "2018-11-24T22:55:41.020", "Body": "

I have a small problem where plastic comes out of the nozzle while the printer is at a standstill (normally towards the end of heating the nozzle for a print), and whilst it moves from the line for clearing the nozzle on the left of the bed (Cura) before the actual print starts. This causes a slight problem where the first few millimetres of the printed line curls upwards when the nozzle comes back around again it goes over it but it causes a slight bump that makes a very small (but noticeable) skip or bump in the print on the bottom layer.

\n\n

I am using the Ender 3 running Marling 1.1.9 with a Bltouch and a glass bed, I didn't seem to have this problem before I upgraded to the glass bed and Marlin for the Bltouch.

\n\n

Any help will be greatly appreciated.

\n", "Title": "Ender 3 extrudes plastic whilst at standstill, and while moving to start of print", "Tags": "|marlin|extrusion|creality-ender-3|", "Answer": "

basics first

\n\n

The viscosity of plastic is temperature dependent: the warmer it is, the lower it gets and thus the more \"runny\". The lower the viscosity is, the less force is needed to move it.

\n\n

In printing, a pressure is applied to the filament from the extruder. Pressure is the force by area, thus for our look pretty much the same: the extruder exerts a force on the filament, to overcome the viscosity keeping it in the nozzle.

\n\n

A secondary effect is, that heated material expands, depending on what kind of material is in the nozzle.

\n\n

what happens

\n\n

The whole problem starts with shutting off the printer after the print: as the filament cools it shrinks. As the motors are turned off, the solidifying and shrinking plastic pulls at the filament. The filament can change its location or be pulled a little through the extruder, keeping the space quite well filled without cavities. Bowden style can change the mere filament path a little to compensate some of the shrinkings by shifting its path from hugging the outer wall to doing the same on the inner wall.

\n\n

As you start to heat up the printer, there is no force applied on the filament from the extruder to push it out of the nozzle. But when you shut it down, there was some filament in the nozzle.

\n\n

The filament melts and its viscosity drops, but at the same time, it expands. The extruder does not yet apply force, but as the material expands, it pushes against the filament stuck above it. Newton's 3rd law is the iconic Actioni contrariam semper et \u00e6qualem esse reactionem or as we know it short: Actio = Reactio, the force you exert in one direction equals a force applied in the reverse direction. Thus, the expanding filament pressing back against the extruder also exerts a force against itself down against the nozzle. The same is true for the nozzle, but the nozzle has one difference: it has a hole, where the forces are bundled to force filament out.

\n\n

At some point, the force from the expanding filament is big enough to overcome the viscosity keeping the filament in the nozzle and it oozes out.

\n\n

fixes

\n\n

There are several ways to fix this in slicing, but I prefer the end-code method.

\n\n\n\n

Example End Code

\n\n

Watch line 2. This is what prevents my Ender 3 from oozing in the first place

\n\n
G91 ;relative position set\nG1 F1800 E-3 ; Retract 3 mm to prevent oozing on startup\nG1 F3000 Z10 ; Move up 10 mm to clear the print\nG90 ;absolute position set\nG28 X0 Y0 ; home x and y axis to clear the print\nM106 S0 ; turn off part cooling fan\nM104 S0 ; turn off extruder\nM140 S0 ; turn off bed\nM84 ; disable motors\n
\n\n

Example Start Code addition

\n\n

This is just a snippet that forces retraction at the start, once the filament is hot. it WILL though make the first three millimeters of push come empty, thus should be combined with a cleaning that uses more than this - check out Writing G-code : swiping at start of print for better nozzle priming.

\n\n
G92 E3\nG1 E-3 F1000\nG92 E0\n
\n" }, { "Id": "7494", "CreationDate": "2018-11-26T11:23:30.187", "Body": "

I have a piece printed in white ABS, which is a half-scale replica of an unvarnished oak sculpture. After sanding off the worst of the layer lines, I'd like to paint it to get a nice wood finish. I believe that most hobby paints should stick fine, but how should I paint it to get a nice oaky appearance? Should I do layers of different colours? Will I get better results with spray or brush paints?

\n", "Title": "How to paint ABS to look like wood", "Tags": "|abs|post-processing|color|", "Answer": "

I found step-by-step instructions on Instructables specific to 3D-printed parts. In summary:

\n\n
    \n
  1. Sand and brush with acetone for a smooth finish (I guess the acetone vapour treatment would work as well.)
  2. \n
  3. Prime with plastic primer
  4. \n
  5. Spray-paint with a satin, nutmeg-coloured base coat
  6. \n
  7. Add grain with darker, acrylic paint using a fine brush
  8. \n
\n" }, { "Id": "7495", "CreationDate": "2018-11-26T11:47:41.900", "Body": "

This is my first printer, a core xy (Tronxy X5S) printer, and it is losing steps during diagonal movements. The curious thing its that fail is even, I've printed 2 calibration cubes and they are almost equals:

\n\n

\"enter

\n\n

When it lose the step, you can ear a \"toc\" sound, as in this footage at 3s: https://youtu.be/aRJB--QBMC4 This is my realy first print, and I don't have much experience with 3d printing, thanks in advance for any information!

\n", "Title": "Tronxy X5S losing steps during print", "Tags": "|belt|tronxy-x5|corexy|", "Answer": "

I had the same issue with my X5S when I first got it assembled. Because it's CoreXY, if one pulley slips, you'll skip steps in the diagonal. Check both set screws on your pulleys for the X and Y.

\n" }, { "Id": "7497", "CreationDate": "2018-11-26T19:29:51.323", "Body": "

I bought a few new nozzles expecting them to come with that little tube that comes out of the nozzle. They didn't come with them after all, so I tried to reuse the tube I originally had in the printer. Turns out my old tube is 4mm OD and 2mm ID, but the new nozzles have 2mm holes for the tube to go in. I use 1.75mm filament, so it seems like to be able to fit the filament through the tube would be impossible barring a tube with an 0.125mm wall.

\n\n

My question is, can I put the tube outside of the nozzle? That is, not stuck in the hole for the nozzle. In theory, the tube would still direct the filament into the right place. It looks like that might be the point of these new nozzles, since it seems so unlikely that someone would be able to stick a filament tube in the nozzle.

\n\n

If not, where can I find the tubes I need? I've looked in a few different places and I can't find it. Or are the nozzles useless, and should I return them? Thanks for the help.

\n", "Title": "Can the filament tube be outside of the nozzle?", "Tags": "|filament|nozzle|ptfe-tube|", "Answer": "

Yes, you can use a direct drive hotend with a bowden tube, but it won't just plug together. You just need a way to secure the end of the bowden tube to be centered above and as close the the hotend mouth as possible. In a pinch, you can spin a 4mm nut onto the tube and secure it down against the hotend mount with zipties, otherwise I would print a nice bracket.

\n\n

It may be more prone to jam on filament swaps than a proper bowden configured hotend but it will work fine in normal use.

\n\n

Edit, I was under the impression you were speaking of the bowden tube between the extruder and the hotend, not the liner inside of the hotend.

\n" }, { "Id": "7501", "CreationDate": "2018-11-26T23:38:43.527", "Body": "

I want to extend all my CR-10S wires. I have two long wire types: 22 and 18 AWG wires. I've done some research and found the following:

\n\n\n\n

Here is where I've problems determining which wire gauge to use:

\n\n\n\n

What's the wire gauge needed for the stepper motor and bed heater wires? Obviously, the bed heater needs more amp so I expect lower wire gauge. Is my 18 gauge wire enough for this?

\n\n

On the Reprap site, it says that 18 AWG or lower is fine for the heating bed.

\n", "Title": "Proper wire gauges for extending all CR-10S wires", "Tags": "|heated-bed|stepper-driver|creality-cr-10|wire-type|", "Answer": "

This answer expands on the heat bed wires question which is not addressed in this answer and gives a foundation for the edit of the question stating that AWG 18 or lower is sufficient.

\n
\n

The current depends on the voltage your machine uses. The question does not state whether you modified the power supply or not. Typical values for the resistance of the bed are in the range of 1.2 - 1.8 \u03a9 (Ohms). This implies a maximum current of $\\frac{12}{1.2}=10 A$ (the stock power supply is 12 V, if the power supply is changed to 24 V, the current doubles). From this reference you can look up the ampacity (maximum current through a wire) which wire gauge fits the current for your application depending on the maximum temperature you want it to reach. For 10 Amps, you will see that AWG 18 can be used for a single wire when you allow the wire to heat up to 60 \u00b0C. Note that bundling wires lowers the ampacity of a wire, because the cooling is less optimal for bundled wires (this is called de-rating).

\n

Please be careful when you read that certain wires are sufficient for your application. It is best to calculate the current your specific application draws from the power supply rather than relying on somebody else's calculation, it could well be that the assumptions are different than in your specific case. A multimeter or multitester is a valuable asset to aid in your calculations.

\n" }, { "Id": "7510", "CreationDate": "2018-11-27T19:34:27.540", "Body": "

I've seen comments about glow-in-the-dark filament being abrasive and would damage a bronze nozzle so a hardened nozzle is recommended.

\n\n

Is the same true for filament that changes colour when warm (for example when held in your hands)?

\n", "Title": "Does color changing PLA require a hardened nozzle?", "Tags": "|pla|nozzle|color|", "Answer": "

The color changing material used in the filament is most likely a Leuco dye. There are several types and some can be abrasive (just like the glow in the dark dyes); but, they aren't like putting sand through your nozzle. Since brass is really soft, even something like the dyes can cause it to wear; but, it will take a while.

\n\n

The safest bet is to use a hardened nozzle; but, if you are just doing a small print then it probably won't be too hard on the brass nozzle either. I have heard of people printing an entire spool of glow-in-the-dark PLA with a brass nozzle then examining the nozzle and not noticing any wear.

\n\n

Also, some people like the better thermal characteristics of the brass over the steel; so, they just stick with the brass nozzle for everything and replace it when needed.

\n" }, { "Id": "7519", "CreationDate": "2018-11-28T12:27:42.440", "Body": "

So I just got a Da-Vinci 3-in-1 Junior Pro 3D Printer, and was excited to start printing my first model.

\n\n

When I open my .STL File in the XYZWare that comes with the printer, and click print, it says that the cartridge inserted into my printer in not genuine, and that it won't print until I order a genuine cartridge. What is interesting is that the cartridge loaded into this printer came with the printer itself, so it is genuine.

\n\n

Is there something I am doing wrong? Here is a picture of the cartridge in my printer -->

\n\n

\"This

\n\n

A Google search about this issue doesn't come up with any results that are of any use to me.

\n", "Title": "Printing refusing to print, citing non-genuine cartridges?", "Tags": "|software|", "Answer": "

I managed to troubleshoot my printer's dilemma, rather quickly I might add.

\n\n

It turns out that the printer's firmware that was shipped with the printer was too old for XYZWare to use. My theory was that the firmware was trying to use a sensor that wasn't on the printer to begin with, and therefore was throwing out the issue of non-genuine cartridges.

\n\n

I updated the firmware on the Printer, and know it works just fine without any hitch!

\n" }, { "Id": "7524", "CreationDate": "2018-11-28T21:08:53.260", "Body": "

I thought as a fun project to make my own 3D printer out of a normal printer parts + some parts out of old CD-ROM drives that are lying around. The printer of my choice is an HP PSC 1315 one.\n\"HP

\n\n

But I have these questions:

\n\n
    \n
  1. Does this printer users stepper motors or is using a combination of DC ones and some sort of position sensor?

  2. \n
  3. What kind of electronics and firmware I can use for this type of builds?

  4. \n
\n", "Title": "3D printer out of normal printer (HP PSC 1315)", "Tags": "|diy-3d-printer|printer-building|e-waste|", "Answer": "

You need not a \"modern\" inkjet printer, but an antique flatbed 2-D printer. These were the state-of-the art in the 1980s, and drove the print head (a felt pen clamped into a mount) in X & Y over the printable area.
\nYou'd still need to hang the whole thing on some Z-drive, of course.

\n\n

See info at the HP museum, or buy one on eBay (Disclaimer: I just searched for that. I do not know the seller) .

\n" }, { "Id": "7527", "CreationDate": "2018-11-29T04:33:33.650", "Body": "

TL;DR: I'm looking to change the steps per mm but I'm not sure what to set it to. Do I decide on a temperature and change the rate based on the percentage for that temp? Or is there a more general setting I can put it to? Or is there a different underlying problem here?

\n\n

I'm running into an issue with my Ender 3 where when I print PLA at different temperatures I'm getting different extrusion rates. I've done the test where marking it at 120 mm and then extruding using PronterFace 100 mm and measuring the distance and this is my results. I did two tests of each to confirm results

\n\n\n\n

When I feed 100 mm through the Bowden tube without the filament going through the hot end I get exactly 20 mm left meaning:

\n\n\n\n

I have been having this problem since I started. I normally print at 210 \u00b0C at 50 mm/s with a flow rate of 103 % which seems to work well for most prints but on flat walls on prints it under extrudes in the same spot each time it comes around to that point.

\n\n

Steps I have taken so far:

\n\n\n\n

The reason I changed the Vref on the extruder stepper is because the system would click back when printing as if the pressure was too great in the tube. Changing this has helped some of the under extrusion a bit.

\n\n

So. depending on the temperature, it's harder or easier to push the filament by hand through the hot end. Maybe I need a better hot end?

\n\n

Examples:

\n\n

My test print to replicate the issue. The ripple bit is brittle and under extruded.

\n\n

\"Test

\n\n

\"Test

\n\n

This was a Prototype piece for an up-sized print I was planning. After going around it under extrudes at the same point each time 210 \u00b0C

\n\n

\"Prototype

\n\n

Most of the print is fine just some parts are different. This is an inner wall.

\n\n

\"Inner

\n\n

Heat towers

\n", "Title": "Different lengths of filament is extruded with different temperatures", "Tags": "|extrusion|hotend|creality-ender-3|stepper-driver|troubleshooting|", "Answer": "

So I feel that I solved the problem thanks to a few sources. Thanks to 0scar for his quick response and help.

\n\n

The problem was two fold. Mechanical problem causing blockage and slicer setting causing ripple.

\n\n
    \n
  1. The PTFE tube wasn't pushed all the way in as far as it could go causing too much pressure in the hotend. This seems to be a really common problem with stock Ender 3's Because of the gap the hotend needed to heat up into the heat break to melt the lump in the gap between the hot end and the PTFE tube.
  2. \n
  3. The slicing settings in cura had combing enabled. I set it to not comb on skin but what it was doing is combing just behind the skin and as a result would leak out filament in little blobs that when it printed the outer layer would be pushed out causing a ripple effect. This caused under extrusion once it finished the move because there wasnt enough filament left in the end.
  4. \n
\n\n

After I fixed these two things I was printing at 190c with no under extrusion, motor slipping, very little stringing and printing much better.

\n" }, { "Id": "7535", "CreationDate": "2018-11-29T18:34:58.660", "Body": "

I have designed this in Blender:

\n\n

\"Screenshot\"

\n\n

The bottom is a regular n-gon with 0.8mm in height. In Slic3r, I see that the bottom-most layer is not solid:

\n\n

\"Screenshot\"

\n\n

The green parts (of the first image) are going into that layer, because I wanted to ensure that I don't have air between the bottom and the green parts. Unfortunately they become printed non-solid / empty.

\n\n

This does not only take extra time for printing, it's also not what I want. What seems to be the bottom here is in fact the top of something and I want it to be a flat surface.

\n\n

How can I tell Slic3r to recognize overlapping items and generate them in a solid way?

\n\n

I have already set the infill to 100%, but that doesn't change anything. The Boolean modifier of Blender is also not very helpful. It creates topologies with gaps.

\n", "Title": "Why are my lowest layers not solid?", "Tags": "|slic3r|blender|", "Answer": "

I printed the thing for some layers to see what the printer actually does:\n\"Inner

\n\n

There are clearly too many retractions and unretractions.

\n\n

This can be seen in advance by letting Slic3r show the retractions and unretractions. \n\"Slic3r

\n\n

\"Green

\n\n

For the green \"walls\", it became clear that it prints them exactly onto the empty space, so in fact the seem to be floating.

\n\n

That made it obvious to me: the normals were pointing into the wrong direction. Therefore, Slic3r seems to have interpreted the material side of the green parts inside out. I had the same issue on some other parts as well.

\n\n

I went to Blender, selected the object(s), chose edit mode and found a function called \"Recalculate normals\". Since then, the bottom layer is solid.

\n\n

Also printing time is reduced by half an hour.

\n\n

\"Result

\n" }, { "Id": "7540", "CreationDate": "2018-11-29T23:39:32.877", "Body": "

I have encountered a problem where the print lines are not joining together, when I remove it from the plate it turns into a stringy mess.

\n\n

\"enter

\n\n

\"enter

\n\n

I am not sure what the problem is because I print out models that have successfully come out with properly joined lines previously, but now are failing. I am using the same nozzle and material that I have been using in past prints.

\n\n

I am using the Ender 3 with Bltouch running Marlin 1.1.9. The settings I use are as follows:

\n\n\n\n

EDIT

\n\n

I have included some higher quality photos as requested, between the original post and this edit I added an aluminium Bowden extruder and a new nozzle but the problem still persists. I have also re-tensioned the x-axis belt and re-flashed Marlin to the printer in case that had somehow corrupted. I have also tried increasing the flow rate in Cura but I had to increase to 130% for there to be any noticeable effect (i was able to print a slightly better model but it still has zero integrity). I also tried adjusting the z-axis height tuning but within a few steps up or down it would go from too close to the bed with not enough material coming out, to a decent height but with the original problem.\nI also cleaned out the hot end.

\n\n

\"enter\n\"enter\n\"enter\n\"enter\n\"enter

\n", "Title": "Print Lines Aren't Joining Together", "Tags": "|print-quality|extrusion|creality-ender-3|", "Answer": "

It is hard to tell from the quality of the picture you added, but this appears to be an example of either an incorrect height setting for your nozzle to touch trigger point (too large) or an under-extrusion problem that could be related to incorrect filament diameter setting, nozzle blockage, too high speed, slipping extruder gear, too less spring force on the extruder roller, etc.

\n\n

As far as can be seen from the second picture, the filament is not properly squished to the build plate, so I expect that you gain from setting a smaller nozzle to sensor trigger point distance. This can be done with the M851 command.

\n\n

Updated after posting high quality pictures:\nFrom the pictures you can see that there is an extrusion problem. Your first layer brim starts with too less material, but seems to make adjoining lines in the brim because it is squished a little too much (nozzle is too close to the build plate). It is advised to calibrate the extruder. The lower walls look fine, these are printed at low speed, the infill however looks very weak, this is probably because of a high printing speed and high temperature. In order to solve the problem you need to lower the speeds, lower the hot end temperature to 200 °C, increase part cooling flow and check and calibrate the whole extruder system/path.

\n" }, { "Id": "7555", "CreationDate": "2018-12-01T17:38:06.567", "Body": "

I made a few models in Autodesk Inventor. When I tried to get them printed, the printer created a model smaller than the model I made in Inventor (270mm x 200mm). Is there a way to keep them from getting resized?

\n\n

Here is the model in Inventor:\n\"The\nAnd here is the model being visualized by the printer driver:\n\"The

\n", "Title": "How do I keep the .stl files from being resized while exporting?", "Tags": "|cad|stl|", "Answer": "

.stl Basics

\n

The .stl format has no inherent sense of which units you use. items are to scale to an ambiguous 1, which could be 1 meter, one millimeter, one lightyear or one inch. To a .stl, only the relative sizing matters. All these faces you see are compared to a line with the length of 1-unit that is

\n

Slicer-Modeling Software interaction

\n

The most common graphic design programs export in millimeters, but some US ones just assume inches, which is a factor of 1"=25.4mm.

\n

Cura, Netfabb, and Slic3r expect that the 1-unit line is one millimeter long - but if it is an inch instead, then the model is shrunk by 1/25.4 or to about 4% of the right size. Scaling up by 2540% one would return to the millimeter scale.

\n

But then there are other programs that use other choices of scales. Blender for example assumes a scene is in meters by default.

\n

Inventor

\n

Inventor can export .stl in a variety of scales, which all just serve as how the length of the inherent but invisible 1-unit line is drawn. The default choice is centimeters, so a scaling factor of 1cm=10mm, which would explain the models being only 1/10th of the expected size in Slic3r. to change the scaling, follow the manual:

\n\n" }, { "Id": "7557", "CreationDate": "2018-12-01T19:54:54.297", "Body": "

When printing a part today on my Prusa i3 MK3, I was prompted to upgrade the firmware. I went to the website and downloaded the firmware as a .hex file.

\n\n

When reading the attached firmware upgrade instructions, it seems there's only one way: use Slic3r and connect the printer via USB. These instructions are also available online.

\n\n

However, my printer is too far away from my PC and I'm printing from SD card only. Can I upgrade from SD card as well?

\n\n

Hopefully next week I'll get the Raspberry Pi Zero W so I can use Octoprint for printing. Does that combination support upgrading the firmware?

\n\n

I have tried:

\n\n\n", "Title": "Upgrade firmware without Slic3r and USB connection?", "Tags": "|prusa-i3|firmware|", "Answer": "

No, you cannot update the firmware from an SD card.

\n\n

Yes you can update from a Raspberry Pi (RPi). A RPi runs a Linux distribution, when you install the correct tools you should be able to update through USB using the RPi.

\n\n

For simplicity and time it would be an idea to lend a laptop for a few minutes.

\n" }, { "Id": "7580", "CreationDate": "2018-12-05T09:18:17.647", "Body": "

First post here, so please forgive me for any silly mistakes.

\n\n

Recently the y-axis (I think, forward-backward) has started skipping forward when printing the first layer of my print. It prints the raft with no issues at all, sticks the bed nice and flat and the raft is perfect every time.

\n\n

Then it starts to print the actual item, but every time it starts to the print the back right corner the Y-Axis jumps or skips on the gears and makes a loud grinding noise. The amount it jumps varies on the print, but always jumps towards the front of the print. It doesn't matter on the size of the print or the position, it always jumps forward never back.

\n\n

The printer is a Makerbot Replikator clone, from eBay. The X & Y moves the extruder. I have tried upgrading and downgrading the software (which uses Makerbot software) I am unsure if I can update the firmware.

\n\n

Any help much appreciated.

\n\n

UPDATE:\nImage has been attached of the failed part, excuse the colour, I use it for my test prints. This is orientation it printed on the bed. First 3 blocks printed fine, 4th one jumped It printed in the following order 1. top-left, 2. bottom left, 3. bottom right, 4. top right.

\n\n

\"Failed

\n", "Title": "Layer shifting on product printing after raft support structure", "Tags": "|troubleshooting|y-axis|", "Answer": "

I'm guessing that the bed in the offending corner is farther away and your raft is curling and lifting off the bed in that corner, and your nozzle it hitting it. Try releveling your bed, paying particular close attention to that corner. In addition, work on bed adhesion in general. Clean the bed, make sure fan is off on first layer, and go slow on first layer, and use proper bed temp for the material. You can also try glue stick.

\n" }, { "Id": "7582", "CreationDate": "2018-12-05T22:59:22.830", "Body": "

I've built a DIY 3D printer similar to Prusa i3. It does homing and bed leveling very well with a PNP Lj12a3_4_Z_BY sensor using G28 and G29.

\n\n

\"Lj12a3_4_Z_BY\"

\n\n

However, when I push the print button on Pronterface or Simplify3D, Z axis goes up about 20mm and starts printing in air. I can't figure out what is going wrong.

\n", "Title": "Why does my DIY printer start printing in air?", "Tags": "|z-axis|bed-leveling|simplify3d|z-probe|pronterface|", "Answer": "

If the cad model was imported from Solidworks into Simplify3D. Push \"Center and Arrange\" button to place the model in the middle of the Heat Bed. There might be a gap between the object and bed level that is hard to see.

\n" }, { "Id": "7591", "CreationDate": "2018-12-07T02:57:07.357", "Body": "

I recently bought my first 3d printer, a monoprice maker select v2.

\n\n

After a couple of print jobs, this piece fell out.

\n\n

\"Printer

\n\n

I have no idea where it came from.

\n\n

But afterwards, I notice that my Z-positioning is all screwed up.

\n\n

When I select \"Home All\" to reset all positions to do bed leveling, the extruder tries to push down pass the build-platform, puncturing a hole into the plastic bed cover.\nThere was a lot of clicking from the motor so I immediately turned it off.

\n\n

I turned it back on and then tried to move the z-position back up. But it moves only about an inch up before the motor starts clicking again.

\n\n

I was reading some other posts about a \"Z-axis limiter switch\". Could that be what the little metal fork-like piece is? \nWhere does it go and how do I put it back?\nAnd how do I reset my z-position?

\n\n

Any help appreciated.

\n", "Title": "What is this part and does it have to do with my my z-position is messed up?", "Tags": "|z-axis|endstop|monoprice-maker-select|", "Answer": "

Limiter Switch/Endstop

\n

That is the lever that is meant to trigger your Z-Endstop. The variant used in your case is on the cheaper end and is meant to look similar to this QIAOH KN12-1 limit switch:

\n

\"a

\n

You will either exchange the endstop as a whole, somehow reattach the/a lever or change the physical position of the endstop so that the moving X-axis is triggering it (again).

\n

Replacement & Wiring advice

\n

Pretty much any switch that can have both an NC/NO (normally closed/normally open) state will function in its state, but it is usually a good idea to have all safety switches (like limit switches) be NC types and have the firmware detect the loss of signal as a safety feature. In case of a broken cable, this will automatically force the machine to halt, instead of keeping the machine moving despite having reached the maximum position.

\n

Normally Open/Closed

\n

It is even possible to wire up a number of redundant switches into one circuit. For NC setup, all the switches need to go in series, so that any switch getting toggled cuts the signal at that point. On the other hand, if only NO switches are used, they have to be wired in parallel: the signal passes when any number of switches is closed.

\n

NO switches, because they are parallel, do cost us in the security department, as a broken lead does not halt the machine's use, but they are easier to troubleshoot as a broken component only takes out its specific arm of the parallel circuit. On the other hand, the NC-series does give us the broken lead security, but a single broken component that stays switched to open disables the whole series and identifying the broken component can take more time.

\n

NOT-Gates

\n

As you can imagine, some clever electronics allow us to use us to turn an NO into an NC and vice versa. This clever piece is a NOT-gate, sometimes also called inverter. They take a logic signal and a supply voltage lead, and only allow the supply voltage to pass into their output if there is no signal in their input side. So, they allow using a NO switch as an NC switch or vice versa. In any case, the NOT-gate needs to be on the output side the switch(es) it shall invert as it only inverts the signal that happens before it in the circuit.

\n

With this clever piece, one can use a combination of NO and NC switches, if one so desires: The NO and NC parts are to be wired separately in groups (the NC group in series, the NO group(s) in parallel). To combine the two signals, there are two ways: Either run the NO output through the NOT-gate into the NC circuit's input, putting it into the series (and making it technically function as a single NC-element), or put the NC group output signal into the NOT-gate and put both groups in parallel. In either case, the NOT-gate 'flips' the function of the switches to behave as if they were the other type: from NC to NO and vice versa.

\n" }, { "Id": "7597", "CreationDate": "2018-12-07T13:02:39.453", "Body": "

A while ago bought a Bowden extruder to replace the old built-in one (It was malfunctioning or something that I cannot remember well). This one in particular has been acquired:

\n\n

\"The

\n\n

The thing is, that almost a year has passed since the printer did something rather than getting jammed and not work properly. The extruder itself seems to work properly, when I heats up and push the filament by hand seems to pass properly.

\n\n

When assembled and start to print, the printer at first seems to work, but after a minute or so, it stops extruding (gets jammed or something).

\n\n

The last time that I removed the filament and I've found out that it was coiled inside (Very strange):

\n\n

\"Coiled

\n\n

My first thought was that the filament was thinner than the extruder's specs, but the seller says that is for 1.75mm, and all my filaments have that diameter.

\n\n

What I am missing? Something maybe that is not properly mounted?

\n\n

The product is not official, bought on Amazon (I didn't know about e3d v6).

\n\n

Printing temperature should be 180\u00baC but I've seen now that Cura sets the first layers to print at 200\u00baC. Material: PLA

\n\n

My printer is a BQ Prusa i3 Hephestos.

\n", "Title": "Filament gets coiled inside the extruder", "Tags": "|filament|extrusion|hotend|bowden|", "Answer": "

If you converted a printer from direct to Bowden feed, it is possible that the cold side of the hot end is too hot, and you are suffering from heat creap. Simple, less expensive printers often use the extruder stepper and the extruder stepper mount as part of the cooling system.

\n\n

If there is any sort of a heat-sink on the cold side of the hot-end, try to rig a fan to blow through the heat sink. If there isn't a heat sink, find one and add it. Heat sinks made for round transistor cases can sometime be fitted to the cold side. A little heat sink great, a couple of transistor heat sinks, and a fan may completely solve your problem.

\n" }, { "Id": "7602", "CreationDate": "2018-12-09T01:44:17.990", "Body": "

I have a thicker glass plate on the bed. Also, I have a z-axis offset at the maximum value 50 yet the nozzle does not clear the plate.

\n\n

Why can't I adjust the axis beyond 50?

\n", "Title": "Ender 3 Nozzle does not clear bed", "Tags": "|z-axis|creality-ender-3|", "Answer": "

There are several ways to get the Z0 into a position the sotware can account for it without needing to write your own firmware if needed:

\n\n\n" }, { "Id": "7606", "CreationDate": "2018-12-09T10:56:26.190", "Body": "

I keep hearing about vase mode, but I have some problem figuring out how to set it up in my slicer; I use Cura slicer.

\n

So, I'd like to get some settings to do it in Ultimaker Cura, using PLA if the material is important, plus any advice on how to properly do it and ensure that the print retains it's shape on print.

\n", "Title": "Settings for vase mode in Cura?", "Tags": "|ultimaker-cura|software|", "Answer": "

Option Spiralize makes your model one layer thick on outer edges.

\n

What I do for a vase is set the top layer count to zero.

\n" }, { "Id": "7609", "CreationDate": "2018-12-09T23:52:58.267", "Body": "

I've successfully flashed Marlin 1.1.9 onto a Melzi v3.5 board (the stock board for my Monoprice Maker Select v2 (v2.1?, a white-labelled Wanhao i3 Duplicator).

\n\n

Initially after the flash succeeded the LCD displayed was \"garbled\". The right side, in particular, had lots of pixels out of place. I was able to resolve this by experimenting with some delay variables. Prior to the flash the LCD was 100 % working; to my knowledge no damage happened while I had the machine apart.

\n\n

Here's my changes to Configuration.h from Marlin 1.1.9, excluding anything to do with X/Y/Z/E, etc (irrelevant stuff).

\n\n
#define MOTHERBOARD BOARD_MELZI\n#define SDSUPPORT\n#define REVERSE_ENCODER_DIRECTION\n\n//\n// LCD for Melzi Card with Graphical LCD\n//\n#define LCD_FOR_MELZI\n\n// Increase delays to fix garbled LCD\n#define ST7920_DELAY_1 DELAY_NS(0)\n#define ST7920_DELAY_2 DELAY_NS(100)\n#define ST7920_DELAY_3 DELAY_NS(200)\n
\n\n

The display is much better, however there are about 4x rows of pixels through center of the display running left to right that mostly don't display. There are little sections--perhaps 20x pixels (maybe 3%) that do appear to be displaying.

\n\n

Here's a photo of the problem:\n\"Marlin

\n\n

Are there any other typical culprits? Based on what I've read it sounds like this is a recent(ish) issue with Marlin, and perhaps would be solved with an older version or a different firmware.

\n\n

Here's a photo, if you look closely you can see the 'dead' rows, with about 6x pixels work

\n\n

PS. While troubleshooting I added shielding to the LCD's ribbon cable (foil wrapping the cable, grounded, and wrapped with electrical tape). It didn't help, but I left it on.

\n", "Title": "\"Missing\" rows on 64x128 LCD after flashing Marlin 1.1.9 onto Monoprice Maker Select v2", "Tags": "|marlin|wanhao|monoprice-maker-select|", "Answer": "

Possibly unrelated to the firmware? This happened to me when I tightened the mount screws on the LCD, after I backed them off a bit it was fine.

\n" }, { "Id": "7634", "CreationDate": "2018-12-13T16:49:09.920", "Body": "

I'm using a Monoprice Maker Select v2 (a Monoprice-branded Wanhao i3 duplicator) and would like to calibrate temperature readings for my hotend.

\n\n

I used a thermal probe on my multimeter to determine the hotend is running about 5 degrees C above it's reported temperature (setting it to 200 degrees reads 205 on the multimeter). I measured this by placing the thermal problem partly into the hole where the printers thermistor resides.

\n\n

I use Marlin 1.9 for firmware and ideally would calibrate it there--I'd prefer the readouts to be accurate rather than relying on offsets to compensate. What's the process for recalibrating the hotend temperature readings?

\n", "Title": "How do I calibrate the temperature of my hotend?", "Tags": "|calibration|hotend|wanhao|monoprice-maker-select|", "Answer": "

Temperature from thermosensors gets collected as a resistance value that changes with temperature. The chip in your board decides the temperature from this value based on a temperature-resistance table.

\n\n

If you are using Marlin Firmware, the setting which table is referenced by your machine to get its values is written under the header Thermal Settings inside Confinguration.h.

\n" }, { "Id": "7636", "CreationDate": "2018-12-13T20:59:38.797", "Body": "

I am attempting to follow these instructions to calibrate the extruder, however when I increase the value for DEFAULT_AXIS_STEPS_PER_UNIT the change in physical output does not correspond with the changes.

\n\n

I started by testing how much filament goes through the extruder when sending G1 E100 F100 to my printer, instead of the 100 mm I get about 38 mm (#define DEFAULT_AXIS_STEPS_PER_UNIT { 80, 80, 400, 93 } in Marlin). I increased the value to DEFAULT_AXIS_STEPS_PER_UNIT { 80, 80, 400, 149.73 } which should take it to 100 mm, when I tested the physical output the increase was only negligible so I repeated the calculation with DEFAULT_AXIS_STEPS_PER_UNIT { 80, 80, 400, 387.8007 } as the new value. The new physical output is about 64 mm, I think this suggests a mechanical problem rather than a firmware configuration problem, but I am not sure where to go from here.

\n\n

My printer is the Ender 3, running Marlin 1.1.9 with a BlTouch and an aluminium bowden extruder mod \n\"extruder\"\n(installed after the extrusion problem began).

\n\n

Any help you can give will be greatly appreciated.

\n", "Title": "When I attempt to calibrate extruder steps the increased values don't correspond to physical increases", "Tags": "|extruder|extrusion|creality-ender-3|", "Answer": "

Mechanical?

\n\n

You basically answer your own question that there could be a mechanical issue.

\n\n

If 93 steps gives you 38 mm, to get to 100 mm, you need to divide 100 by 38 = 2.63 times. This implies that you need 2.63 x 93 = 244.7 steps/mm (not 149.73).

\n\n

You even tried close to 400 steps per mm to see you are not extruding 100 mm of filament. This implies that you are either missing a lot of steps (increasing the stepper current or increasing the temperature of the hotend would help out) or the extruder gear slips on the filament (increasing the pressure on the extruder gear and filament could solve this).

\n\n

Or not mechanical?

\n\n

Furthermore, it could be another issue than mechanical. If your extrusion rate is too fast, the stepper might not be able to cope the high speed and miss steps. I have had this before with a too high of a retraction speed.

\n" }, { "Id": "7642", "CreationDate": "2018-12-14T12:17:54.893", "Body": "

I have this sketch which I have accidantly rotated by some arbitary angle.

\n\n

I need the dotted line to be parallel with the x-axis and the small, left hand side to be flush with the y-axis.

\n\n

I've tried using the Align function, but can't seem to get htat to work.

\n\n

Ive tried rotating the sketch, but I can't see a way to measure the angle it's off by in order to specify how far to rotate it.

\n\n

I'm very new to Fusion 360, and CAD in general so it's probably something really simple.

\n\n

\"Sketch

\n", "Title": "How to align sketch with the x-axis", "Tags": "|fusion360|", "Answer": "

It's rather simple:

\n\n
    \n
  1. Start to edit the sketch with this line via a right-click edit sketch
  2. \n
  3. click the line you want to force horizontal
  4. \n
  5. on the right menu, the lower menu has \"align horitontal/vertical\", click it
  6. \n
  7. DONE
  8. \n
\n\n

To find the button I mean, a picture with the right tool next to the red mark:

\n\n

\"The

\n\n

Hint: in the case of your item, I suggest to use the blue marked symmetry on the two lines you forced to be equal:

\n\n
    \n
  1. first choose the lines (SHIFT)
  2. \n
  3. choose the tool
  4. \n
  5. click the symmetry line (in your case: the construction line)
  6. \n
\n" }, { "Id": "7645", "CreationDate": "2018-12-14T14:52:34.647", "Body": "

A couple of prints after leveling the bed (<10), I noticed some weird sound, and saw the nozzle scratching the build-plate. \nWhat is really weird, is that I noticed (My printer is in quite a dark edge, so the build plate isn't really illuminated), that there were some scratches from a print before, but it worked fine with the prints in between.\nSo my question is, how that might have happened, could it be also software issues (Maybe some mistakes converting to gcode?), or is this propably purely a mechanical issue?\nFurthermore, I'd like to ask whether this may cause any problems, e.g. massively reducing print quality, or weakening the grip of the prints.

\n\n

I'm using the Tevo Michelangelo 3D, with manual leveling (it drives to the positions, but one has to tighten/loose the screws).\nNote that I might have overtightened the screws in the beginning. Another thing I noticed is that the screws on one side tend to be more loose than the other ones.

\n\n

\"Build

\n", "Title": "Scratches in the build plate - How did this happen, and may it cause problems?", "Tags": "|nozzle|bed-leveling|build-plate|", "Answer": "

As the question continued in comments, it can be read that the build plate had over-tight screws and loose screws on the build plate. This uneven tension could lead to stresses that warp the bed, e.g. lower corner and bulging out in the middle of the bed. When levelling on the corners, the nozzle could hit the bulge in the middle when printing. Furthermore, an endstop may have shifted during operation.

\n" }, { "Id": "7650", "CreationDate": "2018-12-14T18:15:41.023", "Body": "

I have a FlashForge Creator Pro. It is equipped with the board revision V SF7.7.

\n\n

My Right extruder prints great, however, the left does not work at all. I have tried to diagnose the cause, but at this point, I feel all the mechanicals are working and it's coming down to logic boards. I don't know what and how to test these. The main problem is like this:

\n\n
    \n
  1. When I order to Load/Unload Filament the motor does not advance
  2. \n
  3. When printing from the left extruder, filament does not come out.
  4. \n
\n\n

I have done the following tests:

\n\n
    \n
  1. The hotend works and the nozzle is clear. I can push filament through when it is hot.
  2. \n
  3. Teeth are in great shape, unclogged and perfectly aligned with the \"V\" on the guide wheel.
  4. \n
  5. The Stepper motor is functional. I can perform the LED test on both pairs of leads and 3V LED lights are up when the motor is turned.
  6. \n
  7. The Stepper cable is OK. I tested continuity on all 4 wires and they are fine.
  8. \n
  9. The Connectors are fine and well-seated.
  10. \n
  11. Voltage to Stepper Driver board: 5.0V when the machine is powered ON
  12. \n
\n\n

I have however encountered these tests that seemed to show a fail:

\n\n
    \n
  1. When I set the left extruder to \"Unload Filament\" the voltage across both pairs of leads is 0V
  2. \n
\n\n

How can I continue troubleshooting and fixing the machine?

\n", "Title": "Extruder_2 Stepper Motor does not work", "Tags": "|troubleshooting|stepper-driver|flashforge-creator|", "Answer": "

The next logical step is to swap the working board with the one that powers the misbehaving motor. If this fixes the motor but makes the other one stop working, it hints that the stepper driver board might be broken. If it does not fix the motor but the right motor works again, while the left is not, then the motors or the board controlling the stepper drivers might be broken.

\n\n

As changing the driver boards did (as OP said) swap the problem from the left to the right extruder, it is extremely likely that the board that was originally left is defective in some way.

\n\n

Getting a replacement part might be in order, and until then, running with a single extruder.

\n" }, { "Id": "7651", "CreationDate": "2018-12-14T18:19:40.210", "Body": "

I have a bunch of ball bearings (608 2RS) that I'm planning on using in a project with printed parts. But I'm not sure how to attach and secure them properly.

\n\n

How is this done usually?

\n", "Title": "How to attach a metal bearing to a print", "Tags": "|3d-design|post-processing|", "Answer": "

Ball bearings are usually fitted in one style, no matter what is the material that they are embedded in: press fit.

\n

Press fit

\n

To get a good press fit, the part will have to have a hole that is the diameter of the bearing (22\u00a0mm) plus a little margin that depends on your printer and filament. From my own experience, a design with 22.1\u00a0mm to 22.4\u00a0mm diameter resulted in a near-enough 22mm hole that gave decent fit. Your design will need to be adjusted depending on the material and resolution of the printer.

\n

Note that in the case of machined metal parts, the hole usually is not exactly 22mm but a coupe thau (=1/1000 inch) smaller as the bearings get pressed into the hole with a hydraulic press. They do deliberately deform the bearing and workpiece a tiny bit to sit perfectly. If you make the hole too much too small, the printed parts could break under the stress such treatment puts them under. However, if done just right the ring itself will deform just enough to fit the bearing's casing, as long as it is flexible enough. I have experienced prints of 2 wall thickness with 15-20% infill to allow a little bit of flex, which resulted in them applying quite some tension on a set of three screws: the Slider for 2040 Openbuild V-slot by FabianFriethjoph does use this effect from PLA to force the wheels into the guide rail just enough to prevent wobble.

\n

Since most filaments shrink, you might still need to use a larger-than 22\u00a0mm hole in design to get just a couple thau under the 22\u00a0mm you want for the perfect press fit.

\n

Adjustable position

\n

In some cases, ball bearings are fitted into larger holes, and then fastened and adjusted with a set of 3 setscrews - their seat can be changed slightly to compensate for warp or shear of the whole item. However, printed plastics are bad at holding a thread, and it would be a very good idea to include a metal insert with the thread. For example, you could include a nut in the middle of the ring holding the bearing, or you might use metal inserts.

\n

Capping

\n

Even if one uses a snug press fit or adjustable position, it can be a good idea to use a cap that makes sure the bearing can't fall out of its area without removing the cap first.

\n" }, { "Id": "7661", "CreationDate": "2018-12-15T15:47:26.457", "Body": "

I have a Monoprice Maker Ultimate 3D printer (a rebranded Wanhao Duplicator 6) and out of the box the hotend temps when set to 200 would vary between 190 and 210. I used repetier-host to run PID tuning and stored the last set of values it gave when it finished, however I'm still seeing around 5 degree temp swings.

\n\n

Here's a screenshot of the temp during a Benchy print from Octoprint:

\n\n

\"Temperature

\n\n

I tried PID tuning again, which gave different values, but basically the same results. Is there anything I can do to improve this? The Maker Select Plus I had only seemed to vary +/- 1 degree so I'm assuming this is more than expected. (I don't actually know what the implications are, so I don't know whether it is/will affect my print quality).

\n\n

Is there another way of tuning (I don't fancy manually tweaking numbers and testing for hours), or if I do more than 8 cycles (the default on the Wiki page linked above) would that improve the accuracy?

\n", "Title": "How to reduce +/- 5 degree temp swings of hotend (after PID tuning)", "Tags": "|calibration|hotend|", "Answer": "

If you are sure that PID is enabled (and not using bang-bang heating, as this gives a very similar heating profile) and did not fix the fluctuations, the best guess for this phenomenon is that the printer board has incorrect capacitors installed on the printer board. This is not very uncommon and can be found on the internet. You should at least show the graph to the vendor and ask for support.

\n\n

If incorrect or faulty capacitors are the cause it requires desoldering, and resoldering new ones. If this is out of your comfort zone, it is better to steer your communication with the vendor to replacing the board.

\n" }, { "Id": "7667", "CreationDate": "2018-12-16T11:54:45.253", "Body": "

I'm setting up my first 3D Printer, a Prusa i3 MK2 clone. \nWhen I try to heat the bed, nothing happens. Then Marlin throws this error after a while:

\n\n
Bed Heating Failed\nPRINTER HALTED\nPlease reset\n
\n\n

In Pronterface I see this in the console:

\n\n
Error:Heating failed, system stopped! Heater_ID: bed\n[ERROR] Error:Heating failed, system stopped! Heater_ID: bed\n\nError:Printer halted. kill() called!\n[ERROR] Error:Printer halted. kill() called!\n
\n\n

There is 0 voltage across D8 on the Ramps board, where it is connected. When I heat up the hotend, on the other hand, which is connected to D10, I see 12V across the board's D10 terminals.

\n\n

The bed is MK42 type.

\n\n

My axes, extruder hotend and fans are working OK.

\n\n

I previously burned out the 5 V regulator on my Arduino Mega by misconnecting one of the end stops. I replaced the regulator on the board and the 5 V supply seems to be working OK again. However I wondered if I might have damaged the board in some other respect? Or is there a config setting in Marlin I should change?

\n", "Title": "RAMPS board - no power to heatbed", "Tags": "|prusa-i3|heated-bed|ramps-1.4|troubleshooting|", "Answer": "

The problem was that one of the pins on the Ramps board power supply header was not soldered. The manufacturer had omitted to solder one of the pins, so there was no power to D8. Fixed by soldering the pin in place.

\n" }, { "Id": "7671", "CreationDate": "2018-12-16T21:27:51.130", "Body": "

I recently got myself a Prusa i3, that I needed to assemble and adjust myself. As expected, the first prints was of quite poor quality. After better adjustment, I improved a bit the print, but I am not quite there yet. I was hoping to get some advice on how to improve the print based on few pictures of the 3DBenchy boat I printed.

\n\n

The most annoying point, on the following image, is the dent that I have in the hull, on the front of the boat. And one layer appears to be missing or close. And I have another one just like it in the top cabin.\nI also think that my specific settings on the first layers shouldn't be there, because they don't improve anything, but that is another topic.

\n\n

\"3DBenchy

\n\n

Also, when I look on the top, I have a lot of filament strings getting here and there, the path of the tip of the printer is quite visible on the roof and floor, the steering wheel is not to clear, etc.\n\"enter

\n\n

Also, at the end of the boat, I am supposed to have some letters, but I cannot read them because of the poor definition of the print...

\n\n

How can I improve the quality of the print? What settings should be looked at?

\n\n

The material used for printing is PLA. The hotend temperature was set to 215 °C for the first layer, 210 °C for the rest. The bed at 65 \u00b0C for the first, then 60 \u00b0C.\nThe setting in Slic3r for the fan is between 35% to 100%. If I understand the rule correctly, under 5 sec for a layer, it is 100%, then it decreases proportionally up to 35% if the layer is done in 60 sec. And off otherwise.

\n\n

The diameter of the filament is set to 1.75, with a nozzle of 0.4 mm. I wonder if I should put it to 0.375 mm.

\n\n

I otherwise think that it is a genuine prusa, but clearly not from the first iteration. I don't have an arduino on it, but the makerbase chip. (That comes from a kit I bought online.) I would wait to be a bit more confident before attempting any modifications.

\n\n

The speed for perimeters is 60 mm/s, 30 mm/s for external perimeters. Infill 80 mm/s. Bridges 60 mm/s. Support material 60 mm/s. Non print move: 130 mm/s. There are a couple of other print speed, but I don't think they are that relevant for the case.

\n\n

After modifications of temperature and cooling, following the answers, I got the following print:\n(Nozzle 200 \u00b0C, fan at 50 %)\n\"New\n\"Burned

\n\n

Weirdly, it was like the PLA got burned here and there, which was not the case at hotter temperature... But at least, the edges are sharper, and the writings are easier to read! I will try to lower a bit more the temperature, but I would need to fight a bit with my extruder motor, which seems to have hard time pushing the PLA when the temperature is too low.

\n\n

I tested at an even lower temperature: 190 \u00b0C. The result is not pretty, the layer were not sticking to each other. And even at 195 \u00b0C, some layer don't attach.\n\"Low

\n\n

I tried again at 200 \u00b0C, but with lower peripheral printing speed. It did help! I can feel the progress.\n\"enter

\n\n

However, I still have a small dent. And on the following picture, we can clearly see that there are sometimes molten half-burned PLA dripping. And there are still strings remaining between the places.\n\"Molten

\n\n

The nozzle was cleaned just before this print.

\n", "Title": "Calibration improvement of the Prusa i3", "Tags": "|print-quality|calibration|", "Answer": "

Your hotend temperature is too high and/or too less part cooling. The part cooling is very important to solidify the hot fluid filament in time to have a solid fundament for the next layer. PLA has a reasonably low glass temperature (at this point the filament is weak and mealable/flexible, at about 60 °C), if the part is not cooled properly, the part temperature can be over the glass temperature when printing the next layer and will distort the previous and current layer.

\n\n

I print PLA at a maximum temperature of 200 °C (for my thermistor reading).

\n\n

A combination or a single of these parameters not being the correct value will cause the dent at the front and the stringing and letters to be faded as the filament is not cooled properly and deforms the previous and current layer, this easily shows up in overhangs like at the bow of your print.

\n\n

Try to lower the hotend temperature by 5 °C per next calibration print (or start at 200 °C and work down from there) and increase the part cooling a little if possible (35 % to 50 %). The build plate generally does not need to be 5 °C higher for the first layer nor do you need an extra 5 °C for the hotend, PLA is not that difficult to print.

\n" }, { "Id": "7674", "CreationDate": "2018-12-17T09:31:20.753", "Body": "

I plugged a new fan into the (TriGorilla v1.4) printer board Fan0 which I found out to be broken.

\n\n

I'm trying to upgrade the part cooling fan for a bigger one. Unfortunately after plugging in the old one, I realize that the fan is constantly \"ON\". On other forum someone told me that MOSFET might be shorted from plugging in the broken fan.

\n\n

This triggered my question: \"Is there any possible way to change the firmware FAN0 to use FAN1 socket?\" and: \"Would that work for part cooling model fan?\"

\n\n

\"Motherboard\"

\n", "Title": "Changing FAN0 to FAN1 in TriGorilla printer board", "Tags": "|marlin|hotend|delta|cooling|", "Answer": "

Yes you can use another fan port, this requires some editing of the configuration files of the firmware and reflash the firmware.

\n\n

If you look at the documentation of the board and an overview of the board layout, you will see that there are 3 PWM controlled FAN ports. The answer is therefore yes, you can use another port (e.g. FAN1 or FAN2) to be used for your broken FAN0 port. The most easy solution is to swap the port numbers in the pins configuration of your firmware, so swap pins 'D9' and 'D7'.

\n\n

\"Trigorilla

\n\n

How you edit the configuration is depending on the version of the TriGorilla board you have, there is a version 1.3 and a version 1.4.

\n\n

Basically you need to define the software FAN (0) to be reached at the hardware location Fan1 using pin 7:

\n\n
#define FAN_PIN        7\n
\n\n

Subsequently, FAN1 in software needs to points to Fan0 in hardware

\n\n
#define FAN1_PIN        9\n
\n\n

If you changed this, the software will think that the Fan1 port on the board is the software FAN.

\n\n

If you are using the v1.4 TriGorilla printer board, the v1,4 specific pin definition is found in pins_TRIGORILLA_14.h. If you look into this file you will see that this is basically a RAMPS board:

\n\n
#define IS_RAMPS_EFB\n...\n#include \"pins_RAMPS.h\"\n
\n\n

So editing for you need to redirect the pins 7 and 9, but you only have a single active fan (as per #define IS_RAMPS_EFB)! Luckily we can forget about the second fan as you want to replace the first by the second.

\n\n

In pins_RAMPS.h you will find:

\n\n
#ifndef RAMPS_D9_PIN\n  #define RAMPS_D9_PIN      9\n#endif\n
\n\n

Note that it is wise not to edit this latter file, instead edit your copy of pins_TRIGORILLA_14.h to include assigning of the pin 7 to the Fan1 header as FAN in the firmware.

\n\n

If you look closely at the assigning of the RAMPS_D9_PIN pin, it says: #ifndef; so if it has been assigned previously, do not override the value.

\n\n

Now edit the pins_TRIGORILLA_14.h file to include:

\n\n
// Comment out this line:\n//#define FAN_PIN 9\n// And enter this beneath that line\n#define RAMPS_D9_PIN 7\n
\n\n

or

\n\n
#define FAN_PIN 7\n// this bypasses setting of the RAMPS_D9_PIN constant\n
\n\n

From this point on, if the software addresses scheduling of the FAN port, the pin 7 schedules the MOSFET attached to the Fan1 header.

\n\n
\n\n

Basically, the above procedure describes how one directs hardware ports by changing the addressing in the firmware configuration.

\n\n

If this does not work, there might be more things broken on your board.

\n" }, { "Id": "7676", "CreationDate": "2018-12-17T10:52:57.067", "Body": "

Where is the correct Marlin firmware file and location to add code that I want to shop up in the LCD menu of my printer, and then execute the function I write when the button is pressed?

\n\n

For example I want to add a menu item that says \"Preheat Custom\" that is in the same menu as \"preheat PLA\" and \"preheat ABS\" and then runs code to heat to values I specify.

\n\n

I'm running Marlin Firmware version 1.1.9 on a Creality Ender 3.

\n", "Title": "How to add menu options to the (Marlin firmware) LCD menu?", "Tags": "|marlin|firmware|", "Answer": "

Preheat Constants - Up to 5 are supported without changing the code just add a new one and then build

\n
#define PREHEAT_1_LABEL       "PLA"\n#define PREHEAT_1_TEMP_HOTEND 215\n#define PREHEAT_1_TEMP_BED     70\n#define PREHEAT_1_TEMP_CHAMBER 35\n#define PREHEAT_1_FAN_SPEED     0 // Value from 0 to 255\n\n#define PREHEAT_2_LABEL       "PETG"\n#define PREHEAT_2_TEMP_HOTEND 235\n#define PREHEAT_2_TEMP_BED    70\n#define PREHEAT_2_TEMP_CHAMBER 35\n#define PREHEAT_2_FAN_SPEED     0 // Value from 0 to 255\n\n#define PREHEAT_3_LABEL       "ABS"\n#define PREHEAT_3_TEMP_HOTEND 270\n#define PREHEAT_3_TEMP_BED    100\n#define PREHEAT_3_TEMP_CHAMBER 35\n#define PREHEAT_3_FAN_SPEED     0 // Value from 0 to 255\n\n#define PREHEAT_4_LABEL       "NYLON"\n#define PREHEAT_4_TEMP_HOTEND 260\n#define PREHEAT_4_TEMP_BED     65\n#define PREHEAT_4_TEMP_CHAMBER 35\n#define PREHEAT_4_FAN_SPEED     0 // Value from 0 to 255\n```\n
\n" }, { "Id": "7677", "CreationDate": "2018-12-17T13:09:31.213", "Body": "

I am at the moment designing some in place mechanical parts. Something like two cylinders that rotate with respect to each other. I need in my design some overhangs that cannot be printed and I was thinking of using soluble supports to be able to print these overhangs. I am afraid that by using something like PVA the PVA will end up between the two parts and glue the together.

\n\n

Does anyone have any experience with this? Can something be done about this apart from 'not using PVA'?

\n\n

Edit:

\n\n

Here is an (minimal) example of an object I would be worried about. The blue and red cylinders are supposed to turn around each other and there is a cutout in the blue cylinder that will need support (the bridge is to long to print without support).

\n\n

\"enter

\n", "Title": "PVA support with small tolerances", "Tags": "|3d-design|support-structures|pva|", "Answer": "

You've not stated this, but one must presume that you have a dual or multi-material extruder equipped printer.

\n\n

Because PVA dissolves in water, the supports will be eroded on contact when the part is immersed. As the PVA dissolves, it increases the concentration of PVA in the water, but only slightly. You would have to agitate the water, change it if you feel it is becoming too concentrated. At some point, you should be able to rotate the cylinders, creating more turbulence, causing more PVA to dissolve into the water.

\n\n

If you are determined enough, you can cause all of the PVA to dissolve, followed by a rinse with clean water. This would remove any residual PVA.

\n\n

The water containing the dissolved PVA is somewhat sticky, but would not glue parts together if the parts were thoroughly rinsed. It's not out of the question to use an ultrasonic cleaner with plain water to provide sufficient turbulence, again changing the water periodically to keep the concentration low.

\n\n

If you use an ultrasonic cleaner, test a sample of your print filament to determine if the heat generated will soften the plastic. I've had success with PVA to the extent that I have not had to use an ultrasonic cleaner to remove it.

\n" }, { "Id": "7678", "CreationDate": "2018-12-17T13:11:56.090", "Body": "

I am trying to get rid of stringing on my prints, to do that I have tried turning on a retraction in Ultimaker Cura, and reducing flow.

\n\n

When I turn on retraction it causes the extruder to skip on the filament. the extruder pulls the material out as it should, but when it pushes back in as it prints the next parts it goes part of the way then it skips making the bumping sound, it seems to do this almost every time that it retracts.

\n\n

To stop this from happening I have tried changing the retraction distance from 10mm to 5mm, the retraction speed from 60 mm/s to 40 mm/s, the flow from 100% to 90% and the temperature from 200 °C to 220 °C.

\n\n

I am using the Ender 3 running Marlin 1.1.9 with an aluminium Bowden extruder upgrade and BLTouch.

\n\n

\"Retraction

\n\n

How do prevent this skipping due to retraction?

\n\n

Update:

\n\n

After changing the setting to what has been suggested in this answer the result of the retraction print resulted in:

\n\n

\"Retraction

\n\n

It has almost completely solved the stringing problem as well.

\n", "Title": "Retraction Causing Skipping", "Tags": "|extruder|creality-ender-3|retraction|", "Answer": "

Fighting stringing will not work by increasing the temperature of the hotend.

\n\n

There could be 2 possible causes for your problem,

\n\n\n" }, { "Id": "7683", "CreationDate": "2018-12-17T17:46:24.213", "Body": "

My Flashforge Adventurer 3 is not printing properly. Filament feeder makes repetitive loud thumping noise and not enough filament is extruded. Resulting print (if completed) is not solid.

\n\n

Results vary with filament. Flashforge brand (red) PLA works best. Any other filament fails completely unless I print at High quality, with head temperature at 235 degrees. Nozzle has been un-clogged repeatedly, but right from the start, when feeding filament in, it does not flow through properly.

\n\n

I had used the included tool to clear blockages in the extruder several times before now, but after posting this question I tried again. Ordinary use of the tool made no difference. So I applied more force, and, pushed filament through by hand directly into the extruder and used a pin to try to clear the extruder. It now appears to be working adequately Not perfectly. I am able to print with default settings. It still thumps -or ticks- a bit. I am alarmed at the amount of force I had to use to clear the extruder blockage.

\n", "Title": "Flashforge Adventurer 3 Filament feeder loud thumping noise, not enough filament extruded", "Tags": "|print-quality|filament|pla|extruder|flashforge-adventurer-3|", "Answer": "

I have had the same issue with mine. And the clicking was indeed calibration. When you're doing the paper part, I make sure I move the axis up one point from the bed allowing enough room for the filament to lay and adhere. If it still thumps after, check for a clogged nozzle (which you\u2019ll have to do anyway if you have attempted countless failed prints from not being calibrated correctly) I can guarantee there\u2019s a blockage at that point it\u2019s just a given if you think about it. Also, I did read above that someone experienced the nozzle being in fact .3\u00a0mm instead of .4\u00a0mm.

\n

I believe this to be true as well. All of their models seem to not be sized right. Therefore, path extrusion needs to be adjusted as well when all is said and done. Good luck

\n" }, { "Id": "7684", "CreationDate": "2018-12-17T18:35:33.537", "Body": "

I am trying to stop the stringing that occurs on one of my prints, I have set it so that it retracts the filament which does stop it during the extrusion of the outer wall layer, however when it comes to print the inner lines of the section it does not retract at all (the extruder gear does not move back).

\n\n

\"Outer

\n\n

\"Inner

\n\n

I am using Cura as my slicer and I cannot find any setting that would change this. My current retraction settings are as follows:

\n\n
Enable Retraction - On\nRetract At Layer Change - On\nRetraction Distance - 7\nRetraction Retract Speed - 40 mm/s\nRetraction Prime Speed - 30 mm/s\nRetraction Extra Prime Amount - 1 mm3\nRetract Before Outer Wall - On\n
\n\n

I am not sure how to stop this from happening, any suggestions that you can make will be greatly appreciated.

\n", "Title": "Extruder doesn't retract on inner wall lines thereby causing stringing", "Tags": "|ultimaker-cura|extrusion|creality-ender-3|retraction|", "Answer": "

Cura has a setting called Combing that is enabled by default. This stops the printer from retracting if the travel is contained within the walls. It does this to speed up the print but you get oozing during the travel since the plastic is still in the melt zone. You can change this setting to no skin which will stop it from combing on the skin layers or turn it of completely.

\n" }, { "Id": "7686", "CreationDate": "2018-12-17T21:47:17.720", "Body": "

I recently switched from Bang-Bang to PID controlled bed heating. The stock firmware pulsed On/Off with about 5 seconds each cycle. Now with PID, my bed keeps its temperature much more stable, the frequency is around 7Hz.

\n\n

Although the total amount of energy used for heating should be less now since the heating is more efficient and doesn't waste energy by cooling down, I am a bit concerned of the fast pulsing of the rather high amperage (around 13 Amps in my case).

\n\n

So I was wondering, do the faster cycles cause more stress on the components (I am thinking MOSFET on the board and components inside the PSU) or is this negligible? I am particularly curious since this is a rather low budget printer (Anycubic i3 Mega, $350 printer with a generic unbranded PSU) and I don't want to push its limits too much.

\n\n

The only thing I can observe is a very faint noise with the same frequency as the pulses. Might have been the same on 5 second cycles, I was just not paying attention to it back then.

\n\n

I'd be very glad if somebody could chime in on this.

\n\n

Thanks in advance.

\n", "Title": "Heatbed PID vs Bang-Bang - which one is more stressing on components?", "Tags": "|heated-bed|electronics|safety|", "Answer": "

The frequency of the cycle shouldn't affect the efficiency as such, however more might be lost due to physical properties. Fast should cause less stress on the printed part, unless it hits a resonance frequency, which it unlikely.

\n" }, { "Id": "7696", "CreationDate": "2018-12-18T15:52:45.867", "Body": "

I'm trying to enter the settings into Cura for the printer.

\n\n

The manual has a screenshot of what they should be:

\n\n

\"Suggested

\n\n

However, when I run Cura and try to enter the settings I'm presented with different options:

\n\n

\"Available

\n\n

\"Available

\n\n\n", "Title": "Adding a custom printer to Cura", "Tags": "|ultimaker-cura|desktop-printer|", "Answer": "

Your manual is in another Castle!

\n

The manual you look at is for Cura (1) 15.04. You are however using Ultimaker Cura 3, most likely the most recent 3.6.

\n

Quick references

\n

The machine setup is for the physical settings of the machine - build volume, nozzle diameter, coordinate system and such.

\n

The other settings you refer to are all set in the right bar, but you will have to switch from simple to advanced mode, and possibly activate the settings you need.

\n" }, { "Id": "7710", "CreationDate": "2018-12-19T21:54:25.530", "Body": "

I've just joined the 3D Printing world and am patiently waiting for my printer to come in next week. I've been thinking about keeping my printer in the garage so that it stays out of the way of every day life.

\n\n

Since I am in the Midwest, I was wondering if this is the best plan. Is it okay to store and use my printer when the ambient temperature can get as low as 10 °F (about -12 °C) and as high as 120 °F (about 50 °C)? Would the cold affect my prints substantially, mainly using PLA, PETG and some TPU? Would larger prints, spanning days, affect the filament on the spool?

\n\n

Note that I am solely talking about the printer. I plan on keeping the filaments inside the house so they won't be bothered by the temp extremes.

\n\n

My google searches came up with nothing and I didn't find anything on the supplier from which I purchased my printer. I doubt that it matters, but in case it does, my printer is the Creality CR-10S

\n", "Title": "Ambient Temperature Range for Printer Storage/Usage", "Tags": "|print-quality|desktop-printer|", "Answer": "

Printers are electrical machines. They don't like water, so they also don't like condensation in them. If you can keep your garage dry and prevent a sheet of metal inside it from rusting, then your printer might survive the problems the shifting temperatures induce by condensing water vapor on the electronics.

\n\n

Another thing to consider is, that at negative degrees Celsius, your printer might trigger a mintemp error, as the minimum temperature allowable is usually defined as 0 °C or higher. On the other hand, low temperatures might allow to bridge further than other days, as the cold air absorbs the heat from the prints faster. It could cause other problems like clogging or under extrusion due to low temperature though.

\n\n

High summer temperatures might impact the print quality, causing quite some extra sagging.

\n\n

The Filament on the machine in a garage might be impacted because of the humidity changes, but might be mitigated with a drybox.

\n" }, { "Id": "7726", "CreationDate": "2018-12-20T15:05:43.267", "Body": "

To check my leveling etc. is correct I was going to print a wide and thin cube. My theory being that I should be able to spot any obvious separation of the layers.

\n\n

Does anyone have any better shape that they print as a dry run?

\n", "Title": "Calibration print shape", "Tags": "|calibration|", "Answer": "

Your calibration shape depends on what you want to calibrate on. Among the most popular:

\n\n\n\n
\n\n

1 - I couldn't find a proper model for an ISO 148-1 Charpy pendulum impact test in thingieverse

\n" }, { "Id": "7740", "CreationDate": "2018-12-21T13:05:11.170", "Body": "

The first layer is very patchy indeed. I've calibrated the build plate pretty accurately but even if it was a badly calibrated build plate I don't think it would have this effect.

\n\n

\"Patchy

\n\n

It doesn't seem to have a negative effect on the print.

\n\n
    \n
  1. Should I be concerned about this?
  2. \n
  3. Is this due to a the build plate fault?
  4. \n
\n", "Title": "Evidence of a warped build plate?", "Tags": "|print-quality|heated-bed|calibration|print-failure|", "Answer": "

I'm having the same problem but worse.

\n

Some parts of the print are squished and towards the middle I will have bad or no bed adhesion. I always have to print with a raft or else the first layer will fail.

\n

Borosilicate glass

\n

From what I've read online the best solution it to replace the build plate or add a pad on top of your build surface. The most recommended one is borosilicate glass. It has good heat conductivity and is very stiff.

\n" }, { "Id": "7749", "CreationDate": "2018-12-21T18:39:06.110", "Body": "

I have a STL file with a rather large roughly-spherical object. I'd like to 3D print it to be fairly large, and hollow inside. Since the printer I'd be using has a max size of (18\u201320 cm)^3, I'd have to print this in pieces and then reassemble them into the sphere-like shape. The textured surface of the object is important, but small seams would seem unavoidable.

\n\n

My Question

\n\n

How could I 3D print a hollow sphere or sphere-like object that's too large for the available basic 3D printer?

\n\n

Because this would need to be done in parts, it's more complicated than the designs I've made. Because the final product is roughly spherical and hollow (i.e. nothing is nicely angular), it's trickier than some other designs.

\n\n

(I have little experience with 3D modelling/CAD, so I'd somewhat prefer solutions with minimal advanced steps such as designing my own joints and whatnot...but advanced steps are definitely okay if part of a great solution!)

\n\n

My Ideas So Far

\n\n

I know I could cut the sphere roughly in (e.g.) eighths via planar slices, print the eight pieces individually, but then the question is how to attach them. I am also concerned that the seams might be too noticeable. (Perhaps there's a better way?)

\n\n

I could just glue the pieces together. My fear then is that it would lack internal structural support if they were attached only at the outer layer (unless the skin thickness were excessive, at least near the boundary).

\n\n

I imagine I could give the pieces internal supports with dowel joints or snap-fit joints (e.g. these), but I lack knowledge of how to do that and don't know whether that would even work well.

\n", "Title": "How could I print a large sphere-like object?", "Tags": "|3d-design|slicing|post-processing|support-structures|", "Answer": "

A sphere can be put together quarters easily, but those need support in the center. However, there is a slightly different cut is more economic:

\n\n\n\n

\"A

\n\n

This way, the support material can be reduced to a minimum - only the top and bottom will need any support, and it is easily accessible to smooth it away.

\n\n

If the walls would be something like one millimeter thick, any good glue should work. If you want to reduce the visible seam, you might bevel the faces, so that the wall has a little gap on the inner wall. As most glues shrink as they harden, it will flatten into the \"gap\", evening out the internal face.

\n" }, { "Id": "7756", "CreationDate": "2018-12-22T14:54:24.457", "Body": "

Current main problem is that at various points during a print, one layer doesn\u2019t attach well to the layer below it. That is what appears to be causing the artifacts in the second picture below, but the picture may not show it clearly.

\n

Background

\n

I have a Makerbot Replicator 5th generation printer. This is the one that is very locked down. As far as I know it can only use the slicer it comes with, Makerbot print, but I'd be happy to be told otherwise. It only prints in PLA and @Trish gave some good advice on drying out the filament. I've also leveled the print bed. Prints have improved from where they couldn't even finish to now where they just aren't very good quality.

\n

The issue I'm having now seems to be primarily that one build layer sometimes doesn't stick to the one below it very well. Then they peel up and the nozzle pulls them and re-melts them into a blob. I've attached two pictures. It's worse in the first one, then I lowered the first model layer fan setting from 50% to 45% and it improved and I was able to get the print in the second picture.

\n

Any help on what settings I can change would be great. Is this because it is underextruding? I think I'm stuck with whatever setting options are available in the Makerbot.print software.

\n

\"First\"\n\"Second\"

\n

Print settings

\n

The default print temp for this printer is 215. This is at 210 degrees, but those blobs aren't actually burned they are a mix of the previous red filament that was on the outside of the nozzle. The travel speed is 150 mm/s, First model layer print speed is 30mm/s, Raft to model shell vertical offset is 0.26mm, Raft to model vertical offset is 0.33mm (I can't tell what the different between those is), z-offset is 0 (default), Layer height is 0.14mm. In the second picture the print was attached but separated easily from the raft.

\n

A couple more settings that might matter is the Print speed: Outlines is 20mm/s, and the Print Speed: Infill is 90mm/s

\n

Update 3/15/19

\n

I made several of the changes suggested including lowering the temperature, leveling the bed, adjusting the Z-offset, and lowering the infill speed. I also continued to dry the filament in a dry box with a lot of desicant that I dry periodically. The desicant seemed to make much more difference than drying the filament at 50C for a couple hours. Print quality has improved a lot, but isn't great. I'm coming to the conclusion that the filament has been damaged by poor storage. It has been left in a drawer in a humid, hot room over the past summer or more.

\n

I'm still using a raft because prints fail completely without it and work reasonably well with one so I have no problem using a raft. Now most of my problem is blobs of filament that I think are running down the nozzle from the heater core. I may have to take some timelapse video to figure that part out. I'm also having some stringing which may be a filament quality issue and some layer shifting.

\n", "Title": "Makerbot Replicator (5th generation) poor print quality", "Tags": "|print-quality|pla|makerbot|", "Answer": "

You can use other slicers with a Makerbot Replicator 5th gen. Simplify works pretty well so I'd look into that. It's not free but a good program.

\n

All the slicer needs to be able to do is export in .makerbot so Simplify is the only other slicer I have found.

\n" }, { "Id": "7765", "CreationDate": "2018-12-23T21:52:08.163", "Body": "

I'm in dire need to reprint a small chunk of a print that got messed up during print because the support for it broke which I cannot explain whatsoever.

\n\n

Anyway. I need to edit the STL file and I don't know what tool to use since Blender is definitely not user friendly for just a simple cut and past.

\n\n

Anyone got an idea on what I should try to do? Reprinting is definitely out of question since that would take at least another 20 hours, that I haven't got, to reprint the whole thing and I would also just waste a lot of PLA.

\n", "Title": "Need help to only print one small part", "Tags": "|slicing|stl|", "Answer": "

I would recommend to use FlashForge in the future(I know that you already solved the problem, but still), the slicer program that I use as well. Blender is not very good for tasks like slicing, as it is rather for creating the actual models you would like to print. Programs like FlashForge provide you with a kind of preview of the model, which is better for slicing as you can see the expected result. \nAnother good thing about FlashForge is that you can push models you created with Blender down in the Z-axis and then slice it off at Z=0 , so you can just use your old model without having to change/modify anything about it.

\n\n

However, as you didn't specify on what printer you are using, I couldn't make sure if FlashForge is compatible with your device. Please send me a comment so that I can come back to this later on, if it is not compatible, there will still probably be a comparable program to FlashForge.

\n\n

Please also keep in mind that in order to fit the two prints together to get the model you would like in the first place, you need PLA glue or something like that for assembling the two pieces, as well as a roughened surface to apply it to.

\n\n

I hope this helps you in the future, Kind regards, Max

\n" }, { "Id": "7771", "CreationDate": "2018-12-24T10:38:41.803", "Body": "

My Anet A8 suddenly had issues with being unable to heat the bed. After ruling out software issues, I disconnected the connector and found this (sorry for the terrible quality):

\n\n

\"Faulty

\n\n

The left most pin on the male connector (bed) is also charred. How could I best repair this?

\n", "Title": "How to repair burnt heat bed connector?", "Tags": "|anet-a8|", "Answer": "

Owning an Anet A8 I confirm that the connectors are not rated for the amount of current that pass through them. You do not need a new bed unless the heat element has burned through (but that is pretty uncommon, it usually is the connector). This burning of the bed connectors is a very well known problem of the Anet A8 printers; these connectors are just not rated for the current and the movement of the bed. It is best not to use a connector at all! And yes, the Anet A8 default printer firmware does not have any build in protection for thermal runaway! It is always advised to immediately flash another firmware, e.g. Marlin firmware.

\n\n

The best repair is to get some high quality silicone AWG 14/16 wires and solder these directly onto the back of the pins of the connector. Also crimp forks to the other end to connect the wires correctly to the printer board.

\n\n

What I did was cutting up the connector to leave only the 2 middle pins (for connecting the bed thermistor, which does not use much power) and soldered the red wire to the left 2 pins and the black to the right 2 pins. You can do that at the back side of the socket where the pins make an angle.

\n" }, { "Id": "7774", "CreationDate": "2018-12-24T17:55:03.473", "Body": "

I have a Monoprice Maker Select v2 printer which has a printing height of 7\".

\n\n

I tried to print this trophy.

\n\n

The first time I printed, it stopped about 3/4 of the way (about 5.5\").

\n\n

I use Ultimaker Cura to move the model's z-position down, effectively splitting it in half to print the top half. But on the 2nd print, it seems to have stopped at the same point. There was a lot of extra PLA filament curled up at the end.

\n\n

\"Printer

\n\n

\"Slicer

\n\n

Did my printer just jam or am I missing a setting that allows it to print the remaining top portion?

\n", "Title": "Why does my model stop printing at the same spot?", "Tags": "|print-quality|ultimaker-cura|monoprice-maker-select|", "Answer": "

The answer is that you have not used support structures. The printing fails as a result of missing support structures.

\n\n

If you look closely to the Ace of Clubs card at the top, you will see that the lower point is being printed from out of nothing, this corner needs to be supported. When unsupported, the extruded filament flows freely and where it deposits is unknown. Usually this extruded filament sticks to the nozzle or ends up stuck at the next piece of the printed layer building up. This build-up can cause the head to hit the print and ruin the print.

\n\n

To enable support in Ultimaker Cura configure your setting accordingly:
\n\"Support

\n\n

These settings are accessible when you select custom settings:
\n\"Advanced

\n\n

If the options aren't visible, use:
\n\"Search
\nand type in the option.

\n\n

Please do note that using supports does not guarantee that the print will not fail! Especially when using long slender support structures, the chance that a support structure fails increases with the amount of support structures and the length of the structure. Sometimes long slender support structures are knocked over.

\n\n
\n\n

Sidenote:
\nYour printer has more issues, if you look at the brim, it is not a continuous bed adhesion layer, it looks like it does not adhere too well, see also the bottom of your trophy.

\n" }, { "Id": "7776", "CreationDate": "2018-12-24T20:26:50.463", "Body": "

I have an object that has cutouts in it (alignment pins fit in the cutouts). The shape of the hole is a right triangle with the point clipped off to create a flat surface in the deep part of the hole.

\n\n

When I print the object in vertical orientation the wall in the deepest part of the cutout is printed properly. But when I print the same object laying down flat on the build plate, the internal wall is not printed and I can see the infill grid through the hole where the wall is supposed to be.

\n\n

I sliced the object with a slicer based off a recent Cura version. I'm using the same STL to print the object vertically and horizontally.

\n\n

Why is this occurring? Is there anything I can do about it other than printing vertically? (Because I have a need to print the real part horizontally. The part shown here is just a thin test piece.) Thank you\n\"Same\n\"The

\n\n

Here is a picture showing what the (Cura-derived) slicer does with the model in horizontally-printed position. I checked the layer display in the vertically-printed instance, and the missing wall was correctly rendered and visible.

\n\n

I guess this is some kind of a slicer bug.

\n\n

\"Layer

\n\n

Here is a large-scale picture of what the cutout shape looks like. This picture is of the protrusion tab that fits into the hole. But the same shape was used to cut the hole in the object.

\n\n

\"The

\n\n

Here is another instance of the same problem. This time, I made up a simple block component, a simple cylinder component, aligned the cylinder into the block volume, and did a combine/cut operation in Fusion to cut a hole in the block (visible in the STL file). As you can see from the (Cura-derived) slicer layer image below, the same problem exists - no wall at the back of the cut when printing in the horizontal position. If I draw a sketch on the block and extrude/cut into the block, I get the same results. I also tried cutting the cylindrical hole, creating a \"tin can\" shell cylinder with an explicit bottom-of-the-can surface, sliding the open can into the cut hole, and joining the can to the block. Same results. The explicit bottom 0.5 mm surface of the can disappears in the horizontal slicing position. Strange.

\n\n

\"Simple

\n\n

UPDATE:

\n\n

The test object is just a simple block with a cylinder extruded into the block as a cutout. As simple as simple can be. I have determined that Cura 3.5.0 works properly (Cura v3.6.0 is available).

\n\n

I am using the Qidi slicer v5.0.7 latest, which uses an earlier version of Cura (I don't know which one; I have sent them an email to find out).

\n\n

I have determined that the Qidi slicer behaves properly if the cutout cylinder diameter is 6mm or larger. But at 5mm, the slicer will not generate a back wall in the cut. The square/triangular cutouts in the example objects below are all 5 mm or less. My installed Cura 3.5.0 generates a proper back wall even at 5 mm. Therefore, it must be a bug in the Qidi slicer, which uses an older version of Cura.

\n\n

As a workaround, I will increase the size of my cutouts to 6 mm. When Qidi updates to a newer version of Cura (they are currently using 3.3), the problem should go away.

\n", "Title": "Internal wall does not print in horizontal print orientation", "Tags": "|ultimaker-cura|slicing|infill|print-orientation|", "Answer": "

I found the answer with the help and inspiration of other people above. Their words reminded me that I could check Cura too, to see if it had the problem. Cura v 3.5.0 did not have the problem. The Qidi slicer v5.0.7 (based on Cura 3.3) correctly generated a back wall if the cutout was 6 mm or larger. At 5 mm, the Qidi/Cura slicer did not generate a back wall. I entered a ticket with Qidi about the problem. I expect the problem will be resolved in their next version. Thank you to everyone for your help.

\n" }, { "Id": "7796", "CreationDate": "2018-12-27T17:16:34.997", "Body": "

I recently downloaded Magicavoxel to give this voxel thing a try, and I was curious if there was a way to turn the voxel files into a file for printing?

\n\n

Magicavoxel doesn't seem to offer a way to export the project as STL.

\n", "Title": "How to print a voxel object?", "Tags": "|3d-design|software|file-formats|", "Answer": "

Magicavoxel supports export as .obj, natively or with the right plugin.

\n\n

While .stl is the standard for 3d-printing exchanges, as it contains a \"1-unit\" length,(typically that corresponds to 1\u00a0mm), .obj is also accepted by most slicers.

\n\n

To get to MagicaVoxel's native export, look in the lower right corner, click export and see this pop up:

\n\n

\"Export

\n\n

Press the top-left OBJ option and you get a save position window where you can store it. Then open it with your slicer and choose a scale - most .obj files do not contain a scale as the format does not contain a unit measure by default like .stl and has to be given one.

\n" }, { "Id": "7804", "CreationDate": "2018-12-28T22:15:47.987", "Body": "

I'm using Simplify3d and a DIY 3D printer. The printer works fine and I've printed very nice objects with very smooth and beautiful surfaces BUT they are twice as tall as they should be! It is like they're scaled double in Z axis! I've checked everything, I changed steps/mm from 4000 to 2000 but it ruined the smooth surface. changing layer height just makes smoother surface and doesn't solve the problem. What am I missing in the settings?\n\"on

\n", "Title": "Calibration of Z axis in Simplify 3D", "Tags": "|diy-3d-printer|calibration|z-axis|", "Answer": "

The steps per mm in firmware is the link to the hardware configuration/setup. Reducing by half must reduce Z advance by half. If your prints are twice as tall, you must have serious under-extrusion problems as when the printer is laying down molten filament for e.g. a 0.2 mm layer, in effect this is a 0.4 mm layer. If this is not the case, you should also calibrate the extruder.

\n\n

Please re-calibrate your machine Z axis in firmware (most firmwares allow command M92 to set the steps per mm), not in Simplify3d, and then the extruder.

\n" }, { "Id": "7818", "CreationDate": "2018-12-29T23:09:59.503", "Body": "

I recently got a Dremel 3D20, and I understand it only takes PLA filament according to the Dremel site. However, I was wondering if anyone has successfully used TPU filament or knows it will work fine. I\u2019m more than happy to use other software to change the temperature, I just don\u2019t want to gunk up or otherwise ruin my printer.

\n", "Title": "TPU Filament in Dremel 3D20?", "Tags": "|tpu|", "Answer": "

I've never used a Dremel printer, thus I cannot guarantee that my solution will work fine, but you might want to give a try to the Ultimaker Cura slicer.

\n\n

Ultimaker printers have Bowden tubes, like the Dremel, unless I am mistaken, and I've been able to successfully print TPU on my Ultimaker 3 Extended printer. I also know that many Ultimaker users have printed using Ninjaflex without much problems on their Ultimaker machines, both old and new.

\n\n

Ultimaker Cura comes with pre-programmed settings for TPU95, which is the Ultimaker brand of TPU, then I guess you'll have to fiddle a bit with settings to find what works best with your Dremel. Take a look around Ultimaker's forums, many users have other printers beyond their Ultimaker printers, you might be lucky and find some useful information. The 'search' feature of the forum works quite nicely.

\n\n

Quick summary of the settings for TPU95A in Ultimaker Cura for a 2.85\u00a0mm Bowden setup:

\n\n\n\n

Depending on how the Dremel performs and the quality of your TPU, you might want to disable retractions, it's been known to help.

\n" }, { "Id": "7823", "CreationDate": "2018-12-30T06:28:21.413", "Body": "

My attempted prints on my Creative Ender3 of model:\n\"This\nhttps://www.thingiverse.com/thing:23686

\n\n

The print uses support structures (generated by Ultimaker Cura) failed in the same location twice. It is printed in PLA (1.75 mm) first print at 200 °C, second print at 196 °C.

\n\n

\"Failed\n\"Failed

\n", "Title": "Basic Spinning Top Prints Fail in Same Location", "Tags": "|creality-ender-3|troubleshooting|", "Answer": "

To me, it looks like your G-code induces an incomplete layer of support on the still standing piece, which later down leads to the print failing.

\n\n

Re-slice the whole thing.

\n\n

As a matter of fact, I would cut the model in its widest place and print both with the large face flat on the surface and glue the two pieces together after printing. That way I can achieve:

\n\n\n" }, { "Id": "7831", "CreationDate": "2018-12-31T00:30:06.857", "Body": "

Recently got an Anycubic I3 Mega Printer and I've been playing with what it can do, but after a model is done it leaves residue on the build plate behind that is bugging me. Do I NEED to remove it? If so, how? Thanks! (I'm using PLA if that matters)

\n\n

My Problem:

\n\n

\"Residue

\n", "Title": "How to remove exess filament from bed after removing model", "Tags": "|pla|heated-bed|post-processing|", "Answer": "

I have had moderate success with a product called Goo Gone (don't laugh). It quite remarkably lifts off any number of \"sticky\" items from various surfaces.

\n\n

I work with PLA filament that is very hard and stubborn to remove from any surface.

\n\n

I own a FlashForge Adventurer 3 that appears to have a carbon fiber build plate. Nothing, except a #11 XActo blade, can harm that.

\n" }, { "Id": "7839", "CreationDate": "2018-12-31T23:30:24.843", "Body": "

What kind of primer should I be using for my PLA prints?

\n\n

I want to be able to sand the object after applying the primer for a smooth finish before painting.

\n\n

Would something like this work?

\n\n

https://www.bunnings.com.au/dulux-duramax-325g-plastic-primer-spray-paint_p1400720

\n", "Title": "Sandable primer suitable for PLA?", "Tags": "|pla|post-processing|smoothing|", "Answer": "

There are generally 2 types of operation you might want to do before priming your object:

\n\n\n\n

Smoothing via Sanding & Filler

\n\n

To smooth your object you generally have two options:

\n\n\n\n

Only after you filled up the structure to be somewhat smooth you apply paint primer. In itself, common spray on primers often are not filling enough to hide away the printing layers.

\n\n

For rough surfaces that need a starting fill, a paste body filler applied with a spatula works best, and for the last pass over a just lightly scratched surface, automotive body filler from a spray can works great. The benefit of spray-can body filler is, that it also acts as a first roughing step, so you don't need to roughen the surface for the primer. If you grab a filler-primer, even skip the primer.

\n\n

Sidenote from experience:

\n\n

Some filler-primer and lacquer spray cans seem to contain solvents that are able to soften PLA.

\n\n

Smoothing via Coating

\n\n

A random find on Thingiverse showed me another way to paint and flatten the surface faster and without sanding, at the cost of details getting smoothed away:

\n\n\n\n

It works by picking up the wet paint and embedding it into the thicker Polyurethane layer, which dries much smoother than the paint itself, filling up the imperfections and the steps between layers. This process will however also fill up non-masked surface details you might want to preserve.

\n\n

Your product in mind

\n\n

The primer you took a look at works on PLA, but it would not smooth out all the dimples. It might work nicely for the PU-Buildup variant.

\n" }, { "Id": "7855", "CreationDate": "2019-01-02T16:02:21.283", "Body": "

I want to print a transparent PET bottle for my homemade lemonades and thought about 3D-printing them.

\n\n

I would like the printed bottle's quality to be as fine as a Coca-Cola PET bottle for coke and the printing substrate or material be something that's cheap or readily available. I thought I might recycle some old PET bottles to print the custom one for my lemonades as I don't want something that would require me buying new materials or re-exporting from the manufacturer.

\n\n

Is it possible to recycle PET and print it into food-certified containers?

\n", "Title": "Can I 3D-print a PET bottle?", "Tags": "|print-quality|print-material|pet|", "Answer": "

No, due to 3 reasons

\n

PET is not (easily) printable.

\n

There is a lot of confusion on what Filaments you can buy: most times filament branded PET is actually PETG, sometimes PETT.

\n

PET is not an easily printable material at all. With expert knowledge and the right machine settings it can be printed, but even then, it is not as easily recyclable into a useable 3D-printer-filament as you might think. You need full reprocessing capabilities, which means the need for machinery to allow thorough cleaning, grinding to dust, melting it up, pelletizing and finally extrusion as a fresh filament.

\n

The closest related material that is easily printable is PETG, a modified PET that also contains glycol. You can't convert PET into PETG with home or hobbyist applications at all - they are totally different in their chemical behavior, even as just one material was added in production. PETG is not brittle like PET, it does not haze on heating, but it ages in UV light, scratches easily and can't be autoclaved like PET. But the chemical modification has to be done during the initial manufacturing of the material, and it is a huge mess to try to recycle the two together, which can and will happen if you try to work with material you source from recycling.

\n
\n

\u201cWhen they\u2019re processed together, PETG melts and becomes sticky while PET remains solid. PETG sticks to PET chips and forms large clumps that pose processing problems.\u201d Resource\u00a0Recycling (magazine/blog)

\n
\n

3D printed objects are very unlikely to become food certified.

\n

You can't easily manufacture (certified) food-rated printed products, like food containers due to the requirements that a machine that manufactures food-certified products needs to comply to. I advise looking at this answer regarding food rating for more elaboration.

\n

It is hard to print really transparent with FDM.

\n

Due to the method how FDM works - extruding lines next to each other - it is often impossible to print fully transparent objects right of the bat - there is almost always air inside a printed object, and there are so many boundaries between the extrusion paths that refract and change the photon paths that the best one can achieve somewhat easily is translucent (=semi-transparent). Read this answer for further information.

\n

But if you manage to get the object really solid, you might get some near-transparent, icy results from some orientations while looking in others still will look matte.

\n

To get them fully transparent you then will have to post-process them to become fully transparent by grinding the surface up to 4000 grit, but that is very labor intensive and most likely not possible for the inside of a bottle. To be clear, you spend hours polishing one surface.

\n

Could it be economic in the slightest?

\n

On a side tangent, the viability of printing a bottle via buying new ones will need to be expored. Shapped PET Bottles with caps start at \\$0.01 per piece and top out at \\$1 per piece - you get the better prices if you order in larger quantities. You will have to compete with getting under \\$1 per bottle, or rather with what the price of a typical bottle you want is.alibaba.com

\n

A typical PET bottle ordered from China weighs 30\u00a0g for a 300\u00a0ml bottle, and the particular example I looked at comes \\$0.22 to \\$0.28, depending on the bottle cap, with a minimum order of one parcel with something around 300 items. That seems to be in the average range.

\n

A roll of 1 kg of PET(G?) filament starts at ~\\$30 at the moment. That is the weight of 33 shaped bottles per roll. Your print will most likely be heavier than the blown up bottle to get it watertight, but let's just assume you might manage the same weight. Then it's about \\$0.90 in the material alone - so we are at more than 300% of a bought product with cap already!

\n

Atop that comes the running cost of the printer, which depends on your print time, printer and electricity price. I know my hobbyist machine comes, maintenance and electricity combined, down to 0.21\u20ac/h, so roughly \\$0.25. Printing a bottle will take several hours.

\n

PET preforms that can be blown up to almost any bottle shape, type and size and ship much cheaper come to prices due to better density. Which means you compete against \\$0.015 to \\0.15 per bottle in material costs.

\n

Conclusion

\n

It is not economically viable to even attempt to print bottles beyond a prototyping stage.

\n" }, { "Id": "7867", "CreationDate": "2019-01-04T00:08:32.683", "Body": "

Here's what it looks like\n\"enter

\n\n

This is the model\nthingiverse linky

\n\n

It looks like it couldn't print the edge, but this happened many many hours after printing the brim.

\n\n

This did not happen with my 1st attempt at this print. The last print lost adhesion and I had to scrap it. This time, adhesion looks good so not sure why this happened.

\n\n

Printing with Monoprice Select V2 with ABS, sliced with Cura. 100C bed / 250C extruder. 15mm/s initial layer speed. 60 mm/s print speed.

\n\n

Update\nIt looks like the printer is starting to smooth it out like so. Still not sure if this will lead to an ok print or will fail because of this layer.\nAnd it seems the stringing area does not have a brim underneath it. Did Cura just not calculate the brim size correctly?\n\"enter

\n\n

Update2\nHere's a few screenshots from Cura to show that the model is lying completely flat.\n\"cura1\"\n\"cura2\"\n\"cura3\"\nI let the print go on overnight and here's where I stopped it\n\"top\n\"side\nIt almost seems like the print shifted completely after printing the initial layer. Have you ever seen anything like this or is there anything in my Cura model that would make it do this?

\n", "Title": "What causes this \"stringing\" and can my print recover from this?", "Tags": "|troubleshooting|print-failure|monoprice-maker-select|", "Answer": "

Both @Oscar and @Trish have identified the problem in their answers. Upon further investigation, I believe I found the root of the problem.

\n\n

The Monoprice Select V2 has what I think is an inherent design flaw with how its wires are routed. The wire can sometimes (1) get caught on the frame (when the y-axis is moving towards its maximum position), (2) get caught between the frame and the print bed preventing the y-axis from reaching the minimum position, and (3) it even hits the y-limit-switch sometimes.

\n\n

Here's a backview of the printer where you can see problem (1) where the wire can get caught by the frame. You can also see the y-limit-switch and I saw first-hand when the wire bundle caught it, forcing another layer shift in my print.\n\"enter

\n\n

For problem (1) above, I found this solution seems to help\nthingiverse link

\n\n

To resolve all of the wiring issues, you have to unwrap the wires and reroute them. People online say this voids the warranty.

\n\n

The wire jamming doesn't always happen. But when you printer moves back and forth along the y-axis hundreds of times for a large print, there's a high probability it snags at some point. This is very frustrating after many hours of printing. And I'm thinking of getting another printer for this reason.

\n\n

I should note that it looks like Monoprice's next version printer, the Monoprice Select Plus, doesn't seem like it would have this problem since the control unit is integrated and the wires don't seem like they would be in the way.

\n" }, { "Id": "7881", "CreationDate": "2019-01-06T04:45:55.790", "Body": "

I have a Monoprice Select v2 that I just bought less than 2 months ago. I've manually leveled it on several prints prior.

\n\n

I noticed a couple of holes in my print bed after my last print, like the extruder pushed into the sheet really hard. They're both on the same side.

\n\n

I then noticed that the extruder's horizontal bar wasn't leveled so I leveled it. I did this by unplugging the wire to one motor and modified the z-position so that the other connected side would go up. (Is this the proper way of adjusting the horizontal bar?)

\n\n

And then I moved the extruder back into its Home position at z-position 0.\nThat's when I noticed that it seems my extruder barely reaches my print bed even though I have the screws almost loosened to the point of detaching from the thread.

\n\n

The extruder z-limit-switch (I think that's what it's called) seems to be pressed so the extruder is as far down as it can go. The controls also show that the z-position is at 0.

\n\n

So I'm guessing my print bed somehow lowered. But I don't see how that could possibly be.

\n\n

Any ideas?

\n\n

Here are some photos but I'm not even sure what to show really

\n\n

you can see the z-limit-switch in the right of this pic next to the letter A's\n\"pic1\"\nyou can see my extruder still has some distance to go before reaching my print bed but you can also see that my screws are loosened as much as possible\n\"pic2\"\nshowing the horizontal extruder bar is now leveled\n\"pic3\"

\n", "Title": "I can't level my bed any more", "Tags": "|bed-leveling|monoprice-maker-select|", "Answer": "

For leveling your bed you do not require the use of a bubble level! This is not necessary, if you do, you need to place the printer on a bubble level underground. It is far easier to level the bed relative to the printer frame rather than to the outside world. Basically the bed and the X-axis (the axis that moves your nozzle) need to be parallel and each side of the X-axis needs to have the same distance to a reference point on the frame.

\n\n

In order to level your X-axis you need to position both of the sides at a certain, same distance from a reference point on your frame, e.g. you could measure this with respect to the top of the Z stepper attachments. If you have 2 steppers, it is easy as you can individually move them by turning the lead screw when the stepper motors are not powered. Once you have each side at the correct distance you can commence a homing sequence and start leveling your bed.

\n\n

You mention that you are running out of threads for proper leveling of the bed while the Z minimum limit switch is pressed/activated. What you need to do is lower the switch a little so that you have some extra height available. If the switch is at the lowest, you could consider drilling extra holes or filing it down, or print one of the Z-adjustment mods that can be found on the web. This should be enough to level your bed properly. Leveling can be done cold or hot, the difference is that the heat expands the plate a little or causes some changes in the shape of the bed due to the expansion or uneven heating. This difference is usually very small, but sometimes causes your prints not to stick to a hot plate when leveled cold.

\n\n

\"Level

\n" }, { "Id": "7891", "CreationDate": "2019-01-07T07:37:34.100", "Body": "

I am trying to print an item with a rectangular grid (using PLA on Ultimaker 2+):

\n\n

\"item

\n\n

Holes are 4x4 mm, distance between holes (wall thickness): 1mm. I am printing with 0.4 mm nozzle.

\n\n

Unfortunately, Ultimaker Cura generates G-code which prints each rectangle on its own and in a quite a bad way:

\n\n

\"print

\n\n

The printer head tries to draw a rectangle, then moves in the direction opposite to a last laid line, which with not perfect adhesion of single line messes up the print pretty badly: it often picks up last rectangle side and drags it.

\n\n

Any way to change the way Ultimaker Cura approaches to lay out the grid? I would imagine that long lines that are connected to other lines would adhere to bed much better than individual squares, but I don't see any options that would allow to alter it.

\n", "Title": "Printing rectangular grid", "Tags": "|pla|ultimaker-cura|slicing|", "Answer": "

Perhaps a slicer is not the ideal tool for this application. It should be possible to use the slicer to generate a 'framework' file, doing coordinate conversions, setup and such, and then write raw G-code or a script to do so.

\n

Whether you find that a desirable approach is another question.

\n" }, { "Id": "7905", "CreationDate": "2019-01-07T15:54:09.763", "Body": "

Recently, I did discover that it might be possible to print fully solid, air free and thus optical homogenous, even with PLA.

\n\n

How can I design a proper focusing lens (or rather: single sided lens with a flat bottom) with this, ignoring the post-processing needed to get the surface smooth?

\n", "Title": "Designing lenses", "Tags": "|pla|3d-design|", "Answer": "

To calculate the focal length of an optical element, the two main factors are the refraction index and the shape of the lens.

\n

For a cylindrical lens with one optic active side (that is, one domed or bowled side), we can ignore the whole bottom cylinder and just take into account the top dome. The shape of the dome is determined by the radius of the circle that created it.

\n

Thin, single sided lenses

\n

For a thin, single-sided lens the rather complicated Gullstrand\u2019s formula to calculate the focal length of lenses becomes rather simple:

\n

$f = \\frac {r}{(n-1)}$ for the bend facing the object

\n

$f = \\frac {r}{(1-n)}= \\frac {-r}{(n-1)}$ for mounting it in reverse.

\n

A Polymer database did give a refraction index of PLA as $n=1.465$.

\n

Thick, single sided lenses

\n

For a thick lens with a total thickness of $d$ and one active side, we solve first for the one active side, and then insert:\n$$f_1=\\frac{r}{n} \\land \\frac {-r}{(1-n)} ; f_2=\\infty ; P_i = \\frac 1 {f_i}$$\n$$P=P_1+P_2 -P_1P_2\\frac d n ; P_2\\to0$$\n$$f=\\frac{1}{P_1}=f_1$$\nAs long as one side of our lense stays flat, the thickness of the lense is mathematically not relevant (save for increasing dispersion).

\n" }, { "Id": "7907", "CreationDate": "2019-01-07T17:21:15.440", "Body": "

I'm trying to build an assembly with one larger part (about 50 mm x 50 mm), two small parts (about 10 mm x 5 mm) each, and one part that starts with two 2 mm x 2 mm squares that eventually bridge into a sort of flap. The larger part has decent adhesion for a couple of layers, but the small parts are slipping off the table right away and the nozzle drags them onto the large part since it's the last part being printed in the group, resulting in some stringing.

\n\n

My bed is adequately leveled and can print larger models well so my suspicion is that the small parts are too small for good bed adhesion and need a brim or a raft to increase the surface area. Is this correct? If so, is there a way in Cura to add a brim or raft to the small parts and not the large part? Ideally I would print all pieces at once instead of the large part separate since I will be doing batches of these assemblies in the future.

\n\n

I'm using Ultimaker Cura 3.6, printing with PLA filament set to 200 °C at nozzle and 60 °C on the bed.

\n", "Title": "Setting up a brim/raft for only a portion of the overall print", "Tags": "|ultimaker-cura|adhesion|rafts|brims|", "Answer": "

This post is quite old, but I describe how I solved this for anyone has the same problem I had: areas moved by the nozzle as the base of some appendices are too thin and tall.

\n

In Cura 4.8 (I don't know since which version this feature is available) it is possible to add some custom shapes (cubes or cylinders) to your model. You can find this tool after move, resize, etc,, in the prepare page (left side icons).

\n

So you can add some custom supports to protect from falling/moving some areas of a single model that need to be kept in place. These added parts can be moved and resized as you would do with the any other imported model.

\n

I found this very useful to avoid that the nozzle moves parts with a small base area that are connected to the rest of the model on a higher level and the lower part is not stable enough. Especially when printing TPU or other flexible filaments.

\n" }, { "Id": "7915", "CreationDate": "2019-01-08T09:26:21.183", "Body": "

I am looking for a font, which is connected and has a good line width (so that it can be 3D printed). Something like in the image below:

\n\n

\"enter

\n\n

Something like in the image above would be great, but I don't know the name. Do you have a suggestion for a font?

\n", "Title": "Font suitable for 3D printing", "Tags": "|3d-models|", "Answer": "

That particular font is Sarina Regular and is part of the Google Fonts collection. Just Google \"connected handwritten fonts\" to find more. Kimberly Geswein does a good selection of handwritten fonts. A lot of them are constant width, but most are unconnected.

\n" }, { "Id": "7918", "CreationDate": "2019-01-08T14:45:17.120", "Body": "

I've been having some bed adhesion problems that I have been trying to solve by leveling the bed. I think that it's pretty level now but when I start a print the lines seem pretty flat. Is this true? I used a feeler gauge to have a 0.2 mm gap between the bed and nozzle, but the center feels as though there's more space, despite the tape (ie I didn't feel any resistance when leveling the center) so I'm not sure if the plastic is too squished or not.

\n\n

\"Top\n\"Side

\n\n

I am printing PLA with 210 \u00b0C at Nozzle and 60 \u00b0C on bed. I also used a 130% extrusion factor for the first layer on a Creality CR-10S. It did come without a black print surface but with a glass sheet and a roll of painters tape instead, so I opted for Blue-tape.

\n\n

Also, I see some stringing, which seems to happen with skirts and brims (this is my first brim) and a bit of under extrusion in the center. But those are probably concerns fit for a separate question.

\n", "Title": "Bed too close to nozzle?", "Tags": "|troubleshooting|adhesion|creality-cr-10|", "Answer": "

I cannot stress enough how much purple glue stick helps with this exact problem.

\n\n

I had a cheap XYZ printer that I used nearly half a spool of filament on trying to align and get prints to stick. After ten hours straight of hair pulling and desperation, I smeared some purple glue stick across the build plate -- worked like a charm. Works great even just on bare glass, although I usually use blue tape underneath.

\n\n

Also had one of those cheap Monoprice MP Mini and a 3D Solutions Cube which both benefited greatly from the glue stick.

\n\n

I've had a few cheap printers that I carried with me for educational purposes in a backpack and I'm sure the alignment wasn't ideal, but the glue always made it work perfectly.

\n\n

Elmer's makes one, I picked up a 3 pack of cheap ones on Amazon with great success.

\n" }, { "Id": "7920", "CreationDate": "2019-01-08T15:28:58.513", "Body": "

After a brownout a print failed and it got the whole hotend covered in PLA. I am now in the process of replacing some parts on the hotend (one of the thermistor legs broke of) and also wanted to take the heater cartridge out. The problem is that the bolt that locks the heater cartridge is stuck and I am now afraid to strip the head.

\n\n

Is there a trick to remove the bolt with the smallest probability of stripping the head? Is it better to apply heat or not? Should I soak it in Acetone or some other solvent? etc.

\n", "Title": "Heatercartridge bolt stuck", "Tags": "|pla|hotend|", "Answer": "

I got a blob of around 5 x 8 cm out of PETG! Setting extruder temp to PETG or even a little higher. I did it all with an older soldering iron.\nSimply (but carefully!) go with the point into the blob and take it slowly off. With a copper 'toothbrush', I cleaned the extruder and nozzle more times.

\n

After mostly all was down I started with the first layer test (Prusa MK3S+)\nThat was surprisingly almost perfect. Then started to print not very high subjects like rings etc. With that several burned drops came down during printing. Use the 'toothbrush' as many times you can between the first prints!

\n

Done without any damage to the printer as a whole!

\n" }, { "Id": "7931", "CreationDate": "2019-01-09T03:35:54.493", "Body": "

Today I was printing some parts and I got these burn marks at the corners:

\n\n

\"1\"

\n\n

This is the first time I see this on this printer (Anycubic I3 Mega). I was using more or less the same settings (in Cura) as always, although I was using a new roll of PLA from a brand I haven't used before.

\n\n

I was printing several parts in one job:

\n\n

\"2\"

\n\n

Most of them are somewhat affected:

\n\n

\"3\"

\n\n
Printing temperature:    200 \u00b0C\nBuild Plate Temperature:  60 \u00b0C\nPrinting Speed:           60 mm/s\nTravel speed:            200 mm/s\n
\n", "Title": "What causes burnt corners?", "Tags": "|print-quality|pla|", "Answer": "

Could be a dirty nozzle and temperature too high. When the head changes direction on the corner, the nozzle wipes off burnt PLA. This might explain why there are no dark marks on the long runs and only the corners.

\n

Heat nozzle up and clean with acetone and then see if the problem persists.

\n" }, { "Id": "7957", "CreationDate": "2019-01-12T12:51:04.407", "Body": "

When I'm printing with my Chromatik filament white, 1.75 mm in diameter, I observe a gap between the infill and the walls (see picture).

\n\n

Everywhere on the web I can find explanations for this kind of problem (apparently it's the symptom of loose belts), but I have this problem for this filament only. I have the filaments Chromatik electric blue and Octofiber black and I don't have this issue with them. I tried to increase the temperature by ~10 °C, but it didn't have much effect.

\n\n

Have you ever seen that guys? I repeat, it's with this filament only.

\n\n

\"Print

\n", "Title": "Gap between infill and walls for one filament only", "Tags": "|filament|infill|", "Answer": "

I have observed similar issues between walls, not necessarily between the infill and the walls.

\n\n

It is most likely that the viscosity of this filament is way different (less fluid) than the other filaments you print. Not only mechanical issues (to be precise: inaccurate positioning e.g. caused by loose belts) could play a part in this, but also printing speed. A more viscous filament needs more pressure and time to get the filament through the nozzle. This is exactly what happened in my case, because of different wall speed line settings (inner and outer), the filament did not flow fast enough leading to under extrusion. In your case you probably also have a higher infill than wall speed, so lowering your infill speed may mitigate your problem. Also, most slicers have an option to define the overlap between the infill and the perimeters/walls, you could also increase that for this filament.

\n" }, { "Id": "7960", "CreationDate": "2019-01-12T15:45:14.487", "Body": "

My ender3 came with a BuildTak-clone surface, and as I was a little too vigorous in getting the print off the bed (I had failed to level right and printed a bit too tight to the bed, resulting in SUPER strong adhesion), I needed to replace it.

\n\n

Peeling off the black was easy. The plastic sheet that held the glue was easy too... but how to clean up the bed to get the residue glue off and prepare for the new 3M sticker?

\n", "Title": "How to clean up my build plate for a new build surface?", "Tags": "|heated-bed|replacement-parts|repair|", "Answer": "

Following up from Charles Duffy's comment, I can confirm that adhesive remover containing D-limonene is an effective way of removing remaining glue when replacing an Ender-3 platform sticker. While \u201cGoo Gone\u201d was named in the comment, I used a similar product, Goo Buster.

\n

Goo Buster turned the glue residue into a water soluble gel that was easily removed using a plastic scraper, with final clean up using a wet paper towel. While D-Limonene has a pleasant citrus fragrance, it is an irritant, and these adhesive removers typically also contain petroleum products and surfactants.

\n

As ever, use care when using solvents near electrical devices such as 3D printers. Ensure the device is isolated from power and all parts are clean and dry before operating.

\n" }, { "Id": "7963", "CreationDate": "2019-01-12T21:16:44.563", "Body": "

I just switched to the Duet Wifi board (used MKS Gen L 1.0 before) and now small structures look terrible. From my observations the hot end moves correctly, but almost no filament gets extruded. On the other hand larger structures look very good. With the MKS board the exact same G-code worked fine.\nAfter that I gradually disabled many features like coasting, wiping and even retractions altogether, the quality only improved by a tiny margin.

\n\n

Example print (the two towers should be cylinders):\n\"Example

\n\n

Additional information:

\n\n\n\n

More info (edit):

\n\n\n", "Title": "Small structures are deformed after switching to Duet Wifi", "Tags": "|print-quality|extrusion|troubleshooting|fdm|", "Answer": "

The OP found the solution and shared this in comments but has not written a proper answer. The OP found:

\n\n
\n

To complete this, the issue was related to the part cooling fan not spinning correctly. After replacing it the problem was gone.

\n
\n" }, { "Id": "7965", "CreationDate": "2019-01-13T21:55:11.213", "Body": "

I have a Robo 3D R1+, and I'm using the default Simplify3D profile for it. When I finish a print, I notice that my X gantry is way out of level. The right side is visibly higher than the left side. So much so that it won't even auto-level on the next print. What that tells me is during the print, the Right z-axis is moving up faster than the left. It also ruins the print.

\n\n

I do not believe this is a hardware issue, as when I use the Craftware slicer, it works properly (I tried the same model with both slicers). Is there a setting or something I need to adjust in Simplify3D to get this to print properly?

\n\n

If you would like to see the G-code for the CW and S3D models I was testing with, you can download them here:

\n\n

G-code download (Google drive)

\n", "Title": "Z-axes out of sync with Simplify3D", "Tags": "|z-axis|simplify3d|", "Answer": "

What you are describing is not possible as a result of changing slicers, this must be an intermittent hardware issue.

\n\n

The slicer has no knowledge of the hardware layout of your machine (other than the build volume and gantry dimensions; if properly configured), it just creates slices of the model you present which are found in the G-code file as Z movements. In case multiple Z steppers are used to move the X gantry, these are usually driven by a single driver, but if they are separately controlled, even in the G-code file the instruction would be to move up Z in total, not per stepper. It is the firmware of the 3D printer that translates this Z level in movement for your 3D printer based on the layout of the machine and the firmware settings.

\n\n

The CraftWare G-code file does show some inconsistencies with respect to the Simplify3D file in that it does not use G29 and has a too large first layer height of 0.45 mm (this is larger than you nozzle diameter of 0.40 mm, you should always limit that to about 75 % of your nozzle diameter).

\n\n

For an X gantry to become unlevel/skew, there must be a mechanical issue that is causing it to miss steps/prevent advancing at one side. It is possible that one of the shafts or lead screws has some more friction than the other (generally it is not a good idea to grease the screws as dirt easily stick to the grease, a light oil may be better suited). This is not uncommon for Prusa i3 clones and is usually fixed by resettling the leadscrew nut by loosening and tightening the screws which attach the nut to the X gantry idler. It is unclear if this is your problem here with this specific machine.

\n\n

To be fair, uneven displacement of any axis powered by 2 separate motors (driven by separate drivers) could be induced by the slicer when unrealistic high accelerations and incorrect hardware/electronic settings are employed.

\n" }, { "Id": "7979", "CreationDate": "2019-01-15T18:45:45.133", "Body": "

Printer: Anet A6

\n\n

I'm gaving some trouble with a loose hot end. I was trying to peel of some PLA when I noticed the whole thing was kinda loose. I could pull it out very easy.\n\"Loose

\n\n

So I tried to put it back, but it's verry loose and I don't know what exactly is wrong.\n\"Put

\n\n

I'm not sure if I broke something, or doing something wrong. Does anyone know how to fix this or do I need a new extruder or hot-end?

\n\n

Kind regards.

\n\n

Update:

\n\n

As Trish said in the accepted answer, the heatbreak is broken. When I've opened the extruder it became more clear:

\n\n

\"enter\n\"enter

\n\n

I'm happy cause this is a fix for about 10 euro.

\n", "Title": "Extruder get loose", "Tags": "|pla|hotend|anet-a6|", "Answer": "

I am sorry to inform you, but your picture looks like your hotend has a broken heatbreak. The heatbreak is the screw that connects the heater block with he coldend.

\n\n

Your first picture clearly shows that the tp 2 or 3 windings of the screw are ripped off. Probably from tightening the nut against the codend carriage and this way applying tremendous force - the lower picture hints that it was tightened far too strong.

\n\n

You are lucky however, as replacement heatbreaks are easy to come by and cheap.

\n" }, { "Id": "7982", "CreationDate": "2019-01-15T19:39:01.503", "Body": "

Printer: Monoprice Select V2.

\n\n

I've done several prints already and swapped out filaments many times but in my most recent swap, I can't feed my PLA through any more.

\n\n

I first preheat my extruder for PLA temps (185 °C). Then I press the plastic thingy to allow me to push as much of the filament into the hole as possible. Then I adjust my extruder position to try to suck it in. Usually after a few mm, I start to see the filament come out of the extruder and I also feel a pull on the filament from the top. But nothing is happening now.

\n\n

However, if I adjust the extruder position in the opposite direction, it eventually pushes the filament back up and out, so I guess the \"stepper motor\" (is that what it's called?) is working (at least in one direction).

\n\n

I'm getting ready to open the extruder module up to see what's going on, but wanted to see if anyone had any simple ideas for me to try before I unscrew anything.

\n", "Title": "Why won't my filament feed through the extruder anymore?", "Tags": "|extruder|change-filament|monoprice-maker-select|", "Answer": "

I figured out the issue. I think I was shoving it in the wrong hole.

\n\n

I took apart my extruder component. There's a good video on it here.\nBut basically, you just have to loosen the two bottom screws on the side fan like this:\n\"enter

\n\n

I checked everything out first. I cleaned the extruder head with the included pin. I also shoved the filament through the heated area and filament came out ok.\n\"enter

\n\n

But then I discovered the filament could go in the wrong way sometimes through the extruder.

\n\n

This is the correct way for the filament to go through. It should come out of that plastic hole.\n\"enter

\n\n

But once in awhile, I accidentally pushed the filament through this way.\n\"enter

\n\n

If the filament was bent and I pushed the lever too hard, it would often find its way down the wrong path.\nSo I played around with how much force I should be holding down the lever and how hard I should be pushing the filament through.

\n\n

I don't know if other 3D printer extruder feeders are designed this way, but seems error prone. Or maybe it's just me.

\n" }, { "Id": "7987", "CreationDate": "2019-01-16T09:20:42.000", "Body": "

I've been 3D printing for a while and I've noticed that, when printing small parts, my colored plastics (PLA, PLA+ and ABS) have better layer adhesion than black ones.

\n\n

Did you notice this?

\n\n

What could be the cause?

\n", "Title": "Are black filaments more brittle?", "Tags": "|filament|filament-quality|", "Answer": "

I have some experience in the textile industry and it is known that items, whether piece goods or yarn, which are black (for most fibers) it is likely an over-dye; colors that failed to match the color that was intended, goes into a pile for black re-dye. It is easy for most fibers to over-dye something black. I imagine that for filaments it is something similar. My experience with black filaments is generally one where black breaks easier, and generally you can find PLA on sale if it is black.

\n" }, { "Id": "7989", "CreationDate": "2019-01-16T12:04:55.947", "Body": "

I'm having a lot of difficulty removing support material without damaging the print.

\n\n

\"Support\n\"Support

\n\n

Are there any tips/tricks to doing this or is it just a case of sanding, cutting, chopping and then cleaning it up as best I can?

\n\n

Settings

\n\n\n", "Title": "Difficult to remove support material", "Tags": "|print-quality|support-structures|monoprice-maker-ultimate|", "Answer": "

It is recommended that you try "tree support" in the "experimental" module of settings.Also,before removing the support, blowing with a hot air gun to soften the support material connected to the model may achieve good results.

\n" }, { "Id": "7992", "CreationDate": "2019-01-16T13:39:33.697", "Body": "

I own a Creality CR-10 (using Ultimaker Cura for slicing) and I am experiencing terrible printing problems.

\n\n

My problem: I am not able to print anything that takes several hours to print. For parts printed in 30 minutes or so, my printer usually works.

\n\n

The prints end up like this (unfinished and with a lot of stringing)

\n\n

\"fail\"

\n\n

This below is actually the best result I got so far.. (important note: there is no under extrusion during the print, it suddenly stops out of nowhere..)\n\"fail2\"\n(Although I have to use terrible retraction settings and need a lot of post processing because of all the stringing - you might have an answer for that too.. I just couldnt get the 3d printer to print parts properly with good retraction settings.)

\n\n

The problem is that at some point of the print, no more (PLA) material is extruded and the printer moves without actually printing anything. The first couple of layers usually work (you can see that in the images) but after wasting almost 600 grams of PLA, I am not able to find a solution myself.

\n\n

.

\n\n

I have some suspicions:

\n\n

Could the problem be the angle, the PLA is inserted into the feeder? (so that it is almost a 90 degree bend)? The filament comes from a spool in the right hand side. But I dont think that this would cause such a problem..

\n\n

\"Angle

\n\n

Is it a software problem? Here are all my cura settings (I even reduced the printing speed to 30mm/s at 205\u00b0C - still didnt work..) :-(

\n\n

\"enter\n\"enter\n\"enter

\n\n

Could the length of the bowden tube and it tangling up be a problem? (as you can see in the image below)\n\"enter

\n\n

And annother important thing: The feeder always grinds into the filament (even at those low retraction settings) and it is always very hard to pull the filament out of the bowden after a failed print. Sometimes its almost impossible and i have to use heavy tools for it.. that should be the probelem\n\"enter

\n\n

I already did some atomic pulls, replaced the nozzle and switched the bowden tube.

\n\n

I have a dream: My printer printing a part without any stringing and actually finishing the print. Please help me to achieve this dream..

\n\n

Thank you for your help in advance. :)

\n\n

(and the filament I used, sorry for the bad quality of the pic)\n\"enter

\n", "Title": "Filament extrusion always stops at some point during print", "Tags": "|print-quality|filament|pla|creality-cr-10|", "Answer": "

I had the same thing! On my anycubic I3 mega.

\n\n

Tried a lot of stuff and got so sick of it so I installed a 10:1 gearbox on my extruder motor. So now it provides 10x more torque and since I havent got the issue anymore where the extruder is just grinding on the filament and the motor is just skipping steps.

\n\n

No because of this and because my anycubic won't connect to a pc I can't change settings on my printer so I resolve the ratio with 1000% flow in my cura settings. And other stuff is also affected.

\n\n

But at least it won't stop and just be sitting there and skipping steps.

\n" }, { "Id": "8022", "CreationDate": "2019-01-20T20:07:06.073", "Body": "

I have an Ender 3 that I have been pretty happy with so far, however it recently started an odd behavior and I can't figure out what's causing it.

\n\n

What happens is that the first ~3 mm of the print comes out \"sloppy\". After that, everything clears up and it prints fine for the rest of the print. (Although it perhaps looks a little under extruded on the top layer)

\n\n

\"enter

\n\n

It looks almost like it's over extruding. But if that's the case, why would it only be for the first 3 mm? Then the top layer looking under extruded makes that possibility even more unlikely.

\n\n

This is consistently happening regardless of what I am printing, the brand of filament (I only print PLA), or the bed temp or hot end temp. I've tried tweaking with the bed leveling and giving a little more gap on the first layer, but that doesn't seem to change anything either. I also calibrated the extrusion multiplier and it's spot on now.

\n\n

I use Ultimaker Cura 3.6 as my slicer. I tried resetting back to defaults to see if maybe I had inadvertently changed something but that didn't help either.

\n\n

I have done a few upgrades - Marlin firmware, Capricorn tubing, glass bed, replaced the (broken) plastic extruder with one of the metal ones, new PTFE fittings. I didn't notice the problem until recently so I can't say if it started corresponding with any of those upgrades.

\n\n

When I first got it, the prints came out beautifully from the first layer, so this is really frustrating me. I'd like to get it back the way it was.

\n\n

Any suggestions on where to look?

\n\n

Update: I did some slightly more controlled experimemts and I did get it looking a little better. It does seem related mostly to bed tempurature. The cooler I make the bed, the better it looks. However as it gets cooler, the prints are also starting to warp and break loose, so the print ends up failing completely. I had a hard time getting a successful print below about 45 degrees, and even at that temperature it still isn't completely clean. I'm using glue stick for adhesion and it just doesn't seem able to hold it without some heat. I traditionally have run around 50 degrees before this problem started though, so it seems odd I have to drop below that now.

\n\n

Also, for more info, the cube dimensions are pretty close in the X and Y, but were about .5mm short in the Z. So the layers do seem to be settling.

\n\n

I did check the bed temp with a non-contact thermometer and it was consistent with what the printer reported, so it doesnt seem to be a bad thermostat throwing things off.

\n", "Title": "First 3 mm prints poorly, then fine after that", "Tags": "|print-quality|extrusion|troubleshooting|creality-ender-3|", "Answer": "

I solved the problem by adding a second Z-axis leadscrew. The problem was that I could have either too tight Z-axis rollers or a sagging gantry. Both of this lead to the nozzle rubbing over the print. In case of sagging, rubbing on the right hand side of the print and a too large distance on the left. In case of too tight rollers, rubbing on the whole print or on whichever side had the tight rollers.

\n

If you don't want to invest in a dual leadscrew setup, turn the excentric nuts that fix the rollers of the gantry to the Z-axis; making sure that they are tight enough but not overtight.

\n" }, { "Id": "8038", "CreationDate": "2019-01-22T22:28:36.557", "Body": "

Is there a way to view more than the last 300 lines in the terminal tab on OctoPrint? Or is there a txt file of a log? Or even a setting/plugin that does either?

\n\n

I keep finding my prints pausing as if I said to change the filament even though that wasn't set in the slicer, but I catch it long after the 300th line in the terminal so I can't see what OctoPrint is trying to do.

\n", "Title": "How to log more than 300 lines of the OctoPrint terminal?", "Tags": "|octoprint|", "Answer": "

Serial logging has to be enabled. Warning: This will impact the performance of OctoPrint. Enabling this feature can be done under Settings -> Serial Connection -> Serial Logging and checking the box for "Log communication to serial.log.\n\"OctoPrint

\n

One can download the log under Settings -> Logging and finding serial.log in the list of log files.

\n

The file can also be found in the following directories (according to this FAQ)

\n
\n

All log files that OctoPrint writes can be found in the logs sub folder in its configuration directory:

\n\n
\n" }, { "Id": "8041", "CreationDate": "2019-01-23T03:19:03.197", "Body": "

I'm unable to use the parts cooler I printed. Every time I turn it on, the hot end temp drops too low, and for some reason the PID can't seem to get it back up:

\n\n

\"Fan

\n\n

This is a test to simulate the issue. The first drop is the fan kicking on, then the last bump is me turning the fan off. It's almost as if the set point drops when the fan kicks in. Any ideas? The PID is tuned (I ran the autotune) and works well without the fan on. This is a RepRap Guru Prusa clone.

\n", "Title": "Unable to hit hot end temperature with part cooler on", "Tags": "|hotend|heat-management|cooling|", "Answer": "

This effect you describe is a commonly known problem that occurs when the print part cooling fan is not correctly positioned, i.e. if it blows air directly onto the nozzle or heater block and is best solved by printing an alternative part cooling fan duct. Alternatively you could insulate the heater block with some insulation cotton or silicone socks that fit over the heater block.

\n" }, { "Id": "8044", "CreationDate": "2019-01-23T12:42:50.497", "Body": "

Which filament is good for a coaster for a hot cup?

\n\n

I don't expect PLA to be good because it can easily melt under a hot cup.

\n\n

Note, I can't use ABS because I don't have an enclosed printer.

\n", "Title": "Filament choice for coasters", "Tags": "|filament-choice|", "Answer": "

Even if you pour in boiling water in a cup, the outside of the cup will have a lower temperature. When resting on a coaster, usually a small part of the cup actually touches the coaster. Also, the design of the coaster could influence the heat transfer, a more open structure of the coaster may be beneficial. Some people print coasters in PLA although the glass temperature (temperature at which the plastic becomes soft, this is usually the temperature of your bed when you print the material) may be lower than than the temperature of the cup, the filament will weaken (for PLA at about 50 °C), but not melt , melting of PLA happens at a higher temperature (for PLA above 150 °C) than your mug can get (unless you pour liquid metal in it). It should therefore be doable in PLA, I have printed a PLA coaster that has very small ridges (about 1 mm) of an embossed image and placed a cup with boiling water on it, to find that the coaster is able to withstand the temperature of the cup without deforming (the ridges do not fail or deform).

\n\n

To elaborate on the filament materials other than PLA; there are many filament types that have higher glass temperatures, but are still very printable. Various types of co-polyesters exist, like PETG, that have a higher glass temperature (> 85 °C), are a substitute for ABS and still very well printable. Nylon is also a material that can be used, there are brands that have low warp nylon.

\n\n

Note that there are a few options to print ABS while you have a non-enclosed printer, you could

\n\n\n\n

Note that coasters have a relative low profile height, it should not be that big of a problem to print ABS coasters on an open printer.

\n" }, { "Id": "8050", "CreationDate": "2019-01-24T00:42:51.380", "Body": "

I need to print a flexible, hollow \"sleeve\" or \"cover\" for an elongated part, kind of like a soft \"skin\" for a \"finger\" (see pic). I'm not sure how to approach the hollow space problem, i.e. how to print a flexible surface that is above an enclosed, hollow space. I could print supports inside, but they'd be hard to remove from inside the space, because I can't quite access all corners with a tool from outside. I could print this in 2 parts (\"bottom\" / \"walls\" + \"ceiling\" separately), but I'd prefer not to glue if I don't have to. Any ideas how to approach this?

\n\n

\"enter

\n", "Title": "Flexible filament how to print a hollow \"sleeve\"", "Tags": "|support-structures|flexible|", "Answer": "

Print orientation is usually the key to print with a minimum of supports. If you print this part with the sharp point down, you will get some support structures on the outside for overhang support to prevent it from tipping over, but no support on the inside.

\n" }, { "Id": "8059", "CreationDate": "2019-01-25T09:36:01.657", "Body": "

I've started printing PETG recently and I'm happy with results so far, awesome strength and good looking (except for stringing). But I've noticed that PETG prints better with more distance nozzle-plate than usual, and under-extrusion make parts looking better than both normal/over-extrusion.

\n\n\n", "Title": "PETG nozzle clearance and extrusion multiplier", "Tags": "|petg|", "Answer": "

On my Ender 3 Pro's I have found the following works well (also remember settings can be effected by different brand/quality of filament):

\n\n

If you use retraction, it may help to slow it down to 25\u00a0mm/s and adjust retract distance if your using Bowden tube or direct drive. Last, a must have, a can of hair spray, works great and less expensive than the glues.

\n" }, { "Id": "8084", "CreationDate": "2019-01-29T06:35:23.410", "Body": "

I'm trying to get the ROKO (SN04-N) sensor to work with my Anet A8. First of all, while trying to screw it to the extruder, I tightened it too much and sort of broke the acrylic plate... sort of. I had to use a very thin steel plate with two holes to enforce the acrylic plate. It still works.

\n\n

Now, I followed instructions in this video. (Please note that the video is not in English.) After the first \"Auto Home\" operation, the guy draws on the bed and then measures. My measurements are slightly different.

\n\n

At the 19-minute mark, the guy is hard-coding the values but I don\u2019t understand how he calculated them? My measured offsets are: X_PROBE_OFFSET_FROM_EXTRUDER 16 and Y_PROBE_OFFSET_FROM_EXTRUDER 58. In his video, his calculations were 18 mm for the X offset and 57 mm for the Y offset. Either way, I\u2019m not able to compile the code as a sanity check fails:

\n\n
\n`static_assert(FRONT_PROBE_BED_POSITION >= MIN_PROBE_Y, \"FRONT_PROBE_BED_POSITION is outside the probe region.\");`\n
\n\n

Here are the sensor limitation values from the configuration file:

\n\n
// Set the boundaries for probing (where the probe can reach).\n#define LEFT_PROBE_BED_POSITION 20 //MIN_PROBE_EDGE\n#define RIGHT_PROBE_BED_POSITION 200 //(X_BED_SIZE - MIN_PROBE_EDGE)\n#define FRONT_PROBE_BED_POSITION 47 //MIN_PROBE_EDGE\n#define BACK_PROBE_BED_POSITION 200 //(Y_BED_SIZE - MIN_PROBE_EDGE)\n
\n\n

What am I doing wrong?

\n\n

Note that I'm using Marlin Firmware v1.1.9

\n", "Title": "ANet A8 running Marlin v1.1.9 Auto Bedlevel with ROKO SN04-N", "Tags": "|marlin|anet-a8|bed-leveling|", "Answer": "

Too bad you broke the acrylic plate (nice temporary fix though), but you can easily print a replacement part once your machine is up and running.

\n\n

Probe positioning is defined in the Marlin configuration as:

\n\n
\n *      +-- BACK ---+\n *      |           |\n *    L |    (+) P  | R -- probe (20,20)\n *    E |           | I\n *    F | (-) N (+) | G -- nozzle (10,10)\n *    T |           | H\n *      |    (-)    | T\n *      |           |\n *      O-- FRONT --+\n *    (0,0)\n
\n\n

This implies that your sensor is located on the back-right when facing the machine and need to have the following constants set:

\n\n
\n#define X_PROBE_OFFSET_FROM_EXTRUDER 16   // X offset: -left  +right  [of the nozzle]\n#define Y_PROBE_OFFSET_FROM_EXTRUDER 58   // Y offset: -front +behind [the nozzle]\n#define Z_PROBE_OFFSET_FROM_EXTRUDER 0    // Z offset: -below +above  [the nozzle]\n
\n\n

In order to calculate the correct limits of travel for the sensor, you need to subtract the offset values from the bed size at the max limits.

\n\n

An additional offset may be required for some sensors, so please add an additional offset in the configuration by defining:

\n\n
#define MIN_PROBE_EDGE 10\n
\n\n

As the sensor is off-center with respect to your nozzle, one can only assume that you have no extra space to move the whole printhead and therefore need to confine the head within the limits of the max/min bed size (there should be some extra space, this can be seen from the offsets for the origin as in values for X_MIN_POS and Y_MIN_POS, but for the sake of simplicity these will not be taken into account).

\n\n

Basically, your positive Y and positive X offset result in the following schematic.\n\"Schematic

\n\n

Or, if you include the #define MIN_PROBE_EDGE [value]\n\"Schematic

\n\n

Bed limits for the sensor then will need to be calculated based on the values of your offset of the sensor. E.g. when your nozzle is at (X=0, Y-0), or (0, 0), your sensor is at (16, 58). If you don't want to move the head further left and forward (to respect to origin as limit!), this is the minimum position of the sensor. When the sensor is at the back-right position of (220, 220), the actual head is at (220-16=204, 220-58=162).

\n\n

This means that the limits for the sensor without a minimum offset are (16, 58) and (220, 220):

\n\n
\n#define LEFT_PROBE_BED_POSITION (X_PROBE_OFFSET_FROM_EXTRUDER + MIN_PROBE_EDGE)\n#define RIGHT_PROBE_BED_POSITION (X_BED_SIZE - MIN_PROBE_EDGE)\n#define FRONT_PROBE_BED_POSITION (Y_PROBE_OFFSET_FROM_EXTRUDER + MIN_PROBE_EDGE)\n#define BACK_PROBE_BED_POSITION (Y_BED_SIZE - MIN_PROBE_EDGE)\n
\n\n

would translate with a MIN_PROBE_EDGE = 0 to:

\n\n
\n#define LEFT_PROBE_BED_POSITION 16\n#define RIGHT_PROBE_BED_POSITION 220 \n#define FRONT_PROBE_BED_POSITION 58\n#define BACK_PROBE_BED_POSITION 220\n
\n\n

and would translate with a MIN_PROBE_EDGE = 10 to:

\n\n
\n#define LEFT_PROBE_BED_POSITION 26\n#define RIGHT_PROBE_BED_POSITION 210 \n#define FRONT_PROBE_BED_POSITION 68\n#define BACK_PROBE_BED_POSITION 210\n
\n\n

The assertion in code: FRONT_PROBE_BED_POSITION >= MIN_PROBE_Y would now translate to (58 >= 58) (or 68 >= 58), in your case it was (47 >= 58) which clearly is not true.

\n\n
\n\n

Please look into this answer, this answer or this answer for more information.

\n" }, { "Id": "8091", "CreationDate": "2019-01-29T23:11:59.720", "Body": "

After uncommenting the REPRAP_DISCOUNT_FULL_GRAPHIC_SMART_CONTROLLER, the display is blinking and no characters are shown on the screen, I have already switched the cables between EXP1 and EXP2, but it did not succeed.

\n", "Title": "LCD full graphic smart controller, no character display and screen blinking", "Tags": "|marlin|full-graphic-smart-controller|", "Answer": "

this problem can be solved by turning the slots on the display, as in the image below. Some Chinese displays are inverted from factory.

\n\n

\"enter

\n" }, { "Id": "8100", "CreationDate": "2019-01-30T21:48:38.103", "Body": "

I made some 3D printed supports for tools, using screws to fix it to the wall, some of them broken because of the screw forces. Is there a way to reinforce only the screw holes where it will have more stress/compress? I am using PLA, Fusion 360 and Ultimaker Cura.

\n", "Title": "Reinforce screw holes", "Tags": "|fusion360|", "Answer": "

In order to add localised extra walls, I will sometimes cut a torus shape around the part that I want to strengthen. This can result in n*wall, infill, n*wall, void, n*wall.

\n\n

See the images in this answer if the description is not clear.

\n" }, { "Id": "8103", "CreationDate": "2019-01-31T05:16:12.187", "Body": "

I am trying to make a box that is 420 mm wide by 86 mm tall by 100 mm long, I wonder if there is a good technique to design, cut, print and fit all parts together to be safe and hard.

\n", "Title": "Print box bigger than the printer bed", "Tags": "|fusion360|", "Answer": "

Are you making a simple box? Or does it have some kind of detail or structure? The photo below exemplifies a structure attached to connectors that have been created outside of Fusion, but you can also use it as an idea to create your own!

\n\n

\"enter

\n\n

Link to OPENSCAD LIBRARY

\n\n

If you are thinking of cutting into parts, you can also create a kind of male / female (puzzle) in the parts that fit. Something like that ...

\n\n

\"enter

\n\n

Here is some interesting information to study How to design Snap-fit Joints for 3D Printing

\n" }, { "Id": "8105", "CreationDate": "2019-01-31T12:11:22.043", "Body": "

Is there any risk of damaging stepper motors if I set too big travel speed?\nWhat is maximum safe travel speed?

\n\n

My printer is a German RepRap Neo.

\n\n

I currently use 120 mm/s. Is it safe to increase this value to 200 mm/s?\nWhat would my printer do if I set very big travel speed?

\n", "Title": "How fast can printer head move without damaging steppers?", "Tags": "|safety|mechanics|speed|", "Answer": "

A high speed is unlikely to be reached unless you also set a high acceleration, and acceleration is generally more likely to cause a problem (unless you reach the pulse rate limit of the drivers).

\n\n

High acceleration will increase vibration, and critically requires higher torque from the motors. At some point, the torque will exceed the motor/drive current capability, and the motor will skip steps. As soon as this starts to happen, your print will become unusable.

\n\n

Before reaching the point of missed steps, you're likely to see other quality issues, but unless you're in a very hot environment, unlikely to see damage to the motor. Depending on the quality and heatsinking of the stepper driver, you might see overheating here (you can check for overheating of the board though).

\n" }, { "Id": "8121", "CreationDate": "2019-02-01T07:03:26.667", "Body": "

I'm pretty new to printing PETG, and my Creality Ender 3 now has a glass print bed. I've done some research, and it looks like a pretty bad idea to try and print PETG on a pure glass surface for fear of the print pulling chunks of glass along with it.

\n

So, I've looked into various bed adhesion options. I've already used blue tape, but I'll need to purchase some wide stuff, and in the meantime, I want to investigate using PVA-based glue stick. However, one question does come to mind: how can I set the bed-nozzle distance after applying the glue stick layer?

\n

Do I even need to? Will the extruded plastic just crush it into the build plate without a problem, saving me the trouble of lowering the build plate? My normal method of sliding a paper sheet under the nozzle at Z\u00a0=\u00a00\u00a0mm would take the glue stick layer along with it if I tried that.

\n", "Title": "How to level bed after applying glue stick for bed adhesion", "Tags": "|heated-bed|bed-leveling|adhesion|petg|glass-bed|", "Answer": "

How much glue do you put on it? I use PVA based spray, barely visible, very evenly spread and no problem whatsoever of sticking paper to the glue layer.

\n

Just level the bed as you normally would and apply a sparsely applied coat of glue, preferably from a spray can. Note that glue stick dissolves in water, so you can distribute the glue with a moist cloth preventing globs or thick layers of glue.

\n

Having printed literally kilometers of PETG on various build platforms (various glass sheets with or without PVA based glue, Aluminium and PEI), chipping of glass is not something I have seen happening (this part is added after comments on adhesion and glass chipping in comments). Perhaps, in case of chipping, if the glass might be of very low quality, then chipping may occur. Whether your glass is of decent quality can be tested, just print on one side, and if it chips you always have the other side. My Ultimaker 3E sheet of glass has a small chip taken out of the surface, but that was caused by not carefully removing the print, tip is to not pry with the corner of a too thick painters spatula, the corner can dig into the glass if force is used. This was not caused by the adhesion of the print but a user error, now, years later and many more kilometers of PETG, no more extra chips are out of the sheet.

\n

Printing on PEI with PETG is another story, PETG just fuses to PEI and is very difficult to remove, a PVA based glue helps, but from experience, still prints were very hard to remove.

\n

Invest in a can of build plate adhesion spray, it lasts long, never gives you problems with adhesion and probably is even cheaper than glue sticks (per mass unit).

\n" }, { "Id": "8124", "CreationDate": "2019-02-01T09:45:57.927", "Body": "

I am new to 3D Printing. We currently have to design in process that I want to finish. The design consists of two parts (top and bottom), the top and bottom that need to snap fit into each other. The arrangement should be similar to the one used in many electronics, for example, the cordless phones in the picture below.

\n\n

My question is, can such clips be done with 3D printing? Will they last (be used 5 to 20 times) and does the material used for 3D printing make a difference? Is there a material that is particularly good for such snap-fit connectors?

\n\n

Note that my design will only be a prototype and eventually we would be creating a mold for our product.

\n\n

\"clip

\n", "Title": "Can snap-fit connector be designed for 3D Printing?", "Tags": "|3d-design|clip|", "Answer": "

yes

\n

You can totally print snap-fit connectors for 3D printing, but you need to keep some things in mind. I assume that since you have looked at these connectors, you have a good idea about the matter, but I nevertheless suggest Angus (MakersMuse) discussion locking devices at the beginning of a tutorial Video on designing buckles.

\n

Your tabs thickness will need to be at least one line width wide at the most narrow point to be printable at all.

\n

Print-Orientation

\n

Another thing besides thickness is orientation. For the strongest tabs, you might want to print the tabs like a C for maximum part strength. Just modeling the part with the tabs, it should print in this orientation then:

\n

\"A

\n

This way the bending is not applying stress against layer boundaries but 90\u00b0 to them, giving even force on each layer. This means that you will need to print parts in awkward orientations just for the tabs usually. You will need lots of support.

\n

You can certainly print in a less awkward position at the expense of strength of the tab, acknowledging that "this is a prototype, we can show you that it closes perfectly like this, though due to FDM limitations we might break the tabs opening it again. So we'll be careful."

\n

prototype-variation of model

\n

Snaples tabs

\n

It might be however easier to print the tabs flat and without the hooks, allowing to do a fitting test, but not a snap-connection. Your benefit is, that you won't have to watch for print orientation, but it won't be locking. Make sure to work with workflow and put the modifications for easy printing/not-locking at the end. Alterations to the general design should come all before this point. Then turn off these steps to create the model that is sent to machining for the mold.

\n

Modular intermediate design

\n

If you need to have working, strong tabs AND a good print, it can pay off to print separate parts that combine into a single piece with a little glue. Maybe the C-clamp is actually a thin bar that is put into the back plate and glued into place or secured with a little friction weld.

\n" }, { "Id": "8126", "CreationDate": "2019-02-01T16:50:27.417", "Body": "

I recently changes the nozzle on my Monoprice Maker Ultimate 3D printer (first time). The brass block that the nozzle screws into is free to rotate around an is wobbly.

\n\n

\"Brass

\n\n

Loose block video

\n\n

I can't see any obvious nut/screw that's come loose though. It could well be that it's not important, but it was definitely tightly attached before it twisted when I removed the nozzle.

\n\n
\n\n

I've taken the assembly apart:

\n\n

\"Silver

\n\n
\n\n

Silver bit removed:

\n\n

\"Silver

\n", "Title": "Loose brass heater block", "Tags": "|hotend|monoprice-maker-ultimate|", "Answer": "

The silver looking cylindrical nut (with the flat faces) between the brass block and the black metal plate is the heat break of the assembly. The brass block should be tightly fit to this heat break. You could turn the brass block to get a close fit again. The heat break itself can move in the upper part, black plate, the cold end by the set screw on the side. Some hotend assembly types allow that, this screw is then used to fixate the orientation of the brass block. This type of hotend is not very common, it is a MK10.

\n\n

Without the brass block it looks like this:

\n\n

\"MK10

\n\n

Maybe this clears it up how the block is attached in between the heat break and the nozzle; it is positioned where you see the threads. The screw to position the brass heater block and the heat break is clearly visible.

\n\n

Don't try to wobble the brass block any further as you will wear out the threads, there appears to be a lot of play already.

\n" }, { "Id": "8129", "CreationDate": "2019-02-02T00:10:43.937", "Body": "

Recently, at work we bought a Guider II printer from FlashForge.\nWhen we try to print models using a high resolution or models with a too high printing time, the feeder gets clogged. And the feeder is too hot. We have concluded that the feeder is getting clogged because the high temperature softens the PLA.

\n\n

We check the feeder fan and is working fine. How can I avoid the jamming? Pausing the 3D printer and wait a few minutes is not working for us. I don't know if it is a common problem for this 3D printer model.

\n\n

Edit:

\n\n

The hotend is an all metal hot end. I can't find more information about the hot end.

\n\n

In the manual of the guider II flashforge recommend a temperature of 210\u00b0C for the head and 30\u00b0C for the bed. I have tried different temperatures. The most common temperature I've used is 190\u00b0C for the head and 55\u00b0C for the bed (I obtain the best results with this temperature).

\n\n

This is the Hotend used by this 3d printer.\n\"Hotend

\n", "Title": "How can I avoid jamming in the feeder?", "Tags": "|extruder|troubleshooting|cooling|", "Answer": "

As Oscar pointed out, this seems to be heat-creep.

\n

What is Heat Creep

\n

Hear Creep happens if the thermal energy deposited in the hotend works up through the heat break and out of the dedicated melt zone, resulting in filament clogging up in the coolend.

\n

Where does Heat Creep come from

\n

Heat Creep is usually a sign of having chosen the settings for the print incorrectly.

\n

The biggest culprit is by having a too high printing temperature. I personally have not yet encountered any PLA that demands to be printed at above 200\u00a0\u00b0C.

\n

In an all-metal hotend, the flow of filament down the path is a serious contributor or keeping heat-creep in check. So as a result, very low extrusion speeds have to be avoided to allow to keep the melting happening only in the meltzone. As the speed of extruded filament is related to the diameter of the extrusion, it is usually better to stay away from very small nozzle diameters.

\n

The anatomy of the heatbreak is also a factor. Take a look at your heatbreak and then at for example the e3D v6 heatbreak below. As you see, it is necked down between the coolend section (the long part) and the part that screws into the heater block (the short part). This reduces the capability of heat to transfer up through the heatbreak, as $I\\propto A =(R_a^2-R_i^2)\\times\\pi$. If $R_a$, the outer radius, shrinks by necking down the heatbreak, then the whole flow of thermal energy is reduced, counteracting heat-creep. But that has to be designed for.

\n

\"an

\n

Another factor that can result in heat creep is insufficient cooling of the cool end. Make sure that the fan that is mounted on the cooling fins spins always and gets 100% of its supply voltage power. Then make sure that it can draw in the maximum amount of air and push it out an unobstructed path.

\n" }, { "Id": "8133", "CreationDate": "2019-02-02T15:21:44.650", "Body": "

I made a model for my 3D clock. Model has few vertical holes so I made one layer thick bridge for every hole. In Fusion 360 everything looks fine, but CURA (version 3.6.0) doesn't want to print those Bridges. Why?

\n\n

The Full Wall is 1.68mm thick(in Fusion), infill is 20%, wall line count is 1, layer height is 0.28mm, the support-bridges are designed to be this thick too.

\n\n

I'm not sure will be material stable above holes without Bridges.

\n\n

\"Bridges

\n\n

\"Model

\n\n

\"Hole

\n", "Title": "CURA doesn't want print bridges", "Tags": "|ultimaker-cura|fusion360|", "Answer": "

Nomenclature help

\n\n\n\n

\"A

\n\n

Ultimaker Cura does ignore too thin walls

\n\n

Your walls are 0.28\u00a0mm thick and your nozzle is most likely 0.4\u00a0mm thick. That can't be printed at all, in fact, you are way too thin to be printed: a printer should never print any object that is thinner than its nozzle as that is a perfect way to generate clogs.

\n\n

Atop that, slicers ignore also what is thinner or exactly as thin as the line width. If the line width is 0.4\u00a0mm, and the wall is 0.4\u00a0mm, it gets ignored. If the wall is 0.45\u00a0mm, it is not. This can be remedied by setting the option print thin walls, but you still need to have one line nozzle-diameter as minimum line width - actually it is common to go about 10% larger for better extrusion.

\n\n

Solution

\n\n

Strengthen the walls to one line width (usually 0.4\u00a0mm) in your CAD-design and activate print thin walls under Shell.

\n\n

\"A

\n" }, { "Id": "8136", "CreationDate": "2019-02-02T17:47:54.567", "Body": "

After I level my Ender 3, the distance between the nozzle and the bed seems fine on both ends, but moving the bed on the Y axis shows that it's increasing and decreasing for three times, which I just cant fix.

\n\n

This only occurs on the left side - the right side is constant from beginning to end.\nAlso I've been using three different beds (the magnetic one and two glasses) to make sure it's really something else.

\n\n

I created this video to demonstrate the problem.

\n\n

I'm sure that this has something to do with the carriage wheel adjustment, but tightening those did not change anything.

\n\n

How do I get rid of this problem?

\n", "Title": "Ender 3 distance between nozzle and bed changing on Y-axis", "Tags": "|troubleshooting|creality-ender-3|bed-leveling|y-axis|", "Answer": "

Blue Painters tape on the left side starting at the center and leaving a band on the far left side and the rear Perfetto side. I checked the before and after and the thickness of the tape shimmed out the deflection on the plate. It sounds like whatever the use to stamp the steel out is creating the bend we are only talking several thousands of an inch but when I put a flat edge on the plate and shined a light from the rear I could see daylight. The blue tape was from Nearys video on creality cr-10 bed leveling.

\n" }, { "Id": "8139", "CreationDate": "2019-02-02T22:09:25.580", "Body": "

I just completed my first print on my Ender-3 and when the print finalized itself the nozzle didn't elevate itself to clear away from the piece. I watched as the nozzle slowly lowered itself into my print and destroy it. Here is the gcode generated by Slic3r used:

\n\n
\n; Filament-specific end gcode\nG4 ; wait\nM221 S100\nM106 S0 ; turn off cooling fan\nM104 S0 ; turn off extruder\nM140 S0 ; turn off bed\nG91\nG1 F1800 E-3\nG90\nG1 Z{z_offset+min(layer_z+30, max_print_height)}{endif} ; Move print head up\nG28 X0 ; home x and y axis\nG1 Y180; Remove Print Position\nM84 ; disable motors\nM300 S2600 P100; Beep\n; filament used = 24040.5mm (57.8cm3)\n; total filament cost = 0.0\n
\n\n

\"preview

\n", "Title": "Why did my printer's nozzle dig itself into my print?", "Tags": "|g-code|creality-ender-3|", "Answer": "

The same thing happened to me several years ago. My print was a fairly tall part for my delta printer, about 220 mm. In many previous prints, the print head would always go up after finishing, but what I didn't know was that the Slic3r finish print routine specified an absolute Z height of about 200 mm. When the tall part finished, the still-hot print head moved down and ruined the top of the print.

\n\n

After figuring out that the routine had an absolute reference, I replaced it with a relative movement 20 mm up. It's worked fine since then. An easy fix.

\n" }, { "Id": "8153", "CreationDate": "2019-02-03T19:06:07.593", "Body": "

In setting up a probe for automatic bed leveling, the limits of the probing area must be entered in the firmware of a 3D printer.

\n\n

When using a sensor, how do you define the bed limits for the sensor in Marlin firmware?

\n\n

E.g. the Marlin (1.1.x) configuration.h contains the following inactive statements:

\n\n
\n  // Set the boundaries for probing (where the probe can reach).\n  //#define LEFT_PROBE_BED_POSITION MIN_PROBE_EDGE\n  //#define RIGHT_PROBE_BED_POSITION (X_BED_SIZE - MIN_PROBE_EDGE)\n  //#define FRONT_PROBE_BED_POSITION MIN_PROBE_EDGE\n  //#define BACK_PROBE_BED_POSITION (Y_BED_SIZE - MIN_PROBE_EDGE)\n
\n\n

How do you set the values for the constants of your custom setup? Is this generic or specific?

\n\n

Probe positioning is defined in the Marlin configuration as:

\n\n
\n *      +-- BACK ---+\n *      |           |\n *    L |    (+) P  | R\n *    E |           | I\n *    F | (-) N (+) | G\n *    T |           | H\n *      |    (-)    | T\n *      |           |\n *      O-- FRONT --+\n *    (0,0)\n
\n\n

The probe could be placed with a positive or negative X and Y value. Considering this position, how to setup the sensor bed limits?

\n", "Title": "How to set Z-probe boundary limits in firmware when using automatic bed leveling?", "Tags": "|marlin|bed-leveling|z-probe|knowledgebase|", "Answer": "

There are a few questions on this topic, so a more generic solution would be informative and will prevent multiple questions of others when their sensor has a slightly different location. This answer intends to cover any position with respect to the nozzle. It also does not matter what kind of sensor it is, it can be 3DTouch, BLTouch, inductive, capacitive, etc., as long as you are able to determine the offset to the nozzle (center to center).

\n

For now, it is assumed that the nozzle can only reach the complete area of the bed, no extra space. So unless the sensor runs of the bed, the nozzle limits are used, otherwise the sensor limits the nozzle with respect to the sensor limits. This is the safest assumption and will prevent the carriage running into the end mounts. But if there is more room for travel, an additional offset may be added to the limits.

\n

First, determine the offset of the sensor (e.g. by measurement or taken from the information of the printable sensor bracket found many share sites on the internet; Thingiverse is a good source for such brackets).

\n
\n

HINT for Marlin 2.x

\n

Note the version (branch) of Marlin! The answer is written at the time of the 1.1.x branch and as such is perfectly valid for the latest 1.1.x (1.1.9) version. For the 2.0.x branch of Marlin, different constant names are in use, and a different strategy is used (more simple for setting up). The constant names are not found in the Configuration.h, the answer (and the theory) is still helpful. Constant probe offset values are now found in Configuration_adv.h:

\n
MIN_PROBE_EDGE_LEFT\nMIN_PROBE_EDGE_RIGHT\nMIN_PROBE_EDGE_FRONT\nMIN_PROBE_EDGE_BACK\n
\n

HINT for Marlin >= 2.0.6

\n

Since version2.0.6 MIN_PROBE_EDGE_* has been renamed again.
\nNow it is:

\n
PROBING_MARGIN_LEFT\nPROBING_MARGIN_RIGHT\nPROBING_MARGIN_FRONT\nPROBING_MARGIN_BACK\n
\n

Note that in Marlin 1.1.x the boundaries are set in absolute positions while in Marlin 2.0.x it is described in offset values from the bed edge.

\n
\n

Sensor Offset

\n

The position of the sensor is set using the following constants:

\n
\n#define X_PROBE_OFFSET_FROM_EXTRUDER [XXX]   // X offset: -left  +right  [of the nozzle]\n#define Y_PROBE_OFFSET_FROM_EXTRUDER [YYY]   // Y offset: -front +behind [the nozzle]\n
\n

where [XXX] and [YYY] are offset values that specify the center of the sensor with respect to the nozzle. If both are positive values, the sensor is located in the back-right, if both negative, the position is front-left (as seen from the front of the machine using the definition in the Marlin configuration file). The other positions that are possible are the back-left and the front-right (one positive and one negative value).

\n
\n

HINT for Marlin 2.x

\n

Note that in Marlin 2.x, these constants are replace by an array definition:

\n
#define NOZZLE_TO_PROBE_OFFSET { XXX, YYY, ZZZ }\n
\n

Where ZZZ is the Z offset. If you use the old constants, the sanity check upon compiling will throw an assertion that these constants are not in use anymore and should be removed.

\n
\n

Second, let's set the sensor area limits!

\n

Sensor Area Limits

\n

In the following images, the nozzle, the sensor and the offsets are defined by:\n\"Legend

\n

There are four possible positions of the sensor, this results in the following schematics for the sensor area limits (transparent red area):

\n

1. Sensor at the back-right:

\n

\"Probe

\n

This implies that the sensor can reach the back and the right borders, so the limits are set by:

\n
\n#define LEFT_PROBE_BED_POSITION (X_PROBE_OFFSET_FROM_EXTRUDER)\n#define RIGHT_PROBE_BED_POSITION (X_BED_SIZE)\n#define FRONT_PROBE_BED_POSITION (Y_PROBE_OFFSET_FROM_EXTRUDER)\n#define BACK_PROBE_BED_POSITION (Y_BED_SIZE)\n
\n

Note that if a minimum probe offset is defined by #define MIN_PROBE_EDGE [value] in the configuration (to account for sensor width/dimension), the bed limits are changed resulting in (where t = MIN_PROBE_EDGE):

\n

\"Probe

\n

Limits are then set by:

\n
\n#define LEFT_PROBE_BED_POSITION (X_PROBE_OFFSET_FROM_EXTRUDER + MIN_PROBE_EDGE)\n#define RIGHT_PROBE_BED_POSITION (X_BED_SIZE - MIN_PROBE_EDGE)\n#define FRONT_PROBE_BED_POSITION (Y_PROBE_OFFSET_FROM_EXTRUDER + MIN_PROBE_EDGE)\n#define BACK_PROBE_BED_POSITION (Y_BED_SIZE - MIN_PROBE_EDGE)\n
\n

In the following sensor position placements, this offset for MIN_PROBE_EDGE will be accounted for.

\n

2. Sensor at the back-left:

\n

\"Probe

\n

This implies that the sensor can reach the back and the left borders (for zero MIN_PROBE_EDGE), so the limits are set by:

\n
\n#define LEFT_PROBE_BED_POSITION (MIN_PROBE_EDGE)\n#define RIGHT_PROBE_BED_POSITION (X_BED_SIZE  + X_PROBE_OFFSET_FROM_EXTRUDER - MIN_PROBE_EDGE)\n#define FRONT_PROBE_BED_POSITION (Y_PROBE_OFFSET_FROM_EXTRUDER + MIN_PROBE_EDGE)\n#define BACK_PROBE_BED_POSITION (Y_BED_SIZE - MIN_PROBE_EDGE)\n
\n

3. Sensor at the front-left:

\n

\"Probe

\n

This implies that the sensor can reach the front and the left borders (for zero MIN_PROBE_EDGE), so the limits are set by:

\n
\n#define LEFT_PROBE_BED_POSITION (MIN_PROBE_EDGE)\n#define RIGHT_PROBE_BED_POSITION (X_BED_SIZE  + X_PROBE_OFFSET_FROM_EXTRUDER - MIN_PROBE_EDGE)\n#define FRONT_PROBE_BED_POSITION (MIN_PROBE_EDGE)\n#define BACK_PROBE_BED_POSITION (Y_BED_SIZE + Y_PROBE_OFFSET_FROM_EXTRUDER - MIN_PROBE_EDGE)\n
\n

4. Sensor at the front-right:

\n

\"Probe

\n

This implies that the sensor can reach the front and the right borders (for zero MIN_PROBE_EDGE), so the limits are set by:

\n
\n#define LEFT_PROBE_BED_POSITION (X_PROBE_OFFSET_FROM_EXTRUDER + MIN_PROBE_EDGE)\n#define RIGHT_PROBE_BED_POSITION (X_BED_SIZE  - MIN_PROBE_EDGE)\n#define FRONT_PROBE_BED_POSITION (MIN_PROBE_EDGE)\n#define BACK_PROBE_BED_POSITION (Y_BED_SIZE + Y_PROBE_OFFSET_FROM_EXTRUDER - MIN_PROBE_EDGE)\n
\n

This should have tackled the basics for defining the sensor area limits. It becomes a little more complicated when there is extra travel space. A very easy use of extra travel space can be found in the configuration file; e.g:

\n
\n// Travel limits (mm) after homing, corresponding to endstop positions.\n#define X_MIN_POS [XX]\n#define Y_MIN_POS [YY]\n
\n

where [XX] and [YY] are offset values from endstop to origin (orange arrows represent X_MIN_POS and Y_MIN_POS):

\n

\"Probe\nResults in offset constants for a back-right probe:

\n
\n#define LEFT_PROBE_BED_POSITION (X_PROBE_OFFSET_FROM_EXTRUDER + X_MIN_POS + MIN_PROBE_EDGE)\n#define RIGHT_PROBE_BED_POSITION (X_BED_SIZE - MIN_PROBE_EDGE)\n#define FRONT_PROBE_BED_POSITION (Y_PROBE_OFFSET_FROM_EXTRUDER + Y_MIN_POS + MIN_PROBE_EDGE)\n#define BACK_PROBE_BED_POSITION (Y_BED_SIZE - MIN_PROBE_EDGE)\n
\n

For all other options, a similar addition of the homing offsets can applied.

\n

A similar addition is possible if the print head is able to travel further on the X or Y axis at the other ends of the axes.

\n
\n

Thoughts for solution for Marlin 2.0.x

\n

As I mentioned earlier, Marlin 2.0.x uses bed edge offsets rather than absolute positions. In analogy of the previous graphs we can draw a bed limits diagram (the example below assumes a back-right mounted probe!):

\n

\"Marlin

\n

Note that the offsets from the edge need to be defined, to properly do this we need access to the definition of the nozzle offset:

\n
#define NOZZLE_TO_PROBE_OFFSET { XXX, YYY, ZZZ }\n
\n

As this is the part that defines the array values, you first need to make an array (note that this is a simple solution that many people should be able to understand with limited programming skills, more elegant solutions use the XYZ struct to access the X, Y or Z properties):

\n

Note that Marlin 2.0.x automatically adjusts your probe area based on the defined offset and the MIN_PROBE_EDGE defined for all 4 sides of the bed.

\n

see:

\n
#if PROBE_SELECTED && !IS_KINEMATIC\n  #define MIN_PROBE_EDGE_LEFT MIN_PROBE_EDGE\n  #define MIN_PROBE_EDGE_RIGHT MIN_PROBE_EDGE\n  #define MIN_PROBE_EDGE_FRONT MIN_PROBE_EDGE\n  #define MIN_PROBE_EDGE_BACK MIN_PROBE_EDGE\n#endif\n
\n
\n" }, { "Id": "8159", "CreationDate": "2019-02-04T02:40:16.637", "Body": "

When I run prints on my Creality CR-10S Pro 3D printer (using Ultimaker Cura as slicer), I set the heated bed to around 70 °C for PETG. after the first few layers, the heated bed set point changes to 0 °C.

\n\n

Is it normal for a 3D printer to turn the heated bed off during a print, or is this a problem?

\n\n

I have had some parts warping, and wonder if this is a potential cause.

\n", "Title": "Creality CR-10S Pro 3D printer heated bed turns off after first few layers: normal?", "Tags": "|heated-bed|creality-cr-10|heat-management|warping|", "Answer": "

I have a new CR10 max.

\n

I can't find any adjustments for the bed temperature settings other than the basic ones in the Creality slicer. I'm not sure if the latest software has got this facility.

\n" }, { "Id": "8163", "CreationDate": "2019-02-04T22:11:56.383", "Body": "

Heads up: I'm not good with electronics and only have a vague idea of it's inner workings.

\n\n

I have a E3D V6 Extruder rated for 24 V, that i plan to use in my 3D printer. Will there be any problems with it if powered by 12 V? Will it take longer to heat up? Will it be able to heat up enough to melt PLA? Will it work at all for that matter?\nIf there are any other quirks or potential problems that I overlooked, please let me know.

\n", "Title": "Can an extruder rated for 24 V work with 12 V ATX power supply?", "Tags": "|extruder|electronics|power-supply|", "Answer": "

It will take longer to heat up. However if you use a boost converter (like I did on my Anet A8 when I upgraded to a Maxiwatt 24 V hot end), then it will work just fine without any further adjustments; to the power supply or the gauges of the wire etc. I set the boost from 12 to 24 volts. Now my A8 heats up in 56 seconds!

\n" }, { "Id": "8166", "CreationDate": "2019-02-05T03:19:35.577", "Body": "

[Edit: My specific question is firmware error or standard layer shifting. using 2 different versions of Cura were getting quite different results from the same stl file.]

\n\n

Per the image below with Cura 15.4 the centered print both shifted to the bottom of the print bed for the first few passes and then did not build the proper internal structure of the bracket. the top loops were then skewed to the top of the bed. The 2nd print in the corner was using Cura 3.1 and the first few passes like usual skewed off towards the bottom but it seemed to start printing normally after that. I am trying to determine if it is a hardware calibration issue or bad firmware on the printer. Any suggestions on what's going on would be appreciated. For some reason the image flipped 180° when posting.

\n\n

Center part completed its print and should be about an inch or so tall. It was not even able to build the internal structure like the 2nd one had started to do. the 2nd one I stopped after about 10 min since it clearly showed different behavior. from the layer shifting links the most likely cause would be over current stepping.

\n\n

\"Failed

\n", "Title": "Print layer shifting on Alunar M508 Prusa i3 clone", "Tags": "|troubleshooting|y-axis|print-failure|", "Answer": "

This effect is called layer shifting . Now that you know what it is called you could look at some other solutions fixing this issue; e.g. here, here or here. The answers of this question describe best what is causing this. Usually (most of all the cases) it means that your belts are not tight enough.

\n\n

An edit of the question shows that the effect happens when a different version of Ultimaker Cura is being used. In such cases you need to check the speeds and acceleration settings. Too high values may lead to skipping steps causing layer shifting. Be sure it is not a mechanical issue, also note that the nozzle does not get caught up by the uneven tape on the bed. In that effect, you may want to look at this unaswered question.

\n" }, { "Id": "8177", "CreationDate": "2019-02-06T13:40:47.473", "Body": "

I\u2019m building a 3D printer and need to understand what calibration and bed leveling is, and I was surprised to see there isn\u2019t a clear explanation on the internet anywhere. Does bed leveling compensate for an uneven bed? So then, what does calibrating do? Is it the same thing?

\n", "Title": "What exactly does calibrating a 3D printer do?", "Tags": "|calibration|bed-leveling|", "Answer": "

First, welcome to the 3D Printing Stack Exchange!

\n\n

On bed leveling

\n\n

Bed leveling, or more accurately bed tramming, adjusts the bed so that it is even in relation to the print head. Typically it is done by sliding a paper between the nozzle and the bed when Z = 0, or the print head is at its lowest. The amount of friction should be similar to the amount of friction of a sheet of paper between two magazines (recommended by Tom Salander on YouTube I believe). One sheet of paper is about 0.1 mm, which is a common gap and allows for good adhesion of the deposited plastic, and typically the gap is measured at the four corners and the center of the bed.

\n\n

But wait, there's more!

\n\n

Mesh bed leveling can account for a physically uneven print bed (like a bowing glass sheet) and can be done manually like above or one can mount a sensor so that everything is automatic. The print bed is broken down into a grid (typically 3x3 or 5x5) and the Z height adjusts so that the gap is 0.1 mm at each point in the grid. A mesh is then made and stored in the printer. As the printer prints it makes the necessary variations in height so that the print is flat. Please note that the bed should be leveled as best as possible before setting up the mesh as it will vastly improve the resulting mesh. Mesh leveling is primarily software. The actual level of the bed remains unchanged, thus needing to level it prior to setting up the mesh.

\n\n

Other calibrations

\n\n

Bed leveling (including mesh bed leveling) is one aspect of calibration. Other important factors to tune in are temperatures of the nozzle and the bed. For example, PLA has a temperature range of 180-230 \u00b0C but each printer is different and may need to print at a higher or lower temperature for best results. Or the printer reports a temperature of 200 \u00b0C but the thermistor is off and is actually printing at 190 \u00b0C. It's up to the user then to compensate if they want/need to print at an actual 200 \u00b0C.

\n\n

Other settings to tune are print speeds, acceleration and jerk, retraction, and extrusion. Faster print speeds can result in poorer quality prints, namely ghosting and under extrusion. Acceleration/jerk works in the same vein since it directly affects speed. Retraction helps with stringing. If the retraction settings are too low then there will be stringing on the model, if it's too high then there can be under extrusion and heat creep. Adjusting the extrusion factor will tell the printer to push more plastic out or push less. The higher the factor the more plastic is pushed. IMO best practice is to leave the factor at 1 in the slicer and calibrate the stepper motor itself (as can be seen here)

\n\n

Calibration models

\n\n

There are several calibration models on thingiverse that can show common print issues. Some of the popular ones are:

\n\n\n\n

Happy printing!

\n" }, { "Id": "8180", "CreationDate": "2019-02-06T15:57:30.457", "Body": "

I have a Creatorbot 3D printer made by 3D PrinterWorks. Their website appears to be down, as well as their Facebook page. To me it appears they are no longer around.

\n\n

I've installed Slic3r as 3D PrinterWorks has recommended in the handbook but cannot download the settings for this from the 3D PrinterWorks website, since that is down.

\n\n

Does anyone know where I can get the Slic3r configuration file for the Creatorbot?

\n", "Title": "Where to get the Slic3r configuration files for the 3D PrinterWorks Creatorbot printers?", "Tags": "|slic3r|3d-printerworks|", "Answer": "

The configuration files were on a flash drive that came with the printer. That being said, here is the file that was located on my flash drive for Slic3r (denoted as deprecated on March 1, 2017).

\n\n

This is for a Creatorbot Pro II.

\n\n

https://pastebin.com/j1dkSt8f

\n\n

Save it as Slic3r_config.ini and import it in to Slic3r.

\n\n

Good luck!

\n" }, { "Id": "8182", "CreationDate": "2019-02-06T18:22:27.210", "Body": "

Is it possible or recommended to print with ASA filament with printer that has no enclosure? I Know that ASA is better than ABS, but is it that better that enclosure is not needed?

\n", "Title": "Print ASA without enclosure", "Tags": "|filament|enclosure|asa|", "Answer": "

According to MatterHackers, Simplify3D, and 3D Insider an enclosure is not necessary but highly recommended as ASA, like ABS, is susceptible to drafts and can warp. A heated bed is necessary (somewhere between 90-110 \u00b0C), and little-to-no cooling fan used. The enclosure helps ensure adhesion and a consistent temperature. ASA also has fumes and an enclosure can help contain unwanted smells.

\n\n

MattHackers has a video (Step 4 in the link) on making an enclosure with some kind of plastic sheeting or trash bag (I didn't watch it). There are videos on YouTube on using cheap Ikea side tables and acrylic sheets to make an enclosure if you want a more permanent solution.

\n" }, { "Id": "8184", "CreationDate": "2019-02-06T18:45:53.690", "Body": "

I have a Monoprice Maker Select Plus, currently using Ultimaker Cura 3.6.0 with the default settings for a Wanhao Duplicator i3 Plus. Right now when a print finishes, the bed retreats towards the back of the machine. I'd rather present the bed forward for easier part removal.

\n\n

Here is my ending G-Code:

\n\n
\nM104 S0      ;extruder heater off \nG91          ;relative positioning\nG1 E-1 F300  ;retract the filament a bit before lifting the nozzle, to release some of the pressure\nG1 Z+0.5 E-5 X-20 Y-20 F{travel_speed} ;move Z up a bit and retract filament even more\nG28 X0 Y0    ;move X/Y to min endstops, so the head is out of the way\nM84          ;steppers off\nG90          ;absolute positioning\n
\n\n

It looks like I need to change that G28 line third from the bottom, but I'm not sure what to change it to. I've not yet done any g-code manipulation of my own. I don't know what units it's using, and it looks like it still has relative positioning, so even then I don't know it's a good idea to just set it for the max size of the bed.

\n\n

So how can I change this code to move the bed as I want?

\n", "Title": "Move print bed to front at the end of a print rather than the back", "Tags": "|ultimaker-cura|g-code|", "Answer": "

Note that this reference states that:

\n\n
\n

Because the behavior of G28 is unspecified, it is recommended not to automatically include G28 in your ending GCode. On a Cartesian this will result in damaging the printed object. If you need to move the carriage at the completion of a print, use G0 or G1.

\n
\n\n

So you need to use a G0 or G1 move.

\n\n

When using Ultimaker Cura (like many other slicers), there is built in functionality known as keywords with a complete list found here.

\n\n

The keyword machine_depth is the one that is of use to you, embed this in your end G-code in between curly brackets and it will expand to the bed size of your machine (replacing G28 X0 Y0):

\n\n

G1 X0 Y{machine_depth}

\n\n

For me this compiles to (e.g. for my coreXY printer):

\n\n

G1 X0 Y300

\n\n

To set the speed, just add the following command prior to the one above:

\n\n

G1 F2500

\n\n

Adding this line before the actual move ensures that the speed is constant, if F2500 would have been included in the move command (like G1 X0 Y{machine_depth} F2500), this defines the end speed, it would start moving at the last speed value prior to the move.

\n\n

This results in adding the following lines in your endscript:

\n\n
\nG1 F2500\nG1 X0 Y{machine_depth}\n
\n" }, { "Id": "8188", "CreationDate": "2019-02-06T21:04:15.697", "Body": "

This is probably the most basic of questions. When I was shopping for a printer I saw models that would list a slew of certain plastics that the printer can print while other printers (different suppliers) would only list PLA/ABS. At the same time, the instruction manual that came with my printer only listed PLA and ABS.

\n\n

What is actually used to determine the possible material (other than diameter)? I assume that if the nozzle can reach the required temperature to melt the plastic and the bed can reach the necessary temperature to maintain bed adhesion, then my printer can print that plastic. Am I correct?

\n\n

For example, my nozzle can reach 250 \u00b0C, but the bed can barely reach 80 \u00b0C so I can print with ABS but not with ASA (since it needs at least 90 \u00b0C on the bed. However, if I switch out the heating element for something beefier then that opens up anything that needs a hotter bed. Right?

\n\n

Let's assume that I have the means to change nozzles and extruder type to fit the need of the filament. The goal of this question is to limit to one or two limiting factors of the printer without major modification.

\n", "Title": "What kind of filaments can I use?", "Tags": "|filament-choice|", "Answer": "

There are other factors besides temperature. Certain \"soft\" filaments won't go well through all extruders, and some other filaments with wood/metal particle additives don't go well through every extruder type and can cause clogging. Even people who do tend to print a lot of these materials will often use a specific nozzle just for those filaments.

\n\n

That said, you still might be able to print even with something like ASA that seems unsupported. You can do this by taking additional measures for bed adhesion, or upgrading or swapping your nozzle or hot end.

\n\n

If you can print PLA and ABS, you can probably also print so-called PLA+ and PETG, at a minimum.

\n" }, { "Id": "8191", "CreationDate": "2019-02-07T15:16:44.943", "Body": "

I installed a BLTouch on my Ender 3 following this video: https://www.youtube.com/watch?v=sUlqrSq6LeY.

\n\n

What I noticed too late was that when installing the BLTouch, I swapped the brown and red wires followed by connecting it to the mainboard.

\n\n

Then I realized that the printer wouldn't turn on. After reading some of the comments, I swapped back the wire and I could see the BLTouch do its self-test.

\n\n

However, it seems that I need to plug into the USB before the LCD + motors can work.

\n\n

So is my mainboard fried? Or how should I fix it?

\n", "Title": "Ender 3 needs Power Supply AND USB to work", "Tags": "|troubleshooting|creality-ender-3|bltouch|", "Answer": "

It seems like I blew the regulator based on the comment in the Reddit post: https://www.reddit.com/r/ender3/comments/ao5m5b/ender_3_can_only_be_powered_on_by_psu_and_usb/

\n" }, { "Id": "8194", "CreationDate": "2019-02-07T20:08:58.047", "Body": "

I've tried printing the Basic Stringing Test on my CR-10S using PETG (250\u00a0\u00b0C nozzle, 80\u00a0\u00b0C bed) with different retraction distances leaving all the rest unchanged.

\n

In the image below you can see 0\u00a0mm, 5\u00a0mm and 10\u00a0mm retractions (in order left-to-right).

\n

\"Basic

\n

The result stunned me because I would have imagined that raising retraction distance would have diminished the stringing.

\n

Why is that?

\n", "Title": "Question about retraction distance and stringing/print quality with PETG", "Tags": "|creality-cr-10|retraction|petg|", "Answer": "

You cannot endlessly increase the retraction distance, doing so leads to different problems as you encountered. As a rule of thumb, the retraction distance should not exceed the length of your nozzle. Depending on the type of extruder, many printers use a value between 2 and 7\u00a0mm (e.g. the Ultimaker Cura retraction length is 6.5\u00a0mm at 25\u00a0mm/s, this is for a Bowden style extruder).

\n

In fighting stringing (in which PETG is very subjected to happening) retraction length is only one of the parameters that you need to adjust to optimize the printing process. Other important parameters (besides retraction length) for stringing are:

\n\n

Furthermore, the type of extruder (direct or Bowden) is also important and other parameters that may reduce the stringing are:

\n\n

Usually it suffices to tune the first parameters. As you see in your results, the retraction of 5\u00a0mm is close to the result you would expect, increasing it, may lead to clogs and in your case leads to a sort of stringing. This latter result is probably due to the fact that the filament does not reach the print in time (you definitely see under-extruded cylinder columns), while leftover pressure after printing the column layer oozes some of the filament creating strings.

\n

Just play with the retraction length in steps of 0.25\u00a0mm (around the 5\u00a0mm) and you may try to change the speed with 5\u00a0mm/s increments if necessary. Also experiment with the flow modifier.

\n" }, { "Id": "8204", "CreationDate": "2019-02-08T21:15:36.847", "Body": "

I have a Rostock Max V2, and I've added a second extruder going into a y-splitter into a single nozzle on my printer. I have both extruders working correctly, but I'm having trouble tuning the retraction settings to prevent stringing when I switch between extruders during a print. My system is essentially identical to the setup seen here. However, I can't get my printer to retract as cleanly as the one in the video

\n\n

What I'm trying to avoid is the long, thin \"tail\" that forms when retracting the filament from the hot end. That \"tail\" binds the other filament during the switch and makes the extruder grind a hollow spot on the filament.

\n\n

I've had limited success tuning my retraction settings, but I find that I need different settings for different extrusion speeds. For example, after an extrusion like G1 E20 F240 a 3mm retraction, 3mm extrusion, then a fast retraction creates a nice, clean break (this routine is recommended here by kraeger on the SeeMeCNC forums). However, after an extrusion like G1 E20 F900 I have to use longer retractions to get a clean break. I think this might have to do with the filament acting like a spring inside the bowden tube. It would make sense to me that the harder you push the filament, the more you need to pull back to compensate for the pent-up spring force.

\n\n

Here's my question: Is there a way to read the value of the extrusion speed, essentially the \"F\" term from the gcode commands, and change my retraction routine accordingly.

\n\n

Example pseudocode:

\n\n
If F value < 500 Then do short retraction\nIf F value > 500 AND F value < 1000 Then do medium retraction\nIf F value > 1000 Then do long retraction\n
\n\n

I'm using the tool change script feature in Simplify3D to store the tool change code.

\n", "Title": "Use Gcode Extrusion Speed in Calculations", "Tags": "|extrusion|simplify3d|multi-material|retraction|", "Answer": "

I don't think you're going to find either a firmware feature or a slicer feature that handles specifically what you want to do. The slicer would probably be the best place to put it, and I'd recommend maybe opening a feature request ticket with Ultimaker, because that sounds like an awesome feature.

\n\n

That being said, there's nothing stopping you from post-processing your GCode file after it's been generated. If you're experienced with python at all, that's the place I'd recommend you start.

\n\n

You'll probably want to do it via the following:

\n\n
    \n
  1. Find the first line number that does a retraction.
  2. \n
  3. Sum up all the extrusion distances between that line and the starting point (the beginning of the file)
  4. \n
  5. Replace the retraction distance and feedrate with whatever your short/medium/long retraction settings are
  6. \n
  7. Store that line number as your new starting point
  8. \n
  9. GOTO 1.
  10. \n
\n\n

If you're using Slic3r, there's actually a post-processing script function built into the app itself, you just need to write the script and give it to the application to make the whole process hands-off. For other slicers you'll probably just have to run the script manually between slicing and printing.

\n" }, { "Id": "8206", "CreationDate": "2019-02-09T02:29:53.833", "Body": "

Using CuraEngine with my Ender 3, I'm getting what I'd call inconsistent inner and outer dimensions - for example, a nominally 3 mm peg is significantly larger than a 3 mm hole, and it takes dimensions something like 2.9 mm for the peg and 3.1 mm for the hole to get them to fit. Is this level of error normal? Is it caused by overextrusion, or does CuraEngine run its paths along the curve of the slice rather than offset by approximately half the nozzle width inside the sliced region? The magnitude of the error being almost exactly 0.2 mm, which is half of the 0.4 mm nozzle diameter, makes me wonder if it's the latter.

\n", "Title": "Inconsistent inner/outer dimensions", "Tags": "|print-quality|ultimaker-cura|slicing|creality-ender-3|", "Answer": "

Cura does correctly account for line extrusion widths wheen positioning the lines, and attempting to fix this with negative xy_offset was a mistake that led to lots of problems: in some cases, it completely eliminated tiny components of the model and left gaps in layers. At some point after asking this question, I did a new test with 8mm peg and hole, and I was actually able to force the 8mm peg into the hole (but not remove it) using tools, without cracking the parts, so I think past tiny sizes where dimensional accuracy is very difficult to achieve, everything is just about right.

\n" }, { "Id": "8210", "CreationDate": "2019-02-09T14:23:58.727", "Body": "

I've been trying to find a 3D printer filament which would not release any chemicals if in contact with heated water for a substantial amount of time. So far, I've easily ruled out both PLA and ABS, as they're not considered food safe from what I can find. I have found PETG filament, which seems to be food safe.

\n\n

My question is: \"Is there's anything special you'd have to do to make sure the print is food safe, or as in my case, to make sure it's safe for usage in a mug?\".

\n\n

I will be using a steel extruder as brass ones may contain lead.

\n", "Title": "Is PETG filament food safe?", "Tags": "|filament|safety|food|petg|", "Answer": "

As far as PET-based filaments go, most of them are FDA approved. So yes, \"food safe\". It's the process of 3D printing that ruins that. all the little layers trap just about everything and are hard to clean. If you wanted to boil your part, that might work, but you'd be better off buying a food-safe coating spray for your parts.

\n" }, { "Id": "8214", "CreationDate": "2019-02-10T14:41:57.790", "Body": "

I currently have my printrbot with a LJ12A3-4-Z/BX-5V Inductive Proximity Sensor ( 61mm M12 DC 5V NPN NO 300mA)

\n\n

That detects my aluminium bed at around 1.2 mm distance. While it works, if the printer does the smallest blob the probe crashes into it.

\n\n

I am considering replacing the probe by the M18 version LJ18A3-8-Z/BX-5V (\nM18 8mm DC 5V NPN NO 300mA).

\n\n

I haven't been able to find real specs for these probes (apart from what the sellers put in eBay or AliExpress - which I do not trust).

\n\n

Will I get a larger detection distance with the M18 version of the 5 V probe? (I wonder if the M18 version is exactly the same M12 version with a larger casing).

\n\n

Or should I move to the 6-36 V versions of the probes - I do not want to do that to not modify my wiring.

\n", "Title": "M12 vs M18 V5 inductive probe", "Tags": "|z-probe|inductive-sensor|", "Answer": "

The larger the diameter and the higher the voltage the larger the detection distance. You do not need to do much wire modification if you use an optocoupler, see this anwer.

\n" }, { "Id": "8216", "CreationDate": "2019-02-10T15:26:53.770", "Body": "

So yesterday I got my power supply in the mail and I thought, let's check it out!

\n

I put all the wires in correctly (as shown in this YouTube video, Anet A8 power supply unit fuse blown, I just searched really quickly, this person has the same problem as me) and "BANG", it blew.

\n

\"Image

\n

Luckily the only thing broken on it is the fuse, for which I can get a new one, but my main question is .. why? why did it blow out? Did I wire it up wrong? The 12\u00a0V output goes to a RAMPS 1.4 board which is working correctly.

\n

It worked the first time I plugged it in; the little green light went on. The second time a nice "big bang". When I tested it the first time, I didn't add the 12\u00a0V wires to the RAMPS board yet. When I did, the fuse of the PSU blew out.

\n

The board is a RepRap RAMPS 1.4, which works fine when connected via USB to laptop.

\n

Upon closer inspection I found a burned out resistor in the circuit of the power supply, maybe this caused the bang?

\n

BIG update!

\n

I opened it up and flipped the board, and noticed that there was a big ol' blob of tin connected to one of the pins I connected my wires to. I am very sure that this blob shouldn't connect to that pin at all, hence shorting my board.

\n

Is this the cause?

\n

\"Close

\n

\"Close

\n

Here is a link to the webpage I bought it from: Anet\u00ae 12\u00a0V 20\u00a0A 240\u00a0W Power Supply Dual-input Centralized Power Monitoring For 3D Printer

\n

Here is how I wired it up at the PSU side:

\n

\"Power

\n

At the RAMPS side:

\n

\"RAMPS

\n

This shows the internals of the PSU, it clearly shows burn marks:

\n

\"Overview

\n", "Title": "Why did my power supply fuse blew out on my Anet A8?", "Tags": "|ramps-1.4|anet-a8|power-supply|", "Answer": "

Unless you know what you're doing, do not remove the covers from a PSU. The components can store a lethal charge long after power is removed, even if the fuse is blown.

\n

My guess is something loose (now vapourised) in the PSU. Electronics have a tendancy of failing either quite soon after manufacture, or lasting reasonably well. This is described with the Bathtub Curve.

\n

Looking at the failure, there was a significant short circuit somewhere - quite likely within the PSU itself. This doesn't mean you can be sure that the RAMPS board survived, but it has a reasonable chance of not being harmed.

\n

There is no significant chance of the PSU failing like that simply due to an excessive load in its output - unless it was overloaded for a significant period of time. Generally these PSUs will have some degree of overload protection even if they are not explicitly designed to survive an overload.

\n" }, { "Id": "8218", "CreationDate": "2019-02-10T20:28:50.273", "Body": "

I printed a temperature calibration cuboid for Hatchbox 1.75 mm PLA, in 1 °C increments from 180 °C to 190 °C.

\n\n

I have two questions related to this image:

\n\n

\"Temperature

\n\n\n\n

Printed on a RepRapGuru Prusa MK2 clone.

\n", "Title": "What calibration options should I look into given these defects", "Tags": "|calibration|", "Answer": "

Your print does not suffer from layer shifts as you call them. This uneven layer deposition is typically caused by the (positioning) accuracy of your printer. All-in-all, this print does not look so bad. You would get better quality prints on a different style of a printer; most high-end printers have a lowering platform instead of a forth and back moving platform.

\n\n

Although 180 °C is at the low side of printing PLA (usually it starts at about 185 °C), the print does look okay. However, looks can deceive, it does not say anything about structural integrity (layer bonding).

\n\n

Note, to calibrate the temperature using a temperature tower, you need a different calibration test print, preferably one that tests overhang. This latter is usually far more important as there is normally not much to see at the walls, you need a slanting part in the print or an overhang to determine the optimal temperature.

\n" }, { "Id": "8222", "CreationDate": "2019-02-11T12:07:57.223", "Body": "

I have just built my first 3D printer, and I am having some problems. It is a Cartesian based 3D printer, using Marlin firmware and Pronterface software. My problem is homing the 3D printer. I do not have a probe to calibrate the z offset but I have all 6 endstops. The problem is that the \"ZMIN\" endstop isn't precise enough and my hotend is always either too far from or too close to the heatbed. Is there a way to manually set the home position, so when I start the printer, It just starts printing and it doesn't have to home again; Or maybe some other way to set the correct offset. It would also be helpful if I could use just the \"ZMAX\" plug, and then manually set the minimum Z position using a piece of paper.

\n", "Title": "Homing the 3D printer", "Tags": "|marlin|diy-3d-printer|z-axis|homing|", "Answer": "

You can reconfigure marlin to use the MAXZ endstop instead of MINZ in Configuration.h, and then reflash it to your printer.

\n\n

https://github.com/MarlinFirmware/Marlin/blob/1.1.x/Marlin/Configuration.h#L877

\n\n

Now this is highly unusual, as most printers only have MIN endstops. If you post a pic of your faulty MINZ endstop, we can probably figure out how you could fix it. Zipties come to mind as a likely solution.

\n" }, { "Id": "8228", "CreationDate": "2019-02-11T21:01:57.003", "Body": "

The Prusa i3 MK3 displays the remaining printing time right from the beginning.

\n\n
\n

Image: 3:38 remaining after 0% printing

\n
\n\n

\"Prusa

\n\n

From about 8 prints until now, I would say that this time is quite reliable.

\n\n

Octoprint, however, does not display the remaining time initially and is then far off.

\n\n
\n

Image: Octoprint saying something like \"too uncertain\"

\n
\n\n

\"Octoprint

\n\n
\n

Image: Octoprint says 8.5 hours remaining

\n
\n\n

\"Octoprint

\n\n

Why doesn't Octoprint just take the remaining time from the printer and display that? Or can I change a setting so that it does?

\n", "Title": "Why can my printer display remaining time but OctoPrint can't?", "Tags": "|octoprint|", "Answer": "

Octoprint is a generic application that has to work with a rather wide variety of printers and printer firmwares. The time estimation that is shipped with Octoprint by default is a very basic method that doesn't rely on any specific printer features. This also makes it kind of useless in some cases, and not very accurate.

\n\n

The estimate that the Prusa i3 Mk3 shows is not actually done by the printer, it is embedded in the GCode generated by Slic3r PE. There are M73 commands added that tell the printer how far along the print job is.

\n\n

As for why Octoprint doesn't do this by default, the major reason is likely that this method only works for specific printers and slicers, and only works well if the slicer can actually do good estimates for a particular printer. This is the case for the Prusa because it supports this feature and the slicer is maintained by them and has enough information to make good estimates. But this is not the case for all printers.

\n\n

The feature is also not entirely standardized as far as I understand, e.g. Prusa uses slightly different M73 commands to give estimates for normal and silent mode.

\n\n

There are plugins that can read the M73 estimates, you could try that. I never tried them myself, this plugin seems to do what you want from a quick glance.

\n" }, { "Id": "8229", "CreationDate": "2019-02-11T21:34:42.023", "Body": "

My i3 MK3 is printing very well for solid parts of an object, but it messes up with infill.

\n\n

\"Infill

\n\n

As you can see in the image, the infill is broken into pieces and bends to that the surface becomes rough and the nozzle touches it next time it moves there, which probably does not make things better.

\n\n

I've read the Prusa problem page, where they suggest three solutions:

\n\n\n\n

I'm printing with 20% infill and I believe that this worked before at the same speed, so I'm tempted to say that both remaining options are options, but it should work without them.

\n\n

It also seems to me as if the infill is thinner that ordinary walls. Is the extrusion speed lower in case of infill? Is there a way of changing the extrusion speed for infill?

\n\n

Side note: my printer always tells me to upgrade to the latest firmware 3.5.1. Since I have just received the printer back from a warranty repair after the last firmware upgrade, I don't want to upgrade the firmware. Could the issue be related to firmware?

\n\n

Infos requested from comments:

\n\n

I'm basically using the Slic3r default settings for PETG. The only thing I adjusted is the temperature, since I'm using HDGlass PETG and there was a recommended temperature written on the spool. Relevant settings seem to be

\n\n\n", "Title": "Infill is rough - how to enhance?", "Tags": "|prusa-i3|infill|", "Answer": "

I have dealt with this on infill as well on multiple MK3s.

\n\n

However, it was not the speed itself, but the hot end having difficulty extruding enough to keep up with the infill.

\n\n

Some things to try:

\n\n\n\n

Good luck!

\n" }, { "Id": "8242", "CreationDate": "2019-02-12T13:52:20.063", "Body": "

I don't understand what's wrong with my G-code. I have set the printing temperature to 195\u00a0\u00b0C but when I try to print, the target temperature is always 0\u00a0\u00b0C and printing never starts.

\n

\"enter

\n

I have tried changing material preset to different PLA profiles but that didn't help at all.

\n

G-code:

\n
;FLAVOR:RepRap\n;TIME:1736\n;Filament used: 0.676205m\n;Layer height: 0.1\n;Generated with Cura_SteamEngine 3.4.1\nT0\nM190 S50\nM104 S195\nM109 S195\nM82 ;absolute extrusion mode\nG21 ;metric values\nG90 ;absolute positioning\nM82 ;set extruder to absolute mode\nM107 ;start with the fan off\nG28 X0 Y0 ;move X/Y to min endstops\nG28 Z0 ;move Z to min endstops\nG1 Z15.0 F9000 ;move the platform down 15mm\nG92 E0 ;zero the extruded length\nG1 F200 E3 ;extrude 3mm of feed stock\nG92 E0 ;zero the extruded length again\nG1 F9000\nM117 Print0ng...\nM83 ;relative extrusion mode\nG1 F1500 E-6.5\n;LAYER_COUNT:96\n;LAYER:0\nM107\nG0 F600 X90.425 Y93.152 Z0.3\n;TYPE:SKIRT\nG1 F1500 E6.5\n...\n
\n

In addition, the extruder and heat sensor are working. An older G-code file correctly sets 200\u00a0\u00b0C as the target temperature for the extruder.

\n

\"enter

\n

This G-code prints just fine

\n
;FLAVOR:RepRap\n;TIME:4628\n;Filament used: 1.44191m\n;Layer height: 0.15\n;Generated with Cura_SteamEngine 3.4.1\nT0\nM190 S60\nM104 S200\nM109 S200\nM82 ;absolute extrusion mode\nG21 ;metric values\nG90 ;absolute positioning\nM82 ;set extruder to absolute mode\nM107 ;start with the fan off\nG28 X0 Y0 ;move X/Y to min endstops\nG28 Z0 ;move Z to min endstops\nG1 Z15.0 F9000 ;move the platform down 15mm\nG92 E0 ;zero the extruded length\nG1 F200 E3 ;extrude 3mm of feed stock\nG92 E0 ;zero the extruded length again\nG1 F9000\nM117 Print0ng...\nM83 ;relative extrusion mode\nG1 F1500 E-6.5\n;LAYER_COUNT:80\n;LAYER:0\nM107\nG0 F4800 X72.96 Y99.484 Z0.3\n;TYPE:SKIRT\n---\n
\n
\n

Note, I have never modified the firmware or changed anything except what I can change with the frontpanel.

\n", "Title": "Anet A8 ignores extruder temperature?", "Tags": "|ultimaker-cura|g-code|troubleshooting|anet-a8|", "Answer": "

I got the same problem today and after a lot of troubleshooting (following the answers here) I found out that the issue was the connection to the hotbed. After I clean out the connection to the hotbed the printer started working as usual. The debugging logic was that the printer heats up the hotbed before setting the extruder temperature.

\n

Looking to your LCD the issue seems to be the same, since 33 degrees is a normal ambient temperature.

\n" }, { "Id": "8250", "CreationDate": "2019-02-12T17:57:02.127", "Body": "

I've done a fair bit of looking around for an answer and have yet been unable to find one.

\n\n

So here is my problem, after about 200-300 mm of extrusion, the PLA stops coming through the nozzle. When I pull the PLA out it looks like this. There are teeth marks going up the PLA and end with a small divot. The head of the PLA is slightly thicker than the rest of the PLA; approximately 1.9 mm.

\n\n

\"Filament

\n\n

I first noticed this when I thought that only the bottom layer was being printed. I moved to printing a small bullet and I got about halfway through the print before it stopped extruding.

\n\n

Next I raised the Z position to about 100 mm to give me some room to work and I manually moved the PLA through the extruder with the dial. No problems until I hit the 200 mm mark or so. I've done this test 4 times now and failed at the approximate locations: 208 mm, 280 mm, 250 mm, and 325 mm.

\n\n

Settings:

\n\n
    \n
  1. Nozzle: 200 °C
  2. \n
  3. Bed: 60 °C
  4. \n
  5. Layer Height: 0.15 mm
  6. \n
\n\n

Other things I have tried with seemingly no effect:

\n\n\n", "Title": "Monoprice Maker Select v2 stops extruding after about 200-300 mm", "Tags": "|extruder|troubleshooting|extrusion|monoprice-maker-select|", "Answer": "

Looking at the tread pattern on the filament, I think it's getting stuck. Note that the teeth marks get closer together as you go up the fillament, and the last one is a big divot, that looks like the gear trying to grab at it, and being unable to move it.

\n\n

Are you seeing any drops in nozzle temp? I had a cooling fan kick in and drop temps low enough to prevent extrusion before.

\n\n

Edit

\n\n

From the discussion in the notes, heat creep was identified as a possible culprit, and seems most likely.

\n" }, { "Id": "8252", "CreationDate": "2019-02-12T19:00:05.360", "Body": "

Kicked off the second long print in a series (printing Lack enclosure components). First 10 hour print was flawless. Started this one, saw the first layer laid down well, went to bed. Woke up to this (you can see the successful prints in the background):

\n\n

\"enter

\n\n

The whole heater block and nozzle is entombed in PLA. The leads to the heater and the thermistor are too. I'm assuming there's no solvent for this, and I'm better off just buying a new hot end.

\n\n

Thoughts?

\n", "Title": "Is this hot end salvageable?", "Tags": "|prusa-i3|hotend|troubleshooting|", "Answer": "

If you can remove the hot-end it is easier. If not, carefully use hot air to soften (not melt) the PLA so you can more easily pull away chunks.

\n\n

When the hot-end is removed from the other plastic parts, you can get more aggressive with hot air. BUT, be careful of the thermistor and heater wires. They are fragile, and can easily be broken or pulled out of their devices.

\n\n

Heat and patience is the key. If you do break a heater or thermister, they are pretty easy to replace. Depending on the hot-end, the heater and/or the thermister may be locked in place by a set screw. That set-screw is probably locked in place by plastic, so use the hot-air gun to melt that plastic and allow you to back out the set-screw.

\n" }, { "Id": "8256", "CreationDate": "2019-02-12T20:17:22.147", "Body": "

I've bought MKS 12864OLED and connected to my DIY 3d printer with Mega/RAMPS combination. In Marlin's configuration.h file there are these lines to uncomment:

\n\n
// MKS OLED 1.3\" 128 \u00d7 64 FULL GRAPHICS CONTROLLER\n// http://reprap.org/wiki/MKS_12864OLED\n//\n// Tiny, but very sharp OLED display\n//\n//#define MKS_12864OLED          // Uses the SH1106 controller (default)\n//#define MKS_12864OLED_SSD1306  // Uses the SSD1306 controller\n
\n\n

I'm not sure to choose between 'SH1106' or 'SSD1306'... either way, the LCD module is not working.

\n\n

\"enter\n\"enter

\n", "Title": "MKS 12864OLED Display setup", "Tags": "|marlin|troubleshooting|reprap|full-graphic-smart-controller|", "Answer": "

First of all let me state that I do not own the module! The analysis below is based on old patches that worked in a version of Marlin in 2015, and translated to the latest version of Marlin of the 1.1.x branch. This is 1.1.9; this is the last version of the 1.1.x branch, all new development takes place in branch bugfix-2.0.x (dated May 2019).

\n\n
\n\n

The reported patches are compatible with an earlier version of Marlin Firmware (a version from 2015). Clearly this doesn't work anymore, but that should not be a problem, if it worked then it should work now provided we configure it correctly. The display you have requires U8GLIB_SSD1306, so the U8GLIB library need to be installed in your Arduino IDE!

\n\n

Let's follow this installation guide for the older version as an example.

\n\n

First, from patch 1 it becomes clear that you'll need to define that you are using a display that is identified by its name/type, you should uncomment the following line in your configuration.h in the section:

\n\n
\n//=============================================================================\n//======================== LCD / Controller Selection =========================\n//========================   (Character-based LCDs)   =========================\n//=============================================================================\n\n
\n\n
\n//#define MKS_12864OLED_SSD1306  // Uses the SSD1306 controller\n
\n\n

to:

\n\n
\n#define MKS_12864OLED_SSD1306  // Uses the SSD1306 controller\n
\n\n

as you are using the SSD1306 controller according to this reference.

\n\n

That is about the only thing you add in the configuration.h file! You only activate the name of the controller type (as in defining a constant) so that it is caught in other source or header files to do/trigger something.

\n\n

With this change, you automatically activated the rest of patch as that is implemented in Conditionals_LCD.h!

\n\n

Next, we need to address patch 2. This is addressed in ultralcd_impl_DOGM.h; here you will find:

\n\n
\n#elif ENABLED(MKS_12864OLED_SSD1306)\n  // MKS 128x64 (SSD1306) OLED I2C LCD\n  U8GLIB_SSD1306_128X64 u8g(DOGLCD_SCK, DOGLCD_MOSI, DOGLCD_CS, DOGLCD_A0);      // 8 stripes\n  //U8GLIB_SSD1306_128X64_2X u8g(DOGLCD_SCK, DOGLCD_MOSI, DOGLCD_CS, DOGLCD_A0); // 4 stripes
\n\n

which clearly differs from the patch:

\n\n
\nU8GLIB_SSD1306_128X64 u8g(23, 17, 16, 25); // SW SPI Com: SCK = 23, MOSI = 17, CS = 16, A0 = 25\n
\n\n

as such that it uses numbers instead of constants. So we need to define these constants first. These constants are defined by the board you are using, more specifically the pin layout.

\n\n

Looking at the pin layout of your RAMPS board:

\n\n
\n      #if ENABLED(MKS_12864OLED) || ENABLED(MKS_12864OLED_SSD1306)\n        #define LCD_PINS_DC     25   // Set as output on init\n        #define LCD_PINS_RS     27   // Pull low for 1s to init\n        // DOGM SPI LCD Support\n        #define DOGLCD_CS       16\n        #define DOGLCD_MOSI     17\n        #define DOGLCD_SCK      23\n        #define DOGLCD_A0       LCD_PINS_DC\n
\n\n

you'll find that the pins are correctly configured with the fore mentioned:

\n\n
U8GLIB_SSD1306_128X64 u8g(DOGLCD_SCK, DOGLCD_MOSI, DOGLCD_CS, DOGLCD_A0);      // 8 stripes\n
\n\n

We move on to patch 3. This patch deals with the reset/initialization of the OLED display. This is also already taken care of in ultralcd_impl_DOGM.h:

\n\n
\n  #if PIN_EXISTS(LCD_RESET)\n    OUT_WRITE(LCD_RESET_PIN, LOW); // perform a clean hardware reset\n    _delay_ms(5);\n    OUT_WRITE(LCD_RESET_PIN, HIGH);\n    _delay_ms(5); // delay to allow the display to initalize\n  #endif\n
\n\n

Next to patch 4, in pins_RAMPS.h you see that pin 25 and 27 are correctly defined (apart from the name LCD_PINS_RST, now without T, but that is fine!):

\n\n
  #if ENABLED(MKS_12864OLED) || ENABLED(MKS_12864OLED_SSD1306)\n    #define LCD_PINS_DC     25   // Set as output on init\n    #define LCD_PINS_RS     27   // Pull low for 1s to init\n
\n\n

The only difference is that pins

\n\n
    #define LCD_PINS_D5\n    #define LCD_PINS_D6\n
\n\n

are not set to -1, so to be consistent, you should change pins_RAMPS.h to:

\n\n
\n      #if ENABLED(MKS_12864OLED) || ENABLED(MKS_12864OLED_SSD1306)\n        #define LCD_PINS_DC     25   // Set as output on init\n        #define LCD_PINS_RS     27   // Pull low for 1s to init\n        // DOGM SPI LCD Support\n        #define DOGLCD_CS       16\n        #define DOGLCD_MOSI     17\n        #define DOGLCD_SCK      23\n        #define DOGLCD_A0       LCD_PINS_DC\n        #define LCD_PINS_D5     -1\n        #define LCD_PINS_D6     -1\n      #else\n        #define LCD_PINS_RS     16\n        #define LCD_PINS_ENABLE 17\n        #define LCD_PINS_D4     23\n        #define LCD_PINS_D5     25\n        #define LCD_PINS_D6     27\n      #endif\n
\n\n

A conclusion from the analysis above is that you enable the display in your configuration file and add the 2 values of -1 for D5 and D6, but I don't think they cause problems if they're not initialized to -1. Be sure that the cables are connected correctly. Some modules have reported upside down connectors.

\n" }, { "Id": "8263", "CreationDate": "2019-02-13T07:05:09.907", "Body": "

\"Failed

\n\n

I have only been able to get my prints this far. They just can\u2019t stop skipping pulling material up towards the nozzle which then melts pulls of another random bit. At first I thought it was motor skipping, so I strengthened the spring, but even after that and calibrating the bed, I couldn\u2019t figure out exactly how to fix this issue. Hopefully someone has some idea what is wrong.

\n", "Title": "Ender 3 print is skipping and dragging up base print lines", "Tags": "|creality-ender-3|print-failure|", "Answer": "

I would get rid of the blue tape and go with glass and hairspray. Glass gives you more of a flat surface. You are printing with the aluminum build plate which over time c\nan warp. The glass with give you a much flatter and level surface to print om. Blue Tape is actually not a great surface to print on. Hairspray using a chemical compound that actually is related to a PLA makeup. This compound in Aquanet Hairspray will bond to the PLA and release when cold. Using both the glass and hairspray will give you and even build plate and a clean smooth surface that it will stick to better than blue tape.

\n" }, { "Id": "8266", "CreationDate": "2019-02-13T10:27:05.650", "Body": "

Octoprint warns me that the objects do not fit into the print volume. I noticed that this happens after a power-off cycle. Since I was overly anxious until today, I always uploaded the GCode file again and it didn't complain any more.

\n\n

Of course, always uploading the files again is also error prone. So today I gave it a try and simply started to print the object. As far as I can tell, it prints nicely.

\n\n
\n

Image: Octoprint saying something like \"Object does not fit into print volume\" in German

\n
\n\n

\"Object

\n\n

Is this a known bug in OctoPrint? Are my files really damaged after a power off cycle?

\n\n

The values (0.00, -3.00, 0.00) seem to be constant in this error message, no matter what object I want to print.

\n\n

I'm using OctoPrint in the PrusaPrint flavor and I'm running version 1.3.10 (hopefully a recent version, since I usually update). I generate the G-Code with Slic3r.

\n", "Title": "False \"Object does not fit into print volume\" warning from OctoPrint", "Tags": "|octoprint|", "Answer": "

This answer is correct, it's normal for Prusa printers to purge at -3 mm on the Y axis.

\n\n

This answer is an addition that describes how to get rid of the error.

\n\n
    \n
  1. Open Octoprint web UI
  2. \n
  3. Go to Settings -> Printer Profiles
  4. \n
  5. Find active profile, click on the pen icon next to it (Edit Profile)
  6. \n
  7. Go to Print bed and build volume
  8. \n
  9. Tick Custom bounding box
  10. \n
  11. Enter -3 to Y Coordinates Min input box
  12. \n
  13. Hit Confirm
  14. \n
\n" }, { "Id": "8288", "CreationDate": "2019-02-16T17:14:12.037", "Body": "

I calibrated my extruder according to the mattshub tutorial and printed the extruder calibration test cube with 2 perimeters of 0.4\u00a0mm. So the walls should all be 0.8mm thick but only 3 are close to 0.8mm and one is about 0.93 mm thick. The pictures are one turn, walls following each other are side by side:

\n

\"Thick\n\"thin\n\"Average\n\"Average

\n

Update 1

\n

I printed a solid, smooth calibration cube with 1 Perimeter of 0.45\u00a0mm, 0 top/bottom layers and 0\u00a0% infill, as suggested and got 0.56\u00a0mm,\n0.65\u00a0mm, 0.79\u00a0mm and 0.54\u00a0mm

\n

\"The

\n

I believe that the thickest side is the one where the layer ends and retraction is done in the corner before layer change. Maybe retraction settings?

\n

Update 2

\n

After setting the Extrusion Multiplier to 70%, I got wall thicknesses of 0.89\u00a0mm,\n0.86\u00a0mm, 0.81\u00a0mm and 0.96\u00a0mm on a 2 Perimeter print of the same object.

\n

Update 3

\n

Fudging with the steps per millimeter changed them from 428 to 306. The resulting one perimeter test was 0.59\u00a0mm, 0.46\u00a0mm, 0.42 mm and 0.39\u00a0mm, pretty much spot on safe for one wall being fat and one thin...

\n

I print at 215\u00a0\u00b0C with 5\u00a0mm retraction to increase strength and get stringing in check.

\n", "Title": "Extrusion test cube resulted in one wall being thicker", "Tags": "|extruder|calibration|tevo-tornado|", "Answer": "

Why not that test print?

\n\n

You don't want a print constraining your walls to 0.8\u00a0mm, since you don't want to print with 1 nozzle diameter for better print quality. Atop that, slicing can induce errors that increase the thickness of walls without us noticing it in the slicer view. Two sources of error (once for each wall)\nmeans an additional 10% error, which would bring the wall as you got perfectly into the expected area: $0.8\\text{ mm}\\times 1,1\\times 1,1=0.968\\text{ mm}$

\n\n

Let's Troubleshoot!

\n\n

You might not have calibrated the extrusion multiplier correctly, or the steps/mm might be off, or other print settings shoot us way off. Let's make sure to find the source.

\n\n

Filament extrusion multiplier

\n\n\n\n

Done that? Math time to calculate the correct multiplier for the filament! The average wall is easy:$$d_\\text {average}={\\frac{\\sum_{i=1}^4 d_i}{4}}$$ $$\\frac{d_\\text{line width}\\times p}{d_\\text {average}}=\\text{Extrusion multiplier}$$\nRemember, that the result is not in % but a float point number! 1 is 100%. So you fill in that value times 100 into the extrusion multiplier.

\n\n

After this, repeat the whole process with 2 perimeters.

\n\n

1 - further reading: Why is it conventional to set line width > nozzle diameter? & Slicer line width vs. extrusion multiplier for layer adhesion?\n

\n\n

Steps/mm

\n\n

Make sure to test the extruder against the heated extruder, then repeat the calibration as your tutorial explained.

\n\n

For some tests, let's fudge to some degree: Trying a filament dependant value of steps per millimeter can help to try to find other sources of errors easier - it makes them at times more pronounced. So we just multiply our steps/mm with the extrusion multiplier that we calculate the way outlined below. For 428 steps/mm and the numbers from Update 3, this gives about 306 steps/mm. This is not a proper calibration, but a means for troubleshooting. After fudging with the parameters reset the extrusion multiplier to 100%, we just want to check what influences our thickness.

\n\n

Other print settings

\n\n\n\n

Lower temperature and less retraction should get prints more consistent, as pressure changes in the nozzle are more consistent. Remember, we are playing with the settings here.

\n\n

And now Properly

\n\n

Back to Calibration!

\n\n

After all the fudging and probably making everything worse, we got to make sure to properly calibrate, those settings will be WAY OFF, in fact, we might go back to the original steps/mm or higher:

\n\n\n\n

Extrusion/Flow Calibration

\n\n

Now, go back to print a single filament extrusion multiplier calibration, as above. It should be somewhat close(er) to 100% now.

\n\n

Slicer Fault?

\n\n

Note that not all Slicers are equal: some slicers are better and more consistent in the results than others with the stock settings, and even with the same settings results may varry. For example the line width for the same settings with Simplify3D and Cura or Slic3r Prusa Edition can differ. The reasons for this are hard to find exactly, but they are most likely rooted in the different slicing processes and optimisation. Sometimes it is some setting that might be overlooked that results in line width variations.

\n\n

The Slicer world changed a lot betwene 2015 and 2019, and even if you got fed up with one slicer years ago, now it might be worth a shot to test it again. See if you might get better results with other slicers, as for whatever reason, your slicer might cause the issiues, whih allows you to hunt down setting dependant items.

\n" }, { "Id": "8293", "CreationDate": "2019-02-17T03:50:02.237", "Body": "

Running my Flsun 3D Cube from a Kubuntu PC using Octopi, and trying to get my webcam to work with it. I've followed the instructions on Github for mjpg_streamer to the letter. Everything I've read says, \"Sure! The Raspberry PI directions will work with Linux, you bet!\" but they don't, at least, not for me, a beginning level Linux user.

\n\n

I know that my webcam works with Kubuntu since I can load it and view it using VLC, but I've been unable to figure out how to get Octopi to read the stream from VLC (the documentation could use some improvement) or even confirm if the stream works.

\n\n

Back to mjpg_streamer: when I issue the command

\n\n
export LD_LIBRARY_PATH=. ./mjpg_streamer -o \"output_http.so -w ./www\" -i \"input_raspicam.so\"\n
\n\n

I get

\n\n
bash: export: 'mjpg_streamer': not a valid identifier\nbash: export: '-o:' not a valid identifier\nbash: export: 'output_http.so -w ./www': not a valid identifier\nbash: export: '-i': not a valid identifier\nbash: export: 'input_raspicam.so': not a valid identifier\n
\n\n

I don't know what I don't know, and I don't know how to proceed. The goal is to view my webcam in Octopi/Octolapse on a Kubuntu PC. (Windows with Yawcam, no brainer; Linux, not so much.)

\n", "Title": "mjpg_streamer, Octopi, Kubuntu", "Tags": "|troubleshooting|octoprint|flsun-3d-cube|", "Answer": "

Going to post the answer here for posterity and for those, like me, trying to cope with Linux after making a career in Windows.

\n\n

I'm running Kubuntu 18 because I didn't want to have to buy an operating system or install a bootleg copy of Windows. The most important thing for a Windows user to know is that the readme.md file is important. Think of it as a .chm file, or a series of linked help documents. What you want to know is THERE, not on the github page.

\n\n

Using the following command in the mjpg-streamer-experimental directory got me where I needed to go.

\n\n
mjpg_streamer -i 'input_uvc.so -d /dev/video0 -r SVGA -f 30' -o 'output_http.so'\n
\n\n

e.g. webcam, 800x600, 30fps to http (port 8080 is the default) The MD files break it down. I also found two web pages very useful:

\n\n

http://skillfulness.blogspot.com/2010/03/mjpg-streamer-documentation.html\nhttps://wiki.ubuntuusers.de/MJPG-Streamer/

\n\n

In OctoPrint the streaming setting is then:

\n\n
http://127.0.0.1:8080/?action=stream\n
\n\n

In my case, I use the network IP rather than local host so I can monitor the print from my upstairs PC. For snapshots:

\n\n
http://127.0.0.1:8080/?action=snapshot\n
\n\n

I'll admit that Octolapse doesn't work for me yet because I'm still trying to work out ffmpeg, but that's another issue. EDIT: got that done, too. Useful command:

\n\n
whereis ffmpeg\n
\n" }, { "Id": "8297", "CreationDate": "2019-02-19T03:11:42.517", "Body": "

I am going to buy an inductive probe for my Creality Ender 3 to enable ABL (Automatic Bed Leveling). And now, I have one last question left on my mind:

\n\n

I am going to buy a Vishay Optokoppler Phototransistor SFH610A-1 DIP-4 Transistor DC.

\n\n

Is it sufficient to put a 2 - 2.2 kΩ resistor between the input of the octocouplers LED side and the 24 V Output of the probe, to lower the current and voltage enough to work with the octocoupler? Or do I need something else?

\n\n

On the transistor side, I will simply attach the 2 wire Z-endstop cable, +5 V on Collector and GND on Emitter.

\n", "Title": "Hooking up an optocoupler in a 24V Machine, is a 2 kOhm resistor sufficient?", "Tags": "|z-axis|electronics|creality-ender-3|bed-leveling|inductive-sensor|", "Answer": "

TL;DR

\n

Yes, a 2\u00a0k\u03a9 resistor is sufficient to limit the current to about 12\u00a0mA through the optocoupler. With a maximum current specification of 60\u00a0mA you require at least 400\u00a0\u03a9; any resistor in between the values should be suitable for this optocoupler component when used at 24\u00a0V. More information is found below.

\n
\n

Having a glass bed and using an inductive probe may not work optimally. In such cases a 3D touch sensor is a very good alternative. For inductive sensors, remember that these sensors optimally detect steel, aluminium is harder to detect (about 60\u00a0% less than iron; a glass sheet in between the bed and the sensor requires you to acquire a sensor with a large detection range, e.g. 8\u00a0mm or more. To optimize the detection distance such sensors need to be powered to the maximum voltage they can handle (usually 36\u00a0V) or the highest voltage you have available (e.g. power supply 12 or 24\u00a0V). To protect the board, that is only allowed to receive up to 5\u00a0V, an optocoupler is an excellent way to guarantee the separation of voltage levels.

\n

Considering your questions, it is fair to assume you are not an experienced electronics tinkerer, purchasing an optocoupler module is the best alternative, you just screw the wires into the respective input screw terminals as shown in this answer.

\n

As the optocoupler separates two circuits, you can safely use 24\u00a0V on the one side and the 5\u00a0V on the other side, you do not need an additional resistor when you use a module board. If you plan to buy separate components to build your own circuit, you need to look at the maximum current that the optocoupler can handle on the input side (that will be 24\u00a0V). From the documentation from the optocoupler one reads that it is limited to 60\u00a0mA. A maximum current would therefore require a resistor of:

\n

$$ R=\\frac{U}{I}=\\frac{24}{60\\times10^{-3}}=400\\ \\Omega $$

\n

My 12\u00a0V optocoupler module uses a 1\u00a0k\u03a9 resistor implying a 12\u00a0mA current. In your case allowing 12\u00a0mA of current yields a 2\u00a0k\u03a9 resistor.

\n" }, { "Id": "8301", "CreationDate": "2019-02-19T10:26:43.260", "Body": "

First, I'm using Linux Mint 18.3 (Sylvia). Until now, I've been using OpenSCAD with the GUI and never experienced issues. Now I try to start OpenSCAD from the command line, but it always opens an empty file, even if a file with the specified name exists.

\n\n

Since I have a rather big script, I'd like to generate the STL's via the CLI. Due to this error, I can not even do any tests for parameter passing.

\n\n

What I'd like to do is to issue a command (flom the command line or in a shell script) that says \"Set objectID to 1, render and export the result to stl\".

\n\n

Here is my M(N)WE:

\n\n
// test.scad\n\nobjectID = 2;\n\n\nif (objectID == 1)  \ndifference(){  \n    cylinder(d=20, h=50, center=true);  \n    cylinder(d=16, h=50.2, center=true);      \n}\n\nelse if (objectID == 2)  \ndifference(){  \n    cube(50, center=true);  \n    cube([35,35,70], center=true);  \n}\n
\n\n

Any help will greatly be appreciated.

\n", "Title": "OpenSCAD 2018 Command line", "Tags": "|stl|openscad|linux|", "Answer": "

I could finally solve the issue. Calling openscad from command line with parameters does NOT work with the AppImage I used. It works fine with a \"standard\" installation, though.

\n\n

Thanks for your support.

\n" }, { "Id": "8313", "CreationDate": "2019-02-21T01:42:44.600", "Body": "

PLA wood has a beautiful natural texture that I wanted to try. But when I used it in my second print, it clogged the nozzle (solved). I know this filament is composed of 70\u00a0% PLA and 30\u00a0% recycled pine wood fibers.

\n\n", "Title": "PLA wood filament: a special treatment?", "Tags": "|filament|filled-pla|", "Answer": "

PLA and wood fibres = wood filament

\n

Most wood filamet consists of about 60-70\u00a0% PLA and 40-30\u00a0% wood fibres. This basically implies that PLA temperatures should be used. It can be printed with standard 0.4\u00a0mm nozzles, but it is adviced to be printed with a larger diameter nozzle. A larger nozzle will less likely to cause nozzles to clog (more area for the fibres to pass through).

\n

Basic printing advice

\n

Start experimenting with relatively large layer heights (0.25-0.3\u00a0mm layer heights). Printing speeds should be held high to ensure relative short residence times in the hot end (in the range 50-80\u00a0mm/s). Short residence times prevent degradation by heat resulting in clogging. It is best for wood fibres filled filaments to not have the filament to stagnate (e.g. pausing). A higher retraction speed and distance might be needed because the filament usually is a little more runny compared to plain PLA.

\n

Beware

\n

After printing, you should retract the filament from the hotend so that during next printing jobs heat up cycle, the filament doesn't degrade and clogs the nozzle. Alter the end script G-code and also use priming scripts at the start and/or do not forget to use a large skirt of multiple lines or a considerable length. As always, you shouldn't leave the printer unattended, regularly check the printer and shut down the printer when there is something wrong, e.g. not extruding. Also note that wood filled filament is reported to be abrasive to brass nozzles, a (hardened) steel nozzle or a Ruby nozzle should mitigate wear problems.

\n

Basic settings

\n

These settings have proven to work, but are not a guarantee, these settings should provide a reference to start experimenting on your own printer:

\n\n" }, { "Id": "8316", "CreationDate": "2019-02-21T13:09:38.600", "Body": "

So, having bought this cube pro duo 3d printer a few days ago I have come to see how significantly limiting the default slicer software that comes with the machine is. For the prints that I am doing I've been looking around at alternative slicers however the cube pro has its own file format for builds and I'm not sure if any other software has the option to export those specifically and I also have no idea id the CubePro can read any files other than its own. On top of this whenever I download some other slicer it gets to the printer setup and there's a whole bunch of settings and parameters for the printer that I haven't been able to find, stuff like Firmware or Nozzle Diameter (Something that id expect to be easy to find).

\n\n

In particularly a software called IdeaMaker is what I'm interested in since I've seen that you can make custom supports and that would be very useful for what I'm doing.

\n\n

Does anyone know what I can do? Or is this a case of there's nothing I can do without some serious modification that I'd rather not do?

\n\n

Any help is appreciated thanks! :D

\n", "Title": "CubePro Alternative Slicer", "Tags": "|software|slicing|firmware|cubepro-duo|", "Answer": "

As far as I understand, the CubePro 3D printers use their own format: .cubepro or .cubex.

\n\n

It appears that a:

\n\n
\n

.cubepro file format and found out that it only does a Blowfish ECB encryption of the .bfb file

\n
\n\n

This implies that you need to find a slicer that outputs .bfb tool path files (similar like G-code files) and a tool to encrypt the tool path file. The dubious CodeX tool and this alternative can do that for you.

\n" }, { "Id": "8320", "CreationDate": "2019-02-22T03:30:26.623", "Body": "

I am trying to make Marlin use the RX1 and TX1 pins that are in the EXP1 connection. I am connecting those pins to the TX, RX of an Arduino.

\n\n

The Configuration.h file has a SERIAL_PORT define, normally configured to be 0, but supposedly changeable.

\n\n

What should be changed in Marlin to make it used the Serial1 port (if possible at all)? I am using the latest code provided in Printrbot's Marlin repository

\n\n

From the Configuration.h file:

\n\n
// SERIAL_PORT selects which serial port should be used for communication with the host.\n// This allows the connection of wireless adapters (for instance) to non-default port pins.\n// Serial port 0 is still used by the Arduino bootloader regardless of this setting.\n#define SERIAL_PORT 0\n//#define SERIAL_PORT 1\n\n// This determines the communication speed of the printer\n// This determines the communication speed of the printer\n#define BAUDRATE 250000\n//#define BAUDRATE 9600\n
\n\n

\"Printrboard\"

\n", "Title": "Printrboard Rev. D - RX1 and TX1 (second serial port) / Marlin", "Tags": "|marlin|firmware|electronics|", "Answer": "

I was able to resolve this issue:

\n\n
    \n
  1. I moved to Marlin 1.1.9 (the final release of the 1.1.x versions).
  2. \n
  3. I made a fork, for those who would like to see the changes. Also summarized below.
  4. \n
  5. The RX1 and TX1 pins are well identified in the question. I connected those directly to the HC-06 bluetooth module (using a voltage divider for the RX pin in the bluetooth, as it is a 3.3 V logic module).
  6. \n
\n\n

Changes in code:

\n\n

Marlin/Conditionals.LCD.h: commented out:

\n\n
// this line conflicted with the bluetooth/serial1 options\n// #define USE_MARLINSERIAL !(defined(__AVR__) && defined(USBCON))\n
\n\n

The rest are the configuration options already provided by Marlin in Marlin.Configuration.h:

\n\n
define SERIAL_PORT 1\ndefine BAUDRATE 115200  // Note: this baudrate MUST match the one set in the bluetooth module. That's done using the AT commands. There is plenty of information on how to do that. \ndefine BLUETOOTH\n
\n\n

As a result of these changes I am able to print with my PrintrBot Simple 2014 (Printrboard Rev. D) with Bluetooth over a distance of ~3.5mts, behind a wall. I haven't tested it exhaustely, but it does print.

\n" }, { "Id": "8325", "CreationDate": "2019-02-22T22:19:48.703", "Body": "

I am looking for a 3.3 V power source in the Printrboard Rev. D pins.

\n\n

I think there should be one somewhere... I do not know how to read the schematics that well.

\n\n

Also, I am going to connect an HC-06, which draws 45 mA or less during operation. So, I need to know if it safe to get the power from the 3.3 V rail (if I find it).

\n\n

So, the questions are:

\n\n\n\n

\"enter\n\"enter

\n", "Title": "Printrboard Rev. D. 3.3 V source", "Tags": "|electronics|", "Answer": "

On the schematic, the 3.3v is marked as \"U7\" and pointed out here:

\n\n

\"enter

\n\n

The 3.3v is marked as pin #2. You can find the three pints from U7 here on the board (blue circle):

\n\n

\"enter

\n\n

I'm not exactly sure if the pins are accessed from the other side of the board, or even if they are marked at all on there. You can always check them with a multimeter to see what their output is. More than likely the bottom single pin is the ground and the top two are one or the other (5v & 3.3v).

\n\n

As far as amperage draw, I'm thinking 45mA is not a lot of draw, but I'm not an expert. I'd think it should be able to handle it, but again, I really don't know.

\n" }, { "Id": "8328", "CreationDate": "2019-02-23T17:25:40.430", "Body": "

Here is an extreme example of notches caused by the printer. They go along all sides but are the strongest on the Y faces. They also happen quite randomly sometimes they are tiny sometimes they are strong.

\n\n

\"enter

\n\n

I have tightened the belts already and while that reduced the ghosting on the X face a lot, it did nothing on the Y and actually never helped with the notches. This test cube has notches and ripples too but not that strong (the skirting on the bottom of that cube is my fault I set the bed level a little too low).

\n\n

\"enter

\n\n

\"enter

\n\n

All models have been sliced with Ultimaker Cura and printed on an Anet A8. Flow = 110 %, layer height of 0.1 mm for the first example and 0.2 mm for the cube, printing temperature = 195 \u00b0C, no change on jerking and acceleration from default settings. Cube size = 20x20x20 mm.

\n\n

The printer has frame support https://www.thingiverse.com/thing:1672959 but no other upgrades yet.

\n", "Title": "Why does my printer create notches on the Y surface?", "Tags": "|print-quality|anet-a8|", "Answer": "

While you may very well be having temperature issues, I see two other issues:

\n\n

First, I think the \"notches\" are z-wobble. Z-wobble is often caused by bent z-axis threaded rods.

\n\n

I have a very similar Geeetech I3 printer. A lot of my z-wobble was cleaned up by removing the threaded rods and making them more straight. You can find videos to help you do that, just search for \"threaded rod straightening\".

\n\n

The other thing I did was decouple the nut on the threaded rod from the x-carriage assembly, allowing it to float in the X-Y plane as it pushes the x-carriage up. The x-carriage ought to be guided up by the smooth rods/bearings. Unfortunately, if the nut is fixed to the x-carriage, the bent threaded rods are strong enough to over-power the smooth rods/bearings and make the whole assembly wobble.

\n\n

The other problem I see is that your printer seems to be extruding too much filament. That is likely causing the little pyramid flair at the base of your print. If you tell the printer to extrude 100mm, it ought to extrude 100mm, not 110mm. Again, a search for \"extruder calibration\" will tell you how to do it. For me it involved extruding, measuring, a bit of math, adjusting some variables in Marlin, compiling and uploading the with Arduino IDE. Repeat until 100mm extrudes 100mm.

\n" }, { "Id": "8334", "CreationDate": "2019-02-24T18:00:25.323", "Body": "

I have an issue with my Anet A8 printer and how it interlocks with Ultimaker Cura.

\n\n

I want to print this file named Loki_h\u00f6rner_v2.stland Cura slices it fine, but when it comes to printing all the preheat happens, but then it stops, not going on at all. What might be wrong here?

\n", "Title": "Anet A8 stops printing", "Tags": "|ultimaker-cura|anet-a8|troubleshooting|print-failure|bug|", "Answer": "

Special characters like \u00c4 \u00d6 or \u00dc in the stl-filename resulted in Ultimaker Cura creating a comment of the filename in the g-code that read like

\n\n
;MESH:Loki_h\u00f6rner_v2.stl\n
\n\n

This apparently could not be parsed by the Anet A8, leading to an error and halt.

\n" }, { "Id": "8350", "CreationDate": "2019-02-26T18:14:11.010", "Body": "

I'm currently trying to implement a data collector on my Replicator+ by utilizing JSON-RPC. Is there an official reference for this? MakerBot used to host a wiki site, but that seems to be gone for their \"troubleshooting\" pages.

\n", "Title": "MakerBot JSON-RPC Command List", "Tags": "|makerbot|replicator+|", "Answer": "

It took some digging, but I was able to start scripting a wrapper in C# for the RPC commands.

\n\n\n\n

I still don't understand what every method does to the machine or necessarily what the results are, so a number of the methods are marked as obsolete until I can test them.

\n" }, { "Id": "8351", "CreationDate": "2019-02-26T18:43:25.330", "Body": "

Issue: My Ender 3 is creating distorted prints with layer separation and deformations.

\n

\"Example

\n

Can anyone point me in the right direction?

\n

Setup:

\n\n

What I have tried:

\n
    \n
  1. Tightening Z axis screw
  2. \n
  3. Tightening Y axis belt
  4. \n
  5. Tightening X axis belt
  6. \n
  7. Switching to a different spool of ABS
  8. \n
  9. Printing a temperature tower (same problem across different temperatures)
  10. \n
  11. Turning off the auto bed leveling.
  12. \n
\n

Update:

\n
    \n
  1. I've measured the temperature of the hot end, it is reading around 205\u00b0C +- 20\u00b0C
  2. \n
  3. As per suggestion from the TH3D support team, I tuned the PID of my hot end. Unfortunately the results did not turn out much better (1,2) and the support technician is suggesting(a long with many people from the comments) that I should try to replace the thermistor. As per suggestion from @Trish, I measured the impedence of the thermistor and it does seem to be somewhat off from stock (118k\u03a9 vs 100k\u03a9). Will update again once the replacement arrives.
  4. \n
  5. As requested, here are some more photos front, back, left side, right side. The cube isn't hollowed out just to save material while I calibrate the dimensions.
  6. \n
  7. I printed the same cube out, rotated 90 degrees. I got similar results though: front, back, left side, right side.
  8. \n
  9. Swapping the nozzle out solved the issue! Thanks for the help everyone!
  10. \n
\n", "Title": "Ender 3 Distorted Calibration Cube", "Tags": "|ultimaker-cura|troubleshooting|creality-ender-3|bed-leveling|", "Answer": "

Resolution: After many trials and errors, I finally replaced the nozzle with one that that was not partially blocked by filament at its entrance.

\n\n
\n\n

Likely cause: A careful examination of the old part hints, that the repeated blockage in the nozzle seems to have been caused by a gap between the PTFE tube and the nozzle, which has considerably moved backwards under the stress of printing as one can see here in a photo of the PTFE tube.

\n\n
\n\n

It took me about a month and I went down a few rabbit holes until @user77232 made a great suggestion to check the nozzle and see if it needs to be cleaned. Thanks for helping me out everyone!

\n" }, { "Id": "8353", "CreationDate": "2019-02-26T20:07:07.140", "Body": "

I currently have a model in fusion 360 of cylinders with the largest being 40\" diameter in a specific pattern as seen below. My printer is a Ender-3x with the build plate dimensions of 8.7\" x 8.7\" x 9.8\"

\n\n

I need to break this model into 5 pieces of exactly equivalent length so I can print same piece 5 times to equal a full cylinder

\n\n

\"enter

\n", "Title": "how to split a 40\" model into 5 parts to print seperate", "Tags": "|3d-models|stl|fusion360|", "Answer": "

You've got several options:

\n\n

Generate full-size STL, cut via software later

\n\n

If you just have an STL, you can use software such as Meshmixer to modify it. In this case, a plain plane cut would suffice.

\n\n

Modifiy the design file

\n\n

A better way is to go into the design file (in your case fusion) and modify it there. In your case, it's rather easy:

\n\n\n\n

If you want to get rid of parts that became superfluous after this, use the remove option, not the delete option, as the later tries to make sure that the object never was made in the workflow.

\n" }, { "Id": "8369", "CreationDate": "2019-02-28T14:43:50.433", "Body": "

I am using a Flash Forge Creator, and when I print big parts (only in this case) I have about 25% of the bottom of the printed object sticking very hard to its support layer. And I spend a lot of time removing it with a cutter.

\n\n

Is there any clue or good practice to avoid that?

\n\n

I use ReplicatorG for my printing settings.

\n", "Title": "3D printed part sticking to the support layer", "Tags": "|pla|support-structures|flashforge-creator|", "Answer": "

I finally found out two causes:

\n\n
    \n
  1. The 3D printer has default settings for ABS, and since there is no printer bed settings in ReplicatorG, the bed temperature was set to 110 \u00b0C instead of  60\u00b0C. I fixed the setting directly on the printer itself.

  2. \n
  3. It seems that one side of the printer bed was little closer to the nozzle than other sides (relatively to the extruder) and this added a pressure on the support layer. I found this by observing the structure of the support layer that was little more compressed in one side. That was easy to fix by iteratively tightening/loosening the screw of that side and observing the support layer (and canceling the print if it is not homogeneous).

  4. \n
\n\n

I installed Ultimaker Cura and Slic3r to try them and I found that Ultimaker Cura was not adapted to my printer (I did not found a complying model), Slic3r seems to be okay but I was not able to fit my big part inside of it.\nI am trying to use the maximum available space for my part, perhaps I need to change some settings for Slic3r and anyway it is already working on ReplicatorG.

\n" }, { "Id": "8373", "CreationDate": "2019-02-28T19:08:22.303", "Body": "

I'm struggling with an upgrade I made to my Creality CR-10. I upgraded the extruder to an E3D V6 hotend. The extruder mount and cooling fans are installed and working, however, I'm doing something wrong with installing the bowden tube into the hotend. Inevitably after I print a few layers, the extruder jambs and the issue is always the same: the bowden tube has backed itself out a few tenths of a millimeter, and the filament has mushroomed into the vacant space and hardened so that the filament cannot go forward or backwards.

\n\n

I've tried several things to fix the issue including:

\n\n\n\n

The only thing that has (partially) worked was when I would ductape and hot-glue the bowden tube into the fitting so it couldn't back out. However, since I'm still tweaking things, I inevitably have to disassemble things and I'm back to square one.

\n\n

I'm trying to figure out what mistake I am making to keep causing this issue. As an example, originally I used the pneumatic fittings wrong and thought I was supposed to pull the plastic part out to release the tube, rather than simply pushing it in to release the tube. (Needless to say, I wrecked a lot of fittings that way.)

\n\n

What else might I be doing wrong to keep causing this issue? What are other culprits to this issue happening repeatedly? Are there firmware settings that may help (or be aggravating the issue)?

\n", "Title": "Installing bowden tube into E3D V6", "Tags": "|extruder|bowden|e3d-v6|", "Answer": "

You may need to secure the pneumatic coupling in the closed position with a small plastic clip (which should be supplied with the hot end).

\n\n

\"Pneumatic

\n\n

You can print your own, providing that your printer will work for long enough (a paper clip might do the trick):

\n\n

Thingiverse: Bowden Tube Clip v3

\n\n

Addendum:

\n\n

Some pneumatic couplers are sprung, so that you have to depress the coupling ring in order to release the tubing. In this case, no clip is usually required. Other pneumatic couplers (such as the one on an E3D V6) are unsprung. This makes it easier to secure and release the tube using the supplied clip. The disadvantage is that you may lose the clip. Sprung couplers sometimes lose their springiness, in which case a clip can be used to secure them in the closed position

\n\n

It's \"swings and roundabouts\", really. You can to choose between the awkwardness of sprung couplers, or the risk of losing a clip. Either way, print some spare clips. You may need them one day.

\n" }, { "Id": "8380", "CreationDate": "2019-03-01T08:02:50.363", "Body": "

Recently started using my kit Prusa I3 Mk3 and noticed that with large horizontal surfaces a wrinkling pattern is emerging.

\n\n

As you can see in the image the wrinkles seem to run parallel, the surface is 10 mm above the build plate with all bridges fully supported.

\n\n

Has anyone seen this before? All other areas seem to be doing well.

\n\n

This print uses PLA filament @ 235 °C hotend temperature and clearly shows a wrinkled pattern on the top layer

\n\n

\"Print

\n\n
\n\n

If it helps, I haven't changed the settings from the normal Prusa Slic3r 0.15 profile.

\n\n

The problem reduced to an acceptable level by reducing the temperature to 205 °C but keeping the fan speed 100 %, I am printing PLA. I might reduce the fan speed if I feel troubled by the result.

\n", "Title": "Prusa i3 Mk3 - Top layer wrinkles (not on buildplate)", "Tags": "|prusa-i3|troubleshooting|", "Answer": "

Waves in printed surfaces with FFF are observed at either the bottom layer (common) or the top layer (less common).

\n

Waves in bottom layer

\n

Rippling/wave generation/wrinkling is a common problem for first layer to occur and has a direct relation to the print nozzle to bed distance; a too short of a distance or over-extrusion can lead to this effect. However, this effect is less commonly observed in top layer finishes. Bottom layer waves are described in more detail in this answer.

\n

Waves in top layer

\n

I have seen this defect before. It is caused by a combination of incorrect hotend temperature and print cooling fan settings. Please reduce the hotend temperature and reduce the fan cooling. The image below clearly shows the differences of such measures.

\n

\"Solving

\n" }, { "Id": "8387", "CreationDate": "2019-03-01T20:32:51.040", "Body": "

I am new to 3D printing and recently got a second hand RF1000 and am busy calibrating it. For this I want to print a basic calibration cube. I noticed the print doesn't always start with enough filament in the nozzle. To make sure the starting point for all prints are the same I added this G-Code in the CuraEngine settings:

\n\n
G92 E0 ; start line\nG1 F300 E-0.5\nG1 X230 Y22 Z0.35 F5000\nG1 F800 E13\nG1 X20 E25 F1000\n
\n\n

This works however after this the slicer automatically adds G-Code to move the extruder to the skirt location. This should be fine however I noticed that during the last part of this movement it retracts filament. This causes the start of the actual print to fail. What causes this and how can I change my settings to prevent this?

\n\n

Thanks for any help or advice you can give!

\n\n

Below is the G-Code from start until the end of the skirt.

\n\n
;Generated with Cura_SteamEngine 15.01\n; Default start code\nG28 ; Home extruder\nG1 Z15 F100\nM107 ; Turn off fan\nG90 ; Absolute positioning\nM82 ; Extruder in absolute mode\nM190 S60\n; Activate all used extruder\nM104 T0 S210\nG92 E0 ; Reset extruder position\n; Wait for all used extruders to reach temperature\nM109 T0 S210\nG92 E0 ; start line\nG1 F300 E-0.5\nG1 X230 Y22 Z0.35 F5000\nG1 F800 E13\nG1 X20 E25 F1000\n;Layer count: 199\n;LAYER:0\nM106 S127\nG0 F9000 X80.200 Y80.200 Z0.200\n;TYPE:SKIRT\nG1 F1800 X119.800 Y80.200 E0.44818\nG1 X119.800 Y119.800 E0.89636\nG1 X80.200 Y119.800 E1.34454\nG1 X80.200 Y80.200 E1.79272\nG0 F9000 X80.600 Y80.600\nG1 F1800 X119.400 Y80.600 E2.23185\nG1 X119.400 Y119.400 E2.67097\nG1 X80.600 Y119.400 E3.11010\nG1 X80.600 Y80.600 E3.54923\nG0 F9000 X81.000 Y81.000\nG1 F1800 X119.000 Y81.000 E3.97930\nG1 X119.000 Y119.000 E4.40937\nG1 X81.000 Y119.000 E4.83944\nG1 X81.000 Y81.000 E5.26951\nG0 F9000 X81.400 Y81.400\nG1 F1800 X118.600 Y81.400 E5.69053\nG1 X118.600 Y118.600 E6.11155\nG1 X81.400 Y118.600 E6.53257\nG1 X81.400 Y81.400 E6.95359\nG0 F9000 X81.800 Y81.800\nG1 F1800 X118.200 Y81.800 E7.36555\nG1 X118.200 Y118.200 E7.77751\nG1 X81.800 Y118.200 E8.18948\nG1 X81.800 Y81.800 E8.60144\nG0 F9000 X82.200 Y82.200\nG1 F1800 X117.800 Y82.200 E9.00435\nG1 X117.800 Y117.800 E9.40726\nG1 X82.200 Y117.800 E9.81017\nG1 X82.200 Y82.200 E10.21308\nG0 F9000 X82.600 Y82.600\nG1 F1800 X117.400 Y82.600 E10.60693\nG1 X117.400 Y117.400 E11.00079\nG1 X82.600 Y117.400 E11.39465\nG1 X82.600 Y82.600 E11.78850\nG0 F9000 X83.000 Y83.000\nG1 F1800 X117.000 Y83.000 E12.17330\nG1 X117.000 Y117.000 E12.55810\nG1 X83.000 Y117.000 E12.94290\nG1 X83.000 Y83.000 E13.32771\nG0 F9000 X83.400 Y83.400\nG1 F1800 X116.600 Y83.400 E13.70345\nG1 X116.600 Y116.600 E14.07920\nG1 X83.400 Y116.600 E14.45495\nG1 X83.400 Y83.400 E14.83069\nG0 F9000 X83.800 Y83.800\nG1 F1800 X116.200 Y83.800 E15.19739\nG1 X116.200 Y116.200 E15.56408\nG1 X83.800 Y116.200 E15.93077\nG1 X83.800 Y83.800 E16.29747\nG0 F9000 X84.200 Y84.200\nG1 F1800 X115.800 Y84.200 E16.65511\nG1 X115.800 Y115.800 E17.01274\nG1 X84.200 Y115.800 E17.37038\nG1 X84.200 Y84.200 E17.72802\nG0 F9000 X84.600 Y84.600\nG1 F1800 X115.400 Y84.600 E18.07661\nG1 X115.400 Y115.400 E18.42519\nG1 X84.600 Y115.400 E18.77378\nG1 X84.600 Y84.600 E19.12236\nG0 F9000 X85.000 Y85.000\nG1 F1800 X115.000 Y85.000 E19.46189\nG1 X115.000 Y115.000 E19.80142\nG1 X85.000 Y115.000 E20.14095\nG1 X85.000 Y85.000 E20.48048\nG0 F9000 X85.400 Y85.400\nG1 F1800 X114.600 Y85.400 E20.81096\nG1 X114.600 Y114.600 E21.14144\nG1 X85.400 Y114.600 E21.47191\nG1 X85.400 Y85.400 E21.80239\nG0 F9000 X85.800 Y85.800\nG1 F1800 X114.200 Y85.800 E22.12381\nG1 X114.200 Y114.200 E22.44523\nG1 X85.800 Y114.200 E22.76665\nG1 X85.800 Y85.800 E23.08808\nG0 F9000 X86.200 Y86.200\nG1 F1800 X113.800 Y86.200 E23.40045\nG1 X113.800 Y113.800 E23.71281\nG1 X86.200 Y113.800 E24.02518\nG1 X86.200 Y86.200 E24.33755\nG0 F9000 X86.600 Y86.600\nG1 F1800 X113.400 Y86.600 E24.64086\nG1 X113.400 Y113.400 E24.94418\nG1 X86.600 Y113.400 E25.24749\nG1 X86.600 Y86.600 E25.55081\nG0 F9000 X87.000 Y87.000\nG1 F1800 X113.000 Y87.000 E25.84507\nG1 X113.000 Y113.000 E26.13932\nG1 X87.000 Y113.000 E26.43358\nG1 X87.000 Y87.000 E26.72784\nG0 F9000 X87.400 Y87.400\nG1 F1800 X112.600 Y87.400 E27.01305\nG1 X112.600 Y112.600 E27.29826\nG1 X87.400 Y112.600 E27.58346\nG1 X87.400 Y87.400 E27.86867\nG0 F9000 X87.800 Y87.800\nG1 F1800 X112.200 Y87.800 E28.14482\nG1 X112.200 Y112.200 E28.42097\nG1 X87.800 Y112.200 E28.69712\nG1 X87.800 Y87.800 E28.97327\nG0 F9000 X88.200 Y88.200\nG1 F1800 X111.800 Y88.200 E29.24037\nG1 X111.800 Y111.800 E29.50747\nG1 X88.200 Y111.800 E29.77457\nG1 X88.200 Y88.200 E30.04166\nG0 F9000 X88.600 Y88.600\nG1 F1800 X111.400 Y88.600 E30.29971\nG1 X111.400 Y111.400 E30.55775\nG1 X88.600 Y111.400 E30.81579\nG1 X88.600 Y88.600 E31.07384\nG0 F9000 X89.000 Y89.000\nG1 F1800 X111.000 Y89.000 E31.32282\nG1 X111.000 Y111.000 E31.57181\nG1 X89.000 Y111.000 E31.82080\nG1 X89.000 Y89.000 E32.06979\nG0 F9000 X89.400 Y89.400\nG1 F1800 X110.600 Y89.400 E32.30973\nG1 X110.600 Y110.600 E32.54966\nG1 X89.400 Y110.600 E32.78960\nG1 X89.400 Y89.400 E33.02953\nG0 F9000 X89.800 Y89.800\nG1 F1800 X110.200 Y89.800 E33.26041\nG1 X110.200 Y110.200 E33.49129\nG1 X89.800 Y110.200 E33.72217\nG1 X89.800 Y89.800 E33.95305\nG0 F9000 X90.600 Y90.600\n
\n", "Title": "Extruder retracts filament while moving from default start line to actual object location", "Tags": "|ultimaker-cura|g-code|extrusion|", "Answer": "

The problem is that you prime the nozzle, but don't reset the filament length.

\n\n

With your last priming action:

\n\n

G1 X20 E25 F1000

\n\n

You deposit a line that is followed by a move to the start of the skirt:

\n\n

G0 F9000 X80.200 Y80.200 Z0.200

\n\n

The printing of the start of the skirt is done by:

\n\n

G1 F1800 X119.800 Y80.200 E0.44818

\n\n

Notice the E term in your final priming line and the skirt print command; it goes from E25 to E0.44818, as these specify absolute movement, this results in a retraction of about 24.5 mm. What you should do is implement a G92 E0 after your last priming command to fix this incorrect retraction behavior.

\n\n
\n\n

Please also do note that you are using a very old version of the CuraEngine, maybe it is possible to update to a newer version as well; depending on the slicer you use.

\n" }, { "Id": "8388", "CreationDate": "2019-03-02T05:11:23.023", "Body": "

I bought an Ender 3, and after assembling it following the description and some YouTube videos and after correct leveling, I printed the test dog gcode on the micro SD card that comes with the printer. PLA 1.75 mm. Attached the image of the printing result. What went wrong? I didn't change or modify any settings what so ever, I just assembled the printer, and printed the test dog. Please help me, I am a beginner in 3D printing.

\n\n

\"enter

\n", "Title": "Ender3 printing tilted layers", "Tags": "|print-quality|troubleshooting|creality-ender-3|", "Answer": "

Ender 3's have a reputation to be not quite square on the base. Make sure that your X, Y, and X axis' are all perfectly squared.

\n" }, { "Id": "8397", "CreationDate": "2019-03-03T23:37:04.290", "Body": "

I am looking for the temperature rating for hardboard. I want to use that as the base for my printer enclosure.

\n\n

It has proven incredible hard get a ball-park figure from Google.

\n\n

So, what is the maximum safe temperature for a hardboard panel at long term? (considering a print job can easily take 6 hours).

\n\n

PS: if you have used a hardboard to build your enclosure, your experience might be helpful.

\n", "Title": "Using hardboard for 3D printer enclosure; what is the temperature rating of hardboard?", "Tags": "|enclosure|", "Answer": "

Masonite or hardboard is a high-density board without a resin. It is listed at around 200 to 275\u00a0\u00b0C for its autoignition temperature. Just for comparison, let's look at similar products.

\n

Medium-density fiberboard (MDF) is similar to hardboard but bonded with a resin, offering a smoother surface and is listed at 200 to 275\u00a0\u00b0C, so pretty close.

\n

The heavier High-density fiberboard variant is listed with about the same temperatures of about 200 to 275\u00a0\u00b0C.

\n

Oriented Strand Board (OSB) is listed at 200 to 260\u00a0\u00b0C.

\n

Conclusion

\n

All the materials have very similar autoignition temperatures, so none of them particularly shines over the other. Using a proper fire safety procedure is highly encouraged, even though the ambient temperature in the enclosure should not reach even close to the 200\u00a0\u00b0C unless the printer has a freak accident and catches fire itself.

\n\n

1 - The RS Catalogue was just chosen for ease of navigation. McMaster Carr does list special heat shielding tape in both thin and aluminium as well as copper and aluminium foil tapes. I am not affiliated to either.

\n" }, { "Id": "8421", "CreationDate": "2019-03-06T20:35:26.030", "Body": "

What is the make/model of a MakerBot Replicator+ motherboard?

\n\n

I'm assuming that the make is now MakerBot since they are now closed-source.

\n", "Title": "Replicator+ Motherboard Model", "Tags": "|makerbot|replicator+|", "Answer": "

It might be this board, MakerBot 5th Generation motherboard, (original image)

\n\n

\"MakerBot

\n\n

The image was very small, so it is rather blurry, I'm afraid.

\n\n

I have contacted the suppliers for confirmation, and will update this answer, when/if I get a reply.

\n\n

However, according to this eBay item, Makerbot Replicator Motherboard Carriage, it could be inferred1 that the same motherboard (MP6292) is used in the Fifth Generation and the Replicator+

\n\n
\n

This part is the aluminum carriage that attaches the main board to the\n printer frame.

\n \n

Compatible with Makerbot Replicator Fifth Generation (5th Gen) and\n Replicator+ (Plus).

\n \n

NOTE: Motherboard show here is for illustration purposes only!\n Motherboard (MP6292) is NOT included.

\n \n

Main board presses right onto the carriage. No special tools are\n needed for assembly. Retaining screw is included.

\n
\n\n

\"Makerbot

\n\n

A much clearer shot of the MP6292 Motherboard

\n\n

\"MP6292

\n\n
\n\n

1 However, that depends on whether you want to trust eBay descriptions.

\n" }, { "Id": "8430", "CreationDate": "2019-03-08T16:51:49.983", "Body": "

I want to 3d print my own icing smoothers, but I'm not sure if its safe to have plastic from a 3D printer in contact with cake icing. Is there any harm in this?

\n", "Title": "Is it safe to use a 3D printed icing smoother?", "Tags": "|safety|food|", "Answer": "

As mentioned, FDM 3D printed parts are flawed for food service and prep use regardless of the material, because of the small gaps between lines where bacteria can grow, and because your printer isn't used in a way to keep it from introducing contaminants into otherwise clean material.

\n

However, there are some ways around this.

\n

Single-use

\n

I'm pretty comfortable using my printer to make items for one time use. If I wanted an icing smoother with a fancy shape for a special cake that I'm not likely to need again soon, I'd go ahead and do that. I would apply all the icing at one time, and then I'd discard the piece, rather than try to clean and save it. Also note that I'm not talking about a commercial kitchen; this would be for a cake I'd eat myself with friends, rather than sell.

\n

The big thing I've done this way so far (I've had my printer less than a year) is make shaped cookie cutters. I'll print the cutters, use them, and then throw them away. If I want the same shape again some time, I'll re-print.

\n

Lining

\n

The other thing you can do with food prep items is print them with the intent to use liners. For example, here is a 3D-printable taco train, where a train car has grooves to hold tacos (yum!). It wouldn't be good to put a taco directly in here, but you could use napkins or similar food-safe liner to separate the 3D part from the food. In the case of the icing smoother, you might be able to print the part and wrap it in aluminum foil.

\n" }, { "Id": "8436", "CreationDate": "2019-03-09T23:12:47.037", "Body": "

I saw on a popular site simple indicators for attaching to the axle of an extruder motor (Prusa printers). Those gadgets are spinning due to movement of motor, both clockwise (pulling filament) or counterclockwise (retracting). But all of those things are attached to motor axis by small neodymium magnet (round, 8x3 mm). I searched informations how such magnets affects for stepper motors and I read that magnet field can significantly change magnetic field of the motor. On presented movies for gadgets which I saw, the motor seems to have no trouble with rotating but AFAIK neodymium magnets have really strong magnetic field and I am curious how its centric orientation due to motor axis:

\n\n\n\n

Currently I use Prusa i3 MK3 printer and during long (~10h) printings an extruder motor is enough hot and I don't want make it hotter. I want to print and attach such \"rotation indicator\" but the fact of used magnets made me started to thinking about magnetic field of motor. Or maybe those changes are so marginal to think about them?

\n", "Title": "How neodymium magnet affect (if any) on extruder motor", "Tags": "|extruder|motor|", "Answer": "

The best way to know is placing the magnet on the shaft; if you see that motor starts shaking or stops this mean that you affects the motor operation, but I think if any magnet interference can be deprecated due internal coils of the motor during operation.

\n\n

The motor works with some coils in the rotor and some magnets in the stator (motor frame). But the shaft is to far from the coils to be affected for the magnetism of one small neodimiun magnet which needs at least 7mm to trap another small metal objet or for induction sensing and 2mm as maximum for creating electrical flows (generation); this values are afected if the magnet is placed over other metal part reducing his atraction field, and the armor is too big for an small magnet 8x3mm.

\n\n

You can attach some indicator with a double sided adhesive tape (3M) if still are worried about affecting your extruder.

\n\n

If your motor extruder is heating during extended usage periods you should adjust the motor current, for example: you motors is rated 0.5A you should calibarte the current for 0.48A or 0.45; this could reduce the torque force of the motor by a little. Or maybe the current current :D is calibrated above 0.5A thats way you are getting over heating.

\n" }, { "Id": "8439", "CreationDate": "2019-03-10T18:38:49.920", "Body": "

I've been using my Ender 3 for about four months now and it's been working wonderfully. The print quality is amazing and all the prints are very strong. Then about three weeks ago, my entire system crashed while Ultimaker Cura was open and it lost the profile for my 3D printer. I recreated the profile to the best of my ability with other people's working profiles online, but none of them worked right. I've been getting severe under extrusion in all my prints, and they're incredibly fragile. For now, I've just been printing a 1\"x1\"x1\" test cube. I've tried many steps from other people's posts online to fix the problem, including:

\n\n
    \n
  1. Raising the print temperature for PLA to 200 °C.

  2. \n
  3. Checking the extruder for signs of too little tension or too much tension. I checked, and the PLA has light tooth imprints on it, and no grinding or damage to the filament.

  4. \n
  5. Clearing out the extruder. I disassembled the whole extruder assemble, and flushed all the plastic from each part with a heat gun, and metal pick, and then tried reprinting, but it didn't work.

  6. \n
  7. Trying a newer Ultimaker Cura version. At the time, I was using Ultimaker Cura 3.1 and hadn't updated because it was working well. I then tried the newest stable release of Ultimaker Cura 3.6, with a few different profiles, and then I also tried the beta version of Ultimaker Cura 4.0, but none of these worked.

  8. \n
  9. Increasing the extrusion rate. I incrementally increased the extrusion rate from 100 % all the way up to 130 %. The prints looked a little better and were a lot stronger, but this still didn't fix it.

  10. \n
  11. Trying a different slicer. I then downloaded Slic3r and created a new profile in that. The prints turned out a lot better, but there was still significant under extruding.

  12. \n
  13. Checking the filament tube for any burns or damage, and ensuring it's inside the extruder assembly all the way.

  14. \n
\n\n

If anyone can help me figure out what's going on with my printer, I'd really appreciate it!

\n\n

Here are some pictures of the prints I've been getting:\nThese were made in Ultimaker Cura with different small changes to the profile made

\n\n

\"Ultimaker

\n\n

These were made in slic3r with a flow rate adjusted up to 130%\n\"Slic3r

\n\n

These were prints I made before I lost all my settings in Ultimaker Cura.\n\"Good

\n\n

Here's some of the material I read/watched and checked before posting myself:

\n\n\n\n

Edit: Here's my printer profile:\nEnder 3 Profile Google Drive

\n", "Title": "Ender 3 severe under-extrusion", "Tags": "|print-quality|ultimaker-cura|slic3r|creality-ender-3|", "Answer": "

My Man, The Ender 3 Marlin firmware saves your E-Steps on the SD Card and you basically have to check the E-Steps every startup because the Firmware sets back the E-Steps Value every shutdown to the minimum 93. Thats a known issue of the Enders and still on the Mk.3 aka Ender 3v2 present. It's an easy fix.

\n" }, { "Id": "8443", "CreationDate": "2019-03-11T07:41:45.090", "Body": "

As an electrician newbie I have a question..

\n

Short question

\n

My power plug is attached to the wall outlet and three wires are at the end of it.. the blue, the brown and the green/yellow oen.

\n

How to determine which is the live, zero and ground wire WITHOUT depending on the colouring?\nI want to know this to always be able to check the wiring, if I don't trust it.

\n

Also, I have these sub questions that follow from my situation:

\n\n

Situation

\n

My tools:

\n\n

In the manual it states that

\n\n

I know that in some countries this colour coding is the standard and can be trusted. I just want to be able to check it.

\n

So, my first hypothesis would be.. if I put the test screw driver on the live (brown) wire, lay my thumb on the end, the internal bulb would glow.\nThis did not happen.\nIt did happen when I put it on the zero (blue) wire. So I am a bit confused by this.

\n

My second trick was using the multimeter. Using the positive and negative probes to determine the polarity and therefore decide how the current was flowing.\nBut there was no sign of polarity... duh.. because I was of course on AC, which is always Alternating.. hence no sign information from a multimeter.\nSo, how CAN I use a multimeter to determine the live/zero line? Maybe measure the current from live/zero to ground? Is that a safe option?

\n

PS: my first post on this forum.. so please correct me where needed

\n", "Title": "Anet A6 stock power wires.. the live, the zero and the ground. Which is which", "Tags": "|anet-a6|wiring|", "Answer": "

This picture shows the correct wiring for a British 13A mains plug:

\n\n

\"UK

\n\n

The green/yellow wire must always be connected to the earth terminal (at the top of the plug). This is most important, and will prevent electrocution if something goes wrong.

\n\n

The brown (live) wire should be connected to the fuse, and the blue wire to the remaining terminal. It is true that, since alternating current is being used, the device will likely work if the blue and brown wires are swapped, but it is better to stick with standard practices to avoid confusion.

\n\n

I would not use a \"test\" screwdriver to determine which terminal is \"live\" in this instance, simply because it is unnecessary. The live terminal is always the one that the fuse is connected to. This is done for safety reasons since the device is automatically disconnected from the live side of the circuit if the fuse blows.

\n\n

How to wire mains plugs is really beyond the scope of this stack. Watch appropriate YouTube videos if you want to make a competent job of it. The only advice that I will give is to always use a proper fuse of the correct rating for the device. Never use something like a paper-clip. Also, don't use the 13 Amp fuse that comes with the plug. The fuse is there to protect the cable, rather than the device. Most printers will draw less than 5 Amps, and will be supplied with a 5 Amp cable, so use a 5 Amp fuse.

\n\n

Also, never work on a plug while it is connected to the power socket. This is most foolhardy, even if the socket appears to be switched off.

\n" }, { "Id": "8447", "CreationDate": "2019-03-11T12:38:14.827", "Body": "

Over the last two days I have tried everything I could come up with to fix the following issues.

\n\n

It all started with bad bed adhesion.(pic.1) Solutions for these problems are readily available, so I calibrated my Z-axis (did the whole wizard from start to finish again) and started a calibration print over the whole buildplate, getting an interesting result.(pic.2) As you can see some area's are squashed nicely whilst others come off directly.

\n\n

I then started looking for alternative Z-levelling solutions and came across an alternative calibration file (link1), at first glance the results seemed allright, -0,8mm seem to provide the best results (pic.3) however notice the darkened area to the left (could be the high temp of this test? 225c?).

\n\n

To validate the number I did a full print on -0,8 and this is where it gets weird, the result shows a non uniform transparancy (pic.4) the mechanical properties are also a lot less. I used my caliper to check the printer and all seems to be well within expected ranges.\nThe bed is also flat, checked with a ruler for deformaties.

\n\n

Printing with the first spool of Prusa fillament, do notice some changes in the Z-level calibration print, the edges now bend upwards, this is done with the same Gcode and spool as previous (pic.5).

\n\n

Any advice on what to do next? \nPrinting on 210c with fan off for the first layer, test square on 225c, all PLA.\nCleaned the bed with Acetone (99% pure) before each attempt.

\n\n

Picture 1:\n\"enter\nPicture 2:\n\"enter\nPicture 3:\n\"enter\nPicture 4:\n\"enter\nPicture 5:\n\"enter

\n\n

EDIT: So Mick mentioned cleaning the nozzle which helped with the inconsistent first layer. In the end it was me being unaware that Aceton doesn't help with grease, a thorough scrubbing of the buildplate with soap and really hot water helped.

\n\n

\"enter

\n", "Title": "Prusa I3 mk3 - Not your average inconsistent first layer", "Tags": "|prusa-i3|", "Answer": "

The Prusa i3 is coated with a PEI sheet. PEI and other build plates stick to the build but don't like to be dirty. Fingerprints can build up and create an interference layer of fats that lessen the adhesion to a point the pieces spontaneously pop off.

\n\n

A good cleaning is often needed. For PEI Isopropylic Alcohol is a fast cleaner, but if you can remove the build plate soap and water work too - but take care not to go too hot as PEI can get damaged. For BuildTak and similar, Acetone also can do the job.

\n\n

When using Gluestick to create a deliberate destroyable adhesion layer (for filaments fusing with PEI that can result in chipping out chunks) clean the surface afterward with a little water. In the choice of gluestick, make sure you get a PVA based one.

\n" }, { "Id": "8466", "CreationDate": "2019-03-14T12:58:11.420", "Body": "

What is Thermal Runaway Protection (TRP) and why should I enable it?

\n\n

How does one do so in Marlin?

\n", "Title": "What is Thermal Runaway Protection?", "Tags": "|marlin|firmware|safety|knowledgebase|", "Answer": "

What is Thermal Runaway?

\n

Let's look at a Thermal Runaway Test (#2) performed by one Chris Bate.

\n

In this video the experimenter drove the heating element non-stop until disaster. The Nichrome wire in the heating element melts at about 1,400 \u00b0C. Only once it melts, will the circuit will break and the current stop. The aluminum heating block however, melts at 660.3 \u00b0C; long before the nichrome melts.

\n

Thermal Runaway Protection

\n

Thermal runaway protection is a piece of code in the firmware of the printer that checks to make sure that once power is being applied to the heater, the thermistor's resistance is changing within a specified frame (time and amount). This is the basic form of a control loop.

\n

If the control system is implemented mechanically then it is called a thermostat, usually via a bimetal strip.

\n" }, { "Id": "8470", "CreationDate": "2019-03-14T15:27:39.700", "Body": "

This question asks about the reasoning behind thermal runaway protection and how to turn it on. However, how do I know if my printer does not have protection in the first place? Either because it is shut off by default or, if such printers exist, is not a feature of the printer.

\n\n

For example, the Creality CR-10S, a popular family of printers, has TRP not enabled (or deactivated) for some reason. Since there are a handful of printers (Creality, Tevo, Anet, Prusa, Ultimaker, etc) that seem pretty ubiquitous in the community and are recommended for beginners, it seems like it'd be handy to know whether or not any given printer has this safety feature.

\n", "Title": "Are there printers that don't have thermal runaway protection?", "Tags": "|safety|", "Answer": "

At the time of this writing (March 2019), many (if not most) cheap printers from the Far East are not delivered with Thermal Runaway Protection enabled as Marlin had this feature disabled as default for a long time (it was for certain so in April of 2018).

\n

I know that Anet printers (A8 from experience) and the Creality Ender-3 printer (experience from another member) come with TRP disabled in the firmware when shipped from the factory. Thomas Sanladerer did a test on his machines and found that it was disabled on the Creality CR-10, Anet E-10 and A-8 had it disabled while the Mini Fabrikator v1 did have no Mintemp/Maxtemp but Thermal Runaway. Even the quite expensive BCN3D Sigma R17 had it disabled in April of 2018 on default.

\n

Among the printers that come with Thermal Runaway Protection enabled are the PrusaResearch builds of the Original Prusa i3.

\n

To test if it is enabled on your printer, you could disconnect the heater elements prior or during printing, see this answer or the process explained in Thomas Sanladerer's Video above or from the start of his safety tutorial:

\n
\n

How to test if TRP is active on my printer?

\n

To test if thermal runaway protection is enabled on your printer, you\ncan disconnect the heater element of the hotend or the heated bed. You\ncan disconnect the heater element while the printer is cold (before\nstart) and also when the heater element is heating up. No heating of\nthe nozzle will take place, so after the period defined by the time\nconstant set in the firmware, the printer will halt if thermal runaway\nprotection is enabled. Power down the machine and reconnect the wires,\nit is not advised to put them back in; when the printer halted, you\nshould power down or reset the printer anyways. If the printer did not\nhalt, power it down as quickly as possible.

\n
\n" }, { "Id": "8480", "CreationDate": "2019-03-16T20:13:59.513", "Body": "

When printing cube.gcode using Pronterface I get:

\n\n
Print started at: 23:07:31\n
\n\n

After 8 seconds I get:

\n\n
Error:Printer halted. kill() called!\nError:Printer halted. kill() called!\n
\n\n

What is the reason?\nHow can I see logs?

\n\n

Additional information:

\n\n

I connected only three motors, and set the following in Configuration.h of Marlin Firmware:

\n\n
#define TEMP_SENSOR_0 999\n...\n#define TEMP_SENSOR_BED 999\n...\n#define X_MIN_ENDSTOP_INVERTING true // set to true to invert the logic of the endstop.\n...\n
\n\n

(Also for min max for XYZ)

\n\n

In 8 second X motor and Z motor were running.

\n\n

This is the link to G-Code print file (cube.gcode).

\n\n

I'm using Marlin 1.1.x on a Arduino Mega 2560 + RAMPS 1.4 without endstops.

\n", "Title": "Error:Printer halted. kill() called!", "Tags": "|marlin|troubleshooting|pronterface|", "Answer": "

Your G-code file contains the G28 command to home the printer, as you do not have endstops, execution of this command fails. If you do not use endstops, you should never home the printer, instead you must position the print head at the origin yourself (generally this would be at coordinate [x=0, y=0, z=\"paper thickness\"]) and remove homing commands from your G-code file.

\n" }, { "Id": "8482", "CreationDate": "2019-03-17T02:26:19.157", "Body": "

I have three stepper motors.\nOne Nema 17 - 2.4 ohm, the second smaller noname from color printer - 9.5 ohm and third the smallest noname from cdrom - 10.5 ohm.

\n\n

I have connected them to arduino mega 2560 with ramps 1.4(set to 1/32 micro stepping) and drivers drv8825. See my previous question.

\n\n

After some time (less than one minute) the first is cold. The second motor is hot. And the third is very hot. I can not even touch it.

\n\n

What can I do to fix it.

\n", "Title": "After some time stepper motor is hot", "Tags": "|marlin|diy-3d-printer|troubleshooting|ramps-1.4|", "Answer": "

Unless you have changed the factory stepper driver settings, they will all be set to deliver the same CURRENT to the motors. Stepper drivers operate as constant current supplies, so the voltage supply does not determine the power sent to the motor.

\n\n

The power dissipation in a circuit is the current squared times the resistance. $P = I^2 R$. Because the current is constant, the 10.5-ohm motor will dissipate over four (4) times the power of the 2.4-ohm motor and will get much hotter more quickly.

\n" }, { "Id": "8504", "CreationDate": "2019-03-20T07:54:06.297", "Body": "

I want to know if it is possible to schedule a 3D Print to start at a specified time so that I don't have to press start. \n
\nMy 3D Printer has an Arduino mega based RAMPS 1.4 control board & is connected to a Raspberry Pi 3B running OctoPi 0.15.1

\n", "Title": "Can I schedule prints to start at a specified time?", "Tags": "|octoprint|", "Answer": "

There are a few options to delay starting using either the OctoPrint environment or directly use G-code for this.

\n\n

The use of G-code is probably the most simple for you to implement. The G4 command defines a \"dwell\" or pausing period for the next command to start:

\n\n

\"G4

\n\n

Depending on the firmware you use, you can use the P or S parameter to specify the pausing period.

\n\n

To pause the print job (for 1 hour) you need to insert the following line as the first line of your G-code print file:

\n\n
G4 P3600000 // One hour pausing; defined in milliseconds\n
\n\n

or (if your firmware supports)

\n\n
G4 S3600 // One hour pausing; defined in seconds\n
\n\n

Other solutions may include the adaption of the OctoPrint controls menu structure or use of the REST API of OctoPrint. These options are more difficult to implement.

\n" }, { "Id": "8506", "CreationDate": "2019-03-20T12:43:03.293", "Body": "

I'm trying to figure out why the print speed changes at one point in this model.

\n\n

I'm using Ultimaker Cura 3.6 and, as shown in the picture, I set the speed to be the same across the entire print. Is there a setting I'm missing?

\n\n

\"Screenshot

\n\n

\"enter

\n\n

0scar is right. Changing the minimum time per layer to 1s (from 3s), makes the entire print print at the same speed, as per settings. However, it's probably a bad idea:\n\"enter

\n", "Title": "Why does the print speed change at specific height? (Using Ultimaker Cura slicer)", "Tags": "|ultimaker-cura|speed|", "Answer": "

Not being able to see the rest of the model (from the first image), if looks as if the light green sliced area displaying a lower speed for the top of the cylinder, is the only part that need to be printed to that height (now confirmed in the second image). This speed reduction is done by the slicer and is not specifically caused by Ultimaker Cura (other slicers do result in similar behavior). Note this is a good thing! Lets explain.

\n\n

Filament needs an amount of time to cool before the next layer is deposited onto the previous layer. When the layers get small (surface area) and there are no other layers the print head shifts to, the print process is slowed down to allow the filament to cool down; hence you see a decrease in print speed. If you deposit too fast, the last part of your print will become too hot and will deform.

\n\n

A print parameter that influences the behavior is the Minimal Layer Time parameter in Ultimaker Cura, please read the hint information of this parameter:

\n\n

\"Hint

\n" }, { "Id": "8512", "CreationDate": "2019-03-21T01:31:57.710", "Body": "

I have some flexible PLA filament (https://www.amazon.com/dp/B00VKSSA4E/, presumably a mix of PLA with some platicizer) that's supposed to be easy to print with settings similar to regular PLA. I've seen recommendations to disable retraction, and indeed I get huge failures to extrude at all for a while after retraction if it's enabled. But with retraction disabled, I get stringing all over the place, and since the material isn't brittle, it's really difficult to remove.

\n\n

I'm using a bed temp of 60 and print temp of 220, increased from 210 for normal PLA since I had trouble getting it to adhere at lower temp. Printer is Creality Ender 3. Using CuraEngine for slicing. The extruder is feeding the material fine; there's no kinking going on or anything.

\n\n

Where should I start trying to improve this? Might retraction work with a really really slow print speed or greatly reduced retraction distance? Or are there other ways to avoid stringing?

\n", "Title": "Avoiding stringing with flexible filament", "Tags": "|print-quality|filament|flexible|", "Answer": "

OK, this turned out to be really idiotic. The main source of the problem was Cura's custom start gcode for my printer (Ender 3), maybe duplicated for other printers too: it crams a massive amount of filament through the extruder to prime it. With flexible filament, this doesn't actually extrude most of it; rather, it just builds up as pressure in the bowden, which makes the first few [tens of] layers ooze like crazy, and then the hot nozzle picks up the ooze and drags it all over the place, and yay, stringing everywhere!

\n\n

A previous version of this answer (see edit history) described a lot of options I was using to try to improve things, some of them rather dubious. Really, the core of the matter, which can manifest in lots of ways, is that if pressure builds up in the extruder due compression of the material being easier than forcing it through the nozzle, it will ooze during travel, or as a glob prior to travel during ineffective retraction, and either way it will eventually lead to catastrophic stringing. So to solve this, I needed to address all the ways it could happen.

\n\n\n\n

In summary, the settings that work for me are:

\n\n\n\n

And the results:

\n\n

\"test

\n\n

\"frog

\n\n

\"frog

\n" }, { "Id": "8513", "CreationDate": "2019-03-21T03:38:17.487", "Body": "

So I got my 3D printer (Anet A8) a few years ago and put it together. I couldn't figure out how to use it. Then about a month ago I decided to try again now that I am older and got it to work. I know more than I did by far. I have printed about 15-20 different times now, but then this problem occurred just now that I have never seen before. I tried looking up what it could be and google results in nothing and this exchange leads to one thread that is close to what I am having difficulty with but doesn't really help. I got the stl file from the internet on thingverse, so I am fairly certain they must have clicked the center and arrange when in the cad software to make the item, which is a phone stand. So my print all of a sudden starting printing in air. Here is a picture and thanks for the help.

\n\n

\"Printer

\n", "Title": "Printer is Confused-Printing in Air Literally", "Tags": "|extruder|software|anet-a8|hardware|print-failure|", "Answer": "

Here's the pattern.

\n\n

The problem is, you didn't slice the g-code using support. It won't print right without it.

\n\n

\"From

\n" }, { "Id": "8519", "CreationDate": "2019-03-22T01:14:22.780", "Body": "

I'm using Ultimaker Cura 3.6.0 and I am getting some annoying \"stringing\" on the first layer. This isn't new to 3.6.0 as I have seen it on pretty much every version I have used.

\n\n

What appears to be happening is that as it is laying down the first layer, it doesn't retract as it moves from one area to another. This leaves a trail of filament which then shows up as an unattractive line embedded in the print. (ignore the other extrusion issues in the example below)

\n\n

A similar thing happens on the top layer. It ruins the look of the nice smooth bottom I get from a glass bed.

\n\n

I don't have stringing issues elsewhere on my prints.

\n\n

I thought at one time I had seen a setting to control it, like \"retraction during travel\" or something, but now I can't seem to find anything that sounds like it in the dizzying array of settings.

\n\n

Does anyone know of a way to get rid it these lines?

\n\n

\"Example

\n", "Title": "Stringing during travel on first layer with Ultimaker Cura", "Tags": "|retraction|", "Answer": "

Yes, if you are using the Combing Mode option, please ensure it doesn't do this in the skin, for a leak/string free first layer, it is required to set the Combing Mode to Not in Skin.

\n\n

\"Ultimaker

\n\n

When the mode of the option is set to Not in Skin, combing is \"off\" for the skin; this implies that the material will retract and move in a straight line to the next print area. When material is retracted (and when properly tuned for your printer), the nozzle will not leak filament causing those (deposited) travel movement lines as indicated by the OP.

\n" }, { "Id": "8534", "CreationDate": "2019-03-23T05:29:42.700", "Body": "

Recently I have been doing more complicated math in OpenSCAD and I have run into something that I find strange. Take a simple math expression: 2 / 2 / 2. By any programming language this will equal 0.5 (1/2), and OpenSCAD agrees. Something like this: 2 / -2 / 2 should also be -0.5 for the same reason. However, OpenSCAD thinks this is -2. That is echo(2 / -2 / 2); gives ECHO: -2. My calculator, other programming languages (and myself) all say its -0.5.

\n\n

Is this a quirk of OpenSCAD, or am I missing something obvious?

\n", "Title": "Why is 2 / -2 / 2 equal to -2 in OpenSCAD? (Mathematical Order of Operations)", "Tags": "|openscad|", "Answer": "

I filed this issue as a bug with the OpenSCAD project, and there is now a fix merged into master, as well as a test case to prevent regression. The latest nightly builds should handle this correctly from here on out.

\n" }, { "Id": "8539", "CreationDate": "2019-03-25T02:17:45.767", "Body": "

3D-printing newbie here. I have a Geeetech's Prusa i3 mk2 B.

\n\n

I'm trying to print this: https://www.thingiverse.com/thing:1358311

\n\n

That's a mold, with 2 external parts and a core. The exterior prints wonderful. But the core is too messy. Take a look at this:

\n\n

\"enter

\n\n

What riddles me is that the side parts, and the pole's base, print fine; so this does not look like a bad calibrated printer, but something else entirely. My guess is this is some precise tuning I don't know yet.

\n\n

I've been trying to print that little pole without success for over a week now. Tried all this:

\n\n\n\n

So far, the best I got is a little pole not-too-deformed so I can make my part anyway, even when the pole is not well printed. But after seeing lots of videos and reading lots of tips online, I still don't understand how to tune my print for that simple little pole.

\n\n

Other that tips, what I would really like to ask is if somebody has a name for that problem I'm facing, so it would be much more easy to search for my tuning options.

\n\n

So... any clue how to fix this?

\n", "Title": "Trouble printing small thin pole", "Tags": "|prusa-i3|ultimaker-cura|slic3r|geeetech|", "Answer": "

I'm sorry for the long time I took to answer, but the ammount of NEW problems I faced since started this question was so big that wanted to write a list of them before answering my own question. I've finally also desisted from the list, as the problems tends towards infinity.

\n\n

Simple version: printing more than a single item did the trick, as several people proposed here. But then the piece broke when tried to remove it from the hot bed, as somebody else also warned me here. By that time I tried a temp tower, had to recalibrate almost the whole printer, had to also relubricate it, had to learn some tricks about sticking and de-sticking pieces from the bed, tried several different glues, started to fine-tune my slicings, and so on and so on... man, 3D printing is laborious.

\n\n

But I wanted to let this link here for all the fellow newbies out there reading this.

\n\n

https://all3dp.com/1/common-3d-printing-problems-troubleshooting-3d-printer-issues/

\n\n

That was exactly the kind of document I was looking for online as a newcomer to this world, and I'm sure it will be useful for a lot of people.

\n\n

Thanks everybody.

\n" }, { "Id": "8545", "CreationDate": "2019-03-25T10:20:22.043", "Body": "

I have an Anycubic i3 Mega-S and when I'm printing something (especially when it's hotter) the temperature increases and decreases by around 3 degrees during the print. Is this normal?

\n\n

I was worried this is a loose thermistor and if it comes out during a print my house will probably catch fire! I have no idea how to tighten it though.

\n", "Title": "Temperature fluctuations, is it normal?", "Tags": "|prusa-i3|hotend|anycubic-i3-mega|", "Answer": "

All printers will have some fluctuation and it's not a concern. This is similar to the fluctuation you'll see in your home temperature around the thermostat setting. There are a couple reasons. One is that the feedback loop (thermistor to control board to heater to extruder block) will always have some lag. Another is that most systems have built-in \"hysteresis,\" i.e. set the \"turn off heat\" a couple degrees above the \"turn on heat\" thresholds. This avoids \"chatter\" from on to off right at the setpoint.
\nSome thermistors (e.g. my AnetA8 clone) can be held in place with a setscrew. If yours doesn't have a similar capability, use Kapton tape to hold it in. This tape is designed for high temperature operations so it won't age or collapse, and it has good holding strength.

\n" }, { "Id": "8548", "CreationDate": "2019-03-25T11:29:23.553", "Body": "

At the moment the outside surface temperature is around 30 \u00b0C. Can I put my PETG spools outside with a fan in order to dry them cheaply?

\n", "Title": "Can I dry PETG filament under sun?", "Tags": "|filament|quality|petg|", "Answer": "

Here is my suggestion for a cheap, well and temperature accurate drying manner of a filament. I've done it several times for PETG that is actually a very moisture absorbing filament and pops it up when using by ruining the job. Why not using your own printer's heat bed to dry the filament (PETG about 6 hours in 65 degrees of Celsius)? I can guarantee that it works:\n\"PETGs

\n" }, { "Id": "8553", "CreationDate": "2019-03-25T17:45:23.737", "Body": "

I am currently running my Tronxy X5s with a MKS Gen L board. So far I have not ran the heat bed over 50 degrees C since I have only printed with PLA so far. I plan to try PETG and/or ABS in the near future and I have a spare power supply 12V/360W laying around.

\n\n

When I first got my printer I purchased this external MOSFET board after reading about X5s \"upgrades\", but so far have not used it.

\n\n

\"enter\n\"enter

\n\n

My plan is to now run the heat bed using a separate power supply than the one running my control board using the external MOSFET to switch it. Since this power supply will only be powering the bed, I would like to bump up the voltage, via trim-pot somewhere from 12V-15V, to gain some watts per square inch on my heat bed.

\n\n

Will this MOSFET isolate the heat bed circuit from my control board to allow it be ran at a higher voltage? Is it safe to run the power supply/heat bed at a higher voltage than it is rated for a significant amount of time?

\n", "Title": "Will this MOSFET allow the heat bed to run at a different voltage than the control board", "Tags": "|heated-bed|power-supply|mosfet|", "Answer": "

For reference,I done tracing this module as shown.\n\"enter\n\"enter\nSo you can use isolated power supply for load.\nAlso you can use up to 24V without any problem.\nEdit:\nThis module still work with higher voltage up to 24V. But according to question. Using higher voltage supply more than rated is same as my question here.\nFor summary. Adding more voltage to resistive load results to higher current flow. Recheck your wire and connectors for current rate so you will know the limit of voltage you can go with.\nOne more thing, mosfet can fail like short circuit. At this situation the temperature of bed cannot be controlled.\nAt normal voltage rate even we left the bed connected to power supply the temperature will rising to about 100C if you add more voltage it will go higher than that can the heated bed will broken or burned or start to fires

\n" }, { "Id": "8568", "CreationDate": "2019-03-28T04:58:13.313", "Body": "

The question is simple - I have a RAMPS 1.4 running Marlin 1.1.9 with the three MOSFETs being used (end, fan, bed), but I'd like to have a couple other Marlin-controlled fans. One of them would be a 4-pin, 6000RPM cooler I got from a dead graphics card. Seeing how it runs perfectly at 12V 350 mA if I keep the control pin disconnected (and ignore the sense pin too, of course), Could I connect 12V and GND directly to the PSU (or RAMPS 12V header) and the CONTROL pin to one of the pwm servo control pins like D11? Or do I need a resistor?

\n\n

I could add some info about the fan if needed, but it's a FirstD 4-pin, 12V 0.35A fan that can run up to 6000 RPM.

\n", "Title": "4pin fan on RAMPS board - direct?", "Tags": "|ramps-1.4|print-fan|fans|", "Answer": "

Yes, this should work.

\n

According to this 4-pin fan specification, such fans use a 5V PWM signal.\nYou would have to make sure the PWM frequency on the pin you use satisfies that 21-28 kHz range specified in the document.

\n

According to 3.3 and 3.4 in the document, you may not be able to turn the fan off completely when using the PWM input signal.

\n" }, { "Id": "8585", "CreationDate": "2019-03-30T21:40:57.987", "Body": "

My printer (Prusa i3 Mk3 rework clone, with a 200 x 200 MK3 ALU-heatbed, RAMPS 1.4) has a car relay for heated bed control. It works just fine using bang-bang on Marlin, but I would like to replace it with a MOSFET anyways. The specialized MOSFET heating modules are way too expensive in my country, so I was thinking about buying a MOSFET that's good enough to handle my 12 V 10 A bed with Marlin's PWM.

\n\n

Would a MOSFET like the STP80NF70 be enough (0.01 \u03a9 at vgs 10 V, 68 V 100 A) or would one of the bigger ones, e.g. in TO-3P encapsulation be needed?

\n\n

It would go in a proper heatsink and ran at 12 V with an optocoupler

\n", "Title": "Minimum MOSFET rating for MK3 heatbed", "Tags": "|prusa-i3|heated-bed|mosfet|", "Answer": "

and welcome to the Stack Exchange 3D Printing site.

\n\n

Let's look at the specs compared with the requirements.

\n\n

How much power is used by the heated bed? You have specified 120W (12V, 10A). That information makes it easy! On my homebrew printer it is higher, but that seems like a good high limit for a typical i3-style design.

\n\n

So, the MOSFET you propose, assuming you completely turn it on, will dissipate 0.01 ohms * 10A^2, or 1 watt. This will warm the part, and you probably should attach it to a heat sink, which could be as simple as the existing extrusions. There is a much more complete answer to this exact question on the Electrical Engineering Stack Exchange site: Dissipating 1W on a TO-220 without heatsink?

\n\n

I doubt the PID frequency is higher than 40 KHz. Since the Vgs rating you referenced is specified with 10V on the gate, you can't drive that directly from an Arduino pin. You need another driver transistor (NPN or FET) to drive the power FET gate. The turn-on time for the power FET will be determined (in the simplest circuit) by the pull-up resistor to +12v connected to the output of the transistor and the gate of the FET.

\n" }, { "Id": "8594", "CreationDate": "2019-04-01T18:37:48.310", "Body": "

I know I did something stupid. I just had to have a SLA 3d printer. The issue being I live in a one bedroom apartment. In the months of owning it I have made lots of amazing pieces, I also for the first time in my life have not only allergies, but sever allergies. I thought I had the flu, and has been most of the last 2-3 months. After making the connection to the symptoms appearing after I got the printer, I sealed the resin vat and removed all cleaning station items from my living space. I had thought I had done \"enough\" by sealing the printers door, and making sure I could not smell any chemicals, and getting a chemical grade air filter.

\n\n

It's been 2 days, and I'm instantly recovering from my symptoms, and have discontinued allergy medication.

\n\n

Other than not own this type of printer, what kind of setup do I need so that I can safely use this printer? Does anyone sell enclosures or setups for businesses or homes that will solve this issue?

\n\n

I can move the cleaning station to my balcony, as it also has a sink and space. Placing the printer even in an enclosure outside would be hard due to the humidity and extreme pollen we get here.

\n", "Title": "Best way to deal with Resin Printers in your living space", "Tags": "|safety|sla|enclosure|resin|", "Answer": "

First things first: Resin is very aggressive. It can very easily make you hypersensitive, even to the fumes of it. So step 1 is easy:

\n

Limit exposure

\n

Wear gloves when working with resin. As you live with your printer in the same room, bottle up the resin right after use and only open it during use to prevent buildup over time and exposure. To further reduce the exposure, leave the room while printing if possible and ventilate the room after bottling the resin again. Possibly even wear breathing protection during operation.

\n

Enclose and seal the machine

\n

To keep the vapors away from you, the machine needs to be enclosed airtight. Any lids need to get a seal, non-opening joints of the frame need to be sealed with a sealant like silicone. Often it is hard to retrofit an enclosure to seal up the workspace.

\n

If you want to enclose the full machine, I suggest using glass sheets and silicone sealant for the whole inside. Brace the construction from the outside with L-profiles along the corners and joints. The most tricky part will be the opening hatch and wiring/ducting access hole. The opening seal needs a sealing lid all around that gets compressed on closing the machine up and some kind of lock to keep it this way. The air filtration and wire access are just hard to make because of their circular shape. You might want to use a wooden or metal base plate, so it is best to put ventilation through the base. In case of wood, afterward coat the inside surface with a thin layer of an airtight material, such as epoxy resin or silicone.

\n

Low presssure operation by ventilating the machine

\n

The next best thing to isolating the machine workspace from the air completely is to make it a low-pressure operation. This means that you evacuate the air from the machine. The imperfect seals now work against a high pressure outside and low pressure inside, meaning that the flow in any non-sealed spot only knows one direction: into the machine.

\n

Ventilation outside...

\n

Fred's answer provides good basics on how to do this in general by using parts for Laser evacuation. This is also the most space-economic way.

\n

...and filtering.

\n

But there are (partial) indoor solutions even, based on ventilating the air from the printer into a multi-stage air filter could reduce or eliminate the amount of chemical exposure. This is not a slim piece of foam, it is a boxy setup with about 3 to 6 stages of filtering. Among dry-filters, a paint-filter in combination with an active coal one should eliminate a large portion of irritants from the stream, but might still need to be vented outside to reduce exposure even more. A 'wet' air filter, where the exhaust of the machine is pearled through a basin of a cleaning liquid (often water or a solvent like isopropylic alcohol) like in an aquarium could help to catch even more chemicals but is bulky.

\n" }, { "Id": "8605", "CreationDate": "2019-04-02T21:07:45.463", "Body": "

I'm having a problem where the relative dimensional error of cylinders is rapidly increasing as the absolute size decreases. Printing a calibration stack of cylinders of diameters 8, 7, 6, 5, 4, and 3 mm, they come out undersized by 5%, 5.3%, 7%, 8%, 10%, and 13%, respectively, as measured by a digital caliper. Cubes exhibit a less severe version of the same pattern: 2.5%, 3%, 5%, 4%, 5%, 7%. The cylinders are generated with OpenSCAD using $fn=180, i.e. they're actually extrusions of 180-gons, so the error should not be caused by poor chord approximation; indeed, measuring projections of the model, or reading the gcode and accounting for nozzle width, everything looks right.

\n\n

Printer is Ender 3, using PLA at 210. Slicing with CuraEngine.

\n\n

Could this be caused by underextrusion or print speed issues - or some effect where the material pulls itself together under tight curvature? What techniques might be able to compensate for it, short of fudging the model?

\n\n

Some additional information: As noted by Trish in the comments, the consistency of the absolute error, which is 0.4 for cylinders and 0.2 for cubes, is likely important. I've also subsequently tested with 110% extrusion rate and the errors for the cylinders dropped to consistently 0.2 mm (still a significant increasing relative error), but the skin layers at the top of the 3 mm cylinder bulged, suggesting the increased extrusion is wrong - an excessive total volume of material.

\n", "Title": "Progressively worse relative size error at smaller absolute sizes", "Tags": "|print-quality|calibration|creality-ender-3|", "Answer": "

I've had this problem recurringly after marking this solved before, so I'm writing a new answer to revisit it.

\n

Based on the magnitude of the error being absolute rather than relative, and not being direction-dependent (i.e. not backlash), it can't be caused by the motion system. I further ruled out inward warping of the material by running tests with 100% infill. When, even at 100% infill, the diameter (thus volume of material) was too low, the only remaining explanation was underextrusion.

\n

Measuring the theoretical rotation distance of my extruder hob (now a different extruder from the original Ender 3 one), accounting for approximate tooth depth along with a precise hob diameter measurement, I got a value 15% lower than my configured value (E-steps 15% higher). I was already aware there was a discrepency here, just not the magnitude, and had purposefully kept it lower because of concerns that the full value would lead to parts that don't fit, due to imperfections in the walls, and especially that it might make the undersized holes problem worse.

\n

In reality, it's done the opposite. The best explanation I can come up with for fixing holes is that, for extrusion widths narrower than the nozzle (which a 15% under-extruded 0.4 mm line is, since it's only about 0.34 mm), the line does not stay centered well, but gets dragged with the direction of curvature. I think I still have some problems with holes being slightly undersized, but they're at least no worse, and seem slightly better.

\n

As for positive cylinders, at 5 mm, printed diameter is nearly perfect, only under by about 40-50 \u00b5m, which is very acceptable and better than being over. Getting this result does depend on having a 100% solid part, though; as soon as infill comes into play, the cylinder is undersized again. Setting Cura's "Minimum Infill Area" high enough, or using enough walls to ensure the cylinder will be solid, can ensure 100% fill without printing the whole part at 100% infill.

\n

So, I think I can conclusively say the main likely cause for this problem is underextrusion due to inaccurate E-steps.

\n" }, { "Id": "8610", "CreationDate": "2019-04-04T03:14:38.240", "Body": "

The expert mode of Slic3r PE has almost a dozen speed values:

\n\n

\"Expert

\n\n

I understand that ultimately print speed at a particular temperature correlates with bond strength, so things like bridges and the first layer having different speeds are reasonable.

\n\n

What about the rest? Why does infill have 3 variations? How are the different categories listed impacted distinctly from any other form of layer/path bonding?

\n", "Title": "Why are there so many print speed parameters in slicers?", "Tags": "|slicing|slic3r|speed|", "Answer": "

There are multiple parameters because of the trade-off at print speeds.\nSlow printing will have less ghosting and a better overall quality, but it takes way more time.

\n

Fast printing will be fast (oh thank you, Captain Obvious), but you will get stronger ghostings and poorer line quality overall. Maybe some strange artifacts.

\n

But you don't need a good print quality in areas that are not visible at all, but you do need it for the outer shell. Also, is the travel mostly just something that you want done as fast as possible but maybe not so fast that that the whole printer starts shaking?

\n

So, you want to set different parameters for different parts.

\n

Next reason: most slicers are used for a large variation of printers, and every printer is a litte bit different\u2014even the same printer type with different addons (like a vanilla Anet A8 vs. a beefed up Anet A8). For the slicer to be able to be used by all these printers, it is necessary to be able to set multiple parameters.

\n

To get deeper into your question about the different infill speeds: if you print a solid infill you will also need to give the printed material more time to cool down, because it is packed tightly and there is not much surface area for heat to get away. If you print these areas too fast you will end up with a part that is too warm and warps and deforms all the time while the next layers were printed on already, inserting even more heat into the part and isolating the warm parts from the outside. You can observe similar behavior when printing overhanging areas with thick walls: there the part will stay in a semi-solid state, and deforms all the time.

\n

But how fast you have to go depends on the cooling system you have on your printer.

\n" }, { "Id": "8635", "CreationDate": "2019-04-06T19:11:09.137", "Body": "

When houses are printed with concrete cement what replaces the steel rebar for reinforcement?

\n

Here's a link referencing printing concrete: https://www.aniwaa.com/house-3d-printer-construction/

\n
\n

House 3D printers use extrusion technology. Some construction 3D printers look like super-sized desktop FFF/FDM 3D printers (gantry style), whereas others consist of a rotating mechanical arm.

\n

In both cases, paste-type components such as concrete are used as filament. The material is pushed out of a special nozzle to form layers. To put it (very) simply, paste extrusion is similar using a piping bag to spread frosting on a cake.

\n

The printer creates the foundations and walls of the house or building, layer by layer. The ground is literally the printer\u2019s build plate. Some concrete 3D printers, however, are used to 3D print brick molds. When molded, the bricks are then piled atop each other manually (or with a robotic arm).

\n
\n

Like most of the people here my experience is with a printer (RepRap) that can use PLA or ABS. With all the materials normally put into concrete, using an extrusion printer to print concrete is puzzling.

\n", "Title": "What is the reinforcement for 3D printing concrete cement?", "Tags": "|print-material|concrete-printers|", "Answer": "

To answer your question, they use steel or fiber glass.

\n\n

Why does concrete need reinforcement? Concrete is very useful to absorb compression loads, but breaks and cracks very easily when subjected to tension loads. Rebar, or steel constructions are added to strengthen the concrete to bear the tension loads. My house has walls that are littered with steel wires/fibers of about 10 cm (about 4 inches) and about 1.5 mm in diameter close to the surface (drilling holes into the wall frequently means hitting such a fiber which comes out and leaves a scar on the wall...). My walls are constructed in a factory flat on the ground, those fibers are added to prevent shrink cracks in the first layer of the wall (the side you always look at); after that, rebar is added and the rest of the wall is cast and after drying the wall is put upright and transported to the build site.

\n\n

Injection of fibers is not new to 3D printing; it is already possible to print short fibers embedded in filament, or continuous where filament and thermoplastic material join together in the nozzle. The company I work for creates molds and aerospace parts this way.

\n\n

You can imagine that it would be possible to print concrete with fibers/wires or continuously with wires. Technically this should be feasible, but I guess that larger diameter wire (rebar) is actually needed instead of relatively small diameter wires. Also, fibers can only be laid down in the direction of travel of the nozzle, not perpendicular to. Rebar often shows strengthening in multiple (three) dimensions, that is not possible with 3D printing.

\n\n

There is some reported success on printing reinforced concrete:

\n\n

According to BAM Infra:

\n\n
\n

\u201cThey have succeeded in developing a process to also print the steel\n reinforcement at the same time. When laying a strip of concrete the\n concrete printer adds a steel cable so that the slab is \u2018prestressed\u2019\n so that no tensile stress can occur in the concrete.\u201d

\n
\n\n

\"Concrete

\n\n

But this means that the print is a part printed at another location, not on location.

\n\n

There is also some success with using horizontal fiberglass reinforcements within the walls during the printing process.

\n\n

See e.g. this reference, this reference for more information on reinforced concrete printing.

\n" }, { "Id": "8641", "CreationDate": "2019-04-08T01:05:38.823", "Body": "

I've been wondering this for a while, and have searched for hours and have found nothing except undetailed explanations and projects.

\n\n

How does a UV LCD 3D resin printer cure resin exactly? I understand there is an LCD screen, but where does the UV light come from? Where is it placed?

\n", "Title": "How does a UV LCD 3D resin printer work?", "Tags": "|sla|knowledgebase|resin|", "Answer": "

I'm surprised your research hasn't answered your question, as the concept is relatively simple. You have most of the answer in the question. The missing item is a light source. Usually the source is an array of ultraviolet LED modules. There are resin printers that would not be called LCD printers, as they use computer display projectors to generate both the image and the UV to cure the resin.

\n\n

A rather extensive list of various resin printers can be found at aniwaa.com along with a clear explanation of the technologies.

\n\n

The image below is courtesy of the link in the previous paragraph.

\n\n

\"printer

\n\n

The light source that answers your question is visible in the third picture. As noted, the LCD panel blocks the light based on the image to be cured. I think the \"uses its own light\" is somewhat misleading, unless the builder has found a high-UV output LED LCD panel or is using daylight resins and has configured for long burn-in times.

\n" }, { "Id": "8662", "CreationDate": "2019-04-11T14:39:30.423", "Body": "

I'm interested in 3d printed reaction chambers, but can't find any good information on chemical resistances of PLA, just vague claims that it \"might not be\" \"because it's biodegradable\" or that it depends on additives (likely true, but it would be nice to know if there's hope of finding PLA without problematic additives if the PLA itself is okay). Is there any published research or even anecdotes (which could suggest it's worth spending effort to investigate further) on this topic?

\n", "Title": "PLA chemical resistances? Especially HCl, other acids", "Tags": "|pla|material|", "Answer": "

See this as well:

\n

Chemical Resistance of 3D Printing Filament by Ravi Toor

\n
\n

Summarised chemical resistance per material below (H= High Resistance, VH = Very High Resistance).

\n

Water \u2013 PLA (H) PETg (VH), CF-PETg (VH), ASA (VH), ABS (VH), ePLA (H), ONE PET (H)

\n

Acids \u2013 PETg (VH), ASA (VH), ABS (H)

\n

Bases\u2013 PETg (VH), CF-PETG (VH), ASA (VH)

\n

Alcohols \u2013PETg (H), CF-PETG (VH), ASA (VH), ABS (H)

\n

Hydrocarbons \u2013 N/A

\n

Ketones \u2013N/A

\n

Ethers \u2013N/A

\n

Fuels \u2013PETg (H), CF-PETG (H)

\n

Salts \u2013 PLA (H), ASA (VH), ABS (VH), ePLA (H)

\n

UV \u2013PLA (H), ASA (VH), ABS (H), ePLA (H)

\n

Oils \u2013 PLA (H), PETg (VH), CF-PETg (VH), ASA (H), ePLA (H)

\n
\n" }, { "Id": "8682", "CreationDate": "2019-04-14T15:19:04.527", "Body": "

My first layers started to have these \"seams\" in them.\nI am printing on the Anycubic Ultrabase.

\n\n

\"Seams

\n\n

Any other layer is fine.

\n\n

Has anyone else experienced similar problems or an idea of how to debug this issue?

\n\n

The picture is of PLA (BQ Easy Black) printed at 215 \u00b0C on the Ultrabase.\nThe the same issue with BQ Easy PLA red, turquoise and white (205 \u00b0C), Zaper PETG Brown 230 \u00b0C and HobbyKing translucent ABS Red (245 \u00b0C).

\n\n

Changing the nozzle did not change anything.

\n", "Title": "\"Seams\" in first layer", "Tags": "|ultimaker-cura|anycubic-i3-mega|anycubic-ultrabase|", "Answer": "

Normally, such patterns are caused by too close printing to the bed (nozzle to bed distance too small), but considering the not fully flattened out extrusion lines (on glass I get a mirror shine fully closed surface) this does not seem to be the reason; you could try to increase the gap a little, in Ultimaker Cura this is very easily done with the Z Offset Setting plugin. Alternatively, you could use a little thicker paper to calibrate the bed levelling.

\n" }, { "Id": "8690", "CreationDate": "2019-04-14T20:53:48.570", "Body": "

I changed the filament, and to adjust filament temperature, I printed a test model and it looked good:

\n\n

\"enter

\n\n

But printing another part did not go so well:

\n\n

\"enter

\n\n

\"enter

\n\n

After the failed print I ran another test: \n\"enter

\n\n

\u0415verything is done with the same settings.\nAnd I think the temperature and settings are okay.

\n\n

Is it possible to have a missed step on the Z axis, and this has caused the crushing of the layers or bad filament quality.

\n\n

Where does the problem come from?

\n", "Title": "Strange problem with quality, what might be the reason for these bubbles?", "Tags": "|print-quality|change-filament|", "Answer": "

From third picture - moisture!

\n\n

Is new filament cheap? I guess it was too long on stash and/or bad package.

\n\n

Look for Maker's Muse's video on Youtube about this topic.

\n" }, { "Id": "8694", "CreationDate": "2019-04-15T12:57:30.940", "Body": "

I'm trying to set up an enclosed (custom enclosure) Prusa i3 style printer for ABS, but having a fair amount of difficulty preventing the part corners from curling.

\n\n

\"Example

\n\n

Detailed specs:

\n\n\n\n

Verified heatbed temperature with calibrated IR imager.

\n\n

Thus far I have been unable to prevent parts from separating from the heatbed during print, primarily at the corners where stress is concentrated. I've tried various heatbed temperatures from 90 \u00b0C to 115 \u00b0C, lower hotend temperatures (which just made the problem worse and caused complete print failure), cleaning the PEI surface with alcohol, etc. to no avail. I'm even seeing this to some extent with Benchy, it shows up as a lift to the stern and bow (slight bend parallel to the keel) -- the print is otherwise basically perfect.

\n\n

I've attached an image of the more extreme curling -- yes, I should probably be using mouse ears on a part like this, but I see the same thing on parts that shouldn't require mouse ears.

\n\n

What is the best way to fix this particular problem? Temperature adjustments, brims, rafts, something else?

\n", "Title": "Best way to fix ABS corner curling on enclosed Prusa style printer with PEI heatbed?", "Tags": "|prusa-i3|troubleshooting|abs|", "Answer": "

A tall skirt (like 1 cm tall or more, even as tall as the part) few millimetres from the part would shield the corners and the outer parts from colder air and keep the part temperature high, reducing curling.

\n\n

It is in principle better than mouse ears, because these just pull the corners, which will still have a lot of internal stress when cooled, but if you keep the part warm as I suggest, the corner will stay flat(ter) by themselves, resulting in a stronger and better print with less internal stresses.

\n" }, { "Id": "8696", "CreationDate": "2019-04-15T18:03:11.117", "Body": "

I have an Anet A8 clone printer with a heated aluminum print bed. I'm considering purchasing a glass bed to add on top of the aluminum. When adding a glass bed on top:

\n\n\n", "Title": "When installing a glass bed, what do you change?", "Tags": "|heated-bed|anet-a8|glass-bed|", "Answer": "

The most common way to install a glass bed (assuming it's literally a piece of borosilicate glass) is with binder clips. Glass is an insulator, so you may need to adjust your bed temps by a few degrees, and it will take somewhat longer to warm up.

\n\n

You shouldn't need any firmware changes, but will need to adjust whatever z homing you do. If you have a limit switch currently, you'll need to move it by the thickness of the glass. If you have an inductive probe, it should still continue to work, but your z-offset will need to be adjusted. If you have BLTouch or a piezo, nothing should need to change there.

\n" }, { "Id": "8699", "CreationDate": "2019-04-15T20:11:55.320", "Body": "

I had problems printing parts and I figured out that my children have turned the knob of my Prusa i3 MK3 and set the printing speed to 112%.

\n\n

Since I'm not experimenting much yet, I'm not printing at different speeds than the original speed. I wonder whether there's a G-Code command that I could let Slic3r insert at the beginning of each print to ensure the print starts with 100% speed.

\n", "Title": "G-Code for resetting to 100% speed", "Tags": "|g-code|slic3r|", "Answer": "

You can put the speed to 100 % by G-code command: M220 S100.

\n\n

The M220command is described here.

\n\n

Know that speed changes sent to the printer have an effect on the next printed layer, it first finishes the current layer at the speed commanded before starting printing the layer.

\n" }, { "Id": "8706", "CreationDate": "2019-04-16T19:40:00.770", "Body": "

I was asked to print something with ESD PLA and the first layer, on a raft, is coming out like this:

\n\n

\"Badly

\n\n

Nozzle temperature was 220 °C and I'm using the default Cura draft profile.

\n\n

Can you provide any tips on how to get a good print with this?

\n", "Title": "Ender 3 first layer of ESD PLA printing on raft fails", "Tags": "|print-quality|pla|rafts|", "Answer": "

The gap between the raft and the first layer is too large (you need to switch to expert mode in Ultimaker Cura to see the values of the option before you can change them) and the hotend is too hot (normal PLA generally prints at 190 °C, ESD PLA prints at higher temperatures; you could aim for 210 °C as a start), This makes it difficult to precisely deposition the first layer on the raft as the nozzle drags the hot filament causing inaccurate first layer deposition.

\n\n

Furthermore, PLA generally doesn't require a raft to print the part. The Creality Ender 3 has a rough bed (like a BuildTak surface), you could print directly onto the bed surface.

\n\n

Source states:

\n\n
\n

Extruder Temp: Typically 210-220\u00b0C. ESD PLA is a filled product and\n has a higher melt viscosity vs. unfilled PLA. Therefore, it is\n sometimes necessary to print at higher temperatures than your standard\n PLA to allow the resin to flow properly.

\n
\n\n

This implies that a hotend of 220 °C mot not be hot enough, in that case the filament is too cold and viscous and gets dragged because the gap is too large.

\n" }, { "Id": "8712", "CreationDate": "2019-04-17T00:04:58.017", "Body": "

The Prusa3d knowledge base mentions that acetone will \"rejuvenate\" PEI in addition to cleaning oil and grease from the surface.

\n\n
\n

About once a week, or when prints stop sticking, use ACETONE to clean the bed. It removes grease better than IPA or Windex. It also rejuvenates the print surface. However, if you use acetone every day, PEI will become brittle and start cracking.

\n
\n\n

From a chemical or physical perspective, how does acetone affect the PEI surface?

\n", "Title": "How does acetone \"rejuvenate\" PEI?", "Tags": "|print-quality|acetone|pei|chemistry|", "Answer": "

Rejuvenate is probably a bit of an exaggerated term. The number one adhesion suppressor is grease. The stuff that comes off of the fingers used to handle the sheet. Even if you are careful and only handle the sides, the grease will be carried to the center of the plate next time its cleaned with less aggressive solvents.

\n\n

Isopropyl alchohol does break down grease but not to the extent that acetone does. Acetone also attacks plastic particles that accumulate on the print surface over time.

\n\n

PEI is resistant to a wide array of chemicals[1] including acetone but it can become brittle if exposed to it too much especially when hot so acetone is not recommended for daily cleaning.

\n\n

[1] Ultem Product Broshure table 4-3

\n" }, { "Id": "8713", "CreationDate": "2019-04-17T10:44:42.763", "Body": "

I'm using the default Ultimaker Cura draft profile but with the nozzle temp at 220 °C because this is a special ESD filament.

\n\n

All was going well after some troubleshooting thanks to some users on 3DPrinting.SE but this happened over night:

\n\n

What would cause an entire layer to lift off the main body, continue to print and then turn to spaghetti?

\n\n

The red circled part was supposed to be part of the blue outline.\n\"failed

\n", "Title": "Layer lifted and separated from ESD PLA", "Tags": "|pla|print-material|", "Answer": "

The spool kept getting caught on itself and tightening. I unrolled some of it and rewound it and the print came out perfect. @anttix was almost correct with the clog theory.

\n\n

\"enter

\n" }, { "Id": "8715", "CreationDate": "2019-04-17T19:00:44.430", "Body": "

I work in a teaching hospital and we have a research project we're interested in pursuing. We'd like to 3D Print tubes we'd implant into rats to help with nerve regeneration. We're interested in the shape of the tubes right now, more so than what material it is or whether it's biocompatible etc..

\n\n

So this question isn't necessarily about what type of plastic or whatever we should print in. My question is more so:

\n\n
\n

We'd like to print a tube that's 1 mm in diameter, about 1 cm\n long and has as many micro \"tubes\" crammed through it as possible,\n something like this:

\n \n

\"Tube

\n
\n\n

I currently have a Stratasys j750 in my lab, a UPrint Se and a Prusa i3 Mk3s. They all work well but for the detail I'm looking for, come up a bit short. They have advertised accuracies of 14 microns (well, the j750) but thats just in the z direction, x and y are more like 200. If I went to get PRECISE, what technology should I look into?

\n", "Title": "Ultra high precision and accuracy printing", "Tags": "|print-quality|resolution|", "Answer": "

Update on this:

\n\n

Per this article: 3D printing strategies for peripheral nerve regeneration

\n\n

There are a few 3d printing technologies beyond your typical FDM/SLA/Polyjet that can get this small.

\n\n\n\n

I found various articles where someone \"printed\" that small, but it was often kind of a misnomer where technically something was made additively, but it wasn't a \"printer\" that you could go buy. I think two photon polymerization may be the \"best\" actual printing method for what I want, though the price tag associated with that style of printers may be out of my range. But it can definitely get that small, this technology can apparently get down into the nanometer range.

\n" }, { "Id": "8719", "CreationDate": "2019-04-18T05:51:05.557", "Body": "

This question concerns an Anet A8 with customized Marlin Firmware 1.1.9.

\n\n

I need to generate an additional digital output to signal move complete to trigger some data acquisition process. The extruder is not connected and was planning to use those outputs.

\n\n

Which section handles G-code processing and is there an already available option to get the job done?

\n\n

Basically, I'm trying to use the Anet A8 as a computer-controlled positioning system. It involves some other elements which need to be triggered in a stationary state. The requirement is: G0 X10 Y10 after this is complete: set a pin high and low after a 10 ms delay. this after every move. Additionally if I could add my own G-code to preserve the original operation would be ideal.

\n", "Title": "Marlin customization; additional output after G0/G1 move", "Tags": "|marlin|anet-a8|", "Answer": "
\n

Which section handles G-code processing?

\n
\n\n

In Marlin Firmware, G-code is processed by procedure void process_parsed_command() in file Marlin_main.cpp

\n\n
\n

I need to generate an additional digital output

\n
\n\n

A G-code that is able to set a port value is M42.

\n\n

You could write a procedure that schedules certain port after a G0 or G1 move, recompile and upload the firmware.

\n" }, { "Id": "8721", "CreationDate": "2019-04-18T10:42:32.927", "Body": "

I've bought a new \"MKS GEN_L V1.0\" and I'm trying to configure it with Marlin 1.1.X. I changed motherboard in configuration.h from previous:

\n\n

#define BOARD_RAMPS_13_EFB 33 //RAMPS 1.3 (Power outputs:Hotend,Fan,Bed)

\n\n

to

\n\n

#define BOARD_MKS_GEN_L 53 //MKS GEN L.

\n\n

I'm getting this error:

\n\n
pins.h:268: error: #error \"**Unknown MOTHERBOARD value set in Configuration.h**\"\n       #error \"Unknown MOTHERBOARD value set in Configuration.h\"\n        ^\n    In file included from sketch\\MarlinConfig.h:42:0,\n                     from sketch\\G26_Mesh_Validation_Tool.cpp:27:\n\nSanityCheck.h:58: error: #error \"MOTHERBOARD is required. Please update your configuration.\"\n       #error \"MOTHERBOARD is required. Please update your configuration.\"\n        ^\n    SanityCheck.h:786: error: #error \"**Z_MIN_PROBE_USES_Z_MIN_ENDSTOP_PIN requires the Z_MIN_PIN to be defined.**\"\n           #error \"Z_MIN_PROBE_USES_Z_MIN_ENDSTOP_PIN requires the Z_MIN_PIN to be defined.\"\n\n        ^\n    SanityCheck.h:942: error: #error \"**LCD_BED_LEVELING requires an LCD controller.**\"\n         #error \"LCD_BED_LEVELING requires an LCD controller.\"\n          ^\n\nSanityCheck.h:1084: error: #error \"**HEATER_0_PIN not defined for this board.**\"\n       #error \"HEATER_0_PIN not defined for this board.\"\n        ^\n    exit status 1\n#error \"Unknown MOTHERBOARD value set in Configuration.h\"\n
\n\n

\"MKS_GEN_L

\n", "Title": "Configuring of MKS_GEN_L V1.0", "Tags": "|marlin|arduino-mega-2650|ramps|", "Answer": "

The motherboard definition in configuration.h should be written like:

\n\n
#define MOTHERBOARD BOARD_MKS_GEN_L\n
\n\n

actually the word MOTHERBOARD was missing and there is no need to write 53 //MKS GEN L at the end.

\n\n

The number definition is declared in boards.h - you just confused the two files, indeed you should not forget to define the motherboard constant itself. Do note that this is basically a RAMPS board, see pins_MKS_GEN_L.h.

\n" }, { "Id": "8729", "CreationDate": "2019-04-18T21:02:41.060", "Body": "

I got a Tronxy X5S-400, I assembled it and when I power it on, I see on the screen one of the below images and gets stuck there.

\n\n

Is the firmware broken? If yes, where can I get a configuration.h file?

\n\n

I verified that the two buses between the screen and the mainboard are not loose or incorrectly connected.

\n\n

\"Marlin

\n\n

\"Screen

\n\n

The board is a MKS Melzi v2.0 clone

\n\n

\"Controller\"

\n", "Title": "Tronxy X5S-400 Marlin stuck at splash-screen", "Tags": "|marlin|firmware|", "Answer": "

It looked like the firmware wasn't flashed correctly in the factory. What I did was to flash a bootloader using an Arduino compatible board and to flash the Marlin software e.g. according to videos from this guy.

\n\n

Next, the menu wasn't displayed correctly so I added, in the Configuration.h the following lines:

\n\n
#define ST7920_DELAY_1 DELAY_NS(63)\n#define ST7920_DELAY_2 DELAY_NS(100)\n#define ST7920_DELAY_3 DELAY_NS(125)\n
\n\n

Now the display is working fine, and the Marlin firmware can be directly flashed from the computer because the bootloader remains there.

\n\n

I also had a problem with the heating bed which was heating very slow and I had to raise consistently the TEMP_BED_RESIDENCY_TIME and decrease TEMP_BED_HYSTERESIS to 1.

\n" }, { "Id": "8735", "CreationDate": "2019-04-19T18:01:26.497", "Body": "

Last week, my X-axis stepper motor died. It was a 42SHD0217-24B model. I ordered the same online, but the vendor made a \"mistake\" and sent me the 17HS3401S model.

\n\n

I know these motors use a different voltage, so I adjusted the screw on the stepper driver and managed to get a smooth rotation with the new model. I know that these 2 motors have a rotation of 1.8\u00b0 per step.

\n\n

My questions are the following:

\n\n\n", "Title": "Replace X axis motor with different model", "Tags": "|stepper|motor|", "Answer": "
\n

Since they have the same rotation angle per step, I shouldn't have to calibrate the new model, right? (I'll use the original wheel)

\n
\n\n

Yes, you do not have to change anything, other than the Vref of the stepper driver.

\n\n
\n

Did the vendor send me a motor with inferior performances?

\n
\n\n

Yes, the one you received has a lower height, so smaller coils and smaller permanent magnets; basically lower torque.

\n\n
\n

Will my performances degrade if I use the new motor?

\n
\n\n

That depends on the loading of the carriage of the X-axis, it could if it is heavy; you now have less torque available to move the carriage. But, this type is frequently found in 3D printer kits and should work.

\n" }, { "Id": "8741", "CreationDate": "2019-04-20T14:52:14.167", "Body": "

I acquired an Anycubic Chiron yesterday. I went through the leveling procedure and I think the level test print came out okay so I printed a 20 mm calibration cube and a benchy. Both of these came out with a sort of spongy consistency.

\n\n

I have no idea what could be causing this so some advice would be appreciated.

\n\n

I'm using Ultimaker Cura 4.0.0 and printing in PLA.

\n\n

\"Spongy

\n", "Title": "Test print coming out spongy", "Tags": "|ultimaker-cura|troubleshooting|anycubic-chiron|", "Answer": "

It turned out I had the wrong filament size set in Ultimaker Cura. Fixing this resolved the issue.

\n\n

\"under

\n" }, { "Id": "9771", "CreationDate": "2019-04-24T19:20:24.317", "Body": "

I'm assembling a 3D printer with the RAMPS 1.4 shield (board) and an Arduino Mega. I have assembled the structure and the electronics (set drivers, placed the jumpers, connected stepper motors, etc.) and have uploaded Marlin firmware (configuring: thermistor, etc.) on to the Arduino Mega.

\n\n

At first I tested my printer without end stops and at that time it worked perfectly.

\n\n

Today I added three end stops and tested again. First it worked fine but after couple of minutes Pronterface gave this error:

\n\n
> Connecting...\nTraceback (most recent call last):\n  File \"printrun\\pronterface.pyc\", line 1053, in connect\n  File \"printrun\\pronsole.pyc\", line 720, in connect_to_printer\n  File \"printrun\\printcore.pyc\", line 46, in inner\n  File \"printrun\\printcore.pyc\", line 197, in connect\n  File \"serial\\serialwin32.pyc\", line 31, in __init__\n  File \"serial\\serialutil.pyc\", line 261, in __init__\n  File \"serial\\serialwin32.pyc\", line 71, in open\n  File \"serial\\serialwin32.pyc\", line 186, in _reconfigurePort\n**ValueError: Cannot configure port, some setting was wrong. Original message: [Error 31] A device attached to the system is not functioning.**\n
\n\n

I tried removing end stops, re-wiring, removing all cables from the RAMPS shield except power cables. Still it gives that error. Although Pronterface connects to the Arduino board when the RAMPS shield isn't powered up. Also the Arduino's regulator is heating up.

\n\n

Do I need to buy a new RAMPS shield?

\n", "Title": "Pronterface not connecting ERROR: A device attached to the system is not functioning", "Tags": "|ramps-1.4|pronterface|", "Answer": "

Finally, I found the solution after frying up a Mega board. The problem is with the Mega board. Part of the board is not functioning properly or not connecting with the RAMPS 1.4 shield properly. So I tried with a new Arduino Mega board and it worked. \nAlso removing the D1 diode is the solution for the voltage regulator overheating on Arduino mega board as mentioned in question \"Arduino Mega voltage regulator overheats with RAMPS board\".

\n" }, { "Id": "9779", "CreationDate": "2019-04-25T16:27:29.330", "Body": "

Can Nylon 6,10 be used for 3D FDM printing? What range of temperatures be used to print with it? How does it compare to the standard 3D printing Nylon material (I think it is 6,6).

\n", "Title": "Printing with Nylon 6,10", "Tags": "|print-material|nylon|", "Answer": "

Based on the information from a Quorra question about what the difference between Nylon 6,10 and Nylon 6,6 is and a ULprospector article, we can establish:

\n\n\n\n

While I see no problem with the technical ability to make a Nylon 6,10 or Nylon 6,12 filament and print with it (the lower hygroscopic of the larger molecules might make that even easier), you will make a compromise in other areas of the material, most liekly cost and availability - to my knowledge no filament that claims to be Nylon 6,10 or Nylon 6,12 is on the market at the time of this writing (April 2019), and as such there are no known benchmarks for print settings needed are available. I would expect the print temperature to be slightly lower than that of Nylon 6,6 though.

\n" }, { "Id": "9780", "CreationDate": "2019-04-25T16:38:27.463", "Body": "

Following advice I read elsewhere, I have covered my (heated) print bed with blue painter's tape, and before each print I apply an Elmer's purple washable glue stick to improve bed adhesion.

\n\n

After the print finishes and I remove the part from the bed, the bottom is covered with a white residue. I'm fairly certain this is the dried glue. It turns purple again when I wet it.

\n\n

Is there an easy way to clean off this residue, or is there a better technique?

\n\n

\"example

\n", "Title": "Removing glue residue on part after printing", "Tags": "|print-quality|adhesion|", "Answer": "

I have two different makes of masking tape. The print stick so well to one of them that I end up ripping the tape up to get it off and the bottom of the print has the tape suck to it. The other roll was the complete opposite, nothing would stick to it. I ended up using the non-stick tape and applied a diluted (30% PVA - 70% water) glue to the (tape covered) hot bed. Works wonderfully.

\n\n

Mike.

\n" }, { "Id": "9783", "CreationDate": "2019-04-25T20:32:56.457", "Body": "

I have acquired a second hand 3D printer, the GEEETECH I3 Pro B. The previous owner tried to change the firmware but he made a mistake and he bricked the board.

\n\n

I want to substitute the firmware of the board (which is an Arduino Mega 2560 based board). Do you think is possible if the board doesn't respond to the controls?

\n\n

I have inquired and found an open source software called Marlin I want to use to restore the printer.

\n\n

If you have an advice or you think that I can do it in any ways tell me.

\n", "Title": "Can I replace a damaged firmware of a 3D printer based on Arduino Mega?", "Tags": "|diy-3d-printer|firmware|", "Answer": "

It is hard to diagnose the board without hands on experience. It is even more difficult if you get a board that has been \"updated/upgraded\" by a previous owner leaving it not in working condition.

\n\n

But, there are at least 2 solutions.

\n\n
    \n
  1. You could buy a new printer controller board, upload new firmware and connect all connectors.

  2. \n
  3. You could try to burn a new bootloader onto your current board and upload a new version of the firmware.

  4. \n
\n" }, { "Id": "9792", "CreationDate": "2019-04-26T15:16:45.453", "Body": "

I'm still new to 3D printing and I want to print something. I expect that I'll mess it up since I find nothing to adjust it but it is now laying around for 4 months and I'm sick of it.

\n\n

So my question is where do I find Windows software to print something and of course where do I get a 3D model?

\n\n

I own a Geeetech i3 Pro W.

\n", "Title": "Which software do I need to start print something?", "Tags": "|3d-models|slicing|fdm|knowledgebase|", "Answer": "

If you're just starting out then Tinkercad (website) is a good place to start designing your own objects. Later you can get to grips with OpenScad for more complex shapes. Both are free.

\n" }, { "Id": "9794", "CreationDate": "2019-04-26T17:56:10.070", "Body": "

Z-fighting is a 3D rendering artifact of coplanar surfaces (means, of triangles located in exactly the same plane and overlapping each other).

\n

It can happen in OpenSCAD's preview mode when doing a difference() or union() operation. In case of difference(), the rendering artifacts can prevent seeing into a hole in the object. The "compile and render" mode in OpenSCAD does not have z-fighting issues. But since it can take some time to render an object in that mode, it is not a practical solution during development.

\n

How best to avoid z-fighting?

\n

If the answer involves changes to my OpenSCAD code, I would love to see an idiomatic answer / established convention of OpenSCAD coders, if that exists.

\n", "Title": "How to prevent z-fighting in OpenSCAD?", "Tags": "|openscad|", "Answer": "

The general advice in the OpenSCAD community is to \"extend your cuts and embed your joins\" (source). The rendering artifacts are one thing but rather just an annoyance; however z-fighting can also cause unexpected errors during STL export (I did not experience that myself so far, just read this somewhere).

\n\n

So you would change the dimensions of your objects very slightly (0.01 mm works fine) so that:

\n\n\n\n

Now you could adjust both the size and position of your parts to keep the mathematically exact dimensions for the resulting part. But I found that for the purposes of 3D printing, such accuracy is not worth it because it complicates the formulas so much.

\n\n

Instead, I adjust either position or size of a part, depending on what is simpler in each case. A measure in the final design will be off by 0.01 mm, which does not matter.

\n\n

And I keep the 0.01 mm offset in a variable called nothing (picked that up somewhere and liked it \u2026). This keeps the calculations intuitively understandable.

\n\n

Example

\n\n

To create a cylinder and cut a hole to half of its depth, I would do this:

\n\n
//!OpenSCAD\nnothing=0.01;\nheight=40;\n\ndifference(){\n    cylinder(h=height, r=20, center=true);\n\n    translate([0, 0, height/4 + nothing]) \n        cylinder(h=height/2, r=15, center=true);\n}\n
\n\n

Now the hole is nothing=0.01 less deep than half of the cylinder \u2013 that's the inaccuracy I accept.

\n\n

(Note: If you don't have OpenSCAD installed, you can also try the above code online by copy & pasting it into OpenJSCAD. Include the magic comment in the first line to switch it to OpenSCAD syntax.)

\n" }, { "Id": "9804", "CreationDate": "2019-04-27T20:12:39.200", "Body": "

I own a Geeetech i3 Pro W and I started printing today. Amazing how it works.

\n\n

I just recognized while printing my first bigger model something very strange: At the beginning some parts are missing and there are huge holes. However when I keep it printing the following layers \"fix\" the issue. I'm wondering what I'm doing wrong.

\n\n

I almost forgot to mention that I think I'm using that PLA, the bed has a temperature of 60 \u00b0C and the extruder 200 \u00b0C.

\n\n

Here are two pictures after about 1 layer and after about 4 layers:

\n\n

\"first

\n\n

\"fourth

\n", "Title": "The first layers while printing look strange", "Tags": "|prusa-i3|quality|geeetech|", "Answer": "

From the first layer image it can be seen that your nozzle to bed distance is just too large:

\n\n
    \n
  1. the lines of deposited filament e.g. in the brim are not touching,
  2. \n
  3. deposited filament lines are not \"squished\" or \"flattened\" to the build plate,
  4. \n
  5. filament is cutting off corners as it is dragged while being hot and not stuck to the bed,
  6. \n
  7. filament leaves the nozzle in \"blobs\"; it sort of free flows from the nozzle as the bed is too far to give resistance.
  8. \n
\n\n

Use the plain paper technique while levelling the bed. The paper should give a slight drag/resistance when pulled/pushed. Proper levelling and a proper nozzle to bed distance for the first layer are essential for successful prints.

\n\n

There is also an option to integrally lower the print level in the slicer software, but it is advised to properly level the bed (hardware solution) instead of using software tricks to sort out the problem.

\n" }, { "Id": "9807", "CreationDate": "2019-04-29T00:55:38.327", "Body": "

The original bed surface of my Ender 3 has become brittle and finally cracked, requiring replacement. I'm trying to figure out what the cause might have been to avoid it happening again. It seems to have started after using \"flex PLA\", which involves both high temperatures (225 °C) and plasticizers mixed in the PLA. Could either of these have contributed to the problem? I'm not sure what material the bed surface is - it's the new one that's removable and held on by clips. If it's PEI, the glass transition temperature is supposedly 217 °C, just above what I use for normal PLA but well below what I'm using for the flex, so perhaps that's the cause?

\n\n

Image of the damage: \"Image

\n", "Title": "Damage to bed surface from heat or chemical or..?", "Tags": "|creality-ender-3|material|bed|", "Answer": "

The build surface on the Ender3 is a BuildTak clone. The picture is a bit unclear, but given my experience with BuildTak (clones) this certainly damage because of heat. You can, as suggested before, replace the bed surface, but I do not think it is necessary at this stage.

\n\n

Normally these surfaces do not get damaged that easily but to prolong the life try to keep the following points in mind:

\n\n\n" }, { "Id": "9820", "CreationDate": "2019-04-30T02:39:39.480", "Body": "

I need to set a Z offset for the Flying bear P902. I calculated the offset (-2.98). But, every time when I try to input this using the LCD screen of my 3D printer, the value jumps to either -2.99 or -2.97. As -2.97 is just a little bit too far from the bed and -2.99 is just a little bit too close to the bed (and -2.98 is perfect), I really want to input this specific value. I have tried many times on the LCD screen and also in the firmware itself.

\n\n

But, even after uploading the firmware, it still displays -2.97.

\n\n

This is the line of code I was changing:

\n\n
#define Z_PROBE_OFFSET_FROM_EXTRUDER -2.98 // Z offset: -below +above  [the nozzle]\n
\n\n

Is there a way I can input -2.98?

\n", "Title": "Specifying Z offset in Marlin firmware", "Tags": "|marlin|calibration|firmware|z-probe|", "Answer": "

Z-offset persitently stored in memory?

\n

Maybe the value of -2.97 for the Z_PROBE_OFFSET_FROM_EXTRUDER is retained in the EEPROM memory when you upload new firmware.

\n

You could try to send the G-code M502 to the machine to re-load the values from the firmware overwriting currently stored values.

\n

Alternative Z-offset using G-code commands

\n

Note that there is a different (and more common) solution to set the Z-offset using G-code M851, you can do this after you uploaded the firmware to the printer. Sending G-code commands can be done using printer software and a USB connection to the printer through a so-called terminal interface. This can be done in OctoPrint, Repetier-Host, Pronterface (Printrun software suite), and probably many more. Alternatively, you could make seperate G-code (basically text files with extension .g) files with each step in a single file and "print" the files through the SD interface of the printer menu.

\n

The following strategy must be followed to specify the Z offset:

\n\n" }, { "Id": "9821", "CreationDate": "2019-04-30T02:57:45.477", "Body": "

I'm not sure how else to describe it. There's probably a name for this but I just don't know it. But the bottom few layers came out great, but the rest came out kind of like a triscuit. Below are pics of my print and settings. I am using a delta style printer. Can someone tell me what this issue is called and how to fix it?

\n\n

\"enter

\n\n

\"enter

\n\n

\"enter

\n\n

\"enter

\n\n

\"enter

\n\n

\"enter

\n", "Title": "Print came out like a wafer", "Tags": "|print-quality|ultimaker-cura|extrusion|", "Answer": "

Just to add to the already made answers:

\n\n

Check out heat-creep!

\n\n

When heat from the hot-end creeps up in the filament, it melts and blocks the extrusion (more or less severely), and under extrusion results.

\n\n

It's basic characteristic is that the print starts out great, and then some time in, under extrusion happens.

\n\n

The solution is to cool down the cold part of the hot-end which is usually done with a small fan directed onto the cold part, e.g the top part where the filament enters the hot-end.

\n\n

Most hot-ends have a heat sink on the cold part, but in my experience this is not enough and for me a fan has always been needed.

\n" }, { "Id": "9833", "CreationDate": "2019-05-01T17:56:17.063", "Body": "

Trying to print a 3D model for my mobile phone, but I see that when printing the sides, being thin, increases the retraction and the recoil seems a little abrupt and makes a coarse sound.

\n\n

I would like to know if it is possible to know what speed and temperature is recommended to print a model.

\n\n

In my case I use Simplify3D, and when I'm going to save the file in .gcode format, I see that there are some ranges shown in colors, how does this apply to the models?

\n\n

\"Screenshot

\n", "Title": "Is it possible to know which is the correct temperature range and speed for any model?", "Tags": "|3d-models|simplify3d|speed|", "Answer": "

So as someone else on here mention, those settings shouldn't be for the model but for the filament. Sadly, you will need to test 99% of filaments to really figure this out. I have a modify tester, and on the description it tells you how to set your temp. https://www.thingiverse.com/thing:3347967

\n\n

You can look at the remix if you want to grab the blank model and put your own numbers on it.\nIt should be noted that things like water in the filament can mess with how the filament reacts to speed and temp. If you have questionable prints coming out of a filament that sat there for a long time. You can easily run it through the test to figure out the temp.

\n\n

Anything else I could add is would just repeat what most of 0scar said.

\n" }, { "Id": "9838", "CreationDate": "2019-05-03T16:20:34.980", "Body": "

I have a generic Prusa clone that I'm not using (my primary issue with it is the lack of rigidity in the frame housing as built). Will the motors (ks42sth40-1204a) and controller that come with it be good for a large-ish Kossel? Something no smaller than 200x200, preferably closer to 300x300? How do I reason about this?

\n\n

Also - I know the Duet board seems to be recommended for Deltas, but can I use the RAMPS board that I already have here?

\n\n

I don't have a design finalized, but have been looking at this Kossel, Building a Large Kossel Delta Printer \u2013 pt. 1: Parts and Planning, which uses a Flex3Drive effector.

\n", "Title": "Can I repurpose most of the components of this Prusa clone for a Delta?", "Tags": "|prusa-i3|diy-3d-printer|printer-building|delta|", "Answer": "

Stepper motors

\n

For equivalence, 4.8\u00a0kg\u22c5cm is 0.471\u00a0N\u22c5m or 47\u00a0N\u22c5cm.

\n

Looking at the RepRapWiki - Nema17, the most common steppers are:

\n\n

The Kossel that you refer to is of a similar size to the Kossel XL. Again looking at the RepRapWiki - Kossel, the recommended stepper is the Kysan 1124090 Nema 17 Stepper Motor which has a Holding Torque of 5.5\u00a0kg\u22c5cm.

\n

So, without knowing the exact make and model of your stepper motors and assuming that the specifications for your stepper motors given in the Prusa clone are correct, your steppers are not as strong as those recommended (apart from the Wantai). However, the recommended steppers may be over-engineered and provide [much?] more torque than is strictly necessary. After all, the holding torque of your steppers isn't that much below the recommended values.

\n

If I were you I'd build the Kossel using your steppers and it might, in all likely hood, work out fine. FWIW, I have used the Rattm 17HS8401 in my Kossel Mini and Kossel XL. I got them on eBay and were quite reasonably priced.

\n

You should probably read this article, RepRapWiki - Choosing stepper for a delta. Whilst no concrete values for holding torque are given, this is interesting:

\n
\n

A good practical setup

\n

The Fisher, a small delta printer was designed by late RRP company. As for all their printers, they were using small and compact steppers with a torque of 2.2 kg.cm. This is lower than most repraps but is sufficient if there is no mechanical problem (friction).

\n

These small motors have a low nominal static torque, but they also have a low inductance (2.5 mH), while due to their small size, the nominal current remains reasonable (1.2A).

\n
\n

as is

\n
\n

High inductance motors

\n

You find on the market steppers sold for 3D printers, with a torque ranging from 2.6 to 4.4 kg.cm and a current of 0.4 A. This low current appeal builders as it make the electronic driver heating much less.

\n

However, it came at a cost, which is a very high inductance which varies from 30 to 35 mH. That means these motors are totally incapable of any speed. They are unusable for a delta and a bad choice for another printer. As an example, a 4.4 kg.cm motor wired for this low current, while having a static torque twice the Fisher motors, simply cannot reach the maximum speed used by the Fisher, effectively having a near zero torque over a given speed. Same motors with a winding giving a nominal current of 1.5 to 2 A will be more usable.

\n
\n

Controller

\n

Also, yes, RAMPS is fine for the Kossel, although the firmware calibration is obviously different, as it is a delta and not cartesian printer. For calibration, refer to How do you calibrate a delta robot 3D printer?.

\n

Although, as Scott Lahteine states early on in this video, How it's Made: The Marlin Firmware!, using an 8-bit controller for delta printers is pushing their capabilities somewhat.

\n

Extruder

\n

I'm not familiar with the Flex3Drive, but it certainly looks interesting. I have used the 3325_0, this NEMA-17 motor has an integrated Planetary gearbox with a 52/11:1 ratio. It generates 16.2\u00a0kg\u22c5cm of torque at 1.68\u00a0A. I wrote a short blog about it here, The extraordinary extruder.

\n

Building Tips

\n

Also, if you are planning on building a Delta/Kossel, then I'd seriously recommend watching the series of videos on YouTube from BuildA3DPrinter.eu as they are extremely informative and helped me a lot, Build manual Kossel XL & Kossel Mini. I just checked their website to try to see which make and model of steppers they use, in order to get an idea of the holding torque, but they seem to have stopped trading. However, their stepper motor page states the following:

\n
\n

The standard motor for most 3D printers including ours, the 42byghw811\nfrom Wantai Motors.

\n

Holding torque is 4.8 kg\u22c5cm or 47.1 N\u22c5cm. Shaft diameter is 5 mm and\nstepping angle is 1.8 degrees.

\n
\n

So, to sum up, your steppers should be fine.

\n" }, { "Id": "9839", "CreationDate": "2019-05-03T17:11:34.603", "Body": "

I've got a printer on the way later this year that's supposed to be able to do both FDM and SLA (with the appropriate accessories). I'm interested in printing some parts for swimming pools (the types that generally cost fifty cents to make, but with markup and shipping end up costing thirty or more dollars).

\n\n

I need to find a material that can withstand exposure to chlorine (up to 15ppm for the rare nuclear shock) and UV from the sun, hopefully for several years.

\n\n

I think that I'd prefer an FDM filament initially in order to test, but understand that FDM processes may require more post-processing in order to keep water from penetrating layers.

\n\n

What are some other considerations that I should be aware of? Having a white color would be preferable, but if it's available in other colors and would be suitable, it might be fun to make some different colored eyeball fittings.

\n", "Title": "What materials would be appropriate for pool parts (chlorine & UV exposure) using SLA or FDM?", "Tags": "|sla|filament-choice|", "Answer": "

Have you thought about using ASA filament? ASA filament is very strong. ASA filament is similar to ABS filament (if you have ever worked with that). When contrasting it to ABS filament, ASA has a higher resistance against UV and chemical exposure. It will also have no problem with the water. Both ASA and ABS filament print at about the same temperatures.

\n\n

Here are some links that might be helpful :

\n\n\n\n

Hope this is helpful.

\n" }, { "Id": "9842", "CreationDate": "2019-05-03T20:25:15.183", "Body": "

I'm wondering if there is some trick to power my OctoPi with the power supply of my 3D printer. I'm using an Geeetech I3 Pro W.

\n\n

The power supply itself should be able, but the output is as far as I'm aware of 3.3 volts. Not my desired 5 V for USB, it would be a shame if I really would need to buy a new power supply when I have a strong one actually running. My current power supply causes a lot of \"Under-voltage detected!\" warnings.

\n\n

After thinking a little about the specs, there are cigarette lighter adapter for cars they use 12 V. Has anyone experience with using that on his printer?

\n", "Title": "Power OctoPi from printer", "Tags": "|prusa-i3|octoprint|power-supply|geeetech|", "Answer": "

I\u2019ve been using a buck converter for quite awhile with no problems. However just as importantly as proper voltage and amperage is using a good cable. I had my step down properly set, but was still getting low voltage warnings until I switched to a thicker cable.

\n" }, { "Id": "9862", "CreationDate": "2019-05-06T00:59:38.303", "Body": "

I printed a case for my phone, a Motorola G4 Plus. I found the model of the casing on Thingiverse

\n\n

I just downloaded the model, used Meshmixer to check for issues, after that, opened Simplify3d and saved it for printing using an SD Card. The printed size of the model was smaller than expected.

\n\n

The model designer, says in the description that he used flexible filament. Is it possible than if I change to that filament, the model result is completely different?

\n\n

Print result - phone casing:

\n\n

\"1\"\n\"2\"\n\"3\"

\n", "Title": "Can a model change size when using a different filament type?", "Tags": "|3d-models|3d-design|simplify3d|", "Answer": "

Of course it can change size... but definitely not in scale you've presented.

\n\n

Filament as same as many other materials can subject something called shrinkage. It depends on physical properties of the material and as its name suggests cause the dimensions to shrink.

\n\n

Here is wikipedia article.

\n\n

The same happens to almost all plastics.\nBut this phenomenon is almost unnoticeable in terms of PLA... well not really...

\n\n

Bending is mostly caused by shrinkage. We have heated beds (among others) to prevent it. And we use adhesives also to prevent bending so to prevent effects of shrinkage.

\n\n

It can be also noticed when you print specific volume (and/or specific base surface) and you left the model on the HB until it cools down - the model can come off the HB on it's own even when adhesive was used.

\n\n

But your \"shrinkage\" is definitely not that one.

\n" }, { "Id": "9865", "CreationDate": "2019-05-06T11:11:55.830", "Body": "

I try to print a stamp with flexible filaments. The problem I encounter is that the filament is flexible, but not soft. This leads to small differences in height to parts of the stamp not working.

\n\n

One solution would be to add a small 'cushion' to add some squishyness to the stamp. I designed the stamp and the 'cushion' but now the question arises: \n\"Which infill will provide the best uniform squishyness (in one axis)?\"

\n\n

I did a test with cubic infill of Cura, and although it becomes quite squishy, some parts are squishier than other parts of the block.

\n\n

TLDR; Trying to print a squishy cube, where in one axis all areas of the cube have the same squishyness.

\n", "Title": "Uniform squishy infill", "Tags": "|slicing|infill|flexible|", "Answer": "

Sinusoidal infill provides great squishiness in the infill direction, but you'll still have the problem that where the infill meets the perimeter wall, it'll be less squishy than where the perimeter wall isn't touching any infill. You can reduce this effect by using a stiffer filament for that wall (if you can print with multiple filaments), by adding more perimeters, and by reducing the gap between infills (i.e. increasing the infill density). You can even explicitly design an extra-thick wall on the face that takes the pressure (the engraved face of the stamp), and then a section behind that with the squishy infill.

\n\n

TBH, I'm not sure that flexible filament is really what you need for a stamp. Soft materials are commonly used for traditional stamp-making more because they're easy to etch than because the stamping works better that way. Print-making uses wooden or metal plates (the equivalent to the stamp) and produces better, more repeatable images than rubber stamps. When you're printing a stamp, you don't need to etch it, so the softness of rubber isn't an advantage for you. My outsider recommendation would be to try using a normal, rigid filament, and sand the surface to the smoothness you need. If you print with the stamp face on the bed, and your first-layer quality is really good on a smooth build plate, you can probably get better results without sanding.

\n" }, { "Id": "9870", "CreationDate": "2019-05-07T10:34:34.253", "Body": "

Sometimes I use a DIY 3D printer running Marlin firmware and I have a hard time to set my heat bed temperature. when I set it to 70 °C for PLA, after a few minutes it decreases to 67 °C and I see these error:

\n\n
READ: Error:Thermal Runaway, system stopped! Heater_ID: bed\nREAD: Error:Printer halted. kill() called!\n
\n\n

Since then the communication with printer is lost, the printing process stops and I have to reconnect to serial port... It's a disaster. I guess I need to lower the sensitivity to 3 degrees at least. I don't know how!?

\n", "Title": "How to decrease sensitivity to heat-bed temperature?", "Tags": "|marlin|diy-3d-printer|thermistor|", "Answer": "

For people from google:\nGo to configuration_adv.h in the marlin firmware source code, and search for "THERMAL_PROTECTION_HYSTERESIS". Then increase the number. The number is how much degress celsius the temperature can be off before thermal runaway

\n" }, { "Id": "9878", "CreationDate": "2019-05-08T14:48:36.270", "Body": "

I have seen that many 3D printers have only one limit switch for each axis, how does it know where to stop on the other end?

\n\n

My first guess is that the machine knows how big the plate is, and calculates it accordingly.

\n\n

If this is true, then if I were to use a RAMPS, I would have to modify the software to figure out the build plate, it won't have the hardware to autocalculate.

\n", "Title": "Why do 3D printers have only one limit switch?", "Tags": "|diy-3d-printer|printer-building|desktop-printer|", "Answer": "

In principle you only need the minimum axis position (or the maximum), the offset to the bed and the size of the bed in the direction of the axes. Fortunately, you can specify this in the firmware:

\n\n

E.g. in Marlin Firmware offsets are defined as travel limits:

\n\n
\n// Travel limits (mm) after homing, corresponding to endstop positions.\n#define X_MIN_POS -33\n#define Y_MIN_POS -10\n#define Z_MIN_POS 0\n#define Z_MAX_POS 240\n
\n\n

Bed size:

\n\n
\n// The size of the print bed\n#define X_BED_SIZE 200\n#define Y_BED_SIZE 200\n
\n\n

Do note that some printers do have maximum endstops on top of minimum endstops. This is handy in case of layer shifting (e.g. caused by the nozzle catching the print as such that the belt skips notches and as such redefining the reference frame) to prevent the carriage from destroying the printer at the maximum of the axis.

\n" }, { "Id": "9883", "CreationDate": "2019-05-08T17:10:45.653", "Body": "

I don't want this to be a specific producer question, but I would like to know if the Sparkmaker is good enough to print small details in OO/HO scale objects.\nI'm referring here to objects like furniture, and other house appliances at scale.\nI wasn't able to find any visuals with very small objects for this printer.

\n", "Title": "Is the Sparkmaker good enough to print OO/HO small detail objects?", "Tags": "|print-quality|resin|sparkmaker|", "Answer": "

I own a Sparkmaker FHD whose X/Y resolution is 57\u00b5m. Z resolution (layer height) is up to the user, 25, 50 or 100\u00b5m being typical values. I have limited experience with it but the level of detail seems to be coherent with the specification.

\n\n

The more popular (but less powerful in therm of UV light, so slower) Anycubic printers have a 2k screen and reach a X/Y resolution of 47\u00b5m. I think some printers with a 4k screen can reach down to 37\u00b5m X/Y resolution, even if the practical resolution is probably larger, due to some horizontal light diffusion in the screen upper layers and in the FEP film.

\n" }, { "Id": "9899", "CreationDate": "2019-05-11T13:42:51.217", "Body": "

Does anyone know how the developers of Marlin decided to name it that?

\n", "Title": "Where did Marlin get its name?", "Tags": "|marlin|terminology|history|", "Answer": "

As far as I know Erik van der Zalm started Marlin. He is from the Netherlands and Zalm translates to \"salmon\". One of the design goals of Marlin was to make it faster than the other firmware available at that time. And a marlin is a very fast swimming fish.

\n\n

Some firmwares developed after Marlin also joined this fish theme: Sailfish, Minnow, ...

\n" }, { "Id": "9904", "CreationDate": "2019-05-12T14:27:38.080", "Body": "

Model: Infitary M508

\n\n

Details: The filament is stuck in the extruder preheated for PLA (the filament is PLA 1.75 white). The extruder's motor works and the filament is in the hole of the extruder (not somewhere else). I took the fan covering the motor apart, to show what is inside, so you might see it on the attached image:

\n\n

\"enter

\n\n

You might also see this video for details: https://www.youtube.com/watch?v=R8rYGhuYWvc

\n\n

I'm able to pull the filament out, when I uncouple the motor's gear, but it doesn't go through the extruder when I push it in.

\n\n

What can I do to fix this? Thanks!

\n\n

P.S. It's the first start of the printer.

\n", "Title": "Stuck filament in the extruder of Infitary M508", "Tags": "|filament|extruder|motor|alunar-m508|", "Answer": "

The hot end can clog in two places.

\n\n
    \n
  1. Heat distortion can cause the filament above the nozzle, at the level of the cooling fins, to melt, expand and prevent further passage.
  2. \n
  3. The nozzle itself may be clogged by impurities.
  4. \n
\n\n

There are two methods available, depending on the type of clogging.

\n\n
    \n
  1. If the hot end is blocked at the level of the cooling fins, a so-called \"Atomic Pull\" helps. This is also a convenient method of changing filaments. The hot end is heated to approx. 90\u00b0C and the filament is pulled out of the hot end with a bold jerk. The filament does not melt completely and remains tough. Thus the complete plastic is pulled out of the hot end.

  2. \n
  3. If the nozzle is clogged, an \"Atomic Pull\" only helps to a limited extent.\nTo clean a nozzle, the hot end should be removed from the holder and the heating block unscrewed. You need a gas burner. With the gas burner the brass nozzle is heated until the plastic is completely carbonized. Wait until it has cooled down and reassemble the hotend.

  4. \n
\n\n

Post-cleaning advice:\nBe sure that the hotend is cooled enough that no heat can creep up the hotend and melt the plastic before it enters the heat block. Without sufficient cooling clogging pre-nozzle is very likely.

\n" }, { "Id": "9912", "CreationDate": "2019-05-13T13:57:16.847", "Body": "

There is pretty much an ecosystem of two Nozzle designs out there that share the M6 thread on the coupler to the Heating block:

\n\n\n\n

What differentiates the two and can one swap one for the other?

\n", "Title": "What is the functional difference between an e3D-Style nozzle, Makerbot MK8 and M6 Chinesium Nozzles?", "Tags": "|nozzle|e3d|knowledgebase|", "Answer": "

Differentiation

\n

The main differences between the e3D-Nozzle family and the "simple" Nozzle are the wrench size, body length and thread length of the nozzle. In fact, I have come across 2 different "Chinese" styles of nozzle, a "big" and a "small" one.

\n

Comparison

\n

For comparison, take a look at this photo, where I aligned the lower ends of the bodies to line up under the wrench needed to handle them.

\n

\"Chinesium

\n

In words, left to right:

\n\n

To differentiate from Makerbot nozzles:

\n\n

Replaceability

\n

Chinese big to Chinese small nozzles in either hotend

\n

Due to the dimensions, one can easily swap the big and small Chinese nozzles for one another. They are virtually interchangeable but differ in the wear patterns.

\n

Chinese nozzles in e3D Hotend

\n

The 3\u00a0mm shorter snout and deeper butting with the heatbreak of the e3D nozzle in its designed hotend make it hard to swap a Chinese nozzle into an e3D setup: neither does the thread allow to screw the nozzle in the right length sometimes, it also extends much further. To accommodate, the whole heater block has to be screwed about 3\u00a0mm more onto the heatbreak, then the nozzle gets screwed in. The result is equal in overall length.

\n

Makerbot Mk7/Mk8 to Chinese Nozzles

\n

But for the different wrench, those generally will fit, but

\n

###e3D Nozzle in Chinese Hotend\nHowever, the long thread of the e3D Nozzle allows it to be mounted in a hotend designed to hold a Chinese nozzle without trouble - the 3\u00a0mm of difference in the body are used for a longer thread and clearance between thread and body, resulting in the same overall length without changing the mounting position of the heater block on the heatbreak.

\n

Internals

\n

Stefan of CNC-Kitchen recently tortured a couple of nozzles for science and investigating wear and tear (video). He found out a couple of differences on the internals:

\n\n

Conclusion

\n

The Chinese Style nozzles can be interchanged for one another. an e3D style nozzle with standard length (aka not-volcano) can be swapped in for any Chinese Style nozzle. A Chinese Style nozzle needs to have the heater block shifted if mounted into an e3D hotend.

\n" }, { "Id": "9914", "CreationDate": "2019-05-13T14:44:58.110", "Body": "

I am not very used to 3D printers and I just bought my first 3D printer today (Creality3D Ender 3). But I am unable to assemble it. The 3D printer doesn't contain instruction on how to built it which I found very weird. I searched a couple of things on the Internet and I found this website, but after some steps I couldn't read it , I didn't understand what I was reading.

\n", "Title": "3d printer set up", "Tags": "|printer-building|creality-ender-3|", "Answer": "

The website you looked at is for an entirely different printer and in general.

\n\n

Your instructions are in a PDF file on the mini-SD card that came with the printer. The official Ender 3 support site is https://creality3d.cn/download/produktdatei_c0002 and it also contains the Official Assembly Instructions (to me, the lower right corner). An alternate setup instruction is in a PDF distributed by Sain Smart.

\n" }, { "Id": "9918", "CreationDate": "2019-05-13T16:31:52.817", "Body": "

The ArchDaily article AI SpaceFactory Wins NASA's 3D-Printed Mars Habitat Challenge shows a working 3D printing apparatus using an unusual material containing

\n\n
\n

basalt fiber extracted from Marian rock (simulant) and renewable plant-based bioplastic.

\n
\n\n

The photo below shows part of the printer. Is this printer just leaving a slurry to dry, or does the mixture somehow catalyze or harden spontaneously? I'm also wondering what the (looks like) twelve black hoses are around the central nozzle.

\n\n

\"AI

\n\n

\"AI

\n\n

\"AI

\n", "Title": "How does this Martian habitat 3D printer built for NASA work?", "Tags": "|print-material|part-identification|autodesk|concrete-printers|", "Answer": "

Let's start with the general design look and feel: This printer contains a robotic arm with a toolhead, pretty similar to a welding robot, and probably is controlled with a similar CAM software.

\n

\"a Picture by Robotics.org

\n

Tool head

\n

The really interesting part here is the tool head. So let's look at it and try to reverse engineer the use of some parts by how they are placed and what one can see about them, together with the information given by OP.

\n

Black pipes

\n

There's a bundle of 12 black pipes that go from the main body to the print head, ending at the side of some distance disk. To me, these look suspiciously like a system to deliver an airstream, so most likely some sort of cooling system. This is further supported by the huge fans at the base of the machine, pumping air into the flexible pipe.

\n

Silver Tank

\n

The first picture shows a silver tank with the label V7 (version 7?) or VT (as in Virginia Tech) or something similar on it. This is connected via a grey hose to the base of the printer. The mounting of it over the extruder hints, that this is a hopper, most likely holding the print material in pelletized or powder form, and that it is fed via the grey hose. From the information given in the question, it might be some sort of PLA (synthesized from cornstarch) or other bioplastic using the Martian dust as a filler material. From here, the print material falls into the central column...

\n

Central Column

\n

...which goes down through the distance disk into the thick nozzle, so it must be the extruder and heater combo. At its top, there is a large stepper motor in Z orientation, which hints that inside of the matte grey tube is an arbor, pressing down the melting pellets past a heating element into the nozzle below.

\n

Print material and further information

\n

The last picture shows proudly "Autodesk" on the side of the printer. Autodesk has an own article about printing in space from August 2018, where Nathan Golino of the NASA owned GMRO states this:

\n
\n

Abrasion has been an issue with the 3D printer we use. It\u2019s very rough on the feed screw and the barrel and nozzle as the material is extruded through the system.

\n
\n

This confirms the general makeup akin to a pellet-style extruder.

\n
\n

Combining a small amount of waste plastic with crushed rock known as regolith can form an additive construction material that\u2019s stronger than concrete. (on a picture caption)

\n

The material we\u2019ve been using in our additive-construction experiments is regolith mixed with waste polymers. You can get polymers in the form of astronaut trash and shipping containers, or you can synthesize polymers. You can use that as a binder for regolith, with a relatively low ratio of polymer to regolith, to make a construction material pretty similar to Portland cement in compression and 20 times stronger in tensile strength.

\n
\n

"Waste polymer" could be anything from ABS to PETG, from ASA over PC to PLA, but it seems that the plastic-to-regolith mix is on the high regolith side. It seems that it behaves more like a plastic than concrete, hardening/solidifying from a molten paste to its hard concrete as it cools.

\n

As an interesting extra tidbit: Golino also states, that the mars-printers are at the moment on level 2 to 3, where 0 is "general concept" and 9 "ready to fly", so in early development.

\n

Further reading into the background of the project - a design competition in 2015 - hinted, that the software for the arm might even be Autodesk PowerMill.

\n

Looking back at the question if that is V7 or VT on the printer's hopper, an article with the same dome printed in OP's question popped up: Virginia Tech had been part of the crews that were taken to the finals of the aforementioned design competition and was part of the finals. They worked with the AI Space Factory team, which came out winning. In a related article, the printer OP showed us can be seen from a different angle and stripped of the cooling pipes and with a different, longer extruder. It tells us a little more about the work distribution of the teams and participants:

\n
\n

Large format vessel printed by AI SpaceFactory in the Autodesk BUILD Space for the construction phase of the competition. The tooling was developed in collaboration with Virginia Tech and Autodesk.

\n
\n

The main company behind it, AI SpaceFactory, showed a different version - the one without cooling pipes - in motion on a YouTube video on April 10th 2019 and the performance of the air-cooled version during the competition finals on May 3rd 2019 (warning, 10 hours of 3D printing galore!) At 9:52:12 we also start to learn what that disk is in picture 1: It is the endcap of the structure which sadly fell through the hole due to navigation issues.

\n

In the following shorter and commented video, we see that the printer also contains some sort of gripper to place the window frames and skylight just as well as the load bearing test performed after the print. Apparently, the material they use is engineered to a point where it can be reused after regrinding it to dust again.

\n" }, { "Id": "9927", "CreationDate": "2019-05-13T23:18:44.857", "Body": "

I am trying to do some edits on an STL file. I am trying to use meshmixer for this. I am essentially trying to move a hole in the following picture \n\"enter\nand widen the hole on the bottom of the part. I was playing around with meshmixer and it seems that it can do this by sculpting rather than precise measures. I would appreciate comments if my observation is correct and if so, what other STL editor would you suggest to do these edits. Thanks!

\n", "Title": "precise transformation using meshmixer", "Tags": "|stl|meshmixer|", "Answer": "

It's possible to close over a hole using Meshmixer. I've done exactly that recently, although the \"hole\" was a depression, the process would be the same. As you've discovered, Meshmixer can be considered somewhat imprecise.

\n\n

Fusion 360 will import the model you wish to modify. You would then turn off edit history, convert the model to BREP, then perform the edits you require.

\n\n

Once you are satisfied with the results, it's an easy matter to export the model as an STL file.

\n\n

I've summarized the steps, which are almost as easy as my description. You'd have to combine Google-Fu with the summary for the detailed portions, but it's something I've done in the past.

\n\n

Fusion 360 is free for hobbyists, renewable each year. There are many YouTube tutorials and text-referenced solutions for the steps required to accomplish your edit.

\n\n

Some will consider Blender to be an alternative method and I agree that it's a viable option, but it's not quite as intuitive as Fusion 360 and was more challenging for me to embrace.

\n" }, { "Id": "9932", "CreationDate": "2019-05-14T09:59:03.603", "Body": "

I recently asked a question in the space section of stackexchange as to if Martian soil, with its decent iron oxide content, could be mined (with magnets) for its iron oxide and the iron oxide used as building material for an SLS/SLM 3d printer.

\n\n

Is this possible? The final result doesn't have to be pure iron, I'm just looking to see if iron oxide is a suitable building material for a SLS/SLM 3d printer.

\n", "Title": "Is iron oxide a suitable material for use in SLS/SLM 3d printing?", "Tags": "|sls|slm|", "Answer": "

First of all, let's start with the basics:

\n

Iron oxide aka rust

\n

The University of Illinois hosts a "Ask the Van", where the question "is rust magnetic" has been asked, and I will quote from Tom J. and Mike W.:

\n
\n

There are several different oxides of iron, with different fractions of oxygen. They are Fe0, Fe2O3, and Fe3O4. Rust consists mostly of Fe2O3, with additional water molecules attached. There are several forms of Fe2O3, and a common mineral composed of Fe2O3 is called hematite, which is a shiny-blackish mineral. Hematite is not ferromagnetic, but it does still respond to a magnetic field and will be attracted to the poles of a permanent magnet.[...] FeO is also not ferromagnetic, but it is pulled about twice as much as Fe2O3 towards the poles of a magnet. Magnetite, Fe3O4, is ferromagnetic, and is about 1/4 as strong as pure iron.

\n
\n

In a followup answer by Mike W., it gets more explicit:

\n
\n

Rust (a collection of some iron oxides) is virtually non-magnetic, unlike plain iron or most types of steel.

\n
\n

This means pretty much, that unlike pure iron, you can't pick up (most) rust particles with a magnet. From all that we know about the Martian soil, we know for sure that it has a very fine grain, so the particles in itself are tiny. This again hints that any exposed iron on Mars has been thoroughly rusted through over the last million years, leaving only non-magnetic rust dust on the surface - dust that is not suitable to be mined with magnets.

\n

We have an analysis of the chemical composition of Mars from some landers, hinting that Martian regolith indeed is colored from its high content of iron in various bonds. So, what we do with that data? We create Maritan regolith simulant, which has been checked back against the findings of the probes. And low and behold: there is not a single percentile of iron in either the findings or the probes or the regolith simulants. Just about 16-18\u00a0% rust.

\n

Regolith for SLS?

\n

Now, we have regolith with a somewhat even distribution of rust in it. And we have a stimulant that can be acquired from Huston. To my knowledge, it has not yet been tested for SLM, but it has been used in powder based extruders, as explored in How does this Martian habitat 3D printer built for NASA work?

\n

With the lack of testing and the relatively low iron oxide content, I am hesitant to say that it will work to print in the usual way. However, with the addition of some polymer, one could create a fast regolith-plastic compound that shows similar behavior to concrete. This material could be made suitable for 3D printing in SLS machines. Another idea might be to go from SLM (selective laser melting) to the older SLS (selective laser sintering) or even simple sintering, in which a compound is pretty much "baked" into shape without fully melting it. We understand well how to sinter materials we have trouble with melting otherwise, and one of the prime examples is tungsten carbide.

\n

Conclusion

\n

While I see problems with mining iron from Martian regolith without a chemical refining process or refining it akin to iron sand, I don't see how martian regolith can't be refined into a suitable SLM or SLS material by addition of some kind of polymer or a thermal and mechanical process to achieve sinterable material. Instead of a polymer, a pure metal (magnesium or aluminium) could be added as a binder too. With the availability of regolith simulant for research, it only takes a research group that is interested in researching the suitability of this material for such applications.

\n" }, { "Id": "9935", "CreationDate": "2019-05-14T11:40:41.190", "Body": "

During a print a lot of plastic ended up ripping the nozzle's yellow insulation strap.

\n\n

Can printing without this insulation around the fusion chamber damage the printer?

\n\n

If there is no chance of damaging the printer, how likely is it that the prints will be affected by the absence of this insulation

\n\n

\"highlight

\n", "Title": "Nozzle insulation gone, can the printer be damaged?", "Tags": "|creality-cr-10|", "Answer": "

Insulation of the nozzle is crucial. I removed it and put a silicon sock on a CR-10. I constantly got heat creeps even at 10C lower temp. Then I removed the sock and printed a 12 hour part with significant quality loss. It did finish the job. With silicon it stopped after 5-6 hours. I suppose that with the sock the upper part of the heat block which is uninsulated leaves more energy go up than without a silicon sock. Then I put cotton all around the heat block. I even put the cotton at the side of the thermistor. And kapton tape. The results are really really perfect. Always talking for the CR-10 stock nozzle and fans. If you don't have insulation you need better fans. All the heat goes up to the heatbreak and softens the fillament.

\n" }, { "Id": "9956", "CreationDate": "2019-05-16T02:35:43.567", "Body": "

I have a Tevo Tornado that I've outfitted with an official BL-Touch auto level sensor. I can see the bed probing run, and I can see the Z axis slowly adjust during x/y moves, so it's doing something. However, you can see that there appears to be a systematic tilt:

\n\n

\"BL-Touch

\n\n

Any ideas what could be causing this? The bed, gantry and print head is tight, no wobble. Here's my start code:

\n\n
G28 ; home all axes\nG29\nG1 Z5 F5000 ; lift nozzle\n
\n\n

I have mesh leveling enabled with a 5x5 grid and correct probe offsets. The bed itself is on PETG printed standoffs instead of springs to eliminate any jitter.

\n", "Title": "BL-Touch bed leveling seems to produce tilted bed level around Y-axis", "Tags": "|bed-leveling|bltouch|", "Answer": "

I know this is incredibly old at this point, but in case anyone stumbles upon this post like I did, I wanted to point out that there is no semicolon behind your G29 code, so it's not being read properly

\n" }, { "Id": "9970", "CreationDate": "2019-05-17T20:19:47.090", "Body": "

I have a Reprap Guru Prusa i3 v2 3D printer. Here is a link to the 192 MB manual. And here is the link to the resources page for the Reprap Guru. Page 58 of the manual discusses electronics.

\n\n

I purchased a 5.5 W laser from an online resource (AliExpress). It has its own controller module. It has one connector with 3 pins

\n\n
    \n
  1. 12 V
  2. \n
  3. Gnd
  4. \n
  5. PWM
  6. \n
\n\n

I want to now connect the laser module in-place of the 3D printer nozzle. I have been able to install it physically, but not sure how to connect it to the Reprap Guru Prusa i3 board.

\n\n

I am not an electrical engineer, but I am capable of connecting wires with clear instructions. Any pointers on how to connect this module to the 3D printer board is appreciated.

\n\n

Updates:

\n\n
    \n
  1. The board of my 3D printer is an Arduino Mega 2560 board.
  2. \n
  3. I have been able to power the laser on using the fan connections and it can burn stuff (so it works). I have connected to the D9 pins
  4. \n
  5. Now need to figure out where to connect the PWM from the laser module to the Arduino Mega 2560 board
  6. \n
\n", "Title": "Add a laser module to Reprap Guru Prusa i3", "Tags": "|prusa-i3|g-code|ramps-1.4|wiring|laser|", "Answer": "

By connecting to the D9 output header (see RAMPS 1.4 shield schematic below) you only have 2 wires that represent a scheduled load and ground. You actually need to connect the positive (red) lead to the power supply 12 V and the negative (black) lead to the ground. The third wire (usually a different color) needs to be connected to the actual D9 in your example; note that this one is connected to the MOSFET! And as such not readily available, it is far more easy to use an other free pin.

\n\n
\n\n

Just use the PWM pin (attached to the MOSFET) of the print cooling fan (that schedules the MOSFET), you can then schedule the laser power with G-code M106, e.g. M106 S127 to select half the power (S255 would be max power). Alternatively, and probably a much better solution is that you can use any free (but exposed) pin of your microprocessor; you can set the value of that pin using G-code M42.

\n\n
\n

M42 switches a general purpose I/O pin. Use M42 Px Sy to set pin x to value y, when omitting Px the LEDPIN will be used.

\n
\n\n

The only electrical wiring you need to do is to attach a wire (solder or connect to a header) to bundle that with a power and ground wire and route that to the laser module.

\n\n

Note that the PWM pins of the Mega are numbered D2 through D13. Also, D44, D45 and D46 are also PWM capable. Checking the RAMPS 1.4 (the board/shield of the Reprap Guru) pinout, you will see that D8, D9 and D10 are used for the MOSFETs (and as such not easily available and would require soldering). E.g. D2 and D3 are used by the X max/min endstops (note that most printers don't use an X-max, so pin D2 may be free on your machine).

\n\n

For your purpose, any of the following pins can be used: D2, D4-7, D1112-13 and D44-46.

\n\n

Best option would be the D11 pin (on second thoughts, D4 might be a much better option as the timer associated with PWM on pin D11 is internally used in Marlin for generating interrupts); it has a pin you can connect to the SERVO header pin.

\n\n

The image shows the location of the pins:

\n\n

\"RAMPS

\n\n

An example to connect a laser module is seen in this image:\n\"Wiring

\n" }, { "Id": "9975", "CreationDate": "2019-05-19T13:27:15.653", "Body": "

I've been thinking, SLS/SLM printers currently use a roller to spread 3d printing substrate, but wouldn't ultrasonic vibrations spread the substrate more cleanly, accurately, and with greater density than a roller?

\n", "Title": "Could you use ultrasonic vibrations instead of a roller with an SLS/SLM printer?", "Tags": "|sls|slm|", "Answer": "

No

\n

The problem is twofold. Resonance and Granular convection

\n

Resonance

\n

Let's start with an empty box. We toss in some powder to create the first layer and use an ultrasonic to create a first layer. What happens? The bed starts to resonate depending on the sound you send into it in patterns - and the powder starts to form valleys and ridges along them as one can see in this video.

\n

Granular Convection

\n

What happens if one shakes a box of fine granulate that contains larger items? Granular convection happens! All items raise simultaneously and the small items start to fall first, resulting in them getting under the larger ones, so as a result end up pushing the large items up.

\n

Because of both effects, there won't be an even layer and it would raise the items printed, even if we managed to get good layers.

\n" }, { "Id": "9981", "CreationDate": "2019-05-20T00:35:25.887", "Body": "

I have been looking at getting some painters tape to use on the glass plate for better print adhesion, and everything I read suggests the blue painters tape, such as this:

\n\n

\"Blue

\n\n

However, this white tape is considerably cheaper:

\n\n

\"White

\n\n

This looks like normal masking tape to me.

\n\n

Is masking tape ok, or is the blue painter's tape preferable? If the latter, then why is that so? What is so special about the blue tape? Is it a different material?

\n", "Title": "Why does the painters tape have to be blue?", "Tags": "|adhesion|masking-tape|", "Answer": "

The second image isn't exactly painter's tape. Both images are types of masking tape, but the common manila/cream-colored masking tape vs the blue or green painter's tape typically has three features that make it less desirable for bed adhesion:

\n\n
    \n
  1. Stronger glue holding the tape to the bed, that will make it harder to change later.
  2. \n
  3. Narrower strips, so it's harder and takes longer to place the tape on the bed.
  4. \n
  5. Thicker, softer material. This is good for filament adhesion, but bad for separating from the filament after the print and accurately leveling the bed.
  6. \n
\n\n

Again: those are only typical arrangements. You can get blue painters tape at the same narrow width as manila masking tape, and you can get wider or thin manila tape. It's more a matter of what you'll commonly find for sale, and in all probability the manila/cream-colored tape will work just fine.

\n" }, { "Id": "9991", "CreationDate": "2019-05-20T22:21:36.067", "Body": "

I\u2019m designing a part that will need to be autoclaved\u2014it will be under steam at 121\u00b0C for about 15 min per job and I will want it to be able to go through the autoclave repeatedly. I ran a test PLA part through the autoclave and it warped noticeably; based on their glass transition temperatures, ABS (105\u00baC) and PETG (80\u00baC) would probably also not hold up. For a consumer-grade FDM printer, what filament materials that could be used for parts that could be autoclaved?

\n", "Title": "Material for autoclave-able part", "Tags": "|filament-choice|", "Answer": "

You need to order the part printed by an SLA machine in PA, preferably with 10 % mineral or glass content. The heat deflection temperature is suitably high for any autoclaving you'll do, and the material will resist most every that your lab and throw at it. I also went down this road with a part for my own lab and found no reasonable solution from a consumer level FDM printer.

\n" }, { "Id": "9994", "CreationDate": "2019-05-21T13:26:03.870", "Body": "

I just bought and build my first 3d printer (HE3D K280 with Marlin) and I'm encountering some problems with Cura 4 and Repetier. When I load and slice a part, the printer does not extrude anything during printing. However, when I manually extrude like 100mm (G1 F100 E100) it does work. Now I'm suspecting the problem lies with the gcode file which is generated with Cura since it contains very small values for E:

\n\n
 ;Layer height: 0.2\n ;Generated with Cura_SteamEngine 4.0.0\n M140 S60\n M105\n M190 S60\n M104 S200\n M105\n M109 S200\n M82 ;absolute extrusion mode\n G28 ;Home\n G1 Z15.0 F6000 ;Move the platform down 15mm\n ;Prime the extruder\n G92 E0\n G1 F200 E3\n G92 E0\n G92 E0\n G1 F1500 E-6.5\n\n;LAYER_COUNT:250\n;LAYER:0\nM107\nG0 F3600 X-7.753 Y4.378 Z0.3\n;TYPE:SKIRT\nG1 F1500 E0\nG1 F1800 X-8.127 Y3.918 E0.01115\nG1 X-8.35 Y3.57 E0.01893\nG1 X-9.088 Y2.287 E0.04677\nG1 X-9.348 Y1.754 E0.05792\nG1 X-9.483 Y1.376 E0.06547\nG1 X-11.413 Y-4.956 E0.18999\nG1 X-11.547 Y-5.534 E0.20115\nG1 X-11.602 Y-6.124 E0.2123\nEtc...\n
\n\n

Does anyone know how to fix this?

\n", "Title": "No extrusion, but manual extrusion works", "Tags": "|filament|extruder|ultimaker-cura|extrusion|repetier|", "Answer": "

I think that you have the incorrect diameter specified (e.g. 2.85 mm instead of 1.75 mm) in your slicer; this also appears from a calculation, see below. Note that you can calculate from extruded volume entering the hotend, or deposited volume. For the first you could calculate the line width of the deposited line and verify that with the settings; from the second you can verify if the volume for the extruded filament equals filament volume based on extruded filament going into the hotend for an assumed line width. Do note that (certainly for first layers!) modifiers may be in place. This is merely to get a ballpark feeling for the chosen filament diameter.

\n\n

If you look at the first move from:

\n\n
G0 F3600 X-7.753 Y4.378 Z0.3\n
\n\n

to:

\n\n
G1 F1800 X-8.127 Y3.918 E0.01115\n
\n\n

You can calculate the travelled distance $ s = \\sqrt{{\\Delta X}^2+{\\Delta Y}^2} = 0.59\\ mm$. Also, from these moves you can see that $0.01115\\ mm$ of filament enters the extruder $(E)$.

\n\n

The deposited volume ($V_{extruded_filament}$) of the printed line equals the cross sectional area $\\times$ length of the deposited filament path. Area could be defined as taken from e.g. the Slic3r reference manual to be:

\n\n

\"Area

\n\n

Basically (as we apply conservation of mass) the filament volume $(V_{filament})$ entering the hotend need to be the same as the extruded filament volume $(V_{extruded_filament})$ leaving the nozzle; so $ A_{filament}\\times E = A_{extruded\\ filament}\\times s $.

\n\n

This latter equation can be solved for $w$ by filling out the known parameters. From this calculation follows that for $1.75\\ mm$ filament you get a calculated line width of about $0.22\\ mm$, and respectively for $2.85\\ mm$ filament you get $0.46\\ mm$ line widths.

\n\n

As the nozzle diameter has not been specified in the question, but most commonly used nozzle diameter often is $0.4\\ mm$, and modifiers for the first layer are at play to print thicker lines; you most probably have the have the wrong filament diameter set if you have a $1.75\\ mm$ extruder setup. Basically it under-extrudes.

\n" }, { "Id": "9996", "CreationDate": "2019-05-21T17:35:11.247", "Body": "

I got some Prusa i3 clones with Melzi boards with Marlin, or nearly so. One of them performed very well (considering that the board would reboot whenever power was applied to the build plate. But I digress) until finally the hot end failed, possibly due to me running it for two days with little respite. The replacement, a typical MK8 clone, installed almost painlessly but failed to start due to over-temperature conditions (we're talking as much as 45 °C over set temperature). I tried the known solutions, continually reducing my P value (got down to 7 before I gave up) and attempting autotune. I also tried autotuning to both higher (230 °C) and lower (150 °C) temperatures.

\n\n

None of these seemed to produce an autotune successfully, and the same error message temp to[sic] high appeared in all cases. Can anyone suggest something besides a new hot end (I have one on order, but what if I have the same problem with another new one)?

\n", "Title": "PID autotune fails under all conditions so far. Any ideas I haven't tried?", "Tags": "|prusa-i3|hotend|pid|", "Answer": "

Turns out I'm using a 12V hot end and should be using 24V. I looked up the resistance to be sure; so no amount of tuning the PID would fix that.

\n" }, { "Id": "10010", "CreationDate": "2019-05-24T01:30:45.453", "Body": "

I am using LightBurn to laser engrave on wood. I am just trying to print some letters.

\n\n

In the softwares preview the output looks correct. The black part is where the laser should burn and the red part are traversal/scan lines\n\"enter

\n\n

When i actually print it the negative space is burnt by the laser (basically where the traversal/scan lines are shown in the preview above)

\n\n

\"\"

\n\n
\n\n

What I was able to figure out is that:

\n\n\n\n

But the issue is when I send the following G-code, the following happens:

\n\n
M42 P4 S255 <--- Laser turns on for a flash of a second\nG1 X15 <--- By the time the movement starts the laser is already off.\nM42 P4 S0\n
\n\n

When i stopped using PWM (via the D11) and instead connected directly to D9 (which is for the fan) this issue stops occurring. So this issue is only occurring when I use PWM. Any Guidance on what to check

\n\n

Update: I read the following on another forum. This might be the root cause here.

\n\n
\n

M42 is an immediate command and would turn on the laser before it reached its intended start point, M106 and M107 are buffered so the on/off can happen in its intended locations.

\n
\n", "Title": "laser is engraving the negative space", "Tags": "|g-code|laser|", "Answer": "

M42 command is an immediate command. This means that it will run before the move GCode commands finish. This is exactly what I was facing.

\n\n

This video has the walk-through of solving the issue:

\r\n \r\n

\n\n

Here is the relevant PDF it talks about: The 2.8 watt, $100 Laser\nUpgrade for MPCNC.

\n\n

Here is the relevant section on page 7 of the PDF:

\n\n
\n
    \n
  1. The laser driver requires a 5 volt TTL input control signal. The\n Marlin fan control Mcodes (M106 and M107) will be used to\n control the laser .Unfortunately, the Ramps fan output (D9) is a\n 12 volt signal so we can\u2019t use it. We'll need a quick firmware edit\n to remap the fan output from pin D9 (12v) to pin 44(5v).

  2. \n
  3. Make a backup copy of your Marlin firmware folder first. Open\n the pins_RAMPS_13.h file in your Marlin firmware folder with a\n text editor (Wordpad). Search for the line where the fan pin is\n assigned and change it from pin 9 to pin 44.

  4. \n
  5. Save the changes and flash the revised firmware back onto your\n Mega board.

  6. \n
\n
\n" }, { "Id": "10013", "CreationDate": "2019-05-24T07:56:30.933", "Body": "

Please check following image, Dog looks smooth from left side but its rough from right side , similar on back too.

\n\n

What could have caused this ?

\n\n

\n\n

Can it be due to moisture due to Air Conditioner in my room ?\n\"dog

\n", "Title": "Ender 3 first print some area is smooth but some is rough?", "Tags": "|print-quality|creality-ender-3|", "Answer": "

I was making following mistakes

\n\n

a) X-axis belt needed a tightening\n( I calibrated all X,Y,Z and they were perfect)

\n\n

b) There was under extrusion .\n ( I had to increase number of steps per mm for extruder motor and store the setting)

\n\n

XYZ calibration cube was really helpful in debugging the problems .

\n" }, { "Id": "10035", "CreationDate": "2019-05-26T20:16:06.960", "Body": "

I've been playing around with PETG for the first time, and everything seemingly worked right just from the start - clean prints, no stringing, no bed adhesion problems, no warping or dimensional accuracy problems, etc. As expected it prints a lot like PLA, and as expected, it's less brittle/stands up much better to crushing/impact, except that it's really brittle when it comes to inter-layer adhesion. Vertical cylinders that were fairly strong in PLA just snap with no effort as PETG.

\n\n

My particular PETG filament is Sunlu, with recommended print temperature 230-250 \u00b0C. I started out with 235 and am now using 250, which does somewhat better. I've used layer heights 0.125 - 0.2 mm.

\n\n

Are these kind of results normal? Is there anything I should be doing to get better adhesion between layers?

\n", "Title": "PETG layer adhesion", "Tags": "|print-material|adhesion|petg|", "Answer": "

While 0scar was right that cooling fan hurts layer adhesion, I've continued to have problems with PETG even with no fan, regardless of temperature. I went looking for advice on the topic, and found a video by CNC Kitchen emphasizing the importance of tuning extrusion rate because of the compressibility of the material in the extruder gear. I'd already found this was a huge issue with TPU and other flexible filaments, so it seemed compelling, and sure enough I just measured that a nominal extrusion of 180 mm only moved the filament by 173.5 mm.

\n

OK. Having extrusion rate off by about 3.5% is plenty to make prints brittle with PLA - I've experimented with this before just to see what would happen. A longer more precise extruder calibration showed more like 4% error. After correcting this, things were better, but I was still getting severe brittleness in some parts of the print but not others.

\n

For a long time, I was able to mitigate most of the remaining problem with reduced speed. I had already reduced travel speeds down from 120 mm/s to 60 mm/s (my normal print speed) because PETG is sticky fast travel over it with the nozzle in contact will tear up the already-printed surface and inhibit adhesion of the next layer. (This seems to be soft PETG acting as a non-Newtonian fluid. Disabling combing, which I'd done for other reasons already with all materials, probably helped with this too.) After also reducing print speed to 40 mm/s, things seemed mostly ok. But I found recently I was still getting serious localized underextrusion in the form of entire lines nearly missing, especially after complex retractions.

\n

I traced this problem down to some extreme extruder speed and jerk, which I'd allowed to mitigate the cost of lots of retractions and linear advance extruder moves. PLA and especially flex materials (where this matters most) can handle ridiculously high extruder speed (150 mm/s) and jerk (25 mm/s "instantaneous" velocity change), but PETG quickly starts slipping in the extruder gear when you do that, and making it easy to "lose" several mm of filament when unretracting. With this fixed (reverted to default 25 mm/s speed and 5 mm/s jerk; 10 mm/s seems to work ok too and performs a lot better), I finally have really strong PETG parts, comparable to PLA.

\n

In the process I also tuned linear advance K factor for PETG, which could impact adhesion. I started with 2.0 which was too high, and dropped to 1.2 which was slightly too low; around 1.4 seems to be ideal. Having this too low could reduce layer adhesion right after acceleration due to localized underextrusion; having it too high could reintroduce extruder gear slippage by putting the filament under more pressure than the gear can reliably hold it to. (If a higher value is needed to get consistent extrusion, this would mean a limit on the speed would also be needed, and going at higher speeds would require an extruder upgrade. For reference, at 0.4 mm line width and 0.2 mm layer height, a K value of 1.2 requires the extruder to be able to compress the filament by about 2.4 mm to print at 60 mm/s.)

\n

TL;DR: Fan completely off, tune extrusion rate to account for compression of PETG in the extruder gear, avoid travel over already-printed material especially at high speeds (limit travel speed to print speed), and keep extruder speed/acceleration/jerk profile conservative.

\n

Update: Almost all of the issues described in this answer seem to stem from the Ender 3's abysmally bad extruder. Some are probably slipping due to really poor grip from the gear; others might be common to all bowden extruders. With the extruder I'm now using (Flex3drive G5) on the otherwise-same printer, I can print PETG at same speed or faster than PLA, with no under- or inconsistent extrusion issues. Cooling does seem to affect layer adhesion, but mostly on very thin (single-wall) parts; otherwise even with fan on at 100% I get better adhesion than I could reliably get with the original extruder. So I think the biggest issue was underextrusion, not over-cooling.

\n" }, { "Id": "10040", "CreationDate": "2019-05-28T04:01:33.180", "Body": "

I'm not really sure where to ask this question as I think it is a design question, but also a printing question. So if there is a better place to post, I'd be happy to harass someone else.
\n I'm (re)designing a sprinkler manifold for a dripper system because the stupid pegs for this stupid manifold are on top of the manifold, which is a prime spot for any old postal person/dog/raindrop to break off. Of course the pegs aren't sold separately so you have to buy a whole new manifold. Seems like a great use for a 3D printer.

\n\n

\"sprinkler

\n\n

I designed a new manifold and decided the pegs were useful in case they broke off. I was thinking having them screw in would be a better design, but for the life of me I can't get them to actually screw in after I print. \nHere is the fusion 360 file.\nThis is generally what it looks like:\n\"MushroomManifold\"\n And here is the resulting stl file.

\n\n

After several prints, the pegs won't screw into the manifold base. I push and I turn and turn but the threads just won't bite. The 3/4\" pipe threads fit just fine, so I know threads can be printed, but these pegs are stubborn.

\n\n

I guess my question is, what's a good design for a peg thingy that needs to attach into a manifold, but also pass water? Should I try to replicate the cantilever thing they have going on, or is a screw better? Any ideas why my pegs won't screw into the base of my mushroom? This is my first attempt at 3d modeling so I'm not totally familiar with all the terminology, so any pointers there would be helpful. \nThanks!

\n", "Title": "Advice for 3D modeling peg for sprinkler dripper", "Tags": "|prusa-i3|3d-design|", "Answer": "

With the suggestions from @R.., I played with a couple of different screw profiles that come with fusion 360, and found these settings to be helpful:

\n\n

\"enter

\n\n

After cutting the hole with these screw settings, I selected all the faces of the hole and off set them by -0.1 mm. Originally, I was offsetting the hole by like -0.02 mm and the peg wouldn't screw. After learning a bit about tolerances of printers, I expanded this to the .1 value and it screwed in magically! I haven't yet worked out if the pegs are water tight, so I may have to revert back to the previous thread settings that seem like they would be tighter, or maybe I'll invest in an o-ring. Suggestions welcome.

\n\n

Thanks to everyone for their input.

\n" }, { "Id": "10045", "CreationDate": "2019-05-28T18:57:20.927", "Body": "

I have a RAMPS 1.6 board. After soldering my stepper drivers, I probed them for bridging and found a short where none should be. Digging deaper into it, my multimeter shows continuity between the 12V +/- connectors (on occasion).

\n\n

I have scanned the whole board repeatedly for solder bridges, but could not find any. Are there any known weaknesses that I should look for or any specific place I should look at to repair it?

\n", "Title": "Short circuit on RAMPS 1.6 board?", "Tags": "|electronics|ramps|ramps-1.6|", "Answer": "

Your multimeter showing continuity doesn't necessarily mean there is a short.

\n\n

All your multimeter is doing is applying a small voltage and then, if the current that flows is over a certain threshold, reporting that there is continuity.

\n\n

The components (stepper drivers, microprocessor) on the board, draw current. That's normal. The current draw might be enough for your multimeter to report continuity. Because the multimeter is only using a small voltage to test (and not the required 12V), the current draw may be intermittent (not enough voltage for the microprocessor to actually start working), and capacitors getting (dis-)charged may also affect things.

\n\n

The multimeter not reporting continuity is a guarantee of no short. However, the multimeter reporting continuity does not mean there is a short. The only way to find out is to apply 12V. If you use a current-limited power supply the possibility of damage if there is a short circuit is limited.

\n" }, { "Id": "10059", "CreationDate": "2019-05-29T17:28:17.730", "Body": "

I've been aware ever since I got it that my Ender 3's X-axis isn't level. Measuring from the top of it to the top of the frame, the right-hand (positive) side is about 4.7mm higher than the left.

\n\n

During assembly, the vertical rails were not entirely parallel, and had to be pulled together to get the X axis on and to bolt the cross beam on the top. I suspect this is related, but I'm not sure.

\n\n

Anyway, aside from the bed having to be tilted to be level with respect to the X axis, this never seemed to cause any problem, so I've left it alone until now. However I have measured almost exactly a 1% dimensional error in the X direction that I've now compensated for by setting the steps per mm, and wonder if the tilted axis could be the cause. Doing the trig, that doesn't make sense - a 4.7 mm error across the width of the bed should translate to something like one part in 2000, not 1%. But maybe something's wrong in my analysis so I'd like second opinions.

\n\n

Aside from that, is this something I should try to correct, or just let be? I suspect it's the base that's warped or tapped/cut incorrectly where the vertical rails bolt on, in which case it seems unlikely there's any way to fix it without replacing that part, which is something I'd rather not get into as long as the printer is working. But if there are other possibilities that are non-invasive to try, I might.

\n\n

\"whole

\n\n

\"x

\n\n

Further update: if this is the cause of the dimensional error (which turned out to be more like 0.41% than 1%, thus closer to plausible) I probably need to fix it. Using firmware steps/mm adjustment is not viable because it produces aliasing patterns in skin layers presumably due to step width no longer dividing nozzle size/line width.

\n", "Title": "Ender 3 X axis not level", "Tags": "|calibration|creality-ender-3|x-axis|", "Answer": "

So I disassembled the printer and first found something very suspicious: the left (Z-motor side) vertical rail was not mounted flush against the base, because the edge of the control board cover panel was under the edge of it. Fixing this made the vertical rails parallel and made the X-axis unit easily slide back on, but it did not fix the issue; the X-axis was still non-level.

\n\n

Next, I started playing with eccentric nuts, which was probably a mistake. They were already the appropriate tightness, and I might have messed them up, in which case I'll have to go back and tune them more later. I then found a second point of adjustment: the attachment of the roughly-triangular bracket that holds the three roller wheels on the right side to the X-axis aluminum extrusion rail. I as soon as I loosened the bolts, I was able to get a plenty play to level the axis. In order to get it level, with the top beam mounted, I raised both sides so that the top outer wheels would hit the plastic end caps of the top beam. This relied on a dubious assumption that the triangular roller wheel arrangements on both sides are symmetric, but it seems correct, and after re-tightening the bracket, lowering it to the bed, and re-leveling the bed, everything seems fully level.

\n\n

Further, after the fix, my dimensional accuracy test piece is 119.6 mm instead of 119.5 mm, which works out to the difference before the fix being undersized by 0.084%, very close to my \"1 part in 2000\" estimate of the error that the non-level axis should cause. Sadly it still wasn't 120.0 mm like it should be, so I went looking for another source of the inaccuracy. Tensioning the X-axis belt seems to have done the job, and I'm now getting 120.0 mm.

\n\n

I took some pictures in the process, which I'll try to attach later to improve this answer for others who may hit the same problem.

\n\n

Caveats: In the process of doing this, I badly messed up the Z axis. I was getting prints coming out almost 1mm shorter than they should be and having to relevel the bed continually. The whole system is \"over-constrained\" by all the points at which it can be tightened, so if anything is inconsistent when it's tightened, everything goes catastrophically wrong. This entry on Maker Steve's blog was very helpful in figuring out how to get it back in order. After making the adjustments, however, I still had a problem: a horrible grinding squeal whenever the Z motor moved in the negative direction. It turned out I'd also over-tightened the screws holding the Z lead screw nut on the carriage (after mistakenly removing it during all this), forgetting that the assembly instructions specify that it shouldn't be tightened down. loosening both screws by about half a turn finally got everything working right again.

\n" }, { "Id": "10061", "CreationDate": "2019-05-29T21:13:49.873", "Body": "

After a few months of printing with my Prusa Mk3 (with plans to get a second one soon), I have been wondering about making my third printer a home-built one was a larger print bed than the Mk3. One thing I wondered about is perfectly expressed in the title question.

\n\n

Are there practical reasons to not use a stepper motor with lead screw for the X and or Y axes?

\n\n

I am certainly happy with the GT2 belts used in my current printer, but I wonder if the design might be simpler with lead-screws on all three axes.

\n", "Title": "Are there practical reasons to NOT use a stepper motor with lead screw for the X and or Y axes?", "Tags": "|diy-3d-printer|lead-screw|", "Answer": "

It is possible to use lead screws; specifically 4 start leadscrews. The only drawback is that you need to be wary of heat.

\n\n

Let's breakdown the concerns

\n\n\n\n

Just from changing the drivers you should be able to get a speed increase without needing to resort to the heavier multi start lead screw. Increasing the voltage will also help, however you would need a driver that can let you vary the holding current, otherwise the motor will heat up and burn when it is not moving.

\n" }, { "Id": "10081", "CreationDate": "2019-05-30T03:13:06.703", "Body": "

I have two Tronxy 2.0 V5 Marlin boards that reboot whenever heat is applied to the bed. The bed has been swapped (because I thought that was the problem) for a new shiny one. The thermistors, too, of course. The same boards (both) work when the beds remain unheated (setpoint = 0 °C).

\n\n

Any ideas what might be causing this, or what I might do to figure it out?

\n\n

Note: I really have no idea which Tronxy board this is; the \"2.0\" is stenciled on the board, so that's all I can figure out. I shamefully admit I tagged it with Tronxy x1 to see if I could generate any interest, and because a \"Tronxy\" tag is not available.

\n", "Title": "Tronxy Marlin boards (two of them) reboot when asked to heat bed", "Tags": "|marlin|heated-bed|firmware|tronxy|", "Answer": "

First, check the power supply. Although it may be specified to deliver the required power, it is possible that the power supply has failed in a way that it can not deliver the rated power. At lower load, the voltage may be correct, but under higher load, it either droops or cuts out completely.

\n\n

To check this, use a voltmeter on the power as it enters the CPU board, not where it leaves the power supply. This accomplishes one additional check. If the voltage droops rather than cuts off, it may be that the connections have corroded and have a higher resistance.

\n\n

If you have any kind of oscilloscope, I would recommend it over a simple voltmeter, because the power interruption or droop time may be very short. When the CPU resets it will switch off the load that causes the problem, and the power may quickly resume the correct value.

\n\n

Second, check that there is not a short in the bed wiring. You might detect that with an ohm meter. You have used two different CPU boards, to it is unlikely to be a common fault on both boards, but you might be using the same wiring.

\n\n

Third, check the routing of the bed heater wires to see that they are not near other wires which connect with the CPU, including thermistor wires and wires to the UI. High-current switching in the bed wires could be coupling into other wires and conducting a RESET signal to the CPU. Ideally, the heater wires will be twisted together with about 3 (or more) twists per inch, and not twisted together with other wires.

\n" }, { "Id": "10142", "CreationDate": "2019-06-01T06:08:38.777", "Body": "

I'm trying to upload a file using Octoprint REST API - it seems to be possible as per the document described here, Upload file or create folder, but I can't figure it out how to do it with the Python request lib.

\n\n

Currently what I'm doing is

\n\n
 import requests\n\n def def Upload_File():\n       fle={'location':\"j:/max.gcode\"}\n       url='http://localhost:5000/api/files/{}'.format('local')\n       payload={'select': 'true','print': 'false' }\n       header={'content-type': 'multipart/form-data','X-Api-Key': 'FD550BD4DA2442BA906AD1850539D6DB' }\n       response = requests.post(url, files=fle,data=payload,headers=header)\n       print(response)\n\nif __name__=='__main__':\n    Upload_File()\n
\n\n

The response returns status is 400

\n\n

My working env is:

\n\n\n", "Title": "Uploading files using Octoprint REST API", "Tags": "|octoprint|", "Answer": "

I installed octoprint locally to help debug this problem.

\n\n
    \n
  1. Your code snippet says \"def def Upload_File\" which is a syntax error.
  2. \n
  3. If you go into Octoprint, under Settings -> Logging, and download octoprint.log, you'll notice it says \"WARNING - 400 POST /api/files/local (::1): No multipart boundary supplied\". A quick google search led me to this StackOverflow question, which states to remove the custom content-type header to fix this problem.
  4. \n
  5. You actually need to open() the file, and assign that value to the 'file' field in the fle object definition, not just tell it where it's currently at on your machine. Additionally, you need to provide a filename field in the fle object.
  6. \n
\n\n

I got your code snippet working as below, went ahead and swapped your API key back into it. You'll need to point the open() call to the actual location of your file on your hard drive, which I assume is j:/max.gcode, rather than mine which is just opening max.gcode from the running directory of the script.

\n\n
import requests \n\ndef Upload_File():\n   fle={'file': open('max.gcode', 'rb'), 'filename': 'max.gcode'}\n   url='http://localhost:5000/api/files/{}'.format('local')\n   payload={'select': 'true','print': 'false' }\n   header={'X-Api-Key': 'FD550BD4DA2442BA906AD1850539D6DB' }\n   response = requests.post(url, files=fle,data=payload,headers=header)\n   print(response)\n\nif __name__=='__main__':\n   Upload_File()\n
\n\n

Finally, if you're going to be doing a whole lot of REST API shenanigans with your Octoprint server, may I offer a prebuilt library?

\n" }, { "Id": "10143", "CreationDate": "2019-06-01T14:22:34.160", "Body": "

I now own the Prusa3D MMU2. The benefits, costs, and experience others have had is well documented. I am interested in rebuilding my large, home-designed delta machine to be multi-material, and don't want to overlook strategies I haven't considered.

\n\n

My original implementation used an E3D Kraken as the hot-end, and handled the inevitable delta tilt by adding two additional degrees of freedom to the head to lower the selected nozzle to the bed. I've been through three generations of mechanisms, and I think the third will work.

\n\n

But, I feel that I am not seeing obvious and better alternatives.

\n\n

So, the question: Through what methods and mechanisms can a multi-material (different polymers, different temperatures) FDM printer operate, and are there available designs or examples of best practices for those methods?

\n", "Title": "Through what methods and mechanisms can a multi-material FDM printer operate?", "Tags": "|delta|multi-material|open-source|", "Answer": "

Another way to combine the simpler geometry of a single nozzle, and to get the reduced mass of a single extrusion tool would be to make it like a CNC machine with a tool changer. One material is printed, then the hot end, extruder, and feed tube are swapped out for another which is primed and ready with the next material.

\n\n

Lots of mechanical precision problems exist for arranging for the nozzles to be in the very same place, plus or minus a small tolerance. This is worsened by the presence of filament bits and strings which seem to eventually pollute the workspace.

\n\n

If that could be worked out, one could have a plethora of extrusion tools, nozzle sizes, materials, multi-material mixing chambers, and other complexities.

\n\n

E3D was talking about such a printer, but I haven't seen a product... only an invitation to send money as a show of interest in such a printer.

\n" }, { "Id": "10159", "CreationDate": "2019-06-03T05:58:00.617", "Body": "

Do you have any specific process recommendations to achieve support for the parts hanging above 45 degrees, using open source software?

\n", "Title": "How to create supports for the parts hanging above 45 degrees?", "Tags": "|3d-design|diy-3d-printer|support-structures|open-source|", "Answer": "

PrusaSlicer has support enforcers you can place on areas that need support.

\n

See this video: Prusa Slicer Support Enforcers.

\n

Automatic supports

\n

There are automatic supports, in the Supports dropdown menu, select Everywhere:

\n

\"Supports

\n

Click Yes in the resulting dialog:

\n

\"Click

\n

Click on the Slice icon in the bottom left:

\n

\"Click

\n

You will end up with a lot of supports:

\n

\"Resulting

\n

However, this method generally results in too much support...

\n

Custom Suport Enforcers

\n

So turn off Supports in the drop down menu, select None:

\n

\"Supports

\n

Now, in the Print Settings tab, under Support Material, disable Auto Generated Supports and enable Generate Support Material:

\n

\"Print

\n

Then right click on the model and select Box from the Add Support Enforcer menu item:

\n

\"Add

\n

You can move by clicking the Move button in the left hand palette,

\n

\"Move

\n

resize by clicking the Resize button

\n

\"Resize

\n

and re-shape the Box as necessary, to support the difficult overhanging parts parts only. Here we can see three boxes have been added - pale blue for the previously added boxes and green for the current box:

\n

\"Three

\n

And a fourth final Box for the tail:

\n

\"Fourth

\n

Now, when you hit the Slice button, there will be much less support structure than when using Automatic supports:

\n

\"Reduced

\n

Parts of the model that the slicer thinks still needs support will be highlighted in dark blue (such as the elbow and back of the head):

\n

\"Unsupported

\n

Add a cylinder support enforcer:

\n

\"Add

\n

Resize and re-shape as before and move into position below the elbow:

\n

\"Putting

\n

Adding a second cylinder as a support enforcer

\n

\"Putting

\n

Upon hitting Slice:

\n

\"Final

\n

Now in Print Settings - Support Material - Pattern change it from Rectilinear to Rectilinear Grid:

\n

\"Print

\n

For prints with curves and details Rectilinear Grid works better, than Rectilinear (which is fine for supporting a plain cube in the air). It is easier to break the support off the print.

\n

Now save your hard work as an AMF file:

\n

\"Save

\n

This file maintains all of the support enforcers so that they can be modified if the actual print needs some adjustments - without having to re-add all of the support enforcers all over again.

\n" }, { "Id": "10171", "CreationDate": "2019-06-04T04:22:43.570", "Body": "

I want to know the math behind how Slic3r calculates E values. $E_{value}$ represents amount of filament in mm (unless volumetric extrusion is selected) that has to be fed into the hotend to obtain a road of specific extrusion width.

\n\n

Consider an example with following parameters:

\n\n\n\n

First part of the question: How is $E_{value}$ calculated for this case?

\n\n

Second part of the question: How is velocity of extruder motor calculated for this case?

\n\n

The Slic3r manual has limited information on flow math but is not comprehensive.

\n\n

Let's assume volume of plastic fed in equal volume of plastic comes out

\n\n

$$Volume_{in} = \\pi\\times{(\\frac{d}{2})}^2 \\times E\u00a0\\times x = \\frac{\\pi\\cdot d^2}{4}\\times E \\times x$$

\n\n

Where,

\n\n\n\n

$$Volume_{out} = (A_{road} \\times L)$$

\n\n\n\n

$$A_{road} = (w - h)\\times h + \\pi\\times{(\\frac{h}{2})}^2 $$

\n\n

Where,

\n\n\n\n

Seems like I am missing something. Math doesn't yield me same result as Slic3r $E$ value.

\n\n
\n\n

Many of you have marked this question duplicate. I know the first question is similar to what has asked before (calculating E value) but the answer doesn't match actual E value in G-code.

\n\n

Also there is a second question on how to calculate extrusion speed given an E value

\n\n

I have added G-code from actual Slic3r with the same settings as above to check the math.

\n\n

The advance extrusion width settings in slic3r are as shown in the picture below:\n\"enter\nThe settings are from a Prusa config for 0.6 mm nozzle

\n\n

Consider a 100 mm x 100 mm x 5 mm part (X x Y x Z dimensions). Following is the output G-code from Slic3r:

\n\n
; generated by Slic3r 1.3.0 on 2019-06-04 at 16:36:24\n\n; external perimeters extrusion width = 0.61mm (6.55mm^3/s)\n\n; perimeters extrusion width = 0.65mm (10.54mm^3/s)\n\n; infill extrusion width = 0.70mm (15.25mm^3/s)\n\n; solid infill extrusion width = 0.65mm (8.78mm^3/s)\n\n; top infill extrusion width = 0.60mm (6.43mm^3/s)\n
\n\n

------ Values of parameters defined in Slic3r -------

\n\n\n\n

------ some initialization lines above --------

\n\n
G1 F1800\n\nG1 X78.400 Y169.100 E8.21483 ; perimeter\n\n**G1 X78.400 Y70.900 E8.21483 ; perimeter**\n\nG1 X176.600 Y70.900 E8.21483 ; perimeter\n\nG1 X176.600 Y169.010 E8.20731 ; perimeter\n\nG1 X177.175 Y169.675 F10800.000 ; move to first perimeter point\n
\n\n

The above code snippet refers to the perimeter of the very first layer in the print. Let us consider the highlighted line in above G-code. According to equations we have above, the values of the variables are:

\n\n\n\n

Area of the depositied road is:

\n\n

$$A_{road} = (0.61 - 0.35)\\times 0.35 + \\pi\\times{(\\frac{0.35}{2})}^2 $$\n$$A_{road} = 0.187211 mm^2 $$

\n\n

For calculating $E_{value}$, We use volume equality

\n\n

$$Volume_{in} = Volume_{out}$$

\n\n

$$E_{value} = \\frac{A\\times L \\times 4} {\\pi\\times d^2 \\times x} = \\frac{0.187211 \\times 98.2 \\times 4} {\\pi \\times 1.75^2 \\times 1} = 7.6432 $$

\n\n

The $E_{value}$ in the G-code is 8.214

\n\n

This is a big difference isn't it? I know about the die swell effect and expansion of molten plastic at the tip, but there seems to be no uniform compensation factor for this!

\n", "Title": "How is E value calculated in Slic3r?", "Tags": "|slicing|extrusion|slic3r|", "Answer": "

To answer your first question:

\n\n

Your calculations are not wrong, they are correct for a normal layer (uncorrected) layer. These calculations should get you very near the solution. The problem is that there are default modifiers at play that modify the extrusion process which become apparent when you change them or look at the hoover hint in the advanced printer settings section. E.g. see the image below of the \"Print Settings\" graphical user interface; specifically look at the hoovering hint:

\n\n

\"Slic3r

\n\n

The hoovering hint tells you that there is a 200 % modifier at play. What! a default modifier without me knowing? Well...., if we had looked at the \nSlic3r Manual (The Important First Layer) a little better, we read that:

\n\n
\n

Fatter extrusion width.
The more material touching the bed, the better\n the object will adhere to it, and this can be achieved by increasing\n the extrusion width of the first layer, either by a percentage or a\n fixed amount. Any spaces between the extrusions are adjusted\n accordingly.

\n \n

A value of approximately 200 % is usually recommended, but note that\n the value is calculated from the layer height and so the value should\n only be set if the layer height is the highest possible. For example,\n if the layer height is 0.1 mm, and the extrusion width is set to 200 %,\n then the actual extruded width will only be 0.2 mm, which is smaller\n than the nozzle. This would cause poor flow and lead to a failed\n print. It is therefore highly recommended to combine the high first\n layer height technique recommended above with this one. Setting the\n first layer height to 0.35 mm and the first extrusion width to 200 %\n would result in a nice fat extrusion 0.65 mm wide.

\n
\n\n

Tada! There we have the modifier from the screenshot; 200 % (this is expressed as a percentage over the layer height, and causes that an additional filament scale factor bigger than 1 is at play; the $x$ in your equations).

\n\n

To answer the second question:

\n\n

That should be rather straight forward, you know how long the path is and at which speed the head is moving (either at constant speed, decelerating or accelerating) and how much of filament you need to deposit, at the end point all filament needs to be deposited so you can calculate how fast the extrusion needs to be to accomplish that.

\n\n
\n\n

If you calculate back from a volume of 8.214 mm2 and solve for unknown $w$ you see that this yields $ w = 0.65\\ mm $, and that is exactly what is stated as first layer width in your Slic3r settings; I quote:

\n\n
\n

first_layer_extrusion_width = 0.65

\n
\n\n
\n\n

P.S. When you look into the source code of Slic3r, if you dig deep, you find that extrusion width is bound by minimum and maximum values, it could well be that that is causing the value to differ from 0.70 mm (200 % of 0.35).

\n" }, { "Id": "10176", "CreationDate": "2019-06-04T17:28:53.913", "Body": "

I have a WhamBam build system on order. A magnet attaches (glues down I think) to the Aluminium printing bed (or add a glass sheet? Separate question), then PEX material on flexible steel sheet gets slapped down for the print surface.

\n\n

\"WhamBam

\n\n

I have been printing PLA on a cold PEI sheet from Vertex, using 4 binder clips to hold it in place vs peeling the backing off and sticking it down. The print job wants to keep heating the bed to 60 \u00b0C, I turn it down, a couple minutes into a job it cranks it back to 60 \u00b0C and I turn it down again. A few times I missed the second turn on, and the PEI has been kinda warped now (or maybe it's just the plastic over the sticky backing), and has also peeled off some surface chunks in the middle, so we've been trying to print around the damaged section. The warping has now made the PEI unusable, so I'm hoping the WhamBam arrives soon.

\n\n

We've been printing for a couple of weeks now (I printed a chess set, largest has 4 cm diameter and is 10 cm tall, some pieces on blue tape, some on the PEI) and are starting to venture into our own designs.

\n\n

Intended project is box tops & bottoms that are ~90 mm x 65 mm x different heights with openings. We tried one on blue tape (a bottom with no openings) and ended up chiseling it off the bed with a steel putty knife (I don't recall if heat was on or not). We tried a top with openings on the PEI, missed that the heat had turned back on, but between the bed not quite level (forgot to re-check it) and the PEI being warped we killed it after the openings were printed around. It was not going to be usable, but we did print enough to be able to confirm the opening spacings (needs work still) so it was not a total loss. Came off the PEI easily (<2 mm thick when we stopped), we managed to miss the damaged parts mostly.

\n\n

So the question: when the WhamBam arrives, is it better to print PLA at 60 \u00b0C, or do I keep playing the game of turning it down (and saving the waiting time of it heating up)?

\n", "Title": "Printing PLA on PEX surface - is heated bed needed?", "Tags": "|heated-bed|", "Answer": "

Received and installed the WhamBam system, and it works great! Have been letting it heat to 60C, which I'm pretty sure is not making it all the way thru the magnet, flex steel plate, and the PEX surface, but parts are adhering great and pop right off the PEX just by the act of picking up the plate, even a large box bottom (~68mm x 95mm) came off so easily I thought it had lost adhesion - my wife printed several items earlier and they came off the same way. I am impressed with the system!

\n\n

We had lost a blade off the hot end fan (and turns out it was 3 blades), initial prints had failed to adhere on the PEX, the PLA didn't appear to be extruding properly, and the fan irregularity was very irritating. Internet searches indicated the plastic melting temp was not accurate enough. Replaced the fan and every print since has been great.

\n\n

EDIT 6/14/2019 Here is a youtube clip of a freshly printed door knob (screen door latch) coming off a 60C bed, no problem!

\n\n

\r\n \r\n

\n\n

And a larger box, ~65 x 98mm, that popped right off.

\n\n

\r\n \r\n

\n" }, { "Id": "10182", "CreationDate": "2019-06-05T15:32:24.787", "Body": "

I have been looking in the Marlin firmware for about an hour or two now to find what pins I should use for UART for my tmc2208 drivers and I have come up with nothing.

\n\n

Does anyone know what they are or how to assign them? BTW, I am using the regular version of the RUMBA board not the RUMBA+ version.

\n\n

For somereason i didn't put that i want to control my stepper motor drivers through uart they are tmc2208

\n", "Title": "What pins do I use for UART control on a RUMBA board for tmc2208?", "Tags": "|marlin|troubleshooting|electronics|stepper-driver|", "Answer": "

The RUMBA schematic is available on the RUMBA wiki.

\n\n
\n

\"RUMBA

\n
\n\n

From the schematic, I see that UART3 (with +5V logic levels, not RS232) is presented on the EXP3 connector. I don't know if Marlin firmware can be controlled through a UART other than UART0, which is converted to USB through an FDDI chip. The Arduino bootloader is not expecting another UART, so you may still need to program it through the USB port (and UART0).

\n" }, { "Id": "10200", "CreationDate": "2019-06-07T17:01:41.877", "Body": "

What's the legal status of Chinese 3D printer part clones such as Hiwin linear rail clones or E3D hotends?

\n\n

While it's clear that Chinese clones are certainly sub-par in quality, no question, what I wonder is whether they violate any laws (they don't use brand names or claim to be the genuine product).

\n\n

In particular, I\u2019m interested whether it is in any way illegal to order such parts in the U.S.

\n\n

Given that there's hardly any 3D printer that doesn't contain at least one Chinese part, I really think this is on topic.

\n\n

I also don't seek legal advice but any information other 3D printer users on here have.

\n", "Title": "3D printer part clones from china - legality", "Tags": "|legal|", "Answer": "

This question is really a legal question, and could apply to any cloned parts/devices rather than being 3D printer specific, and a generic counterfeit consumer goods based question should be asked on SE.Law. However, as you rightly state, a lot of 3D printers from China may contain (whether knowingly sourced or not by the manufacturer1) counterfeit parts, be that ICs, hotend designs (i.e. E3D clones), controllers (i.e. Arduino Mega boards or stand-alone non-RAMPS Arduino shield boards),or what have you. As such this is an issue that may be faced by any unwittingly innocent consumer.

\n\n

Prefacing any statement with I am not a lawyer (IANAL), these sections from Wikipedia entry on Counterfeit consumer goods might help answer your question:

\n\n\n\n

In short, if they have a mind to, Customs could seize it at the border; if your house was raided (for whatever reason) law enforcement could seize your printer (although this seems unlikely, unless they were explicitly raiding your house for knock-offs); and (more worryingly) there is a proposal to fine those people who purchase knock-offs.

\n\n

Of course this is not the only legal issue that may be encountered when buying Chinese devices/parts from less-than-reputable suppliers o eBay or AliExpress, for example, what if:

\n\n\n\n

These questions lead into rather murky grey and legally complex areas, and really would need to be dealt with by a legal professional (solicitor/lawyer).

\n\n
\n\n

Other Stack Exchange posts worth reading

\n\n

These deal more with quality not legality but see

\n\n\n\n
\n\n

1 A bone fide low end oscilloscope manufacturer got stung by a batch of fake regulator ICs just last year, see Re: JYE Tech DSO150 oscilloscope troubles

\n" }, { "Id": "10205", "CreationDate": "2019-06-08T17:20:02.833", "Body": "

Given a 3D boolean array representing voxels, how can it be converted to a 3D-printer-ready file?

\n\n

The end-goal I would like to achieve is to print the 3D shape that the numpy array represents (True coding for fill this voxel, False for leave it empty).

\n\n

For example, the array

\n\n
[\n    [\n        [T, T, T],\n        [T, F, T],\n        [T, T, T]\n    ],\n    [\n        [T, F, T],\n        [F, F, F],\n        [T, F, T]\n    ],\n    [\n        [T, T, T],\n        [T, F, T],\n        [T, T, T]\n    ]\n]\n
\n\n

would encode a level-1 Menger sponge.

\n", "Title": "Convert a 3D NumPy array of voxels to an STL file", "Tags": "|stl|python|", "Answer": "

You can try mayavi.mlab:

\n

Usage

\n
from mayavi import mlab\nimport numpy as np\n\n\ndef draw3d_mayavi(array, path):\n    mlab.contour3d(array.astype(np.int32)) # a window would pop up\n    mlab.savefig(path)\n    mlab.clf() # clear the scene to generate a new one\n
\n

mayavi's recontruction is meant for generating 3D heatmap models of the array, so you have to put in a numeric one with 1s and 0s.

\n

Note

\n

There are some drawbacks:

\n
    \n
  1. A window will pop out, you have to clear it in your code if you want to make multiple models.

    \n
  2. \n
  3. The model reconstructed is .obj and can be very large. If you look closer at the model, you'll see that on the boder the mesh gets 3 layers. I guess the program assumes there to be some gradient.

    \n
  4. \n
  5. The contour3d function can set line_width, but I don't see any sense of using it for binary data.

    \n
  6. \n
\n

Yet, mayavi is very quick, at least compared with voxelfuse. Maybe some post-processing is needed to solve the size problem.

\n

Doc

\n

This function also enables setting color and opacity, etc.\nSee Plotting functions - contour3d:

\n
\n

contour3d

\n
mayavi.mlab.contour3d(*args, **kwargs)
\n

Plots iso-surfaces for a 3D volume of data supplied as arguments.

\n

Function signatures:

\n

contour3d(scalars, ...) contour3d(x, y, z, scalars, ...)\nscalars is a 3D numpy arrays giving the data on a grid.

\n

If 4 arrays, (x, y, z, scalars) are passed, the 3 first arrays give\nthe position, and the last the scalar value. The x, y and z arrays are\nthen supposed to have been generated by numpy.mgrid, in other words,\nthey are 3D arrays, with positions lying on a 3D orthogonal and\nregularly spaced grid with nearest neighbor in space matching nearest\nneighbor in the array. The function builds a scalar field assuming the\npoints are regularly spaced.

\n

Keyword arguments:

\n\n

Example (run in ipython --gui=qt, or in the mayavi2 interactive shell, see Running mlab scripts for more info):

\n
\ndef test_contour3d():\n    x, y, z = np.ogrid[-5:5:64j, -5:5:64j, -5:5:64j]\n\n    scalars = x * x * 0.5 + y * y + z * z * 2.0\n\n    obj = contour3d(scalars, contours=4, transparent=True)\n    return obj ```\n
\n
\n" }, { "Id": "10209", "CreationDate": "2019-06-08T18:59:01.807", "Body": "

I have a Monoprice Maker Select V2.1 (rebadged Wanhao Di3) with a microswiss all metal hot-end and machined lever and extruder plate. It had been printing very consistently for months with this set up - through 5 or 6 kg of filament - until a couple of weeks ago when it has started to under-extrude and then stop partway into a print, after about 30-40 minutes. It seems to clog and grind the filament, skipping steps. I first assumed this was heat-creep, and so disassembled the extruder, cleaned the heatsink and applied new thermal paste before reassembling, but to no luck. I also tried new fans on the cold-end but this didn't help either.

\n\n

Other things I have tried:

\n\n\n\n

I've now run out of ideas of what could be causing the issue and what to try. What other issues could cause the above symptoms or, if it is heat-creep, how else could I solve the issue?

\n", "Title": "Wanhao i3 stops printing after about 40 minutes", "Tags": "|wanhao|underextrusion|", "Answer": "

The nozzle was not seated properly

\n\n

Having failed to identify the problem with my current mods installed on the printer, I decided to remove everything and return the printer to factory condition to identify the problems. In particular this included replacing the all-metal hotend, and I took the opportunity to replace the insulation on the heater block too.

\n\n

On removing the insulation from around my heater block, I discovered a mass of burned-on plastic that had oozed out from around the thread of the nozzle and collected under the insulation, showing that it clearly was not screwed in tight enough. After thoroughly cleaning out the heater block, replacing the insulation and reinstalling the nozzle - and ensuring it was tightly seated against the tube - I have been able to print without issue.

\n" }, { "Id": "10214", "CreationDate": "2019-06-09T00:20:51.997", "Body": "

I noticed foot arches are already digitized, but custom arch supports are usually expensive. PLA and ABS aren't the best material for printing arch supports, especially if they replace the shoe's innersole.

\n\n

Is there a more flexible material for 3D printing that could be used for making custom orthotics?

\n", "Title": "Material for printing orthotics", "Tags": "|3d-design|print-material|material|", "Answer": "

Armadillo by Ninjatek is perfect with 100 % infill and 6 layers 6 walls and 6 the other thing? As a foot pharmacist I can say the cost is associated with the education as pedorthic modalities are different to each person. Ideally it would be easy to 3D print arch supports, but it would also be easy to \"hurt\" the person wearing them. buy a book on pedorthics before u wear them!

\n" }, { "Id": "10226", "CreationDate": "2019-06-11T05:04:22.853", "Body": "

In the process of updating my printer (Alunar M508), I decided to add \"Auto Bed Leveling\" using a BLTouch sensor. After doing a lot of Googling:

\n\n
    \n
  1. I found firmware that will upload and work with printer in default configuration (without BLTouch)
  2. \n
  3. Adding BLTouch configuration, the software compiles and updates the printer, I see all new options in menu and BLTouch on \"power on\" does a self test.
  4. \n
\n\n

I know I have configured the power to the device correctly 2 power sources and ground but where I'm having issues is in identifying Servo0 and ZMax/Min on my board. So far the black and white wire are connected to Z axis \"end stop\" but I cannot identify where to connect the orange signal wire to. I cannot find pinout anywhere of this board.\n\"LSEE

\n", "Title": "How to connect BLTouch sensor to Alunar M508 using LSEE 3D v1.0 board?", "Tags": "|prusa-i3|marlin|bed-leveling|bltouch|alunar-m508|", "Answer": "

The LSEE 3D is basically a RAMPS board (your linked source also shows that the used MOTHERBOARD is a RAMPS board: #define MOTHERBOARD BOARD_RAMPS_14_EFB), this implies that all pins of your board are the same as a RAMPS board. To connect the 3 pin header of the BLTouch sensor you need to connect the red wire to +5 V and the black wire to ground; the orange wire needs to be connected to an available PWM pin. As the LSEE board does not have many exposed (free) pins for you to use, you need to re-use one of the existing PWM pins that you do not use. An example is the pin nr. 2. From the pins_RAMPS.h file you see in the limit switches section:

\n\n
\n//\n// Limit Switches\n//\n#define X_MIN_PIN           3\n#ifndef X_MAX_PIN\n  #define X_MAX_PIN         2\n#endif
\n\n

It appears, from the image, that your board does have max limit end stop switches header pins available. What you could do is use the X_MAX_PIN for the BLTouch sensor.

\n\n

This implies that you need to assign the servo pin to pin nr. 2.

\n\n

From the servos section of the same pins_RAMPS.h file you see that the servos are either connected to pin 7 or pin 11 (depending on the board, your linked sources use the 1.4 version).

\n\n
\n//\n// Servos\n//\n#ifdef IS_RAMPS_13\n  #define SERVO0_PIN        7   // RAMPS_13 // Will conflict with BTN_EN2 on LCD_I2C_VIKI\n#else\n  #define SERVO0_PIN       11\n#endif
\n\n

Using the linked sources, the 11 should be replaced with a 2. This implies that you can connect the orange wire to the \"signal\" pin of the X_MAX end stop connector.

\n" }, { "Id": "10231", "CreationDate": "2019-06-11T13:39:43.343", "Body": "

I have a Monoprice Maker Select Plus, where I'm doing the Gulf Coast Robotics carriage plate upgrade. I was watching through an installation video just to find any "gotchas" I might want to watch for, and the guy in video recommends removing one of the carriage bearings, going from 4 to 3. The idea is to get quieter, smoother travels, with less weight on the belt.

\n

Has anyone else here done this for this model printer? Or for the very-similar Maker Select v2/2.1? Are there any downsides I should watch for? Everything I know seems to indicate 3 bearings are just better (the whole "3 points define a plane" thing), but if it's that simple, why go to the cost of shipping with 4?

\n

Update

\n

I did end up making the switch, and it has worked well. One thing I've noticed is markedly increased ringing/ghosting. However, it should also be noted I switched to a stronger (heavier) build plate at the same time, and that likely is also contributing.

\n", "Title": "3 vs 4 bearings for y axis travel", "Tags": "|y-axis|bearing|monoprice-maker-select-plus|", "Answer": "

Ideally you would use three bearings opposed to four. In principle you will only need 3 fixtures to get a stable reference frame. Look at how most Prusa i3 \"X\" carriages are constructed, they also have just three bearings.

\n\n

Three bearings are way more easy to line out (especially with tight tolerance bearings), with four bearings you will get binding much more easy than with three.

\n\n

I've been using three linear (self printed tight tolerance acetal/POM) bearings on the moving bed (Y direction) for quite some time on a Prusa i3 type of printer made from 2040 Aluminium profiles.

\n\n
\n\n

A similar reasoning applies to fixating the heated bed/glass slate, only three screws are necessary to define the reference plane. This is how my CoreXY heated bed is connected to the \"Z\" platform (a slate of glass is perfectly flat as a result of the production process).

\n" }, { "Id": "10238", "CreationDate": "2019-06-12T07:55:33.040", "Body": "

I am searching for a Linux software to control the 3000 mW laser engraver depicted below. It's a common model you'd find on AliExpress, Banggood, etc. under different brand names.

\n\n

I have already tried nejePrint, LaserWeb, and EzGraver, but they don't work. Any ideas?

\n\n

\"enter

\n", "Title": "Linux software for laser engraver", "Tags": "|software|laser|linux|", "Answer": "

The best is Kiri Moto.

\n

I use CNC and the stack idea is very comfortable.\nLaser and FDM, SLA are available as well.

\n" }, { "Id": "10244", "CreationDate": "2019-06-12T14:38:44.383", "Body": "

The Ultimaker Cura documentation says

\n\n
\n

Tip: If you use an unavailable third party nozzle size, set the line\n width to equal the nozzle size.

\n
\n\n

I sliced an object with a 0.3 mm line width. When the nozzle size is 0.4 mm, the results are almost the same as when the nozzle is 0.3 mm. The filament usage is slightly different and the gcode files have similar, but different, coordinates throughout.

\n\n

What does nozzle size really do besides suggest the line width?

\n", "Title": "What does nozzle size affect besides line width in, e.g., Ultimaker Cura?", "Tags": "|ultimaker-cura|nozzle|", "Answer": "

Searching the CuraEngine source, the only places I can find where nozzle size is used directly involve some arcane logic for merging of infill lines, such as:

\n\n

https://github.com/Ultimaker/CuraEngine/blob/05e93dabce9e863b8742fd69ed87717e1594e7a9/src/MergeInfillLines.cpp#L124

\n\n

So essentially, yes, nozzle size mainly serves as a default value for the line width settings.

\n\n

However, in general it's not always possible for line width different from nozzle size to be honored.

\n\n\n\n

Also, as noted by Tim Kuipers in comments, there are places outside the source, but rather in the json-based configuration tree, where nozzle size plays a role in the defaults and warning ranges for other parameters. Those are mostly line widths, but

\n\n
\n

The nozzle size can affect the values of the following settings besides line width in fdmprinter.def.json: Outer Wall Inset, Outer Wall Wipe Distance and Minimum Support XY Distance. Other than that it only influences the conditions under which setting-values give you a warning.

\n
\n" }, { "Id": "10250", "CreationDate": "2019-06-12T21:25:41.080", "Body": "

Recently on one of her videos a YouTuber stated that prints from large format printers are more brittle than if you were to print them in parts and glue them together. This seems to contradict the testimonials from the customers of a large format printer, who say that they get good prints from those printers (which print have a print area of a meter square).

\n\n

Would a print form a larger format printer be more brittle than a print made of smaller pieces super glued together?

\n\n

(with all other aspects being equal e.g. the nozzle, the temps, the material and the shape of the object).

\n\n

The YouTube didn't cite any source information to back up her claim.

\n", "Title": "Are large format prints more brittle?", "Tags": "|large-format|", "Answer": "

I'd recommend getting the object to fit together by design, rather than glue - though I tend (if the item is never to be disassembled) use Zap-a-gap - that stuff sticks like crazy though you must not squeeze the parts together but let it naturally sit.

\n" }, { "Id": "10272", "CreationDate": "2019-06-15T20:37:43.283", "Body": "

A fillet is like a rounded corner but on the inside of the corner.

\n\n

\"enter

\n\n

Does it make a difference (structurally) to use fillets on a 3d printed part?

\n", "Title": "Should I include fillets on my 3d printed parts?", "Tags": "|3d-models|", "Answer": "

Fillets in X-Y plane (i.e. between two vertical surfaces) work great for 3D prints and increase the strength a lot. They usually also improve the print quality, because the print head can keep a constant speed in the curve instead of slowing down to a sharp corner.

\n\n

However fillets that extend in Z direction (i.e. between a vertical and a horizontal surface) suffer from layer artifacts. Sometimes they can look worse than a sharp corner would. While they do still increase the strength significantly, they are not as strong as similarly sized fillets in X-Y plane are.

\n" }, { "Id": "10288", "CreationDate": "2019-06-17T13:50:30.493", "Body": "

We've been doing some printing with PETG filament on Ender 3 Pro printer and the result were awful:\n\"Overview

\n\n

Here are settings we used:

\n\n\n\n

There are a few types of problems that we had:

\n\n
    \n
  1. Initially filament did not stick to the bed - those 3 items in the middle of the picture are example of this issue. This got fixed by increasing temperature of bed to 80 °C.
  2. \n
  3. At some point a piece would get dis-attached from the bed and would move around together with the extruder around - two prints in the upper right corner of the picture were cancelled for this reason.
  4. \n
  5. Models are very rough, like a cheaply made snowball - that tiny model in the upper left is suppose to be a cattle-bell. Could you tell?
  6. \n
\n\n

Additional info

\n\n

Filament that we used indicated

\n\n\n\n

Question(s):

\n\n\n", "Title": "How to work with PETG? Settings, caveats, etc", "Tags": "|creality-ender-3|petg|", "Answer": "

I use a fine round brass brush, similar to what you would use to clean a gun barrel, to keep my nozzle clean. The brass is softer than the nozzle material, so it doesn't damage it, and the bristles grab strings easily - you barely have to touch them.

\n

I'm using SUNLU PETG filament in my Ender 3. It sticks VERY well to the "Creality Original Ultra Removable Magnetic 3D Printer Build Surface". I'm set at 70\u00a0\u00b0C on the bed. Get your height set well. I had a ton of issues with nothing sticking until I got it dialed in. Now, it almost sticks too well. I often have to remove the magnetic sheet in order to get larger prints off it.

\n" }, { "Id": "10298", "CreationDate": "2019-06-17T21:50:17.157", "Body": "

When printing objects higher than approximately 8-10 cm, sometimes nozzle hits the printed model and knocks over it. After 7-8 hours of printing that's really annoying. I'm using Creality Ender 3 Pro with Ultimaker Cura. How can I avoid this problem?

\n\n

As a note; it happens with thick, wide models without support structure too. I'm using Ender 3 Pro's stock magnetic bed.

\n\n

Here are some photos of printed model.

\n\n

\"Failed

\n\n

\"Failed

\n\n

\"Failed

\n\n

I use Esun PLA+, the part was on baseplate without any loss of contact. It was like one layer missed its coordinates and then all corrupted.

\n\n

I think it's not related with bed adhesion because for example for this model, it didn't knock over the model.

\n\n

There is no roof for the model, I think it doesn't need any support structure. Here you can see the expected finished one:

\n\n

\"Expected

\n", "Title": "How to avoid nozzle from hitting the model?", "Tags": "|ultimaker-cura|creality-ender-3|nozzle|", "Answer": "

It seems the problem was because of Z-axis leveling (level of the X-axis), I found out that the right side was more than 3-4 mm below the left side when the Z-axis height exceeds around 8-10 cm. Below 8-10 cm, the two sides were even. I calibrated the X-axis by turning the eccentric nuts of the wheels and tighten them. I will try printing soon with some test objects.

\n" }, { "Id": "10303", "CreationDate": "2019-06-18T17:52:56.797", "Body": "

I've recently bought myself a preassembled Prusa i3 MK3S printed and made my first projects. One of them was making a cup with my name on it. I want to use it to drink tea, water etc. I know, however, that I need to chose my filament wisely, as using the wrong one might be unsafe. I know that PLA for example is Polylactic acid which is a safe substance and occurs naturally in our body. Another thing is the dye, which can is a chemical substance I know nothing about. Do You recommend any specific type/model? Thanks.

\n", "Title": "Which 3D filament is 100% (or close to being so) food safe and non toxic?", "Tags": "|filament|3d-models|safety|food|", "Answer": "

I think this site will answer some questions about food safe 3D printing: 12 Vital Facts About Food Safe 3D Printing

\n\n

PLA is not a good choice for hot substances because it will deform at hot water temperatures which is no good for a cup (very dangerous!)

\n\n

I would recommend a filament like PETG, PETE, HDPE, and LDPE: What Plastics Are Approved for Food Contact Applications?

\n\n

Those types of plastic require very high printing temperatures and an all metal hot end. You don't want PTFE in your hot end when printing at those temperatures (265 \u00b0C) as it will emit toxic gasses.

\n\n

I've seen food safe filaments for sale so it might be best to search Google for one of them to use.

\n\n

Please keep in mind that printing anything will cause voids and gaps that will allow bacteria to grow. Personally I don't think I would want to risk it for daily use items but a coffee/tea cup might be alright. This is covered in the first link.

\n" }, { "Id": "10310", "CreationDate": "2019-06-19T14:41:55.013", "Body": "

In regards to a part that I'm having printed remotely (by two processes:- stereolithography and laser sintering), I've been advised by the 3d printing company that 'the triangulation of the file is rather rough'.

\n\n

In this particular instance, it probably doesn't matter, but for the future, are there any tips to improving 'triangulation' when generating forms in AutoCAD?

\n\n

Note, AutoCAD's FACETRES variable is set to 10.

\n", "Title": "Improving triangulation on AutoCAD-generated stl files", "Tags": "|print-quality|autodesk|", "Answer": "

This thread and this article, both on Autodesks website, suggest altering the FACETRES system variable to 10 to achieve a higher-quality .stl export.

\n\n

Alternatively, you can use the 3DPRINT command for FACETRES to be increased automatically, which will then offer an .stl export or to send the file directly to a 3D printing service.

\n" }, { "Id": "10317", "CreationDate": "2019-06-20T03:58:03.767", "Body": "

I am printing a small cylinder, but when the object is finished, it's smaller than the measures I used when create the model.

\n\n

I used thincerkad to make a simple model, the measures are:

\n\n\n\n

After the print was done, the actual dimensions were:

\n\n\n\n

Pictures

\n\n

First attempt

\n\n

\"one\"

\n\n

The smaller object that's in the drawn circle was the first one printed, the dimensions I used were:

\n\n\n\n

Then I printed it again, and the result was:

\n\n

\"Two\"

\n", "Title": "Why is the printed object smaller than original model?", "Tags": "|3d-models|", "Answer": "

Let's do the math, you printed something of size 9 cm and got a size of 7 cm. This implies that the scale equals $\\frac{7}{9}=0.778$. In order to print it at the correct size, you should have printed the object at scale $\\frac{1}{0.778}=1.286$; so 28.6 % bigger, i.e. $1.286\\times9=11.6$ cm. You printed at 11 cm, so the print should become smaller than the pencil drawn circle on the paper. This is actually what you see in the image you supplied.

\n\n

This can imply 2 things, you either scale the prints incorrectly to export to stl (but that is unlikely because the Z height is correct), or the steps per mm are incorrectly set in the firmware of your printer. The rotation of the steppers (usually 200 steps) need to be translated into linear movement; this depends on the used pulleys mounted on the steppers (typically used pulleys are: 16 or 20 teeth for belt driven X and Y axes).

\n\n

Calibrating the steps per mm of the extruder is answered in this answer. For the X and Y axis this works the same. If you have a Marlin based printer firmware, send G-code M503 to the printer over a terminal interface as e.g. OctoPrint, Pronterface (as part from Printrun: 3D printing host suite), Repetier-Host have, you can obtain the current values from the reply; these are listed under M92.

\n\n

That value for X and Y needs to be multiplied by 1.286 (as an example) to get the correct dimensions. You do this by sending G-code M92 like M92 X100.00 Y100.00 (see this answer that explains which values you should use based on pulleys you use, either 80 or 100) to the printer, to keep these values they need to be stored in memory using G-code M500 (note that the values 100.00 should be replaced by the values you get by multiplying the return values for X and Y from M503 by the 1.286 multiplication factor, only if the error is systematically increasing with print dimensions, otherwise stick to the calculated values from e.g. the Prusa belt calculator).

\n\n

Without the proper steps per mm, you will not be able to use the full potential of the bed. An alternative as scaling your prints by the appropriate scaling factor will only help if your scaled print is smaller than the bed size divided by that scaling factor, so no use of the full bed. Rather fix the firmware to fit the actual mechanical layout.

\n" }, { "Id": "10323", "CreationDate": "2019-06-20T19:33:42.107", "Body": "

Is there a good rule of thumb for small type on 3d printed pieces?

\n\n

Minimum type size?\nGood typeface for accurate reproduction?

\n", "Title": "Rule of thumb for small type", "Tags": "|print-quality|3d-models|3d-design|graphics|", "Answer": "

I've had better luck with fonts that are heavier, usually sans-serif, and usually bold-face. All-caps can help, too, if it makes sense at all for the text. Impact is one widely-available example, though it's far from perfect and rarely has the look I want. I also usually need to turn on the \"Print Thin Walls\" setting in Cura when handling smaller text.

\n\n

When looking at how small you can get, we'll start with font sizes. It would be easy to get lost here in a discussion of points and measurements. The thing is, font sizes describe the vertical height of the characters. For 3D printing, I believe you'll do better paying more attention to the horizontal width of your text. Most characters are taller than they are wide, so if you can produce legible horizontal features, you can probably handle the vertical features, too.

\n\n

I'll use the letter \"H\" as an example here, because it shows the full size of the box for a typical character. Specifically, since I'm talking about horizontal features if you look at the bottom of the H, it has a three sections: leg, then gap, then leg. Also notice the gap is about 3 times the size of the legs (you can see this better if you zoom in close). This varies by font, but 3:1 is good average ratio. That gives us 5 units of width for the character itself. Additionally, you want to allow some spacing between individual characters; not every character needs it next to every other character, but I find it useful to allocate a 6th unit here.

\n\n

Now consider those 6 units in the context of your nozzle size. With a typical .4 mm nozzle, that means the smallest size character you can legibly produce is about 2.4 mm across. Of course, most fonts are not monospace, where a character might be larger or smaller, but I believe this makes a useful average. Count the number of letters in a line of text you want to print, multiply by 2.4 mm, and that's the minimum amount of horizontal space you need.

\n\n

If you really want to push things, a font specifically designed for 3D printing should theoretically be able to work in terms of 3 nozzle units wide + an extra gap between certain letters. But this is all theory, and for the minimum of what's possible. When you also start to think about what actually looks good, especially if you want to show features like serifs, the real world can really mess this up. In practice, I've found I need to go significantly larger even than the 6 unit / 2.4 mm option... but maybe I've just used the wrong font. You can always try a test print of your text in a small rectangle, to make sure it will be legible before using in a larger object.

\n" }, { "Id": "10331", "CreationDate": "2019-06-21T10:59:30.733", "Body": "

I'm not talking about making something that's outright disproportionate of course. I've been working in Blender and I've use Absolute grid Snap to snap my vertices to the grid. The problem is that it (didn't seem) to always work perfectly for centimeters, and seemed to work better for meters. \n(edit: I've learned what the problem was and it was simply the placement of the vertices in side view, being at slightly different elevations. I'm going to emphasize that the difference was very slight. It was just enough to show up in the measurements. When I switched from front view to side view I was able to adjust the elevation to the grid and that fixed the problem.)

\n", "Title": "How precise do models need to be for 3D printing?", "Tags": "|print-quality|3d-models|", "Answer": "

After some trouble shooting I realized what was wrong, and now it seems more like a non issue if anything. There's actually nothing wrong with the grid, and the vertices were snapped to it appropriately in top view. The problem was that that my vertices were not at the same elevation (in side view). The two seemed very close to being snapped appropriately on the grid in side view which is why it wasn't immediately obvious.

\n" }, { "Id": "10345", "CreationDate": "2019-06-22T14:14:05.373", "Body": "

I just installed my (v3) BLTouch with a vanilla version of Marlin (1.1.9) on my Ender 3 and everything seems to work fine (including auto home) except the probing. It repeatedly does the first three probes (successfully) but then tries the fourth and fails. Even when disabling the axis movement (X/Y stepper motors) the issue persists, which would indicate that it is not a bad connection or physical location dependent. Any ideas on how to troubleshoot?

\n

On second thoughts, it seems that it now fails inconsistently, so it is probably a wiring issue, although there is still the issue of it not retracting before printing and not leveling well when it does (although those issues would be more appropriate for a separate question), among other things.

\n

<Edit: Resolved, I both replaced the BLTouch and updated my firmware, but I believe it was the latter that fixed it.>

\n", "Title": "BLTouch Probing Fails Intermittently", "Tags": "|marlin|bed-leveling|bltouch|", "Answer": "

Reviving old thread...

\n

I have same problem on Ender-3 V2 and two BLTouch sensors. So far, Oscar's idea of screwing in the T10 Torx screw on the BLTouch seems to be working. When the torx screw is screwed in you can see the tip of the probe being pushed out a bit. Evidently, there is a magnet in the BLTouch and there is some metal part on the probe that is attracted to the magnet. When the probe gets close enough to the magnet it snaps to the upper (stowed) position. The Torx screw adjusts the position of the magnet. By screwing the Torx in the magnet is closer to the probe and the probe snaps to the magnet sooner.

\n

The BLTouch firmware can detect when the probe has been been snapped up by the magnet. Apparently, the firmware will only probe downward a limited amount. If the probe is not snapped to the magnet by the end of that travel the firmware will abort the probe. My guess is that from the factory the BLTouch is adjusted at the ragged edge of the firmware's detection travel. By screwing it in a bit it the probe stays within the firmware's travel limit.

\n

Since I haven't disassembled either of my BLTouchs this is mostly speculative. Caveat emptor.

\n

BTW, my printer is connected to Octoprint on Android tablet. Disconnecting the Octoprint and USB cable didn't help.

\n" }, { "Id": "10346", "CreationDate": "2019-06-22T17:32:17.743", "Body": "

Note: The \"TL smoothers\" referred to in the question title are an arrangement of 8 diodes that presumably both provide a voltage drop to address the minimum current output issue of some stepper drivers described in How Accurate Is Microstepping Really, and provide flyback protection and protection of the driver from currents induced by moving the motor in the unpowered state.

\n\n

Now, on to the question:

\n\n

I put together a simple test part to test Y (or X) axis microstepping accuracy:

\n\n

\"Test

\n\n

Every 1mm in the Z direction, the face steps inward 1/80 mm (corresponding to my printer's [micro]steps per mm setting) in the Y direction and 1 mm in the X direction (to clearly show where the steps happen).

\n\n

Here are the results (two runs, near-identical output):

\n\n

\"Photo\n\"Photo

\n\n

Several of the steps are missing entirely, and it's the middle vertical surface, rather than the side two stepped-vertical surfaces, that shows a step-like texture.

\n\n

Note that layers have been printed counter-clockwise, but the design with steps on both sides is to try to reduce the impact of print orientation so as not to depend on what the slicer decides to do. Print speed was 30 mm/s for outer walls, with outer walls set to be printed first so they're not affected by already-printed inner ones.

\n\n

This looks to me like an indication that microstepping is not working accurately, and seems to agree partly with this article: https://hackaday.com/2016/08/29/how-accurate-is-microstepping-really/

\n\n

Would TL smoothers improve this? My printer is an Ender 3, seemingly the latest revision at the time it was sold (December 2018) based on the bed and other aspects. Some threads I found suggested that this was a problem with older Ender 3s that was fixed in later revisions and that TL smoothers won't help, but it looks to me like it's not fixed.

\n\n

Update: With settings adjusted for 0 jerk and 50 mm/s\u00b2 acceleration in the outer walls, I got a somewhat improved result:

\n\n

\"New

\n\n

All steps are clearly visible, but at the reduced and more varying speed, artifacts of the extruder motor accuracy (or maybe just bowden latency) are much worse, and at the steps the \"overshoot and bounce back\" effect is still happening. Is this consistent with a situation where the TL smoothers I asked about could benefit?

\n\n

Update 2: I've further tested with leaving acceleration at default (500 mm/s\u00b2) and just disabling \"jerk\", i.e. \"instantaneous\" change in speed. The results are at least as good as the above with slow acceleration. What strikes me most is that the \"overshoot then bounce back\" happens outward from the model on both edges, rather than inward on the righthand side where the (CCW moving) nozzle is turning inward. Could this indicate that there's no microstepping accuracy problem at all, and that the artifact is purely from excess extrusion as the corner is turned?

\n\n

Updated pic (new one below):

\n\n

\"Old

\n", "Title": "Does this microstepping test result indicate need for TL smoothers?", "Tags": "|print-quality|creality-ender-3|microstepping|", "Answer": "

The missing/inaccurate microsteps seem to be a bug/feature of Marlin:

\n\n
\n
// Moves (or segments) with fewer steps than this will be joined with the next move  \n#define MIN_STEPS_PER_SEGMENT 6\n
\n
\n\n

I'm not sure what the logic for this was in the stock firmware, which I originally performed the tests with, but with Marlin 2.0 I was completely missing the first 5 steps on the right (positive X) side. Changing the value to 1 mostly fixed the problem, but there was also a corresponding bug/feature in Cura:

\n\n
\n
            \"meshfix_maximum_deviation\":\n            {\n                \"label\": \"Maximum Deviation\",\n                \"description\": \"The maximum deviation allowed when reducing the resolution for the Maximum Resolution setting. If you increase this, the print will be less accurate, but the g-code will be smaller. Maximum Deviation is a limit for Maximum Resolution, so if the two conflict the Maximum Deviation will always be held true.\",\n                \"type\": \"float\",\n                \"unit\": \"mm\",\n                \"default_value\": 0.05,\n                \"minimum_value\": \"0.001\",\n                \"minimum_value_warning\": \"0.01\",\n                \"maximum_value_warning\": \"0.3\",\n                \"settable_per_mesh\": true\n            }\n
\n
\n\n

The deviation should be set less than a single microstep, so 0.0125 mm.

\n\n

With both of these fixed, I've got a perfect print:

\n\n

\"successful

\n\n

Sliced printing direction was counter-clockwise for all layers. The steps on the left side are where the slicer decided to put the layer seams (perhaps I should retry forcing it to put the seam somewhere else, or even using vase mode) so they have a little extra bulging from slight ooze during the Z move that you can clearly see from the lighting, but the steps on the right side are dimensionally near perfect.

\n\n

I think I can safely say any past problems were the fault of firmware and slicing software, not anything physical/electrical/electronic.

\n" }, { "Id": "10347", "CreationDate": "2019-06-22T19:50:22.753", "Body": "

I am using a new Prusa i3 MK3S 3D printer kit. I print lots of things using PLA and PETG.

\n\n

After a week of great performance I noticed that when printing some objects with PETG filament I often encountered a problem when there's intense stringing, infill gaps, artifacts, the object sometimes detaches from the plate. I use the Prusa Slicer and Cura and print with the temperature 230/90 \u00b0C, speed max. 300 %.This problem occurs rather regardless of which infill methods I choose. I see this problem more often when printing models sliced with Cura. Sometimes everything goes fine, but most of the time I need to stop the print due to the model collapsing, detaching from the surface, its parts collapsing due to infill gaps. The printer sometimes makes some 'clicking, cracking' sounds when printing with PETG. My filaments come from Fiberology.

\n\n

Surprisingly, I have run the selftest, XYZ, Z calibrations and all the other ones without any errors. The wizard told me that the axes are perpendicular and it gave me congratulations. All the other tests went nearly perfect as well.

\n\n

I do not encounter any problems using PLA, just with PETG. I try to maintain the filament properly (keep it away from moisture, in a closed box). Sometimes (rarely) I get crashes. The filament often builds up on the hotend and I remove it.

\n\n

I do not know what to do, the build went very well and there are no errors, even though I am a new user and this is my very first 3D printer. I have searched the web and I haven't found people reporting this exact same thing.

\n\n

Could You help me? What can I do to improve the quality of the prints, perhaps maintain PETG better (maybe I'm doing something wrong) and most importantly, solve the problem?

\n", "Title": "Prusa i3 MK3S 3D printer PETG printing problems", "Tags": "|print-quality|prusa-i3|3d-models|petg|", "Answer": "

You might try printing with Prusa's recommended settings. They tend to be hotter and slower than I expected.

\n\n

I have made several pet-G prints with the same machine you have.

\n" }, { "Id": "10349", "CreationDate": "2019-06-22T22:06:22.017", "Body": "

I've always been wondering about the actual accuracy of 3D printing devices. When looking for the perfect machine to buy, I looked at the speed, price, filaments supported etc, but also accuracy. I once asked somebody who could give me some advice on what to look at.

\n\n

One of the things I was told about was that many printers don't necessarily have that crazy precision of 0.05 mm (50 micron). Another person told me something different - he said most of those printers actually were capable of putting out 50 micron layer height. How is it really?

\n\n

Another thing is that the official slicers for those machines also claim that this precision is real, for instance the PrusaSlicer v2.0.

\n\n

There are many high-end, very expensive machines and even they sometimes claim their resolution is worse than 50 microns.

\n", "Title": "Do 3D printers really reach 50 micron (0.05 mm) accuracy?", "Tags": "|3d-models|3d-design|fdm|", "Answer": "

A resolution (sometimes called \"accuracy\" for marketing purposes) of 0.05 mm means that if you produce a bunch of 10 mm dice and a bunch of 10.05 mm dice, then the 10.05 mm ones will be statistically larger. Note that dice don't have to actually be anywhere near 10 mm, nor does a random die from the 10.05 mm pile have to be larger than a random die from the 10 mm pile.

\n\n

A repeatability (aka \"repeat accuracy\") of 0.05 mm means that in the experiment above every die from the 10.05 mm pile have to be larger than every die from the 10 mm pile. Note that your dice still don't have to be anywhere near actual 10 mm.

\n\n

A precision (aka trueness) of 0.05 mm means that in the experiment above the average size of a 10 mm die should be within 9.975..10.025 mm. Note that individual dice don't have to be within that interval.

\n\n

Finally, the general accuracy (as defined in ISO 5725) of 0.05 mm means that every 10 mm die should be within 9.975..10.025 mm.

\n\n

To sum it up, the statement from your question is true for the \"commercial accuracy\", but not for the general definition of accuracy. For example here is an article comparing accuracy of 3D printers in dental applications (so we're talking high-end machines), with average accuracy ranging from 0,05 to 0,1 mm and absolute accuracy in range of 0,11 to 0,17 mm.

\n" }, { "Id": "10360", "CreationDate": "2019-06-24T04:56:35.640", "Body": "

To minimize retractions and travel when printing several objects, some slicers produce gcode for sequential deposition when each object is built in a traditional manner bottom up layer by layer before starting a new object. Of course, there are limitations caused by the hotend geometry \"from :

\n\n

So bulky hotends don't allow for this trick to be used wen sequential pieces are closer than several centimeters. If there where hotends with minimal end effector geometry (needle like, much like drill bit on CNC) we might even do things like spiralizing contours of a single object. \nThis has also potential for other tricks like producing interlacing layers for better layer bonding etc. I wasn't able to find any ongoing work on minimal geometry hotends. Any links? And what might be the challenges in making one? heatblocks and heating cartridges are out of the question but nichrome wire and a fast PID heat controller might do the job.

\n", "Title": "Needle Hotend for nonplanar printing", "Tags": "|hotend|fdm|", "Answer": "

Yes, using a needle to extrude will result in a smaller diameter extrusion. All that is needed is to find a one with a suitable bore size. The problem however is in generating enough hydraulic pressure, needed to extrude hot plastic through a nozzle of that size. If you can form an extruder that can, then go for it. The second problem is that the thin wall of such a nozzle won't have the same \"ironing\" effect that current nozzles have. From the picture referenced below you can see the nozzle hole and then a ring of metal around it. That ring flattens the filament out into the desired layer which helps with layer adhesion. With out it, the filament might not even stick to the previous layer, or the bed.

\n\n

\"E3D

\n" }, { "Id": "10369", "CreationDate": "2019-06-24T18:58:46.250", "Body": "

In 3D printing firmware and slicers, jerk settings are expressed in units if mm/s. This is contrary the physical definition of jerk, which is in units of mm/s\u00b3, being the second derivative of speed with respect to time (or the third derivative of position). What is the reason for this discrepancy and how does one interpret jerk in this contect?

\n", "Title": "Why does jerk have units of mm/s rather than mm/s\u00b3?", "Tags": "|firmware|", "Answer": "

In step based motion control, the time between two steps is calculated directly from velocity. If that time is not constant then you are accelerating or decelerating based on a specified acceleration. The next time between steps is calculated from the current velocity based on the desired acceleration. However when moving 2 or 3 axes at once, this can result in very poor and slow performance when moving through complex curves composed of many small moves, because by the math one axis always needs to slow down too much if no jerk is involved. When doing motion calculations for step based systems, actual jerk m/s\u00b3 directly translates into how much velocity 'error' is acceptable in calculation of the next velocity (m/s) to allow turning lo angle corners more quickly, but without missing steps or stalling motors. This velocity error comes directly from the actual jerk between two steps and it does have physical meaning, (and the proper units). Also, low power microprocessors can do the math fast enough, which is not the case if square and cube roots get involved.

\n" }, { "Id": "10371", "CreationDate": "2019-06-24T19:23:53.813", "Body": "

I've recently purchased a Makerbot Replicator Dual clone made by CTC. I'm in the process of upgrading/adding a few parts to it, but noticed that I can't control or print via USB.

\n

The machine prints perfectly from an SD card and I can see information in the terminal from the printer via USB in RepG and through OctoPrint - Such as M105 - but can't send any .x3g files to print or upgrade firmware (I wanted to flash Sailfish 7.7 eventually).

\n

To clarify;

\n

In Octoprint

\n\n

Terminal Output from OctoPrint at connection:

\n
Changing monitoring state from "Offline" to "Opening serial port"\nConnected to: <octoprint_GPX.gpxprinter.GpxPrinter instance at 0x6c9a02d8>, starting monitor\nStarting baud rate detection...\nChanging monitoring state from "Opening serial port" to "Detecting baudrate"\nTrying baudrate: 115200\nRecv: start\nSend: N0 M110 N0*125\nChanging monitoring state from "Detecting baudrate" to "Operational"\nRecv:  Makerbot v7.4\nSend: N0 M110 N0*125\nRecv: echo: gcode to x3g translation by GPX\nRecv: SD card ok\nRecv:  T:27 /0 B:21 /0 T0:27 /0 T1:26 /0 @:0 B@:0\nRecv:  T:27 /0 B:20 /0 T0:27 /0 T1:26 /0 @:0 B@:0\nRecv:  T:27 /0 B:21 /0 T0:27 /0 T1:26 /0 @:0 B@:0\nRecv: ok\nSend: N1 M115*39\nRecv: ok PROTOCOL_VERSION:0.1 FIRMWARE_NAME:Makerbot FIRMWARE_VERSION:7.4 FIRMWARE_URL:https://support.makerbot.com/learn/earlier-products/replicator-original/updating-firmware-for-the-makerbot-replicator-via-replicatorg_13302 MACHINE_TYPE:r1d EXTRUDER_COUNT:2\nSend: M21\nRecv: ok\nRecv: SD card ok\nSend: M20\nRecv: ok\nRecv: Begin file list\nRecv: 2GB\nRecv: System Volume Information\nRecv: mesh_bed.stl\nRecv: xyzCalibration_cube.x3g\nRecv: CTCB_3DBenchy.x3g\nRecv: 3DBenchy.x3g\nRecv: ActiveCoolingDuct.x3g\nRecv: CTCB_ActiveDuctD4_UN.x3g\nRecv: UK_TROLLEY_TOKEN.x3g\nRecv: mesh_bed.x3g\nRecv: z-axis-support.x3g\nRecv: bed-screws.x3g\nRecv: spool_nut.x3g\nRecv: 2016_spool.x3g\nRecv: 2016_spool_no_raft.x3g\nRecv: ActiveDuctD4_UN.x3g\nRecv: Z_Axis_Support_Ends.x3g\nRecv: End file list\nSend: M105\nRecv: ok T:27 /0 B:20 /0 T0:27 /0 T1:26 /0 @:0 B@:0\nSend: M105\n
\n

In ReplicatorG

\n\n

Is my Mightyboard just a dud, or is there something I can do to try and fix it?

\n

The reason I want to try and solve this now, is that I'm planning on adding active cooling and LED lighting control so don't really want to do all that just to find out that I need to replace the board soon.

\n
\n

Additional info

\n

Some information meaning that the current firmware and board is reported, as well as current temperatures of the extruders and heat bed. I can print .x3g files from the SD card, but I can't send G-code commands or .x3g files through USB.

\n

I have just tried a few G-code commands through OctoPrint terminal with mixed results. M105 works, M140 & M106 don't.

\n", "Title": "Can't control printer via USB", "Tags": "|firmware|makerbot|usb|mightyboard|x3g|", "Answer": "

You can find gcode information at the RepRap wiki. The obvious other thing to try is G1 X10 for a move.

\n\n

Octoprint prints by sending the individual file's gcode one line at a time over USB. It appears that the firmware on your printer might not respond to any 'action' commands over gcode, rather than any fault as such with your hardware/software setup.

\n\n

Although your board reports a specific firmware version, it is impossible to know if that firmware was corrupted, or patched by the vendor (and no change reflected in the revision print). Thus, your best option to regain full control might be to flash your own firmware using low-level access. I'm guessing that will require an ICSP lead.

\n\n

The fact that USB works, and the printer works, tends to suggest that the board is properly functional.

\n" }, { "Id": "10375", "CreationDate": "2019-06-25T07:49:58.890", "Body": "

ABS is a very strong material, but it also has some downsides. One of them, which is the necessity of having a printer with enclosure, completely discouraged me from using it, as it would be a waste of money. This is quite sad, because I cannot make prints that will be able to withstand a large load of tension without breaking.

\n\n

Is there any way to print ABS without any enclosure? Maybe there are several types of this material and some are easier to print?

\n", "Title": "Is there a way to print with ABS without enclosure?", "Tags": "|prusa-i3|3d-models|abs|enclosure|", "Answer": "

I print ABS in my basement in an unenclosed Prusa3D i3m3s, just as I print other filaments. Perhaps an enclosure would be helpful, but I don't have problems as it is. Your experience may vary.

\n\n

Before that I printed ABS on a large home-brew delta machine. No problems related to lack of enclosure.

\n\n

Before that I printed ABS on a Thing-o-Matic, also with no problems.

\n\n

I'm not saying that every print was perfect, but in the first two (chronologically) cases, the enclosure was not the largest contributor to print artifacts.

\n" }, { "Id": "10384", "CreationDate": "2019-06-25T18:53:03.010", "Body": "

My anet A8 stepper drivers get very hot after some time printing, so I decided to install a 5V fan to cool them down. I had the idea to get a 12V to 5V regulator to connect a 5V fan, but then i found this image:

\n\n

\"anet

\n\n

(source: lokspace.eu)

\n\n

It looks like the Anet A8 has an ICSP and Serial header that can deliver 5 or 3.3 V directly from the board. Is this correct? If it is, how many amps can i get from this pin? Can I connect a 5V fan directly here?

\n\n

Thanks and sorry for my bad English.

\n", "Title": "Getting 5V directly from the Anet A8 mainboard", "Tags": "|anet-a8|", "Answer": "

You would be better to use a 12V fan.

\n\n

The 5V is for powering logic. It should not have motor loads, even fan motor loads, applied to it. You will not be saving power over using a 12V fan. According to @Tom 's answer, the 5V is derived through a linear regulator.

\n\n

A linear regulator has the property that the current drawn from the regulator at 5V will equal the current drawn by the regulator at 12V. $7/12$'ths of the power will be lost in the regulator chip as heat, which may cause the chip to overheat if there is not enough cooling margin in the thermal design.

\n\n

If you use a 12V fan the current consumption of the fan for the same cooling capacity will be lower, and you won't waste more energy dropping the voltage.

\n\n

May I ask how hot the steppers are? Too hot to comfortably hold for more than a few seconds may still be completely acceptable for motors. So hot that the plastic holding them melts is too hot for the mounting, but may still be ok for the motor. So hot that they cause burns with 5 second contact is probably too hot.

\n\n

If the motors are too hot, it might be better to check the current the motor drivers are programmed to deliver (either through software configuration or a potentiometer -- I don't know the printer). Getting the current right is a better fix that managing the excess heat. It may also improve the linearity of microstepping and improve print quality.

\n" }, { "Id": "10390", "CreationDate": "2019-06-26T11:47:10.083", "Body": "

I've done calibration test with \"Concentric circle test\" (https://www.thingiverse.com/thing:11895) and at specific points there are little bumped points on the print. Also Thingiverse page of the test mentions about these.

\n\n

How can I solve this problem?

\n\n

My printer is Creality Ender 3 Pro, I use Esun PLA+ with 210 celcius extruder and 60 celcius bed temperature.

\n\n

Here is the printed object, both are same print, just took photo on different base.

\n\n

\"enter

\n\n

\"enter

\n", "Title": "How to solve spillage problem?", "Tags": "|ultimaker-cura|creality-ender-3|calibration|nozzle|", "Answer": "

I've not sliced the part in question, but the over extrusion mentioned in the third point (at change of direction/layers) is referring to ooze.

\n\n

This is a result of the hydraulic pressure in the melt zone, and results in over extrusion at any point when the print head is moving slowly (and a corresponding under-extrusion once motion resumes). It can be compensated for by retraction and coasting settings.

\n\n

This effect will probably not be visible in the print you have here, there are other effects which are causing more significant defects.

\n" }, { "Id": "10410", "CreationDate": "2019-06-27T23:05:15.583", "Body": "

How should I describe this part which looks like a small gear so that I can research replacements?

\n\n

\"image

\n\n

This came with my FLSUN 3D printer, which may be based on a Prusa design.

\n", "Title": "What part is this? Circular threading spool gear, about 10 mm diameter", "Tags": "|replacement-parts|part-identification|", "Answer": "

This is a timing belt pulley.

\n\n

Specifically, is a GT2 timing pulley, 2 mm pitch (between teeth), 6 mm wide. The drive diameter is measured by the number of teeth (16 in this case) , the shaft diameter (bore) is measured in mm.

\n\n

The 'GT2' part refers to the tooth profile, some other examples are shown half way down this page.

\n" }, { "Id": "10420", "CreationDate": "2019-06-28T17:00:23.707", "Body": "

I burnt the Z- port for the Z endstop. Can I use the Z+ port? I'm using Jim Brown's Marlin firmware.

\n\n

Any help to change that is highly appreciated. I know nothing about code. Sorry for my ignorance. I'm new on this 3D printing universe.

\n", "Title": "Tevo tarantula Z- port burnt", "Tags": "|z-axis|endstop|tevo-tarantula|", "Answer": "

Assuming line 585 of Configuration.h looks like this

\n\n
// Almost all printers will be using one per axis. Probes will use one or more of the\n// extra connectors. Leave undefined any used for non-endstop and non-probe purposes.\n#define USE_XMIN_PLUG\n#define USE_YMIN_PLUG\n#define USE_ZMIN_PLUG\n//#define USE_XMAX_PLUG\n//#define USE_YMAX_PLUG\n//#define USE_ZMAX_PLUG\n
\n\n

That is to say, if the Maximum endstop ports/plugs aren't currently being used, then you could just redefine the pin used by ZMIN (Z-) to be that currently being used by the ZMAX (Z+), in the appropriate pins_XXXX.h file (contained within MarlinTarantula/Marlin/src/pins/), where XXXX depends upon which board you have (I don't know what board your printer has).

\n\n

For example, if you have a RAMPS board then in pins_RAMPS.h at line 79 you would change

\n\n
//\n// Limit Switches\n//\n#define X_MIN_PIN           3\n#ifndef X_MAX_PIN\n  #define X_MAX_PIN         2\n#endif\n#define Y_MIN_PIN          14\n#define Y_MAX_PIN          15\n#define Z_MIN_PIN          18\n#define Z_MAX_PIN          19\n
\n\n

to become (note the change in the last two lines)

\n\n
//\n// Limit Switches\n//\n#define X_MIN_PIN           3\n#ifndef X_MAX_PIN\n  #define X_MAX_PIN         2\n#endif\n#define Y_MIN_PIN          14\n#define Y_MAX_PIN          15\n#define Z_MIN_PIN          19\n#define Z_MAX_PIN          18\n
\n\n

Then compile and reflash the firmware.

\n\n

Obviously you would then plug the Z_MIN (Z-) wire from the endstop into the Z_MAX (Z+) port.

\n" }, { "Id": "10426", "CreationDate": "2019-06-29T19:30:49.533", "Body": "

I have issues with this overhang:

\n\n

\"Damaged

\n\n

The whole part always breaks in this overhang during the print.

\n\n

I tried to increase the count of wall lines and decrease printing speed, but none of those things help. Do you have any idea how to fix it? Is possible to create support inside a model to print this overhang?

\n\n

(I\u00b4m using Cura 4.1.0.)

\n\n

I\u00b4m using 3 wall\u00b4s lines and 15% gyroid infill. My bed has 70 \u00b0C and nozzle 205 \u00b0C. I am using PLA Prusament galaxy black. The part is oriented the right way. By overhang I mean damaged part above holes.

\n", "Title": "Damaged overhang", "Tags": "|ultimaker-cura|pla|stability|", "Answer": "

When you get upwards facing things failing it can be because the slicer has too few top layers which makes angled faces have gaps. While you may think it is a wall the slicer sees it as top layer. Try increasing the number of top layers and check in the preview how it appears.

\n\n

Infill will normally support those faces so no separate support is needed. Just make sure you have enough top layers that no gaps form, 3-5 layers should be alright.

\n" }, { "Id": "10427", "CreationDate": "2019-06-29T20:29:18.267", "Body": "

I've seen conflicting advice on the correct printing temperature for Microcenter's house-brand Inland PLA+, particularly after a supplier change announced in April 2018.

\n\n

There's an official post from Microcenter, where they state that their PLA+ from both the suppliers they use prints best from 215-225 °C. (The label on the spool specifies 205-225 °C).

\n\n

However, I've run a series of Benchy prints with recently-purchased white inland PLA+ at different temperatures (with a Lulzbot SL toolhead and 0.1 mm layer depth, slicing by Cura LE 3.6.10 after repairing the STL with Microsoft's repair tool, movement 30 mm/sec), and my experience is very different:

\n\n\n\n

Is something wrong with my equipment, such that it's printing with a higher temperature than it reads? Is Microcenter's advice off? Could I have a batch that behaves differently than is expected for the same filament in general?

\n\n

More to the point -- what advice do others have to get good results with Inland PLA+?

\n", "Title": "Inland PLA+ stringing badly at manufacturer-recommended temperature ranges", "Tags": "|filament|pla+|", "Answer": "

Even as PLA+ contains some additives, each printer is different. We usually never know what the actual temperature of the printhead is, but if your printer prints good at 205\u00a0\u00b0C, despite the manufacturer claiming you should use a little more temperature, use it. It might be the perfect combination of temperature and speed for your printer. Your printer is not the benchmark machine the manufacturer of the filament used, and we have no idea what speeds they used if they had an enclosure and what style of hotend they used.

\n" }, { "Id": "10432", "CreationDate": "2019-06-30T10:04:45.887", "Body": "

I just set up a refurbished MP Select Mini V2 and tried to print the test file included by the manufacturer, cat.gcode, from the included SD card. I printed in PLA (I think; the unlabeled sample included with the printer) at the default extruder temperature, 190 \u00b0C. The print bed was set to 50 \u00b0C.

\n\n

It appeared to print the raft fine, and then a couple of layers of the cat. At this point I walked away, and when I came back a few minutes later here's what I found:

\n\n

\"Printer

\n\n

The raft is still stuck to the bed, but the layers of cat that were printed fell off the base and onto the floor. Here's what I picked up off the floor, (next to the detached raft):

\n\n

\"Raft

\n\n

The raft was stuck on the bed just fine, so I don't think it's an adhesion issue. The bed and the desk the printer is sitting on are both very close to level (things don't slide or roll off). There's no breeze and I'm certain nothing bumped or touched the printer while it was running.

\n\n

This is my first attempt at a print so I obviously have no idea what I'm doing. I'm planning to set up a camera to record the next attempt. What other troubleshooting can I do? What is the likely cause of this problem?

\n", "Title": "Why did my print fall off its raft?", "Tags": "|troubleshooting|print-failure|rafts|monoprice-select-mini|", "Answer": "

It looks to me as the model did not have enough surface contact with the raft.

\n\n
\n\n

This can be caused by to big of a gap between raft and model set in the slicer or because of Underextrusion. \nThe part itself does not look underextruded, at least not a lot, so I would say that the slicer settings were not ideal.

\n\n

I would suggest you slice a part by yourself and test the printer again.

\n\n

From personal experience a skirt or brim works better with PLA and uncomplicated models.

\n\n

For general bugfixing, explanation of slicer settings and anomalies the Simplfy3D hompage is a great resource.

\n\n

Here is an article about Rafts, Skirts and Brims and here the general guide page for print quality.

\n\n
\n\n

Your printing temperature is a bit low but shouldn't be the cause for the problem. Maybe check the specs on the PLA you used.\nYour bet temperature seems to be fine as the raft did adhere well.

\n\n
\n" }, { "Id": "10438", "CreationDate": "2019-06-30T18:54:04.080", "Body": "

I'm working on something that I'll share once it's done (then hope it's not a disaster and actually useful to someone).

\n\n

It requires a stepper to rotate a part by a certain amount of degrees, which I plan to achieve by using E1 and simply issuing an appropriate G0 command.

\n\n

The issue I'm having is that I would need Marlin, upon start to check if the part is actually rotated to the 0 deg position.

\n\n

I suspect that there is no way to i.e. use an unused Z max pin and issue something like the well known G28 or like Duet3D does with G0/1 S parameter. Is there maybe a hack I can apply with a filament run out sensor as that axis endstop?

\n", "Title": "Home E / extrude until endstop is triggered", "Tags": "|extruder|endstop|axis|", "Answer": "

You should be able to adapt the \"Dual X Carriage\" feature for your project.

\n\n

That allows a second print head (on the X axis) to move independently of the \"main\" X axis motor - and it can also be homed against its own endstop.

\n\n

You can enable this feature in the configuration_adv.h file under \"Dual X Carriage\".

\n\n

By default, all \"Dual X/Y/Z axis\" features use the first free extruder stepper driver - which would be E1 in your case.

\n\n

You would have to adjust the X2 parameters to allow you to zero the motor on the endstop (X2_MIN_POS 0, X2_HOME_DIR -1, X2_HOME_POS X2_MIN_POS), as well as setting the default mode to \"FULL CONTROL\" (DEFAULT_DUAL_X_CARRIAGE_MODE DXC_FULL_CONTROL_MODE).

\n\n

As far as I understand, you can then use\nT0 and T1 to change between both \"X axes\", with T0 selecting your real X axis, and T1 your custom stepper motor.

\n" }, { "Id": "10439", "CreationDate": "2019-06-30T18:54:10.533", "Body": "

I'm printing part for a chess board from a set on Thingiverse, expanded a little to 50 mm square.\nAfter the 3rd layer, I'm seeing what looks like raised ripples, and you can feel them with a finger too. I didn't see this when printing just 4 pieces earlier.\nPLA at ~200 °C, bed is PEX/flex steel/magnet/AL.

\n\n

\"enter

\n\n

The initial layer also had some streakiness (?) after the 1st layer.

\n\n

Odd, as the bed tests out as pretty level using the paper under the nozzle test.

\n\n

\"enter

\n\n
\n\n

Looking much better now that the infill is starting. Will have to look into calibrating the extruder when this print finishes. Printed a 6 hour iPhone stand yesterday, turned out really nice.

\n\n

\"enter

\n\n
\n\n

Top layer is nearing done (looks like top layer is finishing, then the lip to go for the edge of the board) and all signs of the rippling are gone.\nWill check the 'level' again before we start on the light color squares.

\n\n

\"enter

\n", "Title": "Ripple in layers", "Tags": "|print-quality|", "Answer": "

Printing some more parts tonight. Looks like solution is better bed \"leveling\" (tramming, or basically squareness between X, Y, and Z axis). Getting it dialed in to the correct height, 1/4 turn of a bed leveling screw at a time.

\n\n

Also ordered some metal (stainless steel) shims so we can get and check the bed level more accurately than \"this index card plus a little bit\" as the card measures ~0.16mm and we want 0.18mm to 0.2mm or maybe 0.22mm it seems.

\n" }, { "Id": "10460", "CreationDate": "2019-07-02T02:34:32.817", "Body": "

The printer is \"Monoprice MP Select Mini 3D Printer V2\".

\n\n

This is how my printed fan part looks like:

\n\n

\"Printed

\n\n

These printers are notorious for their Z lead screw assembly (M4 bolt & nut) so I installed flex coupler and aligned the M4 bolt and motor shaft so that the wobble is minimal. It is still there, but I don't think this amount of wobble would produce something like what I experience. Please check out the timelapse video at the end.

\n\n

I also tightened up Z guide rails (no rattle, very little slop)

\n\n

Moreover the banding appears to be less pronounced and have alot greater period for vase prints \"Vase

\n\n

(this is extruded ellipse printed along Y axis).\nI tried:

\n\n
    \n
  1. slowing down print by 50 % - no effect on banding.

  2. \n
  3. monitored hotend temperature with external probe on the heater block (-+5 °C)

  4. \n
\n\n

The banding appears to be almost exclusively in Y axis direction; for example, the same ellipse printed along X axis: \n\"Vase

\n\n

Now, same ellipse in layered mode (seam side) printed along Y axis\n\"Layered

\n", "Title": "Help diagnose Z-banding", "Tags": "|print-quality|troubleshooting|monoprice-select-mini|", "Answer": "

This differs from the traditional banding as observed from Z wobble induced banding as e.g. explained in this answer.

\n\n

Your banding patterns clearly seem to form diagonal bands, this is most probably a combination of the lead and the full rotation of the stepper. The most logical explanation is that the layer shifts as a whole in X-Y direction (when seen against the print height, this movement is concentric seen from the top). This means that the next layer is positioned over the previous layer in a concentric pattern. This hints to some sort of defect in your X/Y-plane assembly and should be investigated further.

\n\n

This is difficult to visualize, but this sketch shows the issue for some layers:

\n\n

\"Diagonal

\n\n

This could be related to the belts of the X and Y-axis, play on the drive pulleys, non-straight lead screw, guide rods with play, play in general, Z-stepper alignment to the threaded rod, etc. Considering the amount of Z-wobble fixes shared by the unofficial MP Select Wiki, the best place to look for is the Z-stepper to lead screw coupling.

\n" }, { "Id": "10472", "CreationDate": "2019-07-03T14:24:33.327", "Body": "

There are several cantilever-style printers around like some DIYs (2018), the TronXY-X1, the Monoprice Select Mini, Tiertime's Cetus line and the 2019 Prusa Minilinks for information, no affiliation.

\n\n

While the specs of the printers are very different, what are the main benefits, tradeoffs and drawbacks that are inherent in the design compared to a more traditional Core-XY or Prusa/Portal design?

\n", "Title": "What are the tradeoffs of a cantilever printer design?", "Tags": "|printer-building|desktop-printer|", "Answer": "

Cantilever printers constrain the X-axis carriage only on one side and have a free hanging side. This means, that the X-beam has to be rather sturdy and is limited in length, making them usually quite small machines.

\n\n

Another tradeoff stemming from this source is, that they have to stiffen this one connection as much as possible. Any play on the connection leads to increasingly large errors the further the printhead is out from the Z-Axis and can lead to bad prints, banding and total failures of prints. This behavior, at least on the TronXY-X1, is sometimes called Z-Wobble and has brought up some aftermarket modifications. For example, this bracket constrains the movement of the X-cantilever.

\n\n

While all axis would be affected by bad constraints, a cantilever suffers the most: The cantilever (if not well constrained) can deflect in both XY and XZ plane, pretty much opening a cone with the tip at the leadscrew nut. A one side-leadscrew portal design can only deflect in the XZ plane, opening an angle along with the center in the leadscrew nut. A double leadscrew design has, constrains both ends of the X-axis and prevents deflection but could suffer from bending the portal bar.

\n\n

The main benefit of a cantilever design is that being of a rather simple design with few moving parts, understanding as well as repairing and modifying the motion control system is fairly easy. The ease of access and low component number also allows reducing manufacturing costs to some degree.

\n\n

Another thing of note is, that these printers usually have a fairly small form factor, allowing them to be used in small workshops without taking up too much space. However, their build-volume to occupied-volume ratio usually is not as good as other designs - printers with a much larger build volume just get a much better ratio there.

\n" }, { "Id": "10487", "CreationDate": "2019-07-04T16:04:27.483", "Body": "

When I print parts in ABS, acetone vapour smoothing is a good technique to get a smooth finish. Is there an equivalent solvent or process for parts printed in ASA? Ideally I'm looking for something as easy to obtain as acetone, and not so awful a chemical that I wouldn't want to work with it, but I'd still be curious to learn about less friendly solvents.

\n", "Title": "What solvents are appropriate for smoothing/finishing ASA?", "Tags": "|smoothing|asa|", "Answer": "

From Simplify3D - ASA:

\n\n
\n

ASA can be smoothed using controlled exposure to acetone vapors (a process called \u201cvapor smoothing\u201d).

\n
\n" }, { "Id": "10508", "CreationDate": "2019-07-06T21:38:16.237", "Body": "

I'm attempting to print some flexible TPE filament. But I failed to imagine TPE was this difficult to print.

\n\n

Specs of the shop-brand filament:
\nRed 1.75 mm TPE (+-0.05 mm).
\nHardness: 45D.
\nPrint temperature: 220-260 °C with 0-95 °C bed.

\n\n

I'm trying to print this on my original Prusa i3 MK3S with powder coated sheet with 0.20 mm layer with PrusaSlicer 2.0.0.

\n\n

What happens? After 3 or 4 layers, the print warps a lot and detaches from the plate. The object is 40 mm long. The next image shows the print detaching from the build plate as well as a skirt of two layers height:

\n\n

\"Image

\n\n

I've tried warmer/colder, more/less fan, faster/slower. I went down to 1 mm3/s, which is 7 mm/s. For reference, PLA prints 15 mm3/s.

\n\n

I readjusted my z-cal, and when I test print a first layer with TPE it's difficult to remove from the bed.

\n\n

I also attempted the glue stick on smooth PEI sheet. Worked until the first few layers of infill, then it still warped.

\n\n

Do I have bad filament with too much shrink, poor settings or is this 45D just too soft for my MK3s?

\n\n

Bonus pile of failures:
\n\"Image

\n", "Title": "TPE warping problems", "Tags": "|warping|tpe|", "Answer": "

I had a devil of a time getting this \"greasy\" tpe85a to print from filaments.ca. Buried in the comments there:

\n\n

85A TPE Warping issue:

\n\n

Mesa K wrote on Filaments.ca to try the following:

\n\n
\n

\"Adhesion: this stuff does not stick to Prusa's smooth PEI sheets at all, it also did not stick to masking tape either. The solution I found was to tape a piece of printer paper to the bed (I used packing tape for this, but I would recommend masking tape or something else easier to remove). I didn't have a glue stick or kapton tape to try, but those may work too.\"

\n
\n\n

That's right, printing on a sheet of printer paper creates amazing adhesion for this stuff. I suspect it to be a bit of a \"greasy\", oily filament at temp. When you drop a piece of popcorn on a sheet of paper, it leaves a grease slick. It is a permanent stain. So if this filament is a bit greasy (but much higher viscosity than melted butter ;-] ), literally, at temp, the paper would actually act as an absorbent material, and even as a degreaser on the point of contact with the filament.

\n\n

I am printing a phone case. I assume you have used a sheet of paper on top of this sheet of paper and leveled the bed perfectly using the paper as feeler gauge method, or all the rest of this advice is useless. Because of the close tolerances I use on the first layer, this is critical.

\n\n

Tips: Print a huge 1 cm brim in case you have to do last ditch intervention on the print. The brim is to prevent warping from the contact surface, but also as a place to tape the print down from the topside in case it decides to warp no matter what you try.

\n\n

Extrusion: I am using a creality CR10V2 I converted to direct drive with a 0.2 mm nozzle. I replaced the stock stiff spring with a pair of ballpoint pen springs I hotglued together. ( I am kind of a barbarian.....) They put just enough tension on the tensioner wheel to reliably push the filament without it sticking to the drive gear. (Single drive gear extruder)

\n\n

Set Cura to extrude at 112 % or in that range. It will not blob at that rate, but will have a nice fat line laid down. It created blobs for me at 115 % extrusion, so avoid going to 145 % like I tried at one point. Nice to know my ballpoint pen printer can do that....

\n\n

The initial layer height will be set to 0.12 mm, with following layers set to 0.19 mm.

\n\n

The wall width will be 0.2 mm and that \"excess\" in the above calc fills gaps and binds two side by side lines together.

\n\n

This stuff also contracts if the adhesion sucks, so extrude extra and it will not contract lengthwise so badly.

\n\n

Heat:

\n\n

Extruder: 220 °C. My filament seems to get more plastic-y at higher temps. Lower temps made a softer, more flexible material. Also, printing at a lower temp reduces contraction related warping.

\n\n

Heat Bed: 85 °C. Keep it there the whole print. This stuff likes the warm and contracts less if kept warm the whole print. I tried starting at 85 °C and letting it cool after the first layer to 40 °C, and all 4 sides came loose from the painters tape surface. Keep it hot!

\n\n

Increasing your fan speed seems to help it contract as soon as it is laid down, laying a bead that has already contracted somewhat by the time it is embedded in the layer.

\n\n

Other tips: Printing infill at 25 mm/s was too fast. I have it making nice prints at 18 mm/s throughout the print. TPE prints slow. Fails print slowly too. Lots of beautiful failures on the path to success... Knowledge gained from watching the machine print.

\n\n

The above guidelines will help you get prints without glue, sometimes with a bit of tape around the brim if it is really cranky. Good luck, very tricky material to work with on large surface area prints.

\n" }, { "Id": "10513", "CreationDate": "2019-07-07T18:04:14.470", "Body": "

I recently discovered this kit after reading this Instructables, Adding More Extruders to Any 3d Printer:

\n\n
\n

\"CNC

\n
\n\n

I'm pretty sure I can use this kit with my board since it uses the same drivers as mine. But that's for motors, not fans. And while I know G-code pretty well, I'm not sure how I would use this to activate and deactivate a fan from G-code. There is probably a better way to do this.

\n\n

The board I am using is from an FLSUN Large Scale 3D printer. Here is a picture of the board:

\n\n

\"FLSUN

\n\n

There appears to be only one labeled pin for the fan. BUT even if there are other pins that I don't recognize, they would have to be controlled by a micro controller (G-code commands). There appear to be a bunch of un-used pins in the bottom right of the board. But if this board just can't do it, there is a newer board here: [link removed].

\n\n

It does seem like it is using Arduino and the newer board might have extra pins for a fan. But at that point, would it be easier (cheaper) to just control the fan from the extruder extender kit? Would I just set it as an extruder with a really high filament extrusion speed and send appropriate G-code commands when needed to run it at max voltage?

\n\n

I know on my Lulzbot Mini there is a \"parts cooling\" fan which allows you to cool off the layers as your structure rises vertically. This is a fan I want. The parts cooling fan must be controlled by the micro controller. It only comes on when printing vertically.

\n\n

I would like to actually add two fans like this to my 3D printer. One of them is a >= 5 V cooling fan like above. Another is a regular 12 V cooling fan for an extra extruder that I am adding.

\n", "Title": "How can I add an extra fan, controlled by a microcontroller, to my board?", "Tags": "|prusa-i3|", "Answer": "

You can use the M42 g-code to manually set any supported digital pin, which can then be used to either enable one of the on-board MOSFETs (D7, D8, D9, D10) or an external MOSFET.\nFor example, M42 P9 S255 would enable the parts cooling fan at 100 %.

\n\n

You should never run any fan or heater directly off of a microcontroller pin (the ATmega2560 on your board supports up to 40 mA. Standard 5 V fans I found online tend to draw 100 mA or more).

\n\n

Your board supports up to four switchable \"power\" outputs - bed, heater 0, heater 1 and FAN.\nDepending on what you use so far, one of those may be usable for your fans.

\n\n

Note that on-board MOSFETs usually switch the ground side of the connected device.\nThis means that you for your 12 V fan, you can connect it directly to one of those connectors.\nThe 5 V would have to receive +5 V from elsewhere (like the +5 V pins near the bottom right mounting hole), but you can still control the fan by connecting its ground lead over one of the on-board MOSFETs.

\n\n

If four MOSFETs are not enough for you, the L298N module provides an easy way to control four additional fans, while using normal digital pins to control the L298N.

\n" }, { "Id": "10515", "CreationDate": "2019-07-07T18:41:11.547", "Body": "

This basic question has been bothering me for a while.

\n\n

Let's say I'm printing an object of height 20.1 mm with 0.2 mm layers. What will the slicer do? in other words, how will it print the last layer? Since I'm printing with 0.2 mm layers, my object will be printed in 11 layers, but the last one would make my object 20.2 mm high. Is there some sort of compensation (in Slic3r or Cura for example), or will I just get an inaccurate object?

\n", "Title": "Object's height not a multiple of layer height", "Tags": "|slicing|dimensional-accuracy|", "Answer": "

Slicers will round off to the next nearest layer, so 20.2 mm in your case. However, you can get to 20.1 mm if you use a 0.3 mm first layer.

\n" }, { "Id": "10524", "CreationDate": "2019-07-08T14:34:07.763", "Body": "

Newbie alert...
\nOn my Ender-5, when I go into the \"Tune\" menu during a print and adjust the \"Speed\" value, that value will later be shown in the display next to a label saying \"FR\". I can also adjust that value by simply turning the knob during printing (and thus started to think of it as the \"speed dial\" ;) ).

\n\n

As far as I was able to tell so far, the \"FR\" percentage value is being applied to all four stepper motor movements and thus allows me to slow down or speed up printing on-the-fly, e.g. to make up for sub-optimal speed settings chosen during slicing (after all, I'm still learning).

\n\n

I only recently learned that FR is actually short for \"Flow Rate\" (or is it \"Feed Rate\"?) and that seems to imply that this is probably about more than just motor speed... Also, there seems to be no equivalent to the Speed setting in Octoprint: All I have on the Control tab are two distinct sliders for \"Feedrate\" and \"Flowrate\". Would I always have to move both to achieve the same effect?

\n\n

Can anyone clarify? What implications of changing FR/Speed might I be missing?

\n", "Title": "What exactly is the relationship between Flow/Feed Rate and (Print) Speed?", "Tags": "|octoprint|speed|", "Answer": "

By turning the knob in the main screen, you're adjusting \"feed rate\".\nThis is essentially a factor that all g-code speed settings are multiplied with - \"speed dial\" seems an appropriate name for it.

\n\n

\"Flow rate\" is something different altogether - this is multiplied with the extrusion commands. It has the same effect as changing your extruders steps-per-mm.\nYou can adjust under- / overextrusion with this on the fly.

\n" }, { "Id": "10538", "CreationDate": "2019-07-10T04:48:36.217", "Body": "

Recently (in 2017) there was a paper that got some publicity by researchers who are using a B spline algorithm to reduce vibrations in 3D printers. But before them, a B Spline implementation seems to have been first been made open-source by an alias named DeepSoic here. I would like to be able to print faster using the method described in the research paper, through post-processing G-code. I'm pretty sure these two sources use basically the same technique but I could be misunderstanding things.

\n

Basically instead of stopping and starting for travel moves, speed changes are done in a curvy fashion, so the head never stops and the printer never shakes. This makes the print smoother and also faster. I think printing 10 times faster is something that is really awesome once you try it. Laser cutting relies on cubic splines for a different reason; to create curves in space. But it seems like these techniques are doing something unique to to 3D printing -- using them to adjust head acceleration/de-acceleration to create smoother movement arcs of the print head. Since laser cutters have a constant head movement, this technique wouldn't help them much.

\n

The downside seems to be that it makes way more G-code commands, overloading the USB port, since it's sending all the points on a curve so quickly. I'm assuming a smart person today would really only use it through an SD card (which has disadvantages) or if they bought a 3D printer with a free Wi-Fi module thrown in (which also has disadvantages). Maybe a high baud rate helps.

\n

I was wondering if there are any more established ways to use this obviously extremely important and beneficial and simple algorithm. Initially I was thinking that this is obviously something that should be added as a checkbox in a slicer, and not something to be implemented in Marlin. But after writing this post I realized that a Marlin implementation would allow you to use this technique over USB, but only if the slicer steedleaders are also using its special G-codes for this optimization. I don't care if it's a post-processing technique like the research paper's or a special Marlin-friendly version, I just want to use this technique even if I have to use this Huawei Wi-Fi module.

\n

Basically I would like to know the best way to get started using this technique through a slicer or other software.

\n
\n

I think there is a miscommunication between users of CNC laser cutters and users of 3D printers. In laser cutting the arcs are used to define the path of the cut, which would be equivalent to filament extrusion. In laser cutting, the motion of the laser itself is constant. But in 3D printing, arcs can be used to smooth the speed of the printhead as it moves across the perimeter, and then to infill. It is using arcs for controlling the head well which isn't a problem in laser cutting. Since it's about the head movement, and not the model itself, I don't see how the STL file really matters.

\n

It's really about using an arc to set head speed (a first derivative of position). Not anything about the shape of the model (which would just be position). At least that's my interpretation.

\n

The Wi-Fi module is interesting because it receives an IP address from my router, then my router stops listing it as a connected device. But it still connected, because I can access it wirelessly. I am going to look into it more once I can fix some other problems with this dual-head. But so far there's a reason to think it might be backdoored.

\n", "Title": "How to post-process G-code to make prints faster using splines and arcs?", "Tags": "|g-code|slicing|", "Answer": "

I would have liked to answer linking to credible official sources, but I cannot add references either on direct B-spline printing. So I'm writing down my thoughts. I've familiarized myself in B-splines to understand what they are and read into the 2 references given by the OP.

\n
\n

Basically, the printer software only allows printing of straight lines. Yes I know we can give orders to the printer to print a curve (using G2 or G3), but these eventually will be converted to printing straight lines. There is no ready made printer firmware available to print cubic curves directly to my knowledge. If it would be possible, these curves should eventually be translated into smaller straight lines by the firmware of timed stepper rotational output. These extra calculations would demand a considerable effort of the printer board processor, most probably far more an 8-bit processor would be able to handle.

\n

Comparing the paper released in 2017 to the G-code pre-processing software reveals that although both seem to refer to B-spline techniques, they are implemented differently. For example, the pre-processing software aims to reduce the linear travel moves by replacing these with B-spline curves (and not affect the actual print object), while the paper focuses on the optimization of the actual printing curves being optimized by B-spline curves (also using a pre-processor). Both eventually would need to create a multitude of small straight lines to have the printer be able to actually print the object as there is no 3D printing firmware solution to print curves. Do note that the method in the paper has been questioned by the RepRap community, which demonstrated that they could print the same object way faster than the B-spline optimized example. Furthermore, do note that the Marlin community is probably moving in that direction as can be seen from e.g. this feature request and this G-code meta overview; G-code instruction G5.

\n

So, both methods rely on pre-processing G-codes by identification of sliced coordinate (print) moves, translation into B\u00e9zier/B-spline curves for (print) moves, which eventually are translated into normal G0/G1 (print) moves. It does not appear that the Marlin community/developers are aiming to implement B\u00e9zier or B-spline curves soon. This implies that if you want to pursuit printing B-splines, you need to make your own pre-processor, or dive into Marlin C++ development; an 8-bit based printer board would not be sufficient indeed like the OP mentioned, up-scaling to 32-bit or interfacing with USB might be the only solution.

\n" }, { "Id": "10540", "CreationDate": "2019-07-10T07:36:56.617", "Body": "

If a customer sends me a non-commercial 3D model to print, am I allowed to charge money for the 3D printing process? I understand I cannot charge anything for the model nor offer it as a part of my business.

\n\n

I cannot download the model myself, print and sell it, but if the customer downloads it and sends it to me for me to print it, is it a violation of the license or not?

\n", "Title": "Printing non-commercial model to make money", "Tags": "|3d-models|legal|", "Answer": "

I agree that intellectual property is complex. I am not a lawyer, but have been in the middle of patent and copyright actions.

\n\n

In my non-legal opinion, you are providing a printing service for which you are free to charge your customer whatever you agree on. You are not responsible for the disposition of the resulting objects. You do not know how your customer intends to use them. They may be using them for commercial or non-commercial activities. They may have a license to use them in a way that is unknown to you, and, again IMO, they have no obligation to disclose their relationship with the IP holder. That license could be part of their trade advantage.

\n\n

If the customer asks you to work with the design to make changes, and the license prohibits derivative works, I would be careful.

\n\n

Never-the-less, the smell test always applies. If it seems to be illegal, unethical, or immoral, you might choose to forgo the customer and avoid any possibility of being tainted by their activities, if it should turn out that they are behaving badly.

\n" }, { "Id": "10546", "CreationDate": "2019-07-10T21:25:07.453", "Body": "

PET-G is the main filament I use. I can achieve high quality of my prints with my Prusa i3 mk3s printer. Recently however, I've recently changed my nozzle from the default 0.4mm to the 1.2mm. I watched the online video tutorial on how to do it properly and did the whole process carefully.

\n\n

Temperature: 250 °C/100 °C\nSpeed: max 200 %, but results are the same even all the higher ones .\nLayer height: 0.35 mm

\n\n

With my first print (and the next ones too) on the new nozzle I noticed lots of under-extrusion with infill (holes). LOTS of stringing although I've increased retraction. What's the reason ?

\n", "Title": "PET-G under-extrusion after changing the nozzle to a 1.2 mm d. one (Prusa i3 mk3s and PrusaSlicer v2.0)", "Tags": "|print-quality|prusa-i3|3d-models|petg|underextrusion|", "Answer": "

Piecing this answer together from the comments on OP's post.\nMyself and R.. noted that a layer height of 0.35 mm, nozzle width of 1.2 mm, and a fairly high feedrate (200 % according to OP, no reference to what 100 % is), is an exceptionally large amount of plastic to try and melt through almost any hotend on the market except possibly the Volcano, or the high-throughput version of the Mosquito.

\n\n

OP responded that lowering the layer height to 0.2 mm seemed to fix the issue. That's still an impressive amount of plastic, but it's a 43 % decrease in total flow compared to 0.35 mm layer height.

\n" }, { "Id": "10550", "CreationDate": "2019-07-11T08:44:58.130", "Body": "

Am just wondering if any conclusions can be drawn from this:

\n\n

\"Photo

\n\n

Three corners are solid, but not the one in the centre of the plate.

\n\n

The bed was levelled before printing (and checked afterwards also). Even though the photo may appear to show a slant or lower corner (where the print is coming off), there is not. The bed is level, relative to the extruder, at room temperature.

\n\n

The temperature of the bed is about 70 \u00b0C. I get inconsistent readings (with laser thermometer) but to the finger it feels about the same everywhere.

\n\n

It's a glass bed, presumably with some coating. Is it degraded? Local temperature variation? Any ideas anyone?

\n", "Title": "Build plate adhesion, PLA", "Tags": "|pla|adhesion|build-plate|", "Answer": "

I cleaned the bed with acetone and it seems to have helped, so presumably it was just a build-up of something.

\n" }, { "Id": "10565", "CreationDate": "2019-07-12T22:32:49.220", "Body": "

I am trying to slice a model that is half a mm less than max width, but not successful.

\n\n

\"enter

\n\n

What am I missing? Is there some minimum value less than maximum allowed, or something?

\n\n

Edit: after changing the width to 220 in machine settings, slicing works. This is a dangerous thing to do, as it could damage the printer.

\n", "Title": "Using maximum width when slicing in Ultimaker Cura", "Tags": "|ultimaker-cura|slicing|", "Answer": "

This answer already addresses that Ultimaker Cura \"eats up\" platform space for e.g. skirt, brim, raft, dual extruder, deposition of priming blob, prime towers, etc. Disabling those features will reclaim platform space so you can print larger prints. However, that will only work when your printer is correctly configured! E.g. the center of the bed needs to be the center of the center in the slicer which needs to have the specific sizes of the bed dimensions. Note that increasing the bed size past the actual dimensions is not considered to be a nice solution, it is an easy work-around that gives you extra space in X+ and Y+, i.e. it does not center this newly created space, furthermore, this can destroy your printer is there is tight space left on those axes! Let's illustrate that with an example, if you have a 200x200 mm build plate and want to slice something of size 200x200 mm, this should be centered around (100, 100), if you change the bed size to 220x220 mm, Ultimaker Cura will center the print around (110, 110) which means that the print maximum coordinates are 210 mm; this is outside the bed area and potentially can destroy your printer!

\n\n

What you should check is if the physical center of your bed actually is the center as defined by the firmware of the printer (surprisingly, many of the cheaper printer have this incorrectly configured). The answers on question \"How to center my prints on the build platform?\" (Re-calibrate homing offset) describe how you could do that.

\n" }, { "Id": "10571", "CreationDate": "2019-07-13T14:26:33.847", "Body": "

Standard Marlin has one problem: when the power suddenly is gone, the print is gone. Prusa and many china printers however come with \"Power Loss Recovery\" or \"Power Out Protection\" or similar. But especially China printers come without Thermal Runaway Protection, so in order to make the printer safe, one often has to get rid of TRP (in the basic shape it comes).

\n\n

How can the PLR be turned on?

\n", "Title": "How to activate Power Loss Recovery in Marlin?", "Tags": "|marlin|firmware|knowledgebase|", "Answer": "

Marlin firmware has such a feature that can be enabled to resume printing after a power outage.

\n

To enable power-loss recovery you should send

\n
M413 S1 \n
\n

to the printer using a console (e.g. using Pronterface, OctoPrint, Repetier-host, etc.) or put commands in a text file with extension .g that can be printed from SD card. To disable power-loss recovery send/print:

\n
M413 S0 \n
\n

To report the state of the power-loss recovery, send through a console:

\n
M413\n
\n

This will result in a returning message in the console of e.g. This Power-loss recovery ON.

\n

To retain the setting, you can use M500 to store it in memory.

\n
\n

If you enable M413 in Marlin firmware, the printer will write a resume printing file to SD card e.g. every layer.

\n

From M413 - Power-loss Recovery documentation I quote:

\n
\n

Enable or disable the Power-loss Recovery feature. When this feature is enabled, the state of the current print job (SD card only) will be saved to a file on the SD card. If the machine crashes or a power outage occurs, the firmware will present an option to Resume the interrupted print job. In Marlin 2.0 the POWER_LOSS_RECOVERY option must be enabled.

\n

This feature operates without a power-loss detection circuit by writing to the recovery file periodically (e.g., once per layer), or if a POWER_LOSS_PIN is configured then it will write the recovery info only when a power-loss is detected. The latter option is preferred, since constant writing to the SD card can shorten its life, and the print will be resumed where it was interrupted rather than repeating the last layer. (Future implementations may allow use of the EEPROM or the on-board SD card.)

\n
\n

This means if you cut the power you can resume the print layer, the only problem is that the part must remain attached to the plate, if it comes loose it is hard to resume printing. This feature is now commonly found on printers these days.

\n

The regular pause and resume functionality of the printer will not work when the power is cut over night, i.e. no recovery file is written in such a case.

\n" }, { "Id": "10573", "CreationDate": "2019-07-13T14:49:41.060", "Body": "

I read that G-code commands can be sent through a console/terminal over USB. What is a console/terminal and how do you use that?

\n", "Title": "What is a printer console/terminal?", "Tags": "|g-code|software|knowledgebase|usb|", "Answer": "

In addition to this answer, the OctoPrint 3D print server software contains a terminal which you can use to send G-code commands from a browser:

\n\n

OctoPrint

\n\n

In the bottom string input box (under the check mark items) you put in a G-code command, which will be send to the printer when you hit the Send button. If the printer gives a reply to that command, it will be displayed in the log window above the check mark interface items.

\n\n

\"OctoPrint

\n" }, { "Id": "10583", "CreationDate": "2019-07-14T02:53:25.627", "Body": "

I've been trying a lot of different things to combat corners curling upward in the first few tens of layers after the bottom skin. To be clear, I'm not talking about corners of the first layer printed on the bed, but rather the points of the outline in layers above the base where direction of print motion changes discontinuously (discrete corner) or abruptly (turn with very tight curvature). Here's an image I found (not mine) that demonstrates:

\n\n

\"print

\n\n

And a pic during print of the type of curling I'm talking about:

\n\n

\"curling

\n\n

And some previous worse prints:

\n\n

\"2

\n\n

My go-to worst test case for this now is a 20mm tall hollow dodecahedron with 0.8mm shell (hollow geometry, not just empty infill; 0% infill on a non-hollow model does even worse, shown above). For everything else I've tried, I've mostly been able to sovle the problem with combinations of

\n\n\n\n

but I can't get all 5 edges of the worst-case dodecahedron completely warping-free without just heavily slowing down the print; during print it's obvious that the curling at the corners in each layer is the source of the warping. Increasing Cura's cool_min_layer_time to 10 seconds (default is 6, and I usually get by fine with 3-4.5 for most things) mostly but not entirely solved it, and going much slower than that seems likely to introduce other surface artifacts from extremely slow extrusion.

\n\n

Are there any additional tricks I'm missing for solving this? I'd like something that's easy to leave on all the time or at least to automate, as opposed to hacks like adding in a junk tower off to the side to waste time between layers.

\n\n

My printer is an Ender 3 with stock gear except for improved fan duct. The problem was worse with the stock fan duct.

\n", "Title": "How do you solve PLA corner-curling short of printing really, REALLY slow?", "Tags": "|print-quality|pla|warping|", "Answer": "

While I tried a lot of things to solve this, including tuning temperature, fan, speed, etc., ultimately the single biggest factor that causes or prevents it is the state of Cura's Outer Before Inner Walls (outer_inset_first) option. With outer walls first, I don't have the problem at all. With the default (inner walls first), I have it to varying degrees depending on geometry and a lot of other factors.

\n\n

I don't have a good explanation for why this happens so I'm asking a new question about it.

\n" }, { "Id": "10585", "CreationDate": "2019-07-14T04:20:17.843", "Body": "

So I am really fed up with inductive probes. The one I am using keeps getting shifted slightly every time I switch nozzles or run an oozy print. That means I have to autolevel again, then manually set a Z-offset (as I would have anyway if I didn't have an inductive probe).

\n\n

On my Lulzbot Mini there is a different scenario. There are four washers at each part of the bed. The nozzle is \"grounded\" so that when the Mini touches the washers, a current is created that seems to act as the Z-stop. Surprisingly there isn't much out there for a DIY implementation of this.

\n\n

Since I have an aluminum bed (and aluminum is conductive), I am thinking of doing the following:

\n\n

1) Put one wire from the Z-stop ground pin to the aluminum bed. Make sure it is away from the wires for the heater / thermister (?)

\n\n

2) Put one wire from the Z-stop 5V into the heating block of my nozzle.

\n\n

When the nozzle probes the bed, a current will be created from the 5V heating block, through the conductive nozzle, into the conductive bed, to the Z-stop ground.

\n\n

I'm always unsure when it comes to circuitry. Will there be any dangerous interference from this technique from, say, the bed heating circuit? I'm not sure what kind of protection circuitry are on each of the Arduino's pins, and I'd rather not fry my board if this sounds like a bad idea to someone.

\n\n

I figure most people don't do this because they have sheets of PEI or some other non-conductive material on their bed. I can use PET tape but still leave holes in the tape for this autobed leveling probe. It would be really great if it worked and wasn't dangerous.

\n\n

I shouldn't even need the third pin?

\n", "Title": "Wiring Z-stop directly to hot end and aluminum bed / spacers", "Tags": "|z-probe|", "Answer": "

This will not work reliably.

\n\n

I know, I have tried it, for a couple of years, with poor consistency.

\n\n

Now, I will tell you that it worked better than the parallax IR sensors. It worked better than trying to slam the head into the bed and listen for the click.

\n\n

I used the brass nozzle and the aluminum bed as a switch to detect the bed position.

\n\n

I used ABS slurry on the bed. With a 100\u00b0C bed the ABS was soft enough for the nozzle to make contact. Elmer's Glue for PLA also was soft enough.

\n\n

But, with the elasticity I had in the synthetic Z-axis of the delta machine, the time delay to much the bed adhesive out of the way, and the general problem of trying to conduct electricity through an aluminum oxide layer, I had variability of about 0.1 mm, which was far to much to give a reliable first layer.

\n\n

To \"level\" the delta bed, I would touch each point several times (with a clean bed) and fit my leveling function to the noisy data. For finding my zero reference at the beginning of a print, I would touch off three time and only use the third one. This helped, but it was still super noisy.

\n\n

I have subsequently incorporated a strain gauge in the triangular delta-bit. It gives much more accurate contact information and is not effected by the bed glue not does the aluminum oxide layer cause problems.

\n" }, { "Id": "10587", "CreationDate": "2019-07-14T11:06:39.267", "Body": "

I have a model that contains a cavity, into which I want to insert a piece of metal, so I can use a magnet to stick to the print. How can I introduce a pause into the G-code without manipulating it manually in Ultimaker Cura?

\n", "Title": "How to pause a print to insert something in a cavity using Ultimaker Cura?", "Tags": "|ultimaker-cura|g-code|knowledgebase|", "Answer": "

This answer already explains how you insert the G-codes to enable a pause into your model. But, this will only work if the printer supports the G-codes that are inserted by Cura. E.g. this question shows that this does not always work!

\n\n

To pause the printer you would need to resort into other methods, e.g. a manually inserted G4 (Dwell) would be a viable solution as shown in this answer.

\n" }, { "Id": "10593", "CreationDate": "2019-07-15T03:37:23.467", "Body": "

I'm new to OpenSCAD, but I can only get default parameters working for simple values.

\n\n

I'm wondering if there is any way to achieve the following, where I have some parameters available to the module caller but by default they are derived from other parameters.

\n\n

I had expected this to work - I thought the compiler to be able to resolve these variables in-scope during compilation since everything here is deterministic - so perhaps I'm just missing some syntax? If not, is there some better way to achieve this?

\n\n
module clasp(length=20, \n             pin_radius=5,\n             mouth = pin_radius * 0.9,\n             inner_radius = pin_radius + 0.25,\n             outer_radius = inner_radius * 1.4,\n             ) {\n  ... do the work ...\n}\n
\n\n

Unfortunately this approach leaves mount/inner/outer all undef.

\n", "Title": "Handing OpenSCAD module parameters derived from other variables", "Tags": "|openscad|", "Answer": "

This seems to be a limitation of the language. One workaround that's only moderately ugly is, in the body of the module:

\n\n
mouth = is_undef(mouth) ? pin_radius * 0.9 : mouth;\n
\n\n

etc. If you'll only be using the file via a use directive in other files, another approach is file-scope variables. These can be overridden when the module is called, just like module parameters, and they do not provide or take values for/from the calling file's file-scope variable namespace.

\n" }, { "Id": "10594", "CreationDate": "2019-07-15T03:56:21.063", "Body": "

While printing PET-G (@ 220 °C nozzle and over 58 °C bed temperature), the outlines always print fine but the infill keeps shoveling. Could I be printing too hot or too cold? I am using a 0.8 mm nozzle.

\n\n

Shoveling is when the plastic is over extruding to the point at which it starts piling up in the path of the nozzle, usually resulting in an uneven surface. Visually, it manifests like a snow plow shoveling snow. I tried adding an image, but, the filament is black so that turned out to be a difficult task.

\n\n

This reason why this is a problem is because the nozzle hits those peaks while printing, which seems to be causing the print to detach from the bed. I've also noticed the print curling up at the edges, but I think that's a separate problem.

\n\n

I am printing at 3000 mm/min. The outline, and the infill is 80 % of that. The layer height is 0.6 mm

\n", "Title": "Infill keeps \"shoveling\", but perimeters are fine", "Tags": "|print-quality|infill|", "Answer": "

Normally, what you're calling shoveling is caused by having the bed too high - when you deposit enough material for a space that should be the nominal layer height high, but significantly less volume is available, it has nowhere to go but up around the edges of the nozzle.

\n\n

However, in your case your temperatures are also seriously wrong for PETG. The normal recommended range of nozzle temperature for PETG is 230-250 \u00b0C, and in my experience, you need the full 250 \u00b0C to have any hope of printing fast. At 220 \u00b0C I'm really surprised you're not having problems with underextrusion instead.

\n\n

The bed temperature is likely even more important. Minimum bed temperature for PETG is 80 \u00b0C. If the material is hitting a 58 \u00b0C bed as it comes out, it's likely to cool way too fast. This may lead to what you call shoveling (especially if you see both pits and ridges rather than just ridges), but even if not, it's going to prevent the material from bonding to previously laid down lines, so that your print will end up more brittle than PLA.

\n\n

One additional detail I initially missed involves your 0.8 mm nozzle. It might be hard for the hotend to keep up with properly melting that much PETG at normal print speeds. As noted in the comments, a 50 mm/s linear extrusion rate with an 0.8 mm nozzle is equivalent, in volumetric extrusion rate, to a 200 mm/s linear extrusion rate with a typical 0.4 mm nozzle, which would be extremely fast for PETG. It's unlikely that any hotend except a \"volcano\" or similar (with extended melt zone length) could keep up with raising that much material to 250 \u00b0C that quickly.

\n" }, { "Id": "10595", "CreationDate": "2019-07-15T04:27:54.393", "Body": "

Goal : create a pair of bevel gears for 90 degrees angle axles.

\n\n

Context : the gears are designed with an OpenSCAD library (https://www.thingiverse.com/thing:1604369, function 'pfeilkegelradpaar'), then I try to modify them using TinkerCAD.

\n\n

The problem : When printing the STL exported from OpenSCAD, everything seems fine. But when I import the STL in TinkerCAD, there is a separation where the angle changes, and I cannot seem to find a configuration that works.

\n\n

\"enter

\n\n

Also, when previewing with Cura (tried with the older v14 as well as the latest v4), I get the same behavior : the exported STL creates a solid object, while the STL from TinkerCAD (even if not modified, just imported and exported) has a 'gap' of about 8 layers.

\n\n

\"enter

\n\n

And, of course, everything fails afterwards.

\n\n

Any help is appreciated. Thanks in advance.

\n", "Title": "Bevel gears 3D model", "Tags": "|ultimaker-cura|3d-design|slicing|", "Answer": "

I don't know how to explain how the problem is occurring with the STL post-processing tools you are using.

\n\n

Try loading the STL directly into a slicer and view the result, then slice and view the toolpath. It is always helpful to eliminate steps in an attempt to narrow down where the problem is generated.

\n\n

Were I to try to patch the OpenSCAD model, I would create a hub component that overlays where the crack is found. This would be small cylinder with a hole that overlays the hub.

\n\n

I have had success adding elements and holes to existing OpenSCAD models and imported STL files using OpenSCAD.

\n" }, { "Id": "10596", "CreationDate": "2019-07-15T05:07:36.373", "Body": "

How should I describe this part which looks like a threaded flange so that I can research replacements? It is the gold piece in the middle of each photo. It is used to create a bed raiser in the FLSUN Cube 3D printer.

\n\n

\"enter

\n\n

\"enter\n\"enter

\n\n

It gets attached to a motor using a flexible bearing

\n", "Title": "What is this called? A tube flange bearing threaded for threaded pushrod", "Tags": "|replacement-parts|part-identification|", "Answer": "

This answer already describes the name of the \"golden\" component you are after, this answer expands upon that answer to note that there are various nuts with different thread sizes that look virtually the same, it would be a pity to order the incorrect one.

\n\n
\n\n

Note that this trapezoidal lead screw nut is made from brass (e.i. in your image, but these nuts are also available in POM/Delrin) and needs to have exactly the same screw threads as your lead screw has. A much used lead screw is using the following designation Tr8x8(p2) ( for a full description of what that means look into this answer (Anet A8 lead screw threads)). Do note that there are many lead screw nut sizes, buying the wrong one will not fit the current lead screw.

\n\n

\"Example

\n\n

From the image you supplied it looks as if that is the same lead screw as used in the linked answer of the Anet A8 lead screw. To verify this, you could measure how much the nut advances on a full rotation of the nut, if it is 8 mm, buy the Tr8x8(p2) if e.g. 4 mm, buy the Tr8x4(p2). Note that if you are going to buy a new nut, you could opt for a nut that has no backlash, BUT, be sure that it would fit the acrylic hole and you have enough space as its height is larger, or be prepared to modify the acrylic part or reconstruct a new one (it wouldn't be a bad idea to do that anyways, acrylic is known for cracking under applying force such as screws and nuts). Note that an anti backlash nut must be used as depicted below.

\n\n

\"image

\n" }, { "Id": "10601", "CreationDate": "2019-07-15T09:06:13.603", "Body": "

I would like to make a 24 V (3D printer board and shield) setup, as opposed to the usual 12 V, and to do so I had been considering using the Taurino Power board, or the clone Eruduino. However, I just found this board:

\n\n

\"Re-ARM

\n\n

The specifications state a DC input of up to 36 V:

\n\n

\"Specifications\"

\n\n

Does anyone know whether that really means it can handle 24 V in the same manner as the Taurino/Eruduino? If so, then that looks like a double win: not only 24 V support, but also a faster processor. Anyone have experience with this board?

\n\n

I was thinking of using with a RAMPS1.6 Plus (maybe), or just a regular RAMPS 1.4 (hacked to support 24 V). I'm just shopping about, and I thought that if I was going to spend \u00a314 on an Eruduino, then I just as well spend that money on something better.

\n\n

It does work with Marlin apparently, as some of the customer reviews would suggest, but none of the reviews that I could find referred to a 24 V setup (heated bed etc.), hence my question.

\n", "Title": "Is this re-Arm controller actually 24 V capable?", "Tags": "|printer-building|power-supply|", "Answer": "

For completion, I've just seen this, Can a ramps 1.6 support 24v? (which basically confirms the 24 V support of the Re-ARM board) although it isn't particularly useful w.r.t. the RAMPS 1.6 side of things, although I would imagine the the 24 V RAMPS hack would still apply.

\n\n

In addition, Alex Kenis does a great review, and he has successfully tried it with 24 V, watch 32-bit series part 4: Re-ARM board \"review?\". Whilst the RE-ARm offers a lot of advantages, some of the main down points to be aware of are:

\n\n\n" }, { "Id": "10611", "CreationDate": "2019-07-16T07:02:36.777", "Body": "

One of the main hacks for converting RAMPS 1.4 boards to use with 24\u00a0V, as stated in RAMPS 24V, is replacing the polyfuses, principally F2 (MF-R1100), with wire and using an inline (car blade or wire) fuse on the heatbed wire (or between PSU and RAMPS) instead1. However, that is for the RAMPS 1.4 boards.

\n

As RAMPS 1.5 notes (as well as 0scar's answer to RAMPS 1.4, 1.5 or 1.6?):

\n
\n

The RAMPS 1.5 uses small surface-mounted fuses rather than the large yellow fuses prone to breakage on the RAMPS 1.4. The downside is that replacing the fuses becomes much more difficult.

\n
\n

Are these SMD fuses rated the same voltages, or greater? Yes, this could be a bit like asking "How long is a piece of string" as it depends upon the manufacturer, but does anyone know what voltage should they be rated for?

\n

Ultimately, if they are both rated at greater than 24\u00a0V, then there should be no need to replace them.

\n

The answer on this thread, Re: Ramps1.4 or Ramps1.5 or ramps 1.6??? states:

\n
\n

OK the ramps 1.6 can only handle 12v OR 24V

\n
\n

so, that would imply that the intention for 24\u00a0V support was there, although, unfortunately, the poster does not post their reference.

\n

However, the PDF of the RAMPS 1.6 schematic shows the same rated fuses as the RAMPS 1.4

\n

\"RAMPS

\n

Nevertheless, that seems like a straight forward copy and paste from the RAMPS 1.4 schematic as it clearly references the MF-R500 PTC, and obviously SMD fuses have been used instead - or are the part numbers the same for the SMD fuses..? I had a google but couldn't see MF-R500 SMD fuses (maybe I didn't look hard enough?).

\n
\n

Footnote

\n

1 This is because the 11\u00a0A fuse is only rated to 16\u00a0V. Note that F1 (MF-R500) is rated for 5\u00a0A at 30\u00a0V, and as such is sufficient for 24\u00a0V operation.

\n", "Title": "Is it necessary to replace the SMD fuses in RAMPS 1.5 or greater, for use with 24 volts?", "Tags": "|printer-building|power-supply|", "Answer": "

I found that at least the following two RAMPS 1.6 derived boards are 24 V capable without modifications:

\n\n

This information is so far only from webshop offers, so might not be the most reliable (and I did not test it myself yet). If somebody can find a confirmation from the manufacturer's spec documents, please let me know.

\n" }, { "Id": "10623", "CreationDate": "2019-07-18T20:06:52.717", "Body": "

Do the properties of 3D printer resin allow it to stick to copper clad surfaces when cured? I am interested in turning my filament based 3D printer into a photo plotter with a laser that can cure a thin layer of resin coated onto a copper surface.

\n", "Title": "Can the resin from Resin based 3D printers be used to make printed circuit boards?", "Tags": "|resin|", "Answer": "

Brushed aluminum is a common bed surface for resin based printers. The aspect of the aluminum that is important to the print is the adhesion, hence the roughness of brushed aluminum. It has to be sufficient to hold the print in place, but not so extreme as to cause destruction on removal.

\n\n

In the case of copper as a print surface, one would certainly want some mechanical adhesion in the form of surface roughness. You've not specified the forces that would be applied to the copper once cured. If you do not plan to manipulate the copper surface in an excessive manner, it's likely that it would adhere. It's also just as likely to pop free if the plate or surface is flexed. I've seen no reference online to copper coated print beds. This would lead me to believe that it was tested and rejected as suitable for 3D printing, but not necessarily unsuitable for your purposes.

\n\n

This is one of those situations where testing is warranted and not particularly difficult to accomplish. You can purchase UV curing resin, build a tin to hold the copper surface and pour the expected layer thickness. Leave it in the sun and let nature's UV do the curing.

\n\n

Consider if you have not yet done so to research safety practice for handling this substance. Wear gloves and eye protection, cover any exposed skin surfaces and have good ventilation.

\n" }, { "Id": "10629", "CreationDate": "2019-07-19T15:07:26.323", "Body": "

I have a Duplicator i3 mini, which has yet to make it a month without breaking. This time it is extra broken because the filament is not extruding properly. the most successful print I've had yet had about a centimeter before turning into an absolute mess. I have a picture. It was not stringy, and had the exact shape i was trying to print, but was like a frame of a sort. I am printing with matterhackers MH build series PLA, which has worked before this started happening. What should I do? What troubleshooting steps should I take?\"It's

\n", "Title": "Why is my 3D printer not extruding properly?", "Tags": "|print-quality|pla|troubleshooting|extrusion|", "Answer": "

Underextrusion and clogs can also be caused by insufficient temperature in the hot end. You've not reference your temperatures, so consider to use a test model and print at different temperatures. Too low temps can result in the problem you present, while too hot temps will increase stringing and peculiar blobs on the print.

\n\n

If your slicer changes print speed at the layer of destruction, it may also be too fast, which is related to temperatures. Simplify3D allows speed variation as well as temperature variation at selected layers, but it requires deliberate action on the part of the operator.

\n" }, { "Id": "10636", "CreationDate": "2019-07-20T15:43:58.080", "Body": "

Please Note: This question is not about the design. It's about deciding print orientation after the design.

\n\n

I have a small, but complex piece which I need to print. Here are two images of different orientation for you:

\n\n

\"enter

\n\n

\"enter

\n\n

No matter how I orient it, it will require a support structure. Any which way I print it, I believe there will be pros/cons to doing so. My question is, Is there a thought process for how to orient the part for printing? What are some of the things to consider when deciding print orientation?

\n\n

Note-1: For a size reference of the part, looking at the second image, it is approximately 60 mm from the top of the long bottom part with the two \"claws\" point down, to the top of the vertical piece which has the two larger chamfered holes in it. In the same image, the left part will be at the bottom when put into use, though will be suspended (the chamfered holes will have wood screws in them, with a block of wood on the other side from the chamfers.

\n\n

Note-2: For this example, I will be using Priline PLA filament on an Anet-A8 printer.

\n", "Title": "How to decide print orientation?", "Tags": "|3d-design|print-orientation|", "Answer": "

(Love the question and here is my 2 cents).

\n\n

Firstly, you want to minimize supports. Even if you have dissolvable supports, you would still want to minimize the usage.

\n\n

For Example:

\n\n

\"Smartphone

\n\n

At first glance of the finished object, it is not obvious at what angle it was printed. Upon close inspection the overhang in the part is designed such that it can be printed without supports. This brings me to my first point:

\n\n

Design with fabrication in mind

\n\n

I've often designed myself into a corner with parts that are complex and are impossible to print properly; or they use up too much filament in the supports. To this end I try to think about the shadow that the part will cast on the bed if there was a light source directly over head. I often orient the part so that it will

\n\n

Cast the smallest possible shadow.

\n\n

Then there are the features. Does the hole need to be round? Does this flange need to be strong? If so then I try to ensure that the feature is oriented in the XY plane as much as possible, because the Z axis is the weakest. Therefore if you have a hole, and it needs to be strong, then it should be printed perpendicular to the Z axis.

\n" }, { "Id": "10641", "CreationDate": "2019-07-21T01:48:04.343", "Body": "

I have a da Vinci miniMaker and am trying to find a way to get XYZware on my computer which is running Ubuntu Linux 19.04. On the XYZprinting website I found references to a version build for Ubuntu, but it was an outdated version with no download link. If anyone knows any way to print to any XYZ 3D printed on Ubuntu, it would be appreciated if you could let me know.

\n", "Title": "Using a XYZ printer on Linux", "Tags": "|software|linux|", "Answer": "

XYZprinting is a company known for vendor lock-in (even filament), so it's not surprising that their software's Linux support is bad, but apparently it is possible to ditch their XYZware and get it to print gcode from whatever slicer you like. Check out the miniMover repository:

\n\n
\n

This is a project that lets you send GCode to an XYZ da Vinci prniter. It will also convert from a .3w file to .gcode and back, and can monitor and setup the printer as well. This works with newer printers that implement the version 3 serial protocol. These include the Nano, Mini W, miniMaker and Jr. line of printers among others.

\n
\n\n

You'll also need to setup a profile for your printer in a slicer (I'd recommend Ultimaker Cura), but it looks like the thread linked from the README has information on settings needed.

\n" }, { "Id": "10643", "CreationDate": "2019-07-21T02:57:43.977", "Body": "

In the RAMPS v1.4 board, it seems like the X, Y, and Z motors all use at least one analog pin, while the extruder motors are entirely digital. See just the top left block of this photo:

\n\n

\"RAMPS

\n\n

Is there any advantage to using analog vs. digital pins in the motors?

\n\n

As far as I can tell the traditional thermistors have to be have an Analog pin input because of the way they are read and processed. The MOSFETs for the heated bed and hot ends (and fans) should be digital because of the way transistors work. The end-stops are also digital because they are on-off switches.

\n\n

But besides that, I can't tell if there is any other requirements or advantages to using some pins in analog / digital, especially for the motors which have seemingly contradictory pin assignments.

\n\n

There are also three digital pins I don't recognize: MISO, MOSI, and SCK.

\n\n

Right now I'm looking to add 2 new motors using 8 total pins (6 Digital pins, 2 for ground). It seems to me I could use any combination of the D pin available in the image:

\n\n

\"Pin

\n", "Title": "Digital vs. analog pins", "Tags": "|ramps-1.4|ramps|", "Answer": "

\"Analog pin\" is misleading nomenclature. Every pin is a digtal pin, but some digital pins happen to also be connected to the analog-to-digital converter and can thus also handle measuring analog signals.

\n\n

Every analog pin can also serve as a perfectly good digital pin. The reverse is not true; if something requires an analog pin you cannot use a digital pin. The only things that require analog pins are the thermistors, for the rest (motors, MOSFETS, endstops,...) you can use either a digital pin or an analog pin.

\n" }, { "Id": "10647", "CreationDate": "2019-07-22T03:57:56.643", "Body": "

I'd like to add an extra motor to my board and I'm not sure where I went wrong. The motor will be used to spin a rotating wheel/carriage of potential hot ends to switch to. Because it's just a motor it doesn't need a heatrod or a temperature sensor.

\n\n

I had just a MKS_BASE 1.0 board, so I purchased a RAMPS 1.4 board from Ebay to be its extender.

\n\n

\"RAMPS

\n\n

(( **Warning ** this board is cheap because it was improperly produced and is a fire hazard: https://reprap.org/wiki/RAMPS_1.4 . I recommend using a CNC shield instead ))

\n\n

This red board is meant to fit an Arduino Mega, but I figure I can use the extra pins on the MKS_BASE1.0 and connect them with jumper wire to the RAMPS 1.4 board. It made sense in case I want to add other things to the original MKS_BASE 1.0 board (like more hot end heater cartridges).

\n\n

I connected the 5V and one GND pin from my MKS_BASE 1.0. I also connected some of the SERVOS pins from the MKS_BASE 1.0: D37 is the 'Dir', D35 is the 'Step', and D17 is the 'Enable'. I also connected the 12V power supply to the RAMPS 1.4 board too.

\n\n

\"RAMPS

\n\n

When it came time to modify Marlin everything was a bit annoying because although Marlin makes it easy to add more extruders, adding just motors is a little more difficult. I had to change the number of extruders to be 3 (from dual extrusion to dual extrusion + extra motor), enable an extra temperature pin (which i am leaving empty) and also modify the pins.h file.

\n\n

I probably wouldn't have had simulate this motor as an extruder if I knew the raw Arduino commands for spinning a motor using calls to D37, D35, and D17, so I figured simulating an extruder would be better, but now I'm second-guessing that decision.

\n\n

Here's my modification to pins.h:

\n\n
#define E2_STEP_PIN 35\n#define E2_DIR_PIN 37\n#define E2_ENABLE_PIN 17\n\n#define HEATER_2_PIN 17  \n\n//#define TEMP_SENSOR_2 3 in Configuration.h\n\n#define TEMP_2_PIN 3\n\n// Marlin 0-indexes these pins, so \"2\" is actually for the \"3\"rd extruder\n\n
\n\n

First thing I have to do is allow for cold extrusions by using M302 S-80. The other (real) extruder motors will all move after this command, so I have that part working.. .

\n\n

In Repetier-Host I am just selecting Extruder 3 and trying to \"push filament\" through it but the motor isn't moving. I'm using an A4988 stepper driver on a Kysan 1124090. Actually, I did this whole process with two motors because I wasn't sure whether the hardware itself would be an issue, so with another set of pins I'm using a Suncor Motor and it also doesn't respond and I also don't know why.

\n\n

It would be really helpful to debug if I could run a single G-code command just to get the motor running at a speed, and take that out of the equation. it doesn't have to be a command to an \"extruder\" but just a command to a pin out, like M42 D35 S100 (but I don't know the raw command for just testing a motor's connections).

\n", "Title": "Where did I go wrong by trying to add an extra motor to my board?", "Tags": "|troubleshooting|ramps-1.4|wiring|", "Answer": "\n\n
    #define E4_STEP_PIN    4 // D4\n    #define E4_DIR_PIN     5//  D5\n    #define E4_ENABLE_PIN  63// D63\n
\n" }, { "Id": "10656", "CreationDate": "2019-07-22T20:16:19.893", "Body": "

I just recently got an Ender 3 and have been trying to learn the do\u2019s and don\u2019ts. My first model looked like it had under extrusion and stringing, but my second model looks like it has frequent \u201cscratches\u201d along the sides. I printed it with two materials, a white sample PLA that came with, and a black Sunlu PLA.

\n\n

\"Print

\n\n

I could use some insight on how to improve it, I\u2019m seeking to make some fairly smooth looking models similar to Fat Dragon games.

\n", "Title": "Models are printing with a scratch on the side", "Tags": "|print-quality|ultimaker-cura|pla|troubleshooting|creality-ender-3|", "Answer": "

The vertical lines you are referring too are the result of ringing or vibration. This is commonly encountered with high print speeds, high acceleration values or if a sudden change in direction takes place.

\n\n

This can be solved by printing slower, decreasing acceleration values and checking for mechanical issues. E.g. the belts may be too flexible (or contain a tension spring) or there may be a loose part somewhere.

\n" }, { "Id": "10659", "CreationDate": "2019-07-22T21:46:15.727", "Body": "

I would like to know if it is possible to change the rollers for linear rails, just replacing or some calculation should be done and take some precautions.

\n\n

In my case, I have a Core XY DIY printer.

\n\n

\"1\"

\n\n

\"2\"

\n\n

I was reading about need to migrate of 8 Bits to 32 Bits, because for get better resolution, results and should be update more things, like add autolevel, etc.

\n", "Title": "Changes rods for linear rails", "Tags": "|diy-3d-printer|", "Answer": "

I've seen mods for the Hypercube (and Hypercube Evolution) using linear rails (e.g. this mod), so yes that is possible. You need to redesign a few parts and make sure you have the correct length, but that shouldn't be too difficult.

\n" }, { "Id": "10669", "CreationDate": "2019-07-23T21:52:28.887", "Body": "

Recently I have been getting some layer shifting starting at layer one. I have had layer shifting at higher layers due to various reasons but mainly for the belts being too loose. But now I am reading that layer shifting can also be caused by belts being too tight.

\n\n

The RepRap wiki page for layer shifting simply gives the mechanical reason for this as \"binding\". Can anyone explain what binding means? I thought it meant that the rails were crashing into something but apparently it doesn't. Then I thought it meant that the X and Y axes weren't perfectly perpendicular.

\n\n

Does it mean that the \"teeth\" of the belts stay \"stuck\" to the gear for too long when moving in one direction? Why would this happen in one direction and not the other? Because the pulleys/gears are at different heights? Or just because of the belts being tighter? Or one of these reasons?

\n\n

Just trying to understand what its happening so I can debug it for my particular 3D printer.

\n", "Title": "What is stepper motor binding? (When belts are too tight)", "Tags": "|troubleshooting|", "Answer": "

I think the RepRap wiki is using the word \"binding\", which translates to \"stick together or cause to stick together in a single mass\" (from Google dictionary), to indicate that some sort of friction is experienced (as you experience when things are sticking together).

\n\n

When there is too much tension in the belt, pulleys and bearings experience a larger radial force stressing the balls of the bearings and pulley shafts. This causes extra friction for the stepper motor to overcome (as the friction force, tangential, is related to the radial force); this means that the stepper has to work harder and can skip steps (for more insight please read below).

\n\n
\n\n

While ball bearings are used to reduce friction (opposed to a bush bearing), each ball has a little friction from a couple of sources according to this reference:

\n\n
\n

The sources of this friction are: slight deformation of the rolling elements and raceways under load, sliding friction of the rolling elements against the cage and guiding surfaces.

\n
\n\n

These effects are generally captured in a single friction coefficient called \"μ\". The relation between friction force (tangential) and bearing loading (radial) is written by $$P_{friction}=P_{load} \\times \\mu$$ so the higher the belt tension ($P_{load}$), the higher the frictional force ($P_{friction}$), the harder the stepper has to work.

\n" }, { "Id": "10680", "CreationDate": "2019-07-25T01:21:10.733", "Body": "

See the pictures below. I have a severe under extrusion when the printer starts the outer wall, which is resolved by the time it finishes the outer wall. It starts the layer in the same place every time, so it results in this vertical line, on one side of which is fine (where it finishes the layer) and the other side has bad gaps and the wall is much thinner.

\n

In this picture, the problem is on the outer wall (see red outer line, the print head is moving counter clockwise.

\n

\"G-Code

\n

And here is the print showing the issue. Just fine on one side, but terrible on the other, precisely where it starts the layer. Strangely, this only occurs on the layers with infill. The top layers seem fine (despite starting in the same place). I have disabled retraction with no effect.

\n

Here you can see it start at the tip and get gradually better as it progresses.\n\"enter

\n

Here you can see that by the time it finishes, it's just fine, and also what a contrast is between the start and end. That should be a flush edge, there.\n\"enter

\n

Another view\n\"enter

\n

Is there some setting that I should be tweaking? I've exhausted my own ideas of different tweaks to no avail.

\n

The printer is a Monoprice Maker Select V2. I'm using Ultimaker Cura 4.1.0.

\n
\n\n

This doesn't happen on layer changes. Although it does line up with the Z seam, you can see from the G-code visualization below that the outer wall is the very last thing it does. The issue occurs at the start of the outer wall (still on the same layer) that it has the issue, but by the time it ends the wall (just before switching layers) it is fine.

\n

I've tweaked a few other settings, one by one, and seeing if any have any effect. So far, not really:

\n\n

What have I done since...

\n

I reset all settings in Cura to a default "Draft" setting and then set layer height to 0.24\u00a0mm, and turned off Infill. Then I have tried prints with different settings for "Retract Before Outer Wall" and printing temperature.

\n

Here are those results:

\n

These pics seem to suggest a very clear lag in extrusion. 1 and 2 are different temps. 2 and 3 are different retraction.

\n

205\u00a0\u00b0C, 0\u00a0% Infill, Retract Before Outer Wall: Off\n\"205

\n

195\u00a0\u00b0C, 0\u00a0% Infill, Retract Before Outer Wall: Off\n\"195

\n

195\u00a0\u00b0C, 0\u00a0% Infill, Retract Before Outer Wall: On\nNote: the retraction setting resulted in a noticeable pause before printing the outer wall. Retraction distance is 6.5\u00a0mm, and this is not a Bowden fed device.\n\"195

\n

There doesn't seem to be anything strange about the G-code, either. Here are the G0 travels just before the outer wall followed by the wall.

\n
...\nG0 F7200 X106.319 Y93.413\nG0 X106.26 Y93.909\nG0 X107.213 Y93.658\nG0 X107.8 Y92.542\nG0 X107.286 Y90.844\nG0 X107.509 Y90.394\n\n; (outer wall of outside)\nG1 F1328 X107.985 Y90.707 E116.98713\nG1 X108.38 Y91.128 E117.01098\nG1 X108.658 Y91.623 E117.03444\nG1 X108.813 Y92.18 E117.05833\nG1 X108.832 Y92.751 E117.08193\nG1 X108.713 Y93.315 E117.10575\nG1 X108.463 Y93.837 E117.12966\n; (first curve complete, on to straightaway)\nG1 X99.631 Y107.716 E117.80936\nG1 X98.912 Y108.59 E117.85612\n...\n
\n

Even more done...

\n

These are retraction off, and 205\u00a0\u00b0C

\n

If I set the "print speed" in Cura to 20\u00a0mm/s (normally 60\u00a0mm/s), the outer wall speed is reduced from 30\u00a0mm/s to 10\u00a0mm/s. The result is quite good.\n\"enter

\n

If I leave the "print speed" at 60\u00a0mm/s and adjust only the outer wall to 10\u00a0mm/s, it's still quite good.\n\"enter

\n

So it seems like an acceleration thing. If I can figure out how to get it to slow down in just the right spots or compensate in some way, then perhaps I can make this problem go away with minimal sacrifice in total speed.

\n", "Title": "Under extrusion when starting outer wall", "Tags": "|print-quality|", "Answer": "

What worked for me was enabling outer wall wipe distance to around 3\u00a0mm, that's all that I enabled and it instantly fixed it, no prime amount or anything, just outer wall wipe distance

\n" }, { "Id": "10681", "CreationDate": "2019-07-25T02:54:18.877", "Body": "

I have a 3D printer hooked up with Fusion 360. I have a 3D drawing in Revit. What format should I export to in order get from Revit to Fusion 360?

\n", "Title": "Export Revit file for Fusion 360 3D Print", "Tags": "|fusion360|autodesk|", "Answer": "

In Revit

\n\n

Use STL Exporter 2019 to customize and export your Revit model to an STL file.

\n\n
    \n
  1. Click Add-in tab -> STL Exporter panel -> STL Exporter.
  2. \n
  3. On the General tab, specify your STL file options.
  4. \n
  5. On the Categories tab, specify which element categories will export to the STL file.
  6. \n
  7. Click Save to create the STL file.
  8. \n
\n\n

In Fusion 360

\n\n
    \n
  1. Click on a valid project then click the Upload button. Then, click the Select files button and navigate to the STL you wish to import.\n\"upload
  2. \n
  3. After selecting the STL file you want to convert, click the Upload button, this time the one in the lower-right corner of the file upload window. Then, the STL file you selected will be imported into Fusion 360. The process can take a few minutes.\n\"browse
  4. \n
\n" }, { "Id": "10683", "CreationDate": "2019-07-25T08:46:26.333", "Body": "

I'm printing on an Ender 5 with the default flex/magnetic build surface.\nI read that PLA and PETG may sometimes be printed without any bed heating at all and also that bed heating is the main contributor to the power consumption of a printer.

\n\n

As I do see that bed heating definitely helps with the first layer adhesion I did not want to turn it off completely, but I did start experimenting with turning off bed heating after all solid bottom layers have printed (using the ChangeAtZ script in Cura) and so far I haven't seen any negative effects, especially no warping (I am usually printing with a brim or raft; I think that might also help in that regard).

\n\n

Am I missing something? Why is anyone keeping the bed heated for an entire print?

\n", "Title": "Why keep the bed heated after initial layer(s) with PLA (or PETG)?", "Tags": "|pla|heated-bed|petg|creality-ender-5|", "Answer": "

its up to you do some own test and see how its working for you:

\n\n

\"enter

\n" }, { "Id": "10690", "CreationDate": "2019-07-25T18:04:30.363", "Body": "

I attempted upgrading my Ender 3 firmware to Marlin 1.1.9, and due to inability to find the directions about the Sanguino variant pin layout file, stupidly built with the \"standard\" (Arduino) file. Various stuff didn't work, and I figured out my mistake and rebuilt, but the extruder motor does not respond even after flashing back to the original firmware.

\n\n

Did I destroy the stepper driver with incorrect pin mapping? (Aside: if so, why just the extruder and not the others?) Is this fixable or should I just order a new motherboard?

\n", "Title": "Botched firmware upgrade - did I destroy the stepper drivers?", "Tags": "|marlin|creality-ender-3|firmware|stepper-driver|", "Answer": "

About 15 minutes after posting this, it started working again. Either the driver had badly overheated and recovered after cooling down, or was never messed up and the firmware was just refusing to operate it with the nozzle cold. For the latter possibility, the stock firmware never had that behavior before, but maybe it's possible that some saved configuration to ignore nozzle-cold got lost in the flashing?

\n" }, { "Id": "10695", "CreationDate": "2019-07-26T09:06:57.497", "Body": "

I'm trying to verify the correct hotend heater for a Reprap x400 Pro V3. I know a 12 V 40 W doesn't work. A working heater measures 4.4 Ω, so I'm guessing it is a 12 V 30 W heater.

\n\n

When I tried replacing a non-functioning hotend heater on a German Reprap X400 V3 with a new 12V 40W heater that heater also wouldn't heat. My guess is the circuitry shut off due to overcurrent. The thermistor temperature didn't change from room temperature, and the heater was not hot to the touch. Afterward, I plugged in the heater measuring 4.4 Ω and it worked.

\n\n

The 100k sensors worked properly. A functioning heater resistance measured 4.4 Ω. The non-functioning heater measured 3.6 Ω. The new 12 V 40 W heater measured 2.6 Ω. Apparently, this model Reprap won't apply power to the heater if the resistance is wrong. I'm assuming this model needs a 12 V 30 W heater, but am trying to confirm this since I can't find a parts bill of material for this model. Used the following equation to estimate heater resistance (voltage^2)/power = resistance. This gives 4.8 Ω for 12 V 30 W and 3.6 Ω for 12 V 40 W.

\n", "Title": "Proper hotend heater for Reprap x400 Pro V3", "Tags": "|diy-3d-printer|hotend|", "Answer": "

\"enter

\n\n

The part causing the hot point failure was the connector for the heater and sensor. The connector showed no sign of burns on the heater pins, but the square tubes (see attached image) accommodating pins from the other connector had started to straighten out toward its original sheet metal state due to heating from current. Bending the tubes back in place allowed the connector to work temporarily and confirmed the failure cause.

\n\n

Details: After getting a 12 V 30 W heater, it also didn\u2019t work. The hot point had stopped working because a wire brush had shorted across the heater terminals. Because a spare hot point worked, the failure wasn\u2019t in the circuitry. The spare was a 12 V 30 W heater (from resistance measurements), while the heater on the failing hot point was a 12 V 40 W heater (from resistance measurements). This made me suspect that the heater resistance had shifted, and the circuitry shut off the heater. However, apparently both 12 V 30 W and 12 V 40 W heaters work on RepRap x400 V3. The brief short caused the bent sheet metal of the connector to start straightening back out of the bend.

\n" }, { "Id": "10724", "CreationDate": "2019-07-30T19:14:31.660", "Body": "

I converted the STL file through an online converter https://www.cadforum.cz/catalog_en/stl2dwg.asp

\n\n

(I work in the program autocad 2017)\nNow I open the resulting DXF and I don\u2019t know how to work with it. How do I merge polygons into a whole model?

\n", "Title": "Getting started with converted STL to DXF", "Tags": "|3d-models|stl|", "Answer": "

I figured it out. Need to go: Network -> conversion to surface

\n\n

Then we get a surface that is already easy to work with.

\n" }, { "Id": "10728", "CreationDate": "2019-07-31T19:08:50.730", "Body": "

I was looking at a BLTouch probe, and saw the recent question about one with intermittent failure. That was kind of scary, where I might spend a bunch of money for something that would make my printer harder to use, rather than easier.

\n\n

But I was curious. I don't have any personal experience using these, but it seems like the probe would only be used during an initial level and when homing the Z axis. Thus, as long as the probe works most of the time, and failure to home properly doesn't damage your nozzle or print bed, maybe this kind of thing isn't too bad.

\n\n

Is this assessment of how the probe functions (that it's not really used after the start of a print) accurate?

\n\n

In the spirit of stack exchange, I'm not interested in the subjective issue of nozzle or print bed damage; I'm only asking about the objective assessment of how a probe is used by the printer.

\n", "Title": "Is a BLTouch probe used during a print after the first layer?", "Tags": "|z-probe|bltouch|", "Answer": "

Your assessment is correct, after probing before printing it deploys several times, after that it is stowed till the end of the print unless you call for deployment.

\n\n
\n\n

The probe is used by G-code command G29; this command is used by a few printer firmwares to perform a bed probing sequence where, depending on the options set in the firmware, the surface of the bed is scanned by deploying the sensor at various locations. From these measurements, the firmware will determine the shape or tilt of the bed to compensate for deviations in Z height during the first couple of layers or millimeters of the print. Basically, it will keep your nozzle (approximately, as it is based on calculations) at the same distance all over the bed area.

\n\n

Once the print starts after the G29 command, the sensor is stowed until the next print calls the G29 command (or M280 with arguments to deploy on Marlin/Duet/Smoothieware, or M340 on Repetier firmware).

\n" }, { "Id": "10738", "CreationDate": "2019-08-01T13:28:42.133", "Body": "

I was having some issues with printing, most noticeably in this picture:

\n

\"Noticeable

\n

The layers are very noticeable and sometimes have gaps, and the overhangs don't print very well (although the former is more of an issue). I just calibrated my E-steps so I don't think that is the issue. It was doing the same thing before I upgraded anything (i.e., I had issues on stock hardware).

\n

My printer is an Ender 3 with the metal extruder upgrade (which replaces the plastic parts as seen here), an E3D v6, printed fan duct (Bullseye), glass bed, BLTouch, and vanilla Marlin. Pictures of it are also in the below album. The printed upgrades were printed on a Prusa MK3S and don't have the same issue.

\n

I am using Hatchbox 1.75\u00a0mm gray PLA, printed at 215\u00a0\u00b0C with my bed at 60\u00a0\u00b0C. I am using Ultimaker Cura 4.1 but was also having the problems on an older version of Ultimaker Cura (maybe 3.6, but I can't remember which it was). The problems also existed with some Hatchbox 1.75\u00a0mm black PLA but I used the same roll on my Prusa MK3S without any issues, so I'm not sure if filament could be the cause (although it is a different printer so it's still a possibility).

\n

I have tried at different printing speeds and the problem still persists.

\n

I also recently tried varying the temperature during printing (first up to 222\u00a0\u00b0C then down to 200\u00a0\u00b0C) with no noticeable difference.

\n

Extra pictures here.

\n

Model is part of Printable Scenery's sorcerer tower.

\n", "Title": "Inconsistent Layer Issues", "Tags": "|print-quality|creality-ender-3|", "Answer": "

The main issue here (the gaps between layers) was solved by reducing combing.

\n\n

Combing was enabled without a limit on the range so a max combing distance of 10 mm was introduced. This prevented too much filament from oozing out during travels.

\n\n

The oozing filament was causing nothing to come out of the nozzle at the beginning of an extrusion, thus creating the gaps that were consistent in location.

\n" }, { "Id": "10742", "CreationDate": "2019-08-02T06:19:05.233", "Body": "

I came across a new Steel infused PLA from Colorfabb. On the store page someone had asked, what happens if it was exposed to water, would it rust. I am actually, not sure what would happen. I am interested what the effects of leaving in the elements a 3D printed object made of the 2 most common type of fused materials.

\n\n
    \n
  1. Wood Infill
  2. \n
  3. Metal Infill (not stainless steel)
  4. \n
\n\n

Would it be preserved by the PLA coating it, or would, over time, rust and dissolve? Would the wood last forever, or will the print get discolored and become mulch?

\n", "Title": "What are the effects of the elements on 3D printed objects made with \"infused\" PLA", "Tags": "|filament|metal-parts|", "Answer": "

The particles on the outmost part of the print which are exposed to air will rust. The ones inside will still be protected somewhat by the PLA. The rust color/effect is part of the idea, since it give certain prints an old look.

\n\n

\"ProtoPasta

\n\n

https://www.proto-pasta.com/products/magnetic-iron-pla

\n" }, { "Id": "10754", "CreationDate": "2019-08-04T22:26:06.673", "Body": "

I have an Ender 5 with an aluminum extruder that was printing beautifully for a few days after building it. I received a Capricorn bowden tube and new couplings as a gift, and replaced those, and now I have stringing issues I can't solve.

\n\n

I've been all over the map with settings (temps, speeds, z-hop, retraction, etc. etc.), I've swapped the tubes, the couplings, the filament, I've checked for clogs, and put clamps on the collars, but no matter what I do, the tube shifts in and out during retractions by about .25 - .5 mm. I suspect that has something to do with the stringing, as nothing else has solved it.

\n\n

Any suggestions are appreciated. I haven't made adjustments to the extruder, but I don't know if that's possible or necessary. I've never had a system using the bowden, so that part of 3d printing is new to me. I'm using 3d Solutech PLA.

\n", "Title": "Ender 5: Bowden tube shifts in/out of extruder during retractions", "Tags": "|pla|extruder|bowden|retraction|creality-ender-5|", "Answer": "

If it's just the pressure fitting on the brass coupler that shifts in/out, or the PTFE tube shifting slightly in/out but remaining in the coupler, that shouldn't be a problem unless it's so loose it can come out entirely. The blue clips which should have come with the printer are intended to stop this motion or at least provide enough pressure to prevent it from falling all the way out.

\n\n

Regarding the small 0.25 - 0.5 mm of play, effectively it just means you lose that much length from your retractions, since part of the retraction goes into pulling the tube back into the coupler. When the retraction is reversed, the effective filament position should go back to what it was before the retraction. As long as your retraction amount is sufficient after the loss, it should not cause stringing.

\n\n

You probably need to look for another cause of your stringing.

\n" }, { "Id": "10762", "CreationDate": "2019-08-06T15:12:19.663", "Body": "

In some situations it useful to have a material in which air passes but water, and water vapor, does not. Printing cell membranes or cases for something that must be immune to humidity changes (like something that holds a highly oxidative metal) are two examples.

\n\n

Teflon (PTFE) is a material that has this property. But PTFE is horrendous to 3D print.

\n\n

Silicone is another material that has this property. 3D printing silicone can be done with a syringe mechanism. But there are also thermoplastic rubbers (e.g. TPU, TPE) that are designed to mimic Silicone.

\n\n

Are there any filaments on the market which naturally form holes with the porous diameter for this feature? Is there any way to manipulate the diameter of this hole, even starting from pellets? Are some filaments more likely than others to have this feature?

\n", "Title": "What filaments allow air to pass but not water vapor?", "Tags": "|filament|print-material|material|filament-choice|", "Answer": "

It is difficult enough to make 3D-printed objects water-tight (unless you paint them). Making them permeable to air but not water vapour is too big an ask, I think.

\n" }, { "Id": "10765", "CreationDate": "2019-08-06T20:01:09.120", "Body": "

Going from 12V electronics to 24V produces a reduction in the overall amount of heat generated in the electronics (that are not intended to get hot). These include the motors, drivers, mosfets and pcb traces.

\n\n

Why then given that 48V power supplies and electronic components cost exactly the same as their 24v counterparts, are there no 48V 3d printers?

\n\n

Is there a saftey aspect to 48V that needs to be considered?

\n\n

EDIT: Yes 48V fans cost a little more but it's not exorbitant. It seems odd to move from 12 to 24 to get half as much heat, when you could move from 12 to 48v and get a quarter as much heat.

\n", "Title": "Can 3d printers be operated at voltages above 24 volts?", "Tags": "|electronics|", "Answer": "

I believe the main issue is the voltage regulators and capacitors on your standard print main board are not rated for 48V. You can find some CNC shields that can handle up to 36V but these are lacking 3d printer features such as heater ports and only have 4 stepper slots.

\n" }, { "Id": "10768", "CreationDate": "2019-08-07T01:33:39.507", "Body": "

Recently my Ender 3 has developed a problem of skipping Y steps (layer shift in the Y direction), and it seems to be getting progressively worse. I'd never experienced layer shift before a few weeks ago when I tried some extreme accelerations (up to 3000 mm/s\u00b2), which mostly worked fine, but I stopped after getting a couple layer shifts, and now have X acceleration at 2000 mm/s\u00b2 and Y acceleration back at the default of 500 mm/s\u00b2.

\n\n

When the layer shifts happen, I usually hear \"grinding\" sounds that seem like they're coming from friction of the nozzle moving over the already-deposited material. Bed leveling/height is correct though.

\n\n

Magnitude of the layer shifts varies between 1 mm and 3 mm. They're all in the positive direction (layers shifts towards the back of the bed).

\n\n

Speeds are reasonable, 30 mm/s for outer walls, 60 mm/s for inner and infill, 120 mm/s for travel.

\n\n

Is there a likely mechanical or electrical cause for the onset of skipped steps/layer shift?

\n\n

Is it possible that the high X acceleration is what's allowing the very high travel speed to be achieved, and that such high travel speed is putting the nozzle on top of cold material that it drags on without giving it a chance to soften it first? Or is the printer underpowered such that there's insufficient current to provide the necessary Y motor torque while the X is under high acceleration? Lowering the X acceleration back to default (500 mm/s\u00b2) does seem to make it less likely or less severe, but it's still happening.

\n", "Title": "Ender 3 has developed a tendancy to skip Y steps", "Tags": "|creality-ender-3|acceleration|layer-shifting|", "Answer": "

While I first experienced layer shifts a couple times before upgrading, only while using extreme acceleration settings (3000 mm/s\u00b2 is fairly extreme for moving the bed), their regular occurrance, which is near-constant with some files, seems to have started after I upgraded the firmware to Marlin 1.1.9. Reverting to the manufacturer's firmware, with no mechanical changes to the printer whatsoever, has the same file printing now with no layer shifts. So I think it's safe to say that this is a firmware problem - either a bug in Marlin, or a problem with the Ender 3 configuration headers for it. The bug tracker suggests this kind of thing is an ongoing issue; here are two among a large number of related bug tracker threads:

\n\n\n\n

I'll try the latest 2.0.x, and/or turning features off one at a time in 1.1.9, and see if I can isolate the cause, then open a new question about bugs in Marlin causing layer shifts if needed.

\n" }, { "Id": "10772", "CreationDate": "2019-08-07T23:16:14.850", "Body": "

Since upgrading from the manufacturer's firmare to recent Marlin (first 1.1.9, then 2.0.x git HEAD), I've been having significant model-dependent (didn't happen for first few things I tried to print; other prints reproduce it reliably) layer shifting on the Y axis. It's always in the positive Y direction, usually by 1-3 mm at a time.

\n\n

\"aborted

\n\n

I first suspected mechanical problems (see: Ender 3 has developed a tendancy to skip Y steps), but reverting the firmware confirmed that it's a firmware problem.

\n\n

A number of bug tracker entries for Marlin indicate that there are significant ongoing layer shift problems:

\n\n\n\n

But most end up getting closed. Some people say reverting to 1.1.9 from 2.0.x helped; others say 1.1.9 is EOL and to use 2.0.x. Is there a solution? How do I get a working version of Marlin (new enough to have Linear Advance 1.5, which was my motivation for upgrading) on Ender 3?

\n", "Title": "Y-axis layer shifts with recent Marlin on Ender 3", "Tags": "|marlin|creality-ender-3|firmware|layer-shifting|", "Answer": "

Buried in a bug tracker entry for a seemingly unrelated issue, I found what seems to be the solution:

\n\n
\n

@johanmga, I've discovered the velocity jumps here come from using \"classic jerk\" computations, and happen whether or not s-curve acceleration is enabled.

\n \n

Try uncommenting //#define JUNCTION_DEVIATION in Configuration_adv.h; that will disable classic jerk and use the junction deviation math instead.

\n \n

Also for reference, issue #12403 is actively tracking the layer shift issue (please comment there too if you have more information on problems/solutions), and #12540 is has more information on classic jerk vs. junction deviation.

\n
\n\n

https://github.com/MarlinFirmware/Marlin/issues/12491#issuecomment-443464603

\n\n

It seems that the \"classic jerk\" code no longer works and produces erroneous motion to the steppers under some conditions (dependent on G-code contents), whereas the Ender 3 default configuration file is still using it.

\n\n

With the default JUNCTION_DEVIATION_MM of 0.02, things worked right away, but printing was so slow that I could plausibly attribute it working just to the slowness. But increasing it up to 0.1 via the menus got the speed closer to what I'm used to, and it's still printing with no layer shifts.

\n\n

I've posted my results in the tracker in hopes that they'll lead to discovery of the root cause: https://github.com/MarlinFirmware/Marlin/issues/12403#issuecomment-519305409

\n\n

Another issue on the tracker seems to have more detail:

\n\n

https://github.com/MarlinFirmware/Marlin/issues/12540#issuecomment-442793326

\n" }, { "Id": "10785", "CreationDate": "2019-08-11T11:17:25.210", "Body": "

I just got my first 3D printer (Creality Ender 3) on Friday, 2 days ago. It works great, but for some reason I'm getting a lot of stringing on my prints, especially the ones where the extruder head has to move a long distance between columns/posts, etc.

\n\n

I'm using Hatchbox \"True White\" PLA, which has a recommended temperature range of 180-210 °C. I've tried printing at 200, 190, and 185 °C and didn't see much improvement. I've also made sure I've enabled the 'retract' setting in the slicer (4.5 mm) and verified the printer is retracting when it should.

\n\n

I'm not sure what else I can try... any suggestions?

\n", "Title": "How to fight lots of stringing with PLA at low temperature (185 \u00b0C)?", "Tags": "|print-quality|pla|creality-ender-3|", "Answer": "

The way to optimise retraction is to use this retraction optimisation tool, which tests various retraction distances and speeds.

\n

Remember to perform this calibration AFTER you set pressure or linear advance, which has a higher priority.

\n

\"enter

\n

You will be able to pick the settings which work the best for that filament brand and type. You will have to do it again if you switch material or brand.

\n" }, { "Id": "10792", "CreationDate": "2019-08-11T20:18:15.967", "Body": "

I heard about the certification program from autodesk website.

\n\n

How can I prepare and write this certification exam as a student? I already started using fusion 360 about a month.

\n", "Title": "How can I become a certified Fusion 360 user?", "Tags": "|3d-design|fusion360|autodesk|", "Answer": "

Start with registering at their website and go from there. There is a PDF of a list of things you have to have mastery in:

\n\n
\n \n
\n" }, { "Id": "10798", "CreationDate": "2019-08-12T12:03:33.250", "Body": "

Problem

\n\n

My CR-10 printer seems to be trying to print the model 4 or 5 layers too low. This means that for the first few layers, the printing nozzle is forced against the bed, preventing extrusion until the print reaches higher layers.

\n\n

Outcome

\n\n

This results in the bottom part of the print having the internal structure visible and the printing head deteriorating. I had to remove the old nozzle because it was clogged up with what I believe to be some residue that was picked up during preceding prints.

\n\n

\"A\nnote: On this print, the top part is almost-well printed. It cannot be seen in pictures, but I say \"almost\" because the well-printed part is still much thinner than expected. This is the result of my purposeful mis-leveling the bed so that the part where the printer \"homes\" is higher than the lower part. I did this to see if there was any obvious bending of the printing bed. Doesn't look like there is.

\n\n

Fix attempts

\n\n\n\n

History

\n\n

The printer used to work perfectly well and I do not remember having changed anything before the problem arose. I recently changed the nozzle and made sure to tighten it as high as possible but the Z-offset problem still persists.

\n", "Title": "Printer head too low when printing", "Tags": "|z-axis|creality-cr-10|", "Answer": "

TLDR; Make sure the bed isn't too high, the z-axis should reach the end-of-rail indicator without forcing against the bed

\n\n
\n\n

So it appears I simply misunderstood (or mis-assumed) the way my printer works.

\n\n

For some reason I believed that it was the printing head, while auto-homeing, that was defining the \"point zero\" for the z axis. Didn't realize, even after disassembled the whole head block, that there was no such mechanism built in.

\n\n

While zeroing it before starting my on-going print I noticed the \"click\" of the end-of-rail switch for the z-axis while the head was forcing against the bed, desperately trying to reach that damn switch. Then it all clicked together ...

\n\n

What happened:

\n\n

For some reason a few weeks ago I decided to level the bed by raising the z-axis by spinning the its driving wheel by hand, taking it very far from the end-of-rail stopper. Ever since then I've been leveling the bed this way, not realizing I was preventing the head from reaching end-of-rail without having to force against the bed.

\n\n

So yeah, just took the bed as far down as possible, made the printer auto-home and leveled the bed from there, as it should be done, and my print is now going very well.

\n" }, { "Id": "10799", "CreationDate": "2019-08-12T18:03:16.770", "Body": "

I am trying to print a 12 hole Ocarina I found on thingiverse. When printing I have to stop it around 25-30 layers because the edge of the shell is higher then the infill.

\n\n

\"Example

\n\n

\"Example

\n\n

G-Code of first 30 layers

\n\n

I tried changing the infill, wall size, speed and retraction settings to no avail. The settings of the example print were:

\n\n\n\n

Does anyone know what might cause this and how I can prevent it from happening?

\n\n

Any help is greatly appreciated! :)

\n", "Title": "Raised shell edges on ocarina print. RF1000 sliced with Cura Engine", "Tags": "|ultimaker-cura|pla|g-code|", "Answer": "

I think the solution in your self-answer is just a partial mitigation for an underlying problem, and your expectations for output quality are way too low.

\n\n

From the pictures in the question, there's serious overextrusion and stringing going on. The stringing could be caused by a secondary problem (bad/insufficient retraction settings), but heavy overextrusion will cause there to be extreme residual pressure left between the extruder gear and the nozzle (especially in a bowden setup, though it's not clear what printer you're using or if it has a bowden tube) that retraction will be unlikely to sufficiently relieve, so it's also a characteristic consequence of overextrusion.

\n\n

As P\u1d00\u1d1c\u029fs\u1d1b\u1d07\u02802 noted in a comment, having the filament diameter set to 1.75 mm while your printer actually uses 3 mm filament could cause this. Cura (especially CuraEngine invoked via command line rather than the GUI) is particularly bad about getting this wrong if you don't pass the options in exactly the right way.

\n\n

Note that lowering the infill percentage as you did would help get somewhat decent results with serious overextrusion, since the excess material has somewhere to go (into the unfilled part of the infill region). But you'll still be getting really bad (what I would call unacceptably bad) prints compared to what you could/should get.

\n" }, { "Id": "10820", "CreationDate": "2019-08-19T10:04:58.590", "Body": "

I've tried to remix this model: https://www.thingiverse.com/thing:90933 (Bauhaus chess set) by scaling it down and inserting little magnet holes into the pieces' underside. My SCAD file looks as follows:

\n\n
difference() { \n\n    scale([0.5,0.5,0.5]) import(\"Bauhaus2Set.stl\");\n\n    translate([ 6, 11  ,0]) cylinder(h=20.5,r=2.5,center=true,$fn=20); // WTH?\n\n    // King/Queen\n    translate([ 6, 11  ,0]) cylinder(h=3.5,r=2.5,center=true,$fn=20);\n    translate([-6, 11  ,0]) cylinder(h=3.5,r=2.5,center=true,$fn=20);\n\n    // Rooks\n    translate([ 6,-11.5,0]) cylinder(h=3.5,r=2.5,center=true,$fn=20);\n    translate([-6,-11.5,0]) cylinder(h=3.5,r=2.5,center=true,$fn=20);\n\n[...]\n
\n\n

Note the third line with \"WTH?\" - I've done quite some trial-and-error, and if I remove that line, then I don't get any subtracted holes anymore when I render the whole thing (F6). In preview (F5), the holes are always present, but in the final render, I need to include the larger subtracted cylinder or it won't work.

\n\n

The STL file seems to be fine in itself, what's going on here?

\n", "Title": "OpenSCAD weirdness with imported STL", "Tags": "|stl|openscad|", "Answer": "

I see you've accepted cmm's answer, but I'd still like to take a stab at the mechanism of the failure based on your comment on it:

\n\n
\n

Excellent explanation, thanks. The \"WTH\" cylinder is centered on the queen piece, and that has a sphere with lots of faces on top. So it looks like the \"kink\" is inside the queen where the sphere and the base cube overlap, and by accident, my trial-and-error removing of cylinders also removed just the right spot.

\n
\n\n

My guess is that if you look at the triangle set of the STL file, you'll find the interior of the queen contains the parts of the cube that are inside the sphere and the parts of the sphere that are inside the cube; they may not even be clipped to meet each other properly where they cross each other's surfaces. Sadly there are lots of tools producing invalid STL files like this. The longer cylinder is probaly sufficient to overlap with where the cube and sphere cross, forcing OpenSCAD to break down the model in that region and recompute the mesh where the components overlap. Without that recomputation, the differences likely end up interacting with just the \"sphere part\" of the STL mesh.

\n\n

This explanation also seems to be consistent with the description of the thing on thingiverse:

\n\n
\n

I much preferred the style of Bauhaus set that TeamTeaUSA's designed, but there was a lot of fiddling with the queen and the knight wouldn't print without supports. So I nestled the queen's sphere further into the body so it would print standing up, added supports for the knight and plated the whole thing.

\n
\n\n

where it looks like the creator took someone else's STL files and moved parts around to create an overlap, likely without proper tooling that could regenerate a valid mesh.

\n" }, { "Id": "10825", "CreationDate": "2019-08-20T13:23:04.320", "Body": "

I've had my anycubic kossel for a good while now, and whenever I print something, it seems like it falls out of calibration very quickly. The biggest example is that the prints tend to scale up as the print progresses, or maybe the print starts to drift, and comes out slanted. This seems to never happen in any similarly priced XYZ printers. Why is this? Do XYZ printers just have an inherent advantage over deltas? Perhaps Deltas have some precision loss?

\n", "Title": "Why are delta bots so finicky?", "Tags": "|delta|", "Answer": "

user77232's points are valid but I have done several things to mitigate these issues on my Anycubic Deltas.

\n\n

First, both of my deltas have linear slide bearings. If your's has the bearing trucks that run in the extrusion slots this will lead to less precise operation and also is a source of wear over time.

\n\n

Second, be sure that the end stop micro switches are positioned precisely and that their fasteners are tight. You may also want to use a mild strength Loctite on their fasteners. When operated at high extrusion rates all components are subject to significant vibration.

\n\n

Third, I found that the universal joints are also a significant source of play. I remedied this to some degree by placing rubber bands across the arms at both ends so that the play was minimized. The rubber bands should be wrapped fairly tightly to perform this function.

\n\n

Forth, I try to tune the belt tension so that all of the belts have the same note when plucked. There are smart phone apps that will help do this. I believe that one that I use is from Gates, a premium manufacturer of drive belts. Also, over time belts stretch so you will need to retension them periodically.

\n\n

Fifth, run through the firmware calibration process regularly for both positional accuracy and for filament extrusion and retraction variables.

\n\n

I don't use my deltas much any more since purchasing a popular XYZ type printer but it cost me three times what they did and it still has issues from time to time.

\n" }, { "Id": "10835", "CreationDate": "2019-08-21T18:12:22.117", "Body": "

During the assembly of my printer, I accidentally shorted the RAMPS 1.4 board, looking for a replacement I found the RAMPS 1.6 which would eliminate the need for a MOSFET power module.

\n\n

My question is, could I simply replace the RAMPS 1.4 for the 1.6 or should I make any other modification?

\n\n

My printer is a Prusa i3 Graber Clone running Marlin

\n", "Title": "Replacing RAMPS 1.4 for 1.6", "Tags": "|ramps-1.4|electronics|ramps|ramps-1.6|", "Answer": "

RAMPS 1.6 is very similar to RAMPS 1.4. Most, if not all, of the changes are hardware improvements, without making the core functionality different. Looking at the incremental changes between versions:

\n

To quote from RepRapWiki: RAMPS 1.5

\n
\n

The RAMPS 1.5 is a variation of the popular RAMPS boards that, like\nall other RAMPS boards, is plugged into an Arduino Mega for operation.\nThe layout of the RAMPS 1.5 is virtually identical to that of the\nRAMPS 1.4. The only real difference between the two are the MOSFETs\nand the fuses.

\n
\n

To quote from RepRapWiki: RAMPS 1.6

\n
\n

The RAMPS 1.6 is the second RAMPS iteration released by\nBIQU/BIGTREETECH. It replaces the original green power connector with\na pair of screw terminals, adds a larger heatsink over the MOSFETS,\nand has a larger bed MOSFET. It maintains the surface-mounted fuses\nand flush MOSFETS of the RAMPS 1.5.

\n

Also, the positions of the D1 and D2 diodes have been swapped from the\npositions in RAMPS 1.4, the D1 diode is now the diode closest to fuse\nF2. The same is true for RAMPS 1.5, also manufactured by\nBIQU/BIGTREETECH.

\n
\n

So, in short, yes, the RAMPS 1.6 should be a direct plug in replacement for the RAMPS 1.4.

\n

That said, you may be able to salvage your RAMPS 1.4, if you can locate the part that has given up the "magic smoke". It could just be a blown capacitor, or diode. There have been a number of questions relating to that, so it might be worth searching for answers here, on SE.3DP1.

\n

It is probably also worth reading RAMPS 1.4, 1.5 or 1.6?

\n
\n

Additional links

\n

An interesting video, where a variety of RAMPS 1.4 boards are compared, is the first of a series, RAMPS series 1 of 5 - Intro and board comparisons - although beware the loud music...

\n

There is also a RAMPS 1.6 Plus, and a RAMPS 1.4.2 which might be worth considering. I wrote up about these, briefly, (disclosure) in my personal blog, a couple of years ago: 3D Printer control boards.

\n
\n

Footnote

\n

1 Such as How drastic is reversing the polarity of the power supply to a RAMPS board?, which might be relevant in your case.

\n" }, { "Id": "10839", "CreationDate": "2019-08-22T15:09:10.833", "Body": "

I'm trying to print a supporting base which will house the spindle for an electrostatic rotor. It's basically just a truncated cone with a hole down it's center to house the spindle.

\n\n

For reasons that I cannot fathom, Ultimaker Cura keeps on adding an unrequested top/bottom layer (color-coded yellow in the screenshots) inside this hole, so instead of a single hollow cylinder of 10 mm depth, the result is a hole only a few millimeters deep with another hollow cylinder behind it.

\n\n

Here is the intended model, note the open space for the hole at the top.

\n\n

\"enter

\n\n

Here is the inner view of the hole being printed as expected:

\n\n

\"enter

\n\n

Finally, here is a layer view of the print a few millimeters from the final top layer with the unrequested top/bottom layer that covers the spindle hole:

\n\n

\"enter

\n\n

The STL file is on Github (with a built-in viewer).

\n\n

Can anyone help me understand why this is happening?

\n", "Title": "Ultimaker Cura is adding redundant top/bottom layers", "Tags": "|ultimaker-cura|slicing|troubleshooting|", "Answer": "

I had the same issue exporting from Sketchup to STL. I imported my STL to Tinkercad and then exported it again and it resolved my issue.

\n

Tinkercad is a free and online tool.

\n" }, { "Id": "10840", "CreationDate": "2019-08-22T16:07:37.677", "Body": "

I want to print a lemon squeezer and I would prefer to use PET-G. I don't know if it is safe to use, because lemons contain lots of citric acid. Does it dissolve PETG? I haven't found an answer anywhere on the Internet. There are generally few things that dissolve PETG. These are aromatic compounds like toluene, phenol etc.

\n\n

I know my model will be food safe, as PETG is food safe, I'm using one without a dye and my nozzle is made out of steel, not brass. I think bacteria growth inside little gaps/between layers is impossible, because the citric acid is quite strong and will kill nearly all of the germs.

\n", "Title": "Does acid dissolve PETG 3D prints?", "Tags": "|3d-models|petg|food|", "Answer": "

The PETG is food safe (plastic water bottles are made of them), however the colour additives may not be a) stable, or b) food safe. If you are going to make a lemon squeezer then I would suggest that you use a virgin material that is just pet-g with no additives.

\n\n

However, you could only ever use it one. Any food particles that get stuck in between the fine layers of the printed part, will cause bacterial growth. If the walls are porous then the juice can get inside of the part and create a breathing ground.

\n" }, { "Id": "10844", "CreationDate": "2019-08-23T02:21:14.880", "Body": "

Is there a command to show the compile date and time of the Marlin 2.0 firmware? Also, is there a way to show this info at boot up?

\n", "Title": "Show Build date and time in Marlin 2.0", "Tags": "|marlin|", "Answer": "

You can show the compile date and time during bootup by changing the constant STRING_SPLASH_LINE2 in Configuration.h:

\n\n
#define STRING_SPLASH_LINE2 __DATE__ \" \" __TIME__\n
\n" }, { "Id": "10847", "CreationDate": "2019-08-23T09:46:58.777", "Body": "

I'm trying to make a water insulated 1 cm3 (1 ml) transparent container and I bought some plexiglass, I cut and glued some pieces together but it looks really crappy and barely holds the water in. I was wondering, is there a transparent material (similar to plexiglass) that can order to 3D print the container out of it? Also, if 3D printing is not the best option, where can I order around a 100 pieces of 1 cm3 transparent water insulating containers with caps?

\n", "Title": "Can I order a 3D print of a 1 ml transparent container?", "Tags": "|3d-models|", "Answer": "

You could order an sla or dlp printed part(which would probably be more expensive, but also quite durable and a little more transparent). Or you could order an fdm printed T-glase print, which would come out pretty clear and not be quite as strong as sla, but still pretty good for holding water.

\n" }, { "Id": "10853", "CreationDate": "2019-08-23T14:04:14.797", "Body": "

Trying to set up Repetier host for Ender 3 on Debian.

\n\n

The /dev/ttyUSBx is found via dmesg, this value goes in config -> printer settings -> port.

\n\n

Printer connects fine, but commands do not send. Status reads

\n\n
xx commands waiting.\n
\n\n

I have tried all the various baud rate settings <= 250k to no avail.

\n\n

Is there something else I should be trying, or what is the correct baud rate to set?

\n\n

dmesg:

\n\n
[2828465.418249] usb 1-13: USB disconnect, device number 77\n[2828465.418480] ftdi_sio ttyUSB0: FTDI USB Serial Device converter now disconnected from ttyUSB0\n[2828465.418492] ftdi_sio 1-13:1.0: device disconnected\n[2828469.171168] usb 1-13: new full-speed USB device number 82 using xhci_hcd\n[2828469.329014] usb 1-13: New USB device found, idVendor=0403, idProduct=6001, bcdDevice= 6.00\n[2828469.329017] usb 1-13: New USB device strings: Mfr=1, Product=2, SerialNumber=3\n[2828469.329018] usb 1-13: Product: FT232R USB UART\n[2828469.329020] usb 1-13: Manufacturer: FTDI\n[2828469.329021] usb 1-13: SerialNumber: A107MDUQ\n[2828469.332455] ftdi_sio 1-13:1.0: FTDI USB Serial Device converter detected\n[2828469.332485] usb 1-13: Detected FT232RL\n[2828469.332756] usb 1-13: FTDI USB Serial Device converter now attached to ttyUSB0\n
\n\n

lsusb:

\n\n
Bus 004 Device 001: ID 1d6b:0003 Linux Foundation 3.0 root hub\nBus 003 Device 003: ID 0a5c:21e8 Broadcom Corp. BCM20702A0 Bluetooth 4.0\nBus 003 Device 001: ID 1d6b:0002 Linux Foundation 2.0 root hub\nBus 002 Device 001: ID 1d6b:0003 Linux Foundation 3.0 root hub\nBus 001 Device 081: ID 18d1:4ee1 Google Inc. Nexus Device (MTP)\nBus 001 Device 002: ID 1b1c:1b45 Corsair \nBus 001 Device 046: ID 1b1c:1b50 Corsair \nBus 001 Device 049: ID 14cd:1212 Super Top microSD card reader (SY-T18)\nBus 001 Device 082: ID 0403:6001 Future Technology Devices International, Ltd FT232 Serial (UART) IC\nBus 001 Device 060: ID 1908:0226 GEMBIRD \nBus 001 Device 059: ID 046d:c52b Logitech, Inc. Unifying Receiver\nBus 001 Device 058: ID 05e3:0610 Genesys Logic, Inc. 4-port hub\nBus 001 Device 001: ID 1d6b:0002 Linux Foundation 2.0 root hub\n
\n\n

usb-devices:

\n\n
T:  Bus=01 Lev=01 Prnt=01 Port=12 Cnt=02 Dev#= 82 Spd=12  MxCh= 0\nD:  Ver= 2.00 Cls=00(>ifc ) Sub=00 Prot=00 MxPS= 8 #Cfgs=  1\nP:  Vendor=0403 ProdID=6001 Rev=06.00\nS:  Manufacturer=FTDI\nS:  Product=FT232R USB UART\nS:  SerialNumber=A107MDUQ\nC:  #Ifs= 1 Cfg#= 1 Atr=a0 MxPwr=90mA\nI:  If#=0x0 Alt= 0 #EPs= 2 Cls=ff(vend.) Sub=ff Prot=ff Driver=ftdi_sio\n
\n", "Title": "Baud rate for Ender 3 on Repetier Host (Debian)", "Tags": "|creality-ender-3|repetier|", "Answer": "

Your firmware configuration may vary (probably worth checking what you have set), but from The Ender 3 configuration is set to 115200 baud rate #12174, the standard baud rate seems to be 115200.

\n\n
\n

I think the default Ender 3 configuration is aligned to the Ender3 official confg, which has 115200 so...

\n
\n\n

Ah, but this is Marlin, and you're using Repetier. Hmmm, maybe ignore that then.

\n\n

According to Recommended baud rate?:

\n\n
\n

Normally 115200 or 250000 baud should work find and usb->serial latency is the bottleneck.

\n
\n\n

Apart from that I don't know. Sorry. I might delete this answer, if it doesn't help.

\n" }, { "Id": "10858", "CreationDate": "2019-08-25T10:41:21.610", "Body": "

I have an Anet A8 and have a problem with my first layer. I printed nice prints but starting today the first layer is tearing in the middle:

\n\n

\"enter

\n\n

\"enter

\n\n

Any idea how to fix this?

\n", "Title": "first layer tearing", "Tags": "|ultimaker-cura|anet-a8|adhesion|", "Answer": "

I found the solution to be the exact opposite. My bed was too low (as in too far from the hotend). All the above mentioned aside, I did also drop the flow rate on the brim and initial layer by roughly 2-3\u00a0%. Now it prints perfectly again. (your mileage may vary)

\n" }, { "Id": "10868", "CreationDate": "2019-08-26T16:57:33.060", "Body": "

On several occasions I've wanted lettering/numbering printed as part of a design, but with the ability to make it stand out more effectively in the printed object without having to do detailed manual finishing. Is there a good material which can catch in and fill sunken (depth 0.4 mm, width 0.6-1.0 mm) lines/strokes of alphanumeric characters without sticking to the surface (including fine layer ridges) of the print? My best result so far has been with crayon wax, but I wonder if there are more suitable materials. (Polymer clay, perhaps?)

\n\n

Results with crayons:

\n\n

\"3D

\n\n

Durability is nice (and essential for some applications), but for many uses I have in mind it's not such a big deal. For example another place I've wanted clear text is on test panels to check nut/bolt thread sizes, in which case the text is unlikely to receive harsh treatment but any heat-based curing processes might effect the dimensional accuracy negatively. So both durable and non-durable solutions are interesting to me.

\n", "Title": "Finishing sunken text in 3D prints", "Tags": "|post-processing|", "Answer": "

As suggested by Davo in a comment, nail polish works very well for this.

\n\n
\n

I've seem people use fingernail polish before, wiping the excess off while it's wet.

\n
\n\n

What works even better than wiping the excess off while it's wet is letting it fill up the pits and fully dry, then using acetone or other solvents to remove it from the non-sunken surface around the text. Acetone doesn't attack PLA, but I've found it does alter some PLA dyes, so alternate solvents might be preferable if that's a problem.

\n\n

Here's the result of my first test:

\n\n

\"White

\n" }, { "Id": "10870", "CreationDate": "2019-08-26T20:33:08.357", "Body": "

Take the standard Hiwin rail, as commonly used in 3D printers:

\n\n

\"Ebay

\n\n

They come in a max length of 2500 mm. Let's say I need twice that. If I connect two rails together end to end (as a \"butt joint\") and just bolt them into place, will it work as though it was one 5000 mm rail? Or are we limited to whatever lengths the manufacturers put out.

\n", "Title": "Can linear guide rails be joined?", "Tags": "|linear-motion|", "Answer": "

The answer is yes; and it's a normal thing to do.

\n\n

https://www.igus.com/info/linear-guides-butting-rails-together

\n\n

\"DryLin\u00ae

\n\n

Basically you should only do it with rails from the same manufacturer and you might have to file the mating surfaces, but butting them is altogether trivial. This method works with Drylin and UHMW-PE bearing surfaces as recirculating balls could get caught in the small gap that is left behind. This would cause the carriage to \"jump\" as it passes over the joint.

\n" }, { "Id": "10874", "CreationDate": "2019-08-27T12:01:01.813", "Body": "

I bought Ender-5 and if I print sample model in bundle SD, it\u2019s just perfect.

\n\n

\"sample_cat\"

\n\n

But when I tested basic cube with raft, its base and top is just ugly. Even thought its 4 sides are perfect.

\n\n

\"raft\"\n\"bottom\"\n\"top\"\n\"side\"

\n\n\n\n

Which setting(s) should I check futher?

\n", "Title": "Problem with first and last layers", "Tags": "|ultimaker-cura|slicing|creality-ender-5|", "Answer": "

You may want to ensure that your filament diameter is correctly set in your printer's CURA profile, and that your hot end is at the correct temperature for the material you are using. Additionally, you should not need a raft to print the cube, and should be able to do without a raft or brim.

\n" }, { "Id": "10875", "CreationDate": "2019-08-27T12:28:00.053", "Body": "

I'm looking to 3D print a structure that won't deform in high heat, up to about 220 °C. The filament itself can be 3D printed all the way up to about 380 °C.

\n\n

PEI seems like it could be a viable option. I found some here. This PEI filament specifies the glass transition temperature at 217 °C.

\n\n

Would this filament work? Are there any other types of materials that would fulfill this engineering requirement?

\n", "Title": "What is the 3D printer filament (or pellet) most resistant to bending at high heats?", "Tags": "|filament|filament-choice|", "Answer": "

An easier but less simple solution might be to make a PLA 'pattern' that is the right size and shape, then use that to cast your item in aluminium (melting point ~ 660C) using the 'investment' or 'lost PLA' process.
\nLinks:- \nhttps://www.instructables.com/id/3D-Printed-Lost-PLA-Investment-Casting-Aluminium/\n

\r\n \r\n

\n" }, { "Id": "10880", "CreationDate": "2019-08-27T16:45:03.360", "Body": "

I recently bought a 3D printer called Dreamer NX from FlashForge. The dealer told me to use FlashPrint software that belongs to the FlashForge printer manufacturer. But, many people advise me to use Ultimaker Cura. Are there many differences between these two software packages?

\n", "Title": "Are there many differences between Ultimaker Cura and FlashForge FlashPrint slicers?", "Tags": "|ultimaker-cura|flashprint|flashforge-dreamer|", "Answer": "

Perhaps this is veering out a little bit from being an answer to the specific differences between software functionality, but one important difference that shouldn't be overlooked is that whatever you learn with Cura is applicable to all FDM 3D printers.

\n\n

Surely some setting tweaks you do might be specific to the properties of your particular printer, but a large amount of them, like choices of infill patterns, shells, adaptive layer heights, using secondary models as custom support and infill masks, breaking up models into multiple pieces for printing, etc. are completely printer-agnostic. If you learn to do these with software that's only usable (or at least only meant to be used) with your particular brand of printers, you'll have to translate/relearn if you later want to use a different printer, or help someone else who has a different brand of printer. If you learn with software that works with any printer, everything you learn is immediately applicable to different printers.

\n" }, { "Id": "10881", "CreationDate": "2019-08-27T18:01:24.013", "Body": "

On a budget, I'm water-cooling a 3D printer. I'm using a 5 V aquarium pump (\\$3). Originally I tried to only use about as much water in a can of chickpeas but then found out i needed a lot more. I have a deliberate need to water cool stepper motors in the first printer, so that I can print with a high temperature filament like PEI (the operating temp of a stepper is maxed at about 53 degrees celsius; PEI requires an enclosure temperature of 80 °C), but on another printer I'm having some other issues with the motors that I think could be solved by better heat dissipation.

\n\n

What I am getting to is a device like this:

\n\n

\"e\"

\n\n

It is the perfect size for a stepper motor. My plan is to zip-tie one of these to each stepper motor and water cool it in a single path across my printer, including the hot end.

\n\n

Can anyone think of a reason why this wouldn't work? i just haven't heard of anyone doing anything like this, but it makes sense to me as a chemistry minor. The specific heat of water is way higher than almost anything else. And it is way less noisy than fans. And it works inside an enclosure, while fans might not

\n\n

Should I ziptie the aluminum block to the back part of the stepper where the metal is, or to one of the darker black sides?

\n\n

Would I be able to 3D print a cooling block like this instead of paying for it? See also this relevant question on thermal conductivity of various 3D printing filaments. It should probably be metal to transfer heat better?

\n", "Title": "Water-cooling stepper motor with aluminum block", "Tags": "|cooling|", "Answer": "

I just did this lol and checked online to see if anyone else did it too. It works really nice.

\n\n

I used thermal paste and zipties to secure each block to the stepper motor. Dont use thermal tape its not as effective as paste.

\n\n

I had to do this operations since my motors were overheating due to the enclosure design.

\n\n

\"enter

\n" }, { "Id": "10888", "CreationDate": "2019-08-28T19:18:51.963", "Body": "

When go to export a model using Fusion 360 or Meshmixer, I see that there are two options. Could the final model be affected by the format chosen at the time of saving?

\n\n

\"enter

\n", "Title": "Export STL as ASCII or binary?", "Tags": "|3d-models|stl|", "Answer": "

The other answers on this thread seem kind of hand-wavy, so I'll give my input.

\n

At its simplest, all we're dealing with here is two different formats of encoding the same data. The 3D file is identical, just described by the file data in different terms.

\n

That being said, there is a multitude of different reasons that 3D prints can fail. Fusion 360 is notorious for having issues with slicers because of fillets, lofts, smooth-curvy type patterns, or intersecting planes.

\n

Binary is a smaller encoding. It almost always works for me. ASCII has never failed me as a backup when binary did.

\n" }, { "Id": "10891", "CreationDate": "2019-08-29T02:59:27.697", "Body": "

Temperature is showing about 680 °C at room temperature. When I make it \"hotter\" by holding my thumb and index finger on it, the temperature does increase. So it is responding to temperature changes.

\n\n

I'm trying to add some thermistors to my MKS GEN L v1.0 board:

\n\n

\"MKS

\n\n

I'm using the A11 pin and GND in the picture above.

\n\n

The thermistor works correctly when plugged into the HE1 temperature pin. So the thermistor is fine hardware-wise.

\n\n

The thermistor is set to be the \"Chamber\" thermistor in Marlin 1.1.x. This just means it is supposed to record the temperature of the enclosure. I don't know if there is some kind of volatility issue with the chamber constant in Marlin.

\n\n

The thermistor has the same settings in Marlin for both HE1 and the chamber one. They are both set to \"thermistor 11\" for the 3950 type of thermistor. So the most visible setting implies they should be treated the same firmware-wise.

\n\n

I've also tried the other extra Analog pins in the AUX-2 port.

\n\n

What could be the issue here?

\n\n

If these are the only available analog pins, how can I add more thermistors to my set-up so that they're available in Marlin?

\n\n

Is there any way I can use the AUX-1 port? It doesn't seem to be an analog pin like in the photo for this particular version of RAMPS. I tried pin A3 but it didn't work.

\n", "Title": "Weird temperature reading using thermistor on MKS GEN L v1.0 AUX-2 analog pins", "Tags": "|marlin|thermistor|", "Answer": "

Thermistors, commonly used as temperature sensors on 3D printers and other DIY things, work by changing their internal resistance depending on the temperature.

\n\n

On the analog ports, the microcontroller can determine the voltage of the connected signal.

\n\n

To actually measure the resistance - and thus temperature - of the thermistor, you need to build a voltage divider:

\n\n

\"Voltage

\n\n

Most, if not all, 3D printer control boards use a resistor of 4.7 kOhms and a capacitor to build this voltage divider:

\n\n

\"RAMPS

\n\n

You'll need at least this 4.7 kOhm resistor to get your thermistors working on any analog pin, if the three thermistor ports on the MKS Gen L don't suffice for you needs.\nYou could also use other temperature sensors, especially for the case, like the DHT11 or DHT22, however I couldn't find whether they're supported by Marlin yet.

\n" }, { "Id": "10893", "CreationDate": "2019-08-29T06:46:09.430", "Body": "

Since retractions above 4 mm can cause jams due to plastic sticking to the cold half of the metal print head, what's the best way to avoid jams when swapping filaments?

\n\n

Cool down and yank out, rapid pull out while melted, pull out at partial heat but not completely melted, or something else?

\n", "Title": "What's the best way to pull out fillament from an all metal print head", "Tags": "|filament|hotend|retraction|change-filament|", "Answer": "

Cold/Luke-Warm pulling can cause damage and wear to the hot end, and should only be used in cases where you suspect there's a jam in the nozzle.

\n\n

Pulling the filament out at a few degrees above its normal printing temperature and pulling quickly is what I use, and what I've seen other's also say, and the procedure most machines use.

\n" }, { "Id": "10906", "CreationDate": "2019-08-30T17:44:56.120", "Body": "

Why don't 3D printer heads use ceramic inner walls? PTFE tubes melt with high enough temperatures and all metal ends risk jamming as heat makes its way up the head.

\n", "Title": "Why don't 3D printer heads use ceramic inner walls?", "Tags": "|hotend|", "Answer": "

It can be done cheaply, as two different users have proven, see

\n\n\n\n

However, as Paulster2 states in his answer, there are some technical issues with using it, which make it rather problematic. Apparently, in comparison with PTFE, the thermal conductivity of the ceramic in spark plugs is too high, to use (according to nophead - a user on the reprap forums), and there are friction/clogging issues, unless the inner diameter is very well polished.

\n\n
\n\n

Synopsis of reference

\n\n

The RepRap user, hp_, encountered the issues above when attempting a design - from Ceramic Hotend - Part 1

\n\n
\n

Research

\n \n

As far as I know there are no ceramic hotends out there, I know\n nophead has tried some spark-plugs for nozzle holders but found them\n not suitable(thermal conductivity is pretty high). I wanted to give it\n a go, confident enough (I hoped), that it would work :)

\n \n

So in my case, a hotend exists out of 2 main parts, a nozzle holder\n and a nozzle.

\n \n \n \n

total length should be in the range of 35-40mm, see my first sketch\n below:

\n \n

\"enter

\n \n

here are many types of ceramic out there, ie. 95% AI2O3, 99% AI2O3,\n Zirconia (see material properties sheet Link)

\n \n

95% AI2O3 is easy to buy but after a few tests the conclusion was its\n to brittle for my taste, second material to try is Zirconia.

\n \n

I've found a few Chinese ceramic manufactures. Only draw back I had to\n order 10 pieces for the first batch.. on something that has never been\n tested, well I'd give it a shot.... and ordered the parts.

\n
\n\n

but the clogging issue mentioned above was encountered:

\n\n
\n

...after the first layer, it just stopped extruding.. ugh!!! what could\n be wrong????

\n \n

Possible root causes\n - Friction coefficient? Meaning after awhile the friction between PLA and the Ceramic became so high it would just jam the nozzle holder.

\n \n \n
\n\n

The user was forced to return to using PTFE.

\n\n

From Ceramic hotend part-2, after some rework done by the Chinese manufacturer, the new hotends worked correctly:

\n\n
\n

Awhile ago i stared working on the ceramic hotend and found out the\n first version wouldn't work for 3.0mm fillament,

\n \n

after some discussion with my chinese counterpart :) i got a new\n version of the ceramic piece.

\n \n

They polished the inside very deep and precise. and i gave it another\n go.

\n
\n\n

and

\n\n
\n

some more tinkering with the hotend and a new nozzle design, with a smaller Inner diameter, and its longer

\n
\n\n

Apart from that the details are a little sparse.

\n\n
\n\n

Additional information

\n\n

From J-head with ceramic body instead of PEEK, specifically this post:

\n\n
\n
\n

Just to be clear, it's Ceramic Zirconium.

\n \n

My concern was that Zirconium becomes brittle when it is exposed to heat for consecutive long periods of time. I would stay with PEEK.

\n
\n \n

MgO or Yttria stabilized grades of Zirconium are very stable.

\n \n

Pure ZrO2 is known to crack, so additives are used to stabilize it.

\n \n

Key Properties of Zirconium Oxide

\n \n \n \n

Typical Uses of ZrO2

\n \n \n \n

Zirconium oxide

\n \n

Zirconium oxide is used due to its polymorphism. It exists in three\n phases: monoclinic, tetragonal, and cubic. Cooling to the monoclinic\n phase after sintering causes a large volume change, which often causes\n stress fractures in pure zirconia. Additives such as magnesium,\n calcium and yttrium are utilized in the manufacture of the knife\n material to stabilize the high-temperature phases and minimize this\n volume change. The highest strength and toughness is produced by the\n addition of 3 mol% yttrium oxide yielding partially stabilized\n zirconia. This material consists of a mixture of tetragonal and cubic\n phases with a bending strength of nearly 1200 MPa. Small cracks allow\n phase transformations to occur, which essentially close the cracks and\n prevent catastrophic failure, resulting in a relatively tough ceramic\n material, sometimes known as TTZ (transformation toughened zirconia).

\n \n

Zirconium dioxide is one of the most studied ceramic materials. Pure\n ZrO2 has a monoclinic crystal structure at room temperature and\n transitions to tetragonal and cubic at increasing temperatures. The\n volume expansion caused by the cubic to tetragonal to monoclinic\n transformation induces very large stresses, and will cause pure ZrO2\n to crack upon cooling from high temperatures. Several different oxides\n are added to zirconia to stabilize the tetragonal and/or cubic phases:\n magnesium oxide (MgO), yttrium oxide, (Y2O3), calcium oxide (CaO), and\n cerium(III) oxide (Ce2O3), amongst others.

\n \n

In the late 1980s, ceramic engineers learned to stabilize the\n tetragonal form at room temperature by adding small amounts (3\u20138\n mass%) of calcium and later yttrium or cerium. Although stabilized at\n room temperature, the tetragonal form is \u201cmetastable,\u201d meaning that\n trapped energy exists within the material to drive it back to the\n monoclinic state. The highly localized stress ahead of a propagating\n crack is sufficient to trigger grains of ceramic to transform in the\n vicinity of that crack tip. In this case, the 4.4% volume increase\n becomes beneficial, essentially squeezing the crack closed (i.e.,\n transformation decreases the local stress intensity).

\n
\n\n

and the following post

\n\n
\n

Thermal conductivity:

\n \n \n
\n\n

Rulon

\n\n

As an aside, again from J-head with ceramic body instead of PEEK, specifically this post:

\n\n
\n

Rulon was one material we used. I think it is a glass filled ptfe. The mechanical strength is far better than solid ptfe and it is easy to machine. There are many grades but Rulon AR for example will withstand 288 deg C.

\n
\n\n

but there are inconsistencies in quality

\n\n
\n

Rulon i looked at a while ago, there are plenty of options with it, however the cost of some of these materials can be incredibly high, and in some cases availability is a serious problem, and the difference country to country is borderline criminal in some cases

\n
\n" }, { "Id": "10907", "CreationDate": "2019-08-30T17:56:55.267", "Body": "

I have an infrared lamp I'd like to use to heat my chamber. Right now I don't have plans to install a cooling fan, but I will if I need to. This question pertains to Marlin (2.0, preferably) in combination with an MKS Gen L v1.0 board.

\n\n

In the documentation for the thermal settings it specifies you can set a heater pin to be used to heat up the chamber. It says this variable is called the CHAMBER_HEATER_PIN but in Marlin 2.0's Configuration_adv.h file there is something called a HEATER_CHAMBER_PIN that has been commented out.

\n\n

Secondly the Configuration.h file: there is something called an AUTO_POWER_CHAMBER_FAN. Is this meant for a fan used for cooling, or a fan used for cleaning out the chamber of particles? In either case, I probably don't want to have this fan always on, but only use it to cool the temperature the heater has been on too long.

\n\n

So really there should be only 2-3 changes I need to make right:

\n\n\n\n

These are the things I can't fully figure out on my own. \nA detailed set of instructions or code snippets for a similar setup would be helpful

\n", "Title": "How to set up Marlin to use a heated chamber?", "Tags": "|marlin|chamber|", "Answer": "

Here is what I found to be the easiest solution. Please use this image for reference. I recommend doing these instructions once from source, since a lot of things can go wrong, then once everything works, go back and integrate them into your existing Marlin codebase.

\n
    \n
  1. Get some 5V relays to run the infrared lamps. These relays take in a 5V digital signal from an arduino pin. When the arduino activates these pins, the lamps will turn on. Get a cardboard box, line it with aluminum foil, and attach a lamp to the top of it using a lamp switch. If you are going to do this NEVER leave the box unattended, as a relay can get fail especially when it gets warm (this happened to me). Dont cause a fire, make sure to keep it attended. Anyway, One wire for the lightbulb goes to NO on the relay, the other goes to D-. One wire from 120V power goes to COM, the other also goes to D-. The yellow "jumper" on the relay boards I posted goes between HIGH and the middle pin. These are pins on the relay, not the MKS GEN board.

    \n
  2. \n
  3. On the MKS GEN board, we will be using the X-MAX endstop as the digital pin to run the chamber. Wire the 5V pin on X-MAX to the D+ of the relay. Wire the GND pin to D- of the relay with the two other wires there. Wire pin D2 on the MKS GEN Board to IN on the relay. Bold means the pin is on the MKS GEN board.

    \n
  4. \n
  5. Clone the Marlin-2.0 repo: git clone -b bugfix-2.0.x https://github.com/MarlinFirmware/Marlin.git

    \n
  6. \n
  7. In Configuration.h, change TEMP_SENSOR_CHAMBER to 1 or 11 (depends on your thermistor, might be other values.) Since we will be using the board's second extruder thermistor as the thermistor in this example, change TEMP_SENSOR_1 to 0 for now.

    \n
  8. \n
\n
#define TEMP_SENSOR_0 1\n#define TEMP_SENSOR_1 0\n#define TEMP_SENSOR_2 0\n#define TEMP_SENSOR_3 0\n#define TEMP_SENSOR_4 0\n#define TEMP_SENSOR_5 0\n#define TEMP_SENSOR_BED 1\n#define TEMP_SENSOR_CHAMBER 1\n
\n
    \n
  1. In the file src/pins/ramps/pins_RAMPS.h, add this line:\n#define TEMP_CHAMBER_PIN 15. Note that this is the thermistor\nslot usually used for a second extruder. If you're already using\nthis thermistor for the second extruder and want to use a different\nanalog pin, see this question. You'll need some resistors.

    \n
  2. \n
  3. Also, around line 95 in pins_RAMPS.h, change X_MAX_PIN to use an unused pin. The number 4 is good. If you don't do this, your Chamber's digital Pin will be always "on" by default. You want it off by default, then activated by the Marlin code.

    \n
  4. \n
  5. In Configuration_adv.h, change //#define HEATED_CHAMBER_PIN 44 to #define HEATER_CHAMBER_PIN 2 to use the X_MAX endstop's digital pin.

    \n
  6. \n
  7. Upload...

    \n
  8. \n
  9. Go into OctoPrint settings. Click Axes and Volume. Next to a checkbox for heated bed, there is a checkbox for heated chamber! Click that, and your chamber is active.

    \n
  10. \n
  11. If some part is wrong (e.g. lamp doesn't go on because wire came loose on relay), debug that and keep working. Check the octoprint log to make sure the C value is shown.

    \n
  12. \n
  13. Once everything works, generalize these solutions to your set-up (e.g. using multiple extruders).

    \n
  14. \n
\n" }, { "Id": "10936", "CreationDate": "2019-08-31T17:55:04.807", "Body": "

You can see small gaps in the print which looks like under extrusion (see image below).\nWhat is the reason for that?

\n\n

I've tried smaller retracting distances. Temperature looks stable.

\n\n

Print settings:

\n\n\n\n

Setup:

\n\n\n\n

Slicer:

\n\n\n\n

\"Small

\n", "Title": "Under extrusion... sometimes", "Tags": "|petg|retraction|underextrusion|e3d-titan|", "Answer": "

I increased the retracting distance to 1.5mm. Speed was set to 30mm/s\nPrinting temperature is still 245\u00b0C

\n\n

The screw which is pressing the filament to the drive gear was too tight.\nSo the stepper lost steps.

\n\n

-> No stringing and no under extrusion anymore. prints are looking good now.

\n" }, { "Id": "10962", "CreationDate": "2019-09-03T20:25:38.783", "Body": "

I run quite a few Ender 3 Pro's using the same slicer settings (Simplify3D), and just recently I have noticed a very odd extrusion problem.

\n\n

I find that at about the same height on several printers the printer under extrudes by quite a margin. After that, it either continues to under extrude for the rest of the print or it will go back to extruding proper amounts of filament with no problem. This destroys the print and makes it both structurally weak and defective. I am wasting quite a bit of PLA trying to fix this problem so any help would be appreciated.

\n\n

Here is what I have done so far:

\n\n\n\n

That's about it. Not sure where to go from here so if anyone out there can think of anything I missed, I would love to hear it!

\n\n

EDIT 1:

\n\n

As per @fred_dot_u asked, the elapsed time at the layer of failure is roughly four hours in. I have also attached a picture of one example of this kind of failure below. I would also like to mention that this is happening on several of these printers as I have 18 printers running in one room. Our current theory is that the power draw is simply too high and so the printers are not getting the heat they need, however, the thermistors still register a solid 195C on my printers that are currently running.

\n\n

\"side

\n\n

EDIT 2:\nHere is another picture of a different model with the same layer failure problem but at a lower layer height. This model was printed along with 11 other identical models on the same bed, all of which failed at the same height.\n\"Model

\n", "Title": "Ender 3 Pro Filament extrusion problems", "Tags": "|pla|creality-ender-3|extrusion|", "Answer": "

There's a known issue across Creality printers with the PTFE tube (bowden) slipping in the coupler on the hotend side, allowing molten material to push back around/up inside it rather than making it out the nozzle. I haven't experienced any such problem myself, even after applying some fairly serious abuse to that part of my Ender 3, but this video from CHEP claims it was the source of his problems and explains the issue well.

\n\n

I believe my Ender 3 came with an extra replacement for this coupler; if yours did, and this seems to be the problem, you can see if the second one they supplied is better. Otherwise it's an easy component to source third-party replacements for.

\n" }, { "Id": "10963", "CreationDate": "2019-09-04T03:31:14.850", "Body": "

I have what I assume to be a brass nozzle, and I only print in PLA. I am starting to have some issues with first layers and stringing. I'm wondering if I should replace the nozzle. What signs should I look for to know when to replace the nozzle?

\n", "Title": "When should I replace the nozzle on my printer?", "Tags": "|nozzle|fdm|", "Answer": "

Replacing the nozzle depends on many things,

\n\n\n\n

To replace a nozzle, there are probably a few reasons for doing so. A nozzle can wear out (see: How to identify nozzle wear; not only from the friction of standard filament, but especially when used with abrasive filament like glow-in-the-dark PLA, and glass or carbon fibre filled filament) causing the outer shape and the inner nozzle diameter to change size. This can result in lower quality prints or difficulty in getting the print to stick to the bed. Having a metal wire or drill bit of the diameter of the nozzle will tell you if the hole itself has worn out. Sometimes, a visual inspection may sometimes be enough for you to tell that it has worn out:

\n\n

\"New

\n\n

(source: filament2print.com)

\n\n

Please note that an interesting video is posted by Stefan from CNC Kitchen on nozzle wear:

\n\n

\r\n \r\n

\n\n

Nozzles can collect burned particles inside, partly clogging the nozzle, causing the extrusion to be non-ideal. A sign for this is when the filament is not leaving the nozzle straight when freely extruded at height, but curls up.

\n\n

Furthermore, when used frequently, nozzles can get coated with filament which is sometimes hard to remove. This sometimes causes freshly extruded filament to stick to the nozzle, certainly if it curls up like mentioned before.

\n\n

Note that nozzles are very cheap with respect to the complete assembly and replacing them is a matter of 2 minutes work. So if you're unsure about how to improve the quality of printing after having tried fixing flow rate and nozzle to bed height, you could replace the nozzle, see e.g.:

\n\n

\"Differences

\n\n

(source: The 3D Print General)

\n\n

Depending on the quality of the nozzle and the filament, a nozzle replacement for one brand might be necessary much sooner than for another printer. E.g. my Ultimaker 3E has literally printed kilometers of PETG without the need to replace the nozzle (or \"core\" as it is called for this printer), while the brass nozzle of my first cheap, now dust collecting, Anet A8 printer has been replaced a few times.

\n" }, { "Id": "10967", "CreationDate": "2019-09-04T14:35:14.170", "Body": "

In this instructable, a user creates a heated bed from a wooden plank, some nails, insulated copper wire, and a DC switch. He does place an aluminum bed on top of it, but for our heated chamber, we won't use any aluminum on top of these wires. We aren't trying to heat an aluminum bed, but an entire chamber.

\n

\"enter\nAssume I place a wire loom like this underneath my 3D printer. (I might place some aluminum foil between the wood and the copper wire, to reflect heat upwards more? Is that safe?)

\n

Then assume I have a cardboard box that has aluminum foil lining the walls on its inside. Assume the total size of my printer is 500mm x 500mm x 500mm. When I place the cardboard box over my printer, do you think it would take less than 1 hour for the top of the cardboard box to reach 60 degrees Celsius?

\n

Do you think there are any other hazards here I am not considering?

\n

Could I, theoretically, do this entirely with cardboard, and no wood?

\n

Could I, theoretically, line the inside of my cardboard box like this? Or might the cardboard ignite?

\n

I'd like to use cardboard because it is both lightweight and insulative. I can remove it easier to check all areas of my printer, unlike an enclosure.

\n", "Title": "Creating a heated chamber from a very large heated bed", "Tags": "|heated-bed|heat-management|chamber|", "Answer": "

Cardboard is fundamentally paper. Paper ignites - just as Ray Bradbury claims - at around 233\u00a0\u00b0C or 451\u00a0\u00b0F.

\n

A thin heater wire usually glows red hot under operation. Not dark red, not blood cherry, not dark cherry, it's usually medium cherry to dark orange in operation, as one can easily tell by looking into a trusty toaster oven, which also shows us how powerful of a heating element this is.

\n

In a typical toaster, heater wires are mounted in a metal frame and backed by some non-flammable material that looks like paper but is not. In a 2004 patent for such a method, they use a sheet of mica. But how hot is our wire? Well, under mains power, the wire of a toaster reaches 700-900\u00a0\u00b0C within seconds, heating the inner chamber of the toaster despite the slots to about 200-250\u00a0\u00b0C so quick that the bread roasts deliciously... until the heat controlling bimetal strip releases it up.

\n

We operate these wires for at best a few minutes. In an electric forge, pretty much the same wires with a little more thickness to them are run at lemon glow for hours... and they are embedded in firebrick material to prevent them from igniting stuff touching them by accident!

\n

How we know the temperature? Incandescence charts of course: \"enter

\n

Conclusion

\n

Paper or cardboard backing on heater wires is a bad idea. Mounting the heater wire to a metal frame that is as nonconductive as possible and without a chance that something accidentally touches it is needed. Fiberglass or rock material would serve as a good carrier, an aluminium block with radiators would give a good heating-to-surface mix, especially is precautions are taken to make the wire surface nonconductive.

\n

Even using wood close to such a heater is a gamble I would not want to take. It's also best to mount the heating element into an enclosure that can't touch the wooden outer chamber and that stays put if the enclosure is opened or removed.

\n" }, { "Id": "10970", "CreationDate": "2019-09-04T21:13:12.973", "Body": "

I'm looking to crimp wires to the AUX-1 and EXP pins of my MKS base. For sanity reasons, i cant directly use Dupont jumpers, they keep coming out.

\n\n

\"MKS

\n\n

BIQU MKS-Base V1.6 Plate Controller Board for 3D Printer Ramps 1.4

\n\n

In the above product image, the type of housing is the \"black\" colored headers on the board, around the top left corner of the board. Each header has ~10 male pins inside of it

\n\n

Most of the other pins are of JST-XH type, for anyone else wondering.

\n", "Title": "What is the MKS Base AUX header connector type?", "Tags": "|wiring|wire-type|", "Answer": "

You need a 2.54 mm pitch (similar pitch to Dupont) female IDC (insulation displacement connectors on a ribbon cable) connector of 2x5P (pins) for the header connectors that have a notch. (for comparison, note that e.g. RAMPS 1.4 has 2x5p dupont headers on the shield for the AUX headers, not the notch type headers)

\n\n

\"Flat

\n\n

These connectors are usually crimped onto flat ribbon cables and used to connect the display to the board. These connectors have a rectangular notch to fit the gap in the housing to correctly position the cable (which the 2x5P Dupont don't have).

\n\n

\"Flat

\n\n

For reference/difference, an image of the Dupont 2x4P and 2x5P is shown below, highlighting both sides (\"up\" on the left, \"down\" on the right):

\n\n

\"Dupont

\n" }, { "Id": "10973", "CreationDate": "2019-09-05T22:47:47.083", "Body": "

I would like to print a vase that has a width/length/height footprint of 100x100x200 mm, and the surface finish is of particular interest to me. Ideally, I would like to do a minimum of sanding/XTC-3D coating after printing, if any.

\n\n

What would be the best material to work with?

\n", "Title": "Smooth finish on large parts", "Tags": "|surface|smoothing|large-format|", "Answer": "

From the printer...

\n

Non-post-processed surface smoothness of a print is mainly a function of the Layer height during print: it is smoother if one reduces the height of the steps taken between the layers. A very good FDM machine can achieve down to 0.05\u00a0mm layer height, but 0.1\u00a0mm is what most hobbyist machines can achieve if well dialed in. SLA printers cure resin in layers down to 25 micron reliably and some can get down to 12.5 micron - a quarter of the best achievable FDM.

\n

Reducing the layer height in FDM comes at the cost of mainly three factors:

\n\n

SLA printers are not affected by the print strength loss and the errors from clogged nozzles and can print entirely clear material without air inclusions. They are affected by the print time increase though. Usually, SLA prints come out of the vat covered in residual resin and curing them in the proper post-processing method results in this resin smoothing over the surface perfectly.

\n

Simple Postprocessing...

\n

Many prints can easily be post-processed.

\n

Sanding

\n

PLA and ABS can be easily sanded down to somewhat smooth, but you'll need to spend time and muscle grease or go electric. A sander with 300+ grit can help a lot. If you have a rotational body, you could sand the vase on a lathe.

\n

Putty

\n

A plastic putty could be smeared over the stepped surface to smooth it and then sanded smooth. I personally like Valejo plastic putty, as it doesn't shrink, but Tamya putty also is a good alternative, bonding strong.

\n

Primer-Filler

\n

Independent of the base material, using a spray coating can easily fill in the notches and crannies left behind during printing. Printing in 0.15\u00a0mm and giving it a short spray coat of primer-filler twice, sanding over rough spots between did a good job of smoothing prints for me.

\n

Vapor Smoothing

\n

If you chose ABS, you could use Acetone vapor to smooth over the surface of a print in almost an instant. This can achieve Super smooth results with tall layer heights, as you practically melt the plastic over, but it comes at the loss of small surface details.

\n

Exposure time is the limiting factor here: the longer the exposure to the acetone vapor, the more the surface is smoothed and the less details remain. Think of exposure time like running the smooth filter in blender once or twice each second.

\n

\"Left

\n

Similarly, ethylacetate or propylene carbonate could be used for PLA prints, though timing might differ from the well established "a couple of seconds" for acetone vapor. Note that these two chemicals are much more expensive.

\n

Costs

\n

Checking prices, I was quoted about 90\u00a0\u20ac/l for (chemical grade) Ethyl Acetate and 130\u00a0\u20ac/l for (chemical grade) Propylene Carbonate via a laboratory supply. Useable Acetone (not chemical grade) comes about 3.70\u00a0\u20ac/l in home depot. For comparison: chemical-grade Acetone comes with a price tag of about 50\u00a0\u20ac/l.

\n

Considering that ABS filaments and PLA filaments typically price similarily, ABS+Acetone Vapor is the least costly chemical Vapor smoothing.

\n" }, { "Id": "10981", "CreationDate": "2019-09-06T22:16:29.717", "Body": "

Yesterday during a print, the printer just stopped working and all motors and fans stopped. My printer is a Graber i3.

\n\n

Just before that I saw a flare coming from somewhere in the extruder or from the power supply, I couldn\u2019t actually see.

\n\n

Checked the voltage coming into the RAMPS and it was 0 V, checked voltage coming from the power supply, also 0 V. Checked the voltage coming into the power supply and it was 217 V... that\u2019s okay, the power supply is switched to 220 V.

\n\n

Today I tried further troubleshooting, by connecting the power supply to 127 V and having it switched to that voltage, before that I disconnected the RAMPS cable in case it was shorted. The result was the power supply did not work, the voltage coming from it was 0 V.

\n\n

Got in contact with the manufacturer since the printer is brand new and he is sending me a new power supply, but I do want to try to find the root cause of the problem, so I started looking for shorts in the RAMPS, hotbed is okay, no continuity. Neither of the other connections have shorts, until I got to the hot end heating cartridge, in this convection I found 5 Ω resistance between phases. This was at first no surprise because it makes sense since it\u2019s a heating element, but wanted to make sure, is it okay to have this resistance in the cartridge?

\n\n

Do you guys have any other suggestion for troubleshooting or possible root causes for this issue?

\n\n

UPDATE 02/OCT/2019:\nI decided to open the power supply, so maybe I could fix it and have it as a back up. First thing I see once opened, the fuse was blown! So I replaced it, since it is soldered and it is a 8A 250V 3x10mm, which happens to be hard to find... took quite a while. Once the repair was done, plugged it in and boom, another firework happened, this time blowing a resistor and not the fuse.

\n\n

Conclusion: it wasn\u2019t the fuse, another thing caused the first and second shortening.

\n", "Title": "Power supply not working", "Tags": "|troubleshooting|hotend|power-supply|", "Answer": "

I would expect the root cause to simply be the power supply itself.

\n\n

5 \u03a9 is a reasonable resistance for a 12 V / 30\u00a0W heater. It seems strange that you are not seeing continuity in the heated bed as this should also have a low resistance (since it's a heater as well). This might be worth investigating further, but I think it's more likely the power supply itself is the cause (and you simply made a measurement error).

\n" }, { "Id": "10986", "CreationDate": "2019-09-08T09:17:05.357", "Body": "

We all know, that the best layer hight is, when you have multiples of full steps. If it is not, sometimes steps get skipped and end up bad layer-to-layer adhesion when one height step missed a tiny bit and then the next catches up, creating an extra-thick layer. For example, this was printed somewhat deliberately, and here, the extra spaced layers are perfect for delaminating the print with just a fingernail:

\n\n

\"Delaminating

\n\n

The Ender 3 I have uses the following Z-Rod:

\n\n\n\n

The firmware (Marlin) I use claims in configuration.h that the NEMA17 motor would be using 400 Steps per mm in Z. configuration_adv.h tells that the microsteps on the Z-axis motor are 16.

\n\n

In the printer's menu, Babystepping is in increments of 0.049\u00a0mm (though some rounding error seems to be there: 5 Babysteps are 0.250\u00a0mm).

\n", "Title": "How to calculate the proper layer height multiples?", "Tags": "|creality-ender-3|z-axis|knowledgebase|", "Answer": "

I see you've already accepted an answer, but based on your comments I think you have some misunderstandings of the topic which are worth clarifying as part of answering this question.

\n\n
\n

0.2125 layer height (+1/4 microstep) and doing all the movements in absolute movements instead of relative forced the result, as the target heights were as a result at 0.2125 mm (for the stepper that's effectively a 0.2 mm), 0.425 (0.4), 0.675 (for the stepper that's, depending on rounding or truncting, 0.6 or 0.7), 0.9 (here they are both 0.9) and so on.

\n
\n\n

It sounds like your understanding is that the stepper driver is \"rounding\"/\"truncating\" to Z positions that are multiples of 0.1 mm. Perhaps that's based on the LCD status display of the Ender 3's firmware, which is based on Marlin 1.0 or something around that version, and shows current coordinates rounded or truncated (I forget which) to one decimal place. This does not have anything to do with the positioning limitations of the actual machine; it's just bad user interface design.

\n\n

The actual firmware position is translated from the floating point value in the gcode to the nearest step/microstep that the stepper driver can represent. With full steps being 0.04 mm, microsteps are 0.0025 mm (1/16 of a step). All of these positions are \"exact\" in a logical sense, but of course subject to physical limits of the mechanical parts and accuracy of microstepping. On the topic of microstepping accuracy, you should read How Accurate Is Microstepping Really? Most if not all models of the Ender 3 have A4988 stepper drivers, one of the chips reviewed in that article. But the important part is that there's no rounding/truncation to whole steps taking place. Rather, the stepper driver is trying to position the motor in between whole steps by balancing the magnetic fields pulling it in each direction, with the goal of producing a linear interpolation between the two adjacent full steps. How well it does this is a matter of the quality of the stepper drivers and the load on the motor.

\n\n

Back to your test, your layer height of 0.2125 mm is not one step plus 1/4 microstep. It's 5 steps (5 * 0.04 mm) plus 0.0125 mm which is 5 microsteps. This is probably a decent test - 5 is 1 mod 16, so you'll end up with a period-16 cycle of microstep positions, at 5, 10, 15, 4, 9, 14, 3, 8, 13, 2, 7, 12, 1, 6, and 11 microsteps mod the whole step. This is pretty close to a period of 3, so you might expect to see some irregularity from poor microstepping accuracy that looks periodic every 3 layers, or you might see it repeating only every 16 layers (every 3.4 mm). But regardless I would not expect delamination problems from this. I think there's another serious extrusion problem behind the photo in your question, and you should probably open a new question about what it might be.

\n" }, { "Id": "10993", "CreationDate": "2019-09-09T19:22:39.693", "Body": "

\"enter

\n\n

Hotend drops from 205 °C to 175 °C, and then the printer errors MINTEMP. How can I fix this? Fan is not starting yet.

\n\n
#define  DEFAULT_Kp 28.65\n#define  DEFAULT_Ki 2.80\n#define  DEFAULT_Kd 73.33\n\n#define HEATER_0_MINTEMP 5\n
\n\n

MINTEMP is 5 °C? It's impossible it's getting that cold.

\n", "Title": "Hotend drops to 175 \u00baC once extrusion starts (MINTEMP error reported)", "Tags": "|marlin|prusa-i3|", "Answer": "

Sudden drops in temperature curves like in your picture are usually missing readings on the thermosensor:

\n\n

Disconnecting the sensor from the board registers as min-temp of the sensor while shorting out as max-temp, so it triggers the mintemp/maxtemp security protocol: shutdown.

\n\n

One could disable mintemp/maxtemp protection, but that wouldn't solve the issue. The issue is usually the wiring of the sensor. To find the fault, put temperature on the sensor but keep the motors disabled. Watch the read temperature. Move the printhead manually and slowly. If it suddenly drops and throws the error, you might have a faulty lead close to the printhead. If not, you need to open the electronics box to check if the connection to the board is ok. In the worst case, you might need to exchange the whole temperature sensor.

\n" }, { "Id": "10998", "CreationDate": "2019-09-11T17:41:31.980", "Body": "

What is the difference between an original E3D V6 rather than a clone?

\n

I know an original would have its own advantages, since there is a huge price difference between the two. Will there be a huge difference in the output? By considering the following:

\n\n

If anyone has used both, please share your experience.

\n", "Title": "E3D V6 Original vs Clone", "Tags": "|print-quality|extruder|e3d-v6|", "Answer": "

While most components (cooling body, heatblock, heater cartridges, thermistors) of knockoff e3d-v6 hotends can be similar enough to not notice in a large part, or at least functionally the same. Note that I don't say the parts are necessarily interchangeable - a lot what is sold under the name is not what it claims to be. The main distinguishing difference is the heatbreak: a proper e3d v6 hotend is hard to manufacture, so let's compare the designs of the real and the knockoffs.

\n

Original

\n

e3D v6 heatbreaks are necked considerably and have a socket with an airgap to the cooling body in top to take the lining and prevent heat flow to the PTFE pipe. This makes it an "all-metal" hotend, because the liner stops in the cold zone, where the cooling block is screwed on, and thus gets the least heat to the bowden, allowing to print hotter than with a lined hotend. The cooling block and the heather block threads are also not the same: M6 and M7. They are flat at the front to securely butt the nozzle thread against.

\n

Their internal and external geometry looks like this:

\n

\"Original\"

\n

Knockoffs

\n

Knockoffs vary a lot how their heatbreak is constructed. Some are lined all the way through. Others are pass-through - and neither is an all-metal hotend. Some knockoffs are necked but use the same thread on both sides.

\n

gallery of knockoffs

\n

\"pass-through\"\"lined\"\"same\"Same

\n

Conclusion

\n

If you print a lot of high-temperature materials, the need for a proper all-metal hotend is there. Many knockoffs don't do this necking properly or invalidate its effect by lining the hotend all the way through.

\n

Note that the machining quality can be quite different.

\n

Due to sizing, not all knockoffs allow to mount a genuine heatbreak with knockoff cooler and heater block.

\n" }, { "Id": "10999", "CreationDate": "2019-09-12T06:20:21.207", "Body": "

Recently my prints come out very messy. I tightened everything up and checked the extrusion multiplier.

\n\n

I have an Ender 3 Pro which I use together with Cura 4.2.1 (and Octoprint). I print in PLA at 190\u00b0C. The print bed is set to 70\u00b0C. The Bed temperature is lower though, since I use a glas bed on top of the heated bed. I use a print cooling fan at 100%. The layer height is set to 0.2 mm, the line width 0.4mm from the 0.4mm nozzle. My retraction is 5mm at 45mm/s.

\n\n

Does anyone have any idea what the reason could be?

\n\n

\"enter

\n", "Title": "Very messy prints with lots of fuzz on edges", "Tags": "|creality-ender-3|print-failure|", "Answer": "

I figured out the problem. I reduced printing speed and printing temperature and the print came out decent. Still some issues but worlds better then before. I will try a few more settings to determine exactly what was the actual Issue. I will also try what @Fernando Baltazar recommended and update here what exactly changed the printing quality.

\n\n

\"enter

\n" }, { "Id": "11001", "CreationDate": "2019-09-12T08:48:42.980", "Body": "

I have an Ender 3 Pro, recently added a BLTouch v3, flashed a bootloader, and I'm now attempting to configure TH3D U1.R2.15 firmware.

\n\n

I have only modified configuration.h, haven't touched any other file, and configured it using the inline comments as a guide.

\n\n

Everything seems to work ok, apart from it doesn't do a grid/mesh level before the print. G29 has been added just after G28 in Cura in the Start gcode box.

\n\n

When I print something, this happens:

\n\n
    \n
  1. Bed + hotend heats up.
  2. \n
  3. X/Y get homed.
  4. \n
  5. Nozzle moves to the center.
  6. \n
  7. BLTouch slowly comes down to probe the bed twice to set the level based on that one single center point.
  8. \n
  9. Primes the filament and begins the print...
  10. \n
\n\n

I'm not sure what the correct sequence is but I would have thought it should probe 9 points in a grid as I have set #define EZABL_POINTS 3.

\n\n

Have I missed something in the config?

\n\n

configuration.h (have omitted all other printer sections to allow stackexchange to accept the rest of the file)

\n\n
/**\n* ************** How to use this firmware - READ THIS, yes actually read this. *********************************\n*\n* Uncomment means removing the 2 // in front of #define.\n* \n* FLASHING NOTES:\n* ALL slicers (Simplify3D, Cura, Slic3r, etc) or anything else that connects to the COM port must be CLOSED for the firmware to be flash.\n* If anything is connected to the COM port when flashing it will fail, typically \"access denied\" is listed in the error section of the Arduino IDE.\n* You MUST use the Included Arduino IDE to flash the firmware if on Windows and if you are on a Mac or Linux follow the guide in our knowledgebase to setup your IDE to work with the firmware.\n* \n* EZABL SETUP NOTES: \n* If you have EZABL uncomment the mount you are using with the printer. \n* If you have a custom/unsupported mount uncomment #define CUSTOM_MOUNT and enter your offsets \n* below in the CUSTOM MOUNT section. Refer to the EZABL guide to get your offsets.\n* \n* STEP 1:\n* Select the correct board from the tools menu for the printer you are flashing.\n* Read the printer title for the model you are flashing, it will show what board to select.\n*\n* STEP 2:\n* Uncomment the printer you want to flash. The printers are sorted A-Z by brand name.\n* If you are using the Creality Dual board with the Ender 3/Ender 5/CR-20 then read the specific section below in that printer section on how to do this.\n*\n* STEP 3: \n* Select the COM port your printer is on from the Tools menu. If you do not see the COM port try\n* downloading the latest drivers from the manufacturer or TH3D site on our knowledgebase.\n* \n* STEP 4:\n* Verify you have the correct board selected, printer model uncommented, and if you are using EZOUT and/or EZABL\n* the lines you need to use them are also uncommented.\n*\n* STEP 5:\n* Once you have your settings verified click the arrow in the upper left to upload to the board.\n*\n* STEP 6:\n* Reset your eeprom. You can send M502 then M500 to reset the EEPROM OR on the printer LCD go to \n* Control > Reset EEPROM to clear out the EEPROM to defaults.\n*\n* BOOTLOADER FLASHING NOTES:\n* For flashing your bootloader with an Uno make sure to select Arduino as ISP for the programmer\n* \n* There are other features in the TH3D Extras section so look there for V6 Hotend,\n* Bootscreen settings, Titan Extruder and more. You only need to edit this file.\n* \n* ERROR NOTES:\n* If you get errors flashing READ the message it gives you and double check that you selected\n* the correct board from the Tools menu in Arduino. Turn off any AV systems and reboot the computer.\n* \n* COMMUNITY REQUESTED FEATURES NOTE:\n* All features in the community requested features section are provided as-is with no support from TH3D.\n*/\n\n#ifndef CONFIGURATION_H\n#define CONFIGURATION_H\n#define CONFIGURATION_H_VERSION 010109\n\n//===========================================================================\n//============================ TH3D Configuration ===========================\n//===========================================================================\n\n// ONLY UNCOMMENT THINGS IN ONE PRINTER SECTION!!! IF YOU HAVE MULTIPLE MACHINES FLASH THEM ONE AT A TIME.\n\n//===========================================================================\n// Creality Ender 3 Options - Select 'Sanguino 1284p' from Tools > Board\n//===========================================================================\n#define ENDER3\n\n// If you are using our EZOut V1/V2 (connected to LCD header) filament sensor kit please follow the install guide\n// and then uncomment the #define EZOUT_ENABLE line below.\n// Do NOT ever connect our filament sensor without the supplied adapter board.\n//#define EZOUT_ENABLE\n\n// EZABL Probe Mounts (Ender 3 uses the same mounts as CR-10)\n//#define CR10_OEM\n//#define CR10_VOLCANO\n//#define CR10_V6HEAVYDUTY\n//#define TM3DAERO\n//#define TM3DAERO_EXTENDED\n//#define PETSFANG  //This is the RIGHT mounted version - if using the left mount please use the CUSTOM_PROBE option.\n#define CUSTOM_PROBE\n\n// TMC2208 Creality Board Setting - uncomment this to set the driver type if you are using the TMC Creality board\n//#define TMC_CREALITY_BOARD\n\n//=================================================================================================\n// README - THE BELOW SETTINGS ARE ONLY FOR USING THE CR-10S DUAL BOARD WITH THE ENDER 3\n// DO NOT UNCOMMENT THE ABOVE #define ENDER3 LINE IF USING THE DUAL BOARD\n// Select 'Arduino Mega 2560' from Tools > Board - NOT Sanguino\n//\n// To use the Ender 3 LCD with the CR-10S dual board board connect the LCD cable to EXP1 on the \n// CR-10S board but rotate it 180 degrees. The LCD end of the cable goes to EXP3 on the Ender 3 LCD.\n// You will have to force it into the EXP1 but it will fit and work.\n// \n// EZABL and EZOut support are still supported just use the lines above this comment section.\n//=================================================================================================\n//#define ENDER3_DUALBOARD\n\n// Dual Extruder Board Options - The below options are for use with the TH3D CR-10S Board with dual extruder support.\n\n// If you are using a single hotend with the 2 into 1 adapter OR mixing nozzle uncomment the below line\n//#define DUAL_EXTRUDER_SINGLE_HOTEND\n\n// If you are using a dual hotend with dual nozzles uncomment the below line\n//#define DUAL_HOTEND_DUAL_NOZZLES\n\n// If you are using a 2nd EZOut V2 (connects to Y+ connector) filament sensor kit please follow the install guide\n// and then uncomment the #define EZOUTV2_DUAL_ENABLE line below. Do NOT ever connect our filament sensor without the supplied adapter board.\n//#define EZOUTV2_DUAL_ENABLE\n\n// Use Ender Bootscreeen instead of TH3D\n//#define ENDER_BOOT\n\n//===========================================================================\n// *************************  END PRINTER SECTION   *************************\n//===========================================================================\n\n//===========================================================================\n// EZABL Advanced Settings\n//===========================================================================\n\n// If you want more or less EZABL probe points change the number below (only used if EZABL enabled)\n// Default is 3 which gives you 3x3 grid for a total of 9 points. STICK WITH ODD NUMBERS\n#define EZABL_POINTS 3\n\n// If you want to probe in on the bed more than 15mm change this below. \n// Do not use 30mm for the Standard CR-10/s or the S4 as you will be on the bed screws.\n// Try 50mm to avoid the binder clips if you use them. Do NOT go under 15mm here.\n// You can do down to 10mm on the Wanhao i3 since it cannot print on the entire bed.\n// You can do down to 5mm on the Wanhao i3 Mini since it cannot print on the entire bed.\n// (only used if EZABL enabled)\n#define EZABL_PROBE_EDGE 15\n\n// If you have issues with your machine running the faster probe setting disable the #define EZABL_FASTPROBE below.\n// DO NOTE: Most machines will work with the fast probe enabled. Use M48 to verify accuracy.\n#define EZABL_FASTPROBE\n\n// This will disable the XYE motors during probing. Can be useful if you have stepper motors causing interference issues with the EZABL sensor.\n//#define PROBING_MOTORS_OFF\n\n// Heaters will stay on during probing - only use if directed to by support. Do not use on AC beds.\n#define HEATERS_ON_DURING_PROBING\n\n// Letting the bed heat recover between probes can increase accuracy due to the bed warping during cooling/heating\n// Enabling the below option will let the bed get back to temperature during probing but will increase probing times.\n//#define WAIT_FOR_BED_HEATER\n\n// If you want a more granular control over the babystepping uncomment the below line.\n// This will make the adjustment finer than the standard setting.\n//#define FINE_BABYSTEPPING\n\n// This will extrapolate the implied tilt of the bed outside of the probe area. Do not comment out unless directed by support.\n#define EZABL_OUTSIDE_GRID_COMPENSATION\n\n// Does your machine make weird noises/vibrations when it is probing the mesh? Enable this to slow down the speed between probe points.\n//#define SLOWER_PROBE_MOVES\n\n//================================================================================\n// IF YOU HAVE A CUSTOM PROBE MOUNT OR ONE THAT IS NOT PRE-SUPPORTED UNCOMMENT THE\n// CUSTOM_PROBE OPTION IN YOUR PRINTER SECTION AND ENTER YOUR PROBE LOCATION BELOW\n//================================================================================\n#if ENABLED(CUSTOM_PROBE)\n  /**\n  *   Z Probe to nozzle (X,Y) offset, relative to (0, 0).\n  *   X and Y offsets must be whole numbers.\n  *\n  *   In the following example the X and Y offsets are both positive:\n  *   #define X_PROBE_OFFSET_FROM_EXTRUDER 10\n  *   #define Y_PROBE_OFFSET_FROM_EXTRUDER 10\n  *\n  *      +-- BACK ---+\n  *      |           |\n  *    L |    (+) P  | R <-- probe (10,10)\n  *    E |           | I\n  *    F | (-) N (+) | G <-- nozzle (0,0)\n  *    T |           | H\n  *      |    (-)    | T\n  *      |           |\n  *      O-- FRONT --+\n  *    (0,0)\n  */\n  #define X_PROBE_OFFSET_FROM_EXTRUDER -44  // X offset: -left  +right  [of the nozzle]\n  #define Y_PROBE_OFFSET_FROM_EXTRUDER -7  // Y offset: -front +behind [the nozzle]\n#endif\n\n//===========================================================================\n//******************** EXTRA FEATURES AND TWEAKS ****************************\n//===========================================================================\n\n// TH3D RGB LED STRIP ------------------------------\n// If you are using the TH3D RGB strip connect to the Z+ endstop connection, power supply connection, and uncomment the below line\n//#define TH3D_RGB_STRIP\n// If you cut the strip shorter please count the LEDs that are left, uncomment the line below, and change the number below to how many LEDs you have.\n#define TH3D_RGB_STRIP_LED_COUNT 20\n\n// EXTRUDER SETTINGS -------------------------------\n\n// If you want to change the Esteps for your printer you can uncomment the below line and set CUSTOM_ESTEPS_VALUE to what you want - USE WHOLE NUMBERS ONLY\n#define CUSTOM_ESTEPS\n#define CUSTOM_ESTEPS_VALUE 138\n\n// If you are using an TH3D Tough Extruder, Bondtech BMG (set steps below to 415), or E3D Titan Extruder\n// uncomment the below line to setup the firmware to the correct steps and direction. Also applicable to Titan/Tough Aero setups.\n//#define TITAN_EXTRUDER\n#define TITAN_EXTRUDER_STEPS 463\n\n// DUAL HOTEND SETTINGS ----------------------------\n\n// This is the distance between each nozzle tip when using a dual hotend like the TH3D Tough Dual Hotend or the E3D Chimera or Dual hotends.\n// This setting only applies to printers using a dual extruder board.\n#define DUAL_HOTEND_X_DISTANCE 18.0\n\n// THERMISTOR SETTINGS -----------------------------\n\n// If you are using an E3D V6 Hotend with their cartridge thermistor (not glass version) uncomment the below line.\n//#define V6_HOTEND\n\n// If you are using a Tough Hotend from TH3D or any thermistors TH3D sells for your hotend uncomment the below line.\n//#define TH3D_HOTEND_THERMISTOR\n\n// If you are using a thermistor TH3D sells for your bed uncomment the below line.\n//#define TH3D_BED_THERMISTOR\n\n// If you are using a Keenovo with SSR and the Keenovo temperature sensor uncomment the below line.\n//#define KEENOVO_TEMPSENSOR\n\n// If you are using a known hotend thermistor value uncomment the below 2 lines and enter the thermistor number replacing the X after the #define KNOWN_HOTEND_THERMISTOR_VALUE\n//#define KNOWN_HOTEND_THERMISTOR\n//#define KNOWN_HOTEND_THERMISTOR_VALUE X\n\n// If you are using a known bed thermistor value uncomment the below 2 lines and enter the thermistor number replacing the X after the #define KNOWN_BED_THERMISTOR_VALUE\n//#define KNOWN_BED_THERMISTOR\n//#define KNOWN_BED_THERMISTOR_VALUE X\n\n// If you want to make thermal protection periods less or more adjust below. The number is in seconds.\n// If you are getting false thermal runaway then increase the protection time. Do not make it over 300 for either setting.\n#define HOTEND_THERMAL_PROTECTION_TIME 60\n#define BED_THERMAL_PROTECTION_TIME 180\n\n// BED SETTINGS ------------------------------------\n\n// If you want PID tuning on your bed you can enable the below line. But PID on a bed is not typically needed. By default BED PID is disabled.\n// This will be disabled when using automatic or manual mesh leveling with a 1284p board due to memory limitations.\n//#define PIDBED_ENABLE\n\n// If you are using an AC bed with a standalone controller (Keenovo) uncomment the below line to disable the heated bed in the firmware\n//#define AC_BED\n\n// Stock bed max is 120C for this firmware. Enable this to allow temps up to 150C. Your bed must support this temp for it to achieve the higher temperatures.\n//#define BED_HIGHTEMP\n\n// MISC --------------------------------------------\n\n// If you have a 5015 fan that whines when under 100% speed uncomment the below line.\n//#define FAN_FIX\n\n// Use your own printer name\n//#define USER_PRINTER_NAME \"Ender 3\" \n\n// If your printer is homing to the endstops hard uncomment this to change the homing speed/divisor to make it less aggressive.\n//#define SLOWER_HOMING\n\n// BOOT SCREEN OPTIONS -----------------------------\n\n// Use TinyMachines Bootscreen instead of TH3D\n//#define TM3D_BOOT\n\n// Disable Bootscreen completely\n#define DISABLE_BOOT\n\n//===========================================================================\n//****************** COMMUNITY REQUESTED FEATURES ***************************\n//===========================================================================\n\n// HOME OFFSET ADJUSTMENT --------------------------\n// If you need to adjust your XY home offsets from defaults then you can uncomment the HOME_ADJUST_LOCATION line below and enter your\n// custom XY offsets. This is provided for convenience and is unsupported with included product support.\n// How to use - measure (home XY then jog using the LCD 1mm at a time) the X and Y distance the nozzle is off\n// the build plate and then put those as NEGATIVE values below, positive values will NOT work (move your endstops to fix a positve offset).\n//#define HOME_ADJUST\n#define X_HOME_ADJUST_LOCATION -10\n#define Y_HOME_ADJUST_LOCATION -10\n\n// LINEAR ADVANCE ----------------------------------\n// See here on how to use Linear Advance: http://marlinfw.org/docs/features/lin_advance.html\n//\n//#define LINEAR_ADVANCE\n// Change the K Value here or use M900 KX.XX in your starting code (recommended).\n#define LINEAR_ADVANCE_K 0\n// NOTE: If using linear advance along with EZABL on a printer with 1284p some Control > Motion menus will not be displayed due to space restrictions.\n// You can still change these via GCode commands.\n\n// BL TOUCH ----------------------------------------\n// If you want to use the BL-Touch install your EZOut Board, uncomment the 2 lines below, uncomment the CUSTOM_PROBE option in your printer section, \n// and then enter your probe offsets in the CUSTOM_PROBE section above. The Pin 27 boards on eBay are clones of our original EZOut. If you want to \n// support the people that originally came up with the board you can get our EZOut breakout board here: http://EZOut.TH3DStudio.com\n// Sales from our shop allow us to allocate time for community firmware development at no charge to you. <3\n// If you have a V3 BL Touch also uncomment the BLTOUCH_V3 line to fix issues with the new V3 probe.\n//\n#define BLTOUCH\n#define BLTOUCH_V3\n// Here is where you set your servo pin. EZOut Servo Pin Numbers: Ender3/5/CR-10 - 27, Ender 2 - 29. For 2560 boards look for the pin you connected the servo wire to and enter below.\n#define SERVO0_PIN 27\n//\n// NOTE: On 1284p boards due to space limitations and the large amount of code the BLTouch requires for the LCD Menus\n// the Bootscreen and some Control > Motion menus will not be displayed due to space restrictions\n\n// MANUAL MESH LEVELING ----------------------------\n// If you want to use manual mesh leveling you can enable the below option. This is for generating a MANUAL mesh WITHOUT a probe. \n// Mesh Bed Leveling Documentation: http://marlinfw.org/docs/gcode/G029-mbl.html If used with a 1284P board the bootscreen will be disabled to save space.\n// NOTE: If you want to automate the leveling process our EZABL kits do this for you. Check them out here: http://EZABL.TH3DStudio.com\n//#define MANUAL_MESH_LEVELING\n\n// POWER LOSS RECOVERY -----------------------------\n// Continue after Power-Loss feature will store the current state to the SD Card at the start of each layer\n// during SD printing. If this is found at bootup it will ask you if you want to resume the print.\n//\n// NOTE: This feature causes excessive wear on your SD card. This will disable junction jerk,  SCurve Acceleration, and Linear Advance due to RAM limitations.\n//#define POWER_LOSS_RECOVERY\n\n// MOTION SETTINGS ---------------------------------\n// There are 2 new acceleration/jerk controls available in this firmware that can result is better print quality and\n// smoother movement. To try each out just enable them below.\n// Due to Anet board restrictions this is always disabled on those machines.\n//#define JUNCTION_DEVIATION_ON\n//#define S_CURVE_ACCELERATION_ON\n\n//================================================================================================\n// Language - This is provided for convenience and is unsupported with included product support.\n// We only test compile with English language. If you run into space issues disable some features.\n//================================================================================================\n\n/**\n * LCD LANGUAGE\n *\n * Select the language to display on the LCD. These languages are available:\n *\n *    en, an, bg, ca, cn, cz, cz_utf8, de, el, el-gr, es, eu, fi, fr, fr_utf8, gl,\n *    hr, it, kana, kana_utf8, nl, pl, pt, pt_utf8, pt-br, pt-br_utf8, ru, sk_utf8,\n *    tr, uk, zh_CN, zh_TW, test\n */\n\n#define LCD_LANGUAGE en\n\n#include \"Configuration_beta.h\"\n#include \"Configuration_backend.h\"\n\n#define UNIFIED_VERSION \"TH3D U1.R2.15\"\n\n#endif // CONFIGURATION_H\n
\n\n

Cura Start GCode

\n\n
M201 X500.00 Y500.00 Z100.00 E5000.00 ;Setup machine max acceleration\nM203 X500.00 Y500.00 Z10.00 E50.00 ;Setup machine max feedrate\nM204 P500.00 R1000.00 T500.00 ;Setup Print/Retract/Travel acceleration\nM205 X8.00 Y8.00 Z0.40 E5.00 ;Setup Jerk\nM220 S100 ;Reset Feedrate\nM221 S100 ;Reset Flowrate\n\nG28 G29 ;Home\n\nG92 E0 ;Reset Extruder\nG1 Z2.0 F3000 ;Move Z Axis up\nG1 X10.1 Y20 Z0.28 F5000.0 ;Move to start position\nG1 X10.1 Y200.0 Z0.28 F1500.0 E15 ;Draw the first line\nG1 X10.4 Y200.0 Z0.28 F5000.0 ;Move to side a little\nG1 X10.4 Y20 Z0.28 F1500.0 E30 ;Draw the second line\nG92 E0 ;Reset Extruder\nG1 Z2.0 F3000 ;Move Z Axis up\n
\n", "Title": "G29 Auto Grid Leveling Doesn't Do Anything", "Tags": "|marlin|creality-ender-3|bltouch|", "Answer": "

G-code is read line-by-line, see e.g. Is G-code read line by line?. If the posted start G-code is the actual used G-code, G29 is not executed because it is on the same line as G28. This means that in:

\n\n
G28 G29 ;Home\n
\n\n

G29 simply is ignored (it is not recognised by the interpreter as parameters for G28). This should be changed into:

\n\n
G28 ;Home\nG29 ;Perform bed leveling\n
\n" }, { "Id": "11003", "CreationDate": "2019-09-12T13:06:02.193", "Body": "

I've just done my first print on an Ultimaker 3 Extended and I'm slightly disappointed with the results, so I'm trying to understand how I can do better next time.

\n\n

My first print

\n\n

Preparing for printing I naively just dropped the two stl files into Cura, set the recommended layer height and infill, selected support (defaulting to Extruder 1), deselected adhesion, ran the slice, saved the g-code and started the print running. Luckily the head 1 did have the same PLA AA 0.4 filament that Cura assumed.

\n\n

When the print finished, I stripped out the support structures, cleaning out the hollow, and cleaning off lots of stringy loose filaments between the lower support and the bottom edge of the print.

\n\n

Even after cleaning up though, the overhanging structure above the support structure turned out to be very rough with many individual filaments visible and in irregular positions, rather than the nice concentric lines in the slice.

\n\n

My first attempt to optimise the print

\n\n

Looking at the completed print I realised that there would have been only a fraction of the support structure, and probably clean edges, if the part had been oriented as a d rather than as a p (the rough surface being the bottom of the p overhang).

\n\n

I re-ran the slice in d orientation and that saved 10 minutes of print time, and a 100 mm of filament, so I know I'll definitely need to look out for that in the future. I can also see how that would fix the problem with the external overhang separating out into loose threads, since that face would no longer be an overhang.

\n\n

Trying to add water soluble supports

\n\n

After the first fix, I wondered what I could do with the second extruder and realised that it was filled with water soluble PVA filament. This made me wonder if this would have helped with the internal overhang.

\n\n

I Configured Extruder 2 as PVA BB 0.4 and selected Extruder 2 for the support structures and re-ran the slice.

\n\n

I was surprised that the it was now taking 40 minutes longer and using almost 470 mm more filament!

\n\n

Looking at the slices, Cura had created a huge PVA scaffolding on the outside of the print, leaving the inside, where the previous PLA support had been, completely empty:

\n\n

\"Printvs.\"Print

\n\n

This was not what I was expecting.

\n\n

Questions

\n\n\n", "Title": "Understanding water soluble support structures", "Tags": "|ultimaker-cura|pla|slicing|pva|ultimaker-3|", "Answer": "
\n

What is the reason for the external scaffolding...?

\n
\n\n

Reading through the Ultimaker support page, I discovered that there is a Support horizontal expansion option in the Support section of the Custom Profile.

\n\n

This appears to default to 0 mm for PLA, but defaults to 3 mm for PVA, which explains the difference in slicing behaviour.

\n\n

If I set Support horizontal expansion to 0 mm, then I get the support I originally expected:

\n\n

\"Print

\n\n

Ultimately though, the solution proposed by Trish and detailed in Oscar's answer using PVA just at the interfaces would be a much better solution, given the cost of PVA.

\n" }, { "Id": "11018", "CreationDate": "2019-09-15T14:15:12.030", "Body": "

The handle of a micro wave oven broke.

\n\n

I can't just order a replacement part because I can't even attach the new one.

\n\n

The problem is that the screw heads are somewhere on the interior side of the door, which cannot be disassembled (non destructively at least). I wouldn't even do it because of safety reasons.

\n\n

I have access to the threads of two loose and captive screws to work with (indicated by the two red lines on picture one). The screws are not machine screws, but screws for plastic like in the attached picture.

\n\n

The plan is to 3d print the plastic part of the handle and reuse the front aluminum cover.

\n\n

I don't want the handle to be loose, so I'm looking for suggestions to attach the new handle. I have a lot of ideas, maybe I will share them later if they are not mentioned at some point. The main problem is: how to attach something when all I have to work with is a loose, non machine screw ?

\n\n

\"enter\n\"enter

\n", "Title": "Attach handle on a loose screw", "Tags": "|3d-design|material|", "Answer": "

If you have enough screw length exposed, clamp onto them via a 2-piece 'nut' that is itself bolted together via fasteners that run perpendicular to the screws. You can then fasten whatever to the 'nut' and maybe even hide the gaggle of fasteners that results. Another alternative for applying the clamping forces needed might be something based off a \"toggle wing nut.\" I do not believe you will get good results via 1-piece print solutions the way the screws are.

\n\n

Loads and nut material will determing how much screw you need inside the 'nut.' The more length you have to work with on top of that, the more room you have to hold the screws' rear ends w/ pliers/vice grip while getting your 'nut' on there.

\n\n

If you don't have enough length, you will just have to use a different method of attaching to the door. Options may include adhesives and/or modifying the actual door - pretty annoying if you can't take it apart.

\n" }, { "Id": "11029", "CreationDate": "2019-09-17T21:04:59.867", "Body": "

I have replaced the stock extruder on my Ender 3 with one of these:

\n\n

\"New

\n\n

The grip gear has a smaller diameter, so I calibrated the esteps as per the top google search: Extruder Calibration \u2013 6 Easy Steps to Calibrate Your Extruder...

\n\n

If I set the esteps so that it's spot on with 100 mm of filament is used up when I ask it to extrude 100 mm, then during a print I get the occasional skip on the extruder.

\n\n

If I dial it back a bit and set it so that it extrudes 90 mm of filament when I ask it to extrude 100 mm, then I don't get the skips.

\n\n

In both cases the print looks normal.

\n\n

I've tried changing the nozzle as well in case there was some blockage, but it doesn't make a difference.

\n\n

Should I just go with the under extrusion? or is these likely to be some other problem that isn't apparent?

\n\n

I didn't notice any issues with the stock extruder and the stock estep setting, but I didn't think to check the calibration.

\n", "Title": "Calibrated Esteps Causes Extruder Skipping", "Tags": "|extruder|creality-ender-3|calibration|", "Answer": "

I have this same extruder on my Voxelab Aquila.\nDefault E-steps are 93. Using the printer menu to feed 100\u00a0mm of eSun black PETG filament I measured a feed length of 66\u00a0mm, so 34\u00a0mm under extruded.

\n

Using the formula 100 / 66 * 93 gave me a new E-step value of 140.9.\nUpon re-testing, I found I was now over extruding by 34mm.

\n

This formula is missing something. I've chased E-step values up and down before so decided to try something different.

\n

Since the new E-step value was over extruding by the same as the original E-step under extruding value, I added the original E-step value to the new E-step value and took the average. 93 + 140.9 / 2 = 116.9.\nI re-tested this value and found it was over extruding by 0.4\u00a0mm. Since I was still over extruding slightly I calculated a new E-step, 100 / 100.4 * 116.9 = 116.4. Took the average between the 2 again and got a final E-step value of 116.6.

\n

Hope this helps anyone else having issues calibrating E-steps.

\n" }, { "Id": "11038", "CreationDate": "2019-09-19T01:26:06.760", "Body": "

Contrary to a lot of other corner related problems (where the corners are bulging), I seem to have a different problem where the corners (ONLY) seem to stick out and appear blobby in the x/y plane. This only happens for corners/edges with a fillet radius greater than 3-4mm and only in the x/y plane. Anything smaller than that radius (such a sharp corner/edge) seems to be fine.

\n\n

Any ideas what could be causing this?

\n\n

Conditions

\n\n\n\n

What I've tried already

\n\n\n\n

Thank in advance for any ideas/suggestions

\n\n

\"blobby

\n\n

\"more

\n", "Title": "What causes these blobby corners", "Tags": "|print-quality|ultimaker-cura|calibration|abs|", "Answer": "

I suspect you are printing through a usb or network connection, and the communication rate it's to slow for any of many reasons. A curve consists of many tiny linear movements, each requiring a command exchange between the PC and printer.

\n\n

If you can, try printing from an sd card plugged into the printer (I'd the printer is so equipped).

\n\n

This could be worse if the uses a Bowden extruder, since there is now compression and windup in the filament.

\n" }, { "Id": "11043", "CreationDate": "2019-09-19T13:49:20.723", "Body": "

Thermal conductivity is how well a plastic conducts heat. Most plastics don't conduct heat very well at all, which is what allows them to be 3D printed. That being said, there are a lot of potential use cases for highly thermally conductive filament, assuming you could print them. A commonly discussed one is computer heatsinks. Similar heatsinks could also be used for stepper motors and extruders in 3d printing.

\n\n

To get a good picture which plastics are useful in such an application (like mentioned in question: \"Water-cooling stepper motor with aluminum block\"), I need to know what is the thermal conductivity of the commonly used thermoplastics.

\n", "Title": "What is the thermal conductivity of various 3D printing filaments?", "Tags": "|filament|filament-choice|", "Answer": "

Trimet3d has a Nano diamond PLA with a claimed thermal conductivity 3-5 times that of PLA. The diamonds are sub-microscopic and smooth. See https://www.tiamet3d.com/product-page/ultra-diamond-pla-1kg\nPrimarily they seem to be doing it to gain strength.

\n\n

I would prefer larger diamonds for higher thermal conductivity. It would be ferociously abrasive even for a diamond nozzle. It would also limit the thermal expansion.

\n" }, { "Id": "11046", "CreationDate": "2019-09-19T22:37:23.797", "Body": "

I am trying to configure Marlin for a Ramps 1.6 board. I have an I2C 4x20 character LCD enabled. I have the LCD working with

\n\n
#define LCD_SAINSMART_I2C_2004\n
\n\n

I have a separate encoder, with push button, but I can't find where to enable it. It seems that combo display-encoder-buzzer boards are enabled as a set. Some common name, like the define I used for the display. But I can't find any useful information that includes a I2C LCD with an encoder.

\n\n

I'm looking for a configuration that would enable my LCD and encoder, or just some way to enable the encoder by itself.

\n\n

For some configurations I found the following. But when I connect my encoder to those pins nothing happens when I try it. So I think I need to enable them somewhere.

\n\n
//encoder pins  \n#define BTN_EN1 31 \n#define BTN_EN2 33  \n#define BTN_ENC 35\n
\n", "Title": "Where to enable user interface encoder in Marlin?", "Tags": "|marlin|", "Answer": "

It seems that ULTIPANEL must be defined. The code for the encoder was included long ago with the Ultipanel (display and encoder) and does not exist on its own.

\n\n
#define LCD_SAINSMART_I2C_2004   //  I2C LCD\n\n#define ULTIPANEL                // enable code for encoder.\n\n//encoder pins\n#define BTN_EN1 37\n#define BTN_EN2 35\n#define BTN_ENC 31\n
\n" }, { "Id": "11051", "CreationDate": "2019-09-20T02:17:29.713", "Body": "

In Finishing sunken text in 3D prints, I found that nail polish works great for filling sunken lettering in 3D-printed parts, but the procedure for using it requires removing the excess from the non-sunken surface using nail polish remover. For PLA, acetone works fine, but with ABS the acetone will obviously dissolve the ABS too (and from my experience trying to get polish off plastic toys made of ABS, it mixes the polish with the ABS really nastily in the process).

\n\n

Is there an alternate nail polish remover that's suitable for use on ABS? Ethyl acetate based ones, maybe?

\n", "Title": "Nail polish remover that won't dissolve ABS?", "Tags": "|abs|print-material|post-processing|", "Answer": "

First of all, Nail Polish comes in 2 types: Acetone based, and acetone-free. The latter is based on Ethyl Acetate filled up with alcohol and water. Sadly, Ethyl Acetate also dissolves ABS, so it is not an option. However, the acetone-free nail polish remover can be used to smooth PLA to some degree. Anyhow, we look for something that is putting Nail polish into solution but not ABS, so it can neither be among the able ketones (like acetone), nor an acetate ester, nor be aniline.

\n

Next up, Turpentine. Turpentine is a crazy mix of chemicals that we know to be able to turn most glues to goop and remove nail polish too. Sadly, Turpentine too can dissolve ABS, and it has been used in this fashion to create ABS Slurry/Cement as a bonding agent.

\n

But what do we actually have in nail polish that needs to be put in solution? Well, to a good part, most nail polishes are a Nitrocellulose lacquer. To keep it in solution, Ethyl Acetate (already ruled out) and Butyl Acetate are used. Sadly, that is an acetate ester, so nope, no gains here.

\n

So, we are left with one way: Mechanical removal. Nail polish is known to bond quite ok, but conventional nail polish lacquer (not the gel stuff) also is known to chip, and we can use that. Letting the lacquer harden and then using a sharp knife o remove most of the surplus, then polishing it up to very fine 1000 grit sandpaper is a possible way to go without needing to open the chemical toolbox. For the last postprocessing step, very short exposure to acetone vapor should restore the surface and put it to shine, including the lacquer.

\n

Other fillers?

\n

An alternative might be other filler materials. For a white filling, white Valejo putty might work, which is an acrylic bound marble dust filler, which can be wiped away with a wet cloth. It doesn't shrink, so gives flat surfaces. It could be colored with other acrylic paints, but the base color is marble-white.

\n

Tamiya Basic Putty is based on Ethylbenzene and often used with ABS models as a gap filler. It is available in several colors. As a downside, it does shrink a little during bonding. Ethylbenzene is not a solvent but incidentally, the same stuff that is used for making ABS so should be safe for the model.

\n" }, { "Id": "11056", "CreationDate": "2019-09-21T07:49:01.270", "Body": "

Often, the pre-generated G-code is enough for start and end. However, sometimes we want to have something different. In this case: how to generate an audible alert of something like 4 bleeps at the end of the print, after putting the printer into the end position and when the bed has reached a \"safe\" 30\u00a0\u00b0C?

\n", "Title": "Writing G-code: Audible Alert at end of print", "Tags": "|ultimaker-cura|g-code|", "Answer": "

Trish's version is good but wait for bed temperature being at 30\u00a0\u00b0C (M190 R30) before setting the temperature to 0\u00a0\u00b0C (M140 S0)

\n

So this is my version:

\n
G91 ;relative positioning\nG1 E-1 F300  ;retract the filament a bit before lifting the nozzle, to release some of the pressure\nG1 Z+0.5 E-5 X-20 Y-20 F9000 ;move Z up a bit and retract filament even more\nG28 X0 Y0 ;move X/Y to min endstops, so the head is out of the way\nG1 Y150 F5000 ;move completed part out\nM84 ;steppers off\nG90 ;absolute positioning\nM190 R30 ;waits until cooling to 30 \u00b0C\nM300 S300 P1000 ;beep\nM300 S300 P1000 ;beep\nM300 S300 P1000 ;beep\nM300 S300 P1000 ;beep\nM104 S0 ;extruder heater off\nM140 S0 ;heated bed heater off \n
\n" }, { "Id": "11068", "CreationDate": "2019-09-25T10:40:53.180", "Body": "

On a RAMPS 1.4 it is possible to mount 5 stepper motors: X,Y,Z steppers are used for cartesian movements, while E0 and E1 are provided for extruders control.

\n\n

I want to use E0 and E1 as additional movement axis in a robotical arm.

\n\n

Is it possible in Marlin and if not, how can one approach the problem?

\n", "Title": "How to extend Marlin to support additional movement axis?", "Tags": "|marlin|ramps-1.4|arduino-mega-2650|axis|", "Answer": "

I found many ways to re-task extruder steppers to \nbehave like cartesian-like controls:

\n\n\n\n\n\n
    M163 S0 P0.6    # Set stepper 0 to ratio 0.6\n    M163 S1 P0.4    # Set stepper 1 to ratio 0.4\n    M164 S2         # Create a virtual stepper with given settings\n    T2              # Select the virtual stepper\n
\n\n\n" }, { "Id": "11070", "CreationDate": "2019-09-25T17:05:29.053", "Body": "

Microstepping reduces the actual torque at partial steps, so I would like to increase the stepper torque as much as possible to ensure better behaviour when high acceleration is needed (I'm installing a heavier print bed).

\n\n

I know which drivers and which motors I have so I know the rated values and how to avoid exceeding them.

\n\n

Besides the additional heating, what are the downsides of using higher current OR current, provided (as said) that the ratings are not exceeded?

\n\n

For example: noise? vibrations? anything else?

\n", "Title": "What are the downsides of high stepper current OR power?", "Tags": "|stepper|", "Answer": "

There is a downside.

\n\n

If the current is too high, you lose the linearity of micro-stepping. If the highest current exceeds the maximum, then the highest current micro-stepping positions will collapse on each other.

\n\n

If you are within the specs of the motors, there are no motor-related problems caused by higher currents.

\n\n

But other problems can be caused by increasing the current. The current must be within the capacity of the motor drivers and their heat sink. If the drivers overheat, many will simply shut off, causing the motors to not move when they should, which will appear as misalignment on X, Y, or Y.

\n\n

Another problem could be higher vibration, which would show as stronger ringing when going around corners. The higher current will give higher torque and higher acceleration, which will cause more vibration.

\n\n

If you are increasing the current to support a bigger and heavier print bed, you will possibly have other troubles. Although the higher torque will be countered by the higher bed mass, the higher bed mass may be coupled with longer belts, which will be more stretchy. You may also be imposing forces over longer frame components, which may cause them to flex more.

\n" }, { "Id": "11071", "CreationDate": "2019-09-25T17:07:52.653", "Body": "

Which of these will heat a bed fastest?

\n\n\n\n

I originally thought Amperage was what mattered until I realized I needed a 24 V power supply to even heat my Lulzbot mini bed by one degree.

\n\n

I know voltage is used to determine insulation thickness on wires. But thin wires with high current in them also get hot. Is insulation thickness on wires only to prevent you from accidentally cutting through them and shocking yourself, or is it for heat reasons?

\n\n

I'd like to power my heated bed with a 19.5 V, 5 A power supply. It's just an old laptop charger - I want to reduce strain on my circuit. It's a big bed and I have a few other laptop chargers lying around so I'd prefer to choose the best one.

\n", "Title": "Does voltage or current affect the time to heat a bed more?", "Tags": "|heated-bed|", "Answer": "

Power is the thing that determines how quickly the bed heats up, nothing else.

\n\n

A specific bed will be defined by it's resistance, this is the only relevant factor which is a constant. Of course, you shouldn't run it at a much higher power than it was designed for, or at a significantly higher temperature (these are related, since the bed looses much more energy to the room as it gets hotter).

\n\n

All 'low voltage' beds will have the same sort of insulation requirements (effectively none, less than 36V is regarded as safe to touch unless the skin is also penetrated).

\n\n

The wires used to connect to the heated bed must have a significantly lower resistance than the bed itself (similarly the connectors). Otherwise the wiring overheats and the bed has to work harder (making the overheating worse). Using a heat bed designed for a higher voltage (and a matching higher voltage supply) puts less strain on the wiring because the current is reduced for the same power output.

\n\n

Since the resistance is constant, increasing a 12V power supply to 13V gives a ~17% power increase (with the same bed) because the current also increases.

\n" }, { "Id": "11074", "CreationDate": "2019-09-26T09:52:11.973", "Body": "

The slicing software knows exactly (given accurate E-step, filament diameter, nozzle diameter values) how much cold plastic is pushed in the extruder and how much mass is extruded.

\n\n

Why should the extrusion multiplier be changed from 100%? Shouldn't the slicer extrude already enough to fill the gaps left by the neighbouring perimeter (see rectangular vs recto-oval extrusion profile).

\n\n

As result of the \"why\" question, implicitly comes up the question \"how\" should I (scientifically, not try and fail) define the extrusion multiplier? it would appear that said value should not be touched if the rest is done properly.

\n\n

The only idea I can have is to print at 100% infill and then saw the part, to check how big are the voids between 4 neighbouring extrusions. Measuring the wall thickness seems very prone to errors (but again, is such calibration really needed?)

\n", "Title": "Why and how am I supposed to change the extrusion multiplier?", "Tags": "|slicing|calibration|extrusion|", "Answer": "

In an ideal world, you will not need to change this parameter once it is properly calibrated.

\n\n

In a non-ideal world, some filament may be out-of-spec, and some filament may slip when it passes through the extruder. So for a flex (or more flexible) filament, you might increase the extrusion multiplier a little to compensate. If the compensation is material specific, it should be consistent (and might even be advised by the filament manufacturer).

\n\n

You might want to increase/reduce the multiplier whilst printing the first layer (in lieu of fixing your bed leveling properly).

\n\n

Maybe you want to fine-tune the top surface (which will be 100% fill) and you prefer to slightly under-fill this (because over-fill results in more noticeable surface defects).

\n\n

It is for quick hacks or fine tuning, there is probably no scientific approach to setting any value other than 100%.

\n\n

When it comes to material specific variations (due to melt viscosity, thermal expansion or drive efficiency), there are many factors which affect the tuning. Machine geometry, temperatures, colourants (and other additives in the filament), as well as the base material.

\n" }, { "Id": "11079", "CreationDate": "2019-09-26T14:43:27.433", "Body": "

I just purchased an Anycubic Photon 3S LCD-based SLA 3D printer. I understand the need for cleaning and curing parts after printing. While the device is in transit, I'm looking to set up a workstation to use it with.

\n\n

Are there any recommendations for accessories to buy?: Particular containers to store materials in, particular products for cleaning prints (or cleaning the machine), particular lamps for curing, etc.

\n\n

I'm trying to avoid gotchas, and I've got time to visit hardware stores or make further online purchases before it arrives. I'm not asking about software or computing hardware, just things that I'll want to have \"on the bench\".

\n", "Title": "What do I need to go along with my new SLA 3D printer?", "Tags": "|sla|", "Answer": "

Safety Gear

\n

Gloves, you want to wear them whenever handling any resin. Single-use gloves are best - dispose of them after use. Consider them contaminated after touching anything in contact with resin and toss them before handling anything that shall not get in contact with resin. That includes door handles.

\n

A good idea is to also wear eye protection, as resin in the eyes could destroy them.

\n

While a dust mask might not be strictly necessary, it could reduce your exposure to the fumes of resin. Some resin fumes are known to create hypersensitivity.

\n

It is also a good idea to put the printer into a dedicated workspace that is well ventilated and not your primary living space. I strongly recommend reading both Best way to deal with Resin Printers in your living space and Safe way of disposing resin

\n

Post-Processing Station

\n

You may want to build a post-processing station. Most pieces can be sourced in any home depot store or made from household items, so I don't recommend specific brands but the requirements.

\n

Washing Station

\n

A typical post-processing station consists of at least 2 vats large enough to submerge your print volume in, so you can wash off your print in the first and then wash it with fresh liquid in the second. The typical liquids for cleaning are isopropyl alcohol and sometimes technical alcohol. Some resins demand special liquids that are specific to the type of resin. Best, the washing vats have securely sealing lids. Glass is preferably as it is easy to clean.

\n

To use the least amount of cleaning liquid, you might want to have a pair of needle spray bottles - one for each bath. Label them!

\n

To avoid spillage and ruining tables, a plastic table cloth can be a good addition. Fold it with the contaminated side onto itself for storage. A different solution would be to put the cleaning station onto a ceramic or steel surface, which can be easily cleaned after use.

\n

Curing Station/Chamber

\n

The next step is curing the print under direct exposure to a UV light source, somewhat akin to how gel nails are hardened. Sometimes the sun is enough.

\n

Since the resin residue from washing is now in the isopropyl alcohol (or other washing liquid), treat it as chemical waste. To reduce the waste of material, flock out the resin in it by exposing the liquid to the UV light and filter the result. The result is Isopropyl Alcohol with some remaining contaminants, which can be used again for the first rinsing step.

\n

Other Tools

\n

Besides cleaning and curing the print, you need to remove the print from the plate, so you need a spatula or scraper, which is reserved only for your SLA printer. Never use it on the build platform of your FDM printers and consider it contaminated with uncured resin after use. Best cure residue on it in the UV chamber and then physically chip off the hardened resin before handling it without gloves again.

\n

Similarly, a tool to stir the resin and remove flakes is often used, and some makers have special spatulas to clean the vat. Clean them well after use.

\n

You will want to have some nice snippets to remove the support structures at some spots and some pliers to break them free - safety first.

\n

Needle files and sanding paper for cleanup where the support stuck are a given.

\n

Resin Recovery

\n

Since the resin in the trays might harden over time, you'll want to have some sort of rig to hold the vat at a tilted angle upside down so it can flow out, back into the resin bottle. A cover might also help to reduce exposure and allow short time storage in the machine. Thingieverse has a couple of solutions for lids, pouring and filtering stations as well as other accessories. Look for those that fit your printer.

\n

The Resin should be stored in airtight and light-blocking bottles. As an extra security measure, you should store the resin in a closed cupboard to prevent light exposure through not totally opaque bottles.

\n

More on Re-using resin can be found here

\n

Further Reading/watching

\n\n" }, { "Id": "11081", "CreationDate": "2019-09-26T16:26:34.587", "Body": "

Are there any browser extensions or printers with OctoPrint built in that would allow me to print straight from the browser?

\n

Thinking of a workflow like this:

\n
    \n
  1. Make something with Tinkercad (or other online service)
  2. \n
  3. download stl or obj
  4. \n
  5. select print from bookmark or dropdown menu
  6. \n
  7. print is sent to printer and starts printing
  8. \n
\n", "Title": "Printing straight from the browser", "Tags": "|desktop-printer|octoprint|", "Answer": "

This works for OctoPI (OctoPrint on a Raspberry Pi).

\n
    \n
  1. Add the Samba package to your OctoPrint machine.
  2. \n
  3. open an SMB connection to that machine from your browser machine
  4. \n
  5. Save your STL to /home/pi/.octoprint/uploads on the Raspberry Pi.
  6. \n
\n

You can save directly from your slicer the same way.

\n" }, { "Id": "11084", "CreationDate": "2019-09-26T16:58:02.780", "Body": "

So I've switched the Trigorilla board in the printer with a SKR 1.3 with TMC2208 drivers and installed the latest Marlin 2.0, with a config based on this one. You can find the Configuration.h here, the only thing I changed in Configuration_adv.h was the pin of the hotend fan.

\n\n

Now when let the printer autocalibrate the delta settings, it tells me that the height is 141.35 mm, instead of the actual ~300 mm and I had to set the radius to 78 mm, instead of the actual 115 mm so that it doesn't try to probe outside the bed.

\n\n

What settings could I have set so horribly wrong that I get these results?

\n", "Title": "Getting wrong measurements on my Kossel Linear Plus after installing SKR 1.3 with Marlin 2.0", "Tags": "|marlin|delta|kossel|skr-v1.3|", "Answer": "

So the problem was that the TMC2208 were wired for UART mode, yet Marlin was configured for standalone, which apparently makes them work, but with completely wrong step sizes. Changing it in the configuration completely eliminated the problem

\n" }, { "Id": "11090", "CreationDate": "2019-09-27T05:21:11.190", "Body": "

I have mounted two radial fan on my printer as a part cooling solution.

\n\n

\"radial

\n\n

As you can see, the fan has input on the left side and blows air down. Does a mirror construction exists? With outlet on the right.

\n\n

I can even print my own casing, but I'm not sure if the fan will work, if I change the rotation direction.

\n\n

I'm using this print cooling fan duct: https://www.thingiverse.com/thing:1850163

\n\n

The fan on the right side has the opening facing the hotend, and there is not much space, so the impeller can catch on wiring etc. If the right fan had opening to the right, there would be no such problem.

\n", "Title": "Does right handed radial fan exist?", "Tags": "|print-fan|", "Answer": "

I did also some research on this and decided to go with this solution. This fan only measures 50x50x10mm and is easy flippable: https://de.aliexpress.com/item/1005001894771961.html

\n

\"enter

\n

Another option was this: https://de.aliexpress.com/item/4001185014078.html

\n

\"enter

\n

Also found a Thingi, where people tried to flip the existing 5015 blower fans. It seems very difficult, since you have to print the fins in flipped direction and they tend to break.. https://www.thingiverse.com/thing:3716277

\n

\"enter

\n" }, { "Id": "11096", "CreationDate": "2019-09-29T03:13:45.687", "Body": "

I'm using a Prusa Slicer with a FlyingBear Ghost 4 and got a wrong printing alignments on the Y axis only on a part of the printing object. What may cause this?

\n\n

Here are the printed model pictures showing the problem:

\n\n

\"Photo

\n\n

\"Close

\n\n

Here are the settings:

\n\n
# generated by PrusaSlicer 2.1.0+win64 on 2019-09-29 at 11:37:50 UTC\navoid_crossing_perimeters = 0\nbed_custom_model = \nbed_custom_texture = \nbed_shape = 0x0,255x0,255x210,0x210\nbed_temperature = 60\nbefore_layer_gcode = \nbetween_objects_gcode = \nbottom_fill_pattern = rectilinear\nbottom_solid_layers = 7\nbridge_acceleration = 0\nbridge_angle = 0\nbridge_fan_speed = 100\nbridge_flow_ratio = 1\nbridge_speed = 60\nbrim_width = 0\nclip_multipart_objects = 0\ncolorprint_heights = \ncomplete_objects = 0\ncooling = 1\ncooling_tube_length = 5\ncooling_tube_retraction = 91.5\ndefault_acceleration = 0\ndefault_filament_profile = \"\"\ndefault_print_profile = \nderetract_speed = 0\ndisable_fan_first_layers = 3\ndont_support_bridges = 1\nduplicate_distance = 6\nelefant_foot_compensation = 0\nend_filament_gcode = \"; Filament-specific end gcode \\n;END gcode for filament\\n\"\nend_gcode = M104 S0 ; turn off temperature\\nG28 X0 Y0 ; home X and Y axis\\nM84     ; disable motors\\n\nensure_vertical_shell_thickness = 0\nexternal_perimeter_extrusion_width = 0.45\nexternal_perimeter_speed = 50%\nexternal_perimeters_first = 0\nextra_loading_move = -2\nextra_perimeters = 1\nextruder_clearance_height = 20\nextruder_clearance_radius = 20\nextruder_colour = \"\"\nextruder_offset = 0x0\nextrusion_axis = E\nextrusion_multiplier = 1\nextrusion_width = 0.45\nfan_always_on = 0\nfan_below_layer_time = 60\nfilament_colour = #FFFFFF\nfilament_cooling_final_speed = 3.4\nfilament_cooling_initial_speed = 2.2\nfilament_cooling_moves = 4\nfilament_cost = 30\nfilament_density = 1.25\nfilament_deretract_speed = nil\nfilament_diameter = 1.75\nfilament_load_time = 0\nfilament_loading_speed = 28\nfilament_loading_speed_start = 3\nfilament_max_volumetric_speed = 0\nfilament_minimal_purge_on_wipe_tower = 15\nfilament_notes = \"\"\nfilament_ramming_parameters = \"120 100 6.6 6.8 7.2 7.6 7.9 8.2 8.7 9.4 9.9 10.0| 0.05 6.6 0.45 6.8 0.95 7.8 1.45 8.3 1.95 9.7 2.45 10 2.95 7.6 3.45 7.6 3.95 7.6 4.45 7.6 4.95 7.6\"\nfilament_retract_before_travel = nil\nfilament_retract_before_wipe = nil\nfilament_retract_layer_change = nil\nfilament_retract_length = nil\nfilament_retract_lift = nil\nfilament_retract_lift_above = nil\nfilament_retract_lift_below = nil\nfilament_retract_restart_extra = nil\nfilament_retract_speed = nil\nfilament_settings_id = \"FlyingBear Ghost 4\"\nfilament_soluble = 0\nfilament_toolchange_delay = 0\nfilament_type = PLA\nfilament_unload_time = 0\nfilament_unloading_speed = 90\nfilament_unloading_speed_start = 100\nfilament_wipe = nil\nfill_angle = 45\nfill_density = 20%\nfill_pattern = stars\nfirst_layer_acceleration = 0\nfirst_layer_bed_temperature = 60\nfirst_layer_extrusion_width = 0.42\nfirst_layer_height = 0.2\nfirst_layer_speed = 30\nfirst_layer_temperature = 210\ngap_fill_speed = 20\ngcode_comments = 0\ngcode_flavor = marlin\ngcode_label_objects = 0\nhigh_current_on_filament_swap = 0\nhost_type = octoprint\ninfill_acceleration = 0\ninfill_every_layers = 1\ninfill_extruder = 1\ninfill_extrusion_width = 0.45\ninfill_first = 0\ninfill_only_where_needed = 0\ninfill_overlap = 25%\ninfill_speed = 80\ninterface_shells = 0\nlayer_gcode = \nlayer_height = 0.1\nmachine_max_acceleration_e = 10000,5000\nmachine_max_acceleration_extruding = 1500,1250\nmachine_max_acceleration_retracting = 1500,1250\nmachine_max_acceleration_x = 9000,1000\nmachine_max_acceleration_y = 9000,1000\nmachine_max_acceleration_z = 500,200\nmachine_max_feedrate_e = 120,120\nmachine_max_feedrate_x = 500,200\nmachine_max_feedrate_y = 500,200\nmachine_max_feedrate_z = 12,12\nmachine_max_jerk_e = 2.5,2.5\nmachine_max_jerk_x = 10,10\nmachine_max_jerk_y = 10,10\nmachine_max_jerk_z = 0.2,0.4\nmachine_min_extruding_rate = 0,0\nmachine_min_travel_rate = 0,0\nmax_fan_speed = 100\nmax_layer_height = 0\nmax_print_height = 200\nmax_print_speed = 80\nmax_volumetric_speed = 0\nmin_fan_speed = 35\nmin_layer_height = 0.07\nmin_print_speed = 10\nmin_skirt_length = 0\nnotes = \nnozzle_diameter = 0.4\nonly_retract_when_crossing_perimeters = 1\nooze_prevention = 0\noutput_filename_format = [input_filename_base].gcode\noverhangs = 1\nparking_pos_retraction = 92\nperimeter_acceleration = 0\nperimeter_extruder = 1\nperimeter_extrusion_width = 0.45\nperimeter_speed = 60\nperimeters = 4\npost_process = \nprint_host = \nprint_settings_id = FlyingBear Ghost 4\nprinter_model = \nprinter_notes = \nprinter_settings_id = FlyingBear Ghost 4\nprinter_technology = FFF\nprinter_variant = \nprinter_vendor = \nprinthost_apikey = \nprinthost_cafile = \nraft_layers = 0\nremaining_times = 0\nresolution = 0\nretract_before_travel = 2\nretract_before_wipe = 0%\nretract_layer_change = 0\nretract_length = 2\nretract_length_toolchange = 10\nretract_lift = 0\nretract_lift_above = 0\nretract_lift_below = 0\nretract_restart_extra = 0\nretract_restart_extra_toolchange = 0\nretract_speed = 40\nseam_position = aligned\nserial_port = \nserial_speed = 250000\nsilent_mode = 1\nsingle_extruder_multi_material = 0\nsingle_extruder_multi_material_priming = 1\nskirt_distance = 6\nskirt_height = 1\nskirts = 1\nslice_closing_radius = 0.049\nslowdown_below_layer_time = 5\nsmall_perimeter_speed = 15\nsolid_infill_below_area = 70\nsolid_infill_every_layers = 0\nsolid_infill_extruder = 1\nsolid_infill_extrusion_width = 0.45\nsolid_infill_speed = 20\nspiral_vase = 0\nstandby_temperature_delta = -5\nstart_filament_gcode = \"; Filament gcode\\n\"\nstart_gcode = G28 ; home all axes\\nG1 Z5 F5000 ; lift nozzle\\n\nsupport_material = 1\nsupport_material_angle = 0\nsupport_material_auto = 1\nsupport_material_buildplate_only = 0\nsupport_material_contact_distance = 0.2\nsupport_material_enforce_layers = 0\nsupport_material_extruder = 1\nsupport_material_extrusion_width = 0.35\nsupport_material_interface_contact_loops = 0\nsupport_material_interface_extruder = 1\nsupport_material_interface_layers = 3\nsupport_material_interface_spacing = 0\nsupport_material_interface_speed = 100%\nsupport_material_pattern = rectilinear\nsupport_material_spacing = 2.5\nsupport_material_speed = 60\nsupport_material_synchronize_layers = 0\nsupport_material_threshold = 0\nsupport_material_with_sheath = 1\nsupport_material_xy_spacing = 50%\ntemperature = 210\nthin_walls = 1\nthreads = 4\ntoolchange_gcode = \ntop_fill_pattern = rectilinear\ntop_infill_extrusion_width = 0.4\ntop_solid_infill_speed = 15\ntop_solid_layers = 9\ntravel_speed = 150\nuse_firmware_retraction = 0\nuse_relative_e_distances = 0\nuse_volumetric_e = 0\nvariable_layer_height = 1\nwipe = 0\nwipe_into_infill = 0\nwipe_into_objects = 0\nwipe_tower = 0\nwipe_tower_bridging = 10\nwipe_tower_rotation_angle = 0\nwipe_tower_width = 60\nwipe_tower_x = 180\nwipe_tower_y = 140\nwiping_volumes_extruders = 70,70\nwiping_volumes_matrix = 0\nxy_size_compensation = 0\nz_offset = 0\n
\n", "Title": "Why is some part of the print not aligned?", "Tags": "|print-quality|y-axis|flyingbear-ghost-4|", "Answer": "

This looks like your problems are lack of retraction and having the slicer's \"overhangs\" setting enabled.

\n\n

Retraction is absolutely essential when your print has layers with multiple disconnected components, and even when it doesn't, it's usually important. Without retraction, material will continue to flow out of the nozzle after extrusion is supposed to have stopped, producing oozing/stringing and subsequent underextrustion (due to the material already having been lost elsewhere).

\n\n

The \"overhangs\" setting is actively harmful unless your printer firmware has \"linear advance\" feature and you have calibrated it properly. Otherwise, when reaching part of the perimeter that's an overhang, the print head motion will slow down (I think PrusaSlicer does 10-20 mm/s for overhangs), but extrusion will continue at the full previous rate until pressure in the nozzle subsides. This is what's causing all the ugly sagging loops around the tail. Disable this misfeature so that print speed remains constant between overhang and non-overhang parts of the perimeters.

\n" }, { "Id": "11103", "CreationDate": "2019-09-30T13:21:56.310", "Body": "

Taulman3D advertises a soluble support material (EVOLV3D\u2122 USM Universal Support Material) that handles temperatures appropriate for nylon and is completely safe to flush down the drain. There's very little about how to print with it successfully, however, or how to remove it after printing.

\n\n

Other than the documented (wide) print temperature range (200-250 °C), what do I need to know?

\n", "Title": "What do I need to know to successfully use Taulman/Dow EVOLV3D\u2122 USM Universal Support Material?", "Tags": "|filament|support-material|", "Answer": "

Advice From Taulman

\n\n

On emailing Taulman asking for advice (after getting some initial failed prints), they responded with the following:

\n\n
\n

[...W]e use the following settings. Support:

\n \n
    \n
  1. Flow 115 %
  2. \n
  3. Infill = 8-10 %
  4. \n
  5. 2 full surfaces at 100 %
  6. \n
  7. Support speed 50 % of print speed.
  8. \n
\n
\n\n

What I've learned trying to apply that advice:

\n\n\n\n

On Support Removal

\n\n

At Taulman's recommended 10 %-or-below infill percentage, any accessible parts can be mechanically removed. At 30 %, that's not really possible anymore -- though 5-10 minutes in boiling water might get the edges loosened up enough to let the bulk of the material be scooped away.

\n\n

If you're just going to let it sit in tap water that isn't being circulated, expect that to take a long time, and to have several cycles of scraping out material that's turned to a gel to allow more to be exposed.

\n\n

Using boiling water speeds the process substantially. If you're using USM with a nylon (or other material that's safe to bring to 100 °C), do that.

\n\n

How's Its Adherence To Nylon?

\n\n

Not as strong as its adherence to itself. If you print a ceiling of USM on top of support infill of nylon, it's possible for that ceiling to come away with the print head; thus, it reduces risk of failed prints to use USM for the infill as well.

\n\n

Thus, the \"2 full surfaces at 100 %\" advice given by Taulman above isn't (as I interpret it) just about ensuring that there's a successfully-printed support floor or ceiling; it's also about ensuring that there's enough surface area between the support floor and the nylon of the main print for them to adhere.

\n" }, { "Id": "11105", "CreationDate": "2019-09-30T13:42:27.720", "Body": "

Let's suppose I want to build a water block shaped exactly for my Ubis 13s hot end

\n\n

\"Ubis

\n\n

(similar to this instructable), or that I want to circulate water around specific objects to be cooled (like Water-cooling stepper motor with aluminum block).

\n\n

CNC allows the use of shaped radiators/water blocks, but it is too expensive for home projects and requires the ability to make a part at the first or second attempt, to keep costs low.

\n\n

3D printing allows me to design jackets which are perfectly matched to the intended part, including gaps for O-rings or matching valleys+ridges between the components which also would result in a mostly waterproof seal (just add grease or hot glue...). In this case, water would be in contact with the object to cool and also provide (where suitable) a better interface. Corrosion can be avoided with some additives (in case of aluminium, water should be acidic).

\n\n

Which common printing material is best for this application? Let's limit the question to materials that can be reliably printed at maximum 270 \u00b0C (nylon ok, but no polycarbonate or PEEK, for example).

\n\n

We are talking about very low pressure pumps, mechanical strength is of little relevance.

\n", "Title": "Which filament to use for an application involving water circulation?", "Tags": "|cooling|filament-choice|water-resistance|", "Answer": "

I saw a lot of selling PETG water cooling pipes.\nSo PETG will be the best choice considering all factors.

\n\n

In addition:\nYou could use any plastic with the oil instead of water for cooling medium.\nAlso automotive anti-freeze (G11, G12, G13 ...) designed to be non corrosive and much better than water in terms of cooling.

\n" }, { "Id": "11116", "CreationDate": "2019-10-03T17:55:12.583", "Body": "

Out of the blue my A6 printer displays a ERR MAXTEMP BED error and shows about 100 degrees on the bed even when just powered on.

\n\n

A quick thermistor check shows that it's working properly : about 80 kOhm at room temperature, similar to other units (didn't bother to really check the specs).

\n\n

Swapping the bed thermistor for the head thermistor connector does not change the temperature readings : the one connected to the bed input gets 100 degrees, while the other one works as expected, proving that the connector and thermosensor are ok.

\n\n

It has to be in the board. How do I fix it?

\n", "Title": "Anet A6 ERR MAXTEMP BED error", "Tags": "|heated-bed|anet-a6|repair|", "Answer": "

I'm posting the answer here hoping that will help anyone that encounters a similar issue. A post on another site indicated that these boards are known to be problematic.

\n\n

After some digging, I came up with the schematics of this part of the mainboard :

\n\n

\"enter

\n\n

I'm not an electronics genius, but clearly it's not the capacitor; so that leaves the pull-up resistor or the microcontroller.

\n\n

Since the printer is already unusable, no harm in trying to replace the resistor, which is located just next to the connectors; for the bed it was the second from left, but if you encounter the same issue with another input, you may need to locate the proper one.

\n\n

Make sure you have the proper tools and knowledge for removing and soldering SMD components; in my case it was a 4k7 resistor in 0805 footprint.

\n\n

Bottomline : replacing a less-than-one-cent resistor saves a month of waiting for the delivery of a $25 board.

\n" }, { "Id": "11118", "CreationDate": "2019-10-04T09:53:07.793", "Body": "

I thought about using a BLTouch probe to do 3D scans of objects.

\n\n

The objects to be suitable require smooth gradients and enough space for the probe, without anything hitting the print head/hot end.

\n\n

What tools do I need to control the printer and obtain such data? are there GCodes scripts for that?

\n\n

I would need to define a grid size, spacing, retraction, maximum expected height increase between adjacent points.

\n", "Title": "How to use a BLTouch or equivalent touch probe to 3D scan objects with a 3D printer?", "Tags": "|software|scanning|bltouch|3dtouch|", "Answer": "

I don't know the probe, but I have used a delta 3D printing machine (of my design) to scan a surface. It takes some time.

\n\n

Your resolution will be limited by the probe geometry of the touch probe tip. You want a Z-probe function that reports the Z-value of the point, not that simply sets Z= when the probe \"hits\". The RepRap firmware has this in the G30 command.

\n\n

You will need to write a loop that probes all points of interest. This is best done by writing a program (or script) that generates the G-code. I have a PERL script that generates the calibration mesh for my Delta machine, and I can share that with you.

\n\n

I edit the script for the area to be scanned. This includes (in my case) the radius and the resolution. I then run the script to produce G-code to do the probing.

\n\n

I send the G-code to the printer with Repetier-Host. Repetier-Host builds a log file of every character returned by the printer. The G30 command returns in the log the Z value wher ethe probe hit.

\n\n

After the G-code is fully executed, I run a PERL script on the log file to extract the Z height of each of the probed points.

\n\n

What you do with that data is up to you.

\n" }, { "Id": "11123", "CreationDate": "2019-10-05T23:16:59.607", "Body": "

I have just started with the FlashForge Creator Pro using the FlashPrint slicer software. I have gotten some nice prints but I am having an issue with a certain print from this model of a knife.

\n\n

See image below:

\n\n

\"enter

\n\n

Everything prints fine up to where the blade starts. As you can see, there is horizontal portion of the blade that juts out and, as I predicted, when it tries to print this, there is nothing for it to adhere to so it hangs there and when the extruder moves, it just pulls the filament around with it and creates a mess.

\n\n

The object file from Thingiverse is designed that way (with it standing upright) and there are pictures on the Thingiverse page of a successfully printed knife. As I am new to this, I am wracking my brain trying to figure out how to print this.

\n\n

I tried to cut the design and print the handle and blade separately (laying the blade down flat on the print surface) but the blade isn't perfectly flat on the sides so it doesn't lay flat. I suppose I could also just cut the blade and lay it vertically on the print surface but it seems as though it was designed to print in full in one piece.

\n\n

Any help or any pointers in the right direction would be greatly appreciated. Thanks!

\n", "Title": "Horizontal line z-axis not printing correctly", "Tags": "|troubleshooting|support-structures|flashforge-creator|flashprint|", "Answer": "

You'll notice in the screen capture that there is an icon marked \"supports.\" This is precisely the solution required. When selected, the software should construct a series of vertical pillars in the locations required to properly print the horizontal portion of the knife blade.

\n\n

It's common for Thingiverse models to have a notation Supports (yes/no) to indicate that this feature should be activated. Depending on the capability of your printer, some angles may require supports that are not required on someone else's printer model. There may also be an adjustment for angle reference to cease supports or to require. My printer will handle up to a 50\u00b0 from vertical without support. Beyond that, more filament is used, but the result is superior to printing without it, as you've discovered.

\n\n

Note also that support is typically categorized as \"from the bed only\" or \"everywhere.\" The former prevents support from being generated between vertically separated parts of the model, while the latter permits it. Some models may have, for example, a foot on the bed not requiring support, but the arm directly above the foot does. \"From the bed only\" means the arm would droop/fail as no support would be generated above that portion of the model.

\n\n

If you are using the ReplicatorG 0400 software, referenced from the manufacturer's web site, page 11 of the manual will have additional information regarding support within the program.

\n" }, { "Id": "11138", "CreationDate": "2019-10-08T23:03:27.653", "Body": "

Normally stainless steel is magnetic. But whenever i order stainless steel nozzles from Amazon, they are not magnetic. This makes me think they could be brass coated in something like aluminum. However, there are many types of steel.

\n\n

I've attached an image of someone who reviewed these nozzles. He says they are not stainless steel because they are not magnetic.

\n\n

\"enter

\n\n

But whenever I order \"stainless steel\" nozzles, even from other sellers, they are not magnetic. i already returned one pack from another seller and just received a nonmagnetic one from a third seller.

\n\n

So that's it? Amazon just sells junk now? Or is there a way I can easily tell whether these are something other than colored brass / good for abrasive or high temp printing. Here is the product that was reviewed: \nAUSTOR 13 Pieces Stainless Steel 3D Printer Nozzles 0.2 mm, 0.4 mm, 0.6 mm, 0.8 mm, 1.0 mm Extruder Nozzle Print Head for E3D Makerbot https://www.amazon.com/dp/B07CHZMGRH/ref=cm_sw_r_cp_apa_i_XtrNDbRMVH8SW

\n", "Title": "How to tell if a nozzle is truly stainless steel?", "Tags": "|metal-parts|", "Answer": "

The other answers are good. For a more practical answer:

\n

Magnet test

\n

As answered by Trish, steel with an austenitic atom structure, which most stainless steels have, are not magnetic. So forget the magnets. Metallurgy is complicated.

\n

\"enter

\n

Density test

\n

Brass has a density of 8.5 - 9, stainless steel is below 8. Depends on the alloy. Might be a bit tricky to detect such small differences.

\n

Hardness test

\n

Better to do a hardness test. Hammer a nail made of mild steel into the nozzle, if the nozzle is softer than the nail, it's brass. If it's harder, it's stainless steel.

\n

There are also HRC hardness scratch tests for that, although more intended for testing hardened steel. Find a chunk of structural steel and scratch the nozzle into it. Stainless steel will scratch the structural steel, brass & aluminium will scratch itself.

\n

Spark test

\n

Use an angle grinder or a dremel tool: if there are sparks, it's steel. This is probably the easiest method.

\n" }, { "Id": "11144", "CreationDate": "2019-10-09T18:21:33.707", "Body": "

I want to make Polyurethane molds for concrete using 3D printed PLA or ABS master object. like this video:

\n\n

\r\n \r\n
(this video is not about concrete of course!)

\n\n

I'm not sure if it will stick to PLA or ABS master or not! if it does stick, whick wax material can solve this problem... Do I need to print my masters with another filament?

\n\n

\"enter

\n", "Title": "Making Polyurethane molds with PLA (or ABS) 3D printed master", "Tags": "|pla|abs|molds|", "Answer": "

In my experience, polyurethane sticks to PLA like super glue, not good. But silicone and alginate doesn't stick at all.

\n

What I do is print the model of the mold with PLA or ABS, no matter. Then, cast a mold of the PLA model of the mold with alginate, then you have the negative of your mold.

\n

Now with this alginate mold of the mold cast your actual mold with silicone. And then you can cast your part on polyurethane in the silicone mold.

\n" }, { "Id": "11147", "CreationDate": "2019-10-10T15:00:44.160", "Body": "

I made two \"OBJ\" 3D models of myself using Sense 2 scanner and using photogrammetry with Meshroom.

\n\n

I would like to extract some information about myself to be able to scale the model properly and to be able to compare the accuracy. For example, distance between shoulders or circumference of the neck/chest. Another option would be to hold a reference parallelepiped and then scale the model based on the known distance between faces.

\n\n

How can I do that? I can calculate the distance between points I can click using Meshmixer, but circumference or the distance between parallel faces is more difficult. The tools Meshmixer offers for the purpose don't seem to work well.

\n\n

What are my options?

\n", "Title": "How to extract 3D information from a 3D model for calibration?", "Tags": "|3d-models|", "Answer": "

Extracting various measurements from 3D models is easily done using the following free online tool:

\n\n

https://0x00019913.github.io/meshy/

\n\n

I scanned a person using the Sense 2 scanner and I compared the values from meshy for the hip and waist with those measured in real life one day after the scan: the difference was about 1 cm, which can easily be explained by the uncertainties in the choice of the measurement point and moment of the day.

\n\n

This shows that both the 3D scanner and meshy are quite accurate. The tool is also very easy to use.

\n" }, { "Id": "11148", "CreationDate": "2019-10-11T05:23:08.063", "Body": "

I am new to 3D printing but have been in CNC Machining for a few years. I have a part I am trying to print that is a cylinder 1.000 in. in diameter and has a .200 in overhang starting at 1.300 in. In other words I am printing a 1.300 in. cylinder that is 1.500 in. tall that at 1.300 in. its diameter increases by .200 in.

\n\n

When I first printed the part the overhang had sunk or fallen out. Not by much and is still usable but made a crappy finish. What would I need to do in order to have the overhang not drop as the base layer extended outward .200 in. at 1.300 in.?

\n\n

I tried slowing the feed rate but that was worse. I also lowered the temp to 195 \u00b0C.

\n\n

I am using a Monoprice Select Mini running at 200 \u00b0C and a 1.0 Speed (Not really sure what that feed rate is in terms of mm/s). Based on what I've seen so far I would increase the speed and keep the temp at 200 \u00b0C.

\n\n

\"enter

\n\n

Any suggestions, I hope I have explained my problem well enough.

\n", "Title": "3D printing overhangs that are over .200 in", "Tags": "|print-quality|extrusion|support-structures|", "Answer": "

It appears that your part could be printable upside down. If possible, I'd highly recommend this, as it mostly avoids supports all together.

\n" }, { "Id": "11160", "CreationDate": "2019-10-13T14:54:44.200", "Body": "

I have a collection of STL files, each containing a separate moving part of an object I want to print. (Imagine a set of gears, or similar, that prints as a single object with multiple moving parts.)

\n\n

My plan was to import them all into Cura, then hit print, then take my fully assembled object off the build plate. However, Cura ignores the coordinate system in the STL files and automatically separates the components from each other on the build plate. This is usually helpful, but it isn't what I want in this case.

\n\n

So I'm looking for a quick and simple way to combine my multiple STL files into a single STL file. I know that the objects don't overlap, so I don't need to do a CSG union operation - it's enough just to concatenate the objects.

\n\n

I tried OpenSCAD, which works, but it takes a really long time, because the meshes are fairly complex and it does the full Boolean operation. Is there another quick and simple way to perform this task?

\n\n

I'd prefer a command line utility, but I'd also be happy if there's a quick and simple way to do it in some free graphical software. (However, I don't want to spend time manually positioning the objects - they're already in the right places in the STL files, so I just want to import them and go.)

\n\n

Edit I've accepted Trish's answer (use Blender), but I'd still appreciate a command-line option if anyone knows one.

\n", "Title": "Combining multiple STL files", "Tags": "|3d-models|software|stl|", "Answer": "

Import them to Tinkercad and then output your combined file from there. You can go into Tinkercad and on the top right side you will see \"import\", click that to import your file (you can repeat the process as desired) and it will put the files on the workspace .. on the screen. When you have your workspace all set the way you want select them all the click \"export\" and it will export all of them to one file for your use.

\n" }, { "Id": "11176", "CreationDate": "2019-10-15T11:26:28.527", "Body": "

When I print with PLA, I get a perfect first layer.

\n\n

However, when I print with PETG, the first layer looks like this:

\n\n

\"enter

\n\n

I've read all the info that suggests reducing the temp, speed, and increasing retraction... I've done all that which has improved things a lot, but I still get this... I can't seem to work out what's causing it.

\n\n

How do I get a perfect first layer with PETG?

\n\n

The latest settings that I've tried, and produced what you see in the picture are, using Cura 4.3 standard Dynamic Quality 0.16 mm profile with these tweaks:

\n\n\n\n

One thought I had, does PETG need a different clearance between the nozzle and the bed than PLA?

\n", "Title": "Bumpy first layer with PETG", "Tags": "|petg|", "Answer": "

When I print with PETG, my bed is 80\u00a0\u00b0C for first 2 layers then I drop it to 65\u00a0\u00b0C

\n

Extruder temp first two layers 240-250\u00a0\u00b0C and then drop to 225-230\u00a0\u00b0C.

\n" }, { "Id": "11178", "CreationDate": "2019-10-15T15:50:15.577", "Body": "

This is about practicality. I'm hearing that people are using their BLTouch not to adjust the Z offset, but as the limit switch for the machine! Why is this so? What are the pros and cons of using a BLTouch (or any touch sensor for that matter) in lieu of a physical limit switch?

\n

(NB: I'm looking for objective reasons, where no other solution will do as opposed to personal preference reasons where people just like to use it).

\n", "Title": "What are the pros and cons of using a BLTouch in place of a limit switch?", "Tags": "|z-axis|bltouch|homing|", "Answer": "

One other way a bed sensor may be preferable over a common limit switch: it automatically adjusts when you change out the build surface.

\n

Most glass build surfaces are anywhere from three to five millimeters thick, while the surface they replace (texture coated or uncoated fiberglass/resin, magnetic sheet, etc.) are around one to 1.5\u00a0mm thick -- that's a enough change in build surface height to be near the limits of the bed adjustment on my Ender 3, for instance.

\n

There are ways to work around this, for instance with spacers that clip onto the bottom of the X gantry to trigger the Z-stop at a higher position -- but if you forget to install the spacer on a build surface change, you may end up scratching the coating on your glass bed (probably a bad thing) by dragging the nozzle across it, as well as potentially damaging the nozzle (cheap, but takes time to replace once you figure out why your prints are suddenly failing). If you're using a bed sensor in place of a limit switch, this oversight can't happen, because the Z homing will automatically detect where to stop regardless what you've mounted on the bed.

\n" }, { "Id": "11182", "CreationDate": "2019-10-16T05:39:49.663", "Body": "

Since I have a 3D printer, I worked primarily with printing ABS. I tried multiple methods for adhesion (various soluble glues, ABS juice) but always had the most success with hairspray on clean borosilicate glass, as long as I print with a bed temperature above 75 \u00b0C and inside an enclosure.

\n\n

So I wonder, if it is known, why and based on which chemicals the hairspray method works so well?

\n", "Title": "Why does hairspray work as an adhesive for ABS?", "Tags": "|fdm|adhesion|", "Answer": "

First of all, not all hairsprays work. The chemicals in hairspray that causes the bonding is "VA/Crotonates Copolymer", it is also called "vinyl acetate/crotonic acid copolymer". Most commonly known as PVA, which is also a filament used as water soluble support material. This is a synthetic polymer created from monomers.

\n

When heated to a certain temperature this ingredient becomes sticky, this makes the filament stick to the plate. It is the double bonded Oxygen molecule that makes this happen. This is not only present in chemicals you put onto the bed, but also the chemistry of certain bed materials. Quoting our own Ryan Carlyle from this thread, A replacement for Aquanet Hair Spray?? Wolfbite by Airwolf:

\n
\n

Most of the popular build surfaces have very similar underlying chemistry. That's because they all rely on diffusion welding for adhesion. The molecular chains at the interface slightly dissolve into each other.

\n

Gluestick active ingredient - http://en.wikipedia.org/wiki/Polyvinylpyrrolidone

\n

\"enter

\n

Aquanet and liquid PVA glue active ingredient - http://en.wikipedia.org/wiki/Polyvinyl_acetate

\n

\"enter

\n

If you're familiar with organic chemistry, what you see here is an identical polyvinyl backbone chain, and functional groups that both have a ketone/ester double-bonded oxygen adjacent to a space-filling component. (The space-filling component probably decreases adhesion so the print can be released.) And guess what? Permanent build plate surfaces rely on the same underlying chemistry:

\n

Lexan also has exposed double-bonded oxygen on a polymer backbone: http://en.wikipedia.org/wiki/Polycarbonate

\n

\"enter

\n

Kapton also has exposed double-bonded oxygen on a polymer backbone: http://en.wikipedia.org/wiki/Kapton

\n

\"enter

\n

Acrylic also has exposed double-bonded oxygen on a polymer backbone: http://en.wikipedia.org/wiki/Poly(methyl_methacrylate)

\n

\"enter

\n

PET also has exposed double-bonded oxygen on a polymer backbone: http://en.wikipedia.org/wiki/Polyethylene_terephthalate

\n

\"enter

\n

Now, if you look at some common solvents for filaments, what do you often see? Double-bonded oxygen on small molecules. Here's ethyl acetate (dissolves PLA) and acetone (dissolves ABS):

\n

\"enter\"enter

\n

These molecules are still diffusing into the filament polymer, but they're so small and mobile that they are able to completely liquefy and dissolve the plastic.

\n

Isn't chemistry great?

\n
\n" }, { "Id": "11187", "CreationDate": "2019-10-17T16:28:29.920", "Body": "

We are attempting to print some single-use prototypes using an SLS printer. The parts must be somewhat water-resistant for short-term usage, but do not have to been waterproof entirely.

\n\n

According to this page, Post processing for SLS printed parts - Water tightness, any silicone type or vinyl-acrylate sealant should work well, but I was hoping that the Stack users here might have some specific suggestions.

\n\n

Does anybody have specific recommendations for sealing SLS prints?

\n", "Title": "Sealant to increase Water-Resistance of SLS print", "Tags": "|post-processing|sls|water-resistance|", "Answer": "

SLS prints from nylon are somewhat porous. This means, that they will let a stabilizer soak into the outer surface to some degree.

\n\n

What kind of stabilizer is needed is depending somewhat on the properties you want, but generally I believe these might be useable depending on the viscosity:

\n\n\n" }, { "Id": "11193", "CreationDate": "2019-10-18T09:17:53.320", "Body": "

I have a Creality Ender 3, I've had it for just under 2 years and during a print one morning it suddenly stopped and powered itself off. After checking the fuse and other electronic components it still didn't power on. I purchased a new switch as I thought the problem was there but after replacing it is still did not turn on.

\n\n

The printer has been working fine and there were no visible or audible anomalies. I urgently need this to be fixed.

\n\n

\"enter

\n\n

\"enter

\n\n

\"enter

\n", "Title": "Ender 3 Power Problem: Not starting up", "Tags": "|creality-ender-3|power-supply|repair|", "Answer": "

@trish Thankyou for your help.

\n\n

I did purchased the new power supply and everything works fine now.

\n" }, { "Id": "11197", "CreationDate": "2019-10-19T07:59:20.990", "Body": "

I'm using Cura to slice my prints. I've noticed that when printing the bottom layer (and also the top layer, if it's flat), it first prints three walls, then fills in the middle by moving back and forth in straight lines.

\n\n

I've noticed that for my parts, the walls look much nicer than the zig-zag pattern in the middle, and it also seems that the zig-zag part detaches from the bed quite easily, whereas the walls don't.

\n\n

My parts would look much better, and possibly also stick better to the bed, if I set the number of walls to 100 or so, so that the parts would be entirely filled in with walls. But then the parts would be completely solid, which isn't what I want. So what I want to achieve is that the bottom layer (and if possible also the top layer) are printed as if the part was composed entirely of walls, but the other layers are printed with three walls as normal. Is this possible in Cura?

\n", "Title": "In Cura, can I make my top and bottom layer be all perimiters?", "Tags": "|ultimaker-cura|", "Answer": "

The latest Cura that I have (4.2.1) includes \"Ironing\" in its options. When I enabled this, it prints the top layer twice. The first time with extrusion and the second time with just a little bit of extrusion (default is 10%) at 90 degrees to the first one. This effectively \"irons out\" the zig-zag pattern giving you a nice smooth top to your parts.\nI was very impressed at how well this works (YMMV of course :-) ).

\n" }, { "Id": "11199", "CreationDate": "2019-10-19T10:21:52.463", "Body": "

I have made some prints with the Ultimaker 2+ and Ultimaker 2 Extended+. The prints are in PLA. For slicing, I use Cura and I check the support checkbox (haven't gone to advanced settings to adjust support yet). I can clearly see that there is a little space between the support and the print. The supports often look like long pillars and such.

\n\n

My question then is: \"What is the best technique for removing the support?\". Is it to use a knife, pliers or perhaps PLA-water? Is it possible to use PLA-water to remove support when printed with Ulitmaker 2+ or is that just the Ultimaker 3? What type of technique would give a good looking print?

\n\n

Ultimaker 3 has support filament that's water-soluble. Is there something similar for Ultimaker 2+?

\n", "Title": "Technique for removing support from Ultimaker 2+?", "Tags": "|ultimaker-cura|pla|support-structures|ultimaker-2|", "Answer": "

The Ultimaker 2+ is a single extruder 3D printer. Without changing the PLA spool and PVA spool continuously during the print you practically cannot make water soluble supports on the Ultimaker 2+ which can be done on the Ultimaker 3. Note that PVA (from experience) is strange material to print, the filament is very hygroscopic and will form bubbles during printing when moist. Also, PVA is prone to clog the nozzle (it cooks easily) it therefore has its own printer core (nozzle assembly) on the Ultimaker 3 (still it clogs easily). Furthermore, it takes a while to dissolve in water.

\n\n

If you have a single extruder and nozzle, your best option is to use the same material for support, but modify the support settings as such that it can be easily removed. E.g. on thin layer heights (0.1 mm) I usually increase the gap between support and product over the default value, see this answer.

\n\n

On dual extruder printers, e.g. on the Ultimaker 3 and S5, my colleagues have better experience using Ultimaker Breakaway filament rather than using PVA. As with the PLA supports, you need to \"break them away\" from the actual product; I use a Leatherman Charge or FREE P4 as these tools have fine pointed pliers. Note that there are removal tools available that are best described as soldering irons that can aid in the removal of support structures:

\n\n

\"Modifi3d

\n\n

Note that I have mixed feelings about this product; it is hard to neatly remove supports using these small soldering irons, but it sometimes works.

\n" }, { "Id": "11201", "CreationDate": "2019-10-19T11:44:15.940", "Body": "

When printing a big base on the bed, I got no warping, but once I print small part (3 or less inches wide), there is a bit of warping on my piece as I remove it.

\n\n

\"Example

\n\n

It is 2 inches wide by 1 mm thick by the base. It seems to be well sticked to the bed while printing, but once removed, it bends a bit. I tried multiple times and it bends at a different place.

\n\n

I was wondering if it may be thicker if it won't bend at all.

\n\n

Can you guys help me with it?

\n\n

Here my Slic3r settings:

\n\n
# generated by PrusaSlicer 2.1.0+win64 on 2019-10-19 at 11:38:38 UTC\navoid_crossing_perimeters = 0\nbed_temperature = 60\nbottom_fill_pattern = rectilinear\nbottom_solid_layers = 7\nbrim_width = 5\ncooling = 1\nend_filament_gcode = \"; Filament-specific end gcode \\n;END gcode for filament\\n\"\nend_gcode = M104 S0 ; turn off temperature\\nG28 X0 Y0 ; home X and Y axis\\nG1 Z210 F5000 ; Send Z to top\\nM84     ; disable motors\\n\nensure_vertical_shell_thickness = 1\nexternal_perimeter_extrusion_width = 0.45\nexternal_perimeter_speed = 50%\nextrusion_multiplier = 1\nextrusion_width = 0.45\nfan_always_on = 1\nfilament_diameter = 1.75\nfilament_settings_id = \"FlyingBear Ghost 4\"\nfilament_type = PLA\nfill_angle = 45\nfill_density = 20%\nfill_pattern = stars\nfirst_layer_bed_temperature = 65\nfirst_layer_extrusion_width = 0.42\nfirst_layer_height = 0.2\nfirst_layer_speed = 30\nfirst_layer_temperature = 200\ninfill_extrusion_width = 0.45\ninfill_overlap = 25%\ninfill_speed = 80\nlayer_height = 0.1\nnozzle_diameter = 0.4\nperimeter_extruder = 1\nperimeter_extrusion_width = 0.45\nperimeter_speed = 45\nprint_settings_id = FlyingBear Ghost 4\nprinter_settings_id = FlyingBear Ghost 4\nprinter_technology = FFF\nretract_before_travel = 1\nretract_before_wipe = 0%\nretract_layer_change = 1\nretract_length = 0.8\nretract_length_toolchange = 10\nretract_lift = 0\nretract_lift_above = 0\nretract_lift_below = 209\nretract_restart_extra = 0\nretract_restart_extra_toolchange = 0\nretract_speed = 40\nseam_position = aligned\n
\n\n

As I said, I got no problem adhering to the bed, it's when I remove it from the bed it becomes deformed.

\n", "Title": "How to get rid of deforming small prints?", "Tags": "|slic3r|warping|", "Answer": "

Pla is flexible and can be shaped at temperatures as low as 50 deg C. To prevent deforming your prints in removing them, you should wait for the bed to reach room temperature. This takes about 5 minutes after a print ended.

\n" }, { "Id": "11202", "CreationDate": "2019-10-19T11:54:53.300", "Body": "

I've made low-poly objects using Reduce function in Meshmixer. My objects are symmetric and I want to keep the symmetry in the low-poly because it's more beautiful, But the software doesn't necessary keep the symmetry.\nusing mirror function results in losing flat faces and many problems...\nIs there any specific way to make a symmetric low-poly?

\n", "Title": "How to make symmetric Low-Poly objects in meshmixer?", "Tags": "|3d-models|stl|meshmixer|", "Answer": "

The best way would be to follow a four-step workflow:

\n\n
    \n
  1. Reduce complexity.
  2. \n
  3. Cut the object along the symmetry plane.
  4. \n
  5. Discard one half of the object.
  6. \n
  7. Mirror the cut object along the cut symmetry plane.
  8. \n
\n" }, { "Id": "11212", "CreationDate": "2019-10-21T03:18:12.477", "Body": "

I have an Epax X1, I have just purchased the 2nd and 3rd bottle of resin, but I don't know what are the best settings.

\n\n

What is the best way to find the best settings for a new resin?

\n\n

Is it possible to print a calibration object that starts with some settings and changes as it goes? For example starts with 12 seconds and decreases half second every 5 mm?

\n\n

EDIT

\n\n

The 2 resins I am dealing with right now are Nova3D and Elegoo. Any help with those two resins would help right now, but I would love to have a more generic answer that would allow me to explore and troubleshoot any resin without asking every time.

\n", "Title": "How do I find the best settings for a resin?", "Tags": "|resin|dlp|", "Answer": "

This test is not just a verification solid, is a program that tries different exposures and shows them all side by side: https://github.com/altLab/photon-resin-calibration

\n\n

The test doesn't move slower and slower, but it does something equivalent: keeps the plate in the same position while changing the bitmap.

\n\n

EDIT

\n\n

I tried the test, and I'm not happy about it. I tested the same black resin with the 0.02 and 0.05 layer thicknesses, and according to the tests I should use 8 seconds with the 0.02 and 6 seconds with the 0.05. This result goes against anything I heard about relation between layer thickness and exposure time.

\n\n

I also read in an issue in the repository that the test is not reliable with clear resins. My 2 tests are so thin that my black resin is transparent. I don't know if this is a factor, it just doesn't feel right.

\n\n

I will keep searching for a better test and update this post if I find one.

\n" }, { "Id": "11214", "CreationDate": "2019-10-21T12:48:55.383", "Body": "

This is GRBL-related. I have a laser-engraver type machine, Chinese, ~550\u00a0mm X axis, aluminium extrusions, 2mm-pitch T belt, Nema 17H3430 motors, one mechanical homing switch for X, Arduino-based.

\n\n

The X-axis accuracy is not up to par. When I make a mark on the X axis and jog to that mark (after homing), depending on the day, the position varies within ~4\u00a0mm or so. Sometimes I have to use G00 X-418, others G00 X-422.

\n\n

I checked the belt tension, it seems OK, not too tight, not too loose. Is there something else I should check?

\n\n

Another idea I have: put a homing switch on the other end of the X-axis, then home both ends of the X-axis, measuring the difference in machine position, then modify the steps/mm value for each session to force the command that puts me on the mark to be constant (e.g. G00 X-420). Any other ideas? Thanks.

\n", "Title": "GRBL: inaccurate on X axis by ~4 mm on 400 mm", "Tags": "|laser|arduino|linear-motion|", "Answer": "

This seems to be either a case of either belt slop or missed steps or it is a case of the accuracy of the limit switch.

\n\n

If the limit switch moves even a little into either direction, you have to account twice that as the maximum error. So the 4\u00a0mm error could come from 2\u00a0mm into either direction from the 0-position.

\n\n

However, there is a silver lining: Laser engraving, just like CNC, usually first dials in the 0 regarding the workpiece instead of the 0 of the machine. As long as the machine's movement is ok, you should be fine even without tightening the mounting of the X-endstop. I suggest to run the following test to find out what kind of error actually hounds you though:

\n\n
    \n
  1. Mount a waste piece
  2. \n
  3. Go to a position on the workpiece
  4. \n
  5. Run the laser for a split second
  6. \n
  7. X10
  8. \n
  9. Run the laser for a split second
  10. \n
  11. Repeat 4&5 till you have 10 points
  12. \n
  13. Measure the real distances between the dots engraved
  14. \n
\n\n

If the distances are always the same but short, you have the wrong steps/mm set and need to adjust them accordingly. If the distances are inconsistent, you have either lost steps or a sloppy belt. To fight lost steps, carefully up the signal to the motor a little. If the results don't change from that, tighten the belt a little - it should sing like a guitar string.

\n" }, { "Id": "11218", "CreationDate": "2019-10-21T16:41:55.947", "Body": "

I have an Ender 3 Pro with upgraded Bowden capricorn tube. The tube will move approximately 1-2 mm during normal operation, though it will not come out of the coupler at all. I have read that the movement if unchanging (constant movement of 1-2 mm) can be compensated for with settings adjustment to avoid retraction issues such that it will extrude or retract \"that much less\"... I do not understand how this is possible...

\n\n

Im confused by comments that a slipping Bowden tube on the extruder side only, say by up to 2 mm, is \"lost retraction\"... I have this problem too with the Bowden tube only at the extruder end, not the hotend, and wonder if it is really an issue at all. Here's why...

\n\n

The filament is in direct contact with the gear and wheel of the extruder. If the Bowden tube is only moving in/out of the extruder end of the coupler, there are no \"gaps\" being created to cause leakage of the filament, etc on the hot end...As the extruder is either pushing or retracting, the filament inside the tube is still moving as much as intended regardless of the amount of play of the Bowden tube... no? therefore, retracting will not be affected at all, nor would the extruding process. Am I wrong and if so, can someone explain to me how this would be?

\n\n

Note, I can see that this movement may cause under-extrusion on the feeding side process, as the machine is expecting say, 0.5 mm extrusion, but then has to compensate (unknowingly) for the slipping out tube during the \"push\", so not enough gets \"out\", but should not affect the retraction amount as the filament is still being pulled directly from the gears/roller.

\n\n

Am I wrong and if so, can someone explain to me how this would affect the retraction along with the under-extrusion?

\n", "Title": "How does slipping Bowden tube affect retraction or does it at all?", "Tags": "|extruder|creality-ender-3|bowden|retraction|", "Answer": "

This is happening to my E3 pro. When the Bowden tube slips back filament fills up the gap causing a heat creep issue. This of course causes clogged nozzle. If retracting I have had the swollen filament lodge in the tube to the point I had to remove tube and heat high enough with heat gun to soften filament for a reverse pull as there was nothing left on the hotend side to pull.

\n

My solution after a week of frustration was to replace the coupling.

\n" }, { "Id": "11228", "CreationDate": "2019-10-23T18:52:53.020", "Body": "

I'm working with an older MakerBot Replicator clone, actually a Flashforge Creator 1, with original Creator firmware.

\n\n

I don't understand where on the build plate the print starts. Is that controlled by the 3d printer's firmware values, or something else in the chain from .stl --> .gcode --> x3g ?? Right now my prints start in the corner nearest 0,0.0, instead of in the middle of the build plate.

\n

How do I control where to place the 3d print on the build plate?

\n

edit: Apologies for the delay. As a result of the answers posted here, I did a whole bunch more testing. The initial response from @mick, seems to indicate that what you see in the Slicer preview is what you'll see on the print bed. That makes sense, but that is definitely not what I'm seeing. I definitely don't see anywhere anything that remotely looks like a check the box [] center. Here's what I do see:

\n

When I drop the object into Slic3r, it goes to middle of build plate.

\n

\"Slicer

\n

\"Slic3r

\n

\"GPX

\n

Unfortunately here's what gets printed.

\n

Attempt #1, off in space

\n

\"print

\n

Attempt #2, Right Hand Margin

\n

\"print

\n

Attempt #3, Near the front.

\n

\"print

\n

I tried moving the print head to center of build plate, thinking that might be a logical start point. No go. Print start moves head to home (rear right corner of print bed) then after elements heat up to temp, it seems to select a random spot on the build plate to start. I never touched the .x3g file between these attempts. So I'm right back to where I started, per the title of this posting "What determines print start location on the build plate?"

\n", "Title": "What determines print start location on the build plate?", "Tags": "|slic3r|flashforge-creator|", "Answer": "

So it turns out there are elements from each of the previous answers that make sense here, but its not a clear picture. I've spent a whole lot of time trying to make sense of this. I also upgraded my firmware to Sailfish.

\n\n

For the MakerBot Replicator (or FastForge Creator) family of 3d printers, the origin of the build plate is NOT at any of the corners, its right in the center of the build plate. Reference here.

\n\n
\n

4.1 Home Offsets:
\n By convention, the center of the build platform is assumed to be the point (0,0,0) in XYZ space. The X, Y, and Z home offsets tell the printer the location of the X, Y, and Z endstops in relation to the build platform\u2019s center.

\n
\n\n

Looks like this:

\n\n

\"Center

\n\n

And it would appear that most other 3D printers are not using this convention. Sigh. From my testing, the main control of print location is within the G-Code generated by the slicer. The tool I was using, Slic3r does give one a chance to correct that\nadjustment. You have to go to top menu Settings --> Printer Settings --> Size and Coordinates (Bed Shape) --> Set to get a nice popup visual tool. See below.

\n\n

Unfortunately the default setting is accurate for the overall bed size, but is a fail for the origin location on the bed. The default origin is set at 0,0, in the corner. Ouch. Big ouch. The origin needs to be located right in the center of the build plate (to be consistent with the firmware controlling the print for this family of printers). It should look like this:

\n\n

\"print

\n\n

Note, when you tell the printer to go to 'home' it doesn't go to origin (0,0,0) it goes to the endstops.

\n\n
 G68 X0 Y0 F500 ; Perform Homing Routine\n
\n\n

So as long as you understand the quirks of these printers things will work out. You have to ensure offsets are set accurately in firmware config files. (Replicator / Sailfish) And you have to indicate the correct center location to the slicer program. I will say the advice given here was of some help in understanding this. Many thanks. I'm posting here in case others using the Replicator or FastForge Creator series of printers is having troubles centering their prints on the build plate.

\n\n

Additional info for anybody using a MakerBot Replicator or Flashforge Creator series printer with Slic3r. I wanted to add my custom G-code stuff. The default Slic3r stuff definitely did not work.

\n\n

Printer Settings --> Custom G-Code --> Start G-Code

\n\n
M103 ; Turn all extruders off, Extruder Retraction\nG21 ; set units to mm\nG90 ; Use absolute coordinates\n(**** begin homing ****)\nG162 X Y F2500 ; home XY axes to maximum stops\nG161 Z F1100 ; home Z axis to minimum stop\nG92 Z-5 ; Set Position Z =-5mm\nG1 Z0.0 ; move Z to \"0\"\nG161 Z F100 ; home Z axis to minimum stop slowly\nM132 X Y Z A B ; Recall stored home offsets for XYZAB axis\n               ; Loads the axis offset of the current home position from the EEPROM and waits for the buffer to empty.\nG90 ; Use absolute coordinates\nG1 X0 Y0 Z50 F3300.0 ; move to waiting position near center of build plate\n
\n\n

Printer Settings --> Custom G-Code --> End G-Code

\n\n
M109 S0 T0 ; Cool down the build platform\nM104 S0 T0 ; Cool down the Right Extruder\nM104 S0 T1 ; Cool down the Left Extruder\nM73 P100 ; End  build progress\nG0 Z150 ; Send Z axis to bottom of machine\nM18 ; Disable steppers\nG162 X Y F2500 ; Home XY endstops\nM18 ; Disable stepper motors\nM70 P30 ; We <3 Making Things! Yipee, you made it...\n; display message above for 30 seconds\nM72 P1 ; Play Ta-Da song\n
\n\n

One other thing I do with Slic3r. I print a single loop of print on the periphery of a phantom skirt. I do this as a printer head clean extrude exercise.

\n\n

Print Settings --> Skirt and Brim --> Skirt --> Loops (minimum): 1, Distance from object: 6mm, Skirt height: 1 This works well. I do this in lieu of the G-code startup used in ReplicatorG software (which went to the lower left hand corner, and did this odd 4mm extrude exercise, with odd timing...) The skirt thing works just fine.

\n" }, { "Id": "11240", "CreationDate": "2019-10-25T15:44:26.073", "Body": "

I have a custom designed and built CoreXY printer. I have noticed that when I order the X to move from one side to the other (right to left) that the Y axis moves back about 0.6 mm as well. When I move the X back (from left to right) the Y also moves forward approximately the same amount.

\n\n

What property of the CoreXY system could be causing this? Note, I am using 608 bearings instead of toothed idler pulleys; which I hope to rectify soon.

\n\n

Steps per mm from the Marlin configuration.h:

\n\n
DEFAULT_AXIS_STEPS_PER_UNIT   { 475.79, 482.87, 468.66, 188.1 }\n
\n\n

Note, they are not the same for X and Y. This is because I am trying to run a calibration cube test on the machine.

\n", "Title": "CoreXY Carriage moves on the Y when moving the X", "Tags": "|diy-3d-printer|corexy|", "Answer": "

For a CoreXY printer to move an axis (X or Y) it requires both stepper motors to turn. If both turn the same direction (at the same speed with the same pulleys), the X-axis will move, if they rotate both in a different direction (at the same speed with the same pulleys) the Y axis will move (see image of CoreXY kinematics below). Rotation of a single stepper would cause a 45° printing pattern.

\n\n

\"CoreXY

\n\n

All four corner points (two top pulleys and two bottom steppers) are fixed to the CoreXY frame

\n\n

It is therefore highly illogical that you need to use different values for your steps per mm (X = 475.79 steps/mm, Y = 482.87 steps/mm). If you need to use such values, this implies that your mechanical layout/mechanics is/are incorrect (skew frame, different pulley diameters, slip on pulley, incorrect tension of belts, etc.). It is advised to make them equal and check the mechanical layout and inspect all parts (pulley diameters) and make the tension equal (e.g. using a Belt Tension Gauge). If the head still moves in the perpendicular direction as commanded, you could try to calibrate from there.

\n" }, { "Id": "11261", "CreationDate": "2019-10-29T20:55:07.047", "Body": "

I have built a 3D printer with Marlin bugfix-2.0.x forked from github.

\n\n

I am attempting to set the default value for junction deviation so that I don't have to change it through the printer's screen/interface every time I power cycle the printer, but what I thought would set it properly isn't doing the trick.

\n\n

My thought was that uncommenting this in Configuration_adv.h would work:

\n\n
#define JUNCTION_DEVIATION_MM 0.02\n
\n\n

I have tried setting this define inside of and outside of the default if block in which it appears, just to make sure the issue wasn't because I didn't have something else set, and I get the same behavior when this define statement is either protected or unprotected by an if

\n\n

Also, the default if block is this:

\n\n
#if ENABLED( JUNCTION_DEVIATION )\n    #define JUNCTION_DEVIATION_MM 0.02\n#endif\n
\n\n

Uncommenting #define JUNCTION_DEVIATION leaves me with a compiler error telling me that I don't need to do that, so I'm not seeing anything that I am supposed to enable to get it to use the JUNCTION_DEVIATION_MM setting. In other words, I can't actually enable \"JUNCTION_DEVIATION\" any more, so I HAVE to break the JUNCTION_DEVIATION_MM setting out of the if but it still has no effect.

\n\n

After setting this parameter, when I power cycle the printer, the junction deviation is set to 0 (although one touch of the adjustment knob makes it jump to 0.010, so I think that what is displayed as \"0\" may actually mean \"unset\"), which is causing a significant stop/start jerk on every angle change until I manually set the junction deviation.

\n\n

My current Configuration.h and Configuration_adv.h are taken from this Marlin fork.

\n\n

In case it matters, I'm running this on an SKR V1.3 with TMC2208 drivers at 24V.

\n\n

Any ideas what I'm missing? I'm sure it's something simple, I just haven't found it.

\n\n

UPDATE: Still no solution after 10 days. Here is additional information from another forum where I posted this request:

\n\n
\n

Did you run M502 and M500 after updating your firmware?

\n
\n\n

I dug into this a little more, and I am either running into a bug, or I'm simply not understanding what I'm supposed to be seeing.

\n\n

I send this command:

\n\n

M205 J0.020

\n\n

Then verify that junction deviation is set correctly (it is)\nThen this:

\n\n

M500

\n\n

And that responds with:

\n\n

Settings Stored (616 bytes; crc 51371)

\n\n

Then I send this:

\n\n

M503

\n\n

And the report for M205 is missing the setting I just confirmed to have changed and been working:

\n\n

...\nM205 B20000.00 S0.00 T0.00\n...

\n\n

And power cycling or resetting the controller leads to the same result: Junction Deviation is reset to 0.

\n\n

Shouldn't I see \"J0.020\" in the M205 line immediately after changing the setting, and verifying that it works?

\n", "Title": "Setting Junction Deviation in firmware has no effect", "Tags": "|marlin|diy-3d-printer|skr-v1.3|", "Answer": "

For the SKR, to store the new Marlin definitions it is necessary to send the comand M502 to restore to the factory settings (will use the configuration on the SD card) and send the comand M500 to save the configurations.

\n" }, { "Id": "11263", "CreationDate": "2019-10-30T00:10:07.520", "Body": "

I've been running this Ender 3 Pro for quite some time with no problem up until recently. The issue was the printer refused to extrude filament (the extruder stepper seems to be rocking back and forth) so I went through a few things to try and fix the problem:

\n\n

I saw somewhere that maybe the wiring to the extruder gear is frayed but I inspected the extruder side of the connector and everything looks fine. I would open up the box and everything but I want to see if anyone has a solution before opening up the circuit box.

\n", "Title": "Ender 3 Pro extruder drive rocking back and forth", "Tags": "|creality-ender-3|extrusion|wiring|extruder-driver|", "Answer": "

Sounds like one end or the other in your stepper cable is not seated correctly or has shaken loose there are 4 wires if even one doesn't make proper contact you will get a shaking back and forth almost like a hard vibration. I always take a cable from one of the other steppers plug into this one move the motor with good stepper wire 10-20 mm see what motor does if it is okay then you know it is a cable issue if not, it could be a bad board or stepper.

\n" }, { "Id": "11265", "CreationDate": "2019-10-30T14:44:22.523", "Body": "

I have a Printrboard rev. D which includes Allegro A4982 drivers. I would like to replace them, but it appears that newer TMC drivers all require several pins for proper operation.

\n\n

\"enter

\n\n

How can I upgrade the drivers in the Printrboard rev. D?

\n", "Title": "How can I upgrade the drivers in the Printrboard rev. D?", "Tags": "|stepper-driver|", "Answer": "

The developers from the Klipper firmware confirmed that Klipper doesn't care about the native functions of each pin, therefore it is indeed possible to use the expansion pins for controlling (including UART) TMC stepper drivers.

\n\n

At this point I cannot confirm, but maybe Marlin allows the same by updating the pin assignments in the source code.

\n" }, { "Id": "11272", "CreationDate": "2019-11-01T02:43:45.477", "Body": "

I know this will be a really obvious question to some people, but I have bricked about 3 boards doing this so I want to be certain before I brick a fourth. I don't have a lot of experience working with AC voltage, especially crimping / hacking it like what's going on here. Trust me it was my last resort to ask this.

\n\n

Basically, as a chamber heater I'm using some 120V heat lamps (here in the U.S.). The problem seems to be that when I connect my laptop to the board (if the laptop is plugged in, which it has to be for long-term serial control), sometimes I:

\n\n
    \n
  1. Hear an electric shock sound -- the board is fried
  2. \n
  3. Don't hear an electric shock sound -- the board is fried, though
  4. \n
\n\n

What I interpret from this is that I'm stupidly wiring the 120V heat lamps incorrectly relative to my 3D printer board, such that when I connect it to my charging laptop via USB, the AC voltages are out of phase, causing my board to be fried. Alternatively, it's the 12V/24V power supply that's wired wrong relative to everything else, which also needs to be wired to the same polarity.\n\"enter

\n\n

I was on the understanding that when crimping a three-wire male electrical cord:

\n\n
    \n
  1. White = Neutral
  2. \n
  3. Black = Hot (or Load)
  4. \n
  5. Green = Ground
  6. \n
\n\n

... reinforced on the left side of the below image as well:

\n\n

\"enter

\n\n

So, I know that much already. The question is, when I connect this AC outlet to a relay like this, I got NO, COM, NC on one side and IN, DC+, DC- on the other. Which wire goes to COM, and which goes to D-? The board uses COM to denote the voltage that is switched 'on' by the relay, and D- to denote whatever the shared ground is among, the 3D printer, heat lamp, printer board, etc... (For more info, see link to relay)\n\"enter

\n\n

This 5V relay says it supports AC loads being managed by an arduino / 3D printer, so I was under the impression that I could connect the white wire to its D- pin (which also should receive the \"GND\" for my 3D printer board) and the black wire to its COM pin. However, what do I do about the green wire? Nothing? Also, was it wrong for me to assume it would support this function when the Arduino is connected to a charging laptop via USB?

\n\n

Basically, I just need to be sure that my laptop (3-prong), power supply (3-prong), heat lamps (2 or 3 prong) and relay (only 2-prong) all have matching polarities and don't cause an electric shock or fry my board. But I am getting really conflicting results and frying a lot of boards, probably by overthinking this.

\n", "Title": "How to wire for AC mains voltage relay, when printer board is connected to AC-charging laptop computer?", "Tags": "|wiring|chamber|", "Answer": "

Mains electric specialist here. When working in mains power, mechanical execution of work is everything.

\n\n

The first rule is do not casually mess around with mains power -- it will kill you! If you've done a lot of low-voltage tinkering with jury-rigged wires splayed all over the place -- do not do that with mains wiring. Ever. Break that habit with mains!

\n\n

The second rule is hard separation. It is absolutely forbidden, for instance, to have low voltage wires intermingled with mains in a jumble, even inside a metal box.

\n\n

A \"Chinese wall\" between mains and low-voltage

\n\n

By which I don't mean low quality. Generally all wires associated with mains should be inside a grounded metal enclosure, and all low-voltage wires should be outside that enclosure, or separated by a physical divider.

\n\n

Here is an ideal setup that will pass muster.

\n\n\n\n

\"enter\n\"enter\n\"enter

\n\n

For now just imagine 3 mains wires coming in: black hot, white neutral and green earth. Earth goes to that #10-32 screw hole on the bump, and the metal case grounds everything else.

\n\n

I pigtail wires on receptacles because it's easier, but it's also clearer for a novice (than using a receptacle as a splice block). On the receptacles, pigtail neutral with white. Hot gets a red pigtail if switched, and black if always-on.

\n\n

Fit the relay into a knockout hole, mains wires inward.

\n\n

Now you have a bunch of black, red, and white wires. Join them by color! Easy peasy! This is why I made such a fuss about getting 3 colors of THHN wire.

\n\n

Button it all up, and the mains work is done.

\n\n

Getting power into our box

\n\n

I glossed over bringing mains supply into this box. Let's review that further.

\n\n

That junction box is made to mount on a wall. It's a code violation to use it as a portable box (but lots of people do). However you must use cordage (SJOW, SJOOW etc.) not in-wall wiring (NM, THHN, UF). I just buy a 14 AWG extension cord and lop the socket off. Visit the electrical supply house (Greybar, not Mouser) and get a strain relief that'll fit that cord and a knockout hole.

\n\n

If you want to mount it on the wall over another box, they make special 4\" square boxes for that. If the existing box is a similar box, they make \"extension boxes\" with open backs. They also make ones designed to go over top of a 1-gang box that is metal or plastic.

\n\n

\"enter

\n\n

If you need the existing outlet space on the wall, you can use surface conduit systems like Legrand Wiremold (not to be confused with cable organizers intended for low-voltage cables only). You extend off an existing outlet, leaving that with its original function, then run surface conduit to another box, where you mount these items.

\n\n

Make the system talk to it

\n\n

That was a lot of heavy lifting, but the rest is downhill.

\n\n

On the above relay, you simply attach 2 wires to W and R. (in fact, common thermostat cable is \"W\"hite and \"R\"ed, coincidence: not). If you short those wires to each other, the relay operates and the lights turn on. Use an appropriate relay if needed. There is low voltage AC on these wires (24VAC between them, 34V peak, isolated from earth and mains) which means you can route these wires without mains-level protection.

\n\n

They make other relay-transformers, but I really like how this one puts the low-voltage on opposite sides of the knockout from the mains. They also make cheaper relays that accept 24VDC or even 12VDC, but then you have to supply that.

\n" }, { "Id": "11285", "CreationDate": "2019-11-03T15:41:37.627", "Body": "

I have a basic Creality Ender-3 with a black rough bed cover. I'm printing with PLA.

\n\n

After assembly, I printed 3 or 4 small toys sliced by Cura with basic settings. All were printed very well!

\n\n

However, after that, extruded filament would not stick to the bed. I tried the following:

\n\n\n\n

Nothing helped so far.

\n\n

What else can I try?

\n", "Title": "Why doesn't PLA stick to heated bed?", "Tags": "|pla|heated-bed|creality-ender-3|adhesion|", "Answer": "

Soap and water are absolutely the wrong stuff for cleaning the print bed. It's a base, and bases are slippery. This helps things like dirt slip off when we clean, but will also make it harder for filament to adhere to the bed. Instead, use rubbing alcohol, which is slightly acidic.

\n

Additionally, not all hairsprays are good for this. There is a certain ingredient you need, and some don't have it. Others do have it, but also have other things that interfere. AquaNet is known to be good for this. You can also buy made-for-purpose spray for 3d printing beds.

\n" }, { "Id": "11290", "CreationDate": "2019-11-04T14:46:33.100", "Body": "

So while doing some research I stumbled upon a wiki page on reprap from a few years back where the user was creating a glass nozzle to replace the brass and PTFE assembly.1

\n\n

Does anyone know the theory behind this? Glass is a great insulator so I could see how that would be beneficial for the heat break part but I can't see how it is appropriate for the nozzle as this is normally brass which is a good conductor.

\n\n

Surely the glass takes much more energy to heat up?

\n\n

On a side note I've seen similar projects using ceramic instead.

\n", "Title": "What is the purpose behind a glass nozzle?", "Tags": "|nozzle|", "Answer": "

One shortcoming would be that when it comes back to lay down a new line next to an existing line, I would think that it would need to be able to melt the previously printed plastic, especially any bumps and strings.
\nHigh thermal conductivity for good heat flow seems important.

\n" }, { "Id": "11316", "CreationDate": "2019-11-08T04:47:34.770", "Body": "

During long prints, my heat block becomes covered in whatever material I am printing. The plastic leaks from the junction between the heat break and the heat block, and runs down towards the nozzle.

\n\n

I frequently have problems with grinding of filament by the Bowden extruder. However, on the E3D Kraken with a Volcano block, if I use water cooling and a fan positioned just above the heat block, there is no grinding, and everything comes out fine. But the heat block grows a beard of plastic regardless if there is a fan over the heat break or not.

\n\n

I check my prints once every 2 hours and can wipe the heat block beard, but for overnight prints this isn't possible, so sometimes my prints will have random blobs from when the plastic leaks onto the print.

\n\n

I have tried multiple nozzles and filaments! I print at only 60 mm/s. Does the volcano block need faster print speeds to prevent this from happening?

\n\n

I cant really figure out the root of the issue. If the rest of the print comes out fine, then why is there an issue?

\n", "Title": "What causes leaking between the heat break and the heat block?", "Tags": "|hotend|e3d-kraken|", "Answer": "

A video I recently viewed (YouTube link) regarding installing a hot end assembly references the proper sequence of parts placement. Starting with the nozzle, thread it into the heater block until it is proud of the surface by a half of a millimeter, perhaps slightly more. The heat break is then threaded into the heater block until it contacts the nozzle. If not already in place, attach the heat sink.

\n\n

The next step is to bring the assembly up to maximum temperature and re-tighten the nozzle assembly, obviously using great care due to the high temp.

\n\n

The video specified that performing this sequence incorrectly would result in leakage from the assembly, which describes your troubles.

\n" }, { "Id": "11321", "CreationDate": "2019-11-08T17:24:25.857", "Body": "

I am trying to increase the flow rate on my Ender 3 from 100 to 108 % but every time I start a new print it reverts back to the old 100 % flow rate. I have clicked through the settings and pressed the \"store settings\" button but it still reverts back at the start of every print. I don't want to do this in my slicer settings as I run 18 Ender 3's so I want to be able to use the same G-code for each.

\n", "Title": "How do I set the flow rate on my Ender 3 Pro and have it stay at that percentage?", "Tags": "|creality-ender-3|firmware|", "Answer": "

You should be able to add a global override to the flow percentage on Marlin firmware printers.

\n

Add this line somewhere in your start code:

\n

M221 S97 ; Flow Percentage hard set.

\n

In Cura, edit the printer's machine settings. The S is the percentage. In my case, 97\u00a0% works for PLA+.

\n

Here is a link you might find useful.\nMarlin Docs

\n" }, { "Id": "11326", "CreationDate": "2019-11-09T16:46:13.840", "Body": "

I've seen articles about the Hermes (now rebranded to Hemera) hotend coming soon from E3D, and was wondering if anyone knows if it's possible to replace the stock Aerostruder on Lulzbot Mini 2 with it. The Aero has been giving me some issues and the printer is currently unusable... I've been printing for a few years but am still pretty new to mods so I don't really know what I'm doing there.

\n", "Title": "Can I replace hotend on Lulzbot Mini 2 with upcoming E3D Hermes?", "Tags": "|hotend|lulzbot|e3d-hemera|", "Answer": "

Yes, E3D has a guide "LulzBot Taz6 Hemera Upgrade" if you'd like to make your own.

\n

The upgrade requires you to print parts beforehand which are found here; note that:

\n
\n

We recommend printing the Hemera Mount in PETG, and the Fan Duct in ABS/ ASA or another high-temperature material.

\n

Use an infill percentage of 25\u00a0% or higher.

\n
\n

The Hemera (the new name for the Hermes) toolheads are also available for purchase, e.g. here.

\n" }, { "Id": "11328", "CreationDate": "2019-11-10T02:18:01.753", "Body": "

I have an STL made out of triangular faces that intersect themselves to create very complicated patterns. It also has a very messy internal structure. I'd like to remove the parts of the faces that are invisible from outside, whilst still maintaining the precise geometry of the model.

\n\n

Any free software I can use is fine by me.

\n\n

I tried using the Hollow Tool in Meshmixer, but that just deleted half of everything, for some reason. I also tried doing a Uniform Mesh Resampling in Meshlab, but that just created a wrinkly, disconnected mess. Disabling infill in Cura seems to do nothing.

\n\n

For reference, the model looks like this.

\n\n

\"Small

\n", "Title": "Delete only invisible parts of faces", "Tags": "|3d-models|stl|", "Answer": "

As long as the STL creates a closed, manifold, watertight body, a good slicer will just slice it. Ultimaker Cura has the option to Union intersecting shells, which lets it take non-manifold parts of a shell and union them to the body in such a way that it creates a closed body. More simplification and fixing the problem in a slicer usually is not needed. If there is still some area where internal geometry is created, it can help to create an overlapping internal structure deliberately, as Easy way to refine a 3D-model for 3D printing by removing internal geometry explains.

\n\n

Let's look at some example: The STL game export of a pauldron is made from the base body (one shell) and the rivets (which are another shell inside the STL). If Union Intersecting Shells is active, the rivets are sliced and printed as part of the pauldron. If Union Intersecting Shells is not active, the rivets are found to be non-sliceable and ignored.

\n\n

Infill is meant to support the upper structures in print.

\n\n

If it is mandatory to join the item into one shell, the STL format isn't the best to go through.

\n\n

If the pattern is produced by a program, that is able to generate and export STEP files, these could be used by CAD software such as Fusion360 which then would interpret the generated body in such a way that it can be unioned easily, getting rid of internal geometry.

\n\n

If the program generating the pattern is working with a vertex cloud such as blender, it should be possible to cut the long outer lines into several pieces and placing the vertices on the intersection of lines. These vertices could then be merged and any internal edges (and faces) removed. This can be a tedious process.

\n" }, { "Id": "11332", "CreationDate": "2019-11-10T16:01:22.870", "Body": "

I'd like to have the 12V PID output of an old Printrboard hotend control a relay that attaches to a 120V crockpot for something I'm making. I have everything working fine using a relay, but because the hotend heater is using PID control, the relay clicks about 3 times per second each time it needs to heat up the crockpot.

\n\n

Not only is this sound annoying, but it will likely reduce the life of the relay significantly. I need the crockpot temperature controlled, so the programmatic thermistor control on an old 3D printer board is a huge help, and probably cheaper than the alternatives. Especially since this board has an SD slot built in and can run any code on power-up. In \"bang-bang\" control, there would be less clicking as the crockpot heats up, and the relay wouldn't be damaged.

\n\n

I can't update the firmware on my board to use a different pin because it is a really old Printrbot board. All the required software is almost 5 years old and is really difficult to find. Fortunately the Printrboard does support M301. So, I should be able to use this command to switch from PID control to bang-bang.

\n\n

In Marlin's documentation, it describes a command called M301, along with a lot of variables for using this command. The problem is, I don't know what any of these variables mean.

\n\n
M301 [C<value>] [D<value>] [E<index>] [I<value>] [L<value>] [P<value>] \n
\n\n
\n[C<value>]  \n\nC term (requires PID_EXTRUSION_SCALING)\n\n[D<value>]  \n\nDerivative value\n\n[E<index>]  \n\nExtruder index to set. Default 0.\n\n[I<value>]  \n\nIntegral value\n\n[L<value>]  \n\nExtrusion scaling queue length (requires PID_EXTRUSION_SCALING)\n\n[P<value>]  \n\nProportional value\n
\n\n

About the only thing I know for sure is that the extruder index is 0.

\n\n

What command would I enter to make this a bang-bang PID controller, so that I don't damage my relay long-term?

\n", "Title": "What are the variables for PID control? How to use M301? How to use this command to switch from PID to bang-bang?", "Tags": "|marlin|heat-management|", "Answer": "
\n

What command would I enter to make this a bang-bang PID controller

\n
\n\n

There's no such thing as a \"bang-bang PID controller\". \"bang bang\" is mutually exclusive of PID. The M301 command is only good for fine-tuning the parameters of the PID controller, but it won't let you switch to bang bang. Unfortunately, you must update the firmware if you wish to use bang bang.

\n\n

You could try setting all values to 0, except P, which you set to an as high value as possible. This will cause the output to be fully on when the temperature is lower than the setpoint, and fully off when higher than the setpoint. However, this can still cause rapid clicking of the relay when the temperature is hovering around the setpoint. Bang bang has some hysteresis built-in (i.e., there is a margin around the setpoint in which the relay will never be toggled); there's no way to emulate this with PID.

\n" }, { "Id": "11341", "CreationDate": "2019-11-11T07:14:42.987", "Body": "

So I just got the Ender 3 and have been trying to print out a part. However, I noticed the printer is trying to print the part, which should be centered in the bed, on the left side.

\n\n

I performed the auto home, leveling procedures, prior to execution, but it still wants to print it on the left side.

\n\n

I checked the values it thinks it has for X, but they are about 60 mm to high (i.e. from the control/move axis menu, I move it towards the zero position and when it touches the switch, it still thinks its around 65-70 mm in the positive direction).

\n\n

I have tried the good old power cycle, but each time it seems to get worse printing closer and closer to the edge. I am sure that I must have fudged something up. Anyone know how to fix the mismatched position?

\n\n

Thanks for any advice!

\n", "Title": "Ender 3 X-axis values are not correct", "Tags": "|creality-ender-3|troubleshooting|x-axis|", "Answer": "

So the issues with the digital display values, was caused by the X-axis binding up and not advancing. I had to move the axis via the control panel in the positive direction and noticed once it got about half-way out, it wouldn't advance for like two or more steps.

\n\n

On the control panel, it said I was 235 mm out from the home position, but in reality I was more around the midway point. When I would move it back to the home position the read-out still displayed that I was offset from the home position in the positive direction.

\n\n

To fix it I tore it down and reconstructed it (made sure everything was plumb and level), I also flipped the belt in the opposite direction (teeth down, so that the stepper pulley grips the belt teeth).

\n" }, { "Id": "11349", "CreationDate": "2019-11-11T23:07:36.620", "Body": "

What is the correct orientation for the X-axis belt?

\n\n

I first thought to place the smooth side down to match the tensioner bearing, but then changed it to match the motor pulley.

\n", "Title": "Ender 3 X-axis belt orientation", "Tags": "|creality-ender-3|x-axis|", "Answer": "

Sadly, the Ender-3 assembly instructions were not clear on step 7, what the right direction is (teeth on inner side of the loop):

\n\n

\"enter

\n\n

The Belt has teeth that need to engage the teeth of the hobbed gear on the motor, just like in the preassembled Y-axis. The teeth have to go aim to the \"inside\" of the loop. This way, the belt will not slip on the motor side and the movements will be repeatable.

\n" }, { "Id": "11351", "CreationDate": "2019-11-12T11:31:53.373", "Body": "

I know that M73 P19 means \"Set completion progress to 19%\", and I suspect that M73 R42 means \"Set remaining time to 42 minutes\", but what is M73 Q17 S43? I can't find description of such syntax.

\n\n

The command is seen in .gcode files produced by PrusaSlicer.

\n", "Title": "What is M73 Q17 S43 G-code command?", "Tags": "|marlin|g-code|slic3r|prusa-research|", "Answer": "

The M73 Set/Get build percentage G-code is only defined for a selected few printer firmwares.

\n\n

As you suspected, next to M73 P19 (tell the firmware at what completage percentage the print is) the M73 R42 tells the firmware the left time to completion.

\n\n

If you look at the description of the M73 G-code, the following parameters may be used:

\n\n\n\n

So, running the Q and S parameters, is similar to the P and R parameters with the exception for referring to the printer percentage/time when in stealth (quiet) mode.

\n\n

These modes, normal and stealth, refer to power modes of Prusa printers:

\n\n
\n

Normal vs. Stealth mode
\n MK3 printers offer two print modes. Normal mode is required for the detection of lost steps (shifted layers), while still being quieter than the silent mode on MK2/S. There is also the Stealth mode, which utilizes Trinamic StealthChop technology, making the printer almost inaudible with the print cooling fan being the noisiest part of the printer. However, Stealth mode does not provide lost step detection.

\n
\n\n

Stealth mode times can be a bit higher than normal mode estimation times. Estimation is done by the slicer. As Prusa maintains this feature, their times are accurate for their printers, but that does not have to be the case for custom printers.

\n\n

To get the current progress, the M73 command is called without parameters:

\n\n
\n

Use \"M73\" by itself to get a report of the current print progress.

\n
\n" }, { "Id": "11360", "CreationDate": "2019-11-13T19:43:55.203", "Body": "

The extruder on my Prusa i3 MK3S keeps clogging while printing. Loading is fine, and each time it clogs, unloading and loading the filament seems to work.\nThe filament extruded just after resolving a clog looks like it has small bubbled on it.

\n\n

The problem only seems to happen when printing certain models. Printing the Batman symbol at 0.15 mm works every time. But the model I'm trying to print always fails at most a few minutes in.

\n\n

I've tried cleaning the nozzle. Cold pull twice, and taken it out and heated it with my airgun and trying to work on it with the acupuncture needle.

\n\n

The print that fails is this \"Small Pill Container\" with 0.1 mm layer height and 100 % infill, but printing a simple cylinder shape (25 mm outer diameter, 2.5 mm walls) creates the same problem. Trying to print them both at the same slicer settings with 0.15 mm layer height still fails.

\n\n

The model was sliced using the newest version of PrusaSlicer 2.1.0 and printed with Prusament PLA at the factory recommended 215\u00b110 °C, and tried 220 °C.

\n\n

Any ideas what could be happening? Anybody experienced a similar issue?

\n\n

Printing the bundled TreeFrog 50 μm sliced by Prusa for the Prusa I3 MK3S cased the same problem. Jam happened about 30 min in. I guess this rules out slicer problems as the root cause.

\n\n
\n\n

After a lot of research, it seems a lot of people have problems with the MK3 clogging, especially with PLA, and especially with prints with a lot of retractions. The issue seems to be the design of the heat-break, which has a throat between 2.2 mm and 2 mm, where the PLA can get stuck. I've ordered a new heatbreak. Please read this answer of my findings.

\n", "Title": "Prusa i3 MK3S keeps clogging during print. Ugly streaks sometimes, and complete clog when printing with lower layer lines", "Tags": "|prusa-i3|extrusion|nozzle|", "Answer": "

My problem was 2 things. The heatbreak, which was switched out for the MK2 version(Explantation below). And the Teflon Tube that runs down the heatsink.

\n\n

Heatbreak

\n\n

Change the heat-break to a generic E3D one. You can order the heatbreak for the MK2 from prusa, or any generic heat break for the E3D hot-end assembly.

\n\n

On the Prusa i3 MK3(s), this component has been given a 45\u00b0 taper in the middle, between 2.2 and 2 mm. This is done to ease filament retraction for the MMU, and will be nothing but problematic if you are not using the multi-material upgrade. Especially with higher nozzle pressures(eq. with lower layer lines), the filament may be squeezed into this taper, clogging the hot-end.

\n\n

You may not experience full clogs, but partial ones that will show themselves as streaks in certain layers on the print.

\n\n

Heatsink Teflon Tube

\n\n

There is a teflon tube that runs down the heatsink. It's crucial that this is mounted correctly and it is not entirely intuative how.

\n\n

First, press the teflon tube into the heatsink all the way to the bottom. Then, try to pull it out slightly. You'll notice that the small plastic ring at the top of the heatsink will pull out slightly along with it. Now, hold this black plastic part at it's current position with your fingernail, and push the teflon tube in the extra amount. When done, there should be no play in the tube.

\n" }, { "Id": "11361", "CreationDate": "2019-11-13T21:19:10.280", "Body": "

In the E3D Kraken cooler block, there is a big, 10 mm grub screw, along the side of the cooler block.

\n\n

I watched the entire Kraken assembly video:

\n\n

\r\n \r\n

\n\n

There was no mention of this very thick grub screw. The E3D Kraken assembly wiki page may refer to this part as the \"stainless plug\".

\n\n

Does the depth of the screw inside the Kraken heatbreak affect the effectiveness of the water cooling?

\n\n

Why was it included in the design at all?

\n\n

I'm asking because water frequently leaks out of this pore for me, ever since I had to repair some damaged tubing. Additionally I often have to use an extra fan when printing at high temperatures. I'm wondering, before I epoxy this grub screw into place, whether the amount it is tightened into the Kraken has some advantages or disadvantages.

\n\n

The video shows the part already assembled on the Kraken. This is what the part looks like - it is much larger than the screws used to secure heat throats. \"enter

\n", "Title": "What does the stainless steel plug do in the E3D Kraken cooler block?", "Tags": "|e3d-kraken|", "Answer": "

This stainless steel plug is necessary because of the fabrication of the cooler block. As this is a water cooled block, cold fluid flows in the inlet brass barb, and hot fluid out of the exit brass barb. The manufactured channel between the two is done through the hole that is later plugged by the stainless steel plug.

\n\n

From the manufacturers website of the stainless steel plugs you can read that a tapered hole and sealant are required to make a tight seal:

\n\n
\n

They have a wide range of applications in sealing and require a tapered hole.

\n
\n\n

and:

\n\n
\n

Replacing these plugs on the Kraken should only be completed if you are confident in your ability to achieve an adequate seal. We use Threadlock and a measured torque setting.

\n
\n\n

Be sure the hole has a taper, if you over-tightened if once it may have worn. Epoxy based glue would be able to be used at the cooler block, temperatures are not expected to be too high, but from the comments below it becomes clear that there are specific sealing products available for sealing the plug other than epoxy.

\n\n

The depth of the grub screw is not likely to affect the cooling properties (it isn't expected to block one of the brass barbs).

\n" }, { "Id": "11369", "CreationDate": "2019-11-14T17:57:47.840", "Body": "

For a model that has raised text on it, how do go about painting the very top layer of text, to make it stand out from the background? Lets say I printed the whole model the same color, what type of paint would work well, and not get on the background too?

\n\n

Here's an example of a model that has raised text: https://www.thingiverse.com/thing:3025280

\n", "Title": "Painting Text on Model", "Tags": "|post-processing|", "Answer": "

I recommend to print stencil for the text. Basically flat thin plate with text holes in it. Just cover your model with it and only text will be painted through the hole. In this way you could do it rough, fast or like Mr Bean did.

\n" }, { "Id": "11376", "CreationDate": "2019-11-15T18:34:29.183", "Body": "

I'm looking for a Printrboard board image with the pin numbers labeled. Specifically for Rev. F.

\n\n

Specifically, I'd like to know what each pin on the EXP1 and EXP2 correspond to numberwise within Marlin.

\n\n

The pins.h file for the Printrboard Rev. F doesn't include any other files and doesn't indicate what pin number corresponds to which part on the board...

\n\n

There is this google sheets file which provides some of the information.

\n\n

\"enter

\n\n

This image has the categories of each pin labeled. But I'm looking for where the pin numbers on the board correspond to the numbers in Marlin, similar to this image for the MKS Gen L:

\n\n

\"enter

\n", "Title": "How to find Printrboard pin numbers in Marlin", "Tags": "|marlin|", "Answer": "

As FarO suggested, from github:Printrbot/printrboard/revF2 (via RepRapWiki - Printrboard - Revision History), the EagleCAD board layout and schematics are contained in the printrboard-revF2.zip:

\n\n\n\n

This is the EagleCAD schematic:

\n\n

\"Printrboard-revF2

\n\n

EagleCAD board layout

\n\n

\"Printrboard-revF2

\n\n

Not quite the annotated diagram that you were after, unfortunately, but it might help.

\n\n
\n\n

There are a couple of configuration files on KevinOConnor/klipper, but unfortunately only for revisions B to D (at90usb1286) and G (SAM3x8c)... E and F are not available.

\n" }, { "Id": "11381", "CreationDate": "2019-11-18T02:13:25.380", "Body": "

I've been printing with my Ender 3 for a while now and it's been great. I've had very few problems - depending on my settings, these are my typical first layers:\"enter

\n\n

\"enter

\n\n

However with no settings or temperature changes and attempting to print the same files, I am now getting this issue with every print - The lines lay down and adhere fine, but if I watch carefully it looks as if the nozzle is causing the previous line to lift and warp. I have checked belt tensions, calibrated all axes.\n\"enter

\n\n

The prints come out fine in terms of dimensions - all within 0.1 mm overall size on large prints - but the quality is now terrible. I'm using the same roll. Prints were back-to-back and humidity is at 20 % in the room I store and print in. I've checked the nozzle, checked the belts, tightened everything, rest the printer settings and put them back, regenerated the G-code with multiple slicers... I'm at a loss now.

\n\n

Any ideas, thoughts, comments, etc would be greatly appreciated.

\n", "Title": "Great prints then suddenly terrible quality", "Tags": "|print-quality|creality-ender-3|troubleshooting|warping|", "Answer": "

Maybe an adhesion issue. I use 320 wet/dry paper on my mats when they fail to work.\nJust circular motions.. Clean well with Windex or alcohol.

\n" }, { "Id": "11388", "CreationDate": "2019-11-19T03:50:07.807", "Body": "

It's fairly common for E3D to sell silicone socks for their hot ends. There are also other companies that sell these silicone socks for their hot end cartridges.

\n\n

According to a brief internet search, it seems the ignition temperature of silicone is surprisingly low - around 450 °C. This surprised me because I was under the impression silicone would just burn / evaporate if it were heated up to a much higher temperature.

\n\n

If my thermistor/heatrod slips off, my heatrod will glow into an orange temperature during thermal runway. This only happens briefly, but its color indicates it is reaching a temperature around 790 °C.

\n\n

So, are silicone socks safe? Couldn't they ignite fairly easily?

\n\n

\"Incandescence

\n", "Title": "Are silicone socks safe?", "Tags": "|safety|", "Answer": "

Silicone socks are safe to use, provided your printer is safely operating and you are using the silicone socks in their operating temperature range.

\n
\n

Your current setup is NOT SAFE!

\n
\n

When the heater element falls out of the heater block (that should not happen in the first place, please secure it correctly) and heats up to about 800\u00a0\u00b0C this means that the printer has no active Thermal Runaway Protection (TRP) enabled. Basically, when the thermistor doesn't measure a temperature rise while the voltage to the heater element is being scheduled, the firmware should shut down the voltage to the heater element. When this fails, the heater element can reach dangerously high temperatures to start burning anything that can catch a flame on touch. In deliberate tests, heaters have been able to melt the aluminium of the hotend:

\n

\r\n \r\n

\n

You should be worried at this point as you see from the link you provided what happens if the heater is not shut off when the heater element is disconnected from the heater block:

\n

\"Burnt

\n

Please fix your printer ASAP by uploading a proper firmware with enabled TRP protection before proceeding to print anything or at least don't let it print without supervision and proper smoke/fire detection devices.

\n" }, { "Id": "11391", "CreationDate": "2019-11-19T07:19:34.300", "Body": "

My rig:

\n\n\n\n

I've updated my start G-code from Cura to include a G29 right after G28 and my Klipper config has the following override for G29:

\n\n
[gcode_macro G29]\ngcode:\n    BED_MESH_CLEAR\n    BED_MESH_CALIBRATE\n    BED_MESH_PROFILE LOAD=default\n
\n\n

Every print now goes through BED_MESH_CALIBRATE sequence and the Bed Visualizer plugin yields this result:

\n\n

\"Bed

\n\n

Based on the info here, Klipper: G-code commands - Mesh Bed Leveling I would expect Klipper to adjust for the differences seen in the visualization above, but the initial layers of my prints are very squished on the right and rear of bed (right and rear of image) and not squished on the left and front of the bed (left and front of image) which make it seem like BED_MESH_CALIBRATE measured the differences in the bed level, but didn't actually do anything about it. Isn't the point of BED_MESH_CALIBRATE to adjust for these differences?

\n\n

You can see in the first pic below that the final move on the initial layer from the outside edge of the print (right front corner) to the inside has the nozzle so low that it melted a line in the first layer as it skidded across.

\n\n

In the second pic below you can see that the left front edge of the print didn't squish properly so it has detached from the bed. What am I missing?

\n\n

\"Melted

\n\n

\"Left

\n", "Title": "Klipper doesn't seem to adjust Z after BED_MESH_CALIBRATE", "Tags": "|creality-ender-3|bed-leveling|octoprint|bltouch|klipper|", "Answer": "

I have identified the problem as lead screw backlash!

\n\n

Bed calibration and Z-axis offset compensation appear to be working as intended, but there is enough backlash in the Ender 3 stock lead screw that the corrections in Z-axis movement commanded by Klipper result in almost no actual nozzle movement.

\n\n

I'm measuring about 0.35 mm of backlash in my lead screw. With differences between the lowest and highest points of my bed being only 0.6 mm the best case scenario (which would only happen if I was using my entire bed) is that the actual nozzle movement would only be about 42 % of what Klipper commanded. The fix for this is to change out the lead screw for a ball screw. The problem could also be mitigated in Klipper if Klipper implemented backlash compensation, but from what I've read on GitHub it doesn't seem like that is a feature the developer of Klipper wants to implement.

\n\n

Update:

\n\n

I installed this anti-backlash nut and it greatly reduced the backlash, but didn't eliminate it. I've uninstalled my solid bed mounts and reinstalled springs until I upgrade to a ball-screw for the Z axis. The auto bed leveling compensation still does something, but the adjustment allowed by having springs is what is really doing most of the leveling work.

\n" }, { "Id": "11407", "CreationDate": "2019-11-20T18:05:00.620", "Body": "

I would like to calculate the maximum safe acceleration for my printer using the following parameters as input:

\n\n\n\n

Elasticity of frame and belts are ignored.

\n\n

The Excel file to obtain the torque at the desired speed is available here.

\n\n

The other parameters are known in advance.

\n\n

The formula I used for the maximum acceleration is the usual $a=F/m$, where $F=torque*radius$:

\n\n

$acceleration = torque\\ *\\ (steps/rev\\ /\\ steps/mm\\ *\\ microsteps\\ /\\ pi\\ /\\ 2) / mass$

\n\n

Using 450 g, 0.15 N/m (as predicted for my stepper at 200 mm/s with 24 V), 200 steps/rev, 80 steps/mm, 16 microsteps I obtain about 2100 mm/s2 which seems reasonable and pretty close to standard values.

\n\n

However, from what I understood the Excel file provides the prediction of a full-step torque, but torque is known to decrease significantly with increasing microsteps.

\n\n

How to introduce the effect of reduced (incremental) torque for microsteps to calculate a safe maximum acceleration value for a printer?

\n\n

For information: my stepper and my printer were sold back then with about that acceleration value preset and with that torque at nominal speed (half voltage, half speed than I assumed here).

\n\n

The print quality was fine, which seems counterintuitive considering the much reduced torque expected at 16X microsteps, about 1/10 of the value I used.

\n\n

I would expect many lost microsteps with the predefined settings. Or maybe it happens: if all the microsteps are lost, the positioning error would be 16 (micro)steps / 80 steps/mm = 0.2 mm.

\n\n
\n\n

Important edit!!!

\n\n

As maybe someone noticed, the formula I used

\n\n

$F=torque*radius$

\n\n

is wrong. The correct one is

\n\n

$F=torque/radius$

\n\n

Taking this change into account, the final formula is:

\n\n

$acceleration = torque\\ /\\ (steps/rev\\ /\\ steps/mm\\ *\\ microsteps\\ /\\ pi\\ /\\ 2) / mass$

\n\n

which results, with the given input values, in 52 mm/s2.

\n\n

This is much less than what is normally set in printers. I doubt that belt stretching and frame flexibility can affect the setting so much, therefore in addition to the microstep aspect, this one should be answered too, since the question is about \"theoretically calculate the maximum acceleration\".

\n", "Title": "How to theoretically calculate the maximum acceleration?", "Tags": "|acceleration|", "Answer": "

You do not appear to have a correct understanding of how microstepping affects torque.

\n\n

What is calculated in the article you refer to is incremental torque. The word \"incremental\" is very important.

\n\n

A stepper motor consists of a permanent magnet rotor and an electromagnet stator. The electromagnets generate a magnetic field, to which the stator wants to align itself. Imagine the stepper motor being at rest. As we apply a torque to it, the rotor will start to deflect from its resting state in which it is aligned with the magnetic field. As you apply more torque, the stator will deflect more.

\n\n

Eventually, if we keep increasing the torque, the motor will no longer be able to hold its position and snap over to the next step. The torque at which this is happens is the holding torque. Essentially, you can think of the holding torque as the torque required to cause a full step deflection of the rotor's position (compared to where it would be aligned with the magnetic field). The deflection in response to a given torque load is called the static load angle.

\n\n

The article calculates incremental torque for microstepping. The incremental microstepping torque is the torque required to cause a microstep deflection. So, if we are using half stepping, the incremental torque is the torque required to cause a half step deflection. Naturally the torque required to cause a half step deflection is (much) lower than the torque required to cause a full step deflection.

\n\n

This is actually irrespective of what level of microstepping the motor is configured for. Causing a half step deflection requires the same amount of torque, regardless of whether the motor is using full or half stepping. All lower incremental torque means is that we're specifying the torque for a smaller deflection. It does not mean torque is reduced overall.

\n\n
\n

I would expect many lost microsteps with the predefined settings.

\n
\n\n

You cannot \"lose\" a microstep. The rotor of a stepper motor has physical increments, and losing a step is when it snaps to the next increment. The only thing you can lose is a full step.

\n\n

The stator of a stepper motor creates a rotating magnetic field. The rotor tries to follow this magnetic field, but (if under load) always lags behind it a little because the coupling between the two magnets behaves like a spring. In full-step mode, the rotating magnetic field moves in discrete steps. All enabling microstepping does is make the field rotate more \"smoothly\". However, it doesn't change the magnitude of the field.

\n\n

If you were to look at the graph of the acceleration of the rotor on a very small time scale, you would get a sawtooth wave. Each time the motor made a full step (i.e., the magnetic field jumps), the acceleration would be high (as the misalignment between the fields would be large) and then gradually drop as the rotor aligned itself again with the magnetic field.

\n\n

If you would use 16x microstepping, you would again see a sawtooth wave, but with a 16x higher frequency and with a lower peak-to-peak amplitude. However, the average value would be the same as for full step mode. The reduction in peak-to-peak amplitude corresponds to the reduction in incremental torque (and, for smooth acceleration, this reduction is actually good).

\n\n

The main reason to be interested in incremental torque is to determine the positioning error. If you imagine the printhead being at rest, a single microstep may not cause any motion because the incremental torque is too low to overcome static friction. So, using 16x microstepping will not allow 16x as precise positioning. However, maximum acceleration is not affected.

\n\n

The second part of your question, where you get the unrealistically low value of 52mm/s2, is based on a simple calculation error. The correct value is 52m/s2 or 52000mm/s2.

\n" }, { "Id": "11410", "CreationDate": "2019-11-22T00:43:44.027", "Body": "

Is anyone using hysteresis with their chamber in Marlin?

\n\n

I have read bang-bang with hysteresis is possible when using a bed heater. I would like to do that for chamber heating. I have some lamps that can't keep flicking on and off so rapidly around a setpoint - this causes the bulb to die out faster and the noise is annoying.

\n\n

Steps to Reproduce:

\n\n
    \n
  1. Start with fresh Marlin Firmware bugfix 2.0
  2. \n
  3. Enable chamber temperature in Configuration.h (TEMP_CHAMBER_PIN 1)
  4. \n
  5. Set chamber hysteresis value to 2 in Confgiuration_adv.h (#define TEMP_CHAMBER_HYSTERESIS 2). Enable #define CHAMBER_LIMIT_SWITCHING in Configuration_adv.h. Decide on pin to use for your heater, if necessary
  6. \n
\n\n

Expected behavior: I would like, when I set my chamber to 30 °C, the chamber lamps to come on until 32 °C, and to stay off until 28 °C.

\n\n

Actual behavior: chamber heater lamps flick on and off at 30 °C (29.9 - on, 30.1 - off).

\n", "Title": "Chamber heating using hysteresis in Marlin", "Tags": "|marlin|heat-management|chamber|", "Answer": "

It does work, but you need to reset EEPROM memory once updating the firmware, if you had it enabled, so the values are transferred into the EEPROM and subsequently used.

\n\n

To reset EEPROM, send the following command in your 3d printer terminal.

\n\n

M502

\n\n

Then, save the firmware default settings

\n\n

M500

\n" }, { "Id": "11415", "CreationDate": "2019-11-22T10:17:00.017", "Body": "

I just scavenged stepper motors from an old laser printer. These have integrated controllers. I'd like to use them without dismantling the controllers. I currently have 4, 6 and 9 wires stepper motors. Any help would be reatly appreciated :) Thanks !

\n", "Title": "Using stepper motors with integrated controllers", "Tags": "|diy-3d-printer|stepper-driver|stepper|motor|arduino|", "Answer": "

Many of the motors that look like stepper motors in laser printers are actually three-phase brush-less DC motors. These look like stepper motors, but are intended to be used differently, controlled differently, and serve a different function. Like stepper motors, they have a permanent magnet rotor surrounded by coils of wire.

\n\n

They typically are used in applications that require a definite rotation speed, rather than a definite position -- velocity rather than location. They create feedback to their controlling board via an encoder.

\n\n

Differences include having a smaller number of poles, and possibly having three coils rather than two. I suspect that the magnetic field profiles of the poles may also be different, since the primary design purpose is not to sharply define the restoring torque curve for a small deviation of the rotor from a rotational position.

\n\n

If these are the type of motors you have in hand, you won't find them very useful for a 3D printer motion control system. They are great for any form of continuous movement where position isn't critical -- maybe wheel motors for robots or for peristaltic pumps on your automated drink mixer.

\n\n

You may ask on what experience I speak. I have done tear-down analysis of several laser printers by different manufacturers.

\n" }, { "Id": "11416", "CreationDate": "2019-11-22T13:00:47.630", "Body": "

I have been experimenting with PETG on my CR-10 upgraded with an E3D all-metal hot end and Marlin firmware on the controller. I am printing with 0.8 mm nozzle at 250 °C and an 85 °C bed.

\n\n

When I print in vase mode, the prints come out beautifully (and quickly with the 0.8 mm nozzle). However, with solid-body prints, I routinely get a thermal runaway fault at about the third layer (the fault references E1). Can anyone suggest how to solve this and why the thermal runaway protection only trips with solid-body prints?

\n\n

For context, when I tried to print the same solid-body print in PLA (205 °C hot end and 50 °C bed) I did not get the thermal runaway fault.

\n", "Title": "Thermal runaway with specific prints", "Tags": "|marlin|creality-cr-10|petg|thermistor|", "Answer": "

Thermal runaway protection (see What is Thermal Runaway Protection?) is triggered when the scheduled voltage to the heater element does not result in a specified increase in temperature within a specified timeframe.

\n\n

The exit of hot filament from the nozzle and the loss of heat of the heater block and the conduction heat loss through the heat break to the cold end need to be supplied by the heater element. When the filament melting temperature is high, heat losses are relatively high as well, this means that the software needs to compensate by scheduling the heater element more quickly/activated sooner (there is some heat capacity in the heater block) and longer. If it is unable to provide enough heat than is being used, the temperature cannot be maintained and the printer notices that scheduling power doesn't result in a temperature rise. This will trigger the thermal runaway protection in the firmware.

\n\n

This can imply that too much heat leaves the nozzle in the form of hot filament, and as such, your printing speed may be too high (or the printing temperature too high).

\n\n

Note that you are using a 0.8 mm nozzle diameter, this means a flow increase of $ \\frac{0.8^2}{0.4^2}=4 $ with respect to a \"standard\" 0.4 mm nozzle! A slower print speed is therefore advised anyways.

\n\n

To solve this, you can either:

\n\n\n\n

To explain the latter option, from the Configuration.h you can read that if you encounter these problems, you can tune this in the Configuration_adv.h

\n\n

Excerpt from Configuration.h:

\n\n
\n * If you get \"Thermal Runaway\" or \"Heating failed\" errors the\n * details can be tuned in Configuration_adv.h\n
\n\n

Excerpt from Configuration_adv.h:

\n\n
\n * If you get false positives for \"Thermal Runaway\", increase\n * THERMAL_PROTECTION_HYSTERESIS and/or THERMAL_PROTECTION_PERIOD\n
\n\n

So you need to change these constants:

\n\n
#define THERMAL_PROTECTION_PERIOD 40        // Seconds\n#define THERMAL_PROTECTION_HYSTERESIS 4     // Degrees Celsius\n
\n\n

The most effective solution for your problem will be limiting print speed and experiment with printing at slightly lower temperatures.

\n" }, { "Id": "11418", "CreationDate": "2019-11-22T13:26:02.340", "Body": "

While I was calibrating my flow rate/extrusion multiplier, I noticed that one layer is shifted slightly. I wanted to re-print the cube in-case it was a fluke, but after two more prints, I saw that the same layer was being shifted in the same way. Also, that is the only layer that is shifted, every other layer is perfect/almost perfect.

\n\n

I'm using Cura 4.3.0, on an Ender 3 which I purchased about a month ago. The extruder is upgraded to a Trianglelabs Bondtech BMG clone. The E, X, Y, and Z steps have all been calibrated. The bed is level (I'm using a Creality ultrabase glass bed). I have upgraded the stock PTFE tubing to Capricorn, upgraded the PTFE coupler fittings (they have plastic tab inserts to keep the tube in place). The X gantry is level. I've also re-lubed the z-rod with super lube, made sure the lead screw nut is loose and printed out a z-spacer for the z stepper motor. That's about it, I'm still using the stock board but my SKR mini is in transit right now. Finally, the filament I'm using is the filament that Creality bundled in with the printer when I purchased it off their official website so I don't know how quality their filament is.

\n\n

Below is a picture of what I'm talking about, I've highlighted in the area with a sharpie. Sorry for the bad image quality my phone is on its last legs at the moment haha:\n\"enter

\n\n

This is the guide I was using to calibrate the flow rate and also where I got the STL

\n", "Title": "Why am I getting Z banding/shifted layer on a single layer?", "Tags": "|print-quality|ultimaker-cura|creality-ender-3|calibration|", "Answer": "

I started this as a comment but it couldn't fit. If it isn't a true answer and should be removed, I understand.

\n\n

You suggest in your question that perhaps the defect could be caused by a problem in the Z-axis lead screw. Indeed, that is possible. If so, it would probably introduce a defect at that position across the entire print bed. If you print a test object and the defect is on all faces at the same height, check your Z-axis. With the power removed, try slowly moving the bed up and down through the region where the defect occurs, feeling for any hangup, hesitation, looseness, or any other non-uniformity.

\n\n

If the defect is along a particular line, check the complementary axis. If the glitch is a X-axis line, check the Y-axis. If a Y-axis line, check the X-axis. Slid the axis back a forth slowly, feeling for any non-uniformity in the motion. It could be a scratch on the linear bearing, or a ding from dropping something. Any discontinuity can cause a linear Z-axis defect, up or down.

\n\n

It could even be cause by the printer not being firmly sitting on the table at an angle, so that after a particular point the balance point changes and the printer rocks, introducing a glitch.

\n\n

3D printers, and many other motion control systems, relay on flat, smooth, uniform bearing surfaces connected to smooth, continuous drive systems.

\n\n

If you attach photos of all four surfaces, we might have a better idea of how to look for the problem.

\n\n

It wouldn't hurt at this point to wipe and clean all sliding surfaces, and then re-lubricate them.

\n" }, { "Id": "11433", "CreationDate": "2019-11-26T14:54:17.227", "Body": "

In Fusion360, how do I align components according to their a midplane?

\n\n

I have 2 complex objects. They have no midpoint defined (is that even possible). How do I align them according to their mid points?

\n", "Title": "In Fusion360, how do I align components according to their a midplane?", "Tags": "|3d-design|fusion360|", "Answer": "

There are several ways to lock items together.

\n\n

Designed in position

\n\n

When you design the parts from the base up, you should lock the very first sketch to the origin in some way or another. Fusion shows this by making any \"well defined\" line as black in contrast to blue, like shown here:

\n\n

\"Left

\n\n

Using Components

\n\n

Once you have bodies, you can pretty much move them around in the timeflow. You then can start to define them as components. Let's assume we have these parts, a bolt, and a part into which it hinges. Of course, these two are modeled \"in place\" but that hasn't to be:

\n\n

\"Bolt

\n\n

Now, we want to lock them together... well, first, let's start and put them into components! Highlight one, then choose the \"new component\" command, and make sure it stays highlighted, from bodies is chosen then hit enter.

\n\n

\"New

\n\n

Repeat for the other part, so your tree should show 2 components now.

\n\n

\"Components

\n\n

Combine Components as modeled

\n\n

Now we want to combine them as modeled, right? Ok, let's do that, it's easy! get out the dropdown from the combine menu and press combine as modeled (Ctrl + J)

\n\n

\"enter

\n\n

Now, click both components and at first it will start to shake the screen, having chosen \"Stiff\" combination. This means the parts don't shift one against the other. If you had them in the right position in the first place, they will stay that way. But there are other ways, which are in the menu that popped up:

\n\n

\"All

\n\n

If for example, I choose \"rotation\", I can choose a rotation center by placing a node as seen here:

\n\n

\"Rotational

\n\n

Creating Joints between Components

\n\n

Now, what if the components are not in position like the bolt is in the wrong position entirely? Well, start with components, as shown above, then we'll see:

\n\n

\"Position\"

\n\n

Now, we should use the joint function:

\n\n

\"Joints\"

\n\n

Choose the first Component, then seek the point which face-position (or for cylinders: position on centerline) shall be the first component's joint-point shall be. For our part, let's take the big part's hole. Click the face, then choose one of the available points. For cylinders that's the ends and the center, for faces, it's the corners, center points of the edges and the center. If you hit the possible alignment points in the first place, it skips the second click to choose it. Once that is done, the body turns transparent.

\n\n

\"Select\n\"Select

\n\n

Repeat the same process for part 2 and the magic starts: Both parts suddenly move till the alignment points are in the same position, the screen shakes and BAM! We are at stiff joint again! The menu changes and we can start to define offsets!

\n\n

\"New

\n" }, { "Id": "11435", "CreationDate": "2019-11-27T06:00:50.600", "Body": "

I have an Anet A8 with the Anet 1.7 board, Skynet3d v2.3.2, and the stock sensor. I originally had this configuration with the stock extruder, but recently purchased an E3D v6 clone with a Bowden extruder. After installing a new bracket I had printed (TNS E3D v6 Bracket) with the adjustable stock sensor mount. I installed the extruder and sensor.

\n\n

I knew I had to adjust sensor position. I went through the 3DStackExchange post on Z probe boundary limits as well, and added all of the end positions and probe to nozzle offsets (I will list below) to my Configuration.h file, saved, checked the changes were there in the Configuration.h tab in the Arduino IDE and uploaded the firmware. After this I went to hit auto leveling in the prepare menu, and the positions were different from the stock extruder and mount, but when it got to the third position, the sensor was off the bed, and the nozzle dug into the bed.

\n\n

The configuration I was editing was from the Anet A8 5 button stock sensor Configuration.h file.

\n\n

My probe is (When looking from the front of the Anet A8) forward and right of the nozzle.

\n\n\n\n

The stock min_probe_edge is 10 mm, so with my math my Left Probe Bed Position is 47, Right is 210, Back is 207, and front is 10.

\n\n

I want to get my probe settings properly set up so I get my bed leveling back and I do not want to ruin my bed while leveling.

\n", "Title": "Anet A8 Z-Probe Bed Positioning going outside of bounds", "Tags": "|anet-a8|bed-leveling|z-probe|skynet3d|", "Answer": "

Do you have eeprom enabled? Because if you do, you can change configuration.h all you want but it's always gonna use the values from eeprom. I advise you to install repetier and change the eeprom values with that. Or maybe you can use the display if you have one.

\n" }, { "Id": "11438", "CreationDate": "2019-11-27T10:20:23.010", "Body": "

From the M502 documentation page can be read that M502:

\n\n
\n

Reset all configurable settings to their factory defaults.

\n
\n\n

Please note that this phrasing from the manual has been used in the question title!

\n\n
\n

To also reset settings in EEPROM, follow with M500.

\n
\n\n

Note that:

\n\n
\n

This command can be used even if EEPROM_SETTINGS is disabled.

\n
\n\n

The question is what is the definition of \"all configurable settings\"?

\n\n

Are these the settings that are displayed with M503, or are there hidden settings?

\n", "Title": "M502 will reset all configurable settings to their \"factory defaults\", which settings are those?", "Tags": "|marlin|g-code|knowledgebase|reset|", "Answer": "

Technically, the description as \"factory settings\" is misguiding, as the settings called up are much better described as \"firmware defined settings\". But since Firmware upgrades usually are rare and far between, these settings can be considered \"factory\" for the usual user, even as we always urge users to test if their firmware has TRP enabled and upgrade if not so.

\n\n

Depending on the firmware, this usually means the settings described in this answer, but it could also be more narrow or extend to different and custom settings inside the firmware. Marlin, when it uses EEPROM_SETTINGS, uses Configurations.h and the additional Configurations_adv.h to define what the factory settings are.

\n\n

For example in this question the firmware defined the additional settings in Configuration_adv.h. Installing firmware does not by itself alter the EEPROM, so these settings needed to be seeded into SRAM via M502 and then saved into EEPROM via M500.

\n\n

The remaining commands in the M50X series are obviously M501 and M503. M501 overwrites the SRAM settings with those from the EEPROM, useful if you toy with the SRAM settings to troubleshoot or play with offsets in a somewhat safe manner. M503 in turn reports all settings currently in the SRAM, which can be changed during running. Most of these settings can be stored into the EEPROM, if EEPROM_SETTINGS is enabled, but they don't necessarily have to come from the EEPROM at the moment, as they can be altered due to a lot of reasons. Your G-code to print could call for example G20 and M149 K just to mess with you by swapping to Inches and Kelvin, but that would be easily fixable by recalling M501. This following M503 output was given as an example by Sebastian.expert:

\n\n
G21    ; Units in mm\nM149 C ; Units in Celsius\nFilament settings: Disabled\nM200 D1.75\nM200 D0\nSteps per unit:\nM92 X100.00 Y100.00 Z398.70 E100.00\nMaximum feedrates (units/s):\nM203 X400.00 Y400.00 Z8.00 E50.00\nMaximum Acceleration (units/s2):\nM201 X1000 Y1000 Z100 E10000\nAcceleration (units/s2): P R T\nM204 P400.00 R1000.00 T1000.00\nAdvanced: S T B X Z E\nM205 S0.00 T0.00 B20000 X10.00 Y10.00 Z0.30 E5.00\nHome offset:\nM206 X0.00 Y0.00 Z0.00\nAuto Bed Leveling:\nM420 S1 Z0.00\nMaterial heatup parameters:\nM145 S0 H195 B55 F0\nM145 S1 H205 B60 F0\nPID settings:\nM301 P52.25 I5.60 D122.00\nZ-Probe Offset (mm):\nM851 Z-2.41\n
\n\n

Note that with a console or terminal, you could sometimes alter EEPROM settings directly via a UI. Among the terminals I know to have this ability is Repetier Host.

\n" }, { "Id": "11458", "CreationDate": "2019-12-02T11:14:10.940", "Body": "

I regularly have 3D models that consist of a Boolean union of balls and cylinders (molecule models, star maps, mathematical designs), that I then want to convert to a STL model for printing at Shapeways. But I don't know how to easily do that! I already have the model as a series of floating point numbers (four for each ball, and seven for each cylinder), so I don't want to have to go into a graphical interface to click/drag points. Instead I want to convert my lists of floating point numbers to STL from the command line.

\n\n

I thought this would be a trivial thing, but I cannot find what is the best tool for this. All I need, I think, is a pointer to what tools are most appropriate (although more detailed help will be appreciated :-)) I thought that \"gmsh\" might be good for this, but it is not immediately clear to me how to use it for that.

\n\n

I would like the software for this to be \"free\", in the sense that it does not cost money. Also, I am not a Windows user, so I would like to be able to run this under Linux or macOS.

\n", "Title": "Convert ball-stick model to STL from the command line", "Tags": "|3d-models|3d-design|", "Answer": "

I would suggest using OpenSCAD for this. OpenSCAD does not natively support reading data files that aren't 3D/2D models or images, but you can write a simple program to generate OpenSCAD code.

\n\n

I would suggest first writing an OpenSCAD file that implements your chosen type of \"ball\" and \"stick\", perhaps like:

\n\n
module ball(x, y, z) {\n    translate([x, y, z]) \n        sphere(d=10, $fn=60);\n}\nmodule stick(x, y, z, length, rx, ry, rz) {\n    translate([x, y, z]) \n    rotate([rx, ry, rz]) \n        cylinder(d=5, length=length, $fn=60, center=true);\n}\n
\n\n

Then, take your data file \u2014 let's suppose it's a CSV \u2014

\n\n
ball,0,0,0\nball,100,0,0\nstick,50,0,0,100,0,90,0\n
\n\n

and convert it into OpenSCAD by adding the appropriate syntax (this could even be done using a basic find-and-replace tool):

\n\n
import <ball-and-stick-definitions.scad>\nball(0,0,0);\nball(100,0,0);\nstick(50,0,0,100,0,90,0);\n
\n\n

Then, producing the STL is just a matter of running the openscad command on your generated OpenSCAD source code.

\n\n

If your stick parameters are not Euler angles but \"start point\" and \"end point\" you'll have to add the \"rotate to point at\u2026\" math yourself, but that is within OpenSCAD's capabilities as it has vector math operations and multmatrix, or trigonometry if you prefer to generate Euler angles; you don't need to do that math at the file-conversion stage.

\n" }, { "Id": "11462", "CreationDate": "2019-12-02T21:44:23.470", "Body": "

I am using a Prusa printer to print parts for prototyping. The project I am currently working on needs parts to be able to slide against each other and they need to be printed together. A bit like these gears, where the whole construction needs to be printed in one go and the gears need to be free to move afterwards. What clearance should one use between such moving/sliding parts to make it snug but not unmovable?

\n", "Title": "Clearance between moving parts", "Tags": "|print-quality|prusa-i3|linear-motion|", "Answer": "

Each printer will have different \"print in place\" tolerances, but you can find such a model with which to determine the numbers you seek.

\n\n

One such item is on Thingiverse:

\n\n

\"tolerance

\n" }, { "Id": "11464", "CreationDate": "2019-12-03T01:01:40.070", "Body": "

I'm new to 3D printing. I've made some projects before just fine, but this project is giving me problems.

\n\n

I 3D printed a trumpet mouthpiece, and the printer made supports inside the mouthpiece funnel. I can easily remove the exterior supports, but I don't know how to get to the supports inside. Does anyone know how I would do that?

\n\n

\"Here's\n\"Here's

\n\n

Here is the link to the model\n\"enter

\n", "Title": "Removing Supports Inside Cone", "Tags": "|print-quality|", "Answer": "

"Don't use supports"<-- that is the best answer.

\n

Warning

\n

Sadly, if you print vertically, you end up with an extremely fragile piece -it'll snap between layers. The OP properly printed horizontally for strength, and possibly by using a very "thin" layer height he can get by without supports. That's the only way to succeed.

\n

I guess I should state that this answer assumes you want to use the print in a trumpet. If it's purely for decoration, then definitely print vertically.

\n" }, { "Id": "11497", "CreationDate": "2019-12-08T11:33:57.167", "Body": "

Just starting with octoprint.

\n\n\n\n

I managed to connect once my pi to the wifi, configured it, but as soon as I reboot my pi, it won't connect to the wifi again.

\n\n

Any hints?

\n\n

Nothing helped me in the wifi troobleshooting FAQ.

\n\n

I thought that I changed a particular config but :

\n\n\n\n

there is no more differences.

\n", "Title": "Octopi connecting just once to wifi", "Tags": "|octoprint|wi-fi|", "Answer": "

What worked for me:

\n\n\n" }, { "Id": "11524", "CreationDate": "2019-12-10T09:07:43.887", "Body": "

I'm new to 3D printing and I'm learning how to build my own 3D printer.

\n\n

I bought polymer LM8UU bearings and I'm trying to fit onto a rod of dimension: D8 mm * L300 mm. Sadly it doesn't fit, I was wondering if I would need to buy new rod?

\n\n

If so, what size of diameter should be used? General guidelines I need to look out for are also appreciated?

\n", "Title": "Building 3D printer: LM8UU bearing can't fit into", "Tags": "|diy-3d-printer|bearing|", "Answer": "

Bearing on a shaft

\n

An 8 mm (ID, Internal Diameter) bearing should always fit on an 8 mm (OD, Outer Diameter) shaft. If it does not fit, one of the 2 is of a different size. Please use a caliper to determine the size of both.

\n

Bearing in a housing

\n

If it is the outer size of the bearing, i.e. fitting in a printed carriage or a bearing housing (e.g. SCxxUU), you should be aware of the different sizes of polymer bearings. E.g. there are RJMP-01-xx and RJ4JP-01-xx types of polymer bearings (xx denotes the shaft diameter, e.g. 08 for 8\u00a0mm shafts).

\n

For RJMP-01-xx bearings (not compatible with LMxxUU bearings):

\n
\n// Parameters for RJMP-01-xx (shaft size xx, all dimensions in [mm])\n// Designation  d1  d2  B   B1      s   dn\n//-------------------------------------------\n// RJMP-01-08   8   16  25  16.2    1.1  15.2\n
\n

For RJ4JP-01-xx bearings (compatible with LMxxUU bearings):

\n
\n// Parameters for RJ4JP-01-xx (shaft size xx, all dimensions in [mm])\n// Designation  d1  d2  B   B1      s   dn\n//-------------------------------------------\n// RJ4JP-01-08  8   15  24  17.5    1.1  14.3\n
\n

d1 is the inner diameter, d2 the outer diameter and B the length of the bearing

\n

The RJ4JP-01-08 bearings are drop-in replacements for the LM8UU linear bearings. Note that the RJMP-01-08 have a larger outer diameter and do not fit into the regular bearing housings. The increased thickness gives the bearings more rigidity.

\n" }, { "Id": "11526", "CreationDate": "2019-12-10T16:43:59.517", "Body": "

I'm trying to find out how \"normal\" filament (not super expensive filament) varies in diameter (or more accurately cross sectional area) over the length of the filament. I've looked around and the only thing I can find, that many filaments are sold with the tolerances, not how \"fast\" they vary along the filament. A +- 0.1 mm over 1 meter is after all qite different from the same alteration over 0.1 meter.

\n\n

I am mainly interested in this, as I want to build a printer with very small Z-height steps, thus a small variation in the filament diameter will lead to a rather large variation in the extrusion width. While I thought over the project, I came to the idea of using some kind of capacitance measuring device to detect the cross-sectional area, however it is only possible (or rather feasible) to measure the average cross-section over a rather long section (10cm+). Hence main part of the question: How \"fast\" does the filament change diameter?

\n\n

The other part is obviously: Are there other (cheap) ways to measure the cross-section? I could use light, but then I would only get the diameter at one point (pretty sure the filaments aren't perfectly round) and using multiple sensors would quickly become expensive. And then there's the issue of transparent filaments. Most mechanical solutions have the same issue, only measuring one point and might have issues with certain types of filaments such as very flexible filaments.

\n", "Title": "Filament Variation Detection", "Tags": "|filament|dimensional-accuracy|", "Answer": "

You would want a thickness gauge with the ability to communicate via serial. Once you managed to modify Marlin to talk to it, and you engineered a system of rollers for the filament to pass through, then you could automatically compensate for the thickness of the filament.

\n\n

\"Neoteck

\n\n

https://www.amazon.com/Neoteck-Digital-Thickness-Electronic-Micrometer/dp/B07Q33RSH6?th=1

\n" }, { "Id": "11548", "CreationDate": "2019-12-12T21:50:28.493", "Body": "

I woke up this morning with my 20 hour long print failed around 15 hour in. I checked the octoprint console, and saw a very weird temperature graph. Later found out thermal runaway protection kicked in, and I thanked the gods for having decided to install a custom firmware to enable that protection.

\n\n

I have an anycubic i3 mega, recently upgraded with an E3D v6. To make that upgrade, I had to splice new connectors on both the thermistor and the cartridge, as the ones they came with weren't going to work.

\n\n

Ever since upgrading I started experiencing some weird Z-wobble, but I am coming to the conclusion that it may actually be a slight consistent underextrusion. The final layers of the parts I had been printing, are all severely underextruded, going as far as delaminating. I have attached both a picture of the temperature reading, and one of the failed parts. You should also see the underextrusion I was talking about in the first layers, and the major underextrusion in the final layers.

\n\n

Now, I don't have a lot of experience with this kind of issue but I think the temperature was jumping around too quickly to actually be a good reading. Sadly I was only able to take the final part of the graph before the printer shut down.

\n\n

I have taken some steps to try to avoid this from happening again

\n\n\n\n

Before this all went down, I had ordered a titan extruder to try to mitigate the underextrusion, as I read online (I think it was from E3D official sources) that the internal pressure of the v6 may be higher than the standard hotend and therefore I may have needed a geared extruder.

\n\n

Do you have any advice for me? And if you could also help me figure out if the first layers actually show underextrusion or Z-wobble, thanks for making it this far.

\n\n
\n\n

I should mention that this had happened to me already but without triggering the protection, the part did heavily underextrude but it was just a test part while trying to find out what was going on with the \"Z-wobble.\" It doesn't seem to consistently happen every time I start a print, but I figure that long prints would certainly make it more likely to happen.

\n\n

\"Temperature\n\"Failed

\n", "Title": "Weird temperature graph and thermal runaway protection triggered", "Tags": "|troubleshooting|hotend|thermistor|underextrusion|e3d-v6|", "Answer": "

Periodic temperature irregularities, such as cycling between a higher temperature and lower temperature slowly enough that it spends at least a layer or two at a different temperature than other layers has a tendency to be mistaken for z wobble. You can actually intentionally modulate print temperature (at least of PLA) every few layers to great a sort of banding or 'wood grain' effect. Higher temperature, for whatever reason, results in thicker perimeters and cooler temperatures thinner ones. This is when always staying above the normal PLA print temperature though. The temperatures in the graph would certainly cause serious under extrusion during the times it was under temperature.

\n\n

Understand though, it is likely that temperature has been behind the underextrusion, rather than it being a problem in of itself.

\n\n

It isn't possible for the temperature to jump around too quickly to get a good reading. The temperature changes relatively slowly, simply due to the thermal mass of the heat block. It takes a little bit for the extruder to come up to temperature, right? When warming up from room temperature, watch how fast the temperature changes. That's the fastest the temperature can change, because that is when the heater cartridge is on at full blast. And even then, it isn't particularly quick.

\n\n

It also does not appear to be a problem with the thermistor. If there was a poor or failing connection (like a wire that was almost broken), this would cause added resistance to the thermistor reading, and thermistors usually lose resistance as they heat up. So extra resistance throwing off the reading would make the printer believe that the hotend was cooler than it actually was, and you would wind up printing at a too high of a temperature. This would cause a number of problems, but your interlayer adhesion would be excellent. Delimitation would not be one of the issues, nor would underextrusion.

\n\n

Thermal runaway protection is kind of a misnomer, because it is really 'something about the temperature of something isn't behaving like it should' protection. Basically all it is is a hard coded temperature and time window. If it has turned on the heater, there is a hard coded number of degrees that the thermistor must increase by within a certain time window, usually 30 seconds to a couple of minutes. If it doesn't, something is wrong. Or, if it drops out of the intended temperature by another hard coded number of degrees for a certain amount of time, then again the thermal runaway protection will be triggered.

\n\n

In cases where the connection to the thermistor is failing, this will prevent the hotend from being heated without any limit (poor thermistor connection means the temperature the printer reads is always much much lower than what it really is, so the printer keeps trying to heat the hotend up hotter).

\n\n

But thermal runaway protection will also get triggered for much more benign problems, like a failing connection to the heater cartridge. If the thermistor is working fine, but the cartridge is not working correctly and either only heats intermittently, or has a poor connection and can't get enough current through it, this will also result in the temperature not going up as expected, thus triggering the thermal runaway protection.

\n\n

Based on your symptoms, the thermistor reading looks to be quite accurate. What you describe is exactly what I'd expect would happen for a print where the temperature of the hotend really did vary exactly as shown in your graph.

\n\n

What can often happen is one of the leads (especially close to the cartridge but really, it could happen anywhere along the length of the two leads) will have broken from repeated wire strain (if you have disassembled your hot end at all or otherwise disturbed the heater wires in anyway, this is more than enough to cause a lot of metal fatigue), but the insulation around the broken wire will hold it together such that wire will still be making contact with itself. But it will be a more resistive connection, and will cycle between an acceptable and poor connection as the print head position moves.

\n\n

Long story short, you get a temperature graph that looks just like that, because the heater cartridge is periodically either developing a poor connection, or losing its connection entirely, only to regain it again as things move just the right way again.

\n\n

I would double check all your connections to the cartridge, checking the actual resistance and not just using a continuity tester. If those seem ok, then you will probably just need to buy a replacement cartridge. I like to keep a few of them on hand since they have a tendency for their wires to break just from a little bit of normal manhandling.

\n\n
\n" }, { "Id": "11549", "CreationDate": "2019-12-13T10:00:11.567", "Body": "

Here is the context
\nI've got an old car for which I have a small plastic piece who is broken. As it's an old car and a very specific piece, I can't find it anymore. So I was thinking about 3D printing it.

\n\n

My problem is this piece is on the carburetor, so close to the engine. This means, it can heat a lot, close to 90-100 \u00b0C.

\n\n

My question
\nDo the pieces created with the common 3D printing techniques melt at 100 \u00b0C? If yes, what kind of other 3D printing technique can I use?

\n\n

Here is the piece I want to recreate (sorry for the bad quality), the scale is in cm.\n\"The

\n", "Title": "What is the melting temperature of a 3D printed part?", "Tags": "|desktop-printer|", "Answer": "

Although metal casting has been briefly mentioned, another approach to your problem would be to create a silicone mold from your print and cast the part using epoxy or polyurethane, both of which have the desired strength and heat properties.

\n

Here is an example of this technique\n

\r\n \r\n

\n" }, { "Id": "11561", "CreationDate": "2019-12-14T17:37:28.810", "Body": "

What is different about cleaning filament to make it effective? What is it made out of? How does it work?

\n", "Title": "What's so special about cleaning filament?", "Tags": "|filament|", "Answer": "

You may discover that \"cleaning filament\" is also described as nylon filament. Nylon requires higher temperatures than most commonly used filaments. As you raise the hot end to the required temperature to melt the nylon (typically 250\u00b0C, all of the other debris has either carbonized or melted.

\n\n

The hot end is allowed to cool (30-50\u00b0C) at which point the nylon filament is pulled out in the reverse direction. Some 'net references suggest a hard pull, but I disagree with violent mechanical forces being applied to delicate mechanical devices. Perhaps that's why my cleaning process takes two to four attempts. Some net references also suggest to start the nozzle heating after reaching the cooling point and to begin applying force upward during the re-heating.

\n\n

My Sigma 16 uses the above method and also suggests a \"strong pull\" which is a translation from \"sharp yank,\" in my opinion.

\n\n

This will collect the debris from the nozzle and heat break and may completely clean the filament path.

\n\n

I use \"natural\" clear nylon for cleaning and perform the sequence two to three times, until the heated portion of the nylon no longer has contamination visible.

\n\n

Even nylon filament that is not dry enough for printing works for cleaning. The moisture bubbles turn the filament into flimsy punctured nylon thread, but it causes no problem with the cleaning process.

\n\n

FilamentOne website references most of what I've posted.

\n\n

\"filament

\n\n

The above image resembles my experience, although the severely \"dirty\" image is much more excessive than my cleanings. The worst is to have PVA support material that has been \"cooked\" at ABS temperatures or higher for long periods. An almost guaranteed nozzle clog is the result of those conditions.

\n" }, { "Id": "11562", "CreationDate": "2019-12-14T18:22:42.047", "Body": "

I'm printing with a Prusa MK3, with the following settings:

\n\n\n\n

The filament is Polyalchemy emerald green (PLA). Nozzle temperature: 210 \u00b0C.

\n\n

On a simple part (it's a keychain), the shell detaches if I apply a bit of force on a zone of the part that is \"fragile\". See picture. You might not be able to see it, but only the 2 external perimeters detach from the rest of the part. I used to print this part on another printer, and I never observed this problem.

\n\n

Any idea on how to solve this problem? It seems the 3 external perimeters didn't fuse properly.

\n\n

\"enter

\n", "Title": "Shell detaches from the rest of the part", "Tags": "|prusa-i3|pla|", "Answer": "

I ultimately concluded that the material isn't great. I printed this part with multiple other PLAs and never observed any problem. The Polyalchemy PLA looks great, but for any part that has some sort of mechanical constraint, it behaves poorly.

\n" }, { "Id": "11576", "CreationDate": "2019-12-17T22:07:17.777", "Body": "

I am attempting to build a 3D printer using the Bear Upgrade for guidance.

\n\n

However, I want to modify some of the parts. I am basing the modified parts on the original designs.

\n\n

I would like to understand the reason behind some design details presented on the original designs so that, if necessary, introduce them into the new designs.

\n\n

\"example

\n\n

Item 1) The holes are not round. Why???

\n\n

Item 2 ) There are some little squares inside the piece which i don't know if they are there for some structural reason

\n\n

Please use this link to the piece depicted above.

\n\n

Here is a drawing of the full assembly, showing that this piece is the y axes linear rods holder:

\n\n

\"enter

\n", "Title": "Help needed to design 3D printer part", "Tags": "|3d-design|diy-3d-printer|fusion360|", "Answer": "

Teardrop shape\nere is a feature there which is inside of the part which you cannot see otherwise. By turning it, you should be able to decipher what these features are for and therefore should be able to

\n" }, { "Id": "11580", "CreationDate": "2019-12-18T13:53:28.950", "Body": "

I have an XYZ DaVinci Mini w which keeps jamming. It does not give a Jam Error. I disassembled the extruder 1 time and found a short piece of PLA stuck in it however that did not fix anything. I've also tried using the Load Filament function repeatedly to try to get the filament out but it does not work. However, after disassembling the extruder, instead of just not printing it makes a loud repeated clicking sound.

\n\n

Any Ideas?

\n", "Title": "XYZ DaVinci 3D Printer constantly jamming", "Tags": "|extruder|xyz-da-vinci-mini-w|", "Answer": "

The clicking sound typically means the drive gears are slipping on the filament because there is a jam in the head/nozzle.
\nFirst thing to do is clear the nozzle. You can use a small drill bit to clear the chamber, but if the extrusion hole (at the tip) is clogged, you'll need to get a set of fine wires designed for extruder clearing. Then heat the head above melting point and slip the wire in (if possible).

\n\n

In general, I find it easier and faster to replace clogged nozzles. They're cheap, and more often than not clearing the exit hole enlarges the diameter (bad thing).

\n" }, { "Id": "11586", "CreationDate": "2019-12-19T01:35:16.663", "Body": "

I have a monoprice select mini v2, and it has worked well up until now. It currently displays 999 degrees for the build plate temperature. This is a glitch, and I don't know how to fix it. Since it thinks the temperature is so high, it thinks that the build plate does not need to be heated. Because of this, many of my prints turn out warped. Is there a way to fix this?

\n", "Title": "Monoprice select mini V2 displaying '999' degrees for build plate temp", "Tags": "|heated-bed|monoprice-select-mini|", "Answer": "

You get this temperature reading when there is broken wire. Not owning this printer type, can't you switch out the bed thermistor for a new one? Or alternatively check if a connection is loose.

\n" }, { "Id": "11595", "CreationDate": "2019-12-20T08:20:42.203", "Body": "

I have this image in STL

\n\n

\"3D

\n\n

But it seems cura skipped the lower panel entirely when slicing the STL.

\n\n

\"Sliced\"

\n\n

The base is thick, as you can see here, but it is still missing in Preview, and Cura seems to ignore the base.

\n\n

\"Base

\n\n

I have tried to decrease to 0.1mm for width or initial layer, but cura still ignored the base panel.

\n\n

I also found this odd behaviour in a number of my models.. such as this:

\n\n

\"Car\"

\n\n

or this

\n\n

\"Car

\n\n

Notice those missing parts? It is like some parts are floating in the air.

\n\n

Can someone help me out here? What kind of setting I have to modify?

\n", "Title": "Cura 4.4 missing object parts", "Tags": "|ultimaker-cura|3d-models|stl|", "Answer": "

Using the preview feature as you have is a good method to determine that the original model is flawed. It's not unusual to discover detailed models have reversed normals or non-contiguous surfaces, which will not print.

\n\n

In a simple example, consider a cube for which five of the six faces are properly described by the design software. This would result in all eight vertices being present, allowing the software to present the \"missing\" face, but the slicer is unable to create this aspect. The cube is no longer solid and is presented as seven zero-thickness panels, which can't be printed.

\n\n

It is necessary to repair the broken models, by closing gaps and reverting/recalculating normals, but that requires skill in other software.

\n\n

Some slicers will attempt to repair damaged models, but some are so severe, the results you've experienced are the result.

\n\n

If you want a confirmation of a failed model, load it into free Meshmixer, run Analysis/Inspector and expect a \"pincushion\" of highlighted failures.

\n" }, { "Id": "11597", "CreationDate": "2019-12-21T04:12:04.200", "Body": "

I have an Ender 3 Pro with a BL Touch.

\n\n

In my G-code should I add an M500 after the G29, to save the results to the EEPROM? I know storage size is an issue, so does storing these results cause an issue?

\n\n

I believe the saved results can be activated before the next print using: M420 S1

\n\n

If I do, does that mean I just auto home (G28) but don\u2019t need to run a G29 until I think the bed has lost its level?

\n\n

I was trying to clarify this after reading:\nBL Touch Probing Fails Intermittently

\n", "Title": "Saving BL Touch settings", "Tags": "|creality-ender-3|g-code|bltouch|", "Answer": "

From this source you can read:

\n\n
\n

After a G29 the leveling data is only stored in RAM. You have to use M500 to save the bed leveling data to EEPROM, otherwise the data will be lost when you restart (or reconnect) the printer. Use M502 to reset the bed leveling data (and other settings to defaults). Use M501 to reload your last-saved bed leveling from EEPROM. This is done automatically on reboot.

\n
\n\n

The source also answers the use of M420 S1:

\n\n
\n

After a G29 bed leveling is automatically enabled, but in all other situations you must use M420 S1 to enable bed leveling. It is essential to include the command M420 S1 in the \u201cStart G-code\u201d in your slicer settings. If you have no bed leveling, or if there is no leveling data, then this command is simply ignored.

\n
\n\n

So, if you're not using a G29 in your start code you must use the G-code M420 S1 to enable the stored mesh from memory.

\n" }, { "Id": "11602", "CreationDate": "2019-12-21T19:02:18.363", "Body": "

I recently purchased a small stepper motor with a hollow shaft, without realizing it ran at 3.8V. I'm looking for the easiest way to integrate it with the ecosystem around the 12V steppers used in 3d printing, including control by Marlin firmware, and also compatibility with mainstream printer boards.

\n\n

The rated specs are as follows:

\n\n
\n    2 phase 4 wires\n    1.8 step angle\n    Rated Voltage: 3.8V\n    Rated Current: 0.8A\n    Body Length: 30mm\n    Shaft Diameter: 5mm\n\n
\n\n

I have only A4988 drivers, and I was under the impression that those drivers could only work at 12V. I have two spare DC-DC buck converters that could drop a 12V output from something to 3.8V. But I'm not sure what wires to do that for.

\n\n

I know that motors have \"A\" and \"B\" coils for movement. Do I just use two DC-DC bucks off an A4988 driver to power this thing? So, after the output from the A4988 driver, I would connect 1A and 1B to IN+ and IN- on the DC-DC buck, and 2A 2B on IN+ IN- on the second DC-DC buck.

\n\n

Is there a better way to integrate this motor into an existing 3D printer board based on RAMPS? I'm using MKS Gen and MKS Sgen boards

\n\n

Using the pins before the driver might not be such a good idea if 800 mA is needed to run the printer. I don't know whether those pins connect to 12V or to the microcontroller output.

\n", "Title": "How to use a 3.8V stepper with Marlin / RAMPS?", "Tags": "|motor|", "Answer": "

The 3.8 V rating does not mean what you think it does. \"Rated voltage\" has a very specific technical meaning. For a 3D stepper motor to work properly, the rated voltage of the stepper motors actually needs to be significantly lower than the supply voltage of the stepper drivers.

\n\n

These steppers are perfectly compatible with a standard 12V RAMPS setup with A4988 system. You do not need to and should not mess around with buck converters. All you need to do is adjust the potentiometer on your stepper drivers to limit the current to at most 0.8 A.

\n\n

The rated voltage in some sense refers to the average voltage the stepper motor should see, but to actually run them at reasonable speeds you need to supply short bursts of higher voltage. A stepper driver and motor actually form a sort of buck converter themselves, and in some sense do the conversion from 12V to the rated voltage for you.

\n" }, { "Id": "11608", "CreationDate": "2019-12-22T15:39:15.720", "Body": "

I bought an Ender 3 two days ago and assembled it today. I think I did it properly, tested the movement of all axes which works for all axes. Then I performed the calibration as described in the manual. I used a piece of paper and adjusted the bed until it barely fits underneath the nozzle for all four corners. Afterwards, I wanted to print my first model so I selected the cat from the usb stick which came with printer. And now comes my problem. I let the printer run for 15 minutes. It moved and moved and moved but there was no filament on the bed. The nozzle and bed were heated properly. The one thing which I noticed was that the stepper which feeds the filament turns for like 30\u00b0 and then flips back: to me it looks like the filament can not be fed in. After canceling the print the extruder moves back to the home position which is like 5\u00a0mm off the bed and then suddenly the filament flows out of the nozzle.

\n

What part of the configuration I'm missing?

\n", "Title": "Brand new Ender 3 does not extrude any filament during printing", "Tags": "|filament|creality-ender-3|", "Answer": "

The stepper motor turning back is retraction. It is a normals process for 3D printers. I suggest you check that the filament is able to extrude at all.

\n\n
    \n
  1. Using the control panel, go to prep then move axis and move the nozzle up(I forget which axis)

  2. \n
  3. Heat the nozzel up to 200 degrees.

  4. \n
  5. Push filament through the nozzel by hand and see if it extrudes.

  6. \n
\n\n

If not, check that the nozzel is unblocked. To unblock the nozzel get a thin needle and clear the hole while still hot. Unless u have a nozzel cleaning tool.

\n\n

If it is not blocked the filament may need changing.

\n" }, { "Id": "11621", "CreationDate": "2019-12-24T08:15:56.250", "Body": "

I have been reading through the Marlin website about ABL (Auto Bed Leveling) and I see what appears to be conflicting information.

\n\n

At https://marlinfw.org/docs/features/auto_bed_leveling.html it states that running G28 (Auto Home) will turn off ABL and that to reactivate it you need to insert the code M420 S1.

\n\n

However at https://marlinfw.org/docs/gcode/G029-mbl.html it states:

\n\n
\n

G28 disables bed leveling. Follow with M420 S to turn leveling on, or\n use RESTORE_LEVELING_AFTER_G28 to automatically keep leveling on after\n G28

\n
\n\n

The latter is referring mostly to manual bed leveling, but in this section is referring to ABL.

\n\n

So what is the difference between the S and the S1 options?

\n", "Title": "What is the difference between M420 S and M420 S1?", "Tags": "|marlin|g-code|bed-leveling|", "Answer": "

TL;DR

\n

Short answer: "There is no difference!".

\n
\n

Long answer

\n

G-code is parsed line by line. The parser in the firmware reads the command and the options (also called parameters) following it, comments are discarded.

\n

Therefore, option S should be followed by a Boolean value 0 for False or 1 for True. Without a value it is undefined and should be reported as incorrect or treated as false (in my opinion). But, the developers have chosen differently. If a Boolean value is expected and the value is not given, the parameter is parsed as true. This has analogy with other software options or switches like e.g. used by Linux commands.

\n

This means that for Marlin the G-code M420 S is exactly the same as M420 S1.

\n

I've checked this with another command (this is not depending on the G-code command as there is a single parser processiong the G-codes and the following parameters); M211 where M211 S is exactly the same as M211 S1.

\n" }, { "Id": "11634", "CreationDate": "2019-12-28T21:09:47.683", "Body": "

I'm trying to increase adhesion of the first layer (as well as to fill gaps for a more even surface) by squeezing more material against the bed. The obvious way of doing that in Cura is by increasing the \"Initial Layer Flow\", i.e. to make the printer push out slightly more material than it normally would.

\n\n

But then there is also a setting called \"Initial Layer Width\" and according to the the Cura Settings Guide (see image below), increasing line width will make the nozzle

\n\n
\n

extrude more material and that material has to flow wider outward. This causes the nozzle to press the material harder on the build plate (...) Not only will the lines be wider ... but they will also be farther apart ... by the same factor, so it would not produce overextrusion

\n
\n\n

This seems to imply that increasing the initial layer width will automatically also increase initial layer flow. If this is so, the question is: which setting is applied first?

\n\n

In any case, it seems that the two settings should not be applied together, if they manipulate the same variable (but I have not seen this recommendation anywhere). Which leads me to my may question: what is the difference between the two settings? More specifically (based on my above reasoning): what else does \"Initial Layer Width\" manipulate, apart from the flow rate in the first layer? Just the distance between the lines so that increasing the setting will lead to fewer lines?

\n\n

\"enter

\n", "Title": "What's the difference between \"Initial Layer Width\" and \"Initial Layer Flow\" in Cura?", "Tags": "|ultimaker-cura|adhesion|", "Answer": "

The typical consensus is that you increase the layer flow initially for better adhesion, though from my experience I decrease it!
\nMy first layer is printed at between 70-75\u00a0% layer flow, this gives the best adhesion and best visuals when printing with ASA or ABS.
\nFrom layer 2 on I've 105\u00a0% layer flow.

\n

The reason is that my first layer is printed "officially" at 0.27mm but in reality, it's more like 0.05\u00a0mm thickness. That's manually finetuned while printing after a material change, basically, the thickness is adjusted for perfect grip on the build plate.
\nWhen left at 100\u00a0% flow rate this will cause ripples or waves on the bottom, it's excess material that builds up along the printed lines.
\nAt 70\u00a0% the wrong initial distance is compensated (visually) while maintaining perfect adhesion.

\n

Conclusion: Fine-tune your printer and settings for each material used, a general answer is not possible. Especially true for difficult materials like ABS, ASS, or Nylon.
\nThe best is to watch the printer while it is printing and adapt the mechanical properties first, then fine tuning the flow rates.

\n" }, { "Id": "11638", "CreationDate": "2019-12-29T16:55:22.417", "Body": "

I printed a model and now I can't remove it.

\n\n

I have been chiselling away with a putty knife and made little or no progress. I even heated the bed up to 70 °C. That really didn't seem to help.

\n\n

Last time, I put it in vice, and tried to free it that way, but instead I broke the glass.

\n\n

Suggestion?

\n", "Title": "How do I remove a 3D print stuck to the glass build plate", "Tags": "|post-processing|glass-bed|", "Answer": "

I don't have a glass bed, but I've had a lot of luck with using dental floss to get stuck prints loose.

\n\n

If you can get the floss under the edge of the print, then you just pull it through to the other side.

\n\n

This technique takes some practice but works really well once you get the hang of it.

\n" }, { "Id": "11650", "CreationDate": "2019-12-31T21:11:24.947", "Body": "

Just added inductive sensor to my printer (MKS Gen L Board) using Marlin 1.1.9\nAfter some tweaking it works great!\nThe problem is when it starts print the sensor detects the bed and stops the print because it gets triggered.

\n\n

Can I tell Marlin to ignore the Z-min readings while printing?

\n", "Title": "Inductive Sensor Causes Print To Stop", "Tags": "|marlin|inductive-sensor|", "Answer": "

Solved the problem, I was missing this line:

\n\n
#define ENDSTOPPULLUP_ZMIN_PROBE\n
\n\n

Apparently the sensor required a pull-up to better define the signal.

\n" }, { "Id": "11657", "CreationDate": "2020-01-02T16:07:03.003", "Body": "

I would like to build a standing shelf where the supports which hold each successive plank are 3D printed (to obtain special shapes).

\n\n

However, I read that PLA flows under constant stress/pressure. Still, this doesn't stop the author of the article from using PLA for a hanging shelf, which obviously is subjected to constant negative pressure.

\n\n

Which material suffers the least from plasticity/non elastic deformation under stress?

\n\n

Answers with data for multiple materials are welcome.

\n\n

I found that the phenomenon is called \"creep\" and is related to ISO 899, but I couldn't find any data for the common filament plastics and I don't know the theory behind it, so I'm not sure whether it's unavoidable or it appears after a threshold stress is reached.

\n\n

Information: it's a living room shelf which will hold books and other stuff and is supposed to last a decade. I will of course add a safety factor and I could even fix the planks to the wall (in hidden places), but ideally the 3D printed material should have NO creep.

\n", "Title": "Which material \"creeps\" (plastic deformation) the least (or nothing at all) under pressure after being printed?", "Tags": "|material|print-strength|", "Answer": "

Your question can not be answered theoretically -- only empirically.

\n\n

You need to print some trial brackets with materials of interest and measure them. The question is: under what conditions should you test them?

\n\n

The problem is that creep involves both compression and tension, and the behavior may be different. It is also impossible to fully translate material specifications into component behavior without a really good model that includes the details of the infill, adhesion to peripheries, and all the microscopic detail of a real 3D-printed part.

\n\n

The problem with typical PLA may be the low temperature. Raising the temperature of a normal PLA printed object to 160\u00b0F (70\u00b0C) softens it to the point of nearly being limp. I have used this for force-fitting PLA parts by warming a pot of water and placing parts in it for a few moments. That temperature is hotter than your room, but a hot summer day in the sunshine could soften the part to the point of failure.

\n\n

For anything load bearing, I would want a material with a higher plastic temperature.

\n\n

There are PLA formulations which are annealed after printing. This is claimed to allow the PLA to slowly recrystalize and become both stronger and usable at higher temperatures. I don't have experience with this.

\n\n

Depending on your printer, you can also consider using a higher temperature filament, such as ABS or PC (polycarbonate). PET-G is a little better than PLA, but it softens at a lower temperature than ABS.

\n\n

As important as the material itself is the anisotrophy of the printer parts. Be sure to print the parts so that the major stresses are along the strongest axes, typically X and Y, and not along the weaker Z asis. Choose your infill to contribute to the strength, and use plenty of it, or design the part so that the infill is not intended to contribute to the strength.

\n" }, { "Id": "11658", "CreationDate": "2020-01-02T17:08:24.880", "Body": "

I was experimenting with building an enclosure using fusion 360.

\n\n

I was wondering if a part with rounded corners of a larger radius would warp less than smaller ones?

\n\n

Also, any recommendation on reading material would be appreciated!

\n", "Title": "Does having more curvier corners help warping more then smaller curves", "Tags": "|3d-models|fusion360|", "Answer": "

In short, yes, it helps a little. Curves provide less surface area per unit volume (a sphere has the least possible surface area relative to the volume of the solid), and that reduces the rate at which the material in that corner will cool relative to elsewhere in the print, and also changes how the material can deform if and when it does cool unevenly. Uneven cooling of fine features, including sharp corners, is what causes warpage, as these areas typically cool faster than others. The curve also helps with bed adhesion, as the more rounded corner is more easily kept down on the bed by the various forces within the object and in the bed adhesion. Sharper corners depend more on the surface area under the corner itself to keep the piece properly stuck to the bed.

\n\n

However, it's not a panacea. It's not always possible, for one; it's usually an option when CADing a \"green field\" design for a household item or other standalone product, but if you're making a replacement part for an existing device, or printing a figurine or other detailed model, you usually have to take the curves (and edges) that design gives you. Also, if you radius an outer edge of a hollow shape, but don't radius the inner corner to keep the material thickness constant, you'll end up with similar cooling problems as the apex of that edge cools fastest.

\n\n

Strategies for avoiding warping are along a couple common lines, but exactly how you implement that strategy depends on the material and on the printer. One overarching strategy is to increase print surface adhesion; the stronger the part sticks to the bed everywhere on its first layer, the harder it will be for corners to lift. Exact techniques depend on the print material, but many of them, like blue painter's tape, Elmer's glue and hairspray, work well for several filament types. Adjusting the printed shape, to print a brim around the shape's base or even a raft under the entire object, are also common anti-warping strategies. The tradeoff is that the stronger the bed adhesion, the harder the part will be to remove from the build surface when complete, which depending on your print bed construction can cause more problems than the warping.

\n\n

Blue tape is nice because it's cheap to buy and easy to apply to the bed surface, both of which mean you don't have to worry too much about damaging the tape as you remove the part, just strip the tape and put down another layer. But, it doesn't work for all filament materials; the only thing I've found that really works well for ABS, for instance, is kapton tape, which is significantly more expensive and takes a lot of effort to lay down a wrinkle-free, bubble-free layer over the entire print bed for a large part (especially on my MakerBot, which doesn't have a removable build surface on it, so I'm laying this tape down in cramped quarters within the printer enclosure). A glass surface painted with ABS slurry is an option I've not yet had opportunity to try for ABS printing, but plenty of people swear by it.

\n\n

Cooling, especially uneven cooling rates, are another major contributor, but again, exactly how you deal with cooling depends on the material. Most plastics, especially PLA, tend to work best when you keep things as cool as possible; the coolest extruder temp that reliably feeds fil, the coolest bed temp that reliably sticks, part cooling fan turned up, and even a standing or box fan blowing through the entire work area to put as much air over the part as possible. What this does is to get the PLA down onto the print surface as a hot \"putty\", then immediately \"freeze\" that bead of plastic as a solid as the heat's removed, so the bead can't shrink as it cools more gradually.

\n\n

Now, with ABS, this cooling strategy doesn't work, and in fact it's the worst thing you can do to an ABS print. The material is much more elastic than PLA, which is quite brittle, and has no true melting temperature, with a very hot glass transition temp. So, as it's laid down, the extruded beads quickly put the part under elastic tension as it cools. Shrinkage is a fact of life with ABS; the only thing you can do is to control the shrinkage by controlling the part cooling, so the part cools and shrinks evenly. ABS calls for a hot print bed for good adhesion, which will keep the first few layers warmer longer, but higher layers of the print will be further from that heat source, so if there's air moving over or within the work volume, these higher layers will cool more rapidly, at which point it really doesn't matter how good your surface adhesion is (I've seen prints split halfway up the model to relieve the tension by warping). So for ABS, a heat enclosure is pretty much a must. My MakerBot is built that way (in fact it's designed for ABS printing and works better with ABS than with PLA, which is supposed to be the more user-friendly material of the two), but most of your open-gantry RepRap-type designs will need something built around it, usually with a separate heating element to heat the work volume more than the bed is capable of doing.

\n" }, { "Id": "11667", "CreationDate": "2020-01-03T21:08:53.273", "Body": "

I'm using a Prusa i3 MK3S printer. After ~8 months of printing PLA, PET-G, ABS I decided to buy some HIPS and print something with it. I cannot print >1 filaments at once, so I'm not using it as a support for ABS, I want to create some high durability working models, like gears, robot parts etc.

\n\n

While the quality of my models is perfect, unfortunately their strength is disappointingly low. They easily undergo plastic deformation or break. I've tried lots of settings, some yielding better or worse results, but the problem is present regardless.

\n\n

My settings:

\n\n\n\n

I've read lots of tutorials regarding HIPS printing and they did not give me the answer to my question...

\n\n

Am I doing something wrong or is it normal for High Impact PolyStyrene? At this point, the High Impact Polystyrene seems less High Impact than generic PLA.

\n", "Title": "Very low strength of HIPS prints - Why?", "Tags": "|filament|3d-models|", "Answer": "

I've been using HIPS for nearly 2 years now to print the housings of EDF units for RC planes. They come out light and strong but have poor layer adhesion, especially on thin-wall parts.

\n

Normally I print at with a 260\u00a0\u00b0C nozzle and 100\u00a0\u00b0C bed but will try higher temperature when I upgrade my Ender 5 to a 32-bit board. Thin cyanoacrylate painted on the surface solves most issues. Sometimes I use a solution of ASA dissolved in acetone to paint the surface and bind layers together.

\n

The EDF rotors I print from Apollo X which is a modified ASA that is easy to print but not soluble in acetone.

\n" }, { "Id": "11673", "CreationDate": "2020-01-04T12:28:15.630", "Body": "

I'm having a problem getting a clean first layer on an Ender 3 with BLTouch auto bed leveling. Thickness seems to fluctuate all over the bed, but in a consistent (repeatable) way. Here's my attempt to print a single layer 5 square bed calibration test:

\n\n

\"Full\nI stopped the print midway through filling the first square, but you get the idea. Lines go from too low so no filament comes out to too high.

\n\n

I printed this several (many) times with slight settings tweaks and it looks pretty much the same every time; the ups and downs aren't random. For example, the center square always is always too low on the left and too high on the right:

\n\n

\"Center

\n\n

The printer is a SainSmart Ender 3 Pro with a BLTouch V3.1 and Creality glass bed, otherwise stock. I flashed a bootloader and Marlin 2.0 using the instructions and pre-compiled firmware from 3D Printing Canada. I'm using the glass bed upside-down on the plain glass side (no coating).

\n\n

I pre-heated and leveled the four corners manually using the paper method. I auto-homed and then lowered the hot end until it would just catch a piece of paper and used that height to set the Z offset using M851 and saved it with M500. It's currently set at -2.80.

\n\n

I added G29 to GCode start in Cura, and it does a 3x3 probe before the print starts. Here's the output when I run M420 V:

\n\n
Bilinear Leveling Grid:\n      0      1      2\n 0 -0.207 +0.172 +0.162\n 1 -0.100 -0.160 +0.220\n 2 -0.118 +0.215 +0.295\n
\n\n

Here's what it looks like in the Bed Visualizer plug-in in Octoprint:

\n\n

\"Bed

\n\n

If I understand this right (dubious) it's showing that the glass is lower toward the front and left, highest at back-right. But it's only 0.4mm from the lowest to highest points. And the whole point of mesh leveling is to compensate for this anyway.

\n\n

At Paulster's suggestion I turned off mesh leveling using M420 S0, leveled manually, and printed again. The result is pretty similar (note that this time I let it run all the way through):

\n\n

\"Second

\n\n

Where should I start looking to diagnose this problem?

\n\n

Update

\n\n

I noticed my X-axis belt was a bit loose, so I tightened it up. It seems to have helped with the odd Z slanting. My test print is still not great though, so this may not be the whole problem. Also I've never seen this effect listed as one caused by loose belts, so it's dubious as the cause. Here's the current test print after tightening the belt:

\n\n

\"Updated\n\"Closeup

\n\n

It's flatter, but I'm still getting (I think) under-extrusion and some odd wobbles at the corners.

\n", "Title": "Ender 3 with BLTouch prints slanted lines", "Tags": "|creality-ender-3|bed-leveling|bltouch|", "Answer": "

This turned out to be a problem with the tightness of the rollers at the left and right sides of the X-axis gantry (that roll up and down the Z rails).

\n

Z-axis motion is driven by a single stepper motor on the left side, so the rollers have to be just the right amount of tight to keep the right side in sync. If the right side is tighter or looser than the left then it lags behind, which gives the gantry a slight slant which changes as it goes up and down.

\n

If the gantry is changing pitch throughout the print, no amount of bed leveling will help. Even auto-leveling is worthless, because the readings the BLTouch takes become immediately out of sync with the gantry as soon as it moves again.

\n

The solution is to adjust the eccentric nuts in the rollers on the left and right. The best description I could find is that they need to be just tight enough that there's some resistance if you roll the top wheel with your finger, but loose enough that you can roll it without forcing the gantry up and down.

\n

I ended up putting a magnetic digital level on top of the gantry rail so I could see exactly how much its incline changed. Send gcode to slide it up and down, then adjust the eccentric nuts a little bit, then repeat. Once I got it so the level didn't change, I re-leveled the bed and printed a beautiful first layer.

\n

That was almost a year ago and it's been working ever since. I've had to re-adjust the eccentric nuts periodically when things start to get off, probably due to thermal expansion when the weather changes.

\n" }, { "Id": "11679", "CreationDate": "2020-01-05T03:28:35.993", "Body": "

I'm getting wavy lines on the first layer only in both the x and y direction identically. The first layer is 0.4 mm with a 0.4 mm tip. The other layers are 0.2 mm. I've tried changing the Z offset all the way from -1.2 to 0.5 mm. I've tried changing the hot end leveling the heated bed. None of these changes affected the wavy lines. The waves have about a 1 mm period. The printer is a German RipRap. The material is ABS. The heated bed is 110 °C. I've tried the hot end at 220 °C and 240 °C. So far, nothing has changed the waves.

\n\n

\"enter

\n", "Title": "Wavy lines on 1st layer only", "Tags": "|troubleshooting|extrusion|", "Answer": "

You have a too thick layer: to get straight lines, the plastic has to be squished together to some degree as it is pushed out of the nozzle. The result is a shape similar to a circle with the top and bottom cut. This works well until your layer thickness is more than 3 quarters of the nozzle diameter - above the \"squishing\" is practically nonexistent, and if you go above the nozzle diameter, there is almost no way to get the desired thickness out of the nozzle at all.

\n\n

To aid in depositing the layers, it is also advised to demand a line width that is about 10\u00a0% larger than the nozzle diameter. As illustration, this is roughly what 0.4\u00a0mm extrusion width with 0.4\u00a0mm layer height (blue) and 0.45\u00a0mm extrusion width with 0.3\u00a0mm extrusion height (yellow) look like:

\n\n

\"enter\"enter\"enter\"enter

\n" }, { "Id": "11690", "CreationDate": "2020-01-06T09:33:33.947", "Body": "

I am designing a part with a material which can bear up to a certain stress.

\n\n

To keep the stress within the design limits, I need to ensure a certain surface of solid material per layer, in other words a minimum amount of extruded filament per layer and a smoothly changing infill ratio so that the infill can transmit the load efficiently to the neighbouring layers.

\n\n

The part has an irregular shape and I cannot simply increase the infill ratio for the whole object because the part is big and it would cost more time and filament. I also cannot build straight pillars of solid material inside the part because there are no regions which are suitable for a continuous pillar.

\n\n

How can I calculate and apply a smoothly changing infill ratio or in general how to ensure that each layer is made of at least a certain amount of material?

\n\n

I use Prusa Slicer for slicing and Fusion 360 for the design.

\n\n

Related questions about variable infill, which however don't answer my question because I need to specify the amount of material and because I need a smoothly changing infill ratio:

\n\n

Different infill in the same part

\n\n

slic3r: Can I vary the infill percentage for different heights of my model?

\n", "Title": "How to slice a model to ensure at least a certain amount of filament per layer?", "Tags": "|slicing|infill|", "Answer": "

I think you have a major XY problem. The amount of material per layer is not what determines the part strength. Unless additional material is placed in a manner that reinforces against the stress you're designing for, it's just wasted. Moreover, infill in particular is not terribly useful this way, as infill has to rest upon the support of existing infill below it. If you suddenly increase infill density at a particular layer, it will not provide any significant additional strength because the added lines will be unsupported and will not bond strongly to anything. Even the next layer above them, and the next after that for quite a few layers, will not bond well because the unsupported lines can just bend downward when the nozzle goes over them, rather than providing a surface for the newly deposited material to press firmly against and bond to.

\n\n

Generally, infill is not your main source of part strength anyway. I would start out (especially if you can test; if this is a one-off thing, the material cost is not going to be an issue anyway and just go with overkill) by increasing the number of wall lines (wall thickness). Walls generally provide the most strength, and the amount of material used will be proportional to cross-sectional perimeter rather than area, which typically will vary \"linearly\" rather than \"quadratically\" (I use these terms loosely since I don't know right off how to make it rigorous - what the independent variable should be thought of as being). My usual default (and I believe Cura's) is 2 walls; I would expect 4-5 to be very strong, and as long as you keep at least 15-20% infill, probably stronger than what you'd get by any increase in infill percent.

\n" }, { "Id": "11691", "CreationDate": "2020-01-06T11:46:21.757", "Body": "

I want to scan a few ceramic items that are shiny and have multiple colors and found that using an artec 3d Spider that doesn't really work due to the shininess and thus want to use a matting spray or sth to be able to get proper results. Ideally sth without color that can be used on the ceramic items and allows to obtain proper texture results for the 3d model too. Any ideas?

\n", "Title": "3d scan matting spray", "Tags": "|3d-models|", "Answer": "

There are many variations of a product generically known as chalk spray. Some of them should be considered permanent or semi-permanent, while the Montana brand product explicitly states \"temporary\" in the description. There is a caveat in that it has to be a non-porous surface to be considered temporary.

\n\n

The product is primarily directed at outside/sidewalk art which can be washed away or will wear away under normal traffic. As such, it is suited for matting shiny surfaces for scanning purposes. White is the best color for scanning as it will accept light and shadows better than the other choices.

\n\n

I have a can of white and can attest that it will wash off. Some of the product will wipe off dry but the small crevices require washing, which should be considered when spraying your item.

\n\n

\"montana

\n\n

The product is available at many online outlets and may be found at local art supply retailers.

\n" }, { "Id": "11692", "CreationDate": "2020-01-06T13:42:57.847", "Body": "

I keep having this issue on prints where the first layer has stringy lines that don't stick together.

\n\n

I've tried the following things

\n\n\n\n

My print settings as shown in this pic are the following

\n\n\n\n

Printer

\n\n\n\n

Let me know if you think there's other relevant settings to this issue. Any help is greatly appreciated, I've been trying to fix this for days. I've tried different models, large and small, but have the same issue no matter the size

\n\n

\"enter

\n", "Title": "Filament spreading apart and stringy on first layer", "Tags": "|print-quality|marlin|ultimaker-cura|prusa-i3|pla|", "Answer": "

I would try two things you didn't mention; a bed leveling (this looks a little close) and an e-stepper calibration. The layer looks thin and under-extruded, and two common culprits are the bed height being too close (\"oversquishing\" the first layer which reduces flow rate and adhesion), and the e-stepper not feeding as much filament as the slicer's asking for (giving you thinner lines than the slicer expected, so they don't adhere to each other or the substrate).

\n\n

I also see problems with that kapton layer. Totally understandable, the stuff is a major pain to lay down bubble-free, however it's also absolutely necessary to do so to avoid first layer issues. That's not your only problem here but it'll keep being a pain after you have filament feeding sorted out.

\n\n

Lastly, I'm seeing slicer error; the floor fill is being laid down over the top of the shell layers on the right side of your image. Remember that the wealth of slicer variables in Cura regarding print speeds, flow rate, filament diameter etc ultimately boil down to a single G1 command per line: \"move from here to here at this speed extruding this length of filament\". So, if the slicer is forgetting where it drew the outline by the time it scripts the floor fill, it's possible it lost the plot on extrusion calculations as well. Alternately, the printer could be the one that lost the plot, either losing track of the steppers or incorrectly interpreting the G-code. Usually you just need to power-cycle the printer, close and reopen Cura and re-slice. Also, if you're printing with a USB cable, try switching to an SD card; there's less to go wrong in the communication between slicer and printer if the slicer tells the printer everything it should do up front.

\n\n

I'd look into each of these, then try to print a calibration shape, like a 20mm XYZ cube, before going back to the print you had on the plate before.

\n" }, { "Id": "11703", "CreationDate": "2020-01-06T20:36:25.940", "Body": "

I'm trying to replace my hotend with the E3D Hemera direct kit. I got to the final step of hot-tightening the hot side and managed to snap my heatbreak. The part that screws into the heat-sink is stuck, though I was able to remove it from the heater block section.

\n\n

Below is a picture of the heatbreak. The red square shows what it in the heat-sink. It's mostly thread (with thermal compound) but a bit of the unthreaded metal is sticking out (above the disk shown).

\n\n

I've tried removing it with a jeweler's drill (which is how I removed the heater block portion, but I cannot get a good grip on the long threaded piece. I've also tried pliers and rounded needle-nose pliers. Neither worked. I couldn't find my regular needle-nose and will go to the store to try that. Are there other options to break this free or am I stuck with buying a replacement heat-sink? I already ordered a replacement, but it'd be nice to have a backup in case something else goes awry (as has been the case with this modification; this is snafu number 4).

\n\n

\"Heatbreak

\n", "Title": "Remove broken heatbreak", "Tags": "|hotend|repair|e3d-hemera|", "Answer": "

Broken screws or screws with damaged screw heads are typically removed with screw extractors. Since there is already a hole you could \"drill\" (unscrew) it out with a screw extractor, as it is counter clockwise, it is about creating enough friction to unscrew it from the threads. Beware that it is not drilling and spalling material, turn slowly.

\n\n

\"enter

\n\n

Alternatively, a grip vice pliers may work if there is enough \"meat\" to clamp on.

\n\n

\"enter

\n" }, { "Id": "11707", "CreationDate": "2020-01-07T01:27:02.153", "Body": "

I have printed two objects with my new 3D printer (Anycubic Mega S) and everytime, my prints are stuck to my bed (sort of glued to it). I cannot remove them by hand. I have tried waiting until it cools off, but the only thing that works is scraping really hard the bed with the spatula.

\n\n

I'm scared that if I have to do that for my next prints, I will break the bed (maybe peel off the element that keeps the plastic and the bed glued together while printing).

\n\n

What is the safest way to remove a print from the bed ?

\n", "Title": "How can I remove my print from the bed safely", "Tags": "|heated-bed|print-material|anycubic-i3-mega|", "Answer": "

I have had good luck using dental floss. If you can get it under the edge of the print, then you can pull it all the way through and prints come off easily.

\n" }, { "Id": "11722", "CreationDate": "2020-01-08T14:06:19.880", "Body": "

I understand that this is probably more of an electronics question, but was hoping that someone with experience of using an Anet A6 in the UK (or a country outside of the US/China) may be able to help... or alternatively, someone knowledgeable in electronics!

\n\n

I recently bought and assembled an Anet A6. I am based in the UK. On the power supply transformer of the Anet A6, there is a switch that allows you to select the input voltage from the mains. There are two options, 100 V or 220 V.

\n\n

When I turn my Anet A6 printer on, nothing happens... I have triple checked all connections and there doesn't seem to be anything wrongly connected or loose.

\n\n

I am wondering if the reason it is not working is because in the UK we use a different mains voltage 230 V (I think) and a different frequency 50 Hz (I think) to the US and China (which I assume the printer was built to accommodate)?... I am not 100 % sure on this, just a guess, I am far from an electronics expert.

\n\n

I don't have a multimeter to test if there is voltage flowing (not that I would even know how to test it lol).\nIs it likely that this difference between voltage/freq is the reason that it is not working? If so, is there anyway to fix this? I would prefer to buy something (some sort of converter) than tinker with the electronics, as I have no experience in electronics and live in a rented flat, which I really don't want to burn down (not that I would if i owned it).

\n\n

Any help is massively appreciated, thanks in advance!

\n\n
\n\n

Update

\n\n

I have done what @Oscar suggested and also bought a multimeter to test the circuitry. I plugged my Anet A6 into the mains power supply and turned it on, but still nothing happens... the LED doesn't light up, not does the LCD screen turn on.

\n\n

I tested the voltage of the power supply whilst it was turned on across connections 6 and 8 in the video below (taken from the assembly instructions video, 12 mins 46 seconds):

\n\n

Assembly Instructions Video, 12:46

\n\n

The 6 and 8 connections correspond to the output from the transformer (ie the connections that would be connected to the mainboard). There was no voltage reading at all when I measured it here with the multimeter. Does this indicate that there is a problem with the transformer/power supply, or is this expected? Or am I testing in the wrong place and there is a better place to test when the printer is on to determine what the problem might be?

\n", "Title": "Using an Anet A6 3D printer in the UK... (mains voltage issue!)", "Tags": "|anet-a8|printer-building|anet-a6|power-supply|", "Answer": "

@Oscar was correct, so long as the switch is set at 220 V, the printer will turn on. I am adding this answer to help anyone else who has a similar problem.

\n\n

I strongly recommend that you buy a multimeter if you have any power supply issues, as this helped me to figure out what was wrong.

\n\n

There were three issues that needed to be rectified before my printer would turn on. The first was that I had bought a fairly cheap EU to UK plug converter from my local supermarket. This was mistake number one, as the quality was low and there was no ground pin for the power supply (which is dangerous). I plugged the EU plug into my converter, and then the converter into the UK mains socket, and it would not turn on. By using my multimeter I was able to figure out that the converter was a piece of rubbish. With the plug still plugged into the converter, but the converter removed from the mains, I touched my multimeter cable, whilst in continuity mode, on one of the three pins on the UK side of the converter (the bit that goes into the wall), and the other cable onto one of the terminals that I had connected the power cable to the power supply with. I touched each terminal in sequence to see if it was electrically connected to the pin on the converter. I repeated this in sequence and identified that the live pin on the converter was not connected, and so no current could flow when plugged into mains. I immediately defenestrated the converter. Here is the replacement that I bought:

\n\n

EU to UK converter

\n\n

The next issue was that the power supply cable was wired up incorrectly (or at least unintuitively). In continuity mode again, I touched one multimeter cable to one pin on the EU plug of my power supply, and the other to one of the terminals (which were connected to the wiring of my power cable.) I discovered that the live and neutral wires of the power cable were wired the wrong way round on the plug (in order for it to be used in a UK converter, not sure what the wiring convention is on mainland EU). In the UK, the right hole in the mains socket is live, the left is neutral, and the top one is ground. On the EU plug I had been provided with, the left pin was live and the right pin was neutral. If I were to plug this in to the new converter in the normal orientation (cable coming out the bottom side), the live and neutral would have been connected incorrectly (plug-live -> converter-neutral, plug-neutral -> converter-live). Hence, in order for the pins to be situated correctly in the converter, and subsequently in the mains socket, the EU plug had to be plugged into the converter upside down (plug-live -> converter-live, plug-neutral -> converter-neutral).

\n\n

Finally, the official instructions for the wiring of the Anet A6 are wrong. If you follow the youtube video that they provide, you will see that the positive terminal is closest to the bottom of the diagram (on the thermistor/endstop side of the board). Mainboard wiring instructions. THIS IS INCORRECT. Check your board, as the polarity will be printed onto the board. If you wire up the mainboard according to the instructions, your V+ wire will go to the negative terminal of the mainboard and, your positive terminal will go to the COM wire of the power supply. This is wrong, as the positive terminal should be connected to the V+ wire, and the negative to COM.

\n\n

Hopefully this helps save someone some frustration and time!

\n" }, { "Id": "11725", "CreationDate": "2020-01-08T18:07:39.557", "Body": "

I have made a temperature \u2a09 fan speed tower which needed 3x9-1 ChangeAtZ post processing scripts and it took me quite much time to configure them all (and check it twice). Is there a way to save this, so that I wouldn't need to make them all again if something went wrong and I needed to start over or if I wanted to do something similar again ?

\n", "Title": "How to save the post processing scripts configuration with Cura?", "Tags": "|ultimaker-cura|", "Answer": "

Thanks to fred_dot_u, I found the solution : \nthe post processing scripts configuration is saved... in the printer profile. So, you cannot directly duplicate it, but re-installing a second occurrence of the same printer will clear the post processing scripts list, you can get them back by selecting the modified one...

\n" }, { "Id": "11738", "CreationDate": "2020-01-11T03:21:19.340", "Body": "

I've recently added a second hotend and extruder assembly to my 3D printer and I've made all the necessary changes in the firmware. I've defined the temp sensor for hotend 2, all the pins for heat and temp as well as defined extruders as 2 instead of 1.

\n\n

The problem is that the printer display in the motion menu is showing

\n\n\n\n

\"Extruder\" and \"Extruder E1\" both control the primary extruder and \"Extruder 2\" controls the second extruder.

\n\n

Any tips, ideas, suggestions?

\n", "Title": "Marlin firmware question for dual extruder", "Tags": "|marlin|extruder|", "Answer": "

There is nothing to worry about, this is a feature not a bug.

\n\n

Extruder refers to the active extruder, the loaded/active tool. Based on the active extruder the Extruder controls either your Extruder E1 (this is known in the firmware as Extruder E0!) or your Extruder E2 (the Extruder E1 from the firmware).

\n" }, { "Id": "11744", "CreationDate": "2020-01-11T20:48:08.260", "Body": "

My printer came with a knock off BuildTak like build surface. What can I use to clean it? I don't want to damage it.

\n", "Title": "What should I use to clean buildtak(knock off)?", "Tags": "|build-plate|maintenance|cleaning|", "Answer": "

For BuildTak, my first recommendation is isopropyl alcohol (aka rubbing alcohol or isopropanol). Readily available just about anywhere basic medical supplies are sold, it cuts though light oils and greases, and dissolves most common build surface adhesives like hairspray or Elmer's glue stick. In fact I often give my Ender's bed a light spray of isopropyl after an initial layer of hairspray, as the alcohol thins the hairspray a bit so I can take a plastic scraper and spread it around a little more evenly, before it sets to a very tacky initial surface that holds well.

\n\n

Depending on where you live, denatured alcohol aka methyated spirits might be easier to get, and this stuff also works well as a cleaner and degreaser. The methanol does a little better job at actually removing old build adhesive instead of just \"reactivating\" it, and it also seems to do a little better than isopropyl at releasing and capturing old print residues that didn't make it off the surface with the rest of the print.

\n\n

Be aware that both the liquid and the vapors of both of these are toxic, denatured alcohol a little more so than isopropanol. You should have plenty of ventilation through your workspace while using either one.

\n\n

For a deep clean, especially with a removable build surface, it's hard to beat a mild dish detergent and water, and a scrub with a Teflon-safe Scotch-Brite sponge (they're usually blue). A clean rinse, pat dry with a towel, then give it a spray of isopropanol to finish drying it and it's ready to go. This is more of a hassle and I don't recommend it as a quick between-prints cleaning, but it can bring a well-used bed surface right back to life when more volatile cleaning agents won't do it.

\n\n

Whether it's worth the frustration getting the plate properly rinsed (soap, being slippery, is not going to aid adhesion), versus just scraping the surface back off and putting a new one on, depends on how easy it is to bring the bed surface to the sink. My Ender's magnetic surface makes this much easier than a simple adhesive-backed surface right on the aluminum. By the same token it also makes the surface easier to replace, and they're not that expensive (so I highly recommend them if you like BuildTak), but \\$12 is \\$12, compared to 5 minutes in the sink with some Palmolive.

\n" }, { "Id": "11747", "CreationDate": "2020-01-12T05:09:11.163", "Body": "

I want to create two piece labels for storage containers. The main piece would be the \u201cbadge\u201d which would have text cut out of it (e.g. \u201cPaint\u201d, \u201cElectrical\u201d etc.). The second piece would be a positive of the text which is would be in a different colour, and would fit inside the cut out on the badge. Because of the tolerance of 3D printers, I need to make the insert slightly smaller than the cut out. Initially, I thought I could just scale the insert but that would affect the letter spacing. Then I thought it would work if I could somehow taper the letters so they are slightly smaller at the top than the bottom. So my question is, how I do that. I did the original in OpenScad but I would try Fusion360 if that\u2019s a better solution. Any and all suggestions are welcome. Thanks.

\n", "Title": "Creating a two part label for storage boxes", "Tags": "|openscad|fusion360|", "Answer": "

You will need to run test prints to see how your printer behaves with your choice of filaments. Then you'll know how much narrower the letter-lines (not the letter dimensions) need to be to fit.

\n\n

The problem with that is one of making letters with narrow solid parts but without \"shrinking\" the open parts so that they fit into the badge receptacle regions. You can't just reduce the dimensions of the letters themselves.

\n\n

Other than what Trish's answer suggests, I'd recommend either using a very small nozzle diameter and setting a narrow linewidth to reduce the \"over-dimensioning\" effects. Even better would be to identify the letters separately from the badge but keep them as a single STL and use a two-color printer (twin extruders, etc) to make the badge in one run.

\n" }, { "Id": "11750", "CreationDate": "2020-01-12T13:51:21.467", "Body": "

I currently only have access to an old, 32 bit OS and need to slice a couple things. While there are a lot of slicers around, most popular ones, for example Ultimaker Cura 4, need to run on a 64 bit operation system.

\n\n

What somewhat recent (late 2018, 2019) options offering common abilities are there that run on older computers?

\n", "Title": "Slicer for 32 Bit Operating systems?", "Tags": "|slicing|", "Answer": "

You can try using C6 Slicer software, which is a Cloud-based one, no need of an application version. I think they are still in development phase, but still does give some good prints.

\n" }, { "Id": "11756", "CreationDate": "2020-01-13T00:32:15.113", "Body": "

I have recently bought an Anycubic i3 Mega S 3D printer. I'm using the stock nozzle (.4 mm) and I print with the plastic it came with (ABS). I'm using Ultimaker 4.4.1.

\n\n

When I print minis, I'm always having issues removing the support the slicer software adds to print the object. Most of the time, I end up breaking parts of my minis. I'm using the tools given with the box.

\n\n

How do I make this process easier ? I'm mostly interested in the settings linked with the support generation, but if there are other ways, I would like to know.

\n\n

Note that the minis are for DnD characters, thus printed objects are often not flat and contain multiple curves, which then creates support that are harder to remove, for example, in between the holder of the mini (the cylinder that holds the piece in a stand up position) and the legs.

\n\n

I usually print models found online for free, but from time to time I buy some models which contain a lot of details than the free counter parts. I don't mind loosing some of those details, as long as the mini is well printed (in short, I don't mind if the sheath of the sword falls off, but I do care if the arm falls off)

\n", "Title": "How to ease the process of removing support for miniatures", "Tags": "|ultimaker-cura|support-structures|anycubic-i3-mega|", "Answer": "

Let's look at the factors that can help us get support parts printed and removed:

\n\n

access

\n

Support structure has to be accessed to be removed. Tree support could help in this. An alternative would be soluable support, which is still accessible if buried deep in a part - as the solvent would be all that needs to access the part.

\n

dimensions/size

\n

Support structure needs to have some crossection to be printed at all. if it gets too small, it will fail to print and fail in supporting. You might alter the support angle to support even 40\u00b0 or 30\u00b0 surfaces to the vertical to forcefully increase the supported area and thus dimension of the support structure.

\n

bonding (to bed)

\n

To make sure the part and its supports don't shift from one another, it is best to print with a brim that makes sure all support trees and the base of the model all share a combined first layer.

\n

debonding

\n

Debonding means, we need to remove the support structure from the printed part. Ultimaker Cura allows to define a gap from support structure to the part, usually 2 layers. With these settings removing supports can be as easy as removing the brim and careful cleanup.

\n

Other things

\n

No support?!

\n

Some miniatures have lots of unsupported areas. For example this knight needs support at the hilt of the sword, the arm and the helmet overhang. This dwarf might need support at his dagger in the back and on the left arm (and was resin printed).

\n

However, with the proper design, there is sometimes no support needed. For example this pirate has no areas that need support for overhangs or free hanging areas.

\n

Printing speed

\n

Printing miniatures and intricate parts is hard. I have set up an older, small TronXY with a 0.2\u00a0mm nozzle just to print small and detailed items, usually engineering models. I print them slower than normal (ca 40\u00a0mm/s, 20\u00a0mm for the outer shell).

\n" }, { "Id": "11771", "CreationDate": "2020-01-14T13:02:07.870", "Body": "

I currently have a single extruder (direct drive) Tronxy X8.

\n\n

I am thinking about adding a second extruder, that I would use only occasionally. I really like the characteristics of the direct drive extruder, so I would not want to switch over to dual Bowden. Also adding the weight of a full second direct drive extruder for only occasional use does not seem useful.

\n\n

Is it possible to use a direct drive extruder as main extruder and a (possibly detachable) bowden extruder as secondary extruder?

\n\n

Is there anything in particular that I need to watch out for when mixing extruder types?

\n", "Title": "Adding a Bowden extruder to a direct drive setup", "Tags": "|extruder|bowden|", "Answer": "

I'm an amateur and I used dual extruder printers but never built any.

\n\n

It's probably a good idea, take care about the purge, retractions distances that may vary. On the filament feed side there shouldn't be interference from the different flexibility. Still it maybe good to place one reel on one side and the other on the opposite.

\n\n

Now the most complex thing is to make both nozzle not interfere. A technique is to rotate by few (like 5\u00b0) the dual setup to give few millimeter more to one of the two nozzles. If both heads are at the same height, you'll need to keep them hot, and avoid the passive one to leak. It would be good also (but not mandatory) to have a purge routine.

\n" }, { "Id": "11775", "CreationDate": "2020-01-15T06:05:27.030", "Body": "

I am wondering if it is possible to print with a 3D printer from an SD card and connect to a serial host at the same time. I want to use the Marlin M118 serial print command to trigger custom functions running on a Raspberry Pi. The RPi would be connected to the printer over serial but would not be sending G-code. Is this possible?

\n\n

My printer is the Monoprice Mini Delta. It runs a Marlin based firmware and I can install standard Marlin on it if necessary.

\n", "Title": "Print from SD and connect to serial host at the same time", "Tags": "|marlin|electronics|repetier|serial-connection|", "Answer": "

This can be done, but you need to have the right order of operations. Octoprint relies on such a setup.

\n\n\n\n

Note that you have to make sure not to dis- and reconnect the raspberry during print, as that might trigger a reboot of the printer!

\n" }, { "Id": "11778", "CreationDate": "2020-01-15T16:59:27.450", "Body": "

I was just informed via a comment that the TH3D Unified firmware a version of Marlin that's no longer updated and considered obsolete (1.9.X in this case) and that, since I'm flashing my firmware to fix my default e-step settings, I might as well flash a non-obsolete firmware.

\n\n

However, something in the back of my mind is telling me that I can't use Marlin 2.0 because of some hardware limitation.

\n\n

I'm using the Creality CR-10S printer (with the Creality 2.0 board, I believe) which is an 8-bit CPU. What should I look out for before upgrading to Marlin 2.0?

\n\n

Marlin's Install website suggests that 8-bit AVR printers can use it (flashing via Arduino IDE). SO I guess I'm double checking before I do something that could potentially brick my printer.

\n", "Title": "What precautions to take when flashing Marlin 2.0?", "Tags": "|marlin|creality-cr-10|", "Answer": "

It wasn't advised to use the version 2.x because it was in development for 32-bit micro processors. Now that it has been released as the official version, you can use it for 8-bit micro processors.

\n\n

But, it totally depends on the amount of options in Marlin you activate (bed leveling, advanced menu, M5xx, etc.). Luckily you can see how large the installation is after you have built it e.g. in PlatformIO. Also, in the configuration files frequently is mentioned how much extra storage activating an option costs (search for PROGMEM in the Marlin sources). Unless you want all options active, you'll be fine. I'm running it on an AVR (MEGA2560) for a CoreXY with bed leveling and some more options; it runs fine.

\n" }, { "Id": "11782", "CreationDate": "2020-01-16T05:55:41.613", "Body": "

Say I wanted to print a plastic credit card like shape (like these), but with a QR code engraved. How could I do that for cheap? You can buy an \"ID card printer\" for $1,000-1,500 on Amazon, but that's way too much for printing one or two cards. Maybe down the road this would be a good option, but I kind of like the option of 3D printing the card from scratch, so the QR code bleeds halfway or all the way through the card, rather than just being printed on the surface. Is this possible for cheap? Maybe like this but not as fancy. Mainly (I'm new to all this) I am wondering what machine would accomplish this for low price yet good quality, and what other equipment I would need.

\n\n

Basically, what printer is best for this type of task?

\n", "Title": "How to 3D print an ID card", "Tags": "|diy-3d-printer|2d|", "Answer": "

One option to create the tag you require can be accomplished with a single extruder and a bit of manual intervention.

\n\n

I've performed the sequence of steps from a Thingiverse creation that resulted in a box lid for a gift card box. It's a single layer of multiple colors placed on the print bed, then consolidated with a backing layer.

\n\n

For your card objective, the single layer provides the contrast and you can determine easily how thick you wish the remaining portion to print.

\n\n

\"snowman

\n\n

The detail level is reasonably good, limited to the printer nozzle and specifications. A glass bed or similarly smooth surface bed will provide optimum results. As 0scar suggests, a 3D printer in the US$100-150 will likely suffice, but check reviews and forums for troublesome products of such a low price.

\n" }, { "Id": "11787", "CreationDate": "2020-01-17T02:32:45.040", "Body": "

I've been trying to print anything for the past few days. Every time I go to print something the printer stops pushing plastic out and usually the motor makes a knocking sound.

\n\n

My original thought was that my E-steps were off (because they were) but I solved that and I still have a problem. Then I noticed that I was struggling with getting the PLA filament to feed into the nozzle. I assumed it was a clogged nozzle and possibly heat creep (because I was a dummy and used my old, bowden retraction settings on my new, direct extruder). I cleared out the clog and I'm able to feed plastic through after disengaging the idler arm. Though, I do struggle getting the filament into the nozzle. Maybe it's due to the Hemera's tight tolerances or due to a misalignment in the hot end, I'm not sure yet. Once I do get the plastic in, it quickly and easily pushes through and spits out of the nozzle so I think that my nozzle isn't clogged. I did do a cold pull the other day.

\n\n

But I've noticed that my extruder is making a knocking sound, as if someone was gently rapping on my chamber door. I also see that the gears move one step back and forth. It looks to me that the motor is stuck or frozen. I'm pretty sure that it's not skipping steps. The sound happens at varying heights and not just the first layer. The first time I noticed it, it was printing the cabin of Benchy. Z > 1 mm when I finally decided to ask this question. The sound is kind of loud, but that's most likely the resonance in the machine than a symptom, stating it nonetheless.

\n\n

My setup:

\n\n\n\n

I'm just not sure what could be wrong here. I feel like I checked all the steps and I'm just missing something.

\n", "Title": "Extruder motor making a knocking sound, nozzle is not clogged", "Tags": "|troubleshooting|creality-cr-10|e3d-hemera|", "Answer": "

I have had the exact same problem on my Ender 3 Pro. I fixed it by adjusting the amount of pressure on the filament by the motor. I did replace the motor too for other problems.

\n" }, { "Id": "11793", "CreationDate": "2020-01-17T20:02:42.850", "Body": "

I'm trying to print a part with thin walls. I've designed it with wall 1.2 mm thick, so that I should get three 0.4 mm lines. This works just fine for the straight lines, but for the radiused corners, Cura 4.3 insists on trying to print infill. This infill is only added where they are going at a tangent to the curve, so it alternates corners on alternate layers.

\n\n

\"enter

\n\n

Worse, it prints the segments entirely out of order, which adds lots of travel and hence print time.

\n\n

I've tried setting the infill to 100 %, setting the wall thickness directly to 3 rather than the default 2. I've tried reducing the wall thickness by 0.2-0.3 mm. I've tried a few other things too - all to no avail.

\n\n

Some layers it gets right:\n\"enter\nSo why can't it get them all right??

\n\n

I presume that this is probably caused by the fact that the curves are actually a series of straight segments, and so the centre gap between outer 0.4 mm walls will not be exactly 0.4 mm all the way round, but is there any way to just force Cura to print three lines all the way round?

\n\n

Now, I know that this will print OK, but the corners will obviously look messier than they need to, and this is not the first time I've come across this problem. I'd like to get this fixed.

\n\n

I have found a similar question, but it's for an older version of Cura, and the recommended solution of 100 % infill doesn't help.

\n", "Title": "How to slice thin curved wall", "Tags": "|ultimaker-cura|slicing|", "Answer": "

Even though each path, in theory, is concentric around the center point of the arc, the pathing does not always work out that way - especially around corners and radii.

\n\n

While your 1.2mm walls should always allow for three 0.4mm paths, if the slicer rounds down the overall thickness to 1.19mm, it will not detect enough room for three 0.4mm paths - but three 0.39mm paths should still fit, even if the slicer determines the thinnest part to be 1.17 mm.

\n" }, { "Id": "11800", "CreationDate": "2020-01-18T15:59:48.430", "Body": "

My Kossel Mini printer was working well. Recently I made some changes including replacing the nozzle and throat, adding a silicone heater cover. Now it has the problem of first layer adhesion.

\n\n

\"\"\n\"\"

\n\n

The perimeter sticks well but the infill line swells upward in the middle, while start and end has no problem.

\n\n

In my experience the adhesion problem only occurs at corner but never in the middle. So I don't know what configuration needs to be changed to fix it.

\n\n

Delta calibration has been done before the print. PVP glue was used on the bed.

\n", "Title": "FDM printer first layer infill swell upward", "Tags": "|adhesion|glass-bed|", "Answer": "

Had tried below and didn't work:

\n\n\n\n

Printed again and watched carefully, then found that it was pull up because the printed PLA slice shrank. I realized that the problem was related to my change on the fan duct. Before, the fan blowed directly to the tip of nozzle; after changed it blows around.

\n\n

Tried to turn off the fan on first layer, the problem was solved.

\n\n

I suspect that the fan duct change made the extruded material not cooling down immediately. It shrank when the air blew on it. I need further test to verify it.

\n\n

\"\"

\n" }, { "Id": "11809", "CreationDate": "2020-01-19T15:51:16.413", "Body": "

I got an Anycubic Predator last month, and after resolving a few mechanical problems, I was able to get it printing decently well. The only significant modification I've made so far is a set of 8-diode TL Smoothers, and I'm now mostly operating it via Octoprint.

\n\n

However, during the last few prints, I've noticed the temperature dropping midway through the print. It warms up and cools down fine, but for some reason it's not able to sustain the temperature throughout the print.

\n\n

\"temp
\nIn this case, the print started out at the correct temperature (200 \u00b0C), held that temp for around 2 hours, then it dropped to a lower temp (174 \u00b0C). It eventually went back up to the target temp, then dropped again 5 minutes later. I tried manually adjusting it to see if that could fix it, but no luck.

\n\n

After this print completed, I restarted it to show how it is easily able to reach the target temp and hold it at the start of the print:

\n\n

\"temp

\n\n

Any tips on diagnosing and resolving this issue?

\n", "Title": "Hotend doesn't maintain temperature", "Tags": "|troubleshooting|hotend|thermistor|anycubic-predator|", "Answer": "

Based on your picture from Octoprint I can assume that you may have the wrong heater.\nWhy? Because heating hotend to 215 takes quite a lot of time (3 minutes) in your case. If you have an appropriate heater it should take around 1 minute

\n

You need to check the resistance of the heater and then calculate the power based on voltage.

\n

Current = Voltage / measured resistance

\n

Power = Current * voltage

\n

For a good working hotend, you need to have at least a 35\u00a0W to 40\u00a0W heater.

\n

Some shops sell 40\u00a0W heaters but these heaters are for 24\u00a0V systems, so in your case, if you have 12\u00a0V system it may be the case why the temperature drops because the heater will work like it has 10\u00a0W of power

\n

But even if you swap the heater, you need to be sure that your power supply/board will be able to deliver appropriate current without damaging itself - MOSFET/SSR (solid state relay) could be helpful sometimes.

\n

As someone mentioned in a different response it's good to have cooling protection like silicone socks or any other protection material.

\n" }, { "Id": "11827", "CreationDate": "2020-01-21T15:38:33.657", "Body": "

I tried printing a \"improved\" calibration cube and noticed that the printer was laying down dots instead of a solid line. This was right after switching out my extruder motor and tweaking retraction settings. I believe my first setting was 0.3 mm, then I tried 0.1 mm. I didn't get a picture of the first setting, but shown below is when retraction was set to 0.1 mm. I think the print was at layer 3 roughly. The difference is 0.1 mm had a higher frequency of dotting, ie more dots that were closer together.

\n\n

This was only present in the first couple of layers. The dots weren't occurring in the same spot so the picture shows dots overlapping each other making a sort of \"twisted\" or braided look. This also happened on both the x-axis movements and the right side of the y-axis movements. The rest of the cube came out relatively well.

\n\n

Settings

\n\n\n\n

I never saw this issue before on my printer nor have I seen it mentioned in various articles/videos online.

\n\n

\"Dotting

\n", "Title": "Odd dotting when printing a line", "Tags": "|troubleshooting|creality-cr-10|e3d-v6|e3d-hemera|", "Answer": "

It's not about printing settings but only about the bed: the bed is too close to the nozzle.

\n\n

Maybe you adjusted it when it was cold, instead of doing it when it's already heated to the desired temperature and the nozzle is heated at about 180\u00b0C to get close to operating temperature without oozing.

\n\n

Or maybe you used a too thin paper sheet.

\n" }, { "Id": "11828", "CreationDate": "2020-01-21T15:42:18.760", "Body": "

I discovered \"diamond\" nozzles (name of the design, not the use of diamond material), which have multiple filament inputs (designs for 3 or 5) and a single output nozzle, see figure.

\n\n

\"Diamond

\n\n

They look like a very good and compact solution to avoid or significantly reduce purge towers, where filament is wasted until the channel is primed, and to avoid a second hot end, which takes space and which could be oozing when unused, but I could find them only made of brass.

\n\n

Are there hardened diamond nozzles, or equivalent systems which are lightweight and can avoid a second hot end, to print composite or abrasive materials?

\n", "Title": "Are there multi-filament hardened nozzles or equivalent systems?", "Tags": "|hotend|nozzle|multi-material|diamond-hotend|", "Answer": "

Yes, but there is no hardened version of the \"Diamond\" nozzle design. Basically when you want to use a hardened nozzle, or a Ruby nozzle in combination with color mixing (I specifically refer to mixing solutions as the melting chamber is smaller than filament changing solutions and your request for none or minimal purging towers) you need to order a design that accommodates the replacement of the nozzle. In such a case you can remove the standard nozzle to replace it for a hardened version.

\n\n

There are a few 2 and 3 filament input designs that are offered on typical auction and Chinese selling sites. Below is an example for the Zonestar M3 mixing color hotend with a replaceable nozzle.

\n\n

\"Zonestar\n\"Zonestar

\n" }, { "Id": "11829", "CreationDate": "2020-01-21T17:27:01.070", "Body": "

Which type of filament material(s) is safe to use as an in-wall box for regular, 120v wiring? For instance, an electrical outlet box.

\n\n

In case it matters, location is the state of Washington, USA.

\n", "Title": "What filament material is safe to use as in-wall housing (US)?", "Tags": "|print-material|safety|legal|", "Answer": "

Safety is not the same as legality

\n

Something might be perfectly safe, but it doesn't make it legal to do or allowed to use. Parking your car over double-yellow lines is one example that is perfectly safe but violates the traffic codes.

\n

Any 3D printed box would violate for example WAC 296-46B-300, as it isn't in compliance with NEC Class 3 Standard.

\n
\n

(1) Cables and raceways for power limited, NEC Class 2 and Class 3 conductors must be installed in compliance with Chapter 3 NEC unless other methods are specifically required elsewhere in the NEC, chapter 19.28 RCW, or this chapter.

\n
\n

The NEC is also known as NFPA 70, and availeable at the Website of the National Fire Protection Agency. You will need to look in Article 725.3 for the exact, current specifications that a cable box would need to follow.

\n
\n

725.3 Other Articles. Circuits and equipment shall comply with the articles or sections listed in 725.3(A) through (N). Only those sections of Article 300 referenced in this article shall apply to Class 1, Class 2, and Class 3 circuits.

\n
\n

Also note, that mains wire work is usually regulated in how it has to be done and you might (in some areas) not even be allowed to do it yourself! Even if you may do it, it might be inadvisable to do so due to insurance reasons: non-professional wiring or non-standard parts can be usually excluded from coverage.

\n" }, { "Id": "11843", "CreationDate": "2020-01-23T19:37:23.463", "Body": "

I could have sworn I read somewhere that when printing with TPU to make sure the part cooling fan is blowing. But I just did a quick Googling and couldn't find anything stating such on Matter Hackers or All3dp.

\n\n

I currently don't have a part cooling fan attached (waiting for square nuts to come in). I've been able to get by printing PLA without the fan. I'm curious if this is going to be a major obstacle with TPU.

\n", "Title": "Part cooling fan on when printing TPU?", "Tags": "|cooling|flexible|tpu|", "Answer": "

Sharing fan percentages like in this answer is only helpful if you use the same printer model, cooling fan and cooling duct. As there are many 3D printers and many cooling fans, ducts and solutions, this cannot be readily adopted to every 3D printer.

\n\n

So, in such a case I would rely on the manufacturers of the filament e.g. the flexible filament I use has settings for different printers listed here. TPU is not very prone to warping. The general rule for cooling of TPU is found to not use it for the first 2 layers and after that proceed with a moderate cooling flow. What that value is for your printer is left as an experiment. Several test/calibration print designs exist (e.g. for bridging) to test this out. It is said that a little cooling aids in better aesthetic prints (finer details) while less cooling results in stronger layer adhesion and thus stronger prints.

\n" }, { "Id": "11844", "CreationDate": "2020-01-24T10:20:39.367", "Body": "

I have an Anet A6, an SN04 sensor for the Z-axis, and Marlin 1.1.x software that has automatic bed leveling enabled.

\n

My question is (looking at the image below): "It seems my bed leveling is not working correctly. What can I do to improve it?"

\n

\"enter

\n

You can see several things below:

\n\n

I used the following procedure to get my leveling "right":

\n

First I set the z-offset with the M851 command, followed by M500. Then I performed the bed leveling with G29 T, followed by M500. Then I started my print.

\n

Maybe I need more grid points for my bed leveling, even though I have this interpolation method on. Or do I need to check the implementation of the bed leveling, maybe something is just plain wrong in the software.

\n

What is your recommendation?

\n

Current bed level status (M420 V)

\n
\nSend: M420 V  \nRecv: Bilinear Leveling Grid:  \nRecv:      0      1      2      3      4      5  \nRecv:  0 +0.709 +0.609 +0.519 +0.456 +0.448 +0.404  \nRecv:  1 +0.525 +0.440 +0.370 +0.325 +0.304 +0.298  \nRecv:  2 +0.368 +0.282 +0.222 +0.177 +0.189 +0.182  \nRecv:  3 +0.221 +0.152 +0.100 +0.055 +0.069 +0.082  \nRecv:  4 +0.086 +0.020 -0.028 -0.060 -0.050 -0.020  \nRecv:  5 -0.027 -0.093 -0.138 -0.187 -0.163 -0.146  \n
\n", "Title": "Anet A6, Marlin 1.1.x, bed leveling with sensor probe", "Tags": "|marlin|bed-leveling|anet-a6|", "Answer": "

Eventually it was a mixture of things that sort of solved this.

\n\n

In the end, I think my bed has become a bit curved over time. So a final solution would be to print on a glass bed, but that was not really an option when using the SN04 sensor.\nI am moving now to a BLTouch sensor + glass bed to make everything perfect again.

\n

EDIT: important note, I also put off bed heating. Since I am working with PLA, it was not really necessary. Doing this allowed me to use painterstape, otherwise it would 'fall off' eventually.

\n

EDIT2: I think I have thought of a better explanation. I am using an Anet A6 and it might be because the x-axis was not parallel to the plate. Meaning that my two z-axis stepper motors were not calibrated properly.

\n" }, { "Id": "11850", "CreationDate": "2020-01-24T21:16:54.593", "Body": "

I am an absolute beginner having issues with my Monoprice Maker select v2 printer. \nThe left half of my prints look fine but the right half always gets messed up. When I watch it print the right half of an object it seems that the PLA isn't sticking even though I level the bed thoroughly beforehand. I don't think the problem is that the right half of my board is not sticky enough, because when I move the print so that the whole thing prints on the left half of the bed the problem persists.

\n\n

I have attatched a photo to show what I am talking about. Any help is appreciated

\n\n

Edit: I'm using inland pla. The extruder is at 220\u00b0 and the bed temp is 60\u00b0\n\"enter

\n", "Title": "My printer is making mistakes on the right half of the print, but not the left", "Tags": "|troubleshooting|monoprice-maker-select|", "Answer": "

I found the problem. This model of printer Monoprice Select v2 has bed warping issues so when the bed heated up it would warp severely. I bought a glass bed and all my problems were solved.

\n" }, { "Id": "11851", "CreationDate": "2020-01-24T22:20:01.610", "Body": "

Configuration

\n\n\n\n

Issue conditions

\n\n

I was printing fine until something happened, do not know what.\nWhat I can tell is I can reach any bed temperature with Pronterface, and for any duration. (my little 450 W PSU provides enough power for my needs, like 60, 70 or 80 \u00b0C, a bit hard for 85 \u00b0C though)

\n\n

But as soon as start the print (when the temperature is already ready extruder and bed -- and as I always did), the motors move the extruder in the bed center and then I get the following error :

\n\n
Error:Heating failed, system stopped! Heater_ID: 0\n[ERROR] Error:Heating failed, system stopped! Heater_ID: 0\n\nError:Printer halted. kill() called!\n[ERROR] Error:Printer halted. kill() called!\n
\n\n

If I check the temperature with M105 ; temp report, the printer gives me:

\n\n\n\n

so bed temp command is fine.....until I start the print

\n\n

Temp command set to 0

\n\n

What causes the printer to be halted is a huge temp difference between command and sensor.\nIn the provided G-code, the bed temp was lowered to 30 \u00b0C, which helps :

\n\n\n\n

See the full log here.

\n\n

So, what does cause the bed temp command to get to 0?

\n\n

Not from G-CODE

\n\n

I checked in the G-code, and there is no M140 *0 nor M190 *0 until the end of the G-code, and I'm sure the problem is not comming from the slice/G-code because I ran some previously successfully run G-code/config and they fail the same.

\n\n

Not COM related

\n\n

I ran the same gcode on sd card (to see if the USB was involved), but the behavior is the same: as soon as the bed temp is reached and print started, the bed temp command is set to 0 (I can see command on LCD screen).

\n\n

Hardware related?

\n\n

I did not change the firmware, and print were going fine, before the printer was doing this, so I guess the root cause is Hardware.

\n\n

Firmware involved

\n\n

Because of the \"0 \u00b0C command\", the firmware react to something but I could not understand what is happening here.

\n\n

when checking the firmware code, the only reasons why bed temp is set to 0 when print is running without asking for it would be

\n\n\n\n

nothing that would really explain this...

\n\n

Any hints?

\n", "Title": "Bed temperature command going to 0 \u00b0C with no obvious reason when print is started", "Tags": "|marlin|heated-bed|troubleshooting|firmware|hardware|", "Answer": "

I tried, by instrumenting code, to know where/when the temp bed is modified.

\n\n

I found that it is called in the gcode M81 when starting the print.

\n\n

Wait M81? isn't it M80 to switch on the PSU?

\n\n

What I did in fact was both wiring this up side down and mixing M80 with M81.

\n\n

But what I didn't know is that M80 and M81 are not strictly opposite functions.

\n\n

Indeed, M81 puts the PS_ON signal to PS_ASLEEP != PS_AWAKE but also disable all heaters.\nI couldn't see it without serial trace because the extruder temp is set back after the M81, but not the bed temp.

\n\n

Problem solved.

\n\n

Hope this can help people in the future to spend less time than me on that simple mistake.

\n\n

Note: What remains a mystery, though, is: why was it working before on previous prints and then, suddenly, broke?!!

\n" }, { "Id": "11857", "CreationDate": "2020-01-26T18:51:09.153", "Body": "

I'm printing flat objects (like rectangular) with PLA on a glass bed and 70 celcius degrees (also tried 75 celcius too, 65 celcius and lower ends with adhesion problem in general). Also I use brims too. Most of the time, after a while it shrinks and warps (when print continues).

\n\n

Room temperature is steady, there is no airflow to cool down things..

\n\n

I tried to slow down to 20mm/sec. I tried to increase heat for first layer... Nothing helps.

\n\n

I am suspicious about moisture of the filaments. Can it be related?

\n", "Title": "Does moisture cause warping?", "Tags": "|pla|creality-ender-3|warping|", "Answer": "

Warping is caused by the plastic shrinking as it cools and inadequate bed adhesion is usually the what lets it warp. Either cleaning your print surface very thoroughly with rubbing alcohol or using something like a glue stick on your print bed will mitigate that warping enough that you won't suffer problems with your print. Printing too hot can also be a problem because the plastic will need to even cool more after it is extruded and could possibly lead to more stresses buildup in the plastic.

\n\n

The dimensional stability of PLA really depends on the quality of the plastic. Storage conditions come into play as well, but it is mostly the quality of the material you need to worry about; I have some cheaper PLA that has gotten brittle due to having absorbed moisture despite being in a (albeit somewhat loosely closed) package with desiccants, and I also have a different brand of PLA that is of much higher quality that I just leave out in the open; this PLA doesn't exibit signs of moisture damage. Higher quality filaments are designed to resist moisture better and be more stable in terms of dimensions. With the cheaper brand of PLA, I have also experienced warping, but that is not due to moisture in the filament; that was actually from a new roll.

\n\n

When a filament absorbs too much moisture, it can become brittle but still print. Excessive moisture will cause any water in the filament to vaporize when passing through the hotend and form bubbles that will ruin the finish quality of a print. You'll know if filament is excessively wet because you will hear quiet and sharp snapping sounds as the result of the bubbles that are formed in the plastic popping. You will also be able to see steam if you examine your hotend with a bright light as it is extruding.

\n\n

I'd suggest trying a different brand of filament if possible, cleaning the print surface / adding glue, or at the very least, a new roll of filament.

\n\n

(When using a glue stick to increase first layer adhesion, it could be worth noting something unusual I found; adding glue to the build plate of a Prusa i3 MK3S actually reduces bed adhesion in my experience. It might be worth playing around to see if super clean works for you, or if super sticky does. The build plate is coated in a very finely textured PEI if that is some information that could help your case.)

\n" }, { "Id": "11859", "CreationDate": "2020-01-26T19:17:13.773", "Body": "

This is my first time asking for some help on here and I have Googled, but only found a few reference to this issue. I know I am doing something dumb but I have no clue what it is.

\n\n

I get an unknown motherboard error every time I try to setup my MKS GEN L. At first I found that in the boards.h file there was no reference to this board and the pins.h file was not there either. I have added both of these in and still nothing. I have checked the configuration.h and as far as I can tell I cannot find the error.

\n\n

I have tried this in 1.1.9 and 2.x of marlin and get the same error.

\n\n

The message is:

\n\n
\n    Arduino: 1.8.9 (Windows 10), Board: \"Arduino Mega or Mega 2560, ATmega2560 (Mega 2560)\"\n\nIn file included from sketch\\MarlinConfig.h:33:0,\n\n                 from C:\\Users\\name\\OneDrive\\Desktop\\Ender 5 1.1.61 version (1)\\Ender 5 1.1.61 version\\Marlin\\Marlin.ino:31:\n\npins.h:235:4: error: #error \"Unknown MOTHERBOARD value set in Configuration.h\"\n\n   #error \"Unknown MOTHERBOARD value set in Configuration.h\"\n\n    ^~~~~\n\nIn file included from sketch\\MarlinConfig.h:39:0,\n\n                 from C:\\Users\\jcgra\\OneDrive\\Desktop\\Ender 5 1.1.61 version (1)\\Ender 5 1.1.61 version\\Marlin\\Marlin.ino:31:\n\nSanityCheck.h:311:6: error: #error \"BABYSTEP_ZPROBE_OFFSET requires a probe.\"\n\n     #error \"BABYSTEP_ZPROBE_OFFSET requires a probe.\"\n\n      ^~~~~\n\nSanityCheck.h:624:8: error: #error \"Z_MIN_PROBE_USES_Z_MIN_ENDSTOP_PIN requires the Z_MIN_PIN to be defined.\"\n\n       #error \"Z_MIN_PROBE_USES_Z_MIN_ENDSTOP_PIN requires the Z_MIN_PIN to be defined.\"\n\n        ^~~~~\n\nSanityCheck.h:909:4: error: #error \"HEATER_0_PIN not defined for this board.\"\n\n   #error \"HEATER_0_PIN not defined for this board.\"\n\n    ^~~~~\n\nexit status 1\n#error \"Unknown MOTHERBOARD value set in Configuration.h\"\n\nThis report would have more information with\n\"Show verbose output during compilation\"\noption enabled in File -> Preferences.\n
\n\n

My config file is as follows Config.h.

\n\n

Please let me know if there is anything else you might need to help with this matter and thank you for your time and attention.

\n", "Title": "MKS GEN L unknown motherboard error", "Tags": "|marlin|creality-ender-5|", "Answer": "

As explained in the answer by @towe, you are using an older version of the configuration file, it appears that you aren't using the latest sources from the 1.1.x and the 2.0.x trees.

\n\n

In addition to the answer, the MKS GEN L is basically a RAMPS board with RAMPS pin layout except for a few pins. The board is defined in both 1.1.9 and 2.0.x (if you scroll down you see that it includes the pin definition for a RAMPS layout: #include \"pins_RAMPS.h\"). This being said, you could get it to work in pre 1.1.7 release of Marlin if you must. Choosing a RAMPS board for MOTHERBOARD constant and overriding/defining MOSFET_D_PIN to 7, X_CS_PIN to 59 and X_CS_PIN to 63 will make the firmware work. But, it is advised to get the latest version, even Marlin 2.0.x will run fine on Arduino Mega 2560 based boards.

\n" }, { "Id": "11875", "CreationDate": "2020-01-27T18:00:24.510", "Body": "

What is the thread pitch of the Ender 3's bed leveling screws? The diameter measures about 4mm. Are they M4 0.7 (coarse) pitch or 0.5 (fine) pitch? I'd like to develop rigorous formulas for the amount to turn the knobs by after measuring (or visually inspecting, since I can see an accurate 0.2 mm first layer decently well) leveling-test patterns in the corners rather than using a closed-loop tune-and-retry approach.

\n", "Title": "Thread pitch of Ender 3 bed leveling screws", "Tags": "|creality-ender-3|bed-leveling|", "Answer": "

Just as Andrew, I did run a quick test to measure it: on my corner closest to <0,0,0>, a little stud is poking out. Just enough to screw on a threadcutter I know without any cutting happening:

\n\n

\"enter\n\"enter

\n\n

Indeed, it is M4x0.7, aka coarse thread.

\n" }, { "Id": "11889", "CreationDate": "2020-01-28T20:12:15.427", "Body": "

We have a Lulzbot Mini at my office (i.e. it's not my personal printer). It's connected to an OctoPi and I've been able to get just one file to print on it (which I had to cancel due to leveling issues). Others have used it successfully in the past.

\n\n

However, right now, every time I try to connect to it through OctoPrint, I just get gibberish back:

\n\n
Connecting to: /dev/ttyACM0\nChanging monitoring state from \"Offline\" to \"Opening serial port\"\nConnected to: Serial<id=0x6f830510, open=True>(port='/dev/ttyACM0', baudrate=115200, bytesize=8, parity='N', stopbits=1, timeout=10.0, xonxoff=False, rtscts=False, dsrdtr=False), starting monitor\nChanging monitoring state from \"Opening serial port\" to \"Connecting\"\nSend: N0 M110 N0*125\nSend: N0 M110 N0*125\nRecv: \ufffdNp|\\x04n6H\\x15\\x06'\nRecv: \\x10ONv\ufffdHO\nRecv: J%\ufffdi\ufffdh\ufffdON,=\\x0c\\x14\\x07\ufffd|i\ufffdMx86B~1~\\x04g\\x1a\\x1e\ufffdHO\nRecv: J%P|H~\\x03\ufffd^\ufffd-\\x08n6\ufffdM\ufffdw\ufffd\ufffdK\ufffd'\nRecv: \\x08\ufffd^\ufffd;V7JO\\x1e\\x0b\ufffd\\x08\\\ufffdH^\ufffdKqp|\\x08\\x13\\x08H\\x08\\x04P|\"]+X{\\x16\ufffd\\x08nl\ufffdP<\\x08\\x14\\x08]\ufffdx\ufffd\nRecv: \ufffdJ\ufffdX\ufffd\\x1a\ufffdH3H6F\ufffdJ\ufffdJ%\ufffdJ/H\\x08^\ufffdI\\x08ZY\\x08\\x07:&h\\x0f^\\x17\ufffd}\ufffd$p|\\x08\\x14\nRecv: ^HJ\ufffdJ\ufffd\\x08\ufffdJ\\x1b\ufffd*':f\\x18\ufffdk\\x1e>H]\ufffdZf\ufffdJ\ufffdZ\ufffdJ\ufffd%\\x08^\\x08W\ufffdX\ufffd\\x1a\ufffd\\x0b\\x1e\\x1eH]\ufffdZ\ufffdKq?N!KP\ufffd.N!Kka\ufffd.a\\x0b'\nRecv: \ufffdJ\ufffdJ\ufffdJH\ufffdI\ufffdJHJP|@_a\\x13\ufffd\\x16\ufffd/m\\x7f\\x1fy\ufffd\\x16=\ufffd6\\x1b\ufffdVO6\ufffd=\ufffd6\\x1b\ufffdf>\ufffdK7f\\x7f\ufffdK{f>\ufffdK\\x1bf\ufffd\ufffd+'J\ufffdz\ufffdJ\\x08@\ufffd\\x08\ufffdJ%\ufffd\\x18%\ufffd(\ufffdJ\\x17\ufffd\nRecv: \\x07\nRecv: \ufffdM!\nRecv: gJ\ufffdj\ufffdI\ufffdj\\x08N\ufffdx\ufffd\\x1a\\x7fB\n\ufffd\\x0b]F\ufffdJ\ufffdJ\ufffdx\ufffd\nRecv: V\nRecv: o\\x16\ufffdF\ufffdJ\ufffdZ\ufffdj\\x0c.\ufffd\\x08%\ufffd\\x05\\x07J\ufffdj\ufffd5\ufffd\\x18HJ\ufffdJ\ufffdX\ufffd*\ufffd\\F]{L$P<\\x08\ufffd\\x08~5Ip|\\x04}&@\\x1faF\ufffdJ\ufffdJ\ufffdIf\ufffd\\x1a33l\\x7f\ufffdN\\x0bYp|\\x088\nNo answer from the printer within the connection timeout, trying another hello\nSend: N0 M110 N0*125\nThere was a timeout while trying to connect to the printer\nChanging monitoring state from \"Connecting\" to \"Offline\"\nConnection closed, closing down monitor\n
\n\n

The only thing I can think of right now is refreshing the firmware, but before I do that, I figured I'd ask here. Does any one know what this gibberish is and how I can restore this printer to regular operation?

\n", "Title": "OctoPrint Unable to Connect to Lulzbot Mini", "Tags": "|octoprint|lulzbot|serial-connection|", "Answer": "

If the printer returns gibberish, the Baud rate of the connection is incorrect. You are using 115200 in the example above. For older Mini firmwares, it should indeed be 115200. For the newer 1.1.5.xx builds, it should be 250000.

\n" }, { "Id": "11898", "CreationDate": "2020-01-29T12:54:17.590", "Body": "

I created an stl file of bathymetry using Matlab and the results can be seen in the two pictures below in Blender. The problem I have is that it is very thin with overhangs. I would like to add some kind of support. For example put a solid block underneath it like in the third picture.

\n\n

\"enter

\n\n

\"enter

\n\n

\"enter

\n\n

I can't seem to figure out how to do this in Blender and if I try to fix the stl file in Blender it puts a surface through my original surface. I also tried to use Microsoft 3D printing repair but it takes hours to queue.\nHow to I change my stl file so it has enough support to be printed?

\n", "Title": "creating support for a surface", "Tags": "|3d-design|support-structures|stl|blender|", "Answer": "

You generated a simple surface. You do however need a closed body to print. You might get away with using that surface as a cutter for a block and removing the top half, but there is an easier way using blender:

\n\n\n\n

Now you have a positive thickness, solid body! Run a simple \"remove double vertices\" on this bottom if you want to reduce file size, but there you go! Export as STL and print!

\n" }, { "Id": "11946", "CreationDate": "2020-02-03T11:28:02.260", "Body": "

Lately I noticed that there is a new type of nozzles (called airbrush nozzles?!?) available; typically found on those online overseas vendor sites. The nozzles are advertised for usage in E3D hardware, but are not found amongst the E3D genuine nozzles.

\n\n

These nozzles look like this:

\n\n

\"Airbrush

\n\n

\n\n

What are the basic physics principles or what is the engineering relevance for application of airbrush nozzles? (Gimmick or actual product improvement?)

\n", "Title": "Is there an engineering/physics foundation for application of airbrush nozzles?", "Tags": "|nozzle|", "Answer": "

An obvious drawback is the lack of a flat surface around the opening, resulting in a severe limitation of the extrusion width to exactly the nozzle width (plus some percent). This nozzle cannot efficiently push the filament against the nearby perimeters because it cannot constrain its height and it cannot flatten the top of the already extruded one. So layer adhesion may not be a big problem, but perimeter-to-perimeter likely is.

\n\n

Also, the thinner wall and extended length will reduce the filament temperature, however I don't know whether a simple compensation would do, since cooling will depend on filament speed and travel speed.

\n\n

Regarding the cooling, you can check also the original designer's goal:

\n\n
\n

A big problem with small. hot and slow prints is the heat radiation of\n the hotend and nozzle itself. To get as little as possible heat into\n the print, the nozzle needs to be as long and pointy as possible. The\n longer the nozzle, the bigger the distance between heater block and the\n print. And the more pointy it is, the less heat radiation can affect\n the print. A nice side benefit: the cooling fan can blow better onto\n the print and \"around\" the nozzle.

\n
\n" }, { "Id": "11951", "CreationDate": "2020-02-05T03:33:12.253", "Body": "

I just started 3D printing a few weeks ago, so I'm still trying to get a handle on the tricks. I printed something with a flat surface and a few raised pieces (shown below). I'm pretty happy with the surface quality overall; however, in the locations where the print head came back for it's second pass the quality is worse. Is there any way to fix this? I'm assuming it's a slicer issue...

\n\n

Using Ultimaker Cura 4.4 & Ender 3 Pro

\n\n

\"enter

\n", "Title": "Poor surface quality when sweep interrupted", "Tags": "|print-quality|ultimaker-cura|creality-ender-3|", "Answer": "

It looks to me like you have underextrusion in these regions probably due to loss (oozing) of material during travel moves prior to printing them. Aside from small gaps in the surface sweep at the part you're talking about, I see a long slightly-diagonal light-gray line between the middle of the right and the middle of the bottom of your image, which appears to be material that oozed during combing.

\n\n

Make sure you have retraction enabled, have sufficient retraction (at least 6 mm for bowden; somewhat less for direct-drive) and either disable combining or set max combing distance very small (like 1.5x nozzle width) and see if this helps.

\n" }, { "Id": "11953", "CreationDate": "2020-02-05T11:40:25.890", "Body": "

In analogy to: What glues for bonding printed PLA to injection-molded plastic?, what are the best glues to use for PETG?

\n

I mostly print in PETG and have occasional failed prints which I usually reprint. But what if I'd like to repair a print e.g. a split between layers or a part broken off?

\n

Knowing that PETG is more "greasy" than PLA, what typical glues can you use to create a good bond; this question excludes using heat to (re-)bond.

\n", "Title": "What glues to use for PETG?", "Tags": "|post-processing|petg|glue|", "Answer": "

A strong epoxy like BSI 5-15\u00a0min works great for me. I use it to bond high power rocket fin halves printed from PETG:

\n

\"enter

\n" }, { "Id": "11956", "CreationDate": "2020-02-05T16:37:30.527", "Body": "

I am developing an image to gcode program, that would recognize edges and generate corresponding G-code to be sent to a plotter. I was able to detect edges using the Sobel operator; then the edges are converted to an undirected graph using a search heuristic of my creation. Converting a graph to functional gcode is not difficult: depth first search does the job. The issue is that using this method the generated paths for the plotter are far from optimal, since they contain many movements that could be removed or shortened just by printing paths in a different order. This can be seen clearly in the images below.

\n\n

Is there an algorithm that can convert an undirected graph to optimal G-code paths? Otherwise, if there are none or the problem is NP complete, what heuristics can be used to generate almost-optimal gcode (e.g. the ones used in programs such as Inkscape)?

\n\n

The graph on the left is converted to the gcode on the right using depth first search on the connected component of the graph. The white and red lines represent, respectively, the visible writes and the invisible movements of the plotter. The G-code can be found here.

\n\n

\"An\n\"The

\n", "Title": "Converting an undirected graph to optimal G-code paths", "Tags": "|g-code|file-formats|algorithm|", "Answer": "

Quote of comment of R.. GitHub STOP HELPING ICE on question reads:

\n\n
\n

Pretty sure it is NP-complete (equivalent to travelling salesman problem), no? \u2013

\n
\n\n

This is correct; this is route optimization at its purest, and is by no means a new problem. You want to travel the shortest total distance between all vertices of what's essentially a totally-interconnected graph; there are no inherent limitations on going from anywhere, to anywhere. The TSP is the general-case statement of this problem, which your problem specializes only slightly by predefining certain movements along edges as being required in the final path (but those edges can be traversed in either direction and in any order.

\n\n

What makes this complex on its face is the sheer number of possibilities that an exhaustive solution to the TSP like Held-Karp has to evaluate. You have no real constraints regarding which points to travel between; you can go from anywhere, to anywhere. Only a relatively small number of edges (your extrusion lines) are known requisites, and those can theoretically be traced in any order.

\n\n

If I'm reading your graph right, you start near the top center, then go to the top left, then to the s-curve, then you jump to the main shape and start traversing it from the \"right arm\", turning downward through the \"body\" and \"left foot\" of the central shape, then up to the \"right hip\", through that leg to the foot, then back up to the \"left shoulder\", through that \"arm\", etc.

\n\n

If I have that right, then you definitely have \"endpoint detection\", where you are identifying points in the graph that are part of only one line segment (and therefore will require a travel move to get to or from them), and are planning travel moves to and from those points. Very smart. I would be interested in knowing exactly how you choose the next one to travel to. Obviously the closest endpoint of an undrawn line is a natural choice, but your algorithm doesn't seem to be doing that. Right from the off it chooses a relatively further point to extrude and then comes back to the rest of the shape. That actually seems to be the most efficient move in the overall graph, because if you don't get it early you will very likely make a big move to get back to it later, but making that decision in a non-exhaustive way doesn't seem intuitive.

\n\n

Anyway, your algorithm was doing pretty well at path choice, up until it finished drawing the \"right leg\". The most efficient move from there would be to go to the bottom of the \"Y\" looking shape to the right of the main figure and trace through that. When that's done, the closest undrawn line segment will be back at the left shoulder of the main figure, which will lead you to the small dots, and you'll end in this region with relatively small travel moves. Overall, I think that a \"closest remaining endpoint\" strategy would be near-optimal at every turn; when you reach the end of a drawn line, look for the endpoint that is closest to your current location. It would make most of the decisions your existing algorithm does, and a few better ones. It's not always the best choice (case in point, the dot at the upper left, which is never closest to the end of any other move and so will be ignored until it's the last one left) but more often than not it is.

\n\n

My programmer savvy says you also have some recursive intersection tracing (\"tree-walking\"); the algorithm sees that there are multiple paths to draw from a single point, remembers that point and then picks a path. When it reaches an end of a chain of extruded lines, it goes back to the most recently-encountered intersection, re-evaluates available paths, and picks the next one until all paths from that intersection are drawn. Then you skip back to the previous intersection, and so on in a recursive LIFO fashion.

\n\n

While that's also generally a smart way to approach it, it makes a couple obviously inefficient moves, such as from the \"right foot\" of the main figure back to the \"shoulder\" (which is the most recent intersection visited but not fully drawn by that point). The more efficient move is simply the closest remaining endpoint, the bottom of the wonky-looking Y to the right of the main figure.

\n\n

How you choose intersection paths to prioritize is also key. In general, taking the route that will lead you to the closest intersection or endpoint will reduce the possible backtracking you have to do. However your algorithm seems to prefer the longest path from a fork (or the one with the most forks along it) and that turns out not to be a terrible way to do it in this particular graph.

\n\n

Now, having drawn the \"left arm\" of the main figure, it is totally beyond me why your algorithm chose to cross the graph to draw the wonky Y, then cross back over to the left side. That is by far the least efficient move it makes and the one you're probably pointing to yourself. The most efficient path from the end of the left arm of the main figure given what's left to draw is straight-up closest-endpoint, filling in dots and lines on the left side, then making one move across the graph to the wonky Y. Closest-endpoint would actually have already filled in that Y as covered earlier, and you'd end your graph traversal in the left region of dots and small lines. You may have one or maybe two relatively inefficient moves between corners of this region on the left of the graph depending on the closest point calculation, but those are minor compared to the moves made across the graph. If your algorithm is producing deterministic results for this graph, I'd debug it and step through to that point, and figure out why on Earth it thought that sequence was preferable. Optimizing that decision may very well be the key to a near-optimal overall graph-walking strategy.

\n" }, { "Id": "11967", "CreationDate": "2020-02-06T19:45:33.203", "Body": "

I'm printing with opaque grey PETG on glass. The intention is to produce a house number plate, so a shiny, production quality finish on the bottom. For this reason, extruding at 245 \u00b0C with a bed at 95 \u00b0C, to give a perfect glass finish with no filament lines showing. Smaller test versions have been very promising; this seems to be the maximum temperatures before warping or a severe elephant's foot arises.

\n\n

However when printing the full-scale version, areas of the first layer of filament seem to go completely \"transparent\"; there seems to be filament there - you can feel the filament \"comb\" when you run your finger over it, and it feels a similar thickness to its neighbours.

\n\n

On the attached photo you might think that those gaps are simply not printed yet, however you can see on the top right corner that it's actually started on the next layer.

\n\n

\"print\"

\n\n

What could be causing this? Is it a blockage which is interrupting flow, and maybe insufficient filament is being \"stretched out\"? Or maybe it could be something to do with temperature? Could it be insufficient layer height (I'm using 0.2 mm, but 0.24 mm on first layer, increasing further reveals filament lines, but tested higher and lower on smaller scale with success).

\n\n

I've tested a range of extrude and temperatures and chosen the temps with the best results; but when I \"go large\" this always seems to happen. I've also calibrated the bed height using the 3 point adjustment screws on this printer (Qidi X-Plus). (The transparent areas are actually occuring in the center where the smaller test prints where working perfectly, so don't know how it could be to do with this).

\n", "Title": "Why could my opaque PETG be printing \"transparent\" in certain places?", "Tags": "|print-quality|petg|glass-bed|qidi-tech-x-plus|", "Answer": "

there are a lot of variable at play here, when are there not in the 3D printing world. Given that I don\u2019t know the answer to your question although very interested to learn how you solve it eventually I will mention the things you can do ,maybe you have to eliminate or identify what parameter is the fault cause.

\n\n

Things I would eliminate - \n1 bed temperature discrepancies - move the origin cycle of the print or rotate it to see that the problems are physical rather than software. If the faults occur at the same place on a reoriented print then I would say you can eliminate nozzle and bed temps as well as material inconsistency.

\n\n

May also be worth trying the print from a different complier I often find a piece prints differently and there fore better from this or that printer app.

\n\n

2 could cooling be the issue? What if you hold a heat gun on the extruded parts to slow down cooling I have done this on tricky parts to get better binding, could be similar issue.

\n\n

3 print a raft or test piece and see what happens to transparency with heat - if the material properties change as a result of temperature you could investigate why the temp is changing at the points in the print that are relevant -

\n\n

Not sure that these ideas are that original but the fewer variable you need to consider the faster you will work out what to do about the rouge one

\n\n

Regards

\n" }, { "Id": "11968", "CreationDate": "2020-02-07T00:05:10.707", "Body": "

The e3D volcano features an extended heater block of length 20 mm with the cartridge heater running parallel to the filament.

\n\n

The purpose of this is to increase the speeds at which filament can be printed (of course the extruder and other factors may still be limiting factors).

\n\n

My question is how capable would this heat block be of printing at slow speeds with a 0.4 mm nozzle?

\n\n

Is printing still possible at lower speeds or is the filament heated too much that jams occur? Is the retraction performance okay?

\n", "Title": "Can you print at low speeds with e3D Volcano hot end?", "Tags": "|hotend|speed|e3d-volcano|", "Answer": "

The worry must be about oozing and stringing. In general, no worries! Nothing a bit of tuning would not fix.

\n\n

I have experience printing with Volcano with 0.4, 0.6, 0.8, and 1.0 mm diameter. \nPLA, PETG, Nylon, TPE, and TPU. (Volcano nozzles have been my default style for several years now. I don't even own a 10 mm heat block anymore (ok, maybe one)).

\n\n

And yes, speaking of the classic 0.4 mm nozzle, you get higher stringing and oozing compared to 10 mm meltzone. You'll need to tune your retraction - it will be significatnly higher than 10 mm meltzone (I'm not giving numbers because it depends on printer style, material, nozzle diameter, temperature, and even slicer and slicer settings...).

\n\n

I do like to print quite hot though with lots of cooling (PLA 220 \u00b0C for example) to increase layer adhesion and strength - so I experience this more.

\n\n

For oozing, you may need to experiment with XY travel speeds, wiping settings, coasting settings, and similar (this also will help with stringing).

\n\n

Also, keep in mind that the bigger the nozzle diameter, the more cooling you'll need (coupled with slower print) just to solidify that massive extrusion flow/mass.

\n" }, { "Id": "11980", "CreationDate": "2020-02-09T10:38:52.470", "Body": "

I'd like to prefix this question with the fact that I know virtually nothing about 3D printers, aside from the general principles of how they work.

\n\n

I've recently seen that SLS printers have become more affordable, to the point where in a few years they might be a compelling investment. I'm mainly interested in 3D printing miniatures for painting, and as such this one:

\n\n

\"Warhammer

\n\n

For scale, the miniature is about 150 mm long. I'm mainly worried about smaller details, such as the faces of the Gunner or Driver. Will a consumer-grade SLS printer be able to print to such level of detail?

\n", "Title": "What level of detail can be expected from a consumer-grade SLS printer?", "Tags": "|print-quality|sls|", "Answer": "

Consumer Market?

\n

While there are no "consumer level" SLS printers on the market currently, the question in itself is very interesting on a scientific level. The pricing edges for the consumer market for 3D printers can be somewhat estimated from the consumer electronics segment. This puts a maximum price tag of about 2000-2500\u00a0\\$ onto it, comparable to a high-end PC.

\n

CurrentlyFeb. 2020, most SLS machines come with 'inquire for price' or with prices of 5000\u00a0\\$ or larger price tags, which indicates they are intended for professional or industrial use. Most SLS printers in consumer hands seem to be phased out older systems from second hand. So while there are for sure tries to get SLS more affordable, it is not there yet.

\n

Resolution of SLS

\n

SLS printers have resolutions based on two factors1:

\n\n

Generally speaking, the finer the grain and more focussed the laser, the better the resolution. Current industrial machines - even cheap ones - work with particle sizes between 20 and 80\u00a0\u00b5m, with the bulk being around 40 to 60\u00a0\u00b5m2.

\n

The laser focus point ranges generally in the "tens of \u00b5m"1, and is listed with values between 50 and 300\u00a0\u00b5m for most ceramic powders in that paper.

\n

Conclusion

\n

As a rule of thumb, 50\u00a0\u00b5m seems to be the average nylon spot size, which is very much comparable with resin printers using SLA/mSLA/DLP technology. Details on miniatures are usually in the area of 100-200\u00a0\u00b5m, so are well achieveable with either.

\n

Comparison SLS to SLA/mSLA/DLP

\n

Resin technology has the benefit of being easier accessible with some entry level printes between 200 and 500\u00a0\\$. Nylon SLS prints do demand a sealant but prints without any supports, Resin does at times need support.

\n

Printing times for DLP/mSLA is not dependant on the ammount of space used, making packing the build surface with as many models as possible benefitial, while SLS, like FDM, works with a moving spot, so the ammount of models increases print time.

\n

Both Technologies work with hazardous material - resin and very fine powders respecively - and demand proper PPE to handle them.

\n" }, { "Id": "11982", "CreationDate": "2020-02-09T17:03:28.673", "Body": "

I'm trying to print this model of a boat:

\n\n

\"enter

\n\n

It has 2 keels joined by 5 bridges, but the top of each keel is curved, so while the middle part is mostly flat, the bridge on the back has a bit of a slope (as you can see with the \"stair\" effect).

\n\n

The problem I have is that I can't get Cura to properly bridge that specific part: It creates a bridge for the first layer, but on the next layer (where there's a part that needs bridging), it just starts drawing a surface:

\n\n

\"enter

\n\n

In this picture you can see layer 86 it has drawn a bridge between both keels, but on layer 87 it tries to print a surface larger than posible. That results in this happening:

\n\n

\"enter

\n\n

and this causes big gaps in the top surface.

\n\n

I've been trying to play a bit with the settings in Cura, but I can't find anything that would fix this... Is there anything that can help me? (maybe even the model has a design problem)

\n", "Title": "Sloped bridges in Cura", "Tags": "|ultimaker-cura|bridging|", "Answer": "

Cura has some options in this area that might or might not help. I've had mixed results. You should make sure Enable Bridge Settings is on, and try adjusting Bridge Skin Support Threshold. By default bridge settings are only used if 50% or more of the area is unsupported. Area is an utterly ridiculous/meaningless metric for whether bridging is needed, so you probably need to set this to just a really high value like 90% or 95%. You may also want to check that Bridge Has Multiple Layers is on.

\n\n

With that said, for your model I would just use supports with a support interface (roof) below the bridges. You could reduce the material cost of them with Support Tree mode. But there are of course places where you can't use supports in similar models (bridge is over another part of the model and there's no access to remove the support material), so having working bridge settings is still desirable.

\n" }, { "Id": "12014", "CreationDate": "2020-02-15T04:04:00.173", "Body": "

I have a Robo R1+ which uses the nozzle contacting the glass print bed to level the printer. When the print head strikes the bed the Z-min endstops open signaling that the bed is touched.

\n\n

I recently upgraded from an Arduino to an SKR 1.3 running Marlin 2.0 and have been trying to make the printer auto level. However all the examples I can find involve a probe. I'm not even sure what this kind of autoleveling is called.

\n\n

Is there a way to configure Marlin 2.0 to perform this kind of autoleveling. And if so what lines should I comment and un-comment?

\n", "Title": "Switch nozzle contact probe auto leveling with Marlin 2.0", "Tags": "|marlin|bed-leveling|skr-v1.3|robo-r1+|", "Answer": "

Basically, you are also using a probe, the nozzle is the probe. So this is very similar to an auto levelling setup using a capacitive or inductive sensor, the difference is that your M851 nozzle to probe distance is zero, and may receive a positive value to slightly raise it to get a sheet of paper in between the nozzle and printing surface.

\n

Please note that below only changes for levelling are addressed, not other specifics in Marlin 2.x for the Robo R1+!

\n

First you define the nozzle offset in Configuration.h, which is exactly at the nozzle, so X and Y (and Z) are zero.

\n
#define NOZZLE_TO_PROBE_OFFSET { 0, 0, 0 }\n
\n

You can also set:

\n
define NOZZLE_AS_PROBE\n
\n

Furthermore, you need to define a levelling method in the same configuration file:

\n
\n//#define AUTO_BED_LEVELING_3POINT\n//#define AUTO_BED_LEVELING_LINEAR\n#define AUTO_BED_LEVELING_BILINEAR\n//#define AUTO_BED_LEVELING_UBL\n//#define MESH_BED_LEVELING\n
\n

For safety, we usually home Z at the center of the printing surface:

\n
#define Z_SAFE_HOMING\n
\n

Also be sure the following statement is active:

\n
#define Z_MIN_PROBE_USES_Z_MIN_ENDSTOP_PIN\n
\n

Next, you need to define the boundaries of the "probe", which is exactly where the nozzle may come (apart from a small safety offset at all edges called MIN_PROBE_EDGE); how you do that is described in question "How to set Z-probe boundary limits in firmware when using automatic bed leveling?", in Marlin 2.x this needs to be set in Configuration_adv.h:

\n
\n#if PROBE_SELECTED && !IS_KINEMATIC\n  #define MIN_PROBE_EDGE_LEFT MIN_PROBE_EDGE\n  #define MIN_PROBE_EDGE_RIGHT MIN_PROBE_EDGE\n  #define MIN_PROBE_EDGE_FRONT MIN_PROBE_EDGE\n  #define MIN_PROBE_EDGE_BACK MIN_PROBE_EDGE\n#endif\n
\n" }, { "Id": "12025", "CreationDate": "2020-02-17T20:35:35.440", "Body": "

I need to smooth some prints with silk PLA filament.

\n\n

The silk PLA has a nice translucent effect but unfortunately if sanded results in an horrible matte finish regardless the sandpaper grade (I have tried with many grades).

\n\n

Since I don't have specific solvents or special equipment, and I don't want to paint it as well, I'm wondering if there are alternative ways to smooth the surface.

\n\n

(Also a my friend speculated about some heat treatment with hair dryer... never heard of something similar and I'm not sure if this could make sense, and in case how should I try).

\n", "Title": "How to smooth PLA Prints without using sandpaper, solvents or paint", "Tags": "|pla|post-processing|", "Answer": "

Unfortunately, there is no way to smooth PLA without sandpaper, solvent or paint - but you can fix the finish after sanding.

\n\n

If you heat the plastic after sanding to just the point the outer shell starts melting the horrible matte finish goes away and the original color of the plastic returns.

\n\n

You have to heat the plastic and then remove the heat source just as it's start melting (right before the original color returns, so when you see it working it's too late) because otherwise the object will deform.

\n\n

I use a heat gun set to 180C and work quickly in short bursts, an hair dryer is probably nowhere as hot, so it will take longer to heat the surface.

\n\n

You have to do this after sanding, applying enough heat to smooth the layer lines will just cause the object to melt and deform.

\n\n

Try on a few failed prints first, you will need to get the feel for when to stop heating and you will still probably ruin prints every once in a while.

\n\n

Another option is to coat the object in an epoxy that will hide the layer lines the best known brand for this is XTC-3D.

\n\n

If you do decide to paint, you get extra thick primer that's supposed to fill the gaps between the layer lines so you don't need to sand as much (sorry, can't remember the brand name)

\n" }, { "Id": "12027", "CreationDate": "2020-02-17T22:00:07.827", "Body": "

I'm facing weird \"pillars\" of underextrusion on outer walls of my XYZ test cube.

\n\n

On the pictures below I`ve printed PLA test cubes with a 0.4 mm nozzle, 0.2 mm height and 210/50 °C hotends/bed temperature.

\n\n

Gaps are appearing in walls parallel to both X and Y sides. The pictured side is parallel to Y face.

\n\n

I'm slicing with Cura, my printer is a homebuilt around Anycubic Kossel with Marlin 2.0 onboard.

\n\n

What have I tried already:
\n1. Temperature from 190 to 210 °C
\n2. Retraction from none to 6 mm 60 mm/sec
\n3. Tuning down Jerk in Marlin from [10,10,0.3] to [5,5,0.3]
\n4. Tuning down acceleration from 3000 to 1000
\n5. Tuning print speed from 60 to 30 mm/sec\n6. Checking belts, nozzle and extruder.

\n\n

Now I just ran out of ideas. Delta is calibrated by G33 autotune. Mechanics looks just fine. What am I missing?\n\"enter

\n\n

\"enter

\n\n

UPD1: tried removing combing - it did not help. But I noticed that my printer accelerates strangely at this parts

\n\n

UPD2: could it be stuttering? I have 320 segments per second with block buffer size of 8 bytes(?)

\n\n

UPD3: lowering segments count to 120 and raising block size to 32

\n", "Title": "Partial underextrusion in walls", "Tags": "|delta|kossel|underextrusion|skr-v1.3|", "Answer": "

Well, I could not track the problem and it lasted till two major updates:

\n\n
    \n
  1. I have changed rods for ones with proper lenght
  2. \n
  3. I have reset all Cura settings to default
  4. \n
\n" }, { "Id": "12033", "CreationDate": "2020-02-19T18:26:15.833", "Body": "

I run a 3D printer farm and I have to replace my Bowden tubes on the printers after about a month or two of use(roughly 1000 hours of use). The Bowden tubes continually melt on the side of the tube very near to where it pushes against the nozzle. I am running Ender 3 Pros and I run at about 205 °C with PLA. The Bowden tubes I have are some I found on Amazon and they are not Capricorn tubes.

\n\n

\"Note

\n\n

EDIT1: I have added more pictures below of a new failure. This time you can see the marks of the teeth of the coupler a good inch below the failure point.\n\"New\n\"New\n\"New\n\"New

\n", "Title": "What causes my Bowden tube to melt on the side?", "Tags": "|creality-ender-3|bowden|", "Answer": "

This isn't melting, it is wear.

\n

And it isn't typical wear you would expect due to abrasion. It is wear caused by retraction. As your filament retracts (usually between parts of the model to reduce stringing), the semi-molten filament is sucked back into the tube.

\n

Then, when the filament is unretracted, the filament is pushed back into the nozzle. However, in the meantime, a little of the filament solidified on the PTFE wall. PTFE is very slippery, so the solidified chunk is peeled off the wall no problems - but it takes with it a tiny bit of the inside of the tube.

\n

Do that hundreds of thousands of times, and the wall of the tube gets thinned at that point until it fails.

\n

One solution to this is to reduce retraction. Notice how the distance between the worn bit and the end of the tube is always the retraction distance in millimeters?

\n

Another solution is an all metal hotend.

\n

Or just trim your bowden tube shorter by a few mm every hundred hours of printing or so.

\n" }, { "Id": "12035", "CreationDate": "2020-02-19T20:27:11.243", "Body": "

Is there a software package that when I have my printer connected directly to my PC via USB could record and export hot end temperature data overtime?

\n\n

Ideally this data would be recorded in a way that I could export it and manipulate it in the likes of Excel.

\n\n

E.g. I see Pronterface has a temperature graph but it doesn't seem possible to export this. I know Simplify3D has a temperature plot in the machine control panel, anyone know if you can export from this?

\n", "Title": "Software to record hot end temperature?", "Tags": "|software|", "Answer": "

I don't know if using OctoPrint is an option. If so, there is a plugin that claims to do exactly this. And you could probably find a few more if you looked for them. Note that I have no first hand experience with this plugin, but I can vouch for OctoPrint being convenient and by default it shows a temperature graph.

\n\n

It might even be relatively easy to write your own plugin to accomplish this. This will mostly depend on your comfort with coding in Python/JavaScript.

\n\n

As a sidenote: if your printer is connected directly to your computer via USB, chances are pretty high it is a simple serial connection. Having multiple programs use this connection at once is not possible as far as I know.

\n\n

This implies that you will not be able to have your current software send it G-code lines while having another one recording the temperature values sent back from the printer.

\n" }, { "Id": "12040", "CreationDate": "2020-02-20T06:40:11.417", "Body": "

I'm trying to do a multifilament print on my single extruder machine. So I separated out the models based on filament, and imported the parts into Cura. I ensured that \"Automatically drop models to build plate\" was disabled and in the \"Prepare\" phase that seems to work. However, when I slice the model it gets pushed back down to the build plate as can be seen in the picture below. Any recommendations? Do I just need to write a script to go in and shift the z location?

\n\n

\"Preview\"

\n", "Title": "Cura projecting floating print onto build plate during slicing", "Tags": "|ultimaker-cura|multi-material|", "Answer": "

I've been playing with this and came up with a solution, so I thought I would share in case anyone else had this issue in the future. In Ultimaker Cura I enabled supports and z-hopping before I sliced the part, then I ran this Python function to remove the supports and get the extruder setup.

\n\n
import re\n\ndef float_part(file):\n    printString = ';LAYER:'\n    partString = ';(.*?).stl'\n\n    with open( file , 'r') as content_file:\n        content = content_file.read()\n\n    printArea = re.search( printString , content ).span(0)[0]\n    partArea = re.search( partString , content ).span(0)[0]\n\n\n    uncommentedLine = partArea - re.search( '\\n.*?(?<!;)\\n' , content[ partArea:printArea:-1 ] ).span(0)[0]\n\n    lastExtrusion = uncommentedLine - re.search( 'E' , content[ uncommentedLine:printArea:-1 ] ).span(0)[0]\n    secondLastExtrusion = lastExtrusion - re.search( 'E' , content[ lastExtrusion-1:printArea:-1 ] ).span(0)[0]\n\n    lastExtrusionAmount = float(re.search( '\\d+(\\.\\d+)?', content[lastExtrusion:] ).group(0))\n    secondLastExtrusionAmount = float(re.search( '\\d+(\\.\\d+)?', content[secondLastExtrusion:] ).group(0))\n\n    ResetCommand = '\\nG92 E' + str(lastExtrusionAmount) + '\\n'\n\n    with open( file , 'w') as content_file:\n        content_file.write( content[0:printArea] + ResetCommand + content[uncommentedLine:] )\n
\n\n

\"Elevated

\n" }, { "Id": "12045", "CreationDate": "2020-02-21T00:47:55.110", "Body": "

I am looking for a cheap 3d printer that can print this: https://www.thingiverse.com/thing:1307100 and is hopefully under US $200, such as the Creality Ender 3, Monoprice Select Mini Pro and V2, the Wanhao Duplicator i3 mini, the Anet A8, or the tronxy p802ma.

\n", "Title": "3d printer recommendations that can print fidgets?", "Tags": "|creality-ender-3|anet-a8|wanhao|monoprice-select-mini|tronxy-p802|", "Answer": "

Pretty much any recent commercially available 3D printer should be able to print that model. Only really old stuff, or poorly built DIY machines, might have trouble. It may take some tuning of slicing settings to avoid problems with the rings bonding together during printing, but a more expensive printer is unlikely to make the tuning any easier.

\n\n

The printers you listed in the question are probably all decent choices. I have an Ender 3 and like it but it's a little over \\$200. There are some printers well under \\$200, like the Monoprice Mini Delta, but I don't have any experience with them and product recommendations are off-topic here anyway. Also keep in mind that you'll have to spend a little bit on consumables, at least one spool of filament that's typically around \\$20 (slightly cheaper for no-name brands, much higher for fancy color/shine/etc. or special materials, but simple PLA should be fine for what you want).

\n" }, { "Id": "12048", "CreationDate": "2020-02-21T08:17:02.820", "Body": "

I've created a model in Sketchup and exported the file as a .stl\nSince the model is 80cm x 24 cm i need to cut it in meshmixer. But when i open it in meshmixer it appears tiny. Why doesn't it open as the real size?

\n", "Title": "Meshmixer size of model", "Tags": "|meshmixer|", "Answer": "

You mention your dimensions in cm. Any chance that the STL export is also in cm where Meshmixer might expect mm?

\n\n

Assuming your Sketchup template was set to cm:
\nTo validate this guess you could measure/eyeball a known dimension and if that comes out to be 10 times too small you could scale your model by 10 times.\nAlternatively you could adjust the template in Sketchup.

\n\n

Additional information on STL files:\nBy it's very nature, the STL file format is just a bunch of unitless numbers in a well defined structure. This structure represents a set of points (vertices), lines (edges) and and triangular surfaces (facet). A few simple rules apply to exactly how all this should be defined. (See here)

\n\n

But the key thing here is that an STL file is not aware of units, the program used to generate the STL file (e.g. CAD software) needs to be told what units to use, and accordingly the reading program (e.g. slicer) needs to use the same settings. Generally in mechanics applications we default to mm, at least in metric land. All this is explained a bit more verbose here.

\n" }, { "Id": "12051", "CreationDate": "2020-02-22T02:34:52.363", "Body": "

Recently I've been having trouble printing properly on my Creality Ender-3 printer. I ran a pretty long print (approx. 15 hours) that turned out really well. I then started printing an attachment for the original print and saw that it was printing layers that were extremely thin.

\n\n

I first scraped off the excess filament left on the extruder nozzle. Then, I heated up the bed and rubbed off the layer with alcohol. I tried printing it again but it still didn't print right.

\n\n

Thin layer
\n\"Thin

\n\n

Weird thing
\n\"Weird

\n\n

From the images above, you can tell it's noticeably hard to see the layer, which shows just how thin it is.

\n\n

I sliced the model in Ultimaker Cura. I set the layer height to 0.15 mm. I've printed models before with this height but the layer wasn't transparent.

\n\n

What should I do to fix this issue?

\n", "Title": "Issue with 3D printer making super thin layers", "Tags": "|ultimaker-cura|creality-ender-3|", "Answer": "

You need to level you bed. Thin prints happen when the extruder is too low and is printing too close to the bed.

\n\n

\"Example

\n\n

Download the following test codes from this address:

\n\n

https://www.chepclub.com/bed-level.html

\n\n

1) The first code is the most important you will want to run moves the extruder to five points on your board - Front Left and Right, Back Left and Right, and Center. Using a folded piece of paper - I use a business card - drag the paper under extruder of each of the four corners. You want to make sure you get a bit of drag when pulling out the paper/card. If you feel have enough of a gap that you can run put the paper/card under the extruder and that you feel a bit of tug when pulling it out. It runs the middle last - if you are having issue with the drag, adjust all four corners slowly until it is right.

\n\n

2) The second runs the extruder in a square pattern on your board. You simply want to run your finger of the print - if it sticks to the bed, you are good - if it doesn't, adjust your corners up and keep testing.

\n" }, { "Id": "12065", "CreationDate": "2020-02-24T20:46:31.447", "Body": "

I've setup OctoPrint with the goal to log temperature data of my hot end. So, I've enable serial logging and examined the serial log file.

\n

The Output for an example line is as follows:

\n
\n

Send: M105

\n

Recv: ok T:20.7 /50.0 B:20.0 /0.0 T0:20.7 /0.0 @:0 B@:0

\n
\n

So, from what I understand:

\n

T = Thermistor temp of hot end \u00b0C

\n

In the example, current temp is 20.7 and target temp is 50.0

\n

B = temp of bed \u00b0C

\n

In the example, current temp is 20.0 and target temp is 0.0

\n

But what is T0?

\n

Or the @ and B@?

\n", "Title": "In OctoPrint when receiving temperature data (M105) what is T0?", "Tags": "|octoprint|", "Answer": "

T is the selected tool, T0 is the first hotend tool. If you only have one hotend, T and T0 are exactly the same.

\n\n

Do note that G-codes are described on the G-code wiki page, for M105 you can find:

\n\n
\n

The parameters mean the following:
\n - T, T0, ..., Tn - extruder temperature. In a single extruder setup, only T will be reported. Some firmware variants will report no T0 in multi extruder setups - in that case T is to be considered the temperature of the first tool. Otherwise, T should be considered the temperature of the currently selected tool (which will be repeated in one of the Tn entries)
\n - B - bed temperature
\n - C - chamber temperature
\n - @ - Hotend power (Prusa only)\n - B@ - Bed power (Prusa only)
\n - P - PINDAv2 actual (Prusa MK2.5/s MK3/s only)
\n - A - Ambient actual (Prusa MK3/s only)

\n
\n" }, { "Id": "12087", "CreationDate": "2020-02-27T08:57:23.307", "Body": "

I am thinking to use closed-loop stepper motors to prevent step loss and make the machine more accurate. What options (preferably low cost) are there for:

\n\n

stepper motor + driver + encoder + microcontoller

\n\n

Is building it from scratch worth it? E.g. Arduino Mega 2560 + RAMPS 1.4/1.5/1.6 + stepper motors (e.g. NEMA17) + drivers (e.g. A4988, DRV8825) + encoders (e.g. AS5047P, AS5047D, AS5048A, TLE5012B) + microcontrollers (e.g. STM32).

\n", "Title": "Closed-loop stepper motors", "Tags": "|stepper|stepper-driver|motor|arduino|ramps|", "Answer": "

It has been 25 or 30 years since I did my first closed-loop system. I was driving a syringe pump from a printer port on a MS-Dos PC that ran hot had to disable the timer interrupts to make the deadlines amount moving the plunger, updating the floppy drive reading and computing the new position, getting the time, and fixing up the timer.

\n

I found moving to a goal with a tolerance of 1 or 2 stepper motor steps was on the money over 95% of the time and I made deadlines over 90% of the time. Any error was made up in the next move. I used the same scheme on an 8 x 32 foot structured light scanning table. I am pretty sure I saw memory errors from cosmic rays on this one. It took 500 hours to scan that with no parity check on the RAM. It had some wooden structures that changed size over time. I could track the movement with the error files from the shaft encoders.

\n

On the x y scanning table, the wait time was7 seconds after a move before the camera stopped asking enough to capture an image. We elected to fix up the position in software and accumulate the errors in the next move in order to be able to finish the project.

\n" }, { "Id": "12097", "CreationDate": "2020-02-28T16:12:25.070", "Body": "

If I set my prints on the window sill (indoors) will the sunlight still be able to cure the resin? The problem with setting them outside is the wind knocking them over.

\n", "Title": "Can you cure resin with sunlight through a window?", "Tags": "|sla|resin|", "Answer": "

Yes.

\n

I frequently leave models made on a Saturn printer with Elegoo gray resin on a surface in the sun to slow cure them. If properly cleaned their finish is indistinguishable from models rapid cured in a UV chamber.

\n

It should be noted that I only do this with small models that are Table top miniature scale. Large models with lots of shadows, overhands or complicated detailing may not cure evenly.

\n

I usually do this if I have a lot of models to cure and am too lazy to keep cycling them through my cure station.

\n" }, { "Id": "12099", "CreationDate": "2020-02-28T23:02:34.030", "Body": "

I've been noticing in some of my linear advance test towers that the (very minor/fine) ripple pattern in walls varies with the K factor, which made me think that it isn't coming from any vibration in the print head motion, but rather from quantization of the extruder into discrete steps (i.e. wall gets slightly thicker right after a step, thins out afterwards until the next step, repeats).

\n\n

By my math, with 1.75 mm filament and 93 steps per mm on my printer (Ender 3), each step is 0.02585 mm\u00b3 of material. At 0.4 mm line width and 0.2 mm layer height, there's 0.08 mm\u00b3 of material per linear mm, so that should give roughly 3 extruder steps per linear mm. That seems comparable to the ripple rate I see, although not exact; I suppose it varies somewhat because of linear advance and perhaps other reasons.

\n\n

Anyway, to get to the question, am I missing something or is this the limiting factor in print resolution on my printer? It seems like isolated small details (smaller than 1/3 mm) won't be extrudable at all except as ooze, or over-extruded if they happen to cross a step threshold, and like things would be far worse if I tried to use a smaller nozzle and thinner layers. It seems that, ideally, you'd want the E-axis steps per mm to be sufficiently high that quantization is a non-issue (i.e. +-1 step is small relative error) for extrusions corresponding to a single X- or Y-axis microstep.

\n", "Title": "Is E-axis steps/mm resolution limiting factor in print quality?", "Tags": "|print-quality|extruder|creality-ender-3|resolution|", "Answer": "

Your calculations about the theoretical extruder resolution are spot on. I did a similar calculation to evaluate which extruder to use with different hot ends, I paste the results. The dark cells are the input cells, the rest is calculated. You can see that for some lines I entered directly the mm/microstep value, since I wanted not a theoretical but practical result for my printer (3 mm filament) or for known extruders (BMG).

\n

\"enter

\n

Concerning the question, the resolution of the extruder matters, but it's a bit complicated to estimate exactly how much.

\n

In general, this are the factors I can think about.

\n

A poor resolution may not impact straight lines much, since the rotation of the extruder is continuous and the extruder is unlikely to snap exactly to the desired microstep position as soon as you ask for it: it's likely a bit behind all the time, that's how torque is obtained (more or less).

\n

The issue may become smaller with drivers which interpolate microsteps up to 256x.

\n

However, whenever there is a change of flow rate, poor resolution implies that you cannot control the exact location/moment where/when the flow changes. This matters mostly at the end and at the beginning or retractions/re-retractions. Maybe you get more ooze?

\n

However, the extruder resolution is not, in practice, as good as you calculated. In fact, as we know, microsteps reduce the incremental torque to very low values. The extruder is a motor which requires quite a lot of torque, since pushing the filament is quite hard, and it is unlikely that you can achieve all the time the 16x microstep accuracy you assumed. For example, due to friction in Bowden, hot end, ... the filament (= the motor shaft) may at a certain point stay "back" more than average. This would cause an increase of effective torque, pushing the filament a bit faster, which would it bring to in sync or so with the desired position, but at that point it would slow down, and so on. Depending on the average speed, this oscillation may be dampened (and then no rippling is visible) or may oscillate constantly, and you see ripples also along straight lines.

\n

This is why I placed the usteps column in my calculations: it is meant to calculate a more realistic resolution assuming that no accurate microstepping is achieved. I assumed higher achievable microsteps the lower the load on the motor is (this means gears, or thinner filament).

\n

Having a high resolution to begin with clearly helps to reduce this issue. You can try to increase the current to the max your drivers and motor and cooling allow, and see if the ripples change. I think it will be reduced.

\n

You may also try to build the Orbiter extruder (linked also in the table) and see how it goes.

\n" }, { "Id": "12100", "CreationDate": "2020-02-29T09:49:35.657", "Body": "

I began build LCD printer and I want make some modifications.

\n\n

What if I will place LCD below VAT\"enter? Will the display break when printing? what are the risks?\nI seen a lot of printers and all of them use PP material for VAT bottom and attach with a lot of screws. I want make more simple VAT-LCD constructions and I think this construction transmis UV light better

\n", "Title": "Attach LCD to VAT", "Tags": "|diy-3d-printer|sla|lcd-screen|", "Answer": "

The way you're thinking about using the LCD directly on the bottom of resin vat and cure the resin is a good idea, but there are several problems that I will focus on the main. as mentioned by @Trish some issues might occur to the LCD and u need to replace it. Except that you need to clean the vat some times after print and with LCD attach to it, you know...

\n\n

But the main reason is that when the resin is cured in the bottom layer can stick to the bottom of the vat so the printers have a solution to raise the bed and again lower it.

\n\n

The FEP (Fluorinated Ethylene Propylene) membrane is an elastic material, so when the bed raises the FEP film get raise a little sticking to the part and detaching. so we cant have this situation with the lcd because the lcd is a solid film that bends just a little.

\n\n

I'm also working on an LCD resin base printer so please post questions if you have any other questions.

\n" }, { "Id": "12108", "CreationDate": "2020-03-01T14:30:33.690", "Body": "

My custom printer is affected by a strange problem. After 30/40 min printing without any problem, PLA starts blobbing from the top of the hotend.

\n\n

\"enter

\n\n

What does it means? Where to start checking?

\n\n

It is possible that there is a little space between nozzle screw and transition screw inside the hotend?

\n", "Title": "PLA from the top of the hotend", "Tags": "|hotend|", "Answer": "

This is a typical sign, that your hotend wasn't tightened properly. Carefully heat it up and remove all the plastic you can while removing the heatbreak - that's what you call transition screw.

\n\n

After cleaning, screw in the heatbreak and the nozzle. The nozzle should butt agains the heatbreak, but have at least 2 threads to the heating block. Reassamble the full hotend now by adding the coolend.

\n\n

Finally heat the hotend on the full assembly to about 240\u00a0\u00b0C and tighten the nozzle against the heatbreak again. This is called hot-tightening. You can add the PTFE liner after the hot tightening.

\n" }, { "Id": "12114", "CreationDate": "2020-03-02T08:01:38.897", "Body": "

A question to those who have a 3D printer. Have you ever needed a spare throat or a heater block? Do they ever break?

\n\n

I just bought some spare parts: heaters, thermistors, nozzles... However, I am not sure if buying throats and heater blocks make any sense.

\n", "Title": "Do spare throats or a heater blocks ever break?", "Tags": "|diy-3d-printer|hotend|replacement-parts|", "Answer": "

Parts don't have to break or bent, they may become unusable by other issues as well!

\n\n

In addition to the answer of @Trish, if you clog your hotend, or the hotend/heater block/nozzle gets buried deep into solidified filament (see image below), it might be much easier just to replace the parts rather than salvaging the parts (or you can salvage the parts later, in the meantime you'll be up and running).

\n\n

E.g. recover from this:

\n\n

\"enter

\n" }, { "Id": "12125", "CreationDate": "2020-03-04T07:45:29.270", "Body": "

I'm using Cura 4.5.0 and an Ultimaker S5

\n\n

The walls in my model are coming out not solid. I don't have a clue why.

\n\n

\"enter

\n\n

\"enter

\n\n

When I open the STL in Sketchup it has all these weird lines. I don't know how they got there and I don't know how to get rid of them.

\n\n

\"enter

\n", "Title": "Uneven walls, not solid", "Tags": "|ultimaker-cura|", "Answer": "

Triangles?!

\n

First of all, the triangles on the STL are not a problem - they are just how STL is defined: a mesh of triangles. you can't save anything but triangles in STL, so let's not bother with that item but the actual elephant in the room: the print.

\n

Underextrusion

\n

Step 1: proper settings

\n

It shows signs of underextrusion. And I can exactly tell you where part of it comes from: You have set the line width to below the nozzle width. However, the line width should be best 10% larger than the nozzle. All of these lines should read between 0.4 to (as I have set it) 0.45\u00a0mm:

\n

\"enter

\n

With a 10% wider line planned into, it is not necessary to have an extra initial layer line width of more than 100\u00a0%, but it can help in adhesion.

\n

Step 2: Other issues?!

\n

There might be other issues at work, though they will show up after setting the width much better. The following two strike me as most likely if the problem persists with the now considerable increased flow:

\n\n" }, { "Id": "12129", "CreationDate": "2020-03-05T08:43:49.570", "Body": "

Using the following code to autotune the PID:

\n\n
M303 E0 S200 C10\n
\n\n

Which is setting my tuning temperature to 200 \u00b0C using 10 cycles (though this is irrelevant in this scenario).

\n\n

The temperature overshoots to 250 \u00b0C then decreases until room temperature.

\n\n

The following error is returned:

\n\n
PID Autotune failed! Temperature too high\n
\n\n

\"PID

\n\n

As a test I reset all PID values to zero.

\n\n
M301 P00.00 I00.00 D00.00\n\nM500\n
\n\n

Then set the printer to reach 200 \u00b0C

\n\n
M104 S200\n
\n\n

This produced the following graph, where oscillations can be seen.

\n\n

\"PID

\n\n

I could attempt to manual tune the PID from here but I'd rather use the autotune command if possible. Any ideas how to fix this?

\n", "Title": "PID Autotuning not working \u2013 large initial overshoot, no oscillations", "Tags": "|pid|", "Answer": "

I recently experienced exactly the same problem. For me it was related to an incorrect heater cartridge. I accidentally used a 12 V cartridge in a 24 V powered system.

\n\n

The heater element got accidentally mixed up between the higher voltage parts container and I forgot to check the resistance. Inserting the correct voltage heater immediately fixed the problem, but it did require a retune of the hotend.

\n" }, { "Id": "12130", "CreationDate": "2020-03-05T16:00:04.440", "Body": "

I have a Phrozen Sonic Mini printer, and of course I'm eventually going to need to replace the FEP film on the resin vat. Is there anything I need to take in to consideration when buying some replacement film? Will any legitimate FEP film I find on Amazon work?

\n", "Title": "Does it matter what kind of FEP film I use for my SLA printer?", "Tags": "|sla|maintenance|", "Answer": "

Yes*

\n

Any FEP film you find will work fine with any SLA printer you come across. Some may work 'better' or 'worse' depending on the quality of the film, but, for the most part they will all just work.

\n

The caveat is that you must ensure the film is of the correct dimension for the resin vat, If it's too small you may be unable to clamp the sheet in taut enough to get a proper seal.

\n

ASWELL, Some SLA printers do not have a standard 'sheet' style FEP, instead opting for a proprietary/semi-proprietary 'cartridge' style FEP. In these cases, unless you can find some form of 'adapter' or aftermarket upgrade, you will be unable to swap FEP films outside of said cartridges sold by your printer manufacturer.

\n

In the case of your Phrozen Sonic Mini, It looks like it uses the standard sheet-style method, You should have no problems with replacing the FEP with an aftermarket sheet.

\n" }, { "Id": "12132", "CreationDate": "2020-03-05T20:50:49.593", "Body": "

I am wondering if it is possible and safe to add a second extruder to my Anet A8 without changing the main board. I was thinking splitting my Z motor wires to free up the Z2 axis motor connector on the main board, using this connector for the second extruder motor. \nIs this possible? If so, how would i configure Marlin to use those extruder pins\nCan I overload the board by using two motor on the same connector?

\n", "Title": "Anet A8 Installing second extruder without changing board", "Tags": "|anet-a8|stepper|multi-material|", "Answer": "

That is not possible without changing to a different printer main board. The Anet A8 board has 4 integrated (A4988) stepper drivers, one for X, one for Y, one for Z and one for E (extruder 0).

\n\n

\"enter\nBoth Z steppers are controlled by a single stepper driver (they are wired in parallel to the single Z stepper driver), there is nothing to free up nor is there to configure in Marlin without replacing the main board.

\n" }, { "Id": "12134", "CreationDate": "2020-03-07T04:42:16.830", "Body": "

About a year ago, a bad print encased my hotend in PLA and I broke the wires due to my impatience trying to get the glob of PLA off.

\n\n

I don't remember the details of the disaster print, but generally the printer was working fine except I now know I was heating the bed too high and causing warped prints that I was trying to avoid by raising the bed temp.

\n\n

I'm trying to salvage the printer now, I bought a new hotend and build plate sticker. I don't want to name the supplier because I'm pretty sure these parts are irrelevant and I don't want this to seem like an advertisement. Anyways, install was pretty easy, I didn't disconnect any of the axis or feeder steppers control wires - just the hotend heater and thermistor.

\n\n

I don't think it should have anything to do with the bad print or the replacement parts, but now when I print I'm having major calibration issues. My entire print seems to be 2x scaled up. This is causing skipped stepper steps, grinding, etc for completely bad prints.

\n\n

I ran at least part of a factory reset program before any trial prints because it was recommended online somewhere (which displayed weird 50 % complete message and that's all). I most suspect the factory reset as my problem. After the reset, I needed to flip the Y axis with an additional G-code instruction before it would home to the correct corner instead of grinding to the top-back-left.

\n\n
M502 ;     Restores default settings\nM562 Y ;     Reverse Y  -- I had to add this to even get it to work as well as it does.\nM500 ;     Saves the settings to EEPROM\n
\n\n

I have read that I can adjust the axis scaling by an arbitrary factor with additional G-codes, but since I didn't mess with the printer other than the program above, I don't expect my scaling factor to be off by a random value, but more a common value that ALL mini select v2s need that I erased. But I am not finding info about this.

\n\n

I am using Cura 4.5 and Cura knows I'm using a Mini Select v2 and the model I've loaded is known to Cura to be the right dimensions.

\n\n

In the picture, you can see that Cura shows it is centered on the bed, but it actually prints off center. Also, the finished print is about 2x scaled in all three axes and lacks the material density of a correct print - it's printing only enough plastic for the correct size print, but spreading it over the scaled-up volume. The steppers are also skipping steps (I guess) so there are a few layer shifts in this print. What a mess! (It's supposed to say it's dimension on the top - \"23 mm\" but it actually measures about 46 mm but hard to say exactly due to the other print issues)

\n\n

\"enter

\n", "Title": "2x scaling of Monoprice Mini Select v2 Firmware v30.50b after factory reset?", "Tags": "|calibration|monoprice-select-mini|", "Answer": "

Thanks for the comments! They led me to find this solution using PuTTY on Windows as my serial (COM) terminal. I guess one of my resets put the settings to 1/16th stepping whereas my hardware has 1/8th stepping.

\n\n
> M115\nNAME: Malyan    VER: 3.0        MODEL: M200     HW: HA04\nok N0 P15 B15\n\n> M503\necho:Steps per unit:\necho:  M92 X93.00 Y93.00 Z1097.50 E97.00\necho:Maximum feedrates (mm/s):\necho:  M203 X150.00 Y150.00 Z1.50 E50.00\necho:Maximum Acceleration (mm/s2):\necho:  M201 X800 Y800 Z20 E10000\necho:Accelerations: P=printing, R=retract and T=travel\necho:  M204 P1500.00 R3000.00 T1500.00\necho:Advanced variables:\nS=Min feedrate (mm/s),\nT=Min travel feedrate (mm/s),\nB=minimum segment time (ms),\nX=maximum XY jerk (mm/s),\nZ=maximum Z jerk (mm/s),\nE=maximum E jerk (mm/s)\necho:  M205 S0.00 T0.00 B20000 X20.00 Z0.40 E5.00\necho:Home offset (mm):\necho:  M206 X0.00 Y0.00 Z0.00\necho:Invert axis: M562 XYZE\nXYZABCD++-+-+-\necho:PID settings:\necho:  M301 P20.00 I0.02 D250.00 C100.00 L20\necho:  M304 P10.00 I0.02 D305.40\necho:Filament settings: Disabled\necho:  M200 D1.75\necho:  M200 D0\nok N0 P15 B15\n\n> M92 X46.50 Y46.50 Z548.75 E48.50 ; from https://www.mpselectmini.com/howto/steps_per_unit_mm\n> M562 Y ; Invert Y because it was Homing to the wrong corner\n> M206 X0.00 Y-2.00 Z0.00 ; Inverting the values from M114 once good Home manually found.\n> G0 X0 Y0 ; testing the range of motion\n> G0 X0 Y120 ; testing the range of motion\n> G0 X120 Y120 ; testing the range of motion\n> G0 X120 Y0 ; testing the range of motion\n> M500 ; this saves the new values to EEPROM\n```\n
\n" }, { "Id": "12135", "CreationDate": "2020-03-07T11:04:27.693", "Body": "

The Y-axis motor is really loud. The pitch of the sound varies depending the angle of the motor (in some angles the motor is silent, in others it's louder than a fan). I am looking for a solution because this is by far the loudest component of my printer. Is there any settings that could help making this motor quieter? If not, should I replace the motor or the board?

\n\n

Here is a short clip of the sound of the motor:\n

\r\n \r\n

\n", "Title": "Y Axis motor is really loud", "Tags": "|anet-a8|motor|nema-17|", "Answer": "

The sounds are mainly caused by the bed resonating with the stepper motor, and this will vary with the speed of the stepper motor. The best solution is to use \"silent\" stepper motor drivers (e.g. Trinamic), but you may be able to reduce the noise by:

\n\n\n\n

I have done all of the above with good results.

\n" }, { "Id": "12142", "CreationDate": "2020-03-08T10:42:17.520", "Body": "

I am trying to print a box shaped object with an ELEGOO Mars SLA printer. I positioned the piece rotating it on X and Y axis 35 and 30 degrees. \nHowever, the print fails because the piece does not stick to the supporting structure (see picture).

\n\n

Do you think that it could be a problem of the piece sticking to the display due to overexposure or deteriorated film? Or is it something else? \nI tried two prints, lowering the exposure time from 8 to 6 seconds, the second print still failed but at least the piece is not totally stuck to the display.

\n\n

Any other idea?

\n\n

First failed print with 8 seconds exposure: the film was completely jammed and all the resin was stuck there (this is why there is no piece on top of the supporting material)\n\"First

\n\n

Second failed print with 6 seconds exposure: the film was perfectly clean, but the object did not stick to the supporting materials therefore collapsing. \n\"Second\n\"Second

\n\n

This is the 3D drawing\n\"3D

\n", "Title": "SLA printing: piece does not stick to supporting structures", "Tags": "|support-structures|sla|print-failure|support-material|elegoo-mars|", "Answer": "

I have tried to print the piece upside-down with the cave part facing down and it worked.\nSince the printer prints upside down, positioning the cave part facing up creates a suction-cup effect on the printer display that makes the whole structure stick to the lcd and detach from the supporting structure, thus making the print fail.

\n" }, { "Id": "12144", "CreationDate": "2020-03-08T18:38:27.507", "Body": "

I've just added a Z-Probe sensor to my custom printer (Arduino - RAMPS 1.4 - Marlin 2 default disabled EEPROM) but now I'm confused. Printer seems to work but I cannot configure the right nozzle height.

\n\n

I use Pronterface and before start printing I send G29 code. Printer succesfully ends bed calibration. At this point I start printing but the printer seems to ignore the calibration (no bed compensation).

\n\n

Do I need to run other commands to tell printer to use the previous calibration?

\n", "Title": "Marlin bed autoleveling confusion", "Tags": "|marlin|bed-leveling|z-probe|", "Answer": "

When the printer ignores the scanned topology (grid) of the bed this means that the levelling is not active. G29 activates the M420 to enable the grid. A G28 after scanning the bed will reset this.

\n\n

You need to make sure that the start G-code (this is an optional script that is pasted before the sliced object) contains both the G28 and the G29, where the G28 is placed above the G29 on separate lines.

\n\n

For each slicer this is defined in different places, bit if you use Pronterface for slicing, a good chance is that it uses the Slic3r engine. If I open an STL for slicing in Pronterface, it starts loading Slic3r.exe; this brings up the GUI for Slic3r which has options to define/modify the start G-code.

\n" }, { "Id": "12153", "CreationDate": "2020-03-10T09:20:02.260", "Body": "

Note: This question is meant as a wiki to include all steps that are needed to update your printer with a self configured Marlin firmware version. The answer is a wiki that is currently in development and can be seen (and edited) by all users.

\n\n
\n\n

I have a printer and need to update my Marlin firmware to get some stuff done to my liking, especially to make sure I have TRP, have power loss recovery and then do a PID-tune. I am also not sure if a bootloader is flashed. So what I need is a step-by-step guide to:

\n\n\n", "Title": "Updating Marlin Firmware - Step by Step Guide", "Tags": "|marlin|firmware|knowledgebase|", "Answer": "

Step 0: extracting old settings & setting up

\n\n

The first step is to get yourself a printing software that has a Console or Terminal like present in Repetier Host, Pronterface (as part of the Printrun software suite), OctoPrint or any other tool (e.g. serial connection with PuTTY also works) that allows to communicate with the printer to extract the settings we already have. Once we have the software installed and the printer connected, send M503 and copy the old settings into a file for later use.

\n\n

Next, we need our development surroundings. Usually, you want to use Arduino IDE (but the PlatformIO plugin as part of Visual Studio Code can be used for both Arduino based microprocessors as 32-bit processors), but you need to know what kind of board the control board of your printer is derived from because some boards have native IDEs that work better for them.

\n\n

Step 1: Choice of Firmware

\n\n

By some metrics of early 2020, about 80 % of all shipped machines run Marlin in some fashionneed citation. The most prolific versions of Marlin at that point are often cited to be 1.1.9 and 2.x. Since anything before 1.1.9 is very much obsolete and needs an update anyway, we will look into 1.1.9 and 2.x only. Version 2.x was developed to include 32-bit microprocessors, but is compatible with 8-bit microprocessor printer boards. As the version jump indicates though, 2.x is pretty much an entire rewrite, so do your choice and jump to the correct next step.

\n\n

Marlin 1.1.x

\n\n

Typically, you start by grabbing a blank Marlin 1.1.9. The next step is to alter the static settings of the printer to match yours in Configuration.h - best use the settings from what we pulled earlier via M503 as a start. Alternatively, you can search for a configuration of your printer between known configuration files. You should at least need to adjust these:

\n\n

For communications and filament diameter:

\n\n
#define BAUDRATE 250000\n\n// Generally expected filament diameter (1.75, 2.85, 3.0, ...). Used for Volumetric, Filament Width Sensor, etc.\n#define DEFAULT_NOMINAL_FILAMENT_DIA 3.0\n
\n\n

Choose your correct temperature tables, and make sure to turn on the one for the bed if you have one!

\n\n
#define TEMP_SENSOR_0 1\n[...]\n#define TEMP_SENSOR_BED 0\n
\n\n

Next come two blocks that set the 'this is ok' temperature area, for the hotends and bed respectively (only hotend shown here).

\n\n
// Extruder temperature must be close to target for this long before M109 returns success\n#define TEMP_RESIDENCY_TIME 10  // (seconds)\n#define TEMP_HYSTERESIS 3       // (degC) range of +/- temperatures considered \"close\" to the target one\n#define TEMP_WINDOW     1       // (degC) Window around target to start the residency timer x degC early.\n
\n\n

The next slot is an important safety feature: Mintemp and Maxtemp. Unless you seriously, positively know your hotend can do more than 275\u00a0\u00b0C (which means you have an all-metal hotend), DON'T touch the Maxtemp, but you might set Mintemp to 0\u00a0\u00b0C if you like.

\n\n

Next come PID-Tuning settings, you only need to work with those if you know what you are doing.

\n\n

The next step is important also: make positively sure that these two lines are exactly as follows, no stray // in front to comment them out. This is TRP.

\n\n
#define THERMAL_PROTECTION_HOTENDS // Enable thermal protection for all extruders\n#define THERMAL_PROTECTION_BED     // Enable thermal protection for the heated bed\n
\n\n

If your printer is a CoreXY or similarily uses 2 belts for moving along 2 axis, you look at the Mechanical Settings tab and alter it there, otherwise we skip further to the Endstop Settings. Enable (remove the leading //) the max-endstops if you have them, the rest is usually not necessary on most consumer-grade machines, then go further to the Movement Settings. From our M503 we grab the settings to fill out the following:

\n\n
#define DEFAULT_AXIS_STEPS_PER_UNIT   { 80, 80, 4000, 500 }\n#define DEFAULT_MAX_FEEDRATE          { 300, 300, 5, 25 }\n
\n\n

If you have a probe, you look into Z-Probe Options and follow 0scar's guide here, skip it otherwise until you find the next snippet. Fix that one up to fit your bed and movement area. You might need to set values for the endstop to bed origin distance. These offsets, X_MIN_POS and Y_MIN_POS, need to contain the correct values to center the bed; see \"How to center my prints on the build platform? (Re-calibrate homing offset)\n\".

\n\n
// The size of the print bed\n#define X_BED_SIZE 200\n#define Y_BED_SIZE 200\n\n// Travel limits (mm) after homing, corresponding to endstop positions.\n#define X_MIN_POS 0 // Value of zero means that the origin of the bed is at the endstop\n#define Y_MIN_POS 0 // Value of zero means that the origin of the bed is at the endstop\n#define Z_MIN_POS 0 \n#define X_MAX_POS X_BED_SIZE\n#define Y_MAX_POS Y_BED_SIZE\n#define Z_MAX_POS 200\n
\n\n

Next, uncomment (remove the leading //) the following line:

\n\n
//#define EEPROM_SETTINGS // Enable for M500 and M501 commands\n
\n\n

If you want to have a special pause position, uncomment and define it in

\n\n
//#define NOZZLE_PARK_FEATURE\n#if ENABLED(NOZZLE_PARK_FEATURE)\n  // Specify a park position as { X, Y, Z }\n  #define NOZZLE_PARK_POINT { (X_MIN_POS + 10), (Y_MAX_POS - 10), 20 }\n  #define NOZZLE_PARK_XY_FEEDRATE 100   // X and Y axes feedrate in mm/s (also used for delta printers Z axis)\n  #define NOZZLE_PARK_Z_FEEDRATE 5      // Z axis feedrate in mm/s (not used for delta printers)\n#endif\n
\n\n

We are on the finishing stretch, just a few things in this file remaining! Select your language with the line:

\n\n
#define LCD_LANGUAGE en\n
\n\n

Turn on the SD-Card slot by uncommenting

\n\n
//#define SDSUPPORT\n
\n\n

The last step we need to alter in the Configuration.h is choosing the correct LCD controller. Uncomment the line corresponding to your printer - you might need to use a generic option.

\n\n

Marlin 2.x

\n\n

Again, grab the 2.x marlin, either the blank base or a preconfigured version. For some printer styles (like Delta), you have to take a specialized set. Then we look at our M503output and set our communications Baudrate and our motherboard (or the board it is derived from), then the number of extruders and the filament diameter:

\n\n
#define BAUDRATE 250000\n\n#ifndef MOTHERBOARD\n  #define MOTHERBOARD BOARD_RAMPS_14_EFB\n#endif\n\n#define EXTRUDERS 1\n\n#define DEFAULT_NOMINAL_FILAMENT_DIA 3.0\n
\n\n

Next we go to thermal settings! We need the correct temperature sensor table for hotend and bed, possibly we could lower MINTEMP to 0. Don't touch MAXTEMP unless you know what you're doing and have a full-metal setup and you know your machine can take more.

\n\n
#define TEMP_SENSOR_0 1\n[...]\n#define TEMP_SENSOR_BED 0\n
\n\n

Our next step is making positively sure that TRP is on. Make sure these lines have no leading //

\n\n
#define THERMAL_PROTECTION_HOTENDS // Enable thermal protection for all extruders\n#define THERMAL_PROTECTION_BED     // Enable thermal protection for the heated bed\n#define THERMAL_PROTECTION_CHAMBER // Enable thermal protection for the heated chamber\n
\n\n

If the printer is a CoreXY or similar, enable the style in the mechanical settings area.

\n\n

Enable (remove the leading //) the max-endstops if you have them, the rest is usually not necessary on most consumer-grade machines, then go further to the Movement Settings. From our M503 we grab the settings to fill out the following:

\n\n
#define DEFAULT_AXIS_STEPS_PER_UNIT   { 80, 80, 4000, 500 }\n#define DEFAULT_MAX_FEEDRATE          { 300, 300, 5, 25 }\n
\n\n

If you have a probe, you need to set it up - 0scar has a partial guide - and it is all in the Z Probe Options area! Otherwise, go on. We need to go down, and in the middle of the Probe setup, we find the bed settings. Set them up to fit your printer and possibly the offset from the home-switches to the build volume corner.

\n\n
// The size of the print bed\n#define X_BED_SIZE 200\n#define Y_BED_SIZE 200\n\n// Travel limits (mm) after homing, corresponding to endstop positions.\n#define X_MIN_POS 0\n#define Y_MIN_POS 0\n#define Z_MIN_POS 0\n#define X_MAX_POS X_BED_SIZE\n#define Y_MAX_POS Y_BED_SIZE\n#define Z_MAX_POS 200\n
\n\n

Down to Additional Features we go! Let's turn on the EEPROM by uncommenting (removing the //)...

\n\n
//#define EEPROM_SETTINGS     // Persistent storage with M500 and M501\n
\n\n

...and think about how you want to set up your preheats or where to have your special park position. But then comes the last part, which we really need to do: Set up the interface. Start by changing the language and turn on the SD-Slot by uncommenting the lower of these lines:

\n\n
#define LCD_LANGUAGE en\n\n//#define SDSUPPORT\n
\n\n

Our last stop on setting up the basics is LCD / Controller Selection. We need to uncomment the right one here. If you don't find yours, use a generic one.

\n\n

Step 2: Preparing the Board

\n\n

There are 2 variants here: either you use a bootloader, or you prepare a .hex file for overwriting the whole firmware. In either case, we need to know what board we have, so we can compile with the correct encoding and setup. You might need to install a proper extension!

\n\n

2.1 - Bootloader

\n\n

A lot of boards come with a pre-flashed bootloader, which makes installing and revising software very fast. But not all boards have one flashed.

\n\n

Flashing a bootloader needs you to have an Arduino and some cables or a different ISP or AVR programming tool. Complete instructions can be found here by Greenonline and here by Robert Lo Giacco and jpa.

\n\n

Or you take your control board to your local maker space and ask someone there to help you flash the bootloader - most maker spaces have at least someone that has an Arduino and can help you!

\n\n

2.2 - .hex file

\n\n

In this case, we don't need to do anything in this step. We'll have a different installation process though.

\n\n

Step 3: Compiling & Installing Firmware

\n\n

Depending on your choice in the previous Step, you have to follow the corresponding branch here:

\n\n

3.1 - Bootloader

\n\n

If you have set your bootloader, now installing firmware is as simple as connecting your computer with the printer using a direct connection and doing a compile & Upload command.

\n\n

3.2 - .hex file

\n\n

After preparing your .hex file, you can upload it with one of the variants shown here by Greenonline, Trish or Thomas Weller

\n\n

Step 4: Finishing touches

\n\n

Seeding

\n\n

Directly after installing up our new firmware on the printer, we need to seed our settings. Connect to the printer via any Console or Terminal (see Step 0) and use these commands

\n\n
M502\nM500\n
\n\n

PID Tune

\n\n

Then we run a PID-Tune. For the first extruder we send:

\n\n
M303 E0 S200 C3\n
\n\n

It will run the machine some and return values named Kp, Ki & Kd. These directly correspond with P I & D. Store them into the EEPROM and save with the following:

\n\n
M301 P##.## I#.## D##.##\nM500\n
\n" }, { "Id": "12158", "CreationDate": "2020-03-10T17:54:59.613", "Body": "

I have a LultzBot TAZ 6 3D printer and the software I use for my prints is \"Cura LulzBot Edition - 3.6.3\". Recently, I have begun to play around with the parameters in Cura relating to initial layer adhesion. One such parameter is the Initial Layer Flow Rate.

\n\n

Now, after manipulating this parameter several times and evaluating the pieces that have been printed subsequently, it seems quite apparent to me that a manipulation in this parameter is affecting all subsequent layers that are printed.

\n\n

For example, if I increase the Initial Layer Flow Rate from the default setting of 100 % to 130 %, layer 2, layer 3,...layer n all appear to be printed at at a flow rate of 130 % resulting in prints that are clearly \"overextruded\" (e.g. the finished surfaces are incredibly uneven, bumpy, etc).

\n\n

Similarly, if I decrease the Initial Layer Flow Rate from the default setting 100 % to 70 %...all subsequent layers are clearly underextruded (e.g. walls are not connected at all and floor/ceiling have visible gaps).

\n\n

Why is this happening? Directly above the Initial Layer Flow Rate parameter is a parent setting that is simply named \"Flow\". (refer to below picture)

\n\n

\"Cura

\n\n

Do I need to manipulate flow in order to avoid this from happening?

\n\n

All I want to do is change exclusively the initial layer's flow rate (for better adhesion on larger pieces)...but my current method is not accomplishing this. Any suggestions?

\n\n

$\\color{red}{\\text{EDIT}}$ - After reading the comments, I just wanted to clarify that I have let my parts \"be printed to completion\". The obersvations that I am making about underextrusion or overextrusion are in reference to the total structure (not just the first several layers). These are large structures (~12 hour prints).

\n", "Title": "Cura Parameter Confusion: Initial Layer Flow Rate", "Tags": "|ultimaker-cura|lulzbot-taz-6|", "Answer": "

If you're over- or under-extruding by a 30% margin, there is going to be so much excess or deficiency of material in the first layer that it will take a few layers to \"catch up\". Normally you would only adjust the flow by 5% or at most 10% unless you're using it to correct for compression of the filament in the extruder gear (for flexible filaments), but adjusting first layer flow only is always a hack to work around incorrect bed height or adhesion problems. You'd do much better to figure out the right solution to whatever problem you're trying to solve with flow.

\n" }, { "Id": "12165", "CreationDate": "2020-03-11T13:30:26.667", "Body": "

I'm getting a pet bird soon, and I know that off-gassing from heating PTFE above 300 \u00b0C creates noxious fumes, which are bird-killer1. To try to prevent even the chance of that I'm replacing my hotend with an all-metal one. I have an E3D v6 1.75 mm, which I noticed still uses a PTFE tube at the cold side (the 3 mm version apparently doesn't, but 1.75 mm filament is what my printer uses, so there we are).

\n\n

If I'm reading the instructions correctly, it sounds like the heatsink shouldn't even get warm to the touch, and if that's the case I'm not worried as long as things go well. What I'm still worried about is what happens if the heatsink cooling fan fails. I plan to plug the fan into the always-on 12 V port on my board (SKR 1.3): if I do, then is it possible to set my firmware (currently Marlin 2.0.4.4) to stop a print if the fan fails?

\n\n

If I can't get the firmware to stop on fan failure, then is there a 1.75 mm all-metal hotend out there that doesn't require the use of PTFE on the cold side?

\n\n
\n\n

1- Here: \"at temperatures above 570\u00b0F (300\u00b0C), Teflon [PTFE] coatings on nonstick cookware start to break down, releasing toxic chemicals into the air (14)\"

\n", "Title": "All-metal hotend, cooling fan failures, and cold-side PTFE", "Tags": "|firmware|hotend|safety|cooling|", "Answer": "

It is doubtful that small PTFE inside hotend could produce that kind of dangerous gas leak.\nBut another thing should be considered: the PTFE tube inside hotend WILL degrade over time and will need replacement.

\n\n

For last several years I had numerous experiments with all kind of solutions including my own designs made. And recently I found the best and all metal solution: Volcano 20mm heater + M6 stainless throat + normal SHORT E3D nozzle (not the Volcano nozzle but the normal one). Basically this solution put the problematic gap (merge of nozzle with throat) deep into hotter zone, you just need longer throat. So far I have best quality and speed with this approach.

\n\n

The most important thing for the all metal hotend is to have fast retracts.

\n\n

For the FAN I recommend to use double deck, that is to put one fan on top of another and connect them in parallel.\nI found this solution in some server power supplies.\nAnd indeed two fans produce really good flow to cool heatsink at higher temperatures (before I was considering water cooling but two fans much cheaper and do the job very good). Also the probability that two fans will fail is very low.

\n" }, { "Id": "12188", "CreationDate": "2020-03-15T18:50:56.100", "Body": "

I notice that if I print from Cura without preheating the printer, it will first raise the bed temperature and then raise the extruder temperature. The G-code it generates is:

\n\n
M140 S55     ; set bed temperature to 55 C\nM105         ; report temperatures\nM190 S55     ; wait for bed temperature to reach 55 C\nM104 S210    ; set hot end temperature to 210 C\nM105         ; report temperatures\nM109 S210    ; wait for hot end temperature to reach 210 C\n
\n\n

The \"Preheat\" feature of Cura presumably send the \"set temperature\" commands without the corresponding \"wait\" command.

\n\n

Wouldn't it be more efficient to do something like this:

\n\n
M104 S210    ; set hot end temperature to 210 C\nM140 S55     ; set bed temperature to 55 C\nM105         ; report temperatures\nM109 S210    ; wait for hot end temperature to reach 210 C\nM105         ; report temperatures\nM190 S55     ; wait for bed temperature to reach 55 C\n
\n\n

Then the bed and extruder heat up simultaneously, and we wait for the higher temperature one first assuming that the other will reach its target temperature in the meanwhile.

\n\n

If this is sound, is there a way to set this in Cura, or would I need to submit a patch?

\n", "Title": "Simultaneous heating of extruder and bed at start of print", "Tags": "|ultimaker-cura|g-code|", "Answer": "
    import os\n    \n    for filename in os.listdir():\n        if filename.endswith(".gcode"):\n            with open(filename, "r") as file:\n                lines = file.readlines()\n            for i in range(len(lines)):\n                if "M140 S" in lines[i]:\n                    lines[i] = "M140 S50\\n"\n                elif "M104 S" in lines[i]:\n                    lines[i] = "M190 S50\\n"\n                elif "M190 S" in lines[i]:\n                    lines[i] = "M104 S185\\n"\n                elif "M109 S" in lines[i]:\n                    lines[i] = "M109 S185\\n"\n            lines = [line for line in lines if "M105" not in line] # remove first two M105 lines\n            with open(filename, "w") as file:\n                file.writelines(lines)\n
\n

I wrote the above Python script to edit my G-codes in bulk for a plane I'm creating. This makes both extruder and bed heat up at the same time. It changes the places the code comes up in the order that @0scar listed above. And removes the grab temp M105 lines. This makes my prints start at least a few minutes faster which is nice when you have many to do.

\n
    \n
  1. Install python3 in Windows or Linux,
  2. \n
  3. Create a file called something.py
  4. \n
  5. Paste the code above and save
  6. \n
  7. Move the script to the same folder of G-code
  8. \n
  9. Open cmd or PowerShell or Linux terminal and type python3 something.py\nor python3 ./something.py
  10. \n
\n

It will update all G-codes to heat simultaneously. For modifying yourself edit the temps I have S50 for the bed and S185 for the extruder.

\n" }, { "Id": "12191", "CreationDate": "2020-03-16T03:19:53.250", "Body": "

I'm using a 200x200 mm PCB Mk2B which connects to the MOSFET of the D8 pin on a RAMPS 1.4 shield. I used 12 V power source for heat bed so I connected positive to pin 1 and negative to both pin 2 & 3 of the bed. Heat bed worked properly. But the wires that connects power source to power supply pin on RAMPS were being heated badly. I think the problem is come from heat bed because when I unplugged heat bed, wires were cool down instantly.

\n\n

Can someone helps me with this problem. I'm just a newbie in this area.

\n", "Title": "Heat bed wires get hot (12 V connected to RAMPS 1.4 shield)", "Tags": "|heated-bed|heat-management|power-supply|", "Answer": "

Check the diameter of the cable. The smaller the diameter the higher the resistance, thus the heat. AWG14 seem a bit small.

\n" }, { "Id": "12199", "CreationDate": "2020-03-18T04:48:46.787", "Body": "

Recently I changed the FEP bottom of my Anycubic Photon S and I'm having problems with prints now. The bottom layer sticks super well, but none of the supports or the prints do.

\n\n

I've tried tensioning the vat, releveling, and changed supports of the structure. Nothing works and I don't know how to proceed.

\n\n

I also tried tightening all of the screws on the build plate, but still errors.

\n\n

I'm using Anycubic black resin, 8 bottom layers at 90 seconds, 7 second exposure with 2 seconds off.

\n\n

The image shows a failed print example:

\n\n

\"enter

\n", "Title": "Photon S DLP first layers adhere but support will not", "Tags": "|sla|dlp|", "Answer": "

I finally reached out to anycubic and their tech support answered and was awesome!

\n\n

They had me update the firmware, tighten the fep film more ( instead of using the small lever I used the big lever this time), then level the bed again. I printed their test print and it worked!

\n\n

After that I looked at their parameters and I saw that mine were very different. The main differences was in the lift distance, speed and retract speed. I only had 1 mm lift distance and much slower.

\n\n

Here are the correct ones in the end

\n\n

\"true

\n" }, { "Id": "12204", "CreationDate": "2020-03-19T11:16:06.490", "Body": "

I've got a peculiar issue today, and was looking for some help with it.

\n\n

Of the six Ender 3s I manage at my college, one of them seems to be stretching the Y axis of all the prints I make with it.

\n\n

Some points about the issue:

\n\n\n\n

The most recent print was a 40 mm fan shroud for a \"hero me\" setup (which I'm working on installing on all six printers).

\n\n\n\n

Here is a side by side of the two finished prints, comparing them along both the X and Y axes. As you can see, the X axis is perfect on the problem print, but the Y is consistently stretched (visible in the shape of the round cone and the screwholes).

\n\n

\"enter

\n\n

This issue has been consistent and I'm honestly stumped, any help is appreciated! If anyone needs me to obtain more documentation or test something, I'll be back in with the printers in the morning (roughly when this post is 10 hours old) and I'm happy to get any documentation needed.

\n", "Title": "Ender 3 Y-Axis Stretched (I don't believe it's layer shifting)", "Tags": "|creality-ender-3|troubleshooting|y-axis|dimensional-accuracy|", "Answer": "

If an axis doesn't print the sizes you command it there are basically 2 options causing this.

\n\n
    \n
  1. The printer is incorrectly configured,
  2. \n
  3. The printer has an hardware issue.
  4. \n
\n\n

To find out which of the 2 is applicable, you need to look into your setup and into the firmware settings. E.g. from the printers display you can read the amount of steps the stepper needs to make to move the axis 1 mm. Alternatively, send M92 through a terminal and look at the steps per mm. If X and Y are the same, the firmware is correctly configured and you are facing a hardware issue. Note that for the stock Ender 3 the value for X and Y needs to be 80 steps/mm. Do not alter these values!!! These values are based on the mechanical layout and the micro-stepping used by the stepper drivers.

\n\n

Since you have flashed a stock hex firmware file it is highly improbable that the firmware contains the incorrect steps per mm value for the Y axis (unless you accidentally changed this through the interface/display). This leaves you with a hardware issue. Common reasons that could identify the source of the problem are:

\n\n\n\n

Considering the stretched Y dimensions, the most logical explanation would be that your belts are stretched, you cannot fix this by changing the steps/mm value, it requires mechanical attention; i.e. replacing for new belts.

\n" }, { "Id": "12213", "CreationDate": "2020-03-20T00:42:32.287", "Body": "

I have had my Ender 3 for a few months now and have loved it! I recently noticed that the build plate, the black removable part of the print bed, is sticking up in the middle. If I poke it, it goes back into place, but will not stay there. It shows especially when I print my leveling squares and find the the corners are near-perfect, but the center is crazy thin.

\n\n

I am using a stock Ender 3 with Cura 4.5.0.

\n\n

For those who may find this to be helpful, I am looking to find a good replacement build plate. In the meantime, I managed to get mine (without the sticky back) to lay flat by increasing the temperature of the bed from 50 Celsius to 60.

\n", "Title": "Ender 3 build surface is sticking up in the middle", "Tags": "|creality-ender-3|build-surface|", "Answer": "

So your (clone) BuildTak bed surface is not sticking to the bed anymore, time to remove the surface and clean the heatbed and apply a new one. These build surfaces (usually) have a sticky 3M sticker to stick it to the bed (or the surface bottom is covered with a sticky surface). When this sticking layer fails of subsequent heating cycles you need to replace the build surface. The higher the temperature the more the build surface expands the higher the surface gets when it doesn't stick in certain places.

\n\n

Note that the build surface is a consumable, it needs to be replaced once in a while.

\n" }, { "Id": "12226", "CreationDate": "2020-03-22T12:51:32.983", "Body": "

\"3D

\n\n

So these past few days I have been 3D printing again with my Ender 3 with PETG and 0.4 mm nozzle and while I have been using the same setting as usual I am seeing an unusual amount of stringing between the prints. Does anyone know why? If you're wondering regarding the settings here they are, I'm using Cura for slicing, see options:

\n\n

\"Quality\" \"Shell\" \n\"Temperature\" \"Retraction\"\n\"Speed\" \"Cooling\"

\n", "Title": "Prints coming out very stringy", "Tags": "|ultimaker-cura|creality-ender-3|nozzle|heat-management|retraction|", "Answer": "

The travel speed of 160 mm/s is a big red flag. PETG is not tolerant of a hot nozzle moving over it at high speeds, especially unretracted (combing). The nozzle will drag material in a stuttering pattern, every so often, marring the surface and pulling what it dug up into strings.

\n\n

Lower the travel speed to the same as the print speed, and then experiment with whether you can increase it without problems. I would not try going over 80 mm/s and probably not even over 60.

\n\n

For what it's worth, this sounds like softened/molten PETG is a non-newtonian fluid, where at low stress (slow moving nozzle pushing/pulling) it deforms gracefully, but at high stress (fast moving nozzle) it strongly resists deformation and has a discontinuous breaking point. A quick Googling turned up this article, Thermal, Rheological and Mechanical Properties of PETG/rPETG Blends, which might explain what's happening.

\n" }, { "Id": "12235", "CreationDate": "2020-03-24T10:55:20.307", "Body": "

I want to make an order with this configuration

\n\n\n\n

\"stepper_motor\"

\n\n

But I am not sure what voltage and current would be enough to make it work. The Ramps 1.6+ board has an input of 12V as you can see in the picture, but I have read that other people had to increase the voltage with a DC-DC converter.

\n\n

\"ramps_voltage\"

\n\n

So, would be 12V and 15A enough to make work that configuration?

\n", "Title": "Is 12V and 15A enough to make work a Ramps 1.6 Plus and two stepper motors?", "Tags": "|stepper|arduino-mega-2650|power-supply|ramps-1.6|tmc2130|", "Answer": "

Yes, A power supply that can deliver 12 volts and up to 15 Amps will work. Since that board says 12 V, that means it is designed to work at 12V. The components on the board might not survive 24V. The TMC stepper motor drivers specified can tolerate a max of 2.5 amps. That current draw is dependent on the per phase winding impedance of the motors that are used. @2.4 ohms phase resistance, that 12 volts could generate a max of 5 amps (more than the Trinamic can tolerate). Therefore, if you configure the stepper drivers to operate at more than 50% power, they will overheat and shutdown.

\n" }, { "Id": "12236", "CreationDate": "2020-03-24T11:36:54.923", "Body": "

I want to buy the board Ramps 1.6 Plus. The description in the link says that is compatible with the driver TMC2130, but I found that there are two versions soldered in SPI mode.

\n\n\n\n

\"comparison\"

\n\n

As you can see in the image other boards like the SKR 1.3 have some socket to plug in the diagnosis pins. But I don't see something similar in the Ramps 1.6, which has male pins instead.

\n\n\n", "Title": "Will the TMC2130 V3.0 stepper driver work with the Ramps 1.6 Plus board?", "Tags": "|stepper-driver|skr-v1.3|bigtreetech|ramps-1.6|tmc2130|", "Answer": "

Well I found that I can use a DuPont cable to connect the diag1 pin to the right endstop

\n\n

\"enter

\n\n

There is also a beta functionality in the Marlin firmware I haven't tried: Just uncommenting SPI_ENDSTOPS definition

\n\n
    * SPI_ENDSTOPS  *** Beta feature! *** TMC2130 Only ***\n    * Poll the driver through SPI to determine load when homing.\n    * Removes the need for a wire from DIAG1 to an endstop pin.\n    *\n    * IMPROVE_HOMING_RELIABILITY tunes acceleration and jerk when\n    * homing and adds a guard period for endstop triggering.\n*/\n#define SENSORLESS_HOMING // StallGuard capable drivers only\n\n#if EITHER(SENSORLESS_HOMING, SENSORLESS_PROBING)\n    // TMC2209: 0...255. TMC2130: -64...63\n    #define X_STALL_SENSITIVITY  8\n    #define X2_STALL_SENSITIVITY X_STALL_SENSITIVITY\n    #define Y_STALL_SENSITIVITY  8\n    //#define Z_STALL_SENSITIVITY  8\n    //#define SPI_ENDSTOPS              // TMC2130 only\n    //#define IMPROVE_HOMING_RELIABILITY\n#endif\n
\n" }, { "Id": "12249", "CreationDate": "2020-03-26T01:26:05.873", "Body": "

I have the original Prusa i3m3 printer. Prusa recommends cleaning the bed before each print with isopropanol (isopropyl alcohol), with only occasional cleaning with acetone. The textured bed prohibits using acetone.

\n\n

Given the SARS-COV-2 situation and COVID-19, isopropanol is impossible to find, and will not be in stock on shelves in the US for months.

\n\n

What would you suggest as an alternative that might still be found on store shelves?

\n", "Title": "PRUSA likes the bed to be cleaned with isopropyl alcohol. What else should I use?", "Tags": "|prusa-i3|acetone|cleaning|bed|pei|", "Answer": "

The Prusa manual also states that you can use Windex for PET-G:

\n\n
\n

use Windex instead as it degreases less thoroughly.

\n
\n\n

Also works with PLA, I've tested it, but for assured adherence, I also use a little swipe of the glue stick included with the printer.

\n" }, { "Id": "12258", "CreationDate": "2020-03-27T13:35:13.377", "Body": "

I work with Fusion360 for designing lots of things. Recently I learned how to work with parameters that I can easily modify all at once, allowing to pretty much make easily customizable pieces.

\n\n

Now, Thingiverse wants customizer pieces in the shape of .SCAD files, and some people just can't work with Fusion360 (.F3D) or proper .STEP files that can be imported by most CAD programs.

\n\n

I have no experience with OpenSCAD. Can I import my .STEP into openSCAD, retain my parameters and export it as a .SCAD, and if yes, how?

\n", "Title": ".STEP/.F3D to .SCAD file?", "Tags": "|file-formats|openscad|fusion360|.step|", "Answer": "

I managed to get openscad-step-reader to compile, but I had to jump through a few hoops:

\n
    \n
  1. Run the apt-get install command from the beginning of the Makefile:
    \napt-get install libocct-data-exchange-dev libocct-draw-dev libocct-foundation-dev libocct-modeling-algorithms-dev libocct-modeling-data-dev libocct-ocaf-dev libocct-visualization-dev libtbb-dev

    \n
  2. \n
  3. Ignore anything to do with cmake. I can get cmake to work by creating a CMakeLists.txt file, but I couldn't get the Makefile it generates to work.

    \n
  4. \n
  5. Edit triangle.h and replace

    \n\n
  6. \n
  7. Edit openscad-step-reader.cpp and replace

    \n\n
  8. \n
  9. Edit explore-shape.cpp and add

    \n\n
  10. \n
  11. Edit the Makefile and change

    \n\n
  12. \n
  13. Run make. The .o files should be created, but the cc command will fail (after a longish time), citing 'undefined reference' 783 times

    \n
  14. \n
  15. Copy the cc command that was printed whilst running make, and put all the .o files before all the -l arguments, like this:

    \n
  16. \n
\n
cc openscad-step-reader.o tessellation.o openscad-triangle-writer.o explore-shape.o -lTKSTL -lTKXDESTEP -lTKBinXCAF -lTKXmlXCAF -lTKXDEIGES -lTKXCAF -lTKIGES -lTKSTEP -lTKSTEP209 -lTKSTEPAttr -lTKSTEPBase -lTKXSBase -lTKStd -lTKStdL -lTKXml -lTKBin -lTKXmlL -lTKBinL -lTKCAF -lTKXCAF -lTKLCAF -lTKCDF -lTKMeshVS -lTKOpenGl -lTKV3d -lTKService -lTKXMesh -lTKMesh -lTKOffset -lTKFeat -lTKFillet -lTKHLR -lTKBool -lTKBO -lTKShHealing -lTKPrim -lTKTopAlgo -lTKGeomAlgo -lTKBRep -lTKGeomBase -lTKG3d -lTKG2d -lTKIGES -lTKSTEP -lTKSTEP209 -lTKSTEPAttr -lTKSTEPBase -lTKXSBase -lTKStd -lTKStdL -lTKXml -lTKBin -lTKXmlL -lTKBinL -lTKCAF -lTKLCAF -lTKCDF -lTKMeshVS -lTKOpenGl -lTKV3d -lTKService -lTKXMesh -lTKMesh -lTKOffset -lTKFeat -lTKFillet -lTKHLR -lTKBool -lTKBO -lTKShHealing -lTKPrim -lTKTopAlgo -lTKGeomAlgo -lTKBRep -lTKGeomBase -lTKG3d -lTKG2d /usr/lib/x86_64-linux-gnu/libTKMath.so.7.5.2 /usr/lib/x86_64-linux-gnu/libTKernel.so.7.5.2 -lfreetype -lpthread -lrt -lstdc++ -ldl -lm   -o openscad-step-reader\n
\n

Now, for me, the program will compile. My system is running Ubuntu Focal Fossa 20.04.6 LTS. I contacted the author (Gordon Assaf) but he said he was no longer working on the project. For me, this has been a godsend, since it allows me to import .step files downloaded from the Swagelok website into my OpenSCAD design.

\n" }, { "Id": "12263", "CreationDate": "2020-03-27T21:18:32.310", "Body": "

I'm having an issue with my Prusa i3 MK2 MMU.

\n\n

It starts printing just fine, and everything gets up to operating temp quite quickly. However, after printing the first few layers, it seems as if the heated bed can't keep up with the demand, and its temperature starts to drop, first it can barely hold 80 °C and then it slowly drops down to 77-76 °C before the printer shuts off due to BED THERMAL RUNAWAY.

\n\n

The LED is continuously on for the bed, so it seems to be supplying power when this happens. Setting the printer down from high power to \"silent\" helps a bit, so could this be that it is not getting enough power?

\n\n

Ambient temp is around 17 °C.

\n", "Title": "Prusa i3 MK2 Heated Bed Thermal Runaway Problems", "Tags": "|prusa-i3|heated-bed|heat-management|", "Answer": "

The local library makerspace (prior to COVID-19 shutdown) has a Prusa i3 MK2 (no MMU) that had a similar problem. The model being printed was relatively small, compared to the bed and was being printed nearly atop the temperature sensor.

\n\n

The part cooling fan was cooling the bed sensor while the rest of the bed was not being used.

\n\n

Moving the model away from the sensor resolved the problem. If you're printing in the center of the bed, consider to re-slice in a position offset from the center.

\n\n

If you are using the PEI sheet as the library machine is/was, consider to shuttle your prints around the bed over time. The library's printer wore away the PEI from always-centered prints. It had to be replaced after about a year. The replacement is going on two years with little complications.

\n" }, { "Id": "12265", "CreationDate": "2020-03-28T11:25:38.903", "Body": "

I want to add an external SD card adapter on my RepRapDiscount Full Graphics LCD. My 3D printer is based on (Arduino Mega + RAMPS v1.4 + RepRapDiscount Full Graphics LCD). \nI buy an SD Card Adapter from Aliexpress (https://www.aliexpress.com/item/32873382195.html?spm=a2g0s.9042311.0.0.4f544c4d5UzB1L) and now I want to add this adapter on my RAMPS 1.4 while not disturbing the working of RepRapDiscount Full Graphics LCD. \nPlease guide me how can I add this to my RAMPS? Thanks!

\n\n

My 3D Printer Details:

\n\n\n\n

\"enter\n\"enter

\n", "Title": "Makerbase MKS Slot2 V1.0 + RepRapDiscount Full Graphics LCD + RAMPS 1.4", "Tags": "|diy-3d-printer|ramps-1.4|reprap|full-graphic-smart-controller|", "Answer": "

A RepRapDiscount Full Graphics LCD already has an SD card slot reader at the back of the board:

\n\n

\"RepRapDiscount

\n\n

Basically one of the 10 pin headers is used for the display and rotary encoder and reset, the other header is used by the SD card reader.

\n\n

The module referenced in the question is a breakout SD card slot reader for the TFT (touchscreen) display controllers. Whether these are compatible with the EXP2 header of the RepRapDiscount Full Graphics LCD should be compared with the schematics. The schematics of the RepRapDiscount Full Graphics LCD are available, but for the SD slot breakout cannot be found. It is therefore unsure if the EXP2 is fully compatible for the SD slot board. To use the EXP2 on a RAMPS shield, you need the L-shaped connector.

\n\n

The description of the Slot2 explicitly says that it is meant to be connected to either a TFT35 or a TFT70 MKS touchscreen display. In order to use it directly to your RAMPS shield requires finding out which pin goes where.

\n" }, { "Id": "13269", "CreationDate": "2020-03-29T09:55:18.983", "Body": "

I'm new to 3D printing and have bought an Ender 3. I have printed a few good prints but I'm noticing a worsening issue:

\n\n

When I select \"autohome\" the axes head towards their limit switches, but the y axis in particular seems to slam into the limit switch, bending it away, meaning that the platform bounces off and doesn't activate the limit switch a second time

\n\n

This causes the machine to slam against the back over and over until the limit switch is triggered or power is removed.

\n\n

I've replaced the limit switch twice

\n\n

I've tried supergluing the limit switch to its PCB but even with a needle and patience this caused the limit switch to be ruined

\n\n

What can I do?

\n\n

Edit: Here are some photos of the switch (2nd replacement). The OEM switch also did the same thing, but I don't have photos of that. The screws are loose in these photos, but this is just because my Allen key is lost - the previous two switches had the screws reasonably tightened with the correct Allen key, provided in the box\"enter

\n\n

\"enter

\n\n

\"enter

\n", "Title": "Homing damages limit switches Ender 3", "Tags": "|creality-ender-3|troubleshooting|homing|", "Answer": "

The solution seems to be simply triggering the switch earlier, and making sure that it is triggered. The thin arm can end up bent over the actual button of the switch, meaning that the arm can be touching the switch without actually "pressing" the switch. For some, bending the arm seems to have worked, but for me I had to wedge a small piece of plastic between the arm and the switch button, meaning that it is a lot more reliable when you press the switch. The piece isn't stuck in there in any way, simply held by friction. This isn't the best solution as you have to keep an eye on that piece with every homing, but until a better answer is available, I recommend this solution as it was more reliable than bending the arm out or into a different shape.

\n" }, { "Id": "13275", "CreationDate": "2020-03-29T23:00:04.660", "Body": "

I'm new to 3D Printing and recently purchased an Ender 3D PRO I'm having an issue with the filament guide tube getting pushed out of the nozzle on the feeding mechanism. The assembly instructions don't include a whole lot of detail about installing this guide tube but there are blue clips that were included along with the spare nozzle. There are no instructions on where to use these blue clips and I have a hunch this might be the problem.

\n", "Title": "Filament feed tube", "Tags": "|extruder|creality-ender-3|bowden|", "Answer": "

The clips or collet clips as they are called are to secure the Bowden tube more solidly to improve printer extrusion; more specifically: extruder retraction performance.

\n\n

E3D has explained this very nicely on their site under Bowden tube physics:

\n\n
\n

For the bowden couplings (which can have some wiggle room in them) we are introducing collet clips, which slide under the toothed collet part of the coupling that physically holds the tube and locks it into place with a little pre-tension to boot. This reduces coupling lash to near zero as the collet and tube it is holding are locked into place.

\n
\n\n

\"Bowden

\n\n

From the animated gif from the E3D website (animated gifs are not allowed on SE) it can be seen that without clips, the tube can move in the tube coupling:\n\"enter\"enter

\n" }, { "Id": "13282", "CreationDate": "2020-03-31T01:07:29.873", "Body": "

Recently my RAMPS power connector caught fire while heating the heated bed. I suspect this was a defect caused by the connector, and de-soldered/removed it as best I could. I'm not an electrical engineer, so I'm looking for advice on whether It is possible to re-solder the power connectors directly onto the board, or I'm just risking another fire. Here are pictures of the top and bottom.

\n

\"Top

\n

\"Underside

\n", "Title": "Can I resolder a power connections with a partially burnt board", "Tags": "|ramps-1.6|", "Answer": "

As MIck wrote, we hope your board isn't damaged - with any luck there aren't any trace shorts in the melted region.

\n\n

My approach would be slightly different. Use some short bits of wire soldered into the board at one end and soldered to a new connector socket at the other end. This way you can put a new plug connector on the power leads and be able to disconnect the board as desired.

\n" }, { "Id": "13290", "CreationDate": "2020-03-31T15:51:25.697", "Body": "

I followed this tutorial on how to set up BLTOUCH v3.0 with my Ender 3 Pro. (v1.1.4 motherboard)

\n\n

I even removed C7 capacitor on Z axes. And no luck. My BLTouch probe is working, it turns on with the Ender and testing is fine. But it doesn't stop the printer on Z axis while auto homing.

\n\n

Firmware is from firmware.th3dstudio.com

\n\n

Any ideas? Any advice is highly appreciated.

\n\n

My configuration.h file: https://paste.ofcode.org/Z4kKiGwYXC6yQ698t9uaeY

\n\n

My configuration_backend.h file: https://paste.ofcode.org/34tuX4mG3Ph57jSnT8YByi3

\n", "Title": "Problem with Creality v1.1.4 (Ender 3 Pro) and BLTOUCH v3.0 (removed C7 capacitor)", "Tags": "|creality-ender-3|bltouch|", "Answer": "

If the probe works, but not on auto homing, it could be that the Z endstop/probe wires are incorrectly wired.

\n\n

From \"The complete BLTouch/3DTouch guide for Creality printers (CR-10/s,Ender 2,Ender 3) for Auto Bed Leveling UPDATED\" can be read:

\n\n
\n

If you connect your bltouch and when auto homing the sensor doesn\u2019t register\n & the z axis keeps dropping pressing into your bed, turn your printer off,\n disconnect the power and invert the black and white connectors in the z limit\n switch header (put black where white was, put white where black was).

\n
\n\n

This can also be found in this Marlin issue tracker:

\n\n
\n

The Problem was the polarity of the sensor wires was reversed.

\n
\n\n

If reversing the polarity does not work, move to the original Marlin firmware.

\n" }, { "Id": "13296", "CreationDate": "2020-04-01T18:13:20.377", "Body": "

I have read that the cable colours depend on multiple factors:

\n\n\n\n

My cable looks like this:

\n\n

\"main_cable\"

\n\n

I live in Spain, in Europe. The colour codes of the International Electrotechnical Commission for AC circuits are these ones:

\n\n

\"enter

\n\n

My PSU has these possible connections

\n\n

\"front_psu\"

\n\n

So I guess these are the connections:

\n\n\n\n

But I am not sure about what are the COM and the V ports. I assume they are for the DC connections, which is the positive and which the negative? The PSU does not have any manual and I am little lost

\n\n

Should I turn the potentiometer to change the voltage to adapt it to the Ramps 1.6 Plus board?

\n\n

\"ramps\"

\n\n

Please correct me if I made some mistake in my question as well

\n", "Title": "Which connectors should I use to connect my Power Supply Unit to the Ramps?", "Tags": "|power-supply|color|wire-type|", "Answer": "

You're quite right :)

\n\n\n\n

The way to go about these things is to connect the AC side and to connect a multimeter to the output terminals (V+ and COM, there are two of them in case you need to wire up more than one connection). I like to lightly tighten down the probes under the terminals, that way you don't have to mess with the connections with the mains terminals exposed.

\n\n

Now you can plug in the power supply (PSU) and make sure it doesn't start smoking or sparking.

\n\n

Verify that the PSU is outputting DC 12V (that seems to be the voltage required by your Ramps board according to the picture). After that, you can connect it to the Ramps board (remember to disconnect the PSU before rewiring). If the voltage is anything but around 12 V (+/- 0.5VDC should be safe), you need to verify that the PSU is the correct type for your application.

\n" }, { "Id": "13300", "CreationDate": "2020-04-02T02:20:31.650", "Body": "

I recently build a 3D printer from scratch and was using an Arduino Mega + RAMPS 1.4 to control it. A few weeks in I installed a heated bed to help my prints stick to the surface. The bed I bought was an Anet bed found on Amazon, I'm told they are prone to causing fires.

\n\n

About a week after I installed the bed, my the MOSFET on my RAMPS overheated and began to smoke. I could no longer perform without heating up too hot and smoking, so I bought another RAMPS 1.4 to replace it. The same thing happened and I returned it...

\n\n

After doing some research and finding out the connectors and MOSFETs on the RAMPS 1.4 are typically undersized, I decided to upgrade to avoid the problem and go with a RAMPS 1.6 instead, which was supposed to better handle the current flow. After about a week of using the RAMPS 1.6, the power connection began to smoke and caught fire(Not the MOSFETs like the RAMPS 1.4, the incoming power connector). I attempted to remove the connector and resolder the wires, I couldn't get it to work. At this point I'm looking for advice on which controller to go with. I don't want to buy another RAMPS out of fear of the same thing happening, but I am also wondering if buying a different heated bed would fix the problem. So any recommendations on where to go from here? I've look at a few boards but one thing is some of them can only control 1 Z stepper driver, while my setup has Z.

\n", "Title": "Which 3D printer controller should I use?", "Tags": "|heated-bed|ramps-1.4|ramps-1.6|mosfet|", "Answer": "

Controller boards

\n

Please note (this is not to bash) that RAMPS shields are not the top of the line printer controller boards, investing in a more modern printer board platform (preferably not a clone of a known board), e.g. a 32-bit board might be a better solution.

\n

Any board with at least four stepper drivers should be sufficient (you can use the two Z steppers in parallel or in series), some boards even offer two Z stepper output connectors controlled by a single stepper, e.g.:

\n

\"Two

\n

No worries if the board doesn't have two Z stepper connectors, you can buy dual stepper breakout boards, e.g. a parallel solution:

\n

\"Dual

\n

Alternatively, an extra stepper driver can drive the extra Z stepper; this requires a board with at least 5 stepper drivers then, the Z2 stepper would then be connected to the E1 extruder driver. The firmware needs to be aware of using 2 stepper drivers, so in Marlin (depending on the version) set Z_DUAL_STEPPER_DRIVERS or assign a value to NUM_Z_STEPPER_DRIVERS .

\n

Anet heatbed

\n

The Anet heatbed is notoriously known for fire accidents/burning caused by the underrated connectors (the connector itself is not rated for 10\u00a0A!), also the connector should require proper strain relief. The best solution is to solder the heat bed wires directly to the backside of the connector (which is what I did on my old Anet A8 printer, I used multiple pins soldered together, both the outer 2 pins on either side), an example from the web (downside is that they only used a single pin) shows this principle (with solution for cable/soldering stress relief using a tie-wrap):

\n

\"Anet

\n

A final notice for connecting heatbeds is to use proper cables; silicone cables of proper gauge (silicone wires are very flexible, e.g. AWG 14) should be used. Also, never solder the ends and put them under screw connectors on the printer board, instead, use ferrules or proper sized fork terminal connectors.

\n

MOSFET Band Aid

\n

In case the MOSFETs on your boards are underrated, you can use external MOSFET boards to relieve the shield/printer board from the high currents. Note that the bed requires the most current (about 10\u00a0A), for the hotend this will not be required, the current draw is much lower, the onboard MOSFETs are rated for those loads.

\n

\"MOSFET

\n" }, { "Id": "13303", "CreationDate": "2020-04-02T06:45:25.030", "Body": "

I've just updated my Anet A8 with the blue bed level sensor to Marlin 2.0 .\nBoth pressing the Level Bed button on the display or using G29 after G28 in the G-code don't do anything.

\n\n

G28 Homing works fine, but doesn't put the nozzle at the center like it did on Marlin 1.1.x, instead it puts the sensor at the center of the build plate.\nG29 worked fine in my previous Marlin 1.1 config.

\n\n

These are my config files:

\n\n\n", "Title": "G29 Bed Level not doing anything on Marlin 2.0", "Tags": "|marlin|firmware|anet-a8|bed-leveling|", "Answer": "

From the linked Configuration.h file the probe X, Y, Z probe offset is set by constant array:

\n\n
#define NOZZLE_TO_PROBE_OFFSET { 75, -35 , 0 }\n
\n\n

So, the sensor is mounted at the right-front (X+, Y- according to the Marlin configuration definition) when facing the printer.

\n\n

This implies that the sensor is limited on the right and at the front.

\n\n

The probing area used to be defined in Marlin 1.1.x in the Configuration.h file. However, Marlin 2.x requires edge offsets rather than absolute bed size constraints. From the Configuration_adv.h of linked file, the following probing limits are set:

\n\n
#if PROBE_SELECTED && !IS_KINEMATIC\n  #define MIN_PROBE_EDGE_LEFT (75 + MIN_PROBE_EDGE)\n  #define MIN_PROBE_EDGE_RIGHT (X_BED_SIZE  - MIN_PROBE_EDGE)\n  #define MIN_PROBE_EDGE_FRONT (MIN_PROBE_EDGE)\n  #define MIN_PROBE_EDGE_BACK (Y_BED_SIZE -35 - MIN_PROBE_EDGE)\n#endif\n
\n\n

This is incorrect, this is what you would do in Marlin 1.1.x. Note that this answer describes in detail how to set the bed probing limits. You need to specify the offset from the edge on each side, in schematics the probing area is defined as:

\n\n

\"Front-right

\n\n

From your printer configuration, the probing limits should be set to:

\n\n
#if PROBE_SELECTED && !IS_KINEMATIC\n  #define MIN_PROBE_EDGE_LEFT (75 + MIN_PROBE_EDGE)\n  #define MIN_PROBE_EDGE_RIGHT (MIN_PROBE_EDGE)\n  #define MIN_PROBE_EDGE_FRONT (MIN_PROBE_EDGE)\n  #define MIN_PROBE_EDGE_BACK (35 + MIN_PROBE_EDGE) ; Note that 35 is absolute(-35)!\n#endif\n
\n" }, { "Id": "13310", "CreationDate": "2020-04-02T18:06:33.543", "Body": "

So, I have created my own heater block for my 3D printer and mistakenly was using a 12 V, 40 W heater cartridge thinking that was the standard for my Ender 3, when in fact it is supplied with a 24 V, 40 W.

\n\n

You may have seen my earlier post about when using autotune the temperature would overshoot by a large margin and returned the following error.

\n\n
PID Autotune failed! Temperature too high\n
\n\n

Graph of Overshoot:

\n\n

\"PID

\n\n

I was unable to remove this overshoot even through manual tuning.

\n\n

Now I have switched from the 12 V cartridge to the 24 V this problem is resolved, and I can now run the autotune. It follows a much more gradual curve when heating up so doesn't trigger the same error.

\n\n

However, I am unsure why this is the case? Can anyone explain why the 12 V heater cartridge results in too high of a temperature increase?

\n\n

It seems counterintuitive to me as I would have thought the higher voltage cartridge would heat up faster as opposed to vice versa?

\n", "Title": "PID autotune fails 'Temp too high' with 12 V heater cartridge but works with 24 V?", "Tags": "|creality-ender-3|hotend|pid|", "Answer": "

It is all about resistance.

\n\n

This requires some formulae:

\n\n

$U = Voltage$
\n$I = Current$
\n$R = Resistance$
\n$P = Power$

\n\n

$U = I \\times R$

\n\n

$P = U \\times R$

\n\n

$ R = \\dfrac{P}{I^2} = \\dfrac{U^2}{P} $

\n\n

The 12 V, 40 W cartridge has a resistance of about 3.6 Ω.

\n\n

If you use this cartridge at 24 V, this caculates to a power of 160 W!

\n\n

This means that there is an enormous influx of heat that is hard to control, hence the overshoot.

\n" }, { "Id": "13318", "CreationDate": "2020-04-03T03:14:04.910", "Body": "

I bought an Ender 3 recently. Auto-home is on the right front moving away from the end stop. I did reverse the wiring of the X axis motor, it did not work. I had Marlin 2.x uploaded, it didn't work too.

\n

Marlin 2.x:

\n\n

The print starts off with a boundary line in the middle and goes off the bed on the right corner to print. Y and Z axes are fine. X axis seems to be bumping into the right front, every time while homing. I had tweaked little bit of Marlin, but I'm a beginner and I don't understand it completely.

\n

I'm using Cura, printer settings, max X=235, max Y=235, max Z=250, origin at the center: unchecked.

\n

This might help...

\n
// The size of the print bed\n#define X_BED_SIZE 235\n#define Y_BED_SIZE 235\n\n\n#define X_MIN_POS 0\n#define Y_MIN_POS 0\n#define Z_MIN_POS 0\n#define X_MAX_POS X_BED_SIZE\n#define Y_MAX_POS Y_BED_SIZE\n#define Z_MAX_POS 250\n\n\n#define MANUAL_X_HOME_POS 0\n#define MANUAL_Y_HOME_POS 0\n#define MANUAL_Z_HOME_POS 0\n
\n

(left this after so many trails)

\n

In Pronterface the mid point X117.5 is at the middle right corner.

\n

I'm thinking the printer is behaving like the origin(0,0) is on the right front, for X at least and it has nothing to do with the slicer.

\n

It's about centering the prints, but it doesn't print on the bed mostly.

\n
\n

Start G-code:

\n
; Ender 3 Custom Start G-code \nG92 E0 ; Reset Extruder \nG28 ; Home all axes \nG1 Z2.0 F3000 ; Move Z Axis up little to prevent scratching of Heat Bed \nG1 X0.1 Y20 Z0.3 F5000.0 ; Move to start position \nG1 X0.1 Y200.0 Z0.3 F1500.0 E15 ; Draw the first line \nG1 X0.4 Y200.0 Z0.3 F5000.0 ; Move to side a little \nG1 X0.4 Y20 Z0.3 F1500.0 E30 ; Draw the second line \nG92 E0 ; Reset Extruder \nG1 Z2.0 F3000 ; Move Z Axis up little to prevent scratching of Heat Bed \nG1 X5 Y20 Z0.3 F5000.0 ; Move over to prevent blob squish\n
\n

End G-code:

\n
G91 ;Relative positioning \nG1 E-2 F2700 ;Retract a bit \nG1 E-2 Z0.2 F2400 ;Retract and raise Z \nG1 X5 Y5 F3000 ;Wipe out \nG1 Z10 ;Raise Z more \nG90 ;Absolute positionning \nG1 X0 Y{machine_depth} ;Present print \nM106 S0 ;Turn-off fan \nM104 S0 ;Turn-off hotend \nM140 S0 ;Turn-off bed \nM84 X Y E ;Disable all steppers but Z\n
\n", "Title": "Creality Ender 3 X axis homing issue", "Tags": "|homing|x-axis|print-axis-offset|", "Answer": "

I realize this is an old post, but in the event someone is having a similar problem and this post turns up in a search....

\n

All three axis-stop switches are normally closed, and go open when the axis carriage hits the little lever. What this switch is doing is providing an easy low-resistance path to ground for the 3.3 (or 5) volt control card rail. You have the rail feeding this circuit through a 10K resistor (or some other relatively high value resistor), then the path forks; one fork to the limit switch, the other through a 100 ohm resistor to the microprocessor pin that handles the stop input. As long as that limit switch is closed and the circuit is good, the 3.3 volts goes through it to ground and the microprocessor never sees the 3.3 volts, the pin is "LOW." Once that switch is opened, the easy path to ground is removed, and the 3.3 volts goes to the microprocessor, and the pin goes "HIGH." If that switch or harness wiring has failed in a manner that opens that branch of the circuit, the card thinks the stop is already tripped, and won't allow motion towards the switch.

\n

You can verify this easily enough by putting a meter on the two contacts in the Z-stop plug where it plugs into the control card (remove it from the card first so you aren't reading the card too.) Using the continuity function (buzzer or just reading ohms), you should get straight continuity with the switch at rest, and it should go open (infinite ohms) when you push the lever.
\nThis checks both the switch and the harness wiring. If you don't see that state-change from continuity to open when you push the lever, go up to the switch itself and pull the wires off it, and read the switch pins directly with the meter. Same deal, should read continuity when in the at rest state, and go open when you push the lever. If it does, then the harness wiring is bad. If not, the switch is junk or failing, soon to be junk.

\n

A LOT of people have a LOT of trouble adding a BLTouch to their machines, with many reporting that the gantry won't go down no matter what they do short of turning the Z-screw(s) by hand. This is a direct result of removing the Z-stop switch wiring and/or switch. That easy route to ground was removed when the switch was disconnected/removed, and even worse, now the microprocessor has conflicting inputs as to the status of the Z axis, one input from the BLTouch telling it the Z-axis is untripped, and the missing switch is telling it that it's already tripped... therefore, no downward motion, only up.

\n

To fix this you have to edit the firmware (configuration.h) in the section covering the state of the stop switches. They are all defaulted to "HIGH," meaning that if the microprocessor sees that voltage, it's tripped. Remember, anything that opens that circuit makes the associated pin to high (including disconnecting the Z-stop wiring from the control board, OR removing the switch and mounting bracket completely.) Doesn't seem to matter if you use the five-pin dedicated Z-probe socket for all five wires or put the black and white wires in the Z-stop socket and the other three in the Z-probe socket. You have to change the "tripped state" of the Z-axis in firmware from "HIGH" to "LOW," then build the new firmware and flash the control card with it. I beat my head against that BLTouch for nearly four months before I stumbled across this little factoid that nobody else was talking about (not anywhere I was looking, anyway, which was pretty much everywhere.)

\n

Schematics are a HUGE help (for some things) if you can read them!

\n" }, { "Id": "13330", "CreationDate": "2020-04-03T16:52:31.170", "Body": "

My printer is calibrated for a certain clearance from the heated bed, which is chosen based on PLA.

\n\n

I would like to try to increase it for the first layer for PETG, so that adhesion is reduced. The printer has M5 screws with 4000 steps/mm, so the resolution clearly allows that.

\n\n

I don't want to modify the printer, I would like a G-Code or another option that I can apply in Prusa slicer in association with the specific filament when desired. I don't want to change the flow rate of the first layer, only the \"zero\" distance.

\n\n

How can I do that?

\n", "Title": "How can I add an offset to the first layer to increase clearance?", "Tags": "|g-code|adhesion|petg|", "Answer": "

If your environment supports it, you can also use a G54 to add a constant offset to any positioning. So, you could use:

\n\n
G53 ; clear offsets  \nG54 Z0.15 ; add 0.15mm to every Z position\n
\n\n

Just remember to clear the offsets at the end of your gcode:

\n\n
G53 ; clear offsets\n
\n" }, { "Id": "13331", "CreationDate": "2020-04-03T21:17:40.410", "Body": "
Printer: Prusa i3 clone / Anet A8\nArduino IDE: Ver 1.8.12\nFirmware: Marlin-2.0.2\nanet-board-master: 1.5.6 (from hardware/anet/avr/platform.txt 2018-03-02)\nConfigurations-relese: 2.0.5 (Configuration.h, -adv.h)\nOS: Windows 10\nBoard selection: Anet V1.0\n
\n\n

I successfully upgraded the stock firmware to a version with bootloader, calibrated and had it running. I started to add features one at a time, recompile and upload. I managed to break it while trying to improve the LCD button debounce which I increased to 25 ms:

\n\n
#if HAS_ADC_BUTTONS\n  #define ADC_BUTTON_DEBOUNCE_DELAY 16  // (ms) Increase if buttons bounce or repeat too fast\n#endif\n
\n\n

The compile worked, transferred, showed the splash screen and went blank. This repeated on power-cycle. I reversed the change but was unable to restore operation.

\n\n

I have tried reinstalling Arduino IDE, copying the folders in again and compiling with the default configurations. I'm getting lots of errors and the compiler exits with:

\n\n
C:\\Users\\username\\AppData\\Local\\Temp\\arduino_build_938465\\sketch\\src\\MarlinCore.cpp:470:11: note: in expansion of macro 'disable_Z'\n       disable_Z();\n       ^~~~~~~~~\nexit status 1\nError compiling for board Anet V1.0.\n
\n\n

Can anyone give me some guidance on how to get going again?

\n", "Title": "Unable to compile Marlin for Anet A8", "Tags": "|marlin|anet-a8|", "Answer": "

I'm almost back in business.

\n\n
    \n
  1. I reinstalled the bootloader.
  2. \n
  3. I switched from Arduino IDE to Visual Studio Code to compile using Crosslink's Anet A8 (Plus) Marlin 2.0 Installation Upgrade YouTube tutorial.
  4. \n
\n\n

1. Bootloader

\n\n

I'm using a Chinese USBasp and there are some confusing tutorials on the web which require upgrading the USBasp firmware using another USBasp or Arduino. I avoided this as follows:

\n\n\n\n

Download and install AVRDUDESS if you haven't got it. I used Ver. 2.11. Connect the USBasp to the Anet board and your computer.

\n\n\n\n

You now have an Anet board with the OptiBootLoader installed.

\n\n

Now follow Crosslink's video tutorial above using the normal USB connection.

\n" }, { "Id": "13337", "CreationDate": "2020-04-04T11:25:14.633", "Body": "

While I was having fun with my ender I burn out UP1 Chip.

\n\n

Does anyone know which chip is that (I can't see because of burnt) and where can I order it?

\n\n

Image: Damaged chip

\n", "Title": "Creality v1.1.5 board replacement UP1 Chip", "Tags": "|creality-ender-3|", "Answer": "

It's an MP1584 chip. \nGuys on Reddit helped me out.

\n\n

link

\n" }, { "Id": "13353", "CreationDate": "2020-04-06T05:40:58.337", "Body": "

I've noticed an interesting behavior on my Ender 3 with SKR Mini E3 mainboard and Marlin 2.0.x bugfix firmware. (otherwise, all other hardware is entirely stock) After the hotend/bed are commanded to cool down, e.g. after a print completes, I have to wait until after they both cool down to ambient before commanding another temperature setpoint.

\n\n

If I don't do this, the printer most often triggers thermal runaway protection. (usually citing the extruder, but also sometimes the bed) I think this might be due to the thermal inertia in the material between the heater and thermistors, causing a 5-10 second delay in sensed temperature rise. I don't see any reason why thermal runaway should trigger; the Octoprint temperature graph looks completely normal, with no perceptible anomalies.

\n\n

Is there some way to tune parameters for thermal runaway protection to alleviate this false-positive situation?

\n", "Title": "Thermal runaway triggers when raising temperature amid cooldown", "Tags": "|marlin|creality-ender-3|troubleshooting|skr-mini-e3|", "Answer": "

Yes there are ways to tune false positives. For Marlin firmware you can find these option in the Configuration_adv.h file.

\n\n

From the file itself you can read what you need to do:

\n\n
\n
 * If you get false positives for \"Thermal Runaway\", increase\n * THERMAL_PROTECTION_HYSTERESIS and/or THERMAL_PROTECTION_PERIOD\n */\n
\n
\n" }, { "Id": "13358", "CreationDate": "2020-04-07T09:19:21.100", "Body": "

I'm trying to print a lithophane with my Ender 3, but the nozzle gets closer and closer to the previous layer, layer after layer.

\n\n

I thought that what was causing the problem was perhaps a gear lock due to the fact that the Z rod wasn't parallel to the vertical axis, but after fixing it the problem remains.

\n\n

The bed is super planar and perfectly levelled and, in fact, the first layer comes out perfectly.\nExtrusion is also okay. The printer seems fine in all aspects.

\n\n

It seems like the seriousness of the problem is proportional to the number of layers, just like the printer is losing a fixed height by each layer.

\n\n

I've ruled out the possibility of wrong calibration of Z steps because after measuring a cube of 12 cm of height and telling the printer to raise 12 cm the cube fits perfectly under the nozzle with the levelling gap (so perfect calibration).

\n\n

What do you think may be the problem?

\n", "Title": "Ender 3 nozzle gets closer and closer to the previous layer as the print progresses", "Tags": "|creality-ender-3|z-axis|", "Answer": "

Finally I've found the damn problem: Adjustable rollers of all axes were flat on one point making layers to shift in all directions.

\n\n

The effect was less visible on the X and Y axes but it was more notable in the Z axis as the flat part was right from the start of the print. I think that flat part was making difficult for the Z axis to move upward, and making my prints fail when printing at higher resolutions (I guess the finer the displacement the weaker the torque).

\n\n

It seemed to me that the nozzle was getting closer to the previous layer but instead it was resting (or barely moving) on the flat part of the Z right adjustable roller first and then on the other adjustable roller of the same axis.

\n\n

To diagnose the problem disable the steppers and moved the axes manually and feel if there are notches, so to speak, in various spots on every axis.

\n\n

The solution is to replace the rollers with new ones and to not close them too tight otherwise they'll deform over time.

\n" }, { "Id": "13366", "CreationDate": "2020-04-08T08:28:42.607", "Body": "

For some time I've been struggling to print models with holes going all the way through from top to bottom. Models without those holes print out just fine. I've spend quite some time on levelling the bed, so I think the levelling is fine. I'm using Cura 4.5 with the default settings (normal profile) for the Anycubic I3 mega.

\n\n

At first I had the problem of the initial circles not sticking to the bed. This is what it looked like:

\n\n

\"lower

\n\n

Then (after doing a lot of searching on the internet) I changed the following settings in Cura:

\n\n\n\n

After that the circles seemed to stick better, but still the bottom layer looks quite messy. This is a picture of the bottom of the object printed with these settings:

\n\n

\"bottom

\n\n

And this is what it looks like from the top:

\n\n

\"enter

\n\n

As you can see, it still gets quite messy around the circles, and they seem to be detached from the rest of the model.

\n\n

After this (and doing a lot more browsing) I decided to make the following change in Cura:

\n\n\n\n

But to be honest, I have not clue what this really does. This is what the initial layer (of another model) looked liked after changing that setting:

\n\n

\"enter

\n\n

And after a couple of layers it looked like this:

\n\n

\"enter

\n\n

Now (finally) my question, does anyone have an idea on how to make my prints (especially the connection of the circles with the rest of the model) better?

\n\n

Thanks!

\n", "Title": "Problem printing circles/holes on Anycubic I3 mega", "Tags": "|ultimaker-cura|pla|adhesion|anycubic-i3-mega|", "Answer": "

From your images can be seen that:

\n\n\n" }, { "Id": "13371", "CreationDate": "2020-04-08T21:36:05.040", "Body": "

I am trying to write a program that edits a 3mf file by defines different slicing parameters for different stls that I have previously merged into my 3mf file, but I am a little bit confused about the configuration of the 3mf file, and where I would look to make changes in the slicing parameters. Can anyone lead me in the right direction as to where I could look? I'm assuming that I wouldn't be making any changes to the .model file because it seems like it only holds information about the meshes.

\n\n

Thanks and let me know if it would be helpful for me to clarify anything.

\n", "Title": "How can I include seperate Slicing Parameters in my 3mf file", "Tags": "|3d-models|stl|prusaslicer|", "Answer": "

Huh. On a hunch I just changed a copy of a .3mf model to have a .zip file extension, and what do you know, it works! 3mf is just a set of compressed xml.

\n\n

The zip folder structure I saw included a MetaData folder. I bet you could put just about anything you wanted in there, as long as it doesn't conflict with things other 3mf parsers are expecting to find.

\n\n

I tested this. I dumped an unrelated pdf file into the folder and re-zipped it. Both MS 3D Builder and Cura were able to open the updated file just fine. However, saving the file again in 3D Builder did not preserve the PDF.

\n\n

I suppose there's a lesson here, too: we may need to be wary of models we download off the internet. They could include literally anything.

\n\n

This also raises an interesting possibility: password protected 3mf files. If 3mf is just a zip container, and zip files can have passwords, it follows logically that 3mf files can have passwords.

\n" }, { "Id": "13376", "CreationDate": "2020-04-10T04:04:38.050", "Body": "

I'm having some issues with my Prusa i3 prints. I'm trying to print the default beer opener print that came with the Prusa's memory card but the infill will break causing clogging and now allowing the print to finish. I've attached a picture of one of the failed prints.

\n\n

\"enter

\n\n

I've checked Prusa's website and tried tightening the extruder gears and made sure the gears are clean. I'm using the PLA sent with the printer (1.75 mm) and with a default G-code file so I'm fairly sure it is a hardware issue, but I'm not sure what the issue could be.

\n\n

Here is the link that I've used to help me troubleshoot.

\n", "Title": "Issues with print Prusa i3 infill breaking and causing clogs", "Tags": "|print-quality|prusa-i3|troubleshooting|infill|", "Answer": "

The issue was the filament sent with the printer was PETG and not PLA like this gcode file called for.

\n" }, { "Id": "13381", "CreationDate": "2020-04-10T17:46:50.973", "Body": "

Is is possible to have alternating numbers of lines for the outer wall between each layer? So, say, one layer will have 3 lines for the wall, the next layer, 2, then the next layer 3, then 2 again, and so on? Even better, if I could group them in twos: two layers with 3 lines, two layers with 2 lines, repeat until done.

\n\n

I use Cura, but if other slicers can do this I'd like to hear about it, too.

\n", "Title": "Alternating number of lines for the shell between layers", "Tags": "|ultimaker-cura|slicing|", "Answer": "

It is possible to get an alternate extra wall in Ultimaker Cura, the option is called Alternate extra wall. It can be used to catch the infill more firmly, strengthening the print.

\n\n

From Ultimaker support:

\n\n
\n

Alternate extra wall

\n \n

This setting adds an extra wall every other layer. This way the infill gets caught between the walls, resulting in stronger prints. For example, if you set the wall line count to two walls and enable alternate extra wall, it will print two walls on even numbered layers and three walls on odd numbered layers.\n \"cross

\n
\n\n

I've used this option frequently, but I'm not aware of any option that you can change the amount of layers alternating. You can do that probably with the option to set different options for different sections of the model, see e.g. \"Different infill in the same part\".

\n" }, { "Id": "13382", "CreationDate": "2020-04-10T17:59:57.533", "Body": "

I've already asked this question somewhere else but unfortunately I had little luck.

\n\n

So... my Ender 3 Pro extruder just started skipping steps, as in the gears (and the gear pinion) will rotate but the filament won't flow. It all started when I changed PLA filament to a new roll; I thought it might have been the roll faulty so I've tried a spool that had been working fine until 2 hours before it all started. Nope, skipping with that one as well.

\n\n

Here's what I've tried doing so far:

\n\n\n\n

None of the above worked, and my cats looked funny at me. Print settings as below:

\n\n\n\n

I've also reverted back to the old PTFE tubing as I noticed that the Capricorn was giving too much resistance to the filament. Nope, still skipping.

\n\n

I've noticed that the extruder gear grips quite firmly onto the filament, so much so that when it starts slipping it actually eats away the filament until it breaks. It's almost like there's a clog somewhere but the tubing is clear, the hot end is clear (I've cleared it and checked multiple times), and the nozzle is brand new.

\n\n

What else can I try? Have I missed something? Apart from the changes listed above (carried out after the extruder started skipping), the printer is absolutely stock, firmware and everything.

\n\n

UPDATE: I've changed the factory hot end bloc with a brand new one, changed PTFE tubing one again, making sure it's as close as possible to the nozzle (unscrew nozzle 1/2 turn, fit PTFE, screw nozzle in) but it didn't change anything at all.

\n\n

The extruder still skips steps as it can't push the filament out of the nozzle. Pushing it manually feels nice and smooth until it hits the nozzle, where I can feel too much resistance.

\n\n

UPDATE 2: I've modifed the following parameters on the EEPROM to limit the filament flow:

\n\n
M203 Z5.00 E25.00\nM201 E1000\n
\n\n

I've also crancked the temperature up to 220\u00b0C but it made no difference whatsoever. What I've noticed is that, after cleaning hot end and tubing, it starts skipping after 1 hour of printing, every single time without fail.

\n\n

UPDATE 3: I've checked the input voltage from the PSU and it's 24V; the Vref for the extruder is 0.744V, so everything looks as expected.

\n\n

UPDATE 4: The extruder idler pulley has a compression washer to hold it in place without impeding idle spinning; it is usually mounted in the order idler pulley, compression washer and bolt. I've noticed that the pulley wasn't spinning freely this way, so I inverted the order to compression washer, idler pulley and bolt. The bolt head is small enough not to stop the pulley from spinning.

\n\n

I've also increased the pressure the spring arm excise on the idler pulley, so that the toothed pulley grips more firmly on the filament.

\n\n

This way I've managed to improve things although not solve them. It's been printing for the last 3 and a half hour without skipping but it's not a solution, as the toothed gear is chewing too aggressively on the filament. In just one hour a good deposit of PLA shavings has formed on the extruder, and I had to blow it away, and this never happened before this all started.

\n", "Title": "Ender 3 pro extruder skipping steps, tried multiple things", "Tags": "|creality-ender-3|troubleshooting|hotend|", "Answer": "

In case anyone else also runs into this problem and has tried everything above, here is how you fix it for good.

\n

While the direct drive extruder might work, it may seem odd to some that the system that previously worked fine now doesn't and needs a complete rebuild.

\n

The problem is caused by a faulty cooling fan for the heat sink. Replace the AXIAL fan on the hotend. That is not the radial fan that is used to cool the print.

\n

The devious thing about this problem is that the symptoms occur seemingly randomly which is caused by the rather slow heat transfer in the heat sink. At some point during the print a critical temperature is reached which deforms/lengthens the heat sink and opens up gaps in the path of the filament. Those gaps cause the filament to get stuck.

\n

The other devious thing is that the cooling fan appears to be working alright but is not. I assume the rotational speed is lower than would be necessary but that can not be verified with the means currently at my disposal.

\n" }, { "Id": "13385", "CreationDate": "2020-04-11T00:18:51.070", "Body": "

I'm printing stacked Prusa COVID-19 face shields on a modded Ender 3. It works fairly well until the second layer of masks, where we get this weird effect that's kind of like stringing. It looks like the curve is becoming \"low poly.\" I'm sorry that I can't be more specific, but I really don't know how to describe it more than this. Please look at the pictures to get more info.

\n\n

One of the strangest things is that it worked earlier with the same slicer and the same profile. I am using PrusaSlicer, but I tried it with Cura and it still didn't work. There are a few more problems with the prints: some under-extrusion on the supports, and some spots had little gaps between the layers.

\n\n

Does anyone have any fixes for any of these problems? I would mainly like to know about the strange kind of stringing because that actually prevents me from stacking prints.

\n\n

I am printing Overture PETG at 240 on the nozzle and 80 on the bed. This is at a speed of 50 mm per second. I have the Basaraba Innovations direct drive mod, the Creality silent board with TH3D Unified Firmware based on Marlin 1.1.9, and the EZABL Pro.

\n\n

Please look at the pictures to see what is happening.\n\"\"\n\"\"\n\"\"\n\"\"\n\"\"\n\"\"\n\"\"\n\"\"

\n", "Title": "Strange stringing-like activity when printing face shields", "Tags": "|creality-ender-3|underextrusion|", "Answer": "

It turns out that the brand of PETG that I was using was not good for stacking prints. I tried another brand of PETG and it worked perfectly. Thank you for all of the suggestions of what could be the problem. If you are stacking face shields (or any other print), do not use Overture PETG.

\n" }, { "Id": "13391", "CreationDate": "2020-04-11T12:07:51.927", "Body": "

I just got an Ender 3 Pro at the beginning of the week and have been printing a lot. I creased the magnetic plate cover, not realizing how it would hold up to being bent. This led to uneven printing and nozzle issues, so I stopped using it.

\n\n

Is there any problem with printing directly onto the platform itself, without a cover?

\n", "Title": "Can I print directly onto the bed plate? (Ender 3 Pro)", "Tags": "|creality-ender-3|bed|", "Answer": "

I've printed for several years direct onto the 3 mm heated bed of the Anet A8 I used; worked perfectly! I did use a specific 3D print spray for adhesion, see e.g. this answer on \"Should you use hairspray on a metal bed 3D printer?\".

\n" }, { "Id": "13393", "CreationDate": "2020-04-11T16:53:48.833", "Body": "

A couple years back I received a Kossel XL kit from builda3dprinter.eu as a gift, but I stopped building it because multiple parts were missing, including printed parts. Now that I'm building a Prusa MK3S, I'm considering buying some of the parts and finding the stl files to print the printed parts. However I'm having a bit of trouble finding the files for the exact model, and finding the printed parts for the XL, rather than the mini. Any help would be greatly appreciated, and advice from people who built the same printer would be incredible. Thanks!

\n", "Title": "Looking for files for printed parts for my Kossel XL kit from Builda3Dprinter.eu", "Tags": "|printer-building|delta|kossel|", "Answer": "

I bought 2 of these kits 4 years ago, and I have been really pleased with the quality of the kits and the results. Sadly this supplier is no longer trading, as he could not compete with low cost imports from the far east.

\n

I have the STL files for the printer kit, so you should be able to print any parts you need.

\n

I have created a Github repository with a ZIP file of the parts files I downloaded from the builda3dprinter.eu website.

\n" }, { "Id": "13401", "CreationDate": "2020-04-12T19:58:24.887", "Body": "

Recently I decided to upgrade my Geeetech A10 (GT2560 v3.0 board) with E3D Chimera, Marlin 2.0.3 and more powerful 50 W heater cartridge. I wired everything up and tried to PID tune the hot end, but I got a problem.

\n\n

When the tuning ends and I use the values with M301, the values are too agressive and the hot end initially overshoots more than 15 °C. After that the temperature is rock steady.

\n\n

\"Overshooting

\n\n

The tuned values are P=9.4 I=0.47 and D=46.4.

\n\n

I tried to change the values according the wikipedia table to no overshoot (P=3,13 I=0.47 and D=123.7), but it just made the temperature unstable.

\n\n

I also tried to tune the values manually but with no succes. I even tried to use zeros for all three gains but it just oscilated ±10 °C and still overshot.

\n\n

My guess is that the cartridge is too powerful for the Marlin PID tuning.

\n\n

Is there a setting in the firmware where I can set the PWM value for the PID tuning? To slow the heating down? I tried to use M301 E1 P0.25 S200 C10 but it seems that it didnt use the P parameter.

\n\n

Or is there a way to manually tune the values without the printer? I've got an Arduin Uno and RPi Zero availible. Or do I have to buy another less powerful cartridge?

\n", "Title": "PID tuning 50 W cartridge in Marlin", "Tags": "|marlin|hotend|heat-management|pid|", "Answer": "

Besides the P, I and D values, you may also have to tune PID_FUNCTIONAL_RANGE and PID_INTEGRAL_DRIVE_MAX.

\n\n

Basically, the functional range disables PID control when more than the set number of degrees away from the target temperature and just puts the heater to zero/maximum power. The integral drive max parameter limits the value of the integral term of the PID controller.

\n\n

The RepRap wiki notes: (note this wiki is dated! some options might not be available)

\n\n
\n

In Marlin, the parameters that control and limit the PID controller can have more significant effects than the popular PID parameters. For example, PID_MAX and PID_FUNCTIONAL_RANGE, and PID_INTEGRAL_DRIVE_MAX can each have dramatic, unexpected effects on PID behavior. For instance, a too-large PID_MAX on a high-power heater can make autotuning impossible; a too-small PID_FUNCTIONAL_RANGE can cause odd reset behavior; a too large PID_FUNCTIONAL_RANGE can guarantee overshoot; and a too-small PID_INTEGRAL_DRIVE_MAX can cause droop.

\n
\n\n

My hunch is that the functional range may be too small; the PID doesn't kick in until close to the desired temperature at which point you will already have overshot the target (due to the high power heater and delay in the measurements). Once you've overshot the target temperature the integral term starts growing (negative) which causes the massive undershoot.

\n" }, { "Id": "13403", "CreationDate": "2020-04-13T04:35:47.093", "Body": "

I have a relatively new Ender 3 Pro, stock except for adding a tempered glass bed. I've fed about 3 kg of PLA filament so far, all Solutech brand. Near the end of the last reel, I started hearing a chunk noise. It varies in rate, between several/minute to going 10-15 between chunks. This has continued into a brand new, just unsealed reel so I don't think it's associated with the filament.

\n\n

I was able to finally discover that the sound is coming from the extruder stepper area and was able to see that when this happens, the filament drive wheel snaps back maybe 30-40 degrees. (This happens in the middle of printing a line, not when the filament retracts between lines.) I'm assuming that the stepper motor is slipping. There is no sign of gaps in the printed model nor is the filament on the reel binding that I can see.

\n\n

I'm guessing maybe that the filament is being fed faster than the hot end can accept it, causing back pressure.

\n\n

I'm using Cura 4.4.0 as my slicer the whole time and while I have changed some settings, everything associated with speed, feed rate, hot end temperature and such are all stock. I did not notice this happening until recently.

\n\n

Has anyone seen this happen before? Anyone have any idea what could be causing this?

\n", "Title": "Ender 3 Pro Extruder Stepper Skipping with a Chunk sound", "Tags": "|extruder|creality-ender-3|stepper|", "Answer": "

UPDATE 3-May-2020: I have now confirmed that the problem was the extruder. Specifically, the input port which was causing the filament to bind. The front of the input port is a metal ring but the back is just plastic and and has worn from the PLA filament, as seen in this photo:\n\"Close\nAfter some delay (see my other question here) I have installed the new, all-metal extruder (original stepper), the clacking is gone as well as the significant under-extrusion. (I also replaced the Bowden tube and print bed springs but I don't believe these were the problem).

\n\n

Having seen what a few km of PLA did, I highly recommend upgrading the extruder.

\n\n

Side point: immediately after the replacement, I started getting corners lifting off the bed which I hadn't seen before. I realized that the new drive gear was slightly smaller than the old so I checked the extruder calibration and it was about 10% low. I adjusted the extruder steps/mm from 93 to 99 to correct for this.

\n\n
\n\n

Original:

\n\n

It appears the problem was the filament binding in the entry to the extruder assembly. I did start seeing signs of under extrudement such as the outer layer of a rectangular object separating of the rest of the body. Later, the problem became worse, with a lot of clanking, to the point that prints became quite fragile.

\n\n

I have ordered a all-metal extruder assembly and will update as I get new results.

\n\n

By the way, I'm really not sure why it's binding but running a strip of filament through the entrance hole, it sure seem to be binding. But in trying to disassemble the extruder and starting by removing the filament, I managed to get molten filament in the bowden tube. Without thinking about it, I pulled the blob all the way into the fitting for the extruder, plugging it so much that I cannot get the fitting off nor push it back into the tube. Anyway, I'm taking the opportunity to upgrade the extruder, tube and springs.

\n" }, { "Id": "13406", "CreationDate": "2020-04-13T19:47:17.077", "Body": "

TL;DR;

\n\n

My initial answer fixed the problem for a while, but it still persists occurring... (see update 2 below)

\n\n
\n\n

Original question body:

\n\n

Print fails at random resulting in a hairy mess although I:

\n\n\n\n

What else to do ?

\n\n
\n\n

First off, the equipment I use:

\n\n\n\n

At first the prints came out as expected.

\n\n

\"first\"

\n\n

There is some bending at the bottom, that was later resolved by adding a brim. The object above and the ones later were printed with roughly the same settings:

\n\n\n\n

Yesterday that same part failed after 4 hours. I had to stop the print because the printer was making \"a hairy mess\" around the object.

\n\n

\"Second

\n\n

As you can see there was some under extrusion earlier in the print which I didn't notice at first.\nI thought the part came loose since it had little to none adhesion to the bed and after cleaning the bed properly and using some glue I started the print again. The result was the same, this time the print failed a bit earlier.

\n\n

\"Second

\n\n

(The small part was printed on top of the loose mess before it failed completly)

\n\n

This time the bottom was quite loose again. After that I decided to print some smaller parts and do some troubleshooting first. I noticed some weird looking \"curly\" lines right away and stopped the print. It looked like \"under extrusion\" and a \"not properly leveled bed\". (Notice that didn't happen in the prints before). So, after leveling the bed and printing the small part again the part got printed well until the top 3 layers which looked under extruded again and I could see the infill pattern through them.

\n\n

I added 5 \u00b0C to the printing temperature and (the smaller parts had 30 % infill) upped the infill to 40 %. That didnt resolve the issue completly but the print looked well enough and I had another go at a slightly bigger print.

\n\n

This time not quite as tall at the ones before and it failed again. The first hour or so everything looked fine, but the printer was getting quite noisy, every retraction was a very loud clicking sound the printer itself was shaking quite a bit during movement and the print failed again. This time the object was very firmly on the bed and I had to use some force to get it off.

\n\n

\"fourth\"

\n\n

The noise and shaking made me check the axis and tightened the wheels a little bit and after that I cleaned the nozzle with a \"cold pull\". The last thing I did was printing a temperature tower which failed in the same way and looks quite bad overall.

\n\n

\"Temp\"

\n\n

So my question is: Where do I go from here? I'm quite new to 3D printing, there still seems to be some under extrusion going on, what are my next steps ?

\n\n

Please note that it seems that the filament curls up at the tip of the nozzle randomly, resulting in a few lines not getting placed properly on the already printed parts. Eventuelly the print then fails. When I push filament throught it curls up and the stream is relatively thin. Not always though, most of the time it looks fine maybe a bit thin.\nCan that be a clogged nozzle if X layers get printed ok, then for 5 seconds it messes up and then prints okay again?

\n\n

[Update]

\n\n

After changing the nozzle the filament stopped curling up and the lines look much, much better. But the problem persists. I started 2 prints, both failed a couple of minutes in. The second one on the first layer. It seems that the nozzle picked the just printed lines up.

\n\n

\"line\n\"line

\n\n

[UPDATE 2]

\n\n

Back to square 1. It seems that I fixed a problem that I created while trying to fix the original problem.

\n\n

After 4 successful prints and ~20 hours of printing the last print failed in the exact same way as shown in the pictures above. The filament is curling up at the nozzle again. The heatblock is not leaking.

\n\n\n\n

I guess the nozzle is clogged ?! Can that be after such short printing times ? I dont want to clean everything for 2 hours after every print. Or worse change everything out after 15 hours or so.

\n", "Title": "Print failing at random, resulting in a \"hairy\" mess", "Tags": "|underextrusion|print-failure|artillery-sidewinder-x1|", "Answer": "

After I finally had the time to disassemble the extruder and I found a possible culprit - which did solve the issue for some time.

\n

\"enter\n\"enter

\n

As you can see on the pictures the heater block leaked and this lead to some filament dripping off of the side and onto the print. The nozzle then eventually caught these drips and this caused the mess.

\n

Here are some tipps for beginners like me:

\n
    \n
  1. How to disassemble the extruder on the Sidewinder X1

    \n
  2. \n
  3. Be carefull when trying to screw the nozzle out. You will most likely need some heat to soften up / melt the filament. I was not carefull and broke the thread. I then had to replace the whole heatblock.

    \n
  4. \n
  5. Examine the tubing that leads the filament through the heatbreak and to the thread of the nozzle mine was a little bit deformed and didn't seem to be cut off straight to begin with.It has to be as flat as possible against the nozzle thread!

    \n
  6. \n
\n

This was most likely the problem, Over time and after several melting ,hardening and melting procedures some filament creeped between the threads and the heatblock started to leak .. first slowly then worse and worse.

\n

Hope that helps anyone. My prints come out great again. Plus I now know a lot more about my printer .

\n

the original problem reappeared, see my update 2 of my question

\n

[UPDATE]

\n

Just to close this off: I can only guess what the original problem was but since I had this issue some times now I am quite certain, that it has to be bad filament (I am using really cheap one) and a relatively dusty environment, mixed with overreacting when trying to solve this issue. I got some needles with which I - fairly regularly - unclog / clean the nozzle especially after longer times without use.

\n" }, { "Id": "13412", "CreationDate": "2020-04-14T13:13:00.820", "Body": "

PLA has a heat capacity of 1.8-2.1 J/g-K, while PETG 1.1-1.3 J/g-K. This means that each gram of PLA needs more energy to heat up. I assume no "melting latent energy", since we talk about plastics.

\n

The density is about the same.

\n

Still, printing speed for PETG is said to be kept at max at 60 mm/s, while PLA can easily go up to 100 mm/s.

\n

Why is PETG supposed to be printed slower than PLA?

\n

Edit: a link to a more recent question may be of interest: Power consumption of filament extrusion

\n", "Title": "Why does PETG require slower speed?", "Tags": "|pla|print-material|petg|", "Answer": "

I'm adding this answer to somewhat challenge the findings of my original answer, and the premise of the question: PETG does not need lower print speeds, and can even be printed at higher speeds than PLA under some conditions due to reduced need for cooling. You can see this from some of the "#speedboatrace" entries printed with PETG. So what was really going on with the original claim and my agreement with it?

\n

I think my original answer is still somewhat true: it's likely that it takes more hotend power to melt PETG at a rate that can be successfully extruded and bonded than to do the same for PLA. But there are other factors at play in the perception that "PETG has to be printed slow".

\n

FarO did not specify details of the printer(s) in question, but I found the big limiting factor for my Ender 3 printing PETG was the stock extruder, which presumably was skipping bad to begin with, and even worse with Linear Advance, trying to keep the filament under high pressure to compensate for its compressibility. Since replacing the extruder with a direct drive one, I've had no problem printing PETG at the same speed as PLA, and both can print much faster than I ever could with the stock bowden extruder.

\n" }, { "Id": "13432", "CreationDate": "2020-04-16T19:42:00.073", "Body": "

I have two options - to buy either the Ender 3 or the Mega Zero. I'm heading towards Ender 3 because the Mega Zero doesn't have a heated bed.

\n\n

My question is, how is a printer without a heated bed (the Mega Zero) a better option than one with a heated bed (the Ender 3)?

\n\n

Why would I even consider buying a printer without a heated bed when the Ender 3 can do the same things and has a heated bed? I want be able to print not only PLA but other materials as well. Doesn't the Mega Zero limit you to using only PLA?

\n", "Title": "Is a heated bed an essential component for printing (difference between Creality Ender 3 or Anycubic Mega Zero)?", "Tags": "|creality-ender-3|heated-bed|", "Answer": "

I used to print PLA with my heated bed turned off, since it was deforming when heated.

\n\n

It works perfectly fine, only detaching the print was terribly difficult.

\n\n

I would not buy a printer without it, because even a weak bed reaching only 50\u00b0C gives you many more possibilities.

\n\n

I mean, you could use it to keep your coffee warm while you work at the computer, or to warm up chemical solutions to make them react faster. You can also use the bed to shake the solution!

\n\n

Go for a heated bed.

\n" }, { "Id": "13437", "CreationDate": "2020-04-17T06:43:18.310", "Body": "

I watched with great interest this video: Temp Tower - Slic3r Gcode Placeholders - How To - Chris's Basement -- Jan 23, 2019 Chris Riley

\n\n

I found a model I'm happy with: Smart compact temperature calibration tower by gaaZolee, December 24, 2017 Uses Layer 0.20 mm infill: 15 %

\n\n

I'm testing a stubborn new spool of PLA. Temp range of test 225 °C to 180 °C. That model above came with recommended ifstatements. I verified that with 100 % thickness on first layer, I'm exactly hitting all the Z targets at exactly the correct place for each temperature story on the tower.

\n\n

I'm using the Slic3r \"Printer Settings\" tab - to place my if statements in place at \"Before Layer Change G-Code\".

\n\n

It's not clear to me on the format required for a G-Code {if} statement. I've tried...

\n\n
{if layer_z == 7.8}M104 S225{endif};   // fail, this converts to: M104 S225 {endif}    \n{if layer_z==1.6}M104 S225             // fail, this converts to: M104 S225    \n{if [layer_z]==1.6} M104 S225          // fails, this converts to: (null)    \n{if [layer_num]==8} M104 S225          // fails, this converts to: (null)    \n{if layer_num==8} M104 S225          // fails, this converts to: M104 S225 \n
\n\n

In detail what I'm seeing is the if statements from all ten towers prints as change temp to 225, 220, 215, 210... down to 180 (last entry) and 180 °C is where ALL my prints get printed at. Total Fail.

\n\n

I'm just not seeing anything in the Gcode output from Slic3r that remotely looks like an if statement. Any ideas here? I've seen this reference, on advanced conditional G-Code but I'm just not understanding what to do here.

\n\n

I do see from Slic3r Custom G-Code Placeholder Reference, that I'm using the correct terms. [layer_z] - Z height of the active layer, and [layer_num] - Integer number of the active layer. It's not clear if their use in \"Before Layer Change G-Code\" require the square brackets or not.

\n\n

Any clues on how to get an if statement into G-Code via Slic3r for a temperature tower print?

\n", "Title": "Temp Tower Setup Troubles -- Slic3r (How do you format G-Code {if} statements?)", "Tags": "|g-code|calibration|slic3r|", "Answer": "

So I think I figured this one out. I was looking at it a bit wrong. And here's my code:

\n
; Note to self:  layer_z = [layer_z]\n; T tower floor 1\n{if [layer_z]==1.6} M104 S225 T0\n; T tower floor 2\n{if [layer_z]==11.6} M104 S220 T0\n; T tower floor 3\n{if [layer_z]==21.6} M104 S215 T0\n; T tower floor 4\n{if [layer_z]==31.6} M104 S210 T0\n; T tower floor 5\n{if [layer_z]==41.6} M104 S205 T0\n; T tower floor 6\n{if [layer_z]==51.6} M104 S200 T0\n; T tower floor 7\n{if [layer_z]==61.6} M104 S195 T0\n; T tower floor 8\n{if [layer_z]==71.6} M104 S190 T0\n; T tower floor 9\n{if [layer_z]==81.6} M104 S185 T0\n; T tower floor 10\n{if [layer_z]==91.6} M104 S180 T0\n
\n

The key to understanding what is going on is the ; Note to self: layer_z = [layer_z] line. The whole block of code above gets repeated for every layer of the print in the G-code, but it gets evaluated. And when I search for "layer z =" and scan thru the code.. I eventually see this...

\n
; Note to self:  layer_z = 1.6\n; T tower floor 1\n M104 S225 T0                  <---- Yowza. Nice. \n; T tower floor 2\n; T tower floor 3\n; T tower floor 4\n; T tower floor 5\n; T tower floor 6\n; T tower floor 7\n; T tower floor 8\n; T tower floor 9\n; T tower floor 10\n
\n

and then again:

\n
; Note to self:  layer_z = 11.6\n; T tower floor 1\n; T tower floor 2\n M104 S220 T0           <---- Yes!\n; T tower floor 3\n...\n
\n

All good. You have to have {} and [] within the code statements...

\n

Addendum: So it turns out things are not quite perfect. I'm noticing at the upper floors, where things should be cooler, the temperature isn't changing. Its locked at 215C. I see this in the code:

\n
; Note to self:  layer_z = 31.5999999999999      <--- not sure how that happened.  \n; T tower floor 1\n; T tower floor 2\n; T tower floor 3\n; T tower floor 4\n; T tower floor 5\n; T tower floor 6\n; T tower floor 7\n; T tower floor 8\n; T tower floor 9\n; T tower floor 10\n
\n

31.6 would have triggered a temperature change. 31.5999999999999 won't do it. I need to modify the {if... == } to a complex test between two values... Just wanted to share that observation here.

\n

I do see from this reference that

\n
\n

Expressions may be chained for an implicit AND:

\n

{if [layer_num] == 10}{if [temperature_1] != 210}M104 S210

\n
\n

And this seems to fix it:

\n
; Note to self:  layer_z = [layer_z]\n; T tower floor 1\n{if [layer_z]<=1.65}{if [layer_z]>1.55} M104 S225 T0\n; T tower floor 2\n{if [layer_z]<=11.65}{if [layer_z]>11.55} M104 S220 T0\n; T tower floor 3\n{if [layer_z]<=21.65}{if [layer_z]>21.55} M104 S215 T0\n; T tower floor 4\n{if [layer_z]<=31.65}{if [layer_z]>31.55} M104 S210 T0\n; T tower floor 5\n{if [layer_z]<=41.65}{if [layer_z]>41.55} M104 S205 T0\n; T tower floor 6\n{if [layer_z]<=51.65}{if [layer_z]>51.55} M104 S200 T0\n; T tower floor 7\n{if [layer_z]<=61.65}{if [layer_z]>61.55} M104 S195 T0\n; T tower floor 8\n{if [layer_z]<=71.65}{if [layer_z]>71.55} M104 S190 T0\n; T tower floor 9\n{if [layer_z]<=81.65}{if [layer_z]>81.55} M104 S185 T0\n; T tower floor 10\n{if [layer_z]<=91.65}{if [layer_z]>91.55} M104 S180 T0\n
\n" }, { "Id": "13466", "CreationDate": "2020-04-20T04:34:04.407", "Body": "

Does a filament exist that can resist ozone (like certain silicone tubing's can). Most glass bottles come with tin or plastic screw hard tops and I would like to replace the screw caps with something that is more resistant to ozone.

\n\n

The reason for this is I make my own homemade ozonated oil in glass bottles and I would like to print out different hard screw top caps for some of the bottles.

\n", "Title": "Filament that is resistant to ozone", "Tags": "|filament|fdm|filament-choice|", "Answer": "

This is highly relevant: PETG is very resistant to Ozone.

\n

https://www.plasticsintl.com/chemical-resistance-chart

\n" }, { "Id": "13472", "CreationDate": "2020-04-20T11:32:15.947", "Body": "\n\n

\"finished

\n\n

Update, I received a hint that an existing question has the answer; that link was not really to my satisfaction -- but it did link to another one that did: What are main differences between rafts, skirts and brims?

\n", "Title": "What's the purpose of the \"outlines\" around prints?", "Tags": "|bed|", "Answer": "
\n

I am curious, what is the purpose of printing a single-height outline around the objects to be printed?

\n
\n\n

The (equidistant) lines at distance from the print object is called the \"skirt\", the skirt is an option found under the \"Build Plate Adhesion\" options in your slicer. The primary function of the skirt is to get the flow going, but there are more benefits you can get from the skirt:

\n\n\n\n
\n

Also, how would it affect the outline if the object to be printed extends to (very near) the very edge of the print area?

\n
\n\n

Do note that a skirt limits the useable build area by the distance and width of the skirt.

\n\n
\n\n

Basically this has been answered (see this answer and this answer) in a different question (\"Random lines are being printed?\"), but it might be beneficial to answer this question rather than closing this for a dupe. This question is focussed on the skirt, the equidistant lines around the print object, while the other question focuses on the priming line.

\n\n
\n" }, { "Id": "13477", "CreationDate": "2020-04-20T18:46:19.953", "Body": "

As I want to print with multiple colors I plan to upgrade my Ender 3 with the diamond hotend.

\n\n

So far I found all information I need, except what power supply I need for my RUMBA board.

\n\n

Can I use the default Ender 3 power supply or do I need an additional power supply or wires?

\n\n

Parts I am going to order:

\n\n\n", "Title": "Upgrade Ender 3 with Diamond Hotend", "Tags": "|creality-ender-3|power-supply|diamond-hotend|", "Answer": "

The RUMBA boards run up to 36 V, so reusing the (24 V) power supply from your printer should be fine. As the 2 extra steppers do not take much extra power (note you are ordering only 1!), your supply should be sufficient. Note that a 12 V heater cartridge is not going to work on a 24 V board (you'll get a massive overshoot and probably over-temperature warnings and printer shutdowns).

\n" }, { "Id": "13495", "CreationDate": "2020-04-23T03:33:33.140", "Body": "

When my Ender 3 prints a top or bottom layer, it leaves a line on the layer. \n\"enter

\n\n

\"enter

\n\n\n\n

My belts are tight enough as I push them they spring back up. I bought a support block for my Z lead screw to help with z banding (I also removed it to see if it will fix the issue but it didn't). I tried 50 mm/s, No Infill, swapped out to brand new 0.6 mm nozzle (the picture has 0.4 mm), Minimum wall-flow to 0 % but no improvement. It has to lead me to think its a mechanical issue but I am not sure where to start.

\n", "Title": "Surface artifacts when formation of object changes (Layer inconsistency)", "Tags": "|print-quality|creality-ender-3|", "Answer": "

I have found the issue. My extruder gear was deformed after 4 months of continuous use which made the extruder have a difficult time when a rapid change in extrusion happened. I swapped it out to a new stainless steel one and it has been working well so far.

\n

EDIT:

\n

These inconsistencies are mainly caused by the pressure in the nozzle not changing fast enough. So to solve this you should look at the extruder setup as that is where the pressure is being controlled. Get any gunk out of there and make sure the path is clear and make sure nothing is grinding and you have a good grip on the filament.

\n

To reduce the change in pressure you should play with printing speed, hotend temperature, linear advance if applicable and you may tune extruder jerk and acceleration although not necessary.

\n" }, { "Id": "13497", "CreationDate": "2020-04-23T06:24:33.977", "Body": "

I'm new to 3D printing, I bought an Ender 5 Pro just recently. Trying to print with 1.75 mm PLA, but the results are very bad, unfortunately. When I got after bed leveling I got a decent result from the demo dog and started to print small things, where the quality was acceptable. Then I tried to switch to bigger items, but the printing failed half-way due to not enough adhesion.

\n\n

There were other quality problems too with this print, and you can already see that in the walls there are holes:

\n\n

\"enter

\n\n

I did another round of bed leveling and Z alignment to make sure the adhesion is good and a test print came out quite fine in all corners and the center. The one layer rectangles and not perfect, in some places the lines separate, but they are mostly fine. But now I have a different problem: all the prints are very messy, less clear, they are not as strong as before and also there are big holes now in the walls. \nThe same statue base (as I had to stop after getting messy earlier) looks like this now:

\n\n

\"enter

\n\n

Another try failed after a few hours (like printing stopped and was printing nothing in the air), but also there is the holes/gaps problem even more visible:

\n\n

\"enter

\n\n

Finally as a test I printed an object with the same G-code as previously used and the result is much different. The object on the left is the new one. It is weak, you can feel by pressing that the walls are not solid, they bend. The rectangle \"eyes\" are also not clear:

\n\n

\"enter

\n\n

There is a difference though, I changed the extruder's nozzle between, the new one was also in the Ender package, it is also 0.4 mm as the original should have been, I changed to try with a new one.

\n\n

Because of the last test with the same gcode, different result, I think the problem might be in hardware adjustment as well, not only software. Anyone has an idea what I'm doing wrong?

\n\n

EDIT 1: after calibrating the extraction amount and reducing the print temperature from 200 C to 190 C, now I get the below result. The values used:

\n\n\n\n

\"enter

\n\n

EDIT 2: Based on the comments, some changes were made and here are the results.\nXYZ cube,

\n\n\n\n

It looks good, the layers are visible though, some ghosting right to X and Y.\nThe sizes are not correct though: X = 20.07 mm, Y = 20.03 mm, Z = 19.84 mm\n\"enter

\n\n

Are X/Y acceptable? What should I do with Z, increase steps/mm ?

\n\n

Finally here is the 3D benchy too, although looks mostly fine, there are some bumps in the walls and small strings in open areas. This was printed earlier and with different settings though:

\n\n\n\n

\"enter

\n\n

EDIT 3 I tried now to print the PolyPearl, that has thin curving lines. The first try failed after 2 hours, a knob developed on the nozzle that ruined the print. This was printing with 190 C. I gave a new try and printed with 200 C, and amazingly it completed the job. See below the pictures, here are my settings for it (changes for Cura 4.6.0's default Super Quality):

\n\n\n\n

The model sticks well to the glass plate even without glue or hair spray, maybe a little too well. I see some problems though, not sure how normal they are:

\n\n\n\n

Here are the images: https://photos.app.goo.gl/ZfuMFFedL171eLeM7

\n\n

Are this problems normal/acceptable?

\n", "Title": "Gaps/Holes in the 3D Print walls", "Tags": "|print-quality|troubleshooting|creality-ender-5|", "Answer": "

Let's see what kind of errors I see (accumulated from what I commented):

\n\n

Underextrusion

\n\n

The legs seem rather underextruded. This could be a clogged nozzle as much as too fast printing or too low a temperature as well as too little pressure (e.g. miscalibrated extruder). Try a cleaning or a fresh nozzle, other playing with the parameter can wait after you fixed the next big issue.

\n\n

To fix the under extrusion if it persists after fixing Ringing, experiment with the following settings:

\n\n\n\n

Ringing/Ghosting

\n\n

The cube with the indentations shows it best: There is a massive ringing happening. This is an effect that happens when the printer changes directions fast - the printhead can't accelerate and decelerate with infinity, as it has momentum, and as a result oscillates around the new path a little, like a sinus ring. This creates a \"ghost\" of the preceding path changes on a flat face, which is why I asked to print a cube.

\n\n

To fix Ghosting and Ringing, there are 2 software fixes:

\n\n\n\n

There is also a hardware design way to reduce it in many designs by shaving off weight from moving parts or stiffening up the construction:

\n\n\n\n

Height inaccuracy

\n\n

This could stem from any of several sources. In descending order of likeness:

\n\n\n" }, { "Id": "13499", "CreationDate": "2020-04-23T13:50:10.240", "Body": "

I just got into 3D printing with an Ender-3 Pro. The test dog printed great, some custom small/thin objects printed good, and now I tried my first big object.

\n

Designed in Tinkercad (link to my (updated) design), exported to .stl, sliced and printed using Cura 4.5.0 with the default 0.2\u00a0mm profile (3 layers wall thickness) and 100% infill (for strength), with the extruder at 200\u00a0\u00b0C and bed at 50\u00a0\u00b0C. Ambient temperature had been 25-ish\u00a0\u00b0C.

\n

The filament is a brand-new Spectrum Premium PLA (Arctic White). On its box it says it prints at 185-215\u00a0\u00b0C.

\n

So I got this:

\n

\"On

\n

\"On

\n

\"One

\n

\"One

\n

\"Other

\n

The top (thinner) sections look OK-ish (there are some visible lines, but meh), but the transition from the bottom sections (thick) to the top has terrible layer separation and warping problems. One side of the base is actually barely holding together when I'm holding it. There is also a smaller problem mid-way at that side of the base, but it's not catastrophic.

\n
    \n
  1. Why did that happen? What did I do wrong? How can I print it again and get it nice and strong?
  2. \n
\n

2) I have a handheld "3D print pen". If I use it to fill the gaps manually, how strong will the part be? The part is meant to "hug" the iPad charging port, so that you can hold the iPad in the bed while charging, without pressing the cable on your body and damaging it. nvm that

\n
\n

Using wall thickness of 4, layer height 0.16\u00a0mm, 30% infill and 205\u00a0\u00b0C extruder, this is my (successful) result.

\n

\"Successful\n\"Successful

\n
\n

I also repaired that broken part. Did it for the aesthetics, but it feels quite strong.

\n

\"Repaired

\n", "Title": "Ender-3 Pro - PLA mid-layer warping (layer separation) - salvageable?", "Tags": "|creality-ender-3|adhesion|infill|warping|", "Answer": "

One hundred percent infill is not necessarily stronger than lower values. By having such a high infill figure, the forces on the model as it cools are magnified and not in a particularly good manner.

\n\n

Consider that you could use twenty to thirty percent infill to get the strength you require for this application, saving filament and time for the print. You've not noted how many wall layers used, but for increased strength, four to five would make for a very strong model.

\n" }, { "Id": "13503", "CreationDate": "2020-04-23T16:14:40.817", "Body": "

I've got a Renkforce RF1000 which should be a good 3D printer. I got it second-hand for my birthday one year ago. I've got no way of contacting the old owner.

\n

I spend a good amount of hours fine-tuning the slicer settings last year but at best got mediocre prints. Between September and a week ago I lived somewhere else and didn't touch my printer.

\n

Now here's what I don't know:

\n\n

Here are some important details:

\n\n

Here are some of the problems I've got:

\n\n

Feel free to give any thoughts you've got. These are the most important questions I've got:

\n
    \n
  1. Should I replace the nozzle with this one so that I know what nozzle I've got and so I'm sure this isn't a problem?
  2. \n
  3. Should I replace the filament with new 1.75\u00a0mm PLA? If so, why?
  4. \n
  5. How do I fix the extruder?
  6. \n
\n\n
    \n
  1. Is the unconnected fan important and if so: what do I do with it? There's no remaining wire to connect it to.
  2. \n
  3. How tight should the 4 screws that hold the filament between the extruder and the rolling disk be?
  4. \n
\n

For now, these are all hardware problems. I can post my Slic3r settings too but I believe the hardware should be fixed before going into slicer settings.

\n

Here are some pictures showing the problems:

\n

\"Top

\n

This is the extruder. The feeding mechanism can be seen in front. It shows the "feed knurl" of which the inside spins while the outside remains stationary (question 3). Next to it are 4 screws which determine how tight the filament is held against the extruder (question 5). On the back it shows a black fan, this got placed by the previous owner but isn't connected (question 4).

\n

\"Image

\n

This image shows the unconnected fan (question 4) to the right. Behind it is the motor that's connected to the extruder. The motor works but the extruder doesn't spin with it. The extruder has a little black hole on top.

\n

\"Front

\n

This shows the extruder from the front. The inner layer spins, the outer layer doesn't (question 3)

\n

\"Failed

\n

These are some of the prints when the extruder was still working.

\n

\"Nozzle

\n

Nozzle and print bed (question 1)

\n", "Title": "Can you help me improve my Renkforce RF1000 print quality?", "Tags": "|extruder|troubleshooting|renkforce-rf1000|", "Answer": "
\n

Should I replace the nozzle with this one so that I know what nozzle I've got and so I'm sure this isn't a problem?

\n
\n\n

It appears the default nozzle size is 0.3 mm. The linked nozzle is a direct replacement.

\n\n
\n

How do I fix the extruder?

\n
\n\n

The \"feed knurl\", usually called the extruder gear is attached to the stepper shaft using a fine grub screw (\"threaded pin\" is a bad choice of words...). The black hole you mention is the hole of the screw (it should be in there, usually an allen key or flat screwdriver should be used). Position it so that the screw is tightened against the flat spot on the stepper shaft.

\n\n
\n

Should I replace the filament with new 1.75 mm PLA? If so, why?

\n
\n\n

If the PLA has been open for a year and not stored properly it could have taken up moisture, I would buy a new spool (3.0 mm) of filament, not 1.75 mm.

\n\n
\n

Is the unconnected fan important and if so: what do I do with it? There's no remaining wire to connect it to.

\n
\n\n

The fan doesn't look as if this is a default fan, this fan is mounted onto cooling fins probably put there by the previous owner to cool the extruder stepper. Maybe the printer has been enclosed and used to print ABS at elevated temperatures. For Printing PLA cooling the extruder stepper would not be necessary as it doesn't require an elevated print temperature.

\n\n
\n

How tight should the 4 screws that hold the filament between the extruder and the rolling disk be?

\n
\n\n

The 4 screws need to be tight enough for the extruder gear to leave indentations on the filament, but not too deep.

\n" }, { "Id": "13505", "CreationDate": "2020-04-23T21:23:04.590", "Body": "

I'm trying to diagnose a print issue that looks like a missed layer (sorry, first-time printer here)

\n\n

\"side-view\"\nand\n\"top-view\"

\n\n

Specs:

\n\n\n\n

Started from a new printer: I was getting imperfect prints (lesser version of this, functional part) and contacted QIDI support. They eventually replied that my bed was too low but I had since lowered the bed on a misdiagnosis. They supplied a slightly modified G-code but that also failed in a similar way.

\n\n

Since then, I've been raising the bed bit by bit to try to fix this issue. I don't care about the little whiskers much, but the solid \"connecting lines\" are a problem.

\n\n

Questions:

\n\n
    \n
  1. Is this actually a bed-leveling/height issue?
  2. \n
  3. What is this defect called (whatever the cause)? I've been googling but can't find something that looks like this.
  4. \n
  5. Is there a better way of fixing bed height issues than just printing->tweaking bed height->printing?
  6. \n
\n\n

I'd appreciate any help diagnosing this issue - even if it's just the name of this problem. I have no experience disassembling/looking at hardware but I could always give it a shot.

\n\n

Update

\n\n

As per comments I took a look at coasting and ran some tests. I first re-leveled the bed manually (I'll call that height=0). I don't want to add tons of pictures so I'll show the baseline, and \"bad\" refers to similar failure as above.

\n\n
+--------+--------------+------------+-----------------------+-----------------+--------------+-------------+\n|        | Combing      | Coasting   | Retraction distance   | Extruder temp   | Bed height   | Results     |\n|--------+--------------+------------+-----------------------+-----------------+--------------+-------------|\n| Test_1 | infill only. | off.       | 2.0 mm.               | 200 C.          | 0.           | Baseline.   |\n| Test_2 | infill only. | off.       | 2.0 mm.               | 200 C.          | +0.05 mm.    | ~ Baseline. |\n| Test_3 | off.         | off.       | 2.0 mm.               | 200 C.          | 0.           | Bad.        |\n| Test_4 | infill only. | off.       | 1.5 mm.               | 200 C.          | 0.           | Bad.        |\n| Test_5 | infill only. | off.       | 2.0 mm.               | 190 C.          | 0.           | TBD         |\n+--------+--------------+------------+-----------------------+-----------------+--------------+-------------+\n
\n\n

(+/- 0.05 mm height achieved with clicks of \"fast leveling\")

\n\n

Test 1 external view:\n\"test_1_out\"

\n\n

Test 1 internal view:\n\"test_1_in\"

\n\n

Update

\n\n

I think I've confirmed an under-extrusion problem. Here is a top layer printing I stopped partway through.

\n\n

\"test_5_top_layer\"

\n\n

This looks exactly like Simplify3D's Guide. So I manually pushed through, removed, and re-inserted the filament - and cleaned the nozzle with a brush.

\n\n

I then ran a new print and noticed a clicking sound in the extruder (I don't know if it's new or if I simply noticed because I was focused on under-extrusion problems) - the print failed in the same \"dragging\" way as above, but the top layers were slightly better.

\n\n

I'm now going through the list of possible causes of clicking extruder + under-extrusion.

\n", "Title": "Partially missed / dragged layers", "Tags": "|print-quality|extrusion|layer-height|qidi-tech-x-plus|", "Answer": "

I managed to confirm the dragged layers were mostly due to under-extrusion.

\n\n

I had a faulty extruder (clicking sound) and once I swapped it out it was greatly improved.

\n\n

It's worth noting I also had a slight z-wobble issue so that could have played a role.

\n\n

Hopefully this can help somebody out as I hadn't seen this kind of symptom in any of the print quality guides.

\n" }, { "Id": "13514", "CreationDate": "2020-04-24T18:35:26.360", "Body": "

I'm building a device that requires five 12 V valves to be programmatically controlled as a switch. So far, I have been able to get outputs from the heated bed, extruder, and fan to be controllable through Marlin, by defining \"SENSITIVE_PINS\" to be an empty array. I can now use M42 to switch these pins on and off, allowing for programmatic control of 3 of the 5 valves.

\n\n

For the other two valves, I'd like to control these directly from the 3D printer board somehow. I noticed the A4988 driver will rapidly flick a 12 V output on and off according to the microstep setting (https://lastminuteengineers.com/a4988-stepper-motor-driver-arduino-tutorial/). But this type of flicking is not really good for controlling a valve, because I want the valve to remain open until I say \"stop\".

\n\n

What I would like is to be able to use the M42 to command to administer a 12 V output on or off from the motor section of the usual boards. The A4988 driver takes in inputs like Enable, Step, and Dir from Arduino pins. Is there any way I can \"hack\" the A4988 driver to provide a constant voltage, similar to a relay switch? Alternatively, can I remove the driver and do something with the female pins that are normally used to connect to the A4988 driver?

\n\n

In researching this I have noticed that when removing the driver, you expose some more 12 V female and GND pins on your board, which can provide an easy connection to a third-party relay or h-bridge. But ideally, I would be able to do this without adding any new hardware to the board.

\n\n

This question is on-topic because it has to do with 3D printer boards that are currently on the market and because it is ultimately a device that is used to 3D print something.

\n", "Title": "Getting a programatically-controlled, constant 12 V from one of the motor outputs", "Tags": "|marlin|stepper-driver|", "Answer": "

It is easy. Leave the driver in. Connect to the first two wires usually used by the motor output. I.e. this is usually where the nema 17 motor goes. The first two wires are your output to the valve.

\n\n

Use M42 to control the ENABLE pin. Bringing the pin high creates a 0V output. Bringing the pin low creates a 12V output.

\n" }, { "Id": "13516", "CreationDate": "2020-04-24T22:12:46.453", "Body": "

I\u2019m building my new 3D printer and I have a doubt. I\u2019m using limit switches on the 3 axes, but only the MIN switches. I also have another three switches which I want to use as MAX switches. How can I do that? I mean, if I insert a specific measure of the plate in Marlin and than the extruded stops before of the limit of the plate due to the MAX endstop what will happen? What should I do, upgrade to MIN and MAX endstops or only keep the MIN ones?

\n", "Title": "How do the MAX endstops work?", "Tags": "|marlin|endstop|axis|", "Answer": "

You should see MAX endstops similarly as MIN endstops, or as an additional option to protect the printer:

\n\n
    \n
  1. Similarly seen as MIN endstops, you would define in the firmware that you are using them to set a reference point at maximum values, your offsets and bed dimensions would be calculated from the MAX positions;
  2. \n
  3. You can configure the MIN endstops and have additional MAX endstops set beyond the bed size dimensions. If the printer would encounter a layer shift causing the nozzle to go beyond the printer (MAX) boundaries, the printer shuts down when the MAX endstop is triggered.
  4. \n
\n" }, { "Id": "13519", "CreationDate": "2020-04-25T02:07:41.443", "Body": "

I recently purchased a BIGTREETECH SKR mini E3 V1.2 and need to adjust some of the settings in configuration h to accommodate for my custom built 3d printer. In the past I've used the RAMPS 1.4 board and adjusted the firmware in the arduino IDE. What is the best way/recommended way to do this for the mini E3 V1.2.

\n\n

Thanks.

\n", "Title": "How to access BIGTREETECH firmware", "Tags": "|marlin|firmware|bigtreetech|", "Answer": "

The previous answer is good, but here's one specific for Bigtreetech.

\n\n
    \n
  1. Install platform.io. I use the command-line interface (CLI)
  2. \n
  3. Modify your marlin files. You can clone existiing firmware for your board from the BigtreeTech Github for your board.
  4. \n
  5. Remove microUSB card from your Bigtree tech board
  6. \n
  7. Plug microUSB card into microUSB reader, and the microusb reader into the computer. You should be able to read your microUSB card
  8. \n
  9. Enter the command in the root directory of your Marlin files: platformio run -e STM32F103RC_bigtree_USB. At least, this is the one for my board. You should have to run this in one folder before the Marlin folder.
  10. \n
  11. It creates a file called firmware.bin in the directory .pio/build/STM32F103RC_bigtree_USB/firmware.bin. Copy it to your microUSB card, replacing and deleting any existing .bin. You can use the name firmware.bin.
  12. \n
  13. Remove microSD card from computer and plug into board.
  14. \n
\n\n

In your Marlin Configuration.h file, there should be a variable called something like MACHINE_NAME. If you make that name custom, then it will appear in the Octoprint terminal when connecting to your board, letting you know that you have updated the firmware.

\n" }, { "Id": "13523", "CreationDate": "2020-04-25T18:30:09.507", "Body": "

I'm new to printing resin miniatures for Dungeons & Dragons and most of my prints are successful, i.e. one or more miniatures print as expected.

\n\n

However when I have multiple minis on the build plate the one in the middle works okay but the ones on the edges don't adhere to the build plate.

\n\n

Should I limit myself to one or two minis in the center of the build plate? Or should it work and I just need to get my settings correct?

\n\n

Note I'm using a Beam 3D Prism printer.

\n", "Title": "Multiple objects on build plate", "Tags": "|resin|", "Answer": "

Check the plate and be sure that it is level. Also, you need to check that your model is level.

\n

The problem will be in the settings if everything is level.

\n" }, { "Id": "13526", "CreationDate": "2020-04-26T05:58:11.507", "Body": "

I was just about to start using my 3D printers heated bed to warm a chemical reaction in a container and was thinking it would be great to be able to get the bed stepping back and forth to stir the pot.\nCan anyone already up to speed in programming G-code walk me through a quick and dirty way to get the X-axis on my old Printrbot metal doing a couple of micro-steps either way in an endless loop? Or suggest some software out there that could achieve the same effect?

\n", "Title": "How to make a printer bed vibrate using g-code (heated chemical stirrer)?", "Tags": "|g-code|desktop-printer|scripts|", "Answer": "

Stefan from CNC kitchen already did it: https://github.com/CNCKitchen/3D-Printer-Vibration

\n" }, { "Id": "13530", "CreationDate": "2020-04-26T09:34:19.323", "Body": "

I previously set up my Ender 3 with a precompiled binary of Marlin that already had the correct X and Y offsets for the attached BLTouch. Now I'm compiling my own, and rather than measure for myself I'd like to just use the same numbers.

\n\n

When I send M851 through the terminal, it gives me the current value for the probe's Z offset from the extruder. I know I can also use M851 to set the X and Y offset, but there doesn't seem to be a way to read the current values.

\n\n

Is there any way to query the printer for the current values of X_PROBE_OFFSET_FROM_EXTRUDER and Y_PROBE_OFFSET_FROM_EXTRUDER? Or, ideally, for any other arbitrary variable?

\n", "Title": "How can I determine the current value of a #defined variable?", "Tags": "|marlin|g-code|bltouch|", "Answer": "

\"#define\" is a feature of the compiler preprocessor and isn't a feature of Marlin. So there is no general way to recover the value of a \"#define variable\" (which isn't actually a variable at all) unless Marlin explicitly provides the option.

\n\n

In the current version of Marlin M851 will report the X and Y offsets. It appears you have an older version which does not do this. In older versions there appears to be no option to recover the value.

\n" }, { "Id": "13546", "CreationDate": "2020-04-28T05:34:59.320", "Body": "

I have and Ender 3 Pro, which I'm enjoying except for an intractable problem. No matter how carefully I level the nozzle and tighten the X belt, using this great guide, the prints seem to be tilting to the right, particularly if they're tall. I realize this is a common problem, but I'm at a loss for what it's called and where the problem is most likely to be.

\n\n

The first sign of trouble in paradise is that, after two layers of a simple rectangular base for a Galton Board I'm trying to build that uses sand, the right side is clearly thinner than the left:

\n\n

\"enter

\n\n

This is with a translucent Overture PETG filament, 1.75 mm.

\n\n

For the board itself, which is about 15 cm tall and looks MUCH better if printed vertically, I notice after about 3-4 cm that the bottom of the left side is becoming detached from the bed:

\n\n

\"enter

\n\n

This gets progressively worse as the print goes on:

\n\n

\"enter

\n\n

The entire Galton Box takes six hours to print using the default PETG settings in Cura for a default level of detail, so I'd really prefer to find a way to diagnose the problem sooner. I know this is not a new problem, but I'd be grateful for any guidance on what it's called or links to how to fix it -- and how to determine if it's a hardware problem or not.

\n\n

Also, out of curiosity: If the machine isn't PERFECTLY level on the table -- my work desk appears to slant by about 2° -- could that be implicated?

\n", "Title": "Diagnosing why the left side of a print detaches from the board", "Tags": "|creality-ender-3|heated-bed|bed-leveling|adhesion|", "Answer": "

The uneven level is explained by the unfortunate choice of the printer manufacturer to not have a second Z lead screw. It is pretty difficult to level these printers, it is all a combination of the correct X belt tension and roller tension. The uneven level in the first picture is what is causing your print to detach, As can be seen you've got a lot of \"squish\" on the right side, but less adhesion (seen by the less transparent color) on the left, therefore it would fail on the left.

\n\n

You need to:

\n\n\n\n
\n\n

Please note that the level of the table top has nothing to do with the leveling of the printer, nor with the leaning of your prints. You're just having adhesion problems as a result of an uneven bed/gantry.

\n" }, { "Id": "13548", "CreationDate": "2020-04-28T19:37:49.080", "Body": "

I recently purchased a BigTreeTech mini e3 V1.2 controller for my custom built 3D printer (as opposed to an pre-purchased ender 3). In doing so, I ran into the problem that the endstops I have been using have three wires: Power, Ground, and signal. The mini e3 V1.2 that I purchased uses endstop connections that only have two pins. This can be seen in the schematic below on the bottom right of the controller. I don't know where to buy endstops that only have two pins and have not been able to find anything online. If anybody knows where these can be bought or how the 3 pinned endstops are supposed to be wired up it would be much appreciated.

\n\n

\"enter

\n", "Title": "BigTreeTech mini e3 V1.2 endstop installation", "Tags": "|wiring|endstop|skr-mini-e3|bigtreetech|", "Answer": "

You do not need to connect the red (+) wire of the endstops. The controller board uses a technique to ensure the signal is 3.3 V or ground, see the top left schematics of this link (will insert an image later!). Specifics about this technology is found here and is explained as using a pull up resistor.

\n\n

What happens is that the signal is always reported as triggered, unless the endstop connects the signal wire to ground.

\n\n

This link shows that only ground and signal are required (will insert an image later!).

\n" }, { "Id": "13557", "CreationDate": "2020-04-29T22:19:08.280", "Body": "

This is the same machine involved in this question: ender-3-pro-extruder-stepper-skipping-with-a-chunk-sound.

\n\n

After reassembling the extruder, I discovered that the extruder stepper did not move, using prepare->move-axis. All three axis steppers did move normally.

\n\n

The first thing is did to test was to exchange the wires to the extruder and X-axis steppers. After this the extruder stepper moved when I moved the X-axis but the X-axis did not move when I moved the extruder. Therefore, I concluded that the stepper is working.

\n\n

I then opened up the main board chassis and exchanged the X-axis and extruder cables there (so the cables are exchanged on both ends). This restored operation to the X-axes (operated as such) and the extruder did not function again (operated as such).

\n\n

From this, I concluded that the cables are also fine and I had somehow blown the extruder driver on the board.

\n\n

I went on Amazon and ordered a \"silent\" Ender 3 Pro motherboard, rev 1.1.5 to replace my current rev 1.1.4 board. I just received the new board and, as far as I can tell, it looks like an authentic Creality product. The only visible differences between the old and new boards are the silkscreened version and the color of the PCB itself. Both are labelled \"Ender 3 Pro\" on the back.

\n\n

I carefully removed the old board, marking where all cables connect and swapped in the new board. Much to my distress, everything acted exactly the same. The extruder stepper did not turn. Swapping the cables at the steppers restored functionality to the extruder (controlled as X-axis) and the X-axis did not move (controlled as extruder). Also swapping the cables at the motherboard restored the X-axis stepper (controlled as such) but the extruder stepper would not move (controlled as extruder).

\n\n

I should add for Completeness that my printer was sold by Sain Smart (Creality OEM, I believe) and was labelled as such, both on the metal and on the LCD start screen. However, the motherboard was a Creality3D board, labelled as \"Ender 3 Pro\" and I believe the only actual difference is the text inside the firmware. With the new board in, the LCD identifies itself as \"Creality Ender 3 Pro\".

\n\n

At this point, the only conclusion I can come to is that the new board has failed in the same way, which doesn't make any sense.

\n\n

I need a sanity check. Does anyone see any flaw in my logic?

\n\n

I should add that there is an oddity on the new board. The fan on front of the hot end seem to run at full speed and the hot end side fan and chassis fan run briefly at power up an then stop. Using control->temperature->fan-speed seems to have no effect on any of the fans. I never observed the chassis fan with the old board so I cannot say this is different but I did previously have control of the hot end fan.

\n", "Title": "Need sanity check debugging non-functional extruder", "Tags": "|creality-ender-3|stepper-driver|stepper|", "Answer": "

The extruder motor will not turn unless the hotend is at a certain minimum temperature. This is a safety feature to prevent the extruder from grinding through the filament with a cold hotend. You can use the M302 command to control this behavior.

\n" }, { "Id": "13559", "CreationDate": "2020-04-30T06:37:04.533", "Body": "

I have been working hard the last year on the model below. I am new to everything 3D that is modeling and especially 3D printing. I have however successfully concluded quite a good number of 3D prints which I created in blender with my Creality Ender 3 Pro so I have a bit of experience.

\n\n

All this new experience for me started with the desire to do this project I have been working on for all this year. A complex 3D model of a knight's tomb which I would like to print.

\n\n

\"enter

\n\n

As you can see Cura is clearly indicating that there is a need for supports in these red areas. The model will be printed in a 15 cm size. I have also managed to resize the model from a 22 million face mesh full of holes to a 900k manifold model.

\n\n

However, when I slice it I get this.

\n\n

\"enter

\n\n

As you can see supports are only generated for the outer column part. None are generated for the arches which are totally absent when the model is sliced.

\n\n

I have tried to alter the model's orientation but with no result. I will try to separate this mesh in parts but it would mean restarting all from scratch since I found no good software to slice it precisely.

\n\n

I am quite sure that the main problem lies in the fact the Ender 3 pro is an FDM printer an that the vertical lines of the arches are too thin. Since I tried to upload my model on 3D printing services to see if it could be printed in other materials and could be printed in finely detailed resin.

\n\n

I would like at least to know if I am right in my problem spotting or if there could be any solution to the present problem so that not to go wrong if I will redesign this part.

\n", "Title": "My 3D model is printing with missing parts when sliced in Ultimaker Cura", "Tags": "|ultimaker-cura|3d-models|3d-design|fdm|support-structures|", "Answer": "

Why there is no support on the top is, that the top of the model, over the gothic arch, has not enough thickness to have any layer laid down there. Because it is too thin, it gets omitted. However, since the area that needs support is only very very small, this area technically doesn't need support and could be printed with bridging alone. Yet, you need to add at least 1 layer thickness to the height of the model by extruding the top surface in blender to make this area printable.

\n

Another way is to knock down the layer height.

\n" }, { "Id": "13566", "CreationDate": "2020-05-01T11:08:31.700", "Body": "

I just changed the motherboard of my CR-10 mini printer to an SKR 1.4 and installed Marlin 2.0.x on it. Everything seems to work except for the automatic bed leveling. I\u2019m using an EZABL probe.

\n\n

The probe is mounted to the left and slightly in front of the nozzle. Therefore I have set the probe-nozzle offset in Marlin as:

\n\n
#define NOZZLE_TO_PROBE_OFFSET { -45, -12, 0 }\n
\n\n

When I run automatic bed leveling with G29 the nozzle moves to a position where the sensor is outside of the bed (but the nozzle is inside the bed). And then it crashes down into the bed with the nozzle because the Z endstop is never triggered.

\n\n

To debug I used the DEBUG_LEVELING_FEATURE in Marlin and ran G29 from Pronterface. This is what I got:

\n\n
SENDING:G29\n  current_position= X140.00 Y90.00 Z5.00 : >>> G29\nMachine Type: Cartesian\nProbe: FIX_MOUNTED_PROBE\nProbe Offset X10.00 Y10.00 Z0.00 (Right-Back & Same Z as Nozzle)\nAuto Bed Leveling: BILINEAR (disabled)\n  current_position= X140.00 Y90.00 Z5.00 : Probe::set_deployed\ndeploy: 1\nProbe::move_z(5.00)\n>>> do_blocking_move_to X140.00 Y90.00 Z5.00\n<<< do_blocking_move_to\nreset_bed_level\n>>> Probe::probe_at_point(30.00, 30.00, raise, 0, probe_relative)\n  current_position= X140.00 Y90.00 Z5.00 :\n>>> do_blocking_move_to X20.00 Y20.00 Z5.00\n<<< do_blocking_move_to\n  current_position= X20.00 Y20.00 Z5.00 : Probe::set_deployed\ndeploy: 1\n  current_position= X20.00 Y20.00 Z5.00 : >>> Probe::run_z_probe\n  current_position= X20.00 Y20.00 Z5.00 : >>> Probe::probe_down_to_z\n>>> do_blocking_move_to X20.00 Y20.00 Z-3.00\n<<< do_blocking_move_to\necho:busy: processing\n  current_position= X20.00 Y20.00 Z-3.00 : sync_plan_position\n  current_position= X20.00 Y20.00 Z-3.00 : <<< Probe::probe_down_to_z\nFAST Probe fail!\n  current_position= X20.00 Y20.00 Z-3.00 : <<< run_z_probe\n  current_position= X20.00 Y20.00 Z-3.00 : Probe::set_deployed\ndeploy: 0\n>>> do_blocking_move_to X20.00 Y20.00 Z-3.00\n<<< do_blocking_move_to\nError:Probing Failed\n[ERROR] Error:Probing Failed\n
\n\n

What I don\u2019t understand is why it says:

\n\n
Probe Offset X10.00 Y10.00 Z0.00 (Right-Back & Same Z as Nozzle)\n
\n\n

Because that is not at all the NOZZLE_TO_PROBE_OFFSET I have defined. Is there another setting related to the nozzle/probe offset besides NOZZLE_TO_PROBE_OFFSET that I have missed? Or why does Marlin get the idea that my probe is to the right and back of the probe when in reality its to the front and left?

\n", "Title": "Automatic bed leveling probes outside of bed despite NOZZLE_TO_PROBE_OFFSET", "Tags": "|marlin|ezabl|", "Answer": "

The problem is that, although you have set the offsets correctly, the boundary limits for the probe to \"probe the area\" have not been adjusted. It is my belief that this is a flaw in ABL of Marlin 2.0.x. If the user has set the probe offsets, the firmware already knows that the probe area is limited!

\n\n

This answer on question \"How to set Z-probe boundary limits in firmware when using automatic bed leveling?\" describes in detail what the boundaries of the probe are for Marlin firmware 1.1.x and 2.0.x.

\n\n

Basically, you need to set the boundaries of the probe using some extra definitions in your Configuration_adv.h file:

\n\n
\n  constexpr int nozzle_to_probe_offset[] = NOZZLE_TO_PROBE_OFFSET;\n  #define PROBE_X_OFFSET_FROM_EXTRUDER nozzle_to_probe_offset[0]\n  #define PROBE_Y_OFFSET_FROM_EXTRUDER nozzle_to_probe_offset[1]\n
\n\n

When including the MIN_PROBE_EDGE this means that the area is defined as:

\n\n
\n  #define MIN_PROBE_EDGE_LEFT (MIN_PROBE_EDGE)\n  #define MIN_PROBE_EDGE_RIGHT (MIN_PROBE_EDGE - PROBE_X_OFFSET_FROM_EXTRUDER)\n  #define MIN_PROBE_EDGE_FRONT (MIN_PROBE_EDGE)\n  #define MIN_PROBE_EDGE_BACK (MIN_PROBE_EDGE - PROBE_Y_OFFSET_FROM_EXTRUDER)\n
\n\n

(Since the offsets are negative in your probe setup, we need to subtract the values in order to add up the absolute value to the edge...)

\n" }, { "Id": "13585", "CreationDate": "2020-05-04T04:39:21.517", "Body": "

I have an Ender 3, I got a new glass bed, the bed comes with glue on the back.

\n\n

Should I stick the glass bed to the aluminium base? or just use it with the clips?\nI saw other people just use the clips, but my glass seems to have a sticky back...

\n\n

\"enter

\n", "Title": "New glass bed, should I glue it?", "Tags": "|creality-ender-3|glass-bed|", "Answer": "

Use Clips on either the left or the right. The aluminum might not be flat. Using clips or the adhesive will cause the glass to warp, defeating the point of using the glass.

\n" }, { "Id": "13603", "CreationDate": "2020-05-05T10:00:47.137", "Body": "

I've got a secondhand Renkforce RF1000 which I can't seem to get working properly. Initially, I asked about hardware issues that required fixing. After fixing those I still got some inconsistent printing results. These problems got worse with a new nozzle and new filament. I've now narrowed it down to one problem: bad feeding

\n\n

To summary:

\n\n\n\n

\"feeding\nThe feeding mechanism

\n\n

The problem seems to be that the pressure for extruding filament through the nozzle is too high. This causes the grub screw to keep spinning without extruding filament. So in the picture, the left screw will spin and the right one will remain stationary.

\n\n

This isn't consistent, it might work for a few millimetres and then stop working for a bit. It can cause the screw to grind into the filament creating a hole that cuts the filament in two.

\n\n

Here's what I've done to try and fix it:

\n\n\n\n

I'm out of ideas of how to fix this. Any help you can give is greatly appreciated.

\n\n

EDIT: here are some video's extruding 50 mm of filament at 2 mm/s and 10 mm/s

\n\n

\"Controls\"\nControls

\n\n

10 mm/s

\n\n

2 mm/s\n(both links to imgur, didn't manage to get the video inline)

\n\n

EDIT 2:

\n\n

I replaced the driving gear and replaced the new nozzle with an old one. I'm not sure about the size of the old one but I assume it is 0.5 mm. I calibrated the extrusion and tested at 1 mm filament per second, I now get the exact results. However, when printing two 20mm calibration cubes I get quite bad results. The first cube (left) is printed at a 1.0 extrusion multiplier and the second one (right) is printed with a 0.95 extrusion multiplier. Any idea what these quality problems are caused by?

\n\n

[\"Progress\n[\"New\n[\"Speed\n[\"Print

\n", "Title": "How to fix bad extrusion that's likely caused by high pressure in the nozzle", "Tags": "|extruder|renkforce-rf1000|", "Answer": "

I think you are pushing filament too fast.

\n\n

Let's go back to the basics: my hotend (old Ubis, ceramic) and extruder can push PLA at 230\u00b0C at max 80 mm/s * 0.5 mm line width * 0.2 mm layer height = 8 mm3/s.

\n\n

At this speed it doesn't extrude very well, there is too much back pressure (see also

\r\n \r\n
). That extrusion speed means (I have 3 mm filament) 1.13 mm/s of filament speed (8 mm3 / 1.52 / pi).

\n\n

You have 0.3 mm nozzle and you print colder, both of which cause more resistance to the flow. If 8 mm3/s is my absolute max, yours should be about 8 * 9/16 (ratio of the nozzle surfaces) = 4.5 mm3/s (really the max).

\n\n

Considering the different print temperature, I would start limit to 3.5 mm3/s, which is 0.55 mm/s extrusion speed.

\n\n

If you print 0.15 mm layers with line width of 0.35 mm, your absolute max printing speed should be 3.5 mm3 / 0.15 /0.35 = 65 mm/s (which is better not to reach, 60 is fine).

\n\n

Try doing the following tests: after cleaning the driving gear, extrude 50 mm filament at 0.3 mm/s and measure how much filament has been extruded. Then repeat 50 mm with 0.5 mm/s, and measure how much filament has been extruded. Try again at 0.75 mm/s. Obviously don't touch E steps and extrusion multiplier. I'm quite sure that at 0.75 mm/s you will notice a measurably shorter length of filament extruded. At 1 mm/s you will see grinding (but not as much as now).

\n\n

If you want, try at regular intervals 0.2, 0.3 0.4, 0.5, 0.6, 0.7 mm/s and plot a graph of the actually extruded filament length. It will look like the one in the video (where he extruded a lot more and weighed the filament, which is time consuming and more expensive).

\n\n

And then switch to 0.4 mm nozzle if it's too slow for your needs.

\n\n

Additional information

\n\n

You use 2.85 mm filament with a direct drive, no gears. Judging from your video the radius if the driving gear (teeth) is about 4 mm, meaning 4*2*pi=25.1 mm circumference. The circumference is controlled with 200 steps * 16 microsteps, as result each microstep controls 4*2*pi/200/16 mm filament length, which is 0.05 mm3 and what you extrude along a 0.95 mm length (at 0.15 mm layer height and 0.35 mm line width). Basically your extruder has no control for moves shorter than 0.95 mm, but in fact it's even 4x worse, since you never get a single microstep precision (4 microsteps of tolerance is more reasonable).

\n\n

You should probably use a much bigger nozzle, or 1.75 mm filament, or a geared extruder, or prints will never be accurate and you will have problems all the time, which you cannot physically solve.

\n" }, { "Id": "13632", "CreationDate": "2020-05-09T20:18:29.180", "Body": "

I've had the Ender 5 Pro, as is, for 3 months.

\n\n

After I tried a cheap PLA filament (maybe too cheap), it clogged the nozzle. I cleaned it, but any other filament I have would have similar problems from then on. After some days doing test prints, it clogged again. This time, I heated it up to 240 \u00baC, I unscrew the nozzle, cleaned it up, chopped the Capricorn tube (it had clear symptoms of a previous tube not reaching the bottom of the hotend and leaking material around it), perfectly aligned, cleaned the heating block with a brass brush, started screwing the nozzle\u2026 and it never reaches the end. It even jiggles a little bit (by \"jiggles\", I mean that when the hotend is hot, you can push the nozzle back and forth, and it does move... like it was a joint, and not a threaded bolt), like it was too small for the block. I try screwing a new nozzle. Same happens.

\n\n

To me it looks like the heating block thread broke, but I can't be sure. I tried a thicker nozzle (0.8 mm, but same thread in the end) and it worked. I could feed filament and it wouldn't look like clogged (at least for 15 cm of feeding filament, when in previous cases it was almost instantaneous).

\n\n

Time to get a new heating block? Any recommendation?

\n\n

Would another kind of heating block be better?

\n\n

Edit: Here's a picture of the bottom of the heating block. Now it's clear there's a metal chip glued with material, and almost (if any) no thread at all.

\n\n

\"broken

\n\n

Another question. Could it have been that cheap material that caused the clogging, or was it just a lack of proper maintenance?

\n", "Title": "Nozzle jiggles when screwing into heating block", "Tags": "|nozzle|creality-ender-5|", "Answer": "

You need to order a new heating block (just a few bucks/euros, so order 2, and spare nozzles, you don't want to install the old nozzles), you completely worn out the threads, this is beyond repair. It looks as if you used too much force to secure the nozzle into place.

\n\n

This is how it is supposed to look:

\n\n

\"enter

\n\n

You can find these cheaply at those typical Chinese vending platforms or online marketplaces. Just search for and Ender / CR-10 heater block.

\n\n

Typical dimensions are found below:\n\"enter

\n" }, { "Id": "13637", "CreationDate": "2020-05-10T12:08:36.423", "Body": "

When I try to print something that is not the test model already on the SD, which means when I try to slice something in Cura 4 and than I start the print, the nozzle stops at Z=15 mm and only the extruder motor goes foward and backward. I tried to modify the G-code but I didn't solve the problem. I need help because it's my first delta 3D printer and I don't know what to do.

\n", "Title": "FLSUN QQ doesn't print and stops at Z=15 mm", "Tags": "|ultimaker-cura|delta|flsun-qq|", "Answer": "

A distance of 15 mm is about the height of the probe or Z-offset. That can be a clue.

\n\n

Compare the G-code from the testprint with the G-code you sliced. It can give you other clues.

\n" }, { "Id": "13638", "CreationDate": "2020-05-10T20:19:32.777", "Body": "

When I load a figure in Cura 4.6.1 I get weird dots all over my figure in Prepare-mode.

\n\n

\"enter

\n\n

How come, and can I turn it off?

\n\n

It's on every model I load :( I am fairly sure the normals are facing the correct way as well.

\n", "Title": "Dots in Cura on Prepare-mode", "Tags": "|ultimaker-cura|", "Answer": "

Solution found!

\n\n

If Display model errors is disabled, the dots appears.\nCheck the setting, hit OK, click on the box and they disappear.

\n\n

\"enter

\n" }, { "Id": "13640", "CreationDate": "2020-05-10T22:10:37.333", "Body": "

I've recently configured my Bigtreetech mini e3 v1.2, and I have every aspect of the printer running well except for the initialized homing sequence (the problem is sometimes small enough to where I can still get a good print). Originally what I was running into was that because I used two Z stepper motors, any movement that wasn't manual input on Pronterface would cause the motors to vibrate and make noise (not enough current). Now the only time the Z motors make noise/vibrate is right after the homing is complete and they briefly move up when the nozzle begins to heat, as well as right after the heating is done and the nozzle dips down to begin the print. I suspect it's because of a fast increase in Z speed that I don't necessary need. I've tried changing a multitude of settings in Configuration.h but haven't had any luck. Does anybody know the command that dictates the Z motors to very quickly move upward just after homing and before heating?

\n", "Title": "Prevent raising Z-axis after homing", "Tags": "|g-code|z-axis|homing|bigtreetech|", "Answer": "

Just solved my own problem. Instead of looking in Marlin for the command, I found out the reason it was lifting up quickly was because of a custom command embedded in the G-code by the slicer. I got rid of the command and the problem was solved.

\n" }, { "Id": "13648", "CreationDate": "2020-05-12T20:38:33.183", "Body": "

I'm trying to compress different 3D files, but find it difficult to find the right software to compress the file.

\n\n

What are the most suitable 3D file compressors to compress 3D files like STL, OBJ and STEP?

\n\n

I have tried Draco, and mac zip compressor.

\n", "Title": "How do I compress 3D files", "Tags": "|3d-models|software|file-formats|open-source|obj|", "Answer": "

3D files differ greatly in size and what they contain:

\n

STL

\n

STL Stereolithography files were invented by 3D Systems to store surfaces. Originally it used ASCII text to store information by naming triplets of vertex positions for each triangle (facet). Since that got too large, newer STL are Binary, which is quite smaller.

\n

Many programs can export them, their size is dependant on the number of surfaces. You can reduce the size of an STL by lowering the number of surfaces at the cost of detail.

\n

OBJ

\n

OBJ was invented by Wavefront as a means of storing 3D information. It stores the data as plain text by storing vertices, to which they connect and what texture is on surfaces spun up by the vertices.

\n

In comparison to STL, they can be bigger if they include surface information. Programs that can't do STL usually support OBJ. Slicers take either. You can reduce the size of the file by reducing complexity.

\n

STEP

\n

STEP files don't save 3D items per se, they store instructions for CAD programs to generate a 3D item. This makes them extremely information-dense and can create highly complex items with a somewhat minimum of file size. They also allow us to easily modify the file.

\n

However, STEP files can't be sliced directly and need to be opened by a CAD program.

\n

Comparison

\n

\""simple"

\n

This is a simple object generated by a mere extrusion, rounding corners, extruding again and a sweep, then copying the item and moving it into position.

\n

\"Tools

\n

But how does that compare as STL and OBJ? Well, the results of this item are rather small in either case, but you get a rough gist of their general comparability.

\n

The STL is 74.3 kB, STEP is 90 kB, OBJ is 95.4 kB.

\n

\"STL\"

\n

\"STEP\"

\n

\"OBJ\"

\n

However, in a maximum compressed .zip archive, things change a lot:

\n\n

\"Compressed\"

\n

Conclusion

\n

STEP is the best to give out in a zip archive if you want others to edit it. OBJ is a tad smaller in a zip archive than STL, but also can contain additional data.

\n" }, { "Id": "13660", "CreationDate": "2020-05-14T08:29:45.800", "Body": "

From what I understand, UV curing of resin prints works by starting a chemical reaction that hardens the resin permanently.

\n\n

Also, a curing step after print is needed to speed up the print and also to reduce the curing during print, which would cure resin beyond the current layer.

\n\n

However, what is the transmissivity of UV light in partially cured prints? If I print a more massive object, or if I use an opaque resin, how deep will the object harden properly? Absorption is always exponential, meaning that it decreases quickly with depth.

\n\n

Depending on the resin, how thick prints can be effectively cured? This information is not provided with the resin, which I actually would expect from reputable manufacturers.

\n", "Title": "Can UV cure inside opaque or more massive printed object?", "Tags": "|resin|", "Answer": "

My background

\n\n

I am not a polymer chemist. I know some, and I've talked about UV curing resins with them. These are my conclusions based on that informal education. Your expertise may well be better.

\n\n

Minimizing curing on the printer is good

\n\n

For the maximum print rate and best layer adhesion, the resin should cure during the printing exposure the minimum amount that is consistent with the object supporting itself during the print, being removable from the bed, being rinsed in isopropyl alcohol, and stand on its own during curing.

\n\n

If you are curing too completely during exposure, you won't get optimal mixing of polymer chains between layers. The layers will be attached by the glue-like action of the new layer on the old, rather than having their polymer chains cross-linked and extending between layers.

\n\n

If you are curing too little, the object won't be strong enough to support itself and survive post-processing. Soft, gummy bits may dissolve, or be swept away by currents in the cleaning solution. Small details may not be robust enough. Supporting structures (as added supports or parts of the model) may not be strong enough to resist gravity and handling.

\n\n

If you are curing too completely, your print will take longer than it could take.

\n\n

Curing is not binary

\n\n

Curing will take place spontaneously over a long time. If it didn't, a bottle of resin might last a very long time. But, curing is not a self-catalyzing process that runs quickly to completion. If it were, then the first stray UV that came through an open bottle top would turn the contents solid. If exothermic (which it seems like it must be), the bottle would get hot.

\n\n

UV curing may fail

\n\n

UV curing the inside of a large, opaque object probably doesn't happen. Before I get all excited, though, I would need to place some bounds on \"large\" and \"opaque\".

\n\n

Absorption of UV light depends on the pigment or dye used in the resin. This absorption is never absolute. It is not 100% gone after the first, thinnest penetration. The light is attenuated by an amount per distance it penetrates.

\n\n

Transmission is the complement of absorption, and the numbers are easier to work with, so lets work with transmission factors of T rather than absorbtion factors of A. $T=1-A$

\n\n

If a factor of T is passed for millimeter, then one centimeter into the object the light intensity is $T^{10}$ of what it is on the surface, which is a small, but non-zero, number.

\n\n

Keep in mind that opaqueness depends on wavelength. For example, Window glass is transparent (very little absorption) to visible light, but highly attenuating to UV light. Were I to design a black resin, I would look for a black pigment that was relatively transmissive to UV.

\n\n

Will it cure?

\n\n

A low UV dose delivered to a 0.2mm layer of resin will partially cure the resin. 1 mm into an object, the dose is lower, but it still exists. 1 meter into the object, the absorption is probably too high pass a useful level of UV.

\n\n

If the transmission factor is 0.8 for a 0.2 mm layer, it is $0.8^5$ for a 1 mm layer (0.33). It would take only three times as much UV exposure to cure a 1 mm thickness as a 0.2 mm thickness. If the object were 1 meter thick, the transmission to the inner bit would be $0.8^{5000}$, which is a very tiny number, roughly $2.82\u00d710^{-485}$.

\n\n

Finally, consider if the object is or must be truly be solid. UV curable resin is relatively expensive. Many UV printable objects include drainage paths for uncured resin to flow out of the object during printing. Perhaps your object could similarly be hollowed out.

\n" }, { "Id": "13678", "CreationDate": "2020-05-16T19:45:36.947", "Body": "

I think that heated bed have some disadvantages - they arent always reliable, they consume additional electrical power.

\n\n

I can imagine one way how HB can be replaced - in some flat surface - glass or iron sheet we can drill some specific holes (probable a bit conic). Then, before detail printing start, printer could fill it with material and continue print detail connecting to these fixations. That can fixate detail in X,Y and probably even Z axis. After printing we can just take detail or cut fixations (if there were Z fixations)

\n\n

I almost sure someone in industry already thought about this bud didnt release and I dont understand why. Defineteley there is pitfails, but I dont see them. Why this method cant be used to replace heated beds?

\n", "Title": "Use fixation holes instead heated bed", "Tags": "|heated-bed|adhesion|", "Answer": "

I've used a 3D printer that had a perforated (and heated) bed for adhesion. It was some incarnation of the Up printer -- and this was several years ago (2014 or 2015) when 3D printing was very hyped but hadn't reached its present level of technological maturity.

\n\n

(this was also before things like BuildTak and PEI sheet.)

\n\n

The bed was made of a perforated fiberglass material similar to PCB boards (without soldermask or copper), black in color, and it was terrible. Prints were printed with a raft, which remained stuck to the bed holes and resisted all efforts to remove them short of a chisel or something similar, and before long all the bed plates (there were several) were marked and gouged from people trying to scrape plastic off of them. (they also warped).

\n\n

I do not think that this is something we want to imitate, especially as it seems like Buildtak-like plastic coatings or glass work so well.

\n\n

As to the use of holes alone to fix parts: They are going to be under stress, possibly too much shear stress for them to hold, you won't get the other benefits of heating, and you'll have trouble as you build upwards. For an extreme example, look at the difficulties of printing the insanely high temperature plastic PEEK, which basically requires not merely a heated bed but a heated chamber.

\n" }, { "Id": "13686", "CreationDate": "2020-05-17T11:26:30.227", "Body": "

I'm trying to print the painter's tripod from this tutorial:

\n\n

\"enter

\n\n

I can't get it to finish the print. It always fails towards the top when it's closing the circles. I've tried brims and even glue for adhesion, and different settings for inlays, perimeters, etc... my bed levelling is correct and my first layer is perfect. The prints come out beautiful, but they always fail there. I have a theory but since I don't have a camera set up I can't confirm, but I feel like the nozzle gets stuck against the previous layer, as if when it's coming from the other side of the \"bridge\" completing the upper layers of the circle, it get's to the other side of the gap and it finds that the filament has hardened just a bit higher than when it's going to print, and therefore knocks the piece to the side (despite using glue!!), and from that point on, obviously, total mess and I have to cancel. Just a theory though. Any ideas of what else could this be or what to do about it?

\n\n

Printer is a Prusa i3 Mark3S

\n", "Title": "Can't print the top part of a circle", "Tags": "|prusa-i3|print-failure|", "Answer": "

You are trying to print an unsupported edge up there - the top edge has nothing to rest on and thus sags down. As a result, the print failed.

\n\n

To remedy this, activate printing with support. With a support angle of 80\u00b0 or tree support, you could minimize the needed material.

\n" }, { "Id": "13691", "CreationDate": "2020-05-18T00:57:59.173", "Body": "

Working on upgrading my Ender 3 to a dual extruder setup using the Winsinn dual hot end and MKS Base V1.6 motherboard using Marlin FW 2.0.5 and am unable to get the PID autotune to work. When I start the autotune, the temperature spikes pretty quickly and quickly fails, either heating to 60 °C or 200 °C (M303 C5 E0 S60) for either hot end (E0 or E1). After looking through other online messages, I used 24 V heater cartridges rather than the 12 V ones that came with the hot end. Aditionally, I have the thermisistor settings to use config 6 (rather than 1) after reading through user recommendations on the hot end. After failing to get the hotend to heat properly, I have the following questions:

\n\n
    \n
  1. Should I try reducing the current to the cartridge? Currently set to default of 255?

  2. \n
  3. Are there any other PID changes I should make to allow the autotune to figure out the right parameters to use?

  4. \n
  5. Are there any other tests recommended to figure out why the hot end heats without being stopped by Marlin?

  6. \n
\n\n

Here is a chart showing the temperatures of the hotends and bed while running autotune on T0 and T1. Weird gap in second attempt was verifying that the right sensor was plugged into the right mainboard pin (no power going to hot end at this time).

\n\n

\"PID

\n\n

Thermistor settings:

\n\n
#define TEMP_SENSOR_0 6\n#define TEMP_SENSOR_1 6\n
\n", "Title": "PID autotune fails with a temp too high", "Tags": "|marlin|creality-ender-3|hotend|pid|", "Answer": "

As the heating curve is very steep, it could be that the wrong cartridges are inserted, or you have been supplied the incorrect cartridges. (Not long ago I've had a similar experience with a 12\u00a0V cartridge in between my 24\u00a0V spares...)

\n

To find out for which voltage the cartridge has to be used you can measure the resistance with a multitester/multimeter. You can calculate the resistance roughly by using the formulas:\n$$ U = I \\times R$$\n$$ P = {U}\\times{I} $$\nCombing these formulas gives:\n$$ R = \\frac{U^2}{P}$$

\n

(P is power in Watt [W], I is current [A], U is voltage [V] and R is the resistance in Ohm [\u03a9])

\n

Your multimeter readings should be close to the calculated values. About 4 Ohms for a 12\u00a0V/40\u00a0W cartridge and about 14 Ohms for a 24\u00a0V/40\u00a0W cartridge.

\n

Since the Ender 3 is running on 24\u00a0V, you need the higher resistance cartridges.

\n
\n

If you are using a 4\u00a0\u03a9 (12\u00a0V/40\u00a0W) cartridge on 24\u00a0V, the power would become:

\n

$$ P = \\frac{U^2}{R}= \\frac{24^2}{4}=144\\ {W} $$

\n

This amount of power will quickly raise the temperature of the nozzle! It then becomes very difficult for the PID control schedule to harness that power (e.g. overshoot control).

\n

From the question is read that:

\n
\n

I used 24 V heater cartridges rather than the 12 V ones that came with the hot end

\n
\n

If the cartridges are truly 24\u00a0V this rapid heat-up is not expected, it could be that you accidentally received incorrect cartridges, you should measure the resistance to be sure.

\n" }, { "Id": "13695", "CreationDate": "2020-05-18T23:31:20.633", "Body": "

I have a Creality Ender 3 that needs a build surface (both the build-tak sticker and the removable stiff backer). My dad has been pestering me to try using paper as a disposable surface. I am hesitant to do so for concerns regarding fires and degrading the bottom of the print. Has anyone here tried this and what were the results?

\n", "Title": "Paper as build surface - how will that work?", "Tags": "|material|safety|build-surface|", "Answer": "

Blue masking tape is a common bed surface, especially for unheated beds. It is effectively an adhesive applied to a paper, although I suspect the paper is treated in some manner to serve as a barrier to paint, as the blue tape is sold as paint masking tape.

\n\n

A sheet of ordinary paper isn't likely to catch fire, as the bed temperature will not reach combustion temperature for paper (more than 230 \u00b0C), but you still have to have the means of providing adhesion of the paper to the bed.

\n\n

A thick enough weight of paper may provide sufficient rigidity to provide for a stable print surface, if the perimeter is well secured and uniformly (smoothly) attached.

\n\n

You'll have to experiment with various types of paper surfaces as well, as some may be treated (calendared) during manufacturing, which could affect the results.

\n\n

Blue tape is your best bet if you want disposable. Also if you use care in removal, it is not a single use bed covering.

\n" }, { "Id": "13702", "CreationDate": "2020-05-19T11:14:17.087", "Body": "

I was printing a lid for a box in my Ender 3 pro printer. But I don't know why it stopped printing and moved aside when it does in finished printings. This happened at midnight and I didn't see it until I woke up. In the hope for a resume I didn't remove the finished part from the printer bed. So now how can I resume this printing from where it left. The total width of the lid was 3\u00a0mm and it printed 2.4\u00a0mm now only 0.6\u00a0mm part left to print.

\n

\"photo

\n

\"photo

\n

\"photo

\n", "Title": "How to resume an unfinished project?", "Tags": "|ultimaker-cura|creality-ender-3|troubleshooting|g-code|slicing|", "Answer": "

Provided that the print hasn't come loose from the bed and you know the layer height or layer number (you could count the amount of layers or measure the height of the print using a caliper), you can edit the original G-code file to print the remainder of the print. The following hints should be taken into account:

\n\n

What you can also do is:

\n\n

Note that you will always see a difference in appearance compared to an object printed in one session, you will see an interface line between the first and the second part.

\n" }, { "Id": "13705", "CreationDate": "2020-05-19T17:25:33.937", "Body": "

Adding custom M Codes to Marlin doesn't work for Marlin 2.0

\n\n

How would one go about adding custom G codes or M Codes to Marlin 2.0? The Marlin_main.cpp file does not exist.

\n\n

In general for Marlin 2.0, things are organized better, but split into more files.

\n", "Title": "Adding custom M or G codes to Marlin 2.0", "Tags": "|marlin|", "Answer": "
    \n
  1. Choose a code in the >10,000 in case new codes are added. But in this example I will choose 13
  2. \n
  3. Navigate to 'src' folder of Marlin
  4. \n
  5. Edit the file gcode.cpp around line 223 to have a new unused number. For example, this will create a new G code function for the label G13.
  6. \n
\n
  ...\n  // Handle a known G, M, or T\n  switch (parser.command_letter) {\n    case 'G': switch (parser.codenum) {\n      case 13: G13(); break;\n      case 0: case 1: G0_G1(  \n  ...\n
\n
    \n
  1. On line 375 of gcode.h add: static void G13(); to declare it.

    \n
  2. \n
  3. In my case i was reading values from an analog system. So I went to src/temperatures and copied M105.cpp to be G13.cpp. Then inside the file I replaced GcodeSuite::M105 to be GcodeSuite::G13. I am using this to take in the weight of something using a [scale][1] but for now I just want to test functionality so here is my test function:

    \n
  4. \n
\n
\nvoid GcodeSuite::G13() {\n\n  SERIAL_ECHOPGM(MSG_OK);\n  SERIAL_ECHOLNPGM("here is where weights are broadcast");\n\n}\n
\n

And again this is the only part I changed in my new copy of M105.cpp (a new file named G13.cpp). There is still more stuff in the file than just these few lines.

\n
    \n
  1. Upload to board

    \n
  2. \n
  3. When going to octoprint and typing in G13 I get:

    \n
  4. \n
\n
\nSend: G13\nRecv: okhere is where weights are broadcast\n
\n

A bit more work can be done to make it look nice, but this was the hard part.\n[1]: https://www.instructables.com/id/Arduino-Bathroom-Scale-With-50-Kg-Load-Cells-and-H/

\n" }, { "Id": "13712", "CreationDate": "2020-05-20T07:22:02.427", "Body": "

Most people complain about the filament not sticking on build plate but mine is vice versa. At first it used to be very good. When I removed the magnetic bed the project would come off easily but for a few days it is like I glue it to the bed with epoxy. It is impossible to remove and when I remove the black projects from the bed I see white color at the bottom of the object printed. Maybe because of too much force but I don't know why this happens.

\n", "Title": "The filament is almost impossible to remove", "Tags": "|marlin|pla|creality-ender-3|troubleshooting|heated-bed|", "Answer": "

Had this problem on my second print. The nozzle was too close to the bed. The only way I got it all off was to heat the bed. Then the PLA became a bit softer and came off pretty fast.

\n" }, { "Id": "13722", "CreationDate": "2020-05-21T13:51:39.333", "Body": "

Sometimes my Extruder will start spinning pointlessly fast and long.

\n\n

Youtube Video showing the problem

\n\n

This Issue only happens:

\n\n\n\n

My regular Setup/Stack:

\n\n\n\n

I have tried to test whether the source of the problem could be Octoprint or the slicer/gcode.

\n\n

Octoprint:
\nThe problem persists, when i print directly from SD

\n\n

slicer/gcode:
\nHere is some gcode, one of a layer change where the problem did occur and one where it was fine:\nProblem occured here:

\n\n
G1 X167.854 Y170.305 E209.95401\nG1 X167.851 Y132.289 E210.24063\n;BEFORE_LAYER_CHANGE\nG92 E0.0\n;3.95\n\nG1 Z3.950 F7800.000\n;AFTER_LAYER_CHANGE\n;3.95\nG1 X168.141 Y131.859\nG1 F3600\nG1 X129.532 Y131.858 E211.24663\nG1 X128.945 Y131.592 E211.26340\nG1 X128.419 Y131.004 E211.28396\n
\n\n

No Problem occured here:

\n\n
G1 X168.288 Y170.558 E474.06508\nG1 X168.224 Y170.259 E474.06950\nG1 X168.213 Y131.857 E474.62495\n;BEFORE_LAYER_CHANGE\nG92 E0.0\n;9.8\n\nG1 Z9.800 F7800.000\n;AFTER_LAYER_CHANGE\n;9.8\nG1 X168.559 Y131.441\nG1 F3600\nG1 X129.622 Y131.440 E475.63947\nG1 X129.198 Y131.248 E475.65161\nG1 X128.776 Y130.777 E475.66807\n
\n\n

I cannot see any problem with the gcode.

\n\n

Now I am at my wits' end - what could be the source of the problem, and how to I proceed to debug it?

\n", "Title": "Extruder spins for a few seconds after layer changes", "Tags": "|extruder|g-code|tevo-tornado|", "Answer": "

Doing some googling, it looks like the BEFORE_LAYER_CHANGE comment in the G-code and stuff that follow come from the before_layer_gcode setting in your PrusaSlicer profile. Somehow, G92 E0.0 got in there. Remove it and the problem should go away.

\n" }, { "Id": "13723", "CreationDate": "2020-05-21T18:00:03.323", "Body": "

Typically in Arduinos, most of the 5 V outputs that have a limited amount of current (40 mA).

\n\n

Are there any 3D printer boards, or is there even a more usual spot, where you can get a 5 V output that isn't capped by the microcontroller?

\n\n

I know that USB 2.0 is (typically) limited to 500 mA especally when connected to a laptop. Just wondering if there was a way to for e.g. get a 200 mA output from one of the 5 V pins, or more if there is a stronger power supply connected to the USB port.

\n", "Title": "Access uncapped 5 V output on 3D printer controller board", "Tags": "|diy-3d-printer|electronics|", "Answer": "

Any 5 V \"output\" on those main boards would be used to power a USB host. This is because the electronics actually runs on 3.3 volts. It would be better to splice the input power (the 12 to 24 V) and use a buck converter to get down to your required 5 V. Select the right one, and you can have all the current you need.

\n" }, { "Id": "13739", "CreationDate": "2020-05-24T16:17:20.553", "Body": "

About a year a go I bought a Tevo Little Monster second hand. It was a little dirty but (as far as I could see) unmodified. I put away that noisy \"MKS SBASE\"-board with smoothieware and installed a nice Duet 2 Ethernet board. I tried for weeks to get it going but without success, then decided to try it again later.\nNow I am on it again and already put a lot more effort in it than last time. Anyway, the problem persists and it is really driving me nuts!

\n\n

Out there on the internet are several people who succeeded with a duet board on the same printer, so it must be possible. For Example this one.

\n\n

My Background

\n\n

I am doing 3D printing for a while now. Besides the Little Monster I own a Makerbot Replicator 2X (running latest Sailfish), a Anycubic Delta Linear Plus (running self-compiled Marlin 1.1.9) and \u2013 for two weeks now \u2013 a Two Trees Sapphire Pro (running self-compiled Marlin 2.0.x). All these printers are running absolutely fine.

\n\n
\n\n

Problem

\n\n

As far as I can tell, my problem boils down to inaccurate z positioning of the nozzle.\nThis makes the nozzle (heavily) scratch on printed parts of the same layer. Small parts are often ripped of the bed by the nozzle before the print is finished. Practically I cannot use the printer in its current state.

\n\n

Here is a video of the Problem.

\n\n
\n\n

Test setup

\n\n

My usual test scenario is a 100 x 100 x 20 mm box with two walls, two top/bottom-layers and 50% Infill. Maximum Speed 90 mm/s. The speed should be fine, as the printer is advertised with 300 mm/s.

\n\n
\n\n

What I changed from stock:

\n\n\n\n
\n\n

What I already tried to solve the problem:

\n\n

Hypothesis: Bad config of duet board

\n\n

Actions: see separate section

\n\n

Hypothesis: Narrow nosed nozzle does not smooth the printed line enough

\n\n

Actions: replaced narrow-nosed nozzle (0.6mm) by broad-nosed nozzle (0.4mm) and adjusted cura accordingly

\n\n

Result: No difference

\n\n

Hytothesis: There is something wrong with the firmware

\n\n

Action: Upgraded duet board from RepRap 2.0 to 3.0

\n\n

Result: No difference

\n\n

Hyptothesis: Geometrical inaccurate build

\n\n

Actions: Measured arm space on effector and carriages (by caliper), \"measured\" rod lengths by eye (b/c my caliper can only measure about 150mm), checked towers by eye for bending.

\n\n

Result: Arm spacing on effector and carriage is the same down to 0.19mm. Rod lengths have no noticeable difference, towers seem to be straight.

\n\n

Hypothesis: Joints on delta Rods have play

\n\n

Actions: Tried to shake/move/wiggle the effector by hand

\n\n

Result: No noticeable play

\n\n

Hypothesis: Overextrusion

\n\n

Action: Intentionally underextruded (80% flow) for testing

\n\n

Result: Printed lines became noticable more narrow (not even touch each other), but scratching persists.

\n\n

Hypothesis: Rubber wheels of the carriages running too rough and are worn out

\n\n

Action: Replaced rubber wheels by linear Rails. (Thingiverse)

\n\n

Result: Carriages run much smoother, but this change does not help with my problem

\n\n

Hypothesis: Belt tension is bad

\n\n

Action: Tensioned the belts thoroughly

\n\n

Result: No noticable difference

\n\n

Hypothesis: Tooth belts are worn out

\n\n

Action: Replaced tooth-belt with new GT-2 Belt

\n\n

Result: Belts look more regular when moving, does not help with my problem

\n\n

Hypothesis: Pulleys and Idlers are worn out or not sufficiently round

\n\n

Action: Replaced Pulleys and Idlers

\n\n

Result: No noticable difference

\n\n

Hypothesis: One or more of the stepper motors are not working properly

\n\n

Action: Replaced all tower motors (incl. wiring) with new ones

\n\n

Result: No noticable difference

\n\n

Hypothesis: One of the stepper drivers is not working correctly

\n\n

Action1: Investigation of signals on the motor-wires by oscilloscope

\n\n

Result1: They all looked about the same (short square pulses), in principal no difference between all motor wires. (I do not know how these signals should look like. I just tried to spot differences.)

\n\n

Action2: Changed the drivers for the tower motors one by one with the unused E1-driver

\n\n

Result2: No noticable difference

\n\n

Hypothesis: Leveling is bad due bad probing

\n\n

Action1: Checked leveling manually (with feeler gauge) and checked BL-Touch repeatability.

\n\n

Result1: Found deviation of 0.2mm on some points between automated and manual leveling. Repeatability is good (accurate to 0.02mm)

\n\n

Action2: Leveling is now done by a DF9-40 weight-resistor (and a voltage divider). This sensor is mounted (manually every time) directly under the nozzle.

\n\n

Result2: The first layer is next to perfect. To my understanding that means a correct calibration.

\n\n

Hypothesis: Flying extruder is too heavy and impacts the accuracy of the carriage movement

\n\n

Action: changed extruder from flying to remote, mounted on one of the towers

\n\n

Result: Problem seems to hit maybe a little less. I am not entirely sure about this.

\n\n

Hypothesis: My problem is no problem, a little scratching is nothing unusual

\n\n

Action: Tried the same print on my smaller Delta printer

\n\n

Result: No scratching on the other printer. Hence it seems not to be usual.

\n\n

Hypothesis: Scratching is related partial overextrusion due to \"extrusion delay\" caused by bowden system

\n\n

Action: Tested linear advance between 0 to 1.2

\n\n

Result: linear advance seems to work as intended (watching the extruder), but does not change anything on my problem

\n\n
\n\n

Configuration

\n\n

I tried to transfer the settings from the original smoothieware config-files as good as possible. Original smoothieware files can be found here on github

\n\n

To have a consistent config, I used the RRF Browser Config Tool.

\n\n

Motor Current:\nThere is no advice printed on the original motors concerning the current. Smoothieware config file says 1.5 Amp. Thats what I configured. (The Replacement-Motors came with a data sheet, they can take 2 Amp. So they got 2 Amp)

\n\n

Steps per mm:\nOn all tower motors there are 20-tooth-GT2-pulleys. RRF Config tool suggests 80 steps/mm (with 16x microstepping). I tried it with and without interpolation from 16x to 256x and with \"native\" 256x microstepping (1280 steps/mm). Also tried native 32x microstepping with 159.53 steps/mm (like in the smoothieware config) and 160 steps/mm. Does not change anything on the problem.

\n\n

Delta Parameters:\nOriginal Smoothieware config suggests:

\n\n
arm_length      397.1900            # This is the length of an arm from hinge to hinge\narm_radius      157.0741            # This is the horizontal distance from hinge to hinge when the effector is centered\n
\n\n

That seems to be (roughly) consistent with what I measure by folding ruler. So that is what I put in my config right away.

\n\n

Auto Calibration:\nTried 4,6,7,8 and 9 factors (using the force-sensor, not BL Touch) and put the results into splunk. Each data point is averaged over 3 calibration runs with exactly the same settings, every time starting from the smoothieware stock-values. (First run was with 159.53 steps/mm, second with 160 steps/mm)

\n\n

\"Splunk

\n\n

Surprisingly the auto calibration finds reliably the same parameters, which differ from the stock-config ones. I have no Idea why, but similar things happen on my smaller delta printer. Hence I do not consider this a problem.

\n\n

Speed and acceleration:\nLowering acceleration (down to 300 mm/s) does not change anything. Lowering speed makes scratching less loud, but still hearable/feelable rumble from the nozzle on the print. Prints are still ripped of the print bed.

\n\n
; Configuration file for Duet WiFi (firmware version 3)\n; executed by the firmware on start-up\n;\n; generated by RepRapFirmware Configuration Tool v2.1.8 on Fri Apr 24 2020 17:00:42 GMT+0200 (CEST)\n\n; General preferences\nG90                                            ; send absolute coordinates...\nM83                                            ; ...but relative extruder moves\nM550 P\"jKossel19\"                              ; set printer name\n;M665 R157.0741 L397.19 B155 H522               ; \nM665 R170 L397.19 B155 H525            ; from forum.duet3d.com with modified hight 520 ==> 525\nM666 X0 Y0 Z0                                  ; put your endstop adjustments here, or let auto calibration find them\n\n; Network\nM552 S1                                        ; enable network\nM586 P0 S1                                     ; enable HTTP\nM586 P1 S0                                     ; disable FTP\nM586 P2 S0                                     ; disable Telnet\n\n; Drives\nM569 P0 S0                                     ; physical drive 0 goes backwards\nM569 P1 S0                                     ; physical drive 1 goes backwards\nM569 P2 S0                                     ; physical drive 2 goes backwards\nM569 P3 S0                                     ; physical drive 3 goes backwards\nM584 X0 Y1 Z2 E3                               ; set drive mapping\nM350 X16 Y16 Z16 E16 I1                        ; configure microstepping with interpolation\nM92 X80 Y80 Z80 E418.00                        ; set steps per mm\nM566 X480.00 Y480.00 Z480.00 E1200.00          ; set maximum instantaneous speed changes (mm/min)\nM203 X240000.00 Y240000.00 Z240000.00 E1800.00 ; set maximum speeds (mm/min)\nM201 X1000.00 Y1000.00 Z1000.00 E1000.00       ; set accelerations (mm/s^2)\nM906 X2000 Y2000 Z2000 E800 I30                ; set motor currents (mA) and motor idle factor in per cent\nM84 S30                                        ; Set idle timeout\n\n; Axis Limits\nM208 Z0 S1                                     ; set minimum Z\n\n; Endstops\nM574 X2 S1 P\"xstop\"                            ; configure active-high endstop for high end on X via pin xstop\nM574 Y2 S1 P\"ystop\"                            ; configure active-high endstop for high end on Y via pin ystop\nM574 Z2 S1 P\"zstop\"                            ; configure active-high endstop for high end on Z via pin zstop\n\n; Z-Probe BLTouch\n;M950 S0 C\"exp.heater3\"                         ; create servo pin 0 for BLTouch\n;M558 P9 C\"zprobe.in+zprobe.mod\" H5 F120 T6000  ; set Z probe type to bltouch and the dive height + speeds\n;G31 P500 X0 Y20 Z0.45                          ; set Z probe trigger value, offset and trigger height\n;M557 R140 S45                                  ; define mesh grid\n\n\n; Z-Probe MANUAL\n;M558 P0 H5 F120 T6000                          ; disable Z probe but set dive height, probe speed and travel speed\n\n; Z-Probe weight resistor\nM558 P1 C\"zprobe.in\" H5 F60 T6000              ; set Z probe type to unmodulated and the dive height + speeds\nG31 P500 X0 Y0 Z0.05                           ; set Z probe trigger value, offset and trigger height\n\n; Heaters\nM308 S0 P\"bedtemp\" Y\"thermistor\" T100000 B4092 ; configure sensor 0 as thermistor on pin bedtemp\nM950 H0 C\"bedheat\" T0 Q10                      ; create bed heater output on bedheat and map it to sensor 0\nM143 H0 S120                                   ; set temperature limit for heater 0 to 120C\nM307 H0 B0 S1.00                               ; disable bang-bang mode for the bed heater and set PWM limit\nM140 H0                                        ; map heated bed to heater 0\nM308 S1 P\"e0temp\" Y\"thermistor\" T100000 B4092  ; configure sensor 1 as thermistor on pin e0temp\nM950 H1 C\"e0heat\" T1                           ; create nozzle heater output on e0heat and map it to sensor 1\nM143 H1 S280                                   ; set temperature limit for heater 1 to 280C\nM307 H1 B0 S1.00                               ; disable bang-bang mode for heater  and set PWM limit\n\n; Fans\nM950 F0 C\"fan0\" Q500                           ; create fan 0 on pin fan0 and set its frequency\nM106 P0 S1 H1 T50                              ; set fan 0 value. Thermostatic control is turned on\nM950 F1 C\"fan1\" Q500                           ; create fan 1 on pin fan1 and set its frequency\nM106 P1 S0 H-1                                 ; set fan 1 value. Thermostatic control is turned off\n\n; Tools\nM563 P0 S\"Vulcan\" D0 H1 F0:2:1                 ; define tool 0\nG10 P0 X0 Y0 Z0                                ; set tool 0 axis offsets\nG10 P0 R0 S0                                   ; set initial tool 0 active and standby temperatures to 0C\n\n; Custom settings\n; set heater parameters\nM307 H0 A213.8 C593.5 D3.3 S1.00 V24.6 B0\nM307 H1 A513.6 C184.3 D8.4 S1.00 V24.4 B0\nM572 D0 S0.5 ; pressure advance\n\n; Miscellaneous\nM501                                           ; load saved parameters from non-volatile memory\n\n\n
\n\n
\n\n

Workarounds:

\n\n\n\n
\n\n

What I still do not understand:

\n\n\n\n
\n\n

EDIT

\n\n

Recently tried:

\n\n\n\n

Result:\nproblem persists and has not changed noticeably.

\n", "Title": "Deltaprinter scratches print", "Tags": "|delta|", "Answer": "

Finally I found an solution. But I still don't know what exactly the problem was.

\n

In my case the problem vanished after the following steps:

\n\n" }, { "Id": "13748", "CreationDate": "2020-05-25T19:17:55.043", "Body": "

I'm using an Ender 3 with a new upgraded board SKR 1.3 Marlin bugfix 2.0 auto bed leveling I use EZABL.

\n\n

for the EZABL I followed their tutorial for Marlin 2.0 since I don't use the unified firmware.

\n\n

Offset info :

\n\n

#define NOZZLE_TO_PROBE_OFFSET { 50, -2, 0 }

\n\n

#define MIN_PROBE_EDGE 20

\n\n

I did try to follow this helpful guide

\n\n

I don't know much about programming so when he said:

\n\n
\n

\"As this is the part that defines the array values, you first need to make an array (note that this is a simple solution that many people should be able to understand with limited programming skills, more elegant solutions use the XYZ struct to access the X, Y or Z properties):\"

\n
\n\n

constexpr int nozzle_to_probe_offset[] = NOZZLE_TO_PROBE_OFFSET;

\n\n

#define PROBE_X_OFFSET_FROM_EXTRUDER nozzle_to_probe_offset[0]

\n\n

#define PROBE_Y_OFFSET_FROM_EXTRUDER nozzle_to_probe_offset[1]

\n\n

I don't understand where to put the lines above and what value to put.

\n\n

update:\nerror message on compiling\n\"enter\nConfig.h : https://pastebin.com/w7CC5eaC\nConfig_adv.h : https://pastebin.com/qep34T1h

\n", "Title": "Auto bed leveling offset issue", "Tags": "|marlin|skr-v1.3|ezabl|", "Answer": "

You do not need to adjust the area yourself for Marlin 2.0, please look into probe.h and into this answer.

\n\n

You only need to enable the offsets:

\n\n
#if PROBE_SELECTED && !IS_KINEMATIC\n  #define MIN_PROBE_EDGE_LEFT MIN_PROBE_EDGE\n  #define MIN_PROBE_EDGE_RIGHT MIN_PROBE_EDGE\n  #define MIN_PROBE_EDGE_FRONT MIN_PROBE_EDGE\n  #define MIN_PROBE_EDGE_BACK MIN_PROBE_EDGE\n#endif\n
\n" }, { "Id": "13750", "CreationDate": "2020-05-26T01:12:58.340", "Body": "

The thermistor of the hotend of my printer broke, at the moment I have no way to replace it, due to the global situation with the virus.\nIn my hands I have one of these industrial PIDs which is Temperature controller+K-type couple+SSR.\nIs there a way to disable the PID in Marlin and start printing manually when the PID reaches the desired temperature? I just need it for PLA right now.\nThis way I can use the printer at least until I can replace the thermistor

\n", "Title": "Disable Marlin PID, and start print manual", "Tags": "|marlin|thermistor|pid|", "Answer": "

You can edit the temperature control commands out of your gcode, or just tell your slicer to omit them, typically by setting temperature to 0. Then, as you suggested, start the print manually once your industrial PID tells you it's at-temperature.

\n" }, { "Id": "13752", "CreationDate": "2020-05-26T04:01:31.273", "Body": "

I want to be able to control my FLSUN QQ over Wi-Fi and don't have OctoPrint or a Raspberry Pi to run it on.

\n", "Title": "How can I print to my FLSUN QQ over Wi-Fi?", "Tags": "|wi-fi|flsun-qq|mks|", "Answer": "

As the auto-install from the Marketplace described in this answer wasn't working , I did have to manually install the MKS WiFi plugin on my Mac (M1 MBPro). To do this I just:

\n
    \n
  1. Downloaded the GitHub files (from the plugin webpage, linked in the Marketplace)
  2. \n
  3. Right-clicked the Cura icon in the Applications folder, to show\npackage contents
  4. \n
  5. I then went into the macOS folder and followed the plugin link
  6. \n
  7. Copy the downloaded GitHub files
  8. \n
  9. Restart Cura
  10. \n
\n

I then just followed the printer setup advice in Cura from this answer.

\n

No more messing around with SD cards :)

\n" }, { "Id": "13767", "CreationDate": "2020-05-28T10:58:44.477", "Body": "

Is there any DRM or license management solution for 3D printing? I'm looking for something, that would help me limit the number of prints someone can make from my projects. Basically, I would like to sell the \"right to make no more than X copies\" of my design. I don't expect it to be bullet-proof (like Widevine L1 for video), but it should at least help me with license management.

\n", "Title": "DRM or license management solution for 3D printing", "Tags": "|file-formats|legal|", "Answer": "

Generally: No

\n

Let's face the obvious problems of the files exchanged and the files used for printing, and then look into why it is a bad idea in the first place.

\n

G-code

\n

G-code is in its design a .txt file that contains specific orders for a machine. There is a g-code command that forces the printer to delete the file (M30) but that needs the exact file path - and just does nothing if the path is incorrect. One could make a slicer profile, that after printing, deletes the file from the SD-card, but doing so as a user is only useful for one-off jobs and it can be prevented by the user by simply shutting down the printer before the line is triggered.

\n

Because .gcode is essentially a .txt without any special features but a custom ending, it can't provide DRM beyond containing a self-delete-after-completion.

\n

STL & OBJ

\n

STL and OBJ are the most relevant exchange files. They are open source and not intended to contain any DRM.

\n

Other formats?

\n

Even with a different format, you would need both a proprietary 3D format that can only be read by your DRM-enabled slicer as well as a proprietary printer file type that both contain DRM. However, using a proprietary printer command type means you lock down your file to be only available to your printer family, which in turn makes this printer family less desirable, as it can't work with the standard g-code format.

\n" }, { "Id": "13774", "CreationDate": "2020-05-29T04:20:19.543", "Body": "

My print area is ~ 17x16 inches (431.8 mm x 406.4 mm = 175,158.4 mm/2). To that end I need four (4) 200mm square heaters, and associated mosfets to run it. Each heater has it's own thermistor and can potentially reach the target temp independently (theoretically they could, since there will be deviations in manufacturing).

\n\n

My mainboard (the SKR Pro) has support for 4 thermistors; however my solution in this configuration has 5.

\n\n

When I double the size of the printer later this year, I expect that I will need 8 heaters.

\n\n

Is there a solution to control that many heaters such that non of them over shoot their temps, or am I going to have to design and build a dedicated thermal management board to control beds of this nature?

\n", "Title": "How to control 4 thermistors in a heated bed?", "Tags": "|thermistor|mosfet|", "Answer": "

You can use the Klipper firmware which accepts an arbitrary number of heaters.

\n\n

Then you can write a Klipper macro which takes one input and applies it to each heated bed (or not, as you prefer).

\n\n

With Klipper you can also add another Arduino and connect it to klipper as well, in case you don't have enough pins on your board: klipper is not limited to controlling one single board per printer.

\n\n

Check the docs.

\n" }, { "Id": "13776", "CreationDate": "2020-05-29T09:20:40.480", "Body": "

Recently I have started using a fairly large LCD resin printer. (Yidimu Falcon Pro)\nIt has a fairly large 260x160mm perforated and coated steel build plate and a decent dual rail Z axis with a ballscrew. It has a 10\" LCD. It prints ChiTuBox files from a USB stick.

\n\n

I have been having a mostly successful print, at good quality, but the bed adhesion keeps haunting me. The best print on this machine had parts curling up from the build plate, however most of the print succeeded.

\n\n\n\n

I have tried:

\n\n

Changing resin (tried Druckwerk Pro D Black and Anycubic Black)

\n\n\n\n

What would be a good next troubleshooting step?

\n\n

\"first

\n\n

\"first\n\"model

\n", "Title": "How to prevent model from peeling off of resin printer buildplate during print?", "Tags": "|sla|resin|", "Answer": "

I had an issue with my Vat Bolts, one being cracked and leveling at a greater tension than the other. With the compressible feet of my Vat this caused a problem in the FEP. This was enough to cause failure on one side more than the other. Cause was verified by switching the left and right knobs, thus shifting the failures to the opposite side of the print.

\n

Just a thought.

\n" }, { "Id": "13788", "CreationDate": "2020-05-31T19:55:13.720", "Body": "

My calibration at my printer is okay. When I try to move 10 mm, it's exactly 10 mm.

\n\n

But I want to know what is my precision for Z-axis.

\n\n

(not posting the link of the product maybe it's not okay for the platform)

\n\n

Spec for leadscrew:

\n\n\n\n

\"calculator

\n\n

My question; is my gear ratio 4:1 or 1:4? It's giving different results.

\n", "Title": "Calculating gear ratio for leadscrew", "Tags": "|calibration|lead-screw|", "Answer": "

Your lead screw nut advances 8 mm every complete rotation of 200 full steps, so a single full step would change the height by 8/200 = 0.04 mm. This implies you need to set the layer height a multiple of 0.04 mm.

\n\n

The gear ratio of 4:1 (a reduction) is required to get to your desired 0.01 mm layer height, but that is a very uncommon and a too small layer height to use.

\n" }, { "Id": "13793", "CreationDate": "2020-06-01T16:21:41.207", "Body": "

After trying to solve this for the whole weekend I've run out of ideas.

\n\n

I've fitted an BLTouch to my Ender-3 with an SKR mini e3 V2.0 board, firmware is Marlin 2.0.

\n\n

My issue is the following: \nwhen I do autohome (G28) it uses the Z-Probe (BLTouch) to home (this is expected). \nI then set Z Probe offset to 0 (using M851 Z0) and move Z down until it the nozzle touches a piece of paper.
\nI then set the Probe Z offset to whatever value Z is at (M851 Z-4.10), save the settings to EEPROM and restart the machine. When doing another G28 and moving Z to 0 afterwards the distance is perfectly the same. This is repeatable.
\nHowever when I do G29 it measures the bed to be sitting around -1.9 mm, therefore moving Z to 0 will crash the nozzle to the bed. I can change my Probe Z offset have this fitting (M851 Z-2.20), however then the nozzle sits ~2 mm above the bed when doing another G28 with \"Bed Leveling\" disabled.

\n\n

All of this is perfectly repeatable, so I think my I've got something off in my Marlin configuration, but can't find anything wrong.

\n\n

What I've tried so far:

\n\n\n\n

Did anyone encounter this issue before or can give me any idea how to fix this? Any help would be appreciated.

\n\n

EDIT: I've just flashed the precompiled stock firmware from the BTT Github and it works. So im quite sure i've got something wrong in my config. But since I can't find the configuration.h for that .bin file, I can't realy compare both

\n", "Title": "Different Probe Z offset for G28 and G29", "Tags": "|marlin|bed-leveling|bltouch|skr-mini-e3|", "Answer": "

I'm answering my own question here.

\n\n

The Issue was that I set Z_MIN_POS to -2.0 I guess these 2 mm were subtracted from G28 but not from G29.

\n\n

I don't know wether this is expected behaviour but changing Z_MIN_POS back to 0 fixed the issue for me.

\n" }, { "Id": "13799", "CreationDate": "2020-06-02T16:53:35.440", "Body": "

I recently started swapping out my nozzle for different sizes. After a few times using a 0.4 mm, then 0.8 mm, then back to 0.4 mm, I now have this \"clumping\" going on when printing. The photos are of PLA but this happens with PETG also. Before I started changing nozzles, my prints were perfect.

\n\n

The blue filament is Overture PLA, printing at 45 mm/s 210 °C (I lowered the temp to 200 °C and it's still happening).

\n\n

\"enter

\n\n

\"enter

\n\n
\n\n

Some things I've noticed (which did not exist when my prints were good):

\n\n\n\n
\n\n

The questions:

\n\n
    \n
  1. Does this \"clumping' have an actual name?
  2. \n
  3. What did I potentially do when swapping nozzles that could have caused this (if that's in fact how it started)?
  4. \n
  5. How can I fix it?
  6. \n
\n\n
\n\n
\n\n

Update:

\n\n

The print stopped and I removed the shroud. My train of thought was spot on as I saw a pool of blue PLA inside the housing and on the nozzle. But I still don't know how this happened.

\n\n

\"enter\n\"enter

\n", "Title": "Material \"clumping\" when printing", "Tags": "|filament|pla|creality-ender-3|nozzle|", "Answer": "

From your initial photos, it looks like the bed level was too high, and there was thereby more material extruded than fit in the space. Even with an \"identical\" nozzle, the distances involved here are on an order of magnitude (less than 0.1 mm) that you're not going to get identical distance to the bed from one nozzle to another.

\n\n

Further, as you found out following up, not having the nozzle tightened all the way will affect its height, and allow molten material to exit around the gap. It's always a problem not having your nozzle tightened all the way to make a good seal (I actually had nearly the same problem the first time I ever changed a nozzle), but having the bed too close probably made even more material ooze out at the gap due to backpressure.

\n\n

Hopefully fixing these two things gets you back to printing right. It might be some work to clean out any material left in the hotend, especially if it worked its way around into the threads where the nozzle screws in. Be gentle if you have to clean that out further, since it's easy to strip the threads if you use abrasive tools, and then you'll be looking at replacing the heater block.

\n" }, { "Id": "13801", "CreationDate": "2020-06-03T10:25:43.747", "Body": "

opposed to what the title of this question might impose, I'm not new to 3d printing itself and have operated more than one printer for years, however, I feel dumb with installing the Thermistor on this new Heated Bed I got from RepRap.me: Aluminum Heatbed MK3 400x300

\n\n

The old Mk2b 214x214 Mendel/Prusa heated bed that I have been using for quite a while now has a through-hole in the center, where the thermistor-leads could be fed to from below.

\n\n

On this new heated bed however, there is a lowering in the very center of the Heated Bed to place the Thermistor in, yet the pads to connect the thermistor leads to right beside it are level with the rest of the Heated bed.

\n\n

I tried using just a tiny amount of SMD solderpaste and just soldering the thermistor leads as flat as possible to the pads, nevertheless, the glass plate on top is not laying flat and stable on the heated bed with the thermistor installed.

\n\n

Any suggestions are greatly appreciated on how improve the thermistor-installation onto this heated bed!

\n", "Title": "How is a Thermistor supposed to be installed between Heated Bed and the Glass?", "Tags": "|heated-bed|thermistor|", "Answer": "

Try to use a silicone heat conductive pad between heater and glass, where you can make a hole for the sensor. It also helps to improve the contact between aluminium heater and glass, which otherwise may be inhomogeneous (as expected between two quite rigid materials).

\n\n

They sell them in rolls and you can choose a 1 mm thick one or even 0.5 mm. They are \"sticky\" but they don't have glue, so you will be able to separate the glass from the pad, even if not so easily as it would be with a clip.

\n" }, { "Id": "13807", "CreationDate": "2020-06-04T05:55:56.823", "Body": "

I have an Ender 3 3D printer.

\n\n

I am having an issue with leveling it, everything I do does not make any change.

\n\n

The right side of the bed is higher than the left side, that will make me loosening the right side springs completely.\nI\u2019ve worked with this printer before and I know it\u2019s should not be like this.

\n\n
    \n
  1. The printer is sitting on a leveled surface.
  2. \n
  3. I have 90° in each of every connection of the stands.
  4. \n
  5. X-Axis is corresponding to the upper stand.
  6. \n
  7. I\u2019ve replaced the regular bed cover for 4 mm \u05f4milky\u201d glass.
  8. \n
  9. I\u2019ve replaced the springs to the stronger ones.
  10. \n
  11. I\u2019ve modify the location of the Z-switch to the lowest corner of the bed.
  12. \n
\n", "Title": "Ender 3, unable to level the heating bed", "Tags": "|creality-ender-3|bed-leveling|", "Answer": "

When you do the homing and then do the leveling how much thread you have left that passes the tuning wheel? Verify this on all the four wheels. If you have no thread left then you may consider to lower the Z end-stop.\nCheck also the Y gantry to see if it is parallel to the base, and make adjustments if needed.

\n" }, { "Id": "13809", "CreationDate": "2020-06-04T20:41:47.253", "Body": "

Could concentrated sunlight be used to build a metal 3D printer, sintering or melting metal powder?

\n\n

Related question on physics stack exchange https://physics.stackexchange.com/questions/143049/what-temperature-is-achieved-in-focus-point-by-5000-flat-1x1cm-mirrors-onto-a-sa

\n", "Title": "Sunlight powered metal 3D printer", "Tags": "|metal-printing|", "Answer": "

Let's start with the obvious: this printer would need to be really big. Not because of a large print volume, but because it needs to collect a lot of sunlight or needs a really big focussing array. The linked question states that the array there, about 0.6\u00a0m\u00b2 large, has roundabout 600\u00a0W of power to focus on that one point.

\n

Power draw needs

\n

What powers are we dealing with?

\n

A typical laser cutter uses a laser tube that at least 20\u00a0W for thin material and up to 300\u00a0W for thicker material. But we need to weld steel, so we need to be roughly equivalent to a cutting laser for the same material. We are not talking mere hundreds of watts, we are talking an industrial 2000 to 20000\u00a0W in a laser of less than a millimeter in diameter. Why do I say the later? Well, power need scales with the area, which scales with the square of the radius.

\n

Let's use a ballpark, a nice round 10000\u00a0W Laser with a somewhat large 1\u00a0mm\u00b2 crossection. We're talking about the ballpark of 100 Gigawatt per square meter.

\n

$L=P/A=\\frac{10000\\ \\text W}{0,001\\times 0.001\\ \\text m^2}=10^{10}\\frac{\\text W}{\\text m^2}$

\n

Solar power harvester size

\n

Luckily, we don't need to illuminate a whole square meter, so we only need some 10 Kilowatts of sunlight for our application. But we need this number to calculate how much sunlight we need to harvest in our smelting machine.

\n

At the stratosphere, Earth gets about 1400\u00a0W/m\u00b2, and on a sunny day, about 1\u00a0kW/m\u00b2 makes it to the surface, we call this the solar constant $S$. Now, if we compare the ballparks, we get to quickly see the ballpark size of our machine:

\n

$P/S=A=\\frac{10 \\text{ kW}}{1 \\frac {\\text{kW}} {\\text m^2}} = 10\\ \\text m^2$

\n

10 square meters of harvesting area gets us the same power. Incidentally, this scales linearly with the item discussed in the linked question, as that one already uses square meters.

\n

Sizing down

\n

But 10 square meters of mirrors into a focal point and then lenses to a focal point is huge, can we make it smaller? And to that I must say somewhat. First of all, we could get our focal point smaller: We need to get 10 Gigawatt per square meter for our $L$, but we can turn two screws here: what if we get from 1 mm\u00b2 to only 0.1 mm\u00b2?

\n

$P=L*A=10^{10}*(0.0001\\times 0.0001)\\ \\text W=100 W$

\n

100 Watts focussed on 0.1 mm\u00b2 is a sixth of the power that array can deliver, so totally feasible in terms of power, as long as you can achieve such a small focal point.

\n

Conclusion

\n

Yes, with a focussing array good enough or a mirror array large enough you could achieve the powers needed to melt metals on a spot focus. Note though, that you need to have a really good focus setup that creates pretty much a solar-powered laser, which means that such a machine will be extremely expensive due to the high precision machinery needed for that - and that this focusing aperture will be most likely the largest part of your machine. We're talking building-size scale. I'd be cheaper and easier to just put lots of solar panels onto the roof of the building you run a conventional metal 3D printer in.

\n" }, { "Id": "13821", "CreationDate": "2020-06-06T13:17:22.703", "Body": "

I have had issues with printing on my i3 MK3, especially when it came to the first layer. In March 2020, I have installed the Prusa Mesh Leveling plugin for Octoprint. With that plugin and a Nylock nut modification, I was able to reduce the bed variance from 0.6\u00a0mm down to 0.014\u00a0mm and prints were great. That took about 25 rounds of calibration and I didn't have any issues with the graphics not updating.

\n

\"Mesh

\n

Shortly after that, my filament sensor stopped working and I ordered a replacement, which I installed yesterday. But today, my prints are bad again, especially the first layer. So I thought I would simply run through the calibration and mesh bed leveling again.

\n

First, I turned the screws in the wrong direction, so the result became worse and I needed several attempts until the results went into the correct direction. I'm currently at ~1.0\u00a0mm bed variance, which is very bad and I need to continue leveling the bed.

\n

\"Bed

\n

Now, the graphics does not update any longer. It always shows the same picture.

\n

I also noticed some different behavior of the mesh bed levelling procedure. Usually, after running the mesh bed levelling, the printer needs some time (~5 secs) before it will react to other commands like a move on the Z axis. I used the following technique to find out when to reload the graphics:

\n\n

Now, the print head moves up immediately after the mesh bed levelling, without the ~5 secs delay and the graphics does not update.

\n

I have already tried:

\n\n

.

\n
pi@octopi:~ $ df -h\nFilesystem      Size  Used Avail Use% Mounted on\n/dev/root       7.3G  1.9G  5.1G  27% /\ndevtmpfs        182M     0  182M   0% /dev\ntmpfs           186M     0  186M   0% /dev/shm\ntmpfs           186M  2.7M  183M   2% /run\ntmpfs           5.0M  4.0K  5.0M   1% /run/lock\ntmpfs           186M     0  186M   0% /sys/fs/cgroup\n/dev/mmcblk0p1   42M   21M   21M  51% /boot\ntmpfs            38M     0   38M   0% /run/user/1000\n
\n

In the log file (octoprint.log) with output level set to DEBUG, I could see an entry:

\n
2020-06-06 12:19:52,261 - octoprint.plugins.PrusaMeshMap - INFO - Generating heatmap\n2020-06-06 12:19:52,288 - py.warnings - WARNING - \n    /home/pi/oprint/local/lib/python2.7/site-packages/matplotlib/pyplot.py:522:\n    RuntimeWarning: More than 20 figures have been opened. \n    Figures created through the pyplot interface (`matplotlib.pyplot.figure`) are\n    retained until explicitly closed and may consume too much memory. \n    (To control this warning, see the rcParam `figure.max_open_warning`).\n    max_open_warning, RuntimeWarning)\n
\n

As you can see, this was at 12:19. The last graphics I saw is from 12:37.

\n

The logs also contain a message on 12:40:

\n
2020-06-06 12:40:39,262 - octoprint.util.comm - ERROR - Error while processing hook PrusaMeshMap:\nTraceback (most recent call last):\n\n  File "/home/pi/oprint/local/lib/python2.7/site-packages/octoprint/util/comm.py", line 2849, in _readline\n    ret = hook(self, ret)\n  File "/home/pi/oprint/local/lib/python2.7/site-packages/octoprint_PrusaMeshMap/__init__.py", line 90, in mesh_level_check\n    self.mesh_level_generate()\n  File "/home/pi/oprint/local/lib/python2.7/site-packages/octoprint_PrusaMeshMap/__init__.py", line 236, in mesh_level_generate\n    fig.savefig(self.get_asset_folder() + '/img/heatmap.png', bbox_inches="tight")\n[...]\n  File "/home/pi/oprint/local/lib/python2.7/site-packages/matplotlib/image.py", line 512, in _make_image\n    output = self.to_rgba(output, bytes=True, norm=False)\n  File "/home/pi/oprint/local/lib/python2.7/site-packages/matplotlib/cm.py", line 259, in to_rgba\n    xx = (xx * 255).astype(np.uint8)\nMemoryError\n
\n

On 12:46 I rebooted the system

\n
2020-06-06 12:46:08,761 - octoprint.server.api.system - INFO - Performing command for core:reboot: sudo shutdown -r now\n
\n

but of course that graphics is still missing and the last available graphics is the one from 12:37. So, after the reboot one needs to run the mesh bed leveling again.

\n

Still, no luck...

\n

OctoPrint version is 1.4.0, OctoPi version 0.15.0PE, Prusameshmap Plugin: 0.3.0. As far as I can tell, that's the latest version available.

\n

What can I do to make mesh bed leveling work again?

\n", "Title": "Mesh bed levelling does not update any more after MemoryError", "Tags": "|prusa-i3|bed-leveling|octoprint|", "Answer": "

I have formatted the SD card and installed Octoprint from scratch. That's nasty, because I lost all the models I uploaded.

\n

It seems to be a bug in __init__.py of OctoPrint-PrusaMeshMap (archived Github repository).

\n

That code saves the heatmap in this line:

\n
fig.savefig(self.get_asset_folder() + '/img/heatmap.png', bbox_inches="tight")\n
\n

It uses Pyplot

\n
import matplotlib.pyplot as plt\n
\n

and thus the code should probably use (untested!)

\n
plt.close(fig) \ndel fig\n
\n

to free the resources.\nUnfortunately it's not possible to file this as an issue because the Github repository is in archived mode and thus readonly.

\n" }, { "Id": "13825", "CreationDate": "2020-06-07T21:32:37.180", "Body": "

I recently heard that the 3D printing lab at my college can do fused-deposition with ABS and PLA, but I would like to use TPU, for greater flexibility.

\n\n

Is it possible to feed a TPU filament into the same machine built for ABS/PLA? Or is there no difference? Assume the diameters of the filaments are the same.

\n", "Title": "Can any filament be used in with any 3D printer?", "Tags": "|print-material|tpu|", "Answer": "
\n

Is it possible to feed a TPU filament into the same machine built for ABS/PLA? Or is there no difference? Assume the diameters of the filaments are the same.

\n
\n

The question is not what the machine is built for but how it is built. Let's break stuff down some into why some filaments work better than others and the challenges with them.

\n

Temperature range

\n

The first obstacle is the temperature range of the printer. For example, if your printer's Extruder can't get over 220\u00a0\u00b0C, you can't print ABS. You need to make sure your printer can match your intended filament's temperature range.

\n

TPU usually prints around between the same temperatures for PLA and ABS, so it should work from that range.

\n

Heated Bed

\n

Most Filaments work better with a heated bed, but some are almost impossible to be printed without. For TPU, a heated bed should be used but is not absolutely mandatory.

\n

Heated Chamber

\n

Some filaments can't be printed without a heated chamber, others like ABS highly benefit from it. TPU is ambivalent on this as far as I know.

\n

Extruder Setup

\n

There's basically 3 extruder setups. Pellet extruders are super rare, so we don't concern about them. The other two are Direct Drive and Bowden. In a Direct Drive, the extruder motor is right over the hotend, and pushes the filament directly into it. In a Bowden setup, the hotend and the extruder motor are connected via a Bowden tube. Both have benefits and drawbacks:

\n

A Bowden makes for a lighter carriage, leading to faster printing speeds. A Direct Drive has much less trouble with elastic filaments and can do much better with retraction.

\n

TPU is one of the filaments that works much better with Direct Drive.

\n

Other considerations

\n

Some Print services do print in machines set up for one filament type and that only, because it prevents cross-contamination of the nozzles, especially when a high-temperature print material remains in it when a low-temperature print comes next. Having for example a little bit of ABS left in a printer that runs PLA next can lead to very very extreme clogging.

\n" }, { "Id": "13829", "CreationDate": "2020-06-08T08:32:49.793", "Body": "

I am building an HEVO printer with a 300x300 build plate and the project includes two T8 lead screws, lead 8 mm (\"Tr8x8(p2)\").

\n\n

I read that such long lead may worsen backlash issues, so anti-backlash bolts are often used. They however increase the torque required to drive the build platform (quite important, since 8 mm lead involves using microstepping for fine adjustments) and they also reduce the usable vertical travel.

\n\n

Since the build plate has already a significant weight (2020 frame, 2x 2 mm aluminium plates, 4 mm borosilicate bed, 8 linear bearings, heater), I wonder what is an approximate weight threshold to decide for using the anti-backlash or to skip it.

\n", "Title": "When to use an anti-backlash bolt for a Z drive gear?", "Tags": "|diy-3d-printer|build-plate|lead-screw|drive-gear|", "Answer": "

First of all, let's look at the geometry: We have a static XY and changing Z on our build plate. This mitigates several problems we might have faced in a different setup, mainly that our lead screw has loads changing from one face to the other: all loads always go into $-Z$, so the lower surface of the Trapezoidal threads in the brass nut of the bed carrier and the upper surface of the screw will always rub against each other.

\n\n

However, let's talk about your lead. Tr8x8(p2) is not a Metric ISO-Trapezoidal pattern that is commonly recognized - the only 8 mm diameter one in that standard is Tr8x1.5, very flat and very well suited for putting high loads onto it because there are several engagement surfaces in the nut and a low pitch angle - the thread only has about a 5\u00b0 (+-1\u00b0)to the horizontal. Assuming a 15 mm nut, that's 10 times the projected engagement surface for a total area of about 214\u00a0mm\u00b2.

\n\n

A Tr8x8 is a very common oddball as in, it's not in the metric tables but available everywhere. It is considerably steeper, and with a 16 mm nut (for easier calculation) would result in 2 times the projected surfaceof a single engagement per thread. But that's only a projected area of 42.8\u00a0mm\u00b2 per thread - about 1/5th of the Tr8x1.5, which correlates directly into that much less friction, because the friction is linear with the surface, which is linear to the length of the nut.

\n\n

Taking an \"intermediary\" TR8x4 gets us about 1/2.5th of the Tr8.1.5's holding power per thread at the cost of half of the speed. From a physical standpoint, I'd take this one, also increasing the precision of the Z-movement by a factor of 2 in comparison to the Tr8x8.

\n\n

Tradeoffs

\n\n

Generally speaking, we have a tradeoff here:

\n\n\n\n

Due to the geometry, the slop in the nut should not be an issue, as the relevant engagement surfaces never change. Further, the heavier the bed, the fewer problems you'll have with fast movements, as gravity will prevent the bed from going \"airborne\" at the end of travel towards the printhead. Travel away from the printhead, as it will do during printing, should never experience such.

\n" }, { "Id": "13836", "CreationDate": "2020-06-09T04:49:32.070", "Body": "

I've been having a problem lately with some minor layer shifting with my Ender 3. It just started recently and I've made no changes to the printer since it was working fine. The layer shifting is always on the X-axis and it's not a large amount. Just enough to render my prints useless. The shift can happen in either direction on the X-axis. See attached photo.

\n

Things I've tried:

\n
    \n
  1. Retensioning the belts

    \n
  2. \n
  3. Printing at slower speeds

    \n
  4. \n
  5. Checking all bed connections to make sure the bed is solid

    \n
  6. \n
  7. Reducing the jerk setting

    \n
  8. \n
  9. Activating Z-hop

    \n
  10. \n
  11. Reducing acceleration

    \n
  12. \n
  13. Increased retraction.

    \n
  14. \n
\n

The problem occurs on multiple STL files that used to work fine.

\n

Any help would be appreciated

\n

\"Photo

\n", "Title": "Ender 3 X-axis layer shifting", "Tags": "|creality-ender-3|", "Answer": "

OK I found the answer. I removed the extruder cover and found that the 2 screws attaching the extruder assembly were loose. A very simple fix to a very irritating problem.

\n" }, { "Id": "13849", "CreationDate": "2020-06-11T03:37:59.430", "Body": "

If I have an issue with my FLSUN printer how can I contact customer support?

\n", "Title": "How can I contact FLSUN support?", "Tags": "|flsun|", "Answer": "

They have a webpage by now www.flsun3d.com

\n" }, { "Id": "13856", "CreationDate": "2020-06-11T15:06:22.703", "Body": "

I'm newbie at this stackexchange and I have a \"FlashForge Finder\"; lately there are many problems with the horizontal surfaces quality (Bottom and Top at the same time).

\n\n

As some issue guides suggests, I changed: number of solid layers, % of filling and extrusion multiplier; I use \"Flashprint\", so some concepts must be translated from Cura in the options menu.

\n\n

P.S: All filaments are PLA.

\n\n

Without any change:\n\"enter\n\"enter

\n\n

After all changes (the \"best\")\n\"enter

\n\n

My settings for last black disk:\n\"enter\n\"enter\n\"enter\n\"enter\n\"enter\n\"enter\n\"enter\n\"enter\n\"enter

\n", "Title": "Bad quality at horizontal faces", "Tags": "|pla|quality|surface|flashforge-finder|", "Answer": "

Finally, I solved all faillures today.

\n

-When the bed is too close from the hotend:

\n

I cheat the machine with a cutter patterns board over the bed at the first extruder calibration; the distance increases several times and then You can do it shorter.\n\"enter

\n

-The another half part of bad quality [low maintenance of the hotend]:

\n

As a good newbie, I didn't know the periods of maintenance and the hotend had much plastic waste the PTFE tube welded and burned at internal, so the only option was to replace it for another same or to upgrade to "All Metal"; I chosen to upgrade with the replacement and I'll more maintenance to this zone.\n

\r\n \r\n
\n\"enter

\n

First attempt of trial piece:\n\"enter\n...PANIC!

\n

But then and FINALLY!\n\"enter\nAlmost all correct, I can continue with my printings.

\n

THANKS SO MUCH TO ALL, for your apportations and patience.

\n" }, { "Id": "13864", "CreationDate": "2020-06-12T19:33:21.343", "Body": "

I installed TH3D to my stock mainboard via this guide, but BLTouch does not work properly.

\n\n

It does not check all 9 points before point it just go middle then check z then start printing. BLTouch normally needs to check all 9 points before printing or with autobedleveling but somehow mine do not work like this.

\n", "Title": "BLTouch with TH3D Ender 3 pro", "Tags": "|creality-ender-3|bltouch|th3d|", "Answer": "

After setting up the BLTouch in firmware, you need to activate the leveling function for every print by inserting the leveling G-code 'G29' in the start G-code script of your slicer. The code should be placed directly after command G28.

\n

Note that you can test the working of the sensor from the printer display: sensor can be deployed, stowed, reset and tested.

\n

An alternative is to use the G29 and M500 command once in a while (for beds that don't change, don't get handled by excessive force and/or are operated at the same temperature) or store the shape of the bed through the display interface and put the command M420 S1 in your start G-code instead.

\n" }, { "Id": "13865", "CreationDate": "2020-06-12T22:30:18.707", "Body": "

I find that white filaments are quite translucent and printing 5 layers of white filament onto 2 layers of black filament (at 0.2 mm layers, the white layers being 100% infilled and the underlying black layer covering about 85% of the whole area) produces a slightly grey color on the top.

\n\n

Is that a limitation of the white colour (or the actual material used)?

\n\n

Are there materials, that address this issue to some extent?

\n\n

Adjusting layer thickness while keeping the overall height won't change things, right?

\n", "Title": "Are there white filaments, that are not so translucent?", "Tags": "|print-material|filament-choice|color|", "Answer": "

PLA filaments are pretty transparent in their pure state. Other filaments, especially fiber-filled ones, are opaque in nature. But in the end, the color is determined a lot by the colorful pigments added and the amount of pigments directly correlates to the opacity. The more pigment is added, the more opaque it becomes. There are several screws for the color though: Ammount and type of pigments. For transparent filaments.

\n\n

Not all pigments are the same. For example, white greatly differs between different brands. For example, my second-favorite brand's white is a little more in the \"cream\" color range, the filament sample that came with my TronXY X1 was very satin gloss and the first spool I ordered was a very white-opaque white. This all is because of the type of pigment used and the amount.

\n\n

How can you tell the amount? From the print properties and the looks of a printed filament: a filament that is very laden with pigments tends to print hotter than one that has few to none, but it also blocks light much better. I have experienced up to 20\u00a0\u00b0C between a transparent and the very heavily pigment laden white filament for the sweetspot. The same white filament was completely opaque after 1.6 mm in walls but most prints are!

\n\n

Remember: walls are generally thicker than roofs. If you really need to make the white covering fully, you'll need to make more white thickness, but thinner layers can help a slight bit as the included air in each layer helps a little.

\n" }, { "Id": "13874", "CreationDate": "2020-06-13T16:09:58.300", "Body": "

I installed the TH3D firmware to my Ender 3 Pro yesterday and today I am trying to configure the extruder rate because I am not using the stock extruder on my printer but have swapped in a BMG extruder. I edited these values and upload it to my board

\n\n
#define CUSTOM_ESTEPS\n//#define REVERSE_E_MOTOR_DIRECTION\n#define CUSTOM_ESTEPS_VALUE 415\n
\n\n

But these values do not change when I start to print something and because of that I need to edit it manually by going in to tune menu and editing flow to 415. How can I fix this issue?

\n", "Title": "Cannot configure default E step for extruder on TH3D", "Tags": "|marlin|creality-ender-3|firmware|th3d|", "Answer": "

Initializing

\n\n

After changing your firmware, you always need to overwrite the old values in the SRAM and EEPROM with those from the Firmware. Which you do with M502 & M500:

\n\n
\n

Installing firmware does not by itself alter the EEPROM, so these settings needed to be seeded into SRAM via M502 and then saved into EEPROM via M500.

\n
\n\n

You could also run a G-code that has only these two lines:

\n\n
M502\nM500\n
\n\n

Altering the EEPROM

\n\n

Via command

\n\n

Alternatively, you could access the printer via a terminal and send the command M92 E415 to overwrite the SRAM directly, then M500 to save the new setting to EEPROM. The associated G-code that only alters the E-steps/mm would read

\n\n
M92 E415\nM500\n
\n\n

Via Software

\n\n

Or you use a terminal that supports direct alteration of the EEPROM, like Repetier Host.

\n" }, { "Id": "13880", "CreationDate": "2020-06-14T15:26:02.713", "Body": "

I installed a BLTouch bed leveling probe on my printer which uses Marlin 2.0.5.3.

\n\n

Now the printer seems to be of two minds when it comes to finding the origin. Homing XY moves to the lower left as it always has, but homing Z moves not only to Z=0, but also to the center of the build plate. The printer knows this is (100,100,0) and is not mistakenly thinking it is (0,0,0).

\n\n

This causes some issues such as now the nozzle wipe at the beginning of a print happens right in the center of where the print is supposed to be.

\n\n

Is this expected behavior?

\n", "Title": "Installed bed leveling probe, now Z homing moves to center", "Tags": "|marlin|bed-leveling|homing|", "Answer": "

I had the same problem, solved it by inserting

\n
// Move X and Y to 0 after homing\nprocess_subcommands_now_P("G1 X0 Y0 F5000");\n
\n

at the end of G28.cpp, just before ui.refresh();

\n

This moves the print head to X0, Y0 and leaves Z untouched after the homing procedure. This way any oozing that might happen while the extruder heats up will be outside of the bed.

\n" }, { "Id": "13886", "CreationDate": "2020-06-15T13:58:45.843", "Body": "

If the right filament material is available, it seems practical for dentist to 3D print temporary crowns. Multiple images of the tooth to crown could be used for a 3D scan. Custom software or settings would probably make the software finish in less time. A 3D-printed temporary crown would probably take less work to get it to fit properly. Anyone know of development to do this? This probably would take a series of clinical trials to get FDA approval.

\n", "Title": "Printing dental temporary crowns", "Tags": "|health|medical|", "Answer": "

You ask about "filament", so I assume you expect fused-filament technologies. These are however not accurate enough, besides being prone to gaps and crevices which are problematic in crowns. The smallest viable nozzle, 0.2 mm, is still too rough for that.

\n

Dental 3D printers need to be very accurate, so the most common technologies used are stereolithography (SLA) and digital light processing (DLP).

\n

Crowns can be made using resin printing. It is possible to use also CNC machining of porcelain, but it's not part of 3D printing.

\n

More info can be found on https://all3dp.com/2/dental-3d-printing-guide/

\n" }, { "Id": "13897", "CreationDate": "2020-06-17T03:13:22.580", "Body": "

I would like to 3D print a small thin tub/mold for an epoxy resin. I have tubings inserted into holes, and I need to fix these tubings securely with epoxy (see picture below). The space is very limited, and the whole assembly must have a smallest possible footprint, so I have to confine the epoxy from spreading to the sides - that's why I need a tub. The tub itself must have as thin walls as possibly for the same reason.

\n\n

\"CAD

\n\n

The wall thickness is constant, so theoretically the nozzle could just make one single loop to print a layer, and then move to the next one. Kinda a spiral motion. It seems to be so simple! How do I get the slicer (I use Ultimaker 2 with 0.4 mm nozzle, CoPA material, and slice in Cura 4.6.1) to produce single outline walls?

\n\n

I tried so many things, but I couldn't get this.

\n\n

With the default settings for 0.2 mm layer a 0.4 mm wall (or thinner) will not be printed at all (left - 0.35 mm wall, middle - 0.4 mm, right - 0.45 mm):\n\"Default

\n\n

Occasionally even the 0.45 mm-thick wall gets excluded from the print, which is really bizarre:\n\"absent

\n\n

If I make the wall thicker, then the slicer tries to pack two discontinued lines next to each other, which is even worse. Cura has an option 'print thin walls', but this results in jerky, discontinued tracks.\n\"discontinued

\n\n

At the moment I print 0.45 mm walls with the 'print thin walls' option turned on, this is the closest to what I need that I could find so far.

\n\n

\"additional

\n\n

This may look fine in Cura, but the result is pretty ugly due to the additional nozzle movements... I really don't understand why the printer has to do them. It prints the outline, then jumps to the 'corner' and deposits a blob there. I can carefully remove these blobs with a scalpel, but come on, this is a disposable part and I need a ton of these!!!

\n\n

\"printing

\n\n

If that helps, here is a link to a sample STL file with 450 μm walls.

\n", "Title": "How to get a quality print of a thin single-walled shell?", "Tags": "|ultimaker-cura|slicing|ultimaker-2|", "Answer": "

You don't need to use vase mode. But vase mode will work.

\n

I'm not familiar with Cura, I use PrusaSlicer, but I'm sure there are equivalent settings.

\n

What you want to do is model the part in two pieces. The first piece will be the same height as the base. The second piece will be the top half. It can all be one model, but it helps to think of it as two.

\n

In the bottom part, add your hole, and print it with however many solid layers as is required to make the base thickness.

\n

For the top part, make it solid, and print it with 1 perimeter and 0\u00a0% infill and 0 top and bottom layers. You can decide the wall thickness by tweaking the extrusion width.

\n

If you want to make the part perfect, you can size the bottom hole by taking the dimensions of the upper portion and subtracting whatever extrusion width you will use from the surfaces.

\n

You can print as many of these as you want as close together as you can because it isn't using vase mode.

\n" }, { "Id": "13909", "CreationDate": "2020-06-19T13:26:03.193", "Body": "

Is there a slicer that can set the print speed for each layer as a function of the layer area? Larger areas give the layer a longer time to cool off before the next print layer. When the layer areas start to get small (usually at toward the end of a print if so) the layer may need a slower speed to cool off.

\n", "Title": "Is there a slicer that can set the print speed for each layer as a function of the layer area?", "Tags": "|slicing|speed|", "Answer": "

Most slicers have a feature in their cooling settings to "slow down if layer print time is below xxx".

\n

Setting this to a higher value should ensure that small / short layers aren't printed too fast, so that cooling is still reliable.

\n

Shorter layers are slowed down linearly to reach the specified minimum time - unless a "minimum print speed" is also set.

\n" }, { "Id": "13915", "CreationDate": "2020-06-21T09:35:34.383", "Body": "

I have a model of an eye that I custom made in Blender 2.83 that when printed only creates roughness on the printed object near where I had the supports. The supports are not the cause of the roughness (at least not completely) since the supports don't even touch the parts of the print where the majority of the roughness and bumps are (refer to my photos of the print)

\n

The roughness is only near the bottom part of the sphere as it prints upwards (refer to photos)

\n

What I'm looking for is a technique or any suggestions for printing this without the roughness so it's smooth like in the rest of the print. I'm also curious what is causing the roughness.

\n
\n

Eye Model in Blender

\n

\"Eye

\n

Blender Scale Ratio: 0.001

\n

Blender Units: mm

\n
\n

Eye Model in Prusa Slicer

\n

Layer Gif

\n

\"Eye

\n

Prusa Slicer Main Settings (Higher Res)

\n

\"Prusa

\n

Variable Layer Heights For Smoothness

\n

\"Variable

\n
\n

Eye Model Prints Photos

\n

Eye Model Small Version 3D Print (Notice it's smooth on top of print)

\n

\"Eye

\n

Eye Model Small Version 3D Print Trouble Area

\n

\"Eye

\n

Eye Model Small & Large Version With Support (Notice roughness on the sphere)

\n

\"Eye

\n

Example of Support Used On Small Print

\n

\"Example

\n

Smooth on inside of print

\n

\"Smooth

\n
\n

--------- Print Details ---------

\n

Printer: Prusa i3 MK3s

\n

Filament: PLA Galaxy Silver (Prusa Reserach)

\n

Slicer Software: Prusa Slicer

\n

Print Temperature First Layer: 205 degrees

\n

Print Temperature Other Layers: 190 degrees

\n

Notes: The suggested temperature for the filament is 205-215, I've adjusted after careful calibration given my environment to a lower temperature to reduce stringing. I created a tower at different temperatures and discovered 190 was the perfect setting to reduce stringing in my case with this material. Refer to my screenshot below. I do not think temperature has anything to do with this since the print is smooth inside and near the top without any issues.

\n

Temperature Tower Test For Filament Photo:

\n

(Note stringing in the cone test areas at 225 to 205)

\n

\"Temperature

\n
\n

--------- Prusa Slicer Settings Photos ---------

\n

Filament settings

\n

\"filament

\n

Extruder Settings

\n

\"extruder

\n

Support Settings

\n

\"Support

\n", "Title": "3D Printed Sphere, How to Remove Roughness", "Tags": "|prusa-i3|troubleshooting|prusaslicer|", "Answer": "

I\u2019ve noticed the best way to get it smooth without support is to make the printer print the inner parts of the model first. That way it gives the hanging layer something to hold on to. I turn my window ac unit up to high aim it at the printer so it cools the filament quickly. Next you\u2019re going to want to mess around with the extruder heat. Try to get it as low as possible but watch out for the good gooey bad gooey range. If the first overhanging layer is close to the bed turn the bed heat off. Then tinker with the flow try to get it low but not super low I go as low as 70 \"enter

\n" }, { "Id": "13931", "CreationDate": "2020-06-23T13:38:58.787", "Body": "

Printing supports, either complete, or partially from PVA have not always resulted in successful prints according to my experience with the filament (used in a dual extruder Ultimaker 3 extended). But, when it works well, the surface finish is perfect as there is no gap between the PVA and PLA.

\n

From my experience with PVA, I conclude it is prone to clog, the filament is very hygroscopic, resulting in popping sounds when printing if too moist which most probably also impacts on clogging. The clogs lead to failed support structures as the extruder grinds through the filament and as such failed prints.

\n

I was wondering if PETG can be used for supports or for the interface layer of supports for printing supports for PLA prints? For example:

\n

\"Non \"Non

\n

This is a sliced view of a print in line color (left or top on small screens) and material color (right or bottom on small screens); black PLA and cream colored PETG. This print contains a larger gap that needs support. Can you make the support from PETG, either the support as a whole or just the top interface.

\n\n", "Title": "Can PETG be used as support material for PLA?", "Tags": "|pla|support-structures|petg|support-material|", "Answer": "

PETG works as support material for PLA, see video

\n

\r\n \r\n

\n

In theory, PLA printed on top of PETG will be fine because PETG softens and gets sticky at higher temperatures.

\n

Printing PETG support on top of PLA may cause remelting of PLA, but if PETG is kept quite cold (220\u00a0\u00b0C) the issue will likely be minor. As shown in the video, it works.

\n

PLA/PETG may still be better than using PLA/PLA because of this difference in extrusion temperatures that, for example in bridges where PLA is printed on top of PETG support, should result in very easily removable supports.

\n

The type of supports used should be tested: tree supports could minimize the contact surface between the two and thus minimize marring by the hotter PETG being deposited onto the PLA at the expense of more PETG and normal supports could be used on a limited surface, so they can be removed easily. If the two materials really don't adhere much to each other, you may even be able to fake dissolvable supports, which increase the contact surface but provide a far better finish for bridges and bottom surfaces.

\n

Using a single nozzle may require experimentation. In my experience, I print PLA at 230 \u00b0C so it wouldn't be an issue, and 220 \u00b0C would also work, but if PLA is printed cold, below 215 \u00b0C, you may need to heat/cool the nozzle. In any case, switching filament (especially PETG -> PLA) requires quite some filament to be discarded, so there is time for the heating. It is however to be kept in mind that cleaning of the nozzle between PLA and PETG (or in fact after any material swap) is difficult: some residue can remain in small gaps or low flow areas of the hotend and will be blended into the stream for quite some time after the swap, resulting in hybrid materials of unknown properties.1

\n

As I said, experimentation is needed for this kind of task: my experience, with a 2012-era hot end, may not even be representative of the behaviour of modern hot ends.

\n
\n

1 - this effect is easily noticeable even with similar materials if switching from dark to light color filament, especially if not doing a cold pull to remove most of the material from the melt zone.

\n" }, { "Id": "13933", "CreationDate": "2020-06-23T17:10:58.707", "Body": "

I have a recently purchased Ender 3 Pro, and it may well already have a bootloader on it. I don't have the various adapters to flash one yet.

\n

Is it destructive to try flashing over USB without the bootloader? Does it fail in a non destructive way?

\n

I have watch some tutorials which claim that it's fine to just try it, but I'd like to be more sure about that.

\n", "Title": "Will I brick my Ender 3 if I try flashing without a bootloader?", "Tags": "|creality-ender-3|firmware|", "Answer": "

My understanding is that if there is no bootloader, you can't flash over USB; you need an ISP programmer attached to the board. So if you're able to flash over USB, that means there is a bootloader, and updating via the bootloader should leave the bootloader in place and just overwrite the rest of the firmware. I'm not sure how strongly this is enforced, though, so you may want to wait for supporting comments/answers from others.

\n

Of course it's always possible to damage your printer in ways that require replacement of the board or at least some parts on it if you flash bad firmware, so you need to be cautious anyway.

\n" }, { "Id": "13937", "CreationDate": "2020-06-24T01:37:39.060", "Body": "

I've seen there are several different types of source material for 3D printing. My question is: Which of those would hold up under regular Loctite superglue or plastic model glue?

\n

I'm asking because I want to try 3D printing, and I have a project in mind. But I don't want to spend too much, so I'm looking to get a small 3D printer. I want to make a clock, and I can design the step down gears to make in a small printer, but I'd need a large base for so many smaller gears. I want to print pieces of the base, and superglue them together.

\n", "Title": "3D printing source material and superglue", "Tags": "|3d-design|print-material|", "Answer": "

CA glue works on PLA, especially if you're gluing parts that fit together rather than small surfaces that just touch, but I'd encourage you to consider alternatives just because there are so many more ways to attach things when you have freedom to design the parts, and non-glue approaches admit disassembly, repair, etc.

\n

Some possibilities include:

\n\n" }, { "Id": "13940", "CreationDate": "2020-06-24T10:22:15.940", "Body": "

I'm having an issue where prints with narrow tolerance come out fused. This makes it pretty much impossible to print anything with narrow parts. It seems to be mostly (maybe only) an issue in the bottom skin layers. Once it gets through those, the rest of the print goes smoothly and tight tolerances are not a problem.

\n

Here's an example where I've tried to print some hinges:

\n

\"First

\n

It's a test print where each hinge has a different tolerance, so the left and right are expected to look different. Below each real image is a preview layer from Cura showing how it's supposed to look.

\n

The first layer appears OK. The second layer looks a little messy, and the gap between inner and outer circles has shrunk. By the third layer the hinge on the right is completely fused.

\n

The printer is an Ender 3 Pro with a glass bed (flat glass, no special surface), BLTouch, and Marlin 1.1.9. The slicer is Cura 4.6.1, and for this test print I used the default for "Super Quality - 0.12\u00a0mm" at 200\u00a0\u00b0C and 60\u00a0\u00b0C bed with no changes. The filament is Mika3D PLA.

\n

Some things I've tried to fix this:

\n\n

What else is there to check?

\n

Edit 2020-06-26:

\n

At R.. GitHub STOP HELPING ICE's suggestion I returned the e-steps to default (93), re-leveled the bed, and adjusted the z-offset tighter. I made 10 attempts with varying z-offsets, and here's the best one:

\n

\"First

\n

The first layer looks better! But the second and third layers are just as bad as before, in fact maybe worse. The circle on the right completely fused on only the second layer. And the top surface is just as ripple-ey and messed up as before.

\n

Here's a closeup of the fourth layer to show how bad it is:\n\"Fourth

\n

So although the re-calibrated e-steps may have been a problem, that clearly wasn't the only problem. What else should I be looking at here?

\n

Edit 2020-06-27:

\n

At Davo's suggestion I double-checked all my slicer settings. Flow is set to 100% everywhere, wall thickness is 0.8 mm for two walls (so 0.4 mm each), and nozzle diameter is correct at 0.4 mm.

\n

At R.. GitHub STOP HELPING ICEs suggestion I double-checked my filament diameter. It is set to 1.75 mm. On the actual filament, my digital caliper measures 1.74 to 1.76, within the expected tolerance. So that doesn't appear to be the issue.

\n

At 0scar\u2666s suggestion I tried a print with 0.2 mm layer height. Here's the first layer:\n\"0.2\nLooks like the same over-extrusion.

\n

Then I tried reducing the flow multiplier to 90% (for both "flow" and "initial layer flow") and printing at 0.2 mm layer height:\n\"90%\nBetter, but it still looks over-extruded!

\n

I don't know what else to try.

\n", "Title": "Expansion in bottom skin after first layer", "Tags": "|pla|creality-ender-3|calibration|glass-bed|", "Answer": "

I had the same problem with my Ender-3 V2.

\n

You need to check if the feeder bracket is square like explained in this YouTube

\n

If that is not the problem maybe you need a custom bracket to change the spacing between the Z-motor and the frame like this, this or this

\n

I have improved mine a lot using:

\n\n" }, { "Id": "13945", "CreationDate": "2020-06-24T18:12:01.233", "Body": "

I have a model that's placed on the bed exactly like on this picture:

\n

\"sample

\n

I have constant quality degradation as the bed moves down to print in the upper left corner (1).

\n

Everything is fine on the X (2)-(3) side. It does not have any visible artifacts. All hell goes along the (1)-(3) curve:

\n

\"hell

\n

Top left corner (1):

\n

\"top

\n

On the way from (1) to (2) lines seem to disappear almost completely.

\n

I used Cura slicer and these printing settings:

\n\n

It looks like a mechanical issue, so I tried tightening/untightening bed bolts. It didn't help. They are a little bit tight, but not too much. The bed does not seem to be wobbling. Also, I tried the bed for wobbling in its top/bottom position. It looks fine along all the way.

\n

What should I try next?

\n

Extruder steps/mm are tweaked for this filament. Extruder produces exactly 97\u00a0mm of 100\u00a0mm of filament.

\n

UPD

\n

I decided to change my software/hardware settings step by step. This time I changed only my software settings to these:

\n\n

Corners have become much sharper and there is a lot less of bulging on the arc.

\n

\"arc\"

\n

However, by X-axis (2) - (3) I see more artifacts:

\n

\"x-axis

\n

Y-axis has become better:

\n

\"y-axis

\n

Currently, I don't have any visible or sensible bed / X play. I tuned rollers to have enough tension not to slip if rotate them separately. So, if I rotate the roller, it moves the whole bed or X carriage. I'll try increasing the tension a little bit and then I'll share the result.

\n

UPD2

\n

I've made belts a little bit tighter and decided to print a new model. The layer height is 0.3 mm. Also, I tried increasing temperature up to 240 \u00b0C and changed the stock vent with a circular vent. The wall count is 50 to make the model solid. Coasting is off.

\n

\"x-axis\"

\n

\"x-axis

\n

Now all artifacts are along the X-axis. There are many fewer of them at (1) than at (2). The model is a doorstep. On the build plate it's placed like this:

\n

\"doorstep

\n

Now I think the problem has nothing to do with X/Y play and these two factors can be eliminated. I'll revert belt tensions back to their previous values and decrease the printing temperature down to 225-230 \u00b0C.

\n

PS. USBASP is still in customs, so I'm doing all this on the stock firmware.

\n

UPD3

\n

I have finally figured out what was wrong. It was insufficient Z-belt tension on both sides. A close look at a DSLR camera shot gave me a clue: there was almost always a straight segment followed by a visible additional step down between layers.

\n

There are still some artifacts but everything looks relatively tolerable now.

\n

\"after

\n

Thanks to all of you guys!

\n", "Title": "Visible lines along Y-axis on Ender 3 Pro", "Tags": "|print-quality|creality-ender-3|petg|y-axis|", "Answer": "

There are a lot of different print quality problems going on here, but the biggest I see is the bulging and sagging at the corners of each layer. This is a result of extrusion not being a zero-delay linear function of extruder motor motion, but subject to compression/pressure. This causes excess extrusion when printing slows down (approaching and rounding a corner) and underextrusion just after speeding back up. There are several possible fixes:

\n\n

Also make sure you don't have your slicer set to slowdown on overhangs, as it makes this phenomenon far worse.

\n

The other surface artifacts I'm less sure about, and you should probably look for other answers about resolving them. Trish is right that the lines look like a retraction problem - material intended to be there got lost somewhere else. It's probably due to combing (skipping retraction inside the model and letting material ooze out there). I would set combing to "not in skin" and set the max combing distance to something very small (around or slightly less than 1 mm) and see if that fixes the problem. Combing is especially bad with PETG in my experience.

\n" }, { "Id": "13961", "CreationDate": "2020-06-27T15:53:20.677", "Body": "

This is regarding the circuit of my 3D printer Tronxy X5SA's stepper motor for X axis.

\n

I had an observation: It has a cooling fan (40X10 24V) which is having issues lately - it stops working sometimes, and when pushed to rotate, spins back.

\n

Problem: Now one of the stepper motors (NEMA 17 ~3.5kgfcm) in my core XY has been malfunctioning, where it misses rotation sometimes out of the blue, which shifts the whole print. I thought this is due to the driver limiting the current to the motor, so I rotated the screw terminal clockwise a bit more to allow more current for the motor. This works fine for a while, but the problem comes back.

\n

Now, I also noticed a few times that this issue occurred when the cooling fan for the drivers stopped working.

\n

Can the stopping of the cooling fan cause the stepper motor to falter?

\n

Another observation is that the stepper motor which is faltering is placed close to another stepper motor. Although I know stepper motors can work in extreme conditions, can this cause the issue?

\n

If you require any more info to investigate the issue, please do let me know

\n", "Title": "X axis stops working sometimes on my Tronxy X5SA", "Tags": "|stepper-driver|stepper|x-axis|fans|tronxy-x5|", "Answer": "

Yes, if the stepper motor drivers overheat, they shut down or skip steps, that is very possibly related to the failing stepper driver cooling fans.

\n" }, { "Id": "13972", "CreationDate": "2020-06-29T00:28:41.437", "Body": "

When a hole is created for a bolt, it might be "tight"; the intent is for the hole to be tapped. Or it might be "loose", and the intent is for the bolt to slide into the hole and attached to a nut on the other side.

\n

What is the terminology for these two kinds of holes?

\n", "Title": "Bolt hole terminology for \"tight\" and \"loose\" bolt holes", "Tags": "|terminology|", "Answer": "

A hole designed for a screw/bolt to slide freely through is called a clearance hole. A hole that is designed to be tapped could be called a pilot hole.

\n" }, { "Id": "13975", "CreationDate": "2020-06-29T02:30:05.760", "Body": "

I'm printing with anycubic i3 mega on an ultrabase bed. When I first got the printer the prints were easy to remove from the bed after it cools down, I didn't need to put any extra effort. However after I used 70% isopropyl to clean it it seems I removed some kind of extra coating as all next prints were sticking to the bed firmly even after the bed cools down. So I tried to heat the bed up to 100 degrees and then cool it down and wait until it gets to something like 35, at that point print comes off quite easily (really helped me with some big parts with huge initial layer) so I wonder if I should just add that extra heating cycle to the end of each print job.

\n

Is there any possible problems with that?

\n

PS my understanding is that PLA should be okay with short temperature spike since it is being melted with twice as high heat. However long exposure to that temperature might cause some deformation (eg if I throw the printed part into dishwasher).\nAnother possible concern is that extra heating cycle could potentially shorten life of the ultrabase, but not sure if it is the case.

\n

UPDATE

\n

so after some experiments I printed at least a dozen calibration cubes adjusting several parameters along the way as it seems each of them contributed to the issue

\n\n

I can say it is easier to remove the cube from the bed now (used to be very difficult and I was using a mallet almost every time in the beginning) but it still doesn't come off on its own. I also noticed that now all three dimensions are almost identical (Z was about 0.5 less). And all X/Y/Z are ~19mm after cube cools down (the model is 20mm) so I wonder if I need to fix that one now

\n", "Title": "Any possible issues with adding heat/cool down cycle in the end of print (PLA)", "Tags": "|adhesion|anycubic-i3-mega|anycubic-ultrabase|", "Answer": "

You did exactly the right thing:

\n
    \n
  1. look,
  2. \n
  3. see,
  4. \n
  5. think,
  6. \n
  7. evaluate and adapt
  8. \n
  9. test
  10. \n
  11. Back to 1. or continue to 7.
  12. \n
  13. Solution found and applied!
  14. \n
\n

Congrats!\nJohan

\n

Ps: I use the tape in paper, used to mask before paint jobs, I was not that keen as you! It always comes off, and I restart every time with a clean glass, no hairspray, glue or others. They cost nothing, and if you are handy, you can use 100mm wide stripes. (or just the amount needed for your part.

\n" }, { "Id": "13981", "CreationDate": "2020-06-29T20:33:34.190", "Body": "

Does anyone know of a (manufacturer independent) software that supports different types of SLA printers?

\n

Since there are many different SLA type printers around, I was wondering if there are any unified or open source methods for slicing in case the manufacturer drops support for the printer software (or my OS upgrades on me overnight and breaks compatability).

\n", "Title": "Do SLA printers have open source slicers available to use?", "Tags": "|slicing|sla|", "Answer": "

From what I have researched, each brand of printer has their own method of slicing objects for their printer to parse. For example:

\n\n

There doesn't appear to be any golden ticket type of method (like G-Code) for slicing and providing instructions.

\n" }, { "Id": "13988", "CreationDate": "2020-06-30T17:18:47.347", "Body": "

Why is cupping bad? (I don't mean hollow parts)

\n

Example:

\n

Lets say that I want to print a simple cup without a handle.

\n

There are two obvious orientations:

\n\n

I don't want to add a drainage hole to my cup for obvious reasons :) Also, I am using a formlabs form 3, if it makes a difference.

\n

A cup:

\n

\"enter

\n", "Title": "Why is cupping bad in SLA prints?", "Tags": "|sla|resin|print-orientation|", "Answer": "

The printer prints, then moves up, then down again. The print surface stays inside the resin vat at all times. As a result, we have this experiment:

\n

\"enter

\n

The "bottle" is resting in a vat of liquid. As we raise it more and more, it does not drain until the lower lid is free of the liquid surface or some point of the shell delaminates. The release of the resin can happen in a rather violent way - which in turn could deform the print in the making. Delamination rips through the part till air can enter the enclosed space, destroying the print in the process.

\n

Even if printing the mouth against the plate you'll have cupping if you have a solid plate to print against. This can be mitigated with a little angle but trap liquid in the print at the end or including a couple of small gaps close to the surface to allow air to get into the print - yet unless the resin can flow out at the bottom some will be trapped in any case.

\n

To prevent cupping, I would turn the cup to print sideways, that way resin and air can be exchanged.

\n" }, { "Id": "13993", "CreationDate": "2020-07-01T17:59:40.483", "Body": "

I'm using FreeCAD, and I would like to duplicate a 2D sketch on another plane, but I can't seem to find an obvious way to do this.

\n", "Title": "In FreeCAD, how do you copy a sketch from one plane to another?", "Tags": "|cad|freecad|", "Answer": "

I\u2019m assuming you are working in the Part Design workbench. Select the sketch and click Edit > Duplicate selected object. Make sure that only the sketch is copied (depending on the version, either deselect the plane or click don\u2019t include dependent objects). This will produce a duplicate sketch in the active body (if you want the duplicate in a different body, make sure it\u2019s active first by double-clicking it if necessary). You can then click Sketch > Reorient Sketch to move it to a different plane.

\n

By the way, FreeCAD has a very active and helpful forum at https://freecadweb.org. I highly recommend asking FreeCAD questions there.

\n" }, { "Id": "14000", "CreationDate": "2020-07-02T10:59:45.537", "Body": "

I am completely new to 3D Printing. I got my first printer, a Creality Ender 5 Pro, yesterday.

\n

My problem

\n

I shutdown the printer without the axis being in home position (X: 0, Y: 0, Z: 320 instead of X: 0, Y: 0, Z: 0). I thought this should be no problem but after turning the printer on again the info screen showed the axis position as 0, 0, 0 again.

\n

So, I can't move the Z axis up now because the printer thinks it's already at 0.

\n

NOTE: On the Ender 5 the bed lowers for increasing values of the Z. So 320 is the lowest and 0 the highest. I know that the motor works because it tries to go down further if I increase the Z position, but I am scared of damaging the motor because it can't move further but it tries to (judging by that weird sound).

\n

My question

\n

Is this a normal behavior that the printer axis cant remember its position? Because I think when I built the printer the axes were also not at the 0, 0, 0 position and on the first start they moved back without any problems.

\n

Does anyone have any ideas on how to solve this? Or is this a broken printer?

\n

PS: I could replicate this behavior on X and Y as well (moving them with prepare->move axis and then shutting down the printer). But in this case, I can easily disable the motors and move them manually back to 0, 0. This isn't the case for the Z Axis.

\n

I hope I explained that understandably.

\n", "Title": "Why can't my Ender 5 Pro remember its axis position or move back to home anymore?", "Tags": "|creality-ender-5|homing|axis|", "Answer": "

(Summed up from several entries in this thread.)

\n

It is an untrue statement that the printer can't move back to home anymore, until the G-Code G28 is executed. The printer can home, but must be ordered to do it.

\n

There are several ways to home the printer (LCD hints are for Marlin firmware):

\n\n" }, { "Id": "14002", "CreationDate": "2020-07-02T19:28:53.050", "Body": "

I'm going to use TMC2208 stepstick in a printer and the firmware can control via UART the current during printing and during holding (static, no movement): the firmware has an explicit setting for running current and for holding current.

\n

In this case, should I care about the physical trimmer which controls the Vref?

\n

I tried to understand the TMC2208 datasheet (page 50) when it's talking about current control but I don't understand whether the Vref becomes superfluous or it still acts as a maximum value which the UART must obey.

\n", "Title": "Is the Vref trimmer relevant on TMC2208 in UART mode?", "Tags": "|stepper-driver|tmc2208|", "Answer": "

The potmeter does not work in UART operation, see e.g. this quote:

\n
\n

You don't have to fiddle with jumpers to set your micro-steps, just\nchange it in the firmware, and you can dynamically change the amount\nof current going to each stepper motor (no more adjusting that small\npotentiometer on the driver board) just by sending a GCode command\n(M906).

\n
\n

So, you cannot change the Vref of a UART operated stepper driver, setting the current directly is the way to go on UART operated stepper drivers; in fact that is actually what you are doing on non-UART operated stepper drivers, you change the Vref to change the current through the stepper motor, the higher the voltage the higher the current through the stepper the more torque the stepper has.

\n" }, { "Id": "14004", "CreationDate": "2020-07-03T10:06:06.487", "Body": "

All my prints come out about 1 mm too short in the Z dimension. So for example a 20 mm cube comes out 19 mm high. A 10 mm cube comes out 9 mm high. The X and Y dimensions are fine. There's a little bit of visible elephant's foot at the bottom, so I assume whatever is happening is in the first couple of layers. The problem is fairly consistently around 1 mm even for larger prints.

\n

\"10\n\"20

\n

The printer is an Ender 3 Pro with a glass bed and BLTouch for automatic leveling, but otherwise stock.

\n

I had a similar issue with another Ender 3 Pro that was resolved thanks to a link to this question about problems in the first 3 mm. The solution was turning the eccentric nuts on the left and right to loosen the rollers that connect the X-axis gantry to the vertical posts. There the Z issue was not as pronounced, and I was getting really messy prints in the first few Z layers. Here that is not an issue; the first few layers look fine while they're printing. Loosening the rollers did not resolve it.

\n

Things I've tried:

\n\n

I'm using the stock Ender 3 Pro profile in Cura, and printing at 0.2 mm layer height. I've kind of run out of things to check. What else can cause Z height loss in the first few layers like this?

\n", "Title": "What can cause Z height loss in the first few layers?", "Tags": "|creality-ender-3|pla|calibration|z-axis|glass-bed|", "Answer": "

Do you have any \u201cslop\u201d on the right side (non motor) of the gantry?

\n

I\u2019ve noticed that my gantry will settle on the right side and lag behind the motor driven - ever so slightly - when it starts to drive up. It will, after that first lag, move fine for the rest of the time. Z axis travel seems barely affected but all my prints are consistently about 0.5 mm short.

\n" }, { "Id": "14007", "CreationDate": "2020-07-03T19:43:47.467", "Body": "

I\u2019m thinking of building a diy H-bot printer but I have some questions...

\n
    \n
  1. How can I tell the firmware that the printer is an H-bot one? I just need to uncomment the CoreXY option? Is it the same?

    \n
  2. \n
  3. Does the print bed need to be square (for example 300x300) or it can be also a rectangle?

    \n
  4. \n
  5. Are the steps/mm the same of the ones on a Cartesian 3D printer?

    \n
  6. \n
  7. How can I recognize which motor goes on which connector on the motherboard? (The X and Y connectors I mean).

    \n
  8. \n
\n", "Title": "How does an H-bot printer work?", "Tags": "|diy-3d-printer|corexy|", "Answer": "

The CoreXY kinematics can be seen as an evolution of the H-bot kinematics. In Marlin, you both need to configure the printer as a CoreXY machine. Note that your steps are determined by the pulleys in the steppers and need to be the same for the steppers. With testing you will find out if you have the correct value.

\n

There are many popular designs out there; e.g. the Hypercube and the Hypercube Evolution (I have built the latter myself).

\n

I would not recommend building an H-bot, these have an inherent design flaw in that the load is asymmetrical causing the carriage to be stressed by a torque causing racking. To minimize this racking, you would require more expensive tight tolerance hardware like proper linear rails (usually not the kind that you find on typical auction or Asian vending sites, but actual pricy Japanese or German hardware). The CoreXY kinematics stress the carriage symmetrically. Note that the difference between an H-bot and a CoreXY printer is only the length of the belts, nowadays good quality belts can be bought for any length you need.

\n

Note that a square or rectangular bed is a non-issue, just specify the dimensions in the configuration. The only thing you need to find out is how to wire the steppers. I connect one stepper to one driver and the other stepper to the other driver. I then did some tests and found out I had to flip one stepper motors connector to get the correct movement. I could have reversed this in firmware as well.

\n" }, { "Id": "14010", "CreationDate": "2020-07-04T11:50:06.563", "Body": "

I just printed my first cube from my newly arrived Artillery Sidewinder X1. I am totally new to 3D printing but I managed to correctly unbox, assemble and prepare it for printing.\nI used PLA filament and the cube ended up great. I removed it using a cutter gently pulling it up from the first layer and it popped right off.

\n

A little layer of material was still stuck on the bed and, I don't know why, my dumb brain decided to use ethyl alchool 90\u00a0% to scrub it off on that little surface. I immediately used some water to wipe the alchool off but it was too late.\nNow on that portion of the bad all I see is a matte area that I can't fix.

\n

Did I irremediably ruin my bed? I am so bummed I can't even describe it. All went great but I decided to do this on my own and I failed. I do know that the bed is replaceable but I just hope is not very expensive.

\n

Here's a couple of pictures to give you a look of the damage.

\n

\"enter

\n

\"enter

\n", "Title": "I mistakenly used 90% ethyl alchool to scrape some PLA off my bed; did I ruin it?", "Tags": "|adhesion|glass-bed|artillery-sidewinder-x1|", "Answer": "

First of all, let's look at what kind of bed you have. According to a review, it is a "porous ceramic coated glass surface."

\n

Your bed is fine...

\n

Glass and almost all ceramics are virtually impervious to most liquids, be them alcohol or even most acids unless that acid is hydrofluoric acid. So on a chemical standpoint, your bed is most likely ok, just the alcohol managed to leech some of the coloration or deposit dust in the surface - which is no problem usually. The matte might even just be PLA stuck in the surface, so if this happens if you print in a different area, you know that is not damage per see it's a normal sign of use.

\n

So from that standpoint, I see no problem.

\n

...but there are safety issues with the bed design

\n

Where I see a problem though is the construction of the bed itself: it runs on mains voltage and is heated directly, which can cause all kind of problems, especially breaking off the cable as there is no proper strain relief on the cable! Atop that, the review points out that the bed heats uneven, which can very quickly lead to stress and breaking of the bed.

\n

Because of this construction, I strongly advise to refit a strain relief and use utmost care no to touch the bed during operation. Make sure not to hammer onto it and don't overheat it to prevent thermal cracking.

\n" }, { "Id": "14014", "CreationDate": "2020-07-05T05:46:46.427", "Body": "

I bought a month ago the Artillery Genius. Everything was pretty good until today, I was printing a test cube and it stopped pulling out filament, when I looked the temperature it was -15\u00a0\u00b0C, a few seconds after the thermistor sensor broke and got stuck on the extruder creating a little bit of smoke. I had bought a thermistor sensor and a heater cartridge original replacement. I changed both and turn the printer back on. As soon I turn it on the extruder start to increase the temperature, almost 300\u00a0\u00b0C and then change to -15\u00a0\u00b0C. I checked the connections multiple times, I even change the sensor with another one, but nothing works. I checked the boards, cables but everything looks great. Don't know what else can be. I already contacted customer support but just want to look for another opinion while waiting for there answer.

\n", "Title": "Extruder increase temperature", "Tags": "|safety|thermistor|", "Answer": "

Your printer does seem to have a firmware problem and lack important safety features - it should have stopped the moment it went 300\u00b0C measurement, beeped like hell, and shut down with an error message about Thermal Runaway Protection. Immediately stop using the printer!

\n

Before you try to print again, you need to make a proper firmware for your printer, one that has both Mintemp, Maxtemp as well as Thermal Runaway Protection actiive and flash that. While setting up your firmware, make sure you use the right temperature table for your thermosensor. How to do that is detailed here.

\n" }, { "Id": "14040", "CreationDate": "2020-07-07T19:45:48.143", "Body": "

\"Sensor\"

\n

\"enter\nI'm just finishing the set-up of a CR-10 Max. It is a new printer.

\n

I don't manage to feed the filament through the material shortage sensor.

\n

I can hear the micro switch click; the LED turns blue, then a few millimeters after that (33\u00a0mm total from the entry point), there is something that prevents the filament from going forward. I don't see any switch on the outside of the sensor, and I applied a reasonable amount of force on the filament.

\n

Can you tell me how to troubleshoot this ?

\n", "Title": "How to feed the filament through the material shortage sensor?", "Tags": "|extruder|creality-cr-10|filament-sensor|", "Answer": "

Here is 3D model that better explains why it was catching and how to remedy the problem:

\n

\"enter

\n

\"enter

\n

\"enter

\n

\"enter

\n

The filament is likely to have some twists and the bevel you create may not be in the correct orientation once it catches the ridge.

\n

The out hole is not chamfered on the inside, as it is drilled from the outside, and I guess a chamfer on the inside is not available at this price range.

\n

I found that twisting the filament 90\u00b0 at most one way or the other always helps the filament finds its way. It can be tricky because of the small diameter/stiffness of the filament, with the spool still attached to the other end.

\n" }, { "Id": "14041", "CreationDate": "2020-07-07T20:04:17.993", "Body": "

So basically, I have 3 different types of steppers.

\n\n

This setup should work right?. I'm using TMC2209's stepper drivers and all are well within the drivers rated phase current limit.

\n", "Title": "Can I use different sized steppers for different axes?", "Tags": "|diy-3d-printer|stepper|nema-17|tmc2209|", "Answer": "

Each axis is fine using a different stepper size and/or quality. You will still need to tune their operating current and steps/mm for each of course. Make sure your motor mounts fit too.

\n

Where it might become challenging is if you wanted two different steppers on the SAME axis. As in, a NEMA17 + NEMA23 for a dual Z axis. I don't think this is what you intend though.

\n" }, { "Id": "14042", "CreationDate": "2020-07-08T01:32:45.770", "Body": "

I have a TEVO Tornado (bought quite recently) and I want to know what is the best type of smoothers MKS smoothers or TL smoothers? Are they the same thing?

\n", "Title": "Smoothers for TEVO Tornado printer", "Tags": "|stepper-driver|smoothing|tevo-tornado|", "Answer": "

The use of smoothers totally depends on the stepper drivers you are using! Note that the TL and MKS smoothers do exactly the same thing. Both use an arrangement of 4 or 8 diodes. Details of the problems with drivers and the working of the smoothers is explained in this blog post. The problem is that some stepper drivers are not able to produce low currents as of the present dead-zone.

\n
\n

if we had a way to modify the motor so that with a voltage of 1.4\u00a0V there would be no current flowing, then the driver would be able to generate all the currents because it would always be spitting out more than the minimum voltage

\n
\n

The diodes prevent current to flow at the cost of a voltage drop; a 1.4\u00a0V voltage drop (2 diodes) would prevent current to flow, as such you see diodes in series on the smoother boards.

\n

These MKS/TL smoothers help with smoothing out the signal going through stepper motors; e.g. the notoriously noisy DRV8825 motor drivers are known for a stepped sine curve rather than a smooth output.

\n

More modern chipsets such as the TMC21xx, TMC22xx, and TMC51xx do a much better job at providing smooth signals, and surprisingly, so do the cheaper drivers like the A4988s!

\n

So if you are using stepper drivers that do not produce a smooth sine wave, like the DRV8825 stepper drivers, you could potentially benefit from installing smoothers. This could help with salmon/zebra skin/moire and ringing print artifacts/defects.

\n

To quote the popular All3DP 3D printing site from "TL Smoother: Should I Add One to My 3D Printer?":

\n
\n

It\u2019s a yes if you\u2019re running DRV8825 stepper drivers. This was\nwhat TL smoothers were designed to do: fix a design flaw in the\nDRV8825. Your mileage may vary based on your printer\u2019s power supply,\nbut you\u2019ll likely see noticeable improvements in print quality.

\n

It\u2019s a no if you\u2019re running newer Trinamic stepper drivers.\nTrinamic drivers have many \u201csmart\u201d features built in that don\u2019t suffer\nfrom the same issues as the DRV8825 and already counter electrical\nnoise. As a post from Trinamic notes, adding TL smoothers doesn\u2019t\nprovide any significant benefit; it only increases power consumption\nand heat generation.

\n

It\u2019s a maybe if you\u2019re running other drivers. Other stepper\ndrivers might not have the DRV8825\u2019s design flaws, but they may\nbenefit from the slight electrical dampening created by the TL\nsmoother\u2019s circuit. Considering the smoothers\u2019 low cost (~\\$8-15 for\npacks of 3 or 4), it doesn\u2019t hurt to try it out and let the results\nspeak for themselves.

\n
\n" }, { "Id": "14047", "CreationDate": "2020-07-08T10:08:43.587", "Body": "

My Creality CR-10 Max is equipped with a BL Touch sensor for bed leveling.

\n

The bed size is 450 mm x 450 mm.

\n

Here is the 16 measurement points taken from a 4x4 grid:

\n
float  v[] = {-1.15625, -0.7625, 0.1525, 1.13,    \n-1.1150, -0.5150, 0.2125, 1.650,    \n-0.8525, -0.215,  0.510, 1.4425,    \n-0.4125, 0.2649, 1.0350, 1.9050};\n
\n

Which looks like this:\"enter

\n

The bed doesn't look flat or level. To what extent should I try to make the bed flat and level?

\n

It's now a day later, and the bed looks like this (image below) after turning the four knobs using AUX leveling (a helper to locate the nozzle above the knobs), so that the nozzle barely scratches a sheet of paper:

\n
float  v[] = {0.0849, 0.0599, 0.1549, 0.2874,\n0.2674, 0.0624, -0.0425, 0.0699,\n0.3374, 0.1199,  -0.0150, 0.0199,\n0.5399, 0.3349, 0.1899, 0.2074};\n
\n

\"enter

\n

I also notice that the bed is slightly concave in its center: using a straight edge (a steel ruler on its side), there is enough room in middle ninth (center square) under the ruler for one sheet of paper.

\n", "Title": "What is the next step after measuring the bed levels?", "Tags": "|bed-leveling|creality-cr-10|bltouch|", "Answer": "

Now that you have leveled the bed you are ready to start printing. To enable leveling for printing, you need to add G-code G29 directly after G28 in your start G-code of your slicer.

\n

The array values you reported are just for information or visualisation of the level of the bed. Although you have ABL you always need to provide a bed that is as level as best as you can, the ABL will take care of the final dents or skewness.

\n" }, { "Id": "14048", "CreationDate": "2020-07-08T17:54:14.560", "Body": "

I have a Felix Tec 4 (Single Extruder) and at the moment I print for a relatively long time compared to my other prints. Recently I noticed that after a few hours of continuous printing the displayed temperature jumped by up to 10\u00a0\u00b0C, which is almost certainly not true. Since it is the temperature that jumps, it will probably be the thermistor that is broken, but what is broken that this only happens after several hours and not immediately?

\n

I think it does not have to do something with the G-code, because the temperature makes jumps of up to 10\u00a0\u00b0C in one second and back again. I had 2 pretty big prints to do. It happend on both of them, but I forgot to take screenshots on the first print which was much worse that the second one. Here is how it looked in OctoPrint on second print:

\n

\"Octoprint

\n

For comparison I have entered the temparatures manually in OctoPrint and made a screenshot of the temparature\n\"Octoprint

\n

I searched the GCode for M104, M109, M140 and M190, but only found M190 one time at the beginning, which I think is normal.

\n", "Title": "Displayed temperature jumping while printing for a long time", "Tags": "|hotend|heat-management|thermistor|", "Answer": "

Your temperature is not just bumping up 10\u00a0\u00b0C. Your hotend is fluctuating in temperature, it drops under and increases over the "set" temperature. The hotend temperature is a result of the amount heat you put into it and how much heat you pull from it (e.g. filament heats up and draws energy from the heater block), apart from a too large printing speed for the heater to follow the energy off-take, this can be the result of one of the following (or combined) issues:

\n\n

Many hotend designs come with silicone "socks" insulating the heater block and to shield airflow from cooling the heater block.

\n" }, { "Id": "14052", "CreationDate": "2020-07-09T18:07:38.317", "Body": "

I was looking at some end high 3D printers (drooling just a little bit) when I found this.\"enter

\n

Now, this was a 3500 \\$ printer when new back in 2016. I'm relatively to the 3D printing (well... kind of) and have not seen this type of design before. What is it? Cartesian? CoreXY? This style, from my limited knowledge, looks like a best of both worlds kind of scenario because if you look at it - it has that traditional simple Cartesian design, but with a light head of a CoreXY type printer (Exclude the circuitry behind the stepper, that is something else). This was designed to be used in a mechanical application, so it has to be precise. Is this a good configuration and does Marlin support it?

\n", "Title": "What type of kinamatics is 3D Gence One using?", "Tags": "|diy-3d-printer|linear-motion|", "Answer": "

I'm can't say for sure, since this is my first time seeing this type of 3D printer configuration too, but I think this is a cartesian style printer. If you think about it, it is similar to a Prusa-style printer, but instead of an x-gantry being raised and lowered by a leadscrew, the heated bed is being lowered and raised with two leadscrews. Software-wise, the printer you mentioned should be configured exactly like a Prusa-style printer, but with a reversed Z-axis, since to make the heated bed move closer to the hotend, it must be raised instead of lowered.

\n

I personally do not think this printer is a CoreXY configuration since a CoreXY printer relies on two motors that work simultaneously to move the X and Y axis, while the printer you showed has the X and Y-axis moving independently of each other (as can be seen by the belt under the heated bed and the belt that moves the hotend left and right). At least, in my opinion, the way the X and Y-axis operate on a CoreXY is what makes the CoreXY unique, instead of the raising and lowering of the heated bed.

\n

You can read about the variations of Cartesian 3D printers here. If you are interested in other styles of 3D printers, you can check the Delta and Polar style printer section of this article.

\n

Hope I piqued your interest in 3D printing :D

\n

Feel free to comment on this if you have more things to ask, and I'll get back to you as soon as I can :)

\n" }, { "Id": "14059", "CreationDate": "2020-07-11T08:37:06.757", "Body": "

I recently got an Ender 3 V2 and when I go to Info -> Version it says V1.0.0. However online, there is V1.0.1 available.

\n

How can I update the Ender 3 V2. It is different as it isn't a ATMEL chip like the ATMEGA 2560 but rather an ARM processor. I own many arduino boards but apparently the chip has the bootloader already on it.

\n

So how can I update the board, through some uploader, specific software or what exactly.

\n", "Title": "Updating an Ender 3 V2", "Tags": "|creality-ender-3|firmware|", "Answer": "

Just copy the firmware to a blank microsd card.\nTurn off the printer and turn it on.\nBoard will flash it automatically and wait the screen to up.

\n

Make sure the name of the firmware file always unique or different from the last one, otherwise you will get the blank screen.\nIf you do, just re-flash by using a rename (different) file of firmware.

\n" }, { "Id": "14073", "CreationDate": "2020-07-13T17:14:53.380", "Body": "

I have a Core XY Custom DIY Printer, but actually I have a problem with the measure in Axis X and Y. When a print a test cube with the measure 20 mm x 20 mm 20 mm, the object printed result with these dimensions.

\n

Z = 20 mm\nX = 15 mm\nY = 15 mm

\n

Well, I know how solved this problem, because of long time ago have the same problem, do a question, and that was answered, and this moment solved my problem. answer old question

\n

Now I use the same process

\n
    \n
  1. Connect the printer to my pc

    \n
  2. \n
  3. Use Simplify 3D for send gcode to the machine

    \n
  4. \n
  5. M503 for get the set values

    \n
  6. \n
  7. M92 for change the values X and Y

    \n
  8. \n
  9. M500 for store the new values for default in the memory of the printer

    \n
  10. \n
  11. M503 for check the actual values, and these are the correct

    \n
  12. \n
\n

So, I print the test cube, the measure are right, but with two problems

\n
    \n
  1. Every time I.m will go to print, if turn off the machine, the process to be need repeat, in other words, the gcode M500 doesn't work.
  2. \n
  3. the nozzle doesn't start the extrusion in the middle when the parameters were changed.
  4. \n
\n

Send M503 for known the parameters. These parameters are the set by default At this moment in the printer.

\n
G21    ; (mm)\nM149 C ; Units in Celsius\nFilament settings: Disabled\nM200 D1.75\nM200 D0\nSteps per unit:\nM92 X80.00 Y80.00 Z400.00 E100.00\nMaximum feedrates (units/s):\nM203 X100.00 Y100.00 Z2.00 E10.00\nMaximum Acceleration (units/s2):\nM201 X2000 Y2000 Z50 E5000\nAcceleration (units/s2): P<print_accel> R<retract_accel> T<travel_accel>\nM204 P2000.00 R2000.00 T2000.00\nAdvanced: Q<min_segment_time_us> S<min_feedrate> T<min_travel_feedrate> X<max_x_jerk> Y<max_y_jerk> Z<max_z_jerk> E<max_e_jerk>\nM205 Q20000 S0.00 T0.00 X5.00 Y5.00 Z0.40 E5.00\nHome offset:\nM206 X0.00 Y0.00 Z0.00\nMaterial heatup parameters:\nM145 S0 H180 B70 F0\nM145 S1 H240 B110 F0\nPID settings:\nM301 P13.70 I0.71 D65.64\n
\n

After sending code M92 X106.64 Y106.64

\n
G21    ; (mm)\nM149 C ; Units in Celsius\nFilament settings: Disabled\nM200 D1.75\nM200 D0\nSteps per unit:\nM92 X106.64 Y106.64 Z400.00 E100.00\nMaximum feedrates (units/s):\nM203 X100.00 Y100.00 Z2.00 E10.00\nMaximum Acceleration (units/s2):\nM201 X2000 Y2000 Z50 E5000\nAcceleration (units/s2): P<print_accel> R<retract_accel> T<travel_accel>\nM204 P2000.00 R2000.00 T2000.00\nAdvanced: Q<min_segment_time_us> S<min_feedrate> T<min_travel_feedrate> X<max_x_jerk> Y<max_y_jerk> Z<max_z_jerk> E<max_e_jerk>\nM205 Q20000 S0.00 T0.00 X5.00 Y5.00 Z0.40 E5.00\nHome offset:\nM206 X0.00 Y0.00 Z0.00\nMaterial heatup parameters:\nM145 S0 H180 B70 F0\nM145 S1 H240 B110 F0\nPID settings:\nM301 P13.70 I0.71 D65.64\n
\n

\"surface\"

\n\n", "Title": "How to start a print in the center position after a change in parameters using M92 and M500 G-code", "Tags": "|diy-3d-printer|g-code|surface|corexy|", "Answer": "
\n

Every time I.m will go to print, if turn off the machine, the process to be need repeat, in other words, the gcode M500 doesn't work.

\n
\n

This tells me that your firmware has the EEPROM support needed for the M500 command disabled.

\n

Fixing the firmware

\n

You need to update your firmware to enable storing the information in the EEPROM: the line should read as follows without any leading //

\n
#define EEPROM_SETTINGS // Enable for M500 and M501 commands\n
\n

When you are already updating your firmware, you should also fix your firmware to have the correct steps/mm, as those are off in your build. An example for the line you look for is

\n
#define DEFAULT_AXIS_STEPS_PER_UNIT   { 106.64, 106.64, 4000, 500 }\n
\n

Workaround

\n

automatic setting of the Steps/mm fix

\n

There's a way to fix it on the user side though: When slicing, your Start-G-code needs to include M92\u00a0X106.64\u00a0Y106.64, best before homing. That way you make sure that your printer sets the correct Steps/mm whenever you load a printjob.

\n

Home position

\n

via offset

\n

If the home offset is wrong, you can fix it as the next line before G28 - if the home position is for example 10 mm of the trigger in both X and Y, you'd add M206 X100.00 Y100.00 Z0.00 - If your printer has "home" properly defined in the center of the build plate and your slicer is set up to respect that, this should do it. You need to

\n

via move and 0-ing

\n

Another way to get the printhead to the center is to use first G28, then insert a movement to the center of the bed (G0 X100 Y100 for a bed 200\u00a0mm across) and then order G92 X0 Y0, defining that point as the origin.

\n

necessary next line!

\n

In either case, the setup of Marlin also prevents you to move to the "negative" area due to the software endstops by default. So you need to add M121 after it to be allowed to go into the negative space.

\n

Start G-code in bulk

\n
M92 X106.64 Y106.64\nG28\nG0 X100 Y100 ; move to center\nG92 X0 Y0\nM121\n
\n" }, { "Id": "14076", "CreationDate": "2020-07-14T03:03:48.837", "Body": "

I need to set the HOME_POS manually since it isn't printing in the center of the bed. But whenever I try to upload it to the Melzi 2.0 board I get an out of sync error.

\n

I am using Arduino IDE on Manjaro Linux, I was able to upload Marlin to the board on Windows 10 but I have since removed that OS.\nI have the correct settings for the board: Sanguino Atmega 1280 or 1280P (16mhz)

\n

Here is what I have tried to get me this far:

\n\n

Here is the (verbose) error message

\n
Arduino: 1.8.12 (Linux), Board: "Sanguino, ATmega1284 or ATmega1284P (16 MHz)"\n\nSketch uses 107854 bytes (82%) of program storage space. Maximum is 130048 bytes.\nGlobal variables use 4054 bytes (24%) of dynamic memory, leaving 12330 bytes for local variables. Maximum is 16384 bytes.\n/root/.arduino15/packages/arduino/tools/avrdude/6.3.0-arduino17/bin/avrdude -C/root/.arduino15/packages/arduino/tools/avrdude/6.3.0-arduino17/etc/avrdude.conf -v -patmega1284p -carduino -P/dev/ttyUSB0 -b115200 -D -Uflash:w:/tmp/arduino_build_963029/Marlin.ino.hex:i \n\navrdude: Version 6.3-20190619\n         Copyright (c) 2000-2005 Brian Dean, http://www.bdmicro.com/\n         Copyright (c) 2007-2014 Joerg Wunsch\n\n         System wide configuration file is "/root/.arduino15/packages/arduino/tools/avrdude/6.3.0-arduino17/etc/avrdude.conf"\n         User configuration file is "/root/.avrduderc"\n         User configuration file does not exist or is not a regular file, skipping\n\n         Using Port                    : /dev/ttyUSB0\n         Using Programmer              : arduino\n         Overriding Baud Rate          : 115200\navrdude: stk500_getsync() attempt 1 of 10: not in sync: resp=0x73\navrdude: stk500_getsync() attempt 2 of 10: not in sync: resp=0x74\navrdude: stk500_getsync() attempt 3 of 10: not in sync: resp=0x61\navrdude: stk500_getsync() attempt 4 of 10: not in sync: resp=0x72\navrdude: stk500_getsync() attempt 5 of 10: not in sync: resp=0x74\navrdude: stk500_getsync() attempt 6 of 10: not in sync: resp=0x0a\navrdude: stk500_getsync() attempt 7 of 10: not in sync: resp=0x65\navrdude: stk500_getsync() attempt 8 of 10: not in sync: resp=0x63\navrdude: stk500_getsync() attempt 9 of 10: not in sync: resp=0x68\navrdude: stk500_getsync() attempt 10 of 10: not in sync: resp=0x6f\n\navrdude done.  Thank you.\n\nAn error occurred while uploading the sketch\n
\n", "Title": "Unable to upload Marlin 2.0 to Melzi 2.0 board", "Tags": "|marlin|firmware|arduino|linux|melzi|", "Answer": "

Okay thanks to @towe for helping me I figured it out. my Baud Rate was set to 115200 but my board is using the old bootloader so it needed to be 57600. I Changed my boards.txt file to that but the verbose console printed:\nOverriding Baud Rate : 115200 turns out you need to change it in the serial monitor as well (the little magnifying glass in the top corner of the IDE)\nafter that everything worked great thanks for all the help!

\n" }, { "Id": "14082", "CreationDate": "2020-07-15T03:00:07.590", "Body": "

I'm using latest stable Marlin 1.1.9.1 with BLTouch. The BLTouch seems a bit damaged so I want to increase its accuracy with multiple measurements.

\n

I've increased #define MULTIPLE_PROBING to 3 but it seems it applies only to mesh ABL and doesn't work with single center homing.\nI want to get something like fastprobe + average for two or three slow probes.

\n

Am I missing something? Or it is not possible?

\n", "Title": "Increase Z-homing probe number", "Tags": "|marlin|bltouch|z-probe|homing|", "Answer": "

Homing and mesh probing are 2 separate actions, by setting #define MULTIPLE_PROBING to 3, you inherently create 3 slow probing points for the mesh probing (not the homing probing), with 2 you get a fast/slow probing action.

\n

Everything is possible, but, that means that you need to dig into the sources; not changing the configuration constants. That is not for the faint hearted.

\n

I've seen the responses of fast and slow probing in action, usually the difference is not existent or very very small, like one or two hundredths of a millimeter, I seriously wonder if it makes sense to use more than 2 probing points.

\n

Please note that a damaged bed may benefit more from increasing the probing array amount (from 3 x 3 to 4 x 4 / 5 x 5). The number of probing points discussed earlier only dictates the amount of probing actions in a single point; if the sensor hits the bed the probe measurement is done, if it hits again, the value should more or less be the same (give or take the accuracy of the sensor), I do not see what damage to a bed would imply more probing measurements in a single spot. The only rationale would be if the sensor itself has a large inaccuracy with an error that is not systematic, but random. My BLTouch (original) is pretty accurate and repeatable, I've had more issues with all those cheap knock-offs.

\n" }, { "Id": "14087", "CreationDate": "2020-07-15T18:48:28.697", "Body": "

I was wondering how WiFi based 3D Printing works since I want to start an open source project and need some information on this. I have done some research, I have now an understanding on how the printer interprets the G-code and how it acts upon the instructions. But what I couldn't find information on is how WiFi based 3D printing works

\n
    \n
  1. What is the communication between the slicer (such as Cura) and the board?

    \n
  2. \n
  3. How are the commands sent to the printer from the slicer, line by line or the entire G-code?

    \n
  4. \n
  5. How do I establish a connection between the slicer and the board?

    \n
  6. \n
\n

I am well aware of the already existent Raspberry Pi idea compatible with Octoprint but I am making a much cheaper one

\n

Thanks for any help

\n", "Title": "How does wireless 3D printing work?", "Tags": "|ultimaker-cura|g-code|wi-fi|", "Answer": "
    \n
  1. Slicers don't talk to the printer. Slicers analyze an STL file and generate a GCODE file, based on your parameters. A print manager sends the commands from the GCODE file to the printer board, which executes them sequentially.

    \n
  2. \n
  3. They are not. Commands are sent to the printer from a print manager of some sort. This varies among printers; some printers can be managed by more than one manager, and some managers can handle many models of printer. Some printer-manager pairs are proprietary and exclusive.

    \n
  4. \n
  5. You do not. See above.

    \n
  6. \n
\n

Note, some programs do have an integrated slicer and print manager. - Thanks, Trish

\n" }, { "Id": "14091", "CreationDate": "2020-07-16T10:36:53.157", "Body": "

I have seen some videos like this one, that uses 3D printing to make a piece that will be used to make a silicone mold that will then be used to make the final object made of some resin.

\n

Isn't possible to make the mold directly from the 3D printing process?

\n

I want to make an object (that's actually another mold to make tofu) using a flexible mold. Can I use 3D SLA printing with a flexible material such as this one and then use it to make my final mold made of some liquid resin? Also, do you have any suggestions about food grand liquid resin or similar?

\n", "Title": "Use SLA 3D printing to make a flexible mold", "Tags": "|sla|resin|molds|flexible|", "Answer": "

Background

\n

SLA relies on the properties of UV-curing Resin. Most currently available UV Curing Resins harden to a solid, hard polymer, but that doesn't mean there are no other UV curing resins that are elastic. Most however will not be suitable for SLA or DLP systems!

\n

Polyurethanes, which can be flexible if cured in the right way, have not come onto the market as suitable SLA/DLP printing material and up to now, I have only found one UV hardening resin on the market - the one offered by FormLabs and identified by OP

\n

Patent History

\n

Indeed, elastic resins are actually quite new. The first patent I could find for an elastic resin was 2003 for an elastic epoxy resin, while subsequent elastic resins were brought to Patent are different like the 2007 elastic olefin resin. In fact, there is a 2013 US patent on SLA Resin, which is still in effect. As a result of such patents being still in effect or just out of protection times, the market is still very much limited because most manufacturers simply lack the knowledge of how to do it or the licenses to be allowed to do it. In fact, Formlabs is named on 43 patents for 3D printers and accessories, of which only 1 is expired as of July 2020. I could not identify the patent that might be in use for their flexible resin, but the marketing material is of May 2020, so it is relatively new on the market and might not have been updated into either the patent database or FormLabs did buy an exclusive license for the material from someone else, meaning they will not show up as Assignee in it. Or it is kept a trade secret.

\n

Material implications

\n

The printed mold will probably be of a different stiffness than a cast mold as you work with a totally different material and your new molds might degrade at a different speed than urethane or latex cast molds. To get a feel for this this, you will need to run some experiments. As FormLabs hands out test specimens of their print materials, you might order the two flexible ones and then test their stiffness and suitability for your uses by having them interact with your casting resin and seeing if they break down and if you can remove them easily.

\n

Food grade

\n

Generally don't consider anything that comes directly from a 3D printer certifiable food safe, as you need to have both a process and a material that is food rated. There are ways to use the resulting parts to manufacture food safe objects, but that's elaborated for example in some of the questions I suggest to look into

\n" }, { "Id": "14100", "CreationDate": "2020-07-18T11:45:41.173", "Body": "

I get the error "Thermal runaway on E1. Reset printer." today on my Tevo Tarantula when powering on the printer. The display shows the error and the speaker beeps loudly everytime I power on the printer or reset it.

\n

I use the firmware\nhttps://github.com/JimBrown/MarlinTarantula, because I can switch E0 heat nozzle to E1 wires, without soldering; PID's for nozzle were configured, and works fine before I replaced the power supply.

\n

Recently my power supply was broken, and today I installed new power supply, clean nozzle, several times start bed leveling, and on fourth time of homing error appeared.

\n

Power supply voltage tested with a multimeter and the voltage of power supply is 12.06V.

\n

Note: several times printer started bed leveling.

\n", "Title": "Thermal runaway when power on", "Tags": "|hotend|tevo-tarantula|", "Answer": "

Issue is solved!

\n

Root cause was a broken thermistor circuit at board and broken thermistor.

\n

I disconnected all wires and power, measured board and thermistors resistance: E0 = ~120\u00a0\u03a9, (E1 and Bed) = ~700\u00a0\u03a9. I swapped pins for TEMP_0_PIN and TEMP_1_PIN within Marlin for Tevo firmware at pins_RAMPS.h as described in answer on question Change hotend thermistor input in Marlin, changed thermistor and everything prints fine.

\n" }, { "Id": "14105", "CreationDate": "2020-07-19T12:26:27.243", "Body": "

I was wondering how slicers work, how their code runs to generate a G-Code file. How is an STL file interpreted through a slicer.

\n

How does the math work for generating supports, build plate adhesion?

\n

I couldn't find any information, so can anyone help with this technical question

\n

Ryan

\n", "Title": "How do slicers convert an STL file to G-Code?", "Tags": "|slicing|", "Answer": "

Slicing in General

\n

An STL is a set of triangle surfaces. A Watertight STL - for slicing purposes - has surfaces that always create closed outlines if cut parallel to the XY plane.

\n

A Slicer does exactly that: it creates plane-cuts at the indicated Z-heights, takes the plane-cut's outline(s), and decides a direction and order in which to follow the generated path. Then it uses this outline to generate the infill pattern, for example, as explained here.

\n

The more paths there are and the smaller the triangles that are cut up, the more complex the solution process becomes and the longer it takes.

\n

Support calculation

\n

A slicer usually identifies areas that need support by calculating at which angle an STL Surface cuts a given plane cut. Under standard settings, this would be about less than 60\u00b0 to the XY-plane with the normal of the surface having a negative Z-component - which means that a needle poking out of that surface points towards the bed.

\n

The most simple form of support generation simply generates a grid pattern between such areas and the bed or next surface below. Tree support on the other tries to generate a support structure that bends around the object without intersecting and only relying on the support of itself.

\n

Build Plate adhesion

\n

A skirt and brim are just taking the outline of the build-plate intersection and surround that with outlines.

\n

A Raft is generated like the simple support case, but taking the whole base of the object, adding a little edge around it and then generating the support grid there.

\n" }, { "Id": "14111", "CreationDate": "2020-07-21T06:32:01.590", "Body": "

TL;DR: Upper layers of a stringing pyramid test are missing retractions in the G-code.

\n

Edit 1: added printer info and additional settings.
\nEdit 2: Clearing up confusing wording.

\n

Update:
\nI wrote a program to parse the G-code and count the retraction-extension pairs between layers. The first 17 layers (base) and 18-95 (pyramid start) have the appropriate number of extruder calls. I expect an extension at the start of the layer, retraction before moving to the second tower, extension to print the second tower, and a retraction before moving back to the first tower.

\n

After layer 95 of 391 the number of retraction pairs starts to decrease from missing a pair every other layer to eventually almost no retraction pairs. The last 70-ish layers have a retraction pair every dozen or so layers.

\n

Perhaps this is related somehow to the shrinking width of the pyramid since the missing retraction-extension pairs increase as the pyramid comes to a point

\n

Disabling Z-Hop shows no difference in this behavior

\n

Original question:
\nAfter switching to a 0.2\u00a0mm nozzle I starting printing some calibration tests to find my stringing pyramids end up looking like a pair of trees. I suspect I've always had this issue, but it's much more evident now with a smaller nozzle that likes to ooze.

\n

This is on an Ender 3, so using a Bowden tube, with a MicroSwiss all metal hot end and the extruder mechanism is aluminum with a pair of toothed drive gears. I'm seeing the problem in 1.75mm PLA of multiple well regarded manufacturers.

\n

Relevant Settings (let me know of other settings I should list):

\n\n

Looking at the G-code, sometimes I see a retraction paired with the Z-hop, but sometimes it's missing. It seems pretty random for when it's missing, but the first quarter or so of the layers do not show the problem. Originally I thought it had something to do with extruder drift over time, or something crazy, until I looked at the G-code and saw it was missing commands.

\n

Snippit of retracting for Z-hop:

\n
G1 F1200 X204.632 Y161.781 E77.9224\nG1 F3000 E74.4224\nG1 F300 Z7.08\n;MESH:NONMESH\nG0 F9000 X190.179 Y161.901 Z7.08\n;TIME_ELAPSED:2354.891407\n;LAYER:171\n;MESH:string_test_fast_pyramid.stl\nG0 X190.179 Y161.901 Z7.12\n;TYPE:WALL-INNER\nG1 F300 Z6.92\nG1 F3000 E77.9224\nG1 F600 X190.179 Y161.555 E77.92356\n
\n

Snippit of not retracting for z-hop:

\n
G1 F1200 X204.473 Y161.666 E78.0239\nG1 F300 Z7.12\n;MESH:NONMESH\nG0 F9000 X190.182 Y161.898 Z7.12\n;TIME_ELAPSED:2359.620803\n;LAYER:172\n;MESH:string_test_fast_pyramid.stl\nG0 X190.182 Y161.898 Z7.16\n;TYPE:WALL-INNER\nG1 F300 Z6.96\nG1 F600 X190.182 Y161.558 E78.02503\n
\n

Test on the left is done with 'Equalize filament rate' enabled, on the right side is done with the option disabled.

\n

\"enter

\n

Any advice you can give as to what settings I am missing or misconfigured would be much appreciated.

\n", "Title": "Ultimaker Cura is not always retracting in exported G-Code", "Tags": "|ultimaker-cura|slicing|retraction|", "Answer": "

It's likely that you're hitting Cura's Maximum Retraction Count (retraction_count_max) within the Minimum Extrusion Distance Window (retraction_extrusion_window). This is a misfeature supposedly designed to avoid grinding/flattening the filament, but of course acting on it will ruin your print in exactly the same way you're seeing. I think setting the window to 0 or the maximum count to some ridiculously high number like 1000000 will fix the problem.

\n

Note that it's exacerbated by the thin nozzle making the number of retractions needed take place over a much smaller extruder-axis move distance than what you'd have with a normal 0.4 mm nozzle. This is probably why nobody has noticed and complained about the badness of Cura's defaults here.

\n" }, { "Id": "14113", "CreationDate": "2020-07-21T17:32:00.113", "Body": "

I hope this kind of questions are tolerated here. Otherwise, please tell me in the comment and I will delete the question - no needs to downvote.

\n
\n

For a new job I need to create a lot of objects like this one:

\n

\"enter

\n

To have an idea of the dimensions:

\n\n

I'm able to make such an item with either a 3D printer or even a standard 4-axis CNC.\nBut because I need to make 100+ of these items I wonder if there are something similar out there.

\n

But I have no idea of how it might be called. I tried with "spacer with slots" without any useful results.

\n", "Title": "Ready to use objects instead of 3D-printing them", "Tags": "|3d-design|cnc|", "Answer": "

If your printer is reliable enough I would suggest printing multiple parts in one go. Since the cylinders are only 12 mm in diameter you can easily fit over a hundred of them on a standard 20 x 20 cm built plate with a couple of millimeters spacing in between.

\n" }, { "Id": "14119", "CreationDate": "2020-07-22T00:34:25.727", "Body": "

I've had this MP Select Mini III for years and years. Lately it's consistently jamming/clogging somewhere between the hot end and the PTFE tubing - but not right away, often when it starts getting to the infill layers.

\n

If I don't catch it in time, the filament twists and twists until even the Bowden tube is clogged (and then it's a whole lot of trouble to clean out).

\n

My temperature has been wobbly of late, so I bumped the temps higher than usual (200-210\u00a0\u00b0C for PLA) thinking perhaps it was dipping low for a spell and allowing it to cool.

\n

Is this the end of the road for this cheap little guy that has been a solid workhorse or are there some known fixes that I should consider?

\n", "Title": "Filament consistently jamming / clogging in Bowden tube on MP mini 3", "Tags": "|extrusion|bowden|monoprice-select-mini|", "Answer": "

If you have it for years, have you ever replaced the Bowden tubes and hotend liner? This printer might be in need of some dire maintenance.

\n

Also check if the Bowden tube is securely fixed, if infill starts, it might be retraction that is giving problems by lifting the tube alongside the rectracted filament.

\n" }, { "Id": "14130", "CreationDate": "2020-07-23T12:23:04.443", "Body": "

I'm printing 6 separate parts in one go, after 4 hours of printing one part failed, but the other 5 are printing nicely.

\n

Is there a way to prevent the print from printing the failed part and continue printing the other 5 parts.

\n

I'm using Cura and an Ender 3 printer.

\n", "Title": "Is there a way to save a multi part print if one fails?", "Tags": "|ultimaker-cura|creality-ender-3|pla|", "Answer": "

In addition to the previously answered "Exclude Region" plugin for OctoPrint, the "Cancel Object" plugin for OctoPrint also works excellently.

\n
\n

Cancel single objects during a print based on code comment lines

\n

This plugin allows the user to cancel single objects during a print while allowing the remaining objects to print normally. Instructions for use with compatible slicers are provided on the plugin\u2019s GitHub Homepage.

\n

\"Cancel

\n
\n

Source: OctoPrint-Cancelobject

\n" }, { "Id": "14141", "CreationDate": "2020-07-26T08:51:43.810", "Body": "

In Cura, if you plug your favourite 3D Printer you will most likely get an option to Print via USB. If you have never heard of what I'm talking about, please see this video:

\r\n \r\n

\n

I wanted to know some information on how this works:

\n\n", "Title": "How does 3D printing over USB work?", "Tags": "|g-code|", "Answer": "

This is just a stub answer. I will try to expand on it later.

\n

The 8-bit microcontrollers used on many 3D printers do not have a USB interface, and so a USB to serial interface chip is used to allow a computer to communicate with the serial port (UART) on the microcontroller. In order for the computer to communicate with the printer, a device driver is needed to allow the operating system to communicate with the interface chip.

\n

The appropriate device drivers should be supplied with your printer, and you should install these drivers before you try to do anything else. The drivers will make the printer look as if it has a serial interface, and the highest speed that most 8-bit microcontrollers can manage is 112800bps.

\n

If the software that you are using has a terminal interface, you can send any G-code commands to the printer. For a list of G-code commands, see here. Note that this list includes commands for CNC machines as well as 3D printers.

\n

For normal printing, G-code commands are sent to the printer using a print manager, such as Repetier-Host. Some slicers may have built-in print managers.

\n" }, { "Id": "14159", "CreationDate": "2020-07-31T22:08:51.527", "Body": "

I am new to 3D printing and have a Monoprice MP Select Mini 3D Printer\u00a0V2. I wanted to be able to 3D-print this VR Adapter from Thingiverse. Before downloading the Cura software, I tried opening the model in Print 3D, and as you can see in this picture, it appears that the model is just under 100 mm, which is well under the 120\u202fmm limit of the MP Select Mini\u2019s 120\u202fmm\u00d7120\u202fmm build plate:

\n

\"Print3D

\n

However, when I downloaded Cura and input the specs for the printer (120\u202fmm for x, y, and z dimensions), the model appears to be much larger than the 120\u00d7120\u00d7120 area. The program tells me that the model is 132.5\u202fmm \u00d7 180.6\u202fmm \u00d7 36.6 mm:

\n

\"Cura

\n

I need the model to fit the controller for the Oculus Quest, so I can\u2019t just scale it down. Can anyone tell me why I am seeing this discrepancy and how I can still print this model?

\n", "Title": "Unable to Determine Proper Print Scale of STL model", "Tags": "|3d-models|slicing|", "Answer": "

The STL format does not define unit information. So there is no way of knowing, from a STL file alone, what size it should be.

\n

However there are only a handful of units people design with. So the unit is most likely one of inches, cm or mm. So if you inverse convert between those combinations you'll likely find the real size.

\n

Your image is somewhat inconclusive. But it seems to indicate your object is in fact larger than your bed. As it it is 18-19 pips of height and i would interpret each as 10\u00a0mm from the image. Which would indicate your model is correct.

\n

Split and glue? Print corner to corner you should have 207\u00a0mm length that way (but not necessarily width).

\n" }, { "Id": "14161", "CreationDate": "2020-08-01T14:55:59.670", "Body": "

I had noisy DRV8825 drivers for my extruder E0 and X, Y, Z axes. I upgraded all to LV8729 drivers but they were still noisy, so I bought two TMC2100 drivers for the X and Y axes and two LV8729 for the Z axis and extruder E0. I updated Marlin (reversing endstops logic and choosing driver types) and I removed the jumpers under the driver pins. But my X and Y axis don't move; eventually the printer halts.\nWhat could be wrong?

\n

\"TMC2100

\n", "Title": "X and Y axes don't move after upgrading to TMC2100 drivers", "Tags": "|stepper-driver|y-axis|x-axis|tmc2100|", "Answer": "

Steps/unit also must be modified. In my case these values made the printer run silent and smooth.

\n
#define DEFAULT_AXIS_STEPS_PER_UNIT   { 100, 200, 6160, 884 }\n
\n

I set Max feed rate and Max acceleration as below:

\n
#define DEFAULT_MAX_FEEDRATE          { 300, 300, 5, 25 }\n#define DEFAULT_MAX_ACCELERATION      { 300, 300, 100, 10000 }\n
\n" }, { "Id": "14162", "CreationDate": "2020-08-02T01:11:22.560", "Body": "

I'm new to this game, and recently upgraded the hotend on my Ender 3 Pro to a clone of an E3D V6, as I'm keen to do nylon prints at some point. I noticed however that this one I got has a teflon liner which seems to negate the advantage of a metal hotend entirely.

\n

I'm wondering what temperature it's safe to run this hot end up to?

\n

\"Teflon

\n

\"Teflon

\n", "Title": "Does a teflon insert in an E3D V6 clone limit it's useable temperature so it can't print nylon?", "Tags": "|creality-ender-3|hotend|e3d-v6|nylon|", "Answer": "

Long story short, you can print Nylon with a Teflon tube. I've done it. P.S. The nylon absorbs water like nothing you've ever seen. Even after the part is printed, it absorbs water, and expands!

\n" }, { "Id": "14182", "CreationDate": "2020-08-04T17:00:34.413", "Body": "

I'm pretty new to 3D printing. I am using Repetier Host 2.6 with Slic3r (printer Geeetech i3 Pro B).

\n

When I slice detailed models I get issues like this as the slice result:

\n

\"Badly

\n

This is the model in the example, Army Men flames of war single version.

\n

I can slice simple objects fine, walls, cubes 2D logos etc.

\n

Why is this doing this?

\n

When I used Cura the slice is fine.

\n", "Title": "Sliced object losing all detail in Cura", "Tags": "|slic3r|repetier|", "Answer": "

This is not "loosing details". What is is is this:

\n

You can not print on air. So, certain things (like the backpack) require SUPPORTS that are removed after printing. Another example is the gun.

\n

Your support setting force the slicer to set up quite a lot of supports and that is what you see - start removing them.

\n

There ALSO is a problem with possibly you using too large a nozzle. Depending on that one you may loose details, but there is not even the basic form visible on your picuture because all I see are supports.

\n

https://all3dp.com/1/3d-printing-support-structures/#:~:text=3D%20printing%20support%20structures%20are,added%20cost%20to%20the%20model. exokauns supports.

\n" }, { "Id": "14184", "CreationDate": "2020-08-05T04:06:47.540", "Body": "

I have an Ender 3 3d printer. It has a bed that wobbles because it came with only 1 bed support beam.

\n

\"Ender

\n

Every model I print, I must print vertically, because the closer it gets to the edges of the build plate, the less adhesion it has.

\n

Is there anything I can to to fix this problem?

\n", "Title": "Bed wobbling on Ender 3", "Tags": "|creality-ender-3|bed-leveling|bed|", "Answer": "

If the V roller wheels aren't tight on the Y axis beam, it means the eccentric nuts are not adjusted correctly. Two of the rollers are mounted centered on the holes in the carriage frame, but the other two are on eccentric nuts which displace them from center slightly depending on the orientation the nut is turned to, to allow tightening and loosening of the grip on the beam. Since the Y axis ones are hard to see under the bed, look at the X or Z ones to get an idea what to expect.

\n

Note that the bolt through the whole roller assembly can loosen when adjusting the eccentric nut. You can probably avoid this by figuring out the right direction to turn it and only going that direction (continuing around just under 360 degrees if you go too far). If you do loosen the bolt then the eccentric nut will move on its own under vibration, so you need to re-tighten it. For the Y axis this might require taking off the bed or taking the carriage off the beam (by removing the belt and tensioner).

\n" }, { "Id": "14186", "CreationDate": "2020-08-05T10:35:49.190", "Body": "

I am going to design and build a 3D printer. I want the highest quality and accuracy so nothing except that is important for me. Which cartesian design has the highest quality and accuracy? CoreXY, Prusa, or Gantry (Ultimaker)?

\n

Also, is it better to have a nozzle that moves in just direction "X", directions "X and Y", or "X, Y and Z"?

\n", "Title": "Which kinematic system has the highest quality?", "Tags": "|printer-building|corexy|", "Answer": "

Just to clarify: Examples of kinematic systems would be Cartesian (which includes CoreXY), Delta, Scara, and Six-Axis. The quality of the system has less to do with the system and more to do with the implementation.

\n

Furthermore, there are 2 main types of desktop/benchtop 3d printers that are commonly available: Fused Deposition Modelling (or fused filament fabrication depending on who you ask) and stereolithography; of which the latter, stereolithography, has better accuracy and quality.

\n

In terms of FDM however, it can be easily argued that CoreXY cartesian printers offer the best quality and accuracy (both of which are subjective btw) than either delta or gantry designs (e.g. gantry would be the Prusa i3). The reason is that in order to get the CoreXY to work at all, the overall engineering and frame rigidity must be at a certain minimum. Once this minimum has been achieved, the quality of the prints typically meets or exceeds the quality/accuracy of even a well-tuned gantry printer; and you are going to see it in the cost of a CoreXY printer.

\n" }, { "Id": "14199", "CreationDate": "2020-08-06T13:21:59.963", "Body": "

I am printing small mechanical pieces in ABS:

\n\n

The first layer is printed correctly, but later, corners warp and first 10\u00a0mm get deformed (See images).

\n

\"enter

\n

\"enter

\n

How do I solve this? Unfortunately, I cannot increase room temperature over 70\u00a0\u00baC

\n

Here is a picture while printing, we can see that the edges get warped even far over the first layer. (Sorry, the picture quality is not so good):

\n

\"enter

\n", "Title": "How to solve ABS deformation at the bottom?", "Tags": "|abs|warping|", "Answer": "

We usually stick the masking tape on the printer's bottom plate, so that the model is formed on the side of the masking tape which without glue, which looks good.\nWhen there is no masking tape, we sometimes manually adjust the height of the bottom plate to make the nozzle appress to the bottom plate when printing the first layer. At this time, the melted material can hardly flow out. The strong pressure will adhere the material tightly to the bottom plate, which can effectively solve this problem. However, this may damage the extruder motor.

\n" }, { "Id": "14200", "CreationDate": "2020-08-06T15:06:06.747", "Body": "

I need to make chocolate busts.

\n

Is it better to make silicon reverse of the bust and cast the bust or directly print it using chocolate printer (I prefer cost and process speed over quality)? I don't have chocolate printer so if this method will be better I will need to use some 3D printing service.

\n

I need to make quite a large amount of the busts.

\n", "Title": "Chocolate printing vs casting?", "Tags": "|chocolate|", "Answer": "

For your application, you should 3d print a positive model, then make a silicone mould negative of that model. Then use the silicone mould to make the final chocolate. Clean the silicone thoroughly before initial use. Fill gaps in the 3d printed model with a light epoxy resin. Use mould release and ensure you wash and sterilize that silicon mould before using it with chocolate.

\n

An actual chocolate printer is not easy to come by, as a normal 3d printer converted to use chocolate is not able to be sterilized for food use.\nThe video in the link illustrates one person's attempt at this process.\n

\r\n \r\n

\n" }, { "Id": "14206", "CreationDate": "2020-08-07T08:15:38.650", "Body": "

I'm trying to model the threads of a "Poland Spring" 500\u00a0ml bottle so I can 3D print an adapter for it. But I can't find information about it. I emailed them but they said they didn't have the information.

\n

How can I find this information out?

\n

The bottle seems to use non standard threads. It uses 3 threads 120 degrees apart that does not go all the way around. Any information on how to get this information?

\n

\"bottle\"

\n", "Title": "Converting / measuring bottle threads for creating an adapter to it", "Tags": "|3d-design|mechanics|", "Answer": "

It's really unlikely that a bottle is using nonstandard threads; the engineering and tooling cost for doing so would not make sense. Most plastic drink bottles use PCO 1881 or PCO 1810 threads. If not, it should be one of a number of other less widely used standards.

\n

The industry term for drink bottle thread is "neck finish". Searching on that, or on one of the standard names like PCO 1881, will find you a lot of information. You can then try to find a match for your threads. Note that the breaks 120 degrees apart do not affect the thread design, and may or may not be part of the neck finish standard, so you can ignore them.

\n

There are existing OpenSCAD libraries for some of these, including

\n\n

And some related articles on Hackaday about their development and use:

\n\n" }, { "Id": "14221", "CreationDate": "2020-08-09T21:17:50.250", "Body": "

Does anyone know of a hot end that is sealed? What I meaning is that the hot end has a rubber seal where the filament enters to keep the top airtight (in order to eliminate oozing).

\n

I am looking to build a dual extruder printer but, I do not want any oozing from the hot end which is not in use. I could build a system to retract and 'close' the nozzle but it would be much more elegant if it would work to just seal the top of the hot end. Thus achieving the same effect as when you pull up water with a straw by covering the top with your finger.

\n", "Title": "Is there such thing as a sealed dual extruder hot end?", "Tags": "|extruder|hotend|dual-nozzle|", "Answer": "

In a true dual-extruder / dual hot-end printer, having a seal isn't necessary.

\n

On my dual-extruder printer, the non-active extruder does an aggressive retraction and then drops the hot-end temp down to a stand by temperature which is lower than the melting point of the filament. Because it had an aggressive retraction ... and it's cooling ... the filament doesn't seem to ever make it out before it has cooled to the point where the filament drops below the melting point. While there is no 'seal' ... it doesn't ooze.

\n

The slicer software (I use Cura) estimates the time remaining before the extruder needs to become active and will start to pre-heat the hot-end so that there wont be much (if any) delay. Once the extruder becomes active, it will re-prime and will usually move to a prime tower to get the filament flowing again ... while the other nozzle (the one that just became inactive) does a retraction and starts cooling down.

\n

With this technique of retracting and lowering the inactive hot-end to a standby temperature, I have never personally experienced an issue with oozing.

\n" }, { "Id": "14238", "CreationDate": "2020-08-11T23:45:21.013", "Body": "

The insulation of the heater block got pulled off...

\n

I was trying to print ABS on my TEVO Tornado, overnight. But I not-so-smartly left the window open overnight, thinking it was alright (as to remove the fumes) but then it warped so much that it pulled off the insulation of the hotend.

\n

It is no longer heating up to over 200 degrees Celcius. It starts showing a message saying PRINTER HALTED: PLEASE RESET Should I just re-insulate it? if I should, what do I use for insulation?

\n", "Title": "Is nozzle no longer heating up correctly related to hotend insulation being pulled off?", "Tags": "|hotend|abs|tevo-tornado|", "Answer": "

Yes I had this same problem myself.

\n

You will need to re-insulate it using a silicone block and next time don't leave the window open while printing. Just keep the fan or AC on to circulate the air.

\n" }, { "Id": "14239", "CreationDate": "2020-08-12T01:11:45.453", "Body": "

I am thinking about getting into the 3D printing craze and currently looking at purchasing a Creality Ender 3 V2 as an entry-level unit. I would like to print using the normal PLA, PETG, ABS, etc. but I'm also thinking of metal-infused PLA or similar for printing jewelry. The suggested modification that I have found in researching is the Micro Swiss All Metal Hotend in order to reach the higher temps for printing metals.
\nAm I correct with this additional mod? Are there other Hotends that would better? Any additional mods for working with metals?

\n", "Title": "Additional mods for printing metal filled filament with Ender 3 V2?", "Tags": "|creality-ender-3|metal-printing|filled-pla|", "Answer": "

There is a lot of misinformation in the 3D printing world about "all-metal hotends" being an upgrade. Some of them, especially clones, are not even all-metal but just poor lookalikes that create all sorts of problems. But even if you get a real one, it's a trade-off, not an upgrade. It lets you print materials that need a hotend temperature over 250 \u00b0C (over which the PTFE liner will begin to degrade and possibly release small amounts of harmful gasses) up to the temperature your heating element can achieve, at the cost of losing the extremely-low-friction PTFE pathway all the way to the nozzle, which aids with smooth extrusion and retraction and avoids jams. Depending on your particular all-metal hotend, it may be harder to get retraction working correctly than with a normal PTFE-lined one. Some users report very good results, others lots of problems.

\n

There are very few materials that need temperatures over 250 \u00b0C to print that don't also need a much more expensive machine (or at least a heavily-modified one) for other reasons, such as requirement of a high-temperature enclosure. The main materials that can be printed on an Ender 3 with an all-metal hotend but not the standard PTFE-lined hotend are polycarbonate (needing 255-300 \u00b0C) and some forms of nylon (which may need up to 250-270 \u00b0C). All the other non-exotic materials, including PLA, ABS, PETG, and TPU, and even some exotic ones like POM (aka delrin or acetal) and many nylons, can be printed just fine with a stock Ender 3 (possibly with a minimal enclosure built around it).

\n

In particular, for the purpose you want - printing metal-infused PLA - there is no point whatsoever in an all-metal hotend. Metal-infused PLA is printed within the temperature range of PLA, which is typically 230 \u00b0C at the absolute highest (and preferably much lower) and the printer you're looking at should print it just fine out-of-the-box (or rather, once you assemble it).

\n

If you're concerned about abrasive wear from the filament, this affects the nozzle not the hotend. Technically the nozzle is "part of" the hotend assembly, but it's easily removable and generally considered a consumable item that you replace periodically. You can get hardened steel nozzles and all sorts of other exotic replacements designed not to wear out from abrasive materials, but the standard brass has the best thermal (heat conduction) properties and is so cheap you're generally better off sticking with it and buying a pack of 20 or so to have on hand when they eventually need replacement. In any case, there's no need to replace the hotend with an all-metal one to deal with nozzle wear; you can get either type of replacement to fit the original one.

\n" }, { "Id": "14244", "CreationDate": "2020-08-12T15:55:59.250", "Body": "

I have an MP Select Mini V2 and when I turn on the printer and select the "print" menu it hangs while saying "please wait" instead of listing the .gcode files stored on the SD card.

\n

I have been using that printer with that SD card for hundreds of prints without error. It started when I took it out and added a new .gcode file (sliced in Cura with the same settings as I always use) and placed it back into the printer to print it. When I connect the SD card into my computer (Windows 10) everything seems normal with the card.

\n

What could be causing this issue?

\n", "Title": "\"print\" menu not loading on Monoprice MP Select Mini V2", "Tags": "|monoprice-select-mini|microsd|", "Answer": "

Too many files on SD card

\n

I removed some files and now it works, it seems like the menu would not load if the SD card contained more files than the printer could display on the print menu (they didn't take up a lot of space in memory, though).

\n" }, { "Id": "14248", "CreationDate": "2020-08-13T02:50:15.843", "Body": "

I'm trying to develop a device to burn a glass slide surface with a special pattern of UV light (between 300\u00a0nm and 360\u00a0nm) in a 5\u00a0cm square surface.

\n

I've read several threads asking questions about LCD printers and their way of functioning but I am not entirely sure how they work. So far I have found two methods that explain LCD printers:

\n
    \n
  1. The LCD screen emits light in UV spectrum that causes the resin to cure. I don't believe there are any LCD screens that can emit at 360\u00a0nm.

    \n
  2. \n
  3. Initially there's a UV light bulb 'behind' the LCD screen which is translucent when off. The pattern is drawn in the LCD screen interfering with UV light's path drawing the image in the plate.

    \n
  4. \n
\n

If option two is the right one, do you know any devices (uv light and lcd screen) to develop what I want within the 360\u00a0nm wavelength?

\n

Alternatively, can a DLP projector to emit UV light (by adding a UV light bulb)? Would it still need to be cured?

\n", "Title": "LCD/DLP resin curing system", "Tags": "|diy-3d-printer|sla|resin|dlp|lcd-screen|", "Answer": "

It's almost always what you call system 2:

\n

The LCD screen is acting as a "mask" for the UV backlight, which is a strong bulb under it. This also is the reason that the screen degrades over time and that the machines need replacement bulbs: the heat from the UV source burns out the screen and itself over time. Both parts are consumable, just like the screen.

\n

You could technically swap out the light source for any one that fits the machine. However, a 360\u00a0nm light source won't help you for printing resins at all: most resins you can buy cure between 395 and 410\u00a0nm.

\n

Yes, even by curing with an UV light source, you need to post-cure the print. The reason for that is to get rid of any uncured, not washed away resin that still sticks to the surface and that a 3D print right of the platform actually isn't cured fully - especially the inter-layer-bonds are not formed fully and curing in the lightbox increases the stability to the print a lot.

\n" }, { "Id": "14250", "CreationDate": "2020-08-14T12:41:14.197", "Body": "

I have stuck resin print that won't detach from the bed of my Creality LD-002R LCD Printer. I guess I could break the print off piece by piece, but it won't budge with pressure, pulling with all my strength, or even hitting it pretty hard.

\n", "Title": "How do I remove a stuck resin print", "Tags": "|resin|creality-ld-002r|", "Answer": "

There are generally 3 ways, in order of least to most desirable, and at times you need to combine them in an escalation:

\n\n" }, { "Id": "14251", "CreationDate": "2020-08-14T21:07:15.707", "Body": "

With resin LCD printers (not filament!), what are the considerations to choose between PLA and "water soluble"? Microcenter carries resins from esun in these two types, and their spec sheets indicate that tyhe water-soluble type has higher tensile strength and other mechanical properties. Which leads me to wonder, under what circumstances one selects PLA then?

\n", "Title": "Resin types: Water Soluble vs PLA", "Tags": "|resin|", "Answer": "

First of all, let's look at what the filaments are:

\n

PLA & PVA Filaments

\n

Normal PLA and Water-soluble PVA contain for the most part the material on the tin, its precursors, and possibly some modifiers. These are only suitable for thermoplastic processes like injection molding or FDM/FFF (Filament deposition modeling/Fused Filament Fabrication) printers - the finished polymer can't be made back into a UV-curable resin easily. Both materials are chemically rather well bonded and are not very reactive at all. They are biodegradable and not a lot of toxic waste. PLA needs very strong chemicals such as dichlormethane to go into solution, but PVA is water-soluble.

\n

Resin

\n

Almost no cured resin (as in post-polymerized) is water-soluble, your webshop might have a misnomer as it meant to write the proper name: water-washable, which is meant to reflect the ability to put the unpolymerized monomers into solution in water.

\n

eResin

\n

eSun offers an eResin-PLA-Bio-Photopolymer, which is similar to PLA, but it is not the same material you get for an FDM printer. You see this most easily by checking the density and the MSDS: Real PLA has a density of 1.21\u20131.43\u00a0g/cm\u00b3, eResin (as the bottles are labeled) has a density of merely 1.07-1.13\u00a0g/cm\u00b3. This is a totally different material in the bottle! It is most likely a resin mix that is based on lactic acid monomers and a UV-active acid that can bond the monomers, creating a structure that does contain lactic acid groups and the binder - but most certainly it is not chemically identical to PLA. It contains, according to the MSDS, Polyurethane acrylate as the 'binder', 1,6-Hexanediol diacrylate Monomers, and about 10\u00a0% photo inhibitors and pigments. The main selling point seems to be, that comparable to PLA, it would be made from a renewable source to a larger part. The chemical reaction that leads to the completed resin is a question I have pitched on Chemistry.SE.

\n

It needs to be cleaned with IPA (isopropyl alcohol) or another organic solvent like any standard resin and you are not allowed to rinse the material into the sewer: it is classed as a LONG-TERM AQUATIC HAZARD - Category 4 and its MSDS contains:

\n
\n

Solubility\uff1aSoluble in ethanol, ethyl acetate, benzene and other organic solvents, insoluble in water

\n
\n

water-washable eSun Resin

\n

eSun offers no water-soluble resin at all, they offer a water washable resin, which means that it is supposed to be less toxic than standard resin and safe to rinse effectively without IPA, creating less toxic waste in the process. The MSDS for this is not (yet)) available so I can't evaluate this. However, I have requested the MSDS for the evaluation of safety procedures needed and hope to hear back from them soon. This resin is most certainly not PVA, but it seems to be chemically somewhat similar to their other resins.

\n" }, { "Id": "14257", "CreationDate": "2020-08-15T14:36:13.300", "Body": "

I have an anycubic photon, and occasionally have trouble removing prints from the build plate. While trying to removing prints I've put some scratches into the surface, as I guess I've come in at the wrong angle trying to get under and leverage the print.\nTypical examples I've seen of people removing prints, they appear to come at it hard with a blunt edge. I've come across scrapers that are blunt and some with a sharper edges.

\n

I've put a bit more a ding into my plate today, using a scraper with more of a blade edge for a difficult to remove print. I had used the same scrapper before with great success, as it slide right under and I could leverage the print, but struggled with it today and scratched up my plate.\nI haven't come across much discussion on this so thought I'd ask about scratches to the build plates.

\n

Are scratches to the build plate something to be concerned about, is it something that it expected when removing prints?

\n", "Title": "Resin printer build plate scratches/ damage", "Tags": "|build-plate|resin|build-surface|", "Answer": "

One of the resources I've found is a forum with a discussion of the impact of scratches.

\n

The general consensus is that scratches are not a problem, as long as there are no burrs above the surface of the plate that would damage the bottom of the vat. One posting party has used 36 grit on a palm sander followed by scotchbrite pad to resurface his plate, but one takes the risk of creating an out-of-plane surface.

\n

A better method to sand a plate would be to place the sandpaper on a glass surface or something equally flat and planar and pass the build plate repeatedly over that.

\n

Another aspect of the discussion was that scratches are good, as the rough surface provides a better bond to the resin.

\n

You're likely to note that your build plate is not a polished smooth surface for the same reason.

\n" }, { "Id": "14259", "CreationDate": "2020-08-15T18:28:53.073", "Body": "

first time here.

\n

I recently bought an Anycubic Mega S and I'm venturing in the 3D world. Lots to learn for sure. I have printed a few items so far and all went pretty well.

\n

My question is about something that caught my attention while browsing on Thingiverse. I was looking for an organizer for a board game that I have and every option I found there shows the piece in a vertical position rather than horizontal which seems like the natural position of the piece.

\n

I would like to know why most of this types of pieces are set to be printed in a vertical position rather than horizontal.

\n

Here is the original piece Betrayal at Baldur's Gate Organizer by Jason Patch. As you can see in the image below the piece is naturally horizontal but the actual files to print (blue ones) are all vertical.

\n

\"enter

\n

PS.: I didn't really know which tags to use so I just added 3D, feel free to suggest any other I will happily edit it. :)

\n", "Title": "Vertical vs Horizontal shapes printing", "Tags": "|3d-models|3d-design|print-orientation|", "Answer": "

When I design parts in CAD software I pick a starting plane and go from there. And that starting plane doesn't correspond to how the part is designed to be printed. That usually comes later.

\n" }, { "Id": "14275", "CreationDate": "2020-08-18T15:07:28.153", "Body": "

After a resin print completes, what is the expected process to finish the print? Is some cleaning expected? I also some manufacturers sell UV chambers to cure the print surface, is this required?

\n

I purchased a low-cost printer from China that is quite high-quality hardware, but sadly short on documentation. Insight on the proper post-print process is appreciated.

\n", "Title": "Curing a resin print", "Tags": "|resin|", "Answer": "

You don't really need a UV curing chamber but wash it with 99\u00a0% isopropyl alcohol\nand put it in bright sun, this should do the trick!!

\n" }, { "Id": "14280", "CreationDate": "2020-08-19T09:15:30.170", "Body": "

I'm pretty new in the 3D printing world, and there are some doubts that have come my way and I've not been able to find anywhere.

\n

Usually, I'm going to try to print pieces bigger than the bed of my Elegoo Mars Pro (which is 115 x 65 x 150\u00a0mm, so it's pretty small). Everywhere I try to look at this they are talking about Fused Filament Fabrication (FFF) printers, and I guess that material will make the answer to these questions to be different, that's why I'm asking here. All of these questions are focused on SLA printers.

\n

Is there any kind of technique / slicer software to be able to print those pieces by parts?

\n

I've read about slicers, but there seem to be a bunch of them, and some of them work best with some printers than others, and have different features but I haven't seen any that helps in this matter.

\n

Also, I've seen the typical plane cut, but this doesn't make it easy later on to "fit" pieces. I would need some kind of female - male joint. Is there any (free) slicer that helps in that, and makes those joints to be accurate and solid?

\n

Thank you!

\n
\n

I removed the "shrink size" part in the question and published it as a separate question. Also moved to a different question "gluing separate pieces" part.*

\n", "Title": "Printing and slicing big pieces with Elegoo Mars (Pro)", "Tags": "|3d-models|slicing|sla|elegoo-mars|", "Answer": "

I actually found it easier to export my model as a single STL file, and then to open up 3D builder. It has a tool that let's you create a flat plane, and then slice a model into pieces using it.

\n

So long as you have a large enough surface area to glue, it shouldn't be a problem.

\n

I tried using booleans and the peg\\hole method in Blender, but it simply wasn't stable enough with complex models.

\n

Where I have used pegs and hole I used round pegs or the simple reason that they are easier to sand down to the right size if they deform and are too tight a fit.

\n" }, { "Id": "14284", "CreationDate": "2020-08-19T12:06:53.827", "Body": "

I've recently purchased an Elegoo Mars Pro 3d printer, and I was wondering when printing large pieces that need to be printed in different steps: is there some kind of post-processing to make it more suitable for gluing them?

\n

I know of joints and so on, but sometimes pieces are big but not thick enough to be able to put a joint somewhere, so I guess the only solution would be to glue them.

\n

Should I use some kind of specific glue to get better results with resins? Is there any process (post-processing, reducing layer height when printing, leave the resin being curated more time...) to improve the sticking of the different parts? Or can we go with any kind of "strong multisurface glue" without any further post-processing steps, and the results should be the same in matters of quality?

\n

Thank you!

\n", "Title": "SLA resin post-processing: gluing parts together", "Tags": "|post-processing|sla|", "Answer": "

As I learned after asking about the chemistry of a 3D printing Resin, the material uses radical polymerisation to get a well connected, branched copolymer. It is comparable to resin cast material but more brittle. So let's look, akin to the PLA-gluing question, what we can do!

\n

Step 0: Safety First!

\n

Some of these methods are working with chemicals that can irritate the skin (resins, cyanoacrylate) or have irritating or flamagle fumes (heated cyanoacrylate).

\n

Use proper protection when working with these! Eye protection and respiratory protection, as well as gloves, are to be used when necessary. Read the manual of the products you are working with!

\n

Preparations

\n

For most glues, it is advisable to prepare the surface: very lightly sand it to increase the surface area, don't touch the prints with bare hands to prevent fingerprints etc. Follow the manual!

\n

Glues

\n

As a result of the chemistry I would suggest the following glues:

\n\n

Other methods?

\n

Unlike PLA, we can not use thermic methods to add inserts, friction-weld or weld/solder two parts together as Resin prints are generally not thermoplastic.

\n" }, { "Id": "14291", "CreationDate": "2020-08-19T21:00:18.697", "Body": "

I've purchased an Elegoo Mars Pro recently, and I've been watching hundreds of video of all kind of processes, and everyone tells the same: resin is toxic, so you should cure it before throwing it to the trash, otherwise it must be treated as toxic waste.

\n

But I've not been able to find anything with "things that are contaminated with it". For example, if I touch with some paper the resin, I guess that I should let it cure in the sun (like putting it in a plastic box and let it cure, maybe?).

\n

But how about the IPA? How do I treat the rests of IPA contaminated with uncured resin? I don't think leaving something flammable into the sunlight during some hours is the best I can do (maybe I'm wrong and it's totally safe?).

\n

Also, are resins water-washable (like this one from Elegoo) as toxic as the rest? Can I wash them in the faucet, or should I use a plastic recipient with just water and wash them there? What do I do with it once I finish? May I throw it into the bath, or let it cure into the sunlight, or...?

\n

Hope this helps me and some others to treat resin disposals as they should be! Thank you!

\n", "Title": "SLA printers: safety with resin contaminated disposals", "Tags": "|post-processing|resin|safety|cleaning|disposal|", "Answer": "

The IPA with the resin in it is a chemical waste and toxic to nature. As such, it needs to be given to a chemical waste handler. But those handlers do charge by volume, so you need to find a way to handle volumes reasonably. On the one hand, this means to not use huge volumes for washing the prints but reasonably small batches, and on the other hand to try to saturate them as much as possible before giving them to disposal.

\n

It is not safe to leave the IPA in the sun or close to a source of flame, and it would certainly be not a good idea to try to torch the IPA to try to reduce the volume, but you might manage to try to concentrate the toxic waste if you might have access to a vacuum distillation apparatus - the IPA would be able to be distilled over to be reused while the original material would end concentrated.

\n

At the moment I am waiting for an MSDS of eSun's Water-Washable resin, but the Eleegoo one is available, and it reads that it is classed as "Aquatic Chronic Class 2" - that's better than the Class 4 which IPA-washing resins have, but it is still not Sewer-safe: Do not allow product to reach sewage system or any water source. However, water is much less of a dangerous base for the waste than IPA, making it easier to handle. This mixture too needs to go to a chemical waste handler or be made inert but the waste also can be concentrated with less danger than IPA using a similar apparatus (cold distilling out water) or even a dedicated waste cookpot/evaporation vat that is exposed to heat - though you clearly should do this in a chemical-grade air filtration unit to mitigate the chance of resin fumes escaping into the atmosphere or your work environment if you heat it more than some. The dissolved resin might not be able to cure anymore as the photoinitiator might be used up during it evaporating.

\n" }, { "Id": "14294", "CreationDate": "2020-08-20T06:47:28.257", "Body": "

I've actually read that resin printed parts are not food-safe in Reddit and Formlabs guide. However, it also says that food-safe can be achieved applying some kind of food-safe coat to the printed parts, or even printing with ceramic resin.

\n

As an owner of a Elegoo Mars Pro, I've searched for the second - ceramic resin - and I've found nothing, so I guess it's only suited for more expensive and professional printers.

\n

But regarding the food-safe coatings, is there any recommendation on which and how to apply them for resin printed objects? Is there any other way to achieve food-safe resin printed parts? Maybe are there food-safe resins, even though they may be more expensive (I haven't been able to find any)?

\n

The scenarios I think that have to be considered are four:

\n\n

I guess that the cleaning and maintenance will depend on the process to make them food-safe.

\n
\n

I've been searching for the net, and I've found these kind of epoxy resins. I see people use them and say it's food-safe, but I cannot see it stated anywhere. Maybe are those what I'm looking for? Maybe not because some of them are for wood (maybe they can be used in other suraces, too?)? Some samples:

\n\n

Hope to find a little bit of knowledge to increase safety when printing some parts and being careful about the different applications that our prints may have.

\n

Thank you!

\n", "Title": "SLA printers: food-safe resin parts", "Tags": "|sla|safety|resin|food|", "Answer": "

Resin basics

\n

Resins are tricky, but probably less tricky than FDM as the manufacturing process is much less likely to include contaminants in the shape of contaminated air, particles, or adding lead into the print. This is all due to the whole process of creating the polymer happening under the protection of the resin, which in its monomeric liquid state is so toxic that it is unlikely any bacteria can survive in it. But before going out on a spending spree, you need to be aware that there are basically 3 kinds of resins on the market, only one of them is for SLA. To know exactly what you get you'd need to read the MSDS, which is usually available by the manufacturer of the resin.

\n\n

Food safety?

\n

There are some resins is on the list of FDA approved plastics for food contact. The list is exclusive: if your plastic doesn't fit one of the listed ones, you can not use it. One example would be polyacrylate, which is defined as being "formed by melt polycondensation of bisphenol-A with diphenylisophthalate and diphenylterephthalate" - which excludes any other method of getting a polyacrylate and it prescribes exactly what basic materials are allowed.

\n

As a result, many two-component resins and light-cured resins that don't match the exact chemistry and method to create an approved plastic/coating will not match the FDA approval list and won't get approval on their own.

\n

Coating

\n

But there is often a way out by applying a proper coating, for example with a sufficiently thick food-grade Polyurethane coating for dry foods. This would render the item food safe under FDA standards for the approved appication. You'd need to adhere to the proper method to apply this coating (manufacturers add those to the labeling usually), which can at times be somewhat complicated.

\n

Insert

\n

Another way might be to add an insert that does follow FDA standards, for example, a steel cup for liquids.

\n" }, { "Id": "14295", "CreationDate": "2020-08-20T15:21:07.623", "Body": "

I've recently noticed that the pulley driving the X-axis belt on my Anet A8 is wobbling quite a lot. I took out my calipers and sure enough the hole wasn't centred. I ordered a new pulley (from a different manufacturer) which arrived today but it was also wobbling, though much less than my old one.

\n

The X-axis motor does not appear to wobble at all, I've tried placing a straight object up against the shaft and ran the motor and could not perceive and wobble at all in the motor, so I know it's not the motor. Before I start ordering more of the same part I wanted to see if anyone else has had this problem or if I'm doing something else wrong (for example if the uncentered pulley is a feature and there's some magical way to mount the belt pulley onto the motor to make sure it does not wobble). My guess is that I've just ordered cheap scrap parts and need to order something more expensive.

\n

Here is a link to a video of the wobble: https://i.stack.imgur.com/hkyxN.jpg

\n", "Title": "Anet A8 X axis drive-gear wobble", "Tags": "|troubleshooting|anet-a8|x-axis|", "Answer": "

There are several possible causes for this. From least to worst:

\n\n

So, let's see the anatomy of a Timing Belt pulley. They exist in basically 3 general types in the McMaster Carr catalog: One flange, two flanges, no flanges. I am not affiliated with them, but they are pretty much one of the parts vendors in the industry that has almost everything, making them the convenience option. In the mantra, they are the "fast-good" option with the "everything" bonus and that the catalog of parts with 3D is integrated into fusion360. A similar supplier would be RS.

\n

\"MXL\n\"The

\n

The part OP ordered is the "With One Flange" design, it is 6mm wide, 5mm axle and 16 teeth. The McMaster Carr catalog has two very close matches to OP's part: 3684N11 & 3684N12, about 6$ for a part machined from a solid piece. Why is that relevant?! well, the part OP ordered is not made from a solid piece but at least two parts: the body and one flange plate, as you can see on the product picture - the plate separation I marked with yellow here:

\n

\"enter

\n

This is where the first error appears: if the flange isn't mounted centered (in poor quality it is rarely) then it appears to wobble but actually doesn't. But the video shows that the axis seems to be straight, while both the front and backside of the pulley wobble in the XY position. This means it is not just the flange that is mis-mounted on the part.

\n

Now, error number 2 is misaligned mounting. In case the bore is good, you can often fix a misalignment by removing the part and mounting it again, more carefully, possibly using some thin shim metal around the axis. In some cases there is a pair of screws already to do this, if not it can help to modify the part with one or two extra alignment screws, allowing to sift the gear's center a slight bit by tensioning the additional screws.

\n

In the case of OP's linked part, the screw that is not radial but protrudes next to the axis is designed to come over the flat of the axis and the other one is to tighten it down.

\n

If the bore for the axis however is really skewed, and even after repeated re-mounting or shimming nothing can be done, a new part is needed. With the measurements of the part, you can order at a lot of online catalogs, the price and quality differ greatly. However, the engineering mantra strikes:

\n

Cheap, Fast, Good. Choose two.

\n

\"enter

\n" }, { "Id": "14301", "CreationDate": "2020-08-21T17:11:25.637", "Body": "

I am a newbie to 3D printing and I am making a 3D printer from scratch (Cartesian). I've got a RAMPS 1.4 shield and an Arduino Mega 2560 board. I have a few questions and it would be very nice if I can get answers.

\n

The onboard MOSFET (STP55NFO6L) for the heatbed is dead and I think that the reason is because of the high resistance of my heat bed (1.8\u00a0\u03a9).

\n

I have searched a lot and I am fully confused about what to do.

\n
    \n
  1. I am thinking of buying an external MOSFET and wire it up with the RAMPS fan MOSFET (STP55NF06L) and use the same heat bed.

    \n
  2. \n
  3. Buy an external MOSFET and replace the onboard MOSFET (dead one) with the fan MOSFET (because of the underlying 11\u00a0A circuit) and use same heatbed.

    \n
  4. \n
  5. Same as 2., but buying a new heatbed also.

    \n
  6. \n
  7. Buying a RAMPS 1.6 and use old heatbed.

    \n
  8. \n
\n

I am totally confused because of lot of searching. Please anybody help me. What should I do?

\n", "Title": "MOSFET burned and high heated bed resistance", "Tags": "|diy-3d-printer|heated-bed|ramps-1.4|arduino-mega-2650|mosfet|", "Answer": "

The resistance of the heated bed being too high can not have caused the MOSFET to burn out. Only a too low resistance could cause that.

\n

Keep in mind that measuring relatively low resistances (such as the one of your heated bed) is difficult, and if you just used a regular multimeter it might indicate a wrong value (e.g. due to the resistance of the test leads or a poor connection between the probe and heated bed). Therefore, the actual resistance might be (slightly) lower.

\n

A 1.8 \u03a9 heated bed at 12 V will draw around 7 A of current. This is well within the capabilities of the MOSFET on board of the RAMPS. So either the resistance of the heated bed is lower than you measured (if the resistance was 1.2 \u03a9 or lower this could cause the MOSFET to burn out), or you simply got unlucky with the quality of your RAMPS board.

\n

The (supposedly) high resistance definitely doesn't make the heated bed compatible with 24 V. Using a 1.8 \u03a9 bed with 24 V would cause a 13 A current draw and 320 W of power. This is a lot more than is sensible for a 214 mm x 214 mm bed.

\n

All of the alternatives 1 and 4 you have listed are reasonable options. There is no reason (option 3) to replace the heated bed as there is no indication it is faulty. I see no reason to take option 2 since it involves desoldering and resoldering the fan MOSFET for no reason (if you are using an external MOSFET the fuse is no longer a limitation). There is a chance you'll damage the MOSFET doing this and option 1 only requires a trivial firmware change.

\n

Option 4 (upgrading to RAMPS 1.6) is purely a matter of personal preference.

\n" }, { "Id": "14310", "CreationDate": "2020-08-23T10:10:05.100", "Body": "

The closest thing I can find is this wiki page that state DRV8825 and A4988 can be mixed. However, the TMCs seem different, requiring more connections and offer more features, so I wonder if they can be used on the same RAMPS board with A4988, particularly for X and Y axis.

\n", "Title": "Can I mix TMC drivers with A4988 on the same RAMPS board?", "Tags": "|stepper-driver|ramps|tmc2130|", "Answer": "

Yes you can mix different drivers, including the TMC drivers (e.g. using for X and Y only).

\n

How you do that is described in this instructable.

\n
\n

Please do note that, from observations, the 8-bit based Arduino boards and shields such as the RAMPS are becoming more or less obsolete; the 32-bit based controller boards are becoming mainstream. Such boards have a lot more potential in execution speed, memory and more available options for peripherals.

\n" }, { "Id": "14322", "CreationDate": "2020-08-24T21:43:23.013", "Body": "

When trying to print (using 215\u00a0\u00b0C for the hotend and 65\u00a0\u00b0C for the bed) there seems to be a clicking and filament stops extruding.

\n

After much work I have cleaned the nozzle and made sure there are no clogs in the machine. However the filament seems to not extrude. When I get the filament out, there are small cuts/marks on the filament. Please see the attached picture for a better view.

\n

I'm not sure if the marks are causing the issue or if there could be something else going on. Any suggestions on how to fix the extruder to push out filament? Everything was working normally until yesterday after I leveled the print bed.

\n

\"Filament

\n", "Title": "Clicking noise from the extruder and no filament coming out", "Tags": "|filament|extruder|monoprice-maker-select|", "Answer": "

There are a number of points missing, but I can offer a few suggestions. The re-leveling of the bed could have resulted in the bed now being in a position to block the nozzle. I have done exactly that in the past. A clean glass bed makes for a perfect clog.

\n

The marks on the filament fit the description of a hobbed gear (extruder drive bolt) chewing into the filament, which would move only slightly from being blocked at the nozzle.

\n

Additionally, filament temperature may be a factor, although it's difficult to determine with the information provided.

\n

For a test, consider to use the controls available to you to raise the nozzle/lower the bed. Bring the nozzle up to your normal filament temperature. Execute an extrusion of sufficient length to ensure that the filament will reach and exit the nozzle.

\n

If this does not provide extrusion, raise the temperature five degrees C and make another attempt. It may be necessary to raise the temperature in steps more than one time.

\n

Use undamaged filament in these tests to ensure that the damage does not factor into the problem.

\n" }, { "Id": "14327", "CreationDate": "2020-08-27T07:11:30.620", "Body": "

I have purchased a Creality LD-002R resin printer. It is solid hardware, with a good experience and great value for the price, but it comes with very little documentation.

\n

I am looking at calibration procedure of the print surface. I have read in reference to other printers that it is customary to interpose a sheet of paper when leveling the print surface against the pane of glass. Is this applicable? Are there any other setup recommendations?

\n", "Title": "Calibrating a resin printer", "Tags": "|resin|creality-ld-002r|", "Answer": "

First of all a caveat:

\n

Leveling an FDM printer

\n

...is done differently than an SLA/DLP printer\nWhen calibrating an FDM printer, you align the bed with the X and Y axis, using a paper, metal shim or another tool (such as a feeler gauge) to make sure the plane that the nozzle moves in is parallel to the bed as much as possible. The paper, shim and gauge all serve in making sure that the nozzle does not push into the bed on the move and to be an easily measurable system. You need to do several measurements and repeat the process till the bed is level. This can take quite some time with paper or shims, which is why I use a feeler gauge in the micrometer area.

\n

Leveling an SLA/DLP printer

\n

when leveling an SLA/DLP (aka Resin) printer, you also want to align the bed, but you want to align it parallel to the screen. The process is rather simple: mount the bed and loosen the retention mechanism, have the printer move down and press the bed against the screen surface. Some printers need to have the (preferably empty and clean) vat installed, others need you to remove it. Tighten the retention mechanism. Move the bed up, you are leveled. Only now comes the resin (and vat!) back. Some printers have their leveling mechanism separated from the mounting screw - that allows removing the bed for taking off the part and keep the leveling intact.

\n

In the case of the Creality ld-002r there's a video Tutorial, the alignment screws are on the side of the carriage. Remove your resin vat before leveling. Loosen only those side screw, keep the top one fast!

\n

The reason why I suggest to level against an empty vat is to make sure that on the one hand the resin doesn't go bad (it should be exposed to as little light as possible), you are exposed to the least resin fumes as possible and to make sure that no pieces are inside the vat when your printer firmly presses the bed against the screen - it could damage the screen or surface would there be chunks of former prints left in the vat! Also, you don't want to contaminate your tools with resin, which means you want to work on the fasteners with no resin in the vat.

\n" }, { "Id": "14331", "CreationDate": "2020-08-28T11:36:16.800", "Body": "

I am currently trying to print a gear with the possibility to connect it to the shaft of a DC motor. The following picture best describes what the shaft of the motor looks like by showing the hole printed in the gear:

\n

\"enter

\n

So far this works well for a short amount of time, however since the gear is connected to another gear that is from time to time exerting quite some resistance already after a few revolutions the shaft starts to wear out the material that is holding it and it turns without powering the gear, i.e. the printed part is not strong enough to withstand the torque of the motor.

\n

I am wondering what the best way forward would be here. I see multiple options:

\n\n

Are there other ideas? What could you recommend me to improve this connection?

\n", "Title": "Design connection to motor shaft for printed gear", "Tags": "|3d-design|print-material|filament-choice|stability|", "Answer": "

If you're thinking of changing materials, you're looking at the wrong parameters. PLA has a higher hardness than either PETG or ABS, but hardness isn't what you want.

\n

The problem you're experiencing is creep, where a material flows in response to pressure. Of the common printing plastics, PLA is by far the most susceptible to creep. Either ABS or PETG will be better in this regard, though unless you need ABS's strength, I'd recommend using PETG because it's easier to get dimensionally-correct prints.

\n

That said, the contact area shown in your drawing is rather small. You'd be much better off using a metal flange to spread the contact out over a larger area.

\n" }, { "Id": "14336", "CreationDate": "2020-08-28T22:52:07.143", "Body": "

Does anybody know an exact difference between E3D High Precision Heater vs a standard one? I mean technical characteristics not only about its naming.

\n

Or maybe somebody has both of them and did some comparison?

\n", "Title": "E3D High Precision Heater difference", "Tags": "|heat-management|e3d|", "Answer": "

E3D themselves specify this on the product page:

\n
\n

Our high precision heater cartridges feature a rounder cartridge with more consistent diameter and surface quality, ensuring greater surface area contact with the heater block for more reliable heat transfer. With the quick change principle in mind, the high precision heater cartridge uses a Molex Microfit 3.0 connector enabling quick HotEnd changes.

\n
\n

E3D also provide a datasheet on the precision heater cartridge which can be compared to that of the regular heater cartridges.

\n

From an electrical perspective, the are identical. They will put out the exact same amount of heat. The main difference is in the dimensional specifications, which for the precision cartridge is guaranteed to be a diameter of 6.0\u00a0mm +/- 0.2\u00a0mm and a length of 20.0\u00a0mm +/- 0.5\u00a0mm, while for the regular cartridge no tolerances are specified though they have the same nominal dimensions.

\n

Apart from this, as we can discern from the product description, the precision cartridge comes with a connector to enable swapping and a better surface finish.

\n" }, { "Id": "14342", "CreationDate": "2020-08-30T04:05:12.090", "Body": "

This is more about using polycarbonate with silicon heated pad as the bed itself, and not as a material that goes onto an existing aluminium bed; i.e. I am not intending to use it as a flexible build surface (although that could be an option as well).

\n", "Title": "Can Polycarbonate be used as heated bed", "Tags": "|heated-bed|", "Answer": "

Since Polycarbonate (PC) has glass transition point of 147\u00b0C (according to wikipedia) where it starts to melt, you could in theory use it as a heated bed for PLA or even PETG. BUT, there are other characteristics:

\n

PC is quite good heat insulator, which would result in uneven heat distribution in the heated bed.

\n

Also it expands quite a lot with rising temperature, which could result in warped bed.

\n

And as per usual with thermoplastic polymers such as PC, heating them and cooling them repeatedly can cause material degradation. That would result in the material becoming brittle, deforming, or changes of other properties....

\n

Next problem would be that it is not stiff enough. Depending on the size of the heated bed you would have to support it on multiple points (I would say at least 5x5 grid for 300x300mm bed) and even then it would be unpredictable.

\n

To sum it up: yes, you probably could use PC as heated bed, but it is much better to stick to traditional materials like aluminium or glass fibre sheets (PCB material), because PC would be very inconsistent and therefore hard to level.

\n

I hope this helps.

\n

Note: I am basing this on my theoretical knowledge, I have not tested it and thus do not know if my assumptions are correct or not.

\n" }, { "Id": "14348", "CreationDate": "2020-08-31T00:56:59.493", "Body": "

I have a brand new Creality CR-10 S5. I'm new to printing.

\n

I'm trying to level the print bed, but after using the Auto-Home function the nozzle is left touching the glass. I understand that to do the leveling, I must move the head manually to the four positions for adjusting. But I really don't want to do that because I don't want to damage the glass, nozzle, or both. From the explanations of leveling I've found, I think I should expect the nozzle to be too high if anything.

\n

Am I conceptually wrong, or have I made some rookie mistake?

\n", "Title": "Can't level CR10 as nozzle is touching the glass", "Tags": "|bed-leveling|creality-cr-10|", "Answer": "

If the nozzle is touching the bed, you need to further screw down the bed. If the springs under the screws are already fully compressed, you cannot lower the bed further and you will need to move the Z-endstop up. Note that there are handy fine tuning aids you can print to help you with this, see e.g. this fine tune part for your printer. Note that this issue is not uncommon, I've read that more people encountered this.

\n" }, { "Id": "14358", "CreationDate": "2020-09-01T19:32:05.367", "Body": "

MY new BLTouch doesn't seem to be actually sensing anything, and I'm not sure how to test it.

\n

Quick background: I have a bit under a decade of experience with printing generally, but essentially none with firmware or any of the specific hardware. I recently got a new Ender 3 Pro, and successfully printed with it for a month.

\n

I recently decided to upgrade my Ender 3 pro motherboard and add a BLTouch. I installed an SKR Mini E3 v2.0, tested briefly (long enough to satisfy myself that it worked as expected, but nothing extensive). I bolted on a BLTouch (not sure if 3.0 or 3.1), built and installed a cable to the dedicated "Z-Probe" port (same pinout as the BLTouch: brown, red, yellow, black, white), and updated the firmware with the binary firmware file from the controller board manufacturer. I have no other mods to electronics.

\n

Right now, I have the following behavior:

\n\n

So far so good, but then this:

\n\n

So, What next? I'd like to check that the BLTouch is actually sending a signal to the board, but I'm not sure how. I've reviewed a bunch of online tutorials and the documentation for the mainboard and the BLTouch, without finding an answer.

\n

Available tools: I have a multimeter and a very cheap oscilloscope. I haven't gotten the maple console installed, but do have a (probably?) functional alternative through the Pango slicer.

\n", "Title": "How can I tell if BLTouch is triggering?", "Tags": "|creality-ender-3|bltouch|", "Answer": "

Problem is solved, though I still don't know how to test the BLTouch.

\n

I had reversed the power connection (black and white wires) at the connection to the extension cable. Swapping that connector back around restored expected behavior.

\n" }, { "Id": "14360", "CreationDate": "2020-09-02T03:10:08.280", "Body": "

I am very new to 3D printing. I am looking to purchase the Ender 3 V2 but before I do I would appreciate some advice on what I should purchase as the upgrades/addons for it. I'm not sure as to what I would need for the V2 since most websites are for the original Ender 3 so I don't know if the V2 already has that upgrade/addons or not when compared to the original. I would rather get all the upgrades/addons and the Ender 3 V2 at the same to save on shipping and all.\u00a0

\n

Pretty much the only upgrade/addons I have constantly seen is the auto bed level.\u00a0

\n

So if anyone can help out a first time 3D printer noob then that would be great!

\n", "Title": "Ender 3 V2 upgrades and addons", "Tags": "|creality-ender-3|", "Answer": "

As per my information, you don't need to upgrade your 3D printer but some of the following steps keep in mind to have your Ender 3 print better:

\n
    \n
  1. Level the bed.
  2. \n
  3. Set the nozzle height.
  4. \n
  5. Try different Build Plates for different effects.
  6. \n
  7. Keep it on the sweet spot for your bed temperature.
  8. \n
  9. Printer adjustment and maintenance are kept on priority.
  10. \n
\n" }, { "Id": "14361", "CreationDate": "2020-09-02T05:13:19.757", "Body": "

I just bought a new RAMPS 1.6 shield to replace my old RAMPS 1.4 shield. The problem is that it's just not working, the motors don't move, heatbed/nozzle don't get heated. It was all working with the RAMPS 1.4 shield.

\n

I checked the RAMPS 1.6 and there's no bridging in the solder joints.

\n

The DRV8255 is drawing current also but the motor doesn't move.

\n

I am using Repetier Firmware. I tried Marlin (default) and even it's not working.

\n", "Title": "New RAMPS 1.6 not working", "Tags": "|diy-3d-printer|arduino-mega-2650|ramps-1.6|", "Answer": "

So In my case, the RAMPS were defective, I returned them and got a new replacement from Amazon. The new one worked just fine.

\n" }, { "Id": "14363", "CreationDate": "2020-09-02T12:54:46.087", "Body": "

I'm not really sure what the type of plug on the heater cable is. Is it a Molex KK or maybe a JST PH?

\n

The printer is a Prusa I3 Hephestos (aka BQ Hephestos). It came with this "BQ HOT-END HEATCORE CLASSIC" hotend from the "BQ Witbox 1" extruder.

\n

\"Hotend

\n", "Title": "What type of plug is this?", "Tags": "|prusa-i3|hotend|electronics|replacement-parts|bq-hephestos|", "Answer": "

As 0scar noted, this looks suspiciously like a JST connector, but the left one is not a JST RCY connector and it is neither one of the common JST PH nor JST XH, JST manufacturer pages show. In fact, it's not a wire-to-wire JST connector. The BQ-store claims it is a 2.5\u00a0mm JST connector, but JST has some 10 dozen different types of connectors, some three dozen of them with a 2.5\u00a0mm pitch.

\n

"JST Quick" / JST RCY

\n

\"enter

\n

This one is rated 3A, and looks like the connector on the right, the thermistor one. So if you need to fix that, you know what to get for that.

\n

So what it is?

\n

The connector however looks at first glance suspiciously like this one:\n\"enter

\n

I found this product on several warehouses, listed as 2-PIN CONNECTOR W/HEADER, .10", and even found a specsheet. Those products appear, in design, to be based on the Molex KK 254 from the 2659 series. A genuine Molex 2659-series connector is rated for up to 2.5 Ampere, and looks somewhat similar.

\n

However, the shop did claim it is a JST 2.5\u00a0mm pin, and they give a side view:

\n

\"enter

\n

That is not a Molex KK. It appears more similar to a JST NV, which however has a 5 mm distance between the peg centers (= pitch) and it's rated for 10\u00a0A (or 120 W at 12V!). While matching in style it does not match in measurements - as OP confirmed, there's a very close to a 4\u00a0mm pitch (+- measurement tollerance) on the connector. So it's not an NV, but something os similar style.

\n

\"enter

\n

But then it has to be the VH! The VH series has a 3.98\u00a0mm pitch, it has that latch and it is rated 10\u00a0A, for 120\u00a0W at 12\u00a0V. In fact, the pins on the Hotend seem to be B2P-VH, matching VHR-2N or VHR-2M "female" adapters.

\n

\"enter

\n

Safety?

\n

I would not trust a Molex KK 2659-series connector with a heater cartridge on a 12\u00a0V Machine! With a 12\u00a0V, 30\u00a0W Heater cartridge draws exactly 2.5\u00a0Ampere, so you'd have a safety margin of 0! That's bad design. A 40\u00a0W heater cartridge would draw 3.3\u00a0A - that's 132\u00a0% of the rating! That'd be a fire waiting to happen!

\n

Only a 24\u00a0V machine could be built with a Molek KK 2659 connector and stay within the 2.5\u00a0A rating (40\u00a0W & 24\u00a0V -> 1.67\u00a0A, 30\u00a0W & 24\u00a0W -> 1.25\u00a0A) with a safety factor of about 1.5 to 2 to the rating (depending on heater cartridge).

\n

However, this is a JST VH with a rating of 10 A. That means, at 12\u00a0V, it's safe for 120\u00a0W load, so plenty safe: That's a safety facor of 3, and on a 24\u00a0V machine it'd be 6. That's Perfectly safe and sane! After all we look for at least a 5\u00a0A rated connector in conjunction with a 12\u00a0V/40\u00a0W heater.

\n

The more tedious variant to connect safely is to use either an even higher rated connector (requiring replacement) or a continuous wire to the board.

\n" }, { "Id": "14388", "CreationDate": "2020-09-06T17:45:07.533", "Body": "

I am using two resin printers based on the ChiTu software (Creality LD-002R and SainSmart Kumitsu KL9), and the former uses ChiTu 1.6.2, while the latter came with 1.6.1. I believe both will work just fine with the latest version, but it occurs to me that having separate copies of the software may be a good idea to keep configurations separate as well: Creality's version of the software came pre-configured (and the settings are undocumented!), while the SainSmart manual documents the software settings.

\n

I don't want to go back-and forth between two sets of settings manually \u2014 the secret recipe approach has poor UX and time's a wastin'. I am wondering if there is any concern with installing multiple copies of ChiTu. And if anyone has a smarter way to accomplish the same (printer profiles? INI files?), I am all ears.

\n", "Title": "Installing multiple versions of ChiTu side by side", "Tags": "|resin|creality-ld-002r|chitu|sainsmart-kumitsu-kl9|", "Answer": "

Running multiple versions of the same software did not sit well with the software person in me, so I dug a little deeper.

\n

ChiTu has a "settings" button to the right side, and under that section it is possible to configure different, separate printer profiles. That may take care of the differences if properly configured, unfortunately only some printer profiles are pre-loaded. ChiTu 1.6.4 includes the Creality LD-002R profile, while the very recent Kumitsu KL9 is not predefined (yet), but the manual comes with a screenshot of the parameters just as they need to be entered.

\n" }, { "Id": "14392", "CreationDate": "2020-09-07T00:41:52.533", "Body": "

Can different resin colors be mixed to generate new colors? I use eSun water washable resins, but I am happy to switch if this is a different vendor's capability.

\n", "Title": "Can photopolymer resin colors be mixed?", "Tags": "|resin|", "Answer": "

Yes and No at the same time

\n

First of all, yes, you can mix resins. However, you should only mix resins that are of the same makeup, as in one brand and type. Why? because different types of resin have different compositions and different polymerization types. Mixing different types can result in unpredictable behavior, and not working at all!

\n

To test, use a tiny amount of your brand and type matched resins and apply to the vat, then print something really tiny, like just 2 or 3 layers of a 20x20x20 mm cube.

\n" }, { "Id": "14402", "CreationDate": "2020-09-09T09:47:39.040", "Body": "

There are tons of fancy filaments around. And a lot are super nice to look at or have super cool properties, like carbon-filled nylon being comparatively far stronger than other material, wood fill is aesthetically pleasing and even just Glow in the dark! But a maker space nearby just banned any of those as abrasive filament. But, how can I know if my filament is abrasive?

\n", "Title": "How do I know if a filament is abrasive?", "Tags": "|filament-choice|knowledgebase|filled-pla|", "Answer": "

A filament made of pure plastic won't be abrasive. The abrasion comes from the added particles.

\n

Filaments with added particles of any kind (there are not so many after all: glass/carbon fibres, metals, glow in the dark, wood, stone) will usually be always be advertised as such because they always carry a higher price tag compared to the plain plastic, therefore you know that it contains potentially abrasive particles.

\n

Once you know that particles are added, most of the time they will be abrasive: as far as I know, only cork is not, any other kind of particles I listed (including wood particles) may easily scratch brass.

\n

If it were your printer, we could discuss how much each kind of particles will abrade, but in your case the ban seems to be complete, therefore only plain plastics (including "plus/+/Pro" blends, like for PLA and ABS) are allowed.

\n" }, { "Id": "14418", "CreationDate": "2020-09-13T05:44:54.303", "Body": "

The printer I am working on is an Artillery Sidewinder X1.

\n

I have fitted it with a 3DTouch (BLTouch clone) bed leveling probe purchased from Bangood.

\n

I have printed a mount for it and plugged it into the main board. Once that was done, I edited the firmware and enabled all settings for the 3DTouch roughly following the directions from Teaching Tech's video here.\nAfter flashing the firmware, an error message appears on every boot saying:

\n
Failed to enable Bed Leveling\necho: Bed Leveling off echo:\nFade Height Off ok\n
\n

After pressing confirm, ABL routine does not work. It homes the X-axis, then the Y-axis, deploys and stows the 3DTouch probe once. It also will show an error message for a split second (so I am unable to know what it says) the comes up with an EEPROM message.

\n

Auto homing does the same thing as ABL routine but with a different error:

\n
STOP called because of BLTouch\nerror - restart with M999\nError:Printer stopped due to errors.\nFix the error and use\nM999 to restart. (Temperature\nis reset. Set it after res[e]t\n
\n

I have also zipped up my Marlin firmware configuration I am currently using. They can be found here: here.

\n

My servo pin is connected to D11 and my two wire pin is connected to the ZMAX endstop. Here is a diagram:

\n

\"3DTouch

\n", "Title": "Artillery Sidewinder X1 3DTouch not working", "Tags": "|marlin|firmware|bed-leveling|3dtouch|artillery-sidewinder-x1|", "Answer": "

Ok so I did a bit of testing with the wiring and it turns out I just had the 3DTouch in the Z- socket instead of Z+ :-) .\nAnother idiot mistake from me!

\n" }, { "Id": "14435", "CreationDate": "2020-09-15T18:22:11.730", "Body": "

I am using "Pretty PETG" along with PrusaSlicer's consecutive print mode.

\n

What I am noticing is that upon finishing the first print, the printer hits MINTEMP BED Fixed. I'm not sure if it's immediate because I let the prints run overnight but I assume the bed cools down and then the error is hit.

\n

I'm just starting to learn G-code and my initial thought was there's an errant bed temperature instruction but the only M140 S0 instructions I see are in the end_gcode and near the bottom of the file. Maybe there a goto in G-code which may be running after M140 S0 which then causes the MINTEMP BED issue? Perhaps there's something else going on?

\n", "Title": "MINTEMP BED Fixed error when running consecutive print", "Tags": "|prusa-i3|g-code|petg|prusaslicer|prusa-research|", "Answer": "

This is most likely a hardware problem, namely the heat bed thermistor cable.

\n

To confirm this as the root cause, here's what you can try

\n\n

If you hit BED MINTEMP or BED_MINTEMP fixed then you have found a cable angle/position that causes the issue. You may need to replace the cable.

\n

The issue could also happen at the connection with Einsey. If the wrapped cable going into Einsey is moved, the connection may be faulty.

\n\n

If you hit BED MINTEMP or BED_MINTEMP fixed then the connection is bad. Try unplugging and replugging the cable. Look at the connectors on both sides to try to determine if the source is the cable or Einsey. You may need to replace the cable or repair the Einsey connector.

\n" }, { "Id": "14438", "CreationDate": "2020-09-16T23:51:57.613", "Body": "

I have a base Creality CR-10 that I have had for 10 months. Since then I have added Z-axis braces and a Z-axis dual lead screw. I have a Filament runout sensor that I want to add and I want to add a BLTouch bed leveling. I know I can probably squeeze those last two upgrades in using the V1.0 board that came with the CR10, but I really want to upgrade boards to the V2.0 that use the TMC2208 drivers.

\n

My questions are:

\n\n

Any insights, comments or links to useful posts/websites very much appreciated!

\n", "Title": "Creality CR-10 board upgrade from V1.0 to V2.0 questions", "Tags": "|marlin|creality-cr-10|bltouch|", "Answer": "
\n

Any issues compatibility wise or firmware wise that I might run into while upgrading

\n
\n

These boards are 8-bit boards with limited storage capacity for your firmware, if you would need an update, as an alternative solution, a 32-bit board may be a much better solution.

\n
\n

Also I know that the V2.5 board uses Marlin firmware, is that the same firmware that the V1.0 uses

\n
\n

I do not own this board so I can't say for sure, a generic remark would be that the Marlin 2.x branch works fine on 8-bit boards (from experience with some of my own boards), but you frequently see (does not have be the case for this board!) that these OEMs use the latest from the 1.1.9 branch. The only drawback is that you sometimes need to be creative to fit the firmware on the board, the more options, the more memory is used.

\n
\n

Is it worth it to upgrade to BLTouch

\n
\n

That depends on the state of your heated bed/build platform. If the build platform is not flat, but is somewhat curved, you may see improvements in bed adhesion when you correctly add a sensor that maps the surface and adjusts for it during printing. If it is flat, manual leveling works fine.

\n
\n

I am not that clear on whether you would have to ever use the manual bed leveling adjustments or does BLTouch take care of that forever?

\n
\n

Even when using a sensor that maps the surface of the build platform, you should always try to deliver a bed that is as level as possible. In videos you often see the bed tilted (very much exaggerated); you need to remember that the printer will print in a plain level after about 10\u00a0mm (or to a different height, determined in your firmware or set by G-code), a skew platform will give you a skew print.

\n" }, { "Id": "14439", "CreationDate": "2020-09-17T01:06:18.653", "Body": "

On 16 September 2020, Autodesk announced changes in the way that Fusion 360 can be used for non-commercial use with their Personal license. As a hobbyist, most of these changes will not affect me very much, since I do not use Fusion 360's advanced features. The most irksome will be only being allowed to have up to ten "documents" active at any one time, the rest having to be archived.

\n

However, Autodesk are also restricting the number of file formats that you can export to. For example, the STEP file format will no longer be available with the Personal license. Will this mean that I will not be able to move my models to another CAD package, such as FreeCAD, once the changes come into effect (without first buying a commercial license)?

\n

Autodesk: Changes to Fusion 360 for personal use

\n

Edit: Good news. Autodesk have announced, on 25 September 2020, that the facility to export models to STEP files will be retained for the free-to-use, personal license.

\n", "Title": "Will I still be able to export my Fusion 360 models to other CAD packages after Autodesk's recent license changes come into effect?", "Tags": "|fusion360|", "Answer": "

If you use the private license: there was supposed to be a cutoff date.

\n

As long as you use the "private" license, you will get some restrictions. Originally, including the lock off of .step and similar files as well as limiting you to 10 active projects. This means, that you will need to deactivate some to make room for new ones, but unless you have many interlocking parts, 10 can be quite a lot for a hobbyist.

\n

They also lock off features that are pretty much only useful for small companies - and if you are in the shoes missing those features, you are on the wrong free license anyway.

\n

In late (25th.) September 2020, the lock of .step was reversed, but other formats will still no longer be available.

\n

This does not apply to all free licenses

\n

Note that this does not apply to the free education license for teachers and students or the free business license for sub 100,000 \\$ companies. You might want to consider swapping to either of these two plans if you qualify.

\n

Last words

\n

I agree with Thomas Sanladerer: Autodesk is under no obligation to offer a free version at all and even a somewhat restricted, locked into the Autodesk-world version is much better than nothing. I will keep using it, but I am also in the education space and thus not directly affected.

\n" }, { "Id": "14474", "CreationDate": "2020-09-23T12:06:31.297", "Body": "

I've been happily printing on my CR-10 for a couple of months but now there is an issue. As soon as I turn it on the hot-end begins heating. I have replaced the heater and the thermistor but the problem remains. The panel on the control box shows 0 for the commanded temperature, but the actual temp just keeps rising.\nAnyone know what could be wrong?

\n", "Title": "CR-10 heating problem", "Tags": "|hotend|electronics|creality-cr-10|heat-management|", "Answer": "

If you power the printer and it starts heating up the hotend, it most probably is caused by a faulty controller board. More specific, the MOSFET (as in an electronic switch component) controlling the current to the heater element is causing this. MOSFET devices usually fail short-circuit implying that when the MOSFET has failed, the current can freely run to the heater element without being controlled by the board.

\n

If you are handy with electronics, you can replace the MOSFET, but it might be better to upgrade to a newer controller board as there may be other damage as well.

\n" }, { "Id": "14490", "CreationDate": "2020-09-24T15:24:28.030", "Body": "

My Ender 3D with BlTouch And Marlin 2.0.1 firmware was working flawless for 3 months.

\n

Yesterday, without any hardware or software change, it started acting weird: on Auto-Home command it first makes expected moves (home X and Y), then moves to the center of the bed and then instead of going down to bed, it rises all the way up and then writes a message about End Stop. This of course happens also on "start print", "level bed", etc.

\n

Before the Auto Home command, I can operate the Z axis normally, after it, the printer does not allow movement down.

\n

Do you have any ideas, what might have gone wrong?

\n", "Title": "Hotend moves up on HOME command", "Tags": "|creality-ender-3|z-axis|", "Answer": "

The mystery is solved. The BlTouch probe was slightly blocked and reported (supposedly) being in touch with the plate. Just touching it released the probe from the stuck position and all works well now.

\n

Due to direct sunlight, I overlooked the warning blinking red colour of the BlTouch.

\n" }, { "Id": "14497", "CreationDate": "2020-09-25T21:06:43.617", "Body": "

Not storing left-over photopolymer resin back with pristine resin in its original shipping can seems to be a common recommendation.

\n

What is the best practice here? Store in a separate bottle and pour this "once around the block" resin first for the next print?

\n", "Title": "left-over photopolymer resins", "Tags": "|resin|", "Answer": "

Safety first

\n

I suggest the following handling of resins, some basic stuff first:

\n\n

Re-cycling

\n

Now, what to do to get the used resin back to the cycle? Any resin that has been exosed to air and light, such as having been in the vat is best considered to be B-Quality. You can use it to cast greeblies or bits (aka disposal by curing), as one would do with leftover casting (2-component) resin, but that is a waste.

\n

Step 1: Re-botteling

\n

So, let's look at some better ways: first of all re-botteling the resin. We need to take in mind, that the quality of our resin will further degrade the longer it stays exposed to light (and to a lesser degree: air), so we need to handle the resin in a way that allows us to eliminate exposition to either. For this, it would be best to keep an empty resin bottle at hand and label it as the leftover bottle. To fill this bottle, you should use a Jig to keep the vat in a position that it pours into the bottle. You might want to use a funnel in some cases!

\n

Step 2: Re-conditioning

\n

Now, we know how to get the stuff back into the (B-quality) bottle. But how to make sure it has the best quality we can? As you notice, many of the jigs involve a funnel. This funnel is used in conjunction with a filter to remove larger particles. The finer the filter, the better. Coffee filters manage to snatch particles down to about 10 to 15 micrometers. It is equivalent to about Grade 4 laboratory filter paper. However, laboratory filter paper of grades 1,2,3 or 602h would allow to catch particles of even lower size, as the mesh gets even smaller, but might clog faster. Tea filters on the other hand have worse filtration ability and should be avoided.

\n

To get the best out of it, use a filtration stack, that starts with a metal mesh filter before going through a rough and a fine filter to get out any chunks and large particles that would clog the fine filter.

\n

It would be best to have this process run in the dark, so mounting the dripping and filtration stack in a box might first sound like overkill, but if you go through a large amount of resin (for example by running several printers) it might be an investment that can save a considerable amount of resin in the long run. However, if you run so many printers, you also might run them continually with the same resin colors and just refill them as needed and only filter if there had been a print failure.

\n

Step 3: Storage

\n

Store your bottles in a closed cupboard. It would be best if this cupboard is ventilated through a filtration unit and then outside. It should also keep a steady temperature above about 10\u00a0\u00b0C to prevent clumping. Just follow the storage manual for the normal resin actually.

\n

Step 4: Re-use

\n

Now comes the tricky part: re-using the resin. While technically the filtered and re-bottled resin should be almost as good as new, it would be best to make sure that we mix it with some virgin resin to make sure we have enough photoinitiator in the resin. For this, I would suggest mixing the recycled resin with between a sixth and half of the fresh stuff. Mix the two well to make sure you get the best possible. Make sure it's the same type and color of the used resin, best even from the same original batch.

\n

Use up the re-cycled resin first, as you should consider it's best before date much shorter than on the virgin bottle.

\n" }, { "Id": "14501", "CreationDate": "2020-09-27T05:53:43.027", "Body": "

My only functional computer at the moment is a raspberry pi, and I was wondering if there was any software that supported it. My printer is a Newmatter mod-t, but I might be able to modify other software to support it

\n", "Title": "Is there any 3D-printing software that supports Raspberry Pi?", "Tags": "|software|raspberry-pi|", "Answer": "

This is a case of Atwood's Law.

\n

Remember that TinkerCAD runs entirely in your web browser. If your Raspberri Pi can browse the web, you can do basic 3D modeling.

\n

Similarly, there is an online slicer available at cnc-apps.com \u2014 probably others, too. If you can browse the web, you can slice an STL to create gcode suitable for most printers. It's free to use (with limits) for guests. However, I haven't used it personally so I can't speak to the quality at this time. (If anyone wants to try this out, I'd love to hear your results).

\n

The final step is getting the printer to run the generated g-code, and here we have our old friend OctoPrint. I'm currently running this on a Raspberry Pi of my own. However, I'd be hesitant to give the Pi over to running the printer if it's your only computer. In most cases I would instead copy the gcode file to an sd card, but as the particular printer here it seems to support wifi it's likely you can upload the gcode directly.

\n" }, { "Id": "14505", "CreationDate": "2020-09-27T12:18:08.727", "Body": "

I want to print a part from Thingiverse. In the description, the creater writes that he used SBS to print it. I did some research because I never heard of SBS.\nI found a description on Filaments.directory that describes it as:

\n
\n

Poly(styrene-butadiene-styrene) is a hard, durable rubber that is commonly used for shoe soles, tires and other products that experience high wear.

\n
\n

But if I search for SBS filaments to buy, there only shows ABS up. Did I misunderstood something and SBS is the same as ABS.

\n", "Title": "What is SBS plastic", "Tags": "|filament|plastic|sbs|", "Answer": "

This response might be way late but no, SBS is not a typo.\nHere are the info:

\n
\n

SBS (styrene-butadiene-styrene)

\n

Poly (styrene-butadiene-styrene) or SBS, is a hard rubber that's used to modify asphalt, to make soles of shoes, tire treads, and other places where durability is important. It's a type of copolymer called a block copolymer. Its backbone chain is made up of three segments. The first is a long chain of polystyrene, the middle is a long chain of polybutadiene, and the last segment is another long section of polystyrene.

\n

Polystyrene is a tough hard plastic, and this gives SBS its durability. Polybutadiene is rubbery, and this gives SBS its rubber-like properties. In addition, the polystyrene chains tend to clump together. When one styrene group of one SBS molecule joins one clump, and the other polystyrene chain of the same SBS molecule joins another clump, the different clumps become tied together with rubbery polybutadiene chains. This gives the material the ability to retain its shape after being stretched.

\n
\n" }, { "Id": "14509", "CreationDate": "2020-09-27T14:33:49.120", "Body": "

I have a Cartesian printer in a cubic format (the bed moves vertically) with Marlin 2.x

\n

I am not finding a way to make it move to Z-max (23\u00a0cm) when finishing the printing.

\n

The idea is to lower the bed all the way to the max, to make easy removal of the printed piece.

\n", "Title": "How to move Z-axis to a specificic position (Z-max) after printing?", "Tags": "|marlin|z-axis|", "Answer": "

To move the bed down, you simply instruct the bed to do so after the print.

\n

The best way to do that is add a G-code line to your "end G-code" script you find in your slicer you use.

\n

Just add the line:

\n
G1 Z230 F500\n
\n

With this command you instruct the bed to lower to your maximum Z height at speed (feed rate) 500\u00a0mm per minute.

\n" }, { "Id": "14512", "CreationDate": "2020-09-28T02:02:51.437", "Body": "

I have an FLSun 3D Cube, running off an MKS GEN V.1.4 main board. With Marlin 1.0, the Z-axis works great (and has been for three years). When I try to upgrade to Marlin 2.0, moving the Z-axis 10\u00a0mm results in extreme motion -50\u00a0mm at least, left motor then the right motor, three or four times, very fast. It's loud, it's jarring, and it's at the very least incorrect.

\n

What setting am I missing?

\n", "Title": "Marlin 1.0 works: Marlin 2.0 destroys Z-axis motion", "Tags": "|marlin|flsun-3d-cube|mks|", "Answer": "

I believe (from reference) the default steps per mm for the FLSUN 3D Cube are:

\n
                                          X,   Y,    Z, E0 \n#define DEFAULT_AXIS_STEPS_PER_UNIT   { 100, 100,  400, 150 }\n
\n

Default marlin 2.0.x are:

\n
                                          X,   Y,    Z, E0 \n#define DEFAULT_AXIS_STEPS_PER_UNIT   {  80,  80, 4000, 500 }\n
\n

Such a setting would explain the excessive speeds; while the printer only needs 400 steps for advancing a single millimeter, the stepper receives 4000, this implies a tenfold, hence larger displacement and higher speeds.

\n" }, { "Id": "14516", "CreationDate": "2020-09-28T16:00:06.823", "Body": "

Of course I'm referring to side by side and not stacking or overlapping. 200 x 200 mm PEI and Kapton sheets are more readily available than 400 x 400 sheets. I'm wondering if anyone has tried this and if the edges of the sheets cause a problem.

\n", "Title": "Has anyone applied four 200 x 200 mm PEI sheets to a 400 x 400 mm bed?", "Tags": "|heated-bed|adhesion|", "Answer": "

Aligning build surfaces isn't the main issue with 1 mm thick surfaces when aligning four 200 x 200 mm surfaces to make one 400 x 400 surface. The main issue is slight bucking at the seams from thermal mismatch. Using this with PETG tends to tear up the build surface at the seams. The image shows aligned surfaces.

\n

\"enter

\n

The above was a poor design because prints tended to always print on the seams, which are more susceptible to damage when removing the print.

\n

More reliable was to put one 200 x 200 mm surface in the center of the bed, cut the other three in half, and tile them along the sides. Glue stice helps fill the seams.

\n

\"enter

\n

The center of the surface tends to wear out first. In that case only the center surface would need changing.

\n" }, { "Id": "14519", "CreationDate": "2020-09-29T06:24:13.407", "Body": "

Background

\n

I bought an Ender 3 v2 printer, everything has been working fine, been experimenting with harder and higher temperature filaments like different variations of PLA and PETG.

\n

Original extruder would bite the filament too hard sometimes and snap it, if it's loose it would grind it, filling its teeth with material that needs to be cleaned out.

\n

I bought a BMG clone (TriangleLab) extruder and set it up according to BondTech instructions but without having to invert the stepper motor rotation and set E-Steps to 419, which seems to be printing fine.

\n

It's still using the stock hotend but with 0.6\u00a0mm hardened steel nozzle. I do have BLTouch add-on and I'm using firmware compiled from tip of bugfix-2.0.x branch of Marlin with stock Creality v4.2.2 board. Firmware Configuration modifications were done to accommodate BLTouch alone.

\n

Problem

\n

I have to manually heat the nozzle and manually extrude about 200-250\u00a0mm from LCD menu, otherwise once print starts, no material will come out.

\n

I checked the bowden tube with a torchlight since it's translucent using a black filament to print and noticed that after machine is stopped (from menu or after print ends), extruder will pull the filament almost all the way out while pulling up Z axis to clear some height, requiring me to manually extrude before next print so tube won't be empty. Previously with stock extruder I didn't have to do anything, the initial print on left hand side of bed or skirt print would take care of putting it back in location.

\n

I've been googling and looking at Cura and Firmware (configuration.h) for a while now but I couldn't figure out why this is happening.

\n

This is my End code in Cura. I do suspect 3rd line has something to do with it but I don't understand what that means.

\n
G91 ;Relative positioning\nG1 E-2 F2700 ;Retract a bit\nG1 E-2 Z0.2 F2400 ;Retract and raise Z\nG1 X5 Y5 F3000 ;Wipe out\nG1 Z10 ;Raise Z more\nG90 ;Absolute positioning\n\nG1 X0 Y{machine_depth} ;Present print\nM106 S0 ;Turn-off fan\nM104 S0 ;Turn-off hotend\nM140 S0 ;Turn-off bed\n\nM84 X Y E ;Disable all steppers but Z\n\nM300 S440 P200\nM300 S660 P250\nM300 S880 P300\n
\n

Retraction settings are stock settings in Cura, haven't touched them.

\n

The Configuration.h file is found here, the Configuration_adv.h file is found here.

\n

Update: This happened again with Benchy print but about 4 cm unlike previously, half of the skirt print failed but actual print was OK. Below is last part of print Gcode. End code isn't different from Cura settings printer end code.

\n
G1 X105.151 Y113.199 E313.92164\nG0 F7200 X105.155 Y112.539\n;TIME_ELAPSED:3369.503317\nG1 F3000 E307.42164\nM140 S0\nM107\nG91 ;Relative positioning\nG1 E-2 F2700 ;Retract a bit\nG1 E-2 Z0.2 F2400 ;Retract and raise Z\nG1 X5 Y5 F3000 ;Wipe out\nG1 Z10 ;Raise Z more\nG90 ;Absolute positionning\n\nG1 X0 Y220 ;Present print\nM106 S0 ;Turn-off fan\nM104 S0 ;Turn-off hotend\nM140 S0 ;Turn-off bed\n\nM84 X Y E ;Disable all steppers but Z\n\nM300 S440 P200\nM300 S660 P250\nM300 S880 P300\nM82 ;absolute extrusion mode\nM104 S0\n;End of Gcode\n;SETTING_3 {"extruder_quality": ["[general]\\\\nversion = 4\\\\nname = Ideagen Nozzl\n;SETTING_3 e-0.6 #3\\\\ndefinition = creality_ender3pro\\\\n\\\\n[metadata]\\\\ntype = q\n;SETTING_3 uality_changes\\\\nquality_type = standard\\\\nposition = 0\\\\nsetting_ver\n;SETTING_3 sion = 15\\\\nintent_category = default\\\\n\\\\n[values]\\\\nfill_outline_ga\n;SETTING_3 ps = True\\\\ninfill_overlap = 10\\\\ninfill_sparse_density = 40\\\\ninitia\n;SETTING_3 l_layer_line_width_factor = 120\\\\nironing_only_highest_layer = True\\\\\n;SETTING_3 nmaterial_flow = 92.5\\\\nmaterial_print_temperature = 210\\\\nretraction\n;SETTING_3 _count_max = 10\\\\nretraction_retract_speed = 50\\\\nspeed_print = 60\\\\n\n;SETTING_3 speed_travel = 120.0\\\\nsupport_infill_rate = 5\\\\ntop_bottom_thickness\n;SETTING_3  = 1.05\\\\ntop_thickness = 1.05\\\\nwall_thickness = 1.2\\\\nzig_zaggify_i\n;SETTING_3 nfill = True\\\\n\\\\n"], "global_quality": "[general]\\\\nversion = 4\\\\nna\n;SETTING_3 me = Ideagen Nozzle-0.6 #3\\\\ndefinition = creality_ender3pro\\\\n\\\\n[me\n;SETTING_3 tadata]\\\\ntype = quality_changes\\\\nquality_type = standard\\\\nsetting_\n;SETTING_3 version = 15\\\\n\\\\n[values]\\\\nlayer_height = 0.35\\\\nlayer_height_0 = 0\n;SETTING_3 .4\\\\nretraction_combing = infill\\\\n\\\\n"}\n
\n", "Title": "Why would my extruder remove the filament all the way out after prints? [Ender 3 v2]", "Tags": "|creality-ender-3|extruder|bowden|retraction|", "Answer": "

As mentioned in the comments, extrusion stopped working altogether at some point halfway through a Benchy print. Once I had opened compressing lever of BMG clone extruder, I observed that gears were loose, after disassembling the extruder noticed that both extruder side and stepper motor side gears were also loosened at hex head screw keys which were keeping them in place. I ordered a thread locker to address that at another time, seems like vibrations due to constant retractions loosened them.

\n

After tightening and reassembling, issue is not bad to the point prints would fail to start, however filament is not primed enough, so some of the skirt print will be blank.

\n

I also added this with 608Z bearings for supporting the smooth movement of spool. Turns out 608 bearings are widely available almost everywhere, roller skates and skateboards use them, so there is a good chance your local sports shop will have it, too.

\n

I installed Octoprint on my laptop to work around this issue as below:

\n
G92 E0 ; Set Extruder position to 0\nG1 E100 F200 ; Extrude 100 mm slowly. This is to make sure hotend is not empty and material flow is consistent, i.e. adjust temperature if necessary.\n
\n

Only then I start my prints.

\n

I learnt that using E<integer> to extrude and E-<integer> retract, I can replace filament too, by using big enough numbers like 350 mm, so I don't have to fiddle with compression lever or its adjustments anymore.

\n

Update: I found more mechanical problems. This issue is not software related. Please see useful advice in comments regarding retraction/priming as an improvement if not solution to this problem.

\n" }, { "Id": "14520", "CreationDate": "2020-09-29T09:25:19.687", "Body": "

I'm trying to create a sketch that is constrained by surface area.

\n

In my model I have a relatively irregular, flat area that I would like to constrain by area so that it never exceeds a certain number.\nThere are a lot of other constraints for angles, lengths, points, lines and the area of a surface is also always available in the properties. As a rather new person to Fusion 360, it feels logical a constraint by surface area should be possible.

\n

Am I missing something or is there a good reason why this constraint doesn't exist?

\n", "Title": "Constrain by surface area in Fusion 360", "Tags": "|fusion360|", "Answer": "

After some testing, Fusion does have a category to put in an Area parameter [edit -> Parameters ->] for Area that takes measurements in acres and circular mil but no useful measurements such as square centimeters or square inch. One circular mil is defined as a circle with a radius of 1/1000 inch, or 0.0254 mm, for an area of $5.067\u00d710^{\u22124}\\text{ mm}^2$.

\n

However, defining the area of an item isn't (currently) straightforwardly possible: areas are not defineable, and with the obvious lack of possible parameters (square metrics and anything but super tiny or super huge) it doesn't seem to be planned. The best you can do is for bodies that you know the formula for the area in the following fashion:

\n

\"enter

\n

Here, the left measurement (d2) is 1 mm. The parameter area is 10000 circular mil. Since we know A=d1*d2, we can go A/d2=d1 for a rectangle.

\n" }, { "Id": "14521", "CreationDate": "2020-09-29T13:02:47.180", "Body": "

I am using an AnyCubic Photon Resin Printer. I have used the (WoW model viewer to export a miniture that I am hoping to 3D print.

\n

However, when I look at the model in photon workshop, certain parts appear in a different colour, and those parts are printing incorrectly on the final version. For example, if you look at the model below the headpiece feathers printed as a solid block all the way to the base of the model.

\n

\"enter\"enter

\n

Looking at the slicing file, this is what photon workshop exported. I believe the issue may be that, because these are models deisigned for games and not 3d printing, they have only one surface and what I am looking at in Blue is what the photon workshop considers to be the inside of the model. Would ye be able to recommend any steps that I could do to adjust the models to allow them be 3D printed?

\n

https://mega.nz/file/p6I00JZK#qhQplzOTh6w7yZxbGw2vtRRKc-iVDp_QHOvaWfPtULs

\n

The above link is the object as exported from WoW Model viewer.

\n", "Title": "How to fix WoW models in photon workshop", "Tags": "|3d-models|software|resin|", "Answer": "

First of all, Fred's answer is very well but some more items to note, and a different way to fix them using blender.

\n

After opening blender, deleting the box with entf and importing the Wavefront .obj, I changed to edit mode and started inspecting the colored areas for artifacts and what they were.

\n

Layered surface only areas

\n

The top feathers are made up from 3 layers of surfaces:

\n

\"enter

\n

This could be easily fixed by removing the middle layer, then connecting the front and back with new faces. In investigating this I also noted, that many of the faces actually are unconnected, so we fix that first, running a Merge intersecting vertices: rightclick -> Merge Vertices -> By Distance. Then, all the lines are blue - this means, that the lines are determined as "sharp edges" so we change to edge select, A, then rightclick -> clear sharp. Much better. Now, select the vertices in between the front and back, and delete them via entf.

\n

After that work-over, the area looks like this:

\n

\"enter

\n

So, next is re-skinning those edges. Select the remaining edges on the outside in pairs, then F to create a face between them, rinse, and repeat. But that does not yet solve the facing issue, because the faces are an odd gradient, not pure color, and as is evident, all the normals (activate them via a menu):

\n

\"enter

\n

All normals are messed up! so we need to recalculate them. Recalculating them is A till everything is selected, then Ctrl+Shift+N, and voila, that area is reworked to a good part:

\n

\"enter

\n

Now, we are going to export the file as .stl: even though Cura can take .obj, it has trouble to analyze those for facing properly at times. So let's look if we fixed that area alright... and in Cura now looks like this (OBJ left, STL right):

\n

\"enter

\n

However, as I will show later, this is the work-intensive way, so don't really bother with those areas, we'll fix them all at once later.

\n

Single surface only areas

\n

Next, the shoulder details. Those are also one surface only, so we need to give them some heft. I am talking these (grey) ones:

\n

\"enter

\n

I hid the bulk of the model (the selected, orange parts) first using H so it is more visible. Then I selected the upper 'wing', then pressed E for extrude. Type in .01 to define that thickness as how much it should extrude it out. Repeat for the other part. ALT+H to unhide the parts we had hidden before and Voila:\n\"enter

\n

However, the newly thickened parts did not intersect the body properly yet, so we need to fix that a little, as seen here:\n\"enter

\n

So, select the area we need to fix and E for Extrude. Slowly pull till the two parts intersect neatly:

\n

\"enter

\n

floating details

\n

Next step: the floating gem in front of the head!

\n

\"enter

\n

Here fixing gets tricky. We might either just delete the crystal and print it separately and fix it with a little piece of wire later, or we need to create a strut. So, let's do the latter! We select the edges on the backside of the crystal like so...

\n

\"enter

\n

rightclick -> Subdivide and then in the lower left corner set it to 2:

\n

\"enter

\n

Select the center and Extrude

\n

\"enter

\n

Keep the area selected and G for moving it down into the head, till we are happy with the look:

\n

\"enter

\n

superfluous surfaces

\n

Then there are some areas that contain "extra" surfaces, those can be deleted, such as this rectangular one and this feather inside the arm:

\n

\"enter\n\"enter

\n

Fixng most surface only areas

\n

Another problematic area is the facemask, which is only a single layer, but wrapped around the face it is a little more tricky to extrude:

\n

\"enter

\n

This area can be fixed by running a solidify operation, which also makes fixing the feathers individually obsolete! So, how to run solidify?

\n

\"enter\nOn the right bar, click the wrench, in the dropdown choose solidify, set the thickness to 0.01 and... voila! Mostly fixed... well, we need to fix a few normals, as you'll see here:

\n

\"enter\nso, let's select everything with A, then recalculate normals with CTRL+SHIFT+N, make sure that inside is not checked in the lower right corner, so the modeled surface is on the outside shell... and we are DONE! Export as STL or OBJ and go to test it!

\n

\"enter

\n

Check in Cura

\n

Now, to check if we still have reversed surfaces, I imported into Acura and... all the red areas (overhangs) are where they belong: on the underside! So... yep, we are done. Make sure to have the unify intersecting shells option on. You also might want to add some more struts for floating shoulder armor and such... It's a labor intensive work, but in the end you'll have a printable object.

\n

\"enter

\n" }, { "Id": "14531", "CreationDate": "2020-10-01T21:49:18.667", "Body": "

My goal is to have a speaker inside a PLA casing to produce a nice hearable sound from a submersible item.

\n

I intended to produce that sound from a Piezo buzzer stuck on a membrane held tight in hollow place between two pieces, but the result is unsatisfactory as the sound gets muffled to inaudible levels.

\n

Is there any known 3D printable methods to permit for a sound to be heard from PLA?

\n

\"enter

\n", "Title": "Can PLA carry sound coming from a loud speaker?", "Tags": "|print-material|", "Answer": "

I did a lot of experimentation in this a couple years ago. The limitation on the sound is the amount of energy you can apply to the plastic, and the ability of the plastic to respond (move to) that energy. I ended up using epoxy to fasten a transducer directly to the plastic wall, and while the sound quality was not great (I could not have used it for speech or music) it was adequate to make the alarm sound, 50% PWM @ 3.3V and I controlled the frequency using the Arduino 'tone' library to control the frequency. I printed that side solid (no infill) and a bit thinner than the other sides (2mm instead of 4mm), printed it separately and then used epoxy to glue in (and its attached transducer) to the cube I was using for the rest of the circuitry.

\n" }, { "Id": "14536", "CreationDate": "2020-10-03T15:43:04.700", "Body": "

After some months of good quality prints, since yesterday my heatbed does not heat anymore.

\n

I've a BQ Prusa i3 Hephestos with this heatbed, controlled by this mechanical relay and powered by this power source. I know that a mechanical relay is not the best choice for a 3D printer, but I only had one of those when I've built the bed and setting bang bang control for the plate gave me some satisfactory prints.

\n

Since if I connect directly the heatbed to the power source, the latter stops working (I think because of its shortage protection), I'm almost sure that in some way I've managed to fry my heatbed.\nBefore buying a new one, though, I'd like to understand how I've broken the current heatbed without doing anything but printing, to avoid similar situations in the future.

\n

I'm afraid I've forgotten some component whose functionality is to protect the heatbed (maybe a diode or something similar, I'm sorry but I have never studied electronics).

\n

I've attached a very simple wiring diagram showing the current configuration.

\n

\"Wiring

\n", "Title": "Short-circuited heatbed", "Tags": "|heated-bed|electronics|bq-hephestos|", "Answer": "

I see four points of error:

\n\n

So, how to check what it might be? We need to measure, but we need to measure in a setup of 12/24\u00a0V circuits while they are live. While not necessarily considered "dangerous", take extra precautions! Remove the parts you will need to access from the machine, ground the power supply, and use your utmost care! Make sure the heatbed is not connected to the Power supply, so we can measure it in the safest way.

\n

Ruling out the Heatbed

\n

The first step can be done without power: the resistance of the heatbed should be non-0, non-open line. It's best to measure directly at the input pins to the bed. If it shows close to 0 (some milli-Ohms), open line (OL), or no continuity (NC), it has a short or is burned out and needs to be replaced.

\n

Ruling out the Power Supply

\n

Once we have reached a safe setup, take a voltmeter and measure the voltage (potential difference) on the pins you reserve for the heatbed. If it is 12\u00a0V with some tolerance, it's good, otherwise you might need to get a new power supply or have a professional fix yours.

\n

Ruling out the trigger signal

\n

Next, move your measuring to the output of the board. Now order your board to heat up the bed. Make no illusions, we will trigger TRP, as the bed is disconnected. This will need us to reset the board between tests. You should see a 5\u00a0V trigger signal. If not, your board or firmware might be faulty and require different investigations.

\n

Ruing out the lines and switch

\n

Next step is to swap to Resistance mode on the multimeter and measure the resistance of the switch, best with the whole power line. Under "heating", the setup should show a close to 0 Ohm resistance, if not it should show an infinite or open line.

\n

What now?!

\n

If it is neither of the 4, reassemble on the test stand, reconnect the bed to the power supply and investigate all connections between the elements we just checked. Now order a heating procedure - as all components should be ok, it has to be one of the interfaces between the parts.

\n" }, { "Id": "14562", "CreationDate": "2020-10-10T18:33:50.613", "Body": "

I am trying to 3D print an object from Shapeways. Confidentiality prevents me from sharing the actual STL files publicly, but what I can say is that it's a shape about 2\u00a0cm tall x 4\u00a0cm wide x 4\u00a0cm deep, and where the design specifies a vertical hole 5\u00a0mm in diameter, the hole comes out fine, but where the design specifies multiple holes 3\u00a0mm in diameter, passing diagonally through the shape (at a 45\u00b0 angle with the ground), the holes seem to collapse (i.e. I can't see through them, and if I push a straightened paper clip through the hole, it's blocked). This has happened repeatedly (e.g. Shapeways offered to re-print some shapes for free when they came out defective, but this happened the second time too).

\n

I am new to 3D printing and trying to figure out:

\n\n", "Title": "Vertical 5 mm diameter hole comes out fine, but diagonal 3 mm holes keep collapsing", "Tags": "|troubleshooting|3d-design|", "Answer": "

A member of the public on Stack Exchange's forum gave me the answer:

\n

"The problem is not the hole 'collapsing' in and of itself. The problem is the material.\nVersatile Plastic is a layered powder fused together with a laser in multiple layers, so the unused power has to come out from the various holes in the model once printing is completed. However, these layers are also very thin and compact. Therefore, if you make a small enough hole, the powder simply won't flow through it and it becomes stuck. Due to the small dimensions you listed (of said hole) it's gonna be difficult to free it due to the sheer compressing forces involved. When working with such fine details Versatile Plastic is not the most indicated material.

\n

The only correction you can make to rectify the problem is changing material. Fine Detail Plastic would be a good choice for a new test, although it is not cheap compared to Versatile, is certainly cheaper than metals.

\n

Fine Detail Plastic should provide you with a good result if you feel like trying it out."

\n

ShapeWays technical support told me basically the same thing, and added that ordering the shape in black might also exacerbate the problem:

\n

He said that when a model has holes in it, a human actually pokes the holes to get residual material out if possible. Then he adds: "The last issue would be that Black indeed causes additional issues. If the production team isn't able to clean the model properly, or didn't do their work right, this could become and issue when we dye your model. All Versatile Plastic is printed in White, when selecting a colored finish such as black, we dye the model using a bath of regular RIT DYE. If the hole still has trapped powder, the bath will make the loose powder wet and make it become sticky, in combination with the dye this causes it to become chunky on the inside."

\n

TL;DR: I can try to fix it by using Fine Detail Plastic, or I could try ordering in white and then painting it black myself to avoid gunking up the holes (the insides of the holes don't need to be painted black).

\n" }, { "Id": "14563", "CreationDate": "2020-10-10T19:46:37.463", "Body": "

I have designed this model:\n\"enter

\n

My goal is to make it static, no vulnarable to shakes.\nThe pitch is 40 mm with 4 revolutions. The dimensions of the 'wire' are 5x5 mm.\nWhich settings for printing should I use to archieve hardness? I plan to make the wire 5x10 mm height, 4 walls, 0.28 layer height, 0.5 nozzle size. PLA. I don't know if this is overkill or it will be useless.

\n

I hope you can give me suggestions about printing settings and model dimensions.\nI know that's spring design, which is made to be springy but I need it for other use.

\n

Second design. Less springy, but not by much. My last hope is to print it horizontaly.\n\"enter

\n", "Title": "How to print spiral(spring) static?", "Tags": "|3d-models|", "Answer": "

If you want an helicoidal object which acts relatively rigid, you should pick a plastic which is very rigid to begin with. In the case of 3D printing, probably PLA is the best choice. Make it as thick as you can in all the directions where you have no constraints.

\n

Still, it's an object which will be hardly printable without dissolving support material. You need a two material 3D printer if you want to get good results.

\n" }, { "Id": "14570", "CreationDate": "2020-10-12T11:27:14.187", "Body": "

I love PrusaSlicer but I am having a problem.\nAs you can see in the picture the head makes movements that seem useless\"Previsualisation\"

\n

(blue lines that come out of the print in the corners and go back instantly in the object) and these make damages to the object during the print.

\n

\"During

\n

How to configure PrusaSlicer so that doesn't happen again ?\nThank you.

\n

Edit : Thank you for your answers but I think we have a misunderstanding. I already have z-hop activated, the real problem is that the slicer makes the head move outside the print (that's what you can see on the first screenshot, every blue line crossing the print shouldn't exist).\nI actually don't have this problem with curaengine on repetierhost as you can see in the following screenshot :

\n

\"enter

\n", "Title": "Useless head movements in PrusaSlicer", "Tags": "|slic3r|prusaslicer|", "Answer": "

I found the solution :\nIt was the option "avoid crossing perimeters" that seems to do exactly the contrary of what it means.\nThank you all for your answers.

\n" }, { "Id": "14576", "CreationDate": "2020-10-12T21:23:57.327", "Body": "

I am currently building a Hypercube Evolution from a kit that was passed on to me. It uses a BIQU KFB2.0 control board and a 24 volt power supply. I have no prior experience in 3D printing.

\n

I was getting to the end of the build. Everything was working. I was testing every component as I added it using Pronterface. I have not covered the aluminium heated bed with anything as I did not want to cause problems for the induction sensor (used as Z Min endstop). The induction sensor has a 4\u00a0mm range for ferrous metal and less for aluminium. I was intending to print on blue tape.

\n

One website I visited suggested using feeler gauges as a better choice when levelling the bed. So I did. What I did not know is that both the hotend and the bed have power running to the outer surface. I found out when a feeler gauge touched the bed and the hotend at the same time. Since then, my LCD display gives the message "Thermal Runaway E1 press reset". I have pressed reset on the control board to no avail. Short press, long (5 and 10 seconds) and double press. None of which work. I have disconnected the hotend and it's thermistor from the control board. I have tried using another thermistor, but to no avail. Pronterface, when trying to connect, gives the following two lines:

\n

"Error: Thermal Runaway, system stopped! Heater_ID: 0\nError: Printer halted, kill() called!"

\n

I have disconnected all wires from the control board other than those leading to the LCD screen. The only power coming from the PSU. Still have the Thermal Runaway E1 error.

\n

I have also re-compiled Marlin 2.0.6.1 using Auto Build Marlin and uploaded to the board, still the error persists. Anybody any idea how to fix this problem?

\n", "Title": "Thermal Runaway E1", "Tags": "|troubleshooting|printer-building|hypercube|", "Answer": "

I found the solution. Controller board was broken, I had to order a new one.

\n" }, { "Id": "14580", "CreationDate": "2020-10-13T19:10:24.833", "Body": "

I've started learning G-code for 3D printing and I'm quite confused.

\n

I'm using an Ultimaker S5 printer and the firmware is : 5.5.12 - 20200226-UltiBot

\n
    \n
  1. Is there a difference between G1 F1200 X188 Y92 E0.01 and G1 X188 Y92 F1200 E0.01?
  2. \n
  3. Most of my code looks like: G1 X[coordinate] Y[coordinate] E[num] without Z!\nHow is this possible?! My model is a 3D-model so how come the letter Z appears about 5 times in tens of papers?
  4. \n
\n

Any help would be greatly appreciated!

\n", "Title": "Order in a line command in G-code", "Tags": "|g-code|3d-printerworks|", "Answer": "

I haven't used every flavor of gcode or firmware, but I've never seen the parameter order be important, provided that:

\n
    \n
  1. The G or M code was the initial entry; and

    \n
  2. \n
  3. Each parameter was clearly listed with one or more spaces between parameters; and

    \n
  4. \n
  5. No spaces were in between the parameter letter and the value.

    \n
  6. \n
\n

We use a custom version of Repetier firmware to run 3D printers and Fadal CNC machines, and in every use G1 F1200 X188 Y92 E0.01 would be evaluated and executed completely identically to G1 X188 Y92 F1200 E0.01

\n

Nearly all gocde variable values are persistent, so if Z remains unchanged because it is only specified once per layer, or if Z remains unchanged because it is explicitly specified (with the same value) on every line, there should be no difference in the behavior of the printer.

\n" }, { "Id": "14596", "CreationDate": "2020-10-15T13:41:46.213", "Body": "

I've a problem with my BQ i3 Hephestos.

\n

I've noticed that in the last few prints that I've experienced some temperature drops every now and then. Yesterday, after 3 of a 4 hours long print, the hotend completely cooled down (I have Octoprint and from the temperature graph it was visible the exponential decrease to 25\u00a0\u00b0C). Apart from realizing that, evidently, I have not any cold extrusion prevention set up on my Marlin firmware (I'll surely fix that asap), yesterday I've shut everything down with the idea to try again today.\nThis morning, though, I could not start my print because the heater didn't even warm up.

\n

After disassembling the hotend I've tried the following things:

\n\n

I've also noticed that while $12\\ V$ is the voltage that the printer sends to the resistance, measured when the resistance is not attached to it, if I repeat the measurement with the resistance attached to the printer I get a few $mV$. I really don't know whether it is the expected behavior or not.

\n
\n

Further inspection...

\n

Trying to inspect the cables better I've noticed a black spot on the connector of my Ramps 1.4 board (as you can see in the attached image).

\n

\"enter

\n

At this point I think I may have damaged something with too much current/too much heat. Too much current it seems strange to me because I have never changed anything in that sense, It is exactly the same as when I have bought the printer. It may be something related to the heat as the problem firstly occurred only on long prints (some hours).

\n

I don't know if this clarify something to you, but to me it says almost nothing...

\n
\n

UPDATE

\n

I've tried to attach the heater again to its original place and now it seems to work (in the sense that it gets hot). I've initially set the temperature to 60\u00a0\u00b0C, and it had no problem reaching and keeping it. Then I've tried to raise it to 180\u00a0\u00b0C but I had to shut down the heater after a few seconds because the cables became very hot and tender and it smelled of burnt

\n", "Title": "Hotend not heating", "Tags": "|extruder|electronics|bq-hephestos|", "Answer": "

In the end it turned to be that the RAMPS board was ruined.

\n

I bought a new RAMPS board and it worked immediately.

\n" }, { "Id": "14599", "CreationDate": "2020-10-16T09:08:05.743", "Body": "

I'm trying to print a small box with 1\u00a0mm thick walls.\nThe box has rounded corners.

\n

Cura slices the rounded corners with straight internal lines instead of rounded lines:

\n

\"Rounded

\n

The reason for these straight lines is that Cura is using infill to fill the walls, because it determines that there's not enough space in a 1\u00a0mm wall for three 0.4\u00a0mm passes.

\n

\"Rounded

\n

This causes the walls to have small imperfections in the final print.\nI was able to solve this by increasing the model wall thickness to 1.2\u00a0mm (to allow for 3 0.4\u00a0mm lines), but even then it wasn't doing it as a single continuous line, but instead breaking it into short lines at the corner:

\n

\"Rounded

\n

Is there a way to avoid this issue without increasing the wall thickness, and have 3 continuous lines?

\n", "Title": "How to slice rounded corners in thin walls with Cura", "Tags": "|ultimaker-cura|slicing|", "Answer": "

Contrary to the other answer, this is not (exclusively) a model / geometry issue.

\n

Frame challenge: There's no good / easy solution to this when using Cura, at least that I know of. Better slicers can easily handle this situation though.

\n

I replicated all the tests in SuperSlicer, which is a PrusaSlicer branch.\nAll tests were done with models with relatively high resolution:

\n

\"enter

\n

As well as a model which I created to replicate a horrible quality .stl:

\n

\"enter

\n

I checked the continuity of the added internal line with Simplify3D's g-code preview.

\n

For a 0.4 mm extrusion width and a 1 mm wall thickness, PrusaSlicer simply adds a 0.2 mm line in the middle. This line is completely uninterrupted:

\n

\"enter

\n

The same happens with the very rough .stl:

\n

\"enter

\n

The same thing happens when using a 1.2 mm wall thickness:\n\"enter\n\"enter

\n

Conclusion: You should not be adding esoteric "fixes" to your models to prop up terrible slicer performance.

\n" }, { "Id": "14601", "CreationDate": "2020-10-17T06:32:00.370", "Body": "

Although it seems this question has been answered before Connecting (Anet A8) 2004 display to MKS GEN L this one differs because the v2.0 board has no obvious AUX-2 connector.

\n

So the question is, how can I connect the Anet A8 LCD Panel (with buttons) to the MKS Gen L V2.0 board? Or where can I find the schematics?

\n", "Title": "Connecting (Anet A8) 2004 display to MKS GEN L V2.0", "Tags": "|lcd-screen|mks|display|", "Answer": "

It probably can be done, but, it requires some splicing of your LCD cables (or using some Dupont male-female cables) and altering of the firmware pins header files.

\n

The MKS GEN L v2.0 lacks the AUX-2 port that is present on the v1.0 controller boards. The reason for this is that they implemented support for more modern stepper drivers like the trinamic drivers that are capable of using UART to set stepper options.

\n

The old v1.0 pin layout is displayed below:\n\"MKS

\n

The new v2.0 pin layout is dislayed below:\n\"MKS

\n

The Anet A8 2004 graphical display (so-called ZONESTAR_LCD) uses (at least one as the buttons are suited with different resistors) analog pin that is used to detect which buttons are pushed.

\n
#elif ENABLED(ZONESTAR_LCD)\n  #define ADC_KEYPAD_PIN 12\n
\n

Pins A5 and A9-12, D40, D42 and D44 are used according to the pin layout.

\n

In the pins_RAMPS.h header file the display usage is coded as:

\n
#elif ENABLED(ZONESTAR_LCD)\n\n  #define LCD_PINS_RS                     64\n  #define LCD_PINS_ENABLE                 44\n  #define LCD_PINS_D4                     63\n  #define LCD_PINS_D5                     40\n  #define LCD_PINS_D6                     42\n  #define LCD_PINS_D7                     65\n
\n

In order for this to work on the MKS GEN L v2.0, you need to source for free pins that can mimic these pins and alter the header file and create a spliced LCD cable.

\n

It might be much easier to buy a new reprap discount display controller for about 10 bucks/euros and connect it to the dedicated EXP1/2 headers. Stay away from the Anet A6 graphical display.

\n" }, { "Id": "14607", "CreationDate": "2020-10-18T18:20:00.700", "Body": "

[SOLVED] on [EDIT 3]

\n

I just finished changing my i3 Mega's PTFE tube and brass nozzle, reassembled it, always taking care not to damage the heating element and thermistor wires.\nThen I check the temperature and it was cooling down, despite the set temperature being higher (210\u00a0\u00baC). I turned it off, let it cool down and turned it back on. I set it again to 210\u00a0\u00baC and it only reaches about 135\u00a0\u00baC-140\u00a0\u00baC, not going over it.\nSo I change the heating element and thermistor for fresh new ones (original ones, that came with the printer) and the same behavior takes place: not heating up beyond 140\u00a0\u00baC.

\n

Any ideas on why this is happening or how to fix it? I checked the connectors and they all seem to be fine. Cooling fans run normally as well.

\n

Printer: Anycubic i3 MEGA. All stock parts, except the new nozzle (standard brass nozzle) and the PTFE tube (a blue one with 1.9\u00a0mm internal diameter). I'm setting the temperatures through the printer's interface, as I have always done.

\n

[EDIT]

\n

I did check some stuff with the multimeter and what I got is that:

\n\n

Form that, I am guessing the problem is the current fed to the HE. Later today I will try to heat both the nozzle and the bed to the (safe) max temperatures and see if the bed heating is problematic too. If so, the problem should be related to current.

\n

If anyone has any other ideas, they are appreciated! :)

\n

[EDIT2]

\n

Heating up the print bed and the hotend at the same time didn't affect the bed at all. It heated up at the same pace as usual. Since the cables are fine, it shouldn't be a problem related to the power supply.

\n

So I'm gessing the problem is the motherboard (Trigorilla 1.1). It didn't seem to have any burned components at a glance but I'm gonna check with a multimeter.

\n

[EDIT3]

\n

Turns out I messed up the heatbreaker, so the heatsink was cooling the hotend way too much... This is one way of learning I need to be careful with the heatbreaker, I guess. I'll leave a solution here for the newbies like me that end up in a similar situation.

\n", "Title": "Anycubic i3 MEGA not heating past 140 \u00baC", "Tags": "|hotend|anycubic-i3-mega|", "Answer": "

There is a part of the extrusion ensemble called heatbreaker, responsible for sepparating the hot- and cold-ends. The problem is that I screwed the heatbreak up into the cold-end heatsinks and the heated block was now touching it. This made it cool off way too hard.

\n

The solution was switching to a new, undamaged cold+hotend.

\n" }, { "Id": "14614", "CreationDate": "2020-10-19T18:01:44.177", "Body": "

I would like to place the TMC drivers (the stepsticks modules, in fact) on the side of my controller board for easier cooling.

\n

What is the maximum length of the data wires? Specifically, Step and UART, the other ones should not pose any problem.

\n

In my case I would extend them by about 10 cm.

\n", "Title": "What is the maximum length of the TMC drivers signal wires?", "Tags": "|wiring|tmc2208|", "Answer": "

10\u00a0cm would be fine. That's the short answer, the real answer is that it depends on the UART speed and the exact conditions that you will be extending them. If you expect noise feedback from the motor coils inducing voltages into the UART lines, then perhaps they should be shorter. Keep the high current wires and the signal (UART) in separate places if possible. Also if you can use a shielded cable (e.g. and old shielded USB cable) then you could get away with a meter thereabout.

\n" }, { "Id": "14616", "CreationDate": "2020-10-19T20:12:14.943", "Body": "

I have a 3D print where there are 5 holes with a diameter of 4\u00a0mm in a cup, and I would like liquid to flow through all 5 holes at once while slowly draining the cup (by slowly I mean: just take a few seconds and not drain instantly). Therefore the holes can not be very large.

\n

When I fill water in the cup it drains fine until there is a small amount of water left, and then it just stops with a small layer of water flowing over the holes.

\n

I'm guessing it is due to surface tension and not enough pressure from water above to push the water through...

\n

Is there a hole design that fixes this problem? I don't know what to Google or if this is the right place to ask the question. It just takes too long to guess my way through and print every attempt at the right size or shape of hole (which I have done so far and still got nothing).

\n", "Title": "Water does not flow through 4 mm hole when there is a small amount, how to let water flow through small hole even when there is low pressure?", "Tags": "|3d-design|post-processing|", "Answer": "

If it is possible to add a downward pointing thin needle (hair, AWG-40 wire etc.) to the edge of (each) hole, do it. The droplets will smoothly glide along it to its end, where the needle circumference becomes so tiny that surface tension cannot hold the droplet anymore. Surface tension won't have a chance!

\n

With a very thin needle and funnel-like hydrophobic surface of the vessel, you may get rid of virtually any (microliter) amount of water. It can be hard to make it by 3D-printing, though.

\n" }, { "Id": "14620", "CreationDate": "2020-10-20T12:15:07.433", "Body": "

I'm using Cura for my 3D printing (an ultimaker S5 printer) and in order to understand 3D printing I'm trying to learn G-code.

\n

I've noticed that even when the model is full of circles, the G-code is full with the G1 command. I haven't seen the G2 or G3 command at all!\nHow could this be?

\n", "Title": "G2 / G3 command in Cura", "Tags": "|ultimaker-cura|diy-3d-printer|g-code|", "Answer": "

The .stl files most commonly used for printing do not have any circles in them at all!

\n

.stl files describe the surface of the things they represent with lots and lots of triangles:

\n

\"STL

\n

That also means that your slicer doesn't really know which parts of the model are supposed to be round (where G2 and G3 might be applicable).

\n

And, another consideration, though historical: Most 3D printers were (and are) based on relatively weak 8-bit microcontrollers. It's much easier for those microcontrollers to calculate the movements needed for the print if it's just straight lines, rather than elaborate curves.

\n

One issue that can arise with this is when your model is of high quality, and your slicers uses all of that quality - a simple circle can then become hundreds or thousands of tiny segments, which all need to be transmitted, parsed and executed - which may lead to stuttering.

\n" }, { "Id": "14626", "CreationDate": "2020-10-20T21:52:53.203", "Body": "

I'm trying to make a printer that will have a second extruder as backup for when the primary one clogs or runs out of filament. It is an ongoing project I'm doing to make a 3D printer as reliable as possible. How could you implement this.

\n", "Title": "Printer with two extruders but one as a backup", "Tags": "|diy-3d-printer|", "Answer": "

You could use a "filament runout sensor" and combine it with a "filament running sensor". If either fails the controller needs to be notified by a signal. If you configure the Configuration.h filament runout setup such that it doesn't call M600 but instead a series of codes to park tool 0, activate tool 1, home tool 1, prime tool 1, and commence printing, you are near a solution.

\n" }, { "Id": "14638", "CreationDate": "2020-10-21T19:45:02.617", "Body": "

I have been printing the same model once with Verbatim PLA and once with an ABS material from Switzerland. For both I was using two brand new 1\u00a0kg spools of filament. I was printing both at the same speed, same additions, same infill percentage, solid layers, solid wall count, etc. Literally everything the same except temperature.

\n

Simplify3D indicated 300\u00a0g for each print, however I already realized that all of Simplify3Ds estimates for time and material are - unlike Cura - very poor. However I was still kinda surprised when I checked the two spools after printing. Originally they had the same diameter of material and same depth. After the print the ABS spool looked almost the same as before the print, in terms of diameter it shrank less than 5\u00a0mm, however the PLA roll was definitely at least 1/3 empty. The difference was also confirmed by a scale. ABS only consumed like 220\u00a0g, while PLA was using a bit over 300\u00a0g. ABS surely did not look like more than 20\u00a0%, but maybe its inner diameter is larger than the PLA spool.

\n

Since I can order ABS considerably cheaper than PLA I was wondering if this is a common phenomenon. Googling for that leads to a million of pages outlining the differences between ABS and PLA where 999'999 of times it says that ABS is more prone to warping than PLA and the other one is about dimensional accuracy, but I could not find anything about material usage.

\n", "Title": "Are there differences in filament consumption between ABS and PLA", "Tags": "|pla|print-material|abs|", "Answer": "

I\u2019ve never actually printed anything other than PLA, but...

\n

Searching the web for filament density, I found this article , among others, that indicate that PLA is nearly 20\u00a0% denser than ABS, on average (1.24 vs 1.04 g/cm3). This indicates that a 1\u00a0kg reel of ABS will have 20\u00a0% more volume (therefore length) than a 1\u00a0kg reel of PLA.

\n

This does not quite explain the difference you see as 300\u00a0g vs 220\u00a0g is 36\u00a0% but maybe your measurement is off or the particular PLA mix you\u2019re using is slightly denser.

\n" }, { "Id": "14639", "CreationDate": "2020-10-21T21:06:31.103", "Body": "

I am using an Ender 3 and I have just upgraded to the Hero Me Gen 5 fan duct adapter. This has shifted the nozzle position. I also flashed some new Marlin 2 firmware and in the Hero Me manual it says to uncomment and change #define MANUAL_Y_HOME_POS -9 to suit this change in nozzle position.

\n

This has, however, not given me the right results. It did not center my prints but actually let the Y-axis grind upon going to Y235. Additionally, I noticed, that my printer must have been off-center from the beginning, because the X position of the nozzle has not changed due to the installation, but the printer homes to a point 5\u00a0mm inside the bed perimeters.

\n

So, if my printers endstops dictate an original home position of 3\u00a0mm inside the bed area for X and 15\u00a0mm outside the bed area for Y, how do I set this up correctly in Marlin 2?

\n

I think the relevant parts are

\n
// The size of the print bed\n#define X_BED_SIZE 235\n#define Y_BED_SIZE 235\n\n// Travel limits (mm) after homing, corresponding to endstop positions.\n#define X_MIN_POS 5\n#define Y_MIN_POS -15\n#define Z_MIN_POS 0\n#define X_MAX_POS 250\n#define Y_MAX_POS 240\n#define Z_MAX_POS 230\n...\n// Manually set the home position. Leave these undefined for automatic settings.\n// For DELTA this is the top-center of the Cartesian print volume.\n//#define MANUAL_X_HOME_POS 0\n//#define MANUAL_Y_HOME_POS -9\n//#define MANUAL_Z_HOME_POS 0\n
\n

This is my idea, I would let the manual home pos be commented out as I do not know what it does. However, I also want the software endstops to correcly work as to not let the printer destroy it aaaaand my BLTouch to probe the bed and not the air or my bed clips using UBL when doing 3-point probing. Would this be correct? Where do I have to look to let the printer consider my bed clips(only 10\u00a0mm on both ends of the y-axis side of the bed)?

\n", "Title": "How to set Home Offsets and Manual Home definitions in Marlin correctly?", "Tags": "|marlin|firmware|homing|endstop|", "Answer": "

Normally you wouldn't use the MANUAL_xyz_HOME_POS at all.

\n

Home your machine.\nUse a caliper or ruler to measure the distance between the nozzle and the edges of the print bed in both the X and Y direction.\nIf the nozzle was off the bed in both directions, input these measurements as negative X_MIN_POS and Y_MIN_POS values.\nIf it was over the bed in either of those directions, you'll need to input them as positive positions.

\n

That should result in your nozzle to be exactly on the edge of the bed after a

\n
G28 X Y\nG1 X0 Y0\n
\n

movement.

\n" }, { "Id": "14649", "CreationDate": "2020-10-23T21:40:56.030", "Body": "

I have this optocoupler.\n\"optocoupler:\"

\n

and this sensor.\n\"sensor:\"

\n

I want to wire them together and use them for auto bed leveling. Note, stock Ender 3 board (Creality v1) has 2 pins for its endstop, not 3, so this:\n\"dosent\nwon't work

\n

However, I found this diagram:\n\"possibly

\n

My optocoupler is rated for 12\u00a0V so I definitely wont attach 24\u00a0V to it. However, everything else seems good. So I could only replace the 24\u00a0V to 12\u00a0V and it would work?

\n", "Title": "How to wire an LJ18A3-8-Z/BX sensor with an optocoupler to a stock Ender 3?", "Tags": "|creality-ender-3|bed-leveling|electronics|wiring|inductive-sensor|", "Answer": "

Both diagrams are exactly the same, the difference is the +5\u00a0V that is not connected on the bottom diagram (displayed in white, labelled "not used"), but since your endstops do not have a +5\u00a0V pin in the socket (only ground, GND, and signal, SIG or S) you don't need to attach it. You should source the +5\u00a0V from elsewhere from your board. As the left and right side of the module are separated circuits, you need to power the board with +5\u00a0V.

\n

Read this answer for applying 24\u00a0V.

\n" }, { "Id": "14656", "CreationDate": "2020-10-25T01:06:43.827", "Body": "

While shopping for TMC2209 stepper drivers I found advertisements "SD2209". Is this the same thing?

\n", "Title": "Is SD2209 the same as TMC2209 stepper drivers?", "Tags": "|stepper-driver|tmc2209|", "Answer": "

From what I can tell, the SD2209 is not a clone of or another name for the TMC2209, but is a board with a TMC2209 on it setup to be used as a drop-in replacement for other stepper drivers. See e.g. this SD2209 a drop-in replacement for Pololu style drivers:

\n

\"enter

\n" }, { "Id": "14660", "CreationDate": "2020-10-25T20:46:17.717", "Body": "

I would like to create a case or a box which has two holes for incoming and exiting water. I want the box to be opened and closed. Therefore it is good to be something like a treasure box.

\n

Is there a way to design the lid of the box to prevent water from leaking around the areas where the box and the lid are meeting without using glue?

\n

\"enter

\n", "Title": "Is it possible to create a completely sealed box?", "Tags": "|water-resistance|", "Answer": "

I know this sounds obvious, but given the box has holes in the top \u2013 does it only need to be watertight when it's the right way up? If not, you could just do something like:

\n

\"enter

\n

Assuming that won't work, I think a really good seal would be hard with just something like PLA, because the printing texture alone means there will always be tiny gaps between the mating surfaces. In some cases, those grooves will actually act as capillaries (if you've ever tried to use a sharpie marker on a print, you can imagine what I mean).

\n

If the lid fits snugly, then just lining it with something like plumber's thread tape would probably do a good job.

\n

If it needs to work straight out of the printer, then I would try something like this (obviously I've exaggerated the detail):

\n

\"enter

\n

The idea being something like the plastic seal you see on some types of refrigerator door, which has a thin wall with a free edge that allows it to flex.

\n

NB in all these cases, sharp corners will cause problems \u2013\u00a0it will be easier to get a seal if you put a radius (say, 5mm) on the vertical edges of the box.

\n" }, { "Id": "14662", "CreationDate": "2020-10-26T13:07:35.723", "Body": "

I have an SKR PRO control board with a dead (shorted, it's burning hot) main processor. I ordered a new board, it was my mistake.

\n

The voltage regulators work, so I ordered a replacement STM32F407 processor from STM (a free sample actually) and I'll repair the board in my free time.

\n

How can I test all pins of the new board, so that I can ensure the board is working when I'll sell it/when I'll use it for my next project?

\n

I think that programmatically turning on and off each pin would be enough, then I would use an oscilloscope or a LED to verify the result. The pins which have special functions (heater, fan, MOSFET in general) would be tested accordingly, but I still need the pulsating input.

\n", "Title": "How to test all pins of a STM32 board?", "Tags": "|part-testing|", "Answer": "

A good starting point could be to use one of the two versions of the RAMPS1.4_TestCode.pde testcode, which is linked to by this thread on the RepRapWiki, RAMPS 1.4 test code:

\n
\n

Please note it will switch on and off your heated-bed and extruder hot-end too besides moving all the motors and flashing the leds. Disconnect them if you do not want any heating while testing.

\n
\n

There is a later version, in this post on the same thread, which also checks the individual stepper motor steps:

\n
#define X_STEP_PIN 54\n#define X_DIR_PIN 55\n#define X_ENABLE_PIN 38\n#define X_MIN_PIN 3\n#define X_MAX_PIN 2\n\n#define Y_STEP_PIN 60\n#define Y_DIR_PIN 61\n#define Y_ENABLE_PIN 56\n#define Y_MIN_PIN 14\n#define Y_MAX_PIN 15\n\n#define Z_STEP_PIN 46\n#define Z_DIR_PIN 48\n#define Z_ENABLE_PIN 62\n#define Z_MIN_PIN 18\n#define Z_MAX_PIN 19\n\n#define E_STEP_PIN 26\n#define E_DIR_PIN 28\n#define E_ENABLE_PIN 24\n\n#define Q_STEP_PIN 36\n#define Q_DIR_PIN 34\n#define Q_ENABLE_PIN 30\n\n#define SDPOWER -1\n#define SDSS 53\n#define LED_PIN 13\n\n#define FAN_PIN 9\n\n#define PS_ON_PIN 12\n#define KILL_PIN -1\n\n#define HEATER_0_PIN 10\n#define HEATER_1_PIN 8\n#define TEMP_0_PIN 13 // ANALOG NUMBERING\n#define TEMP_1_PIN 14 // ANALOG NUMBERING\n\nvoid setup() {\n  pinMode(FAN_PIN , OUTPUT);\n  pinMode(HEATER_0_PIN , OUTPUT);\n  pinMode(HEATER_1_PIN , OUTPUT);\n  pinMode(LED_PIN , OUTPUT);\n\n  pinMode(X_STEP_PIN , OUTPUT);\n  pinMode(X_DIR_PIN , OUTPUT);\n  pinMode(X_ENABLE_PIN , OUTPUT);\n\n  pinMode(Y_STEP_PIN , OUTPUT);\n  pinMode(Y_DIR_PIN , OUTPUT);\n  pinMode(Y_ENABLE_PIN , OUTPUT);\n\n  pinMode(Z_STEP_PIN , OUTPUT);\n  pinMode(Z_DIR_PIN , OUTPUT);\n  pinMode(Z_ENABLE_PIN , OUTPUT);\n\n  pinMode(E_STEP_PIN , OUTPUT);\n  pinMode(E_DIR_PIN , OUTPUT);\n  pinMode(E_ENABLE_PIN , OUTPUT);\n\n  pinMode(Q_STEP_PIN , OUTPUT);\n  pinMode(Q_DIR_PIN , OUTPUT);\n  pinMode(Q_ENABLE_PIN , OUTPUT);\n\n  digitalWrite(X_ENABLE_PIN , LOW);\n  digitalWrite(Y_ENABLE_PIN , LOW);\n  digitalWrite(Z_ENABLE_PIN , LOW);\n  digitalWrite(E_ENABLE_PIN , LOW);\n  digitalWrite(Q_ENABLE_PIN , LOW);\n}\n\n\nvoid loop () {\n\n  // if (millis() %1000 <500)\n  // digitalWrite(LED_PIN, HIGH);\n  // else\n  // digitalWrite(LED_PIN, LOW);\n\n  // if (millis() %1000 <300) {\n  // digitalWrite(HEATER_0_PIN, HIGH);\n  // digitalWrite(HEATER_1_PIN, LOW);\n  // digitalWrite(FAN_PIN, LOW);\n  // } else if (millis() %1000 <600) {\n  // digitalWrite(HEATER_0_PIN, LOW);\n  // digitalWrite(HEATER_1_PIN, HIGH);\n  // digitalWrite(FAN_PIN, LOW);\n  // } else {\n  // digitalWrite(HEATER_0_PIN, LOW);\n  // digitalWrite(HEATER_1_PIN, LOW);\n  // digitalWrite(FAN_PIN, HIGH);\n  // }\n\n\n\n  if ( millis() % 60000 < 30000) {\n    digitalWrite(X_DIR_PIN , HIGH);\n    digitalWrite(Y_DIR_PIN , HIGH);\n    digitalWrite(Z_DIR_PIN , HIGH);\n    digitalWrite(E_DIR_PIN , HIGH);\n    digitalWrite(Q_DIR_PIN , HIGH);\n    digitalWrite(HEATER_0_PIN, LOW);\n    digitalWrite(FAN_PIN, HIGH);\n    digitalWrite(HEATER_1_PIN, LOW);\n    digitalWrite(LED_PIN, HIGH);\n\n  }\n  else {\n    digitalWrite(X_DIR_PIN , LOW);\n    digitalWrite(Y_DIR_PIN , LOW);\n    digitalWrite(Z_DIR_PIN , LOW);\n    digitalWrite(E_DIR_PIN , LOW);\n    digitalWrite(Q_DIR_PIN , LOW);\n    digitalWrite(HEATER_0_PIN, HIGH);\n    digitalWrite(FAN_PIN, LOW);\n    digitalWrite(HEATER_1_PIN, HIGH);\n    digitalWrite(LED_PIN, LOW);\n\n  }\n\n  digitalWrite(X_STEP_PIN , LOW);\n  digitalWrite(Y_STEP_PIN , LOW);\n  digitalWrite(Z_STEP_PIN , LOW);\n  digitalWrite(E_STEP_PIN , LOW);\n  digitalWrite(Q_STEP_PIN , LOW);\n\n  delay (200);\n\n  digitalWrite(X_STEP_PIN , HIGH);\n  digitalWrite(Y_STEP_PIN , HIGH);\n  digitalWrite(Z_STEP_PIN , HIGH);\n  digitalWrite(E_STEP_PIN , HIGH);\n  digitalWrite(Q_STEP_PIN , HIGH);\n\n  delay (200);\n\n}\n\n
\n

As your board isn't a RAMPS you will probably have to change some of the #defines, to correspond with your board, but that should just be a simple search-and-replace exercise.

\n" }, { "Id": "14678", "CreationDate": "2020-10-27T12:38:11.140", "Body": "

I'm working on a project for my masters where I'm trying to detect print errors using object detection and I want to be able to pause the printer when a potential fault is detected. However I can't seem to figure out the correct way to send G-code to my printer from a Python script. Does anyone happen to know how this works?

\n", "Title": "How do you send G-code from a USB port with Python?", "Tags": "|g-code|usb|python|", "Answer": "

The process is pretty straightforward, you open a serial connection and send the instructions over the serial connection. The printer reports "OK" when received (not when the command has been executed).

\n

You can use pySerial to connect to a USB port with the correct speed, then send the command you want to execute (include the newline char).

\n

You need to be aware that commands are buffered, if the buffer is full, the printer sends "OK" when there is a spot free in the buffer.

\n

See this question on a sister SE.

\n" }, { "Id": "14682", "CreationDate": "2020-10-27T22:38:13.310", "Body": "

I have a FlashForge Creator Pro, couple of years old. I'm looking to replace one or both of the heating blocks, as they're getting old and pretty gunked up. Instead of buying two blocks at \\$30 each, I thought I could take some aluminum and make two on my CNC mill. The only thing - are there any manufacturer provided dimensions? I could probably get by measuring with calipers, but I'd prefer the exact specifications.

\n", "Title": "What are the dimensions of the FlashForge Creator Pro heating block?", "Tags": "|flashforge-creator|flashforge|", "Answer": "

The FlashForge Creator Pro uses a MK10 hotend:

\n

\"enter

\n

The Micro Swiss MK10 All Metal Hotend Kit for instance is reported to be a drop-in replacement for the FlashForge Creator Pro, it uses M7 threads.

\n

The heater block for an MK10 has a threaded hole for the thermistor (M4), but some use a machined hole for PT100/3\u00a0mm thermistor cartridges. The block measures around 20\u00a0x\u00a020\u00a013\u00a0mm. The images below should give you enough information to create your custom blocks. However, there appear to be 3rd supplier after-market blocks available on on-line webshops and market places

\n

\"enter

\n

\"enter

\n

\"enter

\n

The Flashforge original spare part heater blocks show that they are slightly different (the top image shows symmetrical heater blocks while the spare parts are not symmetrical), so above drawings and caliper measurements should give you enough information to reproduce the heater block.

\n

\"enter

\n
\n

It would be a great addition if you added your own answer once you measured your blocks and created some drawings!

\n" }, { "Id": "14686", "CreationDate": "2020-10-28T14:44:58.450", "Body": "

Question background:

\n

I am building an extruder / hotend combination to be used with 2.85\u00a0mm filaments.

\n

The extruder I chose is QR Extruder from Bondtech which comes with a stepper motor, with an integrated planetary gear system and a reduction of 5.18 to 1.

\n

The motor's specification states that it has a backlash of less than 1 degree.

\n

I have contacted the manufacturer but I could not get a more specific number.

\n

Here is what I'd like to know (i.e., my question clarified) :

\n\n", "Title": "What are the effects of backlash from a geared stepper motor used to drive a filament extruder?", "Tags": "|extruder|stepper|motor|extruder-driver|drive-gear|", "Answer": "

During normal extrusion backlash has no effect.

\n

During retraction you can perfectly compensate by increasing retraction length slightly.

\n

Backlash cannot be taken into account for pressure advance, but unless it's a lot, it should not cause issues: pressure compensation is a second order effect and does not need to be tuned super accurately to produce results.

\n

One degree does not seem to be enough to cause problems.

\n" }, { "Id": "14702", "CreationDate": "2020-10-31T15:18:29.957", "Body": "

My prints have recently started to come out with a serious skew along the Y axis. This occurs regardless of printing parameters such as speed/acceleration. I don't know the exact cause yet, can someone help me resolve this? What could have caused this?

\n

\"enter

\n

Please don't mind the stringing/blobs, I'm currently running out of filament and I was forced to use an old PET-G spool damaged by moisture.

\n", "Title": "Prusa MK3S Y-skewed prints", "Tags": "|print-quality|prusa-i3|", "Answer": "

Apparently, the possible cause mentioned by @R.. GitHub STOP HELPING ICE turned out to be true. The frame was noticeably skewed in the Y-axis, which is why the prints were leaning in that direction.

\n

I replaced the Z-axis top parts with a 3D printed modified version of them (what came with the printer is orange, what I printed is yellow.) The modified version is slightly longer so as to move the steel rods (ones that hold the X-axis) slightly towards the front of the printer (in the opposite direction to the skew)

\n

This turned out to work perfectly, the skew is still slightly present, but it has been substantially mitigated and I can fix it completely by adjusting the length of the printed parts.

\n

\"enter

\n

\"enter

\n" }, { "Id": "14703", "CreationDate": "2020-11-01T04:34:50.413", "Body": "

I am attempting to make my Ender 3 Pro print significantly faster than it is intended it I fear.

\n

I hear a clicking noise coming from the extruder above 130 mm/s in Cura when it is going in long straight lines (I think my acceleration settings prevent it on shorter lines). I have seen this before when the bed is too close to the nozzle but I have auto bed leveling now and it is happening on all layers.

\n

I have a standard extruder + Bowden tube setup. What can I do to make things go faster? I am less concerned about the quality of the print and more concerned about the speed.

\n

\"Front

\n

\"Top

\n", "Title": "Ender 3 Extruder Speed / Upgrade", "Tags": "|creality-ender-3|extruder|speed|bowden|", "Answer": "

You're likely hitting the thermal limit of your hot end.

\n

The standard Creality hotend used in your printers maxes out at around 8-12 mm\u00b3/s.\nTo calculate your current volumetric speed, you can simply calculate

\n

speed * layer height * extrusion width

\n

With an 0.45 mm extrusion width and 0.2 mm layer heights, printing at 130 mm/s would put you at around 11,7 mm\u00b3/s - very high for the standard hot end.

\n

Increasing temperatures as suggested in the other answer will help, but might reduce print quality on other layers / intricate structures.

\n

The "proper" solution would be a Volcano or SuperVolcano hot end, or any of the other "high flow" alternatives.

\n" }, { "Id": "14706", "CreationDate": "2020-11-01T06:25:00.680", "Body": "

My build with the Hypercube Evolution continues with a replacement BIQI KFB2.0 board. I am not at the printing stage yet. This is due to my latest problem.

\n

The kit contains a \u201cno-name\u201d hotend, complete with round finned heatsink and nozzle. Looks a bit like a V5 J Head Hot End.

\n

Everything is wired up. A 24\u00a0V power supply is being used. The printer is connected to Pronterface via USB. Using Pronterface, I set the hotend temperature to 70\u00a0\u00b0C. The KFB2.0 board lights a bright red LED when power is being sent to the hotend (and/or the heated bed). On initial heating, the red LED is on until the hotend temperature reaches around 65\u00a0\u00b0C and then turns off. The temperature continues to climb to around 120 to 125\u00a0\u00b0C, then starts falling. At 69/70\u00a0\u00b0C the red LED comes on again for about two seconds, the temperature continues to climb after the red LED is off to around 86\u00a0\u00b0C. It then cycles around the 69/70 to 86\u00a0\u00b0C position.

\n

I have removed the thermistor to check its resistance. Reads greater than 100\u00a0k\u03a9. When in situ in the hotend, it was reading around 85\u00a0k&ohm, but a bit of Kapton tape sorted that. I was now reading 100\u00a0k&ohm with the thermistor in place. I then used Pronterface to set the temperature again. This time with a multimeter hooked up to the hotend terminals on the board. It confirmed that the bright red LED was sending 24\u00a0V and nothing when not on, so the hotend is recording a temperature increase from around 65 to 125\u00a0\u00b0C with no power. But, I now got something different when the temperature fell back to 69/70\u00a0\u00b0C. The red LED flickered for about two seconds and the multimeter showed readings fluctuating between 0.1 and 4.2\u00a0V. From this point the hotend kept the correct temperature, even after I had removed the multimeter.

\n

Has anybody any idea why the temperature is going so high and what can I do to fix it?

\n", "Title": "New Build. Hotend Overheating. Not Printing Yet", "Tags": "|hotend|temperature|", "Answer": "

3.9 ohms is a 12V heater. It won't work on 24V.

\n" }, { "Id": "14719", "CreationDate": "2020-11-03T13:56:33.820", "Body": "

I'm building a weather-proof enclosure for some electronics. It's expected to stay exposed outside all the time so I'm worried about its resistance:

\n\n", "Title": "Post-processing and filaments for weather-proof electronic enclosure", "Tags": "|post-processing|filament-choice|water-resistance|", "Answer": "

There are already some questions about the choice of filaments for outdoor use:

\n

Which is more durable to sunlight/weather - PLA, ABS or PETG

\n

3d printing for outdoor use: what types of filament are most weather resistant?

\n

PLA use outdoors?

\n

It may very well be that if you print in PETG you won't need any postprocessing.

\n

If you still want to, you could try to paint or spray the outside with some bitumen spray for car underbody.

\n

I think that the only part which actually needs postprocessing is the spot with exposed moving parts.

\n

It's difficult or impossible to find a solution without having an idea of what we are talking about when you say "some moving parts exposed", but in general this is solved first of all by adapting the design so that water wont' be an issue: rain should flow away without getting inside, and spatwater should be reduced so that it will evaporate when rain stops.

\n

If you only have an exposed pin for rotating parts which are inside, you could also pour some wax: it will seal it while allowing movement. The same bitumen spray may help.

\n" }, { "Id": "14725", "CreationDate": "2020-11-04T09:12:15.110", "Body": "

I couldn't find a reliable source listing the improvements of BLtouch 3.1 vs 3.0.

\n

I can see that BLtouch 3.0 is sold (original) for about 20 Euro, while 3.1 costs about 30 Euro. The difference is significant.

\n

What are the improvements?

\n", "Title": "What are the changes between BLtouch 3.0 and 3.1?", "Tags": "|bltouch|", "Answer": "

Best reference would be the original equipment manufacturer (Antclabs). It appears their sensor has had many versions:

\n

\"enter

\n

So, from 3.0 to 3.1 is a software upgrade. Reported highlights are:

\n
\n

\u2605 Smart V3.0 (Mar. 2019 ~ ) : S10, S60, S90, S120, S140, S150, S160,\nPWM Wiring defect indication(Blue LED), QR, Molex 1.25, Plastic\nPush-pin(PC), Power Wiring defect indication(Red LED)

\n

\u2605 Smart V3.1 (Mar. 2019 ~ ) : S10, S60, S90, S120, S130, S140, S150,\nS160, PWM Wiring defect indication(Blue LED), QR, Molex 1.25, Plastic\nPush-pin(PC), Power Wiring defect indication(Red LED)

\n
\n

If you look at the manuals you will see the differences:

\n

V3.0\n\"enter

\n

V3.1\n\"enter

\n

As can be seen, timings are different and more PWM position G-code options are available in the V3.1.

\n" }, { "Id": "14726", "CreationDate": "2020-11-04T09:13:31.417", "Body": "

I have looked at many tutorials already but I am not sure about how I should wire this properly. I bought a BLTouch V3.1 together with an extension cable

\n

\"enter

\n

Now, the thing is, with that extension cable you get one set of wires for the servo in a Dupont connection (brown, red, yellow) and one for the Z-min in a JST connection (black, white).

\n

The JST connection actually has room for three cables, and when holding it the way you can plug it into the Anet A6 you have the following order: Empty, Black, White. However, IIRC the input Z-min input on the Anet wants the following order: GND, VCC, Z-min.

\n

So if I would use the cable like this. then I would connect the black wire of the BLTouch to the VCC. Will this cause any trouble with my BLtouch?\nBecause I think it does, but then again, why would you sell such a wire.

\n", "Title": "Wiring BLTouch V3.1 on Anet A6 and the JST connector polarity", "Tags": "|bltouch|anet-a6|", "Answer": "

Black is GND and should not be connected to VCC.

\n

Note that you can simply pop out the metal parts (use a pin to press a small clip) from the connector and rearrange how you want them to be.

\n

Note that most printer controller boards have the GND of all the endstops connected, so technically you don't need the black wire at all. I make my own extension cables (from 4 wire cable) where the brown and black are connected at the sensor side and a single white wire is used for the Z-min endstop.

\n" }, { "Id": "14730", "CreationDate": "2020-11-04T14:59:36.520", "Body": "

Painters tape is a common quick fix to adhesion problems, and we know it doesn't need to be blue, but it is often recommended not to do. Why? When is painters tape best used or skipped upon?

\n", "Title": "What are the pros and cons of Painters tape?", "Tags": "|adhesion|knowledgebase|", "Answer": "

Read First

\n\n

Pro

\n\n

Cons

\n\n" }, { "Id": "14737", "CreationDate": "2020-11-06T14:43:31.293", "Body": "

I've started assembling a Sapphire Plus CoreXY printer and produced a nice calibration cube (no layer shift, although a bit of over extrusion in the corner). Then, I started printing a 3D benchy, and got this:\n\"Side\n\"Top

\n

I've checked belt tension and it seems correct (I've printed this belt gauge with another printer and checked that both belts are tensionned the same), with a bass sound when pluked.

\n

I've checked both Z-axis lead screw and it's not 100% perfect (one lead screw has a 1mm deviation on one side when rolled over my desk). When the bed goes down, the effect of the lead screw is only visible when the bed is half the height or more, so it shouldn't matter here.

\n

Also, I had multiple filament breaking before entering the extruder in other prints, so I'm not 100% confident with the extruder.

\n

The machine is a Core XY printer with a Bowden extruder, with linear rails for all axis, and 2 lead screws for Z axis (and 2 stepper motors attached by the same belt). It's level, on a concrete base.

\n", "Title": "Layer Shift with a new CoreXY printer", "Tags": "|print-quality|corexy|", "Answer": "

Shifts in a diagonal direction in CoreXY indicate loss of synch between the two belts.

\n

If it's the belt very loose skipping, you may not hear much but such a loose belt is easy to notice.

\n

If it's the belt a bit loose skipping, you would hear relatively loud noises which make very clear what's happening.

\n

If there are no particular noises, it's the motor skipping steps. One motor move along the X+Y diagonal, one along the X-Y diagonal, so you know which one it is.

\n

If you have missed steps it could be because of excessive friction (maybe one motor is mounted tilted, or it is damaged), or because of excessive acceleration setting, which the motor cannot handle, or because of too low current.

\n

If you decide to increase the current be aware that it may work on simple tests, but during longer prints the stepper driver may overheat and lose steps for self protection.

\n

To check what acceleration and/or current you actually need per each motor and to compare whether motors are significantly different from each other, try this.

\n

Without printing, you set the maximum acceleration relatively high (5000 mm/s^2).

\n

Place a paper square (relatively big) on the printing area, parallel to the axes and tape it.

\n

Align the printing head to one corner of the paper square.

\n

Give a G0 command to move the printing head to the opposite corner of the square (G0 F600 X... Y...) and check that the alignment is correct. Also, only one motor should be moving.

\n

Bring back the head to the opposite corner.

\n

Set the feed rate to high values, like F9000 (150 mm/s) and move the head again. If it reaches the correct spot, the current of the motor is sufficient for that acceleration.

\n

Try for the other diagonal of the square you taped to test the other motor.

\n

You can now reduce the current of the motors which successfully passed the test. If you have Klipper it's super easy and requires only the command "SET_TMC_CURRENT STEPPER=name CURRENT=amps", with Marlin I don't know.

\n

See at what current each stepper fails the test, then decide whether to reduce the acceleration (5000 mm/s^2 is anyway probably too high for your printer frame, you would get artifacts like ringing) or to dial back up the current. I would say that 20% above the current required to pass the test is enough. More than that and you are just overheating the TMC2008.

\n" }, { "Id": "14738", "CreationDate": "2020-11-06T20:23:45.707", "Body": "

I have a Creality CR-10S Pro V2, I've been printing regularly for one month and then, suddenly, I started to notice waves on the Z axis. They're already noticeable with the test cube but deviation increases with the height and objects are ruined (eSUN Gray PETG 20 mm test cube, printed at VERY low speed):

\n

\"PET

\n

It was my first attempt to print with PETG then I thought that could be the problem, I cleaned the nozzle (also using eSUN cleaning filament) and switched back to PLA+. The problem is even more evident (eSUN cold white PLA+):

\n

\"PLA+

\n

Note how the "wave" pattern matches between the two test cubes. Even if I was printing from less than a month (and "just" 5/6 hours per day, not every day) I thought it might be time for some maintenance:

\n\n

I've got this:

\n

\"second

\n

I then moved the printer to the ground and tried again, it helped (possibly a lot) and printing PLA I get a cube almost like the PETG one in the first picture.

\n

I noticed the both threaded rods have some freedom of movement in the Y direction (not the rod itself but the bearing ball they go through at the very top of the printer can move within its plastic enclosure); 0.5/1 mm for the left rod and 1.5/2 mm for the right rod. I don't know if it's an allowed tolerance or the sign that something is wrong (and what).

\n

Searching on-line I read as many opinions as many posts I can find; I wouldn't go and buy replacement parts until I randomly solve the problem. It's frustrating enough to have to deal with it (and a huge amount of material wasted in failed prints and tests) after less than one month of activity...

\n

Update: I secured the two rods using two 0.2 metallic feelers and nothing changed. I still have this shift within the first centimetre (it changes position but it does not go away).

\n

Update 2: everything seems perfectly square (I can't swear this for the rods), cleaned and tight. Everything moves smoothly, and both bed and horizontal bar are fairly leveled. Effect is slightly less noticeable but obviously present.

\n

One concerning point: travel speed was 150 mm/s, if I slow down to 130 mm/s then I can see a slightly less sloped shift; that makes me think that something could be vibrating when moving: top and bottom seem straight and they're printed at a much lower speed (20 mm/s vs 50 mm/s for PLA+, 20 mm/s - 15 mm/s for first layer - vs 30 mm/s for the test cube in PETG; travel speed was 150 mm/s for all of them). I guess that to reduce the printing speed to 10 mm/s for the entire object might help but I wouldn't call it a solution...

\n", "Title": "Wave shift pattern on the Z axis", "Tags": "|z-axis|creality-cr-10|layer-shifting|", "Answer": "

This is classic Layer Shift in the Y plane. This happens in one of three cases:

\n\n

The belt should ring and resonate when tapped, giving a nice tune. I haven't really tuned mines, but my ender3 (250 mm bed movement in total) has about a G major.

\n

The bed movement should be, with motors off, easy and smooth. If there are spots where it hangs, you might need to rearrange cabling or check if there is a bad spot on the linear motion system. If the movement is hard, you might need to lessen the pressure. Since the CR10 is pretty much the same as the ender3, the eccentric nuts should be turned so the bed does not wobble or move side to side but not to a degree the friction slows the movement. Carefully tweak here, use increments of about 10\u00b0 till the layer shift vanishes.

\n

Be careful where you place your printer: cabling can snag on other items or things can brush onto the bed. That should not happen. On the Z-axis it can unplug the X-axis motors and extruder, or in worst case, yank at the heater cartridge and damage it.

\n" }, { "Id": "14742", "CreationDate": "2020-11-07T15:37:46.803", "Body": "

I'm porting Marlin to my Qidi Tech 1 and I'm trying to figure out what part this is. The markings on it read "AAJU". I'm pretty sure its the digital pot to adjust the stepper drivers as there isn't a small adjustable pot on them.

\n

\"enter

\n", "Title": "Which digipot is this with the markings \"AAJU\"", "Tags": "|marlin|firmware|stepper-driver|", "Answer": "

The picture shows, that the marked chip is next to the letters U8 to U12. U is commonly used for Ineparable Assembies, but there are several meanings possible according to the ANSI/IEEE Std 315 (1975). So, let's assume for the moment that this is Assembly 8 to 12.

\n

Note that the X-pot is marked to be down on the right side of each section, right under the sockets, and next to R103 to R107.

\n

The Marking AAJU might belong, according to this reference, to two chips. Both are Voltage uP Supervisory Circuit

\n
AAJU  MAX6339AUT    MAX6339  SOT23-6  Quad Voltage uP Supervisory Circuit\nAAJU  MAX6726KASYD3 MAX6726  SOT23-8  Triple Ultra-Low-Voltage uP Supervisory Circuit\n
\n

The pinout and look of the first chip is very much in line with what I see from the photo:

\n

\"enter\"enter

\n

Sadly, neither a user manual for the board nor the pinout of it seems to be available at the time. My best guess based on these facts is, that it is the Quad Voltage supervisory chip, as the MAX6726 has 8 legs, as the -8 in the entry SOT23-8 indicates.

\n" }, { "Id": "14745", "CreationDate": "2020-11-07T23:55:52.110", "Body": "

Assuming a Cartesian printer with a belt and smooth rod design in which one axis moves another (i.e. the X-axis rails 'sit' on the Y-axis rails), what are the main considerations in sizing the rods and belts? For example, given a base design using 8\u00a0mm diameter rods and 6\u00a0mm belts (assume these are the limiting factors of the printer and that the frame, etc. can handle whatever you throw at it), what is roughly the maximum load, print speed and build size that this should be expected to support? If you were to increase the rod diameters to 10\u00a0mm or even 12\u00a0mm on one or both axes (assume the steppers could handle the increased load), what would the increased rigidity buy you in terms of maximum speed and/or build size and would 6\u00a0mm belts still be appropriate? Ballpark calculations or rules of thumb are fine as I understand the variables are likely not trivial and am looking more for a rough range of guidance to understand the trade-offs involved.

\n", "Title": "Sizing belts and rods for a Cartesian printer", "Tags": "|diy-3d-printer|", "Answer": "

The 3D printing revolution started out with the idea/community project to build self-replicating machines:

\n
\n

RepRap was the first of the low-cost 3D printers, and the RepRap Project started the open-source 3D printer revolution. It has become the most widely-used 3D printer among the global members of the Maker Community.\n(From RepRap.org)

\n
\n

The main aspects for this project was to build self-replicating machines from cheap and "simple" available materials and making them freely available for the benefit of everyone. The rod solution is a simple and affordable solution for linear motion with fair tolerances.

\n

As far as the rigidity of the X carriage, rods aren't the best solution, increasing rod diameter will surely increase the stiffness, but it will be smaller than a design that uses a (quality) linear rail, these are much stiffer. Note that instead of steel, carbon rods can be used, these are stiff and light and reduce the weight of the carriage, allowing for higher acceleration and speed. The solution of using rods is mainly based on availability and being a cheap solution.

\n

Do note that rods flex much more than linear rails, even when you increase the diameter, this will limit the length of the X-axis.

\n

From all3Dp:

\n
\n

Any decent rail will have far less give. In terms of 3D printing, this leads to greatly reduced backlash and ringing artifacts, yielding cleaner print surfaces and edges.

\n
\n

Belts allow the rotational motion to be transferred into a linear motion. Belt width depends on the force (carriage weight) you need to move. Same as for rods, an increased height will allow for a larger load to be driven/moved, but for a fixed load, the increasing height allows for less stretch and thus less printing defects. Note the different types of belts are available, all having their own stiffness and load bearing capacity.

\n" }, { "Id": "14782", "CreationDate": "2020-11-12T12:36:01.050", "Body": "

Marlin has an option to fade out the amount of backlash correction, given as BACKLASH_CORRECTION. It is also available in GCode as M425 [F<value>], where 0.0 = none and 1.0 = 100%.

\n

Assuming backlash correction is otherwise well tuned at 100%, when is useful to reduce its effect? Why not always keep it at 100%?

\n", "Title": "What is the purpose of partial (<100%) backlash correction?", "Tags": "|marlin|calibration|", "Answer": "

The developers explained the use of the M425 code themselves.

\n
\n

We devised a routine for measuring Z backlash automatically during G29\nand found that software backlash compensation does wonders for the\nfirst layer. However, this comes at the expense of artifacts on the\nrest of the print. In particular, any rapid motions of the motor to\ntry to take up any backlash will inevitably create a small pause and\nvibration, leading to a seam in the print. We devised a smoothing\nalgorithm that allowed backlash correction to be gradually applied\nover a distance, this eliminated any harsh transitions, which again\nlead to a huge improvement. Alas, we learned that the feature was very\nsensitive to the variances in the printer build, working amazingly\nwell in some cases, but leading to a degraded quality other printers.\nThis perhaps is a feature that could be used by someone who wished to\nhand tune the algorithm, but not something we could use in a mass\nproduced printer.

\n

...

\n

"M425 F" sets a value from 0 to 1 which is multiplied by the backlash\ndistance. This command is meant for use by the slicer, and allows it\nto "fade" away the backlash compensation gradually over several\nlayers, or to turn it off completely after the first layer (with a\n"M425 F0").

\n
\n" }, { "Id": "14783", "CreationDate": "2020-11-12T17:16:41.670", "Body": "

What is the best technique to improve removing PETG from the print bed?

\n

PETG is known for sticking well to the bed; so well that it does not need a heated bed. If it weren\u2019t for printing other materials, I would try printing directly on the glass bed instead of a build surface. However, the build surface may help protect the glass from excessive force.

\n

To reduce how strong the PETG sticks to the build surface I\u2019ve reduced the bed temperature to 35\u00a0\u00b0C. It is easier to remove the skirt and brim by heating the bed up to 90\u00a0\u00b0C, so that the PETG is soft. However, if one tries to remove the printed object at 90\u00a0\u00b0C, the printed object is like to distort and ruin.

\n

I did sharpen the putty knife on the top side only. This helps, but it still takes excessive time to remove the print job and clear the bed. The excessive force on the bed from the putty knife seems to increase the need to level the bed. It also seems to affect the flatness of the build surface.

\n

I\u2019m looking for techniques to make it easier and less time consuming to remove the PETG print from the bed.

\n

P.S. Additions after input from answers.

\n
    \n
  1. Added keeping the initial Z-height high enough to keep adhesion to the bed from being too high. This resolved the adhesion to the bed issue, but seemed to make adhesion between layers worse.
  2. \n
  3. Set hot end to 230\u00b0C for first layer, then increased hot end temperature to 250\u00b0C for better adhesion between layers.
  4. \n
\n", "Title": "What is the best technique to improve removing PETG from the print bed?", "Tags": "|petg|bed|", "Answer": "

Make sure to use a flexable print bed - when printing is finished wait a few minutes so it can cool down, take the print bed off of the build plate and flex it so the print just pops off for the most part. I also use a laminated PEI sheet over the bed plate

\n" }, { "Id": "14797", "CreationDate": "2020-11-13T15:36:17.320", "Body": "

I am trying to print something that might take 15 hours. I don't want to risk my printer so if I print for 15 hours, what is the worst that can happen? So far, I haven't printed anything for more than 5 hours.

\n", "Title": "Long Print with Lotmaxx SC-10", "Tags": "|creality-ender-3|safety|", "Answer": "

As long as your printer is well designed as far as safety and you didn't get one without thermal runaway protection you should be fine. Worse case scenario: you have a big mess to clean up and a lot of wasted filament.

\n" }, { "Id": "14799", "CreationDate": "2020-11-13T18:20:19.687", "Body": "

I want to check how stable the output of the bedleveling protocol G29 was by running it multiple times. When running it four times, I got this as subsequent output

\n
Recv: Bilinear Leveling Grid:\nRecv:       0      1      2\nRecv:  0 -0.056 -0.475 -0.605\nRecv:  1 +0.124 -0.349 -0.595\nRecv:  2 -0.023 -0.344 -0.673\nRecv:  \n\nRecv:       0      1      2\nRecv:  0 +0.218 -0.183 -0.325\nRecv:  1 +0.399 -0.073 -0.328\nRecv:  2 +0.251 -0.072 -0.399\nRecv: \n\nRecv:       0      1      2\nRecv:  0 +0.498 +0.090 -0.049\nRecv:  1 +0.677 +0.211 -0.038\nRecv:  2 +0.531 +0.217 -0.117\n\nRecv: Bilinear Leveling Grid:\nRecv:       0      1      2\nRecv:  0 +0.772 +0.368 +0.222\nRecv:  1 +0.949 +0.482 +0.231\nRecv:  2 +0.806 +0.495 +0.154\n
\n

Which are quite unstable, and strange results. It seems as if there is an increment of around +0.27/0.28 for each probe point in the next iteration. How can this happen? What are the right bed level settings right now?

\n\n
\n

For reference, here are the Configuration.h and the Configuration_adv.h.

\n", "Title": "Output of G29 bed leveling, Marlin 2.0", "Tags": "|bed-leveling|bltouch|anet-a6|", "Answer": "

Actually the sensor reads the bed correctly in terms of shape, not in actual reproducible distance.

\n

Plotting the grids:

\n

\"enter\"enter\n\"enter\"enter

\n

Results in pretty much the same shapes, so, when correcting for the maximum displacement and plotting all results in a single graph gives:

\n

\"enter

\n

So the sensor does seem to produce reproducible bed geometry, but not the actual values.

\n

Misinterpreting your initial question I assumed a default Anet sensor, this official ROKO SN04-N sensor is reported to not be very accurate and has a maximum hysteresis of 10\u00a0% error of the detecting distance, which is 10\u00a0% of 5\u00a0mm (so \u00b1\u00a00.5\u00a0mm). The overall maximum measurement is in grid 4 with a value of 0.949\u00a0mm and in grid 1 with a value of 0.124\u00a0mm; 0.949\u00a0mm\u00a0-\u00a00.124\u00a0mm = 0.825\u00a0mm is well within the \u00b1\u00a00.5\u00a0mm. This seems odd as the hysteresis should be in play on all measurements as the probe goes up and down. I've used this sensor myself, but replaced it for a more reliable sensor like the LJ18A3-8-Z/BX.

\n

Now that it is clear that an accurate sensor is being used, there might be some play/backlash present in your printer. You should check the probe mount and the Z-axis.

\n" }, { "Id": "14815", "CreationDate": "2020-11-16T02:43:06.180", "Body": "

With a resin printer, the print will occasionally adhere to the bottom sheet of the resin tray (the FEP transparent sheet), instead of the stainless print plate.

\n

How does one remove the failed print from the FEP sheet without causing damage?

\n", "Title": "How to remove a resin print stuck to the FEP tray bottom", "Tags": "|resin|", "Answer": "

Wash the resin tray in the normal way, with hot water and dish soap. Once the FEP sheet is warm, run a finger alongside the bottom of the sheet, opposite the side where the failed print is stuck. The slight warping of the FEP sheet will cause the print to come loose. If this does not occur the first time, try again. Proceed patiently and delicately, making sure to wait for the FEP sheet to be sufficiently warm.

\n" }, { "Id": "14846", "CreationDate": "2020-11-20T11:50:47.697", "Body": "

There are lot of advices on the web how to paint the 3D printed objects, but generally they are advices for manual painting and this required special skills, especially if the object is small. My guess is that maybe 3D printer can lay the color layers as well? I am especially interested in the layering of enamel paints (which can be transparent and which can required high temperature heating afterwards). Medieval art has fine examples how detailed enamel art was created on the metal. Maybe something like this can be achieved with 3D printers as well?

\n

If 3D priner with the paint-printing capability is not available generally then what are the prospects when such printer can be available? Maybe there are some early, experimental efforts to create such printer and maybe test devices are available?

\n", "Title": "Can 3D printer work with enamel paints (or paints generally)? Are such printers available?", "Tags": "|post-processing|", "Answer": "

So, there's not really anything like a 3D painting machine/robot like I think you're looking for, but there are printers that do fine detail with actual paint, usually oils, but not on 3D materials.

\n

I found a thread that talks about canvas printing about 8 years ago, along with a couple of videos that show current machines doing just that, but that's still not what I think you're looking for.

\n

There are also CNC machines that print with enamel paints, but these are usually for 2D again, and not very precise, as they are used for lapel pins that have cavities to hole the paint while it dries. I'm sure something like this could be used without the cavities, but you'd have to do a lot of testing to make sure the paint stays put or mixes as you want it.

\n

Just like the oil printers, these enamel printers are likely very large and costly.

\n

Alternatively

\n

What might work for you is hydrodipping. There's a variety of methods to this, but one company has done a bunch of research on this and can do extremely accurate detail printing to "paint" 3D objects. The below video shows a variety of these hydrodipping techniques, but I've skipped to the most relevant part.

\n

\r\n \r\n

\n

Here's the original video of what I think you're most interested in. It's not 3D printing in the way most of us think, but it's definitely a fantastic outcome.

\n

\r\n \r\n

\n

To explain, if these videos are ever deleted: detailed prints are made of a 3D model to color it exactly the way it needs to be, sometimes using multiple steps and computer positioning to get the object colored/"painted" correctly and seamlessly. One part of the video shows how the software can accurately make straight lines on a human-contoured face mask, while another part shows how a blank, fully 3D cat model can have spots or stripes added in 3 steps with the seams being completely invisible as well as it detailed enough to be mistaken for a real housecat beyond first glance.

\n

As it turns out, you can do (some of) this yourself. After doing some research, I've found that you can actually get blank (instead of pre-printed) films and use an off the shelf printer, as long as it meets certain requirements. (I'm not recommending a site, brand, or anything else, this is just the first/only option I can find. If you do more research, I'm sure there's more options out there.)

\n

https://www.tsautop.com/blank-hydrographic-film/

\n

https://www.tsautop.com/blank-hydrographic-water-transfer-printing-inkjet-printer-6-color-inks/

\n" }, { "Id": "14860", "CreationDate": "2020-11-22T04:44:48.077", "Body": "

I do not have a clear understanding of what causes resin prints to become brittle. Firstly, it appears excessive cold (in the 40s or even 30s, I am in New England) may be a factor. What else can cause brittleness in resin prints? Is there a difference between resin types?

\n", "Title": "Brittle resin prints", "Tags": "|resin|", "Answer": "

disclaimer: I own a company manufacturing 3D printing resins, and all links lead to my products :)

\n

Key reason - resin formula

\n

First of all, it really depends on the composition of your resin. Some of them are more brittle than others. Usually cheaper resins are more brittle.

\n

What causes properties to change?

\n

That being said, some tougher resins even with some flexibility once printed can still become brittle over time. That could happen due to:

\n\n

What can you do?

\n

Best way to protect your models from getting brittle - covering them with paint or other UV resistant coating.

\n

You can also choose resins that don't become brittle over time and is less sensitive to above mentioned causes. For example our TGM-7 resin:

\r\n \r\n

\n" }, { "Id": "14864", "CreationDate": "2020-11-23T02:42:13.553", "Body": "

I just received my new Prusa Mini a few weeks ago and at first it was great. But very soon a afterwards I started having small issues creep into my prints such as shapes becoming elongated, ridges on what should be a smooth surface, and skipped steps along the X axis.

\n

Eventually the entire head carriage started making a thunking sound as it slid across the X axis. This was from the belt, having lost some of its teeth, slipping across the motor.

\n

Here is a photo of the damage to the belt:

\n

\"enter

\n

Prusa service was excellent and with a photo and my order number they shipped out a replacement immediately.

\n

Swapping in the new belt fixed everything and I thought my problems were behind me. But within a day or two of printing I started noticing problems with my prints. Upon inspection, I found that my belt was again damaged:

\n

\"enter

\n

I've tightened belts before on my other printer and have never seen anything like this.

\n

I did notice that the belt provided by Prusa is thinner than the one on my other printer. But the Prusa belt seems to be of a higher quality with some kind of fiber blended into it:

\n

\"enter

\n

The obvious answer is that I've overtightened the belt. But I've tightened belts before and never had this happen.

\n

Interestingly, both belts failed at the exact same location. The position where the damage would be at the X axis motor puts the extruder roughly centered on the build plate.

\n

\"enter

\n", "Title": "How is my Prusa Mini damaging its X axis belt?", "Tags": "|belt|prusa-mini|", "Answer": "

I followed up with Prusa on this recently (a few months later) and they confirmed that there were issues with some belts but that this has been fixed in production.

\n

To provide a better guarantee of quality, I opted to replace both the X and Y axis with genuine GATES 2GT PowerGrip GT3 belts. This has fixed the problem for me.

\n" }, { "Id": "14877", "CreationDate": "2020-11-25T16:56:13.433", "Body": "

I have a number of fairly large printed pieces. The parts, when assembled, form a very large sword. (The model is this: https://www.thingiverse.com/thing:4178060, and is not mine.) It is about 7' from tip to pommel, and individual pieces are fairly large. The pieces are already printed, so advice to modify the model geometry to add reinforcements at this point is moot.

\n

I have printed the pieces in PLA. I'm not asking for glue recommendations, necessarily, but rather something that could help with adhesion in addition to the glue. I'm concerned that after gluing them together, they won't support their own weight very well. I'm not worried about swinging it, just holding it and hanging it on a wall.

\n

If I was very clever, I would have modified the files to add a through-hole that I would've put a pipe through or something, but I didn't think that far ahead. I could drill holes in the printed pieces, but I don't have a good way to make sure they line up.

\n

So, what is the best way to add some kind of support to help keep the parts from falling apart? Is there some trick I could use to figure out how to line up drill holes?

\n", "Title": "What's the best way to connect pieces of a large print after printing?", "Tags": "|3d-models|post-processing|", "Answer": "

A stronger joint than glue can be had by welding, albeit uglier and best used in hidden areas. One way to weld is with friction, using a Dremel to spin a piece of filament so fast it heats up and melts with the two plastic pieces you are joining. You tack the pieces together with a couple spot welds, then move along the joint slowly, melting a bead of plastic that is melted into both sides. Like metal welding, penetration is key to strength. You need a straight piece of filament (heat gun works, and rolling it on a flat surface while it is soft), and 3mm is a better size than 1.75mm, if available. It also helps if the joint is a little \u201cV\u201d shaped, so you melt plastic in the crack between the two pieces. Search \u201cfilament friction welding\u201d for articles.

\n

You can also weld with some filament and a chisel tip soldering iron with variable temperature (lower the temperature to a little above the filament\u2019s printing temp). Melt the filament into the crack between the parts. Do something to get rid of the fumes if using ABS, etc.

\n

Both work, friction welding might get more penetration into the parts being joined, thus a stronger joint, but they are both a sturdy way to do a butt joint between two printed parts, that would be weak if glued.

\n

The friction welding can also be used to \u201crivet\u201d overlapping parts, if you dive in and melt the spinning rod through the two parts, then stop the motor and hold it in place while it cools, then snip the end of the rivet off flush.

\n" }, { "Id": "14881", "CreationDate": "2020-11-25T21:22:11.817", "Body": "

When trying to solve the issue described in this post I followed the advice in the accepted answer and I found that the timing pulley holes for the Y axis (to "secure" the pulley to the motor shaft) were only half drilled (!!!). I suppose that everything worked fine (even without blind screws) for the first month (probably because the belt was extremely tight) but just a bit of wearing and it started to slip.

\n

While I already replaced the pulley I also contacted the Creality technical support to obtain (at lest! funny...) a reassurance that it didn't possibly stressed the belt or the bearing balls. My experience with their technical support has been horrible and I am still waiting (after weeks) my answer (probably it's easier for them to try to ignore their sloppy assembly and QA) but they sent me a procedure to follow to "debug" any issue with my printer.

\n

One of the pictures in that procedure is this:

\n

\"What

\n

To my surprise this is the motherboard I have:

\n

\"What

\n

Now I am puzzled. Which one is the correct motherboard for the Creality CR-10S Pro V2 (not the old version but the newer V2)?\nIs this just [another] sloppy mistake from the technical support or I do have the wrong motherboard? Differences with V1 are not many then I suppose it could even work with the old one...

\n", "Title": "Is this the correct motherboard for my Creality CR-10S Pro V2?", "Tags": "|creality-cr-10|part-identification|", "Answer": "

The board that's installed in your machine appears to be the Creality v2.4 / v2.4.1 board, which is the correct board for your printer.\nIt is distinguishable by the 30-pin ribbon connector, which is used to connect to the rest of the printer.

\n

The board in the tech support pictures is the v2.5.2 "silent board", designed for printers such as the non-Pro CR-10 series. It features two 10-pin connectors for the LCD screen.\nIt also uses individual connectors for all functions, so, contrary to the "Pro" boards, is also easily swappable with aftermarket boards.

\n

A third type of board (e.g. v1.1.5) exists, mostly for the Ender 3 series of printers, which features only one 10-pin connector for the display.

\n" }, { "Id": "14892", "CreationDate": "2020-11-28T01:47:25.993", "Body": "

I'm trying to subtract a polyhedron from a cube, but it is not working (the cube remains solid). However, I can see the cut-out poly in preview mode (but not after a full render).

\n

Preview -- poly cutout shows on the top (and bottom).

\n

\"preview\"

\n

Rendered -- poly cutout not visible.

\n

\"rendered\"

\n

Poly Exploded -- pulled the poly to the right to show its shape.

\n

\"poly

\n

Code

\n
size = 30;\nwall = 3;\nwall_x2 = wall * 2;\nnubGap = .125;\nnubHeight = 8;\nnubOffset = wall + nubGap;\nxCutoutSize = size - wall_x2;\nyCutoutSize = size - wall_x2;\ncutoutLowerY = nubHeight + nubGap;\ncutoutUpperOffset = nubOffset + wall;\n    \ndifference() {\n    cube([size, size, size]);\n    \n    translate([wall, wall, 0]) {\n        polyhedron(\n            points = [\n                [0, 0, -10],\n                [xCutoutSize, 0, -10],\n                [xCutoutSize, yCutoutSize, -10],\n                [0, yCutoutSize, -10],\n        \n                [0, 0, cutoutLowerY],\n                [xCutoutSize, 0, cutoutLowerY],\n                [xCutoutSize, yCutoutSize, cutoutLowerY],\n                [0, yCutoutSize, cutoutLowerY],\n        \n                [cutoutUpperOffset, cutoutUpperOffset, size],\n                [xCutoutSize - cutoutUpperOffset, cutoutUpperOffset, size],\n                [xCutoutSize - cutoutUpperOffset, yCutoutSize - cutoutUpperOffset, size],\n                [cutoutUpperOffset, yCutoutSize - cutoutUpperOffset, size]\n            ],\n            faces = [\n                [0, 1, 2], [2, 3, 0],     // bottom\n                \n                [0, 1, 4], [1, 4, 5],     // side A\n                [1, 2, 5], [2, 5, 6],     // side B\n                [2, 3, 6], [3, 6, 7],     // side C\n                [3, 0, 7], [0, 7, 4],     // side D\n                \n                [4, 5,  8], [5,   8,  9], // slope A\n                [5, 6,  9], [6,   9, 10], // slope B\n                [6, 7, 10], [7,  10, 11], // slope C\n                [7, 4, 11], [4,  11,  8], // slope D\n                \n                [8, 9, 10], [10, 11,  8]  // top\n            ]\n        );\n    };\n};\n
\n", "Title": "OpenSCAD not rendering polyhedron", "Tags": "|3d-design|openscad|", "Answer": "

Usually when there's an overlap in two objects during a difference action, F6 render will resolve the problem. There's something more than that involved here, as reducing the height of the cube creates a non-manifold object from the difference. user R..'s answer has merit but is not going to solve the problem.

\n

Isolating the cube from the code and exporting the result as an STL allows me to determine that the faces are generated in a manner preventing a proper difference action:

\n

\"meshmixer

\n

This image from meshmixer shows the faces have inverted normals. The order of the points are critical when describing a polyhedron. From the wiki page for OpenSCAD:

\n
\n

It is arbitrary which point you start with, but all faces must have\npoints ordered in the same direction . OpenSCAD prefers clockwise when\nlooking at each face from outside inward. The back is viewed from the\nback, the bottom from the bottom, etc. Another way to remember this\nordering requirement is to use the right-hand rule. Using your\nright-hand, stick your thumb up and curl your fingers as if giving the\nthumbs-up sign, point your thumb into the face, and order the points\nin the direction your fingers curl.

\n
\n

EDIT: I reversed some of the points, haphazardly and luckily picked the correct ones:

\n
        faces = [\n            [0, 1, 2], [2, 3, 0],     // bottom\n            \n            [4, 1, 0], [1, 4, 5],     // side A\n            [5, 2, 1], [2, 5, 6],     // side B\n            [6, 3, 2], [3, 6, 7],     // side C\n            [7, 0, 3], [0, 7, 4],     // side D\n            \n            [8, 5, 4], [5,   8,  9], // slope A\n            [9, 6, 5], [6,   9, 10], // slope B\n            [10, 7, 6], [7,  10, 11], // slope C\n            [11, 4, 7], [4,  11,  8], // slope D\n            \n            [10, 9, 8], [8, 11, 10]  // top\n
\n

\"rendered

\n" }, { "Id": "14894", "CreationDate": "2020-11-28T09:08:53.427", "Body": "

\"enterEnder 5 Pro prints are very uneven on base of the model.

\n

In this example I printed an SD card dummy but it is not even flat.

\n

Having a glass bed it is super strange to see this kind of behaviour.

\n

Here are parameters in Cura.

\n
M92 X80.00 Y80.00 Z800.00 E94.5 ;Setup step mm\nM201 X500.00 Y500.00 Z100.00 E5000.00 ;Setup machine max acceleration\nM203 X500.00 Y500.00 Z10.00 E50.00 ;Setup machine max feedrate\nM204 P500.00 R1000.00 T500.00 ;Setup Print/Retract/Travel acceleration\nM205 X8.00 Y8.00 Z0.40 E5.00 ;Setup Jerk\n
\n

Thanks in advance, I tried quite a few calibrations but nothing helped so far.

\n", "Title": "Ender 5 Pro bent/curved print on base", "Tags": "|creality-ender-5|", "Answer": "

This is a classic example of not enough adhesion. Tune bed temperature (usually hotter), use a brim or add an adhesive layer like glue stick or hair spray or a dedicated print adhesion spray.

\n

The question "Warping of bottom of print" has an accepted answer that explains in detail how and why this occurs.

\n" }, { "Id": "14898", "CreationDate": "2020-11-29T10:04:39.287", "Body": "

(Ender 3 Pro with Bigtreetech Board and Touchscreen, BlTouch)

\n

Hello everybody,\nI want to print something for my family but the hotend gets clogged every time in the same part of the print. I tried:

\n\n

The most odd thing about this is that the problem occurs at the same place every time.

\n

On Flat surfaces are some anomalies, that wasn't there when I had the normal clogged nozzle problems (from: retraction settings, dirty Printer, e-steps false). I think it has something to do with the anomalies.

\n

Another thing I don't get behind are missing layers after the layer change,\neven when I don't use retraction at all.\nIn The Picture from the side you see the Support with the Layer change problem.\nAfter around about Layer 40 there is one Layer missing and the next Layers are not connected anymore.\nFrom the top you see the Surface anomaly I don't know how to describe.\nIt would be helpful to know what I did wrong. I am sure I did Everything against a clogged Nozzel but I can be Wrong so tips in this direction are Helpful too.\nI am quite new to 3D Printing(2 Months) I had the usual clogged Nozzel Problem Solved and It Worked Perfectly. I Tried to Fix It with the same Solutions and Nothing Helped, so I think it is a different problem.\nSry for my bad English I am from Germany.\"enter

\n

\"enter

\n

I have an Ender 3 Pro with Bigtreetech Board, Touchscreen and BlTouch which I use together with [insert slicer here]. I print in [PLA/ABS/PETG/Whatever Material] at [Extruder temperature] \u00b0C. The print bed is set to [Bed Temperature] \u00b0C. I use a print cooling fan at [whatever] %. The layer height I set to 0.[x] mm, the line width [line width/extrusion width] from the 0.[x] mm nozzle. The Printing Speed is set to [x] mm/s for walls and [x] mm/s for infill. My retraction is [off / [X] mm at [x] mm/s].

\n", "Title": "I can't figure out how to name this problem or solve it", "Tags": "|print-quality|creality-ender-3|hotend|bigtreetech|", "Answer": "

I noticed that on my Ender 3, if the belts aren't tight enough, I have a similar problem, but mostly when I print anything circular.

\n" }, { "Id": "14900", "CreationDate": "2020-11-29T19:48:50.620", "Body": "

Given a .SCAD file which contains some modules, how can I execute one of those modules from the command line?

\n

example.scad

\n
module One() { ... }\nmodule Two() { ... }\n
\n

render.sh

\n
openscad -q -o one.stl --module One example.scad\n
\n

Note that there is no --module option, but that is what I'm attempting to do. The workaround would be to make another .SCAD file e.g. one.scad which includes example.scad, and simply calls One(); within and render that file from the shell file. But this is not ideal.

\n", "Title": "Export STL from OpenSCAD command line, calling module within script?", "Tags": "|openscad|", "Answer": "

elaborating on the answer from @r-github-stop-helping-ice, I find the following pattern useful:

\n
example.scad
\n
module a() { .. }\nmodule b() { .. }\nmodule c() { .. }\n\nmodule print(item=0) {\n  if (item==1) a();\n  if (item==2) b();\n  if (item==3) c();\n}\n\nitem = 0;\nprint(item);\n
\n
render.sh
\n
#!/bin/bash\nfor i in {1..3}; do\n  openscad -D item=${i} -o item_${i}.stl example.scad\ndone\n
\n" }, { "Id": "14912", "CreationDate": "2020-12-01T03:29:08.103", "Body": "

I am relatively new to 3D printing, but have some background in industrial equipment and test machines.

\n

An important part of printer setup is getting the extruder mm/step rate configured correctly.

\n

I was wondering if anybody has done any research into closed loop drive of the filament where the feedback to close the loop comes from a knurled idler on the filament.

\n

This would mean factors such as drive slippage and different material stiffness could be automatically compensated for.

\n

Going one step further the feedback wheel could be placed at the hotend of Bowden extruder systems negating the need to tune in retract settings for different materials as the closed loop system can just pull back however much material is required to get 1-2\u00a0mm as measured at the hot end.

\n

I have seen there are readily available steppers with drive boards built in for closed loop control, I would imagine it would be a relatively simple task to remove the hall effect sensor from one of these boards and mount it remotely to read off a measuring wheel instead of the drive shaft.

\n

I had a bit of a search but couldn't find anything on this topic online, is this something that has already been looked into and if it hasn't are there any significant reasons why this shouldn't work to improve performance and/or reduce manual tweaking and calibration required.

\n", "Title": "Can closed loop steppers be used for extruders?", "Tags": "|extruder|stepper-driver|stepper|closed-loop|", "Answer": "

Closed loop steppers like S42B are becoming popular for the spatial axes because compensating for skipped steps is the correct way to recover for those axes. For example, if the nozzle knocks into a warped corner of a print and skips, the right thing to do is compensate, commanding additional steps necessary to get to the commanded location.

\n

This is less valuable for extruder, since displacement is not necessarily the target we want to recover, but nozzle pressure. Extruder skip usually occurs when nozzle pressure exceeds the motors ability to drive the filament. Correcting for the skipped steps may result in overcompensating, attempting to drive nozzle pressure up over a longer period of time, since the problem won't clear itself as quickly as it does for the spatial axes. Compensating for skipped steps doesn't recover E errors like they do for X,Y, or Z, since skipped E steps tend to be driven by persistent problems, like low nozzle temp or hotend restrictions, where spatial errors tend to be transient, e.g. hitting a 'speed bump' on the print.

\n

An alternative is filament flow sensors like this BTT Smart Filament Sensor. It uses a similar feedback mechanism like you suggest. It uses a driven wheel and encoder to send pulses to mainboard. When mainboard stopped getting pulses during commanded moves, it pauses print. Instead of trying to correct in a closed loop fashion, its a human-in-the-loop solution

\n" }, { "Id": "14928", "CreationDate": "2020-12-03T04:30:44.787", "Body": "

I'm looking to put a UV-blocking clear coating on some of my resin 3D prints.

\n

I've tried krylon UV-Resistant Clear Coating but it seemed to react with my print making it softer and easy to break. I'm trying to coat a thin Lithophane so I would rather not make it any softer.

\n

Are there any better options for a UV-blocking clear coat for resin 3D prints?

\n

Printer used: AnyCube Photon (dlp)
\nResin used for Print: ELEGOO ABS-Like 3D Printer Rapid Resin
\nLithophane maker: lithophanemaker

\n", "Title": "Clear coating resin 3D prints", "Tags": "|post-processing|resin|uv-printer|", "Answer": "

I'd go for the easy option of using a composite setup: Acrylic lacquers themselves are very low reactive and can serve as a foundation before the UV clear coating is applied. The acrylic lacquer itself doesn't change coloration if exposed to sunlight, but the light can still pass and discolor the lithophane.

\n

Your Krylon clearcoat technically is an acrylic paint, but to be used from a rattlecan, other chemicals are added - a whole lot as the MSDS tells us. The list of chemicals besides the paint in that can are Acetone, n-Butyl Acetate, Propane, Butane, Xylene, Ethyl 3-Ethoxypropionate, Cyclohexane, Ethylbenzene,\nBis(pentamethyl-4-piperidyl)sebacate. Three simple chemicals I put in italics - they are very good solvents that easily react with the resin print. Propane and Butane create the pressure in the rattle can, the other four chemicals make up the actual lacquer. I have observed the mix of Acetone, Cyclohexane, and Ethylbentene (with possibly others) softening PLA, so it is totally possible that they react unfavorably with your resin print. As a result, I suggest using a brush or roller to apply acrylic paint from a can (not spraycan!)

\n" }, { "Id": "14933", "CreationDate": "2020-12-03T19:21:09.300", "Body": "

I had a failed print lastnight and a glob of pla surrounded the hotend. After heating it up and removing the glob the extruder does not release any significant material. I checked with OctoPrint and the extruder stepper does push filament without the hot end on.

\n

After fully heating the hot end I was able to remove the nozzle.

\n

I then pushed the previously melted PLA out of the tube and heater. It was discolored in the heater. I then tried to put the nozzle back on and pull the old PLA out of the nozzle but no success there.

\n

Should the nozzle be left in acetone for a few days before trying again or how should this be dealt with. The nozzle appears to be blocked.

\n", "Title": "After print failure cant extrude PLA", "Tags": "|all-metal-hotend|", "Answer": "

If the hotend is okay, consider yourself lucky and consider the nozzle a loss. Aside from specialty items like ruby tip ones, nozzles are a consumable anyway. Even if you can clean it out well enough to get it working again, it's unlikely to extrude as well as a new clean nozzle. Acetone is probably not going to help; PLA does not dissolve in acetone, and while the pigments/additives might be affected by it enough to weaken and deform PLA in a way that could get it to detach, what's clogging your nozzle is likely burnt/carbonized PLA that's unlikely to be removable by anything that won't also destroy the brass.

\n

If you don't already have replacements on hand, get yourself a pack of 10-20 for $10 or so and be ready to replace when needed. You can also go for a mixed-size pack if you want to try printing fine details with a smaller (0.1-0.3 mm) nozzle or super-fast but rough with a large (0.8-1.0 mm) nozzle.

\n" }, { "Id": "14935", "CreationDate": "2020-12-04T00:58:05.900", "Body": "

For the purpose of cleaning, I need an aggressive solvent for cured or partially cured resin that will degrade resin down to its liquid state. I'm looking for one that would eat out specifically resin (I'm using regular Anycubic green resin) in a rapid fashion but would leave painted / metallic parts and screen of my 3D printer without damage.

\n", "Title": "Aggressive resin solvent", "Tags": "|resin|safety|chemistry|", "Answer": "

I would try hexane, and then Dichloromethane and if those did not work, I would heat up sodium hydroxide to about 70-90\u00a0\u00b0C. These would work better if you print in PLA resin, it's available from a few sources now.

\n

Bucktown polymers and 3Dresyns both have a water-soluble resin. You could also print, make a soft silicone mold, cast in chocolate or isomalt and quickly seal in b72 enamel to hold its shape for your next step.

\n

Another great option is to cast the printed mold with Amazing Remelt as you can microwave it out of your shell afterward. Or heat up/steam.

\n" }, { "Id": "14943", "CreationDate": "2020-12-04T21:09:01.207", "Body": "

This is a specific model for a specific reason. It's shaped like a cylinder and has a recessed bottom. I'm using a Flashforge Adventurer 3 with Flashforge filament. I tried Sunlu silver/silk PLA+ but my Flashforge didn't like it, so I took the recommendation to use Flashforge filament. It worked just fine, but when I put water in it, it has leaks.

\n

I was printing at 210 with a plate temp at 50. Like I stated, I was excited except for the fact that it's not watertight.

\n

Since I'm new at this process, my first thought was there must be a food grade spray, or close to food grade, that could seal the model after I touch it up.

\n

Does anyone know if there is one available? Or are there settings that I need to address?

\n

Thanks in advance for any comments.\"enter

\n", "Title": "Is there a way to make a watertight model? Is there a spray that may work?", "Tags": "|water-resistance|", "Answer": "

There are several ways to ensure water tightness:

\n

Thicker walls

\n

I have experienced that 3 walls and 5 tops/bottoms with an outer shell of Concentric while the core is lines become watertight enough that leakage is almost none.

\n

Lacquer

\n

The oldest known method to seal a piece of porous material is a lacquer, followed by glaze. Glazes require to be burned at some thousand degrees, so are out, but lacquers are simple and easy. Basic clear acrylic lacquer can be used to seep into all the cracks and crannies and then harden out. It's easy to apply and cheap from home depot. Polyurethane lacquers also work. If you look to use it for food, ask if the lacquer is food rated in home depot.

\n" }, { "Id": "14952", "CreationDate": "2020-12-06T13:32:58.037", "Body": "

I am making a bed for my 3D printer. I have bought a silicon heater (31x31\u00a0cm) and I want to glue it to my custom aluminum bed. The tape that it had from factory was bad, so I removed it. I want to glue it to the aluminum and I don't know what type of adhesive to use, I was thinking gasket glue with silicon, but I think that it will have bad thermal conductivity. I found this product, a silicon based, heat transferring paste, but I think that it will not stick good. What is a good adhesive for this purpose?

\n", "Title": "Gluing silicon heater to aluminium", "Tags": "|diy-3d-printer|heated-bed|", "Answer": "

I suggest not gluing it. Starting from the top, make a sandwich this way:

\n

Aluminium with holes for bolts - Silicone heater - Thin cork (the one from IKEA, 2 mm thick for office desks is fine) - Thin plywood with holes for bolts (or other stiff material holding at least 60\u00b0C)

\n

This way you use the aluminium and the plywood to keep the silicone heater well in contact with the aluminium, and the cork insulates so that less heat is lost on the bottom side.

\n

Also, cork is fire-retardant.

\n

If the heater fails replacing it is simple.

\n

Also, you can and should cut away from the cork some space for a thermal non resettable fuse at 180\u00b0C to cut power if the heater overheats.

\n

In my case I should have used one more bolt, as you can see in the photo.

\n

\"enter

\n" }, { "Id": "14963", "CreationDate": "2020-12-07T13:02:45.613", "Body": "

Is this possible? I know that the controller board of the printer is normally controlled (when using USB) by an active computer sending data, but I was wondering if it could be reversed so that the board retrieves files from a USB flash drive? My SD card keeps failing and is overall frustrating to work with, so USB would be a nice option.

\n", "Title": "Printing files over USB drive (Ender 3 Pro)", "Tags": "|creality-ender-3|usb|", "Answer": "

Unfortunately, no.

\n

There are two types of USB devices: host, such as a PC, and device, such as a thumb drive. (Actually, there is a third type, on-the-go, which can act as either.)

\n

The Ender 3 mainboard is a USB device which means that it can only connect to a USB host. You cannot directly connect two USB devices, such as the Ender 3 and a thumb drive) and have them work.

\n

One option would be to set up OctoPrint software on a Raspberry PI single-board computer and connect that by USB to the printer.

\n" }, { "Id": "14986", "CreationDate": "2020-12-08T22:41:48.920", "Body": "

Why does the Ender 3 only have 3 limit switches instead of 6?

\n

How does it handle crashes on other sides?\nIs it worth adding them with a new mainboard?

\n", "Title": "Ender 3, why only 3 limit switches?", "Tags": "|creality-ender-3|endstop|", "Answer": "

Because it knows the side of the build volume, all it needs to know is one physical location to home from. So knowing {x,y,z} (0,0,0), and knowing x(max) = 230, y(max) = 230, z(max) = 250, and knowing 1 step (x) = 0.001 mm, it can know when to stop before hitting the other side.

\n

Because it knows where 0 is, and because it knows the maximum value, it knows where the other limit is through math.

\n

2-sided limit switches are used on industrial equipment that have open-loop control mechanisms, like a lathe with a DRO, the DRO won't stop the lathe, so you need to hook up a limit switch on the other side to stop the lead screw... You could also have a maximum travel limit switch on a CNC router where if you send the spindle too far, you might break the equipment, or cause injury.

\n

I hope this answered your question, and was also informative on how limit switches are used elsewhere... It's not perfect or exact, but I think it should give you a broad idea...

\n" }, { "Id": "14996", "CreationDate": "2020-12-09T09:52:29.507", "Body": "

Recently I started looking at pressure advance and how it works and I'm a bit confused about where it is usually implemented.

\n

My idea of 3D printer was that its firmware is fairly dumb and only replays G-code, not knowing anything about the object being printed, material used, or even the printer itself.

\n

But with pressure advance this whole thing changes and now the firmware needs to know the linear advance factor which combines information about the filament and filament path used. In addition, the E axis is no longer controlled directly by the G-code, but its motion is almost independently determined by the firmware.

\n

Why is this? Is there a reason that slicer (or a post-processor) can't compute all this and directly store the needed extruder axis movements in the G-code?\nDoes the printer have some additional information that the slicer is missing?

\n", "Title": "Why is pressure advance usually implemented in firmware rather than in the slicer?", "Tags": "|firmware|fdm|pressure-advance|", "Answer": "

This is a really good question that sheds a lot of light on 3D printer software/firmware architecture, and Tom already said a lot of the things I wanted to say before getting a chance to write an answer. The basic problem is that, to do pressure advance accurately (and in a way that doesn't get it horribly wrong when inaccurate), you need to know the actual feedrate of the extruder at all times, and that's not available until applying the acceleration profile, which by convention happens in the printer firmware.

\n

With that said, there were primitive and even somewhat advanced attempts to do pressure advance in the slicer. The first seems to have been "coasting", which, along with extra-priming after coast, is pretty much just "pressure advance, assuming a constant feedrate". It gets things horribly wrong if you mix different extruder feedrates (different print speeds or line widths, etc.) or if you have slow acceleration, but if your acceleration is so fast (relative to max speed) that it's approximately instantaneous, it might work okay.

\n

Modern Cura also has Flow Rate Compensation, which is something like pressure advance. It's rate-sensitive, so in theory it can give accurate results with varying line width and print speed as long as acceleration is close enough to instantaneous. Since it appeared after Marlin added linear advance, I never bothered trying to play with it, so I can't speak to whether it actually works decently. There are still a lot of subtleties to when the advance is performed that it could get wrong, and I think you'd want to do some test cases just to read the gcode output and evaluate whether what it's doing is sufficiently close to reasonable.

\n

If you wanted to do full pressure advance in the slicer, you'd need to let the slicer handle acceleration profile, breaking lines up into small segments each with nominal feedrate matching the rate they should end at, and sufficiently close to the rate they should start at, with the firmware acceleration limits set to accommodate the change. Then, knowing a very good approximation of the actual toolhead and thus extruder feedrate for each segment, you'd know the advance to apply, and could apply it as an additional subdivision at the end of the previous print move. And then in theory, it all works out. But this would make the gcode a lot larger/bulkier, and more demanding on the serial link speed and microcontroller's ability to keep up with parsing/planning. So it's almost surely a bad idea.

\n

The Klipper firmware does this differently. It does the gcode parsing and planning (including pressure advance) in Python software (with some C for critical paths) running on a much more capable computer, and sends the precise generated stepper motor timings over the serial link to the microcontroller operating the printer hardware.

\n" }, { "Id": "15002", "CreationDate": "2020-12-09T20:14:20.183", "Body": "

I am using a clear resin to print parts using a Formlabs printer. At the moment, I am using the resin from Formlab which comes in a cartridge that fits in the printer. However, I would like to move to a cheaper option if available. I would like to able to refill the cartridges that I have rather than buying a new cartridge.

\n

Is it possible to buy some clear resin from an outer source or would that damage the printer?

\n", "Title": "Does the resin vary from a printer to another?", "Tags": "|resin|", "Answer": "

According to Formlabs (at least their FormLabs Form 2 and 3 models), their resin is 405 nm which is a fairly standard resin.

\n

There's a wide variety of resins our there that are compatible with 405 nm, and many resin makers/resellers advertise what brand, make, and model of printer they are compatible with. I have an AnyCubic Photon v1 and it uses the same 405 nm resins. You might have to do some testing to see what specific brand or style works for you, including changing your settings (if possible in the software you use). FYI, I use a version of CHITUBOX which does allow settings for exposure time, raise height, pause time, and a few others.

\n

Knowing that it uses 405 nm UV also allows you to know what kind of UV lights to get for easy indoor curing, too.

\n

I'd like to say that it's not likely for these resins to damage the printer, but FormLabs seems to use a plastic tray, instead of the aluminum tray I'm used to so I don't know if there would be a chemical reaction, though I'm skeptical if there would be. I'm assuming it's using a similar FEP clear film for the bottom to the AnyCubic, but I can't confirm this, since the FormLabs film seems to be integral to the tray, rather than replaceable.

\n

I can see why you are looking for other resin options. One place I saw online has a 1 L FormLabs cartridge for \\$150, when 500 mL on Amazon is usually between \\$20-30. I've seen special resin, such as flexible, as high as \\$60 for 500 mL and ABS-like resin for $40, though. Water washable resins seem to be in the \\$40-50 range. There's also "natural" or plant based resin for about the same price as regular resin, but might have a different reaction to your tank.

\n

I'd assume that every brand of resin (except for the rebranded stuff) has their own chemical makeup besides the various types of resin, so you might have to some testing, although I realize that messing up a tank that's \\$150 to replace is not exactly something you want to do every day.

\n

I can tell you that I've used at least 6 different brands of resin and they all react differently to the UV. I mostly have tested clear, and just that produces a variety of results, from how easy/hard it is to cure to what color it turns when it over cures. (I'm still trying to find a clear that fully cures actually clear.) Where I was going with this is that with all these resins I've used, I haven't had any that caused damage to the FEP film. Some have stuck really hard and I had to be really careful pulling it off. However, I've only had 1 film need to be replaced due to clouding, and I have 4 Photons and used all resins I've tested with all my machines.

\n" }, { "Id": "15016", "CreationDate": "2020-12-11T20:20:48.493", "Body": "

I would like to log each line of G-code to the serial port as it is processed.

\n

Steps to achieve:

\n\n

So by the end of the print, on my laptop I would have the reconstructed G-code file (plus whatever other logs the printer outputs).

\n

The printer runs the Prusa Firmware. Ideally I would like to achieve the logging from altering the firmware rather than adding an extra plugin/server (For understanding and experimenting purposes).

\n

What I tried

\n

I have looked in code and found the print functions and examples of them in use in the code. This line is the "command, which is to be excecuted right now", but I think that would be the just one command not the full line.

\n

The cardreader or SdBaseFile are where I would expect a G-code line to be read such that I could add a print statement after it but I did not see where.

\n

Would it be as easy as setting this card.logging bool to true?

\n

I imagine this is quite an easy thing to do and that I have just overcomplicated it by trying to understand the firmware. Any advice would be great!

\n", "Title": "How to log each G-code line read from the SD card to serial on marlin firmware", "Tags": "|marlin|g-code|serial-connection|", "Answer": "

In cmdqueue.h the CMDBUFFER_DEBUG macro is defined (see here) which will log lots of information related to the commands being processed. Probably more information than you need...

\n

To log only the commands as they are processed (i.e. when the printer moves and extrudes etc.) you need this line from marlin_main.cpp: SERIAL_ECHO(cmdbuffer+bufindr+CMDHDRSIZE);.

\n

You can copy this outside of the #ifdef CMDBUFFER_DEBUG condition and compile the firmware then when you connect to the serial port each G-code line the printer processes will be logged.

\n" }, { "Id": "15032", "CreationDate": "2020-12-13T19:59:52.017", "Body": "

I have an MK3S 3D printer, and I use this table as a reference on how to tune my printer for a specific Filament\nbrand.

\n

https://help.prusa3d.com/en/materials

\n

I am trying to find a few missing parameters for a specific brand Filament PLA in the table.

\n

The missing parameters are listed and shown in the image below.

\n\n

\"enter

\n

Where is the best place or reference to find such missing data?

\n", "Title": "Where to find \"Heat deflection temperature\" , \"Impact resistance Charpy\" and \"Tensile strength\" for a specific brand Filament PLA?", "Tags": "|pla|prusa-i3|", "Answer": "

Unless the Manufacturer offers them, you have to extrapolate from other brands. Most PLAs are very close and hit within 10% of one another.

\n" }, { "Id": "15048", "CreationDate": "2020-12-16T00:34:05.570", "Body": "

I 3D modeled a cartoon car using Blender, mostly using add and subtract with boolean tools. When I export to Ultimaker Cura, I get these cuts along the model. Does anyone know why this is happening? Appreciate any feedback on the model.

\n

\"enter

\n

\"enter

\n

The blender file: https://github.com/dantedaiki/jQuery-File-Upload/blob/master/Cartoon%20Car.blend

\n", "Title": "3D model with errors when exporting to Cura", "Tags": "|ultimaker-cura|3d-models|blender|", "Answer": "

In blender, you can occasionally have faces with flipped normals. Those areas then are "inside out". Such areas are just ignored in Cura. Fixing the files is as simple as recalculating the normals, but you'd best use a program like NetFabb for for that.

\n" }, { "Id": "15049", "CreationDate": "2020-12-16T05:28:20.253", "Body": "

I was five hours into a six hour print and got a 28\u00a0mm X direction layer shift. No idea what caused it. I stopped the print cleaned the bed and (via Pronterface) went to home the bed (G28). The gantry homed X and Y and then I got the following message:

\n
\n

Error:Printer stopped due to errors. Fix the error and use M999 to restart. (Temperature is reset. >Set it after restarting)\n[ERROR] Error:Printer stopped due to errors. Fix the error and use M999 to restart. (Temperature is >reset. Set it after restarting)

\n

Error:!! STOP called because of BLTouch error - restart with M999\n[ERROR] Error:!! STOP called because of BLTouch error - restart with M999

\n
\n

Entering M999 followed by G28 just reproduced the error. I powered down and then checked the BLTouch connections. The white signal wire seemed a bit loose, so I tightened that. All other wires are secure. Powering back up, the BLTouch did not do its normal probe up and down routine. The main red light flickered for about three seconds, it then flashed 7 times and then came on fully. At the end of the flashing, the blue LED on the circuit board also came on. Testing, the black and white signal wires gave a reading of 0.975\u00a0k\u03a9.

\n

I had homed and levelled the bed (G28, G29) then saved the data (M500) prior to starting to the six hour print print earlier with no problem.

\n\n

Anybody any idea what has happened?

\n", "Title": "BLTouch Stopped Working", "Tags": "|troubleshooting|bltouch|", "Answer": "

Turned out that the BL Touch had packed it in. Replaced with a new BL Touch v3.1 and everything works.

\n" }, { "Id": "15054", "CreationDate": "2020-12-17T12:30:03.440", "Body": "

Before I start, I'll give you my setup:

\n\n

Since I've updated the Marlin FW on from factory default to 2.0.7.2, my printer stops printing and gives out an thermal runaway exception message. Note that, after the firmware flash, I performed a PID-Tune multiple times.

\n

The problem is absolutely repeatable and happens always on beginning of layer 2 (more precisely: 40 seconds after beginning layer 2). Changing PID values doesn't change anything to the moment of the error occurring.

\n

I managed to run it longer by repeatedly dropping the temperature set-point and making a photo of the temperature plot. First photo is right after the initial drop from 215 to 205\u00a0\u00b0C. Second is when the temperature started rising slowly again.

\n

\"Right

\n

\"Temperature

\n

After this temperature drop, the hotend temperature seemed to be much less stable and reached only 205\u00a0\u00b0C.

\n

At Layer 12, the same thing happened again. But dropping the temperature far too low for PETG and having the "same" issue again, making me stop the print.

\n

This problem is pretty urgent and I haven't found any suitable solution by now. Do you have any ideas of what may cause this trouble? New heaters and thermistors are on their way right now. But I fear that this is not a hardware problem since none of the components are damaged and dysfunctional, nor the moment of failure is random.

\n", "Title": "Thermal Runaway E1 at Layer 2", "Tags": "|marlin|creality-ender-3|petg|pid|thermal-runaway|", "Answer": "

Wrap all the heat system (above nozzle) with heat tape. I solved my problem with this!

\n" }, { "Id": "15060", "CreationDate": "2020-12-19T01:54:31.110", "Body": "

Recently I cleaned up the bowden tube and the nozzle, after that I noticed the 3D was underextruding because the walls of the prints were weak and the first layer didnt adhere on the bed.\nI watched some youtube tutorials on how to fix this and pulled the bowden tube until the nozzle, cleaned inside the tube and removed clogs.\nMy temporary fix is setting the flow on Cura to 200%.

\n

I added a extruder visualizer and it seems to be working just fine.\nAny ideas why it is underextruding?\n\"enter

\n

I'm using PLA at 200C, bed at 60C

\n", "Title": "Ender 5 Underextruding", "Tags": "|ultimaker-cura|hotend|underextrusion|", "Answer": "

If you pulled the Bowden tube out of the hotend and then got this, you almost surely installed it wrong, leaving a gap between the tube and the nozzle for molten filament to fill and jam in. Remove it again with the hotend hot, and if there's a mess inside, look for guides on cleaning it. Then, to reassemble, loosen the fitting from the heat sink by 3/4 to one full turn, press the tube in as far as it will go, then tighten the fitting down to compress the tube against the nozzle.

\n" }, { "Id": "15077", "CreationDate": "2020-12-21T19:26:53.303", "Body": "

Odd question for everyone, hope it has a distinct answer. I'm often printing bone models derived from CT scans (I work in a hospital) and they often have something on the order of 5 million faces +/-. Now, I know from experience that I can decimate them down to 10-20\u00a0% of the original faces and they still pretty much look the same, so I often do that to help my computer run faster. I also know that "GrabCad" (the software for my j750) can handle these face counts and the limiting factor is more so the actual physical print resolution. But it got me curious:

\n

Lets say I was using other software. Lets say more universally available software such as PrusaSlicer or Cura. Now obviously if I'm printing on a Prusa I probably don't need to worry about capturing all the detail from 5 million faces because I doubt I can print that intricately, but lets take the actual printing out of it (and I guess even before it gets to the G-code stage).

\n

Can Cura/PrusaSlicer handle that many faces? Is there a limit? Do files get "dumbed down" at all when they come in? What I'm trying to ask exactly is outside of the G-code and actual printing step, can the software side of things handle something with 10 million faces? 20 million? Is there a limit?

\n", "Title": "File size limit in 3D printing software", "Tags": "|slicing|prusaslicer|", "Answer": "

As a new Cura UltiMaker user, I found that when I tried to load a very detailed STL file, I received a message that it had too many faces. On checking the Cura site, it said that the limit was 30K faces. That may be a limitation only of the version of Cura that I have (it is for a Fokoos Odin-5 printer).

\n" }, { "Id": "15080", "CreationDate": "2020-12-21T23:44:15.693", "Body": "

How do I improve/increase shell coverage so that it covers the entire inner wall?\nI've experimented with shell settings, but I can't seem to get it right.

\n

I'm using Creality Slicer 4.2.1 (Cura)

\n

Picture showing the areas in question below:\n\"Areas

\n", "Title": "Shell doesn't cover the entire inner wall at some areas", "Tags": "|ultimaker-cura|", "Answer": "

You should increase the top layer thickness according to Ultimaker Cura support:

\n
\n

Top/bottom thickness

\n

With the top/bottom thickness you can set the thickness of the solidly printed top and bottom layers of the print. A higher value ensures all gaps on the top and bottom layers are closed completely. However, this can also increase the print time and amount of filament used.

\n

It is advised to always use a multiple of the layer height for the thickness of the top and bottom. This means, for example, that with a layer height of 0.15 mm, it\u2019s better to set the top/bottom thickness to 0.6 mm rather than 0.7 mm.

\n
\n" }, { "Id": "15087", "CreationDate": "2020-12-22T19:24:37.510", "Body": "

Simple question, I want to move my printer head up when the print finishes instead of to the side.\nWhere's the option to do this in Cura?

\n", "Title": "How to move Z up after printing in Cura?", "Tags": "|ultimaker-cura|", "Answer": "

Go towards the right top corner. Click on the printer name. From the drop down, select, "Manage printers". You will get a dialogue box in the middle of your screen. Your current printer's name will be shown in italics. On the right side of the dialogue box, click on, "Machine settings". This will open another dialogue box. In the lower left, you will see a text box with the heading, "End G-Code" scroll down to where you see the line, "G1 X0 Y0". This is the line that moves your print head to the lower left corner. G1 is the command for a linear move. X0 and Y0 are move instructions to move the the 0 coordinate for the two axes. Change this line to G1 Znn where "nn" is the number of the coordinate you wish to move to. Close this dialogue, close the previous dialogue and all newly sliced file will now use this code.

\n

If like to keep track of any changes you make, you can put a semi-colon at the beginning of the line. This turns the line into a comment and will not be actioned. Then press enter and on a new line, put the line, G1 znn.

\n" }, { "Id": "15095", "CreationDate": "2020-12-23T17:06:50.523", "Body": "

High temperature PTFE tape is rated up to 550\u00b0F, which is 288\u00b0C. I'm wondering if it would be useful for components on the hot end to prevent oozing. Has anyone tried it?

\n", "Title": "What happens if you use high temperature PTFE tape with the heater block?", "Tags": "|hotend|", "Answer": "

Whether this is even potentially beneficial depends on the type of hotend you're using. With one like the Ender 3 where the PTFE tube butts all the way against the nozzle, it's not useful, and if it seems useful it means your hotend is misassembled, since filament is never supposed to make contact with the top of the threads.

\n

For an all metal hotend with flaws in the mating surfaces, it's plausible that it might help, but it rather defeats the purpose of having all metal (high temp).

\n" }, { "Id": "15097", "CreationDate": "2020-12-23T21:09:16.677", "Body": "

I need a Prusa MK3S smooth bed and I am not able to get it unless I wait for a few month.

\n

But I can have a Heated Bed Cover for Ender 3/Ender 3 pro/Ender 5 3D Printer 235X235MM shipped to my door tomorrow moring.

\n

Can I use the above Creality bed on MK3S untill I get an original MK3S in a few month?

\n", "Title": "Can I use Creality bed on Prusa MK3S?", "Tags": "|prusa-i3|", "Answer": "

Heaters won't match.

\n

The ender3 is a 24\u00a0V machine. The Prusa 3 is a 12\u00a0V machine. Heater cartridges and Heatbeds are therefore not interchangeable.

\n

Build surfaces can be adapted.

\n

The Ender3 has a build surface that is a little bigger than the Prusa3, and thus you can, in a pinch, use an ender3 sized build surface and install it, possibly cut down, to fit onto the Prusa.

\n" }, { "Id": "15101", "CreationDate": "2020-12-24T13:18:42.023", "Body": "

Thinking this over, the best place to seal the hotend is between the throat tube and nozzle. What are the advantages/disadvantages of an integrated nozzle and throat to make this seal? Does a throat with a Teflon tube improve this seal? Is it reasonable to cut a gasket out of 3mm ID 5mm OD Teflon tubing to make the seal for M6 thread parts?\nI intend these questions to all address the main question to prevent a leaky hot end.

\n

\"Throat\"nozzle\"\n\"Integrated\n\"Throat

\n", "Title": "What technics are you using to seal between the throat tube and nozzle?", "Tags": "|troubleshooting|hotend|", "Answer": "

A seal between the heat break and nozzle should be achieved by tightening them while the hotend is hot. After assembling the (cold) hotend, heat it up to somewhat above the highest temperature you will print at and then tighten the nozzle some more. When it cools down again, you will have a very tight seal. If the nozzle still leaks after this tightening then it is defective.

\n

Using teflon is possible but will limit the maximum temperature you can use your hotend at safely and you won't be able to print some higher temperature materials.

\n" }, { "Id": "15104", "CreationDate": "2020-12-24T14:50:40.900", "Body": "

I'm getting weak prints on Ender 3 Pro with Cura after Cura upgrade.

\n

Prints on my Ender 3 Pro have been good until I was forced to update Cura due to having to update the OS on my laptop. Now my prints are weak even after using Infill 50\u00a0% from 20\u00a0%.

\n

Upgraded to Cura 4.8.0.

\n

I noticed that it seems my print settings were saved from the older non-working Cura version. I went ahead and printed an XYZ Cube with the same settings from my older Cura version where prints were printing fine/acceptable. It fell apart as I tried to lift it off the build plate so I printed a second XYZ Cube with 50\u00a0% infill from previous 20\u00a0% pictured below:

\n

\"enter

\n

The above print is noticeably weak, with layers not bonding or maybe something else.

\n

Below is an image of when an XYZ Cube printed with my older Cura version. It is not perfect but it is much stronger without visible separation and actually noticeably heavier than the grey XYZ Cube printed with Cura 4.8.0 :

\n

\"enter

\n

The green and the grey 1.75\u00a0mm PLA I use and shown in the photos are different brands but both printed very much the same with the older Cura version.

\n

I'm thinking this is an issue with the new Cura 4.8.0 that I'm using as nothing about my printing has changed besides the new Cura version assuming that my original print settings were saved which it looks to me that they were.

\n

Has anyone had a similar issue or is there a known fix for this?

\n
\n

Posting settings pics below:

\n

\"enter

\n

\"enter

\n

12/28/20 :\nWent to https://www.chepclub.com/cura-profiles.html and noted that they mention to use the Cura 4.8.0 Built-In profiles. Below are two pics of test print while using Cura 4.8.0's Built-In Standard Profile :

\n

\"Cura

\n

\"Cura

\n

12/29/20 :

\n

Screenshot showing Nozzle Diameter (still using the 0.4\u00a0mm stock nozzle) & Flow @ 100\u00a0%. Please let me know if these are not the correct settings you asked me to look in :

\n

\"enter

\n", "Title": "Ender 3 Pro Weak Prints after Cura upgrade", "Tags": "|ultimaker-cura|creality-ender-3|", "Answer": "

Total user error on my end. With the new version of this app somehow I did not catch that material selected on the UI dropdown was set to some custom material and not Generic PLA. Once I selected the proper material, all worked as expected.

\n" }, { "Id": "15105", "CreationDate": "2020-12-24T21:21:37.837", "Body": "

I have an Ender 3 Pro and I have been able to get it to work quite well, I installed a 0.8 mm nozzle to try the Vase mode in Cura. I re-leveled the bed and when I tried to print, I got some very strange behavior.

\n

When it starts the "purge line" along the side of the bed, the Z-axis is too high and instead of going down the left side and back up, it will go down the left side and then from left to right along the back edge and then stop and then just extrude filament. If I look at the beginning portion of the G-code it doesn't look any different than the G-Code for the other models I have printed except for in the initialization script it set the Layer height and Min Z to 0.32 instead of 0.2 and the code for drawing the first 2 purge lines is identical. I tried printing an older file that worked correctly and I got the same results where it went along the left side then along the back side and stopped and just extruded filament.

\n

It seems like something in the printer went wonky? What would cause this and what is the fix? A reset of some sort? And how is that accomplished?

\n", "Title": "Ender 3 Pro not interpreting G-code correctly?", "Tags": "|creality-ender-3|g-code|", "Answer": "

It ended up being something with the printer, I powered it down, let it set a few minutes and then powered it up and it printed correctly. Is there a way to "reset" it without the power cycle, and is this something common? Thanks for the response!

\n" }, { "Id": "15111", "CreationDate": "2020-12-26T12:34:09.387", "Body": "

Are there quantitative means of measuring nozzle wear? What steps do you perform to measure this? If so?

\n

I have some digital calipers, but I don't have much confidence in the measurements taken, as it's almost impossible to ensure the nozzle aperture is entirely free of filament residue.

\n

The context here is I've recently been trying out some carbon-fiber-infused PLA filament, but I would like to keep an eye on how much it wears the nozzle and get an idea of nozzle wear rates. I can then work out if the mechanical and aesthetic qualities of the CF filament are worthwhile.

\n", "Title": "Are there quantitative means of measuring nozzle wear?", "Tags": "|nozzle|fdm|maintenance|", "Answer": "

Based on CNC Kitchen measurements,

\n

\r\n \r\n

\n

the wear can be initially estimated by judging the overall length of the nozzle. Abrasive particles wear the nozzle very little on the channel (so the diameter doesn't change, for a while) but they wear out the outer surface in contact with the printed part, resulting in a shortening.

\n

Of course, once the shortening gets severe, the diameter will increase because you reach the inner chamber.

\n" }, { "Id": "15116", "CreationDate": "2020-12-27T00:35:40.840", "Body": "

I have an Anycubic Kossel with Trigorilla Motherboard, Mega2560+RAMPS1.4.

\n

I'm using Marlin 2_0_bugfix

\n

My Y connector is not working anymore so I would like to use E1 connector as Y.\nI found a very good solution here but for me for some reason it's not working.

\n

I did that modification what suggested in the pins_RAMPS.h file (for Y of course instead of X), then I compiled it with Arduino 1.8.13 and uploaded to my printer.

\n

After that I flashed firmware then Y (which I plugged to E1) is not moving anywhere. I have never plugged anything to E1 before, so I think the connector port must be good and working.

\n

What I am missing, what's my mistake?

\n

What I did:

\n
#define Y_STEP_PIN         36\n#define Y_DIR_PIN          34\n#define Y_ENABLE_PIN       30\n#ifndef Y_CS_PIN\n  #define Y_CS_PIN         44\n#endif\n\n\n#define E1_STEP_PIN        60\n#define E1_DIR_PIN         61\n#define E1_ENABLE_PIN      56\n#ifndef E1_CS_PIN\n  #define E1_CS_PIN        49\n#endif\n
\n", "Title": "Marlin: Switch Y to E1", "Tags": "|marlin|stepper-driver|arduino-mega-2650|kossel|", "Answer": "

If you followed the answer to switch steppers in firmware to the letter, but changing X for Y, this should work.

\n

I've looked into the pins files for you and concluded that your proposed changes should work.

\n

This leaves you with the following questions:

\n\n

Swap drivers and cables to troubleshoot the problem.

\n" }, { "Id": "15118", "CreationDate": "2020-12-27T02:36:01.657", "Body": "

I am interested in buying a larger printer, either a Sovol SV01 or a Sunlu S8 (or something else similar).

\n

What I am wondering is, can the control boards and LCD displays be changed to better units later on?

\n

How much are parts interchangeable?

\n

Can the frame, steppers, extruder, base-heater and power supply be run with any control board which has enough outputs? How does this work?

\n

I'm particularly interested in adding silent stepper drivers and BLTouch later on, neither of these units are very popular so not a lot of after market parts are out which list compatibility.

\n", "Title": "Are control boards switchable?", "Tags": "|extruder|stepper-driver|hardware|full-graphic-smart-controller|", "Answer": "

Yes. Stepper motors, heaters, temperature sensors, fans, etc... are all standard parts. Pretty much any control board can be used with the hardware from pretty much every printer.

\n

Some more expensive printers might use more exotic parts (especially temperature sensors), but on the budget printers you're looking at I would be very surprised to find anything non-standard.

\n" }, { "Id": "15120", "CreationDate": "2020-12-27T12:39:38.463", "Body": "

I'm pretty new to 3D printing, so I'm looking for some tips with following problem I have.\nI'm trying to print following model https://www.thingiverse.com/thing:4671256 on my Ender 3 v2.

\n

But as soon as the print reaches the body and the first layer parts are attached, the print comes loose from the bed and the print will be dragged by the nozzle.\nIf you look at the picture you can see the bending of the front paws of the sculpture. So they don't stay sticked onto the bed.

\n

I have changed the model to be 80\u00a0% in size, but also upped the bed temperature from 50 to 75\u00a0\u00b0C.\nRunning at 50\u00a0\u00b0C was even worse...

\n

My printed 20x20x20\u00a0mm test cube was flawless by the way.

\n

\"enter

\n", "Title": "Layer bending with first layer not sticking", "Tags": "|creality-ender-3|adhesion|", "Answer": "

Try a Brim (the model is rather tricky on FDM), which can help with bad adhesion.\nYour first laser appears coarse and not "smooched" as it should. This hints that you leveled either high, or you hit the portal off-angle. If it is just high leveling, a tiny turn "up" can help a lot.

\n

The model has some rather flat overhangs, like the mouth, chin, under the tails and lower body. I suggest printing that with support turned on.

\n

For PLA Filament, I use 200\u00b0C nozzle with 60\u00b0C bed, so unless you go far beyond that, your print should work.

\n

Last words: that the model you chose was made with resin printing in mind, so you will use some details.

\n" }, { "Id": "15124", "CreationDate": "2020-12-27T17:16:29.543", "Body": "

I am currently making an ironman helmet but running into a couple problems.

\n

Whenever I start my print it starts off fine, but then after it builds the bottom support and starts on the build itself things get really wonky. It seems that the build gradually builds away from the support and starts to float in the air.

\n

I also see an issue where the model starts to curve and the filament isn't sticking to the support instead it makes a straight line through the curve.

\n

Lastly the support column on the bottom left starts to clump up.

\n

I am using an Ender 3 printer and Cura.

\n

\"enter

\n", "Title": "Issues with my print", "Tags": "|ultimaker-cura|creality-ender-3|print-failure|", "Answer": "

You are experiencing layer shifting, please check the tension of the X-axis belt, driver current.

\n
\n

A more elaborated answer is found here.

\n" }, { "Id": "15127", "CreationDate": "2020-12-28T02:12:16.623", "Body": "

I haven't ran into the issue yet, but I am sort of expecting it to here soon towards the end of my print. Actually, I am not even sure if it should be something that I should be worried about or not.

\n

Basically I have some overhang on a helmet piece (it's a vent piece) and I see that there is a 90\u00a0% degree overhang. It doesn't look too big but just curious if this should be something that I should be worried about. I forgot to add support in the vent, it's my own fault and I'm definitely going to fix this in my next print for sure, but do you all think it will fail if I keep the current print going?

\n

\"Screenshot

\n

I use Cura, Meshmaker, and I have an Ender 3 printer.

\n

I'm just getting started with 3D printing so I wasn't sure if I should be concerned or not.

\n", "Title": "Overhang Question", "Tags": "|ultimaker-cura|creality-ender-3|", "Answer": "

I wouldn't worry about that overhang. It should settle in a few layers. Just make sure you have a decent cooling fan on it.\nGiven the fact that you are making a helmet, I assume you are going to sand and apply putty to the whole thing at least 2 times and then sand in increasing grit. A little filing on that overhang will be an easy task.

\n" }, { "Id": "15143", "CreationDate": "2020-12-29T13:19:13.127", "Body": "

I am trying to correct the x&y offsets in the Tevo Tarantula that I got from a friend in order that my printer is not printing over the end.\nWhat I got so far is that I need to measure and then enter the offsets into my configuration.h file, build the firmware and flash it to the board.

\n

Because I do not have and can not find the original firmware I found the repository from Jim Brown for the Tarantula.

\n

Looking into the top section of the config file provided by the repo, the board configured is

\n
#define MOTHERBOARD BOARD_MKS_GEN_13        // Original controller board with built in stepper drivers. Works with MKS BASE 1.3, 1.4\n
\n

However since the board in my Tarantula has 1.2 on it I checked the boards.h file for MKS but could not find the v1.2 :

\n
#define BOARD_MKS_BASE          40    // MKS BASE v1.0\n#define BOARD_MKS_BASE_15       405   // MKS v1.5 with Allegro A4982 stepper drivers\n#define BOARD_MKS_BASE_HEROIC   41    // MKS BASE 1.0 with Heroic HR4982 stepper drivers\n#define BOARD_MKS_GEN_13        47    // MKS GEN v1.3 or 1.4\n#define BOARD_MKS_GEN_L         53    // MKS GEN L\n#define BOARD_MKS_GEN_L_V2      54    // MKS GEN L V2\n#define BOARD_ZRIB_V20          504   // zrib V2.0 control b\n
\n

Any idea, what board configuration would be compatible for 1.2? Or is there any other way to correct the x&y offsets other then compiling them into the firmware?

\n", "Title": "What is the correct Marlin firmware setting for Tevo Tarantula with MKS Base V1.2 board", "Tags": "|marlin|firmware|tevo-tarantula|print-axis-offset|", "Answer": "

The MKS Base v1.2 is basically an Arduino MEGA2560 and a RAMPS1.4 on a single board.

\n

You can use the RAMPS pin designation.

\n

Note that the endstop to origin distances is explained in question How to center my prints on the build platform? (Re-calibrate homing offset).

\n

In Jims Marlin fork you can find this at:

\n
#define X_MIN_POS 0 - XTRA_BED_LEFT\n
\n

and

\n
#define Y_MIN_POS 0 - XTRA_BED_BACK\n
\n

Unfortunately, XTRA_BED_LEFT and XTRA_BED_BACK are defined as zero; this implies that the endstops define the origin (not likely) or it is left as an excercise for you to find out. This answer explains how you figure that out.

\n" }, { "Id": "15148", "CreationDate": "2020-12-29T20:53:37.697", "Body": "

I'm relatively new to SLA 3D printing and 3D printing in general. I got a budget SLA resin printer that I've been messing with for a few months now.

\n

So I had a few soft prints laying around that have not been exposed to any UV light. It was sunny out and I decided to bring them outside to harden them up a little bit under the sun. And of course I forgot about them! By the time I remembered, my prints got over cured to the point of being super brittle.

\n

Question: Let's say I properly cure a print (however that is). I'm not looking to paint it. It looks like the print could be accidentally over cured by taking it outside or sunlight shining from an open window. Is that true? Should I coat it with some kind of sealant if I'm done curing it?

\n", "Title": "Accidentally over curing a 3D print", "Tags": "|post-processing|resin|uv-printer|", "Answer": "

You are correct, you can over cure a print by leaving it out in the sun. Prints can even over cure just by the ambient light in a room from the sun (though it would take a while).

\n

One easy option to counter this krylon UV-Resistant Clear Coating which can be found at most hardware stores. I have used it but I found that it made my print a little softer this may have been because of the materials in the spray-can, or the type of resin I printed with.

\n

You should check out the answer posted to Clear coating resin 3D prints as it provides some other options you can try as well such as using a brush on option.

\n" }, { "Id": "15162", "CreationDate": "2020-12-31T05:12:19.470", "Body": "

I've printed the dog that came on the SD card of my new Ender 3 pro, and everything worked great. Today, I turned on the unit and plugged in a Raspberry PI Zero to the Ender's USB port. The rPi is running the latest version of OctoPrint.

\n

Now, when I try to Auto home, I hear a very brief sound of a motor staring to move on an axis, and then the Ender halts. I've power cycled it several times. When I trying to use the panel on the Ender to move on any axis the same thing happens: it sounds like it's starting to move, and then never actually tries to move again until after I power cycle. This is the same no mater which axis I try to move.

\n

I've reinstalled the lated firmware on the Ender. I've opened up the machine to make sure that all of the cables are attaches to the motherboard. I've checked the actuation of the stopper switches and unplugged the switch and motor connectors.

\n", "Title": "No movement on any axis on Ender 3 Pro", "Tags": "|creality-ender-3|troubleshooting|", "Answer": "

I upgraded to the latest firmware, and now everything works again.

\n" }, { "Id": "15177", "CreationDate": "2021-01-02T00:02:00.683", "Body": "

I've searched, and surprisingly I am not able to find this design on Thingiverse:

\n

\"enter

\n

Where can I find this 3d model?

\n", "Title": "Looking for this stl design for a cellphone clamp", "Tags": "|stl|part-identification|", "Answer": "

This file simply is not on Thingiverse. Not all files are on Thingiverse.

\n

A Google reverse-image-search for that picture in all size told me that the image stems from an all3dp article, and they have a reference link to Pinshape as it is presented here:

\n

\"As

\n

The file name of the linked picture is universal-phone-tripod-mount-3d-printing-155113.jpg

\n

Taking that as a search term lead me to pinshape model 37196: 3d printed universal phone-tripod mount by jakejake

\n" }, { "Id": "15180", "CreationDate": "2021-01-02T00:50:37.960", "Body": "

I keep having a recurring problem with my ender 3 pro. The bowden tube keeps popping off here (pictured)

\n

\"enter

\n

I've read elsewhere online where people are having a similar problem, i.e. the ptfe tube is actually popping out, but I don't know if thats the case here. It's staying attached to the metal coupler, but that metal coupler is unscrewing during the course of the print and falling out. Any tips to fix it? New one? Some sort of loc-tite to get it to not unscrew? Any ideas?

\n", "Title": "Ender 3 Bowden tube popping off", "Tags": "|creality-ender-3|", "Answer": "

The fitting can be defective and no longer hold upon the PTFE tube anymore. In that case, you need to cut off a short piece of the tube and most likely replace the fitting.

\n

If the metal coupler unscrews you can fix that by properly screwing it tight. If it does not stay put after applying some torque upon it, then the screw thread is broken, and the whole extruder gearing-setup needs to be replaced. An aluminium swap part costs about 10 to 15 \u20ac.

\n" }, { "Id": "15184", "CreationDate": "2021-01-02T21:30:34.637", "Body": "

I have a base model Ender 5 with the silent v4.2.7 board.

\n

Earlier this week during an overnight print, I came in to find it unfinished and powered off. I turned it back and it complained about an outage. When I started warming the hotend, it squealed and turned itself back off and then back on.

\n

Now anytime, I try to warm up the hotend, I get the same.

\n

I set up the printer to give data me debugging data via the console in Pronterface and in Simplify3D. I don't receive any kind of error, the printer just powers off and then back on.

\n

I'm hoping that the thermister is just dead and I can replace it easily enough. What I'm afraid of is that the board itself is dead. BTW, I am able to warm up the bed with no problem, just not the hot end. Any insight would be appreciated.

\n", "Title": "Dead thermister or dead board on Ender 5?", "Tags": "|hotend|thermistor|creality-ender-5|", "Answer": "

"Heater core" wires are not the same as thermistor wires, are they?

\n

Check if thermistor is properly connected?

\n

Reading -14 C in my case always pointed a disconnection of thermistor. (I didn't had a case of broken one so far.) In case of short it would probably show som max temperature (my guess, I dont want to try), but it is always good to check if wires are do not touch each other and are properly insulated?

\n

Measure thermistor's resistance

\n

If you are able to use multimeter, then:

\n\n

My spare shows 97k Ohms in room conditions. So value should be high (you may refer to specs sheet of given type when in doubt).

\n

Check that hotend is actually powered

\n

If thermistor is OK, then indeed the hotend powering may be broken. Ths definitely require skills and may introduce many risks to electronics and maybe even to your health - but if you are skilled enough, you may measure with voltmeter if there is correct voltage provided to the hotend. For example the MOSFET on the board may be broken (I already replaced one in the past for my heated bed).

\n" }, { "Id": "15186", "CreationDate": "2021-01-02T22:15:20.677", "Body": "

I am new to 3D printing and have a newish Eryone Thinker SE. Its been printing fine with some Eryone Glitter Black (EGB) and I'm happy with the results.

\n

The spool has now finished and I'm trying to use some black Tinmorry and/or Sunlu - both rolls are displaying the same issue.

\n

I can load and purge the new filament just fine but starting a print immediately after results in no filament being extruded.

\n

I have unloaded/reloaded, purged and retried a few times to no success. I also switched back to the take end of the EGB with success.

\n

Extruder temperature is 205\u00a0\u00b0C. Is this just too cold for the new spools?

\n", "Title": "Purge okay but not extruding during printing", "Tags": "|filament|temperature|", "Answer": "

So, I was able to fix this (or it fixed itself overnight as these things sometimes do) with a 5\u00b0C increase in extruder temperature to 210\u00b0C. The Sunlu PLA filament specifies 200\u00b0C - 230\u00b0C as its print range.

\n

I followed the advice of a IRL friend who suggested "What happens if you raise the z a bit so you can see the nozzle and keep raising the heat until it starts to come out on its own. Check temp and then run a print at that temp."

\n

I only needed that 5\u00b0C increase before I could see it oozing. From there I was able to successfully print a calibration cube and further model.

\n" }, { "Id": "15194", "CreationDate": "2021-01-04T09:58:19.717", "Body": "

I've been trying to print some M14 female threads. The male threads I've printed were perfect and fit into a female metal nut, but for some reason I cannot get the female threads to "stick". This means that they end up being hard to screw into, or the male end (both metal and my printed versions) goes in wonky.

\n

I'm printing in PLA on an Ender 3 V2 at 200\u00a0\u00b0C on a 50\u00a0\u00b0C bed. I've tried various resolutions (down to 0.1\u00a0mm layer height) and they all exhibit the issue. Print speed is 50\u00a0mm/s, cooling is just the stock cooling and fan 100\u00a0% after the first layer.

\n

The actual file can be found here (note that it's slightly oversized vs a metric M14 female to allow for printer tolerances) and the follow image shows what I'm talking about:

\n

\"Example\"

\n

It's as though the nozzle is pulling the filament away from the wall, but I've tried Z Hop Retraction and combing (infill only) and that gives the same result.

\n

Answer: The accepted answer and the comments against the question pointed me in the right direction. A combination of turning off combing and reducing the hot end temp to 190\u00b0C gave me perfect threads in 3 consecutive prints.

\n", "Title": "Internal thread not sticking to inner wall", "Tags": "|creality-ender-3|adhesion|", "Answer": "

I do a lot with printed threads, and find this problem creeps up mostly when the filament is wet. Higher temperature (like 220) can partly compensate but it's better to dry your filament and store it properly to keep it dry.

\n

Assuming standard metric thread profile, though, these are also pretty severe overhangs for a concave extrusion (where motion of the head will tend to pull the way you're seeing) unless you use very thin layers. You probably need 0.16 mm or thinner to print reliably (independent of pitch, though finer pitch also needs finer layers for other reasons), and limited acceleration for outer walls (I use 500 mm/s\u00b2). If you slice with "outer walls first" turned off, you may be able to get by with thicker layers, provided you have everything else tuned perfectly to ensure the outer wall sticks to the previously extruded inner one (i.e. flow rate perfect).

\n

A contributor to your problem could also be underextrusion due to oozing during combing. You might try setting max comb distance very low (like 0.6 mm, around what I use) or turning off combing and see if that helps.

\n" }, { "Id": "15197", "CreationDate": "2021-01-04T14:04:54.560", "Body": "

One could make such a controller with 3D printer parts and building a case for them, but is it significantly less than just buying a compatible low-cost printer. The intent is to heat up the all-metal hot end to tighten the nozzle to the heater break.

\n", "Title": "Is it practical to build a separate hotend temperature controller to assemble hotends without taking up printer time?", "Tags": "|hotend|all-metal-hotend|", "Answer": "

You can get a suitable board for $20 or so, print a case for a few cents, and either repurpose an old PC power supply or buy a new one just powerful enough for the hotend (not bed) heater very cheap, so I think it's a lot less costly than a cheap printer, and cobsumes less space. But I'm not clear what you need it for.

\n" }, { "Id": "15213", "CreationDate": "2021-01-05T18:04:35.657", "Body": "

I want to write a couple of very simple G-code test cases in Marlin-Style to make sure my printer works properly. The idea is to test the movement system, then the end-stops, then the heating and ability to hold temperature. But Marlin already runs large, so I can't spare too many lines in the basic setup, if I want to integrate it into the firmware.

\n

The test I have in mind is this order of operations, covering the basis of each axis and the thermal control:

\n\n

What is the best way to set up such a test code in the least amount of lines? Did I forget a crucial test command?

\n", "Title": "Writing G-Code: length-optimized Test code", "Tags": "|troubleshooting|g-code|knowledgebase|", "Answer": "

I can get down to 13(14) lines using 6 commands, one of which is optional: G1 movements, G28 homing, M140 setting the bed temperature without pause, M109 to set the hotend temperature, G4 dwell to pause, and M300 to beep.

\n

The line in [] is optional, but the nozzle isn't safe to touch right away - waiting for the cooldown ensures touch safety.

\n
G1 Z20 F1000 ; Movement Test Z\nG1 X20       ; Movement Test X\nG1 Y20       ; Movement Test Y\nG28          ; Homing Test\nG1 Z20       ; Movement up (safety measure)\nM140 S50     ; Set Bed temperature to 50, directly goto next line\nM109 R180    ; Set Nozzle temperature to 180, wait for achieving\nG4 S60       ; Wait a minute (to see if nozzle temperature is held)\nM140 S0      ; "turn off" bed\n[ M109 R25     ; set nozzle temperature to 25C, wait for achieving ]\nM109 S0      ; actually turn off nozzle!\nM300 S440 P200 ; Beep\nG4 P200      ; short pause\nM300 S660 P200 ; Beep\n
\n" }, { "Id": "15214", "CreationDate": "2021-01-05T20:56:39.513", "Body": "

I'm having an issue with bed levelling since upgrading my stock Ender 3 pro to Marlin 2.0.7.2.

\n

The upgrade process seems to have gone through fine, I get the new options and it shows the FW version when I power on. My issue however, is that after I auto home the printer, disable stoppers and then level the bed, using a piece of paper, any time I attempt to print the nozzle hits the bed which stops all flow as there's no space for the filament to come out.

\n

This process is exactly the same as I used to do before the upgrade and I never had issues, only since the upgrade.

\n

For reference, I've manually adjusted all the wheels on the printer so there's a large gap (for testing) between the paper and the nozzle at any point on the bed (the piece of paper passes below the nozzle with zero resistance). However, when I attempt a print, it homes the printer and then the nozzle drops lower and crushes any paper I put between the nozzle and the bed (I do this while testing so it doesn't damage the bed), the paper is impossible to move by hand.

\n

Anyone know what I'm doing wrong here? None of the guides that I've found for installing Marlin 2.x on the Ender 3 Pro mention anything about having to make any changes to allow the Z axis to work like before so at a bit of a loss as to why this is happening.

\n
\n

Something that occurred to me. I use PrusaSlicer as my slicer and I noticed that it has the following lines in the custom G-code for the template that I always use that reference "Z", now admittedly I'm no expert with G-codes but could it be either of these that's causing the issue:

\n
G1 Z2 F240\nG1 Z0.28 F240\n
\n

Edited to add full prusaslicer template gcode below:

\n

Start G-code

\n
M83 ; extruder relative mode\nM104 S[first_layer_temperature] ; set extruder temp\nM140 S[first_layer_bed_temperature] ; set bed temp\nM190 S[first_layer_bed_temperature] ; wait for bed temp\nM109 S[first_layer_temperature] ; wait for extruder temp\nG28 ; home all\nG1 Z2 F240\nG1 X2 Y10 F3000\nG1 Z0.28 F240\nG92 E0\nG1 Y190 E15 F1500 ; intro line\nG1 X2.3 F5000\nG92 E0\nG1 Y10 E15 F1200 ; intro line\nG92 E0\n
\n

End G-code

\n
{if max_layer_z < max_print_height}G1 Z{z_offset+min(max_layer_z+2, max_print_height)} F600{endif} ; Move print head up\nG1 X5 Y170 F{travel_speed*60} ; present print\n{if max_layer_z < max_print_height-10}G1 Z{z_offset+min(max_layer_z+70, max_print_height-10)} F600{endif} ; Move print head further up\nM140 S0 ; turn off heatbed\nM104 S0 ; turn off temperature\nM107 ; turn off fan\nM84 X Y E ; disable motors\n
\n

Before layer G-code

\n
;BEFORE_LAYER_CHANGE\nG92 E0\n;[layer_z]\n
\n

After layer G-code

\n
;AFTER_LAYER_CHANGE\n;[layer_z]\n
\n", "Title": "Z axis issue after upgrade from stock FW to Marlin 2.0.7.2 on Ender 3 Pro", "Tags": "|creality-ender-3|z-axis|", "Answer": "

This seems like Z homing issue. Sounds pretty frustrating, so I will try to give some hints, though I do not have Ender 3. Also maybe my few questions will be helpful.

\n

Did you "adjusted all the wheels on the printer" after the Z position was zeroed? This could be crucial, especially that firmware may raise Z up after homing (e.g. to 4mm). The G-Code you have sent makes simple Z moves: rises to 2mm, then lowers to 0.28 mm above the sea level (and F is denotion of speed). And obviously this should happen AFTER homing Z axis, not earlier - could you check earlier G-Code for G28 call?\nIf you can manually move the bed out of heatend's range (to avoid crashing it) then I suggest to use own file containing only two commands to experiment with Z homing only, e.g.:

\n
G90             ; absolute positioning (just to be sure it is not incremental G91)\nG28 Z           ; homing Z axis\nG1 Z0.0 F100    ; firmware may have raised Z up slightly, so lower again to 0\n
\n

If homing would cause the bed accident, than I am unfortunately right - or:

\n

(Added after tracing real problem) Check if Z axis is not loose - e.g. coupler screws are tight - so Z axis cannot rotate on its own by gravity and wight of carrige. When Z steppers are enabled, you should not be possible to move carriage up and down with slight pressur of hands.

\n

Does the printer react to Z endstop? This is first thing to check, actually. You can you press it with finger during Z homing to check if it would react properly (Z homing stopped).

\n

If not, then... did you compile Marlin on your own? For example maybe the Z endstop is or should be inverted. More insight is needed then against the Configuration.h, e.g. file for Z_MIN_ENDSTOP settings. Would be good to have its version from previous firmware, or google some valid Ender 3 file to compare with.

\n" }, { "Id": "15218", "CreationDate": "2021-01-06T02:00:01.897", "Body": "

The image below I indicated where the point where the tip of my extruder returns after changing the layer height, I don't know if I am right to call this point "Start Point" ...

\n

\"enter

\n

The problem I am having is possible to see in the photo, because there is a slip of material and sometimes "webs" are created that compromise the quality of the print.

\n

I tried to modify the retraction parameters, such as: speed and retraction length ...

\n

Is there a parameter that I can modify to improve my print?

\n

The slicer software I use is the : FlashPrint_4.5.1 (because I have a FlashForge Dreamer NX)

\n", "Title": "Failure near the start point", "Tags": "|slicing|flashforge|flashforge-dreamer|", "Answer": "

I have recently looked into "print outer walls first" in an attempt to make the seam vanish. But turning that setting on creates a webby structure on the following area for my printer.

\n

Turning the setting off again (and reducing the outer wall speed to 30 mm/s) completely eliminated the ringing again.

\n" }, { "Id": "15223", "CreationDate": "2021-01-06T08:15:42.343", "Body": "

I want to build my own curing station for my resin printed parts.\nI know there are a lot of prebuilt machines but they do not completely satisfy my needs.

\n

I now want to know which material i can use to cover the chamber, but still be able to see inside.\nThe material should therefore block UV light but still be transparent.

\n\n

The questions is now, which material satisfies those points?\nYou can find reference on existing machines like the Elegoo Mercury Pro or the Formlabs Form Wash. Maybe somebody knows which material has been used by either one of them.

\n", "Title": "How to block UV light (transparent material)?", "Tags": "|diy-3d-printer|resin|sla|", "Answer": "

I contacted the manufacturers of curing stations directly and got some interesting feedback. Not all of them were keen on sharing their information, but two responded:

\n\n

I am sure they are using some additional UV blocking on the plastics with a coating, but thats all I could find out.\nThanks to @Greenonline for pointing me in the right direction!

\n" }, { "Id": "15227", "CreationDate": "2021-01-06T13:09:21.887", "Body": "

This question doesn't necessarily have one answer. I'm looking for a build surface that doesn't tear up when removing the PETG print, but still holds the PETG down and keeps it from warping. I am printing PETG with the z-height almost too high. Higher causes the PETG not to stick and warp or even release during printing. But, I still have trouble with PETG prints tearing my build surface. I've tried PEI and black build surfaces that don't identify the material. The build plate on my hot bed is glass.

\n", "Title": "What is the best build surface for PETG?", "Tags": "|petg|build-surface|", "Answer": "

Removable steel spring sheet with PEI textured coatings

\n

I have been using textured PEI string sheet for most of my prints, just love it.\nPrints attach so good but not hard to remove(with proper nozzole Z-offset).

\n

If the prints are small and seems hard to remove,heating up the bed helps a lot.

\n

Almost never used the smooth side, even with proper z-offset, it still damages the surface quite fast.

\n

Here's the link i bought before:\nRemoval Spring Steel Sheet Pre-Applied PEI+Magnetic Base

\n

This is my setup on a ender3pro:

\n

\"three\n\"removable

\n" }, { "Id": "15235", "CreationDate": "2021-01-06T18:58:40.763", "Body": "

I have two otherwise-identical machines (called "Lefty" and "Righty"), so I have a baseline to compare. I am running Marlin 2.0.7.2 on both machines. They are Creality CR-10 printers with BTT SKR e3 Mini v1.2 boards, with Marlin I compiled myself. Both machines are (as far as I know!) running the same exact firmware binary.

\n

When running the same G-code on both printers, Lefty prints fine. But, Righty fires the plastic through -- I need to set the flow rate to 50\u00a0% (!) to be correct. I have checked the esteps, both are identical at 415, and this produces 39.5\u00a0mm of 40\u00a0mm commanded when bumped through 1\u00a0mm at a time via the extrusion menu, even on Righty. The XY movement on both machines is 100\u00a0% correct.

\n

Volumetric extrusion is disabled in the firmware on both machines, and disabled in the slicer. Both machines are running TMC2209 drivers, set to Spread Spectrum at 650\u00a0mA for the extruder. Both machines are set to 1250 accel. The steppers are rated for 1\u00a0A, and they are not skipping steps.

\n

The machines were working side-by-side perfectly until I killed Righty's stepper driver with static, and replaced the board. No other hardware was modified.

\n

Why do both machines behave the same with identical G-code, until extrusion moves? Why does Righty try and extrude nearly double what it should be, only during printing, when Lefty works perfectly?

\n", "Title": "e-steps are wrong only during printing moves", "Tags": "|firmware|extrusion|", "Answer": "

Annoyingly, this went away with a replacement board. I don't have a better fix.

\n" }, { "Id": "15247", "CreationDate": "2021-01-07T23:24:09.293", "Body": "

The Y-axis belt just broke on my Ender 3 v2. I believe that it was over tensioned from the factory. When I initially assembled the printer, I noticed that the Y-axis tensioner was tightened almost all the way. The belt itself felt very stiff. The X-axis belt, which I installed upon assembly, didn't require a lot of tightening. I have ordered replacement belt material and clips to make new belts.

\n

What is the proper tension for both the X- and Y-axis belts?

\n", "Title": "Proper belt tension for Ender 3 v2", "Tags": "|creality-ender-3|maintenance|belt|", "Answer": "

Mine came from the factory tight enough that it doesn't sag at all, but not so tight that it feels hard or stiff or difficult to deflect a bit with gentle pressure. With the bed pushed all the way to the back, I can push the y-axis belt to the side about 1/8" with gentle pressure, and it feels like that's about as far as it's going to go even if I were to push harder. Seems to work really well without a lot of unnecessary force.

\n" }, { "Id": "15256", "CreationDate": "2021-01-08T22:44:57.527", "Body": "

I have an Ender 3 Pro that I installed a BLTouch sensor on. After I auto-home, I level bed and it deploys the probe in the right spot, but doesn't go down far enough, then goes back up and stops leveling.

\n

I am using the wiring for BLTouch for Z homing on SKR mini E3 v1.2. I am using Marlin Bugfix-2.0 with BLTouch and a few other things enabled.

\n\n", "Title": "BLTouch not touching bed while leveling", "Tags": "|creality-ender-3|bltouch|bigtreetech|", "Answer": "

I think I fixed everything. What I did is I used this guide <youtu.be/y_1Kg45APko> which was a guide by makes'n'breaks on setting the probe z-offset using pronterface and then reverted to the original firmware that was on this page<github.com/bigtreetech/BIGTREETECH-SKR-mini-E3/tree/master/\u2026> Which was the marlin firmware for bltouch for z homing for my mainboard.

\n" }, { "Id": "15263", "CreationDate": "2021-01-09T14:39:26.273", "Body": "

From my understanding, the power of heater must higher than heat dissipate to ambient air so the bed can heat up. The reason why a bed heats up too slowly is due to its heat capacity compared to heater power.

\n

As the heater is a resistive load, I think we can put higher voltage to get more heating power.

\n

The PCB heater has two parts: copper and laminate. The reason of failure is that the copper can come off the board due to high temperatures. In this case we can control temperature with firmware. The questions are:

\n
    \n
  1. Will this method work?
  2. \n
  3. What can go wrong or what is the risk of this method?
  4. \n
\n", "Title": "Can we apply more voltage to a PCB heated bed to get faster heating?", "Tags": "|heated-bed|electronics|", "Answer": "

You can add a boost converter to the bed's power supply. Assuming that you are using an external MOSFET to control the bed (and you should). Verify what the MOSFET's max voltage rating is, then obtain a boost converter and configure it to give a voltage that is 10% less than the max. I once used this technique when driving a 24V hot end heater using a 12V power supply. Be forewarned, however, the boost converter I was using exploded during use one day.

\n

\"Commerical,

\n

Or you could get an additional higher voltage PSU and use that as the bed's heater. They make them up to 80V I think now.

\n" }, { "Id": "15274", "CreationDate": "2021-01-10T22:31:08.937", "Body": "

I first want to say thank you for taking the time to read this. I've been trying to print out some parts for another project of mine. These parts use up around 70% of the bed so they are fairly large, unlike the smaller trinkets and things I normally print.

\n

However, I have yet been able to print one of these larger parts without any layer shifting. The part I've been trying to print is a lid to the bottom of a case. It's square, with rounded corners, about 160mm x 160mm.

\n

It will print the first layer without shifting most of the time. However, upon starting the second layer it will almost have at least one layer shift. I have tried a number of things to remedy this, which I will list further down this post.

\n

The layer shifting seems to be unrelated to jerk, acceleration, bed shifting, and z offset at the very least. There is an audible clunk sound that can be heard when it does this. In the image I've linked, you can see the correct path of the print head through the green arrows. However, upon a layer shift, it follows the path of the red arrows. It never shifts during a direction change, only when it is following a straight path. It's almost as if one of the motors gives up for a second. I am aware the print is under extruded, this is one of my trials hoping it was merely the nozzle clipping against the print.

\n

\"enter

\n

Allow me to share with you my setup:

\n\n
\n

Now let me share my tried solutions.

\n\n
    \n
  1. I tightened all the frame screws, hotend screws, fan screws, grub screws. Nothing moves even a millimeter.

    \n
  2. \n
  3. The belts were tensioned so that they can be strummed like a guitar string when I move either the hotend or bed to its end stop.

    \n
  4. \n
  5. I checked my aftermarket fans to ensure they are working. The mainboard and hotend fan run nonstop at their optimal 12V. The PSU fan turns on and off as it needs to cool the PSU...

    \n
  6. \n
  7. The cables are nicely managed in the mainboard compartment and I'm sure that the larger fans provide it plenty of airflow. Not to mention the printer was raised by the use of squash-ball feet.

    \n
  8. \n
  9. I have slowed my print speed from 75mm/s to 60mm/s with this print. This means a 30mm/s inner/outer wall along with 60mm/s infill speed and 150mm/s travel speed. Acceleration and jerk are at their defaults: 500mm/s^2 and 10mm/s.

    \n
  10. \n
  11. I've adjusted my Z offset so that it slightly under extrudes to prevent the filament from bunching up.

    \n
  12. \n
  13. I've calibrated my e steps.

    \n
  14. \n
  15. I've formatted the SD card and changed my firmware from Smith 3D's 4x4 high speed to 5x5 high speed.

    \n
  16. \n
  17. I've updated Cura to the latest version 4.8 and ensured my hero me home offset is correct.

    \n
  18. \n
  19. I've tried moving the model to different corners in Cura and ensured the software's jerk and acceleration control are disabled.

    \n
  20. \n
  21. I've checked the bed movement with its clips, it doesn't move easily and both clips have very good tension.

    \n
  22. \n
  23. I've checked the stepper driver voltages (I did not change them). E: 1.34V Z: 1.16V Y: 0.99V X: 1.15V.

    \n
  24. \n
  25. I have Z hop enabled with a 0.2mm hop, no issues or noises when it moves across the print.

    \n
  26. \n
  27. I've purchased a filament dry-box from SUNLU along with a new spool of SUNLU black filament. The dry box has Capricorn PTFE tubing that leads the filament straight into the extruder. There is very little friction inside the tube.

    \n
  28. \n
  29. I've slightly tightened the tension on the extruder although I didn't have problems with it skipping.

    \n
  30. \n
\n
\n

Again, I'd like to reiterate, nothing is loose on this printer, it doesn't skip when it rapidly changes directions, only when following either a straight or diagonal line. The printer movement is very smooth and near-silent with all of my modifications. It's frustrating to me that I can't solve this problem. I feel like the printer has a mind of its own. I have begun to suspect either the mainboard or something else hardware related is amiss. However, I have chosen not to mess with it until posting this. If you've read through this, I thank you for your patience and time reading it.

\n
\n

Any advice or possible causes to this problem are welcomed. I am at a complete loss and am desperate for a solution.

\n", "Title": "Ender 3 V2 Layer Shifting, No Apparent Reason, Desperate", "Tags": "|print-quality|creality-ender-3|layer-shifting|", "Answer": "

Not sure if you're still actively working on this but my setup mirrors yours almost perfectly, I added an all metal extruder, Micro Swiss hotend, a BLTouch, yellow bed springs and Capricorn tube and got great results but needed more cooling so I changed to the Hero Me Gen 5 using 2 5015 blowers and a Noctua 40x20 fan with a buck converter set to 12\u00a0V and noticed I had shifting so I printed 35\u00a0mm standoffs and wired another Noctua fan, 80x25. It helped reduce shifting but still not perfect and I as well has combed through every possible issue.

\n" }, { "Id": "15281", "CreationDate": "2021-01-11T09:34:20.120", "Body": "

I'm struggling with bed adhesion for nylon on a glass bed (122 \u00b0C measured) in an enclosed chamber (45 \u00b0C near the front, likely more on top of the print bed). I used a glue stick to enhance adhesion, but after around 20 minutes the print comes off the bed.

\n

I tried a no-brand glue stick and a Pritt glue stick.

\n

Now I wonder whether they are suitable for the purpose, because nylon should really be printable in these conditions. Maybe the glue cannot hold those 100+ \u00b0C temperatures.

\n

How to find out whether a glue stick is PVA-based and suitable for nylon (or polycarbonate, ABS) printing?

\n", "Title": "Are all glue sticks PVA-based? How to find out?", "Tags": "|adhesion|", "Answer": "

Not all Glues are PVA based. There is one glue manufacturer call CGMaxed that makes an alcohol based PVB glue. Sold on Ebay exclusively for 22 dollars per 6 oz. I use it every day when my supports fail to stick or having levelling issues.

\n" }, { "Id": "15286", "CreationDate": "2021-01-11T18:43:41.373", "Body": "

Hopefully this isn't against the rules as it may fall under "opinion" more than a concrete answer, but I'll ask anyway.

\n

I live near Boston and it gets cold here. My Ender 3 Pro is out in a detached garage with no heat and I'm pretty sure the garage getting down to about -12\u00a0\u00b0C (10\u00a0\u00b0F) has ruined a few prints for me. Also I'd love to print in ABS at home as well and I think I'd need an enclosure for that.

\n

Now, the printer technically already has an enclosure. It's in a box made out of plywood with some clear acrylic doors on the front, but I think the box itself has too much extra room and doesn't retain heat well enough (thin plywood?).

\n

So my question is, for anyone with experience printing in cold environments, what are some cheap/easy ways to keep the enclosure warm, preferably as safe as possible since I can't always watch the print. I have a small space heater in the garage but I turned it off last night because I wasn't sure how safe it'd be overnight and didn't want to start a fire.

\n

Thoughts on stuff like "reflectix" liners for the enclosure that would help retain heat, small heaters, IR lamps, etc...? All preferably on the cheap side.

\n", "Title": "Safe, cheap, heated enclosure solution for New England garage", "Tags": "|heat-management|enclosure|", "Answer": "

The easiest step would be to add some isolation on the outside of the box. If you have space inside, there might be a good spot to store some non-flammable insulation, for example rock or glassfiber wool.

\n

A different material might also be possible - firebrick is not only non-flammable but also a very good insulator! about 2-3 inches of firebrick can contain the heat produced by a tempering oven while the outside is cool enough to be safe to touch with gloves.

\n

However, you should install an extra thermosensor for the chamber temperature - and make sure that the printer enters print halt mode once the temperature in the chamber gets above a critical temperature to try to mitigate fire risks and possibly start a chamber-cooling protocol - which might include aborting the print or activating coolers that rapidly cool down the chamber.

\n

As you pretty much are going heated chamber, you might find a spot when you might want to use a flexible hose to pull non-chamber air to supply the parts cooling fan.

\n

Also, possibly relocate as much of the electronics to a compartment separate from the printer's heat chamber to ensure the electronics don't get cooked and can be supplied with cooler air.

\n" }, { "Id": "15309", "CreationDate": "2021-01-13T18:52:46.597", "Body": "

I am working on refining my PMMA printing process and I am trying to print as close as possible to the theoretical 100\u00a0% density of the material. I am making good progress and am slowing increasing the extrusion multipliers to try and find a good set of print settings.

\n

I recently have run into an issue where the bottom \u22481/3 of the test cylinder is coming out rather distorted (see picture). The problem seems to sort itself out as the finish around the top of the cylinder looks fine. Do any of you have any experience with PMMA or have seen this problem before?

\n

My initial thought is that it is heat driven and as more material is added it becomes less of an issue as there is more mass to dissipate the heat. For reference I am printing on an Raise3D Pro2Plus, for my prints I am running the extruder at 250\u00a0\u00b0C and the heated bed at 100\u00a0\u00b0C. I am eventually going to need to print rather large cylinders and would like an even surface finish along the length of it.

\n

Test Cylinders, right and left show distortion on the bottom and the center shows a bizarre print where half of the cylinder was fine and half came out very jagged.

\n

\"Test

\n", "Title": "PMMA print distortion and inconsistencies", "Tags": "|print-quality|troubleshooting|print-material|", "Answer": "

Commercial PMMA may soften already at 85 \u00b0C.

\n

The higher you go away from the bed, the colder the air is, so the issues solves itself.

\n

Try lowering the bed temperature, at least to exclude that as cause.

\n

The second cylinder shows the very common issue related to uneven cooling. Probably your part cooling fan is blowing only from one side. Try using a better fan shroud or add a second part cooling fan.

\n" }, { "Id": "15317", "CreationDate": "2021-01-14T14:32:35.767", "Body": "

Let's say, I want to print a box for putting game tokens in.

\n

It is an empty cube, but the top layer is missing.

\n

I do not need full walls. It could have holes in it resulting in a mesh structured wall, like a fence or a shopping cart.

\n

What pattern should I use for the best object stability and print speed?\nWhat programs can I use to design this? (I do not want to manually add 100 holes in my design).

\n", "Title": "Improving print speed by adding holes", "Tags": "|3d-design|speed|", "Answer": "

Definitely the speed will be worse after adding holes (or the quality otherwise, if object's skin is printed fast), though some filament savings may appear.

\n

But regarding "object stability": because adding holes will normally cause the slower printing (because of "skin structures" around holes), then walls might get bit stronger or have better layer adhesion (comparing to line speed, the material, the printing temperature, etc.). Then stability might indeed improve, especially when walls are thin. Or may be reduced, depending on the pattern :) Also number of walls may count (n * nozzle diameter) or lack of skin overlap - then printing holes would support consistency of the structure. Thus the question of stability may be significant. Construction sience would most probably lead to triangles.

\n" }, { "Id": "15323", "CreationDate": "2021-01-14T22:32:30.443", "Body": "

I was having an issue with flat sheets raising up during initial layers. I had my bed at 80\u00a0\u00b0C and kept raising it thinking it was an issue with cold. After reducing the temperature to 50\u00a0\u00b0C the sheets sit flat now but sometimes sections come loose. I ended up using masking tape and a glue stick and everything finally seems to print flat and keep adhesion. I am using 210\u00a0\u00b0C hotend but I might try lowering it to 205\u00a0\u00b0C.

\n

How do you check if your printing temperature is too hot/cold or if your bed temperature is too hot/cold? None of the articles I read ever said that a bed too hot would cause PLA to rise but that's what happened.

\n", "Title": "How to tell if PLA temp is too hot/cold", "Tags": "|pla|build-plate|temperature|", "Answer": "

Too Hot

\n

If you're printing too hot (with any filament, not just PLA) you're going to see stringing and blobs/oozing because the material is getting runny and exiting the nozzle in an uncontrolled manner. Because it's uncontrolled, you will also likely see artifacts showing up in your prints.

\n

You might also see your filament burning. Instead of coming out of the nozzle as whatever color it should be, it will look brown or discolored because it was overcooked.

\n

If your bed is too hot, you might start to see "elephant's feet" where the lowest layer(s) are being heated to the point of becoming soft and the weight of layers on top of them are pushing down, causing that layer to "pooch out."

\n

You might also have problems removing the print from the print bed's surface because the plastic has seeped into the details of the print surface and hardened, essentially welding the part to the surface.

\n

Too Cold

\n

If you're printing filament that is too cold, you're going to run into an issue where the material being pushed into the hot end is not getting melted sufficiently. This means that pressure will build up in the hot end that can't be released through extruding material through the nozzle. If you've ever tried to pipe frosting using a bag/tip and the frosting was too thick, you'll know what I'm talking about.

\n

When material is being fed into the hot end but not being allowed to flow out of it, something has to give. That thing is your extruder. It has a wheel with little teeth on it to grip the filament and feed it into the hot end (or bowden tube which leads to the hot end). It can exert a certain amount of force on that filament. When the hot end's back pressure builds to the point that it becomes greater than the extruder's force, it will start skipping.

\n

Imagine trying to push a large, heavy object. Your feet will begin to slip as your the force of your exertion overcomes the friction between your feet and the ground. That's what will happen to your extruder. It will make clicking/clunking noises as the extruder unsuccessfully tries to push the filament through and the friction between the teeth of the gear and the filament is overcome. This is called grinding.

\n

If your bed is too cold, you simply just end up with problems getting the print to adhere to the surface.

\n
\n

Warping

\n

In your particular case, you're describing warping. Remember from physics 101 that cold things contract and warm things expand. Your print bed is warm, and so, too, are the first layers that are near it because the bed's heat is transferring up into them and keeping them warm.

\n

Obviously, your active (topmost) layer will also be a bit warmer as it as just come out of the hot end. However, in general as you move higher up away from the printed bed, the printed layers get colder. Because they are getting colder, they are undergoing thermal contraction.

\n

This creates a thermal gradient where layers go from greater thermal expansion to greater thermal contraction. The combination means that the bottom of the print will start to curl up (away from the expansion and towards the contraction).

\n

This is not an issue with your printing temperature. It's a problem with your ambient temperature.

\n

The easiest way to fix this issue is to put your printer in an enclosure. This isolates the air immediately around the printer from the rest of the air in the room. Because your heater's bed and nozzle are throwing out a lot of heat, they will heat up the print chamber quite a bit (mine typically runs over 30 degrees celsius, even in the dead of winter).

\n

Because the ambient temperature in the print chamber is so much warmer than the outside air, that temperature gradient is much, much smaller. As a result, warping will stop becoming a problem.

\n" }, { "Id": "15335", "CreationDate": "2021-01-15T16:23:02.720", "Body": "

The following creates an object with two slots:

\n
 translate([-40,-40,0]) difference() {\n     cube ([90,28,12]);\n     translate([12,-.01,5]) cube([12.7,28.02,14.02]);\n     translate([65,-.01,5]) cube([12.7,28.02,14.02]);\n
\n

How can the first main cube be extended to be a complex object of the syntax:

\n
cube([60,20,10],center=true);translate([0,0,5])cube([30,20,10],center=true);\n
\n

and then use difference to create holes?

\n", "Title": "How to combine difference with concatinate object", "Tags": "|openscad|", "Answer": "

The union syntax addresses this:

\n
translate([-40,-40,0]) \ndifference() {\n    union() {\n        cube ([90,28,12]);\n\n        translate([20,0,20]) \n        cube([12,12,12]);\n    }\n\n    translate([12,-.01,5]) \n    cube([12.7,28.02,14.02]);\n\n    translate([65,-.01,5]) \n    cube([12.7,28.02,14.02]); \n}\n
\n

It supports creating an object before the difference syntax is applied.

\n" }, { "Id": "15350", "CreationDate": "2021-01-17T19:25:42.857", "Body": "

After watching every possible YouTube video on the subject and reading any source available, and although I'm a PhD and quite computer savvy, I still can't make my Anet A6 (no probe) behave in terms of Z offset. I upgraded to silicone bed buffers instead of the stock springs; now my bed is ~5 mm raised, and I don't know how to proceed. Some observations:

\n\n

Summary: I urgently need a walkthrough for 6-year-olds. Make that 4-year-olds.

\n", "Title": "How do I set my Z offset?", "Tags": "|marlin|g-code|bed-leveling|anet-a6|print-axis-offset|", "Answer": "

So the new silicone buffers raised the bed by 5\u00a0mm? When this happens, you should raise the endstop also with 5\u00a0mm. Else the printer will go down to the Z endstop that is effectively 5\u00a0mm below the level of the bed. I guess the buffers cannot be compressed by 5\u00a0mm, so you need to move the endstop up to the level your buffer compression is in reach of.

\n

No software offset will work (for your current setup: homing on the bed surface does not work as the switch need to be triggered prior to having any offset in play) other than a hardware change or compression of the buffers of 5\u00a0mm. It would only be possible to use a software offset when the nozzle homes off the bed surface (next to the bed). The only thing you would have had to do is add in your start G-code:

\n
G0 Z5   ; Move the head to 5 mm \nG92 Z0  ; Call this Z = 0 \n
\n

If #define Z_SAFE_HOMING is enabled, you should comment the line in the configuration file to make it home Z at the homed X, Y position.

\n

I will not go into all G-codes, details are read on the G-codes Wiki pages and Marlin firmware G-codes, these won't be able to help you out unless you fix the homing on the bed surface. Currently, you need to do a hardware fix, your endstop is below the surface level of the bed. Alternative is to remove homing Z above the bed surface and redefine the Z offset. A hardware fix is a better solution, and if you manage to print a fancy Z endstop holder and counterpart with a screw you will be able to level the bed more easily.

\n

E.g. M428 can set an offset, yes, but, it needs a reference; that reference is the homing reference or the current position. The current position of a printer that is just turn on is meaningless, it can be everywhere in the print volume. So you need to trigger the endstops first, that is not possible when it is not reachable (without compressing the bed).

\n" }, { "Id": "15362", "CreationDate": "2021-01-18T22:27:52.223", "Body": "

Being new to 3D printing, I have started printing different files from Thingiverse to better my experience. I have noticed on some YouTube videos that certain items/models/STLs should be oriented a certain way on the build plate.

\n

What things should I take into consideration when placing items on the build plate?

\n

Printer: Ender 3v2 (stock)
\nSlicer: Cura

\n", "Title": "Considerations for placing STL on build plate", "Tags": "|ultimaker-cura|3d-models|", "Answer": "

Orienting items

\n

Orienting items on the build plate should consider:

\n\n

So most of it comes from particular shape. Try to look at thing from different sides, spatial awareness helps a lot.

\n

Placing items

\n

Also, placing items on the build plate should consider:

\n\n

So most of it comes from the experiences with particular printer - its technical parameters and observations. Many times it is only a choice - like simple PLA prints sticking well.

\n" }, { "Id": "15370", "CreationDate": "2021-01-19T14:33:41.733", "Body": "

I've recently got an Elegoo Mars-like resin printer which is called Voxelab Polaris. So far, I have a lot of success with it, but one piece is proving really tough to print and I'm hoping for advice!

\n

These Settlers Of Catan tiles are natively about 90x97\u00a0mm.

\n

It's too big to print flat on my 130x82x155\u00a0mm print bed, so I've been trying to print it at a variety of angles including vertically. When printed vertical, it tends to warp along the long axis, and when done at an angle including supports there's not enough landing area to stick it to the print bed.

\n

I've tried printing two together back-to-back, but in Chitubox I can't find a way to add supports between two pieces on the bed (only from each piece to the bed).

\n

Anyone got any insights about how I might print this in resin?

\n", "Title": "Resin printing on Voxelab Polaris", "Tags": "|resin|", "Answer": "

If you want to print flat, you could easily print all parts at 90\u00a0%: the bounding box dimensions now are 81.16\u00a0x\u00a088\u00a0mm - and as a result, you can print some other parts flat next to it but not more than one tile at the same time. Remember to print all cities and roads at 90\u00a0% too.

\n" }, { "Id": "15381", "CreationDate": "2021-01-21T09:23:48.733", "Body": "

I'm printing with Geeetech A20 printer, PLA.

\n

I've started printing the same test piece while changing the following parameters:

\n

Temperature from 215\u00a0\u00b0C to 195\u00a0\u00b0C (lowering by 5\u00a0\u00b0C on each print)

\n

Flow multiplier from 100\u00a0% to 120\u00a0% (increasing 5\u00a0% on each print)

\n

Speed on two first prints was 60\u00a0mm/s and on two last 40\u00a0mm/s

\n

Bed leveling is perfect, the hardware seems to be in tact, the nozzle is clean and still the walls on the outer curve print well all the time, but in the inner circle they seem to be under extruded (even though flow is up to 120\u00a0%)

\n

I am planning on going on with these tests until I get the perfect print, but I am hoping somebody can guide me on what to try next.

\n

I've been reading about the issues on so many articles that now my head is a mess and I just need some clear instructions.

\n

\"enter\n\"enter\n\"enter\n\"enter\n\"enter\n\"enter

\n", "Title": "Top layer and walls have gaps even with 120 % flow (Cura)", "Tags": "|underextrusion|geeetech|", "Answer": "

You are lowering temperature and increase the flow at the same time - which is contradictory (filament is less runny, but you pump more of it). Though 195 \u00b0C seems to be very safe for printing PLA at 40 mm/s. Just doublecheck that when you disable motors, and push filament with hand, it melts quickly and easily goes out. Otherwise you will deal with temperature-related issue.

\n

Outer walls are usually printed slower, and even when underextruded, part of line may appear constant thanks to leaking filament (it has time). Next retraction will cause stop leaking, and the other wall is printed with almost no pressure. Maybe your constant issue with "inner circle" is just a consequence of such moves.

\n

My suggestions:

\n\n" }, { "Id": "15385", "CreationDate": "2021-01-21T16:46:23.413", "Body": "

Recently I\u2019ve been having trouble leveling my bed on my Creality Ender 3 Pro. It\u2019s not that it\u2019s hard to level or anything, it\u2019s that it won\u2019t stay level for more than a day or two. I\u2019ve been leveling it the same way as always with a piece of paper and I\u2019ve been sure to disable steppers. So if anyone knows what I can do about this, please let me know.

\n", "Title": "What\u2019s the best way of leveling the bed?", "Tags": "|creality-ender-3|bed-leveling|calibration|", "Answer": "

Generally, there is no best way, there are only the ways that work for you.

\n

I use a feeler gauge, mounted on a detachable holder. I home to 0, lift the Z by 1\u00a0mm, and then level the bed to the same number on the screws and corners. Then I home back to Z=0 and perform an extrusion test, and if that is not satisfactory, use a different number for the leveling. The number on the plunger does not generally read 1\u00a0mm, as there is an offset between the 0 of the gauge and the printer.

\n

When I don't level from 0 or am in a hurry, I usually only alter leveling by doing about 2-5\u00b0 turns on all the screws to adjust the 0-height via the nuts, using the visual of the printed line as my aide. This only works because I have the experience how a good first layer should look like.

\n

Another typical method that I stopped using once I had the feeler gauge is the paper, but I found it inconsistent.

\n

Do not do these:

\n\n" }, { "Id": "15392", "CreationDate": "2021-01-21T21:49:53.873", "Body": "

I've been dealing with 3D printing for 1.5 years, but now own a CR-6 SE myself since the beginning of 2021. Most things are already quite clear but for 2 days I have had a problem with the adhesion of the prints.

\n

Nearly all prints I have done so far used the filament shipped with the printer (PLA 1.75) and they came off the building plate after some cooling time by themselves. I used the default printer settings for PLA: 200\u00a0\u00b0C nozzle temperature, 60\u00a0\u00b0C printing bed.

\n

Then 2 days ago the prints began to not stick to the bed anymore and I thought this could be because of dust and from touching the bed. So I cleaned the bed with IPA. The microfiber towel was yellowish afterward - so I thought that this must have been printing residues. Since then every print is kind of "welded" to the bed. There is no chance of loosening it without more IPA or way too much force.

\n

I already tried:

\n\n

Today I also tried a spool of brand new PETG, with the following recommended settings: 240\u00a0\u00b0C nozzle temperature, 80\u00a0\u00b0C print bed - but the problem stayed the same.

\n

Am I doing something wrong? Did I destroy the "Carborundum" coating (silicon carbide) of the glass plate?

\n", "Title": "CR-6 SE glass build plate - no lifting possible", "Tags": "|adhesion|build-plate|creality-cr-6|", "Answer": "

I have had similar issues, and I have learned three tricks:

\n
    \n
  1. Some slicers (i.e. Cura) let you specify the height of your first layer, and I have found that slowly incrementing up from 0.16\u00a0mm by 0.2\u00a0mm to find the sweet spot of adhesion without warping. Every brand/type is different. But, usually, for me, fall between 0.18-0.22\u00a0mm.

    \n
  2. \n
  3. Blue painters tape. If you don't need the glass flat finish of printing directly to glass, I always use blue tape. It makes for great adhesion, and also easy removal of super stuck prints, and you can just peel it off the build plate. With blue tape, I tend to add 2-3\u00a0\u00b0C to the bed temp.

    \n
  4. \n
  5. Glue sticks (instead of hairspray). But, Elmer's glue sticks that go on purple and dry clear. When you use these, if a print is stuck, you just need to spritz some water around the print. The print and the glue will wick the water under the print, and reconstitute the glue. Once the glue is tacky again, the print will pop right off. The only downside is a slight texture to the print and not glossy glass flat.

    \n
  6. \n
\n

The reason I like the colored glue, as it reconstitutes it activates the cobalt chloride and turns purple again. This is great for knowing when it is good to start printing again. And also tends to become less of a sticky mess than hairspray.

\n" }, { "Id": "15407", "CreationDate": "2021-01-22T18:16:15.303", "Body": "

Printer: Ender 3\nBed: Creality Glass\nABL: BLTouch\nEverything else: Stock

\n

Few weeks ago I had a meltdown during a failed ABS print. My hotend was completely wrapped in plastic and trying to remove said plastic I snagged the termistor. Anyway I had to replace the nozzle, hotend and thermistor. Everything on here. I went ahead and measured the Nozzle BLTouch needle offsets and re-flashed my Ender 3 with TH3D Firmware and configured BLTouch. After which I adjusted the Z offset again.

\n

All good here. BLTouch appears to work, well at least it appears so.

\n

I started printing and I see that my bed always hangs lower on the right side and BLTouch does not compensate for this difference.

\n

\"enter\n\"enter

\n

Manual leveling (with a heated bed, also tried without)

\n
    \n
  1. Auto home
  2. \n
  3. Move Z axis from 5 to 0
  4. \n
  5. Move the head to each corner of the bed
  6. \n
  7. Adjust the corner hight using the paper method (repeat 2x for each corner)
  8. \n
  9. Auto home
  10. \n
  11. Move Z axis from 5 to 0
  12. \n
  13. Go to Z offset, and move it until it tugs on the paper.
  14. \n
  15. Save and Auto home
  16. \n
\n

But this did not solve the issue. When I move the head to Z0 (with the offset) in the centre it's perfect. Moving it to left gets close to the board, right goes way away from the board.

\n

Belts & screws
\nWent and tightened all the belts and all the screws.

\n

X Axis alignment
\nI set the Z axis to the max 250. And measure the distance from top of the X axis to top of the frame. There is no difference in the left, right or centre of x axis to the top of the printer. Same for distance from bottom part of the frame.

\n

I cleaned the belts and the wheels with a soft brush. Not sure what else to do. I'm giving up on Creality/Ender :(

\n", "Title": "BLTouch does not compensate for differences in bed level", "Tags": "|creality-ender-3|bed-leveling|bltouch|glass-bed|", "Answer": "

The issue was BLTouch i'm not sure what use is this thing if it cannot compensate for differences in bed or frame.

\n

I'm not sure why but the probing seems to be bad. I tried everything clean TH3D install and config, pitch perfect alignment of everything frame, bed, mounts everything. Perfectly calibrated Z offset at the centre of bed. I triple checked the belts. Checked if X axis was off or if any screw was lose, checked wheels to make sure everything was snugg.

\n

Anyhow in the end the issue was with bad BLTouch probing not sure why. I gave up and went and disconneced BLTouch did manual bed leveling with the paper method. Press print and I got a perfect print. Like literally perfect. Even extrusion everywhere.

\n

BLTouch seems to have probed the distance at the centre of bed, I set the offset for it. Than when it went and probed other corners before the print it seems to have read the values wrong.

\n" }, { "Id": "15412", "CreationDate": "2021-01-23T05:38:25.853", "Body": "

I have a Lulzbot TAZ 4 and am using Lulzbot Cura for slicing, printing with HIPS.

\n

Yesterday, I tried a print, only for this to happen:

\n

\"Failed

\n

It appears that the printer got most of the way through printing the part, and then for no apparent reason lowered the nozzle into the part (causing it to detach from the bed), raised the nozzle back up, and then to continue trying to print like nothing happened.

\n

I thought maybe the G-code file got corrupted when transferring to the SD card (a single bit-flip could cause exactly this problem). So I generated the G-code again with exactly the same settings, and the same thing happened again but in a slightly different place and a few layers further up the model:

\n

\"Failed

\n

I have previously printed exactly the same model, only mirrored, with the same settings, and did not have this problem. I've never seen anything like this. Does anyone know what is going on?

\n", "Title": "3D printer destroys part by lowering nozzle", "Tags": "|ultimaker-cura|troubleshooting|lulzbot|", "Answer": "

This is my bet, based on cooling fan failure (I assume it was nozzle fan, not a heatsink fan), though honestly my practical experience with HIPS is zero. Without this fan heated material printed at high angles will definitely curl up, and even flat may be unstable. I suppose it happend, observing layer inconsistency on middle finger close to the nail.

\n

Then, material could curl a bit and build up. Collisions with hotend could result in blob of plastic here or there, and then something bad happened (see @Jack State comment).

\n

I also suppose that the whole print was detached from bed and rotated, because we see surprising shape across the middle finger, and unfinished index finger. Filament was extruded in random locations, causing more blobs, more curling and more mess. There are some gaps which look exactly like hotend run directly into them. I suppose that object rotated both horizontally and vertically. It could be even dragged by hotend at the end (e.g. filament was extruded inside the index finger).

\n" }, { "Id": "15413", "CreationDate": "2021-01-23T08:25:29.673", "Body": "

I use Slic3r to create G-codes for my Labists ET4 printer. Usually, no problem.

\n

Now, as soon as I tried to start a print, the nozzle head went down as usual, but this time it went a bit lower than 0 and the machine made angry noises for less than a second.

\n

It went back up again and heated the bed and the nozzle, only to go back down too strongly with angry noises again. Even left a little pokey hole on the bed.

\n

I got scared and turned the machine's switch off and manually raised the Z axis with ease.

\n

Never had this problem before and now I'm afraid to turn on my machine.

\n

I've had the following setup for G-codes prior to today's incident and it had worked just fine:

\n
G28 ; Home extruder\nG1 Z15 F[travel_speed]\nM106 S[max_fan_speed]; Turn on fan\nG90 ; Absolute positioning\nM82 ; Extruder in absolute mode\n; Activate all used extruder\nM104 T0 S[first_layer_temperature]\nM190 S[bed_temperature]\nG92 E0 ; Reset extruder position\n; Wait for all used extruders to reach temperature\nM109 T0 S[first_layer_temperature]\n
\n

After this, I did try the auto levelling, however it still tried to plow into the bed, albeit not as angrily.

\n

Here's other things to potentially consider:

\n\n

So here's my questions:

\n\n", "Title": "Labists ET4 printer nozzle hits bed after automatic levelling", "Tags": "|z-axis|labists-et4|", "Answer": "

Dirt. It was dirt. Most likely.

\n

\"enter

\n

Apparently the machine establishes bed level with a sensor, and I might be wrong, but I think the ET4 monitors capacitance as a means to notice how far the nozzle is from the bed.

\n

After all else failed I looked at the nozzle closely and noticed that the whole thing looked fuzzy. Looks like soot (from where though!?) had settled on the machine's sensitive components and de facto blind-folded its sensor.

\n

A wipe and a levelling later, all was back to normal. Yay!

\n" }, { "Id": "15417", "CreationDate": "2021-01-24T02:49:42.963", "Body": "

I am using PLA and I am looking for ideas on using the Tune option to tune my Prusa i3 MK3 to increase the speed to 300\u00a0%.

\n

The 300\u00a0% speed works perfectly fine for some of the models.

\n

However, for some of the flat surfaces, the printer cannot handle the speed. The issues are pointed in the image below:

\n

\"enter

\n

It seems that printer is struggling to supply enough filament.

\n

Can I solve the issue by using the Tune menu by changing settings like temperature or flow?

\n

Answer:

\n

While @dandavis is informative, but this is how I did it:

\n\n

Please note that this degrades the print quality, but I am just printing a few organizer boxes. Quality is not essential and not something I expect from a 300% speed.

\n", "Title": "Ideas to tune for 300 % speed", "Tags": "|prusa-i3|speed|", "Answer": "

Provided you find a way to increase the extruder limits, as replied by @dandavis , you will still get underextrusion in the infill if the infill is printed faster than walls.

\n

You need to enable some option in the slicer to label each part of the print, so that you get a comment in the gcode to mark walls, infill, and so on.

\n

Then you need to process the generated gcode, so that when you find the label "infill" you replace it with a flow multiplier to increase flow. How much has to be tuned by try and fail.

\n

You also need to replace every remaining label (walls, ...) with a 100% flowrate setting to bring back the setting to the standard value, obviously.

\n" }, { "Id": "15421", "CreationDate": "2021-01-24T09:28:16.120", "Body": "

I was printing an object and it started to drag so I stopped it.

\n

Went to move the Z-axis up so I could clear the bed and Z-axis would not budge.\nI switched the printer off and manually turned the motors to get the Z up.\nCleared the bed, switched on and homed the printer. When it came to home the Z, BLTouch deployed and then nothing. Motors will not turn.

\n

Things I tried:

\n\n

Info about the printer:

\n\n

Output of M122 for the Z:

\n
Recv:       Z  \nRecv: Address  \nRecv: Enabled       false  \nRecv: Set current   1000  \nRecv: RMS current   994  \nRecv: MAX current   1402  \nRecv: Run current   17/31  \nRecv: Hold current  8/31  \nRecv: CS actual 8/31  \nRecv: PWM scale  \nRecv: vsense        0=.325  \nRecv: stealthChop   true  \nRecv: msteps        16  \nRecv: tstep     max  \nRecv: PWM thresh.   0  \nRecv: [mm/s]        -  \nRecv: OT prewarn    false  \nRecv: triggered  \nRecv:  OTP      false  \nRecv: pwm scale sum 10  \nRecv: pwm scale auto    0  \nRecv: pwm offset auto   36  \nRecv: pwm grad auto 14  \nRecv: off time  4  \nRecv: blank time    24  \nRecv: hysteresis  \nRecv:  -end     2  \nRecv:  -start       1  \nRecv: Stallguard thrs  \nRecv: uStep count   40  \nRecv: DRVSTATUS Z  \nRecv: sg_result  \nRecv: stst  \nRecv: olb  \nRecv: ola  \nRecv: s2gb  \nRecv: s2ga  \nRecv: otpw  \nRecv: to  \nRecv: 157C  \nRecv: 150C  \nRecv: 143C  \nRecv: 120C  \nRecv: s2vsa  \nRecv: s2vsb  \nRecv: Driver registers:  \nRecv:       Z   0xC0:08:00:00  \nRecv:   \nRecv:   \nRecv: Testing Z connection... OK  \n
\n", "Title": "Z axis no longer working on a working printer", "Tags": "|marlin|bltouch|creality-cr-10|tmc2208|", "Answer": "

Issue seems to be resolved. I deleted my Marlin copy, downloaded a fresh copy and I started again.

\n

What worries me is how a working copy and a working printer can suddenly have this issue. Feel like this issue could come back as I never found the root cause.

\n" }, { "Id": "15432", "CreationDate": "2021-01-25T09:27:14.467", "Body": "

I am looking at printing a fair amount of text, ideally using some custom fonts. I quite like Stay Classy but will likely have to consider change if it causes issues.

\n

I am a little stuck on where to start with using these in some tools. I have tried creating an SVG using the font and importing to Tinkercad however that always fails. I only want to print the text, nothing else.

\n

How do I properly convert my font into .svg and import that so I can make my bodies? While I have tried Tinkercad I am open to alternative tools if this can be achieved more easily.

\n", "Title": "Printing 3D letters in any font?", "Tags": "|software|", "Answer": "

Tinkercad and Inkscape is likely the easiest method, requiring very little to learn anew. The key in Inkscape is to use Path, Object to Path, which converts the text format to "ordinary" vectors supported by Tinkercad.

\n

If you're comfortable with both programs, there's no reason to divert to a new package.

\n

Tossing a conversion from vector to bitmap and back to vector is going to "damage" the image, while the suggestion I've made will retain all detail.

\n" }, { "Id": "15439", "CreationDate": "2021-01-25T17:07:16.080", "Body": "

In the past we had printers with poor mechanics and with primitive software algorithms, therefore we used to print inner perimeters faster than the outermost one. See for example (generic, found online):

\n

\"enter

\n

However now we have pressure/linear advance which reduces extra oozing/extrusion in corners or areas with variable speed, and in Klipper we also have resonance compensation which takes care of imperfect mechanics allowing printers to be pushed to higher acceleration without visible artifacts (in my case from 2000 to 6000 mm/s^2), see (generic) photo:

\n

\"enter

\n

However printing slower has a clear disadvantage: E steps calibration is speed dependent with more filament being pushed out at lower speed, see

\n

\r\n \r\n

\n

Is there any reason left to print outer perimeters at a lower speed? Using one speed only except for specific areas (small perimeters, bridges, support) seems to make more sense to me to improve quality and reduce printing times.

\n", "Title": "Is a slower outer perimeter speed still meaningful?", "Tags": "|slicing|calibration|speed|", "Answer": "

Lowering speed on outer perimeter has always been mostly wrong, but possibly useful. Usually, it's a poor approximation for what you really want to do, which is lowering acceleration on the outer perimeter, to avoid surface quality and dimensional accuracy errors due to ringing and backlash. However, on bowden printers without compensation for pressure ("linear advance" in Marlin, aka "pressure advance" in some other firmware), slow acceleration and high speeds give really bad error in extrusion consistency, so you're better off just lowering the max speed too whenever you lower acceleration.

\n

It's also possible that you may want to print at extremely high speeds for inner walls and infill - speeds so high that you get a lot of extrusion consistency problems. If so, it would make sense to retain a sensible max speed on the outer wall. This won't avoid the part strength problems from printing too fast, but if your models are just decorative, it might be worth it.

\n

Generally, though, I'd recommend solving these problems right (using linear advance if your printer needs it, lowering outer perimeter acceleration to get rid of artifacts, printing at a speed your hotend can handle, etc.) and deem "slower outer perimeter" an idea whose time has passed.

\n

Also, note that if you're using a bowden printer without linear advance, sticking to the same speed for all extrusion will largely paper over the problem. This is probably the source of your observation that it "improves print quality" for you.

\n" }, { "Id": "15444", "CreationDate": "2021-01-25T23:45:20.870", "Body": "

I need to print a rotor for a DC motor I'm designing. In the process of testing the behaviors of the motor performances, I would need a material that will not deform at a temperature range between 100\u00a0\u00b0C to 150\u00a0\u00b0C.

\n

Since I don't have a 3D printer yet, I would like to know what would be the best choice for my need.\nI was planning to buy an Ender 3, but I'm not sure this entry-level 3D printer will allow me to obtain the results I'm looking for. I'm excluding PLA material because I think it's the most "fragile" material from this point of view and for my needs.

\n

My questions are:

\n
    \n
  1. Which material should I use in order to have a 3D printed object (rotor) that will not deform at a temperature that varies from 100\u00a0\u00b0C to 150\u00a0\u00b0C?
  2. \n
  3. Can an Ender 3 (planning to use full metal hotend and also hotbed) be used to print the filament that is heat resistant? Should I buy a resin 3D printer?
  4. \n
\n", "Title": "Which 3D printer to print heat-resistant material?", "Tags": "|3d-design|heat-management|", "Answer": "

Using an Ender 3 for high temperature materials is possible but you need to enclose it to be able to heat the air up to 100 \u00b0C.

\n

It's quite involved and it would be much better, if it's something you do seldomly, to have the parts printed professionally.

\n

Many thing start warping or breaking at 100 \u00b0C.

\n" }, { "Id": "15474", "CreationDate": "2021-01-27T17:00:50.830", "Body": "

I have an Ender 3 Pro and I'm about to print a relatively large model on it.\nI've been printing in PETG and I'd like to use up the last of the roll during this print.

\n

When the roll runs out however, I don't currently have any PETG lying around but I do have two brand new rolls of PLA and ABS. I'd love to swap to one of them (lets say ABS) when the PETG is running low, and I'm just wondering if there is anything I should be wary of besides the print temp.

\n

I am aware of general issues with ABS (warping without heated enclosure and stuff) but if I:

\n
    \n
  1. Have heated enclosure
  2. \n
  3. Tune temperature to be higher when I swap to ABS
  4. \n
  5. Have "draft shield" printing along with the model
  6. \n
\n

Are there any other considerations that I need to put in the gcode or anything? How much it pushes the filament or retracts or something? I'm just using "Generic PETG" settings on Cura.

\n", "Title": "How to switch from PETG to either PLA/ABS mid print?", "Tags": "|creality-ender-3|abs|petg|", "Answer": "

You should do a complete calibration for ABS (temperature tower, E steps, flowrate %) before starting the print, then when you change filament remember to apply all the parameters I mentioned.

\n

While I'm not in favour of using the flow rate % to correct the E steps calibration, since you are doing it mid print this may be the easiest way, instead of changing E steps AND flow rate %.

\n

As you said, you need to change temperature too, but both PETG and ABS print well at 235 \u00b0C so it may not be needed. PLA works too, but PLA bridges at 235 \u00b0C are difficult to get properly.

\n

For sure you won't be able to change other parameters, such as fan speed, printing speed and flowrate for bridges, which are all specific to each filament, but hopefully it will work out anyway, since PETG is trickier than ABS or PLA.

\n

Of course you may have issues with adherence: PETG may not bond well to ABS or PLA (in fact, PETG can be used for support for PLA and viceversa because the bonding is not too strong). You may have a weak print with PLA, so go for ABS.

\n

PETG as support for PLA: Can PETG be used as support material for PLA?

\n

Bridge calibration:

\n

\r\n \r\n

\n" }, { "Id": "15495", "CreationDate": "2021-01-29T14:02:54.063", "Body": "

I'm using Cura to slice prints from a biodegradable polyester called PCL (Polycaprolactone).

\n

I need to print @ ~70\u00a0\u00b0C but extruder does not run until nozzle reaches 175\u00a0\u00b0C.

\n

Which setting to change so extruder will turn on when nozzle temperature has reached 70\u00a0\u00b0C?

\n

Here are my settings for the material:

\n

\"strong

\n", "Title": "How to print at low temperatures (filament melting at about 70 \u00b0C)?", "Tags": "|hotend|nozzle|temperature|", "Answer": "

In Cura Machine Settings, add M302 S70.

\n

Apparently, M302 P1 and M302 S0 do nothing, you need to define a non-zero minimum, and in my case, 70. Thanks to 0scar and Trish for your help.

\n" }, { "Id": "15498", "CreationDate": "2021-01-30T00:04:44.543", "Body": "

TL;DR - Please help me rebuild my CR-6 SE so that I can move on

\n
\n

Here's a link to the latest issue that I had to make proof of in a video: CR6-SE failed

\n

Basically, it's failing to reach/maintain temperature (set point of 240\u00a0\u00b0C, fails to heat above 230\u00a0\u00b0C), issues start to happen at temps as low as 210\u00a0\u00b0C.\nCreality support is evasive/dodgy. First, they want to see a video proof for the problem reported, and after a while and emails from me asking, suggested something obvious (set temp to 200\u00a0\u00b0C).

\n

And without any video, they don't respond. Just like last time when I ran into the defective/burned power switch and failed bed leveling.

\n

If you have some spare time, please quickly check the video and provide some feedback.

\n

I already fixed the burned power switch and auto-leveling (Creality ignored me, so I got the parts from Amazon).

\n

This is the last issue that I need to do a "mercy" fix, so that I can either repurpose it or, just give it away (but I still have to make it working again and safe!).\nI already placed an order for an i3 MK3S+, should've bought this in the first place to help me study 3D modeling, not working my degree towards fixing Creality printer failures :)

\n

Thanks in advance.

\n

My troubleshooting steps so far after a screw fell off during print:

\n\n

Ordered a hotend from AliExpress, hopefully, that's it

\n", "Title": "CR6-SE fails to heat hotend to set point", "Tags": "|troubleshooting|hotend|heat-management|creality-cr-6|creality|", "Answer": "

I replaced the hotend with parts bought from AliExpress.

\n

My printer is now able to reach/maintain temperature and complete prints successfully (no more heating failure).

\n

BTW:\nIt's harder to buy replacement parts for CR-6 (as compared to other Creality printers).\nThere were only 2 type of replacement hotend parts for sale on AliExpress at the time I was shopping:

\n\n

There's an Amazon seller and a Canadian 3D printer store that sells individual parts, e.g.: heater block, but the prices are relatively expensive.

\n

I bought a full assembly, but a nozzle kit would have worked as well.

\n

Also, it looks like they're fixing their production process; my new hotend assembly was built better, even with what appear to be the same parts.

\n

Not pretty, but at least it's printing - Hopefully this is it.

\n

\"test

\n" }, { "Id": "15503", "CreationDate": "2021-01-30T15:28:11.253", "Body": "

I've just changed the motherboard on my Ender 3 Pro with a MKS GEN_L v1.0 and flashed the latest Marlin version on it.

\n

I've calibrated my bed manually using the default XY and Z auto home commands on OctoPrint and a piece of paper.

\n

I'm happy with the calibration, however whenever I launch a print the Z axis moves up from the calibrated position by about 4\u00a0mm and starts extruding.

\n

I've checked my Z endstop status with M119 and it's triggered at the right calibrated position.

\n

How can I correct this?

\n", "Title": "Issue with Z homing on Ender 3 Pro with Marlin", "Tags": "|marlin|creality-ender-3|z-axis|homing|endstop|", "Answer": "

I've actually found what the issue was. It turns out that my Z steps were way out of whack (i.e. 4000 steps/mm instead of 400). Apparently, that's the default value in GitHub for version 2.0 of Marlin. Not sure if that's a typo or a valid value, anyhow setting it to 400 fixed it.

\n" }, { "Id": "15513", "CreationDate": "2021-01-31T11:13:46.037", "Body": "

I own a Magic Build 3D Printer. I found out, that the glass is scratched like in the picture.

\n

So, I'm looking for a replacement glass plane to put in front of the projector. Is this a special glass? What are the properties? Where can I find it?

\n

\"enter

\n", "Title": "What type is that glass?", "Tags": "|replacement-parts|dlp|", "Answer": "

This is a sheet of glass you'd most likely need to acquire as a replacement part for your printer's manufacturer:

\n\n

In this case, we have a DLP setup that uses a projector. You best inquire at the manufacturer itself using their website, E-Mail or facebook about a few spare pieces - that will get you the same quality as the bed you already had (save for the scratches) and they usually have such items in stock.

\n" }, { "Id": "15521", "CreationDate": "2021-02-01T06:58:27.727", "Body": "

What should I take care of to replace the nozzle of the hotend assembly the right way?

\n

What kind of data, precautions, tools, steps, and verification are important for replacing the nozzle?

\n

The procedure seems straightforward but tutorials differ greatly and seem often incomplete. Online videos are great but long, sometimes misleading, and difficult to compare if they use another printer model.

\n

For example:

\n\n

So I am looking for a general overview: what should I think of to be in control? And maybe a bit of why, but not necessarily how. (As always, each detail may be a separate study, so please don't to fall into troubleshooting, which could be done in separate questions like E3D V6 nozzle seized into heater block).

\n", "Title": "How to replace the nozzle in my hotend assembly?", "Tags": "|nozzle|fdm|safety|maintenance|knowledgebase|", "Answer": "

Tools

\n\n

Precautions

\n

Beware of touching heater and thermistor wires with tools during operation. Electronics can be damaged by shorting the heater leads, thermistor leads, either with tools, the nozzle, or even a metal bed. Also, neither 12\u00a0V nor 24\u00a0V are pleasant to touch.

\n

Using tools with plastic shielding can help to prevent this, but proper care is needed. Improvised rubber or tape wrappings are not advisable.

\n

Step 1: Unload

\n

Before swapping the nozzle, remove the filament. Follow the procedure of your manufacturer, though often this is heating the nozzle, then triggering the Swap-Filament or unload routine.

\n

Many makers suggest a cold-pull. This is heating up to about the print temperature, then going back to off. Letting the filament cool by about 20\u00a0\u00b0C, they then yank hard at the filament, removing most of the plastic from the nozzle and melt zone. It is not applicable to some nozzles, especially those made from several parts (e.g. Olsson Ruby).

\n

I have found it easier to put in an intermediate step of removing the whole hotend and use clamp it in a small vise to hold it on the heater block. This holds it steady and removes any stress from the heartbreak. However, this is more involved than using a fitting wrench to steady the heater block.

\n

Step 2: Remove the nozzle

\n

Dismount the extruder shroud if it blocks the way to the heater block, or hasn't already been removed to get the hotend dismounted.

\n

If you have a lined hotend, remove the PTFE tube.

\n

If you had oozing, this usually has glued the nozzle and heatbreak to the heater block. Heating the hot end to about the glass transition temperature of the filament will often make it easier (or possible) to remove the nozzle without stress to or damage to parts.

\n

Now use a suitable wrench to remove the nozzle. Make sure to steady the heater block during this step. Otherwise, you might shear off a necked heatbreak, while lined heatbreaks might get deformed.

\n

A Hot removed nozzle needs to be put aside on the safe surface or remain in the block with some windings.

\n

If you work in a vise, you can remove the heatbreak too.

\n

Step 2.5: Cool down

\n

It's best to let the hotend cool down, so you can easier handle the parts. Unless you had to fight with oozing, you can have the nozzle col down still halfway in the block and then do the last windings by hand on a cold nozzle.

\n

Step 3: Install new nozzle

\n

Now, screw in the new nozzle hand tight. There should be at least half a millimeter stick-out between the nozzle flange and the heater block and it needs to butt against the heartbreak. (Nozzle must tighten against the heat break and not stress the heater block). Don't use a wrench yet, just hand tighten the nozzle against the heartbreak.

\n

If you have a lined hotend, re-install the Bowden tube until it butts against the nozzle.

\n

Step 4: Hot tighten

\n

Heat up the hotend to your limit temperature:

\n\n

Now use the wrench to tighten the heated nozzle carefully. Use a second wrench to steady the heater block, if you don't have the setup in a vise. Don't waste too much time, but don't over-torque!

\n

The exchange is done. If the hotend was unmounted, remove it from the vise, reassemble it fully and install it on the printer again. Make sure that your hotend cooling fan does put the heater block in the airstream.

\n

Step 5: Validation

\n

Run some filament through installed hotend. Observe if there is no ooze outside the nozzle.

\n

Step 6: Finishing

\n

If not already done, mount back the extruder shroud and any parts that had been removed.

\n

Check homing of the Z-axis, as you will need to adjust the Z-height. If you have a sensor on the carriage, you might need to adjust its height to compensate for a slightly altered stickout of the nozzle.

\n" }, { "Id": "15528", "CreationDate": "2021-02-01T19:03:35.150", "Body": "

I have an Ender 3 a 3DTouch, with the SKR Mini E3 1.2 board.

\n

From one day to another the probe started producing this pattern, where the right side of my bed (5x5 mesh, tried with 3x3, 7x7, all produce the same pattern) just drops by a significant amount. The bed is the Creality glass bed. I tried switching it back to the stock Ender 3 plate (non-magnetic), but no physical sign of this drop can be seen by the eye.

\n

Probe testing with M048 usually gave me Range ~0.005, and standard deviation between 0.001-0.0025 both in the problematic part, and any other part on the bed.

\n

What could cause this problem?

\n

\"weird

\n", "Title": "Ender 3 weird drop in bed level", "Tags": "|creality-ender-3|troubleshooting|bed-leveling|3dtouch|", "Answer": "

Considering this is an Ender, it is probably caused by the rollers on the X axis or the un-driven right Z post.

\n

I've also seen behavior like this caused by a too constraint PTFE Bowden tube, at the far end the tube pulls on the carriage lifting it slightly upwards, hence a different/larger gap.

\n" }, { "Id": "15541", "CreationDate": "2021-02-02T17:31:07.470", "Body": "

I've noticed when heating up the hotend to maximum operating temperature to tighten the nozzle against the heat break, after putting a wrench on the heater block, the temperature of the heater block drops by as much as 60 \u00b0C. Do you ignore this or have a procedure to correct?

\n", "Title": "Wrench drops temperature of heater block when tightening nozzle", "Tags": "|hotend|nozzle|", "Answer": "

Mitigating "heat loss"

\n

Technically, we don't lose heat, we have a drain of thermal energy out of the parts, but let's stay simple.

\n

To combat the loss into the holding/steadying wrench, I use a vise: it might lessen the heating curve by the added thermal mass if used bare, but by adding two pieces of wood on either side of the heater block, I can insulate the block against the thermal drop from the steadying vise and atop that prevent scratches.

\n

If you use an adjustable wrench for the heater block, one can add wooden "soft jaws" with a little cyanoacrylate glue, otherwise, some painter's tape or Kapton might aid.

\n

It might help that my small wrenches have a super sturdy surface coating making them rather comfortable to use when left out in the cold and prevents rust. This layer also seems to prevent the flow of thermal energy into the wrench. Note that they are not chrome-plated but more like an enamel-look, a glassy powder-coat or actually lacquered.

\n

Conclusion

\n

When I need to mount something installed on the printer, I do the tightening as quickly as possible - the temperature of the measurement point and the temperature of the nozzle getting pressed into the heartbreak do not drop in the same instance, the nozzle will still be at the aimed for temperature when the thermosensor already has dropped.

\n" }, { "Id": "15550", "CreationDate": "2021-02-04T06:56:09.140", "Body": "

Apply / find / create a stainless steel coating to apply to a PETG or PLA part to make it react to a magnet.

\n

My goal is to make a small tubular and conical shapes that can be painted with a stainless steel coating and will react with a magnet.

\n

I know I can buy iron filled PLA but these rust which I want to avoid. I'm trying to get the magnetic properties of stainless steel (no rusting / reacts to magnets)

\n

I have found videos on how to coat with copper / silver / carbon but I'm looking for stainless steel no rust / magnetic properties.\n

\r\n \r\n

\n
\n

I'm looking for a "low-cost" solution just for testing.

\n", "Title": "Apply / find / create a stainless steel coating to apply to a PETG or PLA part to make it react to a magnet", "Tags": "|pla|post-processing|petg|material|", "Answer": "

No rust but has magnetic properties? You could try varnishing the iron filled PLA.

\n" }, { "Id": "15561", "CreationDate": "2021-02-05T06:17:55.813", "Body": "

Could anyone help me to find out what is the brand name of this 3D printer?

\n

\"3D

\n

I found this printer while playing Spider-Man on PS4 in the Lab

\n

\"enter

\n", "Title": "What is the brand of this 3D printer?", "Tags": "|desktop-printer|part-identification|", "Answer": "

The printer on picture looks like LulzBot TAZ.

\n

It could be some prototype or a self-made built or just inspired by it (as it is a digital rendition), because there is no sign of any branding on it. There are similar builds on Thingiverse.

\n" }, { "Id": "15564", "CreationDate": "2021-02-05T19:35:38.683", "Body": "

I hope someone can poke me in the right direction, I\u00b4ve been failing to find a way around this problem and trying for about a week.

\n

My setup is pretty custom, I am using a BTT SKR 1.4 Turbo with TMC2208 driver and a BTT TFT V3 display. At the beginning of this problem I\u00b4ve added a second Z-Axis to my printer and made the mistake to declare both axes in Marlin (as I am only using one driver, I should not have to alter Marlin for this, but I found that out too late). At this point, I updated Marlin to take my new steps/mm for the Z-axis, but no matter how often I did, the firmware didn\u00b4t seem to change anything. Later (out of pure hopelessness) I used the M502 Command, followed by M500 to save and somehow this made my axis work and set the correct steps/mm out of my firmware (at this point, I already found the mistake in Marlin mentioned above).

\n

Relieved I started my first print... But the extruder motor (bowden setup) stopped turning and I don't know why. Now, I\u00b4ve been stuck with this strange behavior for a week:

\n

When I freshly boot the printer, the extruder motor spins (correct length and correct direction). I can do this via M-Command, TFT35 and Pronterface (PC-USB direct to the mainboard). Whenever I start a print, the extruder just stops (motor engaged, but not trying to turn, no skipping steps). No matter how long I wait, even in a 2-hour print, the motor doesn\u00b4t spin a bit. After the print and if I cancel the print the motor won\u00b4t turn again (tried all the ways mentioned before). BUT when I kill the power to the printer and let it reboot I can use the extruder in all the mentioned ways without any problems.

\n

I think this is a software problem, but I can\u00b4t find it. Looked through Marlin a few times, adjusted my Start G-code, looked at the existing G-code, I can not find the mistake. I\u00b4ll attach a few lines of G-code and my Start G-code (I use Cura 4.8). If someone knows a line in Marlin, I\u00b4ll look that up for you - it seems impossible to post the whole firmware somewhere.\nStart G-code:

\n
G21 ;metric values\nG91 ;relative positioning\nM107 ;start with the fan off\nG28 X0 Y0 ;move X/Y to min endstops\nG28 Z0 ;move Z to min endstops\nM900 K0.22 ; set K-primus-factor\nM117 K0.22 ; display K Value on display\nG92 E0 ;zero the extruded length\nG90 ; absolute positioning\nM82 ;set extruder to absolute mode\nG1 Z1 F500 ;move up slightly\nG1 Y60.0 Z0 E9.0 F500.0;intro line\nG1 Y100.0 E21.5 F500.0 ;continue line\n\nG-Code:\nG1 F600 Z0.3\nG1 F1500 E0\nG1 F1200 X99.215 Y92.699 E0.06147\nG1 X99.708 Y92.391 E0.09734\nG1 X100.377 Y92.083 E0.14279\nG1 X100.743 Y91.958 E0.16666\nG1 X101.484 Y91.803 E0.21337\nG1 X102.187 Y91.748 E0.25689\nG1 X103.552 Y91.72 E0.34114\nG1 X118.764 Y91.72 E1.27986\n
\n

If there are any ideas, please tell me. I am completely lost at this point!

\n

Edit: a few mistakes are gone now, formatting and PS.: "blockage of the extruder-motor" after starting a print happens no matter how I start a print (SD TFT/Pronterface/SD Mainboard)

\n", "Title": "Extruder motor does spin - but not while printing", "Tags": "|marlin|extruder|skr-v1.4|", "Answer": "

Extruder stops to spin

\n

The problem you described seem to be reported elsewhere for TMC in relation to stealthChop mode (unsure, see below) and linear advance. For details look at this comment in Extruder stops during print (Bugfix-2.0.x) #17944. An advice from there is to supplement the G-code with (my interpretation):

\n
M569 S0 E ; disable stealthChop for extruder (use spreadCycle)\nM900 K0   ; disable linear advance\n
\n

There is also another discussion TMC2208 extruder driver shuts itself off in spreadcycle mode with pressure advance enabled #1774 for Klipper. Above summary is just a result of my quick look through, I suggest to re-read these articles with more insight.

\n

Also I propose to do following to doublecheck:

\n\n

Extrusion mode

\n

I personally prefer to use extrusion in relative mode, because it allows to manipulate G-code or restart the print from any line. You would need to put M83 in starting G-code instead of current M82, together with enabling Cura setting (Special Modes > Relative Extrusion). I am not sure what happens currently with your filament between in your file's:

\n
  M82 ;set extruder to absolute mode\n  G1 Y60.0 Z0 E9.0 F500.0;intro line\n  G1 Y100.0 E21.5 F500.0 ;continue line\n  ...\n  G1 F1500 E0\n
\n

For me it looks like it's retreating 21 mm of filament just at the start of print? So it cannot really print for several consecutive G-code lines, does it?

\n

Marlin settings vs EEPROM

\n

When updating settings in Marlin like steps/mm you should be aware, that values stored in EEPROM take precedence. Reinstalling firmware will not clear the EEPROM settings. If you want to make persistent change, save to EEPROM and alter the firmware config, to have these values secured for future as "defaults".

\n" }, { "Id": "15571", "CreationDate": "2021-02-06T17:23:59.977", "Body": "

I'm setting up a replacement Melzi 2.0 board from TRONXY for my i3 clone (a Monoprice Maker Select v2 for those curious), which according to the RepRap wiki has A4982 drivers for all the steppers. I'm attempting to configure Marlin 2.0.x for this printer and this is the last big problem I'm encountering.

\n

The problem: The Z-axis (or any motors connected to the Z-axis driver) won't budge no matter what I try. All my other steppers work fine.

\n

What I've tried: I've done everything I can to prompt the Z-axis to move, from Gcode commands to homing via the LCD and Pronterface. I've connected my X-axis motor to the Z-axis driver and it won't move either. Sometimes, when I send a command for them to move, they'll make a small noise - like they're getting power - but won't move. I probed the motor connectors at the board and received nothing but a few stray millivolts here and there, very different from the several volts on my Y-axis motor when I did the same thing. Bear in mind, that was just one test moving them from the LCD, and this time they did not make any noise or attempt to move. I have switched pin definitions in the firmware for X/Z ENABLE/STEP/DIR and the Z driver once again appeared to be the culprit, keeping the Z-axis planted even if it was being controlled as if it were the X-axis.

\n

What I think the problem could be: To me, this seems like a software problem. I'm configuring Marlin for my first time and I think I messed something up somewhere. Could it be a problem with the trim pots, motor currents, motor enabling, or something else? Outside of that, maybe the driver is just dead?

\n

My Marlin 2.0.x config.h for reference:

\n
// These are all excerpts from various parts of Configuration.h\n\n#define DEFAULT_AXIS_STEPS_PER_UNIT   { 80, 80, 400, 500 }\n\n#define DEFAULT_MAX_FEEDRATE          { 300, 300, 4, 25 }\n\n#define DEFAULT_MAX_ACCELERATION      { 3000, 3000, 100, 10000 }\n\n#define DEFAULT_ACCELERATION          2800    // X, Y, Z and E acceleration for printing moves\n#define DEFAULT_RETRACT_ACCELERATION  2800    // E acceleration for retracts\n#define DEFAULT_TRAVEL_ACCELERATION   2800    // X, Y, Z acceleration for travel (non printing) moves\n\n//#define CLASSIC_JERK  //left at default\n\n// :{ 0:'Low', 1:'High' }\n#define X_ENABLE_ON 0\n#define Y_ENABLE_ON 0\n#define Z_ENABLE_ON 0\n#define E_ENABLE_ON 0 // For all extruders\n\n#define DISABLE_X false\n#define DISABLE_Y false\n#define DISABLE_Z false\n\n#define INVERT_X_DIR false\n#define INVERT_Y_DIR false\n#define INVERT_Z_DIR false\n\n#define X_HOME_DIR -1\n#define Y_HOME_DIR -1\n#define Z_HOME_DIR -1\n\n#define X_MIN_POS 0\n#define Y_MIN_POS 0\n#define Z_MIN_POS 0\n#define X_MAX_POS X_BED_SIZE\n#define Y_MAX_POS Y_BED_SIZE\n#define Z_MAX_POS 150\n\n#define MIN_SOFTWARE_ENDSTOPS\n#if ENABLED(MIN_SOFTWARE_ENDSTOPS)\n  #define MIN_SOFTWARE_ENDSTOP_X\n  #define MIN_SOFTWARE_ENDSTOP_Y\n  //#define MIN_SOFTWARE_ENDSTOP_Z    //disabled until the driver works correctly\n#endif\n\n#define MAX_SOFTWARE_ENDSTOPS\n#if ENABLED(MAX_SOFTWARE_ENDSTOPS)\n  #define MAX_SOFTWARE_ENDSTOP_X\n  #define MAX_SOFTWARE_ENDSTOP_Y\n  //#define MAX_SOFTWARE_ENDSTOP_Z\n#endif\n\n//#define Z_SAFE_HOMING\n\n#if ENABLED(Z_SAFE_HOMING)\n  #define Z_SAFE_HOMING_X_POINT X_CENTER  // X point for Z homing\n  #define Z_SAFE_HOMING_Y_POINT Y_CENTER  // Y point for Z homing\n#endif\n\n// Homing speeds (mm/min)\n#define HOMING_FEEDRATE_XY (50*50) //changed from 3000 to 2500\n#define HOMING_FEEDRATE_Z  (4*60)\n\n// Validate that endstops are triggered on homing moves\n#define VALIDATE_HOMING_ENDSTOPS\n\n// Sorry if that was a bit much, I tried to include all the settings that could \n// potentially have an effect on the Z motion.\n
\n

I genuinely have no idea where to even look to solve this problem, so any guidance or advice would be hugely appreciated!! Thanks!

\n

My conclusion is that the driver chip itself is dead and replacing it ought to fix the problem. If this works, I will post an answer to this question.

\n", "Title": "Z-Axis on i3 clone won't move at all, Melzi A4982 driver problem", "Tags": "|marlin|z-axis|stepper-driver|melzi|", "Answer": "

I've replaced the Z-axis stepper driver with an off-board Pololu-style A4988 driver by soldering leads to pins 2 and 3 on the Melzi board, then connecting those to the driver on a separate board. The Z-axis now moves correctly. When I encountered issues with the step pin (pin 3), I connected it to pin 12 (bed heater) instead, leaving the bed to be heated manually. This has worked so far and the only problems I've encountered were minor issues with electrical noise, which I was able to fix with a shielded cable.

\n" }, { "Id": "15574", "CreationDate": "2021-02-07T08:11:23.970", "Body": "

Since I have lots of PETG, I ran tuning to 230\u00a0\u00b0C (average temp for my filaments).\nWhat is it good for, in terms of temperature ranges?

\n

For the same printer configuration, and just different filaments, will I need to run it again and again?\nLet's assume that I'll be printing between 200\u00a0\u00b0C and 240\u00a0\u00b0C.

\n

\"marlin

\n", "Title": "PID tuning and different filament temperatures", "Tags": "|calibration|pid|", "Answer": "

PID tuning can be performed multiple times and the results saved for future use, since the question is about "what are the usable ranges for PID tuning", based on my experience

\n
    \n
  1. a slightly suboptimal tuning will not make the temperature oscillate more than 2-3 degrees, which is more than enough for most traditional filaments
  2. \n
  3. if you have a 30-40 \u00b0C temperature range you can likely keep a tuning in the middle and be done with it
  4. \n
  5. an accurate tuning is needed if you run the hot end at its maximum rated temperature: mine was rated 250 \u00b0C and without a good PID tuning the temperature was overshooting by 2-3 degrees, which was enough to trigger a over-temperature safety shutdown. Using the printer at 245 \u00b0C would have resulted in no issues even with sub-optimal PID tuning.
  6. \n
\n

Of course, people with high temperature hot ends (up to 270-300 \u00b0C or more) will need a tuning for the usual range (200-240 \u00b0C) and one for the higher temperature range to obtain better prints.

\n" }, { "Id": "15576", "CreationDate": "2021-02-07T17:30:09.737", "Body": "

I have upgraded my ER-20 with a Bondtech dual gear feeder.\nIt is not or hardly possible to load/unload filament manually with this feeder, so some .gcode is needed to do it.\nI wanted to implement something similar to the atomic cleaning method for unloading: https://ultimakernasupport.zendesk.com/hc/en-us/articles/115004187066-Atomic-Cleaning-Method

\n

Here is my current .gcode:

\n
G21 ; Metric values\nG90 ; Absolute positioning\nM82 ; Extruder absolute mode\nG28 ; Auto home\nM420 S1\nG1 X100 Y100 Z100 F1000\n; M92 E415 ; 415 steps/mm\nM302 S105 ; Allow extrusion above 105C\nM109 S218 ; Heat hotend to 218C\nM400\nG92 E0 ; Reset extruder positioning\nM104 S160 ; Start the cool down\nM117 Extrude prime blob\nG1 E10 F100 ; Extrude a short before unload to avoid blob forming\nM109 S160 ; Wait for 160C\nM104 S110\nG92 E0\nG1 E0.2 F100 ; Pressurize the hotend\nM117 Pressurize hotend\nM109 S110 ; Heat hotend to 110C\nG92 E0 ; Reset extruder positioning\nM117 Pull out slow\nG1 E-3 F200 ; Pull back a bit, slow\nM117 Pull out fast\nG1 E-430 F2000 ; Pull back 43cm with 2000mm/min\nG92 E0 ; Reset extruder positioning\nM400 ; Wait for command finish\nM117 Remove the filament now\n; M400 ; Wait for command finish\nM302 S170 ; Allow extrusion above 170C\nM104 S0\n
\n

It doesn't work:

\n\n

Could anyone take a look at this code please? Or is there any .gcode validator for Marlin somewhere?

\n

What I intend to do with this code:

\n\n", "Title": "Marlin firmware: unload filament with G-code", "Tags": "|marlin|extruder|g-code|", "Answer": "

There are three things to fix and one suggestion:

\n\n" }, { "Id": "15577", "CreationDate": "2021-02-07T20:16:29.460", "Body": "

I manage four 3D Printers for my college's robotics team. They are used pretty constantly throughout the school year; that is, they are used more as manufacturing printers than hobbyist printers. The environment they are in is less than ideal: The outdoor climate is typically cold and has low humidity, and the room they are in has fine dust particles that are less than 80 microns diameter and have electrostatic properties. The room's temperature also fluctuates week to week. As such, static electricity is fairly common in that room.

\n

What effects does static electricity have on our 3D printers? I have had numerous problems recently with a motherboard with resistors that failed and another printer with an unknown issue that we suspect also has to do with the motherboard. Could the static electricity be causing these or other issues, or affecting the quality of our printed parts?

\n

We use ZylTech filament, and print using a Creality Ender 3 Pro, CR-10S Pro, Ender 5 Plus, and a FlashForge Creator Pro. The dust particles mimic "regolith", or lunar soil, that comes from one of our team's test facilities.

\n", "Title": "Effects of static on 3D printers & print quality", "Tags": "|print-quality|electronics|", "Answer": "

Regolith is one of the worst stuff known to man that could accumulate on electronics and moving parts. NASA replicated Regolith and has a manual. Researchers studied Regolith as a 3D print material - and found it can be used. We even have the formula to make Concrete from it by adding water and refined regolith or even dry refined regolith.

\n

Also, we understand how Regolth behaves electrically. It's not a good conductor in the dry state but at least a conductor. However, just a little water rapidly changes that and makes it a rather good conductor! Tiny amounts rapidly increase the conductivity, making it very dangerous to electronics in combination with dew - which happens in low temperatures.

\n

Dust on bearings gunks them up and then starts to grind away the rails.

\n

Dust on filament gets pulled into the hotend and can create clogs.

\n

So, it's rather clear that we don't want this regolith-dust on our printers.

\n

Danger mitigation

\n

You immediately need to take measures to prevent dust from accumulating on the electronics and secondary on moving parts!

\n

If possible, housing the electronics in a dust-sealed box would be preferential, but\nhousing the printers as a whole in an enclosure might mitigate most of the trouble for the start.

\n

The most simple enclosure that would allow for such would be a simple large wood box, provided that the door contains a seal. For fire protection and to reduce the risk of one spot of the housing getting too high, the inside air should be constantly mixed, best by some sort of permanent running large fan. For further protection, the setup needs to be vented at least when the internal temperature gets too high, allowing to use of the heat of the printers to stabilize the internal temperature - and best even log it.

\n

A venting-trigger temperature should be below 100\u00a0\u00b0C air temperature, better even 80\u00a0\u00b0C. But how to reduce the temperature?

\n

The easiest cooling solution is to intake air from the outside, but we need to get the dust out of that. An easy solution would be a HEPA-filter unit. There are permanently running types, that would keep the printer-box(es) under over pressure compared to the atmosphere, keeping the dust from creeping into the box, but that doesn't give us control to heat the chamber if it gets cold.

\n

But we could salvage an old printer board for that! Setting up the thermosensor in such a way that it measures air-temperature is easy. Using the bed and/or hotend exit of the board to some sort of heating element with grids at the filtered intake should be doable, evening out the temperature in the printer box to a minimum safe temperature.

\n" }, { "Id": "15583", "CreationDate": "2021-02-08T17:07:48.427", "Body": "

I recently purchased a Creality Ender 3 V2 ~1 month ago, and just yesterday installed a BLTouch. When looking at the motherboard, I noticed my board version is 4.2.2. Isn't the Ender 3 V2 supposed to come with a V 4.2.7 board? Did Creality mistakenly include the wrong board with my printer?

\n

I did some googling but couldn't find a definitive answer. If anyone can, please let me know. My plan (if there was a mistake) is to email Creality customer service, but if any of you have a better suggestion, I'm all ears.

\n

PS: I purchased this directly from Creality website, not Amazon or any other marketplace. Thanks, all.

\n", "Title": "Is Creality Ender 3 V2 Supposed To Come With Board V 4.2.2?", "Tags": "|creality-ender-3|firmware|stepper-driver|creality|", "Answer": "

Yes, they are shipping the v2 with the 4.2.2 board. My understanding is that the 4.2.7 board has a change in the stepper driver pinout but is otherwise the same as 4.2.2. The change in the pinout allowed them to change the stepper driver package as the stepper driver vendor appears to be changing that spec. I've used both boards in my V2 with no discernable difference.

\n" }, { "Id": "15592", "CreationDate": "2021-02-09T14:35:40.067", "Body": "

Hello dear Community,

\n

I am trying to print my first Box and I have a problem with the skin of the bottom layers.\nAfter finishing the slicing and while looking into the generated G-code I was wondering why Slic3r increases the E-parameter continuously when 90 % of the lines have the same length and the axis have the same speed. Logically as long as the length of the lines are equal and the speed of the axis is also fixed so the amount of extraction has to be in this case also fix but that is unfortunately not the case. Therefore as you can see in the Result photo, that the printer prints the skin layer at the beginning in the right way with the right amount of extruded material. At the end of the skin layer, it seemed very bad because of the huge amount of extruded material.

\n

Some Infos about my Printer may be interesting for you :\nNozzle diameter = 2.7 mm\nType of Filament= granules\nDiameter of filament = 2.5 mm

\n

Please let me know your suggestions and feedbacks.

\n

\"GECODE \"GCODE

\n", "Title": "Wrong calculation of E-parameter values", "Tags": "|3d-models|slic3r|", "Answer": "

The value you see is the cumulative extrusion, in other words the E value in consecutive G-code commands reflect a position of the filament since you started that print.

\n

It is normal that it continuously increases in "absolute" mode.

\n

You should see, at the beginning of the G-code, also a M82 or G90 command, which means "absolute extrusion". And you should avoid having M83 or G91 later in G-code, which would change an interpretation of E values to "relative extrusion".

\n

It is more accurate than relative extrusion (which is the mode you are thinking about).

\n

From your print it looks like the printing bed is not properly tuned, probably too far.

\n

Also, it is difficult to see properly, but it looks like you get bubbles in the second part of the print, which are the reason there is an increase of extrusion: hot air pushes too much filament out.

\n

Maybe your granules are not properly dry. It is difficult to tell you more, pellet extruders are more difficult to tune than filament extruders.

\n" }, { "Id": "15597", "CreationDate": "2021-02-09T16:45:30.963", "Body": "

Repetier-host has a setting to specify the "print area". That's roughly the size of the bed.

\n

Note that the printer head can go out of those bounds, in my case my bed is very undersized compared to the printer frame, but this would also be an issue if you had clips or some obstacles in the bed.

\n

Is there a similar setting in Cura where I can specify the "print area"/"bed size"/"margins" to be different from the printer width/depth?

\n

Thanks.

\n

Repetier host settings:

\n

\"Repetier-host\"

\n", "Title": "Ultimaker Cura 4.8 / How to specify bed (Print) Area?", "Tags": "|ultimaker-cura|heated-bed|", "Answer": "

I am not sure this is the "official" solution for this, but I was able to define the "disallowed areas" in a custom printer definition.

\n

I created a json file under C:\\Program Files\\Ultimaker Cura 4.8.0\\resources\\definitions for the printer, and added the machine_disallowed_areas section to "overrides".

\n
    "machine_disallowed_areas": {\n    "default_value": [ \n        [ [ -97.5, 90],[97.5,90] , [97.5,80] ,[-97.5,80] ] ,            \n        [ [ -97.5, -57],[97.5,-57] , [97.5,-90] ,[-97.5,-90] ] ,\n        [ [ -97.5, -90], [-49.5, -90], [-49.5,90 ] ,[-97.5, 90] ],\n        [ [ 64.5, 90],  [64.5, -90], [97.5, -90], [97.5,90 ] ]\n        ]\n        }\n
\n" }, { "Id": "15605", "CreationDate": "2021-02-09T23:32:32.100", "Body": "

I currently use a BLtouch 3.1, which is known to stop operating at about 35-40 \u00b0C. In fact, when I preheat the bed at 100 \u00b0C close to the probe, I sometimes get issues with the pin not retracting correctly.

\n

I would like in the future to enclose and heat the printer chamber, therefore I need a probe capable of operating at higher temperature. My target is 100 \u00b0C.

\n

As of now, I'm aware of this solution:

\n

https://hightemp3d.com/products/remoteht-bed-level-probe-3d-printer

\n
\n

This high temperature probe uses a mechanical switch in combination\nwith a servo motor to probe the bed. The servo motor is outside the\nenclosure and retracts the probe pin by pulling on a steel string\ninside a spring guide tube.

\n
\n

Microswitches are known to work at high temperatures (that's what the link above uses), but usually need a servo or other mechanism to extend/retract them when needed. Servos typically don't operate at such high temperature, not to mention that they are usually not very accurate.

\n

I saw a Hall filament sensor which uses Hall effect sensors "ss49e" and which could be easily modified to be used as bed probe. Those Hall effect sensors are rated up to 100 \u00b0C but at that temperature they have up to 8% shift of the null value and +3/-9% change in sensitivity. The linked project uses two of them in differential mode so the temperature compensation should be much better, but it is not clear how much better. A discussion about it on a Russian forum does not talk about this.

\n

Are there other options operating at 100 \u00b0C?

\n", "Title": "What are my options if I want to install a bed levelling sensor capable of operating at 100 \u00b0C?", "Tags": "|bed-leveling|bltouch|chamber|", "Answer": "

There are various designs which can be made to work at 100 \u00b0C.

\n

For example this solenoid probe uses a solenoid to lift a plunger during print, letting it drop during probing. The design uses a rare earth magnet which usually loses strength at 60-80 \u00b0C, but there are high temperature versions or a ferrite magnet will do. It also uses an Attiny13, rated 80 \u00b0C, but there are higher temperature versions of the chip. It is likely that it would work out of specs at 100 \u00b0C too, but it uses an optical endstop, which is the limiting factor with 85 \u00b0C typical.

\n

This other design uses a solenoid to push up and down a tactile switch. I see no reason why the solenoid would not work at 100 \u00b0C, maybe only the external wires may require replacement to withstand the temperature. There is no electronics limiting the operating temperature.

\n

Other designs with a servo are limited by the temperature range of the servo, which is typically 50-60 \u00b0C max. I think they would work fine up to 80 \u00b0C since they are not operated continuously and they are loaded with little force, but there is no guarantee.

\n

Another design working perfectly and with an even lower total weight is based on a microswitch and... the user placing the probe in position manually. It can probably be automated by mounting it on a hinge and by using something on the side of the bed to move it in an upright position and in a tilted position, where it could be held by (ferrite) magnets.

\n" }, { "Id": "15606", "CreationDate": "2021-02-09T23:44:54.657", "Body": "

I have PLA and PETG filament.

\n

I hear that 3D filament absorbs water and causes problems when printing but after printing they can be used with water and they are water proof.

\n

So my question is why is it different after printing/what has changed to make it now waterproof?

\n\n", "Title": "How do filament properties change after printing? (In relation to water absorbtion)", "Tags": "|filament|print-material|water-resistance|", "Answer": "

The properties of the maternal are not what changes after you print. It is how you use the material. PLA and ABS prints aren't usually subject to +100 \u00b0C after printing.

\n

When PLA absorbs moisture it becomes brittle. PLA filament will break when trying to print. Your PLA print may be thick enough so that brittle doesn't cause a problem. However, this can be a problem if you intend your PLA print to be flexible.

\n

The main issue with PETG absorbing moisture is the water changing to steam in the extruder. PETG becomes soft and will distort with force applied at +100 \u00b0C; so generally moisture isn't a issue with PETG prints, due to application.

\n

PLA and PETG absorb moisture, but do not dissolve in water. Usually what a person means by waterproofing is water won't seep through the layers of the material and leak out of a container. Note: most glass will absorb small amounts of moisture, but are still waterproof.

\n

If one wants a hermetic seal this is a similar issue. One person printing hermetic prints said thicker layers make it easier to be hermetic. Thicker layers also tend to make the printer stronger, but can also give less detail in the print.

\n" }, { "Id": "15614", "CreationDate": "2021-02-10T20:47:04.303", "Body": "

I'm using Marlin. Recently my Z-probing randomly doesn't work. Sometimes during Z-Homing it goes up instead of going down to sense the Bed! In Bed Leveling it goes higher and higher every time. It got frustrating so I ignored Bed-leveling.

\n

Sometimes I get a fine Z-homing but it always starts printing in the air!!! I have no Idea why it happens. All home and axis offsets and setting are fine. I had no problem last time I used my printer.\nWhy my printer starts printing 5\u00a0mm in the air?

\n", "Title": "Marlin Z-probe Failure!", "Tags": "|bed-leveling|z-probe|homing|", "Answer": "

I found the problem. I'm using an aluminum structure for my gantry. My Chinese power supply was in contact with this structure. There was a current leak from my power supply that was messing with the Z-probe signal.
\nI detached the power supply, everything's gotten back to normal, except it had my board and TMC2100 drivers fried a week ago.

\n" }, { "Id": "15618", "CreationDate": "2021-02-11T12:41:52.187", "Body": "

I'm quite new to 3D-printing. I need to build a curved piece with a hole pattern and I was wondering whether it was possible to print such a hole pattern accurately, with such a thin (3\u00a0mm) thickness of the piece. In the past, when I tried printing thinner pieces with holes, the piece warped and holes were not printed properly. Therefore is there any changes to the model I need to make to improve the quality of the print, or add any supports?\nI am using an Ender 3 Pro, with PLA.

\n

\"enter

\n", "Title": "How to accurately print a thin piece with a hole pattern", "Tags": "|print-quality|3d-design|support-structures|", "Answer": "
    \n
  1. Use the smallest nozzle possible to get better rounded edges on holes.
  2. \n
  3. Slow down the print speed to increase accuracy and prevent errors.
  4. \n
  5. Print it face down against the bed if possible to reduce support requirements for overhangs and increase hole quality.
  6. \n
  7. Clean the bed well, increase bed temperature, stick glue, enable brim, or add 'mouse ears' to prevent warping.
  8. \n
  9. Make sure the bed is levelled properly to prevent warping and increase finish surface quality.
  10. \n
  11. Use a 'stringing tester' to find the best temperature to print at so that you can prevent stringing between holes.
  12. \n
  13. Enable z-hop retraction to prevent stringing and errors.
  14. \n
  15. You can bore through the holes with a drill to clean them flush.
  16. \n
\n" }, { "Id": "15629", "CreationDate": "2021-02-12T14:40:17.830", "Body": "

Printer configuration and many settings affect heat creep. Heat creep stopping extrusion doesn't necessarily involve melting the filament too high above the nozzle. All it takes is making the filament too soft. The filament may bend and wad up (the importance to avoid gaps in the hot end allowing this). On direct drives (not Bowden tubes) the filament can get soft so that the teeth on the direct drive cuts a notch in the filament instead of pushing it through the nozzle.

\n

[moved to answer]

\n

\"enter

\n

Fig. 1: Some typical signs of heat creep on the filament of a direct drive. 1) notch in filament where gear spins in soft filament. 2) normal teeth marks in filament. 3) soft filament bending. 4) soft filament bunches in open area. With a Bowden tube, the filament widens at the end.

\n

\"enter

\n

Fig. 2.: Another example of heat creep with a direct drive extruder.

\n

Note: Setting the retraction too high can cause the extruder to jam, but this is not heat creep. Although, the symptom could be mistaken for heat creep.

\n

Also note: Printing materials like PETG too fast can clog the nozzle because the filament doesn't have time to melt. This is the opposite cause from heat creep. On some direct drive extruders you can tell the difference in the appearance of the jammed filament moving too fast versus heat creep because the filament is rigid when moving too fast and tends to jam in the nozzle giving a similar appearance to heat creep in Bowden tubes extruders. When PETG jams in a direct drive as shown in Figure 3, both raising the temperature of the extruder (more heat to melt the filament) and improving the cooling of the heat sink (drive teeth digging into stiffer filament) also helps.

\n

\"enter

\n

Fig. 3.: Filament with end expanded in nozzle showing jam from filament moving too fast and not heating enough.

\n

A defect in the filament can jam in the tube to the hot end, as well as the filament hanging up on the spool or in the path to the hot end. This can also stop extrusion.

\n

Of course different materials change the characteristics of heat creep, but what are other things affecting heat creep?

\n

This is expanded from How is heat creep characterized?

\n", "Title": "What are ways to avoid heat creep?", "Tags": "|extrusion|heat|heat-creep|", "Answer": "

Things to consider (This is expanded from How is heat creep characterized?):

\n
    \n
  1. The air volume of the cooling fan on the hotend heat sink affects the temperature gradient across the heat sink. Usually the size/geometry of the fan depends on the printer design, so the main parameter of a fan that controls air volume is the rpm. I've noticed fans fitting hotends from 6,000 to 10,000 rpm. The higher rpm the better as far as preventing heat creep. Of course, make sure the fan is spinning properly.

    \n
  2. \n
  3. The raising the temperature of the extruder increases the chance of heat creep.

    \n
  4. \n
\n\n
    \n
  1. Razing the temperature of the heated bed, especially when the printer is in an enclosure, increases the chance of heat creep because this raises the temperature of the air the fan blows across the heatsink.
  2. \n
\n\n
    \n
  1. The slower the print speed the higher the chance of heat creep. That's because a slower print speed gives the filament more time to heat up in the extruder. Many slicers have a setting that slows down the print speed if a layer will finish below a given time. If this slowed down time causes heat creep, the extrusion will stop when the printer reaches these smaller area layers.
  2. \n
\n\n

Example: This is easy to demonstrate with PETG. Design a table printed top down 100 mm square with four legs 5 mm square and 100 mm long. Set the parameters so that you start to get heat creep when printing the legs. Now put four 1 mm thick walls between the legs to get the heat creep to go away.

\n\n
    \n
  1. The heater block might be too close to the heatsink so that they bypass the heat breaks reduction in conducting heat.

    \n
  2. \n
  3. Hotends for temperatures 245 \u00b0C and below can use such things as Teflon tubing in the heat break to isolate the filament from the metal. But, +250 \u00b0C nozzle temperatures need all metal hotends.

    \n
  4. \n
  5. The PID settings in the firmware might be letting the hotend temperature overshoot too high.

    \n
  6. \n
\n" }, { "Id": "15637", "CreationDate": "2021-02-13T14:04:25.790", "Body": "

I've put together a flashlight mount for a camera coldshoe in OpenSCAD. I originally modeled it in FreeCAD and it was easy to round the edges of the clamp with a fillet and that makes it a little easier to get the light in and out of the mount.

\n

I'm not sure how to do it in OpenSCAD. Naively, I'm sure I could calculate where on my semicircle I would need to add some cylinders in order to round the sharp corners, but it seems like there'd be something a little easier than that.

\n

Am I missing something?

\n

Here's the coldshoe light mount SCAD file and this is what it looks like:\n\"coldshoe

\n", "Title": "Is there an easy way to add rounded edges to semicircle in OpenSCAD?", "Tags": "|openscad|", "Answer": "

I ended up taking a third option I suppose. I decided I was cutting the gap wrong. I was using difference to take a rectangle out of the ring.

\n

I decided I'd simplify that and take out an extruded triangle.

\n
ringRadius = radius + thickness;\nverticalOffset = tan(gapAngle / 2) * ringRadius;\n\nlinear_extrude(height = height)\n    polygon(points = [[0, 0], [ringRadius, verticalOffset], [ringRadius, -verticalOffset]]);\n
\n

That way, the thickness of the plane that needed rounding is uniformely the thickness of the ring. Then, it was just a matter of adding cylinders, centering them in the ring, and rotating them into position.

\n
for(side = [-1, 1]) {\n    cornerRadius = radius + thickness / 2;\n      \n    rotate([0, 0, gapAngle / 2 * side])\n        translate([cornerRadius, 0, 0])\n            color("blue") cylinder(r = thickness / 2, h = height);\n}\n
\n

Eventually I'd like to make it a little more complex and help accommodate a light better, but for now it gets me the rounded ends I was looking for.

\n

\"Rounded

\n

I'm still going to accept the round everything answer because, though I decided it was more complexity than it was worth on this model, it looks like it would solve other rounding issues too.

\n" }, { "Id": "15641", "CreationDate": "2021-02-13T21:09:29.593", "Body": "

I am very comfortable with using PrusaSlicer and having a hard time adapting Cura.

\n

Is there any downloadable well-tested PrusaSlicer profile for some of the Creality printers?

\n", "Title": "Is there a well-tested Ender 3 printer profile for PrusaSlicer available for download?", "Tags": "|creality-ender-3|slic3r|prusaslicer|", "Answer": "

Open PrusaSlicer, go to Configuration -> Configuration wizard, then go to page named "Other Vendors", tick the checkbox next to the name "Creality", then click "Next" at the bottom right of the window. A page with a handful of Creality printers should appear including Ender 3. Tick the checkbox under Creality Ender 3. Then click "Finish" at the bottom right. The profile should appear in the selection.

\n

It imports various print settings (from 0.3\u00a0mm to 0.08\u00a0mm I believe) as well as some basic material presets. You can import more materials by just going to the "Filaments" page in the Configuration wizard and ticking the desired options.

\n

The profiles are tested by PrusaResearch themselves I believe and I think they are based on other profiles found online with some tweaks and adjustments to better fit to PrusaSlicer.

\n

I am using PrusaSlicer 2.3.0

\n

Here is an image, what it should look like. Your colors might be different, I asume you are using Windows, wheras I am using Linux Mint.

\n

\"PrusaSlicer

\n" }, { "Id": "15645", "CreationDate": "2021-02-14T01:56:22.100", "Body": "

Has anyone successfully replaced the inductive Z probe on the Monoprice MP10 (Mini or regular) with a different type, such as BLTouch or IR?\nI find that the stock inductive probe isn't so accurate, and since it's inductive it only works with the aluminum build plate but not with glass or PEI, which I much prefer.

\n", "Title": "Replacement Z Probe for MP10", "Tags": "|z-probe|monoprice-mp-10|", "Answer": "

From Thingiverse you can find a BLTouch mount for the MP10 (and MP10 mini):

\n

\"MonoPrice

\n

Note that in order to use the BLTouch sensor you need a different controller board that supports connecting a BLTouch sensor as the MonoPrice controller board doesn't support connecting a BLTouch sensor.

\n

From ref.:

\n
\n

A BLTouch adapter for the Monoprice MP10 & MP10 Mini. NOTE: you will need an open source control board to get this to work as the stock firmware on the MP10 line of products does not allow for a BLTouch.

\n
\n" }, { "Id": "15649", "CreationDate": "2021-02-14T19:56:45.323", "Body": "

A print that is paused with an M0 can be unpaused with M108.\nThat works with my Chiron running Marlin 2.0.7.2.\nHowever, it also has a display that uses its' own serial protocol to communicate with Marlin. I want that display to be able to unpause my printer when it has encountered M0 in the G-code.\nI tried injectCommands_P(PSTR("M108\\n")); in the code, but it did not work. Perhaps I should not have a new-line at the end of it.

\n

What is the correct command to do this?

\n
void GcodeSuite::M108() {\n  TERN_(HAS_RESUME_CONTINUE, wait_for_user = false);\n  wait_for_heatup = false;\n}\n
\n

Above is the Marlin code for M108. Would be great if someone could decipher what TERN means.

\n", "Title": "How to programmatically resume a paused print in Marlin", "Tags": "|marlin|", "Answer": "

The correct Marlin-command to unpause seems to be setUserConfirmed(). I have tested that it works.

\n" }, { "Id": "15661", "CreationDate": "2021-02-16T10:57:55.650", "Body": "

I'm pretty much a noob to the 3D printing community. I got a Creality CR10 V2 as my first 3D printer and I know it's compatible with BLTouch but not included in the package. So I'm dying to add an ABL feature to my 3D printer but I'm on an extremely tight budget right now. However, I found some industrial IR sensors (two Omron E3Z-D82 and five Keyence PZ-M71) in my toolboxes. I know those sensors are pretty expensive and totally overkill for ABV but I want to use what I got in my hands.

\n

So the question, can I use those sensors for auto bed leveling? I should say that my printer has a glass build plate. By the way, I'm also open to different cheap alternatives

\n

Here are the links to the sensors for more information:

\n\n", "Title": "Can I use photoelectric sensors to make auto bed leveling on a glass bed?", "Tags": "|bed-leveling|bltouch|creality-cr-10|", "Answer": "

3D printers like the German Reprap (see image) already uses a photosensor for homing the axes. The trick will be leveling the leveling sensors.

\n

However, if you use an optical sensor measuring distance, you could mount it on the hotend like we do digital indicators (drop gauge). See Bed leveling method?\nYou would want a resolution of about 0.01 mm. That method will keep you planer to the movement of the hotend. For automatic leveling the photosensor needs to input to your software.

\n

If you want to permanently connect the photosensor to the hotend, probably the most practical communication method would be USB, which could supply power as well as communication to eliminate batteries. The USB cable could run in the path of the other hotend cables. Less weight on the hot end is if you could remote the laser and detector from the electronics.

\n

If only attached during leveling, batteries and wireless connection is better.

\n

I'm finding more digital drop gauges (second image below, data in cable is USB) with computer interfaces than photosensors. Many of the photosensors would need to interface with a meter first. Because a drop meter makes physical contact, it's not practical to leave on the hotend during printing.

\n

\"enter

\n

\"enter

\n" }, { "Id": "15662", "CreationDate": "2021-02-16T13:09:02.410", "Body": "

I'm currently facing a Z banding issue that frustrates me more and more.

\n

My printer is a modified Anycubic I3 Mega.

\n

Customizations:

\n\n

What I tried to fix this issue:

\n\n

Pictures of the XYZ calibration cubes I printed:

\n

\"enter

\n

\"enter

\n

Used slicer parameters (Prusa Slicer):

\n\n

The number of these horizontal lines and their occurrence is different on each print.

\n

Any ideas?

\n", "Title": "Z banding issue", "Tags": "|troubleshooting|z-axis|petg|anycubic-i3-mega|bigtreetech|", "Answer": "

So it looks like I finally resolved that issue. I compiled a complete new firmware based on the Marlin bugfix branch. Before I was working on the sources provided in the Bigtreetech GitHub. I started from scratch and put all the settings I needed into the Configuration.h and Configuration_adv.h. Now all rings are gone. Either there was really a bug or something or I messed up something in the firmware I used before and beginning from scratch solved it.

\n" }, { "Id": "15667", "CreationDate": "2021-02-17T07:32:48.363", "Body": "

I have a Creality Ender 5 Pro which has been working properly for some time. During my last print, the axis motors just suddenly stopped working while the extruder motor continued to work properly.

\n

In fact, when I connect the extruder motor wire to any of the axis motors, they respond correctly (mimicking the extruder motor) but neither works or moves when connected to its own wire(s). As a result the printer does not print anymore.

\n

Here's what I've tried:

\n\n

I'm quite confused why the axis motors would not be working all of a sudden. I'll appreciate your help if you've faced this problem before.

\n", "Title": "Why would all of the Ender 5 Pro's axis motors (X, Y and Z) stop being able to move (all at once)?", "Tags": "|troubleshooting|creality-ender-5|bigtreetech|skr-mini-e3|3d-printerworks|", "Answer": "

Usually, steppers not working can be found when the lines are broken. Often this is an unplugged Stepper, but OP checked that.

\n

That 3 steppers at once failed could indicate a bad firmware update, which OP didn't do. So it is sadly a case of blue smoke from the board. Most likely the stepper drivers are either shortened out or get no signal.

\n

I am sorry, but replacement seems necessary.

\n" }, { "Id": "15684", "CreationDate": "2021-02-18T15:23:18.763", "Body": "

I have run the K-factor Calibration Pattern generator to tune LIN_ADVANCE and found that when K=0, the whole middle part of the line is thinner.\nI was expecting that LIN_ADVANCE would only make a difference short after where the speed was increased or decreased, as described by the first picture on this page.

\n

\"Running

\n

So from this picture, it looks as the best setting is around K=0.8 and that is also what was set as default in Configuration.h for Chiron. But when I tried the Printing Direction-setting set to 45 degrees the result was different. Then the best setting was around K=0.4.

\n

The picture below shows a second test, where I set K=0 and just changed the fast speed. The faster the thinner lines were printed. I have tested printing long lines and seen that thickness will not compensate, even after a long time.

\n

\"Running

\n

So why is my printer printing thinner lines when going fast and not using LIN_ADVANCE? Isn't the number of E-steps per mm printed line constant and not depending on the speed?

\n

Here is a zip-file with the files used and pictures. (let me know if not possible to download)

\n

ADDED 2021-02-22:\nTo learn more about LIN_ADVANCE I made a second test today. To rule out most of the uncertainties that can affect the result, I let the bed stay heated between all four tests. Every test started at 210\u00baC, then 200, 190 and last 185\u00baC. The printer waited more than one minute between each temperature change. Bed temperature set to 60\u00baC. The lines are numbered in the order they are printed. Each test started by printing four lines at 20 mm/s to get "normal" nozzle pressure. Each line is 20+40+20=80mm long. Today I had acceleration set to 1200 and jerk to 8, which are higher than before (500,5). I changed because I use these values in Cura. Unfortunately, the bed is not 100% flat.

\n

LIN_ADVANCE = 0.0\n\"LIN_ADVANCE

\n

LIN_ADVANCE = 0.4\n\"LIN_ADVANCE

\n

LIN_ADVANCE = 0.8\n\"LIN_ADVANCE

\n

LIN_ADVANCE = 1.2\n\"LIN_ADVANCE

\n

What I learned from these today's test:

\n
    \n
  1. Very little ooze when going from 190 to 185\u00baC. (#46 is good)
  2. \n
  3. At higher temperatures after pausing for a minute, the printer needed some 10 to 15cm of printing distance to get the pressure back. (#16, #17)
  4. \n
  5. Next time I will print every second line as a reference and test fewer speed changes.
  6. \n
  7. Next time I will also test higher temperatures.
  8. \n
  9. The lines are printed thinner at higher speed, independent of temperature when K=0. (I thought it only happened at (too) low temperature).
  10. \n
  11. LIN_ADVANCE setting shall probably be lower when printing at higher temperatures (affected by filament viscosity).
  12. \n
  13. It might be different behaviour between the grey and the orange filament. Orange looks better, but the grey one sticks better to the bed.
  14. \n
  15. It is fun to do testing. :)
  16. \n
\n

I did another test the first day, where I tested K=0 to K=0.8 and where the speed was 20+50+20 mm/s and the length was 60+180+60mm. Temp was 190\u00baC. It seems to show that even after 180mm of fast printing, the thickness is not recovered:\n\"180mm

\n

The pictures above are uploaded at a lower resolution. Original pictures and the G-code can be downloaded from here.

\n", "Title": "Why are fast printed lines thinner?", "Tags": "|marlin|print-quality|anycubic-chiron|", "Answer": "

In your case, since linear advance makes the issue disappear, the reason of the thinner lines is the same as explained in the link you provided:

\n

\"enter

\n

The section (2) simply is very long because you probably have a Bowden setup which takes more than the length of the middle section to recover. However it does recover, see the first line (k=0) in the first image.

\n

At higher (constant) speed the flowrate decreases further due to pressure in the nozzle causing a reduced supply of filament, but this effect cannot be corrected by linear advance because linear advance assumes a linear relationship between pressure in the nozzle and extrusion.

\n" }, { "Id": "15690", "CreationDate": "2021-02-19T00:53:54.340", "Body": "

Just as the title says. I feel like I have tried everything. I am compiling the firmware for Marlin on a Megatronics board from RepRap. That shouldn't be relevant, because I have validated that it is a firmware issue (and not a pin assignment/hardware issue).

\n

When I turn it on, the Y-axis is active and just constantly holding. The stepper driver on Y-axis gets pretty warm constantly holding (as does the stepper motor).

\n

When I swap the pin assignments for Y and X then the physical Y-axis moves fine when I manually change the X-axis so I know all the hardware and pin assignments are correct.

\n

All the endstops are correctly reporting open (Z is BLTouch but that is also working).

\n

Relevant software endstops are currently disabled, and default homing position is set to 50 to make sure it doesn't think it is at the bounds on boot.

\n

I have a test firmware that checks all the endstops, temperature, and servos, and all servos work fine. As a sanity check, pin assignments match what is in the pins file.

\n

Does anyone have any idea of what in the latest Marlin firmware would cause a software condition to just hold the y axis and not move? I should note that when I try to manually move it in Repetier-Host it shows that it is moving. When I try to give the home command, no movement on the Y but Repetier changes from red to black indicating it received confirmation of homing.

\n

I am completely out of ideas at this point.

\n", "Title": "Can't get Y-Axis to move with Marlin firmware", "Tags": "|marlin|firmware|y-axis|", "Answer": "

I ran into the same problem. After looking into the pin layout for the board I saw the following:

\n
#elif EXTRUDERS <= 2\n  // Hijack the last extruder so that we can get the PWM signal off the Y breakout\n  // Move Y to the E2 plug. This makes dual Y steppers harder\n
\n

This prompted me to move the stepper driver and the motor cable to the E2 connector on the board. (The one on the other side of the X connector, Order is Z,Y,X,E2,E1,E0)\nThat worked for me.

\n

So, you could move the stepper driver and motor cable. Or you could commment/remove that code from the pins_MEGATRONICS_3.h file

\n" }, { "Id": "15699", "CreationDate": "2021-02-19T23:45:38.477", "Body": "

I have a used Creality CR-10S and I have been using it for one day. I seem to be having problems with my Y-axis. Several times when I selected "Auto home", the bed only moved part of the distance it should have for the head to get to the bottom left corner. It is only the bed movement that has the problem; the head successfully goes to the extreme left each time.

\n

\"This

\n

It does not always happen and when it does, it seems to usually go half as far as it should, but once the bed did not move back at all. I am having a similar problem on "Bed Auto Leveling". Once when it was supposed to go to the back left corner, the nozzle went past the bed by about 5mm. I tried to reposition it with "Move axis" but it thinks it is at 0.00 and it won't let me go negative. One way I can fix it is to "Disable steppers" and then move the bed by hand.

\n

I have done two prints and in both cases there was no Y-axis drift during the print. After first print, "Auto home" only went half way but I was not paying attention to other details. After the second print the machine was off and I moved the bed by hand in response to @octopus8. Then I turned it on and selected "Auto home". The head descended and moved all the way to the left but the Y-axis did not move at all.

\n

When I move the bed by hand through its entire range, either with the machine off or after choosing "Disable steppers", it is pretty smooth. It has some resistance to movement, about the same as the X-axis resistance. I can feel very small "steps" if I move it very slowly. These are both consistent throughout its entire range. There are no individual spots where it behaves differently.

\n

I have made sure that the bed's belt is tight. I check the wheels.under the bed. I cleaned them with a rag. When I turned them by hand, several were slipping against the rail (not causing the bed to move when I turned the wheel.) I tightened their bolts and that tightened some of them against the rail but a couple others still slip against the rail. Is this important? After these adjustments and turning printer on, "Auto home" worked and the steps on "Bed Auto Leveling" also did the correct things. I will report on more post-adjust experience as I get it.

\n

Ever since I tightened the Y-axis belt and wheels I have not had the problem happen again having done 10+ prints. I tried the gcode that @octopus8 suggested at various speeds and the print head always ended up back where it started.

\n

I will keep printing and if I experience the problem again I will investigate more then.

\n", "Title": "Y-axis movement error", "Tags": "|creality-cr-10|y-axis|", "Answer": "

octopus8 recommended doing a series of tests to determine the cause of the problem.

\n

I started with moving the bed by hand (remember to have machine off or choose "Disable steppers" from "Prepare" menu) and the movement was relatively smooth with some resistance, the same as when moving the X-axis. If you move very slowly you can feel slight, small "steps", this is normal. The movement was consistent across the range of movement.

\n

Since this did not indicate a problem, I moved on to checking the drive belt and the bearing wheels. The belt had a small amount of slack which I tightened. Several of the wheels under the bed slipped against the rail they run along. I tightened these. This improved the movement of a couple of them but a couple still slipped a bit.

\n

After this I did not have the problem anymore. For 10+ prints over a week or so, there was no incorrect Y-axis movement and "Auto home" always worked. Nevertheless I continued with the next test suggestion which was to send some simple gcode to the printer via USB. I used both Ultimaker Cura 4.8.0's Monitor mode as well as [sending the commands from the linux terminal][1]. I sent the commands

\n
G28 \nG0 F<speed> \nG0 X200 Y0 \nG0 X200 Y200 \nG0 X0 Y200 \nG0 X0 Y0\n
\n

with values of 900,1800,2700,3600 and 4500 for and everytime the print head returned to the same place.

\n

This is evidence that the tightening of the belt and wheels likely fixed the problem.\n[1]: How to directly send G-code to printer from a Linux terminal?

\n" }, { "Id": "15727", "CreationDate": "2021-02-23T05:24:53.203", "Body": "

Using Prusa I3 MK3S

\n

I am using PLA fillement and I need to do cold pull from time to time.

\n

I noticed many people on Internet suggestusing Nylon fillement for clod pull.

\n

I wanted to reach out to this community and ask if it is better using Nylon fillement for cold pull even if I have been using PLA?

\n", "Title": "Is it better use Nylon fillement for cold pull even if I have been using PLA?", "Tags": "|print-quality|pla|prusa-i3|nozzle|nylon|", "Answer": "

Nylon holds better when pulled, PLA may snap.

\n

Also it can grab particles around it, but it keeps low friction so the force you use to pull goes to detaching the dirt instead of rubbing against the walls.

\n

By getting solid and by maintaining strength at higher temperatures, you can pull when it's hotter (with PLA you should pull at what? 40\u00a0\u00b0C?) and by being hotter can grab particles and remains of other filaments which are potentially softer. It's pulled at around 140-150\u00a0\u00b0C so that ABS and PLA at that temperature are very sticky and soft.

\n

By holding easily higher temperatures (250-260\u00a0\u00b0C easily) you can also extrude it and it will melt any other filament in the nozzle.

\n

Try doing some cold pulls with a light colour PLA to be able to see what it grabs. After a couple of pulls it should be clean. Then do nylon and check how much more stuff it will remove. If it's nothing, no need of using nylon. If it's dirty, it's the proof it can clean everything better than PLA.

\n

A sample of nylon (50\u00a0g usually) will be sufficient for a long time so no need to get a full spool.

\n" }, { "Id": "15731", "CreationDate": "2021-02-23T13:04:44.800", "Body": "

I'm going to add a Raspberry Pi to my printer, which requires a 5\u00a0V source. I want to take this opportunity to also power the Arduino with that same source, as dropping 12\u00a0V -> 5\u00a0V is a quite big waste. I know the electronics don't consume much power, and the regulator can handle the heat, but why not?

\n

I want to know how can I safely do this. Is it sufficient to just bent the pin from the RAMPS to the Arduino's VIN?

\n

I'm also using TMC2209 stepper drivers. According to this: https://learn.watterott.com/silentstepstick/pinconfig/, these requires the 12\u00a0V line to turn on before and turn off after the 5\u00a0V.\nI wonder if this is true, as I haven't seen this anywhere else.

\n", "Title": "How should I connect a separated 5 V source to an Arduino connected to a RAMPS board", "Tags": "|electronics|arduino-mega-2650|switching-power-supply|raspberry-pi|ramps-1.6|", "Answer": "

I would not recommend changing Arduino powering.\nYou can either do as you say and power Arduino by giving it regulated 5V to Vin, but then the ATMEGA2560 still will be powered through the 5V voltage regulator. The voltage regulator (NCP1117ST50T3G) needs to be supplied with 6.5V to give out regulated 5V. So you will be powering the Arduino with some 4V, which might work, but it will be less reliable. But it is electrically safe.

\n

Second option is to power Arduino through it's 5V pin. That is less safe, because the voltage regulator might have an over output-over-voltage-protection that will to load or short-circuit the 5V until it is 5.000V.

\n

Regarding TMC2209. I have not seen any documentation that has any opinion on what order it shall be powered up and down.

\n" }, { "Id": "15740", "CreationDate": "2021-02-24T16:57:35.127", "Body": "

I'm printing a lot of draft parts so I don't care if they fall apart in my fingers, I just need the shape. I can scroll to the Tune menu on my Ender 3 Pro console and set the speed to 200% and it doubles the speed. But when I set the Print Speed setting to 100 instead of 50 mm/s in Cura, it doesn't save much time, even if I adjust the individual first layer speed, wall speed, top layer speed, etc. What is the difference?

\n

Ideally, I would like the first layer to print normally, and then print at 2x speed.

\n", "Title": "What is the difference between tuning the speed on the Ender 3 panel, and the Cura Print Speed setting?", "Tags": "|creality-ender-3|ultimaker-cura|", "Answer": "

I figured it out.

\n

Feed rate, in general, is the speed of a single print move, from point A to point B. It is specified with every G1 command that commands a move. Setting the feed rate on the printer console will scale the feed rate of every move the printer makes. The Cura "Print speed" setting is used as a basis to set the print speed (Feed rate) of different line types. Eg. Infil is 100% of this speed, walls are 50% of this speed, etc.

\n

There are several other settings that will make Cura will decide not to use the speed that you set.

\n\n

In addition, the printer's acceleration setting may prevent it from reaching the maximum speed. You can see this if you issue this command during printing:

\n
M201 X50 Y50 ; Set max acceleration\n
\n

The print will slow down a lot until you set it back to the default. The printer will accelerate and decelerate slowly and simply never reach the maximum speed over short distances. The default in the Marlin firmware (for Ender) seems to be:

\n
M201 X500 Y500 ; Set max acceleration\n
\n" }, { "Id": "15742", "CreationDate": "2021-02-24T20:38:49.417", "Body": "

I need to find a way to design and build a real shopping cart for my project. I have not idea where to start from but I'm thinking maybe I can 3D print the shopping cart which will allow me to quickly iterate on the design. How could go about with 3D printing a project like this?

\n

Please I will appreciate other suggestions better than 3D printing

\n", "Title": "How to 3D print a shopping cart", "Tags": "|diy-3d-printer|3d-design|", "Answer": "

If you need a real shopping cart, please think about actual requirements.

\n

Carts are sturdy devices, build on very consistent frame, because they are meant to carry heavy loads. The built must be focused on good jonts and durable wheels, otherwise forces from load or streatching would quickly damage the construction. Some parts of frame are normally under strain, and a direction of load forces is not always downwards, therefore frame has to be robust.

\n

Cheap additive manufacturing technologies of standard printers are not suitable to produce objects of this size in one piece. If to follow this idea, one chellange would be to design pieces of the print, which could be connected or glued into a ready cart. 3d prints have tendency to snap along layers of print, therefore frame would need to be carefuly built from parts designed to be printed horizontally, rather than vertically. Construction of wheels carrying the cart with assumed load could be not possible, because load forces could mangle them very quickly.

\n

Only the amount of material needed to print the whole thing could exceed cost of the cart. And the same would be about more expensive printing techniques, like printing of metal. The shopping cart is a good example of thing which is not worth printing (at least in 2021).

\n

BTW: Shopping carts have some plastic parts, and printing these could be indeed possible. I suppose they are only protectors, not a part of cart construction frame.

\n

I would advice to look to get a ready cart. Check locally for used, worn out or reconditioned shopping cart for your project. Maybe even try to rent it somewhere. It would be much easier and quicker then trying to 3d print the cart, even if it is possible to do it within some reduced requirements.

\n" }, { "Id": "15748", "CreationDate": "2021-02-26T13:19:13.390", "Body": "

I have an Ender 5 pro and upgraded from the magnetic bed to the glass version. I print with Prusament PLA on 65\u00a0\u00b0C bed temp and 220\u00a0\u00b0C extruder temp. I measured that the glass surface has ~58\u00a0\u00b0C in the center and ~56\u00a0\u00b0C on the corners. That should be in the specs of the spool which has a printed recommendation of 50 +- 10\u00a0\u00b0C for bed and 215 +- 10\u00a0\u00b0C for extruder temps.

\n

Now I also have a BLTouch and use the TH3D firmware so Z offsets should work correctly and are done with the Z offset wizard present (nozzle touching the bed).

\n

The problem I currently have is that the first layers have "lifts" in them. Also, it seems that the layer itself is not 0.2\u00a0mm everywhere. It's close to 0.23\u00a0mm on the edges but on the inner there are variances up to 0.36\u00a0mm.

\n

Currently, I slice with Cura 4.8.0 on Standard Quality 0.2\u00a0mm

\n

How do I get rid of the lifts and irregularity in the layer? Is it a problem with the bed?

\n

\"Lifts\"

\n

\"darker

\n", "Title": "Irregular adhesion on Creality glass bed", "Tags": "|adhesion|temperature|", "Answer": "

The only time I've seen lifting, away from the edges of the part like this, is when the bed is contaminated with something which will stop the print sticking, like fingerprints and such.

\n

A thorough wipe-down with IPA on the heated bed is usually enough to stop it.

\n" }, { "Id": "15757", "CreationDate": "2021-02-28T01:07:19.693", "Body": "

If someone creates a 3D model of a character for 3D printing can I import that model into Unreal engine or Unity 3D for use in a video game? Also is the inverse true? Can I get 3D model of Mario and send that to a 3D printer?

\n

Specifically, it\u2019s more important to know if I can pull a 3D printer model into an unreal game project

\n", "Title": "Can 3D printer models be used in video game engines and vs versa?", "Tags": "|3d-models|3d-design|file-formats|", "Answer": "

Regarding the second question: "Can I get 3D model of Mario and send that to a 3D printer?" meaning successful print, the answer depends on the model and the printer. Basically you can send any model to print, but each technology has its limitations of quality, size, physics. For figurines, resin printers (SLA or DLP technologies) seem best choice.

\n

Printed figurines would be non-movable and empty inside (to some extent), unless are specifically designed as mechanical toys. I think this is the key difference between mechanical movement ability and modern 3d object animations.

\n

As you already noticed, the mesh files could be reused here or there. So static objects like furniture pieces could be suitable for both purposes.

\n" }, { "Id": "15771", "CreationDate": "2021-03-01T16:03:20.437", "Body": "

I have found a video about Invisalign. I saw a 3D printer in the video but I did not understand its type. Can you help me? Do you share a brief piece of information about its type?

\n

Below is the video on YouTube.

\n

\r\n \r\n

\n", "Title": "What type of 3D printer is this?", "Tags": "|resin|part-identification|", "Answer": "

In the video, there is this still from 0:32:

\n

\"enter

\n

The labeling is iPro 8000, which is a 3dSystems resin printer using SLA technology.

\n" }, { "Id": "15772", "CreationDate": "2021-03-01T16:26:21.943", "Body": "

This is my first 3D printer so I'm not entirely sure what could be the cause of this issue.

\n

I recently got a SUNLU S8 3D printer and have been trying to print the first test file, however, the layers end up stringy (for lack of a better word -- if anyone knows whether this is called something else, I'd appreciate the knowledge because Google searching for stringy layers does not yield similar results).

\n

Anyway, I tried to increase the nozzle temperature from 205\u00a0\u00b0C to 215\u00a0\u00b0C and that seems to help slightly (filament is PLA and came with the printer). I also noticed that the extruder's feeder gear is grinding on the filament so I suspect that may be part of the problem.

\n

Before I purchase an extruder kit, I figured I'd reach out and ask this community what they think could be the problem.

\n

Does anyone know what might be causing this issue?

\n

\"Stringy

\n

Edit

\n

I cleaned the tubing and nozzle, and then releveled the bed. The filament was still stringy and whispy, however, I decided to let it print for a while. Although the picture is a top-down view, the walls of the print are also holey.

\n

\"After

\n", "Title": "Stringy first layer (possible extruder issue?)", "Tags": "|extruder|troubleshooting|underextrusion|stringing|", "Answer": "

0scar and Trish both had correct solutions to my problem.

\n

0scar suggested that I clean the nozzle, which I thought I had because the needle that came with the printer was able to slide through the hot end and out the nozzle without any issue. Turns out the hot end was clogged though. This particular hot end has a bowden tube that's separate from the actual bowden tube connected to the extruder gear, and that separate tube was clogged.

\n

\"bowden

\n

Trish was correct about the slicing profile being incorrect and that the problem was under extrusion. The filament provided by Sunlu with the printer seems to be fine. I let the printer run for 25 minutes and only had a slight issue with the filament on the spool not being tight enough when printing the raft. I'm currently running another print and haven't had any issues -- only 15 minutes in.

\n

Anyway, here's the 25 minutes print I was referring to (in the bottom left there's a glob of PLA because that's where the nozzle was when I stopped the print):

\n

\"successful

\n" }, { "Id": "15775", "CreationDate": "2021-03-01T20:35:48.233", "Body": "

I'm trying to get a sequential print going (first time trying this) and Cura won't slice due to build volume issues. From the graphic shown, it looks like this should be sufficiently spaced out but I'm not sure what the problem is. Any tips? I've moved the objects around all over the build volume and it never changes back to "allowable".

\n

\"enter

\n", "Title": "Cura Sequential Printing", "Tags": "|ultimaker-cura|", "Answer": "

If the height of the object is larger than the free space between the gantry and the top of the first printed object, you will not be able to slice the object. The maximum printing height will reduce to the maximum available distance between the gantry rods and nozzle.

\n

Note that the correct Gantry Height needs to be set in the printer properties. For instance, the UM3 I use has 60\u00a0mm free space between the gantry and the nozzle:

\n

\"enter

\n

If you have a smaller height specified, or a taller print object os depicted in the image below, you will not be able to slice the prints (note that a 100\u00a0x100\u00a0x100\u00a0mm is scaled to 20\u00a0x20\u00a0x70\u00a0mm prints, which exceed the 60\u00a0mm):

\n

\"enter

\n

When properly scaled below the 60\u00a0mm, i.e. to 20\u00a0x20\u00a0x50\u00a0mm print objects, you'll see that you are able to slice the objects:

\n

\"enter

\n

Slice preview:

\n

\"enter

\n" }, { "Id": "15776", "CreationDate": "2021-03-01T22:01:28.157", "Body": "

I printed this Curvy vase from Thingiverse and it came out pretty well on my Chiron.

\n

\"Curvy

\n

However, I am not happy with the Z-seam that is very large. When I look at other people's problems with this, they often seem to have too little filament at the seam, but I have too much. What setting should I change to make it less visible?

\n\n

All Cura settings are here (except that I lowered printing temperature to 200\u00a0\u00baC while printing).

\n

All files used and some pictures are here.

\n
\n

2021-03-07 Addendum after I made 19 test-prints of a small portion of the vase's neck. Below are some of my notes:

\n
    \n
  1. Combing Mode=All is better than Off
  2. \n
  3. Speed=60 is worse than 40
  4. \n
  5. Retract Before Outer Wall=On is worse than Off
  6. \n
  7. Outer Wall Line Width 0.45 to 0.35 gave a Z-seam on the outside with more build-up
  8. \n
  9. Inner Wall(s) Line Width 0.45 to 0.35. Some places has less contact between layers, so less appealing and less robust. Also less material use.
  10. \n
  11. Outer Wall Wipe Distance 2.0 spread ot the seam (too much), and also made a ditch before the Z-seam (on the outside of the ring).
  12. \n
  13. Coasting tripled to Vol=0.588 and Wipe Distance 5.0 is more appealing. Two changes at once make it impossible to know which one helped. 5 mm is not enough to completely wipe.
  14. \n
  15. 50% printing speed improved Z-seam and surface smoothness
  16. \n
\n

In the future I will use slower speed for Outer Walls, test Wipe distance=2*Line Width, use Combing (turned Off because of some advice to do so when LIN_ADVANCE is used) and experiment with faster retractions and Z-movement.\nPictures and complete notes are available here.

\n", "Title": "How to get less Z-seam?", "Tags": "|ultimaker-cura|anycubic-chiron|", "Answer": "

A bulging seam is caused by a mix of factors, especially:

\n
    \n
  1. Material oozing while dwelling too long at the point of layer change (including the time spent in the Z move). This can be mitigated by ramping up the max Z speed and acceleration and/or enabling retract at layer change with a very fast retract and unretract speed (without doing it very fast, the time spent retracting has the same effect as the time spent on Z move).

    \n
  2. \n
  3. Excess extrusion due to pressure remaining at the nozzle when decelerating. This can be mitigated by enabling Linear Advance and tuning the K factor for your printer's bowden tube and material properties. A value around 0.5 is probably needed for PLA on your printer. Increasing your acceleration limit to reduce the time spent accelerating/decelerating can also go a long way to mitigate this.

    \n
  4. \n
\n" }, { "Id": "15785", "CreationDate": "2021-03-03T16:50:57.653", "Body": "

I needed to know if typical UV curing resins, such as the ones from Anycubic, can withstand solar radiation over extended periods of time with little to no degradation. It has been quite difficult to find information about the subject online, so I'm wondering whether anyone here has experience here to share.

\n", "Title": "UV degradation of UV curable resins", "Tags": "|resin|", "Answer": "

Yes, it is actually pretty hard to find that information. First thing is that the resin never stops curing. Additional exposure to UV will continue to degrade the material over time. This is true of even plastics (and human flesh), however, in the case of UV resins we're talking about years not weeks. Each manufacturer of resin, even from different batches, will exhibit different performance characteristics with their formulation. As a general rule of thumb resin prints should not be used as an end product. They are used to either create a mould for the final object or used as a fitness test where the dimensions and tolerances are tight. The cured resin is typically not 100% safe to handle, and as such should not be used on bare skin or inside the human body or as any form of eating or cooking utensil. Colour degradation of resins (they become cloudy) will also occur for resins that were once colourless.

\n

EDIT: (Didn't want to expand this but it seems I have to) Companies that release 3d printed with branded resins have custom formulas to match their printers. This is perhaps to ensure that the customer keeps coming back for the consumables. These custom formulations are most undoubtedly patented, however, it seems that they are keeping the formula a secret (until you reproduce it and they claim you've breached their patent). Due to the fact that the formula is unknown to the end-user no one can certify that the cured resin is 100% non-toxic and safe for contact with skin; either on the outside of the body or the inside ;) Speciality dental adhesives have different formulas (and are perhaps more expensive than) 3d printing resins.

\n" }, { "Id": "15796", "CreationDate": "2021-03-04T22:52:14.840", "Body": "

I decided to finally purchase an Ender 3 v2. It arrived today and after a few hours of making sure everything is straight, I tried to power it and do a test print...

\n

I was disappointed to see that the screen stays black after more than 10 seconds from power on.

\n

I did not insert any SD card in the printer or anything that could interfere with the presumably stock firmware on it.

\n

Because of this, I opened up the power source and the motherboard case and checked the voltages/checked LEDs. The motherboard blue led is on, the power source green led is on as well. I did a few measurements - I get 24\u00a0V to the motherboard with a small spike of 35\u00a0V when it starts.

\n

The hotend fan is always on.

\n

I checked the display connection and I am sure it is placed correctly.

\n

I tried to upload firmware onto the mainboard first by clearing the SD card (8\u00a0GB given by Creality) using diskpart. I wrote the 4.2.2 (same as motherboard version) firmware version (stock) onto the SD attempting to upload on the mainboard. I tried renaming it, unplugging the power cable, unplugging the screen cable, etc and nothing would work.

\n

I tried rewriting the DWIN_SET files but the screen would not turn to black/blue/orange colors. I tried more or less everything suggested here.

\n

Is there anything I didn't do? What do you suggest? I already contacted the company where I bought it from, but I don't want to go through the assembly process twice - maybe I can make it work?

\n
\n\n", "Title": "Screen stays black on new Ender 3 V2 after power-on", "Tags": "|creality-ender-3|troubleshooting|firmware|maintenance|", "Answer": "

I solved this by having Creality send me a new main board and new screen.

\n

Sorry if you are experiencing this. Please contact Creality. They were happy to also upgrade my main board to v4.2.7!

\n" }, { "Id": "15797", "CreationDate": "2021-03-05T05:02:39.900", "Body": "

I had the BLTouch (3.1) working with stock firmware on an Ender 3 v2 but wanted to get a 5x5 mesh instead of the 3x3 so I installed the TH3D firmware. With the stock firmware my Z-offset was -2.95\u00a0mm.

\n

After installing TH3D firmware, my Z-offset has to be in excess of -6\u00a0mm to reach the same nozzle-to-bed height. I've tried setting the Z-offset via LCD and through G-code, saving to EEPROM. I'm having 2 problems with this that I assume are connected:

\n
    \n
  1. Z-offset seems to not be applied when setting Z axis position. When I autohome and then set Z axis to 0, even the extended probe from the BLTouch does not touch the bed (it's actually 1-2\u00a0mm away from touching). And this is with the Z-offset set to -6\u00a0mm.
  2. \n
  3. I tried running a print, which includes an ABL request (G29), the print head crashed into the bed immediately after finishing levelling. My hunch is that when starting to print the -6\u00a0mm Z-offset starts being correctly taken into account.
  4. \n
\n

Could this be the result of something I did in the Configuration.h when compiling the firmware?

\n

I made the following changes:

\n

CrealityV42X/Firmware/Marlin/Configuration.h

\n\n

In Configuration_backend.h the only change I made was increasing Z_PROBE_OFFSET_RANGE_MIN from -5 to -20.

\n", "Title": "Z offset not working after flashing TH3D firmware", "Tags": "|creality-ender-3|bltouch|th3d|", "Answer": "

A friend and I ran into this problem with TH3D and this was the workaround for our situation. Pretty sure I know what's causing it, but I haven't had the chance to rebuild the firmware yet to test it.

\n

Autohome, update your bed mesh, either through the menu or via the terminal with G28 & G29. Store settings via menu or M500 via terminal.

\n

Reboot/power cycle the printer, autohome/G28, and insert M420 S into your print file G-code after G28.

\n

Let me know if it works.

\n" }, { "Id": "15799", "CreationDate": "2021-03-05T19:24:51.967", "Body": "

I made two updates to my Ender 3 Pro: Silent Mainboard and BLTouch. My printer head is way out of sync now. After I Auto Home, it moves along the X-axis to the left, hits the stop switch & moves back to the right. It does not move at all on the Z-axis and remains 3 - 4" above the bed during and after Auto Homing.

\n

Here's the detail :

\n

I installed a Creality Ender 3 Pro New Upgrade Motherboard Silent Mainboard V4.2.7 with TMC2225 Driver Marlin 2.0.1 & bootloader pre-installed & then a Creality BLTouch 3D Printer Upgraded Auto Bed Leveling Sensor Kit (the kit that comes from Creality with everything you need to install). I imagine I attempted too many upgrades without verifying the printer worked properly when I did the first upgrade which was the mainboard?

\n

I purchased this Ender 3 Pro back in April 2020. It was working great until I decided to make the upgrades. I assume it came with an 8-bit board but not 100% sure. The new board is 32-bit and not sure what that would negatively impact besides maybe the LCD screen which does work fine after the upgrade.

\n

Here's a video of the printer attempting to Auto Home : \u00a0

\n

\r\n \r\n

\nNote: Disregard the unhooked cable under the printer bed in the video. \u00a0It is totally disconnected and leftover from the BLTouch install.

\n

Here's how I did the upgrade :

\n

First, I simply replaced the new mainboard, and with the mainboard cover and fan back in place, I powered the printer on. The only thing I did here was to verify that the printer would power up ok.\nIt powered up just fine. The nozzle head did not move I don't think when powered on.

\n

Next, I followed the instructions for the BLTouch that came with the kit. I followed the instructions for the Creality V1 Mainboard 32-bit. Here, I upgraded the firmware via an SD card as suggested. The firmware I upgraded to is the Ender-3 Pro_4.2.7_BLTouch_Marlin2.0.1_V1.1.2_TMC2225.bin.\nFinally, I powered on the printer with the BLTouch plugged in and new firmware installed. The BLTouch lights up and the touch sensor clicks out and in and few times.

\n

Now, when I hit auto home the nozzle head moves like I described above & remains in a position that I can not attempt to print from. I attempted to follow step #6 Platform adjustment 32-bit, working through the screenshots in the instructions. This did not work.

\n

If helpful, here are the positions of the nozzle head during the two upgrades :

\n\n

Position after install of BLTouch: remained the same. BLTouch lights up and the sensor tip moved in and out as if it was checking something or verifying it was working.

\n

I'm a bit lost on this issue. Any help will be greatly appreciated!

\n
\n

I'm using 3D4LYF's wiring scheme as I'm using the same set up I believe he is using. The two wires (white & black) on the red connector go in the Z-axis endstop port on the mainboard. For the three wires on the black connector I have: blue=Ground, red=V, yellow=IN that goes in the other 5 pin port on the mainboard. I rearranged the wiring from red/blue/yellow to blue/red/yellow as suggested by 3D4LYF again because I am using the BLTouch Creality Kit. Adding photos of 5-pin port and the connectors:

\n

\"enter\n\"enter

\n", "Title": "Ender 3 Pro will not auto home correctly after mainboard & BLTouch upgrades", "Tags": "|creality-ender-3|bltouch|creality|", "Answer": "

I had the same issue and thought I bricked it, so thank you for getting me on the right path! For Ender 3 Pro v4.2.2 motherboard, no Pinboard A, ISP Pinboard, or Burner required! In my case, I had used the provided pins (white housing) that were on the v4.2.2 motherboard itself.

\n

In doing so I had to switch the positions of the red and blue wires in the 3-pin connector (glad I pay attention to those types of things), connected white and black connectors right beside it as the OP has figured out (the connector my BLTouch came with was a little big so I had to cut a wall of the white housing a little; it fits really well), and re-connected the original Z-axis cable (did not need to remove to start with, but still unused as the BLT pretty much replaces it).

\n" }, { "Id": "15806", "CreationDate": "2021-03-06T11:28:04.237", "Body": "

Comparing trends show that 3D printing stepped over to another level around 2012~2013. Why?

\n

\"3D

\n

\"3D

\n", "Title": "What happened around 2012~2013 in 3D printing field", "Tags": "|history|", "Answer": "

Another opinion received:

\n
\n

That\u2019s the result of MakerBot and others promoting the tech to the general public for the first time after the patents expired. They were just running up stock prices to make a lot of money

\n
\n" }, { "Id": "15816", "CreationDate": "2021-03-08T05:49:16.207", "Body": "

I am building a 3D printer from scratch, the bed will only move on Z and the head will stay at the top of the printer and move X and Y.

\n

How do I modify the Marlin firmware to have the bed lower as it prints instead of lift like most printers.

\n", "Title": "How to setup Marlin to use a moving bed (up-down)?", "Tags": "|marlin|z-axis|", "Answer": "

You can control in Marlin what the direction of the stepper motor is, e.g. my Hypercube CoreXY printer (which has a similar setup like you described) has the following set (in the Marlin Configuration.h file) to ensure the platform raises when it has to decrease height:

\n
// Invert the stepper direction. Change (or reverse the motor connector) \n// if an axis goes the wrong way.\n#define INVERT_Z_DIR true\n
\n

Furthermore, it matters where the Z endstop is located, e.g. using a bed probe sensor or a min Z endstop, you need to home towards a decreasing height (in the direction of your probe/endstop):

\n
// Direction of endstops when homing; 1=MAX, -1=MIN\n#define Z_HOME_DIR -1\n
\n

Don't forget to set a max Z height that falls within the printer volume, e.g.:

\n
#define Z_MAX_POS 345\n
\n

If the bed is heavy, you should also prevent the steppers to lose power when not being used, so at least set Z to false:

\n
// Disables axis stepper immediately when it's not being used.\n// WARNING: When motors turn off there is a chance of losing position accuracy!\n#define DISABLE_Z false\n
\n" }, { "Id": "15827", "CreationDate": "2021-03-08T17:38:25.180", "Body": "

This one is turning out to be a real head-scratcher. I'm running a stock Creality CR-10S and there seems to be a single line approx 45\u00b0 across the print on the X-axis direction.

\n

I have attached images to better explain. Any help would be greatly appreciated.

\n

\"enter\n\"enter\n\"enter\n\"enter\n\"enter

\n", "Title": "Strange vertical lines on X-axis direction of parts", "Tags": "|print-quality|fdm|creality-cr-10|x-axis|", "Answer": "

In your slicer check your z-seam overlap. Lines like that are what happens when a slicer is systematically trying to hide a seam while not adding a ton of time onto the print by adding in a bunch of additional time for travel.

\n" }, { "Id": "15847", "CreationDate": "2021-03-11T22:43:48.540", "Body": "

I'm new to the 3D printing world but I've been using some of the 3D printers at my local library, and am now interested in purchasing my own! One thing that's important for my projects is being able to use variable layer thickness (for example, the first 50 layers are 0.3\u00a0mm, the next 50 are 0.1\u00a0mm, and the last 50 layers are 0.3\u00a0mm).

\n

I know the Prusa printers can do this, but I've been trying to understand whether the Creality Ender 3 allows you to use variable layer thickness. Unfortunately, I can't seem to find this information online so I would like to ask here!

\n", "Title": "Does the Creality Ender 3 allow variable layer thickness?", "Tags": "|creality-ender-3|slicing|layer-height|", "Answer": "

Variable layer height is a setting of the slicer, not an ability of the printer itself.

\n

However, the printer must be able to print at such layer heights. Any FDM (Fused deposition modeling\n) or FFF (fused filament fabrication) printer, which is the type you describe in the question, is able to print at 0.1 to 0.3\u00a0mm with at least a 0.4\u00a0mm nozzle diameter.

\n

There is also notion of optimal layer heights. Optimal hight depend on full steps of Z motor correlated with a leadscrew type. It is especially important when microstepping for Z is disabled or when printer disconnects a Z motor during long printing of single layer, otherwise positioning may get imprecise. Prusa calculator helps to determine these values. For 8 mm/revolution lead of screw (standard T8 in Ender 3) the step for optimal heights would be 0.04 mm.

\n" }, { "Id": "15857", "CreationDate": "2021-03-13T18:16:21.717", "Body": "

So I was thinking of using a closed-loop motor in my 3D printer for better accuracy and high-speed printing without losing steps. I have analyzed options available including just tweaking stepper current and firmware changes to prevent step losses but I want to find the best way to do it with encoders. The problem is I need a pretty high resolution for a 3d printer and usually, high-resolution servo motors use potentiometers that are limited and not suitable for a 3d printer. I am thinking of building my own closed-loop continuous rotation servo using stepper/DC motors and encoders but I can't find any high-resolution encoder at a reasonable price. So is there any way to somehow use low-resolution encoders (like 36 pulse every full turn) or is there any encoder type that I can use for better accuracy at a reasonable price other than optical ones? Also is there any other solution for closed-loop systems at a reasonable price?
\nNote that I am aware that I may need to modify firmware or write my own code for motors and program them from scratch.
\nNote: You might consider this question Closed-loop stepper motors a possible duplicate but I have read that and my main problem is not being able to find any cheap high-resolution encoder for this purpose.
\n
\nAlso for some reason, I can't use products like BTT S42B closed-loop stepper drives, my only option is to build them myself.
\nTL;DR:
\nIs there any type of encoder with high resolution and cheap price for use in 3D printers? (I don't mean brand, I mean technology)

\n", "Title": "What is the best way to build a closed loop continuous rotation servo for 3D printer?", "Tags": "|resolution|closed-loop|servo|", "Answer": "

You can use a magnetic position encoder.

\n

AS5048B High Resolution Position Sensor

\n

14-bit rotary position sensor with digital angle (interface) and PWM output

\n

14 bit means 16k steps/rotation. With a stepper which does 200 steps/rotation and 16x microstepping, you will need only 11 bit, so you have plenty of extra accuracy you can use to filter noise.

\n

You may use AS5600 Positioning Sensor instead, which is 12 bits, since you don't need to track each microsteps in a closed loop, 4x is enough.

\n

AS5600 in PWM mode does up to 920 Hz, AS5048 1 kHz. I'm not sure in I2C mode but surely more. Of course you have to take into account delay between measuring the position and transmission of the position.

\n" }, { "Id": "15859", "CreationDate": "2021-03-14T06:32:03.637", "Body": "

So I was thinking about is it possible to reach higher resolutions with encoders and DC motors? I found a cheap high-resolution magnetic encoder that can be used along with a DC motor to access higher resolutions. The encoder has 8192 PPR meaning that it can measure up to 0.04 degrees if I have correctly calculated. So if for a stepper with 0.9 step angle and no micro-stepping with 20 tooth pulley and 2mm belt, the steps/mm is 10, it means every 9 degrees with this pulley and belt setup makes 1mm and so 0.04 degree makes 0.004mm movement that is about 4 microns. Is this correct and possible? If so, why don't big companies use this method?
\nLink to the encoder:\nRLS RMB20 rotary magnetic encoder module

\n", "Title": "Is it possible to get higher resolutions by using high resolution encoders and custom firmware?", "Tags": "|resolution|closed-loop|servo|", "Answer": "

The mistake in your reasoning is assuming no microstepping. Most 3D printers use 16 microsteps, and in my experience with both cheap A4988 drivers and nice TMC2209 drivers, microstepping is quite accurate. As part of an answer to a question I asked, you can see a test print showing single-microstep features. My motors have 1.8\u00b0 step angle, yielding 3200 steps per rotation at 16 microsteps, or 12.5 microns of linear movement per microstep. With 0.9\u00b0 step angle you could get it down to half that, and you could probably halve it again going to 32 microsteps.

\n

Even if you can't get it as good as your 4 microns with stepper motors though, at 12.5 micron positioning resolution you're already to the point where extrusion error is going to play a much bigger role in dimensional accuracy than toolhead positioning error does. Going past that with FDM requires high resolution extruder axis movement, closed-loop control with a precise filament diameter sensor, direct drive with minimal distance between the extruder gear and nozzle, etc.

\n" }, { "Id": "15884", "CreationDate": "2021-03-17T16:41:51.110", "Body": "

Can I turn off the web interface (i.e. shutting down my computer) during a print? The Raspberry is online still.

\n

Am I correct to say that G-code has already been uploaded via WiFi to my OctoPi, and so turning off my computer now (web interface) will not disrupt the print?

\n", "Title": "Can OctoPrint web interface be turned off during a print?", "Tags": "|octoprint|", "Answer": "

Assuming the OctoPrint server is running on your Raspberry Pi, which is the usual "OctoPi" setup, then yes, once you have uploaded the G-code to OctoPrint, you no longer need to leave the web interface open.

\n

For example, one could upload code from their desktop running the slicing software to OctoPrint, walk to the printer to turn it on, then use a different device (a phone, for example) to connect OctoPrint to the printer and begin the print. At no point does the device accessing the web interface matter. The Raspberry Pi is in control of the printer, not the device that started the print.

\n" }, { "Id": "15888", "CreationDate": "2021-03-17T20:46:59.507", "Body": "

I have a Flex3Drive remote-direct-drive extruder I bought for my Ender 3 (with the original extruder motor), and I'm pretty happy with it except for abysmal retraction performance due to the 40:1 reduction, which requires the motor to turn about 480\u00b0 to retract just 1 mm. This is minimally workable for PLA and rigid materials where I can get by with just 1 mm of retraction, but it takes just as long to retract 1 mm as the original bowden extruder did to retract 6 mm, defeating a lot of the appeal of a direct drive. For TPU I haven't been able to make it workable at all. By the time I retract 3 mm or so, which seems necessary to avoid stringing, the hotend has already melted and/or deposited a blob on the print. Failure to print TPU also defeats much of the point of a direct drive.

\n

I'm running retraction at 8 mm/s, 500 mm/s\u00b2 acceleration, which is already higher than the manufacturer of the extruder recommends, and about the fastest I've been able to get it to work. Based on this calculator and 8.9 mH motor inductance, that seems roughly expected. I have TMC 2209 steppers and tried playing with current but it doesn't seem to make much difference.

\n

Otherwise, I love this extruder, and want to make it work. Is there a way I can salvage it by changing out the motor for something that can do higher RPMs and accelerate just as fast or faster?

\n", "Title": "Can I make this extruder work by changing motor?", "Tags": "|extruder|stepper|motor|", "Answer": "

I was considering buying a Fl3xdrive but had this exact fear, that the speed (rotation) required for retractions was too much.

\n

I stumbled upon this post while looking for a project for a step-up gearbox. I found a Nema 17 Gearbox "Pulleybox" Mod for Extruder on Thingiverse, should be enough to do what you did with your design, I guess.

\n" }, { "Id": "15891", "CreationDate": "2021-03-18T02:02:33.790", "Body": "

So basically I've been having a problem with my Micro+.\nIt will not level / calibrate itself and I can't fix it. The reason I'm here is that I've been using Cura, and somehow it destroyed my bed. (See image)

\n

\"print

\n

I would like to know how to get it off, as I tried freezing, scraping and sandpaper

\n
\n

To clarify some things:

\n\n

My build plate surface got destroyed after trying to use Cura, which sliced wrong and engraved the print into my bed.

\n", "Title": "Options for removing failed prints", "Tags": "|bed|m3d|", "Answer": "

You can try putting it in the oven at 100\u00a0\u00b0C and then peeling it off. At that temperature it's soft as cheese.

\n" }, { "Id": "15895", "CreationDate": "2021-03-18T14:21:53.377", "Body": "

I'm trying to make a frame for a lithograph that I plan to 3D print for my wife for Mother's Day. I typically will have my differences be exactly the right size (rather than oversizing it for the sake of the preview) because it gives me a good feel for how accurate my math is (particularly when I'm accounting for the horizontal margin created by the compressed filament coming out of the nozzle).

\n

In this case, however, it came back to bite me. I debugged this issue for quite a while assuming I had miscalculated something or that I had confused my order of rotations somehow.

\n

Eventually, I oversized my cutout and sure enough, the holes appeared correctly. After poking at it, I realized I only need 0.000005 mm on either side of my cutout for it to render correctly!

\n

I've created a minimal reproducible example of my issue here:

\n
outerRadius = 100;\nheight = 40;\n\nwallWidth = 2;\n\ninnerRadius = outerRadius - wallWidth / sin(60);\napothem = innerRadius * cos(180 / 6);\n\n// change this to 0 and see what happens!!\nholeSizeCorrection = 0.00001;\n\nmodule holes() {\n    for (face = [0:5]) {\n        rotate([0, 0, face * 60 + 30])\n            translate([apothem - holeSizeCorrection / 2, 0, height / 2])\n                rotate([0, 90, 0])\n                    cylinder(h = wallWidth + holeSizeCorrection, r = height / 4, $fn = 4);\n    }    \n}\n\ncolor("orange") difference() {\n    $fn = 6;\n    \n    cylinder(h = height, r = outerRadius);\n    \n    translate([0, 0, -height/4]) // because if it's exactly right, you can't see inside\n        cylinder(h = height * 2, r = innerRadius);\n\n    color("blue") holes();\n}\n
\n

When the holes I'm trying to cut in the polygon are exactly the width of the wall, it renders the preview correctly but the rendered output is incorrect. To verify that the shape was correct, I moved the holes out of the difference and it looks like this:\n\"enter

\n

The preview (aside from the strange exact cut issue that usually happens) is correct as well. But, when I render it, it looks like this:\n\"enter

\n

But, if I have the hole shape stick out 0.000005 mm on either side, it renders just fine!\n\"enter

\n

Now that I know to look for this kind of thing, it'll probably save me debugging time in the future. :) But, it would be nice to know if I've done something wrong as well.

\n", "Title": "Why does OpenSCAD fail to cut holes in polygon sides that are exactly the width of the wall?", "Tags": "|openscad|", "Answer": "

openSCAD simply allows having surface solutions that result in a wall of 0 thickness. The walls appear to clip in those areas and can at times be seen from both sides, like in your example:

\n

\"a

\n

A 0 thickness wall is also exportable into an STL as a set of triangles spun up by vertices that are in each other's plane but have inverted normal vectors for the two sides - which means that the construct exists for the computer - and I have used this in 3D designs for a hologram-effect, even if it does not result in a physically possible property set.

\n

An example of that effect is this cube (made in blender), that has the back wall purple, all others are grey - and the internal wall is at the same X-value as the back wall. You see the grey wall clipping despite the normal of it pointing to +X while the normal of the purple wall is into -X.

\n

\"enter

\n

By having your cookie-cutter extrude not just to but through the back wall, you force a solution that disallows the 0-surface solution and thus solves the problem of creating those artifact walls.

\n

Note that the 0-surface wall persists in the slicer preview:

\n

\"enter\n\"enter

\n

But rest assured: almost all slicers ignore walls that are too thin to be printed, and a 0 thickness wall especially is ignored:

\n

\"enter

\n" }, { "Id": "15908", "CreationDate": "2021-03-20T17:01:11.223", "Body": "

I want to build a mini CNC machine and need some lead screws. I was wondering I can simply 3D print some. There are a few 3D models out there but I want to know if printing it in PLA+ has enough strength for a small CNC. Is it possible?

\n", "Title": "3D Printed Lead Screw", "Tags": "|print-strength|lead-screw|cnc|", "Answer": "

Expanding on some previous comments which are probably enough to warrant an answer:

\n

What Trish said is completely right. Leadscrews are readily available parts and any dimensional errors in the leadscrews will be reflected in the output of your CNC machine unless you have some sort of compensation for them. Moreover, if the material is not highly rigid, the dimensions are subject to change over time, so any compensation would have to be ongoing manual adjustment or closed-loop rather than a one-time calibration. "PLA+" is an especially bad choice because it usually means PLA that's been modified with additives to make it less brittle, deforming under stress instead of holding its shape until it breaks catastrophically. CNC Kitchen's video on PLA+ elaborates on this.

\n

With that said, if you don't need a high level of precision, or if you're in a situation where you're unable to obtain manufactured components, I think 3D printed leadscrews would work ok if you print them in the XY plane rather than along the Z axis. While nozzle width and discrete layers produce a sort of "stairstep" quantization of printed threads in the Z direction, that doesn't happen with the threads in the XY plane; the nozzle width limits feature resolution (oscillations per unit length) but the positioning of the threads is quantized only to the X and Y (micro)step size, which is typically on the order of 10 microns. Moreover, the strength and rigidity of the part printed in this direction can be very high, due to the offset-layered zigzag structure.

\n

Back to accuracy of the part, though, it's important to note that whatever flaws your printer might have in XY positioning accuracy will be reflected in the resulting leadscrew. This includes non-linear effects such as belt paths being slightly trapezoidal instead of having perfectly colinear points of attachment to the carriage. In general, when manufacturing parts that will affect the accuracy of the resulting machine, you want to use processes that amplify the precision your tooling was manufactured with rather than processes that reproduce or amplify its flaws.

\n" }, { "Id": "15913", "CreationDate": "2021-03-21T15:23:20.693", "Body": "

Being new to 3D printing, I started using Cura (which came with my Ender 3v2) to slice models I found on Thingiverse. I know that there are other slicers and have heard positive things about PrusaSlicer.

\n

I know that settings will have different names, but I am asking more about the setup. What things, settings, etc. should I be aware of when using PrusaSlicer? Will I need to re-calibrate anything in PrusaSlicer?

\n", "Title": "Trying PrusaSlicer", "Tags": "|slicing|prusaslicer|", "Answer": "

PrusaSlicer already has pre-tuned profiles for the Ender 3 v2 in the Configuration Assistant. It also has tuned print settings from SUPERDETAIL (0.08\u00a0mm layer height) to SUPERDRAFT (0.28\u00a0mm layer height) so it shouldn't be too hard to set up.

\n" }, { "Id": "15919", "CreationDate": "2021-03-21T22:40:06.427", "Body": "

I have just set up my new 3D printer. However, the build plate keeps slipping while printing. I tried to use binder clips to keep the plate in place, but this makes the build plate unlevel, messing up my prints. What are some other ways to prevent the build plate from slipping. I was considering using duct tape, but am not sure whether this would work.

\n", "Title": "How can you prevent the build plate from slipping?", "Tags": "|build-plate|bed|build-surface|", "Answer": "

Build Plate

\n

If your clips aren't holding down your build plate, make sure your nozzle isn't crashing into the plate and moving it. A build surface between the clip and surface or adhesive might help keep the clip from slipping on the top surface. A rubbery gasket material that can withstand you heated bed temperatures, such as the silicon FarO mentions, can keep the bottom of the plate from slipping. This is the most important surface not to slip.

\n

Apparently, Oscar's option is to tape down the edges of the build plate. I have used Kapton tape to cover torn build surfaces. As Oscar mentioned, it is thin enough for the nozzle to pass over. Kapton tape can be a challenge to adhere at high bed temperatures, such as 110 \u00b0C. For that option it's probably worth checking the adhesive specs if they vary for different Kapton tapes.

\n

Those two options aren't mutually exclusive and could be used together. You could even put clips over the Kapton tape at higher bed temperatures if it is an issue.

\n

Build Surfaces

\n

PEI sheets are the most common build surfaces and already have an adhesive surface. Kapton tape seems to be the surface most resistant to damage, but a sheet that covers the entire build surface is expensive. Kapton seems to do a better job of releasing PETG. Otherwise, I use Elmer's glue sticks to protect the build surface.

\n" }, { "Id": "15922", "CreationDate": "2021-03-21T23:27:56.597", "Body": "

So I have been having an ongoing problem with my printer that I just can't solve. I don't usually ask the questions on stack exchanges until my problem gets unsolvable. Unfortunately like this one

\n

Printer: Creality Ender 3 V2

\n

Current Board: Creality v4.2.7 (Silent Motor)

\n

Background: This started when I accidentally fried the T20 chip on the v4.2.2 mainboard when I tried to install the BLTouch incorrectly (I looked at the photo upside down) and and also accidentally pulled the wire out and what I guess is why the T20 chip was fried. I thought at first the PSU was the one causing the trouble at first so I replaced it. Then found the burnt chip and decided to upgrade the board to a v4.2.7. I installed everything correctly according to a video that I found somewhere on YouTube.

\n

Current Problem(s): When I turn on the machine the light on the mainboard is on but there is continuous beeping coming from the display with small clicks like a clock every 1 sec. or so. The backlight to the display is on but nothing is shown. I just found with a multimeter that the bed and nozzle terminals give no volts back.

\n

Firmware: I have tried to update the firmware to the official one off of Creality's website and also using the source code for Marlin v2.0.7.2 with PlatformIO. The tutorial I followed was here:

\r\n \r\n

\n

Update [3/24]: Found that the firmware seems to not flash even after giving the .bin file a different name.

\n

Extra Thoughts: I currently have no idea what to do. I think I might replace the LCD and its cable. I have only printed a few things and just want to get back to it but I don't want to buy a whole new printer to do so. I feel like I get closer to getting back to it every time I fix something but I'm starting to lose hope that it's just a never ending problem loop. I appreciate any help given.

\n", "Title": "Ender 3 V2 blank screen and LCD continuous beeping with clicks", "Tags": "|marlin|creality-ender-3|troubleshooting|electronics|", "Answer": "

I had the constant beeping, black screen issue.

\n

I went to Creality Cloud Firmware for 3D Printers: Ender 3 v2 Mainboard version 4.2.2.

\n

I installed Ender-3 V2-Marlin-2.0.1 - V1.2.1 - Filament because I have the Ender 3 v2 4.2.2 board. This is the only firmware that loaded. I then reran all the other compatible firmware and it loaded them fine. But it only fixed itself once I ran the one with - Filament. No idea why!?!?! I spent hours and this is what did it.

\n

The site's root is Creality Cloud: Download Creality Firmware and Software.

\n" }, { "Id": "15932", "CreationDate": "2021-03-23T16:17:56.473", "Body": "

So I recently got an M3D Micro+ and have had 2 years of experience in 3D printing. I've had some issues with the printer and the prints produced and would like to know how I can fix them.

\n

The issues include:

\n\n

Here is a screenshot of one of the prints:\n\"enter

\n

The tolerances are also pretty bad. I would like to know recommended settings and such to help me get to actually printing. As of yet, I have not yet gotten a successful print.

\n", "Title": "What am I doing wrong with my M3D Micro?", "Tags": "|troubleshooting|layer-shifting|m3d|", "Answer": "

It's not you - it's the printer!

\n

The M3D Micro is not a very sturdy setup. The X-axis is a single pair of thin rods, hung up on a pair of similarly thin rods in Y and mounted on 3 very thin pillars in Z. While the idea is good, the execution is not particularly well: The rods are too thin and the design is virtually unchanged since 2015 and thus this review from 2016 still applies. As does this from 2018:

\n\n

All in all, you might squeak out better prints with a lot of calibration work, but the printer suffers so heavy from the design flaws that it would be a labor of love.

\n" }, { "Id": "15939", "CreationDate": "2021-03-24T22:46:00.393", "Body": "

I have an SVG file of a line drawing. When I import it into Tinkercad it generates a 3D version with the width of the lines remaining constant along the Z-axis (as expected). I would like to generate a similar STL but with the line width narrowing for increasing values of Z. (Similar to the Bevel option in some of the Tinkercad Text shapes).

\n

Can anyone point me at a tutorial on how to do this in Tinkercad, Blender, Meshmixer or Open SCAD?

\n", "Title": "Vary line with Z-axis for imported SVG file", "Tags": "|openscad|blender|meshmixer|tinkercad|", "Answer": "

In OpenSCAD you could create a bevel using the minkowski transformation by transforming the curve using tapered shape such as a pyramid or as I do below a tapered cylinder:

\n
minkowski() {\n\n  linear_extrude(1)\n    import("drawing.svg");\n  \n  cylinder(h = 20, r1 = 10, r2 = 1); \n}\n
\n

\"Example

\n" }, { "Id": "15952", "CreationDate": "2021-03-26T11:08:20.213", "Body": "

Various methods to scan the top surface exist to improve the bed adhesion to prevent prints to come loose from the bed during printing: e.g. BLTouch and clones or inductive and capacitive sensors/probes.

\n

In order, for the print head, to use this so-called auto-bed leveling (ABL) you need to add the G29 command in your start G-code of your slicer.

\n

Is it necessary to call G29 before every print?

\n
\n

The rationale behind this is that scanning the surface takes up some time, certainly on very short prints, it would be great if the surface geometry could be saved.

\n", "Title": "Using auto-bed leveling, do I need to initiate G29 before every print?", "Tags": "|g-code|bed-leveling|z-probe|knowledgebase|inductive-sensor|", "Answer": "

No, it is not necessary to call G29 before every print to "auto level the bed" 1) provided that:

\n\n

There are several solutions to solve this. You could manually run the G29 command once in a while storing the scanned surface with an M500 command to save the mesh to the EEPROM (memory) of the controller board (this can be done from the printer controller display for Marlin operated printers, an interface like a terminal or a print server application, or from pre-stored .g/G-code files on an SD card). If you use the SD-card, note that it is possible to auto-launch G-code files from the root of the SD-card upon inserting.

\n

Do note to remove the G29 command in the start code of the slicer. The G29 command needs to be replaced with M420 S1 for Marlin firmware operated printers. This command will load the saved mesh at the start of the print from memory. This is especially useful when using a large amount of probing points (e.g. a large bed mesh using a 10 x 10 mesh of 100 probing points, to ensure the mesh is up-to-date, once in a while initiate the scanning sequence to store an updated mesh).

\n
\n

1) Please note that auto-bed leveling might be confusingly indicating that some magic leveling of the build platform/surface itself is taking place (this is also possible in Marlin when there are multiple Z steppers and lead screws used), but, that is not actually what is meant with this phrasing. The process of the auto-bed leveling actually scans the surface of the build surface and compensates the height of the print head/nozzle during a predefined printing height (usually 10\u00a0mm, set in the firmware or through G-code: M420 Z10 ; Gradually reduce compensation until Z=10), during this printing process the nozzle gradually be less and less compensated until there is no compensation and the print nozzle will print parallel to the guide axis (e.g. the X-axis in i3 style printers and X-Y axes in CoreXY kinematics printers.

\n" }, { "Id": "15955", "CreationDate": "2021-03-26T14:42:01.453", "Body": "

I'm new to the 3D printing world and got my first Ender 3 (with the 32-bit controller board).

\n

I have a problem with every single print. I've upgraded to the newest Marlin firmware, did the mesh leveling then started printing and fix the Z bed option in "Tune - Z bed" during the print (so the first layer is perfect). Please note that I save all the data of the mesh before end of the print. Then, when I start the same print again, the nozzle height is wrong. This happens every single print. The strange thing is that I need to adjust the height differently after every print. Like, the first print was -0.055\u00a0mm, second print was 0.30\u00a0mm, then it was +0.25\u00a0mm somehow.

\n

I really like to print without these constant adjustments.

\n", "Title": "Nozzle height adjusting for every print", "Tags": "|creality-ender-3|bed-leveling|z-axis|", "Answer": "

Thanks everybody for their help, but apparently it was a slicer/software/gcode (i don't really know :D) issue...

\n

So i just added this line M420 S1 after the G28 command to my Cura printer settings and now the nozzle height is always at the point i set before, no changing every print.

\n

Hope my solve will help somebody who is struggling with this too!

\n" }, { "Id": "15966", "CreationDate": "2021-03-29T00:01:48.050", "Body": "

I have just acquired my first 3D printer, a BIQU B1. Overall I'm quite pleased with the printing results but I'm having minor defects on the Benchy test. I'm not sure what the problem is, but I guess it's related to the overhang and maybe vibrations.

\n

My print settings are:

\n\n

\"Imperfections

\n

\"Imperfections

\n

\"Imperfections

\n", "Title": "How to improve this Benchy? BIQU B1", "Tags": "|pla|troubleshooting|biqu-b1|", "Answer": "

Code from Celtic's answer:

\n
# generated by PrusaSlicer 2.3.0+win64 on 2021-04-17 at 15:16:02 UTC\n\n[print:BIQU B1 - Standard]\navoid_crossing_perimeters = 1\navoid_crossing_perimeters_max_detour = 75%\nbottom_fill_pattern = monotonic\nbottom_solid_layers = 4\nbottom_solid_min_thickness = 0\nbridge_acceleration = 0\nbridge_angle = 0\nbridge_flow_ratio = 1\nbridge_speed = 50\nbrim_width = 0\nclip_multipart_objects = 1\ncompatible_printers = \ncompatible_printers_condition = \ncomplete_objects = 0\ndefault_acceleration = 0\ndont_support_bridges = 1\ndraft_shield = 0\nelefant_foot_compensation = 0\nensure_vertical_shell_thickness = 0\nexternal_perimeter_extrusion_width = 0\nexternal_perimeter_speed = 50%\nexternal_perimeters_first = 0\nextra_perimeters = 1\nextruder_clearance_height = 20\nextruder_clearance_radius = 20\nextrusion_width = 0.4\nfill_angle = 45\nfill_density = 20%\nfill_pattern = cubic\nfirst_layer_acceleration = 0\nfirst_layer_extrusion_width = 0\nfirst_layer_height = 0.28\nfirst_layer_speed = 20\ngap_fill_speed = 20\ngcode_comments = 0\ngcode_label_objects = 0\ninfill_acceleration = 0\ninfill_anchor = 600%\ninfill_anchor_max = 50\ninfill_every_layers = 1\ninfill_extruder = 1\ninfill_extrusion_width = 0.48\ninfill_first = 0\ninfill_only_where_needed = 0\ninfill_overlap = 30%\ninfill_speed = 60\ninherits = \ninterface_shells = 0\nironing = 0\nironing_flowrate = 15%\nironing_spacing = 0.1\nironing_speed = 15\nironing_type = top\nlayer_height = 0.2\nmax_print_speed = 80\nmax_volumetric_speed = 0\nmin_skirt_length = 50\nnotes = \nonly_retract_when_crossing_perimeters = 1\nooze_prevention = 0\noutput_filename_format = [input_filename_base].gcode\noverhangs = 1\nperimeter_acceleration = 0\nperimeter_extruder = 1\nperimeter_extrusion_width = 0\nperimeter_speed = 50\nperimeters = 4\npost_process = \nprint_settings_id = \nraft_layers = 0\nresolution = 0\nseam_position = aligned\nsingle_extruder_multi_material_priming = 1\nskirt_distance = 5\nskirt_height = 1\nskirts = 1\nslice_closing_radius = 0.049\nsmall_perimeter_speed = 40%\nsolid_infill_below_area = 70\nsolid_infill_every_layers = 0\nsolid_infill_extruder = 1\nsolid_infill_extrusion_width = 0\nsolid_infill_speed = 50%\nspiral_vase = 0\nstandby_temperature_delta = -5\nsupport_material = 0\nsupport_material_angle = 0\nsupport_material_auto = 1\nsupport_material_buildplate_only = 0\nsupport_material_contact_distance = 0.2\nsupport_material_enforce_layers = 0\nsupport_material_extruder = 1\nsupport_material_extrusion_width = 0.35\nsupport_material_interface_contact_loops = 0\nsupport_material_interface_extruder = 1\nsupport_material_interface_layers = 3\nsupport_material_interface_spacing = 0\nsupport_material_interface_speed = 100%\nsupport_material_pattern = rectilinear\nsupport_material_spacing = 2.5\nsupport_material_speed = 60\nsupport_material_synchronize_layers = 0\nsupport_material_threshold = 0\nsupport_material_with_sheath = 1\nsupport_material_xy_spacing = 50%\nthin_walls = 0\nthreads = 24\ntop_fill_pattern = monotonic\ntop_infill_extrusion_width = 0\ntop_solid_infill_speed = 80%\ntop_solid_layers = 4\ntop_solid_min_thickness = 0\ntravel_speed = 150\nwipe_tower = 0\nwipe_tower_bridging = 10\nwipe_tower_no_sparse_layers = 0\nwipe_tower_rotation_angle = 0\nwipe_tower_width = 60\nwipe_tower_x = 180\nwipe_tower_y = 140\nxy_size_compensation = 0\n\n[filament:BIQU B1 - GSDT - PLA+ Silver]\nbed_temperature = 50\nbridge_fan_speed = 100\ncompatible_printers = \ncompatible_printers_condition = \ncompatible_prints = \ncompatible_prints_condition = \ncooling = 1\ndisable_fan_first_layers = 1\nend_filament_gcode = "; Filament-specific end gcode \\n;END gcode for filament\\n"\nextrusion_multiplier = 1\nfan_always_on = 1\nfan_below_layer_time = 100\nfilament_colour = #8B8B8B\nfilament_cooling_final_speed = 3.4\nfilament_cooling_initial_speed = 2.2\nfilament_cooling_moves = 4\nfilament_cost = 1000\nfilament_density = 1.3\nfilament_deretract_speed = nil\nfilament_diameter = 1.75\nfilament_load_time = 0\nfilament_loading_speed = 28\nfilament_loading_speed_start = 3\nfilament_max_volumetric_speed = 0\nfilament_minimal_purge_on_wipe_tower = 15\nfilament_notes = ""\nfilament_ramming_parameters = "120 100 6.6 6.8 7.2 7.6 7.9 8.2 8.7 9.4 9.9 10.0| 0.05 6.6 0.45 6.8 0.95 7.8 1.45 8.3 1.95 9.7 2.45 10 2.95 7.6 3.45 7.6 3.95 7.6 4.45 7.6 4.95 7.6"\nfilament_retract_before_travel = nil\nfilament_retract_before_wipe = nil\nfilament_retract_layer_change = nil\nfilament_retract_length = nil\nfilament_retract_lift = nil\nfilament_retract_lift_above = nil\nfilament_retract_lift_below = nil\nfilament_retract_restart_extra = nil\nfilament_retract_speed = nil\nfilament_settings_id = ""\nfilament_soluble = 0\nfilament_spool_weight = 1000\nfilament_toolchange_delay = 0\nfilament_type = PLA\nfilament_unload_time = 0\nfilament_unloading_speed = 90\nfilament_unloading_speed_start = 100\nfilament_vendor = (Unknown)\nfilament_wipe = nil\nfirst_layer_bed_temperature = 50\nfirst_layer_temperature = 195\nfull_fan_speed_layer = 4\ninherits = \nmax_fan_speed = 100\nmin_fan_speed = 100\nmin_print_speed = 10\nslowdown_below_layer_time = 10\nstart_filament_gcode = "; Filament gcode\\n"\ntemperature = 195\n\n[filament:BIQU B1 - PETG]\nbed_temperature = 70\nbridge_fan_speed = 35\ncompatible_printers = \ncompatible_printers_condition = \ncompatible_prints = \ncompatible_prints_condition = \ncooling = 1\ndisable_fan_first_layers = 3\nend_filament_gcode = "; Filament-specific end gcode \\n;END gcode for filament\\n"\nextrusion_multiplier = 1\nfan_always_on = 1\nfan_below_layer_time = 100\nfilament_colour = #29B2B2\nfilament_cooling_final_speed = 3.4\nfilament_cooling_initial_speed = 2.2\nfilament_cooling_moves = 4\nfilament_cost = 0\nfilament_density = 0\nfilament_deretract_speed = nil\nfilament_diameter = 1.75\nfilament_load_time = 0\nfilament_loading_speed = 28\nfilament_loading_speed_start = 3\nfilament_max_volumetric_speed = 0\nfilament_minimal_purge_on_wipe_tower = 15\nfilament_notes = ""\nfilament_ramming_parameters = "120 100 6.6 6.8 7.2 7.6 7.9 8.2 8.7 9.4 9.9 10.0| 0.05 6.6 0.45 6.8 0.95 7.8 1.45 8.3 1.95 9.7 2.45 10 2.95 7.6 3.45 7.6 3.95 7.6 4.45 7.6 4.95 7.6"\nfilament_retract_before_travel = nil\nfilament_retract_before_wipe = nil\nfilament_retract_layer_change = nil\nfilament_retract_length = 4\nfilament_retract_lift = nil\nfilament_retract_lift_above = nil\nfilament_retract_lift_below = nil\nfilament_retract_restart_extra = nil\nfilament_retract_speed = nil\nfilament_settings_id = ""\nfilament_soluble = 0\nfilament_spool_weight = 0\nfilament_toolchange_delay = 0\nfilament_type = PET\nfilament_unload_time = 0\nfilament_unloading_speed = 90\nfilament_unloading_speed_start = 100\nfilament_vendor = (Unknown)\nfilament_wipe = nil\nfirst_layer_bed_temperature = 75\nfirst_layer_temperature = 235\nfull_fan_speed_layer = 0\ninherits = \nmax_fan_speed = 35\nmin_fan_speed = 35\nmin_print_speed = 15\nslowdown_below_layer_time = 20\nstart_filament_gcode = "; Filament gcode\\n"\ntemperature = 235\n\n[printer:BIQU B1 (No Z Hop)]\nbed_custom_model = \nbed_custom_texture = \nbed_shape = 0x0,235x0,235x235,0x235\nbefore_layer_gcode = ;LAYER:[layer_num]\nbetween_objects_gcode = \ncolor_change_gcode = M600\ncooling_tube_length = 5\ncooling_tube_retraction = 91.5\ndefault_filament_profile = ""\ndefault_print_profile = \nderetract_speed = 0\nend_gcode = ;BIQU Default End Gcode\\nG91 ;Relative positioning\\nG1 E-2 F2700  ;Retract a bit\\nG1 E-2 Z0.2 F2400  ;Retract a bit more and raise Z\\nG1 X5 Y5 F3000 ;Wipe out\\nG1 Z10;Raise Z by 10mm\\nG90 ;Return to absolute positionning\\nG1 X0 Y235  ;TaDaaaa\\nM106 S0 ;Turn-off fan\\nM104 S0 ;Turn-off hotend\\nM140 S0 ;Turn-off bed\\nM84 X Y E ;Disable all steppers but Z\nextra_loading_move = -2\nextruder_colour = ""\nextruder_offset = 0x0\ngcode_flavor = marlin\nhigh_current_on_filament_swap = 0\nhost_type = octoprint\ninherits = \nlayer_gcode = \nmachine_limits_usage = emit_to_gcode\nmachine_max_acceleration_e = 5000,5000\nmachine_max_acceleration_extruding = 500,1250\nmachine_max_acceleration_retracting = 1000,1250\nmachine_max_acceleration_x = 500,1000\nmachine_max_acceleration_y = 500,1000\nmachine_max_acceleration_z = 100,200\nmachine_max_feedrate_e = 75,120\nmachine_max_feedrate_x = 250,200\nmachine_max_feedrate_y = 250,200\nmachine_max_feedrate_z = 10,12\nmachine_max_jerk_e = 2.5,2.5\nmachine_max_jerk_x = 8,10\nmachine_max_jerk_y = 8,10\nmachine_max_jerk_z = 0.2,0.4\nmachine_min_extruding_rate = 0,0\nmachine_min_travel_rate = 0,0\nmax_layer_height = 0\nmax_print_height = 270\nmin_layer_height = 0.07\nnozzle_diameter = 0.4\nparking_pos_retraction = 92\npause_print_gcode = M601\nprint_host = \nprinter_model = \nprinter_notes = \nprinter_settings_id = \nprinter_technology = FFF\nprinter_variant = \nprinter_vendor = \nprinthost_apikey = \nprinthost_cafile = \nremaining_times = 0\nretract_before_travel = 1.5\nretract_before_wipe = 0%\nretract_layer_change = 0\nretract_length = 7\nretract_length_toolchange = 10\nretract_lift = 0\nretract_lift_above = 0\nretract_lift_below = 0\nretract_restart_extra = 0\nretract_restart_extra_toolchange = 0\nretract_speed = 70\nsilent_mode = 0\nsingle_extruder_multi_material = 0\nstart_gcode = M220 S100 ;Reset Feedrate\\nM221 S100 ;Reset Flowrate\\nG28 ;Home\\nG92 E0 ;Reset Extruder\\nG1 Z2.0 F3000 ;Move Z Axis up\\nM109 S{first_layer_temperature[0]}; Preheat hotend\\nG1 X10.1 Y20 Z0.28 F5000.0 ;Move to start position\\nG1 X10.1 Y200.0 Z0.28 F1500.0 E15 ;Draw the first line\\nG1 X10.4 Y200.0 Z0.28 F5000.0 ;Move to side a little\\nG1 X10.4 Y20 Z0.28 F1500.0 E30 ;Draw the second line\\nG92 E0 ;Reset Extruder\\nG1 Z2.0 F3000 ;Move Z Axis up\ntemplate_custom_gcode = \nthumbnails = \ntoolchange_gcode = \nuse_firmware_retraction = 0\nuse_relative_e_distances = 0\nuse_volumetric_e = 0\nvariable_layer_height = 0\nwipe = 0\nz_offset = 0\n\n[printer:BIQU B1 (With Z Hop)]\nbed_custom_model = \nbed_custom_texture = \nbed_shape = 0x0,235x0,235x235,0x235\nbefore_layer_gcode = ;LAYER:[layer_num]\nbetween_objects_gcode = \ncolor_change_gcode = M600\ncooling_tube_length = 5\ncooling_tube_retraction = 91.5\ndefault_filament_profile = ""\ndefault_print_profile = \nderetract_speed = 0\nend_gcode = ;BIQU Default End Gcode\\nG91 ;Relative positioning\\nG1 E-2 F2700  ;Retract a bit\\nG1 E-2 Z0.2 F2400  ;Retract a bit more and raise Z\\nG1 X5 Y5 F3000 ;Wipe out\\nG1 Z10;Raise Z by 10mm\\nG90 ;Return to absolute positionning\\nG1 X0 Y235  ;TaDaaaa\\nM106 S0 ;Turn-off fan\\nM104 S0 ;Turn-off hotend\\nM140 S0 ;Turn-off bed\\nM84 X Y E ;Disable all steppers but Z\nextra_loading_move = -2\nextruder_colour = ""\nextruder_offset = 0x0\ngcode_flavor = marlin\nhigh_current_on_filament_swap = 0\nhost_type = octoprint\ninherits = \nlayer_gcode = \nmachine_limits_usage = emit_to_gcode\nmachine_max_acceleration_e = 5000,5000\nmachine_max_acceleration_extruding = 500,1250\nmachine_max_acceleration_retracting = 1000,1250\nmachine_max_acceleration_x = 500,1000\nmachine_max_acceleration_y = 500,1000\nmachine_max_acceleration_z = 100,200\nmachine_max_feedrate_e = 75,120\nmachine_max_feedrate_x = 250,200\nmachine_max_feedrate_y = 250,200\nmachine_max_feedrate_z = 10,12\nmachine_max_jerk_e = 2.5,2.5\nmachine_max_jerk_x = 8,10\nmachine_max_jerk_y = 8,10\nmachine_max_jerk_z = 0.2,0.4\nmachine_min_extruding_rate = 0,0\nmachine_min_travel_rate = 0,0\nmax_layer_height = 0\nmax_print_height = 270\nmin_layer_height = 0.07\nnozzle_diameter = 0.4\nparking_pos_retraction = 92\npause_print_gcode = M601\nprint_host = \nprinter_model = \nprinter_notes = \nprinter_settings_id = \nprinter_technology = FFF\nprinter_variant = \nprinter_vendor = \nprinthost_apikey = \nprinthost_cafile = \nremaining_times = 0\nretract_before_travel = 1.5\nretract_before_wipe = 0%\nretract_layer_change = 0\nretract_length = 7\nretract_length_toolchange = 10\nretract_lift = 1\nretract_lift_above = 0\nretract_lift_below = 0\nretract_restart_extra = 0\nretract_restart_extra_toolchange = 0\nretract_speed = 70\nsilent_mode = 0\nsingle_extruder_multi_material = 0\nstart_gcode = M220 S100 ;Reset Feedrate\\nM221 S100 ;Reset Flowrate\\nG28 ;Home\\nG92 E0 ;Reset Extruder\\nG1 Z2.0 F3000 ;Move Z Axis up\\nM109 S{first_layer_temperature[0]}; Preheat hotend\\nG1 X10.1 Y20 Z0.28 F5000.0 ;Move to start position\\nG1 X10.1 Y200.0 Z0.28 F1500.0 E15 ;Draw the first line\\nG1 X10.4 Y200.0 Z0.28 F5000.0 ;Move to side a little\\nG1 X10.4 Y20 Z0.28 F1500.0 E30 ;Draw the second line\\nG92 E0 ;Reset Extruder\\nG1 Z2.0 F3000 ;Move Z Axis up\ntemplate_custom_gcode = \nthumbnails = \ntoolchange_gcode = \nuse_firmware_retraction = 0\nuse_relative_e_distances = 0\nuse_volumetric_e = 0\nvariable_layer_height = 0\nwipe = 0\nz_offset = 0\n\n[printer:BIQU B1 Alpha v1]\nbed_custom_model = \nbed_custom_texture = \nbed_shape = 0x0,235x0,235x235,0x235\nbefore_layer_gcode = ;LAYER:[layer_num]\nbetween_objects_gcode = \ncolor_change_gcode = M600\ncooling_tube_length = 5\ncooling_tube_retraction = 91.5\ndefault_filament_profile = ""\ndefault_print_profile = \nderetract_speed = 35\nend_gcode = ;BIQU Default End Gcode\\nG91 ;Relative positioning\\nG1 E-2 F2700  ;Retract a bit\\nG1 E-2 Z0.2 F2400  ;Retract a bit more and raise Z\\nG1 X5 Y5 F3000 ;Wipe out\\nG1 Z10;Raise Z by 10mm\\nG90 ;Return to absolute positionning\\nG1 X0 Y235  ;TaDaaaa\\nM106 S0 ;Turn-off fan\\nM104 S0 ;Turn-off hotend\\nM140 S0 ;Turn-off bed\\nM84 X Y E ;Disable all steppers but Z\nextra_loading_move = -2\nextruder_colour = ""\nextruder_offset = 0x0\ngcode_flavor = marlin\nhigh_current_on_filament_swap = 0\nhost_type = octoprint\ninherits = \nlayer_gcode = \nmachine_limits_usage = emit_to_gcode\nmachine_max_acceleration_e = 5000,5000\nmachine_max_acceleration_extruding = 500,1250\nmachine_max_acceleration_retracting = 1000,1250\nmachine_max_acceleration_x = 500,1000\nmachine_max_acceleration_y = 500,1000\nmachine_max_acceleration_z = 100,200\nmachine_max_feedrate_e = 75,120\nmachine_max_feedrate_x = 500,200\nmachine_max_feedrate_y = 500,200\nmachine_max_feedrate_z = 10,12\nmachine_max_jerk_e = 5,2.5\nmachine_max_jerk_x = 8,10\nmachine_max_jerk_y = 8,10\nmachine_max_jerk_z = 0.4,0.4\nmachine_min_extruding_rate = 0,0\nmachine_min_travel_rate = 0,0\nmax_layer_height = 0\nmax_print_height = 270\nmin_layer_height = 0.07\nnozzle_diameter = 0.4\nparking_pos_retraction = 92\npause_print_gcode = M601\nprint_host = \nprinter_model = \nprinter_notes = \nprinter_settings_id = \nprinter_technology = FFF\nprinter_variant = \nprinter_vendor = \nprinthost_apikey = \nprinthost_cafile = \nremaining_times = 0\nretract_before_travel = 1.5\nretract_before_wipe = 0%\nretract_layer_change = 0\nretract_length = 3\nretract_length_toolchange = 10\nretract_lift = 0\nretract_lift_above = 0\nretract_lift_below = 0\nretract_restart_extra = 0\nretract_restart_extra_toolchange = 0\nretract_speed = 70\nsilent_mode = 0\nsingle_extruder_multi_material = 0\nstart_gcode = M220 S100 ;Reset Feedrate\\nM221 S100 ;Reset Flowrate\\nG28 ;Home\\nG92 E0 ;Reset Extruder\\nG1 Z2.0 F3000 ;Move Z Axis up\\nM109 S{first_layer_temperature[0]}; Preheat hotend\\nG1 X10.1 Y20 Z0.28 F5000.0 ;Move to start position\\nG1 X10.1 Y200.0 Z0.28 F1500.0 E15 ;Draw the first line\\nG1 X10.4 Y200.0 Z0.28 F5000.0 ;Move to side a little\\nG1 X10.4 Y20 Z0.28 F1500.0 E30 ;Draw the second line\\nG92 E0 ;Reset Extruder\\nG1 Z2.0 F3000 ;Move Z Axis up\ntemplate_custom_gcode = \nthumbnails = \ntoolchange_gcode = \nuse_firmware_retraction = 0\nuse_relative_e_distances = 0\nuse_volumetric_e = 0\nvariable_layer_height = 0\nwipe = 0\nz_offset = 0\n\n[printer:BIQU B1 Alpha v2]\nbed_custom_model = \nbed_custom_texture = \nbed_shape = 0x0,235x0,235x235,0x235\nbefore_layer_gcode = ;LAYER:[layer_num]\nbetween_objects_gcode = \ncolor_change_gcode = M600\ncooling_tube_length = 5\ncooling_tube_retraction = 91.5\ndefault_filament_profile = ""\ndefault_print_profile = \nderetract_speed = 35\nend_gcode = ;BIQU Default End Gcode\\nG91 ;Relative positioning\\nG1 E-2 F2700  ;Retract a bit\\nG1 E-2 Z0.2 F2400  ;Retract a bit more and raise Z\\nG1 X5 Y5 F3000 ;Wipe out\\nG1 Z10;Raise Z by 10mm\\nG90 ;Return to absolute positionning\\nG1 X0 Y235  ;TaDaaaa\\nM106 S0 ;Turn-off fan\\nM104 S0 ;Turn-off hotend\\nM140 S0 ;Turn-off bed\\nM84 X Y E ;Disable all steppers but Z\nextra_loading_move = -2\nextruder_colour = ""\nextruder_offset = 0x0\ngcode_flavor = marlin\nhigh_current_on_filament_swap = 0\nhost_type = octoprint\ninherits = \nlayer_gcode = \nmachine_limits_usage = emit_to_gcode\nmachine_max_acceleration_e = 5000,5000\nmachine_max_acceleration_extruding = 500,1250\nmachine_max_acceleration_retracting = 1000,1250\nmachine_max_acceleration_x = 500,1000\nmachine_max_acceleration_y = 500,1000\nmachine_max_acceleration_z = 100,200\nmachine_max_feedrate_e = 75,120\nmachine_max_feedrate_x = 500,200\nmachine_max_feedrate_y = 500,200\nmachine_max_feedrate_z = 10,12\nmachine_max_jerk_e = 5,2.5\nmachine_max_jerk_x = 8,10\nmachine_max_jerk_y = 8,10\nmachine_max_jerk_z = 0.4,0.4\nmachine_min_extruding_rate = 0,0\nmachine_min_travel_rate = 0,0\nmax_layer_height = 0\nmax_print_height = 270\nmin_layer_height = 0.07\nnozzle_diameter = 0.4\nparking_pos_retraction = 92\npause_print_gcode = M601\nprint_host = \nprinter_model = \nprinter_notes = \nprinter_settings_id = \nprinter_technology = FFF\nprinter_variant = \nprinter_vendor = \nprinthost_apikey = \nprinthost_cafile = \nremaining_times = 0\nretract_before_travel = 1.5\nretract_before_wipe = 0%\nretract_layer_change = 0\nretract_length = 7\nretract_length_toolchange = 10\nretract_lift = 0\nretract_lift_above = 0\nretract_lift_below = 0\nretract_restart_extra = 0\nretract_restart_extra_toolchange = 0\nretract_speed = 70\nsilent_mode = 0\nsingle_extruder_multi_material = 0\nstart_gcode = M220 S100 ;Reset Feedrate\\nM221 S100 ;Reset Flowrate\\nG28 ;Home\\nG92 E0 ;Reset Extruder\\nG1 Z2.0 F3000 ;Move Z Axis up\\nM109 S{first_layer_temperature[0]}; Preheat hotend\\nG1 X10.1 Y20 Z0.28 F5000.0 ;Move to start position\\nG1 X10.1 Y200.0 Z0.28 F1500.0 E15 ;Draw the first line\\nG1 X10.4 Y200.0 Z0.28 F5000.0 ;Move to side a little\\nG1 X10.4 Y20 Z0.28 F1500.0 E30 ;Draw the second line\\nG92 E0 ;Reset Extruder\\nG1 Z2.0 F3000 ;Move Z Axis up\ntemplate_custom_gcode = \nthumbnails = \ntoolchange_gcode = \nuse_firmware_retraction = 0\nuse_relative_e_distances = 0\nuse_volumetric_e = 0\nvariable_layer_height = 0\nwipe = 0\nz_offset = 0\n\n[printer:BIQU B1 Alpha v3]\nbed_custom_model = \nbed_custom_texture = \nbed_shape = 0x0,235x0,235x235,0x235\nbefore_layer_gcode = ;LAYER:[layer_num]\nbetween_objects_gcode = \ncolor_change_gcode = M600\ncooling_tube_length = 5\ncooling_tube_retraction = 91.5\ndefault_filament_profile = ""\ndefault_print_profile = \nderetract_speed = 35\nend_gcode = ;BIQU Default End Gcode\\nG91 ;Relative positioning\\nG1 E-2 F2700  ;Retract a bit\\nG1 E-2 Z0.2 F2400  ;Retract a bit more and raise Z\\nG1 X5 Y5 F3000 ;Wipe out\\nG1 Z10;Raise Z by 10mm\\nG90 ;Return to absolute positionning\\nG1 X0 Y235  ;TaDaaaa\\nM106 S0 ;Turn-off fan\\nM104 S0 ;Turn-off hotend\\nM140 S0 ;Turn-off bed\\nM84 X Y E ;Disable all steppers but Z\nextra_loading_move = -2\nextruder_colour = ""\nextruder_offset = 0x0\ngcode_flavor = marlin\nhigh_current_on_filament_swap = 0\nhost_type = octoprint\ninherits = \nlayer_gcode = \nmachine_limits_usage = emit_to_gcode\nmachine_max_acceleration_e = 5000,5000\nmachine_max_acceleration_extruding = 500,1250\nmachine_max_acceleration_retracting = 1000,1250\nmachine_max_acceleration_x = 500,1000\nmachine_max_acceleration_y = 500,1000\nmachine_max_acceleration_z = 100,200\nmachine_max_feedrate_e = 75,120\nmachine_max_feedrate_x = 500,200\nmachine_max_feedrate_y = 500,200\nmachine_max_feedrate_z = 10,12\nmachine_max_jerk_e = 5,2.5\nmachine_max_jerk_x = 8,10\nmachine_max_jerk_y = 8,10\nmachine_max_jerk_z = 0.4,0.4\nmachine_min_extruding_rate = 0,0\nmachine_min_travel_rate = 0,0\nmax_layer_height = 0\nmax_print_height = 270\nmin_layer_height = 0.07\nnozzle_diameter = 0.4\nparking_pos_retraction = 92\npause_print_gcode = M601\nprint_host = \nprinter_model = \nprinter_notes = \nprinter_settings_id = \nprinter_technology = FFF\nprinter_variant = \nprinter_vendor = \nprinthost_apikey = \nprinthost_cafile = \nremaining_times = 0\nretract_before_travel = 1.5\nretract_before_wipe = 0%\nretract_layer_change = 0\nretract_length = 4\nretract_length_toolchange = 10\nretract_lift = 0\nretract_lift_above = 0\nretract_lift_below = 0\nretract_restart_extra = 0\nretract_restart_extra_toolchange = 0\nretract_speed = 70\nsilent_mode = 0\nsingle_extruder_multi_material = 0\nstart_gcode = M220 S100 ;Reset Feedrate\\nM221 S100 ;Reset Flowrate\\nG28 ;Home\\nG92 E0 ;Reset Extruder\\nG1 Z2.0 F3000 ;Move Z Axis up\\nM109 S{first_layer_temperature[0]}; Preheat hotend\\nG1 X10.1 Y20 Z0.28 F5000.0 ;Move to start position\\nG1 X10.1 Y200.0 Z0.28 F1500.0 E15 ;Draw the first line\\nG1 X10.4 Y200.0 Z0.28 F5000.0 ;Move to side a little\\nG1 X10.4 Y20 Z0.28 F1500.0 E30 ;Draw the second line\\nG92 E0 ;Reset Extruder\\nG1 Z2.0 F3000 ;Move Z Axis up\ntemplate_custom_gcode = \nthumbnails = \ntoolchange_gcode = \nuse_firmware_retraction = 0\nuse_relative_e_distances = 0\nuse_volumetric_e = 0\nvariable_layer_height = 0\nwipe = 0\nz_offset = 0\n\n[printer:BIQU B1 Alpha v4]\nbed_custom_model = \nbed_custom_texture = \nbed_shape = 0x0,235x0,235x235,0x235\nbefore_layer_gcode = ;LAYER:[layer_num]\nbetween_objects_gcode = \ncolor_change_gcode = M600\ncooling_tube_length = 5\ncooling_tube_retraction = 91.5\ndefault_filament_profile = ""\ndefault_print_profile = \nderetract_speed = 35\nend_gcode = ;BIQU Default End Gcode\\nG91 ;Relative positioning\\nG1 E-2 F2700  ;Retract a bit\\nG1 E-2 Z0.2 F2400  ;Retract a bit more and raise Z\\nG1 X5 Y5 F3000 ;Wipe out\\nG1 Z10;Raise Z by 10mm\\nG90 ;Return to absolute positionning\\nG1 X0 Y235  ;TaDaaaa\\nM106 S0 ;Turn-off fan\\nM104 S0 ;Turn-off hotend\\nM140 S0 ;Turn-off bed\\nM84 X Y E ;Disable all steppers but Z\nextra_loading_move = -2\nextruder_colour = ""\nextruder_offset = 0x0\ngcode_flavor = marlin\nhigh_current_on_filament_swap = 0\nhost_type = octoprint\ninherits = \nlayer_gcode = \nmachine_limits_usage = emit_to_gcode\nmachine_max_acceleration_e = 5000,5000\nmachine_max_acceleration_extruding = 500,1250\nmachine_max_acceleration_retracting = 1000,1250\nmachine_max_acceleration_x = 500,1000\nmachine_max_acceleration_y = 500,1000\nmachine_max_acceleration_z = 100,200\nmachine_max_feedrate_e = 75,120\nmachine_max_feedrate_x = 500,200\nmachine_max_feedrate_y = 500,200\nmachine_max_feedrate_z = 10,12\nmachine_max_jerk_e = 5,2.5\nmachine_max_jerk_x = 8,10\nmachine_max_jerk_y = 8,10\nmachine_max_jerk_z = 0.4,0.4\nmachine_min_extruding_rate = 0,0\nmachine_min_travel_rate = 0,0\nmax_layer_height = 0\nmax_print_height = 270\nmin_layer_height = 0.07\nnozzle_diameter = 0.4\nparking_pos_retraction = 92\npause_print_gcode = M601\nprint_host = \nprinter_model = \nprinter_notes = \nprinter_settings_id = \nprinter_technology = FFF\nprinter_variant = \nprinter_vendor = \nprinthost_apikey = \nprinthost_cafile = \nremaining_times = 0\nretract_before_travel = 1.5\nretract_before_wipe = 0%\nretract_layer_change = 0\nretract_length = 4\nretract_length_toolchange = 10\nretract_lift = 0\nretract_lift_above = 0\nretract_lift_below = 0\nretract_restart_extra = 0\nretract_restart_extra_toolchange = 0\nretract_speed = 70\nsilent_mode = 0\nsingle_extruder_multi_material = 0\nstart_gcode = M220 S100 ;Reset Feedrate\\nM221 S100 ;Reset Flowrate\\nG28 ;Home\\nG92 E0 ;Reset Extruder\\nG1 Z2.0 F3000 ;Move Z Axis up\\nM109 S{first_layer_temperature[0]}; Preheat hotend\\nG1 X10.1 Y20 Z0.28 F5000.0 ;Move to start position\\nG1 X10.1 Y200.0 Z0.28 F1500.0 E15 ;Draw the first line\\nG1 X10.4 Y200.0 Z0.28 F5000.0 ;Move to side a little\\nG1 X10.4 Y20 Z0.28 F1500.0 E30 ;Draw the second line\\nG92 E0 ;Reset Extruder\\nG1 Z2.0 F3000 ;Move Z Axis up\ntemplate_custom_gcode = \nthumbnails = \ntoolchange_gcode = \nuse_firmware_retraction = 0\nuse_relative_e_distances = 0\nuse_volumetric_e = 0\nvariable_layer_height = 0\nwipe = 0\nz_offset = 0\n\n[printer:BIQU B1 Beta v1]\nbed_custom_model = \nbed_custom_texture = \nbed_shape = 0x0,235x0,235x235,0x235\nbefore_layer_gcode = ;LAYER:[layer_num]\nbetween_objects_gcode = \ncolor_change_gcode = M600\ncooling_tube_length = 5\ncooling_tube_retraction = 91.5\ndefault_filament_profile = ""\ndefault_print_profile = \nderetract_speed = 0\nend_gcode = ;BIQU Default End Gcode\\nG91 ;Relative positioning\\nG1 E-2 F2700  ;Retract a bit\\nG1 E-2 Z0.2 F2400  ;Retract a bit more and raise Z\\nG1 X5 Y5 F3000 ;Wipe out\\nG1 Z10;Raise Z by 10mm\\nG90 ;Return to absolute positionning\\nG1 X0 Y235  ;TaDaaaa\\nM106 S0 ;Turn-off fan\\nM104 S0 ;Turn-off hotend\\nM140 S0 ;Turn-off bed\\nM84 X Y E ;Disable all steppers but Z\nextra_loading_move = -2\nextruder_colour = ""\nextruder_offset = 0x0\ngcode_flavor = marlin\nhigh_current_on_filament_swap = 0\nhost_type = octoprint\ninherits = \nlayer_gcode = \nmachine_limits_usage = emit_to_gcode\nmachine_max_acceleration_e = 5000,5000\nmachine_max_acceleration_extruding = 500,1250\nmachine_max_acceleration_retracting = 1000,1250\nmachine_max_acceleration_x = 500,1000\nmachine_max_acceleration_y = 500,1000\nmachine_max_acceleration_z = 100,200\nmachine_max_feedrate_e = 75,120\nmachine_max_feedrate_x = 250,200\nmachine_max_feedrate_y = 250,200\nmachine_max_feedrate_z = 10,12\nmachine_max_jerk_e = 2.5,2.5\nmachine_max_jerk_x = 8,10\nmachine_max_jerk_y = 8,10\nmachine_max_jerk_z = 0.2,0.4\nmachine_min_extruding_rate = 0,0\nmachine_min_travel_rate = 0,0\nmax_layer_height = 0\nmax_print_height = 270\nmin_layer_height = 0.07\nnozzle_diameter = 0.4\nparking_pos_retraction = 92\npause_print_gcode = M601\nprint_host = \nprinter_model = \nprinter_notes = \nprinter_settings_id = \nprinter_technology = FFF\nprinter_variant = \nprinter_vendor = \nprinthost_apikey = \nprinthost_cafile = \nremaining_times = 0\nretract_before_travel = 1.5\nretract_before_wipe = 0%\nretract_layer_change = 0\nretract_length = 7\nretract_length_toolchange = 10\nretract_lift = 0\nretract_lift_above = 0\nretract_lift_below = 0\nretract_restart_extra = 0\nretract_restart_extra_toolchange = 0\nretract_speed = 70\nsilent_mode = 0\nsingle_extruder_multi_material = 0\nstart_gcode = M220 S100 ;Reset Feedrate\\nM221 S100 ;Reset Flowrate\\nG28 ;Home\\nG92 E0 ;Reset Extruder\\nG1 Z2.0 F3000 ;Move Z Axis up\\nM109 S{first_layer_temperature[0]}; Preheat hotend\\nG1 X10.1 Y20 Z0.28 F5000.0 ;Move to start position\\nG1 X10.1 Y200.0 Z0.28 F1500.0 E15 ;Draw the first line\\nG1 X10.4 Y200.0 Z0.28 F5000.0 ;Move to side a little\\nG1 X10.4 Y20 Z0.28 F1500.0 E30 ;Draw the second line\\nG92 E0 ;Reset Extruder\\nG1 Z2.0 F3000 ;Move Z Axis up\ntemplate_custom_gcode = \nthumbnails = \ntoolchange_gcode = \nuse_firmware_retraction = 0\nuse_relative_e_distances = 0\nuse_volumetric_e = 0\nvariable_layer_height = 0\nwipe = 0\nz_offset = 0\n\n[printer:BIQU B1 Beta v2]\nbed_custom_model = \nbed_custom_texture = \nbed_shape = 0x0,235x0,235x235,0x235\nbefore_layer_gcode = ;LAYER:[layer_num]\nbetween_objects_gcode = \ncolor_change_gcode = M600\ncooling_tube_length = 5\ncooling_tube_retraction = 91.5\ndefault_filament_profile = ""\ndefault_print_profile = \nderetract_speed = 0\nend_gcode = ;BIQU Default End Gcode\\nG91 ;Relative positioning\\nG1 E-2 F2700  ;Retract a bit\\nG1 E-2 Z0.2 F2400  ;Retract a bit more and raise Z\\nG1 X5 Y5 F3000 ;Wipe out\\nG1 Z10;Raise Z by 10mm\\nG90 ;Return to absolute positionning\\nG1 X0 Y235  ;TaDaaaa\\nM106 S0 ;Turn-off fan\\nM104 S0 ;Turn-off hotend\\nM140 S0 ;Turn-off bed\\nM84 X Y E ;Disable all steppers but Z\nextra_loading_move = -2\nextruder_colour = ""\nextruder_offset = 0x0\ngcode_flavor = marlin\nhigh_current_on_filament_swap = 0\nhost_type = octoprint\ninherits = \nlayer_gcode = \nmachine_limits_usage = emit_to_gcode\nmachine_max_acceleration_e = 5000,5000\nmachine_max_acceleration_extruding = 500,1250\nmachine_max_acceleration_retracting = 1000,1250\nmachine_max_acceleration_x = 500,1000\nmachine_max_acceleration_y = 500,1000\nmachine_max_acceleration_z = 100,200\nmachine_max_feedrate_e = 75,120\nmachine_max_feedrate_x = 250,200\nmachine_max_feedrate_y = 250,200\nmachine_max_feedrate_z = 10,12\nmachine_max_jerk_e = 2.5,2.5\nmachine_max_jerk_x = 8,10\nmachine_max_jerk_y = 8,10\nmachine_max_jerk_z = 0.2,0.4\nmachine_min_extruding_rate = 0,0\nmachine_min_travel_rate = 0,0\nmax_layer_height = 0\nmax_print_height = 270\nmin_layer_height = 0.07\nnozzle_diameter = 0.4\nparking_pos_retraction = 92\npause_print_gcode = M601\nprint_host = \nprinter_model = \nprinter_notes = \nprinter_settings_id = \nprinter_technology = FFF\nprinter_variant = \nprinter_vendor = \nprinthost_apikey = \nprinthost_cafile = \nremaining_times = 0\nretract_before_travel = 1.5\nretract_before_wipe = 0%\nretract_layer_change = 0\nretract_length = 7.5\nretract_length_toolchange = 10\nretract_lift = 0.5\nretract_lift_above = 0\nretract_lift_below = 0\nretract_restart_extra = 0\nretract_restart_extra_toolchange = 0\nretract_speed = 70\nsilent_mode = 0\nsingle_extruder_multi_material = 0\nstart_gcode = M220 S100 ;Reset Feedrate\\nM221 S100 ;Reset Flowrate\\nG28 ;Home\\nG92 E0 ;Reset Extruder\\nG1 Z2.0 F3000 ;Move Z Axis up\\nM109 S{first_layer_temperature[0]}; Preheat hotend\\nG1 X10.1 Y20 Z0.28 F5000.0 ;Move to start position\\nG1 X10.1 Y200.0 Z0.28 F1500.0 E15 ;Draw the first line\\nG1 X10.4 Y200.0 Z0.28 F5000.0 ;Move to side a little\\nG1 X10.4 Y20 Z0.28 F1500.0 E30 ;Draw the second line\\nG92 E0 ;Reset Extruder\\nG1 Z2.0 F3000 ;Move Z Axis up\ntemplate_custom_gcode = \nthumbnails = \ntoolchange_gcode = \nuse_firmware_retraction = 0\nuse_relative_e_distances = 0\nuse_volumetric_e = 0\nvariable_layer_height = 0\nwipe = 0\nz_offset = 0\n\n[presets]\nprint = BIQU B1 - Standard\nsla_print = \nsla_material = \nprinter = BIQU B1 (With Z Hop)\nfilament = BIQU B1 - GSDT - PLA+ Silver\n
\n" }, { "Id": "15970", "CreationDate": "2021-03-29T12:24:22.243", "Body": "

I am designing some parts that should modular fit together. I am currently exploring a Lego-like design with octagonal holes and cylindrical pins.

\n

I notice that (depending on the amount of clearance) that the fit is initially tight (to the extent that the pieces are very difficult to remove from each other), but that after a few dozen times connecting and disconnecting the parts the fit becomes very loose. I am currently using PLA. With what material would this occur less quickly/is more resistant to this kind of wear?

\n

The sizes of the pins/holes are slightly bigger than Lego (probably similar to Duplo). Don't think that snap-fit would work in that size. Below of a picture of one of the test pieces (this one later printed in PETG).

\n

\"enter

\n

I have an open printer so I prefer materials that don't require me to build an enclosure first. It is a Prusa i3 MK3S: Direct drive; 1.75\u00a0mm filament; max temp 300\u00a0\u00b0C; heat bed max temp 120\u00a0\u00b0C.

\n", "Title": "What material best to use for press fit parts?", "Tags": "|filament-choice|press-fit|", "Answer": "

If you're aiming for a Lego-like snap fit, look at Lego parts to see how they work. Using the right material is important, but using it the right way is at least as important. Lego bricks are made so that the walls and tubes can deform very slightly as the studs of another brick are pushed into the brick.

\n

The walls of the octagonal holes in your part appear to be solid. Perhaps they're not really, and underneath the surface layer there's a lesser amount of infill, but infill can provide a lot of strength, and that solid surface will prevent deformation at the entrance of the hole.

\n

Try removing the solid bottom layers and reducing or eliminating the infill in the part so that the walls of your holes can flex more.

\n" }, { "Id": "15987", "CreationDate": "2021-03-31T16:17:25.717", "Body": "

The thermoresistor on my Anycubic Mega displays nonsense temperatures (e.g. 700\u00a0\u00b0C), since the resistor in the hotend is working fine the problem is 100\u00a0% in the board (Trigorilla RAMPS 1.4). Someone suggested switching the pin of the thermoresistor from T0 to T1, so I soldered it this way. Now I have to cook a personalized firmware for making it work. I opened a custom firmware in VSCode but I do not know what parameter I have to change, any idea?

\n", "Title": "How to change pin of thermo sensor?", "Tags": "|firmware|ramps-1.4|anycubic-i3-mega|", "Answer": "

If there are 2 slots for temperature measurements, you don't need to solder anything, just plug the thermistor from one into the other and switch pins in the firmware. This board is basically a RAMPS 1.4 board, it includes the pins_RAMPS.h header file, so in order to switch the T0 with the T1 temperature port, you need to change:

\n
//\n// Temperature Sensors\n//\n#ifndef TEMP_0_PIN\n  #define TEMP_0_PIN                          13  // Analog Input\n#endif\n#ifndef TEMP_1_PIN\n  #define TEMP_1_PIN                          15  // Analog Input\n#endif\n
\n

to:

\n
//\n// Temperature Sensors\n//\n#ifndef TEMP_0_PIN\n  #define TEMP_0_PIN                          15  // Analog Input\n#endif\n#ifndef TEMP_1_PIN\n  #define TEMP_1_PIN                          13  // Analog Input\n#endif\n
\n" }, { "Id": "16005", "CreationDate": "2021-04-01T22:55:37.127", "Body": "

Suddenly my prints start having small strings and I'm not sure what to do to eliminate them.

\n

I have already tried to clean the filament tube, cleaned up the nozzle, and played with various retraction settings. Nothing seems to work and I still see these small strings.

\n

\"3D

\n

Any idea how I can eliminate it?

\n", "Title": "Ender 3 - Tiny strings in print", "Tags": "|creality-ender-3|print-quality|troubleshooting|stringing|", "Answer": "

I would say that it IS stringing. If you have a Bowden setup, AND you have a different filament, your retraction may need to be tuned! If this is not a Vase Mode (no separate layers, but a continuous stream, these could be because the start/stop point of the layer change is not "fixed"...Check that setting in the slicer! While it looks like missed layer adhesion, it is probably a movement from one stop point of a layer to the start point of the next layer.

\n" }, { "Id": "16009", "CreationDate": "2021-04-02T10:01:41.570", "Body": "

Recently I bought a printer and failed to find Cura 4.8 settings for it (both in application both myself). My printer Zonestar Z6FB

\n

Currently I doubt in:

\n
    \n
  1. G-code flavor
  2. \n
  3. All printhead settings excluding extruder number (which is obviously 1 in my case)
  4. \n
  5. Start/End G-code
  6. \n
\n

\"enter

\n

Add printer image to eye trap someone who also interested

\n

\"enter

\n", "Title": "Could anyone please provode me settings for Custom FFF for Zonestar Z6FB printer", "Tags": "|ultimaker-cura|", "Answer": "

After some pain, I figured out that if you are a poor Ubuntu/Linux user you should only use the AppImage version of Cura which you can download from official website

\n

Next, make the file you downloaded executable chmod +x *.AppImage

\n

Next, start it from terminal or by clicking in navigator

\n

Ok, proceed as is and then Help \u25b6 Show Configuration Folder.

\n

It will show 2 folders we need ~/.local/share/cura/4.8

\n

Ok now download ZONESTAR_Cura_Resources.zip from here.

\n

Then copy all files from ZONESTAR_Cura_Resources TO ~/.local/share/cura/4.8 but remember that directory structure is messed up but can be figured out.

\n" }, { "Id": "16016", "CreationDate": "2021-04-03T07:46:59.000", "Body": "

I have following G-code to prime the nozzle before start of the print.

\n
G28 ; home all axes\nG0 Z5 F5000 ; lift nozzle\nG0 X30; move to X30\nG28 Y; home Y\nM420 S1; turn on bed leveling\nM109 S220; wait for hotend temperature\nG1 E20 F1800; extrude filament 20mm\n
\n

The idea is to extrude a strand of filament outside of the bed and then start the print. The strand catches on the bed a tears off. So the ooze does not mess up the first layer.

\n

The problem is that the G1 E20 F1800 does not wait for the move to finish and the controller goes to next move immediately. This means the nozzle is going to start the first layer, while spewing filament along the way.

\n

Is there a way to wait for the move to finish?

\n

I have tried M400 which seems not to help. I'm using Marlin Firmware.

\n", "Title": "G-code wait for extrude command to finish", "Tags": "|marlin|g-code|", "Answer": "

Are you sure the move didn't finish? That would be very unusual, not the way 3D printer firmware normally operates. A new G0/G1 move command does not execute until the previous one finishes, whether it's extrude-only, travel-only, or a print move (mix of extrusion and travel).

\n

What's probably happening for you is that the amount of material you're trying to extrude cannot melt and pass through the extruder at the speed you requested. At 1800\u00a0mm/min, assuming 1.75\u00a0mm diameter filament, you're requesting over 72\u00a0mm\u00b3 of material (nearly a whole cubic centimeter!) of material to be extruded per second. According to some back-of-the-envelope calculations I just did, It would take more than 300\u00a0Watts to continuously raise PLA from room temperature to extrusion temperature at that rate, which is not happening without an extreme hotend and power supply.

\n

So, what you're getting is pressure building up between the extruder gear and the hotend (until it starts slipping), causing the material to continue to ooze for some time after the E-axis move finishes, until all the pressure subsides. If your goal is to prime the nozzle for printing, this is not how to do it. It will just end up oozing material all over the start of your print.

\n

You can somewhat fix this by just reducing the feedrate in your command, but that's still not necessarily going to give you great results. The right way to prime is to extrude at nearly the maximum rate your hotend can handle, then retract and wipe before traveling to where the print will begin.

\n" }, { "Id": "16017", "CreationDate": "2021-04-03T08:45:19.673", "Body": "

In slicer G-code is it possible to set the probe Z offset for the first layer only?

\n

For my stock E3 plate I find -4.125\u00a0mm best however for glass I need to go to -4.175\u00a0mm for the first layer to get better adhesion. So it's a manual process every time. Any way to tell the slicer do the first layer at -4.175\u00a0mm and next ones at -4.125\u00a0mm?

\n", "Title": "First layer Z-offset in G-code", "Tags": "|creality-ender-3|g-code|bltouch|z-probe|", "Answer": "

Trying to change the Z level (NOT the offset) during a print isn't a good idea UNLESS you know that all future moves will be "relative".

\n

The M206 (if you use Marlin) is the right way to set it! Depending on the slicer, just create TWO "different" printer definitions and add them to the startup G-codes.

\n

If you don't save it, a reset will return the z-offset to the "saved" value. Or consider resetting to a "standard" value in the ending G-code.

\n

Also, consider using "baby-stepping" (if you can), and manually adjusting things at the start of a print with a different surface.

\n" }, { "Id": "16019", "CreationDate": "2021-04-03T10:50:48.707", "Body": "

My sample filament has just ran out. Luckilly, I've already bought more filament. The only problem I have is that the spool for the new filament won't fit inside my 3D printer. The filament itself works with my printer; it's just the spool that is the problem. Does anyone have any solutions for what to do if a filament spool won't fit in your 3D printer? For reference, I have the FlashForge Adventurer 3.

\n", "Title": "What to do if filament spool won't fit in 3D printer", "Tags": "|troubleshooting|filament|flashforge-adventurer-3|change-filament|", "Answer": "

A spool does not need to be inside a printer. Or on. Or even next to.

\n

My Ender 3 pulls his filament in from the rack above it, my TronXY X1 pulls it from about 80 cm to the left of it, where it hangs from a shelf.

\n

When making a solution that pulls in filament from afar, it is necessary to make sure the path is unobstructed and works for the whole movement range of the printer without getting bent sharply, as that can snap or kink the filament.

\n

In the case of the small printer you have, leave open or remove the door to the filament chamber and make sure to place the spool holder so it drags in the filament straight. There are even spool holders designed specifically for this printer.

\n

There are many designs of spool holders out there, many of them free and with minimal assembly.

\n

RE-spooling

\n

Some printers, especially ones that only take marked rolls, might need their spool cores re-filled. In that case, you need to take utmost care: re-filling a spool needs to be without any twist to the filament or you risk entangling, and you need to make sure to not kink or bend the filament in doing so, or risk breaking at those spots.

\n

Due to the risks involved, this should be avoided whenever possible!

\n" }, { "Id": "16025", "CreationDate": "2021-04-03T22:50:13.897", "Body": "

I have a FlashForge Adventurer 3. I have just finished my sample filament today and tried to replace it with a new spool of filament. This new filament is by Hatchbox, instead of FlashForge. Besides that, the new filament is 1.75\u00a0mm PLA filament, like the sample spool. However, when I tried to insert the new filament, it doesn't feed in. I get a loud, thumping sound from the printer, but the filament doesn't get fed through. Does anyone know why this is happening or what I should do?

\n", "Title": "Unable to load new filament FlashForge Adventurer 3", "Tags": "|filament|change-filament|flashforge-adventurer-3|flashforge|", "Answer": "

First thing I'd try is making sure the nozzle is hot enough and carefully try to feed the new filament where it will be pushed - sometimes there's melted filament that just isn't hot enough to flow out and the new filament is pushing on it, jamming and making your motor make that sound. Usually getting it hot to the point where the old filament is practically dripping out before attempting to insert the new filament will work.

\n

If that doesn't work I might let the thing cool, disassemble the nozzle, and remove the jam mechanically (and reassemble it).

\n" }, { "Id": "16031", "CreationDate": "2021-04-04T20:13:46.087", "Body": "

I've used TinkerCAD (https://www.tinkercad.com/) and was able to easily model objects for 3D printing, despite having no prior experience in 3D modeling.\nHowever it needs to be connected to the internet to work, which is not always available at my location.\nAt minimum, I need an ability to create solid shapes and holes, and with resize, align and rotate options.

\n

Is there an equivalent program that's free, with a similar intuitive interface as TinkerCAD, and works completely offline?

\n", "Title": "Recommendation for free, intuitive offline 3D modeling software for 3D printing", "Tags": "|3d-design|software|", "Answer": "

From trying out several alternatives (Fusion 360, Wings 3D, Blender, and others), the closest free offline equivalent to TinkerCAD turned out to be OpenSCAD.

\n

It has nowhere near the same level of intuition,\nbut based on info from two tutorials ("OpenSCAD Tutorial - Beginners Quickstart", "OpenSCAD - Model a Bearing in less than ten minutes."), OpenSCAD was able to fulfill the minimum requirements that were needed from TinkerCAD most quickly, i.e.,"...ability to create solid shapes and holes, and with resize, align and rotate options"

\n

\"enter

\n" }, { "Id": "16044", "CreationDate": "2021-04-07T12:24:10.323", "Body": "

Long story for hopefully a simple yes/no answer:

\n

I have a lightly modified Ender 3 Pro (BLTouch, glass bed, beefier bed springs and nobs, led lights) and I daydream about someday getting an "Ender Extender" kit to go larger or something, but before I ever do that I need to drastically speed up the prints. Last night the first layer on one of my prints took 40 minutes and I watched the whole time to make sure it laid down well. Can't imagine waiting around with a larger build surface. I understand basic settings like lighter infill/less perimeters/etc... but when it comes to really changing the speed and acceleration settings I'm a bit of a newb.

\n

So after reading this article on improving print speeds, it seems like the Bowden tube style default extruder on my Ender 3 probably doesn't have the grip strength and/or constrained filament path to really push filament faster.

\n

Based on that same article and this one, it seems like there are a lot of options for extruders and hotends to upgrade. The reason the Micro Swiss appealed to me is that it seems like this option is pretty much bolt-on plug and play (additional info here).

\n

And it seems like it already has a convenient spot for my BLTouch.

\n

Which brings me to the title of my question: will that Micro Swiss extruder/hotend combo do what I think it'll do? i.e. allow me to push filament faster and heat it up quicker?

\n

Is there a better alternative extruder/hotend that is also pretty easy to figure out?

\n

Lastly (maybe this should be a different question/too open-ended) are there other relatively simple upgrades/modifications to the Ender 3 Pro that will help it print faster?

\n", "Title": "Will a Micro Swiss extruder and hotend speed up prints on my Ender 3 Pro?", "Tags": "|creality-ender-3|extruder|hotend|", "Answer": "

If you want a single word answer, then no.

\n

Before you ask this question you need to figure out why your prints are slower than you want them to be. With default settings, unless your models are something dead simple like a big cube or cylinder, you're almost certainly limited by acceleration, not extrusion rate or even max print/travel speed (which you'll almost never achieve). Gcode analysis tools like gcodeanalyser.com will help you gauge this by predicting actual speeds the printer will achieve. Note that even if your model is simple, printing infill and top/bottom layers involves a lot of acceleration/deceleration cycles, so even for simple models this may still be your limiting factor.

\n

Until you reach very high accelerations (over 5000 mm/s\u00b2) letting you actually achieve very high speeds (over 150 mm/s or so), the only way to make extrusion your limiting factor is by using really thick layers or wide walls. At 0.2 mm layer height and 0.4 mm line width, even a sustained 150 mm/s is only 12 mm\u00b3/s volumetric extrusion rate which is high but reportedly within the capabilities of the stock Ender 3 extruder and hotend (but probably requires cranking up the temperature).

\n" }, { "Id": "16061", "CreationDate": "2021-04-09T17:59:05.883", "Body": "

I have a Prusa MK3s MMU2s which I was running a 7-hour print job on.

\n

It had two spools of filament in, a blue one (primary, very little left) and a gold one (secondary, quite a lot left). It ran out of blue filament approximately 75% through the print job and rather than simply using the remainder of the blue and switching to the gold, the blue was bent and caught on the output nozzle of my spool container. It says that:

\n
\n

MMU needs user attention, fix issue then press button on MMU unit.

\n
\n

I have fixed the issue (snipped the bent part of the blue filament), however, I can't figure out how to make it continue printing. After fiddling a little bit, the red and green lights are all flashing, but no matter what MMU buttons I press, it doesn't continue printing. How should I fix this without cancelling the print job?

\n", "Title": "How to unload filament without cancelling print", "Tags": "|prusa-i3|prusa-mmu|", "Answer": "

This condition is covered in the MMU2S manual's section on troubleshooting:

\n
\n

13.4 All five LEDs blinking red and green

\n

MMU2S unit can now deal with a state in which the Trinamic drivers were not able to provide enough power for steppers. A possible cause can be a broken MMU2S power supply wiring, connectors etc. Please check that all connectors are properly plugged into the MMU2S board and make sure that the cables are in good condition. Also, make sure that the screws on the Einsy terminal box for the power cables are properly tightened. If this state is detected, the MMU2S unit rehomes and automatically continues printing. If the problem occurs three times in a row, the MMU2S unit stops printing and starts flashing continuously with red and green LEDs. Printing can be restored by restarting MMU2S unit using the button on the right side of MMU2S unit.

\n
\n

So, once you have fixed the problem which prevented the filament from moving, press the reset button on the MMU2S. It is recessed, so you will need a tool to push it (such as the 2.5 mm Allen key included with the printer). You will then need to wait a few seconds for the MMU2 to complete its reboot and be ready to communicate, before telling the printer to continue.

\n

Note that the filament may have been chewed by the MMU2 or extruder drive gears and unable to feed properly. Be prepared to \u2014 if you haven't already \u2014\u00a0disassemble the tubing and manually pull out the remaining filament sections. I find this modified holder for the MMU2's filament tubes that uses push-in couplings instead of a clamp very useful to allow quick access when there's a problem or even when inserting new filament.

\n" }, { "Id": "16065", "CreationDate": "2021-04-10T08:39:04.950", "Body": "

I am looking for a way to easily separate the wires between the power supply and the Einsy board of my Prusa Mk 3S+ with a connector. The power supply delivers 10\u00a0A at 24\u00a0V.

\n

I thought about a YL wire-to-wire electrical connector as it can handle up to 7\u00a0A at 300\u00a0V. This should be enough since the two cables share the total load.

\n

\"enter

\n

The 6 cables (4 power cables & 2 PSU power panic cables) have different diameters. Is it better to use one connector (6 pin) or two connectors (4 pin, 2 pin)?

\n", "Title": "Which connector should I use between the power supply and the Einsy board?", "Tags": "|prusa-i3|prusa-i3-rework|wire-type|", "Answer": "

You could use XT30 or XT60 connectors to sepparate the PSU from the Einsy. I have done it myself a few years ago on 4 Prusa's and it is working great. But I've only got the +/- cables, no power panic though.

\n" }, { "Id": "16067", "CreationDate": "2021-04-10T14:31:51.667", "Body": "

I have bought an Ender 3 Pro in November. I loved 3D printing and even upgraded my machine to improve my experience. However, lately, I can't print anything due to an issue I can't seem to resolve.

\n

I will provide as many details as I can in an orderly fashion.

\n
    \n
  1. My machine: Ender 3 Pro w/ BLTouch clone installed. Running Marlin's latest bugfix branch. Also has a Raspberry Pi with Octoprint built into the PSU so I can control both the Power and functionalities of the printer. I have a glass bed, red aluminum extruder.

    \n
  2. \n
  3. My settings:

    \n\n
  4. \n
  5. The issue: While the first layer is printing, the filament bunches up around the nozzle and if it somehow touches the bed (like I reduce the Probe-Z offset) it doesn't stick and drags on a few mm. It eventually sticks and keeps printing fine for long straight lines but it's impossible to print a little circle inside, for example. The small details aren't printed on the bed, the filament just bunches up around the nozzle.

    \n

    Also when I do a cold pull(heat up to 200\u00a0\u00b0C then pull at 90\u00a0\u00b0C, the tip of the filament looks like this:

    \n

    \"Image

    \n
  6. \n
  7. I have tried the following:

    \n\n
  8. \n
\n

My theory is that the heat traveling up the cold block thins out the filament, causing under extrusion. The filament can't properly stick to the bed because it's too thin. But the hot end fan works. Should I still replace it?

\n

Or is my problem something completely different?

\n", "Title": "Can't print anything! Is this heat creep? (Detailed explanation & Photo)", "Tags": "|creality-ender-3|adhesion|underextrusion|", "Answer": "

This sounds like nothing but a bed leveling (distance from nozzle to bed) problem, though you may have introduced other problems disassembling the hotend. It's normal to have material oozing and bunching up before the print starts; this is why you start printing with a priming line or skirt.

\n

Clean the bed well with isopropyl alcohol, level it (paper method at Z=0 or feeler gauges at Z=0.1, I prefer the latter), then fine tune with a leveling test print. If you're still having problems make sure the PTFE tube is tensioned against the nozzle right inside the hotend. Having any gap will make for all sorts of problems.

\n" }, { "Id": "16077", "CreationDate": "2021-04-11T20:56:16.700", "Body": "

My printer uses an ATMega 2560 with a RAMPS 1.4 shield, A4988 stepper drivers and a 2004 controller LCD interface. The PSU is 360\u00a0W (12\u00a0V, 30\u00a0A).

\n

I bought a BLTouch for my printer, but when installing, Marlin is restarting and is unable to complete the boot.

\n

I realized that this problem happens when it is being powered only by the PSU. When it is started by USB, it works normally.

\n

Note, the USB + PSU combination, when the system (Marlin) has been started by USB, also works normally.

\n

I tried to configure the servo in other positions (0, 1, 2 or 3), but the results were the same:

\n\n

Another possibility that I tested, was to change the BLTouch for a SG90 micro servo, to check if it was not a problem in the equipment. The result was the same:

\n\n

In fact there seems to be some problem between ATMega 2560 and the RAMPS 1.4, when powered by the PSU.

\n

Does anyone know how to solve the problem?\nIn the last case, would the exchange of ATMega 2560 + RAMPS 1.4 be the solution?

\n", "Title": "Marlin is restarting when pins 5 Vcc are enabled and has a servo motor connected (RAMPS 1.4)", "Tags": "|marlin|bed-leveling|ramps-1.4|bltouch|", "Answer": "

PSU is only feeding 12\u00a0V into RAMPS, but (if I remember correctly) RAMPS is using Arduino's onboard regulator for converting 12\u00a0V to 5\u00a0V. That regulator can not provide much power.

\n

If you connect some significant load to any 5\u00a0V pin (like servo, LCD backlight, or BLTouch), the regulator will be overloaded and its output voltage will drop (too low or unstable voltage will prevent Arduino from running correctly).

\n

You can prove this hypothesis by measuring the voltage on any 5\u00a0V pin when Arduino is in "infinite restart". It will likely be far below 5\u00a0V.

\n

Connecting USB helps because it provides additional power for 5\u00a0V rail (but you may be overloading the computer's USB port by doing this).

\n

The solution is to get an external 12\u00a0V to 5\u00a0V regulator (with enough power, something like 3\u00a0A should be ok) and connect it between PSU and some 5\u00a0V pin on RAMPS. (Or get PSU that has both 12\u00a0V and 5\u00a0V output)

\n

If you get an external 5\u00a0V supply, it may be a good idea to then completely disconnect RAMPS from Arduino's 5\u00a0V regulator. Have a look at https://reprap.org/forum/read.php?219,799595

\n

Also, the regulator on the Arduino board will likely be overheating and may get damaged (But regulators usually have some overcurrent protection so it probably will be OK)

\n

The regulator is located somewhere near the power connector on the Arduino Mega board, Google "AMS1117" if you don't know how it looks. Replacement is possible with intermediate soldering skills. The regulator is only used when powering Arduino from power connector or Vin pin (RAMPS uses the Vin pin). If powered from USB or 5\u00a0V pin, it can run without it.

\n" }, { "Id": "16103", "CreationDate": "2021-04-16T02:32:46.070", "Body": "

I recently bought a Titan Aero hot end which came with a 24\u00a0V 30\u00a0W heater cartridge from E3D. I'd like to use this but the cable length is only 1 meter long and I need it to be 2 meters. The ends terminate with prong connections and there is no polarity to the prongs. How can I safely extend the leads one meter and then connect to my Duet 3 Mainboard 6HC?

\n

Should the cable terminate with a JST connector instead of the prongs to connect to the Duet board?

\n

\"heater\n\"duet\"

\n", "Title": "Heater cartridge extension wiring", "Tags": "|hotend|wiring|", "Answer": "

1 meter puts you far enough away from the heater than you don't need high temperature wiring to extend it. The larger the guage(e.g. 20 guage) the less resistance you will add to the heater circuit. This doesn't matter as long as you can still achieve your maximum temperature (if you can still achieve the same current without maxing out your voltage on the heater).

\n

Your sensor is where added series resistance is critical. Series resistance to the thermistor will give a negative error in the temperature.

\n" }, { "Id": "16109", "CreationDate": "2021-04-16T20:02:02.580", "Body": "

I'm new to 3D printing, I just bought an Ender 3 V2 and I am having a lot of trouble with bed adhesion. After a lot of playing around it seems that the base value for Z is changing for every print.

\n

I have been using this Z-offset test model, dialing in my Z-offset during the print until I hit an acceptable value (eg. -0.15)

\n

If I then print the same model again, using the value I found above, it is way off again, and I have to dial it further down to maybe -0.40 and, if I repeat again, to 80.

\n

I have, of course, tried leveling my bed multiple times

\n

I've just been scribbling down numbers and seeing them fail for two days now, so any help is much appreciated.

\n", "Title": "Ender 3 V2 Z-axis base value changes for each print", "Tags": "|creality-ender-3|bed-leveling|z-axis|", "Answer": "

After tightening the screws on the X-axis gantry and doing the bed leveling and test prints again it seems to have gotten better.

\n

Also, while I was under the impression that the V2's heated glass bed should work well without adhesive, I applied a bit of glue stick and now my first print seems to have a perfect first layer.

\n

I may just have been confused because I needed to turn the Z-offset way down to get it to stick in the first place, and then tweaked it upwards to get nice lines, meaning I would have to dial it back down on next test print and so forth.

\n" }, { "Id": "16113", "CreationDate": "2021-04-17T12:11:52.057", "Body": "

My custom 3D printer prints everything inverted. I guess this is a homing problem as the motor moves in correct direction.

\n

In Pronterface,

\n\n

on RAMPS 1.4:

\n\n

(Pin 2, 4 & 6 are not used (are these for MAX_ENDSTOP ?))

\n

Below is my Marlin config

\n
#define X_MIN_ENDSTOP_INVERTING true // Set to true to invert the logic of the endstop.\n#define Y_MIN_ENDSTOP_INVERTING true // Set to true to invert the logic of the endstop.\n#define Z_MIN_ENDSTOP_INVERTING true // Set to true to invert the logic of the endstop.\n#define X_MAX_ENDSTOP_INVERTING false // Set to true to invert the logic of the endstop.\n#define Y_MAX_ENDSTOP_INVERTING false // Set to true to invert the logic of the endstop.\n#define Z_MAX_ENDSTOP_INVERTING false // Set to true to invert the logic of the endstop.\n#define Z_MIN_PROBE_ENDSTOP_INVERTING false // Set to true to invert the logic of the probe.\n\n#define X_HOME_DIR -1\n#define Y_HOME_DIR -1\n#define Z_HOME_DIR -1\n\n#define INVERT_X_DIR false\n#define INVERT_Y_DIR false\n#define INVERT_Z_DIR false\n
\n

I have attached 3 photographs.

\n
    \n
  1. Shows the Home position of hotend. Y Motor on back and Y endstop at front.
  2. \n
\n

\"Home

\n
    \n
  1. Shows inverted print.
  2. \n
\n

\"inverted

\n
    \n
  1. Pronterface screenshot (shows actual G-code file)
  2. \n
\n

\"Pronterface

\n

I tried flipping the motor cables, but that inverts the motor direction\nI also tried INVERT_Y_DIR true, but no luck.

\n

Please help me. What am I doing wrong?

\n", "Title": "Prints are mirrored in X-axis and inverted in Y-axis direction", "Tags": "|marlin|diy-3d-printer|homing|", "Answer": "

Instead of changing firmware or changing the wiring, I flipped the motor direction. See the first photo below. Motor shaft was on the right side, now, the motor shaft is on left side, so the the bed moves backwards. I moved the Y endstop to back and now it prints fine.

\n

only the problem is

\n

In Pronterface,

\n
if I press -Y -- bed moves backward (towards the Y endstop - new position)\nif I press +Y -- bed moves forward (away from Y endstop - new position)\n\n--- is this normal ?\n
\n

\"enter

\n" }, { "Id": "16115", "CreationDate": "2021-04-17T14:32:54.670", "Body": "

There is a material called PETG. In addition, there is a medical solution called Clear Aligner to fix perplexed teeth.

\n

Can I use a 3D printer using PETG and make transparent apparatus?

\n

The program I use gives transparent apparatus, 3-dimensional output in STL format. Do I get results if I send the STL file to the printer?

\n", "Title": "PETG and Clear Aligner?", "Tags": "|petg|medical|", "Answer": "

No

\n

PETG is a material that can only be used in FDM machines. This precludes any internal medical use, as FDM prints can't be made sterile or even hygienic - the sterilization processes would destroy the print, and FDM layers create many spots for germs to grow. Atop that, you can't get the needed precision with FDM - which means any FDM prints are at best waste at worst endangering the patient!

\n

In contrast, 3D printed metals and laser-printed nylon can be created and post-processed in ways that do not have layer lines. Resin prints do not have layer lines that offer these discrete hiding spots for germs. They all are made in a fashion that kills germs or disallows them from being embedded and they can be properly sterilized or made hygienic.

\n

No!

\n

PETG can't be printed completely clear in FDM. By the processing method, air and layer boundaries are included, making prints at least somewhat opaque.

\n

Only Resin prints can be fully transparent as they contain neither boundaries nor air.

\n

NO!!!

\n

STL files contain only surface data and need to be processed into printer readable format. G-code is one of those formats, commonly used in FDM machines and CNC machines.

\n

The processing from STL or other surface data into FDM-3D-printer G-code is done by a program called a slicer. Examples are Cura, PrusaSlicer, Slic3r, and many others.

\n

Resin printers using DLP use different slicers that create images of the model's crossections together with some sort of machine code for the movement. For SLA machines, the code is entirely different again.

\n

Further Reading

\n

I highly suggest you read more about 3D printing dental applications before trying anything for dental use, and especially anything that is printed for any medical application!

\n" }, { "Id": "16124", "CreationDate": "2021-04-18T07:12:53.433", "Body": "

I am new to 3D Printing and I have taken up a summer project. The project requires a fan 3D Printed, If I use a normal fan the airflow will make my project (i.e. a air-propelled car) go backward. Can we modify a fan so that when the fans spin my car goes forward? If yes, how?

\n", "Title": "How can we control the airflow in a fan which can be 3D printed?", "Tags": "|3d-design|", "Answer": "

Look at your fan: it spins in one direction, and the blades push the air from one air to the other side. In fact, the fan is nothing but a propeller! There are ways to optimize them. The Rotation goes for example clockwise, the leading edge grabs the air and the trailing edge releases it on the other side, and if the leading edge is towards you, then the air is pushed away. In the simplest way, it would suffice to physically flip the fan around.

\n

A 3D printed new fan would need to have the whole geometry of the fan blades "flipped" (mirrored around a non-rotational axis). Now the leading edge of the fan is away from you, the trailing edge on your side, the airstream is again from the leading to the trailing edge, and the air comes out on your side.

\n

cave!

\n

Not all fans can be made to reverse the airflow by flipping the fan geometry: Radial fans just because less efficient.

\n

Other fans can be made spinning backwards by swapping the polarity, but others use a simple diode setup and always spin their way. Also note that fans spinning backwards often suffer from reduced efficiency, as the blade geometry is optimized for one direction of spinning.

\n" }, { "Id": "16130", "CreationDate": "2021-04-19T11:44:08.293", "Body": "

Let's say I've made a 3D scan of my face and managed to get that into FreeCAD. How might I then create an object (it's a mask, okay?) that conforms to the shape of my face, with a given thickness, such that I can export and 3D-print that part only?

\n

So if I printed it, it would fit over my face, but still look like my face from the outside, too. It would be, say, 3mm thickness througout.

\n

To be clear, I'm not looking to make a 3D model of my head (the world does not need such a thing), or 3D model of a mold that I can use to replicate my head. I just want to make parts that conform to the shape of other, complex, parts.

\n", "Title": "Creating a 3D modelled \"mold over\" an imported object", "Tags": "|3d-models|blender|freecad|", "Answer": "

Instead of using FreeCAD, I would import the mesh data into a vertex-based 3D modeling software, such as blender.

\n

After stitching the model in any areas where you still have gaps, you remove any vertex belonging to areas you do not want to use for the mask. This leaves you with pretty much a skin-tight base for your mask. Select all the remaining surfaces and scale S them in all directions in regards to the origin so you create a little airgap 5% extra (1.05) should create enough space to sit somewhat comfortably, but you might prefer a little more. As a stand-in for the face, I use this approximation of a forehead...

\n

\"enter

\n

Next, we thicken that surface using the modifier menu: Select the Solidify modifier and set a thickness that looks good to you and that might be printable. Then make sure it pushes away from where your face is. Also make sure the Offset is at 1 or -1 to ensure that it does not stick out from the face-scan surface to the back!\n\"enter

\n

Now you got a perfect base to model on the outside of using the sculpt features while retaining the inner surface! Do yourself a favor though: print at least part of this model as a test fit and adjust the mask as needed.

\n" }, { "Id": "16137", "CreationDate": "2021-04-20T01:33:35.120", "Body": "

I just replaced the board on my Creality CR-10 with a 32-bit SKR Mini E3 V2.0 control board that runs Marlin 2.0. With the new control board, the printer's Y-Axis moves vibrate so much that it blurs the camera I am using that is connected to the bed.

\n

I have checked the mechanical stuff and it seems like the issue is the stepper motors are running choppily instead of smoothly.

\n

I know that 32-bit Marlin has lots of things I can tweak. Are there settings I should be checking? Where do I start?

\n", "Title": "Printer shaking - Marlin 2.0", "Tags": "|marlin|stepper-driver|", "Answer": "

The problem turned out to be mechanical. The arm that I mounted the camera on had developed a crack where it connects to the build platform. I used some superglue to repair the crack and the camera shaking went away.

\n

The lesson here is to check EVERYTHING mechanical before trying to blame shaking on the firmware.

\n" }, { "Id": "16163", "CreationDate": "2021-04-23T01:59:38.850", "Body": "

I was wondering the other day why don't all three motors move at the same time? Don't normal paper printers move 2 motors at a time? they're 2D printers. It makes sense if a 3D printer really does print with all three motors moving. Won't it also be more efficient if they do 3D print in all axes?

\n", "Title": "Is 3D printing really just 2D printing? Because only 2 motors move per layer", "Tags": "|3d-design|mechanics|", "Answer": "
\n

I was wondering the other day why don't all three motors move at the same time?

\n
\n

That is perfectly possible for most printers (with limitations, "3D Printing" with all steppers being used is called "non-planar" printing), but there are some major cons you need to deal with. First, there are not that many software suites that slice objects perfectly well like the "normal" per "fixed/variable layer height" slicing as the slicers are under developed. Second, it requires a specific tall and slender nozzle/print head, else the height of printing is very limited. Last, besides generally not being useful for rectangular prints (except for 90\u00b0 overhanging structures1)), this method is only/best suitable for curved objects as the top layer will follow the contours of the object.

\n
\n

It makes sense if a 3D printer really does print with all three motors moving.

\n
\n

No, it does not make sense, the essentials of 3D printing it that it produces a 3D object, not that all stepper motors should be running simultaneously.

\n
\n

Won't it also be more efficient if they do 3D print in all axes?

\n
\n

Not necessarily, there are limitations to non-planar printing like geometry and print quality, but, you could (in some cases) print with less support material.

\n
\n

1) : From Rene K. Mueller, published March 3, 2021, https://xyzdims.com:\n\"Non-planar\n\"Non-planar

\n

Note that these are demonstration pieces, normally you would rotate the print for 90\u00b0 and print it sliced normal (planar).

\n" }, { "Id": "16168", "CreationDate": "2021-04-23T07:34:29.187", "Body": "

I am creating a bottlecap-like design. The design is sealed by rotating the lid. I am adding a chemical to it for an experiment. I would like the inside of the design to have no air circulating to it through the lid tiny spaces.

\n

My design:

\n

\"enter

\n

Are there any simple solutions I can implement out there? Design-wise, material-wise, or maybe an extra piece?

\n", "Title": "How to create an air-free design?", "Tags": "|3d-design|", "Answer": "
\n

Are there any simple solutions I can implement out there?

\n
\n

A gasket made of rubber or other elastic or deformable material is probably the best option. Printing one or both of the parts using a deformable material like TPU might also work. Finally, if you don't need to open the unit during the experiment, you could use a sealant like silicone caulk might make sense.

\n" }, { "Id": "16171", "CreationDate": "2021-04-23T11:28:07.917", "Body": "

We're thinking of buying a PETG-powered printer. When researching printers available on the market, there are machines with a 100 microns. I was wondering if there are machines with a 50 microns or close to 50 micros? Otherwise, why not?

\n", "Title": "Printer with 50 microns or close to 50 micron (z resolution)", "Tags": "|desktop-printer|resolution|", "Answer": "

PETG is a material type that is only available for Fused Deposition Modeling (FDM) type printers. Those are limited in their achievable resolution by their nozzle size:

\n\n

The smallest available nozzles that doesn't require specialty extruders are 0.2\u00a0mm. That means the absolute minimum detail that could in theory be reproduced on a surface of enough thickness is a 0.1\u00a0mm depression using a design similar to this picture:

\n

\"enter

\n

As a result: NO FDM with PETG is not a solution to your requirements, you want reliable 0.05\u00a0mm resolution in XYZ, which is the area of Resin printers, especially DLP and SLA, but also Stratasys PolyJet can achieve this at the moment.

\n

Estimating from your other questions (like this), I still suggest looking at a resin-based system that has materials that are certified for medical use!

\n" }, { "Id": "16180", "CreationDate": "2021-04-24T23:16:45.253", "Body": "

Does anybody if the Anycube Mega X comes with a nozzle for 1.75 mm filaments or is it 2.85 mm?\nI saw online that it works with 1.75 mm filaments but the Cura settings given by the manufacturer had 2.85 mm.

\n

I should mention that using a 1.75 mm filament works BUT my designs have clear under-extrusion, which is very likely caused by having 2.85 mm in the settings. So at this moment, I am trying to gauge whether to change the settings to 1.75 mm or buying 2.85 mm filaments (this only works if the Mega X comes with the appropriate nozzle).

\n", "Title": "Adequate default filament diameter for Anycubic Mega X", "Tags": "|filament|nozzle|filament-choice|", "Answer": "

This is a well-known problem with Cura. The filament diameter is set to 2.85\u00a0mm by default (probably because Ultimaker printers use 2.85\u00a0mm filament). This will cause extreme under-extrusion if your printer uses 1.75\u00a0mm filament. Simply change the filament diameter to 1.75\u00a0mm in Cura's printer definition, and everything should be fine.

\n

Older versions of Cura would reset the filament diameter to 2.85\u00a0mm if you installed an update or new release, but I think the Ultimaker have fixed that annoying little bug.

\n" }, { "Id": "16183", "CreationDate": "2021-04-25T02:05:56.067", "Body": "

There are a few hits on my issue title but could not find an actual answer/advice.\nI burned my Ender 3 Pro Creality motherboard and replaced it with an SKR 1.4 + TMC 2209 + BLTouch.\nEverything is moving like it should and calibrated.

\n

However, at the end of the homing process, the Z-axis goes up, then it's supposed to move to the printing position.\nAt that point, the acceleration is so great that the Z-axis barely goes down and I end up printing about 4 cm above the bed.

\n

These are the settings I changed on my Marlin 2.0.7 firmware in relation to the Ender 3 setup:

\n
Configuration.h\nset #define STRING_CONFIG_H_AUTHOR "Nicolas Rietsch v 0.1.2"\nenable #define CUSTOM_MACHINE_NAME\nset #define CUSTOM_MACHINE_NAME "Ender-3 PRO"\nset #define TEMP_SENSOR_BED 1\nset #define DEFAULT_MAX_FEEDRATE          { 500, 500, 5, 45 }\nset #define DEFAULT_MAX_ACCELERATION      { 450, 450, 100, 10000 }\nset #default DEFAULT_ACCELERATION          450\nset #define DEFAULT_TRAVEL_ACCELERATION   500 \n\nset #define INVERT_X_DIR true\nset #define INVERT_E0_DIR true\nset #define X_MIN_POS -26\nset #define X_BED_SIZE 230\nset #define Y_BED_SIZE 230\nset #define Z_MAX_POS 250\nenable #define NOZZLE_PARK_FEATURE\n
\n
Configuration_adv.h\nset #define X_CURRENT       500\nset #define Y_CURRENT       500\nset #define Z_CURRENT       500\nset #define E0_CURRENT      650\nset #define CHOPPER_TIMING CHOPPER_DEFAULT_24V\n
\n

What am I missing?

\n", "Title": "First move after homing way too fast", "Tags": "|marlin|creality-ender-3|", "Answer": "

Maybe it would it help if you fix #default to #define in the current line:

\n
set #default DEFAULT_ACCELERATION ...\n
\n" }, { "Id": "16195", "CreationDate": "2021-04-26T14:33:23.217", "Body": "

I'm having some problems with my BLTouch.

\n

\"enter

\n

Yesterday I got the 4.2.7 silent board delivered as well as the BLTouch and a glass bed. I was able to install the silent board without a problem and according to the many, many diagrams I've looked at, I also have my BLTouch installed correctly. I've installed several different firmware versions as well as manually compiling some.

\n

I used the following diagram which was provided in a link by the seller. I also tried the firmware for Ender 3 pro 4.2.7 board on the Creality website as well as a few other firmware versions from various YouTube videos.

\n

\"enter

\n

When I start up my printer the BLTouch does its normal startup thing and my printer says it's ready. But when I try to auto home, bed level, or even manually control the BLTouch it doesn't extend at all. I've spent about 4 hours trying to figure it out on my own and I'm throwing in the towel.

\n", "Title": "Problem with BLTouch on Ender 3 Pro", "Tags": "|creality-ender-3|firmware|bltouch|", "Answer": "

For anyone who has this problem in the future my solution was to loosen the screw on top of the touch just a little bit

\n" }, { "Id": "16229", "CreationDate": "2021-05-01T23:37:11.777", "Body": "

When I first got my 3D printer (a FlashForge Adventurer 3), it came with a sample pack of filament. With this filament, I was able to use skirts for my first layer. When the sample filament ran out, I switched to Hatchbox PLA filament. For some reason, I cannot use skirts with the Hatchbox filament. Now, whenever I try to print something with a skirt, the print moves around, ruining it. The only first-layer that works now are rafts, which I do not like, as they use up more filament and are more of a pain to remove. Is anyone else having this problem? If so, what are some workarounds to this issue?\"Here

\n
\n

Here are the failed prints. I terminated them mid-way, as they started\nto shift on the build plate.

\n
\n", "Title": "My Hatchbox PLA filament only allows me to use rafts", "Tags": "|pla|filament|adhesion|filament-quality|rafts|", "Answer": "

Please try one (or more) of the following:

\n
    \n
  1. Change the temperature of your heated bed (50 - 60\u00a0\u00b0C)

    \n
  2. \n
  3. Check your nozzle height (0.15 - 0.25\u00a0mm is what I use)

    \n
  4. \n
  5. Make sure your bed is level (Maybe use a leaving stick (or whatever those bubble things are called))

    \n
  6. \n
  7. Use adhesion. Start with painter's tape (least damaging, easy to remove), then use a glue stick or even hair spray if necessary.

    \n
  8. \n
  9. Be sure the bed is clean (I recommend glass cleaner 1-3 squirts and then a light scrub of a paper towel)

    \n
  10. \n
\n" }, { "Id": "16246", "CreationDate": "2021-05-05T08:11:32.893", "Body": "

If I buy a printer, it will be going on a small table on a front porch so as to keep the smell from the house. The porch has a roof, so precipitation is not a concern. I will find a way to keep the wind out, so that will also not be a factor. The only concern I can see is the lack of climate control. The printer will probably be an Ender 3 Pro. Will this turn out bad for the prints? Do I need to be concerned about dew? Any other concerns? Solutions?

\n", "Title": "Will there be any negative effects on the print process if the printer is outside?", "Tags": "|creality-ender-3|print-quality|outdoors|", "Answer": "

Unless you enclose the printer fully in some way or another, I see problems, even beyond just print quality:

\n\n" }, { "Id": "16248", "CreationDate": "2021-05-05T15:30:59.503", "Body": "

The Vault (2021) movie has a scene at around 42' in which a laser scanner with a rotating table is used to scan a key in a short time, maybe some minutes:

\n

\"Scene\"

\n

No calibration is done whatsoever. The scan quality must be high enough for the replicated key to be useful.

\n

Is it technically possible? Is it exagerated? Is it fiction?

\n", "Title": "Laser 3D scanner is used in the movie to scan a key: technically possible?", "Tags": "|scanning|", "Answer": "

There have been instances where people have replicated a key from a photograph of a key. A multi-minute scan of a key seems like overkill, but that appears to be a very large key so maybe so.

\n

Is it possible? Yes.

\n

Is it exaggerated? Probably much simpler than is portrayed.

\n

Is it fiction? As portrayed, yes. Practically, no.

\n" }, { "Id": "16249", "CreationDate": "2021-05-05T18:22:00.617", "Body": "

I'm printing a Tower of Pi on my Ender 3, and I woke up to find this near the extruder:

\n

\"Powder

\n

All my attempts to google this led to checking for either a clogged nozzle or the extruder itself stripping the filament, neither of which appears to be the case.

\n

It looks like the upper edge of the filament might be grinding against the intake hole, so I looked into lubrication, but the consensus appears to be "don't".

\n

Has anyone else had this problem? Is there a name for it, and how do I fix it?

\n", "Title": "Excessive filament powder/shavings near the extruder", "Tags": "|creality-ender-3|filament|troubleshooting|", "Answer": "

The shavings come in part from the extruder design: there is no fillet on the pulling in side, the filament goes up in a sharp angle and is dragged over a rather sharp edge. It also brushes against the leadscrew.

\n

To help with the shavings, you should alter the filament path to try and have a flatter angle than the 90\u00b0. A simple rod that pushes out the filament to come in at 80\u00b0 might already reduce the number of shavings.

\n

More might be reduced by opening the entry hole of the filament to have a 1-2 mm phase around it, altering the angle of the edge that the filament runs against to a much lower one and reducing the ability to shave off flakes. I did swap my extruder for an aluminium one, and it has this phase, and since then I have had little to no shavings at that spot anymore.

\n" }, { "Id": "16269", "CreationDate": "2021-05-10T05:01:07.673", "Body": "

Because of the crazy shipping prices, I want to find a cheaper alternative to ordering a 600$ all-metal hot-end.

\n

Is there a material like that which does what PTFE does but till higher temperatures?

\n", "Title": "Can I use a material other than PTFE to print PETG?", "Tags": "|all-metal-hotend|", "Answer": "

PETG does not need an all-metal hotend or alternate bowden/lining material. Unless you're trying to print at #speedboatrace-competitive speeds, the recommended print temperature range for PETG is 230-250 \u00b0C, and the temperature above which you should not use a PTFE-lined hotend is 250 \u00b0C.

\n" }, { "Id": "16271", "CreationDate": "2021-05-10T15:34:12.333", "Body": "

Today I ran into an issue with the filament not sticking to the build plate. However, the strange part is, that this only happens with one filament color (both PLA, khaki, black works just fine) and only in the main print. The test line on the side of the bed and the brim all stick without a problem. I already releveled everything and as I'm using ABL and the other filament works fine, this shouldn't be the issue.

\n

I also tried increasing the first layer thickness in Cura, however, this leads to extreme warp (2\u00a0mm height on 5\u00a0mm width).

\n

From my observations, it seems that the print head is raised after the brim is printed. Is there a setting in Cura to change that?

\n

Printer Details: Modified Ender-3 with MKS Gen L V2 Board and TMC 2209 motor drivers and BLTouch
\nPrint temp: 220\u00a0\u00b0C / 70\u00a0\u00b0C initial after that 200\u00a0\u00b0C / 60\u00a0\u00b0C
\nFirmware: Marlin, details here

\n", "Title": "Print not sticking to bed", "Tags": "|marlin|pla|adhesion|", "Answer": "

Thank you for all your answers! After many failed benchies I fixed the problem now, however I don't know exactly what the problem was so I'll list everything I did:

\n\n

The high temperature is actually a recommendation by the manufacturer (PM Filaments).

\n" }, { "Id": "16283", "CreationDate": "2021-05-13T09:44:07.277", "Body": "

In Prusa i3 clones the Z axis is often problematic. Mechanically unsynchronized dual Z axises which are driven by the same stepper driver easily misalign. Using only one Z motor makes the whole gantry sag on one side. And Z lead screws that aren't perfectly straight lead to z wobble.

\n

The other day I had a simple idea that would fix all of that, but I can't find anyone who implement it before, so I probably there are drawbacks that I don't see.

\n

The idea is to remove the lead screws (possibly replacing it with a second set of smooth rods to counteract X axis vibrations) and instead hang the gantry onto two ropes/wires which are drawn by a winch. That way, both ends of the gantry could be drawn symmetricaly by a single motor. Since there is no Z lead screw, there is no wobble. And since only the gravity is pulling the gantry down, you shouldn't be able to destroy anything by ramming the nozzle into the bed.

\n

What drawbacks are there for this system? What am I missing? Is the gravity strong enough to press the nozzle into the last layer when printing? I am using a direct drive extruder.

\n
\n

Supplemental follow up question (related to, and answered by, Trish's answer):

\n
\n

From my understanding the hangprinter is only attached to ropes, same as a portal crane. I am considering an i3-style printer with the leadscrews removed, but the smooth Z rods intact (maybe even with a second set of smooth rods in place of the leadscrews. So the only forces that gravity has to work against are: output pressure of the extruder and potential bouncing in z direction. So, how could I calculate if gravity is enough for that?

\n
\n", "Title": "Z-Axis on a winch", "Tags": "|prusa-i3|z-axis|", "Answer": "

This concept already exists, it is e.g. found in some high quality 3D printers. Instead of a winch, a belt system (of several belts) is used.

\n

An interesting point from this answer already mentions that a winch isn't a good mechanism to lower/raise the gantry; if the cables are wound over each other the diameter of the wound cables will get larger and thus the movement. This could be circumvented by creating a push pull system and very few windings on the winch.

\n

But, this concept already has been worked out in the VORON 2.4 printer design. This design is based on the work of a group of engineers and is considered to be a high quality printer that is also suitable for professional use. From the following image you can see (on the right side) a belt. This design uses 4 linear rails to support and guide the bed and 4 belts to drive the whole CoreXY gantry up and down.

\n

\"enter

\n

So, a winch will not work, but the type of suspension already exists and is in use.

\n
\n

With respect to the i3 style printers and the free gantry movement the following is from my own experience.

\n

I've build several i3 style printers of which my latest 2 where all having a freely moving gantry upwards as I decoupled the lead screw from the X mounts. The lead screw nuts simply push the gantry up as they are embedded in a separate part; gravity will pull the gantry down. The mass of the guide rods, a stepper motor and the hotend provide enough mass to pull it down. I've had excellent results with these printers, e.g. mentioned in this answer:

\n
\n

poor layer bonding and possibly even collisions between the nozzle and already-printed material, from slight variations in height where extrusion pushes back against gravity

\n
\n

is not encountered. The following images give an impression of the lifting device:

\n

\"enter \"enter

\n" }, { "Id": "16284", "CreationDate": "2021-05-13T11:35:54.940", "Body": "

This problem is driving me crazy, no, seriously speaking. I bought this printer this week and started to calibrate for accurate dimensions. I printed the one-and-only calibration cube (20x20x20 also a 10x10x10) and well, the dimensions for the X & Y axis are perfect, but the Z-axis is a big problem, almost 1.5\u00a0mm bigger than expected.

\n

I tried everything in order to resolve this issue, I checked to see if the steps are correctly calibrated using a ruler and moving my extruder up and down, precise as you expected, no weird sounds coming from the Z-axis and stepper while printing. I calibrated the E-Step, also the bed leveling is done automatically using the BLTouch probe. Also I want to mention that the first layer is perfect (took time to calibrate this too).

\n

I am using Cura for slicing using the default profile for the Ender 3 Pro, Premium PLA, 210\u00a0\u00b0C for the first layer & 200\u00a0\u00b0C for the rest, 50\u00a0\u00b0C bed temperature.

\n

I will leave here some photos, perhaps somebody could give me an idea.

\n

As you can see from the images the Z-axis is scaled-up perfectly. This difference is about 1.5\u00a0mm for both cubes (the one with 20\u00a0mm & 10\u00a0mm).

\n

Has anyone encountered this type of problem before?

\n

EDIT:\nI will attach some pictures with the first layer. Right now I observe some wobbling in the Z axis and saw that the printer is not perfect laid on the table, one corner was lifted. So I tried to flatten. Waiting for the test print.

\n

\"enter

\n

\"enter

\n

\"enter

\n

\"enter

\n

\n", "Title": "Height of the prints are always larger", "Tags": "|marlin|creality-ender-3|ultimaker-cura|pla|", "Answer": "

Since nobody had any idea what cause the problem, I investigated for a full week.

\n

The problem is related to Fade Height and .... a warp bed, since I have ABL (Auto Bed Levelling) and Fade Height enable. The compensation was way too big, something like -1.4 or bigger.

\n

So, after I bought a new glass bed and set the fade height to 0 the problem is gone and the prints are looking normal, like -.2 mm or -.3mm in heights.

\n" }, { "Id": "16298", "CreationDate": "2021-05-16T07:48:06.660", "Body": "

I have a BLTouch clone (3DTouch) on my printer (Artillery Sidewinder X1). I installed it on the printer ages ago and has been working fine since. Recently, I did a BTT smart filament sensor upgrade.

\n

I updated and edited both the Marlin firmware and the MKS TFT28 screen firmware to get it to work with the smart filament sensor. The filament sensor runs through the screen not the motherboard so it is enabled in screen firmware instead.

\n

The sensor works fine and every other part of the printer is fine as well, except that after the BLTouch does a G29 (I have it do that before every print in my start G-code, followed by an M500), It will not adjust the Z-axis to compensate for the unevenness of the bed. I can see the Z motors not move and I cannot feel the lead screws rotate in my hand if I touch my hand against them.

\n

I have been through several other forums, videos, etc. on the internet but none of them have the same problem as me and/or their solutions do not fix the problems.

\n

I have also tried flashing the display with the original firmware, which did not work so I guess is that I have something wrong with the firmware.\nI have checked and double-checked my firmware but maybe there is something I missed or do not know about that could be causing my problem.

\n

Firmware can be found here.

\n
\n

I have tried running the original firmware with changes only made to enable the BLTouch. The Z-axis does still not compensate.

\n
\n

I have also replaced the mainboard. I don\u2019t know if that could affect the operation or not.

\n
\n

Recently, I needed to print a part so I put the default Marlin firmware using the Artillery SWX1 example in Marlin's collection of examples with modifications made only to work with the BLTouch. I can post this copy of the firmware but it is probably identical to the GitHub repo below. This way I could just print with no Z compensation, however, now whenever I try to home the printer or do a G29, the BLTouch doesn't deploy and it will crash into the bed. I posted an unlisted video on YouTube demonstrating the issue here. Also note that in the video, I attempt to move the Z-axis by 1\u00a0mm, but instead, the stepper motors spin at full speed and the gantry comes zooming towards the bed.

\n
\n

I tried flashing the firmware yet again and for some reason, the BLTouch worked. it could home is and to a probing routine. I have been using the printer without the compensation and it works fine but it is really bizarre that this is happening. I know it is unlikely to solve the problem, but I am thinking of purchasing a replacement BLTouch. They are only 20 bucks to replace and I did accidental slightly bend the plastic probe before the Z stopped compensating. So is it possible the BLTouch is cactus?

\n
\n

So recently, I noticed when running a G29 T through Octoprint or Pronterface, etc. that the printer will return a set of numbers for each probing point. Does this indicate that the BLTouch has taken the measurements but the printer will not use them to compensate the bed?

\n

I have tried all the answers below.

\n", "Title": "BLTouch type sensor Z not compensating", "Tags": "|z-axis|bltouch|z-probe|3dtouch|artillery-sidewinder-x1|", "Answer": "

Make sure RESTORE_LEVELING_AFTER_G28 is uncommented in configuration.h.

\n" }, { "Id": "16314", "CreationDate": "2021-05-18T20:07:42.980", "Body": "

So I wanted to print a 3D parabola, with a radius of 15\u00a0cm and a height of 4\u00a0cm. I've told this company, they told me that they need 3.5 days and its price will be 147 USA dollars and 50 cents.

\n

I just want to know what is the average price for this?

\n", "Title": "What is average price for 3D parabola?", "Tags": "|diy-3d-printer|", "Answer": "

The price dependss roughly on material, machine hours, operator labour, profit and administrative overhead.\nSome companies deduce the operator labour, machine hours and overhead to roughly 10 times the material cost. I think that is kind of fair. In your case I assume that you use PLA, the perabola is hollow (just a flat surface) and the company needs to construct the 3D model the parabola to a specific tolerance.\nthis model in PLA should not cost more than \\$5 in filament, so about $ 50 should be an okay price for your parabola. The total price of \\$147 leaves about \\$97 for modeling the parabola. I think that's fair, given that a non-mathician has to find a way to construct a model and test the results.\nIf, for example, you want a metal print and you provide the 3D model I find the price of \\$147 to be very normal. These metal printers are very expensive, labour intensive and eat up energy.\nYou can get instant quotes at the following sites: https://xometry.eu/, https://formalize-am.com/\nAll above is just my best guess, given the data you provided. Expect a better answer when you provide more data.

\n" }, { "Id": "16338", "CreationDate": "2021-05-21T20:58:18.823", "Body": "

SUMMARY: I use a Creality Ender 3 and Cura 4.8 slicer. I need to print multiple objects 'one-at-a-time' (Cura allows this, that is fine) and I must specify the order in which each model is printed but Cura does not have this facility. My question is: how can I change the order of the printed objects by editing the G-code?

\n

DETAIL: Sometimes it is essential or useful to specify the order of printing multiple objects in a 'one-at-a-time' batch print to allow the printing of some objects that are taller than the X-gantry clearance together with other objects that are shorter than the gantry because if you can control the order then you can print the small objects first and then the tall objects (because the gantry won't get in the way of the already-printed small objects but if the tall objects are printed first then the gantry will crash into them). Specifically I am talking about two or more objects printed at roughly the same Y-level on the print bed (I know you can 'get-away' with it if you separate them in Y but I want to print multiple rows of such objects). Without control over print order this cannot be done thereby limiting productivity (number of models per print run).

\n

WHAT HAVE I DONE TO RESOLVE THIS ALREADY: I can control the print order of multiple identical objects in Cura (they are printed in reverse order to their creation as duplicates of an initial model and if there are problems you can sometimes control the order by moving objects around) but this does not work for multiple different objects. I did several Google searches and found some conversations that deal with this issue including on the official GitHub page of the Cura slicer (see for example this Cura GitHub Discussion). The conclusion given by the Cura developers is that they understand the limitation, but it is not an issue for Ultimaker printers so they have no plans to make user-customisable print order a feature in Cura because it would be too much work for them to do the changes and maintain them. One person mentioned a python script to allow post editing of G-code to change print order but I could find no links to it. Another person described a 'workaround' in Cura to do with grouping and ungrouping models in a specific sequence but this does not work for me (?maybe it only worked in older versions of Cura). Perhaps someone on this forum has G-coding knowledge and could shed some light on how it may be done? [PS: I am only interested in G-code editing solutions for this question, or a Cura workaround that works in v4.8 - I don't use Octoprint/Rasperry Pi controllers].

\n

Thanks.

\n

[EDIT] Thanks to the contributors in the accepted answer I managed to do this with 'cut-and-paste' (via a small C program - you could do it manually but it is tedious and there is more risk of making a mistake if you have many models). I used the comments in Cura's G-code and a few other G-code cues to calculate the correct splice points.

\n

It works provided you take care to ensure the new extruder code is either altered according to the new model positions or use the 'relative extrusion' mode when saving the original G-code from Cura (I used the latter method). Also you must splice the G-code at the right points to ensure pre-model start and post-model finish G-code commands are properly carried around and the final model in the original G-code needs a terminal G0 command to be constructed if it is no longer terminal in the new order so that it can properly position the nozzle to go to the next model in the chain. Also you need to ensure M107 is included prior to each model's first layer. There is danger in doing any of this - the new G-code might damage your printer if you get anything wrong - but it can work (at least it has for me).

\n

I made several manual checks on the new re-arranged G-code as well as loading it into Cura and running Cura's nozzle simulator on it to ensure it does what I expected of it prior to risking running it in my printer, but after that it printed OK.

\n", "Title": "Edit G-code to change print order of multiple one-at-a-time print jobs", "Tags": "|creality-ender-3|ultimaker-cura|g-code|", "Answer": "

You can open the G-code file in any text editor and copy-paste the relevant sections of G-code to reorder them. It will be fairly obvious where one object ends and the next one starts because the printhead will move back down to Z=0; you can do a simple search to find the commands that move to Z=0 (in my version of Cura, it inserts a comment ;LAYER:0 at the start of each object).

\n

To ensure that the extruder starts in the correct position, look back at the previous print (in the original order) to see where the extruder finished, then insert (at the start of the new print) G92 Exx.xx where xx.xx is the previous position of the extruder. If looking back to the previous print is too tedious, you can also just use the very first extruder position in the print itself and use that. It will just skip printing the very first section of brim/skirt, which doesn't make a big difference.

\n" }, { "Id": "16341", "CreationDate": "2021-05-22T18:33:38.213", "Body": "

I\u2019m just getting my feet wet with 3D printing, and I\u2019ve had a few prints do this. Curious if anyone has any ideas for me that might help clean this up? (I don\u2019t really have anyone in my personal circle who can help out, so hoping this community can :) )

\n

Edit 5/23\nThe first layer of the print is inconsistent in the way it\u2019s deposited. Some areas feel correct while others are very thin. Usually there are large gaps like in the pictures below. After the first layer, the rest of the print seems to be fine. I\u2019m curious is anyone has any insight into what could be causing the inconsistency.

\n

I try to level the bed before each print, but I\u2019m not sure if it\u2019s at the correct height as the instructions I got with the printer are pretty subjective: \u201cuse a piece of A4 paper and you should be able to pull it with some resistance\u201d.

\n

\"enter

\n

\"enter

\n", "Title": "Help With PLA settings", "Tags": "|pla|filament|extruder|troubleshooting|", "Answer": "

It looks like a bed adhesion problem to me. Some additive information: What is your bed material?

\n

Z-calibration problem isn't the only suspect. I've seen the same problem with my printer before. There were defects in some parts of my glass print bed that prevented it from sticking.

\n

The molten filament from the hot nozzle has two adhesion chances; full adhesion to the bed or adhesion next to the previous extruded filament. If the previous filament is not stuck to the bed, it means you are printing on the air shown in the pictures.

\n

Without forgetting, your images also seem to have a Z-calibration problem, but as I said, I don't think it's the only suspect for blobs. It will be useful to clean the surface.

\n" }, { "Id": "16345", "CreationDate": "2021-05-22T22:53:43.177", "Body": "

I've had my Ender 3 for almost three weeks, gone through about a kilogram of PLA filament (printing a mix of upgrade parts for the Ender 3 and stuff I actually want to use) and made one PETG object, generally with good results.

\n

I've noticed however, as seems relatively common (per YouTubers, anyway), that my bed isn't flat -- that is, the build surface isn't a good approximation of a geometric plane. If I adjust the bed to have correct clearance (good adhesion and correct single-line width) at the corners, I'll get adhesion failure in the center, and if I adjust to give a correct center, the extruded filament will be squished into the build surface texture; the nozzle may even lightly scatch the surface at the corners. That indicates the corners are high, relative to the center, by roundly 0.1 to 0.15 mm.

\n

I'm aware of BLTouch and its clones, but in order to get full use of that system (which automatically compensates for the non-planar bed) I would need to not only install the surface sensing hardware, but flash my printer's firmware (potentially after removing the control module cover and plugging a cable and adapter into the mainboard). As a longtime builder/upgrader of my own computers, this is certainly within my capability, but I'd prefer to make my build surface flat instead of applying software corrections; I see this as upgrading from a 386 to a Core i3 because the computer is overheating -- that is, the problem will go away because of all the other stuff you have to do, but you haven't really solved the problem.

\n

My general idea more or less mimics the self-answer on this question in terms of measuring the excursion and applying shims under the build surface (I've installed the Creality magnetic sheet surface, so shims would be applied between the magnetic base sheet and the removable build surface). I plan to use household aluminum foil, standard weight, which is generally close to 0.63 mil (= .016 mm), applied with repositionable spray adhesive and laid down in layers, using a combination of feeler gages and single-layer test prints to determine where and how much foil to apply.

\n

I've "test flown" this option by putting a single Post-It sheet under the center of the removable build surface, and now I have a much closer match between the center and corners, and can (depending on my nozzle standoff) actually see the outline of the makeshift shim in the first layer where it prints over the edges of the Post-It.

\n

Is there anything I'm missing that would prevent this shimming method using aluminum foil from resolving the warped bed to allow me to depend on an even thickness and correctly adhered first layer?

\n", "Title": "Does shimming my build surface compete with spending a bunch more money and time on an auto-leveling system?", "Tags": "|creality-ender-3|bed-leveling|", "Answer": "

So, the difference between the shimming vs. auto-leveling is that auto-leveling goes around in a 4x4 grid taking measurements of the bed (you do need to set the Z calibration first, as you know) vs. the shimming method where you go to 4 corners on your build plate where you stick a piece of paper or something under there and you adjust the knob at the bottom until you can feel tension on it. What I've found to be the best for me at least is to use both. This is my method for printing every time:

\n
    \n
  1. Clean the build plate. I will take my scrapper and scrape everything off, then I will run it under some hot water and wipe all of the residue off that is still remaining, then I might take some glass cleaner or something and wipe it down.

    \n
  2. \n
  3. Set the Z calibration.

    \n
  4. \n
  5. Do the shimming leveling (I would normally go through all of the corners again plus the center/Z calibration just to make sure that I didn't mess up any of the other corners.

    \n
  6. \n
  7. I will run the auto-calibration (I use OctoPrint so I just run it through there)

    \n
  8. \n
  9. Smear Elmer's glue over the print bed. I do this for a couple of reasons:

    \n
      \n
    1. I use the glass side of the bed, just because I've gotten better results this way, and
    2. \n
    3. it just makes your prints stick better. I also do this after the Z calibration and bed calibration because you get better results this way.
    4. \n
    \n
  10. \n
  11. Start the print and watch the first couple of layers. If you notice that when you are printing and the brim looks stringy, like you could pick apart the individual lines, you should cancel the print, and go back to your slicer and bring your line width down my 0.1\u00a0mm, I typical print with the lines 0.1\u00a0mm or 0.2\u00a0mm below what the nozzle prints at, so a 0.5\u00a0mm nozzle I will print at 0.4\u00a0mm. And if your print messes up here I'd scrape everything off so the filament and the glue off (I'd keep the bed hot and the tip hot just so I don't have to wait 10 min. for it to heat up again) then apply more glue and reprint.

    \n
  12. \n
\n" }, { "Id": "16347", "CreationDate": "2021-05-23T01:37:58.927", "Body": "

I've read that white filament is a lot more brittle, because the pigment percentage is close to 50\u00a0%, e.g. black filament it's only around 8\u00a0%. I was therefore wondering if 'natural' colored filament, which has a somewhat ivory appearance, is also brittle like white filament.

\n

Maybe the color name is a marketing lie or maybe it is really natural colored and there has slim to no pigment in the filament. Making it therefore less brittle. Since the color is closer to white than black I would guess the first one but I would love to be certain.

\n", "Title": "Is 'natural' colored filament equally brittle compared to white filament?", "Tags": "|filament|filament-choice|filament-quality|filament-production|stability|", "Answer": "

No

\n

Natural colored filament has no pigments added, the ivory off-white color is the natural color of the mixture in the filament.

\n

Also note, those white filaments can have much less than 50\u00a0% volume percentage of pigment, depending on the exact hue of white. The starkest cold whites might make that, warmer white can work with a lesser amount. Pigments are part of the filler materials. The other fillers make the filament behave in the designed way and can make up a considerable amount of the filament. Usually, less than 40\u00a0% are fillers compared to the PLA.

\n" }, { "Id": "16355", "CreationDate": "2021-05-24T06:10:46.580", "Body": "

I am trying to print a tank to be used with my RC engine. The material that I have to use needs to have the following properties:

\n\n

I first - foolishly - just printed it with ABS as I do for every fluid container as I can nicely seal it with Acetone. After almost being done with it a friend told me to check the chemical resistance of ABS and as you might guess methanol and nitromethane completely desintegrate it, almost like acetone itself, so that was a waste of time/material.

\n

Next I considered using Nylon. It shows exactly the chemical resistance against all the fuel components I need, however I could not find any (easily available) option to seal the print after printing and after printing a small test container and pouring in some water it leaks after half a minute, so unless I find a way to seal the Nylon containers interior this is also not an option.

\n

I checked various epoxies but the few that I checked all showed poor resistance against methanol/nitromethane.

\n

What I could not test yet but seems like an option is using HDPE. I am using PET bottles to transport the fuel sometimes so it definitely is both resistant and - in theory - watertight, however I am not sure how I can seal an HDPE print, so I am not sure about my first requirement with HDPE. EDIT: I found that Limonene dissolves HDPE and is relatively harmless and easily obtainable. Maybe it can be used to seal the HDPE print surface?

\n

Hence my question: Is there a material that can be printed on a regular desktop printer (heated bed, nozzle up to 255\u00b0C) that satisfies both my requirements above or am I "doomed" to buy a moulded plastic fuel tank?

\n", "Title": "Printing material for methanol tank", "Tags": "|print-material|chemistry|", "Answer": "

As a supplement to the answer (doesn't fit well in the comments). This site https://www.filamentive.com/chemical-resistance-of-3d-printing-filament/ lists PETG has have a very high restance to alcohol, a high resitance to fuel

\n
\n

Is PETG UV Resistant?

\n
\n
\n

In the long run, it is now a well-known fact that it will almost always come out on top when it comes to objects and builds that have to be resistant to ultraviolet (UV) radiation. I.e., UV resistant. This is why it is ideal for objects that have to be kept under the harsh mid-summer sun. This is because UV radiation has minimal effect on it as compared to PLA and ABS.\n--\nFrom https://makershop.co/petg-uv-resistance/

\n
\n" }, { "Id": "16396", "CreationDate": "2021-05-27T23:33:41.123", "Body": "

I live in an arid environment and have never worried about leaving filament out for an extended period of time. To reduce how frequently we receive static shocks I installed a whole house humidifier with a target humidity of 35\u00a0%. Is this enough humidity to warrant storing filament in a dry box? At what point does ambient humidity become a concern?

\n", "Title": "What ambient humidity should one target for filament storage?", "Tags": "|filament|storage|", "Answer": "

The closer to zero humidity the better. Maybe a garage or some place protected from rare precipitation outside would be better. If one room is considerably less humid than the others, you could use it. Keep the bags and dry packs the filament comes in and put them back in when not in use.

\n" }, { "Id": "16405", "CreationDate": "2021-05-30T01:59:01.657", "Body": "

I decided to purchase a BLTouch for my Ender 3 V2. I did all the physical installation of the BLTouch at the same time as adding LED lighting. I plugged in the power to test the lighting (BLTouch was not plugged in yet) and everything came on fine and the lights worked as expected. I plugged in the BLTouch, hot-glued it in place, and finished zip-tying and screwing in everything else back in place.

\n

I connected power once again and powered on my printer. The fans came on, the new lights powered on, but the display didn't come on. I turned it off and back on, still no display. Then I smelled the distinct odor. I quickly powered off the printer and opened it up to look at the mainboard. I couldn't see any major scorch marks at first, so I decided to power it up again. That's when I saw the small spark and magic smoke at F2 in the picture below.

\n

I feared the worst that I had fried my mainboard, but thought maybe it was the BLTouch. So, I unplugged the BLTouch and powered the printer on and miracle of miracles it powered on. All the fans, lights and display were on.

\n

So something with the BLTouch is causing the short but I don't know what it is. YouTube videos that I watched for installation showed a different cable than the one that came with the BLTouch that I purchased. My BLTouch came with a 1 meter extension that is actually 2 sets of cables. One set is a pair of black and white wires that I deduced were for power. The other cable is a set of three wires colored orange, red, and brown. I zip-tied the two connectors together to make sure they stay together. On the BLTouch end, the orange and black wires are next to each other. The connector in the bottom picture is in the same orientation that I plugged it in.

\n

Printer: Ender 3 V2
\nMainboard: 4.2.2
\nBLTouch: Smart 3.1 from Amazon
\nFirmware: Jyers' Ender 3 V2 Extensible UI

\n

\"Ender

\n

\"BLTouch

\n", "Title": "What would cause magic smoke with a BLTouch installed?", "Tags": "|creality-ender-3|troubleshooting|bltouch|jyers|", "Answer": "

Given F2's proximity to the linear 3.3V regulator and some inductor and large diode possibly belonging to a switching regulator, it is likely that some short circuit caused the fuse on a power rail to blow.

\n

According to wiring instructions and your picture (if you're holding the connector as it was plugged in, you had it reversed:

\n
\n

Black to the G pins

\n
\n

NOTE: Sometimes blue or brown are used for ground wires

\n
\n

Red to the V pin

\n

Yellow/Orange to the IN pin

\n

White to the OUT pin

\n
\n

If that is the case and the protection was well designed, swapping the cable and replacing the blown fuse should repair the fault unless it also damaged to probe.

\n

As for identifying the fuse value for F2 (T20), different manufacturers, due to the size limitations of SMD components, unfortunately use different markings, but the T would indicate either 0.5 or 5.0A.\nIt might be best to ask support about the value for the fuse as the manufacturer would surely know.

\n" }, { "Id": "16406", "CreationDate": "2021-05-30T02:25:36.233", "Body": "

My Ender-5 Plus has started failing.

\n

The simplest path to failure is:

\n
    \n
  1. Power On
  2. \n
  3. Settings \u25b6 Leveling (wait till complete)
  4. \n
  5. Tap measuring
  6. \n
\n

The first zone will measure properly, then when starting to measure the 2nd or 3rd zone while the bed is raising the BLTouch, with probe extended, probe will start flashing red before touching the bed, the bed will descend, then my printer remains in that state (auto-leveling reported at zone 1 or 2, head at position 2 or 3, BLTouch flashing red, probe extended, and bed not moving about 2\u00a0cm below probe)

\n

I've also had it fail when attempting repeating levelings, and after starting a print immediately after leveling. (The printer will level, then the head won't move or extrude while the printer starts to repeat progress on the print).

\n

I've tried printing from OctoPrint (which had worked and now fails) then disconnected that completely and powered off then on to try from an SD card, w/ no success. I was able to connect the OctoPrint terminal and do some simple gcodes to see if those worked (G0 Z100, G0 X100) and those worked.

\n

I'm not sure what else to try.

\n", "Title": "Ender 5+ stops while bed leveling", "Tags": "|troubleshooting|bltouch|creality-ender-5|", "Answer": "

After disconnecting and reconnecting every cable, and doing a wiggle check on each, it still didn't work.

\n

So, I went ahead and replaced the BLTouch sensor with an upgrade from Amazon (Creality BL Touch V3.1 Upgraded Auto Bed Leveling Sensor Kit), and everything started working again. Huzzah!

\n" }, { "Id": "16418", "CreationDate": "2021-06-01T16:00:35.677", "Body": "

I recently received my Creality CR-6 SE printer and upgraded the controller board to a Bigtreetech SKR CR6 V1.0 and installed the latest community firmware.

\n

As you can see on the attached image, the corners have this weird elephant foot thing going on the vertical surfaces.

\n

\"enter

\n

Then there is that zig-zag pattern across the flat surfaces on the sides, makes me think the cubic infill pattern is shining through the sides. I am using Cura 4.8.0 with a modified start G-code to put a purge line on the side before print to prevent drooping in the middle of the bed while probing for the Z0.

\n

What causes these issues? I have not tweaked anything regarding the extruder aside from adjusting the screws that clamp the filament as the stock setting broke the filament when retracting because it clamped on too hard.

\n
\n

I have not changed the E-Steps, on the stock firmware this wasn't even possible.

\n

\"these

\n

I use Raise3d Premium PLA filament, I have tried a few and I'm most comfortable with this filament.

\n

If this is over extrusion, I can totally work with that. I have done E-step calibration before on my Ender3. I will test for that after the current print is done.

\n", "Title": "Weird pattern on vertical surfaces on all prints", "Tags": "|print-quality|bigtreetech|creality-cr-6|skr-cr6|", "Answer": "

Seeing the infill from the outside is usually caused by having a thin wall or an incorrect wall thickness compared to your nozzle. Wall thickness should be a multiple of your nozzle diameter; so a .4\u00a0mm nozzle should have 0.4, 0.8, 1.2\u00a0mm thicknesses.

\n

The following list of solutions is from All3DP's article "Troubleshooting Common 3D Printing Problems" under the heading Infill is Visible from the Outside:

\n
\n

CHECK THE SHELL THICKNESS
\nMake sure that the value you have selected for the shell thickness is a multiple of the nozzle size.

\n
\n
\n

INCREASE THE SHELL THICKNESS
\nThe easiest solution is to increase the shell thickness. By doubling the size it should cover any overlap caused by the infill.

\n
\n
\n

USE INFILL AFTER PERIMETERS
\nMost slicing software will enable you to activate Infill prints after perimeters.

\n\n
\n
\n

CHECK PRINT PLATFORM
\nCheck around the model and if you see that the effect is more prevalent on one side than the other, the effect could be due to calibration. If so run through the usual calibration process.

\n
\n
\n

PRINT SHELLS TO YOUR ADVANTAGE
\nDepending on the type of model that you\u2019re printing you can use the internal and shell printing order to your advantage. When you want a high-quality print with a good surface finish where the actual strength of the model isn\u2019t important, select print from the Outside-in. If however the strength of the print is paramount then select Print from in Inside-Out and double the wall thickness.

\n

The reason for the difference in strength is that when you print from the Outside-in you eliminate the small amount of overlap that causes the ghosting issue, but this also means that the actual structure won\u2019t create the same strength of bond between the internal and external structure due to the lack of overlap.

\n
\n" }, { "Id": "16430", "CreationDate": "2021-06-02T20:51:47.847", "Body": "

I am using an Ender 3 with a Micro Swiss hotend and a 0.8\u00a0mm nozzle, Polylite PETG filament and a masking tape covered glass bed. I have used Cura for slicing with settings:

\n\n

\"enter

\n

\"enter

\n

It is also interesting that the problem did not occur on half of the part, but did occur on the other half. Is the problem due to the slight warping up of the part, or due to something else? How can I fix it?

\n", "Title": "How do I fix mini-blobs on the edge of a part?", "Tags": "|creality-ender-3|ultimaker-cura|petg|retraction|", "Answer": "

What you are seeing is what is called the 'seam'. Since this part cannot be printed in vase mode (where the extrusion is essentially done as a compressed spiral), there is a point where the printing on a layer ends, and the printer moves to the next layer. This is where the seam is formed. Blobs that form at this change in layers are not uncommon, but the ones in your print seems larger than expected.

\n

You can experiment with larger retractions settings which normal reduce the size, but these seams cannot be avoided completely.

\n

Another change is that most slicers have the option to place the seam at different locations on your part, including placing them at random locations. This removes the pronounced seam, but put little blobs all over your print. Sometimes that is preferred for accuracy instead of the specific seam.

\n" }, { "Id": "16433", "CreationDate": "2021-06-03T16:29:58.393", "Body": "

I've upgraded my Ender 3 Pro to the point where I'm causing myself problems, one of them specifically is this:

\n

My z axis drops down to the bed because of all the stuff hanging off it (added a bltouch probe, cable chain) whenever it is idle, example video of what I am talking about. When it first started happening I bought an Anti backlash leadscrew nut and installed that and it helped A BIT. The gantry wouldn't drop as often or as far.
\nBut it still happens. I've thought about possibly adding a dual z axis motor and leadscrew anyway (with additional anti-backlash nut), and I wonder if that'll help because the gantry would have to overcome the friction on both sides..... but in the meantime I want to solve a very specific problem it causes:\nI just printed a benchy and right after the print was done (when I wasn't paying attention and before I noticed) the nozzle crashed right down into the print. It must have still been a little hot because it melted right into it. Not far enough to ruin it but I could see this causing an issue if it was something besides the Benchy.

\n

Is there code I can run at the end of a print to tell the hot end to move out of the way? So if it does come crashing down its off in the corner or something? I use Cura and marlin if thats relevant. I did see this question but I guess my even dumber question is: if that code works, how do I add it? Is that in Cura?

\n", "Title": "Gantry crashing into bed after print", "Tags": "|g-code|z-axis|", "Answer": "

What you describe sounds more like a last-layer (or near-last-layer) machine restart than a problem with the Z axis back-driving (though it shouldn't back-drive from the weight of the gantry alone; you may need to tighten things like the anti-backlash nut or Z rollers).

\n

My Ender 3 has, a few times, restarted after 3-6 hours of printing -- leaving the nozzle in a lump of plastic that has oozed as it cooled, and the "resume or stop" selection on the LCD. I haven't seen any indication of overheating, excessive current draw, or other issues with any of my (box stock) components. I have ordered a small 300W UPS to let my machine ignore the (presumed) tiny power flickers that I believe are resetting my mainboard -- but I won't know for certain if this solves the problem for a few weeks after I get the UPS, because the problem has been sporadic even with overnight and longer print times.

\n

With your upgraded mainboard and using the firmware provided by that supplier, you may no longer have the resume feature that was standard on my Ender 3, nor the end of print park (which, in any case, would not happen in the case of a machine reset).

\n" }, { "Id": "16438", "CreationDate": "2021-06-04T10:45:18.690", "Body": "

Is it possible to perform a colour transfer from paper/transparency film to printed resin?

\n

From what I understand there is some sort of transfer process for resin, but I don't quite understand if this works for 3D printed resins.

\n

\r\n \r\n

\n

\r\n \r\n

\n", "Title": "Colour transfer to resin", "Tags": "|resin|color|", "Answer": "

As shown in the second video, the resin is used to coat a surface. The resin is clear, or appropriately tinted to provide for a visually pleasing background to the transfer. The transparency is printed on an inkjet printer which embeds the ink into a surface designed to accept the ink, rather than allowing it to bead up, as in the first video.

\n

When the transparency is applied to the wet resin surface and the resin is allowed to cure, the specially formulated transparency surface is transferred to the resin surface, akin to gluing the entire image, using the backing of the transparency "paper" as the carrier. Once cured, the backing peels off, leaving the image.

\n

There is no analogy to resin 3D printing. If you 3D print a resin model that has a suitable flat surface, you could certainly apply the same types of resin shown in the video and duplicate that process. If the surface was not flat, you'll have irregularities in the transparency which will prevent the image from being transferred.

\n

I can't envision how the inkjet image could be transferred during the 3D resin printing process, as the transparency film would interfere with the bonding of the resin to the build plate or the vat surface.

\n" }, { "Id": "16455", "CreationDate": "2021-06-06T18:50:31.673", "Body": "

I was replacing the nozzle on my E3D V6 clone and the hotend started turning so I held onto it with pliers while I tightened it.

\n

As I did this, the positive wire of the fan that clips onto the E3D V6 melted on the hotend and threw a spark. At this point, the extruder started running so I hit the power switch then unplugged the main cord.

\n

I checked everything to see what caused it, then turned it back on once I figured out what happened and made sure everything was good. Now when it's powered on, it seems like it's still getting power; the BLTouch turns on but the screen doesn't show anything just the backlighting.

\n

Did I fry my board?

\n", "Title": "Did I fry my Ender 3 V2 Main Board?", "Tags": "|creality-ender-3|extruder|electronics|", "Answer": "

Yes, you can damage stepper drivers when you unplug stepper motors. Hot (un)plugging of stepper cables can cause voltage spikes that can destroy the stepper drivers. Furthermore, shorting out the fan may have caused damage also to other parts of the board (voltage regulator, fuse, etc ). These controller boards are not that expensive, ordering a spare would get you up and running quickly. Also order a new fan, mind the voltage.

\n

No, pulling the power cord or switching off the controller board will not destroy the board.

\n" }, { "Id": "16469", "CreationDate": "2021-06-07T18:30:49.797", "Body": "

I have the community edition of the firmware for my CR-6 SE printer installed on a BigTreeTech SKR-CR6 V1.0 board. This firmware has a nice E-step calibration guide you can use to easily get the correct step/mm value.

\n

However when I raise that value, my extruder motor starts to chop, it's like it doesn't have enough strength to push the filament through and starts chopping.

\n

I tried raising the current on the extruder motor, from 600\u00a0mA which is standard, to 900\u00a0mA incrementally, but it didn't make any difference.

\n

I just replaced the stock extruder with a dual gear extruder in hopes that the extruder was causing this. but the new extruder faces the same issue.

\n

I think raising the Vref value for the extruder motor might help, but I don't know how to do that. on the BTT SKR-CR6 V1.0 board all stepper drivers are in UART mode so there is no way to change this on the board itself. and there is no way to change Vref in the UI as far as I know.

\n

What should I do to progress with this issue?

\n", "Title": "Raising E-steps on my CR-6 SE makes the extruder choppy", "Tags": "|stepper-driver|creality-cr-6|skr-cr6|", "Answer": "

You cannot change the Vref of a UART operated stepper driver, setting the current directly is the way to go on UART operated stepper drivers; in fact that is actually what you are doing on non-UART operated stepper drivers, you change the Vref to change the current through the stepper motor, the higher the voltage the higher the current through the stepper the more torque the stepper has.

\n

Depending on the stepper you may increase the current even more, this depends on the stepper motor model/type. There are stepper motorss that allow more than 1000\u00a0mA, I don't know the steppers used in the CR6 E, find out before you test higher values.

\n

Please note that there may be something else causing this issue, possibly your filament path is having a lot of resistance. You should check the extrusion manually and possibly clean the nozzle and heat break.

\n
\n

A similar question is found here: Is the Vref trimmer relevant on TMC2208 in UART mode?

\n" }, { "Id": "16474", "CreationDate": "2021-06-08T18:55:36.850", "Body": "

tl;dr How can I test whether my sporadic in-print restarts are a thermal problem or a firmware issue?

\n

The problem

\n

I've had my Ender 3 (not V2 or Pro) for about a month, and gotten a number of excellent small prints with Amazon Basics and SunLu PLA, as well as Overture PETG, slicing with Cura Slicer 4.9.1; the printer has Marlin 1.0.1 according to the "About Printer" display. I've made no electrical changes to the original hardware, though I've added a few printed parts including a mainboard fan cover, alternative power supply mount, "original screws" display PCB cover, display cable clips, filament guides, storage drawers, and drag chains protecting the cables to the X stepper and extruder/hotend.

\n

However, I've had trouble for most of the time I've had the machine with sporadic reboots in the middle of a multi-hour print. At a minimum, when I resume the print after one of these incidents, the printer will skip the rest of the current layer and start with the next, producing a weak layer (usually it also leaves a significant blob of plastic, and even if I just flatten that blob with the scraper blade as soon as the nozzle is clear, it will stick out of the print and have to be trimmed off later, and if I try to scrape the blob away while it's still soft, it'll result in layer shifts caused by skipping the Y-axis stepper).

\n

Usually, this happens about four to six hours into a 10+ hour print, though I've seen it happen just a half hour or so after a previous occurrence, and near the end of a 65-minute print as well. It's becoming a big issue as I venture into larger prints, as it can ruin the appearance of a print (by skipping a large fraction of a layer that's the visible surface of the part) or causing a weak layer at a stress point -- which wastes a lot of machine time as well as filament.

\n

I've also just upgraded my build surface to a Creality coated glass plate -- which is great in terms of flatness, sticks well, and the part comes off very easily when the plate cools. Unfortunately, if I'm not in the room when the printer restarts, and the bed gets below about 40 \u00b0C, the part will unstick and will start moving with the nozzle when I resume the print.

\n

What I've tried

\n

I had suspected issues with my 120\u00a0V power, so I purchased a small UPS to condition the power for the Ender 3 power supply, but without seeing improvement. Prints running overnight often do better, but not always, and this happens with both PLA (nozzle temp 200\u00a0\u00b0C, bed 50\u00a0\u00b0C) and PETG (nozzle 235\u00a0\u00b0C, bed 75-78\u00a0\u00b0C). I haven't attempted other filaments such as ABS.

\n

I'm usually not watching the printer when it does this, but several times I've caught it immediately after (I can hear it running when I'm at my computer, so when it stops I can get to it in seconds), and once I happened to be watching when it stopped. The sequence is that (usually, but not always, when printing a layer with significant infill -- for which I usually use cubic, for best strength with minimal material and time) the printer will suddenly just stop with a normal info screen display for fifteen to thirty seconds (I don't have the exact time, and have only seen it early enough to have an idea a couple of times), then the display goes dark and within two or three seconds returns to the startup splash (Ender 3 dragon logo), from which it changes to the power failure recovery screen with "Resume Print" and "Stop Print" choices.

\n

This is not a reproducible error (nor am I likely to attempt repeating an overnight or whole-day print multiple times to attempt to reproduce the problem). At no time have I noted the power supply cooling exhaust running hot, nor the steppers getting as warm as a 50\u00a0\u00b0C bed (I don't have an IR thermometer to check the actual stepper temperature). I used a photographic thermometer to check the temperature of cooling air leaving the mainboard housing, and found it never exceeded 30\u00a0\u00b0C during the print I tested.

\n

My hypothesis

\n

I'm aware of two significant possible causes: overheating the mainboard, or overloading the stepper pulse loop in the firmware. The former should (if things are designed sensibly) result in a complete shutdown, though with a hard switch on the power supply it might not be possible to prevent a restart as soon as the sensor reads below critical (which could be the fifteen to thirty seconds of "freeze" I've observed). The latter is something I've read about, where the Marlin firmware will restart if the stepper pulse interrupt loop gets so busy the mainboard can't service the sensor interrupt loop.

\n

The only way I know to test which I'm seeing is to find a way to more efficiently cool the mainboard and see if that improves the situation -- but I'm reluctant to either remove the fan cover or run the printer with the mainboard cover off; the reason for installing the fan cover was to keep fragments of plastic from getting into the mainboard housing as, being directly under the build plate, it's prone to collect strings, hairs, and scrapings.

\n

Update

\n

I thought I had solved this problem -- I reduced the print speed (using the "tune" menu while the print was running) to 90% (=45 mm/s instead of Cura's default 50 mm/s) and started getting longer prints without stoppages; that would have pointed to firmware loop overload as the likely cause. Unfortunately, I still got a freeze about 10.5 hours in to a 13 hour print, but only one rather than the three or so I'd have expected; this may be a clue. Since then I've had freezes in under an hour into a new print once again, once with the bed running to Ymin (a movement not in the gcode) instead of stopping in place.

\n

Due to the required level of disassembly, I haven't attempted to test continuity while in motion on the stepper or hotend wiring, and I cut away the damaged portion of the Bowden tube when I replaced the hotend and nozzle; I get good extrusion and, when they don't stop in the middle, good smooth prints.

\n

Update 18 June 2021

\n

As suggested in comments, I have initiated a "dry run" of a print that previously generated two or three of these stoppages over the course of about fourteen hours. Original print was Overture clear PETG, nozzle at 235\u00a0\u00b0C and bed at 75\u00a0\u00b0C. For this "print" I've retracted the PLA loaded into the machine well clear of the hotend and unplugged the extruder stepper; when I get home from work, I'll be able to see if the machine is still running, and if it has restarted, resuming the print will allow me to tell how long it ran before the incident.

\n

Test Result

\n

This is unexpected. I thought I'd either find the machine still running, or with the power failure resume screen asking to "Resume Print" or "Stop Print" -- instead, I came home from work, some 10 hours after starting the dry run test, to find the printer idle, as if I'd had it properly shut off and then turned the power on. No way to tell when it stopped or why.

\n

Further Update, June 20

\n

I had another of the new type of failure on an overnight print; I was printing PETG (Sun Lu brand), 240\u00a0\u00b0C nozzle and 75\u00a0\u00b0C bed, back on my magnetic textured build sheet because the PETG was warping and coming up from the coated glass -- and when I got up this morning the printer should have still been running, at around 90% complete, but instead it was sitting at the Info Screen, nozzle and bed both reading room ambient temperature with set point 0\u00a0\u00b0C.

\n

I can only presume that either the "resume print" option has a limited life (though I've seen it at what must have been eight or nine hours after the print stopped and print progress in this case suggests it had only stopped four hours or so before I came back), or I'm getting a second interrupt from the resume screen that's causing the printer to return to "fresh power on" state.

\n

I'm reluctant to pursue further warranty help from the Amazon seller because of the hassle of returning a machine I've customized and my feeling that this is an easy fix if I can just determine what to fix.

\n

Followup, June 21

\n

I'm starting to believe my machine is haunted. Over the past twenty-four hours, I've had about twenty-one continuous hours of printing, completely successful (in terms of the machine continuing to run; I've got some corner lifting on the print that was running when I left for work this morning, but that's print settings and/or material vs. build surface). I was more than ten hours into a twelve-plus hour print when I left this morning, and completely a nine and a half hour print yesterday without incident.

\n

In other words, it almost seems as if the problem has resolved.

\n

My partner half jokingly suggested the cats might be somehow resetting the machine, but I don't see any way they could. Just poking at the Ender 3 control knob won't do anything, and even if they managed to turn the knob and poke it again, if they initiated a "stop print" the machine would retract, lift the nozzle, and home the X and Y. If they batted at the power switch (which is after the UPS) and got it to momentarily break power, I'd expect to find the "resume print" screen -- and further, this has happened when I'm in the same room and the sequence isn't compatible with a power break.

\n

Additional Followup 22 June 2021

\n

I thought the machine was back to trying to make a liar out of me, after getting around 27 hours of failure-free prints -- and then it stopped at about 60% of a 2 1/2 hour print in PLA, glass bed, at 200\u00a0\u00b0C nozzle and 50\u00a0\u00b0C bed, 10% gryroid infill. It was printing fine when I went to bed, and when I got up I found it at the info screen (not the resume print choice screen), with the nozzle stuck to the infill.

\n

Additional information 26 June 2021

\n

I was able to take a few minutes to open the main board housing while installing belt tensioners today -- I was slightly startled to discover that despite shipping with Marlin 1.0.1 (correction: it's 2.0.1-V1.0.1, 32-bit build and the newest available from Creality for this mainboard), the mainboard is the 32-bit version 4.2.2. While this is not the most current version, it is the board that was introduced with the Ender 3 V2 in 2020 and almost immediately added to the Ender 3 Pro, and can be updated through the MicroSD or via the USB port.

\n

I'm told (via discussion on the Ender 3 group at Thingiverse) that this rules out firmware overload, as the 32-bit boards are both much faster computationally, and have significantly larger on-board RAM and EEPROM.

\n

I also went over the electrical connections on the mainboard; I found two of the screw terminals weren't as tight as I'd make them (this is probably a result of using tinned wire instead of crimped ferrules); I also was able to push down one of the stepper driver outputs (not sure which one), which wasn't fully seated (though surely making contact, since I've had all steppers operational any time the machine wasn't resetting itself).

\n

Unfortunately, tightening the slightly loose screw connections helped nothing; though I was able to complete a 1 1/2 hour print flawlessly, I had two consecutive failures several hours into a 7 1/2 hour print.

\n

My next thing to check is internals in the power supply.

\n

June 29 update

\n

Power supply check showed perfect 24.0 V output, but I found the two spade terminals on the power switch itself were loose enough to move around with a little tug on the wire. They were making good contact, as far as I could tell, and required effort to pull off the spade on the switch, but once things heat up after some time under load (running X and Y steppers and bed heater at the same time, for instance, along with near-constant nozzle heat) they might well have gotten looser -- and since I moved the power supply off the vertical extrusion to a pair of printed brackets to improve clearance between the magnetic build sheet and power supply, it does move around a bit when the machine is printing (especially with the rapid direction changes of infill).

\n

On that basis, I disconnected both wires and squeezed the terminals with pliers to close them up a little; they were very tight when I reconnected them.

\n

Too early to be sure, but the machine finished a 7.5 hour print flawlessly after this.

\n

Update June 30

\n

It was better -- I got two 7.5 hour prints in a row without a failure. Printed a spacer to allow quick switch between magnetic build surface sheet and glass (clips onto X gantry plate to trigger the Z-stop with the nozzle higher by exactly the extra thickness of the glass plate). Too thick; easier to reprint than to sand or scrape enough.

\n

Machine stopped at the same spot while printing the skirt three attempts in a row. This is less than one minute into the print. Further, doesn't reboot, just freezes until I power it off, then comes back up with "resume" dialog. Tried to reslice, thinking it might be a bad g-code file, and the microSD won't mount on my desktop computer, though it still seems to read okay in the printer and another (full size) SD card mounts on the same USB port and hub (and the problem card doesn't mount even in the microSD slot in the other reader).

\n

This last doesn't appear to be related to the original problem: the microSD had seemingly gotten disconnected without "safely remove" procedure, and Ubuntu apparently isn't capable of recovering from this; that resulted in a corrupted gcode file causing these most recent, very early stoppages, as well as the inability to mount the storage device. Reformatting the storage device corrected that problem.

\n

Question?

\n

What other tests can I run to differentiate between a thermal restart and a firmware overload?

\n", "Title": "Why is my Ender 3 restarting spontaneously in mid print?", "Tags": "|creality-ender-3|firmware|hardware|", "Answer": "

This problem (of spontaneous restarting during printing) appears to be solved.

\n

The Cause

\n

The apparent cause was a combination of push-on spade connectors on the power switch that were looser than ideal, likely specified that way in order to facilitate mass production (the tighter such a connector is, the more difficult it is to assemble, leading to slower work and an increase in damaged parts that then require reworking an assembly), and moving the power supply off its direct mount to the machine frame (in order to increase clearance between the PSU and the heat bed after installation of a magnetic build sheet).

\n

The brackets I used to remount the power supply are free to slide in the 4040 extrusion of the frame base, and have no mechanical connection to the PSU housing or even its plastic extension (which houses the wire connections, power switch, and power cord connector). This allows the PSU to move much more under frame accelerations (Newtonian reaction to movements of the bed and hot end) than with the original mount -- and this movement sometimes pulls against the wires inside the plastic wiring cover.

\n

Combine that with spade connectors that are loose enough to move when the wire is tugged pretty gently, and you have a formula for momentary disconnects. Such a disconnect will reset the main board (like a power flicker can do to a computer with no UPS), resulting in the freeze (actually mainboard startup sequence before it begins to update the display), then a startup.

\n

The Complication

\n

I can't prove it, but I hypothesize that the transition from restarting to the "resume print" screen, to restarting as if freshly powered up after a normal shutdown occurred because the connection issue had progressed to the point of producing two or more disconnects close enough together that the second restart lost the "print in progress" flag.

\n

The Solution

\n

What appears to have solved the problem was to remove the wiring cover on the PSU (ensure power is off and power cord disconnected, mains voltage is present inside this cover), disconnecting the spade terminals at the power switch, and (with pliers) carefully squeezing the female receptacles so they are tighter on the male blade. This must be done carefully, as squeezing too hard will close up the female terminal and prevent reconnection, requiring either much more difficult adjustment with knife and miniature screwdriver, or replacement of the terminal.

\n

Since applying this fix, I've completed a few longish prints (about 7 1/2 hours each) and a number of shorter ones without a single glitch, aside from an unrelated problem that caused a corrupted gcode file.

\n

Adding to this, I did get one in-print reset after this answer; it happened while printing gyroid infill at (IIRC) 40%, which is some of largest ongoing acceleration (=> machine frame movement) of anything you'd do.

\n

My final solution has been to add a third bracket to the power supply, so three corners are supported. Now the power supply doesn't wiggle and wobble when the machine shifts a little; it's back to a near-rigid mount to the frame (if anything, a little sturdier than the original two screws through an upright). I haven't tried gyroid infill again, but I've had 100% reliability (including a couple large prints at 7+ hours) since installing this third bracket.

\n" }, { "Id": "16486", "CreationDate": "2021-06-09T18:03:51.610", "Body": "

I have a Prusa i3 Mk3s with an MMU2s on it. And to be frank, as far as the MMU goes, I have not had much luck with it. I've seen people print amazing MMU prints and it works great for them, but I have had MUCH less success.

\n

If a print requires 900 filament changes to go back and forth between colors, even if I have the MMU dialed-in to work 99\u00a0% of the time, well that's still 9 failures in that print. I'm sure there are things I could do to dial it in, but I have a different question for now:

\n

I have other printers I work with that have dual extruders and work great, and I've been eyeballing an IDEX kit for my Ender 3, which got me thinking that there's gotta be a way to put IDEX on a Prusa.

\n

I was also eyeballing my MMU and thinking I could scrap it for parts (stepper motors etc) for the IDEX. I did a quick Google search and couldn't really find anything, but is there a reason for that? Is the control board not able to handle it, etc?

\n", "Title": "Can a Prusa be converted to IDEX?", "Tags": "|prusa-i3|dual-nozzle|", "Answer": "

It would require an adaptation so extensive that I would define it "rebuild".

\n

You would end up savaging basically only the bare frame and the steppers.

\n

I'm not aware, in fact, of anyone ever doing it.

\n" }, { "Id": "16487", "CreationDate": "2021-06-09T18:37:26.703", "Body": "

I have an Ender 3, currently in stock Bowden extruder configuration.

\n

I want to be able to print nylon and TPU, both of which require temperature too high for the tolerance of the PTFE Bowden tube (as well as the issues with the flexible filament in the tube).

\n

Therefore, I've considered converting my printer to direct drive. However, the conversions I've seen, both DIY/print the parts type and commercial, seem to include a short length of the same PTFE tube between the extruder (now mounted on the hot end carriage) and the actual hot end. This same material ought to have the same temperature limit (about 250 C) as it would have in a Bowden configuration -- and for nylon, at the least, this is a problem, since the PTFE would start to soften from contact with the heat break.

\n

Am I missing something in these conversions, or is the PTFE's glass transition not the limiting factor in printing hotter with a direct drive conversion?

\n", "Title": "How does direct drive conversion allow higher print temperatures?", "Tags": "|creality-ender-3|hotend|nylon|tpu|direct-drive|", "Answer": "

Bowden tubes extending to the nozzle to make a seal will damage the Teflon (PTFE) tube if exceeding 250\u00b0C for an extended time. An all-metal hotend for a Bowden would have either the nozzle sealed against the heatbreak or an integrated nozzle and heatbreak. Then the Bowden tube would attach with a minimum gape with either the heatbreak or heatsink.

\n

Note: The heater block needs to be at the maximum operating temperature When making a seal between the nozzle and heatbreak.

\n" }, { "Id": "16502", "CreationDate": "2021-06-10T15:15:37.933", "Body": "

So I have printed multiple benchies with various slicer settings. I have also made sure my bed is leveled well.

\n

But no matter what I do, I get these over extrusion whenever my nozzle is moving in diagonals (left-right). It's always this same angle where it goes wrong.

\n

Has anyone else who has faced this and has figured out know what the issue can be? I'm new to 3D printing so the help would be very much appreciated.

\n

\"Picture

\n", "Title": "Prints have over extrusion only during diagonals", "Tags": "|extrusion|3d-printerworks|", "Answer": "

The issue was that the X-axis belt was super loose, figured it after someone pointed it out on Reddit:

\n
\n

swordfish45

\n

You have an extremely loose x belt. That's the root cause of all your problems I can see in these pics.

\n
\n" }, { "Id": "16506", "CreationDate": "2021-06-11T04:44:00.000", "Body": "

I just assembled Ender 3 and noticed that the X-axis movement doesn't correspond to the commands. When I make it move 1 mm with the encoder it moves 16 mm. Everything is in the "out of the box" configuration.

\n

The current steps/mm for the X-axis read (from the display) 80 steps/mm.

\n

Am I supposed to manually fix this with steps per mm setting or could it be another problem. Other axes seem to work fine. I also double checked and everything should be built correctly.

\n", "Title": "Creality Ender 3 -X-axis moves too much", "Tags": "|creality-ender-3|x-axis|", "Answer": "
\n

When I make it move 1 mm with the encoder it moves more like 15 mm.

\n
\n

15 is suspiciously close to 16. Perhaps the motor driver is operating in full step mode rather than microstepping. This could be due to a manufacturing defect in the electronics. To diagnose this you would need a multimeter to measure the voltage on the microstepping select pins of the affected motor driver.

\n

Some main boards have jumpers to select the microstepping mode. I am not entirely sure but I don't think the main board that comes with the Ender 3 has them, but (in case I am mistaken) it could be worth checking that you did not forget to put any jumpers in place.

\n

To fix the problem you could also reduce the steps/mm from 80 to 5 but operating in full step mode would seriously degrade print quality, so it is preferrable to find the root cause.

\n" }, { "Id": "16511", "CreationDate": "2021-06-11T23:40:55.463", "Body": "

I have recently started printing with Polymaker's PolyMax PETG on my Ender 3 v2. I have not been able to get the layer adhesion anywhere near as strong as it should be. I would guess it is around 20-25\u00a0% of the strength in the XY direction. The parts snap easily along the layer lines under loads that PLA and nylon hold up to just fine.

\n

Print settings:

\n\n

I made sure to use a nickel-plated brass 0.4\u00a0mm nozzle. I have calibrated my E-steps and tried printing in an enclosure, but nothing seems to help with layer adhesion. I have made sure there is no debris getting on the filament as it comes out of the drybox and even tried taping around the heater block so there is absolutely no part cooling.

\n

Any ideas as to what I could be doing wrong?

\n", "Title": "PETG layer adhesion problems", "Tags": "|creality-ender-3|adhesion|petg|print-strength|", "Answer": "

The layer adhesion with Polymax PETG at 260\u00a0\u00b0C was great so I didn't go any higher.

\n" }, { "Id": "16514", "CreationDate": "2021-06-12T04:41:03.403", "Body": "

The hot end set-up for my printer consist of 2 Titan Aero extruders. The printer relies on Reprap firmware and a Duet 3 controller board. When I finish setting everything up, my primary extruder will use ABS and my secondary extruder will uses HIPS.

\n

If the model I am printing only has supports near its base, once the supports have been completely printed, will the secondary extruder remain hot/at its printing temperature setpoint until the entire print finishes? Or has this been worked out.

\n

Even more specifically say the HIPS extruder isn't needed for several hours during a print. Will it cool down and begin heating up when its needed or just slightly ahead of time?

\n

I feel like the HIPS filament left over in the extruder would be ruined after this and likely to clog going forward.

\n", "Title": "Dual extruder does support/secondary extruder stay hot during entire print?", "Tags": "|reprap|dual-nozzle|", "Answer": "

Slicer Cura has a standby temperature that is used when the hotend is not in use. Furthermore, you can write your own filament/hotend change scripts to do whatever you want. You can even do a full cool down of a hotend and full warmup when it is needed again. You can also always edit your G-code file after slicing, use a standby temperature for the beginning of the print where supports are used, and manually change the G-code at the last change to disable the HIPS extruder.

\n" }, { "Id": "16522", "CreationDate": "2021-06-13T22:02:58.153", "Body": "

I have a Monoprice Maker Select Plus.

\n

It recently started having issues printing. It turns out that the small plastic tube that feeds the extruder with filament was completely clogged. I couldn't get any of the filament out so I need a new feeder tube. I can't find one specifically for this printer anywhere. Does anyone have any ideas?\"enter

\n", "Title": "Can't find replacement nozzle tube", "Tags": "|monoprice-maker-select|", "Answer": "

This printer is using a so-called MK 10 extruder/hotend. These are a little different, there is a liner for the heat break that continues all the way down to inside the MK 10 nozzle:

\n

\"enter

\n

This PTFE tubular liner has an outer dimension of 4\u00a0mm and an inner diameter of 2\u00a0mm. The length of the tube is 34\u00a0mm. These tubes can be bought at pre-cut size or, more economical, bought per meter.

\n" }, { "Id": "16523", "CreationDate": "2021-06-14T08:11:14.640", "Body": "

My Tevo Flash works well. Right now, I'm trying to print a thin, disc-like part on supports. The 3 perimeters at the edge of the disc are OK. The dense fill zigzag pattern makes many U-turns near the perimeter. It all looks OK in Simplify3D. But in the printed part, in several spots, the zigzag pattern doesn't reach the perimeters, leaving a ~1\u00a0mm gap.

\n

My guess: in those spots, the filament has nothing to grab onto underneath, so the U-turn region is dragged back by the nozzle a bit (away from the perimeters) and/or it droops. I'm using the smallest support res in Simplify 3D: 1\u00a0mm. Any options I can try?

\n

\"enter

\n", "Title": "Fill doesn't reach perimeters on supports (Simplify3D)", "Tags": "|support-structures|infill|simplify3d|", "Answer": "

Posted as a wiki answer from a comment from the OP to a different answer.

\n
\n

The Zig Zag vs Lines patterns was only a part of the problem. The 2nd part was that this layer rested on supports only, so the fill, whatever its pattern, didn't have enough surface to "grab on" near the perimeters. If I had switched from Zig Zag to Lines, the air gap would've diminished a bit, but I would've gotten drooping. I solved it with wider, underlying perimeters.

\n" }, { "Id": "16530", "CreationDate": "2021-06-15T15:16:16.197", "Body": "

3D printer use stepper motors for moving print head and extruding filament. They need to have good torque and resolution.

\n

Microstepping improves resolution as much as 32 fold (I think) but reduces torque the higher you microstep.

\n

So...

\n
    \n
  1. Why not rotate the motor with microstepping at high RPM (which also reduces torque) and increase the torque by heavy gear reduction using a worm gear?

    \n
  2. \n
  3. Won't the movement of printhead be even smoother and small errors in microstepping and unevenness of gears be averaged out using high RPM and gear reduction approach?

    \n
  4. \n
  5. Does microstepping indeed provide accurate divisions of steps?

    \n
  6. \n
  7. Can we get by with weaker motors because torque will be increased by gear reduction?

    \n
  8. \n
  9. Can we get by with 48 step stepper motors instead of 200 step because gear reduction provides increased resolution?

    \n
  10. \n
  11. There are extruders that use flex shaft to turn worm gear in direct extruder while motor is mounted on frame which turns flex shaft (zesty nimble comes to mind). Why don't they just use smallest possible stepper motor to rotate worm gear directly, instead?

    \n
  12. \n
\n

Increasing motor RPM and using gear reduction should preserve the precision and torque, letting you use weaker, lighter motors, potentially reducing granularity of movement. I thought this was simpler approach and I wanted to understand what would I be losing as trade offs. I had considered more friction at worm gear and wear, higher heating of motor etc. But may be it's like "don't fix what ain't broken". 3D printers aren't that costly nowadays. I just wish they were even cheaper.

\n", "Title": "Why not use high rpm stepper motors with high microstepping with gear reduction in 3D printer?", "Tags": "|diy-3d-printer|delta|microstepping|", "Answer": "
\n
    \n
  1. There are extruders that use flex shaft to turn worm gear in direct extruder while motor is mounted on frame which turns flex shaft (zesty nimble comes to mind). Why don't they just use smallest possible stepper motor to rotate worm gear directly, instead?
  2. \n
\n
\n

The Flex3Drive, from which the Zesty was purportedly cloned, does admit using a very small NEMA-8 motor directly coupled to the worm gear in place of a flex shaft. This still adds a considerable (from the standpoint of machines aiming for 40 m/s\u00b2 acceleration and such) amount of mass to the toolhead, and I'm not clear what E-axis speed/acceleration is attainable with 40:1 reduction and a low-power motor like that.

\n

While the larger steppers typically used in 3D printers are perfectly capable of any plausible E-axis speed or acceleration you could want without gearing or with small reduction (e.g. 2:1 or 3:1), once you get up to worm gear level ratios, it's a question of the properties of your specific motor whether you can get enough speed for acceptable retraction performance or even normal print moves at very high speeds. For example, I have a Flex3Drive G5 and the orignal NEMA-17 E-stepper from my Ender 3 does not work well with it without further gearing on the motor side of the shaft. However, a "high RPM stepper" like you suggest should manage just fine with worm gear reduction.

\n

Now, is there any reason to do this? If you're transmitting the force in a manner that can't deal with high torque, like a flex shaft, yes. Otherwise, no. As others have said, modern microstepping is ridiculously accurate.

\n" }, { "Id": "16544", "CreationDate": "2021-06-17T02:16:07.533", "Body": "

I am trying to set up RepRap firmware for a CoreXY shown below. There are 4 limit switches in the top plane and one for the Z-axis.

\n

I believe that LS1 & 2 are considered for the X-axis and LS3 & 4 are for the Y-axis. My question is do I need all of these switches? Is it even possible to program this many in the firmware?

\n

\"Image

\n", "Title": "CoreXY limit switch locations", "Tags": "|corexy|", "Answer": "
\n

LS1 & 2 are considered for the X-axis

\n
\n

Yes, these correspond to X-max and X-min, respectfully.

\n
\n

LS3 & 4 are for the Y-axis

\n
\n

Yes, these correspond to Y-min and Y-max, respectfully.

\n
\n

do I need all of these switches?

\n
\n

Technically, no, you need one switch for each axis (if homing is used), but it can be saving your printer when somehow a layer shift has occurred which forces the head to move outside the bed dimensions.

\n

Note that you even can use zero switches, but then you would need to position the print head yourself before each print and condition the G-code with a starting position, this is seen in some CNC machines.

\n

It is a good idea to use min and max switches, but not necessary. (My CoreXY printer doesn't have the Y-max endstop as there was a little too less space to place it, restricting Y bed size...)

\n
\n

Is it even possible to program this many in the firmware?

\n
\n

Yes, firmwares allow for min and max switches, sometimes even multiple min or max switches for a single axis. The problem is whether your controller board has enough pins exposed and connectors added on the circuit board. But, min and max switches for X, Y and Z axis are not uncommon anymore, look at the specs of the controller board.

\n" }, { "Id": "16546", "CreationDate": "2021-06-17T07:58:54.987", "Body": "

I have a Tevo Flash. Normally, I don't care about perpendicularity with respect to the table. But now I have a 5" disc on a ball bearing, held by a 3D printed tube with a flat bottom. If the tube's axis is not 100\u00a0% perpendicular to the bottom, the disc, when spun, wobbles at the edge: ~1/8".

\n

\"Disc

\n

As a test, I printed a vertical sample tube. I took close-up pics, on the printer's table, next to a carpenter's square. Down the table's Y axis, there's a vert. deviation of ~1\u00a0mm over 45\u00a0mm of height, between the sample tube and the square. Down the X-axis, the deviation is small.

\n

\"Deviation

\n

How do I deal with it? Can a slicer (I have Simplify3D) compensate for it? I could gently "skew" the geo in 3D modeling software, but it seems inelegant.

\n

Note: this has nothing to do with bed leveling. The bed is level, the printer has a BLTouch. The first layers look great. The problem is above the bed. The right angles of the aluminum-extrusion frame aren't 100\u00a0% exact. Measured with the carpenter's square, the vertical columns of the frame (Z) deviate 1-2\u00a0mm over 100\u00a0mm from perpendicular, with respect to the bottom frame (X-Y). Trying to fix the whole frame would be hard.

\n

EDIT: I used a 0.127 mm shim (from a sacrificial steel gauge blade), it fixed most of it. With the printer laid horizontally (so I could work with the screws underneath) and the shim in, the vertical posts were 100% true (see pic). When I put the printer back into its vertical, working position, the posts tilted back a bit. I'll try a 0.15 mm shim.

\n

\"Shim

\n", "Title": "Need accurate 90\u00ba angle with respect to the table", "Tags": "|print-quality|tevo-tarantula|dimensional-accuracy|", "Answer": "

The vertical axis of your print is determined by the machine's Z axis, not by the "bed level" compensation of a BL Touch or similar bed error compensation system.

\n

When you print after BL Touch measures the bed height, the firmware moves the Z axis up and down to compensate for errors in the bed (including non-planar bed surface, which is its main utility) -- but as you print and the hotend moves up layer by layer, the movement is determined by the Z axis -- with a common gantry type printer, this is set by the vertical frame. If that frame isn't perpendicular to the X and Y axes, the verticals of the print won't be, either.

\n

To correct your Y axis tilt, you need to correct your vertical frame tilt. The simplest way to do this is likely to be installing an adjustable frame brace system (you can print all the parts except for a couple threaded rods). That will let you make a precise adjustment, print another test like the one in the question, and verify that it's accurate, before printing your "disk on a donut" actual part.

\n" }, { "Id": "16552", "CreationDate": "2021-06-17T13:51:05.610", "Body": "

I am looking to print an HTD Timing Belt pulley to be used in a laboratory setting that can get very cold for extended periods of time. By "very cold" I mean adjacent metal chambers get cooled with liquid nitrogen to lower than -200\u00a0\u00b0C. For the purposes of having a threshold temperature tolerance, assume that the metal chambers coming directly in contact with the pulley may get as cold as mK close to absolute zero.

\n

While it was my intention to print this pulley out of PLA, I am unsure whether or not it will be able to withstand negative temperatures of this magnitude or if it will become brittle--or something else will happen to the structure of the print when it experiences these temperatures. I am open to printing any other material if there are some materials that will hold up better than PLA for low temperatures. It is preferable for me to print this part instead of machine it for the sake of a deadline. I was also wondering if there is some infill pattern, infill density, or other structural print parameters that would help reinforce a printed part against becoming brittle when imposed to such low temperatures.

\n", "Title": "Lowest temperature PLA pulley print will withstand", "Tags": "|filament|filament-choice|temperature|", "Answer": "

You could try PETG. PETG labware works down to -70, and there is a video clip where a PET bottle filled with liquid nitrogen that withstands appreciable pressure.\nBut for whichever material is used, thermal cycling may be a problem.

\n" }, { "Id": "16566", "CreationDate": "2021-06-18T15:55:06.803", "Body": "

Is there any convention on how to mark an all metal hotend to distinguish it from a hotend with a PTFE tube in the heatbreak? Once assembled for a German Riprap 3D printer, they look exactly the same on the outside. I'm trying to decide on a way of marking them to indicate the difference.

\n

Hotend with heatsink\n\"enter

\n

Exposed part of heatbreak between heater block and heatsink\n\"enter

\n

Easy to see PTFE tube on nozzle side of heatbreak (all metal hotend has an all metal heatbreak with no PTFE)\n\"enter

\n

Even after removing heatsink, PTFE tube is difficult to see in heatbreak (all metal hotend has an all metal heatbreak with no PTFE)\n\"enter

\n

Nozzle end of all metal heatbreak\n\"enter

\n

Heatsink end of all metal heatbreak\n\"enter

\n", "Title": "Way to add identity to an all metal hot end", "Tags": "|hotend|all-metal-hotend|", "Answer": "

Usually, the neck of an all-metal heat break would be thinner (as it does not need to contain the PTFE tube, and making it overly thick would conduct heat unneccesarily). This may be observable when one loses track of what is where.\nIf one needs to see at a glance, I do not think there is a convention.

\n" }, { "Id": "16569", "CreationDate": "2021-06-18T20:23:43.510", "Body": "

I'd like to print a mouse shell from food safe PLA.

\n

I need it to be food safe, because sweat can dissolve any harmful chemical from it just like foods. I am not sure about how to finish it after sanding the result. I read that people use food safe epoxy to seal it before painting to get a smooth surface. I read that others seal it after painting to cover the paint, which is possibly not food safe.

\n

So I am not sure if I should seal it before or after painting or maybe both? I read that some people use shellac or even nail polish instead of epoxy to get something that they think is food safe. So the entire topic is starting to get somewhat chaotic for me. What materials should I use for sealing and painting and in which order?

\n", "Title": "How should I finish a food safe print?", "Tags": "|safety|material|food|", "Answer": "

If you're using food safe PLA, there are no "harmful chemicals" soluble in sweat. The food safety issues that remain are surface imperfections where bacteria or mold may grow. These are almost certainly not an issue unless you're actually eating from the object or otherwise putting it in your mouth/body. People touch similar objects and have them in contact with sweaty hands all the time - athletic equipment, bicycle handlebar grips, steering wheels, etc.

\n

For a user with serious medical issues affecting their immune system, this still may be an issue and you need to follow whatever instructions medical professionals have given about contact with objects that may be contaminated. But otherwise there is no need for any coating for a mouse shell except for aesthetics. You can use whatever you like, but be aware that, unlike PLA, most coatings do involve moderate to high levels of VOC (which are ideally down to non-harmful levels after it's finished setting/curing).

\n" }, { "Id": "16571", "CreationDate": "2021-06-18T22:17:27.613", "Body": "

Having started with an Ender 3, it just seemed natural to me that the heatbreak should not be load-bearing; Creality's stock hotend has 2 bolts holding the heat block to the heat sink, which of course waste some heating power and increase the cooling needed to avoid heat creep, but serve the important purpose of keeping the nozzle position rigid relative to the carriage and making it so you don't bend or snap the heatbreak when changing nozzles.

\n

Looking at hotends (especially all-metal ones) for a possible future printer build, I'm surprised to see that many (most?) don't have this property, and have the heatbreak playing a load-bearing role. This seems really undesirable. Only the Mosquito makes a point of doing this right, and supposedly has a patent on this or related design decisions. Is that really the case? Are there basic all-metal hotends that are designed to avoid making the heatbreak load-bearing that don't cost $150?

\n", "Title": "Hotends where heatbreak is not load-bearing?", "Tags": "|hotend|all-metal-hotend|", "Answer": "

You ask in general, not specifically for Ender, so since you mention the Mosquito, which has a characteristic shape and a size, the obvious alternative which doesn't cost that much is the Phaetus Dragon.

\n

It copies the idea of the Mosquito, but it is repackaged in a shape and size fully equivalent to standard v6 hotends so it's a drop-in replacement for any v6 hotend. You don't have one, but in general...

\n

Footnote: Slice Engineering got the patent in US, but not in Europe and China, yet. Also, in Europe their idea apparently was found as not original. Who knows...

\n" }, { "Id": "16572", "CreationDate": "2021-06-19T07:08:32.983", "Body": "

I am pretty sure that sanding makes a lot of microplastics, so it would be nice to collect the dust, and melt it to got a blob of plastic again instead of millions of tiny particles. What is the best way for collecting it, do you use any dust extractor, or are there different techniques like sanding wet surfaces and filtering it from water?

\n", "Title": "What is the best way to collect sanding dust?", "Tags": "|post-processing|safety|", "Answer": "

I decided to collect the dust and treat both PLA and ABS with chemicals to completely break them down. That solves the issue and I won't rely on false hopes that somehow it does not end up in the environment.

\n" }, { "Id": "16585", "CreationDate": "2021-06-21T23:14:21.703", "Body": "

I'm printing a Soma cube I found on Thingiverse -- it's a puzzle toy I remember from the early 1970s, and the kind of thing that ought to be really easy-peasy to print. Each of the seven pieces is made of four cubes (or three for one piece) joined face to face in every possible combination, so that they can be assembled to make a single 3x3 cube (the claim in the day was, in literally tens of millions of ways -- though many of those were reflections or rotations of others).

\n

Problem is, in order for the seven pieces to fit together correctly to make the 3x3 cube, they have to be flat and square on all faces, and dimensionally accurate.

\n

I have an Ender 3 that's just a few weeks old (first print was May 8th to 9th of this year, also my own first 3D print). I'm printing these parts in gray Amazon Basics PLA at 200\u00a0\u00b0C, with the Creality coated glass build surface at 55\u00a0\u00b0C. I'm printing on a raft; the first layer goes down well, but over the course of the first forty or so layers, I get this:

\n

\"Raft

\n

I presume this is due to the upper layers shrinking, though the horizontal dimensions look fine at the level where I stopped the print. I have Cura Slicer 4.9.1 set to print with no part cooling for initial layer, increasing to 100\u00a0% at layer 4. When I removed this part (immediately after stopping the print, so the bed was still in the low 50s), it was still stuck fairly well -- except where it had lifted.

\n

What's causing this warping/lifting?

\n", "Title": "What's causing my PLA prints to lift like this?", "Tags": "|creality-ender-3|adhesion|warping|cooling|", "Answer": "

Increase bed adhesion and/or decrease temperature gradient over the part. As the object builds, and it gets further away from the heated build plate, the plastic gets colder and shrinks. The bottom stays warm and does not shrink, this makes the part want to bow/warp. At the plastic\u2019s glass temperature, where it transitions from liquid to solid (PLA is 60\u00a0\u00b0C), it has little shrink. That\u2019s why heated beds stick to the plastic, but as it cools the object pops off-it shrinks and breaks the bond.

\n

Other answers have good advice-clean oils off the bed, ensure the first layer is squished into the bed. Ensure the bed is level, so the first layer is squished down uniformly. I would disagree that an adhesive is not necessary. I use a glue stick and it works, where glass alone couldn\u2019t keep my corners stuck down. It is common to run a 60\u00a0\u00b0C bed for the first PLA layer and cool it off 5 degrees, as a little cooler can still adhere but have less tendency for the part to \u201celephant foot\u201d from the bottom layers being too warm and gradually deforming. Your 55\u00a0\u00b0C seems fine, you could still cool it off to 50\u00a0\u00b0C. I don\u2019t think there is any benefit to running it extra hot (unless the glass surface runs cooler than the bed thermistor reports) as it will be more prone to elephant foot the part.

\n

What I felt compelled to add to the other answers is that if the upper layers aren\u2019t as cold, i.e. they are closer in temperature to the bed layers, less of a temperature gradient over the part, they won\u2019t shrink as much and will have less strength to warp and peel up the corners. I\u2019ve used a couple of 150\u00a0W heat lamps, shining at the bed, and have been able to print big flat objects without a brim. It especially helps in the winter. The cooling fans, while they help the print quality, increase the coldness/shrink of the upper layers. I think it would be worthwhile to try less cooling fan, so long as the object doesn\u2019t deform/sag. Often the same corner will lift; if something makes it cooler on that corner, like the cooling fan blows on it longer, the heat break fan blows on it, or it\u2019s draftier. If so, a different part orientation could possibly help.

\n

One nuclear option you can try on large flat prints, if you just can\u2019t keep the brim or raft stuck down with adhesive and whatnot, is to pause a few layers in, and tape the brim down with painter's tape.

\n" }, { "Id": "16604", "CreationDate": "2021-06-23T13:21:36.157", "Body": "

A frequently discussed update to a 3D printer is the so-called leveling sensor; this sensor can be inductive, capacitive, optical or mechanical (i.e pressure or physically touching, like a BLTouch).

\n

It is supposed to level your bed, providing improved first layer adherence and thus less failed prints.

\n

How does this automatic bed leveling work?

\n", "Title": "Automatic Bed Leveling (ABL) with a sensor (BLTouch, inductive, capacitive), how does it work?", "Tags": "|bltouch|knowledgebase|inductive-sensor|automatic-bed-leveling|", "Answer": "

The issues/questions raised about the automatic bed leveling using e.g. a BLTouch probe at this stack require a more in depth description of the automatic leveling functionality in general, this answers aims on describing how this works.

\n
\n

Definition of "bed leveling"

\n

Let us begin with saying that such sensors systems alone will never level your bed, this is a misconception that is easily concluded from the terms being used (terms like Auto Bed Leveling, ABL). Using such sensors will not physically change the "level" of your bed.1) Furthermore, the process of leveling your build surface/heated bed is commonly known as tramming.

\n
\n

Tramming is the process of adjusting the build surface to be parallel to the X-axis and Y-axis of the 3D printer.

\n
\n

But, when do you need such a functionality2)? What if the bed surface isn't completely flat/square? Tramming such a surface is difficult and that surface will not be perpendicular to the X-Y plane of the nozzle... Why a bed is not perfectly flat can be caused by the manufacturing process, handling (during shipment, dropping tools on the surface, etc.), the way the bed is mounted and thermal stresses of non-uniform heating.

\n

Basics

\n

Many printers use endstops to determine the origin of the printer, these aren't essential for the printer, but very useful for the operator. Endstops allow for setting the reference from a known position to the origin of the printer. In fact, the sophisticated leveling sensors are nothing more than the simple Z-endstop except that the sensor is mounted onto the print head, it allows for probing the bed in more positions than an endstop limit switch mounted on the frame\u2019s Z-axis.

\n

In order to be able to print consistently, you should provide a bed that is properly trammed (to aid adhesion/prevent objects to not stick to the build plate). But, if for some reason, the surface contains imperfections, the normal tramming of the build surface using just the Z-endstop and the leveling screws might not be sufficient.

\n

What does this so-called "leveling" do?

\n

Using the ABL features from firmware will enable the use of a sensor to scan the surface of the build surface/heated bed to create a digital representation of the topology of the surface. This scanned surface includes the imperfections of the surface (this is depending on the option to process the measurement points of the scanning process, the more probing points the better the geometry can be mapped). E.g. just scanning 3 points will only result in a tilted plane (e.g. Ultimaker), but, scanning multiple points in a grid style process, the surface can be mapped to a complex surface describing the shape. So, if a surface isn't fully flat/square, the scanned surface provides a digital copy of the shape of the build plate.

\n

How is the scanned surface applied?

\n

Note that this scanning of the surface is generally done at a distance from the actual nozzle (except for pressure sensor systems where the nozzle is used as the probe), the images below show this for a BLTouch sensor and an inductive/capacitive sensor. For an inductive sensor this is the distance of detecting metal (minus the distance to the nozzle), for a BLTouch this is based on the triggering point of the extendable probe, see this answer (the image below depict the terminology). The operator of the printer needs to determine at which distance this bed has been scanned. This is commonly known as determining the Z-offset. The Z-offset can be set using a terminal or through the display. Setting this Z-offset is described in detail in "Specifying Z offset in Marlin firmware".

\n

Once the shape of the bed is captured (G-code command G29) and stored and the correct distance (Z-offset) set with M851 Zx.xx (or through the display), the use of the shape needs to be activated in the starting G-code section of the G-code file of the print.

\n

The nozzle will now be following the bed shape to improve the bed adhesion, this compensation is by default set to be smeared out in 10\u00a0mm, but can be disabled or changed to any other value using M420 Zxx. The nozzle will print in the X-Y plane without compensation after the compensation distance. If you do not tram the bed correctly, the bottom of that print object will thus not be square to the height! See question "Does auto leveling result in sheared prints?".

\n
\n

\"BLTouch

\n

\"Inductive/capacitive

\n
\n

1) Do note that actual leveling the bed is possible in some firmware flavors, but this is only possible if more individual controlled Z-steppers are used to move the bed/nozzle assembly!

\n

2) Please note that firmware development has progressed over the years, e.g. using a recent version of Marlin, you can change the firmware to use UBL (see question What is ABL or UBL? Is this the same?). With UBL you technically do not need a leveling sensor, but, this is a laborious process to do this manually.

\n" }, { "Id": "16612", "CreationDate": "2021-06-23T18:10:05.277", "Body": "

The background and printer

\n

I have an Ender 3 Pro that I have made all sorts of modifications to, the important ones being:

\n\n

The problem

\n

I keep getting the error message "SD Init Fail". Sometimes it's right away when I plug the SD card in, sometimes its 5 hours into a print. The print will just suddenly stop and that error will pop up.

\n

Things I've tried

\n

I had previously asked this question here and most of the suggestions were to "take it out and plug it back in again" or to get a new sd card. I've tried it with new, name-brand (Sandisk) cards, and it doesn't work any better/worse than the card that came with the Ender. I've seen various suggestions around the internet (like on Marlin's website) that adjusting the SPI speed can possibly help with "Volume Init" errors, which I assume to mean the same thing:\n\"enter

\n

I have tried the firmware at both normal speed and half and quarter, haven't tried eighth yet. I didn't notice any different behavior for any of them.

\n

I'm at a loss! Help! I'm eventually probably going to jump to Octoprint via a Raspberry Pi, but given the printers location in an area without wifi, is there anything I can do in the meantime? Any other firmware tips? Or suggestions?

\n

Do you think bad hardware could explain a print that goes for a few hours and then errors? I.e. is it reading the SD card during the course of the entire print (its never done?) and then the connection could fail after a few hours of being good?

\n", "Title": "\"SD Init Fail\" on Ender 3 Pro with BTT E3 RRF v1.1", "Tags": "|firmware|sd|", "Answer": "

This is happening with multiple SD cards that work in other ports (your computer, for instance): that tells me the problem must be in the mainboard hardware or firmware.

\n

You're using the stock firmware for that board, with the BLTouch enabled but otherwise original -- assuming you only had to change a compiler directive to enable BLTouch, that pretty strongly points to hardware. And the hardware that's likely at fault here is the SD port on the mainboard.

\n

It's hard to verify, hard to repair (aside from simply replacing the board with another of the hardware revision), but not impossible. With the right tools, you could connect a second SD card port parallel to the existing one (hook jumpers from an oscilloscope?), run the board from the external SD card, and reproduce the error by momentarily disconnecting one of the jumpers.

\n

Repairing the problem might only require reflowing solder connections on the board for the SD port wires, or it might require replacing the port itself.

\n

One thing to check before you do all of that, however: clean the contacts in the port. With power off (of course!) slip a utility knife blade or similar with a sheet of printer paper wrapped over it into the port, work it in and out a couple times to scrub the paper over the spring contacts, then pull it out and carefully vacuum the port.

\n

If that fixes it, good to go! If not, all it's cost you is a few minutes before trying the more drastic stuff.

\n
\n

Do you think bad hardware could explain a print that goes for a few hours and then errors? I.e. is it reading the SD card during the course of the entire print (its never done?) and then the connection could fail after a few hours of being good?

\n
\n

The movement of the machine frame while printing infill or fine detail may jostle the SD card or port when the firmware is trying to read the next line (or batch of lines) of G-code from it.

\n

If you do determine that the SD socket is at fault due to connection problems, it may be worth considering installing an off-frame mainboard housing (I've seen these in printable form for Ender 3 class machines, though if this is your only printer you might have to buy one, since they'll be 10-16 hour prints) to insulate the mainboard from movement induced by bed and hot end/extruder motion. Doing that would be easier than repairing or replacing the SD socket. This has the downside that it requires additional space near the machine, but the upside is that it may solve your "SD Init Fail" problems.

\n" }, { "Id": "16613", "CreationDate": "2021-06-23T21:29:34.073", "Body": "

My 3D printer has got a clog, and i think it is because the nozzle hole has just recently started to only screw in crooked. I\u2019m not sure how this happened. I have tried with multiple nozzles, but they all screw in crooked. Is there a way to fix this? Or is my only option to get a new hotend.

\n

My 3d printer is an Ender 3

\n

\"enter

\n", "Title": "3D printer clog. Nozzle keeps getting screwed in crooked", "Tags": "|filament|hotend|nozzle|", "Answer": "

You should be able to purchase only the heater block for your hot end. The nozzle should thread in by hand without undue force. Having to use a wrench at the outset is an indication of a problem which should have stopped the attempt.

\n

There are many sources for the "heater block for Ender 3" when used in a search. One such result is an Amazon seller of three pieces for US$11.99

\n

\"amazon

\n

Image via linked site.

\n" }, { "Id": "16629", "CreationDate": "2021-06-25T16:53:16.323", "Body": "

A Bowden tube extruder (like the stock one on an Ender 3) is known to have issues with printing the most flexible TPU, and with filaments that (either due to composition or condition) don't take well to too much retraction -- though the latter can be ameliorated somewhat with slicer settings. Direct drive extruders, on the other hand, by reducing the extruder to hot end distance to the practical minimum, greatly reduce the amount of retraction needed as well as the effect of filament compression and stretching.

\n

One potential down side I'm aware of is that putting the extruder stepper and drive on the X carriage adds mass that the X drive motor has to both accelerate and decelerate; this could in theory have an effect on print quality, increasing ringing and overrun on the X axis (though this isn't generally a problem with the steppers used on the Ender 3 and similar printers).

\n

What other reasons might there be to prefer a Bowden tube over direct drive?

\n", "Title": "What reasons are there not to convert Ender 3 to direct drive extrusion?", "Tags": "|creality-ender-3|bowden|direct-drive|", "Answer": "

There are really no reasons not to convert to direct drive, provided you use a good one. Many of the direct drive kits, especially the ones that reuse the existing giant NEMA-17 stepper, are not very good and have tradeoffs due to excessive mass on the toolhead, imbalanced mass, interference with frame reducing print volume, etc. The modern way to do direct drive is with a "pancake" stepper far smaller and lighter, and gearing between 3:1 and 6:1 reduction to get the needed torque - look at the Orbiter, Sherpa Mini, etc. - or even a remote direct drive.

\n

Teaching Tech recently posted a video on this very topic, titled Why direct drive is not automatically better than bowden tube but then pretty much concluding that all the old reasons not to go direct drive no longer apply.

\n" }, { "Id": "16659", "CreationDate": "2021-07-02T23:43:55.787", "Body": "

I've got some sliced models that represent the right-side arms and legs of a robot. I'm really happy with how they printed, so now I'd like to print the left-side arms and legs.

\n

I was thinking it would be pretty trivial to parse the G-code file using Python and change the value of all the Xn commands from n to 2*h - n, where h is in the middle of the bed, say 110 or 120\u00a0mm for an Ender 3.

\n

Before I fire up my favorite IDE, are there any major gotchas I might encounter from such a na\u00efve approach to mirroring the G-code like this? I originally sliced in Cura 4.9.1.

\n", "Title": "Reverse the X-axis of a G-code file", "Tags": "|g-code|python|", "Answer": "

Usage: python3 [filename.gcode] [bed-width]

\n
#!/usr/bin/env python3\n\nimport re\nfrom sys import argv\n\n#first arg is the file, second arg is the bed x width\nifilename = argv[1];\nofilename = ifilename.replace('.gcode', '-reversed.gcode');\n\nnew_data =[]\nBEDX = float(argv[2])\nh = BEDX # Absolute Positioning is default\n\nwith open(ifilename, 'r') as ifp:\n    for d in ifp:\n        new_d = ''\n        tokens = d.split()\n        if len(tokens) == 0:\n            pass\n        else:\n            if tokens[0] == 'G91': # Relative Positioning\n                h = 0\n                new_d += d\n            elif tokens[0] == 'G90':# Absolute Positioning\n                h = BEDX\n                new_d += d\n            elif tokens[0] == 'G1':\n                for t in tokens:\n                    if t.startswith('X'):\n                        if len(t) > 1:\n                            x = h - float(t[1:])\n                            t = 'X'+f'{x:.3f}'\n                    new_d += t+' '\n            else:\n                new_d += d\n\n        new_data.append(new_d)\n\nwith open(ofilename, 'w') as ofp:\n    for d in new_data:\n        ofp.write(d+'\\n') # how does writelines not support a line separator?\n
\n

For those that have a problem with the script above, I think this fixes it.

\n

Also there seems to be a bug with not writting G90 or G91 whenever it finds it.

\n" }, { "Id": "16663", "CreationDate": "2021-07-03T08:18:12.077", "Body": "

As you can see the left side of the edge came out jagged. The right side came out just fine. What happened to the left side?

\n

I am using Ender 3 + BLTouch using Sunlu PLA+ with 200\u00a0\u00b0C hotend.

\n

\"Left

\n

\"Right

\n

Here are my retraction settings (if that matters):

\n

\"Retraction

\n", "Title": "Jagged edge on one side and not on the other side", "Tags": "|creality-ender-3|print-quality|prusaslicer|", "Answer": "

This was happened because I don't use support on the slope. After I use support the jagged edge is gone.

\n" }, { "Id": "16676", "CreationDate": "2021-07-06T12:53:36.570", "Body": "

I have a Prusa i3 MK3S with the MMU on it. At my work, I'm creating a display case to advertise/show off the 3D printing we can do. I was thinking about putting the Prusa in the display case and having it "run" for the two weeks we're in there. I was thinking it'd be cool if I could have the printer pretend to print, i.e. move the axes around, move the bed every so often, etc., but without the need to worry about if a print was messing up (it'll be in a locked cabinet I may not notice more than once a day).

\n

I'd just slice a long print but then the Prusa will wonder where the material is. Any thoughts on how to create that custom G-code?

\n", "Title": "G-code to make printer move around without actually printing", "Tags": "|g-code|", "Answer": "

The simple way to do this is to slice a very long print (near maximum volume should run to 24 hours or longer, anyway), but set your slicer to nozzle temp of 0\u00a0\u00b0C and extrusion flow rate of zero. Back the filament out of the hot end (in case you want to change before resuming actual printing), but you can leave it in the extruder and Bowden tube so everything looks "normal". You can also optionally set the slicer to a lower print speed to draw out the movement, though this may give a distorted impression of how fast or slow the machine is.

\n

Do test before you set up the exhibit, as it's easy to miss something when doing special settings like this...

\n" }, { "Id": "16683", "CreationDate": "2021-07-08T13:30:55.723", "Body": "

Some time ago, I bought an Elegoo Mars 2 Pro, and I really liked the product. This is my first resin printer and I am very satisfied.

\n

What has been bothering me a little is in relation to the slicer. When I slice an item, Chitubox shows 3 hour/20\u00a0g (consumption), but this does not converge, when I print the final result is about 40\u00a0% higher than expected, this in both cases (time and resin consumption). I understand that resin consumption can be a little more relative, but time should converge.

\n

My settings are the defaults provided by Chitubox, changing only the resin settings I use: 8 initial layers in 50\u00a0s and 2.5\u00a0s for other layers.

\n

Is there something I'm doing wrong? Should these values converge with reality?

\n", "Title": "Chitubox estimated time very away from the real time", "Tags": "|resin|sla|dlp|chitu|", "Answer": "

It's quite easy to estimate "close enough".

\n

Compared to FDM printers, movement is way more simple, and there are only 4 different types of movements during a normal print:

\n
    \n
  1. Lowering the print platform at the beginning of print (we can't know the starting position, so time needed cannot be really estimated)
  2. \n
  3. Printing bottom layers with longer exposure times. Speeds can be set differently from normal layers.
  4. \n
  5. Printing normal layers. Just like bottom, move build platform up and back down (slightly higher than last time), and do exposure with constant time.
  6. \n
  7. After the print is complete, lift the print up.
  8. \n
\n

My last print job was 2249 layers, of which 4 were bottom layers. As you can see, everything else pales in comparison of the time needed for "normal" layers. So we can pretty much ignore them.

\n

But the estimate is wrong. Luckily it's very easy to fix closer to the reality. This page shows how to do it, search for "Layer Print Time Compensation"

\n

Since print time is so predictable, it's only a single multiplier value. The easiest way to set this up is to slice a project, write down estimated time AND amount of layers. Then when the print is done record the time it actually took, open the compensation window (gear icon on the right side of Layer Print Time Compensation), and enter all the values there. It will calculate the multiplier for you. Also note that this setting is per-profile, it will be different for different exposure times, movement speeds and layer thickness.

\n" }, { "Id": "16684", "CreationDate": "2021-07-09T08:28:56.900", "Body": "

This might be an odd question, as rather than being disappointed by the raft material that Cura produces, I actually love it.

\n

It prints quickly, is sturdy yet flexible...

\n

I was wondering whether anyone knew of a way to make the walls / base of a print use the raft patterning? Is there a strategy for using this kind of material in CAD apps, or has anyone tried implementing a plugin for Cura to turn all walls into rafts?

\n", "Title": "Use raft pattern instead of wall", "Tags": "|ultimaker-cura|rafts|", "Answer": "

Cura, like many slicers, allows one to set the number of passes for top, bottom and sides. By setting these values to zero, your model becomes completely and only infill. The pattern selection for infill allows you to choose the design which would appeal to you and possibly match that of the raft.

\n

I've just now imported a mostly monolithic STL file and tested the above. By varying the percentage of infill, various patterns result. Low infill percentage values make for a mostly empty print, while the higher numbers provide attractive geometric results.

\n

Of course, one would want the original model to have sufficient wall thickness to create the objective.

\n" }, { "Id": "16686", "CreationDate": "2021-07-09T14:25:26.020", "Body": "

I just noticed the other day that my Ender 3 is printing stuff a lot slower than it can, and realized when I was slicing a part that it's because the default "Wall Print Speed" is half the "Print Speed" even though "Infill Speed" is full speed. I can enter higher speeds, of course, but then Cura puts up a little notification that "this value is normally calculated, but it has been entered instead" and offers to put it back to the calculated default -- which is half the "Print Speed."

\n

I presume there are good reasons (print quality, underextrusion, etc.) for this -- what are they?

\n

I guess it's relevant to note that I have a 4.2.2 mainboard and currently run Creality's version of 2.0.4-1.0.1 Marlin, the newest they offer for this board version without BLTouch.

\n", "Title": "Why does Cura default to printing walls so slowly?", "Tags": "|creality-ender-3|print-quality|ultimaker-cura|speed|", "Answer": "

Cura has a lot of bad defaults, but also a lot of defaults that are chosen to let you get acceptable prints out of a really bad printer, a miscalibrated printer, or a printer with bad firmware.

\n

Printing at anything above very low speeds requires compensation for the differing pressure needed to move material through the nozzle at different flows. The printer firmware feature to do this compensation is known as "pressure advance" or "linear advance", and is absent in stock firmware on most commercially produced printers, especially older ones. Without it, you'll get bulging corners and underextruded middles of your walls. This matters most on the outer wall, which determines the dimensional accuracy and visual quality of your print; errors on the inner walls or infill will mostly be hidden and tend to even out, especially if the inner walls are printed after the outer ones so that the extrusion is constrained against the already-existing outer wall (but here's where Cura's wrong defaults come in: it defaults to printing outer wall last, which replicates the inner wall errors onto the outer wall!).

\n

On a printer without pressure advance compensation, you can use either very low speeds (30 mm/s or preferably even lower) to get a decent outer wall, or you can use high acceleration and cornering velocity (but: most stock firmware doesn't handler cornering velocity correctly either) so that the corners aren't printed significantly slower than the rest of the wall, but at high acceleration/fast cornering you'll get ringing (ghosting) artifacts. These can also be eliminated with better tuning of your printer/firmware, but the point of Cura's defaults is giving decent output on a printer where that work hasn't been done.

\n

If your printer has a version of Marlin (at least 2.0) with linear advance and junction deviation, or if you upgrade to that, or if you switch to Klipper, and then you tune these to match your printer, there's no reason to keep the outer wall speed low. With Klipper you can take the acceleration way up too (using input shaper to eliminate the ringing). This can give you prints many times faster than Cura's defaults. Even if you don't, you can still turn the speed and acceleration way up, but you'll get ugly and possibly structurally weak prints as a result.

\n" }, { "Id": "16696", "CreationDate": "2021-07-09T20:52:05.313", "Body": "

I have three stepper motors each with its own stepper driver. Two of those motors have the same lead screw, while the third has a ball screw.

\n

I need to be able to set the E-steps for the ball screw differently than the two lead screws.

\n

Can this even be done in Marlin 2.0?

\n", "Title": "Separate Z-stepper E-steps", "Tags": "|marlin|", "Answer": "

Unfortunately, It does not appear that any firmware currently available supports this feature, however you can request the feature on the marlin GitHub repository

\n" }, { "Id": "16708", "CreationDate": "2021-07-11T10:25:22.123", "Body": "

I have a set of 3MF files to enclose a MacroPad. They have openings for screws that go through top then PCB then into bottom, but I've been unable to figure out the dimensions of the screws needed and Google keeps trying to encourage me to design new screws, but is not returning results for determining the size and length of the screws needed.

\n

I've tried opening the file in a number of applications (OpenSCAD, LibreCAD, etc.) and am unable to figure this out (though this could just be due to inexperience with more than changing the size of existing 3D designs).

\n

Does anyone know if what I'm trying to do is possible, and if so, where can I read about how to do it, or if it's stunningly obvious, what size am I looking for? If it matters I'm using Linux if it matters.

\n

The MacroPad 3MF files are found here.

\n", "Title": "Determine screw size needed from 3MF file(s)", "Tags": "|3d-models|", "Answer": "

Meshmixer (free, multi-platform) supports and imports .3MF file format. There's something amiss with the file named Macro Keyboard RC Ver2.1 Top MiniCutout. It appears that the object is created from two improperly overlapped objects that have been merged. The result is a hole that isn't quite a hole:

\n

\"object

\n

The other files have holes that appear to be normal. Meshmixer can measure distances but that remains as an exercise to the reader, as it's a royal pain (opinion) to get results with that feature of the program.

\n

\""normal"

\n

The above image is of a different part (note image browser in screen cap) with a normal appearing hole.

\n

The curious thing here is that Fusion 360 (free hobbyist license) also supports the file format and when imported, there is no anomaly with the pictured holes. This would imply that Meshmixer doesn't handle the format as correctly as it should. Also the measure feature of F360 is easier to use, resulting in a hole measurement of about 3.2 mm diameter.

\n

The larger hole measures to 5.0 mm diameter.

\n

\"Fusion

\n

All images created by me via respective programs with files provided by OP.

\n" }, { "Id": "16716", "CreationDate": "2021-07-12T12:27:33.367", "Body": "

I found my Ender 3 was skipping extruder steps a lot, mainly while printing infill (tri-hex at 30%), to the point I saw several layers of infill with spotty extrusion or nothing adhered at all.

\n

When I pulled the filament, I had to use pliers to get enough grip to back it out of the hot end.\nI disassembled the hot end and found melted filament in the end of the Bowden tube, though I'm not sure that indicates heat creep, because I had to dismount the fan in order to reach the heat block and nozzle with the wrenches, and then preheat to get the nozzle out (meaning the heat sink got a lot hotter than it normally would with the constant blast of its fan).

\n

Since the nozzle was full of melted plastic as well, I simply replaced it, made sure the heat break was clear by pushing the Bowden tube through it (and removed all melted plastic from the end of the Bowden tube), then reassembled the hot end with a brand new 0.4 mm nozzle. I then started a new print, gratified initially by the clean, even extrusion.

\n

Until it stopped printing infill about 10 layers into the print.

\n

At that point, I simply aborted the print and turned everything off, as I didn't have time to deal with another disassembly cycle (Sunday evening and had to be up early for work today).

\n

What should I look for in trying to troubleshoot this issue with the nozzle or hot end clogging repeatedly?

\n

I'm printing Amazon Basics PLA, a spool that was purchased the first week in May but only unsealed two days ago; my nozzle is set to 200\u00a0\u00b0C and the coated glass bed (now using a glue stick wash for adhesion assistance) at 55\u00a0\u00b0C. My nozzle clearance is set by homing, jogging the Z axis up by 0.075\u00a0mm, and leveling snug on a 0.08\u00a0mm feeler gauge (set so I can barely push the feeler under the nozzle and have palpable drag when in place, done with bed and nozzle preheated); this should give clearance between 0 and .005\u00a0mm. My first layer (standard 0.2\u00a0mm thickness) was perfect, no extruder step skipping and nice, even line width; this problem started only when the machine began to print infill -- infill prints faster than walls and top/bottom, but I'm not sure this is related to that, as I could see skipping on the walls on the second or third infill layer. This part was printing with 3 line walls and 30% tri-hex infill.

\n

The Bowden tube was replaced when I installed a new hot end two weeks and ten or so prints ago (which included a new nozzle, the same one that I was using until yesterday); I made the cut in the PTFE with a tubing cutter (the kind with a wheel that revolves around the tube), so I'm confident it's square, and I ensured the tubing was butted firmly on the nozzle and the lock ring installed on the push-in coupler.

\n", "Title": "What makes a new nozzle clog before I can finish the print?", "Tags": "|creality-ender-3|extrusion|heat-creep|", "Answer": "

I sounds like you are on the edge of almost heat creep if you start printing before the extruder cools off. See if letting the extruder cool down to near room temperature solves the problem.

\n" }, { "Id": "16718", "CreationDate": "2021-07-12T15:50:29.277", "Body": "

Some people have figured out how to take raw LiDAR data and after going through multiple steps (using LAS tools, converting to digital elevation model (DEM), converting to an STL) getting an STL file that they can then slice and print.

\n

Could you write a program that cuts out all of those intermediate steps and converts raw LiDAR data directly to an STL that can be printed? Could you even cut out the need for slicers and just go straight to a G-code file?

\n

Is this even possible?

\n

From 3D Printing Models derived from Lidar Data:

\n
\n
    \n
  1. Retrieve Lidar Data
  2. \n
  3. Process Lidar Data
  4. \n
  5. Create a DSM
  6. \n
  7. Export the DSM into a .STL
  8. \n
  9. Process for 3D Printing
  10. \n
  11. 3D Print!
  12. \n
\n
\n", "Title": "Building a program to convert LiDAR data directly to an STL or directly to G-code", "Tags": "|g-code|stl|scanning|file-formats|", "Answer": "

The answer is highly depending on the programming skills of the programmer, but in theory, if all pieces of software exist, they can either be tied together in a workflow process (automated) or directly programmed into a new tool.

\n" }, { "Id": "16725", "CreationDate": "2021-07-13T13:15:20.677", "Body": "

I have a Prusa i3 MK3 or maybe it was upgraded to a i3 MK3S.

\n

How can I figure out?

\n

The description of the upgrade kit talks about

\n\n

I'd like to figure that out without taking the printer apart.

\n", "Title": "Distinguish i3 MK3 from i3 MK3S", "Tags": "|prusa-i3|", "Answer": "

On the LCD, the MK3 will show Original Prusa MK3 OK, while the MK3S/+ will show Original Prusa MK3S OK. This will only work if you have the stock firmware installed.

\n" }, { "Id": "16729", "CreationDate": "2021-07-13T18:03:08.173", "Body": "

I have a "classic" home built 3D printer, Arduino Mega, RAMPS 1.4 with Marlin 2.0.

\n

It worked, was put in storage, and now I am trying to get it running again.

\n

During all of the testing phases, the PID autotune, etc., the temperature of the hotend is stable.

\n

When printing, for a few layers (4-5), the temperature is OK, despite the part fan coming on and off. However, after 3-4\u00a0mm of printing the temperature just rises and rises and rises, and after 275\u00a0\u00b0C thermal protection kicks in...

\n

MOSFETs have additional cooling and a fan over them.

\n

I'm out of ideas... Help!!!

\n

My config files and G-code are here

\n

Using PrusaSlicer.

\n

Why do I see a temperature rise on the display? Last time I unplugged the printer and left it on USB power then gave it power again. During USB power temperature was steadily going down, as soon I plugged the main cable it resumed printing and the temperature rising again.

\n

I did another test. 20mm above bed started printing, and got same result. I captured on video entire process Hot end to orbit. Around 2 min 30 sec the show begins.

\n", "Title": "Hotend temperature goes to infinity after few layers", "Tags": "|marlin|hotend|ramps-1.4|reprap|", "Answer": "

So, the problem is in fact of a hardware nature. By replacing RAMPS 1.4, the problem went away.

\n

Arduino, firmware, G-code... everything is the same, only RAMPS was replaced.

\n

I'm going to find a suitable MOSFET replacement.

\n" }, { "Id": "16736", "CreationDate": "2021-07-15T15:27:46.573", "Body": "

I have tried to print Baby Groot , but every time I print the head, there are always layer shifts like the photos below. I have printed three heads and the layer shifts happened all in the same place.

\n

\"enter

\n

\"enter

\n

I tried to print something else that is the (more or less) same height and the layer shifting also happened.

\n

\"enter

\n

I was thinking maybe because z-rod bent. But apparently the layer shift happened in different height.

\n

\"enter

\n

What is my problem and how to solve this ? I am using Ender 3 with BLTouch.

\n", "Title": "Layer Shifting When Printing Tall Prints (Consistent Per-Model)", "Tags": "|creality-ender-3|prusaslicer|layer-shifting|", "Answer": "

What I have done:

\n\n

All of those don't solve this issue. Then I realized that while my z motor rotates there were some occasions where the rod wasn't rotates. I changed the hex bolts and tighten them up properly to solve this issue.

\n" }, { "Id": "16750", "CreationDate": "2021-07-17T14:12:07.403", "Body": "

I\u2019ve been using Thingiverse pretty heavily since I started printing, and I frequently come across the same text block in unrelated models. Am I missing the joke or a hidden meaning in the frequent reuse of the following content?

\n
\n

Designed up this dealer button to use in home games or have just as something laying around for any poker fan. It was inspired by a dealer button in Pokerstars VR.

\n

Printing:

\n

Ring - Infil to add weight, supports needed for spade inset. Letter embossing doesn't need it.
\nSpade - Infil to add weight, no supports
\nLetters - Print flat, already scaled to fit nicely in the embossing
\nStar - Infil to add weight.

\n

Full print with 4x4x4 walls/top/bottom and 38% infil came to about 48g in weight.

\n

Glue in lettering and spade. Star may need minor sanding but should push fit into the space tightly and not need anything else to secure it.

\n
\n

Here is an example of what I mean: Voronoi bear. Just curious if anyone knows why that\u2019s used as a pseudo default description, or if I should avoid those files for any reason I\u2019m unaware of.

\n

Here\u2019s another example: cats

\n", "Title": "Thingiverse dealer button text block", "Tags": "|thingiverse|", "Answer": "

The "dealer button" text is the default text that Thingiverse inserts when you create a new Thing entry. If you get in a hurry and hit "Publish Thing" before you edit that, it'll go up for everyone to see with the "dealer button" default text still in place. Many times, when you see that, you'll also see the defaults still in place for machine specs, filament type, etc.

\n

In other words, it's just a user error, especially prevalent on "quickie" uploads that aren't intended for a wide audience or long term preservation. Based on comments, this may also be a relatively recent change at Thingiverse; I've only uploaded designs there over the past few months, and I've seen it every time over that time frame (since about April 2021).

\n" }, { "Id": "16766", "CreationDate": "2021-07-19T21:01:33.197", "Body": "

A few months ago, I flashed my Ender 3 V2 with JyersUI fork of the Marlin firmware and I like the options and settings that are available with this firmware. The author has put out another update (v1.3.4) and was curious if all my settings will be erased.

\n

Does flashing the firmware erase all settings on the Ender 3 V2? If so, what settings should I record to update the new firmware once installed?

\n", "Title": "Does updating the firmware on an Ender 3 V2 erase its previous settings?", "Tags": "|creality-ender-3|marlin|", "Answer": "

No, settings are stored in memory (or the equivalent storage location).

\n

When a new firmware is uploaded, the values set in the newly configured firmware can be set by Restore Defaults from the Marlin printer LCD the user interface. Alternatively, you could print a G-code file containing M502 and M500, or send them over using a console.

\n
\n

M502
\nReset all configurable settings to their factory defaults.

\n

To also reset settings in EEPROM, follow with M500.

\n
\n
\n

M500
\nSave all configurable settings to EEPROM

\n
\n

See M502 will reset all configurable settings to their "factory defaults", which settings are those? for more information.

\n

This answer hints to a similar description:

\n
\n

Installing firmware does not by itself alter the EEPROM, so these settings needed to be seeded into SRAM via M502 and then saved into EEPROM via M500.

\n
\n

Please note that the EEPROM on 32-bit boards are emulated. There are several ways to emulate EEPROM storage via flash storage in the ARM processor or using an SD card. There is a solution for an actual EEPROM, but that requires buying an EEPROM chip and breakout board on the I2C connector if it has such an output port. Selection of which is done in the Configuration_adv.h config file.

\n" }, { "Id": "16767", "CreationDate": "2021-07-19T23:21:24.307", "Body": "

I\u2019m using the Ender 3 with the Cura slicer and I\u2019ve had success in doing the same print but recently the layers in the middle have smushed out and I don\u2019t know the reason why.

\n

I\u2019ve tried reprinting the piece and the same problem occurs in the same area. I\u2019ve yet to try to let the part finish because I don\u2019t really want to waste filament so whenever I see the problem I stop the print. I tried reslicing the part but the problem still happens.

\n

Here\u2019s the smushed print:

\n

\"Image

\n

And here\u2019s what it\u2019s supposed to look like:

\n

\"Image

\n

Edit: Should also mention that I\u2019m fairly new to 3D printing

\n", "Title": "Layers on printed part have started smushing together mid-print and I don\u2019t know why", "Tags": "|creality-ender-3|", "Answer": "

It looks like a problem with the Z axis. It looks like it isn\u2019t lifting the Z as much as it thinks it is, so each layer gets over extruded because it\u2019s trying to pack too much plastic in a shortened height. I would try cleaning and lubricating the Z rod(s) (with grease, not oil) and run the z axis up all the way and back down a few times, and watch for skipping steps or roughness in the travel.

\n

Look for obstructions to the little wheels on the Z gantry as they roll up the frame posts. You could undo the Z gantry from the Z threaded rod. Be sure to support it on a box or something. Then lift the axis up and down by hand and feel if it\u2019s smooth. A lot of problems with motion on 3D printer be revealed if you can work the motion by hand.

\n" }, { "Id": "16791", "CreationDate": "2021-07-24T16:56:44.953", "Body": "

I am having some issues with my printer skipping/not extruding random layers of my prints. It happens a few layers in and sometimes doesn't even affect the entire layer. I have tried switching from Cura to PrusaSlicer but the problem persists. My printer is an Ender 3 v2 with dual Z-axis screws, a Micro Swiss all-metal hotend, Micro Swiss direct drive extruder, and a firmware update to reach 275\u00a0\u00b0C.

\n

I am currently using Polymaker PC-ABS in an enclosure but I have had the same problem with Hatchbox PETG.

\n

I have heard that Z Hop could be the issue when using Cura. I have tried turning it off while using Cura but it did not solve the problem. I have not tried turning it off in PrusaSlicer.

\n

PrusaSlicer settings are as follows:

\n\n

\"Fan

\n", "Title": "Ender 3 v2 layer skips", "Tags": "|creality-ender-3|extruder|extrusion|underextrusion|direct-drive|", "Answer": "

This is a known issue with the v4.2.2 mainboard relating to overheating. Continued overheating will eventually result in the extruder controller on the mainboard going bad.

\n

Solution: Mainboard change to version 4.2.7 and raise your unit to create more air flow. You could also add a fan under the printer.

\n" }, { "Id": "16793", "CreationDate": "2021-07-25T13:07:20.683", "Body": "

TLDR - I\u2019ve just driven myself insane trying to fix what I thought was a physical problem with my Z-axis, but it seems to have been solved by switching from the TH3D firmware to the Creality firmware. I\u2019m hoping someone can help me see where I went wrong here, so I can learn from it. Have I missed something obvious?

\n

My printer is a Creality Ender 3 Pro. I have a BLTouch. I recently upgraded to the silent 4.2.7 board and compiled my own firmware using TH3D Unified 2 using their instructions. This is my configuration.h.

\n

This \u201cworked\u201d, but after that upgrade, I had severe print problems. (See the photos below) These are supposed to be 20x20x20 calibration cubes (the big one is 200%). They are significantly taller than they should be. It may be hard to see in the photo, but this is because the Z spacing on the bottom layers seems to be too high. But, at the same point on each print (even the bigger one), the problem seems to just stop - and layer spacing is correct again for the end of the print.

\n

\"Comparison

\n

I tried everything to figure out what this was and I was convinced it was a physical problem with my Z-axis. I read lots of questions on here but nothing quite on topic. I printed on different parts of the bed and got the same result. I re-calibrated my Z steps, but these turned out to be pretty much at the firmware default anyway. I tightened every bolt and eccentric nut I could find that related to the X-axis gantry, but nothing made any difference. Same problem, every time.

\n

Out of desperation, I eventually switched to the official Creality firmware for the Ender 3 Pro 4.2.7 board with BLTouch - and the problem was fixed:

\n

\"Printed

\n

Does anyone have any ideas about what caused this? Have I overlooked something obvious? I\u2019d ideally like to go back to the TH3D firmware but it\u2019s seemingly not an option.

\n

https://pastebin.com/MYs8sYGj

\n", "Title": "Creality Ender 3 Pro - Z travel issue with TH3D firmware", "Tags": "|creality-ender-3|marlin|firmware|bltouch|th3d|", "Answer": "

If you check TH3D's site, as of January 12, 2022, there's an updated Unified 2 firmware(Unified 2.36a) that is meant to correct this problem.

\n

Announcement is here: Unified 2.36a is now live! | Firmware Update

\n
\n

Release Notes:

\n

Bugfixes

\n\n

Other Changes

\n\n
\n" }, { "Id": "16810", "CreationDate": "2021-07-28T14:05:45.593", "Body": "

I only have experience with PrusaSlicer, where if I have one .STL file containing multiple volumes, I can click a Split to parts button to then be able to change color of the individual volumes.

\n

Do other slicer softwares handle this similarly or will I face a problem in other softwares?

\n

I'm looking to upload my files for other people to print, and I would prefer to have 1 file per print. Can I upload these .STL files containing multiple volumes or should I rather upload separate .STLs?

\n

Edit: I was mainly looking for typical hobbyist slicers. I was looking to upload the files to a typical online 3d print sharing service for hobbyists.

\n", "Title": "Do all slicers handle multi object .STL files correctly for multi color printing?", "Tags": "|slicing|stl|prusaslicer|", "Answer": "

Splitting up the files into separate objects should preferably be done outside a slicer, there are tools for that. Meshmixer, Blender, any CAD program that imports STL files, etc. will do that. You could upload a single file and let others worry about breaking them apart their selves if their slicer of choice doesn't accept a single file for multi colored printings.

\n" }, { "Id": "17833", "CreationDate": "2021-08-03T01:40:33.073", "Body": "

I have a Creality CR-10 which is running Marlin because I have a BLTouch installed. All was working fine until it suddenly had an issue where it would lose connection to the computer mid-print when another electronic device nearby was plugged in.

\n

For example: when I plug in my lamp or soldering iron which is 3 meters away, it just stops (doesn't reset, just stops moving) leaving the bed and nozzle hot. The software on the printer is still functional, I can still navigate menus, etc. on the screen but the print stops and can't be resumed from Cura.

\n

I also tried Octopi but it does the exact same thing mid-print and the web interface spits out this error:
\ndevice reports readiness to read but returned no data (device disconnected or multiple access on port?)

\n

So it seems I may have some odd interference problem, I guess it could be over the air or through the powerline? I've tried different USB cables, a different power supply for the Pi, but so far, it's still acting up.

\n", "Title": "CR-10 randomly loses connection to computer when printing over USB", "Tags": "|marlin|creality-cr-10|octoprint|", "Answer": "

I had similar issues when I first started, and I think you are on the right track with interference being a problem.

\n

I have a giant bin of USB cables and started going through them. Only 2 worked out of about 20, and 1 of those 2 was intermittent with disconnect issues. The one that ended up working had gold plated leads and was about 8" long. I assume it probably has some additional shielding of the wires inside as well but I can't say for sure. It seems that the USB connection is very sensitive to interference.

\n

Also, if it is anything like my Ender 3 V2, you will probably want to use Kapton or electrical tape on the last pin (+5v) of the USB plug, otherwise the display and motherboard kept powered on after turning off the printer's power supply (it was being powered over the USB cable).

\n

Another suggestion would be to isolate the printer power supply from other devices (for example using a UPS made for a PC).

\n" }, { "Id": "17840", "CreationDate": "2021-08-03T19:31:43.540", "Body": "

Let me preface this with me being quite a newbie to 3D printing, however, I am an engineer and coder, so tinkering with stuff like this is quite the norm for me. Don't be afraid to be too technical, knowledge is power!

\n

I recently purchased a new Creality Ender 3 V2 based on a friend's recommendation and within a week or two, started getting terrible screeching from the print head fan. I already knew I wanted to get better PETG (and in the future, TPU) prints so I went ahead and bought a direct drive extruder (BIQU H2) to replace the Bowden setup.

\n

I got everything tuned pretty well (or so I thought), which included tuning the E-steps (945 which is 10x the original value of 93, however normal for this BIQU H2), my home position, changing the retraction setting, setting my K-factor for linear advance, tuning PID, etc. I flashed these settings into a custom Jyers Marlin firmware. Everything was working fine, and I was starting to get some really high-quality calibration tests, so I then began tuning my slicer profile.

\n

Then, yesterday, all of a sudden it is no longer printing correctly, even older G-code files before I messed with the slicer settings. It prints the prime line, retracts, moves, prints the brim, then retracts again, but doesn't start extruding to actually print the model. I've tried multiple G-code files that were working, and seem to get the same results, nothing prints beyond the first layer retraction or two. There is no clicking/vibrating or any attempts to move the extruder again after it stops printing.

\n

After some more reading, it seems that maybe I should have adjusted the vRef on extruder driver pots, as the new stepper motor is 0.8a (I believe the original 4042 extruder is 2a). Is it possible I damaged my stepper driver for the extruder? From my understanding, it would be moving at 10x the speed of the old stepper attached to the Bowden tube extruder.

\n

Some things I have done:

\n\n

For reference, I have an Ender 3 V2 with a 4.2.2 Motherboard. I'm not sure what stepper drivers it uses as there seems to be conflicting info online and I don't really want to remove the heatsinks if possible (but I will if I need to get this info and can't obtain it directly from the firmware). Unfortunately, the drivers are soldered to the motherboard so I guess I will need a new motherboard if I destroyed the extrusion driver. I do feel I probably should have made some driver adjustments given this stepper motor is 40% the amps, and runs at 10x the speed of the old stepper.

\n

I'm sort of at a loss as to what to try next, any thoughts would be appreciated. I plan on flashing the stock firmware back later this evening to see if that helps, but I doubt it will. I have only made some small tweaks mainly related to home offset, build area, PID tune, and K-factor for linear advance (which were previously working fine).

\n

Edit:\nI did some further testing yesterday. Flashing back to stock firmware didn't help at all. It also seems I may have been mistaken as further testing of the old 4042 stepper was successful if I attached it to the motherboard and ran the G-code which fails on the first layer, however I attached it with a spare cable as to not have to take apart my harness going to the X carriage. Using the same spare cable with the BIQU H2 unfortunately did not help. The issue is reproducible without any filament loaded at all as well. There is an ever so slight play in the motor drive, so I am going to disassemble the extruder again tonight and see if the issue is reproducible with the motor detached from the extruder drivetrain.

\n

Also, just an interesting point, but multiple times after the failure I quickly checked the temp of the heatsinks on the drivers. Ironically, the only "hot" ones were the X/Y/Z steppers, and the extruder driver was only warm. Also, when the issue happens, the stepper becomes completely disengaged (I can turn the wheel freely without feeling the resistance of the stepper holding the position). Not sure if this is indicative of a driver failure, or standard practice for the driver if it encounters significant resistance.

\n

Another interesting point, the motherboard was not grounded. Although attached indirectly to the frame using screws, the panel the motherboard was attached to was not making contact with the frame. Some minor sanding resolved this issue. Once done, I notice the exterior case of my Y motor is grounded, but the X and Z as well as the extruder do not seem to be grounded. Is it typical for the exterior of the motors to be grounded? Perhaps there are some other ground faults I need to sort.

\n", "Title": "Extruder stops printing before finishing first layer (Ender 3 v2 w/BIQU H2)", "Tags": "|creality-ender-3|extruder|extrusion|extruder-driver|jyers|", "Answer": "

Check your retraction and run some experiments with it. I just had a very similar problem, when I switched from 0.6\u00a0mm retraction to 0.8\u00a0mm filament tended to get stuck between rolers and the cooler on layer change.\nI don't have and never had linear advance enabled, so it's not the issue.\nUnless I've misunderstood and your extruder didn't move at all (mine did move properly)

\n" }, { "Id": "17860", "CreationDate": "2021-08-05T19:04:23.157", "Body": "

I purchased a bottle of Siraya-Tech Simple Clear and Siraya-Tech has good documents on their resins with printing guidelines like for the simple clear (see their Simple User Guide).

\n

This gives good curing times for different types of resin printers, but they are all listed for 50 microns layer height. If I wanted to print at different layer heights, I am assuming the curing times would need to be adjusted, but I don't have a good feel for how much they would need to change.

\n

My first print was the Make: Rook Resin Printer Torture Piece, and given how short it was, I thought I would try pushing things to the limit and printed it at a 10-micron layer height, but I used the curing times in the guide above for the Elegoo Mars (figured it was the most similar to my actual printer the Halot-One), and as soon as I started the print I realized that I probably should have changed the layer exposure times. I was shocked how well it turned out for using the times for a 50-micron print, but there were parts of the print that looked like there may have been a little bit of light bleed because of the longer exposures for the thickness, and the print definitely took a long time for how small of a print it was.

\n

So, my question is, if I have good times for a 50-micron print, are there rules of thumb out there, or have experiments been done, for how much to adjust the exposure times if you want to do a 10-micron print, or say a 100-micron print? I would imagine it's not as simple as reducing it by 5x for a 10-micron print or doubling it for a 100-micron print.

\n

Key parameters that would need to be adjusted:

\n
    \n
  1. Exposure for initial layers
  2. \n
  3. Number of initial layers
  4. \n
  5. Layer exposure time
  6. \n
\n", "Title": "Adjusting layer exposure time for different layer thicknesses", "Tags": "|resin|layer-height|", "Answer": "

Based on my experience, cure-time and thickness increase or decrease is not linear and it is mostly curve. For example resin rated for 50 micron 5\u00a0s cure time. You should expect:

\n\n

If a resin manufacturer do not provide specific cure time for thickness you want, you should always print calibration objects.

\n

You can cure specific range of thickness depend on resins chemical properties (eg. uv blocker and photoinitiators), usually for non-specialized resins, you may print 1/2 or 2x of the suggested thickness. So you cannot print 10 micron for resins which designed for 50 micron thickness. For some specialized resins you can only print with the rated thickness and nothing else.

\n" }, { "Id": "17892", "CreationDate": "2021-08-10T13:47:28.350", "Body": "

I'm getting a lot of blobbing and scuffing on top of my prints in PETG on my Ender 3 with AMH/DD. It doesn't seem to do this if the print doesn't have a flat bottom so much.

\n

\"Mid-print

\n

I have the E-steps set to 130, which if anything is slightly under extruding vs the caliper measurements.

\n

Any ideas how I can fix this?

\n

Cura settings:

\n
[general]  \nversion = 4  \nname = Creality Ender 3 - eSUN PETG - Red  \ndefinition = creality_base  \n\n[metadata]  \ntype = quality_changes  \nquality_type = standard  \nintent_category = default  \nposition = 0  \nsetting_version = 17  \n\n[values]  \ncool_fan_full_layer = 3  \ninfill_overlap = 15  \ninfill_pattern = triangles  \ninfill_sparse_density = 10\nironing_enabled = False\nironing_only_highest_layer = True\nmaterial_print_temperature = 230\nretraction_amount = 2\nretraction_speed = 25\nskirt_gap = 6\nspeed_infill = 30\nspeed_print = 35\nspeed_topbottom = 15\nspeed_travel = 250.0\nspeed_wall = 25\nwall_line_count = 3\nz_seam_x = 150\nz_seam_y = 300\n\n----\n[general]\nversion = 4\nname = Creality Ender 3 - eSUN PETG - Red\ndefinition = creality_base\n\n[metadata]\ntype = quality_changes\nquality_type = standard\nsetting_version = 17\n\n[values]\nacceleration_enabled = True\njerk_enabled = True\nlayer_height_0 = 0.21\nmaterial_bed_temperature = 75\nmaterial_bed_temperature_layer_0 = 75\n
\n", "Title": "Flat print blobbing and scuffing with PETG on Ender 3 Pro with Microswiss AMH/DD", "Tags": "|petg|stringing|", "Answer": "

Are you sure the 130 E-steps is correct for your extruder? You don't mention what extruder you're using, but presumably, it's something other than just moving the original extruder to a direct drive mounting position or that would be extreme over extrusion.

\n

Assuming that's right, the most likely cause is having the nozzle too close to the bed, so that there's too little volume to fit the extruded material into.

\n

Also, you almost surely need to increase your nozzle temperature a lot. I'd normally consider 235\u00a0\u00b0C the absolute minimum for PETG, and I can't get it to adhere properly below 245\u00a0\u00b0C with the stock Ender 3 hotend. Since you have an all-metal hotend you can go even higher. PETG likes to "scuff" like that when it's too cool, and when you travel over already-printed material, and the lowest layers are even more susceptible to it since the bed will be sinking a lot of heat out of the material as soon as it's laid down.

\n" }, { "Id": "17895", "CreationDate": "2021-08-10T20:25:51.383", "Body": "

Everytime I export to STL in Fusion 360, a new Version (V1, V2, V3, V4, ...) of the file is created. But I would really like to use that feature to mark bigger changes instead of creating a new version everytime I change a minor detail to see how the Cura reacts to it.

\n

For some reason, no new version gets created when I send the model to Cura via the tools tab. But then no new STL file gets created, but Fusion 360 opens a new instance of Cura instead. Even if Cura is already open.
\nThat's not what I want. I want to overwrite the old STL file so Cura offers me to reload the STL file and I can keep working in the instance of Cura which is already open.

\n

I already deactivated the checkbox to create a new version everytime I save the file. That settings works as expected

\n", "Title": "How to stop Fusion 360 from creating a new version on STL export", "Tags": "|stl|fusion360|", "Answer": "

Not sure if this will solve your issue directly for you but when I save a STL/3MF from F360 I don't use the File/Export in 360.

\n

Instead I just right click the body in the browser tree and save the STL/3MF directly from that menu.\nSelect "save as mesh" This does not add any version number, and also the usage of cloud processing is not required so it's much faster.

\n

\"enter

\n" }, { "Id": "17900", "CreationDate": "2021-08-12T00:53:54.947", "Body": "

I've had my Ender 3 Pro for a few months, and it's been working great. Then, after a failed print (the STL had extra seams, causing the print to get mangled), I've started having issues that I have attributed to under-extrusion.

\n

As evidenced by the below image, some lines in each layer are missing. The gaps align throughout the print, but not between prints (the two squares are identical G-code). This happens in every layer, with a different pattern in each. It also causes some perimeters to not adhere to the previous layer at all.

\n

The extra strands in the right print are present because I ripped off the infill layer that printed on top before I aborted the print.

\n

I first assumed I needed to calibrate my extruder. It turns out I did (93 had to be changed to 150 steps/mm). However, that did not fix the issue.

\n

Some things I've tried:

\n\n

Is there anything else I might try to get this fixed?

\n

\"Evidence

\n

Specs for Reference:

\n\n
\n

If you need any more information, just comment and I'll do my best to supply it.

\n

Please note that I am not open to suggestions to replace my printer. I'd rather fix the one I have.

\n", "Title": "Patterned Under-Extrusion on Ender 3 Pro", "Tags": "|creality-ender-3|underextrusion|", "Answer": "

This is a big red flag and severely wrong:

\n
\n

93 had to be changed to 150 steps/mm

\n
\n

It can be reasonable to tune steps per mm by a few percent, but needing an increase of more than 50% versus the standard value for the same physical gearing indicates either you did something wrong in computing the needed value, or some other serious root problem with the printer.

\n

You should start by fixing that (putting it back at 93) then try to figure out why it seemed to need change.

\n

Likely your extruder isn't gripping the filament. Trying to overextrude that severely likely grinds it so bad the hob fills up with shavings then doesn't grip at all.

\n" }, { "Id": "17906", "CreationDate": "2021-08-13T06:49:52.173", "Body": "

This is my board MKS Melzi v2.0 cloned

\n

\"MKS

\n

I replaced the MCU (the actual IC) with new ATmega1284P 16 MHz, because the old MCU was damaged - the reason was that the fan got dusty and the power line became unstable. I replaced the MCU with the exact same model.

\n

I have tried to flash it many times using usbasp. It works but same issue system menus boot splash are still so slow it's painful. It takes minutes on an old Marlin version 1.1.6, hours on a new Marlin 2.x

\n

I think it's something related to MCU (crystal oscillator frequencies). Maybe the system uses an internal crystal so the MCU got stuck.

\n
    \n
  1. How can I switch to an external crystal?
  2. \n
  3. If it's not, then what can cause the problem?
  4. \n
\n

Firmware links:

\n\n", "Title": "TronXY X5S system (Marlin) - Very Slow menus", "Tags": "|marlin|electronics|tronxy|", "Answer": "

Either:

\n\n

There are some internal fuses that set the clock frequency divider and the clock source and maybe your new IC has the incorrect fuses set.

\n

That is to say, they may be factory default, but they are not the same as the controller board comes shipped with.

\n

You probably want to check both settings as the factory defaults of your replacement IC are probably not what you expect, or desire.

\n

Check the ATmega1284P datasheet for details.

\n
\n

Prescaler

\n

Section 7.12.2 CLKPR - Clock Prescale Register, page 38. That is for the general clock frequency, and shows the register CLKPR in particular Bits 3:0.

\n
\n

\"7.12.2

\n
\n

Or section 7.11 System Clock Prescaler

\n
\n

\"7.11

\n
\n

External clock

\n

Section 7.2 Clock Sources, shows that CKSEL Bits 3:0 set which clock is used, internal or external. You presumably want to use the external clock so set all four bits to zero

\n
\n

\"7.2

\n
\n
\n

To set, or examine, any of the registers, you have to use usbasp. I can't remember what the commands are. If you look on SE.Arduino or SE.Electronics I am sure that there is a question with the answer. I vaguely remember having the same problem with an ATtiny.

\n" }, { "Id": "17916", "CreationDate": "2021-08-14T19:12:20.240", "Body": "

Normally 3D printing is done starting at the base level depositing hot plastic upwards.

\n

In order to create bas relief details, can we deposit hot plastic on an available centered cone base any curve design allowing slow hot end /spinneret movement in vertical or inclined planes (instead of pure Z direction) by any user-defined CNC control program with a 3D printer?

\n

\"Two

\n", "Title": "Inclined plane movement of spinneret", "Tags": "|software|", "Answer": "

I think you would have to rewrite the code specifically for the surface you want to print on, so the printer recognizes the base is not flat.

\n

You'd have to attach an identical pot in exactly the same spot on your printer bed for each print.

\n

For the pots you show, you might try printing just the black design flat, maybe 1 mm thick to keep it flexible, then gluing it to your pot. Using soft filament would make this easier.

\n

Another option would be a 3D pen.

\n

Or use a cutting machine like a Cricut to cut it in self stick vinyl. I believe those machines use SVG files.

\n" }, { "Id": "17920", "CreationDate": "2021-08-15T12:38:05.097", "Body": "

I am currently working through various different iterations ofccreating a propeller for a model ICE (nitro engine). So far I have managed to disintegrate each of them eventually. I have been experimenting mostly with PLA and PETG so far. I am currently thinking about using carbon-infused PLA or using PC (I found one that is printable on my printer). However, I am not so sure what properties I exactly have to look for when it comes to a propeller.

\n

I am making the following assumptions:

\n\n

So far the PLA propeller has held up the longest and I cannot rule out that it accidentally touched something when it broke. That is of\ncourse something it does not have to be able to handle, so I am\nprobably going to give PLA another chance.

\n

The PETG propellers all literally broke apart the second the engine\nstarted, and it did not even hold up one rotation.

\n

The design of the propeller is actually sacrificing some aerodynamic efficiency in order to make sure there are no "thin" parts. It is\ntypically breaking in the section I have highlighted in the\nthe following picture.

\n

\"3D

\n

I have also a picture of the "remains" of the broken PLA propeller.\nIt actually held up many runs before it eventually broke. It actually partially delaminated the entire wall.

\n

\"A

\n

I am looking for two recommendations/tips:

\n\n", "Title": "Improving durability of a printed propeller", "Tags": "|print-material|print-strength|", "Answer": "

For safety on a project like this, it's best to have a self-contained chamber to run a life test until the propeller falls apart. Once a material and design has a reasonable lifetime, then you could use the print.

\n

Reputable propeller manufacturers will already do this. They may even run a shorter burn-in test on product being sold to eliminate defects.

\n

I would suspect PLA as being too brittle. Nylon carbon fiber is probably a better material, but not easy to use. Unless you're wanting to experiment with propeller design, printing propellers will cost you more then buying them.

\n

As Brydon mentioned life testing usually does acceleration such as twice the rate. Temperature is also important. Plastic is more brittle cold and softer hot. You may even want to off balance the axis slightly to accelerate the testing.

\n" }, { "Id": "17926", "CreationDate": "2021-08-16T11:15:21.147", "Body": "

I don't understand the description of this setting in FlashPrint and have been unable to find a better description online. What does it do?

\n

\"Screenshot

\n", "Title": "What does the \"Control Module\" setting in FlashPrint 5.1 do?", "Tags": "|slicing|flashforge|flashprint|", "Answer": "

On Reddit, someone pointed out that this likely refers to which items temperature is being adjusted by the tickbox above (and table below). It seems quite obvious now :-)

\n" }, { "Id": "17934", "CreationDate": "2021-08-17T06:17:31.150", "Body": "

I switched over to a new filament recently (Sunlu PLA) and I can't get the first layer details to stick properly without squishing the first layer way too much. Prints don't always fail, but little circles and the initial skirt rarely stick as they should. The issue appears to be that when it starts to extrude at a new location, it's not always close enough or extruding fast enough for the initial filament to stick and it gets lifted on the next pass. Other things stick just fine, it's just at these transitions where things lift a little. My bed is level and in general things are printing just fine. It's just the start/end of initial layers that tend to have problems and lift just enough to cause problems later.

\n

If I lower my nozzle enough, I can get it to stick more reliably but then I have other problems because the first layer is so squished that the ridges rise and cause problems later.

\n

I've read dozens of articles and tried all these things already:

\n\n

It used to print fine, so I guess it could just be the filament but I'm hoping there's something I can still do. I'm using an Ender 3 v2 with a BLTouch, new springs, replaced Bowden tube and red metal extruder. Printing at 210\u00a0\u00b0C with 60\u00a0\u00b0C bed.

\n

\"Beginning

\n

\"Beginning

\n", "Title": "Stringing on first layer unless I squish it", "Tags": "|print-quality|", "Answer": "

Try using 75 \u00b0C for the bed and 220 \u00b0C or highest recommended extrusion temperature for Sunlu PLA for the first layer, then lower the temperatures to 210 \u00b0C with 60 or 65 \u00b0C bed for subsequent layers.

\n

Other options are Elmer's glue stick, a glue stick specified for a 3D printer bed, or hair spray.

\n" }, { "Id": "17945", "CreationDate": "2021-08-20T04:33:03.290", "Body": "

My designs are invisible on 3D printer LCD screen, i.e. the name of my design is showing up, but the part is missing and if I slice it with the software given by the 3D-printer company it works, but if I use any other slicer like Cura or Matter Control, it's not working.

\n

I'm unable to add images as I'm new here.

\n

I have a Tronxy X5SA 3D printer with a touch display.

\n

I contacted the company, but they can't understand my issue, that's why seeking your help.

\n", "Title": "My 3D printer shows inbuilt designs, but my designs are invisible", "Tags": "|ultimaker-cura|3d-models|3d-design|tronxy-x5|mattercontrol|", "Answer": "

Install the "Chitu code" plugin from the Cura website, some companies use the Chitu board as their printer's CPU and Tronxy does that. Without the plugin, it won't show a preview of your design and also it won't print.

\n" }, { "Id": "17951", "CreationDate": "2021-08-20T23:41:14.893", "Body": "

I've had an Ender 5 for about a year without problems. But just recently, the printer is not lifting the Z-axis before printing. It will home great but when it moves to print the purge line it lifts the bed down about 2-3 millimeters and the nozzle is not even close to touching the bed anymore.

\n

I've checked the starting G-code in Cura and it all looks fine but I'm still having problems.

\n

Current Start G-code:

\n
M201 X500.00 Y500.00 Z100.00 E5000.00 ;Setup machine max acceleration\nM203 X500.00 Y500.00 Z10.00 E50.00 ;Setup machine max feedrate\nM204 P500.00 R1000.00 T500.00 ;Setup Print/Retract/Travel acceleration\nM205 X8.00 Y8.00 Z0.40 E5.00 ;Setup Jerk\nM220 S100 ;Reset Feedrate\nM221 S100 ;Reset Flowrate\n\nG28 ;Home\n\nG92 E0 ;Reset Extruder\nG1 Z2.0 F3000 ;Move Z Axis up\nG1 X10.1 Y20 Z0.28 F5000.0 ;Move to start position\nG1 X10.1 Y200.0 Z0.20 F1500.0 E15 ;Draw the first line\nG1 X10.4 Y200.0 Z0.20 F5000.0 ;Move to side a little\nG1 X10.4 Y20 Z0.20 F1500.0 E30 ;Draw the second line\nG92 E0 ;Reset Extruder\nG1 Z2.0 F3000 ;Move Z Axis up\nM503\n
\n", "Title": "Ender 5 won't lift the Z-Axis before printing", "Tags": "|troubleshooting|z-axis|creality-ender-5|", "Answer": "

After hours of testing (which included reinstalling firmware, formatting various SD cards, and changing G-code and slicer settings), I did find a function to restore factory defaults and that did the trick. I'm still not sure how something wrong got stored in the memory but it did and resetting to factory defaults worked.

\n" }, { "Id": "17979", "CreationDate": "2021-08-26T03:09:13.967", "Body": "

I recently installed the BigTreeTech SKR E3 mini V2.0 on my Ender 5 and I'm having some print quality problems even when using the same settings as before the board swap.

\n

The edges are rough and curling and there is some bulging on the X and Y-axis. I printed a 20\u00a0mm cube and after measuring it, the middle is about 19.9\u00a0mm wide (on the X and Y-axis) but the top half and bottom half of those same sides measure up to be around 19.6\u00a0mm which is a pretty large difference when compared the near-perfect ones I used to achieve.

\n

I've tried calibrating the E-steps and lowering the wall speed to 20\u00a0mm/s but I've had no luck. Is there anything I can do to fix it?

\n", "Title": "Print bulging and curling on the edges", "Tags": "|print-quality|creality-ender-5|", "Answer": "

This is a Wiki answer posting the solution from Carter's comment. If the OP (Carter) wishes to post and accept their own answer, then this wiki answer can be removed.

\n
\n
\n

I did tension the belt the best I could (which wasn't too much from the original) but I think it did help a little. I also took off the Hero Me fan duct I put on and I think that helped a ton too. I was having some clogging issues as well and when I disassembled the direct drive kit I had on, I found the fan duct mount was cracked and was leaning about 20\u00b0 forward. Because of this, I think the fan duct was dragging way too low and touching the print when travelling fast and this caused a ton a problems.

\n
\n" }, { "Id": "17996", "CreationDate": "2021-08-30T15:03:29.150", "Body": "

I'm still confused on this. The Anycubic Photon Ultra is coming out soon, and it features an internal 720p Projector. The 720p resolution seems like a far cry from a 2k LCD (1280\u00d7720 vs. 2560x1440). Despite having 4x as many pixels, many articles are praising DLP as being much higher in resolution / detail. This is very counterintuitive... How can a DLP projector with less pixels produce a more detailed item than a 2k LED exposure?

\n", "Title": "How exactly can a 720P DLP print at a higher resolution than a 2k LCD?", "Tags": "|sla|dlp|lcd-screen|", "Answer": "

How can a digital camera produce images several times the resolution of the sensor? They move the sensor and overlay multiple exposures. Optically or physically moving the DLP chip or its projected image can give a combined "image" from multiple scans anywhere from 2x to several times the linear resolution of the DLP itself.

\n

Such a print might be slower than an LCD, because of multiple exposures for each layer, but it might also be faster, because the light source can be far brighter than can be used with an LCD without damaging the liquid crystals themselvs.

\n" }, { "Id": "18046", "CreationDate": "2021-09-10T10:46:54.557", "Body": "

Backstory

\n

I set up the BLTouch module as explained in the guide that came with it - that included unplugging the Z axis switch, and replacing it with a pair of wires from the BLTouch module.

\n

I managed to burn the firmware available from Creality, however this didn't have an offset for the fact the BLTouch module is offset from the hotend, and I couldn't manually edit the .hex file to add in offsets. I tried using the G Code command to update it on the EEPROM, but that didn't work and it always probed off the edge of the bed.

\n

I then decided to flash new firmware based directly off Marlin 1.1.x, as there are some videos on Youtube about how to do this. I used the pink USB ISP supplied with the BLTouch to flash the Arduino bootloader to the motherboard, and now I can use the Arduino IDE to program the board.

\n

I uploaded my configured Marlin firmware (see below), but it isn't working properly.

\n

Problem

\n

When I home the printer, the X and Y stops go to the switches, bump and zero correctly. However, when the Z axis is homing, the BLTouch probe stays retracted, and it starts heading for home - I forcefully shut down my printer when this happens because I know there's nothing to stop it from crashing into the print head and breaking things.

\n

How do I configure my firmware to use the BLTouch module as the Z stop probe?

\n

Although the probe clicks in and out a few times when I power on the printer, I'm not entirely sure that it is even working - going to the BLTouch menu and choosing Cmd: Deploy doesn't have an effect.

\n

On the up side, when it goes to the centre to home (due to Z_SAFE_HOMING), the BLTouch module is over the centre of the build plate instead of the hotend.

\n
\n

Configuration

\n

You can see the Marlin repository I'm trying to use here on Gitlab, or a direct link to the diff between the inital Marlin configuration and my changes.

\n

These are some hopefully relevant configuration changes I've made:

\n
// Configuration.h\n\n//#define USE_XMIN_PLUG\n//#define USE_YMIN_PLUG\n#define USE_ZMIN_PLUG\n#define USE_XMAX_PLUG\n#define USE_YMAX_PLUG\n\n#define BLTOUCH\n\n#define X_PROBE_OFFSET_FROM_EXTRUDER -44  // X offset: -left  +right  [of the nozzle]\n#define Y_PROBE_OFFSET_FROM_EXTRUDER -6   // Y offset: -front +behind [the nozzle]\n#define Z_PROBE_OFFSET_FROM_EXTRUDER 5.7  // Z offset: -below +above  [the nozzle]\n\n#define Z_HOMING_HEIGHT 4\n\n#define Z_SAFE_HOMING\n
\n
// pins_MELZI_CREALITY.h\n\n#define SERVO0_PIN 20 // Wondering if this is correct?\n
\n

These settings have been left as default, but they look relevant maybe:

\n
/**\n * Z_MIN_PROBE_USES_Z_MIN_ENDSTOP_PIN\n *\n * Enable this option for a probe connected to the Z Min endstop pin.\n */\n#define Z_MIN_PROBE_USES_Z_MIN_ENDSTOP_PIN\n\n/**\n * Z_MIN_PROBE_ENDSTOP\n *\n * Enable this option for a probe connected to any pin except Z-Min.\n * (By default Marlin assumes the Z-Max endstop pin.)\n * To use a custom Z Probe pin, set Z_MIN_PROBE_PIN below.\n *\n *  - The simplest option is to use a free endstop connector.\n *  - Use 5V for powered (usually inductive) sensors.\n *\n *  - RAMPS 1.3/1.4 boards may use the 5V, GND, and Aux4->D32 pin:\n *    - For simple switches connect...\n *      - normally-closed switches to GND and D32.\n *      - normally-open switches to 5V and D32.\n *\n * WARNING: Setting the wrong pin may have unexpected and potentially\n * disastrous consequences. Use with caution and do your homework.\n *\n */\n//#define Z_MIN_PROBE_ENDSTOP\n\n/**\n * Number of servos\n *\n * For some servo-related options NUM_SERVOS will be set automatically.\n * Set this manually if there are extra servos needing manual control.\n * Leave undefined or set to 0 to entirely disable the servo subsystem.\n */\n//#define NUM_SERVOS 3 // Servo index starts with 0 for M280 command\n
\n", "Title": "How to make Ender 5 use BLTouch probe for Z homing?", "Tags": "|marlin|bltouch|creality-ender-5|", "Answer": "

I've solved this now. Here's the problem:

\n
// pins_MELZI_CREALITY.h\n

#define SERVO0_PIN 20 // Wondering if this is correct?

\n

Funnily enough, it wasn't correct. Pin 20 is the Z MIN pin, as defined in pins_MELZI_CREALITY.h, and not the servo pin used by the BLTouch module.

\n

At 8:04 of this youtube video, he says that that value should be set to 27, "if we're using a pin 27 board or splicing pin".

\n

I tried it, and the BLTouch module now responds, and homing works!

\n

For anyone else who comes across this problem (it's strangely very hard to find online), add the following code:

\n
#define BLTOUCH\n#if ENABLED(BLTOUCH)\n  #define SERVO0_PIN 27\n  //#define BLTOUCH_DELAY 375   // (ms) Enable and increase if needed\n#endif\n
\n" }, { "Id": "18063", "CreationDate": "2021-09-13T18:10:11.857", "Body": "

I just bought & received my first 3D printer and built it and tried my first print on it.\nIt is this printer ELEGOO Neptune 2S

\n

I printed the following item (chip clip) as a test.

\n

The top of the item looks exactly correct.

\n

\"top

\n

However, the bottom (where the print started) is slightly "bonded" (still connected).\nIf you look closely you can see the outline of the teeth.

\n

\"bottom

\n

Can you tell me what might have caused this?\nAm I using an improper material? (This was test filament that came with printer).

\n

Was the printer not close enough to (or too far from) the bed?

\n

Is there possibly something I should've changed in the settings of the drawing? I left them the same as they were in the downloaded file.

\n", "Title": "Why does my print have extra layer(s) which keep moving parts locked?", "Tags": "|3d-design|layer-height|elegoo-neptune-2s|", "Answer": "

You have "elephant foot". Look at the left end of the grip section of the clip in your second image -- see how the thickness increases toward the top (which was the bottom, against the build surface, when printing)?

\n

There are a number of causes for this; in your case, it continues for several layers, so the basic solutions (reducing first layer extrusion in slicer settings etc.) won't fix it; you probably have the same issue I do, with the Z axis not moving quite far enough on the first 3-4 layers after the initial.

\n

I'd suggest you start looking for the solution by printing a calibration cube, and measure how much height you're losing. You might also want to draw up some simple calibration prints with one, two, three, and four layers to see the actual height those first few layers are getting, vs. what your layer height setting is.

\n

This will help confirm the problem. Solving it might involve several kinds of tuning, but they'll all revolve around making the first few layer moves up from the start closer to their nominal height. Once layers 2, 3, and 4 are right, once you get a good initial layer, your prints will be significantly improved -- especially print-in-place parts like this one.

\n" }, { "Id": "18066", "CreationDate": "2021-09-13T22:01:51.413", "Body": "

My shiny new Ender 3 V2 seems to ignore the Z-axis limit switch, when I print a model. It moves the nozzle so far down, that it scratches on the bed and destroys the nozzle.

\n

However, when I use the Auto Home function it lowers the nozzle until the limit switch clicks and than stops.

\n

After disabling the stepper motors, the printing platform can be leveled. When after that a printing process is started, Z-axis comes down, the limit switch clicks, but the Z-axis goes about 1.5\u00a0mm further down ignoring the switch.

\n

As a result, the nozzle is pressed with such force to the platform that no filament comes out of the nozzle and is grated with such force that the nozzle and the platform get damaged.

\n

I checked the Z-axis offset in the software to be zero. I moved the Z-axis limit switch higher, to check if the behavior changes without success. The installed firmware version is 1.0.2.

\n

Can someone tell me why it ignores the Z-axis limit switch during printing, when it reacts to it during the leveling process? I just want to print my first ever model.

\n", "Title": "Ender 3 V2 ignories Z-axis limit switch", "Tags": "|creality-ender-3|bed-leveling|z-axis|homing|", "Answer": "

I finally found the problem. The threaded rod of the Z axis was not mounted tight enough to the motor. This resulted in the slipping of the Z axis by large accelerations of the motor.

\n

So... tightening the threaded rod connector socket and redoing the leveling solved the problem.

\n" }, { "Id": "18067", "CreationDate": "2021-09-13T22:10:29.750", "Body": "

I had an idea to make a program for my Arduino Uno that beeps whenever a print is done, but I can't find anything about interfacing it with the printer as all Google gives me is stuff about making a 3D printer from an Arduino.

\n", "Title": "Would it be possible to use an Arduino Uno to interface with my Ender 3 pro so I can program it to beep when done?", "Tags": "|creality-ender-3|arduino|", "Answer": "

Yes you could. But, as an alternative to the Arduino Uno, it might be far easier to send a beep by G-code M300 (or here for Marlin firmware), this will bypass the need for an Arduino Uno entirely. You can even play the soundtrack/theme of Beverly Hills cop from this previous answer.

\n

You can add the (set of) G-code(s) to the "end G-code" in your slicer.

\n" }, { "Id": "18078", "CreationDate": "2021-09-16T15:27:37.270", "Body": "

Is it possible to buy/upgrade the nozzle on my Ender 3 Pro to one that is suited for handling the higher temperatures needed to effectively print with PETG filament?

\n", "Title": "Can the Ender 3 Pro be upgraded to use PETG", "Tags": "|creality-ender-3|nozzle|petg|", "Answer": "

No hotend upgrade is required to print PETG on an Ender 3 (pro or any other variant). The stock hotend can be used at temperatures up to 250 \u02daC, and 230-245 \u02daC is the range typically recommended for PETG. I print it at 245 \u02daC.

\n

You may however want to upgrade the extruder. The stock extruder lacks both torque and grip, and significantly skips/slips when trying to print PETG at any significant speed. Going over 40 mm/s or so is likely to produced failed prints. Look for an extruder with gear reduction and a hob that's designed to grip the filament better (vs the flat one that comes with the printer) or even dual driven hobs.

\n" }, { "Id": "18090", "CreationDate": "2021-09-17T19:17:21.883", "Body": "

I am attempting to print a model that is too tall to print up-right and it has a large flat side that would be my first layer but ultimately would be the side of the object and I'd like it to look presentable. I'm using wood PLA, so I intend to sand and stain it.

\n

I've had no issues with adhesion when using a raft. The slicer lays down a thick first layer that sticks beautifully. Here's the first layer of a raft - nice thick lines:

\n

\"First

\n

But when I try printing without the raft (even when using a brim) that first layer is not only thin, but easily moves away. Also the lines meld together creating gaps in the layer. And that's no skirt around the outside, that's the side of the model that doesn't touch the fill. I have to imagine this would be awful rough and impossible to sand out if it finished.

\n

\"Printing

\n

By the second layer, the friction is picking up the first layer and it catches the extruder and bam:

\n

\"Destroyed

\n

I've done some research suggesting higher temps, both of the extruder and the plate. I did see an improvement in the lines, but it still doesn't stick well. And what is considered a good temp? how do you know if you went too hot?

\n

Any recommended settings in the slicer to make the bottom layer smoother?

\n

The printer is a FlashForge Creator Pro.

\n", "Title": "Need help printing a quality first layer, but having adhesion and fill issues", "Tags": "|print-quality|pla|adhesion|flashforge-creator|", "Answer": "

After some trial and error, the answer seemed to simply be to "add heat." I bumped both the heat of the plate by 15\u00a0\u00b0C, and the extruder by 20\u00a0\u00b0C.

\n

I also increased the size of the first layer to between 0.3\u00a0mm and 0.4\u00a0mm and it's looking much better.

\n

\"Printed

\n" }, { "Id": "18096", "CreationDate": "2021-09-18T20:59:28.847", "Body": "

I have a seemingly unique problem. I'm trying to design models that print with single extrusion walls and are not closed volumes. For example, a topographical map model printed on an edge. It has height and width, but no depth. I want the model to print with a single, non-closed extrusion per layer. Printing back and forth as it moves up the model.

\n

When slicers cut up a model, historically each plane must result in a closed shape. The slicer then determines the tool path to create that shape. But what if, instead of a closed shape, the slice results in lines. No area or volume, just lines. Couldn't the slicer then create a tool path at each line?

\n

Is there anything like this available anywhere?

\n

Also, to be clear, I've been doing this 3D printing thing for a long time and know pretty well how slicers work and their resulting models. I've also been 3D modelling for a long time.

\n

The simplest solution I can think of would be to modify a slicer or find a plugin or something that already does what I'm looking for. The second possible solution would be to find 3D software that can "extrude" an arbitrarily complex surface in such a way that normal slicing software with the "detect thin walls" setting enabled would result in a single extrusion resulting in the desired outer surface. I haven't found either in my searching.

\n

The solution should work with nearly any extrusion width or any nozzle. You have a 0.4mm nozzle, you can print the part with wall widths anywhere from ~0.4mm wide to around 1mm. Your choice when slicing. If you have a 1mm wide nozzle, you could print the part with much thicker walls. It would be configurable in the slicer because the model only defines the surfaces, no volume data.

\n

Like I said, this is a seemingly unique problem.

\n", "Title": "Slicing a non-closed mesh - extrude only on faces, no volume", "Tags": "|slicing|", "Answer": "

You can also have a look at Chisel, it basically does what you want - creates G-code directly from parametric patches (Bezier, Nurbs), you have fine control over extrusion width in every part of your model (perimeter, infill...).

\n

It's optimised for efficient structural panels so currently always expects 2 faces + infill configuration (could be none for hollow structure), but I will very soon add features like specifying arbitrary number patches with no infill, that way you will have full freedom to design part however you like.

\n" }, { "Id": "18100", "CreationDate": "2021-09-19T12:43:49.567", "Body": "

I have a Tronxy X5SA printer upon which I want to install the latest Marlin firmware v2.0.9.1.

\n

I take the configuration example on GitHub for the X5SA printer and modify the lines in configuration.h as recommended in the guide (see this URL).

\n

In Visual Studio Code, when I compile, I obtain this error :

\n
Marlin\\src\\gcode\\motion\\G2_G3.cpp: In function 'void plan_arc(const xyze_pos_t&, const ab_float_t&, bool, uint8_t)':\nMarlin\\src\\gcode\\motion\\G2_G3.cpp:154:7: error: 'MM_PER_ARC_SEGMENT' was not declared in this scope\n       MM_PER_ARC_SEGMENT\n       ^~~~~~~~~~~~~~~~~~\nMarlin\\src\\gcode\\motion\\G2_G3.cpp:154:7: note: suggested alternative: 'MAX_ARC_SEGMENT_MM'\n       ^~~~~~~~~~~~~~~~~~\n       MAX_ARC_SEGMENT_MM\nCompiling .pio\\build\\chitu_f103\\src\\src\\gcode\\sd\\M20.cpp.o\n*** [.pio\\build\\chitu_f103\\src\\src\\gcode\\motion\\G2_G3.cpp.o] Error 1\n========================================================================================= [FAILED] Took 71.95 seconds ========================================================================================= \n\nEnvironment    Status    Duration\n-------------  --------  ------------\nchitu_f103     FAILED    00:01:11.945\n==================================================================================== 1 failed, 0 succeeded in 00:01:11.945 ==================================================================================== \nPS D:\\TRONXY\\Marlin-2.0.9.1\\Marlin-2.0.9.1> echo "done" >C:\\Users\\myUser\\AppData\\Local\\Temp\\ipc\nPS D:\\TRONXY\\Marlin-2.0.9.1\\Marlin-2.0.9.1> \n
\n

Here is the code in G2_G3.cpp on line 154, but I need some help to understand why it's wrong, and how to fix...

\n
 // Start with a nominal segment length\n  float seg_length = (\n    #ifdef ARC_SEGMENTS_PER_R\n      constrain(MM_PER_ARC_SEGMENT * radius, MM_PER_ARC_SEGMENT, ARC_SEGMENTS_PER_R)\n    #elif ARC_SEGMENTS_PER_SEC\n      _MAX(scaled_fr_mm_s * RECIPROCAL(ARC_SEGMENTS_PER_SEC), MM_PER_ARC_SEGMENT)\n    #else\n      MM_PER_ARC_SEGMENT // LINE 154 HERE!!\n    #endif\n  );\n  // Divide total travel by nominal segment length\n  uint16_t segments = FLOOR(mm_of_travel / seg_length);\n  NOLESS(segments, min_segments);         // At least some segments\n  seg_length = mm_of_travel / segments;\n
\n

I have tried to compile the BugFix version successfully, but unfortunately the bugfix version does not work properly with my printer (I can't get the bed to lift to the probe. I have opened another post for the bugfix version issue).

\n", "Title": "Marlin firmware 2.0.9.1 for X5SA example : MM_PER_ARC_SEGMENT Compilation Error", "Tags": "|marlin|firmware|tronxy|tronxy-x5sa|", "Answer": "

The compiler can't see the definition of MM_PER_ARC_SEGMENT. Find the file that it is declared in and ensure that file in included in the compile command, as an include (#include) or path or whatever.

\n

Looking at the current Configuration_adv.h on the Marlin Github repository, the definition is at Line 2033 in the section at Lines 2028-2041

\n
//\n// G2/G3 Arc Support\n//\n#define ARC_SUPPORT                 // Disable this feature to save ~3226 bytes\n#if ENABLED(ARC_SUPPORT)\n  #define MM_PER_ARC_SEGMENT      1 // (mm) Length (or minimum length) of each arc segment\n  //#define ARC_SEGMENTS_PER_R    1 // Max segment length, MM_PER = Min\n  #define MIN_ARC_SEGMENTS       24 // Minimum number of segments in a complete circle\n  //#define ARC_SEGMENTS_PER_SEC 50 // Use feedrate to choose segment length (with MM_PER_ARC_SEGMENT as the minimum)\n  #define N_ARC_CORRECTION       25 // Number of interpolated segments between corrections\n  //#define ARC_P_CIRCLES           // Enable the 'P' parameter to specify complete circles\n  //#define CNC_WORKSPACE_PLANES    // Allow G2/G3 to operate in XY, ZX, or YZ planes\n  //#define SF_ARC_FIX              // Enable only if using SkeinForge with "Arc Point" fillet procedure\n#endif\n
\n

Ensure that you have ARC_SUPPORT enabled - do not have the following line (2031) commented out:

\n
#define ARC_SUPPORT \n
\n" }, { "Id": "18105", "CreationDate": "2021-09-19T17:08:02.777", "Body": "

Prints are generally successful, but I always get a herringbone pattern on the top and bottom surfaces and striations on the sides. The herringbone is finer with finer print qualities, but it is always there. Ditto the sides. See photos.

\n

Always there, regardless of temperature (using a heat tower), speed (even very slow), quality in Cura, etc.

\n

Perhaps this is the best an Ender 3 Pro can do? If so, that's fine. I'm only trying to determine what this printer is capable of.

\n

(Perhaps I can improve the top surface with ironing, but that isn't my question.)

\n

Some things I've done with no effect: Run PLA spool from a drying cabinet; replaced extruder; reset Bowden tube; replaced nozzle; leveled bed numerous times.

\n

One thing I haven't tried yet (will soon): using a better grade of PLA.

\n

\"Top

\n

\"Side\"

\n

It's a heat tower, so the top surface was 190 \u00b0C. Speed was about 50-60 mm/s (don't remember exactly). With the belt tightened, the herringbone is much finer, but still there. The walls are much smoother, by still the layers are visible. I'm just not sure what to expect from this printer. Am I supposed to get surfaces that are perfectly smooth, or is some texture to be expected?

\n", "Title": "Are these normal surfaces from an Ender 3 Pro with PLA?", "Tags": "|creality-ender-3|quality|surface|", "Answer": "

The top image looks as if the (top) layer(s) are under-extruding, the walls don't appear to be suffering from the same problem.

\n

The following image from a recent PETG print is typical for an FDM product. The deposited lines need to touch (actually, slightly overlapping, but the slicer will take care of that) each other:

\n

\"enter

\n

You will see texture (especially when there is a non uniform surface, e.g. with holes, and without an option as monotonic fill in Cura 4.11 or similar disabled), but the lines need to touch each other, you may not see the layer beneath.

\n

There are various reasons for this to happen, for instance: under-extruding because of speed or temperature issues or positioning accuracy. The walls seem to closely adhere, so this may find the cause in slicing parameters for the filling in of the layer.

\n" }, { "Id": "18119", "CreationDate": "2021-09-22T23:30:28.787", "Body": "

I have an STL (Raspberry Pi 4 casing) that automatically places itself like below on the bed surface:

\n

\"enter

\n

Would it be a better and more efficient print if I place it like this:

\n

\"enter

\n

At first, I thought this is no-brainer, the bigger surface should be the bottom layer. However, the horizontal print might result a more efficient head movement.

\n", "Title": "Bed surface stability vs head movement efficiency question", "Tags": "|print-quality|print-orientation|efficiency|", "Answer": "

Long story short, laying the design horizontally is a lot less problematic when compared to laying it vertically. I'll explain:

\n

[ Vertical ]

\n\n

[ Horizontal ]

\n\n

In most cases, it's best to keep your design as low and spread out as possible, though I've had some prints where that wasn't an option.

\n" }, { "Id": "18135", "CreationDate": "2021-09-25T10:01:01.140", "Body": "

I'm trying to 3D print a lattice work or truss, basically some beams forming a rectangle and additional beams forming the diagonals and where those beams cross, they should be fused. So, something like this:

\n

\"simple

\n

The problem is that any slicer I give a form like this to starts drawing triangles around the inner openings and in best case those triangles then touch if you use enough walls, but the pull strength you would get from beams in such a truss is lost because the opposite corners of the rectangle are not connected by a single length of filament laid down. Here as example what PrusaSlicer does for every layer:

\n

\"enter

\n

Basically this gives separate triangles at the top, bottom, left and right with some rectangular walls at the outside. Not bad, but I think that for extra strength it would be better if on even layers there would be long extrusions going all the way from the top-left corner to the bottom-right corner (and hence interrupting extrusion on the other diagonal) and on odd layers having just the opposite (so long extrusions from bottom-left to top-right).

\n

So, my question: is there any way to tell the slicer to do something like that? So, having extra long (alternating) corner to corner extrusions next to the triangular "outer" walls that it normally puts down? Or, is there some other trick I could use to get a similar effect (while also having long extrusions between adjacent corners)?

\n", "Title": "Any way to have slicer make a stronger lattice work?", "Tags": "|ultimaker-cura|3d-design|slicing|fdm|prusaslicer|", "Answer": "

Have a look at Chisel infills there are very strength optimised, limitation is currently that they must be single line width, but multiple-lines thick infill will be supported soon.\nAfter that, it's very simple, basically setting double corrugated infill with corrugation size 1 for your rectangle model.

\n" }, { "Id": "18148", "CreationDate": "2021-09-27T23:02:39.057", "Body": "

OpenSCAD User Manual's section on operators does not mention precedence. Quick Web searches found nothing. I assume the usual order of exponentiation, then multiplication, then addition does hold, but how does e.g. a conditional operator or unary minus interact with these? Is there an official document describing the precedence of operators in OpenSCAD?

\n", "Title": "What is the operator precedence in OpenSCAD?", "Tags": "|openscad|", "Answer": "

Within OpenSCAD expressions, the order of precedence is:

\n
\n\n\n\n\n\n\n\n\n\n\n\n\n\n\n\n\n\n\n\n\n\n\n\n\n\n\n\n\n\n\n\n\n\n\n\n\n\n\n\n\n\n\n\n\n\n\n\n\n\n\n\n\n\n
OperatorsDescription
() []group, vector, or range
() [] .function call, indexing, member lookup
^exponentiation
! + -unary operations
* / %multiplication
+ -addition
< <= >= >ordering
== !=equality
&&logical AND
||logical OR
?: function() let() assert() echo()ternary operator and unary pseudo-operators
\n
\n

There are no bit-wise operators, and neither comma nor "=" are operators.

\n

Source: openscad/src/parser.y

\n" }, { "Id": "18155", "CreationDate": "2021-09-29T12:09:51.647", "Body": "

I am printing Benchies at high speed, I successfully printed one at 300\u00a0mm/s. If I set the speed to 400\u00a0mm/s, the Y axis begins shifting around. This is usually accompanied by a banging sound.

\n

In addition, the extruder motor occasionally clicks. When it clicks, the filament shoots back out a little bit.

\n

Print settings:

\n\n

I am running custom Marlin-based firmware on the stock mainboard.

\n

The printer shakes my desk when printing like this.

\n", "Title": "Creality Ender 3: Layer shifting at high speeds", "Tags": "|creality-ender-3|fdm|", "Answer": "

Ok, so you are having multiple problems, lets break it down:

\n

At the speed you are pushing, it would not be weird to have the Y belt slipping around. Also inertia comes into play, so please, dont do that.

\n

What you are mentioning about the filament going out a bit, also makes sense. You are exceeding the max volumetric flowrate for the ender 3. I would recommend going for a higher flow nozzle/Hotend combo.

\n" }, { "Id": "18173", "CreationDate": "2021-10-02T19:26:16.673", "Body": "

I have used PLA and PLA+ so far and I know that it can use ABS and PETG but I'm curious what other materials could I in theory use with my Ender 3?

\n

It is a stock configuration, for the time being at least until after Christmas, and my grandfather and I have designed an enclosure to build together.

\n", "Title": "What else can I print with my stock Ender 3", "Tags": "|creality-ender-3|material|", "Answer": "

PLA is just about the only common material that prints well on the stock Ender 3. The extruder lacks proper grip and/or torque at the hob for printing PETG well, and printing TPU with a bowden tube is very slow and error-prone. All of these should work if you're willing to go very slow (20-30 mm/s top) though. ABS and ASA are probably doable, at least with a simple enclosure, but I don't have any experience with them. Most other common materials need significantly higher temperatures than the hotend's safe limit of 250 \u02daC.

\n

However there are a couple uncommon materials that should print well:

\n\n

Finally, I guess I should mention that POM (Acetal/Delrin) is within the range of the stock hotend and extruder's capabilities, but is utter hell to print. It doesn't adhere to anything but itself, and warps badly as it cools.

\n" }, { "Id": "18179", "CreationDate": "2021-10-02T23:52:18.500", "Body": "

I am thinking of using it for a project or two but am still somewhat new to this.

\n

So, I was wondering what are the advantages and disadvantages of Eryone's 15% metal PLA and X% metal PLA in general?

\n", "Title": "What are the advantages and disadvantages of Eryone's 15% metal PLA compared to other PLA?", "Tags": "|creality-ender-3|pla|material|", "Answer": "

Advantage: It should print like regular PLA at PLA temperatures.

\n

Disadvantage: the embedded metal particles are abrasive and will widen your standard brass nozzle over time. The fix is to use a hardened steel nozzle, or for the extreme, there are options like ruby-tipped nozzles.

\n

Advantage: It should look "metallic" in a way that is more durable than a paint-on topcoat finish, with a depth instead of a surface-only appearance.

\n

Disadvantage: I can't find the product you're referencing. The nearest I can find is Metal Silk Rainbow PLA and Ultra Silk PLA

\n

That latter one is available in gold, bronze, copper, silver, etc. which sounds like metal, but only printing some will show you in person what it looks like.

\n
\n

See also Additional mods for printing metal filled filament with Ender 3 V2?

\n" }, { "Id": "18191", "CreationDate": "2021-10-04T11:15:33.853", "Body": "

I have been printing toy cars for about a month now and my Ender 3 has stopped extruding plastic even when I insert filament, it has been about a day since it stopped working. Is there any tips for getting it working again?

\n

I tried manual feeding which worked.

\n", "Title": "Printer has stopped extruding", "Tags": "|creality-ender-3|extruder|", "Answer": "

This happened to me once - no plastic would come out, the extruder was jammed. I was stumped as to what was happening, as I couldn't put any filament through the hotend, but it was definitely still hot. The problem: don't ask me how it happened, but a small screw had somehow found its way into the hotend.

\n" }, { "Id": "18194", "CreationDate": "2021-10-04T19:56:35.343", "Body": "

I am making this dog feeder.

\n

My goal is to 3D print it using an Ender 3 V2. I am using Cura to slice it.

\n

When I slice it, though, Cura does not see the Top part (Dispenser box) as a solid piece. And when I am slicing it, it does not create an infill for the ramp. I have tried using the infill support option and setting the infill overhang angle to 0, but it still does not create infill support on the ramp. Here are some prints:

\n

\"Notice

\n

\"Notice

\n

Notice that there is no infill under the ramp.

\n

How do I make Cura understand that this is a solid piece?

\n", "Title": "How do I fix infill support in Cura?", "Tags": "|ultimaker-cura|slicing|infill|", "Answer": "

I've followed the link in the question and exported the dispenser box and sliced it in Ultimaker Cura 4.12.1 and it will slice just fine.

\n

\"enter

\n
\n

From a comment on the answer from the OP I've learned later that the model has been fixed. So the issue was related to the model/STL file, not an issue with Cura.

\n" }, { "Id": "18204", "CreationDate": "2021-10-05T21:02:59.040", "Body": "

Whenever the printer homes, the nozzle ends up digging into the bed and causing damage. This also happens when I start prints, the first layer is shoving the nozzle into the bed, this happens every time.

\n

This only happens when the bed is level. It only stops when I lower the bed completely. And when I do it is too far down and the nozzle is about a centimetre away from the bed.

\n

I haven't thought much about it until now as I am thinking of replacing the bed because of the damage.

\n", "Title": "Ender 3 pro Z axis homing", "Tags": "|creality-ender-3|bed-leveling|", "Answer": "

Sounds like you have incorrectly set up your machine, specifically the Z offset.\nOr your Z endstop is physically too low and not set up properly.

\n

I'd suggest recalibrating your machine from the start as that would help dial it in as well as potentially solve this issue.

\n

I recommend following this guide at the Teaching Tech 3D Printer Site - it's extremely good. (there's also a page for troubleshooting different issues including testing your Endstops to confirm they are working)

\n

Good Luck. 3D printers can be hella finickity but all in good fun.

\n" }, { "Id": "18242", "CreationDate": "2021-10-14T18:45:17.930", "Body": "

Long story I'll try to bullet point to speed up so bear with me:

\n\n

Which brings me to my question:

\n

I want both. I want to be able to do G34 but also eventually run a fully functioning IDEX. I've already bought both the BTT E3 RRF and the IDEX board.... am I out of luck? Do you need an Octopus board or something? Or is there a way to do both with the hardware I have now?

\n", "Title": "Possible to get G34 auto align and IDEX with Ender 3 Pro and BTT E3 RRF?", "Tags": "|creality-ender-3|bigtreetech|", "Answer": "

The basic RRF board has a single Z driver which connects to two Z stepper sockets.

\n

In order to individually drive two steppers, you need an optional stepper driver like in a dedicated Z2 stepper driver or a spare E1 driver to function as the second Z driver. The basic RRF board only has four drivers, so this board alone will not bring the functionality of individual driven Z steppers.

\n

The extension board for support of IDEX gives access to another two stepper drivers, an X2 and a E1. Both are required for the second X-axis, so this expansion board will not give you a spare for the second Z driver.

\n

With the boards you acquired you can therefore not have both dual individual driven Z screws and an additional X-axis with additional extruder (IDEX) using the current onboard drivers (including the breakout board). You need a board with at least 7 stepper drivers, but, if there are free pins available you could add an additional stepper manually in firmware.

\n" }, { "Id": "18247", "CreationDate": "2021-10-17T08:01:03.133", "Body": "

I am not familiar with 3D printing at all, is it possible to print tiny objects with some 3D printing technology that is available to a consumer audience? I am not talking about microscopic scale, but about say a figurine that is 4mm tall. However, the size required for the eye of the figurine, for instance, would be in the ballpark of 0.5 millimeters (500 microns) in diameter.

\n", "Title": "Printing minuscule objects", "Tags": "|resolution|scaling|", "Answer": "

Resin

\n

Today's hobby grade resin printers are capable of amazingly tiny detail. The hole is about 1.5 mm diameter, and the pencil point in the photo below is of the same scale as the part. It was farther out of the frame and composited in to provide a better reference.

\n

This part was constructed on request of a train aficionado desiring to have hinges for the freight cars he was building.

\n

\"door

\n

Details that are not holes would be more easily accomplished. Depending on the orientation of the figurine, you can get between 0.037 mm and 0.050 mm resolution. This is due to different resolutions in the x/y plane and the z-plane. If the figurine's feature was vertical, the z-resolution could be 0.050 mm but the x/y resolution would be 0.037 mm.

\n

There's a great reference page from Chitubox describing the different technologies as well as a couple photos.

\n

\"0.047\nThis image is of a model constructed with 0.047 mm resolution. The following image is of a model constructed at 0.100 mm resolution.

\n

\"0.100

\n

Above images from linked site.

\n

FDM\u2122

\n

Referencing the comment below, it should be noted that the above information is specific to resin printers. It is very much different when discussing filament printers. Filament printers (FDM\u2122 or FFF) will have much coarser resolutions. It's typical to have layer thickness (z-resolution) of 0.100 mm. Every one of my FFF printers can print at that resolution.

\n

The x/y resolution is based on the nozzle diameter. The common nozzle is 0.40 mm diameter, limiting the resolution to slightly greater than that, as the filament will bulge as it is extruded.

\n

One can find 0.25 mm nozzles and expect approximately 0.30 mm width of deposited filament. I've read of 0.10 mm nozzles, but once you reach that level, it's time to think of resin printing.

\n

I had attempted to print the door hinge bracket on my FFF printer, using 0.100 mm layer height on a 0.40 mm nozzle. It was a complete mess.

\n" }, { "Id": "18252", "CreationDate": "2021-10-19T06:56:32.950", "Body": "

Backstory: Like quite a lot of people here, I'm having problems with parts adhering to the build plate of a resin printer. I've tried everything that I can think of: cleaning & re-sanding the buildplate at 80 grit, levelling the plate with and without a sheet of paper between the plate and LCD, warm resin, and silicone spray on the FEP.

\n
\n

In the answer to this question, Michael Wooten says:

\n
\n

PRO TIP to improve bed adhesion: apply a thin layer of resin to the build platform before you start to print. I have not any failed prints after I started coating the plate with resin.

\n
\n

I was wondering if there was any clarification on this layer of resin:

\n\n

Also, is there any known methodology? If I paint some resin onto the build plate and cure with a UV lamp, I worry that the layer won't be particularly flat. I thought about using the "cleaning" function on the printer to shine UV light through, but it would also cure area between the edge of the build plate and the tank.

\n

Any tips on this exact method of making prints stick to the plate would be greatly appreciated!

\n
\n

Update:

\n

I tried again, and it still didn't work.

\n

I left a thin film of resin on the build plate from the previous failed print, and cured it for a minute or two until it felt tacky. The resin was still warm from last print, so I ran again (after removing the failed pancakes of course) and after a few hours I could see that nothing had printed this time either. The irritating bit is that the pancakes slide off the FEP pretty easily!

\n", "Title": "Resin layer for adhesion: how should I do it?", "Tags": "|adhesion|resin|creality|", "Answer": "

According to Uncle Jessy (see Youtube channel), you simply need to pour some liquid resin over the build plate and then leave it for a couple of minutes for the worst to drip off. No

\n

He also recommends that the resin should be kept at a warm room temperature and that the bottle be thoroughly shaken before pouring it onto the build plate or into the resin vat.

\n

He also recommends that resin should not be left to sit in the vat for more than a couple of days as it can settle and separate.

\n" }, { "Id": "18253", "CreationDate": "2021-10-19T10:26:24.363", "Body": "

I'm trying to slice some models using the default HALOT BOX software from Creality, which comes with the Halot One resin 3D printer.

\n

I've oriented them at 45 degrees, and am using supports. When I come to slice the models, the models all jump up a few millimetres, as seen in the below screenshot.

\n

I thought it was just a UI bug, until the same image also appears on the printer screen, with a gap between the supports and the raft.

\n

\"Floating

\n

If I reset the position of the model, it goes back down as I would expect it to be, but when I click "slice" it moves back up again.

\n

\"One

\n

My settings are as follows:

\n

\"Slicer

\n

How do I fix this?

\n", "Title": "HALOT BOX makes objects with support move up", "Tags": "|slicing|resin|creality|", "Answer": "

Turns out I was using an old (and buggy) version of the software.

\n

The version I was using came on the supplied USB memory stick and was version 1.9-something.

\n

I downloaded the latest version (2.0-something) off the Creality website and now it doesn't appear to have the same problem.

\n

\"Yay!\"

\n" }, { "Id": "18270", "CreationDate": "2021-10-23T02:08:33.783", "Body": "

For some reason, my larger prints, or rather the ones that I create, have this "dotted" line in them. And, that line usually splits into two pieces.

\n

I use Ultimaker Cura for a slicer, I use Blender for modeling, and I have an Ender 3 Pro

\n

Let me know if anyone knows the reason for this as it's preventing me from making things on my own.

\n

Picture:

\n

\"Photo

\n

Here's my Cura file, if anyone needs it

\n", "Title": "Why is there a \"dotted line\" in my \"larger\" prints", "Tags": "|creality-ender-3|ultimaker-cura|blender|", "Answer": "

This looks to me just like standard poor reliability of the Ender 3's stock extruder, which is ungeared (torque too low), poorly tensioned, and insufficiently grips the filament (due to flat hob whose teeth only touch the filament in a single point).

\n

If you've had success with other models in the past, is is possible that they were more intricate shapes with small details? My guess is that you have the print speed set higher than your extruder can reliably deliver, but due to acceleration limits, the requested top speed was rarely or never achieved for the models you printed successfully. Now that you're trying to print something with long, straight lines, the toolhead does have time to accelerate to (or closer to) the requested speed.

\n

You didn't mention what material you're printing. If it's PETG, you're just going to have to go a lot slower with the Ender 3's stock extruder. If it's PLA, you might be able to get it to work by increasing the temperature significantly (possibly a tradeoff with other problems). If the speeds you have to drop to for reliable printing end up not being acceptable, look at getting a decent extruder.

\n" }, { "Id": "18272", "CreationDate": "2021-10-23T04:23:38.533", "Body": "

I've noticed my Ender 3 V2 printing odd little bubbles of material on curves. Is this over-extrusion? I'm using Creality PLA at 210\u00a0\u00b0C with a bed temperature of 60\u00a0\u00b0C

\n

\"3D

\n

The opposite side of the same print, with no problem in the curve. Yes, I do have lifting at some corners, but this seems to be a different issue.

\n

\"The

\n

This was printing a center-finder

\n
\n

It's a bit hard to see but here's the same blister-like look on the handles of a grabber-toy. Lighting makes it a bit hard to see, as does printing in black.

\n

\"A

\n

I have calibrated my E-steps; initially, it was extruding 95.5\u00a0mm when told to do 100.0\u00a0mm. By changing the printer's numbers, it's now pushing through 100.1\u00a0mm when requested to do 100.0\u00a0mm. Is that my cause?

\n", "Title": "Pimples/blisters on some curves - cause?", "Tags": "|creality-ender-3|print-quality|pla|overextrusion|", "Answer": "

After much faffing about, I've discovered that OctoPrint is the cause, or specifically running it on underpowered hardware.

\n

I originally thought that OctoPrint sent the job to the printer's internal SD card and it printed from there.

\n

That is incorrect, OctoPrint feeds G-code to the printer. And since I'm using a sub-standard Pi B+, it does not have the resources to keep up.

\n

By attaching a webcam, I doubled down on the resource contention and made it worse.

\n

\"Own

\n

Basically, the OctoPrint minimum-spec is there for a reason, disregard it at your own risk.

\n

The test object can be found at https://www.tinkercad.com/things/1vjIJXoQkde\nand has 5 half-barrels with different segment counts. I originally noticed that the smoother curves suffered worse, and was testing that.

\n" }, { "Id": "18285", "CreationDate": "2021-10-27T12:44:49.727", "Body": "

In PrusaSlicer-2.3.3, when there are two instances of an object closer than the support width, the intersecting surface is filled twice; this is what you can see in the center of the picture. The outer perimeters are indeed correct, but the intersection is printed twice.

\n

It would not be so problematic if this was just for one or two layers, but it's also doing it higher, i.e at the support level, and it ruined the print.

\n

So in PrusaSlicer, how do I slice several objects as one object?

\n

\"PrusaSlicer

\n", "Title": "Instances won't share support or brim", "Tags": "|prusaslicer|support-material|", "Answer": "

I found the relevant answer here

\n
\n

Did you load them as separate objects (which will do what you describe and is wrong) or add the second one as a sub part of the first object (right click, add part->load)?

\n
\n
\n

The latter takes occlusion into account and clips overlapping areas.

\n
\n

Here is what I learned: you can't merge Prusa "instances", they are just a replay of the same G-code

\n

I assume it is a really useful feature if you want to say, fill the bed with dozen of instances of the same product, if you're happy with the settings and you don't want to mess with it. This will just replicate the same g-code at some other place; that makes sense in a production setting.

\n

However, if you're still building then:

\n

Do hard copies (copy paste), reload the same file, or right click on an object, and Add Part / Load. You can merge those ones (as in: CTRL + click / a.k.a multiple selection), right click / merge , and they will slice as expected (as a whole)

\n

Keywords were : overlapping objects, composite object, redundant tool paths

\n

Also, there seems to be a bug in the prusaslicer UI, as you can't reverse the Merge - the objects look like they're trapped under the Merged object once you've committed to it. Set as separate objects is grayed here.

\n" }, { "Id": "18288", "CreationDate": "2021-10-28T13:11:54.453", "Body": "

I have a Creality CR-6 SE running Community firmware. It's printing mostly well, in part because it's improvisationally enclosed (sheets of cardboard between it and the closes window to reduce draft, which was killing my prints on bad days). To make it moderately shock-resistant it's placed on a 50x50\u00a0cm concrete tile which is placed on 3\u00a0mm of foam. The foam reduces vibrations being redirected to the tile while the tile is heavy enough not to care either way.

\n

\"CR-6,

\n

Now it has to move because I want to move my desk into the same room as the printer.

\n

This means 2 things:

\n
    \n
  1. My and my printer will be getting their required ventilation from the same window, but sitting in the same room as a printer for 8+ hours a day isn't a great idea if we breathe the same air. Either I need to scrub the printer exhaust or pipe it out (totally doable, it's not that far).
  2. \n
  3. Either I have to sit very quietly or the printer has to be placed somewhat shock-proof. The floor itself is of wood and on the first floor, so there's some minor bounce in it. It's not 100% level and it gets (slightly) worse if you stand next to it.
  4. \n
\n

I've been thinking about making a towered enclosure (roughly like the famous Lack approach). However, if I construct a tower, the increased position vector between ground and printer could make the printer shake. Besides, a simple Lack won't work because PLA won't like being confined. Where other materials like getting hot, PLA likes to be ventilated. The obvious solution would be a couple of fans that can be shut-off and covered, but maybe I'm thinking too complicated already. Either way, that's a fixable problem.

\n

See-through sheet work is likely going to be done with 2mm plexiglass (acrylic, PMMA).

\n

I'm currently printing PLA 100% of the time but the enclosure should be able to handle PLA, ABS, and PETG. There's a Raspberry Pi available, so requiring different settings for different materials is not a problem either.

\n

So I can fix all problems, except the shock one. More fans mean more vibrations, so it will get plenty of shocks by itself. Add the shocks of me walking past it, especially if the printer is in a tower, and you can wait for failed prints.

\n

I've seen people put their printers on springs, but is that the best way to deal with it in enclosures? Would it make sense to put the tower on top of the existing foam/concrete base or would it be better to catch the shocks as close to the printer as possible?

\n", "Title": "Shocks and enclosures", "Tags": "|enclosure|creality-cr-6|", "Answer": "

First, let's get your physics straight:

\n
\n

To make it moderately shock-resistant it's placed on a 50x50 cm concrete tile which is placed on 3 mm of foam. The foam reduces vibrations being redirected to the tile while the tile is heavy enough not to care either way.

\n
\n

No. The concrete tile gives your printer mass by adding its own to it. Mass by itself results in shifting the point resonance (and thus ringing) occurs, and in conjunction with a flexible mounting (the foam) results in the resonance window getting shifted out of the window where it can be problematic. Nothing about redirecting.

\n

Now, to your problems:

\n

Enclosure

\n

If you want an enclosure, you can mount it directly onto the concrete tile, or you include the buffered concrete in the enclosure. Totally up to you. If you exhaust it, just adding a big PC fan to the back and then a flexible tube to the window to get rid of warmth if you need would be enough.

\n

Dampener

\n

3 mm is way too little. You should get at least one of those rubber mats from the home depot that is put under washing machines. Those thick rubber pads are usually 10 to 20 mm thick and would decouple the printer movement very well from steps on the floor. You might even want more of a different, softer foam between that rubber pad and the concrete tile.

\n" }, { "Id": "18289", "CreationDate": "2021-10-28T15:17:22.480", "Body": "

Can you convert a 3D printer to a laser engraver?

\n

If so is there a specific thing I have to buy?

\n", "Title": "Convert 3D printer to laser engraver", "Tags": "|diy-3d-printer|laser|", "Answer": "

Probably a laser... and a mount for the laser. Then connect its power supply to be switched by the extruder supply/signal. Also, there would be no need for any Z-axis movement, beyond setting the correct height of the laser device.

\n

However, unless you are handy with DIY, messing about with microcontrollers and electronics, modifying firmware and have experience in making your own 3D printer, then it is probably easier to just buy a ready built engraver.

\n
\n

That said, and after reading Trish's (more technically detailed and accurate) answer, it probably isn't all that daunting to accomplish. In addition, if you already have a printer, then a modification would be cheaper than buying a whole new unit.

\n

Here is a handy video tutorial, which shows how easy it, in fact, is: 3D printer to laser engraver for under $60

\n

\r\n \r\n

\n
\n

This unit was tested on the Creality Ender 5, but the plug should work for any Creality printer like the Ender 3 and CR-10

\n
\n

This modification also uses the part cooling fan to control the laser and there is no need to remove the extruder - the laser attaches itself by the use of magnets to the extruder. So you can have a dual-purpose printer without that much fuss at all.

\n

The website used to generate the G-code is http://nebarnix.com/img2gco/, and it takes a simple image file as input. The settings recommended by the video are as follows:

\n
\n

\"Generating

\n
\n

Note: As pointed out in the video, the minimum power should not be zero as this turns off the laser, thereby incurring a delay upon restart. Setting the power to a sufficiently low level - so as not to burn - is preferable, i.e. 10.

\n" }, { "Id": "18295", "CreationDate": "2021-10-29T15:57:32.277", "Body": "

Is it possible to 3D print a QR code? or to engrave it using a 3D printer? I tried to convert it to individual boxes but that takes too long and is very inaccurate. Is there a better way?

\n", "Title": "QR code 3D printing", "Tags": "|3d-models|", "Answer": "

I used the qrcore2stl website for my WiFi access point.

\n

You can easily change the parameters of the QR code and associated .stl, as well as add custom text and keychain holes.

\n

For my WiFi access, I inserted a pause command at 1 mm height, and changed filament from white to black at that point.

\n" }, { "Id": "18300", "CreationDate": "2021-10-30T21:23:20.003", "Body": "

I have an Anet A8 clone printer. I see the throat tube is different on each end. I'm trying to ensure I understand which end should go up and which should go down. To me, it appears (from the photo below) the one on the right is the proper orientation (teflon down, metal up). Is this correct?

\n

\"enter

\n

There are a couple of reasons I ask this question. First, I had the teflon oriented up, and one time the teflon pulled out of the tube while I was pulling the filament out after a print was finished. Secondly, I recently experienced where melted filament was leaking from between heater block and the throat tube right after I replace the tube and nozzle. This was with the teflon down. In this case I think I didn't have the tube tight enough in the heater block (tightened against the nozzle) so there was room for hot filament to ooze out and up past the tube.

\n", "Title": "Which way should the throat tube be oriented?", "Tags": "|anet-a8|", "Answer": "

The PTFE liner should be in contact with the nozzle. Normally, they are cut about half a millimetre too long, to ensure that good contact with the nozzle is made. I used these throats for several years before I switched to an all-metal hot end. If you take care to tighten the nozzle with the hot end at the highest working temperature that you intend to use, you should have no problems with leaking filament.

\n" }, { "Id": "18310", "CreationDate": "2021-11-01T03:56:47.360", "Body": "

Has anyone been able to dye sublimate 3D printed objects? Which materials work and what products are necessary?

\n

I've been trying to find out and apparently it is supposed to be possible. But I'm not finding more detail than that. (Actually, a lot of sites say it isn't possible)

\n", "Title": "Dye sublimation printing on 3D printed object", "Tags": "|color|", "Answer": "

Yes, ABS can be dye sublimated quite easily using a press, timing is the main issue.

\n

PLA can also be dye sublimated if you're careful but will lose a lot of volume due to the heat and pressure. Not too bad if you print solid, but with infill expect the infill to collapse, or melt (unsure which).

\n

PETG is excellent for Dye sublimation. Like ABS it's all in the timing but the result is superior.

\n" }, { "Id": "18312", "CreationDate": "2021-11-01T16:29:14.127", "Body": "

I'm having a very odd problem where my part is not sticking to the raft.

\n

I'm using support material, with a very large overhang angle (70\u00b0). However I don't think this is the cause, as the side of the wheel that is on the build plate is several mm wide, and flat at that end. I need the 70\u00b0 for a weird tread pattern that I have that only needs to be supported on the very first one.

\n

I'm using PLA, with a nozzle temperature of 200\u00a0\u00b0C and a bed temperature of 50\u00a0\u00b0C.

\n

I've checked that the part is snapping to the build plate, so there isn't a gap between the part and the raft, at least not modeled to be. Looking at the slicer (Cura) it seems as though there's a 2 layer gap between my part and the first layer of my wheel, but I'm not sure why it's there? or how to remove that gap

\n

\"Layer\nLayer 5

\n

\"Layer\nLayer 6

\n

\"Layer\nLayer 7

\n

My assumption is that, if I can remove this, everything will work?? The part as it's being printed right now is looking like this, for the first layer of the part. As you can see here, my wheel is very not round.

\n

\"First

\n", "Title": "Part not adhering to the raft", "Tags": "|ultimaker-cura|adhesion|", "Answer": "

I reviewed the G-Code and there was a Z-layer command that was larger than what was supposed to be. The Z-layer change shifted the nozzle up by 3x the distance it was supposed to (0.84 mm instead of 0.28 mm). I changed it to be just the layer distance and the print worked. I have NO idea why this happened, but a solution is a solution.

\n

Interestingly though layers 6 and 7 in the images above were both at the same layer height, but the post-processing viewer identified them as two different layers. They were both at the same Z-value but were called out as different layers even though there was no change in the Z-value in the G-Code.

\n" }, { "Id": "18332", "CreationDate": "2021-11-05T23:31:21.873", "Body": "

Prints on my Ender 3 V2 have a strange visible line along the Y-axis on the top surface. You can't feel it if you run your finger over it, but can see it which spoils the appearance of the part. While printing there's no noticeable change in the motion of the nozzle as it crosses this line. It happens at the same distance across the build plate each time. Has anyone seen this issue or have a suggestion for how to resolve it?

\n

\"Print

\n", "Title": "Visible line along Y-axis on top surface", "Tags": "|creality-ender-3|", "Answer": "

I finally got to the bottom of this. I found a small scratch in the channel of the x-axis gantry (no idea how it got there). The line in the print was at the point where one of the wheels was bumping over the scratch. I carefully filed it down and the line is now barely visible.

\n" }, { "Id": "18340", "CreationDate": "2021-11-06T20:23:21.287", "Body": "

I was wondering if I can/should put my 3D printer into a wardrobe. Would it make my clothes smell? (They won\u2019t be directly next to it) . I was wondering if it could make the printer overheat, or make some other problems. Or if it could be good as an enclosure. I have an Ikea wardrobe. Should I wrap the inside with tin foil? I have an Ender 3 V2 printer.

\n

I\u2019ll be using mostly PLA, ABS, TPU and PETG.

\n", "Title": "3D printer in a wardrobe", "Tags": "|creality-ender-3|print-quality|enclosure|", "Answer": "

Yes, you can use an enclosure, many people do.

\n

PSA: Whatever you use for an enclosure, please take precautions for the potential of fire. Have a smoke detector within the enclosure or nearby. Resist the temptation to leave your printer unattended, give yourself some method for remote viewing (ex: RaspPi + OctoPi + RaspCamera). Even with a tuned printer things still go wrong.

\n

In the long run, you will be better off with a purpose-built enclosure. The primary goals of a good enclosure are visibility of an active print, access to the changing print filament, and heat control (stability). Together these imply a large doorway and windows. The goal temperature to be held within your enclosure is roughly 30-50\u00a0\u00b0C.

\n

A wardrobe is likely a thick material and will hold higher temps, and is not see-thru, making it an undesirable enclosure. DO NOT USE cardboard, unless you want to tempt fate. Aluminum foil, though flame resistant, is not sturdy and makes for poor visibility of the print. In short, don't be super cheap-o here.

\n

The concerns of equipment overheating are mostly for the power supply and controllers (ex: RaspPi). I suggest to detached them, placing them outside of the enclosure. Again, a wardrobe may be too big for the controllers & power supply to be outside and still have the cables reach the printer components. If your enclosure temp never exceeds 32\u00a0\u00b0C (90\u00a0\u00b0F), you could possibly get away with leaving the equipment attached and inside.

\n

Once you have an enclosure for your 3D Printer, determine if your enclosure needs vents. Get yourself a small thermometer and track the internal heat near the roof, during a long-running print. Add passive vents to the corners if the temp reaches 45\u00a0\u00b0C or above. Avoid active fans, as flowing air will lead to warping & separation of prints from the bed. Your goal is to have a stable temperature around your printed plastic.

\n

For reference: The particular enclosure I'm using is a simple clear Acrylic box, with a few small vent holes, measuring 18" wide x 24" long x 18" high, barely big enough for my 3D Printer. I'm running an Creality Ender 3 v1, typically with ABS using 220\u00a0\u00b0C for the hotend and 100\u00a0\u00b0C for the bed. The enclosure internal temperature rises to 35\u00a0\u00b0C, starting from an ambient room temp of 20\u00a0\u00b0C (68\u00a0\u00b0F).

\n

The acrylic makes for a sturdy box, the print progress is very visible, large easy doors and provides moderate insulation. Cost was roughly ~$75 total for 5 sheets of 18" x 24" x 0.093" acrylic (left, right, top, front, back) @ 14\\$/ea.

\n

Hope this helps and best wishes.

\n

P.S. External link to examples of many 3D printer enclosures (some good, some bad): https://www.instructables.com/workshop/howto/3d+printer+enclosure/

\n" }, { "Id": "18341", "CreationDate": "2021-11-07T00:54:32.977", "Body": "

I'm building a workspace for 3D printing for a project. I'm in the tropics so at this point it's just a large concrete pad with a roof next to my workshop. This is a very hot humid country.

\n

I'm assuming keeping it open will be as cool as possible (if that's even something that's needed) unless I air condition a room which will be pretty expensive and I'm not sure I could afford that for the months the project will run. Also less worry about any fumes. The printers will not have enclosures. Thirteen to 14-year-old kids will be doing the actual printing, I'm just setting up a production line for them.

\n

At this point, we're looking at getting Ender 3 printers and printing with PLA filament if the type makes a difference.

\n

Is there anything I need to worry about with a setup like this? I still have a while before we can get any actual printers, so I can wall it if need be.

\n", "Title": "3D printing workspace in the tropics", "Tags": "|desktop-printer|", "Answer": "

Many plastics are hygroscopic so the humidity may be a concern. You should research the effects of plastic that has absorbed too much humidity so that you can recognize it if it becomes a problem.

\n

If you are getting a large amount of plastic, I would recommend getting air tight containers to store it in (I use a 5 gallon bucket with a lid that has a rubber gasket) and desiccant and a method to re-dry the desiccant. (I use a toaster oven to bake the desiccant until it changes color.)

\n

If you actually do have problems with humidity, you may also want to get or build a filament dryer. In extreme cases, some people feed the printer directly from the dryer, but most plastics are not that sensitive.

\n" }, { "Id": "18350", "CreationDate": "2021-11-08T18:37:17.560", "Body": "

This problem started technically just before a major move but I had hoped it was a "one off" issue... I started up my trusty Ender 3 today to find it may have actually gotten worse and I'm not entirely sure what the solution is. I was printing a replica of a weapon from a game and I noticed at one point through the print I had a rather extreme X layer shift now today after finally getting the printer setup I've come to find the problem may have actually gotten worse, additionally I'm having a strange gap issue on the top most layers of my prints and strange waving patterns. Now I should note my printer is printing within an enclosure and its temperatures have been adjusted for it but this is rather extreme.\"major

\n

\"extreme

\n", "Title": "Strange and layer shifts in X axis prints", "Tags": "|creality-ender-3|pla|troubleshooting|", "Answer": "

As others have said, shifts in the X-axis are generally from belts at the wrong tension and loose frame screws. Especially after moving, I would say to check all of the frame screws and retighten them to their original torque. Moving a printer puts a lot of stress on its joints and hardware if it\u2019s not handled correctly.

\n

I highly recommend you print a quick X-axis belt tensioner for your printer. It takes no time at all and it did wonders for my Ender 3 when it was having issues with skipping and layer shifting.

\n

Even if your belts feel tight they may not be at the correct tension. This is generally from the belts stretching out over time or the printer just rattling the belt pulleys loose. After printing the belt-tensioner for the X-axis you\u2019ll be able to adjust the tension mid-print if it\u2019s still skipping.

\n

Honestly go ahead and print one for the Y-axis at the same time. Both shouldn\u2019t take longer than an hour.

\n

X-axis Belt Tensioner for Ender 3

\n

Y-axis Belt Tensioner for Ender 3

\n

If all else fails you can buy a new belt(I\u2019d recommend one with some kind of reinforcements embedded in it) for like $10 on Amazon.

\n

Also, as someone else said - it could be a connection short between your computer and printer If you\u2019re printing straight from the USB cable. I\u2019ve had a ton of trouble with that in the past which is why with the nurse three I decided to just put all of my prints on the SD card and start them from the screen on the printer, it saves on the headaches and you can at least rule out your PC and the Printing software as the culprits.

\n

Good Luck!!

\n" }, { "Id": "18372", "CreationDate": "2021-11-14T00:07:35.113", "Body": "

I have printed some connectors from PLA, in order to make a frame for a "tent" cover for my Ender3 printer

\n

Since printing rods would be a long process with likely poor results, I'm using some cheap thin-walled metal tubes as the main structure, and printing the corners.

\n

I plan on using 2 part epoxy to secure the metal rods into the sockets in my printed part, but can find no real chart of what glue to use with which FDM plastics.

\n

Is 2 part epoxy a suitable adhesive for PLA and other hard items?

\n", "Title": "What is the best glue for the task of assembling a 3D printed part with other materials?", "Tags": "|pla|glue|", "Answer": "

Confirmed - 2 part epoxy works fine between PLA and other things.

\n

I used "selleys" brand, and found that the fastest-setting 5 minute version worked fine, but on paper produces a "weaker" bond than the slow-curing one.

\n

The 5 minute version also has a distinct odour of cat pee, which is not present in the slow version.

\n

Downside, the slow version took 2 days to harden completely, but my item is definitely secured together well.

\n

https://www.selleys.co.nz/products/adhesives/heavy-duty-repairs/selleys-araldite-super-strength/

\n

NOTE that the above link says

\n
\n

Not suitable for bonding plastics such as Polypropylene and Polyethylene

\n
\n

\"Own

\n

\"Own

\n" }, { "Id": "18373", "CreationDate": "2021-11-14T10:36:22.587", "Body": "

My Ender 3 Pro started throwing the Thermal Runaway E1 error last week. Previous to that I had not modified or changed anything on the printer in months, other than move it's location. I replaced the thermistor yesterday and set off a test print but got the same error.

\n

Is this an issue with the motherboard? Or with the heater cartridge?

\n

Printer specs:

\n\n", "Title": "Sporadic thermal runaway E1 error on Ender 3 Pro, continues after thermistor replacement. What could be the cause?", "Tags": "|creality-ender-3|troubleshooting|thermal-runaway|", "Answer": "

I solved this issue by using a multimeter to measure the resistance across both the thermistor and the heater cartridge. The resistance for the Ender 3 Pro are:

\n\n

My thermistor showed a resistance within the correct value, however my heater cartridge had a resistance of 0 ohms.

\n" }, { "Id": "18376", "CreationDate": "2021-11-15T00:04:26.040", "Body": "

I am at a school with several Makerbot Replicator+ \u2013 a total of 9 of them.

\n

So, they seem to print fine and I can hook up to two of them to one laptop (they are some Lenovo models from a few years back) using Makerbot Print. Well and good.

\n

But I wanted to hook them up to my MacBook Pro (2020, OS X Catalina) with the USB cables, and the MacBook doesn't seem to "read" the printers, it's like they aren't even there. Makerbot Print (latest version) doesn't seem to "see" that they are hooked up. I checked the system prefs to see if the Makerbots showed up as connected USB devices and they don't seem to be there either.

\n

Now, I am connecting via the USB cables and then through an adapter that connects the USB-A to USB-C. If I should just use a USB-B to USB-C cable (I ordered one to test it) then fine, I'll do that. I just wanted to check that there wasn't some other problem or if anyone else had this issue.

\n

Next up: USB hubs. Makerbot says they don't recommend it, but sometimes I have to print out lots of stuff at once for student projects and I can't tie up multiple laptops for hours-long prints. I have done the technique of leaving stuff all night but that's hit or miss \u2013 if something goes wrong I am not there to stop the print (at least once something got unstuck from the build plate and I ended up with an extruder with the end encased in hard plastic like a stalactite. Unless I basically blowtorch it off... )

\n

So, the question(s) is/are:

\n

Any recommendations for USB hubs? (I would do wireless but that I am less sure of, and it seems easier, faster, and more reliable to link up through USB. The wireless connection always drops).

\n

Any recommendations for the MacBook issue? Is it just a matter of finding the right cable? (it's certainly possible my $10 USB-A to USB-C adapter plugs aren't well designed, and I should just go for direct cabling)

\n

Any recommendations for a good USB hub to link a Mac (or anything else) to Makerbots?

\n

Thanks for your time and help. I do hope I am not duplicating a post but I don't see anything in my searches that addresses the specific issues I have; though it's possible I didn't use the right search terms.

\n", "Title": "Want to connect Makerbot Replicator+ to Mac and several to USB hub", "Tags": "|makerbot|usb|replicator+|", "Answer": "

You can use Simplify3D to slice your .STL files into .X3G needed for the Makerbot. You can either connect to the printer via USB or export the .X3G file to an SD card for use in the printer. Hope this helps!

\n" }, { "Id": "18381", "CreationDate": "2021-11-16T23:57:43.777", "Body": "

I have some relay boards which I want to build into a case for my Pi. The relays are GPIO controlled and will turn on/off things like a 12 volt DC light, and the 240 volt 50 hertz AC printer itself. The relays are rated for these levels and current draws.

\n

However I would prefer to put the pi and its relays inside a case. No commercial case exists to neatly hold the relays, so I have to design and print something.

\n\n", "Title": "What filaments are appropriate for a case that will contain mains voltage wiring?", "Tags": "|pla|filament-choice|power-supply|", "Answer": "

Any filament except made-to-be-conductive metal infused filaments should be capable of acting as a suitable insulator, but you'll also want to consider things like failure modes under overheating, mechanical damage, whether it might be subject to spills of conductive fluids, etc. These are not so much a matter of filament choice as structural design and safety design on the electrical level (proper fuses, etc.), and may involve compliance with electrical code. I think those questions are outside the scope of 3D Printing SE, but would be appropriate on a sister site like DIY or Electronics.

\n

From the 3D printing side, you might want to avoid PLA unless you can guarantee shutoff if anything exeeds about 55\u02daC, since you'll quickly get warping and loss of structural integrity past that. Although I do have an under-hood automotive part in PLA (wiper fluid connector) that's held up fine for a few years now. PETG or ABS/ASA would probably be preferable.

\n" }, { "Id": "18382", "CreationDate": "2021-11-17T03:22:55.933", "Body": "

Here are the details:

\n\n

I tried dozens of temperatures starting at 205\u00a0\u00b0C and going all the way down to 180\u00a0\u00b0C pretty much 1\u00a0\u00b0C at a time. The present temperature of 188\u00a0\u00b0C seems to generate a higher stickiness to the bed than other temperatures.

\n

I have also adjusted the flow rate down to 47% and that, although it did help with the stringing, didn't fix it completely and made the whole print super weak. I have discovered going below 80% is structurally a bad idea. I wouldn't mind having it at 90% either.

\n

I am trying different retraction speeds and distances now currently testing 5\u00a0mm and 70\u00a0mm/s retraction. Previously I tried 4\u00a0mm with 60\u00a0mm/s and it didn't help.

\n

Using a lower filament flow rate (about <60%) did prevent it from going all the way across at approx 50\u00a0mm/s, but alas <60% is too weak structurally. And didn't solve it completely. The tips had a Y-shaped split top on both sides with the one side of the Y being straight as it is part of the tower.

\n

Any suggestions to rid me of this are welcome. Reminder flow rate below 80% is too weak, and 188\u00a0\u00b0C is preferred.

\n

\"3D

\n", "Title": "Fixing stringing within <80% flow print out is weak", "Tags": "|ultimaker-cura|creality-cr-10|stringing|", "Answer": "

The picture looks like classic Cura misbehavior - it's skipping retractions that need to happen because it's done "too many" already. You should be able to confirm this by looking at the gcode in an analyzer that shows retractions or watching the print. Set "Minimum Extrusion Distance Window" to 0 to fix the problem.

\n

You should not be reducing flow at all, and certainly not by more than a few percent. Put it back to 100%.

\n" }, { "Id": "18385", "CreationDate": "2021-11-17T06:03:21.683", "Body": "

I'm struggling to find the name of a connector I just broke, so I can order a new one.

\n

It's a six pin nylon terminal, that plugs into a set of header pins on a stepper motor.

\n

What do I search for to find these?! Also, how would I go about finding something like this in the future? I seem to struggle to find connectors.

\n

\"Connector\"

\n", "Title": "What is this stepper motor connector called?", "Tags": "|electronics|stepper|", "Answer": "

Short answer: It's probably 6 pin JST PH

\n

Long answer:

\n

Without seeing the socket, it's hard to say for sure, however most Nema 17 stepper motors use 6 pin JST PH connectors with 2.0mm pitch on the motor side. Many control boards use JST XH connectors with 2.5mm pitch on the board side. The two are not compatible with each other due to a difference in pitch and the locking mechanism.

\n

Looking at the sockets there is a clear visual difference: PH has a wide cutout while XH has two slots.

\n

\"PH\nPH vs XH\n\"XJ

\n

The difference on the plug side is more subtle. XH has small hooks that grab onto the slots while PH has not.

\n

There's a good reference at mattmillan.com which helps to identify different types of JST connectors.

\n

EDIT: Usually the easiest and the cheapest option is to buy a pre-made stepper motor cable that fits your control board. However if it's not available in a length you need or re-wiring is hard, you will need connectors and a crimp tool with jaws specifically made for these types of connectors. The cheapest option is probably to borrow one if you can, these can cost a pretty penny.

\n

Answering the second part of your question is tricky. Searching through parts catalogs is too time consuming. Using google image search or google lens to search with images taken from multiple angles is a good start to narrow it down but does not guarantee success.

\n" }, { "Id": "18405", "CreationDate": "2021-11-21T00:08:22.457", "Body": "

Alright, long story but I want to provide as much detail as possible:

\n

I have a heavily modded Ender 3 Pro. Mods include:

\n\n

I'm using Overture Brand white PLA and for the first print that messed up (pic below), I was using the default "generic PLA" Cura settings for an Ender 3 pro.

\n

The problem:

\n

Anyhow, it caused this kind of print:

\n

\"3D

\n

There are a few angles where it actually looks pretty good. The hull is fairly smooth, the first layer went down well, the roofs and bridging look tolerable, but as you can see, especially around the "pillars" it looks I don't know... under-extruded? Like the lines don't connect very strongly. In fact, it might be hard to tell from this photo but the bottom actually ripped off when I took it off the plate, mostly due to weak connections between each layer.

\n

What I've tried:

\n

I pretty much went step by step through Teaching Tech's calibration guide.

\n\n

Long story short, with those few changes I redid the calibration cube and the benchy and they look maybe 5% better but still weird.

\n

My Z-axis squeaks sometimes when moving through a spot 5-10\u00a0mm off the bed, so I lubricated them according to the guide here, but I did that prior to that second benchy so it didn't seem to solve it.

\n

After I first posted this, I decided to dive deeper into that squeaky Z-axis. I triple, super-duper checked that both Z threaded rods were parallel to each other and neither was warped/curved/etc... They seem fine. I lubricated them both a bit more and using G-code told the printer to jog the Z-axis up and down the length of the rods about 20ish times. The squeak did eventually go away so I printed another benchy. No dice. Still looks bad.

\n

Per Criggie's answer, I disabled the steppers and moved the axes around to see if they moved smoothly. Both X and Y move great with steppers disabled and then are pretty firm normally. Z is pretty stiff no matter what but that may be intentional. I previously had problems with my Z-axis falling down when prints were over so I installed anti-backlash nuts and with two anti-backlash nuts (one on each rod) I'm not surprised it doesn't want to fall. But again, when I just tell it to move, it moves very smoothly.\nFor instance, I just got out my micrometer and measured the Z movement. I told it to move 100\u00a0mm up using OctoPrint and measured the actual movement. Seemed to be 100 on the dot.

\n

I have a BTT filament sensor in the filament path prior to the extruder. I noticed it provided a little friction and I was concerned it was making it hard for the extruder to pull the filament. I removed it, but no improvement in print quality.

\n

One other strange thing I've noticed is the benchy always looks bad in the same spot (the pillars). Makes me wonder WHY it's always that exact spot, not randomly all over.

\n

It's got to have something to do with under-extrusion or flow. I think I ruled out the Z-axis. I made a "tall" benchy by modifying the benchy file to have a platform underneath it. This bumped it up a few cm and the problem still happened in the same spot on the pillars of the benchy (which is now in a completely different spot on the Z.\nAlso, given the "flow" preview in Cura:\n\"enter

\n

That circled low flow area is where it always messes up.

\n

So now to figure out why it's got low flow. Again, I calibrated the E-steps and it seems to be accurate. I've also printed benchy's where I told Cura to have 110% flow just to check. The last one I attempted didn't just fail, it failed big time and became a blob at that same spot.

\n

I wonder if the extruder gear is worn and slipping and/or the Capricorn PTFE tube is too restrictive for the "budget" filament brand I'm using that might have trouble sliding through it. I have replacement steel extruder gears showing up Tuesday, will update my question then.

\n", "Title": "Can't 'tune' out my problem, please help!", "Tags": "|creality-ender-3|calibration|", "Answer": "

\"enter\nIt was a worn extruder gear. I didn\u2019t think it was THAT worn, and when I loosened it a bit and slid it a few mm so that the gear hit the filament in a fresh spot it didn\u2019t improve. But new stainless steel gears showed up today and when I installed one it worked perfect. Same file/settings/gcode that had failed several times previously when testing other things

\n" }, { "Id": "18409", "CreationDate": "2021-11-21T13:39:00.673", "Body": "

My printer seems to be responding poorly to some of the prints I provide it with. At a certain time, usually after at least half an hour.

\n

For example, the print I tried today (twice) kept crashing somewhere after 52 minutes. Both times at the same spot. It would instantly reboot, ask to continue and even if I did it would crash again at the end of the backtracking.

\n

A couple of things come to mind:

\n\n

Other prints work fine.

\n

How would I go about finding what's causing the reboots?\nAs a workaround, I'd accept a method of finding out whether a specific print would have this behaviour (incl. looking at the resulting gcode if that helps), without having to waste an hour (or more) of time and material each time.

\n

System:

\n\n

Note: I feel obligated to mention the printer is on an unearthed power socket (old building, only ground floor has a couple of earthed sockets). This might make parts of the system less forgiving than it would otherwise, but I'm not sure about that. Internal wiring of the printer (for as much as it's accessible) seems fine.

\n", "Title": "CR-6 SE rebooting at fixed moments in print", "Tags": "|troubleshooting|power-supply|creality-cr-6|", "Answer": "

Turns out this is the behaviour of files that get corrupted. This has very little to do with the slicer and everything with how some SD cards are just intent on getting themselves a bad name.

\n

In this case, the file was corrupted enough not to be readable on the machine that put the data on there. If the file can't be opened and made look readable by a normal text editor, it's probably corrupted and not worth trying to print. A quick scroll-through to check it's all there might be a good idea as well.

\n" }, { "Id": "18411", "CreationDate": "2021-11-21T17:46:06.583", "Body": "

I'm returning to 3d printing and there's all sorts of new things going on. One of which is a silicone sock. I suspect it's part of the engineering where you want to keep certain parts hot, and certain parts cold, and it assists with that, but I'm not sure. Internet searches have been degrading, and have degraded a lot in the past year or so, so I can't find anything on the subject on how and why, just products.

\n

When do you need a silicone sock? What problem does it solve?

\n

My Ender 3v2 came with one, but apparently previous generations did not. My other printers do not have an option for them.

\n", "Title": "When do you need a silicone sock? What problem does it solve?", "Tags": "|hotend|", "Answer": "

A silicone sock helps keep the heat from the hot end from radiating down onto the freshly-printed parts, which have a blower fan cooling them.

\n

The sock also helps keep the hot end hot and away from that stream of room-temperature cooling air blowing at the part.

\n

As such, the sock is a thermal isolator, helping to keep the hot bits hot and the cooler bits cool.

\n
\n

Additionally, silicone rubber is pretty good at not sticking to hot filament. If your print goes bad and produces "spaghetti" then the sock will mitigate how much plastic gets stuck to the hot end.

\n" }, { "Id": "18413", "CreationDate": "2021-11-21T20:00:59.077", "Body": "

I have a local Microcenter and got some cool orange PETG+. I've never printed with PETG before, but this PETG+ stuff seems flexible and weak. I printed a random part with some thin walls, and when I squeezed it, I broke the thin walls. Interestingly, not across layer lines. The same part in ASA (or ABS? I can't recall) is sufficiently strong and durable, and much less flexible.

\n

I'm reprinting at 100% infill (the first one was 20%), but looking at the place that it broke, I don't see infill. It's ~2\u00a0mm thick there, so I'm pretty sure that it's solid. I printed at 240\u00a0\u00b0C, and my reprint is going at 250\u00a0\u00b0C.

\n

I'd like to check what this PETG+ stuff is better at than regular PETG, so I'd like to look at what the manufacturer has to say about it, and the rest of the internet. Who makes this?

\n

We know that Inland PLA+ is made by eSun, and when I mentioned that to the local Microcenter 3D printer expert, he said as much too. Here's a reddit post speculating.

\n

Product page

\n", "Title": "What OEM is Inland PETG+?", "Tags": "|petg|", "Answer": "

I talked to the guy at the local Micro Center. He said that Inland PETG+ was OEM'd by eSun, and that Inland PETG was OEM'd by Polymaker.

\n" }, { "Id": "18425", "CreationDate": "2021-11-22T14:18:17.270", "Body": "

Started up my Ender 3 today and the print killed halfway because my cat was messing with the cable to my octopi, but when I went to shut it off to reboot and the actual switch seems permanently on (as in stays on regardless of position). do I need a new PSU? and is this a known issue?

\n", "Title": "My Ender 3 Pro power switch is stuck to on", "Tags": "|creality-ender-3|power-supply|", "Answer": "

The mains switch is easily replaced - the part is widely available. Just make sure that the spade connectors go on securely and to the same positions as the old one.

\n

Here's how the wires are connected in my Ender 3's mains input:

\n

\"Ender

\n

\"Ender

\n

The IEC mains connector is on the right in the first image. You can see the yellow wire goes to the earth connection of the PSU. The black wire goes to the neutral connection of the PSU. The live input goes via a wire strip to one end of the fuse, the other end of the fuse is connected to the central pole of the switch, and the on-position of the switch is connected to the live connection of the PSU via the red wire which leaves the picture.

\n

The switch is on the left in the first image, and at the bottom in the second image.

\n" }, { "Id": "18428", "CreationDate": "2021-11-22T23:07:32.380", "Body": "

Will a 3D scanner capture internal details within an enclosure?

\n

I'm contemplating buying a 3D scanner, but they're pretty expensive so I'm looking for advice on whether they will actually be useful for the projects I have in mind. So with the picture below as an example, could I expect to generate a usable STL file by scanning this?

\n

\"Object\"

\n", "Title": "3D scanner capabilities", "Tags": "|scanning|", "Answer": "

Yes, 3D scanners can do that, provided that all surfaces are in sight of the scanner.

\n

Within our company we scan complex turbomachinery parts (e.g. impellers) using a robotic arm with 3D scanner and a rotating platform to scan the whole surface of the parts to compare used parts to new parts and evaluate if they can be used for another maintenance interval.

\n" }, { "Id": "18442", "CreationDate": "2021-11-24T22:52:17.167", "Body": "

I want to put in a date and serial number that would be difficult to erase or change. Is it better to print in the number or engrave it in with a laser engraver.

\n

Material will be anti-bacterial PLA.

\n

It will be on free masks donated to schools, I want to make sure we can identify them when they show up at the market or in shops.

\n", "Title": "Best way to put a serial number on a 3D print", "Tags": "|pla|3d-design|", "Answer": "

Have you considered using a barcode rather than text. It would be easier to print clearly as there are no curves, and would be machine readable.

\n" }, { "Id": "18451", "CreationDate": "2021-11-26T12:17:50.297", "Body": "

When looking at Marlin firmware or reading on leveling of the printer bed the term ABL and UBL are frequently used.

\n\n", "Title": "What is ABL or UBL? Is this the same?", "Tags": "|marlin|bed-leveling|knowledgebase|terminology|", "Answer": "
\n\n
\n\n

Level the bed?

\n

A level bed (twisting knobs under the bed), or a surface that is exactly followed by the nozzle is a prime requisite for 3D printing. Without a proper bed setup (not only level, but also the initial nozzle to bed surface gap; usually the paper thickness distance, but a feeler gauge will do a similar job), adhesion to the print surface might not be optimal and causes the print to get loose during printing or the first layer may not adhere at all.

\n

A leveling solution

\n

Over the years, 3D printing hardware and software has evolved to aid the 3D print operator. One such a solution is the use of a sensor (see e.g. Automatic Bed Leveling (ABL) with a sensor (BLTouch, inductive, capacitive), how does it work?). But a dedicated sensor it not a prime requisite, you can also do it manually.

\n

ABL

\n

Automatic bed leveling comes in many forms, the use of a sensor to sense the proximity of the build surface (inductive, capacitive, optical or through physical touching) enabled scanning of the build surface. In the Marlin firmware, more specific in the configuration.h file of the firmware (note this is a generic one, your printer might need adaptions!), the following on automatic bed leveling is to be found:

\n\n

Note that bilinear leveling is automatic leveling option that doesn\u2019t require you to adjust any leveling knobs (but remember that automatic leveling is not magic, you still need to provide a bed as level as possible! see Does auto leveling result in sheared prints?), but it is laborious to use in the sense you typically have to run the multi probing points prior to printing, which can take some time depending on the grid size (see How to increase the amount of probing points for a BLTouch sensor in Marlin firmware?).

\n

This is where Unified Bed Leveling comes in.

\n

UBL

\n

Unified bed leveling (UBL) introduces a mesh-based software procedure in Marlin firmware similar to (ABL) bilinear leveling but with some extra features. From the configuration.h file:

\n\n

A UBL mesh is generally using more probing points than the (ABL) bilinear leveling equivalent to create a better digital representation (topography) of the used build surface. Bilinear meshes typically use 9-25 probing points, UBL meshes generally use 81-100 probing points to scan the surface more thoroughly. Note that this is up to you how you define this, the bottom line is that UBL is not something you do prior to every print, so you can get away with having some more probing points. Note to choose a value that is in line with the surface (type and size), for a flat glass surface 100 probing points might not be necessary.

\n

Essentially, the operator is presented a means to interact with the 3D printer to control the leveling to allows to easily store (save up to seven meshes), adjust, and swap multiple digitally stored leveling meshes on your printer to encounter the various print surfaces you may use like in multiple build plates.

\n

The main difference between UBL and the ABL systems is that UBL combines features of other leveling techniques to provide users with more control. With Unified Bed Leveling, you\u2019re not technically required to have an automatic bed leveling sensor. Through the interface, you can manually create the mesh! As the machine is homed, the coordinate system is fixed, movement relative to the reference is used by each probing point.

\n

With so many points, creating UBL meshes can take a long time compared to other systems (certainly without a leveling sensor), but once the process is done, you don\u2019t have to redo this again as the UBL features allow you to modify the probing points.

\n

The G26 command is designed to use with mesh enabled leveling procedures (Marlin < 1.1.6 AUTO_BED_LEVELING_UBL and since Marlin 1.1.7 it also works with MESH_BED_LEVELING and AUTO_BED_LEVELING_BILINEAR). The various options are described in the Marlin implementation of G26.

\n
\n\n
\n

To sum up, ABL and UBL are the same in that some sort of procedure is used to scan the topography of the bed surface but they differ in the sense that for ABL you are required to have a automatic leveling sensor and choose less points while you can have more probe points using UBL including advanced features as editing and storing multiple meshes.

\n" }, { "Id": "18453", "CreationDate": "2021-11-26T12:45:59.397", "Body": "

A 3D printer needs to be homed (homing) before the print starts.

\n\n", "Title": "What is homing? What is the purpose of homing?", "Tags": "|homing|knowledgebase|terminology|", "Answer": "

Most 3d printers control head position using stepper motors and end stops with no position feedback. The stepper motor does not actually know its location.

\n

The printer's control system can only know the location of the head by keeping track of the relative number of steps the head has been moved by the stepper motors.

\n

Homing moves all the axes in one direction until it hits the end stops for each axis. Once that has been done, then the control system knows exactly where the head is and future relative motions can be offset from there.

\n

Some control systems will move the axis at a relatively high speed (so you don't get bored, but not fast enough to cause damage) and then overshoot the end stop's activation point and stop. Then they will back off to release the end stop and then move towards it more slowly to get a more exact reading of the activation point.

\n

When the printer is idle, frequently the stepper motors are unpowered to save energy and keep the motors cool. When the motors are unpowered, they can free spin (you can push the axis with your hand or the vertical axis might even slip a bit with gravity), so the position would be uncertain, and it would be necessary to re-home. Once it has rehomed, the control system will keep the motors powered to prevent slipping.

\n

It is only necessary to home if you care about where the printer starts printing. In theory, you could get away with only homing the vertical axis (so the print starts on a surface and the first layer is the correct thickness), and manually set the horizontal start position. However, this might make using the full horizontal limits of the build area difficult if you didn't start it in the right place.

\n" }, { "Id": "18466", "CreationDate": "2021-11-27T20:44:39.323", "Body": "

I encountered the following question in a manufacturing course.

\n

\"enter

\n
G20 G90 G28 \n\nM06 T1 \n\nMO3 S1000\n\nG00 X0 Y0 Z-0.25 \n\nG41 D1\n\nG01 X1 Y1 Z-0.25 F3\n\nG01 X5 Y1 Z-0.25 F3\n\nG01 X5 Y3 Z-0.25 F3\n\nG01 X1 Y1 Z-0.25 F3\n\nG00 X4 Y0 Z0.25 \n
\n

But when you type the G code in NCViewer the shape produced is not a rectangle. Also, I am not sure how the end mill diameter affects that of the rectangle. Can anyone explain how to solve this problem?\n\"enter

\n", "Title": "Interpreting G-code to answer question about geometry of shape produced", "Tags": "|g-code|cnc|", "Answer": "

There is no rectangle, the assignment is wrong. In order to make a rectangle, the G-code should have been:

\n
...\nG01 X1 Y1 Z-0.25 F3\nG01 X5 Y1 Z-0.25 F3\nG01 X5 Y3 Z-0.25 F3\nG01 X1 Y3 Z-0.25 F3 ; <-- missing line\nG01 X1 Y1 Z-0.25 F3\n...\n
\n

Then the rectangle would have been 2 by 4 inches.

\n
\n

As noted in the comments, the assignment is even more unclear. The code shows an offset to be taken into account of tool D1 in the cutter compensation code G41. It further doesn't specify the characteristics of that tool. If it is differently than the end mill currently in the tool head, than the difference in diameter between the D1 tool and the current 0.5" tool needs to be taken into account. The power of using cutter offset definitions is that the same code can be used for different tools, you only needs to set the correct offset. If the D1 tool is defined as a 1" diameter tool, the current tool is half that size, as there is a compensation for 1" at play while a half inch end mill is used, the end product will be larger.

\n" }, { "Id": "18468", "CreationDate": "2021-11-28T00:11:35.417", "Body": "

I am trying to print this really cool flying toy model on my ToyBox 3D printer, but when I try to print the "copter_key-175.stl" file it complains it is a "non-manifold shape". How can I fix this .stl file?

\n
\n

[update START]

\n

Update for future readers:\nI haven't tried it yet, but the Free and Open Source slicer software, Slic3r, boasts this feature worth trying:

\n
\n

auto-repair of non-manifold meshes (and ability to re-export them);

\n
\n

Update again: the best slicers, it seems, based on my research, are:

\n
    \n
  1. Cura (FOSS and professionally supported),
  2. \n
  3. PrusaSlicer (FOSS, forked from Slic3r, and also professionally supported now).
  4. \n
\n

Articles to look at:

\n
    \n
  1. Google search for "slic3r vs cura"\n
      \n
    1. Slic3r vs Cura (Cura wins; Slic3r lags due to no full-time company-sponsored development): https://all3dp.com/2/slic3r-vs-cura-3d-printer-slicer-software-shootout/
    2. \n
    \n
  2. \n
  3. Google search for "prusaslicer vs cura"\n
      \n
    1. PrusaSlicer vs Cura (Cura wins, but just barely, since both are professionally supported with full-time developers): https://all3dp.com/2/prusaslicer-vs-cura-differences/
    2. \n
    \n
  4. \n
\n

[update END]

\n
\n

Note that I have printed many ToyBox-designed models perfectly with this printer over the last 24 hrs.

\n

I have also split the model (to cut the last few cm off the end and shorten it) using this technique here in TinkerCad, then exported the part as a shorter part so I could print on the smaller bed of the ToyBox printer.

\n

\r\n \r\n

\n

Here is what the "key" is supposed to look like:

\n

\"3D

\n

And here is how it comes out instead. Notice the misaligned teeth and layers about halfway through. Once I saw it was botched, I stopped the print early.

\n

\"Side

\n

\"Top

\n

\"Angled

\n

\"Back

\n

How can I make it print properly and/or how can I fix the .stl file?

\n

Notes:

\n\n

Related:

\n\n", "Title": "Non-Manifold .stl file won't print properly on my ToyBox 3D printer", "Tags": "|print-quality|toybox|", "Answer": "

Update 24 Mar. 2023: the maximum usable print volume of the ToyBox 3D printer is approximately 74\u00a0x\u00a079\u00a0x\u00a0100\u00a0mm (width x length x height), and you must add supports to keep tall objects from tipping over and catching

\n

1st, you must add supports to keep tall objects from bending over and catching the nozzle, like this:

\n

\"enter

\n

Notice the side supports. Without them, this ball will not print properly. Between 60% and 80% done, it's so tall that the nozzle will eventually catch on the ball edge, tip it over, and then get jammed against it, causing it to think it hit the edge of the print volume, and forcing the center point to shift several mm in the opposite direction. This results in print sheering and possibly damage to the stepper motors or gears.

\n

2nd, the digital bed size of the ToyBox 3D printer (when you go to load an object to print) is 75 x 80 x 100 mm (7.5 x 8.0 x 10.0 cm).

\n

When printing large objects up to 100mm in height, it seems to work fine. So, that limit is ok. But, when printing large objects which are the full width, ex: a 75mmx75mm circle, the object will get destroyed once the max diameter portion of the circle is reached. This is because the hard mechanical stops (screws, and metal edges of the print space) and/or limit switches of the printer will be hit, causing the print head firmware to adjust the center print point by several mm in the opposite direction. The object will then print with misaligned layers from that point forward. Since the center point just jumped several mm, the end point will be hit again on a future pass on the opposite side, causing it to jump again. It will jump like this repeatedly until the print is complete. You end up with shifted layers as shown in the photo below. This is a 75x75x75mm ball, which printed fine until the max diameter center portion was reached, at which point the mechanical edge stops were hit, the center point was (presumably) readjusted by the firmware, and the layers sheered:

\n

\"enter

\n

So, when printing, shrink the object 1 or 2 mm (5%, or 3.75mm of 75mm is plenty, and 1%~2%, or 0.75mm~1.5mm of 75mm is probably enough) and be sure to add an outer ring as wide as or just wider than the max diameter of the object. Ex: here is a manually-added outer ring to ensure the print head starts and stays centered at the beginning:

\n

\"enter

\n

In conclusion, the maximum usable print volume of the ToyBox 3D printer is approximately 74\u00a0x\u00a079\u00a0x\u00a0100\u00a0mm.

\n
\n

Back to my original answer:

\n

ToyBox 3D printer true print dimensions and limit switch problems

\n

So I figured out that the problem is my print area is so small the printer was occasionally fully pressing and triggering the end-point limit switches! This apparently causes it to stop slightly early, shifting the next layer as it prints.

\n

The printer specs state that it has a print volume of 9 x 8 x 10\u00a0cm. However, the print design area and viewer at https://www.make.toys/ shows the design volume like this

\n

\"Screenshot

\n

In the square grid shown on the base plate there is 1\u00a0cm per square, except that as you can see, the far left and far-right columns are < 1 square. Therefore, looking at this image, the usable bed area appears to be about 7.25\u00a0cm x 8\u00a0cm (the grid size shown in the images), NOT 8\u00a0cm x 9\u00a0cm.

\n

Furthermore, if you do NOT have the "Skip First Ring" option checked on the "Build" tab in the image below, the printer does a "wipe clean" maneuver in the shape of a spiral, circle, or ring around the object to be printed just before beginning the print. This wipes off any dangling stray print material before beginning the print. If your object fully covers the print bed dimensions, however, that ring will be even wider, causing the printer to hit its limit switches.

\n

(The model in this image is 10.4\u00a0cm long, which is too long, causing my printer to hit its limit switches, skewing the layers while printing.)

\n

\"Screenshot

\n

So, the problem is that my part is too big. WithOUT that "Skip First Ring" box checked, the dimensions are probably further reduced by another 5\u00a0mm or so on X and Y dimensions, bringing it down to about 6.75\u00a0cm x 7.5\u00a0cm usable print area. My part was 10.4\u00a0cm long. The Pythagorean Theorem says that $A^2 + B^2 = C^2$, so $C = \\sqrt(A^2 + B^2)$ = $\\sqrt(6.75cm^2 + 7.5cm^2)$ = 10.09\u00a0cm max on the diagonal. My 10.4\u00a0cm long part was too long. The printer hit the limit switches, botching the layers.

\n

Had I checked that box maybe I could have gotten away with a part closer to $\\sqrt(7.25^2 + 8^2)$ = 10.8\u00a0cm long, but that's really pushing the limits of this printer. In the end, shrinking the part a bit more to be about 10\u00a0cm or less was all I needed to do!

\n

UPDATE: I've also proven conclusively by designing in www.TinkerCad.com and printing on the ToyBox that the max allowed print height is exactly 9\u00a0cm, and it will indeed print properly all the way up to that height.

\n
\n

I'd still like a flying propeller

\n

That being said, even though the "key" of the model in my question printed pretty well in the end, the helicopter blade (propeller) printed horribly because the design is flawed and has a bunch of missing material and air gaps around the hub, making the propeller completely unusable!

\n

Instead, I switched to this thing shown below, shrunk it down to 0.6x to fit my printer, set my printer settings from medium to fine resolution, and got pretty good results! I still need to further tweak and edit the design on https://www.tinkercad.com to give the pull key better clearance, and better connection with the hub gear, and I think I'll be able to get a great result! I got it to fly a few times up to 8 ft high or so, but the pull is very rough and inconsistent, so the model needs further tweaking.

\n

\"Screenshot

\n

Keywords: ToyBox 3D printer helicopter and toybox printer clearances, print dimensions, print volume, specs, print settings

\n" }, { "Id": "18479", "CreationDate": "2021-11-29T22:19:47.293", "Body": "

I have a 3D printer and I want to 3D print with metal infused PLA filament. It seems that this filament does not print that well (it won't melt to the right consistency). I referred to my manual and it does not talk about plastic or metal filaments. Is using metal causing the problem or is it something else.

\n

The printer I have is a JGMAKER Magic 3D Printer DIY Kit with Filament Run Out Detection Sensor and Resume Print Metal Base 3D Printers for Hobbyist Education.

\n

I am printing these Christmas balls in different colors for the holidays.

\n

The metal filament is Silk Copper 3D Printer Filament PLA (1.75\u00a0mm) Shine Silky Shiny Copper Like PLA CC3D ZHUOPU

\n", "Title": "Printing metal infused PLA does not work well", "Tags": "|pla|filament|filled-pla|", "Answer": "

This is is PLA, and if you can print regular PLA then this metalised one should print too. However if its a different brand then you may need to increase the hotend temperature in small steps.

\n

I'd suggest you load the filament, and then turn on the heaters to bring it all up to temp. Then use the extruder feedingn knob to force some filament into the hotend. It should come out like hot cheese, and if not then increase the hotend temp till it does.

\n

In your slicer, it should be possible to set up a profile for this material with settings it likes, to ease future printing.

\n
\n

Your comment say it drips, which suggests the hotend temp is too HIGH and the plastic is liquifying. Start at 140 degrees C and then work up from there, not down.

\n" }, { "Id": "18482", "CreationDate": "2021-11-29T23:01:16.873", "Body": "

Is it possible to color a 3D printed object during 3D printing? I am trying to achieve an ombr\u00e9 of many colors.

\n

I tried to stop the 3D printing at individual stages and it looks like blocks. Is there a way to achieve a near perfect ombr\u00e9 or is it not possible yet?

\n

The thing I am printing is Spiral Christmas Balls.

\n

I am trying to achieve something like this effect:

\n

\"Color

\n", "Title": "How to achieve ombr\u00e9 (or shading) effects?", "Tags": "|color|", "Answer": "

The ability to shade 3D printed objects is dependent on the printer, the plastic in use, and available dye.

\n

Several vendors offer filament with a gradient or rainbow-colored patterns, so that printed objects will vary in color over their volume.

\n

Several 3D printers give the option of changing plastics during printing which would allow this.

\n

A few 3D printers allow injecting dye into the plastic during the printing process, allowing full 3d coloring of the final part.

\n

If you can find a natural colored filament and a dye that will dissolve in the filament, it is possible to color the surface of the filament before it goes into the printer, and there is some mixing in the nozzle. (This works well with natural-colored ABS and Sharpie markers, but I haven't found a combination that works well with PLA.)

\n" }, { "Id": "18488", "CreationDate": "2021-11-30T18:54:03.983", "Body": "

I've temporarily acquired two different 3D printers that I'm helping someone reassemble. They are delta style printers that at one point worked great. They were being stored in a large community warehouse type place and somebody that shouldn't have had access pilfered them for parts. Well anyhow I've got them 99\u00a0% of the way reassembled. Everything physical is back into the right place and now I just have to wire it.

\n

Here's a photo of the board:\n\"enter

\n

It looks like a "BT7200 V1.8.26 Ramps Plus2" board. I also recently bought that pictured meanwell power supply.

\n

I googled the board and instead of a wiring diagram or anything just found several similar questions to this like "anyone familiar with?", "marlin pin details?" etc...

\n

If anyone is either familiar with the board that can point me in the right direction for guides as far as these style/brand of boards? What the heck even are "RAMPS" boards? The only thing I've ever wired up is Creality and BTT mainboards meant for an Ender 3.

\n", "Title": "Pin layout of RAMPS Plus2 board?", "Tags": "|electronics|delta|ramps|", "Answer": "

RAMPS boards are one of the oldest types of boards, in Marlin firmware there is a whole section available. Actually, the first were shields that were pressed on on existing Arduino atMega board. Later, the microprocessor was placed onto the shield to become fully integrated boards. Basically, pin layout is very similar for these boards but small differences exist. In Marlin software pin definition you see that the basic layout is read, after which small changes are read over.

\n

The board can be found by e.g. searching for \u201cHE3D Ramp Plus 2 mainboard for the tricolor/K200 DIY 3D printer kit\u201d and appears to be fully compatible with the RAMPS 1.4 pin layout, which is found here.

\n

Snippet from the description by the seller:

\n
\n

Ramps PLus2 is suitable for all kinds of 3D printers, such as UM, Reprap, Detla and Kossel, CoreXY and HBot, SCARA and so on.\nFour layers of exquisite workmanship, from millions of automatic patch production line.

\n

Functional characteristics:

\n\n
\n

\"enter

\n" }, { "Id": "18512", "CreationDate": "2021-12-04T23:22:05.587", "Body": "

I am looking to buy an Ender-3 Pro, but the place I'm looking to buy from has this picture:

\n

\"Confusing

\n

My question is, what is meant by "Junior" in the name?

\n

I'm just wondering if this means it's an alternative kind of version for kids or something, or if it's just the name of that type of 3D Printer (e.g. in the way you might have a Junior Hacksaw or something)? I.e. is this the normal Ender V3 Pro?

\n", "Title": "What is meant by \"Junior 3D printer\"?", "Tags": "|creality-ender-3|", "Answer": "

That is an Ender 3, with the vertical PSU, the older interface, and many tweaks like the drawer and filament loading knob isn't present.

\n

Consider the Ender 3 V2 instead, which is an iteration on the same basic design and is newer.

\n

Personally I found it cheaper to buy the V2 direct from Creality's website than to buy the older model from a local or other online seller, and it had free freight from China, and arrived in about 3 weeks.

\n" }, { "Id": "18528", "CreationDate": "2021-12-07T18:03:27.053", "Body": "

I'm contemplating buying a laser engraver attachment for my Ender 3 Pro. Either the Creality official one or the Comgrow version.

\n

When scanning the respective web pages/answers on here and YouTube videos, it seems like you usually unplug cables for fans and swap them with laser, etc.

\n

So I guess my question is: if I still wanted to be able to easily print, let's say within 10 mins of engraving something, without opening up my printer's electronics case to swap cables every time... what would be the best approach?

\n

Is there a way to have everything permanently plugged in and the G-code file simply dictates what is done (3D printing vs lasering).

\n

I have a different board than the default Creality if that changes anything (a BTT E3 RRF Board). I also have the IDEX expansion board plugged in, that's currently only running a second Z-axis motor (i.e. probably has extra plugs on it).

\n", "Title": "Laser engraver on Ender 3 without taking important ports", "Tags": "|creality-ender-3|laser|", "Answer": "

You don't need to open the electronics casing, why not cut/split the (print) fan cable so that you can plug it quick without too much delay. The power for the laser could be fed directly from a power supply.

\n" }, { "Id": "18531", "CreationDate": "2021-12-09T03:36:56.110", "Body": "

I have been trying to make some small signs, and to highlight the text by changing between black and white filament at a layer just above where the text comes out of the back plate.

\n

I've used Cura 4.12 and the "change filament" script to make the printer pause at the right layer. The change and purge process works fine, no issue there.

\n

However the second colour adheres poorly to the first. Doesn't matter if I print Black then White, or White then Black. Both filaments are the same brand.

\n\n

To save the print I've used superglue to stick the letters back on that have fallen off, but is definitely not ideal.

\n
\n

Here's an example:

\n

\"enter

\n

The loose letters are held on well enough for printing, but only barely. They can knock loose at a touch of a finger afterward.

\n

I have tried changing filament on the first layer that is not the back board, and one or two layers up the letters. One print I even changed filament a layer too low, and surprisingly that did not have adhesion problems. Perhaps it is merely a function of surface contact area ?

\n

Gluestick on the exposed surface while changing filament was useless - the whole thing simply didn't stick and the second colour never got any adhesion at all, merely wiping off.

\n

As for timing, it seems to make little difference if I'm there waiting for the last layer to print, or if I let the printer sit for half an hour before noticing and changing colour. The bed stays hot at 50 degrees, and the hot end is permitted to cool off.

\n

There is a heating cycle, and then a purge/discharge-into-air that shows the transition between colours, so its not that the filament has lost a prime.

\n

At the moment, a much more effective solution is to use a paint marker and colour-in the top layer with a contrasting colour.

\n", "Title": "What techniques are needed to minimise layer separation after filament change?", "Tags": "|creality-ender-3|ultimaker-cura|adhesion|change-filament|", "Answer": "

After much fiddling about, the only positive conclusion I could come up with was to not make the letters too thin. A chunky thick letter has more surface area to adhere, whereas a thin spidery letter is too fragile.

\n

So print fewer words on each label, make the words more-bold, and if they still fall off after printing then CA glue/superglue is good enough to hold them together.

\n" }, { "Id": "18538", "CreationDate": "2021-12-11T17:09:01.297", "Body": "
\n

Update following advice from @R..GitHubSTOPHELPINGICE in the comments - I've increased the Z-offset from -2.97 to -3.10 and all now appears to be much better.

\n
\n\n

The issue, as an example, I'm trying to print a case for a Raspberry Pi which has air vents on the bottom. When the print starts, it does the line down the side which the first ~ 10\u00a0mm doesn't stick, but the rest does perfectly. It prints the skirt, which again the first ~ 10\u00a0mm doesn't stick, but the rest does, then when it gets to doing the vent slots, the same again.

\n

The photo hopefully shows what I'm trying to explain.

\n

\"Initial

\n

When printing a raft, again the first ~ 10\u00a0mm of the extrude doesn't stick, but the remaining does.

\n

\"Raft

\n

Once the raft is printed, the first layer sticks to it perfectly well

\n

\"First

\n

The bed is pretty level, I'm using a BLTouch to assist and has been cleaned with alcohol before the print. Bed has been levelled and the Z-offset calibrated to -2.97.

\n

I have Enable Retraction on, 5\u00a0mm, 45\u00a0mm/s speed. I've tried increasing and reducing these, increasing/decreasing print speed, manual Z-axis offset fine adjustments, etc.

\n

I did think First 3 mm prints poorly, then fine after that was kind of the same issue, but there's no discussion of the issue being for every touch of the bed within the same print job.

\n

Any advice would be appreciated. I've loitered this community for quite a while and lots of Google searching hasn't come up with any suggestions (there's actually very little I could find on this kind of troubleshooting).

\n", "Title": "Filament not adhering to bed for the first 10 mm after every move", "Tags": "|creality-ender-3|heated-bed|adhesion|build-plate|", "Answer": "

From the pictures it looks like the distance between your bed and nozzle is too great. The way I see this is that the extrusion lines that should be pressed against each other have small gaps between them, and look rounded rather than flat. The only other way this could happen is with severe underextrusion/wrong esteps-per-mm, but that seems unlikely.

\n

Note that "automatic bed leveling" fundamentally cannot help you get this right because it depends on a quantity it can't measure, the Z offset between the bed leveling probe and the nozzle tip. At best it can help you get the distance consistent[ly wrong].

\n

This is also consistent with raft fixing the problem, since the raft has extra wide extrusions in the first layer whose width will just vary if the leveling is wrong, and after that, all print heights are just relative to the top of the raft.

\n" }, { "Id": "18540", "CreationDate": "2021-12-12T01:35:50.060", "Body": "

I literally just started OpenSCAD today, so please take it easy on me, but is shell scripting possible with OpenSCAD? as in, to write a script in the OpenSCAD syntax, and have it output images, or animations? with or without having to render the image.

\n

I've been reading the man pages, and I'm not sure if that can be done.

\n", "Title": "Is shell scripting possible with OpenSCAD?", "Tags": "|openscad|scripts|", "Answer": "

No. OpenSCAD is not a scripting language. You cannot use it to generate any kind of executable code. All that it does is to create static geometrical objects that can be exported as STL files (and other formats). If you want to do scripting, use a scripting language such as Python. Note that Python has some wrappers for OpenSCAD that may allow you to do what you want. For example, see OpenPySCAD.

\n" }, { "Id": "18543", "CreationDate": "2021-12-12T09:18:29.137", "Body": "

If I want to make 3D printable objects out of my old drawings, what are my options? Do I need to redraw the entire thing in 3D or is there a better way?

\n

This is an example of the sort of pictures I have, I long ago lost the original Macromedia files so I just have them like this.

\n

\"example

\n", "Title": "3D from 2D picture options", "Tags": "|3d-design|", "Answer": "

I presume that what you are saying is that you only have a rasterized image file. For example a .PNG file, and that you don't have any kind of source file containing vectors.

\n

Since you're already a skilled artist from the looks of it, you're largely limited by the software that you have access to, or how much you're willing to spend on new software.

\n

Probably the simplest and cheapest way would be to use a free program such as Inkscape to redraw your image as lineart only (You can import the image into the software and then trace over it). Draw it completely flat and 2 dimensional with no shading or colors. Only lines to represent edges. Save it as a Structured Vector Image (The default file for Inkskape). Then import it into a free 3D package such as blender.

\n

This will import your image as a series of 2D lines in a 3D space. You can then use traditional 3D art skills (Which you may or may not have yet) to give it 3 dimensions.

\n

A complex shape like that might require a resin printer to print due to the high level of detail. Making it wouldn't be a task for the faint of heart.

\n

A more expensive option would be to commission a professional to do it for you.

\n" }, { "Id": "18552", "CreationDate": "2021-12-13T13:56:08.087", "Body": "

Subtractive manufacturing has been the way of the world for a while, but additive manufacturing, which is synonymous with 3D printing, has disrupted all of that.

\n

If extrusion-based printing is considered additive, then can we call resin printing additive?

\n

Resin printers start with a resin bath and then essentially "subtract" the material it needs (via UV light). This is just a matter of terminology, no big deal.

\n", "Title": "Is resin 3D printing considered \"additive\"?", "Tags": "|extrusion|resin|terminology|", "Answer": "

It really took me a little while to understand why you thought that it was subtractive, but I can now see your point. Which is, that seeing as the resin model is created within the tank/bath, you are saying that the bath is analogous to a block of aluminium which is whittled away, using CNC, to make a 3D model. The resulting aluminium model was once within the block of aluminium, as was the resin 3D model - therefore, you posit, resin 3D printing is subtractive.

\n

The more I thought about it, the more it seemed as if you were, actually, correct.

\n

However, then I realised that we were neglecting the hardening process. It is the hardening of the resin that is done in an additive manner, in order to slowly build up the model, therefore the process is additive.

\n

You aren't removing/subtracting non-hardened resin from the model (which would result in a non-hardened wobbly model), but instead, as user10489's answer states, you are adding hardened resin layers.

\n" }, { "Id": "18562", "CreationDate": "2021-12-14T17:28:14.703", "Body": "

How hot is too hot for any given magnetic layer holding a steel bed surface? Are certain types or grades of magnets more or less suited for this purpose? How can I ascertain which kind I have if the manufacturer doesn't tell me?

\n

I've heard that some magnetic beds should not be used for certain materials because those materials require a very hot bed for adhesion. But in the modern age where resellers and even manufacturers don't usually know what they are selling, what are the rules of thumb, and where are the limits?

\n", "Title": "How hot is too hot for magnetic beds?", "Tags": "|heated-bed|", "Answer": "

Depends on the magnets used

\n

Magnets demagnetize under heat and are fully demagnetized once their Curie temperature is reached. This Curie temperature depends heavily on the type of magnet, and it is not the point where they already suffer demagnetization but where they have none left. This means, that the useable temperature band is much lower - and in some cases, low temperature can negatively impact a magnet's holding force.

\n\n" }, { "Id": "18567", "CreationDate": "2021-12-15T05:54:41.370", "Body": "

I have a new Creality Ender 2 Pro with a Creality 4.2.3 mainboard. I'm attempting to compile Marlin to fix a bug. How can I tell what driver chips I have on this board?

\n

I've narrowed it down to likely A4988 or TMC2208_STANDALONE or possibly the TMC2225. Strangely Creality only has documentation for the 4.2.2 and the 4.2.7 boards (not 4.2.3)

\n

4.2.2 => TMC2208
\n4.2.3 => ?
\n4.2.7 => TMC2225

\n

Some say you can tell the driver just by listening to the noise it makes. Here is a video of the printer running. The motors are nearly silent to my ear.

\n

\"Photo

\n

\"Top

\n

Resources

\n\n", "Title": "How to tell if I have TMC2208 or A4988 Drivers on Creality Silent V4.2.3 Board?", "Tags": "|stepper-driver|creality|", "Answer": "

MS35775 appears to be TMC208 compatible. You can find the data sheet on relmon.com here is the overview:

\n\n" }, { "Id": "18575", "CreationDate": "2021-12-16T21:58:00.310", "Body": "

It seems there are some missing lines on the outer wall on the Z-axis with my prints. I'm not able to pinpoint the problem. Does anyone have ideas about what might be wrong with my setup/settings?

\n

Example:

\n

\"Printed

\n

Here are some settings that I think are relevant:
\nPrinter: Ender 3 v1
\nFilament: Das Filament
\nSlicer: Cura

\n\n

Cheers

\n", "Title": "Missing outer wall on some layers?", "Tags": "|creality-ender-3|ultimaker-cura|z-axis|", "Answer": "

The OP solved the problem by increasing temperature from 210 to "217-218". While it's good to have it working now, this likely suggests other problems with the printer thaat should be investigated.

\n

If the change in temperature made the difference to get this print working, your extruder is just marginally able to push a sufficient amount of material through the hotend at the speed and temperature you're at. Normally I would expect an Ender 3 to do somewhat better, even with the stock extruder and hotend. Here are a few things you might want to check:

\n\n" }, { "Id": "18582", "CreationDate": "2021-12-17T15:33:32.567", "Body": "

What would the best filament for an Ender 3 V2 be? I don't mind about the look but I would like for you to be able to bridge with it and for it to be reasonably cheap. If possible could you give several different options at different prices, different qualities and could you describe which website/company you can get it from.

\n", "Title": "What filament would be best for Creality's Ender 3 v2?", "Tags": "|creality-ender-3|filament|filament-choice|creality|", "Answer": "

Sorry - no one here can tell you to buy brand X from website Y.

\n

However there are an enormous number of options and eliminating some broad categories can help.

\n
    \n
  1. Presuming your printer is stock, it has a brass nozzle, and therefore anything "reinforced" or abrasive is not feasible. That excludes carbon fibre or nylon-reinforced filaments.

    \n
  2. \n
  3. ABS is probably unprintable, unless you've added a heated enclosure

    \n
  4. \n
  5. TPU might work, but it has properties that suit certain kinds of jobs, like flexible phone cases. If you're not printing those things, TPU is wasted.

    \n
  6. \n
  7. PETG is also a maybe - I have no experience with it.

    \n
  8. \n
  9. PLA is the best for printing on an entry-level printer like an ender3.

    \n
  10. \n
\n

You can eliminate all 2.85mm and 3mm filament, because your stock nozzle is a 1.75mm

\n

I've personally not tried TPU or PETG on my ender3v2, mostly because committing to a whole roll is an expense I can't justify.

\n

If I were you I'd ask anyone locally who prints, "where do you get good filament?" and use that as a starting point. Ask your local library if they have a 3d printing service (this is astonishingly common) and where they source filament.

\n

Some people only use the cheapest filament available, others have preferred brands, and others use only premium supplies. Figure out what your personality is.

\n

Going cheap is reasonable if you're only toying about. If this printer is doing real work for people, consider stepping up to something better - cost of failed prints will outweigh the cost of better filament as your skills improve.

\n

I would suggest exploring different styles of PLA, like the metallic-look or Silk mixes. You can also get great effects from Rainbow PLA, which mean you have to have fewer colours in stock. Lastly PLA+ mixtures exist, which are improvements on plain PLA.

\n" }, { "Id": "18584", "CreationDate": "2021-12-17T16:18:43.933", "Body": "

My CR-10 is only a couple of months old. It came factory fitted with auto bed leveling.

\n

Recently, the bed leveling probe has stuck in the withdrawn position whilst it is measuring prior to a print. Whilst stuck in this position the red light flashes. I am able to temporarily resolve the problem by gently pulling the probe down, but then have to start the measuring process all over again.

\n

This problem is occurring more often and if I don't sit through the lengthy measuring process and restart if necessary, the printer will carry on printing in 'mid-air'.

\n", "Title": "Creality CR-10S Pro V2 Auto Bed Leveling probe sticks", "Tags": "|troubleshooting|bltouch|creality-cr-10|", "Answer": "

Having moved the probe up and down gently to ensure it wasn't catching, I restarted the machine and have not been troubled since.

\n" }, { "Id": "18590", "CreationDate": "2021-12-18T20:28:30.537", "Body": "

Recently, my Ender 3D Pro has been unable to print any large models successfully with PLA as the filament starts to expand inside the Teflon tube, causing a clog after about an hour of printing. I am starting to suspect that the problem is heat creep.

\n\n

I have tried many things to patch this problem:

\n\n

If the problem is indeed heat creep, I have plans to control the heat sink temperature with a Peltier and an extra thermometer. Any other ideas are appreciated.

\n", "Title": "Nozzle getting clogged all the time", "Tags": "|troubleshooting|nozzle|", "Answer": "

Since you mentioned "Check if the Teflon tube is touching the nozzle", it sounds like you've disassembled the tube from the couplings and put it back together. This is error-prone and in my experience the main/only likely cause of clogging in Creality hotends. Heat creep is unlikely unless you're operating in a very high ambient temperature or have a failing/failed fan.

\n

The tube can't just be "touching" the nozzle. It needs to be compressed against it. There are various ways to do this, but what usually worked best for me (before I moved to a different setup) was to back the coupler out of the heat sink by at least 1-2 full turns, press the tube all the way in against the nozzle, then tighten the coupler back down to compress the end of the tube against the nozzle mating surface.

\n

The end of the tube also needs to be clean cut, straight, and undamaged. If it's charred, bent, gnarled, whatever, cut it straight with a razor blade while holding it in a jig to make sure the cut is perpendicular. You can find several such jigs (PTFE tube cutting tools) on Thingiverse or make your own or buy one. I like to also take the razor blade and chamfer the outside of the tube end ever so slightly before inserting it. I do this by hand, but I've seen videos of it being done with a jig that looks something like a pencil sharpener, which would probably be the best way.

\n" }, { "Id": "18604", "CreationDate": "2021-12-20T10:40:50.530", "Body": "

the 3D Benchy is everywhere. It is one of the top test prints if you look away from a simple cube.

\n

But what makes the Benchy a good test print at all? It does have almost no critical dimensions that would be measurable to see if the printer is calibrated correctly!

\n", "Title": "Why is a 3D Benchy considered a good test print for FDM printers?", "Tags": "|calibration|knowledgebase|", "Answer": "

Benchy is cute. That goes a long way in attracting attention.

\n

The benchy is one of the earlier STLs that was freely available to download, dating from April 2015, and was released under the Creative Commons Sharealike licence, which clearly states the requirements and limitations/requirements.

\n

Curiously, a printed Benchy can float in still water, provided no print problems crop up. It will also sit nicely on a shelf, with no risk of rolling off and makes a nice ornament.

\n

Links:
https://en.wikipedia.org/wiki/3DBenchy and one of the earliest download sites https://www.thingiverse.com/thing:763622

\n
\n

I've had my printer for a year now, and have never printed a benchy.

\n" }, { "Id": "18611", "CreationDate": "2021-12-21T05:47:16.610", "Body": "

Anyone else seeing this issue? Anyone know a solution?

\n

While editing a part on TinkerCad.com (this part, to be specific), I click the "Send To" button in the top-right --> click Thingiverse --> I see this screen: \"enter

\n

I then click on "Authorize". I'm already signed into Thingiverse, so I click "AGREE & AUTHORIZE APP" --> and I get this error window:

\n

\"enter

\n

It states:

\n
\n

There was a problem authenticating you with Thingiverse

\n

Close this window and return to Tinkercad.

\n

Privacy settings |
\nPrivacy/Cookies

\n
\n

Anyone know what could be the issue? I've tried in Google Chrome, on Firefox, in Chrome with incognito mode, and even with a 2nd pair of separate TinkerCad and Thingiverse accounts! I get the same result each and every time.

\n

Note: I used the Google sign-in for the Thingiverse account. Maybe I need to use the independent account login?

\n
\n

Side note: this question is on-topic:

\n
\n

Websites (could come under Software and/or Tools)

\n

...

\n\n
\n

I just sent in some support request tickets to both sites

\n

Update 22 Dec. 2021:

\n

I posted a help support ticket to both TinkerCad here (choose "Tinkercad Account" from the "Please choose from the options below" dropdown menu) and to Thingiverse here (click link --> choose "Other" from dropdown menu for "What does your support request pertain to?"). I'll add an answer here if I get any useful response or solution from either of them. My support tickets on each site essentially contained the following information:

\n
\n

Subject:

\n

Can't "Send To" Thingiverse from Tinkercad.com

\n

Body:

\n

Please see a full description of my problem here: Can't "Send To" Thingiverse from Tinkercad.com

\n

Essentially, no matter what I do I get into an infinite loop of trying to click the "Send to" button to send a model to Thingiverse, and it repeatedly fails with the error shown in the attachment. Again, see the full description at the link above.

\n

OS:

\n

Linux Ubuntu 20.04

\n

Browser:

\n

Chrome Version 96.0.4664.110 (Official Build) (64-bit)

\n
\n", "Title": "Can't \"Send To\" Thingiverse from Tinkercad.com", "Tags": "|tinkercad|thingiverse|", "Answer": "

Tinkercad replied to my Zendesk ticket I opened (see the question), and as a a result, they seem to have fixed the issue! The "Send To" --> Thingiverse feature seems to be working again!

\n

I sent my request on 20 Dec. 2021:

\n
\n

Subject:

\n

Can't "Send To" Thingiverse from Tinkercad.com

\n

Body:

\n

Please see a full description of my problem here: Can't "Send To" Thingiverse from Tinkercad.com

\n

Essentially, no matter what I do I get into an infinite loop of trying to click the "Send to" button to send a model to Thingiverse, and it repeatedly fails with the error shown in the attachment. Again, see the full description at the link above.

\n

OS:

\n

Linux Ubuntu 20.04

\n

Browser:

\n

Chrome Version 96.0.4664.110 (Official Build) (64-bit)

\n
\n

I got this response on 3 Jan. 2022:

\n
\n

Nicole Smith (Tinkercad)

\n

Jan 3, 2022, 7:16 PST

\n

Hi Gabriel Staples,

\n

Thanks for writing in to let us know about the issue. I'll pass on the information and see what we can figure out.

\n

Regards,

\n

Nicole
\nTinkercad

\n
\n

Then this on 5 Jan. 2022:

\n
\n

Nicole Smith (Tinkercad)

\n

Jan 5, 2022, 15:40 PST

\n

Hi Gabriel Staples,

\n

This should now be working again.

\n

Regards,

\n

Nicole
\nTinkercad

\n
\n

Sure enough; it seems to be fixed!

\n

Next time you have an issue with TinkerCad, submit a help request here (I chose "Tinkercad Account" from the "Please choose from the options below" dropdown menu).

\n" }, { "Id": "18629", "CreationDate": "2021-12-24T17:07:13.920", "Body": "

I have a 3018 Pro CNC and being trying cutting a contour of a simple circular part:

\n

\"Screenshot

\n

G-code:

\n
(TestKnobContour)\n(T1  D=1 CR=0 - ZMIN=-3 - flat end mill)\nG90 G94\nG17\nG21\nG90    \n(2D Contour1)\nZ15\nS5000 M3\nG54\nG0 X10.8 Y0.1\nZ15\nG1 Z5 F10.0\nZ1 F10.0\nZ-2.9\nX10.792 Z-2.938 F10.0\nX10.771 Z-2.971\nX10.738 Z-2.992\nX10.7 Z-3\nX10.6\nX10.562 Y0.092\nX10.529 Y0.071\nX10.508 Y0.038\nX10.5 Y0\nG2 X9.851 Y-3.634 I-10.5 J0\nG1 Z-2.75\nG2 X8.983 Y-5.436 I-9.851 J3.634\nG1 Z-3 F10.0\nG2 X3.301 Y-9.968 I-8.983 J5.436 F10.0\nG1 Z-2.75\nG2 X1.351 Y-10.413 I-3.301 J9.968\nG1 Z-3 F10.0\nG2 X-5.735 Y-8.795 I-1.351 J10.413 F10.0\nG1 Z-2.75\nG2 X-7.299 Y-7.548 I5.735 J8.795\nG1 Z-3 F10.0\nG2 X-10.452 Y-1 I7.299 J7.548 F10.0\nG1 Z-2.75\nG2 X-10.452 Y1 I10.452 J1\nG1 Z-3 F10.0\nG2 X-7.299 Y7.548 I10.452 J-1 F10.0\nG1 Z-2.75\nG2 X-5.735 Y8.795 I7.299 J-7.548\nG1 Z-3 F10.0\nG2 X1.351 Y10.413 I5.735 J-8.795 F10.0\nG1 Z-2.75\nG2 X3.301 Y9.968 I-1.351 J-10.413\nG1 Z-3 F10.0\nG2 X8.983 Y5.436 I-3.301 J-9.968 F10.0\nG1 Z-2.75\nG2 X9.851 Y3.634 I-8.983 J-5.436\nG1 Z-3 F10.0\nG2 X10.5 Y0 I-9.851 J-3.634 F10.0\nG1 X10.508 Y-0.038\nX10.529 Y-0.071\nX10.562 Y-0.092\nX10.6 Y-0.1\nX10.7\nX10.738 Z-2.992\nX10.771 Z-2.971\nX10.792 Z-2.938\nX10.8 Z-2.9\nG0 Z15\nM5\nX0 Y0 Z0\nM30\n
\n

Candle shows that everything is fine for this G-code:

\n

\"Screenshot

\n

However, I am getting weird results (see top right):

\n

\"Photo

\n

What can I do for troubleshooting?

\n", "Title": "Issue milling a circular contour with CNC \u2013 3018 Pro", "Tags": "|g-code|cnc|", "Answer": "

I've ran the code on my own CNC machine. I slightly adapted the code as my machine doesn't understand the movement without the instruction code:

\n
Z1 F10.0\nZ-2.9\nX10.792 Z-2.938 F10.0\nX10.771 Z-2.971\nX10.738 Z-2.992\nX10.7 Z-3\nX10.6\nX10.562 Y0.092\nX10.529 Y0.071\nX10.508 Y0.038\nX10.5 Y0\n
\n

is changed to

\n
G1 Z1 F10.0\nG1 Z-2.9\nG1 X10.792 Z-2.938 F10.0\nG1 X10.771 Z-2.971\nG1 X10.738 Z-2.992\nG1 X10.7 Z-3\nG1 X10.6\nG1 X10.562 Y0.092\nG1 X10.529 Y0.071\nG1 X10.508 Y0.038\nG1 X10.5 Y0\netc...\n
\n

As I used an engraver bit, I made sure the depth was touching the wood (engraving) when running at the lowest depth. The contour it drew was a perfect circle.

\n

\"enter

\n

The code is therefor working as it should (carve a circle, in the photo above, the circle started at the hole in the top left and followed a clockwise path), the result from your milling exercise shows that the final segment of the circle is not giving you a circle segment, instead the milling path is sort of straight. I've seen such paths where the steppers are not powerful enough to mill through the material. As a result they skip steps, and in this case it results in a sort of straight path. You should try running a dry run (in air), or in softer material (this will determine if the code is producing a cirlce in your machine as well), and add more passes to milling the knob (for the final product).

\n" }, { "Id": "18639", "CreationDate": "2021-12-26T06:33:02.953", "Body": "

I recently got a KP3S Kingroon 3D printer and have been trying to set it up.

\n

After a couple of test prints, the Y-axis seems to only move in one direction. At first, I thought it was a motor issue, but when I go into the manual move directions for the Y-axis it seems that both inputs lead to the motor spinning in the same direction.

\n

We have ruled out endstops as a possible issue. I think it might be a hardware issue but lack the skills to confirm the exact issue.

\n
Send:17:40:57.724: @moveRel Y10.00  \nSend:17:40:57.724: N31 G1 Y10.00 F6000  \nSend:17:40:57.728: @updatePrinterState  \nSend:17:41:00.824: @moveRel Y-10.00  \nSend:17:41:00.824: N35 G1 Y0.00 F6000  \nSend:17:41:00.828: @updatePrinterState  \nSend:17:41:07.445: @moveRel Y10.00  \nSend:17:41:07.445: N43 G1 Y10.00 F6000  \nSend:17:41:07.449: @updatePrinterState  \nSend:17:41:09.482: @moveRel Y-10.00  \nSend:17:41:09.482: N46 G1 Y0.00 F6000  \nSend:17:41:09.486: @updatePrinterState   \n
\n

Even though it states that it is increasing and decreasing by 10 it only decreases by 10.

\n

I have updated the firmware to Marlin. I tested switching X and Y inputs and believe the breakdown occurs at the Y input signal.

\n

attached is a picture of the mother board.

\n

\"picture

\n

I am unsure of how to best fix this?

\n", "Title": "Y-axis on KP3S Kingroon only moves in one direction", "Tags": "|y-axis|hardware|linear-motion|", "Answer": "

The answer might just be replace the mother board.

\n

If I understood everything correctly there are 5 things to check to break down the problem:

\n\n

Using Repetier I disproved that the software was broken. By switching the X stepper and Y stepper cables, the motor and connection cable were proven to work. By flashing new firmware it was shown that it was without question that the firmware was the issue.

\n

The backup extractor (E1) is not operational on this board. So it is either the stepper of the mother board.

\n" }, { "Id": "18681", "CreationDate": "2022-01-03T20:44:23.857", "Body": "

I've been trying to do some research on Linear Advance (LA) on Marlin and Pressure Advance (PA) on Klipper, and from my understanding the technology behind each is different, although also similar. If I understand what I have read correctly, then:

\n

LA will alter the extruder acceleration movements so that the extruder can keep relatively the same pressure while printing. The effect of this would be prettier corners and z-seams

\n

PA also seems to alter the extruder flow rate based on the movements. This should also prevent under/over extrusion in the print similar to LA, but where does it differ from LA?

\n

Is my understanding of these correct? I currently have an Ender3v2 with a direct drive setup and am curious if LA or PA would be considered better today (2022)? From what I understand, my 4.2.7 motherboard is not compatible with LA due to its stepper drivers, and I am trying to determine if I should move from Marlin to Klipper and use PA, or whether I should upgrade my motherboard and use LA in Marlin. I may need to upgrade my motherboard anyways as I was also wanting to add some additional temperature sensors and fan controls for a heated enclosure I plan to build down the road.

\n

Any thoughts/advice or reviews and/or current comparisons of the two technologies would be appreciated.

\n", "Title": "What is the difference between Linear Advance and Pressure Advance?", "Tags": "|creality-ender-3|marlin|klipper|pressure-advance|linear-advance|", "Answer": "

Both are implementations of exactly the same concept, even up to the units of the tuning constant being seconds (mm/(mm/s)). The difference is in the implementation details, and particularly how they deal with a mathematically and physically nasty/demanding part of the concept.

\n

In order to compensate for pressure making the filament/filament-path behave like a spring, LA/PA offset the E-axis position by a fraction (K) of the pre-LA/PA E-axis velocity. Mathematically, as an operator acting on the function that's the E-axis position at time t, the basic LA/PA transformation is:

\n

I + K*D

\n

where I is the identity, K is the spring constant, and D is the (time) derivative.

\n

Anyone with a mathematical background in functional analysis or PDEs will recognize this as an unbounded linear operator (at least with respect to most norms, due to the derivative term). Casually speaking, it shifts bad behavior of a particular derivative "down one level". Instantaneous changes of acceleration (starting to accel/decel) become isntantaneous changes of velocity. Instantaneous changes of velocity ("jerk"/junctions) become instantaneous changes of position. (Insert horrified face here.) In short, the result is not physically realizable.

\n

There are two ways to deal with this impossibility:

\n\n

In my opinion, the Marlin behavior is easier to get started with, but hopelessly slow to actually use unless you start cranking up your acceleration and jerk limits. And this ends up being comparable to the manual tuning you have to do with the Klipper PA smooth time window. But in the end, Klipper's approach will always win on print performance, because the smoothing lets you cheat and do moves that wouldn't be possible with Marlin.

\n" }, { "Id": "18683", "CreationDate": "2022-01-04T02:15:19.423", "Body": "

I have a 3DTouch with my Ender 3 3D printer with the Creality 4.2.2 Board. It is running a custom build of Marlin 2.0.9.2. As I have been watching my prints, I noticed that the Z-axis lead screw does not turn at all as the print head traverses the bed.

\n

With the 3DTouch and a mesh of the print bed, the Z-axis lead screw should very slightly turn to move the X-axis gantry up or down to compensate for deviations and irregularities in the print bed as the print head moves around.

\n

I will add that these print head movements travel across a decent portion of the bed. My bed definitely is irregular, and so I would definitely expect the Z-axis screw to adjust the x-axis gantry appropriately.

\n

Here is my start G-code in Cura:

\n
; Ender 3 Custom Start G-code\nM140 S{material_bed_temperature_layer_0} ; Set Heat Bed temperature\nM190 S{material_bed_temperature_layer_0} ; Wait for Heat Bed temperature\nM104 S160; start warming extruder to 160\nG28 ; Home all axes\nG29 ; Auto bed-level (BL-Touch)\nG92 E0 ; Reset Extruder\nM104 S{material_print_temperature_layer_0} ; Set Extruder temperature\nG1 X0.1 Y20 Z0.3 F5000.0 ; Move to start position\nM109 S{material_print_temperature_layer_0} ; Wait for Extruder temperature\n; G1 Z2.0 F3000 ; Move Z Axis up little to prevent scratching of Heat Bed\nG1 X0.1 Y200.0 Z0.3 F1500.0 E15 ; Draw the first line\nG1 X0.4 Y200.0 Z0.3 F5000.0 ; Move to side a little\nG1 X0.4 Y20 Z0.3 F1500.0 E30 ; Draw the second line\nG92 E0 ; Reset Extruder\nG1 Z2.0 F3000 ; Move Z Axis up little to prevent scratching of Heat Bed\n; End of custom start GCode\n
\n

Here are my Configuration.h and Configuration_adv.h files.

\n

I wasn't able to paste my entire configuration file here, so I have provided it in the link above.

\n

What have I done wrong?

\n

To be more concise, here is a list of everything I changed in the default configuration file.

\n
    \n
  1. I ensured #define PDITEMP is not commented so that I can do PID tuning of the nozzle.
  2. \n
  3. Similar to #1, I ensured that #define PIDTEMPBED is not commented so that I can do PID tuning of the bed.
  4. \n
  5. I commented #define Z_MIN_PROBE_USES_Z_MIN_ENDSTOP_PIN since I will be using the 5-pin BLTouch port that is on my 4.2.2. board.
  6. \n
  7. I uncommented #define USE_PROBE_FOR_Z_HOMING since I removed my z-axis endstop and want to use my 3DTouch as the Z endstop.
  8. \n
  9. Uncommented #define BLTOUCH since the 3DTouch is a BLTouch clone.
  10. \n
  11. Changed my x and y offsets in the setting #define NOZZLE_TO_PROBE_OFFSET { -42, -8, 0 }. I left the Z-offset 0, since I will be using the tuning tool to adjust that and observe the squish. For the X and Y, I measured the distance between my probe and the nozzle using a digital caliper.
  12. \n
  13. I adjusted the probe margin from 10 to 15, since I have clips that previously would interfere with the 3DTouch. 15 should give more distance. #define PROBING_MARGIN 15
  14. \n
  15. I enabled and set MULTIPLE_PROBING to 3. I'm paranoid about the current accuracy and am willing to see if that improves anything at the expense of a few additional minutes. #define MULTIPLE_PROBING 3. I think 2 should be fine for general use.
  16. \n
  17. Uncommented #define Z_MIN_PROBE_REPEATABILITY_TEST. I want to test my 3DTouch and uncommenting allows the use of M48 to test it.
  18. \n
  19. Uncommented #define PROBING_FANS_OFF, #define PROBING_ESTEPPERS_OFF, #define PROBING_STEPPERS_OFF, #define DELAY_BEFORE_PROBING 200. The documentation this may improve probing results. I'm all in.
  20. \n
  21. Uncommented #define NO_MOTION_BEFORE_HOMING and #define HOME_AFTER_DEACTIVATE.
  22. \n
  23. Uncommented #define AUTO_BED_LEVELING_BILINEAR
  24. \n
  25. Uncommented #define RESTORE_LEVELING_AFTER_G28. This is to ensure the mesh is applied even after G28, which disables the mesh otherwise.
  26. \n
  27. Ensures that this setting was 10. #define DEFAULT_LEVELING_FADE_HEIGHT 10.0
  28. \n
  29. I set the following: #define GRID_MAX_POINTS_X 7. Ensures a 7x7 mesh grid is created. This could be more or less. 49 points is an improvement over Creality's 9, although a bit much. Worthwhile in my case.
  30. \n
  31. Uncommented #define EXTRAPOLATE_BEYOND_GRID. I was actually wondering if this was causing some of the inconsistent prints near the edge.
  32. \n
  33. Uncommented #define LCD_BED_LEVELING. This is to unlock more options for ABL in the menu.
  34. \n
  35. Uncommented #define LEVEL_BED_CORNERS. This should make moving between corners for manual leveling easier.
  36. \n
  37. Uncommented #define LEVEL_CORNERS_USE_PROBE. This is to achieve exactly what I was doing with G30 in Pronterface. I changed to tolerance with #define LEVEL_CORNERS_PROBE_TOLERANCE 0.03
  38. \n
  39. Uncommented #define Z_SAFE_HOMING, which is important for the BLTouch.
  40. \n
  41. Changed my PLA profile according to what I have determined to be best with #define PREHEAT_1_TEMP_HOTEND 200 and #define PREHEAT_1_TEMP_BED 60
  42. \n
\n", "Title": "Z-axis lead screw not turning to adjust height when using auto leveling?", "Tags": "|creality-ender-3|marlin|bed-leveling|z-axis|3dtouch|", "Answer": "

My firmware actually was working just fine, in terms of probing with G29 before a print and then the Z-axis would adjust during the print.

\n

The way I tested this was by putting an object underneath my probe at only a few of the probing locations at the start of the print G29.

\n

The purpose of this was to simulate very significant deviations in the bed.

\n

Then, when the print was printing its first layer, I would observe the Z-axis and the nozzle, and the printer did definitely make a very obvious motion near the probing points where I put the object underneath the probe.

\n

Looks like when I am printing, it just doesn't seem to make any adjustments at all. I would expect it to make adjustments during the print since my bed is not perfectly level, but I guess it's how things are.

\n

The purpose of this post was to evaluate whether or not my firmware was correctly interfacing with the BLTouch and applying the mesh to my print, and it seems like it is. The question of why it isn't making more noticeable adjustments for my print bed during normal printing is another question.

\n" }, { "Id": "18687", "CreationDate": "2022-01-04T16:50:37.240", "Body": "

I have had my Ender 3 Pro for about 2 years now and it has been working amazingly!

\n

Since November it's struggling very much while printing. I am quite sure the problem lives on the extruder motor.

\n

About 1 of 5 prints come out ok. The main problem is that the first 2-3 first layers are all good. But as the prints develop, under-extrusion problems come. It's not a clog, because I can push the filament and it flows smoothly. It's like the motor cannot push the filament to the feeding line.

\n

Thing's I tried so far:

\n\n

I noticed that the motor is getting pretty hot. I mean, you can't have your hand in there for more than a second. My theory is that this overheating softens the filament and it cannot feed. Is this possible? The aluminum extruder also gets pretty hot. There is no clicking sound or anything weird while printing, the first layers come out perfectly ok.

\n", "Title": "Ender 3 Pro extruder motor failing constantly", "Tags": "|creality-ender-3|extrusion|overextrusion|", "Answer": "

There are a few probable causes I can think of based on your description. That it only starts happening after the printer has been running for a bit makes me think it's a heat issue.

\n

The first problem I'd check out is that extruder motor getting that hot. It should be warm, but not so hot it's uncomfortable to hold. Either the stepper is defective, or you have the current to it set too high. There is a small screw on the motherboard beside each motor driver that allows you to adjust the current to each stepper. Find a guide for your printer to adjust it properly. You'll likely need a multimeter to do it right.

\n

The current being too high on the extruder could also cause the stepper driver (a chip on the mainboard) to overheat. When those overheat they go into thermal shutdown and will stop driving the extruder for a moment until they cool off enough to resume. If your printer has a fan for the mainboard, make sure that is functioning properly as well. Those chips do normally get hot enough they are uncomfortable to touch, so to check if that's the problem you'd need a thermometer and to check what model they are to see what their maximum operating temperature is.

\n

If fixing that doesn't solve it, it could be the heatsink on your hotend is getting hot enough that filament starts melting there, you can get a clog that stops the extruder from pushing filament out properly, but still feels like it can be extruded by hand. Make sure the heatsink on the hotend has sufficient cooling. After a bit of printing try touching the top of the heatsink - if it's uncomfortable to touch, you have a problem there.

\n

-

\n" }, { "Id": "18694", "CreationDate": "2022-01-06T20:14:00.940", "Body": "

I'm using Ultimaker Cura to slice my 3D models, and I often have a problem with object with dimensions over multiple orders of magnitudes.

\n

Basically, if I set an Infill of 20\u00a0%, the infill is calculated for the entire region, regardless of whether for each z-stack, the region is actually a closed one, or multiple separated ones.

\n

As a result, the fragility is increased in those tiny regions that have no filling, because overall, the infill percentage is respected, but out of unfortunate yet inevitable statistics, the parts with no infill ended up colocalizing with the regions that needed infill the most.

\n

\"enter

\n

As you can see in this famous Llamacorn, some closed regions have no infill at all (right side), and the left arrow shows how the infill is calculated based on the entire structure rather than the local one.

\n

Any adaptive option hidden somewhere?

\n

Do I need to tweak the model directly?

\n", "Title": "Adaptive infill density in small region", "Tags": "|ultimaker-cura|infill|print-strength|", "Answer": "

Actually, the right region does have a calculated infill; it just happens to be at the edge of the inner wall.

\n

\"Zoomed

\n

I have drawn a thin black line along the infill lines for this layer and you can see that there is a small infill line.

\n

Cura has a feature called "Support Blocker" that allows you to change the settings for certain areas of your model. You could make those particular areas have an infill of 100\u00a0% if you like.

\n

Chuck Hellebuyck has a video on his YouTube channel that explains how to set this up. It's not that difficult and will take some time to get the area just the way you want it. The information for the "Support Blocker" starts at timestamp 2:01.

\n

\r\n \r\n

\n

Another idea is to try rotating the model on the build plate to "force" an infill line to be generated where you need it. The only problem with this is that you may spend more time trying to get all the areas filled the way you want them.

\n" }, { "Id": "18705", "CreationDate": "2022-01-09T07:49:57.263", "Body": "\n

I saw a couple of tutorials telling one to design whatever needs printing on Fusion 360 and then export it to STL, import on Luban1), and print.

\n

For CNC, I could import a post-processor and the resulting G-code worked fine without having to deal with Luban.

\n

I know Fusion 360 doesn't have the exact machine on the library, but I'm wondering if I could get away with something similar or if there is a post-processor that I could install.

\n
\n

1) Snapmaker Luban is a free, open-source slicing software tailor-made for Snapmaker machines.

\n", "Title": "Fusion 360 + Snapmaker original for printing", "Tags": "|fusion360|snapmaker-original|", "Answer": "

One way to completely avoid Luban is to use OctoPrint, and the OctoPrint plugin for Fusion 360.

\n

Loading the G-code from the USB flash drive can lead to weird behavior, where the Z-axis gets a bit lost. Online printing with luban or OctoPrint worked better so far.

\n

Whatever you do, don't turn off the option in Fusion to start from the home position and set up a reasonable one. Mine went from the homing axis straight to print, including a gash over the plate that was in the way...

\n" }, { "Id": "18711", "CreationDate": "2022-01-10T03:32:05.927", "Body": "

I've been printing a lot of things with 1\u00a0mm offset walls to fit lids.

\n

\"Screenshot

\n

However, in Cura, it's been slicing them but not compensating for the offset creating gaps since the shell thickness (0.8\u00a0mm) is less than the offset. This results in gaps and occasionally allows corners to warp.

\n

\"Screenshot

\n

IMO it should be creating a layer that sits below the inset wall, filling the gap (i.e. making a thicker wall on the last layer before the inset wall). I've looked through the settings in Cura and couldn't find anything to do this.

\n

I could increase the wall thickness but this will result in unnecessary extra printing time/filament.

\n

\"Screenshot

\n", "Title": "Gap between offset walls in print in Cura", "Tags": "|ultimaker-cura|", "Answer": "

Look at the "Skin Removal Width" options in Cura. You might need to unhide them if they're not shown by default. As I understand it, the intent is that the "Skin Expand Distance" feature right next to it is supposed to re-expand the skin areas after shrinking them in a way that results in fewer tiny awkward-shaped regions that are slow to fill; however, as you've found, regions narrower than the wall line width can be lost completely. Setting the "Skin Removal Width" to 0 (and optionally doing the same for "Skin Expand Distance" since it should no longer be needed) will likely fix this.

\n" }, { "Id": "18715", "CreationDate": "2022-01-10T19:59:26.877", "Body": "

When starting the printer, the bed leveling sensor does its little startup thing and then lights up static red. When pressing "Level Bed" or "Auto Home" it homes X and Y and after that sends the Z-axis crashing into the bed.

\n

Is there anything I can change in the firmware to debug or fix this? I have checked the wiring and it's correct.

\n

The BLTouch is not a genuine Creality since at the time it for some reason was not available in my country. The BLTouch is connected like any other BLTouch; 2 wires to the Z endstop connector, and 3 of them to "+5V", "GNF" and "IN". The firmware I got of Marlins GitHub and modified the Ender 3 Pro firmware I got of Marlins configuration GitHub (Marlin BugFix 2.0).

\n

The stuff I modified I put here:

\n

Configuration.h

\n
#define USE_PROBE_FOR_Z_HOMING\n#define BLTOUCH\n#define NOZZLE_TO_PROBE_OFFSET { -42, -5, 0 }\n#define AUTO_BED_LEVELING_BILINEAR\n#define PREHEAT_BEFORE_LEVELING\n#define LEVELING_BED_TEMP 65\n#define GRID_MAX_POINTS_X 5\n#define LCD_BED_LEVELING\n#define Z_SAFE_HOMING\n
\n

Configuration_adv.h

\n
#define BABYSTEP_ZPROBE_OFFSET \n
\n

I compiled this firmware with PlatformIO in VSCode with env: STM32F103RET6_creality.\nThe board I used is version 4.2.2 32-bit.

\n

The video I used can be seen here:\n

\r\n \r\n

\n", "Title": "BLTouch not working on Creality Ender 3 Pro", "Tags": "|creality-ender-3|bltouch|", "Answer": "

I know this is an old post, but I think I figured out the answer.

\n

I had the EXACT SAME PROBLEM TODAY while trying to compile custom Marlin firmware (version 2.1.2.1) for the same exact setup as you have. My BLtouch probe is plugged into the dedicated probe header on the mainboard, and the Z endstop switch is also still plugged in. The bed probe wouldn't trigger while homing or bed leveling, and the nozzle would plow straight down.

\n

The issue is in the firmware. The line #define Z_MIN_PROBE_USES_Z_MIN_ENDSTOP_PIN was not commented out in my Configuration.h. The printer was expecting an interrupt from the Z endstop switch. It wasn't looking for an interrupt from the bed probe, so the probe triggering wouldn't do anything, and it continued to move. Eventually, the nozzle would hit the bed. If you have this issue still, recompile your firmware with this line commented out in your Configuration.h, at around line 1280.

\n

Please let me know if this works for you.

\n" }, { "Id": "18720", "CreationDate": "2022-01-11T14:08:14.693", "Body": "

The Firmware Step-by-step guide remarks to only mess with PID-Tuning settings in the firmware when you know what you are doing. Apparently, defining those settings in the firmware is a somewhat involved process, so this begs the question:

\n

How do you acquire the PID-tuning settings properly and update the boars of a printer so it retains these settings through power cycling?

\n

Let's for simplicity assume there is a Marlin Firmware or a derivative using the same command codes is installed, so we can assume all the commands from the Marlin G-code repository work.

\n", "Title": "How to properly run a PID Tuning and update the firmware?", "Tags": "|marlin|knowledgebase|pid|", "Answer": "

Step 0: Terminal connection

\n

To do any work on the settings of the board, you'll need to run a Terminal - for setting that up, please see the relevant question here.

\n

Step 1: What do we have?

\n

The second step is dissecting the settings that are set already. To do so, send M503 to read the relevant settings from the SRAM of the board and display them in the Terminal of your choice.

\n

Step 2: Try a PID Autotune

\n

To run a PID Autotune, just start by sending this for your first hotend to enforce a 3-cycle PID tuning for the 200\u00a0\u00b0C region:

\n

M303 C3 S200

\n

For your second hotend, you'd send M303 C3 E1 S200 - the E-value is 0-indexed and defaults to 0 for the first hotend, so E1 is the second hotend.

\n

For your heatbed (and PLA) you send M303 C3 E-1 S60 as -1 is the function dependant value for the heated bed.

\n

Step 3: Setting the PID settings

\n

After having obtained the PID settings from running the tuning cycle, we need to send those to the chips. To do so use M301 for a hotend and M304 for the heatbed, using the given values from step 2 as parameters.

\n

Note again, that if you have more than one hotend, you absolutely need to have an E-value if you try to set the second hotend, as the default value is, again, E0 for the first hotend.

\n

Step 4: Saving

\n

To properly save the values into the EEPROM - provided it is enabled - is to just send one command to finalize:

\n\n

If your EEPROM is not enabled, you'd need to take the line you wrote for Step 3 and insert that into your pre-print G-code, thus overwriting the PID settings in the SRAM before each print.

\n" }, { "Id": "18742", "CreationDate": "2022-01-13T09:12:26.117", "Body": "

I need to replace my heatbreak on my Anet A6 since I pushed out the PTFE tube when my filament got clogged...

\n

The current heatbreak has a length of 40\u00a0mm, but, the shop I'm buying from only sells the 30\u00a0mm or 35\u00a0mm (the 40\u00a0mm is out of stock).

\n

Can I just install a 35\u00a0mm heatbreak? Or even a 30\u00a0mm? Or is the 40\u00a0mm crucial in the design of the Anet A6?

\n", "Title": "Using a shorter heatbreak on the Anet A6", "Tags": "|anet-a6|heat-break|", "Answer": "

This hotend design is called a Mk8 type/style hotend, based on the Mk8 Makerbot design. You should get the 40\u00a0mm length heatbreak, this option gives you some extra space, a shorter heatbreak will result in changing the Z-endstop height or raise the bed (if the endstop is fixed).

\n

\"enter

\n

The 30\u00a0mm will for sure be too short and result in a gap in the heat conducting element, the 35\u00a0mm might work. You can source the heatbreaks from typical online market places for about a dollar you get 5 pieces, order these while you try if the 35\u00a0mm version works for you (depending on endstop and bed).

\n" }, { "Id": "18743", "CreationDate": "2022-01-13T14:11:38.770", "Body": "

I had a hard time printing some parts as the brim was printed very well contrary to the first layer, wall, and especially infill. I noticed that filament lines were too thin when printing walls, infill, and the first layer. So the first layer was not sticking to each other.

\n

I suspected that the slicer was the first to investigate, so I printed a model which I already printed before from SD Card and the newly printed part has the exact problems.

\n

Suspecting that filament thread gets cooled very soon so we tried:

\n
    \n
  1. Printing with more Nozzle Temp (225\u00a0\u00b0C).
    \nResult: Walls adhered better but still not strong in addition to weak infill.
  2. \n
  3. Then turned on Fan Automatic Control
    \nResult: more nice walls with still weak infill.
  4. \n
\n

Setup:

\n\n

\"Closeup

\n

\"Small

\n", "Title": "CR-10 Smart with weak infill", "Tags": "|ultimaker-cura|creality-cr-10|infill|", "Answer": "

To solve this problem, I tweaked the following:

\n
    \n
  1. Infill Pattern: Some patterns tend to be more solid than others, going back to Grid instead of Cubic ensuring that there is a solid base for infill to avoid layer shift in infill as Cubic infill is printed in a slanted angle.

    \n
  2. \n
  3. Infill layer height: It seems that the CR-10 Smart is unable to print 0.32\u00a0mm infill layer height.

    \n
  4. \n
  5. Lowering infill speed: going from 75\u00a0mm/sec to 50\u00a0mm/sec as 75\u00a0mm/sec infill speed was too much for the extruder as it was unable to keep up with the speed so you will start to notice under-extrusion on the inside of your part, This under-extrusion will tend to create weak, stringy infill since the nozzle is not able to extrude as much plastic as the software would like.

    \n
  6. \n
\n

Attached below the difference between 0.16\u00a0mm infill layer height on the right, the model is sturdy and strong.\n0.32\u00a0mm infill layer height on the left, the model is weak and stringy.

\n

\"Two

\n

On my try to print a large scale print the same problem occurred, very weak infill, which drove me crazy !

\n

Addressing the real problem "Under-Extrusion" .. testing extruder I noticed that there is a crack on the feeder box

\n

\"Cracked

\n

In my opinion it's a bad decision to go for plastic for the feeder, their design uses the lever to shift the plastic box which has the idler pulley (fixed on it) to relieve the pressure on filament.

\n

\"CR10Smart\n\"inside

\n

The crack on feeder weakened the grip on filament, causing slippage making the filament to be extruded too thin, which made the filament too weak to stick to each other.

\n

Changing extruder to aluminium kit feeder with tight grip on filament ensured that filament is pushed correctly without slipping and also stopped stringing while nozzle is heating.

\n

Print on left after applying the fix.\n\"before&after\"

\n

Hopes this save someone's the trouble.

\n" }, { "Id": "18748", "CreationDate": "2022-01-14T10:16:19.663", "Body": "

I have a weird problem that I can't seems to find an answer for.

\n

I'm using SuperSlicer with my Ender3 + OctroPrint setup.

\n

The prints on the bed in SuperSlicer is perfectly straight

\n

\"enter

\n

However, it seems that when my printer is moving in a straight Y-axis line, there is some kind of linear multiplier on the X-axis because it moves slooowly to the side causing the print to be pretty perfectly rotated on the physical bed as seen in Octoprint and also on the real physical bed (luckily my physical bed is slightly to large so I don't go outside of it).

\n

\"enter

\n

I know there is some kind of setting for this which is used for Z-offset with probe, but I can't seem to find it, and it seems weird that it has been set for the X and Y axis. I don't know if that is what really causing this, because I've never touched those settings on this printer before

\n

Anyone got an idea?

\n", "Title": "Print is rotated perfectly on bed, while straight in the slicer. X-axis moves slowly while printing a straight Y-axis line", "Tags": "|creality-ender-3|marlin|troubleshooting|octoprint|", "Answer": "

The print is not rotated at all on the print bed, the OctoPrint image is exactly the same as the image of SuperSlicer.

\n

What you see is the skirt being printed misleading you into thinking the whole print is rotated! As the rectangles in the corners are aligned in the middle of the rectangles, the skirt is made of 4 skew lines.

\n

If this print prints outside of the bed (in your case the bed is large enough to not cause problems) you are facing the issue that the print bed is incorrectly centered (the hotend to be precise). To have your prints print in exactly the same position as you slice them in the slicer, you could look into How to center my prints on the build platform? (Re-calibrate homing offset), this describes how to determine the offsets from the end stops should be set to get the center of the slicer in the center of your printer.

\n" }, { "Id": "18757", "CreationDate": "2022-01-16T16:50:03.837", "Body": "

I'm running a stock Ender 5 pro with the filament that came with it, and using Creality Slicer 4.8.2, but I'm only able to get reliable bed adhesion if I increase the bed temperature from 50 to 60\u00a0\u00b0C for the bottom layer and decrease the print head speed by about 75\u00a0% from the default profile for the Ender 5.

\n

The machine is absolutely stock, and is fresh out of the box except for bed levelling.

\n

I used the default bed leveling print and that came out well, so I'm reasonably certain that it's not a bed leveling issue. The problem seems to be with models that I've made myself in blender and exported as STL files.

\n

In all cases the raft that was generated by the Creality software has printed out perfectly, but the print has only partially gone down when it came to the model itself. \"enter

\n", "Title": "Why does the first layer only adhere to bed if I increase the temperature by 10 \u00b0C and drop the speed by 75 %", "Tags": "|adhesion|creality-ender-5|", "Answer": "

Your bed is too low - raise it by turning the knobs underneath.

\n

The first layer should not look like strings sitting on the bed as per your photo. Instead it should be a wider strip that looks somewhat like an electronic circuit trace, or like someone has pushed wet paint out of a tube that is being wiped across the surface.

\n

My method is to head the bed with "preheat" in the menu, and let it sit at printing temp for at least 5 minutes. This avoids the heater being at temp but the top of the glass bed being cool.

\n

Then start your job. As the brim or skirt is printed, actively watch it in person and twiddle the height knobs a quarter turn at a time. You want the "end view" or cross sectional view of the printed filament to be like this:

\n
 _____<==>_____\n
\n

and not like this

\n
 ______0______\n
\n

and definitely not like this

\n
       0\n_______________\n
\n

If the head starts scratching the bed, you've gone too far so lower the bed back down again (effectively raising the print head a little)

\n
\n

Here's a print in progress trying to show a better brim. Notice eachgstrand is ovalised and mushed down. That brim will come off in one piece afterward.

\n

\"enter

\n" }, { "Id": "18761", "CreationDate": "2022-01-17T15:44:07.930", "Body": "

Is there a machine (for hobbyists) that will make filament based on the type of plastic I put in. I will sort the plastic before I will put it in the machine.

\n

I have seen the filabot but this uses only plastic from previous prints not plastic types Polyethylene Terephthalate (PET or PETE) or High-Density Polyethylene (HDPE) (these are the #1 or #2 plastic types listed at plasticoceans.org).

\n

To reiterate:

\n\n
\n

I will sort the plastic before I will put it in the machine... so,

\n
sorted waste in ---> sorted filament out\n
\n", "Title": "Plastic bottles to filament", "Tags": "|filament|recycling|plastic|", "Answer": "

There are 2 parameters you need to have good control over when printing any filament:

\n\n

Of these, the melting temperature is directly correlated to the chemical composition of the polymer blend in the filament while diameter control is part of the manufacturing process.

\n

And for 3D printing, we need to take a look at the usability of the material itself. For example, pure PET is not easy to print at all and as used in bottles might be unprintable. PETG (a glycol modified PET) on the other hand is much easier to print - and most filaments sold under PET actually are PETG or PETT.

\n

Troubles of recycling

\n

The melting point of a blend of polymers is often difficult to gauge before doing experiments and in case of recycled material, there are problems with recreating the exact same blend when using small batches unless you use exactly one material as the base for your manufacturing. This brings us to the big problem: errors in the base material. These come in several types:

\n\n

Let's address these piece by piece:

\n

Misidentified base material

\n

Misidentification is when you chuck material into the wrong bin and then process it as if it was the stuff the bin was for. For example, if you'd chuck a chunk of ABS into the PLA bin, your blend will not come out as PLA but as some kind of higher melting composite of the two. The exact details of the result depend on the mixture and how well you mix the processed raw material, but in effect, you just made some kind of PLA+.

\n

This can be overcome by testing and good training as well as knowing the base material well. For example, there is an Austrian company that takes back ski-shoes. Only the hard shells of a particular manufacturer (which used a red ABS) are shredded, pelletized, mixed with some virgin ABS and color for stability and uniformity, then turned into filament, and then printed into flutes.

\n

Another ski-shoe manufacturer takes back their own shoes and recycles the shells back into the current manufacturing, but is silent on what their shells are made from but that they are a long-chain polymer.

\n

When trying to differentiate between PET and PETG, you can not do that unless you do a chemical analysis of every bottle - which leads to a huge problem in reprocessing: PETG melts well before PET and clumps it up, acting as a contaminant (see here for more details)!

\n

Contaminants

\n

Contaminants are a problem that comes with a bad base material. in general, there are two types of contaminants: Chemical and Physical.

\n

Physical contaminants can be avoided by removing them before and after shredding. In the case of Skishoes (that's why I chose that example) is, you'd remove the soft shells and the metal latches, disposing of them in separate ways. Then the plastic shells are roughly sized up, cleaned, and dried before further processing. Most physical contaminants can result in partial clogging during filament production, resulting in an uneven filament. Uneven filament or such containing non-melting particles can result in print failure, for example from being stuck in the extruder or clogging of the nozzle.

\n

Chemical contamination is arguably worse. PET bottles for example: what if the user before used to store chemicals in them that can't be separated from the polymer easily? In the best case, the contaminating raw material is removed, in the worst, it ends up in the stream. This introduced contaminated plastic ends up melting somewhat evenly into a larger portion of the recycled plastic, altering the properties in hard to predict ways. As a countermeasure in industrial PET recycling, the batches are huge and get well mixed before the new plastic product is made. By diluting the chemical contaminants on a vast batch, the effects of the contaminant are vastly reduced and evened out. This is also why even in the case of the recycled ABS-shoes-into-flutes, they mix in some degree of virgin ABS pellets - to buffer against chemical contamination.

\n

Degradation

\n

Not all polymers are suitable for recycling and some of them alter their properties depending on their surroundings. What actually happens depends on the material in question, but let's look at PLA as one example.

\n

While PLA doesn't exactly break down in nature unless put into a high-temperature environment, prolonged UV exposure can bleach out the contained coloration and some blends do become more brittle, others do not encounter this. Angus/Makers Muse had several prints exposed to the harsh Australian sun for up to several years and concluded the worst enemy of PLA over time is the UV light.

\n

A different type of degradation can happen from the environment. The one side of this is cold embrittlement, which means parts become more brittle in cold. This had some experiments done on by Stefan/CNC Kitchen. The other side of this is softening, for example by sitting in a hot car. Usually, this type of degradation is not lasting but could result in embedding contaminants into the mix by embedding them in the plastic, so see there.

\n

Is it a good idea?

\n

Well, from an ecological standpoint, it certainly is a good idea to recycle plastic. But with all the troubles to get any good filament, will it be viable under all viewpoints? You certainly can't sell filament which is of very varying quality unless you make it dirt cheap. Also, all this machinery takes a lot of power and initial investment before you can produce your first spool - which means it might not be economical or profitable.

\n

economical viability

\n

So, let's go back to the main question:

\n
\n

[Is there] a machine that can turn a plastic bottle into usable filament? [...] [Is it] currently on the market?

\n
\n

Yes, you can certainly extrude plastic from bottles into filament shape, and the tools are out there - for a price. However, not all bottles might be useable due to the chemical composition and you will need to make larger batches to reduce chemical contamination.

\n

On an industrial scale, the process consists of several steps: sorting, cleaning, shredding, pelletizing, mixing, extruding, and finally spooling the filament.

\n

Of these, the steps of shredding, pelletizing, and the combo of extruding & spooling need dedicated machinery. Even if hobby projects exist that manage to do this with well-known polymer blends, e.g. recycling 3D prints, such is usually heavy industrial machinery. In hobby-grade machinery, quality control is often problematic, as filament diameter control is the crux, and the price tag to get even filament without readjusting the machine every few minutes is high.

\n

The Shredder might be the cheapest part, only costing several thousand euros professionally and a couple of hundred in hobby grade. A proper pelletizing machine that turns the shreds into pellets for the filament extruder has a price tag of about 10\u00a0000\u00a0\u20ac and I have not yet found a hobbyist kit. A basic inquiry on the absolute minimum investment into a professional filament manufacturing stream without pelletizer came up with about 14\u00a0000 GBP (ca. 16\u00a0800\u00a0\u20ac / 19\u00a0000 USD) plus shipping, while hobbyist kits for only one of the two seem to come up with price tags between 500 and 3000\u00a0\u20ac.

\n

This brings the minimum investment using hobby-grade machinery to roundabout 3000\u00a0\u20ac but without a pelletizer, while an industrial setup comes out starting at about 25\u00a0000 GBP (ca. 30\u00a0000 \u20ac / 34\u00a0000 USD).

\n

Cheaper options?

\n

There are machines out there that turn PET bottles directly into filament by cutting them up directly before entering a filament formation system. This setup is called Pulltrusion, and it turns a plastic strip into an almost-cylindrical, folded-over filament.

\n

While no industrial size machine of this is available, Stefan/CNC Kitchen just released a video investigating the device to manufacture such filament and then tested the print properties of such a filament. Joshua/JRT3D operated the machine in question and manufactured the samples. The base machine is the PetBot engineered by Roman Naskashev, which is commercially available for about 400\u00a0\u20ac assembled plus shipping and import taxes from Russia. Joshua also managed to re-engineer a similar machine using the same method from a 3D printer, so the price point for a self-made machine might be lower.

\n

Each bottle weighs about 20 grams, but neither the mouth nor the bottom can be used, resulting in not 100% useability. The process also means, you can't get any deposit for the bottle back. Assuming an useable portion of about 50%, this would in Germany result in a price of 25 cents per 10 grams, so about 25\u00a0\u20ac per kilogram - which for PET filament would be quite competitive. Some bottles have larger useable portions than others, and others might not require a deposit, making these a very good price, to maybe even free filament.

\n

Do note that the manufacturing path creates a filament that is not solid but contains a void, which is accounted for by increasing the flow multiplier, and it does require a higher temperature than PETG: with settings of 265\u00a0\u00b0C for the nozzle, 80\u00a0\u00b0C for the bed and a 130\u00a0% flow rate, 30\u00a0mm/s extrusion rate, Stefan could use an otherwise PETG profile to gain good results.

\n

However, the higher base temperature requires an All-Metal hotend, which is part of why PET is hard to print with many machines. Other problems are the PET's crystallizing properties, which makes the melting properties at times hard to predict and can induce clogging. Also, Layer adhesion can be problematic.

\n

The biggest problem is the tiny production size of each spool: even if one would manage 15\u00a0grams per bottle in filament, this means that one needs to change the spool 66 to 100 times more often, making larger prints nearly impossible unless one comes up with a good solution for splicing the short pieces.

\n

Final conclusion

\n

While the tools are available, the price tag for a full recycling chain of raw material into filament, either as a hobby or industrially, can be kind of high. This means it might not be economical unless you can manufacture large batches and beat the price point of fresh filament.

\n

However, with small batches and the proper tooling, it might be somewhat viable depending on bottle size and deposit system.

\n" }, { "Id": "18769", "CreationDate": "2022-01-18T20:23:38.553", "Body": "

I am trying to replace a Mk8 heatbreak on my printer. But which orientation do I need to put into the heatblock? The left image or the right image? Note that this specific heatbreak is missing a PTFE tube.

\n

\"enter \"enter

\n

I quickly looked at the other questions on this forum but could not find a clear result with images.

\n", "Title": "Replacing PTFE tube in Mk8 heatbreak", "Tags": "|heat-break|", "Answer": "

The PTFE liner should be in contact with the nozzle, and is normally cut half a millimetre over-size to ensure that it remains in contact.

\n" }, { "Id": "18776", "CreationDate": "2022-01-19T23:59:00.180", "Body": "

Brass has better thermal conductivity compared to steel or hardened steel but it has also far less wearing resistance especially compared to hardened steel that among brass and normal steel has the worse thermal conductivity.

\n

Basically all 3D printer use brass despite is softer and also more expensive metal than steel.

\n

Shouldn't the lower thermal conductivity impact only the time needed to bring it to the established temperature? Is there any noticeable difference in print?

\n", "Title": "Is there any print quality difference switching from brass to steel nozzle?", "Tags": "|extruder|extrusion|nozzle|", "Answer": "

The thermal conductivity will certainly affect the time necessary to bring it to temperature, but will also require adjustment to flow rate with respect to speed of travel. The increased time for heat to travel to the nozzle is reflected in the increased time for heat to be "restored" as the filament transfers it from the nozzle to the bed and to the air.

\n

If you make no other adjustments than waiting a bit longer to heat, you may see print quality changes. You might not, if, for example, the current temperature is a bit higher than needed. Unchanged, the effective temperature change required is compensated by the wider range of temperature allowed by the filament.

\n" }, { "Id": "18778", "CreationDate": "2022-01-20T06:48:30.530", "Body": "

I know that you are not supposed to cure resin prints in the window, but mostly it has been working OK for me. I mostly print minis and props for D&D.

\n

Last week I printed some walls and was a bit impatient to see what they would look like, so I painted them 1 hour after they'd finished. I\u00b4m coming to regret that decision since as of writing they still have that newly printed sticky wetness feel to them. I had calculated with the paint sealing the partially uncured resin in but this doesn't seem to be the case at all.

\n

Does anyone have any ideas that might work? Will they eventually cure enough in the paint layer that the stickiness will go away? Would another layer of paint help? Or leaving them out in the sunlight for a few days? Or do they go into the trash bin? I also considered if some kind of lacquer would help, though I don't want them to be shiny.

\n", "Title": "I painted an uncured UV resin print, now it feels sticky through the paint", "Tags": "|resin|post-processing|", "Answer": "

Scrap the prints

\n

You didn't cure the print, and your paint might interact with the resin in such a way that it might never cure. The paint also will prevent UV rays from accessing the resin.

\n

With this prospect, the only diligent way to go is to treat the item as potentially dangerous and discard it in the proper way.

\n

Layering paints that have not cured fully is also an accident waiting to happen, so better don't.

\n" }, { "Id": "18781", "CreationDate": "2022-01-20T17:18:40.103", "Body": "

Creality Ender 3 Pro \u2013 Z-axis inaccuracy problem.

\n

Before opening this new thread, I did read this question\n(Perhaps there is another question on this site I did not see!)\nBrowsing through comments on that question, I don't really see any clear, verified solution for errors in the Z dimension when printing.

\n

I recently bought a Creality Ender 3 Pro (migrating from a Kingroon KP3, to which I had migrated from a Printrbot Simple Metal).

\n

Today I did a print of a small "testing" object which has a series of rectangular walls (1.5, 2, and 3\u00a0mm) running along both X and Y axis which are exactly 10\u00a0mm tall (in the STL file). I printed this with both the Ender and my older Kingroon. Kingroon had slight inaccuracy with the 2\u00a0mm wall (came out 2.2), but all other dimensions were correct.\nThe Ender had perfect widths for all walls. But the accuracy of the wall height was TERRIBLE- instead of 10\u00a0mm, it was only 9.4!

\n

I should mention two other observations:

\n
    \n
  1. Very often I print with rafts to avoid the "elephant foot" dimension problem. Each time I specify a raft with the Ender, the raft is more or less impossible to snap off!
  2. \n
  3. During the print there are occasional "clicks" at the extruder feed gear (more or less throughout the print process). The only time I encountered that type of symptom previously was with the Kingroon on the very first layer if the height was not quite zeroed correctly and so it was trying to print too close to the bed (The whole leveling/height adjustment on the Kingroon was a constant challenge and needed to be redone each day).
  4. \n
\n

In any case, these two symptoms could also point to improper Z movement during the print job. I should note that to create the G-code file for the Ender I used Cura with the default settings for the Creality Ender 3 Pro machine. Also, during setup I did follow the instructions on a video to "square" the frame.

\n

That earlier thread I mentioned emphasized problems/inaccuracy was in the first few layers.

\n

But in my case the "wall height of 10\u00a0mm" (which came out as 9.4) is not the overall height of the object from the bottom, but rather a measurement from the top of the floor of the object to the top of the walls. So this if well above the first few layers.

\n

I have not yet tried monkeying with the eccentric "tightness" adjusters to the wheels on the two sides (I am a bit cautious about those adjustments because there does not seem to be a way to measure/quantify changes made or even be sure you return to the original state. Hence my use of the term "monkeying").

\n

Also, unlike checking movement on X and Y, I cannot simply raise and lower the Z gantry manually to check for smoothness and freedom of motion).

\n", "Title": "Creality Ender 3 Pro: Problem with Z-axis inaccuracy (squashed layers?)", "Tags": "|creality-ender-3|z-axis|calibration|", "Answer": "

Well, lacking any other suggestions, I took a deep breath and "monkeyed" with the blind eccentric adjusters for the wheeled assemblies for z-movement.
I say this as plural since there is one on the left side (power side) and one on the right side.
As I feared, because you cannot freely slide these, it is really difficult to tell if you are tightening or loosening the "movement".
Based on the potential cause as I identified in the original question,my goal was to slightly loosen them up.
On a positive note, I was able to measure the symptom by using a caliper to check the height of the horizontal gantry above the bed when "homed"(0) then up 10, 20, 30 etc in a non-printing environment. Indeed the height measurements verified that the progression of height did not correspond with the requested/reported movement as shown on the screen.
The variation was much worse on the (slave) right side... Counting on nothing more than dumb luck as I tried adjusting the two sides, I ended up with acceptable performance.
So for now I am done screwing around with it.
If anyone has a great suggestion as to how to better know whether the wheels are getting tighter or looser when making this adjustment, it could help anyone needing to make the adjustment.

\n" }, { "Id": "18792", "CreationDate": "2022-01-22T13:20:06.707", "Body": "

I have been printing a few props for tabletop gaming: a statue, some walls, wine barrels... and I've noticed that resin gets trapped inside. If it stayed firmly inside I wouldn't care too much, but a few times now a model has cracked a tiny amount and liquid resin has started flowing out.

\n

I know the obvious solution is just to make the model solid, but I assumed that since all the slicer tools have an option for hollowing with infill there is a solution to this.

\n

I haven't tried too many things yet, because I am a little anxious to fiddle too much with the printer setting. I went from having 0.5\u00a0mm walls to 0.8\u00a0mm but it still happened. I have tried hex-grid infill and pillars. I also increased the exposure time by a second (from 8 to 9), but this was only to see if the object would be stronger and thus prevent the cracking.

\n

I have considered increasing the Z-lift distance just to have the object longer out of the resin bath to drip off better.

\n", "Title": "Fluid resin gets trapped inside my prints", "Tags": "|3d-models|resin|", "Answer": "

Don't hollow most miniatures below the 40\u00a0mm-scale

\n

Tabletop miniatures are quite small in scale. Often they have very thin details. As a result, hollowing them out is not advisable in the first place and you will have the best results by printing them solidly. Most wargames use something between the 16\u00a0mm to 34\u00a0mm scale, but the problem is still present at the 40\u00a0mm scale. So bite the bullet and print solid for small items - it also gives the model a little weight to stay where they shall be on the table.

\n

Only if you start to print things like small busts or vehicles that have quite some hollow pace, you could conceivably manage to include the needed geometry, as elaborated below, and then hollow out the models accordingly.

\n

Hollow prints properly

\n

In case you do have a hollowed print, you need to include two vents to allow the exchange of air and prevent cupping:

\n

First, you need a vent for air to enter the model. This is best placed at the very top of where the included volume will wit on the printer and needs to be accessible to air once the model is raised out of the vat. For safety two bores to let in the air should be present.

\n

The second vent is somewhat optional unless the print takes really long. If you include it, it needs to be at the very lowest point of the included volume to prevent trapping resin inside and allow it to drain the resin out of the model hanging from the print bed. Should the volume have separate lowest points, you'd need to include multiple drainage vents.

\n

All vents also need to be sizeable enough to allow the viscous resin to drain from them. About 3\u00a0mm\u00b2 (~2\u00a0mm diameter) is the absolute minimum for low viscous resin. High viscous resins require larger holes of about 10\u00a0mm\u00b2 (~3.5\u00a0mm).

\n

Also, no point of the model's interior volume should neck down to below those dimensions or you face the risk of having resin clog those neckings - which is again why you don't hollow small miniatures.

\n

Further listening (especially for larger prints) is here from Angus/Maker Muse, who shows a way to hollow out with a hollow base but skipped the drain vent, and Mark Rhodes, who prefers to use 5\u00a0mm holes where possible.

\n

Avoiding holes?

\n

There are ways to avoid holes altogether, if you can orient the item in such a way that at all times one side of the print is open. Let's take for example a crate or barrel:

\n

We could hollow the item and remove the face we want to be on the table later. This way we shape the item into a cup. We could add air access or resin vent, but we could also just put the missing wall to be facing in X or Y and then angle the item ever so slightly to give the item good resin drainage and totally avoid the need for an extra hole in a surface we want to retain. This is how you'd print a cup without a hole, as I had explained in Why cupping is bad for SLA

\n

Or, you could time your return to the printer to right after the print is finished, you turn the print around so the resin that was trapped inside starts to drain down to the print platform and over that into the vat.

\n" }, { "Id": "18798", "CreationDate": "2022-01-23T19:27:30.450", "Body": "

I'm going to 3D print a series of scaffolds and armatures that I'm going to use as the skeleton\\structure for clay sculptures as an alternative to using twisted wire, because I can print dozens of identical ones off faster than I can twist them out of wire.

\n

They don't need to look pretty, they just need to be able support a little weight.

\n

What is the thickest sensible layer height that I can uses to get them printed off quickly?

\n", "Title": "What is the thickest sensible layer height that an Ender 5 will print at?", "Tags": "|creality-ender-5|layer-height|", "Answer": "

Your maximum layer height is related to the nozzle diameter. Typically, a printer is sold with a 0.4 mm nozzle diameter, unless otherwise specified. One expects to be able to print safely to eighty percent of nozzle diameter, which would be 0.32 mm layer height. Quote from linked Prusa site.

\n
\n

Layer height vs nozzle diameter

\n

Layer height should not exceed 80 % of the nozzle diameter. If you are\nusing the standard 0.4mm nozzle, the maximal layer height is about\n0.32 mm. However, with a 0.6mm nozzle, it\u2019s possible to achieve up to a 0.48 mm layer height.

\n
\n" }, { "Id": "18809", "CreationDate": "2022-01-26T16:10:12.303", "Body": "

I've tried different retraction speeds and distances using calibration models, Z-hop, coasting but nothing prevented stringing. Now I tried printing at 230\u00a0\u00b0C and that seems to do the trick. Even 220\u00a0\u00b0C gives me strings. The filament is quite new.

\n

Has somebody had a similar experience and could you tell me if something else is wrong maybe?

\n", "Title": "Ender 3 print temperature", "Tags": "|creality-ender-3|pla|temperature|stringing|", "Answer": "

It just needed drying, even though it was brand new. Thnx @R.. GitHub STOP HELPING ICE.

\n" }, { "Id": "18816", "CreationDate": "2022-01-28T16:59:56.010", "Body": "

I ran the thickness gave these errors. Now what should I do to fix the errors with Blender or Meshmixer?\nBlender thickness analysis:\n\"enter!

\n

With meahmixer:\nenter image description here

\n", "Title": "Meshmixer or Blender thickness feature", "Tags": "|3d-models|stl|blender|meshmixer|", "Answer": "

The blue error flags are usually trivial. You should be able to click on the blue ones while in Analysis, Inspector and have the model correct those locations. Consider to do one at a time, as you can then Control-Z (undo) to back out if the repair is damaging.

\n

The other flags usually point to a much more severe problem and are likely to remove portions of the mesh. For those, use the select feature to surround the openings, then F to fill. Unfortunately, such a repair may convert what might be a contour into a flat surface, but the options in the Fill feature may provide some relief.

\n

After performing the fill, run the inspector again.

\n

If this is not satisfactory and you are using Windows 10, 3DBuilder will often create repairs that fit the original model quite well.

\n" }, { "Id": "18821", "CreationDate": "2022-01-29T21:46:59.037", "Body": "

I have build a Syringe Pump with a simple stepper motor (NEMA 17), and now I want to actuate it through my 3D printer.

\n

I successfully replaced the main extruder with it and it worked, although I had to still warm up the hot end as Marlin doesn't want to actuate the extruder at cool temperature.

\n

Anyways, I am realizing that my Creality CR10 v3's board has a slot for a second extruder, so I would like to plug the syringe pump there.

\n

I did it but when trying to control it through OctoPrint, I have an error saying that my printer reported a default.

\n

I guess it's not plug-and-play, and it's probably complaining about the absence of second thermistor and so on , right?

\n

What should a complete newbie like me do to be able to control E2 with just a stepper motor connected to it?

\n", "Title": "Plug simple stepper motor NEMA 17 on E2 slot of the main board on Creality CR10", "Tags": "|extruder|creality-cr-10|extruder-driver|paste-extruder|", "Answer": "

To prevent extrusion at cold temperatures, you should enable the feature called "allow cold extrusion" using G-code M302 S0.

\n

Using a spare stepper slot requires building new firmware and uploading it to your board. The most obvious choice is using Marlin firmware, as this is the software that runs on your current board. You need to add an extra extruder in the configuration file.

\n" }, { "Id": "18825", "CreationDate": "2022-01-30T13:38:28.893", "Body": "

How to fill these holes in mandibular jaw in order to 3d print it?

\n

\"enter

\n", "Title": "How to fill big holes that Meshmixer couldn't inspect?", "Tags": "|3d-models|meshmixer|", "Answer": "

I have not used meshmixer, but both blender and meshlab have features to repair non-manifold meshes, including a way to manually fix large holes.

\n

With both of these packages, you can have it select holes, and then either attempt to fill them automatically (which frequently gives terrible results), or manually select a few edges of a specific hole and add a few triangles at a time until it is fully covered (which gives much better results if you choose edges carefully).

\n" }, { "Id": "18843", "CreationDate": "2022-02-02T05:38:25.080", "Body": "

I would need a simple 3D printer for small 3D objects. I have seen that for example ANYCUBIC has very cheap ones. Are they worth it's money?

\n", "Title": "Are those cheap ANYCUBIC 3D printers worth it's money?", "Tags": "|print-quality|", "Answer": "

From what I have heard from a friend using Anycubic printers is that they are good for the money. They claim that the printers are reliable and rarely get any faults in them. Some cheaper printers may not have the most up-to-date spec however if you're going to be doing just basic printing then I'm pretty sure it'll be good enough to get you going. The budget here isn't clear however my current printer for the last 2 years has been the Creality Ender 3 Pro. It is an extremely good printer for the money (around $200) and has everything to get you going as well as being fully upgradeable if you are into that and I personally would argue this is as good as you can get for the money.

\n" }, { "Id": "18846", "CreationDate": "2022-02-02T16:37:04.620", "Body": "

I would like to create something like a treasure box model. I want to print two parts: one is the base and the other one is the lid. However, I want them to snap fit together like LEGO pieces when pressed against each other, creating a perfect seal.

\n

For example, something like this egg where it has two sided, but when pressed against each other they "click-in".

\n

\"enter

\n

Is there a mechanism to attach two printed part?

\n

(Note:I am using Solidworks and a Formlab resin printer)

\n", "Title": "Can you snap fit 3D resin printed parts?", "Tags": "|3d-models|3d-design|resin|", "Answer": "

The mechanisms for "click-in" joining are varied. It's important to note that they are not resin-printer specific, although one should make considerations for the material from which the parts are made.

\n

Some resin printed models will be brittle and would require that the click-in feature be constructed with minimal distortion, while others could be printed with more flexible resin and tolerate much greater distortion, but may also release more easily.

\n

From a Pinshape site:

\n

\"click-joint

\n

Note that in this image, the joint requires a matching spherical portion to accept the male portion. This is presented as a sample, not as a complete answer.

\n

The upper most portion of the male segment is a cylinder of slightly larger size than the main body. The top of the cylinder has a taper which enables the receiving hole to force the sides inward, beyond the matching notch in the receiving hole.

\n

The underside of this larger cylinder can be bluff, making removal difficult, or can be tapered to match, allowing for easier removal.

\n

Consider that the cylinder can be stretched out, flattened in such a way that the cross sections of the male portion and the receiving hole are slots rather than cylinders.

\n

The pages at hubs.com provide greater insight to designing snap-fit joints in 3D printing. The following image represents fairly closely the above dissertation regarding deflection:

\n

\"click

\n

Much more detail regarding design can be found on the linked site.

\n" }, { "Id": "18848", "CreationDate": "2022-02-03T00:40:18.170", "Body": "

In the Ultimaker Cura print settings menu, there is a dropdown menu labeled "profiles", with Super Quality, Dynamic Quality, Standard Quality, and Low Quality. What do these settings do exactly? If I set it to Super Quality for example, it changes things like the layer height settings, but I am able to then change those settings back to whatever I want and profile remains on Super Quality.

\n

What are these quality settings? I know this is probably an obvious question but I am basically a beginner and for some reason can find little info on google.

\n", "Title": "What is the quality setting in Ultimaker Cura do? (Super, Dynamic, Standard, Low)", "Tags": "|ultimaker-cura|", "Answer": "

According to the Ultimaker Support website, these are called "Quality settings". You could think of them as "resolution" settings.

\n

There are 3 different adjustments made when changing these settings:

\n\n

A smaller layer height will produce a smoother surface. Initial layer height can create a stronger adhesion to the build plate. Line width will determine how much plastic material will be extruded.

\n" }, { "Id": "18857", "CreationDate": "2022-02-04T03:58:26.573", "Body": "

I'm new to 3D printing, but my printer supports Linear Advance. I heard that it offers improvements in print quality. I used Marlin Linear Advance Pattern Generator to generate a print with horizontal lines at a variety of k-values.

\n

Which K-Value would be best from my below image?

\n

\"enter

\n", "Title": "Which K-Value would you choose here for linear advance?", "Tags": "|creality-ender-3|marlin|calibration|", "Answer": "

Go with the sort of tower 0scar suggested and look for corner bulge/rounding. It's a lot more useful than the single-layer test pattern from the Marlin site, which I've found can be misleading.

\n

With that said, just looking at your test print, I would go with the lowest K factor that gives acceptable-looking results, so around 0.5 or so. Overshooting is more likely to harm your print quality than undershooting, and will limit your print speed too. When I used the original bowden on my Ender 3, my calibrated K factor for PLA was 0.6, so I think this is in the range of what's expected.

\n" }, { "Id": "18867", "CreationDate": "2022-02-04T21:14:47.957", "Body": "

In Cura slicer, it is possible to display\\visualize where in the print process retractions occur for any given model?

\n

For example, to have them highlighted in the preview window.

\n

I'm making my own models\\sculpts and want to optimise them to reduce the amount of time that retraction occurs.

\n", "Title": "In Cura, is it possible to display where retraction occur on any given model?", "Tags": "|slicing|retraction|", "Answer": "

Yes, it ss possible to visualize the retractions in the preview of the sliced object in Cura.

\n

\"enter

\n

In the "Preview" window mode with "Layer view" option enabled (after you sliced the object), you are able to select the "Line Type" of the "Color scheme" and enable travel lines. Dark blue lines are unretracted and light blue is retracted.

\n

\"enter

\n" }, { "Id": "18876", "CreationDate": "2022-02-07T03:16:59.410", "Body": "

I was getting some weird under extrusion and noticed that the gear on the extruder was a bit worn.

\n

I decided to replace it with a dual-drive extruder, but that makes the E-step way off. I tried to run 100\u00a0mm through without any PTFE tube and only got 67\u00a0mm.

\n

I am trying to figure out where I need to change the number.

\n

I have the 4.2.7 motherboard, and I also put on the Creality touch screen.

\n

It doesn't seem to have any place to enter those settings that I can find.

\n", "Title": "Adjust E-step on Ender 3", "Tags": "|creality-ender-3|extruder|stepper|", "Answer": "

If there is no interface to set the value through the display, you can always set the E-steps per G-code.

\n
\n

M92 Set axis_steps_per_unit

\n
\n

There are at least 2 options, the first is to connect a USB cable and connect to a console (What is a printer console/terminal?) or second "print" the applicable G-codes by creating a text file (with a .gcode/.g file extension) and place the M92 E139 on the first line and M500 on the second line. The latter option requires you to print the file once as it stores the new E-steps value.

\n
\n

E.g. the current value for the Ender 3 is 93 which gives you 67\u00a0mm, to get to 100\u00a0mm you would need: $\\displaystyle \\frac{93\\times100}{67} \\approx 139$ steps, so M92 E139 would set a new value. Store the value with M500. Note that this board stores the settings on the SD card as it has no EEPROM, so have an SD card present in the slot of the board! If you're unable to store the value you can also put it in the start G-code of the slicer, but that is not a preferred method.

\n" }, { "Id": "18888", "CreationDate": "2022-02-08T12:08:53.350", "Body": "

I was wondering what the exact difference between the switches Q and D for the G29 command is? They seem to do the same:

\n
[D<bool>]   \nDry-Run mode. Just probe the grid but don\u2019t update the bed leveling data\n\n[Q<bool>]   \nQuery the current leveling state\n
\n", "Title": "G29 difference between Q and D", "Tags": "|marlin|", "Answer": "

TL;DR
\nYes they are different, but whether Q is active depends on the a configuration setting, else it gets ignored and a regular G29 will be executed.

\n
\n

Option 'D'

\n

With option D the G29 code is being run in dry-run mode:

\n
 *  D  Dry-Run mode. Just evaluate the bed Topology - Don't apply\n *     or alter the bed level data. Useful to check the topology\n *     after a first run of G29.\n
\n

If you look into the source code of G29 for ABL the D option will set the boolean abl.dryrun:

\n
abl.dryrun = parser.boolval('D') || TERN0(PROBE_MANUALLY, no_action);\n
\n

This boolean will cause many parts of the code to be excluded, e.g.:

\n
// Unapply the offset because it is going to be immediately applied\n// and cause compensation movement in Z\n...\n// Auto Bed Leveling is complete! Enable if possible. \n...\nprobe.preheat_for_probing(LEVELING_NOZZLE_TEMP, LEVELING_BED_TEMP);\n...\n// vector_3 planeNormal related code\n...\nextrapolate_unprobed_bed_level();\n...\n// Create the matrix but don't correct the position yet\n...\n// Correct the current XYZ position based on the tilted plane.\n...\n// Unapply the offset because it is going to be immediately applied\n// and cause compensation movement in Z\n...\n// Auto Bed Leveling is complete! Enable if possible.\n
\n

Option 'Q'

\n

Option Q in command G29 code is available when manually probing is active in the configuration.h file:

\n
/**\n * The "Manual Probe" provides a means to do "Auto" Bed Leveling without a probe.\n * Use G29 repeatedly, adjusting the Z height at each point with movement commands\n * or (with LCD_BED_LEVELING) the LCD controller.\n */\n//#define PROBE_MANUALLY\n
\n

From the sources:

\n
 *\n *  To do manual probing simply repeat G29 until the procedure is complete.\n *  The first G29 accepts parameters. 'G29 Q' for status, 'G29 A' to abort.\n *\n *  Q  Query leveling and G29 state\n
\n

Using the Q in the command will set the boolean variable seenQ which enables extra output as if in debugging mode.

\n
  // G29 Q is also available if debugging\n  #if ENABLED(DEBUG_LEVELING_FEATURE)\n    if (seenQ || DEBUGGING(LEVELING)) log_machine_info();\n    if (DISABLED(PROBE_MANUALLY) && seenQ) G29_RETURN(false);\n  #endif\n
\n

When going through the code, you stumble again on:

\n
abl.dryrun = parser.boolval('D') || TERN0(PROBE_MANUALLY, no_action);\n
\n

In the first option ('D') the first part of the statement already was true, so the latter part was of no interest as this statement is an OR statement. Now (for the 'Q' option) the latter part is of interest, the first part rendered false. The latter part is a Marlin ternary macro:

\n
TERN0(OPTION, T)    If OPTION is enabled emit T otherwise emit 0 (false).\n
\n

So, if PROBE_MANUALLY was enabled/defined in the configuration.h file, the ternary macro returns the no_action (a "one"/true, as this a boolean: no_action = seenA || seenQ), otherwise a "zero"/false. And if so, the abl.dryrun is true and all code as listed in the 'D' option will be omitted as well. But, additional exclusion apply for the no_action; this will cause the function to be exited early (line 478):

\n
if (no_action) G29_RETURN(false);\n
\n
\n

So far a generic answer, the following is specific for the Ender 5 Plus.

\n

In looking into another question from you it appears that the printer used is the Ender 5 Plus. If you are using stock firmware, you can peek into the sources as they are being shared (through their Google Drive) by Creality for the Ender 5 Plus.

\n

Digging into the sources, the firmware version is from the 1.1.x branch, they reference to 2016/2017 Marlin development. If you look into the sources, the configuration file doesn't define the needed PROBE_MANUALLY for the Q to become active in a G29 Q sent to the printer, in such a case the Q is ignored and a regular G29 will be executed. Do note that the implementation of the G29 code is able to handle the Q option, but because it is not enabled in this configuration it will not work.

\n" }, { "Id": "18889", "CreationDate": "2022-02-08T17:22:36.207", "Body": "

I would like to create a water chamber (Box) that maintains moisture. My main idea right to use a snap-fit model (Two faces). However, I feel that the areas where the two face snap-fit will allow air to go through and evaporate the water. In simple terms, I would like to create a snap-fit box that can maintain water inside like a water bottle. (Moisture cage)

\n

Note:

\n\n

I there something I can use to perfectly seal the moisture inside? or coat my box from the inside to prevent air exchange?

\n

\"enter

\n", "Title": "Can you have a perfect seal to maintain moisture?", "Tags": "|3d-models|3d-design|resin|water-resistance|", "Answer": "

I don't understand how you can keep moisture, since the model you show is full of holes.

\n

However, assuming you are closing them or something else, then you can use "rotational moulding": put some liquid sealant inside the box, close the box, then start rotating it in every direction so that the sealant coats all the internal surfaces.

\n

\r\n \r\n

\n" }, { "Id": "18891", "CreationDate": "2022-02-09T06:57:57.597", "Body": "

I'm using my printer for some baby toys. My last print was one of those pillars for stacking rings, base 5. The basic format includes a base and a pillar coming up perpendicularly in the middle. A box with a cylinder sticking out of it.

\n

Settings:

\n\n

Three hours of printing later, I took it out of the printer, it feels really solid, all the surfaces are rigid, no compression or anything.

\n

I hand it to my kid, she drops it and the pillar just detaches along the Z layer seam at the base.

\n

Is there anything I can do, either model or printing-wise to reinforce it, aside from printing it horizontally with a bunch of supports?

\n

\"The

\n", "Title": "Reinforcing inter-layer seams", "Tags": "|pla|troubleshooting|adhesion|snapmaker-original|", "Answer": "

The other answers are good, but a big part of your problem is also the gyroid infill, which has very poor layer adhesion strength due to being a 3D infill pattern where there's very low surface area contact between consecutive layers of infill. This isue is amplified by the thick layer height, which makes the gyroid paths differ from layer to layer more than they would with thin layers. If you switch to a fully 2D pattern like triangles where the overlap between consecutive layers is 100%, it should be stronger.

\n

If your extruder is strong and you're not pushing the limits of your printer's extrusion rate, this doesn't matter so much. But I found it was a big issue on the original ungeared extruder of my Ender 3 (before replacing that) when trying to print at anything above 30 mm/s or so. For example, printed M8 bolts came out looking fine, but immediatey snapped at the head when tightened - a failure mode very similar to what you're seeing. Switching to a better infill pattern mitigated it.

\n" }, { "Id": "18898", "CreationDate": "2022-02-09T15:44:02.963", "Body": "

I am using a Formlabs 3 resin printer and I am would like to create a bottle design with a cap. my objective is for the cap to completely prevent any liquid from going in or out of the bottle (even it was shaken) and also prevent dry air from seeping in and effecting the moisture inside.

\n

(Note: I am not planning to cure my print with UV light as I feel it ruins the smoothness and flexibility of the model. tell me if I am wrong)

\n

Is that possible by just printing? or do I need some coatings or extra steps?

\n", "Title": "Can 3D printed bottle caps make a perfect seal?", "Tags": "|3d-models|3d-design|resin|resolution|water-resistance|", "Answer": "

Yes and no

\n

If you can print very tight tolerances and the bottle itself does deform a tad to press into the cap, then yes, you will get a perfect interference fit.

\n

If you want to go absolutely sure, a little seal of rubber into which the mouth of the bottle presses can add the last bit needed. alternatively, a viscous grease on the threads can act as a sealant.

\n

In case you want to pretty much seal the bottle forever and not remove the cap, you might just as well add a few droplets of liquid resin or glue to the threads of the cap before screwing it on, then make sure that it sets and seals. Loctite is one such sealing glue often used.

\n

CURE IT

\n

However you are wrong not to cure your model: unless cured, the model is not safe to the touch without gloves as the bonds are not set and liquid resins are dangerous skin irritants. If you want to use the flexibility of not fully cured resin to seal your containers, you might want to seal the bottle with the raw caps and then cure the combined item right after.

\n" }, { "Id": "18901", "CreationDate": "2022-02-09T18:13:11.570", "Body": "

I am using a FORM LABS 3 printer with clear resin. After printing the model, I wash it with Isopropenyl and dry it. Then I cure it using Formlabs Form Cure for 5 minutes under 60 C\u00b0.\nAfter curing the model, the clear print loses some of its transparency.

\n

Is this normal? can it be avoided?

\n", "Title": "Is it normal for clear resin to lose transparency after curing?", "Tags": "|troubleshooting|resin|color|uv-printer|", "Answer": "

Clouding is a known issue with colored transparent resins, as is yellowing with clear resin.

\n

Uncle Jessy did quite a good video explaining the issue and how to best avoid it.

\n

The conclusion was that you should wash and dry them with as little UV exposure as possible (Drying them inside a box in a warm room rather than in direct sunlight), then coating them with Clear Coat lacquer or a similar product, then curing them.

\n

enter link description here

\n" }, { "Id": "18913", "CreationDate": "2022-02-13T00:03:15.497", "Body": "

So I have little to no idea regarding 3D printing and was building web-based software for a client, I have 3 values in my access,

\n\n

Given this information, what would be the formula to calculate the rough estimation of the time required to 3D print this? I know that it's different for all the 3D printers but still, I would like to have a way to calculate a rough estimation of the 3D printing time, that possibly but not necessarily, touches the reality of most 3D printers.

\n", "Title": "How to calculate a rough 3D printing time?", "Tags": "|cost|estimation|", "Answer": "

You'd certainly need to know more information than that provided. 3D printers have speeds ranging from 20 mm per second to as high as ten times that (extreme 3D printer sports!) and the speed during printing varies with the geometry of the object being printed.

\n

More considerations involve printer nozzle size, object layer thickness (if uniform throughout), printer model, slicer software used, slicer software profile used.

\n

For a rough estimate with the information, you could expect a variance fifty percent or more, if someone was willing to take the risk of making an estimation.

\n" }, { "Id": "18921", "CreationDate": "2022-02-13T17:58:47.500", "Body": "

So I have the following pieces of information based on which I want to calculate the estimated time It would take to 3D print something.

\n\n

Given this info, how to calculate the 3D printing time?

\n", "Title": "Formula of calculating 3D printing time", "Tags": "|estimation|", "Answer": "

In general, there is no easy way to compute 3D printing time from those quantities. Unless you have a printer with extreme kinematic capabilities (and to some extent even then), print speed is highly limited by acceleration and deceleration for toolpaths that are deeply a function of the specific geometry of the model to be printed. This makes it really hard to get a good estimate for time without actually slicing the model and simulating the printer's execution of the resulting gcode.

\n

If the object is sufficiently simple, or if your print speed is so slow as to make acceleration to the max speed nearly instantaneous (but travel speed isn't so slow as to be a dominating factor), you may be able to use any one of the quantities you know to estimate print time half-decently.

\n\n

If the conditions under which you might be able to get a reasonable estimate don't apply, your best bet is to actually slice the model and see what the slicer software estimates, or feed its output to a gcode analyzer/simulation software.

\n" }, { "Id": "18927", "CreationDate": "2022-02-14T20:36:58.183", "Body": "

I recently purchased a Creality Ender 3 Max as my first 3D printer and overall I am quite impressed with the print quality after only leveling the bed.

\n

However, every print I have done so far has had bumps on the outer walls, limited to a section of the lower layers, usually around 6 layers in height.

\n

\"White

\n

Standard print settings:

\n\n

I have tried:

\n\n

\"Black

\n

My next experiment will be to combine a higher extrusion temperature with reduced print speeds however I suspect that this is just making the print more forgiving and masking a different cause.

\n

Does anybody know of a way I can eliminate these artifacts?
\nFurther experiments I can run to debug this or even the correct terminology or search terms for this (I'm having difficulty finding anything that looks like this problem online) would also be greatly appreciated.

\n

Updates:

\n\n", "Title": "Bumps on outer walls consistantly affecting lower layers", "Tags": "|creality-ender-3|print-quality|troubleshooting|", "Answer": "

I think I found an answer.

\n

When the gantry is low (Z less than 100 mm high) the Xmax (right hand) side of the gantry has some play, if I move the gantry higher the rollers tighten up.

\n

I went through Luke Hatfield's X Gantry Rework (PDF section 4.1.2, video tutorial), making deviations for my own machine (the vertical extrusions are mounted to the side of the base on the Ender 3 Max rather than the top).
\nTightening the rollers using the eccentric bolts and trying to get the gantry as level as possible seems to have improved the issue immensely.

\n

\"XYZ\nThe Z measurement for the 20 mm cube benchmark is now 19.7 mm (0.5 mm improvement)

\n

\"3D\nAnd the bumps on the boats are a lot less noticeable.

\n

While reassembling I had some difficulty lining up the screws on the top horizontal extrusion, which would suggest that the vertical extrusions are not aligned correctly as suggested in item 1 of this post discussing problems found on a stock Ender 3 Max:

\n
\n

The vertical extrusions are out, there appears to be a fault in the design and the extrusions are about 2.2mm closer together at the bottom than at the top.. what this means is if you move the carriage to half way up and adjust the wheels perfectly, when you move it to the top the outside wheels will be too tight, and when you move it to the bottom the inside wheels will be too tight!

\n
\n

https://www.reddit.com/r/Ender3Max/comments/qa93m6/problems_i_have_found_with_stock_ender_3_max_and/\n
The suggestion in that post is to install a printed 2.2 mm shim between the vertical extrusion and the base. I'll give the shim fix a try at a later time, for now the quality of the prints is good enough.

\n" }, { "Id": "18932", "CreationDate": "2022-02-15T02:54:12.907", "Body": "

I have just bought a Creality CR-10 Smart 3D printer, and - as part of the rite of passage of newbies - I am struggling with getting a usable initial layer. Whilst trying to make sense of what I was seeing in the net and correlating it with what I was experiencing with my printer, I thought to check the printer's credentials (I had assumed it was a recent model). However, looking at the information on the LED screen, I see:

\n
\n\n\n\n\n\n\n\n\n\n\n\n\n\n\n\n\n\n\n\n\n\n\n\n\n\n
Machine typeCR-10 Smart
Firmware versionHW 1.0.6
Screen versionT18
Hardware versionDWIN 4.3
\n
\n

Can anyone comment on whether this is recent or ancient? Is it a V1 or a V2?

\n

As a software engineer of several decades standing, but a complete newbie to 3D printing, I'm suffering from culture shock, and I'm struggling to understand why my printer doesn't just work "out of the box" like my line printers do.

\n

I find it particularly difficult sorting out the information about Creality's various CR-10* models, partly because of the naming conventions, but chiefly because my CR-10 Smart doesn't look like any that I have seen referred to anywhere in the net. I was told that the CR-10 Smart had auto bed levelling (ABL) built in, but when I search for CR-10S ABL I only find stuff about add-ons like BLTouch, and nothing about the CR-10 Smart with built-in ABL.

\n

Add into the mix the now traditional Internet problem that the date of web pages and whether they are still relevant is often unclear.

\n

Then I found a Youtube video by WillCaddy, which suggested that CR-10S printers with Creality firmware do not have ABL enabled, and that to get functioning ABL you had to install TH3D.

\n

Is this true?

\n

If anyone can help me with any of this, I would be very grateful. I am working my way slowly through Michael Laws' excellent "Teaching Tech" channel, but I'm impatient...

\n", "Title": "How do you make a new CR-10 Smart do auto bed-levelling?", "Tags": "|bed-leveling|creality-cr-10|", "Answer": "

@0scar asked me to post this answer and accept it (see the comment above). I'll try to describe everything I did, most of which helped a little bit but didn't really solve the problem. Bear in mind that I'm a complete newbie to 3D printing.

\n
    \n
  1. Although I knew my printer was a Creality, I didn't know that a CR-10 Smart is not the same thing as a CR-10S. Mine is a CR-10 Smart.
  2. \n
  3. I upgraded the firmware from 1.0.7 to 1.0.10. I managed to brick the computer entirely doing this, but I managed to fix it in the end. I found this post particularly helpful whilst doing this.
  4. \n
  5. I used OctoPrint's Bed Visualizer to update the mesh for my printer's ABL. The mesh is stored in EEPROM, so this doesn't have to be done often.
  6. \n
  7. I didn't understand how OctoPrint's Bed Visualizer managed to produce the shape for the print plate that it did, so I checked the bolts under the glass plate, discovered they were loose, tightened them up, re-ran the mesh generation and got a considerably more plausible plate geometry.
  8. \n
  9. I discovered that Creality Slicer (a variant of Cura) does not include G29 or M420 S1 in the G-code that it generates by default for the CR-10 Smart, so I added M420 S1 to the printer's machine settings which tells it to use the mesh stored by Bed Visualizer.
  10. \n
  11. I have been following Teaching Tech's 3D Printer Calibration and, after upgrading the firmware, I started setting up my first layer generation and got a reasonable result. At this point I thought the problem was solved.
  12. \n
  13. I celebrated by printing a 3D Benchy, but that was a sobering experience and I returned to Teaching Tech.
  14. \n
  15. The next step in Teaching Tech's 3D Printer Calibration that I tried was temperature tuning. That was a complete disaster, so I moved on to retraction tuning, since the Benchy had lots of stringing. Retraction tuning was also disastrous, so I decided to stand back and think a bit.
  16. \n
  17. For some reason I looked more closely at my printer's nozzle. For some reason, I assumed 0.4\u00a0mm was a kind of standard for a hot end nozzle, so I was using that in my settings, also because I thought my printer had a 0.4\u00a0mm nozzle. However, closer examination revealed that the nozzle is at least 1.0\u00a0mm, if not 1.5\u00a0mm. Not surprisingly, once I told the slicer the correct nozzle size, printing improved dramatically.
  18. \n
\n

In the meantime, I believe that my basic problem was always the nozzle size, though \u2013 as you can see above \u2013 there were a number of other things that I corrected on the way.

\n

So now @0scar and I believe that the problem addressed in the title has been solved, and \u2013 in accordance with @0scar's wishes \u2013 I will accept my answer, though his input was instrumental in finding the solution.

\n

I shall now devote myself to the rest of Teaching Tech's 3D Printer Calibration.

\n" }, { "Id": "18940", "CreationDate": "2022-02-16T18:23:28.403", "Body": "

I'm considering buying my first 3D printer. The only place in my house where I have enough space is the garage, and I fear a PLA printer will suffer from the possible temperature differences I may have there.

\n

For this reason (And also the fact that it seems smoother) i'm considering buying a resin printer (like the Elegoo Mars), but I wanted to know if the fact it will be in a garage could be a bad idea? I imagine resin printing is far less sensible to temperature variations, but I may be wrong.

\n

I will only print as a hobby, probably some figurines for me and my kids to paint, or small hardware pieces / boxes for electronic components.

\n", "Title": "Is resin printing correct for my needs & limitations?", "Tags": "|resin|", "Answer": "

Resin likes it warm and dark

\n

Resins increase in viscosity at low temperatures, so having low temperatures may have bad effects on the prints. However, Resin also likes it dark, so if your garage has a temperature above about 10\u00a0\u00b0C and is dry enough to prevent condensation, then your garage would be a good place.

\n

Figurines and high detail models are a common use case for resin

\n

It is near impossible to print miniatures for wargaming and similar in FDM, but resin printers are a very common type for this use case. Similarly, Cosplay props with a very high detail grade can benefit from being printed in resin, either directly or as an intermediate step to create a mold.

\n

With the proper equipment, one could make flexible molds to cast wax positives which then can be used with the lost-wax-technique to create many high detailed metal copies of the printed first positive.

\n

Containers are a possible use case for resin

\n

Resin prints are more brittle than FDM prints. While limited in size, such containers, just as much as casings for electronics, are a possible usecase. In fact, replacement casings for retro electronics (think SNES controllers or N64 Cartridges during restoration projects) are somewhat common, even though they have not the same sturdiness as the original ABS casings. Here, the high detail grade makes the cut.

\n

Tooling is a poor use case

\n

Resin prints are very brittle and have little to no ductile movement. While I have used FDM-printed parts from PLA in a few projects to make adapters between machines and my shop vacuum as well as to create an aluminium bending tool, resin printed parts with their near to no flexibility are generally a poor use case to create hardware or tooling from.

\n" }, { "Id": "18948", "CreationDate": "2022-02-18T03:52:43.093", "Body": "

I am running into what honestly seems like a bug in OpenSCAD, but I can't believe that something so simple could reveal a bug. I am trying to render the following code:

\n
module tri_prism(base1, base2) {\n    polyhedron([\n        base1[0], base1[1], base1[2],\n        base2[0], base2[1], base2[2]],\n        [[0,1,2],[3,4,5],\n        [0,1,4,3],[1,2,5,4],[2,0,3,5]],\n        convexity=5);\n}\n\nunion() {\n    tri_prism(\n        [[0,0,0],[0,1,0],[1,0,0]],\n        [[0,0,1],[0,1,1],[1,0,1]]);\n    tri_prism(\n        [[2,1,0],[2,0,0],[1,1,0]],\n        [[2,1,1],[2,0,1],[1,1,1]]);\n}\n
\n

This should render two (non-overlapping) triangular prisms, like so:

\n

\"Two

\n

However, this image is only the preview. When I hit F6, it renders this, with only the triangular bases being rendered:

\n

\"Only

\n

And the console shows this warning, which I don't know the meaning of because I am a beginner:

\n
UI-WARNING: Object may not be a valid 2-manifold and may need repair!\n
\n

Each of the prisms individually renders just fine, but the two of them together don't. Also, it is not merely a problem of appearance since exporting it as STL and opening it in Cura, I see the same useless nonsense.

\n

Is this a bug or am I doing something wrong? And if it is a bug, how do I work around it?

\n", "Title": "OpenSCAD can't render union of two polyhedrons, but can preview just fine", "Tags": "|openscad|", "Answer": "

This is a case of "Mis-ordered faces".

\n

That means the order in which the vertices making up one or more faces aren't in the correct order.\nThe vertices of a face should be listed in clockwise order from the perspective of a camera looking at the face form the exterior of the polyhedron.

\n

In this case, the problem can be solved by reordering the [0,1,2] face of the tri_prism polyhedron as, for example, [0,2,1].

\n

The misordered face(s) can be found by enabling the Thrown Together option from the View menu and Previewing. The faces in question will be shown in magenta.

\n

Why, then, if the faces are mis-ordered, did the prisms render just fine individually? My guess is that faces have to be properly ordered to be sure to get a correct Render. The behavior of Render is simply indeterminate for polyhedra with misordered faces. It can work in some situations, but it's not guaranteed.

\n

Edit: I forgot to add that, in my opinion, this shouldn't be an issue, and that if it's such a big problem for the Rendering subroutine, it should intelligently reorder the mis-ordered faces in a prelude. It would avoid beginners such needless troubles.

\n" }, { "Id": "18954", "CreationDate": "2022-02-18T18:59:54.770", "Body": "

I have a Creality cr-10 printer. Everything is stock except for the direct drive extrusion system (I bought a conversion kit from Micro Swiss).

\n

I have been working to dial in my settings, and I think they are almost there, with a few exceptions. There seems to be roughness/blobs at the layer transitions on my prints. This was present before my conversion to the direct drive system as well. I calibrated the steps/mm after the change as well. What settings should I focus on to try to get rid of this? Or is it some sort of mechanical issue? I've included some example pictures:

\n

\"enter\"enter\"enter

\n", "Title": "Issues with layer changes/z-seam", "Tags": "|print-quality|creality-cr-10|direct-drive|", "Answer": "

From your pictures, the blobs are (at least mostly) not at layer changes but at speed changes where the toolhead has to drastically slow down and speed back up to go around a sharp corner (including as a special case a corner in the Z direction, i.e. a layer change). This happens because, when the toolhead slows down, there is still pressure in the nozzle due to forces that cause the plastic to resist being extruded to a degree that scales with the speed of extrusion.

\n

The right solution to this problem is enabling and calibrating Linear Advance, which directly models and compensates for the phenomenon. Unfortunately, most printers including yours are still shipping with firmware too old to have this feature, or built without including it. So, your options are:

\n
    \n
  1. Update to firmware with linear advance and calibrate it. The factor for PLA on a direct drive is likely to be around 0.04-0.06. This is the best solution but might be unnerving if you're not comfortable with making that kind of change to your printer.

    \n
  2. \n
  3. Print at lower speeds, so that there's less difference between the cruise speed and the speed at/near the corners.

    \n
  4. \n
  5. Print at higher acceleration/jerk, so that the decel/accel cycle at corners happens much faster giving less time for the residual pressure to push out more material.

    \n
  6. \n
\n

Some of the issues in your pictures, however, look similar to "blobs" but apppear to be warping/curling from insufficient cooling or excess bed temperature. These are the ones in the first few mm of the print, below the lettering on the back (the lettering clearly has actual blobs). Using a lower bed temperature or even unheated bed, and/or improving your cooling (replacing the part cooling fan with a larger one, or pointing a desk fan at the printer during printing) should make these issues go away. You can probably also avoid it by printing significantly slower, or possibly by reconfiguring your slicer to print the outer wall first, before inner walls.

\n" }, { "Id": "18960", "CreationDate": "2022-02-20T22:28:43.913", "Body": "

I want to use magnets to hold the lid of a box down tight enough to keep it relatively airtight (along with a rubber seal etc.), but I am not sure what strength of magnet to use, that will still allow it to be opened without causing damage either by having to be pried open or by crushing the print layers. I cannot seem to find any guide to how magnets are used for 3d printing at various strengths and I cannot afford to buy too many types that I am not then going to use.

\n

Any help that you can provide will be greatly appreciated.

\n", "Title": "How Do I Determine The Appropriate Magnet Strength For A Tight Seal On A Box", "Tags": "|3d-design|", "Answer": "

You can vary the \u201cstrength\u201d of the magnet by making a membrane of 3D printed plastic in between the magnets. The strength of the attraction is something like a logarithmic relationship to distance- small changes in thickness of material in between the magnets have a large effect on how strong they stick to each other. You could dial in your print by buying strong rare earth magnets and experimenting with attenuating the strength with the membrane. Could even use sheets of paper to get the right feel, then measure the thickness of the stack.

\n

If using an fdm printer, it could be helpful to insert the magnets with a pause in the printing, and print over the top, if there isn\u2019t a good way to otherwise capture them.

\n" }, { "Id": "18975", "CreationDate": "2022-02-22T18:22:08.747", "Body": "

Every time I print a large area I get the pattern in the middle.

\n

\"pattern

\n

Below picture was taken during printing. The material isn't evenly laid flat on the bed and some are being scraped up.

\n

It appears from some centimeters away from the edge and extends perpendicular to the nozzle moving direction.

\n

\"during

\n

The pattern appears at the similar position when I print the same model again.

\n

It's a delta printer. Print PETG on glass bed at 75\u00a0\u00b0C.

\n

Update:

\n

After adjusting delta height to add clearance, the first layer is super clean without any wave.

\n", "Title": "What cause the pattern in a large area?", "Tags": "|print-quality|petg|delta|", "Answer": "

The wavy lay down on the first layer usually indicates one of two problems:

\n\n" }, { "Id": "19025", "CreationDate": "2022-02-27T13:20:40.393", "Body": "

I'd like to force Klipper to perform power on (using M80) before homing. For this purpose I'm trying to override G28:

\n
[gcode_macro G28]\nrename_existing: G28_BASE\ngcode:\n  M80\n  G28_BASE { rawparams }\n
\n

But for some reason this does not work, I'm getting the following error:

\n
G-Code macro rename of different types ('G28' vs 'G28_BASE')\n
\n

Isn't G28 overridable? Is there any other way to achieve the desired behavior?

\n", "Title": "Klipper: power on before homing", "Tags": "|g-code|homing|power-supply|klipper|", "Answer": "

Besides using a different macro, it is also possible to use [homing_override] which allows you to redefine the homing sequence.

\n

You can write a simple homing_override like (untested!)

\n
[homing_override]\naxes: xyz\ngcode:\n  M80\n  G28\n
\n

and you are done.

\n

Be aware that this very simple override will home all axes every time homing is called: "G28 X0" will home also Y and Z. You can put checks to home only what is requested, see here but it become more involved.

\n" }, { "Id": "19044", "CreationDate": "2022-03-03T08:59:38.807", "Body": "

\"\"

\n

\"Errored

\n

The left part of the first picture is giving an inverted circle when lying flat, but the right part of the first picture is how it should be. Same STL for both slice orientations. I tried toggling union overlapping volumes with no luck. I tried to invert face normals with Blender with no luck either.

\n

This gives the error: The highlighted areas indicate either missing or extraneous surfaces. Fix your model and open it again into Cura.

\n

Here is a link to the model, this is another link.

\n

I have tried

\n\n

both without success.

\n", "Title": "Little help on this Cura slicing or model error?", "Tags": "|ultimaker-cura|3d-models|3d-design|", "Answer": "

I've discovered via Meshmixer that there are duplicate cylinders for the counterbores. I selected "separate shells" which generated two sets of cylinders in the Objects Browser as well as the main body. Selecting each flawed entry in the browser left behind the main body, which is manifold and passes the Analysis/Inspector test sequence.

\n

You can have the repaired file from my Dropbox, public folder, or you can perform the process on your own. If you select the former, please let me know when it's retrieved, as I endeavor to keep my space in the free category on Dropbox.

\n" }, { "Id": "19057", "CreationDate": "2022-03-06T17:44:24.183", "Body": "

On the last couple of polycarbonate prints with my Prusa, one or more layers have been completely skipped (i.e. no material extruded). The rest of the print is fine save some minor warping. I'm printing in an enclosure that gets up to ~60 \u00b0C with the power supply outside the enclosure. I'm also printing out of a dry box with a fairly long bowden tube. I suspect it has something to do with overheating because this doesn't happen when I print without an enclosure.

\n

I cracked the print in half just to take a look in between the layers.\"Side

\n

\"Bottom\"Top

\n", "Title": "Prusa i3MK3S+ layer skip with polycarbonate (overheating?)", "Tags": "|prusa-i3|adhesion|underextrusion|filament-jam|polycarbonate|", "Answer": "

Turns out I was just ruining the PETG parts that held the extruder gears. They didn't really melt but got soft and crept really badly. I replaced them with new parts everything was fine.

\n" }, { "Id": "19069", "CreationDate": "2022-03-07T22:39:27.283", "Body": "

Does 3D printed Alumide outgas or is it inert? I.e. If I printed a box with it and stored paper in that box for 100 years would the paper become damaged by out gassing?

\n

I noticed that NASA has a list of materials that out gas.

\n

But I'm not sure exactly what polymers are in 3D printed Alumide...

\n", "Title": "Does 3D printed Alumide out gas after printing?", "Tags": "|print-material|safety|alumide|", "Answer": "

Alumide is, as this question explains made up entirely of Aluminium Grit and Polyamide 12 dust.

\n

The Database contains one entry for Polyamide 12 (by filtering for it here):

\n

\"Polyamide

\n

Some other items of similar material (Nylon 6 & Nylon 12) are also in the database:

\n

\"enter

\n

As a result of the material being essentially aluminium and PA12, I would take the PA12 values for an estimation until the material was tested properly.

\n" }, { "Id": "19072", "CreationDate": "2022-03-08T17:28:29.633", "Body": "

Does anyone know what polymers are used in 3D printed Alumide? I know it has Al and nylon, but does it have anything else?

\n

NASA has a database of materials that out gas and I'd like to see if Alumide out gasses.

\n", "Title": "What polymers are used in 3D printed Alumide?", "Tags": "|print-material|alumide|", "Answer": "

Alumide is a registered trademark. It's source words are Alumunium and Polyamide. The first materials known under this name were sold in 2007.

\n

According to my materials provider, the ratio is but for tiny tollerances exactly 50% Polyamide-12 dust that is enriched with 50% aluminium particles and the material is used without special alterations to the machine in a typical nylon Selective Laser Sintering machine using a modified profile for PA12. Its MSDS only lists those two components, so there is nothing but those in it usually.

\n

As a result, the polymer binder is only Polyamide 12

\n" }, { "Id": "19078", "CreationDate": "2022-03-09T21:41:35.673", "Body": "

In every 3D print that I have seen, the bed should be leveled (manually or with some sensor-based system) on multiple points, pretty annoying because often when you reach the perfect distance on a point another point should be adjusted again repeating the procedure multiple time to have perfect leveling on all points.

\n

I'm wondering why height isn't just fixed with the optimal leveling.

\n", "Title": "Why should the bed be leveled and isn't just fixed optimally?", "Tags": "|bed-leveling|", "Answer": "

In addition to the answers above, you can have perfect levelling for one filament that doesn't work with another.

\n

So for instance my normal PLA works perfectly at one bed level, yet if I change to the generic silk I have it won't adhere unless I change the bed height fractionally. The differences are tiny but are make or break changes.

\n

Copper PLA filament is slightly different to both in terms of getting a nice first layer.

\n

ABS also requires slightly tighter bed levelling on my printer.

\n

So while I rarely change bed levelling, I do if I'm changing filament. But quite often I don't bother with the whole levelling thing, I just adjust the knobs while the skirt is printing.

\n

This is from my experience, your mileage may differ.

\n" }, { "Id": "19081", "CreationDate": "2022-03-10T07:16:30.433", "Body": "

There are numerous topics found on first layers that do not adhere properly causing prints to fail or cause print quality defects.

\n

The advice is often to properly level or tram the build surface. How does one tram the build surface?

\n", "Title": "How do you level (or tram) your build surface correctly?", "Tags": "|bed-leveling|tramming|", "Answer": "

For fine tuning I get good results by loading a model of any object with a rectangular footprint and scaling it to a size close to the printer's limits.

\n

With the skirt set to around 10 passes, start the print job, let the skirt print then kill the job.

\n

Peel the skirt from the bed and measure its thickness near the leveling screw locations with digital calipers or a micrometer and adjust the leveling screws as indicated.

\n" }, { "Id": "19103", "CreationDate": "2022-03-16T10:44:16.077", "Body": "

I have a problem with my E 3nder3 (3DTouch, Skr mini e3 v1.2, Dragon hotend with afterburner mount, other parts are original), where after using ABL, at the start of the print the Z-axis just seems to ignore where the 0 point should be and keeps trying to go down for a good second, then it just starts the print as if nothing happened.

\n

The things I've tried and the observations I've made:

\n\n", "Title": "Ender 3 with BLTouch ignores Z0", "Tags": "|creality-ender-3|troubleshooting|z-axis|bltouch|", "Answer": "

Successfully solved the problem. Removed the Z rod from its socket, and greased up the threaded slot it goes into on the x gantry, that seemed to solve the problem.

\n" }, { "Id": "19107", "CreationDate": "2022-03-16T18:13:48.567", "Body": "

I have a (preferably in matlab) 3D array of Booleans describing a voxel structure. I want to turn this into an .stl file but I don't want the final result to have the jagged cubes.

\n

I have tried putting the voxel array into a marching cube algorithm but the resulting STL is not a solid, some parts are infinitely thin sheets. I want every voxel to be represented by something solid.

\n

Is there a way to achieve this?

\n", "Title": "Obtaining a smooth solid .stl from voxel data", "Tags": "|3d-models|stl|", "Answer": "

Voxels are pretty much an interpolation of data points and you can only smooth them out by reducing the fidelity. Why? Because in acquiring the voxel, you already lost fidelity. Let me take you through an example.

\n

The basics

\n

For our basics we take a circle of Radius $R$. It can be fully described the following: $$x^2+y^2=R$$ All solutions to this give us points on the circle.

\n

Its representation as a single voxel with resolution length $r$ is that of a square. The square with its center on $\\{0,0\\}$ we will call the voxel of resolution $r$ around that coordinate. It is defined as $$x =\\{0.5r,-0.5r\\} \\land y=\\{-0.5r \\to 0.5r\\}$$ $$y=\\{0.5r,-0.5r\\}\\land x=\\{-0.5r \\to 0.5r\\}$$ Other squares then can be created by simply adding the respective cell's center's coordinates $\\{a,b\\}$ for $$x' =\\{0.5r,-0.5r\\}+a \\land y'=\\{-0.5r \\to 0.5r\\}+b$$ $$y'=\\{0.5r,-0.5r\\}+b\\land x'=\\{-0.5r \\to 0.5r\\}+a$$

\n

Looks complicated? True, but you see: the first formula is a circle with radius $R$ containing an area of $A=\\pi*R^2$. The latter two blocks define the square of length $r$ and area $A=r^2$. By clever positioning, the grid with $R=r$ has its $\\{0,0\\}$ Voxel wholly contained in that circle. Or in a picture, it looks like this, using $r=R=50$ (and thus a diameter of 100).

\n

\"50

\n

As you see, there is an area between the circle and the square - this is fidelity that is lost when converting to voxels. In a typical voxel transformation, there is only one choice: at which point of filling does a voxel get filled to 100% and at which it gets filled to 0%. Let's assume the circle there is actually a cylinder of 50 units height. Depending on where we set the cutoff, we now get one of three solutions. We could end with one voxel, using 50% as the cutoff point. We could end with a plus shape using 45% as the cutoff point or a square using any filling above 0% as a cutoff, as shown in the following picture.

\n

\"3

\n

It gets even more complicated if you don't have the circle and the grid's center overlap like n the example above: there is a "2x2x1" voxel solution that reflects a differently aligned circle with the same cutoff point as the "1x1x1" solution! That circle is (in the next graphic) centered around $\\{3.5 r,0.5 r\\}$ and thus shifted half a unit up and right so its center overlaps with a corner of the voxels.

\n

\"a

\n

Lost fidelity can't be regained

\n

You see, you have lost a lot of fidelity in voxelizing. All circles have become squares. All squares also are squares. Likewise, all curves in between have become squares.

\n

The problem

\n

So, it is nigh impossible for a computer to identify, what once was a circle square or square before all was made squares. Think... Colors: I take a picture of a smiley. I split it in half and then desaturate the upper half of the right side to black and white and totally saturate the lower. Those are two different ways to totally loose the color information. In the upper case you retain one information more, because the algorithm of making all colors but black white retains the black information, while the lower parts algorithm only retains the outer shape.

\n

\"A

\n

If you only had the black and white smiley, you have lost the information about its color in total. If you only have the black outline, you have lost also the information of the face.

\n

Voxelizaion does loose about that much information, depending on the settings.

\n

Reversing Voxelisation is interpreting the voxel structure

\n

It's not possible for a computer algorithm to simply invert the process. However, there are ways to interpret a voxelized object and try to re-create one of the possible objects that have led to this item, assuming that an "any fill is a voxel" algorithm has been used.

\n

For such, you'd import the model into a software such as Blender, solidify it so that it is one mesh without internal faces, and then running a smoothing operation. You don't get the object that generated that voxel structure, but you get an Object that would generate that voxel structure.

\n

\"Example

\n

How good the interpretation of the voxels is, depends on the resolution chosen in the start. If $r$ is small enough, then the resulting interpretation, together with some artistic definition of which corners are sharp and quite some manual post-processing (which requires the human eye) can lead to a somewhat good approximation of the actual object you believe lead to the voxel structure.

\n" }, { "Id": "19129", "CreationDate": "2022-03-21T10:11:29.947", "Body": "

I'm working on a gadget a bit like a jewelry box. I want the lid on a hinge. Are 3d printed hinges robust enough for daily use long term? Perhaps with a metal pin?

\n

I want to incorporate the hinge into the design but my thinking is that it would be a waste of time if the hinge will break as I'd need to reprint the whole gadget.

\n", "Title": "Tips on incorporating a robust hinge", "Tags": "|3d-design|", "Answer": "

Maker's Muse has a video on how to design hinged objects for printing. It's quite old and might be outdated with regard to materials, slicer functionality, etc. but I think it provides a good background on the topic and a source of ideas. One of the good tips is to look at the abundance of existing hinged designs on Thingiverse and other model sharing sites - that way you can study how they work and test print some before you spend time designing your own thing that might not work.

\n" }, { "Id": "19143", "CreationDate": "2022-03-22T19:16:24.250", "Body": "

\"enter

\n

I want to understand what the infill line distance of 0.4mm signifies. I am printing at 100% infill.

\n", "Title": "Infill line distance at 100% infill density", "Tags": "|infill|", "Answer": "

Presumably your nozzle width (and thereby your default line width) is 0.4 mm. The infill line distance of 0.4 mm means that the centers of lines are spaced 0.4 mm apart from each other, so that they exactly touch their neighbors at their width (0.2 mm from the center in either direction).

\n" }, { "Id": "19153", "CreationDate": "2022-03-25T02:06:58.490", "Body": "

While this is not directly 3D printing related, I intend to use my 3D printer (Labist's ET40) to shake air bubbles off of a resin casting.

\n

I printed parts, made a mould of them in silicon, then cast a copy with resin. All works, great looks and all! Except... bubbles. Especially at ceiling parts that are away from the gas hole.

\n

I tried a few different techniques to get rid of bubbles (heating, cooling, release agent, handshaking, high drop pouring, a mix of the above, different hole positions in the mould), and some I can't due to price restriction (different resin, vacuum removal degassing), but I was explained by an expert I could just have a machine do some low amplitude, high frequency shakin' for me. Again they proposed a specific machinery or DIY project, but I believe I already have the tool for it in the shape of a 3D printer's axes.

\n

Is there a way to create a G-code that would simply move at relatively high speed back and forth on the Y- and X-axis for, say 15 minutes? I'm sure there would need to be some variation to ensure the temporarily repurposed machine doesn't self-destruct in one way or another.

\n", "Title": "G-code to get the 3D printer to shake resin bubbles off from a casting", "Tags": "|g-code|mechanics|axis|labists-et4|", "Answer": "

What you suggest is pretty straightforward with g-code. One would expect to have the item secured properly to the print bed. The printer you've indicated has movement only in the Y direction, which means your g-code should reflect that movement only. X-direction movement means the head will travel left and right and have nearly zero effect on the item. This also applies to Z-movement of the carriage.

\n

Marlin (a common firmware) web page lists a specific g-code for repeat:

\n
\n

The Repeat Marker command is used to define regions of a G-code file\nthat will be repeated during SD printing. A marker is first set with\nM808 L[count], and later in the file a plain M808 command is used\ncount down and loop. (By default up to 10 start markers can be\nnested.)

\n

In slicer software put M808 L to the \u201cStart G-code\u201d and M808 to the\n\u201cEnd G-code.\u201d But this command is not the only requirement. Before\nstarting each whole object it\u2019s important to actually clear the print\narea of obstacles and to reset the coordinate system with G92 or G28,\nso this command is best used with belt printers or other systems with\nautomatic print removal.

\n
\n

From a different page on the same site:

\n
\n

The G0 and G1 commands add a linear move to the queue to be performed\nafter all previous moves are completed. These commands yield control\nback to the command parser as soon as the move is queued, but they may\ndelay the command parser while awaiting a slot in the queue.

\n

A linear move traces a straight line from one point to another,\nensuring that the specified axes will arrive simultaneously at the\ngiven coordinates (by linear interpolation). The speed may change over\ntime following an acceleration curve, according to the acceleration\nand jerk settings of the given axes.

\n
\n

More details specific to this command are also on the linked page. A simple example of a movement g-code:

\n
\n

The most basic move sets a feedrate and moves the tool to the given\nposition.

\n

G0 X12 ; move to 12mm on the X axis G0 F1500 ; set the feedrate to\n1500 mm/min G1 X90.6 Y13.8 ; move to 90.6mm on the X axis and 13.8mm\non the Y axis

\n

There are some caveats related with feedrates. Consider the following:

\n

G1 F1500 ; set the feedrate to 1500 mm/min G92 E0 G1 X50 Y25.3 E22.4 ;\nmove while extruding

\n

In the above example the feedrate is set to 1500 mm/min, then the tool\nis moved 50mm on the X axis and 25.3mm on the Y axis while extruding\n22.4mm of filament between the two points.

\n

G1 F1500 G92 E0 G1 X50 Y25.3 E22.4 F3000

\n

However, in the above example, we set a feedrate of 1500 mm/min on\nline 1 then do the move described above, accelerating to a feedrate of\n3000 mm/min (if possible). The extrusion will accelerate along with\nthe X and Y movement, so everything stays synchronized.

\n
\n

Consider to limit your acceleration based on the weight of the secured item. You would not be worried about precision, but skipped steps due to weight could bring the bed travel to the physical limit stops, causing stepper grinding.

\n" }, { "Id": "19163", "CreationDate": "2022-03-26T19:52:42.177", "Body": "

I recently started printing with an Ender 3 V2. I got a few good prints out of it, and then I have had nothing but issues.

\n

Basically, I can start a print, and after about 10-15 minutes, the temperature will decline from 200\u00a0\u00b0C to 195\u00a0\u00b0C, triggering a thermal runaway alert, (low temp), and then cancel the print.

\n

Here's an example of the hotend temp profile:

\n

\"Screenshot

\n

I have tried testing the hotend thermistor, running PID autotune (multiple times, with up to 10 cycles), replacing the thermistor, running directly from an SD card (instead of OctoPrint).

\n

Both thermistors worked, the first one read 108k\u00a0\u03a9 at room temp, and a new one is at 113.5k\u00a0\u03a9 at room temp.

\n

I'm running firmware v1.0.4 on a stock v4.2.2 board.

\n

One thing that did seem to save a print was that when I noticed the print temp dropping, I paused it for about 30 seconds and then resumed, and then the temp recovered up from 198-200\u00a0\u00b0C for a bit. But that's obviously not a solution.

\n

I'm lost, don't know what else to test/calibrate, any ideas?

\n", "Title": "Ender 3 v2 temperature drop", "Tags": "|creality-ender-3|temperature|thermal-runaway|", "Answer": "

Well, this was a setup error. I am amazed it worked at all, but the reason this was happening was that the power supply was set to 230\u00a0V from the factory.

\n

I am in the US, so the household current is 115\u00a0V. Once I switched the voltage, the printer has been very reliable, and I am not seeing heat drop-offs anymore!

\n" }, { "Id": "19172", "CreationDate": "2022-03-30T11:43:51.800", "Body": "

Prints on an Ender 3 Pro using a standard printing profile results in layer shifting or over extrusion. I can\u2019t figure out what the problem is. I tried reducing the flow as well as print speed but nothing really changes. Any thoughts on the XYZ cube test print below?

\n

\"3D

\n", "Title": "Why layer shifting prints on Ender 3 Pro?", "Tags": "|creality-ender-3|print-quality|layer-shifting|overextrusion|", "Answer": "

This isn't what folks usually refer to as "layer shifts" - those are generally permanent (for the rest of the print) and don't correct unless you get an equal opposite-direction shift, and are caused by things like a stepper motor skipping a step, a belt skipping a tooth, or the print surface shifting on the bed.

\n

Your problem looks more like excess play in the positioning, particularly on the X axis, likely due to a very loose belt. If so, it will manifest differently in different layers due to the geometry and the last direction of motion.

\n

It's also possible that it's Z wobble, where the toolhead experiences a Z-coordinate-dependent displacement in the X and Y directions due to irregularities in the Z motion system. This could be a bent or nicked lead screw or nut, flat spots or other damage to the V roller wheels, or debris on the aluminum V slot extrusions that the V wheels are rolling on.

\n

The fact that the top layer came out very clean and consistent makes me actually suspect it's the latter - some sort of Z wobble.

\n" }, { "Id": "19173", "CreationDate": "2022-03-30T12:01:46.543", "Body": "

I love the idea of the XYZ test cube to help diagnose my bad prints. I\u2019m able to find online very common print issues however am looking for a resource that is more extensive. Often I run into a situation where my issue is not covered.

\n

Would be great if there was a resource that had pictures of numerous bad or less than ideal XYZ prints with cause and fix for each. I\u2019m thinking of more than 50 examples.

\n

Does anyone know of such a resource?

\n

\"My

\n", "Title": "Is there an extensive XYZ Cube diagnosis resource?", "Tags": "|print-quality|", "Answer": "

An XYZ cube is a calibration test for dimensional accuracy testing, it is not meant for extensive trouble shooting. Therefor, there are no specific resources for cube irregularities, the regular resources can be used for trouble shooting.

\n

One of the better resources for troubleshooting print problems is found at Simplify3D or one at All3DP.

\n
\n

Do note that the dimensions in X and Y and Z are based on mechanical ratio's of the steppers with their pulleys or threaded rods, you can calculate the amount of steps required for the dimensional accuracy. It is only necessary to adjust this if e.g. the pulley is of a different diameter than the specification (as in poor quality dimensional pulley accuracy). In the basis you should not adjust the steps per mm based on small sized calibration cubes.

\n" }, { "Id": "19178", "CreationDate": "2022-03-31T21:52:01.607", "Body": "

So, I've gotten the G-code to work regarding the waiting 10 seconds as I wanted it to. However, does anyone know if there is any G-code that exists that will allow me to manually set the height of the heatbed? I'm trying to find out after each layer if I give it 10 seconds to fully 'cure' or 'cool' to see if the results will be better. But I need to move the bed down away from the extruder.

\n", "Title": "Setting Heat Bed Height Manually Mid-Print?", "Tags": "|g-code|heated-bed|", "Answer": "

The way to do this is something like:

\n
G91     ; Relative coordinates mode\nG0 Z5   ; Move +5 on Z axis\nG4 S10  ; Dwell for 10 seconds\nG0 Z-5  ; Move -5 on Z axis\nG90     ; (Return to) absolute coordinates mode\n
\n

You can of course insert this manually in the gcode between layers, but most slicers have functionality (built-in or as plugins) to let you do that automatically, or of course you could use a text-processing utility like sed, awk, or perl to do the insertions automatically if you're familiar with something like that and it's more to your liking.

\n

In a since-deleted answer, Trish raised some reasons you might not want to do this, including significantly increasing the time your print takes and possible difficulties getting the next layer to adhere to a fully cooled layer. I'm not convinced the latter is a real problem, but these concerns are worth considering. Some other reasons not to do what you're asking about include:

\n\n

If you want to cool the entire layer effectively before the next layer goes down, I would recommend "side fan" setups that produce a thin layer of airflow (ideally laminar) at the current Z height. You can find examples of this in lots of the #speedboatrace speed Benchy entries on YouTube, especially on Vorons and other CoreXY machines since it's the easiest to setup on a printer where the bed moves down (like yours) rather than the nozzle moving up, being that you can just mount the fans and ducts at fixed height.

\n

If you want to experiment with this without actually making fancy fan ducts and mounts, just get a desk fan or box fan and point it at your printer. This works surprisingly well.

\n" }, { "Id": "19191", "CreationDate": "2022-04-04T15:25:08.643", "Body": "

I am trying to print a shape (e.g. square container) using clear resin. However, I am facing the problem of the design sinking when I put it water. I am right now trying to make it hollow from the inside in some spots to reduce its density, but not sure how far can that take me. It is necessary for me that the print floats!\n\"enter

\n

Do you guys have any tips? like a different print mater? or a change in shape that will help the design float?

\n", "Title": "Which 3D printing material is best for floating designs?", "Tags": "|3d-models|3d-design|resin|water-resistance|", "Answer": "

The material doesn't matter*

\n

Floating designs are exactly that: designs. If the design is made to float, and your printer can create a watertight shell (which is a different problem than the material), then you can use any material. Well, almost any, because:

\n

Only one caveat remains

\n

The only type of material that isn't suitable is any material that will dissolve in the fluid it will swim in. Like PVA in water.

\n

Otherwise, the material only matters for other properties, but not for "it can float".

\n

How to design a floating item?

\n

When does a design float? Well, the rule described by Archimedes is often described as "An item floats if when the volume of water it displaces is lighter than the item." In other words: if its density is lower than water.

\n

However, it's not that simple after the first look: What is the volume and weight of the boat? After all, we can make steel, which weighs upwards of 3 tons per cubic meter float while water is only 1 ton a cubic meter. What is the mystery part? It is simple: the underwater ship does not allow ingress.

\n

Draft depth

\n

So, we want a solid wall from the lowest part to the line the item will sink into when it reaches equilibrium. That's the draft depth. For ships, there is a formula to quickly calculate the draft depth based on knowing how much water will be displaced:

\n\n

In the alternative:

\n\n

The second method also works for tilted or very asymmetric shapes (like a bot listing), while the first is roughly over the thumb.

\n

Now, how does that help with the design? Remember, we had sealed the slots earlier. If the slots are in any way below our draft depth, then water ingress happens through that slot - and the slot will have to be moved above the draft depth.

\n

Dasign adjustments

\n

There are several adjustments to get the item to have a lower draft, but all of them boil down to reducing the density.

\n

In the case of your design, the hanging floor of the basin can have a lot of it cut away and turned into a grid of sorts, which would reduce the impact this part of the print has to the total density.

\n

You could also move the slots up enough so that it will be above the waterline, even if filled to an expected degree.

\n

If you really need to include voids, you will need to alter your printing code in between layers, because resin printers don't typically raise the print fuly out of the resin vat and have such hollows filled with resin as a result. And please try to avoid Cupping

\n" }, { "Id": "19198", "CreationDate": "2022-04-06T01:36:46.673", "Body": "

Using cyanoacrylate to glue PLA parts sometimes leaves a white residue or haze near the glue locations. Is there an easy way to remove it?

\n

I've tried water and alcohol swabs but after drying the haze remains.

\n

\"Photo

\n", "Title": "How can cyanoacrylate haze be removed from PLA?", "Tags": "|pla|glue|", "Answer": "

Acetone or alcohol hasn't worked at all for me, if anything it made it worse by stripping the PLA of the sheen its supposed to have. I have tried various types of Acetone as well, all with similar results.

\n

I know this will sound crazy, but I have been doing this for about 6 months. I use Sesame Oil, applied generously with a Q-tip, and then I "rub it in/dry it off" with a paper towel. This has worked on various sheens of PLA and PLA+ and completely removes any evidence of super glue. I haven't seen the oil dry out yet either. At first I thought the oil would wear off or dry out, but it has not yet and its been 6 months since I've been doing this.

\n" }, { "Id": "19208", "CreationDate": "2022-04-08T11:30:13.170", "Body": "

I am new at this and maybe my model is not the best, I adapted it for another one actually. Can you tell me why I can't get the holes printed? I already checked the faces and they are all in the correct orientation (I think)

\n

What happens is that I start printing with the holes facing down and they are not printed at all. I never let it keep going for long but it seems to be completely filled inside.

\n

You can check the file here.

\n

\"enter

\n

\"enter

\n", "Title": "Not printing the holes", "Tags": "|3d-models|slicing|", "Answer": "

The problem is internal geomoetry

\n

The body you modeled consists of a non-manifold shell: There exists a fully enclosed shell on the inside of the item that tries to define an "outside" of the body. In the following picture, I have hidden part of the geometry to better show the problematic internal surfaces in orange:

\n

\"internal

\n

Automatic processes such as Meshmixer or Windows 10 3D builder interpret such an internal, one-sided open cylinder as "This probably is missing a surface on both ends". This solution leads to two intersecting and manifold shells - a cylinder overlapping the drilled holes and the body with the drilled holes - which then promptly get treated with a boolean union... and voila! No more holes. Or even no more outer shell as the easiest solution is to just stitch that lower surface and discard the rest. This is what happens when Meshmixer does just that: you are left with the cone half and some inverted artifact areas.

\n

\"Meshmixer

\n

So the best solution is to ensure the parts don't contain such volumes encased by a non-manifold surface in the first place. Due to the nature of the part, in this case, it is rather simple: Simply removing the circle of vertices that spans up both the plane, as well as the cylinder, marked orange results in all internal surfaces getting removed. Note that due to the orange parts sharing (at least partially) vertices with the wanted outside, this has to be done manually. Would both surfaces share no vertex, a simple "separate shells" operation could result in a very quick way to remove offending structures.

\n

\"Without

\n

Without the internal geometry, the model gets interpreted correctly - the mere presence of such superfluous internal geometry makes the slicer believe that some surfaces are inverted or missing, and thus need to be inverted or stitched - and the solution to the slicing is... utter mess.

\n

\"Cura\n\"Slicing

\n" }, { "Id": "19210", "CreationDate": "2022-04-08T18:26:00.620", "Body": "

About a day ago my Ender 3 v2 just shut off while printing. It didn\u2019t really bother me since this has happened many times before and switching the power supply off and then turning it on would fix the issue. This time though the printer is no longer turning on. I opened up the power supply and the indicator light is turning on, but when I check the outputs with a voltage meter it says 0. What could be the problem here?

\n", "Title": "My Ender 3 v2 has randomly turned off and no longer turns on. Any suggestions on how to fix this?", "Tags": "|creality-ender-3|", "Answer": "

I'm not familiar with that particular power supply, but usually they have a fuse. In your case it's more likely to be a capacitator causing the problem though as fuses are not intermittent problems and they're usually situated before any board lights for obvious reasons. They normally either work or fail.

\n

I'm assuming you checked for loose wires and connections.

\n

This is a job for an electrician, capacitators cannot easily be tested without proper equipment and need some good soldering skills to replace properly. If they're liquid filled then you can sometimes see doming on the top of the capacitator where it looks a bit deformed. Or even a leak. If they're solid state then there is no easy way to see if they're failing.

\n" }, { "Id": "19215", "CreationDate": "2022-04-10T07:33:36.303", "Body": "

This problem has been occurring for a while. On the top of round objects, you can see the individual layers. Maybe I just need a lower layer height.

\n

\"Photo

\n", "Title": "Weird top layers on Ender 3 Pro", "Tags": "|creality-ender-3|", "Answer": "

Kilisi is absolutely right that you necessarily (without advanced non-planar slicing techniques that aren't available in production slicers) have a "stairstep" effect whenever you have a shallow angle top surface like that. However, it looks from your picture like you also have some gaps that are accentuating the problem and making your top surface non-watertight. This can be fixed.

\n

Slicers (at least Cura) are fairly bad about figuring out where they have to put material under the very top layer to ensure that you have a solid wall of the desired thickness. Where the outer wall face is pointing almost-upward, you would need either a lot more outer perimeters than the shell thickness you want, because they're significantly offset from each other (often by as much or more than the whole 2D wall width) at each successive layer. Using excessively many walls will solve this, but wastes a lot of print time and material. Using more top layers is the easiest fix I know. I find that 5 top layers at 0.2 layer height pretty much always gives solid curved tops, even with spherical top shape. The only way that might fail is if you have really low infill and they all "sink in" rather than bonding properly.

\n

Of course these gaps could also be caused by underextrusion or misplaced extrusion. Check instructions for enabling and calibrating Linear Advance/Pressure Advance on your printer for one of the big ingredients in fixing top gaps and related extrusion inaccuracy problems.

\n" }, { "Id": "19219", "CreationDate": "2022-04-10T20:13:44.160", "Body": "

I just bought and built a Creality Ender 3 V2. The last step of the setup is to flip this little yellow switch on the power supply to be either 115\u00a0V or 230\u00a0V. From my other research, I think that I have to flip the switch to 115\u00a0V because I live in the US. Since US standard outlets give 120\u00a0V of electricity and not 115\u00a0V, I am confused. Why is 115\u00a0V "correct" if the voltage I will supply is actually 120\u00a0V? Will this extra 5 volts burn out the printer?

\n", "Title": "Creality Ender 3 V2 little yellow switch on power supply", "Tags": "|creality-ender-3|power-supply|", "Answer": "

It won't burn it out. The voltage is between 110 to 125. Power supplies are designed to work in that range. So sometimes you'll see appliances rated for 110\u00a0V, 115\u00a0V, 120\u00a0V... they actually are all the same.

\n" }, { "Id": "19221", "CreationDate": "2022-04-10T20:52:14.880", "Body": "

I am using an Elegoo Saturn S and have leveled the build plate per manufacturer's instructions a handful of times. However, I have yet to get a successful print except for the rook model that came with the printer and a couple Validation Matrix prints, which are pretty thin. I've switched resins and am finding more luck with grey vs white. White always resulted in delamination at some point early in the print and supports not printing (even with thicker supports). Grey is printing alright for the most part but most of the raft falls off the build plate. There's just enough adhesion that the print does not outright fail (surprisingly) but a good 60%-70% of the raft is hanging loose. I had one print that could have finished and the current print is about half loose

\n

I assume that this is a leveling issue, but as I said, I've leveled the bed numerous times. I bought a new build plate (to counteract another problem) and I've recently changed the FEP. It wasn't until this change and switching to grey resin that I've had any remote success. One thing I've noticed is that the resin pools more to the left side suggesting the work area is not level. Would this matter? I figured that if the build plate is level with the LCD, it shouldn't matter if everything is off. The angle seems to be about 2 or 3 degrees.

\n

My current settings:

\n\n

Originally sliced with ChituBox, but that stopped working for some reason. Slicing with Lychee now.

\n", "Title": "Does an uneven resin printer matter if the build plate is level with LCD?", "Tags": "|bed-leveling|adhesion|resin|elegoo-saturn|", "Answer": "

Within reason, yes

\n

If you can't put the printer down 100\u00a0% even, that's ok. The most crucial thing is, that during the whole operation the resin level in the printer does never get over the edge of the print basin and that at the lowest level, everything is covered still. It's still better to settle the printer as even as possible.

\n" }, { "Id": "19225", "CreationDate": "2022-04-11T10:00:15.477", "Body": "

What are the real overhang limits? I see a lot online about 45\u00a0%, then up to 60\u00a0%, but I'm routinely doing them at up to 90\u00a0% for "shortish" distances and 80%+ for several centimeters at a time. I haven't tried to see how far I can do it, since I don't have filament to waste on that sort of thing.

\n

It's making me wonder how believable all the YouTube and website experts are. Same thing with stringing, they all talk about it on Ender 3's but this print has been going for 25 hours and has maybe two tiny strings.

\n

This is the second time I've printed this same STL with no supports.

\n\n

\"Overhang\"

\n", "Title": "Practical overhang limits", "Tags": "|print-quality|", "Answer": "

"Real overhang limits" are hard to define. If you want accurate extrusion, such that precision parts fit together correctly or angled geometric surfaces that are supposed to be flat come out flat, each extrusion line must have at least some minimal portion of itself (probably including its center line) printed on top of and against existing material underneath that already has sufficient rigidity (both geometrically and in terms of cooling) not to deflect when printing against it. In this sense, the overhang angle is arctan(lw * (1-k) / lh) where lh is layer height, lw is line width (normally nozzle width), and k is the portion of overlap you demand. For example at 0.4 line width, 0.2 layer height, and 50% overlap, you get out exactly 45\u00b0.

\n

If you just want the printed part to have basic structural integrity, things get a lot more fuzzy, and dependent on the geometry - particularly, the convexity/cocavity of any overhanging extrusions. Concave overhangs, like the inside of a spherical dome, will quickly fail as soon as you lose most or all of the overlap - expect them to hard-fail at arctan(lw/lh) (63\u00b0 in 0.4/0.2 case) since the material will just be dragged inward around the curve with nothing to stick to. You might get a little bit more overhang if there's already a horizontally adjacent extrusion in the new layer for the material to stick to, but in my experience it will be unreliable.

\n

Convex overhangs, on the other hand, can work out even when they're extreme. This is because the curvature of the toolhead path pulls the new material towards/against a region where it has existing material to bond to.

\n

Keep in mind that layer height is a free parameter you can tune, that greatly increases the overhang available to you. Some slicers also have "adaptive layer height" settings to use thinner layers precisely in the layers that have severe overhangs. Line width is also a parameter you can tune, and increasing it works in your favor up to a point. But once you get to a point where the "wider than nozzle" line is attempted over thin air, it will fail badly, sagging down rather than expanding to the desired width, and not bonding to adjacent lines. So if you use wider lines to get better overhangs, you need to be very attentive not to go over angles that would place their centers off of the previous layer.

\n" }, { "Id": "19228", "CreationDate": "2022-04-11T12:23:33.497", "Body": "

I made a hook recently that I put up with VHB tape. It fell off around 2 weeks later (the hook come loose from the tape). It's just a hook for an empty backpack, so let's say it is under medium load.

\n

Every time I design something like this I wonder if it helps to roughen up the surface before applying the tape. I'd imagine that the tape grabs onto grooves made with a knife or sandpaper but I am not sure tbh. I've seen some videos where someone preheats the tape before applying it but without explaining why.

\n

Does anyone know how that works best?

\n", "Title": "Does VHB tape stick better on 3D prints if the surface is completely flat or roughed up?", "Tags": "|pla|post-processing|surface|stability|", "Answer": "

If you ook into the VHB design document, the section "How to Prepare Specific Surfaces" describes how to treat the surface:

\n

\"enter

\n

For higher adhesion, a primer should be used according to the document.

\n

This reference describes that roughing up plastic parts can be beneficial for the adhesion of some tapes:

\n
\n

Abrade the Surface: Roughing up the surface (i.e. sanding) will loosen up any accumulated dirt, rust, or chipped paint. It will aid in the adhesion to painted surfaces or plastic items. A finely- abraded surface with shallow scratches created by a circular motion (rather than straight lines) has the best potential for a strong and persistent bond. This method can create up to a 40 % increase in surface area and can result in greater immediate and long-term bonding potential. Scrub pads, fine steel wool, or sandpaper can achieve the right level of abrasion. A palm sander could be helpful for larger jobs. Avoid using coarse abrasive materials because a too-rough substrate may inhibit the adhesive flow onto the surface. Always clean with the IPA/Water solution, or other solvents, and make sure all loose particles are removed. While it is not typical, some high-bond tapes adhere best to smooth, glassy surfaces, so double check with the manufacturer before you abrade the surface and compromise the bond strength.

\n
\n

Both references describe the use of a cleaning solution based on 50\u00a0% water and 50\u00a0% IPA.

\n
\n

A higher temperature is beneficial for obtaining the bond strength faster, the VHB design document shows that increasing temperature shortens the time when the full bond strength is reached:

\n

\"enter

\n" }, { "Id": "19238", "CreationDate": "2022-04-15T13:32:31.390", "Body": "

As an engineer I was initially interested in making parts. For example I designed and printed a better part for something which wasn't available locally, and even had a client who wanted 150 of them. But print time was 23 hours per part. and I didn't have full confidence in the robustness of the part. The layer lines are a big weakness and anything less than 3\u00a0mm is so flimsy that it's a waste of time. So robust performance parts are out. As are high tolerance ones. Build volume makes it even less useful. And the design compromises you have to make are difficult to justify if there are other ways.

\n

Then with other network parts I thought of designing the vast majority needed other bits and pieces, screws, shafts, connectors in metal that I'd need to source and assemble. Enclosures were okay, but weak, and I can fabricate those stronger and faster in other ways.

\n

So now I mainly just print to what I perceive to be 3D printings strengths and have almost given up on parts.

\n

Has anyone had a different experience?

\n", "Title": "Under what circumstances are 3D printed parts actually viable with a desktop printer in comparison to other production methods?", "Tags": "|print-quality|", "Answer": "

I've only just started my journey in 3d printing, but I have a little hobbyist experience in other forms of "structural crafting". I can see a few ways you can use 3d printing to enhance other techniques - the main one being that a printed part can be used to make a mold, which you can then use to cast aluminum, bronze, or whatever else.

\n" }, { "Id": "19252", "CreationDate": "2022-04-18T10:50:11.667", "Body": "

I want to print out a flat object without any support structure straight onto the build plate of my ender 5. It's going to be PLA and I need it to be thin enough to still be flexible.

\n

I don't have a picture available, but imagine that I wanted to print out the Coke Cola and then wrap it around a bland soda can, so that the logo is raised up slightly?

\n

Alternatively, what is the best layer height to use, and how many layers should I use?

\n", "Title": "How thin should I print a logo so that it is flexible enough to wrap around a curved object?", "Tags": "|layer-height|", "Answer": "

1 layer, of whatever thickness your device can print.

\n

I did something like this to print letters for a flat sign. The letters were 1mm thick and didn't bend, but I'd accidentally printed quite a lot of brim lines which were a single layer thick, and they'd merged into each other.

\n

The brim worked superbly as a "net" to hold the letters in place, so in theory you could print as little as one layer thick.

\n\n" }, { "Id": "19256", "CreationDate": "2022-04-19T01:43:10.483", "Body": "

Will build surfaces coated in PEI lose their qualities, such as adhesiveness, if left unused but removed from packaging for long periods of time?

\n", "Title": "Do unused PEI sheets degrade over time?", "Tags": "|build-plate|build-surface|pei|", "Answer": "

The PEI will be fine. It's widely used in industrial high impact, high heat and high repetitiveness parts because it doesn't degrade easily. Probably some in your car.

\n" }, { "Id": "19267", "CreationDate": "2022-04-20T10:18:24.720", "Body": "

I'm trying to figure out if it's worth buying a 3D printer or using an online printing service like e.g. this one in the long run. Anyone have any cost analysis?

\n", "Title": "Advantages of buying a 3D printer versus using online services", "Tags": "|cost|services|", "Answer": "

It is probably cheaper for a business to order 3D prints than to pay an employee to get up to speed, unless that business is doing new designs regularly. This is especially true if the parts need to look professional, or need to use challenging materials (like anything other than PLA, PETG, maybe TPU).

\n

If the business does buy a 3D printer, it can be economical to get a more prosumer model, than to pay an employee to futz with a consumer printer to coax good print quality out of it, or spend additional time modifying it- a 300 dollar printer can turn into a 1500 dollar printer with enough things going wrong or needing to be upgraded.

\n

When evaluating the true price of an in house 3D printed part to a business, it\u2019s important to include employee time interacting with the machine and post processing: file prep, slicing software work, prepping bed, loading filament, preheating bed, watching the first layer go down. Oh wait, it got messed up? Start again. Once printing, time spent checking in on it, then time spent unloading filament, hand post processing (trimming a brim, removing goobers, zits, strings, drilling out precision holes). It is a hands on process, especially with consumer machines. Producing parts on the clock, I figure between 40 minutes (everything runs perfect) to two hours (Murphy\u2019s law) of human interaction for a one-off part. Batches of more than one can be less, I had little clam shell enclosures done in batches of 6 come out to about 45 minutes of labor apiece to the company, ultimately (once 3D printer was dialed in).

\n

Another factor of overhead is a 3D printer needs a space for it to live.\n-Near ventilation if using noxious materials\n-with enough table space to have a computer near it, and some hand tools, and a clear area to post process the parts\n-away from employees bothered by the sound\n-where it can be checked on easily\n-where it doesn\u2019t get cold in the winter, to the point where low ambient temperatures cause warping

\n

Note, some of these space requirements can be sidestepped by printers with prosumer features: WiFi transfer of gcode to the machine (computer doesn\u2019t need to be near printer), full insulated enclosures (cold ambient temperature resistance and less noise) with air filtration (may not need ventilation for fumes).

\n

Business ownership of a 3D printer can be a waste if there isn\u2019t someone on staff that is mechanically inclined and can use a CAD package. Without CAD skill prints are limited to what independent contractors design for the company (and who may themselves own their own 3D printer), or things that can be downloaded from the internet.

\n

As an asset, it can be difficult to sell a 3D printer for anywhere near what it is worth, especially with aftermarket modifications. They are large and heavy and delicate so local sales are preferred over shipping, which severely restricts the market, especially outside urban areas.

\n

Not a business? Time on your hands? Go for it.

\n" }, { "Id": "19300", "CreationDate": "2022-04-26T19:45:35.450", "Body": "

I recently installed Octopi on my Raspberry Pi 4 and noticed some unusual behavior in that I lose the Wifi connection every 5-10 minutes immediately after boot. Once disconnected, I cannot re-establish the connection because my network's SSID doesn't even appear in the network list anymore. The only way I can re-establish the connection is to reboot the device.

\n

With that said, I did find a troubleshooting discussion of similar problems at octoprint.org: OctoPi losing network connection mid-print.

\n

Following the various advice, I must have tried about 12 different things, but none of them have fixed my issue. At first, I thought that wifi power-save mode was the most likely culprit. iw wlan0 get power_save indicated that power-save mode was turned on, but then I turned it off with iw wlan0 set power_save off and the wifi still disconnects.

\n

Going a step further, I set up a script to run the iw command right after boot so that the change is made permanent, but that didn't work either.

\n

Other troubleshooting attempts I tried:

\n\n

I am at my wit's end.

\n

As for my setup, I have an 8 GB Raspberry Pi 4 and am using an image of OctoPi 0.18.0 with OctoPrint 1.7.3. This I downloaded and imaged onto a 128 GB micro-SD card using the Raspberry Pi Imager. My wifi network is 2.5 GHz secured with WPA2 with a visible SSID and is definitely within close range. One way that I know that it is not a hardware issue is because I have another image with the Raspberry Pi OS 64-bit version and wifi works just fine when I run that.

\n

As for Octopi, one atypical difference is that I am running it with a desktop. It may be that, for whatever reason, perhaps that particular distribution of RPi OS has a major bug in it? If so, then maybe I do have a solution, but I don't want to run without a desktop because I have a nice setup on my 3D printer that includes a touch screen. Given that is the case, could I maybe use the 64-bit Raspberry Pi OS and just load OctoPrint onto it with sudo apt-get [package-name] or something like that?

\n

Any additional troubleshooting advice is much appreciated, but I suspect that not much else will work. I am not a greenhorn when it comes to linux-bases systems, but this is my first time trying out an image using Octopi.

\n", "Title": "Wifi goes down in Octoprint after 5-10 minutes with reboot required to reconnect", "Tags": "|octoprint|raspberry-pi|wi-fi|", "Answer": "

I think I have a solution for this. Please follow the steps mentioned in this Github page of mine for the Wifi connectivity issue. I rarely have any issues with the wifi signal dropping randomly.

\n
\n

Wi-Fi connectivity issue

\n\n

check_wifi.sh - script to check if Raspberry Pi is still connected to the wifi or not. If not then it restarts the wlan0.

\n
#!/bin/sh\n# keep wifi alive\nif ping -c3 192.168.0.1 #router ip address\nthen\n    echo "......"\n    echo "No network connection, restarting wlan0"\n\n    sudo ifconfig wlan0 down\n    sleep 30\n    sudo ifconfig wlan0 up\n\nelse\n    echo "Wifi working normally."\nfi\n
\n\n

Additional resources:

\n\n" }, { "Id": "19301", "CreationDate": "2022-04-26T20:10:26.357", "Body": "

I need to print parts that fit together very well on the Longer LK5 Pro. However, after printing a Benchy, I noticed that whatever I print has a lot of imperfections. Is there any way I can fix this? All I know about the printing conditions was that I was printing at 230\u00a0\u00b0C nozzle temperature with 60\u00a0\u00b0C bed temperature. I was using PLA+. I was also printing the Benchy file that comes with the Longer LK5 printer. I tried tightening the Y-axis belt, that moves the bed, and the wheels on the bottom of the bed.

\n

Here are pictures of my Benchy:

\n

\"Top

\n

\"Side

\n

\"Bottom

\n

\"Right

\n", "Title": "How to tune Longer LK5 Pro for smoother prints?", "Tags": "|extruder|bed-leveling|stepper|belt|longer-lk5|", "Answer": "

Just got an LK5 and printed the included Benchy gcode. It looks pretty similar to the benchy in the pics with some of the blobs in the same location. Then I downloaded the benchy from Thingiverse and sliced it with the Longer 1.3 Slicer. Pretty good results from doing that but it took longer to print.

\n" }, { "Id": "19306", "CreationDate": "2022-04-28T12:45:04.880", "Body": "

I have a very eccentric, weird, unusual and strange idea. I need some advice and serious professional help.

\n

I'm interested in 3D printing in PLA a hollow complex structure with 0.2 thickness walls (Yes! That thin!). Fill it with very fine copper powder with a little borax powder thoroughly mixed. Use superglue to join halves or other shell pieces together, making sure the powder is very well compacted. Then in a separate container I want to make some thin plaster of Paris (calcium sulfate with a lot of water). Mix in it, some of my trimmed hair (about 5mm in length). No joke. Seriously. Please, I'm begging you with all my heart, hear me out! There's a very good useful reason for doing it. I then place the object (3D printed flimsy crappy shell filled with copper powder) in a DIY drywall box and pour in the plaster over the 3D printed shell object until the box is filled and object completely covered. Leave it to dry and completely solidify for a day. Then I bake the entire thing in a furnace making sure I'm over the copper melting temperature and voila! 3D printing in copper very complex intricate models with ease. Can it be that easy? Or am I deluding myself?\nThe hair purpose, after it will burn inside the plaster while in the furnace, is to create very thin tubules or air holes for water and gases to escape and to prevent cracking of the plaster under intense heat. I don't want to use hay because the straws are too thick. I have to use very thin organic straws. I just can't think of anything more accessible than my hair. Do you know of something even thinner and more accessible than human hair? Please let me know. I know it sounds and looks very odd, weird and strange. I'm opened to alternatives or other suggestions, otherwise I wouldn't be here making a fool of myself with such an insane ridiculous idea.

\n

I was thinking to add some form of additional volume above the object, which is connected to the model by some thin hollow tube. All this volume (like an empty cube (shell) ) will also be filled with very fine copper powder providing additional melted copper to the model, in the case if the powder was not very well compacted inside the shell model.

\n

Could this absurd ridiculous insane crazy idea work? I have never heard of anything like this. This is so bizarre and strange. It seems to be some form of odd mix of multiple techniques. But besides all this, will it work in the end? Will the plaster hold while some of it(depends on the model) will be inside molten copper? Or do I have to mix in the plaster, not just hair, but also some individual singular fine strands of steel wool?

\n

I don't know who and where to ask such a thing. Am I in the right place? I don't know what this idea is, I don't know how to name it, I don't know how to ask or formulate this idea, I don't know how to google it or search it. I don't know anything. I really need some guidance, help and advice.

\n", "Title": "The weirdest DIY 3D metal printing", "Tags": "|pla|metal-printing|", "Answer": "

I don't see why it wouldn't work. It doesn't seem to be the optimal way but I haven't tried it.

\n

Only thing that might be an issue is that PLA doesn't burn away clean (not for me anyway) which can leave defects in the product. But there are filaments specifically made for casting which apparently burn away with no residue.

\n

This is assuming you can actually successfully print a complex object with walls that thin.

\n" }, { "Id": "19320", "CreationDate": "2022-05-02T15:45:28.323", "Body": "

I build and export my model using ZBrush and as STL files.
\nTo fix the mesh for 3D print, I try to use 3D Builder which can automatically repair my parts.
\nAs it saves as a single file, if I import all parts at once,
\nI import the files one by one, repair them, then save them as a new file.
\nAfter all the parts were repaired, I import all parts to see the result
\nbut find some repaired parts' positions shifted.
\nHow should I handle these issues?

\n

\"Rendering

\n", "Title": "3D Builder: position shifted after auto repair", "Tags": "|repair|print-preparation|", "Answer": "

I came across this thread as I was facing the exact same issue. I am also using 3D Builder to repair my STLs before 3D printing them.

\n

The previous answers were however not sufficient (for me) so I went on a trial-and-error spree to determine what was happening, I think I found the answer! (hope that you can still use it!)

\n

An STL file has certain characteristics regarding its location (origin point): X, Y, Z and rotational for the same axes. When you import one or multiple STL files in the 3D builder, it keeps all of these origin points except for the z axis, it automatically places the model/models on the bed so that the lowest point in your (combined) model has a point of Z = 0.

\n

To ensure that all your parts have the same "origin point" you should import the model all at once and then remove all parts except for one. Then, auto-repair it and save it under a certain name.

\n

For example: You have a combined file consisting of three parts.

\n\n

You now have a repaired model for all parts and their origin points should all be the same!

\n" }, { "Id": "19326", "CreationDate": "2022-05-03T15:49:01.490", "Body": "

I have an old notebook computer that works just fine, but the outside of the lid is badly damaged and needs to be replaced. The screen and wiring are fine, so I only need to replace the housing that is exposed to the outside world.

\n

What is the best filament for an impact-resistant printed housing? Should I consider other options that may prevent damage to the internal components? Are there any alternatives with cosmetic benefits?

\n

Edit:\nSince I was asked, presume I may be willing to buy a new part to upgrade or accommodate a new filament type.

\n", "Title": "Best filament to print the housing for a laptop?", "Tags": "|filament-choice|", "Answer": "

For casings I use a combination of TPU and PETG or PLA. PETG shell gives it rigidity and TPU gives it a bit of impact protection. So corners and inside layers of TPU within a hard PETG or PLA shell (shell has no corners).

\n

I haven't had a problem with either but obviously PLA won't withstand heat very well, so it depends on environment.

\n

For a laptop case you'd maybe want to do it the other way around with the outside shell of TPU and inside layers of PETG for rigidity.

\n" }, { "Id": "19333", "CreationDate": "2022-05-04T03:00:20.000", "Body": "

The printer is a Creality CR-10 S5.

\n

Marlin 2.0.7.2

\n

\"enter

\n

\"enter

\n

\"enter

\n

\"enter

\n

Here is the highest point.
\n\"high

\n

Deepest valley
\n\"deep

\n

Further the leveling screws on the right X0,0 and X=0, Y=467 are barely at the very bottom of the screw. The ones on the left side are cranked super tight nearly at the maximum tightness. The BLTouch is doing a 10x10 grid so the resolution is pretty high.

\n

Any suggestions as to how to get it flatter? Does anyone else have this issue and resolve it?

\n

I can print with a raft. Any way to confirm if my bed is actually this bad or sensor glitch. I obviously can't see at sub-millimeter levels with my eyes.

\n

Could this be an issue with the Z-axis?

\n

So here are the wheel pictures:\n\"enter\n\"enter\n\"enter

\n

\"enter

\n

So, I changed the wheels, and re-leveled. I got this:

\n

\"after\"\nExcept for about 1 cm of the corner opposite the red side it is level enough to print on.

\n", "Title": "Bed leveling suggestions given this mesh generated by BLTouch", "Tags": "|bed-leveling|bltouch|creality-cr-10|", "Answer": "

Understanding the pattern

\n

Indeed, it looks like the error is induced by the X-Axis, as the pattern is very uniform over the Y-axis. The most likely reason for such an error is either damage to the rail, or damage to the motion system using the rail.

\n

Damage to the rail would either be a deformed or a deep spot in it.

\n

However, in your case, it is quite easily the wheels being worn down on the leading and lower roller while the trailing one does show to be not pressed against the rail properly. As a result, the print head can possibly might either tilt a little or the contact surfaces of the wheel might no longer run perfectly concentric with the wheel's hub itself and the printhead. In both cases, the head does a little wave dance around the ideal Z-position.

\n

I would try to tighten the rollers, and in either case swap them out for spares.

\n

After re-fitting

\n

The new mesh leveling tells me that the back right corner needs to go down quite some (about a millimeter) and the whole left can go up a little. The front right looks about right, maybe a little on the low end, but not too much. A BL-touch printer still needs leveling in itself.

\n" }, { "Id": "19340", "CreationDate": "2022-05-04T22:30:40.983", "Body": "

I recently broke the cooling fan on my Voxelab Aquila while doing some maintenance, and I'm in the middle of a multi piece print that I would like to finish soon. I found the fan replacement I need, but it's not going to arrive for over a week. I was wondering if it would be ok to print a few things with the broken fan, or if that would be bad for the motor or anything else.\n\"enter\"enter

\n", "Title": "Can I Print With a Fan Missing 3 Blades?", "Tags": "|print-fan|", "Answer": "

I think you'll get weird print artifacts and strange surface errors.

\n

The fan is not balanced and will buzz. This added vibration could be seen as an effect in the finish.

\n

Personally I'd suggest patience and wait for the replacement to arrive before continuing. Or scavenge a suitable fan from something else in the meantime.

\n" }, { "Id": "19353", "CreationDate": "2022-05-07T02:55:07.467", "Body": "

Most of the guides I can find are just canned responses to specific questions. Instead I'm looking for something meant to teach good fundamental understanding and core needed skills. Beginner's guides are common in other hobbies but I am having trouble finding one for 3d printing.

\n", "Title": "Has anyone written a primer for hobbyists new to 3D printing?", "Tags": "|knowledgebase|", "Answer": "

Here's a brief outline I threw out in chat once. I'm marking this as a "community Wiki" answer so feel free to edit.

\n

It is not a full Primer, so should date better than a Word6.0 manual.

\n
\n

Start by reading the instructions that came with your printer. There's a high chance that some assembly is required, and if you get something wrong then things may nor work right later. Some brands come complete, some are better than others in this regard. Take your time.

\n

For most people, they spend the first couple of weeks failing prints for multiple reasons. For me it was bed levelling and getting the first layer-adhesion, and filament tension.

\n

So work on getting the bed levelled, work out how much gluestick or tape your filament needs to work, and what temperatures work in your environment.

\n

I use 210\u00a0\u00b0C on the hotend for PLA+ and 60\u00a0\u00b0C bed temp, though others get away with 190\u00a0\u00b0C on the hotend and 50\u00a0\u00b0C on the bed. My printer is in a garage though.

\n

Try and print a 20\u00a0mm cube or a benchy.

\n

After that, explore http://thingiverse.com or http://thangs.com looking for pre-made stuff that you would benefit from. Start small.

\n

The Grab Toy Infinite is a great starter - it's very forgiving about tolerances, and kids like it. Expect rough handling to break it.

\n

When you're happy printing other people's things, identify some needs of your own. In fact, make up a document / draught email / notepad of ideas of things to print. I add stuff to mine all the time.

\n

When you've got a need that no one else can fill, you can start designing your own item and do the whole

\n
idea -->  ||:  (re)design --> implement --> test --> curse :|| success!!    loop.    \n
\n

Many people bang on about expensive fancy software, but you can make a perfectly adequate part using http://tinkercad.com/ as a grounding.

\n

For example, I had too many spare hacksaw blades and none of the "holders" I could buy were perfect, nor even close. Here's my output:

\n

https://www.tinkercad.com/things/9yQMmxRv4Lz-spare-hacksaw-blade-holder

\n

Like many things in making, expect to fail and learn and do it again.

\n

Sometimes it looks like we buy printers to print things for the printers for printing things for the printers...repeat.

\n

Look for needs in your life and design something to fill them. It's most satisfying.

\n

There's a huge gap between Functional prints, which do a job, and pretty prints which are just to look nice.

\n

Functional things are great - you can therefore justify the cost of more printer upgrades. LOOK AT ALL THE MONEY WE SAVED!

\n

But overall enjoy yourself and the time you spend making things.

\n" }, { "Id": "19359", "CreationDate": "2022-05-08T04:23:23.410", "Body": "

Looking to print a new part for a home appliance. There's going to need to be a new model created with the customizations made, but the model (after printing) will have to fit where the old part was. Is there any 3D modeling software that is better for this purpose? Will I just have to guess at proper proportions and hand-adjust the scaling of each dimension and angle through trial and error until a version fits?

\n", "Title": "Best way to make a new model for custom part printing?", "Tags": "|3d-models|rapid-prototyping|", "Answer": "

Use a CAD software

\n

Whenever you need to make a part fit given dimensions, it is best to set those. There are tons of Computer Aided Design software packages around, starting at free and ending at thousands a year for a single PC license. There are so many, that Wikipedia made a comparison list

\n

all3Dp always curates a list of free 3D software. Among them, I can point to the following as options:

\n\n

Not on the curated list of all3dp is one entry I personally worked with and which is somewhat potent but easy to use:

\n\n" }, { "Id": "19363", "CreationDate": "2022-05-10T00:29:00.807", "Body": "

Should I be able to hand turn the stepper motor for the extruder of an Ender 3?

\n

Trying to figure out why the motor isn\u2019t turning on a new to me, never used, but out-of-warranty Ender 3.

\n

Swapping controller cables I discovered the extruder port on the motherboard is dead, but even if I put it on the X axis and manually move the axis it makes the sound like it should move, but doesn\u2019t actually move at all.

\n

Trying to figure out if it\u2019s seized or something. Doing a resistance test with a multimeter shows a resistance of 4 for either of the two pairs of wires. I am not sure what else to test.

\n

I have a hard time believing I need a new control board and a new stepper motor, but maybe two things are broke.

\n

Thanks for the help!

\n
\n

EDIT - Got the Extruder Working

\n

After the comments here mentioned that "Yes, it should be able to be moved by hand", curiosity got the better of me and I said, "Well, if its broken, let's see why".

\n

I did the following:

\n
    \n
  1. I tried to turn the stepper motor by hand again, just to confirm I wasn't crazy from the day before when I tried it. It wouldn't budge.
  2. \n
  3. I removed all four screws on the bottom of the stepper motor and attempted to pry things apart. While fiddling with it, I thought I saw the stepper motor turn.
  4. \n
  5. Sure enough, I now tried to spin the stepper motor and it moved relatively easily.
  6. \n
  7. I put the 4 screws back in place and validated I could still hand turn the motor. I could in fact do so.
  8. \n
  9. I hooked the stepper motor back up to the X axis controller and told it to move, and sure enough now it moves and works!
  10. \n
  11. Just as a sanity check, I then hooked it up to the extruder controller and it again wouldn't turn.
  12. \n
\n

I'm going to try what @towe recommended to make sure the controller board is in fact fried, but I think I might JUST have a fried board and not a fried motor.

\n", "Title": "Ender 3: Extruder Stepper Motor - Hand Turn", "Tags": "|creality-ender-3|troubleshooting|extruder|motor|", "Answer": "

Criggie's answer is basically correct, but I disagree with the conclusion that it:

\n
\n

Could be expensive - you might want to compare cost of parts with cost of a new printer, remembering there may be other non-functional components still undiscovered.

\n
\n

If you want to turn the Ender 3 into a decent printer, the controller board is one of the components you want to replace anyway, since it comes with either (old models) extremely loud and poorly performing A4988 stepper drivers or (newer models) TMC2208 stepper drivers hard-wired in a mode where they don't work well and malfunction if you enable Linear Advance (which is critical to getting decent prints on a bowden extruder system). Good boardsthat are exact fits for the housing and cable connectors, with TMC2209 steppers that lack the above problems, can be had for $35 or so.

\n

If the motor is dead, that's a pain but not expensive to replace. Equivalent motors are available for $15 or so all over the place, or you could make the upgrade to a light-weight geared direct drive extruder with pancake stepper instead of the large NEMA 17 (which negates pretty much all of the disadvantages of direct drive and gives you a much better printer than you started with).

\n" }, { "Id": "19367", "CreationDate": "2022-05-10T15:31:32.807", "Body": "

I need to check the belts on my Ender 3 to confirm they are properly set up, functioning, and undamaged. How do I go about doing this?

\n", "Title": "How do I check the belts on my Ender 3?", "Tags": "|creality-ender-3|maintenance|", "Answer": "

Loosen the screws holding the far right hand side of the belt cog in place on the gantry, tighten the belt by moving the cog outwards until the belt looks horizontal, tighten the screws back up and test. It's a pretty intuitive procedure.

\n

To see if the belt is OK, just visually inspect it. It's just a belt, if it has splits or something then it has issues.

\n" }, { "Id": "19378", "CreationDate": "2022-05-12T15:32:38.177", "Body": "

Blender software has a nice mesh analysis add-on called \u201d3D Print Toolbox". It is making us give it a second look for our 3D printing workflow.

\n

On the other hand, MeshLab is very nice to have mesh repair tools.

\n

I see the errors in the Blender, and then I close the program. And then I open MeshLab and make the corrections. The goal is to produce the model with a 3D printer.

\n

Is there a possibility to do the analysis in MeshLab as well? How?

\n", "Title": "Blender add-on \"3D Print Toolbox\" and MeshLab", "Tags": "|3d-models|blender|", "Answer": "

Meshlab has a bunch of tools for that under 'Filters', you'd need to read the documentation for specifics as your needs may vary from model to model.

\n

But it's much the same as the Blender addon with the 'Cleaning and Repairing' options to merge vertices, close holes etc,.

\n" }, { "Id": "19390", "CreationDate": "2022-05-17T08:14:10.560", "Body": "

I'm using an Ender 3 v2 with Jyers firmware.

\n

I'm looking for a firmware / OctoPrint plugin which allows browsing G-codes stored in my OctoPrint storage and starting them directly from my printer's screen like it's stored on SD card. I would like to have all advantages that OctoPrint provides but not need to use my computer/smartphone to start printing.

\n

Is there any way to do this?

\n", "Title": "Browsing and printing print files (G-code) stored in OctoPrint from printer's screen", "Tags": "|creality-ender-3|octoprint|jyers|", "Answer": "

Accessing the print files stored on OctoPrint managed locations other than the SD card (i.e. where OctoPrint is installed; RPi or laptop, etc.) is not possible.

\n

The other way around is possible, you can access SD stored codes from OctoPrint. This is possible because the there are M-codes in place to list the files on the SD-card (M20: List SD card) or handle files to load and start them.

\n

To access files from the printer UI to an external storage space would require many information on where it can get the files, through which connection; there are no M-codes in place to do that. Basically you are either printing from OctoPrint or from the printer. You state that: I would like to have all advantages that OctoPrint provides but not need to use my computer/smartphone to start printing, how would that be possible if the printer itself initiates the print? OctoPrint is a print manager, it sends your G-code line by line to the printer, you are requesting to start a print from the print manager through a command on the printer itself. If you want OctoPrint managed prints to benefit from the plug-ins, you need to start the print using your phone or a browser. Personally, I never look or use the printer display on my OctoPrint managed printers, you don't need the display if OctoPrint is able to present all the data to you through a browser.

\n" }, { "Id": "19409", "CreationDate": "2022-05-21T10:35:36.947", "Body": "

I recently had a print failure/crash, where the print stuck to the nozzle and forced molten filament back into the print head, fans, and heater block.

\n

I changed the nozzle for a new 0.4mm same as existing. I shortened the bowden tube by ~8mm to remove some crispiness, and the push-lock connector on top of the print head, which was full of solidified PLA.

\n

The hotend was scraped clean of PLA, and the wiring was inspected. The silicon sock was unhappy but I managed to get it to stay in place.

\n

The part-cooling fan duct was deformed, but I have reshaped it as well as I could. The part-cooling air is probably slightly less than it was.

\n

\"enter

\n

I'm printing some Gridfinity bins, and the base just isn't filling in completely and there is also more stringiness.

\n

The sides are not joining up at all, and are just a series of separate strands. They do merge somewhat at the corners.

\n

\"enter

\n

My printer worked much better before the crash - what do I have to focus on to improve this?

\n

All print jobs since the reassembly are lower in quality, with one in three showing these large "wire bundles" look but all of them are not as good as before-prints.

\n", "Title": "What's causing this lack of layer adhesion after a crash?", "Tags": "|creality-ender-3|print-quality|stringing|gridfinity|", "Answer": "

SOLVED!

\n

\"enter

\n

Turns out I hadn't pushed the bowden tube completely home into the hotend on the print head. So there was a space where molten plastic was spreading out, interfering with both extrusion and retraction.

\n

I've flipped the bowden tube end-for-end now, and my test job is well-underway.

\n" }, { "Id": "19414", "CreationDate": "2022-05-21T23:13:51.640", "Body": "

I have a 1.5 inch screen, and I would like to create an adjustable angle plate for the screen, similar to Game boy advanced (shown below)

\n

Does anyone know how the mechanism works?\nBy any chance, is there an adjustable connector that I can put on a 3D printed part to make an adjustable angle plate?

\n

\"enter

\n", "Title": "Adjustable Angle for a 1.5 inch OLED screen", "Tags": "|3d-design|", "Answer": "

You'll need to design a hinge in two parts, probably with a third and fourth part as a cover.

\n

The unit holds position by friction, so it will get sloppier with use as the plastic wears, or you may want to use a metal shaft and bushings. This will also be affected by the mass of the screen and the torque onto the hinge.

\n

From your image, you can clearly see which hoops in the hinge are part of the top and which are coming up from the bottom.

\n

The ribbon cable will pass between the lower and upper in the middle where the two wide hoops are, and not through the ends which will provide the bulk of the support.

\n
\n

If you want to make a case exactly like this, then consider buying a shell instead. The ifixit step-by-step instructions at https://www.ifixit.com/Guide/Nintendo+Game+Boy+Advance+SP++Shell+Replacement/137582 are clear and helpful for a complete organ-transplant.

\n

The hinge/clutch mechanism used in these is a marvel, you're unlikely to be able to print one without a lot of trial & error, and looks like:

\n

\"enter\n
From\nhttps://console5.com/store/game-boy-advance-sp-hinge-replacement-case-hinge.html

\n

However people have designed GBA cases - search your favourite STL websites for relevant links. You may be able to find a case someone else has already designed and done debug.

\n

Potentially useful tool https://3dmixers.com/m/275635-gba-sp-hinge-removal-tool-with-handle

\n" }, { "Id": "19428", "CreationDate": "2022-05-24T09:33:41.747", "Body": "

Printer: Elegoo Mars 3 LCD Resin Printer

\n

I have a similar problem to this question, First / bottom layers not printing? and this issue on Reddit, Elegoo Saturn Squishing Bottom Layers Together - Models Too Short.

\n

I am trying to print an object that is a cylinder with an engraving on it (the cylinder is not very high though, - it looks like the cylinder shown in this question on SE.Blender, How can I engrave this Text on the side of this cylinder (without changing the style of the font)?):

\n

\"Similar

\n

The cylinder is about 3.6\u00a0mm high only. It has an engraving on the side of it.

\n

I haven't printed with supports but directly on the built plate (similar to the rook model Elegoo gives). The problem is that the first layers are not printed properly (it cuts off the engraving). This results in a reduction in the height of the cylinder.

\n

Any solutions for how the first layers could come out properly?

\n", "Title": "Resin LCD print not printing first layers properly, resulting in a reduction of the height of the object", "Tags": "|troubleshooting|z-axis|resin|", "Answer": "

This is a known issue with printing directly on the build plate. The first few layers have to be cured for much longer than the rest of the layers in order to ensure that the model adheres properly to the build plate. This causes the layers to be thinner but wider. Often resulting in a slight brim around your model (Called an Elephant's foot in some circles).

\n

Normally you would use a skate or raft and supports, so the distortion wouldn't be to the model itself.

\n

You could reduce the number of bottom layers in your slicer, and reduce the exposure time, which will give you much closer to your expected height, but it may require a lot of trial and error printing to find the right settings for your printer. I think that some people have made calculators to help with this, or you could try to find a torture calibration model to help you find your optimum settings.

\n

Personally, as a person who make models for printing, I would recommend that you changed the dimensions of your model, rather than changing your printer settings.

\n

I often make the parts of my models that are in contact with the base slightly thicker, but with a slight bevel to the edge to allow for expansion (Elephant foot) , then I simply sand down the extra after printing till the model is of the desired size.

\n

Again, this is mostly a matter of trial and error.

\n" }, { "Id": "19450", "CreationDate": "2022-05-27T23:36:42.800", "Body": "

I have an Ender 3 Pro with the BTT SKR E3 V2.0 mini with Marlin firmware 2.0.8.2.x.\nI am trying to print PETG, which requires decently high temperatures.

\n

I initially replaced the stock board after a thermal runaway event that seemed to have damaged it. After installing the new board and getting all the settings dialed in (typically 260\u00a0\u00b0C hotend and 90\u00a0\u00b0C bed), it worked great for about 2 weeks until I got the thermal runaway event error again.

\n

Here is what I have tried so far

\n\n

I PID tuned the printer using M303 E0 S260 C10 and stored new PID values in EEPROM + firmware. A note, running this multiple times seemed to constantly increase the P and D values. I stuck with the initial values given (kP 13.97 kI 0.84 kD 57.96). I still continued to get thermal runaway events.

\n

I then tested the heater cartridge and thermistor with my multimeter. The heater was 13.5 ohms which seems about right. I was unable to measure the thermistor value. Searching online shows I likely need a better multimeter to do so. It's possible it is bad, but I find that hard to believe considering this issue was happening prior to my replacing it.

\n

Example log of the failure happening. All I did was heat the printer up, leave it on for a bit, set it to cool down briefly, then tell it to heat up again. The printer was heated for ~5-8 minutes before this log starts.

\n

Could this be the board again, or is there something else I'm missing?

\n", "Title": "Thermal Runaway issues on Ender 3 Pro even after replacing thermistor and heater cartridge", "Tags": "|creality-ender-3|marlin|troubleshooting|thermal-runaway|", "Answer": "

I'm fairly certain I have solved this issue, and it ended up having nothing to do with the printer and everything to do with what it was plugged into!

\n

I had it on a smart outlet with some automations set up to kill the power if there was ever a fire. Unfortunately, the outlet I was using was only rated for 8A, while the Ender 3 Pro can draw up to 15 amps. When it was unable to draw more than 8A to heat the hotend, this likely caused the printer to think there was a problem, triggering the thermal runaway failsafe.

\n

After moving it to an outlet with a higher amperage rating, I have had no more issues.

\n" }, { "Id": "19451", "CreationDate": "2022-05-28T04:32:10.337", "Body": "

I'm new to 3D printing and I was wondering about the risks of leaving my printer to print overnight? I'm aware that if something goes wrong I'll wake up to spaghetti for breakfast but what are the other things like having the nozzle and bed heated that long? And, when it's done it just sits there on so what could that do to the screen? What if the printer runs into some physical problem and damages itself and/or the print? I'm not looking for answers to these questions specifically just feedback on what I should do when printing overnight.

\n", "Title": "What are the risks of leaving to print overnight?", "Tags": "|safety|life-expectancy|", "Answer": "

Assuming the printer is operating correctly, there are no lifetime/wear concerns from running it that long. Many-hours and even -days prints are normal usage, and print-farms run as businesses run these things basically 24/7 for months on end, modulo routine maintenance. Having the heaters on for 8 hours is really no big deal.

\n

Of course, that's assuming the printer is operating correctly. If not, there are various things that can go wrong.

\n

The worst possible is thermal runaway: the printer losing track of the bed or hotend temperature and running the heating element always-on until it melts the metal. This is a severe fire hazard. Printers with properly built firmware have thermal runaway protection to perform an emergency-stop and turn off heaters if they can't detect a reasonable temperature measurement response to operating the heater, but some popular machines have this feature intentionally turned off by the manufacturer. If you will be operating your printer unattended, absolutely make sure you test that thermal runaway protection is present and working (there should be good ways to simulate it and see that the safety is triggered).

\n

Even with thermal runaway protection working, there is a very slight chance the power mosfet controlling a heater fails in a way that leaves it always-on, which the firmware cannot protect you against. For this reason you also want to have the printer away from flammable materials and may even want an active fire suppression system (there are "balls" you can hang above your printer that activate and release fire suppressant if it's on fire). This is a very low probability event - I've never heard of it happening - but I'm including it for completeness.

\n

Now, the less severe stuff. As long as the first few layers go down well, it's unlikely your print will detach from the bed mid-print, but it can end up having (usually small) parts that warp upward and collide with the nozzle. This can cause lots of different types of problems:

\n\n

Most of these things do not impose any major safety risk like fire, but they can do varying degrees of damage to the printer, requiring repair or at least maintenance. For example endstop collisions themselves usually don't damage anything but they might knock your frame out of square/alignment, requiring tinkering before the next print.

\n" }, { "Id": "19461", "CreationDate": "2022-05-29T16:43:41.977", "Body": "

I am a beginner and looking for software to easily pose a 3D character, which I currently have in an FBX file, and export the pose to an OBJ file.

\n

For example, I could move the character's arm upward, then export that as an OBJ and print it.

\n

Every software I know of either can not pose FBX characters (such as Fusion360) or can not export poses (such as Blender). How can I do this easily?

\n", "Title": "Software to Pose and Export FBX Character", "Tags": "|3d-models|3d-design|software|cad|", "Answer": "

If you pose it in Blender you can export it and print it with the pose. It's very simple, you just export as a STL file and then print.

\n" }, { "Id": "19468", "CreationDate": "2022-05-30T21:00:18.440", "Body": "

I've tried printing in vase mode (or "spiralise outer contour" in Cura) and while the floor looks fine, the vertical sides look "saggy"

\n

I'm using a 0.4\u00a0mm nozzle, with eSUN PLA+ at 218\u00a0\u00b0C and a bed temp of 60\u00a0\u00b0C. This combination works fine for normal printing. Layer height is 0.28\u00a0mm (Low Quality mode in Cura) with a line width of 0.4\u00a0mm

\n

\"enter
Image is backlit by a monitor to show the laciness in the walls.

\n

\"enter

\n

The original model was a 1x1x6 Gridfinity bin that uses less plastic than the original.

\n

https://thangs.com/designer/LittleHobbyShop/3d-model/%23gridfinity%20Vase%20Mode%20Single%20Box-65828

\n
\n

Is this insufficient cooling, or too fast a print speed letting the filament sag under gravity before it cools? Or is a 0.4\u00a0mm nozzle too small?

\n

This reminds me of brickwork where the mortar is too wet. The bits that work right look fine, but all four sides have bad parts.

\n

What's the trick to vase mode printing?

\n", "Title": "Saggy vase walls. What are the tricks to successful vase mode printing?", "Tags": "|print-quality|vase-mode|gridfinity|", "Answer": "

The trick was to change a lot of settings and save as a separate set.

\n

What works for "normal" prints does not work well for vase-mode work.

\n

\"enter

\n

I had to:

\n\n

Vase mode also cannot deal well with printing multiple parts at the same time, and if you use "one-at-a-time" mode in your slicer then the max height caps out at 25 mm for me, which is not a lot of use.

\n" }, { "Id": "19476", "CreationDate": "2022-05-31T13:42:53.543", "Body": "

My Ender 3 was printing great. However, I'm now getting all sorts of globbing up on the Extruder nozzle. This is causing the prints to fail, and if I come back after a print I'm often getting 1/3 of the print printed with gaps, etc before it totally fails. Any idea what is going on? Its the same roll of PLA that was printing fine at this temperature, etc; but we are getting down to the last 15% of the roll if that matters.

\n

\"PLA

\n

------------------- EDIT With More Information ---------------

\n

Switched out the PLA and my printer is printing great again! Per the suggestion from someone, I'm currently baking the old PLA and will report back on the results!

\n", "Title": "PLA Globbing up on the Extruder Nozzle", "Tags": "|creality-ender-3|troubleshooting|pla|extruder|", "Answer": "

I would think the filament has absorbed too much moisture. You can bake it and then try again.

\n

I bake 2 hours at 50 degrees which works ok for me. But the first time I did this some of the roll tried to fuse together and the innermost 1/4 was wasted. So I've found that it's best to loosen the filament on the roll before baking if it's not that much. If it's a full roll I actually unwind it onto an empty spool and roll it loosely.

\n" }, { "Id": "19477", "CreationDate": "2022-05-31T14:01:28.520", "Body": "

Now that V6 nozzles are getting more advanced, as in Nickel coated brass/copper nozzles (e.g. Bondtech CHT\u00ae nozzles) the availability for Volcano hot ends seem to lag behind...

\n

A Volcano heat break adapter/nozzle extender has been spotted on the online market places to be used together with the shorter E3D V6 nozzles.

\n

\"Volcano

\n

There seems to be confusion about the mounting of the adapter.

\n

Either from the top of the heater block:

\n

\"Volcano

\n

or, from the bottom:

\n

\"Volcano

\n

What is the correct positioning of the adapter considering a hex key insert is in the path of the filament, e.g. effects on retraction performance?

\n

\"Volcano

\n", "Title": "Volcano hot end adapter orientation?", "Tags": "|nozzle|e3d-v6|e3d-volcano|", "Answer": "

I'm not sure what the intended mounting is, but I would put the hex socket downward towards the nozzle. Especially with a CHT, the opening to the nozzle (3 openings rather) is significantly wider than the normal filament path (usually actually 1.85-2.00 mm), and having the adapter widen just before it should help the filament reach the entire opening smoothly once it's molten and expanded in the space.

\n

Conversely, mounting it with the hex socket pointed upwards creates a point in the melt zone where the path widens then narrows again. Due to the way viscous fluids work, material that gets into this space is very unlikely to merge back with the main flow; it's going to stay there and bake. This could lead to jams or at least increased friction from the molten filament having to pass by carbonized gunk. Even if not, it could leave mismatched color/material filament inside the hotend after a purge, only to have it randomly come out later and ruin your print (or at least make it look ugly).

\n" }, { "Id": "19492", "CreationDate": "2022-06-03T15:17:24.977", "Body": "

I am currently adding a fillet to the base of an object (the plane that's touching the bed) and I was curious if the radius of the filet contributed to any mis-prints. I've had luck so far but was wondering if the intensity of the radius had mattered.

\n

I am using the Ender3 Pro.

\n
\n

I may do some test prints and see for myself and provide an answer to share experience .

\n", "Title": "Can you fillet the base of an object without needing supports? Is there a recommended radius?", "Tags": "|3d-design|", "Answer": "

You can print a fillet without support as support material causes other issues like problems with removing supports and ugly scarring on you print. However, a fillet will cause an overhang when you slice the object

\n

\"enter

\n

and may result into poor results as well, see e.g. this:

\n

\"enter

\n

If your design allows it you should better use a chamfer than a fillet for the base of the print object.

\n

A chamfer prints better than a fillet because a fillet creates an overhang (see indicated area on the left part of the image below.

\n

\"enter\nImage from 3DVerkstan

\n

A chamfer, which normally is a 45\u00b0 straight cut-off, doesn't create an overhang and, as such, prints better. If you still want a fillet, you could start with a chamfer of which you fillet the top, see the right part of the image above.

\n

A chamfer with the height of the first and second layer is generally a good idea to reduce the slightly over \u201csquished\u201d first layer issues that create a lip around the base of the part.

\n" }, { "Id": "19522", "CreationDate": "2022-06-06T16:26:08.540", "Body": "

I am working on a prototype and I have a need for a motor shaft to Lego axle adapter for use with a stepper motor.

\n

My overall project entails controlling a stepper motor using C# via USB connection to make very small rotations which will adjust the height of a specimen stage (constructed from Legos) for a stereo microscope.

\n

I need adapters similar to 6mm D shaft to Lego axis adapter Free 3D print model but for 4\u00a0mm dia motor D shaft and 5\u00a0mm dia motor D shaft

\n

The purpose of the adapter is to allow very small and slow rotations.

\n\n", "Title": "Stepper motor D shaft to Lego axle adapter", "Tags": "|3d-models|3d-design|bricks|", "Answer": "

OpenSCAD would be well suited for creating something made up of relatively simple shapes, where different dimensions are needed for some parts of the shapes - like the diameters and offsets of your stepper motor shafts.

\n

A solution in OpenSCAD could look something like this:

\n
outer_diameter = 8.5;   //Outer diameter of the adapter\nstepper_length = 12;    //Length of the stepper shaft\nstepper_diameter = 4;   //Diameter of the stepper shaft\nstepper_d_offset = 1.6; //Offset from the center of the shaft to the plane of the D\n\n//4mm shaft: d = 4, offset = 1.6\n//5mm shaft: d = 5, offset = 2\n//6mm shaft: d = 6, offset = 2.5\n\nthickness_mid = 2;      //Thickness of the massive section between stepper and lego shafts\n\nlego_length = 10;           //Length of the lego shaft\nlego_diameter = 4.9;        //Outer diameter of the lego shaft\nlego_internal_width = 1.9;    //Width of the slots for the shaft\nlego_corner_radius = 0.5;\n\ncutout_size = lego_diameter;\ncutout_translate = cutout_size / 2 + lego_internal_width / 2;\n$fn = 128;               //Accuracy / resolution of circles\neps = 0.01;\n\nmodule fillet_square(width, radius) {\n    translate([radius - width / 2, radius - width / 2, 0])\n        minkowski() {\n            square(width - 2 * radius);\n            circle(radius);\n        }\n}\ncolor(0,0.5)\nunion(){\n    linear_extrude(height = stepper_length + eps) {\n        difference() {\n            circle(d = outer_diameter);   \n            difference() {\n                circle(d = stepper_diameter);\n                translate([0, stepper_d_offset + stepper_diameter / 2, 0]) {\n                    square(size = stepper_diameter, center = true);\n                }\n            }\n        }\n    }\n    translate([0, 0, stepper_length]) {        \n        linear_extrude(height = thickness_mid) {\n            circle(d = outer_diameter);\n        }\n    }\n    translate([0, 0, stepper_length + thickness_mid - eps]) {        \n        linear_extrude(height = lego_length + eps) {\n            difference() {\n                circle(d = outer_diameter);\n                difference() {\n                    circle(d = lego_diameter);\n                    translate([cutout_translate, cutout_translate, 0]) {\n                        fillet_square(cutout_size, lego_corner_radius);\n                    }\n                    translate([cutout_translate, -cutout_translate, 0]) {\n                        fillet_square(cutout_size, lego_corner_radius);\n                    }\n                    translate([-cutout_translate, cutout_translate, 0]) {\n                        fillet_square(cutout_size, lego_corner_radius);\n                    }\n                    translate([-cutout_translate, -cutout_translate, 0]) {\n                        fillet_square(cutout_size, lego_corner_radius);\n                    }\n                }\n            }\n        }\n    }\n}\n
\n

\"enter

\n

You can then export your .stl file (or any other format) for 3D-printing from OpenSCAD.

\n" }, { "Id": "19532", "CreationDate": "2022-06-09T16:27:35.160", "Body": "

What are the meanings of the red, green, and blue light prompts on the Ender 6 motherboard? My Ender 6 is stuck at the boot screen. I followed the instructions provided in the "Creality After-Sale" videos on YouTube and took apart the base of the printer to look if any wiring was loose.

\n

I found that the red light was continuously on at the D2 position on the motherboard. Creality's video showed the light on this location of the motherboard to be blue. I also checked on the internet and it appears some users also have a green light. Users also report the light blinks but in my case, it remains red throughout.

\n

I am wondering if this indicates what is wrong with my printer and why it is stuck on the boot screen.

\n", "Title": "Meaning of light prompts on Ender 6 motherboard", "Tags": "|repair|creality-ender-6|", "Answer": "

I found out that the motherboard had gotten damaged and the green light indication had to do with showcasing this. The problem was resolved when I bought a new motherboard online and replaced the damaged motherboard with the new one. The printer works fine now.

\n" }, { "Id": "19536", "CreationDate": "2022-06-11T10:26:53.657", "Body": "

I have radial fan with 2 pins connector and always kept it connected to GND/5V pins (marked J5 at the left bottom on pinout image for my controller board).

\n

I've stumbled upon the USE_CONTROLLER_FAN feature of Marlin which allows setting some pin to be used for PWM-controlling a control board fan. I think SERVO0_PIN should do fine (D11 at the bottom center on the pinout image), but I'm not sure how to connect it.

\n
    \n
  1. Should I split connector and use GND + D11 pins?
  2. \n
  3. Or should I use some proper PWM fan for that (which are always 4pin and then how would one connect THAT to those pins?)
  4. \n
  5. Is it safe to run a fan directly from board pins or should I resort to using either unused MOSFET outputs (e.g. FAN MOSFET pin marked as D9 on the left)?
  6. \n
  7. Should I use dedicated MOSFET board to drive that fan using that SERVO0_PIN?
  8. \n
\n

\"MKS

\n", "Title": "Connecting controller fan on RAMPS 1.4 for Marlin", "Tags": "|marlin|wiring|fans|", "Answer": "

Servo pins are PWM pins, so yes D11 can be used, but not directly connected to the fan as the pins only allow a very low current. You'll need a MOSFET to drive the fan. You don't need 4-pin fans, 2-pin fans will suffice.

\n" }, { "Id": "19551", "CreationDate": "2022-06-13T03:06:18.897", "Body": "

The printer is a Prusa I3. I'm running it directly from a PC with Repetier 2.2.4. Repetier has a soft Emergency\nStop but that requires grabbing the mouse and getting the pointer to the hot spot on the screen. I'd be more comfortable with a physical button.

\n

If all else fails I can rig a panic button to connect a 30 ohm resistor between hot and ground downstream of the GFI, causing it to trip. That seems a bit extreme. Also not sure if the capacitors in the power supply might keep a motor going for another few steps after the mains power goes away. Interrupting the stepper motor power just ahead of the motor drivers seems like the optimal way to go.

\n", "Title": "Where/how can I connect a physical Emergency Stop panic button directly to a RAMPS 1.4 board to quickly stop all stepper movement?", "Tags": "|ramps-1.4|", "Answer": "

Just pull the main power on the machine if it bombs and goes haywire. It isn\u2019t a computer that needs a soft shutdown.

\n

The only thing you want to do is turn it back on quickly so the hotend fan goes back on and you don\u2019t get heat creep with filament melting and then solidifying in the heat break.

\n

Disconnecting motor power isn\u2019t great, because the reason you are hitting the panic button is probably because the machine bombed is trying to wreck itself, in which case the processor needs to be reset. Could make a DC disconnect between the power supply and main board, but it should stay off for a few seconds for the caps to drain, so if it\u2019s a momentary switch all users would have to know to hold it down for a spell. If it is an on/off switch you have a possible source of confusion, why the printer isn\u2019t turning on, because now there are two power switches, and you wouldn\u2019t want to use it as a general use power switch because the power supply would be left on all the time.

\n

One alternative would be a reset button on the processor, if there is a breakout for that pin or a tact switch on the pcb you could wire a big, official momentary switch (normally open type) in parallel with. I would suggest some kind of shroud so it doesn\u2019t get bumped accidentally.

\n" }, { "Id": "19559", "CreationDate": "2022-06-15T15:11:28.510", "Body": "

How can I replace the line in the time display with a dot? Using Marlin 2.1

\n

\"Photo

\n", "Title": "How to change the time format on LCD screen in Marlin 2.1?", "Tags": "|marlin|firmware|", "Answer": "

The apostrophe or single quote character is officially used to indicate minutes, if there are two, this indicates seconds (but, in the image from the question, the double quote cannot be seen).

\n

Replacing the single quote for a decimal isn't a good idea to display the time as the following number is in seconds, not a fraction of a minute.

\n

Either way, the remaining time is converted from a number into a human readable format by the toDigital() function of the duration_t.h file (referring to the Marlin 2.1 bugfix branch). Even with little programming skills if is easy to find where and what you want to change ( hint, see comment: // 12'34), the function is quoted for reference below:

\n
uint8_t toDigital(char *buffer, bool with_days=false) const {\n    const uint16_t h = uint16_t(this->hour()),\n                   m = uint16_t(this->minute() % 60UL);\n    if (with_days) {\n      const uint16_t d = this->day();\n      sprintf_P(buffer, PSTR("%hud %02hu:%02hu"), d, h % 24, m);  // 1d 23:45\n      return d >= 10 ? 9 : 8;\n    }\n    else if (!h) {\n      const uint16_t s = uint16_t(this->second() % 60UL);\n      sprintf_P(buffer, PSTR("%02hu'%02hu"), m, s);     // 12'34\n      return 5;\n    }\n    else if (h < 100) {\n      sprintf_P(buffer, PSTR("%02hu:%02hu"), h, m);     // 12:34\n      return 5;\n    }\n    else {\n      sprintf_P(buffer, PSTR("%hu:%02hu"), h, m);       // 123:45\n      return 6;\n    }\n  }\n
\n" }, { "Id": "19568", "CreationDate": "2022-06-19T12:29:22.180", "Body": "

I have this .stl that I downloaded and I need to create a solid of this 3D. However, if I click "Make solid", it doesn't work.

\n

How can I do? I think the problem is the black layer but I don't know how to resolve.

\n

Before make solid:

\n

\"Screenshot

\n

After make solid:

\n

\"Screenshot

\n

I need to make solid and the black layer is deleted when I do it.

\n

https://drive.google.com/file/d/1g-YaF1k_X5FU1Y-SgWiGCXC4k_nADohk/view?usp=sharing

\n

PS: The other object are different item, so if they are there or not is the same

\n", "Title": "Meshmixer create solid", "Tags": "|3d-models|3d-design|meshmixer|", "Answer": "

Make solid requires an enclosed volume

\n

To run the make solid operation, the selected parts need to enclose a volume. In case the volume is not fully enclosed, the program tries to solve a solution that closes the open surface.

\n

The black layer is most likely failing to compute because its normals are flipped. This means it does not enclose a surface, it excludes anything between the surfaces from being inside the body defined by it - it is everything but. This is solved as "this surface does not enclose anything, so I cut it out" but for where it creates a valid solution in the area of the white surface before the operation.

\n

This leaves you with the white retained part after the operation.

\n

To fix this is an in-depth project

\n

Fixing such errors is quite involved. You will need to do the following steps, depending on your program to alter:

\n\n" }, { "Id": "19617", "CreationDate": "2022-07-02T05:08:49.173", "Body": "

I bought an Ender 3 recently and it has the 4.2.2 Creality board. Then I bought a BLTouch to upgrade it with bed leveling. It turns out that the BLTouch now comes with a standard 5-pin connector instead of the separate wires that all the tutorials talk about. How do I install the thing?

\n", "Title": "Configuring Marlin 2.x with Ender 3 4.2.2 and BLTouch", "Tags": "|creality-ender-3|marlin|firmware|bltouch|", "Answer": "

Don't forget to add

\n
M420 S0\n
\n

after the G28 command in the G-code, else you have to level bed before every print.

\n" }, { "Id": "19619", "CreationDate": "2022-07-02T15:44:16.010", "Body": "

After printing some parts in TPU, I encountered continuous clogging after returning to PLA. Not right out of the bat, but after about an hour or such, all prints I started since the swap back clog after about an hour. I use an Ender-3, Bowden Style, and print my PLA generally at 200\u00a0\u00b0C. The TPU had been listed as 220-240\u00a0\u00b0C on the roll, so I printed at 230\u00a0\u00b0C.

\n

How can I regain normal printing behavior?!

\n", "Title": "My printer clogs after Printing TPU! How can I fix it back up?", "Tags": "|troubleshooting|pla|tpu|", "Answer": "

What's the cause of the problem?

\n

The problem is the dissimilar printing temperatures:

\n\n

As a result, when the PLA is molten and well printable already, residue of TPU in the hotend is at an awkward spot: it is molten enough to seep down along the filament path with molten PLA, but it is not soft enough to get easily extruded from the nozzle. This is what leads to clogging.

\n

Problem solution

\n

To fix the clogging, I took the following steps after the very first time I encountered it:

\n\n

There you go! One restored printing behavior!

\n

Technically, the nozzle swap and cold pull were overkill, but reduced the amount of TPU that needed to be purged out of the nozzle.

\n

Refined problem solution

\n

Since the problem occurred first, I managed to refine my procedure to prevent clogs in the first place.

\n\n

While the extruder pushes the PLA through at an elevated temperature, it clears the whole path while cooling down, and can cycle right into the next print. It can help to use a different color PLA than the TPU to have a visible confirmation of the last residue being gone.

\n

Another helpful indicator is the cooling down filament's flexibility: as long as TPU is in the mix, the extruded string is bendy but becomes stiff as soon as there is almost no more residue in it.

\n" }, { "Id": "19657", "CreationDate": "2022-07-16T15:53:11.103", "Body": "

Lately I've been having a lot of trouble with printing overhangs in TPU. The failures are very geometry/toolpath dependent, entirely reproducible in their location (same gcode gives same failures each time it's printed), and seem to occur where there is a convex (model-inward) curve over an overhang of more than 40\u00b0 or so. My original test case for this was frog model with more severe overhang, but I since made a simplified conical overhang test piece at only 50\u00b0 that's faster to reprint and shows the issue:

\n

\"Two

\n

The frog print on the left is pretty much a complete failure. The one on the right is mostly a success, but shows some moderate flaws around one side (left, as viewed) and the front of the belly, and even slight flaws (hard to see in pic) around the other side of the belly. It was done using some of the mitigations described below.

\n

\"3

\n

The test piece on the left is nearly perfect. The other two show varying degrees of the overhang problem I'm experiencing, and make it easier to see what exactly is happening than with the frog. The extrusion seems to bunch up, then get stretched out too thin, in an oscillating pattern that builds up and shifts with each layer.

\n

I've tried printing overhangs significantly slower and reducing speed and acceleration quite a bit on the outer walls and even on the inner walls too, and none of that seems to help. Nor does increasing the number of walls or the wall line thickness help. However a number of things do seem to help, and it takes two or more of these in combination to get a mostly acceptable result at 0.2 mm layer height:

\n\n

Generally I'm able to choose some subset of the above that works, but it ends up being a matter of per-model trial-and-error, wasting lots of time and materials if the settings don't work, so I'd like to figure out what's really going on here so I can make predictions about what will work and ideally get a base configuration that "always" works. Also, I still don't have this working at thinner layer heights, which I'd like to be able to use for better detail, as TPU is flow-bound not motion-bound and I could in theory print much higher detail at the same speed with thinner layers.

\n

My best guess at the root cause so far is that the overhanging walls simply do not have enough rigidity to avoid being displaced by the toolhead attempting to extrude against them, so any oscillation of the toolhead velocity or extrusion pressure causes them to deform in the pattern of the oscillation. Does this seem plausible, and if so, what might some other possible mitigations be?

\n

For completeness, my printer is a heavily modified Ender 3 with (remote) direct drive extruder and fairly extreme cooling, but turning the cooling way down or even off (assuming sufficient layer time for passive cooling) does not seem to affect the behavior here significantly. So I think the question is mostly printer-agnostic and is really a matter of material behavior and slicing.

\n", "Title": "Root cause fix for TPU overhang failures", "Tags": "|print-quality|filament|print-material|tpu|", "Answer": "

I fixed my overhangs by increasing the nozzle temp. You need to heat it up enough that the elasticity is lost which makes it misbehave on overhangs and curl up.

\n" }, { "Id": "19666", "CreationDate": "2022-07-17T17:08:52.663", "Body": "

I've just bought my first 3D printer (Malyan M200 V2). For the most part, it's been really good and I've had no issues, apart from when printing the first few layers the printer doesn't seem to extrude enough material and doesn't form the correct shape. Whether it's a circle, rectangle, or anything else. So for example I've printed the below part.

\n

The raft prints perfectly:

\n

\"Photo

\n

Then when it starts the first few layers of the actual print it extrudes a bit of material which then hangs from the nozzle and is dragged about the surface of the raft before stopping as more material is extruded. So when the print finishes the first layer looks messy like the one below:

\n

\"Photo

\n

But everything after the first few layers is perfect for example the top of the same print:

\n

\"Photo

\n

I've tried adjusting the temperature of the nozzle and the print bed neither has made a difference. The bed is level I've double-checked that. Trying to find the issue online keeps bringing me back to temperature or bed levelling.

\n

I'm using this PrimaValue PLA Filament

\n

The leaflet in the box recommends printing at 210\u00a0\u00b0C, I've tried 210, 215, and 220\u00a0\u00b0C.

\n

The print bed I've been keeping at 60\u00a0\u00b0C which seems to have been working but I've tried printing down to 45\u00a0\u00b0C on the print bed and the same issue occurs. I'm not sure what an ideal temperature is for PLA, I've seen some posts saying that a heated bed isn't needed for PLA and others saying that it should be heated to between 50-70\u00a0\u00b0C (my printer only heats up to 60\u00a0\u00b0C).

\n

I've looked at some of the settings in Cura and there are various settings for things like initial speed and first/last layer speed which sound like they might help but I don't know anything about them. I have tried slowing down the print but again I just watched it happen more slowly, albeit that did help some as that's when I've realised it doesn't appear to be pushing out enough material on the first few layers. Any help or suggestions would be greatly appreciated.

\n", "Title": "First layers going funny", "Tags": "|print-quality|", "Answer": "

Hi Thanks for all the suggestions. I've been experimenting with various prints using some of the different settings and it appears that I needed to increase the initial layer height. Now the first layers are printing as they should.

\n" }, { "Id": "19671", "CreationDate": "2022-07-18T07:41:50.733", "Body": "

I've created a little enclosure for a project in sketchup\n\"enter

\n

I then exported the .STL

\n

When I open it in Creality slicer 4.8 or Cura 5.0 It looks like this\n\"enter

\n

I thought that the red was "overhang that needs support" (but can also mean a shell?)

\n

Hoever, that's not the irritating part. That would be the grey in the middle where the "hole" should be.

\n

When I slice it, it looks like this:\n\"enter

\n

I didn't notice this before I started printing and 4 hours into the print I noticed that it was just completely ignoring the window and was considering it as a part of the base.

\n

Why is it doing this and how can I fix it?

\n

I've tried editing the original sketchup model a number of times, but I keep getting the same result.

\n

I read the question and answer at Empty space in model is getting filled but I'm not 100% sure that this is the solution I'm looking for.

\n

The slicer, knows enough to see that the hole should be there, as it is rendering in a shaded area. It feels like there is a setting or some such that is "print the shaded area" with a checkbox (on or off) - and if that setting is there, someone please tell me where to find it!

\n

I've also tried importing the STL into other programs (like fusion 360) and RE-exporting the STL, but the issue persists.

\n
\n

Please note that this question seems to be similar to mine, however, there isn't an accepted answer. The one provided below is actually a much better answer.

\n", "Title": "Disparity between Sketchup STL and Slicer", "Tags": "|sketchup|", "Answer": "

Use Solid Inspector to fix problems

\n

Before exporting your Sketchup model to STL use Solid Inspector. It is available for free from the Sketchup extensions page and will detect reversed faces, stray edges, surface borders. The "main problems" mentioned above are then eliminated.

\n" }, { "Id": "19673", "CreationDate": "2022-07-19T01:20:44.100", "Body": "

The print is a collapsible sword I'm using as a test. Everything seems fine on the outside, but inside it seems the segment parts of the sword inside the hilt are fusing together at the seam lines causing them to stick together.

\n

Model: https://thangs.com/designer/3dprintingworld/3d-model/Collapsing%20Katana-22696

\n

Finished Print (looks fine)\n\"enter

\n

Concentric blade pieces (fuse marks that I sheared off and broke to get out)\n\"enter

\n

Seams on exterior of hilt look great\n\"enter

\n

Print settings/info

\n

Printer: Prusa MK3S+

\n

Slicer: PrusaSlicer 2.5.0a3

\n

Settings preset: 0.15\u00a0mm QUALITY

\n

Filament: Prusa PLA

\n

Nozzle temp: 205\u00a0\u00b0C (default is 215\u00a0\u00b0C, but lower eases stringing and has never been a problem for many past prints)

\n

Nozzle size: 0.4\u00a0mm

\n

Full config: https://pastebin.com/ECa6KkYK

\n", "Title": "Why are these concentric parts fusing together at seam lines?", "Tags": "|prusaslicer|stringing|", "Answer": "

After some trial and error I found that the issue was stringing due to excess moisture in the filament from being stored outside of a sealed low-humidity container for long periods.

\n

After placing it in a heated dehydrator for 2 days, my next print had low stringing and did not bond interlaced parts together significantly.

\n" }, { "Id": "19689", "CreationDate": "2022-07-22T02:19:35.210", "Body": "

Are there any safety risks inherent to PLA plastics used for 3D printing?

\n

The material safety data sheet of some PLA plastics indicates low risks at a toxicological level, but I'd like to make sure some other factor isn't overlooked.\n(1, 2, 3)

\n
\n
\nSECTION 11: TOXICOLOGICAL INFORMATION \n
\nPRINCIPLE ROUTES OF EXPOSURE: Eye contact, Skin contact, Inhalation, Ingestion. \nACUTE TOXICITY: None noted during use. \n

LOCAL EFFECTS: Product dust may be irritating to eyes, skin and\nrespiratory system. Particles, like other inert materials, are\nmechanically irritating to eyes. Ingestion may cause gastrointestinal\nirritation, nausea, vomiting and diarrhea.

\n

SPECIFIC EFFECTS: May cause skin irritation and/or dermatitis.\nIngestion may cause gastrointestinal irritation, nausea, vomiting and\ndiarrhea. Inhalation of dust may cause shortness of breath, tightness\nof the chest, a sore throat and cough. Burning produces irritant\nfumes.

\n

CHRNOIC TOXICITY: None noted during use.

\n

REPRODUCTIVE TOXICITY: No data is available on the product itself.
\nCARCINOGENIC EFFECTS: None of the components of this product are\nlisted as carcinogens by IARC, NTP, or OSHA.

\n
\n", "Title": "Are there any major safety risks of PLA plastic?", "Tags": "|pla|filament|safety|", "Answer": "

If you are concerned about inhalation (and I think you should be), you should use a hierarchy of controls to mitigate the risk.

\n

NIOSH (part of the CDC) have a good document outlining how to mitigate the risks of 3D printing particulate emissions.

\n

Note that the smell you experience may be VOCs, not particles. Both are important to block. To do so, use a respirator with both particulate and organic vapour filters, in addition to an air purification/local exhaust ventilation system. The \u2018carbon filters\u2019 often seen in 3D printers do almost nothing for particulates, only nuisance levels of vapours.

\n" }, { "Id": "19712", "CreationDate": "2022-07-31T18:55:04.073", "Body": "

I'm quite new to 3D CAD and printing.\nI own a Dremel 3D45 and I use FreeCad / Ultimaker Cura as softwares.

\n

My question is pretty simple.\nSay you have to make one object with a pin and another with a hole. They should be coupled together. Of course if you set the diameters of the pin and the hole equal the won't fit!

\n

Right now I'm setting the hole larger of 0.2 mm and the pin smaller of 0.2 mm. This allow a quite good coupling (not so hard but with some resistance).

\n

I guess this tolerance (0.4 mm in my example) depends on a lot of variables: 3D printer settings, material, etc... so it may change using different setup.

\n

How to correctly handle this?

\n

Should I add a variable in my CAD spreadsheet and use it to change the nominal diameter of the coupling items?

\n

I don't think so, but anyway: is there a settings in Ultimaker Cura that allow to compensate an hole or a pin by a specified amount?

\n

Any other suggestion is gladly accepted.

\n", "Title": "How to take in account tolerances when coupling", "Tags": "|ultimaker-cura|3d-design|cad|freecad|", "Answer": "

Once you understand how the parts will need to fit together to meet their purpose, you will need to define allowances on your parts in order to create clearance between them.

\n

You will need to understand your printer's capabilities and accuracy by printing some test parts and measuring them. Tolerance is the amount of variation from the specified dimension that is acceptable on a part.

\n

If your printer isn't accurate enough to achieve the tolerances specified on the part, you'll have to find some way to improve the parts so that they are within tolerance. Often this is done as a finishing step: filing, sanding, or grinding an oversized printed part; drilling out an undersized hole; etc. You should also consider other options: buy a more accurate resin printer, redesign the part to make the pin out of a commercially available metal rod or tube, pay someone else to make the parts for you, etc.

\n

Armed with this information you can "design for manufacturing". That means you alter the design of your parts enough so they can be successfully produced with the tooling available to you.

\n

I just posted a little Q&A that discusses this very topic of the difference between allowances and tolerances, and ways to achieve that.

\n" }, { "Id": "19717", "CreationDate": "2022-08-02T08:22:09.037", "Body": "

I'm using Ultimaker Cura and a Dremel 3D45S with an eSun Nylon filament.\nAs starting point I'm using the default settings of the printer, 260\u00a0\u00b0C for the hotend and 80\u00a0\u00b0C for the bed.

\n

These are the results:

\n

\"enter\n\"enter\n\"enter

\n

In addition to the poor print quality, the resulting dimensions are wrong. For example, the cylinder is supposed to have a wall thickness of 5\u00a0mm, with an inner diameter of 35\u00a0mm and the outer diameter of 45 mm. With ABS they are quite good (just a 0.2\u00a0mm difference). With nylon they are off by 1\u00a0mm! The wall has a thickness of 6\u00a0mm.

\n

For both materials the flow is set to 100\u00a0%

\n

Any ideas on what I should change to improve the printing?

\n

Here my current settings for Nylon:

\n

\"enter

\n", "Title": "How to fix these issues with Nylon?", "Tags": "|ultimaker-cura|nylon|dremel-3d45|", "Answer": "

If you look up the hardness of Nylon with respect to ABS you will find that ABS is generally much harder. The effect of softer filament is that the teeth of the extruder feeder dig in more into the filament (so for every rotation of the gear, less material is extruded than in hard filament), if you do not correct for this (lower the extrusion flow modifier) you are under-extruding material.

\n

This is clearly not happening in your case!

\n

The effects you see are caused by over-extrusion. This can cause the effects you see on the outside of your print and also result in dimensional inaccuracies. You need to tune the printer for different materials. Try to extrude 100\u00a0mm of filament and see how much is being extruded (mark some filament from a reference point and redo the measurement after extrusion).

\n

Also make sure the filament is dry, Nylon tends to take up moisture which can bubbling (boiling the moisture) during extrusion.

\n" }, { "Id": "19719", "CreationDate": "2022-08-02T12:01:16.207", "Body": "

I have an Artillery Hornet printer that I am very satisfied with. I make my models in Blender, export them as STL files, and import them into Cura slicer. Everything works great with no problems, mostly. But yesterday, I made a model that I used a reference image in Blender (deleted it before exporting then). And when I printed it, direct afterward, the printer unloads the filament. I tried to print it multiple times and every time it did the same. No big problem then, but a bit annoying.

\n

Does anyone have any idea what could cause this, and how to prevent it? Other models I make in Blender have not given me that result then. I still have not tried making other models with a reference image to compare the result. So I found this place first and try to ask if someone knows about that problem.

\n

This is the end G-code:

\n
G1 E-6 F9000\nM104 S0 T0 ; turn off temperature\nM140 S0 ; turn off bed\nG1 X110 Y220 F10000\nM84 ; disable motors\n
\n", "Title": "My printer unloads the filament, after I finished the print", "Tags": "|filament|", "Answer": "

I haven't used your printer type, but this link suggests:

\n
\n

If your printer unloads the filament after each printing session\nautomatically, you might want to check the end code of your machine\nand disable the retraction (;G1 E-6 F9000).

\n
\n" }, { "Id": "19721", "CreationDate": "2022-08-03T16:06:35.867", "Body": "

A couple of days ago I got my first 3D printer: Creality 3D Ender 3 Pro. I finished assembling it last night. I booted it up, but ran into problems (which I guess is not a common thing for a beginner in 3D printing). After booting and running the motors, it seemed that the lead screw got stuck about halfway down the Z-axis. I heard a rattling sound. So I turned the 3D-printer off. I disassembled the lead screw and applied some Lithium lubricant on it. But the problem persisted. Looking more closely at it, I noticed some notches on the lead screw:

\n

I tried to spin the Creality Z-axis stepper motor, and it felt smooth and did not have any resistance. The bolts on the rod holder were slighly loose so that the lead screw had a bit of play inside.

\n

Im wondering if the lead screw may have been defect during manufacturing process? Are lead screws supposed to have notches like these, or are these manufacturing defects? Here's two photos of the lead screw:

\n

\n

Im guessing the notches are too deep for the z-axis to actually work the way it is supposed to.

\n", "Title": "Assembled my first 3D-printer: Lead screw problem?", "Tags": "|creality-ender-3|z-axis|", "Answer": "

Both of the above answers are likely correct but also, I own an Ender 3 and it's kinda/sorta tricky to get it set up right. I had a similar problem and I was certain it was the lead screw -- and mine indeed does not have the marks that yours has -- but in the end the issue was that I had assembled the printer ever-so-slightly out of true, and I had to do a lot of adjusting and wobbling to get things lined up right. So just be prepared for that.

\n

Yes I think the rod is damaged and send back for a replacement, but when it comes back be sure and triple-check that everything is adjusted properly following videos like this. You might also need to spend a little bit more money to replace some of the stock parts to really get it dialed in but once you do you won't be disappointed!

\n" }, { "Id": "19733", "CreationDate": "2022-08-06T05:22:49.700", "Body": "

I upgraded my Ender 3 Pro from the stock 8-bit motherboard to a new 32-bit 4.2.7 motherboard. No problems were encountered with the swap, flashing of the new firmware, or calibration.

\n

When I went to start the first test print, the firmware throws the E1 Error: "Heating Failed: E1; PRINTER HALTED; Please Reset".

\n

When the print started, the bed heated up to the requested 60\u00a0\u00b0C, and then started warming the print head to 210\u00a0\u00b0C. Once the print head reached 205\u00a0\u00b0C, the temperature started to drop. It dropped back down to 200\u00a0\u00b0C, then up to 202\u00a0\u00b0C then down to 198\u00a0\u00b0C and so on. It kept doing the up and down heating until it dropped to 190\u00a0\u00b0C at which point the E1 error was displayed. I have run through the cycle several times with different prints, and through the menu options; and it always fails in the same up and down way.

\n

Printer Specs:

\n\n

Prior to the upgrade, everything worked fine. I printed several small prints the day before without a problem. After the upgrade the E1 error.

\n

Thoughts? Suggestions?

\n", "Title": "Heating Failed: E1 after upgrade to 4.2.7 motherboard - Ender 3 Pro", "Tags": "|creality-ender-3|marlin|bltouch|", "Answer": "

Problem Solved

\n

After checking in with the Ender 3 group on Facebook and doing quite a bit more research I tracked the problem down to PID calibration. The default settings for the board were not heating the print head properly. Because it was coming in short, the thermal runaway logic on the new board cut in and shut everything down.

\n

To fix the problem, the printer needs to run a PID autotune. This is a good idea when you replace any part between the nozzle and the logic board. To accomplish the autotune you need to access the printers console through an app such as Pronterface. You should also take care when hooking in a USB cable to the printer. The 4.2.7 board runs power to the USB port. To get the printer connected to your computer, you will need to mask off the power pin on the USB cable (with the possibility of blowing the chip on the board if you skip this step)

\n

The Marlin command for PID autotune is M303. Follow this up with an M301 to set the results from the autotune. And don't forget to finish up with an M500 to save the settings to the EEPROM.

\n

I would suggest this video from Teaching Tech for an example of the process, and this video from 3D Print General to conduct the same autotune on the build plate.

\n

Post Script

\n

Important Safety Tip - The 4.2.7 board draws more power than the old 8 bit board. During my first test print after the PID calibration, the connector between the power supply and the motherboard melted down. It seems that the wires were crimped rather than soldered. This was fine with the old board, but it was too much with the added power draw. The poor connection caused the wires in the connection to over heat. This is a fire hazard.

\n

Check the power connector. remove the shrink wrap and take a close look. If you don't see solder on the joint, replace it. It is worth the slight expense and hassle to avoid ruining your printer and potentially starting a fire.

\n

The issues is discussed in this Makers Muse video.

\n" }, { "Id": "19740", "CreationDate": "2022-08-08T03:33:13.100", "Body": "

I'm printing a ring that's a replacement for the non-slip base of a mixing bowl. The ring is about 130mm in diameter, with a rectangular cross section, like this:

\n

\"rubber

\n

I'm using Cura as the slicer, and I've set the infill to 100% and concentric, but after slicing it looks like Cura used lines instead; the ring is filled with parallel straight lines:

\n

\"lines

\n

Is this a problem with Cura? Is there something I can do to encourage it to use concentric infill? I don't really care what the infill pattern is, but I think concentric would print a lot faster since the head wouldn't have to switch directions all the time.

\n", "Title": "Cura uses lines instead of concentric infill", "Tags": "|ultimaker-cura|infill|", "Answer": "

This is a known issue. Cura's profile variable logic sets the number of bottom layers to 999999 if infill is set to 100%, overriding infill by replacing it with additional bottom layers. If you go find the setting for number of bottom layers and set it back to the number you actually want, overriding this, infill should work as expected.

\n

Alternatively, setting top/bottom pattern to concentric should also fix it, and you probably want that anyway so that you don't have distinct bottom layers that are printed as lines.

\n" }, { "Id": "19747", "CreationDate": "2022-08-09T15:00:01.930", "Body": "

I have to have a pin fit in a hole, and there needs to be a gap so the pin can turn freely. Can I make the pin smaller by changing the tolerance?

\n", "Title": "What is the difference between allowance and tolerance?", "Tags": "|3d-design|terminology|", "Answer": "

There are four terms that many people find confusing:

\n\n

Use tolerance when specifying the amount of error permitted in making a part. Use allowance when specifying a gap between two mating parts.

\n
\n

Two other related terms that are important when specifying dimensions for manufacturing:

\n\n

A machine cannot reliably produce parts with a smaller tolerance than its accuracy. If elephant foot expands your parts by 0.1\u00a0mm, then your accuracy is only 0.1\u00a0mm. You will not be able to repeatably print parts with 0.05\u00a0mm tolerance. (You might get lucky and one out of ten might accidentally fit the dimensions required, but that's not a good manufacturing plan.)

\n

Precision is often confused with accuracy. People think that if their machine is advertised as having 0.05\u00a0mm step size (its precision is 0.05\u00a0mm) that they can print parts with 0.05\u00a0mm tolerance. But if your printer has a precision of 0.05mm and an accuracy of 0.1\u00a0mm, you cannot count on it to repeatedly position itself to within 0.05\u00a0mm of the desired dimension.

\n

To measure your machine's accuracy, you'll need to print some pins and holes and carefully measure the differences between what you defined and what you printed. The difference between the largest and smallest measurements you take is the accuracy. And be sure to check the accuracy in your X, Y, and Z dimensions; your printer might have a difference between them that would impact the roundness of the parts.

\n
\n

In the case of the sizes needed to fit a pin in a hole so that it can pivot freely, you need to define an allowance in order to create the clearance you desire.

\n

What is the minimum gap between parts you are looking for, and what is the maximum you can accept? That's the clearance.

\n

Let's say you want a clearance of at least 0.2\u00a0mm between the pin and hole, but no more than 1.0\u00a0mm. And let's say you measured your printer's accuracy to be \u00b1\u00a00.2\u00a0mm. If you print a 5\u00a0mm pin, your pin would be anywhere between 5.0\u00a0mm\u00a0\u00b1\u00a00.2\u00a0mm, so the hole must therefore be 5.6\u00a0mm \u00b1\u00a00.2\u00a0mm. The minimum clearance of 0.2\u00a0mm would be an minimum sized hole (5.4\u00a0mm) and a maximum sized pin (5.2\u00a0mm); the maximum clearance of 1.0\u00a0mm would be a maximum sized hole (5.8\u00a0mm) and a minimum sized pin (4.8\u00a0mm).

\n

Note that a clearance of 1.0\u00a0mm is really loose, and is just too sloppy for your application. You might think to tighten the tolerances to 0.05\u00a0mm in order to reduce the clearance. But since your printer's accuracy can't produce a part that meets your specified tolerances, you would need to find a different way to manufacture or "finish" the parts.

\n

The traditional way to solve the problem of inaccuracy in production is to create the part larger than the maximum material condition, then use a subtractive method to finish it to the desired dimensions.

\n

Say we need the pin and hole to have 0.2\u00a0mm clearance, but we've already established that our machine only has 0.2\u00a0mm of accuracy. How do we print the parts to fit? We print the hole undersized and the pin oversized by the amount of accuracy in our machine, plus we include an allowance to ensure we always have some material to remove in the finishing step.

\n

Let's establish the hole's final dimension to be 5.0\u00a0mm\u00a0\u00b1\u00a00.05, so we print the hole to 4.7\u00a0mm\u00a0\u00b1\u00a00.2\u00a0mm (resulting in a hole that's between 4.9\u00a0mm and 4.5\u00a0mm). After printing we run a 5\u00a0mm drill bit through it to finish it to 5.0\u00a0mm\u00a0\u00b1\u00a00.05\u00a0mm.

\n

Then we do a similar operation with the pin. Print it to 5.1\u00a0mm\u00a0\u00b1\u00a00.2\u00a0mm (giving us a pin between 5.3\u00a0mm and 4.9\u00a0mm), then chuck it in a lathe or drill and carefully sand or file it until it becomes 4.8\u00a0mm\u00a0\u00b1\u00a00.05. Now we finally have achieved our clearance of 0.2\u00a0mm\u00a0\u00b1\u00a00.1\u00a0mm, which is a good enough fit for our purpose.

\n

These types of secondary finishing steps have been used by craftspeople for hundreds of years.

\n" }, { "Id": "19754", "CreationDate": "2022-08-11T23:19:44.767", "Body": "

Both items were printed at the same time. The item on the right was perfect while the one on the left has crazy thick strings. Sorry I didn't keep the build plate's orientation, you can see how they were positioned in the Cura screen cap.

\n

\"Photo

\n

\"Screenshot

\n

I thought the stringing was from nozzle travel but if that were true the strings would be coming from the center pillar which they are not. Some of the strings shoot out from the left which doesn't make any sense.

\n

I checked the bottom and it seems like the first layer is perfect or pretty damned close to it.

\n

Sovol Sv01 pro

\n\n

Settings

\n\n", "Title": "Thick stringing next to a perfectly printed model", "Tags": "|creality-ender-3|petg|stringing|", "Answer": "

225\u00b0C is way too cool to print PETG, especially at 60 mm/s if your printer's extruder is similar to the Ender 3's stock extruder (going off what you said; I'm not familiar with your specific printer). It will be having serious trouble extruding, slipping in the filament gear, at which point you'll have too little material, so what does get extruded gets stretched out too thin and is under a lot of tension, and since it's not hot enough to bond well with the previous layer and also not thick enough to press well against the previous layer, it gets pulled across a diagonal rather than following the toolhead path.

\n

Drop your speed for PETG to 30 mm/s or lower and increase the temperature to 235\u00b0C at a bare minimum. I would really call 245\u00b0C the minimum for PETG, but that's borderline too hot for the stock PTFE-lined heatbreak and will degrate the PTFE (and arguably offgas harmful fumes, although probably at levels way too low to actually be harmful) over time.

\n" }, { "Id": "19756", "CreationDate": "2022-08-12T13:59:43.077", "Body": "

On my Ender 3 v2 printer I recently and consistently get some knocking.\nThis happens in only two scenarios.

\n

First, it now occurs all the time when printing the initial test strip gets near the top (high Y value), and knocks several times.

\n

Secondly, it occurs if the model (sliced with Cura) has a high Y value (eg: if the model occupies most of the bed).\n(If there is room and I move the model - in Cura - closer to the front there is no knocking.)\nOn the first 10 (or so) layers the printer sometimes knocks when a high Y value is reached and the entire model is thereafter shifted to the front by a few millimeters.

\n

There is a third scenario. At the end of a print the print head is in the middle of the bed and moves up 20 mm then travels directly to the top left corner. At this corner there are 4 or 5 "knocks" (and the nozzle is 20 mm above the bed).

\n

Any suggestions to diagnose/fix this problem will be much appreciated.

\n", "Title": "Why is my Ender 3 v2 knocking & misaligning for high Y?", "Tags": "|creality-ender-3|print-quality|y-axis|", "Answer": "

Embarassingly, I discovered that the cable to the heatbed was sometimes caught between the on/off switch and the adjacent power plug. So, for high Y values the cable was very tight and the bed could not be moved. Presumably the "knocking" came from the Y-axis motor. The problem was fixed by attaching this cable to the adjacent hotend/X axis motor cable.

\n

Hopefully this will be helpful to others who have a similar problem. Please add a comment if you experienced this.

\n" }, { "Id": "19762", "CreationDate": "2022-08-13T17:54:16.610", "Body": "

I have an Ender 3 V2 which is around 6 months old. Recently, extruding slowed to an almost complete halt seconds into a print for no apparent reason. The extruder works perfectly when extruding manually (using the extrude setting) and I see no reason for it to not extrude properly. I am using Ultimaker Cura and a new filament (my old one has the same issue). The print speed is 10mm/s and the temperature is correct for the filament which is PLA+.

\n

After some testing it seems random when it stops extruding but it always happens within the first 10 seconds of the print starting.

\n", "Title": "Why Does My 3D Printer Stop Extruding PLA?", "Tags": "|creality-ender-3|ultimaker-cura|extrusion|underextrusion|print-failure|", "Answer": "

If it only extrudes the priming line, then nothing comes out for the model, this sounds like you have your slicer configured for extremely low or no flow, or wrong filament diameter. Check that any flow settings are at or near 100% and that the filament diameter in the machine, extruder, and material settings is correct (1.75 mm for most printers).

\n" }, { "Id": "19768", "CreationDate": "2022-08-15T11:26:15.457", "Body": "

I have 12 parts for a model I want to print but I would like to know if I can put all of them in a single G-code file and print that on its own. Would this affect the model in any way?

\n

I\u2019m using PLA on my Ender 3 Pro

\n", "Title": "Can I print multiple parts in a single G-code file?", "Tags": "|g-code|", "Answer": "
\n

I have 12 parts for a model I want to print but I would like to know if I can put all of them in a single G-code file and print that on its own.

\n
\n

You certainly can. The printer doesn't care how many parts there are. Many single parts, like those with holes, will have layers that have areas that aren't contiguous. To the printer, multiple parts look just like a single part that happens not to be connected.

\n

That said, printing multiple parts at once means that the job will be larger and take longer, and a problem printing any of those parts can force you to stop the whole job. Because small parts have less area in contact with the bed, small parts are more likely to come loose from the bed during the print, so running a job with many small parts can be risky -- if any one part comes loose, you might lose all the time and material you put into the whole job.

\n

One tool that can help mitigate that risk is the Cancel Objects plugin for OctoPrint. If you use OctoPrint to manage your printer, you can use the plugin to stop further work on any objects that have problems during the print and continue with the rest. Here's a video about using Cancel Objects.

\n

Also, when printing multiple parts, be sure to check that you have enough material (filament, resin, etc) available to complete the whole job.

\n" }, { "Id": "19784", "CreationDate": "2022-08-18T17:38:44.113", "Body": "

I recently purchased a spool of PETG to try working with it. I have managed to dial in most of the settings in Prusaslicer but one, in particular, is giving me a problem. As seen in the photo, the clip I printed has extra extrusion on the inside and outside. I have noticed that the nozzle will pause at the seam for about 5 secs before continuing. (The bottom is not Elephant's foot, I just didn't clean off all the brim)

\n

\"Photo

\n

I positioned the seam on the inside of the model. I know that the extra extrusion is caused by the seam but why would it also appear on the outside of the model?

\n

\"Screenshot

\n

I have printed the same clip in PLA without any printing errors. What setting within Prusaslicer needs to change so I can get rid of the extra plastic on the inside and outside of the print?

\n

[I don't know what relevant print settings are needed to solve this problem, but will edit the question when I get some guidance.]

\n", "Title": "Extra extrusion at seam of print with PETG", "Tags": "|print-quality|petg|prusaslicer|", "Answer": "

After checking several places online, I finally got an answer in a Discord chat.

\n

The solution was to turn off the Power-loss recovery setting on the printer itself.

\n

After that was done, the print came out beautifully.

\n" }, { "Id": "19789", "CreationDate": "2022-08-19T11:56:47.827", "Body": "

I did my homework reading similar questions, like this, this and this.\nHere a video that shows the issue:

\n

\r\n \r\n

\n

The filament is PETG from JAYO and the printer is a Dremel 3D45.\nAs you can see, the filament does not stick on the buildplate. The manufacturer suggests to use 220-250 \u00b0C for the nozzle and 70-80 \u00b0C for the bed.\nHere what I tried so far:

\n\n

Honestly I don't know what to do further.\nFrom your experience what should I do to avoid what you see in the video?

\n", "Title": "No way to stick PETG on buildplate", "Tags": "|adhesion|petg|build-plate|dremel-3d45|", "Answer": "

I agree with fred_dot_u, you are too far away from the bed. If you can't get it after that, I would suggest using Bed Weld by Layerneer.

\n

\"Product

\n" }, { "Id": "19839", "CreationDate": "2022-08-29T17:50:08.960", "Body": "

When I'm printing sometimes there is some "transparency" between the layers as you can see in the picture, I'm not sure how to name the problem.

\n

\"enter

\n

I've looked on google, somes people said It could be the nozzle that is clogged, I cleaned it and it doesn't seem clogged.

\n

I've also seen that it could be the pressure of the spring in the extruder arm, I adjusted it.

\n

I also checked the bed level and it looks fine.

\n

With all thoses changes et verifications I still have the problem, not even a small improve.

\n

Some time ago I didn't have any problem, I didn't change the cura parameters, and the problem is for every 3D files, not only this one.

\n

In your experience what could cause this ? Did I do something wrong ?

\n

I have a artillery sidewinder x1 which I use together with Cura. I print in PLA at 200 \u00b0C. The print bed is set to 50 \u00b0C . I use a print cooling fan at 100 %. The layer height I set to 0.16 mm, the line width 0.44 from the 0.4 mm nozzle. The Printing Speed is set to 30 mm/s for walls and 60 mm/s for infill. My retraction is 5 mm at 45 mm/s.

\n

EDIT 30/08/2022:

\n

I have tried to print the gcode given by artillery (which is the manufacturer of the 3d printer), there is two files a cube and a chick, I have the same problem except that it's only the infill.

\n

\"enter

\n

This file is supposed to be perfect for the printer so to my understanding the problem is not from the software setting in cura, but from the printer itself.

\n", "Title": "Why is there some transparency between the layers of my print?", "Tags": "|print-quality|artillery-sidewinder-x1|", "Answer": "

I found out what was the problem here, like @0scar said I think it was the tension spring that was wrongly set, even if I had checked it before.

\n

Also the real name of the problem was "underextrusion" and not "transparency between the layers"

\n

Here is the steps I followed to fix it:

\n

I disassembled the extruder by removing the screw on the side like on the picture, there is also two behind the FAN.

\n

\"enter

\n

After that I had access to every gear that was pulling the fillament.

\n

I cleaned everything, there was some dust but not a big quantity so I don't think I was the problem.

\n

I also removed the spring and the spring handle. I change the tension of the spring to be as low as possible. Then I reassembled every parts.

\n

After that I have redone the bed level and the spring tension.

\n

Compared to the first time that I change the spring tension, this time the gear to change it felt easier, there was less resistance than last time. Even so I changed the tension to the lowest possible each time.

\n

After doing all that I printed the manufacturer print file, It was ok, there was no longer under extrusion. Then I tried multiple file of mine, which didn't print well before all that, this time it was perfect.

\n

I think the real problem here was that some time ago I have disassembled the extruder and when I reassembled it I must have badly re-assembled it. Especially the spring part, since the feeling was different.

\n" }, { "Id": "19866", "CreationDate": "2022-09-06T21:30:15.640", "Body": "

Photos exist where the orange controller of Prusa printers is shown.

\n

In most photos, it looks as if the surface is somewhat rough.

\n

Here is such a photo:

\n

\"enter

\n

It sparkles a bit, so I would assume that it's not perfectly even.

\n

I do not own such a controller or Prusa printer, so I would like to ask if somebody could show a close up of what this surface really looks like and perhaps give me some information about how it could be re-created.

\n

Thank you very much!

\n", "Title": "How is the Prusa typical orange surface created?", "Tags": "|prusa-i3|print-material|surface|", "Answer": "

That texture you see is from the build surface, all prints you see are printed with the plane you see downwards.

\n

There are several options to create such a surface finish. From texture coated heated beds to magnetic flexible build surfaces.

\n" }, { "Id": "19868", "CreationDate": "2022-09-06T22:36:18.933", "Body": "

I have an Anycubic Kobra Max. It has a bed size of L430*W410.

\n

I would like to print a rough surface as shown here:

\n

\"Photo

\n

To do that, I need a coated PEI build plate, and I need to print the surface pointing towards the build plate.

\n

The coated PEI build plate that is shown in the photo was designed for an Ender 5.

\n

I would like to know if somebody could point me to such a solution for a Kobra Max as I don't know what would be needed to "fix" it on the build plate.

\n

Here is the full data for the product shown in the image just for clarification of what it shows:

\n\n", "Title": "Rough surface coated PEI textured build surface to fit Anycubic Kobra Max?", "Tags": "|build-surface|anycubic-kobra-max|", "Answer": "

Considering that the PEI coated build surface is smaller than your actual build platform size you should be able to fit the build surface onto the build platform at the expense of a smaller print surface area. In the past I have secured sheets of glass with Kapton tape (very thin high temperature resistant electronics tape), but glass is stiff and rigid. You might be able to use tape to tape the build surface corners to the build platform. However, from the website: supplied without magnetic platform side B, the plates are coated on both sides so you can save one side for flawless parts while you need the other side. The performance may not be good without the soft magnetic base to maintain flatness. Do not recommend using them separately., so you need to get a magnetic platform to use this surface according to the seller.

\n

This makes sense, as the sheet is thin, and some filaments warp (by shrinkage) considerably, the sheet may deform. You could try to use double sided high temperature tape, but that defies the the purpose of the build surface, you need to get it off easy and bend the surface to release the print. Therefore, a magnetic base is required.

\n" }, { "Id": "19870", "CreationDate": "2022-09-07T07:44:12.830", "Body": "

The Anycubic Kobra Max comes with a glass plate.

\n

I need to use a textured PEI plate for my Kobra Max.

\n

As I understand it, the PEI plate needs to be used on a magnetic bed plate.

\n

Currently, there is no hotbed with a spring steel magnetic plate available for it yet.

\n

I am still a complete newbie at plates.

\n

Could I use an other magnetic plate for this, or do they have to specific to a certain printer (apart from the size of course)?

\n", "Title": "Are magnetic hotbed plates interchangable?", "Tags": "|heated-bed|build-surface|anycubic-kobra-max|", "Answer": "

Usually the build surface is not the magnetic part, it is just a flexible steel PEI coated plate, so yes you could use a magnetic build platform sourced somewhere else.

\n
\n

Currently, there is no hotbed with a spring steel magnetic plate available for it yet.

\n
\n

Actually there are solutions, note that complete build plate and surface systems are sold for the size of your printer, e.g. look into manufacturer BuildTak.

\n" }, { "Id": "19881", "CreationDate": "2022-09-08T09:44:14.710", "Body": "

I would like to print a custom version of something akin to this rugged case that was originally created using injection molding:

\n

\"ea

\n

The outside consists of a material that is a bit softer than the main body.

\n

It is used to protect the electronics against drops when the case falls onto the floor.

\n

Unfortunately, I don't know which material this is, and I don't know which method I could use to measure its softness.

\n

I would therefore like to ask if anybody has experience with such a softer outer hull and can tell me which material could be used when I want to 3D print it.

\n

I would like to use this case in a hospital environment.

\n", "Title": "Which material can be used to print a drop protector?", "Tags": "|print-material|filament-choice|", "Answer": "

Case designers usually use TPU for flexibility and polycarbonate for stiffness

\n" }, { "Id": "19890", "CreationDate": "2022-09-09T16:31:57.257", "Body": "

\"enter

\n

When I print the gcode repeatly, under extrusion happens at the same place.

\n

The under extrusion not only occurs on first layer. I noticed the second layer also has some part under extruded, but not at the same place as first layer, because the infill line direction is rotated 90 degrees.

\n

I have tried the following but could not solve the problem:

\n
    \n
  1. Delta calibration (leveling)
  2. \n
  3. z=0 gap adjustment
  4. \n
  5. Filament flow calibration
  6. \n
\n

It's a delta printer running Marlin 2.1. Slice with SuperSlicer.

\n

The top layer doesn't seem to have under extrusion. There is even a bit over extrusion at the corner.

\n

\"enter

\n", "Title": "What could be the reason of under extrusion on some areas?", "Tags": "|print-quality|troubleshooting|delta|underextrusion|", "Answer": "

low-spots in the bed

\n

If the bed is lower in some areas, the line is not squished there and more rounded. As a result, a line appears to be "under extruded"

\n

High-spots in the bed

\n

In other areas, the bed might be too high and block the nozzle - the line looks under extruded because it is too thin.

\n

Uneven filament

\n

When filament is uneven and below nominal diameter, those areas create a real under extrusion in the area when that area is fed.

\n" }, { "Id": "19895", "CreationDate": "2022-09-10T14:45:15.957", "Body": "

It is difficult to describe with words what's happening, so take a look at the picture. Somehow PrusaSlicer decides to move and print in the air (blue line), where instead it can continue going from outwards to inward. I understand it wants to print first the outer layer but in this case it is obvious it will not hold that layer.

\n

The final position is shown in the second image.

\n

\"prusa

\n

The final step of that layer:\n\"enter

\n

Any idea how to configure PrusaSlicer to avoid this situation?

\n", "Title": "How to configure PrusaSlicer to prevent printing in the air", "Tags": "|prusaslicer|", "Answer": "
\n

How to configure PrusaSlicer to prevent printing in the air

\n
\n

You need either support or change the print orientation.

\n

You cannot print into the air, you can however print lines on top of a prior layer with a certain overlap, or bridge a certain gap. E.g. (screw) holes are often covered with 2 layers over the screw hole so that you use bridging to print (the cutout shows that the hole has a bottom):

\n

\"enter\n\"enter

\n

After printing you need to punch through the layers to open the hole.

\n

Starting and stopping extruding in thin air will result in deposition of the extruded material on the location the nozzle crosses a prior layer (often seen when support structure pillars have fallen over).

\n

What is used frequently together with support in Ultimaker Cura is the option called Enable support roof, this will create a dense top layer on top of the support interface which allows for pretty good bottom surface of the overhang.

\n

This e.g. results in the following slice:

\n

\"enter

\n

When applied to a recently printed flanged tube you see that the bottom surface is pretty good (note the concentric lined top surface, first image shows the product with support, second image, on the left a view of the bottom above support layer and right the support with roof layer):

\n

\"enter\"enter

\n
\n

How to configure PrusaSlicer to prevent printing in the air

\n
\n

In PrusaSlicer you should be able to something pretty similar with Interface pattern spacing set to zero:

\n

\"enter

\n" }, { "Id": "19896", "CreationDate": "2022-09-10T15:02:32.360", "Body": "

I got my first 3D printer, a Creality Ender 3 S1.\nI followed the Quick Installation Guide for the initial configuration. I also had to adjust the corners of the bed manually (the very first print ended up as spaghetti).

\n

Now I wanted to try the first print, so I decided to print the "rabbit" that was included as a G-code with the SD card for the printer. I used the filament that was included with the printer.

\n

The problem is that after half of the rabbit body is printed, the nozzle moves the rabbit, and the rest of the print ends like a spaghetti mess and I have to cancel it. This has happened three times now and I don't understand how to fix the problem or what causes it.

\n

Why does the print of the "rabbit" fail mid-way up?\nAre there any settings I should adjust? How should I figure out what settings I need?

\n

I used the G-code that was already stored on the SD card, with default settings after adjusting my bed.

\n", "Title": "Ender 3 S1 fails printing mid ways", "Tags": "|creality-ender-3|", "Answer": "

I have now got a print of the "rabbit" working.

\n

I did a few things.

\n\n

These alone did not solve the problem.\nThen I did these adjustments:

\n\n

After these adjustments, I finally successfully printed the rabbit.

\n" }, { "Id": "19898", "CreationDate": "2022-09-10T19:29:50.233", "Body": "

I searched about the subject in this website and could only find this question here asking "if there is an FDM Resin 3D printer", not how to build it.

\n
\n

The idea is to replace the inks for other fluids, such as resin and its hardener (and other fluids) and then add a Z axis "somehow".

\n
\n

The thing is: I couldn't find any tutorial on the subject.

\n

The closest thing I could find was this project here called Oasis 3DP, an open source powder and inkjet 3D printer, but the objects are incredibly fragile.

\n

The other type was this DIY that converts a creality 3D into a ink printer, but it uses homemade nozzles actuated by piezoelectrics instead of a inkjet printer nozzle, and the results aren't as good as a conventional inkjet printers (not to insult the creator this is hella cool still).

\n", "Title": "How to convert a conventional Inkjet printer into a FDM resin 3D printer?", "Tags": "|diy-3d-printer|resin|fdm|", "Answer": "

Sounds like you are asking for a kit. There isn't one, and there isn't likely to ever be one, because conventional resin printing and FDM printing use completely different approaches to take advantage of respective material properties, and combining them is likely to get you the worst features of both rather than something better.

\n

Additionally, ink jet printing and things based on it inherently get you brittle parts, and attempts to combine this with FDM methods won't fix that.

\n

Without a kit, you are designing this from scratch. This basically means you either need to be an engineer that can design solutions for all the problems up front, or you will be doing a lot of trial and error with repeated redesigns as you discover new problems -- not that an engineer wouldn't have the same issue without a lot of foresight.

\n" }, { "Id": "19907", "CreationDate": "2022-09-11T21:53:43.643", "Body": "

I have an Ender 3V2 running a custom version of Jyers Marlin, and using a CR Touch as a Z-stop, and with a heated glass print bed. This is my first printer, and it mostly prints fine except when printing near the edges where some imperfections with bed leveling mess with bed adhesion and have definitely ruined a couple of my prints. The main issue is that the first layer is very uneven, where in some places filament it basically smeared onto the bed whereas in other places the filament is barely touching the bed. What's stranger still is that generally speaking, the infill seems to be more consistent in terms of z-height on the first layer, whereas the walls have much more variation. I've included some photos to illustrate the issue:

\n

\"\"\nThis is the entire leveling test I printed (it only tests the edges because the center is fine). This was run without UBL - I verified this as the Z axis motor didn't move during the print (it's only 1 layer).

\n

\"enter\nThis is what has been causing me the most issues. In the top right corner of the print, the wall filament basically disappears for about 1-2 inches because the nozzle is too close to the bed. This is pretty close to the top right leveling screw as well. You can also see that only the walls are affected, as the infill is pretty even across the same area.

\n

\"enter\nKind of the opposite issue in the bottom right compared to the top right, where the walls are printed too high off of the bed and start to string from poor adhesion. Once again, the infill seems to be ok (except in the corner).

\n

Things I've tried:

\n\n

Given that this is my first 3D printer it's really kind of put a damper on the experience for me as even though 75% of the things I print are small enough to avoid the problem area, I want to be able to use all of the print bed without issue. Any help, troubleshooting, or advice would be greatly appreciated!

\n

Edit 1: Took off the X/Z carriages and found two abnormalities - not sure if these are causing the issue. The nozzle carriage has a bit of space between the bottom of the X-axis profile and the roller. It doesn't really wiggle during printing, but it could if I touched it; right now, the two rollers on top sit nicely on the profile. Also, these rollers all have a slightly discoloured ring where a little bit of material has been shaved off of the roller. The Z-axis rollers on the side opposite the lead screw are the opposite case, where the rollers have to be forced onto the structure since it's so tight.

\n", "Title": "Inconsistent Z-height on Ender 3 V2 print bed", "Tags": "|creality-ender-3|bed-leveling|jyers|", "Answer": "

Just for anyone stumbling across this thread now: as it turns out, the wheels on the x-carriage (print head) were installed at a slight angle and not screwed in as tight as they were supposed to be, which probably led to some inconsistencies in printing.

\n" }, { "Id": "19913", "CreationDate": "2022-09-13T05:20:51.723", "Body": "

I'm struggling to find out an enclosed 3D printer that allow to load the common 1 kg filament bobbins. Most of the enclosed ones accept only proprietary bobbins (like Dremel or Flashforge).

\n

Do these bobbins ( = 1 kg) rely on a specific standard I can search for?\nHow to filter out the printers that match my request when making a Google search?

\n

I'm aware I can "easily" put them out of the machine using a custom support (or even directly in a dryer as I do for my Dremel) but this partially nullifies the advantage of the enclosure.

\n", "Title": "How to find enclosed 3D printer that can host 1 kg filament bobbins?", "Tags": "|filament|desktop-printer|enclosure|", "Answer": "

There are standards for spools/bobbins/reels

\n

Looking for standards for spools, I started at Alibaba and then came to an actual European manufacturer or retailer of reels: H\u00e4fner. They manufacture reels for wires, which are also used for 3D printers. They helpfully provide a chart of their products from about 250\u00a0cm\u00b3 to a whooping 98454\u00a0cm\u00b3 - which is about 250 grams of ABS to in theory 98 kilos. However, typically the largest spool you will find for privates is the 300/20 K type with about 3800\u00a0cm\u00b3, which is more than enough to get a 4-kilo reel of PLA onto it. At times you might find something in the order of 8000 to 10000\u00a0cm\u00b3 for a rather heavy 8-kilo spool, though that is industrial sizing.

\n

As this single offerer shows, spools are very varied, and the hub diameter of them is not standardized, but the boreholes only come in very distinct sizes.

\n

However, some of the items that are sold as 3D filament spools are actually welding wire spools, for which standards exist, such as this dictating "2\u20101/32\u00a0inch" arbor holes.

\n

Scale of spools is dependant on the market

\n

Typical enclosed consumer FDM machines are small.

\n

But spools below 1 kilo are not useful for mass-production of large parts and even then, 3 kilos is a sweetspot for handling between ease of moving and time spent changing spools. So industrial machines usually take larger spools - or propriatory cannisters/cartridges with a spool.

\n

One of the big ones is the Stratasys F900. It has a print-volume of 914\u00a0x\u00a0610\u00a0x\u00a0914\u00a0mm and takes "up to 500\u00a0cubic\u00a0inch" cartridges, apparently the largest size of FDM Filament cartridge offered by Stratasys. For the record: 500 cubic inch weigh about 8 kilos in ABS, and about 10 kilos in PLA.

\n

Modification of existing printers

\n

It is almost trivial to modify an existing printer that takes non-proprietary spools to allow taking in filament from the outside without keeping the door closed. A piece of PTFE tube can easily take the position of the spool inside the machine to guide the filament into the machine-mounted intake. The modification might only need a single small hole in the door for the tube or its fitting to get into the machine.

\n

This way you mount pretty much an adapter for larger spools, but you bypass for example automatic filament detection with the spool unless you also open the door and slot in a "disk" of sorts that contains the RFID with the configuration of the mounted spool.

\n

A random example setup, mounting the "outer intake" in an angled block at the center of the former door:

\n

\"an

\n" }, { "Id": "19914", "CreationDate": "2022-09-13T07:30:28.763", "Body": "

I know that a single mixing nozzle with dual filament input can blend them in order to make gradients. I also know that IDEX means you have two independent nozzles and you can make objects with different materials (or colors).

\n

But I don't understand if an IDEX printer can still "mix" (on the layer of course) the two filaments to create a gradient. My common sense says it's not possible, but I didn't find a proof of that.

\n

I'm NOT interested in a mixing nozzle as most of the time I will need two different kind of filaments. The gradient with IDEX is just a "nice-to-have" feature. Not mandatory, but I would know before choose my next printer.

\n", "Title": "Are IDEX printers able to make gradients?", "Tags": "|filament|desktop-printer|dual-nozzle|", "Answer": "

Consider the task from the creation reference. An IDEX printer requires the slicer to direct each nozzle to a specific location of the model while it is being created. No true gradient could be accomplished as it would require both nozzles to be in nearly-instant-extrusion proximity.

\n

I suppose one could create a matrix of colors in a manner similar to that of RGB LED color displays, but that's a severe stretch of the imagination. It would require perhaps a strong understanding of full g-code control to place the individual lines. Additionally, there would be a limitation to two colors and the combinations thereof, although with the bi-color Quantum style filament, results might be interesting.

\n" }, { "Id": "19917", "CreationDate": "2022-09-13T16:08:16.637", "Body": "

I am having some trouble with model adherence to my print bed. As shown in the attached photo, my model is pulled away at the sides leaving it bowed even though it is designed to be flat. Other than this "bowing", the model is of good quality.

\n

The model is printed solid on an Anycubic Photon M3 Plus with Anycubic 3D Printing UV Sensitive Resin. My print settings are:

\n\n

The bed is aligned. Could anyone suggest what is wrong and what I could try to remove this "bowing" effect?

\n

\"Resin

\n", "Title": "How to keep model flat in resin printing?", "Tags": "|print-quality|adhesion|resin|build-plate|", "Answer": "\n" }, { "Id": "19933", "CreationDate": "2022-09-16T20:45:56.450", "Body": "

So far, I have only printed with SLS printers.\nThe results were great, but it was such a horrible mess, and precautions took so much time (gas mask, chemistry gloves, etc.) that I am now switching to FDM.

\n

I would like to ask how I could print the following model:

\n

\"enter

\n

When you look at this from the side, you notice that there is a gap, and I wonder how I could print this. I am not even sure how I could do this with supports.

\n

As you can see, Cura also marks this region red.

\n

Thank you for your any input how I could deal with this.

\n

But I would like to keep this beautiful angle / gap. Having a totally flat would be my last option.

\n

I have uploaded my model here, if you somebody needs it to estimate how to deal with it.

\n

\"enter

\n

This is a screenshot of the support that Cura suggests. But I didn't understand it as I couldn't recognize anything.

\n

\"enter

\n", "Title": "How would I print a non-flat surface using FDM?", "Tags": "|fdm|support-structures|", "Answer": "

The "typical" angle for overhangs that can be printed without support is considered to be 45\u00b0, although some printer/plastic combinations can exceed that value. Your angle appears well beyond 45\u00b0 and is closer to 90\u00b0 which requires support.

\n

I have printed models with similar overhangs and the support material creates an unsightly underside. This can be mitigated with a dual material printer, using water soluble filament for the support. I've recently seen videos and images of dual material prints in which the second material (support) was PETG. In both cases, a decent underside can be expected.

\n

Additionally one could use a method promoted by Angus of Maker's Muse in one of his videos.

\n

I have used this method with quite good results. It effectively converts an overhang to a bridging situation.

\n

In your use case, consider that your model is a rounded rectangle. Create another ordinary rectangular prism slightly larger than the main object, perhaps two to four millimeters larger. Remove a shape from the inside matching the main object, but create it in such a manner as to have an overlap of one to two nozzle widths.

\n

Unfortunately, I see that you have a double overhang, as the initial rim is horizontal and the secondary is angled. This would mean you'd want to create a single nozzle width rounded rectangle at the point of the slope change.

\n

The angled portion is going to require support or a substantially increased amount of added-in "crutches" of similar design.

\n

For something like this, I'd recommend a plane cut at the point where the angled surface meets the vertical surface. Create two objects, perhaps add alignment pins and then glue it together.

\n

If you print it in ABS, you can then acetone smooth the result and make the glue joint disappear, especially if it is glued together with acetone/ABS slurry.

\n

\"sliced

\n

\"sliced

\n

Images created from OP's model, screen captured via Meshmixer.

\n

The resulting internal faces are perfectly flat, although possibly too thin for alignment pegs.

\n" }, { "Id": "19936", "CreationDate": "2022-09-17T00:03:12.903", "Body": "

I am just using Cura for the first time.

\n

This is what my model looks like in Fusion 360:\n\"enter

\n

Here is a close-up look:

\n

\"enter

\n

And this is what Cura creates for me.

\n

I assume the turquoise area is a support structure.\nWhy does it do that?

\n

Also, I don't understand why it marks the edges as red. I assume this means that there is problem.

\n

Thank you!\n\"enter

\n

\"enter

\n", "Title": "Why does Cura create this turquoise support structure?", "Tags": "|ultimaker-cura|", "Answer": "

Each color has a special meaning:

\n\n

And so on ; you can see them if you click on "Color Scheme: Line Type" on the Preview Interface, and select what you want to see.

\n

The turquoise is your support for bed adhesion (see the Build Plate adhesion tree on your Print Settings interface), the red one is your shell.

\n

Nothing is wrong, please read the doc, it could be useful sometimes ...

\n" }, { "Id": "19958", "CreationDate": "2022-09-22T12:42:01.263", "Body": "

I'm new in the 3D priting and I bought a BIQU B1 printer :-)

\n

I printed the Pokemon with the white filament that come as a sample with the printer (PLA) and after that I bought the Inland PLA+ and PETG+ from Microcenter.\nThe first thing that my son asked me to print is the toaster.\nWell, I tried to print three times with the PETG+ filament and always I end up after one or two layers with oozie everywhere and I had to stop printing.\nI replaced the filament with the PLA+ and now it's printing correctly (It's 91% complete right now :-) )

\n

So, I set the correct temperator for both filaments:

\n\n

I'm thinking that for this type of object (torture toaster) it doesn't work with PETG because of the complexity.

\n

Is that correct? If not, what I could be doing wrong with PETG+ filament?

\n", "Title": "Torture toaster with PETG", "Tags": "|petg|biqu-b1|", "Answer": "

Like mentioned, one of the most important things with PETG and other materials such as PLA with woodfill, Nylons, some other materials usually with similar properties to PETG, drying is your friend. The main reason being is that the materials listed above are all hygroscopic. They have a tendency to absorb moisture in the air very well and it should almost be a regular thing to dry it for some ours before printing it. Some materials like Nylon cant be used even after just three days of sitting in open air.

\n

The issue arises when you are printing and the moisture trapped in the filament is super-heated. If you print some and its been out you will get those pops and cracks while its printing, that's the result of the moisture heating and the water bubble exploding making cracks, leaving inconsistent layers and sometimes small craters.

\n

I wouldn't say that the toaster is impossible but its a challenge, as it should be. The premise of the toaster is to challenge your printer and give you and insight into how your printer is working, in terms of your settings. Printing this model will be great to see how well your retraction, bridging, tolerance, and temps are. While I would generally say try it out, it does take some time so going with some easier prints like retraction cubes, temp towers, and simple bridges may be better and easier on filament use when starting off.

\n

If the extruder is slipping, attempt to add a slight more tension but not too much. As mentioned before, it really is a better thing to grab a cheap aluminum kit from Amazon or a hobby store online. The reason being is that the idler will no longer be a smooth roller bearing but instead have teeth as well to help hold on, the aluminum is also much stronger and can hold up in the case you get a jam for any reason.

\n" }, { "Id": "19974", "CreationDate": "2022-09-26T12:17:19.277", "Body": "

I have an HBot 3D 1.1 printer (it's a CoreXY style printer, newer versions are produced by ZMorph). I think that a filament guide tube inside the hotend got damaged, resulting in decreased diameter, which means I can't push the filament through it. It stops halfway through the heatsink (black marker in the attached photo).

\n

I need some help, I'm not sure how to disassemble this type of hotend. With my Ender 3 which I have at home, I can just unscrew the nozzle since it's simply a hexagonal nut, but here it seems that the nozzle and heat block are one part and I don't think I can unscrew the heat block and the heatsink. I'm not sure what to do.

\n

I'm sure the nozzle itself isn't clogged. I've done some cold-pulling on one end, inserted a thin wire from the other, and examined the insides with a flashlight.

\n

\"Photo

\n", "Title": "Changing a teflon tube inside a hotend, help with disassembly", "Tags": "|hotend|nozzle|ptfe-tube|", "Answer": "

This is an old hotend type, it is called a J-Head (see e.g. the J-Head Nozzle Mk V, I'm unsure which exact version you have). The hotend is serviceable, you can buy separate "nozzles" (with integrated heater block) for it in some e-shops. You should be able to unscrew the "nozzle" from the PEEK nozzle holder. The milled flat surfaces indicate that you can use a 13\u00a0mm or 1/2" open-end wrench to disassemble the PEEK nozzle holder.

\n

The "nozzle":

\n

\"enter

\n

The instruction to assemble such a hotend are:

\n
\n

Mk V

\n
    \n
  1. Secure the brass nozzle in a vise by the heater section.
  2. \n
  3. Wrap a couple of turns of PTFE tape (plumbing tape) around the brass threads.
  4. \n
  5. Screw the nozzle holder down onto the nozzle. If no flats are milled, use a pair of pliers to tighten the nozzle. The nozzle holder can be protected from the pliers by first wrapping it with a rag or paper towel. If there are flats milled, a 13 mm (1/2") open-end wrench can be used to tighten the nozzle.
  6. \n
  7. Remove the brass nozzle from the vise.
  8. \n
  9. Slide the PTFE liner down into the nozzle holder. The PTFE liner needs to be inserted such that the flat end is making contact with the brass and the internally tapered end is towards the top.\nInstall the washer.
  10. \n
  11. Screw in the hollow-lock socket set screw. Ensure that the washer stays centered while tightening this set screw. Use a piece of filament to ensure that the set screw is not too tight as the liner can become compressed and obstruct the passage. If this happens, slightly loosen the set screw.
  12. \n
\n
\n

To disassemble you need to reverse the order.

\n

You need to ask yourself it you want to change to a newer type of hotend, but generally, these are higher, e.g. compared to a V6:

\n

\"enter

\n" }, { "Id": "19982", "CreationDate": "2022-09-27T17:15:18.513", "Body": "

Recently, I accidentally shorted out my heater cartridge when trying to do my first nozzle swap on my Prusa MK3S+. I just ordered a new E3D v6 Hotend with 30\u00a0W Heater Cartridge and it will be here in a few days. I just cut the wire from my heater cartridge to my heat block (long story short, I can't remove it from the block). I'm not sure I understand exactly the wattage though. The heater cartridge that came with my Prusa MK3S+ is 40\u00a0W. Does that mean the actual cartridge or the plug into the board? Also, can I just solder the existing 40\u00a0W wires to the new 30\u00a0W cartridge? If so, does it need to be a precise solder (I'm not very good at precise soldering) Finally, can I just wrap the connection in electrical tape? Sorry for so many questions; this is my first time trying to understand the electrical component of 3D Printing. Any help would be appreciated.

\n

I can't replace my individual heater cartridge because the screw was melted in. I checked my mainboard fuses and they are fine. My printer still powers on, all the motors work fine, and even the bed heater works. I cut the wires at the heater cartridge and left them in the air not touching, and now it thinks it's heating up, so I think that I can deduce that the wires were touching the heater cartridge, and I wasn't able to separate them. Unfortunately, I don't have a wire crimper and am trying to keep this fix as simple as possible.

\n

Note:

\n

After using a 30\u00a0W heater on my MK3S+ for a while, I started getting thermal runaway problems somehow, and I have switched over to E3D Revo Six for the safer PTC heating element.

\n\n", "Title": "Using 40W adapter on 30W heater cartridge?", "Tags": "|prusa-i3|hotend|wiring|", "Answer": "

There's a lot going on in this question, and I think you would have done better to ask first before ordering anything how to fix your printer. It's not even clear what part is damaged. A heater cartridge itself can't be damaged by shorting, as that's basically the normal mode of operation for it. However, if you shorted the leads going into it to one another, bypassing the heater, those wires or more likely your mainboard could be toast. Or you might just have broken a wire.

\n

Moreover, it doesn't make sense why you ordered a whole replacement hotend, much less one that's a downgrade for your printer. The E3D V6 is woefully underpowered at 30W. 40W is a bare minimum nowadays.

\n

To answer your specific question points, normally heater cartridges and preassembled hotends come with wires long enough, and proper gauge, to go all the way to the terminals on the mainboard. Some, however, have short wires and some sort of connector. If it has long wires, it's best to just run them all the way to the mainboard as intended rather than splicing. If you do need to splice wire or add a connector, wiring that was made for a higher current will be fine for lower current (as long as it's undamaged). Solder joints are generally not a good idea, though, as they will undergo wear when the wire moves with the toolhead. My understanding is that it's better to use some sort of crimp splice.

\n

And of course, before you do any of this, try to determine whether your controller board is what's damaged. If so, which I think is fairly likely, then you need to either repair or replace it, not the hotend. Connecting a multimeter (in voltage mode) or light bulb of the appropriate voltage to the heater terminal on the board and trying to run the heater would tell you immediately if it's bad (no output) but won't necessarily tell you that it's good.

\n" }, { "Id": "19987", "CreationDate": "2022-09-29T07:43:24.603", "Body": "

I'm facing a weird problem I cannot understand why it happens.\nMost of my objects have a large planar base so I don't have any adhesion issue.\nI'm using a PLA Silk gold filament from eSun (filament dryer is on while printing) and a Dremel 3D45 printer.

\n

This is the bottom (first layer) of an object printed directly on the (glass) bed:

\n

\"enter\n\"enter

\n

Then I tried another object (very similar) using a raft. The raft itself is almost perfect:

\n

\"enter

\n

But, again, the first layer of the actual object (only 0,1 mm air gap from the raft) has the same problems of the first one:

\n

\"enter

\n

The most relevant settings I'm using in Cura are:

\n\n

I enabled the auto-calibration before each print.

\n", "Title": "Why raft is printed much better than the actual first layer?", "Tags": "|pla|rafts|dremel-3d45|", "Answer": "

Based on the scarring and the appearance of the printed holes, it looks as if the nozzle to bed distance is not good, it is probably too large (could be too close as well, if you consider the scarring where the nozzle might have picked up already deposited material). The bed is also quite hot, for PLA you could drop this to 50\u00a0\u00b0C

\n

Note that an initial layer height of 0.3\u00a0mm is also quite large, it causes a lot of material to be deposited while there might be too much/too little room for the amount. Printing on glass should give you a shiny mirror glaze finish (unless the glass is textured); the images don't express that feature from printing on glass build platforms.

\n

A raft generally shouldn't be necessary for PLA (but some exceptions may apply), but certainly not for large planar based prints with a decent amount of surface area in contact with the bed. It is hard to get a nice surface on top of a raft/support structure.

\n

Rafts generally print very well as there is extra space in between the deposited lines, and there usually is some over-extrusion.

\n" }, { "Id": "19989", "CreationDate": "2022-09-29T13:41:26.263", "Body": "

I had a model I had originally printed in PLA, and then lost the STL... but I still have the G-code. Now I want to print it in PET-G. Do I need to do anything other than update the numbers on the M104 and M90 instructions near the top of the file to set for new hot end and bed temperatures to get it to print properly?

\n

I know some materials I might also need to adjust speeds, but it seems like with PLA<=>PET-G those two items ought to be enough. Is there anything else I should check?

\n", "Title": "Editing a G-code file for different material", "Tags": "|g-code|print-material|", "Answer": "

Tired of waiting, the print is now 40% done and seems to be just fine.

\n

Thinking through my own problem more, there are three other things I might want to check (other than base speeds, as mentioned in the question) if I do this in the future:

\n
    \n
  1. Cooling. Some materials need the cooling fan set differently. This should also be a single line edit near the top of the file.
  2. \n
  3. Retraction. Some materials need different retraction settings to control stringing, and in fact I do find I often need retraction to be a little more aggressive for PET-G vs PLA, even if this one print came out okay. This change is needed for every retraction instruction, instead of just once at the top of the file, but I could probably adjust it via a simple find/replace in the file.
  4. \n
  5. Travel Speed. Some materials need faster travel moves, also for stringing. This manages a trade-off between speed and quality. One could just always set for the fastest travel speed, which both improves print times and stringing, but faster travel moves sometimes result in ringing/ghosting at the corners, and so there's a balance that can be different per material type. This might be adjustable with a find/replace, but I would want to read through the file some to be sure we don't use similar gcode/speed numbers for anything else. It may also be possible to use strong accelleration/jerk controls to compensate for consistently faster travel moves, in which case I could set my slicer to use fast speeds that still always work, regardless of material.
  6. \n
\n" }, { "Id": "19998", "CreationDate": "2022-10-01T16:36:32.280", "Body": "

Are 3D prints safe to touch right after printing? If not should I do some kind of curing (like in resin printing) when my prints are done or wash them with alcohol or something.

\n", "Title": "Are 3D prints safe to touch right after printing?", "Tags": "|print-material|safety|material|health|", "Answer": "

FFF/FDM: Yes

\n

FDM/FFF runs by melting a filament and letting it cool again. As soon as the printed parts are cooled down to below 50\u00b0C (which is rather quick) handling them is safe. There's no chemical process involved, it is a pure physical, thermoplastic process, and the plastic re-solidifies as the part is cooled down.

\n

It is far more likely to get damage from touching the hotend than from the printed part.

\n

SLS/DLMS aka Fused Powder prints: Yes but...

\n

SLS/DLMS runs by pointing a high-powered laser on a fine powder and melting/smelting/welding/sintering it together. While such powder prints, no matter what type of powder they are made from, are safe for your hands to touch, the fine powders are very bad for your lungs. Handling such prints requires breathing protection until they are dusted off, usually using compressed air.

\n

For stability, some are impregnated with resins after print, which requires hand protection till the resins are cured.

\n

DLP/SLA aka "Resin": NO

\n

Resin parts are made by curing a liquid resin.

\n

The most common type is DLP/SLA, where a laser or light source cures the resin at the bottom of a vat and the model is pulled up from. As a result, they come coated in liquid, uncured resin. These resins are a known skin irritant, and their fumes can be an irritant too. In general, keep them in a dedicated workshop.

\n

Because of this, these prints are not safe to handle barehanded and need to be washed and cured after printing.

\n

Polyjet Resin Printing: kindoff

\n

There's a method of resin printing that is more akin to an inkjet printer, such as the Stratasys Polyjet system. Those prints don't come coated in resin and come out cured to a point where handling them is safe, but those are (as of 2022) industrial machines, and even as the PolyJet core patent expired in 2019, it is a market that nobody jumped at, especially considering many of the advanced features like full-color Polyjet using CMYB is under patent still. Oh, and apparently, even in 2022, only one other company managed to make a functional Polyjet machine that is ripe for the market, which is super tiny in the first place.

\n

Back to topic. While the parts come out fully cured, polyjet printing itself is messy. You absolutely need support material everywhere. It is safe to touch, but in the dissolution stage of the support material, it is most certainly not safe to touch.

\n" }, { "Id": "20007", "CreationDate": "2022-10-02T12:51:41.797", "Body": "

I am trying to print ColorFabb XT-Filament PETG on my Kobra Max, and I get the following results:

\n

\"Mess\"

\n

These are my Cura settings:

\n

\"Cura

\n", "Title": "What is this situation called and how do I avoid it?", "Tags": "|ultimaker-cura|filament|petg|colorfabb-xt|", "Answer": "

WAY too low print temperature/wrong material Profile

\n

That's a PLA profile with 195\u00a0\u00b0C on the printhead. PETG is recommended between 230\u00a0\u00b0C and 250\u00a0\u00b0C. Load a PETG-Profile. in Cura, the material (which dictates print temperature) is a separate thing and can be swapped independent of many other settings, especially how detailed you print. However, very careful piece of advice:

\n

Save your profile before swapping materials!

\n" }, { "Id": "20015", "CreationDate": "2022-10-04T12:20:24.290", "Body": "

I just purchased the PrimaFix heated bed adhesive for my 3D printer. I read on the back of the box that it's dangerous when it comes in contact with the eyes, skin, mouth and when swallowed.

\n

After printing, is it safe to touch the first layer? If not, what should I do to make it safe to touch?

\n

\"enter

\n", "Title": "Is the first layer safe to touch after printing with heated bed adhesive (PrimaFix)?", "Tags": "|adhesion|safety|health|", "Answer": "

This is a general purpose catch all warning that some people with sensitive skin might have a rash like reaction, and you should never put these things in your mouth or eyes anyway.

\n

It just means that you should treat it like you would super glue.

\n

Once it's dry it's inert and safe to handle

\n" }, { "Id": "20016", "CreationDate": "2022-10-04T12:42:31.440", "Body": "

I have a PLA 3D-printed object made of several rings with a diameter of 150 mm and a thickness of 6 mm. The height of each ring is about 30-35 mm but it does not matter so much.

\n

I'm looking for a way to keep the rings together (one on top of another). I cannot place anything inside the ring (like a thicker area to host a nut). Most of the rings may have a closed top or bottom, so I cannot access the inner part.

\n

I'm using three tabs (at different angles to force the correct alignment) that fit inside matching slots on the counterpart:

\n

\"enter

\n

\"enter

\n

They are a bit stronger than pins and holes due to the greater surface of the base layer, but of course are useful only for alignment.

\n

Due to the very small thickness of the ring I cannot place a screw inside them to keep in position, the inner wall cannot hold a screw!

\n

I was thinking about L-shaped tabs and slots to fix the rings rotating them a bit, but:

\n
    \n
  1. it would not easy to clean the horizontal leg of the slots
  2. \n
  3. nothing would prevent to rotate the rings in the opposite way (even accidentally) so the disconnect
  4. \n
\n

For my use-case scenario, I don't need anything too strong. Just avoid to disconnect the rings when taking the object in hands. I tried with magnets but:

\n
    \n
  1. to make an acceptable resistance I needed 7-8 per ring (due to their small size)
  2. \n
  3. the electronics inside uses Hall sensors so I cannot use magnets!
  4. \n
\n

Any idea?

\n", "Title": "Make a joint to keep together rings of PLA", "Tags": "|pla|cad|", "Answer": "

You could use velcro. I find it very easy to work with because you can cut it precisely with scissors or craft knife, and it's great for just holding things together but won't come undone by accident.

\n

It's cheap and there are lots of different thicknesses at sewing shops. The ones I use come in sheets and have an adhesive backing. Takes a day for the adhesive to cure but once it's stuck on it won't come off.

\n" }, { "Id": "20022", "CreationDate": "2022-10-05T07:43:20.790", "Body": "

I use eSUN (Silk) PLA filament daily and I've never had an issue. I use the parameters suggested by the manufacturer:

\n\n

The extrusions are perfects (the printing itself may be improved, but here I'm talking about the extrusion) even if it runs for 20h+.

\n

Instead with the Silk Rainbow PLA filament, after few hours the extruder is clogged. No more extrusion. I have to stop the printing (and waste all) preheat the extruder, remove the nozzle and extract the filament. Then, feeding another kind of PLA I'm able to recover the functionality.

\n

This happens ONLY with this specific filament!

\n

I tried to increase the nozzle temperature up to 220 \u00b0C. It lasted maybe one hour longer but eventually failed again.

\n

The filament is stored inside a dryer that is on during printing.\nWhat can cause such a behavior? I have a Dremel 3D45.

\n

UPDATE

\n

Here some pictures. Some parts are broken because I had already thrown it away. The bottom layer is quite good:

\n

\"enter

\n

Details of the surface of the base:

\n

\"enter\n\"enter

\n

Details of the infill and support structures:

\n

\"enter

\n

Weird enough both the comment and the answer suggest to lower the temperature but here the answer I received from the manufacturer:

\n
\n

You can adjust the printing temperature to 230 \u00b0C higher, and then\nadjust the printing speed to 40 mm/s slower. The plug is generally\ncaused by bad material fluidity, the temperature is set higher, you\ncan increase the fluidity.

\n
\n", "Title": "eSun Silk Rainbow PLA leads to nozzle clogging", "Tags": "|pla|extruder|nozzle|print-failure|dremel-3d45|", "Answer": "

I suggest you lower the temperature. There's a different sweet spot for some filaments even if they're the same brand. And it's not always the middle recommended heat on the label I have found.

\n" }, { "Id": "20024", "CreationDate": "2022-10-06T03:14:25.903", "Body": "\n

I have printed 1 model 3 times with different settings (different temperatures and different bed temperatures).

\n\n

All 3 models show 1 "line" at some point.\nThe models are smooth all around, but then there is this 1 line on each model.

\n

What is this error called, please?

\n

\"Photo

\n

\"enter

\n", "Title": "A vertical scar on the print surface, what is the name of this problem?", "Tags": "|print-quality|ultimaker-cura|pla|filament|anycubic-kobra-max|", "Answer": "

I had the same concern to make mechanical connections where we tolerate a distance of 0.1 mm and these so-called "sewing points" making it impossible. I started to compare between the preview before printing in the slicer and the part printed before.

\n

The problem is recurrent along a vertical line and always repeats itself at the same point. So by looking to the layer settings, there is a junction between the outer and the first inter shell. This junction could be avoided by modifying the "shell print order" parameter.

\n

Here is a video shows the difference abd before / after picture. The line remains visible but the points disappear completely. You can set the layer height to 1.2 mm for better results.

\n

\r\n \r\n

\n" }, { "Id": "20029", "CreationDate": "2022-10-06T13:59:01.680", "Body": "\n

I have printed 1 model 3 times with different settings (different temperatures and different bed temperatures).

\n\n

All 3 models show a seam.

\n

How / where do I best hide this seam?

\n

\"Photo

\n

These are some screenshots of the model in Fusion 360:

\n

\"enter

\n", "Title": "How / where do I hide the seam?", "Tags": "|ultimaker-cura|pla|", "Answer": "

Hiding the seam isn't really possible when the object is rotationally symmetric. The closest you can get is hiding it "statistically" by distributing the seam randomly so it's fractionally-bad everywhere rather than fully-bad in one place. Depending on your needs, this might be good, or it might be awful. If you're hoping to clean up the seam in postprocessing, it's probably awful since there's not just one isolated point that needs fixing with a razor blade or something, but the whole print surface.

\n

One thing you can try, though, is turning on wipe. In Cura this is "Outer Wall Wipe Distance". Setting it to 1-2 mm can go a long way to concealing the seam, especially if it's not bulging from ooze, which yours doesn't seem to be. Combined with random distribution, this might come out looking pretty good for your particular need.

\n

In general, how to deal with seams depends on whether the problem is cosmetic or functional. I'm assuming in your case it's cosmetic, but if it is functional (affecting fit of mechanical parts together, making a printed gear rotationally asymmetric, etc.) then one trick you can do is make an artificial cut into the surface at an arbitrary point and selecting to put the seam on the sharpest corner. This can help ensure the entire print fits inside the intended print volume, and is only missing a small amount of material on a scale smaller than the functional scale of the print.

\n" }, { "Id": "20037", "CreationDate": "2022-10-07T10:25:43.763", "Body": "

I need to build me a small support wall 250x80\u00a0cm mounted on two metal supports approximately 200\u00a0cm apart.

\n

I was looking into WPC (Wood Plastic Composite) boards and then it hit me that I may be able to make it functional AND pretty with my 3D printer (Prusa MK3S + MMU2).

\n

However, I have zero idea on how to calculate whether the board will be able to handle the weight of the soil I would shovel behind it, if at all.

\n

My current design proposal is to make a hexagon mesh of PETG filament. The hexagons would have cylinder cutouts in all three directions, through which I would then hammer in cylinders to bind them together. Perhaps even some edge contours so that they fit perfectly.

\n

But, my problem is that I don't know how to calculate the strength of such a solution (or any other).

\n

Is this even a viable idea?

\n

The filament type needs a bit more explanation:\nMy current proposal is PETG since I read that PLA will become brittle with time (as I can attest to myself having to replace my flag pole holder every 3 - 4 years)

\n

I don't want ABS since I have horrible experience printing it.

\n

I did not find anything of particular use when searching for "3D print load-bearing" on this site and similar with Google searches...

\n", "Title": "Can I 3D print a wall assembly and expect it to be load-bearing?", "Tags": "|print-strength|", "Answer": "

Here are some general issues with load bearing 3d prints for this type of usage:

\n\n" }, { "Id": "20060", "CreationDate": "2022-10-13T01:53:05.310", "Body": "

If I am 3D printing on my Ender 3, and I pause the print, the extruder lets out more filament for like 2 seconds. This leaves a glob of dry filament on my print, which the could contact the nozzle tip, causing it to slide the build platform.

\n

I have fixed the build platform sliding problem by attaching the included clips. However, another problem remains. Once the glob is made, the filament dropped on top of it gets pushed up, making the print not be even, which ruins the print. I don't think I did anything wrong on the setup, but it might just be that the extruder doesn't stop in time? Did I do anything wrong is is this an issue among all Ender 3s?

\n\n", "Title": "Extruder oozes filament out and slides build platform on contact with nozzle tip", "Tags": "|creality-ender-3|extruder-driver|", "Answer": "

Working pause-and-resume without damaging the quality of your print depends on the printer doing a lot to preserve the primed/ready-to-print state of the printer and avoid heat damage to the part you're printing and to the filament, and on your using very well-dried filament. If any of these requirements aren't met, you're going to have a bad day.

\n

In particular, the printer needs to do extra retraction (at least an extra 10 mm, probably more like 20+) to get the filament completely out of the melt zone while paused, and needs to quickly move the nozzle away from the print so as not to melt it. If this is done right, unretracting by the same amount just when starting will get you back to the necesssary nozzle pressure to resume extrusion nearly uninterrupted. Marlin firmware can be configured to do this right, but I doubt it's done right in the version of the firmware shipped with your printer. You didn't describe what happens in detail when you pressed pause, but it sounds like this isn't happening, so it's likely doing just a "dumb pause" in-place.

\n

If you really want good pause functionality, you probably need to investigate customizing the firmware. This is not terribly difficult, but also not dead simple, and can go badly wrong if you mess it up. You'd need to weigh these risks against how much you want the functionality.

\n" }, { "Id": "20079", "CreationDate": "2022-10-15T20:11:39.957", "Body": "

I upgraded my Ender 3 V2 with a BIQU H2 hot end and extruder head and a BLTouch probe. I updated the firmware from Ender website using Ender-3 V2_32bit_4.2.2_BLTouch_Marlin-2.0.1 - V1.1.1.bin. All hardware seems to work flawlessly, incl. all axis movements, extruder movements, heating, fans controlling,...

\n

However, I don't know how to really proceed with changes to extruder settings. I found no YouTube videos or internet articles, particularly about "Ender 3 V2" and "BIQU H2" and what I found is not compatible.

\n

From what I gathered, I need to change the speed ratio of extrusion and the gear ratio of extrusion. But I don't know how to set it on Ender 3 V2. I tried to configure the gear ratio. There's a nice YT manual for Ender 3 V2, however for the old extruder, so I couldn't measure the extruded filament beyond the extruder. I made a line 100\u00a0mm above the extruder with a marker and set the extrusion to +100. I got something like 10\u00a0mm of snail-pace slow extrusion. So I went to the extruder's gear ratio and tried to raise it, but the value should be roughly $100 / 10 * 93 = 930$ (93 was the default and current value). However, the adjustment goes only to 186, and with that, I get a little bit less than 20\u00a0mm of extrusion. And I also suspect that the extruder's speed is about 7 times smaller (1:7 gear ratio) than what it is supposed to be.

\n

So, could you advise or direct me to materials that explain how to set BIQU H2 on Ender 3 V2? Is there anything else I have to set, apart from extruder speed and gear ratio? Some tutorials mentioned about stepper motor voltage if I remember correctly, but again, that cannot be set in the menu. I'm fine with settings over G-code, as long as I have some clear instructions on what actually to set.

\n
\n

Note:

\n

I marked Oscar's post as an answer, though to me it tells only part of the story. But helped to clarify some stuff. Be careful and study the matter yourself, however, you should come to this conclusion and those steps for Ender 3 V2 and BIQU H2, particularly for setting the extruder motor current.

\n

Disclaimer: Those were my steps and I do not take any responsibility for anything if you follow them. Check everything yourself and advice in the comments below, if anything doesn't look right.

\n
    \n
  1. Check you have TMC2208 - you should have the large letter A written on your card reader

    \n
  2. \n
  3. Check your BIQU H2 stepper motor nominal voltage is 800\u00a0mA in the attached manual

    \n
  4. \n
  5. Check the resistors at the stepper motor (marked E) on your main board are R150 (150\u00a0m\u03a9)

    \n
  6. \n
  7. If all above is true, you should get Vref = 1.05. Based on

    \n
  8. \n
\n

$\\frac{0.8}{\\sqrt{2}} = {\\frac{325mV}{150m\u03a9+20m\u03a9}*\\frac{1}{\\sqrt{2}}*\\frac{V_{ref}}{2.5V}}$

\n

I separated voltage:

\n

$V_{ref} = \\frac{0.8A/\\sqrt{2}}{325mV/((150m\u03a9+20m\u03a9)*\\sqrt{2}*2.5)} = \\frac{(150m\u03a9+20m\u03a9) * 2.5 * 0.8A}{325mV} = 1.046V$

\n
    \n
  1. Measure your current Vref with positive contact on the potentiometer and negative on the card reader cover, using an appropriate scale on your Voltmeter, i.e. up to 20V DC. You should get something like 1.4\u00a0V, if you have the default factory setting. One hint: I put a piece of insulation tape on the cooler next to the potentiometer in order to avoid a short circuit in case the cooler is connected to the negative body - it's easy to touch it with a screwdriver. Remove it after the correct voltage is set. Or use a plastic or ceramic screwdriver.

    \n
  2. \n
  3. Turn slightly the potentiometer anti-clockwise, about 1/5 of the circle for start, and measure the voltage again, then do a fine adjustment, measure again, etc., until you get desired value. I could set it from 0.03 to 1.02, meaning that I set 780\u00a0mA, slightly below the nominal voltage.

    \n
  4. \n
\n

\"Photo

\n", "Title": "Ender 3 V2 calibration for BIQU H2", "Tags": "|creality-ender-3|calibration|", "Answer": "

The Ender you have uses the TFT display. This display operates its own firmware, so when you update the controller board, you only update the printer function not the GUI interface. It is the interface that limits the input you require for setting the correct E-steps per millimeter as these are limited in the region of the stock extruder; it has no anticipation for using a much larger value like you need when using a large gear ratio extruder. When the correct steps per milliliter are set, you do not need to worry about the speed other than that you cannot use very high retraction speeds with high gear ratio extruders, if too fast it will skip steps.

\n

Question Adjust E-step on Ender 3 already explains how you set the correct value for the E-steps, it does not require the display to set this. You do require either a connection over USB to a computer/laptop or an SD-card.

\n

Do note that the stepper you use on the direct extruder is different from the one installed stock as extruder motor, the reference voltage must be set manually using a multimeter and small screwdriver, your driver's don't allow to adjust through firmware or G-code. This stepper should be limited to 0.8 A. You need to calculate which Vref you need for your driver (you probably have TMC2208 stand alone drivers) and adjust the potentiometer to measure the calculated Vref. You do not measure current directly, this is done through measuring the Vref (this is a measure for the current). There are many sources to be found to set this Vref, e.g. this one.

\n" }, { "Id": "20082", "CreationDate": "2022-10-17T02:20:31.737", "Body": "

I was trying to fasten the top screw on the power supply of my Ender 3, but it won't fasten. So, I took the screw out and found something like a little yellow ring stuck onto it. I think it broke because I may have over fastened it, but I didn't tighten too much... Is this the "thread" of the hole? Is it supposed to be so fragile?

\n

\"Img

\n", "Title": "Ender 3 power supply top screw isn't tightening", "Tags": "|creality-ender-3|power-supply|", "Answer": "

Sounds like you've stripped the thread by overtorquing the fastener.

\n

Your options are to use a slightly larger fastener and drill/tap the larger hole for that bigger thread. And don't confuse the screws next time you open this connector.

\n

The little yellow ring might be a retainer so the screw is held captive in the part, or it could be a load spreading washer, or a soft area to soak up vibration and prevent buzzing/rattling and resonance.

\n" }, { "Id": "20088", "CreationDate": "2022-10-18T16:57:03.020", "Body": "

I recently ran into some issues where my Mac (macOS 12.6) was no longer recognizing the USB drive that came with my Elegoo Mars 3 Pro. The printer also wouldn't recognize the drive, so I used Disk Utility to reformat the drive to MS-DOS (FAT). However, after doing this, the printer still would not recognize the drive.

\n

After some web searching, I found that the drive needs to be formatted using a Master Boot Record scheme, but Disk Utility was not displaying this option when formatting a drive.

\n

How can I format a USB drive for an Elegoo Mars using macOS?

\n", "Title": "Elegoo Mars 3 Pro USB drive formatting?", "Tags": "|usb|elegoo-mars|", "Answer": "

Disk Utility can be used to format the drive, but first, you'll need to change the view settings to enable the Scheme options when erasing the drive.

\n

In Disk Utility, from the View menu, select Show All Devices:

\n

\"Screenshot

\n

Next, you'll need to select the parent device on the left-hand pane, and not the volume:

\n

\"Screenshot

\n

Finally, after using the Erase button in the top bar of the window, you'll now have the option to select the scheme:

\n

\"Screenshot

\n

Erase the drive using the MS-DOS (FAT) format and Master Boot Record scheme. The name of the drive likely doesn't matter. Using these settings, I was able to copy some files back to the newly-erased drive and the printer immediately found the files and was able to begin printing again.

\n" }, { "Id": "20092", "CreationDate": "2022-10-19T14:03:52.960", "Body": "

I feel this is a relatively simple thing, but I am having some difficulty. The model is simple. It is 2 x 127 x 28 x \u03c0 mm. It has one corrugated side and all others flat. The non-flat side has a flat area for text.

\n

(Later deformed into a cylinder, hence pi (\u03c0))

\n

The problem I'm having in Blender is a lot of non-manifold vertices after the union of the text into that area.

\n

Is it possible to have the text and widget as two different meshes touching and come out as one object in the print?

\n

Use another program besides Blender?

\n

Manually editing the mesh... seems excessively time consuming.

\n", "Title": "How can I incorporate text in my model?", "Tags": "|3d-models|blender|", "Answer": "

You have a number of options with respect to adding text to a shape. Top of the list would be the easy-to-use Tinkercad, but it may be too limiting for your purposes.

\n

More complex, and coming in next would be the free hobbyist version of Fusion 360. In very many ways, a tougher program to learn, but YouTube tutorials may get you started on your specific objective.

\n

My favorite would be to learn the wonders of OpenSCAD, a text based descriptor program that uses parametric modeling to create 3D objects.

\n

It's not impossible to create non-manifold with the above three programs, but very much easier to create a working model.

\n

If your final objective is a cylinder, it might be good to start with the desired shape and apply the text in the program of your choice.

\n" }, { "Id": "20093", "CreationDate": "2022-10-19T18:37:35.863", "Body": "

I was running my Ender 3 just fine with good first-layer adhesion/everything else but my setup changed after my Pi SD card got corrupted so I had to re-install and reconfigure my bed leveling via OctoPrint. After this my first layer started to look like this:

\n

\"Image\n* this is the bottom of the completed print

\n

IIRC I changed the following (and have tried undoing it but to no avail):

\n\n

This is with PLA

\n

Running a modified Ender3 w/

\n\n

Sliced on Cura 4.11.0

\n\n

I've tried:

\n\n

After the first couple of imperfect layers get ironed over, the upper layers will not have any extrusion problems, or whatever this is, and are basically fine.

\n", "Title": "Ender 3 first layer inconsistent layer lines", "Tags": "|creality-ender-3|marlin|ultimaker-cura|troubleshooting|bltouch|", "Answer": "

Solved, the nozzle was set too low and was causing pressure buildup at certain points since the PEI plate wasn't perfectly flat.

\n

After raising it and swapping for a glass bed I was able to stop this behavior.

\n

Note: While this helped this issue a little bit, it was also making adhesion really poor on faster prints. A better fix I found was that I recently swapped filament spools, it looks like the average spool thickness was greater than the previous one, and after reducing the flow 5\u00a0%. I was able to eliminate this problem entirely, thanks to the question What is causing 'droplets' on first layer?

\n" }, { "Id": "20105", "CreationDate": "2022-10-21T09:08:51.197", "Body": "

The seams on my print are much larger than they were a few months ago and I'm not sure what's going on or how to fix it.

\n

Pic with seams random:

\n

\"https://i.imgur.com/PvmpsWb.png\"

\n

With aligned seams:

\n

\"https://i.imgur.com/1GIDDz9.png\"

\n

I'm using the 0.2\u00a0mm QUALITY profile with the following adjustments:

\n\n

My full slicer settings found here.

\n

I've tried lowering the retraction speed, increasing it, increasing the detraction speed to 2x, raising temp, and calibration of the linear advance according to the calibration test print on Prusa's website.

\n

What I can do to make gaps smaller?

\n", "Title": "Seams of print are underextruded causing large pits/gaps? (Prusa Mk3+, Prusa Slicer)", "Tags": "|prusa-i3|underextrusion|prusaslicer|", "Answer": "

I was able to more or less solve it by using the "extra length on restart" property set to 0.1 mm. Sort of "primes" the filament after a retraction by moving it forward a bit.

\n

To lesser effect I reduced retraction distance from 0.8 to 0.5, lowered retraction speed from 35 to 10, and upped detraction speed from 35 to 50.

\n

\"enter

\n

Left object is with new settings, right object is original issue.

\n" }, { "Id": "20122", "CreationDate": "2022-10-25T04:10:45.107", "Body": "

I have a Creality LD-006 resin printer and my FEP breaks very often. When I'm printing I heard a hard "tac\u00a8" (it's because the cured resin is taking of the FEP when the printing bed goes up).

\n

I'm using a Creality standard resin white with 2.7 exposure time.

\n

This is how my FEP breaks (I replaced the FEP (it was original) and made maybe 2 or 3 prints and it broke).

\n

\"Broken

\n

Could it be that I'm printing a lot of small pieces at once?

\n

I have a smaller printer (Creality Halot-One) and I made more prints and I never have this type of problem. Maybe I can't print small pieces in a big printer?

\n", "Title": "How to solve broken FEP", "Tags": "|troubleshooting|resin|repair|", "Answer": "

FEPs are consumables

\n

Once an FEP film on the foil has developed cracking, it's a lost cause and has to be replaced in due time - repair to pre-damage conditions is impossible even if you manufacture them.

\n

You can try to avoid the affected areas and eek a few more prints out of the film, but the damage will spread and make the film unusable in the longer term.

\n

Damage to FEP can occur due to bad resin

\n

If your resin contains any chunks due to print failure, those can result in damage of the FEP.

\n

Bad Placement and high speed can damage FEPS

\n

Another reason can be too pointed pressure, which happens for example if the only spot is pulling up very high, and then suddenly releases.

\n

The position of the printed objects can be as much the factor in this case as the size or geometry of the release area.

\n

The more controllable factors that play into this are the lifting speed and height of the bed. It takes some non-zero amount of time for the film to release the model. During this time the film adheres to the model, and can be simulated as a spring. The force of a spring is dependent on a constant and its elongation: $F=k \\times \\Delta d(t) $. Our foil would have a variable factor $k$, dependent on the coordinate in the vat. However, for us, the elongation at the moment t $\\Delta d(t)$ is relevant. By lifting faster and higher, the film is experiencing more elongation (stretch) and a sharper release from the model happens. Higher stress means damage to the film.

\n" }, { "Id": "20127", "CreationDate": "2022-10-26T10:42:35.897", "Body": "

I am finally unable to reach adequate printing with my Creality 3D. This is how it prints a raft

\n

\"enter

\n

As you see, thread is non-uniform.

\n

What can be a reason?

\n

Printing setup:

\n
    \n
  1. Nozzle is brass, 0.4 width, new from Amazon

    \n
  2. \n
  3. Filament is PLA

    \n
  4. \n
  5. Nozzle temperature is 220, bed temperature is 60.

    \n
  6. \n
  7. I am using autolevelling with BLTouch

    \n
  8. \n
\n\n", "Title": "Print thread is non-uniform", "Tags": "|creality-ender-3|", "Answer": "

I found a problem: it is with extruder motor driver.

\n

I tried to control the extruder with the knob and saw it that rotates with strange sounds and jerkily. Then I connect extruder cable to the socket of Z-axis on motherboard and also tried to control it with the knob. This time I found it rotates smoothly.

\n

I.e. motor is good, cable is good, motherboard 4th driver is not good.

\n" }, { "Id": "20138", "CreationDate": "2022-10-27T22:26:20.400", "Body": "

I want to know how simple is it to install CR6Community firmware

\n

https://github.com/CR6Community/Marlin/releases

\n

I do not want to make low-level modifications for it. And also I want to know how simple is it to get back to official firmware?

\n

I want to update because I came to know it has better calibration options and also I'm having issue with Z Axes that does not remember its position.

\n", "Title": "Installing Creality CR6 SE Community firmware", "Tags": "|firmware|creality-cr-6|", "Answer": "

If you can reformat a (micro)-SD-card on a Windows PC (there are built-in system tools) and copy files to it, then you have enough skills.

\n

Going back to the stock firmware is as easy as copying the files to an SD-card and having it inserted to the printer while it starts up.

\n

After that you have to reflash the display, but this is similar to flashing the printer, as you have to copy files on a micro-SD-card and unscrew the back of the display to get to the card-slot on the back of the display-control-board.

\n" }, { "Id": "20141", "CreationDate": "2022-10-30T02:22:11.867", "Body": "

I need to print a thin piece.\nTheoretically, it could work, but Cura prints the walls as single lines.\nIn my case, it is just this line that breaks apart as you can see here.

\n

Is there a setting that would instruct Cura to avoid creating such a line as the outer wall, or how else could I resolve this problem?

\n\n

Speeds:

\n\n

\"Photos

\n

\"Screenshot

\n", "Title": "How do I avoid straight lines for walls in Cura?", "Tags": "|ultimaker-cura|anycubic-kobra-max|", "Answer": "

I'm pretty sure your core problem is underextrusion from printing PETG faster than your extrusion system can actually keep up with. Unless you've cranked up the acceleration, long straight lines are the only place in a print where the actual print speed will reach cruise at the requested speed; otherwise, all the time is spent speeding up towards that requested speed, then slowing back down for the next turn before reaching it.

\n

If you have a single-flat-filament-gear, no-reduction-gearing, bowden extruder like the Ender 3 and most of these Ender 3 clones have, you'll be lucky to get reliable, consistent extrusion with PETG even at 50 mm/s (assuming 0.4 nozzle and 0.2 layer height). 80 mm/s is almost surely too much without upgrading your extruder.

\n

Also, you didn't say anything about temperature, but low temperature will exacerbate this problem and lead to the failed layer bonding you're experiencing. Anything below 245\u00b0C is "very low" for PETG - despite what manufacturers recommendations may say. They write the recommendations low so that customers with PTFE-lined hotends (which can't safely go above 245-250) will buy their product, not because it actually works well at those temperatures.

\n" }, { "Id": "20143", "CreationDate": "2022-10-30T11:23:18.843", "Body": "

I'm using Creality Slicer, brims and other anti-warming or adhesion settings are all turned off. when I place an object on the bed a shaded area appears around it (See picture).

\n

\"enter

\n

Is this simply the area in which you can't print anything else in sequential print mode due to the risk of the print head hitting it, or is there some other meaning?

\n", "Title": "What is the shaded area that appears around the outside of objects in Creality Slicer?", "Tags": "|slicing|creality|", "Answer": "

Creality Slicer is a derivate of Cura. As such, all standard Cura things apply.

\n

Cura uses red surfaces to denote overhangs.

\n

Your upper surface is denoted as an overhang, this indicates that your item has its surfaces at least partially "upside down". The model needs to be repaired, as such can result in the print solution being "That's to be a closed surface", ignoring the hole in the center. The problem isn't the slicer, it is the generation of the STL that messed up.

\n

About inverted surfaces, see also: here, here and here

\n

Boxes around items come from sequential printing

\n

The box around parts is also usually created when you try to print multiple items in sequence and relates to the dimensions of your printhead. It pretty much marks the exclusion zone where you are not allowed to place the next item. Check if you have turned print order from all at once to in sequence and adjust as needed.

\n" }, { "Id": "20148", "CreationDate": "2022-10-31T02:37:53.427", "Body": "

I have a request to create a jigsaw puzzle based on a road layout, for a customised christmas gift. Imagine a Secret Santa present.

\n

The city in question is Dunedin, New Zealand, which is on the waterline and has surrounding hills. I'd like to combine a download from https://touchterrain.geol.iastate.edu/?DEM_name=JAXA%2FALOS%2FAW3D30%2FV2_2 (available in OBJ, STL Binary, STL ASCII, or GeoTIFF) with an overlay like this in JPG/PNG.

\n

\"enter

\n

Last step before printing would be to carve out individual jigsaw shapes for printing separately.

\n

I imagine each road would be a slight "trench" or hollow in the STL's surface. Clearly a filament swap would not work to paint the roads as they climb, so either coloured marker or a fine paintbrush would be used, followed by clearcoat to preserve.

\n

What software workflow would give a useful result?

\n

I've looked at blender and freecad and tinkercad but nothing gives any result close to what I imagine but I haven't the experience.

\n
\n\n

I suspect the STL is not "closed" which is leading to all manner of difficulties.

\n

What would get me close to my desired print outcome?

\n", "Title": "Software workflow for merging a flat image and a STL?", "Tags": "|software|stl|pre-processing|", "Answer": "

I was not able to perform the download at your link. Instead, I visited the touch_terrain site and approximated the location. The modifications allow for a thicker base, which I selected to 4 mm arbitrarily, eliminating the non-closed (probably) as well as exaggerated the vertical scale to 4x.

\n

\"NZ\nThe result is satisfying and appears to have a minor validation problem surrounding the perimeter of the base, but no problems with the terrain.

\n

If adjusting the base thickness does not resolve the problem you experience, consider also to increase the overall size of the model and use the tiling function to segment as needed.

\n

For the roadway application, you can create an SVG of the map at appropriate scale. Thin lines will be problematic, but the thicker lines can be boolean-subtracted from the STL file. It will be necessary to extrude the SVG to sufficient thickness to impact the terrain STL in order to create the trenches.

\n

It might also be possible, but far more complex, to use a feature in Fusion 360 that overlays an extrusion onto a surface in a uniform manner. Angus from Maker's Muse has a video explaining the process, but it's a pair of primitives, not a complex terrain and may not apply.

\n

With the permissions corrected, the file downloaded without complications. I note that the composition/analysis is in excess of 140K triangles (Meshmixer) and could create problems with some programs. Fusion 360 will either refuse to function or provide an alert.

\n

I reduced the mesh using Meshmixer set to 70 percent, resulting in no apparent loss of detail, but fewer than 40K triangles. Tinkercad also dislikes high triangle count objects. The reduced file will be available for download for a limited time.

\n

\"STL

\n

Image from released (reduced) STL file.

\n
\n

But wait, there's more. After some additional thought, with consideration for your constraints, I think I have a workable approach. I'm not sure how you'd approach this first step, though. I used Windows 3D Builder to repair the broken base. Meshmixer shows the flaw but does not repair it. I suspect other online repair facilities will resolve this simple problem. I could have tried Meshmixer (plane cut) and certainly Fusion 360's plane cut, but only thought of that as I type this addendum.

\n

Once repaired, I saved your map image and traced it in a suitable program to create the SVG file. Both in Inkscape and in LightBurn (a laser editing/burning program), the tracing went well. LightBurn allowed me to confirm that the SVG generated had closed shapes, but Tinkercad would not import any variation I created. I used Inkscape's Path to OpenSCAD which previews fine the extrusion to 10 mm, but it would not render, complaining of non-closed shapes.

\n

\"extruded

\n

There are "speckles" all over the map, even though I enlarged it to 200% scale to reduce that aspect of the lines. I suspect this will be problematic and suggest to locate a better resource, to eliminate excessive detail in the map.

\n

Once you have that solved and can extrude your map, scale and align it to match the terrain STL which has been repaired. Create a duplicate of the terrain, also properly aligned and elevate it. The object of this step is to use the terrain to carve away the extruded map. If the boolean subtract does not destroy the terrain, great, otherwise, lower the extruded map into the terrain to the desired depth and perform another subtract.

\n

This should give you the channels you require for painting.

\n" }, { "Id": "20153", "CreationDate": "2022-10-31T19:15:55.457", "Body": "

I bought a new 32-bit board for my old Creality printer, and now I am going to connect my old old BLTouch sensor to it. I got some instructions from the internet and found that it needs to be connected to the central pin on MoBo.

\n

\"enter

\n

This pin is marked "IN" while its purpose is to control BLTouch effector, i.e. it should be "OUT", not "IN".

\n

Why?

\n
\n\n", "Title": "Why is the pin, which is intended to control the BLTouch, marked \"IN\" on Creality board?", "Tags": "|creality-ender-3|bltouch|wiring|", "Answer": "

TL; DR - The pins are clearly named in reverse, and the IN pin is obviously an output and the OUT pin is obviously an input.

\n

However, as to the why, it would be hard to guess, apart from the lazy deduction of either:

\n\n

Without one of the board's designers posting an answer here, I guess we will never know for sure. A more generic labelling of both of the pins, such as IO1 and IO2, would have been better, and less confusing.

\n

In addition, the guide, where the image that you posted comes from, seems to be rather poorly written and I would caution you against following it - see below for more details.

\n
\n

Notes

\n

I may be wrong but the image that you show is presumably from this guide, Creality V4.2.2 & V4.2.7 Motherboard BL Touch Wiring Options, which shows two differing wiring suggestions (5 wire and 3+2 wire).

\n

The 5 wire

\n

A BLTouch with a 5 pin connector attaches to the 5 pin socket on the controller board:

\n

original image

\n

\"5

\n

The 3+2 wire

\n

A BLTouch with both a 3 pin and a 2 pin connector attaches to part of the 5 pin socket (the three left most pins, G, V and IN) and the Z-axis minimum endstop (Z-), respectively:

\n

original image\n\"3+2

\n

Note: If the image is expanded and examined carefully, it can be seen that "IN" has actually been written on the diagram, just over the yellow wire - obviously this is just a blind repetition of the mis-labelled pin.

\n

Deduction

\n

Either the z-axis endstop or the right most two pins of the 5 pin connector (G and OUT) can be used to connect the BLTouch probe sensor.

\n

The right most connector (OUT) is therefore equivalent to the endstop pin, and is (most probably) an active low input, using a pull-up resistor either on the board or internal to the MCU - even though it is labelled as OUT.

\n

This leaves the remaining connector (the three left most pins) for the BLTouch motor/servo. Therefore, obviously, the IN pin is actually an output used to actuate the motor/servo.

\n

Note: As dandavis points out in their comment, the IO pins of the MCU IC are configurable in the firmware, and so could be reconfigured as either inputs or outputs.

\n

Alternative 3+2 wiring

\n

The much more useful and detailed tutorial, BLTouch Installation for Ender 3 with 32-bit V4.2.2 Board (linked to by towe in their comment) shows a similar set of connections, except that the 5 pin plug is replaced by the dual 3+2 wire connector, with both plugs (the 3 pin and the 2 pin) connected to the 5 pin socket on the controller board (rather than having the 2 pin going to the Z-axis endstop connector, as shown in the previous guide):

\n

\"Disconnecting

\n

\"Connecting

\n

\"Connecting

\n

Enhanced 3+2 diagram and wire swapping warning - 32-bit board only

\n

A better diagram of the 3+2 wiring configuration is shown below:

\n

(original image)

\n

\"Enhanced

\n

I found this diagram via the cautionary tale, Creality v4.2.2 board and BLtouch problem - board fried! on Reddit, which highlights the mismatch of the wiring of the various BLTouch probe/MoBo versions (the V and G pins may need to be swapped on the servo connector and must be verified before connecting and powering).

\n

This mismatch, for the 32-bit board is also highlighted in BLTouch Installation for Ender 3 with 32-bit V4.2.2 Board, under the Swap the Wiring section:

\n
\n

This BLTouch Kit is meant for 8-bit and 32-bit board, however, the default wiring is for 8-bit (green) board with adapter (hack the buzzer pin for LCD). No swapping of wires is neccessary for 8-bit board.

\n

For 32-bit board, we will need to do a minor adjustment to the wiring. This is a MUST step, as the swapped wires are VCC (+ve 5VDC) and GND (0V). If you do not swap the wires, you will damage the BLTouch and it is not under warranty.

\n
\n

(original image)

\n

\"Wire

\n

Firmware

\n

For the sake of completion, and because the guide where your image came from does not seem to be particularly well-written - the firmware will differ, depending upon where the probe sensor is connected to, i.e. the z-axis endstop connector or the right hand pins of the 5 pin socket.

\n

Either a different pre-compiled firmware binary is required or (if compiling the firmware yourself) some lines need to be uncommented in the Marlin source code, in configuration.h - which, according to the first guide, are the following lines:

\n
#define BLTOUCH\n...\n#define AUTO_BED_LEVELING_BILINEAR\n...\n#define RESTORE_LEVELING_AFTER_G28\n
\n

However, I'm not convinced by that rather overly simplistic first guide, unless the right most two pins of the 5 pin socket and the Z-axis endstop pins are hard-wired together on the board, which may be possible, but seems unlikely. I would have assumed that some pin changes are required in the firmware (but I may be wrong).

\n

Indeed, the far superior guide, How to Set Up Marlin & BLTouch for Auto-Bed Leveling, shows that those three lines are only for the bed-levelling settings and a multitude of other changes are required.

\n

Of particular note, if the Z-axis endstop is not used and the G and OUT pins are used instead, then make sure that the following line is commented out:

\n
#define Z_MIN_PROBE_USES_Z_MIN_ENDSTOP_PIN\n
\n

and uncomment

\n
#define Z_MIN_PROBE_ENDSTOP\n
\n

There are a number of other lines that need modifying, which are beyond the scope of this answer. Please refer to this guide or some other well written guide.

\n

References

\n\n" }, { "Id": "20159", "CreationDate": "2022-11-01T11:22:36.663", "Body": "

I bought motherboard version 4.2.7 and had BLTouch version 3.1

\n

Printer version is probably Creality Ender 3 Pro (although I don't know how it can matter if I got brand new motherboard from Amazon).

\n

On firmwares page I dont see appropriate BLTouch verions. May be I need no firmware upgrade? How to know?

\n", "Title": "How to determine correct firmware for Creality MB and BLTouch?", "Tags": "|creality-ender-3|firmware|bltouch|", "Answer": "

You linked to the incorrect directory, if your printer is a Creality Ender 3 Pro, the correct location is this, here you can find versions of the firmware for your printer type and controller board.

\n

Depending on the version of the touch sensor, you either have or have not got an adapter board. If you have an adaptor board you have the components in the red rectangles:

\n

\"enter

\n

If so you need this firmware file: Ender-3 Pro-4.2.7-TMC2225, Marlin2.0.1-BLTouch-V1.3.1(with adapter board), otherwise you need this firmware file: Ender-3 Pro- 4.2.7-TMC2225, Marlin2.0.1-BLTouch-V1.1.2(without adapter board).

\n

An alternative is to build your own firmware, you then no longer are bound to pre-compiled files from others, the Marlin configuration file for the Ender 3 Pro is found near the Marlin sources, see here, you probably only need to enable the BLTouch in the configuration file. There are even more solutions, you can look into TH3D software or many other forks from Marlin on GitHub.

\n" }, { "Id": "20166", "CreationDate": "2022-11-02T14:03:07.623", "Body": "

How much weaker does a PETG part become when it absorbs as much moisture as it can hold?

\n", "Title": "How much weaker does PETG become when it absorbs as much moisture as it can hold?", "Tags": "|print-material|petg|material|knowledgebase|print-strength|", "Answer": "

To my knowledge, there is no reason to expect a printed PETG object to become weaker by virtue of absorbing moisture. Unless you keep it in a controlled environment with desiccant or unless it's solid (100% infill) and so thick that it's hard for moisture to migrate deep into the interior, the printed object will rapidly reach an equilibrium with a certain moisture content. It's not something that happens slowly over a long time of exposure to excess moisture. Even an air conditioned indoor environment with 45-50% relative humidity has plenty for it to absorb.

\n

Where moisture affects the part strength is when it's present in the filament you're trying to print. This is because:

\n
    \n
  1. Water has both an incredibly high specific heat and an even more extreme phase transition energy for the transition from liquid to vapor. This means a lot of your hotend's heat will be spent boiling off the water rather than getting the plastic sufficiently melted to flow well and bond to itself. This can largely be compensated for by increasing the temperature 15-30 \u00b0C. However...

    \n
  2. \n
  3. When the water trapped in the filament boils, it produces bubbles. These either remain inside the extruded lines, making them less than solid, or they burst out, making holes in the extrusion. And while more temperature makes the melting issue above mostly go away, more temperature increases the violence of the bubbling and the corresponding damage to the print's integrity.

    \n
  4. \n
\n

Neither of these is a concern with moisture absorbed into the part after printing.

\n" }, { "Id": "20194", "CreationDate": "2022-11-09T14:43:21.747", "Body": "

I have created a Mengersponge in OpenScad which is a 3D object. I wanted to get an SVG export from it for the laser cutter but I receive this error:"Current top level object is not a 2D object." How do I select each side of this sponge to export it as SVG for laser cutting?

\n

Code:

\n
module MengerSponge(side = 270, order =3){\n    difference(){\n        cube([side,side,side],center=true);\n        MengerSponge_aux1(side, order);\n    }\n    }\nmodule MengerSponge_aux1(side, order){\n    rotations=[\n    [90,0,0],\n    [0,90,0],\n    [0,0,90],\n    ];\n    for(rotation=rotations){\n        rotate(rotation)\n    MengerSponge_aux2(side, order);\n        }\n}\n\n\n module MengerSponge_aux2(side, order){\n     if(order>0){\n        translate([0,-(side-(side/pow(3,order-1)))/2,0])\n            for(i=[0:pow(3,order-1)-1]){\n                translate([0,(side/pow(3,order-1))*i,0])\n                    translate([-(side-(side/pow(3,order-1)))/2,0,0])\n                        for(i=[0:pow(3,order-1)-1]){\n                            translate([(side/pow(3,order-1))*i,0,0])\n                                cube([(side/pow(3,order)),(side/pow(3,order)),side+1],center=true);\n            }\n        }\n        MengerSponge_aux2(side, order-1);\n     }\n     else{\n         echo("Fail");\n     }\n }\nside=270;\norder=3;\nMengerSponge(side, order);\n\n```\n
\n", "Title": "OpenSCAD ERROR: Current top level object is not a 2D object", "Tags": "|openscad|", "Answer": "

Use the projection() feature of OpenSCAD to effectively generate a plane cut. Position the cube structure in such a manner that the x/y plane intersects the desired shape to be exported. Rendering the code will then generate a 2D image. Previewing the code will generate a 1 mm thick 3D model which will "flatten" when rendered.

\n

For the image below, I did not translate the cube, merely placed the cube generation within the projection feature.

\n
projection(){\nMengerSponge(side, order);\n}\n
\n

This rendering will export to SVG as desired. Different positioning of the cube (rotate, translate) will create different patterns.

\n

\"projection

\n" }, { "Id": "20201", "CreationDate": "2022-11-11T08:44:04.783", "Body": "

Does anyone know of a method, app, technique to "dumb down", if you will, larger models to do a quick initial print to check that it fits the real world proof of concept that it was designed for.

\n

I am not talking about 0.3\u00a0mm draft printing, I already do that... But is there a way or method, to kind of "skeleton" the large print, print it in let's say, 25\u00a0% of the actual time to test its physical application. Scaling down works, in some situations, but I would prefer a 100\u00a0% model, just "simplified / skeleton'ized", still with the correct dimensions.

\n

I primarily use Fusion360 for designs.

\n", "Title": "Convert model to \"skeleton / prototype\" model for quick real world \"Proof of Concept\"", "Tags": "|fusion360|rapid-prototyping|", "Answer": "

There are techniques to remove material from an exiting model to create less dense models (like in a skeleton). One such a technique is applying Voronoi tessellation (also known as a Voronoi decomposition, a Voronoi partition, or a Dirichlet tessellation), basically the model is broken up in polyhedrons of which the ribs are kept with a certain thickness and smoothed. See e.g. the following example from https://www.voronator.com/:

\n

\"enter

\n

You can modify files yourself or use an online application to do that for you.

\n

However, you may create skeleton like models, but it is not always faster to print these models; they may have less material, but retraction may cause the printing time to be actually longer than printing the original at low density infill.

\n" }, { "Id": "20226", "CreationDate": "2022-11-18T18:00:46.553", "Body": "

I've just bought my first 3D printer, and I've started playing with it. But apparently, I have the problem that the raft won't stick. I mean, it sticks, for iteration (layer) or two, and after that it just disconnects. This is what I've checked until now:

\n\n

Now, I've came up to the idea: I would like to try to interconnect each line in one layer (iteration) with thicker dot of melted filament for better adhesion. Unfortunately, I don't know how to setup this parameter in slicing software (Cura). Does anyone eventually know where I can find this parameter in Cura? In addition, here it is the extract of the GCODE:

\n
;FLAVOR:Marlin\n;TIME:8750\n;Filament used: 3.97836m\n;Layer height: 0.15\n;MINX:45.401\n;MINY:52.901\n;MINZ:0.36\n;MAXX:154.6\n;MAXY:147.099\n;MAXZ:31.035\n;Generated with Cura_SteamEngine 5.2.1\nM140 S45\nM105\nM190 S45\nM104 S210\nM105\nM109 S210\nM82 ;absolute extrusion mode\nG28 ;Home\nG1 Z15.0 F6000 ;Move the platform down 15mm\n;Prime the extruder\nG92 E0\nG1 F200 E3\nG92 E0\nG92 E0\nG92 E0\nG1 F1500 E-6.5\n;LAYER_COUNT:199\n;LAYER:-7\nM107\nG0 F7200 X49.177 Y57.705 Z0.36\n;TYPE:SUPPORT-INTERFACE\nG1 F1500 E0\nG1 F1350 X48.447 Y58.574 E0.05124\nG1 X47.809 Y59.466 E0.10075\nG1 X47.23 Y60.43 E0.15151\nG1 X46.778 Y61.31 E0.19618\n\n...\n\n;TIME_ELAPSED:8750.646320\nG1 F1500 E835.92386\nM140 S0\nM104 S0\nM140 S0\n;Retract the filament\nG92 E1\nG1 E-1 F300\nG28 X0 Y0\nM84\nM82 ;absolute extrusion mode\nM104 S0\n;End of Gcode\n;SETTING_3 {"global_quality": "[general]\\\\nversion = 4\\\\nname = Normal #2\\\\ndefi\n;SETTING_3 nition = custom\\\\n\\\\n[metadata]\\\\ntype = quality_changes\\\\nquality_ty\n;SETTING_3 pe = fast\\\\nsetting_version = 20\\\\n\\\\n[values]\\\\nadhesion_type = raft\n;SETTING_3 \\\\nmaterial_bed_temperature = 45.0\\\\n\\\\n", "extruder_quality": ["[gen\n;SETTING_3 eral]\\\\nversion = 4\\\\nname = Normal #2\\\\ndefinition = custom\\\\n\\\\n[me\n;SETTING_3 tadata]\\\\ntype = quality_changes\\\\nquality_type = fast\\\\nsetting_vers\n;SETTING_3 ion = 20\\\\nposition = 0\\\\n\\\\n[values]\\\\ncool_fan_enabled = False\\\\nin\n;SETTING_3 fill_pattern = grid\\\\ninfill_sparse_density = 80\\\\nmaterial_print_tem\n;SETTING_3 perature = 210.0\\\\nretraction_hop_enabled = True\\\\nspeed_print = 60.0\n;SETTING_3 \\\\n\\\\n"]}\n
\n", "Title": "Stronger connection (thicker point) between two lines", "Tags": "|ultimaker-cura|pla|adhesion|", "Answer": "

After some "experimenting", I've managed to resolve the problem. There were several contributors to the problem I had:

\n\n" }, { "Id": "20230", "CreationDate": "2022-11-20T14:33:59.757", "Body": "

When designing for 3d FDM printing, I'm wondering what is best practice for items with large overhangs which cannot have (or would be fairly impractical) support structures. Consider my following design:

\n

\"enter

\n

This item is about 7" long, and you'll notice the narrow slot running through the middle of it. The slot is 0.100" wide.

\n

I was about to print this when I suddenly realized printing it would be a problem. So I started messing around with painting on supports, but since something slides in this slot, I would have to do some extensive cleanup to make sure it's smooth.

\n

What I ended up doing was splitting the body at the start of the overhang, and placing some alignment pins in the body (with corresponding holes in the upper portion of the body. e.g.

\n

\"enter

\n

I will print the top and bottom separately and glue them together.

\n

My question is whether this is my only option or if I'm missing something in the design process which might work better? This design is in Fusion360, in case that matters when discussing some specific tools that are available which I'm unaware of.

\n", "Title": "Design Considerations for Large Overhangs", "Tags": "|3d-design|support-structures|fusion360|", "Answer": "

While slicing into slabs and the use of alignment pins is a perfectly viable solution, including the alignment features organically into a single print can make assembly faster. In this case, the model itself acts as an alignment feature or might even act as the clamp, at the cost of creating a more complex cut. An additional benefit of a more complex cut is the elimination of separate alignment pins and getting the glue surfaces away from parts that need a critical dimension

\n

As an example, I used an approximation of your model and shifted the cut layer around some (more complex) to get a natural left-right alignment feature from the deep cut, and then added a high "peak" on the model to act as a front-back alignment feature. The yellow faces ensure alignment in this case, and the red faces are main glue faces.

\n

\"enter

\n

Also, keep in mind point orientation can alter things a lot: printing on the "back" as we both modeled, the slot needs support. However, there is an orientation that remove the need to support the slot's top and instead only requires support in the pocket, possibly eliminating the need for a two-part print at the cost of needing to remove the support in the area marked red, but which, as a pocket without undercuts, should be decently easy:

\n

\"enter

\n" }, { "Id": "20233", "CreationDate": "2022-11-20T17:18:53.757", "Body": "

I have an Ender 3 Pro with the direct drive upgrade, aluminum extruder (The same problem happened with the plastic one), double-sided bed default black on one side and PEI on the other (Happens with both), aluminum adjustment knobs for bed with better springs, and upgraded spool holder with ball bearings.

\n

Things I have done to try to fix this:

\n\n

Those all failed and I have no idea what this is or how to get rid of it, please help.

\n

Settings:

\n\n

I'm afraid there is literally nothing else I could possibly do at this point except replace/tune mechanical parts, which I have tried.

\n

\"Photo

\n", "Title": "Ender 3 Pro Direct Drive weird/small blobs?", "Tags": "|creality-ender-3|print-quality|calibration|direct-drive|", "Answer": "

I bought a TL smoother which fixed the problem!

\n" }, { "Id": "20245", "CreationDate": "2022-11-24T19:20:51.553", "Body": "

Please reference the following page on Thingiverse:\nhttps://www.thingiverse.com/thing:5600770

\n

This is an insert for a bandsaw (the replaceable part that the blade passes through in the middle of the bandsaw's table), with holes to allow for dust to fall down and also be pulled down by airflow from a shop vacuum or dust collector.

\n

I am modeling my own version of this with my specific dimensions, but I thought I'd ask if it would be easier for the 3d printer to do round holes instead of the hexagonal ones.

\n

(For reference, the printer that I have access to is a Prusa i3 MK3S+, with a 0.4 mm nozzle).

\n

Edit: The fluidity of movement being better for circles is what I was expecting. When doing a circle, each small segment of straight line is only slightly different than the previous one, whereas the hexagonal hole would require abrupt changes. In my case, the final appearance is not critical since it's just to let dust fall through. The version with circular holes might be a bit stronger as it removes a bit less material. I used OpenSCAD to create the model, since my geometry is pretty straightforward. I was starting to mess around with FreeCAD (out of curiosity) but then I remembered that in OpenSCAD I used the trick of 'facet numbers = 6' to get the hexagons, so I just changed those to 60 (thank goodness for being able to define and use variables) to have the same placement of holes.\n\"Pic

\n

The printer isn't mine, it's in a public library and I don't want to abuse it so I will only print the one with circular holes.

\n", "Title": "Vertical holes: round vs hexagonal", "Tags": "|prusa-i3|", "Answer": "

The g-code that drives a 3D printer, with some exceptions, converts circles to multiple segments of straight lines, very short straight lines. There is going to be a greater amount of math to be performed on many circles, while one could consider that if the hexagonal segments are sliced in a suitable manner, the result is going to be more linear, with lower demands in the mathematics aspect.

\n

That's one aspect of the difference. Another aspect is the fluidity of movement. You may get a smoother extrusion with the circles as the travel distance versus speed is going to be more uniform in creating a circle, while creating a hexagon will result in a speed change at the corner.

\n

It's only one person's opinion, but I feel that it's half of one, six dozen of the other. You get a trade-off from the processing load with a hexagon, but you might get a better appearance with the circles.

\n

If you are creating your own design (good!), would it be more difficult to create one of each? What program are you using to create this work?

\n" }, { "Id": "20258", "CreationDate": "2022-11-28T02:04:34.673", "Body": "

I'm a student artist trying to use 3D printing in my practice a bit. I'm working with liquid light, which is essentially a photosensitive liquid that you can apply to surfaces and later develop in the darkroom. I'm hoping to do this on 3D models that I print and was looking for advice on a good filament to print with that would absorb the liquid light. The manufacturer of the liquid recommends using a semi-gloss or glossy clear polyurethane to treat nonabsorbent surfaces, but I was hoping to avoid this. Anything would be appreciated and helpful!

\n", "Title": "3D printing filament that can absorb photosensitive liquid", "Tags": "|filament|filament-choice|", "Answer": "

PrusaSlicer (free, multi-platform) supports a feature known as fuzzy skin. Text below from linked page. Additionally, Cura slicer and Super Slicer also support this feature.

\n
\n

The\u00a0Fuzzy skin feature lets you create a rough fiber-like texture on\nthe sides of your models. If enabled, the perimeter will be resampled\nwith a random step size and each new sample point will be shifted\ninside or outside of the perimeter by a random length limited by the\nFuzzy skin thickness. This simple algorithm produces surprisingly nice\nresults suitable for tool handles or just to give the print surface a\nnew interesting look or to hide print imprecisions. You can also use\nmodifiers to apply fuzzy skin only to a portion of your model.

\n
\n

This would result in a roughened surface, if the right parameters were used, allowing surface tension to secure itself while the "outward" or fuzzy portion might prevent shearing or peeling.

\n" }, { "Id": "20263", "CreationDate": "2022-11-28T15:48:48.973", "Body": "

I have an Elegoo Mars.

\n

I have printed a figure in Resin, but after some months I found this - the figure is as if it is not completely cured. The base is open (see picture below). it wasn't open before!

\n

It was printed a month ago, I am not precisely sure. I think I only machine cured and sunlight.

\n

How is this possible?

\n

\"Base

\n", "Title": "Old printed object deformed over time", "Tags": "|resin|elegoo-mars|", "Answer": "

You should probably cure more, but curing should not be complete during print, or the layer will not stick properly.

\n

Also, when you cure with UV light or the sun after the print is completed, you can cure only up to a certain depth, likely around a couple of millimetres.

\n

If your object was massive and you didn't "hollow it" then some resin got trapped inside it and it didn't cure properly initially. It may slowly cure later, but by the time the object cracks due to stress, some resin may still be liquid enough to cause the issue.

\n" }, { "Id": "20272", "CreationDate": "2022-11-30T13:11:23.057", "Body": "

I would like to print an object on an Elegoo Mars 3, object which is generated layer by layer via other means.

\n

So as input I have a series of BMP or PNG black and white images, one for layer, and I need to somehow assemble them to obtain a CTB printable file.

\n

The solution can either involve generating a 3D model to be loaded in Chitubox, or it can involve generating directly a CTB file.

\n

How can do that?

\n", "Title": "How to generate a file for resin printing from layer by layer images?", "Tags": "|slicing|resin|", "Answer": "

You can do this with the open-source tool UVtools. I don't know if there is a built-in method but you can certainly achieve it by writing a short script.

\n

If you don't find the option in the menu, you can ask in the forum. Tiago, the father of UVTools, is a very kind and knowledgeable person.

\n" }, { "Id": "20277", "CreationDate": "2022-12-02T16:33:17.783", "Body": "

I replaced my hotend's thoat and I made a mistake I hope it's not fatal for the new install. After reassembling the hotend I left a marging of about 5mm of the throat out of the heatsink. I noticed something was off when leveling the bed and then confirmed it after seeing more underextrusion artifacts than expected on a test print. I fixed it but now I'm worried about the debris left by the pocket made when I missed the throat depth (see pic below)

\n

\"throat

\n

The material used was PLA so that'll be most of the debris I guess. Is it that bad? I'm printing a second benchy right now which looks OK, but I'm worried about possible clogging in the future. This is an all-metal thoat, by the way

\n", "Title": "Missed hotend's thoat depth during install, is it too bad?", "Tags": "|hotend|", "Answer": "

It\u2019s probably fine.

\n

If you have it tightened down all the way now, you\u2019ve squished most of the plastic out. You might burn up some plastic if your block gets too hot, but you could easily do that anyway. You might also have some issues with cold pulling and cold-ish unloading, but I wouldn\u2019t expect it.

\n

If you\u2019re worried, your only recourse is to remove the nozzle and clean it out, probably while hot. Q-tips work fairly well for that. Unless you\u2019re doing something fancier than PLA though, I think you\u2019ll be fine.

\n" }, { "Id": "20288", "CreationDate": "2022-12-06T02:07:30.243", "Body": "

I bought a 12V 33.3A switching power supply for my DIY 3d printer, it looks like this:\n\"enter

\n

I connect all of the L, N, EGND to outlet. When I flashing firmware, I got electrical shocks several times from SD slot and screen in one hour. It is so frequent so I do not think it is static electric discharge.

\n

When measuring around using AC voltage mode of multimeter, I found touching one probe and another probe connected to DC+ or DC-, it measures 3-4 volts. When I measure AC voltage between EGND and DC output, it reads around 28-30 volts. Is this why I got electric shocks? Should I connect DC- to EGND dirctly to prevent this?

\n", "Title": "Do you ground your motherboard and screen?", "Tags": "|grounding|", "Answer": "

TL; DR

\n

You could try, powering off and connecting the V- to the AC Earth (either with, or without, a 100\u00a0\u03a9 resistor - although the use of resistor would be preferable).

\n

\"Connect

\n

Then switch the PSU on to see if:

\n\n

Personally, this is what I would try... and then get a replacement PSU anyway, when funds allow.

\n
\n

Notes

\n

It's obviously a dodgy earth/ground but without seeing inside, it might be difficult to determine exactly what the issue is.

\n

Some of these LED power supplies are notoriously badly fabricated and there may be a poorly soldered connection inside, loose conenction, or a faulty part.

\n

It is probably easily remedied, if you know what to look for... but not so easily fixed if you don't. I've had a lot of these LED PSUs in the past and some are fixable, some not. Normally the output faisl and just the power transistors need replacing. I haven't come across one which shocks me yet, though, so I couldn't say what the issue is (likely to be), apart from a bad ground.

\n

If you think you could manage it, open it and check for bad ground/earth connections. If you don't feel confident, then, seeing as they are relatively cheap so, maybe obtain another PSU from a different source/manufacturer and try that one instead.

\n

If you do open it up, remember to disconnect from the mains supply..!

\n

To be honest, it sounds like a dodgy unit, but... As a sanity check: Does the wall socket that you are plugging into actually have an Earth connection. I have a rented flat (in SE Asia), that has "three-hole" wall sockets, but the earth isn't actually earthed, and I'm constantly getting shocks from everything. As a shoddy workaround, you could connect the earth to the plumbing (if you have metal pipework), to get a good earth... However, seeing as the unit is detective, then this could result in a shocking shower... so not advised!!!.

\n

I can't help thinking that this could be quite a serious issue, and that (in the absense of a definitive answer here) maybe you should ask, or look, on SE.Electrical Engineering.

\n

Further reading

\n

Should I ground the DC output negative terminal to the chasis on my 12 volt project?, in particular, this answer (but all of the answers are both informative and worth a read).

\n" }, { "Id": "20292", "CreationDate": "2022-12-06T11:09:21.350", "Body": "

I recently picked up a second hand/defective 'Renkforce RF100' printer. A pretty obscure German brand. The problem with it was: broken hotend fan cable which caused clogging that the previous user was unable to fix. I have thoroughly cleaned the hotend, rewired the fan and now it works again.

\n

The mainboard it used is not very well known, designated as MPX.3 which seems similar to this HICTOP brand RAMPS 1.4 board. Renkforce's documentation is very sparse. Through some digging I found a GitHub repo with some customizable Marlin firmware and this diagram: \"RF100

\n

No real circuit diagram to be found for this specific Renkforce board, however. It does look similar to the MKS Base 1.0 board. As you can see, it has support for up to two extruders, a fan and a heated bed ("EEFB"). Only one extruder (E0, labeled 5 on the picture above) is in use on this printer.

\n
\n

Here is where I need some advice:

\n

I would like to add a part cooling fan to this printer. Right now, only the hotend fan (40\u00a0mm, 12\u00a0V 0.15\u00a0A) is connected to the 'Fan' (labeled 7 in the picture above) header on the board and it will run at 100\u00a0% as soon as the hotend reaches >50\u00a0\u00b0C. Now, I could of course wire the hotend and parts cooling fans together, but I would like to have separate control over the parts cooling fan, in order to only run it at specific times (e.g. ramp it up after the first few layers are printed).

\n

My question: Could I, for instance, just screw the parts cooling fan leads into the spare extruder terminal (E1) and re-program the firmware to let it know that that pin (D7, labeled 6 on the picture above) is a FAN_PIN? Which I could then ideally address/control with the G-code M106. In a sense I would repurpose the E1 terminal for a fan - do I need to worry about (fire) safety here? Since it's an extruder terminal? Or is that terminal "simply" a 12\u00a0V connection that I can connect to whatever? Do I need to add a MOSFET next to the terminal? Or am I overthinking this and should I use another position / pin on the board? This is really the limit of my electronics knowledge right now and safety is a primary concern. Any input is appreciated!

\n

Edit for future reference: this GitHub repo has a modern (at the time of writing) version of Marlin suitable for this printer.

\n", "Title": "Repurpose extruder pin(s) for a parts cooling fan?", "Tags": "|marlin|ramps-1.4|electronics|print-fan|", "Answer": "
\n

In a sense I would repurpose the E1 terminal for a fan - do I need to worry about (fire) safety here?

\n
\n

No, fans do not use much power, heater cartridges do.

\n

Considering the fifth stepper driver is not installed, this board is not equipped for using the E1 heater port/terminal in its current state.

\n

E.g, the MOSFET for the E1 heater is not installed (E0 and the bed MOSFET are installed):

\n

\"enter

\n
\n

Or is that terminal "simply" a 12\u00a0V connection that I can connect to whatever?

\n
\n

No, it is a PWM controlled terminal by the microprocessor, however, the VCC and GND of the MOSFET connection holes should be connected, so will deliver 12\u00a0V.

\n
\n

Do I need to add a MOSFET next to the terminal?

\n
\n

Yes, if you want the firmware to control the speed.

\n

The chance that the microprocessor is connected to the MOSFET gate port is quite big. I don't think that this board has missing traces from the processor to the peripherals.

\n

So, you could repurpose the pin that is dedicated to driving the E1 MOSFET, but, you should not run the fan directly from that pin. You should solder a MOSFET or add an external MOSFET module to protect the processor of over-current.

\n

Changes to the firmware are simple, you need to add the correct port to the functioning of the fan, you may find some help in question Controlling more fans with RAMPS board or here How to use second hotend for bed heating? (this may be helpful to modify the sources, or ask another question).

\n" }, { "Id": "20303", "CreationDate": "2022-12-08T10:12:16.377", "Body": "

Is it safe to give a 3D printed toy (printed using food-safe filaments) to a toddler?\nAre there any recommendations/studies on what is considered safe?

\n", "Title": "Is it safe to give 3D printed toys to a toddler?", "Tags": "|filament-choice|", "Answer": "

The model election is a non-trivial issue when dealing with to a toddler but also, the extruder and hotend should be 100% free of debris from any previous, non food-safe filaments, although I don\u00b4t know how plausible that may be from a procedure point of view and I don\u00b4t know if there are any regulations with thresholds on this, but it\u00b4s not like we will be making a chemical analysis of a print to verify neither, so moving on to next point

\n

Depending on what you want to do (and your printer flexibility), you might as well have 2 sets of extruder/hotend/bed if your really want to go this route on a production scale: one for food-safe prints and the other for the rest of the filaments like ABS

\n" }, { "Id": "20316", "CreationDate": "2022-12-13T18:27:13.623", "Body": "

I've noticed there have been some strange patterns occurring in my prints lately, sort of like warping. I've attached a picture of a Benchy below and an image of a project I'm working on. But as you'll see on the Benchy, the front is messed up. Never had this issue before, it's only started occurring recently. Any ideas as to what it looks like?

\n

\"enter

\n

\"enter

\n

\"enter

\n

\"enter

\n", "Title": "Issues with my 3D prints", "Tags": "|print-quality|ultimaker-cura|filament|anet-a6|", "Answer": "

Looks like insufficient cooling. If it only just started, your part cooling fan might be clogged with dust and debris, or have a failing bearing making it spin slow and give low flow, or might have broken fins, etc.

\n

It's also possible that a difference in ambient air has made the difference. Especially if you're printing PLA on a hot bed, it's hard to get low-to-the-bed overhangs like the Benchy bow sufficiently cooled that they won't deform like this, and having the ambient temperature a few degrees higher can make the difference in not being able to.

\n

If the latter is your problem, lowering the bed temperature (and finding alternatives to heat to get your part to stick) is probably your best option. Or, get better fans.

\n" }, { "Id": "20318", "CreationDate": "2022-12-13T21:17:28.873", "Body": "

\"enter

\n

While I was setting up a new Ender 3 v2, and tuning the z-steps value, I noticed that some layers were extending beyond the others in the x and y axes. I didn't notice any pattern related to which layer, or how far from the print bed, or how often they might repeat during the same print.

\n

My current setup is OctoPrint running on a raspberry pi 3B, connected via USB to the recently-assembled Ender 3v2.

\n

My current theory is that something related to the z-axis stepping isn't lifting the print-head. It might be that the wrong number of steps are being performed, or that the screw isn't turning along with the stepper, or that there is some kind of friction or binding keeping the x-axis beam from raising.

\n", "Title": "How to I eliminate these \"squished\" layers that appear on my prints?", "Tags": "|creality-ender-3|z-axis|extrusion|", "Answer": "

If your nozzle is too close to the bed, there is not enough space between the nozzle and the bed to extrude the proper amount of filament. Excess filament is either squished upwards between individual printed lines or isn't extruded at all creating additional pressure in the hotend.

\n" }, { "Id": "20322", "CreationDate": "2022-12-14T17:37:34.800", "Body": "

This summer I bought a white PLA+ filament from Sunlu.\nAfter some trials, I found these settings in Ultimaker Cura for my Dremel 3D45:

\n

\"enter

\n

The layer thickness is 0.2 mm.\nThe objects were printed almost perfectly for my needs:

\n

\"enter

\n

Now, after 3 months, I tried to print with the same material and of course with the same parameters. Even if the infill seems quite good:

\n

\"enter

\n

the walls are terrible:

\n

\"enter

\n

I'm using a dryer before and during the printing.\nI'm lost because I cannot think what can be happened.

\n

The other variable is the environment temperature in the box: this summer was about 20 \u00b0C, now it is 12-13 \u00b0C. But given the use of the dryer and given that the Dremel is closed it should not matter.

\n

Could the filament be damaged?\nAny other idea?

\n", "Title": "Same parameters lead to very different results", "Tags": "|ultimaker-cura|pla|filament|dremel-3d45|", "Answer": "

This looks like a clogged nozzle.

\n

Clean your nozzle, and if you have a setup with a Bowden tube coming up to the nozzle, like an Ender 3, check that it is clean, and has good, strong contact with the nozzle.

\n

The low ambient temperature is not helping, but should create warping, not a total lack of material.

\n

In addition, make sure that you have a silicone sock on your heat block to protect from too much heat loss and have a more even temperature.

\n" }, { "Id": "20330", "CreationDate": "2022-12-17T16:21:32.607", "Body": "

Is there a way to get rid of the (yellow) internal support, Cura is generating for overhangs?

\n

\"Top/Bottom\n\"Top/Bottom

\n

I am already printing with 3 walls, so there should be no additional benefit in this case and it is harming my print.

\n

(Reason: I want to print this in silk PLA, which is very temperature sensitive. These overhangs tend to curl up, due to the additional material and heating. Adding more part cooling will hurt the layer adhesion dramatically.)

\n", "Title": "In Cura remove Top/Bottom on overhang", "Tags": "|ultimaker-cura|", "Answer": "

Skin Removal Width should be the setting you are looking for.

\n

\"enter

\n

PrusaSlicer's equivalent is Ensure vertical shell thickness.

\n" }, { "Id": "20333", "CreationDate": "2022-12-18T00:11:21.240", "Body": "

My Ender 3v2 stops mid-print; every print. The head just stops moving. The nozzle and bed temperatures are still on.

\n

I then can cancel and start the job but it's stopping again, even when using totally different G-code files.

\n

I'm using OctoPrint and didn't change anything lately. This never happened before.

\n

Any idea why this is happening?

\n", "Title": "Ender 3v2 stops mid-print", "Tags": "|creality-ender-3|", "Answer": "

I reinstalled the firmware and also reinstalled octoprint. Since then everything seems to be printing fine.

\n" }, { "Id": "20355", "CreationDate": "2022-12-21T06:54:15.640", "Body": "

My Dremel 3D45 stopped the extrusion. I removed the nozzle thinking about it was clogged. But still it didn't solve. Also the nozzle was harder to remove than usual.

\n

So I removed the bottom cover and I found this:

\n

\"enter

\n

\"enter

\n

It seems the heater is completely burned and messed up.\nUnfortunately the replacement parts are hard to find.

\n

I'm wondering what could have caused this damage.\nI didn't run the nozzle over 220 \u00b0C (usually 195 \u00b0C) and the printer has less than 250 hours of working time.

\n

It was just a defect? Or could I have prevented it?

\n", "Title": "What could have caused this damage?", "Tags": "|troubleshooting|extruder|nozzle|dremel-3d45|", "Answer": "

As you say that the nozzle removal was harder than normal it can be concluded that it has been removed before.

\n

Looking at the hotend when new,\n\"enter\nit evidently looks as if filament has been entrapped onto and on top of the heater block.

\n

It could be that in a prior attempt to remove or replace the nozzle, the heat break has come loose or un-tightened as such that molten filament leaked on top of the heater block.

\n

This hypothesis is more likely in this case than filament that has been ricochetted back from the nozzle opening back to the cavity containing the hot end (I recently experienced this latter issue on an Ultimaker destroying the core...).

\n" }, { "Id": "20357", "CreationDate": "2022-12-21T11:28:12.883", "Body": "

Recently, I've installed a new BIGTREETECH SKR mini E3 V3.0 board on Creality Ender-5. Previously, I was using SKR mini E3 V1.2 board, but it broke down and I had to replace it. I connected new one exactly like the previous board and calibrated the printer. Everything seemed to be working properly.

\n

Now, I'm getting unexpected M122 errors while printing from OctoPrint:

\n

\"M122

\n

This is OctoPrint's console output:

\n
Recv: ok\nRecv: Error:checksum mismatch, Last Line: 30869\nRecv: Resend: 30870\nShould resend line 30870 but no sufficient history is available, can't resend\nChanging monitoring state from "Printing" to "Error"\nSend: G1 X109.711 Y118.883 E.02691\nSend: M112\nSend: N31035 M112*37\nSend: N31036 M104 T0 S0*38\nSend: N31037 M140 S0*99\nChanging monitoring state from "Error" to "Offline after error"\nConnection closed, closing down monitor\nClosing down send loop\n
\n

This checksum mismatch error appears randomly at different stages of printing, but always around 30000 line.

\n
\n

What is my setup?

\n\n

I customized the firmware from BIGTREETECH official repository for this board. This problem also occurs, when I upload official firmware for this board. OctoPrint is connected to the board via GPIO pins - that's how it was connected before and I have no way to connect it via USB cable.

\n
\n

What have I done to resolve this issue?

\n\n

I suppose that the problem is in the connection between the printer and OctoPrint. What can cause this error to happen?

\n", "Title": "What causes checksum mismatch error while printing from OctoPrint?", "Tags": "|octoprint|skr-mini-e3|serial-connection|", "Answer": "

According to Marlin docs, these checksum mismatch errors were due to the baud rate being set incorrectly.

\n
\n

The serial communication speed of the printer should be as fast as it can manage without generating errors. In most cases 115200 gives a good balance between speed and stability. Start with 250000 and only go lower if \u201cline number\u201d and \u201cchecksum\u201d errors start to appear.

\n
\n

Lowering baud rate solved the problem.

\n" }, { "Id": "20365", "CreationDate": "2022-12-23T16:04:02.453", "Body": "

How to use organic supports in PrusaSlicer or Cura? I heard that it is possible, but I can\u2019t find this option in these slicers.

\n", "Title": "How to use organic supports in the slicer?", "Tags": "|ultimaker-cura|support-structures|prusaslicer|", "Answer": "

As far as I know, PrusaSlicer organic tree support isn't available to the public yet. PrusaSlicer is rumored to use the same code as the Pull request made by Thomas Rahm on his Cura fork who started working on this code around 2020.

\n

The "organic" tree support contributed by Thomas Rahm is also available in the 5.3.0-alpha+xmas Cura version in Cura, which showcases that new tree support. You can find more information in the article Special alpha: new (Xmas) tree supports for UltiMaker Cura

\n

Full disclosure I'm one of the Cura developers.

\n" }, { "Id": "20373", "CreationDate": "2022-12-25T00:12:52.673", "Body": "

Well, I know what my critical error was, I was in a hurry and set a print running before leaving it unattended for about 8 hours; came home to a great enormous disaster.

\n

\"Photo

\n

\"Photo

\n

What on earth happened? Perhaps more importantly, is anything here salvageable, or do I just need to get a new printhead?

\n", "Title": "What happened during this print disaster, and is it salvageable?", "Tags": "|creality-ender-3|print-failure|repair|", "Answer": "

I'd say you are quite lucky, the hotend (as seen in the second photo) is fairly intact!

\n

I've had a similar problem recently, but in my case it destroyed an Ultimaker core module hotend, and salvage would have taken too much time.

\n
\n

What happened during this print disaster

\n
\n

What probably happened is that the print has become unstuck from the build plate or a part of the support is caught by the hotend blocking the deposition of filament to the print object and instead it ricocheted back against the nozzle. This is not uncommon.

\n
\n

, and is it salvageable?

\n
\n

I'd say yes. You should be able to get the silicone sock off and clean the hotend. Heating the hotend and scraping it off seems do-able from the photo you provided, it may require some time and elbow grease.

\n" }, { "Id": "20389", "CreationDate": "2022-12-28T10:04:10.580", "Body": "

I started playing with OctoPrint and I already have it configured for external access and management, using OctoEverywhere.

\n

I would like it to be an easy to use printer, because it is shared between people that don't use Cura or PrusaSlicer, so I want them to upload STL files and having a slicer built into the print server (printer is an Ender 3) for them to click a button and convert these STL files into a printable G-code. I have tried to add the printer to Cura but it says that the API connection key is invalid. Can anyone help me with this?

\n", "Title": "How can I upload STL files to OctoPrint?", "Tags": "|ultimaker-cura|octoprint|stl|", "Answer": "

The latest Octoprint web UI (as of writing, Version 1.9.3) allows for drag n' drop for file upload.

\n

Slice and export locally, then drag it over to the Web UI, or use the embedded upload button.

\n" }, { "Id": "20399", "CreationDate": "2022-12-30T09:44:21.947", "Body": "

I'm looking for a tool to modify STL files to make changes, before printing it. I know that it exists, but I don't know the name of the software.

\n

For example, I have a STL file for a ball and I want to modify it to be more egg shaped by extruding or pushing. I'm looking for something like in The Sims games, where I could edit terrains just by clicking mouse and creating holes and hills.

\n", "Title": "Tool for modifing STLs?", "Tags": "|3d-design|software|stl|", "Answer": "

Sculpting should be the tool you are looking for.

\n

You can find a lot of sculpting software on the web, including free ones. I personally use Meshmixer and Blender for simple works.

\n" }, { "Id": "20403", "CreationDate": "2023-01-01T17:46:26.217", "Body": "

I am pretty new to 3D-printing, and I have just finished assembling my Prusa i3 a few days ago.

\n

I finished up basic calibrations, which went fine, but noticed my prints were coming out with a bunch of rough lines on the sides. I looked online, and saw some things about adjusting flow rate, so I took that down from 95 to 90. I am not really sure what else to try right now. I have messed with layer height, which is at 0.2 mm now.

\n

I have double-checked belt tensions and messed around with it, but the belt status seems to be hovering around 267 pretty consistently. I did try loosening it a lot more, but got the around the same, ~270 at max, even when extremely loose. Not sure what the belt tension should be feeling like, but it seems to check out with the guides.

\n

I would appreciate any help or advice.

\n

\"Test

\n

Calibration cube printed after tensioning the belts:\n\"calibration

\n", "Title": "Prints coming out with jagged lines on the sides, rough edges", "Tags": "|prusa-i3|calibration|prusaslicer|layer-shifting|", "Answer": "

It must have been something with the G-code, or how I was slicing it. I tried printing from the files that were already loaded on the SD card when I got it and it came out much better. I then modeled a cube and sliced it up carefully in PrusaSlicer and it also came out almost perfectly. Thanks to everyone who chimed in, really appreciate it and hope this can help someone else.

\n" }, { "Id": "20406", "CreationDate": "2023-01-01T19:51:18.443", "Body": "

First time trying the EasyPrint Flex TPU filament. When printing a solid, I can see clear lines with plenty of air on both sides. This gives me the idea that it is not extruding enough filament.

\n

I've tried many tweaks:

\n\n

The last tweak makes the most apparent improvement. Printing at 25\u00a0mm/s, it's somewhat decent. There's still some air going through each layer, but much less. The result is still dissatisfactory.

\n

As this printer uses a direct extruder there shouldn't be any problem with the feeding.

\n

Can I now conclude that this is simply a crappy filament and my best bet is to buy something better? If not, then am I missing something obvious here?

\n", "Title": "TPU on Ender 3 S1 Pro only works when flow is set to 600 %", "Tags": "|creality-ender-3|ultimaker-cura|klipper|tpu|", "Answer": "

This is the Creality Sprite Extruder Pro.

\n

The tension is controlled by a bolt with a square-shaped nut that is pressing on the lever with a spring. To increase tension to the gears that feed the filament the adjustment bolt is turned clockwise.

\n

However, you have to be very careful not to turn it too far, or the bolt will unhook from the internal nut. Turning anti-clockwise in this situation does nothing.

\n

If you're lucky, like I was, it will get stuck in a position of max tension no matter what you do. If unlucky, the various parts can go into the gears and cause permanent damage.

\n

It is a most unfortunate design as there is no way to tell if you turned it too far!

\n

In my case, having it at max tension at all times caused it to under-extrude. Adding more flow alleviated the problem somewhat. I had to open everything up to see what was going on there. In my case, just putting the nut back on the bolt solved my problem.

\n

TPU is much softer than PLA so it's more sensitive to tension. While PLA can print okay at max tension, TPU cannot.

\n

\"Tension

\n" }, { "Id": "20408", "CreationDate": "2023-01-02T01:21:00.430", "Body": "

I just finished assembling my Ender 3 Pro and am having trouble with bed leveling. The extruder doesn't even come close to the bed, the z-limit stops the nozzle about 3-4\u00a0mm away.

\n

\"Extruder

\n

The leveling springs are already pretty uncompressed. Is there any way to decrease the gap? Should I be checking for a specific assembly step to see if I assembled it incorrectly somewhere?

\n", "Title": "Ender 3 Pro has too much space between extruder and bed", "Tags": "|creality-ender-3|bed-leveling|", "Answer": "

Based on your picture, I can tell that the leveling springs aren't standing straight up, rather they are leaning at an angle, making it so that full extension doesn't push the platform up as high as would be possible if they were straight. I had the exact same problem. I simply took a pen and nudged all the springs to make them stand up straight, and it solved the problem.

\n

I know it doesn't look like it will help much, but trust me it does.

\n" }, { "Id": "20419", "CreationDate": "2023-01-05T18:46:10.763", "Body": "

Is there a convention for the scale of the 3D printing STL files that are shared online and/or expected by 3-D printers?

\n

I found a model on Thingiverse that is useful for my project, a connector that must fit with other parts, so precision is important. Eyeballing, it seems that each STL unit corresponds to 1 millimeter, is this a safe/conventional assumption?

\n", "Title": "Is there a conventional scale for 3D printing models?", "Tags": "|3d-models|stl|", "Answer": "

STL files are based on coordinates rather than individual units of measurement as they're supposed to be completely unit agnostic in order to allow them to be evenly scaled on any platform, and to not be bogged down with unit translations.

\n

Typically, the units are set by whoever makes the model, and by whatever the default of the software that they use is. In most cases, creators use a one-to-one scale.

\n

So, the answer to your question is that the conventional scale is "100%". Though probably isn't the answer that you're looking for.

\n

If you print it at 100% it will be whatever size the creator intended it to be. For example, if you were printing a Warhammer miniature and it has a base attached to it, and you print it at 100% then that base will be 25mm wide, and the figure will be the exact height that the creator intended it to be. Whatever that height is.

\n

For the purpose of your question, if the part "looks" like it was created in MM then it almost certainly was.

\n

If you take that STL file and put it into your slicer, then use the default scale (100%), you can use the slicer to determine how its width or height or length in MM. Just set the scale of the model to whatever makes that dimension the same as the dimension you need it to be and the rest of the model will be scaled accordingly.

\n

Your biggest problem might actually be the material that you use, as some will shrink after printing, or shrink unevenly.

\n" }, { "Id": "20425", "CreationDate": "2023-01-06T04:55:10.233", "Body": "

This would be my first ever 3D printing\nI previously worked in Sketchup to generate simple objects, mostly floor plans and various small objects mostly for visualizing and designing things.

\n

I am now trying to print a dust collector (hose) adapter that would connect my vacuum to my table saw guard. I have the model already (looks like below), I have managed to get quotes from 3D printing service providers so apparently everything looks OK

\n

I still have doubts regarding the stability of the object during the 3D printing.\nHow can I verify if everything is OK in that respect? I plan to print at our local public library which offers free printing (you pay the materials)

\n

Here is my model, the OD at the base is 64.3\u00a0mm (2 17/32", the height is 288.1\u00a0mm):

\n

\"enter

\n

Here is the link to the file.

\n

If it matters the local public library branch has the following 3D printers which I can use:

\n\n", "Title": "How can I test if my model can be 3D printed without having a printer?", "Tags": "|support-structures|part-testing|", "Answer": "
\n

How can I test if my model can be 3D printed without having a printer?

\n
\n

Find out what printer is used and slice the print yourself using free tools like PrusaSlicer, Slic3r, Ultimaker Cura, etc..

\n

From the mentioned printers, none of these allows for the tube to be printed upright, the TAZ has the largest height of 285\u00a0mm (depending on the exact version they have), your object is 288\u00a0mm...

\n

e.g. One of my printer cannot take the object upright:\n\"enter

\n

This means that you are left to two options, cut the object into two pieces and design a coupling between the two pieces, or you need to modify the orientation of the object.

\n

Do note that orientation of the print on the plate can do a lot to print object that sometimes do not appear to fit on the plate or in the volume.

\n

E.g., you could print the tube other than upright, see below (it fits my 200x300\u00a0mm printer):

\n

\"enter

\n

This is sliced with support, but, a smaller scaled version printed fine without support. The larger version and the shallow incline of the tube need support for the full size print.

\n

Scaled version of the tube printed without supports:\n\"enter

\n

I think the best solution is to break up the part into two pieces and design an appropriate coupling. This would give you 2 parts, but the printing time and the amount of support will be minimum. The less time and the less support the higher the chance the parts print in one go without failing. It is also best to print the two parts after each other, else you will have the nozzle to retract (takes time) in between the two parts if printed together.

\n" }, { "Id": "20429", "CreationDate": "2023-01-06T21:44:26.463", "Body": "

I would like to add neodymium magnets during print. The model, which I would like to print, has fully enclosed holes made for placing magnets.

\n

\"Hole

\n

I added pause on layer (M601) in slicer. Since, this is a very long print, I would like to avoid any problems. I read that the magnet can attach to the nozzle while printing. How can I avoid this, and what else should I watch out for? How should I prepare in advance for magnets placing?

\n", "Title": "How to add magnets to 3D print mid-print?", "Tags": "|3d-models|", "Answer": "

I'll share some experience, I got while printing with magnets:

\n
    \n
  1. Firstly, check if the magnet can stick to something around the hotend, especially when using neodymium magnets. It may be necessary to replace the nozzle with a non-magnetic one.

    \n
  2. \n
  3. Apply a small amount of glue before placing the magnets. The magnet should be free of metal filings that may stick to the magnet, because the glue will not stick to the magnet properly. In this situation, the magnet will not stick to the print and may frivolously move around in the hole.

    \n

    I used a small amount of hot glue on the bottom and top of the magnet, completely surrounding it in the hole. Hot glue should be used carefully as it can deform the plastic surrounding it.

    \n
  4. \n
  5. Check magnets pole orientation. This one is important, if you want to use magnets in pairs, e.g. for parts that should connect and disconnect. Magnets should either attract or repel each other, depending on the application. Before placing the magnets, check that they are correctly positioned. You can also mark the poles first with a marker.

    \n
  6. \n
  7. Make sure the magnet fits in the hole. For this purpose, you can make a quick test print and check if the magnet fits correctly into the hole. This will save potential drilling in the printout and other unwanted problems.

    \n
  8. \n
\n" }, { "Id": "20435", "CreationDate": "2023-01-07T22:57:19.543", "Body": "

I just got an Ender 3 Pro for Christmas and, aside from firmware, I have made no changes to it whatsoever. Until yesterday, my printer was running all prints flawlessly. The only thing that changed was the filament, but I don't think that's the issue because the problem only starts to occur at a certain Z height.

\n

What I've ensured:

\n\n

What I've noticed aside from the crappy prints:

\n\n

Images of a good print and a bad print. These are the same version of the same G-code file printed at the same settings on the same printer in the same conditions. (EDIT: I removed the good print images because of a logo they contained that I am trying to limit public exposure to.)

\n

\"Side

\n", "Title": "My prints have started to turn to mush at a certain Z height", "Tags": "|creality-ender-3|print-failure|stringing|", "Answer": "

The thump and the appearance of the print are indications of underextrusion. Tracking the source of this failure can be challenging, especially considering how many factors you've included in your question.

\n

Consider to create a model of smaller or larger diameter than the existing troublesome object. Adjust the height of the model in inverse proportion to the diameter change, that is, if the diameter is smaller, increase the height.

\n

For example, if the problem is heat creep in the nozzle, it can appear at a specific duration of the print. By decreasing the diameter of the object, the failure point should appear at a higher level, more or less confirming that time is a factor, pointing to heat creep.

\n

Consider also to download and print a suitably configured temperature tower. This will allow you to reference the time factor again, but also allow you to expand the range of printing temperatures you can use for that specific color and manufacturer of filament.

\n" }, { "Id": "20444", "CreationDate": "2023-01-09T22:18:13.170", "Body": "

I haven't used my Creality Ender 3 S1 in a week. But I started it now. During auto leveling and auto-home the stepper motor for the build-plate (Y axis?) goes over the limit and a unhealthy sound is heard (the stepper pulley skips the belt).

\n

All worked well last time I used the printer. I have tried to "Disable motors" and manually moved the plate, but it did not fell in "alignment", so the noise will come e.g. with "auto-home" again.

\n

Is there a way I can reset this or may something be broken?

\n
\n

I did "Control -> Reset Configuration" and tried to use "auto-home" again, but the same problem occurred. I also tried to tighten the belt for the motor that moves the build-plate, but it did not help.

\n

It sounds like something is broken. In some way, it sounds like the stepper motor has the "wrong position", if that can happen. Perhaps there are some "encoder" that is broken?

\n

Any suggestions on how to move on from here?

\n
\n

To reproduce my problem:

\n
    \n
  1. I disable the stepper motors.
  2. \n
  3. Move the build-plate as close to the motor as possible.
  4. \n
  5. I do "Control -> Reset Configuration" to reset the printer.
  6. \n
  7. I do "auto-home". The end result of this is that the build-plate is centralized and the "level-pin" on the nozzle is in the center of the build plate.
  8. \n
  9. When I do "auto-level" now, it eventually tries to move the build-plate too far away from the motor, so the pulley skips the belt and noise is heard.
  10. \n
\n
\n

Auto home

\n

When looking at this video of Creality Ender 3 S1 doing auto home, it moves the build-plate as close to the motor as possible, before moving the build-plate to home. This does hot happen on mine.

\n

Instead, if I first put the build-plate in the center, then start the auto home. It never moves the build-plate as close to the motor as possible, it only moves the build-plate away and finally the motor pulley skips the belt.

\n", "Title": "Y-Stepper motor pulley skips the belt", "Tags": "|creality-ender-3|motor|", "Answer": "

When a stepper keeps on spinning and skipping the belt, it usually indicates that there is something wrong with the end stop. If it is not pressed, the stepper will rotate the length of the bed usually making the pulley skip the belt.

\n

Please check your Y-end stop, you find it in the image below left of the stepper (you see the cable of the endstop, just above where the sticker is being pulled off).

\n

\"enter

\n

You could try and press this switch the moment you auto home to see if it works. If it works, make sure the carriage hits the switch.

\n
\n

The updated question explains that the bed moves in opposite direction when homing, if so, this hints to a defective end stop switch or cable (or unconnected). When the end stop is defective the firmware (when properly configured and proper switch state routing is used) the printer assumes the end stop is triggered and moves away from the end stop.

\n" }, { "Id": "20448", "CreationDate": "2023-01-10T23:00:54.433", "Body": "

I know why and can understand the logic of changing E-steps for a new extruder- obviously it is related to the number of turns of the stepper.

\n

What I don't understand is how the hotend could alter the E-steps so drastically.

\n

For example in Marlin the default e-steps for the stock MK8 hotend on a Creality printer is 93 (which is always too low IME) but then if you have a MicroSwiss it sets the E-steps to 137.6

\n

How does simply changing of the hotend (keeping the same nozzle diameter) require such a high increase in the E-steps? Can someone explain the physics of it?

\n

Edit:

\n

This is what I was referring to:\n\"enter

\n

Based on your advice I did some further searching and found that Microswiss makes an extruder as well. I initially assumed that it was referring to the hotend, since the most popular all-metal replacement hotend for the Ender line is the Microswiss, since it is a direct drop-in and requires no additional mount hardware.

\n

If it is the Microswiss extruder that they are referring to- I find that odd that they would include only that one since there are perhaps a dozen popular extruder options for the Ender line and they don't list firmware e-step options for any of them.

\n", "Title": "Why do we change E-steps based on the hotend?", "Tags": "|extruder|hotend|", "Answer": "

Actually we don't.

\n

Under the assumption that the same extruder is used, the hot end doesn't matter. The question is unclear about the reuse of the same extruder. If there are different hot ends that have the extruder incorporated into the hot end design, the gearing solution used for these different extruders explain why the E-steps per mm are different. But, if the same (Bowden) extruder is used, you don't need to change the E-steps.

\n

To explain, the E-steps per mm is a property that belongs to the extruder, not the hot end. The E-steps per mm expresses how many steps need to be send to the extruder stepper to extrude a mm of filament. This filament is pushed while it is 1.75\u00a0mm or 2.85\u00a0mm. For this reason the nozzle diameter doesn't have a part in the equation either; the slicer will calculate the amount of length to extrude a volume is needed, the firmware recalculates this extrusion length (or volume if the filament area is taken into account) and expresses this with the E-steps per mm into an amount of steps for the stepper.

\n

You can calibrate your extruder by disconnecting the hot end or unmounting the nozzle (and telling the firmware to extrude at low temperature using G-code M302; you just need to make sure when 100\u00a0mm extrusion is requested, 100\u00a0mm is spit out. Any deviations from that can be fixed with the extrusion multiplier.

\n

In the early days, when extruders didn't use dual gear filament gear solutions (which many these days do now), the filament was pushed by a single gear and used a counter bearing that pressed onto the filament with a spring like mechanism. What frequently happened with the cheaper extruder solutions is that friction in the path up to the nozzle caused some slip, so it may look like more steps per mm were needed, but in fact this was just an extruder issue.

\n

Also beware of soft filament and pressure on the filament. See e.g. the following image of 2 different (blue and red) filament types or pressures on the filament sketched in the same image:

\n

\"enter

\n

This image shows 2 types of filament or pressures on the filament. You see that the softer filament (red) or more pressed on (resulting in the gear teeth digging deeper into the filament), has an effective radius of R2 while the harder (blue) or less pressed on has an effective diameter of R1. R2 is smaller than R1, so for the same amount of steps per mm less filament is extruded (if the gear makes one turn, the extrusion difference is $ E_{diff}= 2\\times\\pi\\times(R_1-R_2)$. Such effects should preferably be adjusted with the extrusion modifier in the slicer.

\n" }, { "Id": "20459", "CreationDate": "2023-01-14T11:50:38.833", "Body": "

I am trying to print an object with this cross section (the full object is just this mirrored):

\n

\"Object\"

\n

The problem is that big overhang layer. Cura slices it like this:

\n

\"Bad

\n

This is obviously going to fail. Is there any way to make Cura slice it in a spiral inwards from the outside, something like this (excuse the terrible diagram):

\n

\"Good

\n", "Title": "Print overhangs from the edges in", "Tags": "|ultimaker-cura|slicing|", "Answer": "

Here are your settings. The ones with the arrows are especially important. Just copy everything I've underlined with red and everything will work fine ;-)

\n

\"enter

\n" }, { "Id": "20463", "CreationDate": "2023-01-16T15:25:07.870", "Body": "

I have created my own tool changer for my 3D Printer but I'm having an issue with the Z-offset for each head.

\n\n

I have used the offset wizard to get the Z-offset of each head from the bed and then I tried to use the following G-code (M851 Z-4.3) in the Extruder Start G-Code and I have done this for each head, I can see it has changed on the screen in the configuration menu but it looks like it has had no effect as the print has already started.

\n

What is the best way for me to set each offset for all my extruders?

\n", "Title": "Z-Offset Multiple Heads on Tool Change", "Tags": "|marlin|ultimaker-cura|extruder|z-probe|", "Answer": "

There is a specific code to set the offset between the various tools. This G-code is called M218 - Set Hotend Offset.

\n

\"enter

\n" }, { "Id": "20466", "CreationDate": "2023-01-17T08:25:03.470", "Body": "

I have recently purchased an Anycubic Kobra, and the first two prints worked out fine. On the third print, the Y-axis belt snapped, so I contacted Anycubic support and they gave me a replacement one. I did a few test owl prints, but there was major Y-axis shifting on the prints.

\n

\"Images

\n

After I replaced the belt, I made sure that they were as tight as possible, but there was some slack, but I couldn't tighten them further. There is only shifting in the Y-axis, but it mostly only shifts in one direction (towards front).

\n

What other reasons would cause a Y-axis shift like this? Is this due to incorrect belt replacement, as there was no shifting prior to the belt snap?

\n", "Title": "Major Y-axis skipping (Anycubic Kobra)", "Tags": "|troubleshooting|layer-shifting|", "Answer": "

I found a solution to this layer shifting. Even though the belt was too loose, I managed to use a rubber band to tighten the belt slightly. Now to prints go off without a hitch. Thanks for your help.

\n" }, { "Id": "20470", "CreationDate": "2023-01-18T00:21:41.403", "Body": "

Suddenly, I can't seem to print anything on my Flashforge Adventurer 4. No matter what I try, the raft looks like this. I'm re-calibrated and tried all kinds of different heat/speed settings. What am I doing wrong?

\n

\"enter

\n", "Title": "raft balling up", "Tags": "|print-quality|rafts|flashforge-adventurer-4|", "Answer": "

Thank you for all the suggestions.\nMy problem ended up being that the filament tube was loose (not screwed in all the way) into the extruder head. Eventually, the printer started giving me errors about irregular printhead readings and to check it. The print head was loose so I took the whole thing apart and put it back together tightly. My next print was perfect.

\n" }, { "Id": "20472", "CreationDate": "2023-01-18T10:01:15.700", "Body": "

I'm using PrusaSlicer 2.5.0 and printing the sliced G-code on a Creality CR-6 SE via a USB-connected OctoPrint/OctoPi. As you can see, the slicer sets the temperature by G-code commands in two steps (final temperature is 205\u00a0\u00b0C). This seems strange to me, as the printer can go directly to the final temperature. Also, this screenshot shows the case where I'm switching filament, therefore I have to heat up the extruder and when I start printing, the hotend cools for some time, because the target temperature is way lower set by PrusaSlicer.

\n

Is there a setting for configuring this in PrusaSlicer?\nThese are the first few lines in the G-code file:

\n
; generated by PrusaSlicer 2.5.0+win64 on 2023-01-18 at 12:15:56 UTC\n\n; \n\n; external perimeters extrusion width = 0.42mm\n; perimeters extrusion width = 0.44mm\n; infill extrusion width = 0.44mm\n; solid infill extrusion width = 0.44mm\n; top infill extrusion width = 0.40mm\n; support material extrusion width = 0.36mm\n; first layer extrusion width = 0.42mm\n\nM201 X500 Y500 Z100 E5000 ; sets maximum accelerations, mm/sec^2\nM203 X500 Y500 Z10 E60 ; sets maximum feedrates, mm / sec\nM204 P500 R1000 T500 ; sets acceleration (P, T) and retract acceleration (R), mm/sec^2\nM205 X8.00 Y8.00 Z0.40 E5.00 ; sets the jerk limits, mm/sec\nM205 S0 T0 ; sets the minimum extruding and travel feed rate, mm/sec\nM107\n;TYPE:Custom\nG90 ; use absolute coordinates\nM83 ; extruder relative mode\nM109 S180 ; set temporary nozzle temp to prevent oozing during homing\nM190 S70 ; wait for bed temp to stabilize\nM140 S70 ; set final bed temp\nG28 ; home all axis\nG1 Z50 F240\nG1 X2.0 Y10 F3000\nM104 S230 ; set final nozzle temp\nM109 S230 ; wait for nozzle temp to stabilize\nG1 Z0.28 F240\nG92 E0\nG1 X2.0 Y140 E10 F1500 ; prime the nozzle\nG1 X2.3 Y140 F5000\nG92 E0\nG1 X2.3 Y10 E10 F1200 ; prime the nozzle\nG92 E0\nG21 ; set units to millimeters\nG90 ; use absolute coordinates\nM83 ; use relative distances for extrusion\n; Filament gcode\n
\n", "Title": "Why does Prusaslicer set the final temperature in two steps?", "Tags": "|octoprint|prusaslicer|creality-cr-6|", "Answer": "

Since the bed usually takes longer to heat than the nozzle, the nozzle is in a pre-warmup stage so that it doesn't ooze filament waiting for the bed while homing and/or finding the Z limit.

\n

Once the bed is warmed up and the Z limit is reached the nozzle heats up to the final temperature and the print starts.

\n

If you go to the Printer Settings tab of Prusaslicer and click on Custom G-Code, you'll see the initial temp in the Start G-Code section. Mine even says:

\n
M104 S150 ; set temporary nozzle temp to prevent oozing during homing\n
\n

A temperature of 150\u00a0\u00b0C is low enough that the filament doesn't melt but close enough to the final temperature so that you're not waiting for a full heat cycle.

\n" }, { "Id": "20480", "CreationDate": "2023-01-20T18:12:11.107", "Body": "

I'm thinking about trying to use a 12\u00a0V power supply for the board and 24\u00a0V for the hotend and heatbed triggered by some MOSFETs modules like this Power Expansion Board Heating Controller MOSFET High Current Load Module 25A 12V or 24V for 3D Printer Parts.

\n

Has something like this worked for anyone? I'm wondering if for triggering 24\u00a0V voltage the module needs also 24\u00a0V input or 12\u00a0V will do.

\n

Maybe I'll start from RAMPS just to kick off, in the long run I thought about Monster8 (I wanted to check out 3-4 motors autolevelling someday) on 12 V but I have also a spare 24 V PSU but just 300W, so I wanted to try a double PSU build and 24 V only for 310x310 240-260W heated bed.

\n

Note: I intend using a heated bed designed for 24 V.

\n", "Title": "Controlling MOSFET with 12 V from board to heat bed with 24 V PSU", "Tags": "|heated-bed|mosfet|", "Answer": "

You can certainly use a 24\u00a0V PSU for the heatbed, controlled from a 12\u00a0V board (i.e. RAMPS) - assuming that the heatbed is either designed for 24 V or is of a type that accepts dual supply voltage (be sure to use the correct input).

\n

You have three choices:

\n\n

Modifying the RAMPS

\n

Assuming that you are actually using a RAMPS board:

\n

From RepRapWiki - RAMPS 24V:

\n
\n

Modifying for 24v heated bed

\n

You have to replace the 11 amp polyfuse, MF-R1100, with something else. This device is only rated for 16v. It will burn out and catch fire on 24volts. Most simply replace the polyfuse with a thick wire. Then wire in a 24v fuse with a inline fuse holder on the heated bed cables. Wire up 24 volt supply to the top two 11amp power sockets and its completed.

\n

You can also use an external MOSFET with a separate 24v supply without any changes.

\n
\n

Thus, you can use 24 V, for just the heatbed, on a standard RAMPS board directly. However to do so, you need to modify the board and replace the fuse, as the standard fuse is only rated for 16 V and the current rating would be too high for a 24 V supply.

\n

Just swap out the 11 amp polyfuse, MF-R1100, and replace it with a more suitably rated fuse. Alternatively, just replace the MF-R1100 with a thick wire and use an additional inline fuse holder with a 12/24\u00a0V fuse, and reduce the current rating of the fuse by half, i.e. 5\u00a0A.

\n

As a 24 V heatbed will actually use less current than a 12 V heatbed, this is perfectly safe. In addition, the heatbed circuit on the RAMPS is isolated from the rest of the RAMPS control circuit, so there is no need to worry about using the two different voltages on the same board.

\n

SSR

\n

However, instead of a MOSFET, you might be better off using a relay, such as a FOTEK SSR-100 DD (which is a solid state relay module). Using a relay will isolate the two circuits. Nevertheless, you need to be aware that some (a lot?) of these Fostek SSRs are relabelled - that is to say that a 5 A SSR has the label replaced with one that says 10 A of 15 A, and the unscrupulous reseller bumps up the price accordingly. So, maybe choose a different brand of relay.

\n

MOSFET

\n

Assuming that you do not have a RAMPS board:

\n

Wire up an external PCB with an appropriate MOSFET. For instance, you could just use the same MOSFET as the RAMPS board uses, which is one of these three (from RepRapWiki - RAMPS 1.4 - BOM):

\n\n

From Re: best ramps 1.4 headbed mosfet upgrade:

\n
\n

IRLB8743pbf and IRLB3034pbf are good choices. IRF3205 and P55NF06 are bad choices, because they don't have guaranteed performance at 4.5V gate drive.

\n
\n

See also RAMPS 1.4 and external mosfets.

\n" }, { "Id": "20483", "CreationDate": "2023-01-20T22:26:56.053", "Body": "

I'm building a product which requires a dowel pin inserted into a plastic piece. I'm planning to 3D print the plastic piece with nylon (must be nylon for biocompatibility), but the tolerances won't be sufficient to ensure a consistent press-fit for the pin. To get around this, I plan to undersize the hole, and then ream it out with a reamer on the drill press. There will be 1000 pieces on this run. Any flaws you can see with this design?

\n", "Title": "Inserting steel dowel pin into hole with interference fit", "Tags": "|3d-design|", "Answer": "

It is not possible to insert a steel dowel into a hole of a plastic workpiece with a press fit: at least not for long. Tremendous force is exerted on the plastic by the steel, and the plastic will soon begin to deform in a process called cold flow or creep.

\n" }, { "Id": "20486", "CreationDate": "2023-01-22T00:51:47.217", "Body": "

I'm experiencing an issue with the auto homing where the Z-axis will ignore the endstop when I use the auto home button in Pronterface. Strangely, homing the axes individually I don't seem to have any issues, but when I use G28 or home the axes sequentially I observe the Z-axis motors to continue moving the gantry down despite definitely having hit the Z-endstop. Below is a link to a video demonstrating this issue:

\n

\r\n \r\n

\n

Triggering the Z-endstop manually (with my finger) the M119 command shows "TRIGGERED". I tested, starting from a high Z position and descending towards the endstop, manually triggering the endstop (with my finger) and I observed the Z-axis motors stop.

\n

I used Marlin Bugfix 2.0.9.3.x preconfigured release from BTT's GitHub corresponding to this board (BTT SKR 3 EZ), only changing things like motor direction and build area so as to correspond to the printer.

\n

I am able to copy/paste the configuration.h file if that will help.

\n", "Title": "Auto Homing issue (Z-axis ignores endstop but individually homing each axis there is no problem)", "Tags": "|marlin|homing|endstop|bug|", "Answer": "

For anyone looking I figured out what the issue was. Turns out that I had miswired the JST connectors coming from the switch to the board, having not double-checked and proceeding to connect wires to the board from the switch based on the color of the wires and my assumptions (black to ground, red to VCC, other color to signal). Actually for my switches black was VCC, red was signal, and the other color was ground. Thankfully it seems the board isn't worse for wear despite my errors in wiring previously, and now the printer is functional.

\n" }, { "Id": "20510", "CreationDate": "2023-01-29T11:19:52.507", "Body": "

I recently bought a Anycubic Vyper which prints overall not bad but has a real nasty issue with stringing.

\n

I tried out several temperatures, settings for retract distances and retraction speed but nothing seems to really work. All of my filament is new and stringing occurs with different brands so it does not seem to be caused by the filament.

\n

When the nozzle heats up there is molten filament coming out of the nozzle even the extruder is not doing anything at all.

\n

Is there something I can try to fix the stringing apart from taking the hotend apart?

\n

Retraction tower with different retract-values:

\n

\"3D

\n

Retract test which shows the stringing:

\n

\"3D

\n", "Title": "Stringing on Anycubic Vyper", "Tags": "|stringing|anycubic-vyper|", "Answer": "

Thanks for the answers. In this special case the printer had a hardware-defect,so I had to send it back.

\n

I don't know which part in detail was defect but the hotend was switched off in the middle of the print, so either the hotend or something at the mainboard.

\n" }, { "Id": "20513", "CreationDate": "2023-01-29T23:14:59.240", "Body": "

Looking for a way to taper an imported STL model off toward the bottom (or top, if easier).\nThe same transformation would turn a straight cylinder into a cone.

\n

The intent is to make a vase like the model in the image that tapers off so it will fit as an outer layer to my clay pots.

\n

The model is symmetric around the XZ and YZ plane and only has positive z values.

\n

I assume this would require some non-linear transformation, but I'm unable to find results when googling.

\n

\"Model\"

\n", "Title": "Conical scaling in OpenSCAD or Blender", "Tags": "|openscad|", "Answer": "

To install Blender go to Blender.org and download the version for your platform.

\n

No need to install just extract and run "Blender"

\n

to enable importing stl files go to edit -> preferences
\nin the addons tab search for stl, check the box.

\n

\"enter

\n

Now import your stl:\nFile -> Import -> STL(.stl)(experimental)
\nand select your file

\n

You may need to zoom out quite a bit because of the scale.
\nDo this with the mouse wheel, while the pointer is over the view where the part is. Or with View -> frame all

\n

\"enter

\n

If the part is not selected (it has no orange outline) select it by clicking on it.
\nAnd go into "edit mode" by hitting TAB

\n

Now rotate the model so you can see the bottom plane.
\nHold the middle mouse button and drag.
\nIf all vertices are selected when you open the model, hit "a" twice quickly to deselect all.
\nSelect the bottom vertices by shift clicking them.

\n

Now comes the actual conical scaling
\nActivate "proportional editing" By hitting "o" or clicking the icon, and select linear in the drop-down menu.

\n

\"enter

\n

Scale by hitting "s" and moving the mouse but do not click yet
\n\"enter\nNow hit "Shift z" This will constrain scaling to the xy plane.
\nYou will see a circle that indicates the region of influence of the proportional edit. Adjust it to your linking with the mouse wheel, and click to confirm. Or if you want a precise ratio type for instance 0.8 and hit enter to confirm.\n\"enter

\n

TAB to exit edit mode.
\nCheck that only your object is selected. If other objects are orange, hit "a" twice to deselect all and select your object bay clicking.
\nAnd finally File -> Export -> STL
\nGive it a different name than the original check "Selection Only" and "Export stl"\n\"enter

\n" }, { "Id": "20538", "CreationDate": "2023-02-04T13:43:35.887", "Body": "

I'm producing a series of brackets to allow a laser cutter to be reliably positioned on a waste board\\spoil board in that has a measurement grid on it, and to allow a honeycomb bed to be reliably positioned within it in relation to the laser cutter.

\n

Essentially, I'm making 4 large right angle piece with mounting screws and stepped sides. Which will be screwed down to a piece of timber so that things can be fixed to them in the same position each time they're used.

\n

It has a lot of corners, and 90\u00b0 angles, and it looks like it was designed by a Brutalist architect.

\n

Will bevelling1) the corners for purely aesthetic reasons have any practical effect on the print or the printing process. For example will rounder corners make the print stronger or weaker, or will it make warping more or less likely. Will it improve or harm bed adhesion, will it have any substantial effect on print time or quality?

\n

Version 1.0

\n

\"enter

\n

I'm using an Ender 5 with PLA and a 0.2 layer height. Because that's what I have on my shelf right now, and I don't want to spend any extra money on this.

\n
\n

1) I'm using the word "bevel" because I'm using Blender to make the model (Because it's what I'm most familiar with), and you create rounded edges using the Bevel tool in Blender.

\n", "Title": "Do bevelled corners hold any practical advantages or disadvantages over square ones for blocky functional prints?", "Tags": "|print-quality|3d-models|3d-design|", "Answer": "

Bevel chamfer fillet, whatever. For correct semantics see the other excellent answer.

\n

What are advantages of rounded (or at least, less sharp) edges and corners?

\n\n" }, { "Id": "20551", "CreationDate": "2023-02-06T15:46:08.877", "Body": "

I spent way too much time trying to get webcam working with Octoprint. Since my webcam is not supported by mjpegstream and similar streamers, I had to use Motion. Finding the URL for the video stream was quite fast, but it seems no-one in the world except for the Motion's author themselves knows the URL for the stills for timelapse.

\n

All the attempts in several places of the net canter around trying to connect to http://octopi.local:8080/0/action/snapshot which correctly produces a still jpg of the camera, places it in /var/lib/motion and returns a confirmation that the image was produced to web UI, with no image itself in sight. There are workarounds - python scripts, cgi for webserver, and so on. No-one seems to have found Motion's URL that would return a still jpg.

\n", "Title": "How to set up Octoprint with Motion as camera streaming service?", "Tags": "|octoprint|", "Answer": "

For camera 0, if the hostname of your Octopi is octopi.local the default stream URI is http://octopi.local:8081/ (or whatever address your Octopi has). If you use a higher-number camera, use 8081+cam number as the port.

\n

The snapshot uri for camera 0 is http://octopi.local:8081/0/current

\n

If you use camera numbered higher, use whatever camera number you use in place of this 0 in the URL. It's especially tricky because literally every "special" URI of this format sits on 8080, the control port. This one is attached to the port of the first camera stream, regardless of which camera number you want.

\n

\"screenshot

\n" }, { "Id": "20563", "CreationDate": "2023-02-10T20:32:06.823", "Body": "

I made a successful print before this incident with my Ender 3 S1 running Klipper firmware. After that print, it started not extruding. Well, the motor started wiggling back and forth rapidly. After the prime line, it stopped completely. I thought it might be a problem with Klipper, so I switched back to stock firmware and used a different slicer too. But it is still there. I uploaded a video of it to YouTube:

\n

https://youtube.com/shorts/q-ZLMg6NHng

\n", "Title": "Extruder motor wiggling", "Tags": "|creality-ender-3|", "Answer": "

I replaced the cable to the extruder and the extruder motor.\nNow it works fine.

\n" }, { "Id": "20575", "CreationDate": "2023-02-14T01:52:43.267", "Body": "

I use a procedural 3D animation package (SideFX Houdini) for my 3D object design. After exporting my design to STL then importing to PrusaSlicer, I manually place modifier objects to adjust the infill percent on certain areas. I'd like the ability to procedurally designate the infill percentage in certain areas in my design.

\n

Is there a way to flag an object in STL/3MF/Other file format so it's interpreted as an infill adjustment modifier in PrusaSlicer?

\n", "Title": "Define PrusaSlicer infill modifiers in STL/3MF?", "Tags": "|3d-design|stl|infill|prusaslicer|cad|", "Answer": "

So worked out you can't directly do this with metadata, however you can add modifiers to objects in PrusaSlicer, and you can use external STLs as the geometry for the modifier. Then you can add an infill property to the modifier and set different parameters there.

\n

\"enter

\n" }, { "Id": "20577", "CreationDate": "2023-02-14T14:21:49.730", "Body": "

I'm playing with the QiDi Print software to compare the slicing results with mine (Dremel 3D45). One of the reasons I'm going to buy it is the printing in parallel using the two nozzles:

\n

\"enter

\n

But I'm not able to set up the slicer correctly. As an example I set both extruders with the same PLA. I did the following steps:

\n
    \n
  1. load the objects (two instances of the same or even two different objects)
  2. \n
  3. select the left one and press "print with extruder 2"
  4. \n
  5. select the right one and press "print with extruder 1"
  6. \n
  7. in the slicer parameters window, if I try to set the extruder for build plate adhesion it is set for both extruders:
  8. \n
\n

\"enter

\n

\"enter

\n

So I cannot set Extruder 1 for object printed with extruder 1 and the same for the other.\nHence I tried to select one object to enable the "per model settings". But, looking at the settings to customize, the Dual extrusion panel shows nothing:

\n

\"enter

\n

I enabled the visibility of all settings.\nIn any case, trying to slice with these settings, it add a weird wall around all the objects:

\n

\"enter

\n

I never seen such a wall and I don't understand why is there and how to avoid it.

\n

Finally, how to set up the slicer in order to print two objects in parallel?

\n", "Title": "QiDi i-Fast print in parallel", "Tags": "|ultimaker-cura|slicing|dual-nozzle|", "Answer": "

\"ooze

\n

The question regarding the wall is covered in the advanced settings for Additions. In the screen capture above, you can see that it is turned on. Ostensibly, it's used to scrub off the previous filament oozing as it sits idle.

\n

I don't have an i-fast and cannot identify the setting you require for duplication printing. Terminology is tricky; my IDEX printer slicer calls it duplication printing and mirror printing.

\n

As an aside, you should have a contact email for QIDI tech support. My experience with the email system is next-day response.

\n" }, { "Id": "20591", "CreationDate": "2023-02-17T19:27:11.943", "Body": "

I recently got into 3D printing.

\n

Printer Setup

\n\n

After doing some test prints and figuring out my leveling and z-offset.\nI had some print residue on the bed, where you could clearly see the first layer fails.

\n

This morning, I decided to clean the print bed with acetone after reading several other forums and watching some reputable youtubers.

\n

After wiping the bed down with a cloth, I know have frosting on my bed.\nI found this out from some other posts, but no post on how to properly clean the frosting.

\n

I've added a picture, so hopefully that helps too.

\n

\"enter

\n

Is this bed surface trashed??

\n", "Title": "How to clean / remove frosting from build-surface", "Tags": "|build-plate|cleaning|build-surface|acetone|pei|", "Answer": "

Acetone should not be used for cleaning build surfaces. It attacks most plastics, probably including whatever Buildtak-clone is made of. It probably won't immediately ruin it, but it will degrade the surface texture.

\n

The right agent for cleaning bed surfaces is IPA (isopropyl alcohol). If you have print material stuck to the bed texture from printing with the nozzle way too close to the bed, your best best for removing it is printing more of the same material on top of it, and hoping it adheres to the new print and pulls off with it. Not using aggressive solvents.

\n

Is the bed surface ruined? Probably not ruined, but likely somewhat degraded. I've seen much worse though, so I'd expect it's still good for a few months of moderate use, if you take good care of it and don't keep using inappropriate cleaners or printing with the nozzle scraping the bed.

\n" }, { "Id": "20595", "CreationDate": "2023-02-19T07:24:50.197", "Body": "

For the benefit of a novice friend who wants to get into 3D printing, I am following an excellent YouTube course by Paul McWhorter, on Fusion 360 (playlist) which uses a Creality Ender-3 v2 to process the resulting designs.

\n

However, the esteemed Mr McWhorter often refers to the "threaded rod" during the video that covers the assembly of the printer (Fusion 360 for 3D Printing LESSON 2: Assembling the Creality Ender 3 Version 2 3D Printer).

\n

Is this just a slip of the tongue, or does the Ender-3 v2 actually come with a threaded rod, and not a lead screw?

\n

Looking at the official dual Z-axis upgrade kit for the Ender-3 v2 on Amazon, Creality Official Ender 3 Dual Z-Axis Upgrade Kit ( 42-34 Stepper Motor Included ) for Ender-3, Ender-3S, Ender 3 Pro, Ender 3 V2 3D Printer, it would seem that the kit comes with a lead screw:

\n

\"Lead

\n

Therefore it would seem logical that the base printer also comes with a lead screw. However, I've not really been able to find a clear enough image of the printer's component parts in order to ascertain whether that is true or not.

\n

I don't want to recommend that my friend purchases an Ender-3 v2, only to realise that it is supplied with a threaded rod, when it arrives.

\n

Can anyone, who either owns an Ender-3 v2, or who has seen one "in-the-flesh", confirm whether it comes with a lead screw?

\n

Also, without wishing to broaden the scope of this question, do all Creality printers come with lead screws? Are there any Creality printer models that do actually come with a dreaded threaded rod (which, IMHO, should be avoided, as that would require an immediate upgrade to lead screw)?

\n

Note: I'm not looking for a (shopping) recommendation, I just need a simple yes or no confirmation.

\n", "Title": "Does the Creality Ender-3 v2 come with a threaded rod or a lead screw?", "Tags": "|creality-ender-3|", "Answer": "

The Ender 3 v2 does have a lead screw.

\n\"Photo\n

These are the main differences:

\n\n" }, { "Id": "20596", "CreationDate": "2023-02-19T09:44:51.160", "Body": "

I just recieved a roll of eSUN black PLA+ and I have found out it has moisture damage after using it for 2 days. I was previously using a roll of eSUN orange PLA+ and it worked really well. So I ran a stringing test just to make sure the settings were right for the black roll and the result was terrible. So I spent a full day messing with slicer settings as moisture damage was last on my list for the issue since the roll was brand new. But with no improvement I decided to dry out a sample of filament just to try and this was the result.

\n

\"enter

\n

Left is filament straight from the roll, middle is dried out filament and orange as a control. So now I know the problem is this normal for eSUN filaments? Should I dry out filaments as soon as I take them out of the box? I didn't have this issue with my orange roll.

\n", "Title": "Black eSUN PLA+ has moisture damage straight out of the box. Is this normal?", "Tags": "|filament|stringing|", "Answer": "

Yes this is plausible. Please weight your bobin and dry it then weight it again to know how much water you extracted.

\n" }, { "Id": "20606", "CreationDate": "2023-02-22T13:40:03.770", "Body": "

I have a new Ender 3 S1 printer with CR Touch sensor (never had experience with auto levelling before) and I'm confused about how it works. Also, I updated the firmware to Professional Firmware.

\n

I ran a couple of tests with different approaches and printed a two-layer square the size of the bed.

\n
    \n
  1. I do manual bed tramming using paper. Decent tolerance, all four corners of the test print have approximately the same height of ~0.45\u00a0mm
  2. \n
  3. Then I run the tramming wizard, it touches four corners of the bed and says the back-left corner has an elevation of 0.15\u00a0mm and the front-right corner is -0.16\u00a0mm. I adjust the bed using screws until the wizard says all four corners are within ~0.03\u00a0mm and I print the same square again. The result is worse than paper levelling back-left corner is ~0.64\u00a0mm thickness and the front-right is ~0.45\u00a0mm. It even visually looks inconsistent
  4. \n
  5. I tram the bed using paper again, print the square, and all is good. Then I built auto-leveling mesh, enabled it in starting G-code, and printed the square. Once again, the result is bad.
  6. \n
\n

How it could be that manual tramming does not correspond to the probe levelling? I don't think it's a mechanics issue, since both the nozzle and the probe are sitting rigidly on the same extruder.

\n

Here is what the mesh looks like after nice and consistent manual tramming.

\n

\"Photo

\n", "Title": "Inconsistency of manual tramming and auto tramming/auto bed levelling", "Tags": "|creality-ender-3|marlin|bed-leveling|" }, { "Id": "20613", "CreationDate": "2023-02-23T19:03:55.567", "Body": "

I've been getting deeper and deeper down the rabbit hole of UBL and I am losing my mind!

\n

Just installed the CR Touch and I am having print issue after print issue with larger prints despite manually leveling the bed (bed adjustment not UBL manual) to the best of my ability.

\n\n

The height map clearly shows an issue - I just don't know how to fix it.

\n

\"Screenshot

\n

Of note - G29 P1 will only get through 44 of the 100 touchpoints

\n

Here is the code I ran through to generate the mesh:

\n
G28       ; home all axes\nM420 S0   ; Turning off bed leveling while probing, if firmware is set\n          ; to restore after G28\nM155 S30  ; reduce temperature reporting rate to reduce output pollution\nM190 S65  ; (optional) wait for the bed to get up to temperature\nG29 P1    ; automatically populate mesh with all reachable points\nG29 P3    ; infer the rest of the mesh values\nG29 P3    ; infer the rest of the mesh values again\n@BEDLEVELVISUALIZER ; tell the plugin to watch for reported mesh\nM420 S1 V ; enabled leveling and report the new mesh\nG29 S0    ; Save UBL mesh points to slot 0 (EEPROM).\nG29 F 10.0 ; Set Fade Height for correction at 10.0 mm.\nG29 A     ; Activate the UBL System.\nM500      ; save the current setup to EEPROM\nM155 S3   ; reset temperature reporting\nM140 S0   ; cooling down the bed\n
\n", "Title": "Ender 3 CR Touch UBL Fails to Produce Viable Mesh", "Tags": "|creality-ender-3|marlin|firmware|", "Answer": "

I don't know how useful this will be for you now, but I do have a couple of tidbits of info to help others who may have this question and find yours:

\n
    \n
  1. You can replace the stock screws on the Plate that carries the extruder (Carriage). The stock ones are (I believe) M5x30\u00a0mm screws, you can replace the top right and bottom (when looking at the carriage from the front) screws with M5x25\u00a0mm screws. If you put them in from the back, and use a shorter nut, you can achieve the same range of motion you normally have with the stock extruder.

    \n
  2. \n
  3. If you're configuring your own firmware, in Configuration_adv.h in the Marlin Firmware source files, you will find a section called @section Leveling. Here, you can modify the probing margins per axis, per value. You can set both the MESH_MIN and MESH_MAX distances for both X and Y axis.

    \n
  4. \n
\n

It should look something like this:

\n
    #if EITHER(MESH_BED_LEVELING, AUTO_BED_LEVELING_UBL)\n      // Override the mesh area if the automatic (max) area is too large\n      #define MESH_MIN_X 20\n      #define MESH_MIN_Y 20\n      #define MESH_MAX_X X_MAX_POS - 48\n      #define MESH_MAX_Y Y_BED_SIZE - (20)\n    #endif\n
\n" }, { "Id": "20618", "CreationDate": "2023-02-24T15:44:11.757", "Body": "

I'm using the eSun Silk PLA and I printed a temperature tower to find out the best settings. For print speed, the default speed of 50\u00a0mm/s and 25\u00a0mm/s for walls and surfaces are used.

\n

The problem is none of the temperature levels seem to be the best! As you can see from the following pictures:

\n

\"enter\n\"enter

\n

The 195 \u00b0C step is the "less worse" but still far from good.\nAre there any other parameters I should tune?

\n

I have a Dremel 3D45 and I use Cura.

\n", "Title": "PLA temperature tower failed", "Tags": "|pla|calibration|temperature|", "Answer": "

For the most part, a well dialed-in printer should be able to print PLA just fine along that entire range of temperatures. You seem to have various other problems going on which make a temperature tower a poor place to start with tuning. Also, this test makes heavy use of bridging, which is really hard to get right because it involves things like suddenly disappearing backpressure against the extrusion, lack of surface to squish the material against, and how you deal with that. Cura has a lot of settings to tune this, but getting it right is an advanced topic, and hard even then. Judging that you have the wrong temperature because bridging had problems, when you haven't done anything to get bridging to work, doesn't make sense.

\n

The catastrophic stringing at the bottom indicates to me that the filament is fairly wet. I would suggest drying it before doing anything else, because so many of the common print problems folks hit have wet filament as the root cause or at least a contributing cause, and you can waste so much time chasing other things like tuning retraction, temperatures, etc. when the problem was just water all along. Silk filaments are particularly sensitive to moisture, by the way.

\n

You also have a severe ripple artifact in the surface of your print. This could be ringing from having your acceleration set too high for your printer. Probably any more than 500 is way too much for outer walls on this printer; inner and other parts could perhaps go a bit higher. But it's possible you already have acceleration plenty low and that the issue here is poor belt tensioning or a loose screw somewhere in the toolhead or motion system assembly.

\n

It looks like you have some places where the material didn't stay in the right place at corners. This might indicate poor cooling, either something wrong with the fan (filled with dust, broken blade, bad bearing with too much friction) or just that the printer lacks good cooling to begin with. This would also contribute a lot to bridging not working well.

\n" }, { "Id": "20626", "CreationDate": "2023-02-25T21:57:16.077", "Body": "

Printer: Ender 3 with a metal extruder - PLA

\n

After a long break from printing anything, I started printing again, but the results had obvious underextrusion issues:\"XYZ

\n

But after I tweaked temp, flow, and speed for multiple test prints, the issues disappeared - even if the settings were returned to default.

\n

Sadly, after a couple good prints, it started having issues again.

\n

Why is my printer having these issues?

\n

EDIT: The cause was that I didn't install the spring in the tension arm correctly-it was not compressed enough to deliver the necessary force to grip the filament.

\n

Since my printer was working fine until recently, it leads me to believe that the spring must have weakened. So new question: is that the case? And do you recommend I unscrew the arm and release the spring when I'm not using the printer to improve lifetime?

\n", "Title": "Underextrusion Cause", "Tags": "|creality-ender-3|pla|underextrusion|", "Answer": "

This is unlikely to be a settings issue. It's almost surely a physical issue with the extruder.

\n

Usually on an Ender 3, this is a problem with the tension arm. The stock one is made of plastic and the arm, or mount that's holding the arm can crack.

\n

It's also possible you have a weak spring tensioning it.

\n

Aside from issues with the arm, the teeth of the extruder gear can get gummed up with chewed-up plastic, or worn down from printing abrasive materials, to the point they're no longer able to dig into the filament and push it effectively.

\n" }, { "Id": "20634", "CreationDate": "2023-02-28T22:00:27.500", "Body": "

I have a BLV Core XY, I'm trying to print different models, but the machine moves the center once it goes to start the process.

\n

This is my G-code of the beginning:

\n
G21\nG90\nM82\nM107\nG28\n\nM413 S1 ;Power-loss recovery\n\n;M190 S60    ;bed heat on\nG34              ;Auto Z Level\nG29              ;Bed Levelling\n\nG28 Z\n \nG1 Z2 F240\nG1 X2 Y10 F1200\nG1 Z0.2 F240\nG92 E0.0\nG1 Y150 E15.0 F1000.0 ; intro line\nG1 X2.3 F5000\nG1 Y10 E30 F1200.0 ; intro line\nG92 E0.0\n\nM117 Printing...\n
\n

\"one\"

\n

\"two\"

\n

However, if I do a simple command like, to send the (0,0) on Z axis, it happens fine.

\n

\"center\"

\n", "Title": "How center the nozzle before start printing", "Tags": "|ultimaker-cura|g-code|corexy|", "Answer": "

You have defined your bed incorrectly in the firmware.

\n

First line you print is at X=2, while it is printed a couple of centimeters from the edge. Second the final cube is not in the center of the bed while it presumably is sliced in the center.

\n

You should look into the definition of the bed size, the end stop offsets and the probe offsets to fix this. A detailed question and answer is found in How to set Z-probe boundary limits in firmware when using automatic bed leveling?.

\n" }, { "Id": "20636", "CreationDate": "2023-03-01T03:01:15.377", "Body": "

I've got one particular print which is causing me problems. I've printed other things without changing the filament or printer, so the problem is at least partially the print itself, which has a lot of supports and unconnected sections. What sorts of things can I do to fix this? I'm printing on an Ender 3 Pro and using Ultimaker Cura for slicing.

\n

I print in PLA at 200\u00a0\u00b0C. The print bed is set to 40\u00a0\u00b0C. I use a print cooling fan at 100%. The layer height I set to 0.2\u00a0mm, the line width 0.4 mm from the 0.4 mm nozzle. I'm not positive what my print speed was set to, but I think I tried it at the default (50 mm/s infill) and then a second time slower (20-30 mm/s, probably). Regardless, the wall speed was set to half whatever the infill speed is. My retraction is 6.5 mm/off at 25 mm/s.

\n

\"half-completed

\n", "Title": "I have one print that's underextruding, what settings should I change?", "Tags": "|creality-ender-3|print-quality|ultimaker-cura|underextrusion|", "Answer": "

I have noticed issues like this to happen for me due to what I mention below. I would check each of these and attempt to further narrow down the problem. It is the nature of 3D printing with how different each print shape, machine, and filament being used are, so it is difficult to provide exact settings advice to fix an issue.

\n
    \n
  1. Retraction speed & distance - with the incorrect combination you can cause your printer over time to extrude less at certain layers. The problem compounds over time and becomes apparent on prints that have a lot more retraction steps than others. It is important that any time you change to a new brand or type of filament, you run certain temperature and speed test prints. There are certain models that can be downloaded online on Thingiverse or Printables websites that test for print quality based on differences of retraction speeds, temperature, overhang angle, cooling settings, etc. Teaching Tech (YouTube link), for example, offers a very comprehensive array of tests and procedures for calibrating your printer. I suggest visiting his website.

    \n
  2. \n
  3. Bowden tube lubrication - Something like a Capricorn\u2122 tube offers less resistance as the filament is sliding through the tube as compared to the Bowden tube that comes with your printer. Increased friction in the Bowden tube will cause inconsistent extrusion of the filament due to impeding the acceleration and speed as the filament is moving back and forth in the tube. Furthermore, it will exhibit different resistance based on how the filament is resting within the Bowden tube while it is flexing as the print head is moving. Lubricating your filament with a little bit of olive oil every few prints will help in this regard in addition to purchasing a higher quality Bowden tube.

    \n
  4. \n
  5. Extruder drive gear tension - the point where your filament is biting into the cogs of the extruder needs to be very tight. The spring on the extruder pulley clamp needs to constantly exert enough pressure for the cogs to really dig into the filament. This is important because if the bite is not good enough, it will cause inconsistent extrusion when rapid accelerations happen such as when retractions occur. It is good practice that every few prints to manually push on the pulleys together as over time the spring will degrade and become weaker.

    \n
  6. \n
\n" }, { "Id": "20640", "CreationDate": "2023-03-01T11:20:46.197", "Body": "

I am following the tutorial Slicing a 3D object to SVG.

\n

My code looks like so:

\n
import("/Users/petarivcec/Desktop/art/king of sun/1.stl");\n\n z_min = 0;\n z_max = 100;\n x_max = 50;\n y_max = 55;\n slice = 1;\n \n n = floor(sqrt((z_max - z_min)/slice)+1);\n for(z = [-z_max:slice:z_min]) { \n   i = (z + z_max) / slice;\n   x = x_max * (i % n);\n   y = Y_max * floor(i / n);\n   translate([x,y,0]) {\n       projection(cut=true) \n           translate([0,0,z]) thing();\n   };\n };\n
\n

When I hit Render, then I get warnings like:

\n

WARNING: Ignoring unknown module 'thing' in file , line 16

\n

I guess that I just need to figure out how to run the loop on my imported stl instead of the non-existent thing module which exists in the example code but I'm not really sure.

\n", "Title": "I am getting an error when trying to slice an STL object into multiple vector files", "Tags": "|openscad|", "Answer": "

According to the linked site, it appears that you are missing the "identifier" module for the item to be cut into layers, aka thing();

\n
\n

module thing() // a random object, in this case a hollow sphere on top\nof a hollow cube.

\n
\n

I suspect that if you encase your STL import in a module named thing() you will have a solution. An alternative to naming the module would be to remove the thing() call and use the import at that specific location.

\n

The second sample in the linked site is a failure similar to your for the same reason.

\n" }, { "Id": "20647", "CreationDate": "2023-03-02T11:56:17.853", "Body": "

I'm trying to print with ABS plastic. I've only used PLA before. I'm using Anycubic Kobra Neo (Heated, PEI-coated bed). Originally, I just changed my plastic to ABS and tried printing with manufacturer-provided profile (80/230 degrees), without changing Z-offset or any other details. However, plastic just didn't stick to the bed. Since then, I've tried:

\n\n

Nothing seems to help. Every time, it starts printing the "border" (not sure how it is called) more or less fine:

\n

\"Border

\n

But then it either leaves peaces of plastic on the bed, or they just stick to the extruder:

\n

\"Extruder

\n

Are there any other steps I should check to ensure proper adhesion? I'm aware that it is recommended to use things like glue to increase adhesion when printing ABS, but I thought it is possible to print without them if the bed is heated and has PEI (or other) special coating.

\n", "Title": "What should one do to ensure ABS sticks to bed?", "Tags": "|extrusion|abs|", "Answer": "

I use glue sticks around the footprint of what you will be printing. Never had a problem after that. Another alternative is to use some masking tape.

\n" }, { "Id": "20652", "CreationDate": "2023-03-03T16:20:03.610", "Body": "

What are some science/engineering journals on the topic of 3D printing with high impact factor? To publish a paper about the analysis of 3D models before printing them.

\n", "Title": "Science/engineering journals with high impact factor", "Tags": "|3d-models|", "Answer": "

I think this one is acceptable: Additive Manufacturing

\n

It has an 11.632 Impact Factor.

\n

Also, it has the preprint feature mentioned here:

\n
\n

In support of Open Science, this journal offers its authors a free\npreprint posting service. Preprints provide early registration and\ndissemination of your research, which facilitates early citations and\ncollaboration.

\n

During submission to Editorial Manager, you can choose to release your\nmanuscript publicly as a preprint on the preprint server SSRN once it\nenters peer-review with the journal. Your choice will have no effect\non the editorial process or outcome with the journal. ...

\n

You will be notified via email when your preprint is posted online and\na Digital Object Identifier (DOI) is assigned. Your preprint will\nremain globally available free to read whether the journal accepts or\nrejects your manuscript.

\n
\n" }, { "Id": "20660", "CreationDate": "2023-03-08T16:28:14.613", "Body": "

I have a classic Ender 3 running Marlin-2.1.1 on a BTT SKR v1.3 board with TMC 2130 drivers. Wifi is connected with ESP3D on an ESP32-CAM board connected to a TFT port. The X endstop is on the left side.

\n

When I send G28 or G28 X, the hotend does not home properly.\nRather the hotend just moves a little to the right side (like 3\u00a0mm) and stops.\nSo if I keep on sending G28 X, the hotend just moves little by little to the right side.

\n

configuration files

\n", "Title": "G28 X does not home the hotend", "Tags": "|marlin|homing|axis|ender3|", "Answer": "

This indicates that the printer thinks the endstop is already triggered and is trying to back off to untrigger it, so that it can probe. If a minimal move to the right does not untrigger it, it gives up, assuming something is wrong with the endstop. Either your endstop switch is broken or it's not connected. A disconnected endstop switch will behave identically to one that's stuck down (open circuit).

\n" }, { "Id": "20664", "CreationDate": "2023-03-09T04:33:35.677", "Body": "

I've an Ender 3 and a piece broke.

\n

The part that is broken is on top of the Z stepper motor that guides the filament to the extruder.

\n

The metal "guide wheel" that hugs the filament as its pushed through, the plate beneath it broke. I dropped a socket wrench on it. -- it's just sitting there in the photo, but it's not doing it job.

\n

Do I have to replace the whole stepper motor or can I just get the feeder thing that sits on top of the motor?

\n

\"This

\n

I don't know what it's called so it's tough to search for

\n", "Title": "Replacing (and identification) of a broken piece of the extruder", "Tags": "|creality-ender-3|extruder|", "Answer": "

Sadly this is a known-weak part on all Creality extruders of this design.

\n

The tension arm is ABS plastic and will work fine for a few months, maybe a year and will then break around where the bearing is secured to the arm.

\n

That equates to 5-10 kg of printing before the arm breaks. Since its a higher pressure point, gluing will not succeed.

\n

Your best option is to buy a replacement tension arm, in Aluminium. There are many options on Aliexpess, though fit and finish can be a mixed bag. I had to swipe some bolts from my original extruder-motor mount because the new one was short.

\n

It is telling that Creality offer a metal one and not a plastic one, but sells new printers with just plastic ones.

\n" }, { "Id": "20672", "CreationDate": "2023-03-11T08:38:11.530", "Body": "

I'm working on tuning an old Ultimaker 2 Go, and I'm having some weird extrusion issues. The printer uses a Bowden extruder. Extrusion seems to increase and decrease. I don't think it is the filament quality, as I am using Prusament PLA that I am storing in a dark closet inside a sealed bag with silica gel. I also did multiple cold pulls until no debris. I do notice that sometimes the filament appears to bubble. The extruding motor also sometimes slips/snaps back while changing filament, but not during prints.

\n

However, I am using 1.75\u00a0mm filament while the printer is designed for 2.85\u00a0mm. In order to do this, I followed some steps I found from All3D.

\n

I also have a small PVC jig to hold the filament, but I think it turns pretty well and shouldn't be the issue.

\n

Filament: PLA, Temp: 215\u00a0\u00b0C, Slicer: Cura

\n

\"1\"

\n

*Update:\nI calibrated the e-steps and all of the axes, but the problem still remains.

\n

I also printed an XYZ cube, and there is a line right under the letters X and Y. It also appears that the extrusion is more inconsistent outside of the letters. I'm pretty sure this issue is related to the weird first-layer pattern, or this is just a cooling issue due to varying cooling %.

\n

Filament: PLA, Temp: 215\u00a0\u00b0C, Slicer: PrusaSlicer

\n

\"1\"

\n", "Title": "Weird Pattern First Layer Extrusion", "Tags": "|extrusion|bowden|ultimaker|", "Answer": "

It turns out that it was caused by the filament being ground at regular intervals. I solved it by lowering the printing speed (I did change the bowden tube to a 1.75 mm one though). Still working on fully tuning this printer.

\n" }, { "Id": "20675", "CreationDate": "2023-03-12T04:36:50.020", "Body": "

Resin specifications

\n

There are datasheets available for SLA resin mechanical properties. Like this Formlabs' General Purpose Resin datasheet.

\n

They present mechanical properties for two cases.

\n

First case

\n
\n

Data was obtained from green parts, printed using Form 2, 100 \u03bcm, Clear settings, without additional treatments.

\n
\n

Second case

\n
\n

Data was obtained from parts printed using Form 2, 100 \u03bcm, Clear settings and post-cured with 1.25 mW/cm\u00b2 of 405 nm LED light for 60 minutes at 60 \u00b0C.

\n
\n

Table

\n

\"Screenshot\"

\n

Question

\n

I need to know the mechanical properties immediately after the resin is hardened by the projected light. Would the first case above provide my desired info?

\n", "Title": "Resin mechanical properties during the print process, immediately after light hardens it", "Tags": "|resin|print-material|sla|print-strength|dlp|", "Answer": "

I contacted Formlabs at support@formlabs.com and received answers:

\n

Formlabs Services answer 1

\n
\n

The green tensile strength is valid only for layers that have finished solidifying during the 3D printing process. So in effect, as the part is being printed, each section that has solidified is in its green state where the green tensile strength applies.

\n

Other that that, there is a moment in time for each scanline of a layer while it is being lased/polymerized, where the tensile strength will be something less than that green tensile strength, until the laser moves from that location and that section solidifies.

\n
\n

Formlabs Services answer 2

\n
\n

We have not tested compressive strength for our materials at the moment, so we would not be able to provide any data on that specifically.

\n
\n
\n

Regarding mass density of our standard resins, they will be 1.08, 1.13, and 1.14 [g/cm^3] in their liquid, green, and postcured states respectively.

\n
\n" }, { "Id": "20681", "CreationDate": "2023-03-13T18:58:15.983", "Body": "

I recently upgraded to Simplify3D version 5. With this version, I can now print over WiFi to my Qidi X-Pro. I run an Eero mesh network and the printer is about 6 feet from the router.

\n

I tried printing a model with this version over WiFi and received the following error on the Qidi console:

\n
Illegal gcode in line: xxxx, P_4, T_10470103!\n
\n

On a second attempt, I received:

\n
Illegal gcode in line xxxx, P_$, T10470000!\n
\n

The same models print correctly if saved to a USB stick first and then inserted into the printer. I've been in contact with Simplify3d support, but, they can't recreate the problem.

\n

Has anyone encountered this problem? Do you know of a fix?

\n", "Title": "Illegal G-code when trying to print", "Tags": "|g-code|simplify3d|wi-fi|", "Answer": "

I ran another test. This time I used the USB device that came with the printer. The print finished without errors. I'm not sure why using a different USB device would impact the WiFi printing.

\n

Note: I have some suspicions that using a quality setting of High may have actually caused the problem.

\n" }, { "Id": "20689", "CreationDate": "2023-03-16T01:34:19.817", "Body": "

When my Ender 3 was on the stock motherboard, I went through two BLTouch sensors. In both cases, we had the same result: sometimes, when homing with the sensor, the printer would jam the gantry down into the bed because it would miss the sensor. When leveling, it would intermittently stop before it had completed all the steps.

\n

Alas, since I have a glass bed I cannot use the Z-axis limit switch for homing (without mechanical modification, anyways.)

\n

I installed a BigTreeTech SKR mini e3 motherboard. Motion control works fine. In fact, it's much, much quieter. Sadly, still have lousy results (although I've only tried one sensor on this one.)

\n\n

Does anyone have any suggestions?

\n

(I think I've been using the same wire this whole time, for what that's worth.)

\n", "Title": "Ender 3 BLTouch not working with BigTreeTech SKR mini e3 or stock motherboard", "Tags": "|creality-ender-3|bed-leveling|bltouch|bigtreetech|", "Answer": "

I had this same issue and fixed it by removing the z-stop cable and using the black and white (the combined 2 wires on the BLTouch). Below is a diagram that may help.

\n

\"BLTouch

\n" }, { "Id": "20693", "CreationDate": "2023-03-17T06:40:20.603", "Body": "

I use "Flashprint 5" as a 3D print pre-processing software. I want to have a 0-degree raster angle for all its layers (100 % infill density), in the attached object below. But what I found is that each layer always alternates to 90 degrees. Any solution for this case?

\n

\"Raster

\n", "Title": "How to make constant raster angle on Flashprint 5?", "Tags": "|diy-3d-printer|flashforge-creator|flashprint|", "Answer": "

Looking at Flashprint expert mode, I don't seem to find options to change this.

\n
\n

Any solution for this case?

\n
\n

Yes there is by using a different slicer, e.g. Ultimaker Cura. In Cura you will be able to specify the directions of the infill layers as you want them to be using the Infill Line Directions printing property. Default the property is empty ([ ]), but could be changed to [90, 90] or [0, 0] (or any other arbitrary angle).

\n

Print option is found under Infill:
\n\"enter

\n

Layer 60:
\n\"enter\n
Layer 61:
\n\"enter

\n

You can take this even further in Cura by specifying directions for more than 2 layers, e.g. [-45, 0, 45, 90] will give you an alternating pattern of 4 layers.

\n" }, { "Id": "20704", "CreationDate": "2023-03-18T23:17:25.910", "Body": "

Issue:

\n

I realized that the filament extrusion always stops at the same spot for a couple seconds then starts back, I know it's the printer and not the filament, I can see the gear literally stop for a couple seconds. The fan also stops during that time, I suspect it has caused the printer to activate it's thermal runaway protection shutdown a couple times. I have no idea what to google to fix it. Here is a picture:

\n

\"Filament

\n

As you can see it's like it stops before the curve, but to be clear, unlike all the questions I have read here, it starts back a couple seconds after.

\n

For context:

\n\n

Maybe related?

\n

I've had my printer for maybe 2 months, and I thought I finally had it setup, prints were consistent and never failing, then yesterday it's like every single setting fell apart at the same time.

\n

The print is no longer sticking to the bed, and I constantly get a "AB Temp." shutdown. I switched the nozzle, no success. I always level it with a sheet of paper, but now I see that after the bed probe hits (as it does before every print), it causes it to raise the nozzle a couple tenth of a millimeter, which explains why it's not sticking. I fixed that by changing the Z-Index, but it's weird that it randomly changed? (by 0.3mm which is significant)

\n

I have no idea how to fix the extrusion stopping randomly though, but I think when the extrusion and fan stop together, it suddenly changes the nozzle thermal properties, so its temperature shoots up, I think that's what's causing the "AB Temp." shutdown.

\n

I print via USB-C so it's not a corrupt file, I have been printing this way many times, no issues. I'm stumped, thank you for your help!

\n

Update:

\n

This time I got quite far before thermal shutdown, it doesn't look random so I think the problem might be in the G-Code, at the same time, I don't get the pattern. Here are the pictures:\n\"Gap\"Gap

\n", "Title": "Printer stops extruding at same point on each pass", "Tags": "|creality-ender-3|ultimaker-cura|extrusion|thermal-runaway|", "Answer": "

The error you encounter is related to a partly broken wire or problem with the seating of connector plugs in their sockets. The fact that the fan also stops hints to this. You need to check all cables and connectors, especially the thermistor.

\n

Such problems arise after a few months of usage.

\n" }, { "Id": "20709", "CreationDate": "2023-03-20T14:33:16.890", "Body": "

Alright, a few assumptions about the printer.

\n
    \n
  1. It can handle the momentum of the nozzle moving so fast.
  2. \n
  3. I have good enough bed adhesion to do this.
  4. \n
  5. The nozzle can handle the amount of filament going through it.
  6. \n
  7. Im using PLA filament.
  8. \n
  9. I have the thing anchored to my desk with 10 foot long 3 inch wide bolts so it wont achieve liftoff.
  10. \n
\n

I am somewhat interested in building this printer so that why im wondering. Would it be possible that the filament will move so fast as it exits the nozzle that is doesn't attach the the model itself, just fly off of it? Thanks for any help!

\n", "Title": "If you had good enough motors could you print at 1000 mm/s with good quality?", "Tags": "|diy-3d-printer|speed|", "Answer": "

Yes you can print at 1000 mm/s with good quality, but motors really have nothing to do with it. The cheap motors you get on an Ender 3 can go at 1000 mm/s if you have a controller board supporting 48V power supply for the stepper drivers. In general, max motor RPM before you hit pullout (the speed at which torque rapidly falls off because you don't have enough voltage to drive a reversal of the magnetic field for the next step in time) is proportional to voltage. With 24V these motors can take you up to 500 mm/s or higher, so at 48V I'd expect to hit 1000 mm/s without much problem.

\n

However, to reach 1000 mm/s, you need enough acceleration to reach that speed before you have to start slowing back down to stop or change direction. High acceleration, not high speed, is what has tradeoffs with quality, because no physical system is perfectly rigid, and under high forces (F=ma), there will be significant physical stresses on the printer frame, belts, motor shafts, bearings, etc. affecting the accuracy of your print. But fortunately, the accelerations you need to reach 1000 mm/s aren't all that high. You can reach it on 20 mm lines with just 50000 mm/s\u00b2, which, although 100x the default on typical bedslingers like an Ender 3, is well within the capabilities of good CoreXY, CrossXY, or delta printers, especially with input shaping available on modern firmware. You can find plenty of videos on YouTube of the K3 (by Annex Engineering), VzBot, etc., as well as some RatRigs and Vorons, doing this kind of acceleration at pretty much perfect quality.

\n

Now, the hard part of getting really high speeds is pushing and melting the plastic that fast. Fortunately, you didn't specify a layer height, so if by 1000 mm/s, you mean 1000 mm/s at 0.05 mm layer height, that's only 20 mm\u00b3/s, and doable even for a very standard hotend, and easy if it has a CHT nozzle. But if you want to do 1000 mm/s at 0.2 mm layers by 0.4 line width, that's 80 mm\u00b3/s. There are very few hotends which can achieve that kind of flow, but the Goliath by VzBot can do nearly twice that, and various hybrid volcano approaches like the Volcomosq can probably reach 60-80 mm\u00b3/s.

\n

People don't make high-end gear like this to print ugly boats (well, they do that sometimes for fun, but if you're putting the effort into making it, you almost surely want it to be useful too). The printers that can print at this kind of speed absolutely do it at high quality.

\n

Here's one example I just found of printing at 50k acceleration, 1500 mm/s:

\r\n \r\n

\n" }, { "Id": "20720", "CreationDate": "2023-03-22T10:08:26.133", "Body": "

I am new to 3D printing, and are having a problem with wall thickness in vase mode.\nI would like to print this vase model: https://www.printables.com/model/60696-blossom-vase

\n

As seen in the "Makes & comments" section, people have printed the model with quite thick walls, and I would like this as well, maybe thicker.

\n

I have a Prusa i3 MK3S+ printer, and are using the Prusa Slicer software. When I import the .stl model and select vase mode in print options, I cannot alter the layer height, solid layers etc. options to increase wall thickness. This means that the 3D print is almost paper thin.

\n

So how can I make a print of this model, with thick walls?

\n", "Title": "How to adjust vase mode thickness", "Tags": "|prusa-i3|prusaslicer|", "Answer": "

Vase mode implies a single wall, so if you need to have a thicker wall you need to set an extrusion width larger than the nozzle width (you cannot do that unlimited), but usually a 0.4\u00a0mm nozzle can print at 0.8\u00a0mm width (twice the nozzle diameter), or you could replace the nozzle for a wider nozzle, e.g. a 0.8\u00a0mm nozzle (and also print at larger diameter). Note that when using larger nozzles/diameters, the extrusion volume increases, so you need to print slower.

\n" }, { "Id": "20723", "CreationDate": "2023-03-23T12:29:37.680", "Body": "

My printer recently started lowering the bed an additional step for every measurement on the G29 procedure. The only change I did, at least purposely is, adjusting the nozzle offset and MIN/MAX positions for X and Y because I installed the "Hero Me" parts.

\n

Here's a video of what happens:

\n

\r\n \r\n

\n", "Title": "Weird bed movement when doing ABL (G29)", "Tags": "|automatic-bed-leveling|", "Answer": "

As it turned out, it is actually a bug: https://github.com/MarlinFirmware/Marlin/issues/25565

\n" }, { "Id": "20730", "CreationDate": "2023-03-26T00:11:18.273", "Body": "

I'm very new into 3D printing.

\n

I brought an Ender 6 and a cover for ABS printing, but the standard white Bowden tube was too short to pass through the hole of the cover. So I'm replacing the Bowden tube for a Capricorn one.

\n

However, the manufacturer (Captubes.com) only states that their 2 meter kit is compatible with CR-10 and Ender 3.

\n

Is this kit also compatible with an Ender 6?

\n", "Title": "Upgrading Bowden Tube on Ender 6 to Capricorn", "Tags": "|bowden|creality-ender-6|", "Answer": "

I ended up getting in contact with the sales team at Captubes.com, and they replied promptly, what I did not expect.\nThey claim that the Capricorn tube is indeed compatible with the Ender 6 printer, and that they use the same fittings, with the exception of pro printers and ones that use something called a Bondtech extruder.\nI'd like to thank everyone that helped to clear this matter.

\n" }, { "Id": "20734", "CreationDate": "2023-03-26T17:44:31.027", "Body": "

I'm trying to build a cover for my Hypercube Evo's Meanwell LRS 350W power supply.

\n

The power supply input is 120V AC and it provides 24V DC output to a Einsy Rambo 1.2 board.

\n

I want to protect the Meanwell mains with a cover and a rocker switch with IEC receptacle like this one.

\n

The receptacle has a 5 AMP Fuse between mains and switch. The Einsy also has three fuses, one for Hotend 5 AMP and one for Fans, etc. 5 AMP, and also a 15 AMP fuse for the heatbed. And there is a dedicated power input for the bed on the board. I plan to run a MK2a heatbed, which I think will draw about 13 AMPS at least.

\n

My question is, will the receptacle fuse blow when I use the heatbed, or does the amperage come from the power supply (after the receptacle)?

\n

Any other thing I should worry about in this configuration?

\n", "Title": "Can I run a 15 AMP heatbed behind a 5 AMP IEC Receptacle?", "Tags": "|power-supply|", "Answer": "

The fuses on the board are for current pulled from the 24V DC. 15 amps of 24V DC is 3 amps of 120V AC, plus a little lost in conversion inefficiency in the power supply. If all of them were maxxed out, you'd be right at the 5 amp from mains, and inefficiency could perhaps throw you over. But presumably they're not supposed to be maxxed out, so you're probably okay.

\n" }, { "Id": "20737", "CreationDate": "2023-03-27T09:38:03.233", "Body": "

I got an Ender 3 Pro yesterday and have been having a problem with all of my prints. Whenever the extrusion stops and the X-axis moves, it moves too far. For example, I tried printing a Benchy with 2 skirt lines, and the lines were the right size but too far apart (I measured it and they turned out to be 5\u00a0mm apart).
\nTo be clear, this issue only occurs when I'm printing something. When I use the 'move axis' function on the X-axis, it moves appropriately.

\n\n

Update: I've attached a picture of this. I threw the original Benchy in the trash so I made another one. I'll keep experimenting and see if I can fix it.\n\"Picture

\n", "Title": "Ender 3 Pro: X-axis moves too much during printing", "Tags": "|creality-ender-3|ultimaker-cura|axis|x-axis|ender3|", "Answer": "
\n\n
\n

The issue came from the way I assembled my printer, not from the G-code or slicer that I was using.

\n
\n\n
\n

It turns out that I put the belt on the X-axis upside down, and that was causing it to move incorrectly. I detached the belt from the 2 points that it was clipped onto, then re-applied it correctly and tightly, and now I can print things properly.

\n" }, { "Id": "20743", "CreationDate": "2023-03-29T13:59:46.777", "Body": "

I have created an action figure and would like to print it.\nAnyway, I am not sure the best way to print it, the action figure can be printed only once but the duration is 10 hours. If I separate the part, the time is concise, but I need to do some changes to my file. What is the best way to print an action figure?\nIf the second method is the better (separate arm, legs, etc.), does exist a good way to do the join between parts?

\n

My 3d printer is Elegoo Mars, so I use Chitubox.

\n

\"enter

\n", "Title": "What is the best way to print an action figure?", "Tags": "|3d-models|3d-design|", "Answer": "

In my experience, you should never print a figure like that upright as it A) takes too long and B) can cause problems with supports.

\n

I would recommend that you cut it on two pieces. One with the legs and one with the body. This will cut your print time almost in half.

\n

Next, tilt both parts backwards by at least 45 degrees. This will further reduce print time and will give you better supports as you can spread them out across a larger surface area while having a smaller surface area in contact with the FEP for each layer.

\n

My printer ins an Elegoo Saturn. I use Chitubox.

\n" }, { "Id": "20748", "CreationDate": "2023-03-30T13:42:11.850", "Body": "

My printer's hot end heats up way faster than the bed. Still, it takes its sweet time, and Cura's default startup sequence: heat the bed, wait for bed to reach target temperature, heat hot-end, wait to reach hot-end temperature, proceed - takes considerably more that it should.

\n
M140 S{material_bed_temperature_layer_0}      ;Start heating bed\nM190 S{material_bed_temperature_layer_0}      ;Wait for bed to reach temp\nM104 S{material_print_temperature_layer_0}    ;Start heating extruder \nM109 S{material_print_temperature_layer_0}    ;Wait for extruder to reach temp \n
\n

Sure I could start heating both to target temperature simultaneously and then wait for both, but then I'll be baking unmoving filament in the nozzle for much longer than necessary. What I'd like to do is to start heating the nozzle when the bed is about 20\u00a0\u00b0C short of its target temperature - that causes them to reach their target temperatures nearly simultaneously. Something along the lines of:

\n
M140 S{material_bed_temperature_layer_0}      ;Start heating bed\nM190 S{material_bed_temperature_layer_0 - 20} ;Wait for bed to be 20 degrees before target\nM104 S{material_print_temperature_layer_0}    ;Start heating extruder \nM190 S{material_bed_temperature_layer_0}      ;Wait for bed to reach temp\nM109 S{material_print_temperature_layer_0}    ;Wait for extruder to reach temp \n
\n

Except S{material_bed_temperature_layer_0 - 20} isn't recognized as a valid substitution.

\n

How can I achieve the desired effect?

\n", "Title": "Math in Cura start G-code expressions?", "Tags": "|ultimaker-cura|g-code|heated-bed|", "Answer": "

None of the above examples work, because the settings parser is more complex than that.

\n

So you can achieve preheated temperatures by adding the following modification to the fdmprinter.def.json file:

\n
"material_bed_temperature_pre":\n    {\n        "label": "Build Plate Preheat Temperature",\n        "description": "The temperature used for preheating the heated build plate.",\n        "unit": "\u00b0C",\n        "type": "float",\n        "resolve": "material_bed_temperature_pre",\n        "default_value": "material_bed_temperature_layer_0 - 8",\n        "value": "material_bed_temperature_layer_0 - 8",\n        "minimum_value": "0",\n        "minimum_value_warning": "max(build_volume_temperature, max(extruderValues('material_bed_temperature')))",\n        "maximum_value_warning": "130",\n        "maximum_value": "200",\n        "enabled": "false",\n        "settable_per_mesh": false,\n        "settable_per_extruder": false,\n        "settable_per_meshgroup": false\n    },\n
\n

A probable reason why the previous suggestions fail is that the code completion will see the inclusion as some sort of conflict/default as it is not listed in the settings ("enabled = false"). When there is conflict, it attempts to resolve it based on the "resolve" field, equating to the layer_0 setting in an incorrect example.

\n

So the value can be something else than what is listed in the "value" category depending on circumstances.\nWith this addition, the value has no other choice than to compute the preheated temperature based on layer_0 temperature.

\n

For nozzle temperatures, I use fixed value which does not yet cause oozing of most materials during the small mismatch of sync period.

\n

The setting in the G-code generator is as follows:

\n
M190 S{material_bed_temperature_pre} ;Initial bed temperature, wait\nM140 S{material_bed_temperature} ; start heating the bed\nM104 S180 ;Hotend initial target, no wait\nG28 ;Home\nG1 Z15.0 F1200 ;Move the platform down 15mm\nM104 S{material_print_temperature} ;Hotend actual target, no wait\nM190 S{material_bed_temperature} ;Target bed temperature, wait\nM109 S{material_print_temperature} T0 ; wait for hotend\n
\n

So first we wait for the bed to reach preheat temperatures, which takes some time.\nThen, the bed temperature is set to target and the nozzle is to preheat levels, but in the background.

\n

During this, we moved to the home position.

\n

Finally, in sequence ultimate target for the nozzle is no wait, and then we stay until the bed temperature is final. To make sure the nozzle is at the final temperature (if it was not reached during bed heat up), we wait for it until it goes to actual printing.

\n" }, { "Id": "20756", "CreationDate": "2023-04-01T01:15:32.677", "Body": "

I'm trying to combine two different G-code files into one. The reason I'm doing this is that I'd like to print my part with different layer heights. I've managed to combine the two G-codes together, but when it gets to the layer where the layer height changes, it stops moving and keeps extruding for 10-15 seconds, creating a blob of plastic, then moves on destroying the print.

\n

I've tried using with PrusaSlicer's manual layer height feature, but their normal supports are unusable on my part and their organic supports are not compatible with the variable layer heights. Cura simply does not have this capability, and their adaptive layer height feature is not the answer. Here is what I'm trying to do:

\n
    \n
  1. Create G-code with layer height set to 0.3\u00a0mm
  2. \n
  3. Change layer height to 0.1\u00a0mm and create G-code from that
  4. \n
  5. Load 0.1\u00a0mm G-code into Notepad++ and find the layer I want to change it at
  6. \n
  7. Delete all G-code past that layer
  8. \n
  9. Load 0.3\u00a0mm G-code into Notepad++ and find where that layer would start based on Z height
  10. \n
  11. Copy/Paste the rest of the G-code into the 0.1\u00a0mm file
  12. \n
  13. Save and run on the printer
  14. \n
\n

It mostly works but the layer height change stops. Does anyone know what is happening?

\n

Here are a bunch of lines from the g-code before and after where the transition happens. I have it marked in the g-code exactly where:

\n
G1 F2400 E2234.44369\nG1 F3600 X105.992 Y179.63 E2234.66835\nM204 S2000\nG1 F2400 E2233.86835\nG1 F300 Z5.8\nG0 F9000 X106.068 Y183.796 Z5.8\nM204 S500\nG1 F300 Z5.4\nG1 F2400 E2234.66835\nG1 F3600 X99.843 Y190.022 E2234.8704\nM204 S2000\nG1 F2400 E2234.0704\nG1 F300 Z5.8\nG0 F9000 X101.42 Y192.688 Z5.8\nM204 S500\nG1 F300 Z5.4\nG1 F2400 E2234.8704\nG1 F3600 X106.635 Y187.472 E2235.03967\nM204 S2000\nG1 F2400 E2234.23967\nG1 F300 Z5.8\nG0 F9000 X107.871 Y190.479 Z5.8\nM204 S500\nG1 F300 Z5.4\nG1 F2400 E2235.03967\nG1 F3600 X104.311 Y194.039 E2235.15521\nM204 S2000\nG1 F2400 E2234.35521\nG1 F300 Z5.8\nG0 F9000 X108.544 Y194.049 Z5.8\nM204 S500\nG1 F300 Z5.4\nG1 F2400 E2235.15521\nG1 F3600 X109.947 Y192.646 E2235.20075\nM204 S2000\nG1 F2400 E2234.40075\nG1 F300 Z5.8\nG0 F9000 X120.783 Y90.912 Z5.8\nM204 S500\n;TYPE:WALL-OUTER\nG1 F300 Z5.4\nG1 F2400 E2235.20075\nG1 F1978 X120.784 Y92.216 E2235.22798\nG1 F1939 X120.794 Y92.736 E2235.23906\nG1 F1901.4 X120.794 Y122.763 E2235.89141\nG1 F1939 X120.783 Y123.279 E2235.90241\nG1 F1978 X120.784 Y124.585 E2235.92968\nG1 X120.974 Y124.314 E2235.93659\nG1 X121.199 Y123.844 E2235.94748\nG1 X121.327 Y123.323 E2235.95868\nG1 F1939 X121.362 Y122.774 E2235.9704\nG1 F1901.4 X121.362 Y92.736 E2236.62299\nG1 F1939 X121.327 Y92.172 E2236.63503\nG1 F1978 X121.201 Y91.663 E2236.64598\nG1 X120.97 Y91.177 E2236.65722\nG1 X120.819 Y90.963 E2236.66269\nG0 F1620 X120.783 Y90.912\nM204 S2000\nG1 F2400 E2235.86269\nG1 F300 Z5.8\n;MESH:NONMESH\nG0 F9000 X100 Y39.255 Z5.8\n;TIME_ELAPSED:4586.593663\n;-------------------------------------------------------------\n;-----------------Transition Happens Here---------------------\n;-------------------------------------------------------------\n;LAYER:18\nG0 X100 Y40.153 Z6.1\nG1 F300 Z6.1\nG0 F9000 X97.297 Y34.468\nM204 S500\n;TYPE:SUPPORT\nG1 F300 Z5.7\nG1 F2400 E2715.82188\nG1 F1800 X97.297 Y34.468\nG1 X97.47 Y35.08 E2715.86566\nG1 X97.834 Y36.138 E2715.9427\nG1 X98.102 Y36.794 E2715.99148\nG1 X98.421 Y37.518 E2716.04595\nG1 X98.79 Y38.243 E2716.10196\nG1 X98.96 Y38.555 E2716.12642\nG1 X99.366 Y39.232 E2716.18077\nG1 X99.605 Y39.602 E2716.2111\nG1 X100 Y40.153 E2716.25778\nG1 X100.057 Y40.232 E2716.26448\nG1 X100.343 Y40.609 E2716.29706\nG1 X100.73 Y41.147 E2716.34269\nG1 X100.934 Y41.458 E2716.3683\nG1 X101.342 Y42.136 E2716.42278\nG1 X101.476 Y42.382 E2716.44206\nG1 X101.861 Y43.147 E2716.50103\nG1 X102.193 Y43.913 E2716.55851\nG1 X102.303 Y44.19 E2716.57903\nG1 X102.602 Y45.029 E2716.64035\nG1 X102.687 Y45.297 E2716.65971\nG1 X102.939 Y46.186 E2716.72332\nG1 X103.19 Y47.204 E2716.79551\nG1 X103.692 Y47.104 E2716.83075\nG1 X103.724 Y47.394 E2716.85084\nG1 X103.799 Y48.272 E2716.91151\nG1 X103.818 Y48.652 E2716.9377\nG1 X103.839 Y49.553 E2716.99975\nG1 X103.839 Y56.239 E2717.46007\nG1 X100 Y56.239 E2717.72438\nG1 X97.297 Y56.239 E2717.91048\nG1 X97.297 Y34.823 E2719.38494\nG0 F1620 X97.297 Y34.468\nM204 S2000\nG1 F2400 E2718.58494\nG1 F300 Z6.1\nG0 F9000 X100 Y40.153 Z6.1\nM204 S500\nG1 F300 Z5.7\nG1 F2400 E2719.38494\nG1 F1800 X100 Y56.239 E2720.49244\nM204 S2000\nG1 F2400 E2719.69244\nG1 F300 Z6.1\nG0 F9000 X130 Y89.241 Z6.1\nM204 S500\nG1 F300 Z5.7\nG1 F2400 E2720.49244\nG1 F1800 X130 Y125.641 E2722.99852\nG1 X127.425 Y125.641 E2723.1758\nG1 X127.425 Y89.241 E2725.68189\nG1 X135 Y89.241 E2726.20342\nG1 X135 Y125.641 E2728.7095\nG1 X142.825 Y125.641 E2729.24824\nG1 X142.825 Y89.241 E2731.75433\nG1 X140 Y89.241 E2731.94882\nG1 X140 Y125.286 E2734.43047\nG0 F1620 X140 Y125.641\nM204 S2000\nG1 F2400 E2733.63047\nG1 F300 Z6.1\nG0 F9000 X100 Y175.345 Z6.1\nG1 F300 Z6.1\nG0 F9000 X97.278 Y181.097\n
\n", "Title": "Splicing G-code issue", "Tags": "|g-code|prusaslicer|", "Answer": "

This is completely expected from your description. You just cannot bluntly paste a different section of a G-code file into another without tweaking the interface layer (unless the extruder length is reset at every layer).

\n

You need to make sure that the extruder distance is set to the right value. Usually the extruder length is reset to zero once in a while or at layer change to prevent too large E values (G92 E0), but when you paste a layer with a mismatch in E value between the two, the extruder will extrude (or retract, this is equally valid, but in this case extrude) first before it will move further.

\n

What you need to check for is the E value, look if it is reset or not and set the value by defining the correct value with G92 Exxx, where xxx is the correct value from the pasted layer (this will be the starting point for E extrusion).

\n

From the updated question with the G-code snippet we see that the above described is actually happening, layer 17 has a final move laying down filament in

\n

G1 F2400 E2235.86269

\n

and layer 18 has the first filament deposition with

\n

G1 F2400 E2715.82188.

\n

You can see that almost 500\u00a0mm of filament is wasted (2235 -> 2715). This means that you need add

\n

G92 E2715.82188

\n

before

\n

G1 F2400 E2715.82188.

\n\n" }, { "Id": "20758", "CreationDate": "2023-04-01T09:31:04.370", "Body": "

Just started 3D printing. Switched from a 0.4\u00a0mm nozzle to a 0.2\u00a0mm nozzle and am printing my first object. It has this pattern/inconsistency. Thinking it might be temperature variation but not sure. Anyone know what it is?

\n

\"Image

\n

I changed the speed to 60\u00a0% cause not enough was coming out at a time, it improved the results. Layer height 0.2\u00a0mm but it may be a little off from because of the leveling. I increased flow 15\u00a0% just now and seems to be printing better with more adhesion.

\n", "Title": "Why does my print have this unusual pattern?", "Tags": "|print-quality|", "Answer": "

Layer height should be max. 3/4 of the nozzle width, so you should print at max. 0.15\u00a0mm layer height with a 0.2\u00a0mm nozzle. So the problem is that you are using a too large of a layer height (0.2\u00a0mm). If you push more flow it appears to work, but, for the wrong reasons.

\n

With a layer height as large as the nozzle width, you extrude cylinders/tubes with much space between the perimeters, by increasing the flow percentage you somewhat flatten the tubes, but it is better to lower the layer height.

\n" }, { "Id": "20776", "CreationDate": "2023-04-03T20:11:27.823", "Body": "

I just swapped out my MicroSwiss direct-drive extruder and hotend for a Creality Sprite Pro on my Ender 3 with an upgraded mother board to the BigTreeTech Octopus v1.1 and I'm experiencing thermal runaway on power-on.

\n

Previous (working) configuration:

\n\n

New configuration:

\n\n

Steps that I'm taking to hit thermal runaway:

\n
    \n
  1. Turn on power switch for the Ender 3 Pro.
  2. \n
\n

NOTE: Thermal Runaway Protection does not work here. The TRP detects and triggers, but Klipper doesn't seem to have any ability to control the heating tube. Instead, Klipper shuts down and the heating tube keeps running away. Each time I've had to cut the power manually.

\n

Additional things that I've tried:

\n\n", "Title": "Thermal runaway without error on power-on - Ender 3, BTT Octopus V1.1, and Sprite Pro", "Tags": "|creality-ender-3|heat-management|creality|bigtreetech|klipper|", "Answer": "

This question seems to be solved, this (community) answer is posted as a whole from comment until the OP adds a more detailed answer.

\n
\n
\n

I swapped back in the original heating tube and it had the same issue. I was able to get it all working by using the second heating element port on the Octopus board.

\n
\n" }, { "Id": "20778", "CreationDate": "2023-04-03T21:12:10.720", "Body": "

I'm working on converting an Ultimaker 2 Go to use 1.75 mm filament. So far, I have replaced the bowden tube, replaced the feeder with this one: https://www.youmagine.com/designs/alternative-feeder-1-75mm-version#information, and changed slicer/machine settings.

\n

An issue I am having is grinding at regular intervals, causing severe inconsistent extrusion. Messing with the feeder tension does not do anything. So, I am wondering if upgrading the hotend to an e3D V6 will solve it. Is there anything else to consider?\n\"enter\nI also wonder if the problem has to do with the knurled drivegear. Would this work fine with 1.75 mm filament?

\n

\"enter

\n

`

\n", "Title": "Ultimaker 2 Go - Converting to Use 1.75 mm Filament", "Tags": "|extruder|hotend|e3d-v6|ultimaker-2|", "Answer": "

Figured this out a while ago, but I thought I might as well update this.

\n

So the issue was simply there was not enough tension, and that pretty much solved all the problems. I also did end up using an E3D v6 1.75\u00a0mm hotend.

\n

Right now, I'm working on making it print faster. I am using klipper on a Raspberry Pi Zero W, and I'm playing with different fan ducts to improve part cooling.

\n" }, { "Id": "20784", "CreationDate": "2023-04-05T17:01:32.697", "Body": "

After fiddling with the settings on my Cura profile, my Ender 3 S1 is now messing up my prints in a weird way. While printing, it just pauses at a corner on all the layers of the print for a second or so, leading to large blobs of material building up there. What setting(s), if any could be causing this?

\n

As a note, my Cura settings are anything but default. I mess around with a lot of different options to try to optimize my prints. I could reset the profile to default, but I'd rather keep most of my settings.

\n

Here is a picture of the issue:\n\"issue\"

\n", "Title": "What Cura setting is messing up my prints?", "Tags": "|creality-ender-3|print-quality|ultimaker-cura|troubleshooting|", "Answer": "

Similar to what @kosteklvp suggested, it turns out the problem was related to the SD card I was using. It was a microSD card with an adapter, which may have been the problem. After swapping it out with for a different SD card, the printer now functions properly.

\n" }, { "Id": "20788", "CreationDate": "2023-04-06T17:38:22.200", "Body": "

So, the G-code preppended to my slicer output (by Cura itself) looks like this:

\n
; Ender 3 Custom Start G-code\nG92 E0 ; Reset Extruder\nG28 ; Home all axes\nM104 S{material_standby_temperature} ; Start heating up the nozzle most of the way\nM190 S{material_bed_temperature_layer_0} ; Start heating the bed, wait until target temperature reached\nM109 S{material_print_temperature_layer_0} ; Finish heating the nozzle\nG1 Z2.0 F3000 ; Move Z Axis up little to prevent scratching of Heat Bed\n;G1 X0.1 Y20 Z0.3 F5000.0 ; Move to start position\n;G1 X0.1 Y200.0 Z0.3 F1500.0 E15 ; Draw the first line\n;G1 X0.4 Y200.0 Z0.3 F5000.0 ; Move to side a little\n;G1 X0.4 Y20 Z0.3 F1500.0 E30 ; Draw the second line\n;G92 E0 ; Reset Extruder\n;G1 Z2.0 F3000 ; Move Z Axis up little to prevent scratching of Heat Bed\n;G1 X5 Y20 Z0.3 F5000.0 ; Move over to prevent blob squish\n
\n

But this code seems to be causing issues where it ignores the ABL (auto bed leveling.)

\n

In my case, this means that the extruder starts clicking, and I suspect that with the particular hotend I have, this causes issues. For now, I have commented out all the code to draw the primer lines, as you can see.

\n

Is there a way to make them respect auto-leveling?

\n", "Title": "CURA prints some primer lines, but they don't seem to respect bed automatic bed leveling", "Tags": "|creality-ender-3|bed-leveling|g-code|automatic-bed-leveling|", "Answer": "

You should add M420 S command after G28 to Start G-code.

\n

According to Marlin docs, G28 command disables bed leveling by default.

\n
\n

G28 disables bed leveling. Follow with M420 S to turn leveling on, or use RESTORE_LEVELING_AFTER_G28 to automatically keep leveling on after G28.

\n
\n

Alternatively, you can change the RESTORE_LEVELING_AFTER_G28 value to true in firmware.

\n" }, { "Id": "20810", "CreationDate": "2023-04-10T09:41:49.927", "Body": "

As these are the two most commonly used extruder systems in FDM printers, I would like to know what is the difference between them. What are the advantages and disadvantages of using each of them?

\n", "Title": "What is the difference between a Bowden extruder and a direct drive extruder?", "Tags": "|extruder|bowden|knowledgebase|direct-drive|", "Answer": "

A Bowden extruder and a direct drive extruder are two different types of extruder systems used in FDM 3D printers. Both feed filament into the hotend, but differ in the way the filament is pushed into it. The main difference is the location of the extruder motor relative to the hotend, which results in some pros and cons in the usage of both of them.

\n
\n
\n

Bowden extruder

\n

A Bowden extruder has the motor located remotely from the hotend, usually on the printer's frame. Filament is fed from the motor to the hotend through a Bowden tube. This setup is typically used in 3D printers that require high speeds and accelerations, as the reduced weight of the hotend allows for faster and more accurate movement.

\n

\"Visualization

\n

The main advantage of a Bowden extruder is that it reduces the weight of the printhead. It can improve print speed and accuracy, especially for faster print speeds. It also reduces the amount of space required in the printhead, allowing for larger hotends or additional features to be added.

\n

However, Bowden extruders can suffer from problems with filament control, such as oozing or stringing of the filament. The friction in the Bowden tube and distance between the extruder and the hotend causes the reaction time of each motor movement to be extended.

\n
\n
\n

Direct drive extruder

\n

A direct drive extruder has the motor located directly above or beside the hotend, which eliminates the need for a Bowden tube. The Bowden tube is typically very short or totally absent. This setup is used in printers that require precise filament control and are capable of printing with a wide range of materials.

\n

\"Visualization

\n

The main advantage of a direct drive extruder system is that it minimizes the distance between the extruder and the hotend. It allows for better control of the filament and reduces friction due to a shorter distance between the extruder and the hotend. This allows to use smaller values of retraction settings, which can reduce stringing and oozing. It results in better control of demanding filaments. Direct drive extruders can handle flexible and soft filaments more effectively.

\n

However, the added weight of the extruder motor can result in slower printing speeds and reduced accuracy during high-speed printing. A heavier printhead can also cause artifacts and ringing on the surface of the prints.

\n
\n
\n

In summary, Bowden extruders are better suited for high-speed printing and have a lower risk of filament contamination, while Direct Drive extruders are better suited for more precise printing and more demanding materials. The choice between these two systems ultimately depends on the specific requirements of the printer and the printing application. Costs also have to be taken into account, because direct drive extruders are relatively more expensive than Bowden extruders.

\n\n" }, { "Id": "20812", "CreationDate": "2023-04-10T14:47:28.193", "Body": "

We have an old Anet A8 3D printer. It took us a while to fix it and get it running. But when it prints, the filament curls up and sticks to the nozzle end and doesn't stick to the bed.

\n

Sometimes it doesn't curl up, if I wipe the nozzle after the printer starts. But when it prints, a portion of the filament sticks to the bed, but as the head moves, the whole thread of the filament dries up quickly and drags the part that did stick with it, messing everything up and curling up the nozzle like the first case.

\n

My Anet A8 has a heated bed, but although the bed heats up, it seems like the filament dries up too quickly. Is that the problem, how to fix it ?

\n", "Title": "Filament curls up in the nozzle and doesn't stick to the bed", "Tags": "|troubleshooting|adhesion|anet-a8|", "Answer": "

It looks like it is a problem with temperature. After changing filament temperature to 260\u00a0\u00b0C it prints excellent now. Printing PLA at 215\u00a0\u00b0C also works pretty well. Fresh new spool does magic or PLA does magic.

\n" }, { "Id": "20815", "CreationDate": "2023-04-10T20:06:02.577", "Body": "

I am having a lot of troubles with running my Prusa Mk3S at my student dorm, because of the extremely loud stock part cooling fan - it annoys people A LOT). So I started working on a design using one or two Noctua 40x20 axial fans (2 to get more static pressure) as part cooling fans.

\n

So I wanted to ask if it/they would provide sufficient cooling to replace the centrifugal fan currently on the extruder assembly.\nDoes anyone have some experience on these kinds of builds?

\n", "Title": "Is a 40x20 Noctua fan with a shroud good for part cooling?", "Tags": "|prusa-i3|cooling|print-fan|", "Answer": "

This probably does not make sense.

\n

Noctua fans are quiet because they're really low-flow, and aside from really high-power (and noisy!) models like 4028 server fans, axial fans in general are unsuitable for part cooling because they have essentially no static pressure, meaning you can't run them through narrowing ducts. If you can just mount them to blow directly where you need the air, without a duct compressing it at all, they might work, but this tends to lead to a bulky toolhead that collides with ends of the axis, reducing your build volume, or that's cantilevered in ways that subjects the whole toolhead to vibrations.

\n

I would look for a quieter radial blower fan that meets your needs. Note that the high quality dual-ball-bearing fans are generally a lot noisier than the garbage sleeve-bearing ones; the advantage is that the bearings don't wear out after a year or so. But if your goal is minimizing noise, you might do best to just buy new sleeve-bearing fans when they wear out. Knowing Prusa, they probably put the high quality fans on there, which would be why they make more noise.

\n" }, { "Id": "20818", "CreationDate": "2023-04-12T09:05:18.153", "Body": "

I tried to print my first object, which is the calibration cube downloaded from here, XYZ 20mm Calibration Cube.

\n

However, the output has a visible infill on all sides:

\n

\"Photo

\n

I think this is because some settings are not correct in the slicer.

\n

I am using:

\n\n", "Title": "Print output missing sides", "Tags": "|creality-ender-3|ultimaker-cura|troubleshooting|slicing|infill|", "Answer": "

It looks like perimeters and solid layers are not set in the Slicer. Make sure that these settings are greater than zero:

\n

\"Settings

\n" }, { "Id": "20820", "CreationDate": "2023-04-12T14:55:49.610", "Body": "

I have a Ender3V2 Creality 3D printer. I recently changed the extruder for a metal one because the previous was broken.

\n

I have troubles to print now. The adhesion layer peels off easily (kinds of bubbles inside that lift it) and the overall quality is poor.

\n

While looking for a solution, I noticed that my extruder was not calibrated correctly. However, following the standard procedure does not fix my problem. Below is what I did and the results:

\n

\"Extruder

\n

The PTFE tube is disconnected to avoid eventual nozzle problem.\nI asked for 10 cm extrusion severals times, and the results were as follows (distance in mm):

\n

\"Experimentation

\n

Ratio is Transmission ratio E, that I found in Control -> Motion -> Transmission ratio -> Transmission ratio E.

\n

I don't understand these results. The initial ratio change should have corrected the under-extrusion. Returning to 93 should also have given me the same result as initially.

\n

I didn't use any G-code, I just used the printer panel (for both ratio changing & asking for extrusion).

\n

I followed this video, How TO Calibrate The Esteps On A Direct Drive Extruder The EASY Way!

\n

\r\n \r\n

\n

and used this EStep Calculator \u2013 V1.1.

\n", "Title": "Creality Ender3V2 extruder calibration", "Tags": "|creality-ender-3|extrusion|calibration|", "Answer": "

Thanks to @kosteklvp's instructions in commentary, I was able to correctly calibrate my extruder.

\n

Using a third-party app (octoprint via raspberry pi in my case) gave me more flexibility with settings.

\n

Following @kosteklvp's proposed procedure (at this link) allowed me to achieve my calibration.

\n

I still have other troubles while printing, but no more related to my extruder.

\n" }, { "Id": "20821", "CreationDate": "2023-04-12T16:56:04.757", "Body": "

I got my first 3D printer this week, a Creality Ender 3.

\n

Tried to print a few small samples to test it, but every shape I printed had an extra external layer. Please, check these images:

\n

\"Photo

\n

\"Photo

\n

I used Freecad to draw the objects, and Slic3r to convert them to G-code.

\n

Am I doing something wrong? Is my printer not a good one?

\n

Any comment will be very helpful.

\n\n", "Title": "Extra external layer (shell) on my first prints", "Tags": "|creality-ender-3|troubleshooting|underextrusion|", "Answer": "

If you do some quick calculations it appears that this is caused by under-extrusion.

\n

Comparing the extruded volume to the calculated volume for the deposition there is a mismatch of about 2.6. This hints to the use of an incorrect diameter for the filament $\\frac{2.85^2}{1.75^2}=2.65$.

\n" }, { "Id": "20823", "CreationDate": "2023-04-12T20:25:28.247", "Body": "

I'm using a 65W heater cartridge and T-D500 type NTC thermistor. This setup has been working fine since I installed it sometime last year. Suddenly today, the temperature reading started jumping around 5-25\u00b0 below the set temperature, but only while printing. It holds temperature solid at idle, and PID autotune produces numbers like normal.

\n

How should I go about diagnosing what component is bad? Is it necessarily the cartridge or thermistor (or rather their wiring) or could it be noise or voltage instability or something else?

\n", "Title": "Diagnosing sudden problem with hotend temperature", "Tags": "|troubleshooting|hotend|thermistor|thermal-runaway|", "Answer": "

Well, it could be a bad thermosensor...

\n

Bad Thermosensors do show themselves in multiple ways, among them fluctuating their readings before fully failing.

\n

Thermosensors have ranges

\n

Every thermosensor comes with a range for which it actually can operate reliably. For example, the TS2289 is rated for -40 to 90\u00a0\u00b0C, and thus would be not usable for a 3D printer at all, while TS2452 is rated for -100 to 600\u00a0\u00b0C.

\n

The glass bead sensor on a Creality Ender3 is a \u200entc 100k thermistor. That is only a class, and it contains multiple entries. This is not actually a specific sensor but a class that contains multiple sensors. For example, NTC 3950 is listed as -40 to 300\u00a0\u00b0C, which means it might be barely useable in that area, yet there might be larger fluctuations than normal as the resolution at the ends of the measuring range gets higher. In about the same dimensions, you might however also get a very similar sensor like thos from the Datasheet of the NTC Type BR Series. This document lists BR11/14/16/25 with a ceiling of 200\u00a0\u00b0C and B32/42/55 with a ceiling of 300\u00a0\u00b0C, all of which are in a very similar size range.

\n

Not all sensors are glass-bead style, like the standard e3D v6 Thermosensor cartridge, which is listed as a Semitec 104NT thermistor the ceiling listed as 300\u00a0\u00b0C, but the same form factor also fits other cartridges. The sensor listed by e3D actually does come as a glass bead style too, listed as -50 to 300\u00a0\u00b0C

\n

Such other sensors might be PT100B or PT1000 are claimed to accurately work up to 450\u00a0\u00b0C and might operate reliably even at 500\u00a0\u00b0C. A similar E3D Thermosensor is the PT100, and would also operate fine in that range.

\n

Simply put: to reliably print at those high temperatures, you might want to change the thermosensor for a type that actually does operate reliably in those high-temperature areas.

\n

P.S.

\n

I do not use the word thermistor (or thermocouple) here, because those describe specific types of sensor. The more generic term Thermosensor describes anything from a simple thermometer, over thermistors and thermocouples to an IR sensor.

\n" }, { "Id": "20830", "CreationDate": "2023-04-14T20:47:05.893", "Body": "

Going through the printer options, I came across the Fade Height option under Bed Leveling section.

\n

\"Fade

\n

What is it and what is its use in Marlin firmware?

\n", "Title": "What is the Fade Height option in Marlin?", "Tags": "|marlin|bed-leveling|", "Answer": "

Fade Height is used to specify the height above the bed at which the printer should stop fading the bed leveling correction. Bed leveling is gradually decreasing with each successive layer until fade height is reached. From this height, bed leveling compensation becomes turned off.

\n

\"Presentation

\n

For example, if the bed is slightly warped or uneven, the printer may need to compensate by lowering the nozzle at the center of the bed but less so near the edges of the bed. By setting Fade height, the printer will gradually decrease the correction as it moves away from the center of the bed, resulting in more accurate print.

\n
\n

According to Marlin docs:

\n
\n

With Fade height enabled, bed leveling correction is gradually reduced as the nozzle gets closer to the Fade height. Above the Fade height no bed leveling compensation is applied at all, so movement is machine true.

\n
\n" }, { "Id": "20841", "CreationDate": "2023-04-17T07:48:21.847", "Body": "

I have an organic but more complex STL model. It is so detailed and every detail is important to us.

\n

We have a remesh tool in Meshmixer. It says:

\n
\n

With Meshmixer, you can create an even tessellation using the Remesh command. Click the Select Icon in the menu and click to create an area to be remeshed, or use Ctrl + A to select the entire object. On the Popup menu, click Edit \u2192 Remesh or hit R.

\n
\n

Question:\nWhen applying this tool to my STL model, does it happen that any detail will be gone off? Or, will I get same complex model?

\n", "Title": "With remesh tool in Meshmixer; What happens to the details of the model?", "Tags": "|stl|meshmixer|", "Answer": "

While doing this, you may encounter the loss of details. To avoid this, precise settings must be made. For example,

\n\n" }, { "Id": "20847", "CreationDate": "2023-04-20T08:37:48.857", "Body": "

After encountering an issue where the original heater cartridge (~40\u00a0W) for my generic MK6 hotend was unable to reach/maintain stable temperature at ~260\u00a0\u00b0C (for PETG) I replaced it with a higher wattage heater (60\u00a0W). The new heater heats up faster and can maintain temperature, but I noticed that if I try to print too fast it will become unstable and the printer stops with a heater error, even with a lower temperature of 250\u00a0\u00b0C. This can be seen in the temperature plot below.

\n

I have performed PID tuning (see the nicely flat spots on the temperature plot), so I don't think that's the issue. The thermistor is well-secured to the block.

\n

I am using a BTT SKR Pico V1.0 control board with Klipper over UART to a Raspberry Pi A3.\nDoes anyone have experience with an issue like this? Could it be that the SKR Pico is not supplying sufficient power while also running the motors and heated bed?

\n

\"HeaterFailure\"

\n", "Title": "Print failure due to heater inconsistency", "Tags": "|troubleshooting|hotend|temperature|", "Answer": "

This is very similar to what I recently experienced in the question Diagnosing sudden problem with hotend temperature, where the root cause seems to have been a bad thermistor - internal conductors probably broken in a way that they intermittently make reduced contact during toolhead motion. But I didn't have a problem while heating like your graph shows, only during motion, so it's also plausible that the wiring to the heater is bad. Since these aren't expensive components it might make sense to just replace the thermistor, heater cartridge, and any asssociated wiring if you don't want to spend time fighting with the problem.

\n" }, { "Id": "20851", "CreationDate": "2023-04-20T19:38:59.363", "Body": "

I've got a 12\u00a0V bed (MK2A) and a 24\u00a0V PSU (Meanwell 350). I've also got a 12-24\u00a0V Einsy Rambo controller, and I'd like to run my hotend at 24\u00a0V. My HEVO is still in development, so when I tested the bed, anything over 32\u00a0\u00b0C restarted the board (I forgot it was 12\u00a0V).

\n

Must be a voltage drop from the bed draw? The Einsy has a dedicated rail for the bed, so I was thinking I could use a dropdown converter between the PSU and the board rail. So I could continue to run the 24\u00a0V PSU for everything else and 12\u00a0V for the bed. Is it a bad idea from a safety perspective to run a simple 24\u00a0V->12\u00a0V converter in line there?

\n

Also, what would that do to my amperage, I read something about converting volts down causing high amps and wattages requirements. Is that true?

\n

UPDATE:

\n

I wanted to check if changing the bed config in Marlin would allow me to reduce the voltage and it appears that if I change MAX_BED_POWER to 127 (half) I can indeed run the bed as is. I can bring it up to 70 degrees at least at the moment. Admittedly I'm not sure what this does exactly and if this is safe. Thoughts?

\n

This thread appears to discuss exactly my problem and my temporary solution with MAX_BED_POWER limit too.

\n

https://reprap.org/forum/read.php?4,252638

\n

Here's a similar discussion

\n", "Title": "Is it a bad idea to run 24 V to 12 V step down to heated bed?", "Tags": "|heated-bed|power-supply|", "Answer": "

It will come with issues

\n
    \n
  1. Efficiency of good step down converters is 90%. Cheap ones used to start at 70%. What that means is:\n\n
  2. \n
  3. Additional point of failure:\n\n
  4. \n
  5. Lower voltage means thicker or hotter cables.\n\n
  6. \n
\n

It is your choice if you are OK with these drawbacks. I would not, but none of these screams "don't do it!". It's quite probable that you will be able to pull it off and make a decent printer.

\n" }, { "Id": "20876", "CreationDate": "2023-04-26T02:02:14.913", "Body": "

My CoreXY style printer doesn't have perfect belt alignment with its pullies, so there is some occasional squeaking on at least one pulley where the belt rubs against a flange. Since I don't have an easy way of adjusting the pulleys (maybe some tiny shim washers) I was hoping to just apply some oil to minimize the wear.

\n

Could I just throw some 3-in-1 oil on the belt? Is there a better lubricant to use? I doubt that the belt would start skipping, but I am worried about damaging the belt due to some chemical incompatibility with the oil.

\n

The belt is the most generic GT2-style timing belt you would find on a 3D printer, probably the same as this HICTOP 3D Printer Belt.

\n", "Title": "Can I lubricate the timing belts to prevent squeaking against pulleys?", "Tags": "|maintenance|linear-motion|belt|", "Answer": "

The pulleys on nearly every 3D printer I've seen are held in place on the stepper motor shaft with a grub screw. A bit of rotation of the pulley should bring into view this screw, which will use a fairly small hex wrench. Loosen the screw and slide the pulley in the direction appropriate to relieve the friction.

\n

\"pulley

\n

photo courtesy random selection from pinterest

\n

If you are unable to access the screws and choose to lubricate the belt, consider to use a non-reactive PTFE (teflon) type or silicone type substance and apply it sparingly to the belt edges. If the teeth of the belt and pulley receive the substance, you may not be able to avoid belt jumps under high acceleration movement.

\n" }, { "Id": "20888", "CreationDate": "2023-04-30T16:37:30.113", "Body": "

I have recently acquired a 'CraftBot XL' 3D printer (sold by CraftUnique), which is a CoreXY 3D printer with linear rails. It is a second-hand purchase and the seller were a company that had several. This particular unit has an issue where sometimes during X-axis movement, the Y-axis would move as well. They replaced the stepper motors, but the problem persisted so they opted to sell the machine.

\n

I have found a possible cause of this issue. When I tried to move the carriage around manually, I noticed a particular heavy resistance / "drag" on approx. 1/3rd to 1/2 of the X-axis motion, toward the left extent of the build space. The remainder of the X axis and the Y and Z-axes are completely free to move as they ought to.

\n

The force required to move the carriage in the problematic part of the X-axis is (subjectively) quite large. Much much larger than any other movement. I believe that during some combined XY-manouevers (diagonal movement), this force is too large, causing the carriage to only move in the Y-axis, resulting in errant behavior (at best: layer shift, at worst: carriage getting stuck and the printer needing to be reset). The behavior arises during homing often, as the home position is in the problematic area. An additional reason that I think the X-axis force is too large is that, when this problem occurs and I ever so slightly help the carriage along the X-axis with my hand, the movement can complete without issue.

\n

Steps I have undertaken to no avail yet:

\n\n

This is a diagram of the setup:

\n

\"CraftBot

\n

What else could be causing this? I don't hear any out of place noises like metal grinding.

\n

The next thing I want to check is the squareness of the gantry. I think I know how to measure it, but I'm not sure how to adjust it if it's off. Any help there?

\n", "Title": "CraftBot XL (CoreXY) part of x-axis movement resistance / high force required", "Tags": "|linear-motion|belt|corexy|carriage|", "Answer": "

I'm going to answer this myself, as I believe I have found the issue. It was purely a mechanical issue.

\n

On this particular model, the extruder motor rides just on top of / above the crossbar that the linear rail is mounted under. The extruder motor was mounted very close to this crossbar, there was barely any clearance between the two. I figure that due to a slight deviation in height or non-squaredness in the ZY-axis, the motor started to heavily rub on the bar toward the left extent of the X-axis. I tested with a piece of paper, which I slid between the motor and the bar. This immediately caused the same "resistance" I was feeling before, but now everywhere along the X-axis.

\n

After re-mounting the extruder motor about 1 mm higher, which the frame already allowed for, there is no longer any improper resistance noticable along any axis:

\n

\"Adequate

\n" }, { "Id": "20890", "CreationDate": "2023-05-01T18:56:20.907", "Body": "

I can't seem to find wiring diagrams anywhere, so I am asking here.

\n

Which pins do I use to connect my Arduino Uno to my Bigtreetech board?

\n

I am connecting the Arduino to use it as an ICSP programmer, so that you can flash the BigTreeTech board. I have a V3 board version.

\n", "Title": "How to connect Arduino Uno R3 to Bigtreetech SKR Mini E3 V3", "Tags": "|bigtreetech|wiring|arduino|", "Answer": "

First, make sure that you have flashed your Uno to be an ICSP programmer.

\n

Then you just connect up the SPI pins of the two boards.

\n

The pins on the Uno that you must use, are these, circled in yellow (you can ignore the red circles):

\n

\"Uno

\n

The pin out is as follows:

\n

\"Arduino

\n

The pins on the BigTreeTech board are shown on the right hand edge/side of the board, half way down, where it says SPI1:

\n

\"BigTreeTech

\n

The SPI pins on the v3 version of the board are still in a similar location, but in a different order (along with 2 other pins, that can be ignored):

\n

\"BigTreeTech

\n

Connect these pins on the Uno, to the "like-for-like" pins on the BigTree - no crossing over of wires is required. So, connect:

\n\n

On the V3 board ignore the additional NSS and 3.3V pins on the SPI1 connector

\n
\n

References

\n\n" }, { "Id": "20897", "CreationDate": "2023-05-03T12:52:46.613", "Body": "

This is kind of hard to explain but I have a 2\u00a0mm disc that has text on the top, it's a stamp effectively. I want my text to start right on the top after the top layer of the base (if that makes sense), but it's starting the text a few layers down into the top which is making the infill lines a right mess

\n

The text seems to start a few layers down in the top of the base, and it's causing it to look a mess as it's going around it all.

\n

\"3D

\n

\"Zoomed

\n", "Title": "I am pulling my hair out, why does my text start a few layers in to the top of my base?", "Tags": "|creality-ender-3|layer-height|print-failure|", "Answer": "

Your problem has to do with the top layers of your part surrounding the raised text. The infill for your raised text continues a bit because it's not yet close to the top.

\n

The print becomes a mess because the inner walls for the raised text don't hit any infill material like the top layers in this Benchy: \"Example

\n

To remedy this, you can try several different things.

\n
    \n
  1. Connect Infill Lines. Assuming you use the Cura slicer, you can use the Connect Infill Lines option which is specifically designed to create extra support for top layers.\n\"enter\nIf the option isn't visible yet you can find it in the additional Infill settings:\n\"enter

    \n
  2. \n
  3. Increase infill density. You can increase the infill density ans check if the corners hit (or are very close to) the infill after slicing. In the example below, I raised the infill density from 10% to 25% and all corners are well supported by the infill. \"increased\nThis is often not the best solution because it will severely increase the print time and used material.

    \n
  4. \n
  5. Change infill pattern. You can play around with different infill patterns to see if another pattern is more favourable for the unsupported walls. I changes the infill pattern to Lines in this example and it supports the top layers a tiny bit better.\n\"enter

    \n
  6. \n
  7. If all else fails, you can add another piece and set the Per Model Settings for the overlapping regions. I added a simple cube and increased the infill density for the overlaps to 50% and got this result:\n\"enter

    \n
  8. \n
\n

Of course there are al lot more things you can try but these are some options that have worked for me in the past. Hope it helps!

\n" }, { "Id": "20904", "CreationDate": "2023-05-05T08:14:46.670", "Body": "

Can you suggest me some good material choices for printing some mountains models for educational and fun purposes?

\n

My goal is to mantain the best accuracy without spending too much, I have many models to print: they don't need to be super-resistant.

\n

I was thinking about High-Detail Resin or simple PLA / Recycled rPLA.\nI also checked Rigid Resin and Color-Changing PLA.

\n

Here are some examples of my models:\n\"mountain\n\"mountain

\n", "Title": "Best materials for printing mountains", "Tags": "|3d-models|print-material|dimensional-accuracy|", "Answer": "

Material is not really going to affect the accuracy unless you're using very exotic things like lightweight foaming materials where the expansion is variable. You don't even have overhangs. So accuracy is pretty much entirely a function of the process, the quality of your machine, and the settings you use.

\n

Resin will generally have higher resolution than FDM if that's what you're looking for. With FDM you can increase resolution somewhat by using a smaller nozzle, but that will make large prints like this very slow. Within the broad classes of resin or FDM, though, which specific resin or which specific polymer or color or brand you use is not going to make a difference. Pick the one that meets your aesthetic needs.

\n" }, { "Id": "20922", "CreationDate": "2023-05-10T08:07:26.050", "Body": "

I am interested in 3D printing projects and I use the Cura program for that purpose. Lately, when I preview my models in the Cura program to prepare for printing, I encounter an error and my prints come out distorted. I am using version Cura 4.10.0. Does anyone have experience with this version? Could there be an issue with the integrity or the geometric compatibility of my files in terms of dimensions and size? If anyone has experienced a similar problem, could you please help me?

\n

Thank you in advance.

\n", "Title": "Preview bug in Cura 4.10.0", "Tags": "|ultimaker-cura|", "Answer": "

You are using a fairly old version of UltiMaker Cura, you should try a more recent version of Cura to test if it happens in the newer version as well.

\n

Currently, at the time of writing, the latest version of Cura is 5.3.1.

\n" }, { "Id": "20935", "CreationDate": "2023-05-11T21:41:25.310", "Body": "

I've recently gotten into Resin 3d printing with a refurbished Elegoo Mars 1 but I've run into a weird bug where the printer would not expose any resin to the LCD.

\n

I tested it by taking off the build plate and basin and looked at the LCD when it was exposing the layers. I've tried updating the firmware and tested about 20 odd prints of various sizes and detail. With those I saw that prints with large individual layers essentially stopped the LCD entirely (The exposure would flash on the LCD for a second then the LCD would not turn on again until I completely restarted the printer). What is more bizarre is that the first test print I did worked perfectly and still does for some reason (which was the tugboat and a random miniature).

\n

For specific printer details I have an Elegoo Mars with the C type motherboard. There are no burn marks on the wiring or circuits and the LCD screen was replaced recently with a brand new one. And nothing is misaligned or unplugged. I even double checked that the LCD was still functional with the test exposure tools on the printer.

\n

From my observations it is a data streaming issue with how it is connected with the USB drive. But that is a software issue that I am not capable of knowing.

\n

Any ideas on what could be wrong or if this has happened before?

\n

EDIT: Discovered that the Firmware needed to be updated to the latest version. Also, after testing out several very different print files the original print file that was having this issue was corrupted in some way as rebuilding it and printing the new version more or less worked. I also did another go around with cleaning everything and making sure that both the plate was level/tightened, FEP replaced, and that there was no IPA leftover on either.

\n", "Title": "Resin printer LCD breaks if layer is too big", "Tags": "|resin|lcd-screen|elegoo-mars|", "Answer": "

If the user, Illogical Consistency (who suggested the edit) posts their own answer, then this wiki answer can be deleted

\n
\n

From this (rejected) suggested edit:

\n
\n

I had to dig deep into several other websites and found out that I had to partition my newer 32gb usb with a FAT32 4096 partition and only put the .SH4 .CBD and .TXT files in the root. On then would the firmware update properly.

\n
\n

From this comment:

\n
\n

I dug deep into several other websites and found out that I had to partition my newer 32gb usb with a FAT32 4096 partition and only put the .SH4 .CBD and .TXT files in the root. Then the firmware finally updated

\n
\n" }, { "Id": "20936", "CreationDate": "2023-05-11T21:41:56.253", "Body": "

\"shape\"

\n

I would like to print the outer contour without extraction, and after that print the straight lines between the contour. I attached a picture how Cura does the slicing. It does several extractions from the outer contour to print the straight lines.

\n

Is there a setting so that Cura slices so that the outer wall is done first, without extractions and finally does the straight lines in between?

\n

I am not looking for "vase mode", but almost. I would prefer a nice outer contour.

\n", "Title": "How to print the outer contour before inner lines with Cura?", "Tags": "|ultimaker-cura|slicing|", "Answer": "

As far as I understand Cura, you can't. It optimizes the path, this isn't always how we would think or like it to be.

\n

To fix this, you'll need to write your own G-code, for such a simple geometry this should be very doable if you have some programming skills. It is printing an elliptical shape with cross lines, the latter all having a little offset from the previous layer.

\n
\n

With limited programming skils you could also achieve some success. When the current sliced G-code is inspected in e.g. an online G-code interpreter/visiulizer, you could extract the correct path for the elliptical circumference (cutting out/move the parts that break the printing of the circumference as a whole) to ensure continuity of printing the outer elliptical shell. Note that you need to take care of the E parameter in the code as this must be reset e.g. using (G92)[https://reprap.org/wiki/G-code#G92:_Set_Position] to the correct starting point for extrusion. Once obtained you have a continuous printing code for the outer shell. Now you need to add the cross line paths to be placed after a move to the correct position. It would be beneficial to have the formula of the elliptical shape to determine the length of the cross line and the position it needs to start, otherwise you need to move the cross line printed parts here. This may be a lot of work to achieve.

\n" }, { "Id": "20954", "CreationDate": "2023-05-15T17:18:15.680", "Body": "

A small [7\u00a0cm long x 5\u00a0cm high x 10\u00a0cm deep] hard plastic piece of my old luggage broke (see attached). The manufacturer does not have spares for this old model.

\n

\"Broken

\n

Is there any way to scan the working piece on the left with an Android app and order a 3D printed one-off one?

\n", "Title": "Printing spare plastic parts", "Tags": "|scanning|", "Answer": "

I used some thermoplastic. It does not look exactly like the original as I had to give it shape with my own hands, but certainly does the trick. It also costs less than $10 USD. Let's see how long it lasts.

\n

\"Fixed

\n" }, { "Id": "20973", "CreationDate": "2023-05-19T20:28:16.370", "Body": "

I'm printing for the first times on my new Sovol SV04, using their PLA @ 210 \u00b0C (after printing a temp tower).

\n

The walls and top layers are almost perfect, but the infill has a lot of gaps between layers:

\n

\"enter

\n

I'm not sure which parameter needs to be tuned, since the walls are ok and I'm afraid to make a mess...

\n

What could cause this behavior?

\n", "Title": "Gaps in infill only - walls are perfect", "Tags": "|print-quality|ultimaker-cura|pla|infill|", "Answer": "

The infill speed might be set too high, but it's possible this is just some of the normal badness of self-intersecting infill like "grid" and "triangles". If you notice, these patterns have the extrusion path cross back over itself once or twice at the same points. Hitting this barrier of material already being there when the nozzle gets to it causes a rapid spike then drop in backpressure in the nozzle, and tends to make material push upward around the sides, and then underextrude right after the intersection point. This could be the cause of what you're seeing.

\n

If so, there are several possible ways to improve the situation:

\n\n

If this isn't the issue, other things to check for:

\n\n" }, { "Id": "20979", "CreationDate": "2023-05-21T07:08:24.087", "Body": "

I'm currently using Cura for one printer (Hellbot Magna SE, essentially an Ender clone) and PrusaSlicer for another (Artillery Hornet).

\n

I first learned this method for multicolored prints with Cura on CHEP's channel since I have had awful accidents with manual filament changes in the past, given that my Hellbot printer doesn't understand the M600 command and I have to use M0 pauses instead. this way works great and gives me a lot of peace of mind since now I can leave the printer unattended for a bit knowing it will finish printing then turn the nozzle and bed off, home on all axes and just sit there idle until I come back and start the next print on top, this one sliced with the corresponding Z Offset value so as to not collide with the one currently sitting on the bed.

\n

I would like to do the same on my Hornet, but I use PrusaSlicer with it since the builtin profile is so much better, and I can't figure out how to do it. I have located the Z Offset setting, but the only way to be able to lift objects from the platter seems to be by making them "parts" of another one, and in that case I lose the option to disable the "Printable" flag that lets me hide the different components of the object so I can slice them and export them as separate files.

\n
\n

Note: While typing this it just occurred to me maybe setting the\ncorresponding Z Offset for the object that goes on top of the one\nalready on the bed without making it a part but rather the only\nthing on the platter might work? Perhaps this isn't necessary for\nPrusaSlicer to generate the code that I need. I'll test this tomorrow\nbut anything that implies possible collisions will always make me a\nbit nervous. I will update the question with the result of the test.

\n
\n

Thanks in advance!

\n", "Title": "Trying to print separate models one on top of the other using PrusaSlicer", "Tags": "|prusaslicer|multi-material|", "Answer": "

I'm gonna answer myself since I tested my hypothesis today.\nTurns out in PrusaSlicer you don't need to "elevate" the model from the build plate and make it sit on the other model in order to be able to print on top of it. Just the Z Offset option is enough.

\n

Here's what I did:

\n
    \n
  1. Load the first part that will sit at the bottom.
  2. \n
  3. Right click it and Add Part(s). This will be just for reference.
  4. \n
  5. Take note of the height of each part (this will be the Z Offset for the next print)
  6. \n
  7. Remove all parts and import the files as separate models.
  8. \n
  9. Now you can generate separate files for each of them. First one will of course have an offset of 0, next one will have an offset equal to the previous ones' heights, and so on.
  10. \n
\n

As an important note, only have your bed on for the first print, afterwards always leave it off as this will start to deform the print from below. I recommend you use glue stick or hairspray to help the adhesion instead.

\n" }, { "Id": "20996", "CreationDate": "2023-05-29T02:57:57.827", "Body": "

I recently got a used Creator Pro printer, and I just did a few prints amounting to about 10 hours of total print time, with no issue. But suddenly, the extruder started jerking filament back and forth, not feeding. If I force the filament with my hand, it goes through the hot extruder easily, and when I do filament loading, this still happens. I disassembled the extruder to make sure the motor isn't stuck on anything, and even with nothing touching the shaft, it does this wiggling when I tell the printer to load the filament. I doubt the sequencing for the motor spontaneously changed after hours of printing. I pulled the cable and plugged it back in, same thing. The firmware is Sailfish if that matters. I'm beginning to suspect the stepper motor is toast, but I wanted to ask here first if anyone thinks it could be something else.

\n", "Title": "Extruder motor jerking back and forth", "Tags": "|extruder|stepper|", "Answer": "

Plug the extruder cable into one of the other motors and send a filament feed signal.\nIf the problem manifests the same, then you can probably rule out the motor as the culprit. You could have a bad cable or a bad controller board. To rule out a bad cable you'd just swap out the cable from the board to the extruder.\nHowever, if the problem doesn't recur, then the motor might be bad. It might also be mounted poorly. You may be able to remove it from it's mounting and test if the problem persists.

\n" }, { "Id": "21009", "CreationDate": "2023-06-01T18:49:40.480", "Body": "

I am following the tutorial of Prusa for using Octoprint with my Prusa i3 MK3. I have a Raspbery Pi Zero and I'm using the built-in GPIO pins, not an USB connection.

\n\n

But in the Octoprint website, there's no serial port to select. It just shows "AUTO" and after a while it displays an error message that there's no Serial port found.

\n

Via SSH I can find quite some devices which look like Serial:

\n\n

I haven't had any issues with Serial connection since 2019 and despite all warnings, the Pi Zero did a great job. However, after the last Octoprint update, Octoprint didn't come up any more, so I did a reinstall from scratch.

\n

I'm using OctoPrint 1.9.0

\n", "Title": "Octoprint does not detect Serial port of RPi zero", "Tags": "|prusa-i3|octoprint|serial-connection|", "Answer": "

Neither the tutorial of Prusa nor the Octoprint setup instructions mention this. But it's common Raspberry Pi knowledge, for everyone how experiments with the RPi.

\n

The Serial interface that works over GPIO needs to be activated via

\n
sudo raspi-config\n
\n

In the menu, go to "Interface Options" (3), then "Serial Port" (I6). Answer "No" for the login shell and "Yes" for the Serial port hardware. After that, reboot your Raspberry when being asked.

\n

The Octoprint web interface should then detect /dev/ttyS0 and be able to connect.

\n" }, { "Id": "21012", "CreationDate": "2023-06-03T16:18:02.293", "Body": "

When calibrating my printer and configuring my slicer there are many different values that can affect the final distance between the print bed and the nozzle for the first layer. I'm trying to figure out where I should be making different adjustments. I'm having some first-layer issues, and I'm trying to get my layer height configured correctly to make changing materials and print settings easier.

\n

Intuitively it seems like I should be configuring the printer so that when Z = 0 the nozzle is just touching the bed and leaving the height of the first layer entirely up to the slicer. That way the slicer can accurately calculate the first layer height. But a lot of the instructions online imply that Z = 0 is calibrated slightly above the bed (the thickness of the paper). If I were setting zero on a milling machine I'd subtract the thickness of the feeler gauge to get a 'true zero' to the table.

\n

Some of the Marlin calibration features also seem to assume that Z is configured somewhat high. For instance when leveling the bed manually or when editing a UBL mesh it homes the printer and then moves to each point to verify the bed height, but it doesn't seem to account for the thickness of the feeler gauge. This makes me think that the feeler gauge height is expected to be built into the probe Z-offset. I'm using a 0.2\u00a0mm metal gauge as it's more repeatable than paper. If I have my Z-offset set to 'true zero' I can work around this by homing the bed to my feeler gauge and then calibrating.

\n

As far as I can tell the following values can affect the final Z height of the first layer:

\n\n

For reference, I have a Sovol SV01, with a BLTouch and Marlin 2.0.5.3. My pre-calibration process is as follows:

\n
    \n
  1. Power up the motors and measure the height of each side of the gantry from the printer frame.
  2. \n
  3. Turn one of the Z-axis lead screws until both sides sides of the gantry are the same height.
  4. \n
  5. Home the printer.
  6. \n
  7. Run the Marlin "level corners" wizard. I use a 0.2\u00a0mm steel feeler gauge, and go through the corners until they all have the same amount of friction (the centre of the bed is a bit loose now, so the center is slightly concave).
  8. \n
  9. Home the printer.
  10. \n
  11. Measure the distance to the bed using the feeler gauge.
  12. \n
  13. Adjust the 'Z-offset' value to get a distance of 0.2\u00a0mm. This will mean that the surface of the bed is at Z = 0.
  14. \n
  15. Go back to 5 until no adjustments are needed.
  16. \n
  17. Heat up the bed and the hotend (not necessary yet, but can't hurt).
  18. \n
  19. Use the Marlin assistant to generate a 'Cold Mesh' UBL mesh.
  20. \n
  21. Save the mesh.
  22. \n
\n", "Title": "Should I calibrate Z-offset so that Z = 0 has the nozzle touching the bed?", "Tags": "|slicing|z-axis|bltouch|calibration|", "Answer": "

No, you should not, unless you adapt your slicer settings.

\n

This answers explains the Z-offset. If your nozzle is at true Z=0 when touching the bed, then you need to find out all the property's in your slicer that adjust for the paper thickness. E.g. first layer is slightly overextruded and lines are usually wider, with your nozzle at the true zero, these kind of aids need to be changed else you will get too much flow in the first layer, pressure will build up and this will not help you to get a perfect first layer. This may result in rippling, balling up of filament, etcetera.

\n

Commonly used printing paper is about 0.1\u00a0mm, this is pretty negligible on most prints you will print.

\n

But, yes you can, could also answer your question, this may prove to give you additional work to get your prints printing.

\n" }, { "Id": "21017", "CreationDate": "2023-06-04T18:20:38.660", "Body": "

Being a newbie in 3D printing, I have some trouble with my prints. After removing wiggle room from the X axis and the bed, there are deformities even on simple prints. The first layer turns out okay-ish but there seems to be some stringing that makes the print deviate consisently when there is a turn. How could I improve?\nMy setup:

\n
Ender 3 V2 - stock printing bed, extruder/nozzles\nFirmware: mriscoc\nAuto bed leveling (BL touch) - Z offset -2.18mm\nSlicer: Cura - 0.2mm layers, 20% infill\nMaterial: PLA - 200C nozzle, 60C printing bed.\n
\n

The Gcode of the XYZ cube:

\n
;FLAVOR:Marlin\n;Layer height: 0.2\n;Generated with Cura_SteamEngine 5.3.0\nM140 S50\nM105\nM190 S50\nM104 S200\nM105\nM109 S200\nM82 ;absolute extrusion mode\n; Ender 3 Custom Start G-code\nG92 E0 ; Reset Extruder\nG28 ; Home all axes\nM420 S1 ; Use mesh level stored\nG1 Z2.0 F3000 ; Move Z Axis up little to prevent scratching of Heat Bed\n
\n

Sample output (XYZ cube):

\n

\"X

\n

\"Y

\n

\"Z

\n

\"Side

\n", "Title": "How do I fix deformities when printing on my Ender 3 V2?", "Tags": "|creality-ender-3|print-quality|pla|stringing|mriscoc|", "Answer": "

Update: I identified the issue as a lack of tension in the X and Y timing belts. After tightening the belts, the prints are much smoother !

\n" }, { "Id": "21019", "CreationDate": "2023-06-04T22:19:47.263", "Body": "

Is 3D printing progress (the one you check on the display of a 3D printer) linear or not?

\n

By linear I mean that equal differences in percentual progress will take the same clock time.

\n", "Title": "Is 3D printing progress linear?", "Tags": "|display|", "Answer": "

Short answer: Usually not.

\n

Longer answer: Depends on the printer.

\n

Many consumer grade 3D printers show progress by how many layers are printed in relation to how many layers in total. But some layers take a lot longer than other layers, so it's just an approximation, at best.

\n

Some higher tier 3D printers calculate differently, and as a result the progress indicator is more precise.\nFor example, the current models of Bambu Lab printers are pretty good at showing precise progress on the display. I think most resin printers are fairly accurate as well.

\n

I think that if we look back at this question in 5 years or so, the answer should probably be changed to: "Yes, usually", since printers get better and better each year.

\n" }, { "Id": "21032", "CreationDate": "2023-06-08T00:56:23.220", "Body": "

I have PrintrBot Simple Metal and over the course of today's use, I managed to get a piece of filament stuck in the hotend, just below the extruder gear. There's not really much to grab at, and my tweezers aren't that strong. How should I go about getting the filament out of the hotend? If it matters, I'm using 1.75mm MH Build Series PLA.

\n

Though I solved my problem, I would still like for members of the 3D community to share solutions that work for them.

\n", "Title": "How do I remove filament from the hotend of a direct-drive printer?", "Tags": "|hotend|filament-jam|", "Answer": "

Cold Pull

\n

The technic is nigh the same as for a bowden machine:

\n\n

If you want to get rid of residue, especially after printing a high-temperature material, you might want to purge the nozzle too.

\n

But the filament is short...

\n

If the filament is too short already, purging it is the solution:

\n\n

Purging

\n

I mentioned above purging. Purging is just pushing extra filament through. As it melts and gets pushed out with extra force, it takes with it all the residue and old filament. You pretty much purge the old material from the system. It might be easier for you to use a different color, as then you can see when you have purged the previous filament fully.

\n

If you still need to clean more or want to swap the nozzle, you can let the hotend cool down to about 20\u00a0\u00b0C below the printing temperature of your now-loaded filament and return to a cold pull.

\n

I generally have good enough success by performing a cold pull and then purging some centimeters more after the filament is of even color.

\n

Nozzle geometry warning

\n

Core-Heating nozzles are very complex and doing a cold pull in them is very hard to impossible. It is best to not cold-pull and instead just purge forward if possible.

\n" }, { "Id": "21037", "CreationDate": "2023-06-09T12:19:18.557", "Body": "

I designed some line art figure in Blender using several Bezier Curves, converted them all to Meshes, joined them to one Mesh and converted it back to a Curve.\nThen I extracted the curve and set a depth and at the end I saved it as a STL file.

\n

Now in Cura (5.2.2.2) using the standard settings I have the following struggle:

\n

When opening up the STL everything seems normal and as designed:

\n

\"Before

\n

But after slicing some lines/walls are ignored:

\n

\"After

\n

Judging by the preview visualization and time those inner structures disappear.

\n

Do you know how to correct by tweaking the Cura settings?

\n

I think the problem comes from the Blender Project due to the multiple converting and joining process but maybe I can save the current model instead of starting over.

\n", "Title": "Missing walls in Cura after slicing", "Tags": "|creality-ender-3|ultimaker-cura|blender|", "Answer": "

As suspected the error came from the Blender project. I tried using only Bezier curves and after joining and extruding the exported STL file behaves as expected.

\n" }, { "Id": "21043", "CreationDate": "2023-06-11T20:23:51.587", "Body": "

Recently, I've been noticing what looks like under extrusion. My first thought was a clogged nozzle, but it only occurs in some places:

\n

\"(Photo

\n

My printer is a PrintrBot Simple Metal, using MH Build Series 1.75\u00a0mm PLA and the model was sliced with Cura 5.1.0. What could be causing this, and how should I fix it?

\n

Slicer settings:

\n\n

I'm not printing faster than usual, and I watched some of this print print, and there were no obvious issues like a clicking extruder. I find it strange that it occurred regularly along the edge of one piece, but the piece next to it was perfect. It seems to occur around two sharp corners. The other piece didn't have those sharp corners.

\n

Another doubtful theory by me:

\n

I've had some issues with a loose plug recently. Could this be causing a drop in temperature?

\n", "Title": "Under-extrusion in Certain Locations", "Tags": "|print-quality|underextrusion|", "Answer": "

Solved! Finally...

\n

So after having this issue off and on, an extreme case appeared. It turns out that filament was resisting coming off the spool, and when the extruder and hotend (the Simple is a direct-drive) pulled away from the spool it created tension in the filament, effectively pulling it out of the hotend, though only just enough to cause this issue. In this specific case, it seems that the long edge before the side of the piece was pulling the filament.

\n

Shoutout to @metacollin and @DragonDon for their answers. I'm still calibrating my machine, so they'll useful.

\n" }, { "Id": "21070", "CreationDate": "2023-06-19T15:20:45.210", "Body": "

I am new to 3D printing, so this is probably a simple one.\nI am using Freecad to design and Bambu slicer for printing. The object (simple cable clip) basically consists of vertical walls with no relevant overhangs. However, for printing, I have stacked seven of them in a vertical tower separating all seven objects by a small gap.

\n

\"Bambu

\n

When I slice this with supports enabled, I get a preview that somewhat looks like Devil's Tower. I tried to draw the support manually and even blocked support at the walls which allows me to draw exactly what I want, but this doesn't change anything. The slicer still throws material at it like crazy.

\n

What I ideally want is a single vertical support/breaking layer separating the parts. I know, I could model this breaking layer in Freecad, but the curved nature of the upper side makes sketching and modeling this quite difficult.

\n

So my question is, is there an easy way to do this in a slicer?

\n", "Title": "Add minimal support or breaking layer in multi part", "Tags": "|slicing|support-structures|support-material|", "Answer": "

The most easy solution is to edit the G-code file. In the G-code file you can see when the next stacked item starts to be printed. If there is no gap between the objects, the objects fuse together. If you insert G-code to raise the Z height by 0.1 - 0.2 mm and redefine the new Z to the old value prior to move you have created a weak/break layer similar to a support roof with a gap to the printed part.

\n

So after the last layer of an object you insert:

\n
// E.g. last Z is 15 mm, layer height is 0.2 mm\nG1 Z15.15 // Raise Z height by 0.15 mm\nG92 Z15   // Define this the Z you left\n// Continue printing next object\n
\n" }, { "Id": "21075", "CreationDate": "2023-06-20T16:47:10.123", "Body": "

All the scanners I have seen, are scanning objects that stand on a platform. What about something like a plastic part which doesn't stand on it's own? for example this plastic piece I broke on my K\u00e4rcher pressure washer

\n

\"enter

\n

Do you just have to be skilled at modelling enough to create a model from scratch?

\n", "Title": "3D scanning parts that don't stand on their own", "Tags": "|3d-models|scanning|", "Answer": "

At work we print holders for parts that don't standup themselves. This part you broke is probably not suitable for being printed yourself, this is most probably Nylon (or even worse, POM), when you print this yourself you will get into some challenges, like warping. If this has been broken, I doubt whether you find a suitable replacement material. Personally, I would buy this part, it is too difficult to scan and print yourself. And, if printed outside using a print service, it will be more expensive then buying a spare.

\n" }, { "Id": "21084", "CreationDate": "2023-06-22T11:40:56.633", "Body": "

I\u2019m fairly new to 3D printing but have managed about 10 parts successfully so far. On this print, there were some issues printing the raft, but by the final layer, it looked ok, so I let it continue. I was surprised to see that when trying to print the circular pattern, the filament seems to come unstuck from the raft and pull across.

\n

Is this down to bed adhesion, or some post-processing setting? I\u2019m running PLA at 50\u00a0mm/s. The bed temp is 50\u00a0\u00b0C and the nozzle temp is 210\u00a0\u00b0C.

\n

\"Partial

\n", "Title": "Why are the circular parts of this print not sticking to the raft properly", "Tags": "|creality-ender-3|", "Answer": "

Likely the extruder gear is slipping on the filament. The tension arm may be weak or broken. It's also possible that there's a partial clog.

\n

Whenever you get curved paths not adhering in place but getting "pulled" straight across the curve, the forces pulling on the extruded material from the nozzle moving are exceeding those making it adhere to whatever it was supposed to adhere to below. This can especially happen on overhangs, where there's low contact with what's below, or in cases like over a raft or support where there's a gap and it's intended not to strongly adhere to anything below. However, if the right amount of material is being extruded, and if it's sufficiently melted before extrusion, there should be very little force trying to pull it out of place. This leads me to believe there's some extrusion problem going on.

\n

If you can't find anything wrong, you can try just going slower on the first layer after the raft, or using a hotter nozzle temperature. In the big scheme of things, 50 mm/s is a really low print speed, but for someone new to 3D printing, using a stock Ender 3 that likely has lots of little problems you're going to have to find and fix, it's at least an "above average" speed for a first layer, and I'd try just reducing that.

\n" }, { "Id": "21088", "CreationDate": "2023-06-22T22:02:36.757", "Body": "

I'll start off with some info in case it's needed, but my questions are a bit lower. I have an Ender 3 Pro with the 4.2.2 motherboard. Grabbed a Marlin firmware bin from Shiny Upgrades and seems to work great (built my own once and wasn't a fan). Got my Z-offset sorted out with a combination of stealing other people's numbers and testing a few prints out. Tried auto homing, but got weird numbers and instead resorted to that other approach. Printer seems work well now and my Z-offset is around -0.8\u00a0mm.

\n\n", "Title": "How does my printer auto level based off the sensor I installed?", "Tags": "|marlin|bed-leveling|z-probe|", "Answer": "
\n

Is the Z-offset I set in printer UI the distance between the sensor and the nozzle?

\n
\n

From this answer on question "Z Offset on autoleveling sensor setup" we read that:

\n
\n

the offset between the nozzle and the sensor trigger point (to the bed)

\n
\n

If you replace the build surface for a different surface the offset doesn't have to be changed.

\n
\n

Is my bed leveling kind of useless?

\n
\n

No, automatic bed leveling can be very useful if the build surface isn't as flat as a sheet of glass, it will map the surface and ensures the first layer is deposited at the correct distance.

\n
\n

I don't recall seeing any X or Y offsets in the printer UI

\n
\n

These settings are in the firmware, that is where these are defined. Note that these shouldn't be in the UI, as this is a hardware change. Also note that Marlin is pretty well programmed and is aware of this sensor offset, and even will use these offsets to have the print head (nozzle to be exact) stay within the specified print area. This is a good thing, e.g. when the sensor offset is large, and it would probe to the edge of the bed, the head could potentially wreck one side of the printer. This is described in question "How to set Z-probe boundary limits in firmware when using automatic bed leveling?".

\n" }, { "Id": "21093", "CreationDate": "2023-06-23T11:27:52.630", "Body": "

I have the issue that G29 does work as expected but then fails a couple of times at different measuring points after taht it succeeds again.

\n

So I decided to enable debugging for leveling by M111 S32:

\n
Recv: <<< do_blocking_move_to  X291.1250 Y96.0000 Z1.1500\nRecv:   current_position= X291.1250 Y96.0000 Z1.1499 : sync_plan_position\nRecv: <<< Probe::probe_down_to_z  X291.1250 Y96.0000 Z1.1499\nRecv:  Probe fail! - No trigger.\nRecv: <<< Probe::run_z_probe  X291.1250 Y96.0000 Z1.1499\nRecv:   current_position= X291.1250 Y96.0000 Z1.1499 : Probe::set_deployed\nRecv: deploy=0 no_return=0\nRecv: Raise Z to 5.0000\nRecv: do_z_clearance(5.0000 [1.1499 to 7.1500], 0)\nRecv: do_blocking_move_to_z(7.1500, 4.1667)\nRecv: >>> do_blocking_move_to  X291.1250 Y96.0000 Z1.1499\nRecv: >  X291.1250 Y96.0000 Z7.1500\n[...]\nRecv: <<< do_blocking_move_to  X291.1250 Y96.0000 Z7.1500\nRecv: >>> Probe::probe_specific_action  X291.1250 Y96.0000 Z7.1500\nRecv: BLTouch STOW requested\nRecv: BLTouch from 10 to 90\nRecv: bltouch.stow_proc() end\nRecv: <<< Probe::probe_specific_action  X291.1250 Y96.0000 Z7.1500\nRecv: >>> do_blocking_move_to  X291.1250 Y96.0000 Z7.1500\nRecv: >  X291.1250 Y96.0000 Z7.1500\nRecv: <<< do_blocking_move_to  X291.1250 Y96.0000 Z7.1500\nRecv: //action:notification Probing Failed\nRecv: Error:Probing Failed\nWARNING! Received an error from the printer's firmware, ignoring that as configured but you might want to investigate what happened here! Error: Probing Failed\n
\n

Does Probe fail! - No trigger. mean the probe has a defect so that it sometimes does not trigger?

\n", "Title": "G29 Sometimes fails - Defective probe?", "Tags": "|bltouch|", "Answer": "

I've changed the probe to a CR-10 model. The one with the metal pin. No Issues anymore. I didn't even change the wire. So either the probe or the pin was bad. \u200d\u2642\ufe0f

\n" }, { "Id": "21095", "CreationDate": "2023-06-23T23:08:06.603", "Body": "

I've had a magnetic sheet on my Ender 3 for some time. It came with a Creality 3D flexible bed. I've since got a couple of PEI coated steel plates and use it with that.

\n

I noticed it had a lot of crud embedded in it, so since each of the coated steel beds came with a magnetic pad, I have a choice of pads to apply.

\n

Since there seems little variation in the magnetism once the plate is fully over the pad. would I be better going for the lightest at around 120\u00a0g, or the heavier ones (at around 240\u00a0g) which do have a slightly stronger pull?

\n

I'm intending to use a couple of ADXL345's to tune the resonance after I've reinstalled the bed.

\n", "Title": "Is a lighter magnetic pad better?", "Tags": "|creality-ender-3|bed|build-surface|", "Answer": "

If you want/intend to print fast (high acceleration), adding mass to the moving bed is a big problem. It both decreases the resonant frequencies of the system (making it harder for input shaping to suppress them without introducing significant errors into the toolpath) and increases the force needed to accelerate the bed (making it easier to skip steps). So, all other things equal, you want to pick the lighter magnetic plate.

\n

With that said, if you really want performance, the lightest plate is no plate. Myself and at least several other people I know doing high performance printing on bedslingers have dropped (or skipped adopting) the magnetic plates and gone to clip-on plates or no plates at all. I'm running my Ender 3 with "blue tape"/"painter's tape" directly on the aluminum bed (no separate buildplate) and others are using various surface treatments (variants on the old "ABS slurry", hairspray, etc. concepts) to print directly on aluminum. This saves a lot of mass, but can be less convenient for popping prints off.

\n" }, { "Id": "21115", "CreationDate": "2023-07-01T21:14:28.613", "Body": "

I recently spilled waste water from my resin printing onto a fabric cooler. The water was in a container I was using to cure leftover resin from cleaning finished prints by leaving it in the sun for a while. So there was an unknown amount of uncured resin in it after I had left it out for a few hours.

\n

Is it likely that the cooler is permanently contaminated or should I be able to wash it and use it again? It isn't a really expensive one but also not really something I wanted to replace yet.

\n", "Title": "Water washable resin safety", "Tags": "|resin|safety|", "Answer": "

I ended up calling the cooler a loss. I don't know what that resin will do embedded in the fabric of the cooler and I don't really want to drink poison when I am out on the beach or develop some sort of sensitivity from it. Ended up getting a new cooler as a gift so it all evened out.

\n" }, { "Id": "21131", "CreationDate": "2023-07-04T04:16:24.387", "Body": "

1.Active chamber heating (fan forced convection +heating bed +PTC heater) vs 2.Passive chamber heating(natural convection+ heating bed)

\n

In the book "3D Printing Failures_ 2020 Edition_ How to Diagnose and Repair ALL Desktop 3D Printing Issues" writen by Aranda, Sean, it is said less cooling rate could let polymer have more temperature and time to release stress. This is the reason why chamber heating is used. But

\n
    \n
  1. In the design 1 chamber above bed is already >50\u2103 with 120\u2103 heating bed.
  2. \n
  3. In another design "120\u2103 bed with PTC supplyment which is cycled by a fan and the chamber can be up to >60\u2103". It forms forced convection, so the printed part may be cooled quickly even though the chamber has up to 60\u2103 temperature(the temperature difference between printed layer and ambient changes slightly).
  4. \n
\n

\"enter

\n

So why we use chamber heating with fan forced convection design other than natural convection deign?

\n\n", "Title": "Will active chamber heating with heating bed and PTC heater (forced convection by fan) be better than passive chamber heating with only bed?", "Tags": "|heated-bed|abs|heat-management|warping|chamber|", "Answer": "

Having forced convection allows to flow air on top of the heater so that more heat will be pushed into the chamber. Also, it makes the chamber uniform.

\n

If you are barely heating the chamber (50 \u00b0C) the bed can already do it, while forced will cool down the part more quickly. Forced convection has almost only downsides, no advantages.

\n

If you are heating the chamber to higher temperatures like 70-120 \u00b0C or more, the printed part has no problem about being cooled too much, and at the same time the bed set at 120 \u00b0C will not be able to heat up the chamber that much (or you need 2 cm thermal insulation around the chamber). Forced in this case has no downsides and only advantages.

\n" }, { "Id": "21164", "CreationDate": "2023-07-09T16:14:05.073", "Body": "

\"Red
\nRed wheel does not touch aluminium beam

\n

I was checking the POM wheels of my printer, since I have a minor banding issue with my prints.
\nI noticed that the wheel marked in red never touches the beam no matter how tight the eccentric nut on the right green wheel is tightened.

\n

Things I have already tried:

\n
    \n
  1. Replacing all three POM wheels
  2. \n
  3. Tightening eccentric nut on the right wheel
  4. \n
  5. Check if the z-axis beams are square
  6. \n
  7. Making sure the holes on the bracket and on the gantry are square/centered
  8. \n
\n

Any ideas on what the root cause the problem could be? I am thinking the easiest solution would be to add a second eccentric nut to the offending wheel?

\n", "Title": "One of my Ender 3 gantry POM wheels never touches z-axis aluminium beam", "Tags": "|creality-ender-3|troubleshooting|pom|", "Answer": "

Your right (undriven) side Z carriage is out of square, and is deflecting the gantry.

\n

Imagine the left side Z carriage wasn't attched to the gantry at all, just on its own. The only way one of the two wheels without the eccentric nut, on the outer side, could be loose, is if the wheel with the eccentric nut, on the inner side, were not tight.

\n

So start by disassembling the system and attaching the gantry only on the driven-side carriage, i.e. in a cantilever configuration. Do this without the X carriage attached so gravity isn't fighting it, and ensure it's square to the frame and that the eccentric nut is tightened enough there's no play. You should be able to rotate any one of the wheels with manual force while holding another still, but if you don't hold one still, rotating any wheel by hand should move the carriage and cause all three wheels to turn.

\n

It's possible that the bolts holding the gantry to the carriage don't have it aligned square; if it's off, you'll have to take it off the top to adjust. These bolts need to be really tight or they'll tend to self-loosen and throw everything out of wack again.

\n

Once you have the driven-side carriage bolted to the gantry tightly square, it's time to do the undriven side. Start by adjusting the eccentric nut to get the carriage snug, and only then bolt it to the gantry.

\n

Confirm that it moves all the way up and down the Z axis without binding. If the Z extrusions are not parallel or if either of them is bent, you'll have problems and you need to go back and fix that before connecting the undriven side to the gantry again.

\n" }, { "Id": "21177", "CreationDate": "2023-07-14T13:56:27.610", "Body": "

I would like to create a profile, in the Prusa Slicer software, that could be preset so that in the last layer the nozzle is at 210\u00a0\u00b0C of temperature so that the printer ensures that the inner and outer walls are well together and do not become loose in the layer higher. I have very thin 0.7\u00a0mm walls and I usually have problems with the walls opening every now and then...

\n", "Title": "In the last layer, the nozzle should be at 210 \u00b0C of temperature configured in the Prusa Slicer profile", "Tags": "|prusaslicer|", "Answer": "

In the printer settings custom G-code you could add a custom G-code there

\n
{if layer_num == (total_layer_count-1) } M104 S210 {endif}\n
\n

But that does not sound right why not start with 210\u00a0\u00b0C in the first place? It is perfectly OK to print PLA at 210\u00a0\u00b0C.

\n" }, { "Id": "21216", "CreationDate": "2023-07-25T05:05:03.123", "Body": "

I recently installed the CR-Touch onto my CR10-V3 printer. I updated my firmware to Marlin 2.1.2M and set my probe offsets and selected level bed which checked 81 points on the print bed and it said bed leveling was complete.

\n

I tried printing something that I had already created G-code for from Prusa Slicer, and on the initial wipe and perimeter print the nozzle was way too high (maybe 10-15\u00a0mm off the print bed surface)

\n

I searched for how to enable using the auto bed leveling mesh using Prusa slicer, and I found a couple of places mentioning adding M420 S1 to the G-code to enable using the auto bed leveling mesh, but even after adding that, my print did the same thing and during the wipe and perimeter print it was way too high.

\n

Below is the start G-code that I'm using in Prusa Slicer

\n
G90 ; use absolute coordinates\nM83 ; extruder relative mode\nM104 S150 ; set temporary nozzle temp to prevent oozing during homing\nM140 S{first_layer_bed_temperature[0]} ; set final bed temp\nG4 S30 ; allow partial nozzle warmup\nG28 ; home all axis\nM420 S1 ; Enable use auto bed leveling saved mesh\nG1 Z50 F240\nG1 X2.0 Y10 F3000\nM104 S{first_layer_temperature[0]} ; set final nozzle temp\nM190 S{first_layer_bed_temperature[0]} ; wait for bed temp to stabilize\nM109 S{first_layer_temperature[0]} ; wait for nozzle temp to stabilize\nG1 Z0.28 F240\nG92 E0\nG1 X2.0 Y140 E10 F1500 ; prime the nozzle\nG1 X2.3 Y140 F5000\nG92 E0\nG1 X2.3 Y10 E10 F1200 ; prime the nozzle\nG92 E0\n
\n

If I'm understanding the last part of that G-code, it should be 0.28\u00a0mm above the print bed when it does its nozzle wipe, but it's 10\u00a0mm or more off the bed when it does that.

\n

I did some additional testing this morning, and the problem seems to be the z-values after auto homing. I have a z-probe offset entered of -2.122\u00a0mm that I got using the z-probe offset wizard in the Marlin firmware, and that offset visually seems reasonable for where the probe triggers vs the nozzle height.

\n

After auto homing all axis, if I manually change the z-axis to be 0.28\u00a0mm, then the nozzle is still about 7\u00a0mm above the print bed which is where my prints are starting off.

\n

If I manually deploy the touch sensor and lower the print head, the touch sensor engages with the print surface at around a z position of -5\u00a0mm, and then with the z-probe offset of -2.112\u00a0mm it's about 7\u00a0mm too high.

\n

Where is this extra height coming from? I've checked and rechecked the z-probe offset, and like I said, the values seem reasonable, but I'm not sure where this extra height is coming from.

\n", "Title": "Absolute Z values not 0 at print surface after auto homing with CR-Touch", "Tags": "|marlin|bltouch|creality-cr-10|automatic-bed-leveling|", "Answer": "

I have no idea what happened the first time I initialized everything, but I started over from the beginning and ran the Initialize EEPROM command from the UI, and then saved the settings, and then went to the probe offsets and reset my probe offsets to exactly what they were before, but now after auto homing the absolute Z values look accurate and if I manually move the Z axis to 0 it is right at that 0.1\u00a0mm height of a sheet of paper.

\n

The first time I initialized the EEPROM I ran auto home immediately after and I was getting an error on the main page ERR: too far and maybe in the process of trying to fix that I messed something else up, but it appears that everything is working how it should now.

\n" }, { "Id": "21252", "CreationDate": "2023-07-29T18:06:12.837", "Body": "

When shipped from the factory to retail the Ender 5 comes packaged in a series of layers using shaped foam.

\n

In order to repackage an Ender 5 in its original box (for example, for transport or storage) it must be dismantled and placed back in this foam packaging in a specific order and arrangement.

\n

What does this order and arrangement look like, with pictures if possible.

\n

This is "how do I deconstruct" question, the opposite to "how do I build".

\n", "Title": "What does an Ender 5 look like in its original foam packaging?", "Tags": "|creality-ender-5|", "Answer": "

Watch this in reverse:

\n

\r\n \r\n

\n

The "unboxing" ends at 2:45

\n

Screenshots in reverse order:

\n

\"enter

\n

\"enter

\n

\"enter

\n

\"enter

\n

\"enter

\n

\"enter

\n

\"enter

\n

\"enter

\n

\"enter

\n

\"enter

\n

\"enter

\n

\"enter

\n

\"enter

\n

\"enter

\n

\"enter

\n

\"enter

\n

\"enter

\n

\"enter

\n

\"enter

\n

\"enter

\n

\"enter

\n

\"enter

\n" }, { "Id": "21263", "CreationDate": "2023-08-02T22:06:14.120", "Body": "

After replacing the nozzle on my Creatbot F430 printer, I auto-leveled the bed and initiated a test print - a simple square with a 2 mm thickness. However, I observed that the thickness of the printed model is inconsistent, varying between 1.5 mm and 1.7 mm. I'm wondering the Z-Offset is affecting this, or I shouldn't really worry about 0.3 mm / 0.5 mm difference.

\n

Nozzle Size is 0.4 mm

\n

\"Printer

\n", "Title": "The thickness of my model is not always accurate. How do I solve this?", "Tags": "|z-axis|nozzle|simplify3d|", "Answer": "

Yes the Z offset needs to be redone each time a nozzle is removed even if you re install the same size.

\n

Another thing to verify is to ensure the slicer is actually set with a 0.4mm nozzle

\n" }, { "Id": "21285", "CreationDate": "2023-08-05T13:44:47.947", "Body": "

Problem: The first layer on the bed was printed fine, but it seems the filament got stuck or something and wouldn't stick to the bed, but instead to the nozzle. Note, it also got behind the blue rubber part. This is the work of about half an hour of printing.

\n

Info:

\n\n

Question: How would I go on about removing the filament around the nozzle?

\n

Attempt: I tried to cut it away, but it has already dried out, making it quite hard. I did some research, but I could only find ways to unclog the nozzle itself (given that the nozzle can be reached, which is not the case). My fear is, if I continue to try to break it off, I'll break the whole part. I could heat up the nozzle hoping for the filament to melt again, but that'll probably make it worse.

\n

\"Photo

\n", "Title": "How do I remove clogged filament around a nozzle", "Tags": "|troubleshooting|pla|nozzle|", "Answer": "

You need heat to remove that. If you're hesitant to heat up the hotend, you can use a soldering iron or a butter knife heated over a lighter to start attacking the blob from the outside to get it down to a more managable size. A heat gun or even a hair dryer is also an option; the latter won't melt it, but may soften it enough to be pliable.

\n

Once you get the bulk of the blob off and are able to remove the silicone sock, a brass wire brush is the tool to use to clean off the remainder. To do this, the hotend needs to be hot, and you need to avoid brushing around where the wires go in so you don't accidentally break the wire insulation and cause a short. If there's plastic stuck there at the base of the wires, a toothpick can be used to remove it while the hotend is hot.

\n" }, { "Id": "21315", "CreationDate": "2023-08-13T15:25:58.830", "Body": "

I am using an Ender 3 Pro that I bought near the end of March 2023, and I have not modified any of its parts. About an hour ago, I tried preheating the hotend (using the 'Preheat PLA End' setting) and heard a buzzing sound coming from the parts cooling fan.
\nI tried cleaning the fan and made sure it was connected properly, which made the noise a bit quieter but didn't make it go away.

\n\n

I will keep trying to understand and fix the problem, and will add updates if I find more info.

\n", "Title": "Ender 3 Pro: Parts cooling fan making strange noises", "Tags": "|creality-ender-3|print-fan|", "Answer": "
\n\n
\n

The sound was caused by a buildup of filament near the motherboard and in the motherboard fan. After getting rid of the strands, the part cooling fan has stopped making the buzzing noise.

\n" }, { "Id": "21316", "CreationDate": "2023-08-14T13:23:58.597", "Body": "

I am having trouble with my print here, specifically with the part laying on the green support:

\n

\"Print

\n

It seems the edge running on the supports does not stick to it and it wants to curl upwards :

\n

\"Photo

\n

I have reduced layer height to 0.1\u00a0mm and support Z distance to 0.05\u00a0mm. I am using the tight support setting in prusa slicer. The perimeter is printed at 35\u00a0mm/s and the temperatures are 200\u00a0\u00b0C and 73\u00a0\u00b0C for the bed to print PLA+.

\n

Here is what I would like to get close to side by side with the current result. The good one is printed upside down and takes me 23h to print instead of 12h, so I would like to find a good fix for this ;)

\n

\"Photo

\n", "Title": "How to improve quality of edge over support", "Tags": "|pla|creality-cr-10|support-structures|prusaslicer|", "Answer": "

This looks like inadequate cooling. If this is PLA, it's probably a matter of PLA this close to a hot bed being unable to cool enough not to still be soft - the heat from the bed will continually heat the part, and it doesn't take much heat to keep it soft. If you're using a bed temperature of 60\u00b0C (common recoommendation for PLA) this is hopelessly too hot for any small parts with overhangs to print close to the bed. Try reducing it or printing with a completely cold bed using a non-heat-based method for adhesion. Some good ones are a textured surface like painter's tape/"blue tape", especially roughed up with light sanding, or an adhesive aid like glue stick or hair spray.

\n" }, { "Id": "21333", "CreationDate": "2023-08-18T18:08:48.160", "Body": "

A long time ago, I went through the experience of adding extra thermistors to my 3D printer board. Ultimately, I did get it to work, but it resulted in extra wiring and was not as clean as I would like.

\n

I was wondering if there are any ready-made 3D printer boards that support adding extra thermistors. I need 5 thermistors for a large 3D printer with a long heated bed. I need the thermistors to measure the temperature of an extruder, the chamber (in 1-2 spots), and 2-3 spots along the bed.

\n

Since I am looking for a new 3D printer board anyway, are there any boards that support adding extra thermistors using a premade PCB or that come with specific JST connectors for up to 5 thermistors?

\n", "Title": "Are there any 3D printer boards or expansions that support more than 3 thermistors?", "Tags": "|electronics|temperature|thermistor|part-identification|", "Answer": "

Yes. Generally, it will be the boards with lots of steppers (or rather stepstick sockets) which also support lots of heaters/thermistors. For example the BTT Octopus Pro has 4 hotend heater ports, heated bed terminals with dedicated bed power supply, and 5 thermistor ports TB/T0/T1/T2/T3. I suspect other 8-stepper boards are similar, though I haven't checked.

\n

If you'll be using Klipper, though, there's no need for lots of ports on the same board. You can use cheap RAMPS boards or old Creality boards or whatever and connect as many as you like to the Klipper host for it to control in sync.

\n" }, { "Id": "21350", "CreationDate": "2023-08-23T00:39:12.310", "Body": "

I've been having tremendously bad print quality lately and have been doing everything I can think of to fix it, none of it is helping. I tried printing a Temp Tower for the first time and since I was paying close attention to the temperature, I noticed that the printer is making some fairly wild temperature swings. My target temp is 200C and it's swinging between 192C and 207C back and forth, back and forth.

\n

\r\n \r\n

\n

That is a video of test printing a cube. Took about 30 mins, video sped up 8x so you can watch the temp changes. It takes about 4 seconds in the video to swing from 192 to 207 so that was about 32 seconds in real time.

\n

\"temperature

\n\n

While trying to address the print quality, here are the things I've done before noticing the temperature fluctuations:

\n\n

Here's the thermistor connection on the hotend:

\n

\"thermistor

\n

Any suggestions are welcome.

\n", "Title": "Bad print quality, Temperature Fluctuates about +/- 8 degrees", "Tags": "|print-quality|pla|temperature|thermistor|monoprice-maker-select-plus|", "Answer": "

This looks like a an extruder PID tunning issue.

\n

In Marlin ( I assume this printer use Marlin) there is a auto-tune command for the temperature PID (M303) you can check this page: https://www.3dmakerengineering.com/blogs/3d-printing/pid-tuning-marlin-firmware

\n

You mentioned that you changed the sensor you need to be careful and ensure it is exactly the same type as the stock or to know exactly what type of sensor it is and ensure it is adequately set in the marlin settings\na wrong sensor will give you an offset on the measured value but will not generate fluctuations by itself

\n" }, { "Id": "21358", "CreationDate": "2023-08-24T00:18:41.143", "Body": "

I produced a cube model (XYZ) using a printer to test its capabilities. Could someone offer guidance concerning the issues visible in the image?. \"enter

\n

Printer Used : Createbot F430\nSlicer Used: Simplify 3D\n\"Slicer

\n

\"enter

\n", "Title": "Help me diagnose these print problems?", "Tags": "|3d-models|simplify3d|", "Answer": "

These artifacts look like oozing during retract or unretract due to an excessively slow retract/unretract speed, but might also be related to misconfigured or lack of pressure advance/linear advance.

\n

Based on the settings info you added to the question, I think this is a very plausible explanation.

\n

The "extra restart distance" setting is probably the primary culprit. This is an extremely misguided feature and, with the configured value, is spewing out enough plastic to fill a whole 1.25 mm worth of linear movement (at typical layer height and line width). All this is ending up in one place as a blob. Turn that off (set it to zero) and the problem will almost surely go away. There is never a legitimate reason to extrude a fixed excess amount of plastic after each unretract, and doing so will cause catastrophic nozzle collisions dependent on your model geometry.

\n

Beyond that, 2.0 mm is a very long retract distance for a direct drive, which I believe this machine has, and 15 mm/s is very slow retraction. Those combined make it take at least 1/7 of a second dwelling in place during retract/unretract, which could also ooze material. 0.3-0.8 mm is the usual reasonable range of lengths for direct drive, and 30 mm/s (1800 mm/min) or so a reasonable speed. Play with these, but set it back up closer to where you had it if you get stringing or other problems.

\n

The coasting setting of 2 mm is really, really dubious too. Coasting is a hack to make up for printers with extremely outdated/bad firmware by underextruding. This doesn't seem to be the cause of your bulges, but it could be making them more noticeable by contrast between slightly over-extruded (from oozing) and severely underextruded lines juxtaposed. Coasting should always be off.

\n

Assuming your printer has it (I would hope so at that price, but sometimes pricy printers are running really old firmware), you should also make sure linear advance is enabled and calibrated. This solves the problem that coasting was trying to solve, but does it in a way that's actually accurate and still extrudes the correct total amount of material, just modulating pressure up and down as it does so that it flows in the right places.

\n

TL;DR: Set extra restart distance to 0, coasting to off, and maybe speed up and reduce the length for your retractions.

\n" }, { "Id": "21364", "CreationDate": "2023-08-24T17:59:56.193", "Body": "

I bought a Ender 5 S1, 3d printer (without limit switch version), I could be able to asemble it. Every axis works fine, however I cannot make the printing platform move upwards from the GUI.

\n

I can make Y axis and X axis move without any problem.But Z axis moves only down, not up.

\n

I tried to fix it by installing the latest firmware 1.0.7, but it didn't worked.What can I do to debug or fix?

\n", "Title": "Creality Ender 5 S1, I can move z axis downwards but not upwards, Why?", "Tags": "|z-axis|creality-ender-5|creality|", "Answer": "

I found the problem, My wide CRTOUCH power cable was loose. I am a begginer so I didn't reliazed that loose CRtouch Power cable was the problem

\n

When CRTouch is loose, you cannot move the Z axis probably because of security issues.

\n" }, { "Id": "21368", "CreationDate": "2023-08-25T13:22:53.297", "Body": "

I'm using an original Prusa MK3S and the PrusaSlicer as my slicing software.
\nI want to print this net shape with only one layer and a thickness of 0.3mm.

\n\n

Intuitively, I would expect the slicer to trace the diagonal lines one after the other, e.g., from left to right, and then follow up with the perpendicular ones.

\n

But when sliced with the PrusaSlicer, it traces the individual squares of the net in a seemingly random ordering.

\n

\n

I'm using all the default settings and a layer height of 0.3mm. Any ideas why the slicer does that?\nPrinting the squares instead of the lines is probably more prone to breaking when bent.

\n", "Title": "Why does the PrusaSlicer trace random squares instead of the straight lines?", "Tags": "|prusaslicer|", "Answer": "

Your model is not printable the way you "intuitively" think it is:

\n
\n

I would expect the slicer to trace the diagonal lines one after the other, e.g., from left to right, and then follow up with the perpendicular ones.

\n
\n

Once one direction of diagonal lines was finished, the other direction could only be drawn by "picking up the pen" over and over to draw each broken-up segment one at a time. Such a strategy has a lot of travel/retraction, and is thereby slow/inefficient and also higher risk for print errors. A pattern that snakes around in a continuous path, avoiding self-crossing as long as it can, is preferable.

\n

However, the slicer doesn't even have that much high level reasoning at its disposal to choose between "these two options". It's just solving for a decent best effort at an optimization problem for covering all the perimeters which need to be printed, which is something akin to the travelling salesman problem or similar graph theoretic problems which are NP complete and thereby do not admit any efficient search for an optimal solution.

\n

As an aside, you may notice that grid infill doesn't have this "problem". It prints the way you intuitively expected it to. This is because the slicer cheats and prints all of the crossing points twice. That often works out okay for infill, where it won't be visible, although lots of users, especially when printing at high speeds, run into problems with "self-intersecting infill patterns" where the sudden nozzle occlusion and resulting change in backpressure create broken extrusions and material sticking up past the layer height and colliding with subsequent layers. This kind of problem makes such a strategy a bad idea for the actual outer geometry of your print. But even if it weren't a bad idea, the slicer simply doesn't have a way of knowing "these features are ones the user wants me to print like infill". It's just solving the general problem of "print the perimeters for these polygonal regions", and in general, the solution doesn't look like an infill-specific hack.

\n" }, { "Id": "21372", "CreationDate": "2023-08-27T09:55:44.870", "Body": "

When I give the print command from Pronterface this how it's printing. The Z value is 10.00, can't solve the issue even after first layer calibration being carried out.

\n

\"print

\n", "Title": "Z-axis Calibration Fault in Prusa i3 MK3S", "Tags": "|print-quality|print-failure|", "Answer": "

Reading your question, it seems that the Prusa i3 mk3 (which I also have) is set up for z=0 to be the proper height.

\n

I do not use pronterface, so I don't know what special failures it may be causing. If the Prusa is calibrated for z=0 to be at bed level, and the printer is printing at z=10, and shows z=10 on the UI, then it is likely that it is being programmed for z=10. The question is, why?

\n

Because the printer is calibrated, and is reporting that Z=10 at the point of failure, I do not suspect a mechanical problem with the printer.

\n

I have sliced objects that were not properly touching the bed, which has caused this kind of offset. Perhaps you could check that the object appears to be flat on the bed before you slice. I had one case where a very thin spike stuck out of the mesh and lifted the object up (in the virtual slicer space) so that only the spike touched the bed, and since the spike was so small, it generated no extrusions.

\n

There may also be calibration functions of pronterface to allow for adjustment to printers that are not properly calibrated. Since I don't use pronterface, I don't know it, but I would suggest reviewing all the deep and hidden menues for something to tweek.

\n" }, { "Id": "21382", "CreationDate": "2023-08-31T06:42:58.887", "Body": "

So I'm having a bit of a strange issue where I'm getting gaps between perimeter walls, but only on part of the print. Even more strangely, it doesn't happen with every print, though it will happen consistently on the same model if I print it more than once.

\n

I know the go-to answer for an issue like this is "You're under-extruding. Fix your e-steps or increase your extrusion multiplier", however I'm not sure that's the only issue at play here. I did try increasing my extrusion multiplier (tried both 1.05 and 1.10), and while the issue did improve, it didn't fully solve it. The print shown below was printed with an extrusion multiplier of 1.00.

\n

Also worth noting is that there is a bulge in the side of the print toward the bottom, but I'm not sure if it's related to the perimeter wall gaps issue. I have no idea what would cause that.

\n
\n

Basic Info

\n

Printer: Ender 3 Pro
\nFilament: Hatchbox PLA ("gray blue")
\nSlicer Software: Slic3r

\n
\n

Slicer Settings

\n
\n

Note: I am only adding the settings I believe are relevant to the question. If you would like to see a setting not shown here, please let me know and I'll add it.

\n
\n\n

\"Print

\n

\"Print

\n

\"Printer

\n
\n

Print Images

\n

This first picture is to show the orientation of the print on the print bed:\n\"Print

\n

\"Print

\n", "Title": "Gaps between perimeter walls on only part of the print", "Tags": "|creality-ender-3|print-quality|", "Answer": "

You are using a larger than "standard 0.4\u00a0mm" nozzle. Large diameter nozzles cause flow volume to increase when the extruder etrusion speed is kept the same as for the standard nozzle which can cause extruding problems like underextruding.

\n

The nozzle diameter increase appears marginal (as in "it is just an extra 0.2\u00a0mm in diameter", but actaully the nozzle increases 50\u00a0% in diameter), but has significant effects on the volume flow of the extrusion.

\n

As seen from the table below, when increasing the nozzle diameter for the same printing speed, the volume flow increases from 1.26\u00a0mm\u00b3/s to 4.24\u00a0mm\u00b3/s which is a 337.5\u00a0% increase of the flow for the selected layer height (0.3\u00a0mm).

\n

\"enter

\n

This increased flow must be created by your extruder/nozzle assembly and might fall out of the range it can produce for the current setting. Generally this requires you to increase the printing temperature or decrease the printing speed.

\n" }, { "Id": "21387", "CreationDate": "2023-09-01T15:11:46.800", "Body": "

Speaking about any printer, in a general sense.
\nIf I load up the X carriage with two hotends and two stepper motors in a direct-drive-way... Then surely the force applied by the x stepper motor will not translate to the intended distance of the carriage.

\n

What I am getting to is: How do you ensure changing carriage weight does not change travel distance?

\n

Where would you change something? Do you start at software-level, like there being a variable and an algorithm which you feed the weight to, or do you change the stepper motor, and so on?

\n", "Title": "How much weight can the x motor support?", "Tags": "|x-axis|", "Answer": "

Load does not change the distance a stepper travels unless it misses steps. That's why it's called a stepper and not a normal open-loop motor - it moves in discrete steps that are completed in their entirety or not at all.

\n

If the motor can't move (actually: accelerate) the load you've given it, no amount of software-level stuff will change that. Giving it more current might. But generally, unless you've built an extremely rigid machine for extremely high usable acceleration/speed, these motors have more power than they can use without going way beyond the point where your print quality becomes unacceptable. For example the X motor on a stock Ender 3 can accelerate the stock carriage at nearly 100000 mm/s\u00b2 if you really want it to (with input shaping; without it, the resonances will give you drastically increased reaction forces that make it skip steps much sooner, maybe at 6000-10000 mm/s\u00b2).

\n

A more massive toolhead will be harder to accelerate, but the way you solve this is by using lower acceleration, or putting on a more powerful X motor. However, unless you're also doing serious vibration-reduction modifications to go with the increased mass, you won't even want to have more acceleration. The ringing will be bad.

\n" }, { "Id": "21398", "CreationDate": "2023-09-05T09:10:05.610", "Body": "

I just want to print this shower curtain ring:

\n

\"enter

\n

For some reason SuperSlicer doens't like the circles and after slicing the model looks like this in preview.\"enter

\n

What is the issue with the slicer? Is there any setting that I might have and ruin the model?

\n

Thanks

\n", "Title": "SuperSlicer model malfunction?", "Tags": "|3d-models|slicing|superslicer|", "Answer": "

At the moment I didn't find the exactly what option from config did this but after reimporting the preset everything is back to normal(possible something got corrupted in the preset or the app, I don't know for sure).\nI will still research to see what caused the issue for feature.\nThanks all

\n" }, { "Id": "21400", "CreationDate": "2023-09-05T21:32:31.437", "Body": "

I should mention that this is my experience with Marlin. I can adjust the Z offset all I want in the UI, but not the X/Y offset. If you reinstall the probe, I've heard you may need to adjust the Z offset (makes sense, you might not mount in the exact same way). I assume X/Y offsets get affected as well, so it doesn't make much sense to me that you'd have to rebuild the firmware to adjust those values. Maybe they're less sensitive to change? Anyways, not sure what makes them special enough to exclude from the UI.

\n

My printer seems to work okay using a prebuilt firmware, though it could just as easily be pointlessly leveling 50\u00a0mm off of the nozzle for all I know.

\n", "Title": "Why is the Z probe offset easily adjustable in the printer UI, but not the X/Y probe offset?", "Tags": "|z-probe|automatic-bed-leveling|", "Answer": "
    \n
  1. Usually you install the sensor once and keep it unchanged for years. Nozzle, though, is an element people, depending on printer usage, replace semi-frequently and the Z offset will change with different nozzles, different tightness, etc.

    \n
  2. \n
  3. XY probe offset has very little importance - your bed level changes very little over distances of order of 1\u00a0cm, so 1\u00a0cm of error in XY probe offset is negligible. Meanwhile, even 0.05\u00a0mm Z offset difference impacts first-layer adhesion significantly.

    \n
  4. \n
\n

On a separate note, you don't need to rebuild the firmware to change the probe offset. You just need an interface (USB cable) and terminal (Pronterface) that allows you to communicate with the printer using G-code. First issue M501 to load the configuration from EEPROM, then M851 with the right X, Y values to set the XY offset, then M500 to save the changes to the config to EEPROM. No firmware update is needed.

\n" }, { "Id": "21416", "CreationDate": "2023-09-11T04:42:58.493", "Body": "

I want to operate a 3D printer on a ship. This brings up some issues. I want to estimate how likely it is to get usable results under conditions on a moving vessel.

\n

First, it is impossible to level the platform and keep it balanced on which I want to install the printer. Any ballasting operation will tilt the plane even when berthed. This, of course, rules out several printing technologies, such as stereolithography, laser sinter printing, etc., which rely on gravity to keep the materials in a fixed position.\nIn my opinion, the only technology that is more or less tolerant of slight inclinations of the printer bed is FDM.

\n

However, a ship is also moving. Even when berthed, it will respond to waves and wind. I do not intend to use a printer in rough sea listing to 10\u00b0 or more. However, under rather calm conditions (2\u00a0m wave height), we reach +/- 2\u00b0 list and +/- 1\u00b0 pitch easily at turn rates of 50\u00a0mrad/s. Linear accelerations are +/- 0.1\u00a0m/s2 (X and Y) and +/- 0.4\u00a0m/s2 (Z).

\n

I'm aware that an FDM printer will be affected by acceleration and position. Position changes will alter the vector of gravity pulling on the printer's mechanics, and acceleration will superimpose another force.\nOf course, an error will be introduced. How big that error is will depend primarily on the stiffness of the mechanics and slackness of bearings and support.

\n

Has anybody had experience with a moving coordinate system (most likely on board any vehicle) while printing with an FDM printer?\nWhat problems do I expect when using a printer with actors in the most common cartesian coordinate system (X, Y, Z) on a ship?

\n", "Title": "What problems to expect when operating a 3D printer on a ship?", "Tags": "|bed-leveling|fdm|", "Answer": "

While your conceptions of "leveling" might apply to SLA or SLS, they are misconceptions with respect to FDM. "Leveling" the bed in FDM is not about leveling but about squaring it with respect to the rest of the motion system. There is no need for it to be "level" with respect to gravity. FDM printers operate leaning diagonally or hanging sideways off of walls (typical in some print farm configurations with auto ejection), upside down (e.g. the Positron), or in any other configuration you like. There are even printers setup to be carried as backpacks, printing while the wearer is in motion. Being on a ship, with extremely slow changes in orientation from the movement of water and the vessel, is completely irrelevant to FDM.

\n

If you want some numbers, the accelerations you cited, 100 mm/s\u00b2 in X/Y and 400 mm/s\u00b2 in Z, are at least an order of magnitude below what a decent FDM printer experiences constantly as part of printing. Nowadays, good printers print at 5000-15000 mm/s\u00b2 nominal acceleration, and experience additional forces from pseudo-instantaneous changes of velocity at the boundaries between linearizes segments comparable to another 2000+ mm/s\u00b2.

\n" }, { "Id": "21425", "CreationDate": "2023-09-12T12:00:40.737", "Body": "

I am completely new to 3D printing and want to start with SLA printing.\nI found the Anycubic Photon Mono 2 printer, but on the ordering page, there are options for "Airpure" and "Wash & Cure" machines.

\n

I do not know what these are, nor if they are needed for SLA printing.\nCan someone explain the function of these items, and if they are needed for SLA?

\n", "Title": "Equipment for starting with SLA printing", "Tags": "|sla|uv-printer|anycubic-photon|", "Answer": "

Airpure can be replaced with a good fan and duct that sucks fumes out from the printer and vents them outdoors away from any living creatures. Not practical in many situations, but can be much cheaper if your situation allows.

\n

I've seen a hobbyist making excellent prints where his "Wash & Cure" were a big jar with a lid, filled with isopropanol (to be shaken vigorously with the print inside) and a UV flashlight (to shine at the print from all directions).

\n

What you really shouldn't skimp on is a good quality rebreather and eye protection.

\n" }, { "Id": "21432", "CreationDate": "2023-09-13T23:54:04.470", "Body": "

I have a 3D object that I print with 0.6\u00a0mm nozzle and white PETG. At the moment I can't print the layers in another direction since I use 3D printed threads. The object is a de-seeder bit for the sorrel fruit. Some of the teeth break off when using it (see red arrow showing the missing tooth that breaks off when used below).

\n

If you want to see how it's used, I have provided an animation/video of it.

\n

\"Photo

\n

\"Upclose

\n

I was wondering if there was a way to increase the strength of the teeth on the bits from breaking. I was thinking of using some type of nail hardener but was wondering if that would work or if I should try and use something else.

\n

I'm also willing to try a different 3D material but it needs to be able to withstand summer heat in Florida in a garage which gets to be 97\u00a0\u00b0F or 36.11\u00a0\u00b0C and be printed with an E3D Revo 6 nozzle (which is like a hardened steel nozzle). PLA will just melt/turn rubbery.

\n", "Title": "How to harden / strengthen the teeth of a 3D print", "Tags": "|print-material|petg|print-strength|", "Answer": "

Alter geometry

\n

You experienced one of several factors of print strength:

\n\n

So, we need to make sure to strengthen that as much as possible. Let's start with the shape as it is:

\n

\"Teeth

\n

Now, how can we increase the strength? Layer-layer strength is linear to area, so can reduce the number of teeth. Next, thicken the walls - two is already more than twice as strong while 4 to 5 is hefty strong.

\n

\"reduced

\n" }, { "Id": "21437", "CreationDate": "2023-09-14T14:33:06.890", "Body": "

My Ender 3 v2 with a CR Touch/BLTouch installed is only moving down, but not up!\nThe display shows P Endstop when I try to move below around halfway up the gantry. What's going on, and how do I fix this?

\n", "Title": "Ender 3 v2 with CR Touch/BLTouch moving down but not up", "Tags": "|bltouch|ender3|", "Answer": "

Your CR Touch/BLTouch is not connected properly. Check the cable that runs from the mainboard to the probe on both ends. It may be slightly loose, so make sure to push both ends of the cable all the way in.

\n" }, { "Id": "21448", "CreationDate": "2023-09-17T05:55:30.393", "Body": "

I'm using the new Elegoo Neptune 4 Pro printer. I have the issue with both Cura and PrusaSlicer alike (I'm a Cura user).

\n

The issue is simple, instead of printing a perfect circle, the final print will have a diameter range of around 0.3\u00a0mm. For example, if I print a 15\u00a0mm diameter cylinder, the longest diameter will be 15\u00a0mm but the shortest will be 14.7\u00a0mm.

\n

I measured these with my caliper and I simply turned the cylinder around in the calipers; the cylinder will push up against the calipers as it reaches the thickest part of the cylinder and then come down as it goes to the thinner part.

\n

It's not visible to the eye; it looks like a very nice and perfectly round cylinder. But I can feel it with the calipers. The height is dead on 25\u00a0mm as designed.

\n

However, I don't get this result from the holes inside that same print. The holes are perfect circles and measure the same diameter roughly no matter what angle I measure them (at least I believe so, measuring the holes can be a bit tougher).

\n

Calipers are accurate and go to two decimal places for mm.

\n

I've tried:

\n\n
\n

I remeasured and found that the thickest part of every cylinder is not directly on the Y-axis, instead, it's diagonal. The bottom right and top left of the cylinder is the thickest portion while every other angle is roughly the same.

\n

I found the same was true for the cube I made. Measuring from the bottom right to the top left resulted in 26\u00a0mm, but the bottom left to top right resulted in 25.67\u00a0mm.

\n

Calibrated the the Y-axis (the only one that was off), issue persists. Again, this 0.3\u00a0mm deviation exists in the cubes printed as well, and always diagonally. Bottom Right to Top Left is always about 0.3\u00a0mm larger than Bottom Left to Top Right. Looked at an old calibration cube I made with my Ender 3V2 previously and the same measurements were only off by about 0.03\u00a0mm.

\n

This is very strange, I think I may simply return this printer for a replacement.. I have no idea where to begin investigating this issue.

\n
\n

I took some measurements. I found that the right and left sides of the printer (which are separate pieces of aluminum) were roughly 0.2-0.3mm in difference from their distance to the aluminum center piece.

\n

Now, without taking the entire printer apart, I can't say for certain that this measurement even matters, its possible that things aren't attached the way they seem from the outside.

\n

However, the measurement perfectly aligns with the error margin on the prints. It also correlates to the X/Y axis misalignment, as the measurement indicates the bed X-Axis is slightly tiled up on the right side by 0.2-0.3 mm. Had it indicated it was tilted up on the left side, the print errors wouldn't have made sense as the diameter would have increased/decreased on the wrong diagonals.

\n
\n

They agreed to a replacement after I sent pictures of the inconsistent diameter and measurements on the printer itself. I will update on if the issue is gone on the new printer once I have it.

\n", "Title": "Cylinders have an inconsistent diameter", "Tags": "|dimensional-accuracy|elegoo-neptune-4-pro|", "Answer": "

It's all but confirmed that the printer was defective. I got a new printer and loaded up the sample G-code it comes with, but the bed's belt snapped 10 minutes into the print (it was only finger-tight). This makes two Neptune 4 Pro printers in a row I've received with defects.

\n

I can only imagine that their quality control is terrible or I have terrible luck. They're also not replying to my emails for the next week due to a week-long holiday. This is an entire month wasted on defective printers.

\n

After some shenanigans, I got a new belt on it. The belt didn't snap as I thought, instead, the metal that was crimped on it failed to hold the belt and it slipped out. This was when I noticed that the Z-axis movement was very noisy. To my surprise, I find that the lead screws were entirely unlubricated.

\n

I printed new cylinders/cubes. The measurements are inconclusive, which probably means the machine is fine. On a 15\u00a0mm cylinder I found nearly a 0.2\u00a0mm variance, diagonally. This time, the bottom left to the top right (opposite of the last printer). A 30\u00a0mm cylinder showed a definite 0.4\u00a0mm variance on the same diagonal.

\n

I tried readjusting the X-axis, but nothing changed. I pulled out a cylinder that my old Ender 3v2 printed (25\u00a0mm diameter), the variance was <0.1\u00a0mm, a perfect circle (ignoring the Z-seam).

\n

This printer has not been worth the time spent on it, let alone the hassle. What a shame. The printer is fast and has beautiful quality prints otherwise. But it's worthless if they can't properly align the axes at the factory.

\n" }, { "Id": "21450", "CreationDate": "2023-09-17T22:26:05.243", "Body": "

In order to save some money on a project, I'm considering using 3\u00a0kg spools on my Ender 6.\nHowever, I never attempted this before, and could not find a firm stance on the web.

\n

I know that it is possible to build a filament hanger/holder separately, but is it necessary on the Ender 6?

\n

For reference, the spool hanger/holder on my Ender 6 extends 9\u00a0cm outwards of the printer.

\n

The filament loaded in the printer (see photo) is a 1\u00a0kg white PLA. The dimensions of the filament drawing are for the 3\u00a0kg filament to be used.

\n

\"Dimension\n\"Side\n\"Internal\n\"Close-up

\n", "Title": "Can I use 3 kg filament spools in an Ender 6 3D printer?", "Tags": "|filament|creality-ender-6|", "Answer": "

While editing this question and analyzing the input the community provided, I've concluded that it is not possible to use a 3\u00a0kg filament spool directly on the Ender 6.

\n

The actual answer depends on the filament spool dimensions you are going to work with.

\n

For one thing, the spool hanger has a length of only 9\u00a0cm, while the filament spool has a width of 10.2\u00a0cm. I don't think it is a good idea to leave the spool hanging loose.

\n

Secondly, the filament spool would hit the filament shortage sensor.

\n" }, { "Id": "21478", "CreationDate": "2023-09-28T13:05:07.923", "Body": "

I have been printing on and off for years. Generally, I have had no issues until recently. Upon thinking back I don't think I used the Z-offset at any point and only manually leveled my Ender 3 S1 Pro.

\n

I'm not sure if it's material or me, or both.

\n

I have to learn this. I bought a new, expensive printer which is ABL only. No knobs, only Z baby-stepping in software. The card reader in my S1 broke so I bought a new board and discovered I needed to do this process again. Since I purchased a Sonic Pad, I changed over to Klipper and printed TPU for 2 weeks fine.

\n

I bought a spool of Overture PLA, it's been a disaster. Before trying to dial this in last night the spool had been drying at 45\u00a0\u00b0C for 12 hours. Doesn't seem to print any differently.

\n

Instead of the config steps, after manually leveling and getting the mesh, I made a one-layer print of concentric squares with a solid 1" box in the center. I'm baby stepping trying to find the sweet spot and think I do. I begin printing my model, 5 copies, layer 1: 1 - doesn't stick; 2 - sticks; 3 - sticks; 4 - sticks. I stop and clean the bed with IPA. Copy 1 - sticks; 2 + 3 - doesn't stick. Stop printing, manually level, and find no resistance with paper. Re-level manually only, print, skirt doesn't lay down right.

\n

Tried the Creality "BuildTak" bed first, and worse.

\n\n

What's the process? 1) Manual 2) Probe 3) Mesh? Does the order of 2 and 3 matter?

\n

Mesh and manual are easy enough. I won't be at my printer until 7 pm Central be eager to read any advice during the day. I have Googled this also; obviously, I missed something.

\n

\"Photo

\n

\"Photo

\n", "Title": "First layer disaster: auto-leveling the bed. Need guidance", "Tags": "|creality-ender-3|adhesion|automatic-bed-leveling|", "Answer": "

I'm not claiming this is the definitive answer but I made a successful print with these changes.

\n

There were several things I did that may have helped but I don't think solved the problem. I eliminated the elephant's foot compensation. I brought up the flow ratio to 1.01. Even though I had indicated no cooling on the first layer, I unchecked the box that indicated fan Always on.

\n

The real solution I believe, increasing the nozzle temperature to 230\u00a0\u00b0C and bed temperature to 70\u00a0\u00b0C for the first layer and afterward dialing it back to 210\u00a0\u00b0C and 60\u00a0\u00b0C respectively.

\n

I've had rolls of PLA for a long time. Perhaps the formulation has changed over time but I didn't have to go through such steps before, nor print so hot on layer one.

\n" }, { "Id": "21494", "CreationDate": "2023-10-09T06:02:58.893", "Body": "

I think my heat bed thermistor is broken (infinite resistance measured) on my Prusa MK2, so I bought a new one, and I'd like to know which connector I shall use. Where can I find the references of this connector, please ?

\n

\"Picture\nRegards,

\n", "Title": "Prusa i3 MK2 thermistor connector reference", "Tags": "|prusa-i3|", "Answer": "

Those are JST SYP 2 pins female.

\n" }, { "Id": "21499", "CreationDate": "2023-10-10T17:58:08.657", "Body": "

I just received a Fusion3 EDGE printer and tried printing ABS with a 0.4\u00a0mm extruder at 260\u00a0\u00b0C and bed at 115\u00a0\u00b0C (the temperatures set by the demo print). The bed started heading on the first print, then stopped heating after a few layers and hasn't heated since after repeated tries. Does anyone know if the fault state is causing this or if a failed heater is causing the fault state?

\n

I've been going through the document Fusion3 EGDE-Troubleshooting: Heater Faults.

\n

Reset the heater fault and get the heater is heating too slowly.

\n

I'm waiting for tech support's response before moving this to an answer, but this is the result so far.

\n

I measured the voltages on the SRDD100 relay. When the bed heater was turned on 24\u00a0V was across the input and the output passed 24\u00a0V to the heater (relay on). When the bed heater changed to the fault condition, the input had 0\u00a0V and the output did not pass the 24\u00a0V to the heater (relay off). Turning the power off on the 3D printer, unplugging the bed heater, and measuring the resistance of the bed heater showed an open circuit.

\n

\"Photo

\n

\"Photo

\n

\"Photo

\n

\"Photo

\n

\"Photo

\n

\"Photo

\n

\"Photo

\n

\"Photo

\n", "Title": "Heater on bed stopped working on Fusion3 EDGE printer", "Tags": "|heated-bed|", "Answer": "

I measured the voltages on the SRDD100 relay. When the bed heater was turned on 24 V was across the input and the output passed 24 V to the heater (relay on). When the bed heater changed to the fault condition, the input had 0 V and the output did not pass the 24 V to the heater (relay off). Turning the power off on the 3D printer, unplugging the bed heater, and measuring the resistance of the bed heater showed an open circuit. The OEM also concluded that the bed heating element failed and is shipping a replacement.

\n

OEM comment:

\n
\n

every machine that is built runs through at least 12 hours of printing high temperature materials before it is allowed to ship. Having it basically show up not functioning is. . really weird.

\n
\n

My response:

\n
\n

This looks like it was on the tail of infant mortality for the heater exceeding the burnin time or a burnin escape. It only lasted a few minutes into a print with bed at 110C. Otherwise, it would be a defect caused in the process between burnin and arriving here.

\n
\n

The packaging showed no sign of mishandling and nothing on the unit showed signs of mishandling. There was no sign of damage to the heater. The lack of response to this question here supports that this is a rare incident.

\n

The OEM has also requested the return of the failing heater element for analysis (shipping container included with replacement part).

\n

Received new bed from OEM, installed bed, and printed successfully. First print after installing bed:

\n

\"enter

\n

Below is normally a difficult print to not have strings between the stringers, but the Fusion3 Edge printed with no strings on the first attempt.

\n

\"enter

\n
\n

When looking at SRDD100 application notes, I wondered why the 3D printer designer switched the positive supply voltage instead of the ground as in the application notes. But, thinking about this, the heater is attached to the grounded aluminum bed. If the designer switched the ground instead of the positive supply, a short to the bed could cause thermal runaway. Thus, the designer had safety in mind.

\n" }, { "Id": "21512", "CreationDate": "2023-10-14T06:17:01.367", "Body": "

I'm getting frequent layer shifts, always in the Y-axis. It's happening on more than one model, at least four to date, and on multiple attempts at the same models. Some of the models are mine (Tinkercad), some from Thingiverse. I can't think of any model that has not failed recently except some very small, simple ones.

\n

The printer is a RepRapGuru I3 clone, from a kit, purchased, built, and put in service in Summer 2018. I've been using it almost daily since. It's gone through at least 50*1 kg spools of filament in that time so has seen a lot of use.

\n

The printer has a single direct drive extruder, not Bowden. The frame is acrylic/plexiglass. The frame is solid, with no loose fasteners and no cracks that I can detect. There's no "soft foot".

\n

If a model fails, it fails consistently on every attempt to print it. The layer shifts occur often, but not always, at exactly the same point in the print job. In those cases where I've witnessed the shift, there seems to always be a very rapid "jitterbug" motion going on, as in inter-wall infill or while laying down a long narrow section of the model or while building overhang. It happens sometimes, but not exclusively, on the first continuous layer of a "ceiling" above infill and often at or near a corner of the model. I don't believe I've seen it happen on the bottom few layers where the model has a solid "floor" nor during "steady steaming" printing of a large surface.

\n

The magnitude of the shift varies but is typically roughly 5 mm, give or take. One, however, was about 30 mm. When it happens the sound suggests to me that the motor is slipping poles, not that the belt is slipping on the pulley, but that's only an impression. [NOTE: It's just occurred to me that I can put match marks on the belt and pulley. After a layer shift, if they're still aligned, I can pretty much rule out belt slippage. There's only a 1 in 20 chance they'd still be aligned after a shift. Will do that tonight.]

\n

The failures I've seen do not occur at the beginning of a layer, i.e. during perimeter printing; they occur somewhere in the middle details of a layer.

\n

The layer shifts began a few months ago. Prior to that, the machine had no chronic problems.

\n

I few months ago (note the coincidence with the above paragraph) I rebuilt the printer with all new guide rods, linear bearings, belts, and pulleys, in stages over several weeks.

\n

Before and after those updates, the Y carriage moved/moves smoothly and freely through its entire range with no tight spots.

\n

Later still, I replaced the original quarter-inch acrylic Y carriage plate with a 3 mm thick (220 X 220 mm) aluminum one, with four linear bearings (new ones again) in new aluminum pillow blocks vice the original three bearings in split plastic pillow blocks on the original. I replaced the original heated bed with a new one that has, unlike the original, a 3 mm aluminum backing plate. The very wimpy original heat bed springs were replaced with new, stiffer ones.

\n

The Y layer shifts started sometime shortly before or during the first series of updates. I'm certain the problem predates the heat bed and Y carriage replacement, but I've included that info above to communicate the current state of the machine.

\n

I'm unsure whether this issue began before, during, or after the guide rod, bearing, belt, and pulley replacement, which were done in phases over several weeks. I can't definitely tie the issue to any particular phase of those upgrades. The layer shifts definitely began before the heat bed and Y carriage plate replacements.

\n

Y belt tension is pretty normal, certainly not loose enough to cause it to jump teeth yet not overly tight. Tension is similar to that of the X belt, but that has a shorter span so maybe apples and oranges.

\n

Speeds slow/fast Settings:

\n\n

The only changes to these speeds have been to reduce some of them since the layer shift problems began. None were particularly high to begin with except Travel/High was 150 mm/s vice the current 80 mm/s. Reducing them had no effect on the problem.

\n

I typically run with retraction (2.5 mm; 40 mm/s) and Z-hop (0.5 mm) turned on.

\n

The Marlin firmware on the 2560 main board has never been updated or modified in any way. I don't know the version but it's whatever RepRapGuru shipped with their kits in mid-2018. The shield board is RAMPS; I'm almost sure I remember "Version 1.4" on the box it came in within the kit.

\n

I've never attempted to modify acceleration or jerk settings. They're as they've always been.

\n

I use Repetier V2.3.2 software. I use the embedded "Cura Engine", which I believe is really an older Cura Steam Engine slicer, not the current Cura slicer. I have not tried other slicers.

\n

The machine is controlled directly from a PC using the Repetier software and the embedded CuraEngine slicer; there is no SD card slot, digital display, or rotary encoder control. Gcode comes directly from the Repetier software on the PC to the printer via the USB connection.

\n

Things I've tried:

\n\n", "Title": "Need help identifying cause of frequent Y layer shifts", "Tags": "|layer-shifting|layer|", "Answer": "

Thanks to some help from an expert this issue is resolved. The culprit was crosstalk between the X motor cable and the sense line of the Y endstop. Re-routing cables to separate the two has solved the problem.\nMarlin monitors the endstop switches while printing unless inhibited by an option in the firmware; default is monitoring enabled. If a false low is detected during a move that move is stopped, resulting in a layer shift

\n" }, { "Id": "21515", "CreationDate": "2023-10-15T15:41:49.893", "Body": "

I'm printing a model, sliced in Cura, on a Monoprice Mini Delta v2.

\n

The model is long, about the width of the print bed. When I print it, the extruder drags the filament (which should stay in place) into the wrong position.

\n

To summarize, my white PLA won't stick to the print bed (which is heated to 40\u00a0\u00b0C). Is the bed temp too low? Is the nozzle too high (200\u00a0\u00b0C)?

\n

If someone could tell me how to get the PLA to stick- preferably a long-term option like print or bed temp - it would be greatly appreciated.

\n", "Title": "How can I get PLA to stick to my printer bed?", "Tags": "|pla|monoprice-mini-delta|", "Answer": "

I would go with 60 degrees on the bed and check the bed levelling. You shouldn't be having a problem with PLA if the levelling is fine unless your filament has gone bad.

\n" }, { "Id": "21528", "CreationDate": "2023-10-18T12:35:11.527", "Body": "

Printer: Creality CR-10 Smart
\nSlicer: Cura

\n

My axial fan, which cools the heatsink, broke down. I replaced it with a cheap knock-off fan, as it was the only option available. However, this replacement caused temperature fluctuations that went beyond the acceptable range.

\n

Here's what I tried to fix the issue:

\n
    \n
  1. I initially tried flipping the direction of the axial fan to make it blow hot air instead of surrounding air. This seemed to work for a while, but it caused heat to build up, and my 3D printer's filament stopped extruding from the nozzle.

    \n
  2. \n
  3. I attempted to lower the current supplied to the fan, thinking it might help. However, this turned out to be a bad idea, as I wasn't sure how much current the main board could safely supply through this port. The result was that the resistor I used got fried.

    \n
  4. \n
  5. I then decided to mount a 740\u00a0\u03a9 resistor in series to create a voltage drop, leaving about 10 volts for the fan. Unfortunately, this also led to heat build-up.

    \n
  6. \n
  7. Finally, I tried mounting a 320\u00a0\u03a9 resistor, leaving about 14.5 volts for the fan, and this seemed to work fine.

    \n
  8. \n
\n

I'm concerned about heat dissipation and the overall reliability of this solution. What are your thoughts on this?

\n

The issue stems from the small size of the heatsink and the large size of the axial fan, which directly faces the heat block. I'm now considering the option of mounting a 24\u00a0V, 3x3\u00a0cm fan and creating a custom mount for it.

\n", "Title": "Changing axial fan on CR-10 smart caused temp fluctuation/heat creep", "Tags": "|hotend|creality-cr-10|creality|fans|heat-creep|", "Answer": "

After encountering issues with an unknown brand fan overpowering the Creality fan, I experimented with resistors. Initially, a 330-ohm resistor resolved the thermal runaway warning but led to under-extrusion due to inadequate cooling.

\n

\"enter

\n

A 100-ohm resistor addressed under-extrusion but triggered thermal runaway.\n\"enter

\n

The optimal solution was a 220-ohm resistor, resolving both issues.\n\"enter

\n

I've tested this setup for approximately 1 day, 11 hours, and 7 minutes with successful results.

\n" }, { "Id": "21534", "CreationDate": "2023-10-21T15:01:13.687", "Body": "

On my Creality Ender 3 v2, I have performed bed levelling and am fairly confident that the nozzle height is correct - a piece of paper under the nozzle can be pulled out with a gentle tug. For some reason, the first layer that prints always has the nozzle too far off the bed and so it's not adhering correctly.

\n

I'm not sure if I've missed something with bed levelling, or if there is some other setting I need to tweak.

\n

FYI I'm printing with a 0.8\u00a0mm diameter nozzle with 0.5\u00a0mm layer height and 0.45\u00a0mm initial layer height. When I look at the printer in action, the first pass looks like the nozzle is about 1\u00a0mm off the bed.

\n", "Title": "Creality Ender 3 v2 1st layer too high off bed", "Tags": "|creality-ender-3|bed-leveling|", "Answer": "

I think there were two things wrong here:

\n
    \n
  1. I was not levelling the bed correctly - I was manually moving the z-axis down to the lower limit - which can be slightly different to the printer lowering until the limit switch activates (Menu > Prepare > Auto home).

    \n
  2. \n
  3. Looking at the suggested answer Nozzle not level and first print failed I did check the position of my z-axis limit switch and it was pushed all the way downwards. I decided to try filing a bit off the notch which rests on the base rails so that the z-limit would be hit a bit lower.

    \n
  4. \n
\n" }, { "Id": "21552", "CreationDate": "2023-10-29T07:12:13.210", "Body": "

I have planned a simple object in TinkerCAD from a box with radius set to 1\u00a0mm.

\n

However I would need to get a 3\u00a0mm rounding on the vertical axis, therefore I have added a quarter circular eraser.

\n

Please see both of these in this shot:

\n

\"enter

\n

Now it's time to round the red one:

\n

\"enter

\n

Looks good from top-angle:

\n

\"enter

\n

But it's not rounded properly from other angles:

\n

\"enter

\n

I wish to maintain the 1\u00a0mm rounding on the whole edge, how can I do that?

\n

I tried many things:

\n

1.) Inverse ring eraser: almost good, but I would need 2\u00a0mm width, and 3\u00a0mm outer radius which is impossible (for a ring).

\n

\"enter

\n

2.) Inverse half sphere: not good, as it's not the shape I need

\n

Finally

\n

I succeeded with a "hack": ring with 4\u00a0mm width and 6\u00a0mm radius, then scaled back to 50\u00a0%, making it 3 radius and 2\u00a0mm width. Then create variuos inverse-quarters from it (eraser) and tune them even further removing unnecessary halves:

\n

\"enter

\n

Using these erasers I could finally reach my goal:

\n

\"enter

\n

However this is super-tedious, is there a better software out (I'm sure it is), which can do this "rounding" effect on any object? Now I'm satisfied, but if I need to

\n\n

I'm screwed up: will have to "slice" this object to 3 pieces, increase the size of the middle, and then put them back together. Do this for every single axis. (= scale operation I'll need to slice this object to 9 pieces).

\n", "Title": "How to maintain 1 mm rounding when I need 3 mm in a single axis?", "Tags": "|tinkercad|", "Answer": "

TinkerCAD is a fine tool for simple designs, but when you need to fillet or chamfer corners, this involves too many actions where you cut out the parts to get the requested fillet/chamfer. It might be interesting to save time to move to a different design tool.

\n

This geometry is recreated in a few minutes in e.g. Fusion360, considering this is a simple design, you need to change the order of the actions to fillet the object properly. Fillet the corners of the box prior to filetting the box itself:

\n

\"enter

\n

\"enter

\n

\"enter

\n

In case the design is way more complex an alternative solution would be to join a cube in the corner and round that:

\n

\"enter

\n

\"enter

\n

\"enter

\n

\"enter

\n" }, { "Id": "21558", "CreationDate": "2023-10-30T16:57:58.700", "Body": "

I'm using (as pure beginner, please be respectful to my little knowledge in this topic) FreeCAD v0.21.1 for a simple design I wish to 3D print.

\n

This is how it looks like from the front (at least partially):

\n

\"enter

\n

Text is made with ShapeText and I applied a small Pocket onto it in Part Design:

\n

\"enter

\n

I have many operations after this text pocket in the tree.

\n

I'm satisfied with my results.

\n

I however wished to make another one, with different text, so I followed a tutorial to copy my whole "Body_of_ND32" to a new "Body_of_ND16":

\n

\"enter

\n

After this operation, I have got an error complaining "Recompute failed", and text pocket is containing the original "ND32" pocket, instead my new value.

\n

Of course, I could rebuild the whole thing from scratch adding the text only at the end, to keep everything in shape, but can I somehow detect what's wrong with the recomputation?

\n", "Title": "Compute failed? Why?", "Tags": "|freecad|", "Answer": "

A "Recompute failed" is not your fault. Freecad is a free CAD software that is maintained by more than 400 volunteers. It is a bug in the software and happens often, for example because of floating point rounding errors. Just moving or changing a shape or body a tiny amount might fix the problem. It could happen when intersecions happen exactly on a line, outlines make a shape self-intersect or beveled objects produce unexpected results or for other unexpected reasons. If you see a pattern in the cause of the error, you might file a bug report

\n" }, { "Id": "21564", "CreationDate": "2023-11-01T08:45:01.660", "Body": "

I want to end the bed heating approximately X minutes before the print is finished so that the bed starts cooling and releasing the print.

\n

Since an object can have different sizes and printing one layer can take very long or very quickly, I cannot simply insert an M140 S0 before the last layer. What is the best time interval and method to achieve this? Is there a Cura slicer extension for it, an OctoPrint plugin, or should I write a custom script for Cura? I mostly print with a 50\u00a0\u00b0C heated bed and PLA using OctoPi on a Creality Ender 3 v2 with Marlin firmware. With a printer bed at room temperature, the print could come loose too early while still printing, so 20\u00a0\u00b0C might be too extreme. Make sure the printer is finished when the bed is around 35-40\u00a0\u00b0C? Is there any theory and method to back this up? Does this method have a term or name?

\n", "Title": "What is the best time interval and method to stop heating the bed to start releasing before the print is finished?", "Tags": "|marlin|ultimaker-cura|heated-bed|octoprint|cooling|", "Answer": "

In addition to Oscar's answer, there is an OctoPrint plugin that attempts to achieve exactly that. The plugin uses the following and-condition to determine the moment to stop bed heating:

\n

"printing finished for at least 90% and remaining print time is below 5 minutes".

\n

The plugin is called OctoPrint-BedCooldown. Its page describes:

\n
\n

Turns off the bed heater toward the end of a print

\n

For filaments such as PLA, many printers have more than enough stored thermal mass in the bed to keep bed adhesion throughout the print. Therefore, you may want to turn off the bed heater automatically before the end of a print, saving cooldown time.

\n

The bed heater will be turned off during a print, when both conditions are met:

\n\n

Be sure to monitor your print, as turning off the bed heater could cause the print to come loose prior to completion.

\n
\n" }, { "Id": "21565", "CreationDate": "2023-11-01T09:28:38.737", "Body": "

I've noticed several times that one of my stepper motors is making a slight noise after a print is finished, indicating it's still enabled; also, the axes are still statically positioned and unmovable.

\n

I meant that the stepper motors are still powered. Usually, after a print, the extruder moves up and away, and the bed moves to the front. However, some X seconds after that, I would like to automatically turn off the steppers because it could potentially preserve the lifetime of my stepper motors.

\n\n", "Title": "What are the pros and cons of automatically turning off stepper motors after a print? Does it increase the stepper motor's lifetime?", "Tags": "|creality-ender-3|marlin|ultimaker-cura|stepper|octoprint|", "Answer": "

While it's plausible that it might increase lifetime, I doubt it. Unless you're driving them out of spec and allowing them to overheat, motor coils should last pretty much indefinitely. The main wear component in a stepper motor is the bearings, not the coils, and the bearings do not get any additional wear from having the coils energized, only from actually turning.

\n

The advantage you do get from turning off the steppers is energy savings. As long as the stepper is energized, you're burning roughly $I^2R$ watts, where $I$ is the current you're driving the motor at and $R$ is the coil resistance. You also eliminate the stepper hum, which can be really infuriating to some people's ears, and if your board has properly firmware-switchable fans, you let the fans power down too, which also saves energy and cuts noise.

\n

Another aspect that can be an advantage or a disadvantage is that, once steppers are powered off, you can move the motion components by hand. This can be nice for adjusting the toolhead and bed to remove the completed print, but it can also cause the toolhead or bed to fall due to gravity, which can lead to damage to the printed part or the printer. If this is an issue, on most kinematic systems (pretty much anything but delta or corexz) you can leave the Z stepper(s) energized between prints, and only disable the X/Y ones. The reduction gearing/leadscrew lead on most printers makes it so Z doesn't need anywhere near the current of X/Y, so turning off X/Y and just leaving Z engaged still gets you most of the power conservation.

\n" }, { "Id": "21567", "CreationDate": "2023-11-01T10:24:46.457", "Body": "

I recently started using this Slicer Thumbnails plugin in OctoPrint and added the steps to my Cura 5.5 slicer. However, my thumbnails are in one color, like this:

\n

\"Screenshot

\n

And are not showing any depth as shown for example on the plugin page:

\n

\"Screenshot

\n

Am I missing something here or is there a better way to create and add beautiful thumbnails?

\n

For reference, I used the model Female Scart Dust Cap for Male Scart Connector.

\n", "Title": "Why are my Cura generated thumbnails showing in one flat color without any depth?", "Tags": "|ultimaker-cura|g-code|octoprint|", "Answer": "

After a lot of trial and error I figured it out. I thought it was the result of having very small models but it turns out that simply changing the Filament Color in Cura Filament Profiles also translates to the Thumbnail color. Using a filament profile with the color black apparently doesn't show depth.

\n

Below is the thumbnail using the original Cura Thumbnail Script and another Filament Color which resulted in the following transparent PNG Thumbnail:

\n

\"SCART

\n

Instead of the original script I also tried using the Cura script Cura JPEG Thumbnail creator as described at How to generate a gcode preview which resulted in the following JPG Thumbnail:

\n

\"SCART

\n" }, { "Id": "21570", "CreationDate": "2023-11-02T07:32:08.690", "Body": "

Plastic straws are forbidden in the EU so it will get harder to buy them. I wonder if it is possible to 3D print plastic straws? I would likely use them as tools when woodworking, but if you can use them to drink beverages that would be a bonus. I am not familiar with 3D printing so I don't know if it is possible to get them bendable and so on like usual plastic straws.

\n", "Title": "Is it possible to 3D print plastic straws?", "Tags": "|plastic|", "Answer": "

It indeed is possible to print a plastic straw via 3D printing, however it requires a considerable amount of precision and supports during 3D printing, modelling and slicing.

\n

This is because as the 3D printers build the plastic straw layer-after-layer, the circular structure could result in instabilities and therefore requires some time to condense.

\n

Overall, these structures are possible to be 3D printed but not as easy as it seems, and way more expensive to print as well.

\n" }, { "Id": "21583", "CreationDate": "2023-11-05T12:06:54.643", "Body": "

I have little experience with CAD software, only use Thinkercad and only recently started using the Revoscan app. I know how to rotate files when slicing (in Cura) but that only has an effect on the generated gcode file, not the STL/OBJ file. But as I'm trying to publish a few things I scanned using Kiri Engine (photogrammetry app) where the original rotation is wrong.

\n

I want to know if I have an object in OBJ/STL format how do I rotate the position in those files so I can publish my OBJ/STL files in the proper rotation. I'm fine to use another app or tool, I just don't know how to do it yet.

\n", "Title": "How to change the orientation of an object inside the STL or OBJ file?", "Tags": "|3d-design|stl|cad|obj|rotation|", "Answer": "

Many 3D editing programs exist that will accomplish your objective. Tinkercad will do this as well, using the rotate feature, followed by moving it to the build plane and exporting it as an STL as desired.

\n

Fusion 360 is available for free personal use, but has a bit of a learning curve.

\n

Meshmixer from Autodesk will also rotate in the x/y/z axes and also place the model on the build plane.

\n

You've presented your objective well enough. I suggest to pick a single program and ask specifically how to accomplish that which you are unable to achieve.

\n" }, { "Id": "21584", "CreationDate": "2023-11-05T12:12:20.563", "Body": "

When scanning objects like sculptures using Kiri Engine (photogrammetry) often the bottom is open. What I currently do after exporting to OBJ, is opening it in my RevoScan app and use the "fill holes" feature to select it and fill it. However that's usually not entirely flat and suitable for printing and results in artifacts. What I then do it simply lower the object when slicing to sub-zero/a negative value to have a small part of the object be under the bed, basically cutting it off and leaving a perfectly flat surface in my gcode.

\n

However, I want to really be able to do this in my STL/OBJ file already for publishing to Thingiverse. How do I alter the STL/OBJ file to basically achieve the same, but in my object rather than in the final gcode?

\n", "Title": "How to I give my mesh object (STL/OBJ) a flat surface?", "Tags": "|3d-models|3d-design|stl|cad|obj|", "Answer": "

Associated with your other question, Meshmixer (and Fusion 360) will perform plane cuts. I feel that Meshmixer will accomplish easier the plane cut you seek, as one simply imports the model, selects the model by clicking on it, then from the tool bar on the left, selects Edit/Plane Cut. Manipulating the plane which appears with respect to translation and rotation is done with gizmos within the main window. Note the dialog box which should be set to re-mesh in order to properly close the cut.

\n

One of the gizmos will allow you to select which side of the plane remains after the cut. If you make an error (in most programs), Control-Z will allow you to reverse the last few actions to try again.

\n

Tinkercad can be used by creating a cube larger than the object, turning it into a "hole" and placing it to cut in the desired manner. Tinkercad will also glitch with too many triangles/facets, which leaves Meshmixer a better option in my opinion.

\n

Please note that for both questions, you'll find helpful videos with tutorial approaches, to make your objectives that much easier. Too many to link in a practical manner.

\n" }, { "Id": "21589", "CreationDate": "2023-11-05T17:33:55.923", "Body": "

What is the average or recommended lifespan for standard Creality printer nozzles used with non-abrasive standard PLA only? What is a proper metric for that, time used, time heated, filament meters that passed through?

\n

Does the nozzle diameter affect the lifespan?

\n

I know nozzles are cheap, but I'm asking because I'm thinking of writing a maintenance tracking plugin for printer parts in OctoPrint.

\n", "Title": "What is the lifespan of nozzles, and does the nozzle diameter affect the lifespan?", "Tags": "|creality-ender-3|nozzle|maintenance|", "Answer": "

Print material matters more

\n

Non-abrasive materials can leave a soft brass nozzle live for dozens of filament spools without noticeable degradation, especially if the nozzle tip is hot enough to keep it above melting point and does not scrape over cooler areas. However, abrasive filaments can eat a nozzle within just a couple gross meters. One of the worst offenders is Carbon-Fiber or Glass-Fiber infused material, requiring, for example, ruby-tipped nozzles, or glass-nozzles to withstand more than about 300 meters (~2 Gross).

\n

Note that any filled filament is abrasive to some degree, and nobody advertises with the fact that the filament is abrasive.

\n

Nozzle formfactor impact

\n

A larger nozzle allows the extrusion of more plastic per time. The plastic itself, as it cools, can act as an abrasive itself. So to a degree, nozzle geometry will have an impact on the print. A flat around the nozzle orifice such as with the venerable e3D v6 nozzle design can help in keeping the orifice intact a little longer (as the nozzle has more thermal mass and can press down the filament further away from the orifice), but a thinner tip has other benefits (such as reacting faster to temperature change) at the cost of longevity in this one regard.

\n

Nozzle diameter impact?

\n

Typically, nozzles wear much faster and lose their tip from grinding on the printed material than that the material getting pushed through them starts to bore out the nozzle. This can be seen very well in the experiments Stefan/CNC Kitchen conducted. Also here.

\n" }, { "Id": "21597", "CreationDate": "2023-11-06T10:57:40.443", "Body": "

I recently started using the new mriscoc firmware for my Creality Ender 3 V2 and it is truly performing above expectations. Even considering that Jyers UI was already a step up from the original Creality firmware.

\n

I'm trying to understand how the new feature, that seems specific to this firmware, called Adaptive Mesh Leveling (AML) actually works in the firmware.

\n

Below is a screenshot of how the Cura Plugin shows AML options:

\n

\"Cura

\n

I have successfully tested AML and I think it's a truly transformative and great way to do fast and more precise bed leveling, only to the relevant print areas on the bed. Also, while probing, the screen shows each probing point in real time being filled in a grid.

\n

However, I noticed that when using the OctoPrint Bed Visualizer instructions to obtain just the mesh data using the following g-code in the OctoPrint Plugin Settings:

\n
M155 S30  ; set temperature reporting delay, use a value longer than the time it takes for your leveling command to complete.\n@BEDLEVELVISUALIZER  ; instruct plugin to start recording responses from printer.\nM420 V    ; Get bed leveling mesh data.\nM155 S3   ; set the temperature reporting delay back to a shorter time span.\n
\n

My previously generated 5x5 of 9x9 full bed grid is no longer showing, instead the mini grid that was made on the print area is showing, as a full bed. Which makes me wonder, how does the firmware deal with AML? Is it basically tricked into using a subarea of the printer as a virtual 'new bed', the new maximum bed size being only a portion of the full bed?

\n

Additionally would there be a way for OctoPrints Bed Visualizer plugin to even know if it deals with the full bed grid or the AML 'mini grid' and if so would it be exactly on which part of the full-bed the mini-grid was generated?

\n", "Title": "How does Adaptive Mesh Leveling (AML) actually work?", "Tags": "|marlin|octoprint|automatic-bed-leveling|aml|mriscoc|", "Answer": "

Bed Visualizer shows the mesh leveling area regardless of the bed size, that is good because it provides a good level of detail if the mesh area is very small compared to the bed size.

\n

But it could be a useful addition if displaying the mesh area over set to the bed size were possible through a configurable option.

\n" }, { "Id": "21598", "CreationDate": "2023-11-06T11:07:55.847", "Body": "

With the following thought in mind: 'Two eyes are better than one' does additional Thermal Runaway detection in OctoPrint add any benefit?

\n

This OctoPrint-ThermalRunaway Plugin seems to add a basic ThermalRunaway detection in OctoPrint. I understand all the smart disclaimers he makes. Security-wise, of course, we should prefer mechanical over firmware, and firmware over external software (OctoPrint). And I think printers that don't have at least any kind of firmware runaway protection should be trashed instantly.

\n

However, assuming you already have a printer running firmware runaway protection would this OctoPlugin add any value, 'in case firmware runaway protection' somehow fails?

\n

Related: Firmware runaway detection as well as the mentioned 'solution' in OctoPrint are based on sending an emergency gcode to the board and performing an emergency shutdown.

\n
\n

Wouldn't it be better, more secure if the power was cut from the printer entirely? Eg. instead of an OctoPrint plugin sending an emergency g-code, it sending an power disable command to a smart plug (API)?
Source: Octoprint-ThermalRunaway issue

\n
\n", "Title": "Does additional thermal runaway in OctoPrint add anything and shouldn't it use a smart plug to kill power instead?", "Tags": "|firmware|octoprint|thermal-runaway|", "Answer": "

It can save prints in specific situations. If you set maximum temperature lower than the one configured in the firmware, and configure the command set to cause printer to stop heating instead of stopping altogether, it is possible that plugin will stop printer from overheating before printer will panic and stop mid-print.

\n

This, of course, does not replace other protections, but I recently lost a couple of hours of printing due to stupid error on my part, and this plugin might have prevented it.

\n

It shouldn't be used instead of firmware protection. But I can see how it can be use to supplement it.

\n" }, { "Id": "21600", "CreationDate": "2023-11-06T11:27:44.370", "Body": "

Given that, a printer is well-maintained and calibrated.

\n

I'm searching for a particular nozzle accuracy and filament quality test:

\n\n", "Title": "Can a print test be performed to determine nozzle and filament quality?", "Tags": "|print-quality|filament|nozzle|quality|filament-quality|", "Answer": "

Mostly a Relative measure

\n

Assuming you have a well-known, well-printing nozzle and filament combination, then you can print a benchmark print, change one factor and then test with altered settings.

\n

One of the most ubiquitous prints for a benchmark is Benchy. Benchy isn't so much a calibration test, but has all aspects you need in a benchmark. Overhangs, rounded corners, sharp corners, small diameter parts and sharp corners followed by longer stretches: it's all in there! and with those one gets a decent idea of print quality for other parts.

\n

Another typical benchmark test is a cube, which has the sharp corners and stretches.

\n

Depending on which factors you switched, you get a resulting relative quality comparison between the two prints. But you won't get a measurable metric, unless you invent a score based on artifacts.

\n

Filament factors

\n

Among the factors that would show up based on bad filament. The three most noticeable I can think of are:

\n\n

Nozzle factors

\n

Nozzle problems generally are systemic and would show up on the whole print.

\n\n" }, { "Id": "21603", "CreationDate": "2023-11-07T06:23:00.757", "Body": "

What is considered a 'good' layer height for the first layer? Should it be relatively smaller than the other layers, or should it be considered relative to the nozzle diameter? Can the community suggest an appropriate layer height for the first layer with justification?

\n", "Title": "What is an appropriate layer size for the first layer?", "Tags": "|print-quality|layer-height|layer|", "Answer": "

If the first layer is so thin the bed clearly shows through, your extruder is skipping steps unable to push enough filament, or the nozzle drags over the bed surface, you're too low.

\n

If the cross-section profile of the extruded thread of filament is round, first layer of a print results in separate threads of filament with gaps in between or parts bulging, detached from the bed, filament doesn't form clear sharp corners where it turns 90 degrees, or becomes detached, you're too high.

\n

The right height is when the filament is smeared somewhat flat, but still with noticeable thickness. And considering inaccuracy of printer rails, bed, drive trains, achieving such profile throughout the entire bed surface, never getting into 'too thin' or 'too thick' is what you strive for. Whether it's 0.07 or 0.13mm, doesn't really matter, what matters is it doesn't go below or above the range at any point of the bed.

\n

A typical 0.4mm nozzle with 0.1mm first layer height is most likely to achieve this, but when in doubt, too thin will only result in dimensional inaccuracy and thinner bottom; too high will result in poor adhesion and may end up with the whole print detaching from the bed.

\n" }, { "Id": "21615", "CreationDate": "2023-11-10T18:59:43.917", "Body": "

I updated firmware on my Ender3 v2. Previously I used to have precompiled 2.0.7. Now I compiled 2.0.9 from source. Everything seems to work fine except of Move menu. Previously, I pushed the button to select axis in menu, rotated the button to set move value, and pushed again to confirm and only after confirm my head started to move. Using new firmware, I push the button to select axis, rotate the button - and head starts moving during rotation, not waiting until I push button to confirm move.

\n

Could anyone help me out, which config parameter is responsible for this behavior? Or was it changed inside firmware?

\n", "Title": "Ender3 v2 on Marlin moves head while rotating button", "Tags": "|creality-ender-3|marlin|firmware|", "Answer": "

Depending on the firmware you may have the option to disable live motion. Marlin has different UI for the Ender3V2 (DWIN type display), which you can choose in the configuration.h file just after the line:

\n
// Ender-3 v2 OEM display. A DWIN display with Rotary Encoder.\n
\n

You can enable the DWIN_LCD_PROUI to get the option to desactive live motion

\n
//#define DWIN_CREALITY_LCD           // Creality UI\n#define DWIN_LCD_PROUI              // Pro UI by MRiscoC\n
\n

You can also look for other Marlin forks with more implemented features, for example:

\n

\"Move

\n" }, { "Id": "21620", "CreationDate": "2023-11-11T17:51:30.300", "Body": "

I am really new to 3D printing. I started with downloaded stuff about 2 months ago and now I am starting to do my own "easy" things.

\n

While looking at tutorials I heard one of them talking about doing a chaffer of a specific size/height because he had a 0.40\u00a0mm Nozzle (the same as the one I have with an Anycubic Vyper). This got me thinking about if I should also take into consideration my Nozzle size for when I try to design something.

\n

For example, should I think about multiples of 0.40\u00a0mm when deciding overall lengths or heights?

\n

If I need to create space between 2 pieces, or joint pieces, should I always have to leave a multiple of 0.40\u00a0mm space between those pieces?

\n

I have tried to find out info about this but I just found info about the importance of the Nozzle size at the time of print, but not at design time.

\n

I am using Fusion 360 if that info is needed.

\n", "Title": "Does one has to take into consideration the nozzle diameter while designing something that will be 3D printed?", "Tags": "|3d-design|nozzle|fusion360|", "Answer": "

It would depend on what you are printing and the tolerances that you require.

\n

For example, if you are producing engineering parts, and are a skilled designer, then you should take the nozzle size and layer height into consideration. Though the order of wall printing will probably be of more interest to you (If you print the inner walls first then the outer walls might bulge outwards slightly, and vice versa).

\n

If you ware making garage kit style sculptures and busts, then you don't really need to think about the nozzle at all. Your slicing software will automatically adjust the print to fit what is possible and the slight differences won't make much of a difference.

\n

If you're trying to print warhammer size figures, then yes you do need to take the nozzle width into consideration as it will effect the small details that you print. Which is why people mostly choose resin printers for this scale.

\n

If you're only an average skill designer making items that don't require extremely delicate details or high tolerances for practical parts, then don't really need to even think about your nozzle.

\n

I have an Ender 5 that's still using the 0.4\u00a0mm nozzle that it shipped with quite happily.

\n

TLDR: If you're new to 3D printing and aren't trying to print Warhammer figures or engineering parts, you can largely forget about your nozzle. You won't notice the difference until you're much more skilled at modelling and are looking to do resin quality tollerances.

\n" }, { "Id": "21631", "CreationDate": "2023-11-13T18:55:40.017", "Body": "

I have a 3D printer (Artillery Genius Pro) with a 24\u00a0V motherboard. I want to connect this motherboard a 12\u00a0V laser engraver module. If my motherboard was 12\u00a0V (with 12\u00a0V, GND, and PWM pins), I believe I could do it. In that case, I would connect 12\u00a0V, GND, and PWM of the laser module to 12\u00a0V, GND and Blower Fan (as PWM) of the motherboard. But my 3D printer has a 24\u00a0V board. How can I connect my 12\u00a0V laser module to my 24\u00a0V motherboard?

\n

Your guidance in this direction will be highly appreciated.

\n", "Title": "Connecting 12 V Laser Module to 24 V Control Board", "Tags": "|electronics|laser|", "Answer": "

You'll need a 12\u00a0V step-down regulator to convert your 24\u00a0V to 12V, or a separate power supply for the laser module (make sure to connect each PSU's ground terminals).

\n

It also depends on what sort of PWM your module expects.\nThe fan control on 3D printer boards is done by turning on and off the ground connection to the fan, while the positive supply (12/24\u00a0V) is on constantly. This may not be what your module requires.

\n

Maybe it's fine with a 5\u00a0V PWM signal (usually called "TTL signal"), in which case you could use any available digital pin (e.g. from the endstop connectors) and set your firmware up to use that pin as the "fan". Depending on the firmware, there may be delays between the change in "fan speed" / laser power being scheduled and it actually being applied. If it needs 12\u00a0V PWM, I like using L298N-based "motor drivers" to drive loads like this. Hook up the inputs to the motor driver to the output of the step-down regulator and connect the corresponding input to one of your motherboard's PWM pins.

\n" }, { "Id": "21633", "CreationDate": "2023-11-15T11:45:25.660", "Body": "

Recently added a probe to my Anet A8 for auto bed levelling, calculations have been added based on position of probe to hotend as required. I'm using Marlin 1.1.9.

\n

When I go into 'Prepare' in the printer settings to select auto home, it doesn't centre on the bed, seems to go to the back right. I presume this should be to the centre of the bed? I can't find where to change this in the config. Note the LCD screen shows X:153, Y:114. Config below.

\n
//============================== Endstop Settings ===========================\n\n// @section homing\n\n// Specify here all the endstop connectors that are connected to any endstop or probe.\n// Almost all printers will be using one per axis. Probes will use one or more of the\n// extra connectors. Leave undefined any used for non-endstop and non-probe purposes.\n#define USE_XMIN_PLUG\n#define USE_YMIN_PLUG\n#define USE_ZMIN_PLUG\n//#define USE_XMAX_PLUG\n//#define USE_YMAX_PLUG\n//#define USE_ZMAX_PLUG\n\n// Enable pullup for all endstops to prevent a floating state\n#define ENDSTOPPULLUPS\n#if DISABLED(ENDSTOPPULLUPS)\n  // Disable ENDSTOPPULLUPS to set pullups individually\n  //#define ENDSTOPPULLUP_XMAX\n  //#define ENDSTOPPULLUP_YMAX\n  //#define ENDSTOPPULLUP_ZMAX\n  //#define ENDSTOPPULLUP_XMIN\n  //#define ENDSTOPPULLUP_YMIN\n  //#define ENDSTOPPULLUP_ZMIN\n  //#define ENDSTOPPULLUP_ZMIN_PROBE\n#endif\n\n// Mechanical endstop with COM to ground and NC to Signal uses "false" here (most common setup).\n#define X_MIN_ENDSTOP_INVERTING true  // set to true to invert the logic of the endstop.\n#define Y_MIN_ENDSTOP_INVERTING true  // set to true to invert the logic of the endstop.\n#define Z_MIN_ENDSTOP_INVERTING true  // set to true to invert the logic of the endstop.\n#define X_MAX_ENDSTOP_INVERTING false // set to true to invert the logic of the endstop.\n#define Y_MAX_ENDSTOP_INVERTING false // set to true to invert the logic of the endstop.\n#define Z_MAX_ENDSTOP_INVERTING false // set to true to invert the logic of the endstop.\n#define Z_MIN_PROBE_ENDSTOP_INVERTING true  // set to true to invert the logic of the probe.\n\n// Enable this feature if all enabled endstop pins are interrupt-capable.\n// This will remove the need to poll the interrupt pins, saving many CPU cycles.\n//#define ENDSTOP_INTERRUPTS_FEATURE\n\n/**\n * Endstop Noise Filter\n *\n * Enable this option if endstops falsely trigger due to noise.\n * NOTE: Enabling this feature means adds an error of +/-0.2mm, so homing\n * will end up at a slightly different position on each G28. This will also\n * reduce accuracy of some bed probes.\n * For mechanical switches, the better approach to reduce noise is to install\n * a 100 nanofarads ceramic capacitor in parallel with the switch, making it\n * essentially noise-proof without sacrificing accuracy.\n * This option also increases MCU load when endstops or the probe are enabled.\n * So this is not recommended. USE AT YOUR OWN RISK.\n * (This feature is not required for common micro-switches mounted on PCBs\n * based on the Makerbot design, since they already include the 100nF capacitor.)\n */\n//#define ENDSTOP_NOISE_FILTER\n\n//=============================================================================\n//============================== Movement Settings ============================\n//=============================================================================\n// @section motion\n\n/**\n * Default Settings\n *\n * These settings can be reset by M502\n *\n * Note that if EEPROM is enabled, saved values will override these.\n */\n\n/**\n * With this option each E stepper can have its own factors for the\n * following movement settings. If fewer factors are given than the\n * total number of extruders, the last value applies to the rest.\n */\n//#define DISTINCT_E_FACTORS\n\n/**\n * Default Axis Steps Per Unit (steps/mm)\n * Override with M92\n *                                      X, Y, Z, E0 [, E1[, E2[, E3[, E4]]]]\n */\n#define DEFAULT_AXIS_STEPS_PER_UNIT   { 100, 100, 400, 100 }\n\n/**\n * Default Max Feed Rate (mm/s)\n * Override with M203\n *                                      X, Y, Z, E0 [, E1[, E2[, E3[, E4]]]]\n */\n#define DEFAULT_MAX_FEEDRATE          { 400, 400, 8, 50 }\n\n/**\n * Default Max Acceleration (change/s) change = mm/s\n * (Maximum start speed for accelerated moves)\n * Override with M201\n *                                      X, Y, Z, E0 [, E1[, E2[, E3[, E4]]]]\n */\n#define DEFAULT_MAX_ACCELERATION      { 2000, 2000, 100, 10000 }\n\n/**\n * Default Acceleration (change/s) change = mm/s\n * Override with M204\n *\n *   M204 P    Acceleration\n *   M204 R    Retract Acceleration\n *   M204 T    Travel Acceleration\n */\n#define DEFAULT_ACCELERATION          400     // X, Y, Z and E acceleration for printing moves\n#define DEFAULT_RETRACT_ACCELERATION  1000    // E acceleration for retracts\n#define DEFAULT_TRAVEL_ACCELERATION   1000    // X, Y, Z acceleration for travel (non printing) moves\n\n/**\n * Default Jerk (mm/s)\n * Override with M205 X Y Z E\n *\n * "Jerk" specifies the minimum speed change that requires acceleration.\n * When changing speed and direction, if the difference is less than the\n * value set here, it may happen instantaneously.\n */\n#define DEFAULT_XJERK                 10.0\n#define DEFAULT_YJERK                 10.0\n#define DEFAULT_ZJERK                  0.3\n#define DEFAULT_EJERK                  5.0\n\n/**\n * S-Curve Acceleration\n *\n * This option eliminates vibration during printing by fitting a B\u00c3\u00a9zier\n * curve to move acceleration, producing much smoother direction changes.\n *\n * See https://github.com/synthetos/TinyG/wiki/Jerk-Controlled-Motion-Explained\n */\n//#define S_CURVE_ACCELERATION\n\n//===========================================================================\n//============================= Z Probe Options =============================\n//===========================================================================\n// @section probes\n\n//\n// See http://marlinfw.org/docs/configuration/probes.html\n//\n\n/**\n * Z_MIN_PROBE_USES_Z_MIN_ENDSTOP_PIN\n *\n * Enable this option for a probe connected to the Z Min endstop pin.\n */\n#define Z_MIN_PROBE_USES_Z_MIN_ENDSTOP_PIN\n\n/**\n * Z_MIN_PROBE_ENDSTOP\n *\n * Enable this option for a probe connected to any pin except Z-Min.\n * (By default Marlin assumes the Z-Max endstop pin.)\n * To use a custom Z Probe pin, set Z_MIN_PROBE_PIN below.\n *\n *  - The simplest option is to use a free endstop connector.\n *  - Use 5V for powered (usually inductive) sensors.\n *\n *  - RAMPS 1.3/1.4 boards may use the 5V, GND, and Aux4->D32 pin:\n *    - For simple switches connect...\n *      - normally-closed switches to GND and D32.\n *      - normally-open switches to 5V and D32.\n *\n * WARNING: Setting the wrong pin may have unexpected and potentially\n * disastrous consequences. Use with caution and do your homework.\n *\n */\n//#define Z_MIN_PROBE_ENDSTOP\n\n/**\n * Probe Type\n *\n * Allen Key Probes, Servo Probes, Z-Sled Probes, FIX_MOUNTED_PROBE, etc.\n * Activate one of these to use Auto Bed Leveling below.\n */\n\n/**\n * The "Manual Probe" provides a means to do "Auto" Bed Leveling without a probe.\n * Use G29 repeatedly, adjusting the Z height at each point with movement commands\n * or (with LCD_BED_LEVELING) the LCD controller.\n */\n//#define PROBE_MANUALLY\n//#define MANUAL_PROBE_START_Z 0.2\n\n/**\n * A Fix-Mounted Probe either doesn't deploy or needs manual deployment.\n *   (e.g., an inductive probe or a nozzle-based probe-switch.)\n */\n#define FIX_MOUNTED_PROBE\n\n/**\n * Z Servo Probe, such as an endstop switch on a rotating arm.\n */\n//#define Z_PROBE_SERVO_NR 0   // Defaults to SERVO 0 connector.\n//#define Z_SERVO_ANGLES {70,0}  // Z Servo Deploy and Stow angles\n\n/**\n * The BLTouch probe uses a Hall effect sensor and emulates a servo.\n */\n//#define BLTOUCH\n#if ENABLED(BLTOUCH)\n  //#define BLTOUCH_DELAY 375   // (ms) Enable and increase if needed\n#endif\n\n/**\n * Enable one or more of the following if probing seems unreliable.\n * Heaters and/or fans can be disabled during probing to minimize electrical\n * noise. A delay can also be added to allow noise and vibration to settle.\n * These options are most useful for the BLTouch probe, but may also improve\n * readings with inductive probes and piezo sensors.\n */\n//#define PROBING_HEATERS_OFF       // Turn heaters off when probing\n#if ENABLED(PROBING_HEATERS_OFF)\n  //#define WAIT_FOR_BED_HEATER     // Wait for bed to heat back up between probes (to improve accuracy)\n#endif\n//#define PROBING_FANS_OFF          // Turn fans off when probing\n//#define DELAY_BEFORE_PROBING 200  // (ms) To prevent vibrations from triggering piezo sensors\n\n// A probe that is deployed and stowed with a solenoid pin (SOL1_PIN)\n//#define SOLENOID_PROBE\n\n// A sled-mounted probe like those designed by Charles Bell.\n//#define Z_PROBE_SLED\n//#define SLED_DOCKING_OFFSET 5  // The extra distance the X axis must travel to pickup the sled. 0 should be fine but you can push it further if you'd like.\n\n//\n// For Z_PROBE_ALLEN_KEY see the Delta example configurations.\n//\n\n/**\n *   Z Probe to nozzle (X,Y) offset, relative to (0, 0).\n *   X and Y offsets must be integers.\n *\n *   In the following example the X and Y offsets are both positive:\n *   #define X_PROBE_OFFSET_FROM_EXTRUDER 10\n *   #define Y_PROBE_OFFSET_FROM_EXTRUDER 10\n *\n *      +-- BACK ---+\n *      |           |\n *    L |    (+) P  | R <-- probe (20,20)\n *    E |           | I\n *    F | (-) N (+) | G <-- nozzle (10,10)\n *    T |           | H\n *      |    (-)    | T\n *      |           |\n *      O-- FRONT --+\n *    (0,0)\n */\n#define X_PROBE_OFFSET_FROM_EXTRUDER -43\n#define Y_PROBE_OFFSET_FROM_EXTRUDER -4\n#define Z_PROBE_OFFSET_FROM_EXTRUDER 0\n// X offset: -left  +right  [of the nozzle]\n// Y offset: -front +behind [the nozzle]\n// Z offset: -below +above  [the nozzle]\n\n// Certain types of probes need to stay away from edges\n#define MIN_PROBE_EDGE 10\n\n// X and Y axis travel speed (mm/m) between probes\n#define XY_PROBE_SPEED 6000\n\n// Feedrate (mm/m) for the first approach when double-probing (MULTIPLE_PROBING == 2)\n#define Z_PROBE_SPEED_FAST HOMING_FEEDRATE_Z\n\n// Feedrate (mm/m) for the "accurate" probe of each point\n#define Z_PROBE_SPEED_SLOW (Z_PROBE_SPEED_FAST / 2)\n\n// The number of probes to perform at each point.\n//   Set to 2 for a fast/slow probe, using the second probe result.\n//   Set to 3 or more for slow probes, averaging the results.\n#define MULTIPLE_PROBING 3\n\n/**\n * Z probes require clearance when deploying, stowing, and moving between\n * probe points to avoid hitting the bed and other hardware.\n * Servo-mounted probes require extra space for the arm to rotate.\n * Inductive probes need space to keep from triggering early.\n *\n * Use these settings to specify the distance (mm) to raise the probe (or\n * lower the bed). The values set here apply over and above any (negative)\n * probe Z Offset set with Z_PROBE_OFFSET_FROM_EXTRUDER, M851, or the LCD.\n * Only integer values >= 1 are valid here.\n *\n * Example: `M851 Z-5` with a CLEARANCE of 4  =>  9mm from bed to nozzle.\n *     But: `M851 Z+1` with a CLEARANCE of 2  =>  2mm from bed to nozzle.\n */\n#define Z_CLEARANCE_DEPLOY_PROBE   10 // Z Clearance for Deploy/Stow\n#define Z_CLEARANCE_BETWEEN_PROBES  5 // Z Clearance between probe points\n#define Z_CLEARANCE_MULTI_PROBE     5 // Z Clearance between multiple probes\n//#define Z_AFTER_PROBING           5 // Z position after probing is done\n\n#define Z_PROBE_LOW_POINT          -2 // Farthest distance below the trigger-point to go before stopping\n\n// For M851 give a range for adjusting the Z probe offset\n#define Z_PROBE_OFFSET_RANGE_MIN -20\n#define Z_PROBE_OFFSET_RANGE_MAX 20\n\n// Enable the M48 repeatability test to test probe accuracy\n//#define Z_MIN_PROBE_REPEATABILITY_TEST\n\n// For Inverting Stepper Enable Pins (Active Low) use 0, Non Inverting (Active High) use 1\n// :{ 0:'Low', 1:'High' }\n#define X_ENABLE_ON 0\n#define Y_ENABLE_ON 0\n#define Z_ENABLE_ON 0\n#define E_ENABLE_ON 0 // For all extruders\n\n// Disables axis stepper immediately when it's not being used.\n// WARNING: When motors turn off there is a chance of losing position accuracy!\n#define DISABLE_X false\n#define DISABLE_Y false\n#define DISABLE_Z false\n// Warn on display about possibly reduced accuracy\n//#define DISABLE_REDUCED_ACCURACY_WARNING\n\n// @section extruder\n\n#define DISABLE_E false // For all extruders\n#define DISABLE_INACTIVE_EXTRUDER true // Keep only the active extruder enabled.\n\n// @section machine\n\n// Invert the stepper direction. Change (or reverse the motor connector) if an axis goes the wrong way.\n#define INVERT_X_DIR false\n#define INVERT_Y_DIR false\n#define INVERT_Z_DIR true\n\n// @section extruder\n\n// For direct drive extruder v9 set to true, for geared extruder set to false.\n#define INVERT_E0_DIR false\n#define INVERT_E1_DIR false\n#define INVERT_E2_DIR false\n#define INVERT_E3_DIR false\n#define INVERT_E4_DIR false\n\n// @section homing\n\n//#define NO_MOTION_BEFORE_HOMING  // Inhibit movement until all axes have been homed\n\n//#define UNKNOWN_Z_NO_RAISE // Don't raise Z (lower the bed) if Z is "unknown." For beds that fall when Z is powered off.\n\n//#define Z_HOMING_HEIGHT 4  // (in mm) Minimal z height before homing (G28) for Z clearance above the bed, clamps, ...\n                             // Be sure you have this distance over your Z_MAX_POS in case.\n\n// Direction of endstops when homing; 1=MAX, -1=MIN\n// :[-1,1]\n#define X_HOME_DIR -1\n#define Y_HOME_DIR -1\n#define Z_HOME_DIR -1\n\n// @section machine\n\n// The size of the print bed\n#define X_BED_SIZE 220\n#define Y_BED_SIZE 220\n\n// Travel limits (mm) after homing, corresponding to endstop positions.\n#define X_MIN_POS -33\n#define Y_MIN_POS -10\n#define Z_MIN_POS 0\n#define X_MAX_POS X_BED_SIZE\n#define Y_MAX_POS Y_BED_SIZE\n#define Z_MAX_POS 240\n\n/**\n * Software Endstops\n *\n * - Prevent moves outside the set machine bounds.\n * - Individual axes can be disabled, if desired.\n * - X and Y only apply to Cartesian robots.\n * - Use 'M211' to set software endstops on/off or report current state\n */\n\n// Min software endstops constrain movement within minimum coordinate bounds\n#define MIN_SOFTWARE_ENDSTOPS\n#if ENABLED(MIN_SOFTWARE_ENDSTOPS)\n  #define MIN_SOFTWARE_ENDSTOP_X\n  #define MIN_SOFTWARE_ENDSTOP_Y\n  #define MIN_SOFTWARE_ENDSTOP_Z\n#endif\n\n// Max software endstops constrain movement within maximum coordinate bounds\n#define MAX_SOFTWARE_ENDSTOPS\n#if ENABLED(MAX_SOFTWARE_ENDSTOPS)\n  #define MAX_SOFTWARE_ENDSTOP_X\n  #define MAX_SOFTWARE_ENDSTOP_Y\n  #define MAX_SOFTWARE_ENDSTOP_Z\n#endif\n\n#if ENABLED(MIN_SOFTWARE_ENDSTOPS) || ENABLED(MAX_SOFTWARE_ENDSTOPS)\n  //#define SOFT_ENDSTOPS_MENU_ITEM  // Enable/Disable software endstops from the LCD\n#endif\n\n/**\n * Filament Runout Sensors\n * Mechanical or opto endstops are used to check for the presence of filament.\n *\n * RAMPS-based boards use SERVO3_PIN for the first runout sensor.\n * For other boards you may need to define FIL_RUNOUT_PIN, FIL_RUNOUT2_PIN, etc.\n * By default the firmware assumes HIGH=FILAMENT PRESENT.\n */\n//#define FILAMENT_RUNOUT_SENSOR\n#if ENABLED(FILAMENT_RUNOUT_SENSOR)\n  #define NUM_RUNOUT_SENSORS   1     // Number of sensors, up to one per extruder. Define a FIL_RUNOUT#_PIN for each.\n  #define FIL_RUNOUT_INVERTING false // set to true to invert the logic of the sensor.\n  #define FIL_RUNOUT_PULLUP          // Use internal pullup for filament runout pins.\n  #define FILAMENT_RUNOUT_SCRIPT "M600"\n#endif\n\n//===========================================================================\n//=============================== Bed Leveling ==============================\n//===========================================================================\n// @section calibrate\n\n/**\n * Choose one of the options below to enable G29 Bed Leveling. The parameters\n * and behavior of G29 will change depending on your selection.\n *\n *  If using a Probe for Z Homing, enable Z_SAFE_HOMING also!\n *\n * - AUTO_BED_LEVELING_3POINT\n *   Probe 3 arbitrary points on the bed (that aren't collinear)\n *   You specify the XY coordinates of all 3 points.\n *   The result is a single tilted plane. Best for a flat bed.\n *\n * - AUTO_BED_LEVELING_LINEAR\n *   Probe several points in a grid.\n *   You specify the rectangle and the density of sample points.\n *   The result is a single tilted plane. Best for a flat bed.\n *\n * - AUTO_BED_LEVELING_BILINEAR\n *   Probe several points in a grid.\n *   You specify the rectangle and the density of sample points.\n *   The result is a mesh, best for large or uneven beds.\n *\n * - AUTO_BED_LEVELING_UBL (Unified Bed Leveling)\n *   A comprehensive bed leveling system combining the features and benefits\n *   of other systems. UBL also includes integrated Mesh Generation, Mesh\n *   Validation and Mesh Editing systems.\n *\n * - MESH_BED_LEVELING\n *   Probe a grid manually\n *   The result is a mesh, suitable for large or uneven beds. (See BILINEAR.)\n *   For machines without a probe, Mesh Bed Leveling provides a method to perform\n *   leveling in steps so you can manually adjust the Z height at each grid-point.\n    With an LCD controller the process is guided step-by-step.\n */\n//#define AUTO_BED_LEVELING_3POINT\n//#define AUTO_BED_LEVELING_LINEAR\n#define AUTO_BED_LEVELING_BILINEAR\n//#define AUTO_BED_LEVELING_UBL\n//#define MESH_BED_LEVELING\n\n/**\n * Normally G28 leaves leveling disabled on completion. Enable\n * this option to have G28 restore the prior leveling state.\n */\n//#define RESTORE_LEVELING_AFTER_G28\n\n/**\n * Enable detailed logging of G28, G29, M48, etc.\n * Turn on with the command 'M111 S32'.\n * NOTE: Requires a lot of PROGMEM!\n */\n//#define DEBUG_LEVELING_FEATURE\n\n#if ENABLED(MESH_BED_LEVELING) || ENABLED(AUTO_BED_LEVELING_BILINEAR) || ENABLED(AUTO_BED_LEVELING_UBL)\n  // Gradually reduce leveling correction until a set height is reached,\n  // at which point movement will be level to the machine's XY plane.\n  // The height can be set with M420 Z<height>\n  #define ENABLE_LEVELING_FADE_HEIGHT\n\n  // For Cartesian machines, instead of dividing moves on mesh boundaries,\n  // split up moves into short segments like a Delta. This follows the\n  // contours of the bed more closely than edge-to-edge straight moves.\n  #define SEGMENT_LEVELED_MOVES\n  #define LEVELED_SEGMENT_LENGTH 5.0 // (mm) Length of all segments (except the last one)\n\n  /**\n   * Enable the G26 Mesh Validation Pattern tool.\n   */\n  //#define G26_MESH_VALIDATION\n  #if ENABLED(G26_MESH_VALIDATION)\n    #define MESH_TEST_NOZZLE_SIZE    0.4  // (mm) Diameter of primary nozzle.\n    #define MESH_TEST_LAYER_HEIGHT   0.2  // (mm) Default layer height for the G26 Mesh Validation Tool.\n    #define MESH_TEST_HOTEND_TEMP  205.0  // (\u00c2\u00b0C) Default nozzle temperature for the G26 Mesh Validation Tool.\n    #define MESH_TEST_BED_TEMP      60.0  // (\u00c2\u00b0C) Default bed temperature for the G26 Mesh Validation Tool.\n  #endif\n\n#endif\n\n#if ENABLED(AUTO_BED_LEVELING_LINEAR) || ENABLED(AUTO_BED_LEVELING_BILINEAR)\n\n  // Set the number of grid points per dimension.\n  #define GRID_MAX_POINTS_X 3\n  #define GRID_MAX_POINTS_Y GRID_MAX_POINTS_X\n\n  // Set the boundaries for probing (where the probe can reach).\n  //#define LEFT_PROBE_BED_POSITION MIN_PROBE_EDGE 10\n  //#define RIGHT_PROBE_BED_POSITION (X_BED_SIZE - MIN_PROBE_EDGE) 167\n  //#define FRONT_PROBE_BED_POSITION MIN_PROBE_EDGE 10\n  //#define BACK_PROBE_BED_POSITION (Y_BED_SIZE - MIN_PROBE_EDGE) 202\n  #define LEFT_PROBE_BED_POSITION 10\n  #define RIGHT_PROBE_BED_POSITION 167\n  #define FRONT_PROBE_BED_POSITION 10\n  #define BACK_PROBE_BED_POSITION 206\n\n\n  // Probe along the Y axis, advancing X after each column\n  //#define PROBE_Y_FIRST\n\n  #if ENABLED(AUTO_BED_LEVELING_BILINEAR)\n\n    // Beyond the probed grid, continue the implied tilt?\n    // Default is to maintain the height of the nearest edge.\n    //#define EXTRAPOLATE_BEYOND_GRID\n\n    //\n    // Experimental Subdivision of the grid by Catmull-Rom method.\n    // Synthesizes intermediate points to produce a more detailed mesh.\n    //\n    //#define ABL_BILINEAR_SUBDIVISION\n    #if ENABLED(ABL_BILINEAR_SUBDIVISION)\n      // Number of subdivisions between probe points\n      #define BILINEAR_SUBDIVISIONS 3\n    #endif\n\n  #endif\n\n#elif ENABLED(AUTO_BED_LEVELING_UBL)\n\n  //===========================================================================\n  //========================= Unified Bed Leveling ============================\n  //===========================================================================\n\n  //#define MESH_EDIT_GFX_OVERLAY   // Display a graphics overlay while editing the mesh\n\n  #define MESH_INSET 1              // Set Mesh bounds as an inset region of the bed\n  #define GRID_MAX_POINTS_X 10      // Don't use more than 15 points per axis, implementation limited.\n  #define GRID_MAX_POINTS_Y GRID_MAX_POINTS_X\n\n  #define UBL_MESH_EDIT_MOVES_Z     // Sophisticated users prefer no movement of nozzle\n  #define UBL_SAVE_ACTIVE_ON_M500   // Save the currently active mesh in the current slot on M500\n\n  //#define UBL_Z_RAISE_WHEN_OFF_MESH 2.5 // When the nozzle is off the mesh, this value is used\n                                          // as the Z-Height correction value.\n\n#elif ENABLED(MESH_BED_LEVELING)\n\n  //===========================================================================\n  //=================================== Mesh ==================================\n  //===========================================================================\n\n  #define MESH_INSET 10          // Set Mesh bounds as an inset region of the bed\n  #define GRID_MAX_POINTS_X 3    // Don't use more than 7 points per axis, implementation limited.\n  #define GRID_MAX_POINTS_Y GRID_MAX_POINTS_X\n\n  //#define MESH_G28_REST_ORIGIN // After homing all axes ('G28' or 'G28 XYZ') rest Z at Z_MIN_POS\n\n#endif // BED_LEVELING\n\n/**\n * Points to probe for all 3-point Leveling procedures.\n * Override if the automatically selected points are inadequate.\n */\n#if ENABLED(AUTO_BED_LEVELING_3POINT) || ENABLED(AUTO_BED_LEVELING_UBL)\n  #define PROBE_PT_1_X 20\n  #define PROBE_PT_1_Y 160\n  #define PROBE_PT_2_X 20\n  #define PROBE_PT_2_Y 10\n  #define PROBE_PT_3_X 180\n  #define PROBE_PT_3_Y 10\n#endif\n\n/**\n * Add a bed leveling sub-menu for ABL or MBL.\n * Include a guided procedure if manual probing is enabled.\n */\n//#define LCD_BED_LEVELING\n\n#if ENABLED(LCD_BED_LEVELING)\n  #define MBL_Z_STEP 0.025    // Step size while manually probing Z axis.\n  #define LCD_PROBE_Z_RANGE 4 // Z Range centered on Z_MIN_POS for LCD Z adjustment\n#endif\n\n// Add a menu item to move between bed corners for manual bed adjustment\n//#define LEVEL_BED_CORNERS\n\n#if ENABLED(LEVEL_BED_CORNERS)\n  #define LEVEL_CORNERS_INSET 30    // (mm) An inset for corner leveling\n  //#define LEVEL_CENTER_TOO        // Move to the center after the last corner\n#endif\n\n/**\n * Commands to execute at the end of G29 probing.\n * Useful to retract or move the Z probe out of the way.\n */\n//#define Z_PROBE_END_SCRIPT "G1 Z10 F12000\\nG1 X15 Y330\\nG1 Z0.5\\nG1 Z10"\n\n\n// @section homing\n\n// The center of the bed is at (X=0, Y=0)\n//#define BED_CENTER_AT_0_0\n\n// Manually set the home position. Leave these undefined for automatic settings.\n// For DELTA this is the top-center of the Cartesian print volume.\n//#define MANUAL_X_HOME_POS 0\n//#define MANUAL_Y_HOME_POS 0\n//#define MANUAL_Z_HOME_POS 0\n\n// Use "Z Safe Homing" to avoid homing with a Z probe outside the bed area.\n//\n// With this feature enabled:\n//\n// - Allow Z homing only after X and Y homing AND stepper drivers still enabled.\n// - If stepper drivers time out, it will need X and Y homing again before Z homing.\n// - Move the Z probe (or nozzle) to a defined XY point before Z Homing when homing all axes (G28).\n// - Prevent Z homing when the Z probe is outside bed area.\n//\n#define Z_SAFE_HOMING\n\n#if ENABLED(Z_SAFE_HOMING)\n  #define Z_SAFE_HOMING_X_POINT ((X_BED_SIZE) / 2)     // X point for Z homing when homing all axes (G28).\n  #define Z_SAFE_HOMING_Y_POINT ((Y_BED_SIZE) / 2)     // Y point for Z homing when homing all axes (G28).\n#endif\n\n// Homing speeds (mm/m)\n#define HOMING_FEEDRATE_XY (100*60)\n#define HOMING_FEEDRATE_Z  (4*60)\n\n// @section calibrate\n\n/**\n * Bed Skew Compensation\n *\n * This feature corrects for misalignment in the XYZ axes.\n *\n * Take the following steps to get the bed skew in the XY plane:\n *  1. Print a test square (e.g., https://www.thingiverse.com/thing:2563185)\n *  2. For XY_DIAG_AC measure the diagonal A to C\n *  3. For XY_DIAG_BD measure the diagonal B to D\n *  4. For XY_SIDE_AD measure the edge A to D\n *\n * Marlin automatically computes skew factors from these measurements.\n * Skew factors may also be computed and set manually:\n *\n *  - Compute AB     : SQRT(2*AC*AC+2*BD*BD-4*AD*AD)/2\n *  - XY_SKEW_FACTOR : TAN(PI/2-ACOS((AC*AC-AB*AB-AD*AD)/(2*AB*AD)))\n *\n * If desired, follow the same procedure for XZ and YZ.\n * Use these diagrams for reference:\n *\n *    Y                     Z                     Z\n *    ^     B-------C       ^     B-------C       ^     B-------C\n *    |    /       /        |    /       /        |    /       /\n *    |   /       /         |   /       /         |   /       /\n *    |  A-------D          |  A-------D          |  A-------D\n *    +-------------->X     +-------------->X     +-------------->Y\n *     XY_SKEW_FACTOR        XZ_SKEW_FACTOR        YZ_SKEW_FACTOR\n */\n//#define SKEW_CORRECTION\n\n#if ENABLED(SKEW_CORRECTION)\n  // Input all length measurements here:\n  #define XY_DIAG_AC 282.8427124746\n  #define XY_DIAG_BD 282.8427124746\n  #define XY_SIDE_AD 200\n\n  // Or, set the default skew factors directly here\n  // to override the above measurements:\n  #define XY_SKEW_FACTOR 0.0\n\n  //#define SKEW_CORRECTION_FOR_Z\n  #if ENABLED(SKEW_CORRECTION_FOR_Z)\n    #define XZ_DIAG_AC 282.8427124746\n    #define XZ_DIAG_BD 282.8427124746\n    #define YZ_DIAG_AC 282.8427124746\n    #define YZ_DIAG_BD 282.8427124746\n    #define YZ_SIDE_AD 200\n    #define XZ_SKEW_FACTOR 0.0\n    #define YZ_SKEW_FACTOR 0.0\n  #endif\n\n  // Enable this option for M852 to set skew at runtime\n  //#define SKEW_CORRECTION_GCODE\n#endif\n
\n", "Title": "Anet A8 Auto Bed Levelling - Auto home not centring with new Marlin 1.1.9 config", "Tags": "|marlin|anet-a8|automatic-bed-leveling|", "Answer": "

Your config looks fine for the most of the definitions that have been compared to the known settings when using a probe.

\n

You should be aware that with #define Z_SAFE_HOMING you define that the sensor is in the middle of the bed, however, the nozzle is then offset from the middle of the bed per the #define X_PROBE_OFFSET_FROM_EXTRUDER -43 and #define Y_PROBE_OFFSET_FROM_EXTRUDER -4 defined offsets.

\n

So, when G28 is commanded, the carriage will move to (110, 110) for the probe which is displayed as (110 - -43, 110 - -4) = (153, 114) for the nozzle.

\n

So there is nothing wrong with the configuration, it works as it should.

\n" }, { "Id": "21660", "CreationDate": "2023-11-21T12:53:22.077", "Body": "

I'm looking into Input Shaping (Resonance Compensation) to compensate for vibration on an Ender 3 V2. I found the "Official Creality G-Sensor ADXL345 Vibration Compensation Sensor for Sonic Pad 3D Printed Smart Pad" which I guess, unlike the name suggests, can work on more than just the Sonic Pad.

\n

\"Official

\n

I have a fairly stock Creality Ender 3 V2 printer running the Marlin based mriscoc firmware. Marlin Firmware supports Input Shaping (M593) and so does the mriscoc firmware.

\n

The Marlin documentation states:

\n
\n

It\u2019s not always easy to attach an accelerometer to most printer boards, so Marlin doesn\u2019t provide accelerator-based tuning. If you have that information you can use it.

\n
\n

The mriscoc firmware documentation, which is specifically for Ender 3 V2 and Marlin based, states:

\n
\n

The IS parameters are the resonance frequencies of your printer\u2019s X and Y axes, which can be measured using an accelerometer or a test print

\n
\n

I'm thinking to buy the bare ADXL345 accelerometer. Is it possible to wire this up, and if so, how do I wire this accelerometer to my 4.2.2 board? I guess I can than continue by simply flashing the IS enabled firmware of mriscoc to get it supported in the firmware.

\n", "Title": "How to physically wire a (ADXL345) accelerometer for Input Shaping (Resonance Compensation) support on for a Ender 3 V2 printer?", "Tags": "|creality-ender-3|input-shaping|resonance-compensation|mriscoc|vibration-compensation|", "Answer": "

Currently Marlin does not provide a way to get the resonance frequencies by using an accelerometer and possibly never will because of the lack of resources in current printer boards. You can get those values by other medium and save them as Marlin IS parameters. It is possible to estimate the resonance frequencies by test prints.

\n" }, { "Id": "21661", "CreationDate": "2023-11-21T13:03:28.320", "Body": "

In addition to my question How to physically wire a (ADXL345) accelerometer for Input Shaping (Resonance Compensation) support on for a Ender 3 V2 printer?.

\n

Does an accelerometer in a Marlin-based Input Shaping setup compensate in real-time (during print) for vibration? Or is it a one time (for example before print) measurement to determine a level of vibration and use that as the compensation scheme during a print, as in "not adapting to unexpected vibration"? Similar to how bed leveling is used.

\n

If it does not do real-time compensation, what is the benefit of having an accelerometer compared to doing Input Shaping test prints instead?

\n", "Title": "Does an accelerometer in a Marlin-based Input Shaping setup compensate in real-time (during print) for vibration?", "Tags": "|marlin|input-shaping|resonance-compensation|vibration-compensation|", "Answer": "

Input shaping in 3D printers is typically implemented prior to the actual printing operation, not in real-time. It's based on the known characteristics of the printer rather than dynamic adjustments made during printing. This requires to be set-up and determined before printing. The printer's firmware is where you can define input-shaping.

\n

3D printer firmware based input shaping

\n

The resonant frequencies of the printer are determined through test prints and input into the firmware. The firmware then automatically applies the necessary adjustments to the motion commands during the printing process (it reads the G-code and applies the algorithms on motion changes). This is still not real-time adjustment; it's a predefined modification based on the printer's known characteristics.

\n

Real-time or not?

\n

Input shaping is not a real-time control system that dynamically adjusts to changing conditions during a print. Instead, it's a method of optimizing the printer's motion commands based on its known mechanical properties and behavior (hardware determined). Why not real-time? Real-time control for input shaping in 3D printing would be complex and would require additional sensors and processing capabilities (controller board). The printer would need to continuously monitor its own vibrations and adjust its movements on the fly, which is not typically feasible with current consumer level 3D printing technology.

\n" }, { "Id": "21662", "CreationDate": "2023-11-21T13:28:42.003", "Body": "

The Marlin Firmware supports Input Shaping (M593) as described in the Marlin documentation writing:

\n
\n

Set the Input Shaping damping factor and/or frequency (in Hertz) for axes that support it.

\n
\n

But the parameters shown under the Usage section suggest only axis X and Y are supported? It is possible to use input shaping (in Marlin) to compensate vibration on all X, Y and Z axes?

\n", "Title": "Does Marlin input shaping compensate vibration on all X, Y and Z axes?", "Tags": "|marlin|input-shaping|resonance-compensation|vibration-compensation|", "Answer": "

No, as of December 2023, Z-axis Input Shaping is not possible in Marlin firmware. Marlin only supports Input Shaping on the X and Y axes.

\n

There is a feature request on GitHub to implement Z-axis Input Shaping on Marlin.

\n" }, { "Id": "21668", "CreationDate": "2023-11-22T22:49:02.310", "Body": "

I have an odd issue with Creality Slicer 4.8 (Cura slicer?)

\n

It wants to cut up my model and add surfaces that do not exist in the model.

\n

These screenshots are without infill or support:

\n

\"Head

\n

\"layer

\n

\"layer

\n

Wire mesh view from behind and to the left (Blender)\n\"Blender

\n

I assume that since this model is a base 3D model from a game with the body, clothes, hairstyle, etc put on top, that I have overlooked something which is being interpreted as a surface break. I had a similar problem with the dress and boots, which I solved by tediously removing the inner details.

\n

I have removed the back part of the "skull" and the top of the neck, trying to closed any gaps.

\n

\"solid

\n

STL

\n

How do I fix this issue?

\n

I am printing on a Creality Ender 2.

\n", "Title": "Issues with slicer creating gaps with surfaces that are not in the model", "Tags": "|3d-models|slicing|creality-ender-2|", "Answer": "

Some Surfaces are likely inverted - which needs to be made visible.

\n

To see that in blender, you need to turn on "surface normals". The blue spikes should go out of the surfaces, showing the item is properly aligned.

\n

\"position

\n

To flip those surfaces and make all show into the same direction, follow for example this answer Slicer is adding additional floor bed - the relevant part for it here:

\n
\n\n
\n

Another way to get the normals displayed is:

\n
\n\n
\n" }, { "Id": "21672", "CreationDate": "2023-11-24T12:41:17.290", "Body": "

I'm exploring the concept of Pressure Advance and its ability to predict and adjust pressure changes in the extruder. The image below shows the effects of Pressure Advance:

\n

\"Pressure

\n

Source: Teaching Tech via YouTube

\n

Since there are no pressure measurements performed, how does Pressure Advance algorithmically anticipate these pressure variations without real-time pressure data? Which underlying principles are used to estimate the pressure in advance?

\n", "Title": "How does pressure advance predict pressure without having actual pressure data?", "Tags": "|extrusion|nozzle|pressure-advance|pressure-prediction|pressure|", "Answer": "

Short answer: by calibration.

\n

Expanding on this short answer: pressure advance (or also called "linear advance") can be used to compensate for the elasticity of the filament and the extruder system. There are at least three sources of elasticity, the filament in a Bowden tube, the compression of the filament itself and the conversion of stepper rotation into torque itself lagging behind.

\n

To counter affect an unwanted effect, you need a measure to change the unwanted effect so that after tuning the unwanted effect is gone.

\n

The build-up of pressure in the nozzle (as a result of the springiness of the complete extrusion system causes the end of a line to over-extrude when movement slows or stops. Consequently, a lack of pressure at the beginning (first need to compress the filament) will result in a lack of filament at the nozzle.

\n

In order to compensate, we use test prints, not sensors, in which the printer owner/tuner selects the best extruded line based on various settings of the pressure advance ratio. This works well, the extrusion system doesn't generally change over time unless you change the extruder, Bowden tube lengths, etcetera. There is no need for a sensor if you can determine how to counter the effect

\n" }, { "Id": "21674", "CreationDate": "2023-11-24T17:58:45.553", "Body": "

I have a tooth and a nerve inside it as two different objects.

\n

The first picture is a molar tooth and the second image is a nerve inside the molar tooth.

\n

My question is; how to generate supports so that the nerve inside the molar tooth will be printed out? (3D printing object inside another object)

\n

The purpose of the aim is to use the printed object as patient education material. (Not for sale)

\n

\"3D

\n

\"3D

\n", "Title": "How do I 3D print an object inside another object?", "Tags": "|3d-design|support-structures|prusaslicer|", "Answer": "

Similar to the answer above, but with a bit of a twist, it may be simpler to print both parts out as molds, and then to cast the parts that you want in two-part casting resin.

\n

There are various tutorials on the internet that go into detail on how to do this.

\n

For example: 3D Printed Molds Tutorial: How to DIY Your Own

\n

Cura slicer even has a function for this. Mold mode though I've never used it.

\n

Use a Boolean function to create a cavity inside the tooth using the nerve as a cutter, and then export it. Then use Cura's mold function to create a mold of the tooth with the cavity inside it.

\n

Create a resin tooth using clear resin, and then once that is set mix up some red resin and pour it into the cavity.

\n

It's still using 3D printing, just not as you originally thought. IT may be easiest to do the tooth mold in two parts and then join them together.

\n" }, { "Id": "21679", "CreationDate": "2023-11-27T10:27:04.893", "Body": "

My heaterblock is in a poor state after the heater malfunction. Before I'll buy a spare, I want to ask what material is the original block made of? I don't see it on the product page.

\n

Available spares and replacements are made of aluminum, brass, and copper, the latter two in plated and not plated versions. Original is, obviously, aluminum or plated something. Aluminum 3rd party replacements are 10 times cheaper than brass or copper ones. So, before I'll buy expensive one, I want to know if I really have to.

\n

If original is aluminum, the supplemental question would be why to buy plated copper despite the price difference.

\n", "Title": "What material is Prusa mk3s+ hotend heaterblock made of?", "Tags": "|prusa-i3|hotend|replacement-parts|", "Answer": "

Aluminum

\n

At 9.20 grams (around 0.32 ounces) including screws and some soot I wasn't able to clean out, it can't be anything but aluminum.

\n

\"Photo

\n" }, { "Id": "21681", "CreationDate": "2023-11-27T14:06:30.703", "Body": "

The YouTube video 'PID vs MPC' from Mark Misin Engineering Ltd perfectly demonstrates that Model Predictive Temperature Control (MPC) is more powerful than Proportional-Integral-Derivative (PID). Marlin implemented MPC as described in Marlin's MPC documentation. The Marlin-based firmware version I use also describes MPC in the mriscoc MPC documentation and as follows:

\n
\n

In the latest releases, MPC is being incorporated in all versions. MPC has proven to be a better algorithm for keeping the nozzle temperature stable, and it is also very useful for high-power heaters.
Source: GitHub mriscoc repository for Ender 3 V2/S1

\n
\n

Since 3D printers, especially the heated parts (bed and nozzle), are relatively simple, with only simple control and sensor readings, why exactly is MPC a better choice than PID? And why is it more useful for 'high-power heaters'?

\n", "Title": "Why is MPC better than PID for 3D printers, and (why) is it more useful for 'high-power heaters'?", "Tags": "|marlin|temperature|pid|mriscoc|mpc|", "Answer": "

Short answer

\n

PID reacts only after the temperature changes are registered by the sensor. MPC instead calculates the expected heat loss before the temperature actually changes and then applies heat to prevent those changes.

\n

Here's a graph comparing PID and MPC, created by Reddit user yelleck:

\n

\"Graph

\n

Slightly derogatory answer

\n

PID and its modifications1 is the duct-tape solution to the hotend temperature regulation problem2, and MPC is the engineering-PhD solution.

\n

Slightly longer answer

\n

MPC is a predictive model that compensates before the temperature actually changes. It calculates a physical heat model (effectively a Thermal Resistance Circuit). Using this, it figures out how much heat energy (in Joules) will leave the hotend in a given time\u2014through radiation, convection, and extruded molten plastic\u2014and thus it knows exactly how much heat energy (again in Joules, or "Watts times Seconds") it has to put in to keep the temperature the same or move to a new temperature. The properties of this physical model (heat capacity, emissivity, etc.) can be obtained automatically with fairly good accuracy or looked up in a table (filament heat capacity per mm).

\n

PID on the other hand is a reactive model that can only work after the temperature has already changed. It is more like a vigilant operator who stares only at the measured difference between the actual and desired temperature. If this temperature difference changes by just a tiny bit, or by a large step, it will immediately try to counteract that error by applying its three rules: P, I, and D.

\n

The three PID coefficients can also be auto-tuned, but the results are... well, let's say there is a reason that there are dozens if not hundreds of PID tuning guides and forum questions out there. The coefficients are not linked to any distinct physical property of your hotend: The geometry, fan, heater power, silicone sock, and filament are all mushed together into three abstract numbers. So far I have tried a handful of times to get a deep, intuitive understanding of PID behavior and tuning, but have failed each time, and honestly, I don't consider it worth my time anymore. MPC just makes so much more sense.

\n

The PID controller does not know or care why the temperature difference changed. It could be due to faster extrusion speed, switching on the fan, or a changed temperature setpoint (M104), for example. A PID controller has to react to any changes using the same rigid 3-rules-system1, although the three situations will in practice require different reactions. The MPC model, on the other hand, can figure out precisely how much each of those changes would influence the temperature and how it must react to prevent that.

\n

So in a PID-based system, the temperature will always fluctuate a little by design. It needs those temperature changes in order to know how to react (unless you assume a completely static situation), because it is the only measurement it has. Consequently, a noisy temperature measurement, which looks like a lot of tiny, very fast changes, will force the PID controller to react to them, which leads to even more noticeable fluctuations because the system reacts to a change that isn't really there, since it's just noise. Alternatively, you could make it react slower to ignore these fast noise fluctuations, but that will make it... well... slower.

\n
\n

And why is it more useful for 'high-power heaters'?

\n
\n

As explained above, the PID controller must react to noise. With a high-power heater, even a tiny adjustment in the PWM can lead to a large increase in temperature. So the controller must react more slowly and carefully so as not to overshoot or oscillate. But by going more slowly, you are throwing away the main benefit of your high-power heater, namely: faster heating. Since MPC largely ignores noise, it won't oscillate. And since it knows your heater power in Watts and the exact amount of energy needed in Joules, it can just go "full throttle" when a higher temperature is needed3, stopping just in time to avoid overshooting. With a lower-powered heater, this problem is not so apparent, because it can only go slowly anyways.

\n
\n

1 Yes, Marlin has PID_FUNCTIONAL_RANGE, PID_EXTRUSION_SCALING, and PID_FAN_SCALING to work around these issues. Those are hacky, ad-hoc solutions that try to do the same thing as MPC, but without a correct and complete physical model underneath. It's rather like adding more duct tape to the problem instead of stepping back, understanding the situation, and engineering a proper solution. Also, each of these functions has its own configuration parameters, which require separate tuning, while the auto-tuning in MPC can take care of all of them together (except filament heat capacity, which you can look up or calculate using physics instead). So if you consider using those additional PID features, you are essentially using MPC, but without a solid theoretical foundation, so you could as well go all the way and just use MPC instead.

\n

2 Don't get me wrong: PID control is a flexible and well-established tool that is for many, many different tasks everywhere. Just like duct tape. But it is not the best tool for every job. A well-tuned PID can control the hotend temperature within a few degrees with little overshoot. It just wasn't specifically designed for this case, and thus it is outperformed by tools that were.

\n

3 Marlin PID tries to do the same thing with PID_FUNCTIONAL_RANGE: it goes full power when the temperature difference is greater than this configured value. But with a high-power heater, this is an additional value that you have to tune. And after the PID kicks in, it needs some time to "wind up", which is another time during which the heater is not using its full potential. Also, the PID coefficients have to be tuned for different, partially conflicting goals: heating up quickly, avoiding overshoots, and avoiding oscillations. Again, all are lumped together into three abstract numbers with no physical meaning.

\n" }, { "Id": "21683", "CreationDate": "2023-11-27T19:22:34.770", "Body": "

I'm using Cura Thumbnails using a plugin in Cura 5.5. Also, I use the Marlin-based mriscoc firmware on a Creality Ender 3 V2 with OctoPrint using OctoPi on a Raspberry Pi 4.

\n

The firmware supposedly supports Thumbnail's preview when printed from an SD card, although I never tried. I can also view the Thumbnail in the OctoPrint UI. I print only using USB, controlled by OctoPrint on my Raspberry Pi.

\n

Is it possible to send the Thumbnail to the printer LCD when a print is started from OctoPrint? In a similar way to how I have to add 'Show Progress on printer LCD' using a Cura plugin to follow the progress on the printer LCD when OctoPrint controls it.

\n", "Title": "Is there a way to send a thumbnail to the printer LCD screen when printing over USB from OctoPrint?", "Tags": "|marlin|octoprint|usb|mriscoc|octopi|", "Answer": "

No, it is currently not possible to see the thumbnail in the printer screen if the G-code is being printed from a host like Octoprint. It is technically possible if the same G-code exists on the SD card or if the user can wait about 5 to 10 minutes for the binary jpeg image to be transferred to the SD card using the serial protocol. But I don't think it's worth the effort.

\n" }, { "Id": "21686", "CreationDate": "2023-11-27T23:30:23.550", "Body": "

I've got a line with a point constrained to it. I would like to split my line at the point. The "Split Edge" tool will allow me to split the line anywhere but at that point. How can I split at line at an existing point?

\n

\"enter

\n", "Title": "How can I split at line at an already existing point?", "Tags": "|freecad|", "Answer": "

I figured it out. Splitting at that specific point makes no sense because the split wouldn't be constrained to it at all. It would just be confusing.

\n

The way to do this is to split somewhere else on the line, then constrain the split point to the existing point.

\n" }, { "Id": "21691", "CreationDate": "2023-11-28T11:01:08.587", "Body": "

Why do most slicers slice, and printers print in 2.5D, a planar method, rather than an actual 3D non-planar method? Doesn't this mainly come down to slicer software limitations rather than hardware limitations?

\n

The results, in many cases, seem so much better. As shown below:

\n

\"planar\n\"planar

\n", "Title": "Why is 'planar-2.5D-slicing' most commonly used instead of 'non-planar-3D-slicing'?", "Tags": "|slicing|fdm|planar|non-planar|2.5d|", "Answer": "

Degrees of freedom

\n

2D has 3 degrees of freedom: X, Y, and a single rotation. As a result, you can describe any object in 2D by defining its center position and the rotation its surface start point is at and then referencing its shape.

\n

3D has 6 degrees of freedom: X, Y, and Z position, and the rotation around each of the axes. Describing a 3D object is much more complicated.

\n

Printers usually work with fully constrained rotations after the slicing. Traditional machining has limited Z accessibility without a 4th axis or using a 5-axis arm, and is limited in depth by the length of tooling.

\n

Slicing is a number of mathematical 2D processes

\n

When slicing, the slicer creates slices of a layer thickness. These then are interpreted by projecting to the center the smallest outline that would fit everywhere in the body and raising that outline to the center of the slice.

\n

The stack of outlines is pretty much the outer surface reduced to a number of 2D objects. As placing the object already fixed its rotation, this leaves each outline to be solved for 2 variables for the toolpath: X and Y.

\n

Just stacking all these results in a 3D object in the end.

\n

2.5D is a machining term

\n

Traditional machining creates 3D objects. However, machining with a 2.5D setup does not create undercuts, as it only uses cylindrical tooling in most cases. This machining works by projecting the top item onto a plane and having at most one Z coordinate. That means one can easily create a "height map" of the item's surface.

\n

The surfaces in nonplanar mapping are similar: the top examples have a lot of traditional 2D solved printing and a single top layer that follows a 2.5D machine path - which is, remind you, a simplified 3D coordinate system.

\n

By only solving the last layer for X, Y, and Z path coordinates and banning undercuts, the solving time can be massively reduced, leading to faster slicing and allowing more time spent on refining the print paths. 2.5 D path solution algorithms also are much better explored from traditional machining, while true 3D machining requires the machining tool to move with all degrees of freedom.

\n

Layered 2.5 D

\n

Example (a) below breaks from traditional 2.5 D but can be reduced back to 2.5 D. Cutting the item in non-planar layers that follow the bottom geometry, that geometry can be solved as a 2.5 D machine path, and then the next layer is solved with an offset above that. However, cutting an item non-planarly can become very complicated, especially with more complex geometry than a simple arch.

\n" }, { "Id": "21698", "CreationDate": "2023-11-29T11:46:07.370", "Body": "

Just as the title says - I'm in the need of a new heater block, and need to decide which one to buy. Plated copper is 10 times more expensive now, at least where I live (70 PLN which is about 16 Euro or 18 USD vs 7 PLN for aluminum), but in the scale of the cost of a printer, difference is negligible.

\n

Copper has better thermal transfer, but it also has a greater thermal mass, so it heats up and cools down slower, but should be able to melt plastic faster. Or so it seems. I don't know how these theoretical properties translate into practical pros and cons for 3D printing.

\n

I'm not looking for "buy this" answer. I'm looking for comparison that will let me, and future readers, make informed decision.

\n", "Title": "What are the differences between aluminum and copper heater block and why would I use one vs another?", "Tags": "|heat-management|replacement-parts|", "Answer": "

I\u2019ll add from my experience: for printing with composite materials, when using a tungsten carbide nozzle, if screwed into an aluminum hotblock (even when screwed hot), the nozzle quickly unwinds and then plastic leaks. In the case of a copper hotblock, a similar effect is not observed (apparently, this is explained by different thermal expansion of the materials).

\n" }, { "Id": "21703", "CreationDate": "2023-11-29T21:19:31.243", "Body": "

I'm trying to modify this code to create a small 2\u00a0mm x 2\u00a0mm hole on the short side of the box (to fit 2 wires coming out of a battery):

\n
// Which one would you like to see?\npart = "both"; // [box:Box only, top: Top cover only, both: Box and top cover]\n\n// Size of your printer's nozzle in mm\nnozzle_size = 0.35;\n\n// Number of walls the print should have\nnumber_of_walls = 3; // [1:5]\n\n// Tolerance (use 0.2 for FDM)\ntolerance = 0.2; // [0.1:0.1:0.4]\n\n// Interior dimension X in mm\ninterior_x=70;\n\n// Interior dimension Y in mm\ninterior_y=21;\n\n// Interior dimension Z in mm\ninterior_z=17;\n\n// interior corner radius in mm\nradius=0; // [0:20]\n\n// What fraction of the flat X side should the hook take up? (0 for no hook)\nx_hook_fraction = 0.5; // [0:0.1:1.0]\n\n// What fraction of the flat Y side should the hook take up? (0 for no hook)\ny_hook_fraction = 0.5; // [0:0.1:1.0]\n\n// What fraction of the hooks should have a slot behind them? (0 for no slot)\nslot_length = 0.7; // [0:0.1:1]\n\n/* Hidden */\n$fn=100;\n\nwall_thickness=nozzle_size*number_of_walls;\n\n// Outer dimensions\nx = interior_x + 2 * wall_thickness;\ny = interior_y + 2 * wall_thickness;\nz = interior_z + 2 * wall_thickness;\n\nhook_thickness = 3 * nozzle_size;\n\ntop_cover_wall_thickness = hook_thickness + wall_thickness;\n\ny_hook_length = (y - 2 * radius) * y_hook_fraction;\nx_hook_length = (x - 2 * radius) * x_hook_fraction;\n\nmodule box_interior () {\n    offset(r=radius) {\n        square([interior_x-2*radius, interior_y-2*radius], center=true);\n    }\n}\n\nmodule box_exterior () {\n    offset(r=wall_thickness) {\n        box_interior();\n    }\n}\n\nmodule bottom_box () {\n    difference(){\n        // Solid box\n        linear_extrude(z-wall_thickness){\n            box_exterior();\n        }\n        \n        // Hollow out\n        translate([0,0,wall_thickness]) linear_extrude(z){\n            box_interior();\n        }\n        left_slot();\n        rotate([180,180,0]) left_slot(); // right slot\n        front_slot();\n        rotate([180,180,0]) front_slot(); // back slot\n    }\n    left_hook(); // left hook\n    rotate([180,180,0]) left_hook(); // right hook\n    front_hook(); // front hook\n    rotate([180,180,0]) front_hook(); // back hook\n}\n\nmodule left_hook () {\n    translate([(x-2*wall_thickness)/2,-y_hook_length/2,z-wall_thickness]) rotate([0,90,90]) {\n        base_hook(y_hook_length);\n    }\n}\n\nmodule front_hook () {\n    translate([-x_hook_length/2,-y/2+wall_thickness,z-wall_thickness]) rotate([90,90,90]) {\n        base_hook(x_hook_length);\n    }\n}\n\nmodule base_hook (hook_length) {\n    difference(){\n        linear_extrude(hook_length){\n            polygon(points=[[0,0],[2*hook_thickness,0],[hook_thickness,hook_thickness]]);\n        }\n        translate([hook_thickness, hook_thickness, 0]) rotate([45,0,0]) cube(2*hook_thickness, center=true);\n        translate([hook_thickness, hook_thickness, hook_length]) rotate([45,0,0]) cube(2*hook_thickness, center=true);        \n    }\n}\n\nmodule left_slot () {\n    slot_length = y_hook_length*slot_length;\n    epsilon=2; // ensure it definitely protrudes\n    translate([x/2+epsilon,-slot_length/2,z]) rotate([0,90,90]) {\n        cube([2*hook_thickness, wall_thickness+epsilon, slot_length]);\n    }\n}\n\nmodule front_slot () {\n    slot_length = x_hook_length*slot_length;\n    epsilon=2; // ensure it definitely protrudes\n    translate([-slot_length/2,-y/2-epsilon,z]) rotate([90,90,90]) {\n        cube([2*hook_thickness, wall_thickness+epsilon, slot_length]);\n    }\n}\n\nmodule right_groove () {\n    translate([-tolerance/2+(x-2*wall_thickness)/2,-y_hook_length/2,wall_thickness+hook_thickness*2]) rotate([0,90,90]) linear_extrude(y_hook_length) {\n        base_groove();\n    }\n}\n\n\nmodule front_groove () {\n    translate([-x_hook_length/2,-y/2+wall_thickness+tolerance/2,wall_thickness+hook_thickness*2]) rotate([90,90,90]) linear_extrude(x_hook_length){\n        base_groove();\n    }\n}\n\nmodule base_groove () {\n    polygon(points=[[0,0],[0, -1], [2*hook_thickness, -1],[2*hook_thickness,0],[hook_thickness,hook_thickness]]);\n}\n\nmodule top_cover () {\n\n    // Top face\n    linear_extrude(wall_thickness) {\n        box_exterior();\n    }\n    \n    difference(){\n        // Wall of top cover\n        inset = wall_thickness + tolerance/2;\n        linear_extrude(wall_thickness+hook_thickness*2){\n            offset(r=-inset) {\n                box_exterior();\n            }\n        }\n        \n        // Hollow out\n        translate([0,0,wall_thickness]) linear_extrude(z){\n            offset(r=-wall_thickness*2) {\n                offset(r=-inset) {\n                    box_exterior();\n                }\n            }\n        }\n        \n        right_groove();\n        rotate([180,180,0]) right_groove();\n        front_groove();\n        rotate([180,180,0])  front_groove();\n    }\n  \n\n}\n\n// left_hook();\nprint_part();\n\nmodule print_part() {\n    if (part == "box") {\n        bottom_box();\n    } else if (part == "top") {\n        top_cover();\n    } else if (part == "both") {\n        both();\n    } else {\n        both();\n    }\n}\n\nmodule both() {\n    translate([0,-(y/2+wall_thickness),0]) bottom_box();\n    translate([0,+(y/2+wall_thickness),0]) top_cover();\n}\n
\n

I've tried something like this but it doesn't work:

\n
// Adding a hole to the short side of the box\ntranslate([interior_x / 2, interior_y - hole_size / 2, interior_z / 2])\n    cube([hole_size, hole_size, interior_z], center=true);\n
\n", "Title": "How to add a 2x2 mm hole to a SCAD file?", "Tags": "|3d-models|3d-design|openscad|", "Answer": "

You can use the difference() function in OpenSCAD to subtract one or more 3D shapes from another. Source: The documentation for the difference() function.

\n

For reference, your code already does that several times. Something like this:

\n
module bottom_box () {\n    difference(){\n        // Existing code ...\n\n        // Subtract the hole\n        translate([hole_position_x, hole_position_y, hole_position_z])\n            cube([hole_size, hole_depth, hole_size], center=true);\n    }\n    // Rest of the code ...\n}\n
\n

Two cube examples

\n

Below are two examples of a cube with and without a hole (in the bottom) that, for testing, can be rendered in your browser on openscad.cloud:

\n

1. Model without a hole

\n
cube_size = 20;\nmodule simple_cube() {\n    cube(cube_size);\n}\nsimple_cube();\n\n
\n

\"model

\n

Example of a model without a hole on openscad. cloud.

\n

2. Model with a hole

\n
cube_size = 20;\nhole_size = 2;\nmodule cube_with_hole() {\n    difference() {\n        cube(cube_size);\n        translate([cube_size / 2, cube_size / 2, 0])\n        cube([hole_size, hole_size, cube_size], center=true);\n    }\n}\ncube_with_hole();\n
\n

\"model

\n

Example of a model with a hole in the bottom on openscad. cloud.

\n" }, { "Id": "21705", "CreationDate": "2023-11-30T09:30:55.337", "Body": "

I use the Cura Thumbnails plugin to create and embed thumbnails in my G-code to view them using OctoPrint or my printer LCD. However, I recently found much higher quality Thumbnails that were generated using fast approximate anti-aliasing (FXAA). I was wondering if there is a way to enable or use another tool (slicer) to embed higher-quality thumbnails that were generated using anti-aliasing methods such as FXAA. For example, as generated by the tool unlimitedbacon/stl-thumb as shown below:

\n

An example of thumbnails generated with that tool:

\n

\"example

\n

An example of a thumbnail generated without and with FXAA:

\n

\"example

\n", "Title": "Are there any slicers that use anti-aliasing (such as FXAA) for higher quality thumbnail generation?", "Tags": "|slicing|thumbnail|", "Answer": "

As of May 2023, Cura is working on it as described in this Github issue Ultimaker/Cura/issues/7447. One user suggests the use of the spatial anti-aliasing method supersampling anti-aliasing (SSAA) instead.

\n" }, { "Id": "21708", "CreationDate": "2023-11-30T12:56:20.580", "Body": "

The Creality Ender 3 V3 SE, apart from auto bed leveling (ABL), also has a way to calculate the Z-offset automatically. A feature I didn't hear about before.

\n

According to a Maker's Muse video, it's based on a force sensor as shown below.

\n

\"'Force

\n

How does this concept work? How can a force sensor automatically determine the Z-offset?

\n", "Title": "How can a force sensor automatically determine the Z-offset?", "Tags": "|creality-ender-3|automatic-bed-leveling|z-offset|sensors|", "Answer": "

The Z-offset is the distance between the nozzle and the triggerpoint, see e.g. this answer of question Automatic Bed Leveling (ABL) with a sensor (BLTouch, inductive, capacitive), how does it work?.

\n

A "force sensor" as called in the question is actually a strain guage; deflection will cause a different electrical resistance which is a measure for the applied stress. These sensors are frequently found in aircraft to measure the strains (as a measure for the stresses) in the structure, e.g. wing spars, (pressure) bulkheads or frames, as part of structural health monitoring (e.g. to compare to the design spectrum of the OEM).

\n

In theory the Z-offset with a strain gauge is non existent if you use such a sensor for ABL except that you may need to corect for possible flexing of the bed or the hot end assembly and a paper offset (default distance to allow better flow and adhesion).

\n

However, the system referenced in the question is an Ender 3 V3 SE with an ABL system and a strain gauge.

\n
\n

Relax, Let Auto Leveling Do It
\nLeveling is the basis of print quality.\nEnder-3 V3 SE features a CR Touch sensor for auto leveling\nand a strain sensor for auto Z offset.\nNo need to turn screws or pull a paper.\nJust lie back and enjoy the print success.

\n
\n

This implies that the strain gauge is not used to determine the level of the bed, instead the surface of the build plate is scanned/mapped by the CR Touch (BLTouch clone) sensor. In order to map this scanned surface to the correct height, a test point with the nozzle hitting the buildplate (and as such causing the strain gauge to measure that the nozzle touches the surface) is used to set the Z-offset based on this measurement. Without such a strain gauge, the printer user needs to do that manually; this is the process where you set the offset using the M851 G-code command. Instead of the user lowering the nozzle until a piece of paper can be dragged with a little resistance to determine the "true zero position" of the Z-axis, the triggering of the strain gauge and the automatic handling by the Creality software will do that for you.

\n

To explain the automatic calculation of the Z-offset in this printer is that the Creality Ender 3 V3 SE first calibrates the Z-offset with the help of a pressure of strain-gauge sensor mounted under the front left corner of the print bed, there is a strain-gauge sensor which can detects deformation on the structure that is mounted on.

\n

\"enter

\n

Knowing the exact position of the nozzle and the position (height) of the trigger point of the leveling sensor, the Z-offset can be calculated and set. That height is then put into the M851 G-code command to set the Z-offset. This is exactly how you would do this manually other than you usually use a sheet of paper or a feeler gauge between the nozzle and the bed. The automatic calculation will surely make a correction for that (e.g. there could be play in the nozzle assembly).

\n" }, { "Id": "21712", "CreationDate": "2023-12-01T03:34:02.990", "Body": "

I'm looking for an SLA resin that's electrically conductive. But, there don't seem to be any DIY or cheap formulations for electrically conductive resin.

\n

If it were easy to do, we'd all be making circuits with electrically conductive resin. So I think that level of conductivity is probably not going to be available for a while. And I guess beyond the scope of this question. Likewise, a resin that was conductive enough for electroplating is also beyond the scope of this question.

\n

A guy on YouTube mixed 5% graphite with resin and got 1.4\u00a0M\u03a9 on his multimeter when the electrodes were placed about 2\u00a0cm apart. This is useful for electrostatic discharge but not much else. I'd like something more conductive than that.

\n

I would like something similar to ProtoPasta conductive PLA, which has about 400-2000\u00a0\u03a9 of resistance when you measure it 2\u00a0cm apart on a multimeter. It's not that good, but it's enough for a touch screen or button, for example. Someone on Reddit mentioned that people had formulated this before, but I can't find any evidence of that. At least, not with SLA resin - just that FDM stuff.

\n

I even tried my hand at this a few months ago. I mixed some kind of generic resin with chopped carbon fibers, but it was kind of weird. The chopped fibers stuck out the edges of the final print, and were never really spread around that well. It was conductive but highly ansiotropic. It also looked horrible. The fibers clumped together in the uncured (and cured) resin, and it was not possible to keep them well-dispersed in the mixture throughout the print.

\n

Does anyone have a good 1) cheap supplier or 2) cheap & simple DIY formulation for creating this kind of resin?

\n", "Title": "Are there any electrically-conductive SLA resins?", "Tags": "|resin|sla|", "Answer": "

You could just use conductive paint, it's under $20 usually for the basics.

\n

Bareconductive is the only one I've used, worked well enough for my simple project. Other products might be better.

\n" }, { "Id": "21714", "CreationDate": "2023-12-01T10:34:34.347", "Body": "

In addition to the question How can a force sensor automatically determine the Z-offset?.

\n

Is there any well-documented 'auto z-offset' (DIY) upgrade available for 3D printers? I'm particularly interested in using a strain gauge and upgrading the Creality Ender 3 V2. But examples for adding this capability, using other methods or to other printers are also welcome because I can modify it.

\n", "Title": "Is there any well-documented 'auto z-offset' (DIY) upgrade available for 3D printers?", "Tags": "|diy-3d-printer|automation|z-offset|", "Answer": "

It's not a common thing because it's usually something you do once and then it works for years without need to touch it, and I don't know of generic upgrades that provide just that in conjunction with whichever probe you have.

\n

Bambu X-1 Carbon does it automatically with a piezzo sensor for a rough estimate and then using its lidar to analyze baby-stepped lines on the build plate to find the perfect Z height. This is not something to implement in any printer though.

\n

The Prusa Mk4 approach is much more viable: it uses a load cell sensor in the extruder to find the moment the nozzle presses into the bed. I'm pretty sure this can be done with a strain gauge just as well, thing is they don't produce a neat binary on/off, just analog signal in form of varying resistance. You'd likely need a small microcontroller with an ADC input to read the gauge and convert that to binary On/Off on the Z axis input. And you'd still need to add (small, fixed) Z offset to get the height of the nozzle touching the bed as opposed to pressing into it.

\n

I'm not aware of any ready-made solutions that do this, but it should be a project well within skill range of a hobbyist.

\n" }, { "Id": "21726", "CreationDate": "2023-12-02T13:56:38.753", "Body": "

I have two Creality Ender 3 V2 printers with a version 4.2.2 board. One printer has the official CRTouch, and one is nearly original without a probe. I found a similar product, a DIY project called BFPTouch, and I have almost all the parts needed to make it.

\n

\"BFPTouch\"

\n

Model and photo source: Thingiverse.com

\n

I use the Marlin-based mriscoc firmware on both printers. Version Ender3V2-422-MM-MPC-20231202.bin, and on my printer with the CRTouch, I use the Ender3V2-422-BLTUBL-MPC-20231202.bin version.

\n

Is the BFPTouch interchangeable with the CRTouch, like the BLTouch and CRTouch are? Is the wiring to the board the same, and would it suffice to use the BLTUBL firmware version of mriscoc that I've mentioned without further altering Marlin?

\n", "Title": "Is the BFPTouch interchangable with the BLTouch for mriscoc firmware without custom changes to Marlin?", "Tags": "|creality-ender-3|marlin|bltouch|mriscoc|crtouch|", "Answer": "

No, it is not interchangeable without changing the configuration of the firmware. These sensors do not require you to set the BLTOUCH directive, so this must be in your configuration:

\n
//#define BLTOUCH \n
\n

For the CR Touch this directive is required.

\n
#define BLTOUCH\n
\n

I am building the Tiny Touch an even smaller version of the BFPTouch, in fact it is a derivative! In the last link you find instructions to configure the probe, which redirects to the setup of the BFPTOUCH.

\n" }, { "Id": "21729", "CreationDate": "2023-12-02T18:50:09.257", "Body": "

I thought I had two identical Ender 3 V2 printers, the main board in both is v4.2.2 but it turns out they came with a different LCD screen.

\n

I flashed the firmware on both printers to mriscoc firmware and tried to also flash both LCD screens.

\n

DWIN

\n

The printer with the DWIN screen worked fine, everything went smooth and instead of the original blue, flat GUI, I now look at a colorful GUI something like this:

\n

\"mriscoc

\n

DACAI

\n

On the other printer that I identified as a DACAI LCD as per mriscoc instructions documented.

\n

\"DACAI

\n

I then flashed the firmware and the private folder separately, as shown below, in the end you can see it fails to load. Not shown on the video but if I rotate the knob, the buttons appear on top of the load screen:

\n

\"failed

\n

After rebooting I see this:

\n

\"failed

\n

I am able to view a test thumbnail on screen, as provided in the documentation and shown below:

\n

\"thumbnail

\n

But I expect the same colors and icons as on my DWIN screen, did I miss something?

\n

I made sure the right cluster size was used when formatting, I tried three different SD cards and two different SD card readers, as well as unmounting from PC before removing it.

\n

How to successfully flash a DACAI LCD screen of an Ender 3 V2 to achieve the same as on the DWIN LCD?

\n", "Title": "How to successfully flash a DACAI LCD screen of a Creality Ender 3 V2?", "Tags": "|creality-ender-3|firmware|lcd-screen|mriscoc|", "Answer": "

The process as shown in the question was all performed correctly and successfully however, simply the wrong icon set was used. The documentation isn't very clear and I should have personally documented this better when I did it months ago.

\n

For flashing the LCD screen, the SD card will be placed in the SD card slot on the back of the LCD screen, not the printer slot.

\n

This is what fixed it:

\n\n

Also, it's required to have micro SD card of 8GB or smaller formatted explicitly as MBR, FAT32, and with a 4 KB sector size.

\n

For the DACAI screen specifically, I also flashed another DACAI screen firmware, also available in mrisoc Git repo: dacai_update.zip. Place the firmware.zlib file in the root of the SD card and flash.

\n

Repower the printer after flashing both firmware and private folder. To test if the firmware and screen work correctly you can place this test gcode on the printer SD card to check if the thumbnail preview on the printer LCD is shown.

\n

Result shown below:

\n

\"mriscoc

\n" }, { "Id": "21737", "CreationDate": "2023-12-03T16:50:46.180", "Body": "

I'm building this simple DIY filament runout sensor for Ender 3 V2 based on this model from Thingiverse.com. I am not sure about the wiring, it seems trivial, I probably need to use 'normally closed' (NC). Below is the result so far.

\n

\"DIY

\n

\"DIY

\n

I found this schematic from a YouTube video which uses a different sensor:

\n

\"schematic\"

\n

Based on the switch I used and the schematics shown, I guess I need to connect only two wires as follows:

\n\n

Is that wiring correct?

\n

Lastly, I wonder how the connector is called that is used on the Creality v4.2.2 board, and if I need to use a transistor?

\n", "Title": "How to wire a DIY filament runout sensor to a Creality v4.2.2 board?", "Tags": "|creality-ender-3|wiring|filament-sensor|sensors|runout-sensor|", "Answer": "

That\u2019s a cool print for your filament runout sensor. I\u2019m planning on doing this same mod to my Ender 3 V2 in the next week or two as well.

\n

I\u2019ve done a decent amount of research on how to implement this so hopefully I can answer some of your questions.

\n

Wiring

\n

I found this schematic of the V4.2.2 motherboard online.

\n

\"Schematic

\n

Here is the pdf download link to the Ender 3 V2 v4.2.2 schematic (clicking on the link immediately downloads the pdf).

\n

It looks like for our motherboard (if you also have the V4.2.2), the pinout is:

\n\n

\"Upclose

\n

Based on the schematic, the Sensor Signal pin is pulled HIGH. This means to trigger the sensor, you want to connect the Sensor Signal pin to Ground.

\n

The wiring for the switch is:

\n\n

The 'NO' pin on the switch is left unconnected.

\n

When the filament runs out, the switch opens and the \u2018NC\u2019 pin is connected to the \u2018C\u2019 pin. This pulls the Sensor Signal pin on the motherboard to LOW, which is what triggers the printer to pause.

\n

Here is a diagram of how the switch should be wired to the motherboard.

\n

\"Photo

\n

Firmware

\n

You\u2019ll also need to update the firmware on the motherboard. This isn\u2019t too hard if you\u2019ve never done it before.

\n

I\u2019d recommend installing the JyersUI firmware as detailed in the video:\n

\n \n

\n

Just follow along with the guide to install it. The Jyers firmware also has some really nice UI improvements. Check the video description for links to the firmware.

\n

If you don\u2019t want to go that route, or you if have the wrong kind of motherboard chip (see video), you can also download the official creality firmware to enable the filament runout sensor.

\n

The link to the firmware list is here: Official Ender 3 V2 firmwares

\n

I\u2019m pretty sure the one you\u2019ll want is:

\n

Marlin-2.0.1-HW-4.2.2-mainboard-V1.1.2-Compatible with BLTouch and filament detection

\n

I didn\u2019t use the official firmware, and there aren\u2019t any release notes (of course), so I\u2019m only 90% sure that's the right one .

\n

At any rate, it might be wise to check your current firmware and make sure you download that too. That way you can revert your printer to its original firmware, just in case something goes wrong.

\n

JWST connectors

\n

Finally, the connectors on this board are the very common JST connectors.\nHere is a link to a relatively inexpensive JST connector Kit and crimping tool:\nConnector kit w/ crimp tool

\n

It\u2019s pretty easy to crimp and install the connectors yourself. Here is a quick video on how to do it:

\n \n

\n

If you\u2019re good at soldering you could also just hard solder the switch to the board if you don\u2019t want to buy the connectors and crimper. Although I\u2019d recommend against it.

\n

Hopefully that helps. Good luck :)

\n

P.S. I'll post how mine turns out once I'm finished.

\n" }, { "Id": "21738", "CreationDate": "2023-12-03T17:21:58.550", "Body": "

Is there a method such as fetching data from to stepper motor drivers, that can automatically detect or count stepper motor skips or stepper motor stalling?

\n

Preferably for a Creality Ender 3 V2 board v4.2.2, but any related information is welcome. If something like this exists, how does it work and what are common use cases or implementation of it?

\n

The reason I am asking is because it is a performance indicator that could help to track or measure print consistency and quality.

\n", "Title": "Can stepper motor skipping or stalling be detected automatically?", "Tags": "|stepper-driver|stepper|sensors|", "Answer": "

You seem to be looking for closed-loop stepper motors.

\n

Detection of stalling and skipping on open-loop (aka regular) steppers is possible - that's how Prusa mk3s+ finds home position without endstops*. But it is not reliable and precise - that's why Prusa firmware tests every axis a couple of times.

\n

If quality matters, and you want to reliably detect stalls and skips during print, closed-loop motors are what you need. Per one of the manufacturers description, they signal back to the controller what they are doing, so that controller can correct for skips and stalls:

\n
\n

On the contrary, NEMA17 closed loop stepper motor feedback signal, according to the signal stepper motor driver to take the corresponding action measures or signal feedback to PLC, by PLC to determine the next step of the program.

\n
\n

Open-loop steppers are much cheaper, so most manufacturers just massively over-spec their open-loop steppers to make them force their way thorough any obstacles like blobs of plastic, and not need to care about feedback signal, and pay for it. As far as I know, closed-loop steppers are not used in hobby or even entry business machines, cost of motors and controllers being prohibitive.

\n
\n

* When the motor stalls, power draw changes. If your driver can monitor power draw, firmware can make educated guess about the stall.

\n" }, { "Id": "21748", "CreationDate": "2023-12-04T22:50:22.197", "Body": "

I see mixed terms such as 'rPLA, Recycled PLA, Recycled rPLA' used by different filament producers without a clear explanation or a standard definition.

\n

Does it all mean Recycled PLA, or is rPLA a different material?

\n", "Title": "Does rPLA stand for Recycled PLA, or is it a different type of material?", "Tags": "|pla|filament|recycling|terminology|rpla|", "Answer": "

All brands and shops that I know of use rPLA name for recycled PLA. However, this is neither a chemical name abbreviation, like PLA, nor a naming standard that's enforced by any agency. Thus, every time you want to buy rPLA, you must check if it really means recycled PLA. Because it can mean something else and manufacturer won't break any laws by doing it.

\n

Hopefully the name will solidify soon, and anyone who sells anything but recycled PLA as rPLA will risk getting sued for false advertising. But we'll have to wait for that.

\n

Be aware that "up to 100% recycled material" means any non-zero number equal or lower than 100%. So 1% of recycled materials meets this declaration as well as 90%. Only when manufacturer gives straight number, or "at least" number, we can be sure that amount of recycled material is what we expect it to.

\n" }, { "Id": "21752", "CreationDate": "2023-12-05T09:08:56.660", "Body": "

In addition to the question Does rPLA stand for Recycled PLA, or is it a different type of material?.

\n

Does PLA lose mechanical properties when it is being recycled into rPLA? If so, how often can it be recycled before being unusable for recycling? And how would filament manufacturers know or account for this? How would they know if I returned the new PLA or rPLA waste? What if, in theory, the same rPLA batch is sent back repeatedly for recycling?

\n", "Title": "Does PLA lose mechanical properties when recycled, and if so, how often can it be recycled?", "Tags": "|pla|material|recycling|filament-production|rpla|", "Answer": "

Yes, recycled PLA loses mechanical properties.

\n

According to Dr. D-Flo's excellent video on Recycling 3D Prints and Waste plastic into Filament (PET & PLA) it loses mechanical strength during every thermal cycle - i.e. when it is printed, when it is melted down for recycling, when it is next printed, etc.

\n

Manufacturers account for this by mixing in "virgin" PLA. So "recycled PLA" is really a mix of recycled and new PLA. This will lessen the extent to which the recycled portion will affect overall strength of the result.

\n

I do not know if/how manufacturers would account for how many thermal cycles the PLA you supply has been through.

\n

If the mix of virgin/recycled PLA has more virgin than recycled, then it may not matter. If the mix favoured recycled then each batch would be worse than the one before. I suppose manufacturers can test a sample from the beginning of their re-extrusion, and if they find it too brittle (etc.) they can change the mix before continuing.

\n" }, { "Id": "21755", "CreationDate": "2023-12-05T23:30:33.407", "Body": "

I found two great designs for a DIY sensor that achieve both filament runout and filament jam detection in one unit.

\n

A clever design that only uses one microswitch as follows:

\n

\"Filament

\n

Source: Thingiverse.com

\n

Another design using two microswitches, as follows:

\n

\"Filament

\n

Source: Thingiverse.com

\n

I am not sure about the reliability of either yet, and I initially liked the design with one more switch. Then I realized there was no way to distinguish what event occurred (runout or jam), so I plan to make the second two microswitch versions instead. However, how would I wire this in a way so that my Ender 3 V2 running Marlin, specifically mriscoc firmware, would be able to interpret both signals separately and interpret them as two distinguishable events so that the exact event is reported (runout or jam)?

\n

Alternatively, I could wire it to my Raspberry Pi, since I always print from OctoPrint only, and implement it there instead. However, I'd prefer to wire it to the printer board and have the firmware handle it.

\n

Lastly, in comparison to these two specific designs, would an encoder wheel approach add benefits other than runout and jam detection?

\n

Suggestions for better designs or approaches are also welcome.

\n", "Title": "How to distinguish filament runout and filament jam using a microswitch-based filament sensor in Marlin?", "Tags": "|marlin|electronics|filament-jam|sensors|runout-sensor|", "Answer": "

Blue design

\n

The blue design can only detect a lack of filament. It is boolean: Filament or No Filament.

\n

Pink design

\n

The pink design can have multiple states, combining two boolean values.

\n

The right of the sensors is "Filament there".

\n

The left is using a ramp. On pull along the ramp, the sensor is triggered once a certain amount of pull is overcome, either by pulling up, left or right. As a result of passing through the hole in the lever of known diameter. Any tangle or over thickness will act on the lever to the right and trigger the sensor, and so will strong yanking up or left, through those might self reset. As a result, this sensor will trigger a signal that could be called Tangle or No Tangle, or of course No Jam/Jam.

\n

Your interpretation that there is no way to distinguish the results depends on one thing: that you put the sensors through a single loop back, for the effect of an AND gate.

\n
\n\n\n\n\n\n\n\n\n\n\n\n\n\n\n\n\n\n\n\n\n\n\n\n\n\n\n\n\n\n\n
FilamentJamOutput of the Series
Filament inNo JamRun
Filament inJamStop
no FilamentNo JamStop
no FilamentJamStop
\n
\n

The design is set up such, that the filament sensor shows connectivity on being pressed, while the jam sensor is connected in reverse: triggering it opens the loop. While the designer wired the two in parallel, they could also be connected in series for the same effect.

\n

To overcome this design deficiency of only sending a stop message, you need to route the signal to your control board and distinguish the two signals. However, for all but the error message, the practical effect should be the same: the machine shall stop if either of the Stop conditions is triggered.

\n

If you have other sensory equipment on the printer, you might quickly run into problems in that the number of free pins is usually extremely limited, so you might not have enough pins to route the signals separately.

\n

is tangle trigger separate a good idea?

\n

Now, is the additional requirement of pins and wiring a good idea when it comes to error handling? No.

\n

In either detected case - no more filament or tangled filament - you need to stop printing and access the very same area to deal with the lack of feeding filament. Only in large operations, there is any reason to try and detect the two separately for logging purposes.

\n" }, { "Id": "21759", "CreationDate": "2023-12-06T09:01:12.307", "Body": "

I've been using Klipper and Cura for a while now (somewhat new to 3D printing), and I notice whenever I import a new G-code file to Klipper, none of the Slicer settings from Cura display in the G-code files in Klipper. However, when I imported a G-code file from my friend (the PETG one) I noticed it showed up right away:

\n

\"Screenshot

\n

I looked at the difference in the G-code formats, and I noticed that the file sliced with Prusa had the settings in the bottom of the file in the below (truncated) format:

\n
;TYPE:Custom\n; Filament-specific end gcode\n; filament used [mm] = 5342.18\n; filament used [cm3] = 12.85\n; filament used [g] = 16.32\n; filament cost = 0.45\n; total filament used [g] = 16.32\n; total filament cost = 0.45\n; estimated printing time (normal mode) = 2h 49m 42s\n; estimated printing time (silent mode) = 2h 50m 36s\n; prusaslicer_config = begin\n; avoid_crossing_perimeters = 0\n; avoid_crossing_perimeters_max_detour = 0\n; bed_custom_model =\n; bed_custom_texture =\n; bed_shape = 0x0,250x0,250x210,0x210\n; bed_temperature = 90\n; before_layer_gcode = ;BEFORE_LAYER_CHANGE\\nG92 E0.0\\n;[layer_z]\\n\\n\n; between_objects_gcode =\n; bottom_fill_pattern = monotonic\n; bottom_solid_layers = 5\n; bottom_solid_min_thickness = 0.5\n; bridge_acceleration = 1000\n; bridge_angle = 0\n; bridge_fan_speed = 50\n; bridge_flow_ratio = 1\n; bridge_speed = 25\n; cooling = 1\n; cooling_tube_length = 5\n; elefant_foot_compensation = 0.2\n; external_perimeter_extrusion_width = 0.45\n; external_perimeter_speed = 25\n; external_perimeters_first = 0\n; extra_loading_move = -2\n; extra_perimeters = 0\n; extrusion_multiplier = 1\n; extrusion_width = 0.45\n; fan_always_on = 1\n; fan_below_layer_time = 20\n; filament_colour = #FF8000\n; filament_cooling_final_speed = 3.4\n; filament_cooling_initial_speed = 2.2\n; filament_cooling_moves = 4\n; filament_cost = 27.82\n
\n

Whereas the G-code generated by cura has them in the below format:

\n
;End of Gcode\n;SETTING_3 {"global_quality": "[general]\\\\nversion = 4\\\\nname = Filament_Sample-\n;SETTING_3 PET-dev.1\\\\ndefinition = creality_ender3pro\\\\n\\\\n[metadata]\\\\ntype =\n;SETTING_3 quality_changes\\\\nquality_type = standard\\\\nsetting_version = 22\\\\n\\\\\n;SETTING_3 n[values]\\\\nadhesion_type = skirt\\\\nklipper_experimental_enable = Tru\n;SETTING_3 e\\\\nklipper_ui_temp_support_enable = True\\\\nretraction_combing = nosk\n;SETTING_3 in\\\\nsupport_enable = False\\\\nsupport_type = buildplate\\\\n\\\\n", "extr\n;SETTING_3 uder_quality": ["[general]\\\\nversion = 4\\\\nname = Filament_Sample-PET\n;SETTING_3 -dev.1\\\\ndefinition = creality_ender3pro\\\\n\\\\n[metadata]\\\\ntype = qua\n;SETTING_3 lity_changes\\\\nquality_type = standard\\\\nintent_category = default\\\\n\n;SETTING_3 position = 0\\\\nsetting_version = 22\\\\n\\\\n[values]\\\\nbrim_gap = 0.17\\\\\n;SETTING_3 nbrim_line_count = 14\\\\ninfill_enable_travel_optimization = True\\\\nin\n;SETTING_3 fill_sparse_density = 100\\\\ninset_direction = outside_in\\\\nironing_en\n;SETTING_3 abled = True\\\\nironing_flow = 5.0\\\\nironing_inset = 0.2\\\\nironing_mon\n;SETTING_3 otonic = True\\\\nironing_pattern = zigzag\\\\nmaterial_flow = 90.0\\\\nret\n;SETTING_3 raction_amount = 3.0\\\\nretraction_hop = 1.0\\\\nretraction_hop_enabled\n;SETTING_3 = True\\\\nretraction_hop_only_when_collides = True\\\\nroofing_layer_cou\n;SETTING_3 nt = 1\\\\nroofing_line_width = 0.2\\\\nroofing_material_flow = 75.0\\\\nro\n;SETTING_3 ofing_pattern = lines\\\\nskin_monotonic = True\\\\nskin_overlap = 4.0\\\\n\n;SETTING_3 speed_print = 100.0\\\\nsupport_angle = 65.0\\\\nsupport_conical_enabled\n;SETTING_3 = True\\\\nsupport_interface_density = 20\\\\nsupport_skip_some_zags = Tr\n;SETTING_3 ue\\\\ntop_bottom_pattern_0 = lines\\\\ntop_layers = 1\\\\nwall_line_count\n;SETTING_3 = 4\\\\nwall_overhang_angle = 55.0\\\\nwall_overhang_speed_factor = 80.0\\\n;SETTING_3 \\n\\\\n"]}\n
\n

Prusa has them in a more sensible format, whereas Cura stores them in INI format, stored as strings in a JSON object, which is then stored as a G-code comment (which seems ridiculously overcomplicated)

\n

I already have a bash/jq function I use to parse those comments from the Cura sliced G-code and display it to me, but I'm trying to find a way to have it done automatically while the G-code file is getting generated. I have looked at some plugins for Cura, but I didn't see one that applied to this.

\n

I know I could possibly modify the metadata.py (Moonraker) to have it parse for the Cura style G-code comments, or even write a Cura plugin, but it occurred to me that I can't be the first one to have this issue. And I didn't find anything for it upon Googling it.

\n

The question: Does anyone have a simple workaround for this that doesn't require writing plugins or modifying existing Moonraker files?

\n

And a bonus question: Why in the world would they store slicer settings nested in three different formats? Why?!

\n

TL;DR: Cura stores the print config in the G-code comments, but Klipper/Moonraker can't parse them due to the format. Is there a simple way around this? (aside from writing a plugin or modifying Moonraker files)

\n

P.S. I do see that some plugins seem related to this but I don't see one that formats and saves the settings to the G-code file. They either just extract them from the G-code, or require you to execute a script every time (or even launch a Node server..)

\n

Update (Answer... Kinda)

\n

So after looking into the Moonraker python, I can see that there actually is a Cura class that extends BaseSlicer class, and it's what handles the comment parsing:

\n
class Cura(BaseSlicer):\n    def check_identity(self, data: str) -> Optional[Dict[str, str]]:\n        match = re.search(r"Cura_SteamEngine\\s(.*)", data)\n        if match:\n            return {\n                'slicer': "Cura",\n                'slicer_version': match.group(1)\n            }\n        return None\n        \n    # .. Truncated some rows..\n\n    def has_objects(self) -> bool:\n        return self._check_has_objects(self.header_data, r"\\n;MESH:")\n\n    def parse_filament_weight_total(self) -> Optional[float]:\n        return regex_find_float(r";Filament\\sweight\\s=\\s.(%F).", self.header_data)\n\n    def parse_filament_type(self) -> Optional[str]:\n        return regex_find_string(r";Filament\\stype\\s=\\s(%S)", self.header_data)\n\n    def parse_filament_name(self) -> Optional[str]:\n        return regex_find_string(r";Filament\\sname\\s=\\s(%S)", self.header_data)\n\n    # .. Truncated some rows..\n\n    def parse_nozzle_diameter(self) -> Optional[float]:\n        return regex_find_float(r";Nozzle\\sdiameter\\s=\\s(%F)", self.header_data)\n
\n

And if you look at the regex, it's looking for some config names like Filament weight, Filament name, Filament type, etc. But Cura stores it as just name, weight, and type, but puts it under a [metadata] section.

\n

Also, the Cura-style metadata is all just concatenated into one string, which has a line return injected every 80 characters. This means that depending on the values, some of the names of the metadata items get truncated halfway through, and have another ;SETTING_3 injected as the prefix, which makes it somewhat of a nightmare to simply parse with RegEx.

\n

I was trying to avoid having to modify any Moonraker code, but it looks like I'll just have to alter the Cura class to extract all lines that start with ;SETTING_3 , then concatenate > JSON Parse > INI Parse.

\n", "Title": "How can I get Cura's slicer settings to save into the G-code in a way that Moonraker/Klipper can read them?", "Tags": "|ultimaker-cura|g-code|prusaslicer|klipper|", "Answer": "

Not sure why I wasn't able to find this initially, but here's a list of replacement strings that can be used in the Start/End G-code settings of the machine/printer profile.

\n

In my Machine profile settings, I inserted the below text at the top of the Start G-code section:

\n
; Cura/Material/Profile settings\n;Layer Height: {layer_height}\n;MINZ: {layer_height_0}\n;Print Speed: {speed_print}\n;first_layer_bed_temp: {material_print_temperature_layer_0}\n;Nozzle diameter = {machine_nozzle_size}\n;Filament id = {material_id}\n;Filament type = {material_type}\n;Filament name = {material_name}\n;Filament brand = {material_brand}\n;Filament amount = {filament_amount}\n;Filament weight = {filament_weight}\n;Filament cost = {filament_cost}\n;Jobname = {jobname}\n\n; Ender 3 Custom Start G-code\n; ... Rest of my start G-code was here\n
\n

And now, when I generate a new G-code file, it has the below data in the file:

\n
; Cura/Material/Profile settings\n;Layer Height: 0.2\n;MINZ: 0.2\n;Print Speed: 30.0\n;first_layer_bed_temp: 260.0\n;Nozzle diameter = 0.4\n;Filament id = generic_abs_175 #2\n;Filament type = ABS\n;Filament name = ASA\n;Filament brand = iSANGHU\n;Filament amount = [1.01]\n;Filament weight = [3.013193642578125]\n;Filament cost = [0.05423748556640625]\n;Jobname = TN1-148_SHAFT_ADAPTOR\n\n; Ender 3 Custom Start G-code\n
\n

(Note: I had to save an ASA material as ABS because apparently, Cura thinks my printer profile isn't compatible with ASA, so it's saved as ABS. but that value is accurate to what's in the configs)

\n

And it works fine in MoonRaker as well

\n

\"Screenshot

\n

\"Screenshot

\n

Source: How to access material type (eg PLA, PETG etc) from Cura post-processing plugin?

\n" }, { "Id": "21760", "CreationDate": "2023-12-06T10:35:51.380", "Body": "

I found this great DIY 'filament diameter sensor' or 'filament width sensor'. It is called Inline Filament Diameter Estimator Lowcost (InFiDEL), as shown below. The GitHub project states:

\n
\n

A cheap, yet precise filament diameter sensor, intended to compensate for filament diameter deviations in real-time.

\n
\n

\"InFiDEL

\n

Model source: Printables.com or Youmagine.com

\n

It can essentially be used for flow control. In a YouTube video the maker (Thomas Sanladerer) says:

\n
\n

... the firmware can then compensate for any differences in filament diameter and filament cross-sectional area essentially by adjusting the extrusion multiplier ...

\n
\n

The video also explains how three different sized drill bits are used to calibrate the measurement.

\n

To my understanding, it uses a principle where a hall effect sensor measures magnetic strength that is translated into a voltage that reflects the diameter, such as 1.78 Volt for 1.78 mm. I plan to implement something like this, however, I don't know how Marlin implements this and I'd like to understand it first. The Marlin documentation does contain several G-codes for a filament width sensor, such as M405, M406, and M407, but I cannot find anything about how Marlin implements this. Also, shouldn't the firmware know the distance between the nozzle and the filament width sensor to know when to compensate? The distance seems to be not implemented by Marlin entirely.

\n

Lastly, I think that this sensor could also serve as a filament runout sensor by noting that the diameter is simply close to 0 mm.

\n", "Title": "How does Marlin firmware use filament width sensor data for flow control?", "Tags": "|filament-sensor|sensors|filament-width|filament-diameter|width-sensor|", "Answer": "

The Marlin firmware is ready for connecting such a filament sensor. You need to adjust the configuration (Configuration_adv.h) to enable the FILAMENT_WIDTH_SENSOR definition (remove the // before #define FILAMENT_WIDTH_SENSOR):\n\"enter

\n

The distance from measurement to melting chamber should and is taken into account.

\n
\n

shouldn't the firmware know the distance between the nozzle and the filament width sensor to know when to compensate

\n
\n

The firmware does do that. You can either set the distance inside the firmware compile-time (which is pretty safe as the distance doesn't change much unless you have a Bowden setup where you exchange the tube for a different length tube or use a tool changer/multiple tool heads with different tube lengths) or through G-code:

\n

M405 D<cm> specifies that distance through G-code, otherwise the firmware set value is used:

\n

\"enter

\n" }, { "Id": "21766", "CreationDate": "2023-12-06T20:10:10.643", "Body": "

I just discovered automatic bed leveling (ABL) probes that only use a microswitch.

\n

For example, the FreeABL as shown below:

\n

\"Photo

\n

Source: Thingiverse.com

\n

Or, the magnet mounted KlackEnder as shown below:

\n

\"3D

\n

Source: Thingiverse.com

\n

Both use the simple concept of probing the bed using a microswitch instead of an optical or retractable touching probe, such as the popular BLTouch. However, since no measurement other than touch can be made, how reliable and accurate are such methods? Did someone ever run a Probe Repeatability Test (M48) for such microswitch based automatic bed leveling probes, or have experience regarding its reliability?

\n

For reference, a 10-probe M48-test using my CRTouch results in a deviation of 0.000750.

\n", "Title": "How can microswitch-based automatic bed leveling probes be possibly reliable and accurate?", "Tags": "|bed-leveling|bltouch|z-probe|automatic-bed-leveling|sensors|", "Answer": "

I use them both for my Ender 3 home-against-bed and for my delta-calibration probe, and they work reasonably well. Of course, as noted in a comment by Fritz, you should remove the metal lever arm, as that amplifies any error, as well as introducing its own non-reproducibility through arm stiffness that may vary with temperature.

\n

I forget the actual spread of values I get probing my delta, but my Klipper probe configuration has samples_tolerance set to 0.015 mm, meaning that, when it probes each point 5 times, it will start over if it gets measurements that differ by more than that amount, and normally it does not restart. However, with it previously set lower (probably 0.005 mm but I don't remember for sure), I did on occasion hit instances of it repeatedly failing. So, in my usage, it was repeatable with an error margin of something greater than 5 microns but less than 15. I don't think that's too bad.

\n

The particular switch/PCB I use was a cheap generic "Ender 3 endstop switch replacement" off Amazon.

\n" }, { "Id": "21768", "CreationDate": "2023-12-06T20:34:15.807", "Body": "

A few closely related questions about the straightness of linear rods. The 'straightness' of linear rods is vital for a 3D printer to function correctly and prevent things like Z-Banding or Z-Wobble. So, how do we measure the 'straightness' of linear rods? What measurement method or metric can be used?

\n

I can imagine using some reference material (a piece of wood or a table), but then again, how would we know the reference material is perfectly straight? Lastly, how straight does it even have to be? What's a standard or acceptable range they should be in, and is there a standard that defines, classifies, and indicates this?

\n", "Title": "What defines the 'straightness' of linear rods and how to measure it?", "Tags": "|z-axis|linear-motion|axis|linear-rod|", "Answer": "

A common metric for straightness of shafts in industrial equipment is Total Indicated Runout (TIR). The shaft is placed in V-blocks or ball bearing rollers at its bearing journals. Runout is checked at various positions along its length with a dial indicator as the shaft is rotated.

\n

This method could be easily applied to guide rods supported at the same axial locations where they are held in the printer.

\n" }, { "Id": "21770", "CreationDate": "2023-12-07T12:55:05.390", "Body": "

Prusa says the MK3S+ draws up to 120\u00a0W. So let's say 150\u00a0W of inrush current.

\n

$$150 W \\div 230 V \u2248 0.65 A$$

\n

An order of magnitude less than a typical Raspberry Pi optically isolated relay shield can do.

\n

Is there anything I should be aware of when choosing the board? Anything I'm missing here?

\n", "Title": "Is it safe to use 230 V AC, 10 A relay shield to turn my Prusa MK3S+ on/off using OctoPrint?", "Tags": "|prusa-i3|power-supply|raspberry-pi|octopi|", "Answer": "

Yes it is safe to use such relay modules. Note that the printer will use less power than the specified power, when in idle it nearly doesn't use any power at all, the inrush current will be ver low when switching the relay. Furthermore, the relay is capable of handling the current according to its specification. Do note that you are using mains power, which can be dangerous, please make precautions working on mains (pull the plug from the socket) and shield the wires and use proper crimping tools, no solder.

\n

I have done this with 2 printers, one where I cut the 230\u00a0V, but the latest one uses a computer 12\u00a0V PSU which has a separate wire (I think this is 5\u00a0V) which you can short to start the PSU; this is much safer. I used and use OctoPrint with custom shell scripts to turn on the printer and the hot end cooler fan as described in this answer.

\n

\"enter

\n" }, { "Id": "21774", "CreationDate": "2023-12-07T21:35:16.313", "Body": "

What are the differences between the bed leveling probes BLTouch, CRTouch and 3DTouch in terms of reliability and accuracy?

\n

Is there some kind of benchmark available? Also, how often should the plastic 'probes' in the BL and 3D touch be replaced? As that problem is solved in CRTouch.

\n", "Title": "What's the accuracy and reliability difference between BLTouch, CRTouch and 3D touch?", "Tags": "|bltouch|automatic-bed-leveling|crtouch|", "Answer": "

This answer goes into the reliability aspects of the often-discussed "Touch sensors" within the 3D printing community based on experience with the BLTouch and the 3D Touch..

\n

Throughout my journey in 3D printing, I've had the opportunity to experiment with various iterations of clones (from metal pins to plastic pins), referred to as the "3D Touch", attempting to emulate the renowned BLTouch sensor. One undeniable allure of these clones is their significantly lower price point, which tricked me into buying them being a budget-conscious 3D print enthusiast. However, it's essential to describe my experiences with these clones, which spanned up until 2019.

\n

At first glance, these clones seemed promising, operating as expected and demonstrating competence in their initial usage. However, this initial phase was short-lived, as many of them eventually caved in to issues that severely impacted their reliability. These problems typically manifested in one of two ways: they either ceased to function altogether or exhibited erratic behavior by unpredictably deploying and retracting the probing pin during critical operations such as G28 and G29 G-code executions. The latter issue, characterized by premature pin deployment when the print bed was not adequately lowered (CoreXY), often resulted in disrupted Auto Bed Leveling (ABL, What is ABL or UBL? Is this the same?) sessions, leading to subpar print outcomes (actually no print at all...) and frustration.

\n

In contrast, after making the switch to a genuine BLTouch sensor, the transformation in reliability was remarkable. It has faithfully served its purpose on my CoreXY printer for a continuous span of approximately five years, without a hint of trouble. The consistent and error-free performance of the authentic BLTouch has not only elevated the quality of my 3D prints but has also instilled confidence in the reliability of my 3D printing endeavors.

\n

In essence, while cost-effective alternatives may initially appear enticing, investing in a genuine BLTouch sensor can undoubtedly pay itself in terms of frustration, reliability and long-term satisfaction. In hindsight, buying a genuine BLTouch would have actually saved me; "Penny wise, pound foolish" comes directly to mind.

\n" }, { "Id": "21789", "CreationDate": "2023-12-10T21:00:49.320", "Body": "

Some parts of this model when slicing in Cura are just being deleted.

\n

Here is the part of the model before slicing; notice how there is a wall between the threads and the inside\n\"enter

\n

And after slicing, that wall is just gone, and this wouldn't be practical to print.\n\"enter

\n

And turning on supports doesn't fix the problem in the slightest.

\n

\"enter

\n

\"enter

\n", "Title": "Why does Cura remove a part of a model when slicing?", "Tags": "|ultimaker-cura|slicing|", "Answer": "

As mentioned in the comments. Probably the model contains too little space for the slicer to be able to slice for that nozzle size and resolution.

\n

You have a few options:

\n\n

Maybe you can increase wall thickness in slicer settings and achieve the same, although I am not sure of it.

\n

As mentioned by you, apparently PrusaSlicer did work. It's interesting to figure out what the actual difference is. Isn't it just the wrong visualization in Cura? Did you try to print that Cura-sliced model or preview it top-down layer for layer?

\n" }, { "Id": "21805", "CreationDate": "2023-12-12T22:00:38.703", "Body": "

As per the Marlin documentation for G-code M413 - Power-loss Recovery:

\n
\n

When Power-loss Recovery is enabled, and Marlin runs a print job from the SD Card or Flash Drive, it periodically saves the print job state to the SD Card / Flash Drive. If the machine crashes or a power outage occurs, Marlin presents the option to resume the interrupted print job.

\n
\n

If the printer is controlled by a host over USB instead (such as OctoPrint), can you still use Power-loss Recovery?

\n", "Title": "Can you use Power-loss Recovery when printing from OctoPrint?", "Tags": "|marlin|octoprint|power-loss-recovery|", "Answer": "

There is an OctoPrint plugin that attempts to achieve exactly that. The plugin is called OctoPrint-PowerFailure and it's page describes:

\n
\n

OctoPrint-PowerFailure attempts to recover a print after a power failure or printer disconnect. It is intended as a last resort and does not replace the use of proper power backup and appropriate communication setups. Recovered parts are certain to show small defects, but this may be acceptable in some cases.

\n
\n" }, { "Id": "21807", "CreationDate": "2023-12-13T03:51:49.457", "Body": "

I've been attempting to 3D print a hollow cube, but I'm encountering issues with noticeable gaps between the walls. Despite adjusting the Extrusion multiplier and width settings, the results remain unchanged. Can someone provide guidance on how to address this issue or suggest alternative parameters to tweak for a more solid print?

\n\n

\"enter

\n

\"enter

\n

\"enter

\n", "Title": "What could be the underlying reasons for the spaces or voids between the layers?", "Tags": "|print-quality|creatbot-f430|", "Answer": "
\n

What could be the underlying reasons for the spaces or voids between the layers?

\n
\n

Underextrusion is what is called the phenomenon you see.

\n

For a calibrated printer, which is pretty much how you get them delivered, there should not be any fiddling with the extrusion multiplier parameter necessary to get reasonable prints.

\n

When underextrusion appears in your prints, this usually suggests that something is wrong in the extruder to spool to nozzle path.

\n

Many of the underextrusion can be related to e.g. (this is an example list and may not be complete, but, given the question it is hard to pinpoint to a specific issue):

\n\n

You need to do some troubleshooting, inspecting and cleaning of the complete extrusion path and address any problem you encounter.

\n" }, { "Id": "22810", "CreationDate": "2023-12-13T15:23:50.083", "Body": "

It is my first time printing flexible filaments.

\n

I have been trying to print a flexible case for my mini-PC using the TPE 83A by eSun on my KP3S pro.

\n

Whenever I try to print it starts well, then after printing a few strokes, the print systematically fails.\nIt is not an adhesion/levelling problem as I installed a BLTouch probe and made a 12x12 points grid of my 200x200 printing bed, which works perfectly with PLA.\nIt seems that after printing a few lines, the nozzle just stopped extruding filament anymore (see photos); although the extrusion gear still seems to be turning.

\n

FYI my nozzle is a hardened steel nozzle, that I recently acquired to replace the original brass nozzle (because I wanted to print abrasive filament). Could it be the problem? I read that a hardened steel nozzle may not conduct heat as well. I did try a wide range of temperatures and saw the same results; also given the small size of the MK8 nozzle used by the KP3S, I would wager that heat losses due to poor heat conductivity of the nozzle may not be as important no? (I guess that the delta in temperature between the contact point with the heating also depends on the length of the nozzle)

\n

Other information:

\n\n

\"Photo

\n

Any idea of what might be happening? I gleaned a lot of info from Proper settings for printing rubber and rubber-like materials? on printing flexible filaments. I will perform a bunch of tweaks based on that question to hopefully troubleshoot my way to a proper print through trial and error; however, it would be nice if someone has a better idea that could direct me as to what could be the problem given the symptoms observed.

\n", "Title": "TPE print fail - filament fails to extrude after normal start", "Tags": "|filament|fdm|tpu|flexible|tpe|", "Answer": "

After several months of trial and error with this filament, I finally managed to understand the issue.\nIt was a mechanical issue due to the filament sensor of the KP3S Pro.

\n

The out of the box setup of the KP3S pro is with the filament roll being set on rollers and passed into a filament sensor to anticipate situations where filament is running out.

\n

\"Out

\n

However, this sensor applies pressure on the filament, and it creates a mechanical strain that prevents more elastic materials from being pushed through the extruder.

\n

In order to lessen the mechanical strain on the filament I mounted it over the machine with a spool holder, and stopped using the filament sensor when printing my eSun eLastic TPE 83-A. I found this video, Kingroon KP3S Pro Top Mount Spool Holder in Detail\n, which uses a KP3S spool mount model which can easily be adapted to the KP3S pro.

\n

Thus the filament feeds directly in the drive, and I have no problem printing this flexible filament.

\n

\"Top

\n

I may (or may not) explore a bit more in the future to see if I can tinker with the pressure of the filament sensor so that I can take advantage of it also with flexible/elastic filaments. But for now I will be printing flexible filament without using the filament sensor.

\n

I won't go in detail about the parameters I used yet for this filament, because I am still exploring the best parameters for this filament. But this issue really was the core of my problem and I could already obtain very satisfactory results with my KP3S Pro printing this eSun TPE-83A.

\n" }, { "Id": "22818", "CreationDate": "2023-12-14T20:07:22.750", "Body": "

I need to convert the rotation movement of a motor to a linear movement (it has to be in the same axis of the motor).

\n

I made a 3D model in Tinkercad consisting of two parts:

\n\n

I printed this model but the plunger gets stuck inside the first part and do not move.

\n

So it fails.

\n

I want a simple way to convert the rotary move to linear in the same axis of motor.

\n

I broke part #1 ("the first part") from behind to see what happened inside. I found it sticks as part #2 jumps above the screw shape.

\n

\"enter

\n

I think this mechanism is hard to be done.

\n

So I switched to scotch yokes mechanism.

\n

Hope it works!!

\n

The vertical gear will pull the horizontal gears to move the articulator in a linear movement.

\n

\"enter

\n", "Title": "Convert rotation movement to a linear movement, in the same axis", "Tags": "|rotation|", "Answer": "

The following might seem obvious, but it shouldn't be overlooked either.

\n

Unless you have one of those Wire EDM cutting machines, with very precise tolerances1, that can seamlessly cut a shape out of blocks of metal that then, seemingly, glide out of the surrounding block, with little to no stiction2, then your printed surfaces of the screw will have imperfections that may cause the binding. The first step would be to shave, sand and smooth the surfaces. Maybe even treat with another chemical, such as acetone (depending upon your printing material) to smooth the surfaces even further.

\n

However, in addition to that, some lubricant may be in order. Every gear box or mechanism contains a little oil or grease.

\n

So, your original design may still work, with a little help from smoothing and lubricating any mating surfaces. As Trish states, your design has a lot of surface friction, so every bit of polishing of the surfaces will help.

\n
\n

1 Approaching $\\frac{5}{1000}$ (i.e. five thousandths) of a millimetre

\n

2 Such as the objects shown at the start of this video: How these impossibly thin cuts are made

\n" }, { "Id": "22827", "CreationDate": "2023-12-18T20:41:08.390", "Body": "

Looking at a bare Creality Ender 3 V2, I realised that there are three end-stop switches, one for each axis, X, Y, and Z. But this only helps to determine the start (0) position of each axis. Wouldn't doubling the end stops to six instead, two for each axis, one on each side, help to determine not only the start but also the end position? (Besides, to my understanding, the switches are called 'end stops' but are used as 'start stops' in this case.)

\n

If each axis has an 'end stop' at the start and at the end, it can dynamically determine how many steps can be taken. Isn't this more elegant and adaptable than using fixed sizes? Also, since end-stop switches are simple and cheap, why don't all printers use them? I think it could also help in precision calibration and still stay operational when hardware tweaks are made without firmware changes.

\n

Moverover, wouldn't it prevent wrongly configured firmware (by means of a 'hard stop' where stepper motor are trying to move something moving beyond the physical possible axis potentially resulting in catastrophic damage?

\n", "Title": "How could dual end-stop switches on each axis improve the functionality and precision of 3D printers like the Creality Ender 3 V2?", "Tags": "|firmware|calibration|endstop|hardware|sensors|", "Answer": "

First the questions in the body are addressed:

\n
\n

I realised that there are three end-stop switches, one for each axis, X, Y, and Z. But this only helps to determine the start (0) position of each axis.

\n
\n

No not necessarily, you could mount an endstop to the other end of the axis to determine the maximum value for that axis without the need for a start endstop, Marlin is already equipped with functionality for this:

\n
// Direction of endstops when homing; 1=MAX, -1=MIN\n// :[-1,1]\n#define X_HOME_DIR -1\n#define Y_HOME_DIR -1\n#define Z_HOME_DIR -1\n
\n

E.g. Ultimaker printers have their Z-endstop at the bottom, so the maximum value of the Z axis.

\n
\n

Wouldn't doubling the end stops to six instead, two for each axis, one on each side, help to determine not only the start but also the end position?

\n
\n

No, not necessarily, the end is fixed by the start position and the defined axis length (e.g. bed size of Z movement). It might be interesting to use a second endstop in case something goes wrong. E.g. layer shift as a result of the nozzle hitting something redefines the origin and could result in thinking the origin is somewhere on the build plate, adding the maximum bed sizes, the printhead could run into the end and destroy your printer, a second endstop, when triggered will stop the printer. I have used such an endstop on one of my printers, but I have never ran into problems with layer shifting and as such the endstop was never hit; I removed this endstop recently.

\n
\n

Besides, to my understanding, the switches are called 'end stops' but are used as 'start stops' in this case

\n
\n

The start is also an end of the axis, there are two ends on an axis.

\n
\n

If each axis has an 'end stop' at the start and at the end, it can dynamically determine how many steps can be taken.

\n
\n

No, steps are determined by your hardware (common hardware configuration for stepper motors is e.g. 200 steps per revolution or 400 or even more/less steps per revolution) together with the stepper driver which allows for substeps of the afore mentioned values.

\n
\n

Moverover, wouldn't it prevent wrongly configured firmware (by means of a 'hard stop' where stepper motor are trying to move something moving beyond the physical possible axis potentially resulting in catastrophic damage?

\n
\n

Yes it would, I already described that in case of layer shifting. But if you are bold enough to compile your own software you should known what you are doing and configure it correctly. Also test the printer before you use it and keep the powerswitch nearby.

\n

Now to answer the main question,

\n
\n

How could dual end-stop switches on each axis improve the functionality and precision of 3D printers like the Creality Ender 3 V2?

\n
\n

The precision will not be improved by adding another endstop to the other side of the axis. However, the functionality can be improved by adding another endstop but this is mainly protecting the printer in case something went wrong, as in the printhead has changed its position relative to the origin, e.g. in case of layer shifting. Knowing that the Creality Ender 3 are printers on the cheap end with their shortcomings (see all the questions) this may prevent damaging the printer, but, generally the belt skips when the head is obstructed in X-Y range. For the Z direction this may be different as there are lead screws involved which can procude more force (but I have yet to see a layer shift in Z direction...).

\n" }, { "Id": "22832", "CreationDate": "2023-12-20T08:46:34.653", "Body": "

I've 3D printed a box with a hole in it, the hole looks like a horse. My idea was that I then 3D print that horse in a different color and put it into the hole. The problem was that the horse didn't fit the hole, as they were the same size. I then made the horse smaller, but that made a new problem. Now the feet of the horse don't align with the hole. How can I scale the horse easily, like how can I just make everything thinner?

\n

\"Vector

\n

The image above is how the horse looks, and what I want is to make all sides go in 3\u00a0mm, I'm not sure how to do it without the legs moving.

\n", "Title": "How to fit a model into a hole with the same shape?", "Tags": "|scaling|", "Answer": "

Scaling in FreeCAD is very easy but you must remember that it's CAD not art. You can offset the sub-binders off a face or sketch. You can't offset the pad or the sketch without playing with parameters, a lot, which you won't want to do.

\n

Give the sub-binder (the green one) a negative offset of say -0.5mm. Pad it as if it were a sketch and it should fit the hole.

\n" }, { "Id": "22845", "CreationDate": "2023-12-24T13:14:16.807", "Body": "

Recently both belts on my Prusa MK3S+ became loose. Quite suddenly, too. One print was okay, the next nozzle wobbled in the Y and bed in the X causing layer shifting. More precisely, each movement was shifted pretty randomly.

\n

I checked belt tension and there was basically none. It was good a month ago, and it must've been good enough just one print before. Of course I will tension them again, but I would be glad to know what are the possible causes of belts losing tension suddenly, to check them during the maintenance.

\n

\"Shot

\n", "Title": "What could have caused both belts becoming loose?", "Tags": "|prusa-i3|repair|belt|", "Answer": "

I figured it out.

\n

Long hours of printing ASA parts for my upcoming Voron Trident, in an enclosure, made printer parts softer and belts skipped on the teeth. I needed to glue and shim belt ends in place. Next time I'll need to replace these parts, and belts. Hopefully I'll have another printer by then.

\n" }, { "Id": "22846", "CreationDate": "2023-12-25T06:13:05.207", "Body": "

I'm working on printing the 3D Printable Jet Engine w/ Minimal Printing Supports and I'm having a surprising amount of difficulty printing the fan. The fan seems to print fine, right up until the last 20 or so layers, where it starts to gap and blob-like I've never seen before.

\n

I have quite a few photos available to view, but I'm including a few below:

\n

\"Photo

\n

\"Upclose

\n

Settings/components:

\n\n

What I have tried:

\n\n

If it was something like a clogged nozzle, then I would assume that that same pattern would be seen everywhere on the print. That goes for the temperature and retraction as well.

\n", "Title": "Encountering some weird layer gaps on the last few lines of a particular print", "Tags": "|ultimaker-cura|slicing|filament-quality|layer-shifting|", "Answer": "

I re-printed the model using Curas overlay settings and some stretched cubes to apply different settings to some of the fan blades:\n\"enter\nAnd once printed, the blade seen here at the very bottom looks like it was the best:\n\"enter\nThe changes on that blade were just 4 bottom layers, 0 top layers and 7 walls (effectively making everything after the bottom 4 layers 100% density with just walls).

\n

I'm not sure why exactly the gap between the top layers and the walls was showing up, but regardless, this seems to be the fix.

\n" }, { "Id": "22848", "CreationDate": "2023-12-25T15:57:41.857", "Body": "

I'm thinking about implementing some kind of nozzle wear sensor. My question is simple: Do such sensors or automatic approaches already exist? Although I expect not and I know nozzles are cheap, it's just a fun educational challenge, not financially viable but that's not the purpose anyway.

\n

I thought about using a simple camera with a proper lens and computer vision to do either one or more of the following things:

\n
    \n
  1. Have the extruder retract, perhaps brush the nozzle, then have a zoomed camera and light pointing up from under the nozzle, in an attempt (with some reference image next to it) to determine nozzle diameter size.
  2. \n
  3. Have a camera from the side of a nozzle, brushing it clean, extruding a few cm of filament, and analyzing how straight the filament flows down when extruded
  4. \n
  5. Take two precision measurements from under the nozzle somehow, one of the heat block and one of the nozzle-end, and determine the difference, although I have no way to account for other variables such as the nozzle not being screwed in properly, maybe non-standardized nozzle-height et cetera.
  6. \n
\n

Does something like this or another approach to achieve the same already exist? If not, is this a proper approach or are there other things to consider?

\n", "Title": "What is a good approach to develop a 'nozzle wear sensor'?", "Tags": "|nozzle|printer-building|sensors|", "Answer": "

It makes a huge difference if you want to measure the internal nozzle diameter or the "typical" wear when using an abrasive filament. CNCKitchen has a nice article,How Much Abrasive Filaments Damage Your Nozzle!, about the typical wear.

\n

TLDR is that most of the wear comes from moving your nozzle through parts of the last layer during travel moves. This way, your nozzle tip gets shorter.

\n

If that's what you want to measure, you might use your bed-leveling sensor for that. If your leveling sensor is precise enough, you could measure the distance to the bed with your BLTouch or other sensor. When your nozzle is new, you can take a second measurement by "pressing" the nozzle into the bed and detecting when motor current spikes, or by lowering it to a conductive edge of your print bed. This way, you can save the distance of your bed leveling sensor in relation to your nozzle tip. If your nozzle wears out, it should be shorter, and you should see that you can move your hotend lower before closing an electrical circuit or before your stepper driver uses an increased current (because it pushes into the bed).

\n

I need to mention that this is just an idea and I have never seen a solution like this. Doing this might pose problems I didn't think of.

\n

As you mentioned in possibility 3, a loose hotend screw can make the measurement of the distance difficult. Maybe you can remove that error by probing against a metal that has the inverted shape of a nozzle (just an inverted cone) which shouldn't change with nozzle wear, and then probe against a flat surface afterward and examine the difference. For this to work, you would of course have to clean the nozzle before any measurement.

\n" }, { "Id": "22850", "CreationDate": "2023-12-25T21:15:09.453", "Body": "

Polytetrafluoroethylene (PTFE) is one of the best-known and widely applied PFAS commonly described as persistent organic pollutants or "forever chemicals".

\n

If I want to avoid PFAS due to environmental concerns, is there an alternative to:

\n
    \n
  1. PTFE Bowden tube used?
  2. \n
  3. PTFE coated nozzles?
  4. \n
  5. PTFE spray used for rod maintenance?
  6. \n
\n", "Title": "Is it possible to 3D print without PFAS/PTFE?", "Tags": "|maintenance|ptfe-tube|ptfe|pfas|sustainability|", "Answer": "

None of those items are needed.

\n

If you have a bowden extruder, upgrade to a direct drive.

\n

Even on direct drive systems, PTFE tube is commonly used as a "reverse bowden" or "feed tube" to deliver the filament from a drybox or spool mounting point to the toolhead. Other tube materials may be used, but may contribute too much friction; you'll have to experiment if you want to go that path. You can usually forgo having one at all if youu mound the spool with a good direct path, or setup a system of filament guides/pulleys.

\n

If you have a PTFE-lined hotend, replace it with an all-metal one.

\n

Direct drive extruders may have a small piece of PTFE between the extruder and hotend, even when the hotend is all-metal. This part of the filament path being low-friction is not critical, and it can be replaced with a 2mm ID metal or even printed plastic tube without seriously compromising extrusion capability.

\n

Nozzles should never be PTFE-coated. Don't buy ones that are. The best nozzles are 100% tungsten carbide (never wears out, never wears into air or into your printed parts), but pretty much no nozzle should contain any PTFE unless explicitly marketed as such (eew).

\n

As a lubricant, there are lots of choices other than PTFE sprays, most of which are probably better. PTFE is highly recommended against for linear rails, as it's reported to lead to sliding by the balls instead of rolling. For smooth rods it might be a good choice, but I would expect there are grease, graphite, or other options that work just as well. I've never used a printer with smooth rods so I can't recommend anything in particular from experience.

\n" }, { "Id": "22863", "CreationDate": "2023-12-27T20:35:27.320", "Body": "

I've been repeatedly printing the same G-code on two fairly identical machines (Creality Ender 3 V2) with the same settings, the only difference being one is printed from a Micro SD card and one is printed using OctoPi over a USB cable.

\n

Although I didn't measure the exact times, I have the idea that the SD card prints were consistently faster.

\n

Is it possible that prints from Micro SD card are indeed faster than prints sent from OctoPrint and if so, by how much percent could it be and why?

\n", "Title": "Is printing from a micro SD card faster than printing over a USB cable?", "Tags": "|g-code|usb|microsd|sd|performance|", "Answer": "

The slowest USB standard is "low-speed USB 1.0", at 1.5 megabits per second. The slowest SD card I've been able to find has a rated read speed of 900 kilobits per second.

\n

The average line of G-code is about 200 bits. Even the slowest USB connection or SD card can transfer thousands of lines of G-code per second, much faster than your printer can carry out the relevant motions. Any speed difference is going to come from how a printer handles those data sources, not anything inherent in them.

\n" }, { "Id": "22875", "CreationDate": "2023-12-30T14:35:30.853", "Body": "

To my understanding, all settings including those that resulted from the Nozzle Auto MPC tuning and Bed PID tuning and the created UBL mesh are all stored I the EEPROM by the mriscoc Marlin firmware on a Creality Ender 3 V2.

\n

I'm asking because I recently encountered a bad (or very slow) micro SD card, where the printer took long to boot to the start screen and start beep (30 sec), consistently with a bad SD, and without a SD it booted almost instantly.

\n

So, does a Creality Ender 3 V2 running mriscoc firmware require a micro SD card at all, for other things as I previously mentioned?

\n

I'm printing only over USB using OctoPrint on a Raspberry Pi (OctoPi).

\n", "Title": "Does a Creality Ender 3 V2 running mriscoc firmware require a micro SD card at all?", "Tags": "|creality-ender-3|marlin|microsd|mriscoc|eeprom|", "Answer": "

If you only print from Octoprint, the SD card is not used.

\n" }, { "Id": "22883", "CreationDate": "2024-01-01T14:48:01.887", "Body": "

I'm using the mriscoc firmware for Creality Ender 3 V2. The firmware has the option to run more PID Autotune cycles. The default is 5 cycles. Does running a PID Autotune with an increased cycle count result in more accurate PID values? For example running 50 cycles instead of the default 5?

\n", "Title": "Does running a PID Autotune with an increased cycle count result in more accurate PID values?", "Tags": "|temperature|pid|mriscoc|", "Answer": "

In short, yes, More PID cycles will generally yield 'better' results.

\n

However, the PID values are being averaged over these multiple runs, and it will very quickly pass the point of diminishing returns after 5 cycles, Hence why it is the default.

\n

By the time you reach the hypothetical 50 cycles, you have likely far exceeded the margin of error present as a result of tolerances in your setup, and are only averaging noise.

\n" }, { "Id": "22888", "CreationDate": "2024-01-02T13:30:33.927", "Body": "

I want to make a print where I swap color halfway through. I have tried doing something similar with a 2+ Connect before, and what the internet told me then was to go into Ultimaker Cura and modify the G-code to add a filament change. And the printer blew right past that layer and printed the whole thing in one go. (How Cura would allow me, completely without warning, to insert G-code unsupported by the chosen printer, especially since the chosen printer is of their own make and entirely under their control, is beyond me, and also a different question.)

\n

So naturally, now that I'm working with an Ultimaker 2+ (Extended, with original firmware up-to-date according to Cura), I am inclined to distrust Cura with these things. I have tried to look into modifying the G-code manually to add a pause, and then by hand interrupt the print to change filament. The two main options I have found are the commands M0 and G04. But none of the sources I have found have mentioned printer support. I haven't been able to find anything online that tells me whether M0 or G04 are actually supported by the firmware.

\n

The G-code that Cura makes, with the settings I chose, uses M0, as shown below:

\n
G0 F9000 X19.305 Y19.27\nG0 X19.27 Y19.27\n;TIME_ELAPSED:25576.615046\n;TYPE:CUSTOM\n;added code by post processing\n;script: PauseAtHeight.py\n;current layer: 22\nM83 ; switch to relative E values for any needed retraction\nG1 F300 Z4.57 ; move up a millimeter to get out of the way\nG1 F9000 X190 Y190\nG1 F300 Z15 ; too close to bed--move to at least 15mm\nM0 ; Do the actual pause\nG1 F300 Z3.57\nG1 F9000 X19.27 Y19.27\nG1 F300 Z3.57 ; move back down to resume height\nG1 F1800 ; restore extrusion feedrate\nM82 ; switch back to absolute E values\nG92 E3066.65549\n;LAYER:22\n;TYPE:FILL\n;MESH:brett.stl\nG11\nG1 F3600 X34.487 Y34.487 E3067.94669\n
\n

Would this even work? And if it turns out that it doesn't, is it as easy to fix as swapping out that M0 command with a G04 (possibly with a specified time argument)? Finally, I want this change to happen between what Cura, in the slice preview, calls layers 23 and 24, where it seemingly gives the first layer the name 1. The G-code insertion wizard asks me to insert the last layer I would like finished before the pause and specifies that 0 is the first layer printed. So I enter 22 in there. But in the above G-code, it seems to have inserted the pause before it begins to print its layer 22 (which in the Cura preview would be layer 23?) I need some help untangling this.

\n", "Title": "Does Cura's M0 pause work to pause an Ultimaker 2+ (Extended) print?", "Tags": "|ultimaker-cura|g-code|ultimaker-2|", "Answer": "

I decided to go ahead and try anyways, to moderate success.

\n

The main takeaways:

\n\n

In the rightmost corner you can see a little bit of the damage from the leaking while swapping filament, but most of it is hidden below the black (and you can also see that it didn't stick to the build plate, but that's entirely unrelated to this issue).

\n

\"enter

\n" }, { "Id": "22889", "CreationDate": "2024-01-02T20:55:22.867", "Body": "

Yesterday, I upgraded my Ender 3 with a Creality touchscreen that is compatible with it. However, since the installation, it gets stuck on the loading screen once the bar fills up. I tried different versions and even built my own, but they all gave me one of three responses:

\n\n

And I can't reinstall the old LCD because I had to cut its cable to disconnect it.

\n

I also asked the Bing AI, which gave me tips that didn't work.\nAny tips from this community?

\n", "Title": "My Ender 3 malfunctions with touchscreen", "Tags": "|creality-ender-3|troubleshooting|firmware|", "Answer": "

Agarza was right.

\n
\n

Have you checked to see if the screen itself needs a firmware update?

\n
\n

I inserted the SD card into the wrong place. I finally got it to work by inserting the SD card with the DWIN folder into the touchscreen itself (after unscrewing the housing) and successfully flashed the working version of the firmware.

\n" }, { "Id": "22897", "CreationDate": "2024-01-04T10:20:46.293", "Body": "

The new OctoPi camera stack is reported to be lighter, more efficient, and overall better. Old camera stack sometimes loses access to my camera during print, and only regains it after restart. Thus, obviously, I want to try the new one. Maybe it'll help with my specific problem, maybe it won't, but it has to be at least slightly better. I do not want to lose my uploaded files, Octolapse lapses, plugin config etc.

\n

So, is there a way to upgrade to the new camera stack without reinstall? Or to save plugins, configs and files to restore them after reinstall?

\n", "Title": "How to migrate OctoPi to new camera stack without reinstall?", "Tags": "|octopi|", "Answer": "

I think many problems could arise from doing this manually such as missing or incompatible dependencies. As I had to do the same I just tested the following successfully:

\n

Making a backup of the OctoPi configuration (required)

\n\n

Make a backup of the entire SD card (optional)

\n

You can make a full bit-to-bit copy (image) of the SD card as a backup or use another SD card for testing it first. On Windows you can for example download Win32DiskImager and make a full image/backup of the SD card.

\n

Flashing OctoPi with the new camera stack

\n\n

\"Raspberry

\n

Restoring the OctoPi configuration

\n\n" }, { "Id": "22907", "CreationDate": "2024-01-05T14:21:49.017", "Body": "

When pushing the limits of the bed size (235x235\u00a0mm) for two Creality Ender 3 V2 printers running the same mriscoc firmware and the same config and upgrades. I noticed that despite using the same G-code for these two fairly identical printers resulted in offsets that was not accounted for.

\n

I sliced and printed a test model 175mm_bed_center_calibration_crosshair.stl from Thingiverse.com and when slicing in Cura it is perfectly centered as shown below:

\n

\"test

\n

However when printed on each printer and measured, the position in mm for both printers (namely Owl and Fox) are off as shown below:

\n

\"measurement

\n

I would like to setup the printers fairly similar, I only print from OctoPi over USB to these printers, one Pi for each printer, so that I can slice once and use the same G-code on both printers. How do I calibrate this without recompiling the firmware? Can I setup a start or end G-code in OctoPi per printer to set a proper center offset? And moreover, why does this happen for identical printers? Shouldn't the tolerance be much smaller than 3\u00a0mm difference horizontally and 2\u00a0mm vertically (shown from above)?

\n

Additional measurements

\n

For reference, based on request in the comments some measurements of the end stops relative to the frame of the XY-axes of both printers.

\n

Measurement of the Z-axes are irrelevant for this purpose.

\n

Owl Y-axis Endstop (85mm)\n\"Owl

\n

Fox Y-axis Endstop (85mm)\n\"Fox

\n

Owl X-axis Endstop (45mm)\n\"Owl

\n

Fox X-Axis Endstop (45mm)\n\"Fox

\n

I also measured the size between the bed and the frame as shown below and it's precisely 7mm on each side of the bed on both printers.

\n

\"Space

\n

I think these are the only measurements that make sense, but correct me if you have other suggestions for measuring it.

\n

Also, the only offset that is reported after running the M503 G-code in G-cod M851 it not relevant for centering, the Z-offset. For Fox M851 X-41.50 Y-7.00 Z-1.13 ; (mm) and for Owl M851 X-41.50 Y-7.00 Z-1.06 ; (mm).

\n", "Title": "Why do two identical Creality Ender 3 V2 printers have a center that is 3 mm off from each other and 7 mm off expected and how to calibrate it?", "Tags": "|creality-ender-3|calibration|bed|mriscoc|octopi|", "Answer": "

Perhaps the hot-end or end stops of one of the printers are located in a slightly different location. That's why there is a calibration guide to be able to correct that.

\n" }, { "Id": "22965", "CreationDate": "2024-01-20T11:20:03.007", "Body": "

I've bought an additional second hand printer and I noticed that the lead screw has been bent. I did not have any issues so far with prints that are only a few cm tall.

\n

I filmed it from the top, next to a ruler, as shown below and put a grid over it for reference. I think it should be replaced. How bad is this? When should it actually be replaced?

\n

\"rotating

\n", "Title": "When is a leadscrew too bent and should be replaced?", "Tags": "|lead-screw|", "Answer": "

Cause and effect

\n

Bent screws can cause an effect known as Z-wobble. But, to mitigate this, the non-driven side of the lead screw is left unconstrained. This is intentionally designed as such.

\n

This means that as long as the linear guide (being V-slot with rollers, linear shafts or linear rails) are sufficiently guiding the carriage, a little bent screw would not cause too much problems in the print result.

\n

Note that the animated gif shows the end displacement, this exaggerates the actual displacement experienced by the lead screw nut.

\n
\n

When is a leadscrew too bent and should be replaced?

\n
\n

If it causes quality issues in your prints.

\n

E.g.:\n\"enter

\n

Mitigation

\n

Buying new, straight leadscrews. Usually, the more straight the more expensive, the lead screws from the far East range from acceptable to unusable. Ball screws are much more precise, but also cost more, especially the ground screws.

\n

Some people straighten the lead screws, there are videos to be found how you do that.

\n

But, note that there is an option not to replace the screws!

\n

E.g., to completely remove any wobble of the lead screw, you can use something called an Oldham coupler to decouple the X-Y movement from the Z movement. For Creality Ender printers, there are specific couplers to order to install so that you will not need to buy new lead screws.

\n

Custom printer designed Oldham couplers look similar to:\n\"enter

\n

There are also printable versions that work very well, in fact I designed and used my own, inspired by this design:\n\"enter

\n" }, { "Id": "22973", "CreationDate": "2024-01-21T21:24:04.473", "Body": "

I already switched my standard 0.4\u00a0mm nozzle to a standard 0.6\u00a0mm nozzle with a huge printing time reduction without losing much detail. I also would like to try using Core Heating Technology (CHT) 0.6\u00a0mm nozzle instead, to be able to increase printing speeds and reduce printing time even more.

\n

CNC Kitchen tested the performance with the following results:

\n

\"Extrusion

\n

I found some E3DV6 CHT nozzles, do they fit on a stock Ender 3 V2 hotend or do I need a specific E3D hotend for that? Or since I don't want to upgrade my hotend, are there MK8 CHT nozzles, or are the sizes the same?

\n", "Title": "Are there any Core Heating Technology (CHT) nozzles available for a stock Ender 3 V2 hotend?", "Tags": "|nozzle|speed|e3d-v6|cht|mk8|", "Answer": "

Creality style "MK8" hotends (including the Ender 3's) can use "MK8" or e3D V6 nozzles.

\n

Bondtech sells genuine CHT in both Creality MK8 and e3D V6 forms. If you prefer matching what your machine has exactly, you can buy that, but it will only work on the Creality-style "MK8" hotends, not anything else. If you buy the e3D V6 version, it will work on your Ender's Creality MK8 hotend and any hotend that's compatible with e3D V6 nozzles.

\n

I've always bought the e3D V6 version for my Ender and it's worked fine.

\n" }, { "Id": "22979", "CreationDate": "2024-01-22T19:47:24.187", "Body": "

I noticed when scanning objects using photogrammetry or the AI-based Neural Radiance Field (NeRF) in apps such as Kiri Engine that the original size is not retained in the object.

\n

How can I actually retain the original size, or set the proper sizes when or after scanning objects like that? So that, when 3D printed it is the same size as the original?

\n", "Title": "How to set or retain original object size in photogrammetry scans with apps such as Kiri Engine?", "Tags": "|cad|scanning|artificial-intelligence|photogrammetry|nerf|", "Answer": "

Reference objects

\n

When scanning something, including a simple reference geometry, such as a cube, is best. This can then be used to scale the final model to the correct size and as a reference for if the item has the correct angles. Besides cubes, coins (esp. matte ones) make handy markers and scale references.

\n

If your scan contains color values, a simple photo scale angle could suffice. That item would be taken directly from photography. There it is sometimes required to contain size information and a pointer to the angle the photo was made at. This is made by using a square or angle with scale markings and distortions of the item can be solved later till the angle appears properly and to the right scale.

\n" }, { "Id": "22980", "CreationDate": "2024-01-23T05:59:15.687", "Body": "

My son is printing a simple block with a small (2\u00a0mm high) bump on top - see image below of the cross section. The left shows the 3D model and the right shows where the plastic actually prints, using a grid infill at around 10\u00a0%, with a top height of 1.6\u00a0mm

\n

\"Grapic

\n

As you can see, this is very weak, and the bump very easily snaps off, as it has very little plastic securing it to the layer below. When we increase the top/bottom thickness (in Creality Cloud) to 4\u00a0mm this issue goes away, and the bumps are well connected. But then the print takes a very long time.

\n

Questions:

\n
    \n
  1. Is this the expected behavior of a slicer? Why does it not add material to firmly connect the bump to what is beneath it?

    \n
  2. \n
  3. How exactly is the "top thickness" setting applied? E.g. if it's set to 2\u00a0mm, does that only apply to the layers comprising the topmost 2\u00a0mm of the model, or does it stipulate that there must be 2\u00a0mm of plastic at the 'top' of the model at all X, Y coordinates?

    \n
  4. \n
  5. Is there a way to set the top thickness separately from the bottom thickness in Creality Cloud?

    \n
  6. \n
\n", "Title": "Effect of top thickness on print with bumps on top", "Tags": "|slicing|", "Answer": "

Modern versions of Cura have a feature called "skin edge support thickness" which will build extra perimeter-like layers under the edge of your "bump" down into the infill area, making it attach much better. I believe it defaults to on.

\n

Your problem is probably that "Creality Cloud" is based on an ancient fork of Cura and does not do this, but you could check and see if they have anything similar available. The right solution, though, is to use real up-to-date slicing software like recent Cura (5.x at the time of this writing; I believe the feature was introduced near the end of the 4.x series), PrusaSlicer, or anything but the junk your printer vendor ships (or doesn't even ship but expects you to interface with via the cloud?)

\n" }, { "Id": "22986", "CreationDate": "2024-01-24T02:07:53.767", "Body": "

I have been trying to print fiber-filled nylons (Polymaker PA6GF and Taulman PA CF) on my Prusa i3 MK3S+ with a steel E3D 0.4\u00a0mm nozzle at a reasonable speed. I print at 290\u00a0\u00b0C for the Polymaker and 265\u00a0\u00b0C for the Taulman. This is at the upper end of the specified print temperature range. I can only print up to 25\u00a0mm/s before the extruder gears slip and never regain traction. They are tightened pretty much all the way (not that it makes much of a difference). Can the print speed be improved somehow without moving to a larger nozzle, or is this the limit? I was thinking a bimetal nozzle or maybe even a volcano-length nozzle with nuts might improve the viscosity. I would like to get up to maybe 50\u00a0mm/s.

\n", "Title": "How can I improve print speed with carbon fiber nylon and an 0.4 mm nozzle?", "Tags": "|extrusion|nozzle|speed|nylon|carbon|", "Answer": "

I got a nickel plated copper meltzone extender which solved the problem. I was able to print a decent benchy with the fastest print speeds at 70 mm/s. The melt zone extender results in the same melt zone length as a volcano nozzle in a normal v6 heater block. A volcano nozzle with an adapter or just a couple m6 brass nuts should work the same. I got the Polymaker filament to work but not the Taulman. I would avoid it.

\n" }, { "Id": "22988", "CreationDate": "2024-01-24T14:08:34.890", "Body": "

I drew this object by myself using Blender:

\n

\"enter

\n

Importing the STL in Cura v5.3.1 under Ubuntu 23.10 I'm able to slice it for a Sovol SV04 printer, but if I select the Dremel 3D45 printer it fails:

\n

\"enter

\n

I don't see any "red" spot on the model that would tell me it's damaged.

\n

What could prevent the slicing changing the model of the printer?\nHow to understand why it cannot slice and how to fix then?

\n", "Title": "Why Cura cannot slice this object?", "Tags": "|ultimaker-cura|slicing|dremel-3d45|", "Answer": "

While without the files it's a guessing game, we can make an educated guess: the most frequent and likely culprit for this sort of faults is the model not being a manifold.

\n

Most likely you've created several objects in Blender, and visually they comprise the shape you want, but internally they intersect, creating a lot of unnecessary internal geometry under the surface, and Cura fails to deduce what parts of the geometry are the actual "skin" of the object.

\n

First, if your shape is composed of multiple objects, use the Boolean Union modifier, to merge them into a single object. Besides that, install "3D Printing Toolbox" add-on in Blender, find it in the sidebar tabs, and use the "Make Manifold" option, and inspect your shape to make sure it worked correctly and removed everything "under the surface", leaving only hollow skin. If it failed, you may have to do that manually.

\n" }, { "Id": "22993", "CreationDate": "2024-01-26T07:46:27.007", "Body": "

Is it possible to laser engrave an object that was 3D printed with PLA or will the structure of the whole object simply melt and fail?

\n

How does PLA behave when being engraved? Does it require special treatment of the PLA first or is, for example, more infill required?

\n", "Title": "Can a PLA print be laser engraved?", "Tags": "|pla|post-processing|infill|laser|", "Answer": "

Yes, it is possible

\n

Jered Adams shared, in a Youtube Video from 2023, that he could, in fact, engrave prints with a laser, and his method. The problems he encountered were mostly related to getting good reference points and affixing the part. He did use a large industrial-size machine with a 40\u00a0W CO2 laser module. For the positioning, he used a printed block of known height to adjust the laser module. After making an initial high-power pass with air assist, he added powder-coat paints to the depth to make the engraving easier to see and turned off the air.

\n

During first engraving with 12\u00a0%, notice the thin white smoke at the center where the laser impacts, getting pulled away by the air assist. The engraving itself is visible as a slightly different color.\n\"Laser\n\"later

\n

After adding the powder and burning it in with a lower power setting (9\u00a0%), the result is very legible:\n\"engraved

\n

Power settings

\n

Mr. Adams used 12\u00a0% power on the initial pass and 9\u00a0% on the "inking" pass. Taking his 40\u00a0W Diode as a base, that'd correspond to 4.8\u00a0W and 3.6\u00a0W respectively, so could be done with a 5\u00a0W laser module.

\n

Do note that the initial pass power could also depend on the color of the laser and the exact composition of your polymer. This is because some colors and materials absorb wavelengths from the laser differently, so it is hard to say for sure the same setting will work for all colors the same.

\n

Print settings

\n

The engraving, with the right settings, should at most be one wall thickness deep, which would compromise that one wall's carrying capacity. A 2-wall print might see significant weakness due to the engraving, but with 3 or more walls, the impact would be massively reduced.

\n" }, { "Id": "22996", "CreationDate": "2024-01-26T11:13:40.067", "Body": "

Everything worked normally until the printer finished with a model after a 22-hour print job. As usual, I made everything like for the previous models to print the next one, but I suddenly noticed that this time it was printing only to the left side, the coordinates of Y-axis. I tried to change some parameters, opened and cleaned the nozzle from the PLA, powered it on-and-off to rest, checked everything, and performed some research on the net and youtube but it's still the same.

\n

Additionally, I'm sharing this photo as an example of the current situation, with labeling that works only in Y-axis:

\n

\"A

\n

Any suggestions, or solutions if someone has had the problem before?

\n", "Title": "Creality Ender 3 after normal printing, lost the ability to print in X and Z axis", "Tags": "|creality-ender-3|troubleshooting|z-axis|x-axis|", "Answer": "

I unscrewed parts that move X and Z axis, took off E and X connectors and then put everything in place. Gently moved the head on X axis, turned the printer on and it was working fine. Maybe, something was stuck or not in place while printing previous large model.

\n" }, { "Id": "23003", "CreationDate": "2024-01-27T21:30:24.177", "Body": "

I upgraded two printers (Creality Ender 3 V2) with a dual Z-axis upgrade using an additional stepper motor. However, I realized that this could potentially still cause the two screws to be moved individually.

\n

I figured that a mechanical dual Z-axis with a timing belt would have probably be a better upgrade instead of adding an additional stepper motor.

\n

Anyhow, when two stepper motors are used, how to accurately synchronise them so that both screws on the Z-axis are and stay synchronised?

\n", "Title": "How to accurately synchronise dual Z-axis when two steppers motors are used?", "Tags": "|z-axis|stepper|belt|", "Answer": "

There are a few options when adding an extra stepper motor, the usual upgrade is using a single stepper driver, driving 2 steppers either serial or parallel connected, or using 2 stepper drivers, this requires an extra stepper on the controller board (in that case one needs to replace the board; the standard board has the ability to control 4 stepper motors individually: X, Y, Extruder and Z).

\n

The forces in 3D printing of moving the carriage along the various axes are relatively low, far less than e.g. a CNC machine where a spinning tool is pushed through a material. In 3D printing the nozzle hovers over the printing object connected to the print by a molten drop of filament, virtually no resistance. This implies that it is very unlikely to miss steps on one of the driven sides so that is becomes out of sync. Note that you can automatically tune the two sides if you have two steppers on separate stepper drivers and have a BLTouch (I'm using this on my CoreXY).

\n

The two stepper motors push up a reasonably low mass for what the stepper motors can generate, certainly for the Z-axis as these are driven by lead screws. Driving a gantry with one stepper is the design flaw of the cheap-end printers, a timing belt with a single stepper works very well (I'm using this on my take of a Prusa i3 I designed), but must be mechanically tuned to be as straight in relation to the frame (if not, a test cube would look more as a 3D skewed cube).

\n
\n

Anyhow, when two stepper motors are used, how to accurately synchronise them so that both screws on the Z-axis are and stay synchronised?

\n
\n

Print a fixed distance leveling aid, see e.g. these and level the X-gantry to the frame first, remove the levelling aids and then level the bed through the bed screws. The screws will not likely get out of sync, but if so, re-level with respect to the frame. Or, buy a larger controller board with more spare stepper drivers and automatically tune Z1 and Z2; be sure the bed is parallel to the frame then.

\n" }, { "Id": "23004", "CreationDate": "2024-01-27T22:26:36.510", "Body": "

I'm trying to understand both Oldham couplers and a Anti-Recoil Nuts.

\n\n

In case of the Creality Ender 3 V2 both are a replacement for the original nut:

\n

\"Original

\n

Why is there no part that utilises both functions in order to have the benefits of both? Better alignment to reduce Z-Wobble using the Oldham Coupler, and less backlash and better motion precision using the Anti Recoil Nut.

\n

To my understanding they cannot be used both at the same time and I could not find a part that seems to combine those functions. Am I missing something? Does one make the other obsolete?

\n", "Title": "Can a Oldham coupler and a Anti-Recoil Nut be used at the same time?", "Tags": "|z-axis|lead-screw|z-wobble|anti-recoil|coupler|", "Answer": "

I don't see why you couldn't use them together. They are both dealing with eliminating a different effect.

\n

The Oldham deals with the movement in the X-Y plane, the backlash nut with the Z-axis.

\n

The backlash effect is not something to worry about, the Z-axis generally moves just in one direction and the gantry has reasonable mass.

\n" }, { "Id": "23011", "CreationDate": "2024-01-28T22:39:17.150", "Body": "

I have a Lotmaxx Shark v2, which has developed the problem of constantly saying the filament has ran out when it hasn't. I've narrowed down the issue to the main board, and tried to work around the problem:

\n\n

If, for whatever reason, the main board is damaged, I'd love to just disable this sensor. I don't really need it. Also unused is the second extruder set, so if I can't disable the sensor maybe I could configure the printer to use extruder #2 all the time instead of extruder #1?

\n", "Title": "How to disable a filament sensor?", "Tags": "|marlin|g-code|filament-sensor|runout-sensor|lotmaxx|", "Answer": "

Via firmware

\n

Looking at your M412 - Filament Runout link, it says that it requires FILAMENT_RUNOUT_SENSOR.

\n

Try disabling FILAMENT_RUNOUT_SENSOR in the firmware by commenting out (//) the following line, in Configuration.h, like so

\n
//#define FILAMENT_RUNOUT_SENSOR\n
\n

From Filament Runout Sensor

\n
//#define FILAMENT_RUNOUT_SENSOR\n#if ENABLED(FILAMENT_RUNOUT_SENSOR)\n  #define NUM_RUNOUT_SENSORS   1     // Number of sensors, up to one per extruder. Define a FIL_RUNOUT#_PIN for each.\n  #define FIL_RUNOUT_INVERTING false // Set to true to invert the logic of the sensor.\n\n...\n
\n
\n

Via UI

\n

Watch this video, The BEST Ender-3 V2 Firmware EVER?! MRiscoC Professional Firmware, at 2:43,

\n

\n \n

\n

It shows the Filament sensor can be disabled via the UI:

\n

\"Filament

\n" }, { "Id": "23015", "CreationDate": "2024-01-29T12:31:20.743", "Body": "
What I saw
\n

Last night, I sliced a Califlower in Cura, used the "Send to OctoPrint" option to send it to the printer via the OctoPrint Connection plugin (the standard Cura OctoPrint plugin at v3.7.3), made sure the first layer printed fine, and then put my PC to sleep.

\n

That's when I noticed the print had stopped.

\n
What I expected to see
\n

I expected the print to carry on in the background, as I thought the print had already been sent to the OctoPrint 'server'.

\n

It seems that rather than Cura uploading the G-code to OctoPrint (running on a Raspberry Pi 5\u2020, plugged into the printer via USB) telling it to print and then just monitoring progress, it streams the whole G-code in real time, using OctoPrint as a glorified network attached USB serial port, which rather defeats the purpose of having a print 'server' at all\u2021.

\n
What I tried
\n

I tried setting the option to "Store G-code on the SD card of the printer", but the Note: wasn't kidding when it said this would take a long time. Since there was no option to cancel, I ended up restarting OctoPrint, forcing a disconnect.

\n
What I want to understand
\n

Is there any way to get Cura to upload the G-code to OctoPrint and then tell it to print rather than stream it to the printer?

\n

I would prefer my power efficient Raspberry Pi pushing G-code commands to the printer for hours, not my power hungry gaming PC.

\n
\n† Specifically a Raspberry Pi 5, running Raspberry Pi OS the from 2023-13-05 Image, fully `apt` upgraded, and running OctoPrint 1.9.3, installed using the recommended `octoprint_deploy` script.\n

\u2021 When I send a 50 page document to my networked HP printer, I don't expect that to stop printing after I put my PC to sleep either!

\n", "Title": "Getting OctoPrint to continue printing when I put my PC to sleep?", "Tags": "|ultimaker-cura|octoprint|", "Answer": "

The obvious, a bit expensive but probably best and most common solution is to use Octopi. (it's so common it took me a while to realize you're running Octoprint from a PC!)

\n

You get a Raspberry Pi 3b, or 4, or any of a large number of cheaper and more available alternatives (I personally use Orange Pi zero 2) - then you install Linux with Octoprint on it. You power it up from the printer's power through some sort of regulator, connect the printer with it over a stub of USB cable, and connect it to your home LAN over Ethernet or Wifi. It's powered on and running while the printer is running. You link up Cura to it to upload gcode to its sd card (over LAN, so very fast) and you control it over web interface. That means the PC can be in another room (with Octoptint page displaying camera view of the print bed on second screen, in case anything goes wrong), you can design and slice your projects in silence and shut down your PC when you're done, while the small Pi works overnight feeding the printer gcode over its USB connection.

\n

Configuration:

\n\n

\"enter

\n\n

\"enter

\n\n

\"enter

\n\n

\"enter

\n

From then on the PC is connected to Octopi over the net, so the transmission between the two is vastly faster than over serial, and after slicing you get the drop-down option "Print with Octoprint" besides "Save to disk". Selecting "Print with Octoprint" allows you to type in the directory on Pi, edit the file name, and pick whether to select and start the print immediately - without that it will just be placed in the storage medium of the Octoprint device, to be started from Octoprint at a later time.

\n

\"enter

\n" }, { "Id": "23041", "CreationDate": "2024-02-03T13:34:35.970", "Body": "

Starting from this sketch:

\n

\"Sketch

\n

I've revolved and extruded it to result in the body shown:

\n

\"3D

\n

This is destined to be printed, so I need to fill in the cavity of the stick-y-out-y bit. I can use Surface->Patch to make it look correct from the outside, but there's still a void within:

\n

\"Close-up

\n

\"3D

\n

\"Sectional

\n

I've searched using phrases such as "fusion join surface to body" "fusion fill void within body" and have only found results talking about making a body exclusively from surfaces. One promising sounding option was "boundary fill", but it seems to have no effect in this case.

\n

What is the correct tool / technique to use here?

\n", "Title": "How to fill a void within a body?", "Tags": "|fusion360|", "Answer": "

Simplify the order of operations

\n

Sometimes, it is way easier to do the order of operations in a different order:

\n

First, create the thickness of the body. Only then then subtract the rotation profile and last sweep the the outer profile subtractively

\n

\"enter\n\"enter\n\"enter

\n" }, { "Id": "23070", "CreationDate": "2024-02-15T01:53:48.250", "Body": "

I have an Ender 3 Pro, board version 4.2.2, and a CR Touch with 5 pin connector. I also removed the z-axis endstop connector at the endstop.

\n

I've been building and flashing Marlin 2.1.2.2, but I haven't found a configuration that makes the CRTouch work as expected. Typically, during auto homing, the machine will ignore the probe triggering and drive Z into the bed either 100% of the time or most of the time. I didn't take detailed enough notes to know for sure.

\n

As a test case, I tried following along with this answer exactly and still had the same problem. Configuration changes from the Marlin Ender 3 Pro default:

\n\n

On my first boot after inserting the SD card with firmware:

\n
    \n
  1. The probe deployed and retracted a couple of times, ending retracted with the red LED on
  2. \n
  3. The machine seemingly flashed the firmware as it sat on a blue screen for a while
  4. \n
  5. The CRTouch LED turned purple I think when flashing ended, though I can't tell from my video
  6. \n
  7. When I got to the main menu, I started auto homing
  8. \n
  9. X and Y axes worked as expected
  10. \n
  11. For the Z axis, the probe centered on the build plate, deployed, and the LED turned blue
  12. \n
  13. The probe started lowering, contacted the plate and retracted. The LED turned to blinking red, maybe steady red first. The Z motor kept driving down so I killed the power
  14. \n
\n

The extruder fan also made a loud buzzing noise that gradually grew quieter. In the past I've noticed the buzzing seemingly get quieter when the probe is deployed or retracted.

\n

Anyone know what could be causing this, or how to troubleshoot it? Creality's firmware works as expected, so it's not a hardware problem. I was thinking maybe PB1 is wrong for the Ender 3 Pro, but from what I see online I gather it's the same mainboard between both printers.

\n", "Title": "What do I need to configure to get Marlin to run an Ender 3 Pro with CR Touch?", "Tags": "|creality-ender-3|marlin|bltouch|z-probe|crtouch|", "Answer": "

I think this is an issue with Marlin v2.1.2.2, which I was trying to use. https://github.com/MarlinFirmware/Marlin/issues/26809 this issue points to another bug with probing. My issue is different, but downgrading to 2.1.1 solved it.

\n" }, { "Id": "23104", "CreationDate": "2024-02-29T01:28:02.490", "Body": "

I printed this on an Ender 3 S1 Pro at 215\u02daC with a 20% infill (actually 5 were printed at once). The print bed was leveled prior to print.\"The

\n

When I tried to remove the supports, they took the bottom of the sphere off with them.

\n

How typical is this, and what parameters can I change to have the best chance of a successful print?

\n", "Title": "Help With Bottom of Sphere Coming off With Supports", "Tags": "|support-structures|", "Answer": "

Have you seen question 3D Printed Sphere, How to Remove Roughness? Although the question is centered around the roughness, not the supports, the answers hint to a solution for this question.

\n

If you look at the answers, you see that in case you have have unsupported perimeters (see image below), they sag out (as seen in your print where the lower perimeters of the sphere are not perectly round) and may fuse with the support causing the support and the perimiters (walls) to bond quite well.

\n

Furthermore, if you use too few perimeters and a low infill percentage, the print object will become weak and have an increased change that removing the print from the plate or the support from the print cause the print to break.

\n

\"enter

\n

Picture credits to Trish, this is 180\u00b0 rotated

\n" }, { "Id": "23117", "CreationDate": "2024-03-04T04:32:39.117", "Body": "

My Setup

\n\n

Wiring

\n\n

Problem

\n

The hotend immediately starts heating at 100% (24\u00a0V DC) on startup without any command to do so.

\n

What I've tried

\n\n

Question

\n

How do I stop the hot end from heating up on startup?

\n

Klipper printer.cfg:

\n
[include moonraker_obico_macros.cfg]\n# This file contains pin mappings for the Creality "v4.2.7" board. To\n# use this config, during "make menuconfig" select the STM32F103 with\n# a "28KiB bootloader" and serial (on USART1 PA10/PA9) communication.\n\n# If you prefer a direct serial connection, in "make menuconfig"\n# select "Enable extra low-level configuration options" and select\n# serial (on USART3 PB11/PB10), which is broken out on the 10 pin IDC\n# cable used for the LCD module as follows:\n# 3: Tx, 4: Rx, 9: GND, 10: VCC\n\n# Flash this firmware by copying "out/klipper.bin" to a SD card and\n# turning on the printer with the card inserted. The firmware\n# filename must end in ".bin" and must not match the last filename\n# that was flashed.\n\n# See docs/Config_Reference.md for a description of parameters.\n\n[skew_correction]\n\n# [filament_switch_sensor runout_sensor]\n# pause_on_runout: True\n# switch_pin: PA4\n\n[stepper_x]\nstep_pin: PB9\ndir_pin: PC2\nenable_pin: !PC3\nmicrosteps: 16\nrotation_distance: 39.83\nendstop_pin: ^PA5\nposition_endstop: 0\nposition_max: 235\nhoming_speed: 50\n\n[stepper_y]\nstep_pin: PB7\ndir_pin: PB8\nenable_pin: !PC3\nmicrosteps: 16\nrotation_distance: 39.78\nendstop_pin: ^PA6\nposition_endstop: 0\nposition_max: 235\nhoming_speed: 50\n\n[stepper_z]\nstep_pin: PB5\ndir_pin: !PB6\nenable_pin: !PC3\nmicrosteps: 16\nrotation_distance: 8\n# position_endstop: 0.0                     # disable to use BLTouch\n# endstop_pin: ^PA7                         # disable to use BLTouch\nendstop_pin: probe:z_virtual_endstop    # enable to use BLTouch\nposition_min: -5                        # enable to use BLTouch\nposition_max: 250\n\n[safe_z_home]                         # enable for BLTouch\nhome_xy_position: 157.5,120.5\nspeed: 100\nz_hop: 10\nz_hop_speed: 5\n\n[bltouch]                             # enable for BLTouch - fast-mode\nsensor_pin: PA7\ncontrol_pin: PB0\npin_up_touch_mode_reports_triggered: True\nprobe_with_touch_mode: True\n# x_offset: -44                         # modify as needed for bltouch location\n# y_offset: -6                          # modify as needed for bltouch location\nx_offset: -28                           # For the sprite\ny_offset: -40                           # For the sprite\n#z_offset: 0.0                         # modify as needed for bltouch or run PROBE_CALIBRATE\nspeed: 10\nsample_retract_dist: 5.0              # Can be set lower, example 2.5 depending on height of bltouch from bed\nlift_speed: 40\nsamples_tolerance_retries: 3\nspeed: 10\nsamples: 1\n\n[bed_mesh]\nspeed: 80\nhorizontal_move_z: 5\nmesh_min: 18,18\nmesh_max: 175,202\nprobe_count: 5,5\nalgorithm: bicubic\n\n# manual Bed adjustment via BED_SCREWS_ADJUST\n[bed_screws]\nscrew1: 72.5, 41.5\nscrew1_name: front left screw\nscrew2: 198.5,35.5\nscrew2_name: front right screw\nscrew3: 198.5,205.5\nscrew3_name: rear right screw\nscrew4: 28.5,205.5\nscrew4_name: rear left screw\nhorizontal_move_z: 10\nspeed: 50\n\n[screws_tilt_adjust]\nscrew1: 72.5, 41.5\nscrew1_name: front left screw\nscrew2: 220,41.5\nscrew2_name: front right screw\nscrew3: 220,212.5\nscrew3_name: rear right screw\nscrew4: 72.5,212.5\nscrew4_name: rear left screw\nhorizontal_move_z: 10\nspeed: 50\nscrew_thread: CW-M4\n\n[input_shaper]\nshaper_freq_x: 100\nshaper_freq_y: 100\nshaper_type: mzv\n\n[gcode_macro G29]\ngcode:\n    G28\n    BED_MESH_CALIBRATE\n    G0 X0 Y0 Z10 F6000\n    BED_MESH_PROFILE save=default\n    SAVE_CONFIG\n\n\n[extruder]\nmax_extrude_only_distance: 100.0\nstep_pin: PB3\ndir_pin: PB4\nenable_pin: !PC3\nmicrosteps: 16\nrotation_distance: 33.500\nnozzle_diameter: 0.400\nfilament_diameter: 1.750\nheater_pin: PA1\nsensor_type: EPCOS 100K B57560G104F\nsensor_pin: PC5\ncontrol: pid\npid_Kp: 21.527\npid_Ki: 1.063\npid_Kd: 108.982\nmin_temp: 0\nmax_temp: 250\n\n[heater_bed]\nheater_pin: PA2\nsensor_type: EPCOS 100K B57560G104F\nsensor_pin: PC4\ncontrol: pid\npid_Kp: 54.027\npid_Ki: 0.770\npid_Kd: 948.182\nmin_temp: 0\nmax_temp: 130\n\n#[fan]\n#pin: PA0\n\n[fan]\npin: PA0\n\n[mcu]\nserial: /dev/serial/by-id/usb-1a86_USB_Serial-if00-port0\nrestart_method: command\n\n[printer]\nkinematics: cartesian\nmax_velocity: 300\nmax_accel: 3000\nmax_accel_to_decel: 3000\nmax_z_velocity: 5\nmax_z_accel: 100\n\n[display]\nlcd_type: st7920\ncs_pin: PB12\nsclk_pin: PB13\nsid_pin: PB15\nencoder_pins: ^PB14, ^PB10\nclick_pin: ^!PB2\n\n[gcode_macro G29]\ngcode:\n    G28\n    G1 Z10 F600\n    BED_MESH_CLEAR\n    BED_MESH_CALIBRATE\n    BED_MESH_PROFILE SAVE=default\n    SAVE_CONFIG\n\n[temperature_sensor raspberry_pi]\nsensor_type: temperature_host\nmin_temp: 10\nmax_temp: 100\n\n[temperature_sensor mcu_temp]\nsensor_type: temperature_mcu\nmin_temp: 0\nmax_temp: 100\n\n[board_pins]\naliases:\n  EXP1_1=PC6,EXP1_3=PB10,EXP1_5=PB14,EXP1_7=PB12,EXP1_9=<GND>,\n  EXP1_2=PB2,EXP1_4=PB11,EXP1_6=PB13,EXP1_8=PB15,EXP1_10=<5V>,\n  PROBE_IN=PB0,PROBE_OUT=PB1,FIL_RUNOUT=PC6\n\n[exclude_object]\n\n[include mainsail.cfg]\n[include timelapse.cfg]\n\n[gcode_macro do_the_mesh]\ngcode:\n  BED_MESH_PROFILE LOAD="default"\n  SKEW_PROFILE LOAD=my_skew_profile\n\ndescription: Either load the default mesh or perform a mesh load operation.\n\n#*# <---------------------- SAVE_CONFIG ---------------------->\n#*# DO NOT EDIT THIS BLOCK OR BELOW. The contents are auto-generated.\n#*#\n#*# [bltouch]\n#*# z_offset = 1.420\n#*#\n#*# [bed_mesh default]\n#*# version = 1\n#*# points =\n#*#       0.005000, -0.022500, -0.015000, -0.015000, -0.017500\n#*#       0.032500, -0.022500, -0.015000, -0.042500, -0.047500\n#*#       0.105000, 0.022500, 0.020000, -0.045000, -0.077500\n#*#       0.140000, 0.080000, 0.055000, 0.015000, 0.022500\n#*#       0.225000, 0.182500, 0.147500, 0.077500, 0.067500\n#*# x_count = 5\n#*# y_count = 5\n#*# mesh_x_pps = 2\n#*# mesh_y_pps = 2\n#*# algo = bicubic\n#*# tension = 0.2\n#*# min_x = 18.0\n#*# max_x = 175.0\n#*# min_y = 18.0\n#*# max_y = 202.0\n#*#\n#*# [skew_correction my_skew_profile]\n#*# xy_skew = -0.0008053242270892168\n#*# xz_skew = 0.0\n#*# yz_skew = 0.0\n```\n
\n", "Title": "Creality Sprite Pro Immediately starts heating hotend on startup", "Tags": "|creality-ender-3|hotend|klipper|", "Answer": "

After much gnashing of teeth, I determined that the problem revolved around the Z-stop wire from the 40 conductor cable being connected.

\n

The Sprite Pro is equipped with a 5-pin BL Touch connector. For whatever reason, having the 2-pin connector plugged into the Z-stop port causes the nozzle heater output to peg at 24\u00a0V DC.

\n

Updated printer.cfg

\n
[include moonraker_obico_macros.cfg]\n# This file contains pin mappings for the Creality "v4.2.7" board. To\n# use this config, during "make menuconfig" select the STM32F103 with\n# a "28KiB bootloader" and serial (on USART1 PA10/PA9) communication.\n\n# If you prefer a direct serial connection, in "make menuconfig"\n# select "Enable extra low-level configuration options" and select\n# serial (on USART3 PB11/PB10), which is broken out on the 10 pin IDC\n# cable used for the LCD module as follows:\n# 3: Tx, 4: Rx, 9: GND, 10: VCC\n\n# Flash this firmware by copying "out/klipper.bin" to a SD card and\n# turning on the printer with the card inserted. The firmware\n# filename must end in ".bin" and must not match the last filename\n# that was flashed.\n\n# See docs/Config_Reference.md for a description of parameters.\n\n[skew_correction]\n\n# [filament_switch_sensor runout_sensor]\n# pause_on_runout: True\n# switch_pin: PA4\n\n[stepper_x]\nstep_pin: PB9\ndir_pin: PC2\nenable_pin: !PC3\nmicrosteps: 16\nrotation_distance: 39.83\nendstop_pin: ^PA5\nposition_endstop: 0\nposition_max: 235\nhoming_speed: 50\n\n[stepper_y]\nstep_pin: PB7\ndir_pin: PB8\nenable_pin: !PC3\nmicrosteps: 16\nrotation_distance: 39.78\nendstop_pin: ^PA6\nposition_endstop: 0\nposition_max: 235\nhoming_speed: 50\n\n[stepper_z]\nstep_pin: PB5\ndir_pin: !PB6\nenable_pin: !PC3\nmicrosteps: 16\nrotation_distance: 8\n# position_endstop: 0.0                     # disable to use BLTouch\n# endstop_pin: ^PA7                         # disable to use BLTouch\nendstop_pin: probe:z_virtual_endstop    # enable to use BLTouch\nposition_min: -5                        # enable to use BLTouch\nposition_max: 250\n\n[safe_z_home]                         # enable for BLTouch\n# home_xy_position: 157.5,120.5\nhome_xy_position: 139.5,169\nspeed: 100\nz_hop: 10\nz_hop_speed: 5\n\n[bltouch]                             # enable for BLTouch - fast-mode\n# sensor_pin: PA7                     # 3 pin connector plus separate z endstop\nsensor_pin: ^PB1                      # 5 pin connector: https://www.reddit.com/r/ender3v2/comments/pw1eyq/printercfg_with_bltouch_for_klipper/\ncontrol_pin: PB0\npin_up_touch_mode_reports_triggered: True\nprobe_with_touch_mode: True\n# x_offset: -44                         # modify as needed for bltouch location\n# y_offset: -6                          # modify as needed for bltouch location\nx_offset: -30                           # For the sprite\ny_offset: -41.5                         # For the sprite\n#z_offset: 0.0                         # modify as needed for bltouch or run PROBE_CALIBRATE\nspeed: 10\nsample_retract_dist: 5.0              # Can be set lower, example 2.5 depending on height of bltouch from bed\nlift_speed: 40\nsamples_tolerance_retries: 3\nspeed: 10\nsamples: 1\n\n[bed_mesh]\nspeed: 80\nhorizontal_move_z: 5\nmesh_min: 18,18\nmesh_max: 175,202\nprobe_count: 5,5\nalgorithm: bicubic\n\n# manual Bed adjustment via BED_SCREWS_ADJUST\n[bed_screws]\nscrew1: 72.5, 41.5\nscrew1_name: front left screw\nscrew2: 198.5,35.5\nscrew2_name: front right screw\nscrew3: 198.5,205.5\nscrew3_name: rear right screw\nscrew4: 28.5,205.5\nscrew4_name: rear left screw\nhorizontal_move_z: 10\nspeed: 50\n\n[screws_tilt_adjust]\nscrew1: 72.5, 41.5\nscrew1_name: front left screw\nscrew2: 220,41.5\nscrew2_name: front right screw\nscrew3: 220,212.5\nscrew3_name: rear right screw\nscrew4: 72.5,212.5\nscrew4_name: rear left screw\nhorizontal_move_z: 10\nspeed: 50\nscrew_thread: CW-M4\n\n[input_shaper]\nshaper_freq_x: 100\nshaper_freq_y: 100\nshaper_type: mzv\n\n[gcode_macro G29]\ngcode:\n    G28\n    BED_MESH_CALIBRATE\n    G0 X0 Y0 Z10 F6000\n    BED_MESH_PROFILE save=default\n    SAVE_CONFIG\n\n\n[extruder]\nmax_extrude_only_distance: 100.0\nstep_pin: PB3\ndir_pin: PB4\nenable_pin: !PC3\nmicrosteps: 16\n# rotation_distance: 33.500  # for stock extruder\nrotation_distance: 7.53      # for sprite pro\nnozzle_diameter: 0.400\nfilament_diameter: 1.750\nheater_pin: PA1\nsensor_type: EPCOS 100K B57560G104F\nsensor_pin: PC5\n#control: pid\n#pid_Kp: 21.527\n#pid_Ki: 1.063\n#pid_Kd: 108.982\nmin_temp: 0\nmax_temp: 250\n\n[heater_bed]\nheater_pin: PA2\nsensor_type: EPCOS 100K B57560G104F\nsensor_pin: PC4\ncontrol: pid\npid_Kp: 54.027\npid_Ki: 0.770\npid_Kd: 948.182\nmin_temp: 0\nmax_temp: 130\n\n#[fan]\n#pin: PA0\n\n[fan]\npin: PA0\n\n[mcu]\nserial: /dev/serial/by-id/usb-1a86_USB_Serial-if00-port0\nrestart_method: command\n\n[printer]\nkinematics: cartesian\nmax_velocity: 300\nmax_accel: 3000\nmax_accel_to_decel: 3000\nmax_z_velocity: 5\nmax_z_accel: 100\n\n[display]\nlcd_type: st7920\ncs_pin: PB12\nsclk_pin: PB13\nsid_pin: PB15\nencoder_pins: ^PB14, ^PB10\nclick_pin: ^!PB2\n\n[gcode_macro G29]\ngcode:\n    G28\n    G1 Z10 F600\n    BED_MESH_CLEAR\n    BED_MESH_CALIBRATE\n    BED_MESH_PROFILE SAVE=default\n    SAVE_CONFIG\n\n[temperature_sensor raspberry_pi]\nsensor_type: temperature_host\nmin_temp: 10\nmax_temp: 100\n\n[temperature_sensor mcu_temp]\nsensor_type: temperature_mcu\nmin_temp: 0\nmax_temp: 100\n\n[board_pins]\naliases:\n  EXP1_1=PC6,EXP1_3=PB10,EXP1_5=PB14,EXP1_7=PB12,EXP1_9=<GND>,\n  EXP1_2=PB2,EXP1_4=PB11,EXP1_6=PB13,EXP1_8=PB15,EXP1_10=<5V>,\n  PROBE_IN=PB0,PROBE_OUT=PB1,FIL_RUNOUT=PC6\n\n[exclude_object]\n\n[include mainsail.cfg]\n[include timelapse.cfg]\n\n[gcode_macro do_the_mesh]\ngcode:\n  BED_MESH_PROFILE LOAD="default"\n  SKEW_PROFILE LOAD=my_skew_profile\n\ndescription: Either load the default mesh or perform a mesh load operation.\n\n#*# <---------------------- SAVE_CONFIG ---------------------->\n#*# DO NOT EDIT THIS BLOCK OR BELOW. The contents are auto-generated.\n#*#\n#*# [bltouch]\n#*# z_offset = 4.350\n#*#\n#*# [bed_mesh default]\n#*# version = 1\n#*# points =\n#*#     0.005000, -0.022500, -0.015000, -0.015000, -0.017500\n#*#     0.032500, -0.022500, -0.015000, -0.042500, -0.047500\n#*#     0.105000, 0.022500, 0.020000, -0.045000, -0.077500\n#*#     0.140000, 0.080000, 0.055000, 0.015000, 0.022500\n#*#     0.225000, 0.182500, 0.147500, 0.077500, 0.067500\n#*# x_count = 5\n#*# y_count = 5\n#*# mesh_x_pps = 2\n#*# mesh_y_pps = 2\n#*# algo = bicubic\n#*# tension = 0.2\n#*# min_x = 18.0\n#*# max_x = 175.0\n#*# min_y = 18.0\n#*# max_y = 202.0\n#*#\n#*# [skew_correction my_skew_profile]\n#*# xy_skew = -0.0008053242270892168\n#*# xz_skew = 0.0\n#*# yz_skew = 0.0\n#*#\n#*# [extruder]\n#*# control = pid\n#*# pid_kp = 19.488\n#*# pid_ki = 1.101\n#*# pid_kd = 86.236\n```\n
\n" }, { "Id": "23119", "CreationDate": "2024-03-04T11:54:16.633", "Body": "

I recently started experimenting with sending G-code to my printer via USB cable. I downloaded Printrun and it works great! However, I want to be able to do this from my web browser because the app im developing is web based.

\n

I found this Chrome app called chrome-gcode-sender that does this but it unfortunately no longer works because Google sadly ended support for all Chrome apps in June 2022.

\n

The maker of this app suggested on GitHub that the same thing should be achievable using Web Serial API. I have been trying to get this to work for almost two days now but with little success. I found several tutorials online about how to use Web Serial API but nothing for this specific use case. Here is a sample of the code I have developed so far, just a simple HTML document with some vanilla JavaScript:

\n
<html>\n  <head>\n    <title>Web Serial API Demo</title>\n  </head>\n  <body>\n    <button id="connect">Connect</button>\n    <script>\n\n        var device\n        let button = document.getElementById("connect")\n\n        document.addEventListener("DOMContentLoaded", event => {\n            button.addEventListener("click", async() => {\n                try {\n\n                    device = await navigator.serial.requestPort()\n\n                    await device.open({baudRate: 9600})\n\n                    console.log("Opened: ", device)\n                    console.log("Info: ", device.getInfo())\n\n                    const encoder = new TextEncoder()\n                    const writer = device.writable.getWriter()\n\n                    await writer.write(encoder.encode("G28"))\n\n                    writer.releaseLock()\n\n                    await device.close()\n\n                } catch (error) {\n\n                    console.log(error)\n\n                }\n            })\n        })\n\n    </script>\n  </body>\n</html>\n
\n

When I click the 'Connect' button and select the device I want it appears to connect fine. However, when I try to send a simple G-code like G28 (autohome) the printer does nothing. If anyone has an idea how I could get this working please let me know! Im also 100\u00a0% open to an alternative approach! The main idea is just to be able to send G-code from a browser to a printer.

\n

Notes:

\n\n", "Title": "Can I use Web Serial API to send G-code to my Ender 5 Pro?", "Tags": "|g-code|creality-ender-5|usb|serial-connection|", "Answer": "

Finally, got it! I was very close, but two changes needed to be made to the above code for the example to work.

\n
    \n
  1. The baudRate needed to be changed to 115200. This value can vary from printer to printer but 115200 is fairly common, and thus a good default. If you find it's not working for you try looking up the recommended baudRate for your printer/firmware version.

    \n
  2. \n
  3. I needed to add \\n to the end of my G-code string. In this example there was only one command, G28\\n but if you want multiple commands in a single string you will need to add \\n after each one.

    \n
  4. \n
\n

Awesome!!! It is possible to send G-code to a printer from a web browser with just a few lines of JavaScript!

\n

Update: If anyone is interested I built this simple terminal-style app. It should work with any compatible browser and is also usable offline if you clone the repo and open the sender.html file in your browser.

\n" }, { "Id": "23122", "CreationDate": "2024-03-04T20:00:28.877", "Body": "

I am getting this under extrusion on my prints but only after travel moves. The rest of the print looks fine.

\n

\"enter\n\"enter

\n

My printer is a Monoprice Maker Select V2 (rebranded Wanhao Duplicator i3 V2) with a DiiiCooler with a radial fan. I was using Repetier 0.92.9 but have upgraded the firmware to Marlin bugfix-2.1.x commit 9e879a5b.

\n

I'm using Cura 5.6.0 and the default Draft profile with the following settings changed:

\n\n

I'm printing with Inland Turquoise PLA at 215\u00a0\u00b0C and the build plate at 60\u00a0\u00b0C.

\n

I am printing Kersey Fabrications Speed Test.

\n

I'm pulling my hair out. I've tried the following without making much headway:

\n\n

For the first print, I did 0.5-3\u00a0mm distance, 50\u00a0mm/s retract, 20\u00a0mm/s advance. There was very slight stringing at 0.5\u00a0mm and none in the other sections. All of the sections had slightly jagged edges except for 1\u00a0mm.

\n

For the second print, I did 1\u00a0mm distance, 20-70\u00a0mm/s retract, 20\u00a0mm/s advance. There was no stringing and I didn't see any apparent difference between the sections.

\n

For the third print, I did 1\u00a0mm distance, 50\u00a0mm/s retract, 10-50\u00a0mm/s advance. Again, no stringing and no differences between the sections.

\n

None of the three prints showed any signs of under-extrusion.

\n\n

It didn't spin freely, it turned but felt mushy. I cleaned the grease from the wheel and the surrounding area and added a drop of sewing machine oil to each side of the wheel. After that, it spun freely.

\n

I did a cold check of my extruder steps with the extruder removed and found that it matched what I found earlier when I checked it hot through the hotend.

\n

After reassembling the extruder, I reprinted the acceleration tuning tower from the same G-code as before but I saw no improvement to the under extrusion.

\n\n", "Title": "What is causing this under extrusion that only happens after travel moves", "Tags": "|extruder|underextrusion|retraction|monoprice-maker-select|wanhao|", "Answer": "

Printing with a brand new roll of Polymaker PolyLite grey PLA seems to have fixed the problem. I'll have to redo all of my tuning and do some more tests, but there's no sign of the under extrusion so far.

\n

I ran out of the Inland turquoise PLA doing all my testing but I have tried drying the Inland black PLA+. I've dried it a total of 36 hours now at 50 \u00b0C and it made no improvement to the under extrusion issue. 24 hours of that was in a Sunlu S1 filament dryer (the original model without a fan) in 6 and 12 hour increments. 12 hours of that was in a convection toaster oven converted to a reflow oven with a Controleo3 kit. I tried drying with and without desiccant packs in both the filament dryer and the reflow oven.

\n

I don't know how or why the filament degraded but it does not seem to be due to any current moisture content.

\n" } ]